You are on page 1of 1943

Which one of the following is not a differential diagnosis of shoulder tip pain?

A. Pulmonary embolism.

B. Myocardial infarction.

C. Emphysema.

D. Pneumothorax.

E. Peptic ulcer disease.

Incorrect. Correct answer is C


45% answered correctly

Explanation:

Correct Answer Is C

Shoulder-tip pain is an important clinical sign and can be caused by local musculoskeletal trauma or inflammation or referral.

The following are the differential diagnoses of referred shoulder-tip pain:

Pulmonary embolism
Pneumothorax
Myocardial infarction
Perforation of peptic ulcer disease
Diaphragmatic irritation

Emphysema is not a cause of shoulder-tip pain, unless a spontaneous pneumothorax occurs.

References

• http://www.webmd.com/pain-management/tc/referred-s

• http://www.uptodate.com/contents/evaluation-of-the

Last updated:
Time spent: QID:14
2023-2-12

1 of 1943
After percutaneous coronary intervention (PCI) in a patient with ST segment elevation myocardial infarction (STEMI), which one the following
is the recommended period for antiplatelet therapy?

A. Two weeks.

B. Four weeks.

C. Six weeks.

D. 12 months.

E. Lifelong

Incorrect. Correct answer is D


45% answered correctly

Explanation:

Correct Answer Is D

In nearly all patients in whom intra-coronary drug-eluting stent is placed, dual antiplatelet therapy is indicated for at least 12 months. Dual
therapy is with aspirin and any of the following:

Clopidogrel
Prasugrel
Ticagrelor

References

• RACGP - AFP - Dual antiplatelet therapy

Last updated:
Time spent: QID:25
2023-2-12

2 of 1943
Which one of the following is of predictive value for prognosis of a patient with systolic heart failure?

A. Jugular venous pressure.

B. Peripheral edema.

C. Shortness of breath.

D. Orthopnea.

E. Chest pain.

Correct
45% answered correctly

Explanation:

Correct Answer Is A

Prognosis of systolic heart failure can be predicted by jugular venous pressure (JVP) and the third heart sound (S3). Increased JVP or the
presence of S3 sound indicates poor prognosis.

Other options are of diagnostic, not predictive values.

References

• http://www.uptodate.com/contents/prognosis-of-hear

Last updated:
Time spent: QID:26
2023-2-12

3 of 1943
You are called to see a 50-year-old Aboriginal woman with history of congestive heart failure, who has developed severe dyspnea and is in
respiratory distress. On examination, she is pale and sweaty with a blood pressure of 110/75 mmHg and pulse rate of 120bpm. The
respiratory rate is 26/min. Chest auscultation is significant for bilateral crackles. A chest X-ray shows bilateral whiteout of lungs from the
lower lobes up to middle of the lung fields. An ECG shows no abnormality. Troponin level is normal on arrival and eight hours later. Which one
of the following is the most likely diagnosis?

A. Acute right heart failure.

B. Acute pulmonary edema.

C. Pleural effusion.

D. Acute myocardial infarction.

E. Pulmonary embolism.

Incorrect. Correct answer is B


45% answered correctly

Explanation:

Correct Answer Is B

Acute pulmonary edema presents with dyspnea, bibasilar coarse crackles, tachycardia, pallor and cold limbs due to hypoperfusion. The
background history of congestive heart failure supports the diagnosis of pulmonary edema in this patient as the most likely diagnosis.

(Option A) Acute right heart failure presents with ankle edema, raised JVP, hepatomegaly, shortenss of breath. With only lung congestion and
no other findings that can be present in right heart failure, this diagnosis less likely.

(Option C) Pleural effusion progresses slowly and usually does not cause acute-onset symptoms. Even an acute pleural effusion takes hours
to days to develop.

(Option D) With acute myocardial infarction ECG changes and positive cardiac enzymes should have been present.

(Option E) A massive pulmonary embolism (PE) can be the second likely diagnosis on the list differential diagnoses. PE presents with
shortness of breath, tachypnea and tachycardia as the hallmark symptoms. Pleuretic chest pain or shoulder-tip pain may be other finding.
ECG may be normal or show S1Q3T3 pattern (prominent S wave in lead I, Q wave and T wave inversion in lead III), sinus tachycardia, T wave
inversion in leads V1 –V3 and right bundle branch block.

References

• http://www.heartfoundation.org.au/SiteCollectionDo

Last updated:
Time spent: QID:28
2023-2-12

4 of 1943
Which one of the following is not indicated in management of pulmonary edema?

A. Continuous positive airway pressure (CPAP).

B. Bilevel positive airway pressure (BiPAP).

C. Glyceryl trinitrate.

D. Oral forusemide.

E. Morphine.

Incorrect. Correct answer is D


45% answered correctly

Explanation:

Correct Answer Is D

Management of acute pulmonary oedema is as follows:

Oxygen 10-15 L/min by Hudson mask and reservoir bag.

Once the patient is stable continue oxygen 2-6 L/min by nasal cannula.

Intravenous forusemide - it is one of the most essential steps in treatment of pulmonary oedema by decreasing the volume overload.
If taken orally, forusemide takes time to work and is not effective in treatment.

Glyceryl trinitrate – it reduces the preload; it is essential to titrate the dose to maintain systolic blood pressure above 100mmHg.

Morphine – by decreasing the sympathetic tone, it results in vasodilation and reduction of preload

CPAP and BiPAP – these non-invasive methods of ventilation are used to reduce alveolar and pulmonary edema by reducing the
venous return and preload.

References

• Therapeutic Guidelines - Cardiovascular

• http://www.racgp.org.au/download/documents/AFP/201

• https://www.mja.com.au/journal/2011/194/8/2011-upd

Last updated:
Time spent: QID:29
2023-2-12

5 of 1943
A 65-year-old woman presents with several episodes of acute lightheadedness, especially shortly after getting off the bed or a chair for the
past 3 months. Her medical history is otherwise unremarkable. A table tilt test is arranged that is positive. Which one of the following would
be the first-line management of this patient?

A. Oral fludrocortisones.

B. Oral hydrocortisone.

C. Intravenous fluids.

D. Increased salt and water intake.

E. Indomethacin.

Incorrect. Correct answer is D


45% answered correctly

Explanation:

Correct Answer Is D

A positive table tilt test is highly suggestive of orthostatic (postural) hypotension. Orthostatic hypotension is an excessive fall in blood
pressure when an upright position is taken on. The consensus drop is >20mmHg in systolic pressure, 10mmHg in diastolic pressure or both.
The condition may be acute or chronic.

Orthostatic hypotension is a manifestation of aberrant blood pressure regulation due to various conditions, not a specific disorder. A myriad
of causes can lead to orthostatic hypotension.

The most common causes of acute orthostatic hypotension include:

Hypovolemia (e.g. blood lost)


Drugs
Prolonged bed rest
Adrenal insufficiency

The most common causes of chronic orthostatic hypotension:

Age-related changes in blood pressure regulation


Drugs
Autonomic dysfunction

The symptoms are related to diminished blood flow to central nervous system, especially the brain and include:

Faintness
Lightheadedness
Confusion
Blurred vision
Syncope, falls or even seizures may be seen in severe cases

Other symptoms may be caused by the underlying etiology of orthostatic hypotension rather than hypotension itself. The symptoms start
within seconds to a few minutes of standing and resolve rapidly after lying down.

Work-up for orthostatic hypotension starts with measuring the blood pressure and heart rate after 5 minutes in supine and at 1 and 3 minutes
after standing (or sitting upright if the patient is not able to stand).

Hypotension without a compensatory increase in heart rate (<10 bpm) is suggestive of autonomic impairment
Hypotension with compensatory increase in heart rate (>100 bpm or by >30 bpm) suggests hypovolemia
Reproduction of symptoms in the absence of hypotension and an increase of more than 30 bpm in heart rate points towards postural
orthostatic tachycardia syndrome (POTS)

POTS, also known as postural autonomic tachycardia, or chronic or idiopathic orthostatic intolerance is a syndrome of orthostatic intolerance
in younger patients. Various symptoms (e.g. fatigue, light-headedness, exercise intolerance, cognitive impairment) and tachycardia occur with
standing; however, there is little or no fall in BP. The reason for symptoms is unclear.

Table tilt test:

This test may be done when autonomic dysfunction is suspected. It gives more consistent results than supine and upright blood pressure
assessment and eliminates augmentation of venous return by leg muscle contraction.
6 of 1943
What is table tilt test and how it is interpreted?

Tilt table testing is used to evaluate syncope in younger, apparently healthy patients and, when cardiac and other tests have not provided a
diagnosis, in elderly patients.

Tilt table testing produces maximal venous pooling, which can trigger vasovagal (neurocardiogenic) syncope and reproduce the symptoms
and signs that accompany it, including nausea, light-headedness, pallor, hypotension, and bradycardia.

After an overnight fast, a patient is placed on a motorized table with a foot board at one end and is held in place by a single strap over the
stomach; an IV line is inserted. After the patient remains supine for 15 min, the table is tilted nearly upright to 60 to 80° for 45 minutes. If
vasovagal symptoms develop, vasovagal syncope is confirmed. If they do not occur, a drug (eg, isoproterenol) may be given to induce them.
Sensitivity varies from 30 to 80% depending on the protocol used. The false-positive rate is 10 to 15%.

With vasovagal syncope, heart rate and BP usually decrease. Some patients have only a decrease in heart rate (cardioinhibitory); others have
only a decrease in BP (vasodepressor). Other responses include a gradual decrease in systolic and diastolic BP with little change in heart rate
(dysautonomic pattern), significant increase in heart rate (> 30 beats/min) with little change in BP (postural orthostatic tachycardia
syndrome), and report of syncope with no hemodynamic changes (psychogenic syncope).

Relative contraindications to the test include:

Severe aortic or mitral stenosis


Hypertrophic cardiomyopathy
Severe coronary artery disease (CAD)

In particular, isoproterenol should not be used in patients with hypertrophic cardiomyopathy or severe CAD.

Conservative management

Conservative measures should always be considered as the first-line management. These measures include increased sodium and water
intake in the absence of heart failure or hypertension. This may expand intravascular volume and decreases the severity of symptoms. This
approach carries the risk of heart failure, particularly in the elderly and patients with impaired myocardial function.

NOTE - development of dependent edema without heart failure is not a reason to stop the treatment.

Other conservative measures include:

Patients requiring prolonged bed rest should be advised to sit up first on wakning and exercise in bed when possible.

Elderly patients should avoid prolonged standing. Sleeping with the head of the bed raised may relieve symptoms by promoting Na
retention and reducing nocturnal diuresis.

Patients should rise slowly from a recumbent or sitting position, consume adequate fluids, limit or avoid alcohol, and exercise
regularly when possible, because modest-intensity exercise promotes overall vascular tone and reduces venous pooling.

If the case is postprandial hypotension, it should be recommended that the size and carbohydrate content of meals be reduced. The
patient must minimize alcohol intake and avoid sudden standing after meals.

Waist-high fitted elastic hose may increase venous return, cardiac output, and BP after standing.

Medical management

If conservative measures fail, the following medications may be uses to treat orthostatic hypotension:

Fludrocortisone – being a potent mineralocorticoid, fludrocortisone exerts it effects through sodium retention, which results in volume
expansion and relieving or decreasing the symptoms. It is only effective if sodium intake is adequate.

This drug may also improve the peripheral vasoconstrictor response to sympathetic stimulation. Supine hypertension, heart failure, and
hypokalemia may occur; K supplements may be required.

Other drugs used include midodrine, NSAIDs, L-Dihydroxyphenylserine (a norepinephrine precursor), and propranolol or other beta blockers.

For this patient with no heart fialure, increased water and sodium intake will be the next best step in management.

References

• http://www.merckmanuals.com/professional/cardiovas

• http://www.merckmanuals.com/professional/cardiovas

7 of 1943
Last updated:
Time spent: QID:136
2023-2-12

8 of 1943
John, 35 years old, presents to the emergency department with pain and swelling of his left thigh since this morning. Investigations establish
the diagnosis of deep venous thrombosis for which he is started on heparin in hospital. He has diabetes and hypertension and his wife
mentions that is very busy and distracted and always forgets to take the drugs he is prescribed for treatment of his hypertension and
diabetes. Which one of the following options would be the most appropriate management for him after the course of heparin is completed?

A. No more treatment is needed.

B. Warfarin for 6 months.

C. Aspirin for 6 months.

D. Surgical intervention.

E. Caval filter.

Incorrect. Correct answer is E


45% answered correctly

Explanation:

Correct Answer Is E

Treatment of DVT starts with either unfractionated or low molecular weight heparin. Warfarin could be started at the same day (or within 48
hours). Heparin therapy should be continued for 5 days and stopped once INR is above 2 in two consecutive days.

NOTE – Since anticoagulation is contraindicated in the presence of a bleeding diathesis, the following tests should be performed prior to
heparinization:

Activated partial thromboplastin time (APTT)


International normalised ratio (INR)
Platelet count
Thrombophilia screen including: activated protein C resistance, fasting plasma homocysteine, prothrombin G20210A, antithrombin III,
protein C, protein S, lupus anticoagulant, anticardiolipin antibody and lupus anticoagulant

Warfarin should be continued for at least 3 months or more depending on the patient’s risk of recurrent venous thromboembolism.

The objectives of anticoagulation therapy are treating the current DVT and prevention of pulmonary embolism. Studies have shown that as
many as 33% of patients may develop PE while receiving adequate anticoagulation therapy.

Cava filters are an alternative to systemic anticoagulation with warfarin (or heparin) in the following situation:

DVT or PE in patients with contraindications to anticoagulation therapy; these patients include those with:

Hemorrhagic stroke
Recent neurosurgical procedure or other major surgery
Major or multiple trauma
Active internal bleeding (e.g. upper or lower gastrointestinal bleeding, hematuria, hemobilia)
Intracranial neoplasm (either primary or metastatic)
Bleeding diathesis (e.g. secondary thrombocytopenia, idiopathic thrombocytopenic purpura, hemophilia)
Pregnancy
Unsteady gate or tendency to fall (as seen in patients with previous stroke, Parkinson disease)
Poor patient compliance with medications

DVT or PE in patients with a complication of anticoagulation therapy (e.g. bleeding)

Failure of anticoagulation therapy – the patient develops venous thromboembolism while on therapeutic dose of anticoagulant.

Free-floating iliofemoral or caval thrombus


PE prophylaxis - IVC filter placement has been advocated as a means of preventing PE in patients at high risk for thromboembolic
events. Traditionally, such patients have included the following populations:

Patients with DVT who are about to undergo surgery (lower-extremity orthopedic surgery, major abdominal surgery,
neurosurgery)
Patients with chronic pulmonary hypertension and a marginal cardiopulmonary reserve
9 of 1943
Patients with cancer
Trauma patients, including those with (1) severe head injury with prolonged ventilator dependence, (2) major abdominal or
pelvic penetrating venous injury, (3) spinal cord injury with or without paralysis, (4) severe head injury with multiple lower-
extremity fractures, or (5) pelvic fracture with or without lower-extremity fractures

As this patient is known be noncompliance with his medications he should have a inferior vena cava filter for prevention of PE.

References

• http://emedicine.medscape.com/article/419796-overv

• http://www.ncbi.nlm.nih.gov/pubmed/22698970

Last updated:
Time spent: QID:145
2023-2-12

10 of 1943
A 32-year-old pregnant womam presents to the Emergency Department at 32 weeks pregnancy with right calf swelling and tenderness which
is diagnosed as deep vein thrombosis (DVT). Which one of the following is the most appropriate management?

A. Therapeutic dose of low molecular weight heparin for 3 months.

B. Prophylactic dose of low molecular weight heparin for 3 months.

C. No treatment is needed.

D. Termination of pregnancy.

E. Start her on warfarin and continue for 6 months with strict INR monitoring.

Correct
45% answered correctly

Explanation:

Correct Answer Is A

Current guidelines recommend that pregnant women with deep vein thrombosis (DVT)) should be started on therapeutic dose of
subcutaneous low molecular weight heparin (LMWH) for the remainder of their pregnancy and at least 6 weeks postpartum. It must be
ensured that a minimum of 3 months (12 weeks) of treatment with therapeutic dose is provided.

In patient with PE, whether pregnant or not, unfractionated heparin (UH) is the initial choice which is then followed by LMWH once the patient
is hemodynamically stable. Proximal DVT and PE require extended duration of treatment (usually a minimum of 6 months.)

This patient has developed DVT at 32 weeks of her pregnancy. She needs to be started on therapeutic dose of LMWH for the rest of the
pregnancy (approximately 8 weeks) and 6 weeks postpartum which in total adds up to 14 weeks of treatment after which she may be
consider for prophylaxis with prophylactic dose of heparin for another 3 months.

Warfarin is contraindicated in pregnancy due to risk of warfarin embryopathy. It occurs in approximately 5% of the fetuses exposed to warfarin
between 6-13 weeks gestation.

Warfarin embryopathy (fetal warfarin syndrome, aka DiSala syndrome) is associated with:

Hypoplasia of the nasal bridge, depressed nasal bridge and deep groove between the nostrils and the nose tip
Laryngomalacia
Pectus carinatum
Widely spaced nipples
Congenital heart defects
Enlargement of brain ventricles
Agenesis of corpus callosum
Stippled epiphyses – stippling of unclacified epiphyses during the first year of life
Brachydactyly (shortened fingers) and mild hypoplasia of nails
Severe mental retardation
Deafness
Reduced muscle tone
Feeding difficulty
Low birth weight and growth retardation (failure to thrive)

Exposure to warfarin during second or third trimester carries an additional 5% risk of fetal intra-cerebral hemorrhage.

11 of 1943
A 76-year-old man comes to your clinic for a routine health check-up. He has blood pressure of 110/90 mmHg and a pulse rate of 92 bpm. He
is on no medications except daily multivitamins. As a part of evaluation, an ECG is obtained which is shown in the following photograph.
Which one the following is the next best step in management?

A. Aspirin.

B. Warfarin.

C. Reassurance.

D. Metoprolol.

E. Atropine.

Incorrect. Correct answer is C


45% answered correctly

Explanation:

Correct Answer Is C

The ECG shows a sinus rhythm with increased PR interval characteristic of first-degree atrioventricular block. The condition can be caused by:

Age-related fibrosis and degeneration of AV node – the most common cause


Drugs: digoxin, beta blockers, calcium channel blockers
Increased vagal tone.

First-degree heart block is characterized by PR interval> 200 ms (0.2 s) on ECG, normal QRS complexes in terms of duration and spacing, and
the presence of a P wave before each QRS.

This is quite common among the elderly due to age-related fibrotic changes of the cardiac conductive system. Asymptomatic patients do not
need treatment and must be reassured. With symptoms (e.g., dizziness, shortness of breath, chest pain, etc.) atropine or pacemaker (if
unresponsive to atropine) is considered.

References

• Therapeutic Guidelines – Cardiovascular

Last updated:
Time spent: QID:222
2023-2-12

12 of 1943
A 79-year-old man collapses on the floor while waiting in the Emergency Department and becomes unresponsive. He has a blood pressure of
84/47 mmHg and rapid and barely perceptible pulse. Cardiopulmonary resuscitation is started immediately. The rhythm, obtained by
defibrillator is shown in the accompanying photograph. Which one of the following is the next best step in management?

A. Continue CPR until the patient regains consciousness.

B. Cardioversion.

C. Amiodarone infusion.

D. Intravenous adrenaline.

E. Defibrillation.

Incorrect. Correct answer is B


45% answered correctly

Explanation:

Correct Answer Is B

The first step in management of a collapsed patient is management is calling for help (if possible) and starting CPR with chest compression
and ventilation. Once the defibrillator is available obtain the cardiac rhythm with the pedals. The rhythm of this patient is characteristic of
ventricular tachycardia (VT). In patients with unstable hemodynamic status, the next best step in management is synchronized cardioversion
if the patient has a pulse, or defibrillation if no pulse is detected.

Hemodynamic instability is manifested as:

Chest pain
Dyspnea
Hypotension
Perfusion-related confusion
Collapse and/or unresponsiveness

(Option A) Continuing the CPR without defibrillation is unlikely to help in this condition.

(Option C) Amiodarone and other anti-arrhythmic drugs used for VT are indicated if the patient is hemodynamically stable.

(Option D) Intravenous adrenalin is the first step in management, in conjunction with chest compression and ventilation, in patients with
asystole, or ventricular fibrillation after two attempts of DC shock fail to convert the rhythm.

(Option E) Defibrillation is the option when there is ventricular fibrillation or if the patient has pulseless VT.

References

• Therapeutic Guidelines – Cardiovascular

Last updated:
Time spent: QID:223
2023-2-12

13 of 1943
Which one of the following is NOT a contraindication to thrombolytics use in a patient with ST elevation myocardial infarction?

A. Ischemic stroke in the past 3 months.

B. Hemorrhagic stroke in the past 10 years.

C. Heavy menstrual bleeding.

D. Coagulation disorders.

E. Gastrointestinal bleeding within the past 4 weeks.

Incorrect. Correct answer is C


45% answered correctly

Explanation:

Correct Answer Is C

Absolute contraindications for fibrinolytic use in STEMI include the following:

Prior intracranial hemorrhage (ICH)


Known structural cerebral vascular lesion
Known malignant intracranial neoplasm
Ischemic stroke within 3 months
Suspected aortic dissection
Active bleeding or bleeding diathesis (excluding menses)
Significant closed head trauma or facial trauma within 3 months
Intracranial or intraspinal surgery within 2 months
Severe uncontrolled hypertension (unresponsive to emergency therapy)
For streptokinase, prior treatment within the previous 6 months

Relative contraindications for fibrinolytic use in STEMI include the following:

History of chronic, severe, poorly controlled hypertension


Significant hypertension on presentation (systolic blood pressure >180 mm Hg or diastolic blood pressure >110 mm Hg
Traumatic or prolonged (>10 minutes) cardiopulmonary resuscitation (CPR) or major surgery less than 3 weeks previously
History of prior ischemic stroke not within the last 3 months
Dementia
Recent (within 2-4 weeks) internal bleeding
Noncompressible vascular punctures
Pregnancy
Active peptic ulcer
Current use of an anticoagulant (e.g., warfarin) that has produced an elevated international normalized ratio (INR) higher than 1.7 or a
prothrombin time (PT) longer than 15 seconds

Of the given options, only heavy menstrul bleeding is not an absolute contraindication to thrombolytic therapy.

References

• Medscape - Thrombolytic therapy - Thrombolytic Therapy for Acute Myocardial Infarction

• Queensland Government - Thrombolysis for STEMI Clinical Pathway

Last updated:
Time spent: QID:224
2023-2-12

14 of 1943
A 70-year-old man presents to the Emergency Department with complaint of chest pain starting 15 minutes ago, central in location, and dull
and aching in nature. He is given aspirin, sublingual glyceryl trinitrate, and oral antacid, and is put on supplemental oxygen by nasal cannula.
These measures ameliorate the pain to a significant extent.

On examination, his blood pressure is140/90mmHg, pulse 110 bpm, and respiratory rate 20 breaths per minute. A 12-lead ECG strip reveals no
abnormality. He mentions that he has had these pains every time he exceeded a certain amount of physical activity, and that each time the
pain subsides with rest or sublingual glyceryl trinitrate. He rates this current episode no more than the previous ones. Which one of the
following is the most appropriate next step in management?

A. Reassure and discharge him home.

B. Book for an outpatient echocardiography.

C. Admit him to coronary care unit (CCU), measure cardiac enzymes and repeat the ECG.

D. Refer him for a stress ECG and echocardiography.

E. Refer him to a gastroenterologist.

Incorrect. Correct answer is D


45% answered correctly

Explanation:

Correct Answer Is D

Central chest pain described as heaviness, dull and aching that may or may not radiate to the jaw, left arm, or epigastrium is more likely to be
ischemic in nature. The duration of pain (less than 20 minutes), being brought on by a predictable amount of exertion and relieved by rest or
nitrates is characteristic of stable angina, a condition in which an increased oxygen demand of the heart, induced by activity, leads to
ischemia because the stenotic coronary arteries cannot keep up with increased need for oxygen. On the other hand, unstable angina is
defined as any new ischemic chest pain, or one with deviation from the typical pattern of previous pains, in terms of either duration, intensity,
frequency, or decreased amount of exertion required for its reproduction.

The characteristics of the pain (duration, reversibility, and response to rest and nitrates), in addition to a normal ECG establishes the diagnosis
of stable angina in this patient. In approaching to such patients, the next best step in management is urgently performing a stress test (either
conventional treadmill, chemical, or nucleic) to establish the coronary artery stenosis if the ECG or diagnosis is equivocal (as is in this
patient). A positive stress test (reproduction of the chest pain, ST segment depression>2mm or a drop in blood pressure>10mmHg) is then
followed by angiography for further evaluation and treatment with either ballooning with or without stent placing or coronary artery bypass
grafting surgery.

(Option A) Reassuring and discharging the patient is not an appropriate action before the patient has been fully assessed.

(Option B) Echocardiography may be a part of plan now or later on in outpatient setting, but is not the most appropriate management now.

(Option C) Admission to CCU, cardiac enzymes and follow-up ECGs were indicated if the patient had any changes in the intensity of the pain
or its duration, unresponsiveness to rest and nitrates, or aggravating or relieving factors.

(Option E) Referring the patient to gastroenterologist is not correct because the pain is typical for cardiac ischemia.

15 of 1943
A 50-year-old man ipresents to the Emergency Department with chest pain felt behind the sternum radiating to his jaw. A 12-lead ECG strip is
obtained and is as the following photograph. You give him aspirin and sublingual nitroglycerine and start him on supplemental oxygen by
nasal cannula. A troponin level is ordered which comes back negative. Which one of the following is the next best step in management of this
patient?

A. Repeat troponin in 8 hours.

B. Immediate reperfusion therapy.

C. Repeat the ECG in 6 hours.

D. Start him on beta blockers.

E. Serial ECGs.

Incorrect. Correct answer is B


45% answered correctly

Explanation:

Correct Answer Is B

With ST elevation in leads II, III, and aVF on the given ECG, this patient has sustained an inferior ST-elevation myocardial infarction (STEMI) for
which the next best step in management is either immediate percutaneous coronary intervention (PCI) as the preferred option, or thrombolytic
therapy if not contraindicated.

(Option A) Even with negative troponin, ST elevation of more than 1mm in two or more contiguous leads and chest pain makes the diagnosis
certain. In fact with chest pain and ECG changes, no troponin level was required to guide the management, and reperfusion therapy should
have been performed even without waiting for the results.

(Option C) This patient has acute inferior MI and should be treated immediately. Waiting for 6 hours to obtain an ECG is definitely an incorrect
answer.

(Options D and E) Serial ECGs are indicated as well to further assess the possible evolution of the myocardial infarction. Beta blockers are
effective in reducing the mortality, but neither ECG, nor beta blockers takes precedence over PCI.

References

• Australian Prescriber - Management of acute coronary syndromes

• Medscape - Acute coronary syndrome

Last updated:
Time spent: QID:225
2023-2-12

16 of 1943
A 60-year-old man presents with complaints of increasing tiredness and abdominal distention for the past four months. His past medical
history is remarkable for smoking 20 cigarettes a day for the past 20 years. On examination, there is bilateral ankle edema and ascites. The
liver is palpated 3cm below the costal margin. His jugular pulse is noted to drop on expiration and rise on inspiration. Which one of the
following is the most likely diagnosis?

A. Cardiac tamponade.

B. Budd- Chiari syndrome.

C. Superior vena cava obstruction.

D. Constrictive pericarditis due to TB in the past.

E. Hepatic cirrhosis.

Incorrect. Correct answer is D


45% answered correctly

Explanation:

Correct Answer Is D

Drop of jugular pulse on inspiration and its rising during expiration is a normal physiologic response. Dropped jugular venous pressure (JVP)
during expiration and its rise on inspiration is a pathological sign called Kussmal sign. Kussmal sign is seen in restrictive cardiomyopathy,
constrictive pericarditis, and cardiac tamponade. Of the options, only constrictive pericarditis due to TB infection can present with Kussmal
sign.

(Option C) Superior vena cava (SVC) obstruction leads to edema of the face not ankle edema. On the other hand, although the JVP is raised in
SVC obstruction, there is no pulsation of the jugular vein.

Budd-Chiari syndrome (option B) is associated with thrombus formation in the hepatic vein, leading to portal hypertension. It may present
with fatigue, right upper quadrant pain, mild jaundice, and hepatosplenomegaly; however, JVP remains normal, without pulsation.

In hepatic cirrhosis (option E), the liver is usually shrinked and not enlarged. Although fatigue, edema and ascites are commn findign, the
Kussmal sign is not a feature.

References

• http://emedicine.medscape.com/article/157096-clini

Last updated:
Time spent: QID:226
2023-2-12

17 of 1943
A 30-year-old man presents to the emergency department with chest pain that has started this morning and worsened over time. He mentions
that deep breathing increases the pain intensity. On examination, pleuretic chest pain and a temperature of 38°C is noted. The BP is
140/85mmHg and the pulse 100bpm. He takes shallow breaths in a rate of 20/min. The rest of physical examination is inconclusive. A 12-
lead ECG is obtained and is shown. Which one of the following is the most likely diagnosis?

A. Myocardial ischemia.

B. Pulmonary embolism.

C. Infectious endocarditis.

D. Acute myocardial infarction.

E. Pericarditis.

Incorrect. Correct answer is E


45% answered correctly

Explanation:

Correct Answer Is E

The clinical picture of pleuretic chest pain (chest pain worsened with breathing), mildly elevated respiratory rate and a borderline pulse rate
can be either to pericarditis or pulmonary embolism, but diffuse ST elevation in pericordial and limb leads favors pericarditis as the most likely
diagnosis.

(Option A) Although myocardial ischemia causes chest pain, the nature of the pain and the pattern of ST segment elevation makes this
diagnosis less likely.

(Option B) Pulmonary embolism can present similarly; however, the ECG changes are not consistent with this diagnosis.

(Option C) Infectious endocarditis presents with a murmr and fever. The ST changes in the ECG are characterisitc for pericarditis. Infectious
endocarditis does not cause such an ECG abnormality.

(Option D) Acute myocardial infarction (MI) causes ST elevation in specific lead groups, depending on the coronary artery involved and its
territory. The pleuretic nature of the chest pain, on the other hand, is against MI as a likely diagnosis.

References

• http://www.racgp.org.au/download/documents/AFP/201

• http://www.australiandoctor.com.au/cmspages/getfil

Last updated:
Time spent: QID:227
2023-2-12

18 of 1943
A 70-year-old man in brought to the Emergency Department because of light-headedness for the past 4 hours. On examination, he is found to
have bradycardia with an irregular pulse of 45 bpm and a blood pressure of 85/60 mmHg. Atropine is used as the treatment of symptomatic
bradycardia but the pulse rate remains the same and the lightheadedness persists. An ECG strip is obtained which is shown in the following
photograph. Which one of the following is the next best step in management?

A. Metoprolol.

B. Dopamine.

C. Intravenous pacemaker.

D. Permanent pacemaker.

E. Adrenaline.

Incorrect. Correct answer is C


45% answered correctly

Explanation:

Correct Answer Is C

The rather constant PR intervals, and ‘p’ waves that are not followed by a QRS complex seen on ECG is characteristic of Mobitz II AV block.

Symptomatic AV block should be initially be treated with temporary pacing. Percutaneous or intravenous pacemakers are often available in
the emergency department and can be applied. If not, atropine is used instead.

In the following situations temporary pace makers are the most appropriate option for the initial management:

History of asystole
Mobitz II AV block
Complete heart block
Ventricular standstill>3 seconds

(Option A) Metoprolol is definitely the wrong option because using it in this situation can dramatically worsen the condition.

(Option B) Dopamine is a drug used for increasing cardiac contractility and vascular tone and is not used in treatment of symptomatic
bradycardias.

(Option D) Permanent pacemakers are definitive treatment of such blocks and are considered after initial temporary pacing.

(Option E) Adrenaline infusion is sometimes indicated to maintain an adequate heart rate after atropine, while waiting for pacemaker
insertion.

TOPIC REVIEW

Second-degree (Mobitz) heart block

Click here, for a useful video explaining this type of heart block and its ECG characteristics.

The typical ECG findings in Mobitz I (Wenckebach) AV block—the most common form of second-degree AV block—are as follows:

Gradually progressive PR interval prolongation occurs before the blocked sinus impulse
The greatest PR increment typically occurs between the first and second beats of a cycle, gradually decreasing in subsequent beats
Shortening of the PR interval occurs after the blocked sinus impulse, provided that the P wave is conducted to the ventricle
Junctional escape beats may occur along with nonconducted P waves
A pause occurs after the blocked P wave that is less than the sum of the 2 beats before the block

19 of 1943
During very long sequences (typically >6:5), PR-interval prolongation may be inapparent and minimal until the last beat of the cycle,
when it abruptly becomes much greater
Post-block PR-interval shortening remains the cornerstone of the diagnosis of Mobitz I block, regardless of whether the periodicity
has typical or atypical features
R-R intervals shorten as PR intervals become longer
The typical ECG findings in Mobitz II AV block are as follows:
Consecutively conducted beats with the same PR interval are followed by a blocked sinus P wave
PR interval in the first beat after the block is similar to the PR interval before the AV block
A pause encompassing the blocked P wave is equal to exactly twice the sinus cycle length

The typical ECG findings in Mobitz II AV block are as follows:

Consecutively conducted beats with the same PR interval are followed by a blocked sinus P wave
PR interval in the first beat after the block is similar to the PR interval before the AV block
A pause encompassing the blocked P wave is equal to exactly twice the sinus cycle length

References

• Therapeutic Guidelines – Cardiovascular

• Medscape - Second-Degree Atrioventricular Block

Last updated:
Time spent: QID:229
2023-2-12

20 of 1943
A 47-year-old man has developed central chest pain one hour ago. On examination in the Emergency Department he is sweating profusely,
has a BP of 90/60 mmHg and a pulse rate of 50bpm. An ECG shows 2-mm ST elevation in leads II, III, and aVF. Which one of the following is
the most common cause of death in pre-hospital setting in this condition?

A. Ventricular tachycardia.

B. Ventricular fibrillation.

C. Bradycardia.

D. Asystole.

E. Hypotension.

Incorrect. Correct answer is B


45% answered correctly

Explanation:

Correct Answer Is B

The clinical and ECG findings are characteristic of inferior ST-elevation myocardial infarction (STEMI).

Of all patients experiencing acute myocardial infarction (MI), usually in the form of ST-elevation MI, 25–35% will die of sudden cardiac death
(SCD) before receiving medical attention, most often from ventricular fibrillation; however, ventricular tachycardia is the most common
arrhythmia early in the course of MI.

Patients suspected of having STEMI should be connected to defibrillator on the way to the hospital. Most ventricular fibrillations occur in the
first 24 hour post-MI, with a half occurring within the first hour.

References

• Research Gate - The clinical challenge of preventing sudden cardiac death immediately after acute ST-elevation myocardial infarction

Last updated:
Time spent: QID:230
2023-2-12

21 of 1943
A 59-year-old man in brought to the emergency department of a tertiary hospital with compressing chest pain that has started 30 minutes
ago.The pain is central in location and radiates to his jaw and left arm. A 12-lead ECG is obtained and is as follows. Which one of the following
is the most appropriate management of this patient?

A. Fibrinolytic therapy.

B. Morphine.

C. Aspirin.

D. Cardiac catheterization.

E. Rescue angioplasty.

Incorrect. Correct answer is D


45% answered correctly

Explanation:

Correct Answer Is D

The history is highly suggestive of angina pectoris. On ECG, there is ST elevation in leads I, aVl, V5 and V6 (lateral aspect of the heart) and ST
elevation in V3 and V4 indicative of involvement of the anterior wall. The ECG is characteristic of an anterolateral ST-elevation myocardial
infarction (STEMI).

The most appropriate management of patients with STEMI is emergency reperfusion therapy either by fibrinolytics or percutaneous coronary
intervention (PCI) if the presentation is within 12 hours after the onset of chest pain. Since the patient is in a tertiary hospital, PCI is the
preferred method of reperfusion therapy.

Provided that proper facilities and an experienced cardiac interventionist is available a PCI could be performed with 90 minutes after
presentation. Cardiac catheterization is a general term that includes angioplasty, PCI, and balloon angioplasty.

(Option A) Fibrinolytics are the preferred method if PCI cannot be performed and the patient has no major risk of bleeding.

(Options B and C) Aspirin and morphine have to be administered for every patient with acute coronary syndrome as important steps but they
are the primary steps and can be given while the patient is arranged to be transferred to catheterization laboratory. Aspirin could be the
correct answer if the question asked about the next step.

(Option E) Rescue angioplasty is a term used to describe emergency angiographic coronary intervention after fibrinolytics fail to control the
ischemia.

References

• MJA - Guidelines for the management of acute coronary syndromes

• Therapeutic Guidelines – Cardiovascular

• Medscape - Acute Coronary Syndrome

Last updated:
Time spent: QID:231
2023-2-12

22 of 1943
Steven is a 65-year-old man, who is a known case of congestive heart failure. He had been stable on enalapril, metoprolol and digoxin until
three weeks ago when his wife passed away from breast cancer. Since then, he stopped taking his medication. Today, he is brought to the
emergency department by his son, with complaints of shortness of breath, night coughs and ankle edema. On examination, he has a blood
pressure of 90/75mmHg, heart rate of 68 bpm, and respiratory rate of 26 breaths per minute. On auscultation, an S3 gallop is noted, but there
is no crackle. You decide to start him on medication again. Which one of the following would be the best option to start with?

A. Start him on enalapril.

B. Start him on metoprolol.

C. Start him on digoxin.

D. Start him on enalapril, digoxin and metoprolol at the same time.

E. Start him on enalapril and metoprolol.

Correct
45% answered correctly

Explanation:

Correct Answer Is A

The case describes a patient, with congestive cardiac failure, who has been under control with three medications, but his heart condition has
been decompensated due to drug withdrawal.

At this moment, and based on physical findings, the patient has not pulmonary edema (no comments on basal crackles).

Angiotensin converting enzyme (ACE) inhibitors (e.g. enalapril) improve prognosis in all patients in all grades of heart failure and should be
used as initial therapy in all patients. Angiotensin II receptor blockers (ARBs) such as losartan are used when ACE inhibitors cannot be
tolerated .

Diuretics are added to ACE inhibitors to help control congestive symptoms and signs.

Beta blockers should only be started when the patient is stable and euvolemic. As this patient has S3 gallop and ankle edema (signs of
hypervolemia), any options suggesting beta blockers (e.g. metoprolol) as initial treatment would be incorrect.

Digoxin is used in patients with heart failure if:

Heart failure is caused by atrial fibrillation (often in conjuction with beta blockers if the patient is euvolemic)
In patients with heart failure, who have sinus rhythm, but medications such as ACE inhibitors, diuretics have not adequately controlled
their symptoms
In patients with significantly decreased ejection fraction (EF) (<35%)

References

• Therapeutic Guidelines – Cardiovascular; available on http://www.tg.org.au

Last updated:
Time spent: QID:233
2023-2-12

23 of 1943
Which one of the following drug groups is the mainstay of therapy in diastolic heart failure?

A. ACE inhibitors and beta blockers.

B. Beta blockers and calcium channel blockers.

C. Calcium channel blockers and diuretics.

D. Angiotensin receptor blockers and beta blockers.

E. Beta blockers and diuretics.

Incorrect. Correct answer is B


45% answered correctly

Explanation:

Correct Answer Is B

Diastolic dysfunction is defined as decreased compliance of the left ventricle during the diastole, resulting in decreased cardiac output. A
hypertrophied left ventricle due to chronic hypertension remains the leading cause of diastolic dysfunction. The disease is most common
among the elderly women. Other causes of diastolic dysfunction include restrictive cardiomyopathy and constrictive pericarditis. Pericardial
tamponade causes acute diastolic dysfunction. Ejection fraction essentially remains unaffected or even increases.

Mainstay of therapy in the diastolic dysfunction, also termed as ‘heart failure with preserved systolic function (HFPSF)’ largely depends on the
underlying cause. Mainstay of pharmacotherapy is with cardiac selective beta blockers (e.g. atenolol, carvedilol, metoprolol) and/or calcium
channel blockers. Beta blockers, by slowing the heart rate, give ventricles more time to fill and enhance cardiac output. Calcium channel
blockers serve the same purpose and can be used as second-line treatment or added to beta blockers later in the course of treatment.

The following drugs should not be used in treatment of diastolic dysfunction:

Diuretics – they may decrease the already prone-to-decrease cardiac output. They are indicated if there is pulmonary congestion and
edema.
Vasodilators (e.g. nitrates) – they decrease the venous return and decrease cardiac output.
Arterial vasodilators (hydralazine) – they may cause dynamic obstruction against the left ventricle outflow.
Digoxin and other inotropic drugs – there is no role for these drugs as the ejection fraction is normal or even elevated. Digoxin may be
considered for co-existing atrial fibrillation (but not as the first-line therapy).

(Option A) No clear evidence exists to support ACE inhibitor therapy for diastolic heart failure. There has been no direct improvement in
overall morbidity and mortality in patients with diastolic heart failure associated with ACE inhibitors; however, ACE inhibitors may have an
important role in the treatment of the diseases underlying diastolic heart failure. Moreover, patients with diastolic heart failure frequently have
comorbidities such as renal insufficiency and care should be taken when using ACE inhibitors, as there is the risk of renal function
deterioration.

(Option C) Calcium channel blockers are useful for treatment of diastolic heart failure, but diuretics decrease the cadiac output and should be
avoided, unless there is volume overload.

(Option D) Beta blokcers are mainstay of therapy in patients with diastolic heart failure, but there is no clear evidence from randomized
clinical trials that treatment with angiotensin receptor blockers directly improves overall morbidity or mortality, or diastolic function in patients
with diastolic heart failure diastolic heart failure.

(Option E) Diuretics should not be used in patients with diastolic dysfunction, unless in the presence of volume overload and congestion.

References

• Therapeutic Guidelines – Cardiovascular; available from http://tg.org.au

• http://www.uptodate.com/contents/treatment-and-pro

Last updated:
Time spent: QID:234
2023-2-12

24 of 1943
An 17-year-old girl sustained sudden loss of consciousness and collapsed while playing basketball at school. She is now in the emergency
department. She mentions that her father died suddenly at the age of 37 years. Which one of the following investigations is most likely to
establish the diagnosis of the underlying cause of her collapse?

A. ECG.

B. Cardiac biomarkers.

C. Angiography.

D. Chest X-ray.

E. Echocardiography.

Incorrect. Correct answer is E


45% answered correctly

Explanation:

Correct Answer Is E

Sudden loss of consciousness in a young person while on exertion is most likely due to either an arrhythmia or hypertrophic obstructive
cardiomyopathy (HOCM). With a positive family history for sudden death at young age, HOCM would be considered as the most likely
diagnosis.

CXR, ECG and electrocardiography are all part of the work-up for this patient, but among them, echocardiography is more likely to establish
the diagnosis of HOCM. Echocardiography is the best tool for detecting structural cardiac abnormalities (e.g. valvular lesion,
cardiomyopathies).

References

• http://www.csanz.edu.au/wp-content/uploads/2014/01

Last updated:
Time spent: QID:235
2023-2-12

25 of 1943
Patients with a variety of cardiac conditions may require special care in pregnancy. Which one of the following cardiac lesions is mots likely to
cause problems during pregnancy?

A. Mitral stenosis.

B. Tricuspid regurgitation.

C. Mitral regurgitation.

D. Aortic regurgitation.

E. Ventricular septal defect.

Correct
45% answered correctly

Explanation:

Correct Answer Is A

Among given options, mitral stenosis (MS) is more likely to complicate a pregnancy. Plasma volume is increased by 50% during pregnancy
leading to more pronounced left atrial congestion and backflow of blood to the lungs resulting in pulmonary congestion and its typical
presentation (fatigue, dyspnea, orthopnea, paroxysmal nocturnal dyspnea, pulmonary oedema, etc).

Other mentioned lesions are well tolerated during pregnancy owing to the fact that decreased peripheral vascular resistance causes
accumulation of blood in the periphery. This eliminates a part of the work load the heart should have undergone if all that blood returned to
the heart for recirculation.

References

• http://emedicine.medscape.com/article/155724-clini

Last updated:
Time spent: QID:237
2023-2-12

26 of 1943
A 70-year-old man, farmer by profession, presents to the Emergency Department after he experienced chest pain of 20 minutes duration
following physical activity. When the pain started, he took 3 puffs of glyceryl trinitrate but there was no relief. In the Emergency Department
though, the pain alleviated with oxygen. On further questioning, you realize that he is on aspirin, metoprolol and sublingual glyceryl trinitrate
spray on an 'as needed' basis. During the past 6 months, he has had only 3 episodes of chest pain subsided just by rest. He saw his GP 6
months ago when he received his last prescription. Which one of the following is the most likely explanation of failure of glyceryl trinitrate to
relieve his symptoms?

A. Onset of unstable angina.

B. Nitrate tolerance.

C. Reduced drug potency.

D. Poor absorption through the mouth.

E. Onset of myocardial infarction.

Incorrect. Correct answer is C


45% answered correctly

Explanation:

Correct Answer Is C

The given scenario is more likely due to expiration of the glyceryl trinitrate. It is important to remember to remind the patients to update their
GTN sprays or pearls. Expired GTN has reduced potency and effect on relieving the ischemic pain.

The given scenario is more likely to have occurred due to expiration of the glyceryl trinitrate. It is important to remind the patients to update
their GTN sprays or pearls. Expired GTN has reduced potency and effect on relieving the ischemic pain. Since this patient has refilled his
prescription 6 months ago, it is likely that the drug is past its expiry date and lost the desired potency.

Nitrate tolerance (option B) is a possibility and concern in patients who take GTN or other nitrates on a regular basis. After some time,
resistance to nitrates develops and the ability of pain control diminishes. In this patient, however, tolerance is very unlikely, as he is not been a
regular user of nitrates and has used them occasionally on an ‘as-needed’ basis.

In patients with unstable angina or myocardial infarction, the pain may not respond to nitrates. However, this patient is not likely to have these
because it is unlikely for unstable angina (option A) or myocardial infarction (option E) to respond to oxygen but not to nitrates.

(Option D) GTN adequately and effectively absorbs through the mouth.

27 of 1943
A 25-year-old woman comes to your practice with complaints of ‘skipped heart beats’. On examination, she has a blood pressure of 110/85
mmHg and pulse of 88 bpm. Heart auscultation is remarkable for a mid-systolic click followed by a late systolic murmur. Echocardiography
shows mitral valve prolapse (MVP). Which one of the following is correct about her condition?

A. MVP is more common in men.

B. Prophylaxis against infectious endocarditis is recommended.

C. MVP is present in up to 10% of population.

D. Risk of pulmonary embolism is high.

E. Ventricular arrhythmias do not occur.

Incorrect. Correct answer is C


45% answered correctly

Explanation:

Correct Answer Is C

Mitral valve prolapse is seen in up to 10% of population and is more common in women. Although it is asymptomatic most of the time, it can
present with palpitation and shortness of breath, or rarely even chest pain or more severe complications.

(Option A) MVP is more common in women than men.

(Option B) Antibiotic prophylaxis against endocarditis is no longer recommended for valvular lesions in the 2007 AHA guideline. Indications
for prophylaxis antibiotic therapy include bicuspid aortic valve, acquired aortic or mitral valve disease (including mitral valve prolapse with
regurgitation), and hypertrophic cardiomyopathy with latent or resting obstruction.

(Option D) If MVP leads to atrial enlargement and blood stagnation, there is an increased risk of systemic thromboembolism; however, risk of
pulmonary embolism is extremely low.

(Option E) Although very rarely, supraventricular and ventricular tachyarrhythmias are found in patients with MVP.

References

• Medscape - Mitral Valve Prolapse

Last updated:
Time spent: QID:238
2023-2-12

28 of 1943
A patient with aortic stenosis has undergone cardiac catheterization. You are told that the aortic systolic gradient is 55 mmHg. Which one of
the following additional piece of information is necessary to determine the significance of the gradient?

A. Presence of coronary artery disease.

B. Left ventricular ejection fraction.

C. Left ventricular end-diastolic pressure.

D. Cardiac output.

E. Left atrial pressure.

Incorrect. Correct answer is D


45% answered correctly

Explanation:

Correct Answer Is D

An aortic systolic gradient of 55 mmHg means that during systole, the pressure in the aorta is 55 mmHg lower than the pressure in the left
ventricle. A normal gradient is less than 10 mmHg. With aortic stenosis, the cardiac output is diminished because the stenotic valve impedes
blood outflow from the left ventricle. Therefore, cardiac output would be determinant as to whether immediate surgical valve replacement
should be considered. It should be born in mind that even in the presence of a normal cardiac output, a gradient more than 50 mmHg needs
surgical replacement of the aortic valve.

References

• AMC Handbook of Multiple Choice Questions – page 435

• Medscape - Aortic Stenosis

Last updated:
Time spent: QID:239
2023-2-12

29 of 1943
A 39-year-old woman presents with dizziness after she left the gym. An ECG is obtained and is as the accompanying photograph. Which one
of the following is the next best step in management?

A. Reassurance.

B. Intravenous atropine.

C. Transcutaneous pacemaker.

D. Intravenous pacemaker.

E. Adrenaline.

Incorrect. Correct answer is B


45% answered correctly

Explanation:

Correct Answer Is B

The rhythm strip shows sinus bradycardia with a rate of 50 bpm. Asymptomatic bradycardias are usually left alone. If symptomatic,
symptoms can include:

Pre-syncope or syncope (faint)


Shortness of breath
Lightheadedness (dizziness)
Chest pain

Every patient with symptomatic bradycardias should be managed as follows:

Medical management:

Atropine (IV 500-600mcg – repeat every 3-5 minutes up to 3mg) – the best initial management

Adrenaline (IV infusion 2-10 mcg/min) to maintain satisfactory heart rate – the second-line medical management

Pacemaker insertion

Consider pacemaker for the following patients:

Patients who do not respond adequately to medical therapy


Patients at risk of asystole:

History of recent asystole


Mobitz II AV block on ECG
3rd degree AV block on ECG
Pulse<40 bpm
Ventricular standstill> 3 seconds

The initial pacing is with percutaneous pacing in the emergency department. Intravenous pacemaker insertion needs specialist referral.
Permanent pacing is then followed electively if needed.

This patient should be given atropine as the best initial management.

References

• Therapeutic Guidelines – Cardiovascular

30 of 1943
• Medscape - Sinus Bradycardia
Last updated:
Time spent: QID:240
2023-2-12

31 of 1943
A 70-year-old man presents to the Emergency Department with chest discomfort, shortness of breath and pre-syncope. An ECG is obtained
and is as follows. Which one of the following is the most appropriate immediate management of this patient?

A. Atropine.

B. Adrenaline.

C. Transcutaneous pacemaker.

D. Synchronized DC cardioversion.

E. Intravenous fluids.

Incorrect. Correct answer is C


45% answered correctly

Explanation:

Correct Answer Is C

The ECG is suggestive of third-degree (complete) heart block. Although atropine is used as first-line therapy for symptomatic bradycardias, in
complete heart block emergency rescue transcutaneous pacemaker is the management of choice. Atropine should be given if emergency
pacing cannot be done immediately.

References

• Therapeutic Guidelines – Cardiovascular

Last updated:
Time spent: QID:241
2023-2-12

32 of 1943
A 4-year-old boy is brought to the Emergency Department after he told her mother 'my head is light and I feel like falling down'. On
examination, his blood pressure is 80/50 mmHg and he has a regular pulse rate of 60 bpm. An ECG is obtained which is shown in the
following photograph. Which one of the following is the next best step in management?

A. Give him adenosine.

B. Give him amiodarone.

C. Give him atropine.

D. Immerse his face in cold water and massage his carotid sinus.

E. Cardioversion with DC shock.

Incorrect. Correct answer is C


45% answered correctly

Explanation:

Correct Answer Is C

They ECG shows a sinus rhythm at rate of 60bpm, rather wide QRS complexes and the presence of typical ‘delta waves’ characteristic of a pre-
exciting syndrome (e.g., Wolff-Parkinson-White). Although a heart rate of 60 bpm is considered normal in adults, it definitely signifies
bradycardia in a 4-year-old child (normal 80-120bpm). Since this child has symptomatic bradycardia (light-headedness), he should be treated
initially with atropine.

The normal range of heart rate for different age group is as follow:

Age (years) Hear rate


<1 100-160
1-2 90-150
2-5 80-140
6-12 70-120
>12 60-100

(Options A and D) Adenosine or immersion of the face in cold water and carotid sinus massage are treatment options for supraventricular
tachycardias, which is not the case here.

(Option B) Amiodarone is used for ventricular tachyarrhythmias; this child does not have tachycardia.

(Option E) Cardioversion is the treatment of choice for tachyarrhythmias that have resulted in hemodynamic instability, namely chest pain,
shortness of breath, hypotension or hypoperfusion-related confusional state.

References

• Therapeutic Guidelines – Cardiovascular

Last updated:
Time spent: QID:242
2023-2-12

33 of 1943
A 28-year-old man has had several episodes of palpitations associated with mild dizziness and sweating over the past year. These attacks
occurr usually after exercise and last between 10 and 20 minutes, and often stop after he immerse his face in cold water. He now presents
with an attack that has been going on for 2 hours. His ECG is shown in the following picture. Which one of the following medications is most
likely to be effective?

A. Digoxin.

B. Quinidine.

C. Flecainide.

D. Verapamil.

E. Propranolol.

Incorrect. Correct answer is D


45% answered correctly

Explanation:

Correct Answer Is D

The rhythm strip shows paroxysmal supraventricular tachycardia (PSVT) at a rate of almost 180 bpm. The mechanism of PSVTs is either a re-
entry circuit or automatous focus involving the atria. The best initial step in management of PSVTs is performing maneuvers that suppress
conductivity in atrioventricular (AV) node such as Valsalva maneuvers, immersion of the face in cold water, etc. If non-medical measures
failed, preferred medical management is with intravenous adenosine (preferred) or verapamil as the first-line medications.

(Option A) Digoxin is no more recommended for management of PSVT.

(Options B and C) Quinidine and flecainide are antiarrhythmic drugs, but not used for management of PSVT.

(Option E) Beta blockers such as metoprolol, propranolol and atenolol are second-line drugs.

References

• AMC Handbook of Multiple Choice Questions – page 433

• Therapeutic Guidelines – Cardiovascular

Last updated:
Time spent: QID:244
2023-2-12

34 of 1943
A 78-year-old man presents to your clinic with a 3-week history of palpitation. On examination, he has a blood pressure of 135/80 mmHg and
heart rate of 115 bpm and irregular. ECG reveals atrial fibrillation (AF). An echocardiography is arranged that shows an ejection fraction (EF)
of 38%. Which one of the following is the most appropriate treatment option for him?

A. Electrical cardioversion.

B. Metoprolol.

C. Digoxin.

D. Flecainide.

E. Verapamil.

Incorrect. Correct answer is B


45% answered correctly

Explanation:

Correct Answer Is B

The main goals of treatment of patients with atrial AF who also have heart failure are control of symptoms and prevention of arterial
thromboembolism. In patients with heart failure, hemodynamic consequences of AF results in decreased exercise capacity and the
decompensation of heart failure.

Similar to the general population either rate control or rhythm control are mainstay of therapy. Previously, rhythm control was considered the
superior method for treatment of AF in patients with heart failure; however, recent data has challenged this approach. Currently, rate control is
the most appropriate initial management in patients with AF and symptoms related to rapid ventricular response. Rate control to prevent rapid
AF often leads to an improvement in symptoms in patients with heart failure. Moreover, slowing of the ventricular rate often leads to a
moderate or even marked improvement in left ventricular function. Based on current evidence, cardiac-selective beta blockers (metoprolol,
atenolol, carvedilol, bisoprolol) are preferred options to use for rate control in patients with AF and heart failure because they are also
recommended for treatment of heart failure itself. Digoxin can be used as adjunctive therapy if beta blockers fail to adequately control the
rate.

If rate control cannot be achieved with beta blockers or combination of beta blockers and digoxin, amiodarone may be effective either alone
or in combination with other rate-slowing medications.

NOTE – Initiation or increase of beta blocker is contraindicated in patients with decompensated heart failure. If such patients need rate,
digoxin is the recommended agent; however, digoxin often fails to control the rate when used alone, especially in patients with elevated
sympathetic tone (hypertension, tachycardia, etc.).

Beta blockers are contraindicated if any of the following is present:

Heart rate <60 bpm


Symptomatic hypotension
Greater than minimal evidence of fluid retention: increased jugular venous pressure, pulmonary crackles indicative of interstitial
pulmonary edema, presence of S3, S4 or both, or a new or changed murmur
Signs of peripheral hypoperfusion
PR interval>0.24seconds
Second- or third-degree AV block
History of asthma or reactive airways
Peripheral artery disease with resting limb ischemia

NOTE - The non-dihydropyridine calcium channel blockers verapamil and diltiazem are not usually recommended due to risk of an
exacerbation of heart failure. If their use is considered, extreme caution is required.

This patient has both AF and heart failure; however, there is no finding in history or on examination to indicate that his heart failure is
decompensated. On the other hand, he has no contraindication to beta blockers. A reduced ejection fraction of 38% alone is not a
contraindication for beta blockers. In fact, beta blockers, by reducing heart rate and cardiac oxygen demand, will add to benefits of their use in
this patient for rate control as the most appropriate management option.

(Option A) Electrical cardioversion is used as an initial emergency treatment only if any of the 4 following conditions are present:

Active ischemia (symptomatic or electrocardiographic evidence)


Evidence of organ hypoperfusion
Severe manifestations of heart failure including pulmonary edema

35 of 1943
The presence of a pre-excitation syndrome that may lead to an extremely rapid ventricular rate due to the presence of an accessory
pathway

This patient has none of the above conditions to necessitate such treatment.

(Option C) Digoxin is the initial management of patients with AF and decompensated heart failure. This patient’s heart failure is not
decompensated. Even so, digoxin is not likely to slow the rate in the first hours of treatment.

(Option D) Flecainide is an option for conversion to or maintenance of sinus rhythm and is used for rhythm control. Unless the patient has
severe symptoms caused by the arrhythmia rather than the rapid rate, cardioversion should be performed on an elective basis after adequate
anticoagulation has been achieved.

(Option E) Verapamil is a non-dihydropyridine calcium channel blocker. These agents are appropriate options for rate control in the general
population but should be avoided in heart failure due to significant risk of myocardial suppression and worsening of the heart failure
associated with their use.

References

• Medscape - Management of Atrial Fibrillation in Patients With Heart Failure

• UpToDate - The management of atrial fibrillation in patients with heart failure

• AHA Journals - Atrial Fibrillation and Acute Decompensated Heart Failure

• RACGP - Management of atrial fibrillation

Last updated:
Time spent: QID:245
2023-2-12

36 of 1943
A 65-year-old man presents to the emergency department with complaints of palpitation for the past 3 days which is worse today. He is on
thyroxin 50 mcg for hypothyroidism. On examination, he has a blood pressure of 140/90 mmHg and an irregular pulse. An ECG is obtained
and is as shown in the picture. Which one of the following would be the next step in management?

A. Stop thyroxin.

B. Start him on metoprolol.

C. Arrange for measuring plasma TSH level.

D. Give him intravenous atropine.

E. Place a percutaneous pacemaker.

Incorrect. Correct answer is B


45% answered correctly

Explanation:

Correct Answer Is B

The clinical findings of palpitation and irregular pulse in conjunction with ECG establish the diagnosis of atrial fibrillation (AF). AF may be
idiopathic or due to a variety of conditions, with chronic hypertension being the most common cause. Hyperthyroidism is another etiology for
AF. For every patient with AF, TSH should be measured (option C) and an echocardiography performed, but these steps are considered once
the patient is started on appropriate treatment. For patients with symptomatic AF, treatment either with rate control (preferred) with
metoprolol or calcium channel blockers or rhythm control should be started immediately.

Considering anticoagulation for prevention of thromboembolic events is another important step for AF with more than 48-hour duration or of
unknown duration. Investigation for reversible causes of AF is considered once the patient is given appropriate initial management.

A TSH level below the normal lower limit is associated with an increased risk of AF, as is hyperthyroid state associated with excess doses of
thyroxin. Even in the presence of increased levels of T4 in the plasma due to excess thyroxin, stopping the thyroxin (option A) will not lead to
rapid improvement of symptoms.

Atropine (option D) and pacemakers (option E) are used for treatment of symptomatic bradycardia, which is not the case here.

References

• RACGP - Management of atrial fibrillation

• Medscape - Atrial Fibrillation

Last updated:
Time spent: QID:246
2023-2-12

37 of 1943
A 69-year-old man just had a syncopal episode in the background history of aortic valve stenosis. He is now in the Emergency Department.
Which one of the following is the most immediate management?

A. Electrocardiography.

B. Electrocardiography and cardiac monitoring.

C. Chest X-ray.

D. Emergency echocardiography.

E. Stress test.

Incorrect. Correct answer is B


45% answered correctly

Explanation:

Correct Answer Is B

Every patient over the age of 65 with an underlying cardiac disease is at risk of developing fatal arrhythmias such as complete heart block,
ventricular tachyarrhythmias, etc. Although aortic stenosis is frequently associated with episodes of loss of consciousness and falls due to
decreased perfusion on activity, the patient’s condition should not be merely attributed to this pathophysiology. An ECG should be obtained
and the patient placed on cardiac monitoring for detection of potential arrhythmias that might have not been present when the ECG was
taken.

(Option A) ECG alone may miss the causative arrhythmia if it is transient.

(Option C) Chest X-ray is not useful in detecting physiologic or structural heart anomalies.

(Option D) Echocardiography may be considered later for additional pieces of information but not urgently. It has no role in detecting an
arrhythmia. On the other hand, the diagnosis of aortic stenosis as a possible cause of syncope is already made.

(Option E) Stress test is not helpful for detection of an arrhythmia. It can cause myocardial ischemia in this patient due to aortic stenosis.

References

• RACGP - Dizziness and loss of consciousness

• Medscape - Syncope

Last updated:
Time spent: QID:248
2023-2-12

38 of 1943
A 62-year-old woman is seen in the ward 24 hours after a non-ST segment elevation myocardial infarction (NSTEMI). She complains of light-
headedness. The radial pulse is difficult to feel, blood pressure is 90/60 mmHg, heart sounds are muffled, and there is evidence of
biventricular cardiac failure. Which one of the following rhythm strips would be of most concern in this situation?

A.

B.

C.

D.

E.

Correct
45% answered correctly

Explanation:

Correct Answer Is A

The clinical picture of weak pulses, tachycardia and lightheadedness are suggestive of a tachyarrhythmia of ventricular origin, i.e. ventricular
tachycardia (VT) or ventricular fibrillation (VF). The latter is always associated with loss of consciousness; therefore, cannot be considered in
this woman.

The only rhythm strip consistent with the clinical picture is option A, which shows VT. VT is the most common arrhythmia occuring after
myocardial infarction (MI). They are in particular common in the first 24 hours post-MI.

References

• ACC/AHA/ESC Practice Guidelines

• Medscape - Ventricular Tachycardia

Last updated:
Time spent: QID:249
2023-2-12

39 of 1943
By definition, which one of the following best describes the condition termed as acute coronary syndrome?

A. Non-ST elevation myocardial infarction and ST elevation myocardial infarction immediately following cardiac surgery.

B. Unstable angina, non-ST elevation myocardial infarction and ST elevation myocardial infarction.

C. Stable angina, unstable angina and non-ST elevation myocardial infarction.

D. Stable angina, unstable angina and ST elevation myocardial infarction.

E. Unstable angina.

Incorrect. Correct answer is B


45% answered correctly

Explanation:

Correct Answer Is B

Acute coronary syndrome consists of unstable angina, non-ST elevation myocardial infarction and ST elevation myocardial infarction. They
all present with similar clinical symptoms.

References

• Mayo Clinic Proceedings - Acute Coronary Syndromes: Diagnosis and Management, Part I

• UpToDate - Acute coronary syndrome: Terminology and classification

Last updated:
Time spent: QID:17
2023-2-12

40 of 1943
Which one of the following is not associated with prolongation of QT interval?

A. Hypothyroidism.

B. Hypercalcemia.

C. Haloperidol.

D. Sotalol.

E. Methadone.

Incorrect. Correct answer is B


45% answered correctly

Explanation:

Correct Answer Is B

The following is a list of conditions that can cause QT prolongation:

Hereditary syndromes (rare)


Electrolyte/metabolic abnormalities including hypocalcemia, hypokalemia or hypomagnesemia
Intrinsic cardiac disease
Medications:

Antiarrhythmics: amiodarone, disopyramide, dofetilide, ibutilide, procainamide, quinidine, sotalol


Antihistamines: astemizole, terfenadine
Antibiotics: clarythromycin, erythromycin, pentamidine, sparfloxacin
Anti-malarials: chloroquine, halofantrine
Anti-psychotics: chlorpromazine, haloperidole, mesoridazine, pimozide, thioridazine
Gastrointestinal drugs: cisapride, domperidone
Opiate agonists: methadone, levomethadyl
Other: Arsenic trioxide, bepridil, droperidol, probucol

CNS disorders
Systemic illnesses
Myocardial Infarction

Hypercalcemia is associated with shortening of QT interval and does not cause QT interval prolongation.

References

• UpToDate - Acquired long QT syndrome

Last updated:
Time spent: QID:250
2023-2-12

41 of 1943
A 60-year-old man presents to the Emergency Department with complaint of vague abdominal pain. On examination, there is mild abdominal
tenderness in the epigastric area with a vaguely palpated abdominal mass. On auscultation, a bruit is heard over the mass. Which one of the
following is the next step in management?

A. Aortogram.

B. Emergency bedside ultrasonography.

C. Non-contrast abdominal CT scan.

D. Erect and supine abdominal X-rays.

E. Immediate transfer of the patient to the operating room.

Incorrect. Correct answer is B


45% answered correctly

Explanation:

Correct Answer Is B

The findings are consistent with provisional diagnosis of abdominal aortic aneurysm (AAA). Physical examination is only moderately sensitive
for diagnosis; therefore, a bedside ultrasonography for confirming the diagnosis of AAA is the next best step in
management. Ultrasonography is the preferred diagnostic method for AAAs with high accuracy.

Spiral CT scan gives visualization of the aneurysm and surrounding structures. It is only indicated prior to elective surgical repair. It is never
used to make a diagnosis of AAA. CT angiography with intravenous contrast can be used for assessment, but non-contrast CT and abdominal
X-rays are of no use to diagnose AAA.

References

• RACGP - Aortic aneurysms Screening, surveillance and referral

Last updated:
Time spent: QID:252
2023-2-12

42 of 1943
A 55-year-old man with history of smoking and hypertension has been diagnosed with a 4.2 cm abdominal aortic aneurysm (AAA). Which one
of the following is the investigation of choice for monitoring and surveillance of his AAA?

A. MRI angiography.

B. CT angiography.

C. Ultrasonography.

D. Digital subtraction angiography.

E. Non-contrast abdominal CT scan.

Incorrect. Correct answer is C


45% answered correctly

Explanation:

Correct Answer Is C

Ultrasonography is the most accurate and the best investigation for diagnosing, screening, and surveillance of aortic abdominal aneurysms.
Ultrasonography is a noninvasive and readily available test with approximately 100% sensitivity and specificity. CT angiography is performed if
elective surgical correction is planned, once the diagnosis is made by ultrasonography.

References

• RACGP - Aortic aneurysms Screening, surveillance and referral

Last updated:
Time spent: QID:254
2023-2-12

43 of 1943
A 55-year-old man presents with concerns about developing abdominal aortic aneurysm (AAA), as his father was diagnosed with the same
disease at the age of 57 years. He has smoked 20 cigarettes a day for the past 15 years. On examination, he has a blood pressure of 130/75
mmHg. Which one of the following is the best advice to give?

A. Reassure as AAA has no genetic predisposition.

B. Arrange for regular screening with ultrasonography.

C. Order a CT angiogram.

D. Start him on aspirin.

E. Start him on anti-hypertensive medications.

Incorrect. Correct answer is B


45% answered correctly

Explanation:

Correct Answer Is B

Patients with a first- degree relative with abdominal aortic aneurysm (AAA) have a 20% increased risk of developing AAA compared to normal
population; for this reason, he needs to be screened for AAA. Screening is performed with ultrasonography as the best investigation with very
high index of sensitivity and specificity.

NOTE - Advancing age is the most common risk factor for AAA. Other risk factors include male gender, family history, smoking,
hypercholesterolemia, diabetes mellitus, and chronic hypertension.

(Option A) Reassurance is inappropriate because he has increased risk of AAA due to the family history.

(Option C) CT angiogram is indicated once the surgical repair is planned.

(Option D) Aspirin is used neither for prophylaxis, nor treatment of AAA.

(Option E) Anti-hypertensive medications are of no benefit because the patient is normotensive.

References

• RACGP - Aortic aneurysms Screening, surveillance and referral

Last updated:
Time spent: QID:255
2023-2-12

44 of 1943
A 65-year-old man presents with history of two episodes of abdominal pain after meals. As a part of workup plan, an abdominal
ultrasonography is performed, which showed gallstones in the gallbladder and a 3.8 cm abdominal aortic aneurysm (AAA). Which one of the
following is the recommended management of this asymptomatic AAA?

A. Ultrasonography every 12 months.

B. CT scan every 12 months.

C. Ultrasonography every 24 months.

D. CT scan every 24 months.

E. CT scan or ultrasonography every 12 months.

Incorrect. Correct answer is C


45% answered correctly

Explanation:

Correct Answer Is C

Ultrasonography is the preferred method for screening and surveillance of asymptomatic AAA. The screening intervals depend on the size of
the AAA and are according to the following table:

AAA diameter (cm) Surveillance interval (months)


3-3.9 24
4-4.5 12
4.6-5 6
>5 3

Given the size of the AAA (3.8 cm), it should be monitored every 24 months with ultrasonography.

References

• RACGP - Aortic aneurysms Screening, surveillance and referral

Last updated:
Time spent: QID:256
2023-2-12

45 of 1943
Abdominal ultrasonography of a 68-year-old man for renal colic reveals an abdominal aortic aneurysm 4.5 cm in size as a chance finding.
Which one of the following is the most appropriate management of his aneurysm?

A. Refer him for elective surgical repair of the aneurysm.

B. Repeat sonography in 6 months.

C. Repeat sonography in 12 months.

D. Repeat sonography in 24 months.

E. Arrange for CT angiography.

Incorrect. Correct answer is C


45% answered correctly

Explanation:

Correct Answer Is C

Ultrasonography is the preferred method for screening and surveillance of asymptomatic abdominal aortic aneurysms (AAA).
Recommended screening intervals for an asymptomatic AAA depends on the size of the aneurysm and is according to the following table:

AAA diameter (cm) Surveillance interval (months)


3-3.9 24
4-4.5 12
4.6-5 6
>5 3

Given the size of the AAA (4.5 cm), it should be monitored every 12 months with ultrasonography.

References

• RACGP - Aortic aneurysms Screening, surveillance and referral

Last updated:
Time spent: QID:257
2023-2-12

46 of 1943
Which one of the following is not an indication for surgical repair of an abdominal aortic aneurysm (AAA)?

A. AAA more than 5.0 cm in a male patient.

B. AAA more than 5.0 cm in a female patient.

C. AAA growth more than 1.0 cm/year.

D. AAA with back pain.

E. AAA with distal thromboembolism.

Correct
45% answered correctly

Explanation:

Correct Answer Is A

The following are indications for surgical repair of an aortic aneurysm:

Asymptomatic abdominal aortic aneurysm greater than over 5.5 cm in men


Asymptomatic abdominal aortic aneurysm over 5.0cm in women
Thoracic aortic aneurysm over 6.0 cm
Growth of more than 1.0c m/year
Symptomatic abdominal aortic aneurysm (abdominal/back pain, distal thromboembolism)

An asymptomatic aortic aneurysm of 5.0 cm in a male is not an indication for surgical repair.

References

• RACGP - Aortic aneurysms Screening, surveillance and referral

Last updated:
Time spent: QID:258
2023-2-12

47 of 1943
A 72-year-old man comes to the emergency department complaining of epigastric pain that radiates to his lower back. While waiting for
further evaluation in the emergency department, he suddenly collapses. Physical examination shows a blood pressure of 80/50 mmHg and
pulse of 110 bpm. He is pale, cold, and sweaty. Which one of the following is the next best step in management?

A. Take the patient to the operating theatre immediately.

B. Arrange for CT angiography.

C. Non-contrast abdominal CT scan.

D. ECG.

E. Bedside ultrasonography.

Incorrect. Correct answer is E


45% answered correctly

Explanation:

Correct Answer Is E

The location of the pain and its radiation to the lower back, followed by signs of shock is consistent with a ruptured abdominal aortic
aneurysm as the most likely provisional diagnosis. Before emergent transfer of the patient for surgery (option A), vascular surgeon should be
contacted, and the diagnosis should be confirmed with bedside ultrasonography as the most readily available with acceptable sensitivity and
specificity. In the meanwhile, resuscitative measures such as intravenous access, fluid administration, and supplemental oxygen should be
taken in the emergency department. An ECG (option D) must be obtained pre-operatively but not the priority as the first step in management.

Other diagnostic measures such as CT angiography (option B) or non-contrast CT scan (option C) are unnecessary and does not change the
management plan.

References

• RACGP - Aortic aneurysms Screening, surveillance and referral

Last updated:
Time spent: QID:259
2023-2-12

48 of 1943
You are called to see a 58-year-old patient, who has presented with complaints of weakness and shortness of breath. A blood test reveals
hemoglobin of 85 g/L (120-160). As a part of the workup for finding the cause of his anemia, an abdominal CT scan is performed showing a
10.5 cm infra-renal abdominal aortic aneurysm as an incidental finding. You tell the patient that he needs surgery for treatment of his
aneurysm. He wants to know how dangerous it could be if he does not have the surgery. Which one of the following is the mortality rate of a
ruptured abdominal aortic aneurysm?

A. 10%.

B. 25%.

C. 50%.

D. 80%.

E. 100%.

Incorrect. Correct answer is D


45% answered correctly

Explanation:

Correct Answer Is D

Rupture of an AAA into the peritoneal cavity is often usually rapidly fatal, whereas, retroperitoneal rupture often stabilizes transiently providing
a window of opportunity for life-saving surgical intervention and repair. Collectively, a ruptured AAA is catastrophic and carries a mortality risk
of 80-90%.

References

• RACGP - Aortic aneurysms Screening, surveillance and referral

Last updated:
Time spent: QID:260
2023-2-12

49 of 1943
A 72-year-old man is brought to emergency department with ambulance after he sustained a sudden severe chest pain radiating to his back.
On auscultation there is a diastolic murmur best heard over the left sternal border in the second intercostal space. ECG shows ST elevation in
leads II, III and aVF. A CXR is remarkable for a widened mediastinum. Which one of the following is the most appropriate management?

A. Give alteplase.

B. Give aspirin, intravenous morphine, and beta blockers.

C. Give morphine and beta blockers and arrange for emergency trans-esophageal sonography.

D. Refer the patient to cardiology clinic.

E. Arrange for emergency angiography.

Incorrect. Correct answer is C


45% answered correctly

Explanation:

Correct Answer Is C

The clinical findings of abrupt chest pain radiating to back and a widened mediastinum on CXR are consistent with diagnosis aortic
dissection. The diastolic murmur signifies involvement of the proximal aorta resulting in aortic regurgitation and decreased blood flow to the
coronary arteries and a consequent myocardial infarction (MI). Thrombolytics (e.g. alteplase) (option A) are absolutely contraindicated when
aortic dissection is suspected.

Aspirin (option B), clopidogrel and other anticoagulation medications are contraindicated as well due to increased risk of deterioration of the
dissection, which is temporarily stable. The cornerstone of initial therapy in this situation is morphine for pain control (and its partial effect on
lowering blood pressure) and beta blockers to maintain the systolic blood pressure below 120 mmHg. Trans-esophageal sonography should
be emergently performed to confirm the diagnosis.Angiography using CT angiogram (CTA) (option E) is the preferred method if trans-
esophageal sonography is not available.

(Option C) Referring the patient to a cardiology clinic while he is in the hopsital and in urgent need for intervention is not an appropriate
option.

References

• http://www.ncbi.nlm.nih.gov/pmc/articles/PMC186140

• http://www.ecinsw.com.au/aortic-dissection

Last updated:
Time spent: QID:261
2023-2-12

50 of 1943
Which one of following is the most common cause of acute coronary syndrome?

A. Cocaine toxicity and consequent coronary artery vasospasm.

B. Acute thrombosis.

C. Chest trauma.

D. Arterial inflammation.

E. Anxiety.

Incorrect. Correct answer is B


45% answered correctly

Explanation:

Correct Answer Is B

Atherosclerotic changes of coronary arteries remain the most common cause of ischemic heart disease. The most common cause of acute
coronary syndrome is acute rupture of an atheroma followed by platelet aggregation and thrombus formation. Although other options can
lead to acute coronary syndrome, they are far less common.

References

• Medscape - Acute Coronary Syndrome

Last updated:
Time spent: QID:18
2023-2-12

51 of 1943
Which one of the following is the most common cause of acute limb ischemia?

A. Congestive heart failure.

B. Thrombosis.

C. Vasculitis.

D. Embolism.

E. Smoking.

Incorrect. Correct answer is B


45% answered correctly

Explanation:

Correct Answer Is B

Thrombotic occlusion is the most common cause of acute lower limb ischemia accounting for more than 80% of cases.

(Option D) Occlusion from embolus is not as common, partly due to decline in rheumatic heart disease and prompt management of patients
with atrial fibrillation with prophylactic anticoagulation. Atrial fibrillation accounts for two-thirds of acute limb ischemias due to embolism.

(Options A, C and E) Congestive heart failure, vasculitis and smoking are less common risk factors of acute limb ischemia.

References

• http://www.ncbi.nlm.nih.gov/pmc/articles/PMC11177

Last updated:
Time spent: QID:262
2023-2-12

52 of 1943
Which one of the following signs mandates emergency surgical intervention in acute limb ischemia?

A. Pallor.

B. Coldness.

C. Paralysis.

D. Pain at rest.

E. Pulselessness.

Incorrect. Correct answer is C


45% answered correctly

Explanation:

Correct Answer Is C

Paralysis is present when a patient with acute limb ischemia is unable to wriggle the toes or fingers. It is the most reliable sign indicating
acute limb ischemia requiring emergency surgical intervention. Paresthesia of the affected limb is another indicator of acute limb ischemia
and can be used in the absence of paralysis.

(Option A) Pallor is among the signs suggesting acute limb ischemia; however, the color of the limb may be affected by the ambient light.

(Option B) Coldness of the limb is the least reliable sign as it can be due to decreased body temperature of the patient to cold weather.

(Option D) Pain (either with exertion or at rest) and pallor can be caused by chronic ischemia as well, and are not reliable indicators of the
need for emergency surgical treatment.
(Option E) Pulselessness can be a sign of acute limb ischemia, but paralysis remains the most critical indicator.

References

• http://www.ncbi.nlm.nih.gov/pmc/articles/PMC111776

Last updated:
Time spent: QID:263
2023-2-12

53 of 1943
A 65-year-old man presents to the emergency department with sudden onset right-sided leg pain and paresthesia. On physical examination,
the distal pulses of the right leg are absent and the limb is cold. A CT angiography is arranged showing thrombotic occlusion of the right
femoral artery. Which one of the following is the next best step in management?

A. Intravenous infusion of heparin and emergency embolectomy.

B. Intravenous infusion of heparin for 24 hours and review.

C. Complete bed rest with compression bandage and leg elevation.

D. Start the patient on warfarin.

E. Start the patient on aspirin and clopidogrel.

Correct
45% answered correctly

Explanation:

Correct Answer Is A

The clinical and imaging findings are quite consistent with acute limb ischemia. Paralysis, paresthesia and compartment syndrome are
ominous signs that demand emergency surgical intervention after heparin has been started. The golden time for surgical intervention is
4 hours. Signs of ischemiaare are reversible if prompt action is taken within this period. Prolonged acute ischemia(>6 hours) leaves
irreversible and permanent deficits. Heparin cover should be maintained and warfarin started. Heparin then can be safely withheld once the
INR is 2-3. Warfarin alone is not recommended as it is pro-coagulation at the beginning.

References

• http://www.ncbi.nlm.nih.gov/pmc/articles/PMC111776

Last updated:
Time spent: QID:264
2023-2-12

54 of 1943
A 70-year-old man presents with acute pain and paralysis of the right leg diagnosed to have been caused by acute leg ischemia. Heparin is
started immediately. After emergency imaging, he is transferred to the operating room for embolectomy. Surgical intervention successfully
restored blood supply to the affected limb after 3 hours. This patient is at risk of developing reperfusion injury as a result of prolonged
ischemia. Which one of the following is not a characteristic feature of reperfusion injury?

A. Hyperkalemia.

B. Hypokalemia.

C. Metabolic acidosis.

D. Myoglobinuria.

E. Elevate creatine kinase.

Incorrect. Correct answer is B


45% answered correctly

Explanation:

Correct Answer Is B

Reperfusion injury is a complication of blood restoration to a limb, which has been ischemic for a while. Characteristic features of reperfusion
injury are all related to ischemia and its impacts on tissue (especially muscle cells).

Features of reperfsuion syndrome include:

Metabolic acidosis (lactic acidosis)


Elevated creatine kinase
Hyperkalemia
Myoglobinemia and myoglobinuria

These findings are caused by hypoxemia resulting in metabolic acidosis, and muscle cell breakdown and release of its cell into the blood.
Hypokalemia is not a characteristic feature of reperfusion injury.

The extent of the reperfusion injury depends on the following:

Duration and the site of arterial blockage


The extent of collateral flow to the affected area
The previous health of the affected limb

Approximately one third of all deaths from arterial occlusions are due to metabolic complications after revascularisation.

References

• http://www.ncbi.nlm.nih.gov/pmc/articles/PMC111776

Last updated:
Time spent: QID:265
2023-2-12

55 of 1943
A 55-year-old man presents to the emergency department after sudden onset of pain and paresthesia in his left leg. On examination, the distal
pulses of the left lower limb are absent and the limb is cold and painful. A CT angiogram is performed and establishes the presence of
femoral artery embolism. The patient was started on intravenous heparin and embolectomy was carried out. Which one of the following is the
next best step in management?

A. Aspirin and warfarin for 6 months.

B. Lifelong aspirin.

C. Warfarin for 3-6 months.

D. Low molecular weight heparin for six months and lifelong warfarin afterwards.

E. Heparin for one week.

Incorrect. Correct answer is C


45% answered correctly

Explanation:

Correct Answer Is C

Every patient with acute limb ischemia must be started on anticoagulation therapy with warfarin after embolectomy is performed. Heparin
cover should not be withheld until the INR is in the therapeutic range of 2-3.

Long-term warfarin should be considered following embolism from a cardiac source; otherwise 3-6 months warfarin may suffice.

Aspirin is indicated in patients with atherosclerotic changes of the peripheral arteries and chronic limb ischemia. It has no role in
management of acute limb ischemia.

Combination of aspirin and warfarin is not shown to benefit the patient more than warfarin alone in terms of preventing further
thromboembolic events. Furthermore using these two drugs in combination increases the risk of bleeding.

Heparin is only used until the INR is in the therapeutic range and withheld afterwards. This goal is usually achieved in 2-3 days.

References

• http://www.ncbi.nlm.nih.gov/pmc/articles/PMC111776

• Therapeutic Guidelines – Cardiovascular; available on: http://tg.org.au

Last updated:
Time spent: QID:267
2023-2-12

56 of 1943
A 55-year-old man is on heparin after he developed an acute limb ischemia. Which one of the following tests is used for monitoring the
response to anticoagulation therapy with heparin?

A. Fibrinogen level.

B. Prothrombin time (PT).

C. INR.

D. Activated partial thromboplastin time (aPTT).

E. Bleeding time (BT).

Incorrect. Correct answer is D


45% answered correctly

Explanation:

Correct Answer Is D

Activated partial thromboplastin time (aPTT) is used to assess the integrity of the following coagulation pathways:

Intrinsic pathway that includes prekallikrein, high molecular weight kininogen and factors VIII, IX, XI, and XII.
Common pathway including factors II, V, X, and fibrinogen.

aPTT is also used for monitoring the effects of maintenance therapy with heparin. Usually the goal of heparin maintenance therapy is to keep
aPTT level 1.5- 2.5 times above patient’s aPTT baseline. When starting heparin therapy, aPTT is measured 6-hourly until it is within therapeutic
range of 65-100 seconds, and 24 hourly afterwards.

INR is used for monitoring warfarin therapy. It assesses extrinsic coagulation pathway, as does PT. Normal range of PT in an individual not on
blood thinners is 11-13.5 seconds. PT and INR are prolonged if the patient is on warfarin, or deficient in factors II (prothrombin), III, IX, X, or
fibrinogen, or has vitamin K deficiency.

BT is used as a platelet function test with a normal range of 1-9 minutes. It is prolonged if the patient is on antiplatelet medications (aspirin,
clopidogrel), has thrombocytopenia, or in those with platelet aggregation disorders e.g. Von-Willebrand disease. BT is not used routinely
anymore.

References

• Therapeutic Guidelines - Cardiovascular; available on: http://tg.org.au

Last updated:
Time spent: QID:268
2023-2-12

57 of 1943
A 65-year-old man comes to your practice with complaint of pain in buttock and leg. He has the pain for the past three months, and describes
it as cramping felt in the left buttock. The pain is only present when he starts walking and comes on after five minutes or so every time. It
radiates to the left thigh and goes away few minutes after he stops walking. Which one of the following is the most likely diagnosis?

A. Neurogenic claudication resulting from spinal canal stenosis.

B. Stenosis of his left common iliac artery.

C. Stenosis of the left superficial femoral artery.

D. Osteoarthritis of the right hip joint.

E. L4/L5 disc prolapse.

Incorrect. Correct answer is B


45% answered correctly

Explanation:

Correct Answer Is B

The nature of the pain (cramping) and its pattern (brought on by walking, relieved by rest) is characteristic of chronic limb ischemia. Exertional
buttock pain that is ischemic in nature (buttock claudication) is due to obstruction of either common iliac or external iliac artery. Stenosis of
superficial femoral artery leads to ischemic pain of lower leg on walking (calf claudication), not the buttock.

(Option A) Spinal canal stenosis usually affects the mid or lower lumbar spine and can cause nerve root impingement presentation, which is
pain brought on by standing and relieved by recumbency. Pain distribution is related to the nerve root distribution rather than muscle groups
supplied by an involved artery.

(Option C) Stenosis of the superficial femoral artery gives rise to leg rather than buttock claudication.

(Option D) Osteoarthritis of the hip is associated with pain on activity and relif by rest. The pain is felt in the hip and can radiat to the knee.
Buttock pain is not common.

(Option E) L4/L5 disc prolapse can result in unilateral nerve root entrapment leading to neurological findings, not necessarily related to
walking. The pain of L4/L5 disc prolapse is worse on lifting heavy objects, coughing, sneezing or straining at stool. Numbness and parsthesia
are features that are commonly present in nerve root impingement, but not in chronic limb ischemia.

References

• http://emedicine.medscape.com/article/460178-clini

• http://www.racgp.org.au/afp/2013/june/peripheral-a

• http://www.awma.com.au/publications/2011_awma_vlug

Last updated:
Time spent: QID:269
2023-2-12

58 of 1943
A 76-year-old man comes to your office for evaluation. He mentions that he has difficulty walking because of the left leg pain. The pain is
brought on after walking two blocks and gets better when he stops to rest. On examination, the leg skin is shiny and dark. The legs hair is lost
and the muscles are atrophied. Distal pulses are difficult to palpate. Which one of the following is the most likely diagnosis?

A. Acute limb ischemia.

B. Deep vein thrombosis.

C. Superficial vein thrombosis.

D. Chronic obstructive arterial disease.

E. Spinal canal stenosis.

Incorrect. Correct answer is D


45% answered correctly

Explanation:

Correct Answer Is D

Leg pain brought up by walking and relieved by rest and weak or absent distal pulses are characteristic of chronic limb ischemias as a result
of chronic obstructive arterial disease. Atrophied muscles and shiny hairless skin supports the diagnosis.

The clinical findings in chronic limb ischemia include:

Weak or absent distal pulses – the hallmark finding


Shiny and hyperpigmented skin
Hair loss and leg ulcers
Thickened nails
Muscular atrophy
Vascular bruits

(Option A) Acute limb ischemia presents with sudden onset pain, pallor, paralysis, paresthesia, pulselessness and poikilothermia. This patient
has features of chronic limb ischemia.

(Option B) Leg pain due to deep vein thrombosis can be brought on by walking and relieved by rest (similar to chronic limb ischemia), but
other features such as sparse leg hair, pigmentation, muscle atrophy, etc are not feautres of DVT. DVT presents with leg pain and tenderness,
swelling and warmth.

(Option C) Superficial thrombophlebitis presents with pain, erythema, induration and tenderness along the course of a superficial vein.

(Option E) Leg pain caused by neurogenic claudication due to spinal canal stenosis tends to get worse with erect posture and relieved by
recumbency. Absence of neurological deficits makes this diagnosis less likely.

References

• http://emedicine.medscape.com/article/460178-clini

• http://www.racgp.org.au/afp/2013/june/peripheral-a

• http://www.awma.com.au/publications/2011_awma_vlug

Last updated:
Time spent: QID:270
2023-2-12

59 of 1943
A 60-year-old man presents with leg pain for the past 6 months. The pain becomes worse with walking and is relieved when he rests. There is
no pain at rest. He has smoked 20 cigarettes per day for the past 30 years. On examination, he is obese with a BMI of 31 and has a blood
pressure of 160/110 mmHg on two readings 20 minutes apart. Distal pulses of the left lower limb, including dorsal pedis, are barely
perceptible. The skin of the legs is shiny and hairless. Mild muscular atrophy of the leg is noted. Ankle-brachial index (ABI) is 0.7. Which one
of the following would be the most appropriate management and advice?

A. Smoking cessation, exercise and follow-up in three months.

B. Smoking cessation, statins and ACE inhibitors.

C. Duplex Doppler venous ultrasonography.

D. Aspirin, metoprolol and statins.

E. Referral for vascular surgery.

Incorrect. Correct answer is B


45% answered correctly

Explanation:

Correct Answer Is B

With intermittent calf claudication, presence of the risk factors and an ABI of 0.7, the diagnosis of chronic peripheral arterial disease is almost
established. Management of peripheral arterial disease includes the following:

Smoking cessation - smoking is the most important predisposing factor for peripheral arterial disease (PAD). Smoking cesation alone is
associated with an improvement in the distance of pain-free walk, doubled 5-year survival and better post-op outcomes.

Exercise – exercise on an as tolerated basis, improves the pain-free walking distance and time and should be advised for all patients.

ACE inhibitors – evidence suggests that ACE inhibitors may improve walking ability in patients with intermittent claudication. The ACE
inhibitor with greatest evidence of benefit is ramipril. It is unknown if the improvement in walking distance associated with ramipril is due to a
class effect of ACE inhibitors or whether it is specific to this medicine. The ABI does not seem to improved though.

Statins – statins improve revascularisation, pain-free walking distance and survival. Of all lipid-lowering agents, only statins have been proved
to lower the mortality in patients with vascular diseases due to atherosclerotic changes. It should be started for patients with coronary artery
disease, PAD, aortic disease (e.g. abdominal aortic aneurysm), carotid artery disease and diabetes mellitus.

Clopidogrel and aspirin – they are often prescribed to reduce the overall risk of myocardial infarction and stroke, but are not associated with
improvement of PAD symptoms.

Beta blockers (e.g. metoprolol) are not indicated in the absence of cardiac disease. They have no effect on PAD.

More detailed management of PAD is highly dependent on the severity of the disease. ABI is the mostly used predictor for this purpose. ABI is
interpreted as follow:

ABI value Interpretation Action Nature of ulcers if present


Abnormal vessel stiffness (usually
>1.4 Refer routinely
from peripheral vascular disease)
Venous ulcers
1.0-1.4 Normal
Borderline – discussion with a
Use full-compression
vascular surgeon may be None
0.9 bandage
appropriate depending on the
patient’s symptoms and risk factor
Manage risk factors
Mixed ulcers – do not use
<0.9 Abnormal and diagnostic of PAD compression bandage if
Routine specialist referral is ABI<0.8
indicated
Arterial ulcers – no
<0.4 Critical limb ischemia Urgent specialist referral
compression bandage

Among the given option, smoking cessation, statins, and ACE inhibitors are the best possible management and the advice for this patient.

References

60 of 1943
• http://www.bpac.org.nz/BPJ/2014/April/ankle-brachi

• http://www.awma.com.au/publications/2011_awma_vlug

• http://www.bpac.org.nz/BPJ/2014/April/ankle-brachi
Last updated:
Time spent: QID:271
2023-2-12

61 of 1943
A 56-year-old man comes to your practice with an eight-month history of left leg pain, which is brought on by walking, and used to relieved
with rest. Recently, he gets the pain even at rest, particularly at night. He has to dangle his leg for relative pain relief. He has smoked for the
past 30 years and been on hydrochlorothiazide for the past 10 years. On examination, he is an overweight man with a BMI of 29, blood
pressure of 145/100mmHg and pulse of 98bpm. His left leg is hairless with shiny and hyper-pigmented skin. The left femoral pulse is barely
perceptible and the right dorsal pedis pulse cannot be felt. He has an ankle-brachial index (ABI) of 0.4 on the left side. Which one of the
following is the most appropriate management?

A. Advise him to stop smoking, do more exercise and review in three months.

B. Advise him to stop smoking, do more exercise and start him on statin.

C. Arrange Duplex Doppler ultrasonography and review.

D. Urgent referral for vascular surgery.

E. Arrange for arteriography and review.

Incorrect. Correct answer is D


45% answered correctly

Explanation:

Correct Answer Is D

The clinical picture is consistent with chronic peripheral artery disease (PAD) and limb ischemia suggested by calf claudication and the
presence of risk factors (hypertension, smoking and obesity). The rest pain and ABI of 0.4 suggests a critical ischemia, for which surgical
intervention is required to be planned; otherwise he will end up gangrene and amputation. This patient should be urgently referred to a
vascular surgeon for assessment and treatment.

(Options A and B) Smoking cessation is the most appropriate advice that should be given to patients with PAD. It increases the exercise
tolerance and walking distant and slows progression to more advanced disease. Statins should be started for patients with PAD as well as
those with coronary artery disease, diabetes mellitus, aortic diseases due to atherosclerosis, and carotid artery disease. Pentoxifyline (not an
option) might have a role, but trials have shown moderate benefit with this drug.

(Option C) Duplex Doppler ultrasonography has already been used for determination of the ABI. It is unable to locate the anatomy of the lesion
and a road map for surgery. Duplex Doppler ultrasonography is used to assess the blood flow to the leg and calculation of ABI, and the
presence of a clot in the venous system when DVT is suspected. It cannot accurately spot the site of occlusion in an artery.

(Option E) Once surgical intervention is considered by a surgeon, the presence and site of occlusion should be indentified and documented
prior to the surgery. This is achieved by either less invasive CT angiography or MRI angiography, or through more invasive procedure of
conventional arteriography.

References

• http://www.imagingpathways.health.wa.gov.au/index.

• http://www.bpac.org.nz/BPJ/2014/April/ankle-brachi

Last updated:
Time spent: QID:272
2023-2-12

62 of 1943
A 28-year-old man develops rapid palpitations and mild light-headedness following a 20km run. He denies any cardiac disease and mentions
that a he had a similar episode 3 months ago for which he underwent cardiovascular examination by another doctor but everything was
normal. On examination, he has regular but rapid pulse of 180 bpm. Which one of the following rhythm strips would be expected for him?

A.

B.

C.

D.

E.

Incorrect. Correct answer is D


45% answered correctly

Explanation:

Correct Answer Is D

An arrhythmia induced by exercise in a young, otherwise healthy person, presenting with a rapid regular pulse is very likley to be a paroxysmal
supraventricular tachycardia (PSVT).

The presence of ventricular tachycardia (option A) in the absence of an underlying cardiac disease is unlikely, as are complete heart block
(option E) and atrial flutter (option C).

Atrial fibrillation (option B) presents with irregular pulse.

References

• MJA - Supraventricular tachycardia

• Medscape - Paroxysmal Supraventricular Tachycardia

Last updated:
Time spent: QID:21
2023-2-12

63 of 1943
You are assessing the risk of thromboembolism in a 75-year-old woman, who is booked for elective surgical replacement of her left knee joint.
Which one of the following, if present in history, is the most significant risk factor for thromboembolism during her peri-operative stay?

A. History of colon resection due to toxic megacolon.

B. Irritable bowel syndrome.

C. Stage II kidney disease.

D. Transient ischemic attack with full recovery three years ago.

E. Heart failure.

Incorrect. Correct answer is E


45% answered correctly

Explanation:

Correct Answer Is E

Peri-hospital risk assessment of a venous thromboembolic events (VTE) such as deep vein thrombosis or pulmonary embolism is
stratified according to the following table:

Stratification of deep vein thrombosis and/or pulmonary embolism during hospital stay
Risk category Medical Surgical
High
Stroke Orthopedic surgery of pelvis, hip, or
Age>75 lower limb
Congestive heart failure Major surgery* with age >60
Shock Major surgery with age 40-60 and
Cancer cancer or history DVT/PE or other
History of DVT/PE risk factors**
Thrombophilia Thrombophilia***

Major surgery with age 40-60, or


other risk factors
Immobilized patients with active Minor surgery with age>60, or age
Moderate disease 40-60 and history of DVT/PE or
estrogen therapy or other risk
factors

Major surgery with age<40 years


Minor medical illness
Low Minor surgery with age<60 years

*major surgery: any intra-abdominal operation and all other operations lasting more than 45 minutes

** infectious diseases, varicose veins, obesity or general immobility

***deficiency of antithrombin, protein C, protein S, Factor V Leiden mutation, hyperhomocysteinemia, and prothrombin 20210A

Among the options, congestive heart failure and the surgery are of the highest risk for venous thromboembolism. None of the other options
are risk factors for VTE.

References

• Therapeutic Guidelines – Cardiovascular; available on: http://tg.org.au

Last updated:
Time spent: QID:273
2023-2-12

64 of 1943
Five days after surgical resection of a colonic tumor, a 73-year-old man develops pain, swelling and tenderness in his right calf. Physical
examination reveals swelling of the ankle and the calf and tenderness over the gastrocnemius muscle. Which one of the following is the most
appropraite diagnostic tool to establish the diagnosis?

A. CT angiography of the leg.

B. MRI of the leg.

C. Conventional venography of the leg.

D. Serum D-dimer.

E. Duplex Doppler ultrasonography.

Incorrect. Correct answer is E


45% answered correctly

Explanation:

Correct Answer Is E

With the clinical picture suggestive of deep vein thrombosis, Duplex Doppler ultrasonography should be used to confirm the diagnosis.
Although pain, swelling, and tenderness are suggestive of DVT, but treatment should never be started solely on clinical basis.

(Option A) CT angiography is used for visualization of clots in arteries not veins.

(Option B) MRI is being used increasingly for assessment of DVT, and probably is the preferred investigation for suspected caval and iliac
venous thrombosis, particularly in pregnancy, but is expensive, time-consuming and not always available.

(Option C) Conventional venography is the criterion standard for diagnosis of DVT, but is rarely used now because less invasive and
convenient diagnostic modalities are widely available.

(Option D) D-dimer assay, although sensitive, lacks specificity. Increased levels of D-dimer can be seen in several conditions, with post-surgery
state as one. It is basically used to rule out DVT or pulmonary embolism in low risk patients (Wells score<2). A positive test demands more
specified evaluation, but a negative one excludes the possibility of DVT or PE.

References

• AMC handbook multiple choice questions – page 173

• http://www.imagingpathways.health.wa.gov.au/index.

Last updated:
Time spent: QID:274
2023-2-12

65 of 1943
A 73-year-old man presents with swelling of his left leg for the past three hours. Evaluation establishes the diagnosis of a deep vein
thrombosis (DVT) in the left lower leg. A Doppler ultrasound reveals deep vein thrombosis in the calf as well as popliteal and femoral veins.
His past medical history is negative for previous DVT. Which one of the following is the next best in management of this patient?

A. Angiography.

B. Angioplasty.

C. Heparin.

D. Warfarin.

E. Aspirin.

Incorrect. Correct answer is C


45% answered correctly

Explanation:

Correct Answer Is C

All patients with DVT are initially managed by injectable heparin (either intravenous unfractionated or subcutaneous low molecular weight
heparin). Warfarin can be started within 24-48 hours with close monitoring of INR. Once INR is in the therapeutic range of 2-3, heparin can be
withheld. Warfarin should be continued for 6 months at least. Patients should see their GP on a regular basis for close monitoring of the INR.

(Option A) DVT is the presence of clot in the deep veins. Arteriography evaluates arterial structure and thromboses within arteries.

(Option B) Angioplasty is performed for revascularizaion in arterial problmes with no role in DVT.

(Option D) Warfarin is indicated in this patient after initial anticoagulation with heparin.

(Option E) Antiplatelet therapy with aspirin or clopidogrel does not provide adequate anticoagulation for prevention of DVT.

References

• Therapeutic Guidelines – Cardiovascular; available on: http://tg.org.au

Last updated:
Time spent: QID:275
2023-2-12

66 of 1943
An 82-year-old man presents to your office complaining of an ulcer over the medial side of his leg just above the internal malleolus. He
mentions that the wound has been there for the past 6 months, and worsened recently. The examination confirms the diagnosis a venous
ulcer. The ipsilateral ankle-brachial index is 0.7. Which one of the following is correct regarding the management of this patient?

A. Compression stockings are unsafe for this patient.

B. Prescribe compression stockings and refer him to a vascular surgeon.

C. Give antibiotics and review in two weeks.

D. Check fasting glucose.

E. Check HbA1C.

Correct
45% answered correctly

Explanation:

Correct Answer Is A

Venous leg ulcers are the most common chronic wounds seen in general practice. They account for 70-90% of chronic wounds. They tend to
develop over the medial distal leg (just above the internal malleolus), and are usually painful.

The most important risk factor for developmet of a venous ulcer is previous deep vein thrombosis (DVT) with consequent damage to venous
structure and insufficiency.

Most venous ulcers respond adequately to a conservative management called Bisggard regimen. This regimen has four components:

1. Patient education
2. Elevation of the foot
3. Elastic compression
4. Evaluation

Elastic compression is considered for very patient unless:

ABI is below 0.8


The patient is diabetic

In the above situations, compression stocking causes more compromised arterial flow and makes the limb ischemia worse; therefore,
specialist advice and close monitoring is strongly recommended.

Antibiotics are not indicated in uncomplicated venous ulcers. Diabetes is one of the most important predisposing factors in development of
arterial ulcers. Although fasting glucose level may be considered for screening of diabetes, it is not a step in management here. HbA1C is
used to monitor efficacy of or adherence to anti-hyperglycemic regimens, which is not the case here.

References

• http://www.racgp.org.au/afp/2012/may/venous-leg-ul

• http://www.racgp.org.au/afp/2014/september/managem

• http://www.awma.com.au/publications/2011_awma_vlug

Last updated:
Time spent: QID:276
2023-2-12

67 of 1943
A 55-year-old man presents to the Emergency Department with episodes of palpitation and light-headedness. An ECG is obtained and is
inconclusive. He is placed on telemetry monitoring. Evaluation of the records shows nonsustained paroxysms of ventricular tachycardia.
There are many of these paroxysms during the night. Which one of the following is the next best step in management?

A. Amiodarone.

B. Bisoprolol.

C. Verapamile.

D. Synchronized DC cardioversion.

E. Digoxin.

Correct
45% answered correctly

Explanation:

Correct Answer Is A

For this patient with symptomatic non-sustained VTs, Amiodarone is the drug of choice. Amiodarone is considered first-line treatment option
for those with sustained VT who are hemodynamically stable, or those with symptomatic non-sustained VT. Sotalol and lignocaine are
second- and third-line medications, respectively.

Synchronized cardioversion (option D) is the next best in management of patients with sustained VT and consequent hemodynamic instability
i.e. chest pain, dyspnea, hypotension, and perfusion-related confusion. Pulseless VT should be treated similar to ventricular fibrillation (VF)
with defibrillation.

In outpatient setting, beta blockers (atenolol, bisoprolol, or metoprolol) (option B), orally, are used for nonsustained episodes of VT associated
with symptoms as preferred options. Amiodarone, sotalol or flecainide, orally, are second-line options. Calcium channel blockers (verapamil)
(option C) or digoxin (option E) are not used for management of ventricular tachycardia.

References

• Therapeutic Guidelines – Cardiovascular

Last updated:
Time spent: QID:277
2023-2-12

68 of 1943
A-79-year-old woman undergoes total hip replacement surgery. After 4 days, she develops a swollen right leg. The circumference of the right
calf is 4 cm greater than that of the left leg. The right leg feels warm to touch and is slightly tender. Which one of the following is most
appropriate diagnostic tool to use for a diagnosis?

A. Duplex Doppler ultrasound.

B. X-ray.

C. Contrast venography.

D. CT angiography.

E. D-dimer assay.

Correct
45% answered correctly

Explanation:

Correct Answer Is A

This patient has suspected diagnosis of deep vein thrombosis (DVT) and requires urgent Doppler ultrasound to establish the diagnosis.

Doppler ultrasound is more than 90% sensitive and more than 95% specific for thromboses within the femoral and popliteal veins, but less
accurate for iliac or calf vein thromboses.

D-dimer is used for assessment of patients with Well score of <2.

A patient with symptoms and signs consistent with DVT should be assessed and the clinical (pretest) probability of acute DVT established by
using a validated scoring system. If the clinical probability of DVT is low (<2), a D-dimer assay should be performed; otherwise, the next step
would be Doppler Duplex sonography.

Modified Wells' score for predicting probability of deep vein thrombosis


CLINICAL CHARACTERISTIC SCORE
Active cancer (treatment ongoing, administered within previous 6
1
months or palliative)
Pralysis, paresis or recent plaster immobilization of the lower
1
extremities
Recently bedridden >3days or major surgery within previous 12 weeks
1
requiring general or regional anesthesia
Localized tenderness along the distribution of the deep venous system 1
Swelling of entire leg 1
Calf swelling >3cm larger than asymptomatic side (measured 10 cm
1
below tibial tuberosity)
Pitting edema confined to the symptomatic leg 1
Collateral superficial veins (nonvaricose) 1
Previously documented DVT 1
Alternative diagnosis at least as likely as DVT -2
A score of ≥2 indicates that the probability of DVT is likely; a score of <2 indicates
that the probability of DVT is unlikely

In patients who have symptoms in both legs, the more symptomatic leg is used

This patient has been bedridden for more than 3 days (1 point), has localized tenderness (1 point) and the circumference of the affected leg is
more than 3 cm greater than the unaffected leg (1 point). With a Wells' score or 3, she has high probability diagnosis of DVT and needs
Doppler ultrasonography of the leg as the next step.

If the Wells' score was <2 the next best step would have been D-dimer assay (option E). A negative D-dimer test excludes DVT (and pulmonary
embolism), but positive values is only suggestive of DVT and further investigations would be needed.

Previously, contrast venography (option C) was the definitive test for the diagnosis of DVT but has been largely replaced by ultrasonography,
which is noninvasive, more readily available; however, it still remains the gold-standard for diagnosis of DVT (not in practice). While a Doppler
ultrasound can establish the diagnosis, there is no need for CT angiography (option D) unless accurated anatomical site of the thrombus is
required for surgical intervention (very rarely indicated).

69 of 1943
References

• http://www.racgp.org.au/download/documents/AFP/201

• http://emedicine.medscape.com/article/1911303-work

• http://www.emedicine.medscape.com/article/1911303-

Last updated:
Time spent: QID:279
2023-2-12

70 of 1943
A 40-year-old man presents to the emergency department after sudden onset of the right calf pain and paralysis. The right dorsal pedis pulse
is not perceptible. The limb feels cold and is pale. The patient is given analgesics. After starting the patient on heparin, which one of the
following would be the most appropriate step in management?

A. Abdominal CT.

B. CT angiogram.

C. Doppler Duplex ultrasound of the calf veins.

D. Pelvic ultrasound.

E. Echocardiography.

Incorrect. Correct answer is B


45% answered correctly

Explanation:

Correct Answer Is B

This patient has developed critical acute limb ischemia requiring urgent vascular surgery for revascularization and restoration of blood supply.
Clinical signs of acute arterial occlusion include (6 P's):

Pain
Paralysis
Pulselessness
Pallor
Paresthesia
Poikilothermia.

With any of the above signs or symptoms the patient is considered to have acute limb ischemia.

To confirm the diagnosis of acute arterial occlusion and the extent of obstruction, the patient should have an urgent CT angiogram as the road
map to the surgery. Magentic resonance angiography with contrast is an alternative to CT angiography with about the same sensitivity and
specificitiy.

(Option A) Abdominal CT scan is not required as this patient did not present primarily with abdominal problems.

(Options C and D) Pelvic ultrasound or Doppler Duplex ultrasound of the calf veins adds nothing to the management strategy because firstly
the diagnosis is already made, and secondly these modalities do not provide adequate information regarding the anatomical site of the
occlusion. Furthermore, Doppler Duplex ultrasound is highly operator-dependent.

(Option E) Echocardiography is the investigation to consider once acute phase of arterial occlusion has been managed. Thombi from the
heart is a main source of acute limb ischemia.

References

• http://www.imagingpathways.health.wa.gov.au/index.

• http://www.ncbi.nlm.nih.gov/pmc/articles/PMC111776

Last updated:
Time spent: QID:280
2023-2-12

71 of 1943
A 55-year-old man presents to a regional hospital complaining of chest pain. ECG shows ST segment elevation in leads I, V1, V5 and V6. The
nearest tertiary hospital is 300 km away. He has been given aspirin, oxygen, and morphine. Which one of the following is the most appropriate
step in management of this patient?

A. Transfer the patient to a tertiary hospital for PCI.

B. Give him intravesnous tissue plasminogen activator.

C. Give him beta blockers.

D. Give him clopidogrel.

E. Give him glyceryl trinitrate.

Incorrect. Correct answer is B


45% answered correctly

Explanation:

Correct Answer Is B

With chest pain and ST elevation in two contiguous leads, the diagnosis of ST elevation myocardial infarction (STEMI) is established and
emergency reperfusion therapy is indicated as the first-line management.

Reperfusion is best achieved with PCI if it can be performed within 90 minutes of presentation. In this case, however, the time needed for
travelling 300km to a tertiary hospital for PCI is far beyond the 90-minute limit; therefore, reperfusion with fibrinolytics such as tissue
plasminogen activator (tPA) is the best option.

Beta blockers, nitrates are part of the management plan but not the most appropriate step in this situation. Clopidogrel is best avoided if the
patient is undergoing fibrinolytic therapy.

TOPIC REVIEW

Choice of reperfusion therapy depends mostly on the time delay (both to first medical contact and potential PCI or fibrinolytic therapy):

In general, PCI is the treatment of choice, providing it can be performed promptly by a qualified interventional cardiologist in an
appropriate facility.

In general, the maximum acceptable delay from presentation to balloon inflation is:
60 minutes if a patient presents within 1 hour of symptom onset; OR
90 minutes if a patient presents later.

NOTE - For patients who present late (between 3 and 12 hours after symptom onset) to a facility without PCI capability, it is appropriate
to consider transfer for primary PCI if balloon inflation can be achieved within 2 hours (including transport time).

All PCI facilities should be able to perform angioplasty within 90 minutes of patient presentation.

Fibrinolysis should be considered early if PCI is not readily available, particularly in rural and remote areas.

When there are major delays to hospitalization (i.e., more than 30 minutes), pre-hospital fibrinolysis should be considered.

Reperfusion is not routinely recommended in patients who present more than 12 hours after symptom onset and who are
asymptomatic and hemodynamically stable.

References

• Heart Foundation - Acute Coronary Syndrome Guideline

• Medscape - Acute Coronary Syndrome

Last updated:
Time spent: QID:281
2023-2-12

72 of 1943
A 45-year-old man comes to your practice for a routine health check. He has no remarkable past medical history. On examination, you feel his
pulse is irregular with an average rate of 84 bpm. The rest of the exam is unremarkable. He denies any history of lightheadedness, shortness
of breath, palpitation or chest pain. An ECG is ordered which is shown in the accompanying photograph. Which one of the following is the
most appropriate next step in management of this patient?

A. Start him on metoprolol.

B. Start him on warfarin.

C. Start him on digoxin.

D. Start him on aspirin.

E. Do an echocardiography.

Incorrect. Correct answer is D


45% answered correctly

Explanation:

Correct Answer Is D

The ECG shows atrial fibrillation (AF) without tachycardia, and the given scenario is that of an asymptomatic AF. The priority in management
of patients with asymptomatic AF centers around prevention of thromboembolic events, until elective cardioversion is performed either
medically or by application of synchronized cardioversion. Patients with AF in the setting of valvular heart disease should be started on
warfarin. Other patients with AF should be stratified and managed based on CHA2DS2 - VASc index. CHA2DS2 - VASc is calculated according
to the following table:

Letter Condition Score


C Congestive heart failure / Left ventricular dysfunction 1
H Hypertension 1
A2 Age ≥75 years 2
D Diabetes mellitus 1
S2 Previous history of stroke or TIA 2
V Vascular disease i.e., peripheral arterial disease, previous MI, aortic plaques 1
A Age 65-74 years 1
Sc Sex category (i.e. female sex) 1
NOTE - maximum score is 9 since age may contribute 0, 1, or 2 points 9

The decision as to whether anticoagulation therapy is started is based on the CHA2DS2 - VASc score and according to the folloiwing table:

Score VTE Prophylaxis


0 No prophylaxis at all or low-dose aspirin
1 Evidence of treatment limited in this group. Options include no antithrombotic treatment, aspirin 75
-300mg daily or oral anticoagulant (OAC) such as warfarin. Aspirin or OAC is unlikely to have a net
clinical benefit unless the risk of hemorrhage as an adverse outcome is low (HAS-BLED score is low)
≥2 OAC such as warfarin. New oral anticoagulants (NOAC) such as dabigatran or rivaroxaban are preferred
to warfarin.

73 of 1943
This patient has a CHA2DS2 - VASc score of 0. For him both no anticoagulation phrophylaxis or anticoagulation therapy with low-dose aspirin
are acceptable options. There is no option suggesting 'no anticoagulation therapy'; therefore, the only acceptable option would be
anticoagulation therapy with aspirin.

For symptomatic patients or older patients with no or mild symptoms, who have rapid response AF evident by tachycardia, rate control should
be started with beta blockers (first-line) such as metoprolol (option A) or calcium channel blockers (second-line). Digoxin (option C) is no
longer recommended for rate control.

Echocardiography (option E) and TSH are two most important steps considered for every patient with new onset or new diagnosed AF, but
not as the next best step in management.

NOTE - Rhythm control (vs. rate control) may be considered in the following patients groups:

Patients with unacceptable arrhythmia-related symptoms


Patients presenting for the first time with non-valvular disease
AF secondary to an identifiable and correctable underlying cause
Patients who are intolerant of rate-controlling medications
Patients with congestive heart failure - Arrhythmia can decompensate CHF, or cause CHF in an already sick heart
Patients <65 years

References

• Quick reference guide: Atrial Fibrillation Information for the Health Practitioner

• Therapeutic Guidelines - Cardiovascular

Last updated:
Time spent: QID:282
2023-2-12

74 of 1943
Which one of the following is not associated with increased serum troponin level?

A. Acute sepsis.

B. Renal failure.

C. Pulmonary embolism.

D. Aortic dissection.

E. Bradyarrhythmias.

Incorrect. Correct answer is E


45% answered correctly

Explanation:

Correct Answer Is E

There is a long list of conditions that can lead to elevated serum troponin levels in the absence of acute coronary syndrome and myocardial
infarction. Bradyarrhythmias do not cause elevated serum troponin levels.

The causes for elevation of serum troponin levels are summarized in the following table:

System Causes of troponin elevation


Aortic dissection.
Sustained tachyarrhythmias
Medical ICU patients
Hypotension
Heat failure
Apical ballooning syndrome
Cardiovascular Endocarditis and pericarditis
Hypertension
Sarcoidosis, Amyloidosis, haemochromatosis
Scleroderma and lupus
Left ventricular hypertrophy

Blunt chest trauma


Cardiac surgeries and other invasive
interventions
Chemotherapy with cardiotoxic drugs
Myocardial injury Hypersensitivity drug reactions
envenomation

Pulmonary embolism
Respiratory ARDS

Sepsis / Systemic inflammatory response


syndrome (SIRS)
Viral illness
Infections/immune Thrombocytopenic thrombotic purpura

Severe GI bleeding
Gastrointestinal

Acute stroke (haemorrhagic or ischaemic)


CNS Head trauma

Chronic kidney disease


Renal

Diabetes mellitus
Endocrine hypothyroidism

75 of 1943
Rhabdomyolysis
Musculoskeletal

Burns
Integumentary

Neurofibromatosis
Duchenne muscular dystrophy
Inherited Kippel-Feil syndrome

Endurance exercise
Environmental: carbon monoxide, hydrogen
Other sulfide

References

• European Heart Journal - Troponin elevation in coronary vs. non-coronary disease

• BMJ - Differential diagnosis of elevated troponins

Last updated:
Time spent: QID:284
2023-2-12

76 of 1943
Which one of the following conditions would be associated with silent (painless) myocardial infarction?

A. Polycythemia vera.

B. Hypertension.

C. Diabetes.

D. Migraine.

E. Hypercholesterolemia.

Incorrect. Correct answer is C


45% answered correctly

Explanation:

Correct Answer Is C

Typical presentation of myocardial infarction (MI) is with chest pain and discomfort that may radiate to the jaw, left arm or epigastrium.
Nausea, vomiting and weakness are other features. Atypical presentation is with fatigue, weakness, syncope and sometimes shortness of
breath.

Approximately one in three MIs are silent and the only presentation is changes on ECG in the absence of symptoms or presence of vague and
non-specific symptoms. Silent MI is more common among the following patient groups:

Women
The elderly
Diabetics

References

• https://www.mja.com.au/insight/2012/35/silent-mi-d

• http://clinical.diabetesjournals.org/content/21/1/

• http://circ.ahajournals.org/content/93/12/2089.ful

Last updated:
Time spent: QID:22
2023-2-12

77 of 1943
A 76-year-old woman presents with complaint of palpitation. She in a known case of persistent atrial fibrillation for the past one year and is on
rate control medications. She is not diabetic and has never had a stroke or TIA. She denies any episode of chest pain or shortness of breath.
On examination, her BP is 130/85 mmHg with an irregular pulse of approximately 90 bpm. An ECG shows atrial fibrillation (AF). Which one of
the following is the most appropriate management regarding prevention of stroke in this patient?

A. Digoxin.

B. Metoprolol.

C. Aspirin.

D. Warfarin.

E. Heparin.

Incorrect. Correct answer is D


45% answered correctly

Explanation:

Correct Answer Is D

AF is manage either by rate control as the preferred method using beta blockers (first choice) or calcium channel blockers (diltiazem,
verapamil), or rhythm control with flecainide, or amiodarone. Systemic thromboembolism is the most feared complication of AF, as one out of
five patients with AF will develop cerebrovascular accidents (stroke or TIA) in one year. For this reason, the risk of stroke should be assessed
and prevention considered for every patient with AF longer than 48 hours. CHA2DS2 -VASc scoring system is used for this objective in patients
with AF without underlying valvulopathies. CHA2DS2 -VASc score is calculated according to the following table:

Letter Condition Score


C Congestive heart failure / Left ventricular dysfunction 1
H Hypertension 1
A2 Age ≥75 years 2
D Diabetes mellitus 1
S2 Previous history of stroke or TIA 2
V Vascular disease i.e., peripheral arterial disease, previous MI, aortic plaques 1
A Age 65-74 years 1
Sc Sex category (i.e. female sex) 1
NOTE - maximum score is 9 since age may contribute 0, 1, or 2 points 9

The decision as to whether anticoagulation therapy is started is based on the CHA2DS2 - VASc score and according to the folloiwing
table:

Score VTE Prophylaxis


0 No prophylaxis at all or low-dose aspirin
1 Evidence of treatment limited in this group. Options include no antithrombotic treatment, aspirin 75
-300mg daily or oral anticoagulant (OAC) such as warfarin. Aspirin or OAC is unlikely to have a net
clinical benefit unless the risk of hemorrhage as an adverse outcome is low (HAS-BLED score is low)
≥2 OAC such as warfarin. New oral anticoagulants (NOAC) such as dabigatran or rivaroxaban are preferred
to warfarin.

This woman is 76 years old (2 points), has no previous history of stroke or TIA (0) and is not diabetic (0) or hypertensive (0). There is no
history of MI, peripheral arterial disease, or aortic plaque (0). She does not have congestive heart failure or left ventricle dysfunction (0). She
receives one additional point for being female. Collectively, this woman has a CHA2DS2 - VASc score of 3; therefore, she should be started on
warfarin as the most appropriate option. Heparin should also be started to counteract the initial prothrombotic effects of warfarin until INR is
within the therapeutic range of 2-3.

NOTE - Although the cut-off point for type 1 hypertension is defined as a systolic blood pressure of 130 mmHg or higher by the American
Heart Association (AHA), the cut-off value for hypertension in CHA2DS2 - VASc score remains 140mmHg or higher.

78 of 1943
(Option A) Digoxin is occasionally used for rate or rhythm control in AF. It does not prevent thromboembolic events.

(Option B) Metoprolol is used for rate control not anticoagulation and thromboembolism prophylaxis.

(Option C) Aspirin is used in patients with nonvalvular AF and a CHAD2S2 score of 0 or 1 (in carefully selected patients where anticoagulation
outweighs the risk of bleeding).

(Option E) Heparin is used initially in conjunction with warfarin and stopped once the INR is within the therapeutic range (2-3).

References

• Quick reference guide: Atrial Fibrillation Information for the Health Practitioner

• Medscape - Atrial Fibrillation

• RACGP - Management of atrial fibrillation

Last updated:
Time spent: QID:285
2023-2-12

79 of 1943
According to current Australian Resuscitation Council Guidelines, which one of the following would form the basis for diagnosing an acute ST
segment elevation myocardial infarction (STEMI)?

A. Chest pain with elevated troponin.

B. Chest pain/discomfort that often radiates, is provoked by exercise and relieved by rest.

C. Chest pain/discomfort suggestive acute myocardial infarction in the presence of ST segment elevation more than 1mm in two
contagious leads, or newly developed left bundle branch block.
D. Chest pain/discomfort, ST depression or T-wave inversion, and elevated troponin.

E. Unprovoked sustained chest pain with 12-lead ECG that might be normal, with ST segment depression, or T-wave inversion.

Incorrect. Correct answer is C


45% answered correctly

Explanation:

Correct Answer Is C

According to guidelines of Australian Resuscitation guideline (#14), the diagnosis of an STEMI is established if there are myocardial ischemia
symptoms (non-specific in silent MI) and either or both of the following:

ST segment elevation greater than one millimeter in two contagious leads in the presence of symptoms.
Development of new left bundle branch block.

References

• Australian Resuscitation Council Guidelines - Guideline 14: ACS: Overview & Summary

• The New Zealand Medical Journal - ST Elevation Myocardial Infarction

Last updated:
Time spent: QID:286
2023-2-12

80 of 1943
A new systolic murmur is noted one week after an acute ST segment elevation myocardial infarction in a 65-year-old man. Echocardiography
reveals an ejection fraction (EF) of 55%. Which one of the following is the most likely underlying cause for this murmur?

A. Papillary muscle rupture.

B. Papillary muscle dysfunction.

C. Mitral valve stenosis.

D. Aortic regurgitation.

E. Ventricular septal defect (VSD).

Incorrect. Correct answer is B


45% answered correctly

Explanation:

Correct Answer Is B

Post-myocardial infarction (MI) papillary muscle dysfunction (rupture included) is the most common and well-known complication of the left
ventricle MI, resulting in mitral valve insufficiency and regurgitation.

Dysfunction of the papillary muscle is often associated with a normal or minimally decreased ejection fraction (EF), while the rupture of this
muscle (option A) causes significant drop in EF. This patient has a normal EF.

Mitral valve stenosis (option C) is not a complication of MI.

Aortic valve regurgitation (option D) and VSD (option E) are not common complications in MI. Furthermore, these conditions are associated
with decreased EF. This patient has a normal EF.

References

• Medscape - Complications of Myocardial Infarction

Last updated:
Time spent: QID:287
2023-2-12

81 of 1943
Which one of the following is the most appropriate course of action for pulseless electrical activity?

A. Adrenaline 1mg intravenously and investigating for reversible causes.

B. Atropine up to 3 mg intravenously and looking for the reversible causes.

C. Start cardiopulmonary resuscitation, give 1mg adrenaline intravenously and look for reversible causes.

D. Defibrillation, CPR, defibrillation, CPR, and IV adrenaline.

E. No cardiopulmonary resuscitation as pulseless electrical activity in not likely to reverse.

Incorrect. Correct answer is C


45% answered correctly

Explanation:

Correct Answer Is C

Pulseless electrical activity (PEA) is managed by starting CPR (30:2) immediately, securing an airway and providing intravenous access. If not
responsive, IV adrenaline should be given. Looking for a correctable cause of PEA and correcting that is always an important part of
management.

Defibrillation has no role in management of PEA as the organized electrical activity of the heart is preserved.

There is no recommendation to support the use of atropine in PEA or asystole.

References

• Medscape - Pulseless Electrical Activity

Last updated:
Time spent: QID:288
2023-2-12

82 of 1943
A 67-year-old woman presents to your practice with complaint of dyspnea, especially on exertion. On heart auscultation, there is a systolic
murmur heard over the apex, and a diastolic murmur in the left upper sternal border. The pulmonary component of S2 is split. Which one of
the following is the most likely diagnosis?

A. Aortic valve stenosis.

B. Mitral valve regurgitation.

C. Pulmonary valve stenosis.

D. Tricuspid valve incompetence.

E. Mitral valve stenosis.

Correct
45% answered correctly

Explanation:

Correct Answer Is A

Typically, a split S2 is characteristic of either mitral valve regurgitation (MR) or aortic valve stenosis (AS) (may be seen in other cardiac
conditions such as bundle branch block as well). The murmur of mitral regurgitation occurs during systole when the mitral valve fails to
completely close and there is blood flow turbulence around the valve.

The typical murmur of the AS is an early systolic harsh murmur best heard over the right upper sternal border (in the second intercostal
space) that can radiate to one or both carotid areas. However, in elderly patient the murmur may be best heard over the apex and resembles
murmurs caused by MR.

On the other hand, calcified stenotic leaflets of the aortic valve may fail to tightly close during the diastole and give rise to valve regurgitation
and a diastolic murmur.

With the history of exertional dyspnea, and the above mentioned murmurs and more importantly the split S2, AS will be the most likely
diagnosis.

References

• Medscape - Aortic Stenosis

• UpToDate - Clinical manifestations and diagnosis of aortic stenosis in adults

Last updated:
Time spent: QID:388
2023-2-12

83 of 1943
A 63-year-old man visits you because of progressive shortness of breath brought on by exertion. He does not smoke and denies any chest
pain, known cardiac disease, or hypertension. On examination, a systolic murmur over the left sternal border, a diastolic murmur over the left
lower sternal border, and a systolic murmur over the apex are heard. Which one of the following is the most likely diagnosis?

A. Mitral valve stenosis.

B. Mitral valve regurgitation.

C. Aortic valve stenosis.

D. Aortic valve regurgitation.

E. Tricuspid valve regurgitation.

Incorrect. Correct answer is D


45% answered correctly

Explanation:

Correct Answer Is D

Of all the valvular lesions, aortic valve regurgitation (AR) is capable of producing three different murmurs.

AR results in backflow of blood from the aorta into the left ventricle during diastole. This causes a diastolic crescendo murmur. Since this
murmur is caused by spillage of blood from the aorta into the left ventricle it is best heard over the left sternal border.

The backflow of blood from the aorta during the diastole results in increased end-diastolic left ventricular volume. The increased volume gives
rise to a functional flow murmur during the systole which is often best heard between the apex and the left sternal border.

Additionally in sever AR, the backflow of the blood during the diastole causes a rumbling mid-diastolic jet murmur (Austin-Flint murmur). The
jet flow strikes the anterior leaflet of the mitral valve and closes it prematurely.

(Option A) Mitral valve stenosis is characterized by an opening snap after the S2 and diastolic rumbling murmur best heard in the apex.

(Option B) Mitral valve regurgitation causes a holosystolic systolic murmur radiating to the axilla and best heard over the apex.

(Option C) Aortic stenosis is associated with an ejection murmur in aortic area that may be associated with thrills. It has a harsh quality and
may radiate to the carotids.

(Option E) Tricuspid valve regurgitation is associated with a holosystolic murmur in some patients.

References

• Medscape - Aortic Regurgitation

Last updated:
Time spent: QID:389
2023-2-12

84 of 1943
A 67-year-old man presents to the emergency department with facial puffiness and swelling of the right arm and upper chest. He is
moderately short of breath. On examination, his face, arm and upper chest are slightly cyanosed and puffy. The rest of the exam is
inconclusive. Which one of the following would be the most appropriate next investigation?

A. Chest X-ray.

B. Echocardiography.

C. ECG.

D. RAST.

E. Angiography.

Correct
45% answered correctly

Explanation:

Correct Answer Is A

The clinical picture is consistent with superior vena cava (SVC) syndrome. SVC syndrome results from any condition that leads to obstruction
of blood flow through the SVC. Obstruction can be caused by external compression of the SVC by adjacent pathologic processes involving the
right lung, lymph nodes, and other mediastinal structures, or by thrombosis within the SVC. In some cases, both external compression and
thrombosis coexist.

Conditions that can lead to SVC syndrome can be malignant or non-malignant:

Malignant causes - malignant mediastinal tumors are the most common cause (>80%). Of malignant tumors bronchogenic carcinoma
accounts for 75-80% of cases, with most of these being small cell carcinomas. Non-Hodgkin lymphoma is responsible for 10-15% of
mediastinal malignancies resulting in SVC syndrome. Rare malignant causes include Hodgkin lymphoma, metastatic cancers, primary
leiomyosarcoma of the mediastinal vessels and plasmocytomas.

Non-malignant causes - some of these causes are:

Mediastinal fibrosis
Vascular diseases, such as aortic aneurysm, vasculitis, and arteriorvenous fistulas
Infections, such as histoplasmosis, tuberculosis, syphilis, and actinomycosis
Benign mediastinal tumors such as teratoma, cystic hygroma, thymoma, and dermoid cyst
Cardiac causes, such as pericarditis and atrial myxoma
Thrombosis related to the presence of central vein catheters

Early in the clinical course of SVC syndrome, partial obstruction of SVC may be asymptomatic, or the symptoms are so minor that are
overlooked, but as it advances toward complete SVC obstruction, the classic symptoms and signs become more obvious:

Dyspnea - the most common symptom (63% of patients)


Facial swelling
Head fullness
Cough
Arm swelling
Chest pain
Dysphagia
Orthopnea
Distorted vision
Hoarseness
Stridor
Headache
Nasal stuffiness
Nausea
Pleural effusion
Lightheadedness

The characteristic physical findings include venous distention of the neck and chest wall, facial edema, upper extremity edema, mental status
changes, plethora, cyanosis, papilledema, stupor and even coma. Bending forward or lying down can aggravate the signs and symptoms.

Since most cases of SVC are due to mediastinal malignancies, a chest X-ray is always the most appropriate initial investigation). Chest X-ray
may reveal a widened mediastinum or a mass in the right side of the chest. One study showed that only 16% of patients with SVC syndrome
had a normal chest X-ray.

85 of 1943
CT scan, MRI, and angiography (option E) can be use for more detailed evaluation, but not as initial assessment.

RAST (option D) stands for radioallergosorbent test and is a blood test for finding the allergen a patient is allergic to, and has no role in
diagnosis of SVC syndrome.

ECG (option C) and echocardiography (option B) are not diagnostic for SVC.

References

• Medscape - Superior Vena Cava Syndrome

• UpToDate - Malignancy-related Superior Vena Cava Syndrome

Last updated:
Time spent: QID:604
2023-2-12

86 of 1943
A 45-year-old man presents to your clinic with hand and arm swelling and pain which developed a few hours after working with a chainsaw. He
has no family or personal history of thrombophilia or deep vein thrombosis. On physical exam, he is in good health otherwise. Generalized
non-pitting edema of the right hand and arm up to the shoulder is noted. The arm is slightly cyanosed, but not warm to touch. The remainder
of the exam is inconclusive. Which one of the following is the most likely diagnosis?

A. Lymphedema.

B. Subclavian vein thrombosis.

C. Brachial plexus injury.

D. Cellulitis.

E. Lymphangitis.

Correct
45% answered correctly

Explanation:

Correct Answer Is B

The clinical picture is suggestive of upper extremity deep vein thrombosis (UEDVT) (thrombosis of axillary or subclavian vein). UEDVT is rare
but increasing in incidence with considerable morbidity. Pulmonary embolism may occur in as many as one-third of the patients.

The increased incidence of UEDVT in the recent decades is directly related to increasing use of central catheterization for chemotherapy, bone
marrow transplantation, dialysis and parenteral nutrition. UEDVT has been reported in 25% of those with central catheters.

UEDVT is classified as primary or secondary on the basis of pathogenesis:

PRIMARY UEDVT

Primary UEDVT is a rare disorder (2 per 100 000 persons per year) which is subdivided into two other categories:

Effort thrombosis (the so-called Paget-Schroetter Syndrome) - Patients with Paget-Schroetter Syndrome develop spontaneous UEDVT, usually
in their dominant arm, after strenuous activity such as rowing, wrestling, weight lifting, or baseball pitching, but are otherwise young and
healthy. The heavy exertion causes microtrauma to the vessel intima and leads to activation of the coagulation cascade. Significant
thrombosis may occur with repeated insults to the vein wall, especially if mechanical compression of the vessel is also present.

Idiopathic UEDVT - Patients with idiopathic UEDVT have no known trigger or obvious underlying disease. Idiopathic UEDVT can be,
however, associated with occult cancer. In one study, one fourth of patients presenting with idiopathic UEDVT were diagnosed with cancer
(most commonly lung cancer or lymphomas) within one year of follow-up.

Thoracic outlet obstruction refers to compression of the neurovascular bundle (brachial plexus, subclavian artery, and subclavian vein) as it
exits the thoracic inlet. Although this disorder may initially cause intermittent, positional extrinsic vein compression, repeated trauma to the
vessel can result in dense, perivascular, fibrous scar tissue formation that will compress the vein persistently. Compression of the subclavian
vein typically develops in young athletes with hypertrophied muscles who do heavy lifting or completely abduct their arms. Cervical ribs, long
transverse processes of the cervical spine, musculofascial bands, and clavicular or first rib anomalies are sometimes found in these patients.
Therefore, cervical spine and chest plain films should be obtained in all patients undergoing evaluation for thoracic outlet syndrome.

Regardless of the etiology, UEDVT can be associated with the following complications:

Pulmonary embolism (in one-third of patients)


Persistent upper extremity pain and swelling
Superior vena cava syndrome
Loss of vascular access.

SECONDARY UEDVT

Secondary UEDVT develops in patients with central venous catheters, pacemakers, or cancer and accounts for most cases of UEDVT.
Catheter-related thrombosis is caused by several factors including damage to the vessel wall during catheter insertion or infusion of
medication, or the catheter impeding the blood flow through the vessel and stasis.

Regardless of the etiology, UEDVT can have the following clinical features:

Vague shoulder or neck discomfort


Pain and discomfort of the arm
Non-pitting edema of the hand and arm
87 of 1943
Extremity cyanosis
Dilated cutaneous veins
Supraclavicular fullness
Fever (often low-grade) – higher fevers may be caused by thrombophlebitis or the underlying malignancy
Facial edema and cyanosis if there superior vena cava syndrome has occurred

If the thoracic outlet syndrome is the etiology of the UEDVT the additional following findings are other possible findings:

Brachial plexus tenderness


Pain radiating to the arm and forearm
Arm or hand atrophy
Hand weakness
Referred pain from the brachial plexus to the fourth and fifth fingers

NOTE (1) – At occasions, UEDVT may be asymptomatic.

NOTE (2) – Thrombophilia seems to be associated with increased risk of UEDVT.

(Option A) Lymphedema can be a differential diagnosis, but there is no predisposing factor in history favoring this diagnosis.

(Option C) With brachial plexus injury, neurologic symptoms would have been present. However, UEDVT caused by thoracic outlet syndrome
may be associated with concurrent neurologic dysfunctions of the brachial plexus.

(Options D and E) Lymphangitis and cellulitis are often associated with more systemic symptoms e.g. fever. The patient may look ill. Erythema
and warmth of the limb are other features that are absent here.

References

• http://circ.ahajournals.org/content/106/14/1874.fu

• http://emedicine.medscape.com/article/424777-overv

Last updated:
Time spent: QID:640
2023-2-12

88 of 1943
Worried parents of a 4-year-old boy bring him to the Emergency Department because he is breathless and his heart is racing. An ECG strip is
obtained and is as follows. Which one of the following is the most appropriate immediate management of this child?

A. Reassure the parents as it is sinus tachycardia.

B. Intravenous adenosine.

C. Intravenous lignocaine.

D. Intravenous beta blockers.

E. Immerse the child's face in cold water.

Incorrect. Correct answer is B


45% answered correctly

Explanation:

Correct Answer Is B

The ECG shows episodes of tachycardia with wide monomorphic QRS complexes that do not sustain and terminate spontaneously. This is
called non-sustained wide QRS complex tachycardia. A wide complex for this matter is defined as a QRS of >0.09 second (90 msec). Wide
QRS complex tachycardia is either due to ventricular tachycardia or a supraventricular tachycardia (SVT) with aberrancy. While most wide QRS
complexes in adults originate from ventricles, in children they often represent SVT with aberrancy. However, due to the potential progression
of ventricular tachycardia (VT) to ventricular fibrillation and cardiac arrest, any wide QRS complex tachycardia should be considered VT and
managed accordingly until proven otherwise.

Asymptomatic non-sustained wide QRS complex tachycardia does not often require treatment other than identifying and correction of the
underlying cause. Symptomatic non-sustained wide QRS complex tachycardia and sustained wide QRS complex tachycardias whether
symptomatic or asymptomatic require treatment.

Treatment options depend on the patient’s hemodynamic status:

In patients with wide QRS complex tachycardia who are hemodynamically unstable, represented by hypotension, acutely altered
mental status or signs of shock, the treatment is with electrical cardioversion if there is a pulse. Defibrillation is the choice in the
absence of a pulse.

In adult patients with sustained wide QRS who are asymptomatic or have symptoms other than those mentioned above, or non-
sustained wide QRS complex tachycardia with symptoms other than those mentioned above, pharmacotherapy with an antiarrhythmic
medication is the treatment of choice. Amiodarone is first-line. Lignocaine or sotalol can be used alternatively.

In children with sustained wide QRS complex tachycardia who are asymptomatic, or have symptoms but are hemodynamically stable
(no hypotension, no acutely altered mental state, no signs of shock), or in children who have non-sustained wide QRS complex
tachycardia with symptoms other than those related to hemodynamic instability, it is recommended that treatment start with
adenosine if the rhythm is regular and QRS complexes are monomorphic. The rationale behind this is the fact that such tachycardias
are often SVTs with aberrancy; hence, responsive to adenosine. However, if there is no response to adenosine, VT should be
considered and treated with amiodarone or procainamide after expert consultation or under expert supervision.

This child has symptomatic non-sustained wide QRS complex tachycardia but his symptoms are not any of those representing hemodynamic
instability; so, the most initial treatment to consider for him must be intravenous adenosine.

(Option A) This child has symptomatic non-sustained wide complex tachycardia, probably with an underlying pathology. His parents cannot
be reassured, and this child’s tachycardia requires treatment as well as investigations for an underlying cause.

(Option C) Lignocaine, amiodarone, procainamide, etc. will be considered after adenosine proved ineffective to control the tachycardia, and
the wide QRS complex tachycardia is likely to be VT. These medications should be used after consultation or under supervision of an expert.

NOTE - of these medications, amdiodarone is the safest to use.

89 of 1943
(Option D) Oral (not intravenous) beta blockers are used as first-line in treatment of non-sustained wide QRS complex tachycardias in
outpatient setting if the symptoms are mild or not representative of hemodynamic instability.

(Option E) Vagal stimulation by maneuvers such as immersion of face in cold water, Valsalva maneuver, carotid massage etc. are used as the
initial management for patients with narrow QRS complex tachycardia because such tachycardias are not likely to have ventricular origin. This
child has wide QRS.

References

• RCH - Supraventricular Tachycardia SVT

• Therapeutic Guidelines – Cardiology

• UpToDate - Management and evaluation of wide QRS complex tachycardia in children

Last updated:
Time spent: QID:680
2023-2-12

90 of 1943
A 72-year-old man presents to the Emergency Department with severe retrosternal chest pain and shortness of breath. ECG reveals ST
segment elevation myocardial infarction (STEMI). Angiography is performed which shows 70% stenosis of the right coronary artery and 30%
of the left. Which one of the following would be the most appropriate management option?

A. Pacemaker.

B. Thrombolytic therapy.

C. Angioplasty of the right coronary artery.

D. Angioplasty of the left coronary artery.

E. Angioplasty of both right and left coronary arteries.

Incorrect. Correct answer is C


45% answered correctly

Explanation:

Correct Answer Is C

Timely angioplasty is the gold-standard treatment of STEMI. Angioplasty commonly includes passing an arterial catheter from the femoral or
radial artery into the aorta and from there into the coronary arteries. Contrast material is then injected to visualize a map of coronary arteries
and possible lesions.

Significant stenosis (>50%) may be considered for ballooning, atherectomy, stenting, etc. Insignificant coronary artery disease is defined as a
stenosis of less than 50% (consensus-based). Insignificant stenosis does not often require intervention.

In this patient, only the right coronary artery (70% stenotic) requires angioplastic intervention.

(Option A) Pacemaker insertion is not a treatment option for myocardial infarction; however, it might be used for treatment of heart block as a
complication of MI at particular occasion.

(Option B) Thrombolytic therapy was the correct option if angioplasty would not be available in a timely fashion.

(Options D and E)The left coronary artery does not require intervention for now.

References

• PubMed - What percent luminal stenosis should be used to define angiographic coronary artery disease for noninvasive test evaluation?

• AHA - ACC/AHA Guidelines for Percutaneous Coronary Intervention

Last updated:
Time spent: QID:729
2023-2-12

91 of 1943
A 56-year-old Aboriginal Australian man is brought to the Emergency Department of a local hospital with severe chest pain radiating to the left
arm, hypotension, and bradycardia. An ECG confirms ST elevation in leads I, aVL, V5 and V6. Which one of the following fibrinolytic agents
cannot be used for treatment of this patient?

A. Alteplase.

B. Tenecteplase.

C. Urokinase.

D. Streptokinase.

E. Reteplase.

Incorrect. Correct answer is D


45% answered correctly

Explanation:

Correct Answer Is D

There is a high prevalence of IgG anti-streptokinase antibody among Aboriginal Australian patients, making this drug an inappropriate option
for thrombolytic therapy in acute ST elevation myocardial infarction in this patient group. This antibody makes treatment with this agent
ineffective.

References

• MJA - Guidelines for the management of acute coronary syndromes 2006

• Therapeutic Guidelines - Cardiovascular

Last updated:
Time spent: QID:23
2023-2-12

92 of 1943
A 71-year-old man with a history of congestive heart failure for the past 10 years, controlled on perindopril and hydrochlorothiazide, presents
to the Emergency Department with a one-day history of palpitation. He has a blood pressure of 135/90 mmHg, an irregular pulse rate of
98 bpm, and a respiratory rate of 18 breaths per minute. Lungs are clear to auscultation. No ankle edema or raised jugular venous pressure is
noted. An ECG is obtained and is shown in the following photograph. Which of the following would be the next best step in management?

A. Aspirin.

B. Metoprolol.

C. Warfarin.

D. Digoxin.

E. Diltiazem.

Incorrect. Correct answer is B


45% answered correctly

Explanation:

Correct Answer Is B

The ECG is characteristic of atrial fibrillation (AF). The most appropriate initial management of AF in a patient with stable hemodynamic
status is rate control. Rate control often leads to an improvement in symptoms in patients with heart failure. In addition, slowing of the
ventricular rate often leads to moderate or, in some cases, marked improvement in left ventricular function.

Cardioselective beta-blockers (metoprolol, atenolol, and carvedilol) are first-line medications for this purpose. Anticoagulation should be
considered if AF persists beyond 48 hours or if the duration is unknown. Digoxin should be added if a second agent is necessary to achieve
adequate rate control. However, the efficacy of digoxin decreases with physical activity.

If rate control with either beta-blockers or a combination of beta-blockers and digoxin has not been achieved, amiodarone may be useful
either alone or in combination with other rate-slowing agents.

It should be noted that in patients with decompensated heart failure the initiation or increase of beta blockers is contraindicated. In such
patients, the use of digoxin is suggested. However, digoxin is often ineffective if used alone, especially in patients with an elevated
sympathetic tone. Also, for patients with acute AF with the rapid ventricular response associated with heart failure, amiodarone may be
helpful to slow the ventricular rate. Although usually used for rhythm control, amiodarone can also provide a degree of rate control while it is
being loaded and thereafter. Amiodarone is not recommended as a chronic rate-control medication, but in the acute setting can assist with
rate control as it is being loaded, or can be used as a temporary rate-control medication in a patient who is unable to tolerate other therapies.

Even though this patient has heart failure, since the heart failure is rather stable and not decompensated, beta-blockers (metoprolol) are the
most appropriate next step in management.

NOTE - Acute decompensated heart failure (ADHF) can be defined as the sudden or gradual onset of the sign and/or symptoms of heart
failure requiring unplanned office visits, emergency department visits, or hospitalization. Regardless of the underlying precipitant of the
exacerbation, pulmonary and systemic congestion due to increased left- and right-heart filling pressure is nearly a universal finding in
ADHF.

Acute decompensated heart failure (ADHF) may present with:

Pulmonary edema
Cardiogenic shock
Weight gain and fluid retention

93 of 1943
Exertional dyspnea
Orthopnea
Raised jugular venous pressure
Peripheral edema

(Option A) Aspirin is used for patients in whom AF is not caused by a valvular lesion and has a CHA2DS2 - VASc score of zero (or 1 in selected
cases).

(Option C) Anticoagulation with warfarin (after reaching an INR of 2-3 by initial administration of heparin) is considered the second most
important step in whom AF has lasted over 48 hours or is of unknown duration. With an AF of only one-day duration anticoagulation is not
required for now.

(Option D) Digoxin could have been an appropriate option if the patient’s heart failure was decompensated and beta blockers were
contraindicated.

(Option E) Although non-dihydropyridine calcium channel blockers (verapamil and diltiazem) are second-line medications for rate control in
patients with AF, extreme caution should be considered in patients with heart failure, due to the risk of heart failure exacerbation and
decompensation.

References

• Atrial Fibrillation Management - The Heart Foundation

• UpToDate - The management of atrial fibrillation in patients with heart failure

• AHA Journals - Atrial Fibrillation and Acute Decompensated Heart Failure

Last updated:
Time spent: QID:730
2023-2-12

94 of 1943
A 72-year-old woman is brought to the Emergency Department with chest pain of several hours duration. Of physical findings on examination,
a blood pressure of 98/50 mmHg and an irregular pulse of 120 bpm are remarkable. An ECG shows ST elevation in leads V2 and V4. Which
one of the following will be the most appropriate initial management option?

A. Cardioversion.

B. Thrombolysis with TPA.

C. Lidocaine.

D. Verapamil.

E. Digoxin.

Incorrect. Correct answer is B


45% answered correctly

Explanation:

Correct Answer Is B

Chest pain and ST elevation in leads V2 through V4 suggests ST-segment elevation myocardial infarction (STEMI) of the anterior wall. On the
other hand, an irregular pulse of 120 bpm suggests atrial fibrillation (AF). AF is seen in 10-15% of the patients with acute myocardial infarction
(MI) as a complication.

The onset of AF in the first hours of MI is usually caused by left ventricular failure, ischemic injury to the atria, or right ventricular infarction.
Pericarditis and all conditions leading to elevated left atrial pressure can also lead to AF in association with an MI. The presence of AF during
an MI is associated with an increased risk of mortality and stroke, particularly in patients who have anterior wall MI.

Although both conditions are simultaneously present in this patient, urgent management of the MI takes precedence over that of AF. If the
patient presents within the first 12 hours of symptoms onset, reperfusion therapy either by percutaneous coronary intervention (PCI) or
thrombolytic therapy with thrombolytic medications such as tissue plasminogen activator (TPA), reteplase, alteplase, tenecteplase, etc.
should be considered as the most appropriate next step in management. PCI (not an option), if performed in a timely fashion and by an
experienced interventional cardiologist, has better outcomes and fewer complications compared to thrombolytic therapy.

AF persisting after reperfusion therapy should managed accordingly:

Immediate electrical cardioversion is indicated for all patients who are hemodynamically unstable, such as those with new or worsening
ischemic pain and/or hypotension. Synchronized electrical cardioversion to treat atrial fibrillation begins with 200 J (or the biphasic
equivalent). Conscious sedation (preferred) or general anesthesia is advisable prior to cardioversion.

For patients who do not develop hypotension, a beta-blocker can be used. For example, metoprolol may be given in 5-mg intravenous boluses
every 5-10 min, with a maximum dose of 15 mg. Intravenous diltiazem is an alternative for slowing the ventricular rate, but it should be used
with caution in patients with moderate-to-severe heart failure. In patients with new-onset sustained tachycardia (absent before MI),
conversion to sinus rhythm should be considered as an option.

Atrial fibrillation and atrial flutter confer an increased risk of thromboembolism; therefore, anticoagulation with either unfractionated heparin
or low molecular weight heparin (LMWH) should be started if contraindications are absent.

References

• Therapeutic Guidelines - Cardiovascular

• Medscape - Complications of Myocardial Infarction

• European Heart Journal - Atrial fibrillation in acute myocardial infarction

Last updated:
Time spent: QID:830
2023-2-12

95 of 1943
Which one of the following is the most appropriate treatment for venous ulcers of lower limb?

A. Bed rest with limb elevation.

B. Antibiotics.

C. Tissue debridment.

D. Compression stocking and a walking program.

E. Aspirin and statins.

Incorrect. Correct answer is D


45% answered correctly

Explanation:

Correct Answer Is D

Venous hypertension is associated with morphologic changes in the capillary and lymphatic microcirculation, resulting in important
physiologic changes such as capillary leak, fibrin deposition, erythrocyte and leukocyte sequestration, thrombocytosis and inflammation.
Collectively, these processes impair oxygenation of the skin and subcutaneous tissue. Sever venous hypertension and hypoxia leads to
edema, subcutaneous fibrosis, hyperpigmentation and ulcer formation.

Continuous compression therapy, typically achieved with gradual compression stockings or compression bandage is a very important and
essential part of treatment of chronic venous disease with or without ulceration.

The mechanism by which compression therapy produces beneficial effects is not yet understood; however, the following have been
suggested:

improvement in venous hemodynamics by reducing venous reflux

increasing deep venous flow velocity

improvement in lymphatic flow and cutaneous microcirculation

decreasing ambulatory venous pressure

alteration in certain pro-inflammatory matrix metalloproteinases present in chronic ulcers that result in favorable changes promoting
ulcer healing

compression that is intermittent enhances fibrinolysis, a potentially important mechanism in reducing fibrosis and enhancing ulcer
healing

Compression therapy is contraindicated in patients with moderate to severe peripheral artery disease (PAD), and should be used with extreme
caution in patients with mild to moderate PAD, evident by an ankle-brachial index (ABI) of <0.9 but >0.6.

In the presence of arterial disease, compressiom stocking can cause skin necrosis that in few instances may lead to amputation. Active
cellulitis is another contraindication for compression therapy.

References

• http://www.racgp.org.au/afp/2014/september/managem

• https://www.nhmrc.gov.au/_files_nhmrc/publications

Last updated:
Time spent: QID:920
2023-2-12

96 of 1943
A 57-year-old man presents to the Emergency Department with sudden onset of severe pain and pallor in the left lower limb. He is a known
case of ischemic heart disease (IHD) and peripheral vascular disease (PVD). On examination, left femoral pulse is felt, but the left popliteal
pulse is absent. Which one of the following is the most appropriate next step in management?

A. Intravenous unfractionated heparin (UFH).

B. Low molecular weight heparin (LMWH).

C. Warfarin.

D. Verapamil.

E. Embolectomy.

Correct
45% answered correctly

Explanation:

Correct Answer Is A

Sudden onset of pain and pallor suggests acute limb ischemia. Absence of popliteal pulse confirms the diagnosis with high certainty. In
approach to acute limb ischemia, administration of bolus dose of unfractionated heparin (UFH), intravenously, is the most appropriate next
step in management once the diagnosis of acute arterial occlusion has been made by history and physical examination.

Anticoagulation prevents further propagation of thrombus and inhibits thrombosis distally in the arterial and venous systems due to low flow
and stasis. Administration of heparin should not be delayed while waiting for diagnostic procedures to be performed. A loading dose of 5000
IU is given, which is then followed by 1250IU/hour. Further adjustment is guided by active partial thrombin time (APTT).

(Option B) LMWH can be uses with the same efficacy of UFH, except in unstable patient or where a surgical are other invasive procedure
may follow, in which case UFH must be used instead because the anticoagulation effect of UFH can be readily reversed by protamine sulfate.
This patient will require either vascular catheterization and embolectomy, or thrombolytic therapy or, in worst case scenario, amputation of his
leg. This makes UFH the most appropriate option.

(Option C) Warfarin is an oral antagonist of vitamin K. In early stages after initiation, it is associated with increased risk of clot formation (pro-
thrombotic). Warfarin is indicated for long-term management of this patient.

(Option D) Verapamil is a non-dihydropyridine calcium channel blocker (CCB). Its use has no therapeutic benefit in patients with acute limb
ischemia.

(Option E) Embolectomy is one of the options for definite treatment of this patient, but heparin, as the crucial initial management, must be
started before decision as to definite treatment is made.

References

• Medscape - Peripheral Vascular Disease

• BMJ - Acute limb ischaemia

Last updated:
Time spent: QID:972
2023-2-12

97 of 1943
A 65-year-old woman presents to the Emergency Department after she collapsed at work. She was walking to her desk when she suddenly
lost consciousness and fell down. According to her colleague, who is accompanying her, she was ‘out’ for only a few seconds, and regained
consciousness shortly. One examination, she is overweight, has a blood pressure of 110/75 mmHg, pulse rate of 110 bpm and respiratory rate
of 18 breaths per minute. You are arranging for an ECG when she collapses again. On a quick review, she is found to have a blood pressure of
80/45 mmHg. You feel no radial or carotid pulse. An ECG is obtained which is shown in the following photograph. Which one of the following
is the most appropriate next step in management?

A. Intravenous bolus of normal saline.

B. Intravenous amiodarone.

C. Synchronized cardioversion.

D. Cardiopulmonary resuscitation and defibrillation.

E. Cardiac catheter ablation.

Incorrect. Correct answer is D


45% answered correctly

Explanation:

Correct Answer Is D

The ECG is typical for ventricular tachycardia (VT). In each lead, QRS complex are of same morphology; therefore, this patient has a
monomorphic VT.

Approach to sustained VT (VT lasting more than 30 seconds) depends on the patient’s hemodynamic status. Those with hemodynamic
instability are in emergency need for reversion of the rhythm to a sinus rhythm. In the presence of a palpable pulse, the initial management is
by synchronized cardioversion if a pulse is palpable, and defibrillation (unsynchronized) if no pulse is felt.

Since this patient is pulseless, CPR should be immediately started and defibrillation performed when available and ready.

In patients with a shockable rhythm (ventricular fibrillation and pulseless VT), the first action is start chest compression while waiting for
defibrillator to be ready. First defibrillation should be followed by 2 minutes of CPR. After the second defibrillation, adrenaline should be
started and repeated every second cycle.

NOTE - Hemodynamic instability is defined as the presence of any of the following:

Chest pain
Hypotension (systolic blood pressure <90mmHg)
Perfusion-related confusion or unconsciousness
Unconsciousness
Heart failure

(Option A) In a patient with VT, the cause of hypotension is decreased cardiac output as result of pump failure not hypovolemia; therefore,
intravenous fluids are useless.

(Option B) Antiarrhythmic medications such as amiodarone are indicated as first-line management option in patients who are
hemodynamically stable despite presence of VT, or for sustaining a sinus rhythm after treatment with cardioversion or defibrillation.

(Option C) Synchronized cardioversion was initial management and the correct option if the patient had a palpable pulse.

(Option E) Cardiac catheter ablation is the definite treatment in patients with regular supraventricular tachyarrhythmias if the cause is found to
be an accessory pathway bypassing the normal cardiac conductive system e.g. Wolff-Parkinson-White syndrome.

References
98 of 1943
• Australian Resuscitation Council - Guideline 11.9 - Managing Acute Dysrhythmias

• http://www.racgp.org.au/afp/2012/june/basic-and-ad

• Therapeutic Guidelines - Cardiovascular


Last updated:
Time spent: QID:985
2023-2-12

99 of 1943
A 60-year-old man presents to the Emergency Department with complaint of palpitation for the past 2 weeks. On examination, he has blood
pressure of 125/95 mmHg, and an irregular heart rate of 160 bpm. An ECG shows atrial fibrillation (AF). He is started on metoprolol for rate
control, and electrical cardioversion is planned. Which one the following should be used for anticoagulation?

A. Heparin, 24 hours before the procedure.

B. Warfarin 4 weeks before, to 4 weeks after the procedure.

C. Aspirin.

D. Apixaban from 48 hours before, to 48 hours after the procedure.

E. Flecainide.

Incorrect. Correct answer is B


45% answered correctly

Explanation:

Correct Answer Is B

Converting AF to sinus rhythm is associated with a small risk of thromboembolic events. The risk can be due to either pre-existing thrombus
(more common) or de novo thrombus formation.

Embolization after conversion to sinus rhythm is often caused by the dislodgement of left atrial (LA) thrombi that has been formed during
atrial stagnation; however, in a minority of patients the clot forms at the time of cardioversion.

This risk is increased in the immediate post-cardioversion period, whether planned or spontaneous. Patients undergoing cardioversion of AF
of more than 48 hours duration represent a high-risk group, with an embolic risk of 1-5% in the first month if prophylactic anticoagulation is
not given.

The 1-month risk of post-cardioversion thromboembolism can be decreased to less than 1% with anticoagulation starting from 1 month (4
weeks) before to 1 month (4 weeks) after the procedure. Oral anticoagulation with warfarin with a target INR of 2-3 is the most common
method used.

Heparin is used initially before, or at the same time with warfarin to counteract the prothrombotic effects of warfarin early in the course of
use, and is stopped once INR is within the therapeutic range of 2-3.

(Option A) Heparin is also used for acute anticoagulation if emergency cardioversion has been considered, such as in patients with
hemodynamic compromise due to AF, or who have failed to respond to rate control medications and are symptomatic.

(Option C) Aspirin is unlikely to provided adequate prophylaxis against thromboembolism prior to and after cardioversion.

(Option D) Apixaban and rivaroxaban are oral factor Xa inhibitor coagulants. Rivaroxaban has shown equal efficacy to warfarin. Apixaban is
suggested to be superior to warfarin; however, their efficacy and safety prior to and immediately after cardioversion has not been adequately
studied.

(Option E) Flecainide is an antiarrhythmic medication used for pharmacological cardioversion with no effectiveness in preventing
thromboembolism.

References

• UpToDate - Prevention of embolization prior to and after restoration of sinus rhythm in atrial fibrillation

Last updated:
Time spent: QID:1016
2023-2-12

100 of 1943
A 65-year-old female patient undergoes percutaneous coronary intervention and stent placement through femoral artery, and is started on
aspirin andclopidogrel. After 24 hours, she develops a pulsatile painful mass in the groin though which the catheter was sent in. Which one of
the following is the definitive treatment of this mass?

A. Massage and application of compression.

B. Surgical repair.

C. Vitamin K.

D. Angiography.

E. Injection of thrombin into the mass.

Incorrect. Correct answer is E


45% answered correctly

Explanation:

Correct Answer Is E

The scenario is a typical description of a pseudoaneurysm as a complication of femoral artery catheterization.

A pseudo aneurysm is a hematoma that forms as the result of a leaking hole in an artery. The hematoma forms outside the arterial wall and is
contained by the surrounding fibromuscular tissue. The hematoma must continue to communicate with the artery to be considered a
pseudoaneurysm.

Pseudoaneurysm occurs in up to 7.5% of femoral artery catheterizations and can cause distal embolization, extrinsic compression on the
neurovascular structures, rupture, and hemorrhage.

A pseudoaneurysm presents with a painful pulsatile groin mass. A bruit over the mass may or may not be heard. Duplex Doppler ultrasound
may provide evidence of extra-arterial flow or there may be classic ‘to-and-fro’ Doppler waveform in the neck of the pseudoaneurysm.

Ultrasound-guided thrombin injection for the pseudoaneurysms of the iliac, femoral and peroneal arteries is safe, effective, and associated
with few complications. It has emerged as the preferred treatment modality for pseudoaneurysms occurring as a result of percutaneous
femoral arterial interventions (success rate: 97%).

The procedure should be performed by a physician and an ultrasonographer to enable continuous visualization of the pseudoaneurysm.
Thrombin is injected into the sac of the pseudoaneurysm away from the neck under direct ultrasound guidance.

(Option A) The ultrasound-guided compression is successful in 90% of case and was the treatment of choice previously; however, this
approach is not favorable anymore because it needs prolonged compression time (up to 120 minutes), makes the patient uncomfortable, is
associated with early recurrence and has limited success in treating large pseudoaneurysms.

(Option B) Urgent surgical exploration is indicated for a threatened limb and when a percutaneous approach is not feasible. This method was
the treatment of choice before 1985.

(Option C) Vitamin K is not a treatment option. Any options suggesting cessation of clopidogrel is incorrect as this is associated with high
mortality rate in a patient who has just undergone angioplasty. Moreover, it does not treat the aneurysm.

(Option D) Angiographic intervention via a retrograde approach from the contralateral common femoral artery is only indicated if acute vessel
occlusion due to distal emboli occurs. The event presents with the patient complaining of pain, pallor, parenthesis or decreased movement in
the respective limb. Clinical examination may reveal a cold ischemic limb with absent pulses, an ankle-brachial index (ABI) <0.5, or absent
color flow and Doppler waveform on duplex ultrasound in the index artery.

References

• http://www.medscape.com/viewarticle/580670_2?pa=yC

Last updated:
Time spent: QID:1074
2023-2-12

101 of 1943
A 30-year-old school teacher presents to the Emergency Department after she collapsed at school during exercise. She was unconscious for
few seconds with full recovery afterwards. She now complains of shortness breath but denies chest pain. On examination, a systolic murmur
is heard over the right second intercostal space radiating to the apex. Which one of the following could be the most likely diagnosis?

A. Patent ductus arteriosus.

B. Aortic stenosis.

C. Hypertrophic obstructive cardiomyopathy.

D. Ventricular septal defect.

E. Mitral stenosis.

Incorrect. Correct answer is B


45% answered correctly

Explanation:

Correct Answer Is B

Exercise-induced collapse and a murmur on examination are most likely to be caused by a structural heart disease such as valvular lesions or
hypertrophic cardiomyopathies. Arrhythmias can also be a cause of exercise-induced hemodynamic collapse; however, arrhythmias usually
do not give rise to murmurs.

This patient has a systolic murmur best heard over the right second intercostal space radiating to the apex. Of the options, only aortic
stenosis (AS) is capable of producing such murmur.

The murmur associated with AS is a systolic ejection murmur. It is typically heard best at the base of the heart in the ‘aortic’ area that is the
right second intercostal space. The murmur has a harsh quality and can be transmitted equally to the carotid arteries. The murmur may also
radiate to the apex of the heart where it may have a different quality. It is also common for patients with AS to have mild mitral regurgitation
with a systolic murmur best heard at the apex.

(Option A) The murmur in patent ductus arteriosus (PDA) is heard in both systole and diastole (Gibson’s murmur or machinery murmur) with
greatest intensity in the left infraclavicular region. Furthermore, PDA is congenital heart disease and very unlikely to have presented this late.

(Option C) Hypertrophic obstructive cardiomyopathy (HOCM) can cause exertional syncope; however, characteristics of the murmur
described in the scenario make HOCM less likely. Patients with HOCM may develop several types of systolic murmurs, but the two most
common are related to left ventricular outflow tract (LVOT) obstruction and mitral regurgitation. The LVTO obstruction murmur is a harsh
crescendo-decrescendo murmur best heard over the left sternal border that can radiate to the axilla but not the neck.

Papillary muscle and/or chordae tendineae abnormalities associate with HOCM results in mitral valve regurgitation that can produce a mid- or
late-systolic murmur that is best heard at the apex.

(Option D) Ventricular septal defect is associated with a systolic murmur with greatest intensity in the left sternal border.

(Option E) Sever mitral stenosis can cause syncope during exercise but the murmur is diastolic with the greatest intensity in the apex.

References

• Medscape - Aortic Stenosis

• Clinical manifestations and diagnosis of aortic stenosis in adults

Last updated:
Time spent: QID:1183
2023-2-12

102 of 1943
A 54-year-old man presents to the emergency department with complaint of retrosternal chest pain that worse on inspiration and lying back
and relieved by bending forward. On examination, he has a blood pressure of 138/97mmHg, pulse rate of 110bpm, shallow respiration at a
rate of 26 breaths per minute and a temperature of 38.5°C. An ECG is obtained which is shown in the following photograph. Which one of the
following is the most appropriate next step in management after administration of supplemental oxygen?

A. Non-steroidal inflammatory drugs (NSAIDs).

B. Corticosteroids.

C. Arrange for cath lab.

D. Heparin.

E. Thrombolysis.

Correct
45% answered correctly

Explanation:

Correct Answer Is A

The clinical picture and the ECG findings are characteristic of pericarditis as the most likely diagnosis. Pericarditis presents with pleuritic
chest pain (worsening with respiration) that often intensifies on lying down and relieved by bending forward. A friction rub (not a given finding
in this patient) may be present. Fever, often low-grade, can be a feature.

Pericarditis is an important differential diagnosis to consider in patients with chest pain. It is diagnosed in 5% of patients presenting to
emergency departments with chest pain in the absence of a myocardial infarction. Viral infections and idiopathic pericarditis are the most
common etiologies.

Troponin can be elevated in some patients; however, unlike in myocardial infarction, raised troponin levels in pericarditis are not associated
with negative prognosis.

Pericardial effusion is common complication that can follow in as many as 60% of patients. Cardiac tamponade can be a serious life-
threatening condition complicating pericardial effusion.

Findings in pericarditis can include diffuse PR depression and diffuse ST elevation with upward concavity. Unlike pericarditis, myocardial
infarction typically produces ST elevation with upward convexity. ST elevation in pericarditis is often seen in pericardial and limb leads and
involves more than one coronary vascular territory.

Nonsteroidal anti-inflammatory drugs (NSAIDs) are the first-line medications for treatment of pericarditis. Of NSAIDs, ibuprofen is the drug of
choice due to its favorable effect on the coronary flow and large dose range. Depending on severity and response, 300–800 mg every 6–8
hours may be initially required and can be continued for days or weeks, ideally until the effusion has disappeared. Gastrointestinal protection
may be considered.

Colchicine (0.5 mg twice daily) added to an NSAID or as monotherapy also appears to be effective for the initial attack and the prevention.

The role of systemic corticosteroids (option B) in pericarditis is controversial. High dose corticosteroids (i.e., prednisolone 1 mg/kg/ day) with
a 2–4-week taper is considered in pericarditis secondary to connective tissue disease, uremia or autoreactivity.

Referring the patient to cath lab (option C), heparin (option B), or thrombolytic agents (option E) may be considered for patients with
myocardial infarction or other conditions within the spectrum of acute coronary syndrome. The pleuritic nature of the chest pain, presence of
fever, and the characteristic ECG findings favors pericarditis with high index of certainty.

References

103 of 1943
• RACGP - Pericarditis: Clinical features and management
Last updated:
Time spent: QID:1252
2023-2-12

104 of 1943
A 67-year-old man, who is a known case of congestive heart failure and well controlled on treatment, forgets to take his medication while on a
10-day trip to France. Today he is in your office with complaint of bilateral edema of his legs. On examination, he has a blood pressure of
165/110mmHg, heart rate of 106bpm and respiratory rate of 23 breaths per minute. There is bilateral pitting edema of both legs up to the
knees. Which one of the following options is the drug to consider for him first?

A. Spironololoctone.

B. Furosemide.

C. Digoxin.

D. Beta blockers.

E. ACE inhihitors and beta blokcers but not diuretcis.

Incorrect. Correct answer is B


45% answered correctly

Explanation:

Correct Answer Is B

This patient has presented with a decompensated heart failure which has been previousuly well controlled. In approaching such patients, the
first thing to consider is the volume status. For patients who are in hypervolemic state, the fisrt step is volume reduction using a loop diuretic
such as furosemide, bumetanide, or torsemide. It is also important to start the patient on an ACE inhibitor (e.g. captopril, enalapril).
Angiotensin receptor blockers (ARBs) such as losartan, erbisartan or valsartan can be used alternatively in those who are intolerant of ACE
inhibitors.

Indicators of hypervolemic state neccessiating commencement of a diuretic are:

Increased jugular venous pressure


Increased weight
Dyspnea
More than minimal peripheral edema
Crackels
Elevated brain natriuretic peptide
Hepatomegaly, ascites or anasarca in severe cases

Once the patient is euvolemic, addition of a cardioselective beta blocker to the above regimen is the next best step in management.

For euvolemic patients, an ACE inhibitor (or ARB) plus a beta blocker is the management of choice.

NOTE – beta blokers should only be started for patients who do not have respiratory crackles and no more than minimal peripheral edema.

With significant bilateral edema in this patient, he is hypervolemic and should be started on a loop diuretic such as furosemide as the most
appropriate next step in management. An ACE inhibitor or ARB should be used in conjunction with a diuretic for mortality benefit.

(Option A) Spironolactone (an aldosterone antagonist) is used in euvolemic patients who have a GFR>30ml/min and serum potassium of
<5mmol/L with close monitoring of potassium levels. Spironolactone provides additional mortality benefit in patients with CHF. Since this
patient is hypervolemic, he cannot be started on spironolactone now.

(Option C) Digoxin is indicated in patients with EF<35%. Digoxin should be added only after maximally-tolerated dose of a beta blocker.

(Options D and E) Beta blockers are contraindicated in patients with CHF who have volume overload evident by significant peripheral edema,
pulmonary crackles, increased JVP, ascites, hepatomegaly, etc. In euvolemic patients, however, addition of beta blockers has shown mortality
benefit. This drug class should be considered for all patients with euvolemic CHF.

References

• https://www.ncbi.nlm.nih.gov/pmc/articles/PMC4347206/

Last updated:
Time spent: QID:1255
2023-2-12

105 of 1943
A 75-year-old man comes to the Emergency Department because of severe chest pain starting 20 minutes ago. His past medical history
includes type II diabetes mellitus and hypertension. On examination, he has a blood pressure of 110/65 mmHg and pulse of 110 bpm. An ECG
shows ST elevation in leads I, aVL, V5 and V6. With the diagnosis of ST elevation myocardial infarction, he undergoes thrombolytic therapy
with tenecteplase followed by heparin infusion. ST segment elevation resolves within 20 minutes as does the chest pain. While heparin is
being infused, he suddenly develops drowsiness and confusion. His blood pressure is now 185/110 mmHg and pulse rate 50 bpm. There is no
focal neurological finding. Which one of the following is the best explanation for this clinical picture?

A. Intracranial hemorrhage.

B. Silent myocardial re-infarction.

C. Anaphylactic reaction to tenecteplase.

D. Hypertensive encephalopathy.

E. Aortic dissection.

Correct
45% answered correctly

Explanation:

Correct Answer Is A

Sudden onset of lethargy and confusion, as well as raised blood pressure and bradycardia are consistent with the diagnosis of elevated
intracranial pressure due to intracranial bleeding.

Intracranial hemorrhage is a devastating complication of thrombolytic therapy seen in 1% of patients. 53-60% of patients die of the event
during hospitalization and 25% of the survivors will suffer permanent neurological deficits.

(Option B) Silent myocardial reinfarction may present with bradycardia and hypotension if an inferior reinfarct occurred, but not with
hypertension and bradycardia.

(Option C) Anaphylactic reaction to tenecteplase would have caused hypotension, not hypertension.

(Option D) Hypertensive encephalopathy has a more insidious onset and is seen in higher blood pressures (>180/120mmHg). It presents with
headache, restlessness, vomiting, confusion and coma.

(Option E) With Aortic dissection there should have been compensatory tachycardia.

References

• AHA - Guidelines for Thrombolytic Therapy for Acute Stroke: A Supplement to the Guidelines for the Management of Patients With Acute
Ischemic Stroke

Last updated:
Time spent: QID:24
2023-2-12

106 of 1943
A 56-year-old man presents to the emergency department with complaints of chest pain and lightheadedness. On examination he has a blood
pressure of 85/50mmHg, rapid and barely perceptible pulse of 160bpm and respiratory rate of 24 breaths per minute. An ECG is obtained
which is shown in the accompanying photograph. Which one of the following is the most appropriate next step in management?

A. Atropine.

B. Intravenous diltiazem.

C. Intravenous metoprolol.

D. Pace maker insertion.

E. DC cardioversion.

Incorrect. Correct answer is E


45% answered correctly

Explanation:

Correct Answer Is E

The ECG is characteristic of supraventricular tachycardia (tachycardia with regular and monomorphic QRS complexes and absence of P
wave).

SVT is a common condition that occurs in individuals of all age groups and presents with a variety of clinical manifestations. Patients may be
asymptomatic or may present with minor palpitations or more severe symptoms.

If symptomatic, symptoms can include:

Palpitations - >96%
Dizziness - 75%
Shortness of breath - 47%
Syncope - 20%
Chest pain - 35%
Fatigue - 23%
Diaphoresis - 17%
Nausea - 13%

Rare complications of paroxysmal SVT include myocardial infarction, congestive heart failure, syncope, and sudden death.

Acute management of supraventricular tachycardia (SVT) includes controlling the rate and preventing hemodynamic collapse. Patients with
unstable hemodynamic status (hypotension, chest pain, dyspnea and perfusion-related confusion) should be treated with immediate
cardioversion after sedation. This patient has a systolic blood pressure of less than 90 mmHg indicating hemodynamic instability.
Lightheadedness per se is not a sign of instability.

If the patient is stable, vagal maneuvers can be used to slow the heart rate and to convert it sinus rhythm. If vagal maneuvers are not
successful, adenosine can be used in increasing doses. If adenosine does not work, atrioventricular (AV) nodal blocking agents such as
calcium channel blockers (diltiazem or verapamil) (option B) or beta-blockers (e.g. metoprolol) (option C) should be used, as most patients
who present with SVT have AV nodal reentrant tachycardia (AVNRT) or AV reentrant tachycardia (AVRT). These arrhythmias depend on AV
nodal conduction and therefore can be terminated by transiently blocking this conduction.

(Options A and D) Atropine and pace maker are used for patients with symptomatic bradycardia which is not the case here.

References
107 of 1943
• Medscape - Paroxysmal Supraventricular Tachycardia
Last updated:
Time spent: QID:1279
2023-2-12

108 of 1943
A 50-year-old man presents to your practice for treatment of his newly-diagnoses hypertension. His medical history is significant for asthma
and reflux nephropathy. Laboratory studies shows a serum urea of 18 mmol/L (normal: 2.9-8.2 mmol/L), creatinine of 190 µmol/L (normal: 50-
110 µmol/L) and proteinuria of 900 mg/d. Which one of the following would be the antihypertensive medication of choice for him?

A. Amlodipine.

B. Losartan.

C. Perindopril.

D. Indapamide.

E. Atenolol.

Correct
45% answered correctly

Explanation:

Correct Answer Is A

Important determining factors for selection of antihypertensive medications include the presence of concomitant medical conditions and/or
contraindications to a specific antihypertensive drug class. For example, patients with migraine and hypertension may benefit from beta
blockers because beta blockers can simultaneously lower the blood pressure and prevent migraine attacks. Patients with proteinuria and
hypertension are likely to benefit most from ACE inhibitors or angiotensin receptor blockers (ARBs), while these agents are not appropriate
management options for treatment of patients with chronic kidney disease associated with significantly decreased GFR, because these drugs
act by relaxing of the efferent glomerular arteriole that will result in deterioration of an already impaired GFR.

Although the proteinuria makes ACE inhibitors (e.g. perindopril) (option C) or ARBs (e.g. losartan) (option B) the most appropriate option for
him, the polycystic kidney disease associated with significantly decreased GFR (evident by increased BUN and creatinine) makes them
inappropriate options as well (for calculation of eGFR based on the serum creatinine, age and sex click here). In patients with bilateral renal
artery stenosis, hypertensive nephrosclerosis, polycystic kidney disease or chronic kidney disease, intra-renal perfusion pressure is already
reduced, and GFR is maintained in part by an angiotensin II-induced increase in resistance of the efferent (post-glomerular) arteriole. Blocking
this response with an ACE inhibitor or ARB will relax the efferent arteriole, lower intra-glomerular pressure, and reduce the glomerular filtration
rate.

Indapamide (option D) is a diuretic. The antihypertensive effect of diuretics is reduced in patients with renal disease making them
inappropriate and less reliable options for hypertension control in such patients.

In patients with renal disease associated with decreased GFR, calcium channel blocker such as amlodipine and nifedipine and beta blockers
such as atenolol and metoprolol are acceptable options; however, this patient has asthma that makes beta blockers contraindicated for him.
Of the options, the only appropriate medication to consider for hypertension control in this man is the calcium channel blocker amilodipine.

References

• UpToDate - Major side effects of angiotensin-converting enzyme inhibitors and-angiotensin II receptor blockers

• Drugs.com

Last updated:
Time spent: QID:1395
2023-2-12

109 of 1943
Robert, 67 years old, presents to your clinic for evaluation of increasing left leg pain for the past 6 months. He describes the pain as aching
which was brought on after walking one or two blocks and alleviated with rest. However, it has been worse recently, starting after walking
shorter distances and taking more time to get better. He also mentions that sometimes he wakes up in the middle of the night due to leg
pain. He smokes 25-30 cigarettes a day and drinks socially. He denies any remarkable medical condition except hypercholesterolemia for
which he is on atorvastatin 10 mg, daily. On examination, he has a blood pressure of 142/91mmHg, pulse rate of 88 bpm and a BMI of 32.
Compared to the right side, left leg has less hair. He has a left ankle/brachial index (ABI) of 0.35 on the left side and 0.7 on the right. Which
one of the following investigations is the option of choice to establish a certain diagnosis in Robert?

A. Duplex ultrasound.

B. Digital subtraction catheter (DSA) angiography.

C. Arteriography.

D. CT angiography.

E. Magnetic resonance angiography.

Correct
45% answered correctly

Explanation:

Correct Answer Is A

Robert has typical features of ischemic leg pain in history and physical examination. An ABI of 0.3 (normal: 0.9-1.1) signifies the need for
endovascular intervention. However, confirmatory investigations are still required to establish and document such diagnosis.

To make a diagnosis of arterial disease noninvasively, ultrasound examination is the mainstay investigation and the best option here. Each
mode of ultrasound provides specific information. Ultrasonography is used to evaluate the presence of vascular disease, anatomic location,
and the extent and morphology of the lesion(s). Duplex ultrasound scan is the most appropriate diagnostic tool to use for Robert to establish
a diagnosis of vascular disease.

Real-time ultrasound scan uses reflected sound waves to produce images and assess blood velocity. Two ultrasound modes are routinely
used in vascular imaging: (1) B Mode and (2) doppler mode.

B mode (brightness mode) is the conventional, grayscale, or 2-dimensional ultrasound scan. This mode is used for anatomical details. Color
doppler or simply doppler ultrasound scan is the mode used for assessment of blood flow and velocity. The combination of these two (B
mode + doppler mode) is called duplex ultrasound.

There also other advanced imaging modalities such as CT angiography (option D) or Magnetic resonance angiography (MRA) (option E). The
main goal of these modalities is provision of information necessary for intervention either by open surgery or endovascular intervention.
These modalities are not used for diagnosing vascular disease because the non-invasive and readily available ultrasound can diagnose
vascular disease in most anatomical sites with high sensitivity and specificity.

Catheter DSA (option B) is the gold standard for imaging peripheral arteries but is rarely used for diagnosis because of its invasive nature and
the availability of non-invasive or less invasive imaging modalities such as duplex ultrasound, CT angiography and MRA.

DSA is used to clearly visualize blood vessels in a bony or dense soft tissue environment. A catheter is introduced into an artery (e.g. tibial
artery) and contrast medium is injected into that artery. Images are produced by subtracting a pre-contrast image from post-contrast images.

(Option E) Arteriography is another term used for angiography. In general, conventional angiography or arteriography consists of injection of
contrast matter into arteries and taking X-ray images. As mentioned before, these techniques are used mainly (if ever these days) before
interventions. For diagnosis, ultrasound is the best option.

NOTE - In cases with traumatic limb ischemia or acute limb-threatening ischemia where endovascular or open surgery is required, CT
angiography is the modality of choice because urgent intervention is considered and should be planned. MRA is another option but it is
less available. In such cases, duplex ultrasound is used only if CT angiography or MRI are not available or the patient has any
contraindications for contrast matter. However, some surgeons still prefer the duplex as the modality of choice to guide the operation.

References

• RACGP – AFP – peripheral arterial disease, diagnosis and management in general practice

• UpToDate – Noninvasive diagnosis of arterial disease

• Diagnostic Imaging Pathways – Acute limb ischemia


110 of 1943
Last updated:
Time spent: QID:1463
2023-2-12

111 of 1943
A 62-year-old man presents to your clinic for a follow-up visit. He had a myocardial infarction (MI) 4 years ago. He is currently on aspirin 80
mg daily. He follows a healthy diet and does exercise regularly. Laboratory studies are all within normal parameters. An ECG is obtained that is
normal. Which one of the following is the most appropriate advice for him?

A. Reinforce the importance of a healthy diet and regular exercise.

B. Stop aspirin.

C. Add statins.

D. Tell him he does not need any further follow-up.

E. Add warfarin.

Incorrect. Correct answer is C


45% answered correctly

Explanation:

Correct Answer Is C

According to the guideline by National Heart Foundation of Australia, every post-MI patient have to should be provided with the following
medications and advice upon discharge. Of these, some are indicated in all post-MI patients and some are only used under certain
circumstances (read explanations for each entity):

Aspirin

All patients should take 75–150 mg daily unless contraindicated.

Clopidogrel

There is evidence that clopidogrel should be prescribed for up to 12 months after an acute coronary syndrome, in particular after
stent implantation, with the duration of therapy depending on the particular type of stent and circumstances of implantation (level II
evidence). Clopidogrel may also be prescribed as an alternative when aspirin is contraindicated, or in addition to aspirin, particularly in
patients with unstable angina or recurrent cardiac events.

Beta-blocker

Should be prescribed for most patients after a myocardial infarction unless contraindicated, and continued indefinitely, especially in
high-risk patients. Carvedilol, bisoprolol or metoprolol (extended release) should be used in patients with heart failure.

Angiotensin-converting enzyme inhibitor

Should be given early after an acute coronary syndrome, and its use reviewed later.

Statins

Statin therapy should be initiated in hospital for all patients with coronary heart disease.

Warfarin

Recommended after myocardial infarction for those at high risk of systemic thromboembolism because of atrial fibrillation, mural
thrombus, congestive heart failure or previous embolization. Warfarin may sometimes be combined with aspirin, but in this
circumstance patients should be observed closely for signs of bleeding.

Nitrates

All patients should be prescribed a short-acting nitrate (unless contraindicated) and provided with a written action plan for chest pain.

Insulin/oral hypoglycemics

Good glycaemic control should be obtained and continued in patients who have had acute coronary syndromes and who have
diabetes.

Aldosterone antagonists

112 of 1943
Initiation of eplerenone therapy should be considered early after myocardial infarction in those with left-ventricular systolic
dysfunction and symptoms of heart failure.

Lifestyle advice

All patients should be given advice on lifestyle changes that will reduce the risk of further cardiac events, including smoking
cessation, good nutrition, moderate alcohol intake, regular physical activity and weight management as appropriate.

Ongoing prevention and cardiac rehabilitation programs

Cardiac rehabilitation is a proven effective intervention. All patients with cardiovascular disease should have access, and be actively
referred, to comprehensive ongoing prevention and cardiac rehabilitation services.

Chest pain action plan

All patients should be provided with a written action plan for chest pain which includes:

Rest and self-administration of short-acting nitrates


Self-administration of aspirin unless contraindicated
Calling an ambulance if chest pain or discomfort is not completely relieved within 10 minutes
Individualised clinician notification and action plan for those living in areas where an ambulance is not readily available.

Fish oil

A diet high in omega-3 fatty acids from fish and the use of fish oil tablets is recommended (grade B recommendation).

Psychosocial factors

Depression, social isolation and lack of quality social support are likely to lead to significantly worse outcomes in those with coronary
heart disease. All patients with coronary heart disease should be assessed for comorbid depression and level of social support.

Diabetes

An early glucose tolerance test should be considered in those without diagnosed diabetes.

Implantable cardiac defibrillators (ICDs)

ICDs should be considered in some patients who, despite optimal medical therapy, have persistently depressed left ventricular
function after ST-segment-elevation myocardial infarction.

This patient is already taking aspirin and should continue it for life unless contraindicated. Therefore, cessation of aspirin (option B) would be
inappropriate. He is already following a healthy life style including diet and exercise. It is a good idea to encourage him and reinforce the
importance of a healthy diet and regular exercise (option A). He should have been discharged on statins. In fact, statins must have been
started during his hospital stay and continued indefinitely unless contraindicated. Of the options, adding statins to his medications is the first
priority and should come first. The next best advice would be reinforcing the importance of a healthy diet and regular exercise.

(Option D) This patient requires regular follow-ups and monitoring. Telling the patient that he does need follow up is an incorrect advice.

Warfarin (option E) is only recommended after myocardial infarction for those at high risk of systemic thromboembolism due to atrial
fibrillation, mural thrombus, congestive heart failure, or previous embolization. This is not indicated in this patient.

References

• MJA - Guidelines for the management of acute coronary syndromes 2006

Last updated:
Time spent: QID:1554
2023-2-12

113 of 1943
A 65-year-old man presents to the Emergency Department with complaint of chest pain radiating to his jaw for the past 2 hours. The patient is
given aspirin, sublingual nitroglycerine (TNG), and supplemental oxygen. With these measures the chest pain subsides. An ECG is obtained
and is as follows. A troponin level is requested with the result still pending. Which one of the following is the next best step in management?

A. Send the patient home on aspirin.

B. Coronary angiogram and angioplasty.

C. Thrombolysis.

D. Admit and wait for the troponin results.

E. Perform stress test.

Incorrect. Correct answer is B


45% answered correctly

Explanation:

Correct Answer Is B

This patient as a typical ischemic chest pain as well as an ECG showing ST segment elevation in leads II, III, and aVF. With these, an inferior
ST segment elevation myocardial infarction (STEMI) is a definite diagnosis prompting immediate treatment and reperfusion therapy.

Therapy should be started regardless of the troponin result (option D) because troponin may take up to 6-8 hours to become positive, and its
negativity does not change the diagnosis and the management plan in this scenario. In other words, even with a negative troponin result, the
diagnosis is still STEMI and urgent reperfusion therapy is required.

According to National Heart Foundation of Australia, Australian Clinical Guidelines for the Management of Acute Coronary Syndromes For
patients with ST elevation myocardial infarction (STEMI) presenting within 12 hours of symptom onset, and in the absence of comorbidities
that impact the individual’s overall survival, emergency reperfusion therapy with either primary percutaneous coronary intervention (PCI)
(formerly called angioplasty) or fibrinolytic therapy is recommended.

Since this patient is still in the 12 hours window, angiography to located the coronary stenosis or blockage and stent placement to maintain
the vessel patent is the first-line treatment choice. In case PCI cannot be performed with acceptable delay, thrombolysis (option C) using
agents such as alteplase, tenecteplase or reteplase are the treatment of choice.

With the definite diagnosis of MI and the need for urgent intervention, sending the patient home on aspirin (option A),or performing stress
test (option E) are incorrect.

References

• Australian Clinical Guidelines for the Management of Acute Coronary Syndromes

Last updated:
Time spent: QID:1574
2023-2-12

114 of 1943
A 32-year-old woman presents to you practice with complaint of an upper respiratory tract infection evident by a fever of 38.3°, rhinorrhea, and
dry cough. On examination, her blood pressure is 170/110 mmHg on two separate readings 15 minutes apart. On auscultation, an abdominal
bruit is heard. The bruit radiates to the right flank. A Doppler duplex sonography shows right renal artery stenosis. Which one of the following
would be the most appropriate next step in management?

A. Conventional renal arteriography.

B. Magnetic resonance angiography (MRA).

C. Start her on enalapril.

D. Start her on thiazide diuretics.

E. Perform percutaneous renal artery angioplasty.

Incorrect. Correct answer is C


45% answered correctly

Explanation:

Correct Answer Is C

This woman has been incidentally found to have unilateral renal artery stenosis (RAS) once the cause of her hypertension has been
investigated.

Renal artery stenosis (RAS) is the major cause of renovascular hypertension. Apart from its role in the pathogenesis of hypertension, RAS is
also being increasingly recognized as an important cause of chronic renal insufficiency and end-stage renal disease. In older individuals,
atherosclerosis is by far the most common etiology of RAS. As the renal artery lumen progressively narrows, renal blood flow decreases.
Eventually, the decreased perfusion compromises renal function and structure. Less commonly, RAS may have been caused by fibromascular
dysplasia. This more often is seen in younger patients.

RAS may present with one or more of the following:

Abdominal bruit
Azotemia
Sudden worsening of hypertension or renal function
Acute kidney injury or decreased renal function after initiation of antihypertensive therapy, especially with angiotensin-converting
enzyme (ACE) inhibitors or angiotensin receptor blockers
Unexplained renal insufficiency, in elderly patients.
Congestive heart failure, with poor control of hypertension and renal insufficiency in the absence of a significant decrease in ejection
fraction (so-called flash pulmonary edema)

Investigations to consider in patients with suspected RAS include laboratory and imaging studies. Laboratory studies include renal function
test such as creatinine, urea, and electrolytes, 24-hour urine exam, and urinalysis. Serologic tests may also be considered for systemic lupus
erythematosus or vasculitis if these conditions are suspected.

The initial imaging to consider for patients with suspected RAS is Duplex ultrasound scanning which is a combination of B-mode
(conventional) ultrasound for assessment of anatomical structures and Doppler unit for obtaining data about renal blood flow velocity. This
technique is non-invasive and has a high sensitivity and specificity (~98%).

The general approach to treatment of RAS includes control of hypertension and hypercholesterolemia in patients with RAS due to
atherosclerosis. Preferred antihypertensive medications in RAS are angiotensin-converting enzyme (ACE) inhibitors or angiotensin receptor
blockers (ARBs). These two classes of medications can lead to impaired renal function evident by increased serum creatinine and
hyperkalemia. It is recommended that such patients should be monitored closely, and if creatinine increases by >25% of baseline within the
first month of use, these drugs are ceased and replaced with calcium channel blockers as the second-line medications for hypertension
control. Diuretics including thiazide diuretics (option D) are not recommended for hypertension control in patients with RAS.

Also, strict serum cholesterol control is recommended for patients with RAS in the setting of atherosclerosis usually with statins.

In the absence of trials showing benefit from revascularisation therapy such as percutaneous renal artery angioplasty (option E) over
conventional non-surgical treatment and the significant risk of complications, revascularization treatments is only considered and advised for
patients who fail medical therapy with resistant or poorly-controlled hypertension; recurrent flash pulmonary edema; dialysis-dependent
kidney failure resulting from renal artery stenosis; chronic renal insufficiency and bilateral renal artery stenosis; or renal artery stenosis of a
solitary functioning kidney. This patient may or may not need revascularization after further assessment is undertaken. In the meanwhile, she
should be started on enalapril (ACE inhibitor) for hypertension control as the most appropriate next step in management.

Other imaging studies such as conventional renal arteriography (option A), spiral CT angiography, MR angiography (option B), or radionuclide
scans may be considered for further assessment and treatment. None of such investigations take precedence over starting therapy with
115 of 1943
antihypertensive medications as this stage.

References

• The CARI Guidelines – Caring for Australasians with Renal Impairment: Renal Artery Stenosis

• Medscape - Renal Artery Stenosis

Last updated:
Time spent: QID:1576
2023-2-12

116 of 1943
Three months ago you started Mary, aged 73, on residronate 35mg weekly, after she was diagnosed with osteoporosis confirmed with bone
mineral density (BMD). Her presenting symptom at that time was back pain. Examination revealed height decrease and mild kyphosis, as well
as tenderness over thoracic vertebrae. Despite being on residronate, her symptoms persisted and she also developed pain over new areas of
her thoracic vertebrae. A new X-ray established new osteoporotic fractures. Which one of the following would be the most appropriate
management option for her?

A. Continue residronate at the same dose.

B. Switch to alendronate.

C. Increase the dose of residronate.

D. Switch to zoledronic acid.

E. Switch to teriparatide.

Incorrect. Correct answer is E


45% answered correctly

Explanation:

Correct Answer Is E

Bisphosphonates are currently the most commonly anti-resorptive agents prescribed as first-line treatment for most of osteoporotic patients.
Of this drug family, alendronate (10mg/day or 70mg/week, orally), residronate (5mg/day or 35mg/week, orally) and zoledronic acid are
available in Australia.

Bisphosphonates should be used for at least 12 months before their efficacy on treatment of osteoporosis is assessed. However, in cases
where the patient suffers two or more minimal trauma fractures despite being on sufficient doses of an anti-resorptive drug, e.g.
bisphosphonates, commencement of teriparatide is justified as the most appropriate option ( E is correct).

Teriparatide is the synthetic parathyroid hormone that predominantly acts by increasing the osteoblasts (bone-forming cells) and by inducing
new osteoblasts formation. This drug is costly and at least 18 months of continuous use is needed effectiveness. Based on these, this
medication is only reimbursed by the PBS for patients with severe osteoporosis and very high risk of fractures who have:

A BMD T-score of =<-3

OR

had two or more minimal trauma fractures

OR

experienced at least one symptomatic new fracture after at least 12-months of continuous therapy with an anti-resorptive agent at
adequate doses.

(Option A) While Mary has developed new osteoporotic fractures due to severe osteoporosis, continuation of the same agent is not a wise
management.

Switching to other bisphosphonates such as alendronate (option B) or zoledronic acid (option D) does not add any benefit, as all members of
this drug family have almost the same effectiveness.

Mary is already on the recommended weekly dose for residronate and increasing the dose of residronate (option C) does not seem to benefit
her.

References

• https://www.osteoporosis.org.au/health-professional-resources

• https://www.racgp.org.au/osteoporosis

Last updated:
Time spent: QID:1271
2023-2-12

117 of 1943
Jane, a 65-year-old patient of yours, is being assessed for osteoporosis. A Dual Energy X-ray Absorptiometry has revealed T-scores of -2.5 and
-2.7 for the femoral neck and the vertebral column, respectively. She was diagnosed with the cancer of her right breast 6 years ago for which
she underwent right mastectomy, chemotherapy and radiation therapy. In addition to advice regarding calcium and vitamin D, which one of the
following medications would be the best option for treatment of her osteoporosis?

A. Raloxifene.

B. Alendronate.

C. Teriparatide.

D. Strontium ranelate.

E. Hormone replacement therapy (HRT).

Incorrect. Correct answer is B


45% answered correctly

Explanation:

Correct Answer Is B

According to WHO criteria and based on Jane’s T-scores at vertebra and femoral neck, she is suffering from osteoporosis. Currently,
bisphosphonates are the first-line treatment option for primary and secondary treatment of vertebral and non-vertebral fractures due to
osteoporosis. Alendronate, residronate and zoledronic acid are the available bisphosphonates in Australia .

(Option A) Raloxifene is a selective estrogen receptor modulator (SERM) reimbursed by the Pharmaceutical Benefit Scheme (PBS) for
treatment of postmenopausal osteoporosis. SERMs also have a preventive effect on breast cancer; therefore, an appropriate option for post-
menopausal women with family or personal history of breast cancer in whom vertebral fracture due osteoporosis is a concern. Although there
is excellent evidence (Grade A) that raloxifene reduces the risk of vertebral fractures, limited evidence support its effect on non-vertebral
fractures. Jane is osteoporotic in femur as well; hence raloxifene would not an appropriate first-line option for her.

(Option C) Teriparatide is the synthetic parathyroid hormone that predominantly acts by increasing the osteoblasts (bone-forming cells) and
by inducing new osteoblasts formation. This drug is costly and at least 18 months of continuous use is needed effectiveness. Based on these,
this medication is only reimbursed by the PBS for patients with severe osteoporosis and very high risk of fractures who have:

A BMD T-score of =<-3

OR

had two or more minimal trauma fractures

OR

experienced at least one symptomatic new fracture after at least 12-months of continuous therapy with an anti-resorptive agent at
adequate doses.

Jane does not any of the above criteria to meet the need for teriparatide.

NOTE – BMD starts declining approximately 12 months after cessation of therapy with teriparatide; hence, continuation of therapy with an
anti-resorptive, generally a bisphosphonates.

(Option D) Strontium ranelate is an effective second-line option for reducing the risk of further osteoporotic fractures in postmenopausal
women with prevalent fractures. This agent should not be used in patients with previous or clinically active cardiovascular disease or
uncontrolled hypertension and should only be used when other drug classes for treatment of osteoporosis are not appropriate. Strontium
ranelate is prescribed at a dose of 2 grams per day.

(Option E) Breast cancer is estrogen dependent; therefore, prescribing estrogen for patients with a personal or strong family history of breast
cancer is not an appropriate option. Estrogen is an option (available on PBS) for the prevention and treatment of osteoporosis in women who
are near or at menopause, in particular for those patients suffering from menopausal vasomotor symptoms, e.g. hot flushes. When
considering this therapy, potential adverse effects i.e. increased risk of thromboembolism, cardiovascular diseases and breast cancer should
be weighed and discussed with the patient.

References

• https://www.osteoporosis.org.au/health-professional-resources

118 of 1943
• https://www.racgp.org.au/osteoporosis
Last updated:
Time spent: QID:1270
2023-2-12

119 of 1943
A 54-year-old woman presents to your clinic for advice regarding osteoporosis. You order a DEXA scan showing a T-score of -1. Which one of
the following is the most appropriate management for this patient?

A. High-calcium diet.

B. Bisphosphonate.

C. Calcium and vitamin D supplement.

D. Adequate exercise.

E. Exposure to sunlight.

Correct
45% answered correctly

Explanation:

Correct Answer Is A

A T-score of -2.5 of less indicates osteoporosis. T-scores between -1 to -2.5 suggests osteopenia.

A T score of -1 or above is considered normal. So this woman is neither osteopenic nor osteoporotic. For postmenopausal woman without
evident bone loss, maintaining a high-calcium diet is the most appropriate advice for prevention of osteoporosis (grade A recommendation).
Diet is the preferred source of calcium, and patients should aim for a total daily calcium intake of 1300 mg. Dairy products are rich in calcium
and one of the best sources.

(Option B) Bisphosphonates are used for treatment of established osteoporosis or osteopenia in ceratin patient groups . This woman does
not have osteoporosis.

(Option C) Calcium supplementation (with or without vitamin D, depending on the patient’s vitamin D status) is used when adequate calcium
intake cannot be ensured solely through diet.

(Option D) Exercise may have additional benefits including weight control, reduction of blood pressure, pain relief and improvement in quality
of life. Exercise can also be beneficial for improving balance as a part of a 'falls reduction program'. Particularly in patients diagnosed with
osteoporosis, supervision by a physiotherapist, exercise physiologist or other appropriately trained and qualified health professional is
recommended. Overall, moderate exercise against gravity, such as brisk walking for 30 minutes 4 times a week, jogging or tennis may make a
small contribution to retarding bone loss. Non-weight bearing exercises such as swimming, cycling, etc are not effective in bone loss
prevention.

(Option E) Exposure to sunlight is necessary for vitamin D production. Vitamin D is necessary for calcium absorption through the gut.

References

• http://www.racgp.org.au/download/documents/Guideli

• http://www.racgp.org.au/download/documents/Guideli

• Therapeutic Guidelines – Endocrinology; available from: http://tg.org.au

Last updated:
Time spent: QID:768
2023-2-12

120 of 1943
Which one of the following breast diseases can be caused by vitamin A deficiency?

A. Fibroadenoma.

B. Intraductal papilloma.

C. Periductal mastitis.

D. Ductal carcinoma in situ.

E. Paget disease.

Incorrect. Correct answer is C


45% answered correctly

Explanation:

Correct Answer Is C

Of the given options, periductal mastitis is found to be caused by vitamin A deficiency.

Periductal mastitis, also known as subareolar abscess, squamous metaplasia of lactiferous ducts, or Zuska disease is an inflammatory
condition of the subareolar ducts. Periductal mastitis primarily affects young women, but can occur in men as well. Periductal mastitis is a
separate entity from duct ectasia, which usually affects older women and is characterized by dilated ducts and sticky toothpaste-like
dischagre.

The cause of periductal mastitis is unknown. However, vitamin A deficiency and smoking have been implicated as potential causes.

The majority of patients who get periductal mastitis are smokers. It is postulated that smoking leads to or is associated with damage
to subareolar ducts with subsequent tissue necrosis and later infection. The toxic substances in cigarette smoke may damage the ducts
directly, or there may be a localized hypoxic effect.

There is increasing evidence that vitamin A (or retinoids) have a significant effect on mammary duct epithelial cell proliferation and
differentiation. Vitamin A deficiency impairs blood clearance of bacteria and results in decreased phagocytic activity in vitro.

References

• Science Direct - Mammary Duct Ectasia

Last updated:
Time spent: QID:690
2023-2-12

121 of 1943
A 12-year-old boy is brought to the emergency department with loss of consciousness. His blood glucose on arrival is 2.5 mmol/L. His past
medical history is significant for type I diabetes mellitus, for which he is on insulin. Which one of the following is the most appropriate
management?

A. Dextrose 5% intravenously.

B. Dextrose 10% intravenously.

C. Dextrose 50% intravenously.

D. Normal saline.

E. Oral sweet juice.

Incorrect. Correct answer is B


45% answered correctly

Explanation:

Correct Answer Is B

In children with type I diabetes mellitus, hypoglycemia is the most common encountered complication. hypoglycemia may lead to seizures
and loss of consciousness.

In childhood (other than neonatal period), hypoglycaemia is defined as a blood glucose level of <2.6 mmol/L. The most appropriate treatment
of severe hypoglycemia when the patient is unconscious and unable to take oral glucose is with a bolus of intravenous dextrose 10%, 2.5 to 5
mL/Kg followed by 0.03 to 0.05 mL/Kg/minute until the patient is stable.

In adults with hypoglycemia, 50% glucose solution is used for treatment of severe hypoglycemia. This solution is not recommended for
children because it can result in serum hyperosmolarity and death.

In a conscious and cooperative child, oral route is preferred.

References

• http://www.rch.org.au/clinicalguide/guideline_inde

• Therapeutic Guidelines – Endocrinology; available from http://tg.org.au

Last updated:
Time spent: QID:521
2023-2-12

122 of 1943
A 38-year old lady presents to your pratice with fatigue, lethargy and weight gain of 5 kg over the past few months. She also has irregular
heavy periods. She feels flushed and tremulous. She is hypertensive, and her blood sugar is 10 mmol/L. Which one of the following is the
most appropriate choice of investigation?

A. Thyroid function tests.

B. 24-hour urine cortisol level.

C. Serum FSH and LH.

D. Serum FSH and estradiol.

E. Serum prolactin level.

Incorrect. Correct answer is B


45% answered correctly

Explanation:

Correct Answer Is B

This patient has clinical features suggestive of Cushing's syndrome.

Cushing's syndrome in women may present with:

Anxiety
Tremulousness
Weight gain
Severe fatigue
Menstrual irregularities
Hypertension
Hyperglycemia
Thin easily-bruising skin
Purplish pink striae may be present on buttocks, thighs, abdomen and breast
Proximal muscle weakness

When Cushing syndrome is suspected, overnight dexamethasone (1 mg) challenge test is performed as screening. Patients in whom cortisol
level fails to become suppressed should undergo high-dose dexamethasone suppression test. If high-dose dexamethasone fails to suppress
serum cortisol levels, of 24-hour urine cortisol level should be measured as the most definitive diagnosis.

References

• UpToDate - Establishing the diagnosis of Cushing's syndrome

• Medscape - Iatrogenic Cushing's Syndrome Workup

Last updated:
Time spent: QID:566
2023-2-12

123 of 1943
A 57-year-old woman presents to your clinic complaining of low back pain. She has a history of breast cancer, successfully treated 4 years
ago. She does not go out much because she is afraid of developing melanoma. A Dexa scan shows a T score of -2.8 g/cm2. An X-ray reveals
2 vertebral fractures. Which one of the following would be the most appropriate next step in management?

A. Alendronate.

B. Raloxifene.

C. Vitamin D.

D. Calcium.

E. Strontium ranelate.

Incorrect. Correct answer is C


45% answered correctly

Explanation:

Correct Answer Is C

With two fractured vertebrae and a T-score of -2.8, this patient is suffering from osteoporosis. Medical treatment is indicated in the following
patient groups:

Those with a minimal trauma fracture


Those aged 70 years or over
Those with a T-score of –2.5 or lower
Those currently on prolonged (at least 3 months) high dose corticosteroid treatment (at least 7.5 mg/day prednisolone or equivalent)
and with a T-score of –1.5 or lower.

There are different options for medical prevention and treatment of osteoporosis such as bisphosphonates (e.g. alendronate, risedronate),
estrogen receptor modulators (e.g. raloxifene), strontium ralenate, and teriparatide.

Supplementation of calcium and vitamin D should be considered for all patients, as studies suggest that the optimal efficacy of medical
treatment, to a considerable extend, depends on the availability of these two.

Bisphosphonates are always first-line medications in postmenopausal women (even those with personal history of breast cancer).

Whenever bisphosphonates are considered for treatment, close attention should be paid to vitamin D status and its deficiency be corrected
prior to starting treatment. Achieving a 25-hydroxy vitamin D plasma level of 50nmol/L is the target before the patient is started on
bisphosphonates.

Vitamin D supplementation is indicated for the following patient groups:

Those with proven vitamin D deficiency


Institutionalized or housebound people
Women who are shrouded for cultural reasons

Since this woman does not expose herself to sunlight due to fear of developing melanoma, she should be categorized as housebound and
vitamin D deficient because exposure to sunlight is the most significant source of vitamin D production in the body. She definitely needs
alendronate as first-line medical treatment for her osteoporosis, but not before her vitamin D deficiency is corrected to a level of 50nmol/L or
above.

References

• http://www.racgp.org.au/afp/2012/march/osteoporosi

• http://www.racgp.org.au/download/documents/Guideli

• https://www.nos.org.uk/NetCommunity/Document.Doc?i

• Therapeutic Guidelines – Endocrinology; available from http://www.tg.org.au

Last updated:
Time spent: QID:464
2023-2-12

124 of 1943
Jennifer is a 67-year-old female patient of yours in the GP clinic you work. She has had type 2 diabetes mellitus for the past 10 years. Despite
advice and treatment, she has poor glycemic control with the last HbA1C ordered 2 months ago being 10.7%. She also has
hypercholesterolemia and hypertension for which she is on treatment. Which one of the following would be the most appropriate screening
advice for her regarding retinopathy?

A. Review by an ophthalmologist every 2 years.

B. Yearly review by an optometrist.

C. Review by ophthalmologist every 5 years.

D. Tight glycemic control followed by retinal photography.

E. Review by an optometrist every 2 years.

Incorrect. Correct answer is B


45% answered correctly

Explanation:

Correct Answer Is B

Type 2 diabetes mellitus is associated with several complications including, cardiovascular disease, peripheral neuropathy, diabetic
nephropathy, diabetic foot, and diabetic retinopathy. There are recommendations for appropriate screening of patients with type 2 diabetes for
early detection and treatment of such complications.

Current guidelines by the National Health and Medical Research Council (NHMRC) recommends the following:

Diabetic retinopathy

All patients with type 2 diabetes should have dilated fundus examination and visual acuity assessment at the diagnosis of diabetes and at
least every two years. Higher risk patients (e.g., longer duration of diabetes, or poor glycemic control, blood pressure or blood lipid control)
who do not have diabetic retinopathy at least every year.

Diabetic peripheral neuropathy

All patients should be screened for distal symmetric polyneuropathy starting at diagnosis of type 2 diabetes and at least annually thereafter,
using simple clinical tests

Diabetic nephropathy

Kidney status in people with type 2 diabetes should be assessed by yearly screening for albuminuria (note that dipstick urine test is not
adequate to identify albuminuria) and estimated glomerular filtration rate (eGFR)

Glycemic control:

Apart from routine screening of blood sugar by the patient, HbA1C should be tested every 3-6 months

Lipids:

If the patient has low CVD risk, these tests can be performed every three years. More frequent testing can be justified if the clinical situation
varies or if therapeutic changes have been made e.g., a change in type or dose of the lipid lowering agent. Some guidelines suggest yearly
testing of lipids when the patient is deemed to be at clinically high risk.

Other tests:

Based upon a clinical risk assessment, individually assess the need for further investigations such as liver enzyme abnormalities for hepatic
steatosis.

Jennifer has long-standing diabetes with poor glycemic control. Besides, she has hypertension and hyperlipidemia as well. These, collectively,
put her at higher risks of diabetic retinopathy. For that, she needs to be screened yearly by an optometrist, ophthalmologist or another health
professional with enough expertise in the field. The assessment can be done by pupillary dilation and ophthalmoscopy or retinal photography.

Options suggesting 2-yearly screening (options A and E) could be the correct answers for people with diabetes an no increased risk of
retinopathy. Any option suggesting longer than 2 years, e.g., 5-yearly (option C) is wrong in all circumstances and for all patients with type 2
diabetes.

125 of 1943
(Option D) Although tight glycemic control should be advised and emphasized on, screening is not dependent on it and screening should be
advised and carried out regardless of the glycemic control.

References

• RACGP & Diabetes Australia – General practice management of Type 2 Diabetes

• Health Department – Diabetic retinopathy

Last updated:
Time spent: QID:1674
2023-2-12

126 of 1943
Which one of the following is least likely to be found on clinical examination of patient with thyrotoxicosis?

A. Insomnia.

B. Fine tremors.

C. Painful hip.

D. Atrophy of interosseous muscles.

E. Wide pulse pressure.

Incorrect. Correct answer is C


45% answered correctly

Explanation:

Correct Answer Is C

Thyrotoxicosis can be caused by the folloiwng four conditions, of which the first two are the most common causes in Australia (percentages
for Australian population):

1. Graves' Disease (70%)


2. Toxic multinodular goitre (15%)
3. Toxic adenoma (5%)
4. Thyroiditis (5%)

Symptoms of thyrotoxicosis depend on the severity and duration of the disease, the specific cause of the thyrotoxicosis, and the age of the
patient. Excessive thyroid hormone affects virtually all body systems through the stimulation of metabolic processes and activation of the
sympathetic nervous system.

In general, the clinical features include:

Weight loss (weight gain in 10% due to increased appetite)


Heat intolerance
Palpitations
Breathlessness
Heart failure in the elderly
Irritability and insomnia (option A)
Nervousness, anxiety
Tiredness and lethargy
Diarrhea
A fine tremor (option B), with proximal muscle weakness (unable to climb stairs or rise from a sitting position)
Sweating
Tachycardia
Goitre
Vitiligo and alopecia
Pretibial myxedema: skin thickening like orange peel, common over the shins
Wide pulse pressure (option E)
Eye changes - related to sympathetic stimulation and (in Graves) to antibody reactions. Thyroid eye disease of Graves' disease may
present with proptosis, eye muscle involvement, corneal involvement, and optic nerve compression. lid retraction and lid lag are
common to all causes of thyrotoxicosis)

The muscular symptoms of thyrotoxicosis vary from mild myasthenia to profound muscular weakness and atrophy, especially of proximal
muscle groups. This weakness forms the basis of a useful clinical test. If a thyrotoxic patient seated in a chair is asked to hold one leg out
straight and in a horizontal position, he or she may be able to do so for 25 - 30 seconds only while one with a normal thyroid sons can
maintain such a position for 60 - 120 seconds. Toe standing and step climbing may also bring out muscle weakness that is otherwise not so
apparent. In the more extreme forms of muscular involvement, there is not only weakness but also atrophy. Wasting of the temporal and
interossei muscles (option D) may be noted in a considerable number of patients, and in a few, wasting of all skeletal muscles. This wasting
may go so far as to bear a close resemblance to progressive muscular atrophy. Occasionally, the myopathy may shade into the picture of a
polymyositis.

Joint pain such as hip pain is one of the symptoms of and underactive thyroid and is seen in hypothyroidism rather than hyperthyroidism. The
cause is often excess fluid accumulation in the joint space and swelling.

References

127 of 1943
• Thyroid UK – Graves and thyrotoxicosis: Muscles

• UpToDate – Overview of the clinical manifestations of hyperthyroidism in adults

• RACGP – AFP – Evaluating and managing patients with thyrotoxicosis


Last updated:
Time spent: QID:1630
2023-2-12

128 of 1943
A 30-year-old woman presents to your practice with complaint of episodes of headache, sweating, palpitation, tingling sensation in her hands,
shortness of breath‫ و‬and severe anxiety. A thorough medical workup reveals no pathological finding. During an episode of these symptoms,
which one of the following laboratory findings is most likely to be found?

A. Increased free plasma epinephrine.

B. Decreased serum glucose.

C. Decreased serum calcium.

D. Increased serum amylase.

E. Lymphocytosis.

Correct
45% answered correctly

Explanation:

Correct Answer Is A

Episodic attacks of a constellation of symptoms resembling anxiety are consistent with pheochromocytoma as the diagnosis.

Pheochromocytoma is a rare tumour of adrenal gland tissue that results in increase in the level of adrenal catecholamines. Catecholamines
are responsible for sympathetic (fight and flight response) and affect the heart rate, metabolism, and blood pressure. Patients with
pheochromocytoma may be asymptomatic or have episodic symptoms of palpitations, flushing, sweating, headache, tremor, and anxiety due
to catecholamine excess. Severe hypertension is often seen; however, up to 15% of patients are normotensive. Pheochromocytomas can be
associated with familial syndromes such as von Hippel–Lindau disease, multiple endocrine neoplasia type 2 (MEN II), familial paraganglioma
syndrome, and neurofibromatosis.

Biochemical findings during an attach are increased serum epinephrine and metanephrine, hyperglycemia, hypercalcemia, and erythrocytosis.

There is currently no consensus as to the optimal biochemical test for pheochromocytoma.

Plasma metanephrine testing has the highest sensitivity (up to 99%) for detecting a pheochromocytoma, but it has a lower specificity (85%).
This test is superior to measurement of circulating epinephrine and norepinephrine because plasma metanephrines are elevated continuously,
unlike epinephrine and norepinephrine, which are secreted intermittently.

Measurement of 24-hour urinary metanephrines and catecholamines yields a sensitivity of 90% and specificity of 98%. It is often used as a
screening test for low-risk patients.

Test selection criteria include the following:

Use plasma metanephrine testing in patients at high risk (ie, those with predisposing genetic syndromes or a family or personal
history of pheochromocytoma)
Use 24-hour urinary collection for catecholamines and metanephrines in patients at lower risk

NOTE - Thyrotoxicosis, hypoglycemia, anxiety or panic attacks, hyperthyroidism, adrenal medullary hyperplasia, familial dysautonomia, and
intracranial lesions may also have similar symptoms. Various tumors including neuroblastomas, ganglioneuroblastomas, and
ganglioneuromas may mimic pheochromocytomas.

References

• RACGP – AFP – Incidental adrenal masses – A primary care approach

• MSD Manuals – Pheochromocytoma

• Medscape – Pheochromocytoma

Last updated:
Time spent: QID:1624
2023-2-12

129 of 1943
A 55-year-old woman presents to your clinic with a swelling in the left lower part of her neck. The swelling has developed over the past 4
months, is painless, and feels rubbery and regular on palpation with no tenderness. She is otherwise healthy and asymptomatic. Ultrasound
confirms the presence of a thyroid nodule; therefore, you perform a fine needle aspiration (FNA) sampling and send it for pathology
assessment. The pathology result reports only the presence of follicular cells. Which one of the following would be the most appropriate next
step in management?

A. CT scan of the neck.

B. Direct laryngoscopy.

C. MRI of the neck.

D. Repeat the FNA.

E. Excisional biopsy of the lump.

Incorrect. Correct answer is E


45% answered correctly

Explanation:

Correct Answer Is E

Nodular thyroid disease is a common clinical problem. Steps to consider in approaching to a thyroid nodule and excluding malignancy include
(1) a thorough clinical evaluation, (2) thyroid function tests (TFT), and (3) an ultrasound exam of the thyroid gland and fine needle aspiration
(FNA) biopsy if the nodule is larger than 1 cm.

Thyroid FNA biopsy is the most accurate test for determining malignancy, and is the essential and integral part of evaluation of a thyroid
nodule. When guided by ultrasound, there results are even more accurate.

FNA results are classified as diagnostic (satisfactory) and non-diagnostic (unsatisfactory). Unsatisfactory smears, comprising 5-10% of all
results, are due to hyocellularity of the specimen, usually caused by cystic fluid in the specimen, bloody smear, or inappropriate preparation.

Diagnostic smears are conventionally classified as:

Benign

Benign cytology comprise 60-70% of all reports, is negative for malignancy, and incudes colloid nodule, cysts, or Hashimoto thyroiditis.

Malignant

Malignant cytology comprise 5% of all reports and is positive for malignancy including primary thyroid tumors or non-thyroid metastatic
cancers. Papillary carcinoma is the most common type of thyroid malignancy.

Indeterminate (atypical)

Indeterminate or suspicious specimens, comprising 10-20% of all reports, include atypical changes, Follicular neoplasms, or Hurthle cells.
Overall, the indeterminate is associated with a risk of malignancy anywhere between 15% to 60% risk of malignancy (average 30%), depending
on the specifics of the report.

Of these conditions, the main difficulty arises in cases of follicular neoplasm because it is impossible to distinguish a follicular adenoma (a
benign lesion) from a follicular carcinoma on cytological assessment. Therefore, follicular lesions therefore often require excision and full
examination of the lesion and its capsule before a definitive diagnosis can be made.

In general, lesions with atypical features are best completely excised for full evaluation and making a definite diagnosis.

Repeating the FNA (option D) under ultrasound guide is used in cases of non-diagnostic reports in a reattempt to obtain an adequate and
appropriately prepared sample.

(Option B) Laryngoscopy adds no diagnostic vale to evaluation of a nodule within the thyroid gland and a completely irrelevant option here.

CT scan (option A) or MRI (option C) are not routinely indicated in the assessment of thyroid nodules because they cannot reliably distinguish
benign from malignant lesions. The main indication for CT scan or MRI is to determine the presence and extent of retrosternal extension and
the presence and degree of tracheal compression in the presence of obstructive symptoms.

References

130 of 1943
• RACGP – Thyroid lumps and bumps
Last updated:
Time spent: QID:1546
2023-2-12

131 of 1943
A 54-year-old woman presents with a swelling in the neck which has progressively enlarged for the past few months. Investigations reveal the
mass to be goiter. Which one of the following symptoms, if present, should make you think of urgent surgery?

A. Retrosternal extension.

B. Hoarseness.

C. Dysphagia.

D. Dyspepsia.

E. Puffiness of her face on raising her arms above the shoulder.

Incorrect. Correct answer is E


45% answered correctly

Explanation:

Correct Answer Is E

Goiter refers to an enlarged thyroid gland. Causes of goiter include autoimmune disease, the formation of one or more thyroid nodules and
iodine deficiency. Goiter occurs when there is reduced thyroid hormone synthesis secondary to biosynthetic defects and/or iodine deficiency,
leading to increased thyroid stimulating hormone (TSH). This stimulates thyroid growth as a compensatory mechanism to overcome the
decreased hormone synthesis.

Elevated TSH is also thought to contribute to an enlarged thyroid in the goitrous form of Hashimoto thyroiditis, in combination with fibrosis
secondary to the autoimmune process in this condition. In Graves’ disease, the goiter results mainly from stimulation by the TSH receptor
antibody.

Substernal goiter may obstruct the thoracic inlet. Pemberton sign refers to faintness with evidence of facial congestion and cyanosis due to
external jugular venous obstruction when the arms are raised above the head, a manoeuvre that draws the thyroid into the thoracic inlet.

Of the option, urgent surgery in only indicated who have troublesome compressive symptoms (positive Pemberton sign) and/or fail to
respond to medical therapy.

Pemberton sign - Note the congestion of the face on arm raising

(Option A) Retrosternal extension of the thyroid gland per se is not an indication for emergency surgery unless it causes compressive signs
and symptoms as explained above.

(Option B, C and D) None of these signs or symptoms is an indication for emergency surgery. Dyspepsia is not likely to be a direct mass effect
of goiter. Dysphagia or hoarseness might be caused by a direct mass effect but they do not indicate the need for emergency surgery.

TOPIC REVIEW

Causes of goiter:

132 of 1943
Hashimoto thyroiditis
Graves’ disease
Familial or sporadic multinodular goiter
Iodine deficiency
Follicular adenoma
Colloid nodule or cyst
Thyroid cancer

References

• RACGP – AFP – Goitre

Last updated:
Time spent: QID:1562
2023-2-12

133 of 1943
Ethan is a 16-year-old boy who has presented to your clinic with breast enlargement shown in the following photograph. The breast
enlargement was first noticed four months ago as a small swelling under the nipple that has enlarged gradually in the next two months and
has remained the same size since then. He has been your patient since the age of nine years with the complaint of asthma that is currently
controlled on inhaled salbutamol on an as-needed basis and fluticasone three times daily. He takes no other medications. Physical
examination reveals mild tenderness of breasts. The rest of the examination is unremarkable. Which one of the following is the most
appropriate management for him?

A. Changing his asthma medication.

B. Reassurance.

C. Mastectomy.

D. Testosterone injection.

E. Ultrasound of the testes.

Incorrect. Correct answer is B


45% answered correctly

Explanation:

Correct Answer Is B

The photograph shows bilateral enlargement of breast with a discoid fashion in a pubertal boy consistent with gynecomastia as the most
likely diagnosis. Gynecomastia can be physiologic or nonphysiologic (pathologic).

Physiologic gynecomastia has three phases of occurrence, corresponding to times of hormonal changes. The first peak occurs in the
neonatal period due to transplacental passage of estrogen. Gynecomastia of the neonate is seen in 60% to 90% of neonates and almost
always resolve spontaneously within the first year of life.

The second peak occurs during puberty when palpable breast tissue and increase in breast size occurs. It can present as early as 10 years,
but the peak of onset is between the ages of 13 and 14 years. The incidence declines in late teenage years. By the age of 17, only 10% of boys
have persistent gynecomastia. The last peak of occurrence is in adults between the ages 50 and 80 years.

For a patient presenting with gynecomastia, a thorough history, including drug history and physical exam should be undertaken to exclude
pathologic causes of gynecomastia such as medications and drug abuse, hypogonadism, chronic liver or kidney disease, hyperthyroidism,
tumors (e.g. testicular, adrenal) and malnutrition. In 25% of cases with gynecomastia no identifiable cause is found (idiopathic
gynecomastia).

Ethan is 16 years old and has no remarkable clues in history indicating a pathologic cause. This suggests he has pubertal gynecomastia.
Pubertal gynecomastia often resolves between 6 months and 2 years of onset and requires no active management except providing
explanation and reassurance.

134 of 1943
(Option A) Although medications are the cause of gynecomastia in 10 to 25% of cases, salbutamol and fluticasone (and almost none of other
asthma medication) has shown a relation with development of gynecomastia, and an advice for a change of asthma medications is not a
correct option

(Option C) Mastectomy is considered for patients with persistent gynecomastia who are distressed by their breast appearance due to
cosmetic reasons.

(Option D) Testosterone injection may be an option for patients with hypogonadism for general treatment of hypogonadism and not merely
for the gynecomastia.

(Option E) Testicular cancers should be considered in younger adults presenting with pathologic gynecomastia. While some authors advocate
routing use of ultrasound for detection a non-palpable testicular mass, the majority advocate routine use of this modality in patients who have
abnormal scrotal exam findings such as a testicular mass, signs of feminization, and other indicators of testicular malignancies. Ethan has
the provisional diagnosis of physiologic gynecomastia and there is no clues in the history and physical examination to mandate a scrotal
ultrasound.

References

• PubMed – Gynecomastia in adolescent males

• Medscape – Gynecomastia

• AFP – Gynecomastia

• PubMed – Gynecomastia: Clinical evaluation and management

Last updated:
Time spent: QID:1494
2023-2-12

135 of 1943
Adrian is a 17-year-old boy, who has presented to your GP clinic with complaint of painful breast enlargement for the past three months as
shown in the following photograph. He has asthma that is well-controlled on inhaled salbutamol and beclomethasone. He has no other
medical condition, otherwise takes no other medications, and enjoys a good health. He plays football in his school team. He denies using
illicit drugs. Physical examination shows unilateral right breasts enlargement starting from the nipple and spreading concentrically. Touching
the breast causes mild pain. There is no discharge or lymphadenopathy. The rest of the examination including testicular examination is
unremarkable. Which one of the following is the most likely cause of this presentation?

A. Inhaled salbutamol.

B. Inhaled beclomethasone.

C. Misuse of anabolic steroids.

D. Physiologic gynecomastia of adolescents.

E. A testicular tumor.

Incorrect. Correct answer is C


45% answered correctly

Explanation:

Correct Answer Is C

The photograph and the description of physical findings in in favor of gynecomastia as the most likely diagnosis.

The three major causes of gynecomastia are physiologic, idiopathic and drug/substance abuse related. Physiologic gynecomastia comprise
25% of all cases and is seen in neonatal, pubertal, and in males between 50 to 80 years of age. Medications including drugs of abuse such as
alcohol, anabolic steroids, amphetamine and opiated account for 10 to 25% of cases. In 25% of cases gynecomastia is idiopathic and no
identifiable cause is found. Other less likely causes are primary and secondary hypogonadism, chronic liver disease, chronic renal disease,
tumors (e.g., testicular, adrenal), malnutrition/ starvation, and hyperthyroidism as well as other rarer causes.

In this boy, persistent pubertal gynecomastia (not an option) can be the best option, given the absence of other physical findings that may
suggest a medical condition such as hypogonadism or testicular tumors. Persistent pubertal is classified under idiopathic gynecomastia, and
defined as pubertal gynecomastia not resolving after 2 years, or persisting beyond 17 years of age.

NOTE – by the age of 17, pubertal gynecomastia is resolved in 90% of boys. Gynecomastia beyond this age is no more termed physiologic
and is categorized under idiopathic pathologic gynecomastia.

In the absence of idiopathic gynecomastia that statistically seems the likely explanation, medications and specific substances should be
considered as a possibility. Many medications, based either on anecdotal reports or studies of different quality, are known to cause
gynecomastia. Of Adrian’s medications, neither inhaled salbutamol (option A) nor inhaled betamethasone (option B) can cause
gynecomastia. Another possibility to consider, especially in younger patients with athletic behavior, is misuse of anabolic steroids. This boy is
a football player and is likely to have used or is using anabolic steroids for enhanced performance. In younger patients with gynecomastia
who also have athletic behaviors, misuse of anabolic steroids should always be considered as a possibility, an in fact the best option among
others in this scenario.

(Option D) Physiologic gynecomastia of adolescents is expected to resolve between 6 months to 2 years of onset and usually does not
persist beyond 17 years of age. On the other hand, the breast enlargement in the photograph exceeds 5cm in diameter (macromastia).
Macromastia suggest pathologic rather than physiologic gynecomastia.

(Option E) Testicular tumors are an important but not common cause of gynecomastia, especially in younger patients. Although less likely,
testicular tumors should be considered a possibility and investigated thoroughly, especially if there is a testicular lump. However, in the
absence of signs and symptoms, a testicular tumor is less likely to be the cause of such presentation.

136 of 1943
References

• PubMed – Gynecomastia in adolescent males

• Medscape – Gynecomastia

• AFP – Gynecomastia

• PubMed – Gynecomastia: Clinical evaluation and management

Last updated:
Time spent: QID:1492
2023-2-12

137 of 1943
A 68-year-old man has bilateral breast enlargement as shown in the accompanying photograph. His medical history includes hypertension,
rheumatoid arthritis (RA), and gastroesophageal reflux disease (GERD). He takes nifedipine and hydrochlorothiazide for hypertension,
ibuprofen for the RA, and ranitidine for the reflux. He is a heavy alcohol drinker and takes 8-12 standard units of alcohol every day. He also
smokes two packs of cigarettes per day. On examination, he has a blood pressure of 145/98 mmHg, pulse rate of 92 bpm and respiratory rate
of 18 breaths per minute. There is bilateral breast enlargement consistent with gynecomastia. In addition, he has bilateral parotid
enlargement. The abdomen is soft and non-tender with no ascites or organomegaly. Which one of the following is most likely to have caused
this presentation?

A. Smoking.

B. Ibuprofen.

C. Ranitidine.

D. Nifedipine.

E. Alcoholic liver disease.

Incorrect. Correct answer is E


45% answered correctly

Explanation:

Correct Answer Is E

The photograph and the physical findings are highly suggestive of gynecomastia. Gynecomastia, is the development of female-like breasts in
a male caused by proliferation of breast glandular tissue.

Gynecomastia can be physiologic in neonates, adolescents and older males between the age of 50 to 80 years, or pathologic (nonphysiologic)
and caused by a medical condition, medications, or substances of abuse (alcohol, marijuana, heroin, anabolic steroids). In idiopathic
gynecomastia, no identifiable cause is found. The three major etiologies of gynecomastia are physiologic (25% of all cases), idiopathic (25%
of all cases), and medications/ substance use (10-25%).

Other contributing factors include chronic liver disease, malnutrition, primary or secondary hypogonadism, tumors (e.g. testicular, adrenal),
hyperthyroidism, chronic kidney disease/ dialysis, and other rarer causes.

It is important to differentiate gynecomastia from pseudogynecomasita that is fat deposition in obese men and also from breast cancer,
although male breast cancer is rare and comprise only 1% of all breast cancers. Nonetheless, it should always be considered as possibility in
men presenting with gynecomastia, especially if it is unilateral and painless.

Medications are another common etiology for breast cancer. The most commonly reported medications with a high association with
gynecomastia are inhibitors of testosterone production (e.g. cyproterone acetate, spironolactone, finasteride, flutamide, ketoconazole, etc.),
anabolic steroids, cimetidine, digoxin (digitalis), captopril, and methyldopa.

The following table lists medications that have been reported to be associated with gynecomastia. Most of the listed the listed drugs are
based on anecdotal reports and do not have high quality evidence supporting a cause-and-effect relationship. Using evidence-based criteria,
there is good evidence of a true association with estrogens, spironolactone, cimetidine, ketoconazole, growth hormone, gonadotropins,
antiandrogens therapies and 5-alpha reductase inhibitors (shown in red in the table).

Medications with fair evidence include first-generation and atypical antipsychotics in adults, calcium channel blockers, omeprazole, HIV drugs,
alkylating agents, anabolic steroids, alcohol and opioids (shown in green in the table). Other drugs only have been reported anecdotally
without quality supporting evidence.

Antiandrogens/ inhibitors of testosterone synthesis Drugs of abuse

138 of 1943
Cyproterone acetate Alcohol
Flutamide, nilutamide, bicalutamide Amphetamines
Finasteride, dutasteride Heroin
Spironolactone Marijuana
Ketoconazole Methadone
Lavender oil
Tea tree oil

Hormones
Anabolic steroids
Chorionic gonadotropin
Antibiotics
Estrogens
Ethionamide Growth hormone
Isoniazid
Ketoconazole
Metronidazole

Antiulcer drugs Psychoactive drugs


Cimetidine Diazepam
Ranitidine Haloperidol
Omeprazole Phenothiazines
Tricyclic antidepressants
Atypical antipsychotics
Cancer/ chemotherapeutic drugs
Alkylating agents
Methotrexate Other
Vinca alkaloids Auranofin (gold salt – often used for RA)
Combination chemotherapy Sulindac (NSAID – often used for RA))
Imatinib Metoclopramide
Theophylline
Cardiovascular drugs Phenytoin
Penicillamine
ACE inhibitors (enalapril, perindopril, captopril)
Domperidone
Amiodarone
Etretinate (used to treat severe psoriasis)
Calcium channel blockers (diltiazem, nifedipine,
HAART (highly active retroviral therapy
amlodipine)
Digoxin
Methyldopa
Reserpine

This patient is on multiple medications and also taking excess alcohol. Of his medications, nifedipine (option D) can be the culprit for the
gynecomastia (fair evidence). Ranitidine could be a less likely, yet not impossible cause of this presentation. Ibuprofen (option B) has not
shown any associated with gynecomastia, neither does smoking (option A).

In this patient, the gynecomastia and bilateral parotid enlargement can be early indicators of alcoholic liver disease and the best option
among others. Later in the course of the disease, other signs such as ascites, jaundice, pruritus, etc. can develop. It should be noted though
that alcohol alone (not an option) even in the absence of liver disease can produce such clinical picture. Alcoholic people can develop
gynecomastia due to phytoestrogen and parotid enlargement through disrupted fat metabolism and fat deposition in parotid glands.

NOTE – given the absence of other features of liver disease in the question, and alcohol in the options, it is only a matter of justifying the best
option among of the others. The only two options that fit the scenario best are alcoholic liver disease and nifedipine. Since the patient has
been on long-term nifedipine but just recently has developed gynecomastia, it is less likely (but not impossible) that nifedipine has caused
gynecomastia compared to liver disease for which an additional fair clue (parotid enlargement) is provided in the scenario.

References

• Medscape – Gynecomastia

• AFP – Gynecomastia

• PubMed – Gynecomastia: Clinical evaluation and management

• UpToDate – Gynecomastia, clinical features, diagnosis, and evaluation of gynecomastia in adults

Last updated:
Time spent: QID:1490
2023-2-12

139 of 1943
Benjamin is 65 years old and a regular patient in your GP clinic. He has presented with complaint of left breast enlargement for the past three
months. He has congestive heart failure (CHF) diagnosed two years ago, for which he was started on metoprolol, captopril, and digoxin. Four
months ago, he developed ankle edema, nocturnal cough, and exertional dyspnea. Based on the diagnosis of CHF decompensation,
spironolactone was added to his medications with adequate control of his symptoms. He does not smoke, nor does he drink alcohol. His
family history is significant for breast cancer in his mother at the age of 58 years and in his sister at the age of 50 years. On examination, a
tender round rubbery mass 4x3 cm in size and concentric with the left nipple is palpated. The mass is regular and mobile. The other breast is
normal. There is no lymphadenopathy, nipple discharge or skin changes. Which one of the following would be most appropriate next step in
management?

A. Ultrasound of the breast.

B. Cessation of spironolactone.

C. Fine needle aspiration (FNA) of the breast lump.

D. Cessation of digoxin.

E. Excision of the breast lump.

Correct
45% answered correctly

Explanation:

Correct Answer Is A

The clinical findings in in Benjamin is highly suggestive of gynecomastia as the most likely diagnosis.

Gynecomastia is characterized by proliferation of breast glandular and adipose tissue causing a female -like breast in a male. Gynecomastia
can be physiologic in neonates, pubertal, and in adult males aged 50-80 years. Physiologic gynecomastia accounts for 25% of all cases of
gynecomastia. Pathologic (nonphysiologic) causes of gynecomastia include medications and substance abuse (10-25%), liver disease or
cirrhosis (8%), primary/secondary hypogonadism (10%), tumors (3%), hyperthyroidism (1.5-2%), renal disease/ dialysis (1%), and other
identifiable causes (6%). In 25% of cases, including persisting pubertal gynecomastia (gynecomastia not resolving within 6 months to 2 years,
or persisting after 17 years of age), no identifiable cause is found (idiopathic gynecomastia).

Gynecomastia is not palpable unless the diameter of the glandular tissue exceeds 0.5 cm. Gynecomastia has four characteristic features on
physical examination:

1. centrally located
2. symmetrical in shape
3. usually bilateral
4. tender to palpation

On physical examination the clinical findings include palpation of a rubbery or firm mass concentric with the nipple and areola. Tenderness
and discomfort are often evident on palpation. It is bilateral in over 90% of patients, but sometimes patients present with unilateral
enlargement or bilateral enlargement but one side large than the other. One side may enlarge weeks to months later before the other.

Although, unilaterality alone is not an indicator of pathologic gynecomastia, some studies suggest that it could be more common with specific
etiologies such as medications.

Gynecomastia with a diameter exceeding 5 cm is referred to as macromastia. Macromastia in more cases has a pathologic rather than
physiologic etiology, especially if unilateral.

For patients presenting with gynecomastia, a thorough history and physical examination should be undertaken. In history, important points to
underpin are pubertal development and stage (in younger adults), any medications taken or substances used by the patient, medical
conditions such as thyroid, liver or kidney disease, libido, and history of cancer. In physical examination, differentiation between true
gynecomastia and other conditions such as pseudogynecomastia and, most importantly, breast cancer is very important. Other clues to look
for are signs of liver disease (jaundice, pruritus, ascites, enlarged parotid glands, spider nevi, etc.), renal disease, hypogonadism (feminization,
decreased libido and sexual function), hyperthyroidism, testicular cancer (e.g., testicular mass), and drug abuse (opiates, alcohol, anabolic
steroids).

If based on the history and physical findings, the case turns out to be physiologic gynecomastia, no further evaluation is required. Similarly,
asymptomatic and pubertal gynecomastia do not require further tests and should be reevaluated in six months. On the other hand, further
evaluation is indicated in the following situations:

Macromastia (breast size exceeding 5 cm in diameter)


A lump that is tender, of recent onset, progressive, or of unknown duration
Signs of breast malignancy i.e. stony hard mass, irregular borders, being fixed to the surrounding tissues, lymphadenopathy,
unintentional weight loss, loss of appetite, skin changes such as dimpling, or nipple discharge.
140 of 1943
There is concerns about testicular cancer or other non-breast malignancies

A serum chemistry panel may be helpful for renal or liver disease is suspected. Free or total testosterone, luteinizing hormone (LH), estradiol,
and dehydroepiandrosterone sulfate levels are used to evaluate a patient with possible feminization syndrome. Thyroid-stimulating hormone
(TSH) and free thyroxine levels should be obtained if hyperthyroidism is suspected.

Routine use of sonography for patients with nonphysiologic gynecomastia has been a place of debate; however, recent studies as well as
expert opinion suggest that ultrasonography as a readily available, non-invasive and non-expensive investigation be used to establish a
diagnosis of true gynecomastia and its size and also exclusion of other differential diagnoses such as cancer, pseudogynecomastia, lipoma,
hematoma, etc., especially if findings are suggestive.

Also, scrotal sonography for possible testicular cancer is endorsed and recommended for the those with a testicular mass found on physical
examination, increased serum estradiol or HCG levels, macromastia, and otherwise unexplained gynecomastia, especially in younger patients.

Benjamin’s breast enlargement is very likely to have been caused by spironolactone. There is strong evidence for spironolactone as an inducer
of gynecomastia mostly through inhibition on testosterone production, but cessation of spironolactone (option B) on which his CHF is stable
is not a wise advice as it can cause deterioration of his CHF symptoms. Digoxin has been reported as a cause of gynecomastia, but compared
to spironolactone, the evidence is fair. Nonetheless, it can still be the cause; though less likely than spironolactone. Even though, cessation of
digoxin (option D) can result in CHF decompensation, and should not be advised.

Of the option, a breast ultrasound is a reasonable choice because it can safely exclude other causes of breast enlargement, confirm a certain
diagnosis, and provide relief in case it confirms the provisional diagnosis of gynecomastia.

FNA of the breast lump (option C) must be considered if imaging suggests malignancy. Excision of the breast lump (option E) for now is
unnecessary because it is invasive and adds no diagnostic benefit.

References

• Medscape – Gynecomastia

• PubMed – Gynecomastia: Clinical evaluation and management

• AFP – Gynecomastia

Last updated:
Time spent: QID:1488
2023-2-12

141 of 1943
A 62- year-old woman presents with bloody discharge from her left nipple. The family history is significant for a cousin recently diagnosed
with breast cancer. On examination, no mass is palpated. Which one of the following is the most likely diagnosis?

A. Breast duct papilloma.

B. Breast duct ectasia.

C. Paget disease of the breast.

D. Mastitis.

E. Breast duct carcinoma.

Correct
45% answered correctly

Explanation:

Correct Answer Is A

Nipple discharge, as a sign, may have a variety of underlying causes. Some of these causes are as follows:

Bloody discharge – bloody discharge from nipple is usually caused by benign ductal papilloma, but breast neoplasms including breast
duct carcinoma and Paget disease of the breast should always be excluded.

Serous, green, yellow-brown discharge – it is usually due to benign fibrocystic changes

Toothpaste (worm)-like discharge – it is characteristic of mammary ductal ectasia.

The bloody nipple discharge of this woman is most likely to have been caused by benign ductal papilloma; however, breast duct carcinoma
(option E) and Paget disease of the breast (option C), as less common possibilities, should be considered as well.

Breast duct ectasia (option B) presents with worm-like or toothpaste-like but not bloody nipple discharge.

Mastitis (option D) presents with local tenderness and redness of the breast. The clinical picture is completely different from this case
scenario. Moreover, it is a very uncommon condition in a woman of this age.

References

• Anthology of Medical Conditions – AMC Publication – page 105

• RACGP - AFP - Nipple Discharge

Last updated:
Time spent: QID:459
2023-2-12

142 of 1943
Lucas is 67-year-old patient of yours in your GP clinic. He has congestive heart failure (CHF) diagnosed three years ago for which he has
been on carvedilol, perindopril, and digoxin with improvement in symptoms. Six months ago, due to decompensation of his CHF, evident by
increased dyspnea and ankle edema, and after specialist advice, he was started on spironolactone. Today, he has presented with complaint of
painful unilateral breast enlargement. He does not smoke, and drinks only on social occasions and within safe limits. His past medical history,
besides the CHF, is myocardial infarction seven years ago. His family history is remarkable for breast cancer in his mother, of which she died
15 years ago, and in his sister at the age of 52. On examination, you notice a left breast lump concentric with the nipple which is firm and
rubbery and slightly tender. There is no nipple discharge, ulceration or regional lymphadenopathy. The other breast is normal. Which one of
the following could be the most likely cause of this presentation?

A. Breast cancer.

B. Digoxin.

C. Spironolactone.

D. Carvedilol.

E. Perindopril.

Incorrect. Correct answer is C


45% answered correctly

Explanation:

Correct Answer Is C

The description of the breast enlargement in Lucas is consistent with gynecomastia as the most likely diagnosis. Gynecomastia is defined
histologically as proliferation of breast glandular and adipose tissue, and clinically as the development female -like breast in a male.
Gynecomastia, in most cases is due to increased ratio of female sex hormones to testosterone, and presents with a firm rubbery mass
concentric with the nipple. It is bilateral in most cases but in 5-10% of men it can be unilateral and for unknown reasons more often on the left
side.

Gynecomastia can be physiologic in neonates, adolescents and older males between the age of 50 to 80 years, or pathologic (nonphysiologic)
and caused by a medical condition, medications, or substances of abuse (alcohol, marijuana, heroin, anabolic steroids). In idiopathic
gynecomastia, no identifiable cause is found. The three major etiologies of gynecomastia are physiologic (25% of all cases), idiopathic (25%
of all cases), and medications/ substance use (10-25%).

Other contributing factors include chronic liver disease, malnutrition, primary or secondary hypogonadism, tumors (e.g. testicular, adrenal),
hyperthyroidism, chronic kidney disease/ dialysis, and other rarer causes.

It is important to differentiate gynecomastia from pseudogynecomasita that is fat deposition in obese men and also from breast cancer.
Breast cancer (option E) is rare in men and comprise less than 1% of all cases of breast cancer. On the other hand, breast cancer presents
differently with a stony hard lump that is usually immobile and attached to the surrounding structures with possible skin dimpling,
lymphadenopathy and nipple discharge, or ulceration in advanced disease. Nonetheless, it should always be considered as possibility in men
presenting with gynecomastia, especially if it is unilateral and painless.

Medications are another common etiology for breast cancer. The most commonly reported medications with a high association with
gynecomastia are inhibitors of testosterone production (e.g. cyproterone acetate, spironolactone, finasteride, flutamide, ketoconazole, etc.),
anabolic steroids, cimetidine, digoxin (digitalis), captopril, and methyldopa.

The following table lists medications that have been reported to be associated with gynecomastia. Most of the listed the listed drugs are
based on anecdotal reports and do not have high quality evidence supporting a cause-and-effect relationship. Using evidence-based criteria,
there is good evidence of a true association with estrogens, spironolactone, cimetidine, ketoconazole, growth hormone, gonadotropins,
antiandrogens therapies and 5-alpha reductase inhibitors (shown in red in the table).

Medications with fair evidence include first-generation and atypical antipsychotics in adults, calcium channel blockers, omeprazole, HIV drugs,
alkylating agents, anabolic steroids, alcohol and opioids (shown in green in the table). Other drugs only have been reported anecdotally
without quality supporting evidence.

Antiandrogens/ inhibitors of testosterone synthesis Drugs of abuse

143 of 1943
Cyproterone acetate Alcohol
Flutamide, nilutamide, bicalutamide Amphetamines
Finasteride, dutasteride Heroin
Spironolactone Marijuana
Ketoconazole Methadone
Lavender oil
Tea tree oil

Hormones
Anabolic steroids
Chorionic gonadotropin
Antibiotics
Estrogens
Ethionamide Growth hormone
Isoniazid
Ketoconazole
Metronidazole

Antiulcer drugs Psychoactive drugs


Cimetidine Diazepam
Ranitidine Haloperidol
Omeprazole Phenothiazines
Tricyclic antidepressants
Atypical antipsychotics
Cancer/ chemotherapeutic drugs
Alkylating agents
Methotrexate Other
Vinca alkaloids Auranofin (gold salt – often used for RA)
Combination chemotherapy Sulindac (NSAID – often used for RA))
Imatinib Metoclopramide
Theophylline
Cardiovascular drugs Phenytoin
Penicillamine
ACE inhibitors (enalapril, perindopril, captopril)
Domperidone
Amiodarone
Etretinate (used to treat severe psoriasis)
Calcium channel blockers (diltiazem,
HAART (highly active retroviral therapy
nifedipine, amlodipine)
Digoxin
Methyldopa
Reserpine

Of the medications Lucas is taking, spironolactone, digoxin (option B), and perindopril (ACE inhibitor) (option E) have been reported to have a
potential to cause gynecomastia. Spironolactone has the best evidence to cause gynecomastia, and most likely to have caused this
presentation. Although gynecomastia can occur even after prolonged use of a potential medication, the fact that Lucas has developed
gynecomastia after being started on spironolactone, is in line with this option as well.

Carvedilol (beta blocker) (option D) has not shown a meaningful association with gynecomastia.

References

• Medscape – Gynecomastia

• PubMed – Gynecomastia: Clinical evaluation and management

• AFP – Gynecomastia

• UpToDate – Gynecomastia, clinical features, diagnosis, and evaluation of gynecomastia in adults

Last updated:
Time spent: QID:1486
2023-2-12

144 of 1943
Mia, 45 years of age, presents for advice about reducing the risk of breast cancer after her mother was diagnosed with breast cancer at the
age of 60 years. She started menstruation at the age of 17 and has three children, the first born when he was 37 years. She has been on
combined oral contraceptives for the past four years and is happy with that. She smokes 10 to 15 cigarettes a day. Her past medical history
incudes benign breast disease including simple breast cyst and fibrocystic changes of the breast. Of the factors in the history, which one
could not be a risk factor for development of breast cancer in her?

A. The history of previous breast diseases.

B. Smoking.

C. Family history of breast cancer in her mother.

D. Combined oral contraceptives.

E. Her first pregnancy at the age of 37.

Correct
45% answered correctly

Explanation:

Correct Answer Is A

Of the given option, only Mia’s history of previous breast diseases cannot be a cause for breast cancer. All other options, with different relative
risks, has been shown to be associated with development of breast cancer as risk factors.

Some breast diseases have a relation with breast cancer and are associated with a slight increase in the risk of a woman catching breast
cancer while others not. Benign breast diseases represent a spectrum of breast disorders noticed incidentally during imaging, because of the
symptoms or presence of a breast lump. Some of such diseases are associated with an increase in the patient’s future risk of developing
breast cancer. These diseases are rather risk markers than premalignant, because those cancers that subsequently develop are not
necessarily in the area the breast disease exists and the malignancy may occur in the contralateral breast.

Benign breast disease can be categorized histologically into three categories:

1. Nonproliferative

These diseases are not generally associated with an increased risk of breast cancer and include: simple cyst (the most
common non-proliferative breast disease), papillary apocrine change, epithelial related calcification, and mild hyperplasia of
usual type. It is important to note that terms such as fibrocystic changes, fibrocystic disease, chronic cystic mastitis, and
mammary dysplasia refer to nonproliferative lesions. These terms are not clinically useful because they encompass a
heterogeneous group of diagnoses, not a single diagnosis.

2. Proliferative without atypia

Proliferative lesions without atypia include usual ductal hyperplasia, intraductal papillomas, sclerosing adenosis, and radial
scars. These lesions are associated with a small increased risk of developing breast cancer (approximately 1.5-2 times that
of the general population). Fibroadenomas are associated with increased risk of breast cancer only if the fibroadenoma is
complex, if there is adjacent proliferative disease, or if there is a family history of breast cancer. For the majority of women
with fibroadenomas the cancer risk is not increased.

3. Proliferative with atypical hyperplasia

Proliferative breast diseases with atypical hyperplasia includes atypical ductal hyperplasia and atypical lobular hyperplasia.
Atypical hyperplasia is pathologic diagnosis, usually found as an incidental on biopsy of mammographic abnormalities or
breast lumps. These lesions have some, but not all, features of ductal carcinoma in situ or lobular carcinoma in situ. These
lesions, especially if multifocal are associated with a significantly increased risk of breast cancer.

Mia’s breast cyst and the fibrocystic disease are both nonproliferative lesions from category one and does not confer an increased risk of
breast cancer.

(Option C) Most women with breast cancer do not have a family history; however, having one or more first-degree relatives or second-degree
relatives on the same side of family with breast cancer is associated with increased risk of breast cancer. Mia, with one first-degree relative
(mother) diagnosed with breast cancer after the age of 50, is at slightly increased risk of breast cancer (1.8 times the normal population).
Despite being modest, family history can play a role as cause of breast cancer.

(Option B) Many studies have shown that tobacco smoking may be associated with an increased risk of breast cancer. While there are some
limitations to the evidence, studies have been generally consistent in supporting a link between tobacco smoking and increased risk of breast,
particularly in women who have started smoking when they were younger or have started smoking many years before having their first child.

145 of 1943
(Option D) Combined’ oral contraceptives (COCs) are associated with a small increase in risk of breast cancer while being taken. This risk is
estimated to be 7% for every 5 year use and goes down again when stopped. It is estimated that 0.7% of breast cancers each year in Australia
are attributable to the use of COCs.

(Option E) First full-term pregnancy after the age of 35 is associated with increased risk of breast cancer (RR: 2.1-4).

TOPIC REVIEW

The most important risk factors for development of breast cancer include the following:

Factors associated with the highest risk for development of breast cancer (relative risk [RR] >4.0):

Advanced age (65 years and older)


Atypical hyperplasia of breast (biopsy proven)
Certain inherited genetic mutations ( BRCA1, BRCA2, TP53, ATM, CDH1); (RR: 4-8)
Ductal or lobular carcinoma in situ (DCIS/LCIS); (RR: 8-10)
Family history of early ovarian cancer (age <50)
Multiple first-degree relatives with breast cancer
Ionizing radiation exposure before age 30 (RR: 22-40)
Personal history of early breast cancer (age <40)

Factors associated with RR of 2.1-4.0 for breast cancer:

High endogenous estrogen or testosterone level (postmenopausal)


First full-term pregnancy after age 35 years
Very dense breasts
One first-degree relative with breast cancer
Atypical proliferative breast diseases (e.g. atypical ductal hyperplasia)
Certain inherited genetic mutations (e.g. CHEK2, PTEN)

Factors associated with RR of 1.1-2.0 for breast cancer:

Alcohol consumption
Age 30-35 at first full-term pregnancy
Diethylstilbestrol exposure in utero
Early menarche (age <12 years)
Height (>5 feet 3 inches)
High socioeconomic class
Ashkenazi Jewish heritage
Personal history of breast cancer (age of onset >40)
Dense breasts
Benign breast conditions: typical ductal hyperplasia, fibroadenoma (in certain groups of women), sclerosing adenosis, microglandular
adenosis, papillomatosis, radial scar

References

• Cancer Australia – Breast Cancer Risk Factors

• UpToDate – Overview of Benign Breast Diseases

Last updated:
Time spent: QID:1484
2023-2-12

146 of 1943
Barney, 67 years old, presents with a left breast mass he noticed felt a few days ago. On physical examination, there is a non-tender left breast
mass which is firm and non-mobile. There is no lymphadenopathy. An ultrasound scan is arranged, the result of which reports a 2x3 cm
hypoechoic mass with irregular borders. Which one of the following is the most appropriate next step in management?

A. Excisional biopsy.

B. Mastectomy.

C. Fine needle aspiration (FNA) biopsy.

D. Follow-up ultrasound in 6 months.

E. Follow-up mammography in 6 months.

Incorrect. Correct answer is C


45% answered correctly

Explanation:

Correct Answer Is C

The approach to a breast lump in male patients is almost the same as for females and include physical examination, imaging and (if
indicated) biopsy and histological studies.

Clinical findings suggestive of a malignant lesion are the stony-hard consistency of the lesion, irregular borders, being non-tender, immobility,
and skin changes (e.g. dimpling). Ulceration of the skin is seen in advanced disease. Lymphadenopathy could be a feature but it could be
absent in early disease.

In approach to a suspicious mass, imaging should follow the physical examination. Mammography is the standard modality; however, it can
be associate with technical difficulties due to small size of breasts in men. Ultrasound can reliably be used for such purpose. On ultrasound, a
malignant lesion has characteristic features including:

Hypoechogenicity
Irregular and ill-defined borders
Spiculated margins
Being taller than broader
Posterior acoustic shadowing
Microcalcifications

Any suspicious mass should be biopsied to confirm the diagnosis and to assay for estrogen and/or progesterone receptors as well as human
epidermal growth factor receptor 2 status.

Barney’s ultrasound study shows hypoechogenicity and irregular borders of the mass suggestive of malignancy. In addition, firmness of the
mass and its immobility are in favor of malignancy as well. Therefore, the next step to consider is sampling via FNA or core needle biopsy.

FNA biopsy is convenient and minimally invasive and can provide adequate tissue for diagnostic purposes in many cases; however, in up to
25% of patients, the sample is insufficient for diagnosis. Compared to FNA, core needle biopsy offers a more definitive histological diagnosis,
avoids inadequate sampling, and usually distinguishes between invasive versus in situ cancer.

Open biopsy (excisional biopsy) (option A) may be an option if less invasive measures such as FNA or core needle biopsy fail to obtain
adequate samples.

While the mass has suspicious features of malignant lesion, follow-ultrasound (option D) or mammography (option E) are inappropriate, and
action should be taken for a definite diagnosis and management. Mastectomy (option B) would be the likely surgical component of the
treatment if biopsy establishes a definite diagnosis of breast cancer.

References

• Medscape – Breast Cancer in Men Overview of Male Breast Cancer

• UpToDate – Breast cancer in men

• PubMed – Ultrasound characterization of breast masses

Last updated:
Time spent: QID:1474
2023-2-12

147 of 1943
All of the following options are features of diabetic autonomic neuropathy EXCEPT:

A. Bradycardia.

B. Urinary retention.

C. Sexual impotence.

D. Diarrhea.

E. Foot ulcers.

Incorrect. Correct answer is E


45% answered correctly

Explanation:

Correct Answer Is E

Autonomic neuropathy is one of the complications of diabetes mellitus. Diabetic autonomic neuropathy is common and often a debilitating
condition. Clinical manifestations of diabetic autonomic neuropathy include the following:

Skin - dry skin (due to lack of sweating), or excessive sweating in defined areas
Eye - poor dark adaptation, sensitivity to bright lights
Cardiovascular - postural lightheadedness, fainting, orthostatic tachycardia, orthostatic bradycardia and orthostatic hypotension
Urinary - urgency, incontinence, dribbling, and bladder emptying abnormalities
Gastrointestinal - diarrhea, constipation, nausea and vomiting
Sexual - erectile dysfunction and ejaculatory failure in men, loss of ability to reach sexual climax in women

Although foot ulcers, a major source of morbidity in patients with diabetes, are a feature of diabetic neuropathy, it is unrelated to autonomic
dysfunction. Diabetic foot ulcers have a different pathophysiology, mostly including peripheral neuropathy and ischemia from peripheral
vascular disease. Poor resistance against infections is also a contributing factor.

References

• UpToDate – Diabetic autonomic neuropathy

• RACGP - General practice management of type 2 diabetes - Neuropathy

• PubMed – Cardiac autonomic neuropathy: Risk factors, diagnosis and treatment

Last updated:
Time spent: QID:1413
2023-2-12

148 of 1943
A 29-year-old woman, who has been recently diagnosed with nodular thyroid disease, is worried about developing thyroid cancer. Which one
of the following does not increase the risk of thyroid cancer?

A. Family history of benign thyroid disease.

B. Family history of thyroid cancer.

C. Graves' disease.

D. Chronic goiter.

E. Familial adenomatous polyposis.

Correct
45% answered correctly

Explanation:

Correct Answer Is A

While personal history of a benign thyroid diseases, such as thyroid nodules, is associated with increased risk of thyroid cancer, such
relationship has not been established for family history of benign thyroid diseases.

(Option B) A family history of thyroid cancer in a first-degree relative is a proven risk factor for thyroid cancer. The genetic basis is still
undetermined.

(Option C) Some research have concluded that Graves' disease can be associated with increased risk of thyroid cancer. It is not known
whether Garves' disease by itself or the treatments used (radiotherapy or thyroid ablation) increases such risk.

(Option D) Chronic iodine deficiency is a risk factor for development of thyroid cancer. Chronic goiter, as a presentation of such deficiency, can
increase the risk of thyroid cancer.

(Option E) Familial adenomatous polyposis (FAP) is a well-known risk factor for thyroid cancer. FAP is associated with colonic polyposis and a
very high risk of colon cancer. There is also an increased risk for other cancers such as papillary thyroid cancer. Gardner syndrome, a subtype
of FAP, is also associated with such risk as well.

TOPIC REVIEW

Risk factors for thyroid cancer:

Gender – thyroid cancer are 3 times more common in women than in men
Hereditary conditios
MEN IIa syndrome (pheochromocytoma, parathyroid tumors, and medullary thyroid cancer)
Familial non-medullary thyroid carcinoma
FAP (including Gardner syndrome)
Corney complex type1
Cowden disease (papillary and foliular thyroid cancers)
Family history
Autoimmune thyroiditis
Acromegaly
Systemic lupus erythematosus
Graves' disease
Diabetes
Personal (not family) history of benign thyroid disease such as thyroid nodules.
Low-iodine diet
Radiation exposure
Obesity
Taller height
Cigarette smoking

References

• Research UK - Thyroid cancer risk

Last updated:
Time spent: QID:1315
2023-2-12

149 of 1943
A 32-year-old woman presents to your GP clinic for advice on breast cancer after her maternal aunt was diagnosed with breast cancer
three months ago. She wants to know about the risk factors of breast cancer. She is obese and has a body mass index (BMI) of 45. She
smokes 20 cigarettes a day and is on oral contraceptive pills (OCP). The rest of examination, including breast exam, is unremarkable. Which
one of the following pieces of advice is the best to reduce the risk of breast cancer in her?

A. Smoking cessation.

B. Weight loss.

C. Regular exercise.

D. Using low-estrogen dose OCPs.

E. Using non-hormonal methods for contraception.

Incorrect. Correct answer is B


45% answered correctly

Explanation:

Correct Answer Is B

It has been estimated that approximately 20%of all cancers are caused by excess weight. The Million Women Study, the largest study of its
kind on women has shown that approximately half of cancers can be attributed to obesity in postmenopausal women. The International
Agency for Research into Cancer (IARC) and the World Cancer Research Fund (WCRF) reports show that common cancers in obese people
are predominantly endometrial, esophageal adenocarcinoma, colorectal, postmenopausal breast cancer, prostate and renal cancers. Less
common malignancies associated with obesity are melanoma, thyroid cancer, leukemia, non-Hodgkin’s lymphoma, and multiple myeloma.

The role of obesity in development of breast cancer in women is hypothesized to be due to increased production of endogenous estrogen in
the fat tissue and excess exposure of the breast tissue to unopposed endogenous estrogen. For this patient, weight reduction is the most
appropriate advice both for general health and risk reduction for breast cancer.

(Option A) Smoking have a positive link to a number of malignancies. Some reports advocate tobacco smoking as a risk factor for the
development of breast cancer; however, this risk is not as significant as obesity.

(Option C) Regular exercise is associated with improved overall health as well as decreased incidence of malignancies. The effect does not
seem to be superior to weight reduction for breast cancer.

(Options D and E) Use of contraceptive pill has been associated with only a slightly increased risk of breast cancer after 5 years of use.
Changing the contraception method or reducing the dose of estrogen is not likely to significantly decrease such risk.

TOPIC REVIEW

Many risk factors for breast cancer have been identified, including genetic, environmental, and lifestyle factors. Of these risk factors some are
modifiable while others not. The physician should take a detailed and through family and personal history to estimate the patient's risk of
breast cancer. After risk assessment and establishment, physician and patient can discuss the best preventive, screening and management
plan involving measures such as addressing modifiable risk factors or genetic counseling. Screening is important, because early detection
may lead to better outcomes.

The most important risk factors for development of breast cancer include the following:

Factors associated with the highest risk for development of breast cancer (relative risk [RR] >4.0) are as follows:

Advanced age (65 years and older)


Atypical hyperplasia of breast (biopsy proven)
Certain inherited genetic mutations ( BRCA1, BRCA2, TP53, ATM, CDH1); RR 4-8
Ductal or lobular carcinoma in situ (DCIS/LCIS); RR 8-10
Family history of early ovarian cancer (age <50 years)
Multiple first-degree relatives with breast cancer
Ionizing radiation exposure before age 30 (RR 22-40)
Personal history of early breast cancer (age <40)

Factors associated with RR 2.1-4.0 for breast cancer are as follows:

High endogenous estrogen or testosterone level (postmenopausal)


First full-term pregnancy after age 35 years
150 of 1943
Very dense breasts (>50%, compared with 11-25% mammographically)
One first-degree relative with breast cancer
Proliferative breast diseases (eg, atypical ductal hyperplasia)
Certain inherited genetic mutations (eg, CHEK2, PTEN)

Factors associated with RR 1.1-2.0 for breast cancer are as follows :

Alcohol consumption
Age 30-35 at first full-term pregnancy
Diethylstilbestrol exposure in utero
Early menarche (age <12 years)
Height (>5 feet 3 inches) [6]
High socioeconomic class
Ashkenazi Jewish heritage
Personal history of breast cancer (age of onset >40)
Dense breasts (25-50%, compared with 11-25% mammographically)
Benign breast conditions:Non-atypical ductal hyperplasia, fibroadenoma, sclerosing adenosis, microglandular adenosis,
papillomatosis, radial scar
Never breastfed a child
Nulliparity (no full-term pregnancies)
Late menopause (age >55)
Type II diabetes mellitus
Obesity (post-menopausal)
Personal history of uterine, ovarian, or colon cancer
Recent and long-term use of hormone replacement therapy (HRT) containing estrogen and progestin
Recent oral contraceptive use
Occupation: night shift
Tobacco abuse
Sedentary lifestyle
Inferior cardiovascular health
High bone mineral density

Factors that reduce risk of breast cancer (RR <1) include the following:

Asian, Hispanic, or Pacific islander race


Breastfeeding
Age <20 at first pregnancy
Tamoxifen use
Prior risk-reduction breast surgery
History of cervical cancer
History of oophorectomy
Exercise/active lifestyle
Low bone mineral density

References

• PubMed - Obesity is a Major Risk Factor for breast Cancer

Last updated:
Time spent: QID:1278
2023-2-12

151 of 1943
Three months ago you started Mary, aged 73, on residronate 35mg weekly, after she was diagnosed with osteoporosis confirmed with bone
mineral density (BMD). Her presenting symptom at that time was back pain. Examination revealed height decrease and mild kyphosis, as well
as tenderness over thoracic vertebrae. Despite being on residronate, not only did her symptoms persist, she also developed pain over new
areas of her thoracic spine. Investigations have established new osteoporotic fractures. Which one of the following would be the most
appropriate management option for her?

A. Continue residronate at the same dose.

B. Switch to alendronate.

C. Increase the dose of residronate.

D. Switch to zoledronic acid.

E. Switch to teriparatide.

Incorrect. Correct answer is E


45% answered correctly

Explanation:

Correct Answer Is E

Bisphosphonates are currently the most commonly anti-resorptive agents prescribed as first-line treatment for most of osteoporotic patients.
Of this drug family, alendronate (10mg/day or 70mg/week, orally), residronate (5mg/day or 35mg/week, orally) and zoledronic acid are
available in Australia.

Bisphosphonates should be used for at least 12 months before their efficacy on treatment of osteoporosis is assessed. However, in cases
where the patient suffers two or more minimal trauma fractures despite being on sufficient doses of an anti-resorptive drug, e.g.
bisphosphonates, commencement of teriparatide is justified as the most appropriate option.

Teriparatide is the synthetic parathyroid hormone that predominantly acts by increasing the osteoblasts (bone-forming cells) and by inducing
new osteoblasts formation. This drug is costly and at least 18 months of continuous use is needed effectiveness. Based on these, this
medication is only reimbursed by the PBS for patients with severe osteoporosis and very high risk of fractures who have:

A BMD T-score of =<-3

OR

had two or more minimal trauma fractures

OR

experienced at least one symptomatic new fracture after at least 12-months of continuous therapy with an anti-resorptive agent at
adequate doses.

(Option A) While Mary has developed new osteoporotic fractures due to severe osteoporosis, continuation of the same agent is not a wise
management.

Switching to other bisphosphonates such as alendronate (option B) or zoledronic acid (option D) does not add any benefit, as all members of
this drug family have almost the same effectiveness.

Mary is already on the recommended weekly dose for residronate and increasing the dose of residronate (option C) does not seem to benefit
her.

References

• NPS - Teriparatide (Forteo) for severe osteoporosis

• Healthy Bones Australia

Last updated:
Time spent: QID:1254
2023-2-12

152 of 1943
Jane, a 65-year-old patient of yours, is being assessed for osteoporosis. A Dual Energy X-ray Absorptiometry has revealed T-scores of -2.5 and
-2.7 for the femoral neck and the vertebral column, respectively. She was diagnosed with the cancer of her right breast 6 years ago for which
she underwent right mastectomy, chemotherapy and radiation therapy. In addition to advice regarding calcium and vitamin D, which one of the
following medications would be the best option for treatment of her osteoporosis?

A. Raloxifene.

B. Alendronate.

C. Teriparatide.

D. Strontium ranelate.

E. Hormone replacement therapy (HRT).

Incorrect. Correct answer is B


45% answered correctly

Explanation:

Correct Answer Is B

According to WHO criteria and based on Jane’s T-scores at vertebra and femoral neck, she is suffering from osteoporosis. Currently,
bisphosphonates are the first-line treatment option for primary and secondary treatment of vertebral and non-vertebral fractures due to
osteoporosis. Alendronate, residronate and zoledronic acid are the available bisphosphonates in Australia .

(Option A) Raloxifene is a selective estrogen receptor modulator (SERM) reimbursed by the Pharmaceutical Benefit Scheme (PBS) for
treatment of postmenopausal osteoporosis. SERMs also have a preventive effect on breast cancer; therefore, an appropriate option for post-
menopausal women with family or personal history of breast cancer in whom vertebral fracture due osteoporosis is a concern. Although there
is excellent evidence (Grade A) that raloxifene reduces the risk of vertebral fractures, limited evidence support its effect on non-vertebral
fractures. Jane is osteoporotic in femur as well; hence raloxifene would not an appropriate first-line option for her.

(Option C) Teriparatide is the synthetic parathyroid hormone that predominantly acts by increasing the osteoblasts (bone-forming cells) and
by inducing new osteoblasts formation. This drug is costly and at least 18 months of continuous use is needed for effectiveness. Based on
these, this medication is only reimbursed by the PBS for patients with severe osteoporosis and very high risk of fractures who have:

A BMD T-score of =<-3

OR

had two or more minimal trauma fractures

OR

experienced at least one symptomativ fracture after at least 12-months of continuous therapy with an anti-resorptive agent
in adequate doses.

Jane does not any of the above criteria to meet the need for teriparatide.

NOTE – BMD starts declining approximately 12 months after cessation of therapy with teriparatide; hence, continuation of therapy with an
anti-resorptive, generally a bisphosphonates.

(Option D) Strontium ranelate is an effective second-line option for reducing the risk of further osteoporotic fractures in postmenopausal
women with prevalent fractures. This agent should not be used in patients with previous or clinically active cardiovascular disease or
uncontrolled hypertension and should only be used when other drug classes for treatment of osteoporosis are not appropriate. Strontium
ranelate is prescribed at a dose of 2 grams per day.

(Option E) Breast cancer is estrogen dependent; therefore, prescribing estrogen for patients with a personal or strong family history of breast
cancer is not an appropriate option. Estrogen is an option (available on PBS) for the prevention and treatment of osteoporosis in women who
are near or at menopause, in particular for those patients suffering from menopausal vasomotor symptoms, e.g. hot flushes. When
considering this therapy, potential adverse effects i.e. increased risk of thromboembolism, cardiovascular diseases and breast cancer should
be weighed and discussed with the patient.

References

• https://www.osteoporosis.org.au/health-professional-resources

153 of 1943
• https://www.racgp.org.au/osteoporosis
Last updated:
Time spent: QID:1253
2023-2-12

154 of 1943
A 22-year-old man is brought to the emergency department after he collapsed during surfing. On examination, he is tanned, has a blood
pressure of 80/60 mmHg and a pulse rate of 100 bpm. You draw blood and start him on intravenous fluids. A bolus of 500 ml normal saline is
given, and since there is no response, you give him another 2 liters of normal saline in 2 hours; however, his blood pressure remains low. His
laboratory results are back and are remarkable for a serum sodium level of 124 mmol/L and potassium of 6 mmol/L (3.5-5.5). Which one of
the following is the most appropriate next step in management?

A. Give him more fluids.

B. Transfuse packed cells.

C. Give him corticosteroids.

D. Start him on dopamine.

E. Give him a bolus of atropine.

Incorrect. Correct answer is C


45% answered correctly

Explanation:

Correct Answer Is C

The scenario describes a patient with persistent hypotension despite adequate volume replenishment and without any evident source of fluid
or blood loss. However, in the light of the laboratory results, another possibility finds its way into the differential diagnoses. This patient has
hyponatremia and hyperkalemia making adrenal insufficiency a likely possibility diagnosis. Aldosterone, secreted by adrenal glands, maintains
intravascular volume via reabsorption of sodium and water from renal tubules while increasing potassium and hydrogen ions excretion at the
same time. Aldosterone insufficiency results in sodium and water loss and hyperkalemia. Accumulation of hydrogen ions gives rise to
metabolic acidosis.

This patient has also hyperpigmentation than can either be due to tanning caused by surfing in the sun or adrenal insufficiency. If the latter is
the case, hyperpigmentation is a support for the diagnosis, and also indicates that the adrenal insufficiency is primary (Addison disease).
Hyperpigmentation is a finding in nearly all patients with primary adrenal insufficiency, and is the most characteristic physical finding.
Physiologic response to cortical adrenal insufficiency causes an increase in production of ACTH with melanocyte stimulating hormone (MSH)
as a co-product. The elevated MSH results in increased melanin synthesis and causes hyperpigmentation.

The resulting brown hyperpigmentation is generalized, but is most prominent in areas exposed to light (such as the face, neck, and backs of
hands) and areas exposed to chronic friction or pressure (such as the elbows, knees, spine, knuckles, waist, and shoulders (brassiere straps).
Pigmentation is also prominent in the palmar creases, where it escapes being worn away by friction, and in areas that are normally pigmented,
such as the areolae, axillae, perineum, and umbilicus.

Management of adrenal crisis is with parenteral administration of corticosteroids. Hydrocortisone is the drug of choice for patients with
known diagnosis of adrenal insufficiency, while dexamethaosne is the preferred medication for those without a previous diagnosis of adrenal
insufficiency, because it is not measured in serum cortisol assays.

(Options A and B) In patients with adrenal insufficiency, fluid replacement does not increase the blood pressure. Corticosteroids alone will
treat the hypotension without any further need for fluid replacement.

(Option D) Vasopressors such as dopamine or epinephrine are not useful for treatment of adrenal crisis because the vascular tone is already
normal. Vasoconstriction will not replenish the deficient volume.

(Option E) Atropine is an anticholinergic agent that counteracts parasympathetic effects such as bradycarida. Atropine will not treat
hypotension caused by a non-cardiovascular pathophysiology.

References

• UpToDate - Clinical manifestations of adrenal insufficiency in adults

• UpToDate - Treatment of adrenal insufficiency in adults

Last updated:
Time spent: QID:1179
2023-2-12

155 of 1943
A 40-year-old woman presents to you clinic with a painless lump in her right breast that she noticed three days ago. Which one of the
following is the next best step in management?

A. Fine needle aspiration (FNA).

B. Mammography.

C. Ultrasonography.

D. Core biopsy of the lump.

E. Complete excision of the lump.

Incorrect. Correct answer is B


45% answered correctly

Explanation:

Correct Answer Is B

In approach to a palpable mass, mammography is the initial diagnostic test to exclude occult abnormalities in the noninvolved tissue, and to
rule out carcinoma. If the patient is 30 years old or younger, ultrasonography (option C) would be the initial diagnostic modality, because the
dense breast tissue in this age group makes it difficult for mammogram to detect small lesions and can leads to false negative results.

Simple FNA (option A) is usually performed after mammography. If FNA is done before mammography or ultrasound, a 2-week interval should
be waited because small hematomas from the needle aspiration may cause false-positive results.

Although an ultrasound can distinguish a cyst from a solid mass, FNA is the preferred method when a mass is palpable because the
procedure is both diagnostic and therapeutic. FNA frequently establishes the etiology of a cyst, provides pain relief in a symptomatic cyst, and
results in a clinical breast exam free of interfering masses. This method also allows easy diagnosis in the office setting, which, in some cases,
relieves anxiety for the patient.

A complete excision of the lump (option E) is indicated if a malignant breast lesion is highly suspected.

References

• Cancer Council - The investigation of a new breast symptom

• Medscape - Primary Care Guide to Managing a Breast Mass: Step-by-Step Workup

Last updated:
Time spent: QID:152
2023-2-12

156 of 1943
A 55-year-old diabetic woman presents with a foot ulcer shown in the following photograph. On examination, she looks ill and has a fever of
38.6°C. Which one of the following is the most appropriate next step in management after wound debridment and taking swabs?

A. Oral amoxicillin – clavulanate.

B. Intravenous ticarcillin - clavulanate.

C. MRI of the foot.

D. Daily wet dressing.

E. Daily wound toilet with povidone iodine.

Incorrect. Correct answer is B


45% answered correctly

Explanation:

Correct Answer Is B

The photograph shows an ulcer on the fifth toe of the right foot with marked inflammation and cellulitis of the surrounding tissue. Moreover,
the patient has fever and tachycardia (systemic toxicity). In such cases, after debridement and taking swabs for cultures, antibiotics are of
significance importance as the next best step in management. The antibiotics of choice for patients with severe infection evident by systemic
toxicity is piperacillin + tazobactam or ticarcillin + clavulanate.

The following table outlines choice of antibiotics for different types of diabetic foot ulcers:

Wound Characteristics Antibiotic therapy


Not required unless the ulcer is
odorous. Treat as mild
infection.
Uninfected Wound does not have purulence or any manifestations of inflammation
Without odor, only daily wet
dressing.
Oral Amoxicillin + clavulanate or
Presence of 2≥ manifestations of inflammation e.g. purulence,
Mild erythema, pain, tenderness, warmth or induration BUT no Cephalexin PLUS metronidazole
infection cellulitis/erythema greater than 2cm AND no extension beyond or (if penicillin allergic)
superficial subcutaneous tissue AND no systemic illness
Ciprofloxacin +clindamycin
Dicloxacillin/ flucloxacillin
Infection as above PLUS cellulitis extending >2cm OR lymphangitic
Moderate streaking OR extension beyond superficial fascia, deep-tissue abscess,
infection gangrene OR extension to muscle, tendon, joint or bone; NO systemic Metronidazole must be added if the
involvement wounds discharges are odorous
(anaerobe activity)
Severe Infection as above AND systemic toxicity or metabolic instability e.g. Piperacillin + tazobactam or
infection fever, chills, tachycardia, hypotension, confusion, vomiting,
leukocytosis, sever hyperglycemia, or azotemia Ticarcillin + clavulanate or (if
penicillin allergic)
157 of 1943
Ciprofloxacin PLUS either
clindamycin or lincomycin

(Option A) Oral amoxicillin-clavulanate is used for mild infections or ulcers without infection but odorous discharge.

(Option C) MRI is indicated if osteomyelitis, as a complication of the ulcer, is suspected. It is not a priority at this stage. Even if was indicated,
it would not be a priority compared to debridement, taking swabs, and starting antibiotics.

(Option D) Daily wet dressing is used as adjunct therapy to antibiotics for wound care.

(Option E) Cytotoxic antiseptics such as povidone iodine delay wound healing and should not be used.

References

• Australiam Prescriber - Managing foot infections in patients with diabetes

• MJA - Australian Diabetes Foot Network: management of diabetes-related foot ulceration — a clinical update

• Therapeutic Guidelines – Antibiotics

• IWGDF Guidelines

Last updated:
Time spent: QID:1135
2023-2-12

158 of 1943
A 60-year-old diabetic woman, whose diabetes is well controlled on insulin, comes to your clinic with an ulcer on the dorsum of her right fifth toe which she noted 5 days ago. On
examination, a purulent odorless discharge is noted. There is marked inflammation of the fifth toe, extending to the lateral aspect of the foot and ankle. She has a blood pressure
of 110/85 mmHg, heart rate of 104 bpm, and temperautre of 38.7 C. Which one of the following is the most appropriate next step in management?

A. Intravenous ticarcillin + clavulanate.

B. Analgesics.

C. X-ray of the foot.

D. MRI of the foot.

E. Debridment of the ulcer.

Incorrect. Correct answer is E


45% answered correctly

Explanation:

Correct Answer Is E

In approach to diabetic-related foot ulcers (DRFU), the most important initial step in management is debridement of the ulcer. Debridement allows comprehensive evaluation of
the wound, wound bed, and actual wound size. Debridement can convert a chronic wound to an acute one with accelerated healing. It also removes local pressure off the wound.

After the wound is debrided, swabs are taken for bacteriological assessment and culture. This should be performed before antibiotics are started. In the next step, antibiotics
should be stared if indicated.

(Option A) The decision as to empirical antibiotic treatment is guided by the characteristics of the ulcer. The following table summarizes the types of diabetic ulcers, associated
clinical features and antibiotics of choice for antimicrobial treatment:

Wound Characteristics Antibiotic therapy


Not required unless the ulcer is odorous. Treat as mild
Uninfected
Wound does not have purulence or any manifestations of infection.
inflammation
Without odor, only daily wet dressing.
Oral Amoxicillin + clavulanate or
Presence of 2≥ manifestations of inflammation e.g. purulence,
erythema, pain, tenderness, warmth or induration BUT no
Mild infection Cephalexin PLUS metronidazole or (if penicillin allergic)
cellulitis/erythema greater than 2cm AND no extension beyond
superficial subcutaneous tissue AND no systemic illness
Ciprofloxacin +clindamycin
Infection as above PLUS cellulitis extending >2cm OR lymphangitic Dicloxacillin/ flucloxacillin
streaking OR extension beyond superficial fascia, deep-tissue
Moderate infection
abscess, gangrene OR extension to muscle, tendon, joint or bone; NO Metronidazole must be added if the wounds discharges are
systemic involvement odorous (anaerobe activity)
Piperacillin + tazobactam or
Infection as above AND systemic toxicity or metabolic instability e.g.
Severe infection fever, chills, tachycardia, hypotension, confusion, vomiting, Ticarcillin + clavulanate or (if penicillin allergic)
leukocytosis, sever hyperglycemia, or azotemia
Ciprofloxacin PLUS either clindamycin or lincomycin

This woman’s ulcer is severely infected evident by significant cellulitis and systemic symptoms of fever and tachycardia. She should be started on intravenous antibiotics, and
ticarcillin + clavulanate is an appropriate option after debridement and swabs.

(Option B) Analgesics can be given for pain control. It is not a priority before debridement, swabs and antibiotics.

(Option C) In the presence of an infection/inflammation, lateral, antero-posterior, and oblique X-ray views of both feet should be performed with the X-ray request specifying the
anatomical location of the ulcer and mentioning the possibility of underlying osteomyelitis, diabetic arthropathy, and gas formation. However, it can be performed after more
important steps (debridement, swabs, and antibiotics) have been taken.

(Option D) MRI is indicated if osteomyelitis is suspected based on the following:

Ulcers unresponsive to antibiotic therapy


Probing the wound results in bone being touched
Bone is visible in the base of the wound
X-ray suggest bone involvement

In this scenario, MRI is not a priority in management.

References

• Australian Prescriber - Managing foot infections in patients with diabetes

• MJA - Australian Diabetes Foot Network: management of diabetes-related foot ulceration — a clinical update

• Therapeutic Guidelines – Antibiotics

Last updated:
Time spent: QID:1134
2023-2-12

159 of 1943
A 62-year-old diabetic man presents with a foot ulcer shown in the following photograph. On examination, he is not febrile. Debridement is perfomred, swabs for cultures are
taken, and antibiotics started. Which one of the following is the most appropriate next step in management?

A. X-ray of the foot.

B. Blood culture.

C. MRI.

D. Daily wet dressing of the wound.

E. Ultrasonography.

Correct
45% answered correctly

Explanation:

Correct Answer Is A

A step-wise approach to management of diabetic foot ulcers includes:

1-Debridement

Management of diabetic foot ulcers starts with debridement as a very important initial step in management. It allows comprehensive evaluation of the wound, wound bed, and
actual wound size. Debridement may also convert a chronic wound to an acute one with accelerated healing. Reducing the local pressure on the wound is another benefit.
Debridement can be surgical or non-surgical. It must be noted, however, that in the presence of peripheral vascular disease wound debridement should be performed after
comprehensive assessment of the vasculature status. Debridement in an ischemic limb is associated with complications.

2-Wound swabs

If infection is evident or suspected, deep wound swabs or needle aspiration of the exudate should be taken for bacteriological analysis before starting antibiotic treatment. If an
ulcer can be probed down to bone, osteomyelitis is likely to be present (MRI is indicated).

It is usual to take a wound swab even in the absence of the clinical signs of infection or in an ulcer which is clean, does not probe to bone, is not producing large amounts of
exudate, and has granulation tissue. The swab should be taken as localized and as deep as possible. If a pathogen is found without any clinical signs of infection, a topical
bactericidal dressing, such as one containing nanocrystalline silver, may help clear the wound of bacteria.

3- Antibiotics

Antibiotics are indicated if there is minimal inflammation but odorous wound secretions, or in the presence of moderate to severe infection. Choices of antibiotics depend on the
wound characteristic and associated symptoms. The following table summarizes antibiotic therapy for different diabetic ulcers:

Wound Characteristics Antibiotic therapy


Not required unless the ulcer is odorous. Treat as mild
Uninfected
Wound does not have purulence or any manifestations of infection.
inflammation
Without odor, only daily wet dressing.
Oral Amoxicillin + clavulanate or
Presence of 2≥ manifestations of inflammation e.g. purulence,
erythema, pain, tenderness, warmth or induration BUT no
Mild infection Cephalexin PLUS metronidazole or (if penicillin allergic)
cellulitis/erythema greater than 2cm AND no extension beyond
superficial subcutaneous tissue AND no systemic illness
Ciprofloxacin +clindamycin
Infection as above PLUS cellulitis extending >2cm OR lymphangitic Dicloxacillin/ flucloxacillin
streaking OR extension beyond superficial fascia, deep-tissue
Moderate infection
abscess, gangrene OR extension to muscle, tendon, joint or bone; NO Metronidazole must be added if the wounds discharges are
systemic involvement odorous (anaerobe activity)
Piperacillin + tazobactam or
Infection as above AND systemic toxicity or metabolic instability e.g.
Severe infection fever, chills, tachycardia, hypotension, confusion, vomiting, Ticarcillin + clavulanate or (if penicillin allergic)
leukocytosis, sever hyperglycemia, or azotemia
Ciprofloxacin PLUS either clindamycin or lincomycin

4- Imaging

160 of 1943
When clinical signs of inflammation are evident, lateral, antero-posterior, and oblique X-ray views of both feet should be obtained, with the X-ray request specifying the anatomical
location of the ulcer and mentioning the possibility of underlying osteomyelitis, diabetic arthropathy and gas formation. Bone infection usually must be present for several weeks
before it is detectable on plain X-ray films, so serial X-rays at one to four weeks intervals may be necessary if clinical infection fails to resolve and the initial X-ray is clear.

MRI of the affected area may be useful for differentiating infectious from non-infectious inflammatory conditions. However, if MRI facilities are not available, a surgeon should be
able to decide regarding exploration and debridement of an ulcer based on clinical examination.

5- Other tests

Surgical biopsy of the affected area may be useful for collection of subsequent specimens for microbiological examination if deterioration occurs after empirical
antibiotic therapy is started.
Full blood count, ESR, electrolytes, blood glucose and HbA1c, renal and liver function tests are indicated. Weekly measurement of C-reactive protein titer during treatment
of a foot infection may help monitor the response to treatment.

In this patient, swabs have been taken and antibiotics started, and the most appropriate next step will be antero-posterior, lateral and oblique X-rays of both feet as mentioned
earlier.

(Option B) Blood cultures may be indicated if the patient had systemic manifestations such as fever and chills (absent in this patient).

(Option C) MRI is indicated if osteomyelitis is suspected. A probe reaching the bone, bone being visible through the ulcer, or inadequate response to antibiotics makes
osteomyelitis likely.

(Option D) Daily wet dressing of the wound is the treatment of choice for non-infected ulcers, and adjuvant treatment to antibiotics for infectious wounds.

(Option E) Ultrasound is not recommended for initial assessment of diabetic foot ulcers.

References

• Australian Prescriber - Managing foot infections in patients with diabetes

• MJA - Australian Diabetes Foot Network: management of diabetes-related foot ulceration — a clinical update

• Therapeutic Guidelines – Antibiotics

Last updated:
Time spent: QID:1133
2023-2-12

161 of 1943
A 65-year-old woman with long-standing history of type II diabetes mellitus (DM) presents with a right foot ulcer which she noticed 7 days ago. You admit her to the hospital for
treatment based on the diagnosis of a diabetes-related foot ulcer (DRFU). Wound debridment is carried out. The following day, examination reveals that the ulcer is 1 cm in
diameter and oozing an odorless clear fluid. Which one of the following is the most appropriate next step in management?

A. Oral amoxicillin-clavulanate + metronidazole.

B. Intravenous ticarcillin + metronidazole.

C. MRI of the foot.

D. Daily wet dressing of the wound and follow-up.

E. Wound toilet with povidone iodine.

Incorrect. Correct answer is D


45% answered correctly

Explanation:

Correct Answer Is D

Foot ulceration is a major cause of mortality and morbidity in diabetic patients. Management of every diabetic-related foot ulcer (DRFU) starts with debridement as an essential
component of wound bed preparation by removing non-viable tissue. This allows for comprehensive examination of the wound bed and assessment of actual wound size.
Additionally, debridement may convert a chronic wound to an acute one with accelerated healing. Reducing the local pressure on the wound is another benefit.

Debridement can be surgical or non-surgical. Surgical sharp debridement is indicated when there is deep necrotic tissue, gangrene, drainage of a collection, or deep infections.
Non-surgical debridement is generally required every 1-2 weeks to remove non-viable tissue, including the surrounding callous. Non-sharp debridement with enzymatic, autolytic,
or biological preparations can be used as an adjunct to sharp debridement and can be helpful for adherent slough, or when pain limits surgical sharp debridement.

NOTE – In the presence of peripheral vascular disease, debridement should be avoided and the patient referred for specialist vasculature assessment. The only exception to
this rule is the presence of an abscess, in which case debridement and drainage should be performed regardless of vascular status.

The rest of management relies upon whether the wound is infected or inflamed, and if so, the wound characteristics. The following table outlines further steps for wound
management based on the presence of infection/inflammation and its severity:

Wound Characteristics Antibiotic therapy


Not required unless the ulcer is odorous. Treat as mild
Uninfected
Wound does not have purulence or any manifestations of infection.
inflammation
Without odor, only daily wet dressing.
Oral Amoxicillin + clavulanate or
Presence of 2≥ manifestations of inflammation e.g. purulence,
erythema, pain, tenderness, warmth or induration BUT no
Mild infection Cephalexin PLUS metronidazole or (if penicillin allergic)
cellulitis/erythema greater than 2cm AND no extension beyond
superficial subcutaneous tissue AND no systemic illness
Ciprofloxacin +clindamycin
Infection as above PLUS cellulitis extending >2cm OR lymphangitic Dicloxacillin/ flucloxacillin
streaking OR extension beyond superficial fascia, deep-tissue
Moderate infection
abscess, gangrene OR extension to muscle, tendon, joint or bone; NO Metronidazole must be added if the wounds discharges are
systemic involvement odorous (anaerobe activity)
Piperacillin + tazobactam or
Infection as above AND systemic toxicity or metabolic instability e.g.
Severe infection fever, chills, tachycardia, hypotension, confusion, vomiting, Ticarcillin + clavulanate or (if penicillin allergic)
leukocytosis, sever hyperglycemia, or azotemia
Ciprofloxacin PLUS either clindamycin or lincomycin

This patient has a 1cm ulcer. There is no comment regarding surrounding inflammation or cellulitis to suggest infection. Moreover, the wound secretions are clear and non-
infective. Management of such wounds, after initial debridement, includes deep wound swabs (if not already taken) and wound dressing and daily check to assess the healing
process.

Although numerous topical medications and gels are promoted for ulcer care, relatively few have proved to be more efficacious than warm wet dressing.

NOTE – If the presence of peripheral vascular disease, dry wound dressing should be applied and the patient referred for specialist vascular assessment and care.

(Options A and B) Antibiotics are used when the wound is associated with infection/inflammation. Oral amoxicillin-clavulanate is indicated for ulcers with minimal infection or
those without signs of infection but with odorous secretions. Parenteral antibiotics are indicated when there is marked infection. For deep or widespread infections or those
associated with osteomyelitis, antibiotics such as piperacillin- tazobactam, ticarcillin-clavulanate, meropenem, ertapenem, carbapenem, moxifloxacin, or ciprofloxacin PLUS
metronidazole, or a third-generation cephalosporin PLUS metronidazole are used.

(Option C) MRI is indicated where osteomyelitis is suspected based on clinical or laboratory studies.

(Option E) Topical antiseptics, such as povidone iodine, are toxic to the healing wound.

References

• NPS - Managing foot infections in patients with diabetes

• MJA - Australian Diabetes Foot Network: management of diabetes-related foot ulceration — a clinical update

• Therapeutic Guidelines – Antibiotics

162 of 1943
Last updated:
Time spent: QID:1132
2023-2-12

163 of 1943
Which one of the following tests is most likely to differentiate hypercalcemia caused by hyperparathyroidism and cancer?

A. PTH.

B. Calcium level.

C. Phosphate.

D. Alkaline phosphatase.

E. Vitamin D level.

Correct
45% answered correctly

Explanation:

Correct Answer Is A

Primary hyperparathyroidism and malignancy are the most common causes of hypercalcemia, accounting for more than 90% of cases.
Primary hyperthyroidism is the more common of these two.

It is usually not difficult to differentiate between them. Malignancy is often evident clinically by the time it causes hypercalcemia, and patients
with hypercalcemia of malignancy have higher calcium concentrations and are more symptomatic from hypercalcemia compared to
individuals with primary hyperparathyroidism.

However, it may be difficult to differentiate the two problems clinically when the presentation is less typical. As an example, some patients
with occult malignancy may present with mild hypercalcemia. Alternatively, patients with hyperparathyroidism can occasionally have acute
onset of severe, symptomatic hypercalcemia (parathyroid crisis). In such cases, measurement of intact PTH usually distinguishes these two
conditions.

Intact PTH concentrations are generally undetectable or very low in hypercalcemia of malignancy and are elevated or high-normal in primary
hyperparathyroidism.

It is uncommon for patients with hypercalcemia of malignancy to have elevated PTH levels. A suppressed PTH level is often indicative of
malignancy. However, in rare cases there might be concomitant primary hyperparathyroidism or a PTH secreting tumor.

References

• http://www.uptodate.com/contents/primary-hyperpara

Last updated:
Time spent: QID:1072
2023-2-12

164 of 1943
A 30-year-old man presents to your practice with complaints of anxiety and palpitation. He mentions that he is always afraid that something
bad is going to happen soon. On examination, he has a blood pressure of 160/80mmHg and heart rate of 110bpm. His palms are wet. Which
one of the following is the most likely diagnosis?

A. Hyperthyroidism.

B. Panic disorder.

C. Pheochromocytoma.

D. Generalized anxiety disorder.

E. Hypothyroidism.

Correct
45% answered correctly

Explanation:

Correct Answer Is A

There are some points to take into consideration in this scenario. First is the feeling of ‘always afraid that something bad is going to
happen soon’. This subjective feeling also described as ‘impending doom’ often indicates the presence of anxiety and can be seen during a
panic attack, in patients with pheochromocytoma, and in those with hyperthyroidism and thyrotoxicosis. Impending doom is described by
patients as if something very bad is just about to happen but they do not know what it is, or as if the world is going to end, or like they are
going to die of a heart attack.

Some other conditions that can cause such sensation include:

Generalized anxiety disorder


Temporal lobe epilepsy
Excessive caffeine use
Sleep deprivation
Depression in adults
Agoraphobia
Thyroid storm
Hyperthyroidism
Hyperventilation
Hypopituitarism
Anaphylaxis

Increased perspiration and palpitation are features seen almost in every hypermetabolic and hypersympathetic state. Sweaty hands can be
seen in episodic attacks of pheochromocytoma, hyperthyroid states, anxiety, and during a panic attack.

Pheochromocytoma, hyperthyroidism and acute panic attack can present with tachycardia and hypertension. Unless the patient is
experiencing an acute panic attack right now, it does not seem to be the case. Elevated blood pressure on examination goes against
generalized anxiety disorder (option D) and panic disorder (option B), making these two less likely, but still possible. Sustained elevation of
blood pressure is not a typical feature of anxiety disorders.

In fact, of the options, pheochromocytoma and hyperthyroidism top the list of differentials. The most common presentation of
pheochromocytoma is with episodic headache, tachycardia, hypertension, anxiety and sweating. Although some patients with
pheochromocytoma appears to have sustained increased blood pressure, the absence of headache, which is one of the components of the
classic triad of the disease (headache, palpitation, and diaphoresis), makes pheochromocytoma (option C) less likely. Moreover, there is no
comment regarding episodic nature of the symptoms.

Having said these, hyperthyroidism would be the most likely diagnosis. Anxiety, the senses of something bad is going to happen, palpitations,
sweatiness and elevated systolic blood pressure are reasonably justified by such a diagnosis.

The most frequent symptoms of hyperthyroidism are nervousness (anxiety), heat intolerance, palpitations, and fatigue and weight loss.
Common signs on examination include agitation, sinus tachycardia, elevated systolic blood pressure, fine tremors and hyper-reflexia.

Hypothyroidism (option E) causes bradycaria and dry skin, and is not associated with anxiety and the sense of impending doom

References

• UpToDate - Overview of the clinical manifestations of hyperthyroidism in adults

165 of 1943
• Medscape - Hyperthyroidism and Thyrotoxicosis

• Medscape - Panic Disorder

• Medscape - Pheochromocytoma
Last updated:
Time spent: QID:1068
2023-2-12

166 of 1943
A 30-year-old man presents to your practice with complaints of anxiety and palpitation. He mentions that he is always afraid that something
bad is going to happen soon. On examination, he has a blood pressure of 160/80mmHg and heart rate of 110bpm. His palms are wet. Which
one of the following is the most likely diagnosis?

A. Hyperthyroidism.

B. Panic disorder.

C. Pheochromocytoma.

D. Generalized anxiety disorder.

E. Hypothyroidism.

Correct
45% answered correctly

Explanation:

Correct Answer Is A

There are some points to take into consideration in this scenario. First is the feeling of ‘always afraid that something bad is going to
happen soon’. This subjective feeling also described as ‘impending doom’ often indicates the presence of anxiety and can be seen during a
panic attack, in patients with pheochromocytoma, and in those with hyperthyroidism and thyrotoxicosis. Impending doom is described by
patients as if something very bad is just about to happen but they do not know what it is, or as if the world is going to end, or like they are
going to die of a heart attack.

Some other conditions that can cause such sensation include:

Generalized anxiety disorder


Temporal lobe epilepsy
Excessive caffeine use
Sleep deprivation
Depression in adults
Agoraphobia
Thyroid storm
Hyperthyroidism
Hyperventilation
Hypopituitarism
Anaphylaxis

Increased perspiration and palpitation are features seen almost in every hypermetabolic and hypersympathetic state. Sweaty hands can be
seen in episodic attacks of pheochromocytoma, hyperthyroid states, anxiety, and during a panic attack.

Pheochromocytoma, hyperthyroidism and acute panic attack can present with tachycardia and hypertension. Unless the patient is
experiencing an acute panic attack right now, it does not seem to be the case. Elevated blood pressure on examination goes against
generalized anxiety disorder (option D) and panic disorder (option B), making these two less likely, but still possible. Sustained elevation of
blood pressure is not a typical feature of anxiety disorders.

In fact, of the options, pheochromocytoma and hyperthyroidism top the list of differentials. The most common presentation of
pheochromocytoma is with episodic headache, tachycardia, hypertension, anxiety and sweating. Although some patients with
pheochromocytoma appears to have sustained increased blood pressure, the absence of headache, which is one of the components of the
classic triad of the disease (headache, palpitation, and diaphoresis), makes pheochromocytoma (option C) less likely. Moreover, there is no
comment regarding episodic nature of the symptoms.

Having said these, hyperthyroidism would be the most likely diagnosis. Anxiety, the senses of something bad is going to happen, palpitations,
sweatiness and elevated systolic blood pressure are reasonably justified by such a diagnosis.

The most frequent symptoms of hyperthyroidism are nervousness (anxiety), heat intolerance, palpitations, and fatigue and weight loss.
Common signs on examination include agitation, sinus tachycardia, elevated systolic blood pressure, fine tremors and hyper-reflexia.

Hypothyroidism (option E) causes bradycaria and dry skin, and is not associated with anxiety and the sense of impending doom

References

• UpToDate - Overview of the clinical manifestations of hyperthyroidism in adults

167 of 1943
• Medscape - Hyperthyroidism and Thyrotoxicosis

• Medscape - Panic Disorder

• Medscape - Pheochromocytoma
Last updated:
Time spent: QID:1068
2023-2-12

168 of 1943
A 64-year-old woman presents to your practice with complaints of palpitation and anxiousness. She has past medical history of hypertension
and ischemic heart disease. He also has hypothyroidism, for which she is currently on levothyroxine 150 mcg, daily. Based on the history and
clinical findings, you suspect hyperthyroidism caused by overtreatment of her hypothyroidism. A serum thyroid stimulation hormone (TSH)
confirms the diagnosis. You decrease the dose of levothyroxine to 100 mcg, daily. When you should perform a follow-up TSH level again?

A. After one day.

B. After one week.

C. After two weeks.

D. After four weeks.

E. After six months.

Incorrect. Correct answer is D


45% answered correctly

Explanation:

Correct Answer Is D

Thyrotoxicosis is common in the Australian community and is frequently encountered in general practice.

The most frequent symptoms of thyrotoxicosis are nervousness, heat intolerance, palpitations, fatigue and weight loss despite increased
appetite (weight gain occurs in 10% of patients). Common signs on examination include agitation, sinus tachycardia, fine tremor and hyper-
reflexia. Elderly patients often present with nonspecific symptoms. Up to 20% of the elderly patients have atrial fibrillation (AF).

Most presentations of thyrotoxicosis are due to Graves disease, toxic multinodular goiter, toxic adenoma and thyroiditis. Graves disease is by
far the most common cause of thyrotoxicosis.

Exogenous thyroid hormone is another cause of thyrotoxicosis. Exposure to excess thyroid hormone usually occurs with overtreatment of
hypothyroidism and intentional ingestion in factitious disorder.

Overtreatment with thyroid hormone should always be suspected when a patient, who is on with thyroxin replacement therapy for
hypothyroidism, presents with signs and symptoms of hyperthyroidism.

This patient has presented with the history and clinical findings consistent with hyperthyroidism and thyrotoxicosis. The diagnosis is
established with presence of depressed TSH levels. As the most appropriate option, thyroxin should be discontinued or the dose reduced.
When therapy is stopped, serum T4 concentration falls about 50% in 7 days.

Response to treatment should be monitored at 4-to 6-week intervals, and dose adjustment performed until the patient is euthyroid again.

NOTE - After starting the treatment of hypothyroidism, the first follow-up with TSH should be performed in 6-8 weeks. This is different
from the above where followup after dose reduction is required.

References

• RACGP - Evaluating and managing patients with thyrotoxicosis

• UpToDate - Exogenous hyperthyroidism

Last updated:
Time spent: QID:1041
2023-2-12

169 of 1943
A 29-year-old woman presents to your GP clinic with complaints of mood swings, palpitation, and fine tremors for the past few weeks.
Laboratory tests are ordered and the results are remarkable for a decreased TSH level and normal T3 and T4. Which one of the following is
most likely to be the cause of this presentation?

A. Grave’s disease.

B. Pregnancy.

C. A toxic thyroid nodule.

D. Hyperthyroidism.

E. Hypothyroidism.

Correct
45% answered correctly

Explanation:

Correct Answer Is A

The constellation of symptoms is suggestive of hyperthyroidism. The laboratory findings consistent with the diagnosis are expected to be a
subnormal TSH with elevated serum T4, T3, or both. This patient has a subnormal TSH level in the presence of normal T3 and T4 values – an
entity termed subclinical hyperthyroidism.

It should be noted that subclinical versus overt hyperthyroidism are only biochemical definitions because hyperthyroid symptoms may be
present in those with subclinical hyperthyroidism, and absent in those with overt hyperthyroidism. In other words any patient with suboptimal
TSH and symptoms of thyrotoxicosis such as palpitations, sweating, increased appetite, tremors, etc. has hyperthyroidism despite of normal
T4 and T3.

The cause of subclinical hyperthyroidism are the same as the causes of overt hyperthyroidism with Graves disease being the most common
cause for both.

(Option C) A toxic nodule is another cause but is less common than Graves disease.

(Option B) During pregnancy, production of the thyroid hormones increases by approximately 50%. As a result TSH level during pregnancy is
lower than the normal non-pregnant level, but still within the normal range. The increase in T4 and T3 are due to an increase in thyroid binding
globulin (TBG). Free T4 and T3 remain unchanged.

(Option D) Hyperthyroidism is a general term describing a set of symptoms associate with excess thyroid hormones production that can be
caused by a variety of diseases, of those Grave's disease is the most common one.

(Option E) Hypothyroidism presents with elevated TSH and decreased T3 and T4. Subclinical hypothyroidism is defined as elevated TSH
beyond the normal limit but normal values of thyroid hormones.

References

• UpToDate - Subclinical hyperthyroidism

• RACGP (AFP) - Evaluating and managing patients with thyrotoxicosis

Last updated:
Time spent: QID:1032
2023-2-12

170 of 1943
An 83-year-old man fell off a slippery toilet bowl in the bathroom while trying to reach toilet paper, and had his left femoral neck fractured. He
was treated with open reduction and internal fixation. Today, he is being discharged. Which one of the following pieces of advice is most
important to give?

A. He should start alendronate.

B. He should have a bone scan.

C. He should take low molecular weight heparin (LMWH) for 6 weeks.

D. He should take warfarin for 6 months.

E. He should take supplemental calcium and vitamin D.

Correct
45% answered correctly

Explanation:

Correct Answer Is A

Femoral neck fracture following falling off a short height is in fact a fracture following a minor trauma. Minor (low) trauma fractures are
always pathologic. Osteoporosis is the most common cause of pathologic fractures. Unlike in women, osteoporosis in man is often
secondary and a manifestation of an underlying cause. In men with clinical manifestations of osteoporosis e.g., low trauma fracture or with
low bone mass (T-score less than -2.0), an initial evaluation for secondary causes of osteoporosis should be performed. Conditions to
consider are renal or liver disease, hyperparathyroidism, Cushing's syndrome, celiac disease, and other forms of malabsorption, or
hypercalciuria.

The initial evaluation should include routine biochemical tests to for renal or hepatic disease, a full blood count (FBC), serum testosterone,
calcium, alkaline phosphatase, 25-hydroxyvitamin D, and 24-hour urine calcium.

I addition to assessment for the underlying cause and treating them, if possible, patients with pathological fractures due to osteoporosis,
should be started on osteoporosis-specific treatment for prevention and treatment of osteoporosis.

Bisphosphonates are (e.g., alendronate, residronate, zoledronic acid) are first-line treatment options for this purpose and the best advice to
give for prevention of further fractures and bone density enhancement.

(Option B) Although it is recommended that bone mineral density scan be measured, treatment can be started without a bone scan.

(Option C) Patients undergoing hip fracture surgery are in the highest risk category for development of venous thromboembolism (VTE),
based on the procedure itself. Without thromboprophylaxis, reported rates of VTE are high following hip surgery. This necessitates
thromboprophylaxis following surgery for all patients admitted to hospital with hip fracture. The recommended regimen is low molecular
weight heparin (LMWH) (first-line) or fondaparinux for up to 35 days starting 12 hours after the surgery. Although, thromboprophylaxis is more
vital than alendronate, it has already been started post-operatively and continued during hospital stay, which is often 10-14 days. Six weeks of
anticoagulation with LMWH is beyond the maximum duration recommended and not a correct option.

(Option D) Warfarin may be less effective than LMWH, and it has the drawback of the need for INR monitoring; however, patients who wish to
avoid the discomfort of a twice daily injection may be more compliant with warfarin. Six months, however, is way beyond the recommended
duration of prophylaxis.

(Option E) Calcium and vitamin D supplementation should be advised if dietary calcium and vitamin D are inadequate. Supplementation with
calcium and vitamin D has not shown to reduce the risk of further fractures in the future.

References

• MJA - Evidence-based guidelines for the management of hip fractures in older persons: an update

• RACGP - Detection, prevention and treatment of osteoporosis

Last updated:
Time spent: QID:1012
2023-2-12

171 of 1943
Daniel, a 78-year-old patient of yours, presents for consultation. He has a fall last month and had his femoral neck fractured. He underwent
surgery and intramedullary nail placement. The fracture occurred when the chair leg broke and he fell off onto the ground from a height of
approximately 70cm. In laboratory assessment, significant findings are a testosterone level of 8ng/dL (normal:8-50ng/dL) and vitamin D of
12mmol/L (≥75mmol/L). Serum calcium is normal, as are other laboratory parameters including liver and renal function tests. Which one of
the following is the most important management to consider as the next step?

A. Alendronate 70mg, once weekly.

B. Calcium 1000mg, daily plus vitamin D 600IU, daily.

C. Calcitriol 0.25mg, daily.

D. Testosterone replacement therapy.

E. Adequate sun exposure.

Incorrect. Correct answer is C


45% answered correctly

Explanation:

Correct Answer Is C

The femoral neck fracture caused by falling off a short chair, as described, is in fact a fracture following a minor (low) trauma. This is always
pathologic. Osteoporosis is the most common cause of pathologic fractures. Unlike in women, osteoporosis in men is almost always
secondary and a manifestation of an underlying disease. Therefore, in men with clinical manifestations of osteoporosis e.g. low trauma
fracture or with low bone mass by DEXA (T-score less than -2.0), an initial evaluation for secondary causes of osteoporosis should be
performed.

Conditions to consider are renal or liver disease, hyperparathyroidism, Cushing's syndrome, celiac disease and other forms of malabsorption,
or hypercalciuria.

The initial evaluation should include routine biochemical tests for renal or hepatic disease, a full blood count (FBC), serum testosterone,
calcium, alkaline phosphatase, 25-hydroxyvitamin D , and 24-hour urine cortisol.

Based on the test results, Daniel is suffering from severe vitamin D deficiency and vitamin D should be replenished as the most appropriate
next step in management. The treatment option of choice is calcitriol 0.25mg, daily.

(Option A) Bisphosphonates are medications of choice for treatment of osteoporosis and prevention of further fractures in patients with
osteoporosis, whose osteoporosis is not due to hypogonadism (low testosterone levels), or those with hypogonadism in whom testosterone
replacement therapy is contraindicated. Indications are a T-score or less than -2.5 or history of pathologic fracture regardless of the T-
score. The two most widely used drugs are alendronate and residronate. Alendronate should be given 70 mg, weekly (or 10mg, daily). It is,
however, necessary that vitamin D be corrected to a level of above 50nmol/L before therapy with bisphosphonates is started.

(Option B)This patient has normal serum calcium levels and does not need calcium supplementation now. Moreover, calcium should not be
added to calcitriol therapy due to the risk of hypercalcemia. Dietary calcium intake should be maintained at 800 to 1000 mg, daily.

(Option D) Testosterone replacement therapy is the treatment of choice in osteoporotic men with hypogonadism. Daniel’s testosterone level is
in the lower limit normal range. Judged by his testosterone level, he is unlikely to have hypogonadism.

(Option E) Adequate sun exposure should be recommended to all patients as a means of preventing vitamin D deficiency. Daniel has severe
vitamin D deficiency and needs to be treated more aggressively rather than just with sun exposure.

References

• http://www.racgp.org.au/afp/2012/march/osteoporosi

• http://www.racgp.org.au/download/documents/Guideli

• http://www.uptodate.com/contents/treatment-of-oste

• http://www.uptodate.com/contents/clinical-manifest

Last updated:
Time spent: QID:1011
2023-2-12

172 of 1943
A previously healthy and fit 35-year-old man presents with complaints of recent-onset palpitations and feeling edgy. On examination, he looks
anxious. His blood pressure is 160/80mmHg and heart rate 102 bpm. The rest of the exam is inconclusive. Which one of the following could
be the most likely diagnosis?

A. Pheochromocytoma.

B. Panic attack.

C. Anxiety.

D. Hyperthyroidism.

E. Carcinoid syndrome.

Incorrect. Correct answer is D


45% answered correctly

Explanation:

Correct Answer Is D

Pheochromocytoma, panic attack, anxiety and hyperthyroidism can all present with palpitation, anxiousness and nervousness, and
palpitation.

The classic history of a patient with a pheochromocytoma (option A) includes spells of headache, palpitation, and diaphoresis,
and hypertension. These four characteristics together are strongly suggestive of a pheochromocytoma. The spells of pheochromocytoma
may vary in occurrence from monthly to several times per day, and the duration may vary from seconds to hours. Typically, they worsen with
time, occurring more frequently and becoming more severe as the tumor grows. There is no comment regarding episodic nature of the
symptoms in the history. Furthermore, the patient mentions no headache or excessive sweating. Considering these facts, pheochromocytoma
is less likely to be the diagnosis.

Panic attack (option B) can also present with palpitations and pounding heart, anxiousness, sweating, trembling or shaking, sensation of
shortness of breath, choking or smothering, chest pain or discomfort, nausea and abdominal discomfort, and dizziness. Patients with panic
disorder have recurrent episodes of panic, with the fear of recurrences resulting in significant behavioral changes and worry about the
consequences of the attack. They may feel they are going crazy, losing control, or dying. Feeling of impending doom (somatic concerns of
death from cardiac or respiratory problems) may be a major concern of patients during an attack. Patients may seek emergency care in the
emergency department. There are no signs specific for panic disorder. If the patient presents in an acute state of panic, hypertension,
tachycardia, mild tachypnea, mild tremors, and cool, clammy skin might be noted. Panic attack normally lasts 20 to 30 minutes but rarely in
can continue for 1 hour.

Again, there is no comment regarding episodic nature of symptoms as expected in panic disorder. Also, the patient does not appear to have
been experiencing an acute attack while being examined to justify the elevated blood pressure.

Patients with anxiety disorders (option C) often experience anxiousness and excess worry. They also experience bouts of palpitation and hear
pounding, but hypertension is not often an associated feature.

Carcinoid syndrome (option E) most commonly presents with diarrhea, flushing and wheezing which are absent here. In fact carcinoid
syndrome is the least likely diagnosis among others.

With the given constellation of symptoms, hyperthyroidism is the best fit for this scenario and the most likely diagnosis among others.

The most frequent symptoms of hyperthyroidism are nervousness, heat intolerance, palpitations, and fatigue and weight loss. Common signs
on examination include agitation, sinus tachycardia, elevated systolic blood pressure, fine tremor and hyper-reflexia. Elderly patients often
present with nonspecific symptoms. Up to 20% of the elderly patients have atrial fibrillation (AF).

References

• UpToDate - Overview of the clinical manifestations of hyperthyroidism in adults

• Medscape - Hyperthyroidism and Thyrotoxicosis

• Medscape - Panic Disorder

• Medscape - Pheochromocytoma

Last updated:
Time spent: QID:1067
2023-2-12

173 of 1943
A 37-year-old woman, who underwent thyroidectomy 4 days ago, presents with complaints of tingling of her hands and feet and around her
mouth. Her total calcium level was 2.02 (normal 2.1- 2.6 mmol/L) on discharge. Which one of the following is the best immediate
management of this patient?

A. Intravenous calcium.

B. Calcium gluconate, orally.

C. Calcium gluconate and vitamin D, orally.

D. Diazepam.

E. Calcitriol.

Correct
45% answered correctly

Explanation:

Correct Answer Is A

Every patient with symptomatic hypocalcemia (paresthesia of hands, feet and perioral area are sensory equivalents of tetany) with any
calcium concentration below the reference and those with asymptomatic hypocalcemia, whose total calcium levels have acutely fallen below
1.9 mmol/L are recommended to urgently have calcium replaced intravenously.

Ten to 20ml of 10% calcium gluconate in 50-100 ml of Dextrose 5% should be given intravenously over 10-20 minutes with ECG monitoring.
This can be repeated until the patient is asymptomatic. It should be followed with a calcium gluconate infusion. Calcium chloride can be used
as an alternative to calcium gluconate, but it is more irritating to veins and should only be given via a central line.

If the case was asymptomatic hypocalcemia (with calcium above 1.9 mmol/L) or latent tetany (represented by positive Trousseau and/or
Chvostek sign) oral calcium with or without vitamin D supplementation was the choice.

References

• http://www.endocrinology.org/policy/docs/13-02_Eme

• http://www.ncbi.nlm.nih.gov/books/NBK6967/

• http://ceaccp.oxfordjournals.org/content/7/2/55.fu

• http://emedicine.medscape.com/article/852184-overv

• http://www.merckmanuals.com/professional/endocrine

• Davidson’s Principles & Practice of Medicine – Churchill/ Livingstone– 20th edition – pages 774-775

• Therapeutic Guidelines – Endocrinology; available on http://tg.org.au

Last updated:
Time spent: QID:98
2023-2-12

174 of 1943
A 55-year-old woman presents to your practice after noticing a clear nipple discharge from her right breast. On physical examination, no
palpable mass is found in her breasts. Both nipples appear normal, however, pressure over the right areola causes discharge of clear fluid
from the nipple. Which one of the following could be the most likely diagnosis?

A. Benign ductal papilloma.

B. Paget disease of the breast.

C. Ductal ectasia.

D. Intraductal breast carcinoma.

E. Breast fibroadenoma.

Correct
45% answered correctly

Explanation:

Correct Answer Is A

Nipple discharge is always an abnormal finding except in late pregnancy or the postpartum period. Based on characteristics, there are seven
types of nipple discharge:

1. Milky: white discharge; sometimes fat globules are seen under microscopy
2. Multicolored gummous: sticky discharge
3. Purulent: pus with white cells seen under microscopy
4. Watery: colorless discharge
5. Serous: faintly yellow, thin discharge
6. Serosanguineous: thin, clear discharge with pink tint, RBCs seen under microscopy
7. Bloody (sanguinous): pure blood

Conditions associated with each type of discharge are as follows:

Milky Discharge

Galactorrhea, or nonpuerperal lactation, usually results from multiple duct discharge from both breasts. The most common cause of
nonpuerperal lactation is hyperprolactinemia associated with pituitary adenomas, medications or other causes resulting in increased
production of prolactin (e.g. primary hypothyroidism). In many women, galactorrhea can be idiopathic.

Multicolored and Sticky Discharge

Duct ectasia or comedomastitis is the most common cause of a multicolored, sticky discharge that is commonly bilateral and usually in
perimenopausal woman. It begins as a dilation of the terminal ducts with an irritating lipid fluid collection and producing an inflammatory
reaction resulting in discharge from the nipple.

Duct ectasia is most frequently associated with pain, itching, and swelling in the nipple. Palpation of the areola can often reveal a tubular
mass, reflecting the dilated ducts. Often a history of nipple manipulation is elicited. If the disease progresses, a mass can develop (plasma
cell mastitis) that can mimic cancer. Surgery is indicated only if a mass forms or the discharge changes to serosanguinous or bloody.

Purulent Discharge

In patients with acute puerperal mastitis, chronic lactation mastitis, central breast abscesses, or plasma cell mastitis, nipple discharge is
purulent and usually unilateral. Breast cultures and smears may reveal a causative organism.

Abscess formation usually requires incision and drainage if appropriate antibiotics and warm compreses are not effective. It is important to
remove a portion of an abscess wall for histologic study to exclude an underlying cancer associated with secondary necrosis and infection.

Watery, serous, serosanguinous, and bloody Discharges

Intraductal papillomas are the most common cause of these discharges, but fibrocystic disease, advanced duct ectasia, breast cancer, and
vascular engorgement in near-term pregnancy can also be the causes.

In patients over 50, malignancy becomes increasingly common, especially if the discharge is unilateral and associated with a mass. Surgical
exploration is mandatory in this group of patients with this type of discharge, even if cytologic and mammographic findings are negative.

NOTE - surgical referral is required if the nipple discharge is spontaneous and unilateral, or the patient is older than 60 years.

175 of 1943
(Option B) Paget disease of the breast most commonly presents with a scaly, raw, vesicular, or ulcerated lesion that begins on the nipples and
then spreads to the areola. Nipple discharge, if present at all, tends to be bloody rather than clear.

(Option C) Ductal ectasia presents with multicolored and sticky discharge. Toothpaste like discharge is a classic description of discharge
associated with duct ectasia.

(Option D) With unilateral clear discharge in this woman, the most likely cause is a benign Intraductal papilloma. Although advanced duct
ectasia and intraductal carcinoma are among other etiologies, they are less likely to be the cause compared with intraductal papilloma.

(Option E) Fibroadenoma does not cause nipple discharge.

References

• http://www.ncbi.nlm.nih.gov/books/NBK284/

• https://canceraustralia.gov.au/sites/default/files

• RACGP - Management of Benign Breast Conditions - Part 3

Last updated:
Time spent: QID:995
2023-2-12

176 of 1943
A 30-year-old Australian woman presents to your practice complaining of fatigue, constipation, and weight gain for the past few months. She
has no prior history of neck surgery or radiation. On examination, she has hoarseness and her skin is dry. You order thyroid function tests
(TFT) that reveals an elevated serum thyroid stimulating hormone (TSH) and decreases T4. Which one of the following is the most likely
cause of this presentation?

A. Thyroid cancer.

B. Post-ablative hypothyroidism.

C. An autoimmune disease.

D. Pituitary hypofunctioning.

E. Iodine deficiency.

Incorrect. Correct answer is C


45% answered correctly

Explanation:

Correct Answer Is C

The clinical findings and the history suggest hypothyroidism as the most likely diagnosis. The diagnosis is confirmed by typical laboratory
findings of an elevated serum Thyroid Stimulating Hormone (TSH) and decreased T4.

Iodine deficiency remains the most common cause of hypothyroidism worldwide. However, in Australia and other iodine-replete countries,
autoimmune chronic lymphocytic thyroiditis (Hashimoto’s thyroiditis) is the most common etiology.

Primary hypothyroidism refers to hypothyroidism caused by the thyroid gland problems. Primary hypothyroidism accounts for almost 95% of
cases of hypothyroidism.

References

• UpToDate - Clinical manifestations of hypothyroidism

Last updated:
Time spent: QID:914
2023-2-12

177 of 1943
A 29-year-old woman presents to your office with concerns about osteoporosis. She is an elite athlete and has a body mass index (BMI) of 15
kg/m2. She has two daughters aged 2 and 4 years. You order a bone densitometry with DEXA scan, the result of which reveals a T-score of -3
and -1.7 for the femur and vertebrae, respectively. In addition to supplemental calcium, which one of the following should be given to her?

A. Alendronate.

B. Eestrogen replacement therapy.

C. Vitamin D.

D. Weight bearing exercise.

E. Anabolic agents.

Incorrect. Correct answer is C


45% answered correctly

Explanation:

Correct Answer Is C

The low BMI of this woman, as well as being a professional athlete makes anorexia nervosa (AN) a very likely diagnosis. If the diagnosis is
AN, it is most likely to be of restricting type, where the patients reduces weight through restricted calorie intake or excessive exercise.

Regardless of the diagnosis, one thing is certain: this woman is at increased risk of osteopenia/osteoporosis due to low body weight and
prolonged menopause and the DEXA scan proves that osteoporosis of the hip and osteopenia of the vertebrae have already occured.

Such patients should be approached similar to those who have osteopenia/osteoporosis as a complication of AN. All patients with AN should
be started on calcium (1200-1600mg, daily) and vitamin D. Calcium and vitamin D, will not restore the bone density, but they prevent further
bone loss. For this patient, vitamin D is the best option to give in combination with calcium.

(Option A) Alendronate and other bisphosphonates are first-line treatment for postmenopausal women with established osteoporosis. In
women of reproductive age, bisphosphonates should not be used routinely as they are teratogenic and may remain in the body for long
periods. Adverse effects of these drugs are not adequately studied in premenopausal women.

(Option B) Estrogen therapy comes second after weight gain, as the most effective treatment. This woman, however, has already two children,
making estrogen deficiency a less concerning feature.

Option D) Weight-bearing exercises have been shown equivocal results in bone mass restoration to date. Not to mention that this patient is
already an athlete, making this option less effective for her. Furthermore, excessive exercise can be associated with more probability of
traumatic fractures in a osteopenic/osteoporotic woman.

(Option E) Androgens, in any form, have shown no benefit in treatment of AN-related osteoporosis.

TOPIC REVIEW

Osteoporosis is one of the significant complications of anorexia nervosa (AN) seen in at least 30% of female patients. Osteoporosis is
associated with risk of fractures.

Osteoporosis in AN is characterized by increased bone resorption without concomitant increased bone formation. Trabecular bone, found in
the spine and hips, is affected more than cortical bone. The most severely affected site is the lumbar spine. Bone loss occurs at a rate of 4%
to 10% per year.

Multiple factors contribute to osteoporosis in AN. Peak bone mass is generally acquired during adolescence and young adulthood, which
frequently coincides with the onset of AN. The peak bone mass acquired by a patient with AN depends on the time of onset and duration of
the eating disorder, the degree of nutritional depletion, and changes in body composition.

The AN-associated osteoporosis is different from that found in the typical postmenopausal state. In AN, patients have low serum levels of
estrogen, androgens, IGF-1, and leptin. In addition, there are high levels of cortisol and normal serum levels of calcium, vitamin D, and
parathyroid hormone.

In all patients with AN longer than 6 months, a bone mineral density (BMD) should be measured with dual-energy X-ray absorptiometry
(DEXA). For patients who continue to suffer episodes of AN, a follow-up test should be done every 2 years.

178 of 1943
To date, weight gain and resumption of menstrual function have been shown to be the most effective treatment for osteoporosis in patients
with AN. There is limited evidence supporting the use of hormone therapy in the form of estrogen, recombinant human insulin-like growth
factor 1 (IGF-1), or both.

A systematic review identified five types of treatment that have been investigated:

Weight gain - Restoration of weight and menses is the cornerstone of treatment for AN, and is associated with improved bone mineral density
in patients with AN and osteoporosis. However, it is not clear if bone loss is fully reversible.

Hormone therapy - Supplemental estrogen in the form of oral contraceptive (estrogen plus progestin) or estrogen replacement therapy has a
secondary role in minimizing and perhaps reversing osteoporosis in patients with AN. There is evidence from multiple studies to support this
widespread practice. This is in contrast to the unequivocal benefits seen in postmenopausal osteoporosis.

Bisphosphonates - Bisphosphonates should not be used routinely in AN. Long-term side effects in young patients are unknown, and
bisphosphonates are teratogenic and can persist in the body for many years after discontinuation of treatment. However, Bisphosphonates
have been used for severe osteoporosis, especially when AN was unlikely to remit in near future. This is based on the known effectiveness of
bisphosphonates in decreasing bone resorption and increasing bone mineral density in osteopenic postmenopausal women and adolescent
patients with osteogenesis imperfecta.

Calcium - Calcium and vitamin D are not effective in restoring bone mineral content in patients with AN and osteoporosis. In addition, studies
have found no correlation between calcium intake and bone mineral density in AN, suggesting that adequate calcium and vitamin D do not
prevent osteoporosis. However, calcium is necessary for bone maintenance and restoration, and is considered the standard treatment for
patients with AN. The American Academy of Pediatrics and the Society for Adolescent Medicine both recommend that all adolescents ingest
1200-1500 mg/day of calcium and 800 IU of vitamin D to achieve peak bone mass.

Exercise - Generally, studies suggest very limited, if any, relationship between physical activity in patients with AN and their bone mineral
density. A few observational studies, however, have found that exercise exerts a protective effect upon bone mineral density in patients with
AN, consistent with the known effect that weight bearing exercise has in the general population.

Patients with osteoporosis should refrain from excessive exercise and activities that significantly increase the likelihood of fractures.

Other treatments such as anabolic medications, calcitonin, raloxifene has not been proved effective in treatment of AN-related osteoporosis.

Limited evidence suggests that recombinant human insulin-like growth factor 1 (IGF-1) is beneficial with greatest benefits in patients who
received IGF-1 plus estrogen and progestin.

References

• Medscape - A Review of Osteoporosis Management in Younger Premenopausal Women

• UpToDate - Anorexia nervosa in adults and adolescents: Medical complications and their management

Last updated:
Time spent: QID:937
2023-2-12

179 of 1943
Which one of the following is a feature more likely to be seen in hyperthyroidism?

A. Course tremors.

B. Pain in shoulders.

C. Narrow pulse pressure.

D. Delayed reflexes.

E. Weight gain.

Incorrect. Correct answer is B


45% answered correctly

Explanation:

Correct Answer Is B

Muscle weakness and proximal myopathy is a feature associated with hyperthyroidism and thyrotoxicosis. Another infrequent complication
seen in patients with hyperthyroidism is chronic periarthritis and calcific tendinitis. Both conditions tend to occur in the shoulder, causing
limitations of range of motion. The condition may lead to frozen shoulder (adhesive capsulitis).

NOTE – Unlike in hypothyroidism, joint pain is not as common in hyperthyroidism and if occurs is more related to periartiular structures
such as tendons and muscles. Joint pain is more common in hypothyroidism and is often due to hypometabolic state leading to join
effusion and swelling.

(Option A) Course tremors are not a feature of hyperthyroidism, but fine tremors are. Fine tremors are best noted when the patient stretches
out the fingers, and the examiner touches the finger tips with of palm of his/her hand.

(Option C) Hyperthyroidism is associated with widened pulse pressure. Narrow pulse pressure is seen in hypothyroidism.

(Option D) Delayed reflexes are a feature of hypothyroidism. Hyperthyroidism has hyper-reflexia as a feature.

(Option E) An elevated metabolic state may lead to weight loss, despite increased appetite. In about 10% of patients, weight gain can be seen.

References

• Murtagh’s General Practice – McGraw Hill – 5th Edition – page 214

• Nature - Hyperthyroidism is a Risk Factor for Developing Adhesive Capsulitis of the Shoulder: A Nationwide Longitudinal Population-Based
Study

Last updated:
Time spent: QID:838
2023-2-12

180 of 1943
Which one of the following clinical features is seen in both hypothyroidism and hyperthyroidism?

A. Premature grey hair.

B. Decreased libido.

C. Atrial fibrillation.

D. Cardiomegaly.

E. Hyperlipidemia.

Incorrect. Correct answer is B


45% answered correctly

Explanation:

Correct Answer Is B

The following table outlines the clinical features associated with hypothyroidism and hyperthyroidism:

Hyperthyroidism Hypothyroidism
Tachycardia bradycardia
Heart failure Cardiomegaly
Anxiety sleepiness
Irritability lethargy
Anxiety Husky voice
Heat intolerance Cold intolerance
Warm skin Cold skin
Premature grey hair
Decreased libido Decreased libido
Psychosis psychosis
Hyperlipidemia
Macrocytic anemia

Decrease libido is a presenting feature in both hyperthyroidism and hypothyroidism. Psychosis is another common feature.

References

• Murtagh’s General Practice – McGraw Hill – 5th Edition – page 214

Last updated:
Time spent: QID:837
2023-2-12

181 of 1943
Which one of the following is NOT correct regarding oral bisphosphonates?

A. They reduce bone loss.

B. They increase the mineral density.

C. They are firs-line treatment option for prevention of osteoporosis.

D. They cannot be continued more than 2 years.

E. They are first-line treatment for osteoporosis in postmenopausal women.

Incorrect. Correct answer is D


45% answered correctly

Explanation:

Correct Answer Is D

Oral bisphosphonates are first-line medications for both prevention (prescribed under certain circumstances) and treatment of osteoporosis
in post-menopausal women. They slow bone loss, improve bone mineral density and reduce fracture rates.

Oral bisphosphonates should be continued for at least 5 to 7 years, with review of the bone mineral density every 1 to 2 years.

References

• http://www.racgp.org.au/afp/2012/march/osteoporosi

• http://www.racgp.org.au/download/documents/Guideli

• Therapeutic Guidelines – Endocrinology; available from http://www.tg.org.au

Last updated:
Time spent: QID:773
2023-2-12

182 of 1943
A 65-year-old female presents with Colles fracture of the right wrist. Further workup shows that she is osteoporotic. She has past history of
breast cancer for which she was successfully treated with mastectomy and radiotherapy. Which one of the following would be the most
appropriate management of her osteoporosis after taking care of the fracture?

A. Bisphosphonates.

B. Raloxifene.

C. Tamoxifen.

D. Premarin.

E. Calcium supplements.

Correct
45% answered correctly

Explanation:

Correct Answer Is A

Currently, bisphosphonates are first-line treatment for osteoporosis in patients with or without history of breast cancer. Bisphosphonates do
not interact with estrogen receptors.

Although estrogen (premarin) (option D) has been used for prevention of osteoporosis in women, its use has been associated with increased
risk of breast cancer if used beyond recommended duration of therapy (7 years). Estrogen-containing regimens are inappropriate options for
patients with personal history of breast cancer.

Raloxifene (option B) is a selective estrogen receptor modulator (SERM) used for treatment of osteoporosis if bisphosphonates are
contraindicated. It has the benefits of estrogen effects on bones without increasing the risk of estrogen-dependent cancers i.e. breast and
endometrial cancers. As a matter of fact, raloxifene reduces the risk of breast cancer in high-risk postmenopausal women. This drug is often
used as the most appropriate option for treating younger postmenopausal women with only vertebral osteoporosis because it is not effective
for prevention of non-vertebral fractures.

Neither raloxifene, nor tamoxifen (option C) are used as first-line medications for treatment of osteoporosis.

References

• http://www.racgp.org.au/afp/2012/march/osteoporosi

• http://www.racgp.org.au/download/documents/Guideli

• Therapeutic Guidelines – Endocrinology; available on http://www.tg.org.au

Last updated:
Time spent: QID:836
2023-2-12

183 of 1943
A 55-year-old woman comes to your clinic because she is concerned about developing osteoporosis after menopause. Which one of the
following is correct regarding prevention of osteoporosis?

A. Bisphosphonates are recommended only for women and just for primary prevention of osteoporosis.

B. Strontium ranelate is recommended only for treatment of severe osteoporosis in postmenopausal women.

C. Bisphosphonates are recommended only for women and just for treatment of osteoporosis.

D. In women with minimal-trauma fractures, bisphosphonates are contraindicated.

E. Alendronate is the second-line therapy in the treatment of postmenopausal osteoporosis.

Incorrect. Correct answer is B


45% answered correctly

Explanation:

Correct Answer Is B

Bisphosphonates (e.g. alendronate, residronate, zoledronic acid) can be used in both men and women, not only for primary prevention of
osteoporosis, but also for treatment of established osteoporosis and prevention of more fractures. Bisphosphonates are always first-line
therapy unless contraindicated.

Raloxifene is used in post-menopausal women as the second-line therapy, where bisphosphonates are contraindicated. It can be used for
both prevention and treatment of osteoporosis.

Strontium ranelate is used to treat severe osteoporosis or those with minimal trauma fratures while on adequate dose of bisphosphonates. It
works by increasing bone formation and decreasing bone loss. Myocardial infarction is a worrisome adverse effect that limits the use of this
drug to severe cases of osteoporosis under strict care and surveillance.

References

• NPS - New drugs for osteoporosis

• RACGP - Detection, prevention and treatment of osteoporosis

• Therapeutic Guidelines – Endocrinology; available from: http://www.tg.org.au

Last updated:
Time spent: QID:772
2023-2-12

184 of 1943
A 62-year-old woman presents to your clinic for discussing treatment options after she sustained a wrist fracture while playing tennis a few
weeks ago. On DEXA scan she is found to have a T-score of -1.7. Which one of the following would be the most appropriate management?

A. Zoledronic acid.

B. Vitamin D supplements.

C. Advise that she should have more exercise.

D. Reassure that she does not need treatment at this stage.

E. Raloxifene.

Correct
45% answered correctly

Explanation:

Correct Answer Is A

Different guidelines and authors suggest various indications for commencement of medical therapy for osteoporosis. This sometimes leads
to confusion as to whether which patients are candidates for medical therapy. Literally, osteoporosis is defined as a T score equal to or less
than -2.5 on BDM.

Established osteoporosis is the preferred term for those with osteoporosis (a T-score less than -2.5 at the hip or spine), and one or more
fragility fractures. Recognized fractures for this definition include Colles’, hip, vertebral and low-trauma fractures at the ankle. (While various
other terms, such as ‘severe’, ‘advanced’ or ‘frank’ osteoporosis appears in reports, they have no specific meaning and should not be used).

A first fragility fracture doubles the risk of another fracture. With two previous fractures, there is 12-fold increase in the risk of consequent
fractures. Because of such risk, current guidelines recommend treatment of those who have had a fracture and a T-score is below -1.5. In
other words, the diagnosis of osteoporosis is not required to treat these individuals (a T-score≤-2.5 is not needed to start treatment)

All the following patients need medical treatment:

Those with a minimal trauma fracture (even if the T-score is not <-2.5 ; a T-score of <-1.5 and a fracture necessitate medical treatment)
Those 70 years old or older
Those currently on prolonged (at least 3 months) high dose corticosteroid treatment (at least 7.5 mg/day prednisolone or equivalent)
and with a T-score of –1.5 or lower.

For others, ensuring adequate dietary calcium and vitamin D intake (or supplementation if there is adequate dietary intake) and lifestyle
modification such as exercise, reducing alcohol consumption and stopping smoking.

All other patients need to be treated medically. Unless contraindicated, the first-line option for treatment of osteoporosis is always
bisphosphonates. Although alendronate was considered the drug of choice, recent recommendation favors the use of intravenous zoledronic
acid due to more compliance. Calcium and vitamin D should be given to all patients undergoing medical treatment regardless of the specific
agent.

If the patient is vitamin D deficient, correction of vitamin D level (25-hudroxy vitamin D>50 nmol/L) is the most important step in management
prior to commencement of treatment with bisphosphonates. The patient should receive vitamin D first if:

There is documented vitamin D deficiency based on serum 25-hydroxy vitamin levels


Housebound women
Nursing home residents
Women who are shrouded due to cultural reasons

References

• Australian Prescriber - New drugs for osteoporosis

• RACGP - AFP - Osteoporosis Pharmacological prevention and management in older people

• RACGP - Detection, prevention and treatment of osteoporosis

• Therapeutic Guidelines – Endocrinology

Last updated:
Time spent: QID:771
2023-2-12

185 of 1943
A 34-year-old woman, who is a known case of schizophrenia currently controlled on risperidone, presents to your
practice with a 4-month history of amenorrhea. Physical examination is inconclusive. Among the laboratory tests, a
raised prolactin level at 7845 mU/L is significant. Which one of the following is the most likely diagnosis?

A. Hyperprolactinemia due to risperidone.


B. Prolactinoma.
C. Hypothyroidism.
D. Stress-related illness.
E. Chronic renal failure.

Incorrect. Correct answer is B


45% answered correctly

Explanation:

Correct Answer Is B

The signs and symptoms of this patient and the prolactin level on the test results is suggestive of
hyperprolactinemia. The causes of hyperprolactinaemia are listed in the following table:

pituitary adenomas (micro or macro) - disruption of stalk


hypothalams tumors (craniopharyngioma, glioma), infiltration (sarcoidosis), irradiation
drug- cimetidine, methyldopa, metoclopramide, phenothizine, risperidone, other neuroleptics,
induced SSRIs, TCAs, opiates and marijuana
pregnancy, hypothyroidism, renal failure, liver disease, polycystic ovarian syndrome,
systemic
epileptic seziures
neurogenicc breast stimulation, chest wall trauma or lesion, stress

186 of 1943
According to this table, all of the options can be underlying causes of hyperprolactinemia, but with prolactin level as
high as 7845 mU/L, the most likely cause would be a prolactin-secreting pituitary adenoma (prolactinoma). A
prolactin level of above 5000 mU/L is more likely to have been caused by a prolactinoma.

Although drugs such as risperidone are a potential cause of hyperprolactinemia, the prolactin levels are expected to
be below 5000 mU/L. This is true about hyperprolactinemia due to stress-related illness and hypothyroidism.

References

• Medscape - Hyperprolactinemia

• Therapeutic guidelines – Endocrinology; available from http://tg.org.au

Time spent: QID:51 Last updated:


2023-2-12

187 of 1943
A 32-year-old woman presents to your clinic for a skin checkup. On examination, she is found to have a skin lesion on her upper back. The lesion is highly suspected of being melanoma. An excisional biopsy is performed with 2 mm margins and the
specimen is sent to a laboratory for histopathological studies. The result is a melanoma of 2.5 mm in depth. The margins, however, are clear. Which one of the following would be the next best step in management?

A. A wider excision.

B. Radiation therapy.

C. Sentinel node biopsy.

D. CT scan of the head.

E. Chemotherapy.

Correct
45% answered correctly

Explanation:

Correct Answer Is A

Every lesion suspected of being melanoma should be surgically excised with 2 mm margins both as the initial management and the most appropriate step in diagnosis. Further management is then guided by tne results of the biopsy. If the lesion is
proved to be melanoma, a wider excision is needed. The margins of the second excision depend on the reported depth of melanoma, and is according to the following table:

Depth of the lesion Margins of the second excision


Melanoma in situ (restricted to epidermis) 5 mm
<1.0 mm 1 cm
Minimum 1 cm and maximum 2 cm. for depths of 2-4mm, 2 cm is
1-4 mm
more desirable
>4mm 2 cm ( maximum 3 cm)

For melanomas greater than 1mm in depth, a sentinel node biopsy is recommended during the second excision to assess the potential metastasis to the subcutaneous tissue and lymph nodes.

References

• RACGP (AJGP) - Diagnosis and management of cutaneous melanoma

Last updated:
Time spent: QID:27 2023-2-12

188 of 1943
Which one of the following is the most important prognostic factor in basal cell carcinoma?

A. Depth of the lesion.

B. The color of the lesion.

C. Residual cancer cells in the margins of the excised lesion.

D. The width of the lesion.

E. The site of the lesion.

Incorrect. Correct answer is C


45% answered correctly

Explanation:

Correct Answer Is C

Generally, the prognosis for patients with BCC is excellent, with a 100% survival rate for cases that have not spread to other sites. Typically, basal cell tumors enlarge slowly and relentlessly and tend to be locally destructive. Periorbital tumors can
invade the orbit, leading to blindness, if diagnosis and treatment are delayed. BCC arising in the medial canthus tends to be deep and invasive and more difficult to manage; this type of BCC can result in perineural extension and loss of nerve
function.

Although BCC is a malignant tumor, it rarely metastasizes. The incidence of metastatic BCC is estimated to be less than 0.1%. The most common sites of metastasis are the lymph nodes, lungs, and bones.

Treatment of BCC is curative in more than 95% of cases; however, BCC may recur, especially in the first year, or develop in new sites. The prognosis of BCC is, therefore, mostly based on the likelihood of recurrence.

There are several prognostic factors affecting the chance of recurrence of a BCC. Of which, the clinical location, the architectural pattern and excision margins are the most important factors. Of these three, most reports consider the presence or
absence of tumor cells in the excision margins as the most significant prognostic factor regarding recurrence.

(Options A and D) Although the width and depth of tumors are important, as long as they can be excised with tumor-free margins the prognosis is good with recurrence being less likely.

(Option C) Although different BCCs can vary in color and some with specific morphological or histological characteristics more likely to recur, color alone is not of great significance in determining the prognosis.

(Option E) The site of the lesion is important due to the fact that lesions in specific areas are less likely to be excised with tumor-free margins. Some of these areas are nose, eyelids, temples, pre- and post-auricular regions and lower legs.

TOPIC REVIEW

The following parameters affect, to different extends, the outcome and prognosis of BCC:

Recurrent tumors (poorer prognosis)


Multiple tumors (poorer prognosis)
Size and depth of invasion (stage)
Morphemic, infiltrating and micronodular (poorer prognosis)
Morphological and histological subtype
Treatment modality (Mohs surgery has been associated with best prognosis)
Incomplete excision (probably the poorest prognosis)
Perineural spread
Nevoid basal cell carcinoma syndrome (poorer prognosis)
Special sites (poorer prognosis):

Nose
Eyelids
Temple
Pre- and post-auricular
Lower legs

References

• Cancer Council Australia - Basal cell carcinoma, squamous cell carcinoma (and related lesions) – a guide to clinical management in Australia

Last updated:
Time spent: QID:47 2023-2-12

189 of 1943
A 59-year-old farmer presents to your clinic with a lesion on his upper chest. The lesion has appeared and progressively enlarged in the past 2 months. Examination establishes a diagnosis keratoacanthoma with high certainty. Keratoacanthoma is
more likely to be confused with which one of the following?

A. Basal cell carcinoma.

B. Squamous cell carcinoma.

C. Pyogenic granuloma.

D. Seborrheic keratosis.

E. Granuloma fissuratum.

Incorrect. Correct answer is B


45% answered correctly

Explanation:

Correct Answer Is B

Keratoacanthoma (KA) is a rapidly growing tumor of keratinocytes. They are almost exclusively seen in sun-exposed areas. The characteristic feature is the crater; the central part of the lesion is necrotic, giving the lesion the appearance of a volcano.

KAs are now considered a low-risk variant of squamous cell carcinoma (SCC). The major diagnostic problem is confusion with SCC, especailly for KAs of the nose and the lips. Interestingly, sometimes KAs cannot be told apart from SCCs based on
cytological studies and the whole specimen is needed for differentiation.

(Option A) An ulcerated nodular BCC may resemble KA, but SCC remains the most confusing differential diagnosis.

(Options C and D) Pyogenic granuloma and seborrheic keratosis are very unlikely to be confused with KA.

(Option E) Granuloma fissuratum is a firm red fissured fibrotic granuloma found in the gums and buccal mucosa. It is usually caused by ill-fitting dentures.

190 of 1943
A 32-year-old female patient presents to your practice complaining of a skin lesion on the ventral aspect of her right forearm, which has developed during the past 6 weeks. The lesion is shown in the accompanying photograph. She had a successful
renal transplant last year. Which one of the following would be the most appropriate treatment?

A. No active treatment is needed, as it resolves spontaneously.

B. Surgical removal of the lesion.

C. Radiotherapy.

D. Topical podophyllin.

E. Removal with liquid nitrogen.

Incorrect. Correct answer is B


45% answered correctly

Explanation:

Correct Answer Is B

The lesion in the photograph is a domed nodule with a necrotic plug in the center. The appearance is characteristic of Keratoacanthoma (KA). KAs are keratinizing skin tumors which grow more rapidly (6-8 weeks) compared with basal cell carcinoma,
squamous cell carcinoma and melanoma. They are usually seen as a solitary nodule in sun-exposed areas. It often develops later in life with a predilection for women.

If the lesion is left untreated, spontaneous healing and resolution may occur within 3 to 6 months; however, at instances it may continue to grow or even metastasize.

Since KA is clinically indistinguishable from malignant lesions, especially squamous cell carcinoma, the preferred management would be the same as for squamous cell carcinoma which is elliptical surgical excision with margins of 3-5mm (the
same that would be done for squamous cell carcinoma of the skin).

KAs share features such as infiltration and cytological atypia with SCCs; hence they are considered to be a variant of SCC called SCC-KA type.

Although a shave biopsy may be used for diagnosis, it is not an adequate final treatment and complete excision should eventually follow.

The need for complete surgical removal is even more in patients who have undergone organ transplantation, because these patients are on immunosuppressive medications drugs; therefore, the lesion is more likely to be malignant. Even if the case
is KA, spontaneous resolution is far less likely in the presence of immunosuppression.

191 of 1943
A 36-year-old woman presents to your clinic concerned about a lesion on her right shin. The lesion appeared several days after the site was stung by a bee. On examination, there is a 0.6mm nodule on the lateral aspect of the right shin as illustrated
in the photograph. It is not tender to touch. Which one of the following could be the most likely diagnosis?

A. Basal cell carcinoma.

B. Squamous cell carcinoma.

C. Dermatofibroma.

D. Molluscum contagiosum.

E. Pyogenic granuloma.

Incorrect. Correct answer is C


45% answered correctly

Explanation:

Correct Answer Is C

The appearance, history, and exam findings are suggestive of dermatofibroma as the most consistent diagnosis.

Dermatofibroma, also called sclerosing hemangioma or histiocytoma, is a common pigmented nodule in the dermis due to the proliferation of fibroblasts, usually following minor trauma. Dermatofibroma is more commonly seen in women on the
lower leg. The lesion is a button-like nodule that is firmwell-circumscribedribed. The size may vary from 0.5 to 1cm. It can be pink, brown, tan, gray, or violaceous. The nodule is freely mobile over the deeper structures. The characteristic feature on
examination is a dimpling of the nodule when it is pinched laterally (dimple sign). The lesion is often asymptomatic but may be itchy or tender.

Other options have different characteristic features.

192 of 1943
A 47-year-old man presents because of a lesion on the back of his right shoulder. The lesion is illustrated in the following photograph. Which one of the following is the most appropriate advice?

A. The lesion should be excised, as it is malignant and can extend locally, but not through lymph nodes.

B. The lesion is benign and does not need to be excised.

C. The lesion is benign, but should be excised because it can become malignant.

D. The lesion is benign, but should be excised because it can become infected.

E. The lesion is malignant and should be excised because it can metastasize through adjacent lymph nodes.

Incorrect. Correct answer is D


45% answered correctly

Explanation:

Correct Answer Is D

The photograph shows a well-circumscribed lesion, which appears to arise from within the epidermis. It also has a punctum in the center. These features are characteristic of an epidermoid (sebaceous) cyst.

Epidermoid cysts originate the from dermis and are attached to the skin. Since they are related to the pilosebaceous follicle, they can occur in any hair-bearing region such as the scalp or scrotum.

Of note, the yellow cheesy material within the cyst is desquamated keratin, not sebum.

Epidermoid cysts are benign and do not progress to malignancy; however, it is recommended that they be removed, as superinfection may lead to suppuration and abscess formation.

193 of 1943
A 6-year-old boy is brought to you by his mother because she is concerned about skin lesions on the boy’s back and trunk. The lesions have started to appear and increase in number for the past week. The rash over his back is shown in the
accompanying photograph. On examination, the child is well and healthy with no other abnormal findings. Which one of the following is the most likely diagnosis?

A. Varicella zoster.

B. Impetigo.

C. Molluscum contagiosum.

D. Herpes simplex.

E. Papilloma virus (warts).

Incorrect. Correct answer is C


45% answered correctly

Explanation:

Correct Answer Is C

The lesions in the photograph are pearly dome-shaped papules with central umbilication characteristic of molluscum contagiosum as the most likely diagnosis. Molluscum contagiosum is a common viral infection of childhood caused by
molluscipoxvirus a member of the poxvirus family.

The infection presents with firm, smooth, spherical papules that are pearly white and have a central dimple (umbilicus). Most papules range from 1 to 3 mm; however, lesions of up to 1-2 cm have been reported mostly due to coalescing smaller
lesions.

Lesions can develop anywhere, but flexures and areas of friction are more frequently involved. Lesions may also occur in the anogenital area and are not usually associated with sexual abuse of the child. Involvement of the eyelid margins may lead
to chronic conjunctivitis.

Molluscum contagiosum has a benign course and nature. Most patients experience the spontaneous resolution of the lesions within 3 to 6 months, but on occasion, it may take up to 3 years.

(Option A) Varicella zoster infection in children (chickenpox) presents with blisters at different stages (intact, ruptured, crusted, or sometimes infected) which are often itchy. The eruption usually follows prodromal symptoms such as malaise and
fever. None of the history and physical examination features is consistent with varicella zoster infection.

(Option B) Impetigo is a superficial skin infection presenting with lesions often with honey-colored crusting.

(Option D) Herpes simplex presents with painful vesicles that may become unroofed to produce a raw appearance.

(Option E) Papillomavirus infection (wart) presents a completely different picture. Often it is cauliflower-like with a rough surface.

194 of 1943
There are a number of skin lesions which are related to cumulative sun-exposure. Which one of the following is most likely to be caused by chronic sun exposure?

A. Actinic (solar) keratosis.

B. Junctional nevus.

C. Seborrheic keratosis.

D. Tinea versicolor.

E. Keratoacanthoma.

Correct
45% answered correctly

Explanation:

Correct Answer Is A

Actinic keratosis and Bowen’s disease are seen frequently in light-skinned individuals, who have had significant sun-exposure. They are precancerous lesions for squamous cell carcinoma.

(Option B) Junctional nevus is not associated with sun-exposure.

(Option C) Seborrheic keratosis does not seem to have strong association with sun-exposure because it frequently appears in areas not exposed to and affected by the sunlight.

(Option D) Tinea versicolor is a skin infection caused by Malassezia furfur and is aggravated by heat and damp, but not related to sun exposure.

(Option E) Keratoacanthoma is a rather benign tumor arising from pilocebaceous glands, mostly in sun-exposed areas such as face. The association with sun-exposure is not as significant compared with actinic keratosis or Bowen's disease.

References

• Medscape - Actinic Keratosis

• DermNet NZ - Actinic Keratosis

• RACGP - Actinic Keratoses

Last updated:
Time spent: QID:57 2023-2-12

195 of 1943
A 50-year old farmer presents to your practice with a dark mole on his left cheek. The mole has been there for 20 years, but has enlarged and become slightly lumpy and itchy over the past 4 months. Which one of the following would be the most
appropriate management option for this patient?

A. Treat the lesion using liquid nitrogen.

B. Ask the patient to return for review in 3 months.

C. Remove the lesion using the laser.

D. Use topical imiquimod for 6 weeks.

E. Excisional biopsy of the lesion for histopathology.

Incorrect. Correct answer is E


45% answered correctly

Explanation:

Correct Answer Is E

A changing mole, a mole with an irregular border, bleeding, itching, or color variegation is melanoma until proven otherwise. When melanoma is suspected, the next best step is an excisional biopsy of the lesion with 2 mm margins. Elliptical incisions
are made so that reconstruction of the wound is more straightforward. If histopathological results confirm the diagnosis, referral to a plastic surgeon is necessary for a wider excision with margins of 5 mm to 3 cm depending on the depth of the
lesion.

NOTE - in GP settings, patients with suspected lesions on sensitive areas such as the head and neck should be referred to a plastic surgeon even for the initial excision. If the referral was an option, it would be the correct answer.

196 of 1943
Which one of the following is the most important risk factor for melanoma?

A. Family history of basal cell carcinoma or squamous cell carcinoma of the skin.

B. A cousin with melanoma in family history.

C. Working outdoors since the age of 18.

D. Multiple sunburns in childhood.

E. Presence of solar keratosis.

Incorrect. Correct answer is D


45% answered correctly

Explanation:

Correct Answer Is D

The following table classifies risk factors for developing melanoma in a descending order are as follows:

Risk factor Comments Relative Risk


With a history of previous melanoma, the person is more than 10 times likely to develop another melanoma.
Previous history of melanoma >10
The risk is greatest at the first 1-2 years
A simple melanotic nevi count of greater than 100 is associated with a 7-fold increase in risk of contracting
Multiple melanotic nevi (benign) 7
melanoma compared to general population
A dysplastic nevi count of greater than 5 is associated with a 6-fold increase in risk of contracting melanoma
Multiple dysplastic nevi 6
compared to general population
A history non-melanoma skin cancer is associated with 4-time increase in melanoma compared to general
Previous history of nonmelanoma skin cancer 4
population.
Family history of melanoma
Family history of melanoma in a first-degree relative almost doubles the risk of melanoma. This risk is higher
2
if the family member has had melanoma at younger age as it could be an indicator of familial melanoma.

Compared to individuals with Fitzpatrik type IV skin, those with type I (pale skin, bright eyes, freckles who
Fair complexion easily burn and never tan) have near 2-fold increase in risk of contracting melanoma compared to general ~2
population.
There is higher rates of melanoma in people with extensive or repeated intense exposure to sunlight. The risk
is highest if exposure occurs intermittently in adolescence or childhood. One episode of severe blistering
UV exposure 1.5
sunburn confers about the same risk of intermittent sun exposure. However, the relative risks for the highest
categories of exposure, compared with the lowest, are rarely > 1.5.

Among the given options multiple sunburns in childhood predispose to the most significant risk (it is associated with only slightly elevated risk factor, yet the greatest among other options).

(Option A) A family history of non-melanoma skin cancers (NMSC) is an important risk factor for NMSC but not for melanoma; however, personal history of NMSC is a significant risk factor for melanoma.

(Option B) Family history of melanoma is not that significant if not in the first-degree relatives (parents, siblings).

(Option C) Working outdoors since the age of 18 years is another risk factor, but not as important as multiple sunburns.

(Option E) Solar keratosis is a significant risk factor for squamous cell carcinoma, not melanoma.

197 of 1943
The mother of a 5-year-old boy, who has recently undergone removal of a melanoma on her back asks you what can play a major role in developing melanoma in her child in the future. Which one of the following options would be your answer to her
question?

A. Family history of melanoma.

B. Sunburn.

C. Fair skin.

D. UV exposure.

E. The presence of multiple dysplastic moles.

Incorrect. Correct answer is E


45% answered correctly

Explanation:

Correct Answer Is E

The following table categorizes the risk factors for developing melanoma in a descending order of significance:

Risk factor Comments Relative Risk


With a history of previous melanoma, the person is more than 10 times likely to develop another melanoma.
Previous history of melanoma >10
The risk is greatest at the first 1-2 years
A simple melanotic nevi count of greater than 100 is associated with a 7-fold increase in risk of contracting
Multiple melanotic nevi (benign) 7
melanoma compared to general population
A dysplastic nevi count of greater than 5 is associated with a 6-fold increase in risk of contracting melanoma
Multiple dysplastic nevi 6
compared to general population
A history nonmelanoma skin cancer is associated with 4-time increase in melanoma compared to general
Previous history of nonmelanoma skin cancer 4
population.
Family history of melanoma
Family history of melanoma in a first-degree relative almost doubles the risk of melanoma. This risk is higher
2
if the family member has had melanoma at younger age as it could be an indicator of familial melanoma.

Compared to individuals with Fitzpatrik type IV skin, those with type I (pale skin, bright eyes, freckles who
Fair complexion easily burn and never tan) have near 2-fold increase in risk of contracting melanoma compared to general ~2
population.
There is higher rates of melanoma in people with extensive or repeated intense exposure to sunlight. The risk
is highest if exposure occurs intermittently in adolescence or childhood. One episode of severe blistering
UV exposure 1.5
sunburn confers about the same risk of intermittent sun exposure. However, the relative risks for the highest
categories of exposure, compared with the lowest, are rarely > 1.5.

All the given options are potential risk factors for developing melanoma, but the presence of multiple moles (>100 nevi or more than 5 dysplastic nevi) is the most significant risk factor.

Red hair and blue eyes are associated with a highly- and moderately-increased risk of melanoma respectively. Fair complexion is another risk factor, but the degree of association with melanoma depends on the age of the patient. In patients over the
age of 45 years, fair skin can be as significant a risk factor as multiple nevi.

UV exposure and sunburns are associated with a slightly increased risk of melanoma. In sun-related melanomas, acute intense and intermittent blistering sunburns, especially on areas of the body that only occasionally receive sun exposure, are the
most significant risk factors for the development of sun exposure-induced melanoma. This sun-related risk factor for malignant melanoma differs from squamous or basal cell carcinomas.

NOTE - lentigo malignant melanoma (LMM) is an exception to this rule because it frequently appears on the head and neck of older individuals who have a history of long-term sun exposure; therefore, prolonged sun (UV) exposure is the
greatest risk factor for LMM.

Exposure to ultraviolet radiation (UVR) is a critical factor in the development of sun-related melanomas. Ultraviolet A (UVA), wavelength 320-400 nm, and ultraviolet B (UVB), 290-320 nm, potentially are carcinogenic and can attribute to melanoma
induction. The suggested mechanisms through which UV exposure can play its role are:

Suppression of the immune system of the skin


Induction of melanocyte cell division
Free radical production
Damage to melanocyte DNA

Interestingly, melanoma does not have a direct relationship with the amount of sun exposure because it is more common in white-collar workers than in those who work outdoors. Also, it is more common on the back of men's and women's lower
legs when there is no significant sun exposure.

198 of 1943
On a routine health examination, the lesion shown in the following photograph is found on the inner side of the lower lip of a 37-year-old man. The lesion is painless. Which one of the following is the most likely diagnosis?

A. Sebaceous cyst.

B. Peutz - Jegher's syndrome.

C. Squamous cell carcinoma of the lip.

D. Mucous cyst.

E. Basal cell carcinoma.

Incorrect. Correct answer is D


45% answered correctly

Explanation:

Correct Answer Is D

The lesion shown has a bluish-glistening color and is dome-shaped. These are characteristics of a benign mucoid cyst. A mucous cyst, also known as a mucocele forms when mucus or saliva escapes into surrounding tissues. A lining of granulation
or connective tissue is formed to create a smooth, soft round fluid-filled lump. They most commonly occur on the inner surface of the lower lip (75% of cases) but may also appear on the floor of the mouth or on the gums, buccal mucosa, and
tongue. If persistent or bothersome, an incision and evacuation of the cyst are performed.

(Option A) Sebaceous cysts are caused by obstruction of a sebaceous gland in hair-bearing skin. It does not occur in the lip.

(Option B) Peutz - Jegher's syndrome is associated with melanocytic spots on the buccal mucosa and in the gastrointestinal tract.

The lip of a female patient with Peutz-Jegher's syndrome

(Options C and E) Both squamous cell carcinoma (SCC) and basal cell carcinoma (BCC) of the lip occur at the mucocutaneous junction of the lip. The mucus membrane of the lip is unlikely to be the site an SCC or BCC arises. Moreover, patients with
SCC or BCC typically are older.

References

• DermNet NZ - Mucocoele of the lip

• Medscape - Dermatologic Manifestations of Mucocele (Mucous Cyst)

Time spent: QID:35 Last updated:


2023-2-12

199 of 1943
Which one of the following lesions of squamous epithelium is not premalignant?

A. Solar (actinic) keratosis.

B. Leukoplakia.

C. Intradermal nevus.

D. Bowen's disease.

E. Chronic radiation dermatitis.

Incorrect. Correct answer is C


45% answered correctly

Explanation:

Correct Answer Is C

Of the given options, the only one not associated with skin cancers is intradermal nevus. Benign melanocytic nevi have three major types, classified according to the position of the melanocytes in relation to epidermal:

Intradermal nevus: all of the nevus cells are within dermis. This type accounts for a majority of benign congenital nevi.
Junctional nevus: the nevus cells are located at the junction of the basal epidermal layers and dermis.
Combined nevus: nevus cells are both intradermal and junctional.

(Option A) Solar (actinic) keratosis is usually a raised plaque usually on sun-exposed area of the skin. It is a precursor of SCC.

(Option B) Leukoplakia is whitish lesion in the oral cavity and associated with SCC.

(Option D) Bowen disease is considered SCC in situ, and is malignant.

(Option E) Chronic radiation dermatitis may result in SCC.

References

• Fitzpatrick’s Color Atlas & Synopsis of Clinical Dermatology – McGraw Hill - 6th Edition

• http://emedicine.medscape.com/article/279269-overv

• http://emedicine.medscape.com/article/1099775-over

• http://www.therapeutique-dermatologique.org/spip.p

Last updated:
Time spent: QID:61 2023-2-12

200 of 1943
Which one of the following skin malignancies is most likely to arise from a burn scar?

A. Basal cell carcinoma.

B. Malignant melanoma.

C. Squamous cell carcinoma.

D. Fibrosarcoma.

E. Sweat gland adenocarcinoma.

Incorrect. Correct answer is C


45% answered correctly

Explanation:

Correct Answer Is C

The most common skin malignancy arising from a burn scar is squamous cell carcinoma (SCC). Marjolin ulcer is a less common type of SCC of the extremities found on chronic ulcers or burn scars.Marjolin ulcers occur on average approximately 30
years after an injury to the skin that results in a scar or an ulcer (range 10–75 years). Rarely, an acute Marjolin ulcer may develop between 6 weeks and 1 year of injury. It is estimated that around 2% of thermal burns scars turn into Marjolin ulcers.

Marjolin ulcer can affect people of all ages, most commonly between 40 and 60 years of age. Men are 2–3 times more likely be diagnosed with Marjolin ulcer than women. All races and skin types can develop Marjolin ulcers. The most common sites
for Marjolin ulcers are the legs and feet. The ulcers can also form on the head and neck.

Other options are malignancies not seen or very rarely seen in areas with a burn scar.

201 of 1943
Which one of the following is the most likely diagnosis of the lesion shown in the accompanying photograph?

A. Keratoacanthoma.

B. Basal cell carcinoma.

C. Seborrheic dermatitis.

D. Malignant melanoma.

E. Simple nevus.

Incorrect. Correct answer is D


45% answered correctly

Explanation:

Correct Answer Is D

The photograph shows a pigmented lesion on the left side of the nose. With pigmentation, the two most likely diagnoses are simple nevi or malignant melanoma. Early melanomas may be differentiated from benign nevi by the ABCD:

A - Asymmetry
B - Border irregularity
C - Color that tends to be very dark black or blue and variable
D - Diameter ≥ 6 mm

The lesion is asymmetrical (A) and has irregular borders (B). It shows variegation (variation in colors) (C) and seems to be larger than 6mm; therefore, melanoma seems to be the most likely diagnosis.

Keratoacanthoma, basal cell carcinoma, and seborrhoeic dermatitis present quite differently.

References

• Cancer Council Australia - Clinical practice guidelines for the diagnosis and management of melanoma

• Medscape - Malignant Melanoma

Last updated:
Time spent: QID:615 2023-2-12

202 of 1943
A 52-year-old golfer man presents to your GP clinic with an ulcer on his right pinna. He says the lesion is itchy and easily bleeds on scratching. On examination, deeply sunburned areas around the lesion are noted. The lesion is shown in the
accompanying photograph. Which one of the following can be the most likely diagnosis?

A. Basal cell carcinoma.

B. Squamous cell carcinoma.

C. Malignant melanoma.

D. Bowen’s disease.

E. Keratoacanthoma.

Incorrect. Correct answer is B


45% answered correctly

Explanation:

Correct Answer Is B

The photograph shows a flat amelanotic lesion on the left pinna that has undergone ulceration and is slightly crusted. The appearance of the lesion is consistent with cutaneous squamous cell carcinoma (SCC) as the most likely diagnosis.

The classic presentation of a cutaneous SCC includes a shallow ulcer with heaped-up edges, often covered by a plaque usually in a sun-exposed area. Typical surface changes may include the following:

Scaling
Ulceration
Crusting
A cutaneous horn

Less commonly, cutaneous SCC presents as a pink cutaneous nodule without overlying surface changes. Regional spread of head and neck cutaneous SCCs, may result in enlarged preauricular, submandibular, or cervical lymph nodes.

NOTE - Although, the appearance of the lesion resembles SCC, it should be noted that at times SCC, keratoacanthoma, BCC, or even amelanotic forms of melanoma may look similar and the definite diagnosis cannot be made unless biopsy and
histologic studies are performed. However, because of the classic features of the lesion and also the fact that squamous SCC is the most common skin cancer, SCC would be the most likely diagnosis in this case.

(Option A) Although basal cell carcinoma (BCC) of this area is more common than SCC, the characteristic features of BCC, especially in its most common from – nodular, is different.

(Option C) Malignant melanoma is often pigmented. This amelanotic lesion is less likely to be melanoma.

(Option D) Bowen’s disease (SCC in situ) presents with an asymptomatic well-demarcated erythematous patch or plaque. The presence of symptoms makes Bowen’s disease a less likely yet possible diagnosis. Moreover, ill-defined borders of this
lesion is not in favor of Bowen’s disease.

(Option E) Keratoacanthoma often has a nodular structure with central crater which is absent here.

203 of 1943
References

• Cancer Council - Clinical Practice Guide

• DermNet NZ - Cutaneous squamous cell carcinoma

• http://emedicine.medscape.com/article/1965430-overview

Last updated:
Time spent: QID:616 2023-2-12

204 of 1943
Which one of the following statements is correct regarding nail apparatus melanoma?

A. It is a common form of melanoma.

B. It has a good prognosis.

C. The 5-year survival depends on the depth (thickness) of the lesion.

D. Nail removal maybe curative.

E. There is a strong association to sun exposure.

Incorrect. Correct answer is C


45% answered correctly

Explanation:

Correct Answer Is C

Nail apparatus melanomas (subungual melanoma) are rare but frequently fatal neoplasms. They account for between 1.5-3% of all melanomas. All age groups can be affected; however, it is most common in the 7th decade of life.

Melanoma of the nail initially presents as a longitudinal pigmented streak. Later, the proximal nail fold may become pigmented (Hutchinson’s sign). As the disease advances, the nail plate may be destroyed. By this stage, the disease is often
advanced, with lymph node metastasis frequently present.

As with other melanomas, increased tumor depth and the presence of ulceration are associated with a worse prognosis and decreased five-year survival.

70% of subungual melanomas occur on the thumb or great toe. Subungual melanomas involving the toes can be managed easily with digital amputation at the metatarsal-phalangeal joint. If the first toe (big toe) is involved, complete amputation
should be avoided when the lesion is amenable to less invasive oncological treatments because of the importance of the toe in balance.

Whenever possible, subungual melanomas of the fingers should be resected at the distal interphalangeal joint to preserve function. Melanomas located more proximally on the fingers can often be managed with wide local excision of soft tissue,
skin grafts, or local flaps for soft tissue coverage.

NOTE - Longitudinal melanonychia occurs in other conditions, including benign nevi of the nail matrix and the multiple linear streaks that are common in people with dark skin. However, patients with a solitary, widening, irregularly shaped, or
irregularly pigmented streak in the nail should undergo a biopsy of the nail matrix to evaluate for melanoma.

(Option A) Nail apparatus melanoma accounts for 1.5-3% of all melanomas. It is not the most common form of melanoma.

(Option B) Nail apparatus melanoma has a poor prognosis.

(Option D) Nail removal has no role in the management of melanoma, as the lesion is in the subungual matrix.

(Option E) Nail apparatus melanomas occur with almost equal incidence in people with black skin, Asians, and Caucasians, and in people in tropical versus nontropical climates, sun exposure does not seem to be an etiology.

References

• Therapeutic Guidelines – Dermatology; available from http://tg.org.au

• UpToDate - Surgical management of primary cutaneous melanoma or melanoma at other unusual sites

Last updated:
Time spent: QID:617 2023-2-12

205 of 1943
A 42-year-old male, accountant by profession, comes for screening and risk assessment for melanoma. Which one of the following, if present in the history, would be of highest risk for developing melanoma?

A. Sunburns in childhood.

B. Irish background.

C. Family history of basal cell carcinoma.

D. A cousin who has recently been diagnosed with melanoma.

E. Working outdoors in early adulthood.

Correct
45% answered correctly

Explanation:

Correct Answer Is A

The following table categorizes risk factors for developing melanoma in a descending order of significance::

Risk factor Comments Relative risk


With a history of previous melanoma, the person is more than 10 times likely to develop another melanoma.
Previous history of melanoma >10
The risk is greatest at the first 1-2 years
A simple melanotic count of greater than 100 is associated with a 7-fold increase in risk of contracting
Multiple melanotic nevi (benign) 7
melanoma compared to general population
A dysplastic nevi count of greater than 5 is associated with a 6-fold increase in risk of contracting melanoma
Multiple dysplastic nevi 6
compared to general population
A history nonmelanoma skin cancer is associated with 4-time increase in melanoma compared to general
Previous history of nonmelanoma skin cancer 4
population.
Family history of melanoma
Family history of melanoma in a first-degree relative almost doubles the risk of melanoma. This risk is higher
2
if the family member has had melanoma at younger age as it could be an indicator of familial melanoma.

Compared to individuals with Fitzpatrik type IV skin, those with type I (pale skin, bright eyes, freckles who
Fair complexion easily burn and never tan) have near 2-fold increase in risk of contracting melanoma compared to general ~2
population.
There is higher rates of melanoma in people with extensive or repeated intense exposure to sunlight. The risk
is highest if exposure occurs intermittently in adolescence or childhood. One episode of severe blistering
UV exposure 1.5
sunburn confers about the same risk of intermittent sun exposure. However, the relative risks for the highest
categories of exposure, compared with the lowest, are rarely > 1.5.

Of the given options, multiple sunburns in childhood are the most significant risk factor for melanoma. Of UV-related risk factors, sunburns in the first and second decades of life, especially if acute and blistering, carries the most significant risk
factor compared with other options, but not in general because UV exposure is associated with only a slightly increased risk of developing melanoma (relative risk ~1.5).

(Options B and E) Irish background (Caucasian) and working outdoors since early adulthood may be associated with slightly increased risk, but not as significant as sunburns in childhood.

(Option C) A family history of non-melanoma skin cancers (NMSC) does not appear to be a risk factor; however, personal history of NMSC is a risk factor with a relative risk of approximately 4.

(Option D) A family history of melanoma in a second-degree relative is not a risk factor for melanoma, but melanoma in first-degree relatives at least doubles the risk of developing melanoma in a person.

References

• Cancer Council Australia - Clinical practice guidelines for the diagnosis and management of melanoma

• Medscape - Malignant Melanoma

Last updated:
Time spent: QID:623 2023-2-12

206 of 1943
Ben, 10 years old, is brought by his mother to your GP clinic for advice on melanoma screening. Which one of the following puts Ben at the highest risk of developing melanoma?

A. Family history of melanoma in first-degree relatives.

B. Red hair.

C. Freckles.

D. Multiple sunburns.

E. Outdoor activities.

Correct
45% answered correctly

Explanation:

Correct Answer Is A

The following table classifies risk factors for developing melanoma in descending order:

Risk factor Comments Relative Risk


With a history of previous melanoma, the person is more than 10 times likely to develop another melanoma.
Previous history of melanoma >10
The risk is greatest at the first 1-2 years
A simple melanotic nevi count of greater than 100 is associated with a 7-fold increase in the risk of contracting
Multiple melanotic nevi (benign) 7
melanoma compared to the general population
A dysplastic nevi count of greater than 5 is associated with a 6-fold increase in the risk of contracting
Multiple dysplastic nevi 6
melanoma compared to the general population
A history of nonmelanoma skin cancer is associated with a 4-fold increase in the risk of melanoma compared
Previous history of nonmelanoma skin cancer 4
to the general population.
Family history of melanoma
A family history of melanoma in a first-degree relative almost doubles the risk of melanoma. This risk is higher
2
if the family member has had melanoma at a younger age as it could be an indicator of familial melanoma.

Compared to individuals with Fitzpatrick type IV skin, those with type I (pale skin, bright eyes, freckles who
Fair complexion easily burn and never tan) have a nearly 2-fold increase in the risk of contracting melanoma compared to the ~2
general population.
There are higher rates of melanoma in people with extensive or repeated intense exposure to sunlight. The risk
is highest if exposure occurs intermittently in adolescence or childhood. One episode of severe blistering
UV exposure 1.5
sunburn confers about the same risk of intermittent sun exposure. However, the relative risks for the highest
exposure categories, compared with the lowest, are rarely > 1.5.

The following table classifies risk factors for melanoma as high, moderate, and lower than average:

High Risk

Changing mole
Dysplastic nevi in familial melanoma
Multiple numbers of naevi (>100) or >10 dysplastic naevi

Moderate Risk
One family member with melanoma
Previous history of melanoma
Sporadic dysplastic naevi
Congenital naevus

Lower than average risk


Immunosuppression
Sun sensitivity
History of acute, severe blistering sunburns
Freckling

Of the given options, a family history of melanoma carries the most significant risk (RR of at least 2), followed by red hair/freckling and then multiple sunburns, and prolonged sun exposure caused by outdoor activity.

References

• Cancer Council Australia - Clinical practice guidelines for the diagnosis and management of melanoma

• Medscape - Malignant melanoma

Time spent: QID:624 Last updated:


2023-2-12

207 of 1943
The following photograph shows skin lesions on the hand of a 73-year-old woman, who resides in a nursing home. The lesions are severely itchy and she keeps scratching them, especially in the night. The itch prevents her from a good night sleep.
On examination, she looks disheveled and poorly cared for. Which one of the following would be the most appropriate treatment for her?

A. Permethrin 5% cream.

B. Itraconazole cream.

C. Topical corticosteroids.

D. Gamma benzene hexachloride.

E. Topical erythromycin.

Correct
45% answered correctly

Explanation:

Correct Answer Is A

The itching, the appearance of the lesions and living in a nursing home all favor of scabies as the most likely diagnosis. First-line medication for treatment of scabies is permethrin 5% cream. If treatment with permethrin fails or the patient is sensitive
to this drug, benzyl benzoate can be used as second-line therapy. Benzyl benzoate is more irritating to skin and the treatment is likely to fail due to inadequate adherence.

(Option B) Itraconazole cream is used for treatment of fungal skin infections. It has no role in treatment of scabies.

(Option C) Topical corticosteroids of moderate potency may be used in conjunction with permethrin or benzyl benzoate, with no effect on the scabies.

(Option D) Gamma benzene hexachloride (Lindane®) was formerly used for treatment of scabies. It is no longer approved for this purpose in Australia, as well as many other parts of the world.

(Option E) If a bacterial infection superimposes scabies, the antibiotics of choice would be topical mupirocin or oral cephalexin in more severe cases. Erythromycin is not routinely used for

References

• Therapeutic Guidelines - Dermatology; available from http://tg.org.au

Last updated:
Time spent: QID:625 2023-2-12

208 of 1943
A 50-year-old woman presents to your clinic with multiple lumps. She has had these lumps for 20 years. The only complaint, apart from her concerns about the look of the lumps, is that they become painful at times. On examination, about 30 mobile
subcutaneous lumps feel soft, rubbery, and lobulated when palpated. They are distributed bilaterally in her upper and lower extremities, chest, and back. She mentions that her mother has the same problem. She is otherwise healthy. Which one of the
following is the most likely diagnosis?

A. Adipose dolorosa.

B. Multiple desmoid tumors.

C. Multiple epidermoid cysts.

D. Multiple symmetrical subcutaneous lipomas.

E. Neurofibromatosis type I (Von Recklinghausen disease of nerve).

Incorrect. Correct answer is D


45% answered correctly

Explanation:

Correct Answer Is D

The clinical findings such as mobility, subcutaneous location, and the consistency of the lumps make the multiple symmetrical lipomas the most likely diagnosis. Being lobulated is another important clue, as is the positive family history.

Lipomas are benign tumors of mature fat cells. They are quite common and can be found in subcutaneous tissue. Although the subcutaneous fat layer is the most common site for lipomas to arise, they may be subfascial, intramuscular, or be found
in many other sites. There is usually a genetic predisposition.

(Option A) Adipose Dolorosa (Dercum disease) is a syndrome characterized by diffusely painful subcutaneous fat deposition without focal discrete lumps. This syndrome is most common in middle-aged women. The painful fat deposits are mostly
confined to and seen in the abdomen and thighs.

(Option B) Desmoid tumors arise from the deeper layers of the abdominal wall.

(Option C) Epidermoid cysts contain keratinized material and are confined to the dermis. They are invariably attached to the overlying skin and are not mobile. They have a central punctum and may become infected and form abscesses. Like lipomas,
epidermoid cysts do not progress to malignant lesions.

(Option E) Neurofibromatosis type I, also called Von Recklinghausen disease, is a disease of nerves presenting with café-au-lait spots and pedunculated and sessile skin lesions (molluscum fibrosom). Given the history and characteristics of the
lesions, neurofibromatosis is unlikely to be the diagnosis.

References

• DermNet NZ - Lipoma and liposarcoma

• Medscape - Lipomas

Last updated:
Time spent: QID:626 2023-2-12

209 of 1943
The lesion shown in the following photograph is found on the back of a 50-year-old man. It has been present for the past 8 years and has grown slowly. Which one of the following is the most appropriate next step in management?

A. Review in 12 months.

B. Resection.

C. Excisional biopsy with 2 mm margins.

D. Radiotherapy.

E. Topical imiquimod.

Incorrect. Correct answer is C


45% answered correctly

Explanation:

Correct Answer Is C

A changing mole, especially this one with color variegation evident in the photograph, should be considered melanoma until proven otherwise. Even when a clinical diagnosis of melanoma is made with certainty, an excisional biopsy should be
performed. This confirms the diagnosis and allows further planning of definitive treatment in terms of the width and orientation of excision margins, and whether or not a sentinel lymph node (SLN) biopsy is performed.

Immediate wide excision with margins based on a clinical estimate of tumor thickness may result in inadequate or excessive tumor clearance. It may also compromise subsequent management by making it impossible to perform accurate lymphatic
mapping to identify draining lymph node fields and SLNs within those fields.

The Clinical practice guidelines for the management of cutaneous melanoma in Australia and New Zealand, endorsed by the Australian National Health and Medical Research Council (NHMRC), recommend excision biopsy with 2 mm margins whenever
possible.

Partial biopsies such as punch biopsies, incisional biopsies and shave biopsies are frequently unsatisfactory and may result in misdiagnosis due to unrepresentative sampling. However, an incision, punch or shave biopsy from the most suspicious
area of a large pigmented lesion may be appropriate when complete excision is difficult.

References

• RACGP - AJGP - Diagnosis and management of cutaneous melanoma

Time spent: QID:627 Last updated:


2023-2-12

210 of 1943
The lesion shown in the accompanying photograph has been present on the finger of a 56-year-old woman for the past 8 months. It has slowly enlarged since then and is causing mild discomfort. Which one of the following is the most likely
diagnosis?

A. Chronic paronychia.

B. Heberden’s node of osteoarthritis.

C. Pyogenic granuloma.

D. Mucous (synovial) cyst of the finger.

E. Rheumatoid nodule.

Incorrect. Correct answer is D


45% answered correctly

Explanation:

Correct Answer Is D

The shiny nodule shown in the photograph which appears to be fluid-filled is suggestive of the digital synovial cyst (mucous cyst of the finger ).

Mucous (synovial) cysts of the finger are subcutaneous cystic lesions found on the dorsal aspect of the distal phalanx, distal to the distal interphalangeal joint, and overlying the germinal nail bed. They may cause distortion of the nail growth. They
are located in the midline or laterally. They grow slowly and may undergo spontaneous resolution. Recurrence is likely if they are inadequately excised.

(Option A) Chronic paronychia is nail fold inflammation usually caused by staphylococcus aureus or fungi. The characteristic feature is a painful swelling and erythema of the whole nail fold (crescent-shaped).

(Option B) Heberden’s node of osteoarthritis is a bony lesion over the distal interphalangeal joints.

(Option C) Pyogenic granuloma is a fleshy red vascular structure usually followed by minor trauma to the fingers. It is painless.

(Option E) Rheumatoid nodules are periarticular nodules seen in patients with rheumatoid arthritis.

References

• DermNet NZ - Cutaneous cysts and pseudocysts

• Medscape - Digital Mucous Cyst

Last updated:
Time spent: QID:36 2023-2-12

211 of 1943
Which one of the following statements is correct about lipomas?

A. Lipoma is a premalignant lesion.

B. Lipomas are usually lobulated.

C. Lipomas cannot be tethered to the skin.

D. Lipomas often occur in the scrotum.

E. Lipomas are invariably subcutaneous.

Incorrect. Correct answer is B


45% answered correctly

Explanation:

Correct Answer Is B

Lipomas are common benign tumors of mature fat cells that can be seen in any site of the body containing fatty tissue. They are not premalignant (option A).

Although they frequently occur in the subcutaneous fat tissue, it is not uncommon to see lipomas in the deeper layers such as beneath the fascia or within muscles. Therefore, they are not invariably subcutaneous (option E) and can be found in
deeper layers as well

As there is very little fat in the scrotum, lipomas almost never occur there (option D), but fat deposition within the spermatic cord may be seen. Unlike lipomas, epidermoid cysts of the scrotum are common.

Lipomas are usually freely mobile and not attached to the overlying skin; however, in areas such as the back of the neck or the trunk, where the skin has less mobility, they could be less mobile or even immobile. This fact makes option C incorrect.

A lipoma's most characteristic physical feature is the lobulated contour, which can reliably differentiate it from an epidermoid cyst and its smooth contour.

References

• Medscape - Lipomas

Last updated:
Time spent: QID:628 2023-2-12

212 of 1943
On a health check, a 75-year-old man is found to have a lesion on his left cheek. He mentions that he has had it for three years. The lesion is illustrated in the accompanying photograph. Which one of the following is the most likely diagnosis?

A. Benign nevus.

B. Seborrheic keratosis.

C. Bowen’s disease.

D. Lentigo maligna melanoma.

E. Pigmented basal cell carcinoma.

Incorrect. Correct answer is D


45% answered correctly

Explanation:

Correct Answer Is D

The irregular borders and different color shades of the lesion are suggestive of melanoma. Since the lesion has been stable for 3 years, lentigo maligna melanoma (Hutchinson melanotic freckles) is the most likely diagnosis.

Lentigo maligna melanoma is classically a pigmented macular skin lesion found on the sun-exposed facial skin of the elderly. The color varies from black to light tan and is often variegated.

(Option A) Benign nevus tends to have smoother borders and more even coloration.

(Option B) Seborrheic keratoses are well-defined lesions with surfaces ranging from velvety to verrucous. The color can vary from black to light tan. Color variegation is not usually a feature.

(Option C) Bowen’s disease (squamous cell carcinoma in situ) is a scaly plaque most commonly in sun-exposed areas. The plaque of Bowen’s disease can be mistaken for psoriasis or dermatitis.

(Option E) Basal cell carcinoma has different characteristic features such as being pearly, raised, and associated with telangiectasis in nodular BCC, pearly thready borders in superficial BCC, and scar-like appearance in morphoeic BCC, none of
which are present here.

References

• DermNet NZ - Lentigo maligna and lentigo maligna melanoma

• Medscape - Lentigo Maligna Melanoma

Time spent: QID:630 Last updated:


2023-2-12

213 of 1943
A 45-year-old woman presents with a mole on the lower eyelid of her left eye as shown in the following photograph. Which of the following would be the most appropriate management?

A. Excision of the lesion under local anesthesia.

B. Review in one month.

C. Reassure that the lesion is benign.

D. Refer the patient to a plastic surgeon.

E. Apply topical imiquimod.

Incorrect. Correct answer is D


45% answered correctly

Explanation:

Correct Answer Is D

The photograph shows an irregular pigmented lesion on the lower eyelid of the left eye. The appearance of the lesion is suggestive of melanoma and complete excision and histopathological studies are indicated. Due to the anatomical location of
the lesion, excision should be performed by a plastic surgeon to avoid tissue destruction or loss of function in such an anatomically delicate area.

When melanoma is suspected, excisional biopsy should be done as soon as possible; therefore ‘review in one month’ (option B) and ‘reassurance’ (option C) are incorrect answers. Imiquimod (option E) is not an appropriate management option for
melanoma.

References

• Cancer Council Australia - Clinical Practice Guidelines for the Management of Melanoma in Australia and New Zealand

Time spent: QID:631 Last updated:


2023-2-12

214 of 1943
An 8-year-old boy is brought to your practice by his mother, who is concerned about a patch of hair loss on his head. According to the mother, the hairfall started 2 weeks ago and left the area bald. On examination, there is a 2x3cm patch of hair loss
on the scalp. There is no scarring, inflammation, or flaking of the area. The region is completely bald with no hair. Which one of the following is the most likely diagnosis?

A. Psoriasis.

B. Tinea capitis.

C. Alopecia areata.

D. Trichotillomania.

E. Discoid lupus erythematosus.

Incorrect. Correct answer is C


45% answered correctly

Explanation:

Correct Answer Is C

Sudden-onset patchy hair loss, no hair growth, and a normal-appearing scalp favor alopecia areata as the most likely diagnosis.

Alopecia areata is a term used for a hair disorder characterized by one or more discrete areas of hair loss. Although hair loss can occur anywhere in the body, hair loss of the scalp, eyebrow, or beard often brings the patient to medical attention.

The physiopathology of alopecia areata is an autoimmune inflammation of anagen hair bulbs leading to the cessation of hair growth, but not the destruction of the hair follicle. The etiology of this autoimmune reaction is unknown. Probably, a genetic
factor is involved, as 20% of the patients have a positive family history of alopecia areata.

The most common presentation of alopecia areata is the appearance of one or many round-to-oval completely hairless patches. The presence of exclamation point hairs (i.e. hairs tapered near the proximal end) is pathognomonic but is not always
found. No epidermal changes are associated with hair loss, and the scalp remains intact. Additionally, hair loss in other hair-bearing areas supports the diagnosis. A positive ‘pull test’ in the periphery of the hair loss patch indicates that the disease is
active and more hair loss can be expected.

Nail involvement is seen in up to 50% of patients and is associated with poor prognosis. Nail pitting is the most common form of nail involvement but other abnormalities have been reported as well. Fingernails (rather than toenails) are
predominantly affected.

The course of the disease is extremely variable. The patch(es) may resolve spontaneously, remain unchanged, or expand and coalesce with other patches to result in alopecia totalis (entire scalp) or alopecia universalis (entire body).

With a single patch of hair loss, there is a 33% chance of hair re-growth in 6 months which will increase to 50% in one year.

Poor prognostic factors for hair re-growth include the following:

Younger age of onset (<10 years of age)


Alopecia totalis
Peripheral rather than central hair loss
Associated nail pitting
Coincidental atopy
Organ-specific autoimmune disease
Persisted hair loss for many years

The chance of relapse is very high (86% in one study), although the time of relapse cannot be predicted.

(Option A) Psoriasis is characterized by erythematous scaly plaques, usually with a shiny silver appearance. In this patient, the scalp is intact; therefore, psoriasis is not likely.

(Option B) Tinea capitis is associated with flaking and hairs of different lengths and different stages of growth, often broken and damaged.

(Option D) Trichotillomania is compulsive pulling out of the hair. Although the scalp appears normal, hairs of different lengths are observed.

(Option E) Discoid lupus erythematosus would have inflammation of the scalp of the affected area and probably scarring.

References

• Therapeutic Guidelines – Dermatology

• Medscape - Alopecia Areata

Last updated:
Time spent: QID:632 2023-2-12

215 of 1943
A 45-year-old woman presents to your clinic concerned about a solitary patch of hair loss sized 5x6 cm. On examination, the scalp of the affected area is completely hairless, but normal-appearing with no scarring, inflammation, or flaking. Which one
of the following is the next best step in management?

A. Hydrocortisone 1% cream.

B. Intradermal injection of triamcinolone.

C. Scraping of the scalp and microscopy after preparation with KOH.

D. Reassure her that the condition is self-limiting and she will not go bald.

E. Clotrimazole cream.

Incorrect. Correct answer is B


45% answered correctly

Explanation:

Correct Answer Is B

Unaffected scalp skin and complete hair loss are suggestive of alopecia areata. Alopecia areata is used for a hair disorder characterized by one or more discrete areas of hair loss. Although hair loss can occur anywhere in the body, hair loss of the
scalp, eyebrow, or beard often brings the patient to medical attention.

The physiopathology of alopecia areata is an autoimmune inflammation of anagen hair bulbs, leading to the cessation of hair growth, but not the destruction of the hair follicle. The etiology of this autoimmune reaction is unknown. Probably, a
genetic factor is involved because 20% of patients have a positive family history of alopecia areata.

The course of the disease is highly variable. Hair growth may occur and patches resolve spontaneously; however, patches may remain unchanged, or expand and coalesce with other patches to result in alopecia totalis (entire scalp) or alopecia
universalis (entire body).

With a single patch of hair loss, there is a 33% chance of hair re-growth in 6 months which will increase to 50% in 12 months.

Alopecia areata is managed as follows:

Intralesional or topical potent corticosteroids are the initial treatment for most patients with patchy alopecia areata.

Intralesional corticosteroid injection has a high probability of success. It is appropriate for eyebrows and small areas on the scalp, but not for wide or multiple scalp or eyelash involvement. Injection of corticosteroids for alopecia of the eyelashes can
cause cataracts, glaucoma, and cutaneous atrophy. The two commonly used preparations for intradermal injections are:

Triamcinolone acetonide 10mg/ml


Betamethasone (acetate + sodium phosphate) 5.7mg/ml

The injections should be 6 weeks apart.

While potent topical corticosteroids are frequently used to treat alopecia areata, evidence for their effectiveness is limited. Topical corticosteroids are usually reserved as first-line therapy for children and adults who cannot tolerate intralesional
injections.

The choices of topical therapy are outlined in the following table:

Very potent
Betamethasone dipropionate 0.05% in optimized vehicle
Potent
Betamethasone dipropionate 0.05%
Betamethasone valerate 0.1%
Methylprednisolone aceponate 0.1%
Mometasone furoate 0.1%
Triamcinolone acetonide 0.1%

For those with extensive lesions (>50% of scalp hair loss) or recurrences, topical immunotherapy is probably the most effective treatment and should be considered first-line. A potent contact allergen is applied to the scalp weekly to precipitate an
allergic contact dermatitis. The consequent mild inflammatory reaction is associated with hair regrowth through an unknown mechanism.

For children younger than 10 years, minoxidil solution and/or topical corticosteroids are considered first.

Dithranol cream can be used for those chronic stable areas of hair loss that may have previously failed to respond to corticosteroid therapy.

(Option A) Hydrocortisone 1% is a mildly potent topical corticosteroid. Mildly potent corticosteroids are not effective in the treatment of alopecia areata.

(Option C) Scraping of the scalp and preparation with KOH for microscopy would be the correct answer if tinea capitis were suspected. Tinea capitis would have flaking and hairs of different lengths as other presenting symptoms in addition to
patches of hair loss.

(Option D) While in a good percentage of affected individuals alopecia areata can persist or even progress to alopecia totalis or universalis, reassurance is not an appropriate option.

(Option E) Clotrimazole cream is used for fungal skin infections. It is not effective against alopecia areata. Even if tinea was the case, this option was incorrect because tinea of hair should be treated with oral antifungal agents.

References

• Therapeutic Guidelines – Dermatology

• UpToDate - Management of alopecia areata

Last updated:
Time spent: QID:633 2023-2-12

216 of 1943
A 17-year-old boy presents to your practice with lesion shown in the accompanying photograph. The lesions have failed to improve with conventional therapy. Which one of the following would be the most appropriate management?

A. Doxycycline.

B. Hydrocortisone 1% cream.

C. Isotretinoin.

D. Benzoyl peroxide.

E. Occlusive makeup.

Incorrect. Correct answer is C


45% answered correctly

Explanation:

Correct Answer Is C

The photograph shows severe cystic acne and scarring. Systemic isotretinoin (Raccutane®) is the treatment of last resort for severe cystic acne or acne with scarring if conventional therapy fails. In this patient with no response to conventional
therapy, systemic isotretinoin is the most appropriate management option.

For mild papulopustular acne, the use of either a topical retinoid or benzoyl peroxide is recommended. If the treatment fails, topical clindamycin or erythromycin is added. Salicylic acid, 3% to 5% in 70% ethanol, can be used for mild truncal acne.

For moderate papulopustular acne +/- trunk involvement +/- nodules, not only the strength of topical retinoids or benzoyl peroxide should be increased, but oral rather than topical antibiotics should be used. Oral doxycycline is the first-line option.
Tetracycline or minocycline can be used alternatively as second- and third-line options, respectively. Erythromycin (250-500mg, twice daily) is the choice if tetracyclines are not tolerated or contraindicated (e.g. in pregnancy).

For moderate to severe acne +/- nodules +/- cysts, topical adapalene or tretinoin plus antibiotics is used. If there is no improvement by 6 weeks or insufficient response by 12 weeks, the antibiotic is changed or the dose is increased. If such measures
fail, referral for systemic treatment with isotretinoin is the most appropriate management.

References

• Therapeutic Guidelines – Dermatology

Last updated:
Time spent: QID:634 2023-2-12

217 of 1943
Which one of the following parts of the nail is more commonly affected by tinea unguium?

A. Nail fold.

B. Nail bed.

C. Nail plate.

D. Cuticle.

E. Entire nail including nail bed.

Incorrect. Correct answer is C


45% answered correctly

Explanation:

Correct Answer Is C

Tinea unguium is the infection of the fingernails or toenails caused by dermatophytes. Generally, fungal infection of the nails is termed onychomycosis (plural: onychomycoses). Onychomycoses which are specifically caused by dermatophytes are
referred to as tinea unguium. There are three distinctive presentations:

Distal subungual

This presentation is the most common form of fungal nail infection. The most common culprit to this presentation is Trichophyton rubrum. It begins with a whitish, yellowish or brownish discoloration of the distal corner of the nail, which gradually
spreads to the entire width of the nail plate and then slowly extends toward the cuticle.

Proximal subungual

The most common cause is Trichophyton rubrum. It presents with whitish, yellowish, and brownish discoloration of the nail, starting from around the cuticle and spreading distally.

White superficial

The most common cause is Trichophyton mentagrophytes. It starts with a dull white spot on the surface of the nail plate. The whole nail plate may eventually be involved. The white areas are soft and can be scraped gently to obtain a sample.

In all three types, the nail plate is the often affected part.

References

• Medscape - Onychomycosis

Last updated:
Time spent: QID:635 2023-2-12

218 of 1943
A 15-year-old boy presents to your practice with a lesion shown in the following photograph. The lesion is itchy. Which one of the following is the most likely diagnosis?

A. Psoriasis.

B. Tinea of the face.

C. Seborrheic dermatitis.

D. Allergic reaction.

E. Tinea versicolor.

Incorrect. Correct answer is B


45% answered correctly

Explanation:

Correct Answer Is B

The photograph shows a circular erythematous scaly lesion with a central clearing. This picture along with the itchiness as the main symptom is highly suggestive of tinea infection of the face (Tinea faciei) as the diagnosis.

Tinea is caused by dermatophytes - fungi that affect any part of the skin. The typical rash of tinea has the following characteristic features:

Annular or arcuate (arc-shaped)


Scaly and itchy
Definite edges and central clearing as it expands

(Option A) Psoriasis is a systemic disease. Although the rashes of psoriasis may resemble tinea at some stages, they tend to appear bilaterally and in a symmetrical fashion. In addition, silvery scaling is often present; however, scaling may be absent
in some forms of psoriatic lesions. Central clearing is not a feature of psoriatic rash.

(Option C) Dermatitis may present similar to tinea infection, but central clearing makes tinea a more likely diagnosis.

(Option D) Allergic drug reactions can cause various clinical pictures such as urticaria, different types of rash, and dermatoses. The photograph however illustrates a typical case of tinea.

(Option E) Tinea versicolor is a common skin fungal infection caused by Malassezia furfur. It presents with a well-demarcated macular rash that is hyperpigmented or hypopigmented and slightly itchy. It is more commonly seen in the upper trunk,
however, it may involve the whole trunk, neck, and even upper limbs. Tinea versicolor lesions do not have central clearing.

Tinea versicolor - note the widespread hypopigmented lesions

References

219 of 1943
• Medscape - Tinea Faciei

• Therapeutic Guidelines – Dermatology


Last updated:
Time spent: QID:636 2023-2-12

220 of 1943
An 8-year-old boy is brought to your practice with a complaint of itchy lesions shown in the following. He has been recently prescribed cotrimoxazole. Which on of the following could be the most likely diagnosis?

A. Tinea infection.

B. Psoriasis.

C. Contact dermatitis.

D. Allergic drug reaction.

E. Tinea versicolor.

Correct
45% answered correctly

Explanation:

Correct Answer Is A

The photograph shows multiple round lesions of different sizes over the face, chest, abdomen, and arm. The lesions have prominent circular edges with centers being clear. With itchiness in the history, dermatophytic infection (tinea) is the most
likely diagnosis.

Tinea is caused by dermatophytes fungi that affect any part of the skin. The typical rash of tinea has the following characteristic features:

Annular or arcuate (arc-shaped)


Scaly and itchy
Definite edges and central clearing as it expands

(Option A) Psoriasis is characterized by erythematous patches distributed symmetrically. A unique feature of psoriasis is the silvery scaling of the patches absent here. Moreover, psoriatic lesions do not have central clearing.

(Option C) Contact dermatitis can cause an itchy scaly erythematous rash without central clearing.

(Option D) Allergic reactions to the medication (e.g. cotrimoxazole) can cause various clinical pictures, including an itchy erythematous rash but without central clearing. Wheels (hives) have clear centers, but the lesion's appearance differs from
those shown in the photograph. There is usually no scaling.

(Option E) Tinea versicolor presents with hypo/hyperpigmented macules of different sizes with slight scaling and itching. There is no central clearing of the lesions.

References

• Medscape - Tinea Corporis

• Therapeutic Guidelines

Last updated:
Time spent: QID:637 2023-2-12

221 of 1943
A 23-year-old woman presents to your GP clinic with a rash on her back. The rash appeared two months ago as a 3x4 cm oval patch on her upper back that later on was followed by an eruption of smaller spots with occasional itching. The spots are
paler than the surrounding skin. She has tried tanning oils and salons but the rash persists. On examination, there are multiple 3- to 5-mm macules on her back distributed in a ‘Christmas tree pattern. Which one of the following is the most likely
diagnosis?

A. Lichen simplex chronicus.

B. Pityriasis rosea.

C. Allergic contact dermatitis.

D. Atopic dermatitis.

E. Tinea corporis.

Incorrect. Correct answer is B


45% answered correctly

Explanation:

Correct Answer Is B

The clinical picture and the course of symptoms are highly suggestive of pityriasis rosea (PR) (fine pink scale) is a common skin disorder seen in otherwise healthy people, especially in children and young adults. The cause is thought to be a viral
infection (possibly herpes virus 6 or 7).

The eruption begins with a 'herald patch' that may be mistaken for tinea. This rash is then followed in approximately 2 weeks by the development of multiple, scaly, salmon-colored macules, each about 1 to 2 cm in size and oval in shape. The eruption
is symmetric and most commonly involves the chest, back, abdomen, and adjoining areas of the neck and limbs. Lesions are not usually seen on the face, hands, and feet. The lesions are arranged along the skin creases to form an appearance
similar to a Christmas tree. The rash may be itchy.

The herald patch of pityriasis rosea

The herald patch of pityriasis rosea and subsequent evolving macules of different sizes

222 of 1943
Christmas Tree appearance of pityriasis rosea

Pruritus is commonly evident in 25-75% of patients and is usually of mild-to-moderate severity. Secondary eczematous changes can occur if pruritus is severe. A small number (approximately 5%) of patients with PR have mild prodromal symptoms
(e.g. malaise, fatigue, headache, nausea, anorexia, chills, fever, and arthralgia) that precede the appearance of the herald patch. Lymphadenopathy may occur before the onset of the rash.

As a rule, PR only needs symptomatic management. Menthol is often effective for mild itching. Moderately potent corticosteroids are used for more severe cases of itching. Exposure to sunlight may be effective for the alleviation of itching as well as
hastening the resolution of the rash.

The condition resolves spontaneously in 6-8 weeks in most cases, but some lesions may persist for as long as 3-4 months. With the resolution of the eruption, post-inflammatory pigment change can be observed. Both hypopigmentation and
hyperpigmentation can follow the rash. Dark-skinned people may experience hyperpigmentation that takes a few months to resolve.

PR-like eruptions are differential diagnoses. This condition is associated with many drugs such as acetylsalicylic acid, barbiturates, bismuth, captopril, clonidine, gold, imatinib, isotretinoin, ketotifen, levamisole, and metronidazole, as well as vaccines
such as BCG, HPV, and diphtheria.

References

• DermNet NZ- Pityriasis rosea

• Medscape - Pityriasis Rosea

• Therapeutic Guidelines – Dermatology

Time spent: QID:638 Last updated:


2023-2-12

223 of 1943
Jane, 21 years old, presents with a painless lesion on the distal interphalangeal joint of her right middle finger. The lesion has been present for the past 2 months and increased in size. The lesion is shown in the following photograph. On examination,
she is otherwise healthy. Which one of the following is the most likely diagnosis?

A. Abscess.

B. Pyoderma gangrenosum.

C. Pyogenic granuloma.

D. Basal cell carcinoma.

E. Epidermoid cyst.

Incorrect. Correct answer is C


45% answered correctly

Explanation:

Correct Answer Is C

The fleshy red lesion on the distal interphalangeal joint of this patient is consistent with the diagnosis of pyogenic granuloma (PG). PG is a common benign acquired vascular neoplasm of the skin and mucous membrane. The etiology is unknown.
Some think it might be due to minor trauma, especially for PGs of fingers. Pyogenic granuloma is a misnomer because the lesion is neither pyogenic nor granuloma.

It is an erythematosus (fleshy red) dome-shaped papule or nodule that bleeds easily and is prone to ulceration, erosion, and crusting. It is most often seen in adolescents and young adults. Up to 5% of pregnant women may experience it.

Although very rare, PGs have been reported in the gastrointestinal tract, the larynx, nasal mucosa, conjunctiva, and cornea.

(Options A, D, and E) Abscess, epidermoid cyst, and mucous cyst are covered with skin.

(Option B) Pyoderma gangrenosum typically presents with a well-defined border, which is usually violet or blue.

References

• DermNet NZ - Pyogenic granuloma

• Medscape - Pyogenic Granuloma (Lobular Capillary Hemangioma)

Last updated:
Time spent: QID:37 2023-2-12

224 of 1943
A 45-year-old woman presents to your GP clinic for review. A few months ago, she developed redness around her nose and cheeks that became worse after she drank alcohol. Recently, she was asked, by one of her colleagues at work, if she has
alcohol problems because her appearance resembles those with excessive alcohol use. She drinks an average of 10 units of alcohol per week. Her facial appearance is shown in the accompanying photograph. Which one of the following is the most
likely diagnosis?

A. Seborrheic dermatitis.

B. Erysipelas.

C. Acne rosacea.

D. Systemic lupus erythematous.

E. Alcohol-related skin disease.

Incorrect. Correct answer is C


45% answered correctly

Explanation:

Correct Answer Is C

The photograph shows an erythematous butterfly rash. Additionally, the chin is also involved. Several papules and pustules are noted as well. The clinical picture presented in the photograph, along with the history, is suggestive of rosacea (also
known as acne rosacea) as the most likely diagnosis.

Rosacea is a common chronic disorder that mainly involves the face. It tends to present in middle-aged people but may start at earlier ages. It is more common in cold climates.

The condition often begins as an exaggerated or prolonged flushing tendency, with erythema affecting the central face or the butterfly area in particular. Sometimes erythema can be seen in the chin and forehead. Initially, the erythema occurs
intermittently but later becomes persistent. Sometimes, rosacea is associated with edema. Telangectiasis is often present. Sterile inflammatory papules, pustules, and nodules may be present mimicking acne. The distinguishing feature is the
absence of comedones.

Patients often complain that their faces feel hot, burn, sting or itch. The patient often reports that their face is increasingly easily irritated by topical products.

The etiology of rosacea is unknown. Alcohol is not a cause but it can trigger the flushing and worsen the symptoms. Other triggering factors include:

Hot or cold temperatures


Wind
Hot drinks
Caffeine
Exercise
Spicy foods
Emotions
Topical products that irritate the skin and decrease the barrier
Medications that cause flushing

In about 50% of the patients, blepharoconjunctivitis is observed. It presents with itching, burning, grittiness or foreign body sensation in the eye and erythema and swelling of the eyelid. More advanced cases can develop enlarged sebaceous glands
and connective tissue changes resulting in a bulbous, rhinophymatous nose.

Seborrheic dermatitis, SLE, and erysipelas are among the differential diagnosis but not consistent with the history. None of these conditions have alcohol as a triggering factor.

(Option A) Seborrhoeic dermatitis has scaling of the skin as a prominent feature.

(Option B) Erysipelas is associated with the abrupt onset of erythematous butterfly rash almost always caused by streptococcus pyogenes. There is often fever and constitutional upset. It is not a chronic condition and is not triggered by alcohol use.

(Option D) Although the rash resembles that of SLE, the lack of other signs and symptoms favoring SLE makes it least likely.

(Option E) Alcohol-related skin disease is a general term, not a diagnosis. Rosacea worsens by alcohol but is not caused by it.

References

225 of 1943
• DermNet NZ - Rosacea

• RACGP - AFP- Rosacea


Last updated:
Time spent: QID:682 2023-2-12

226 of 1943
A 17-year-old girl presents to your GP practice with redness and swelling of the sun-exposed areas of her face after he spent 60 minutes in the sun. She is on treatment for acne. On examination, erythema and edema of the sun-exposed areas of her
face, neck and upper chest are noted. There are bullae all over the area. Which one of the following is the most likely diagnosis?

A. Doxycycline phototoxicity.

B. Allergic contact dermatitis.

C. Erythromycin phototoxicity.

D. Stevens – Johnson syndrome.

E. Benzoyl peroxide phototoxicity.

Correct
45% answered correctly

Explanation:

Correct Answer Is A

The scenario describes a typical case of photosensitivity associated with medication, namely drug-associated photoeruption.

There are two basic types of photoeruptions, which differ in clinical appearance and pathogenesis:

Phototoxic eruptions — by far, phototoxic eruptions are the most common drug-induced photoeruptions. They are caused by absorption of ultraviolet light by the
causative drug resulting in cell damage. Ultraviolet A light (UVA) is the most common wavelength implicated; ultraviolet B light (UVB) and visible light can elicit
reactions with some drugs. The eruption is typically an exaggerated sunburn, often with blisters. The following drugs are the most common causes of phototoxic
eruptions:

NSAIDs
Quinolones
Tetracyclines (e.g.doxycycline)
Sulfonamides
Phenothiazines

Photoallergic eruptions — photoallergy is a lymphocyte-mediated reaction caused by exposure to UVA. It is postulated that the absorbed radiation converts the drug into an immunologically active compound that is then presented to lymphocytes,
causing a spongiotic dermatitis (eczema). The eruption is characterized by widespread eczema in photo-exposed areas: face, upper chest, and back of the hands.

Most photoallergic reactions are caused by topical agents including biocides added to soaps (halogenated phenolic compounds) and fragrances such as musk ambrette and 6-methyl coumarin. Systemic photoallergens such as the phenothiazines,
chlorpromazine, sulfa products, and NSAIDs can produce photoallergic reactions; however, most of their photosensitivity reactions are phototoxic.

The characteristic skin lesion in this patient (edema, erythema and blistering) suggests a phototoxic eruption to one of the medications she is on for treatment of acne. Of the options, doxycycline (a tetracycline) is most likely to have caused this
presentation.

(Option B) Allergic contact dermatitis, as the name implies, require skin contact with an specific allergen to trigger a cutaneous immune response, most often characterized by eczema (spongiotic dermatitis). Moreover, contact dermatitis is not
associated with UV exposure.

(Option D) Stevens – Johnson syndrome, sometimes called erythema multiforme major, is a hypersensitivity response that can be triggered by the same drugs mentioned above, but with a different presentation. Furthermore, UV exposure does not
play a role. Stevens – Johnson syndrome presents with epidermal and muscosal necrosis and sloughing of the skin and mucous membrane.

(Options C and E) Benzoyl peroxide and erythromycin have not shown proven association with photoeruptions.

References

• UpToDate - Drug eruptions

• Medscape - Drug-Induced Photosensitivity

Time spent: QID:711 Last updated:


2023-2-12

227 of 1943
A 2-year-old boy is brought to the emergency department, by his mother, after he developed a rash following a bee sting 20 minutes ago. The appearance of the rash is shown in the following photograph. On examination, there is no breathlessness,
wheezing, abdominal pain, or hemodynamic instability. Which one of the following is the most appropriate management?

A. Intravenous epinephrine.

B. Intravenous diphenhydramine.

C. Intramuscular epinephrine.

D. Oral promethazine.

E. Oral corticosteroids.

Incorrect. Correct answer is D


45% answered correctly

Explanation:

Correct Answer Is D

The picture shows generalized urticaria (hives) of the trunk. The rash also has involved the buttocks and arms but not the eyelids. In approach to patients with immediate hypersensitivity, the initial assessment should be focused on airway, breathing
and circulation (ABC).

In the presence of any of the following situations, intramuscular epinephrine is always the most appropriate immediate management:

Any acute onset of hypotension or bronchospasm or upper airway obstruction, even if typical skin features are not present.
Any acute-onset illness with typical skin features (urticarial rash or erythema/flushing, and/or angioedema) PLUS Involvement of respiratory and/or cardiovascular and/or persistent severe gastrointestinal symptoms.

NOTE - An adrenaline autoinjector (EpiPen® or Anapen®) may be used instead of an adrenaline ampoule and syringe. For children 10–20 kg (aged 1–5 years) EpiPen Junior® or Anapen Junior® should be used. Intravenous boluses of
epinephrine are not recommended without specialized training, as they may increase the risk of cardiac arrhythmias.

In the absence of the above mentioned presentations, management revolves around treatment of urticaria (and not anaphylaxis). Oral antihistamines (e.g. promethazine) are mainstay of therapy in most cases. Intravenous route is used when the
urticaria is severe or eyelids are involved. If there is no response to antihistamines, oral corticosteroids are considered.

References

• Therapeutic Guidelines

• ASCIA Guidelines - Acute management of anaphylaxis

Last updated:
Time spent: QID:740 2023-2-12

228 of 1943
Jane, 53 years, presents to your GP practice with facial weakness, right ptosis, and drooling from the right corner of her mouth since this morning. She mentions a history of right ear pain a few days earlier. On examination, an erythematous vesicular
rash is noted in the right ear canal. Which one of the following would be the most appropriate next step in management?

A. Oral aciclovir.

B. Oral prednisone.

C. Methotrexate.

D. Clindamycin.

E. Gabapentin.

Correct
45% answered correctly

Explanation:

Correct Answer Is A

The clinical findings and history suggest Ramsay Hunt syndrome as the most likely diagnosis. The syndrome is defined as an acute peripheral facial neuropathy caused by the reactivation of a latent varicella-zoster virus infection. The syndrome is
associated with an erythematous vesicular rash of the skin of the ear canal, auricle (also known as herpes zoster oticus), and/or oropharyngeal mucous membrane.

Varicella zoster virus (VZV) infection causes two distinct clinical syndromes. Primary infection, also known as varicella or chickenpox, is a common pediatric viral disease. After a chickenpox infection, VZV remains latent in the neurons of cranial
nerves and dorsal root ganglia. Subsequent reactivation of latent VZV can result in a vesicular rash in a dermatomal distribution, known as herpes zoster. VZV infection or reactivation involving the geniculate ganglion of the 7th cranial nerve (CN VII)
within the temporal bone is the main pathophysiological mechanism of Ramsay-Hunt syndrome.

Ramsay-Hunt syndrome is estimated to account for 16% of all causes of unilateral facial palsies in children, and 18% of facial palsies in adults. Ramsay Hunt syndrome is thought to be the cause of as many as 20% of clinically diagnosed cases of
Bell palsy. Although VZV reactivation is the cause in most cases of this syndrome, herpes simplex virus (HSV) II has been implicated in some cases.

The syndrome usually presents with paroxysmal pain deep within the ear. The pain often radiates outward into the pinna of the ear and may be associated with a more constant, diffuse, and dull background pain. The onset of pain usually precedes
the rash by several hours or even days.

The classic triad includes (1) ipsilateral facial palsy, (2) ear pain, and (3) vesicles in the auditory canal and auricle. In some patients taste perception, hearing (tinnitus, hyperacusis), and lacrimation may occur.

Ramsay-Hunt syndrome is generally considered a polycranial neuropathy, with frequent involvement of cranial nerves V, IX, and X. With the involvement of the geniculate ganglion, the inflammation can spread to the 8th cranial nerve (CN VIII),
resulting in auditory and vestibular disorders.

Clinical manifestations of Ramsay Hunt syndrome may include:

Vesicular rash of the ear or mouth (as many as 80% of cases)


The rash might precede the onset of facial paresis/palsy (involvement of the seventh cranial nerve [CN VII])
Ipsilateral lower motor neuron facial paresis/palsy (CN VII)
Vertigo and ipsilateral hearing loss (CN VIII)
Tinnitus
Hyperacusis
Otalgia (ear pain)
Headaches
Dysarthria
Gait ataxia
Fever
Cervical lymphadenopathy
Facial weakness usually reaches maximum severity by one week after the onset of symptoms.
Other cranial neuropathies might be present and may involve cranial nerves (CNs) VIII, IX, X, V, and VI.
Ipsilateral hearing loss has been reported in as many as 50% of cases.
Blisters of the skin of the ear canal, auricle, or both may become secondarily infected, causing cellulitis.

Antiviral treatment should be started in all patients within 72 hours of the onset of the rash. This time limit does not apply to patients with herpes zoster ophthalmicus and immunocompromised patients, meaning that such patients should receive
antiviral therapy even beyond 72 hours of presentation.

Treatment options include famciclovir, valaciclovir, and aciclovir.

NOTE - Aciclovir is the preferred medication in children and pregnant women.

For this patient, oral acyclovir is the most appropriate option to consider as the next step in management.

(Option A) Therapy with oral corticosteroids (e.g. prednisolone daily for 7 days, tapering to 5 mg daily over the next 2 weeks) can reduce symptoms in the acute (inflammatory) phase of herpes zoster, but only when used in combination with an
antiviral agent. Corticosteroids are recommended for patients over the age of 50 if not contraindicated and should be used with caution in patients with comorbid conditions such as diabetes.

(Option C) Methotrexate has no role in the treatment of herpes zoster.

(Option D) Antibiotics other than mentioned antiviral agents are ineffective in treating herpes zoster.

(Option E) Gabapentin and tricyclic antidepressants are the most effective treatment options for neuropathic pain associated with herpes zoster infection, but given the time of presentation, antiviral therapy should be considered as the cornerstone
of therapy first.

References

• Medscape - Ramsay Hunt Syndrome

• DermNet NZ - Ramsay Hunt Syndrome

• UpToDate - Epidemiology, clinical manifestations, and diagnosis of herpes zoster

L t d t d

229 of 1943
Last updated:
Time spent: QID:819 2023-2-12

230 of 1943
A 65-year-old man presents with an exquisitely painful vesicular rash, which has been present for the past 7 days and is increasingly painful. The rash is shown in the following photograph. His past medical history is unremarkable. Physical
examination is otherwise inconclusive. Which one of the following is the most appropriate immediate management?

A. Oral famciclovir.

B. Intravenous famciclovir.

C. Oral amitriptyline.

D. Intramuscular immunoglobulin.

E. Oral Phenergan® (promethazine).

Incorrect. Correct answer is C


45% answered correctly

Explanation:

Correct Answer Is C

The appearance of the rash and the history suggest herpes zoster (shingles) infection as the most likely diagnosis.

Herpes zoster (shingles) is caused by the reactivation of the varicella-zoster virus (VZV). It usually occurs in adults but can be seen in children and in the first 2 years of life if there has been a history of maternal varicella.

Pain is a significant complaint in patients with shingles. Tricyclic antidepressants and the anticonvulsant gabapentin are the most effective medications for pain control in neuropathic pain associated with shingles.

(Options A and B) Antiviral agents such as famciclovir, valacyclovir, or acyclovir should be used in any patient seen within 72 hours of the onset of vesicles, all patients with ophthalmic herpes zoster, and in immunocompromised patients. For
pregnant women, only acyclovir (first line) or valaciclovir (second line) are used because the safety of famciclovir during pregnancy has yet to be established.

(Option D) Varicella zoster immunoglobulin (VZIG) and vaccine are used for prophylaxis. Their use is not effective in the treatment or for pain control after the infection has established.

(Option E) Promethazine is not effective for pain control in shingles.

References

• Medical Journal of Australia - The prevention and management of herpes zoster

• Therapeutic Guidelines – Antibiotics

• Therapeutic Guidelines – Neurology

Last updated:
Time spent: QID:820 2023-2-12

231 of 1943
A 60-year-old man presents to your clinic with complaints of fever and a painful swollen right thigh. He has diabetes well controlled on metformin 500mg 8-hourly and hypertension for which he is taking valsartan 80mg, daily. He is on atorvastatin 20
mg daily for hypercholesterolemia as well. His recent medical history is remarkable for deep vein thrombosis (DVT) of his right leg, for which he is currently on warfarin. One week ago, he was admitted to the hospital after one episode of syncope
and started on amiodarone after he was diagnosed with ventricular tachycardia (VT). Physical examination is remarkable for a temperature of 38.3°C and a warm swollen tender right thigh that is erythematous. The right thigh circumference is 3 cm
greater than that of the left thigh. Which one of the following could be the most likely cause of this presentation?

A. Deep vein thrombosis (DVT).

B. Cellulitis.

C. Drug interaction.

D. Hematoma.

E. Rhabdomyolysis.

Incorrect. Correct answer is B


45% answered correctly

Explanation:

Correct Answer Is B

At first glance, there are a number of differential diagnoses for a swollen painful thigh including DVT, cellulitis, and hematoma. This patient is on warfarin for treatment of a previous DVT, which is a risk factor for another venous thromboembolic
event (VTE) such as another DVT or pulmonary embolism (PE). However, it is less likely, yet not impossible, for this patient to develop another VTE while on treatment with warfarin. Therefore, DVT of the right leg (option A) is a weak possibility.

The interaction between warfarin and amiodarone is well-known. Amiodarone results in the decreased metabolization of warfarin through hepatic pathways and leads to increased bleeding tendency. Considering that the patient has been recently
started on amiodarone while on warfarin, a hematoma (option D) can also be a possibility. However, the presence of fever, warmth, and erythema of the swelling makes this diagnosis less likely. Pressure from a hematoma on surrounding tissues
such as skin or deeper layers (e.g. muscles can compromise circulation and results in complications such as necrosis or a superimposed infection that can justify, to some extent, the clinical picture; however, the expected sequence would be
hematoma formation first and then the development of signs such as tenderness, warmth, and erythema later. It is also worth reminding the fact that hematomas present as bruises or bluish not red discoloration. Hematomas are not warm and are
often non-tender. Swelling is a feature though.

Drug interaction (option C) between warfarin and amiodarone can result in increased bleeding tendency and the development of a hematoma, and interaction between atorvastatin and amiodarone can lead to an increased potential for the
development of statin-related muscle problems. None of these events are associated with the given presentation and are very unlikely to be the cause of this clinical picture.

Rhabdomyolysis (option E) is a serious condition followed by muscle fibers breakdown and release of muscle cell contents such as myoglobin and potassium. Myoglobinuria results in the deposition of myoglobin in the kidneys and renal failure.
Release of excess potassium from damaged muscle cells leads to hyperkalemia and serious complications such as cardiac arrhythmias. Statins such as atorvastatin and simvastatin in particular are metabolized by cytochrome P450-3A4 (CYP3A4),
and amiodarone is a potent inhibitor of this cytochrome. Concomitant use of amiodarone and statins can result in increased activity of statins and rises the likelihood of statin-related adverse effects such as muscular pain, myopathy, and rarely
rhabdomyolysis. However, in the event of statin-induced rhabdomyolysis, a systemic presentation is expected. Rhabdomyolysis never causes the focal signs mentioned in the scenario.

Of the given options, this clinical picture is most consistent with cellulitis of the left thigh as the most likely diagnosis. Cellulitis presents with systemic signs such as fever and focal findings of induration, warmth, redness, and tenderness of the
affected area. Diabetes can be a risk factor.

References

• Medscape - Cellulitis

• RACGP - Warfarin: indications, risks and drug interactions

Last updated:
Time spent: QID:1208 2023-2-12

232 of 1943
John, 60 years of age, presents to your office because he is concerned about a skin lesion on his forearm. The dermatoscopic view of the lesion is shown in the following photograph. Which one of the following could be the most likely diagnosis?

A. Actinic keratosis.

B. Bowen’s disease.

C. Malignant melanoma.

D. Dermatosis papulosa nigra.

E. Seborrheic keratosis.

Incorrect. Correct answer is E


45% answered correctly

Explanation:

Correct Answer Is E

The photograph shows an oval brown plaque on a sun-exposed area in an old man. There is also keratotic plugging in the surface of the lesion (yellowish-colored). These features are characteristic of seborrheic keratosis as the most likely diagnosis.

Seborrheic keratosis (SK) is the most common benign skin tumor in older adults with the incidence increasing with age. It is typically oval or round and brown lesions mostly in sun-exposed areas with adherent greasy scales. The color may range
from black to tan. Usually, SK has a velvety verrucous (warty) surface and appears stuck on. However, some lesions are so verrucous that may resemble warts.

(Option A) Actinic keratosis is characterized by flat small scaly lesions on the sun-exposed area of old adults, especially the back of the hands. Lesions can be multiple. Actinic keratosis is precancerous with the potential to become squamous cell
carcinoma.

Multiple actinic (solar) keratoses on the back of the hand and forearm

233 of 1943
Actinic keratosis - Note the flat appearance and scales of the lesion

(Option B) Bowen’s disease, similar to AK, is seen in the sun-damaged area of skin in older adults and is precancerous. It is flat and scaly such as AK but is larger in diameter and tends to be thicker with well-demarcated borders.

Bowen disease - similar to actinic keratosis but


larger and well-demarcated

Bowen disease

(Option C) Malignant melanoma commonly presents with a rather flat lesion of various colors and irregular borders and shapes. Although nodular melanomas can present with a pigmented dome-shaped lesion, the commonality of SK, the presence
of keratotic plugs, and the lack of color variety make malignant nodular melanoma a far less likely diagnosis compared to SK.

234 of 1943
Nodular melanoma - note the different colors and lack of
keratotic plugs which differentiate this lesion from seborrheic keratosis

Nodular melanoma

(Option D) Dermatosis papulosa nigra is a variant of SK consisting of multiple brown-black dome-shaped, small papules found in the face of young to middle-aged people with colored skin, especially black. This single lesion on the forearm of an old
man is SK not its variant dermatosis papulosa nigra.

Dermatosis papulosa nigra

NOTE – Stucco keratosis is another variant of SK that consists of large numbers of superficial grey-to-light brown flat keratotic lesions usually on the tops of the feet, ankle, and the back of the hands and forearms.

References

• Medscape - Seborrheic Keratosis

Time spent: QID:1447 Last updated:


2023-2-12

235 of 1943
The 7-year-old Evie is brought to your general practice by her mother who is concerned about a rash she has developed and is shown in the following photograph. The rash appeared almost two months ago on her abdomen and has spread to the
extent evident in the photo. There is no complaint of pain or itching, but a few lesions have ulcerated (one shown in the photograph). Evie is otherwise healthy and has not had any serious illnesses so far. Her growth charts are all normal. Physical
examination is otherwise normal. She goes to school and has two younger brothers and sisters at home. Which one of the following is the correct advice to give?

A. She should be excluded from school until the resolution of the rash.

B. She can go to school but should use separated towel and other personal items.

C. She should be given oral acyclovir.

D. Cryotherapy with liquid nitrogen is the treatment of choice for the rash.

E. She should use topical steroids.

Incorrect. Correct answer is B


45% answered correctly

Explanation:

Correct Answer Is B

The lesions shown in the photograph are pearly dome-shaped papules with central umbilication characteristic of molluscum contagiosum as the most likely diagnosis. Molluscum contagiosum is a common viral infection of childhood caused by
molluscipoxvirus a member of the poxvirus family.

The infection presents with firm, smooth, spherical papules that are pearly white and have a central dimple (umbilicus). Most papules range from 1 to 3 mm; however, lesions of up to 1-2 cm have been reported mostly due to coalescing smaller
lesions.

Seeking medical attention can be due to concerns about the rash itself, or more frequently the development of eczema in the surrounding skin. The latter can make a diagnostic challenge because eczema can obscure the primary lesions.

Lesions can develop anywhere, but flexures and areas of friction are more frequently involved. Lesions may also occur in the anogenital area and are not usually associated with sexual abuse of the child. Involvement of the eyelid margins may lead
to chronic conjunctivitis.

Molluscum contagiosum has a benign course and nature. Most of the patients experience the spontaneous resolution of the lesions within three to six months, but on occasion, it may take up to three years.

In most cases, active treatment is not recommended because spontaneous resolution usually occurs upon activation of the immune response which may take from three months to three years. For individual lesions, removal by cryotherapy (option
D) or curettage may be considered but is not generally recommended as a routine treatment for the lesions.

Children with molluscum contagiosum do not require isolation or exclusion from school (option A). however, it is better to avoid sharing towels or bathing together as these may increase the risk of spread to their brothers and sisters or other
children.

(Option C) Active topical or oral antiviral therapy, such as acyclovir, is unnecessary and recommended.

Option E) Short-term use of a topical steroid for the treatment of associated eczema is recommended. There is no associated eczema in the photograph, so topical steroids are not indicated.

NOTE - Inflamed lesions do not require antibiotics either orally or topically.

References

• RCH – Molluscum

• Health – VIC – Molluscum contagiosum

Time spent: QID:1468 Last updated:


2023-2-12

236 of 1943
A 51-year-old man presents to the hospital with a swollen hand shown in the following photograph. He was gardening in his courtyard yesterday when his left index finger was injured. This morning, he noticed that his left hand is tender, swollen, and
red. On examination, he has a blood pressure of 134/78 mmHg, a pulse of 88, a respiratory rate of 16 breaths/min, and a temperature of 38.1°C. Which one of the following is the most appropriate next step in management?

A. Ultrasound of the hand.

B. MRI of the hand.

C. Admission to the hospital and commencement of intravenous antibiotics.

D. CT scan of the hand.

E. Discharge him home on oral antibiotics and review in 24-48 hours.

Incorrect. Correct answer is E


45% answered correctly

Explanation:

Correct Answer Is E

The swollen, red, and tender hand following a recent injury makes cellulitis the most likely diagnosis. Cellulitis is a clinical diagnosis and no imaging studies such as ultrasound (option A), MRI (option B), or CT scan (option D) are required.

Streptococcus pyogenes (S.pyogenes) or other Streptococcus species such as group B,C, or G are the most common causes of nonpurulent, recurrent (e.g., associated with lymphedema), or spontaneous rapidly spreading cellulitis. Staphylococcus
aureus is less common for cellulitis and is often associated with penetrating trauma, ulceration, or an abscess. Purulent cellulitis is typically caused by Staphylococcus aureus.

Antibiotics are the mainstay of treatment. Oral antibiotics are adequate for cellulitis not associated with systemic features and are usually appropriate for patients with a single systemic feature of infection (e.g., fever alone).

For adults with two or more of the following clinical features, admission to the hospital and commencement of intravenous antibiotics (option C) should be used initially:

Temperature more than 38°C or less than 36°C


Heart rate of more than 90 bpm
Respiratory rate of more than 20 breaths/minute
White cell count more than 12x109/L or less than 4x109/L, or more than 10% immature (band) forms

Other indications for initial intravenous antibiotics (even without having 2 or more of the above) are:

Intolerance of oral therapy


Immunocompromised patients
Comorbidities that could be associated with the rapid progression of cellulitis such as diabetes mellitus
Early follow-up is not possible

Patients who are discharged on oral antibiotics require early assessment (within 24-48 hours) for treatment response.

For this patient with a fever of 38.1°C as the only concerning factor besides the cellulitis, discharging home on oral antibiotics with an early review is the most appropriate option to consider at this stage. Intravenous antibiotics should be considered
next if the response to oral therapy is unsatisfactory.

Other advice to give this patient is rest and elevation of the affected area.

References

• Therapeutic Guidelines – Antibiotics

Time spent: QID:1654 Last updated:


2023-2-12

237 of 1943
A 27-year-old man presents with extremely itchy rash on his buttocks and elbows as shown in the following photograph. He has long-standing history of celiac disease. He was also diagnosed with major depression seven months ago for which he is
under treatment with Zoloft. Which one of the following could be the most likely diagnosis?

A. Linear IgA dermatosis.

B. Neurotic excoriation.

C. SSRI-induced dermatitis.

D. Scabies.

E. Dermatitis herpetiform.

Incorrect. Correct answer is E


45% answered correctly

Explanation:

Correct Answer Is E

The photograph shows papules and vesicles of which some are eroded. Presence of such features, the clinical picture, and the presence of long-standing celiac disease in history make dermatitis herpetiform the most likely diagnosis. Dermatitis
herpetiform is a rare cutaneous condition presenting with persistent immunobulous disease with a strong link to celiac disease.

The condition is called herpetiform because blisters/vesicles develop in clusters resembling herpes simplex.

Dermatitis herpetiformis (aka Duhring-Brocq disease) is a rare skin condition presenting with persistent immune-related bullous formation. The name herpetiformis is derived from the tendency for blisters to appear in clusters, resembling
herpes simplex. However, dermatitis herpetiformis is not due to viral infection.

Dermatitis herpetiform and celiac disease are linked as both are due to intolerance to the gliadin fraction of gluten. Gluten triggers production of IgA and an autoimmune response that targets the intestine and skin. Over 90% of patients with
dermatitis herpetiform have gluten-sensitive enteropathy and 15-25% of patients with celiac disease have concurrent dermatitis herpetiform. These patients tend to have a more severe intestinal pathology in comparison to those without dermatitis
herpetiformis.

Dermatitis herpetiformis symmetrically affects scalp, shoulders, buttocks, elbows, and knees. Lesions come in form of extremely itchy papules and vesicles on normal-looking or erythematous skin. Lesions form in clusters (herpes simplex-like).
Vesicles are often unroofed and flat due to scratching by the patient.

Other forms in which dermatitis herpetiform can present are digital petechia or flat red patches, or wheels imitating other skin conditions such as scabies (option D), dermatitis, or linear IgA dermatosis (option A).

Linear IgA dermatosis (LAD) is an autoimmune subepidermal vesiculobullous disease that may be idiopathic or drug-induced. The clinical presentation is heterogeneous and appears similar to other blistering diseases, such as bullous pemphigoid
and dermatitis herpetiform. The list of factors implicated in development of LAD continues to grow; however, vancomycin is the most documented agent causing LAD thus far.

Dermatitis herpetiform can cause hypopigmentation or hyperpigmentation after resolution.

In neurotic excoriation (option B), skin abnormalities are caused by traumatizing effects of scratching on skin. In this case presence of vesicles make such diagnosis unlikely.

SSRIs can cause an increase serotonin concentration in skin which results in allergic reaction presenting with pruritus and other skin abnormalities including macular rash, xanthoma, purpura, etc. However, given the history of celiac disease and the
presence papules and vesicles on elbows and buttocks make celiac disease more likely than SSRI-induced dermatitis (option C).

References

• DermNet NZ- Dermatitis Herpetiform

Last updated:
Time spent: QID:1708 2023-2-12

238 of 1943
A 61-year-old woman presents to your practice complaining of a rash on her left shin. On examination, there is a well-demarcated 3x2cm erythematous lesion. It is neither painful, nor itchy. The lesion is shown in the accompanying photograph. Which
one of the following is the most likely diagnosis?

A. Seborrheic keratosis.

B. Actinic keratosis.

C. Invasive squamous cell carcinoma.

D. Psoriasis.

E. Bowen’s disease.

Incorrect. Correct answer is E


45% answered correctly

Explanation:

Correct Answer Is E

The large raised scaly plaque shown in the picture has the characteristic features of Bowen’s disease.

Bowen’s disease is squamous cell carcinoma in situ of the skin. The disease is more commonly seen in those over 60 years of age, but it can be found at any age (rare before 30 years). Sun exposure is the most significant risk factor.

Bowen’s disease presents as a well-demarcated erythematous and scaling plaque that may be mistaken for psoriasis or dermatitis. It is usually asymptomatic, but ulceration is a possibility. In 10-20% of cases there are multiple lesions. Although
Bowen’s disease can virtually occur anywhere in the body, the sun-exposed areas, particularly the lower legs are the most common site of presentation.

The diagnosis is confirmed and documented by punch or shave biopsy. All of the following options can be successfully used for treatment with about the same efficacy and a recurrence rate of 5-10%:

Topical fluorouracil
Imiquimod
Cryotherapy
Curettage
Cautery
Excision
Photodynamic therapy
Laser therapy
Radiation therapy

Mohs’ surgery is used when the anatomic location of the lesion is of functional consideration (e.g. penis, fingers) or for cosmetic issues. Occasionally, observation may be considered if the lesion is progressing slowly and is located in areas with
poor healing ability such as the anterior leg of an old person.

239 of 1943
Jesse, 57 years old, presents to your GP clinic with concerns about a spreading brown spot on his left cheek. Physical examination suggests melanoma. Which one of the following is the most appropriate step in management?

A. Excisional biopsy.

B. Punch biopsy.

C. Referral to a plastic surgeon.

D. Radiotherapy.

E. Chemotherapy.

Incorrect. Correct answer is C


45% answered correctly

Explanation:

Correct Answer Is C

Based on current guidelines, any suspicious lesion for melanoma should undergo an initial excisional biopsy (option A) for confirmation. Other types of biopsies such as punch biopsy (option B) are neither as accurate nor recommended.

However, lesions in sensitive anatomical areas such as the face and neck should be biopsied by a plastic surgeon; therefore, referral to a plastic surgeon is the best action to take at this stage.

The initial management of melanoma is excision. Other modalities such as chemotherapy (option E) or radiotherapy (option D) might be considered later for advanced or metastatic melanomas, but not at this stage.

References

• RACGP – Melanoma and other skin cancers: a guide for medical practitioners

Last updated:
Time spent: QID:1780 2023-2-12

240 of 1943
Charlie, 65 years old, is concerned about recent changes in a mole on his chest. He had the mole for a long time but recently it has grown bigger. The mole is shown in the following photograph. Which one of the following is the most likely diagnosis?

A. Squamous cell carcinoma.

B. Superficial spreading melanoma.

C. Benign nevus.

D. Basal cell carcinoma.

E. Age-related skin changes.

Incorrect. Correct answer is B


45% answered correctly

Explanation:

Correct Answer Is B

The photograph shows a pigmented lesion with color variation with recent growth. Of the options, superficial spreading melanoma is the most likely diagnosis.

Superficially spreading melanoma, is the most common type of melanoma that presents with an irregularly-pigmented plaque or macule. The first sign of melanoma is usually a new or existing mole or spot which changes in color, shape, or size.

Squamous cell carcinomas (option A) are usually flat and have no pigmentations.

Basal cell carcinomas (BCCs) have different forms of which nodular BCC is the most common. This type presents with a pearly nodule with telangiectasis. BCC is not usually pigmented.

Benign nevi (option C) often are of uniform color as do age-related pigmented skin lesions (option E).

References

• DermNet – superficial spreading melanoma.

• RACGP - AJGP – Diagnosing and management of cutaneous melanoma

Time spent: QID:1782 Last updated:


2023-2-12

241 of 1943
John, 42 years old, presents to your GP clinic to discuss skin cancer prevention. His past medical history is significant for a cutaneous squamous cell carcinoma (SCC) excised which was successfully treated years ago. He has fair skin with a
tendency to tan rather than burn. He denies a history of sunburns as a child. Which one of the following is the most significant risk factor for developing another SCC?

A. Tendency to tan rather than burn with sun exposure.

B. History of SCC.

C. His age.

D. Absence of sunburns in childhood.

E. Fair skin.

Incorrect. Correct answer is B


45% answered correctly

Explanation:

Correct Answer Is B

SCC is the second most common skin cancer in Australia after basal cell carcinoma. Risk factors for SCC are:

Unprotected exposure to ultraviolet (UV) radiation from the sun or tanning bed
Age over 50
Male gender
Fair skin (Fitz Patrick I and II)
Immunosuppression
Personal history of skin cancers (non-melanoma or melanoma)
History of human papillomavirus (HPV) infection
Precancerous skin lesions such as Bowen’s disease and actinic keratosis (AK)
Chronic infections and skin inflammation from burns, scars, and other conditions

Of these risk factors, the presence of another SCC in history has the most significance (high risk). Fair skin (option E) is another high-risk factor for SCC but not as significant as the history of another SCC. In the absence of past history of skin
cancer, it would be the correct answer.

He is under 50 and his age (option C) is not a risk factor for now. He tends to tan rather than burn (option A) which is a protective factor. While childhood sunburns are a risk factor, their absence (option D) is not one.

References

• RACGP – AJGP – Skin checks in primary care

Last updated:
Time spent: QID:1784 2023-2-12

242 of 1943
A 43-year-old man presents with a painful rash on his back as shown in the following photograph. The rash erupted one week ago and was itchy and slightly sore. The soreness built up in intensity and now he has severe sharp pain. Which one of the
following is the most appropriate treatment option for him?

A. Gabapentin.

B. Opioids.

C. Amitriptyline.

D. Systemic steroids.

E. Famciclovir.

Incorrect. Correct answer is D


45% answered correctly

Explanation:

Correct Answer Is D

The rash presents a typical case of shingles caused by varicella zoster virus (VZV) infection.

Varicella zoster virus (VZV) infection causes two distinct clinical syndromes. Primary infection, also known as varicella or chickenpox, is a common pediatric viral disease. After a chickenpox infection, VZV remains latent in the neurons of cranial
nerves and dorsal root ganglia. Subsequent reactivation of latent VZV can result in a vesicular rash in a dermatomal distribution, known as herpes zoster (shingles). The lesions are painful, and the pain may last long even after the resolution of the
rash.

Pain that persists for 3 months or more is defined as post-herpetic neuralgia (PHN). The pain associated with acute zoster infection and PHN can be burning, sharp, stabbing, and constant or intermittent. More than 90 percent of patients with PHN
also have allodynia, defined as pain evoked by normally nonpainful stimuli such as light touch.

Of note, pain management for acute zoster infection is different. For pain associated with acute herpes zoster infection, simple analgesia or systemic steroids are used as first-line options. The tricyclic antidepressant (TCA) Amitriptyline or
opioids (option B) may be considered as the second line.

On the other hand, pregabalin, gabapentin, TCAs, or lignocaine patches are preferred first-line options for PHN. Opioids and tramadol are used as second-line alternatives.

Antiviral agents such as famciclovir, valacyclovir, or acyclovir should be used in any patient seen within 72 hours of the onset of the rash, all patients with ophthalmic herpes zoster, and in immunocompromised patients. For pregnant women, only
acyclovir (first line) or valaciclovir (second line) are used because the safety of famciclovir during pregnancy has yet to be established. Antivirals are often given for 7 days.

This patient has presented beyond the 72-hour window, and has no other indications for antiviral therapy, and giving him famciclovir (option E) is futile. For him, pain control is the most significant concern. Of the options, systemic steroids such as
prednisolone (50mg daily for 7 days) are the most appropriate. Gabapentin (option A) or amitriptyline (option C) was the correct option if the case was PNH (pain persisting for 3 months or beyond.

References

• Australian Prescriber - Herpes zoster: epidemiology, clinical features, treatment and prevention

• Medscape - Herpes Zoster

Time spent: QID:1788 Last updated:


2023-2-12

243 of 1943
A 52-year-old man presents with intense pain in his chest wall for the past 4 months. The problem started with the eruption of a rash over the area. The rash resolved in 2 weeks, but the pain has persisted and increased in intensity. He describes the
pain as constant, sharp, and unbearable, affecting his life. He has used Panadol and ibuprofen for pain control with minimal effect. He also has diabetes, hypertension, and stage III chronic kidney disease, and denies any pain or pins and needles
elsewhere. Which of the following is the most appropriate option for him?

A. Gabapentin.

B. Tramadol.

C. Amitriptyline.

D. Systemic steroids.

E. Pregabalin.

Incorrect. Correct answer is C


45% answered correctly

Explanation:

Correct Answer Is C

The scenario represents post-herpetic neuralgia, defined as chronic pain associated with herpes zoster infection (shingles) caused by the varicella zoster virus.

Varicella zoster virus (VZV) infection causes two distinct clinical syndromes. Primary infection, also known as varicella or chickenpox, is a common pediatric viral disease. After a chickenpox infection, VZV remains latent in the neurons of cranial
nerves and dorsal root ganglia. Subsequent reactivation of latent VZV can result in a vesicular rash in a dermatomal distribution, known as herpes zoster (shingles). The lesions are painful, and the pain may last long even after the resolution of the
rash.

Pain that persists for 3 months or more is defined as post-herpetic neuralgia (PHN). The pain associated with acute zoster infection and PHN can be burning, sharp, stabbing, and constant or intermittent. More than 90 percent of patients with PHN
also have allodynia, defined as pain evoked by normally nonpainful stimuli such as light touch.

Pain control in the acute phase of the infection is often achieved by simple analgesics and/or systemic steroids (e.g., prednisolone 50mg/d for 7 days). Opioids or amitriptyline can be used as second-line options if adequate pain control is not
achieved.

For PHN, the anticonvulsants gabapentin or pregabalin, tricyclic antidepressants (TCAs), or lignocaine dermal patches are used as first-line options. Of the TCA, amitriptyline is the most effective and commonly prescribed one.

With pain lasting for 4 months, this patient has PHN. Gabapentin (option A) and pregabalin (option E) are eliminated by renal excretion and should be used with caution and meticulous dose adjustment in patients with renal insufficiency and avoided
if better first-line options are applicable. For him, amitriptyline is the most appropriate option for pain control.

TCAs inhibit the reuptake of norepinephrine and serotonin in the central nervous system. They are thought to increase the inhibition of nociceptive signals from the periphery. The preferred tricyclic drug for PHN is amitriptyline, starting at 10 mg each
night and titrating slowly as tolerated, with a maximum dose of 150 mg daily. Amitriptyline is excreted via the liver and its metabolism and is the most appropriate option in this scenario. Its anticholinergic adverse effects such as sedation and dry
mouth) may limit the patient’s compliance.

Opioids (option B) or tramadol can be used as second-line agents, in those with contraindications to the first-line options.

While steroids (option D) are used for treating acute herpes zoster pain where inflammation seems to have a role, they have no effect on PNH as it has different pathophysiology.

References

• RACGP - Focus - Neuropathic pain A management - an update

• UpToDate - Post-herpetic neuralgia

• Australian Prescriber - Herpes zoster: epidemiology, clinical features, treatment and prevention

Last updated:
Time spent: QID:1790 2023-2-12

244 of 1943
Which one of the following is not correct regarding contact dermatitis?

A. Plants are the most common cause of contact dermatitis.

B. It can be treated with topical steroids.

C. It can be treated with oral steroids.

D. The application of skin moisturizers can assist the healing process.

E. Avoidance of the allergen is essential to the prevention of further flare-ups.

Correct
45% answered correctly

Explanation:

Correct Answer Is A

Allergic contact dermatitis is a type 4 or delayed hypersensitivity reaction and occurs 48–72 hours after exposure to the allergen. The mechanism involves CD4+ T-lymphocytes and the release of cytokines that activate the immune system and cause
dermatitis.

Contact allergy occurs predominantly from an allergen on the skin rather than from internal sources or food. Of note, a patient might have been in contact with the allergen for years without it causing dermatitis. Even small quantities of an allergen
can induce dermatitis.

Allergic contact dermatitis is especially common in metal workers, hairdressers, beauticians, healthcare workers, cleaners, painters, and florists.

Patients with impaired barrier function are more prone to contact dermatitis.

Allergic contact dermatitis develops hours after contact with the responsible material and settles down over some days provided the skin is no longer in contact with the allergen. The condition is often limited to the site of contact with the allergen,
but it may extend outside the contact area or become generalized. Transmission from the fingers can lead to dermatitis on the eyelids and genitals. Dermatitis is unlikely to be due to a specific allergen if the area of skin most in

Some typical examples of allergic contact dermatitis include:

Eczema in the skin in contact with jewelry items, due to contact allergy to nickel
Reactions to fragrances in perfumes and household items
Eczema under adhesive plaster, due to contact allergy to rosin
Swelling and blistering of face and neck in reaction to permanent hair dye, due to allergy to paraphenylenediamine
Hand dermatitis caused by rubber gloves
Itchy red face due to contact with methylisothiazolinone, a preservative in wash-off hair products and baby wipes
Fingertip dermatitis due to acrylates used in hair extensions and nail cosmetics.
Reactions after dental implants containing acrylates
Localized blistering at the site of topical medications
Swelling and blistering on exposed sites (e.g., face and hands) due to contact with plants such as poison ivy

Therefore, prevention of further contact with the allergen is the mainstay of the treatment. A skin patch test may be required to find the culprit. Active lesions are often treated with topical steroids (option B). More severe cases may necessitate a
short course of oral steroids (option C).

Depending on the severity and the clinical course, other treatment options can include phototherapy/photochemotherapy, immune suppressants such as azathioprine or ciclosporin, or immune system modulators tacrolimus ointment.

The application of skin moisturizers (option D) can promote general skin health conditions, alleviate the symptoms, and accelerate the healing process.

Although plants could result in contact dermatitis, they are not the most common cause of contact dermatitis. Nickel and other metals/ chemicals are the most common cause of contact dermatitis.

References

• DermNet - Allergic contact dermatitis

• Medscape - Allergic Contact Dermatitis

Last updated:
Time spent: QID:1796 2023-2-12

245 of 1943
A 61-year-old male farmer presents to your clinic because he is concerned about a skin lesion on his face. The lesion is shown in the accompanying photograph. Which one of the following is the most likely diagnosis?

A. Keratoacanthoma.

B. Basal cell carcinoma.

C. Implantation dermoid cyst.

D. Amelanotic malignant melanoma.

E. Squamous cell carcinoma.

Incorrect. Correct answer is B


45% answered correctly

Explanation:

Correct Answer Is B

The appearance of the lesion is consistent with nodular basal cell carcinoma (BCC). BCC is the most common skin cancer. Unlike squamous cell carcinoma (SCC), BCC can be seen in non-photoexposed areas as well. Nodular BCC is the most
common subtype. The pearly appearance, shiny surface and telangiectasis are characteristic features. Additionally, the lesion in the photograph has undergone ulceration at parts.

(Option A) Keratoacanthoma has the central crater filled with necrotic tissue, which is not seen here.

(Option C) Implantation dermoid cysts are firm subdermal cystic nodules that often occur following a penetrating trauma. The fingertips of mechanics are the most common site for such lesions.

(Option D) Amelanotic melanoma can resemble a nodular BCC; although it will lack the pearly appearance of BCC, which can be noted in this photograph.

(Option E) Squamous cell carcinoma does not fit the clinical picture. It usually presents with an ulcerating plaque.

246 of 1943
A 41-year-old farmer has presented with a lesion on his right sole. The lesion appeared 4 months ago and has progressively enlarged since then. The lesion is shown in the accompanying photograph. Which one of the following could be the most
likely diagnosis?

A. Neuropathic ulcer.

B. Malignant melanoma.

C. Simple nevus.

D. Pigmentation from heat.

E. Squamous cell carcinoma.

Incorrect. Correct answer is B


45% answered correctly

Explanation:

Correct Answer Is B

The size and color of the lesion, the irregularity, and the rapid progression are highly suggestive of malignant melanoma.

(Option A) Neuropathic ulcers occur over pressure points of areas with impaired sensation due to conditions such as diabetes mellitus (the most common cause), syphilis, leprosy, and other neuropathies. They present a completely different picture.

Neuropathic ulcer

(Option C) The rapid progression, size, and irregularity are inconsistent with melanoma rather than simple nevus as a diagnosis. A simple nevus usually has regular borders, is often unicolor, and does not spread so fast.

(Option D) Pigmentation from heat, also called ‘erythema ab igne’ is tanning of the skin, where it is chronically exposed to heat.

247 of 1943
Erythema ab inge (left thigh)

(Option E) Cutaneous squamous cell carcinoma (SCC) presents with a usually amelanotic skin lesion in sun-exposed areas.

References

• DermNet NZ - Melanoma

• Medscape - Malignant Melanoma

Time spent: QID:41 Last updated:


2023-2-12

248 of 1943
A 43-year-old farmer from Queensland presents with a pigmented lesion on his upper chest, shown in the accompanying photograph. The lesion appeared 4 months ago, but recently has enlarged. The lesion has been itchy and oozing and painful for
the past 3 weeks. Which one of the following is the most likely diagnosis?

A. Malignant melanoma.

B. Keratoacanthoma.

C. Basal cell carcinoma.

D. Blue nevus.

E. Benign pigmentation.

Correct
45% answered correctly

Explanation:

Correct Answer Is A

Although the lesion shown in the photograph may be mistakenly considered as a pigmented nodular basal cell carcinoma, a more careful inspection of the lesion indicating pigmentation of different colors as well as the history of the rapid
progression over 4 months suggests malignant melanoma (nodular type) as the most likely diagnosis.

(Option B) Keratoacanthoma has the typical crater which is absent here. The crater often has keratin or dead skin cells inside.

Keratoacanthoma - note the nodular structure and the presence of a crater

(Option C) Pigmentation and color variegation of this lesion favors melanoma rather than basal cell carcinoma. Moreover, the rapid progression and evolution of symptoms such as itching and tenderness are more commonly seen in melanoma.

(Options D and E) Clinical presentation and the history (rapid course of progression and evolving of size and symptoms) definitely exclude blue nevus and benign pigmentation as the likely diagnoses.

249 of 1943
On a routine health examination of a 72-year-old man, you notice lesions depicted in the following photograph on his back. Which one of the following could be the most likely diagnosis?

A. Superficial spreading melanoma.

B. Mycosis fungoides.

C. Hutchinson melanotic freckles.

D. Seborrheic keratosis.

E. Solar keratosis.

Incorrect. Correct answer is D


45% answered correctly

Explanation:

Correct Answer Is D

The appearance of the lesions are suggestive of seborrheic keratosis. Characteristic features include variegated pigmentation from black to tan, the verrucous greasy surface and occurrence in crops.

Seborrheic keratoses are well-defined benign lesions which are often pigmented and can occur on the head, neck, trunk and limbs and areas not always significantly exposed to sunlight. Although the etiology is unknown, sun exposure may play a
role.

They usually start to appear in the third decade of life and increase in number as the age advances. Seborrheic keratoses are the most common benign pigmented tumors in those aged 50 years and over.

Seborrheic keratoses are typically round plaques with adherent greasy scale. They seem to be stuck on the skin. Their color may vary from black to tan. The surface appears velvety to verrucous. Sometimes they are so verrucous that may be
mistaken with warts, particularly if not heavily pigmented. Sometimes there are surface cracks and horn cysts.

Some seborrheic keratoses are flat. Flat lesions, particularly those of the face and the limbs, may be difficult to be distinguished from melanoma or lentigo maligna. This is even more true if the lesions are inflamed and consequently, enlarged and
itchy with tendency to bleed.

(Option A) Superficial spreading melanoma is another likely diagnosis, especially if the lesions were more superficial. The large number of the lesions, smooth margins and the greasy warty look of the lesion in the center make seborrheic keratosis a
more likely diagnosis. Biopsy and histological studies are required if the diagnosis is in doubt.

(Option B) Mycosis fungoides is a misnomer describing a rare involvement of the skin with lymphoma. The appearance would be irregular eczematous dermatitis progressing to form plaques.

(Option C) Hutchinson melanotic freckles have more irregular borders and are more flat than the lesions depicted. Furthermore, Hutchinson melanotic freckle more commonly tends to be solitary and on sun-exposed areas.

(Option E) Solar keratoses are firm, hyperkeratinous plaques of 2 to 10mm in size seen in sun-exposed areas e.g. dorsum of hands.

References

• Medscape - Seborrheic Keratosis

• DermNet NZ - Seborrhoeic Keratosis

• Therapeutic Guidelines – Dermatology; available from http://tg.org.au

L t d t d

250 of 1943
Last updated:
Time spent: QID:44 2023-2-12

251 of 1943
A 42-year-old woman sustained a motor vehicle accident (MVA) 2 days ago and was brought to the Emergency Department by ambulance.
She was resuscitated accordingly, and was admitted to the hospital due to fractures of ribs and her left humerus. Today, she is found to be in
respiratory distress. Pulse oxymetry shows oxygen saturation of 89% on room air. She has a blood pressure of 130/85 mmHg, pulse rate of
100bpm in both arms, respiratory rate of 30 breaths per minute, and a temperature of 36.7°C. Lungs and heart are clear on auscultation.
Supplemental oxygen by facial mask is started, and a chest X-ray obtained that shows whitening of the left pleural angle. Which one of the
following is the most likely diagnosis?

A. Traumatic rupture of the aorta.

B. Pulmonary contusion.

C. Pneumonia.

D. Hemothorax.

E. Atelectasis.

Incorrect. Correct answer is B


45% answered correctly

Explanation:

Correct Answer Is B

Except for traumatic rupture of the aorta, all the given options have respiratory distress and hypoxia as a common and early presentation.
Furthermore, equal pulses of both arms make traumatic rupture of the aorta a very remote possibility.

With atelectasis (option E) and post-obstructive pneumonia (option C), fever is expected to be present. This patient is afebrile making these
two less likely.

In hemothorax (option D), chest exam is not normal, and the following are present:

Mild to moderate shortness of breath (commonly)


Absent breath sounds on the base of the affected side
The base of the affected lung is dull to percussion
Faint and distant breath sounds on the apex of the affected side

With respiratory distress and hypoxia following blunt chest trauma 24 -48 hours after the incident and a normal chest exam, pulmonary
contusion would be the most likely diagnosis.

Pulmonary contusion is the bruising of lung parenchyma due to trauma. It is most commonly caused by direct blunt trauma to the chest wall,
or in explosions, or a shockwave associated with penetrating trauma.

The condition may not show up immediately after the injury and becomes evident 1 or 2 days after the trauma. This necessitates close
monitoring of every patient with considerable trauma to the chest wall. Hypoxia and respiratory distress are main manifestations.

Pulmonary contusion is very difficult to diagnose only based on chest exam because exam findings are almost always inconclusive.

Chest X-ray is the initial diagnostic tool; however, chest X-ray often underestimates the size of contusion and tends to lag behind the clinical
picture. In pulmonary contusion, X-ray shows whitening of the affected area(s).

In one-third of the patients, radiological characteristics may take an average 6 hours to develop. The true extent of injury takes 24-48 hours to
develop. When the radiologic appearance is evident in a short time after the incidence, a CT scan must be performed for assessment of
associated injuries.

Pulmonary contusions usually resolve in 3 to 5 days, provided no secondary insult occurs. The main complications of pulmonary contusion
are acute respiratory distress syndrome (ARDS) and pneumonia. Approximately 50% of patients with pulmonary contusion develop ARDS.
This percentage increases to 80% if more than 20% of the lung is affected. Direct lung trauma, alveolar hypoxia, and blood in the alveolar
space are all major activators of an inflammatory pathways resulting in acute lung injury.

Pneumonia is also a common complication of pulmonary contusion. Blood in alveolar spaces provides an excellent growth medium for
bacteria. Clearance of secretions is decreased with pulmonary contusion, and this is augmented by any chest wall injury and mechanical
ventilation. Good tracheal toilet and pulmonary care is essential to minimize the incidence of pneumonia in this susceptible group.

NOTE - Ruptured aorta (option A) presents with hemodynamic instability (hypotension and tachycardia) and widened mediastinum on
chest X-ray.

252 of 1943
References

• Medscape - Blunt Chest Trauma

Last updated:
Time spent: QID:95
2023-2-12

253 of 1943
A 24-year-old man is brought to the Emergency Department after he sustained a roadside accident. On examination, he is conscious and fully
alert, has a blood pressure of 135/90 mmHg, pulse rate of 100 bpm, and respiratory rate of 22 breaths per minute. There are several tender
bruises over his chest. A part of the chest wall on the right side has paradoxical respiratory movements. He is started on opiate analgesics
and supplemental oxygen by face mask. A few hours later he develops drowsiness and severe dyspnea. Bedside pulse oximetry shows an
oxygen saturation of 87%. Which of the following is the most appropriate management for this patient?

A. Tracheostomy

B. Intubation and ventilation.

C. Strapping the chest.

D. Tracheostomy.

E. Cricothyroidotomy.

Incorrect. Correct answer is B


45% answered correctly

Explanation:

Correct Answer Is B

The clinical picture suggests flail chest. Flail chest occurs when 3 or more adjacent ribs are each fractured in 2 places, creating a floating
segment comprised of several rib sections and the soft tissues between them. This unstable section of chest wall moves in the opposite
direction of the uninjured, normal-functioning chest wall (paradoxical movement), and is associated with significant morbidity from pulmonary
contusion. Abnormal motion can be difficult to detect and make the diagnosis difficult.

Initial management of flail chest includes oxygen and close monitoring for early signs of respiratory compromise, ideally using both pulse
oximetry and clinical observation.

Pain management and pulmonary toilet is mainstay of therapy in patients whose respiration is not yet compromised. Options for pain control
include patient-controlled analgesia (PCA), oral pain medications and indwelling epidural catheters.

Respiratory compromise, if develops, is a result of underlying pulmonary contusion. In alert, otherwise uncompromised patients, continuous
positive airway pressure (CPAP) may spare them from the need for intubation and mechanical ventilation. However, intubation and
mechanical ventilatory support is required in:

Patients with severe injuries


Patients with respiratory distress
Progressively worsening respiratory functions
Patients with respiratory suppression because of excessive narcotic pain control

NOTE – Intubation and mechanical ventilation should be followed by prophylactic chest tube insertion (often bilateral), because the
ragged edges of broken ribs my lead to pneumothorax.

This patient has developed respiratory distress despite being on analgesics and oxygen due to either respiratory suppression from analgesia
or underlying pulmonary contusion. Therefore, he should be intubated and mechanically ventilated. This then should be followed by
prophylactic chest tube insertion.

Stabilization of the segment with manual or object pressure restricts chest wall expansion and interferes with appropriate respiratory
mechanics and is no longer used.

Other options have no role in management of flail chest and its complications.

References

• Medscape - Flail Chest

• Medscape - Trauma

• UpToDate - Inpatient management of traumatic rib fractures

Last updated:
Time spent: QID:526
2023-2-12

254 of 1943
A 34-year-old gang member sustains a penetrating chest trauma and is brought to the Emergency Department. On examination, his vital
signs are stable. He has no shortness of breath. A wound is noted in the right hemithorax. Initial evaluation is consistent with an open chest
wound without any air-sucking or bubbling. Auscultation reveals decreased breath sound in the right lower lung field, and percussion is
associated with a dull note. Which one of the following would be the most appropriate next step in management?

A. Strap the wound with pressure.

B. Underwater-sealed chest tube.

C. Needle aspiration.

D. Intubation.

E. Thoracotomy.

Incorrect. Correct answer is B


45% answered correctly

Explanation:

Correct Answer Is B

The main concern in the scenario is possible hemothorax evident by decreased breath sounds and dullness on percussion. Water-sealed
thoracostomy drainage is the primary mode of treatment for hemothorax.

Injuries leading to massive hemothorax include aortic rupture, myocardial rupture, and injuries to hilar structures. Other causes include
injuries to the lung parenchyma and intercostal or mammary blood vessels. A volume of 30 mL is needed for hemothorax to manifest on an
upright chest X-ray. In skilled hands, ultrasound can diagnose hemothorax accurately.

Although the chest wound seems open on exam, there is no sign or symptom suggestive of a sucking open chest wound potentiating tension
pneumothorax for now. The main concern with open chest wounds is the fact that air tends to enter the pleural cavity through the wound,
rather than into the lung through the trachea once the wound diameter exceeds 75% of the diameter of the trachea during inspiration. This will
quickly lead to pneumothorax and tension pneumothorax.

NOTE - Immediate management of a sucking chest wound is with a three-side-sealed wound covering to produce a one-way valve
mechanism

(Option A) Wound strapping is not an option for management of hemothorax.

(Option C) Immediate decompression with needle aspiration is used for urgent management of tension pneumothorax.

(Option D) Intubation is not an option for management of patient with hemothorax.

(Option E) Thoracotomy is the procedure of choice for surgical exploration in the following situations:

An immediate bloody drainage of ≥20 mL/kg (approximately 1500 mL)


Persistent bleeding (generally >3 mL/kg/hour)
Shock despite initial treatment

References

• Medscape - Hemothorax

Last updated:
Time spent: QID:622
2023-2-12

255 of 1943
A 2-year-old Aboriginal boy has ingested 2 lithium batteries 2 hours ago, and is brought to the Emergency Department. Chest and abdominal
X-rays confirms the batteries to be still in the esophagus. Which one of the following would be the most appropriate management?

A. Emergency endoscopic removal of the batteries.

B. Watch for the passage for 3 days.

C. Serial chest and abdominal X-rays.

D. Urgent laparotomy.

E. Reassure the parents.

Correct
45% answered correctly

Explanation:

Correct Answer Is A

Lithium batteries can cause liquefaction necrosis as rapidly as six hours. This occurs due to production of sodium hydroxide production
caused by the current of the battery. If the battery is in the esophagus, it should be endoscopically removed as soon as possible. Esophageal
burns and subsequent perforation occur adjacent to the negative battery pole (anode). Injury can continue even after endoscopic removal for
days to weeks due to residual alkali or weakened tissue.

References

• Poison Control - Button Battery Ingestion Tirage and Treatment Guideline

Last updated:
Time spent: QID:767
2023-2-12

256 of 1943
A 71-year-old woman underwent a hip replacement surgery 3 days ago. Tonight, you are called to visit her because she is unwell. She is
agitated, cannot sleep and is exquisitely sensitive to noise. She has a history of anxiety and depression. Physical examination is
unremarkable, and the vital signs are stable. An air blood gas (ABG) analysis reveals a PaO2 of 91 mmHg (normal: 80-95 mmHg). Which one
of the following diagnoses best describes this clinical picture?

A. Urinary tract infection.

B. Pulmonary embolism.

C. Paroxetine withdrawal.

D. Benzodiazepine withdrawal.

E. Transient ischemic attack.

Incorrect. Correct answer is D


45% answered correctly

Explanation:

Correct Answer Is D

Agitation, sleeplessness, and more importantly exquisite sensitivity to noise suggests benzodiazepine withdrawal as the most likely
explanation to this presentation. History of anxiety and depression endorse this diagnosis as well, because it is common for patients suffering
from anxiety to be on benzodiazepines as a means of relief.

The benzodiazepine withdrawal syndrome is highly variable. Common symptoms include insomnia, irritability, palpitations and sensory
disturbances. Abrupt discontinuation in patients on high doses, e.g. greater than diazepam 50mg daily or equivalent, may result in seizures.

(Option A) Urinary tract infection (UTI) would have presented with more pronounced symptoms such as fever, frequency, urgency and/or
dysuria. Without these, and considering the normal exam results, UTI seems a remote possibility.

(Option B) Considering the type of the surgery and subsequent immobilization, pulmonary embolism (PE) should be doubted and investigated;
however, it is unusual for a patient with PE to have stable vital signs and normal physical exam findings, and more importantly a normal
arterial oxygen pressure. Additionally,sensory disturbances are not a feature seen in PE.

(Option C) SSIR (e.g. paroxetine) withdrawal syndrome can cause dizziness, vertigo, headache, nausea and flu-like symptoms as well as
anxiety, confusion, irritability, excessive dreaming and insomnia. Although paroxetine can be the cause as well, the presence of sensory
disturbances makes benzodiazepine withdrawal more likely.

(Option E) Without any focal neurological deficits in history and clinical findings, TIA is unlikely.

References

• The benzodiazepine withdrawal syndrome

• Therapeutic Guidelines – Psychotropic

Last updated:
Time spent: QID:829
2023-2-12

257 of 1943
A 42-year-old man, who was involved in a motor vehicle accident, is being evaluated in the Emergency Department. On examination, he has
stable vital signs and an oxygen saturation of 96% on room air; however, he has marked difficulty in breathing due to intractable chest wall
pain, which is worse on inspiration. He only takes shallow breaths. On examination, multiple bruises and tender points are noted on his chest.
A non-contrast CT scan of the chest reveals multiple bilateral rib fractures as well a small pneumothorax on the right side. Which one of the
following options is the most appropriate immediate treatment to help him with breathing?

A. Intubation.

B. Morphine.

C. Chest strapping.

D. Needle aspiration of the pneumothorax.

E. Underwater-sealed chest drain.

Incorrect. Correct answer is B


45% answered correctly

Explanation:

Correct Answer Is B

Pain control has a fundamental role to play in the management of rib fractures to decrease chest wall splinting and alveolar collapse. Patients
with pain due to rib fractures, such as this, try to minimize their chest wall motion by shallow breathing and avoiding coughing efforts.
Adequate pain management improves tolerance for deep breathing and coughing that will result in enhanced lung volume and clearance of
secretions. These will reduce the risk of lung collapse and pneumonia.

A number of inpatient strategies have been proposed to optimize pain control in patients with rib fractures, including:

Regional anesthesia - Regional anesthesia techniques available for the management of multiple rib fractures include continuous epidural
infusion, paravertebral block, intrapleural infusion, and intercostal nerve block.

Intravenous narcotics - Intravenous narcotics e.g. morphine can provide rapid adequate pain control. Intravenous route is preferred over
subcutaneous or intramuscular injections because of the rapid and predictable onset of action. Although there is no evidence to support
use intravenous nonsteroidal anti-inflammatory drugs (e.g. ketorolac) for pain control in rib fracture, they can be used to supplement and
reduce narcotic use. However, these agents are avoided in patients who have a significant bleeding risk (e.g. hemothorax, solid organ injury)
and those with renal insufficiency or hypovolemia. Patient-controlled analgesia (PCA) is advocated for patients with rib fractures because of a
more timely access to pain medication by the patients and a reduced risk for excessive sedation.

The size of the pneumothorax and O2 saturation in this patient, make pain caused by fractured ribs the only respiratory problem for now,
and adequate pain control is the first priority forcomfortable and painless breathing. This can be achieved by measures described above. Of
the options only morphine can be used for pain control.

(Option A) Intubation would be the most appropriate option for patients with fail chest with contraindications or inadequate response to less
invasive measures such as CPAP or BiPAP.

(Option C) Chest strapping by decreasing chest wall movements can be somewhat useful but it decreases adequate ventilation which is in
disagreement with the management objectives. It is not an appropriate treatment.

(Option D) Needle aspiration is the most appropriate next step in patients with tension pneumothorax for immediate relief. This then is
followed by chest tubes and continues underwater-sealed drainage.

(Option E) Water-sealed chest drain must be inserted for treatment of the pneumothorax; however, the pneumothorax is small and has not
compromised oxygenation yet. It is not a priority at this stage.

References

• Medscape - Rib Fracture

• UpToDate - Inpatient management of traumatic rib fractures

Last updated:
Time spent: QID:864
2023-2-12

258 of 1943
A 35-year-old man is brought to the Emergency Department after he sustained a car crash. On examination, he has a blood pressure of 80/50
mmHg, heart rate of 110 bpm, and respiratory rate of 19 breaths per minute. He is pale and diaphoretic. Bedside pulse oxymetry shows an
oxygen saturation of 95% on room air. Which one of the following is the most appropriate next step in management?

A. Supplemental oxygen.

B. Chest X-ray (CXR).

C. CT scan.

D. Intravenous colloid fluids.

E. Blood transfusion.

Correct
45% answered correctly

Explanation:

Correct Answer Is A

In all trauma patients (like other patients) the ABC (airway, breathing, and circulation) protocol should be followed in a sequential manner.
There is no comment regarding airway compromise. He also does not appear to have any respiratory distress. In addition, the normal oxygen
saturation of 95% (94-100%) excludes respiratory compromise - at least for now.

The patient, however, is in pre-shock state evident by a systolic blood pressure of less than 90 mmHg. Reflexive tachycardia is
confirmatory. Although tension pneumothorax and cardiac tamponade can cause shock in the absence of hemorrhage, there are no clues to
such conditions in the scenario. These make hemorrhagic shock the most likely cause of hypotension and tachycardia.

In all trauma patients in hemorrhagic shock, oxygen supplementation is beneficial and should be considered as the very next best step in
management even if oxygen saturation is within the normal limits but not maximum. Maximizing the blood oxygen pressure compensates, to
some extent, the decreased tissue oxygenation due to diminished perfusion caused by hypovolemia.

(Options B and C) CXR and other imaging modalities such as CT scan should be withheld until the patient is hemodynamically stable.

(Option D) Replenishing the lost volume is the most essential step after adequate oxygenation is ensured. Classically, isotonic solutions such
as normal saline or ringer is used. Preference of colloids over isotonic fluids is still a place of significant debate.

(Option E) Blood transfusion is considered when initial fluid resuscitation using 2 liters of intravenous fluid fails to restore and maintain
adequate perfusion.

References

• BMJ Best Practice

• Medscape - Hypovolemic Schok Treatment and Management

• ADULT TRAUMA CLINICAL PRACTICE GUIDELINES: Management of Hypovolaemic Shock in the Trauma Patient

Last updated:
Time spent: QID:944
2023-2-12

259 of 1943
A 27-year-old woman is rushed to the Emergency Department by paramedics after she sustained a motor vehicle accident. Upon arrival, she is
unconscious, has a blood pressure of 80/60 mmHg, pulse of 120 bpm, and respiratory rate of 22 breaths per minute. Stridor is noted on
phsical examination. The patient has an oxygen saturation of 88% on room air. Supplemental oxygen is provided immediately, intravenous
fluids are started, and a portable cervical X-ray obtained that reveals a fractured and dislocated cervical vertebra. Which one of the following
is the most appropriate step in management of her airway?

A. Cricothyroidotomy.

B. Tracheostomy.

C. Intubation.

D. CPAP.

E. Increasing the oxygen flow.

Incorrect. Correct answer is C


45% answered correctly

Explanation:

Correct Answer Is C

This patient has concomitant compromised airway and a critical spinal injury at C2 level. Unconsciousness and stridor together mean that
this patient should have an airway secured. This could be achieved by endotracheal intubation as the most effective and convenient method.
However, intubation could be associated with immediate respiratory paralysis if the cervical lesion is above the C3 level or delayed phrenic
nerve paralysis from ascending edema of the spinal cord in lower cervical lesions.

According to Advanced Trauma Life Support (ATLS®) guidelines, orotracheal intubation is the preferred method of airway management for
patients with traumatic cardiopulmonary arrest, even with evidence of spinal injury. Orotracheal intubation using rapid sequence intubation
protocol is recommended for patients who are breathing but unconscious and in need for airway control or ventilatory support. In-line spinal
stabilization should be maintained throughout the procedure to minimize the spinal columns movement and reduce the risk of causing or
exacerbating a spinal cord injury.

(Option A) Cricothyroidotomy is indicated when an airway is required immediately in a patient who is not a candidate for orotracheal or
nasotracheal intubation. Severe facial trauma is the most common indication. It is absolutely contraindicated in children younger than 12
years due to the risk of consequent tracheal stenosis. Relative contraindications include:

Airway obstruction distal enough to the cricoid membrane that a cricothyroidotomy would not provide a secure airway with which to
ventilate the patient

Presence of a SHORT neck, which includes Surgery (history or prior neck surgery), Hematoma, Obesity, Radiation (evidence of
radiation therapy), or Trauma/burns, making it difficult to locate the patient’s anatomical landmarks or producing an increased risk of
further complications

Tumor, infection, or abscess at the site of incision

Lack of operator experience with the procedure

(Option B) Tracheostomy should be performed in the operating room and is indicated for patients who are planned for prolonged mechanical
ventilation.

Tracheostomy may be considered in the following situations:

Congenital anomaly (e.g., laryngeal hypoplasia, vascular web)


Upper airway foreign body that cannot be dislodged with Heimlich and basic cardiac life support maneuvers
Supraglottic or glottic pathologic condition (e.g., infection, neoplasm, bilateral vocal cord paralysis)
Neck trauma that results in severe injury to the thyroid or cricoid cartilages, hyoid bone, or great vessels
Subcutaneous emphysema
Facial fractures that may lead to upper airway obstruction (e.g., comminuted fractures of the mid face and mandible)
Upper airway edema from trauma, burns, infection, or anaphylaxis
Prophylaxis (as in preparation for extensive head and neck procedures and the convalescent period)
Severe sleep apnea not amendable to continuous positive airway pressure devices or other less invasive surgery
Tracheostomy may also be performed to provide a long-term route for mechanical ventilation in cases of respiratory failure or to
provide pulmonary toilet in the following cases:

Inadequate cough due to chronic pain or weakness


Aspiration and the inability to handle secretions

260 of 1943
(Option D) CPAP cannot provide a secure airway and is not an appropriate option in this patient.

(Option E) Increasing the oxygen flow while the airway is compromised is futile.

References

• IJCHS - Airway management in cervical spine injury

• Medscape - Surgical Airway Techniques

• Medscape - Tracheostomy

Last updated:
Time spent: QID:945
2023-2-12

261 of 1943
A 23-year-old man is brought to the emergency department by ambulance paramedics after he sustained a stab wound in the chest in a street
fight. On examination, he as a knife stuck in the left hemithorax. The patient is awake, oriented and cooperative. His blood pressure is
110/65mmHg, hear rate 100 bpm and respiratory rate 18 breaths per minute. There are no raised neck or forehead veins. Which one of the
following is the most appropriate next step in management?

A. Remove the knife int he emergencey department and place deep sutures.

B. Remove the knife in the emergency department after you obtained a chest X-ray (CXR).

C. Transfer the patient to the operating room (OR), remove the knife in the OR and insert chest tube.

D. Insert chest tube in the emergency department and then remove the knife.

E. Perform a needle thoracocentesis for immediate decompression and then transfer the patient to the OR.

Incorrect. Correct answer is C


45% answered correctly

Explanation:

Correct Answer Is C

As a rule, all impaled foreign bodies should be secured in place and not removed until the patient is in the operating room (OR). The rationale
behind this approach is that the object could have damaged major blood vessels, which are temporarily blocked by the object. If the object is
removed, the pressure over the vessels is released, and life-threatening, potentially uncontrollable hemorrhage could ensue.

Such patients should be taken to the OR for removal of the impaled object in a controlled environment where potential bleeding after removal
can be promptly controlled. A chest tube should be left in place for successful drainage of blood leak or air in the pleural space. As many as
80% of patients with penetrating chest trauma have hemothorax, pneumothorax, or both.

Any options suggesting removal of the knife in any place other than the OR is incorrect.

Immediate needle aspiration is the most appropriate next step in patients with tension pneumothorax which is not the case here. Tension
pneumothorax presents with hypotension and difficulty breathing as well as other findings such as diminished air entry into the affected side,
tracheal deviation away from the site of the injury, and diminished breath sounds and hyper-resonance of the affected hemithorax.

References

• JEMS - Use Caution with Impalement Injuries

• Canadian Respiratory Journal - Removal of an impaled knife under thoracoscopic guidance

Last updated:
Time spent: QID:951
2023-2-12

262 of 1943
A 27-year-old man is brought to the Emergency Department after he sustained a stab wound to the left side of his upper chest. On
examination, blood pressure is 120/90 mmHg, heart rate 110, and respiratory rate 19 breaths per minute. Bedside pulse oxymetry shows an
oxygen saturation of 91% on room air. On inspection, the trachea is deviated to the left and breathing appears labored. Auscultation reveals
decreased air entry into the left lung. The left side of the chest is dull to percussion. Which one of the following is the most appropriate next
step in management?

A. Give him supplemental oxygen.

B. Insert chest tube in the left hemithorax.

C. Thoracocentesis.

D. Intravenous fluids.

E. Take him the operating room (OR) immediately.

Correct
45% answered correctly

Explanation:

Correct Answer Is A

This patient has labored breathing and decreased oxygen saturation (normal 94%-100%) in the setting of penetrating chest trauma. Air entry is
also diminished on the injured side. These can be caused by either hemothorax or pneumothorax. With dullness over the affected lung,
hemothorax is the diagnosis, as in pneumothorax the presence of air in the pleural space gives rise to hyper-resonance on percussion on the
affected side.

Regardless of the etiology, however, this patient is in obvious respiratory compromise evident by decreased lung air entry, elevated respiratory
rate, and decreased oxygen saturation (<94%) in the range of mild hypoxia; therefore, the first action must be application of supplemental
oxygen according to the ABC protocol (airway, breathing , and circulation). This then should be followed by immediate chest tube insertion for
evacuation of the air/blood form the pleural cavity as the definite treatment.

(Option B) Chest tube must be inserted for management of hemothorax after supplemental oxygen is given.

(Option C) Needle thoracocentesis would be indicated for emergent decompression if there was tension pneumothorax. In tension
pneumothorax, the excess air in the pleural space leads to increased pressure within the affected hemithorax resulting in compression of the
mediastinum away from the site of the injury. The heart then twists around its superior-inferior axis and kinks veins and decrease venous
blood return to the heart. This leads to hypotension and shock. As mentioned earlier, the mediastinal structures and trachea are pushed away
from the site of the injury, unlike this patient whose trachea is shifted towards the side of the injury.

(Option D) Intravenous fluids are not indicated for now as the patient has normal blood pressure and heart rate.

(Option E) Pneumothorax or hemothorax alone can be treated in the Emergency Department with chest tube, and is not an indication for
transferring the patient to the OR per se.

References

• BMJ - Pneumothorax: an update

Last updated:
Time spent: QID:952
2023-2-12

263 of 1943
A 32-year-old man presents to the Emergency Department with a knife impaled in his back under the right scapula. On examination, he has
blood pressure of 85/50 mmHg, heart rate of 110 bpm and respiratory rate of 22 breaths per minute. There is adequate air entry into both
lungs, but the breathing is labored and the patient can say one sentence at a time. Pulse oxymetry shows an O2 saturation of 91% on room air.
After starting him on supplemental oxygen by face mask and intravenous fluids, which one of the following will be the next best step in
management?

A. Urgent CT scan of the chest.

B. Intubation.

C. Cross match.

D. Removal of the knife under general anesthesia.

E. Chest tube.

Incorrect. Correct answer is C


45% answered correctly

Explanation:

Correct Answer Is C

Low blood pressure and tachycardia in this patient indicate internal hemorrhage. Although tension pneumothorax and cardiac tamponade can
cause similar picture, there is no clue to such conditions in the scenario. Tension pneumothorax is associated with shifted trachea away from
the site of the injury, hyper-resonance, and/or decreased chest wall movements on the affected side. Cardiac tamponade results in decreased
diastolic flow to the heart, pulsus paradoxus, hypotension, and tachycardia. Both tension pneumothorax and cardiac tamponade can lead to
raised neck and forehead veins; however, veins might be flat due to concomitant hemorrhage and hypovolemia.

This patient has another problem as well – the impaled knife. As a rule, any impaled foreign body should be secured in place and not removed
until the patient is in the operating room (OR). The rationale to this approach is that the object could have damaged major blood vessels,
which are temporarily sealed by the object. If the object is removed, the pressure over the vessels is released and life-threatening and
potentially uncontrollable hemorrhage could occur. This would be a grave condition in this patient which is already hemorrhaging.

In this patient blood should be drawn for typing, cross matching, and reservation after initial resuscitation with oxygen and fluid. Blood should
be available in the OR in case the repair is prolonged, and bleeding cannot be stop immediately.

(Option A) In this patient, measures such as initial resuscitation, blood typing and cross match, and immediate transfer to the OR should not
be delayed for imaging studies such as CXR or CT scan.

(Option B) Intubation of this patient should be considered in the OR if general anesthesia is planned. The knife is then removed, and any
possible damage repaired.

(Option D) This patient must be transferred to the OR for removal of the impaled object in a controlled environment and under general
anesthesia. However, blood group typing and cross matching should be performed before that.

(Option E) A chest tube will be secured in the OR after the knife is removed and damages repaired because approximately 80% of patients
with penetrating chest injuries have hemothorax, pneumothorax, or both.

References

• JEMS - Use Caution with Impalement Injuries

• Canadian Respiratory Journal - Removal of an impaled knife under thoracoscopic guidance

Last updated:
Time spent: QID:953
2023-2-12

264 of 1943
A 32-year-old man is brought to the Emergency Department by ambulance after he sustained a house fire. On examination, the patient is
conscious but in severe distress due to pain. He has voice hoarseness, burns around the neck, and burned nose hair. Which one of the
following is the next best step in management?

A. Intubation.

B. Oxygen by face mask.

C. Intravenous morphine for analgesia.

D. Debridment of the burns and application of topical antibiotics.

E. Intravenous fluids.

Incorrect. Correct answer is B


45% answered correctly

Explanation:

Correct Answer Is B

In victims of a close environment burns, smoke and heat inhalation is associated with significant risk of airway obstruction. Any patient with
sooth in the mouth or nose or burns in the face or mouth or around the neck should be monitored very carefully for airway compromise due to
edema. Burn victims should be intubated before edema results in complete airway obstruction.

This patient has hoarseness that indicates airway involvement, and intubation should be considered. In meanwhile he should receive
supplemental oxygen as the most important next step in management while arrangements for intubation are undertaken.

All other options are considered as parts of management, but none takes precedent over oxygenation and intubation.

References

• NSW Government - Clinical Guidelines - Burn Patient Managemebnt

• AMC Handbook of Multiple Choice Questions – page 381

Last updated:
Time spent: QID:141
2023-2-12

265 of 1943
A 19-year-old man is brought to the Emergency Department after he sustained a stab wound to the right side of his chest. The knife is still in.
On examination, his vital signs are within normal range and there is no respiratory distress; however, air entry to the right lung is diminished. A
chest X-ray (CXR) reveals a 25% pneumothorax on the right side. Which one of the following is the most appropriate next step in
management?

A. Admit and observe.

B. Perform serial chest X-rays.

C. Removal of the knife in the operating room under general anesthesia and chest tube insertion.

D. Removal of the knife in the operating room under general anesthesia.

E. Thoracotomy.

Incorrect. Correct answer is C


45% answered correctly

Explanation:

Correct Answer Is C

As a rule, any impaled foreign body should be secured in place and not removed until the patient is in the operating room (OR). The rationale
behind this approach is that the object could have damaged major blood vessels, which are temporarily tamponaded by the object. If the
object is removed, the pressure over the vessels is released, and life-threatening, potentially uncontrollable hemorrhage could occur.

Treatment of this patient’s pneumothorax should be undertaken in the operating room by inserting a chest tube and water-sealed chest
drainage after the knife is removed and any damage is repaired

(Option A) Admission and observation is incorrect because not only this patient requires emergency removal of the impaled knife in the OR,
his traumatic pneumothorax should be treated with chest tube.

(Option B) Admission and observation with serial chest X-rays is considered in selected patients with occult asymptomatic pneumothoraces.
A 25% symptomatic pneumothorax needs definite treatment with chest tube insertion.

(Option D) Knife removal should be followed by chest tube insertion for drainage of blood and post-op pleural secretions even in the absence
of pneumothorax.

(Option E) Thoracotomy may be indicated for acute or chronic conditions.

Acute indications include the following:

Cardiac tamponade
Acute hemodynamic deterioration/cardiac arrest in the trauma center
Penetrating truncal trauma (resuscitative thoracotomy)
Vascular injury at the thoracic outlet
Loss of chest wall substance (traumatic thoracotomy)
Massive air leak
Endoscopic or radiographic evidence of significant tracheal or bronchial injury
Endoscopic or radiographic evidence of esophageal injury
Radiographic evidence of great vessel injury
Mediastinal passage of a penetrating object
Significant missile embolism to the heart or pulmonary artery
Transcardiac placement of an inferior vena caval shunt for hepatic vascular wounds

Chronic indications for thoracotomy include the following:

Non-evacuated clotted hemothorax


Chronic traumatic diaphragmatic hernia
Traumatic cardiac septal or valvular lesion
Chronic traumatic thoracic aortic pseudoaneurysm
Non-closing thoracic duct fistula
Chronic (or neglected) post-traumatic empyema
Infected intrapulmonary hematoma (e.g. traumatic lung abscess)
Missed tracheal or bronchial injury
Tracheoesophageal fistula
Innominate artery/tracheal fistula
266 of 1943
Traumatic arterial/venous fistula

References

• JEMS - Use Caution with Impalement Injuries

• Canadian Respiratory Journal - Removal of an impaled knife under thoracoscopic guidance

• Medscape - Emergency Bedside Thoracotomy

Last updated:
Time spent: QID:954
2023-2-12

267 of 1943
A 36-year-old man is rushed into the Emergency Department by paramedics after he sustained a head-on collision as an unrestrained driver.
He has a hard collar and his head is fixed to a spine board. On a quick review, he is conscious and fully oriented, but in distress as he is not
able to feel his arms and legs. There is no visible site of active bleeding or limb deformity. Chest is clear to auscultation, neck veins are not
raised, and abdominal exam is inconclusive. His blood pressure is 90/40 mmHg, heart rate 50 bpm and respiratory rate 18 breath per minute.
His oxygen saturation is 88% on room air. Sphincter tone is decreased and there is no sensation or deep tendon reflexes below the neck. He is
immediately placed on supplemental oxygen via face mask. Which one of the following would be the most appropriate next step in
management?

A. Placement in Trendelenburg position.

B. Intravenous colloid fluids.

C. Blood transfusion.

D. Adrenaline.

E. Atropine.

Correct
45% answered correctly

Explanation:

Correct Answer Is A

The scenario suggests neurogenic shock. The most common cause of shock in a trauma patient is hypovolemic shock even in the presence
of obvious spinal injury. In comparison, neurogenic shock is much less common and when the condition exists it can mask the presentation
of hypovolemic shock; therefore, it is imperative that hemorrhage, as the most likely cause of decreased blood pressure is excluded.
Neurogenic shock is always a diagnosis of exclusion.

In this patient, no site of active bleeding is noted. There is also no limb deformity to point towards long bone (or pelvic) fractures as the
source of bleeding and decreased blood pressure. Abdominal exam is unremarkable as well.

Tension pneumothorax and cardiac tamponade are other conditions that can lead to decreased blood pressure by impeding venous return to
the heart. For these to exist, chest should be involved. There are no comments as to chest involvement in the scenario; moreover, breathing is
normal, and no abnormalities are found on auscultation.

Although the possible sources of occult bleeding should be thoroughly investigated, with no clues pointing towards other causes of
hypotension, especially hemorrhage and obstruction, the next possible cause to consider in this patient must be neurogenic shock.

Neurogenic shock is classically characterized by hypotension, bradycardia, and peripheral vasodilatation. Neurogenic shock is due to loss of
sympathetic vascular tone and happens only after a significant proportion of the sympathetic nervous system has been damaged – as can
occur with lesions above the 6th thoracic vertebra. In a quadriplegic patient, blood pressure normally ranges between 80/40mmHg to 100/60
mmHg and pulse rate is down to 50 bpm.

The first measure to consider in patients with neurogenic shock is close attention to airway, breathing and circulation (ABC). Hypoperfusion to
an injured spine can be associated with poor outcomes. Oxygenation should be monitored carefully, and oxygen be given. It is recommended
that patients with traumatic spinal injury receive 15L/m oxygen via a non-rebreathing mask.

Hypotension and bradycardia are common features of neurogenic shock. According to guidelines by Neurosurgical Society of Australia, the
first measure to consider is placement of the patient in Trendelenburg position. In neurogenic shock, the main mechanism of hypotension is
pooling of blood in the peripheral venules and small vein; therefore, such maneuver can correct hypotension by increased venous return to the
heart.

This patient has hypoxia evident by an oxygen saturation of 88% (normal ≥95%); therefore, supplemental oxygen should begin. In patients
with neurogenic shock, systolic blood pressure should be maintained above 90 mmHg. Placement of the patient in Trendelenburg position (if
there are no contraindication e.g., head trauma) should be considered next.

(Option B) Isotonic fluids (not colloids) should come next after oxygenation and placement in Trendelenburg position. Careful monitoring for
volume overload is a ‘must’.

(Option C) Blood transfusion is considered in hemorrhagic patients if there is minimal response to adequate fluid resuscitation using
crystalloids (e.g. normal saline). Patients with neurogenic shock as their sole underlying cause of their hypotension do not require blood
transfusion.

(Option D) Vasopressors or inotropes such as dopamine, nor-adrenaline (norepinephrine) or phenylephrine are considered for patients with
hypotension resistant to position or adequate volume resuscitation. Urinary output can be an appropriate guide (urinary output of
<0.5cc/kg/hour). Adrenaline is not used for such a purpose.

268 of 1943
(Option E) Atropine is reserved for patients with bradycardia of <50bpm. As mentioned earlier, it is quite common for patients with neurogenic
shock to have bradycardia down to 50bpm. Atropine is only indicated if there is severe bradycardia (<40 bpm), or when there is significant
hemodynamic instability caused by it. Atropine should also be considered in patients who are undergoing maneuvers that can induce
bradycardia by vagal stimulation e.g., nasopharyngeal suction or intubation.

References

• Medical Online - The Acute Management of Spinal Injuries

Last updated:
Time spent: QID:988
2023-2-12

269 of 1943
A 30-year-old woman is brought into the Emergency Department after falling from a horse while riding. She has a hard collar on and her head
is taped to a hard spine board. She is fully conscious and oriented but unable to move any of her limbs. She answers the question in a normal
voice. On a quick review, no visible active bleeding is noted. Chest is clear to auscultation and abdominal exam is inconclusive. No limb
deformity is noted either. She has a blood pressure of 85/40 mmHg, heart rate of 50 bpm, and oxygen saturation of 90% on room air. Which
one of the following is the most appropriate next step in management?

A. Atropine.

B. Adrenaline.

C. Oxygen.

D. Trendelenburg position.

E. Intravenous colloid fluids.

Incorrect. Correct answer is C


45% answered correctly

Explanation:

Correct Answer Is C

In all trauma patients, initial survey and management starts with the ABC protocol (airway, breathing, and circulation).

This patient is fully conscious and oriented and answers the questions in a normal voice; therefore, her airway is not compromised, and no
further airway management is indicated - at least for now.

Breathing comes next. An oxygen saturation of 90% on room air indicates mild hypoxia and requires supplemental oxygen as the most
important next step in management. Additionally, hypoxia in the face of cord injury can adversely affect neurologic outcome. Arterial
oxygenation should be monitored and supplemented as needed.

No site of active bleeding or any other clues to hemorrhage as the cause hypotension is present in this patient. Although, thorough
investigation is required to exclude overt or occult hemorrhage as the most common cause of shock in trauma patients, given the clinical
findings on the quick review, neurogenic rather than hemorrhagic shock is more likely.

Patients with neurogenic shock often have bradycardia down to 50bpm and a blood pressure ranging from 80/40 mmHg to 100/60 mmHg.

(Option A) Patients with neurogenic shock have bradycardia of down to 50 bpm. No specific therapy is required unless there is significant
hemodynamic instability or severe bradycardia (<40 bpm).

(Option B) Inotrope medications such as dopamine, norepinephrine, or phenylephrine maybe indicated in patients with resistant hypotension.

(Options D and E) This patient may be initially placed in Trendelenburg position (head down) to relocate the pooled blood in the peripheral
venules and small veins to the heart. This results in an increase in cardiac output and blood pressure. Longer term management includes
judicious administration of crystalloids (not colloids) such as Hartman’s solution or normal saline to maintain a blood pressure of 90-100
mmHg. This is critical as hypoperfusion of the injured spine is associated with neurological sequelae and poor outcomes. These, however, do
not take precedence over oxygen supplementation as the most appropriate next step in this patient.

References

• Medical Online - The Acute Management of Spinal Injuries

Last updated:
Time spent: QID:989
2023-2-12

270 of 1943
A 56-year-old man undergoes hip surgery. In the following 24 hours, he only passes 200 ml of urine. During the surgery he was given 1.5 liters
of normal saline and 2 units of packed red cells. Which one of the following investigations is more likely to find the cause of the oliguria?

A. Administration of 1 liter of normal saline in 1 hour and monitoring the response.

B. CT scan of the abdomen and the pelvis.

C. Retrograde pyelogram.

D. DSMA scan.

E. Urine analysis.

Correct
45% answered correctly

Explanation:

Correct Answer Is A

Oliguria, defined as a urine output less than 30 ml/hour (0.5ml/kg/hour) is a common problem in post-operative period. This can be a
consequence of fluid and blood loss resulting in increased release of aldosterone and antidiuretic hormone (ADH) by the adrenal cortex and
posterior pituitary, respectively, in the first 24 hours after the surgery. Oliguria should be temporary and not last more than 24 hours. A urinary
output of less than 500 ml within the first 24 hours warrants prompt investigations.

Before the oliguria is attributed to diminished urine output, it is important to ensure that the patient is not in urinary retention. A full, tender
bladder is often a typical presentation of urinary retention. If in doubt, a catheter should be passed to check the urine volume in the bladder.
Ultrasound can be used as an alternative.

Oliguria is caused by either of the following:

Poor renal perfusion (pre-renal failure e.g., hypovolemia or pump failure)


Renal failure (e.g., acute tubular necrosis)
Renal tract obstruction (post-renal failure)

Most cases of post-operative oliguria are secondary to hypovolemia. This should always be considered first until proven otherwise. This holds
true about this patient as well. More so because he had developed hypovolemia during the surgery requiring resuscitation with fluid and
blood.

In such circumstance, the next best step would be fluid challenge. One liter of normal saline is administered, and urine output monitored
closely. An increase in urine output in response to fluid administration is both therapeutic and diagnostic for oliguria due to hypovolemia.

With renal failure as the underlying cause, oliguria will not respond to fluid challenge. If that is the case, investigation should include urine and
plasma osmolality and electrolytes as well as an ultrasound for assessment of urinary tract obstruction as a possible cause of renal failure.

(Option B) CT scan is not useful for diagnosis of renal failure. If imaging is required, ultrasonography is the preferred option.

(Option C) Retrograde pyelogram requires contrast media. If renal failure is a concern, any option suggesting contrast imaging is incorrect.
Contrast media is nephrotoxic and a hazard in patients with suspected renal failure.

(Option D) DSMA scan has no role in workup for oliguria or renal failure.

(Option E) Urinalysis for urine specific gravity is a part of routine investigation in patients with suspected renal failure and is indicated if the
patient does not respond to fluid resuscitation. Other tests are urine and serum osmolality and urea, creatinine, electrolytes.

References

• NSW Health - Western Sydney Local Health District

Last updated:
Time spent: QID:1043
2023-2-12

271 of 1943
A 48-year-old man with history of heavy smoking and alcohol drinking had a laparotomy with oversewing of a perforated duodenal ulcer. 24
hours after the operation, he is confused and disorientated. Which one of the following is the most appropriate initial investigation?

A. Blood alcohol level.

B. Arterial blood gases (ABG).

C. Serum electrolytes.

D. Chest X-ray.

E. Liver function tests.

Incorrect. Correct answer is B


45% answered correctly

Explanation:

Correct Answer Is B

Post-operative confusion/disorientation is one of the serious post-operative complications. The most common cause of post-operative
confusion is hypoxia that must be excluded or confirmed by an arterial blood gas (ABG) analysis as the most appropriate initial step in
management.

Hypoxia is most likely due to hypoventilation as a result of sedative drugs and post-operative analgesics; however, other etiologies such as
electrolyte disturbances, atelectasis, and sepsis with gram negative organisms should always been considered and excluded. Sepsis with
gram negative organism can lead to acute respiratory distress syndrome (ARDS) and shock in a short time. Drugs or alcohol withdrawal is
another cause to consider.

(Option A) Delirium tremens (DT) or alcohol withdrawal can also be associated with post-operative delirium and confusion. However, the
diagnosis is clinical and bases on the history of alcohol consumption. Blood alcohol level do not help with the diagnosis.

(Option C) Electrolyte derangements (particularly hypo- and hypernatremia) are other common cause of post-op confusion. An electrolyte
panel is an important part of investigation in patients with post-operative confusion after hypoxia is excluded.

(Option D) Chest X-ray is indicated in patients with post-operative confusion after the cause is confirmed to be hypoxia by ABG. Pulmonary
atelectasis or pneumonia may be evident on chest X-ray.

(Option E) Without the history of liver disease, confusion is unlikely to have been caused by liver disease. Liver function tests are not among
routine investigations in patients with post-operative confusion.

References

• AAFP - Delirium in Older Persons: Evaluation and Management

Last updated:
Time spent: QID:1115
2023-2-12

272 of 1943
A 40-year-old woman underwent cholecystectomy. On the first post-operative day, she develops fever and shortness of breath. After chest
physiotherapy which one of the following is the next best step in management?

A. Continuation of chest physiotherapy.

B. Commencement of antibiotics.

C. Postural drainage.

D. Supplemental oxygen.

E. Bronchoscopy.

Incorrect. Correct answer is D


45% answered correctly

Explanation:

Correct Answer Is D

The clinical findings of shortness of breath and fever in the early post-operative period (within 24 hours) are suggestive of atelectasis.
Management of post-operative atelectasis starts with chest physiotherapy, followed by supplementation of oxygen to achieve an oxygen
saturation of above 90%.

(Option A) Continuation of chest physiotherapy is an appropriate step in management of atelectasis; however, after initial physiotherapy,
supplemental oxygen should be given to maintain adequate oxygen saturation.

(Option B) Antibiotics are not indicated in treatment of atelectasis unless chest infection follows.

(Option C) Postural drainage helps with clearing of the secretions. The patient is asked to rest on the non-affected side to let the secretions
drain by gravity. This is an important step in management of patients with atelectasis after initial physiotherapy and oxygen supplementation.

(Option E) Bronchoscopy would be indicated if continuous chest physiotherapy fails. Routine use of bronchoscopy for removing secretions is
not recommended.

References

• Medscape - Atelectasis

• UpToDate - Overview of the management of postoperative pulmonary complications

Last updated:
Time spent: QID:1116
2023-2-12

273 of 1943
In the first 24 hours after elective cholecystectomy, a 50-year-old woman develops a fever of 38.2°C and pulse rate of 110 bpm. The
respiratory rate is 22 breaths per minute. She is smoker and had no pre-operative problems except the biliary pain. Which one of the following
can explain this presentation?

A. Wound infection.

B. Pulmonary embolism.

C. Pulmonary atelectasis.

D. Bile leakage.

E. Pneumonia.

Incorrect. Correct answer is C


45% answered correctly

Explanation:

Correct Answer Is C

Based on the history and clinical findings, pulmonary atelectasis is the most likely cause of this presentation. Pulmonary atelectasis is a quite
common cause of post-operative fever in the first 24 hours of surgery, particularly in smokers.

Postoperative atelectasis is usually caused by decreased compliance of lung tissue, impaired regional ventilation, retained airway
secretions, and/or postoperative pain that interferes with spontaneous deep breathing and coughing. Operations in the upper abdomen are
most prone to development of atelectasis.

(Option A) Wound infection presents more delay around 7th day post operation. Clinical signs of infection such as erythema, local tenderness,
and/or purulent secretions are often found on clinical examination.

(Option B) Pulmonary embolism typically occurs around 5th day post-operation, and presents with pleuritic chest pain, tachycardia, and
tachypnea. The patient may or may not be febrile. It is less likely that a patient develops pulmonary embolism within the first day of surgery.

(Option D) Leakage of bile presents with general malaise, abdominal pain, low-grade fever, and mildly elevated liver function tests. Shortness
of breath and tachycardia are not common features.

(Option E) Pneumonia often complicates atelectasis at around 3rd day post-op. It is early for development of pneumonia.

References

• UpToDate - Overview of the management of postoperative pulmonary complications

• Medscape - Atelectasis

• AMC of Handbook Multiple Choice Questions – pages 183-184

Last updated:
Time spent: QID:1117
2023-2-12

274 of 1943
Eighteen hours after an uneventful cholecystectomy on a 45-year-old woman, she develops a fever and a cough. A chest X-ray (CXR) is
obtained that is shown in the following photograph. Which one of the following is the next best step in management?

A. Chest physiotherapy.

B. Oral antibiotics.

C. Intravenous antibiotics.

D. Intravenous steroids.

E. Morphine.

Correct
45% answered correctly

Explanation:

Correct Answer Is A

Of significance on the CXR, is the loss of the right heart border shadow, indicating collection in the right middle lobe. The onset of symptoms
within the first 24 hours post-operatively, in addition to the CXR findings is consistent with the diagnosis of atelectasis. Atelectasis is common
in abdominal and trans-thoracic procedures. Mucus retention in airways leads to post-obstructive collapse of the distal airways, particularly in
the lower parts of the lung. Pneumonia can complicate atelectasis.

Post-operative atelectasis is managed as follows:

Removal of impacted secretion by percussion and encouraging the patient to actively cough.
Passive postural drainage.

The most important step in management is chest physiotherapy, followed by oxygen supplementation. If the above measures failed, a
catheter can be passed through, guided by bronchoscope, for more vigorous removal of the secretions.

(Options B and C) Antibiotics are not indicated unless fever persists more than 48 hours, suggesting superimposed pulmonary infection
(pneumonia).

275 of 1943
(Option D) Steroids have no role in management of atelectasis.

(Option E) Morphine is not indicated for management of atelectasis. Furthermore, morphine can decrease respiratory drive and worsens the
atelectasis.

References

• Medscape - Atelectasis

• UpToDate - Overview of the management of postoperative pulmonary complications

Last updated:
Time spent: QID:1123
2023-2-12

276 of 1943
A 60-year-old man develops fever, tachypnea and shortness of breath 8 hours after a surgical procedure. A Chest X-ray show atelectasis of the
lower lobe of the right lung. He has an oxygen saturation of 89% on room air. The patient undergoes chest physiotherapy, deep breathing, and
incentive spirometry, but there is no significant improvement. Which one of the following is the next best in management?

A. Antibiotics.

B. CT angiogram.

C. Sputum examination.

D. Continuous positive airway pressure (CPAP) while arranging for bronchoscopy.

E. Continuation of physiotherapy.

Incorrect. Correct answer is D


45% answered correctly

Explanation:

Correct Answer Is D

Atelectasis is common among patients who have undergone surgery especially in the setting of general anesthesia and use of opiate
analgesics.

The initial management of atelectasis is with chest physiotherapy, deep breathing, active coughing, and incentive spirometry, followed by
administration of supplemental oxygen. At occasions, these measures fail resolve the problem, mostly due to blockage of respiratory tract
with a large mucus plug. The treatment of choice in this situation is bronchoscopic removal of the plug. In the interim, the patient must be put
on continuous positive airway pressure for improved oxygenation.

(Option A) Antibiotics are not required unless chest infection is suspected.

(Option B) CT angiogram would be indicated if pulmonary embolism (PE) was suspected. PE typically develops around 5 days post-operation.
Although not impossible, PE is less likely of a diagnosis given the early onset of symptoms. PE presents with sudden-onset pleuritic chest
pain, shortness of breath, tachypnea, and tachycardia often around day 5 post-operation. With a massive PE, hemodynamic instability
develops.

(Option C) Sputum examination might be indicated if chest infection is suspected. It is not a part of workup for atelectasis.

(Option E) After stabilization of the patient, continuous physiotherapy is recommended for prevention of further atelectasis; however, at this
acute setting, it is unlikely to immediately relieve the shortness of breath and hypoxia.

References

• Medscape - Atelectasis

• UpToDate - Overview of the management of postoperative pulmonary complications

Last updated:
Time spent: QID:1124
2023-2-12

277 of 1943
On the sixth day after an uneventful appendectomy on a 23-year-old man due to gangrenous appendicitis, he develops a fever. On
examination, he has a blood pressure of 135/90 mmHg, pulse rate of 94bpm, and a temperature of 39°C. A small fluctuating tender swelling is
noted on the lateral side of the wound. Which one of the following would be the most appropriate next step in management?

A. Oral antibiotics for 14 days.

B. Ultrasonography of the wound area.

C. CT scan of the abdomen.

D. Incision of the wound and drainage.

E. Intravenous antibiotics for 3 days, followed by oral antibiotics.

Incorrect. Correct answer is D


45% answered correctly

Explanation:

Correct Answer Is D

Wound infection and abscess formation are common post-operative complications of surgical procedures. Usually wound infections are
expected on day 7 post-operation or thereabouts. Abscess formation usually takes longer; nonetheless these timings are inaccurate, and
surgical complications can potentially occur at any time during post-operative period. Simple wound infections present with fever, erythema,
and local tenderness.

Abscesses, on the other hand, are more indurated and fluctuating, and characteristically contain puss. Abscesses tend to be associated with
higher fevers.

Superficial wounds infections can be managed by suture removal and no antibiotics. Wound infections with cellulitis alone but no fluctuance
to indicate underlying abscess can be treated with a course of antibiotics without drainage. However, when the clinical findings suggest an
abscess, incision and drainage is the mainstay of therapy.

The fluctuating tender swelling in this patient is highly likely to be an abscess. Moreover, he has systemic symptoms of infection. In such
circumstance, the most appropriate management is incision of the abscess and drainage. He also needs intravenous antibiotics. The wound
should be left open for daily sterile packing.

(Option A) Oral antibiotics are insufficient for management of this wound infection. Such infections often require intravenous antibiotics after
incision and drainage has been carried out.

(Options B and C) Wound infection can often be differentiated from abscesses on clinical basis. If they cannot be differentiated,
ultrasonography is the investigation of choice to distinguish them. CT scan is not the favored diagnostic tool for this purpose. Here, the
clinical findings are sufficiently diagnostic for an abscess without need for further evaluation.

(Option E) Antibiotics alone are not sufficient for management of wound abscesses but are used in conjunction with incision and drainage.

References

• RACGP - AFP - Post-operative wound management

Last updated:
Time spent: QID:1125
2023-2-12

278 of 1943
While doing exercise, a 16-year-old girl fell from a machine in the gym and had her head hit against the floor. She did not lose consciousness
and remembers the incident. On examination, she is fully oriented, and has no neurological deficits. However, there is a swelling over the left
side of the scalp. Which one of the following would be the next best step in management?

A. CT scan of the head.

B. Skull X-ray.

C. Observation for 6 hours and discharge if no neurological derangement develops.

D. Admit her to the hospital and observe for 24 hours.

E. Reassure and discharge home.

Incorrect. Correct answer is C


45% answered correctly

Explanation:

Correct Answer Is C

Management of head trauma depends on its severity. Clinically, head trauma is categorized as minor, moderate or severe. Clinical features of
each type are shown in the following table:

No loss of consciousness
Up to 1 episode of vomiting
Stable and alert conscious state
Minor May have scalp bruising or laceration
Normal examination otherwise

Brief loss of consciousness at the time of injury


Currently alert or responds to voice
May be drowsy
Two or more episodes of vomiting
Moderate Persistent headache
Up to one single brief (<2min) convulsion occurring immediately after the impact
May have a large scalp bruise, hematoma or laceration
Normal examination otherwise

Decreased conscious state – responsive to pain only or unresponsive


Localizing neurological signs (unequal pupils, lateralizing motor weakness)
Signs of increased intracranial pressure:

Uncal herniation: Ipsilateral dilated non-reactive pupil due to compression of the oculomotor nerve

Central herniation: Brainstem compression causing bradycardia, hypertension and irregular


Severe
respirations (Cheynes-Stokes)
Decorticate: arms flexed, hands clenched into fists, legs extended, feet turned inward
Decerebrate: head arched back, arms extended by the sides, legs extended, feet turned inward

Penetrating head injury


CSF leak from nose or ears

Management

Management of head trauma depends on the severity and is as follows:

Minor head injury

Observation for up to 6 hours with examination of vital signs, GCS, pupils, and limbs every 30 minutes
Patient may be discharged from the emergency department for home care after carers are instructed about alarm signs
If there is any doubt as to whether there has been loss of consciousness or not, assume there has been and treat as for moderate
head injury.

279 of 1943
Adequate analgesia

Moderate Head Injury

If, on the history from the carers and/or ambulance, the patient is not neurologically deteriorating, he/she may be observed in the
emergency department for a period of up to 6 hours after trauma with neurological observations every 30 minutes(conscious state,
PR, RR, BP, pupils and limb power).
The patient may be discharged home if there is improvement to normal conscious state, no further vomiting occurs, and patient is
able to tolerate oral fluids.
A persistent headache, large hematoma or possible penetrating wound may need further investigation and discussion with a
consultant.
Adequate analgesia.
Consider anti-emetics - consider a longer period of observation if anti-emetics are given.

NOTE - The is no clear guidelines as to the exact time of observation of a patient with head trauma at the hospital.

Severe head injury

The initial aim of management of a patient with a serious head injury is prevention of secondary brain damage. The key steps are to maintain
oxygenation, ventilation, and circulation, and to avoid rises in intracranial pressure (ICP). Management includes:

Urgent CT of head and c-spine. Ensure early neurosurgical and ICU intervention.

Cervical spine immobilization should be maintained even if cervical spine imaging is normal.

Intubation and ventilation if:

Unresponsive or not responding purposefully to pain


GCS persistently <8
Loss of protective laryngeal reflexes
Respiratory irregularity

In consultation with the neurosurgeon consider measures to decrease intracranial pressure:

Head elevation by 20-30 degrees (after correction of shock) and head in midline position to help venous drainage.
Ventilate to a target PCO2 35mmHg (4-4.5 kPa)
Ensure adequate blood pressure with crystalloid infusion or inotropes (e.g. noradrenaline) if necessary.
Consider mannitol or hypertonic saline
Consider phenytoin loading dose (20 mg/kg over 20 minutes, intravenously).

Control seizures if they occur


Correction of hypoglycemia
Analgesia: sufficient analgesia should be administered by careful titration

The following conditions need emergency transfer to a tertiary medical facility (if not already done):

All severe head injuries


Deteriorating conscious level (especially motor response changes)
Focal neurological signs
Seizure without full recovery
Definite or suspected penetrating injury
Cerebrospinal fluid leak
Care required beyond the comfort level of the hospital

With no loss of consciousness, no focal finding on neurological exam, but a swelling (probably hematoma), this patient probably has a
moderate head injury and requires to be closely observed for 4 hours.

References

• The Royal Children's Hospital (RCH) - Head trauma

• RACGP (AFP) - Minor head injuries in children

Last updated:
Time spent: QID:320
2023-2-12

280 of 1943
An otherwise fit 57-year-old woman spikes a fever of 39°C five days after an appendectomy for acute appendicitis. On examination, there is a
tender, erythematous and fluctuant swelling at the medial end of the wound. Which one of the following options would be the most
appropriate next step in management?

A. Administer high dose, broad spectrum antibiotics, intravenously.

B. Return the patient to the OR for resuturing the wound.

C. Start the patient on oral antibiotics.

D. Make an incision over the swelling to allow free draining.

E. Remove the underlying suture material to prevent sinus formation.

Incorrect. Correct answer is D


45% answered correctly

Explanation:

Correct Answer Is D

The tender erythematous fluctuant swelling at the surgical site suggests abscess formation as a post-operative complication of the surgical
wound. Once abscess is formed, incision and drainage are the most appropriate management to considered first. Intravenous antibiotics are
indicated as an adjunctive therapy.

Wound infection and abscesses are common post-operative complications of surgical procedures. Often, wound infections are expected on
the around the 7th day post-op. Abscess formation usually takes longer; nonetheless, these timings are inaccurate, and surgical complications
can potentially occur at any time during the post-operative period. Simple wound infections present with surrounding cellulitis (erythema and
local tenderness). Abscesses, on the other hand, are more indurated and fluctuating and characteristically contain puss, and tend to cause
higher fevers.

(Option A) Intravenous antibiotics are used after incision is made to let free drainage of the abscess. Intravenous antibiotics are necessary,
particularly in this patient with systemic infection and high fever but are not sufficient if used alone. Wound infections with cellulitis but no
fluctuance can be treated with a course of antibiotics without drainage. However, when clinical findings suggest an abscess, incision and
drainage is the mainstay of therapy.

(Option B) Opening the wound, irrigation and re-suturing is not an effective management for abscesses. The sutures are removed, wound
debridement and irrigation is performed, but the wound is left open and packed.

(Option C) Oral antibiotics are insufficient for management of this wound infection. Such infections often require intravenous antibiotics after
incision and drainage has been carried out.

(Option E) Removal of the underlying suture material to prevent sinus formation is important but does not treat the patient’s current condition.

References

• RACGP - AFP - Post-operative wound management

Last updated:
Time spent: QID:1126
2023-2-12

281 of 1943
A nurse from the surgery ward calls you to visit a 65-year-old inward patient for agitation. When you arrive at the ward you are informed
that she underwent cholecystectomy 48 hours ago. Her file shows that she received prophylactic amoxicillin prior to the surgery. On
examination, she is confused and delirious, has a blood pressure of 135/87 mmHg, heart rate of 110, and temperature of 38.4°C. She is
agitated and difficult to deal with. Which one of the following is the most appropriate next step in management?

A. Start her on ceftriaxone.

B. Give her haloperidol.

C. Intravenous fluids.

D. Chest X-ray.

E. ABG.

Incorrect. Correct answer is B


45% answered correctly

Explanation:

Correct Answer Is B

Delirium following a major surgery in the elderly is a common postoperative complication. Delirium and confusional state after surgery are
most commonly caused by hypoxia. Hypoxia can be caused by the effect of hypoventilation due to anesthetics or analgesics, or simply by
shallow breathing due to pain.

Fluid and electrolyte disturbances, hypoglycemia, or infections can also cause post-operative delirium. Alcohol withdrawal or delirium tremens
are other important etiologies not to miss. Urinary retention or fecal impaction should be thought of as well.

Management of delirium is by identification of the underlying etiology and treating it. This woman is febrile, making infection a likely cause of
her delirious state. Although prophylactic antibiotics decrease the risk of post-operative infections, they do not elimnated such risk.

A confused and delirious patient can be difficult to deal with at times, and pharmacological treatment should be considered for sedation
before other diagnostic or therapeutic measures are carried out. An ABG is often the very initial step to exclude hypoxemia as the most likely
cause of delirium but attempts to obtain an arterial sample in an agitated delirious patient is not easy and may result in arterial injuries in a
combative patient. An agitated patient may not let an oxygen face mask to be fixed in place. Under such conditions, medications are used to
sedate and calm the patient.

Haloperidol is a convenient drug, which is most frequently used for this purpose and is the most appropriate next step in management.

Atypical antipsychotics such as risperidone, olanzapine or ziprasidone are second-line choices that can be used as alternatives; however, in
the presence of extrapyramidal symptoms, they should be use in preference to haloperidol. Intravenous diazepam (not an option) is the
medication of choice if alcohol withdrawal or delirium tremens are suspected from the history and clinical findings.

NOTE - Pharmacological therapy should only be considered in a delirious person with severe behavioral disturbance and/or severe emotional
disturbance if their behavior threatens their own safety or the safety of others, is likely to interfere with essential medical or nursing care, or
if the disturbance is causing significant distress

(Option A) Although the patient is febrile, antibiotics should not be administered unless bacterial infections are suspected after initial
investigations such as full blood count (FBC), chest X-ray, etc.

(Option B) This patient has a normal blood pressure and does not seem to be in urgent need for fluid resuscitation. However, it may be
considered if further studies indicate otherwise.

(Options D and E) ABG is often the first-line investigation to exclude hypoxia as the most common cause of post-operative
delirium/confusion. A chest X-ray is indicated for evaluation and workup of conditions such as atelectasis, pneumonia, or pulmonary
embolism if the patient is found to be hypoxic/hypoxemic. Again, obtaining an arterial sample for ABG can be difficult in this patient and
should be attempted once the patient is easier to deal with.

References

• Clinical Practice Guidelines for the Management of Delirium in Older People

• Delirium Care Pathways

Last updated:
Time spent: QID:1131
2023-2-12

282 of 1943
John, 5 years of age, falls from a swing and injures his right leg. Accompanied by his parents, he is now in the Emergency Department. He has
a 5 mm deep laceration in a 2x3 abrasion on his right leg. The wound is contaminated with soil. You disinfect the wound and ask his parents
about his tetanus vaccination status. He has received 2 doses of DTPa at 2 and 4 months of age, but not the third dose at 6 months or
thereafter. Which one of the following is the appropriate management plan for him regarding tetanus vaccination?

A. One dose of DTPa now and a booster dose after 2 months.

B. One dose of DTPa plus tetanus immunoglobulin (TIG) now.

C. One dose of dT plus TIG now.

D. One dose of DTPa now plus a 5-day course of topical antibiotic cream.

E. One dose of DTPa nowplus a 5-day course of oral penicillin.

Incorrect. Correct answer is B


45% answered correctly

Explanation:

Correct Answer Is B

This child has a dirty tetanus-prone wound. Since there is no straight and clear definition for a tetanus-prone wound, all wounds other than
clean, minor cuts should be considered tetanus-prone, with emphasis on the fact that some certain types of wounds are more prone to growth
of C. tetani (dirty wounds). Such wounds include:

Bite wounds
Compound open fractures
Foreign-body containing wounds (wood splinters in particular)
Wounds complicated by pyogenic infections
Wounds with significant tissue damage such as contusions and burns
Deep penetrating wounds
Superficial wounds contaminated with feces, dirt, soil, manure and dust, especially if disinfection is delayed more than 4 hours
Wounds older than 6 hours

These wound are considered as dirty wounds.

NOTE - Reimplantation of an avulsed tooth is also considered a tetanus-prone event. The reason is that minimal washing and cleaning of
the tooth is mandatory to increase the likelihood of successful reimplantation.

Management of a wound for tetanus prophylaxis depends mainly on the history of prior tetanus vaccination and whether the wound is clean
or dirty, and is outlined in the following table:

283 of 1943
Clean wound and less No tetanus containing vaccine is needed
than 10 years has No tetanus immune globulin (TIG) is needed
elapsed since the last
primary or booster
dose
If up-to-date with schedule vaccines:

No TIG is needed, but


Give dT - if aged 9–10 years OR immunization event
Clean wound and 10 due, replace dT with Schedule dTpa
years or more has
elapsed since the last If not up-to-date with Schedule vaccines:
primary or booster
dose No TIG is needed, but
Give age appropriate tetanus immunization
immediately and if needed, arrange additional catch-
Complete primary up immunization*.
course (3 doses at
ages 2, 4 and 6 Dirty wound and less No tetanus-containing vaccine is needed
months) than 5 years has No TIG is needed
elapsed since the last
primary or booster
dose
If up-to-date with schedule vaccines:

No TIG is needed, but


Give dT - if aged 9–10 years OR immunization event
Dirty wound and 5 due, replace dT with Schedule dTpa
years or more has
elapsed since the last If not up-to-date with Schedule vaccines:
primary or booster
dose No TIG Is needed, but
Give age appropriate tetanus immunization
immediately and if needed, arrange additional catch-
up immunization*.

No TIG is needed, but


Give age appropriate tetanus immunization
immediately and if needed, arrange additional catch-
No prior tetanus up immunization*.
immunization
Clean wound

OR

Incomplete primary
course Give TIG

OR AND

Unknown primary Dirty wound


Give age appropriate tetanus immunization
vaccination status immediately and if needed, arrange additional catch-
up immunization*.

Age up to 10 years (i.e. up to the 10th birthday): a DTPa-containing vaccine


Age 10-18 years (i.e. up to the 18th birthday): dTpa
Age 18 years or older: dT

Tetanus toxoid is available in Australia only in combination with diphtheria, with or without other antigens such as pertussis, inactivated
poliomyelitis, hepatitis B and hemophilus influenza type b(Hib).

While the acronym DTPa (capital letters) signifies child formulations of combined diphtheria, tetanus and acellular pertussis vaccines, dTpa is
used for formulations containing significantly lesser amounts of diphtheria and acellular vaccines. Such formulations are used for
284 of 1943
adolescents and adults.

John has not completed his primary course of tetanus vaccination. On the other hand, his wound is contaminated with soil and is dirty. With
an incomplete course of primary immunization and a dirty wound, he should receive TIG for passive and a tetanus-containing vaccine
immediately. Since he is under the age of 10 years, the appropriate vaccine for him is a DTPa containing vaccine.

(Option A) John needs an immediate dose of a DTPa vaccine now (as the missed third dose). He also needs another dose as booster at least
4 weeks (one month) later. Although,the vaccination program offered is correct, the absence of TIG as a very important and crucial step in this
option makes it incorrect.

(Option C) Although immediate vaccination and TIG are steps to take for management of the tetanus-prone wound in this child, dTpa is not
the age-appropriate vaccine to use in this setting and DTPa should be used instead. dTpa is used for those older than 10 years of age but
younger than 18 years.

(Options D and E) Antibiotics are not required for management of tetanus, but may be used for prevention of other wound infections. The
decision as to whether prophylactic antibiotic coverage is given is a clinical decision made on a patient-to-patient basis. However, since these
two option do not offer TIG, they cannot be accepted as correct options.

References

• Immunisation Hanbook - Guide to tetanus prophylaxis in wound management

Last updated:
Time spent: QID:1222
2023-2-12

285 of 1943
John, 15 years of age, is a patient of yours since he was born. Today, his mother has brought him to your office after his left hand was bitten
by his dog. The bite happened when John accidentally stepped on the dog's tail and provoked the animal. On examination, there is a puncture
wound which slightly bleeds when you remove the bandage the mother put on at home. No associated bone or tendon injury is found. You
check his for his immunization status and know that his primary and booster vaccines (a total of 5 doses) has been given with the last
booster at the age of 4 years. You debride the wound, disinfect it, and put a non-adherent absorbent plaster as covering. You also prescribe a
5-day course of co-amoxiclave for prevention of infections. Since it is a deep bite wound, you decide to also be on the safe side and give him
prophylaxis against tetanus. Which one of the following would be the best course of action for tetanus prophylaxis for him?

A. A booster dose with DTPa (child formulation of diphtheria, tetanus, and acellular pertussis).

B. Tetanus toxoid (TT).

C. Adult diphtheria and tetanus toxoids (dT).

D. TT plus tetanus immune globulin (TIG).

E. A dose of combination vaccine of diphtheria toxoid plus tetanus toxoid plus whole cell pertussis (DTPw).

Incorrect. Correct answer is C


45% answered correctly

Explanation:

Correct Answer Is C

Due to high rates of mortality associated with C. tetani infection, it is important that every person with a tetanus-prone wound be assessed for
the need of post-exposure prophylaxis against tetanus.

There is no clear definition of a tetanus-prone injury, and every wound, other than clean minor cuts, is considered tetanus-prone. Of these
wounds, the following are defined as dirty with higher potentials for growth of the germ, severe infection, and poorer outcomes:

Bite wounds
Compound open fractures
Foreign-body containing wounds (wood splinters in particular)
Wounds complicated by pyogenic infections
Wounds with significant tissue damage such as contusions and burns
Deep penetrating wounds
Superficial wounds contaminated with soil, manure and dust, especially if disinfection is delayed more than 4 hours
Wounds older than 6 hours

NOTE - Reimplantation of an avulsed tooth is also considered a tetanus-prone event. The reason is that minimal washing and cleaning of
the tooth is mandatory to increase the likelihood of successful reimplantation.

Management principles of a tetanus-prone wound relies on the previous history of tetanus immunization of the injured person as well as the
nature of the wound: clean versus dirty.

The following table summarizes the management plan for tetanus-prone wounds based on the immunization history and the nature of the
wound:

286 of 1943
Clean wound and less No tetanus containing vaccine is needed
than 10 years has No tetanus immune globulin (TIG) is needed
elapsed since the last
primary or booster
dose
If up-to-date with schedule vaccines:

No TIG is needed, but


Give dT - if aged 9–10 years OR immunization event
Clean wound and 10 due, replace dT with Schedule dTpa
years or more has
elapsed since the last If not up-to-date with Schedule vaccines:
primary or booster
dose No TIG is needed, but
Give age appropriate tetanus immunization
immediately and if needed, arrange additional catch-
Complete primary up immunization*.
course (3 doses at
ages 2, 4 and 6 Dirty wound and less No tetanus-containing vaccine is needed
months) than 5 years has No TIG is needed
elapsed since the last
primary or booster
dose
If up-to-date with schedule vaccines:

No TIG is needed, but


Give dT - if aged 9–10 years OR immunization event
Dirty wound and 5 due, replace dT with Schedule dTpa
years or more has
elapsed since the last If not up-to-date with Schedule vaccines:
primary or booster
dose No TIG Is needed, but
Give age appropriate tetanus immunization
immediately and if needed, arrange additional catch-
up immunization*.

No TIG is needed, but


Give age appropriate tetanus immunization
immediately and if needed, arrange additional catch-
No prior tetanus up immunization*.
immunization
Clean wound

OR

Incomplete primary
course Give TIG

OR AND

Unknown primary Dirty wound


Give age appropriate tetanus immunization
vaccination status immediately and if needed, arrange additional catch-
up immunization*.

Age up to 10 years (i.e. up to the 10th birthday): a DTPa-containing vaccine


Age 10-18 years (i.e. up to the 18th birthday): dTpa
Age 18 years or older: dT

John has had a full course of tetanus vaccination on recommended schedule. His wound is a deep bite wound which by definition (above) is
dirty. The decision as to the need for tetanus prophylaxis and how to do that is based on the time that has elapsed since his last booster. For
dirty wounds in a person with complete primary vaccination and the last booster dose within the previous 5 years neither vaccine nor TIG is
indicated; however, if more than 5 years has passed since the last tetanus containing vaccine, an immediate booster dose will be required.
The preferred vaccine depends on the age:

287 of 1943
If ≤ 10 years the booster dose should be given using a DTPa vaccine. The acronym DTPa (capital letters) signifies child formulations
of combined diphtheria, tetanus and acellular pertussis.
If >10 years, dTpa is used. dTpa signifies formulations that contain significantly lesser amounts of diphtheria and acellular pertussis
vaccines. Such formulations are used for adolescents and adults.

For John, who is older than 10 years, a single dose of dTpa (preferred but not an option) or a combination of diphtheria and tetanus toxoid
vaccines (dT) is the most appropriate action to take.

(Option A) DTPa is a child formulation tetanus vaccine (up to 10th birthday) and not used for older persons.

(Option B and D) TT alone is not available in Australia and other age-appropriate vaccines should be considered. On the other hand, John does
not need TIG.

(Option E) Whole-cell pertussis vaccines are suspensions of the entire B. pertussis organisms that has been inactivated, usually with formalin.
Whole-cell pertussis is available in combination with diphtheria and tetanus vaccines (DTPw). Immunization with vaccines containing whole-
cell pertussis have been frequently associated with minor adverse reactions such as erythema and swelling at the site of injection, fever and
agitation. DTPa replaced DTPw for booster doses in 1997, and for all doses from 1999; therefore, not an appropriate option for John.

References

• Immunisation Hanbook - Guide to tetanus prophylaxis in wound management

Last updated:
Time spent: QID:1223
2023-2-12

288 of 1943
While working in a factory, a 26-year-old machinist sustains a hand injury with a cutting machine. The wound is deep with bone exposure and
crushed surrounding tissue. The wound is irrigated with copious amounts of normal saline and dressed accordingly. The patient is arranged
to have his wound repaired in the operating room. The surgeon advises intravenous antibiotics for prophylaxis and tetanus prevention. He had
a full course of tetanus vaccination as a child and also received a booster dose 3 years ago. Which one of the following is the most
appropriate option for him regarding tetanus prophylaxis?

A. Human tetanus immune globulin (TIG), 250 IU, via intramuscular injection.

B. A single dose of dT (diphtheria/ tetanus vaccine).

C. Tetanus prophylaxis is not indicated for this patient.

D. dTpa plus 250 IU of TIG.

E. A single dose of DTPa.

Incorrect. Correct answer is C


45% answered correctly

Explanation:

Correct Answer Is C

All wounds other than clean minor cuts should be considered tetanus-prone and promptly managed according to current recommendations.

The decision as to tetanus prophylaxis is based on two very important features:

1. Tetanus-immunization history
2. Dirty versus clean nature of the wound

In addition to being tetanus-prone, the following wounds are considered dirty:

Bite wounds
Compound open fractures
Foreign-body containing wounds (wood splinters in particular)
Wounds complicated by pyogenic infections
Wounds with significant tissue damage such as contusions and burns
Deep penetrating wounds
Superficial wounds contaminated with soil, manure and dust, especially if disinfection is delayed more than 4 hours
Wounds older than 6 hours

The following table summarizes the management plan for tetanus-prone wounds based on the immunization history and the nature of the
wound:

289 of 1943
Clean wound and less No tetanus containing vaccine is needed
than 10 years has No tetanus immune globulin (TIG) is needed
elapsed since the last
primary or booster
dose
If up-to-date with schedule vaccines:

No TIG is needed, but


Give dT - if aged 9–10 years OR immunization event
Clean wound and 10 due, replace dT with Schedule dTpa
years or more has
elapsed since the last If not up-to-date with Schedule vaccines:
primary or booster
dose No TIG is needed, but
Give age appropriate tetanus immunization
immediately and if needed, arrange additional catch-
Complete primary up immunization*.
course (3 doses at
ages 2, 4 and 6 Dirty wound and less No tetanus-containing vaccine is needed
months) than 5 years has No TIG is needed
elapsed since the last
primary or booster
dose
If up-to-date with schedule vaccines:

No TIG is needed, but


Give dT - if aged 9–10 years OR immunization event
Dirty wound and 5 due, replace dT with Schedule dTpa
years or more has
elapsed since the last If not up-to-date with Schedule vaccines:
primary or booster
dose No TIG Is needed, but
Give age appropriate tetanus immunization
immediately and if needed, arrange additional catch-
up immunization*.

No TIG is needed, but


Give age appropriate tetanus immunization
immediately and if needed, arrange additional catch-
No prior tetanus up immunization*.
immunization
Clean wound

OR

Incomplete primary
course Give TIG

OR AND

Unknown primary Dirty wound


Give age appropriate tetanus immunization
vaccination status immediately and if needed, arrange additional catch-
up immunization*.

Age up to 10 years (i.e. up to the 10th birthday): a DTPa-containing vaccine


Age 10-18 years (i.e. up to the 18th birthday): dTpa
Age 18 years or older: dT

This man has completed his schedule vaccines and his wound is deep and associated with crushed tissue both of which, by definition, make
the wound a dirty one. For dirty wounds in a patient with complete vaccination, the decision relies on the time elapsed since the last booster
dose. While those who have received their last booster dose more than 5 years ago need another immediate booster dose with a tetanus-
containing vaccine, people with a booster vaccine within the past 5 years do not need further action for tetanus prophylaxis; therefore, no
treatment for tetanus prevention is required for this patient.

290 of 1943
(Options A, B and D) TIG is only indicated, for passive protection against tetanus in conjunction with a tetanus-containing vaccine, in persons
with no, incomplete or uncertain tetanus vaccination history, who have a dirty wound. With complete vaccination this man does not need TIG.
This man does not require any form of tetanus vaccines either.

(Option E) DTPa is a child formulation tetanus vaccine (up to 10th birthday) and not used for older persons. dTpa is the preferred formulation
for children aged 10 to 18 years. For older individuals (≥19 years) dT (diphtheria and tetanus but no pertussis) is also an option. Tetanus
toxoid alone (TT) is not currently available in Australia.

References

• Immunisation Hanbook - Guide to tetanus prophylaxis in wound management

Last updated:
Time spent: QID:1224
2023-2-12

291 of 1943
A 50-year-old man presents to the Emergency Department with a laceration in his left lower leg caused by pickax while farming. The wound is
1.5 cm deep and is contaminated with soil. You debride the wound, disinfect it, and put a non-adherent bandage on it. On enquiry about
tetanus vaccination, you realize that he had his tetanus vaccination completed. He also had a hand cut 5 weeks ago, for which he received a
single dose of dT. Which one of the following is the appropriate action for management of this patient?

A. Start him on a 5-day course of oral penicillin for wound infection prophylaxis.

B. Give him250 IU of human tetanus immune globulin (TIG).

C. Give him one dose of tetanus toxoid (TT).

D. Arrange for follow-up to check the need for antibiotics and discharge home.

E. Give him one dose of dT.

Incorrect. Correct answer is D


45% answered correctly

Explanation:

Correct Answer Is D

Tetanus is an acute disease which is often fatal. The disease is caused by clostridium tetani (C. tetani), a non-capsulated gram-positive rod
and the neurotoxin it produces. The neurotoxin acts on the central nervous system resulting in muscle rigidity and painful muscle spasms.
The incubation period is 3 to 21 days with a median of 10 days; however, it could be as short as 1 day or as long as several months. Shorter
incubation periods are seen with significantly contaminated wounds. In such instances. a more severe clinical course and a poorer prognosis
would be expected.

The disease initially presents with spams of the masseter muscles, dysphagia, neck, shoulder and back muscles pain and stiffness that may
progress to generalized muscle spasms. Reduced ventilation, apnea and/or laryngospasm can occur. The cause of death is respiratory failure
in most patients. Deep vein thrombosis, pulmonary embolism, rhabdomyolysis, fractures, muscle rupture, hypotension, hypertension, and
cardiac arrest are other possible complications.

Due to high rates of mortality associated with C. tetani infection, it is very important that every person with a tetanus-prone wound be
assessed for the need for post-exposure prophylaxis against tetanus.

There is no clear definition for a tetanus-prone injury, and every wound other than clean minor cuts is considered tetanus-prone. Of these
wounds, the following are defined as dirty with higher potentials for growth of the germ, severe infection, and poorer outcomes:

Bite wounds
Compound open fractures
Foreign-body containing wounds (wood splinters in particular)
Wounds complicated by pyogenic infections
Wounds with significant tissue damage such as contusions and burns
Deep penetrating wounds
Superficial wounds contaminated with soil, manure, and dust, especially if disinfection is delayed more than 4 hours
Wounds older than 6 hours

NOTE - Reimplantation of an avulsed tooth is also considered a tetanus-prone event. The reason is that minimal washing and cleaning of
the tooth is mandatory to increase the likelihood of successful reimplantation.

Management principles of a tetanus-prone wound relies on the previous history of tetanus immunization of the injured person as well as
whether the wound is clean or dirty.

Clean minor cuts are not categorized as tetanus-prone wounds and for these wounds TIG is unnecessary, independent of previous tetanus
vaccination history.

Adults with injuries considered to be tetanus-prone (all wounds other than clean minor cuts) should receive a booster dose of dT if more than
5 years have passed since their last dose of tetanus-containing vaccine. dTpa can be used as an alternative for provision of additional
protection against pertussis. In children <10 years of age, this dose of vaccine should be given as DTPa or a DTPa-combination vaccine,
consistent with the child’s vaccination history and the recommended schedule. If there is any doubt about the adequacy of previous tetanus
immunization in a person who has a tetanus-prone wound, TIG must be given as soon as possible, as well as tetanus toxoid-containing
vaccine to provide both immediate passive and active protection.

292 of 1943
The recommended dose for TIG is 250 IU, through intramuscular injection as soon as practicable following the injury. The dose should be
increased to 500 IU if more than 24 hours have passed since the injury.

NOTE - Individuals with significant humoral immune deficiency may not have developed or maintained adequate immunity to tetanus,
despite vaccination. Such persons require TIG for tetanus-prone wounds.

This man’s tetanus vaccination is complete. He also has received a booster dose 5 weeks ago (less than 5 years); therefore, he does not need
further tetanus prophylaxis either active (vaccine) or passive (TIG) despite the fact his has a tetanus-prone and dirty wound. However, follow-
up is required for treatment of potential wound complications including other wound infections.

(Option A) Antibiotic prophylaxis is not indicated for prevention of tetanus, but its use for prevention of other bacterial infections is matter of
clinical judgement. The best action, as offered as an option, is a follow-up to check if there are any signs of wound infection before the
decision as to commencement of antibiotics is made.

(Option B) TIG is indicated in cases with unknown, uncertain, or incomplete tetanus immunization with dirty wounds.

(Options C and E) Tetanus toxoid is available in Australia only in combination with diphtheria, with or without other antigens such as pertussis,
inactivated poliomyelitis, hepatitis B and Hemophilus influenza type b(Hib). Tetanus toxoid (TT) alone is not an available option for this
patient. Moreover, he does not need tetanus vaccine (dT or dTpa) or immunoglobulin.

References

• Immunisation Handbook - Guide to tetanus prophylaxis in wound management

Last updated:
Time spent: QID:1225
2023-2-12

293 of 1943
A 57-year-old farmer presents to the Emergency Department after he accidentally steped on a rake in the farm and injured his left foot. On
examination, there are 3 puncture wounds on his left sole which are not actively bleeding but the edges of the entry wounds are contaminated
with soil. He has had a full course of tetanus vaccination including several boosters with the last one being 6 years ago. After disinfecting the
wounds and putting non-adherent bandage over them, which one of the following would be the best action regarding post-exposure
prophylaxis of tetanus in this patient?

A. Another tetanus booster with DTPa.

B. No tetanus immunization is required.

C. Another tetanus booster with dT.

D. Tetanus toxoid in form of dT and tetanus immune globulin (TIG).

E. TIG and a course of oral penicillin.

Incorrect. Correct answer is C


45% answered correctly

Explanation:

Correct Answer Is C

In management of a tetanus-prone wound there are two important features that guide the prophylaxis plan. The first one is the immunization
status of the patient, and the second is whether the wound is clean or dirty.

Since there is no straight and clear definition for a tetanus-prone wound, all wounds other than clean, minor cuts should be considered
tetanus-prone, with emphasis on the fact that some certain types of wounds are more prone to growth of C. tetani. Such wounds include:

Bite wounds
Compound open fractures
Foreign-body containing wounds (wood splinters in particular)
Wounds complicated by pyogenic infections
Wounds with significant tissue damage such as contusions and burns
Deep penetrating wounds
Superficial wounds contaminated with soil, manure and dust, especially if disinfection is delayed more than 4 hours
Wounds older than 6 hours

NOTE - Reimplantation of an avulsed tooth is also considered a tetanus-prone event. The reason is that minimal washing and cleaning of
the tooth is mandatory to increase the likelihood of successful reimplantation.

Management of tetanus-prone wounds is outlined in the following table:

294 of 1943
Clean wound and less No tetanus containing vaccine is needed
than 10 years has No tetanus immune globulin (TIG) is needed
elapsed since the last
primary or booster
dose
If up-to-date with schedule vaccines:

No TIG is needed, but


Give dT - if aged 9–10 years OR immunization event
Clean wound and 10 due, replace dT with Schedule dTpa
years or more has
elapsed since the last If not up-to-date with Schedule vaccines:
primary or booster
dose No TIG is needed, but
Give age appropriate tetanus immunization
immediately and if needed, arrange additional catch-
Complete primary up immunization*.
course (3 doses at
ages 2, 4 and 6 Dirty wound and less No tetanus-containing vaccine is needed
months) than 5 years has No TIG is needed
elapsed since the last
primary or booster
dose
If up-to-date with schedule vaccines:

No TIG is needed, but


Give dT - if aged 9–10 years OR immunization event
Dirty wound and 5 due, replace dT with Schedule dTpa
years or more has
elapsed since the last If not up-to-date with Schedule vaccines:
primary or booster
dose No TIG Is needed, but
Give age appropriate tetanus immunization
immediately and if needed, arrange additional catch-
up immunization*.

No TIG is needed, but


Give age appropriate tetanus immunization
immediately and if needed, arrange additional catch-
No prior tetanus up immunization*.
immunization
Clean wound

OR

Incomplete primary
course Give TIG

OR AND

Unknown primary Dirty wound


Give age appropriate tetanus immunization
vaccination status immediately and if needed, arrange additional catch-
up immunization*.

Age up to 10 years (i.e. up to the 10th birthday): a DTPa-containing vaccine


Age 10-18 years (i.e. up to the 18th birthday): dTpa
Age 18 years or older: dT

This patient has a dirty wound (deep and soil-contaminated) and full course of tetanus vaccination with the last booster dose 6 years ago.
Since more than 5 years has passed since his last booster dose, he needs another immediate booster dose with dT (preferred) or dTpa as the
most appropriate management option. He does not need TIG.

(Option A) DTPa is a child formulation and not age-appropriate for this patient.

(Option B) This option was the correct one if less than 5 years would have elapsed since his last dose of tetanus-containing vaccine.
295 of 1943
(Options D and E) TIG as a means of passive immunization is not required in individuals who has complete primary vaccination against
tetanus (such as this man); therefore, any options suggesting TIG is incorrect. Antibiotics, on the other hand, may be used for prevention of
wound infections in addition to tetanus prophylaxis. The decision regarding prescribing antibiotics for other infections than tetanus should be
individualized for every patient.

References

• Immunisation Hanbook - Guide to tetanus prophylaxis in wound management

Last updated:
Time spent: QID:1226
2023-2-12

296 of 1943
A few hours after insertion of a permanent pacemaker for a 55-year-old man, he develops shortness of breath and pleuritic chest pain. On
examination, he has a blood pressure of 80/50 mmHg, heart rate of 110 bpm, and respiratory rate of 25 cycles/min. He has an O2 saturation
of 96% on room air and a jugular venous pressure (JVP) of 6 cm (normal<3cm). On auscultation, lungs are clear but heart sound are distant
and muffled. Which one of the following is the investigation of choice to consider as the next best in management?

A. Serum troponin level.

B. ECG.

C. Chest X-ray.

D. Echocardiography.

E. CTPA.

Incorrect. Correct answer is D


45% answered correctly

Explanation:

Correct Answer Is D

Permanent pacemaker insertion into the heart chambers is performed through venous access. Most commonly, subclavian or cephalic veins
are used for venous access. Rarely, internal jugular vein is used.

The procedure is minimally invasive and carried out under local anesthesia. There are, however, complications associated with the procedure
such as bleeding, infection, pneumothorax, hemothorax, myocardial perforation, etc.

With chest pain, raised JVP, clear lungs, and importantly the distant muffled heart sounds, pericardial tamponade probably due to myocardial
perforation is the most likely diagnosis. Myocardial perforation during or after the procedure is often a benign self-limiting condition, but rarely
it can result in the disastrous cardiac tamponade. Pneumothorax is a more common complication of the procedure compared with
myocardial perforation; however, if the case was a tension pneumothorax, decreased respiratory sounds in the affected lung and a normal
heart auscultation would be expected.

Although cardiac tamponade is a clinical diagnosis, two dimensional and Doppler echocardiography play major roles in the identification of
pericardial effusion and assessing the hemodynamic significance. Typical finding on two-dimensional echocardiography are a moderate to
large effusion and swinging of the heart within that effusion. Also, echocardiography can be used as a guide for pericardiocentesis, which is
the immediate and most vital treatment for the condition.

(Options A and B) Serum troponin level and ECG would have been considered if the if the scenario was suggestive of myocardial ischemia.
The presentation is very typical for cardiac tamponade rather than ischemia.

(Option C) Chest X-ray is considered if the clinical presentation suggests pneumothorax, or other lung pathologies are thought of. Of note, in
case of tension pneumothorax, the very first step in management is always urgent decompression with needle aspiration. Any further
investigation should be reserved until the patient is hemodynamically stable.

(Option E) CTPA is used when pulmonary embolism (PE) is suspected. PE is associated with tachycardia, tachypnea, and pleuritic chest pain.
A normal lung auscultation does not exclude PE as a diagnosis; however, distant muffled heart sounds is not an expected finding in PE.
Moreover, this patient has normal O2 saturation that makes PE even a weaker possibility.

TOPIC REVIEW

Potential complications of permanent pacemaker insertion include:

Access-related complications

Early access-related complications:

Bleeding
Hematoma
Phlebitis or thrombophlebitis of the vein
Local infection
Arterial injury or puncture
297 of 1943
Hemothorax
Pneumothorax
Catheter-related thrombosis (which may lead to pulmonary embolism)
Air embolism
Dysrhythmias
Atrial wall puncture from guide wire (which may lead to pericardial tamponade)
Lost guide wire
Anaphylaxis
Chylothorax (possible with left-side lead insertion)

Late access-related complications:

Chylothorax
Hematoma
Venous thrombosis or occlusion
Phlebitis
Atrioventricular fistula
Hemothorax
Infection

Pocket-related complications

Early pocket-related complications:

Swelling
Hematoma
Bruising and local pain
Infection

Late pocket-related complications:

Pocket erosion
Pocket infection
Chronic pain at the site

Lead-related complications

Early lead-related complications:

Atrial or ventricular arrhythmias


Chamber perforation
Pneumothorax and pneumopericardium (with an atrial lead)
Intercostal or diaphragm pacing
Pectoral muscle stimulation
Lead dislodgement
Tricuspid valve laceration
Cardiac tamponade
Pericardial friction rub
Hypotension
Bleeding

Late lead-related complications:

Infection of pacer lead


Endocarditis
Systemic infection
Perforation
Access vein thrombosis
Inferior vena cava thrombus
Pectoral muscle stimulation
Right atrial thrombus
Loss of capture and sensing
Tricuspid regurgitation
Lead fracture
Intercostal or diaphragm pacing

References

• Medscape - Cardiac Tamponade

• Medscape - Permanent Pacemaker Insertion Technique


298 of 1943
Last updated:
Time spent: QID:1240
2023-2-12

299 of 1943
An 11-year-old boy is recovered from a beach in Northern Queensland and brought to the Emergency Department of the local hospital. Upon
arrival he has severe pain in the right foot and leg. Jellyfish tentacles are noted stuck in the skin of the right sole on a whip-like manner, and
there is surrounding erythema and slight swelling. Which one of the following options would be the most important step in management?

A. Intramuscular morphine.

B. Pour vinegar on the foot and tentacles.

C. Immerse the foot in hot water.

D. Administer antivenom.

E. Start intravenous antibiotics.

Incorrect. Correct answer is B


45% answered correctly

Explanation:

Correct Answer Is B

Of all marine envenomation incidents, Jellyfish sting is the most common in Australia. Stings are caused by simultaneous discharge of many
hundreds to thousands of nematocytes that are the venom-contained microscopic capsules in the jellyfish tentacles.

Jellyfish sting often causes acute sharp pain and an inflammatory response at the site of the sting. Such response may vary from local
redness and tenderness of different intensity, urticaria, and swelling. Skin necrosis and destruction can follow. In severe envenomation,
caused by significant number of stings, cardiorespiratory collapse can occur.

Jellyfish stings can be caused by a variety of jellyfish species, most of which cause minor stings with insignificant and self-limiting
presentations. In Australia, most serious jellyfish stings are caused by the following two types of tropical jellyfish:

Box jellyfish – the Australian Box jellyfish (Chironex fleckeri) has a large box-like body of up to 20x30 cm and many tentacles. Contact with
these tentacles results is whip-like skin marks and immediate pain which is often severe. Stings with multiple tentacles can lead to significant
envenomation and cardiopulmonary arrest.

Jellyfishes causing Irukandji syndrome – stings from approximately 10 species of jellyfish can cause this syndrome that presents with severe
generalized and often cramping pain within 5 to 40 minutes (typically 20-30 minutes) of the sting, nausea and vomiting, perspiration,
restlessness and a sense of impending doom, and shortness of breath. Hypertension and hypertensive cerebrovascular events, pulmonary
edema and acute heart failure are other possible presentations.

When stings occur, there might be non-discharged nematocysts left in the skin. These often release more venom into the victim’s body and
further envenomation upon handling; therefore, it is important to prevent nematocysts form discharging their content. Vinegar (acetic acid 4-
6%) can neutralize nematocysts discharge in some species such as Box jellyfish and many types of those jellyfishes causing Irukandji
syndrome and is the most appropriate initial step if stings by such species occur. Stings in the tropical regions of Australia such as Northern
Queensland is almost always caused by such species.

Vinegar can result in more discharge from non-discharged species such as bluebottle jelly fish; therefore, this measure should be considered
for tropical regions only.

NOTE - Vinegar is only used to neutralize the non-discharged nematocysts and does not provide pain relief or counteract the effects of
already-injected venom.

(Options A and C) Pain relief is of paramount importance if there is pain. Measures considered for pain control in jellyfish sting victims
depends on the pain severity. While immersion of the affected area in warm water is the only method required for some patients, some other
may need more analgesia, e.g. by morphine. Immersion of the stung area (if possible) is a useful pain control method for jellyfish stings
especially non-tropical species. However, pain control should be considered after neutralization of non-discharged nematocysts.

(Option D) In Australia, antivenom is available for Box jellyfish and other multi-tentacled jellyfish species. It can be used, especially for
suspected severe envenomation. However, it is not a priority compared to pouring or spraying vinegar on the sting site.

(Option E) Antibiotics might be considered later on if an infection occurs at the sting site or somewhere else.

References

300 of 1943
• RACGP - AFP - Marine envenomations

• Australian Resuscitation Council Guidelines - Guideline 9.4.5 Envenomaton - Jellyfish Stings


Last updated:
Time spent: QID:1317
2023-2-12

301 of 1943
A 46-year-old man undergoes laparoscopic cholecystectomy due to recurrent episodes of cholecystitis in the setting of gallstones. After 12
hours of the surgery, he develops chest pain. In addition to a complete focused history and physical examination, which one of the following
would you consider first for this patient?

A. Chest physiotherapy.

B. CTPA.

C. ECG.

D. Echocardiography.

E. Chest X-ray.

Incorrect. Correct answer is C


45% answered correctly

Explanation:

Correct Answer Is C

Chest pain in the post-operative period is a well-known complication that can be related or non-related to the surgery. Usual causes of
operation-related include:

Atelectasis
Pneumonia
Myocardial infarction
Pneumothorax
Pulmonary embolism

Of these, myocardial infarction and pulmonary embolism are potentially fatal etiologies to consider and exclude first. Among patients 45
years of age or older undergoing noncardiac surgery, complications of cardiac death, myocardial infarction, heart failure, or ventricular
tachycardia occur in up to 5 percent. Of these, perioperative myocardial infarction is the most common and should be ruled out as a
possibility; therefore, an ECG would be the most appropriate next investigation to consider for this patient.

(Option A) Chest physiotherapy is the initial treatment option in patients whose pain is due to pulmonary atelectasis. Pulmonary atelectasis
presents with pleuritic chest pain and shortness of breath often within the first 24 hours of the operation. Pulmonary atelectasis can be a
diagnosis in this patient; however, unless other more serious causes of chest pain are not excluded safely, and patient is proved to have
pulmonary atelectasis, this could not be a priority in management.

(Option B) CTPA is the modality of choice for patients with high probability of pulmonary embolism as the diagnosis. Although not a rule,
pulmonary embolism does not develop so early in the post-operative period; nonetheless, even if pulmonary embolism is suspected in this
patient, an ECG and probably a chest X-ray should be done first to exclude other causes of the chest pain.

(Option D) Echocardiography might be considered later in patients who are suspected to have myocardial infarction is the cause of the chest
pain based on ECG findings or positive cardiac markers such as troponin.

(Option E) A chest X-ray could be a correct in patients with respiratory symptoms and an unclear diagnosis. Also, patients with suspected
pulmonary embolism require a chest X-ray to exclude other conditions than pulmonary embolism such as pneumonia as the etiology. A clear
chest X-ray is on the other hand, a requirement for V/Q scan to be a reliable modality if decided as an investigation for pulmonary embolism.

References

• UpToDate - Perioperative myocardial infarction or injury after noncardiac surgery

• AHA - Perioperative Myocardial Infarction

Last updated:
Time spent: QID:1335
2023-2-12

302 of 1943
A 27-year-old man is brought to the Emergency Department after he sustained a motor vehicle accident. He is unable to move his arms and
legs, and seems to have cervical spine injury. On examination, he has a blood pressure of 90/60 mmHg and pulse rate of 50bpm. He is alert
and is breathing spontaneously. There is no visible bleeding source or limb deformity. His chest has multiple bruises and is tender to touch
over the bruises but is normal on auscultation. The abdomen is quite normal on exam with no tenderness, rebound, or guarding. Which one of
the following would be the next best step in management?

A. Intravenous colloid solutions.

B. Atropine.

C. Trendelenburg position (head down).

D. Adrenaline.

E. Dopamine.

Incorrect. Correct answer is C


45% answered correctly

Explanation:

Correct Answer Is C

To answer this question correctly, familiarity with the mechanisms responsible for hemodynamic instability in a traumatic patient, who also
has spinal cord injury is necessary.

In any trauma patient, hypotension should be assumed to be caused by hypovolemia until proven otherwise, even in a patient with an overt
spinal injury. Sources of bleeding must be aggressively sought and controlled. Neurogenic shock is rare in comparison.

According to the scenario, there is no visible source of bleeding. Furthermore, absence of limb deformity makes long bone fractures as a
source of bleeding unlikely. Abdomen is soft and non-tender, making intra-abdominal hemorrhage less likely, at least for now.

Other possibilities to consider as a cause of decreased blood pressure in this patient are bleeding into the chest (e.g., hemothorax) or
obstruction of venous return associated with tension pneumothorax or cardiac tamponade. For these to be the cause, chest should have been
involved, as is in this patient, evident by multiple bruises over the chest wall. However, the patient is breathing spontaneously and there is no
abnormality on chest auscultation.

Although tachycardia is often a significant feature seen in hemorrhagic or obstructive shock, the presence of bradycardia does not exclude
these conditions, as concomitant neurogenic shock can prevent compensatory tachycardia in response to decreased blood pressure. In this
patient, however, with no evidence pointing towards other causes of hypotension, especially hemorrhage and obstruction, attention should be
turned to other cause justifying the hypotension – the neurogenic shock.

Spinal shock and neurogenic shock are two different conditions that are often confused. Spinal shock is caused by acute spinal cord injury.
Spinal shock is not a true form of shock. It refers to the flaccid areflexia that may occur after spinal cord injury and may last hours to weeks. It
may be thought of as ‘concussion’ of the spinal cord. Spinal shock resolves as the soft tissue swelling improves. Associate features are
absent deep tendon reflexes, sensation, and flaccid paralysis below the level of spinal injury.

Neurogenic shock, on the other hand, is classically characterized by hypotension, bradycardia, and peripheral vasodilatation. Neurogenic
shock is due to loss of sympathetic vascular tone, and happens only after a significant proportion of the sympathetic nervous system has
been damaged, as can occur with lesions above the 6th thoracic vertebra.

In a quadriplegic patient, the blood pressure normally ranges between 80/40 mmHg to 100/60 mmHg and pulse rate is down to 50 bpm.

NOTE – Spinal shock and neurogenic shock often coexist in patients with high spinal injuries.

Clinical features of neurogenic shock include:

Hypotension – due to massive vasodilation


Bradycardia – due to unopposed parasympathetic stimulation
Poikilothermia (unable to regulate temperature)

Unlike hemorrhagic and obstructive shocks, patients with neurogenic shock have warm and flushed extremities because the sympathetic tone
is lost and blood pools in the extremities.

These features occur within 30 minutes of cord injury above the 6th thoracic vertebra and can last up to 6 weeks.

303 of 1943
The treatment goal for neurogenic shock is adequate perfusion and oxygenation. High flow oxygen should be administered (15 lit/min via
non-rebreathing mask). Lesions above C5 often require intubation.

According to guidelines published by Neurosurgical Society of Australia, placing the patient in Trendelenburg position (head down) is the most
important next step for initial management of decreased blood pressure and bradycardia. Trendelenburg position increases the return of
pooled blood in the periphery to the heart and increases cardiac output and blood pressure.

Until the neurogenic shock resolves, it is of great significance to maintaining a systolic blood pressure of 90-10 mmHg. Isotonic fluids such as
Hartman’s solution or normal saline are solutions of choice but should be administered with great caution. Excessive fluid administration may
cause pulmonary edema because these patients are at risk for the acute respiratory distress syndrome (ARDS).

If the above measures fail to establish adequate perfusion, addition of inotropes is indicated. Dopamine or norepinephrine is used for
treatment of protracted hypotension and bradycardia caused by higher spinal lesions. In lower lesions, vasopressors such as phenylephrine
can be used.

(Option A) Judicious administration of isotonic fluids are indicated in some patients, but colloids are almost never indicated for management
of neurogenic shock.

(Option B) Atropine is reserved for patients with bradycardia of <50 bpm. As mentioned earlier, it is quite common for patients with
neurogenic shock to have bradycardia down to 50 bpm. Atropine is only indicated if there is severe bradycardia (<40 bpm), or when there is
significant hemodynamic instability caused by it. Atropine should also be considered in patients who are undergoing maneuvers that can
induce bradycardia via vagal stimulation e.g. nasopharyngeal suction, intubation, etc.

(Options D and E) Inotropic support with dopamine or adrenalin (epinephrine) is rarely used and should be reserved for patients who have
decreased urinary output (<0.5cc/kg/hour) despite adequate fluid resuscitation.

TOPIC REVIEW

Trendelenburg position

Although widely used for hemorrhagic shock in the past, Trendelenburg position is now considered of no use in treating hypovolemic or
inflammatory shock.

By placing the patient in Trendelenburg position, it was believed that translocation of blood from periphery to the systemic circulation occurs,
and blood flow to the brain will be increased,

Trendelenburg position increases the blood pressure in the carotid arteries, but since there is no valve between carotid arteries and internal
jugular vein, the difference between these two remains constant and the perfusion pressure of the brain unchanged.

In hypovolemic shock or untreated inflammatory shock, the venules and small veins have been already depleted of their blood content, so
even in Trendelenburg position, the amount of blood returning to the heart is not considerable.

Patients in neurogenic shock, however, can benefit from Trendelenburg position, provided that such placement does not compromise other
aspects of care (e.g., securing the airway). This position causes blood to be rapidly translocated from denervated, engorged venules and
small veins back to the heart. With increased ventricular end-diastolic volumes, stroke volumes, cardiac output, and blood pressure will
increase.

References

• The Acute Management of Spinal Injuries

Last updated:
Time spent: QID:396
2023-2-12

304 of 1943
During your night shift in a rural hospital in Northern Territory, you receive a 27-year-old man 30 minutes after being bitten by a brown snake in
the left ankle. Upon arrival, he has no symptoms. On examination, you notice scratches over the left ankle, but no fang marks are present. The
nearest tertiary hospital is 150 km away. Which one of the following is the most appropriate next step in management of this patient?

A. Discharge him home because he is asymptomatic and there is no fang mark indicating a bite.

B. Air-transfer him to the tertiary hospital.

C. Give him a vial of antivenom now and another if symptoms develop.

D. Application of a tourniquet above the bite site.

E. Give him a vial of antivenom once he starts developing the symptoms.

Incorrect. Correct answer is B


45% answered correctly

Explanation:

Correct Answer Is B

It is very important that every suspected or definite snake bite is taken very seriously and assumed that envenomation has occurred until
proven otherwise.

Unless massively envenomated, especially in children, where signs and symptoms may appear in a matter of minutes, life-threatening effects
may not be apparent until hours later. Therefore, every patient with suspected or definite envenomation should be very carefully observed and
monitored in a tertiary center with intensive care unit and access to anti-venom kits. Absence of visible bite marks does not reduce the
importance of emergency care for this patient because there might be severe envenomation with absent or insignificant bite wound, fang
marks, or other signs of a bite. Since a tertiary hospital is 150 km away, transferring the patient, preferably by air, is the most appropriate next
step in management.

(Option A) As mentioned earlier, there is often a time window between the envenomation and emergence of symptoms; hence, just because
the patient is asymptomatic at present, he cannot be discharged. On the other hand, even with envenomation, snake bites may be painless
and without visible marks. There might be no significant wound, local signs, or fang marks. In fact, in most cases there are no fang marks and
often only a single mark or a scratch mark may be present (localized redness and bruising are uncommon in Australian snake bites)

(Option D) Application of arterial tourniquet in snake bites is discouraged and not recommended.

(Options C and E) Data suggests that antivenom might prevent certain envenoming syndromes if used early but may have little if any effect
once major envenoming syndromes are established. There is no high-level evidence as a guide in this regard and clinicians have to balance
the risks of giving antivenom (such as anaphylaxis and serum sickness) against the potential benefits of giving it early without waiting for
confirmation of envenoming. The following are absolute and relative indications for antivenom administration:

Absolute indications:

Reported sudden collapse, seizure or cardiac arrest


Abnormal international normalized ratio (INR) (reference interval, 0.9–1.3)
Any evidence of paralysis, with ptosis and/or ophthalmoplegia being the earliest signs

Relative indications:

Systemic symptoms (vomiting, headache, abdominal pain, diarrhea)


Leukocytosis
Abnormal activated partial thromboplastin time (outside laboratory’s reference interval)
Creatine kinase level > 1000 U/L

For now, this patient is asymptomatic and does not have a clinical (not laboratory) indication for antivenom; however, laboratory tests may
show results such as an increased INR that mandate antivenom. Investigations for laboratory investigations to establish an absolute or
relative indication for antivenom should not delay arrangements for transferring the patient to a tertiary center.

TOPIC REVIEW

Many of the snakes found in Australia are capable of lethal bites to humans. These include Taipans, Brown snakes, Tiger snakes, Death
Adders, Black snakes, Copperhead snakes, Rough Scaled snakes, and many Sea snakes. Venom is produced in the modified salivary glands of
the snake and forced out under pressure through paired fangs in the upper jaw. Snake venoms are complex mixtures of many toxic
305 of 1943
substances which can cause different effects in human victims. The life-threatening early effect in Australian snake bite is neurotoxic muscle
paralysis which kills by causing breathing failure. Other significant effects include:

bleeding due to coagulation failure


muscle damage causing kidney failure
headache
nausea and vomiting
occasionally, initial collapse or confusion followed by partial or complete recovery
abdominal pain
blurred or double vision, or droopy eyelids
difficulty in speaking, swallowing, or breathing
swollen tender glands in the groin or axilla of the bitten limb
limb weakness or paralysis
respiratory weakness or respiratory arrest

Important tips in initial management of snake bites:

Keep the victim at rest, reassured and under constant observation


Commence resuscitation if necessary
Immobilize the affected limb
Transport the victim to a medical facility, preferably by ambulance
As a rule, all victims of suspected or definitive snake bites should be assumed to have been envenomated until proven otherwise.
DO NOT cut or incise the bite
DO NOT use an arterial tourniquet
DO NOT wash or suck the bite because identification of venomous snakes can be made from venom present on clothing or the skin
using a Venom Detection Kit.

NOTE - It is not recommended to kill the snake for purposes of identification, because medical services do not rely on visual identification
of the snake species.

References

• Medical Journal of Australia - Snakebite in Australia: a practical approach to diagnosis and treatment

Last updated:
Time spent: QID:1385
2023-2-12

306 of 1943
A 35-year-old man presents to the Emergency Department after he sustained a bite by a drunk woman in a pub. The woman was a stranger
who walked away after the incident. On history, he is fully immunized against tetanus with the last dose being 3 years ago. He is unaware of
his hepatitis B immunization. On physical examination, there is a 2-cm bite wound 5 mm deep in the right arm that is actively bleeding and
teeth marks around the wound. Which one of the following is the most appropriate next step in management of this patient?

A. Tetanus immunization.

B. Start him on zidovudine for HIV prophylaxis.

C. Give him hepatitis B vaccine and immunoglobulin.

D. Start him on a 5-day course of prophylactic azithromycin.

E. Suture the wound and review in 24-hours.

Incorrect. Correct answer is C


45% answered correctly

Explanation:

Correct Answer Is C

Human bites have a higher complication and infection rate than animal bites. Although not true bites, clenched fist injuries are categorized
under bite wounds. Clenched fist injury commonly presents as a small wound over the metacarpophalangeal joint of the dominant hand as a
result of the patient striking another person's teeth with a clenched fist. Human bite wounds to the hand more commonly develop bacterial
infection than human bites at other sites, with clenched fist injuries conferring the highest risk, particularly because of the potential for
breaching the metacarpophalangeal joint space to produce septic arthritis or osteomyelitis.

Management of human bites include the following:

STEP 1 - Wound care

The wound should be irrigated with normal saline and washed with water and soap. Gross foreign bodies should be removed and
debridement considered if necessary. There is limited evidence regarding wound closure and suturing and decision should be made in a case-
by-case bases; however, the following are general recommendations:

Primary closure could be considered in carefully selected bite wounds where cosmesis is an issue.
Primary closure of head and neck wounds with antibiotic prophylaxis : the risk of infection is only 1% due to enhanced blood supply
and lack of dependent edema in these areas.
Suturing is not recommended in wounds at high risk of infection.

NOTE - Patients with bites in the hand (either animal or human bite) should be referred early to hand surgeons to evaluate the need for
exploration to prevent loss of function. Admission to hospital for intravenous antibiotic therapy may be required.

Elevation and immobilization is another important part of wound management to consider.

STEP 2 - Tetanus prophylaxis

All patients should be assessed for tetanus vaccination history and be given tetanus prophylaxis if necessary.

STEP 3 - Post-exposure prophylaxis for hepatitis B virus (and very rarely HIV virus)

The risk of transmission HIV infection following a human bite is extremely rare and practically negligible. Post-exposure prophylaxis for HIV is
not routinely performed unless there is high index of suspicion that a biter’s saliva who is HIV positive has been mixed with his/he blood.
There is no post-exposure prophylaxis for hepatitis C.

For hepatitis B, all patients, who has suffered a human bite and are not adequately immune to hepatitis B, or their immune status is unknown,
it is recommended that they receive hepatitis B immunization with a full course of hepatitis B vaccine and hepatitis B immunoglobulin.

STEP 4 - Antibiotic prophylaxis

For otherwise healthy individuals, antibiotic therapy is usually not necessary for bites and clenched fist injuries with a low risk of infection (e.g.
small wounds not involving tendons or joints that present within 8 hours and that can be adequately debrided and irrigated).

307 of 1943
Antibiotic therapy is only necessary for bites and clenched fist injuries with a high risk of infection. These include:

wounds with delayed presentation (8 hours or more)


puncture wounds that cannot be debrided adequately
wounds on the hand, feet, face or genitalia
wounds involving deeper tissues (e.g. bones, joints, tendons)
wounds in immunocompromised patients.

STEP 5 - Patient education and provision of reading materials

STEP 6 - Review in 24-48 hours

Of the given options, the most appropriate step for this patient would be giving him hepatitis B vaccine and immunoglobulin. However, if
possible, a hepatitis B antibody titer to assess his immunity before commencement of immunization would have been the most appropriate
option.

(Option A) This patient has received a full course of tetanus vaccination with the last dose being 3 years ago; therefore, he does not require
tetanus immunization.

(Option B) Since the risk of transmission of HIV by human bites is practically negligible, prophylaxis with zidovudine is not necessary for this
patient.

(Option D) This patient does not have any of the aforementioned criteria for antibiotic prophylaxis and does not need it. Even so, azithromycin
is not good choice.

(Option E) As mentioned before, wound closure will be decided on a case-by-case basis and might be considered for this patient because the
presentation is early and the wound is in an anatomical area where adequate debridement can be performed; however, only suturing the
wound and review in 24 hours is not an appropriate management.

References

• RACGP – Management of mammalian bites

• Medscape – Human Bites

• Therapeutic Guidelines

Last updated:
Time spent: QID:1441
2023-2-12

308 of 1943
A 32-year-old man is brought to the emergency department of a local hospital after he was recovered from a fire in a building. He was working
in his office in that building when the fire started and was barricaded in the room for approximately 20 minutes before firemen were able to
break into the room surrounded by fire and full of smoke. Which one of the following is the correct method of giving him oxygen?

A. 2L/min via nasal cannula.

B. 6L/min via Venturi mask.

C. 5-10L/min via simple face mask.

D. 10-15L/min via non-rebreathing mask.

E. Transferring him to hyperbaric oxygen chamber.

Incorrect. Correct answer is D


45% answered correctly

Explanation:

Correct Answer Is D

In every victim of a fire especially in closed spaces, carbon monoxide poisoning should always be consider a possibility to investigate and
treat immediately. Carbon monoxide (CO) is an extremely poisonous, odorless, tasteless and colorless gas, which is produced when there is
incomplete combustion of carbon containing fuels (e.g. coal, petroleum, peat, natural gas, etc.). Inhalation is the most common route of
exposure.

CO binds to hemoglobin with a much higher affinity than that of oxygen to form carboxyhemoglobin. CO-bound hemoglobin will not be able to
transport oxygen anymore. This results in in tissue hypoxia. Moreover, CO combines with other hem compounds such as myoglobin and
cytochrome oxidase.

Treatment of patients with CO poisoning depends on the severity of the toxicity, but for every patient with suspected CO poisoning high flow
(10-15L/min) of normobaric oxygen via non-rebreathing mask should be started immediately. Intubation may be required in more severe
cases.

NOTE - It is important to note that oxygen saturation by pulseoxymetry should not be used to guide oxygen therapy because pulseoxymeters
cannot differentiate between CO-bound and O2-bound hemoglobin. So, any option suggesting that the need for and mode of oxygen therapy
should be based on O2 saturation reading on pulseoxymeters would be incorrect.

Oxygen 2L/min via nasal cannula (prongs) (option A), 6L/min via Venturi mask (option B) or 5-10L/min via simple face mask (option C) will
not provide adequate high flow oxygen to this patient and are not correct options.

(Option E) Hyperbaric oxygen therapy using hyperbaric oxygen chambers are indicated for severely poisoned patients who are comatose, have
seizures or other neurologic abnormalities, or have myocardial ischemia. Pregnancy is another indication for hyperbaric oxygen therapy. For
such patients, hyperbaric oxygen therapy should be started as quickly as possible, ideally within 6 to 8 hours. There is no comments on
symptoms or physical examination findings in this patient pointing towards the need for commencement of hyperbaric oxygen therapy.
Moreover, even if he is in need for that, he must be started and maintained on high flow, normobaric oxygen while arrangements and
preparations for hyperbaric oxygen therapy are made.

TOPIC REVIEW

Symptoms of CO poisoning depend on the extent of poisoning:

The main manifestations of carbon monoxide poisoning develop in the CNS and heart which are organs most dependent on oxygen. The
symptoms are as follows:

Mild toxicity - throbbing temporal or frontal headache, fatigue, dyspnea on exertion, light-headedness and dizziness. In general, clinical
symptoms of mild poisoning are non-specific and may mimic those of a non-specific viral illness, with vomiting, headache, malaise, weakness,
fatigue and shortness of breath.

Moderate toxicity - severe headache, weakness, dizziness, nausea, vomiting, tachycardia, tachypnea, flushing, perspiration, decreased
vigilance, diminished manual dexterity, impaired sensorimotor task performance, prolonged reaction time, difficulty thinking, impaired
judgment, blurred or darkened vision, ataxia, loss of muscular control, tinnitus and drowsiness.

309 of 1943
Severe toxicity - may produce syncope, seizures, confusion, disorientation, incontinence, ventricular dysrhythmias, cardiorespiratory
depression, respiratory failure, coma and death. Delayed effects, attributable to hypoxia, usually result in neuropsychiatric effects.

Investigations

Mild poisoning

Based on history and clinical picture.

Significant exposures

Determining carboxyhemoglobin (COHb) level when the patient is first seen and repeat every 2 to 4 hours until patient is
asymptomatic or level is within the normal range.
ECG, EUC (electrolytes, urea, and creatinine), CK (creatine kinase), ABGs, and chest X-ray if symptomatic or if the COHb level is greater
than 20%.

NOTE - COHb levels correlate poorly with signs and symptoms of toxicity. Interpretation may be confounded by delays in obtaining blood
samples and therapeutic interventions such as oxygen administration.

CT scan or MRI if there are persistent neurological symptoms.

References

• Department of Health – Australian Clinical Guidelines for Acute Exposures to Chemical Agents of Health Concern: A Guide for the
Emergency Department Staff

Last updated:
Time spent: QID:1478
2023-2-12

310 of 1943
While doing painting on a scaffold, John, a 48-year-old painter, falls off from a height of approximately three meters and lands on his both
heels before he fell onto the ground. His coworkers called an ambulance to transfer him to hospital. In the emergency department, he denies
any loss of consciousness, which is confirmed by one of his coworkers present at the scene who has accompanied him to the hospital. He is
fully alert and responsive but in severe left foot pain and modest right foot discomfort as well as back pain. There are abrasions on the palms
and a minor wound on the forehead. Vital signs are all within normal limits. There is severe tenderness, bruising and swelling below the left
ankle and moderate tenderness over the lumbar area. Neurological examination is performed as far as feasible with unremarkable results.
After giving him pain killers for pain, which one of the following would be the most appropriate next step in management?

A. CT scan of the head to exclude intracranial hemorrhage.

B. AP and lateral x-ray films of the left and right foot.

C. CT scan of the left foot.

D. Spinal x-ray series.

E. Immediate referral to orthopedic surgeon.

Incorrect. Correct answer is D


45% answered correctly

Explanation:

Correct Answer Is D

Given the mechanism of injury (fall from height) and severe pain and bruising and swelling of the left foot, calcaneal fracture (fracture of the
calcaneus bone of the heel) could be the most likely diagnosis.

Calcaneal fracture is uncommon and comprises only 1-2% of all fractures. Axial loading of the foot following a fall is the most common
mechanism of injury. Calcaneal fractures are important from two different aspects. First is that this fracture is associated with long-term
disability. The more important feature, however, is the fact that an axial load significant enough to cause a heel fracture can be associated
with other injuries that may initially remain unnoticed, mostly due to distraction by severe foot pain. The patient might have sustained internal
organs injury, spine injuries, and injuries of knees, hips and femur (mostly femoral head and neck). One study based on data from the
American College of Surgeons National Trauma Data Bank (NTDB), showed that in patients with calcaneal fracture, 23% had spinal fractures,
18% head injuries, and 15% thoracic organ injuries. Most of the spinal fractures occurred in the lumbar spine.

Based on these facts, it is necessary that every patient with calcaneal fracture undergoes a thorough assessment for other possible injuries.
Imaging studies should be ordered if any other injury is suspected in history and physical examination.

Given the commonality of spinal injury as an association with calcaneal fracture, and the presence of back pain, X-ray of the spine for
assessment of any spinal fracture or injury is the most appropriate next step in management for this patient.

(Option A) CT scan of the head would have been indicated if John had loss of consciousness or other signs or symptoms of an intracranial
pathology. John is completely lucid and has not had any alteration in consciousness. He also have unremarkable neurologic examination; so,
brain imaging is not indicated for now.

(Option B) Plain films of the foot are the cornerstone of confirming the diagnosis. CT scan of the foot (option C) is indicated in equivocal
cases or where more details are required to decide about the management (extra-articular and intra-articular fracture are often managed
differently). Although imaging of the calcaneus is necessary, more serious injuries must be excluded first.

(Option E) Referral to orthopedic surgeon for definitive management of the calcaneus fracture is appropriate after other more serious injuries
such as spinal injuries are ruled out.

References

• Radiopedia – Calcaneal fracture and associated spinal injuries

• Medscape – Calcaneus Fracture Imaging

Last updated:
Time spent: QID:1500
2023-2-12

311 of 1943
A 65-year-old man underwent an uneventful right carotid endarterectomy 4 days ago. Today, he presents with shortness of breath which is
progressively becoming worse. Which one of the following would be the best immediate step in management?

A. Intubation.

B. Tracheostomy.

C. Opening all the wound layers in the Emergency Department.

D. Supplemental oxygen via face mask.

E. Opening all the wound layers in the operating theater.

Incorrect. Correct answer is C


45% answered correctly

Explanation:

Correct Answer Is C

Carotid endarterectomy is the procedure of choice in some patients with carotid stenosis. This procedure is associated with complications,
the most important of which are as follows:

Perioperative stroke
Myocardial infarction
Hyper-reperfusion syndrome
Cervical hematoma
Nerve injury
Infection
Carotid restenosis

Postoperative cervical wound hematoma is a potentially lethal complication of carotid endarterectomy (CEA). The hematoma can rapidly
distort the airway anatomy, making tracheal intubation difficult, and ultimately causing fatal airway obstruction. Post-CEA patients might
develop airway emergencies related to hematoma formation either in the hospital or after discharge.

The progressive short of breast in this patient is most likely to have been caused by an expanding cervical hematoma. As always,
management starts with the ABCD of life support (airway, breathing, circulation, and disability). The first step in this approach is ensuring and
maintaining a patent airway. This patient has a rapidly deteriorating airway in need of urgent attention with intubation (option A). Initial
method of airway management is based on the location, severity, and progression of the hematoma. Most patients can be managed
successfully using a stepwise approach, beginning with awake fiber-optic intubation if time permits. If this fails, laryngoscopy, with either a
direct or video laryngoscope, is a reliable backup. However, in a time-sensitive critical situation where there is impending respiratory
compromise or airway loss, opening of all the wound layers at the spot (patient’s bedside in the Emergency Department in this scenario) to
decompress the trachea and facilitate airway management should always be considered first as the most immediate action.

Surgical management of the airway using tracheostomy (option B) comes next if attempts to intubate patient fail.

Oxygen (option D) should be given after ensuring the patency of the airway or prior to intubation as a part of rapid sequence intubation. With
an obstructed airway, supplementation with oxygen has minimal, if any, benefit.

(Option E) Opening the wound layers in the operating theatre is the definite and most specific management of a patient who has a cervical
hematoma and is clinically stable which is not the case in this scenario.

References

• Medscape - Complications of Carotid Endarterectomy (CEA) in the Postanesthesia Care Unit (PACU)

• NEJM - Airway Management in Postoperative Carotid Endarterectomy Patients

• UpToDate – Complications of carotid endarterectomy

Last updated:
Time spent: QID:1584
2023-2-12

312 of 1943
A 19-year-old man is brought to the Emergency Department by ambulance after he sustained a gunshot wound to his abdomen. On
examination, he has normal blood pressure and pulse rate with no sign of hemodynamic instability. The entry wound is in the epigastrium.
There is no exit wound. The abdomen is tender to palpation. On anterior-posterior and lateral abdominal X-ray films, the bullet is found to be
lodged in the right psoas muscle. Which one of the following would be the most appropriate next step in management?

A. Emergency ultrasound scan.

B. Abdominal CT scan.

C. Close clinical observation.

D. Exploratory laparotomy.

E. Diagnostic peritoneal lavage (DPL).

Incorrect. Correct answer is B


45% answered correctly

Explanation:

Correct Answer Is B

Gunshot wounds are destructive to tissue. Destructive and wounding capability of a bullet is directly related to its kinetic energy and the
damage caused by:

Passage of missile
Secondary shock wave
Cavitation
Exponential increase in injury with increasing velocity and efficient energy transfer
Fractures (even in the absence of direct impact)

In every patient with gunshot wound(s), the very first step attending to airway, breathing, and circulation as usual. Hemodynamically unstable
patients should be resuscitated immediately and vigorously, and any visible bleeding should be controlled while urgent transfer to the
operating room for emergency laparotomy is planned and arranged.

In a stable patient, the next step would be a careful neurovascular exam. A high index of suspicion should always be kept for compartment
syndrome secondary to increased muscle edema, especially in wounds inflicted by higher-velocity bullets. All wounds must be examined and
documented.

For stable patients, plain radiographs and tripe contrast CT scan should be performed:

Plain radiography

A chest radiograph should be obtained on all patients to exclude penetration of the chest cavity, even when the wound entry and exit are in the
abdomen. Clues to look for on chest X-rays are hemothorax or pneumothorax or irregularities of the cardiac silhouette, air under the
diaphragm (indicative of peritoneal penetration.)

Abdominal radiographs in 2 views (anterior-posterior [AP], lateral) are also obtained on all patients with gunshot wounds to help determine
missile trajectory and to account for retained missiles (i.e., bullets, shrapnel, and foreign bodies).

CT scan

Multidetector CT (MDCT) scanners have revolutionized the role of CT scanning in abdominal gunshot wounds. Intravenous and oral contrast,
and in cases with suspected colorectal injuries. Contrast matter visualizes hollow and solid viscus organs injuries. In stable patients, this
imaging modality can be applied relatively fast and efficient as an excellent test for those who do not need urgent exploratory laparotomy.

This patient is currently stable and has had abdominal X-rays performed showing a bullet lodged in the right psoas muscle. In its trajectory,
the bullet may have injured other internal organs such as the liver, bowel, diaphragm, etc. For him, an abdominal MT scan to assess such
injuries is the most appropriate next step in management.

(Option A) Ultrasonography is an invaluable study in initial assessment of trauma patients, including those with gunshot wounds to the
abdomen who are clinically unstable. It can rapidly determine the presence or absence of blood in the pericardial and peritoneal spaces.
Focused assessment with sonography for trauma (FAST) is used for the evaluation of penetrating abdominal trauma because of its speed,
noninvasiveness, and reproducibility in diagnosing intraperitoneal injury that requires laparotomy. FAST uses 4 views of the chest and the
abdomen that are pericardial, right upper quadrant, left upper quadrant, and pelvis to evaluate for pericardial fluid indicative of cardiac injury
and for free peritoneal fluid. Free fluid in the abdomen can be a sign of hemorrhage secondary to liver or splenic laceration or, less commonly,

313 of 1943
of spillage secondary to hollow viscus injury. FAST cannot exclude intra-abdominal injury, especially injury to the diaphragm or hollow viscus
organs. This patient is clinically stable and ultrasound is not likely to provide specific information for clinical decision making.

(Option C) With a bullet lodged in the psoas muscle that may have injured to the liver, diaphragm, kidneys, etc. this patient requires exploratory
laparotomy, and close clinical observation as the only approach is definitely incorrect.

(Option D) Exploratory laparotomy is emergently indicated for patients with abdominal gunshot wounds (or penetrating abdominal traumas in
general) who are unstable, or for the stable patients who, through imaging studies, are found to have internal organ injuries such as
perforations, lacerations, etc. This patient may be planned for exploratory laparotomy if CT scan shows injuries requiring surgical repair.

(Option E) The role of diagnostic peritoneal lavage (DPL) in the evaluation of abdominal gunshot wounds is extremely limited. Latest
generation multi-detector CT scanners (MDCT) provide high-resolution images with great speed, and have made DPL almost obsolete. DPL is
only reserved for special circumstances – for example, assessment of a stable patient in a hospital where no MDCT is available.

References

• UpToDate - Initial evaluation and management of abdominal gunshot wounds in adults

• OrthoBullets – Gunshot wounds

Last updated:
Time spent: QID:1604
2023-2-12

314 of 1943
A 56-year-old man was diagnosed with non-Hodgkin lymphoma (NHL) and started on chemotherapy. After two years and a half, he was
planned for stem cell transplant following a relapse. Four days after transplant, he develops a fever of 38.0 °C and severe pain while
defecating. There is no blood covering or mixing with the stool. The stool is of normal consistency. Blood tests show a white cell count of
0.1x109/L (4-11x109/L), platelet count of 30x109/L (150-400x109/L), and hemoglobin of 90 g/L (120-160 g/L). Which one of the following is
the most appropriate next step in management?

A. Complete isolation and monitoring.

B. Start him on broad-spectrum antibiotics.

C. Arrange for blood cultures and chest X-ray and plan further steps based on the results.

D. Platelet transfusion.

E. Sterilization of the gut with special care for anus.

Incorrect. Correct answer is C


45% answered correctly

Explanation:

Correct Answer Is C

Bone marrow destruction using radiation or chemotherapy, is a process in bone marrow transplant before new bone marrow is injected. It
takes a while for the newly transplanted bone marrow to take over reproduction of blood cells; therefore, there will be deficiency of all three
cells lines (red blood cells, white blood cell, and platelets) during this period. Low blood cell count (leukopenia) places the patient at risk of
infections including opportunistic infections, while low platelet count (thrombocytopenia) poses a risk of bleeding.

Post-transplant fever is a common post-op complication of bone marrow transplant without infection. Such fever often develops 4-5 days
after the transplant even in the absence of infection and resolves by days 5-6. However, since infections are quite common in post-transplant
patients and are associated with high mortality, any post-transplant fever should be attributed to infection and thoroughly investigated until
proven otherwise. This is so important that commencement of empirical broad-spectrum antibiotics (option B) could also be considered;
however, recent studies recommend that fever on days 4-5 that often is cytokine-related rather than infectious, and antibiotics are better used
if there is high suspicion of infection through investigations. For this patient, blood culture and chest X-ray to investigate a bacterial infection
before starting antibiotics is the most appropraite option to consider first.

(Option A) At day 4 post-transplant, this patient is already in isolation and monitored closely.

(Option D) With a platelet count of 30x109/L, this patient has thrombocytopenia. Thrombocytopenia is the term applied when the peripheral
blood platelet count falls below 150x109/L. The problem is common in practice, but generally it presents no major clinical problems. However,
if the platelet count falls below 30x109/L, significant bleeding might occur. In this patient no obvious bleeding has happened yet. While
bleeding in the presence of higher platelet of demands platelet transfusion, in the absence of active bleeding, platelet transfusion for this
patient is not necessary at this stage.

(Option E) Colorectal problems are not uncommon in post-bone marrow transplant patients and may be caused by direct effect of radiation or
chemotherapy, graft-versus-host disease of infections. Although, painful defecation in this patient should be seen to and investigated, it is not
the most important priority in order.

References

• UpToDate - Early complications of hematopoietic cell transplantation

• High Fever Occurring 4 to 5 Days Post-Transplant of Haploidentical Bone Marrow or Peripheral Blood Stem Cells

Last updated:
Time spent: QID:1636
2023-2-12

315 of 1943
A 25-year-old man is brought to the Emergence Department after he was retrieved from a car accident by the paramedic staff. He had a head-
on collision at high speed. Upon arrival, he has stable vital signs, and initial surveys clear airway, breathing, and circulation. His left elbow is
significantly swollen and deformed and exquisitely tender to touch. The left hand feels cold, clammy, and numb and is pale. Radial and ulnar
pulses are absent. Which one of the following is the most appropriate next step in management?

A. Reduction in the Emergency Department.

B. X-ray of the elbow.

C. Arrangements for immediate referral to the operating theatre.

D. Simple analgesia.

E. Ulnar nerve decompression.

Incorrect. Correct answer is C


45% answered correctly

Explanation:

Correct Answer Is C

This patient has an exquisitely painful deformed and swollen elbow that suggests a dislocated elbow with or without fracture. Dislocations
can cause neurovascular injuries that pose a significant risk to the affected limb. The deadliest risk is acute limb ischemia which presents
with the following (the 6 Ps):

1. Pain – sudden and severe


2. Pallor – commonly mottled
3. Pulselessness – loss of peripheral pulses
4. Paresthesia – decrease in sensation or loss of sensation
5. Paralysis
6. Poikilothermia – coolness of the affected limb

Once acute limb ischemia is suspected the following steps should be taken immediately:

Limb elevation to the heart level


Loosening of any restrictive bandages or dressings
Notify the specialty registrar immediately without hesitation to make arrangements for urgent transfer to the operating theatre
Placing the patient nil by mouth until review.
Increasing the frequency of neurovascular assessment to every 15 minutes until review.
Adequate analgesia.

This patient has classic signs of acute limb ischemia and requires urgent management before any further complications follow. Of the
options immediate arrangement for being transferred to the operating theatre is the most appropriate next step in management. In the
theatre, any vascular damage is repaired first, followed by nerve salvation (e.g., ulnar nerve decompression (option E) if compromised) or
repair. Fractures and/or dislocations are fixed last.

(Option A) While gentle and very careful alignment of the fractured/ dislocated limb is one of the priorities in the Emergency Department,
reduction is a decision to be made after careful assessment by the specialist. Blind reduction in the Emergency Department may render
further neurovascular injury.

(Option B) Plain X-rays are important to obtain and are a part of routine physical examination in fractures/ dislocation. However, they do not
change the management plan in this scenario. This patient needs to be transferred to the operating theatre due to acute limb ischemia. X-rays
can be obtained after arrangements are made or en route to or in the operating theatre.

(Option D) Although analgesia is of paramount importance for this patient, it does not take precedence over immediate arrangements for
review and treatment by specialist. Moreover, the pain associated with such a significant trauma is very unlikely to be responsive to simple
analgesia. In such instances, opiates are often used.

References

• RCH – Neurovascular observation

• Neurovascular assessment – A guide to using the NSW electronic observation form

Last updated:
Time spent: QID:1650
2023-2-12

316 of 1943
Irukandji syndrome could potentially be caused by:

A. Wasp sting.

B. Bee sting.

C. Spider bite.

D. Jellyfish sting.

E. Snake bite.

Incorrect. Correct answer is D


45% answered correctly

Explanation:

Correct Answer Is D

Irukandji syndrome is a painful, potentially lethal condition caused by certain jellyfish from the Cubozoa class (box jellyfish) species. This
species has a small body (size of a coin) and usually four tentacles. The sting is usually mild and often goes unnoticed by the victim; however,
after 20-30 minutes of the sting, a surge of catecholamine resulting in tachycardia, hypertension, agitation and generalized severe pain and
cramps especially in the abdomen and back. Hypotension and, pulmonary edema, and potentially life-threatening cardiac complications can
follow.

Victims should be transferred to the hospital for:

Analgesia (oral and IV)


Investigation and treatment of cardiac effects (echocardiogram, troponin)
Cardiac monitoring

References

• NCBI – Irukandji Syndrome

• RACGP – AFP – Marine envenomation

Last updated:
Time spent: QID:1706
2023-2-12

317 of 1943
A 24-year-old man presents to the Emergency Department of a rural hospital after he was bitten by a brown snake 15 minutes ago. On
physical examination, he has a laceration of 3 cm on the lateral side of his left lower leg, but no fang marks can be found. Apart from pain and
tenderness of the affected area, the rest of clinical assessment is inconclusive. The nearest tertiary hospital is 150Km away. Which one of the
following is the most appropriate next step in management?

A. Application of pressure bandage and immobilization.

B. Application of venous torniquet.

C. Administration of antivenom.

D. Transferring the patient to the tertiary hospital by ambulance.

E. Irrigation and dressing of the wound.

Correct
45% answered correctly

Explanation:

Correct Answer Is A

In all definite or suspected snake bites, envenomation must consider having occurred until proven otherwise. Neither absence of fang marks
nor lack of envenomation symptoms is not against envenomation because firstly, in many envenomation cases, no fang/bite marks are found.
Secondly, unless massive envenomation occurred there is a time window during which the victim is completely symptomatic.

All snake bite victims should be transferred to a tertiary hospital (option D) where appropriate facilities such as laboratory, intensive care unit,
antivenom kits and experts are available. Arrangements for immediate transfer of the patient to the tertiary hospital 150 km away should be
made as soon as possible. In the meanwhile, the most important first aid measure is application of pressure bandage with immobilization
(PBI). The bandage needs to be broad (15 cm) and preferably elasticized, rather than the previously recommended crepe bandage. The
bandage is applied first over the bite and then to cover the whole limb at a pressure similar to that used for a sprained ankle. Immobilization is
essential; the limb and whole patient must remain immobilized from the time of the bite or as soon as applicable. The use of a PBI more than
4 hours after the bite is unlikely to be effective.

The PBI can be removed when the initial clinical and laboratory assessment shows no evidence of envenoming, and the patient is in a facility
where antivenom is available. For patients with envenoming, the PBI can be removed after administration of antivenom. As there are reports
of cases where envenoming appears to be delayed by early application of a PBI but becomes evident soon after its removal, careful
observation of the patient in the hour after PBI removal is essential.

The following should ALWAYS be avoided

Application of torniquet (option B).


Cleaning, washing, and dressing the wound before the patient is in a facility for definitive care and treatment (option E).
Incision if the bite site.
Administration of antivenom anywhere but the tertiary hospital (if indicated) (option C).

TOPIC REVIEW

Many of the snakes found in Australia are capable of lethal bites to humans. These include Taipans, Brown snakes, Tiger snakes, Death
Adders, Black snakes, Copperhead snakes, Rough Scaled snakes, and many Sea snakes. Venom is produced in the modified salivary glands of
the snake and forced out under pressure through paired fangs in the upper jaw. Snake venoms are complex mixtures of many toxic
substances which can cause different effects in human victims. The life-threatening early effect in Australian snake bite is neurotoxic muscle
paralysis which kills by causing breathing failure. Other significant effects include:

bleeding due to coagulation failure


muscle damage causing kidney failure
headache
nausea and vomiting
occasionally, initial collapse or confusion followed by partial or complete recovery
abdominal pain
blurred or double vision, or droopy eyelids
difficulty in speaking, swallowing, or breathing
swollen tender glands in the groin or axilla of the bitten limb
limb weakness or paralysis
respiratory weakness or respiratory arrest

Important tips in initial management of snake bites:


318 of 1943
Keep the victim at rest, reassured and under constant observation
Commence resuscitation if necessary
Immobilize the affected limb
Transport the victim to a medical facility, preferably by ambulance
As a rule, all victims of suspected or definitive snake bites should be assumed to have been envenomated until proven otherwise.
DO NOT cut or incise the bite
DO NOT use an arterial tourniquet
DO NOT wash or suck the bite because identification of venomous snakes can be made from venom present on clothing or the skin
using a Venom Detection Kit.

Antivenom

Data suggests that antivenom might prevent certain envenoming syndromes if used early but may have little if any effect once major
envenoming syndromes are established. There is no high-level evidence as a guide in this regard and clinicians have to balance the risks of
giving antivenom (such as anaphylaxis and serum sickness) against the potential benefits of giving it early without waiting for confirmation of
envenoming. The following are absolute and relative indications for antivenom administration:

Absolute indications:

Reported sudden collapse, seizure or cardiac arrest


Abnormal international normalized ratio (INR) (reference interval, 0.9–1.3)
Any evidence of paralysis, with ptosis and/or ophthalmoplegia being the earliest signs

Relative indications:

Systemic symptoms (vomiting, headache, abdominal pain, diarrhea)


Leukocytosis
Abnormal activated partial thromboplastin time (outside laboratory’s reference interval)
Creatine kinase level > 1000 U/L

References

• MJA – Snakebite in Australia: a practical approach to diagnosis and treatment

• SA Health – Snake bite and spider bite: management guidelines

Last updated:
Time spent: QID:1722
2023-2-12

319 of 1943
A 12-year-old schoolgirl is brought to the Emergency Department of a tertiary hospital after she collapsed at school. En route to the hospital,
she was started on dextrose 5% drip at a rate of 60 ml/minute. On examination after arrival, she has blood pressure of 180/110 mmHg, pulse
rate of 50 bpm and respiratory rate of 12 breaths per minute. She has an O2 saturation of 95% on room air. Neurological exam shows the
presence of doll eye reflex is present. Which one of the following would be the next best step in management?

A. Arrange for emergency CT scan of the head.

B. Stop the dextrose drip and start her on saline.

C. Give intravenous steroids.

D. Intubate her and start mechanical ventilation.

E. Neurosurgical reference.

Incorrect. Correct answer is B


45% answered correctly

Explanation:

Correct Answer Is B

The findings of high blood pressure and bradycardia (Cushing reflex) point towards increased intracranial pressure (ICP) as the most likely
cause. Cushing reflex (also the vasopressor response, Cushing effect, Cushing phenomenon and Cushing reaction) is a physiological nervous
system response to ICP. Cushing triad is: (1) hypertension, (2) bradycardia and (3) irregular breathing e.g., Cheyne-Stoke. This triad may
indicate imminent brain herniation.

Increased ICP is more underpinned by the presence of the ‘doll eye’ sign (movement of the eyes in the same direction as the head) signifying
involvement of brainstem, probably due to increased intracranial pressure.

The raised ICP is highly likely to be compromised by dextrose drip which has already been inappropriately started for the patient. Dextrose is
rapidly consumed by cells and the remaining free water shifts into the brain extravascular tissue, and results in worsening of the edema,
swelling, and more rise in the ICP. For this reason, the dextrose drip should be replaced with a normal saline as the most important immediate
management.

An unconscious patient is not able to maintain airway patency. Furthermore, there is significant risk of aspiration; therefore, the patient should
be intubated (option D), but not as the first priority at this stage, considering the fact that the patient is breathing spontaneously and is not
hypoxemic (O2 saturation 95%). The patient should then be taken for CT scan of the head (option A) for determination of the likely causes of
her problem. Consultation with or referral to the neurosurgery specialist (option E) should be arranged.

Intravenous methylprednisolone (option C) has shown effective in spinal cord compressions and cases of increased ICP due to tumors and
abscesses. If, after neuroimaging, the cause of ICP was found to be an abscess or a tumor, corticosteroids may be considered as a part of
management plan.

References

• Beat - Intravenous Fluid Therapy in Traumatic Brain Injury and Decompressive Craniectomy

Last updated:
Time spent: QID:458
2023-2-12

320 of 1943
A 3-year-old child is brought by his parents to the Emergency Department (ED) after he swallowed two button magnets. The child was initially
well upon arrival at the ED and did not have any lethargy, abdominal pain, nausea, or vomiting. The parents explain that the event was
witnessed and that they had brought the child over as soon as they could. The child does not have any illnesses and is otherwise well. An
initial plain abdominal film was requested with results as shown:

The child was placed on NPO (nil per mouth), and intravenous (IV) hydration was started. Twelve hours later, you repeat the x-ray and note the
same findings as the initial x-ray. The patient has stable vital signs all throughout and is still asymptomatic. Which of the following is the most
appropriate management?

A. Perform serial x-rays every 4 hours.

B. Give the child laxatives.

C. Discharge him, and advise the parents to come back if he develops symptoms.

D. Refer to surgery for laparotomy.

E. Refer to a specialist for endoscopy.

Incorrect. Correct answer is E


45% answered correctly

Explanation:

Correct Answer Is E

Children of all ages can ingest a foreign body, but the incidence is most common between the ages of 6 months to 3 years old, and in those
with behavioral or developmental problems. The management of foreign body ingestion depends mainly on three factors:

1. the site of obstruction and


2. the physical properties of the object, including size, shape, and composition
3. the timing of ingestion.

Site of obstruction

Assessment of the site of obstruction can be determined by asking about the following symptoms:

321 of 1943
Esophagus:

Dysphagia
Food refusal, drooling
Chest pain
Vomiting, hematemesis
Throat pain
Unexplained fever
Altered mental state.

Esophageal obstruction usually occurs at these 3 sites:

Cricopharyngeal sling at the thoracic inlet (between the clavicles on CXR) – 70% of cases
Mid-esophagus (at the junction of aortic arch and carina on CXR) – 15% of cases
Lower esophageal sphincter – 15% of cases

NOTE - Impaction at other sites in the esophagus suggests an underlying esophageal abnormality.

Stomach and lower GI tract:

Abdominal pain, abdominal distension


Vomiting, hematemesis
Hematochezia
Melena
Unexplained fever

NOTE - Coughing, choking, and dyspnea are considered emergencies and are best treated as foreign body inhalation rather than
digestion.

Determination of risk

High-risk objects

button batteries: most are asymptomatic, but they may erode GI mucosa and lead to fatalities
NOTE: parents may be advised to offer honey to children older than 12 months of age with suspected button battery
ingestion; continuing at regular intervals until reaching the hospital
large objects (>6 cm long and/or >2 cm wide): potential entrapment at the pylorus
superabsorbent polymers (e.g., desiccants)
magnet plus a metal object or >1 magnet ingestion
lead-based objects that fail to transit through the stomach
multi-component objects (e.g., toys with lights, motors, and batteries) as they may break apart

High-risk children

Pre-existing GI tract abnormalities (repaired or unrepaired congenital abnormalities, eosinophilic esophagitis)


Neuromuscular disease

Timing

The timing of the ingestion would also be helpful in determining whether the object has passed through the esophagus or the stomach. In
general, it takes about 6 to 8 hours for food to pass through the stomach.

The next step after the initial assessment is to determine whether imaging is needed if the suspected object is radiopaque. The following
algorithm provides the next steps:

322 of 1943
Royal Children's Hospital ( https://rcg.org.au)

In this case, a witnessed ingestion of button batteries, PA chest, or abdominal x-ray is the correct initial step. A lateral- chest x-ray may be
indicated as well. On the x-ray, batteries are seen past the esophagus and in the stomach. However, button batteries are high-risk objects;
therefore, urgent referral to specialist services (e.g., ENT, surgery, or gastroenterology) for removal of the batteries would be the most
appropriate next step in management.

Passage of a foreign body lodged at the lower esophageal sphincter may occur after several hours, and the stable patient with normal
anatomy may be observed in the hospital, and allowed to eat, with a follow-up x-ray in 12-24 hours.

In this child with a high-risk foreign body that has not passed after 12 hours of observation and conservative management, longer waits and
repeating x-rays (option A) is inappropriate.

Although he is well and asymptomatic now, button batteries still have a risk of mortality and require referral for removal. Therefore,
discharging the child home and advising him to return if there are symptoms (option C) is incorrect as well.

Laxatives (option B) are never used in the treatment of foreign body removal. Medications such as glucagon 0.02-0.03 mg/kg IV (max 0.5 mg),
benzodiazepines, and nifedipine have been reported to assist foreign body passage through the lower esophageal sphincter in adults.
However, their role in children has not been established, and they are unlikely to be successful in children with anatomic abnormalities.

Laparotomy (option D) to remove foreign bodies is an unnecessarily invasive procedure for foreign body removal in most cases. Unless the
child is having severe symptoms or is rapidly deteriorating, laparoscopic removal is the method of choice.

NOTE -If the child were of older age, consider intentional foreign body ingestion. This may warrant an initial mental health review and
possible referral to psychiatry for further evaluation

References

• RCH - Foreign body ingestion

• WA Health - Perth's Children Hospital: Foreign body - oesophageal and ingested

Last updated:
Time spent: QID:1764
2023-2-12

323 of 1943
Which one of the following is not seen in zygomatic bone fracture?

A. Horizontal diplopia.

B. Vertical diplopia.

C. Enophthalmos.

D. Unstable temporomandibular joint.

E. Inability to open the mouth.

Incorrect. Correct answer is D


45% answered correctly

Explanation:

Correct Answer Is D

Zygoma bone has a prominent and important position in the facial skeleton. It forms a significant portion of the floor and the lateral wall of
the orbit, and forms a portion of the zygomatic arch (malar eminence) which plays a key role in the determination of facial morphology.

The fractures of zygoma are often complex, and may affect either if the following portions:

Zygomaticofrontal suture (along the orbital rim – lateral orbital wall)


Zygomaticomaxillary suture (medially)
Zygomatic arch

Zygoma bone and its relation with frontal, maxillary and relevant structures

Most zygomatic fractures give fractures of maxillary sinus, zygomatic arch, inferior orbital wall and lateral orbital wall. Clinical picture in
zygomatic fractures can include:

Pain (70% of patients)

Binocular diplopia (30% of patients) – The diplopia can be predominantly vertical, horizontal or oblique depending on the part of the
zygomatic complex which is mostly affected. Diplopia can be secondary to muscle entrapment, neuromuscular injury, or
intramuscular haematoma.

Periorbital and/or subconjunctival ecchymosis (50% of patients)

Paresthesias in the distribution of the infraorbital, zygomaticofacial, or zygomaticotemporal nerves can occur.

Enophthalmos – Significant malar depression can be seen with step defects at the infraorbital rim, frontozygomatic suture, and
zygomatic buttress of the maxilla intraorally.

Inferior displacement of the lateral canthal tendon is common.

Proptosis may be present due to orbital oedema or haemorrhage. Acute orbital hematoma may cause vision compromise and should
be managed appropriately.

Difficulty opening mouth – Posterior displacement of the fracture fragment may impinge on movement of the mandible causing
difficulty with mastication and mouth opening. In addition reflexive spasms of masseter muscle may contribute to difficulty mouth
opening and mastication as well.

324 of 1943
Origins of the masseter muscle and its relation to zygoma bone and mandible

Crepitus from subcutaneous emphysema or proptosis and visual loss from orbital emphysema may occur with forceful nose blowing.
Patients should be cautioned against this.

In some patients, ipsilateral epistaxis also is noted as a result of lacerated maxillary sinus mucosa.

NOTE - Although zygomatic fractures are not true orbital blowout fractures, entrapment of orbital contents, enophthalmos, and diplopia
with eye movement restriction may occur because of the contributions of the zygomatic bone to the orbital floor. Lateral orbital rim is
displaced in most of zygomatic complex fractures.

Temporomandibular joint (TMJ) is a synovial joint between the mandible and temporal bone. Zygomatic fractures does not affect the TMJ
structure (e.g. instability), but mouth opening and chewing may be restricted through mechanisms other than TMJ instability or involvement.

References

• Medscape - Zygomatic Complex Fractures

• UpToDate - Initial evaluation and management of facial trauma in adults

Last updated:
Time spent: QID:514
2023-2-12

325 of 1943
A 26-year-old man presents to the Emergency Department after he was hit in the face by a racket in a squash game. Based on clinical
findings, diagnosis of orbital floor fracture (blow-out fracture) is made. Which one of the following is the most consistent finding suggesting
blow-out fracture?

A. Diminished vision.

B. Subconjunctival hemorrhage.

C. Anesthesia of the cheek.

D. Inability to open the mouth.

E. Horizontal diplopia.

Incorrect. Correct answer is C


45% answered correctly

Explanation:

Correct Answer Is C

Inferior orbital wall fracture is associated with the following features:

Binocular vertical diplopia


Subconjunctival hemorrhage
Peri-orbital ecchymosis
Anesthesia or paresthesia of lower eyelid, upper lip, and maxillary area due to infraorbital nerve injury
Limited vertical movement
Enophthalmos
Proptosis if there is severe orbital edema may lead to proptosis -once the edema subsides (usually 1-2 weeks) enophthalmos may be
seen.

Of the given options, anesthesia of the cheek is the most specific finding indicative or orbital floor fracture and injury to the infraorbital nerve.

(Options A and B) Diminished vision and subconjunctival hemorrhage can be found in blow-out fracture, as well as many other conditions that
have eye injury as a feature.

(Option D) Inability to open the mouth is a finding in complex zygomatic fractures, particularly if the zygomatic arch is involved.

(Option E) Binocular horizontal diplopia is seen in lateral orbital wall fractures.

References

• Medscape - Orbital Floor Fractures (Blowout Fractures)

• UpToDate - Initial evaluation and management of facial trauma in adults

Last updated:
Time spent: QID:515
2023-2-12

326 of 1943
A 32-year-old man was involved in a motor vehicle accident and sustained multiple injuries including tibia and pelvis fractures. The patient is
managed accordingly. On the second day after hospital admission, you are called by the ward nurse because he has developed acute
confusion and shortness of breath. A chest X-ray shows bilateral lung infiltrates. Which one of the following is the most likely diagnosis?

A. Pulmonary embolism.

B. Fat embolism.

C. Air embolism.

D. Pneumonia.

E. Drug withdrawal.

Incorrect. Correct answer is B


45% answered correctly

Explanation:

Correct Answer Is B

Considering the history of pelvic and tibial fractures, clinical, and chest X-ray findings, fat embolism is the most likely diagnosis.

Fat embolism is a common problem following trauma to or surgery of lower extremities. These conditions can result in fat escaping from
bone marrow into circulation and embolus formation.

The symptoms of fat embolism syndrome often present 24 to 72 hours after the insult, and involve lungs, brain and skin, and results in a triad
of respiratory, neurological, and cutaneous manifestations:

Lung problems including rapid breathing, shortness of breath (dyspnea), and hypoxemia – respiratory signs are the earliest findings in
fat embolism and virtually exist in all patients.

Neurological problems often follow respiratory involvement, and include irritability, agitation, headache, confusion, seizures, or coma

Petechial skin rash is caused by occlusion of small blood vessels with emboli. Upper torso is the most common site involved. Eyes
may be affected as well. Petechial skin rash is seen in approximately 60% of patients.

Diagnosis is made based on the history and physical exam findings, as well as exclusion of other possible causes.

Investigations include chest x-rays, blood tests, and CT scan a, and cardiac assessment with ECG and echocardiography to exclude cardiac
causes of the presentation.

Bilateral chest lung infiltrates in a patient with fat embolism

327 of 1943
purpruic lesions in the brain (post-mortum) of a patient with fat embloism

purpric lesions of fat embolism in the upper toros and axillary area of a patient with fat embolism

(Option A) Pulmonary embolism is the second most likely differential diagnosis; however, the history and chest X-ray findings are more
consistent with acute respiratory distress syndrome caused by fat embolism.

(Option C) Minor cases of air embolism are common and cause minimal or no symptoms. Severe cases are characterized by hemodynamic
collapse and/or acute insufficiency of certain organs, including the lungs, brain, and spinal cord. Dyspnea is present in almost all patients and
may be accompanied by substernal chest pain, a sense of impending doom, lightheadedness, or dizziness. Signs of air embolism include a
gasp or cough when the bolus of air enters the pulmonary circulation, a sucking noise as air is sucked into the intravascular space, a mill
wheel murmur (a churning sound heard throughout the entire cardiac cycle), tachypnea, tachycardia, hypotension, wheezing, crackles,
respiratory failure, altered mental status, focal neurological findings, crepitus over superficial vessels, livedo reticularis, and bubbles within
retinal arteries. Although air embolism can cause the presentation in the scenario, with tibial and pelvic fractures in the history fat embolism
is more likely to be the diagnosis.

(Option D) Hospital acquired pneumonia can be seen in patients after 48 hours of hospital admission. The time of symptoms onset, bilateral
infiltrates and absence of fever, cough and sputum makes pneumonia a less likely diagnosis.

(Option E) Drug withdrawal symptoms tend to occur earlier. Besides, dyspnea and chest X-ray findings are inconsistent with the condition.

References

• Medscape - Fat Embolism

• Journal of Emergencies, Trauma, and Shock - Emergency management of fat embolism syndrome

Last updated:
Time spent: QID:524
2023-2-12

328 of 1943
A 22-year-old man has sustained multiple injuries during a motor vehicle accident. On examination, you find a laceration in his scalp, blood in
the urethral meatus, and a left-sided pneumothorax. He has a blood pressure of 90/60 mmHg, pulse rate of 110 bpm and respiratory rate of
26 breaths per minute. The breath sounds over the right hemithorax are diminished. Neck and forehead veins are not distended and the
trachea is normal in position. Which one of the following would be of greatest importance as initial management of this patient?

A. Management of possible urethral injury suggested by the presence of blood in the meatus.

B. Management of the pneumothorax.

C. Management of the scalp laceration.

D. Checking the tetanus immunization status.

E. Evaluation for possible blunt abdominal injury.

Incorrect. Correct answer is B


45% answered correctly

Explanation:

Correct Answer Is B

Current guidelines suggest initial management of traumatic patients in a sequence of A (airway), B (breathing), and C (circulation).

The clinical findings suggest a pneumothorax in the right hemithorax as well as pre-shock indicated by low blood pressure and compensatory
tachycardia. Pneumothorax can compromise breathing and result in death if not treated promptly by chest tube insertion.

(Option A) Blood in the urethra indicates urogenital injuries and should be investigated after the patient is hemodynamically stabilized.

(Option C) Scalp laceration is another step to take after intravenous fluid resuscitation has been started and chest tube inserted. This can
prevent ongoing blood loss. In the meanwhile, bleeding can be controlled by applying pressure on the laceration if required.

(Option D) Tetanus immunization, if indicated, is important once the patient is clinically stable.

(Option E) Although copious bleeding from scalp laceration can result in hemorrhagic shock and justify the low blood pressure and
tachycardia, possible abdominal injuries should not be overlooked, and prompt investigation must be undertaken.

NOTE - Given the pre-shock status of the patient. evident by the decreased blood pressure and the tachycardia, intravenous fluid
resuscitation would come prior to correction of the pneumothorax; however, if the pneumothorax was of tension type, immediate relieve
by needle thoracotomy (not an option) would take precedence.

References

• UpToDate - Initial management of trauma in adults

• Medscape - Trauma

Last updated:
Time spent: QID:525
2023-2-12

329 of 1943
A 56-year-old man presents to the emergency department with acute right eye pain, redness and blurred vision. On examination, he has eye injection and corneal haziness, and the
eye feels hard to palpation. Which one of the following is the most appropriate immediate management?

A. Oral acetazolamide.

B. Topical pilocarpin.

C. Carteolol.

D. Laser iridotomy.

E. Topical corticosteroids.

Incorrect. Correct answer is C


45% answered correctly

Explanation:

Correct Answer Is C

The clinical findings are a classic description of acute closed-angle glaucoma – an acute rise in the pressure of the anterior chamber. This condition is real emergency and if left
untreated catastrophic results can ensue. Treatment of acute closed-angle glaucoma is with immediate application of topical agents that inhibit aqueous production. The following
topical agents might be used:

Topical beta blockers (first-line): timolol, carteolol


Alpha adrenergic agonists: e.g. apraclonodine
Topical prostaglandins

Other management options depend on the setting:

If the patient can be seen within 1 hour of presentation, urgent referral to an ophthalmologist will be the next best step.
If the referral is delayed, the patient should be given acetazolamide PO (250mg x2) (option A). After one hour of treatment, topical pilocarpine (option B) can be started as
well (2 doses 15 minutes apart).

NOTE – The most frequently drug group in the emergency department is topical beta blockers (timolol, carteolol) and intravenous acetazolamide.

Of the given options carteolol, is the most appropriate management option.

(Option D) Laser iridotomy will be the definite treatment of closed-angle glaucoma and is considered after the acute attack subsides. This procedure is not applicable in acute
setting.

(Option E) Topical corticosteroids have shown no benefit in an acute attack but might be helpful in reducing the corneal inflammation after the acute phase has subsided.

References

• Eye Emergency Manual

• RACGP – Ocular emergencies

• Medscape - Acute Angle-Closure Glaucoma in Emergency Medicine

• Therapeutic Guidelines

Last updated:
Time spent: QID:306
2023-2-12

330 of 1943
A 65-year-old woman presents to the emergency department with sudden onset decrease in visual acuity and floaters in the visual field of her right eye. The eye appears normal on
inspection, and is not painful. Which one of the following could be the most likely diagnosis?

A. Age-related macular degeneration.

B. Closed-angle glaucoma.

C. Cataract.

D. Retinal detachment.

E. Presbyopia.

Incorrect. Correct answer is D


45% answered correctly

Explanation:

Correct Answer Is D

The symptoms are highly suggestive of retinal detachment. In retinal detachment, the retina peels away from its underlying layer of support tissue. Initial detachment may be
localized or broad, but without emergency treatment the entire retina may detach and result in loss of vision.

Retinal detachment presents with sudden onset of visual impairment, floaters and flashes. The patient describes black dots in the visual field. The patient may describe the visual
loss as a curtain coming down in front of the eyes. Visual field will be completely lost if macula is detached.

Vision through the affected eye of a patient with retinal detachment

Ophthalmoscopy may show detached retinal fold as a large grey shadow in the vitreous.

Retinal detachment - slit lamp view

If the detachment is not repaired within 24-72 hours, permanent damage may occur.

Risk factors for retinal detachment include:

Aging
Congenital eye diseases
Cataract surgery
Diabetic retinopathy
Focal retinal atrophy

331 of 1943
Family history of detachment
Hereditary vitreoretinopathy
Myopia (axial)
Prematurity
Trauma
Uveitis

(Option A) Age-related macular degeneration presents with slowly progressive visual loss in an elderly. There are no floaters or flashes and compared to acute-onset of symptoms in
retinal detachment, the visual impairment occurs insidiously.

(Option B) Open-angle glaucoma is associated with gradual vision impairment that often initially affects the peripheral vision, progressing to involve the entire visual field.

(Option C) Cataract is the opacification of the lens. The patient will have difficulty in seeing both far and near objects with the affected eye. Dazzling is a common complaint and
vision is worse in bright light.

(Option E) In presbyopia, the lens becomes stiff and less easy to accommodate, resulting in declining ability to focus on near objects. Presbyopia presents with eye strain, difficulty
seeing in dim light, and problems in focusing on small objects and fine prints.

References

• http://www.racgp.org.au/afp/2014/april/flashes-and

• http://www.aafp.org/afp/2004/0401/p1691.html#afp20

• http://emedicine.medscape.com/article/798501-overv

• http://emedicine.medscape.com/article/798501-clini

Last updated:
Time spent: QID:291
2023-2-12

332 of 1943
A 47-year-old man presents to the emergency department with acute onset of pain in his right eye, associated with lacrimation, nausea and severe unilateral headache. Further
assessment establishes the diagnosis of acute closed-angle glaucoma. The patient is managed initially with timolol drop, intravenous acetazolamide and pilocarpine eye drop.
Which one of the following would be to most appropriate long-term management of this patient?

A. Laser iridotomy.

B. Laser trabeculectomy.

C. Topical cortisone.

D. Topical pilocarpine 4%.

E. Topical carbonic anhydrase inhibitor.

Correct
45% answered correctly

Explanation:

Correct Answer Is A

Long term management of closed-angle glaucoma is with iridotomy, either by laser or surgery. A small piece of iris is removed at 12 o’clock, allowing free drainage of aqueous.

(Option B) Laser trabeculectomy is treatment of choice in patients with chronic open angle glaucoma with failed conservative management. The procedure includes establishing a
pressure valve at the limbus, so that aqueous can flow freely into a conjunctival bleb.

(Option C) Corticosteroids are not used for treatment of glaucoma.

(Option D) Topical pilocarpine is used, in conjunction with intravenous acetazolamide, for urgent decompression in acute closed-angle glaucoma.

(Option E) Carbonic anhydrase inhibitors such as topical acetazolamide and dorzolamide, or oral acetazolamide are used in pharmacological management of chronic simple
glaucoma. Oral acetozolamide is also used in pre-referral management of acute closed-angle glaucoma.

References

• Therapeutic Guidelines

• Medscape - Acute Angle-Closure Glaucoma in Emergency Medicine

Last updated:
Time spent: QID:307
2023-2-12

333 of 1943
A 60-year-old man presents to the emergency department with painful red right eye and photophobia. On examination, the right eye pupil has normal reaction to light. Which one of
the following is the most likely diagnosis?

A. Acute iritis.

B. Acute close-angle glaucoma.

C. Acute keratitis.

D. Bacterial conjunctivitis.

E. Viral conjunctivitis.

Incorrect. Correct answer is C


45% answered correctly

Explanation:

Correct Answer Is C

The clinical picture is consistent with acute keratitis (inflammation of the cornea) as the most likely diagnosis.

Acute keratitis or corneal ulcer presents with:

Red eye
Eye pain
Reflex lacrimation
Photophobia
Normal reactive pupils

(Option A) Acute iritis is associated with a red painful eye and irregular constricted pupil. Photophobia can be a feature. The cornea is often normal, but the vision is usually blurry.

(Option B) Acute closed-angle glaucoma presents with a painful red eye, which is hard to touch. There is photophobia and haziness of the cornea, as well as headache, nausea and
vomiting. The pupils are dilated fixed and not reactive to light or accommodation. Light reflex is absent.

(Options D and E) Conjunctivitis (either bacterial or viral) is characterized by red eye(s), normal cornea and normal reactive pupils. There is no visual abnormality or photophobia.

References

• Eye Emergency Manual

• Optometrist Australia - Herpetic epithelial keratitis

Last updated:
Time spent: QID:308
2023-2-12

334 of 1943
A 32-year-old man presents to your office following trauma to his left eye. On slit-lamp examination, hyphema in the anterior chamber is noted. Which one of the following is the
most appropriate next step in management?

A. Immediate referral to ophthalmologist.

B. Give antibiotics and arrange follow-up in a week.

C. Advise ice packs and review in the emergency department tomorrow.

D. Prescribe NSAIDs and follow up in 3 days.

E. Drain the hyphema in the emergency department.

Correct
45% answered correctly

Explanation:

Correct Answer Is A

Hyphema is the presence of blood in the anterior chamber. It occurs usually after trauma to the eye. If there is no history of trauma, non-accidental injury in children or coagulation
disorders should be suspected.

According to Australian therapeutic guidelines, an urgent ophthalmology review on the same day of the presentation is mandatory and the most appropriate management
option. While awaiting an ophthalmology review, the patient should be instructed to rest in bed with the head elevated 30 to 45° and be given an eye shield to protect the eye from
further trauma. Intraocular pressure must be monitored regularly.

Hyphema is associated with recurrent bleeding, glaucoma, and blood staining of the cornea. Any of these three may result in permanent vision loss.

(Option B) Antibiotics are not indicated in the treatment of hyphema.

(Option C) 'Ice packs and review' is not appropriate. Urgent referral should be arranged as the most appropriate step.

(Option D) Oral or topical NSAIDs are contraindicated, as they may cause re-bleeding.

(Option E) Draining a hyphema in the emergency department is not recommended. It should be performed by an ophthalmologist in the operation room.

References

• RACGP: Ocular emergencies

• Medscape - Hyphema

Last updated:
Time spent: QID:309
2023-2-12

335 of 1943
Which one of the following conditions is most consistent with the funduscopic findings shown in the accompanying photograph?

A. Age-related macular degeneration.

B. Central retinal artery occlusion (CRAO).

C. Chronic simple glaucoma.

D. Acute angle glaucoma.

E. Diabetic retinopathy.

Incorrect. Correct answer is C


45% answered correctly

Explanation:

Correct Answer Is C

The photograph shows an increased ratio of the optic cup (light yellow circle) to the optic disc (orange circle). Also, the retinal vessels seem to be cut when they enter the optic disc.
These findings are characteristics of chronic simple glaucoma.

Normal optic cups occupy less than 50% of the optic disk. The median cup-to-disc ratio is 0.2 to 0.3 (20-30%). With chronically increased intraocular pressure (IOP), this ration
increases. In chronic simple glaucoma, the cup size increases (especially along the vertical axis); therefore, this ratio increases as a result. Individuals with a cup-to-disc ration of
over 50% need further evaluation.

As damage progresses ,the optic disc becomes pale (atrophied) and the disc widens and becomes deeper, so the blood vessels of the disc appear to be broken as they enter the
cup and disappear, then appear at the base again.

(Option A) The retina in age-related macular degeneration has drusen in dry type, and vascular proliferation in wet type.

(Option B) In central retinal artery occlusion, the retina becomes pale and there is cherry-red discoloration of the macula (cherry-red spot).

(Option D) Acute closed-angle glaucoma is not associated with significant retinal changes.

(Option E) Diabetic retinopathy presents with microaneurysms, exudates, cotton-wool spots and flame hemorrhages on funduscopy.

References

• Medscape - Primary Open-Angle Glaucoma (POAG)

Last updated:
Time spent: QID:310
2023-2-12

336 of 1943
Which one of the following is the most appropriate definite management of cataract?

A. Intraocular lens(IOL).

B. Phaecoemulsification.

C. Use of protective measures such as sunglasses.

D. Topical corticosteroids.

E. Pilocarpine drop.

Incorrect. Correct answer is A


45% answered correctly

Explanation:

Correct Answer Is A

Nothing can be done for an opacified lens and it should be removed and replaced with a synthetic intraocular lens. The best method for lens replacement is phaecoemulsification.

(Option B) Phaecoemulsification is a modern cataract surgery for lens replacement, in which the lens is emulsified with an ultrasonic handpiece and aspirated from the eye.
Aspirated fluids are replaced with irrigation of balanced salt solution, maintaining the anterior chamber, as well as cooling the handpiece. Phaecoemulsification and extracapsular
cataract extraction (ECCE) is the best method for intraocular lens replacement.

(Option C) Protective measures such as wearing sunglasses are advised to prevent further damage to the lens, but not for treatment.

(Option D) Short-term topical corticosteroid may be used for management of post-operative inflammation. Corticosteroids are not useful for treatment of cataract itself.

(Option E) Pilocarpine drop causes pupil constriction, and is used for initial management of closed-angle glaucoma or long-term medical management of chronic simple glaucoma.

References

• Medscape - Senile Cataract (Age-Related Cataract)

Last updated:
Time spent: QID:311
2023-2-12

337 of 1943
Five days after phaecoemulsification surgery and intraocular lens replacement due to cataract, a 68-year-old man presents with redness and blurred vision of the treated eye. The
eye is illustrated in the accompanying photograph. Which one of the following is the condition shown in the photograph?

A. Hyphema.

B. Hypopyon.

C. Retinal detachment.

D. Central retinal artery occlusion.

E. Keratitis.

Incorrect. Correct answer is B


45% answered correctly

Explanation:

Correct Answer Is B

Phaecoemulsification, the current standard procedure for lens replacement in cataract, has a number of potential post-operative complications. Anterior uveitis is one of such
complications. Anterior uveitis presents with eye pain and redness (conjunctiva and episclera in a circumcorneal fashion), blurred vision, photophobia, and reflex lacrimation. The
pupil is often constricted but regular unless adhesions develop. Hypopyon is a feature that could be seen in anterior uveitis, and is accumulation of leukocytic exudate in the anterior
chamber. The exudate settles at the dependent aspect of the eye due to gravity. The eye illustrated in the photograph has collection of exudate material at the bottom of the anterior
chamber due to gravity, characteristics of hypopyon.

The most common post-operative complication associated with phaecoemulsification is thickening of the posterior capsule. During the surgery, the posterior part of the capsule is
deliberately left to make the surgery safer. Over a few months to few years, this capsule opacifies in 5-30% of the patients and resembles the recurrence of the cataract. This
condition is simply treated with capsulotomy with Yag laser in an outpatient setting.

Few patients may have post-operative irritation and inflammation. A short course of topical corticosteroids will take care of the problem. Other very rare complications include
vitreous hemorrhage, retinal detachment, glaucoma and endophthalmitis. Central retinal artery occlusion is not a complication of phaecoemulsification.

References

• Wiley Online Library - Complications of cataract surgery

Last updated:
Time spent: QID:312
2023-2-12

338 of 1943
A 66-year-old woman presents with sudden decrease of vision in her right eye 45 minutes ago. She has a 15-year history of type II diabetes mellitus and hypertension for the past
five years, for which she is on enalapril. On examination, the visual acuity of the left eye is 12/18, and that of the right eye is limited to only finger counting. Fundoscopic examination
of the right eye is shown in the following photograph. Which one of the following is the most likely diagnosis?

A. Central retinal artery occlusion (CRAO).

B. Central retinal vein occlusion (CRVO).

C. Age-related macular degeneration.

D. Diabetic retinopathy.

E. Hypertensive retinopathy.

Incorrect. Correct answer is A


45% answered correctly

Explanation:

Correct Answer Is A

The findings in the photograph are a pale retina and a cherry-red macula (cherry-red spot) associated with central retinal artery occlusion (CRAO).

(Option B) Central retinal vein occlusion (CRVO) has a more insidious onset, unless the vein has acutely undergone complete obstruction. The fundoscopic findings in CRVO are
tortuous retinal vessels, edema of the optic nerve and diffuse hemorrhages in all the four quadrants of the retina.

(Options C, D and E) Age-related macular degeneration, diabetic retinopathy and hypertensive retinopathy are not usually associated with sudden onset of visual loss. Moreover,
funduscopy will show different patterns.

References

• Medscape - Central Retinal Artery Occlusion (CRAO)

Last updated:
Time spent: QID:313
2023-2-12

339 of 1943
A patient presents to your practice with gradual loss of vision. The accompanying photograph is the fundus of one of his eyes on funduscopic exam. Which one of the following is
the most likely diagnosis?

A. Diabetic retinopathy.

B. Hypertensive retinopathy.

C. Age-related macular degeneration.

D. Central retinal artery occlusion.

E. Central retinal vein occlusion.

Incorrect. Correct answer is B


45% answered correctly

Explanation:

Correct Answer Is B

Findings on funduscopic examination are papilledema (C-shaped head of the optic nerve), straightening of the vessels, thickened opacified vessel walls (copper-wiring) and
arteriorvenous nicking. These findings are suggestive of hypertensive retinopathy.

Straightening and constriction of vessels due to arteriosclerosis is the first funduscopic finding in hypertensive retinopathy. This is followed by arteriorvenous nicking - when the
sclerotic arteriole presses against the thin wall of an adjacent venule, the venule will appear as hourglass.

Further progression of hypertensive retinopathy will lead to:

Retinal hemorrhages (either flame or dot-blot)


Cotton-wool spots
Optic nerve edema
Star-shaped macula and hard exudates

(Option A) Some features of hypertensive retinopathy can be seen in diabetic retinopathy, as well as vascular proliferation in proliferative diabetic retinopathy, but arteriorvenous
nicking, copper-wiring and more importantly papilledema are characteristic of hypertensive retinopathy.

(Option C) Age-related macular degeneration presents with drusen, pigment and sometimes hemorrhage of the macula in dry type, and proliferation of vessels from choroid into the
neurosensory retina and macular hemorrhages and exudates in macule in wet type.

(Option D) Central retinal artery occlusion (CRAO) gives a different picture with pale retina and the cherry-red spot.

(Option E) Retinal vein occlusion gives tortuous congested veins and hemorrhage as the typical picture.

References

• Merck Manuals - Hyeprtensive Retinopathy

• Medscape - Ophthalmologic Manifestations of Hypertension

Last updated:
Time spent: QID:314
2023-2-12

340 of 1943
A 60-year-old woman presents to your practice, complaining of visual problems. She describes the gradual loss of the ability to see clealry and that the center of her vision is blurry.
Furthermore, she sees horizontal lines wavy. Which one of the following would be the next best step in management?

A. Referral to ophthalmologist.

B. Pilocarpine drops.

C. Laser emulsification.

D. Duplex Doppler sonography of the carotid artery.

E. Iridectomy.

Correct
45% answered correctly

Explanation:

Correct Answer Is A

Central visual impairment is characteristic of macular degeneration, as is the distortion of objects such as seeing straight lines wavy. Once the diagnosis of macular degeneration is
suspected, urgent referral to ophthalmologist would be the next best step in management.

Amsler grid as it might appear to some one with AMD. Patients with
Vision of a patient with AMD - notice the blurring of central vision
AMD may see the horizontal lines wavy

Age-related macular degeneration is the leading cause of blindness in the elderly population in developed countries. There are two types of ARMD:

Dry ARMD

It comprises 90% of AMD cases, and is characterized by drusen, pigmentation and sometimes hemorrhages at macula. The progression is slow.

Drusen (singular, ‘druse’) are tiny yellow or white accumulation of extracellular material that build up between Bruch’s membrane (a layer of retina) and the retinal pigment
epithelium. The presence of a few small drusen is normal with advancing age, and most people over 40 years have some hard drusen. The presence of large and multiple drusen in
the macula is pathologic and a common early finding in AMD.

Wet ARMD

Wet ARMD occurs when abnormal vessels grow from the choroid into the neurosensory retina and leak at macula. It is associated with rapid deterioration in vision and visual
distortion.

References

• Medscape - Nonexudative (Dry) Age-Related Macular Degeneration (AMD)

• Medscape - Exudative (Wet) Age-Related Macular Degeneration (AMD)

Last updated:
Time spent: QID:315
2023-2-12

341 of 1943
A 50-year-old man presents to you complaining of difficulty in reading books and seeing in dim light. His problem started six months ago and has worsened progressively. His sight
improves when he look through a 1 mm pinhole. Which one of the following would be the most likely diagnosis?

A. Myopia.

B. Presbyopia.

C. Hypermetropia.

D. Cataract.

E. ​Chronic simple glaucoma.

Incorrect. Correct answer is B


45% answered correctly

Explanation:

Correct Answer Is B

The history and the age are suggestive of presbyopia. Presbyopia is a Greek word meaning ‘the eye of an old man’. This condition is almost always seen after age 40 and is
completed at 60. With age, the lens becomes stiff and less easy to accommodate. The ability to focus on near objects continuously declines from 50 mm in a child to 100 mm at the
age of 25 and eventually to only 1-2 meters at 60 years of age.

Presbyopia presents with the following:

Eye strain
Difficulty seeing in dim light
Problems in focusing on small objects and/or fine prints

(Option A) A person with myopia has not difficulty seeing near objects, but vision for distant objects is diminished.

(Option C) Although persons with hypermetropia have the same presentation as this clinical scenario, the late age of onset is more suggestive of presbyopia. Hypermetropia usually
occurs at younger ages.

(Option D) Patients with bilateral cataract have difficulty in seeing both far and near objects. Furthermore, they tend to be visually impaired in bright rather than dim light.

(Option E) Chronic simple glaucoma presents differently with impaired peripheral visual fields initially. The vision may be completely lost as the disease progresses continuously.
The following photograph shows characteristic visual field impairment in chronic simple glaucoma. A similar pattern would be seen in those with retinitis pigmentosa and hysteria.

Vision through the eye of a patient with chronic open-angle glaucoma

References

• Medscape - Presbyopia - Cause and Treatment

Last updated:
Time spent: QID:292
2023-2-12

342 of 1943
A 27-year-old woman presents to your practice with sudden onset of pain, redness and tearing of her right eye as illustrated in the following photograph. Further inquiry revleas that
she also has had pain and stiffness of her lower back for the past 1 year that she attributes to bad sitting at work. Her brother has similar back pain. On examination, she is found to
be photophobic. Limited ability to bend forward is the other significant finding. Which one of the following would be the next best step in management?

A. Anti-double stranded DNA antibody.

B. HLA-B27.

C. X-ray of the lumbosacral spine.

D. Anti-nuclear antibody (ANA).

E. Check ESR and CRP.

Incorrect. Correct answer is C


45% answered correctly

Explanation:

Correct Answer Is C

The photograph shows an eye with 360-degree perilimbal congestion that is more intense at limbus. There is also an iris with irregular contour. These findings, along with symptoms
of acute onset of pain, redness, photophobia and tearing, establish the diagnosis of anterior uveitis with high certainty.

Anterior uveitis is caused by several conditions such as trauma, seronegative spondyloarthropathies (i.e. ankylosing spondylitis, reactive arthritis, psoriatic arthritis and
inflammatory bowel disease), idiopathic juvenile arthritis, and very rarely infections such as herpes, syphilis and tuberculosis. In 50% of cases, no clear cause is found (idiopathic).

The presence of low back pain and stiffness and the family history of similar condition in this patient’s brother make ankylosing spondylitis (AS) the most likely underlying etiology
of the anterior uveitis. When AS is suspected, X-ray of the lumbosacral spine is always the next best step in management

The findings on X-ray include the following:

Sacroiliitis evident by haziness of the sacroiliac joint (the earliest finding)


Fusion of sacroiliac joint (a late finding)
Bamboo spine and squaring of the vertebral bodies (a late finding)

(Options A and D) As the name seronegative implies, serologic tests such as ANA, anti-double stranded DNA antibody, rheumatoid factor (RF), etc. are negative in seronegative
arhtropathies.

(Option B) ​Although HLA-B27 is positive in 90% of patients with AS, it is not used as a diagnostic tool because it is also positive in 10% of people without AS.

(Option E) Seronegative arhtropathies are inflammatory; therefore, ESR and CRP are expected to be elevated. However, normal values do not exclude AS. On the other hand, positive
levels can be found in several other conditions and are not diagnostic for AS or other seronegative arhtropathies.

References

• RACGP - AFP - Ankylosing spondylitis

• Medscape - Ankylosing Spondylitis and Undifferentiated Spondyloarthropathy

• Medscape - HLA-B27 Syndromes

Last updated:
Time spent: QID:558
2023-2-12

343 of 1943
A 40-year-old man presents to your practice with a mass in front of his left ear that he noted three months ago. On examination, the mass is painless, firm and mobile. When the
patient is asked to smile, the left corner of his mouth does not elevate. Furthermore, trickling of saliva from the same side is noted. Which one of the following is the most
appropriate next step in management?

A. CT scan of the head and neck.

B. CT scan of the chest.

C. Fine needle aspiration (FNA) biopsy.

D. Chest X-ray.

E. CT scan of the abdomen.

Correct
45% answered correctly

Explanation:

Correct Answer Is A

The examination is remarkable firm and painless lump in front of the ear where the parotid gland lies. There is also neurologic findings associated with facial nerve dysfunction on
the same side.These findings make a malignant tumor of the parotid gland the most likley diagnosis.

When a salivary gland tumor is suspected, investigation starts with imaging studies. CT/MRI of the head and neck is best for imaging of parotid gland. CT/MRI can differentiate
neoplastic from benign disease, define intra- versus extra-glandular location, assess local extension and invasion, and detect nodal and systemic metastases.

An exception to this recommendation is the tumors of the submandibular salivary gland and parotid gland tumors within the superficial lobe. Of note, tumors limited to the
superficial lobe of the parotid gland usually do not invade the facial nerve.

In this patient, who had evidence of facial nerve involvement, CT scan or MRI or the head and neck is the most appropriate next step to take.

(Option C) FNA biopsy (or core biopsy) is the definitive means of diagnosing a parotid malignancy. Biopsy is performed after imaging studies, usually before surgical treatment.

(Options B, D and E) In addition to local spread, salivary gland tumors can metastasize to distant sites. The most common of such site are lungs, liver, and bones. Chest X-ray and
CT scan of the chest and abdomen may be used later for further assessment and staging.

References

• Western Australia - Imaging Pathways

• UpToDate - Salivary gland tumors: Epidemiology, diagnosis, evaluation, and staging

• UpToDate - Malignant salivary gland tumors: Treatment of recurrent and metastatic disease

Last updated:
Time spent: QID:702
2023-2-12

344 of 1943
A 75-year-old woman presents with a 16-hour history of a painful tender swelling under the right side of the jaw. The accompanying photograph shows the intraoral appearance. On
examination, the swelling is exquisitely tender to palpation. Which one of the following would be the appropriate investigation to confirm the diagnosis?

A. Ultrasonography of the submandibular area.

B. Intraoral plain X-ray.

C. CT scan of the head and neck.

D. MRI of the head and neck.

E. Sialogram of the submandibular salivary duct.

Incorrect. Correct answer is B


45% answered correctly

Explanation:

Correct Answer Is B

The unilateral erythema and the tender swelling shown in the photograph are characteristic of acute sialadenitis in the setting of salivary duct calculi.

Salivary duct calculi are composed predominantly of calcium salts, and form due to stasis and ductal inflammation. More than 80% of salivary duct stones occur in the
submandibular gland due to its long and torturous course. Another reason that most salivary stones form in the in submandibular glands is the mixed mucous and serous
composition of the saliva in this gland compared to other salivary glands.

For submandibular calculi, plain X-ray is the most useful intial study, as 80% are radio-opaque. In contrast, most parotid duct calculi are radiolucent. It is important to note that X-ray
is considered the choice if bases on clinical findings, stones are highly suspected. Otherwise, CT scan or ultrasound are used for initial assessment.

(Option A) Ultrasound is increasingly used, as it is able to give information about the gland, the presence of duct dilation and can identify stones as small as 1mm; however, it is
difficult to perform in submandibular area, especially where the lesion is away from the mandible. It is best used for parotid gland problems.

(Option C) CT is complementary and can provide superior images in complex cases or in patients with recurrent disease. However, in some patients artefacts from dental implants
may limit the usefulness of CT scan. In most cases, CT scan and MRI are unnecessary.

(Option D) MRI is not routinely used for diagnosis of sialadenitis and sialolethiasis.

(Option E) A Sialogram is a useful study for delineating the exact size and location of stones in a salivary gland duct. The stone will be visualized as a filling defect within the duct.
Active infection of salivary gland is a contraindication for sialography because it can exacerbate the extent of the infection. While other means such as X-ray or CT-scan can be used,
sialograms are less frequently used these days.

References

• http://www.australiandoctor.com.au/cmspages/getfil

• AMC Handbook of Multiple Choice Questions – pages 106-107

Last updated:
Time spent: QID:735
2023-2-12

345 of 1943
A 42-year-old man presents to your office with a painful swelling under the jaw, which becomes more painful and prominent after eating. Bimanual examination reveals a slightly-
tender mass in the region. Which one of the following is the most appropriate next step in management?

A. Intraoral X-ray of the submandibular area.

B. Sialogram.

C. CT scan of the submandibular area.

D. Ultrasonography.

E. OPG.

Correct
45% answered correctly

Explanation:

Correct Answer Is A

The scenario is highly suggestive of chronic sialadenitis. Patients with chronic sialadenitis experience recurrent pain and swelling of the affected salivary gland. This is most
commonly due to duct obstruction, with salivary duct calculi being the most frequent cause. Patients with sialolithiasis often complain of postprandial pain and swelling and may
have a history of prior episodes of acute suppurative sialadenitis.

On palpation, the gland is often enlarged, may be tender, and minimal saliva will be expressed from the obstructed duct. Large salivary-duct stones will usually be palpable with
bimanual examination. If calculi are palpable in the floor of the mouth, further examination of the mouth and gland massage should be avoided, as this may lead to the pushing of
the stone more proximally into the duct and decrease the likelihood of a successful transoral excision.

Salivary duct calculi are composed predominantly of calcium salts, and form due to stasis and ductal inflammation. More than 80% of salivary duct stones occur in the
submandibular gland due to the long and torturous nature of the duct, as well as the mixed mucous and serous composition of the saliva (in comparison to the serous saliva from
the parotid gland).

Confirmatory imaging can include plain X-ray, ultrasound or CT scan. For submandibular stones, intraoral plain X-ray is the most useful study, as 80% are radio-opaque. In contrast,
most parotid duct calculi are radiolucent.

(Option B) A sialogram is a useful study for delineating the exact size and location of stones in a salivary gland duct. The stone will be visualized as a filling defect within the duct.
Active infection of salivary gland is a contraindication for sialography because it can exacerbate the extent of the infection. While other means such as X-ray or CT-scan can be used,
sialograms are less frequently used.

(Option C) CT is complementary and can provide superior images in complex cases or in patients with recurrent disease. However, in some patients artefacts from dental implants
may limit the usefulness of CT scan.

(Option D) Ultrasound is increasingly used, as it can provide information about the gland, the presence of duct dilation and identify stones as small as 1mm; however, it is difficult to
perform in submandibular area specially where the lesion is away from the mandible. It is best used for parotid gland problems.

(Option E) OPG (orthopantomograph) is panoramic dental X-ray of upper and lower jaws. OPG has no role in diagnosis of salivary gland diseases.

NOTE - If a plain X-ray does not reveal a stone as the underlying etiology, other modalities such as CT scan, ultrasound, MRI, or MRI sialogram are used for furtehr assessment.

References

• AMC Handbook of Multiple Choice Questions – pages 106-107

• Medscape - Submandibular Sialadenitis/Sialadenosis

Last updated:
Time spent: QID:736
2023-2-12

346 of 1943
A 47-year-old man presents with a mass in his neck that he noticed 6 months ago. On examination, the mass is painless, firm and mobile. A fine needle aspiration is performed with
inconclusive result. Which one of the following would be the next best step in management?

A. Chest X-ray.

B. CT scan of the neck.

C. CT scan of the chest.

D. Excisional biopsy of the mass.

E. Ultrasonography.

Incorrect. Correct answer is B


45% answered correctly

Explanation:

Correct Answer Is B

Painless lateral neck lumps in adults are malignant until proven otherwise. A diagnosis can be readily made in the vast majority of cases through careful history-taking and a
thorough clinical examination. Fine-needle aspiration biopsy (FNAB) and CT scan should be considered as the most useful investigations before referral to a specialist center.

FNA will often establish the diagnosis; however, occasionally the results are nonspecific and non-diagnostic. In these cases, excisional biopsy is required after CT scan of the neck is
performed. CT scan is very helpful in investigating neck lumps because it demonstrates relational anatomy in addition to pathological and benign lesions. The use of intravenous
contrast enhances diagnostic capability. CT scan without contrast is of limited usefulness. Spiral or helical CT scanning allows faster, higher-quality image acquisition with less
radiation exposure. CT scan may also assist FNA by accurately localizing the lesion for sampling and avoiding damage to vital structures during the sampling procedure.

(Options A and C) Chest X-ray and CT scan of the chest may be required later as a part of workup if the swelling is suspected to be a metastatic lesion due a primary cancer within
chest.

(Option D) Excisional biopsy is performed for definite diagnosis after CT scanning as mentioned earlier.

(Option E) Although ultrasound is appropriate for differentiation between solid and cystic masses, it contributes little to the diagnosis of neck lumps. It is very useful in evaluating
the thyroid gland to determine whether or not a nodule is solitary or a part of a multinodular goiter. Ultrasound is also useful during FNA when a neck lump is small or difficult to
localize by palpation. Its benefits include ease of access, low cost and not exposing the patient to ionizing radiation.

References

• AAFP - The Adult Neck Mass

Last updated:
Time spent: QID:737
2023-2-12

347 of 1943
A 42-year-old man presents to the emergency department with floaters and flashes in his right eye. Ophthalmologic examination reveals a detached retina as the cause. Which one
of the following is the most appropriate initial management?

A. Atropine drop.

B. Pilocarpine drop.

C. Acetazolamide.

D. Intravenous prednisolone.

E. Nursing the eye with head-up position.

Incorrect. Correct answer is E


45% answered correctly

Explanation:

Correct Answer Is E

Detachment of the neurosensory retina may occur spontaneously or in the setting of trauma. The most common form is due to a tear or break in the retina. Patients may describe
sudden onset of new floaters or black dots in their vision, often accompanied by flashes of light (photopsias). In its early stages, a detachment may present as a persistent missing
portion of the monocular visual field. Once the macula (central retina) becomes involved, visual acuity will be severely compromised.

Retinal detachment is not painful and does not cause a red eye. There may be a dulling of the red reflex, and ophthalmoscopic examination may reveal the retina to be elevated with
folds. If the detachment is extensive, there may be a relatively abnormal pupilary reflex caused by affected afferent limb of the reflex.

Early referral to ophthalmologist for dilated retinal examination is mandatory. Patients should be advised to tilt their head in an attempt to prevent the progression of the
detachment, as well as to promote the chance of the detached retina fall back into place.

Reattachment of the retina with surgery, cryotherapy, or by injecting an expansile gas are definitive management options. If these fail, a band is placed around the eye to
approximate the retina close to the sclera.

References

• RACGP - Flashes and floaters: A practical approach to assessment and management

• Master the Boards STEP 3 (USML) – Conrad Fischer - Sonia Richert – Kaplan Medical Publications – Pages 501-502

Last updated:
Time spent: QID:807
2023-2-12

348 of 1943
A 40-year-old man with long history of alcohol consumption and smoking presents with a 2-cm painless lump in his left tonsil. Which one of the following is the most likely
diagnosis?

A. Squamous cell carcinoma (SCC).

B. Nasopharyngeal cancer.

C. Carotid artery aneurysm.

D. Metastasis.

E. Non-Hodgkin lymphoma (NHL).

Correct
45% answered correctly

Explanation:

Correct Answer Is A

Presence of alcohol and smoking in the history is highly suggestive of SCC. Smoking and alcohol are the strongest contributing factors to SCCs of the head and neck.

(Option B) Nasopharyngeal carcinomas are different from SCCs of the head and neck. Epstein-Barr virus (EBV) infection is the most common risk factor. Other risk factors include
Human Papilloma Virus (HPV) and genetic predisposition.

(Option C) Carotid artery aneurysm does not present with a lump in the tonsil.

(Option D) Although metastasis can be among differential diagnoses, given the history of alcohol and smoking, SCC is more likely as the underlying cause of this presentation.

(Option E) Although lymphoma is the second most common cause of a tonsillar lump in adults, a single lump in tonsil is a very unusual presentation for NHL.

References

• UpToDate - Epidemiology, etiology, and diagnosis of nasopharyngeal carcinoma

Last updated:
Time spent: QID:833
2023-2-12

349 of 1943
The parents of a 6-week-old baby has brought her to your clinic after they noticed a swelling on the right side of her neck. She is otherwise healthy, is being breastfed, and is gaining
weight appropriately. Which one of the following should not be considered in the differential diagnosis of the lateral cervical swelling in this child?

A. Acute cervical adenitis.

B. Branchial cleft cyst.

C. Cystic hygroma.

D. Sternocleidomastoid muscle hematoma.

E. Thyroglossal duct cyst.

Incorrect. Correct answer is E


45% answered correctly

Explanation:

Correct Answer Is E

Of the given options, thyroglossal duct cyst is invariably in the midline (or just slightly off of it) and not seen on the lateral aspect of the neck.

(Option A) Cervical adenitis (lymphadenitis) is defined as enlarged, inflamed, and tender lymph node(s) of the neck. Cervical lymph nodes are distributed along the course of the
sternocleidomastoid muscle, in the posterior neck, supraclavicular, or submandibular region.

(Option B) A branchial cleft cyst is a congenital epithelia cyst that arises on the lateral part of the neck due to failure of obliteration of the second branchial cleft (or failure of fusion
of the second and third branchial arches) during embryonic development. A branchial cyst can also be among differential diagnoses of a lateral neck swelling in children.

(Option C) Cystic hygroma, also known as cystic lymphangioma or macrocystic lymphatic malformation, is a congenital multiloculated lymphatic lesion that can arise anywhere, but
is classically found in the left posterior triangle of the neck (especially left side) and axillary regions. It can cause a swelling observed in the lateral side of the neck.

(Option D) Hematomas of the sternocleidomastoid present with lateral swellings along the course of the muscle.

References

• AAFP - Evaluation and Management of Neck Masses in Children

Last updated:
Time spent: QID:936
2023-2-12

350 of 1943
A 52-year-old man presents to the emergency department with sudden-onset right-sided vision loss. His medical history is remarkable for right-sided headache of 2 weeks duration,
which is exacerbated by chewing. Fundoscopic exam is performed with the findings shown in the following photograph. Which one of the following is the most likely cause of the
visual loss?

A. Central retinal artery occlusion (CRAO).

B. Branch retinal artery occlusion (BRAO).

C. Central retinal vein occlusion (CRVO).

D. Branch retinal vein occlusion (BRCO).

E. Circumciliary vein occlusion.

Correct
45% answered correctly

Explanation:

Correct Answer Is A

The photograph shows a pale retina and a red macula. This fundoscopic picture is most consistent with CRAO.

A majority of patients with CRAO develop unilateral sudden-onset and significant vision loss. The vision loss is often painless. CRAO almost never occurs in both eyes
simultaneously. Occasionally, CRAO is preceded by transient monocular blindness ,or there is a stuttering or fluctuating course. The vision loss is severe, and most affected patients
can see only hand motions and rarely can count fingers.

On fundoscopic examination, ischemic retinal whitening is seen immediately after an occlusion of the central retinal artery. A "cherry red spot" appears in the macula, where the
retina is thinner and the retinal pigment epithelium and choroidal vasculature can be seen more easily as the ischemic retina becomes less translucent.

Common causes of CRAO include:

Atherosclerotic changes of the carotid artery


Emboli from the heart
Atheroma within the central retinal artery
Hematologic diseases such as polycythemia, sickle cell disease, leukemia and lymphoma
Inflammatory processes such as giant cell arteritis, Wegener’s granulomatosis, polyarteritis nodosa, lupus, etc.

With history of right-sided headaches aggravated by chewing, and now sudden-onset loss of vision, giant cell arteritis (GCA) is most likely to have caused this visual loss.

GCA is a chronic vasculitis of large- and medium-sized arteries, occurring among people over the age of 50 years. It is more common in women than men. The vascular involvement
can be generalized; however, cranial branches of the arteries branching from the aortic arch are most prominently involved.

GCA can presents with headache, jaw claudication, temporal region tenderness, weak pulses of the temporal artery, or jaw claudication. Arm claudication may occur in a minority of
patients as a result of subclavian or axillary arteries involvement. Polymyalgia rheumatica can be an association.

Visual symptoms in GCA are common and if not managed promptly, may have devastating outcomes such as permanent loss of vision.

Visual manifestations of GCA is due to ischemia of central or branch retina artery occlusion and ischemia, ischemia of choroid, retina or a combination of these conditions. In many
patients amaurosis fugax (transient monocular loss of vision) precedes acute vision loss.

Despite effective therapy, visual loss remains common in GCA and permanent partial or complete loss of vision in one or both eyes occurs in 15-20% of cases. This is due to the fact
that visual loss is the presenting feature in many patients who could have been treated if diagnosed earlier based on other presenting symptoms.

(Option B) BRAO presents differently on fundoscopic exam with whitening and paleness of the retina only and in segments that are supplied by the occluded branch of the retinal
artery.

351 of 1943
(Options C and D) Fundus examination in CRVO and BRVO reveals retinal hemorrhage, edema and dilated retinal veins. This pattern is generalized in CRVO and focal in BRVO.

(Option E) Circumciliary vein occlusion is seen in glaucoma. Glaucoma has a different fundoscopic exam appearance.

References

• UpToDate - Pathogenesis of giant cell arteritis

• Medscape - Giant Cell Arteritis (Temporal Arteritis)

• UpToDate - Clinical manifestations of giant cell temporal arteritis

• RACGP - Polymyalgia rheumatica and giant cell arteritis

Last updated:
Time spent: QID:1137
2023-2-12

352 of 1943
Which one of the following is the most appropriate management option for the eye abnormality shown in the accompanying photograph?

A. CT scan of the head and the orbit.

B. Warm compress.

C. Anesthetic eye drops.

D. Antibiotic eye drops.

E. Corticosteroid eye drops.

Incorrect. Correct answer is B


45% answered correctly

Explanation:

Correct Answer Is B

The photograph shows a diffuse beefy red discoloration of the sclera not crossing the limbus (the border or the cornea and sclera). This is typical for subconjunctival hemorrhage.
Subconjunctival hemorrhage can be caused by trauma to the eye, a sudden increase in intrathoracic pressure such as during sneezing or coughing, anti-coagulation disorders or
medications, or spontaneously (idiopathic). The condition is usually asymptomatic, unless caused by trauma, in which case pain can be a feature. In non-traumatic cases, mild
irritation can be feature at occasions.

NOTE – Subconjunctival hemorrhage is directly related to hypertension but measuring the blood pressure to reassure the patient is worth doing.

In cases of subconjunctival hemorrhage with no history of trauma, no active treatment is required. Artificial tears can be applied if irritation is a complaint. It is important to note that
use of aspirin and NSAIDs should be avoided due to increased risk of rebleeding. Like a bruise, the hemorrhage may become green or yellow with time. The discoloration usually
disappears within 2 weeks. Application of warm compress, after cold compress for the first 24 to 48 hours, several times a day, can facilitate the process.

Patients should be advised to return if the bruise-like appearance does not resolve completely, if pain ensues, or if there is rebleeding.

CT scan of the orbit may be indicated in patients with subconjunctival hemorrhage following trauma to exclude orbit fractures or other possible associated injuries.

References

• Eye Emergency Manual

• Medscape - Red Eye

Last updated:
Time spent: QID:1241
2023-2-12

353 of 1943
A 67-year-old man presents to your practice complaining of difficulty seeing in daylight. He explains that when he is out in the daytime, the light is dazzling. He, however, does not
have any problem in reading or watching TV. He has the history of type II diabetes mellitus and hypertension, for which he is on metformin and enalapril, respectively. Which one of
the following could be the most likely diagnosis?

A. Cataract.

B. Chronic simple glaucoma.

C. Hypertensive retinopathy.

D. Presbyopia.

E. Retinal detachment.

Correct
45% answered correctly

Explanation:

Correct Answer Is A

The presentation is most consistent with diagnosis of cataract. Cataract is a term used for any lens opacity. The prevalence of cataract increases with age. Sixty five percent of
those aged 50 to 59 years and all people older than 80 years will have lens opacities.

Cataract presents with gradual loss of vision with preserved direct pupillary light reflex. Unilateral cataract may remain unnoticed, but the mono-ocular impaired vision may affect
distance judgment due to stereopsis. Bilateral cataracts lead to gradual vision loss that can cause frequent change of glasses.

Dazzling, especially in sunlight is seen initially. With bilateral cataract difficulty in reading, recognizing faces, watching TV and driving, particularly at night, become the more
prominent symptoms.

Appearance of the lens in a patient with cataract

(Option B) Chronic glaucoma presents with gradual loss of vision often starting with blurring in the peripheiry of the visual field, progressing to invlove the central vision. Dazzling is
not a problem.

(Option C) Hypertensive retinopathy leads to vision loss and retinal hemorrhage, but dazzling is an unusual feature.

(Option D) Presbyopia is associated with ageing and is characterized by the progressively diminishing ability to focus clearly on an object close to the eye. The first symptom of
presbyopia is eyestrain. Others incluse difficulty seeing in dim light, problems focusing on small objects and fine print that are usually noticed between the ages of 40 and 50. There
is no dazzling.

(Option E) Retinal detachment is associated with floaters and flashes, but no dazzling.

References

• Health Department - Cataract

• Medscape - Senile Cataract (Age-Related Cataract)

Last updated:
Time spent: QID:293
2023-2-12

354 of 1943
Three days after a successful lens replacement in a 67-year-old man due to the cataract of his right eye, he wakes up in the morning with a painful red eye and photophobia. On
examination, the affected eye has decreased and blurry vision. Funduscopic exam is normal on both sides. The picture of his eye is illustrated in the following photograph. Which
one of the following is the most likely diagnosis?

A. Hypopyon.

B. Conjunctivitis.

C. Acute glaucoma.

D. Uveitis.

E. Suture infection and abscess.

Incorrect. Correct answer is D


45% answered correctly

Explanation:

Correct Answer Is D

The photograph shows a red eye with constricted and more importantly a deposit of white matter in the anterior chamber. These alongside the eye pain, photophobia, and blurred
vision, make anterior uveitis the most likely diagnosis.

The uvea is the pigmented part of the eye including iris, ciliary body, and choroid. The iris and ciliary body are called the anterior uvea. Iris inflammation invariably involves the ciliary
body too; so the inflammation is best referred to as anterior uveitis rather than iritis. Anterior uveitis is idiopathic in 50% of cases. The remaining half are caused by an underlying
condition such as seronegative spondyloarthropathies, Behçet disease, Lyme disease, tuberculosis, sarcoidosis, herpes simplex and herpes zoster infections, juvenile rheumatic
arthritis disease, or trauma.

Clinical features suggestive of anterior uveitis include the following:

Red eye: the redness in anterior uveitis is circumcorneal, meaning that there is 360° injection around the limbus. The injection has the highest intensity in the perilimbus
and decreases towards the periphery. The injection in conjunctivitis is reverse.
Pain
Blurred vision
Photophobia (both direct and consensual) on examination
Constricted pupil – there might be pupil irregularities due to adhesion of the iris to the anterior aspect of the lens
Lacrimation

Anterior uveitis can also be a complication of cataract or other eye surgeries. Currently, phacoemulsification is the standard procedure for lens replacement in people with cataract.
This procedure is associated with a number of complications and anterior uveitis being one of them. The eye illustrated in the photograph has collection of exudate material at the
bottom of the anterior chamber due to gravity, namely hypopyon, a feature seen in anterior uveitis.

(Option A) Hypopyon is accumulation of leukocytic exudate in the anterior chamber. The exudate settles at the dependent aspect of the eye due to gravity. Although this patient has
hypopyon in the affected eye, this is a sign seen in anterior uveitis not a diagnosis.

(Option B) Conjunctivitis presents with conjunctival redness giving rise to a red eye; however, the condition is not a cause of blurred vision, pupil abnormalities. Superficial
photophobia can be a feature though.

(Option C) Acute closed-angle glaucoma presents with acute eye pain and ipsilateral headache, nauseas and vomiting, hazy cornea and an irregular dilated pupil. The orbit is hard to
palpation. This patient does not have clinical features of acute glaucoma.

(Option E) Suture infection, if occurs, is most likely to cause endophthalmitis (the infection of the entire orbit including both the anterior and posterior chambers) with a different
presentation especially on fundoscopic exam.

NOTE – Patients with anterior uveitis need urgent referral for review by an ophthalmologist and follow up to prevent damage from prolonged inflammation including disruption
to aqueous flow inside the eye that can result in glaucoma and adhesions between the iris and lens. Treatment incudes topical corticosteroids such as prednisolone eye drop

355 of 1943
that significantly reduces the inflammation, and subsequently, the pain and redness. Cyclopentolate drop is often considered as well to keep the pupil dilated and prevent from
lens-iris adhesions.

References

• Wiley Online Library - Complications of cataract surgery

Last updated:
Time spent: QID:1242
2023-2-12

356 of 1943
A 48-year-old woman presents to your practice with complaints of a painful red eye and decreased vision that has started and progressed since 3 days ago. On examination, the left
eye is red, but the cornea is normal in appearance with preserved light and red reflexes. Vision is 4/12 on the left and 10/12 on the right side. The orbit movement are preserved and
painless. Which one of the following could be the most likely diagnosis?

A. Glaucoma.

B. Conjunctivitis.

C. Blepharitis.

D. Uveitis.

E. Keratitis.

Incorrect. Correct answer is D


45% answered correctly

Explanation:

Correct Answer Is D

A red eye is a finding in many ophthalmological diseases such as those of the conjunctiva (e.g. conjunctivitis), cornea (e.g. corneal ulcers), inflammation of the eye chambers (e.g.
anterior uveitis), or episclera and sclera. Each condition, however, often has specific features with which the differential diagnoses could be narrowed down. The cornea is normal in
this patient and the red reflex is intact. These findings make acute closed-angle glaucoma (option A) an unlikely diagnosis. In patients with acute closed-angle glaucoma, the cornea
is hazy and the pupil is partially or fully dilated and unresponsive to light. The red reflex is lost. Other accompanying features include hardness of the orbit, ipsilateral headache and
nausea and vomiting. Chronic closed-angle glaucoma does not cause a red eye.

There is also no clear or purulent discharge in the history. These make conjunctivitis (bacterial, viral or allergic) (option B) a less likely diagnosis as well. In conjunctivitis, the eye is
red but not painful. The vision is preserved and there is no corneal or pupillary abnormalities.

Blepharitis (option C) is the inflammation of the eyelids. The condition is associated with inflamed and often itchy eyelids. If there is any abnormal eye finding, it will be related to
concomitant conjunctivitis.

Keratitis (option E) is the inflammation of the cornea due to infection (e.g. herpes simplex, herpes zoster) or other inflammatory processes. There are corneal abnormalities such as
circumcorneal dendritic ulceration in herpes infection. Features of keratitis include redness, pain and a sense of foreign body or grittiness, photophobia, and lacrimation. The vision
and pupillary reflexes are normal.

In fact, in this patient with such constellation of signs and symptom, anterior uveitis is the most likely diagnosis. The uvea is the pigmented part of the eye including iris, ciliary body
and choroid. The iris and ciliary body are called the anterior uvea. Iris inflammation invariably involves the ciliary body too, so the inflammation is best referred to as anterior uveitis
rather than iritis. Anterior uveitis is idiopathic in 50% of cases. The remaining half are caused by an underlying condition such as seronegative spondyloarthropathies, Behçet
disease, Lyme disease, tuberculosis, sarcoidosis, herpes simplex and herpes zoster infections, juvenile rheumatic arthritis disease, or trauma.

Acute anterior uveitis presents with eye pain and redness. Eye pain generally develops over a course of few hours or days unless the cause is trauma, in which case it develops
almost immediately. There is 360º perilimbal injection, which increases in intensity as it approaches the limbus. This pattern is different from the redness in conjunctivitis, in which
the injection decreases as it approaches the perilimbal area. Visual acuity may or may not be decreased in the affected eye. On pupillary examination, the patient may experience
direct photophobia when the light is shone into the affected eye, or consensual photophobia when the light is directed into the unaffected eye. Consensual photophobia is very
helpful in distinguishing the photophobia due to iritis from superficial photophobia such as seen in conjunctivitis. Hypopyon, the white cells precipitate in the anterior chamber, is
another feature that may be seen in anterior uveitis. Pupil may be irregular due to adhesion of the iris to the anterior aspect of the lens.

Chronic anterior uveitis presents primarily with blurred vision, mild redness, and no or little pain or photophobia except during an acute exacerbation.

References

• Medscape - Iritis and uveitis

Last updated:
Time spent: QID:1243
2023-2-12

357 of 1943
A 26-year-old woman presents with red right eye since this morning. For the past few hours she also has had mild discomfort and irritation of the eye. She denies any discharge or
itchiness. On examination, eye movements, orbital pressure and visual acuity are normal. Cornea shows no abnormality and has preserved light and red reflexes. A funduscopic
exam reveals no retinal abnormality either. She has unaffected visual acuity. There is a patchy redness of the sclera as shown in the following photograph. Which one of the
following would be the most appropriate eye drop to consider for her?

A. Ketotifen.

B. Hypomellose.

C. Sodium cromoglycate.

D. Prednisolone.

E. Chloramphenicol.

Incorrect. Correct answer is B


45% answered correctly

Explanation:

Correct Answer Is B

The clinical presentation, as well as the appearance of the eye (a patchy redness) makes epicscleritis the most likely diagnosis.

Epicscleritis is the inflammation of the episclera, the tissue that lies between the conjunctiva and the sclera. The condition is usually mild and self-limiting but often recurrent. Two-
thirds of the cases are idiopathic and the remaining one-third can have an underlying systemic condition such as infections (tuberculosis, syphilis, Lyme disease or cat scratch
disease) or a rheumatologic disorder (e.g. rheumatoid arthritis, polyarteritis nodosa, seronegative spondyloarthropathies, etc.). In patients with epicscleritis, visual acuity and fields
are unaffected. Mild photophobia and lacrimation might exist; however, these are not significant findings in most patients.

The initial management usually starts with artificial lubricants (tear) such as Hypomellose or Artelac 4 to 6 times a day. For more discomfort and irritation not responsive to this
measure, topical NSAIDs such as diclofenac would be the next line of treatment. Topical corticosteroids are reserved for those not responding to the above. Over 80% of the patients
will respond to topical corticosteroids.

(Options A and C) There is no itchiness or discharge as pointers towards allergic conjunctivitis to justify the application of drugs used for treatment of the condition. Ketotifen is an
antihistamine and sodium cromoglycate a mast cell inhibitor. These drugs have no role in management of epicscleritis.

(Option D) Prednisolone can be a choice if the condition does not show a satisfactory response to artificial tear drops and topical NSAIDs.

(Option E) Chloramphenicol is an antibiotic used for treatment of bacterial conjunctivitis. The absence of purulent discharge and/or sticky eyelids, especially on waking up, excludes
bacterial conjunctivitis.

NOTE – A distinction between epicscleritis and scleritis is very important because while the former is a mild self-limiting condition, the latter can result in serious damage to the eye
and sight. Unlike epicscleritis, scleritis is associated with severe penetrating pain in the eye that radiates to the jaw, eyebrows, forehead or sinuses. Eyeball tenderness is also
characteristic of scleritis and absent in epicscleritis. Scleritis is almost always associated with an underlying systemic disease. In scleritis, redness increases gradually over a
course of several days. There is a bluish red tinge which is best seen in the natural light rather than through the slit lamp. The discoloration can be focal or involve the entire sclera.
The interpalpebral area is the most common site. The discoloration does not blanch with application of sympathomimetic dilating drugs.

References

• Medscape - Episcleritis

• Medscape - Scleritis

Last updated:
Time spent: QID:1244
2023-2-12

358 of 1943
An 80-year-old woman presents to your GP clinic with 'funny spots' affecting her right vision. Over the past week, she has noticed a number of flashes and floaters in the visual field
of the right eye. On physical examination, visual acuity and fields are normal. Which one of the following could be the most likely diagnosis?

A. Vitreous hemorrhage.

B. Retinal detachment.

C. Depression.

D. Optic neuritis.

E. Posterior vitreous detachment.

Incorrect. Correct answer is E


45% answered correctly

Explanation:

Correct Answer Is E

Flashes (photopsias) refer to the perception of light in the absence of external light stimuli. Flashes are typically described as a momentary arc of white light, similar to a bolt of
lightning or a camera flash. They are more noticeable in dim lighting, may be triggered by eye movement and are usually in the temporal visual field. Flashes can be generated
anywhere along the visual pathway. In the eye, flashes are the result of mechanical stimulation of the retina by vitreoretinal traction.

Floaters refer to the sensation of dark spots. These spots are caused either by opacities in the vitreous, which cast shadows on the retina, or by light bending at the junction
between fluid pockets and the vitreous. Floaters may be caused by vitreous debris from infection, inflammation and hemorrhage, but are typically due to the age-related
degeneration of the vitreous that forms condensations of collagen fibers. Floaters are often described by patients as ‘flies’, ‘cobwebs’ or ‘worms’ that are more pronounced against
light backgrounds. Floaters may also result from hemorrhage of retinal vessels into the vitreous and may be described as small black or red spots.

NOTE - The most common cause of vitreous hemorrhage is proliferative diabetic retinopathy.

The most common cause of flashes and floaters in elderly patients is posterior vitreous detachment (PVD). PVD is present in approximately 66% of patients over the age of 70
years. PVD is age-related vitreous degeneration, shrinkage and separation from the retina. During separation, the vitreous may tug and cause mechanical stimulation of the retina
that will results in flashes. Patients with PVD often have normal vision and visual fields otherwise. There is no afferent pupillary defect, and patients have normal pupillary light
reflex.

NOTE - Patient with PVD are be at higher risk of retinal detachment if PVD is associated with vitreous hemorrhage. Near 70% of these patients have been found to have at least one
retinal tear.

(Option A) Vitreous hemorrhage is mainly associated with floaters. Flashes are less common. On the other hand, compared with PVD, vitreous hemorrhage is less common.

(Option B) Retinal detachment is also associated with flashes and floaters, especially of sudden onset. Compared with PVD, retinal detachment is a rare condition (1 in 10,000
people per year). Furthermore, patients with retinal detachment often have visual acuity and visual fields abnormalities.

(Option C) Depression can cause a variety of somatic symptoms. However, flashes and floaters are unlikely to be somatic presentation of depression.

(Option E) Optic neuritis can be associated with flashes, especially on eye movements. Floaters are not a feature because the pathology is limited to the optic nerve and does not
affect the vitreous. Moreover, retro-orbital pain, worse on eye movements, is a common presentation of optic neuritis which is absent in this patient.

TOPIC REVIEW

Differential diagnosis of flashes and floaters:

Flashes

Ophthalmic

Posterior vitreous detachment


Retinal tear/hole
Retinal detachment
Optic neuritis – photopsia on eye movement, retrobulbar pain

Non-ophthalmic

Migraine – scintillating (sparkling or shining) scotomas, colored lights, bilateral, evolves over 5 to 30 minutes before resolving with onset of a headache, normal visual
acuity
Postural hypotension – bilateral temporary dimming of vision and light-headedness
Occipital tumors
Vertebrobasilar transient ischemic attacks

Floaters

Ophthalmic

359 of 1943
Vitreous syneresis
Vitreous hemorrhage
Posterior vitreous detachment
Retinal detachment
Vitritis
Tear film debris

NOTE - There is no non-ophthalmic cause for floaters

References

• RACGP – Flashes and floaters

Last updated:
Time spent: QID:1437
2023-2-12

360 of 1943
A 75-year-old man presents to your practice with complaint of persistent right ear pain for the past few weeks. He has been a heavy a smoker for the past 50 years. He denies any
recent hearing loss. Physical examination, including a full otoscopic exam, is unremarkable. No significant hearing loss is noted. Which one of the following could be the most likely
diagnosis?

A. Cholesteatoma.

B. Chronic otitis media.

C. Tongue cancer.

D. Acoustic neuroma.

E. Ramsay-Hunt syndrome.

Incorrect. Correct answer is C


45% answered correctly

Explanation:

Correct Answer Is C

Ear pain (otalgia) is a common complaint and comprise approximately 4% of general practice setting encounters alone.

The nerve supply for sensation in ear originates from several cranial nerves including the trigeminal nerve (CN-V), facial nerve (CN-VII), glossopharyngeal nerve (CN-IX), and vagus
nerve (CN-X) as well as branches from the cervical plexus (mostly C2 and C3). These nerves also supply multiple head and neck structures; therefore, in many cases with ear pain,
pain is referred from somewhere else. This may pose a diagnostic challenge.

The etiology of ear pain can be primary or secondary. Primary otalgia originates from the ear structures itself while secondary otalgia is the pain felt in the ear but in fact referred
from other structures. In children, primary otalgia is more common while in adults ear pain is often due to a secondary cause.

The following table outlines the primary and secondary causes of ear pain:

Primary Secondary
Ear infections Dental inflammation and infections
Trauma Temporomandibular joint dysfunction
Foreign bodies Trigeminal neuralgia
Impacted cerumen (wax) Head and neck cancers
Otologic neoplasms (extremely rare) Temporal arteritis
Eagle’s syndrome

Primary otalgia often has benign causes that at experienced hands are often diagnosed through history and physical examination. With negative ear findings on examination,
secondary causes of ear pain should be considered and investigated. Odontogenic problems, especially those of posterior teeth, are the most common cause of secondary otalgia
accounting for approximately 63% of cases.

Temporomandibular joint (TMJ) dysfunction is the second most common cause of secondary ear pain. Vertigo and tinnitus are other possible presentations of TMJ dysfunction.
Pain related to chewing or other jaw movements is often a clue in history.

Trigeminal neuralgia usually has other associated symptoms such as unilateral attacks of abrupt onset that last up to 2 minutes and are excruciating.

All cancers of the head and neck, usually squamous cell carcinomas, can cause secondary otalgia in patients who have an otherwise normal otology history and examination. Of
importance are neoplasms in the oropharyngeal region (soft palate, posterior pharyngeal wall, palatine tonsil or tongue base), which can present with deep, intense otalgia. Patients
with cancers in this region may have additional symptoms of dysphagia, odynophagia, and sore throat, or may be otherwise asymptomatic. The presence of risk factors for these
cancers such as chronic alcohol use and tobacco exposure is a red flag and a clue to the diagnosis. In advanced stages, cervical lymphadenopathy is a common examination
finding.

Given this patient’s age, normal ear examination, presence of heavy smoking in the history, and lack of clues for other secondary causes of ear pain, squamous cell carcinoma of the
tongue is the most likely explanation for this presentation. This patient need referral to an otolaryngologist for further assessment and management.

(Option A) A cholesteatoma consists of squamous epithelium that is trapped within the skull base and can erode and destroy important structures within the temporal bone. Its
potential for causing central nervous system (CNS) complications (e.g., brain abscess, meningitis) makes it a potentially fatal lesion. Hearing loss is also a common symptom of
cholesteatomas. Large cholesteatomas fill the middle ear space with desquamated epithelium, with or without associated mucopurulent discharge, interfering with sound
transmission. Ossicular damage is frequently present and can cause conductive hearing loss. In this patient, normal exam findings are inconsistent with cholesteatoma as
diagnosis.

(Option B) Chronic otitis media presents with purulent ear discharge and often conductive hearing loss. Otoscopy shows perforated tympanic membrane and pus in the ear canal.
This patient has none of these findings. Moreover, the condition is usually seen in children.

(Option D) Acoustic neuromas are intracranial tumors arising from the Schwann cell sheath of either the vestibular or cochlear nerve. As acoustic neuromas grow, they eventually
occupy a large portion of the cerebellopontine angle. Unilateral hearing loss is the most common symptom present at the time of diagnosis. Other symptoms of acoustic neuroma
include headaches, and imbalance. Absence of unilateral hearing loss in this patient, and ear pain rather than headache make such diagnosis very unlikely.

(Option E) Ramsay Hunt syndrome (herpes zoster oticus) is the viral infection of the facial nerve and presents with ear pain, vesicles involving pinna/external auditory meatus and
facial nerve palsy. The absence of vesicles as well as other features of facial nerve involvement make such diagnosis unlikely.

361 of 1943
References

• RACGP – AFP – Otalgia

Last updated:
Time spent: QID:1614
2023-2-12

362 of 1943
Jane, 65 years old, presents to your GP practice with a lump she noticed the other day above her left collar bone. She denies any symptoms. ON examination. A 1.5x2 rubbery lump
is felt in the left supraclavicular fossa which is none tender and mobile. There rest of the physical examination is inconclusive. Which one of the following is the next best step in
management?

A. Fine needle aspiration (FNA).

B. CT scan of the chest.

C. CT scan of the brain.

D. Mammography.

E. Colonoscopy.

Correct
45% answered correctly

Explanation:

Correct Answer Is A

Neck masses in adults are a common presentation for head and neck cancer. Head and neck cancer accounts for 3.4% of all malignancies in Australia, and the incidence of
oropharyngeal squamous cell carcinoma is rising. Early diagnosis is essential to prevent worsening prognosis.

All neck masses should be considered malignant until proven otherwise. Detailed history and examination is crucial in the initial work-up.

It is of paramount importance to notice the presence of any red flag symptoms and risk factor for head and neck malignancies in the history.

Red flag symptoms for head and neck cancer include:

a mass that has been present for >2 weeks


a recent change in the voice
dysphagia or odynophagia
ipsilateral otalgia, nasal obstruction, or epistaxis
unexplained weight loss or loss of appetite

Risk factors for head and neck cancer include:

smoking
alcohol use
age >40 years
past history of previous head and neck malignancy
past history of head and neck cutaneous lesions

In physical examination, assessment of assessing the location of a neck mass is essential as it provides clues to the etiology. Characteristic features of the lump such as size,
shape, consistency, compressibility, mobility, changes in overlying skin, are very important and must be documented thoroughly.

In terms of investigations, fine-needle aspiration (FNA) of the mass for cytology and computed tomography of the neck with contrast are the mainstay of first-line investigation.

CT scan of chest (option B), colonoscopy (option E) and gastroscopy, CT scan of the brain (option C), and mammography (option D), are considered for further investigation to
detect a primary malignancy in case initial assessments suggest the lump to be a metastatic lesion.

References

• RACGP – AJP - An approach to neck masses in adults

Last updated:
Time spent: QID:1686
2023-2-12

363 of 1943
Jane is 67 years old and a known case of type 2 diabetes mellitus for the past few years. She has presented to your clinic in distress because she has started bumping into objects
since this morning. She also says that she has been seeing floaters and flies in front of her eyes for the past three days. Eye examination reveals vision loss and decreased red reflex
of the right eye. The right retina cannot be visualized. The vision of the left eye is preserved; however, there are retinal changes consistent with pre-proliferative diabetic retinopathy.
Which one of the following could be the most likely diagnosis?

A. Proliferative retinopathy.

B. Vitreous hemorrhage.

C. Occlusion of the central retinal vein.

D. Retinal detachment.

E. Cataract.

Incorrect. Correct answer is B


45% answered correctly

Explanation:

Correct Answer Is B

Floaters refer to the sensation of dark spots in the vision, which are caused either by opacities in the vitreous that cast shadows on the retina, or by light bending at the junction
between fluid pockets and the vitreous. Patients often describe floaters as “flies”, “cobwebs” or “worms” that are more pronounced against light backgrounds. Hemorrhage from
adjacent retinal vessels into the vitreous body leads to floaters, a sudden onset of decreased visual acuity (bumping into objects since this morning), and a darkened or diminished
red reflex consistent with this patient’s symptoms.

The most common cause of vitreous hemorrhage is proliferative diabetic retinopathy. As this patient has long-standing diabetes mellitus and her left eye presents retinal changes
consistent with pre-proliferative diabetic retinopathy, it is very likely that her right eye also has diabetic retinopathy with vitreous hemorrhage that leads to these symptoms. Though
vitreous hemorrhage is the right diagnosis with proliferative retinopathy (option A) being the most likely underlying pathophysiology for that in this case.

Occlusion of the central retinal vein (option C) presents with variable visual loss and has an insidious onset unless the vein is acutely and completely obstructed. The fundoscopy
will reveal retinal hemorrhage, dilated tortuous retinal veins, cotton-wool spots, macular edema, and optic disc edema.

Retinal detachment (option D) presents with sudden onset of visual impairment, floaters, and flashes. Patients describe black dots in their visual fields and visual loss as a curtain
coming down in front of their eyes. The visual field will be completely lost if the macula is detached. Ophthalmoscopy may show detached and floating retinal folds as a large grey
shadow in the vitreous. Dulling of the red reflex may be seen in retinal detachment.

Cataract (option E) often presents with a history of gradual progressive visual deterioration and disturbance in the night and near vision. Patients may also complain of decreased
visual acuity, glare, myopic shift, and monocular diplopia.

References

• RACGP- Flashes and floaters

• Medscape- Diabetic Retinopathy

Last updated:
Time spent: QID:1748
2023-2-12

364 of 1943
A 37-year-old woman with a history of recurrent oral and genital ulcers presents with a red eye as illustrated in the following photograph. Which one of the following is the most
appropriate action to take?

A. Oral corticosteroids.

B. Topical corticosteroids.

C. Oral fluconazole.

D. Referral to ophthalmologist.

E. Topical antibiotics.

Incorrect. Correct answer is D


45% answered correctly

Explanation:

Correct Answer Is D

The photograph shows conjunctival redness and congestion which is more intense at the limbus, iris with irregular contour, and more importantly a collection of exudate material at
the bottom of the anterior chamber due to gravity, characteristics of hypopyon. The presence of recurrent oral ulcers, genital ulcers, and uveitis in this patient makes the Behcet
syndrome the most likely diagnosis.

Patients with anterior uveitis need an urgent referral for review by an ophthalmologist and follow-up to prevent damage due to inflammation, including disruption to aqueous flow
inside the eye that can result in glaucoma and adhesion between the iris and lens. Behcet syndrome itself is also a condition requiring prompt referral for an ophthalmological
opinion, which may be sight-saving.

The treatment of Behcet syndrome is high-dose steroids and specific ulcer treatment. Ocular manifestations require systematic, not topical corticosteroids (option C).

Fluconazole (option C) is an antifungal medicine, and topical antibiotics (option E) are used for bacterial infections, none of which is the case in this scenario.

Although oral corticosteroids (option A) are the treatment of choice, they should be started by the specialist after a thorough assessment of the patient.

References

• Dermnet NZ – Behcet Disease

• Murtagh’s General Practice, John Murtagh, 8th Edition, p.212

Last updated:
Time spent: QID:1750
2023-2-12

365 of 1943
A 78-year-old male comes to your general practice clinic complaining of seeing wavy lines whenever he tries to directly look at his grandchildren. He is otherwise well and only wears
corrective lenses for his myopia. Fundoscopy findings are shown below:

Which of the following could be the best diagnostic test for this patient?

A. Amsler grid.

B. Visual field testing.

C. Optical coherence tomography.

D. Fluorescein angiography.

E. Digital tonometry.

Correct
45% answered correctly

Explanation:

Correct Answer Is A

The history and clinical findings, including the fundoscopic exam, suggest age-related macular degeneration (ARMD).

Fundoscopy findings show the characteristic drusen. Drusen are extracellular deposits made of lipids, proteins, and cellular debris under the retina. They appear as yellow or whitish
spots on fundoscopy. Having drusen increases an individual’s risk of developing ARMD and is considered a sign if the patient develops characteristic symptoms. Other fundoscopy
findings in ARMD include the following:

retinal edema and localized elevation


detachment of the retinal pigment epithelium (RPE)
a gray-green discoloration under the macula.

ARMD is the leading cause of vision loss in Australians over 50 years of age. It is a chronic, painless disease that mainly affects the retinal pigment epithelium.

Symptoms can include the following:

difficulty reading or any other activity which requires fine vision (despite wearing glasses)
distortion, where wavy lines appear bent
difficulty distinguishing faces
dark or blurred patches in the center of the vision

The Amsler grid is the best initial test to screen and monitor the progression of symptoms for ARMD. The patient can be given a piece of paper and asked to check for distortions in
the lines.

Below is a sample of an individual with normal eyesight next to that of a patient with ARMD.

366 of 1943
Visual field testing and optical computed tomography (options B and C) are ancillary procedures to check for the integrity of the RPE but are not required to diagnose ARMD.

Fluorescein angiography (option D) is used in the evaluation of ARMD to check for the vascular integrity of the retinal vessels. This involves dye injection and a highly specialized
camera. This may help in the detection of early disease but is not the initial step.

Digital tonometry (option E) has no role in the evaluation of ARMD and is indicated if glaucoma is suspected.

References

• Macular Disease Foundation Australia

• Medscape - Age-related Macular Degeneration

Last updated:
Time spent: QID:1778
2023-2-12

367 of 1943
A 21-year-old woman presents to your office with itchy and watery eyes. Her symptoms started 2 days ago with redness of both eye, lacrimation and itching. She wears contact
lenses. On examination, her upper and lower lids are erythematous bilaterally. Which one of the following, if in history, is most consistent with the diagnosis?

A. Mucopurulent discharge.

B. A lump in the upper eyelid.

C. Reduced visual acuity.

D. Using a new preserving solution for her contact lenses.

E. Bacterial conjunctivitis 3 months ago.

Incorrect. Correct answer is D


45% answered correctly

Explanation:

Correct Answer Is D

The clinical picture of bilateral itchy and watery eyes is consistent with either viral or allergic conjunctivitis. With erythematous eyelids, an allergic reaction would be the most likely
diagnosis. Allergic conjunctivitis may be the response of a previously sensitized individual to an allergen, including but not limited to the following:

Household dust
Pollen from trees or grass
Mold spores
Animal dander
Chemical perfumes
Contact lens solutions
Certain medications (systemic or topical)

​This woman has used a new contact lens solution that is very likely to have caused an allergic reaction.

(Options A and E) A mucopurulent discharge will be consistent with bacterial conjunctivitis which is unlikely in this scenario. A previous history of bacterial conjunctivitis is unrelated
to the current presentation.

(Option B) A lump in the upper eyelid is a finding in stye or chalazion, which have quite different presentations.

(Option C) Conjunctivitis does not affect vision. If vision is impaired, some other diagnoses other than conjunctivitis must be considered.

References

• Eye Emergency Manual

• Medscape - Allergic Conjunctivitis

Last updated:
Time spent: QID:296
2023-2-12

368 of 1943
A 42-year-old machinist engineer comes to the emergency department with a foreign body in his left eye. On examination, a metal speck is found stuck in the cornea. Which one of
the following is the most appropriate management?

A. Remove the foreign body under local anesthesia.

B. Obtain an orbital X-ray.

C. Refer the patient to an ophthalmologist.

D. Cover they eye with a pad.

E. Remove the foreign body under general anesthesia.

Incorrect. Correct answer is C


45% answered correctly

Explanation:

Correct Answer Is C

Any embedded foreign body should be removed by an ophthalmologist, and the next best step in management would be a referral to an ophthalmologist.

(Option B) An X-ray of the orbital area is required if a foreign body is suspected, but it is not visible on examination.

(Option D) No evidence supports the usefulness of an eye patch when the foreign body is still in the eye. After removal, however, padding the eye for 24 hours is often
recommended.

(Options A and E) Stuck foreign bodies of the eye should be removed by an ophthalmologist. Moreover, removal of an embedded foreign body usually does not require application of
anesthetics or general anesthesia.

References

• Eye Emergency Manual - An Illustrated Guide

• RCH - Acute eye injuries in children

Last updated:
Time spent: QID:299
2023-2-12

369 of 1943
A 38-year-old woman presents to the emergency department after getting hit in her right eye with a squash ball in a match. On examination, the visual acuity of the affected eye is
decreased. The eye is shown in the accompanying photograph. Which one of the following is the most appropriate next step in management?

A. Application of an eye patch.

B. Topical antibiotics.

C. Urgent referral to an ophthalmologist.

D. Evacuation of the blood under local anesthesia.

E. Reassurance as it resolves within 5 to 7 days.

Incorrect. Correct answer is C


45% answered correctly

Explanation:

Correct Answer Is C

The photograph shows blood in the anterior chamber consistent with the diagnosis of hyphema.

Causes of hyphema include:

Trauma – Trauma is the most common cause of hyphema. Of all traumatic hyphemas, most occur in sports. In traumatic hyphema, the impact forces the globe inward
and instantaneously increases the pressure of the anterior chamber. This pressure pushes the lens, iris, and ciliary bodies backward and disrupts the vasculature with
tearing of and bleeding from the blood vessels.

Blood clotting disturbances


Medications – anticoagulants
Neovascularization – such as in diabetic retinopathy, previous eye surgery, etc.
Melanoma or retinoblastoma
Abnormal vasculature

Hyphema is not usually painful unless it is caused by trauma. Visual acuity is often impaired in the affected eye. The presence of blood in the anterior chamber is characteristic.

Every patient with hyphema should be urgently assessed by an ophthalmologist as the most appropriate step in management. In the meanwhile, the eye should be protected from
more damage (e.g. by application of eye patches or shields), the head elevated and the patient advised about restricted physical activity.

Although most hyphemas resolve in 5 to 7 days, reassurance is not an appropriate action, as there may be serious complications without prompt assessment and treatment.

Topical antibiotics are not part of the management plan in hyphema.

NOTE – The following conditions require urgent referral to an ophthalmologist as the most appropriate management:

Significant trauma to the eye / penetrating trauma/chemical burns (severe) /embedded foreign body in the cornea or intraocular foreign body
Hyphema (>3mm)/hypopyon
Corneal ulcer
Severe conjunctivitis
Uveitis/acute iritis
Behçet syndrome
Acute glaucoma
Giant cell arteritis
Acute dacryocystitis
Endophthalmitis
Herpes zoster ophthalmicus – when the tip of the nose is involved, consider the possibility of the internal eye involvement as well

References

• Eye Emergency Manual

370 of 1943
• Medscape - Hyphema
Last updated:
Time spent: QID:300
2023-2-12

371 of 1943
Which one of the following is the most appropriate initial step in management of chemical burn injuries to the eyes?

A. Instillation of local anesthetics and padding the eye.

B. Irrigation with water.

C. Irrigation with a neutralizing agent.

D. Topical antibiotics.

E. Urgent referral to ophthalmologist.

Incorrect. Correct answer is B


45% answered correctly

Explanation:

Correct Answer Is B

In chemical injuries of the eye, the most important initial step is irrigation with plenty amount of water to wash out the offending agent, followed by urgent referral to an
ophthalmologist for further management.

(Option A) While the eye might have been seriously damaged, the application of topical anesthetics and padding are inappropriate actions. The anesthetic agent might have a
chemical interaction with the offending agent and cause more injury. Application of topical anesthetic should be considered after thorough irrigation of the eye if the patient still has
pain.

Option C) Neutralizing agents should never be used in this situation, as they can result in heat production and more damage caused by thermal injury.

(Option D) Topical antibiotics may be considered by the ophthalmologist for further management, but they have no role in initial management.

(Option E) Urgent referral to ophthalmologist is the most appropriate next step after vigorous irrigation.

References

• RACGP: Ocular emergencies

• RCH - Burns / management of burn wounds

Last updated:
Time spent: QID:301
2023-2-12

372 of 1943
A 45-year-old man comes to the emergency department with complaint of severe pain in his left eye. The pain started 3 hours ago, and is associated with blurred vision and nausea.
On examination, the patient has stable vitals. The left eye is reddened and hard to palpation. The visual acuity of the left and right eyes are 6/18 and 6/12 respectively. There is no
history of trauma to the eye. Which one of the following is the most likely diagnosis?

A. Acute retinal artery occlusion.

B. Acute retinal vein thrombosis.

C. Open-angle glaucoma.

D. Acute closed-angle glaucoma.

E. Cataract.

Incorrect. Correct answer is D


45% answered correctly

Explanation:

Correct Answer Is D

The clinical picture of a painful red eye and headache, blurred vision and decreased visual acuity and a hard orbit is highly suggestive of acute closed-angle glaucoma.

Acute-angle glaucoma results from an obstruction preventing the aqueous humor from being circulated normally; this leads to increased pressure within the orbit. It is a genuine
emergency and hesitation in prompt treatment can result in permanent loss of vision.

(Options A and B) Acute retinal artery and vein thrombosis can cause sudden loss of vision, but they are painless.

(Option C) Open-angle glaucoma presents with gradual painless visual loss. It does not develop acutely.

(Option E) Cataract is opacification of lens. It develops gradually. Cataract is not painful.

References

• Eye Emergency Manual

• RACGP – Ocular emergencies

• Medscape - Acute Angle-Closure Glaucoma in Emergency Medicine

Last updated:
Time spent: QID:304
2023-2-12

373 of 1943
A 56-year-old man presents to the emergency department with acute-onset pain in the left eye associated with headache and nausea. He has a bluury vision in the left eye. On
examination, the left eye is red, with an irregular and unresponsive pupil. The accompanying photograph shows the affected eye. Which one of the following is the most likely
diagnosis?

A. Acute iritis.

B. Bacterial conjunctivitis.

C. Acute closed-angle glaucoma.

D. Chronic open-angle glaucoma.

E. Cataract.

Incorrect. Correct answer is C


45% answered correctly

Explanation:

Correct Answer Is C

The sign and symptoms found on exam are suggestive of acute closed-angle glaucoma as the most likely diagnosis.

Acute closed-angle glaucoma presents with sudden onset unilateral eye pain, ipsilateral headache and, nausea and vomiting. The attack is often preceded by blurred vision or halos
around lights, at night time. Pupil dilatation in dim light results in more increased pressure of the anterior chamber and worsens the symptoms.

Findings on examination include the following:

Red eye
Irregular semi-dilated fixed pupil. The pupil may look vertically ovoid
Hazy cornea
The affected eye feels hard to touch, as the intraocular pressure has risen to over 60 mmHg (normal 15-20 mmHg)
Decreased visual acuity

(Option A) Acute iritis, better termed anterior uveitis, presents with acute pain, photophobia (not found in acute closed-angle glaucoma), lacrimation, circumcorneal redness, and a
constricted pupil.

(Option B) Conjunctivitis may present with pain and photophobia, but the rest of the exam is normal. Visual acuity is preserved and cornea and pupil look normal.

(Option D) Chronic open-angle (simple) glaucoma is optic nerve damage caused by chronically increased intraocular pressure. Patients, if symptomatic, will of complain of gradual
loss of visual fields; first, the nasal and superior, and eventually the temporal. Simple galucoma is not painful and the pupil and iris are normal. The eye is not red.

(Option E) Cataract is neither painful nor associated with red eye or pain. The lens is hazy on exam.

TOPIC REVIEW

The following table compares differentiating features of different underlying aetiologies of a redy eye:

Site of inflammation Pain Discharge Vision Photophobia Pupil Cornea Ocular


tension

374 of 1943
Conjunctiva, including lining Purulent, lids stuck
Bacterial conjunctivitis Irritation, gritty Normal No Normal Normal Normal
of the lid (usually bilateral) in the morning
Conjunctiva, lining of lids
Viral conjunctivitis often follicular (uni or Gritty Watery Normal No Normal Normal Normal
bilateral)
Conjunctiva, papillary
Allergic conjunctivitis swelling on lid linings Gritty, itching Watery Normal No Normal Normal Normal
(bilateral)
Contact hypersensitivity Conjunctiva and eyelids
Itching Watery Normal No Normal Normal Normal
(Dermato-conjunctivitis) edema
Subconjunctival Beefy red area fading at edge
No No Normal No Normal Normal Normal
hemorrhage (unilateral)
Unilateral – Circumcorneal No, reflex
Herpes simplex keratitis Yes / gritty Normal Yes Normal Abnormal Normal
dendritic ulcer lacrimation
Unilateral – Circumcorneal No, reflex
Corneal ulcer Yes Normal Yes Normal Abnormal Normal
(exclude foreign body) lacrimation
Localised deep redness/
Scleritis/episcleritis Yes No Normal No Normal Normal Normal
tenderness
Yes – radiates
No, reflex Constricted, maybe Normal or
Acute uveitis / iritis Maximum around cornea to brow, Decreased Yes Normal
lacrimation irregular low
temple, nose
Yes, severe
Diffuse but maximum No, reflex Dilated, absent light Hard,
Acute glaucoma with nausea Decreased Yes Hazy
circumcorneal lacrimation reflex elevated
and vomiting

References

• Eye Emergency Manual

• RACGP – Ocular emergencies

• Medscape - Acute Angle-Closure Glaucoma in Emergency Medicine

Last updated:
Time spent: QID:305
2023-2-12

375 of 1943
Sue, an 18 year old girl, presents to your clinic with lower abdominal pain and fishy-smelling vaginal discharge for the
past three days. You are a male doctor and a female nurse is on duty at the clinic. You mention to the patient that
she needs a vaginal examination, but she declines to be examined by you and says that she wants a female doctor to
examine her. Which one of the following is the most appropriate action in this situation?

A. Call her boyfriend to be present while you are examining her.


B. Treat the patient without vaginal examination.
C. Convince the patient that female chaperon will be present during the examination.
D. Defer the examination and inform the patient about the associated risks of not doing the examination.
E. Ask the nurse to perform the vaginal examination.

Incorrect. Correct answer is D


45% answered correctly

Explanation:

Correct Answer Is D

Informed consent is required before an intimate examination is carried out on a patient. Intimate examinations
include examination of the breasts, genitalia and anus/rectum. Patients have a right to decline examination as long
as the decision is informed.

This patient does not want to be examined by a male doctor; therefore, her wish should be respected, and the
examination is deferred, but she must also be informed about how delayed treatment might affect her health and
alter the course of treatment.

Patients may ask for the presence of a chaperon, but it is not the patient’s concern here. The chaperon or a female
nurse can be present during the examination, but they cannot do the examination on your behalf. The patient is
unwilling to be examined by a physician of opposite sex and presence of a female nurse or chaperon does not fulfill
her wish. A patient may ask for a particular chaperon to be present or a particular health practitioner (maybe gender-
based) to undertake the examination. Such requests are complied with where possible.

While the patient is uncomfortable with being examined by a male doctor, presence of his boyfriend (option A) or a
nurse or chaperon (option C) is incorrect. She does not want you to perform the examination; so, her wish should be
respected and any attempt to convince her otherwise is avoided.

(Option B) Treating without appropriate investigation and just based on a speculated diagnosis is not an acceptable
option.

(Option E) A nurse cannot perform the examination on your behalf as it is your responsibility to treat her based on
your own direct findings during the exam.

NOTE - It is wise to have a chaperon, nurse, or observer of the same sex in general present when performing
intimate examination, particularly when examining a patient of the opposite sex. If the patient refuses another
health practitioner to be present during the examination, this should be documented. Having a staff member
within your hearing is an advisable practice as a protective measure against possible accusations.

References

• AMA - Patient examination guidelines

Last updated:
Time spent: QID:716 2023-2-12

376 of 1943
A 42-year-old woman comes to your clinic complaining of headache and asking for a sick leave certificate. She has
separated two years ago and currently lives with her two children Sarah and Beth, 7 and 4 years old respectively. She
admits to smoking cigarettes and using alcohol on a regular basis. During the examination, she looks depressed.
More probing causes her to break in tears and confess that she cannot resist gambling. She has lost all of her
money and cannot provide care for her children anymore. Which one of the following would be the most appropriate
next step in management?

A. Refer her to specialist gambling treatment bodies (e.g., gamblers anonymous).


B. Inform the Child Protection Service.
C. Arrange for financial support.
D. Arrange for cognitive behavior therapy.
E. Perform motivational interview.

Incorrect. Correct answer is B


45% answered correctly

Explanation:

Correct Answer Is B

This patient has features of problem gambling requiring help and support, but the main issue and the most
important step would be the two little children she has back at home. Pathological gambling is not normally
associated with violence against children; rather the abuse tends to exist in the form of ‘neglect’.

This woman’s problematic behavior has posed her children at risk; therefore, the next step in management would be
protecting the children from the harm threatening them by informing child protection authorities.

(Options A and E) Once the children’s safety is ensured, attention should be turned to the patient. Motivational
interview would be the most important initial step, followed by referral to specialist gambling treatment agencies if
the patient is willing or has persuaded so.

(Option C) Options suggesting ‘arranging for financial support, local funding-for-poor programs, etc.’ are incorrect
because such measures, if necessary, will be taken by authorized agencies after the patient has been referred to
them.

(Option D) Cognitive behavioral therapy or other methods such as cognitive therapy or brief interventions can be
applied when felt necessary.

References

• RACGP - AFP - Problem gambling

• SA Health Department - PROBLEM GAMBLING A GUIDE FOR FRIENDS AND FAMILY

Last updated:
Time spent: QID:99 2023-2-12

377 of 1943
A mother brings her 18-year-old mentally retarded daughter at eight weeks pregnancy. The mother requests abortion
for her daughter as she is unable to look after herself. The patient does not want an abortion. Which one of the
following is the most appropriate action to take?

A. Perform medical abortion as it is close to nine weeks; otherwise she will require surgical abortion.
B. Do not perform abortion as the patient did not consent.
C. Refer the matter to Family Court.
D. Get consent from the local council.
E. Discuss the matter with the father of the girl in a week.

Incorrect. Correct answer is C


45% answered correctly

Explanation:

Correct Answer Is C

Because of mental retardation, this patient does not have capacity to make decisions and give consent. Although the
mother can give consent on her behalf, but there are particular procedures for which no relative, no matter how
close, can give consent on the patient’s behalf and the case must be referred to either the Supreme Court or the
Family Court. In Australia, these two courts can exercise their role as the supreme parents of children.

An application to the court should be considered in situations so serious that neither the incompetent young
person, nor the parents or guardian can give valid consent to. These situations are as follow:

The procedure is very high risk (for example, separating conjoined twins).
There may be life-changing effects such as in:
Sterilization of mentally disabled young persons
Abortions
Removal of life support
Removal of organs for transplants
Gender re-assignment
Bone marrow harvest
There is a strong objection from a dissenting parent
A child with capacity to make decisions is refusing healthcare and there is significant risk of harm in them
doing so
The procedure involves invasive, irreversible (oophorectomy) or major surgery. Life-saving emergency
surgeries are exceptions

Abortion in individuals who are incompetent due to mental disability should be consented to by the Supreme
Court or the Family court. Neither the patient, nor the carer consent is not valid for this procedure.

(Option A) Performing the abortion without a court order is illegal and inappropriate.

(Option B) The refusal of an incompetent patient does not eliminate the need for further action while the carer’s
concerns have not been addressed yet.

(Option D) Local councils are not authorized to make decision on this issue. As mentioned earlier, the decision is in
the power of the Supreme Court or Family Court.

(Option E) Like the mother, the father of the child is not authorized to make decisions in this regard.

378 of 1943
References

• NSW Health Department - Health Care in People with Intellectual Disability

Last updated:
Time spent: QID:717 2023-2-12

379 of 1943
A 34-year-old man is brought to the Emergency Department with fever, headache and a change in mental status
leading to significant disorientation. A head CT scan is obtained which is normal. Based on the diagnosis of
meningitis, he is planned to be started on intravenous antibiotics after a lumbar puncture is performed. The patient is
agitated and is fighting with anyone who tries to get near him. Some of his friends from work accompany him. Every
time an LP is about to be attempted, the patient pushes away the LP needle. Which one of the following would be the
most appropriate action to take in this situation?

A. Sedate the patient and perform the LP.


B. Wait for his relatives to arrive for consent.
C. Use blood cultures as an alternative.
D. Arrange for a brain MRI.
E. Ask his co-workers to sign the consent form.

Correct
45% answered correctly

Explanation:

Correct Answer Is A

The scenario describes a patient who is unable to give either an informed consent or informed refusal to the
procedure. He does not have the capacity to understand his medical condition and the consequences of deferring
the LP or antibiotics. On the other hand, there is no valid substitute decision maker (proxy) or family member to
make decisions on the patient’s behalf. Under such circumstances, when there is an urgent life-threatening or even
severely painful medical condition and the patient is not competent to give consent to or refuse the treatment and
there is no substitute decision maker or family member, the patient’s best interest will guide the management.

Since performing an LP followed by intravenous antibiotics is the most important and essential step in management
of suspected meningitis, which is potentially life-threatening, the patient should be sedated, and undergo LP.

(Option B) Waiting for the relatives to arrive for consent before starting the treatment is not appropriate as untreated
meningitis carries significant risk of morbidity and mortality.

(Options C and D) MRI or blood culture is not accurate for diagnosis and guidance of further treatment of meningitis.

(Option E) Co-workers are not qualified to consent on the patient’s behalf.

References

• Australian Medicolegal Handbook / Elsevier

• Good Medical Practice: Ethics, Law and Professionalism - Cambridge - pages 64-65

Last updated:
Time spent: QID:814 2023-2-12

380 of 1943
A 25-year-old man is involuntarily admitted to the mental health ward for treatment of psychosis. On physical
examination, he is found to have a 5x5 cm ulcer on the plantar surface of his left foot, requiring wound debridment,
irrigation and intravenous antibiotics. Which one of the following is the most appropriate approach regarding
consent for treatment of his wound?

A. No consent is required as the patient is already involuntarily admitted.


B. Take consent from the treating psychiatrist.
C. Take consent from the patient.
D. Contact the patient's family for informed consent.
E. Apply to the mental health tribunal for consent for wound debridment.

Incorrect. Correct answer is C


45% answered correctly

Explanation:

Correct Answer Is C

Patients who are subject to involuntary treatment under mental health act, may still have capacity to give or decline
consent for a specific healthcare matter not related to their current mental illness (for example the use of antibiotics
for chest infection, or a surgical procedure on the limb). Assessment of the patient’s capacity to make decisions
about every other treatment other than that for the initial problem should be performed in the standard way and
documented appropriately.

If a patient lacks capacity to give consent to health care for a condition unrelated to the current mental illness, the
consent should be sought from a substitute decision maker in the same order as for other patients who are not
legally able to give informed consent. Involuntary treatment order only applies to the current mental health issue and
does not cover other areas of treatment.

References

• Australian Medicolegal Handbook - Elsevier Australia (2008) – page 201

Last updated:
Time spent: QID:818 2023-2-12

381 of 1943
An Indigenous woman brings her four-month-old boy for vaccination. During the interview, she does not make any
eye contacts and avoids conversations. When you ask her to hold the child for you, so that you can inject the vaccine
she denies. Which one of the following would be the most appropriate management?

A. Send a nurse for a home visit.


B. Check her behavior in the next visit.
C. Ask one of her family members to accompany her in the next visit.
D. Call the Child Protection Service.

E. Involve her in a parental program.

Incorrect. Correct answer is C


45% answered correctly

Explanation:

Correct Answer Is C

In visiting indigenous patients, cultural differences should always be borne in mind. These differences sometimes
are a significant barrier in establishing appropriate communication and rapport with the patient.

In some cultures, any relationship between people of opposite sex is considered a taboo and unethical. In some
cases, outsiders cannot easily be trusted, and therefore it is recommended that an Aboriginal health worker be
involved when assistance is required with a cultural issue. Here, there are some pointers towards the failure in
communication between the doctor and the patient. Firstly, she avoids eye contact. Second, she is reluctant to
converse and finally she refuses to hold the child for vaccination because when you get close to the baby you may
get close to her and, by this, breach her circle of safety.

In situations like this, the best step can be involvement of another family member in next visits to make the patient
feel more comfortable.

Thus far, no concern regarding child abuse or neglect has risen, because the mother has brought her child for
vaccination and this indicates that the mother cares about the baby and his safety. For this reason, calling the child
protection authorities or sending a nurse for a home visit would not be necessary.

The patient’s behavior is not likely to change in the next visit if no active measures are taken.

A summary of communication tips for dealing with indigenous people are as follow:

Do not assume English is a first language, particularly in remote areas.


Do not assume a nod means understanding and/or agreement to treatment.
Check hearing because it might have been impaired due to chronic ear infection.
Appreciate the different family network, particularly the tendency of grandmothers and aunts to care for
children.
Do not assume a broken appointment means the patient will not return for treatment. Often family and cultural
duties take precedence.
Be aware of cultural sensitivities.
Do not touch a patient, particularly of the opposite sex, without seeking permission and explaining what you
are doing.
Be aware that patients may not be comfortable with direct questions about their family and health.
Do not be to stern or authoritative during a consultation.
Ensure receptionists and other staff understand the cultural sensitivities of Indigenous patients.

382 of 1943
Be accepting, respecting and non-judgmental.

References

• Murtagh’s General Practice – 5th Edition – page 1400

Time spent: QID:834 Last updated:


2023-2-12

383 of 1943
A 73-year-old woman has a living will clearly mentioning that she does not wish to be admitted if she is terminally ill.
Today, she is brought to the Emergency Department after she sustained a fall at home and had a femoral neck
fracture. In the emergency department, she becomes drowsy after a morphine shot is given to her for pain control.
Regarding her will, which one of the following would be the next best step in management?

A. Arrange for transferring to the operating room for surgical fixation of the fracture.
B. Arrange a family meeting.
C. Admit her.
D. Refer her for palliative care.
E. Check the validity of her will.

Incorrect. Correct answer is C


45% answered correctly

Explanation:

Correct Answer Is C

Based on the concept of autonomy, every individual has every right over their body if they are competent or
capacitated. In circumstances where the patient lacks competence to decide, a previous instruction by him/her such
as a valid living will or advance directives will guide the treating team as to treatment.

This patient has a living will that she should not be admitted if she is terminally ill. Her decision then should be
respected and acted upon if such circumstance arises, but as neither femoral neck fracture nor a completely
reversible adverse effect of opiates (drowsiness) is concordant with definition of a terminal illness, she should be
admitted for treatment of opiate overdose and reversal of the current condition. Once she is out of this state, further
management plan including fixation of her fracture by surgery or other measures can be discussed with her.

References

• Australian Medicolegal Handbook - Elsevier– pages 134-148

Last updated:
Time spent: QID:840 2023-2-12

384 of 1943
A 26-year-old immigrant man stole a car and while on the run he hits a woman on the road. Eventually, he ended up
hitting the guardrail in a highway and injuring himself. He is in the hospital now and behaves aggressively and rude.
He requires orthopedic attention and care. While in the ward, he insists to smoke a cigarette but smoking is not
allowed in the ward. He warns you, as his treating doctor, that he will sue you because he knows many influential
people and will make you lose your job. Which one of the following is the most appropriate action in this situation?

A. Discharge him because he is not cooperative.


B. Contact immigration authorities to take over for legal proceeding.
C. Tell him that you can only give him nicotine gum or patch.
D. Call the police to arrest him and take over.
E. Take him to a safe place and let him smoke under supervision.

Incorrect. Correct answer is E


45% answered correctly

Explanation:

Correct Answer Is E

In mental health setting, cigarettes have been used as a patient management tool by staff, mediating exchanges and
relationships between staff and patients and between patients. Examples include using control over supply of
cigarettes to patients to comply with requests such as taking medications, getting dressed, agreeing to speak to the
treating doctor, etc. Although it has been a place of debate if implementation of non-smoking policies in psychiatric
wards is of benefit, it is still in practice.

In this situation though (emergency setting) where a severely agitated and disturbed patient is approached, allowing
him to smoke may help in de-escalation of the patient without unnecessary coercive treatments. A common practice
in Australia, when such a situation arises, is to allow the patient to smoke under appropriate supervision and in place
that the risk of others being exposed to cigarette smoke is nil or at least minimum.

Discharging an uncooperative agitated patient not only is inconsistent with the duty of care, but it also may pose the
patient and others at significant risk; therefore, not an appropriate action.

Discharging the patient (option A) or surrendering him while he is in need of medical care both for his psychiatric
and orthopedic problems to the immigration office (option B) or the police (option D) is not appropriate. The police
can deal with the patient, if necessary, after adequate care has been taken medically.

Nicotine gums and patches (option C) may be helpful in reduction of agitation in patients who are quitting smoking,
but unlikely to satisfy this patient.

References

• NCBI - Achieving Smoke-Free Mental Health Services: Lessons from the Past Decade of Implementation Research

• NCBI - Verbal De-escalation of the Agitated Patient: Consensus Statement of the American Association for
Emergency Psychiatry Project BETA De-escalation Workgroup

Last updated:
Time spent: QID:847 2023-2-12

385 of 1943
You are the on-call psychiatrist on a night shift when you are informed by the nurse of the psychiatry ward that a 48-
year-old lady with severe depression has refused to eat or drink for the past two days. She believes she does not
have bowels. She was brought to the hospital by her son. On examination, she is severely dehydrated, but refuses to
receive any oral intake. Electroconvulsive therapy (ETC) is considered for her by you as an authorized psychiatrist for
ECT, but she refuses to give consent to the procedure. Which one of the following is the most appropriate next step
in her management?

A. Take consent for ECT from her son.


B. Take consent from the Mental Health Tribunal.
C. Take consent from hospital administrator.
D. Treat her with ECT without consent under duty of care.
E. Give her nasogastric feeding and anti-depressants.

Incorrect. Correct answer is D


45% answered correctly

Explanation:

Correct Answer Is D

ECT is widely used as a treatment option. In Australia, the most frequent indication for ECT has been major
depression, especially if associated with psychotic features (such as in this patient). Regulations regarding when to
use ECT and how to obtain consent vary from state to state.

For voluntary patients who have adequate capacity to make decisions for themselves, ECT can be administered
by authorized physicians if:

The procedure and techniques have been fully explained to the patient
All associated discomforts and potential adverse effects have been explained to the patient
Alternative treatments (if available) has been explained to the patient
The patient has been offered to ask any question regarding the procedure
The patient is aware of his/her right to withdraw consent and discontinue the procedure at any given time
The patient has been noted that she/she can have legal and medical advice before giving consent

NOTE - Consent to ECT should be in writing.

For involuntary ECT, consent process and authorization varies in different states. ECT is different from other
procedures in terms of substitute decision making processes. In South Australia, for example, ECT should be
authorized by the Guardianship Board; however, should emergency arise, ECT can be given without applying the case
to the Guardianship Board. In the Australian Capital Territory and New South Wales, Mental Health Tribunal is the
authorized body for approval of ECT on involuntary patients.

Despite differences among different Australian states (and for the exam purposes) the following rules Produced by
Australian Health Ministers Advisory Council (AHMAC) can be applied in general:

The Tribunal may approve the performance of electro-convulsive therapy upon a person who is an involuntary
patient, a forensic patient, a patient under supervision or is subject to a community treatment order but it must not
give its approval unless satisfied that:

The person is not capable of giving informed consent

AND

386 of 1943
Two medical practitioners (at least one a psychiatrist) have formed the opinion after considering the person’s
clinical condition, history of treatment and any appropriate alternative treatments that electro-convulsive
therapy is reasonable and a proper treatment to administer for a the person and that without that treatment
the person is likely to suffer serious mental or physical deterioration.

Electro-convulsive therapy in life saving emergencies:

The authorized psychiatrist may authorize the performance of electro-convulsive therapy upon a person who is an
involuntary patient, a person under supervision or a forensic patient without having obtained the approval of a
tribunal if the authorized psychiatrist has the opinion that electroconvulsive therapy is necessary to save life of a
person or to prevent the person from suffering irreparable harm.

The authorized psychiatrist must report electroconvulsive therapy to the tribunal after it is performed.

In simple words, involuntary patients in an emergency condition, where delay can lead to serious harm to them or
others, are treated with ETC by an authorized psychiatrist without any need for approval from Mental Health
Tribunal, Guardianship Board, etc.

Where the situation is not likely to result in serious harm to the patient or others, the decision as to whether ECT
is performed as involuntary treatment should come from authorities.

It is strongly recommended that emergency ECT be avoided if possible. It is also strongly recommended that
emergency ECT is not given without a second opinion. If practical, the matter should be discussed with the chief
psychiatrist or his/her authorized representative before proceeding.

This patient is suffering from severe dehydrated that can be life threatening or at least pose substantial risk to her
health; therefore, she is in an emergency. Under this circumstancees, ECT should be proceeded with without consent
and under the duty of care as per the above.

Again, it is of paramount importance that physicians seek advice regarding the state legislations in practice.

References

• SA Health Department - Electroconvulsive Therapy Policy Guideline

• Queensland Health Department - The Administration of Electroconvulsive Therapy

Time spent: QID:874 Last updated:


2023-2-12

387 of 1943
A 52-year-old woman is involuntarily admitted to the psychiatric ward due to major depression with psychotic
features. She refuses to take anything by mouth including her medications because she believes that she will die if
she eats. Electroconvulsive therapy (ECT) has been decided for her as a life-saving measure and explained to her but
she refuses to consent to this treatment. She is clinically stable for now. Which one of the following is the next best
step in her management?

A. Take consent from the Mental Health Tribunal.


B. Give ECT without consent under duty of care.
C. Obtain consent from her husband.
D. Give her nasogastric feeding and anti-depressants.
E. Obtain consent from the hospital administrator.

Correct
45% answered correctly

Explanation:

Correct Answer Is A

ECT is widely used as a treatment option. In Australia, the most frequent indication for ECT has been major
depression, especially if associated with psychotic features (such as in this patient). Regulations regarding when to
use ECT and how to obtain consent vary from state to state.

For voluntary patients who have adequate capacity to make decisions for themselves, ECT can be administered
by authorized physicians if:

The procedure and techniques have been fully explained to the patient
All associated discomforts and potential adverse effects have been explained to the patient
Alternative treatments (if available) has been explained to the patient
The patient has been offered to ask any question regarding the procedure
The patient is aware of his/her right to withdraw consent and discontinue the procedure at any given time
The patient has been noted that she/she can have legal and medical advice before giving consent

NOTE - Consent to ECT should be in writing.

For involuntary ECT, consent process and authorization varies in different states. ECT is different from other
procedures in terms of substitute decision making processes. In South Australia, for example, ECT should be
authorized by the Guardianship Board; however, should emergency arise, ECT can be given without applying the case
to the Guardianship Board. In the Australian Capital Territory and New South Wales, Mental Health Tribunal is the
authorized body for approval of ECT on involuntary patients.

Despite differences among different Australian states (and for the exam purposes) the following rules Produced by
Australian Health Ministers Advisory Council (AHMAC) can be applied in general:

The Tribunal may approve the performance of electro-convulsive therapy upon a person who is an involuntary
patient, a forensic patient, a patient under supervision or is subject to a community treatment order but it must not
give its approval unless satisfied that:

The person is not capable of giving informed consent

AND

Two medical practitioners (at least one a psychiatrist) have formed the opinion after considering the person’s
clinical condition, history of treatment and any appropriate alternative treatments that electro-convulsive

388 of 1943
therapy is reasonable and a proper treatment to administer for a the person and that without that treatment
the person is likely to suffer serious mental or physical deterioration.

Electro-convulsive therapy in life saving emergencies:

The authorized psychiatrist may authorize the performance of electro-convulsive therapy upon a person who is an
involuntary patient, a person under supervision or a forensic patient without having obtained the approval of a
tribunal if the authorized psychiatrist has the opinion that electroconvulsive therapy is necessary to save life of a
person or to prevent the person from suffering irreparable harm.

The authorized psychiatrist must report electroconvulsive therapy to the tribunal after it is performed.

In simple words, involuntary patients in an emergency condition, where delay can lead to serious harm to them or
others, are treated with ETC by an authorized psychiatrist without any need for approval from Mental Health
Tribunal, Guardianship Board, etc.

Where the situation is not likely to result in serious harm to the patient or others, the decision as to whether ECT
is performed as involuntary treatment should come from authorities.

It is strongly recommended that emergency ECT be avoided if possible. It is also strongly recommended that
emergency ECT is not given without a second opinion. If practical, the matter should be discussed with the chief
psychiatrist or his/her authorized representative before proceeding.

Impaired judgment and false bizarre beliefs of this patient, along with the major depression is indicative of major
depression with psychotic features, which is one of the most common and well-known indications for ECT as a
highly effective treatment. This patient, who clearly lacks decision-making capacity, is subject to involuntary
treatment after approval from Mental Health Tribunal.

(Option B) Since the patient is clinically stable, no emergency situation is present to mandate ECT without approval
from the aforementioned authority.

(Options C and E) ECT widely is different from other medical procedures where consent from relatives of a patient
who is subject to involuntary treatment can be obtained; therefore, consent from other people such as relatives,
hospital administrator, another colleague, a medical senior, etc. are incorrect options.

(Option D) Nasogastric tube to feed or give medications does not eliminate the need for ECT where it is clearly and
necessarily indicated.

Only medical practitioners are permitted to provide ECT, and it must be performed in a hospital approved for this
purpose, whether public or private. A minimum of two medical practitioners must be present, of whom one should be
experienced in the administration of ECT and the other in anesthesia.

References

• SA Health Department - Electroconvulsive Therapy Policy Guideline

• Queensland Health Department - The Administration of Electroconvulsive Therapy

Time spent: QID:876 Last updated:


2023-2-12

389 of 1943
One of your patients is a 37-year-old doctor, who has just recently found out he is HIV positive. You are the only one
that knows about this. Which one of the following you are legally obliged to inform?

A. His hospital administrator.


B. Medical Board.
C. His patients.
D. His patients only if he performs surgeries where transmission is possible.
E. No one without his consent.

Incorrect. Correct answer is E


45% answered correctly

Explanation:

Correct Answer Is E

Healthcare workers (HCWs), who are HIV positive, have a right to privacy as long as they are not posing others at the
risk of the infection. Therefore, you cannot inform anyone of his condition without his consent if he is not posing
others at risk.

(Options A, B and C) There are guidelines and protocols for HCWs with blood-borne viruses, and it is assumed that
they follow these guidelines and precautionary measures as a part of their job. Neither the treating physician, nor the
patient is obliged to inform any authorities including Medical Board, state government, the insurance company or his
employer.

(Option D) An HCW is not legally obliged to inform his/her patients of his/her HIV positivity. The risk of transmission
of the infection to the patients is extremely rare, especially when the physician takes precautionary measures and/or
is under treatment with antiretroviral therapy. However, HCWs should understand their obligation to report their
infections with blood-borne viruses (BBV) status if required under jurisdictional legislation and should be informed of
relevant policies. They should understand their obligation to report all sharp injuries, whether or not there was a risk
of patient exposure.

TOPIC REVIEW

The following should be considered for HCWs with BBVs:

All HCWs infected with a BBV should remain under regular medical supervision.

HCWs must not perform EPPs (exposure-prone procedures) if they are human immunodeficiency virus (HIV)
antibody positive.

HCWs must not perform EPPs while they are hepatitis C virus (HCV) RNA positive but may be permitted to
return to EPPs after successful treatment or following spontaneous clearing of HCV RNA.

HCWs must not perform EPPs while they are HBV DNA positive, but may be permitted to return to EPPs
following spontaneous clearing of HBV DNA or clearing of HBV DNA in response to treatment.

References

• Australian National Guidelines for the Management of Healthcare Workers Living with Blood Borne Viruses and
Healthcare Workers who Perform Exposure Prone Procedures at Risk of Exposure to Blood Borne Viruses

390 of 1943
Last updated:
Time spent: QID:908 2023-2-12

391 of 1943
A 26-year-old female comes for cervical cancer screening by HPV testing. She never had sexual activity with a man,
but is a lesbian and has a girlfriend. She prefers to see a lesbian-friendly health care provider for her test. Which one
of the following would be the most appropriate action to take in this situation?

A. She does not need cervical cancer screening.


B. Refer her to a lesbian-friendly clinic in the area.
C. Do the test yourself and then refer her to a gynecologist.
D. Ask your clinic nurse to do the test in your clinic.
E. Advise her to follow safe sex practice principles even with a female partner.

Incorrect. Correct answer is B


45% answered correctly

Explanation:

Correct Answer Is B

Specific subtypes of human papilloma viruses (HPV), most commonly types 16. 18, 31, 33, 35, are associated with
premalignant and cancerous cervical lesions. HPV can spread through sexual intercourse, as well as skin-to-skin
contact in female-to-female sexual relationship. Therefore, lesbians need pap smears like heterosexual women.

There is no evidence to suggest that HPV infection rate is lower in lesbians, and rates of cervical abnormalities for
lesbians are like those of heterosexual women.

As this patient prefers to see a lesbian-friendly healthcare provider, the next best step would be encouraging her for
screening and referring her to a lesbian-friendly clinic in your area. Failing to do so may result in the patient not
seeking screening. This not only poses the patient at risk; it may also lead to lawsuit for the physician once the
patient contracts cervical cancer.

(Option A) Lesbians need cervical cancer screening like heterosexual women. Telling her that she does not require
pap smears is not correct.

(Option C) Performing a cervical cancer screening test without the patient's consent is an act of battery and should
be avoided.

(Option D) It is a wise practice to have a female nurse or chaperon present while the doctor takes sample for HPV
testing, particularly if the doctor is male, but taking samples is the responsibility of doctors not nurses.

(Option E) Advising precautionary measures such as safe sex practice is appropriate but does not eliminate the need
for screening tests as the most important issue here.

References

• Murtagh’s General Practice – McGraw Hill – 5th Edition – pages 929 and 932

• Good Medical Practice – Professionalism Ethics and Law – Cambridge – page 165

Last updated:
Time spent: QID:424 2023-2-12

392 of 1943
You are a resident at the Emergency Department. An angry father approaches you because the social worker has
been asking him if he has punished his child physically. The child is five years old and has been in the Emergency
Department four times this year with several episodes of trauma not consistent with the alleged history given by the
parents. Today, the child is brought with a complaint of ‘slipping into a hot bathtub’ with a burn wound on his legs.
The father threatens to sue you and says ‘how dare you think that about me, I love my son!’ Which one of the
following would be the most appropriate next step in management?

A. Admit the child to remove him from the possibly dangerous environment.
B. Call the police.
C. Ask the parents if there has been any abuse.
D. Speak to the wife privately about possible child abuse.
E. Report the family to child protective services.

Incorrect. Correct answer is E


45% answered correctly

Explanation:

Correct Answer Is E

Once a healthcare worker forms an opinion, on reasonable grounds, that child abuse has occurred or is in progress,
reporting to Child Protection Service is mandatory. The physician is legally protected if the case is found out not to
be due child abuse, if reporting has occurred in good faith.

In this case, with several episodes of injuries with unfitting accounts, child abuse is very likely and the family should
be reported to the Child Protection Service immediately.

NOTE - The power of removing the child from the parents is not within the physician’s authority. This is
undertaken by authorities such as child protection services or courts of law.

(Option A) Admitting the child for protection would be unnecessary as the child can be satisfactorily safeguarded
while in hospital.

(Option B) Calling the police would have been indicated if the assault is in progress, which is not the case here.

(Options C and D) When the belief of child is formed, talking to the parents would be incorrect as it is unlikely to
change the course of action. Abusive parents are not likely to give the exact account of the event and admit to child
abuse, nor are they likely to change their behavior without intervention.

References

• RACGP - The White Book: Child abuse

• RCH - Child abuse

• AIFS - Mandatory reporting of child abuse and neglect

Last updated:
Time spent: QID:913 2023-2-12

393 of 1943
During visiting an 82-year-old man for an upper respiratory tract infection, you notice multiple bruises on
different body parts. He lives with his daughter and her boyfriend, and confides in you that the bruises are the result
of being physically abused by her daughter’s boyfriend. He adamantly insists that you should not inform the police or
any other authorities because he can deal with his problem by himself. You perform a mental status exam, including
a ‘serial seven’ the result of which is normal. Which one of the following would be the most appropriate management
option in this situation?

A. Discharge him home after management of the bruises.


B. Obey his wish but arrange for regular follow-ups.
C. Ask him to see a social worker before he leaves the hospital.
D. Inform the daughter about abuse so that she knows what is happening with her father.
E. Notify the police immediately.

Incorrect. Correct answer is B


45% answered correctly

Explanation:

Correct Answer Is B

Elder abuse must be considered by any health practitioner seeing elderly patient as they have an essential role in the
recognition, assessment, understanding and management of elder abuse and neglect. Once faced with
elderly abuse, the first thing to consider is to assess if the patient has the capacity to make decisions (as in this case
where the mental status of the patient has been evaluated).

The elderly should be consulted about the criminal nature of abuse and that it is unacceptable and there is always
means to prevent it. They should be made aware of they legal rights and that they can seek legal action and
protection if they wish so at any time.

If the elderly patient has the decision-making capacity and refuses any intervention, their decisions must be
respected, but he/she should be advised to contact you or other support agencies for help in the future. The status
of the elderly should be checked through regular follow-ups.

References

• RACGP - The White Book: Elder abuse

Last updated:
Time spent: QID:916 2023-2-12

394 of 1943
A 22-year-old Aboriginal man is in the waiting list for kideny transplant due to end-stage renal disease (ESRD). In the
meanwhile, he is on dialysis three times a week. He has presented to you as his treating physician and says he does
not want to undergo dialysis and wants to withdraw from treatment. Which one of the following would be the most
appropriate action to take?

A. Arrange for a donor for him.


B. Ignore his wish and treat him.
C. Arrange a family meeting.
D. Discuss his decision with him to make sure he understands the consequences.
E. Refer the case to the court.

Incorrect. Correct answer is D


45% answered correctly

Explanation:

Correct Answer Is D

Patient autonomy is the cornerstone of all healthcare ethics. Every competent adult has the absolute right to do what
they desire with their own health and life. Competency is a legal term and not determined by the healthcare workers.
Capacity, which is a different term, is what used instead in medical decision-making process. These two, however,
can be used interchangeably.

When a patient decides not to accept or withdraw from a treatment, the most appropriate next step is always a full
discussion with the patient about the potential consequences of his/her decision and making sure that they
understand them.

(Option A) Arranging for a donor just because he does not want to go on with the treatment is not appropriate. Every
patient should follow specific protocols.

(Option B) Ignoring a competent patient’s expressed wish and acting differently is an act of battery and punishable
by the law.

(Options C and E) Arranging a family meeting for discussion about a competent patient’s wish is not appropriate.
Neither is referring the case to the court because the law is quite straightforward on this matter.

References

• MJA - Consent, capacity and the right to say no

• Good Medical Practice – Ethics / Professionalism and law – Cambridge – page 333

Last updated:
Time spent: QID:929 2023-2-12

395 of 1943
A mother brings her two-year-old daughter to your clinic because she is concerned about a lesion on her
child's external genitalia. On examination, the lesions turn out to be genital wart (condyloma acuminata). The mother
has the history of treatment for a CIN1 cervical dysplasia. Which one of the following can be the most likely cause of
her genital wart?

A. Perinatal infection from the mother.


B. Sexual abuse.
C. Primary infection.
D. Breastfeeding.
E. Autoinoculation.

Correct
45% answered correctly

Explanation:

Correct Answer Is A

Sexually transmissible diseases (STIs) are rarely seen in cases of sexual child abuse but if present, strongly suggest
the possibility. In other words, a child with STI has suffered sexual abuse until proven otherwise; this, however, does
not mean that sexual abuse is the most likely explanation.

Anogenital warts or condyloma acuminata are caused by the human papilloma virus (HPV).

Studies indicate that in adults, genital HPV infections are primarily sexually transmitted. In children, the mode of
transmission of HPV infection is not as straightforward. Sexual transmission is recognized as a possibility in
children, but other possible modes of transmission have been documented as well.

In summary, the modes of HPV transmission in children include:

Sexual abuse

Oral-genital contact
Genital-genital contact
Genital-anal contact
Fondling
Digital penetration of the vagina or anus

Nonsexual transmission

Autoinoculation
Direct contact with caretaker
Contact with objects or surfaces contaminated with HPV

Vertical transmission (from mother to infant)

Via bloodstream prior to birth


During vaginal delivery through infected birth canal
Via cesarean section with or without early rupture of membranes

Sexual abuse must never be eliminated when considering possible modes of transmission for anogenital HPV. Many
forms of sexual abuse can result in transmission of HPV, including genital-genital contact, genital-anal contact, oral-
genital contact, fondling, and digital anal/genital penetration.

However, Adams' (2001) classification scale for evaluating medical findings of suspected sexual abuse lists
anogenital warts/condyloma in a child younger than two years of age as a nonspecific finding for sexual abuse. In
396 of 1943
such cases perinatal transmission must be considered first as the most likely explanation.

Vertical transmission of the HPV virus does not mean that warts must be present at birth or shortly after birth. HPV
is a latent virus and can reside in the skin and mucous membranes without causing warts. The warts may not appear
until months or even years after birth. Some authors believe the time between infection and the presentation can be
as long as five years. Some believe in a shorter period of up to two years. In general, vertical transmission of HPV
can still be the main cause even if lesions first appear years after birth.

Vertical transmission can occur through the bloodstream prior to birth, or at the time of birth as the infant passes
through the infected birth canal. Delivery via cesarean section (with or without premature rupture of membrane)
does not eliminate the possibility of vertical transmission of HPV. There are even reports of congenital condyloma
after caesarean section without premature rupture of membranes.

Some authors also believe that that HPV transmission can occur in utero through semen, ascending infection from
the mother's genital tract, or transplacentally.

Anogenital warts (HPV) also can be transmitted via autoinoculation. Children with a common wart on their hands or
elsewhere on their body can transmit the virus by touching their warts and then touching their own genitals.

Non-sexual transmission can also occur from direct contact with caretaker contaminated with genital HPV or
common warts. For example, caretakers with genital warts who touch or scratch their genitals and then, without
washing their hands, change a baby's diaper or assist a child with toileting/bathing may transmit the virus to the
child's genitals. HPV transmission via contact with contaminated objects or surfaces is also possible.

The mother has been treated with CIN1. Although HPV serotypes associated with cervical cancer are different from
thosecausing anogenital warts, presence of cervical neoplasia could suggest co-infections with other types of HPV
as well. In this child, with the mother’s possible infection and the child’s age, the most likely cause to the child’s
anogenital warts appears to be perinatal infection from the mother.

If the child was older the likelihood of sexual abuse would be more pronounced, as perinatal infection must have
presented by 2 years of age, as most authors believe. Although not a rule, the younger the child, the more likely the
HPV infections is due to perinatal infections rather than sexual abuse.

References

• Medscape - Ano-Genital Warts in Children: Sexual Abuse or Not?

• NCBI - Genital warts in children: what do they mean?

Time spent: QID:934 Last updated:


2023-2-12

397 of 1943
You are assessing a 16-year-old girl, who has cut her wrist intentionally. You talk to her and after a thorough
assessment you are reassured that she is not suicidal, nor does she have any suicidal ideations at the moment. Her
parents separated two years ago. Currently, she is living with her mother, but she wishes to live with her father. Both
her parents and the school psychologist are concerned about her and insist to be informed of her condition. Which
one of them should you inform?

A. Only the father.


B. Only the mother.
C. Both parents.
D. Only the school psychologist.
E. None of them.

Incorrect. Correct answer is E


45% answered correctly

Explanation:

Correct Answer Is E

The case represents self-mutilation in the absence of an intention to die or suicidal thoughts. This is termed non-
suicidal self-injurious behavior. The behavior is purely for non-suicidal reasons, either to relieve distress or to make a
change in others or the environment, or for anxiety relief.

This girl is 16 years old and considered an adult in most areas of healthcare. She has a legal right to confidential
healthcare.

The exception is when there is a significant concern of risk to self or others. This patient, based on the opinion of a
healthcare professional, is not suicidal and does not pose any harm to herself, at least for now; therefore, she is
excluded from this exception. Her current problem should not be informed to anyone without her expressed consent;
however, she should be encouraged to seek help and counselling from professionals (e.g., school psychologist), or
her supporting resources such as her parent.

Confidentiality is a means of providing the client with safety and privacy and therefore, protects client autonomy. For
this reason, any limitation on the degree of confidentiality is likely to diminish the effectiveness of counselling.

It should be noted though that if the patient was acutely suicidal, she lacked competence and involuntary actions
could have been considered.

NOTE - Notifying the parents of a dependent minor cannot be performed without their consent. Should any
dispute arise between the duty of the health professional and the child’s refusal of parental notification, the issue
should be referred to an authorized third party such as a court.

References

• RCH - Engaging with and assessing the adolescent patient

• MJA - Confidential health care for adolescents: reconciling clinical evidence with family values

Last updated:
Time spent: QID:958 2023-2-12

398 of 1943
A 34-year-old construction worker presents to your clinic after a foreign body entered his left eye while working. He is
from Algeria and does not speak English. His supervisor is accompanying him in the visiting room and asks if he
could translate for the patient. Which one of the following is the most appropriate action to take in this situation?

A. Ask for an accredited translator to be present.


B. Bring an accredited interpreter on the phone.
C. Ask the supervisor to leave the room and do not intervene because the patient’s privacy could be
breached.
D. Ask the patient if he wants his supervisor to do the interpretation.
E. Let the supervisor to do the interpretation.

Incorrect. Correct answer is B


45% answered correctly

Explanation:

Correct Answer Is B

Language differences pose a challenging situation on doctor-patient relationship where the treating doctor and the
patient speak different languages. The problem is prominent in countries such as Australia where the number of
those unable to efficiently speak and/or understand English is considerable owing to high rate of immigration from
overseas.

It is essential that in situations where a patient has some difficulty or uncertainty in understanding English, a
qualified healthcare interpreter is used. The fact that a healthcare interpreter/ translator has been used should be
noted in the patient’s medical records. Most consent forms have a space for statement that the translator has
translated the contents of the form and the information given by the patient.

It should be noted that a member of the patient’s family, a friend or another non-accredited person should not, in
general, act as interpreter, as both legal and ethical questions could be raised about the validity of any consent
obtained. An exception is when the medical issue is minor and use of a close friend or family member is the
expressed wish of the patient.

In this case scenario, the treating doctor should ask for an accredited healthcare interpreter to be present; however,
since it is often impossible to have a qualified interpreter available on the spot, the Telephone Interpreting Service
(TIS) should be used as the most convenient means of accessing to an interpreter. This service has a dedicated
telephone number for doctors in private practices and the service is free when doctors are providing care. The
service is claimable under Medicare to Australian citizens or permanent residents.

If an unqualified interpreter has been used, for example in an emergency, a qualified interpreter should be obtained
as soon as possible to ensure that the patient has understood what has taken place.

It is not appropriate to use the supervisor to translate despite the fact that the patient might have consented to it.
When the translator is present, the doctor could reliably ask the patient about his preference is he prefers the
supervisor to be present.

References

• RACGP - Using interpreters

• RACGP - Standards for general practices (4th edition)

Last updated:
Time spent: QID:975 2023-2-12

399 of 1943
All of the following situations allow the doctor-patient confidentiality to be breached except:

A. When the patient consents to allow personal details to be revealed to a third party.
B. If there are other health professionals who have a legitimate therapeutic interest in the care of the patient
including medical students.
C. If there are other health professionals who have a legitimate therapeutic interest in the care of the patient
excluding medical students.
D. If there is overriding public interest.
E. Where disclosure of the information is required or permitted by operation of the law.

Incorrect. Correct answer is B


45% answered correctly

Explanation:

Correct Answer Is B

For a proper doctor-patient relationship it is important to ensure that information provided by patients to the treating
doctor will remain strictly confidential.

The general rule is that doctors may not, without the consent of their patients, disclose to any third party information
acquired in the course of their professional relationship. This rule of confidentiality extends also to disclosure to
family members.

However, there are exceptions where confidentiality can be breached. These exceptions are as follows:

Where the patient gives valid consent for his/her medical information to be revealed to a third party - e.g. the patient
asks you to reveal his medical information to his/her employer, insurance company, etc.

Sharing information in the healthcare team - in many health care situations, consent for sharing confidential
information between members of the ‘health-care team’ is implied and it is presumed that patients know and accept
that this will happen. These members include other health professionals who have a legitimate therapeutic interest
in the care of the patient. Medical students cannot be considered legitimate in this sense and are excluded, unless
the patient consents to. In fact, medical students should be considered thirds parties.

Exceptions established by law - these include the notification of infectious diseases, births and deaths, and deaths
reportable to the coroner. In some states, doctors are obliged to notify the relevant registration authority if a health-
care professional, who is a patient, is ill and the community is believed to be at risk; this exception is backed by
immunity from civil action.

Overriding community interest - when community interest overrides that of the patient, disclosure of the patient’s
medical information is not considered a breach of confidentiality. An example is when a doctor advise the police of
a patient they believe should not be driving a motor vehicle, or a psychiatrist believes a patient is a serious threat to
others.

References

• Good Medical Practice – Professionalism Ethics and Law – Cambridge – 1st Edition – pages 74-75

• Medicl Board odf Australia - Good Medical Practice: A Code of Conduct for Doctors in Australia

Last updated:
Time spent: QID:976 2023-2-12

400 of 1943
A mother brings her 6-month-old daughter for advice regarding vaccination of her baby. The child has not received
any vaccine so far because the mother had believed it is more natural, but she was a little concerned after she read
in an article that vaccination is beneficial for children. You explain to her the benefits of the child being vaccinated. At
the end, she decides not to vaccinate her child. Which one of the following is the most appropriate approach in this
situation?

A. Inform child protection services.


B. Call the police as she is putting the baby’s health at risk.
C. Inform the local community council.
D. Respect her decision.
E. Apply for guardianship to the guardianship court.

Incorrect. Correct answer is D


45% answered correctly

Explanation:

Correct Answer Is D

According to current regulations in Australia, vaccination is not compulsory and parents can choose not to vaccinate
their children. Physicians are required to fully explain the benefits and risks of vaccination to parents, and respect
their wishes if they refuse vaccination of their children.

Any option suggesting reporting of such parents to authorities such as child protection services, police, court, etc is
incorrect.

References

• VAIS - Vaccination Awareness

Last updated:
Time spent: QID:990 2023-2-12

401 of 1943
A 6-year-old child is presented with multiple bruises. Based on reasonable grounds, you form the idea that the child
has sustained physical abuse. Which one of the following is the most appropriate next action you should take?

A. Full blood exam.


B. PT, APTT.
C. X-ray.
D. Take photographs of the lesions.
E. Notify the Child Protection Service.

Incorrect. Correct answer is D


45% answered correctly

Explanation:

Correct Answer Is D

When a healthcare professional, based on reasonable grounds, forms a belief that child abuse has occurred,
immediate action should be taken.

The priorities in dealing with child abuse are:

1. To diagnose, treat and document the child injuries


2. To notify and involve the Child Protection Services immediately
3. To provide, when consent is given, a verbal or written report to Child Protection Service and the Police (this is
different from notifying the Child Protection Services on perceived child abuse)

This child does not appear to be in immediate need for treatment as the first priority; therefore, taking photographs
of the lesions to document them is the next best step in management.

NOTE - Notifying the Child Protection Services does not need any consent from the parents, care or guardian. In
fact, it is advisable that the doctor withhold from the parent that he/she has made a notification if it is believed
that the accompanying parent has been involved in child abuse. However, the doctor must establish that consent
has been given (by one of the parents or the child's legal guardian) to perform a clinical examination and to
provide a report (not notification) to Child Protection and the Police. Ideally this should be in writing. If consent is
unobtainable, the child should only be examined if a medical emergency exists.

(Option A, B and C) Full blood exam and coagulation profile would be needed if the bruises were considered to have
been caused by a medical condition with hemorrhage tendency. These tests might later be indicated for further
assessment after the bruised are proved not to be a consequence of injuries. X-ray exam might be considered
somewhere during assessment process if indicated. It is not a priority now.

(Option E) Notification to child protection services comes next after documenting the injuries and treating them.

References

• RCH - Child abuse

• RACGP - The White Book: Child abuse

Last updated:
Time spent: QID:1035 2023-2-12

402 of 1943
A 13-year-old girl comes to your GP clinic asking for options regarding abortion. She left home 12 months ago and is
living with his 18-year-old boyfriend. Last evening, she went to a party where she became drunk and was forced to
have unwanted sex with a man. She wants to know if she could have an abortion in case she gets pregnant. On
examination, there is no evidence of trauma. Which one of the following should be notified first?

A. Sexual assault service.


B. The police.
C. Her parents.
D. School.
E. Child protection services.

Incorrect. Correct answer is E


45% answered correctly

Explanation:

Correct Answer Is E

According to current legislation in Australia, doctors, nurses and midwives are obliged to lodge a mandatory report to
the child protection services whenever they have formed a ‘reasonable belief’’ that a child under 18 years has been or
is being sexually abused or assaulted. This holds valid for all peoples younger than 18 years of age regardless of
whether they are dependent or independent (mature) minors; therefore, the next step in management in this scenario
is reporting to child protection services.

The doctor, to whom a mature minor discloses the sexual assault, should initially inform the child of legal
requirements of mandatory reporting and the limitations on doctor-patient confidentiality.

The key point to appreciate and fully absorb is the legal concept of ‘child’ in the term ‘mature child’. No matter how
mature or independent they might be, they are children as long as they are younger than 18 years and reporting any
alleged abuse/assault is a ‘must’.

NOTE - This is different from incidences when a mature child voluntarily involves in a sensual sexual relationship.
In such cases no mandatory reporting is required if the child is 13 years or older.

When a belief of child sexual abuse or assault is formed, the practitioner should discuss the mandatory reporting
requirements with the mature minor and include the following in the discussion:

How the minor would like a report to be progressed


Their preferences for alerting (or not) their carers/parents
Their preference for informing (or not) the Police
If the minor views themselves at any ongoing risk of (further) sexual abuse
Any alerts for the child protection service or the police
Any further information the mature minor would like to have included in the report

Child protection services should make a mandatory report to the police; however, if the mature minor in discussion
with the mandatory reporter, determines they do not want the police to be involved, this information needs to be
included in the report. Child protection services and the police will take this into account whenever there is no
ongoing risk to this child or any other child.

NOTE - It is always appropriate and advisable to persuade or encourage a mature minor to inform her
parents/cares or to allow the doctor to do so on their behalf, but if the mature refuse to do so, it should be
avoided as it unnecessarily breaches the doctor- patient confidentiality.

Her school (option D) is not required to be informed in any case; even if the child is dependent and not mature.

403 of 1943
Sexual assault services (option A) are dedicated bodies that provide the victims of sexual assault with counselling
services, forensic examinations and psychological care. Although it is advisable that sexual assault victims be
counselled about the benefits of such services and offered referral, no mandatory reporting to sexual assault
services is required.

TOPIC REVIEW

A child or minor is a person who is younger than the age of majority. In Australia the age of majority is 18 years. This
is the age at which citizens can exercise all the civil rights available to Australians; however, a child of or over 16
years of age can give consent for medical treatment. In certain situations, a child younger than 16 years of age may
give consent as well if he/she is considered competent by Gillick competence rule.

Based on Gillick competence principle, a minor (<16 years, but not younger than 13 years) is considered competent
to consent to treatment if:

she/he lives independently of her/his parents (i.e. emancipated minor), AND


she/he appears to have sufficient understanding and intelligence to enable her or him to understand fully what
is proposed and the risks and benefits.

If these criteria are met, a minor can consent to treatment without any requirements to informing a parent or
guardian or obtaining consent from them.

To put it in a more clear and straightforward way, always approach a minor as a consenting adult if:

1. she/he is 13 years of age or older, AND


2. she/he lives independent of his/her parents, AND
3. she/he appears to fully understand the situation, treatment options and risks and benefits.

References

• Western Australia Department of Health - Guidelines for the mandatory reporting of child sexual abuse when the
child is a mature minor

• NSPCC Learning - Gillick competency and Fraser guidelines

Time spent: QID:1036 Last updated:


2023-2-12

404 of 1943
You are a medical officer in a tertiary hospital. A patient is about to undergo abdominal surgery. When the patient is
being transferred to the operating theatre, the nurse informs you that the consent for the surgery has not yet been
obtained. Which one of the following is the most appropriate step regarding consent?

A. Send the patient to the operating room and ask the anesthesiologist to obtain the consent.
B. Obtain the consent yourself and send the patient for the surgery.
C. Download information from the internet and discuss it with the patient and obtain consent.
D. Call the treating surgeon and ask him to obtain consent.
E. Ask the attending nurse to obtain consent from the patient on his way to the operating theatre.

Incorrect. Correct answer is D


45% answered correctly

Explanation:

Correct Answer Is D

The ultimate responsibility of obtaining consent is with the clinician directly in charge of the treatment; therefore, the
operating surgeon should be called for obtaining consent from the patient. However, it is possible for that clinician to
delegate authority to other healthcare professionals,such as more junior members of staff. The proviso for such
delegation is that the person obtaining consent must be fully equipped to deal with the consent process.

References

• Australian Medico-legal Handbook – Elsevier Australia (2008) – page 68

Last updated:
Time spent: QID:605 2023-2-12

405 of 1943
A 13-year-old girl presents to your clinic asking for termination of pregnancy. Three weeks ago, she was in a party
where she was coerced to have unwanted sex with a stranger. Yesterday, using a home pregnancy test, she
realized she is pregnant. She is a school girl and lives with her parents. Which one of the following is the most
appropriate next step in management?

A. Inform the Child Protection Service.


B. Inform her parents of the event.
C. Terminate the pregnancy.
D. Inform the police.
E. Inform the Sexual Assault Services.

Correct
45% answered correctly

Explanation:

Correct Answer Is A

According to current legislation in Australia, doctors, nurses and midwives are obliged to lodge a mandatory report to
the child protection services whenever they have formed a ‘reasonable belief’’ that a child under 18 years has been or
is being sexually abused or assaulted.

NOTE - Mandatory reporting in cases of sexual assault DOES NOT take into account whether the child is
dependent or independent (mature). Gillick competence rule does not exclude any child from being reported to
child protection services whatsoever. All children younger than 18 years are subject to mandatory reporting by
the treating doctors, nurses and midwives, teachers, principals and the police.

Gillick competence rule, on the other hand, applies for termination of pregnancy, and a Gilick-competent minor can
request an abortion without parental consent.

Based on Gillick competence principle, a minor (<16 years, but not younger than 13 years) is considered competent
to consent to treatment if:

she/he lives independently of her/his parents (i.e. emancipated minor), AND


she/he appears to have sufficient understanding and intelligence to enable her or him to understand fully what
is proposed and the risks and benefits.

If these criteria are met, a minor can consent to treatment without any requirements to inform a parent or guardian
or obtaining consent from them.

To put it in a more clear and straightforward way, always approach a minor as a consenting adult if:

he/she is 13 years of age or older, AND


he/she is independent of his/her parents, AND
he/she appears to fully understand the situation, treatment options, risks and benefits.

(Options B and C) This child does not fulfill criteria for termination of pregnancy without parental consent or
informing. Although she is 13 (minimum age required), she lives with her parents and cannot consent to termination
of pregnancy. Regulations related to termination of pregnancy in a minor varies in different states but generally
parents should consent or at least be informed (according to the state regulations); however, a child aged 13 or more
can still refuse her/his parents being informed. As the healthcare professional involved in the treatment, you should
respect her/his wish in this regard. No mandatory reporting to the parents is required. The decision as to whether the
parents are informed and how to be so is the responsibility of the child protection service and not of the primary
healthcare provider.

406 of 1943
If she is willing to terminate the pregnancy and does not want her parents/carer/guardian involved, a court order,
often issued by Children’s Court is required before proceeding to termination of pregnancy.

(Option D) Informing the police is only acceptable if the victim consents to, or when there is immediate threat to the
victim’s health. Informing the police is not helpful if the child is not willing to disclose the assault to the police
because police cannot take action if there is no evidence; however, in most cases there is close collaboration
between the police and the child protection service.

(Option E) Sexual assault services are dedicated bodies that provide the victims of sexual assault with counselling
services, forensic examinations and psychological care. Although it is advisable that sexual assault victims be
counselled about the benefits of such services and offered referral, no mandatory reporting to sexual assault
services is required.

References

• Western Australia Department of Health - Guidelines for the mandatory reporting of child sexual abuse when the
child is a mature minor

• NSPCC Learning - Gillick competency and Fraser guidelines

• RACGP - AFP - Disclosures of sexual abuse: what do you do next?

Time spent: QID:1037 Last updated:


2023-2-12

407 of 1943
A 17-year-old girl presents to your clinic requesting an abortion at 12 weeks pregnancy. Her pregnancy is the result of
a rape that happened a while back in a party while she was drunk. You are against abortion morally and think that
abortion can be only ethical in cases of severe congenital anomalies. Which one of the following should the most
appropriate step in management of this patient?

A. Refer to another GP for a second opinion.


B. Inform her parents.
C. Inform the Sexual Assault Services.
D. Refer to a tertiary hospital for further management.
E. Perform the abortion despite your will.

Correct
45% answered correctly

Explanation:

Correct Answer Is A

The very first step in management of this patient is reporting the incidence of sexual assault to child protection
services. Any options suggesting mandatory reporting to child protection services would be the most appropriate
option.

According to current legislation in Australia, doctors, nurses and midwives are obliged to lodge a mandatory report to
the child protection services whenever they have formed a ‘reasonable belief’’ that a child under 18 years has been or
is being sexually abused or assaulted WITHOUT taking into account whether the child is dependent or independent
(mature). Gillick competence rule does not exclude any child from being reported to child protection services
whatsoever if the case is sexual assault or abuse. All children younger than 18 years are subject to mandatory
reporting by the treating doctors, nurses and midwives, teachers, principals and the police in such incidences.

The main point of the scenario appears to be the conflicting ethical issues regarding termination of the pregnancy
between the doctor and the patient. In instances where such conflicts exist and the doctor does not feel comfortable
in dealing with consultation regarding abortion, he/she should advise the patient to see another GP or a women’s
health center as early as possible (ideally before 12 weeks gestation).

Termination of pregnancy should be performed by approved clinics and abortion service providers.

NOTE - Beyond a specific gestational age (20 weeks is South Australia, 22 weeks in Queensland, 18-20 weeks in
NSW, etc.) the termination of pregnancy might be subject to further review and assessment e.g., requiring approval
from a panel appointed by the Minister of Health in South Australia after 22 weeks.

(Option B) This girl is older than 16 years and can consent to most medical treatments/procedures including
termination of pregnancy. No parental consent is required to do so, nor is there any obligation to inform them.

(Option C) Sexual assault services are dedicated bodies that provide the victims of sexual assault with counselling
services, forensic examinations and psychological care. While offering referral is appropriate, informing them
without the patient’s consent is a breach of doctor-patient confidentiality.

(Option D) Referral for further management is considered after discussion with the patient about the procedure
and obtaining informed consent for referral after the patient is fully informed of the risks and potential
complications. While the doctor is against termination of pregnancy, such discussion should be taken over by a third
party (e.g., another doctor) who is impartial and does not feel uncomfortable with the issue of termination of
pregnancy.

(Option E) The doctor is not to obliged to become involved in abortion if it is against his/her ethical principles.

408 of 1943
References

• Termination of pregnancy: Information and legal obligations for medical practitioners

• Western Australia Department of Health - Guidelines for the mandatory reporting of child sexual abuse when the
child is a mature minor

• NSPCC Learning - Gillick competency and Fraser guidelines

• Angloinfo - Termination of pregnancy/ abortion

Last updated:
Time spent: QID:1038 2023-2-12

409 of 1943
Melisa, 14 years old, presents to your practice for termination of pregnancy at 13 weeks gestation. She left home at
the age of 12 years and has been living with her 20-year-old boy friend for the last eight months against her parents’
wishes. Her boyfriend left her after he knew she is pregnant. Which one of the following is the most appropriate next
step in her management?

A. Inform the Child Protection Service.


B. Refuse her request because she is underage.
C. Refer her for termination of pregnancy after consultation and obtaining informed consent.
D. Tell her that her parents should consent to pregnancy termination.
E. Contact the police because the sexual relationship was illegal due to the age difference.

Incorrect. Correct answer is C


45% answered correctly

Explanation:

Correct Answer Is C

A child or minor is a person who is younger than the age of majority. In Australia, the age of majority is 18 years. This
is the age at which citizens can exercise all the civil rights available to Australians; however, a child of or over 16
years of age may give consent for medical treatment. In certain situations, a child younger than 16 years of age may
give consent.

Based on Gillick competence principle, a minor (<16 years, but not younger than 13 years) is considered competent
to consent to treatment if:

She/he lives independently of her/his parents (i.e. emancipated minor), AND


She/he appears to have sufficient understanding and intelligence to enable her or him to understand fully
what is proposed and the risks and benefits.

If these criteria are met, a minor can consent to treatment without any requirements to informing a parent or
guardian or obtaining consent from them.

In summary, according to Gillick competence rule, a person under the age of 16 years is still able to give consent for
medical treatments including operative procedure on him/her providing that:

He/she is 13 years of age or older, AND


lives independent of his/her parents, AND
he/she appears to fully understand the situation, treatment options, risks and benefits.

Although the relationship is illegal, because she is under the age of 16 years, they have been living together with their
own consent, and the boyfriend has not been a teacher, relative, or someone in whom the child puts her trust
because of their position. Providing this is the case, the police in Australia or the various child protection authorities
would not normally take action against either the girl or her parent. Informing the Child Protection Services would be
the correct answer and the next best step in management if the child was sexually assaulted.

Unless she gives permission to do so, it would be inappropriate for her parents to be advised of the pregnancy and
unnecessary for them to give consent to the procedure.

References

• Termination of pregnancy: Information and legal obligations for medical practitioners

• NSPCC Learning - Gillick competency and Fraser guidelines

410 of 1943
• Western Australia Department of Health - Guidelines for the mandatory reporting of child sexual abuse when the
child is a mature minor
Last updated:
Time spent: QID:1039 2023-2-12

411 of 1943
Nicole, 72 years, is one of your patients, who presented with a breast lump. Physical exam finding of an inverted
nipple and pea-au-de-orange made you highly suspicious of breast cancer. You ordered a mammogram, the result of
which confirmed the diagnosis. Her eldest son calls and tells you that he thinks that her mother has breast cancer
and asks you that you do not tell her mother of diagnosis, if it turned out to be cancer, because she might become
depressed. Which one of the following would be the most appropriate action in this situation?

A. Tell her son that you should meet Nicole alone and you have to inform her of the diagnosis anyway.
B. Tell her son to bring all family members for a family meeting.
C. Arrange an appointment with the son for further discussion.
D. Call Nicole and ask her to bring a family member with her for the appointment.
E. Follow the son’s wish, as breaking the news might put Nicole at risk.

Correct
45% answered correctly

Explanation:

Correct Answer Is A

As a rule, a patient is entitled to be informed of the diagnosis as soon as it is made. The information should not be
withheld on requests of relatives and carers. In this case, you should meet Nicole alone and you must inform her of
the diagnosis anyway.

It is Nicole’s decision whether to share any information about her diagnosis, health condition, or treatments
proposed.

(Options B, C and D) Any option suggesting family meetings or arranging a meeting with the requesting relative is
definitely wrong. In fact, telling about the patient's medical information to a third party, irrespective of their closeness
or intimacy is an act of breach of doctor-patient confidentiality. Nicole does not need to bring a family member
unless she wishes so.

(Option E) There may be circumstances under which providing information could cause the patient harm, and the
treating doctor decides to withhold information from the patient. This is frequently referred to as therapeutic
privilege. Particular information may be withheld where the practitioner believes, on reasonable grounds, that
providing it may damage the patient’s health. The responsibility is on the practitioner to show that providing the
information would be reasonably likely to cause significant harm. This decision is made in very limited situations by
the treating doctor, not on a relative’s or carer’s request.

References

• NHMRC - General Guidelines for Medical Practitioners on Providing Information to Patients

• Cameron Stewart; Ian Kerridge; Malcom Parker; The Australian Medicolegal Handbook; Churchill Livingstone
Elsevier – pages 74-75

Last updated:
Time spent: QID:1040 2023-2-12

412 of 1943
You have a patient with severe multiple sclerosis that is advanced and progressive, who now has developed renal
failure secondary to diabetes. The patient is alert and has elected to put the DNR order in place at her own
discretion. Today, he has presented for follow-up and you notive that he has a markedly elevated serum potassium
of 8 mmol/L. Which one of the following is the most appropriate management of this patient?

A. Dialysis cannot be done because of the DNR order.


B. You can do the dialysis if the DNR is reversed for the procedure.
C. Proceed with the dialysis; ignore the DNR order.
D. Give insulin and glucose until the DNR status is discussed with the family.
E. Seek a court order to overrule the DNR order.

Incorrect. Correct answer is C


45% answered correctly

Explanation:

Correct Answer Is C

A "Do Not-Resuscitate' (DNR) order is very specifically defined as refraining from resuscitative efforts, such as chest
compression, antiarrhythmic medications (e.g. amiodarone, atropine, adrenaline, etc.), and electrical cardioversion in
the event of the patient’s cardiopulmonary arrest. A DNR order has nothing to do with any other forms of care the
patient is receiving.

DNR order has no impact on the use of dialysis, and DNR order should be ignored when assessing the patient for
dialysis. Hyperkalemia is life-threatening. It is not reasonable to use an inferior therapy such as insulin and glucose
or resins for management of hyperkalemia when dialysis is indicated. In this scenario, DNR order should be ignored
and dialysis performed after obtaining the patient’s consent. The patient, however, still has every right to refuse the
dialysis. This is different from the DNR order.

This patient is awake, alert and able to understand his own medical condition; therefore, the patient’s family is not
relevant in the process of decision making if the patient has the capacity to understand his or her own medical
condition.

References

• Merck Manual - Do-Not-Resuscitate (DNR) Orders and Physician Orders for Life-Sustaining Treatment (POLST)

• ETHICS IN MEDICINE University of Washington School of Medicine - Do Not Resuscitate (DNAR) Orders

Last updated:
Time spent: QID:1083 2023-2-12

413 of 1943
Robert, aged 68 years, is in the operating theatre undergoing a colon resection due to colon cancer. Prior to the
surgery, the risks of the surgery was fully explained to him. After discussion with the treating surgeon, he decided to
sign a “Not for Resuscitation (NFR)” form, if anything happens to him during the surgery or afterwards. During the
operation, he starts bleeding suddenly and profusely and in a matter of seconds and before anything can be done,
he becomes pulseless and his heart stops beating. The blood pressure is not recordable. Which one of the following
is correct regarding this situation?

A. Transfuse fluids and blood products, and resuscitate him as this has occurred as a complication of the
surgery.
B. Do not resuscitate because the patient’s blood pressure and pulse are not recordable and he has
arrested.
C. You can give fluid, blood products and medications, but no chest compression or electric cardioversion.
D. Only administer intravenous fluids and blood products to restore circulating volume but do not perform
chest compression or cardioversion.
E. DNR orders are not valid when they are not related to the underlying disease.

Incorrect. Correct answer is B


45% answered correctly

Explanation:

Correct Answer Is B

It is common to have patients presenting for surgery with a 'Do Not Resuscitate' (DNR) order written in their files.
Physicians and patients suffer from misconception about the potential benefits and harms of resuscitation in the
operating room (OR), and even definition of resuscitation in the OR requires clarification prior to surgery.

Cardiopulmonary resuscitation (CPR) in the OR has a very different prognosis than CPR in other areas. The
percentage of patients resuscitated in the OR, who return to their pre-CPR functioning, is 50-80% versus 4-14% for
those patients who are resuscitated in other areas. This difference is due to several factors such as the fact that the
arrest is always witnessed in the OR and the cause is often known that allows prompt effective intervention targeted
at the cause.

Another reason for this difference is that causes of arrest in the OR are often reversible effects of anesthesia or
hemorrhage, and not due primarily to the patient’s underlying disease. This fact makes physicians even more
uncomfortable with DNR orders in the OR because they may feel that their actions has led to the arrest, and they are
ethically obliged to resuscitate the patient, even if the patient has clearly expressed wishes to the contrary. This is a
significant misunderstanding. Physicians should be aware of the fact that competent patients or their appropriate
surrogates have the right to refuse medical procedures and care, even if the care is to counteract the effect of
previous medical intervention.

In this case scenario, the patient has been fully informed of the risks of the operation, yet he insists on a DNR order
to be in place. DNR means if the patient suffers a cardiopulmonary arrest, represented by undetectable blood
pressure and pulse and inability to breathe spontaneously, resuscitative measures such as chest compression,
electrical cardioversion, acute administration of antiarrhythmic drugs such as epinephrine, or atropine must NOT be
performed in respect to the patient’s wish, even if the arrest could be reversed rapidly and effectively. This holds true
even if the arrest is just a simple consequence or complication of the procedure.

Just using intravenous fluids and blood products without establishing artificial circulation, at least initially, by means
such as chest compression is futile and not appropriate.

Again, once the patient arrests, measures considered as a part of basic life support (BLS) or advanced life support
(ALS) such as chest compression, assisted ventilation, use of medications, or electric cardioversion cannot be used
for the patient.

References

414 of 1943
• ETHICS IN MEDICINE - University of Washington School of Medicine - Do Not Resuscitate (DNAR) Orders
Last updated:
Time spent: QID:1084 2023-2-12

415 of 1943
A patient with Alzheimer disease from a skilled-nursing facility is admitted to the hospital for severe upper
gastrointestinal bleeding. The bleeding has not stopped despite multiple transfusions, octreotide, proton ump
inhibitors, and endoscopy. He needs monitoring and evaluation in an intensive care unit. The patient has a DNR order
in place by his legal guardian. Which one of the following is correct regarding this situation?

A. Reverse the DNR order and transfer to the ICU as needed.


B. He cannot be DNR and be in the ICU.
C. Transfer to the ICU as needed; it is okay to be DNR in the ICU.
D. He can go to the ICU with the DNR but cannot go for surgery.
E. He can go to the ICU with the DNR but cannot be intubated.

Incorrect. Correct answer is C


45% answered correctly

Explanation:

Correct Answer Is C

A ‘Do Not Resuscitate’ or DNR order means no cardiopulmonary resuscitative efforts if the patients suffers a
cardiopulmonary arrest. There is no other automatic limitation in therapy besides this. DNR orders are often
confused with generalized withholding of care such as transfusion or ICU admission. DNR orders are not the same
as stopping the active management of the patient. DNR orders do not automatically mean that the patient is
immediately terminal or that there is an automatic assumption of palliative care only. This patient should be
transferred to the ICU, but in case CPR is indicated, it should be withheld.

Intubation, surgery, and any other form of treatment can be given to this patient as long as he is not in
cardiopulmonary arrest.

NOTE - When the patient has a cardiopulmonary arrest, it is always presumed that the patient has given consent
to CPR unless the patient has specifically and expressively refused this therapy in advance. In other words, you
do not need to ask for consent from anybody regarding CPR unless the patient is already DNR.

References

• Merck Manual - Do-Not-Resuscitate (DNR) Orders and Physician Orders for Life-Sustaining Treatment (POLST)

• ETHICS IN MEDICINE - University of Washington School of Medicine - Do Not Resuscitate (DNAR) Orders

Last updated:
Time spent: QID:1085 2023-2-12

416 of 1943
Sophia is an 82-year-old patient of yours, who has been under your care for the past eight years due to chronic
obstructive pulmonary disease (COPD). A few months back she informed you that she had written a “Do Not
Resuscitate (DNR)” order. Today, he is brought to you by her family member with complaint of severe dyspnea. She
looks drowsy and confused. Her family members appear quite concerned and ask you to treat her by any means.
Which one of the following is the most appropriate action?

A. Arrange a family meeting.


B. Do not give any treatment as she has a DNR order in place.
C. Admit her and assess for any need for ICU care.
D. Refer the case to guardianship tribunal.
E. Insert intravenous line and nasogastric tube and nothing more.

Incorrect. Correct answer is C


45% answered correctly

Explanation:

Correct Answer Is C

DNR means that patient should not receive cardiopulmonary resuscitation using chest compression, electric
cardioversion, or acute administration of antiarrhythmic medications when he or she sustains cardiopulmonary
arrest. This means absence of pulse, blood pressure and inability to breathe spontaneously which is not the case in
this scenario.

A common misunderstanding about DNR is that it is mistakenly interpreted as withholding appropriate treatment
from patients at end-stage conditions. Another common misunderstanding is that a person who is DNR should not
get other aspects of an appropriate medical treatment and care. such as biopsies, surgeries, or dialysis.

Since this patient is not clinically dead yet, DNR does not apply, and appropriate treatment, which is admitting her
and assessment for the need for any intensive care in the ICU, should be considered.

NOTE - When the patient has a cardiopulmonary arrest, it is always presumed that the patient has given consent
to CPR unless the patient has specifically and expressively refused this therapy in advance. In other words, you
do not need to ask for consent from anybody regarding CPR unless the patient is already DNR.

(Option A) Since this patient is not subject to DNR yet, and the wishes of family are not against those of the patient, a
family meeting will be unnecessary.

(Option D) Referral to Guardianship Tribunal would be the correct answer if there was any conflict between the
patient’s best interests and the family members or between family members with equal levels of decision making
regarding the patient’s care.

(Option E) Just insertion of intravenous line and nasogastric tube, and not giving the patient appropriate medical
management is incorrect because neither the patient nor her family has requested withdrawal of treatment.

References

• Merck Manual - Do-Not-Resuscitate (DNR) Orders and Physician Orders for Life-Sustaining Treatment (POLST)

• ETHICS IN MEDICINE - University of Washington School of Medicine - Do Not Resuscitate (DNAR) Orders

Last updated:
Time spent: QID:1086 2023-2-12

417 of 1943
An 86-year-old woman with longstanding history of chronic obstructive pulmonary disease (COPD) has an advance
directive of DNR (do not resuscitate) order in place. She is now in the emergency department with a femoral neck
fracture sustained at home. She is drowsy and disoriented. Her son and daughter are in the emergency department
asking you to do whatever it takes to treat her. Which one of the following should be your next appropriate action?

A. Inform her son and daughter that she will be given all necessary treatment.
B. Wait until she regains her consciousness and discuss the matter with her.
C. Inform your senior doctor.
D. Inform her son and daughter that there is nothing you can do as she is DNR.
E. Refer the case to Guardianship Tribunal.

Correct
45% answered correctly

Explanation:

Correct Answer Is A

A "Do Not Resuscitate" (DNR) order means if the patient dies i.e. cardiopulmonary arrest, the treating physician(s)
does not perform chest compressions, attempt electrical cardioversion, or use acute antiarrhythmic medications. In
other words, DNR is defined as not attempting any treatment once the patient is dead, indicated by loss of pulse,
blood pressure and spontaneous breathing. DNR orders cannot be considered valid and should not be followed
unless the patient loses all of these.

The most common misunderstanding about DNR is that being DNR must mean the patient is preterminal and is just
about to die. Another common misunderstanding is that a person who is DNR should not get other aspects of an
appropriate medical treatment and care such as biopsies, surgeries, or dialysis.

DNR does not mean that the patient should let be dead just because a DNR order is in place. DNR just means that
the treating physician should consider death as the end point of life and take no further actions to reverse it.
Therefore, DNR is not valid until the patient is clinically dead evident by loss of pulse, blood pressure and the ability
to breath spontaneously.

This patient has a DNR order in place, but she is still clinically alive. She has a femoral neck fracture and the DNR
does not preclude an appropriate medical management of her current condition. She is not conscious now, but her
next of kin can decide on her behalf. As they have given you the consent to proceed with treatment at your discretion,
the next appropriate action is telling them that you will give her all necessary treatments in her best interest.

There is no need to inform the senior doctor, nor referring the case to Guardianship Tribunal, as the path is clear and
there is no conflict over the patient’s best interest in this scenario.

References

• Merck Manual - Do-Not-Resuscitate (DNR) Orders and Physician Orders for Life-Sustaining Treatment (POLST)

• ETHICS IN MEDICINE - University of Washington School of Medicine - Do Not Resuscitate (DNAR) Orders

Last updated:
Time spent: QID:1087 2023-2-12

418 of 1943
An 82-year-old man is admitted to the hospital with complicated bowel obstruction and booked for emergecy
surgery. He has a DNR order in place after he had a previous admission for another reason last year. He still wishes
to be DNR. What should you do about the surgery?

A. No surgery can be done on a patient who is DNR.


B. Reverse the DNR order for the surgery.
C. DNR is acceptable only if the surgery does not require intubation.
D. Surgery is acceptable while DNR if an additional consent is signed.
E. DNR does not preclude surgery; proceed with the surgery.

Incorrect. Correct answer is D


45% answered correctly

Explanation:

Correct Answer Is D

A 'Do not resuscitate order' (DNR) is not meant to be a generalized limitation on all forms of therapy. You can still
intubate patients even if they are DNR.

DNR just means you are taking death as the endpoint of giving treatment. In the event of cardiopulmonary arrest, do
not give the additional therapy of cardiopulmonary resuscitation, defibrillation, and chest compressions. DNR is not a
general equivalent for withholding any other forms of therapy.

This patient needs surgery, and a DNR does not preclude this; therefore, proceeding with the surgery is the most
appropriate management option. However, like any other medical procedure, consent should be obtained from the
patient for the surgery if the patient is alert and competent to understand the treatment options, their risks and
complications and the consequences that might arise if treatment is withheld.

If the patient insists that DNR order be in place despite being informed of the consequences, no CPR should be
attempted in the event of cardiopulmonary arrest during the surgery or in the perioperative period.

References

• Merk Manual - Do-Not-Resuscitate (DNR) Orders and Physician Orders for Life-Sustaining Treatment (POLST)

• ETHICS IN MEDICINE University of Washington School of Medicine -- Do Not Resuscitate (DNR) Orders

Last updated:
Time spent: QID:1088 2023-2-12

419 of 1943
A 67-year-old man presents to the emergency department with severe chest pain. An ECG is consistent with inferior
myocardial infarction. Initial management with aspirin, oxygen, and morphine is done. While arrangements are made
to transfer the patient for percutaneous coronary intervention, he becomes pulseless and unconscious. Cardiac
monitoring shows ventricular fibrillation. He is immediately managed with defibrillation and regains his
consciousness and pulse again. He then asks you to not resuscitate him again if anything happened to him. Which
one of the following is the most appropriate approach?

A. Assess his mental competence.


B. Write “do not resuscitate” order in his file.
C. Overrule his request as this decision is irrational.
D. Ask his wife to convince him otherwise.
E. Refer the case to the Guardianship Court.

Correct
45% answered correctly

Explanation:

Correct Answer Is A

Every competent adult patient has full control over his/her body. They may refuse a medical treatment even though it
is life-saving. They may ask for withholding or withdrawal of treatment, including cardiopulmonary resuscitation.
When such requests are faced, the next best step is assessing the patient’s mental competence. Once the patient is
found competent, their wish should be respected and followed.

(Option B) A ‘do not resuscitate’ order can be put in place after he is found to be mentally competent and it is
assured that he is fully informed of the consequences.

(Option C) By overruling a competent patient’s wish and doing the resuscitation, the crime of battery is committed,
and the perpetrator may face criminal charges.

(Option D) While a competent adult patient can make decisions for himself/herself asking his/her relatives to
convince them otherwise is not correct. This is an issue related to them and they need to address it between
themselves at their discretion.

(Option E) The rules and regulations on this issue are quite clear and straightforward and there is no need to involve
the Guardianship court or any other authorities.

References

• Australian Medico-legal Handbook – Elsevier Australia (2008) – page 131

Last updated:
Time spent: QID:606 2023-2-12

420 of 1943
A 76-year-old lady is brought to the Emergency Department by her daughter with fractured femoral neck. She is
unconscious and the “Do Not Resuscitate (DNR)” status is unknown. Based on evaluations, she is in need for surgery
for the fracture. Which one of the following is the most appropriate action in this situation?

A. Ask if there is a DNR order in place first.


B. Proceed to the surgery without consent.
C. Ask for consent from her daughter.
D. Ask for consent from guardianship tribunal.
E. Refer her for palliative care.

Incorrect. Correct answer is C


45% answered correctly

Explanation:

Correct Answer Is C

A "Do Not Resuscitate (DNR) order means if the patient dies evident by cardiopulmonary arrest, the doctor does not
perform chest compressions, attempt electrical cardioversion, or use acute antiarrhythmic medications. In other
words, DNR is defined as not attempting any treatment once the patient is ‘dead’, indicated by loss of pulse, blood
pressure and breathing. DNR orders cannot be considered valid and should not be followed unless the patient losses
all three components (pulse, blood pressure, breathing).

The most common misunderstanding about DNR is that being DNR must mean the patient is pre-terminal and is just
about to die.

Another common misunderstanding is that a person who is DNR should not get other aspects of an appropriate
medical treatment and care such as biopsies, surgeries, dialysis, etc.

DNR does not mean that the patient should let be dead just because a DNR order is in place. DNR just means that
the treating physician should consider death as the end point of life and attempts no further actions to reverse it. So
DNR is not valid until the patient is clinically dead evident by loss of pulse, blood pressure and the ability to breathe
spontaneously.

In this case scenario the patient is unconscious, not dead; therefore, asking if there is a DNR order in place (option A)
will be inappropriate because even so, it does not apply in this situation. The patient, however, needs surgery and
consent is required for that matter. Since the patient is unconscious, her next of kin (daughter) can give consent to
the surgery on her behalf.

(Option B) Proceeding to the surgery without consent could be the correct answer in emergency cases, where no one
is present to consent on the patient's behalf, and to his/her best interest, and if failing to take prompt action would
put the patient’s health at serious risks. In such circumstances it is presumed that the care givers would consent to
the patient’s best interest if he or she was present.

(Option D) Referral of the case to guardianship tribunal is appropriate only if there is conflict between family
members or care givers or when the family wish is in conflict with the patient’s best interest.

(Option E) Palliative care would be the correct answer if the patient or his/her caregiver/next of kin based on sound
grounds has refused active treatment.

References

• Merck Manual - Do-Not-Resuscitate (DNR) Orders and Physician Orders for Life-Sustaining Treatment (POLST)

• ETHICS IN MEDICINE - University of Washington School of Medicine - Do Not Resuscitate (DNAR) Orders

421 of 1943
Last updated:
Time spent: QID:1089 2023-2-12

422 of 1943
A 42-year-old patients of your, who suffers schizophrenia and is on controlled-release ziprasidone, presents for a
follow-up. During the visit you notice a mole on his face that is highly suspected to be melanoma. You explain that
the mole should be excised and send for pathology and further follow-up will be required, but he refuses and says
the mole will heal on itself. Which one of the following is the most appropriate next step in management?

A. Tell him about the risks of melanoma and benefits of treatment.


B. Formal mental state examination.
C. Refer him for a psychiatric opinion.
D. Admit him involuntarily.
E. Refer the case to Mental Health Tribunal.

Correct
45% answered correctly

Explanation:

Correct Answer Is A

At common law and under some statutes, adults (people over 18) are presumed to be competent, although it is
possible to rebut the presumption by showing that an adult lacks competence. A functional test, settled on by most
common law jurisdictions, is used to assess the competency of a patient by healthcare professionals to examine the
ability of a patient to consent to or refuse a specific treatment. A patient is competent if he/she is able to:

understand and retains treatment information


believe the information
weigh the information and reach a decision and communicate their decision

NOTE - even if the reasoning is not sound or even delusional, the doctor cannot consider the patient incompetent to
make decisions. Reasoning is a very important indicating the patient's competency even if the reasoning is irrational
or delusional to the treating doctor. The following examples demonstrate how the capacity to reason advocates the
patients rights for autonomy:

(Adult: Refusal of Treatment) [1994] 1 WLR 290 concerned a patient in the Broadmoor Asylum who refused consent to
having his gangrenous leg amputated. The doctors questioned his competence to make a decision about treatment,
given that he was suffering from a mental illness. The judge defined capacity as a sufficient understanding of ‘the
nature, purpose and effects of the proffered [treatment]’. The mechanics of understanding were split into three stages:

comprehending and retaining treatment information


believing the information
weighing it among other factors to reach a decision.

As the patient was able to complete these three steps he was found to have capacity, even though he was suffering
from schizophrenia and the delusional belief that he was a doctor.

(Adult: Refusal of Medical Treatment) [2002] EWHC 429, where a ventilator-dependent patient in an intensive care unit
sought to refuse treatment and be allowed to die. Some of the patient’s doctors could not accept her decision because
they feared that she was experiencing a ‘psychological regression’ brought on by her level of disability, her reaction to
being totally dependent on others, her anger, and the effect of being in an intensive care unit rather than a specialist unit.

The arguments against competence were rejected by the court. There was no evidence of psychological regression, and
the patient’s lack of experience in rehabilitation did not go to the issue of whether she understood the nature and effect
of refusing treatment. It was said that doctors should not confuse the question of capacity with their own emotional
reaction to the patient’s decision.

In contrast, incompetence is implied when the patient cannot reason for their refusal of treatment:

423 of 1943
[1997] 2 FLR 426 concerned a woman who had consented to an emergency cesarean section but could not consent to
the insertion of a needle for the preliminary anaesthetic because of an acute needle phobia. The court found that her
phobia made her incompetent because it prevented her from being able to reason through the issue of consenting to the
anesthetic.

This patient has been offered treatment for a suspected melanoma and falsely believes that he does not need
treatment. In such situation, the most important step is discussing with him the risks melanoma can pose to his
health, treatment options and material risks associated with the treatment. He should then be able to believe the
information, weigh them and reach a decision and communicate his decision.

(Options B and C) This patient should not be assumed incompetence. However, if the patient, after discussion, still
refuses treatment against his best interest competence should be assessed comprehensively. This may include:

Assessment of the patient’s understanding and beliefs through discussion with them
Formal tests of cognitive capacity (such as the Standardized Mini-Mental Status Examination)
Clinical psychological or neuropsychiatric assessment
Consideration of corroborative history from other people; for example, family members and health
professionals such as the patient’s general practitioner

(Option D) Involuntary admission is the appropriate option if the patient is considered to be at immediate risk of self
harm or harm to others, or seem that due to a psychiatric condition is not capable of taking care of himself/herself.
None of these conditions are present and involuntary admission is not appropriate.

(Option E) Referral to Mental Health Tribunal might be considered for patients when an order for involuntary
treatment is required, and if they are found to be incompetent for making decisions.

References

• The Australian Medicolegal Handbook; pages 80-85

• MJA - Consent, capacity and the right to say no

Time spent: QID:1127 Last updated:


2023-2-12

424 of 1943
A 38-year-old man, who is a known case of schizophrenia and currently well-controlled on ziprasidone, sustains
bilateral fracture of his legs after he is hit by car as a pedestrian. He is now in the emergency department and
required surgery to fix the fractures, but he refuses the surgery despite being fully informed of his condition and
consequences of refusal. Which one of the following concepts should guide further management?

A. Informed consent.
B. Utilitarianism.
C. Patient’s autonomy.
D. Patient’s best interest.
E. Incompetency.

Incorrect. Correct answer is C


45% answered correctly

Explanation:

Correct Answer Is C

Every competent adult has the right to refuse proposed treatment(s), no matter how the treatment is crucial or
beneficial for them to save their lives or health. This patient seems to understand the consequences of not receiving
treatment, yet he refuses it. In this case no treatment can be given based on the concept of autonomy.

Patient’s autonomy is the most fundamental principle underlying all healthcare ethics. Autonomy grants every
competent adult patient the absolute right to do what he wishes with his own health care. The concept of autonomy
is fundamental to the entirety of the Australia legal system and has complete acceptance as an operating principle
of decision making.

Autonomy over one's own medical care is seen in the same light as freedom of religion, freedom from illegal search
and seizure, freedom of speech, and freedom of assembly. Patients have the right to refuse undesired therapy, and
they have the right to choose whether or not they will participate in experimentation. Each patient has the right to
have his wishes carried out even in the event that he loses consciousness or the capacity to make decisions for
himself. Autonomy represents a patient's right to determine his or her own health-care decisions.

(Option A) Considering the concept of patient’s autonomy, informed consent is an essential part of medical ethics.
Consent means the patient’s approval before you proceed to touch, examine or treat them. Consent must be
informed, meaning the patient should be fully aware of treatment options, possible complications and material risks.

(Option B) Utilitarianism is a form of consequentialism stating that consequences of any action are the only
standard of right and wrong. A wrong action is good if the outcome is good and bad if the outcome is not good.
Action based on utilitarianism in this scenario means that the patient should be treated against his expressed will to
otherwise (bad action), because the outcome of treatment is best for him (good outcome). This is not an acceptable
concept in medical ethics because it is against the wishes and wills of a competent patient and his/her autonomy.

(Option D) No form of a treatment can be pursued without the patient’s agreement, even if the proposed therapy is in
the patient’s best interest. Although beneficence or doing what is good for people is a high aim and ethical principle,
autonomy is considered more important and takes precedence. Each patient has the right to refuse a treatment even
if that treatment has no adverse effects and will help them.

(Option E) Competency is a legal term. Only a court of law can decide if a patient is competent; however, for the
medical purposes as long as a patient appears to understand what is proposed and the consequences, criteria for
competency are met. Delirious state, dementia, intellectual disability, the effects of medications, psychotic
symptoms such as delusions and hallucinations and other forms of acute thought disorder makes a patient
incompetent for decision making. This patient has schizophrenia, but is well-controlled on medications and has not
thought disturbances at the moment, and cannot be considered incompetent and be overruled.

425 of 1943
References

• ALRC - Informed consent to medical treatment

• MJA - Consent, capacity and the right to say no

Last updated:
Time spent: QID:1180 2023-2-12

426 of 1943
Nathalie, 72 years old, is a patient of yours with Alzheimer disease. Today, she is in your office for a health check
before she goes on a European tour for bushwalking. On examination, she has a mini-mental status exam (MMSE)
score of 21/30. Which one of the following is the most appropriate action for you to take?

A. Report her to immigration department.


B. Tell her that she should cancel her trip as she is unfit.
C. Telll her that she can go to the trip but should use her medications regularly.
D. Call her family and discuss the issue with them.
E. Reassure that she can take the trip.

Incorrect. Correct answer is B


45% answered correctly

Explanation:

Correct Answer Is B

As a rule, patients with dementia, regardless of the severity, should not travel alone. Even in its mildest form,
Alzheimer disease can be associated with forgetfulness and impaired visuospatial skills and appropriate navigation
and carries the risk of getting lost especially in an unfamiliar environment. Furthermore, bushwalking includes treks
in the nature which makes this tour even more risky for this patient.

Nathalie has an MMSE of 21 and can be categorized as having mild dementia. An MMSE score between 21 and 24 is
associated with mild functional dependence. An MMSE score between 10 and 20 is seen in patients with moderate
Alzheimer disease and is associated with more immediate dependency, such as inability to drive, difficulty with
hygiene and shopping, and remote memory impairment. Severe disease (MMSE score under 10) is associated with
total dependence and the need for constant supervision. Motor impairments, notably gait and balance impairment,
incontinence, and myoclonus, develop at this late stage.

(Option A) Mandatory reporting to immigration department is not required. It is not a doctor’s duty or responsibility
to provide such information to authorities. On the other hand, immigration department has nothing to do with this
woman’s travel authorization.

(Option C) There are approved medications such as donepezil, rivastigmine, or memantine, with modest effect on
dementia. They may delay the decline in cognition temporarily but cannot reverse the disease or prevent it from
progression. Even by taking her medications, Nathalie’s current cognitive status will not improve.

(Option D) Calling Nathalie’s family for discussing the issue with them is not a correct option because although her
cognition is mildly impaired, she is still competent adult and informing her family without her consent will be a
breach of confidentiality.

(Option E) Reassurance is not appropriate because a trip long away from home while unaccompanied by a carer is
associated with risks to her health and well-being.

References

• IHPA - Standardised Mini-Mental State Examination (SMMSE)

• Dementia Australia - Dementia and your legal rights

Last updated:
Time spent: QID:1188 2023-2-12

427 of 1943
A 78-year-old woman, who is a patient in your GP clinic, presents to you for a health check-up before she goes on a
European tour that includes bush walking and travelling through mountains. On examination, you notice a decline in
her recent memory, as well as having trouble in remembering new people’s names and addresses. You perform a
Mini mental state exam (MMSE) for her, the result of which is 24/30. You recommend that she cancel her tour until
further assessment is carried out because due to her current condition you think she is unfit to travel alone. She
refuses and tells you that she will stick to her plan. Which one of the following is the next best step in management?

A. Apply the case to Guardianship court.


B. Admit her to the hospital.
C. Contact the department of immigration and tell them about her condition.
D. Send her to a nursing home.
E. Do nothing.

Incorrect. Correct answer is E


45% answered correctly

Explanation:

Correct Answer Is E

Patients with dementia thrive in familiar environments and by following a familiar routine. Adherence to routines
produces a sense of safety by knowing what is going to happen next and avoidance of confusion. Routines are
interrupted when traveling. For patients with cognitive impairment, traveling can be confusing. Being away from
familiar surroundings, eating and sleeping in unfamiliar places, disrupted sleep pattern, having to speak and interact
with strangers (such as airport and hotel staff) and having to follow directions that may not be fully understood and
become confusing for dementia patients. Confusion causes distress that may result in an unusual, potentially
catastrophic behavior. The first rule to consider about travelling is that no person with dementia should ever travel
unaccompanied because there are many decisions to make, directions to follow, and unfamiliar surroundings to
navigate that a person with dementia will find it overwhelming.

This woman has an MMSE score of 24, suggesting mild dementia. Examination also reveals cognitive function
impairment associated with this MMSE score.

She also needs thorough investigation for exclusion of organic and possibly reversible causes of dementia such as
hypothyroidism and vitamin B12 deficiency. Depression should be considered as a cause of pseudodementia that
can mimic dementia. Another important step is a full and formal assessment of her mental state and psychological
status because MMSE is a screening rather than an accurate diagnostic tool. MMSE should be assessed and
interpreted according to the patient’s educational status because while an MMSE score of 23 can be normal for a
patient with 7th grade education, it is clearly abnormal for another patient with college education.

In the meanwhile, this patient should be strongly advised to refrain from travelling alone, especially to unfamiliar
places. She should also be offered further assessment and follow-up; however, if she refuses to do so, nothing more
can be done. She is still a consenting adult and mild cognitive impairment will NOT preclude her from making
decisions for her.

(Option A) Applying for guardianship is not appropriate because this woman has only mild cognitive impairment and
is competent and can make decisions for herself.

(Option B) Hospital admission is not necessary. Further assessment, if planned, is performed in outpatient setting.

(Option C) Mandatory reporting to immigration department is not required. By traveling alone, this patient is not
putting anyone but herself at risk. Also, it is not a doctor’s duty or responsibility to provide such information to such
authorities. More importantly, immigration department has nothing to do with this woman’s travel authorization.

428 of 1943
(Option D) An MMSE score of 24 or a recent decline in memory is not an indication for placing this patient in a
nursing home.

TOPIC REVIEW

Interpretation of MMSE score and the potential associated impairments is summarized in the following table:

MMSE Stage of cognitive


Impairments in activities of daily life (ADL)
score dysfunction
Probably none
25-30 Probably normal
Problems coming up with the right word or name
Trouble remembering names when introduced to new people
Having greater difficulty performing tasks in social or work settings
21-24 Mild dementia Forgetting material that one has just read
Losing or misplacing a valuable object
Increasing trouble with planning or organizing

Forgetfulness of events or about one's own personal history


Feeling moody or withdrawn, especially in socially or mentally
challenging situations
Being unable to recall their own address or telephone number or
the high school or college from which they graduated
Confusion about where they are or what day it is
The need for help choosing proper clothing for the season or the
Moderate occasion
10-20
dementia Trouble controlling bladder and bowels in some individuals
Changes in sleep patterns, such as sleeping during the day and
becoming restless at night
An increased risk of wandering and becoming lost
Personality and behavioral changes, including suspiciousness and
delusions or compulsive, repetitive behavior like hand-wringing or
tissue shredding

Require full-time, around-the-clock assistance with daily personal


care
Lose awareness of recent experiences as well as of their
surroundings
Require high levels of assistance with daily activities and personal
1-9 Severe dementia care
Experience changes in physical abilities, including the ability to
walk, sit and, eventually, swallow
Have increasing difficulty communicating
Become vulnerable to infections, especially pneumonia

References

• IHPA - Standardised Mini-Mental State Examination (SMMSE)

• Dementia Australia - Dementia and you legal rights

Time spent: QID:1189 Last updated:


2023-2-12

429 of 1943
You are asked by a community service to check on an old woman who lives alone and is reported by her neighbors.
When you arrive, you knock but there is no response. After you find the front door open, you call her by name and
introduce yourself as you step in. When you enter her house, the first thing you notice is a messy living room and a
dirty kitchen with unwashed dishes piling upin the sink and rotting food on a plate. An old woman, who is disheveled
and unkempt, walks out of a room, comes towards you, and calls you by another name. You have just started
introducing yourself when she suddenly shouts at you to get out and leave her alone. When you try to calm her down
and explain who you are and why you are at her home, she suddenly attacks you and takes a swing at you. Which one
of the following should be the immediate response in this situation?

A. Leave the house immediately.


B. Call the police.
C. Restrain her.
D. Sedate her.
E. Get a court order.

Correct
45% answered correctly

Explanation:

Correct Answer Is A

The safety of healthcare staff is of paramount significance when dealing with an aggressive patient.

Both the house and the patient’s condition indicate that she is very likely to have a mental illness either caused by a
medical or psychiatric condition. Your presence at her home has made her aggressive to the extent of attempting
assault on you. In this situation, the most appropriate immediate response should be leaving the house immediately
to prevent further aggression and possible assault.

(Option B) Calling the police for support is the next best step after your safety is ensured. In the presence of the
police, you can try verbal de-escalation or other necessary measures such as sedation.

(Options C and D) Sedation or restraining deprives the patient of autonomy and should only be considered as the last
resort. When physical restraint is required, a coordinated team approach is essential, with roles clearly defined and
swift action taken (restraint protocol). While you are alone, trying to restrain or sedate an aggressive patient may
pose harm both to you and the patient.

(Option E) Getting a court order is not necessary at this stage. Court order for guardianship may be required later for
guardianship issues or consent to treatment.

References

• RACGP - General Practice - A safe pace: A guide for the prevention and management of patient-initiated violence

Last updated:
Time spent: QID:1190 2023-2-12

430 of 1943
You are a surgeon in the emergency department. A 71-year-old man with documented history of Alzheimer disease,
who lives with his daughter, is brought to you. The patient is confused, cannot communicate with you, and does not
appear to understand the situation. He is in urgent need for an immediate colostomy. You try to reach his daughter
on the phone for consent but she is not available. Which one of the following is the most appropriate next step in
management?

A. Proceed with the surgery.


B. Wait until you can find the daughter and obtain consent.
C. Do not perform the surgery.
D. Perform the surgery after signatures from two other doctors.
E. Apply the case to the court for guardianship.

Correct
45% answered correctly

Explanation:

Correct Answer Is A

In emergency situations, when the patient is permanently or temporarily unable to give consent, and there is no carer
or guardian or next of kin available to obtain consent from, and delaying the treatment will pose risk to patient’s
health, treatment can be provided without consent. This is based on the principle of duty of care and the patient’s
best interest, and assuming that the patient’s or his/her carer would consent to treatment if he/she was present.
However, if there is an advanced care directive made in the past by the patients, the instruction contained in that
must be followed.

(Option B) In emergency situations, slightest delay may be associated with increased risks to the patient’s health;
therefore, wasting time waiting for the daughter to arrive for consent would be inappropriate.

(Option C) While this patient is in need for emergency treatment, deferring the surgery is not appropriate.

(Option D) You are the healthcare professional responsible for the care and health of the patient and no approval
from other doctors is required.

(Option E) Application of the case to the guardianship court in instances where a patient is in need for emergency
treatment is not appropriate, as delay in treatment is against duty of care and the patient’s best interest.

References

• SA Health - Consent to Medical Treatment and Health Care Policy Guideline

Last updated:
Time spent: QID:1191 2023-2-12

431 of 1943
A pregnant woman has presented for an antenatal visit. During the session, she admits to being beaten by her
husband at home. She adds that her husband is an alcoholic and aggressive and sometimes he even punches and
kicks her. However, he is very careful not to leave any marks or bruises as a proof of physical abuse. Which one of
the following is the most important step in response to this woman’s alleged situation?

A. Call the police.


B. Admit her to the hospital.
C. Talk her into calling the police.
D. Talk to her husband.
E. Refer them for marital counselling.

Incorrect. Correct answer is B


45% answered correctly

Explanation:

Correct Answer Is B

Domestic violence in pregnancy is a common, chronic, complex social problem which is present in all cultures. For
many women, pregnancy and the postpartum period exacerbates the violence and threats within their relationship.
A violent and jealous partner may form an aggressive behavior towards the pregnancy because he is not prepared to
‘share’ her. Financial or sexual pressures may also be more compromised by the pregnancy.

An abusive partner will often target the breasts, stomach and genitals of their pregnant partner. Often the abuse will
start with the first pregnancy, and as a result the woman may avoid prenatal check-ups. Women who do not seek
antenatal care until the third trimester should raise a suspicion of domestic violence.

Abused pregnant women are twice as likely to miscarry compared to non-abused pregnant women. Other obstetrics
complications such as preterm labor and placental abruption are also more likely to occur in such women. In
addition to measures taken for non-pregnant victims of domestic violence, obstetric assessment and interventions
should also been considered, including an assessment of risk of physical harm to a pregnant woman and her fetus
from domestic violence. Most of the time, it is not safe for the pregnant woman to return home. In such situation, an
emergency admission can be made as a ‘place of safety’.

Admission secures the safety of the woman and her fetus for now, and provides a window of opportunity not only for
a full assessment of the potential injuries to her and the fetus, but also for discussing the effects of abuse on her
pregnancy and the fetus’s wellbeing as well as offering different options for help and improving her condition.

(Option A) This woman is a competent adult. Calling the police without her consent is not appropriate unless there
is imminent risk to her health or when there is serious injuries such as lacerations, broken bone, gunshot wounds or
stab wounds. She does not have such injuries and is safe while in your office.

(Option C) This woman should be advised to inform the police directly and report her problem because the police
may be able to provide more information about the patient’s legal options. The patient should be informed that in
many cases she will be able to activate or withdraw from criminal proceeding later. Medical practitioner should also
offer to report the incident to the appropriate authorities, including the police if the patient wants this. However, it is
important to respect their wishes and not pressure them into making any decisions. Any options suggesting
reporting to the police by the treating doctor without the patient’s consent or urging or pushing the patient to call the
police is incorrect. Options should be fully explained but decision must be left to the patient.

(Option D) Talking to the husband is not appropriate at this stage because in most cases such intervention escalates
the situation and jeopardizes the woman’s condition.

(Option E) Referring the woman for marital counselling may imply that the violence is the woman’s fault. However, it
may be considered later in the course of treatment.

432 of 1943
References

• NCBI - Acknowledging a persistent truth: domestic violence in pregnancy

• RACGP - The White Book - Intimate partner abuse: identification and initial validation

Last updated:
Time spent: QID:1192 2023-2-12

433 of 1943
One of your friends calls and tells you that he has come across some negative comments about you as a general
practitioner on Facebook. You check and realize that a Tom has written many negative posts about you and the way
you practice and approach your patients. One of his posts is a detailed story about how you mismanaged him for a
sore throat 3 weeks ago. You check your files and you recognize him; a patient you treated for a viral pharyngitis and
refused to give him antibiotics despite his insist. Which one of the following is the most appropriate action you
should take in management of this situation?

A. Inform your medical defense organization and seek advice.


B. Give feedback on his posts by explaining the nature of his condition and why you refused his requested
treatment.
C. Write to ask him to remove the post.
D. Ignore him and his posts.
E. Contact Facebook and use its policies for removing the post.

Correct
45% answered correctly

Explanation:

Correct Answer Is A

Doctors have always been subject to reputational risks from negative comments or rumors. Nowadays and with the
enormous speed at which words are spread on social media, the risk is even more significant. The issue is not only
how but whether the healthcare entity or an individual physician should respond at all. Such situations make a
dilemma: the patient or his/her family has revealed his/her history or story on the social media but a response by the
doctor to his/her comments and explaining the account of events could be a violation of patient’s privacy. On the
other hand, ignoring such comments could be perceived as an unspoken agreement with the complaints or lack of
concern with the feedback. The desire to “correct the record” or give the other side of the story when faced with the
negative online posting is natural. However, doing so may lead to an allegation of breach of confidentiality or other
legal consequences.

When facing such problem, as a medical center or an individual doctor, there are options to consider:

Ignoring by simply doing nothing.


If the patient can be identified with certainty, contact the patient directly to discuss their concerns and see if
they will remove the post.
Use the website (Facebook here) policies for removal of the post(s).
Send a letter to the patient and/or website owner and request the and/or threaten that you will take legal
action for defamation.
Start defamation proceedings.

Depending on the situation and the nature of the posts and/or comments or whether the patient can be identified,
any of the above options can be choice; however, it is very important that you seek advice from an expert colleague
or your medical defense organization as the most appropriate initial step in the challenge.

(Option B) Giving feedback on the post means that you are discussing the patient’s private information, released to
you as his doctor based on the doctor-patient confidentiality. This is a breach of this confidentiality and very
incorrect. Unlike complaints that are made by a patient directly to an organization, many online complaints are
anonymous. You may think you know who made the complaint while it may have been made by a relative or friend
who knows about the patient, and not the patient himself.

(Options C and E) These options can be your choices as well; however, the decision as to how to respond is best
made after consultation with an expert colleague or medical defense organization.

434 of 1943
(Option D) In most cases, ignoring such negative comments and/or posts are advisable if the nature of such
posts/comments are benign. However, in this case, where your reputation or that of the center you are working at
might be at risk, the best action to take can be considered after consultation with more expert resources of advice.

References

• RACGP - AFP - Patients’ use of social media: e-rating of doctors

Time spent: QID:1203 Last updated:


2023-2-12

435 of 1943
Martha, 54 years old, is a patient of yours, who has come to your office today for a health check.
During the session you notice that she is sad and worried, and is vaguely trying to communicate
about her concerns. Further probing reveals the issue. Her 16-year-old son is an addict and a drug
dealer. She has been frequently beaten by him when she refuses to give him money for drugs.
Which one of the following is the most important next step to consider regarding her problem?

A. Call the police.


B. Tell her to seek shelter in a women’s refuge center.
C. Inform the child protection service.
D. Admit the son to a rehabilitation center.
E. Arrange an appointment with the son.

Incorrect. Correct answer is B


45% answered correctly

Explanation:

Correct Answer Is B

Of the options, advising Martha to seek shelter in a women’s refuge center is the most appropriate
one. She must move from the violent environment to stay with a relative or friend or a women’s
refuge center to reduce the risk of further harm while further action and follow-up is planned for
her.

(Option A) Although it is important that Martha is informed of her legal options, including calling
the police, this cannot be done without Martha’s consent. Every adult person is mandated by law to
report to the police or relevant authorities if a belief is formed, based on reasonable grounds or
upon disclosure by the victim that family violence has occurred or is occurring. However, if such
issue is disclosed to a healthcare professional as a part of a therapeutic relationship, the
healthcare worker often should not call the police without the patient’s consent. Exceptions must
be made when there is an imminent serious risk to one’s health, or if there are children involved and
in immediate risk.

(Option C) Informing child protection services in not appropriate because the one requiring
protection in this scenario is Martha not his son.

(Option D) Admission of the boy to a rehabilitation center is not a priority compared with Martha’s
safety and wellbeing, nor can it be done without the son’s consent.

(Option E) Arranging an appointment with the son may be planned later on after discussion with
Martha but does not take precedence over a safety plan for Martha’s safety.

References

• RACGP - The White Book - Abuse and violence - Working with our patients in general practice

• Govenrment of Western Australi - Department of Health


Last updated:
Time spent: QID:1228
2023-2-12
436 of 1943
A 49-year-old married man is brought to the Emergency Department with internal bleeding from injuries sustained in
a motor vehicle crash. The patient has been stabilized but requires blood transfusion prior to planned emergency
surgery. When he is asked to give consent for the transfusion, he states that blood transfusion is forbidden in his
religion. Which one of the following is the most appropriate next step in management?

A. Assess the patient’s decision-making capacity to refuse the transfusion.


B. Call the patient’s wife to obtain consent for the transfusion.
C. Call the Guardianship Court for consent.
D. Proceed with transfusion against the patient’s wish.
E. Talk to the patient about blood substitutes and ask him if he would like to consider them.

Correct
45% answered correctly

Explanation:

Correct Answer Is A

According to Australian law every competent adult individual has full control over his/her body even if this means
refusing life-saving treatment. There is no exception to this rule if the patient is competent; therefore, the first step
would be assessing the patient’s competency to make decisions. Decision-making capacity includes understanding,
appreciation and reasoning.

Once the patient is proved to be competent, other measures such as transfusing Hemacil® or other intravenous
fluids may be considered and discussed with the patient.

It should be borne in mind that although parents of an underage individual are required to consent to the
treatment, this will not apply if the treatment is life-saving or limb-saving. In other words, if the patient in the
vignette was a minor, blood transfusion should be performed against the parent’s will.

Guardianship court is consulted when patient lacks competency either due to impaired judgment or suppressed level
of consciousness and there is no advance directives and there is conflict between family members and the treating
physician, or between the family members themselves.

References

• Australian Medico-legal Handbook – Elsevier Australia (2008) – page 131

Last updated:
Time spent: QID:607 2023-2-12

437 of 1943
A 32-year-old woman presents to your GP practice with complaints of disturbed sleep, colicky
abdominal pain and headaches. During examination, you notice several bruises over her arms, legs
and chest. Upon further inquiry, she confides in you that she has been beaten by her husband and
this happens almost every few days. She then breaks in tears and tells you that she feels helpless
and hopeless and is living in constant fear and stress. She insists that you not tell anybody about
this, especially the police. Which one of the following is the most appropriate next step in
management?

A. Take photos of the bruises.


B. Call the police.
C. Tell her that she should call the police.
D. Give her the contact numbers for a safe shelter.
E. Counsel her about domestic violence.

Incorrect. Correct answer is E


45% answered correctly

Explanation:

Correct Answer Is E

Family violence is a coercive and controlling behavior by a family member that causes physical,
sexual and/or emotional damage to others in the family, including causing them to live in fear and
threatening to harm people, pets or property. Family violence is most perpetrated by one partner
towards another, or by an adult towards a child. Other forms include elder abuse or sibling abuse.

Establishing trust and rapport is of significant importance when domestic violence is disclosed by
a patient. Doctors should be empathic, avoid judging the patient and be direct about the illegal
nature of violence. Some patient might believe that the bad attitude toward them is their fault. The
victim of domestic violence should be counselled properly and openly about the nature of domestic
violence. They should fully understand that assault, in any form, occurring between family
members is a criminal offence, unacceptable and prosecutable, and that they can take steps to
control the situation as the most appropriate next step in management of the situation.

(Option A) Recording the patient’s explanations, physical exam findings and, with the patient’s
consent, taking photographs of the injuries is an important step. Such documents can be used as
invaluable pieces of evidence in a court of law if the patient decides to press charges against the
perpetrator. However, she will feel more confident and experiences mutuality if they are consulted
beforehand. Although documenting the account of the event and the injuries is important, it is
better done after counselling.

(Option B) It is important that she learns her legal options, including calling the police. However,
this cannot be done without her consent. By law, every adult person is mandated to report to the
police or relevant authorities if they are sure or highly suspected that family violence has occurred
or is occurring. Healthcare professionals, especially doctors, are exceptions to this rule because
such disclosures to a healthcare professional occurs based on a therapeutic relationship, with
confidentiality as a cornerstone of this relationship. Confidentiality cannot be breached without the
patient’s informed consent unless there is an imminent serious risk to their health, or if there are
children involved and in immediate risk.
438 of 1943
(Option C) Victims of domestic violence have legal options including informing the police. Patients
should be made aware of such options and encouraged to use them based on their own decisions.
This, however, is best if advised after appropriate counselling.

(Option D) If the patient is reluctant to contact authorities such as the police, family violence
services, or other relevant authorities, making a safety plan with the patient is a very important next
step. A safety plan may include making a list of emergency numbers and considering a safe place
for the patient to go to and how the patient will get there. Such issues can be brought up and
discusses after appropriate counselling. She should also be advised to leave the violent
environment and stay with a family or friend, or to seek shelter in a refuge center as soon as
possible.

References

• RACGP - The White Book - VIolence and the Law

• Western Australia -Department of Health - Policy Frameworks


Last updated:
Time spent: QID:1229
2023-2-12

439 of 1943
After a night shift, an emergency department physician goes to a public bar where he meets his
friend. There, they use amphetamines for pleasure and spend some hours together in the bar
drinking alcohol. You, as another doctor working in the same hospital, see him using drugs. His
next shit starts in two hours. Which one of the following is the most appropriate next action to
take?

A. Inform the Medical Board.


B. Inform the Australian Health Practitioner Regulation Agency (AHPRA).
C. Leave him and say nothing.
D. Talk to him about not going to work under drug influence.
E. Report to the director of the emergency department.

Incorrect. Correct answer is D


45% answered correctly

Explanation:

Correct Answer Is D

According to section 140 of National Law, ‘notifiable conduct’ occurs when a medical practitioner
has:

practiced the practitioner’s profession while intoxicated by alcohol or drugs; or


engaged in sexual misconduct in connection with the practice of the practitioner’s profession;
or
placed the public at risk of substantial harm in the practitioner’s practice of the profession
because the practitioner has an impairment; or
placed the public at risk of harm because the practitioner has practiced the profession in a
way that constitutes a significant departure from accepted professional standards.

Although this doctor has used drugs and alcohol, he is not practicing under influence and has used
illicit drugs and alcohol outside the workplace and in his private life. You have not seen him
practicing while intoxicated, neither have you formed a reasonable belief that he is doing so. Unless
he practices under influence, mandatory reporting to AHPRA (option B) or medical board (option A)
is not necessary.

On the other hand, you know that he is attending the emergency department in 2 hours and might
be still under influence when he starts working; thus putting his patients at risk. Now, this is your
duty to prevent harm to patients. For this reason, leaving him and saying nothing (option C) is not
an appropriate option as well.

In this case, talking to him about not going to work is the most appropriate next thing to do. If he
seems so under influence to not understand the situation and you form a belief that he may
practice while intoxicated, notifying the director of the emergency department of the situation
(option E) would be a wise decision.

TOPIC REVIEW

Approaching a colleague who is practicing under alcohol or drug influence:

440 of 1943
STEP 1 – Duty of care to patient(s) (patients’ safety) – talk to the colleague and send him home. If
difficult seek help from another colleague or the clinical director.

STEP 2 – Duty of care to colleague – make sure that your colleague is safe and check to ensure
he/she has reached home safely. Suggest occupational health referral and insist that his/her
behavior was not appropriate.

STEP 3 – Duty of care to your workplace (hospital/clinic) – keep accurate record of the incident
and inform the clinical director. Consider notifying AHPRA if required in cases of obligatory
notifications.

References

• https://www.medicalboard.gov.au/codes-guidelines-policies/guidelines-for-mandatory-
notifications.aspx

• https://www.racgp.org.au/afpbackissues/2002/200212/20021201khong.pdf
Last updated:
Time spent: QID:1333
2023-2-12

441 of 1943
You are an intern in the surgery ward. There is an attending surgeon, who has had several anger
outburst since you started the ward. Once he becomes angry, he shouts at ward nurses and other
staff. At one occasion, he confronted an operating room nurse during a surgery due to what he
believed was irresponsibility from the nurse. When she tried to explain to him, he became angry
and left the operating room after he shouted and insulted the nurse. Which one of the following is
the best action you should take?

A. Talk to the surgeon directly.


B. Talk to one of the attending surgeons.
C. Raise the problem at the interns meeting.
D. Report to the director of clinical training.
E. Report to the hospital manager.

Incorrect. Correct answer is D


45% answered correctly

Explanation:

Correct Answer Is D

In case an intern has any concerns regarding any supervision, lack of guidelines, problems with
education and training, observation of misconduct by other interns, registrars, consultants, and
other colleagues, or any conflict in workplace, the problem should be reported directly to the
supervisor of intern training or the director of clinical training and advice is sought from him/her.
These persons can also play a pastoral role in such events in giving advice regarding the situation
or reporting the matter if deemed necessary. An supervisor of intern training or a director of clinical
training is often the first point of contact for interns.

Talking to the surgeon directly (option A), or to one of attending surgeons (option B), or reporting to
hospital manager (option E) are decisions that should be left to or at least made after consultation
with the supervisor of intern training or the director of clinical training.

Raising the issue at the intern meeting (option C) leads to unnecessary publicizing of the issue that
could be addressed in a more personal and confidential manner.

References

• https://pmcv.com.au/computer-matching-service/resources/74-a-guide-for-interns-in-victoria/file
Last updated:
Time spent: QID:1341
2023-2-12

442 of 1943
During your shifts, you come to know that one of the interns in your ward has been giving
inappropriate doses of medications to patients at different occasions for the past several weeks.
Which one of the following is the best action to take?

A. Talk to the intern about your concern and his mistake.


B. Report to the supervisor of intern training.
C. Tell the ward nurse to help the intern.
D. Report to Medical Board.
E. Report to the hospital manager.

Incorrect. Correct answer is B


45% answered correctly

Explanation:

Correct Answer Is B

Issues related to interns in a hospital or clinic, whether it is impairment, misconduct, or mistakes


are often directly reported to the supervisor of intern training (SIT) or director of clinical training.
One exception is where there is immediate and serious risk to the wellbeing or safety of patients or
other staff, in which case action should be taken immediately. For example, if you witness that an
intern is about to administer an inappropriate dose of the drug you should tell him/her about it
before he/she can put more risk to the patient’s health.

NOTE – It is the responsibility of the SIT or the director of clinical training to notify the relevant
authorities such as AHPRA in case of any notifiable misconduct or impairment.

References

• PMCV - A guide for interns in Victoria


Last updated:
Time spent: QID:1389
2023-2-12

443 of 1943
You are a general practitioner in a general clinic. Albert and Mandy, husband and wife, are your
regular patients. Albert has diabetes and recently survived a myocardial infarction. Mandy has
come to you today, and shows you a will that she says is signed by Albert. She says that Albert has
made changes to his previous will and now this his new will signed by him. She asks you to sign
the will as witness. What should be your response to her request?

A. Sign the will.


B. Refuse to sign the will.
C. Call Albert and ask if he confirms the will.
D. Seek legal advice from your medical indemnity.
E. Ask you clinic manager if you can sign the will.

Incorrect. Correct answer is B


45% answered correctly

Explanation:

Correct Answer Is B

Under succession law, a will must be signed in front of at least two people as witnesses. This is
required by the legal formalities for making a valid will. The purpose of witnesses for a will is
provision of a safeguard for prevention of fraud and forgery.

Any person above the age of 18 years with mental capacity and credibility to give evidence in a
court of law can witness a will. The only exceptions are those who are unable to see (for example
are visually impaired) to see the act of signing and those who will inherit under the will. The latter
however has been changed is some states. For example, in Victoria, someone who will benefit
under the will can sign as a witness.

The process of witnessing a will is as follows:

First, the will-maker must sign the will in front of two or more witnesses, all present at the
same time and in the same place.
Witness must be mentally competent and be able to see the will-maker make their signature.
At least two witnesses having attested the will then sign their names in confirmation that the
will-maker’s signature, made in their presence, was genuine.

For you, the most appropriate response to Mandy’s request must be refusal to sign the will,
because you have not witnessed Albert’s signing the will in person. You must tell Mandy that you
cannot witness the will because the signature was not made in your presence and you cannot
witness something you have not witnessed. Moreover, you should tell Mandy that only your
signature as a witness cannot validate the will because at least two witnesses must be present at
the same time and in the same place for a valid witnessing process to the will.

Calling Albert to ask if he confirms the will (option C) is incorrect too. Even if he says that: “it is OK
doc. This is my signature and you can go ahead” you cannot still witness the will because you have
not seen him in person signing the will.

444 of 1943
Signing the will (option A) is the most inappropriate action to take. Signing as a witness what you
have not witness is ethically and professionally wrong.

It is better to be familiar with some basic legal concepts that may arise in your daily practice. In
this case, the issue is quite straight forward and based on common sense. As an ethical basic you
cannot witness something you have not personally witnessed. The case is so clear that no further
action including seeking legal advice from your medical indemnity (option D), or asking your clinic
manager if you can sign the will (option E) is required. Calling your medical indemnity for this issue
is like you call them to ask if you can tell a lie.

However, in cases where you are unsure about what to do, seeking legal advice from available
reliable resources such as your medical indemnity sounds reasonable and wise.

References

• WillsHub- successions and inheritance law

• Citizens’ Advice Bureau – Make a will


Last updated:
Time spent: QID:1480
2023-2-12

445 of 1943
Jane is in your office for her laboratory test results you ordered 15 days ago when you suspected
she may have systemic lupus erythematosus (SLE) based on her history and clinical findings. The
test results confirm your diagnosis. You write a referral letter to a local rheumatologist for
consultation. She wants to know for how long this referral letter is valid. Which one of the following
would be you correct response to her question?

A. Three months from the date of issue.


B. Three months from the first visit by the specialist.
C. Six months from the first visit by the specialist.
D. One year from the date of issue.
E. One year from the first visit by the specialist.

Incorrect. Correct answer is E


45% answered correctly

Explanation:

Correct Answer Is E

If supplied by a general practitioner (GP), the law states that a referral is valid for a single course of
treatment for a period of 12 months (one year) after the first service given in accordance with the
referral. Despite this limitation, the referring practitioner can specify an alternative time period such
as three, six, or 18 months or indefinite. Indefinite referrals are not usually made because patients
often have a specific condition requiring a certain amount of time and treatment from specialists.

Note that the referral period begins on the date of the first specialist visit, not on the date the
referral was written. This is frequently misunderstood, including by general practitioners,
specialists and their receptionists.

Other rules to remember are:

Referrals from a specialist are valid for only three months (after the first visit by the practice
the patient has been referred to).
A referral for admitted patients is valid for three months, or the duration of the admission,
whichever is the longer.

References

• AMA - Medicare requirements for referrals to specialists and consultant physicians

• The rules on referrals


Last updated:
Time spent: QID:1482
2023-2-12

446 of 1943
You are a hospital medical officer (HMO) in a public hospital. An intern in your ward approaches
you and says he has just started his rotation in this ward and has been under lots of stress lately.
He cannot get enough sleep and feels down and irritated most of the time. He asks you for a
prescription of some benzodiazepine to help him out. Which one of the following should be the
most appropriate response at this situation?

A. Report him to the coordinator of interns program.


B. Report him to the medical board.
C. Ask him to see his general practitioner (GP).
D. Prescribe benzodiazepines for him.
E. Refer him to a support group.

Incorrect. Correct answer is C


45% answered correctly

Explanation:

Correct Answer Is C

Most interns find their intern year enjoyable and satisfying but it will also be intellectually, physically
and emotionally challenging at times. It is advised that interns speak to others around them who
have been or are going through similar experiences for the best advice.

When a doctor, including interns and medical students, feels unwell either physically or emotionally,
the best advice to give is seeking professional help. Like everybody else, healthcare professionals
should have a regular general practitioner (GP) as their first point of contact with the healthcare
system. This holds true about this situation as well. You should encourage this troubled intern to
see his GP, or if he does not have any, it is about time he had one.

The Medical Board, as a part of code of conduct and good medical practice for medical
practitioners recommends against self-assessment when it comes to the doctor’s own medical
care and advocates consulting with an independent doctor for professional advice.

(Option A) Reporting to the coordinator of interns program is the option when you form a
reasonable belief that the intern’s condition is jeopardizing the patients’ care and health, which is
not the case here. He, with good intentions, has sought help for his problem, and you have not
witnessed or been informed of him putting patients at risk, or deviating from professional manner.

(Option B) Although interns hold a provisional registration with the Australian Health Practitioner
Regulation Agency (AHPRA), notifications should be made to the manager or coordinator of interns
program in the hospital they are working, and not directly to the AHPRA and the medical board.

(Option D) This intern should be advised to go through the standard path of healthcare as for every
member of the community. Any prescription of therapy for him should only be done after a full
evaluation of different aspects of his problem. Therefore, prescription of therapies of any kind, prior
to this assessment is inappropriate and an incorrect option.

(Option E) Although referral to a support group seems a benign and harmless option in this
situation, it is better to be left to the intern’s GP for discussion, and offered by someone who is fully

447 of 1943
in charge of his medical care.

References

• Medical Board – Good Medical Practice – a code of conduct for doctors in Australia

• Australian Medical Council - Guide to intern training in Australia


Last updated:
Time spent: QID:1498
2023-2-12

448 of 1943
A 14-year-old girl presents to you, accompanied by her mother, for a health check. She asks to be
examined alone without her mother in the room. During the examination, she requests a Chlamydia
test. Based on your assessments, she appears to fully understand the test and the impact of the
results. You take a swab to send to a local laboratory. After one week you receive the test result
which is negative. Today, the mother has called your office, and tells you that she knows about her
daughter being tested for Chlamydia and she wants to know the result. Which one of the following
would be the most appropriate response to her request?

A. Do not tell her.


B. Tell her that the test result is negative.
C. Tell her that the results have not come through yet.
D. Ask her to come with her daughter for the next appointment.
E. Tell her that you can disclose the results only to her daughter.

Incorrect. Correct answer is E


45% answered correctly

Explanation:

Correct Answer Is E

According to Common Law, there is a presumption that minors (people under 18) lack the
competence to consent to medical treatment; however, if the minor can show they have
competence, the law will treat them as having the right to consent to treatment. In other words, a
person aged 18 or older is competent until proven otherwise, while a person under 18 years is
defined incompetent unless the treating physician, through assessment, forms a belief that the
minor has sufficient understanding and intelligence to enable him or her to understand fully what is
proposed. This test is sometimes referred to as the Gillick competence test, and the minor is
referred to as Gillick-competent minor or mature minor.

There is another important issue in the scenario that should be considered as well. Sexually
transmissible diseases (STIs), unwanted pregnancies, prenatal care, substance misuse and mental
disorders are major health problems which can occur in the adolescence. These are delicate and
sensitive issues for which early intervention is desirable. However, adolescents’ concerns about
confidentiality can be a barrier to their accessing health services. When adolescents understand a
service is confidential, they are more likely to disclose information about behaviors that may put or
have put their health at risk, to seek health care, and to return for follow-up. For termination of
pregnancy in minors, however, additional specific rules apply.

This girl has concerns about Chlamydia (an STI) and asked you to be tested in private and without
her mother knowing of that. Since she has not given you the explicit consent to inform her mother
of the issue, giving any information in this regard to the mother would be a breach of confidentiality
and inappropriate. This could ruin the trust the girl has put in you and make her reluctant to ever
come back for treatment and consultation should she have any other health issues. Considering
this fact, the most appropriate response to the mother’s request is to mention her daughter’s right
of confidentiality and the fact that you can disclose the results only to her daughter. The mother
knows about the test and it is likely that the daughter has told her about it. Nonetheless, this should
not lead to disclosure of the test results just based on this probability.

449 of 1943
More importantly, once a minor is considered Gillick-competent or mature, he/she is entitled to the
same confidentiality of medical information as an adult patient.

(Option A) Although by not telling the mother confidentiality is preserved, this option is not
appropriate because not only should you not tell her, but you had better explain why you cannot do
so. You need to come clear with her about the confidentiality issue and that it is her daughter’s
decision is she wants to inform her of the test results.

(Option B) Once a mature minor is considered mature, as in this case, he/she entitles
confidentiality, and giving any information regarding diagnosis, treatment or prognosis to anyone
but the patient is a breach of this confidentiality and inappropriate.

(Option C) This option includes giving false statements which is unprofessional and inappropriate.

(Option D) Asking the mother to come with her daughter for the next appointment is not
appropriate. This is a decision the daughter should make. The girl might want to come alone for
discussion in a more private visit while she is not concerned about her mother’s presence.

References

• Confidential health care for adolescents: reconciling clinical evidence with family values

• RACGP - Consent to medical treatment: the mature minor


Last updated:
Time spent: QID:1518
2023-2-12

450 of 1943
Glen is a 32-years old patient of yours who has been recently found out to be HIV positive. Maggie,
his wife, is also a patient in your office. In the last visit and after telling Glen about his diagnosis
and counseling about treatment options and preventive measures, you advised that he should tell
her wife about his diagnosis and that she needs HIV testing, and if positive, treatment. Despite full
explanations about the risks to his wife and the methods to inform her, he still refuses to tell his
wife because he believes this could ruin their marriage and that he cannot afford losing her in such
situation. You again explain to him that it is necessary for her good. He still refuses. Which one of
the following would be the most appropriate next step in management?

A. Inform health authorities.


B. Inform his wife.
C. Respect his confidentiality.
D. Tell him that she should practice safe sex as of now.
E. Apply for a court order for notification.

Correct
45% answered correctly

Explanation:

Correct Answer Is A

Contact tracing of sexual partners has a crucial role in management of sexually transmissible
infections (STIs). Initiation of contact tracing is the responsibility of the diagnosing clinician.

The objectives of contact tracing include the following:

To prevent re-infection of the index case


To treat the possibly infected contacts and minimize complications in them
To reduce the population prevalence of STIs in the community

Contact tracing starts with a conversation with the index patient about informing their partners.
The patient can decide to inform their own contacts (patient referral) or organize for someone else
to inform them (provider referral). Patient referral is the most common type of contact tracing used
in general practice. For this type of contact tracing to be successful, it is important that the
diagnosing doctor informs the patient about who needs to be informed and what information
needs to be given. If the patient decides to use provider referral, the diagnosing doctor can collect
the contact person's details and either notify the contacts themselves or pass the details to a
practice nurse or a sexual health clinic who can undertake this.

Both methods can be anonymous or not, and both can be performed using a range of methods
including in person, telephone, SMS, email or letter. Either method is acceptable. While patients are
usually willing to inform regular partners, for casual contacts or ex-partners often other non-direct
methods are elected.

However, due to significance of HIV, provider referral is the preferred method for contact tracing in
a patient with HIV infections. Contact tracing starts with recent sexual or needle-sharing partners.
Outer limit is the onset of the risky behavior or last known negative HIV test result. If the index
patient has donated or received blood products, semen, or body tissue, the relevant authority
451 of 1943
should be contacted as well. For those whose HIV has been detected due to a recent TB diagnosis,
contact tracing for TB contacts is also important.

If, despite adequate consultation about the importance and means of informing sexual contacts,
the index case refuses to use either patient referral or provider referral to inform his/her contact, it
is the responsibility of the diagnosing doctor to start contact tracing without the patient’s consent.
The method to consider first is notification of the authorized body in that state. Such authorities
either take the matter in hand, or delegate the matter to the diagnosing doctor, a nurse practitioner,
or a suitably qualified and experienced counselor.

Depending on the state such authorized bodies are:

Australian Capital Territory (ACT): Chief Health Officer


New South Wales (NSW): Director General of NSW Health
Queensland (QSL): Chief Executive of QLD Health
South Australia (SA): Clinic 257 Contact Tracing Officer – the doctor can notify directly or may
refer the patient
Victoria: Partner Notification Officer
Western Australia (WA): regional Public Health Units

NOTE – The Tasmanian Act states that a medical practitioner who becomes aware that his or her
HIV-positive patient has not taken all reasonable measures and precautions to prevent the
transmission of HIV may, after consultation with an approved specialist medical practitioner,
inform any sexual contact of that HIV-positive person, and will not be liable to any civil or criminal
liability in relation to that action.

This patient has refused to inform his wife despite being consulted about the importance of
contact tracing and the means it can be done but still ignores your instructions. In such
circumstance, the most appropriate action would be informing the wife through notifying an
authorized body defined by the health department.

(Option B) Informing the patient’s wife directly is not an appropriate option unless such
responsibility is delegated by the authorized body in the health department, or after consultation
with institutional ethics committee and discussion with the medical defence organization before
disclosure. If direct notification is approved after the above processes, the doctor should provide
the patient with written advice that he/she must notify the partners because if he/she refuses the
doctor will do that.

(Option C) Refusing to inform sexual partners is a decision that puts others at risk and forms one
of the exceptions to patient-doctor confidentiality. By the law, the diagnosing physician has a duty
of care for the patient’s sexual partners and has to notify the relevant authorities if the patient
refuses to do so.

(Option D) Safe sex practice is an appropriate advice to give every patient with or without an STI
but does not eliminate the need for contact tracing and mandatory notification.

(Option E) Contact tracing for a notifiable disease against the patient’s will does not need a court
order. The diagnosing physician can notify relevant authorities if, after complete explanation of the
importance of contact tracing and its different methods, the patient refuses to inform contacts.

References

452 of 1943
• ASHM - Guide to Australian HIV Laws and Policies for Healthcare Professionals

• Murtagh’s General Practice – McGraw Hill- 6th Edition – page 267


Last updated:
Time spent: QID:1520
2023-2-12

453 of 1943
You are counseling a 42-year-old patient who was diagnosed with HIV infection last month. He has
been in a stable relationship with his girlfriend for the past eight months. He has had three visits to
your office for consultation regarding referral, antiretroviral therapy, and further investigations for
other sexually transmissible infections (STIs). He has no STI other than HIV. In the past visits, you
talked about the importance of contact tracing. You told him that he should speak to her girlfriend
about his disease and brings her in for HIV testing but every time he adamantly insists that it is his
secret and he does not want anyone to know about it, her in particular. During this visit, her
girlfriend accompanies him and sits in the waiting room. You again emphasize that he should
inform her; otherwise, her health is at more risk. He again refuses to tell her. Which one of the
following is the most appropriate action you should take?

A. Respect the patient's wishes.


B. Obtain a court order to inform his partner.
C. Consult with the ethics committee.
D. Personally notify her or notify the health department.
E. Advise safe sex.

Incorrect. Correct answer is C


45% answered correctly

Explanation:

Correct Answer Is C

HIV is a sexually transmissible infection (STI). Contact tracing of sexual partners has a crucial role
in the management of sexually transmitted infections (STIs). Initiation of contact tracing is the
responsibility of the diagnosing clinician.

Contact tracing starts with a conversation with the index patient about informing their partners.
The patient can decide to inform their contacts (patient referral) or organize someone else to
inform them (provider referral). Patient referral is the most common type of contact tracing used in
general practice. For this type of contact tracing to be successful, it is important that the
diagnosing doctor informs the patient about who needs to be informed and what information
needs to be given. If the patient decides to use a provider referral, the diagnosing doctor can collect
the contact person's details and either notify the contacts or pass the details to a practice nurse or
a sexual health clinic who can undertake this.

Both methods can be anonymous or not, and both can be performed using a range of methods
including in person, telephone, SMS, email, or letter. Either method is acceptable. While patients are
usually willing to inform regular partners, for casual contacts or ex-partners, other non-direct
methods are often preferred by the patients.

However, due to the significance of HIV, provider referral is the preferred method of contact
tracing. Contact tracing starts with recent sexual or needle-sharing partners. The outer limit is the
onset of the risky behavior or the last known negative HIV test result. If the index patient has
donated or received blood products, semen, or body tissue, the relevant authority should be
contacted as well. For those whose HIV has been detected due to a recent TB diagnosis, contact
tracing for TB is also important.

454 of 1943
On some occasions, however, the patient refuses to use either patient or provider referral to inform
his/her contacts despite full discussion and explanations. In such situations, it is the responsibility
of the diagnosing doctor to start contact tracing even without the patient’s consent. The method to
consider first is the notification of the authorized body in that state. Such authorities either take the
matter in hand directly, or delegate the matter to the diagnosing physician, a nurse practitioner, or a
suitably qualified and experienced counselor.

One important point to consider is that direct notification of the index patient’s sexual contacts
should be performed after the ethics committee's approval and counseling with medical indemnity.
Once approved, the patient should be provided with written advice that the patient must notify the
partner, and if the patient still refuses to do so then the doctor has the right to inform his/her
sexual contacts.

Of the given options, consulting with the ethics committee for a legally-supported means of
contact tracing is the most appropriate one; however, notifying the health authorities would be the
best action if it was among the options.

(Option A) The patient's wishes should be respected unless they put others at risk. If the patient
ignores instructions about informing his/her sexual contacts, his wishes cannot be followed
because you have a duty of care for his sexual contacts as well and must notify the relevant
authorities.

(Option B) Contact tracing for a notifiable disease against the patient’s will does not need a court
order. The diagnosing physician can notify relevant authorities if the patient refuses to inform
contacts after completely explaining the importance of contact tracing and its different methods.

(Option D) While notifying the health authorities is the most appropriate option, direct notification
of contacts of this patient without his consent is not an appropriate option unless permitted and
approved by health authorities and after careful consultation with the ethics committee and/or
medical indemnity. Even so, the patient should be provided with written advice that you will notify
his contacts personally if he fails to do so.

(Option E) Safe sex practice is appropriate advice to give every patient with or without an STI but
does not eliminate the need for contact tracing and mandatory notification.

References

• ASHM - Guide to Australian HIV Laws and Policies for Healthcare Professionals
Last updated:
Time spent: QID:1526
2023-2-12

455 of 1943
Mother of an eight-year-old boy brings him to your practice with complaints about his behavior. She believes that her
son has recently become rude and does not listen to her. She sometimes beats him and locks him up in a room as a
punishment for his behavior. The mother also mentions that she has become irritable recently and cannot sleep as
usual. Which one of the following is the most appropriate step in management?

A. Talk to the boy.


B. Notify the Child Protection Service.
C. Refer the mother to Good Parenting programs.
D. Inform the police about the child abuse.
E. Refer the mother for psychiatric counselling.

Incorrect. Correct answer is B


45% answered correctly

Explanation:

Correct Answer Is B

Family violence is coercive and controlling behavior by a family member that causes physical, sexual and/or
emotional damage to others in the family, causing them to live in fear or being threatened. Family violence is most
commonly committed by one partner towards another (‘domestic violence’ or ‘intimate partner abuse’) and/or by an
adult towards a child or children. Other forms include elder abuse or sibling abuse. Any kind of abuse may have long-
term detrimental effects.

There is a legal obligation on both medical and nursing staff to notify Child Protection Service if they have
formed the opinion that a child is need of protection. The child in need for protection is defined by ‘Children,
Youth and Families Act 2005 - SECT 162 PART 4.1’:

(1) For the purposes of this Act a child needs protection if any of the following grounds exist:

(a) The child has been abandoned by his or her parents and after reasonable inquiries-

i. the parents cannot be found; and

ii. no other suitable person can be found who is willing and able to care for the child;

(b) The child's parents are dead or incapacitated and there is no other suitable person willing and able to care
for the child;

(c) The child has suffered, or is likely to suffer, significant harm as a result of physical injury and the child's
parents have not protected, or are unlikely to protect, the child from harm of that type;

(d) The child has suffered, or is likely to suffer, significant harm as a result of sexual abuse and the child's
parents have not protected, or are unlikely to protect, the child from harm of that type;

(e) The child has suffered, or is likely to suffer, emotional or psychological harm of such a kind that the child's
emotional or intellectual development is, or is likely to be, significantly damaged and the child's parents have
not protected, or are unlikely to protect, the child from harm of that type;

(f) The child's physical development or health has been, or is likely to be, significantly harmed and the child's
parents have not provided, arranged or allowed the provision of, or are unlikely to provide, arrange or allow the
provision of, basic care or effective medical, surgical or other remedial care.

(2) For the purposes of subsections (1)(c) to (1)(f), the harm may be constituted by a single act, omission or
circumstance or accumulate through a series of acts, omissions or circumstances.
456 of 1943
The way the mother is using to punish the child for his behavior is likely to pose risks to both his physical and
emotional well-being. According to the above, the mother's action should be reported to child protection authorities.

Reporting child abuse of any kind is mandatory for all medical practitioners and must be reported to authorities as
soon as possible (e.g., Child Protection Service). In this case, it is even more paramount because the mother has the
symptoms of a psychiatric problem.

Talking to the boy for obtaining collateral history or referring the mother for counselling and good parenting program
may be considered later in the course of action; however, securing the child’s safety, is of significance priority.

In this scenario, contacting the police directly would not be necessary. Calling the police may be justified in cases of
in emergency situations, but not here.

References

• RCH - Children in need of protection

Time spent: QID:639 Last updated:


2023-2-12

457 of 1943
A mother brings her 16-year-old daughter for assessment of her knee pain. In the examination
room and while you are examining the girl, she privately tells you that she has had unprotected sex
and asks for Chlamydia testing. You take a swab for testing and send it to the laboratory, the result
of which comes back positive. Few days later, the mother calls you and says that her daughter has
told her all about it and the test you performed and wants to know the test results. Which one of
the following is the most appropriate action you should take?

A. Tell the mother about the test result.


B. Tell her: “I cannot tell you anything about your daughter’s test results. No STD test result
can be discussed or revealed over the phone, not even to the patient.”
C. Tell her: “please ask your daughter to call for the results.”
D. Tell her: “please bring your daughter for another appointment.”
E. Tell her: “you should come over for test results because the issue cannot be discussed
over the phone.”

Incorrect. Correct answer is C


45% answered correctly

Explanation:

Correct Answer Is C

Although the age of maturity is 18 years old according to the Common Law, younger individuals
can still consent to their own medical and dental treatments if they adequately understand what is
proposed as treatment, its consequences and the material risks associated with it. This is referred
to as ‘Gillick competence rule.’ The term Gillick-competent or mature minor is used for such
minors. As a rule, minors mature enough to consent are medicolegally owed the same duty of
confidentiality as adults. Confidentiality becomes even more important in this age group because
major health problems which can occur in adolescence include sexually transmissible diseases,
unwanted pregnancies, substance misuse and mental disorders. These are delicate and sensitive
issues for which early intervention is desirable. However, adolescents’ concerns about
confidentiality can be a barrier to their accessing to health services. When adolescents understand
a service is confidential, they are more likely to disclose information about behaviors that may put
or have put their health at risk, to seek health care, and to return for follow-up.

Understandably, parents have an interest in being informed of and knowing about their children’s
health problems. However, disclosure of such information without the adolescents’ consent will
have negative effect on therapeutic relationship between the treating doctor and the patient. Most
studies show that a considerable number of adolescents did not visit their healthcare providers –
despite wanting to do so – because they were worried that their parents would find out.

In approach to such health issues in adolescents, the most appropriate approach is encouraging
the young person to involve at least one parent as an understanding parent is an invaluable source
of support and relief; however, if the minor refuses, confidentiality should be respected and not
breached.

NOTE – There are exceptions to confidentiality even if the minor has not consented to. These
exceptions may arise when there is a serious and imminent threat to the life or health of the

458 of 1943
individual (e.g. suicide) or another person (e.g., homicide or transmission of serious infectious
diseases.

NOTE – In practice, doctors should always encourage adolescents to inform their parents,
particularly for complex or complicated issues because parents are generally best source of
support for their children. However, if, despite encouragement, an adolescent refuses to inform
his or her parents, confidential health care can be provided as long as the doctor is satisfied that
the adolescent is a mature minor and that the treatment offered is in the adolescent’s best
interests. In profound or life-altering procedures, such as sterilization (in a person with
intellectual disability) or gender reassignment, not even parents can consent on behalf of their
child; the Family Court must decide.

This girl requested Chlamydia testing in private, implying that she wanted the issue to be
confidential. However, even if the mother was present and aware of the test, disclosure of the test
result to anybody other than the girl without her expressed consent would be inappropriate and
should be avoided; therefore, the most appropriate response to the mother’s request is that you are
not allowed to tell anybody but her daughter about the results and she should personally call or
come for that. She can ask the test results from her daughter after they are discussed.

(Option A) Telling the mother about her daughter’s test result is a breach of confidentiality and not
appropriate.

(Option B) Although you should not tell the mother about the test result, this is due to
confidentiality issues, and not because an STD test result cannot be discussed over the phone.

(Option D) The decision as to whether the girl’s mother accompanies her in the next appointment
should be made by the girl. The next visit is very likely to include discussion about treatment option,
safe sex education and arrangement for follow-up visits. These cannot be productively and
efficiently discussed in the presence of the mother unless the girl willingly consents to.

(Option E) You cannot tell the mother about the test results over the phone, in person or in any
other way, unless the girl has consented to disclosure.

References

• MJA - Confidential health care for adolescents: reconciling clinical evidence with family values

• RACGP - Consent to medical treatment: the mature minor


Last updated:
Time spent: QID:1532
2023-2-12

459 of 1943
An 18-year-old Aboriginal man is brought to the Emergency Department of a local hospital. He is
very agitated, hostile, and aggressive. When approached for care, he becomes even more
distressed and starts yelling at the staff and threatening to kill them if they touch him. Verbal de-
escalation fails to calm him down. After calling the security staff to take over the situation, which
one of the following would be the most appropriate next step in management?

A. Discharge him.
B. Call the police.
C. Involuntary admission to the hospital.
D. Asking for an Aboriginal health worker.
E. Restrain him.

Incorrect. Correct answer is D


45% answered correctly

Explanation:

Correct Answer Is D

According to risk management protocol in Australia, calling the security is the most appropriate
step if verbal de-escalating fails. However, the situation with Aboriginals and Torres Strait Islanders
is different. Aboriginal hospital experience can be very stressful for Aboriginals and also
bewildering for health workers.

In such cases, Aboriginal and Torres Strait islander hospital liaison officers (IHLO’s) play an
important role in making rapport and building relationship with Aboriginal patients. Therefore, it is
strongly recommended that prior to any physical restraining or contacting the police, the IHLO or
Aboriginal health worker is contacted to be present and de-escalate the situation.

The main reason for such necessity is the cultural differences and the familiarity of Aboriginal
health workers with the background and sensitivities of this group. For example, while a caring eye
contact is usually perceived as empathy from an aggressive patient, it is unacceptable and
provoking for Aboriginal Australians.

The security staff also must be present in case they are needed but no intervention should take
place as long as possible, and until the Aboriginal health worker takes over.

Other options may be considered after appropriate intervention by and advice from an IHLO or
Aboriginal health worker.

References

• Aboriginal and Torres Strait Islander Patient Care Guideline


Last updated:
Time spent: QID:1592
2023-2-12

460 of 1943
You, as medical doctors in a hospital, are assigned to review the files of doctors who have applied
for a certain position in that hospital and report the eligible doctors to the hospital manager for
further assessment. You realize that one of your colleagues, who is not involved in this process, is
reading the applicants’ files. Which one of the following is the most appropriate next step in this
situation?

A. Inform the medical board.


B. Report the issue to the hospital administrator.
C. Report the issue to AHPRA.
D. Talk to your colleague.
E. Report to your supervisor.

Incorrect. Correct answer is D


45% answered correctly

Explanation:

Correct Answer Is D

Anyone can make a voluntary notification (raise a concern), but under the National Law only
registered health practitioners, employers, and health education providers are required to make a
mandatory notification. In other words, they have a legal duty and are obliged to do so.

Separate guidelines have been developed for mandatory notification about registered health
practitioners and registered students. These guidelines apply to both (1) registered health
practitioners and (2) employers of registered health practitioners (even if they are not registered
practitioners themselves).

There are four concerns that may trigger a mandatory notification. Depending on the type of
concern, you must assess the risk of harm to the public when deciding whether to make a
mandatory notification.

These four concerns are:

Impairment of the practitioner


Intoxication while practising
Significant departure from accepted professional standards
Sexual misconduct.

There are different thresholds that trigger a mandatory notification depending on whether you are
making a notification as a treating practitioner, non-treating practitioner, employer, or education
provider.

A treating practitioner is a practitioner who becomes aware of the concern while providing
treatment to another practitioner. The threshold for making a mandatory notification as a treating
practitioner is higher than for other notifier groups. This is to give practitioners the confidence to
seek help without the fear of a mandatory notification. The threshold for treating practitioners to
make a mandatory notification about impairment, intoxication while practising, and significant

461 of 1943
departure from accepted professional standards is when there is a substantial risk of harm to the
public.

Before making a mandatory notification, a notifier must form a reasonable belief that the incident
or behavior that led to a concern actually occurred and that a risk to the public exists. As this
practitioner has not revealed the information in the applicants’ files to anyone yet, the most
appropriate step is to talk to him first and advising against his conduct.

If you form a belief on reasonable grounds that he would continue this misconduct, mandatory
reporting to medical board via AHPRA (options A and C) or to the administration or your supervisor
(options B and E) depending on the workplace policies and order of hierarchy can be considered
next.

References

• AHPRA – Making a mandatory notification


Last updated:
Time spent: QID:1594
2023-2-12

462 of 1943
You are assessing a patient presenting with pilonidal sinus cyst. He gives consent to you taking a
photo of the lesion and use it only for assessment of his condition. Regarding his consent, which
one of the following are you legally allowed to do with the photos?

A. Saving the photo on your mobile phone to assess the clinical course of the condition.
B. Transferring the photo to a colleague’s phone for consultation.
C. Sharing the photo on your social media.
D. Sharing the photo in a group of medical doctors as a case.
E. Using the photo for an academic conference.

Correct
45% answered correctly

Explanation:

Correct Answer Is A

These days, with the widespread use of smartphones and tablets with digital camera, clinical
photography has become part of day-to-day clinical practice. Using this feature requires meticulous
consideration of the following:

Patient’s consent

Before taking any images, appropriate consent from the patient must be sought and obtained.
When seeking consent, it should be made sure that the patient has all the information they need to
make an informed decision. This includes information about how the image might be used and
disclosed in the future. The patient can decide for what purposes, and under which circumstances
the image(s) are or may be used. Any agreement with the patient should be strictly respected and
any breach of that constitutes a serious breach of confidentiality.

Security of the information

Reasonable steps should be taken to protect the personal information from misuse, interference,
unauthorized access, modification, or disclosure. A healthcare provider who stores photos
involving personal information on a mobile phone or tablet will need to make sure that their
security settings are adequate to protect the information. Images of patients showing medical
conditions are likely to be highly sensitive and it could be difficult to control how images are used
and disclosed once they are shared through an app or on a social media. A health service provider
should carefully consider whether they are able to maintain control of images and review the app’s
or any other platform’s privacy policy, so they understand how the images will be used, disclosed
and stored.

In this scenario, the patient’s consent only allows you to use the photos for assessment of the
clinical course of the disease; therefore, you are not allowed to share your social media (option C),
in a group of medical doctors as a case (option D) or use it for an academic conference (option E).
Even if the intention of sharing the information is patient’s best interest such as consultation with
another healthcare professional with a legitimate interest in the patient’s care, it should be carried
out in an approved safe environment or platform.

463 of 1943
On the other hand, there are instances when you need to consult a colleague with a legitimate
interest in the patient’s care for the same purpose the patient has consented to. Even in such
situations, transferring the photos to other healthcare providers’ smartphones (option B) is not
appropriate because you can no longer ensure the safety and security of patient’s information after
it is sent out, unless the information is shared where privacy and security of the data is guaranteed.
By sending the photos to a colleague’s smartphone, you will lose full control of the data while still
liable for any breach or misuse of them. If sharing the patient’s information is inevitable or
necessary for the patient’s care, it should be taken place in an environment where safety and
security of patient’s privacy could be guaranteed.

NOTE – De-identified information is not considered ‘personal information’ under the Privacy Act.
An image can be de-identified by removing any information that might allow the individual to be
identified, including rare characteristics or a combination of unique characteristics. This might
include facial features and other distinctive physical details like a rare visible medical condition,
physical marking or tattoo.

References

• Australian Government – OAIC – Taking photos of the patient


Last updated:
Time spent: QID:1670
2023-2-12

464 of 1943
Mary and John, an infertile couple, finally manage to conceive IVF. They are in your office today for
prenatal care and also to discuss whether they can sell the leftover eggs, sperms, and embryos.
Which one of the following is correct regarding selling sperms, eggs, or embryos in Australia?

A. It is illegal to sell the embryos, but eggs and sperms can be sold.
B. It is legal to sell the eggs, sperms, and embryos.
C. It is illegal to sell sperms, egg, or embryos.
D. It is illegal to sell eggs and embryos but legal to sell sperms.
E. It is illegal to sell sperms, but embryos and eggs can be sold.

Incorrect. Correct answer is C


45% answered correctly

Explanation:

Correct Answer Is C

In Australia, buying or selling organ tissues of any kind, including eggs, sperms, and embryos is
illegal. Donations are allowed (no profit involved). However, the donor can request reimbursement
for out-of-pocket expenses such as times away from work, travel expenses, etc. This is different
from the United States where it is legal to sell or by sperms and eggs, but trading embryos is illegal.

References

• NHMRC - Organ and tissue donation by living donors


Last updated:
Time spent: QID:1684
2023-2-12

465 of 1943
Concerned parents of a 2-week-old male baby have brought him to the Emergency Department
after he was found dead in the cot. Which one of the following should be considered first?

A. Notifying the coroner.


B. Notifying the police.
C. Reporting to the registry of deaths.
D. Filling out the cause of death form and issuing a death certificate.
E. Notifying the child protection service.

Correct
45% answered correctly

Explanation:

Correct Answer Is A

The scenario represents neonatal death in a setting outside the hospital. Neonatal death is defined
as death u until 28 days of age.

Investigation of any sudden unexpected neonatal death should include:

Coroner notification
Thorough maternal and infant medical histories
Full autopsy examination by a forensic pathologist skilled in perinatal autopsy or a forensic
pathologist in conjunction with a perinatal pathologist
Investigation of the various scenes where incidents leading to the death might have occurred
including the neonate/infant’s sleeping environment

Of the given options, notifying the corner is the next step to take.

There is no imminent risk involved; therefore, notifying the police (option B) is not justified, and
neither is informing the child protection service (option E) as there no longer exists a child to
protect.

Filling out the” cause of death” form (option D) when the cause is not still clear is incorrect and
must be avoided.

References

Last updated:
Time spent: QID:1698
2023-2-12

466 of 1943
A 56-year-old woman presents to your GP practice requesting a prescription of oxycodone for her
painful knees due to long-standing osteoarthritis. She is from interstate and arrived in Victoria 2
weeks ago to stay with her daughter. She hands over a prescription letter from her local GP back at
home. On further questioning about the medication history and if she is having any concerns, she
admits to having an addiction to narcotics years ago but not anymore. Which one of the following,
would be the most appropriate approach in this situation?

A. Give her a prescription for oxycodone.


B. Call her doctor and ask about the prescription.
C. Deny her request.
D. Call your local pharmacy and ask if you can give her a prescription.
E. Give her a non-opiate painkiller instead.

Incorrect. Correct answer is B


45% answered correctly

Explanation:

Correct Answer Is B

This scenario represents a common situation GPs face in their practice: requests for prescribing
drugs of dependence such as opiates. The challenge intensifies when it comes to first-time
patients with such requests as in this case.

In such circumstances, it is of paramount importance to rule out drug-seeking behavior, especially


if there is a past or recent history of addiction to narcotics. Another important step to consider is
thorough history and assessment of the indication for which oxycodone started in the first place,
the duration of use, any adverse effects including dependency, and if the prescription is still valid.
These would be achieved through contacting and involvement of her regular doctor in decision-
making.

It is important to note that oxycodone is a schedule 8 drug and is strictly regulated; therefore,
giving this patient a prescription for it (option A) without verification through proper channels
would be inappropriate. Following verification of the prescription and its validity from her local
doctor, you may write a prescription for her medication, if your state regulations permit it.
Pharmacists have no role in making such decisions and contacting the local pharmacy to seek
permission (option B) for a prescription is not correct. Prescription is decided at the doctor’s
discretion.

This patient could potentially be managed with alternative pain control measures including non-
opiate painkillers, lifestyle modification, physiotherapy, and eventually joint replacement; however,
abrupt withdrawal of opiates and replacing them with non-opiate painkillers (option E) is not
advisable.

Optimal pain management is an essential part of the duty of care doctors have towards their
patients. Therefore, denying the patient’s request (option C) without proving her with an appropriate
pain management plan is neither ethical nor proper.

467 of 1943
TOPIC REVIEW

The 6 Rs of managing high-risk opioid prescribing:

1. Rotation of opioids
2. Reduction (tapering)
3. Replacement pharmacotherapy
4. Reversal with naloxone
5. Referral to allied health practitioners/other specialists
6. Restriction of supply

Under the Pharmaceutical Benefits Scheme (PBS), oxycodone is a Schedule 8 drug.

Legislative requirements

There are strict legal requirements around the prescription of drugs of addiction or controlled
drugs, known as Schedule 8 (S8) medicines. The legislative requirements vary in each state and
territory. Importantly, the legislative requirements for prescribing S8 drugs vary depending on the
person’s dependence:

For drug-dependent persons, S8 medications (and in some states and territories some
benzodiazepines) cannot be prescribed without a permit or appropriate approval from the
relevant state or territory health department’s pharmaceutical services unit (PSU), via an
authority/permit/approval, to patients who are known or suspected to be drug dependent.
For non-drug-dependent persons, S8 medications cannot be prescribed for a period greater
than 2 months without appropriate approval in some states or territories.

Before prescribing an S8 drug, GPs must take all reasonable steps to ensure a therapeutic need
exists.

Once a therapeutic need is established, GPs must comply with state- or territory-specific health
legislation and where necessary, obtain an authority/permit from the relevant PSU. These
authorities are distinct from and in addition to, any authority under the PBS for scripts. When a PBS
or Repatriation Pharmaceutical Benefits Scheme (RPBS) authority application is for an S8 medicine
(other than dexamphetamine sulfate or methylphenidate), the following guidelines apply:

the maximum quantity is generally for 1 month’s supply (e.g., 1 week’s therapy with three
repeats)
where supply for a longer period is warranted, quantities are usually for up to 3 months of
therapy
telephone approvals are limited to 1 month’s supply.

Prescribers need to state the interval of repeat where repeats are called for and ensure state or
territory health authorities are notified about ongoing treatment. A review by a second doctor is
required for a PBS prescription of an opioid beyond 12 months.

Inappropriate S8 prescribing may result in criminal prosecution, financial penalties, the loss of a
doctor’s authority to prescribe S8 drugs or disciplinary action.

References

• RACGP – Prescribing drugs of dependence in general practice


468 of 1943
• RACGP - AJGP: The six Rs of managing high-risk opioid prescribing
Last updated:
Time spent: QID:1724
2023-2-12

469 of 1943
You are an intern in a psychiatric ward in a teaching hospital where you are approached by Melany
who is a ward patient’s sister. She is quite distressed and worried as her brother just confided in
her that he is going to kill himself and ‘end’ it tomorrow. She asks for help with that. Which one of
the following would be the most appropriate approach in this situation?

A. Inform the police immediately.


B. Do not interfere as it is the patient’s wish and decision.
C. Advise her to persuade her brother to talk to you.
D. Call the intern program supervisor.
E. Talk to a senior doctor in the ward.

Incorrect. Correct answer is D


45% answered correctly

Explanation:

Correct Answer Is D

Suicidality is a grave risk in psychiatric patients. Studies suggest that suicide risk increases in the
days and weeks following a psychiatric inpatient admission.

When dealing with patients at risk of suicidality, prompt measures should be taken to mitigate such
risk. All such measures should be taken by experienced registered health practitioners. Since
interns work under provisional registration and limitation, it is always recommended that they seek
help from their assigned supervisors. Some tasks might not be included in the accredited intern
position. In this scenario calling the intern program supervisor is the best option. They can also
seek help from the senior doctor assigned to them, but not any random senior doctor in the ward
(option E). The role of supervisors in intern training is to oversee, guide, and support the interns to
ensure the patients’ safety and that of the interns.

There are occasions where the safety of the patient or others is at imminent serious risk and
prompt action is required. In such circumstances, there are security and restraint protocols in
hospitals that could be called to action. Informing the police (option A) is mostly limited to threats
outside hospitals. Even such a decision should be made after seeking advice from the intern
program supervisors if feasible. On the other hand, the patient’s plans are for ‘tomorrow’; therefore,
it is not an imminent threat.

Suicidality always rules out competence. In other words, a suicidal patient is not capable of making
reasonable decisions for themselves, and decision-making is almost always delegated. Suicide is
not a wish or decision to be respected and followed as it comes from a lack of competence;
therefore, not interfering because it is the patient’s wish and decision (option B) is incorrect.

Suicidal patients require thorough evaluation and therapeutic intervention. Only advising the
patient’s sister to persuade him to talk to you (option C) is neither proper nor enough.

References

• Queensland Health – Caring for a patient who is suicidal

470 of 1943
• Australian Medical Council - Guide to intern training in Australia
Last updated:
Time spent: QID:1742
2023-2-12

471 of 1943
4. An 85-year-old woman, a frequent patient of the hospital where you work, visits you to inquire
about the contents of her living will. You have previously attended to her case as one of her
physicians. She is very friendly with the hospital staff, nurses, and other medical workers and tells
you that “she sees you as family.” She lives alone and has an estranged relationship with her
children, who live abroad. She would like to know if you would be willing to inherit her possessions
should the time come for her death. What is the most appropriate course of action?

A. Politely ask her to leave the hospital.


B. Call security to escort her out.
C. Refer her to a lawyer who is knowledgeable regarding a living will.
D. Tell her that you can only explain the medical terms.
E. Help her write the living will.

Incorrect. Correct answer is D


45% answered correctly

Explanation:

Correct Answer Is D

A living will or an advanced directive is a voluntary statement outlining the types and conditions of
medical care that a person would prefer before requiring such care. It is considered valid if:

The patient had the capacity when they wrote it, or has expressed these preferences
previously
The living will has clear and specific details about treatments that you would accept or refuse
It is recent and relates to a current condition

In case a person does not wish to make their own decisions, an Enduring Guardian may be
appointed to make medical and dental decisions in case the patient is incapacitated.

A person may also nominate one or more substitute decision-makers (through Power of Attorney)
to make decisions on their behalf. In NSW, this is only for financial reasons, and medical or dental
concerns are excluded. Naming a financial decision-maker will entail writing the following:

When and under what conditions, the agreement of Power of Attorney will take effect
Who is to have responsibility for substitute decision-making, and what decisions
What cross-consultation is to occur
The circumstances when revocation is possible

The living will may also concurrently cover financial, personal, and medical decisions. It is not a
legally binding document by itself, however, it is a key piece of information when considering the
treatment of a person who is unable to provide consent at the time of care. Medical practitioners
are required to abide by the living will unless it conflicts with other laws or professional
responsibilities. Health professionals and family members also do not have the authority to
override the living will.

472 of 1943
NOTE - Living wills can be spoken or written, but signed written documents are preferred to
ensure that the patient’s wishes are recorded.

Doctors may assist patients in explaining the medical terms, and ensure that patients who write a
living will use terms that will be clearly understood by their medical providers. The patient’s current
health status and prognosis will also be valuable information for the patient in her own guiding her
own decisions to write the will. Therefore, the best answer in this scenario is to offer assistance in
explaining the terms medical terms.

Politely asking her to leave the hospital (option A) is not appropriate. Instead, the patient should be
guided appropriately regarding her rights and professional boundaries you have.

Calling security to escort the patient out (option B) is an inappropriate response. The patient is not
posing a threat to the doctor or the other staff members.

(Option C) Living wills are not required to be witnessed but may strengthen the validity. Likewise, a
lawyer is not necessary when writing a legal will, although they may be able to provide assistance
and legal advice once the patient is aware of their values and wishes. In accoByw, an adult is
presumed to have decision-making capacity unless there is evidence to the contrary.

Helping her write the will may pose an ethical dilemma in this scenario. There is a conflict of
interest wherein you have attended to her as her doctor or may attend to her again in the future,
and she has offered to give her possessions when she passes. Doctors are required to act in their
patient's best interests despite their professional opinions. Therefore, those directly involved in the
care of patients would be discouraged from becoming a patient’s Enduring Guardian (option E).

References

• Advance care planning Australia – Health professionals: roles and responsibilities


Last updated:
Time spent: QID:1786
2023-2-12

473 of 1943
A 25-year-old man is brought to the emergency department by an ambulance after he was found on
the streets behaving bizarrely and reported to the police. During the initial assessment, he
cooperates nicely and interacts appropriately and seems to be oriented to time, place, and people
but at some point, he mentions that he is feeling the urge to punch you in the face as you do not
understand him. He believes neutrons represent the spirit and electrons are a way of
telecommunication with the higher realm. When you ask him how he knows these, he says God
speaks to him and gives him directions to save the world from a network of sinister beings
crawling over the internet. Which one of the following is the most appropriate next step in
management?

A. Give him 10mg of haloperidol and wait to see if it works.


B. Ask a nurse to help you restrain him.
C. Call security immediately.
D. Perform a urinary drug screen test.
E. Involuntary psychiatric admission.

Incorrect. Correct answer is C


45% answered correctly

Explanation:

Correct Answer Is C

This patient seems to have a disordered thought process and probably auditory hallucinations
(hearing God speaking to him) which highly suggest psychosis. Most importantly, he has
expressed explicitly his urge to harm you. In this situation, calling security is the most important
aspect of the treatment. Although he is cooperative and interactive for now, this might escalate at
any moment due to his unstable judgment and thought process.

Once the environment is secure, further steps can be prioritized. A disturbed patient can be
sedated to protect him and the personnel using haloperidol (option A) may be considered. If the
patient escalates, physical restraint can be considered based on local protocols, but calling the
nurse to help you restrain him (option B) in the absence of trained security is incorrect.

A urinary drug screen is usually necessary and ordered among other investigations wants the
environment is secure and safe to proceed. This patient also requires consultation with the
psychiatry registrar and further evaluation such as the need for involuntary psychiatric admission
(option E).

References

• Australian Commission on Safety and Quality in Healthcare – National standards in mental health
services
Last updated:
Time spent: QID:1802
2023-2-12

474 of 1943
A 17-year-old boy is admitted to the emergency department due to progressive right lower quadrant abdominal pain.
The patient is febrile and confused. Investigation establishes the diagnosis of a perforated appendicitis. He is
booked for emergency laparotomy. His 18-year-old sister accompanies her. Which one of the following is correct
regarding the consent for the surgery?

A. The consent should be obtained from the patient.


B. The consent should be obtained from the sister.
C. Two doctors are required to give consent.
D. The consent should be obtained from the parents.
E. There is no need for the consent as the case is an emergency.

Incorrect. Correct answer is B


45% answered correctly

Explanation:

Correct Answer Is B

The age at which an individual is considered an adult is 18 years old, but when it comes to medical treatment, one
can give consent a younger age, provided that they have a good understanding of the condition, treatment options
and potential consequences. The age at which a minor can consent for medical treatment is 14 or 16, depending on
the state. The validity of consent of a minor is based on the Gillick competence rather than the age. If this patient
was alert and mentally competent, the consent could be obtained from him, but since he is confused, he is
incompetent for this purpose.

In cases where there is an emergency and the patient is either underage to give consent, or is a consenting adult but
is mentally incapacitate due to any reason, and medical treatment is required to save their life or prevent serious
damage to their health, treatment may be provided without consent if there is no advance directives or substitute
decision maker available (guardian, adult relative, etc.). In common law it is referred to as ‘defense of necessity’.

The treatment provided must be required urgently and not be given just because it is convenient. It must be
proportionate to the patient’s needs. Treatment that goes beyond what is necessary to avert the crisis should not be
given.

Importantly, treatment cannot be provided where the patient has made a valid advance directive or refusal of
treatment certificate that clearly says they have refused the intervention, providing that certain conditions are met.

Perforated appendicitis is a genuine emergency putting the patient’s health at serious risk if surgery is not performed
immediately; however, since his adult relative (her sister) is available, consent should be obtained from her as her
substitute decision maker.

(Option A) This patient is 17 years old and would have been capable of giving consent if he was alert and seemed to
understand his condition, the proposed treatment options and their consequences, but his confusion precludes him
from such an understanding at present. He is temporarily incompetent to give informed consent.

(Option C) Since an adult relative as the patient’s substitute decision maker is available, consent should be sought
from her. It is important to note that the substitute decision maker should act based on the patient’s best interest,
which in this case is surgery. If she refuses the surgery, the doctor should not comply with her wish because it is not
in the patient’s best interest. If such conflicts arise, the doctor can proceed to the surgery while following the
guidelines of the state he/she is practicing. In South Australia, for example, the doctor can proceed with the
treatment if another physician who has visited the patient in person confirms (in writing) the need for emergency
intervention. In other states different rules have been put in place.

(Option D) Parents of the boy were the most appropriate options from whom to obtain consent if they were available.
Even an option suggesting obtaining consent from the parents over the phone could be the most appropriate one;

475 of 1943
however, the question does not mention if the parents could be available in person or by telephone immediately;
therefore, his next of kin who is currently available is the most appropriate option.

(Option E) Consent is not required in cases of genuine emergencies, where the patient cannot consent, there is not
advance directive, and no substitute decision maker is available, or if the substitute decision maker refuses a
treatment that is in the patient’s best interest.

References

• Australian Medico-legal Handbook – pages 76-77

• Good Medical Practice – Professionalism Ethics and Law; Cambridge; page 64

Time spent: QID:713 Last updated:


2023-2-12

476 of 1943
A 75-year-old man is brought to the hospital with reduced level of consciousness. CT scan of the head shows
epidural hematoma requiring non-urgent surgery. You are unable to discuss the procedure with the patient due to his
decreased level of consciousness. His spouse and the eldest son are present. His spouse says that her husband
never wanted to come to the hospital and would never want to be resuscitated or have any surgery on him. His son
requests you to do everything possible to save his father’s life. Which one of the following is the most appropriate
action?

A. Listen to his son.


B. Arrange a family meeting to reach a consensus.
C. Apply for guardianship.
D. Listen to his wife.
E. Request the medical superintendent to assess the patient capacity.

Incorrect. Correct answer is D


45% answered correctly

Explanation:

Correct Answer Is D

If a person cannot give consent for their own treatment and there is no advance health directive present, a health
practitioner should obtain consent from the “person responsible”. In this situation, patient’ spouse can consent for
the patient’s future treatment. A spouse’s opinion is considered more valid and appropriate than that of sons or
daughters; therefore, the directives should be obtained from the wife. One other important step is trying to find out
whether the patient was competent or not when he expressed such wishes.

In instances where no advance health directive or a spouse, family members, or carer is present, a medical
superintendent or authorized medical officer can decide for life-saving emergency surgery if the patient is
incompetent to make a decision.

Application to Guardianship is required for all those patients who lack the capacity to decide about their treatment;
however it usually takes few days.

Under the Guardianship Act 1987, a 'person responsible' can make decisions about most medical or dental
treatments.

A 'person responsible' is, in order of priority:

(1) - The legally appointed guardian of the person (including enduring guardian) with the function of consenting to
medical/dental treatment

If there is not one then

(2) - Their spouse or de facto spouse or same sex partner, or if there is no spouse or de facto spouse or same sex
partner

If there is not one then

(3) - Their unpaid carer

If there is not one then

(4) - The patient’s nearest relative over the age of 18 year, which means (in order of preference):

Son or daughter
477 of 1943
Father or mother
Brother or sister (including adopted people and ‘step’ relationships)
Grandfather or grandmother
Grandson or granddaughter
Uncle or aunt
Nephew or niece

When there are two relatives in the same position (for example, a brother and a sister) the elder will be the person
responsible.

References

• Good Medical Practice – Professionalism, Ethics and Law – Cambridge – pages 61-62

Time spent: QID:714 Last updated:


2023-2-12

478 of 1943
After a course of full investigation on a 67-year-old woman, the diagnosis of pancreatic cancer is established. You,
as the treating doctor, are approaching the patient’s room to break the news to her when the patient’s son steps
forward and asks you to not tell her mother about the diagnosis. He argues that his mother is very fragile at the
moment and telling her about the diagnosis will make her worse. You assess the patient’s file. The consent form
does not have any instructions regarding the patient’s wish as to whether she wants to be informed of the diagnosis.
Which one of the following is the most appropriate action to take?

A. Tell the son that your decision cannot be based on the relatives’ recommendations.
B. Do not inform the patient of the diagnosis as it is not mentioned in the file whether she wants to be
informed of the diagnosis.
C. Arrange a family meeting to further discuss the issue.
D. Refer the case to the Guardianship Court.
E. Withhold the information from the patient as it may lead to an emotional breakdown.

Correct
45% answered correctly

Explanation:

Correct Answer Is A

It is a patient’s right to know what you know and as soon as you know, and no recommendation from relatives, no
matter how rational it may sound, can breach this indisputable fact. The only exception is when the patient clearly
states that he/she does not want to be informed of the diagnosis.

There are very limited circumstances in which information may be deliberately withheld from a patient. This is
frequently referred to as ‘therapeutic privilege’. Particular information may be withheld where the practitioner (and
not the relatives) believes, on reasonable grounds, that giving the information to the patient may damage the
patient’s health. The responsibility is on the practitioner to show that providing the information would be reasonably
likely to cause significant harm. It is not acceptable to argue that if the risks associated with a procedure or
condition were disclosed, the patient may choose not to go ahead with it.

(Option B) It is always assumed that the patient has the right and wants to use the right of being informed of the
diagnosis and actively participate in management plan. It should never be assumed otherwise unless the patient
clearly asks it.

(Option C) Arranging a family meeting is futile because the patient should be informed regardless of the meeting
conclusion.

(Option D) Referring the case to the Guardianship court is not appropriate because the rules are quite clear in this
regard. The Guardianship court will confirm your decision.

(Option E) Fearing of the patient's emotional or physical reactions to bad news can never be an excuse
for withholding information from them.

References

• Australian Medcolegal Handbook – Elsevier Australia (2008) - pages 74-75

• Queensland Health - Clinical Excellence Division: Guide to Informed Decision-making in Health Care

Last updated:
Time spent: QID:715 2023-2-12

479 of 1943
In a small town in Sydney, suddenly a number of people fell sick with bloody diarrhea, severe abdominal pain, and oliguria. In the Emergency Department, most of them are severely dehydrated and confused. The panel of doctors recommended
emergency blood tests. Which one of the following, if present on test results, is diagnostic of the condition?

A. Thrombocytopenia.

B. Microangiopathic hemolytic anemia.

C. High creatinine levels.

D. Severe ADAMTS13 deficiency.

E. High LDH levels.

Incorrect. Correct answer is B


45% answered correctly

Explanation:

Correct Answer Is B

Renal failure reflected by oliguria and abdominal pain following invasive diarrhea is classic presentation of hemolytic uremic syndrome (HUS). Addition of CNS symptoms (confusion) to this constellation makes thrombotic thrombocytopenic purpura
(TTP) another possibility if it is caused by ischemia because of thrombi and not sodium derangement caused by severe dehydration. The exact etiology of HUS and TTP is not clear, but the role of Shiga toxin in HUS and ADAMTS13 (a
metalloproteinase) in TTP have been implicated.

HUS, and to some extent TTP, commonly occur following a diarrheal illness with enterohemorrhagic Escherichia coli O157:H7 and Shigella dysenteriae serotype I. These bacteria, in addition to causing bloody diarrhea, are capable of secreting Shiga
(Shigella) and Shiga-like toxin (E-coli). These toxins can bind to certain cell membrane receptors, which, depending on the cell type, can result in:

1. Chemokine or cytokine secretion (colonic and renal epithelial cells)


2. Cellular activation (monocytes and platelets)
3. Secretion of unusually large Von Willebrand multimers (glomerular endothelial cells)

Clinical differentiation of hemolytic-uremic syndrome (HUS) and TTP can be problematic; however, central nervous system involvement is more common in TTP, and more severe renal involvement in HUS.

In HUS, an antecedent history of diarrheal illness is often present. In fact, some authors suggest a clinical classification of HUS based on the presence or absence of diarrhea.

In children, the distinction between HUS and TTP may be more important because general supportive measures (with dialysis as needed) are the standard therapy for HUS, while TTP is treated with plasma exchange. In adults, however, HUS is also
often treated with plasma exchange; therefore, differentiating between HUS and TTP is not as important as it is in children.

There is not a single diagnostic test for HUS and TTP. These are diagnosed based on clinical presentation and presence of microangiopathic hemolytic anemia presenting with:

1. Anemia, elevated bilirubin and LDH (often significantly high)


2. Presence of schistocytes on peripheral smear

In this case, presence of micro-angiopathic hemolysis, is the most important finding that suggests either HUS or TTP. The distinction between the two, however, neither is possible with certainty, nor is necessary as the therapeutic approach is almost
the same for both HUS and TTP in adults.

(Option A) Thrombocytopenia is almost always a feature of HUS and TTP, as it is in other conditions such as ITP; therefore, it is not diagnostic.

(Option C) Renal function tests, including creatinine are part of workup for suspected HUS or TTP, but not diagnostic because high creatinine levels are seen in TTP and HUS as well as dehydration and many other conditions.

(Option D) ADAMTS13 is a metalloproteinase that cleaves Von Willebrand factor (VWF). Its deficiency results in circulating large multimers of VWF. Large molecules of VWF multimers by adhering circulating platelets together leads to microthrombi
in the organs, ischemia, and end organ damage. Majority of patients (>90%) with acquired TTP have circulating antibodies against ADAMTS13 making them ADAMTS13 deficient; however severe ADAMTS13 deficiency is more common in sporadic
forms rather than outbreaks. As the test is time-consuming and more prognostic rather than diagnostic, it is not routinely ordered. Furthermore, ADAMTS13 deficiency alone does not seem to cause TTP, and a contributing factor such as pregnancy,
infection, drugs, etc. is required to trigger TTP.

(Option E) Regardless of the etiology, elevated LDH is seen in hemolysis. LDH is neither sensitive, nor specific for HUS/TTP.

References

• Medscape - Thrombotic Thrombocytopenic Purpura (TTP)

• MSD Mannua - Thrombotic Thrombocytopenic Purpura (TTP)

Last updated:
Time spent: QID:50 2023-2-12

480 of 1943
A 72-year-old man presents to the emergency department with complaint of perianal pain for the past 2 days. His anal area is illustrated in the accompanying photograph. Which one of the following is the most likely diagnosis?

A. Thrombosed internal hemorrhoid.

B. Thrombosed external hemorrhoid.

C. Rectal carcinoma.

D. Crohn’s disease of the anus.

E. Perianal abscess.

Correct
45% answered correctly

Explanation:

Correct Answer Is A

Traditionally, hemorrhoids are classified as internal and external; however, some authors believe that since these two have different origins and mechanisms of development, they are better termed hemorrhoids (instead of internal hemorrhoids) and
perianal hematoma (instead of external hemorrhoids). Perianal hematoma and external hemorrhoids are often interchangeably used.

Hemorrhoid (internal hemorrhoid):

The anus is mainly lined by discontinuous masses of spongy vascular tissue termed anal cushions, which contribute to anal closure and differentiating flatus from stool. Viewed from the lithotomy position, these cushions are located at 3, 7, and 11
o’clock. These cushions are attached together and to the surrounding structures by supporting fibromascular tissue. Hemorrhoids occur when these structures become bulky and protruded due to gravity, straining, or increased tone of anal sphincter
(unlike the common belief, hemorrhoid is not a varicose anal vein).

Bulky and protruded cushion are at risk of trauma from hard stool and bright red bleeding from the capillaries of the underlying lamina propria.

Constipation and straining are the most common causes of hemorrhoids; however, bowel habit is normal in many patients. Congestion from a pelvic tumor, pregnancy, congestive heart failure, nephrosis, or portal hypertension plays a role in only a
minority of patients.

Hemorrhoids are classified as following:

1st degree – remains in the rectum.


2nd degree – prolapses through the anus on defecation but reduces spontaneously afterwards.
3rd degree – like 2nd degree but needs digital reduction.
4th degree – remains prolapsed persistently.

Hemorrhoids are painless unless acute thrombosis superimposes, in which case it may become painful. Thrombosed hemorrhoids are managed conservatively with analgesics, stool softeners, bed rest (elevation of the bed foot) and ice packs for 2-
3 weeks. The drawback of this method is the long time off work. Hemorrhoidectomy is the second option if conservative management is not an option for any reason.

Perianal hematoma (external hemorrhoid)

Perianal hematoma is not hemorrhoid; however, it is usually called an external hemorrhoid. It presents as a painful tense blue swelling at the anal verge caused by a recent thrombosis of an anal vein, often after straining at stool.

The picture in the question shows a fleshy red lesion protruding out of the anus consistent with an (internal) hemorrhoid. Presence of pain suggests acute thrombosis.

(Option B) External hemorrhoid has a different appearance.

(Option C) Although hemorrhoids can be associated with rectal carcinoma, the lesion itself is not a carcinoma. Moreover, a carcinoma this large would have been associated with more pronounced systemic and local symptoms.

(Option D) It is important to note that for every anorectal lesion, thorough investigation should be conducted in an attempt to exclude serious underlying pathologies such as inflammatory bowel disease or malignancies.

(Option E) Perianal abscesses present with painful and tender red perianal swelling and induration, not a lesion protruding out of the anus.

References

481 of 1943
• Medscape - Hemorrhoids

• Therapeutic Guidelines – Gastroenterology


Last updated:
Time spent: QID:322 2023-2-12

482 of 1943
Ten years ago, a 75-year-old man underwent a successful right hemicolectomy for colon cancer, followed by chemotherapy after the cancer was found to be Duke C stage. Now, he has presented for surveillance. He has no specific complaint and the
physical examination is completely normal. Which one of the following would be the investigation of choice for him?

A. Abdominal CT scan.

B. Colonoscopy.

C. Liver function tests (LFT), renal function tests (RFT) and full blood exam (FBE).

D. Carcinoembryonic antigen (CEA).

E. Sigmoidoscopy.

Incorrect. Correct answer is B


45% answered correctly

Explanation:

Correct Answer Is B

Colonoscopy should be performed one year after the resection of a sporadic cancer, unless a complete post-operative colonoscopy has been performed sooner. Recommendations for familial adenomatous polyposis (FAP) and hereditary
nonpolyposis colorectal cancers (HNPCC) are different from sporadic cancers.

If a peri-operative colonoscopy performed at one year reveals advanced adenoma, next colonoscopy should be performed in 3 years. If the colonoscopy performed at one year is normal or identifies no advanced adenomas, the next colonoscopy
should be performed in 5 years. Since this man has not have a colonoscopy in the past 10 years, he should undergo colonoscopy now.

Apart from colonoscopy according to the above recommendations, patients undergoing either local excision (including transanal endoscopic microsurgery) of rectal cancer or advanced adenomas or ultra-low anterior resection for rectal cancer
should be considered for periodic examination of the rectum at 6-month intervals for 2-3 years with digital rectal examination, rigid proctoscopy, flexible proctoscopy, and/or rectal endoscopic ultrasound. These examinations are considered to be
independent of the colonoscopic examination schedule described above.

Other tests that may be used during this period for early detection of metastases may include:

Chest X-ray
CEA (carcinoembryonic antigen) – diagnostically nonspecific but useful for monitoring recurrence.
PET scan (distant metastases will light up on PET scan)
Pelvic/abdominal CT scan

Abdominal CT scan (option A) and CEA (option D) are acceptable options for surveillance but not as crucial as colonoscopy.

(Option E) Sigmoidoscopy cannot visualize beyond the sigmoid colon. Patients with the history of colon cancer need to have visualization of their entire colon for possible tumors or premalignant lesions by colonoscopy.

References

• UpToDate - Surveillance after colorectal cancer resection

Last updated:
Time spent: QID:324 2023-2-12

483 of 1943
Which one of the following is the most common cardiac manifestation of hemochromatosis?

A. Supraventricular tachyacardias.

B. Congestive heart failure.

C. Atrial fibrillation.

D. Atrial flutter.

E. Atrioventricular (AV) block.

Incorrect. Correct answer is B


45% answered correctly

Explanation:

Correct Answer Is B

The most common cardiac manifestation of hemochromatosis is congestive heart failure. The underlying pathology is deposition of iron in the myocardium leading to restrictive cardiomyopathy. Other manifestations are supraventricular
tachycardias (option A), conductive disorders such as AV block (option E) and atrial fibrillation (option C) and atrial flutter (option D). However, these are not as common as congestive heart failure.

References

• Medscape - Hemochromatosis

Last updated:
Time spent: QID:326 2023-2-12

484 of 1943
A 45-year-old man presents with complaints of polyarthritis, impotence and decreased libido. Which one of the following investigations is the most appropriate step to consider?

A. Iron studies.

B. Fasting blood sugar.

C. HFE gene testing.

D. CT scan of the head.

E. Prolactin level.

Correct
45% answered correctly

Explanation:

Correct Answer Is A

It is very important to consider hemochromatosis in patients with decreased libido and manifestations related to other sites of the body such as joints, liver, CNS, etc.

Hemochromatosis, through iron overload and deposition of iron in several organ systems, can present with the following features:

Chronic hepatitis and cirrhosis (the most common cause of mortality).

Abdominal pain.

Arthralgia – often MCPs and large joints, due to chondrocalcinosis and pseudogout. The pain is like that of osteoarthritis.

Skin darkening (tanned skin) – deposition of iron in the skin.

Small testes, infertility, impotence, and decreased libido –small testes are due to hypopituitarism and/or liver disease. Iron deposition in gonads occurs but is not the cause hypogonadism.

Damage to the pancreas and diabetes mellitus (known as bronze diabetes).

Restrictive cardiomyopathy and ensued congestive heart failure (15% of patients). Other less common cardiac manifestations are supraventricular tachycardias, atrial fibrillation, atrial flutter and varying degrees of atrioventricular
block.

Accumulation of iron in the pituitary and panhypopituitarism.

Hepatocellular carcinoma (hepatoma) – in 10% of cases with liver involvement.

Osteoporosis (25% of cases) and osteopenia (41% of case)

Sparse body hair especially pubic hair (62% of patients)

Spoon nails (50% of cases)

NOTE – the most common cause of death from haemochromatosis is liver cirrhosis, followed by cardiac disease, which affects approximately 15% of the patients.

When hemochromatosis is suspected clinically, iron studies are the most appropriate next step to make a diagnosis.

The transferrin saturation (ratio of serum iron to iron binding capacity) reflects increased absorption of iron, which is the underlying biological defect in hereditary hemochromatosis (HH). A fasting transferrin saturation >45% is the most sensitive
test for detecting early iron overload, but not diagnostic of HH. Ferritin can be used to assess iron overload, but it is not as accurate because it is an acute phase reactant and may be elevated in response to several physiologic stresses, alcohol
consumption, and liver disease. Serum ferritin is abnormal when it is >250 µg/L in pre-menopausal women and >300 µg/L in men and post-menopausal women.

If fasting transferrin saturation or serum ferritin is increased on more than one occasion, HH should be suspected, even if there are no clinical symptoms or abnormal liver function tests (LFT). In this situation, the HFE gene test (option C) should be
considered as the next and also most diagnostic tool.

(Option B) Fasting blood sugar is a good step for this patient because diabetes is a possible feature with hemochromatosis; however, it is not diagnostic for the condition.

(Option D) CT scan of the head adds no diagnostic value in this patient.

(Option E) Although hemochromatosis can cause panhypopituitarism and decreased pituitary hormone levels, measurement of prolactin level alone does not add any diagnostic value.

References

• Medscape - Hemochromatosis

Last updated:
Time spent: QID:327 2023-2-12

485 of 1943
Which one of the following is not correct regarding hemochromatosis?

A. The incidence of hemochromatosis in Australia is 1:200.

B. C282Y homozygous accounts for approximately 90% of hemochromatosis cases in Australia.

C. The majority of patients with one copy of C282Y and H63D mutation never develop hemochromatosis.

D. Ninety percent of those with homozygous C282Y will develop symptoms at some stage of their lives.

E. Carriers of only one copy of the mutated HFE gene will not be affected clinically.

Incorrect. Correct answer is D


45% answered correctly

Explanation:

Correct Answer Is D

(Option A) Correct - The incidence of hereditary hemochromatosis (HH) in Australia is 1 in 200-300.

(Option B) Correct - Appromixately 80-90% of HH cases in Australia have homozygous C282Y mutations of their HFE gene.

(Option C) Correct - Most of patients with heterozygous C282Y/H63D mutations never develop clinical symptoms or will have only mild symptoms.

(Option D) Incorrect - Only 28.4% of males and 1.2% of the females with C282Y mutation will develop clinically significant presentation of iron overload some time in their lives.

(Option E) Correct - Carriers of a gene mutation never will be symptomatic

TOPIC REVIEW

Hemochromatosis has two forms. It is either due a genetic condition, namely hereditary hemochromatosis (HH), or secondary to conditions leading to iron overload in the body such as chronic hemolysis and multiple transfusions. Secondary iron
overload is referred to as hemosidrerosis.

HH is an autosomal recessive genetic disease with variable penetrance and delayed age of onset (rare before the age of 30 years), in which the body iron content exceeds enormously beyond the normal limit (20-60g compared to normal amount of
4g) due to increased absorption through gastrointestinal tract. HH is the result of a mutation in HFE gene, which is located on the short arm of chromosome 6.

The two most common mutations of HFE gene are termed C282Y and H63D. Different possible combinations of these mutations and their risk for development of clinical HH is summarized in the following table:

Homozygous C282Y High risk for HH


Homozygous H63D Unlikely to develop clinical HH
Heterozygous C282Y & H63D Milder form of HH

Homozygous C282Y is the most common form of HH (80-90%), followed by heterozygous C282Y/H3D. 28.4% of males and 1.2% of the females with C282Y mutation will develop clinically significant presentation of iron overload some time in their
lives, but rarely before the age of 30 years.

Those with homozygous H63D are very unlikely to develop clinical disease. Heterozygosity of C282Y and H63D leads to HH with milder clinical forms.

Epidemiology

The prevalence of hemochromatosis in the Australian of Northern Europe background is 1 in 200-300 (250). Every 1 in 8 is a silent carrier of one mutated HFE gene.

Clinical disease is seen more in men than women. The age of onset for women is more advanced. The age of onset is rarely before 30 years.

Pathophysiology

HH results in deposition of iron in different body organs leading to a multi-organ/multisystem involvement and presentation.

Presentation

Hemochromatosis, through iron overload and deposition of iron in several organ systems, can present with the following features:

Chronic hepatitis and cirrhosis (most common cause of mortality)

Abdominal pain

Arthralgia – often MCPs and large joints, due to chondrocalcinosis and peusogout. The pain is similar to the pain associated with osteoarthritis.

Skin darkening (tanned skin) – deposition of iron in the skin.

Small testes, infertility, impotence and decreased libido –small testes are due to hypopituitarism and/or liver disease. Iron deposition in gonads occurs, but is not the cause hypogonadism.

Damage to the pancreas and diabetes mellitus (known as bronze diabetes).

Restrictive cardiomyopathy and congestive heart failure (15% of patients). Other less common cardiac manifestations are supraventricular tachycardias, atrial fibrillation, atrial flutter and varying degrees of atrioventricular block.

Panhypopituitarism - caused by iron deposition in the pituitary

486 of 1943
Hepatocellular carcinoma (hepatoma) – in 10% of cases with liver involvement.

Osteoporosis (25% of cases) and osteopenia (41% of case)

Sparse body hair especially pubic hair (62% of patients)

Spoon nails (50% of cases)

Diagnosis

The transferrin saturation (ratio of serum iron to iron binding capacity) reflects increased absorption of iron, which is the underlying biological defect in HH. A fasting transferrin saturation >45% is the most sensitive test for detecting early iron
overload, but not diagnostic of HH. Ferritin can be used to assess iron overload, but it is not as accurate because it is an acute phase reactant and may be elevated in response to several physiologic stresses, alcohol consumption, and liver disease.
Serum ferritin is abnormal when it is >250 µg/L in pre-menopausal women and >300 µg/L in men and post-menopausal women.

If fasting transferrin saturation or serum ferritin is increased on more than one occasion, HH should be suspected, even if there are no clinical symptoms or abnormal LFTs. In this situation, the HFE gene test should be considered as the next
diagnostic tool.

Although liver biopsy is the most accurate test to diagnose hemochromatosis, an MRI of the liver in conjunction with HFE gene testing for mutations are diagnostic enough to eliminate the need for liver biopsy.

NOTE - Iron studies may be normal in individuals with a genetic predisposition to HH, who have not developed iron overload. Up to 40% of homozygotes have normal iron studies, which may be due to overt (blood donation) or covert (gynecological or
gastrointestinal) blood loss

References

• Medscape - Hemochromatosis

Time spent: QID:328 Last updated:


2023-2-12

487 of 1943
A 42-year-old woman comes to your clinic seeking advice on screening for hemochromatosis. Her 32-year-old brother has been recently diagnosed with hereditary hemochromatosis. She has two children, aged 9 and 18 years old. Which one of the
following is the best action regarding screening for hemochromatosis and assessing the chance of her children developing the disease?

A. Screen her for hemochromatosis.

B. Screen both children for hemochromatosis.

C. Screen the 18-year-old child for hemochromatosis.

D. Screening is not needed at this stage.

E. Screen her and his husband.

Correct
45% answered correctly

Explanation:

Correct Answer Is A

The gene involved in hereditary hemochromatosis (HH) is called the HFE gene. Mutations in the HFE gene can lead to impaired regulation of iron storage and clinical manifestations of hemochromatosis. There are two types of mutation in HFE gene:
C282Y and H63D.

Terminology

Those with only one copy of the mutated HFE (either C282Y or H63D) gene are called heterozygote.

Those with two copies of C282Y mutation are called ‘homozygote’. Since those with two copies of H63D mutation never develop clinical hemochromatosis, homozygote, refers to a person with both copies of HFE with C282Y mutation
only.

Those with one copy of HFE gene with C282Y mutation and the other with H63D mutation are called ‘compound heterozygote’. These individuals are often asymptomatic and if symptoms are present they are mild.

About 90% of people of Northern European ancestry with symptoms of HH have the C282Y mutation in both copies of their HFE gene (homozygote). Two percent are compound heterozygote (see above).

Since HH follows an autosomal recessive pattern of inheritance, there is often no family history, or affected family members may appear to be scattered in generations. If both parents heterozygous (carriers for a mutation in the HFE gene), there is a
25% chance for their children to be affected and genetically predisposed to HH.

This woman’s brother is diagnosed with hereditary hemochromatosis; meaning that both their parents have been at least carriers of HFE gene mutation. She can be a heterozygote, homozygote or compound heterozygote. Genetic testing will reveal
that.

It is recommended that first-degree and second-degree relatives of individuals, who have HH or are homozygous for the C282Y gene mutation, are tested with iron studies and the HFE gene test. Based on the recommendation she needs testing.
Further steps depend on her test results:

If she does not have a faulty HFE gene, no further testing of the children would be indicated, because even if the father is homozygote, the children would only be carriers in the worst case scenario.

If she is found to be homozygote, then the next step would be testing the father. If the father is found to have no HFE mutations the children are not at risk of HH and could only be carriers and not susceptible to HH.

Of the optins, testing the mother for HH would be the next best step in management.

References

• https://www.nhmrc.gov.au/_files_nhmrc/file/your_he

• http://www.australianprescriber.com/magazine/34/3/

Last updated:
Time spent: QID:329 2023-2-12

488 of 1943
A 35-year-old woman presents to your practice with complaint of right upper quadrant discomfort for the past 3 months. She smokes 10 cigarettes a day and drink alcohol at weekends. On examination, she is otherwise healthy, with no palpable
abdominal mass or tenderness. An abdominal CT scan is arranged and obtained, which is shown in the accompanying photograph. Which one of the following is the most likely diagnosis?

A. Liver abscess.

B. Simple hepatic cyst.

C. Hepatic hemangioma.

D. Hepatocellular carcinoma.

E. Hydatid cyst.

Incorrect. Correct answer is B


45% answered correctly

Explanation:

Correct Answer Is B

The homogenous hypoattenuating (darker than the surrounding liver parenchyma) oval-shaped lesion in the photograph is characteristic of a simple hepatic cyst.

Simple hepatic cysts are common benign liver lesions and have no malignant potential. They can be diagnosed on ultrasound, CT, or MRI.

Simple hepatic cysts are one of the most common liver lesions, occurring in approximately 2-7% of the population. It is slightly more common in women. Hepatic cysts are typically discovered incidentally and are almost always asymptomatic, unless
they are large enough to cause symptoms (such as in this patient).

Simple hepatic cysts may be isolated or multiple and may vary from a few millimeters to several centimeters in diameter. Simple hepatic cysts are benign developmental lesions that do not communicate with the biliary tree. They can occur anywhere
in the liver, but there may be a greater predilection for the right lobe of the liver.

Certain diseases are associated with multiple hepatic cysts and include:

Polycystic liver disease


Autosomal dominant polycystic kidney disease (ADPKD) - hepatic cysts may be seen in ~40% of those with ADPKD
Von-Hippel-Lindau disease

Findings on ultrasonography include:

Round or ovoid anechoic lesion (may be lobulated)


Well-marginated with a thin or imperceptible wall and a clearly defined back wall
May show posterior acoustic enhancement if large enough
A few septa may be possible, but no wall thickening is present
A small amount of layering debris is possible
No internal vascularity on color Doppler

On CT scan, a hepatic cysts is characterized by its homogenous hypoattenuation (water attenuation). The wall is usually imperceptible, and the cyst is not enhanced after intravenous administration of contrast material.

(Option A) Liver abscess is associated with fever, leukocytosis and more pronounced symptoms. They are solid and hyperattenuated on CT scan.

(Option C) hemangiomas have less homogenocity and well-demarcation compared to hepatic simple cysts.

(Option D) Hepatocellular carcinoma (HCC) present with less demarcated hepatic lesions that are often hypoattenuated on CT and hypoechoic on ultrasonography. The radiologic findings are inconsistent with HCC as a possible diagnosis.

(Oprion E) Hydatid cysts presents as a multiloculated cyst (daughter cysts within the main cyst)

489 of 1943
References

• http://emedicine.medscape.com/article/190818-overv

• http://radiopaedia.org/articles/simple-hepatic-cys

• http://gi.org/wp-content/uploads/2014/08/ACG_Guide

Last updated:
Time spent: QID:353 2023-2-12

490 of 1943
Which one of the following is the most common cause of a hyperechoic mass on liver ultrasonography?

A. Hepatoma.

B. Simple cyst.

C. Hemangioma.

D. Echinococcal cyst.

E. Metastatic liver disease.

Incorrect. Correct answer is C


45% answered correctly

Explanation:

Correct Answer Is C

Hemangiomas, benign proliferation of vascular tissue, are the most common cause of a hyperechoic liver mass on ultrasound.

Hepatic hemangiomas (also known as hepatic venous malformations) are benign non-neoplastic hypervascualr lesions. They are frequently diagnosed as an incidental finding on imaging in asymptomatic patients. It is very important to differentiate
hemangiomas from hepatic neoplasms.

On ultrasound, they typically manifest as well-defined hyperechoic lesions; however, a small proportion (10%) are hypoechoic, which may be due to a background of hepatic steatosis, where liver parenchyma has increased echogenicity

On CT scan, most hemangiomas are relatively well defined.

Features of typical lesions on three phasic CT scan include:

Noncontrast: often hypoattenuating relative to liver parenchyma


Arterial phase: typically show discontinuous, nodular, peripheral enhancement (small lesions may show uniform enhancement)
portal venous phase: progressive peripheral enhancement with more centripetal fill in
Delayed phase: further irregular fill in and therefore iso- or hyper-attenuating to liver parenchyma

References

• http://radiopaedia.org/articles/hepatic-haemangio

• http://www.uptodate.com/contents/hepatic-hemangiom

Last updated:
Time spent: QID:355 2023-2-12

491 of 1943
A 28-year-old man presents with increasing dysphagia and odynophagia. Endoscopy reveals inflamed esophagus, which easily bleeds on contact. Several biopsies are taken showing eosinophilic infiltrations on histology. Which one of the following
would be the most appropriate next step in management?

A. Proton pump inhibitors.

B. Swallowed fluticasone.

C. Oral prednisolone.

D. Albendazole.

E. Helicobacter pylori eradication.

Correct
45% answered correctly

Explanation:

Correct Answer Is A

Primary eosinophilic esophagitis (EoE) is an increasingly recognized medical condition characterized clinically by symptoms related to esophageal dysfunction, and histologically by eosinophilic inflammation in the esophagus.

EoE is hypothesized to be an atopic inflammatory disease caused by an abnormal immune response to antigenic stimulation, mostly foods. Normally, eosinophils are normal component of mucosal infiltrates in all-length of the gastrointestinal tract
except the esophagus. Eosinophils in the esophageal mucosa are always pathologic.

Generally, the clinical symptoms of EoE are nonspecific, and the patients are in good physical condition resulting in a delayed diagnosis (years) in some cases.

The presenting symptoms vary depending on the age of the onset:

Children - children tend to present with nausea and vomiting, weight loss, anemia, and failure to thrive. In neonates and infants, refusal of food is the most common presenting symptom.

Adults - the characteristic symptoms in adults include dysphagia for solid foods, retrosternal pain and food impaction. Some patients also present with gastroesophageal reflux disease (GERD) symptoms unresponsive to medical anti-reflux therapy.
A subset of patients have been recognized to have a typical clinical presentation of EoE in the absence of GERD who show a clinicopathologic response to PPIs. This condition is currently referred to as PPI-responsive EoE.

There is no Australian guidelines for diagnosis and management of EoE and current recommendation is based on the guidelines by the American College of Gastroenterology (ACG).

According to the ACG, diagnostic criteria for EoE include all of the following:

Symptoms related to esophageal dysfunction.


≥15 eosinophils/hpf on esophageal biopsy
Persistence of eosinophilia after a proton pump inhibitor (PPI) trial
Secondary cause of esophageal eosinophilia excluded

This patient has symptoms related to esophageal dysfunction (dysphagia and odynophagia) and established eosinophilia on histological studies. In order for EoE to be the definite diagnosis, it is necessary that eosinophilia persists after an 8-week
trial of a PPI as well and other causes of eosinophilia are excluced.

The rationale behind the trial of PPI is that GERD may mimic EoE, coexist with it, or contribute to it. Conversely, EoE may contribute to GERD; therefore, the diagnosis of EoE is generally made after the symptoms persist after an 8-week course of
proton pump inhibitors (PPIs) as the best initial step in management. PPIs may benefit patients with EoE either by reducing acid production in patients with co-existent GERD, or by other unknown anti-inflammatory mechanisms.

The main three components of treatment in established EoE are (1) dietary advice and alteration, (2) pharmacotherapy, and (3) surgical intervention. For pharmacological intervention topical swallowed steroids (e.g., fluticasone, budesonide) are
considered the main treatment options, once the diagnosis of EoE is established either after failed PPI therapy or normal pH studies.

For patients unresponsive to the above measure, oral (systemic) predniso(lo)ne maybe indicated.

TOPIC REVIEW

Causes of esophageal eosinophilia:

Eosinophilic esophagitis
GERD
PPI-responsive eosinophilic esophagitis
Achalasia
Crohn’s disease
Parasitic infections
Drug hypersensitivity
Connective tissue disease (e.g., scleroderma, dermatomyositis)
Celiac disease
Hypereosinophilic syndrome

References

• RACGP - Eosinophilic oesophagitis – A guide for primary care

• PubMed - Eosinophilc esophagitis

• Medscape - Eosinophilic Esophagitis

Last updated:
Time spent: QID:465 2023-2-12

492 of 1943
A 65-year-old woman presents to your practice because she has been noting streaks of blood on her stool on different occasions in the past week. These occasions were preceded by a period of constipation. The only medication she is currently on
is panadeine for a back pain started 3 weeks ago. She is otherwise healthy. Which one of the following would be the most likely diagnosis?

A. Ulcerative rectal cancer.

B. Colorectal cancer metastasizing to the lumbar spine.

C. Internal hemorrhoid.

D. External hemorrhoid.

E. Ulcerative colitis.

Incorrect. Correct answer is C


45% answered correctly

Explanation:

Correct Answer Is C

Blood on stool can be caused by several conditions, some being benign and others malignant. Colorectal cancer, particularly if arising from rectum, can cause blood covering the stool. The blood may be bright or dark red, depending on the site of the
tumor. The darker the blood, the more proximal the tumor. Proximal colon cancers tend to present with lethargy and fatigue rather than blood-stained stool because blood mixes with the stool and is barely visible. Rectal bleeding may be the only
symptom of colorectal cancer; however, weight loss, abdominal pain or discomfort, bloating, anorexia, and other symptoms may be seen.

By the time a colorectal is clinically evident, it has often already metastasized to the liver (the most common site) and lymph nodes most of the time. Lungs and bones are rarely involved several months after the tumor has metastasized to the liver
and/or lymph nodes.

Internal hemorrhoids are one of the most common causes of benign rectal bleeding. Patients may notice blood covering the stool, as streaks on the stool, dripping in the toilet bowl, or staining the toilet paper. The most common predisposing factor
for development of an internal hemorrhoid is chronic constipation. This woman has been on panadeine (paracetamol 500mg + codeine 8 mg). This can justify the constipation and internal hemorrhoid as the most likely cause of this presentation.

Ulcerative rectal cancer (option A), or colorectal cancer (option B) in general, can cause rectal bleeding (often painless), but a rectal tumor so large to cause constipation is expected to be associated with more pronounced presentation including
weight loss, anemia, fatigue, or decreased stool caliber.

Ulcerative colitis (option E) is often diagnosed at an earlier age is associated with other manifestations such as bloody diarrhea, weight loss, join pain, uveitis, etc. With bloody stool as the mere complaint in a 65-year-old woman, ulcerative colitis is
an unlikely diagnosis.

As a general rule external hemorrhoids (option D) do not bleed but are painful; therefore, less likely to be the diagnosis.

NOTE - Although internal hemorrhoid is the most likely diagnosis, colorectal cancer should be excluded by thorough investigation.

References

• RACGP - Haemorrhoids – a review

• Medscape - Hemorrhoids

• Medscape - Rectal Cancer

Last updated:
Time spent: QID:469 2023-2-12

493 of 1943
A 70-year-old man presents with difficulty in swallowing for the past 6 months and 4kilogram weight loss in this period. He describes that the most difficult part of swallowing for him is when he tries to start getting the food down his mouth. He had
been a smoker for most of his adult life but has quit 10 years ago. Which one of the following would be the most appropriate management option at this point?

A. Endoscopy.

B. Surgery.

C. Upper series barium study.

D. Helicobacter pylori testing.

E. Manometry.

Incorrect. Correct answer is C


45% answered correctly

Explanation:

Correct Answer Is C

No matter what the clues point towards, every patient with dysphagia should undergo appropriate investigation. Just because of weight loss, the patient’s cannot be told to have esophageal cancer. Although the patient’s age is a red flag for
dysphagia, the fact that it occurs at initiation of swallowing makes oropharyngeal dysphagia a more likely probability. On the other hand, every patient with dysphagia, regardless of the etiology, may have weight loss due to decreased calorie intake;
nonetheless, a thorough and judicious assessment should be considered for every patient with dysphagia.

The best initial step in management of dysphagia depends on provisional diagnosis based on the history and clinical findings. When esophageal cancer is suspected, evaluation starts with upper endoscopy and biopsy. With oropharyngeal and
motility-related dysphagia, barium studies should come first.

In this scenario, oropharyngeal dysphagia, probably caused by a retropharyngeal pouch (Zenker's diverticulum), is the most likely diagnosing; therefore, barium swallow would be the best initial assessment tool. If a retropharyngeal pouch is
diagnosed on barium studies, endoscopy should be avoided due to the significant risk of the scope perforating the pouch.

(Option A) Endoscopy is the initial investigation when cancer is suspected based on history and clinical features.

(Option B) Surgery is indicated if the cause of dysphagia is found to be cancer or Zenker’s diverticulum. Achalasia unresponsive to conservative measures may eventually need surgical intervention as well.

(Option D) Helicobacter pylori can cause peptic ulcer and consequently strictures of the gastric outlet (more common) or inlet (less common). Stricture at the junction of the esophagus to the stomach may cause dysphagia, but difficulty in initiation
of swallowing goes against this diagnosis.

(Option E) Manometry can be used once barium meal study suggests a motility disorder such as achalasia.

References

• Diagnostic Imaging Pathways - Dysphagia

Last updated:
Time spent: QID:96 2023-2-12

494 of 1943
Which one of the following is a sign of pyloric stenosis due to peptic ulcer?

A. Vomiting within the first 1 hour of eating.

B. Vomiting immediately after eating.

C. Vomiting after 2 hours of eating.

D. Regurgitation.

E. Epigastric pain radiating to back.

Correct
45% answered correctly

Explanation:

Correct Answer Is A

Pyloric obstruction also known as gastric outlet obstruction (GOO) is the consequence of any disease producing a mechanical barrier to gastric emptying. Clinical entities that can cause GOO are generally categorized into benign and malignant.

Peptic ulcer disease (PUD) is among the benign causes of GOO. The incidence of ulcer-induced GOO has dramatically declined owing to the adequate and efficient treatment of PUD. The mechanism of obstruction by PUD can be either edema around
the ulcer or scar formation after the ulcer heals.

The most common symptoms of GOO, regardless of the underlying etiology, are bloating, anorexia, nausea and vomiting. Vomiting is usually nonbilious, and characteristically contains undigested food particles.

Patients with gastric outlet obstruction from a duodenal ulcer or incomplete obstruction typically present with symptoms of gastric retention, including early satiety, bloating or epigastric fullness, indigestion, anorexia, nausea, vomiting, epigastric
pain, and weight loss. They are frequently malnourished and dehydrated and have metabolic insufficiency. Weight loss is frequent when the condition is chronic and is most significant in patients with malignant disease. Abdominal pain is not
frequent and usually relates to the underlying cause, e.g., PUD, pancreatic cancer.

The time of vomiting can suggest the site of obstruction. In pyloric obstruction (more proximal) the time of vomitus is usually within the first hour of eating, whereas in pyloric stenosis, or duodenal stenosis or obstruction (more distal) the vomiting
occurs after one hour because normally it takes 1 hour for the food to reach the duodenum. Within 2-4 hours of eating the food is in the small intestine. Vomiting after 2-4 hours should raise suspicion against another cause.

References

• Medscape - Gastric Outlet Obstruction

Last updated:
Time spent: QID:470 2023-2-12

495 of 1943
Which one of the following is the most common cause of acute bile duct obstruction in tertiary care hospitals?

A. Choledocholithiasis.

B. Benign strictures.

C. Tumors.

D. Post-biliary access/manipulation by ERCP or PTC.

E. Acute Cholecystitis.

Incorrect. Correct answer is D


45% answered correctly

Explanation:

Correct Answer Is D

The most common cause of bile duct obstruction in community is choledocholithiasis. Gallstones contribute to approximately 70% of cases presenting with biliary tree obstruction. Benign strictures and tumors are responsible for 15% of cases with
obstructed bile passage. However, it should be noted that the question does not ask about the most contributing factor to obstruction in tertiary hospitals rather than the community.

The most common cause of bile duct obstruction in tertiary hospitals is biliary access/manipulation by endoscopic retrograde cholangiopancreatography (ERCP) or percutaneous transhepatic cholangiography (PTC).

References

• http://www.uptodate.com/contents/endoscopic-retrog

• http://emedicine.medscape.com/article/186850-overv

Last updated:
Time spent: QID:471 2023-2-12

496 of 1943
Which one of the following is correct about hereditary hemochromatosis?

A. Venepuncture should not be performed when the patient is not symptomatic.

B. The incidence is around 1/500,000.

C. Ninety percent of patients are homozygous for C282Y.

D. The incidence is more common among Asians.

E. The life expectancy is reduced regardless of hepatic involvement.

Incorrect. Correct answer is C


45% answered correctly

Explanation:

Correct Answer Is C

Hereditary haemochromatosis (HH) affects 1:200-300 people in Australia. Of the affected persons, approximately 90% are C282Y homozygotes.

HH is much more prevalent among those of Northern European descent and is rare in Asians and Africans. Screening of the population at risk is strongly recommended to identify those with the disease, who are not still clinically symptomatic for
early intervention and preventing the progression of disease; therefore, venepuncture as the first-line therapy should be started in those with abnormal iron studies but no symptoms yet.

The most common cause of mortality in HH is liver involvement and liver cirrhosis. If treatment is started before liver is involved, those with HH can expect a normal lifespan.

References

• RACGP - AFP - Hereditary haemochromatosis

• Medscape - Hemochromatosis

Last updated:
Time spent: QID:472 2023-2-12

497 of 1943
A mother of two children aged 9 and 18 years is concerned about her children developing hemochromatosis, as his husband has recently been diagnoses with the disease. Regarding screening for hemochromatosis, which one of the following is the
next best in management?

A. Screen the 9-year-old boy.

B. Screen the 18-year-old boy.

C. Screen both children.

D. Screen the mother.

E. No screening is needed as the disease becomes clinically evident between 30-60 years.

Incorrect. Correct answer is D


45% answered correctly

Explanation:

Correct Answer Is D

The gene involved in hereditary hemochromatosis (HH) is called the HFE gene. Mutations in the HFE gene can lead to impaired regulation of iron storage and clinical manifestations of hemochromatosis. There are two types of mutation in HFE gene:
C282Y and H63D.

Terminology

Those with only one copy of the mutated HFE (either C282Y or H63D) gene are called heterozygote.

Those with two copies of C282Y mutation are called ‘homozygote’. Since those with two copies of H63D mutation never develop clinical hemochromatosis, homozygote, refers to a person with both copies of HFE with C282Y mutation
only.

Those with one copy of HFE gene with C282Y mutation and the other with H63D mutation are called ‘compound heterozygote’. These individuals are often asymptomatic and if symptoms are present they are mild.

About 90% of people of Northern European ancestry with symptoms of HH have the C282Y mutation in both copies of their HFE gene (homozygote). Two percent are compound heterozygote (see above).

Since HH follows an autosomal recessive pattern of inheritance, there is often no family history, or affected family members may appear to be scattered in generations. If both parents heterozygous (carriers for a mutation in the HFE gene), there is a
25% chance for their children to be affected and genetically predisposed to HH.

Current guidelines advise that first-degree and second-degree relatives of individuals, who have HH or are homozygous for the C282Y gene mutation, are tested with iron studies and the HFE gene test. Based on this recommendation, both children
should be considered for testing; however, in this case testing the mother would be more convenient and reasonable. If she does not have a faulty HFE gene, no further testing of the children would be indicated, because even if the father is
homozygote, the children would only be carriers in the worst case scenario.

References

• http://www.australianprescriber.com/magazine/34/3/

• https://www.nhmrc.gov.au/_files_nhmrc/file/your_he

Last updated:
Time spent: QID:474 2023-2-12

498 of 1943
The photograph is one of a barium swallow series performed in a 78-year-old man. He has presented with symptoms of 12 months duration. Which one of the following could be the most likely presenting symptom?

A. Recurrent chest infection.

B. Progressive weight loss.

C. Retrosternal burning sensation.

D. Gurgling in the neck.

E. Food regurgitation.

Incorrect. Correct answer is E


45% answered correctly

Explanation:

Correct Answer Is E

The photograph shows a pocket of contrast material at the root of the neck, as well as the contrast in the esophagus characteristic of retropharyngeal pouch (Zenker’s diverticulum). The condition is most commonly found in the elderly population.

Patients with Zenker’s diverticulum usually have dysphagia because the primary problem is an overactive upper esophageal sphincter, which fails to relax. Despite long-standing dysphagia, patients usually do not have significant weight loss.

When the pharyngeal pouch becomes large enough to retain contents such as mucus, pills, sputum and food, the patient may complain of pulmonary aspiration and recurrent chest infections, foul-smell breath, gurgling in the throat, appearance of a
mass in the neck, or regurgitation of food into the mouth.

Of these symptoms, however, food regurgitation is the most distressing symptoms for which medical attention is usually sought..

Retrosternal burning sensation is a characteristic feature of gastro-esophageal reflux disease (GERD) and is not an associated symptom in Zenker’s diverticulum.

References

• AMC Handbook of Multiple Choice Questions – pages 449-450

• UpToDate - Zenker's diverticulum

Last updated:
Time spent: QID:476 2023-2-12

499 of 1943
A 69-year-old woman is diagnosed with carcinoma of the cecum. Which one of the following is more likely to have been her initial presenting symptoms?

A. Right iliac fossa (RIF) mass.

B. Altered bowel habit.

C. Weakness and fatigue.

D. Melena.

E. Bright rectal bleeding.

Incorrect. Correct answer is C


45% answered correctly

Explanation:

Correct Answer Is C

Colorectal cancers may present with a wide variety of symptoms. The presenting symptoms, to a great extent, depends on the location of the tumor.

A change in bowel habits is a less common presenting symptom for right-sided tumors because feces is liquid in the proximal colon and the lumen caliber is larger. Right-sided tumors present with anemia and fatigue due to chronic blood loss, while
tumors of the left side are associated with altered bowel habits and rectal bleeding.

(Option A) If a right-sided tumor is large enough, a right iliac fossa mass may be palpated. But a tumor that large has already caused significant symptoms for which the patient has already sought medical attention.

(Option B) Altered bowel habit is more commonly seen in left-sided colon cancers, including rectal tumors.

(Option D) Melena is associated with upper gastrointestinal (GI) bleeding with prolonged passage time of the blood through the gastrointestinal tract. Colorectal tumors are very unlikely to cause melena.

(Option E) Rectal bleeding is often caused by a rectal cancer or more distal left-sided colon cancers.

References

• UpToDate - Clinical presentation, diagnosis and staging of colorectal cancer

• Merck Manuals - Colorectal Cancers

Last updated:
Time spent: QID:477 2023-2-12

500 of 1943
Which one of the following is the most useful investigation for detection of gallstones and dilatation of the common bile duct?

A. HIDA scintigraphy.

B. Ultrasound.

C. Endoscopic retrograde cholangiopancreatography (ERCP).

D. X-ray.

E. Liver isotope scan.

Incorrect. Correct answer is B


45% answered correctly

Explanation:

Correct Answer Is B

Ultrasound is the most useful investigation for detection of gallstones and dilatation of common bile duct. It is also useful in detection of hepatic metastases and some liver diseases.

(Option A) HIDA scan is used if sonographic studies are equivocal. On scan, the liver, CBD, and duodenum light up; the gallbladder will not if inflamed.

(Option C) ERCP is used for visual detection and retrieval of stones in the CBD as well as other causes of obstructive jaundice.

(Option D) Since only 10% of gallstones are radio-opaque, abdominal X-ray will be able to pick up gallstones in only up to 10% of cases; hence, not a preferred method.

(Option E) Liver isotopic scan is useful for evaluation of hepatic cirrhosis.

References

• http://emedicine.medscape.com/article/175667-worku

Last updated:
Time spent: QID:478 2023-2-12

501 of 1943
A 48-year-old male presents to your practice for evaluation of his liver disease. He has the past medical history of chronic alcoholism, intravenous drug abuse and hepatic cirrhosis. Laboratory studies show deranged liver function tests. Which one of
the following would be the best indicator of chronic liver disease?

A. Alkaline phosphatase.

B. Albumin.

C. Alanine aminotransferase (ALT).

D. Aspartate aminotransferase (AST).

E. Bilirubin.

Incorrect. Correct answer is B


45% answered correctly

Explanation:

Correct Answer Is B

Albumin is synthesized in the liver and has a half-life of around 20-22 days. Of the given options, albumin is the only indicator of chronic liver disease.

(Option A) Alkaline phosphatase is not specific to the liver and can be elevated in the following conditions:

Paget’s disease
Fractures
Cholestasis (bile duct obstruction, cirrhosis, etc)
Malignant diseases with bony metastasis

(Options C and D) Alanine aminotransferase is specific to the liver and is raised in metabolic syndrome, obesity, fatty liver and liver failure. ALT and AST are indicators of hepatocellular damage and can be elevated in both acute (e.g. viral hepatitis)
and chronic liver disease. Interestingly, ALT and AST may be normal until the very last stages of chronic liver disease.

(Option E) Bilirubin can be elevated in both acute and chronic liver diseases.

References

• PubMed - Liver enzyme alteration: a guide for clinicians

• UpToDate - Approach to the patient with abnormal liver biochemical and function tests

Last updated:
Time spent: QID:479 2023-2-12

502 of 1943
You are one of the senior residents in surgery. You are called to see Mr. Kingsley, a 67-year-old man, who has just been diagnosed with acute cholangitis. Which one of the following statements is not correct regarding the management of acute
cholangitis?

A. Plan for immediate decompression if the patient does not respond to initial measures.

B. Plan for biliary decompression on semi-urgent basis (<72 hours) if the patient is responding to initial resuscitation.

C. Plan for urgent decompression (within 24-48hrs) if the patient is older than 70 years.

D. The most appropriate method of biliary decompression is ERCP, sphincterectomy and stenting.

E. Initial aggressive resuscitation and antibiotics usually fail to get good response in majority of cases.

Incorrect. Correct answer is E


45% answered correctly

Explanation:

Correct Answer Is E

Acute ascending cholangitis is initially managed with aggressive fluid resuscitation and intravenous antibiotics followed by biliary decompression.

Since the infectious organisms responsible for acute ascending cholangitis are enteric gram negative bacteria, the selected antibiotic of choice should provide appropriate coverage against these germs.

All patients with ascending cholangitis require biliary drainage. In about 85-90% of patients, there is respond to medical therapy. In this group decompression may be performed semi-electively during the same admission (and ideally within 72 hours);
however for the following patients urgent decompression may be considered:

Patients older than 70 years


Patients with diabetes
Patients with other comorbid conditions

Approximately 10% to 15% of patients (not the majority) fail to respond within 12 to 24 hours or deteriorate after initial medical therapy and need urgent biliary decompression. Delay to do so increases the chance of an adverse outcomes.

References

• http://www.uptodate.com/contents/acute-cholangitis

• http://emedicine.medscape.com/article/774245-treat

• http://www.stritch.luc.edu/medicine/sites/default/

Last updated:
Time spent: QID:480 2023-2-12

503 of 1943
Which one of the following complications of acute diverticulitis carries the highest mortality rate?

A. Bleeding.

B. Abscess formation.

C. Peritonitis.

D. Perforation.

E. Intestinal obstruction.

Incorrect. Correct answer is D


45% answered correctly

Explanation:

Correct Answer Is D

Diverticular disease carries a number of potential complications including:

Bleeding, especially in the elderly


Bowel perforation
Intra-abdominal abscess formation
Peritonitis
Fistula formation
Intestinal obstruction

Rupture of an inflamed diverticulum with fecal contamination of the peritoneum occurs in only 1 to 2% of cases but is associated with a 20% mortality rate, which is the highest when compared to other complications of diverticulitis.

Perforation of diverticula into the abdominal cavity presents with the following features:

Abdominal distention
Diffuse tenderness of the abdomen even to light palpation.
Guarding
Rigidity
Rebound tenderness
Absent bowel sounds

References

• http://www.ncbi.nlm.nih.gov/pmc/articles/PMC140235

Last updated:
Time spent: QID:482 2023-2-12

504 of 1943
A 71-year-old man presents with progressive jaundice, pale stool and dark urine. On examination, a mass is palpated in the right upper quadrant that moves with respiration. Ultrasonography shows a dilated common bile duct (CBD) and no masses in
the head of the pancreas. Which one of the following could be the most likely diagnosis?

A. Chronic pancreatitis.

B. Carcinoma of the tail of pancreas.

C. Peri-ampullary tumor.

D. Biliary cirrhosis.

E. Budd-Chiari syndrome.

Incorrect. Correct answer is C


45% answered correctly

Explanation:

Correct Answer Is C

The clinical picture is highly suggestive of common bile duct obstruction (CBD). CBD obstruction presents with obstructive jaundice and often a palpable mass (distended gallbladder) in the right upper quadrant that can be tender or non-tender
depending on the underlying etiology. The following are the most common causes of CBD obstruction:

Stones – the most common cause


Strictures
Periampullary tumors – these tumors arise within 2cm of the ampula of Vater in the duodenum and include the following:

Pancreatic head / uncinate process tumors: includes pancreatic ductal adenocarcinoma involving head and uncinate process of the pancreas
Lower common bile duct tumors: includes types of cholangiocarcinoma involving the intra-pancreatic distal bile duct
Ampullary tumors: those originating from the ampula of Vater
Periampullary duodenal carcinoma

The pancreatic head tumors are the most common periampullary tumors, but not in this case, as sonography shows that the pancreatic head is clear; therefore, other types of periampullary tumors should be considered as the most likely diagnosis.

(Option A) Chronic pancreatitis presents with abdominal pain, malabsorption and diarrhea. Obstructive jaundice is not a presentation.

(Option B) Anatomically, tumors of pancreatic tail are far from biliary system and do not cause biliary obstruction.

(Option D) Biliary cirrhosis does not cause CBD obstruction.

(Option E) Budd-Chiari syndrome is thrombotic occlusion of hepatic veins, presenting with a different clinical picture.

505 of 1943
A 34-year-old woman presents to your clinic complaining of abdominal pain and diarrhea one week after she returned from a trip to Thailand. While she was on the trip, she first noticed abdominal pain in the right iliac fossa which resolved
subsequently. On examination, the abdomen is non-tender and soft with no rigidity or guarding. However, digital rectal exam is tender. Which one of the following is the most likely diagnosis?

A. Giardiasis.

B. Celiac disease.

C. Rotavirus infection.

D. Urinary tract infection.

E. Appendicitis.

Incorrect. Correct answer is E


45% answered correctly

Explanation:

Correct Answer Is E

Although giardiasis, viral gastroenteritis and celiac disease can all present with diarrhea and abdominal pain, no rectal tendernesson is elicitedon rectal exam. Urinary tract infection is not associated with tenderness on rectal exam either.

Of the given options, the only one that can justify the clinical presentation is acute pelvic appendicitis.

Clinical features of pelvic appendicitis are:

Absence of abdominal wall rigidity and tenderness


Tenderness in the rectovesical pouch and/or pouch of Douglas on rectal examination
Right-sided spasm of psoas muscle
Diarrhea due to irritation of the rectum by the inflamed appendix
Increased frequency of urination caused by irritation of bladder due to an inflamed appendix
Hypogastric pain brought on by internal rotation of a flexed hip due to contact of the inflamed appendix with the obturator internus muscle

References

• http://www.aafp.org/afp/1999/1101/p2027.html

• http://emedicine.medscape.com/article/773895-clini

• http://www.uptodate.com/contents/acute-appendiciti

Last updated:
Time spent: QID:483 2023-2-12

506 of 1943
A 35-year-old woman is admitted with a hematemesis. On examination she has a blood pressure of 95/60 mmHg and a pulse rate of 100 bpm. She appears reasonably well-nourished, her sclerae are yellow and she has scratch marks and bruises on
her arms. Two spider nevi are noted on her chest. Her abdomen is distended and soft with evidence of shifting dullness. Dilated veins are easily visible in the subcutaneous tissue and the liver is palpable three finger-breadths below the costal margin.
Her hemoglobin is 85g/L (115-165), serum albumin 28g/L (32-45) and the serum alanine aminotransferase 50U/L (<35). Which one of her clinical and biochemical findings is likely to have the greatest impact on her long-term prognosis?

A. Her blood pressure on admission.

B. The serum albumin concentration.

C. The number of spider nevi.

D. The size of the liver on admission.

E. The serum aminotransferases concentration.

Incorrect. Correct answer is B


45% answered correctly

Explanation:

Correct Answer Is B

The clinical picture is suggestive of ascites in the background of chronic liver disease. The Child-Pugh system is used to assess the prognosis of patients with liver disease and cirrhosis by calculating 1-year survival rate. This system takes into
account the following 5 parameters for determination of the prognosis:

1. The presence of ascites


2. Bilirubin level
3. Albumin level
4. Prothrombin time
5. Encephalopathy

The Child-Pugh score is then calculated according to the following table:

Points assigned
Parameter
1 2 3
Ascites Absent Slight moderate
Bilirubin <2mg/dL (<34.2μmol/L) 2-3mg/dL(34.2-51.3μmol/L) >3mg/dL (>51.3μmol/L)
Albumin >3.5 g/dL 2.8-3.5 g/dL <2.8 g/dL
Prothrombin time (Seconds over control) <4 4-6 >6
INR <1.7 1.7-2.3 >2.3
Encephalopathy None Grade I-II Grade III-IV

Based on the Child-Puch score, the patients are categorized as follows:

Class A (scores 5-6): well-compensated disease – one- and two-year survival are 100% and 85% respectively.
Class B (scores 7-9): functional compromise – one- and two-year survival are 80% and 60% respectively.
Class C (scores 10-15): decompensated disease - one- and two-year survival are 45% and 35% respectively.

Of the given options, the only one contributing to prognosis is serum albumin concentration.

References

• http://www.uptodate.com/contents/cirrhosis-in-adul

• http://www.medscape.com/viewarticle/572659_3

Last updated:
Time spent: QID:484 2023-2-12

507 of 1943
A 42-year-old man is admitted with a hematemesis. On examination she has a blood pressure of 95/60 mmHg and a pulse rate of 104 bpm. He has yellowish sclerae, scratch marks and bruises on his arms. Several spider nevi are noted on his chest
and abdomen. His abdomen is distended and soft with evidence of shifting dullness. Dilated veins are easily visible in the subcutaneous tissue and the liver is palpable two finger-breadths below the costal margin. Which one of the following could be
the most likely underlying cause of his problem?

A. Alcoholic liver disease.

B. Biliary cirrhosis.

C. Schistosomiasis.

D. Budd-Chiari syndrome.

E. Hepatitis C cirrhosis.

Correct
45% answered correctly

Explanation:

Correct Answer Is A

Upper gastrointestinal bleeding may have different causes, but I this man having the clinical features of liver disease and cirrhosis, bleeding from esophageal varices would be the most likely cause of bleeding. Cirrhosis results in portal vein
hypertension.

When the portal vein is obstructed, shunting between the portal vein an systemic veins start to develop. This shunts lead to dilated veins around the umbilicus (caput medusa), in the lower gastrointestinal tract (hemorrhoids), or in the upper
gastrointestinal tract (esophageal varices).

In Australia alcoholic liver disease is the most common cause of chronic liver disease and cirrhosis, followed by hepatitis C infection.

Biliary cirrhosis, schistosomiasis, and Budd-Chiari syndrome can all potentially lead to liver cirrhosis, but are much less common than alcoholic liver disease.

References

• http://emedicine.medscape.com/article/182098-overv

• https://www.nhmrc.gov.au/health-topics/alcohol-gui

• AMC Handbook of Multiple Choice Questions – page 455-456

Last updated:
Time spent: QID:485 2023-2-12

508 of 1943
A 66-year-old man presents with intermittent right upper quadrant pain. An ultrasound, performed for revealing the cause gallstones, reveals a lesion in the liver. Triple phase CT scan is performed for more evaluation showing a 35 mm subcortical
lesion with early prominent dense enhancement, which spreads through the lesion in the late portal venous phase. Which one of the following conditions would fit this description best?

A. Isolated metastatic lesion.

B. Hemangioma.

C. Hepatocellular carcinoma.

D. Hepatic cyst.

E. Hydatid cyst.

Incorrect. Correct answer is B


45% answered correctly

Explanation:

Correct Answer Is B

The patient has the provisional diagnosis of the biliary colic, most likely due to biliary stones. The ultrasound scan – as the best initial diagnostic tool –has been used to confirm the diagnosis. Furthermore, any associated inflammation would be
evaluated. Inflammation of the gallbladder manifests as the thickening of the wall of the gallbladder and the presence of pericholecystic fluid. Stones might be seen in the common bile duct as well; however, the sensitivity of ultrasound for detection
of ductal stones is low (30%-50%).

As a routine procedure, when scanning for biliary problems, the sonographer will scan the liver as well. In this case the sonographer has encountered an incidental finding, irrelevant to the presenting symptoms, for which a triphasic CT scan of the
liver has been performed.

Early prominent dense enhancement of the lesion during the arterial phase is characteristic of liver hemangioma (the most common benign liver tumor). Hemangiomas are seen in approximately 20% of the general population. They may be solitary
or multiple. The lesions typically show intense enhancement during the arterial phase of triphasic CT scan and retain a blush of contrast during the portal venous phase.

(Options A and C) Most malignant liver tumours (primary or metastatic) are hypovascular and will not have the early enhancement during the arterial phase; rather, they become more pronounced during the portal venous phase.

(Option D) Cystic lesions in the liver may be simple, multiple (polycystic liver disease), neoplastic or infective (hydatid cysts). Simple cysts are extremely common and usually asymptomatic. On imaging, these cysts have a low-density homogenous
appearance. With polycystic disease, the number and size of the cysts often lead to symptoms.

(Option E) Hydatid cysts have a characteristic septate appearance and heterogenous appearance if they contain daughter cysts. Liver abscesses are usually symptomatic and more likely to have a heterogenous appearance.

References

• AMC Handbook of Multiple Choice Questions – pages 448-449

• http://radiopaedia.org/articles/hepatic-haemangiom

• http://www.uptodate.com/contents/solid-liver-lesio

Last updated:
Time spent: QID:486 2023-2-12

509 of 1943
A 55-year-old man presents with a 6-month history of increasing dysphagia for solid foods. He has a previous history of gastroesophageal reflux for many years. He has managed his reflux with antacids but since the dysphagia started, his reflux has
not been so troublesome. Which one of the following is the most likely diagnosis?

A. Esophageal cancer.

B. Achalasia.

C. Gatro-esophageal junction stricture.

D. Para-esophageal hernia.

E. Ulcerative esophagitis.

Incorrect. Correct answer is C


45% answered correctly

Explanation:

Correct Answer Is C

Dysphagia to solid food is more likely to be caused by mechanical obstruction due to strictures, tumors, rings or webs.

With the history of protracted gastro-esophageal reflux disease (GERD), the most likely cause of this presentation would be stricture. Inflammation and scarring of the esophagus result in stenosis of the esophagus most often at the site of junction
to the stomach. Resolution of GERD symptoms supports the diagnosis: when the stricture develops, the amount of acid reflux is decreased.

(Option A) Esophageal cancer is another important differential diagnosis that has to be excluded, but not the most likely diagnosis given the absence of other symptoms such as significant weight loss, anemia, etc.

(Option B) Achalasia causes dysphagia to both solids and liquids. Dysphagia to solids, but not to liquids is against out achalasia as a probability diagnosis.

(Option D) Para-esophageal hernias may present with GERD and GERD, over time, may result in this clinical picture. The stricture, however, is caused by the GERD, not directly by the hernia.

(Option E) Ulcerative esophagitis is very rarely complicated by strictures, especially in patients with HIV; hence, an unlikely diagnosis in this HIV negative patient.

References

• Medscape - Esophageal Stricture

Last updated:
Time spent: QID:487 2023-2-12

510 of 1943
You are about to perform a femoral venepuncture and you should take precaution not to damage the adjacent structures. Which one of the following is the order of structures in the groin under the inguinal ligament from medial to lateral?

A. Lacunar ligament, femoral artery, femoral vein, femoral nerve.

B. Lacunar ligament, femoral vein, femoral nerve, femoral artery.

C. Lacunar ligament, femoral vein, femoral artery, femoral nerve.

D. Femoral vein, femoral artery, femoral nerve, lacunar ligament.

E. Femoral vein. Femoral artery, lacunar ligament, femoral nerve.

Incorrect. Correct answer is C


45% answered correctly

Explanation:

Correct Answer Is C

Femoral triangle consists of three borders:

Upper border: inguinal ligament


Medial border: lateral border of adductor longus
Lateral border: medial border of sartorius

Femoral trinagle and its contents

The contents of the femoral triangle from medial to lateral are:

1. Lacunar ligament and deep femoral lymph nodes


2. Femoral vein
3. Femoral artery
4. Femoral nerve

Remember LEVAN: Lymph node chain / lacunar ligament, Empty space, Vein, Artery and Nerve for the order of contents from medial to lateral.

The lacunar ligament is a ligament in the inguinal region that connects the inguinal ligament to the pectineal ligament near the point where they both insert on the pubic tubercle. This ligament comprises the medial border of the femoral canal.

511 of 1943
Location of the lacunar ligament, and the femoral canal: lacunar ligament comprises the medial border of the femoral canal

References

• http://teachmeanatomy.info/lower-limb/areas/the-fe

• http://www.bartleby.com/107/pages/page412.html

Time spent: QID:489 Last updated:


2023-2-12

512 of 1943
Which one of the following is the most common early complication of hemorrhagic pancreatitis?

A. Pseudocyst.

B. Infection.

C. Obstructive jaundice.

D. Pancreatic fistula.

E. Renal failure.

Incorrect. Correct answer is E


45% answered correctly

Explanation:

Correct Answer Is E

Acute pancreatitis can be classified into acute interstitial (most common) and acute hemorrhagic (least common). In the first type, the gland architecture is preserved but is edematous. Inflammatory cells and interstitial edema are prominent within
the parenchyma. In hemorrhagic type, there is marked necrosis, hemorrhage, and fat necrosis. There is marked pancreatic necrosis along with vascular inflammation and thrombosis.

Hemorrhagic pancreatitis can rapidly result in severe hemorrhage, hypovolemia, shock and acute renal failure. Other options are also potential complications of acute pancreatitis but often do not occur as early as acute renal failure.

References

• http://www.hopkinsmedicine.org/gastroenterology_he

• http://www.uptodate.com/contents/management-of-acu

Last updated:
Time spent: QID:494 2023-2-12

513 of 1943
A 57-year-old man presents to your practice complaining of abdominal discomfort and pain for the past 6 months. The pain is predominantly felt in the epigastric area. He does not smoke but admits to chronic alcohol use. On examination, no
abdominal tenderness is elicited. The remainder of the exam is inconclusive. An abdominal ultrasound scan is arranged that reveals the presence of a 10 cm cystic lesion in the epigastric area. Which one the following is the most appropriate
management option?

A. Endoscopic gastrostomy.

B. Laparotomy.

C. Percutaneous drainage.

D. Drainage through ERCP.

E. Conservative management and re-evaluation in 6 months.

Incorrect. Correct answer is D


45% answered correctly

Explanation:

Correct Answer Is D

A pancreatic pseudocyst is a collection of pancreatic juice encased by reactive granulation tissue (not epithelial tissue) in or around the pancreas. Pseudocysts can be single or multiple, small or large, and can be located either within or outside the
pancreas. Most pseudocysts communicate with the pancreatic ductal system and contain high concentrations of digestive enzymes such as amylase and lipase.

The walls of pseudocysts are formed by adjacent structures such as the stomach, transverse mesocolon, gastrocolic omentum, and pancreas. The lining of pancreatic pseudocysts consists of fibrous and granulation tissue; the lack of an epithelial
lining distinguishes pseudocysts from true pancreatic cysts.

The mechanism by which a pseudocyst is formed is necrosis and liquefaction of the pancreatic necrosis of pancreatic or peripancreatic tissue.

Pseudocyst can be seen in the following situations:

After an episode of acute pancreatitis (in 10% of patients) - necrosis of peripancreatic tissue progresses to liquefaction and pseudocyst formation. Alternatively, a pseudocyst may result from parenchymal necrosis leading to the
complete ductal disruption, and gross leakage of pancreatic juice.

In patients with chronic pancreatitis – pseudocysts may develop after acute attacks of pancreatitis or after the pancreatic duct is obstructed. The latter causes increased intraductal pressure and leakage of pancreatic juice.

After blunt or penetrating abdominaltrauma (including iatrogenic injuries) - The injury can directly disrupt the duct and causes leakage.

Clinical manifestations and complications of pancreatic pseudocysts include:

Expansion of the pseudocyst can produce abdominal pain, duodenal or biliary obstruction, vascular occlusion, or fistula formation into adjacent viscera, the pleural space, or pericardium.

Spontaneous infection.

Digestion of an adjacent vessel can result in a pseudoaneurysm, which can produce a sudden, painful expansion of the cyst or gastrointestinal bleeding due to bleeding into the pancreatic duct.

Pancreatic pleural effusion -can result from disruption of the pancreatic duct with fistulization into the chest.

Pancreatic peritonitis - can be caused by disruption of the pancreatic duct with fistulisation to the abdomen.

NOTE - up to 40% of pseudocysts resolve without intervention; however, they can produce a wide range of clinical problems depending on the location and extent of the fluid collection and the presence of infection.

Pancreatitis pseudocysts are diagnoses with CT or ultrasound scan. Where the diagnosis is in doubt, the content can be aspirated (under endoscopic ultrasonography or CT scan) and examined.

The old rule mentioning that intervention is needed if the cysts are larger than 6 cm or persist beyond 6 weeks is no longer in use, and surgical intervention should be considered if any of the following is present:

Compression of large vessels (clinical symptoms or seen on CT scan)


Gastric or duodenal outlet obstruction
Stenosis of the common bile duct due to compression
Infected pancreatic pseudocysts
Hemorrhage into pancreatic pseudocyst
Pancreatico-pleural fistula
Pancreatic pseudocysts and symptoms:

Early satiety
Nausea and vomiting
Pain
Upper gastrointestinal bleeding

Asymptomatic pancreatic pseudocyst AND either of the following:

Pseudocysts> 5cm, unchanged in size and morphology for more than 6 weeks
Diameter> 4cm and extrapancreatic complications in patients with chronic alcoholic pancreatitis
Suspected malignancy

514 of 1943
Surgical drainage is the criterion standard against which all other interventions are measured in terms of success rate, mortality and recurrence rate. In recent years, however, endoscopic drainage has been introduced and can be applied provided
that the cyst is near the stomach or duodenal wall:

There are two main types of endoscopic drainage:

Transmural drainage: in this method, using endoscopy, a small incision is made in the stomach (endoscopic cystgastrostomy [ECG]) or in duodenum (endoscopic cystduodenostomy [ECD]) to let the pseudocyst drain into the stomach
or duodenum. ECD is preferred over ECG.

NOTE – pseudocysts should have a mature capsule (wall thickness>3mm and <1cm), bulge the lumen and have minimum size of 5-6 cm to become eligible for endoscopic drainage.

Transpapillary drainage: this method is safer and more effective than transmural drainage, but requires that the cyst communicates with the pancreatic duct because this method includes entering the pancreatic duct by ERCP, and from
there, into the pseudocyst. Stents may be left in place to facilitate drainage.

Generally, endoscopic drainage methods are preferred over open surgical treatment if eligibility is met and there is no contraindication because these methods are less invasive and associated with fewer complications.

Laparotomy with cyst excision and internal and external drainage is still the gold standard management option; however it is ONLY considered first-line therapy for surgical intervention in the following conditions:

Complicated pseudocysts i.e. infected or necrotic


Pseudocysts associated with pancreatic duct stricture and a dilated pancreatic duct
Suspected cystic neoplasia
Presence of pseudoaneurysm, unless it has been embolised before the procedure
Coexistence pseudocysts and bile duct stenosis
Complications such as compression of the stomach or the duodenum, perforation or pseudoaneursyms

NOTE – pseudoaneurysm is an absolute contraindication to endoscopic drainage unless it is embolized prior to the procedure.

Generally, patients with symptomatic pseudocysts should undergo interventional measures for pseudocyst drainage. The procedure of choice is endoscopic drainage. This cyst is 10 cm in size (>5cm) and amenable to endoscopic drainage either by
endoscopic transmural or transpapillary drainage. Transpapillary drainage has the lowest complication rate of all the mentioned procedures and is the method of choice if the pseudocyst communicates with the pancreatic duct. Fortunately, 80% of
pseudocysts communicate with the pancreatic duct.

(Option A) Endoscopic cystgastrostomy (ECG) or duodenostomy (ECD) are methods of choice if the pseudocyst is not communicating with the pancreatic duct.

(Option B) Laparotomy and surgical removal of the cyst is considered if endoscopic methods fail or there is a contraindication.

(Option C) Percutaneous catheter drainage has low success rate and high recurrent rates. It is never considered for treatment of a pancreatic pseudocyst. However, in infected pseudocysts it is the procedure of choice for sampling and examining the
material as the most appropriate initial step.

(Option E) Conservative management is not an appropriate option for symptomatic pseudocysts.

References

• HPB - Pancreatic pseudocysts – when and how to treat?

• Medscape - Pancreatic Pseudocysts

• Schwartz Principle of Surgery – McGraw Hill – 10th Edition – pages 1375-1378

Time spent: QID:495 Last updated:


2023-2-12

515 of 1943
A 34-year-old alcoholic man, who survived an episode of acute pancreatitis 4 weeks ago, has presented with mild discomfort in the epigastrium for the past few days. He is concerned that the disease might have recurred. He denies nausea and
vomiting and describes the pain as constant and nagging. Physical examination is unremarkable. A contrast abdominal CT scan is performed showing a 4-cm pancreatic pseudocyst. Which one of the following is the next best step in management?

A. Endoscopic decompression.

B. Open surface decompression.

C. Observation.

D. Percutaneous catheter drainage.

E. Urgently take him to the operating room.

Incorrect. Correct answer is C


45% answered correctly

Explanation:

Correct Answer Is C

A pancreatic pseudocyst is a collection of pancreatic juice encased by reactive granulation tissue (and not epithelial tissue) in or around the pancreas. Pseudocysts can be single or multiple, small or large, and can be located either within or outside
of the pancreas. Most pseudocysts communicate with the pancreatic ductal system and contain high concentrations of digestive enzymes such as amylase and lipase.

The walls of pseudocysts are formed by adjacent structures such as the stomach, transverse mesocolon, gastrocolic omentum, and pancreas. The lining of a pancreatic pseudocysts consists of fibrous and granulation tissue. Lack of an epithelial
lining distinguishes pseudocysts from true pancreatic cysts.

The mechanism by which a pseudocyst is formed is necrosis and liquefaction of the pancreatic necrosis of pancreatic or peripancreatic tissue.

Pseudocyst can be seen in the following situations:

After an episode of acute pancreatitis (in 10% of patients) - necrosis of peripancreatic tissue progresses to liquefaction and pseudocyst formation. Alternatively, a pseudocyst may result from parenchymal necrosis leading to the
complete ductal disruption, and gross leakage of pancreatic juice.

In patients with chronic pancreatitis - pseudocysts may develop after acute attacks of pancreatitis or after the pancreatic duct is obstructed. The latter causes increased Intraductal pressure and leakage of pancreatic juice.

After abdominal blunt or penetrating trauma (including iatrogenic injuries such as pancreatic surgery - injury can directly disrupt the duct and causes leakage.

Clinical manifestations and complications of pancreatic pseudocysts include:

Expansion of the pseudocyst can produce abdominal pain, duodenal or biliary obstruction, vascular occlusion, or fistula formation into adjacent viscera, the pleural space, or pericardium.

Spontaneous infection.

Digestion of an adjacent vessel can result in a pseudoaneurysm, which can produce a sudden, painful expansion of the cyst or gastrointestinal bleeding due to bleeding into the pancreatic duct.

Pancreatic pleural effusion -can result from disruption of the pancreatic duct with fistulization to the chest.

Pancreatic pleural effusion: can be caused by disruption of the pancreatic duct with fistulization to the abdomen.

NOTE - up to 40% of pseudocysts resolve without intervention; however, they can produce a wide range of clinical problems depending on the location and extent of the fluid collection and the presence of infection.

Pancreatitis pseudocysts are diagnoses with CT or ultrasound scan. If the diagnosis is in doubt, contents can be aspirated and examined.

The dictum mentioning that intervention is needed if the cysts are larger than 6 cm or persist beyond 6 weeks is no longer in use, and surgical intervention should be considered in (even one criterion is sufficient):

Compression of large vessels (clinical symptoms or seen on CT scan)


Gastric or duodenal outlet obstruction
Stenosis of the common bile duct due to compression
Infected pancreatic pseudocysts
Hemorrhage into pancreatic pseudocyst
Pancreatic-pleural fistula
Pancreatic pseudocysts and symptoms:

Early satiety
Nausea and vomiting
Pain
Upper gastrointestinal bleeding

Asymptomatic pancreatic pseudocyst AND either of the following:

Pseudocysts> 5cm, unchanged in size and morphology for more than 6 weeks
Diameter> 4cm and extrapancreatic complications in patients with chronic alcoholic pancreatitis
Suspected malignancy

Surgical drainage (laparotomy and internal and external drainage) is the criterion standard against which all other interventions are measured in terms of success rate, mortality and recurrence rate. In recent years however endoscopic drainage has
been introduced and can be applied provided that the cyst is near the stomach or duodenal wall:

There are two main types of endoscopic drainage:

516 of 1943
Transmural drainage: in this method, using endoscope a small incision is made in the stomach (endoscopic cystgastrostomy [ECG]) or in duodenum (endoscopic cystduodenostomy [ECD]) to let the pseudocyst drain into the stomach
or duodenum. ECD is preferred over ECG.

Transpapillary drainage: this method is safer and more effective than transmural drainage, but requires that the cyst communicates with pancreatic duct. Using ERCP and through the pancreatic duct, the cyst is reached and poked so
that its content can drain into the pancreatic duct. Stents may be used to facilitate drainage.

NOTE – pseudocysts should have a mature capsule (wall thickness>3mm and <1cm), bulge the lumen and have minimum size of 5-6 cm to become eligible for endoscopic drainage.

Generally, endoscopic drainage methods are preferred over open surgical treatment if eligibility is met and there is no contraindication, because these methods are less invasive and associated with fewer complications.

Laparotomy with cyst excision and internal and external drainage is still the gold standard management option; however it is considered first-line therapy for surgical intervention if:

Complicated pseudocysts i.e. infected and necrotic pseudocysts


Pseudocysts associated with pancreatic duct stricture and a dilated pancreatic duct
Suspected cystic neoplasia
Presence of pseudoaneurysm unless it has been embolized before the procedure
Coexistence pseudocysts and bile duct stenosis
Complications such as compression of the stomach or the duodenum, perforation or pseudoaneursyms

NOTE – pseudoaneurysm is an absolute contraindication to endoscopic drainage unless it is embolized prior to the procedure.

Generally, patients with symptomatic pseudocysts should undergo interventional measures for pseudocyst drainage. The procedure of choice is endoscopic drainage, but as mentioned before, for a pseudocyst to be amenable to this procedure, it
has to be at least 5-6 in size as well as bulging into the lumen. This cyst with 4 cm in size is unlikely to be drained by endoscopic measures; therefore, open decompression (surgical drainage) should be considered; however, since the pain is mild, a
watchful observation would be best management here in an attempt to avoid the high rates complications associated with surgical drainage. If the symptoms were more pronounce, intolerable or indicative of more serious complications, surgical
drainage (open surface drainage) would have been the option of choice.

Percutaneous catheter drainage has low success rate and high recurrent rates. It is never considered for treatment of a pancreatic pseudocyst. However, in infected pseudocysts it is the procedure of choice for sampling and examining the material
as the most appropriate initial step.

References

• HPB - Pancreatic pseudocysts – when and how to treat?

• Medscape - Pancreatic Pseudocysts

• Schwartz Principle of Surgery – McGraw Hill – 10th Edition – pages 1375-1378

Time spent: QID:496 Last updated:


2023-2-12

517 of 1943
A 51-year-old Aboriginal male with history of acute pancreatitis presents with persistent abdominal pain and loss of appetite. On examination, an abdominal mass is found. A contrast abdominal CT scan is performed one cut of which shown in the
following photograph. Which one of the following is the most likely diagnosis?

A. Chronic pancreatitis.

B. Acute pancreatitis.

C. Pancreatic pseudocyst.

D. Gastric adenocarcinoma.

E. Gastric lymphoma.

Incorrect. Correct answer is C


45% answered correctly

Explanation:

Correct Answer Is C

The round lesion with hypodense homogenous content and the thin smooth wall in the vicinity of and obliterating the pancreas is highly suggestive of a pancreatic cyst or pseudocyst. With the history of previous acute pancreatitis, a pseudocyst
would be the most likely diagnosis.

A pancreatic pseudocyst is a collection of pancreatic juice encased by reactive granulation tissue (and not epithelial tissue) in or around the pancreas. Pseudocysts can be single or multiple, small or large, and can be located either within or outside
of the pancreas. Most pseudocysts communicate with the pancreatic ductal system and contain high concentrations of digestive enzymes such as amylase and lipase.

The walls of pseudocysts are formed by adjacent structures such as the stomach, transverse mesocolon, gastrocolic omentum, and the pancreas. The lining of pancreatic pseudocysts consists of fibrous and granulation tissue. Lack of an epithelial
lining distinguishes pseudocysts from true pancreatic cysts.

The mechanism by which a pseudocyst is formed is necrosis and liquefaction of the pancreatic necrosis of pancreatic or peripancreatic tissue.

Pseudocyst can be seen in the following situations:

After an episode of acute pancreatitis (in 10% of patients) - necrosis of peripancreatic tissue progresses to liquefaction and pseudocyst formation. Alternatively, a pseudocyst may result from parenchymal necrosis leading to the
complete ductal disruption, and gross leakage of pancreatic juice.

In patients with chronic pancreatitis - pseudocysts may develop after acute attacks of pancreatitis or after the pancreatic duct is obstructed. The latter causes increased Intraductal pressure and leakage of pancreatic juice.

After abdominal blunt or penetrating trauma (including iatrogenic injuries such as pancreatic surgery - injury can directly disrupt the duct and causes leakage.

Clinical manifestations and complications of pancreatic pseudocysts include:

Expansion of the pseudocyst can produce abdominal pain, duodenal or biliary obstruction, vascular occlusion, or fistula formation into adjacent viscera, the pleural space, or pericardium

Spontaneous infection

Digestion of an adjacent vessel can result in a pseudoaneurysm, which can produce a sudden, painful expansion of the cyst or gastrointestinal bleeding due to bleeding into the pancreatic duct

Pancreatic pleural effusion -can result from disruption of the pancreatic duct with fistulisation to the chest

Pancreatic peritonitis - can be caused by disruption of the pancreatic duct with fistulisation to the abdomen

References

• HPB - Pancreatic pseudocysts – when and how to treat?

• Medscape - Pancreatic Pseudocysts

• Schwartz Principle of Surgery – McGraw Hill – 10th Edition – pages 1375-1378

L t d t d

518 of 1943
Last updated:
Time spent: QID:497 2023-2-12

519 of 1943
A 45-year-old man presents with signs and symptoms of gastro-esophageal reflux disease (GERD) and is started on a course of omeprazole 20 mg BID for 6 weeks with complete resolution of the symptoms. Because of long-standing symptoms, an
upper endoscopy is performed. The endoscopic view of the distal esophagus is shown in the following photograph. Histopathology shows intestinal metaplasia, but no dysplasia. Which one of the following is most appropriate next step in
management?

A. Pneumatic dilation.

B. Continue PPIs with doubled dose.

C. Endoscopy after 24 months.

D. Endoscopy after 6 months.

E. No intervention is needed.

Incorrect. Correct answer is C


45% answered correctly

Explanation:

Correct Answer Is C

The photograph shows Barrett’s esophagus. Barrett’s esophagus refers to metaplasia of the squamous epithelium of the distal esophagus to columnar epithelium of the intestinal type. The metaplasia is believed to be due to GERD in a susceptible
individual.

Anatomical landmarks on endoscopic view of the Barrett's esophagus - www.barrettsadvice.com

Barrett’s esophagus is often associate with severe reflux disease, esophagitis, or strictures, but is also seen in asymptomatic or minimally symptomatic patients.

Barrett’s esophagus is a premalignant condition with a 40- to 125-fold increase in the incidence of adenocarcinoma of the lower esophagus; however, the absolute risk is relatively small and approximately 0.5% per year.

Every patient with the alarming symptoms of weight loss, anemia, heme-positive stool, and dysphagia, or GERD for more than 5 years should undergo endoscopy and biopsy. Further management depends on the histological findings and is
summarized in the following table:

Endoscopic Findings Management


Barrett’s esophagus (metaplasia) Repeat endoscopy every 2-3 years for lesions ≥ 3cm and every 3-5 years for lesions <3 cm.
Low-grade dysplasia Repeat endoscopy in 6 months.
High grade dysplasia Distal esophagetomy.

This patient has metaplastic changes involving an area of more than 3cm; therefore, the next step in management would be repeating the endoscopy in 2-3 years.

520 of 1943
(Option A) Pneumatic dilation is not a management option for GERD or Barrett's esophagus.

(Option B) Although proton pump inhibitors (PPIs) may be associated with the appearance of squamous islands and possibly some degree of regression, there is no evidence that PPIs reduce the incidence of esophageal cancer in patient’s with
Barrett’s esophagus.

(Option D) Endoscopy after 3-6 months (current recommendations: every 6 months) was the choice of management if low-grade dysplasia (not metaplasia) would be found on histological studies.

(Option E) This patient needs close surveillance and follow-up and action should be taken for early detection of esophageal malignancies.

References

• Cancer Council Australia - Barrett's Oesophagus without Dysplasia

• Therapeutic Guidelines – Gastroenterology; available from http://tg.org.au

Time spent: QID:156 Last updated:


2023-2-12

521 of 1943
A 39-year-old Aboriginal man presents to your practice with a 6-week history of abdominal pain, nausea, vomiting and bowel motions that are difficult to flush down the toilet. The abdominal pain is described as a constant and disabling radiating to
back. The pain is neither brought up, nor alleviated by eating. He is a chronic alcoholic and continues to drink alcohol in large amounts. Which one of the following investigations is most likely to reveal the underlying cause of his symptoms?

A. Investigations for exocrine function of pancreas.

B. Stools exam.

C. Transabdominal ultrasound.

D. ERCP.

E. Plain abdominal X-ray.

Incorrect. Correct answer is C


45% answered correctly

Explanation:

Correct Answer Is C

The clinical picture and history of excessive alcohol use makes chronic pancreatitis the most likely diagnosis. Chronic pancreatitis presents with epigastric pain as the most dominant feature. The natural history of pain in chronic pancreatitis is
highly variable. Most patients experience intermittent attacks of pain at unpredictable intervals, while a minority of patients have chronic pain. In most patients, pain severity either decreases or resolves over 5-25 years. In alcohol-induced disease,
alcohol cessation may reduce the severity of pain.

The pain is constant, often radiates to back and may be associated with nauseas and vomiting. It is often felt in the epigastric area but may be felt on the left side or even right side. The pain may or may not be related to eating. If related, there may
be weight loss due to fear of eating. At times of pain patients bend forward to the so-called “pancreatic position” or lie in the knee-chest position on their right or left side to decrease the pain intensity.

Patients with severe pancreatic exocrine dysfunction cannot properly digest complex foods and absorb partially digested breakdown products. Nonetheless, clinical significant protein and fat deficiencies does not develop until more than 90% of the
pancreatic function is lost. However, fear of eating due to pain brought up by eating, can result in early weight loss. With severe chronic pancreatitis, insulin deficiency and diabetes mellitus may also develop.

Abdominal CT scan, MRI, ultrasound, plain X-ray films and ERCP can be used for diagnosis, but CT scan is the best initial test used for imaging studies when chronic pancreatitis is suspected. CT scans are 75-90% sensitive and 85% specific.
Ultrasound is the second-line initial imaging study with a sensitivity of 60-70% and specificity of 80-90% and the best among option in the absence of CT scan.

Calcifications within the pancreatic duct are present on plain films in approximately 30% of cases, making palne films a less desirable option.

The characteristic finding on imaging is pancreatic duct calcifications, ductal dilation, enlargement of the pancreas and fluid collection.

Calcium deposition is most commonly seen in alcoholic pancreatitis, but is also present hereditary and tropical forms of the disorder. It is rare in idiopathic pancreatitis.

Magnetic resonance cholangiopancreatography (MRCP) is becoming the diagnostic test of choice since it can show calcification, without any radiation risks, but is expensive and not readily available. For this reason, it is not an ideal option for initial
assessment.

Endoscopic retrograde cholangiopancreatography (ERCP) is reserved for situations where non-invasive modalities are not available, are equivocal and for intervention.

References

• http://www.uptodate.com/contents/clinical-manifest

Last updated:
Time spent: QID:501 2023-2-12

522 of 1943
A 45-year-old man presents to the emergency department with acute onset epigastric and right upper quadrant abdominal pain that he describes as constant and severe. He is nauseous and has vomited 3 times since the pain started. He admits to
chronic heavy alcohol consumption. On examination, he has a blood pressure of 140/90mmHg, pulse rate of 110bpm and temperature of 38.4°C. The epigastric area is mildly tender to palpation. Which one of the following is the most likely
diagnosis?

A. Acute cholecystitis.

B. Acute pancreatitis.

C. Acute gastritis.

D. Acute gastro-esophageal reflux disease.

E. Alcohol intoxication.

Incorrect. Correct answer is B


45% answered correctly

Explanation:

Correct Answer Is B

The clinical picture and the history of heavy alcohol use suggest acute pancreatitis as the most likely diagnosis.

The cardinal symptom of acute pancreatitis is abdominal pain. The pain is characteristically described as dull, boring and constant. The pain is often sudden-onset and gradually becomes worse. Most often, the pain is felt in the upper abdomen
usually in the epigastric region, but sometimes is perceived more on the right or left side, depending on which part of the pancreas is involved. In about 50% of the patients, the pain directly radiates through the abdomen to the back. The duration of
pain is variable but typically lasts more than 1 day.

Restless and agitation may be noted. At times of pain patients bend forward to the so-called “pancreatic position” or lie in the knee-chest position. Anorexia, nausea and vomiting, and diarrhea are other likely symptoms. Acute pancreatitis secondary
to alcohol, frequently occurs 1 to 3 days after a binge drinking.It can also occur after cessation of drinking.

The patient should be asked about recent operative or other invasive procedures such as ERCP, family history of hypertriglyceridemia, previous biliary colic and binge alcohol drinking as the major causes of acute pancreatitis.

(Option A) Acute cholecystitis causes pain in right upper quadrant, fever and leukocytosis. Murphy’s sign is usually positive: the patient is asked to inspire deeply while the right subcostal area is palpated. The Murphy sign is cessation of inspiration
due to pain. Patients with acute cholecystitis may experience increased discomfort and hold in mid-inspiration. Abdominal examination may show voluntary and involuntary guarding.

(Option B) Acute gastritis usually can present with pain in epigastrium associated with nauseas and vomiting and epigastric tenderness but fever would not be a feature.

(Option D) Gastroesophageal reflux disease (GERD) presents with heartburn, chronic cough, a metalic taste in the mouth. The given clinical picture is completely different from that of GERD.

(Option E) Patients with lcohol intoxication presents with slurred speech, nystagmus, disinhibited behavior, incoordination, unsteady gate, memory impairment, stupor, or coma depending on the severity of intoxication.

References

• http://www.uptodate.com/contents/acute-pancreatiti

Last updated:
Time spent: QID:503 2023-2-12

523 of 1943
The following photograph is one cut from an abdominal CT scan in a 68-year-old man. Which one of the following is the most likely diagnosis?

A. Gastric outlet obstruction.

B. Cancer of the stomach.

C. Simple hepatic cyst.

D. Pancreatic pseudocyst.

E. Hepatic hemangioma.

Correct
45% answered correctly

Explanation:

Correct Answer Is A

The picture shows a contrast CT scan of the abdomen. On the right side of the picture (left side of the patient) a distended contrast-filled stomach is seen. Duodenum is on the left side of the picture. It also contains contrast media.

The right kidney is another structure seen on the CT scan. In front of the vertebral body and slightly to the left, the aorta (containing contrast material) is noticed. Other structures seen on this cut of the CT scan are lower part of the right hepatic lobe,
left hepatic lobe, spleen and parts of the pancreas.

The distended and fluid filled stomach is suggestive of gastric outlet obstruction (GOO). The distended duodenum indicates that the obstruction has occurred at the duodenum level.

Clinical entities that can result in GOO generally are categorized into two groups – benign and malignant. In the past, peptic ulcer disease (PUD) was the most common cause of GOO, but currently, 50-80% of GOOs are due to malignancies such as
pylorus adenocarcinoma, lymphoma and gastrointestinal stromal tumour (GIST).

Benign causes of GOO include pancreatic pseudocysts, gastric varices, infections such as tuberculosis, and rarely gall stones.

At this level no pancreatic or hepatic lesion is seen. Cancer of the stomach would have given an obstruction in upper parts if it involves the pylorus. Besides, this might be the underlying cause of the CT scan findings not the interpretation.

References

• Medscape - Gastric Outlet Obstruction

• UpToDate - Gastric outlet obstruction in adults

Last updated:
Time spent: QID:528 2023-2-12

524 of 1943
A 35-year-old woman presents with a 4-month history of dysphagia to both solid and liquid foods. A barium study is done demonstrating dilated esophagus with tapering at the lower end. Based on the history and radiological findings, achalasia is
the diagnosis. Which one of the following is the most appropriate management in this patient?

A. Pyloromyotomy.

B. Pneumatic dilation.

C. Botulinum toxin injection.

D. Calcium channel blockers.

E. Nitrates.

Incorrect. Correct answer is B


45% answered correctly

Explanation:

Correct Answer Is B

Achalasia is a primary esophageal motility disorder characterized by the absence of esophageal peristalsis and impaired relaxation of the lower esophageal sphincter (LES) in response to swallowing. These abnormalities cause a functional
obstruction at the gastroesophageal junction.

The goal of treatment is to reduce the resistance of the LES and overcome the obstruction.

Currently, standard of care for achalasia is based on guidelines from the American College of Gastroenterology and is as follows:

Initial therapy should be either graded pneumatic dilation (PD) or laparoscopic surgical myotomy in patients who are fit for such surgeries.

Botulinum toxin injection into the LES is considered for patients who are not appropriate candidates for surgery.

Pharmacologic therapy can be used for patients not undergoing PD or myotomy and who have failed botulinum toxin therapy (nitrates and calcium channel blockers most common)

For this patient, either pneumatic dilation or endoscopic myotomy (not an option) is the most appropriate initial management option.

(Option A) Pyloromyotomy is the procedure used for treatment of hypertrophic pyloric stenosis or obstruction.

(Option C) In patients with mild symptoms or in those who are not appropriate candidates for pneumatic balloon dilation or surgery, injection of botulinum toxin is considered the next most appropriate treatment. Botulinum toxin injection has been
associated with symptoms improvement, but may need to be repeated at intervals of 3-12 months.

(Options D and E) Pharmacological therapy can be used for those patients not undergoing pneumatic dilation or myotomy, and who have failed botulinum toxin therapy. These medications, however, may not be desirably effective. First-line medical
treatment is with glyceryl trinitrate (oral or sublingual spray). If the episodes are associated with spasms or are frequent or disabling, a trial of diltiazem, nifedipine, or isosorbide dinitrate can be considered.

References

• Medscape - Achalasia

• Therapeutic Guidelines – Gastrointestinal; available from http://tg.org.au

Last updated:
Time spent: QID:529 2023-2-12

525 of 1943
A 72-year-old woman presents to the emergency department with sudden-onset severe pain in the right lower quadrant and and back pain. She has the history of deep vein thrombosis (DVT) two months ago, for which she was started on warfarin. On
examination, she has a blood pressure of 110/75 mmHg, pulse rate of 140 bpm and temperature of 37.2°C. There is a tender mass in the right iliac fossa (RIF). Plain X-ray of her abdomen is obtained and is shown in the following photograph. Which
one of the following is the most likely diagnosis?

A. Abscess of the appendix.

B. Rectus sheath hematoma.

C. Cecal volvulus.

D. Leaking abdominal aortic aneurysm.

E. Fecal impaction.

Incorrect. Correct answer is C


45% answered correctly

Explanation:

Correct Answer Is C

The X-ray shows an extensively air-filled dilated proximal colon arising from the right side and extending to the left. With these X-ray findings, along with the history and physical findings, cecal volvulus is the most likely diagnosis.

Volvulus occurs when a segment of viscus twists around its axis and results result in obstruction as well as blood supply compromise.

The most common types of volvulus are sigmoid volvulus (more common) and cecal volvulus (less common). Cecal volvulus tends to occur in younger patients (30-60 years) compared to sigmoid volvulus that often is seen in elderly patients, who
are bedridden or nursing home residents.

526 of 1943
Cecal volvulus – significant dilation of the cecum (dilation arising from the right side).
Distal to the dilation, the colon is prolapsed.
Other prominent finding is thick-walled dilated small bowel loops highly
suggestive of either the distal obstruction of the small bowel
or proximal obstruction of the large bowel.

(Option A) There are no suggestive clues to acute appendicitis as a precipitating event for an appendix abscess to form.

(Option B) Although the clinical findings and history of warfarin use can make rectus sheath hematoma a likely diagnosis, the abdominal X-ray is consisetent with csecal volvulus.

(Option D) With leaking abdominal aortic aneurysm, a tender pulsatile mass would be expected in the midline, not in the right iliac fossa.

(Option E) With fecal impaction, fecal material was expected to be seen in colon (left side). Focal tenderness is not a dominant feature in fecal impaction.

References

• BMJ - Diagnosis and treatment of cecal volvulus

• Radiopaedia - Caecal volvulus

Time spent: QID:589 Last updated:


2023-2-12

527 of 1943
A 65-year-old man is being evaluated for assessment of the source of gastrointestinal bleeding after he presented with complaint of melena at several occasions and was found to be anemic. Due to the very low level of hemoglobin and symptoms
such as severe weakness and chest pain on exertion, he received several transfusions; however, he was not able to maintain a satisfactory hemoglobin level. Upper and lower endoscopy and small bowel barium studies were performed failing to find
a source of bleeding. Which one of the following is the next best step in management?

A. Capsule endoscopy.

B. Interventional angiography.

C. CT angiography.

D. Repeat endoscopy.

E. Radionuclide imaging.

Incorrect. Correct answer is C


45% answered correctly

Explanation:

Correct Answer Is C

The case scenario is an example of obscure gastrointestinal tract (GIT) bleeding. Obscure GIT bleeding is defined as bleeding from the GIT that persists or recurs without an obvious etiology found on upper endoscopy and colonoscopy and
radiologic evaluation of the small bowel (such as by small bowel follow-through or enteroclysis).

Obscure GIT bleeding accounts for approximately 5% of patients with GIT bleeding. In 75% of these patients, the source of bleeding is in small bowel. The remainder of cases are due to missed lesions in either upper or lower GI tract.

Obscure bleeding is subdivided into overt or occult, depending upon the presence or absence of clinically-evident bleeding:

Obscure occult (inactive) bleeding – This is manifested as iron deficiency anemia or recurrent positive Fecal Occult Blood Test (FOBT) results.

Obscure overt (active) bleeding – This is manifested as recurrent episodes of clinically evident bleeding (e.g., melena or hematochezia), or persistent blood loss. Patients with ongoing blood loss so significant to necessitate transfusion are
considered to have life-threatening active bleeding.

Further management of obscure GI bleeding depends on whether the bleeding is clinically active (overt) or inactive (occult):

Active obscure GI bleeding:

In patients presenting with active bleeding, radiological investigations are the mainstay of diagnostic approach, given their high sensitivity and non-invasiveness. CT Angiography (CTA) can detect bleeding rates of 0.5 ml/min and above. A negative
CTA signifies that catheter angiography is not indicated since the latter is less sensitive than the CTA and has been shown to detect the source of bleeding only if the rate of blood loss is greater than 0.5 ml/min.

With a negative CTA and continuous bleeding, technetium 99m – labeled RBC nuclear scan is used as the second-line investigation at some institutions while others may use this as the first-line investigation. Among available modalities, RBC nuclear
scan is most sensitive for active GI bleeding and can detect bleeding rates as low as 0.1 mL/min.

NOTE - Life-threatening hemorrhage should indicate catheter angiography as first-line management. A hemorrhage so significant is very likely to light up on catheter angiography. With catheter angiography, the site of bleeding can be found
and controlled at the same time.

Inactive obscure GI bleeding:

Capsule endoscopy is the option of choice. However, a negative test demands CT angiography or other radiologic modalities (radionuclide scan, etc.) for further assessment.

This patient has low hemoglobin despite several transfusions; however, he does not have urgent life-threatening hemodynamic instability such as low blood pressure, tachycardia, shortness of breath, chest pain, or hypoperfusion-related altered
mental state. For him a CTA would be the most appropriate next step in management to find the source of bleeding.

(Option A) Capsule endoscopy is the first option for patients with occult obscure GI bleeding i.e. iron deficiency anemia or recurrent positive Fecal occult blood test (FOBT) results.

(Option B) Interventional angiography was the option of choice if the patient had immediate life-threatening hemodynamic instability at the moment. Although this patient has low hemoglobin, more transfusion can temporarily stabilize him to fulfill
the criteria for CTA.

(Option D) Repeating the endoscopic studies is unlikely to add any information regarding the site of bleeding, as this measure has already failed to do so.

(Option E) Radionuclide imaging is used when there is obscured GI bleeding of unknown origin despite CT angiography or capsule endoscopy. The test takes a long time to perform. This patient with brisk and significant blood loss is not a good
candidate for this test.

References

• Diagnostic Imaging Pathways - Gastrointestinal Bleeding (Obscure)

• Diagnostic Imaging Pathways - Gastrointestinal Bleeding (Acute)

Last updated:
Time spent: QID:644 2023-2-12

528 of 1943
Which one of the following is the most common presenting symptom of a patient with duodenal stricture secondary to duodenal ulcer?

A. Vomiting 3-4 times a day.

B. Abdominal distension.

C. Right upper quadrant pain.

D. Vomiting after one hour of meals.

E. Vomiting early in the morning.

Incorrect. Correct answer is D


45% answered correctly

Explanation:

Correct Answer Is D

Gastric outlet obstruction (GOO), is the consequence of any disease producing a mechanical obstruction to gastric emptying. Clinical entities that can cause GOO are generally categorized into benign and malignant.

Peptic ulcer disease (PUD) is among the benign causes of GOO; however, the incidence of ulcer-induced GOO has dramatically declined owing to adequate and efficient treatment of PUD. The mechanism of obstruction by PUD can be either edema
around the ulcer, or scar formation after the ulcer heals.

The most common symptoms of GOO, regardless of the underlying etiology, are bloating, anorexia, nausea and vomiting. Vomiting usually is described as nonbilious, and characteristically containing undigested food particles.

Patients with gastric outlet obstruction from a duodenal ulcer or incomplete obstruction typically present with symptoms of gastric retention, including early satiety, bloating or epigastric fullness, indigestion, anorexia, nausea, vomiting, epigastric
pain, and weight loss. They are frequently malnourished and dehydrated and have a metabolic insufficiency. Weight loss is frequent when the condition is chronic and is most significant in patients with malignant disease.

Abdominal pain is not frequent and usually relates to the underlying cause, e.g., PUD, pancreatic cancer.

The time of vomiting can suggest the site of obstruction. In pyloric obstruction (more proximal) the time of vomitus is usually within the first hour of eating, whereas in pyloric stenosis, or duodenal stenosis or obstruction (more distal) the vomiting
occurs after one hour because normally it takes 45 minutes to 1 hour for the food to reach the duodenum.

(Option A) If the patient has 3 or 4 meals a day he may vomit 3 to 4 times within one hour after each meal, but vomiting 3 to 4 times a day by itself is not a common presentation of gastric outlet obstruction.

(Option C) Right upper quadrant abdominal pain is seen in hepatobiliary diseases and is not a characteristic feature of gastric outlet obstruction.

(Option B) Abdominal distension is a feature of bowel obstruction, particularly obstruction of the large bowel.

(Option E) Early morning vomiting is associated with raised intra-cranial pressure and pregnancy, and is not a feature of gastric outlet obstruction.

References

• Medscape - Gastric Outlet Obstruction

Last updated:
Time spent: QID:645 2023-2-12

529 of 1943
A 56-year-old man presents to the emergency department with acute-onset severe epigastric pain, fever, nausea and vomiting. Past medical history includes type 2 diabetes mellitus, cardiomyopathy, and hypertension. Abdominal ultrasound shows a
stone in the bile duct, multiple gallstones in the gallbladder and a dilated common bile duct of 6mm. Laboratory studies are significant for elevated conjugated bilirubin, elevated white cell count, and a normal hemoglobin level. Liver function tests
show a 6-time rise in alanine aminotransferase and aspartate aminotransferase. Lipase and amylase are also elevated. Which one of the following options is the most appropriate next step in management?

A. Abdominal CT scan.

B. ERCP.

C. MRCP.

D. Upper endoscopy.

E. Monitor liver function tests.

Incorrect. Correct answer is B


45% answered correctly

Explanation:

Correct Answer Is B

The clinical and laboratory picture suggests acute pancreatitis secondary to gallstones and common bile duct obstruction that has led to LFT abnormalities as well.

Gallstone pancreatitis necessitates specific therapeutic consideration in addition to conventional treatment of acute pancreatitis.

Multiple studies suggest that early endoscopic retrograde cholangiopancreatography (ERCP), within 24-48 hours, with papillotomy, or surgical intervention to remove bile duct stones is beneficial in patients with acute biliary pancreatitis and the
presence of the following:

Concomitant acute cholangitis


Persisted common bile duct obstruction evident by worsening signs or symptoms of obstruction (clinical deterioration), or deteriorating liver function tests

Endoscopic papillotomy is typically accompanied by placement of a plastic biliary stent to decrease the risk of post-ERCP pancreatitis.

With increased liver function tests (LFTs), especially the elevated conjugated bilirubin, this patient should be considered to have concomitant cholangitis until proven otherwise. Considering the likelihood of acute cholangitis caused by CBD
obstruction, this patient should undergo early ERCP as the most appropriate next step in management, after rehydration, analgesia, and antibiotics. LFTs are used to monitor the response to treatment.

An ultrasound scan (not an option) is the very first imaging investigation for patients with signs and symptoms of pancreatitis or pnacreatic cancer if manifestations of biliary obstruction are also present. Otherwise, Abdominal CT scan is used;
However, in the absence of clinical features of biliary obstruction, CT scan is the preferred diagnostic modality.

CT scan is the most important imaging test for the diagnosis of acute pancreatitis, its severity and potential complication Patients with clinical and biochemical features of acute pancreatitis. CT scan is indicated if:

there is no improvement with conservative therapy


complications are suspected
other diagnoses than pancreatitis is suspected

References

• Medscape- Acute Pancreatitis Treatment & Management

• UpToDate - Management ofacute pancreatitis

Last updated:
Time spent: QID:646 2023-2-12

530 of 1943
A 70-year-old man presents to the emergency department with acute epigastric pain, nausea and vomiting. He has the history of gallstones. Laboratory tests show moderate elevation of lipase, amylase, alanine aminotransferase (ALT) and aspartate
aminotransferase (AST). Both conjugated and unconjugated bilirubin are within the normal range. Inflammatory markers are not elevated. Abdominal ultrasound reveals a small stone in the common bile duct (CBD), but CBD is not dilated. Which one
of the following is the most appropriate next step in management?

A. Abdominal CT scan.

B. MRCP.

C. ERCP.

D. Intravenous fluids, analgesics and antiemetics.

E. Monitoring the liver function tests.

Incorrect. Correct answer is D


45% answered correctly

Explanation:

Correct Answer Is D

This patient has clinical diagnosis of acute pancreatitis, most likely secondary to gallstones; however, a non-dilated common bile duct (CBD) and normal bilirubin excludes CBD obstruction. Management of the acute pancreatitis should be started
with intravenous fluids, analgesia and antiemetics. The patient should be nil by mouth (NPO) and NG tube inserted for GI decompression.

Further attention then should be paid to biliary stones as the underlying cause. Early ERCP (within 24-48 hours) is recommended for patients with acute biliary pancreatitis and either of the following:

Concomitant acute cholangitis


Persisted common bile duct obstruction evident by worsening sign or symptoms of obstruction, or deteriorating liver function tests

Elevated liver enzymes and/or CBD stone on ultrasound are pointers towards a biliary cause for acute pancreatitis; however, normal level of bilirubin and the non-dilated of CBD on ultrasound exclude cholangitis with high certainty. For this patient
early ERCP is not indicated, but ideally the patient should undergo semi-elective cholecystectomy during this hospital stay.

CT scan is the most important imaging test for diagnosis of acute pancreatitis and its intraabdominal complications and also for assessment of severity. Patients with clinical and biochemical features if acute pancreatitis, who do not improve with
initial conservative therapy or who are suspected of having complications or other diagnoses should undergo CT scan of the abdomen. This patient may need CT scan later (usually not earlier than 72 hours) in the course of the disease but not now
as the most appropriate next step.

Monitoring LFTs are required for assessment of response to treatment, but not a priority in initial management.

References

• http://emedicine.medscape.com/article/181364-overv

• http://www.ncbi.nlm.nih.gov/pmc/articles/PMC299908

• http://www.uptodate.com/contents/management-of-acu

Last updated:
Time spent: QID:647 2023-2-12

531 of 1943
A 49-year-old man with smoking history of 30 pack/year and longstanding gastro-esophageal reflux disease (GERD) presents with progressive difficulty in swallowing for the past 6 months. Initially, he had difficulty swallowing solid foods, but later he
also developed dysphagia to liquids. He has malaise, fatigue, and has lost 15 kg in the past 3 months. On clinical examination, he looks pale and anemic. Which one of the following would be the next best step in management?

A. CT scan of the chest.

B. Barium Swallow.

C. Esophagoscopy.

D. Bronchoscopy.

E. Chest X-ray.

Incorrect. Correct answer is C


45% answered correctly

Explanation:

Correct Answer Is C

The clinical picture including the history of heavy smoking and GERD, as well as the characteristics of the dysphagia is suggestive of mechanical dysphagia most likely caused by esophageal cancer. Weight loss can be seen in any kind of dysphagia;
however, a significant weight loss of 15 kg in only 3 months suggests a more malignant etiology. Anemia is another clue to malignancy.

When esophageal cancer is suspected, the best initial test is endoscopy (esophagoscopy) and biopsy for definite diagnosis.

Barium swallow (option B) is very sensitive for detection of strictures and intraluminal masses but does not allow staging and biopsy. It is now rarely used when cancer is suspected.

Bronchoscopy (option D), chest X-ray (option E), and CT scan of the chest (option A) are all used as tools for further evaluation and staging of the tumor, once esophagoscopy established a diagnosis of esophageal cancer.

Bronchoscopy may be used for malignancies of upper (and middle) esophagus because it is common for tumors of these parts to invade the tracheobronchial tree.

NOTE - The extent of tumor spread to the mediastinum and para-aortic lymph nodes should be assessed by CT scanning of the chest and abdomen and endoscopic ultrasound. Endoscopic ultrasonography is the most sensitive test for
assessment of the tumor depth. Other studies to consider for staging are bone scan and liver function tests.

References

• Diagnostic Imaging Pathways - Dysphagia

• Medscape - Esophageal Cancer

• UpToDate - Clinical manifestations, diagnosis, and staging of esophageal cancer

Last updated:
Time spent: QID:648 2023-2-12

532 of 1943
Which one of the following factors has the greatest impact on development of esophageal adenocarcinoma?

A. Achalasia.

B. Barrett’s esophagus.

C. Low-fiber diet.

D. Smoking.

E. Gastro-esophageal reflux disease.

Incorrect. Correct answer is B


45% answered correctly

Explanation:

Correct Answer Is B

While smoking and alcohol are the two most common risk factors for squamous cell carcinoma of the esophagus, Barrett's esophagus is the most significant risk factor for development of esophageal adenocarcinoma.

(Option A) Achalasia has been associated with esophageal cancer but is not as significant as other factors such as smoking and alcohol in squamous cell carcinoma and Barret's esophagus in adenocarcinoma of the esophagus.

(Option C) Although implicated in some studies, low-fiber diet is not as significant as other risk factors for esophageal cancer.

(Option D) Smoking and chronic alcohol ingestion are the most common and important risk factors for development of esophageal squamous cell carcinoma.

(Option E) Gastro-esophageal reflux disease is not a risk factor for esophageal adenocarcinoma unless it results in Barrett’s esophagus (indirect effect).

NOTE - Obesity is considered another important risk factor, mostly because it predisposes to reflux disease. However, the outcome of these conditions – GERD and the consequent Barrett’s esophagus, remains the main and most significant
risk factor through producing metaplastic changes that can progress to dysplasia and eventually cancer.

References

• Medscape - Esophageal Cancer

• BMJ - Adenocarcinoma of oesophagus: what exactly is the size of the problem and who is at risk?

Last updated:
Time spent: QID:158 2023-2-12

533 of 1943
A 42-year-old woman presents to the emergency department with right sided abdominal pain, mild jaundice and fever. She has a blood pressure of 130/85 mmHg, pulse rate of 90 bpm and temperature of 38.1°C. She is admitted to the hospital. After
3 hours, she develops rigors and chills. On re-exam, her blood pressure has fallen to 90/60 mmHg, her pulse has risen to 110 bpm and her temperature is 39.5°C. Which one of the following is the most likely diagnosis?

A. Septic shock.

B. Choledocholithiasis.

C. Acute pancreatitis.

D. Acute cholecystitis.

E. Acute cholangitis.

Incorrect. Correct answer is E


45% answered correctly

Explanation:

Correct Answer Is E

The clinical picture is highly suggestive of acute cholangitis as the most likely diagnosis. This patient has signs and symptoms consistent with diagnosis of acute cholangitis. Fever is present in approximately 90% of patients. Abdominal pain and
jaundice occurs in 70% and 60% of patients, respectively. The patient can become quickly septic and hypotensive with altered level of consciousness.

(Option A) Septic shock can result from acute cholangitis, and is a clinical syndrome not a diagnosis.

(Option B) Choledocholithiasis is the presence of gallstones within the common bile duct. A patient with uncomplicated common bile duct stones typically develops abdominal pain with mild liver function abnormalities. Fever is not a usual fever in
uncomplicated patients. The two major complications associated with choledocholithiasis are acute cholangitis and acute pancreatitis.

(Option C) In acute pancreatitis, the abdominal pain is typically accompanied by nausea and vomiting, which may persist for many hours. The patient is often restless. Jaundice is not a feature unless there is concomitant common bile duct (CBD)
obstruction and cholangitis.

(Option D) Acute cholecystitis is inflammation of the gallbladder, characterized by right upper quadrant pain and tenderness, fever, and leukocytosis. Gallstones are the most common etiology. Jaundice is not a feature of acute cholecystitis.

References

• http://www.uptodate.com/contents/acute-cholangitis

• http://emedicine.medscape.com/article/774245-clini

• http://www.stritch.luc.edu/medicine/sites/default/

Last updated:
Time spent: QID:649 2023-2-12

534 of 1943
A 56-year-old man presents with complaints of 10 kg weight loss in the past three months and jaundice. He denies any abdominal pain, but he describes symptoms of early satiety. Recently, he has noticed dark urine and pale stool. On examination,
he has icteric sclera and a palpable right upper quadrant mass that moves with respiration. No abdominal tenderness is elicited. Which one of the following is the most appropriate initial investigation?

A. Abdominal CT scan.

B. Abdominal ultrasonography.

C. Chest X-ray.

D. ERCP.

E. MRCP.

Incorrect. Correct answer is B


45% answered correctly

Explanation:

Correct Answer Is B

The clinical presentation is highly suggestive of pancreatic head carcinoma. This is evident by painless obstructive jaundice and a distended gallbladder (the right upper quadrant mass that moves with respiration).

Pancreatic cancer can present with the following (percentages are from one recent study of 186 patients with pancreatic cancer):

Malaise and fatigue – 86%


Weight loss – 85%
Anorexia – 83%
Abdominal pain – 79%
Epigastric pain – 71%
Dark urine – 59%
Jaundice – 56%
Nausea – 51%
Back pain – 49%
Diarrhea- 44%
Vomiting – 33%
Steatorrhoea – 25%
Thrombophlebitis – 3%

The most frequent signs are:

Jaundice – 55%
Hepatomegaly- 39%
Right upper quadrant mass – 15%
Cachexia – 13%
Courvoisier’s sign (nontender but palpable distended gallbladder at the right costal margin) – 13%
Epigastric mass – 9%
Ascites – 5%

Of all pancreatic malignant tumors, most are found in the pancreatic head as the most common site. Pancreatic body and tale are second and third most common sites, respectively.

Initial imaging study for assessment of suspected pancreatic cancer depends on the presenting symptoms. For patients with jaundice, the initial imaging study is typically a transabdominal ultrasound. It has high sensitivity for detecting biliary tract
dilation and determining the level of the obstruction. It also has high sensitivity (>95%) for detection of a mass in the pancreas. The sensitivity is lower for tumours <3 cm.

Abdominal CT (option A), however, is the preferred initial imaging modality in patients with epigastric pain and weight loss, but without jaundice. A contrsast-enhanced CT scan of the abdomen is alwasy considered for such patients.

(Option C) Chest X-ray is not indicated for diagnostic imaging of pancreatic cancer.

(Options D and E) ERCP and MRCP may be indicated in further assessment, in cases with obstructive jaundice.

References

• Diagnostic Imaging Pathways - Cholestatic Jaundice

• UpToDate - Clinical manifestations, diagnosis, and staging of exocrine pancreatic cancer

Last updated:
Time spent: QID:672 2023-2-12

535 of 1943
A 32-year-old woman presents to the Emergency Department with abdominal pain and fever since yesterday, which now has localized in the pelvis. On examination, he has a blood pressure of 110/85 mmHg, pulse rate of 98 bpm, respiratory rate of
22 breaths per minute, and fever of 38.2°C. Palpation of the abdomen elicits no tenderness, rebound, or guarding, but there is mild tenderness on rectal exam. Which one of the following would be the next best step in management of this patient?

A. Laparoscopic drainage and appendectomy.

B. Open drainage.

C. CT scan of the pelvis and the abdomen.

D. Ultrasonography of the pelvis and the abdomen.

E. Intravenous antibiotics.

Incorrect. Correct answer is D


45% answered correctly

Explanation:

Correct Answer Is D

The clinical picture is suggestive of pelvic appendicitis. The most common symptom of appendicitis is abdominal pain. Typically, symptoms begin as periumbilical or epigastric pain migrating to right lower quadrant of the abdomen. This pain
migration is the most characteristic feature in the patient’s history. However, in up to 30% of the time, the appendix may be hidden from the anterior peritoneum by being in pelvic, retroileal or retrocolic (retroperitoneal retrocecal) position. The hidden
position of the appendix may significantly alter the clinical manifestation of the appendicitis.

A patient with a pelvic appendix may show no abdominal signs, but rectal examination may cause tenderness in cul-de-sac. Additionally, an obturator sign (pain on passive internal rotation of the flexed right thigh) may be present in a patient with
pelvic appendix.

With typical presentation of appendicitis, imaging studies are not necessary and the patient will be taken to the operating room for appendicectomy, but when diagnosis of appendicitis is not certain (such as in this case) imaging studies are required
for confirmation.

CT scan, specially the technique of appendiceal CT, is the most accurate imaging study (more accurate than ultrasonography). Appendiceal CT gives the highest diagnostic yield. If appendiceal CT is not available, standard abdominal/pelvic CT with
contrast remains highly useful and may be more accurate than ultrasonography.

However, because of concerns about patient exposure to radiation during CT scans, ultrasonography has been suggested as a safer primary diagnostic modality for appendicitis, with CT scanning used secondarily when ultrasounds are negative or
inconclusive. A healthy appendix usually cannot be viewed with ultrasonography. When appendicitis occurs, the ultrasonogram typically demonstrates a noncompressible tubular structure of 7-9 mm in diameter. CT scan is the next step if ultrasound
is non-diagnostic.

NOTE - Pelvic appendicitis may present with:

An absence of abdominal wall rigidity and tenderness


Tenderness in the rectovesical pouch and Pouch of Douglas on rectal examination
Spasm of psoas muscle on right-hand-side
Diarrhea due to rectal irritation by the inflamed appendix
Frequency of micturition due bladder irritation by the inflamed appendix
Hypogastric pain on flexing and internally rotating the hip - due to contact of inflamed appendix with obturator internus muscle

References

• AAFP - Acute Appendicitis: Review and Update

• Medscape - Appendicitis

Last updated:
Time spent: QID:673 2023-2-12

536 of 1943
A 56-year-old man is brought to the Emergency Department with upper gastrointestinal bleeding evident by bright-red hematemesis. He is a known case of alcoholic liver disease. Two endoscopic attempts to stop the bleeding fail. Which one of the
following management options should be considered if the least likelihood of hepatic encephalopathy is desired?

A. Octreotide.

B. Repeat endoscopy.

C. Transjugular intrahepatic portosystemic shunt (TIPS).

D. Distal splenocaval shunt.

E. Renocaval shunt.

Incorrect. Correct answer is D


45% answered correctly

Explanation:

Correct Answer Is D

The history of alcoholic liver disease and the bright red bleeding indicates variceal bleeding with high certainty. Varicose esophageal veins are the most common cause of variceal bleeding. Although at occasions gastric varices can be the source.

Currently, endoscopic therapy is the definitive treatment for active variceal bleeding. It can be performed at the same time of the diagnostic endoscopy. Two forms of endoscopic treatment are commonly used:

Sclerotherapy - Sclerotherapy includes injection of a sclerosant solution into the varices through an injection needle that is placed through the endoscope.

Variceal band ligation – The procedure is similar to hemorrhoidal banding and involves placing small elastic bands around varices in the distal 5 cm of the esophagus.

Surgical intervention is another option. Surgical procedures fall into either of these two categories:

1- Shunt surgeries – a shunt is created between portal and systemic venous system

2- Non-shunt surgeries - the two procedures used are (a)esophageal transection and (b) devascularization of gastro-esophageal junction

Shunt surgeries can be categorized as:

Nonselective - those that decompress the entire portal tree and divert all flow away from the portal system, such as portacaval shunts

Selective - those that compartmentalize the portal tree into a decompressed variceal system while maintaining sinusoidal perfusion via a hypertensive superior mesenteric-portal compartment, such as a distal splenorenal shunt, or a distal
splenocaval shunt

Partial - those that incompletely decompress the entire portal tree and thereby, maintain some hepatic perfusion

Selective shunting maintains near normal portal flow while decompresses the varicose veins. Since the portal flow is normal the risk of hepatic encephalopathy is minimal compared to TIPS, non-selective, or partial shunting.

Of the options, the only one that can be applied as the next best step in management to both stop the bleeding and decrease the risk of hepatic encephalopathy is splenocaval shunting. Of all patients with selective shunting, only 10-15% develop
encephalopathy; however, since retroperitoneal lymphatics are diverted, ascites is a common complication.

NOTE - The routine procedure after endoscopic measures fail is TIPS despite the higher risk of hepatic encephalopathy; however, the question does ask about the procedure to choose with the lowest risk of hepatic encephalopathy.

Attempting a third endoscopy (option B) or medical treatment with octreotide (option A) are not likely to stop the bleeding and are incorrect options.

Renocaval shunting (option E) does not lead to decompression of portal system.

References

• UpToDate - Methods to achieve hemostasis in patients with acute variceal hemorrhage

• Medscape - Portal Hypertension

• BMJ - UK guidelines on the management of variceal haemorrhage in cirrhotic patients

Last updated:
Time spent: QID:694 2023-2-12

537 of 1943
A 57-year-old man presents to the emergency department with absolute constipation for two days and ongoing vomiting for the past eight hours. On examination, the abdomen is not distended or tender. The rest of the exam is unremarkable except
signs of mild dehydration. Gastrograffin studies confirm the presence of small bowel obstruction. Which one of the following is the most common cause of small bowel obstruction?

A. Postoperative adhesions.

B. Bowel cancer.

C. Crohn’s disease.

D. Hernias.

E. Hypokalemia.

Correct
45% answered correctly

Explanation:

Correct Answer Is A

Post-operative adhesions are the most common cause of small bowel obstruction (SBO). The risk of developing an obstruction after surgery from postoperative adhesions is estimated to be 9% within the first year after abdominal surgery, 19% by 4
years, and 35% by 10 years.

The incidence of adhesive SBO seems to be higher in open surgeries than in laparoscopic surgeries. The cause for such difference is not clear, but is likely to be related to the degree of manipulation and trauma to the intraabdominal tissues.

Malignant tumors are the second most common cause of SBO, accounting for about 20% of cases. SBO has been described in as many as 42% of women with ovarian carcinoma and 28% of patients with colorectal carcinomas.

Hernias are the third leading cause of SBO and account for approximately 10% of all cases. Ventral and inguinal hernias are most commonly associated with obstruction; however, internal hernias, femoral, obturator and parastomal hernias also can
contribute to SBO.

Other less common causes of small bowel obstruction include:

Strictures e.g. caused by Crohn’s disease


Small bowel tumors
Trauma
Intussusception
Bezoars
Gallstone ileus (pseudo-obstruction)
Superior mesenteric artery syndrome
Hypokalemia (pseudo-obstruction)

References

• UpToDate - Etiologies, clinical manifestations, and diagnosis of mechanical small bowel obstruction in adults

• Medscape - Small-Bowel Obstruction

Last updated:
Time spent: QID:695 2023-2-12

538 of 1943
A 57-year-old man presents to the emergency department with fever and right upper quadrant pain. On examination, he has a blood pressure of 80/60mmHg, pulse rate of 110 bpm, respiratory rate of 22 breaths per minute and temperature of 38.9°C.
The right upper quadrant is tender to palpation. Laboratory studies reveal high white cell count and mildly elevated bilirubin. Based on the findings you suspect acute cholangitis as the diagnosis. Which one of the following organisms is most likely to
have caused sepsis?

A. Staphylococcus aureus.

B. Pseudomonas aeruginosa.

C. Enterococcus fecalis.

D. Escherichia coli.

E. Treponema pallidum.

Incorrect. Correct answer is D


45% answered correctly

Explanation:

Correct Answer Is D

Acute cholangitis is a clinical syndrome characterized by fever, abdominal pain and jaundice, developing as a result of stasis and infection in the biliary tract. It can rapidly progress to sepsis. Biliary system obstruction is the most common
predisposing factor.

Culture of bile, ductal stones, and blocked biliary stents are positive in over 90% of cases, yielding a mixed growth of gram negative and gram positive bacteria. The most common bacteria isolated have colonic origin.

Escherichia coli is the major bacterium isolated (25 to 50%), followed by Klebsiella (15-20% percent) and Enterobacter species (5-10%). Enterococcus species (e.g. enterococcus fecalis) are the most common gram positive bacteria (10-20%).

Anaerobes, such as bacteroides and Clostridia, are usually present as isolated pathogens , but they are rarely the sole infecting organisms and it is not clear if they play a role in acute cholangitis.

References

• http://www.uptodate.com/contents/acute-cholangitis

• http://emedicine.medscape.com/article/774245-overv

Last updated:
Time spent: QID:696 2023-2-12

539 of 1943
Which of the following is a contraindication to surgical intervention in esophageal malignancies?

A. Invasion to tracheobronchial tree.

B. Invasion to small blood vessels.

C. Lesions greater than 5 cm.

D. Odynophagia.

E. Aspiration pneumonia.

Correct
45% answered correctly

Explanation:

Correct Answer Is A

Surgery is the cornerstone of treatment for esophageal cancer; however, in the following situations surgery is contraindicated:

Distant metastases – the presence of peritoneal, lung, bone, adrenal, brain, or liver metastasis, or extraperitoneal lymph node spread (e.g., paraaortic or mesenteric lymphadenopathy). Approach to patients with celiac lymph node involvement
remains controversial.

Invasion to adjacent structures – invasion of the tumor to the aorta, trachea, heart, great vessels, or the presence of tracheoesophageal fistula (tumor stage: T4b)

NOTE – With invasion to the pleura, pericardium, or diaphragm the cancer is classified as T4a, and considered potentially resectable.

In addition, the presence of severe, associate comorbid conditions such as cardiovascular or respiratory diseases decreases a patient’s chance of survival and success of esophageal resection. Every patient should undergo cardiac and respiratory
function assessment prior to the surgery. A forced expiratory volume in 1 second (FEV1) less than 1.2 L and a left ventricular ejection fraction of less than 40% are relative contraindications to the surgery.

Of the options, only invasion of the tumor to the tracheobronchial tree is a contraindication to surgical resection.

References

• Medscape - Esophageal Cancer

• UpToDate - Management of locally advanced, unresectable and inoperable esophageal cancer

Last updated:
Time spent: QID:697 2023-2-12

540 of 1943
During endoscopic evaluation of a 65-year-old man for dyspepsia, he is found to have an abnormal-appearing patch of mucosa in the distal part of his esophagus. Biopsy result is significant for Barrett's esophagus and metaplasia. Which one of the
following is correct regarding his condition?

A. He shoudl be monitored every 2-3 years by endoscopy and biopsy.

B. He should be monitored by endoscopy every third month.

C. There is 70-80% risk of progression to adenocarcinoma.

D. Hiatus hernia has no association with Barrett’s esophagus.

E. Barrett's esophaus is an indication of urgent surgical intervention.

Correct
45% answered correctly

Explanation:

Correct Answer Is A

With metaplasia present in this patient, surveillance with endoscopy and biopsy should be performed every 2-3 years. Other options are incorrect.

Barrett’s esophagus is a condition in which metaplastic columnar epithelium replaces the stratified squamous epithelium of the distal esophagus. The metaplastic epithelium is acquired as a consequence of chronic gastro-esophageal reflux disease
(GERD). Hiatus hernia predisposes to GERD and indirectly to development Barrett’s esophagus.

Barrett’s esophagus is a premalignant condition with a 40- to 125-fold increase in the incidence of adenocarcinoma of the lower esophagus; however, the absolute risk relatively small and approximately 0.5% per year.

Every patient with the alarming symptoms of weight loss, anemia, heme-positive stool, and dysphagia, or GERD for more than 5 years should undergo endoscopy and biopsy. Further management depends on the histological findings and is
summarized in the following table:

Endoscopic Findings Management


Barrett’s esophagus (metaplasia) Endoscopy every 2-3 years for lesions ≥ 3cm and every 3-5 years for lesions <3 cm.
Low-grade dysplasia Repeat endoscopy 6 months.
High grade dysplasia Distal esophagetomy.

References

• Cancer Council - Clinical question:How frequently should patients with BO undergo endoscopy

• Therapeutic Guidelines – Gastroenterology

• UpToDate - Barrett's esophagus: Surveillance and management

Last updated:
Time spent: QID:698 2023-2-12

541 of 1943
Which one of the following is most important to look for on an ultrasound scan in a patient with jaundice?

A. Gallstones.

B. Head of the pancreas.

C. Dilation of bile ducts.

D. Hepatic masses.

E. Thickening of the gallbladder wall.

Incorrect. Correct answer is C


45% answered correctly

Explanation:

Correct Answer Is C

Jaundice can be caused by a variety of etiologies that broadly are categorized into extrahepatic or intrahepatic.

A careful history and physical examination is essential. Laboratory tests follow for further evaluation.. The most important laboratory values to consider are bilirubin (total and unconjugated), hepatic transaminases, gamma glutamyl transpeptidase
(GGT), alkaline phosphatase and coagulation profile.

If the history, physical exam, and initial laboratory studies suggest obstruction of the biliary tree as the underlying pathophysiology of jaundice, an imaging study is indicated to differentiate extrahepatic and intrahepatic causes of cholestatic
jaundice. The imaging tests that can be used are abdominal ultrasonography, endoscopic ultrasonography, abdominal CT scan, ERCP, and MRI.

Abdominal ultrasonography is the initial imaging modality of choice, becasue it is inexpensive and readily available. In evaluation of jaundice, the most important finding to look for is obstruction of the biliary tree, evident by biliary duct dilation.
Sensitivity of abdominal sonography ranges between 55% and 91% according to different studies. The sensitivity increases with increased serum bilirubin and duration of the jaundice.

Ultrasound can also show stones; however, common bile duct (CBD) stones may not be visualized enough due to the duodenal gas obscuring the distal CBD. Gallstones, hepatic masses and pancreas may also be visualized
on ultrasonography. Gallbladder thickening suggests cholecystitis. Cholecystitis per se does not result in jaundice, unless cholangitis or distal biliary tree obstruction occurs.

References

• http://www.uptodate.com/contents/diagnostic-approa

Last updated:
Time spent: QID:747 2023-2-12

542 of 1943
A 70-year-old woman presents to the Emergency Department with progressive abdominal distention for the past 3 days. She has not passed any stool or flatus. She has vomited several times since this morning. She relates that symptoms started
3 days ago after she fell off her bed. On examination, she has a blood pressure of 100/75 mmHg, pulse rate of 90 bpm, respiratory rate of 21 breaths per minute, and a temperature of 37.6°C. There is no tenderness, rebound, or guarding on
abdominal examination. Which one of the following is most likely to be the cause of this presentation?

A. Sigmoid volvulus.

B. Cecal volvulus.

C. Pneumatosis coli.

D. Pseudo-obstruction.

E. Toxic megacolon.

Incorrect. Correct answer is D


45% answered correctly

Explanation:

Correct Answer Is D

Of the options, the most consistent one with the history of fall (trauma) and the given clinical presentation is acute colonic pseudo-obstruction, also known as Ogilvie syndrome.

Acute colonic pseudo-obstruction is a disorder characterized by gross dilatation of the cecum and the right hemicolon in the absence of an anatomic lesion obstructing intestinal flow.

Acute colonic pseudo-obstruction is associated with an underlying disease in 95% of patients, including:

Non-operative trauma
Infection
Myocardial infarction/ heart failure
Obstetric or gynecologic disease
Abdominal/pelvic surgery
Neurological problems e.g. Parkinson disease, spinal cord injury, multiple sclerosis, Alzheimer disease, etc
Orthopedic surgery
Miscellaneous medical conditions e.g. metabolic, cancer, respiratory failure, renal failure
Miscellaneous surgical conditions e.g. urologic, thoracic, neurosurgery

Studies suggest that non-operative trauma is among the most common causes of acute colonic pseudo-obstruction.

Acute colonic pseudo-obstruction is more common in men and in patients over the age of 60 years. Nausea, vomiting, abdominal pain, constipation, and, paradoxically, diarrhea are the primary manifestations. Abdominal distention is always present
and can cause difficulty breathing. On physical examination, the abdomen is tympanic; however, bowel sounds are present in almost 90% of patients. Peritoneal signs are absent in the early stages of the disease; if they develop, they suggest
impending perforation.

(Options A and B) Volvulus (cecal or sigmoid) is associated with acute onset of pain and obstruction, but a volvulus of three-day duration would have resulted in a very ill and toxic patient due to ischemia of the affected part of the colon and signs of
peritoneal irritation.

(Option C) Pneumatosis coli is the presence of gas in the large intestinal wall. It is a condition not a diagnosis.

(Option E) Patients with toxic megacolon typically appear very ill with fever, tachycardia and abdominal tenderness. There is often a history of bloody diarrha or other signs and symptoms of chronic inflammatory bowel disease.

References

• UpToDate - Acute colonic pseudo-obstruction (Ogilvie's syndrome)

• Medscape - Pneumatosis Intestinalis Imaging

Last updated:
Time spent: QID:812 2023-2-12

543 of 1943
Which one of the following is the most common type of esophageal cancer worldwide?

A. Adenocarcinoma.

B. Squamous cell carcinoma.

C. Lymphoma.

D. Metastatic lesions.

E. Sarcoma.

Incorrect. Correct answer is B


45% answered correctly

Explanation:

Correct Answer Is B

The epidemiology of esophageal carcinoma has markedly changed over the past several decades. Worldwide, squamous cell carcinoma is the most common type of esophageal malignancies. Smoking and alcohol are the most significant risk
factors for this type of esophageal cancer.

However, in developed countries such as United States of America, UK, and Australia, the incidence of adenocarcinoma of the distal esophagus and gastro-esophageal junction has progressively increased and accounts for over 70% of all cases of
esophageal cancer. Gastroesophageal reflux disease (GERD) can result in Barrett’s esophagus which is the most significant risk factor for adenocarcinoma of the esophagus.

References

• Medscape - Esophageal Cancer

Last updated:
Time spent: QID:159 2023-2-12

544 of 1943
A 24-year-old woman develops moderate, generalized abdominal pain of sudden onset, and faints shortly thereafter. On examination, she is pale and diaphoretic, with a blood pressure of 80/50 mmHg, pulse rate of 110 bpm, respiratory rate of
22 breaths per minute and temperature of 37.6°C. The abdomen is mildly distended and tender to touch. The rest of the exam is unremarkable. She has no history of recent abdominal trauma, but has been on oral contraceptive pills (OCP) since the
age of 15 years. Which one of the following could be the most likely diagnosis?

A. Strangulated hernia.

B. Mesenteric artery occlusion.

C. Rupture of abdominal aortic aneurysm.

D. Ruptured a hepatic adenoma.

E. Acute appendicitis.

Incorrect. Correct answer is D


45% answered correctly

Explanation:

Correct Answer Is D

The age of the patient, the acute onset of the presentation and the history of OCP use make a ruptured hepatic adenoma the most likely diagnosis.

Hepatocellular adenomas (HCAs), also known as hepatic adenomas or liver cell adenomas, are rare, benign tumors of epithelial origin and occur in less than 0.004% of the population at risk.

HCAs occur mostly in women of childbearing age and are strongly associated with the use of oral contraceptive pills (OCPs) and other estrogens. This is reflected by a dramatic increase in the incidence of this disease since OCPs were introduced in
the 1960s.

The clinical presentation varies widely. Main features in history and physical examination may include the following:

Pain in the right upper quadrant or epigastric region in 25-50% of patients with HCAs
Lesions may be noticed by patients as a palpable mass. Lesions may also be discovered incidentally during an abdominal imaging study for an unrelated reason
History of birth control or anabolic steroid use should be checked in patients with suspected hepatocellular adenomas.
Patients may present with severe, acute abdominal pain with bleeding into the abdomen, which results in signs of shock such as hypotension, tachycardia, and diaphoresis
Hemoperitoneum occurs more frequently if the patient is taking a high-dose OCP, is actively menstruating or pregnant, or is within 6 weeks postpartum. Location of the lesion also is important, with those near the surface of the liver
more prone to causing hemoperitoneum.

(Option A) A strangulated hernia is very unlikely to present with this clinical picture and an onset so acute.

(Option B) Mesenteric artery occlusion is usually seen in elder patients with risk factors for thrombosis or embolus formation. There is often a concomitant medical condition predisposing the patient to embolus formation such as atrial fibrillation.
The condition presents with postprandial diarrhea, abdominal pain and abdominal exam findings that are usually disproportionate to the symptoms. Rectal bleeding may be found on digital rectal exam.

(Option C) Abdominal aortic aneurysm (AAA) is a disease seen in older male patients. Rupture of an AAA may present similarly; however, the age and gender of this patient make this diagnosis very unlikely.

(Option E) Acute appendicitis presents with vague periumbilical pain that gradually localizes in the right lower quadrant. Symptoms of peritonitis and shock develop later following rupture of the inflamed appendix. Acute appendicitis is a very unlikely
explanation for this presentation.

References

• http://emedicine.medscape.com/article/170205-overv

• http://emedicine.medscape.com/article/170205-clini

Last updated:
Time spent: QID:817 2023-2-12

545 of 1943
A 48-year-old man presents to your practice several months after gastrectomy with complaint of dyspepsia. This problem occurs mostly 30 minutes after meals. Dyspepsia is worse when he takes toast and cereal for his breakfast. Which one of the
following will be the most important advice to give?

A. He should use more pillows when sleeping.

B. Revision of gastrectomy.

C. Ranitidine.

D. Esomeprazole.

E. Advice dietary changes.

Incorrect. Correct answer is E


45% answered correctly

Explanation:

Correct Answer Is E

With gasterectomy in history, dumping syndrome is the most likely underlying cause to this presentation. Dumping syndrome is the effect of altered gastric reservoir function, abnormal gastric motor function, and/or pyloric emptying mechanism.

In terms of temporal relation to eating, dumping syndrome can have early or late manifestations:

Early dumping

Symptoms of early dumping syndrome occur 30-60 minutes after a meal, and are hypothesized to result from accelerated gastric emptying of hyperosmolar contents into the small bowel. The hyperosmolarity of the bowel content leads to fluid shift
from the intravascular compartment into the bowel lumen, resulting in rapid small bowel distention and increase in the frequency of bowel contractions.

With rapid shift of fluid into the intestinal lumen, circulating volume contraction follows, triggering a vasomotor response that presents with tachycardia, lightheadedness, etc

Late dumping

Late dumping occurs 1-3 hours after a meal, probably through development of hyperinsulinemic (reactive) hypoglycemia. Rapid delivery of a meal to the small intestine results in an initial high concentration of carbohydrates in the proximal small
bowel and rapid absorption of glucose, hypoglycemia, and a hyperinsulinemic response. The high insulin levels stay for longer period and are responsible for the subsequent hypoglycemia.

The following diagram summarizes the mechanism through which the dumping syndrome develops:

The clinical presentation of dumping syndrome can be categorized as abdominal versus vasomotor symptoms, and based on the time of onset as early versus late.

Early dumping systemic symptoms:

Desire to lie down


Palpitations
Fatigue
Faintness
Syncope
Diaphoresis
Headache
Flushing

Early dumping abdominal symptoms:

Epigastric fullness
Diarrhea
Nausea

546 of 1943
Abdominal cramps
Borborygmi

These symptoms can be collectively referred to as dyspepsia.

Late dumping symptoms:

Perspiration
Shakiness
Difficulty to concentrate
Decreased consciousness
Hunger

For patients with dumping syndrome, dietary modification is the first-line management option. The following pieces of advice are appropriate:

Daily energy intake is divided into 6 meals.


Fluid intake during and with meals be restricted and liquids be avoided for at least half an hour after a meal.
Avoiding simple sugars.
Milk and milk products are generally not tolerated and should be avoided.
Because carbohydrate intake is restricted, protein and fat intake should be increased to fulfill energy needs.

Additional points to consider:

Supplementation with dietary fiber has proven effective in the treatment of hypoglycemic episodes. Many medical therapies have been tested, including pectin, guar gum, and glucomannan. These dietary fibers form gels with carbohydrates, resulting
in delayed glucose absorption and prolongation of bowel transit time.

Dietary change to a low-carbohydrate, high protein diet, as well as the use of alpha-glucosidase inhibitors, may be useful to control the symptoms of dumping. This is preferential to subtotal or total pancreatectomy in those persons with severe
symptoms.

Most patients have relatively mild symptoms and respond well to dietary changes. In some patients with postprandial hypotension, lying supine for 30 minutes after meals may delay gastric emptying and also increase venous return, thereby
minimizing the chances of syncope.

References

• RACGP - AFP - The bariatric surgery patient

• Medscape - Dumping Syndrome

Time spent: QID:828 Last updated:


2023-2-12

547 of 1943
A 65-year-old woman presents to the emergency department with a painful discrete swelling below and lateral to the pubic tubercle. On examination, the swelling is non-reducible and mildly tender to touch. Coughing or straining does not make any
changes in the swelling. Which one of the following is the most appropriate next step in management of this woman?

A. Surgical exploration.

B. Ultrasonography of the swelling.

C. NPO, nasogastric tube and bowel rest.

D. Fine needle aspiration (FNA) of the swelling.

E. Excisional biopsy of the swelling.

Correct
45% answered correctly

Explanation:

Correct Answer Is A

The clinical picture is suggestive of a femoral hernia as the most likely diagnosis. Irreducibility and the fact the swelling is not altered by cough or straining suggests that the hernia has been incarcerated. Tenderness to palpation, if present, indicates
strangulation.

Femoral hernias account for 2-4% of groin hernias, are more common in women, and are more likely to present with strangulation requiring emergency surgery. An incarcerated hernia needs watchful monitoring for signs of strangulation while semi-
elective surgery is arranged. Where clinical picture suggests strangulation, emergency surgical intervention should be performed without delay.

For most hernias (as in this case), the location will be obvious on physical examination. Femoral hernias most commonly present inferior to the inguinal ligament and medial to the femoral artery. They present with a swelling below and lateral to the
pubic tubercle, in contrast to inguinal hernias that present as a swelling medial to the pubic tubercle.

Most of the time, the diagnosis is evident by physical exam; however, in situations when the physical exam is uncertain, such as in obese patients, additional imaging studies are recommended. Ultrasonography of the groin region is particularly
useful if confusion exists as to the location of the hernia.

This is particularly important owing to the fact that femoral hernias are approached differently from inguinal hernias. While watchful waiting may be an option for asymptomatic or minimally symptomatic inguinal hernia, this is not recommended for
femoral hernias due to the high risk of complications.

(Option A) Ultrasonography is used when the diagnosis is uncertain clinically. ​CT scan is used if ultrasound results are equivocal and non-diagnostic. MRI has the highest accuracy but is not readily available and cost-effective.

(Option C) Femora hernias have a high rate of incarceration and strangulation and it is unlikely that an incarcerated femoral hernia resolves with bowel rest.

(Options D and E) The diagnosis is certain in this case scenario. Fine needle or excisioanl biopsies are not necessary when hernia is suspected.

References

• http://www.uptodate.com/contents/classification-cl

Last updated:
Time spent: QID:898 2023-2-12

548 of 1943
A 45-year-old refugee Somali man presents with rectal bleeding. He describes the bleeding as bright blood covering the stool. Which one of the following should be the next investigation?

A. Rectal examination.

B. Fecal occult blood test.

C. CT scan of the abdomen and pelvis.

D. MRI of the abdomen and pelvis.

E. Colonoscopy.

Correct
45% answered correctly

Explanation:

Correct Answer Is A

In approach to every patient with rectal bleeding, a complete history should be taken and physical examination performed. Inspection of the anus and a digital rectal exam must be carried out for every patient presenting with rectal bleeding as the
most important initial step. If available, anoscopy or proctoscopy is an appropriate means of inspecting the anorectal area, especially during active bleeding.

Although often benign in nature, rectal or occult bleeding should always be attributed to more serious causes until proven otherwise by endoscopic testing. Flexible sigmoidoscopy in low-risk younger patients or colonoscopy in most other patients
establishes the diagnosis of hemorrhoids, and excludes other causes of bleeding such as colitis, tumors, polyps, or vascular malformations.

(Option B) Occult blood testing in the presence if overt bleeding is not indicated.

(Options C and D) Imaging modalities such as CT scan or MRI are not routinely used for evaluation of rectal bleeding.

(Option E) Colonoscopy is considered if the source of bleeding is suspected to be more proximal. With colonoscopy the entire colon can be often visualized.

References

• UpToDate - Approach to minimal bright red blood per rectum in adults

Last updated:
Time spent: QID:900 2023-2-12

549 of 1943
A 56-year-old man presents to your practice with complaint of fresh rectal bleeding covering his stool. He has long-standing history of constipation. He mentions no pain on defecation. Which one of the following can be the most likely cause?

A. Colorectal cancer.

B. External hemorrhoid.

C. Internal hemorrhoid.

D. Anorectal fistula.

E. Anal fissure.

Incorrect. Correct answer is C


45% answered correctly

Explanation:

Correct Answer Is C

Fresh blood covering the stool in the background history of prolonged constipation is most likely caused by internal hemorrhoid.

Typical pattern of rectal bleeding in internal hemorrhoids is usually associated with a bowel movement. Bright-red blood typically coats the stool at the end of defecation. Blood may also drip into the toilet bowl or stain toilet paper. Hemorrhoidal
bleeding is almost always painless, and painful defecation prompts investigation for other causes such as anorectal fissures.

Although the pattern of rectal bleeding in this man is more likely to have been caused by hemorrhoids, overt or occult rectal bleeding should not be merely attributed to hemorrhoids until other less common but more serious causes of rectal bleeding
have been excluded by endoscopic testing. Flexible sigmoidoscopy or anoscopy in low-risk younger patients or colonoscopy in most other patients establishes the diagnosis of hemorrhoids, and excludes other causes of bleeding such as colitis,
tumors, polyps, or vascular malformations.

(Option A) Presence of rectal bleeding in this patient should also raise suspicion against colorectal cancer. However, this diagnosis is less likely because firstly, there are no other suggestive findings such as weight loss, altering bowel habits, etc. and
secondly, hemorrhoids are more common than cancers statistically.

(Options B and E) External hemorrhoid and anal fissure are associated with significant anal pain and are very unlikely to be the diagnosis in this patient who has no pain on defecation.

(Option D) Anorectal fistulae are associated with soiling rather than rectal bleeding. It is unlikely that painless fresh rectal bleeding is the only presentation of anorectal fistulae.

References

• RACGP - Haemorrhoids – a review

• UpToDate - Hemorrhoids: Clinical manifestations and diagnosis

Last updated:
Time spent: QID:901 2023-2-12

550 of 1943
A 67-year-old man presents to your GP clinic with complaint of fresh rectal bleeding covering the outer surface of his stool for the past 3 weeks. He is otherwise in good health. Which one of the following is the most appropriate next step in
evaluation?

A. Digital rectal exam (DRE).

B. Proctoscopy.

C. Flexible sigmoidoscopy.

D. Rigid sigmoidoscopy.

E. Colonoscopy.

Incorrect. Correct answer is A


45% answered correctly

Explanation:

Correct Answer Is A

Rectal bleeding is often caused by benign conditions such as hemorrhoids or anal fissures. However, it is possible that this assumption masks serious and more proximal pathologies such as colorectal cancer. To avoid missing patients with sinister
conditions, either as the cause of bleeding or a concomitant problem, all such patients should be assessed even if based on the history the etiology is clear.

Approach to patients with rectal bleeding starts with a thorough history followed by physical exam. The physical examination should be directed at identifying possible or definite sources of bleeding and at finding worrisome distal lesions that may
be detectable on examination. A detailed physical examination must include external inspection of the anus and a digital rectal exam as initial investigations.

Having the patient bear down during the examination may induce prolapse of a hemorrhoid or bleeding of a superficial lesion. Moreover, when available, office-based anoscopy or proctoscopy (option B) should be performed in patients who present
with acute rectal bleeding. These are simple measures not requiring bowel preparation.

Sigmoidoscopy (options C and D) or colonoscopy (option E) is indicated in patients in whom initial assessment suggests more proximal lesions as the cause.

References

• UpToDate - Approach to minimal bright red blood per rectum in adults

Last updated:
Time spent: QID:902 2023-2-12

551 of 1943
A 38-year-old man underwent gastric bypass surgery due to obesity (BMI>42) a few months earlier. Now, he has presented with complaints of lightheadedness, palpitation, and diarrhea within the first hour of meals, especially after he takes milk,
toast, or cereal. Which one of the following would be the most appropriate management option?

A. Addition of vitamin B12.

B. Endoscopy.

C. Diet counselling.

D. Esomeprazole.

E. Ranitidine.

Incorrect. Correct answer is C


45% answered correctly

Explanation:

Correct Answer Is C

The presentation and the history of gastric bypass surgery make dumping syndrome the most likely diagnosis. Dumping syndrome is the effect of altered gastric reservoir function, abnormal gastric motor function, and/or pyloric emptying
mechanism.

In terms of temporal relation to eating, dumping syndrome can have early or late manifestations:

Early dumping

Symptoms of early dumping syndrome occur 30-60 minutes after a meal, and are hypothesized to result from accelerated gastric emptying of hyperosmolar contents into the small bowel. The hyperosmolarity of the bowel content leads to fluid shift
from the intravascular compartment into the bowel lumen, resulting in rapid small bowel distention and increase in the frequency of bowel contractions.

With rapid shift of fluid into the intestinal lumen, circulating volume contraction follows, triggering a vasomotor response that presents with tachycardia, lightheadedness, etc

Late dumping

Late dumping occurs 1-3 hours after a meal, probably through development of hyperinsulinemic (reactive) hypoglycemia. Rapid delivery of a meal to the small intestine results in an initial high concentration of carbohydrates in the proximal small
bowel and rapid absorption of glucose, hypoglycemia, and a hyperinsulinemic response. The high insulin levels stay for longer period and are responsible for the subsequent hypoglycemia.

The following diagram summarizes the mechanism through which the dumping syndrome develops:

The clinical presentation of dumping syndrome can be categorized as abdominal versus vasomotor symptoms, and based on the time of onset as early versus late.

Early dumping systemic symptoms:

Desire to lie down


Palpitations
Fatigue
Faintness
Syncope
Diaphoresis
Headache
Flushing

Early dumping abdominal symptoms:

552 of 1943
Epigastric fullness
Diarrhea
Nausea
Abdominal cramps
Borborygmi

These symptoms can be collectively referred to as dyspepsia.

Late dumping symptoms:

Perspiration
Shakiness
Difficulty to concentrate
Decreased consciousness
Hunger

For patients with dumping syndrome, dietary modification is first-line management option. The following pieces of advice are appropriate:

Daily energy intake is divided into 6 meals.


Fluid intake during and with meals be restricted and liquids be avoided for at least half an hour after a meal.
Avoiding simple sugars.
Milk and milk products are generally not tolerated and should be avoided.
Because carbohydrate intake is restricted, protein and fat intake should be increased to fulfill energy needs.

Additional points to consider:

Supplementation with dietary fiber has proven effective in the treatment of hypoglycemic episodes. Many medical therapies have been tested, including pectin, guar gum, and glucomannan. These dietary fibers form gels with carbohydrates, resulting
in delayed glucose absorption and prolongation of bowel transit time.

Dietary change to a low-carbohydrate, high protein diet, as well as the use of alpha-glucosidase inhibitors, may be useful to control the symptoms of dumping. This is preferential to subtotal or total pancreatectomy in those persons with severe
symptoms.

Most patients have relatively mild symptoms and respond well to dietary manipulations. In some patients with postprandial hypotension, lying supine for 30 minutes after meals may delay gastric emptying and increase venous return, thereby
minimizing the chances of syncope.

References

• RACGP - AFP - The bariatric surgery patient

• Medscape - Dumping Syndrome

Time spent: QID:909 Last updated:


2023-2-12

553 of 1943
A 57-year-old man presents with constipation and abdominal distention for the past three days. On examination, a lump is noted in the left inguinal region. The lump has no cough impulse and is not tender or painful. Which one of the following is the
best next investigation to consider in this patient?

A. Ultrasound of the lump.

B. Ultrasound of the abdomen.

C. Erect and supine x-ray films of the abdomen.

D. CT scan of the abdomen.

E. Biopsy.

Incorrect. Correct answer is C


45% answered correctly

Explanation:

Correct Answer Is C

The lump can be a segment of a small bowel loop entrapped in the inguinal region, namely inguinal hernia. Absent cough impulse is a pointer towards incarceration of the hernia.

There are two points in the question. The first and most important one is the presence of constipation and abdominal distention consistent with bowel obstruction. The second point is the presence of an inguinal lump with no cough impulse, no pain,
and no tenderness.

The history and physical findings make small bowel obstruction (SBO) the most likely diagnosis and the first condition to exclude. The initial imaging of choice in assessment of suspected bowel obstruction is plain X-rays films. At least two views
are required: upright view and supine view. Plain abdominal films can confirm the diagnosis in most patients without the need for further diagnostic testing, but they can be equivocal in 20-30% of patients, or normal or misleading in 10-20%.
Abdominal X-rays are always the first imaging step in diagnosis.

Currently the best and most accurate diagnostic test for SBO is abdominal CT scan (option D) with both intravenous and oral contrast. CT scan can simultaneously provide information about the presence, level, severity and cause of obstruction. In
addition, other abdominal pathologies can be detected. In some cases, closed-loop or strangulating obstruction may be revealed. CT scanning has been replaced with barium follow-through, which previously was the gold-standard diagnostic test for
SBO. Follow-through series are now reserved for cases in which CT is not diagnostic. The question, however, asks about the initial investigation not the most accurate one.

Abdominal ultrasound scan (option B) is sometimes used for assessment of bowel obstruction, and is the only modality with the ability to assess the bowel pristaltism. However, plain X-rays remain the standard initial investigation.

This lump is very likely to be an incarcerated inguinal hernia which justifies both the lump and the obstruction. However, other differential diagnoses such as an inguinal lymph node should be considered as well once SBO, as the most likely cause, is
excluded. In that case, ultrasound of the lump (option A) and biopsy (option E), if the lump turns out to be a lymph node, are indicated.

References

• Medscape - Small Bowel Imaging

Last updated:
Time spent: QID:912 2023-2-12

554 of 1943
You prescribed triple therapy against Helicobacter pylori with clarithromycin, amoxicillin and esomperazole for a 42-year-old patient of yours after his test results came back positive for the infection and you considered it as the cause of his
intractable dyspepsia. One week after completion of the treatment, he comes back with reduced yet persisting symptoms. You arrange for a breath urea test. The result of the test is positive indicating treatment failure. Which one of the following is
the most likely cause of the failed treatment?

A. Resistance to clarithromycin.

B. Resistance to amoxicillin.

C. False positive breath urea test result.

D. Poor adherence to the therapy.

E. Misdiagnosis of the cause of dyspepsia.

Correct
45% answered correctly

Explanation:

Correct Answer Is A

Triple therapy with clarithromycin, amoxicillin and a proton pump inhibitor (PPI) for 7-14 days is the regimen of choice for treatment of H.pylori infection with over 90% response. Clarithromycin is the most effective antibiotic against H.pylori.

The most common cause of treatment failure with this regimen is the increasing rates of resistance of H.pylori to clarithromycin. This resistance cannot be overcome by dose increase or lengthening the duration of therapy.

NOTE - In those with penicillin hypersensitivity, in whom amoxicillin cannot be used, metronidazole can replace amoxicillin. So the regimen whould be a combination of metronidazole, clarithromycin and a PPI. I such cases, resistance to
either metronidazole or clarithromycin, or even both, can be the cause of treatment failure.

References

• Antibiotic-resistant H. pylori infection and its treatment.

Last updated:
Time spent: QID:933 2023-2-12

555 of 1943
Which one of the following conditions is least likely to be associated with Helicobacter pylori infection?

A. Gastric carcinoma.

B. B cell lymphoma of MALT tissue.

C. Gastroesophageal disease (GERD).

D. Atrophic gastritis.

E. Peptic ulcer disease.

Incorrect. Correct answer is C


45% answered correctly

Explanation:

Correct Answer Is C

(Option A) Correct- There is strong evidence that H. pylori contributes to the development of gastric cancer.

(Option B) Correct- H. pylori infection can also lead to the development of mucosa-associated-lymphoid-tissue (MALT) lymphoma, a type of cancer of the stomach. Treatment and eradication of H. pylori infection can result in regression of
this malignancy in up to 75% of cases.

(Option C) Incorrect- To date, there has been no study suggesting a strong link between H.pylori and GERD. Eradication of H.pylori has not shown effective in treatment of GERD.

(Option D) Correct- Chronic H.pylori-induced inflammation can eventually lead to loss of normal gastric mucosal architecture, with destruction of gastric glands and replacement by fibrosis and intestinal-type epithelium. This process of atrophic
gastritis and intestinal metaplasia occurs in approximately 50% of the H.pylori-colonized population.

(Option E) Correct- Helicobacter pylori (H.pylori) has been has been strongly linked to the development of gastric and duodenal ulcers. Eradication of H. pylori can prevent ulcers forming. Indeed patients presenting with ulcers should be tested for H.
pylori and treated because eradication of H. pylori in patients with pre-existing ulcers cures ulcer disease and can prevent most recurrences.

References

• http://www.cdd.com.au/pages/disease_info/heliobact

• http://www.ncbi.nlm.nih.gov/pmc/articles/PMC153910

Last updated:
Time spent: QID:940 2023-2-12

556 of 1943
Which one of the following is the most common type of esophageal malignancy in Australia?

A. Squamous cell carcinoma.

B. Lymphoma of the esophagus.

C. Adenocarcinoma.

D. Sarcoma.

E. Metastatic tumors.

Incorrect. Correct answer is C


45% answered correctly

Explanation:

Correct Answer Is C

The epidemiology of esophageal carcinoma has markedly changed over the past several decades. Previously, squamous cell carcinoma was the most common type of esophageal malignancies worldwide (90-95% of cases); however, over the past 4
decades and in developed countries such as United States of America, UK, and Australia, the incidence of adenocarcinoma of the distal esophagus and gastro-esophageal junction has progressively increased. Currently, adenocarcinomas account for
more than 70% of all cases of esophageal cancers in Australia. Barrett’s esophagus caused by gastroesophageal reflux disease (GERD) is the most significant risk factor for development of esophageal carcinoma of adenocarcinoma type.

NOTE - Squamous cell carcinoma remains the leading type of esophageal carcinoma worldwide with smoking and chronic alcohol as the most significant risk factors.

References

• Medscape - Esophageal Cancer

• Esophageal cancer: A Review of epidemiology, pathogenesis, staging workup and treatment modalities

Last updated:
Time spent: QID:160 2023-2-12

557 of 1943
A 42-year-old man presents with 10-month history of nocturnal gastric acid reflux. He smokes 40 cigarettes a day, drinks alcohol on a regular basis, and 4 cups of coffee a day. He loves spicy foods and eats a lot of them. On examination, his height is
178 cm and weighs 110 kg. He has a waist circumference of 115 cm. Which one of the following is the most appropriate advice for him regarding long-term management of the condition?

A. He should stop drinking alcohol.

B. He should stop smoking.

C. He should avoid spicy foods.

D. He should reduce weight.

E. He should stop drinking coffee.

Incorrect. Correct answer is D


45% answered correctly

Explanation:

Correct Answer Is D

Advice regarding lifestyle modification has been historically a part of management plan, in conjunction to antacid therapy, for gastroesophageal reflux disease (GERD). Mild cases will respond to simple measures (e.g., bed head elevation), while most
will need proton pump inhibitors (PPIs) as the cornerstone of the treatment.

Lifestyle modifications that are often advised include:

Weight reduction
Bed head elevation during sleep
Avoiding spicy foods
Last meal not later than 2-3 hours before bed time
Avoiding or reducing alcohol and caffeine intake
Quitting smoking

Of these, however, only elevation of the head of the bed and weight reduction seems to have an effect more than placebo. While widely encouraged, other measures appear to be associated with improved overall health with no specific effect on
GERD, at least not in all patients.

References

• UpToDate - Medical management of gastroesophageal reflux disease in adults

• RACGP - Gastro-oesophageal reflux disease

Last updated:
Time spent: QID:973 2023-2-12

558 of 1943
A 71-year-old man is being evaluated for iron deficiency anemia. On initial laboratory tests, he as a positive fecal occult blood test (FOBT) result. An upper endoscopy is negative, and he is planned for colonoscopy. During the procedure, the
colonoscope does not pass beyond the hepatic flexure. No abnormal finding is noted up to there. Which one of the following is the most appropriate next step in management?

A. Force the colonoscope to pass the hepatic flexure.

B. Tagged red blood cells.

C. Colon capsule endoscopy (CCE).

D. CT colonography (CTC).

E. Barium enema.

Incorrect. Correct answer is C


45% answered correctly

Explanation:

Correct Answer Is C

In the elderly, gastrointestinal bleeding should be attributed to colorectal malignancies until proven otherwise. In such patients, the whole length of the colon should be thoroughly investigated with colonoscopy as the most appropriate diagnostic
modality.

An incomplete colonoscopy examination may occur in up to 5-10% of cases due to patient discomfort, a colon with many twists, postoperative adhesions, or hernia. In such cases, either a CT colonography (CTC) or colonic capsule endoscopy (CCE)
may be performed. The latter, based on recent studies, is associated with higher diagnostic value.

(Option A) Forcing the colonoscope is likely to cause damages such as mucosal tear or even perforation.

(Option B) Tagged red cells are used in cases of acute lower gastrointestinal bleeding when colonoscopy cannot spot the bleeding site. it is not useful for tumor detection.

(Option D) Compared to first-generation capsules, CTC was superior to CCE, but with the advent of newer capsules, diagnostic value of CCE has surpassed that of CTC.

(Option E) Double contrast barium enema was traditionally used in cases of incomplete colonoscopy, but retrospective studies demonstrated that this method could miss more malignant lesions than CTC and CCE; hence, these procedures have
superseded double contrast barium enema.

References

• Diagnostic Imaging Pathways - Gastrointestinal Bleeding (Obscure)

Last updated:
Time spent: QID:978 2023-2-12

559 of 1943
Kim, 9 years old, is diagnosed with hepatitis A after she developed jaundice and abdominal pain 8 days ago. Today, her mother calls and asks you when she can go back to school. Which one of the following would be your answer?

A. When her tests are normal.

B. She can go back to school now if there is a separate sanitation system.

C. She can go to school now.

D. After 4 to 6 months.

E. After one month.

Incorrect. Correct answer is C


45% answered correctly

Explanation:

Correct Answer Is C

The incubation period of hepatitis A is 15 to 50 days, but more commonly 28 to 30 days. The usual clinical presentation is acute fever, malaise, anorexia, nausea and abdominal discomfort (prodromal symptoms), followed by dark urine and jaundice
a few days later. Symptoms usually last several weeks.

The likelihood that symptoms will follow infection increases with age: jaundice occurs in only a small proportion of infants and young children, but a majority of adults. Infants and children infected with HAV may have mild or no symptoms.

The viral load in the stool (and infectivity) is highest just before the onset of jaundice. Patients are considered infectious from a few days before onset of prodromal symptoms to a few days after onset of jaundice, and non-infectious one week after
onset of jaundice (if it occurs) or two weeks (14 days) after onset of prodromal symptoms, whichever comes first.

Kim has had jaundice 8 days ago and she does not seem to be infectious anymore because more than 7 days has pasted since jaundice. She can go back to school now.

Serum IgM anti-HAV is the gold standard for diagnosis. The antibody becomes positive at the onset of symptoms, peaks during the acute or early convalescent phase, and remains positive for approximately 4 to 6 months. Liver function tests may
also remain deranged beyond the infections period. Liver function tests or HAV antibodies are not appropriate indicators for infectivity.

NOTE - exclusion of contacts is not necessary.

References

• http://www.sahealth.sa.gov.au/wps/wcm/connect/publ

• http://www.health.gov.au/internet/main/publishing.

Last updated:
Time spent: QID:1031 2023-2-12

560 of 1943
Which one of the following is not in favor of achalasia as a diagnosis?

A. Dysphagia to solids.

B. Dysphagia to liquids.

C. Regurgitation of food.

D. Painful swallowing.

E. Nocturnal regurgitation.

Incorrect. Correct answer is D


45% answered correctly

Explanation:

Correct Answer Is D

Achalasia is a primary esophageal motility disorder characterized by the absence of esophageal peristalsis and impaired relaxation of the lower esophageal sphincter (LES) in response to swallowing. The LES is hypertensive in about 50% of patients.
These abnormalities cause a functional obstruction at the gastroesophageal junction.

Dysphagia for solids (91%) (option A) and liquids (85%) (option B) are the primary clinical feature of achalasia. Although dysphagia for liquids can occur in patients with other esophageal motility disorders (e.g., progressive systemic sclerosis), this
symptom is most characteristic of achalasia and strongly suggests the diagnosis.

Regurgitation of food (option C) is another important feature of achalasia. It can be worse at night (nocturnal regurgitation) (option E) when the patient is recumbent. Patients can find undigested food particles on their pillow in the morning.
Regurgitation is a problem in 40-60% of patients with achalasia.

Weight loss in the range of 5-10 kg is common (40-60%). Profound weight loss can also occur. Difficulty belching is another important symptom that is seen in 85% of patients. Chest pain and heart burn can also be present.

Painful swallowing (odynophagia) is not a feature of achalasia and, if present, another diagnosis must be thought of.

References

• UpToDate - Achalasia: Pathogenesis, clinical manifestations, and diagnosis

Last updated:
Time spent: QID:1073 2023-2-12

561 of 1943
A 55-year-old man presents to your clinic with complaint of recurrent epigastric pain radiating to back. Each episode of pain lasts for 3 to 4 hours. No specific trigger is mentioned, and the pain does not seem to be related to eating or hunger. An ECG
is obtained, and is normal. Abdominal ultrasound exam is inconclusive. Laboratory tests, including serum amylase, lipase and troponin, are all within normal parameters. Which one of the following is the most likely diagnosis?

A. Duodenal ulcer.

B. Esophagitis.

C. Acute coronary syndrome.

D. Chronic pancreatitis.

E. Cholecystitis.

Incorrect. Correct answer is D


45% answered correctly

Explanation:

Correct Answer Is D

Epigastric pain can have a myriad of etiologies. Cardiac ischemia, peptic ulcer disease, hepatobiliary diseases, pancreatitis, and lower lobe pneumonia can all cause epigastric pain.

Of the options, however, chronic pancreatitis is the most likely diagnosis.

Chronic pancreatitis presents with epigastric pain that typically radiates to back. The pain may or may not be triggered by eating and lasts for several hours. Serum amylase and lipase are often normal, or just slightly elevated. Characteristic finding
on imaging is calcification of the pancreas. Ultrasound shows calcifications in only 60 to 70% of patients. 30 to 40% of patients may have normal ultrasound scan results.

(Option A) Duodenal ulcer presents with epigastric pain. It can radiate to back. Laboratory findings and ultrasound scan are inconclusive; however, it is worse with hunger and relieved by eating.

(Option B) Esophagitis presents with dysphagia (difficulty swallowing) and/or odynophagia (pain on swallowing). Laboratory tests and ultrasound exam findings are unremarkable; however, it is uncommon for the pain to radiate to back. Retrosternal
chest pain and heart burn are other features associated with esophagitis.

(Option C) A normal ECG makes acute coronary syndrome (ACS) less likely. On the other hand, chronicity of the pain and the episodic nature excludes ACS with high certainty.

(Option E) Cholecystitis can presents with pain often aggravated by eating, especially eating fatty foods. The pain can be felt in the right upper quadrant or epigastrium. The pain can radiate to the tip of right shoulder. Furthermore, fever is a common
feature. Alkaline phosphates and gamma glutamyl transpeptidase (GGT) are often elevated. Lipase and amylase are normal or only slightly elevated. It is unlikely that sonography misses the diagnosis.

References

• MEdscape - Chronic Pancreatitis

Last updated:
Time spent: QID:1091 2023-2-12

562 of 1943
Which one of the following is the most common finding on ultrasound examination in patient with right upper quadrant pain?

A. Gallbladder wall thickness and extravasation.

B. Gallstones.

C. Pericholecystic edema.

D. Bile duct dilation.

E. Normal findings.

Correct
45% answered correctly

Explanation:

Correct Answer Is B

Most RUQ pains in adults are usually due to biliary or hepatic diseases.

Biliary causes of the RUQ pain include:

Gallstones
Acute cholecystitis
Acute cholangitis
Dysfunction of the Sphincter of Oddi

Hepatic etiologies include:

Hepatitis
Perihepatitis
Liver abscess
Budd-Chiari syndrome
Portal vein thrombosis

Gallstones are very common among different populations. They could be asymptomatic and incidentally found during evaluation for another reason, or cause problems such as biliary colic, cholecystitis, choledocholithiasis, and cholangitis.

Of the options, gallstones are the most common ultrasonographic findings in patients with RUQ pain.

Thickened gallbladder wall and extravasation is the diagnostic finding seen in patients with RUQ with cholecystitis. Gallstones, either as the cause, or as an incidental finding may be noted. Pericholecystic edema can be noticed as well.

Bile duct dilation can be seen in patients with RUQ caused by bile duct obstruction.

References

• http://www.uptodate.com/contents/epidemiology-of-a

Last updated:
Time spent: QID:1095 2023-2-12

563 of 1943
Sarah, 48 years of age, underwent cholecystectomy 6 weeks ago for frequent episodes of biliary colic caused by gallstones. An intra-operative cholangiogram was clear with no left stones evident at that time. Today,0 she has presented because
after being symptom-free for one week after the operation, she has started to have bloating and belching again. Which one of the following best explains her symptoms?

A. Pancreatitis.

B. Cystic duct stump syndrome.

C. Stone left in the common bile duct.

D. Symptoms are not related to gallstones.

E. Post-cholecystectomy syndrome.

Incorrect. Correct answer is E


45% answered correctly

Explanation:

Correct Answer Is E

The dyspepsia manifested by bloating and belching after cholecystectomy is suggestive of post-cholecystectomy syndrome (PCS) as the most likely diagnosis.

PCS affects between 10-15% of patients with cholecystectomy and is characterized by a heterogenous group of symptoms including:

Upper abdominal pain


Nausea and vomiting
Diarrhea
Jaundice
Bloating
Excessive gas
Dyspepsia.

These symptoms can be the continuation of symptoms thought to be caused by gallbladder pathology, the development of new symptoms normally attributed to the gallbladder, or symptoms caused by removal of the gallbladder.

In 90%of the time an etiology can be found. The most common etiologies are:

Choledocholithiasis – stones remained or formed in the common bile duct or cystic duct remnant
Biliary dyskinesia
Continuously increased bile flow to the GI tract
Dilation of cystic duct remnant

NOTE – choledocholithiasis is the most common cause of PCS. It can involve the common bile duct of the cystic duct remnant. Choledocholithiasis is classified as retained, if found within 2 years of cholecystectomy or recurrent, if the stone id found
2 years after the surgery. Recurrent stones formed as result of the biliary stasis are often caused by strictures, papillary stenosis, and biliary dyskinesia.

NOTE – ultrasonography is the initial imaging study of choice for patients with suspected PCS.

PCS is a provisional diagnosis and the presentation should be renamed after a specific cause as an explanation for the symptoms is clinched.

(Option A) The presentation is quite different from acute pancreatitis due to chronicity of symptoms and lack of the typical presentation. Chronic pancreatitis can cause dyspepsia but other features suggestive of the diagnosis are absent. Chronic
pancreatitis presents with abdominal pain that can radiate to the back, chronic diarrhea due fat malabsorption and other signs and symptoms caused by pancreatic insufficiency.

(Option B) Cystic duct stump syndrome or cystic duct remnant syndrome refers to gastrointestinal symptoms of bloating, dyspepsia, nausea, etc caused by dilation of the remnant cystic duct. It is among the etiologies for PCS. The patient is likely to
have PCS either due to this condition or other causes. Unless further studies establish the exact etiology, it cannot be said that cystic duct stump syndrome is the most likely cause to this presentation.

(Option C) Retained stones can also be an etiology for PCS and the presentation; however an intra-operative cholangiogram has been clear, making this condition less likely, yet not impossible.

(Option D) A clear cholangiogram makes this option correct but again this is not an etiology.

References

• http://www.ncbi.nlm.nih.gov/pmc/articles/PMC254647

• http://emedicine.medscape.com/article/192761-overv

Last updated:
Time spent: QID:1114 2023-2-12

564 of 1943
Robert, 54 years of age, has presented to your practice with complaint of painless intermittent rectal bleeding. You arrange for colonoscopy. After a good colonic preparation, the entire colon is visualized showing one pedunculated polyp in
transverse colon, one in descending colon and one in the rectum. All three polyps are resected endoscopically and sent for histopathology exam. The polyp resected from the transverse colon shows high-grade dysplasia. Which one of the following
would be the most appropriate investigation to consider for Robert?

A. CT colonography.

B. CT scan of the abdomen and pelvis.

C. Rectal ultrasound.

D. Surveillance colonoscopy in 3 years.

E. Serum alpha-feto protein (AFP) and carcinoembryonic antigen (CEA) tumor markers assay.

Incorrect. Correct answer is D


45% answered correctly

Explanation:

Correct Answer Is D

Although adenomatous polyps (adenomas) of the colon and rectum are benign growths arising from the colonic and rectal mucosa, there is a risk of progression to carcinoma in-situ and eventually to invasive colorectal cancer, therefore, surveillance
is required.

The nature and interval of such surveillance depends on the size, number, and histopathological examination of the polyps (adenomas). As per current National Guidelines by the National Health and Medical Research Council (NHMRC), any
colorectal polyps observed during colonoscopy should be resected and sent for histopathological exam, and surveillance program is considered.

Current Australian guidelines recommend the following surveillance program for colorectal polyps:

Group A (low risk) – any patient with the following falls into this group:

one to 2 adenomas; AND


all <10mm; AND
no villous features; AND
no high-grade dysplasia

For group A surveillance includes colonoscopy at 5-year intervals, or fecal occult blood test (FOBT) every 1-2 years.

Group B (high risk) – any patient with any of the following falls into this group:

three to 4 adenomas; OR
any adenoma ≥10mm; OR
villous features; OR
high-grade dysplasia

For group B surveillance includes colonoscopy at 3-year intervals.

Group C (multiple) – this group includes patients with ≥5 adenomas.

For group C, surveillance includes colonoscopy at 1-year intervals if the number of adenomas is 5-9 and sooner if ≥10.

Group D (possible incomplete or piecemeal excision of large or sessile adenomas)

Surveillance in this group consists of colonoscopy at 3- to 6-months intervals until no residual adenoma is detected after that time surveillance continues at 1-year intervals.

For Robert, either the number of adenomas (3) or the high-grade dysplasia histopathology of the adenoma resected from the transverse colon puts him in group B (high-risk); therefore, colonoscopic surveillance at 3-year intervals is the most
appropriate investigation for him.

(Option A) CT colonography is a new safe and well tolerated tool for diagnosis of colonic polyps and masses and, when performed by experienced hands, is as accurate as traditional colonoscopy. However, cost and training issues are needed to be
addressed before it is widely accepted beyond the current indications of incomplete colonoscopy or patients who are unfit to undergo optic colonoscopy.

(Options B and C) CT scan of the abdomen and pelvis is not an effective tool for surveillance of colorectal adenomas. It is used, however, in cases of colorectal cancer to assess the extent of the disease and staging. Rectal ultrasound is not an
assessment tool for surveillance of colorectal adenomas. It is neither sensitive nor specific.

(Option E) Serum tumor markers such as AFP and CEA are used if the diagnosis is invasive carcinoma. Such markers are used for assessment of the tumor burden as well as monitoring for any post-treatment recurrence.

References

• Colonoscopic Surveillance Intervals - Adenomas

• RACGP - Diagnosing colorectal polyps and masses

Last updated:
Time spent: QID:1204 2023-2-12

565 of 1943
A 72-year-old man is brought to the emergency department with abdominal pain. He reports constipation in the past two months and absolute constipation and not even gas pass
since yesterday. On examination, the abdomen is mildly distended, tympanic to percussion, and there is mild tenderness in the lower abdomen in particular. On digital exam, the
rectum is empty and otherwise normal. An abdominal X-ray shows sudden cut –off the sigmoid gas at its lower end and proximal dilation of the colon. Which one of the following is
the most accurate tool to reach a diagnosis with high certainty?

A. CT scan of the abdomen.

B. Barium enema.

C. Colonoscopy.

D. Abdominal ultrasound scan.

E. Barium meal and follow through.

Correct
45% answered correctly

Explanation:

Correct Answer Is A

This patient is very likely to have bowel obstruction, and with the 2-month history of constipation and radiological finding of a sudden cut-off of the sigmoid and dilated proximal
colon, sigmoid volvulus is the most likely diagnosis.

Once colonic volvulus is suspected based on the history and physical findings and the patient is stable, confirmatory imaging should follow for confirmation of the diagnosis and
prompt action should follow to reduce the risk of complications (if not already developed). The initial imaging to consider is a plain abdominal x-ray. Abdominal x-rays reveal a
distended loop of colon that may resemble a coffee bean or a dilated U-shaped colon with a cut-off point at the site of obstruction. Plain abdominal x-rays may also show distention
of the small bowel with air-fluid levels and decompressed colon distal to the point of volvulus. Further diagnostic modalities to consider for more accurate diagnosis and
determination of the exact anatomical site of obstruction are contrast-enhanced CT scan and water-soluble contrast enema.

Currently, contrast-enhanced abdominal CT scan is the preferred confirmatory diagnostic tool for both cecal and sigmoid volvulus because it is noninvasive, almost readily available,
accurate for both cecal and sigmoid volvulus, and has the advantage of identification of incidental pathology that may be missed with plain radiographs or contrast studies. In
addition, abdominal CT may facilitate the diagnosis of colonic ischemia (A is correct).

(Option B) Barium, as a contrast matter, is contraindicated if bowel perforation is suspected. Although this patient does not have signs of bowel perforation or even ischemia,
barium should be avoided. Should enema be considered as a diagnostic tool, a water-soluble contrast is the option.

NOTE – Avoid barium if viscus perforation is suspected or if the goal is to assess the post-operative leak.

(Option C) Rigid or flexible sigmoidoscopy is the most appropriate initial treatment option for patients in whom bowel perforation or ischemia has been excluded with high certainty.
After endoscopic detorsion, a rectal tube is left in place for 1 to 3 days to maintain reduction. Colonoscopy is only used in unusual cases in which the obstruction point is beyond
the reach f shorter scopes. Definitive treatment is with surgery.

(Option D) Ultrasound scan is not an appropriate diagnostic tool for suspected colonic volvulus. It has no benefit over plain radiograph for this purpose.

(Option E) Barium meal and follow through is an investigation to consider for suspected small bowel obstruction.

References

• http://www.imagingpathways.health.wa.gov.au/index.php/imaging-pathways/gastrointestinal/acute-abdomen/suspected-bowel-obstruction#pathway

• https://www.aci.health.nsw.gov.au/networks/eci/clinical/clinical-resources/clinical-tools/abdominal-emergencies/bowel-obstruction/large-bowel-obstruction

• https://www.aci.health.nsw.gov.au/networks/eci/clinical/clinical-resources/clinical-tools/abdominal-emergencies/bowel-obstruction/large-bowel-obstruction

Last updated:
Time spent: QID:1205
2023-2-12

566 of 1943
A 76-year-old man is brought to the Emergency Department with colicky abdominal pain and abdominal distention for the past 24 hours. He has not passed any stool or gas in this
period. He gives the history of constipation for the past month. His past medical history is significant for appendectomy at the age of 42 years. On examination, he has stable vital
signs. Abdominal exam reveals a distended abdomen, tympanic to percussion. No tenderness, rebound tenderness or guarding is elicited. She has also a reduceable mass in her left
inguinal region suggestive of an inguinal hernia. Rectum is empty with no mass. Which one of the following can be the most likely cause to this presentation?

A. Sigmoid volvulus.

B. Sigmoid cancer.

C. Adhesions from previous surgery.

D. An obstructed hernia.

E. Fecal impaction.

Correct
45% answered correctly

Explanation:

Correct Answer Is A

The history of prolonged constipation, as well as the exam findings of a tympanic distended abdomen, colicky abdominal pain and an empty rectum is more consistent with sigmoid
volvulus as the most likely diagnosis.

A colonic volvulus occurs when a part of the colon twists around its mesentery and causes colonic obstruction. Such obstruction can be acute, subacute or chronic. Although
volvulus can occur in any portion of the large bowel, the sigmoid colon is the most frequently affected part, followed by volvulus of the right colon and terminal ileum, namely cecal
or cecocolic volvulus. In very rare cases, volvulus may develop in the transverse colon or the splenic flexure.

Patients with colonic volvulus are commonly elderly, debilitated, and bedridden. A history of dementia or neuropsychiatric impairment is often present. The symptoms are acute in
more than 60-70% of patients, while the rest present with subacute or chronic symptoms. A history of chronic constipation is common. The patient may describe previous episodes
of abdominal pain, distention, and obstipation, which suggest repeated subclinical episodes of volvulus.

Regardless of its anatomic site, colonic volvulus presents the same way with cramping abdominal pain, distention, constipation and/or obstipation. Abdominal distention often
increases progressively. The distention is characteristically tympanic over the gas-filled, thin-walled colon loop. Tenderness or rebound tenderness indicates that peritonitis has
occurred or is imminent.

With progressive obstruction, nausea and vomiting occur. The development of constant abdominal pain is an ominous sign indicating the development of a closed-loop obstruction
with significant intraluminal pressure. This leads to the development of ischemic gangrene and bowel wall perforation.

Depending on the extent of bowel ischemia or fecal peritonitis, signs of systemic toxicity may be present. Because of the massive abdominal distention, the patient may have
respiratory and cardiovascular compromise.

Plain abdominal films are the initial imaging choice. Massive dilation of the sigmoid colon loop arising from the pelvis and extending to the diaphragm is a typical finding of sigmoid
volvulus. The walls of the loop are evident as three bright lines converging in the pelvis to create a beaklike appearance.

(Option B) A sigmoid tumor can also be the underlying cause of this clinical picture. Left-sided colon cancer can present with bloody stool, changes in bowel habits and abdominal
pain specially if the tumor has caused partial or complete obstruction. However, a sigmoid tumor large enough to cause complete obstruction of the colon to result in obstipation is
expected to have more pronounced exam findings.

(Option C) Obstruction caused by the entrapment of a loop of the small bowel in an adhesion band formed from previous surgery can cause small bowel obstruction. In fact,
adhesions from previous abdominal surgeries are the most common cause of small bowel obstruction. In this patient, however, the clinical picture more favors large bowel rather
than small bowel obstruction because in small bowel obstruction nausea and vomiting are prominent features occuring earlier in the course of the process. This patient has not
vomited until after 24 hours of symptoms onset.

The clinical presentation of small bowel obstruction includes the following:

Nausea/vomiting (60-80%): The vomitus can often be bilious in nature


Constipation/absence of flatus (80-90%): Typically, a later finding of SBO
Abdominal distention (60%)
Fever and tachycardia: Late findings and may be associated with strangulation

(Option D) An obstructed hernia is expected to cause constant abdominal pain and tenderness. In early early stages and before the strangulation occurs, there is no
tenderness. This patient has a mass suggestive of hernia. However, the mass is reduceable indicating that the hernia is not yet incarcerated or strangulated. A reducible hernia does
not cause obstruction. Even in case of incarceration, it is the small bowel that becomes entrapped; therefore, symptoms would be expected to be in favor of small bowel
obstruction. This patient has the clinical presentation of a large bowel obstruction.

(Option E) As the name implies, this kind of obstruction is caused by impaction of fecal matter often in the rectum. An empty rectum excludes this diagnosis.

References

• Medscape – Small-Bowel Obstruction

• Medscape – Large Bowel Obstruction

• Medscape – Colon Cancer

567 of 1943
• Emergency Care Institute - Large bowel obstruction
Last updated:
Time spent: QID:1206
2023-2-12

568 of 1943
Which one of the following conditions is not associate with dysphagia?

A. Achalasia.

B. Parkinson's disease.

C. Esophageal varices.

D. Esophageal cancer.

E. Esophageal stricture.

Incorrect. Correct answer is C


45% answered correctly

Explanation:

Correct Answer Is C

Dysphagia means difficulty in swallowing and can be classified as three types:

Oropharyngeal dysphagia – difficulty in passing liquids more than solids due to neurologic or muscular problems such as:

Parkinsonism
Myasthenia gravis – the upper one-third of the esophagus has striated muscle. If myasthenia gravis affects this part, dysphagia ensues.
Prolonged intubation
Zenker’s diverticulum

Esophageal dysphagia – difficulty in passing solids more than liquids due to local obstruction in the following conditions:

Strictures
Schatzki ring
Webs
Carcinoma.

Motility-related dysphagia – difficulty in passing both solids and liquids at the same time, seen in conditions such as:

Achalasia
Scleroderma

At occasions, there is odynophagia that is painful swallowing. In fact, in odynophagia the difficulty in swallowing is caused by the pain, not by structural abnormalities. Monilial (candida) esophagitis and pill esophagitis are two examples of
conditions leading to odynophagia.

Of the given options, the only condition which is not associated with dysphagia (or odynophagia) is esophageal varices.

References

• Medscape - Dysphagia

Last updated:
Time spent: QID:161 2023-2-12

569 of 1943
Which one of the following is the most likely presentation of a rectal cancer?

A. Melena, constipation and colicky lower abdominal pain.

B. Diarrhea, bloody stool and abdominal pain.

C. Constipation.

D. External hemorrhoids.

E. Fatigue.

Incorrect. Correct answer is B


45% answered correctly

Explanation:

Correct Answer Is B

Bloody stool (rectal bleeding), bowel habit changes mostly in form of diarrhea or altered stool caliber, and abdominal pain are the most common presenting symptoms in order of
incidence in rectal cancer.

Rectal cancers present with a variety of sign and symptoms including the following:

Rectal bleeding – bleeding is the most common presenting symptom of rectal cancer, occurring in 60% of patients. It should be noted, however, that profuse bleeding and
anemia are rare. Occult bleeding detected through a fecal occult blood test (FOBT) is present in 26% of all cases.

Changes in bowel habits – this is the second most common presenting symptom, occurring in 43% of patients. Bowel habit changes is often in form of diarrhea rather
than constipation. Some patients report a decrease in stool caliber. Large tumors can cause obstruction and obstructive symptoms. Tumors located low in the rectum can
cause a feeling of incomplete evacuation and tenesmus.

Back pain is usually a late sign caused by a tumor invading or compressing nerve trunks.

Urinary symptoms may also occur if the tumor is invading or compressing the bladder or prostate.

Malaise is a nonspecific symptom and present in 9% of rectal cancer cases.

Bowel obstruction due to a high-grade rectal lesion is rare, occurring in 9% of all cases.

Pelvic pain is a late symptom, usually indicating nerve trunk involvement, and is present in 5% of all cases.

Other manifestations include emergencies such as peritonitis from perforation (3%) or jaundice, which may occur with liver metastases (< 1%).

(Option A) Melena is the black stool caused by oxidation of blood in the GI. Melena is a feature in upper gastrointestinal rather than lower gastrointestinal bleeding, and not a
presenting symptom in rectal cancers. Although a change in bowel habit is the second most presenting symptom of rectal cancers, this change is more often in form of diarrhea
rather than constipation. Lower abdominal pain due to partial or complete obstruction can be a feature.

(Option C) As mentioned earlier, diarrhea is more common than constipation in rectal cancers.

(Option D) External hemorrhoids (a thrombosed anal vein) is not a common presenting complaint in patients with rectal cancer.

(Option E) Fatigue is feature more commonly seen in right-sided colon cancers. As the blood mixes with the stool, bloody stool is a less frequently encountered presentation. On the
other hand, bleeding from right-sided lesions result in diarrhea (blood is cathartic) and anemia. The latter is the main cause of fatigue in patients with right-sided malignant lesions.
Fatigue or malaise is seen in only 9% of patients with rectal cancer.

References

• Medscape - Rectal Cancer

Last updated:
Time spent: QID:1207
2023-2-12

570 of 1943
Which one of the following is the most common mode through which hepatitis B infection is transmitted in Australia?

A. Sexual intercourse.

B. Transfusion of blood and blood products.

C. Injecting drug use.

D. Vertical transmission from the mother to the infant.

E. Breast feeding.

Correct
45% answered correctly

Explanation:

Correct Answer Is A

Hepatitis B virus is a blood-borne virus, meaning that the virus should enter the blood stream in order to cause infection. Modes of transmission include:

sharing contaminated objects that pierce the skin or mucous membranes, such as needles, tattoo equipment, body-piercing equipment, acupuncture equipment, razor
blades and toothbrushes
sexual contact (heterosexual or homosexual, more common with homosexual contact)
perinatal transmission from an infected mother to her infant (vertical transmission)
needlestick injuries
contact between infective fluid and mucous membranes, such as a splash of blood into eyes or mouth
invasive medical or dental procedures if there has been inadequate infection control
transfusion of infected blood or blood products
human bites and other direct contact with the blood or open spores of an infected person

Of these routes, some are more common than the others in different parts of the world, depending on the regional prevalence of the chronic hepatitis B infection:

Low-prevalence areas: Such areas include the United States, Canada, Western Europe, Australia, and New Zealand. The prevalence of hepatitis B in such areas is ≤2% (0.1-
2%). Less than 12% HBV infected individuals live in these areas. The most common route of transmission in these countries are sexual transmission and percutaneous
transmission.

Intermediate-prevalence areas: Sexual and percutaneous transmission and transmission during delivery are the major transmission routes in areas of intermediate
prevalence (rate of 3-5%). These regions include Eastern and some areas of Northern Europe, Japan, some Mediterranean countries, Latin and South America, and Central
Asia.

High-prevalence areas: Such areas include China, Southeast Asia, sub-Saharan Africa, Pacific Islands, Parts of Middle East and Amazon Basin. The prevalence in ≥8%
(often 10-20%). Perinatal infection including vertical transmission from the mother to the infant is the predominant mode of transmission.

Australia has a chronic hepatitis B prevalence of less than 0.5% and is among low-prevalence regions of the world. The most common mode of transmission of hepatitis B infection
in Australia is heterosexual and homosexual sexual intercourse with the latter being associated with higher risk of transmission. This is followed by percutaneous transmission such
as in injecting drug use and non-sterile procedures such as piercing, tattooing, etc.

(Option B) In Australia, screening of blood and organ donors using serological, and subsequently nucleic acid amplification testing, has virtually eliminated the risk of transmission
of hepatitis B through blood transfusion and organ transplants.

(Option C) Injecting drug use predominates other modes of transmission in newly infected individuals with hepatitis C in Australia. It is also an important mode of transmission in
hepatitis B after sexual intercourse.

(Option D) Perinatal transmission including vertical transmission from the mother to the infant is the predominant route of transmission of hepatitis B infection in areas with high
prevalence of HBV infection. In such areas the prevalence is ≥8% (generally 10-20%). These areas include China, Southeast Asia, Indonesia, sub-Saharan Africa, Pacific Islands, parts
of the Middle East, and the Amazon Basin. Due to perinatal care and administration of hepatitis B vaccine within the 12 hours of birth, this route of transmission is not common in
Australia.

(Option E) Breast feeding does not appear to be a significant route of transmission, neither are fecal-oral and vector-borne modes.

NOTE - Saliva may contain levels of virus that are likely to be infective only if inoculated directly into tissue (ocular or mucous membranes). The risk of transmission by inadvertent
inoculation by other means, such as by toothbrush, razor etc., or through close personal contact in households in which one or more infected persons reside, is low but not
negligible.

References

• https://emedicine.medscape.com/article/177632-overview#a5

• http://www.immunise.health.gov.au/internet/immunise/publishing.nsf/Content/Handbook10-home~handbook10part4~handbook10-4-5

Last updated:
Time spent: QID:1230
2023-2-12

571 of 1943
A 24-year-old man is being evaluated for dysphagia as his main chief complaint. He has past medical history of childhood asthma. Endoscopy is arranged showing narrowing and
inflammation of middle esophagus. Biopsy is significant for the presence of eosinophils in the esophageal mucosa. Which one of the following is the most appropriate treatment
option to consider for this patient at this stage?

A. Oral antifungal agents.

B. A trial of proton pump inhibitors.

C. Swallowed budesonide.

D. Systemic corticosteroids.

E. Triple therapy against H.pylori.

Incorrect. Correct answer is B


45% answered correctly

Explanation:

Correct Answer Is B

Primary eosinophilic esophagitis (EoE) is an increasingly recognized medical condition characterized clinically by symptoms related to esophageal dysfunction, and histologically by
eosinophilic inflammation in the esophagus.

EoE is hypothesized to be an atopic inflammatory disease caused by an abnormal immune response to antigenic stimulation, mostly foods. Normally, eosinophils are normal
component of mucosal infiltrates in all-length of the gastrointestinal tract except the esophagus. Eosinophils in the esophageal mucosa are always pathologic.

Generally, the clinical symptoms of EoE are nonspecific, and the patients are in good physical condition resulting in a delayed diagnosis (years) in some cases.

The presenting symptoms vary depending on the age of the onset:

Children - children tend to present with nausea and vomiting, weight loss, anemia, and failure to thrive. In neonates and infants, refusal of food is the most common presenting
symptom.

Adults - the characteristic symptoms in adults include dysphagia for solid foods, retrosternal pain and food impaction. Some patients also present with gastroesophageal reflux
disease (GERD) symptoms unresponsive to medical anti-reflux therapy. A subset of patients have been recognized to have a typical clinical presentation of EoE in the absence of
GERD who show a clinicopathologic response to PPIs. This condition is currently referred to as PPI-responsive EoE.

There is no Australian guidelines for diagnosis and management of EoE and current recommendation is based on the guidelines by the American College of Gastroenterology (ACG).

According to the ACG, diagnostic criteria for EoE include all of the following:

Symptoms related to esophageal dysfunction.


≥15 eosinophils/hpf on esophageal biopsy
Persistence of eosinophilia after a proton pump inhibitor (PPI) trial
Secondary cause of esophageal eosinophilia excluded

This patient has symptoms related to esophageal dysfunction (dysphagia and odynophagia) and established eosinophilia on histological studies. In order for EoE to be the definite
diagnosis, it is necessary that eosinophilia persists after an 8-week trial of a PPI.

The rationale behind this trial is that GERD may mimic EoE, coexist with it, or contribute to it. Conversely, EoE may contribute to GERD; therefore, the diagnosis of EoE is generally
made after the symptoms persist after an 8-week course of proton pump inhibitors (PPIs) as the best initial step in management. PPIs may benefit patients with EoE either by
reducing acid production in patients with co-existent GERD, or by other unknown anti-inflammatory mechanisms.

The main three components of treatment in established EoE are (1) dietary advice and alteration, (2) pharmacotherapy, and (3) surgical intervention. For pharmacological
intervention topical swallowed steroids (e.g., fluticasone, budesonide) are considered the main treatment options, once the diagnosis of EoE is established either after failed PPI
therapy or normal pH studies.

For patients unresponsive to the above measure, oral (systemic) predniso(lo)ne maybe indicated.

(Option A) The condition is not infectious and antifungal or antimicrobial agents have no role in treatment.

(Option C) Topical swallowed steroids (e.g., budesonide) are considered as the next step, once the diagnosis of EoE is established either after failed PPI therapy or normal pH
studies.

(Option D) Systemic corticosteroids are a treatment option for patients unresponsive to PPIs or swallowed corticosteroids.

(Option E) Eradication of H.pylori have not shown efficacy in treatment of EoE.

TOPIC REVIEW

572 of 1943
Causes of esophageal eosinophilia:

Eosinophilic esophagitis
GERD
PPI-responsive eosinophilic esophagitis
Achalasia
Crohn’s disease
Parasitic infections
Drug hypersensitivity
Connective tissue disease (e.g., scleroderma, dermatomyositis)
Celiac disease
Hypereosinophilic syndrome

References

• RACGP - Eosinophilic oesophagitis – A guide for primary care

• World Journal of Gastroenterology - Eosinophilc esophagitis

• Medscape - Eosinophilic Esophagitis

Last updated:
Time spent: QID:1260
2023-2-12

573 of 1943
After evaluation of a 67-year-old woman with right upper quadrant abdominal pain, dyspepsia and weight loss, she is found to have a solitary tumor in the right lobe of her liver. CT
scanning of the liver suggests the tumor to be malignant. Which one of the following could be the most likely diagnosis?

A. Hepatoma.

B. Metastasis from colon cancer.

C. Metastasis from stomach cancer.

D. Metastasis from breast cancer.

E. Metastasis from ovarian cancer.

Correct
45% answered correctly

Explanation:

Correct Answer Is A

Approximately 90% of malignant liver tumors are metastases from other primary malignancies elsewhere. Hepatoma, also known as hepatocellular carcinoma (HCC) is a primary
liver tumor accounting for 10% of malignant liver lesions.

In metastatic liver disease, metastases are usually multiple rather than single; therefore, a solitary malignant liver tumor is more likely to have caused by HCC rather than metastasis.

References

• https://gi.org/guideline/diagnosis-and-management-of-focal-liver-lesions/

• https://emedicine.medscape.com/article/197319-overview

Last updated:
Time spent: QID:1269
2023-2-12

574 of 1943
A 35-year-old man is brought for assessment by his wife with complaints or frequent episodes of confusion, dysarthria and wide-based gait. He is an office manager and has been
behaving peculiarly recently at work. Of importance, his father and paternal aunt have died with the same symptoms at ages of 50 and 53, respectively. Which one of the following is
most likely to make a diagnosis?

A. CT scan of the head.

B. MRI of the head.

C. Serum copper and ceruloplasmin level.

D. Lumbar puncture.

E. Liver function tests.

Incorrect. Correct answer is C


45% answered correctly

Explanation:

Correct Answer Is C

Neuropsychiatric problems in a young patient in the presence of a family history of such presentation is suggestive of Wilson disease. Wilson disease is a rare (3 in 100000)
inherited disease (autosomal recessive) which is caused by accumulation of copper in the liver and central nervous system. The genetic basis is a mutation of a gene on
chromosome 13 that codes for a copper transporting ATPase, ATP7B. Over 200 mutations have been identified so far.

Total body copper content is 100-150mg with an average intake of 1-5mg/day. The dietary copper is absorbed in the proximal small bowel. Copper is incorporated into
ceruloplasmin in the liver. In Wilson disease copper absorption and transport in the liver are intact but incorporation into ceruloplasmin in the hepatocytes and biliary excretion is
impaired. This results in accumulation of copper in the liver and later on other organs (e.g. CNS). The most common presentations are with liver disease or neuropsychiatric
disturbances. Asymptomatic patients are most often detected by family screening.

The clinical hallmark of Wilson’s disease is the corneal Kayser–Fleischer ring, which is present in 95% of patients with neurologic symptoms and somewhat over half of those
without neurologic symptoms.

Kayser-Fleischer ring - deposition of copper in the limbus

While most affected children present with hepatic involvement (hepatitis, cirrhosis, or fulminant liver failure) adults often present with neuropsychiatric manifestations as the
presenting symptoms.

Neuropsychiatric manifestations of Wilson disease include dysarthria, dysphagia, dyskinesias, purposeless stereotyped movements, dementia, Parkinsonism, micrographia,
clumsiness. There can be also decreased memory, anger outbursts, labile emotions, decreased libido and personality changes.

Other possible clinical feature are:

Coombs negative hemolytic anemia


Polyarthritis
Hypermobile joints
Grey skin
Abortions
Hypoparathyrodism

Diagnostic tests include serum copper (increased) and ceruloplasmin (decreased). It should be noted, however, that ceruloplasmin is an acute phase reactant and may be increased
in response to hepatic inflammation, pregnancy, estrogen use, or infection.)

The urinary copper excretion rate in patients with Wilson disease is greater than 100 mcg/d (normal<40mcq/day). Although the sensitivity and the specificity of this test are
suboptimal for use as a screening test; it may be useful to confirm the diagnosis and to evaluate the response to chelation therapy.

NOTE - The criterion standard for diagnosis of Wilson disease is liver biopsy. Copper levels of more than 250mcq per each gram of dry weight of the liver in an asymptomatic patient
establishes the diagnosis.

Chelation with D-penicillamine or trientine are mainstay of therapy. Patients should be on diets with decreased sources of dietary copper.

575 of 1943
CT scan of the head (option A) or preferably MRI of the head (option B) are used for the degree of brain involvement in Wilson disease.

Lumbar puncture (option D) has no role in diagnosis of Wilson disease.

Liver function tests are impaired in most patients with Wilson disease. These changes are, however, nonspecific and can be seen in a variety of other liver diseases; hence, not
diagnostic.

References

• https://emedicine.medscape.com/article/183456-overview

• https://www.aasld.org/sites/default/files/guideline_documents/Wilson%20Disease2009.pdf

Last updated:
Time spent: QID:1277
2023-2-12

576 of 1943
A 35-year-old woman presents to the emergency department with acute onset right upper quadrant pain which started one hour ago. When visited by the emergency physician, she
mentions that the pain has gone away. An ultrasound scan of the liver and biliary tree reveals a simple cyst in the liver. Which one of the following is the most appropriate next step
in management?

A. Reassure her.

B. Aspiration of the cyst.

C. Abdominal CT scan.

D. Laparoscopic removal of the cyst.

E. MRI of the liver.

Incorrect. Correct answer is C


45% answered correctly

Explanation:

Correct Answer Is C

Simple liver cysts are often congenital and hypothesized to develop from intrahepatic biliary ducts because they have a lining similar to biliary ducts.

Simple cysts generally cause no symptoms but may produce dull right-upper-quadrant pain if large in size. Patients with symptomatic simple liver cysts may also report abdominal
bloating and early satiety. Occasionally, a cyst is large enough to produce a palpable abdominal mass. Very rarely, jaundice caused by bile duct obstruction or rupture and acute
torsion of a mobile cyst may occur. Patients with cyst torsion may present with an acute abdomen. When simple cysts rupture, patients may develop secondary infection, leading to
a presentation similar to a hepatic abscess with abdominal pain, fever, and leukocytosis.

It is very important to differentiate a simple cyst from malignant hepatic lesion. One differential diagnosis to exclude is a liver tumor with central necrosis that is often misdiagnosed
as simple hepatic cyst. Genuine intrahepatic neoplastic cysts are rare.

If a cystic lesion is found on ultrasound, often contrast-enhanced CT scan must be considered to characterize the lesion. Images are taken during the pre-contrast, arterial and portal
venous phase.

(Option A) Reassuring a patient with a symptomatic cystic lesion is not appropriate unless further investigations are conducted the other causes of such presentation has been
excluded. Symptomatic hepatic cysts usually require treatment.

(Option B) Percutaneous aspiration guided by ultrasonography or computed tomography (CT) is technically simple but has been abandoned because the recurrence rates are nearly
100%. Aspiration combined with sclerosis with alcohol or other agents has been successful in some patients but has high failure and recurrence rates. Successful sclerosis
depends on complete decompression of the cyst and apposition of the cyst walls. This is not possible if the cyst wall is thickened or if the cyst is large.

Percutaneous catheters should not be placed to drain simple cysts, because the cavity becomes contaminated, leading to the development of hepatic abscess. Unlike the typical
pyogenic hepatic abscess, this problem is difficult to resolve with repeated catheter placements because of continued secretion from the cyst epithelium.

(Option D) Laparoscopic unroofing of the cyst by excising the portion of cyst wall that extends to the surface the liver is the treatment of choice for symptomatic liver cysts. This,
however, should be considered once further investigations establishes the diagnosis of a simple cysts and excludes other differential diagnoses.

NOTE - No medical therapy has proved effective in reducing the size of simple hepatic cysts.

(Option E) MRI has a limited role in evaluation of simple cysts and is often unnecessary.

References

• https://emedicine.medscape.com/article/190818-overview

• http://www.imagingpathways.health.wa.gov.au/index.php/imaging-pathways/gastrointestinal/liver/investigation-of-a-focal-liver-nodule?tab=HS7#pathway

Last updated:
Time spent: QID:1301
2023-2-12

577 of 1943
A 57-year-old man is brought to the Emergency Department of the local hospital you are working in after an episode of vomiting large amount of fresh blood. A couple of hours
before presentation, he had binge drinking which was followed by retching several times and vomiting before he vomits blood. This is the first time he is having such problem. He
has enjoyed good health otherwise and has not had any complaints except indigestion after meals for which he is taking over-the-counter antacids. He does not smoke but drinks
equivalent to 30 grams of alcohol every day. Which one of the following could be the most likely cause of his presentation?

A. Duodenal ulcer.

B. Gastric cancer.

C. Gastric ulcer.

D. Mallory-Weiss tear.

E. Esophagitis.

Incorrect. Correct answer is D


45% answered correctly

Explanation:

Correct Answer Is D

Although peptic ulcer disease including duodenal ulcers (option A) and gastric ulcers (option C) is the most common cause of bloody vomiting, the fact that the bleeding has
followed frequent retching, makes Mallory-Weiss tear more likely as the diagnosis.

NOTE - Without the retching in history, peptic ulcer disease would have been the most likely diagnosis.

Mallory-Weiss tear is a linear laceration at the gastroesophageal junction. This tear is hypothesized to occur either by a rapid increase in intragastric pressure and distention, which
increases the forceful fluid ejection through the esophagus, or secondary to a significant change in transgastric pressure, which is a difference in pressure across the gastric wall
due to negative intrathoracic pressure and positive intragastric pressure. Such change leads to distortion of the gastric cardia, resulting in a gastric or esophageal tear.

Mallory-Weiss tears are the cause of hematemesis in 3-5% of cases in adults. The condition is frequently encountered in binge alcohol drinkers, mostly due to repeated retching
and/or vomiting; however, even one single episode of retching or vomiting can result in the tear.

Hematemesis is the presenting symptom in virtually all patients with Mallory-Weiss tears. The diagnosis is not dependent on the amount of hematemesis because it can vary from
blood flecks or streaks of blood mixed with gastric contents or mucus to a significantly large amount of bright red bloody emesis. Melena is another presentation reported to occur
in as many as 10% patients. Other possible symptoms include:

Patients with severe vomiting can develop light-headedness or dizziness, or even syncope. These symptoms usually occur secondary to dehydration from the underlying cause of
vomiting and are not secondary to blood loss from the Mallory-Weiss tear. Only in rare cases a Mallory-Weiss tear lead to anemia requiring transfusions.

Abdominal pain and dyspepsia - As many as 40% of patients may experience epigastric pain or heartburn. These symptoms are often related to the underlying cause of vomiting and
not specifically to the Mallory-Weiss tear.

(Option B) vomiting of a large amount of fresh blood in an unusual presentation for gastric cancer. Additionally, if the cancer was the cause, features such as weight loss as well as
other cancer-associated symptoms would have been expected.

(Option E) Esophagitis can result in a frail mucosa that bleeds easily; however, symptoms related to such bleeding are expected to be iron deficiency anemia (in the long run) and
melena (or occult blood in the stool) rather than copious bloody vomitus.

References

• Medscape - Mallory-Weiss Syndrome

Last updated:
Time spent: QID:1318
2023-2-12

578 of 1943
A 45-year-old patient presents to your GP clinic with complaint of dyspepsia. Evaluations unveil peptic ulcer disease and H.pylori infection. You start him on a 14-day course of triple
therapy with amoxicillin, metronidazole and pantoprazole. Ten days after completion of the treatment, the patient is only partially improved and the dyspepsia still exists, though to
a lesser extent. You arrange for a breath urea test which turns out positive. Which one of the following could be the most likely cause of treatment failure in this patient?

A. Amoxicillin resistance.

B. Metronidazole resistance.

C. Inappropriate timing of the test after treatment.

D. Unreliable urea breath test.

E. Patient’s non-compliance to treatment.

Incorrect. Correct answer is B


45% answered correctly

Explanation:

Correct Answer Is B

The most common cause of treatment failure against H.pylori infection is antibiotic resistance.

Antibiotics used in multiple drug regimens against H.pylori are clarithromycin, metronidazole, and amoxicillin. Of these three, resistance to metronidazole and clarithromycin are the
most common causes of treatment failure.

While resistance to metronidazole can be overcome by increasing the dose of metronidazole and duration of therapy, this method is not effective where clarithromycin resistance is
the issue.

(Option A) H.pylori is still susceptible to amoxicillin and resistance to this antibiotic has not been reported as a cause of treatment failure.

(Option C) A breath urea test is often performed within 4-6 weeks of the commencement of therapy. This patient has presented 10 days after completion of a 14-day course of
antibiotics (so 24 days after the commencement of treatment) and nearly about the time of the test.

(Options D and E) Patients' non-compliance or an unreliable test result with a standard kit are less likely explanations for this patient’s persisting symptoms.

References

• Antibiotic-resistant H. pylori infection and its treatment.

Last updated:
Time spent: QID:1363
2023-2-12

579 of 1943
A 40-year-old man presents to your practice with an inguinoscrotal swelling. Which one of the following findings is consistent with the diagnosis of indirect inguinal hernia?

A. Swelling of the posterior part of the scrotum.

B. Swelling of the anterior part of the scrotum.

C. The examiner’s finger cannot get above the swelling.

D. The swelling is transilluminable.

E. The scrotum is painful.

Incorrect. Correct answer is C


45% answered correctly

Explanation:

Correct Answer Is C

In indirect inguinal hernia, the bowel enters the scrotum through the inguinal canal. The hernia is often felt as a swelling filling the scrotum. Since the swelling originates from the
abdomen and the inguinal area, the examiner’s finger cannot get above the swelling and this the most important finding that distinguishes a swelling arising from within the scrotum
such as a testicular tumor, an epididymal cyst, hydrocele, sebaceous cyst from a swelling originating from out of scrotum.

(Options A and B) An indirect inguinal hernia fills the entire scrotum not just the anterior or posterior part of it.

(Option D) Although a gas-filled loop of bowel in the scrotal sac, as a result of indirect inguinal hernia, can be transilluminable, transillumination is more likely to due to a hydrocele.
Even so, what differentiates between these two is that the examiner’s finger cannot get above the swelling in indirect hernia.

(Option E) A painful scrotum can be due to a variety of reasons such as epididymo-orchitis, incarcerated or strangulated indirect hernias, testicular torsion or trauma. Presence of
pain in a patient with scrotal swelling is not diagnostic for an indirect inguinal hernia.

References

• Medscape – Abdominal Hernias

Last updated:
Time spent: QID:1369
2023-2-12

580 of 1943
A 27-year-old woman presents to the Emergency Department with severe perianal pain started 10 hours ago that has increased progressively since then. The pain is exquisite and
the patient is unable to sit. The anal region is shown in the following photograph. Which one of the following would be the most important part of management for this patient?

A. Incision under local anesthesia.

B. Hemorrhoidectomy under general anesthesia.

C. Topical antihemorrhoid agents.

D. Antibiotics.

E. Hemorrhoidectomy under topical anesthesia.

Correct
45% answered correctly

Explanation:

Correct Answer Is A

The picture shows an erythematous raised area near the anus almost covering it. Such appearance, as well as the symptoms and clinical findings, is consistent with a perianal
abscess as the most likely diagnosis. Physical examination in patients with perianal abscess demonstrates an erythematous, well-defined, fluctuant, subcutaneous mass near the
anal orifice. Vital signs are often within the normal ranges.

As a rule, abscesses have to be treated with incision and drainage. Simple perianal abscesses can be treated in the emergency department under local anesthesia followed by
administration of systemic antibiotics. More complex perianal abscesses may need treatment in the operating room.

(Options B, C and E) Hemorrhoids have a different clinical picture depending on their grade. Bright rectal bleeding is the most common symptom in internal hemorrhoids. Other
manifestations include pruritus, mucous discharge and anal pain and discomfort. A thrombosed anal vein (formerly called external hemorrhoid) presents with a painful and tender
bluish mass protruding out of the anus. This patient does not have hemorrhoid; therefore, any option suggesting treatment of hemorrhoids is incorrect.

(Option D) Antibiotics alone are inadequate for treatment of a perianal abscess. Clinical suspicion of a perianal abscess warrants surgical incision and drainage. Delaying surgical
intervention results in chronic tissue destruction, fibrosis, and stricture formation that may result in impaired anal continence.

References

• Medscape - Anorectal Abscess

Last updated:
Time spent: QID:1387
2023-2-12

581 of 1943
Which one of the following is the most important diagnostic feature of achalasia?

A. Dysphagia for solids.

B. Dysphagia for liquids.

C. Dysphagia for both solids and liquids.

D. Regurgitation.

E. Weight loss.

Incorrect. Correct answer is C


45% answered correctly

Explanation:

Correct Answer Is C

Achalasia is caused by lack of a relaxation of the lower esophageal sphincter and aperistalsis of the esophageal body. The condition is characterized by degeneration of the nerves innervating the esophagus of unknown cause.

Symptoms of achalasia include the following:

Dysphagia – the most common symptom: like other motility disorders of the esophagus there is dysphagia for both liquids and solids at the same time.
Regurgitation – regurgitation of achalasia is worse at night when the patient is in lying position, but that of retropharyngeal pouch (Zenker’s diverticulum) occurs even when the patient is standing.
Chest pain
Heart burn
Weight loss – weight loss can occur with all types of dysphagia

Of the symptoms, dysphagia to both liquids and solids is the cardinal and the most common symptom of achalasia.

(Option A) Dysphagia for solids as the prominent feature is characteristic of conditions that cause a narrowing within the lumen such as a tumor.

(Option B) Dysphagia for liquids as the cardinal symptom seen in oropharyngeal dysphagia such as in Zenker’s diverticulum, Parkinsonism, stroke and myasthenia gravis.

(Option D) Regurgitation in recumbency is also a feature of achalasia but it is not as common as dysphagia (~40% vs. 80%). Regurgitation is also seen in other conditions such as retropharyngeal pouch and is not a feature specific to achalasia.

(Option E) Weight loss can be a manifestation of all esophageal diseases. It is not an exclusive feature of achalasia.

582 of 1943
A 57-year-old man presents to your GP clinic with complaints of dysphagia and hoarseness. The first symptom was hoarseness, which started almost one month ago, followed by
development of dysphagia after 2 weeks. His past medical history is significant for surgery for achalasia 10 years ago. On examination, normal-appearing but paralyzed vocal cords
are noted. A CT scan is arranged that shows a mass in the thoracic inlet. Which one of the following could be the most likely diagnosis?

A. Stricture formation due to the previous surgery.

B. Thyroid cancer.

C. Esophageal cancer.

D. Laryngeal cancer.

E. Lung cancer.

Incorrect. Correct answer is C


45% answered correctly

Explanation:

Correct Answer Is C

Dysphagia and hoarseness can be features seen in esophageal cancer, thyroid cancer or laryngeal cancer. In esophageal cancer, the tumor growth obstructs the esophagus.
Dysphagia often occurs to solid foods first, followed by to liquids, as the lumen progressively becomes narrower. Dysphagia often becomes a symptom once the lumen diameter is
less than 13 mm. In esophageal cancer, hoarseness occurs if the tumor invades the recurrent laryngeal nerve. This is an indicator of advanced disease and unresectability.

A malignant thyroid tumor can also invade the recurrent laryngeal nerve, and cause hoarseness. Dysphagia can also occur due to impingement of the proximal esophagus.

Laryngeal cancer, by invasion to structures within the larynx, causes hoarseness or aphonia. Once large enough in size, a laryngeal tumor can compress the proximal esophagus and
cause dysphagia as a symptom.

Of these three, and given the history of achalasia as a risk factor for esophageal cancer and the presence of a mass in the thoracic inlet on CT scan, esophageal cancer is the most
likely diagnosis compared to thyroid and laryngeal cancers. Because a stricture from the previous surgery (option A) in esophagus can explain the dysphagia but not the
hoarseness.

(Option B) As a malignant thyroid tumor grows, it can compress the adjacent structures in the neck. It can also invade the laryngeal nerve and cause hoarseness. However, a thyroid
cancer is most likely to present with a thyroid nodule. It is unusual for a thyroid tumor to be so large to compresses the esophagus but is missed on physical examination of the
neck.

(Option D) Although laryngeal cancer can cause hoarseness and, if large enough, dysphagia, a neck mass would be expected on imaging. This patient has a mass in the thoracic
inlet. This makes laryngeal cancer almost unlikely. Furthermore, the presence of previous achalasia as a risk factor for esophageal squamous cell carcinoma makes esophageal
cancer a more likely diagnosis. Important risk factors for laryngeal cancer are smoking and alcohol consumption.

(Option E) An apical lung cancer can present with a mass in the thoracic inlet on imaging. A lung cancer can cause hoarseness but is unlikely to cause dysphagia.

TOPIC REVIW

Common clinical manifestations of lung, laryngeal, esophageal, and thyroid cancers:

Lung cancer

Cough
Chest pain
Shortness of breath
Coughing up blood
Wheezing
Hoarseness
Recurring infections such as bronchitis and pneumonia
Weight loss and loss of appetite
Fatigue
Metastatic signs and symptoms may include the following:

Bone pain
Spinal cord impingement
Neurologic problems such as headache, weakness or numbness of limbs, dizziness, and seizures

Laryngeal cancer

Dysphonia/ aphonia
Dysphagia
Dyspnea
Aspiration
Blood-tinged sputum
Fatigue and weakness
Cachexia
Pain
Halitosis

583 of 1943
Expectoration of tissue
Neck mass
Otalgi (ear pain)

Esophageal cancer:

Dysphagia (most common); initially for solids, eventually progressing to liquids


Weight loss (second most common) due to dysphagia and tumor-related anorexia.
Bleeding (leading to iron deficiency anemia)
Epigastric or retrosternal pain
Bone pain with metastatic disease
Hoarseness (due to the involvement of the recurrent laryngeal nerve)
Persistent cough or frequent pneumonia due to tracheobronchial fistulas caused by direct invasion of tumor through the esophageal wall and into the main stem
bronchus

Thyroid cancer

A painless, palpable, solitary thyroid nodule (the most common presentation)


Hoarseness (suggests involvement of the recurrent laryngeal nerve and vocal fold paralysis)
Dysphagia (a sign of esophageal involvement)
Heat intolerance and palpitations (in autonomously functioning nodules)

References

• Medscape – Esophageal Cancer

• Medscape – Thyroid Cancer

• Medscape – Malignant Tumors of the Larynx

• Medscape – Non-Small Cell Lung Cancer

Last updated:
Time spent: QID:1445
2023-2-12

584 of 1943
A 32-year-old woman, who has just recently migrated from Somali to Australia, presents to your practice with complaint of severe anal pain, particularly on defecation, for the past 3
months. She mentions that at times she notices blood streaking her stool or dripping in the toilet bowl after a painful bowel movement. Examination, along with the history, confirms
the diagnosis of anal fissure. Which one of the following options is the most expected predisposing factor for this presentation in this patient?

A. Rectal cancer.

B. Hemorrhoids.

C. Perianal abscess.

D. Rectal schistosomiasis.

E. High-fiber diet.

Incorrect. Correct answer is D


45% answered correctly

Explanation:

Correct Answer Is D

Rectal cancer (option A) can cause altered bowel motions and bleeding. Anal fissures are not a feature of rectal cancer though. Given the patient is young and has anal fissure,
rectal cancer is unlikely to be the predisposing factor to this presentation.

Hemorrhoids (option B) present with painless rectal bleeding, and there is no pain on defecation. Although chronic constipation can be a predisposing factor for both hemorrhoids
and anal fissure, hemorrhoids do not cause anal fissure.

Perianal abscesses (option C) can present with painful or tender swellings around the anus. Although they might cause pain on defecation as the feces passes and compresses an
abscess, they do not cause rectal bleeding or anal fissure. Perianal fistulae, however, can develop as a consequent of perianal abscess.

High-fiber diet (option E) do not predispose to anal fissure. In fact such diet, by constipation prevention, is a management step in both hemorrhoids and anal fissures.

Given the fact that the patient is an immigrant from Somali, where schistosomiasis is endemic, rectal schistosomiasis is most likely to have predisposed to the anal fissure and such
presentation.

Schistosomiasis is either intestinal or urogenital, depending on where the adult flukes are located. In intestinal schistosomiasis, adult worms occupy mesenteric veins, and their
eggs pass into the lumen of the intestine and reach the faeces. In the Eastern Mediterranean Region, Somalia and Sudan remain the most endemic countries.

Gastrointestinal schistosomiasis due to S. mansoni, S. japonicum and S. mekongi can cause bowel lesions such as ulceration, pseudopolyps, and microabcesses. Gastrointestinal
schistosomiasis clinically presents with abdominal pain, altered bowel habits, and blood in stool. Liver enlargement and periportal fibrosis are common in advanced cases, and is
generally linked with ascites and portal hypertension. Other well-documented clinical signs include superficial abdominal blood vessel dilatation, spleen enlargement, and bleeding-
prone esophageal varices.

Patients with severe hepatosplenic schistosomiasis may die from ruptured esophageal varices. Some studies suggest an association of S. mansoni and S. japonica and cancer
development in the liver and colon, but no hard evidence exists to support the association.

Chronic schistosomiasis commonly affects the sigmoid colon and rectum and usually presents with abdominal pain, diarrhea, bleeding, or ulcerations. Given its location of infection,
it can also lead to anal fistulas, fissures, or abscesses.

References

• WHO – Schistosomiasis

Last updated:
Time spent: QID:1550
2023-2-12

585 of 1943
In which one of the following conditions of the colon, occurrence of malignant changes is most likely?

A. Adenomatous polyp.

B. Melanosis coli.

C. Diverticulitis.

D. Ulcerative colitis.

E. Familial polyposis coli.

Incorrect. Correct answer is E


45% answered correctly

Explanation:

Correct Answer Is E

Of the given options, Familial polyposis coli has the strongest association with colorectal cancer and is very significant risk factor for that.

Familial polyposis coli, also termed familial adenomatous polyposis (FAP) is caused by mutations in the APC gene that a person inherits from his or her parents. About 1% of all
colorectal cancers are caused by FAP. In the most common type of FAP, hundreds or thousands of polyps develop in a person’s colon and rectum, often starting at ages 10 to 12
years. Cancer usually develops in 1 or more of these polyps as early as age 20 years. By age 40, almost all people with FAP will have colon cancer if their colon hasn’t been removed
to prevent it. People with FAP also have an increased risk for cancers of the stomach, small intestines, pancreas, liver, and some other organs.

(Option A) Adenomatous polyp: About 70 % of all polyps are adenomatous, making it the most common type of colon polyp. When this type of polyp is found, it is tested for cancer.
Only a small percentage actually become cancerous, but nearly all malignant polyps began as adenomatous.

(Option B) Melanosis coli: is a condition that can be caused by chronic laxative abuse. Studies have not found an association between melanosis coli and an increased risk of
colorectal cancer in humans. Or it may be that the factors that contributed to constipation itself, such as a diet that's too low in fiber, are the reason why this possible higher risk of
cancer exists

(Option C) Diverticulitis: the risk of colorectal cancer is increased in the short-term period after hospitalization related to diverticular disease. According to a recent systematic
review and meta-analysis, the prevalence of colorectal cancer is 1.6% in patients with acute diverticulitis who underwent colonoscopy.

(Option D) Ulcerative colitis: The risk of colorectal cancer for patients with ulcerative colitis is increased, and is estimated to be 2% after 10 years, 8% after 20 years and 18% after 30
years of disease

References

• Cancer Council Australia – Familial Bowel Cancer

Last updated:
Time spent: QID:1558
2023-2-12

586 of 1943
A 56-year-old man has been brought to the Emergency Department with a 12-hour history of abdominal pain and distention since this morning. He mentions no bowel movement or
gas pass since last night, and describes the pain as colicky and mostly around the umbilicus. He has vomited 3 times since this morning. On examination, he has stable vital
sounds. On abdominal exam, there is moderated abdominal distention with no tenderness, rigidity, guarding or rebound tenderness. On auscultation, hyperactive bowel sounds are
noted. Digital rectal exam reveals an empty rectum with no blood or tenderness. There are dark spots on his lips and buccal mucosa. He remembers having these spots since
childhood. The spots on his lip are shown in the following photograph. Which one of the following could be the most likely diagnosis?

A. Cecal cancer.

B. Acute appendicitis.

C. Sigmoid volvulus.

D. Intussusception.

E. Bowel perforation.

Incorrect. Correct answer is D


45% answered correctly

Explanation:

Correct Answer Is D

The dark spots on the lips and buccal mucosa of this patient is highly suggestive of Peutz Jeghers syndrome.

Peutz Jeghers syndrome (PJS) is an autosomal dominant inherited condition characterized by freckling, gastrointestinal polyposis, and increased risk of certain cancers. Features
seen in PJS are:

Freckling - Most people with PJS have flat deeply pigmented spots around and inside their mouth, on their lips, fingers and toes. Less commonly, these spots can be seen on hands
and feet, in the mucosa of the nose, conjunctiva or rectum. Freckles are benign and develop early in childhood and may disappear into adulthood.

Polyposis - Patients with PJS will typically develop dozens to thousands of hamartomas, usually beginning in childhood. These polyps primarily develop in small bowel (jejunum,
ileum and duodenum); however, they can also occur anywhere in the in the gastrointestinal (GI) tract.

GI problems - GI symptoms arise mostly from the complications of hamartomas. Bleeding can occur as a GI hamartoma rubs against intestinal lining. GI bleeding in PJS may preset
as blood in stool either occult or overt, anemia, etc. Bloating and abdominal pain may be seen.

One important well-known complication in GI polyps of JPS is intussusception (infolding of one part of the intestine into another) of the small bowel.

Other - Very early puberty can occur and occasionally male patients develop gynecomastia.

The disease is different in each patient meaning that not all patients show all of the features of the disease or any particular feature to the same extent as other patients.

NOTE - PJS is associated with an increased risk of specific cancers such those of stomach, small intestine, colon, pancreas, breasts, ovaries, cervix or testes. Not everyone
with PJS will develop cancer, but those affected are at an increased risk and with a younger age of onset than those in the general population.

This patient has the clinical picture of small bowel obstruction. Given the PJS in history, small bowel intussusception would be the most likely diagnosis. Small bowel obstruction
presents with central colicky abdominal pain, early development of nausea and vomiting, mild to moderate abdominal distention, and constipation/ obstipation. As the condition
deteriorates, bowel ischemia, necrosis, perforation, and peritonitis can develop.

(Option A)Although people with PJS are at increased risk of GI cancers at younger ages, cecal cancer is a less likely diagnosis in this scenario. Cecal cancer presents with fatigue
due to chronic blood loss rather than signs and symptoms of bowel obstruction.

(Option B) Acute appendicitis can also present with periumbilical pain that gradually shifts and fixes in the right lower quadrant. Nausea and probably vomiting may follow the pain.
The patient usually has anorexia and regional tenderness and often rebound in the right lower quadrant. However, bowel obstruction, as seen in this patient, is not a feature.

587 of 1943
(Option C) In sigmoid volvulus, the sigmoid colon twists around the mesenteric axis and causes bowel obstruction. Abdominal distention is a prominent feature as are other signs
and symptoms of large bowel obstruction such as an empty rectum on DRE. Sigmoid volvulus is typically seen in elderly patients who are bed-ridden and have chronic constipation.
The location of the pain, the age of the patient, and the quick start of the nausea and vomiting are against large bowel obstruction on this patient. Moreover, with Peutz Jeghers
syndrome, intussusception of the small bowel and small bowel obstruction are stronger possibilities.

(Option E) Bowel perforation presents with signs and symptoms of peritonitis such as abdominal tenderness and rebound tenderness, board-like rigidity, and guarding in an ill and
toxic patient.

References

• Cancer Council – Peutz-Jeghers syndrome

• Medscape – Peutz Jeghers Syndrome

Last updated:
Time spent: QID:1586
2023-2-12

588 of 1943
Alfred is a 40 years old patient in your clinic, who has presented with heart burn and chest pain of 12 months duration. The chest pain is unrelated to physical activity, but both the
heart burn and chest pain are worse with meals. He started developing difficulty swallowing to both liquids and solids and has lost 3 kg in this period. He also has regurgitation,
specially when he lies down which he finds the most disturbing of his problems. He also mentions that rolling back his shoulders or raising his arms eases swallowing for him.
Physical examination is unremarkable. Which one of the following could be the most likely diagnosis?

A. Esophageal cancer.

B. Esophageal strictures.

C. Scleroderma.

D. Esophagitis.

E. Achalasia.

Incorrect. Correct answer is E


45% answered correctly

Explanation:

Correct Answer Is E

This scenario is a very typical picture of achalasia. Achalasia is a primary esophageal motor disorder of unknown etiology characterized manometrically by insufficient relaxation of
the lower esophageal sphincter (LES) and loss of esophageal peristalsis; radiographically by aperistalsis, esophageal dilation, with minimal LES opening, ‘bird-beak’ appearance on
barium studies, poor emptying of barium; and endoscopically by dilated esophagus with retained saliva, liquid, and undigested food particles in the absence of mucosal stricturing
or tumor. Achalasia occurs equally in both genders with prevalence that ranges up to 1 per 10,000 persons. The majority of cases are idiopathic, but the syndrome can be associated
with malignancy (especially involving the gastroesophageal junction) and as a part of the spectrum of Chagas disease. Rarely, achalasia is genetically transmitted.

In the early stages of the disease, dysphagia may be very subtle and misinterpreted as dyspepsia. The presence of heartburn due to food stasis can add to the diagnostic confusion.
As achalasia progresses, dysphagia to both solids and liquids develops but it is worse to solids. To ease the progression of the food bolus, patients may modify their eating habits
including eating more slowly or arching the back (rolling back of the shoulders) or raising the arms when swallowing.

Substernal chest pain during meals in the setting of dysphagia, weight loss, and even heartburn may be accompanying symptoms that often lead to misdiagnosis of achalasia as
gastroesophageal reflux disease (GERD). Achalasia must be suspected in those with dysphagia to solids and liquids and in those with regurgitation unresponsive to initial trial of
proton pump inhibitor (PPI) therapy.

NOTE - Of the symptoms, regurgitation is often more bothersome and more likely as a presenting symptom.

(Options A) occasionally, achalasia may be a result of esophageal malignancy, especially at the gastroesophageal junction. However, esophageal cancer in a young patient with no
known risk factors is far less likely than idiopathic achalasia. Moreover, with malignancy, a more rapid course of dysphagia and a more profound weight loss compared to benign
causes of dysphagia are expected.

(Option B) In esophageal stricture, dysphagia for solids occurs first, and dysphagia to liquids develop later as the stricture progressively narrows the esophageal lumen.

(Option C) Scleroderma (systemic sclerosis) is a complex and heterogeneous disease with clinical forms ranging from limited skin involvement (limited cutaneous systemic
sclerosis) to forms with diffuse skin sclerosis and severe and often progressive internal organ involvement (diffuse cutaneous systemic sclerosis). Skin involvement (including
Raynaud’s phenomenon) is almost always present at the time of presentation and diagnosis. Esophageal involvement in scleroderma may mimic the symptoms of achalasia;
however, in the absence of other features of scleroderma, it is a remote possibility for this patient.

(Option D) Esophagitis (inflammation of the esophagus) is associated with pain on swallowing (odynophagia) rather than dysphagia.

References

• UpToDate – Achalasia: Pathogenesis, clinical manifestations, and diagnosis

• Gut and Liver – The Pathogenesis and Management of Achalasia: Current Status and Future Directions

Last updated:
Time spent: QID:1658
2023-2-12

589 of 1943
A 43-year-old man presents to your Genral Practice with a midline abdominal swelling shown in the following photograph. The swelling is not painful but feels uncomfortable. It
significantly decreases on lying down and accentuates when he coughs or raise his legs or shoulder while supine. On physical examination, the lump feels soft and non-tender.
Which one of the following is the most appropriate next step in management?

A. Physiotherapy.

B. Abdominal truss.

C. Hernioplasty with mesh insertion.

D. Laparoscopic surgery.

E. Classic open abdominoplasty.

Correct
45% answered correctly

Explanation:

Correct Answer Is A

The photograph shows a midline swelling starting from the umbilicus extending to halfway between the umbilicus and the xiphoid process of the sternum. The appearance
alongside the history and physical finding makes abdominal recti diastasis (ARD) the most likely diagnosis.

Abdominal rectus diastasis is a condition where the abdominal muscles are separated by an abnormal distance due to widening of the linea alba, which causes the abdominal
content to bulge. The rectus fascia is intact, and the condition should therefore not be confused with a ventral hernia. Rectus diastasis can be congenital but is most commonly
acquired during pregnancies and/or larger weight gain causing laxity of linea alba.

ARD presents with visible bulging on exertion or when the intrabdominal pressure is increased (Valsalva maneuvers, coughing, straining, etc.) DRA often develops around the
umbilicus but can also occur anywhere between the xiphoid and the pubic bone.

It is a common condition in pregnancy with an incidence as high as 66% in the third trimester. In 30–60% cases it persists post-partum. Not all patients with abdominal rectus
diastasis are symptomatic. Symptoms, if present, include pain and discomfort in the abdomen, musculoskeletal problems like pelvic instability and lumbar back pain, and
urogynecological symptoms such as urinary incontinence, fecal incontinence, and pelvic organ prolapse. In addition, patients with rectus diastasis also experience lower perception
of body image, lower body satisfaction, and lower quality of life compared with the general population.

Patients with symptoms should receive treatment. Although the efficacy has not been studied adequately, treatment starts with physiotherapy of abdominal wall muscles.

If a course of months of physiotherapy fails to improve the condition surgical repair comes next. The procedure can be performed via classic abdominoplasty (option E) or
endoscopic surgery and stabilization of the defect (option D).

Hernioplasty and mesh insertion (option C) is used for hernias. DRA is not a true hernia.

An abdominal (hernia) belt or truss (option B) is used to keep the content of a hernia inside the abdominal wall y application of pressure over the defect. They are used to relieve the
symptoms of hernia, bridge the surgery, or provide additional support during recovery from the surgery. They are not used as a treatment modality of DRA.

References

• Frontiers in surgery – Treatment Options for Abdominal Rectus Diastasis

Last updated:
Time spent: QID:1704
2023-2-12

590 of 1943
Brad is 55 years old and has presented to the Emergency Department with complaint of sudden onset abdominal pain. The pain was initially felt in the center of the abdomen but
progressed to become generalized. He denies other symptoms including diarrhea, rectal bleeding, and nausea and vomiting. On physical examination, he has blood pressure of
145/95 mmHg, pulse rate of 120 bpm and irregular, respiratory rate of 18 breaths per minute, and temperature of 37.4°C. Abdominal examination, reveals mild generalized
tenderness with no rebound, guarding, or rigidity. The rest of the exam in unremarkable. Which one of the following is most likely to assist to help with a diagnosis?

A. Abdominal ultrasound scan.

B. Abdominal CT scan.

C. Plain abdominal film.

D. Electrocardiogram (ECG).

E. Colonoscopy.

Incorrect. Correct answer is B


45% answered correctly

Explanation:

Correct Answer Is B

The scenario represents a case of acute-onset abdominal pain with minimal abdominal exam findings but the presence of a rapid irregular pulse suggestive of atrial fibrillation. This
constellation of symptoms in a patient over the age of 50 years, especially if male, suggests acute mesenteric ischemia as a likely diagnosis.

Acute mesenteric ischemia (AMI) is interruption of intestinal blood flow by embolism, thrombosis, or a low-flow state. It leads to mediator release, inflammation, and ultimately
intestinal infarction, bowel perforation and peritonitis.

The early hallmark of mesenteric ischemia is severe pain which is surprisingly out of proportion to abdominal physical findings. The abdomen is often soft, with mild or no
tenderness at early stages. Later, as necrosis develops, signs of peritonitis appear, with marked abdominal tenderness, guarding, rigidity, and absent bowel sounds. The stool may
become heme positive. The usual signs of shock develop eventually and frequently result is high mortality.

Cited causes for AMI are arterial embolus, atherosclerosis, non-occlusive ischemia such as in hear failure, shock, etc. In some patients, no cause is identified. AF is infamous cause
of arterial embolus and a classic risk factor for AMI.

Early diagnosis of mesenteric ischemia is particularly important because mortality increases significantly once intestinal infarction sets in. Mesenteric ischemia must be considered
in any patient > 50 with known risk factors or predisposing conditions who develops sudden, severe abdominal pain (AF in this scenario).

Patients with clear peritoneal signs should proceed directly to the operating room for both diagnosis and treatment. For others, selective mesenteric angiography or CT scan with
angiography is the diagnostic procedure of choice. Other imaging studies and serum markers can show abnormalities but lack sensitivity and specificity early in the course of the
disease when diagnosis is most critical. CT scanning has also the added benefit of assessment of other abdominal pathologies.

Plain abdominal films (option C) are useful mainly in ruling out other causes of pain (e.g., perforated viscus), although portal venous gas or pneumatosis intestinalis may be seen
late in the disease.

Doppler ultrasonography can sometimes identify arterial occlusion, but sensitivity is low. MRI is very accurate in proximal vascular occlusion, less so in distal vascular occlusion.
Serum markers (e.g., creatine kinase, lactate) rise with necrosis but are nonspecific findings that are seen later.

Abdominal ultrasound scan (option A) is the choice if conditions such as gallbladder disease or acute appendicitis suggested based on the presentation.

While an electrocardiogram (ECG) (option D) should be in the initial workup of this patient to further assess the rapid irregular rhythm as well as cardiac ischemic pain as a possible
cause of abdominal pain, it is not diagnostic for AMI.

Colonoscopy (option E) is specifically valuable in assessment of intracolonic pathologies such as in gastrointestinal bleeding but is of little if any diagnostic value in this patient with
suspected AMI.

References

• Medscape – Acute Mesenteric Ischemia

• John Murthag General Practice - Abdominal Pain

Last updated:
Time spent: QID:1736
2023-2-12

591 of 1943
Tim, 46 years old, is an established case of AIDS who has presented with a complaint of painful swallowing. The condition started two months ago and has been progressive.
Endoscopy is arranged and reveals linear large ulcers in the middle and distal parts of the esophagus. A biopsy of the abnormal mucosa is remarkable for tissue destruction and the
presence of intranuclear and intracytoplasmic inclusion bodies. Which one of the following would be the most appropriate pharmacological treatment option for him?

A. Proton pump inhibitors (PPIs).

B. H2-receptor antagonists.

C. Swallowing inhaled corticosteroids.

D. Ganciclovir.

E. Acyclovir.

Incorrect. Correct answer is D


45% answered correctly

Explanation:

Correct Answer Is D

Individuals with HIV infection are at an increased risk for esophagitis caused by cytomegalovirus (CMV) as an AIDS-defining illness when the CD4+ lymphocyte counts decline below
100 cells/µL.

Such as in any other type of esophagitis, CMV esophagitis presents with odynophagia (pain on swallowing) as the main symptom.

The most constant endoscopic finding in CMV esophagitis is mucosal ulceration. The ulcers may be single or multiple, deep, or shallow, and could be several centimeters or more in
diameter. The ulcers mostly affect the middle and distal parts of the esophagus.

Histopathologic findings include:

Basophilic inclusion bodies in the nucleus and cytoplasm (intranuclear and intracytoplasmic inclusion bodies)
Arrogates of macrophages

The treatment of choice for CMV esophagitis is ganciclovir or valganciclovir, usually in intravenous form. Foscarnet is used as an alternative if resistance occurs. Acyclovir (option
E) is not potent enough for the treatment of CMV esophagitis.

Combination therapy with IV ganciclovir and IV foscarnet is used when monotherapy fails; however, this regimen is associated with significant toxicity. Longer-term oral ganciclovir
is considered for patients who require long-term suppressive treatment, and patients with HIV infection with CD4+ lymphocyte counts of less than 50 cells/µL.

PPIs (option A) and H2-receptor antagonists (option B) (e.g., ranitidine) are used for reflux disease, peptic ulcer disease, or gastritis none of which is the case here.

Swallowing inhaled corticosteroids (option C) is the treatment of choice, after a trial of PPIs, for eosinophilic esophagitis. The clinical picture does not fit eosinophilic esophagitis.

References

• Medscape - Cytomegalovirus Esophagitis

• NIH - Cytomegalovirus Esophagitis

Last updated:
Time spent: QID:1752
2023-2-12

592 of 1943
A 46-year-old man presents with complaint of neck lump that appears and disappears every few weeks. Apart from the lump, there is no other symptom. On examination, a lump is noted in the right anterior triangle of the neck. The lump is rubbery in
consistency and non-tender. The rest of the exam is normal. Which one of the following options could be the most likely diagnosis?

A. Acute lymphoblastic leukemia (ALL).

B. Acute myeloid leukemia (CML).

C. Hodgkin lymphoma (HL).

D. Follicular lymphoma.

E. Chronic lymphoblastic leukaemia (CLL).

Incorrect. Correct answer is D


45% answered correctly

Explanation:

Correct Answer Is D

The waxing and waning nature of the lump, that is very likely to be an enlarged lymph node, is most consistent with follicular lymphoma.

Follicular lymphoma (FL, previously called follicle center lymphoma) is a heterogenous clinicopathologic entity that includes tumors derived from germinal center B cells. FL is the second most common subtype of non-Hodgkin lymphoma (NHL).

Most patients FL present with painless peripheral adenopathy in the cervical, axillary, inguinal, and/or femoral regions. The adenopathy commonly waxes and wanes spontaneously, but does not altogether disappear. While hilar and mediastinal
nodes are often involved, large mediastinal masses are rare.

Some patients present with relatively asymptomatic large abdominal masses with or without evidence of gastrointestinal and/or urinary tract obstruction. Staging studies usually demonstrate widely disseminated disease with overt involvement of
the following extralymphatic sites:

Spleen (40%)
Liver (50%)
Bone marrow (60-80%)

Despite the presence of widespread disease at diagnosis, most patients are asymptomatic other than for lymph node enlargement. Only approximately 20% presents with B symptoms (i.e. fevers, night sweats, or unintentional weight loss). There are
no characteristic laboratory abnormalities specifically associated with FL and, despite the large tumor burden, fewer than 25% of patients present with an increased serum lactate dehydrogenase (LDH) or cytopenias in the peripheral blood.

References

• http://www.uptodate.com/contents/clinical-manifest

Last updated:
Time spent: QID:778 2023-2-12

593 of 1943
Joseph, 5 years old, is brought to the emergency department by his mother. About one hour ago and while running at home, he hits his face against the coffee table and gets a nose bleed. The mother tried to stop the bleeding by compressing the
nose and applying ice, to no avail. On physical examination, the child looks otherwise quite healthy with no remarkable finding. A full blood exam (FBE) is inconclusive. Which one of the following could be the most likely cause of this persistent
bleeding?

A. Immune thrombocytopenic purpura (ITP).

B. Von Willebrand disease.

C. Hemophilia.

D. Leukemia.

E. Factor V Leiden mutation.

Incorrect. Correct answer is B


45% answered correctly

Explanation:

Correct Answer Is B

The Scenario describes a healthy child with a platelet type of bleeding. In bleeding disorders with faulty platelet or platelet-mediated coagulation, the bleeding tends to occur immediately after insult and is superficial.

In patients with factor deficiency (factor type of bleeding) the bleeding is deep (e.g. into muscles and joints) and delayed, because initially platelets will establish homeostasis.

Von Willebrand disease (VWD) is an inherited disease with many different types (22 types) and clinical pictures. Almost all types cause mild bleeding problems with excellent prognosis. The most common types often have an autosomal dominant
inheritance.

This disease is very common (1 in 100 population) and is the most common inherited bleeding disorder.

Von Willebrand factor is a circulating factor that is attached to factor VIII. This factor by gluing platelets together and to the vascular lining plays the earliest role in coagulation.

Because the clinical disease can be very mild, most cases will never be diagnosed, but if symptomatic, the symptoms can include:

Easy bruising
Mucosal bleeding (e.g. epistaxis, menorrhagea, gastrointestinal bleeding, etc)
No history of haemarthroses or intramuscular hematomas (except for type 3 which is very rare but can have musculoskeletal manifestations)
Prolonged bleeding after trauma or surgery

The clinical picture and a normal platelet count suggests VWD as the most likely diagnosis.

(Option A) Immune thrombocytopenic puprpura (ITP), as the name implies, should have low platelet count as a feature, which is not present in this patient.

(Option C) Hemophilia presents with factor type of bleeding (deep and delayed).

(Option D) With isolated thrombocytopenia and an otherwise healthy child leukemia is very unlikely if possible at all.

(Option E) Factor V Leiden mutation presents with hyper-coagulability not bleeding disorders.

594 of 1943
A 45-year-old woman is evaluated as a part of routine health insurance medical examination. The past medical history is significant for a gastric bypass surgery 12 years ago. Her blood test report is remarkable for hemoglobin of 115 g/L (normal
130-160g/L), an MCV of 110 fL (normal 80-100 fL) and a platelet count of 110000/mm3 (normal 150000-400000/mm3). The reticulocyte count is 0.5% (normal in adults 0.5-1.5%). Which one of the following is the most appropriate next step in
management?

A. Colonoscopy.

B. Thyroid stimulation hormone (TSH) level.

C. Serum vitamin B12 level.

D. Iron studies.

E. Bone marrow biopsy.

Incorrect. Correct answer is C


45% answered correctly

Explanation:

Correct Answer Is C

The laboratory findings of macrocytic anemia (MCV>100) and thrombocytopenia in association with the history of gastric bypass surgery makes pernicious anemia the most likely diagnosis, and serum vitamin B12 level for confirmation the best next
step in management.

Pernicious anemia is a result of significant decrease in intrinsic factor, a glycoprotein secreted by parietal cells in the gastric mucosa, which is necessary for the absorption of vitamin B12 in the ileum. Antibodies against parietal cell in autoimmune
diseases or gastric bypass surgery are common causes of pernicious anemia. It presents with macrocytic anemia, low platelet and normal reticulocyte count. A strict vegetarian diet is another cause.

(Option A) Colonoscopy is the best diagnostic tool when approaching to patients with unexplained iron deficiency anemia because GI tract is a main cause of occult bleeding in such patients. This patient, however, has macrocytic anemia. Iron
deficiency presents with microcytic anemia.

(Option B) Although hypothyroidism is one of the differential diagnosis of macrocytic anemia, but the history and the low platelet count makes it less likely as the etiology of the anemia.

(Option D) Iron study is not the most appropriate step as the anemia is not typical of iron deficiency which presents with microcytosis.

(Option E) Bone marrow biopsy is indicated where no cause for the anemia can be found or is the cause is suspected to originate from bone marrow problems. It is not indicated here, at least for now.

References

• Medscape - Pernicious Anemia

• UpToDate - Treatment of vitamin B12 and folate deficiencies

Last updated:
Time spent: QID:780 2023-2-12

595 of 1943
A 54-year-old man from India presents to your clinic with tiredness and fatigue for the past 12 months. He is a strict vegatarian. He does not consume alcohol or smoke. He denies abdominal symptoms. On examination, he is very pale. The rest of
the examination is unremarkable. Blood test show a low hemoglobin (90 g/L) and an MCV of 130 fL (normal 76-96 fL). Which one of the following is the most likely cause of his presentation?

A. Folic acid deficiency.

B. Thalassemia.

C. Vitamin B12 deficiency.

D. Lead poisoning.

E. Iron deficiency.

Incorrect. Correct answer is C


45% answered correctly

Explanation:

Correct Answer Is C

With a hemoglobin of 9 g/L and n MCV of 130 fL, this patient has macrocytic anemia. Macrocytic anemia is most commonly seen in B12 and B9 (folate) deficiency. This patient is a strict vegetarian. Plants are poor in B12; therefore, B12 deficiency
and consequent megaloblastic macrocytic anemia best explains his anemia.

Vitamin B12 deficiency can increase serum homocysteine that is an amino acid implicated as a strong risk factor for cardiovascular diseases. Neurological problems are other complications.

Major risk factors for vitamin B12 deficiency include:

Age more than 65 years


Gastric bypass surgery or gastric resection (intrinsic factor deficiency)
Terminal ileum disease (Crohn’s disease)
Metformin use
Strict vegetarian diet

Folic acid (option A) deficiency can also cause macrocytic anemia, but vegetarians receive adequate amounts of it through their diet. So, folic acid deficiency is less likely to be the cause of anemia in this man.

Thalassemia (option B), lead poisoning (option D) and iron deficiency (option E) present with microcytic anemia and cannot explain the macrocytosis in this patient.

References

• Medscape - Pernicious Anemia

• UpToDate - Treatment of vitamin B12 and folate deficiencies

Last updated:
Time spent: QID:781 2023-2-12

596 of 1943
A 25-year-old woman presents with a history of recurrent nose bleeds and easy bruising. Investigations confirms the diagnosis of acute idiopathic (immune) thrombocytopenia purpura (ITP). Her hemorrhage tendency is best initially treated with
which one of the following?

A. Gamma globulin infusion.

B. Platelet transfusion.

C. Plasma infusion.

D. Corticosteroids.

E. Azathioprine.

Incorrect. Correct answer is D


45% answered correctly

Explanation:

Correct Answer Is D

Initial therapy for bleeding tendency in ITP in adults is with corticosteroids. Corticosteroids are thought to reduce the clearance of platelets from the circulation.

(Option A) Intravenous gamma globulin infusion is indicated if there is inadequate response to high-dose corticosteroids.

(Option B) Since platelets are rapidly cleared from the blood in ITP, platelet transfusion is of little benefit in long-term management of the disease. However, it may be considered for life-threatening significant bleeding or prior to emergency
splenectomy.

((Opion E) Azathioprine is not used as initial therapy, but may be considered as a corticosteroid-sparing agent for long-term management.

(Option C) Plasma infusion has no role in management of ITP.

References

• http://www.ebmt.org/Contents/Resources/Library/Res

• http://www.rch.org.au/clinicalguide/guideline_inde

Last updated:
Time spent: QID:782 2023-2-12

597 of 1943
A patient is booked for splenectomy due to chronic idiopathic thrombocytopenic purpura. Which one of the following is the most appropriate management regarding platelet transfusion?

A. Give 6 units of platelets and 6 units of FFP the night before the surgery.

B. Give 10 units of platelets the night before surgery.

C. Give 6 units of platelets the night before surgery.

D. Give 6 units of platelets after clamping the splenic artery.

E. Give 6 units of platelets the night and another 6 units in the morning before surgery.

Incorrect. Correct answer is D


45% answered correctly

Explanation:

Correct Answer Is D

The problem in ITP is that platelets are cleared from the blood by the spleen; therefore giving platelets while the spleen is in place is futile and is reserved for occasions when bleeding is life-threatening. On these occasions, platelets are considered
as a short-lived management until more effective measures could be taken.

As there is no comment regarding the severe bleeding, the platelets should be given to the patient for post-op homeostasis once the splenic artery is clamped and the spleen is no more able to sequestrate platelets.

References

• http://emedicine.medscape.com/article/1829892-over

Last updated:
Time spent: QID:783 2023-2-12

598 of 1943
Which one of the following is the best test to diagnose Fanconi’s anemia?

A. Bone marrow biopsy.

B. Full blood count.

C. Chromosome fragility test.

D. Genetic testing.

E. Presence of neutropenia.

Incorrect. Correct answer is C


45% answered correctly

Explanation:

Correct Answer Is C

Fanconi anemia is the most frequently reported of the rare inherited bone marrow failure syndromes. There has been only 2000 cases reported in the medical literature. Fanconi anemia is the most common inherited cause of aplastic anemia. The
transmission mode is autosomal recessive.

Fanconi anemia is different from Fanconi syndrome which is also a rare hereditary kidney disease. The pathophysiology of the disease is a faulty mechanism in DNA repair. In Fanconi anemia, chromosomes are susceptible to DNA cross-linking
agents.

About 75% of patients with Fanconi anemia have birth defects, such as skin problems (e.g. café au lait spots) (>50%), short stature (50%), thumb or thumb and radial anomalies (40%), abnormal male gonads (30%), microcephaly (25%), eye anomalies
(20%), structural renal defects (20%), low birth weight (10%), developmental delay (10%), and abnormal ears or hearing (10%).

Once Fanconi anemia is suspected, the patient should be referred to a hematologist and/or geneticist, who can arrange for diagnostic testing. The laboratory should be accredited and certified to perform Fancoi anemia, and should have evaluated
many patients with and without FA because evaluating a large number of patients enables a laboratory to validate its Fanconi anemia testing procedures, and to establish ranges for normal and abnormal test results.

The recommended testing procedures includes:

1. Chromosome breakage tests also termed chromosome fragility test


2. Mutation analyses
3. Bone marrow chromosome analyses

Of these three set of tests, a chromosome fragility test is the first and most important test to consider and perform. In this test, DNA cross-linking agents are used on cells cultured from the patient’s blood sample. Normal cells can correct most of
the chromosomal damage caused by the DNA cross-linking agents, whereas cells from patients with Fanconi anemia A typically show multiple chromosomal breaks and rearrangements.

References

• Fanconi Anemia: Guidelines for Diagnosis and Management

• Medscape - Fanconi Anemia

Last updated:
Time spent: QID:784 2023-2-12

599 of 1943
A 6-year-old girl with sickle cell disease is brought to your practice for routine follow-up. She is completely asymptomatic and has never had a sickle cell crisis. She takes no medications. Physical examination is unremarkable. Which one of the
following complications is more likely to occur earliest in this patient?

A. Splenic infarction.

B. Bone infarction.

C. Splenic sequestration.

D. Stroke.

E. Acute coronary ischemia.

Incorrect. Correct answer is B


45% answered correctly

Explanation:

Correct Answer Is B

Vaso-occlusive phenomena and hemolysis are the clinical hallmarks of sickle cell disease (SCD) that is an inherited disorder due to homozygosity for the abnormal hemoglobin S (HBs). Vaso-occlusion results in recurrent painful episodes and a
variety of serious organ system complications that can lead to lifelong disabilities and early death. Clinical signs and symptoms typically develop at an early age.

Bone infarction is the most common earliest presenting symptoms. This often presents as acute pain in the hands and/or feet, and is often the initial symptom occurring in 40% of all patients in general and 50% of children who become symptomatic
before the age of two years. The cause is bone marrow infarction. Since pain tends to involve bones with highest bone marrow activity, and because marrow activity changes with age, different bone pain patterns are predictable. During the first 18
months of life, the metatarsal and metacarpals can be involved, presenting as dactylitis or hand-foot syndrome.

An acute episode of pain is the second most common initial presentation and the most common symptom after the age of two years. Splenic sequestration is the third most common presenting symptom, occurring in 20% of patients overall and
one-third of children before the age two years.

Over time, vaso-occlusion can occur in virtually every organ system, accounting for the characteristic acute and chronic multisystem failure associated with the disease.

References

• UpToDate - Overview of the clinical manifestations of sickle cell disease

• Medscape - Sickle Cell Anemia

Last updated:
Time spent: QID:785 2023-2-12

600 of 1943
A 23-year-old woman presents to your practice with complaint of breathlessness while climbing stairs. On blood tests, she has hemoglobin of 7 g/dL. Which one of the following is the most appropriate next step in management?

A. Transfusion of packed cells.

B. Colonoscopy.

C. Small bowel biopsy.

D. Parenteral iron.

E. Fecal occult blood test (FOBT).

Correct
45% answered correctly

Explanation:

Correct Answer Is A

According to guidelines, transfusion of packed red cells is not indicated for those with Hb >10 g/dL. The lower threshold varies from 6 g/dL to 8 g/dL; however, hemoglobin level alone cannot be used to determine the need for packed cell transfusion.

It is recommended that symptomatic anemia be treated with transfusion of packed cells in all patients with Hb<10 g/dL, regardless of the Hb level, provided that the symptoms are severe enough and are clearly related to the anemia rather than the
underlying condition. This is true regardless of the underlying etiology of the anemia.

Colonoscopy (option B), FOBT (option E), or small biopsy (option C) may be indicated as attempts to spot the source or iron loss or deficiency where iron deficiency is the underlying cause of anemia. Before proceeding to such tests, iron deficiency
should be diagnosed by iron studies. Iron replacement (most commonly by oral supplementation) is the treatment of established iron deficiency anemia. Parentral iron (option D) may be used in very severe cases of iron deficiency anemia.

NOTE – According to most references, exertion-related symptoms of anemia are not indications for transfusion of packed cells. In this question, however, the only acceptable option will be transfusion of packed cells, as no other option is
correct regarding this clinical scenario.

References

• MJA - Diagnosis and management of iron deficiency anaemia: a clinical update

• NCBI - Blood Transfusion - Recommendations for the transfusion of red blood cells

Last updated:
Time spent: QID:787 2023-2-12

601 of 1943
A 23-year-old African man presents to your clinic with pallor and dark urine after he took cotrimoxazole for an acute respiratory infection. Blood investigations are significant for hemoglobin of 90 g/L and a reticulocyte count of 5%. Both direct and
indirect Coombs tests are negative. Serum electrophoresis shows type A hemoglobin. There is no family history of such presentation. Which one of the following is the most likely diagnosis?

A. Glucose-6-phosphate dehydrogenase deficiency.

B. Autoimmune hemolytic anemia.

C. Sickle cell anemia.

D. Thalassemia.

E. Hereditary spherocytosis.

Correct
45% answered correctly

Explanation:

Correct Answer Is A

Of the options, glucose-6-phosphate (G6PD) deficiency is the most likely diagnosis.

G6PD deficiency is an X-linked enzymatic defect that can cause hemolysis after oxidative stresses such as acute illness or ingestion of specific drugs.

The most commonly implicated drugs are sulfa drugs (e.g., cotrimoxazole), primiquine, dapsone, quinidine and nitrofurantoin. The most common type of oxidant stress, however, is infections not drugs.

Type-A G6PD deficiency is the milder form and is more common among black people, whereas type B is mostly seen in patients of north Mediterranean origin. Type B has a more severe presentation.

The usual presentation is similar to all hemolytic anemias i.e. low hemoglobin, high LDH, decreased haptoglobin, elevated bilirubin (mostly indirect) and increased reticulocyte count in the presence of a normal MCV. An elevated reticulocyte count is
a criterion for hemolysis but is not specific for it.

(Option B) Negative Coombs test rules out the possibility of autoimmune hemolytic anemia.

Options C and D) With normal hemoglobin (hemoglobin A, which includes two normal α chains and two normal β chains) on serum electrophoresis, sickle cell disease and thalassemia are excluded.

(Option E) With a negative family history of hereditary spherocytosis, this disease is less likely to be the cause of hemolysis in this patient. Another clue against such diagnosis is that hereditary spherocytosis manifestation occur much earlier in life.

References

• UpToDate - Diagnosis and management of glucose-6-phosphate dehydrogenase (G6PD) deficiency

• Medscape - Glucose-6-Phosphate Dehydrogenase (G6PD) Deficiency

Last updated:
Time spent: QID:797 2023-2-12

602 of 1943
A 27-year-old man presents with sudden onset of jaundice, pallor, and dark urine three days after taking of primaquine for malaria. On blood tests, hemoglobin is 52 g/L (120-160g/L) and unconjugated bilirubin is elevated. Blood film shows 'bite
cells'. Which one of the following is the most likely diagnosis?

A. Iron deficiency anemia.

B. Glucose-6-phosphate dehydrogenase deficiency.

C. Anemia due to blood loss.

D. Acute hepatitis.

E. Chronic renal failure.

Incorrect. Correct answer is B


45% answered correctly

Explanation:

Correct Answer Is B

Development of jaundice after use of primaquine is suggestive of glucose-6-phosphate dehydrogenase (G6PD) deficiency. The typical presentation is sudden onset of jaundice, pallor, and dark urine, with or without abdominal and back pain. This is
associated with an abrupt fall in the hemoglobin concentration of 30-40 g/L during which time the peripheral blood smear reveals red cell fragments, microspherocytes, and eccentrocytes or "bite" cells.

Special stains show Heinz bodies, which are collections of denatured globin chains often attached to the red cell membrane.

Episodes of acute hemolytic anemia are triggered by oxidative stress. Infections are the most common triggering factor.

Iron deficiency anemia, anemia due to blood loss and anemia of chronic renal failure are not associated with elevated unconjugated bilirubin. This fining is a feature of hemolytic anemia.

References

• UpToDate - Diagnosis and management of glucose-6-phosphate dehydrogenase (G6PD) deficiency

• Medscape - Glucose-6-Phosphate Dehydrogenase (G6PD) Deficiency

Last updated:
Time spent: QID:798 2023-2-12

603 of 1943
A 32-year-old woman, accompanied by his brother, presents with complaint that every time she gets a cut, it takes a long for the bleeding to stop. She also mentions that she is distressed with her heavy periods, for which she has to use 10 pads a
day. Her brother mentions that she has been like that since he can remember. She is otherwise healthy with no significant finding on physical examination. Which one of the following is the most likely diagnosis?

A. Hemophilia A.

B. Stuart disease.

C. Von Willebrand disease.

D. Factor IX deficiency.

E. Immune thrombocytopenic purpura (ITP).

Incorrect. Correct answer is C


45% answered correctly

Explanation:

Correct Answer Is C

Long-standing history of prolonged bleeding after trauma and history of menorrhagea in an otherwise healthy woman is highly suggestive of Von Willebrand disease as the most likely cause.

Von Willebrand disease (VWD) is an inherited disease with many different types (22 types) and clinical pictures. Almost all types cause mild bleeding problems with excellent prognosis. The most common types often have an autosomal dominant
inheritance.

This disease is very common (1 in 100 population) and is the most common inherited bleeding disorder.

Von Willebrand factor is a circulating factor that is attached to factor VIII. This factor by gluing platelets together and to the vascular lining plays the earliest role in coagulation.

Because the clinical disease can be very mild, most cases will never be diagnosed, but if symptomatic, the symptoms can include:

Easy bruising
Mucosal bleeding (e.g. epistaxis, menorrhagoea, gastrointestinal bleeding, etc)
No history of hemarthroses or intramuscular hematomas (except for type 3 that is very rare but can have musculoskeletal manifestations)
Prolonged bleeding after trauma or surgery

(Option A) Hemophilia A presents with factor type of bleeding (deep and delayed). As both haemophilia A and B are X-linked, women can all be carriers. Homozygous females never born.

(Option B) Stuart disease or factor X (Stuart-Prower factor) deficiency is one of the world’s rarest factor deficiencies. Factor X is a vitamin K-dependent factor that serves as the first enzyme in the common pathway of thrombus formation. It can
cause both platelet and factor types of bleeding.

(Option D) Like hemophilia A, factor IX deficiency (haemophilia B) is never seen in an alive woman.

(Option E) ITP presents with petechiae and bruising rather than prolonged bleeding. Furthermore, compared to WVD, chronic ITP in adults is less common.

604 of 1943
A 32-year-old African male presents to the Emergency Department with fatigue, jaundice, dark-colored urine and acute decrease in hemoglobin for the past four days. Liver function tests are normal, except for an elevated unconjugated bilirubin level.
Which one of the laboratory results would be most consistent with the diagnosis of hemolysis due to glucose-6-phosphate dehydrogenase deficiency?

A. Spherocytosis.

B. Schistocytosis.

C. Positive Coomb’s test.

D. Heinz bodies.

E. Elliptocytosis.

Incorrect. Correct answer is D


45% answered correctly

Explanation:

Correct Answer Is D

Glucose-6-phosphate dehydrogenase (G6PD) deficiency is the most common enzyme deficiency in human, affecting 400 million people worldwide, with a high prevalence in people of African, Asian, and Mediterranean descent. The condition is
inherited as an X-linked recessive disorder. It is polymorphic with more than 300 variants.

G6PD deficiency can present as neonatal hyperbilirubinemia. People with this disorder can experience episodes of brisk hemolysis in response to oxidative stresses or, less commonly, have chronic hemolysis. However, many individuals with G6PD
deficiency are asymptomatic. G6PD deficiency confers partial protection against malaria.

The diagnosis of G6PD deficiency can be made on the basis of a well-documented history, evidence of hemolysis, a peripheral-blood smear showing Heinz bodies (erythrocytes with denatured hemoglobin) and ‘bite cells’. Measurement of G6PD
activity while the patient is in remission is the gold-standard diagnostic test for G6PD deficiency.

G6PD deficiency should be suspected in all patients with non-immune acute hemolysis and no spherocytosis on laboratory testing.

(Option A) Spherocytes are characteristic feature in hereditary spherocytosis.

(Option B) Schistocytes are seen in microangiopathic hemolytic anemia (MAHA) associated with hemolytic uremic syndrome and thrombotic thrombocytopenic purpura (TTP).

(Option C) Positive Coombs test is the characteristic feature of immune-mediates hemolytic anemia.

(Option E) Elliptocytosis, also known and ovalocytosis, is a feature of hereditary elliptocytosis – an inherited disorder in which an abnormally large number of patient’s red blood cells are elliptical rather than the typical biconcave disc-shaped.

References

• UpToDate - Diagnosis and management of glucose-6-phosphate dehydrogenase (G6PD) deficiency

• Medscape - Glucose-6-Phosphate Dehydrogenase (G6PD) Deficiency

Last updated:
Time spent: QID:799 2023-2-12

605 of 1943
Which one of the following is the most common cause of anemia in geriatric population?

A. Iron deficiency anemia from blood loss.

B. Nutritional anemia from vitamin B12 deficiency.

C. Anemia of chronic disease.

D. Hemolytic anemia.

E. Myelodysplastic anemia.

Incorrect. Correct answer is C


45% answered correctly

Explanation:

Correct Answer Is C

Chronic diseases are the most common causes of anemia in the geriatric population accounting for 30-45% of cases. Iron deficiency is the second most common cause.

Causes of anemia in geriatric population and the percentage of each are summarized in the following table:

Cause Percentage of cases


Chronic diseases 30-45%
Iron deficiency 15-30%
Post-hemorrhagic 5-10%
Vitamin B12/B9 deficiency 5-10%
Chronic leukemia and lymphoma 5%
Myelodysplastic syndromes 5%
No identifiable cause 15-25%

References

• AAFP - Anemia in the Elderly

• Evaluation and Management of Anemia in the Elderly

Last updated:
Time spent: QID:800 2023-2-12

606 of 1943
A 70-year-old man from a low-level-of-care nursing home is brought to the hospital after he had a fall 3 hours ago. He is on warfarin for atrial fibrillation (AF). Laboratory studies show that he has an INR of 4.9. A CT scan of the head is ordered which
is normal. Other investigations are unremarkable. Which one of the following is the next best step in management?

A. Stop warfarin.

B. Skip one dose of warfarin.

C. Give fresh frozen plasma (FFP).

D. Give vitamin K.

E. Repeat CT scan of the head in 2 days.

Incorrect. Correct answer is B


45% answered correctly

Explanation:

Correct Answer Is B

The therapeutic range of warfarin for most patients varies between 2 to 3.5 (2-3, or occasionally 2.5-3.5). Increased INR beyond therapeutic levels are associated with higher risk of bleeding.

For patients with an INR above the therapeutic range but less than 5, who are not bleeding, skipping the next dose of warfarin and resuming lower doses once the INR is within the therapeutic range is the recommended management.

Ceasing warfarin, FFP, vitamin K (intravenously) and Prothrombinex are used in situations where there is active bleeding or the risk of bleeding is high.

References

• https://www.health.qld.gov.au/publications/clinica

• http://www.uptodate.com/contents/management-of-war

• https://www.mja.com.au/journal/2013/198/4/update-c

Last updated:
Time spent: QID:827 2023-2-12

607 of 1943
A group of college students are admitted to theemergency department with bruises over their skin, dark-colored urine and shock one week after they were back from camping in a forest. Which one of the following is most likely to be the cause of
this presentation?

A. Giardia lamblia.

B. Escherichia coli.

C. Disseminated intravascular coagulopathy.

D. Staphylococcus aureus.

E. Ross River fever.

Incorrect. Correct answer is B


45% answered correctly

Explanation:

Correct Answer Is B

The bruises over the skin (petechial rash), kidney involvement, and mental status alteration are consistent with thrombotic thrombocytopenic purpura (TTP) as the most likely diagnosis.

TTP is a rare blood disorder characterized by clotting in small blood vessels resulting in a low platelet count. In its full blown form, the disease consists of the pentad of (1) microangiopathic hemolytic anemia, (2) thrombocytopenic purpura, (3)
neurological abnormalities, (4) fever (non-infectious), and (5) renal disease.

TTP can affect any organ system, but involvement of the peripheral blood, the central nervous system, and the kidneys causes the clinical manifestation.

Neurological manifestations include alteration in mental status, seizures, hemiplegia, paresthesias, visual disturbances, and aphasia. Patients may notice dark urine from hemoglobinuria. Severe bleeding from thrombocytopenia is unusual, but
petechiae are common. Fever may occur in 50% of patients

E-coli O157:H7, E-coli O104:H4 and some other Shiga toxin-producing bacteria are the most common cause in children with TTP and hemolytic uraemic syndrome (HUS). In adults, many cases are idiopathic, but the same bacteria can cause these
diseases as well. In adults immunosuppression and pregnancy can be other possible causes.

Since a group of persons are affected simultaneously, it is very likely that E-coli has been the etiology of the TTP.

(Option A) Giardia lamblia infection manifest with a different clinical picture including diarrhoea (non-bloody), flatulence, and crampy abdominal pain.

(Option C) DIC can present with similar picture (bleeding, shock, renal dysfunction, hepatic dysfunction, and central nervous system problems); however, the coincidence makes E-coli infection a more likely diagnosis.

(Option D) Staphylococcus aureus causes acute gastroenteritis with vomiting (more prominent) and diarrhea. Petechiae, neurological manifestations and shock are not present.

(Option E) Ross River fever is a mosquito-borne disease presenting with fever, rash, and polyarhtralgia.

References

• http://www.merckmanuals.com/professional/hematolog

• http://emedicine.medscape.com/article/206598-clini

Last updated:
Time spent: QID:918 2023-2-12

608 of 1943
A 27-year-old man presents to the emergency department with pallor and jaundice. On physical examination, the spleen is palpated two cm below the costal margin. A blood exam reveals a reticulocyte count of 18% (normal 0-2%) and
microspherocytosis. A Coomb's test is positive. Which one of the following is the most likely cause of this clinical picture?

A. Acquired autoimmune hemolytic disease.

B. Hereditary spherocytosis.

C. Congenital autoimmune hemolytic disease.

D. Myeloproliferative disorder.

E. Glucose-6-phosphate dehydrogenase (G6PD) deficiency.

Correct
45% answered correctly

Explanation:

Correct Answer Is A

The pallor and jaundice indicate the hemolytic nature of the process. With a palpable spleen and a high reticulocyte count, the process has been chronic and probably long-standing. With the positive Coombs test, the autoimmune nature of the
condition is ensured with high certainty; therefore, of the options acquired autoimmune hemolytic disease is the most likely cause to consider. Microspherocytosis can be seen in cases of autoimmune hemolytic disease.

(Option B) Hereditary spherocytosis leads to episodes of jaundice and pallor caused by hemolytic anemia, and characteristic spherocytosis on peripheral smear; however, Coombs test is negative because the hemolysis is not autoimmune in nature.

Option C) Congenital autoimmune hemolytic disease would have presented much earlier in life.

(Option D) Myeloproliferative disorders are not associated with hemolysis.

(Option E) G6PD would have demonstrated its clinical features earlier in life. The condition is not autoimmune and Coombs test would be negative.

References

• Medscape - Hemolytic Anemia

Last updated:
Time spent: QID:921 2023-2-12

609 of 1943
An 80-year-old man is planned for emergency surgery due to a strangulated direct hernia. He has been on clopidogrel after he sustained an acute myocardial infarction 10 months ago. Which one of the following is the next best in management prior
to the surgery?

A. Give platelets and proceed to the surgery.

B. Give DDAVP and proceed to the surgery.

C. Give cryoprecipitate and proceed to the surgery.

D. Stop clopidogrel and proceed to the surgery.

E. Give vitamin K intravenously.

Correct
45% answered correctly

Explanation:

Correct Answer Is A

Strangulated hernias are genuine emergencies requiring immediate surgery. Failing to do so can lead to bowel perforation, peritonitis and death. This patient, however, is on clopidogrel and has an increased risk of intra- or post-operative bleeding due
to effect of clopidogrel on platelet function. Since abdominal surgery is associated with marked risk of bleeding in this patient, platelets should be given prior to the surgery to counteract the effect of clopidogrel.

If this patient had a coronary stent, antiplatelet therapy should have not been ceased. Exceptions to this include patients with coronary stents who are undergoing spinal, intracranial, extraocular TURP or major plastic reconstructive procedures. For
these operations, patients at low risk of stent thrombosis should have their antiplatelet therapy routinely ceased perioperatively.

(Option B) Desmopressin (DDAVP), often in conjunction with fresh frozen plasma, is used for preoperative management of patients with Von-Willebrand disease in whom there is deficiency of factor VIII.

(Option C) Cryoprecipitate replenish coagulation factors with no effect on platelet activity inhibited by clopidogrel.

(Option D) It takes approximately 7 days for effects of antiplatelet drugs to suitably diminish. Stopping clopidogrel now will not decrease the chance of bleeding immediately and is not useful.

(Option E) Vitamin K is used for warfarin reversal and has no effect on platelet function.

TOPIC REVIEW

Coronary stent thrombosis is an uncommon but clinically devastating complication of coronary artery stenting that usually results in significant myocardial infarction or death. Approximately 40% of reported cases have occurred in the context of
non-cardiac surgery (NCS) performed in patients with coronary artery stents, in whom dual antiplatelet therapy or clopidogrel alone has been ceased.

In patients with coronary disease, cessation of aspirin or clopidogrel is associated with an approximate 2-3 fold increase in subsequent death or myocardial infarction. This risk is further elevated in patients with intracoronary stent and is of added
concern because the dramatic consequences of stent occlusion.

There is uncertainty regarding the risks of stent thrombosis in individual patients, and in particular how to balance this risk against that of surgical complications if antiplatelet therapy is continued throughout the perioperative period.

The following are current recommendations regarding antiplatelet agents and non-cardiac surgery:

Elective non-cardiac surgery should be deferred for at least 6 weeks and ideally 3 months following PCI with bare metal stenting.

Elective surgery should be deferred for 12 months following drug eluting stents because of a likely increased risk of death/myocardial infarction/stent thrombosis.

Despite the observation that dual antiplatelet therapy increases the likelihood of bleeding for most surgical procedures, the consequences of bleeding are less significant than those of stent thrombosis.

The risk benefit ratio would favor continuation of aspirin in most patients and dual antiplatelet therapy in many patients with prior coronary artery stenting who are undergoing non-cardiac surgery. Exceptions to this include patients
undergoing spinal, intracranial, extraocular TURP or major plastic reconstructive procedures. For these operations, patients at low risk of stent thrombosis should have their antiplatelet therapy routinely ceased perioperatively.

In patients with coronary artery disease in whom NO stent has been placed, antiplatelets can be stopped 7 days before the surgery.

In the event of emergency, surgery should be proceeded to without cessation of antiplatelet therapy. platelet transfusion might be considered in selected patients in anticipation or occurrence of major bleeding.

In other situations than bare metal stents and drug eluting stents placed within the past 6 weeks and 12 months respectively, antiplatelet medications should be stopped 1 week (7 days) before the procedure.

References

• http://www.csanz.edu.au/wp-content/uploads/2014/12

• http://www.racgp.org.au/afp/2014/december/new-oral

Last updated:
Time spent: QID:938 2023-2-12

610 of 1943
Accompanied by his wife, a 68-year-old man is brought to the emergency department. The wife mentions that her husband woke up with a nagging headache. One hour later, he started to develop slurred speech and weakness of the right arm. She
mentioned that her husband has been on a pill for his cardiac condition for the past 3 months, but she cannot remember the name. Since his pulse seems irregular, you assume that the pill is warfarin and check an INR which comes back 4.5. A non-
contrast CT scan of the head reveals intracranial hemorrhage. Which one of the following is the next best action to take?

A. Give fresh frozen plasma (FFP).

B. Give FFP and vitamin K.

C. Give protamine sulfate.

D. Give vitamin K.

E. Switch to unfractionated heparin.

Incorrect. Correct answer is B


45% answered correctly

Explanation:

Correct Answer Is B

Bleeding is the most common complication of warfarin therapy and is related to the INR value. Although incremental rises in INR increase the risk of bleeding, most intracranial bleedings occurs in patients with an INR in the therapeutic range. Such
events occur in 0.5-1.0% of patients with AF per year.

In the event of major or life-threatening bleeding associated with warfarin use, prothrombin complex concentrate (PCC) with or without FFP is the most appropriate next step in management. FFP in large volumes should be used if PCC is not
available. In major or life-threatening bleedings, 5-10mg of vitamin K should also be given in conjunction with PCC or FFP to sustain the reversal effect.

Cessation of warfarin should be considered in all patients with bleeding; however, cessation of warfarin alone is not going to change the immediate management plan that is reversing its effects immediately.

(Option A) FFP alone immediately reverses the warfarin effect, but vitamin K is aslo necessary to sustain the response after the effect of FFP wears out.

(Option C) Protamine sulfate is the antidote to heparin with no effect on warfarin.

(Option D) Vitamin K is an effective antidote to the anticoagulation effect of warfarin. Despite this, data are lacking to show that its use improves outcome in life-threatening bleeding. The usual dose is 5-10mg administered orally or intravenously.
Intravenous route achieves a more rapid response compared with oral administration, with an onset of action between 6-8 hours. However, both routes achieve a similar correction of the INR by 24 hours. Vitamin K is the treatment of choice if the
goal is to normalize the INR and no immediate counteraction against major organ bleeding or life-threatening bleeding is prompted. Intravenous administration is the preferred method.

(Option E) Switching to heparin does not decrease the risk of bleeding; nor does it counteract warfarin effects.

TOPIC REVIEW

Prothrombin complex concentrate (PCC) comes in two forms. One formulated with three factors (II, IX and X) and the other with four factors (II, VII, IX and X). Advantages of PCC over FFP include rapid reconstitution into a small volume for infusion
over 20–30 minutes, fast onset of action (10-15 minutes), no requirement to check a patient’s blood group, minimal risk of viral transmission due to pathogen reduction and inactivation steps during manufacturing, and reduced risk of other clinical
adverse reactions such as transfusion-associated circulatory overload or transfusion-associated acute lung injury.

Prothrombinex-VF, a three-factor PCC, is the only product currently in routine use in Australia and New Zealand for warfarin reversal. Despite instructions by the Warfarin Reversal Consensus Guidelines published in 2004 recommended that it be
supplemented with FFP, there have been several reports of successful use of Prothrombinex-VF without addition of FFP. Prothrombinex-VF has been used successfully to electively reverse anticoagulation in patients on warfarin therapy with a stable
INR, and achieved the target INR in over 90% of patients. However, since the efficacy of Prothrombinex-VF alone has not been extensively evaluated for patient with major or life-threatening bleeding or INR > 10, current recommendation is that
Prothrombinex-VF is supplemented with FFP for addition of adequate amounts of factor VII to ensure optimal reversal of the anticoagulant effect of warfarin when major or life-threatening bleeding is a concern.

For life-threatening (critical organ) and clinically significant bleeds, the consensus is to use the maximum dose of Prothrombinex-F (with vitamin K1 and FFP) and the maximum amount of FFP when Prothrombinex-VF is unavailable.

Recommendations for managing patients on warfarin therapy with bleeding are summarized in the following table:

Clinical setting Recommendations


1. Stop warfarin therapy AND give:
2. Vitamin K (5-10mg IV) AND
3. Prothrombinex-VF (50IU/kg IV) AND
INR ≥1.5 with life-threatening (critical organ) bleeding 4. FFP (150-300ml)

If Prothrombinex-VF unavailable, give FFP (15ml/kg)


1. Stop warfarin therapy AND give:
2. Vitamin K (5-10mg IV) AND
3. Prothrombinex-VF (35-50IU/kg IV)
INR≥2 with clinically significant bleeding (not life-threatening)

If Prothrombinex-VF unavailable, give FFP (15ml/kg)


Omit warfarin, repeat INR the following day and adjust the warfarin dose to maintain INR within the therapeutic range
Any INR with minor bleeding
If bleeding risk is high or INR>4.5 give vitamin K (1-2mg orally or 0.5-1mg IV)

Recommendations for managing patients on warfarin therapy with a high INR but no bleeding are summarized in the following table:

Clinical setting Recommendations


INR above the therapeutic range but <4.5 and no bleeding Lower or omit the next dose of warfarin

Resume therapy at a lower warfarin dose when INR approaches the therapeutic INR

611 of 1943
If INR is minimally above the therapeutic range (up to 10%) no action is required
Stop warfarin; consider reasons for elevated INR and patient specific factors. Vitamin K is not generally necessary unless the patient is
high risk for bleeding*, in which case:

INR 4.5-10 and no bleeding 1. Consider vitamin K (1-2mg orally or 0.5-1mg IV)
2. Measure INR within 24 hours
3. Resume warfarin at a reduced dose once INR approaches the therapeutic range

Stop warfarin therapy AND

1. Give 3-5mg vitamin K orally or IV


2. Measure INR in 12-24hours; close monitoring of INR daily to second daily over the next week
3. Resume warfarin therapy at a reduced once INR approaches the therapeutic range

INR >10 and no bleeding


If high risk for bleeding*:

1. Give Prothrombinex-VF 35-50IU/kg


2. Measure INR within 12-14hours; close monitoring over the next week
3. Resume warfarin at a reduced dose once INR approaches the therapeutic range

*Risk factors for major bleeding are:

A major bleed within the previous 4 weeks


Surgery within the previous 2 weeks
Platelet count< 50,000/mm3
Known liver disease
Concurrent antiplatelet therapy

NOTE- PCC does contain small amounts of heparin and its use to reverse warfarin anticoagulation should be carefully considered in patients with heparin-induced thrombocytopenia.

References

• https://www.mja.com.au/journal/2013/198/4/update-c

Time spent: QID:946 Last updated:


2023-2-12

612 of 1943
A 67-year-old man in brought to the emergency department with sudden-onset severe headache and confusion. He was diagnosed with atrial fibrillation (AF) 4 months ago, for which he has been on warfarin since then. Blood studies show an INR of
3.5. A CT scan of the head reveals intracerebral hemorrhage. Which one of the following is the most appropriate next step in management?

A. Stop warfarin.

B. Vitamin K.

C. Fresh frozen plasma (FFP).

D. Increase the dose of warfarin.

E. Reduce the dose of warfarin.

Incorrect. Correct answer is C


45% answered correctly

Explanation:

Correct Answer Is C

Bleeding is the most common complication of warfarin therapy and is related to the INR value. Although the risk of hemorrhage is directly related to the INR value, most events of intracranial hemorrhage occur in patients with an INR within the
therapeutic range. Such events occur in 0.5-1.0% of patients with AF per year.

In the event of major or life-threatening bleeding associated with warfarin use, prothrombin complex concentrate (PCC) with or without FFP is the most appropriate next step in management. FFP in large volumes should be used if PCC is not
available.

(Option A) Cessation of warfarin should be considered in all patients with bleeding; however, cessation of warfarin alone is not going to change the immediate manage plan that is reversing its effects immediately.

(Option B) Vitamin K is an effective antidote for the anticoagulation effect of warfarin. Despite this, data are lacking to show that its use improves outcome in life-threatening bleedings. The usual dose is 5-10mg administered orally or intravenously.
Intravenous route achieves a more rapid response compared with oral administration, with an onset of action between 6-8 hours. However, both routes achieve a similar correction of INR by 24 hours. In major or life-threatening bleeds, 5-10mg of
vitamin K should be given in conjunction with PCC or FFP to sustain the reversal effect.

Vitamin K is the treatment of choice for patients on warfarin therapy with bleeding in whom the aim is to normalize the INR (not immediate counteraction against major organ bleeding or life-threatening bleeding), vitamin K given intravenously is the
preferred treatment.

(Option D) Increasing the dose of warfarin worsens the condition and not a correct option. Reducing the dose of warfarin can lead to decreased INR in long-term. It does not counteract the bleeding.

(Option E) While the patient is bleeding, dose reduction of the warfarin is not an appropriate option. Dose reduction is an option for patients with INRs beyond the target but no active bleeding.

TOPIC REVIEW

Prothrombin complex concentrate (PCC) comes in two forms - one formulated with 3 factors (II, IX and X) and the other with 4 factors (II, VII, IX and X). Advantages of PCC over FFP include rapid reconstitution into a small volume for infusion over
20–30 minutes, fast onset of action (10-15 minutes), no requirement to check the patient’s blood group, minimal risk of viral transmission due to pathogen reduction and inactivation steps during manufacturing, and reduced risk of other clinical
adverse reactions such as transfusion-associated circulatory overload or transfusion-associated acute lung injury.

Prothrombinex-VF®, a 3-factor PCC, is the only product currently in routine use in Australia and New Zealand for warfarin reversal. However, since the efficacy of Prothrombinex-VF® alone has not been extensively evaluated for patients with major or
life-threatening bleeding, or INR > 10, it is currently recommended that Prothrombinex-VF® be supplemented with FFP for addition of adequate amounts of factor VII to ensure optimal reversal thrapy when major or life-threatening bleeding is a
concern.

For life-threatening (critical organ) and clinically significant bleedings, the consensus is to use the maximum dose of Prothrombinex-F® (with vitamin K1 and FFP) and the maximum amount of FFP when Prothrombinex-VF is unavailable.

Recommendations for managment of patients on warfarin therapy with increased INR and bleeding are summarized in the following table:

Clinical setting Recommendations


1. Stop warfarin therapy AND give:
2. Vitamin K (5-10mg IV) AND
3. Prothrombinex-VF (50IU/kg IV) AND
INR ≥1.5 with life-threatening (critical organ) bleeding 4. FFP (150-300ml)

If Prothrombinex-VF unavailable, give FFP (15ml/kg)


1. Stop warfarin therapy AND give:
2. Vitamin K (5-10mg IV) AND
3. Prothrombinex-VF (35-50IU/kg IV)
INR≥2 with clinically significant bleeding (not life-threatening)

If Prothrombinex-VF unavailable, give FFP (15ml/kg)


Skip one dose of warfarin, repeat INR the following day and adjust the warfarin dose to maintain INR within the therapeutic range
Any INR with minor bleeding
If bleeding risk is high or INR>4.5 give vitamin K (1-2mg orally or 0.5-1mg IV)

Recommendations for managing patients on warfarin therapy with a high INR but no bleeding are summarized in the following table:

Clinical setting Recommendations


INR above the therapeutic range but <4.5 and no bleeding Decrease or skip the next dose of warfarin

Resume therapy at a lower warfarin dose when INR is within the therapeutic range

613 of 1943
If INR is minimally above the therapeutic range (up to 10%) no action is required
Stop warfarin; consider reasons for elevated INR and patient specific factors. Vitamin K is not generally necessary unless the patient is
high risk for bleeding*, in which case:

INR 4.5-10 and no bleeding 1. Consider vitamin K (1-2mg orally or 0.5-1mg IV)
2. Measure INR within 24 hours
3. Resume warfarin at a reduced dose once INR is within the therapeutic range

Stop warfarin therapy AND

1. Give 3-5mg vitamin K orally or IV


2. Measure INR in 12-24hours; close monitoring of INR daily to second daily over the next week
3. Resume warfarin therapy at a reduced dose once INR is within the therapeutic range

INR >10 and no bleeding


If high risk for bleeding*:

1. Give Prothrombinex-VF 35-50IU/kg


2. Measure INR within 12-14hours; close monitoring over the next week
3. Resume warfarin at a reduced dose once INR is within the therapeutic range

*Risk factors for major bleeding are:

A major bleed within the previous 4 weeks


Surgery within the previous 2 weeks
Platelet count< 50,000/mm3
Known liver disease
Concurrent antiplatelet therapy

NOTE- PCC contains small amounts of heparin and its use to reverse warfarin anticoagulation should be carefully considered in patients with heparin-induced thrombocytopenia.

References

• MJA - An update of consensus guidelines for warfarin reversal

Time spent: QID:947 Last updated:


2023-2-12

614 of 1943
A 73-year-old man presents to the emergency department with fracture of the right femoral neck after he sustained a fall at home. He underwent coronary artery drug eluting stent placement 2 months ago, and has been on clopidogrel since then. A
full blood count (FBC) is normal; however, he has several bruises over his body. He requires surgery for fixation of the fracture. Which one of the following is the most appropriate management of this patient?

A. Stop clopidogrel and proceed with surgery in one week.

B. Do the surgery now.

C. Administer fresh frozen (FFP) plasma and proceed with the surgery.

D. Give platelets and proceed with the surgery.

E. Switch to heparin and perform the surgery in 7 days.

Incorrect. Correct answer is B


45% answered correctly

Explanation:

Correct Answer Is B

Femoral neck fracture requires emergency surgical fixation, because other measures such as traction or rest pose a more significant risk to the patient than does the risk of bleeding associated with antiplatelet therapy. Patients for whom surgery
cannot be deferred should be operated on immediately. Any option suggesting deferral of the surgery is inappropriate. On the other hand, this patient has undergone coronary stenting with drug-eluting stent and requires at least 12 months of
antiplatelet therapy. Cessation of antiplatelet therapy is associated with significantly increased risk of stent thromobosis and grave complications; hence antiplatelet therapy should be continued. He should undergo emergecny surgery while he is on
clopidogrel.

Of note, an Australian retrospective review of 181 patients with proximal femoral fracture demonstrated no significant difference in the amount of bleeding, transfusion requirement, complications rate, or length of stay in 16 patients taking
clopidogrel and in 48 taking aspirin compared to others.

(Options A and E) Deferral of an emergency surgery is incorrect.

(Option C) FFP reverses the effect of heparin and warfarin. It has no effect on inhibited action of platelets.

(Option D) Platelet transfusion might be considered in selected patients in whom the risk of major bleeding clearly outweighs the benefits of counteracting the antiplatelet therapy. However, transfusion of platelets in this patient may result in stent
thrombosis and is not advisable. The only exception is when the risk of bleeding is so high that remarkably outweighs the risk of stent thrombosis.

TOPIC REVIEW

Coronary stent thrombosis is an uncommon but clinically devastating complication of coronary artery stenting that usually results in significant risk of myocardial infarction or death. Approximately 40% of reported cases have occurred in the context
of non-cardiac surgery (NCS) performed in patients with coronary artery stents, in whom dual antiplatelet therapy or clopidogrel alone has been ceased.

In patients with coronary disease, cessation of aspirin or clopidogrel is associated with an approximate 2-3 fold increase in subsequent death or myocardial infarction. This risk is further elevated in patients with intracoronary stent and is of added
concern because the dramatic consequences of stent occlusion.

There is uncertainty regarding the risks of stent thrombosis in individual patients, and in particular how to balance this risk against that of surgical complications if antiplatelet therapy is continued throughout the perioperative period.

The following are current recommendations regarding antiplatelet agents and non-cardiac surgery:

Elective non-cardiac surgery should be deferred for at least 6 weeks and ideally 3 months following PCI with bare metal stenting.

Elective surgery should be deferred for 12 months following drug eluting stent because of an increased risk of death/myocardial infarction or stent thrombosis.

Despite the observation that dual antiplatelet therapy increases the likelihood of bleeding for most surgical procedures, the consequences of bleeding are less significant than those of stent thrombosis.

The risk benefit ratio would favor continuation of aspirin in most patients and dual antiplatelet therapy in many patients with prior coronary artery stenting who are undergoing non-cardiac surgery. Exceptions to this include patients
undergoing spinal, intracranial, extraocular TURP or major plastic reconstructive procedures. For these surgeries, patients at low risk of stent thrombosis should have their antiplatelet therapy routinely ceased perioperatively.

In patients with coronary artery disease in whom no stent has been placed, antiplatelets can be stopped 7 days before the surgery.

In the event of emergency, surgery should be proceeded to without cessation of antiplatelet therapy. platelet transfusion might be considered in selected patients in anticipation or occurrence of major bleeding.

In other situations THAN bare metal stents and drug-eluting stents placed within the past 6 weeks and 12 months respectively, antiplatelet medications should be stopped 1 week (7 days) before the procedure.

References

• http://www.csanz.edu.au/wp-content/uploads/2014/12

• http://www.aafp.org/afp/2010/1215/p1484.html

• http://www.racgp.org.au/afp/2014/december/new-oral

• http://bja.oxfordjournals.org/content/111/suppl_1/

Last updated:
Time spent: QID:948 2023-2-12

615 of 1943
A 66-year-old man presents with altered bowel habits, decreased stool caliber and rectal bleeding in the form of blood covering the stool. Investigations show that he has a colorectal cancer. He is planned for surgical tumor resection. Currently, he is
on warfarin due to atrial fibrillation (AF) and has an INR of 2.5. Which one of the following is the most appropriate option to consider for warfarin reversal prior to the surgery?

A. Proceed with the surgery.

B. Give vitamin K before the surgery.

C. Give fresh frozen plasma (FFP) and proceed with the surgery.

D. Wait for 3 months.

E. Switch to clopidogrel and perform the surgery in one week.

Incorrect. Correct answer is B


45% answered correctly

Explanation:

Correct Answer Is B

This patient, as a candidate for a surgical procedure, is at increased risk of intra- or post-operative bleeding due to warfarin therapy. Resection of a colonic tumor is an elective procedure that allows for planned action for warfarin reversal. In those
surgical candidates with an INR of 2-3, cessation of warfarin 4-5 days before the surgery and administration of vitamin K (3mg, intravenously) the evening before the surgery is the preferred plan.

(Option A) Proceeding with a major surgery without warfarin reversal is associated with a significantly increased risk of bleeding and not appropriate.

(Option C) FFP was the correct answer if this patient required emergency surgery. Prothrombinex-VF (or if unavailable FFP) is used for rapid reversal of warfarin effect (in minutes) for urgent surgical procedures associated with increased risk of
bleeding. Using FFP for warfarin reversal in elective procedures is not appropriate.

(Option D) Deferring the surgery for 3 months is unnecessary and inappropriate because warfarin reversal can be achieved in 5 days, maximum. There is also no other medical condition to preclude surgery in this patient.

(Option E) Clopidogrel is an antiplatelet medication. Switching to clopidogrel not only does not counteract the effect of warfarin, but it also adds to the risk of bleeding by inhibiting platelet aggregation.

TOPIC REVIEW

Management of patients on warfarin therapy who are undergoing an invasive procedure is according to the following table:

Risk of bleeding Before surgery After surgery


Withhold warfarin 4-5 days before surgery

Check INR the day before surgery; if INR is 2-3, administer 3mg vitamin K IV

Day of surgery:

Recommence warfarin on the night after the surgery at the previous maintenance dose.
INR≤1.5 – surgery can proceed
Low
Consider VTE prophylaxis as per local guidelines
INR>1.5 – defer surgery or if urgent administer Prothrombinex-VF (use FFP if
Prothrombinex-VF unavailable)

Consider preoperative VTE prophylaxis using LMWH if immobilized

For procedures with low risk of bleeding (e.g. cataract, dental, dermatological) continue
warfarin
High OPTION 1 Recommence warfarin on the night of surgery at the previous maintenance dose.

Withhold warfarin 4-5 days before surgery

Once INR<2 start LMWH e.g. 1.5mg daily or 1mg twice a day , OR unfractionated heparin Consider bleeding risk against thrombosis
(UFH) infusion

If using LMWH, begin with prophylaxis dose; if UFH is used avoid bolus dose and aim to prolong the APTT
If LMWH is used, the last dose should be 24 hours before surgery by 1.5 times.

If UFH is used, stop infusion 4-6hours before surgery Delay resumption of therapeutic dose of LMWH for 48-72 hours after surgery in the absence of bleeding

OPTION 2 Continue LMWH or UFH for a minimum of 5 days and cease 48 hours after target INR is reached.

If INR has been stable within 2-3 in the past 2-4 weeks administer vitamin K, 3mg IV the day In surgery with high bleeding risk, consider using prophylactic dose LMWH or UFH only and cease 48 hours
before surgery after target INR is reached.

Day of surgery:

If INR≤1.5 - surgery can proceed

If INR>1.5 - defer surgery or if urgent administer Prothrombinex-VF (use FFP if


Prothrombinex-VF unavailable)

For urgent surgery, check INR before surgery and administer Prothrombinex-VF. Use FFP if
Prothrombinex unavailable.

616 of 1943
For procedures with low risk of bleeding (e.g. cataract, dental, dermatological) continue
warfarin

The following table summarizes peri-operative arterial and venous thromboembolism risk stratification:

Risk group Atrial fibrillation VTE Mechanical heart valve


CHADS21 score: 5-6
Recent VTE (<3 months)
Any mechanical mitral valve or older aortic
High Recent stroke or TIA (<3 months) valves
High risk thrombophilia2

Rheumatic valvular heart disease


Recurrent VTE
CHADS2 score: 3-4 Bileaflet aortic valve prosthesis with one or
Moderate
more risk factors3
VTE in the past 3-12 months
CHADS2 score: 0-2
Bileaflet aortic vale prosthesis without any risk
Low VTE>12 months previously
factors
No history of stroke or TIA
1
CHADS2 score: congestive heart failure, hypertension, age>75yrs, diabetes 1 point each; previous stroke or TIA 2 points

2
Deficiency of anithrombin, protein C, or protein S, antiphospholipid syndrome, or homozygous or double heterozygous factor V Leiden and prothrombin variant

3
Risk factors: AF, diabetes, cardiac failure, age>75, hypertension, previous stroke or TIA

References

• https://www.mja.com.au/journal/2013/198/4/update-c

Time spent: QID:949 Last updated:


2023-2-12

617 of 1943
A 22-year-old woman comes to the emergency department complaining of epistaxis for the past 12 hours. She denies any trauma to her nose and face. On examination, she is noted to have several bruises over her chest, arms and legs and
petechiae on the inner side of her lower lip. Laboratory tests revealed a platelet count of 223000/mm3, a PT of 12 seconds, and a prolonged APTT of 57 seconds. Her blood tests are otherwise normal. Which one of the following could be the most
likely diagnosis?

A. Hemophilia.

B. Von Willebrand disease.

C. Factor XII deficiency.

D. Hemolytic uremic syndrome (HUS).

E. Idiopathic thrombocytopenic purpura (TTP).

Incorrect. Correct answer is B


45% answered correctly

Explanation:

Correct Answer Is B

On approach to a bleeding disorder, the first thing to consider is evaluating whether it is platelet versus factor type of bleeding.

Platelet type of bleeding is superficial and causes gingival bleeding, epistaxis, ecchymoses, petechiae and purpura.
Factor type of bleeding occurs more deeply in joints or muscles causing hematomas.
Gastrointestinal, genitourinary, or central nervous system bleeding could be caused by both types.

This patient has platelet type of bleeding in the presence of a normal platelet count, a normal PT and prolonged ATTP. The clinical and laboratory picture is highly suggestive of Von Willebrand disease as the most likely diagnosis.

Von Willebrand disease is the most common hereditary bleeding disorder affecting 1% of population and characterized by a defect in production of Von Willebrand factor (VIIIa). Of all affected persons, 1% may become symptomatic at any age. Von
Willebrand presents with superficial bleeding tendency in the presence of normal platelet count and PT, but an often elevated APTT. The reason for increased ATTP is the fact that defect in Von Willebrand factor results in decreased factor VIII level of
activity.

(Option A) Hemophilia is only expressed in male individuals. Females can only carry the gene, as the pattern of inheritance is X-linked recessive.

(Option C) Factor XII deficiency is a very rare genetic disease which never causes bleeding. Hemolytic uremic syndrome is characterised by the presence the helmet cells (schistocytes or fragmented re cells) on the peripheral blood smear with
features of haemolytic anemia such as decreased red cell counts and hemoglobin, reticulocytosis, elevated LDH, and probably jaundice.

(Option D) The primary clot formation and homeostasis is the function of platelets. After endothelial lining is damaged, Factor VIII and Von Willebrand factor are released from underneath the epithelial cells and cause the platelets not adhere to each
and to the endothelial lining. Several hours later (after the fibrin clot is formed) a metaloprotease (ADAMTS13) dissolves the Von Willebrand factor to let the blood stream washes the platelets away. The deficiency of this enzyme leads to hemolytic
uremic syndrome (HUS) and thrombotic thrombocytopenic purpura (TTP). A microangiopathic hemolytic anemia is the essence of the diagnosis. This patient does not have hemolytic anemia.

(Option E) Immune thrombocytopenia (ITP) typically occurs in a young otherwise healthy patient and is characterized by isolated fall in platelet count and platelet type of bleeding.

618 of 1943
A 56-year-old woman is brought to the emergency department with sudden-onset severe epigastric pain. On examination, she has a blood pressure of 90/55mmHg, heart rate of 110bpm, and respiratory rate of 22 breaths per minute. There is
abdominal guarding and rigidity, as well as marked tenderness and rebound tenderness over the epigastric area. A chest X-ray reveals free air under the right hemi-diaphragm. Based on clinical findings, a perforated peptic ulcer is diagnosed and the
patient is planned for emergency laparotomy. She is on warfarin due to deep vein thrombosis (DVT) that developed 2 weeks ago. Which one of the following is the most appropriate next step in management?

A. Stop warfarin, give vitamin K and do the surgery.

B. Proceed with the surgery.

C. Give fresh frozen plasma (FFP) and proceed with the surgery.

D. Add intravenous heparin and perform the surgery.

E. Stop warfarin, give her heparin and proceed with the surgery.

Incorrect. Correct answer is C


45% answered correctly

Explanation:

Correct Answer Is C

This patient is in need of emergency life-saving surgery in the setting of perforated peptic ulcer disease and clinical manifestation of peritonitis (abdominal rigidity and guarding). In the event of need for life-saving or emergency surgeries where
deferring the surgery is not possible, warfarin effect should be reversed immediately using prothrombin complex concentrate (CCP) (Prothrombinex-VF®), or fresh frozen plasma (FFP) if CCP unavailable, regardless of the risk of potential VTE if
warfarin is temporarily stopped. Any option offering addition of vitamin K to cessation of warfarin and administration of CCP (or FFP) woudl be the most appropraite management (not an option).

(Option A) Cessation of warfarin and administration of vitamin K was the method of choice if the patient had an INR of 2-3 within the past 2-4 weeks and there was a time window for deferral of the surgery for at least 24 hours. In such cases, vitamin
K is given intravenously the evening before the surgery and INR is checked on the day of the surgery. An INR of ≤1.5 is safe to proceed with the surgery. If INR is >1.5 Prothrombinex-VF® should be given (FFP is used if Prothrombinex-VF is not
available)

(Option B) Proceeding with the surgery without warfarin reversal is associated with significant risk of intra- or post-operative bleeding and not recommended.

(Option D) Addition of heparin adds to the risk of bleeding.

(Option E) Cessation of warfarin and bridging with heparin is indicated in patients with high risk of VTE, who are planned for elective surgery. The protocol is not used for emergency procedures.

TOPIC REVIEW

Management of patients on warfarin therapy who are undergoing an invasive procedure is according to the following table:

Risk Before surgery After surgery


Withhold warfarin 4-5 days before surgery

Check INR the day before surgery; if INR is 2-3 administer 3mg vitamin K IV

Day of surgery:

Recommence warfarin on the night of surgery at the previous maintenance dose.


INR≤1.5 – surgery can proceed
Low
Consider VTE prophylaxis as per local guidelines
INR>1.5 – defer surgery or if urgent administer Prothrombinex-VF (use FFP if
Prothrombinex-VF unavailable)

Consider preoperative VTE prophylaxis using LMWH if immobilized

For procedures with low risk of bleeding (e.g. cataract, dental, dermatological) continue
warfarin
High OPTION 1 Recommence warfarin on the night of surgery at the previous maintenance dose.

Withhold warfarin 4-5 days before surgery

Once INR<2 start LMWH e.g. 1.5mg daily or 1mg twice a day , OR unfractionated heparin Consider bleeding risk against thrombosis
(UFH) infusion

If using LMWH, begin with prophylaxis dose; if UFH is used avoid bolus dose and aim to prolong the aPTT
If LMWH is used, the last dose should be 24 hours before surgery by 1.5 times.

If UFH is used, stop infusion 4-6hours before surgery Delay resumption of therapeutic dose of LMWH for 48-72 hours after surgery in the absence of bleeding

OPTION 2 Continue LMWH or UFH for a minimum of 5 days and cease 48 hours after target INR is reached.

If INR has been stable within 2-3 in the past 2-4 weeks administer vitamin K, 3mg IV the day In surgery with high bleeding risk, consider using prophylactic dose LMWH or UFH only and cease 48 hours
before surgery after target INR is reached.

Day of surgery:

If INR≤1.5 - surgery can proceed

If INR>1.5 - defer surgery or if urgent administer Prothrombinex-VF (use FFP if


Prothrombinex-VF unavailable)

For urgent surgery, check INR before surgery and administer Prothrombinex-VF before
surgery. Use FFP if Prothrombinex unavailable.

619 of 1943
For procedures with low risk of bleeding (e.g. cataract, dental, dermatological) continue
warfarin

The following table summarizes peri-operative arterial and venous thromboembolism risk stratification:

Risk group Atrial fibrillation VTE Mechanical heart valve


CHADS21 score: 5-6
Recent VTE (<3 months)
Any mechanical mitral valve or older aortic
High Recent stroke or TIA (<3 months) heart valves
2
High risk thrombophilia

Rheumatic valvular heart disease


Recurrent VTE
Bileaflet aortic valve prosthesis with one or
Moderate CHADS2 score: 3-4
more risk factors3
VTE in the past 3-12 months
CHADS2 score: 0-2
Bileaflet aortic vale prosthesis without any risk
Low VTE>12 months previously
factors
No history of stroke or TIA
1
CHADS2 score: congestive heart failure, hypertension, age>75yrs, diabetes 1 point each; previous stroke or TIA 2 points

2
Deficiency of anithrombin, protein C, or protein S, antiphospholipid syndrome, or homozygous or double heterozygous factor V Leiden and prothrombin variant

3
Risk factors: AF, diabetes, cardiac failure, age>75, hypertension, previous stroke or TIA

References

• https://www.mja.com.au/journal/2013/198/4/update-

Last updated:
Time spent: QID:950 2023-2-12

620 of 1943
A 55-year-old man has been on warfarin for AF for the past 3 months. He presented with an incarcerated inguinal hernia and was booked for emergent surgery. Warfarin was stopped and fresh frozen plasma was given. Which one of the following is
the time to resume warfarin therapy?

A. 12 hours post-op.

B. 48 hours post-op.

C. Immediately after recovery from anesthesia.

D. 5 days post-op.

E. When INR is less than 1.8 again.

Correct
45% answered correctly

Explanation:

Correct Answer Is A

For those, whose warfarin therapy has been stopped before major surgical procedures, it is recommended that the previous maintenance dose of warfarin be resumed on the night of surgery (12-24 hours). In addition to warfarin, low molecular
weight heparin (LMWH) in prophylactic dose or unfractionated heparin (UFH) with slow infusion is started at the same time. The target APTT is 1.5 times the normal.

LMWH or UFH is continued for at least 5 days and is ceased 48 after the target INR is reached (≥1.8).

References

• https://www.mja.com.au/journal/2013/198/4/update-c

• https://www.seslhd.health.nsw.gov.au/rhw/manuals/d

Last updated:
Time spent: QID:1062 2023-2-12

621 of 1943
A 65-year-old woman underwent an emergency surgery for a strangulated femoral hernia 2 days ago. She was on warfarin that was ceased before the surgery and fresh frozen plasma was given. Which one of the following is the most appropriate
action to take regarding resuming anticoagulation?

A. Start the patient on LMVH.

B. Start the patient on unfractionated heparin.

C. Resume warfarin.

D. Resume warfarin and start LMWH.

E. Resume warfarin after one week.

Incorrect. Correct answer is D


45% answered correctly

Explanation:

Correct Answer Is D

For patients whose warfarin therapy has been stopped before major surgical procedures, it is recommended that the previous maintenance dose of warfarin be resumed on the night of surgery (12-24 hours) and prophylactic dose of LMWH started at
the same time.

Unfractionated heparin (UFH) can substitute LMWH as a second-line option. Unlike LMWH, UFH use requires monitoring to a target APTT of 1.5 times the normal.

If unfractionated heparin (UFH) is used bolus injection should be avoided and slow infusion used. LMWH or UFH is continued for at least 5 days and is ceased 48 after the target INR is reached (≥1.8).

The reason for co-administration of LMWH or UFH is the fact that warfarin initially increases the risk of thromboembolism.

References

• https://www.mja.com.au/journal/2013/198/4/update-c

• https://www.seslhd.health.nsw.gov.au/rhw/manuals/d

Last updated:
Time spent: QID:1063 2023-2-12

622 of 1943
Which one of the following hereditary conditions is associated with highest risk of venous thromboembolism (VTE)?

A. Anti-thrombin deficiency.

B. Protein C deficiency.

C. Protein S deficiency.

D. Factor V Leiden.

E. Lupus anticoagulant.

Correct
45% answered correctly

Explanation:

Correct Answer Is A

In addition to conditions such as immobility, obesity, oral contraceptive pills, etc., there are a variety of inherited factors that contribute to VTE. These factors are also known as strong, medium and weak risk factors.

Anti-thrombin deficiency, protein C deficiency (option B), and protein S deficiency (option C), are associated with high risk of VTE. Among this group members, anti-thrombin deficiency, formerly known as anti-thrombin III deficiency, confers the
highest risk of VTE..

(Option D) Factor 5 Leiden is a moderate risk factor for VTE.

(Option E) Lupus anticoagulant is also a factor for both VTE and arterial thromboembolism but is an acquired condition not an inherited one. The prevalence is 1 to 5% of population and even more in the elderly and those with a comorbid condition
such as cancer. The risk of developing VTE in an individual with lupus anticoagulant is approximately 6-8%.

References

• American Heart Association - Risk Factors for Venous Thromboembolism

• MedScape - Genetics of Venous Thromboembolism

• PubMed - Venous Thromboembolism: Classification, Risk Factors, Diagnosis, and Management

Last updated:
Time spent: QID:1213 2023-2-12

623 of 1943
Which one of the following conditions is the most common inherited hypercoagulable state leading to venous thromboembolism (VTE)?

A. Anti-thrombin deficiency.

B. Protein C deficiency.

C. Protein S deficiency.

D. Factor V Leiden.

E. Lupus anticoagulant.

Incorrect. Correct answer is D


45% answered correctly

Explanation:

Correct Answer Is D

Venous thromboembolism (VTE) is a multifactorial disease driven by environmental/acquired risk factors such as age, obesity, oral contraceptives, and immobility, as well as inherited risk factors such as genetic polymorphisms.

Of genetic factors, factor V Leiden is the most common genetic cause of VTE . Prothrombin G20210A, gene mutation and deficiencies in protein S, protein C, and anti-thrombin account for most of the remaining cases.

It should be noted however that although factor V Leiden is the most common genetic cause of VTE, it is associated with only a medium risk of VTE in carriers.

References

• MedScape - Genetics of Venous Thromboembolism

• PubMed - Venous Thromboembolism: Classification, Risk Factors, Diagnosis, and Management

Last updated:
Time spent: QID:1214 2023-2-12

624 of 1943
A 7-year-old girl is brought to your practice by her parents, who are extremely concerned about several bruises on the child’s arms, legs and trunk she has developed over the past 2 days. She usually enjoys good health and has had no recent medical
problem except an upper respiratory tract infection (URTI) 2 weeks ago with complete resolution. On examination, there are several non-blanching petechial over the trunk and limbs and some bruises. There has been no bleeding event other than
bruises and the petechial rash. The rest of the exam is insignificant with no lymphadenopathy or hepatosplenomegaly. A full blood exam is arranged, the result of which is as follows:

WBC: 10500 (normal: 4,000-11,000)


RBC: 5x1012/L (normal: 3.9-5.6x1012/L)
Hb: 13g/dL (normal : 11.5-16g/dL)
MCV: 85fL (normal: 76-96fL)
Platelet count: 35x109/L (normal: 150-400x109/L)

Which one of the following is the most appropriate next step in management of this child?

A. Strict bed rest.

B. Systemic steroids.

C. Intravenous immunoglobulin.

D. Platelet transfusion.

E. No treatment.

Incorrect. Correct answer is E


45% answered correctly

Explanation:

Correct Answer Is E

The clinical picture of bruising and petechial rash following a viral illness in an otherwise healthy child (no pallor, no lymphadenopathy, and no splenomegaly) is suggestive of immune thrombocytopenic purpura as the most likely diagnosis. Isolated
thrombocytopenia found on FBE makes this diagnosis almost certain.

ITP is acute in approximately 90% of cases and spontaneous resolution often occurs without any active treatment. ITP persisting beyond 6 months is termed as chronic ITP.

Management of acute ITP in children includes no treatment at all or oral steroids. Without treatment, the platelet count will return to an acceptable level at which normal activity can be recommended within 4 to 6 weeks. Addition of systemic
steroids raise the platelet count faster compared to no treatment. Use of steroids has not shown any significant benefit in terms of mortality, morbidity and risk of chronicity of ITP. Immunoglobulins are considered if there is significant bleeding.
Considering this fact, watchful waiting is the treatment option of choice for acute ITP in children where possible.

In general, conservative management with no active treatment should be considered the first-line treatment option the in children with ITP with a platelet count of >20x109/L and some with a platelet count of <20x109/L provided that the following
criteria are met:

The diagnosis is certain.


There is no pallor, hepatosplenomegaly, or lymphadenopathy.
There is only isolated thrombocytopenia on FBE.
There is no active bleeding other than bruising and petechial in isolation. Any mucosal, gastrointestinal, or genitourinary bleeding, even if ceased, excludes the patient from this approach.
The child is otherwise well.
Parents can be reassured and educated adequately and appropriately.
There is confidence about the degree of parental supervision and safety of home environment, particularly for younger children.
Follow up within a few days is guaranteed by the on-call pediatrician who must be contacted and agreed with the management plan.

In this child a definitive diagnosis of ITP can be made with high certainty. There is only isolated thrombocytopenia of greater than 20x109/L on FBE. He is otherwise healthy has no bleeding other than bruising and petechiae. For her, no treatment and
watchful waiting is the most appropriate option provided than follow up is guaranteed, her carers can be reassured and educated, and there is appropriate home safety for the child.

(Option A) While limited activity as means of decreasing a bleeding event is recommended, strict bed rest is not an appropriate advice.

(Option B) Systemic corticosteroids are first-line treatment in children who have active bleeding other than bruises and petechiae, even if ceased now. Prednisolone (2-4mg/kg/day for 2 weeks then tapered) is the regimen of choice. This child has no
active bleeding and a platelet count of >20x109/L. For now, he can be managed conservatively with no active treatment.

(Option C) Intravenous immunoglobulin (IVIG) is the second-line treatment option. An important indication is severe active bleeding unresponsive to steroids.

(Option D) In ITP, platelets are sequestrated by the spleen. Platelet transfusion is only rarely considered in cases with severe life-threatening bleeding as a temporary measure, or after splenectomy due to severe persistent ITP where the risk of intra-
or post-operative bleeding is significant.

References

• http://www.rch.org.au/clinicalguide/guideline_index/Immune_Thrombocytopenic_Purpura/

• https://emedicine.medscape.com/article/202158-treatment

Time spent: QID:1261 Last updated:


2023-2-12

625 of 1943
A 4-year-old boy is brought to the emergency department by his parents with bruising and a generalized rash for the past two days. He also had an episode of nose bleed this morning that stopped after 10 minutes of pressure and ice application. He
also had a runny nose, mild fever and sneezing 10 days ago that was resolved after one week of conservative management. On examination, the child is otherwise healthy and in good shape. His vital signs are stable. There are three bruises over the
abdomen, right arm and left buttock as well as generalized non-blanching petechial rash over the trunk and limbs. There is no pallor, lymphadenopathy, or hepatosplenomegaly. A full blood count is arranged with the following results:

WBC: 9700 (normal: 4,000-11,000)


RBC: 4.2x1012/L (normal: 3.9-5.6x1012/L)
Hb: 13.7g/dL (normal : 13-18g/dL)
MCV: 90fL (normal: 76-96fL)
Platelet count: 56x109/L (normal: 150-400x109/L)

Which one of the following is the most appropriate next step in management?

A. Oral prednisolone.

B. Intravenous immunoglobulin.

C. Platelet transfusion.

D. Strict bed rest.

E. No treatment.

Correct
45% answered correctly

Explanation:

Correct Answer Is A

Petechial rash, easy bruising and isolated thrombocytopenia on FBE in an otherwise healthy child without hepatosplenomegaly, lymphadenopathy or other alarming findings is suggestive of immune thrombocytopenic purpura as the most likely
diagnosis. The presence of a recent viral illness in the history makes such diagnosis even more certain.

Management of acute ITP in children includes no treatment at all or oral steroids. Without treatment, the platelet count will return to an acceptable level at which normal activity can be recommended within 4 to 6 weeks. Addition of systemic
steroids raise the platelet count faster compared to no treatment. Use of steroids has not shown any significant benefit in terms of mortality, morbidity and risk of chronicity of ITP.

In general, conservative management with no active treatment should be considered the first-line treatment option the in children with ITP with a platelet count of >20x109/L and some with a platelet count of <20x109/L provided that the following
criteria are met:

The diagnosis is certain.


There is no pallor, hepatosplenomegaly, or lymphadenopathy.
There is only isolated thrombocytopenia on FBE.
There is no active bleeding other than bruising and petechial in isolation. Any mucosal, gastrointestinal, or genitourinary bleeding, even if ceased, excludes the patient from this approach.
The child is otherwise well.
Parents can be reassured and educated adequately and appropriately.
There is confidence about the degree of parental supervision and safety of home environment, particularly for younger children.
Follow up within a few days is guaranteed by the on-call pediatrician who must be contacted and agreed with the management plan.

In this child a definitive diagnosis of ITP can be made with high certainty. There is only isolated thrombocytopenia of greater than 20x109/L on FBE. He is otherwise healthy but has had an episode of nose bleed. For this child, oral steroids are
indicated because he does not fulfill the criteria for “no treatment” option. Prednisolone (2-4mg/kg/day for 2 weeks then tapered) is the regimen of choice.

(Option B) Intravenous immunoglobulin (IVIG) is the second-line treatment option. An important indication is severe active bleeding unresponsive to steroids.

(Option C) In ITP, platelets are sequestrated by the spleen. Platelet transfusion in limited cases with severe life-threatening bleeding as a temporary measure, or after splenectomy due to severe persistent ITP where the risk of intra- or post-operative
bleeding is significant.

(Option D) While limited activity as means of decreasing a bleeding event is recommended, strict bed rest is not an appropriate advice.

(Option E) ‘No treatment’ could be an acceptable option if there was no bleeding in history other than the bruises and petechial rash. With the nose bleed, even though it was minimal and self-limited, systemic steroids should be started as there is the
risk of further bleeding episodes that could be significant or even life-threatening.

References

• http://www.rch.org.au/clinicalguide/guideline_index/Immune_Thrombocytopenic_Purpura/

• https://emedicine.medscape.com/article/202158-treatment

Time spent: QID:1262 Last updated:


2023-2-12

626 of 1943
After a long flight from London to Sydney, a 35-year-old man, who is a known case of sickle cell disease, presents to the emergency department with severe right calf pain. The pain has started acutely two hours ago and is 6 out of 10 on a scale of 1
to 10, according to the patient. He denies any chest pain or shortness of breath. On examination, he has a blood pressure of 134/78 mmHg, pulse rate of 98 bpm, respiratory rate of 17 breaths/min, a temperature of 37.2℃, and O2 saturation of 94%
on room air. Chest examination is normal and there is no abnormal respiratory or heart sound. Which one of the following would be the next best step in management?

A. Enoxaparin.

B. Adequate analgesia.

C. Oxygen.

D. Hydration.

E. Limb elevation.

Incorrect. Correct answer is B


45% answered correctly

Explanation:

Correct Answer Is B

This is a rather challenging question, describing a person with sickle cell disease, who has developed calf pain after a long flight. Long distance flights are associated with prolonged immobility and, consequently, venous stasis that predisposes to
deep vein thrombosis (DVT). On the other hand, especially during unpressurized long flights, hypoxia can develop and trigger a sickle cell crisis. There is no additional information such as leg circumference discrepancy, warmth, erythema, etc., as
pointers towards DVT versus sickle cell crisis.

DVT can present with local tenderness, erythema, pain, and increased limb circumference. Pain and tenderness is also a feature in patients with pain crisis due to sickle cell disease. However, the most important notion to consider in this scenario is
not a clear diagnosis. It is about the general rules to consider in approaching every single patient.

Evaluation of a patient always starts with checking if he/she is stable or not. A stable patient has a patent airway, breathes normally and has normal circulatory parameters such as blood pressure and pulse rate in adults and pulse rate and capillary
refill time (CRT) in children. Chest pain and altered consciousness are other indicators of instability to look for. This patient is speaking and giving history. These indicate a patent airway. He is breathing normally and has an acceptable O2 saturation.
Therefore, oxygen (option C) is not necessary for now. His pulse rate and blood pressure also provide assurance as to his stable circulatory status. He does not have chest pain and is conscious and oriented. This patient is stable for now. After
stability is assured, the next step in priority should be removing any discomfort or pain if possible and putting the patient at ease. In practice and when pain is a symptom, the patient should be asked about the pain characteristics, and if severe and
discomforting for the patient, analgesia have to be offered. So, for this patient, whose main complaint is pain, the most appropriate next step in management after the ABC is analgesia regardless of the etiology. Analgesia is also the mainstay of
treatment if the diagnosis turns out to be a pain crisis due to sickle cell disease.

Other options should be considered next based on the diagnosis. Enoxaparin (option A) and limb elevation (option E) may be considered if further assessment establishes DVT as the diagnosis. It is important to note that anticoagulation only starts
if the a diagnosis of DVT is certain via pretest probability tools and appropriate imaging studies. Hydration (option D) to maintain the plasma volume may be considered if the patient is found to have a pain crisis due to sickle cell disease.

References

• NCBI – Sickle Cell Disease and Venous Thromboembolism

• Merck Manual – Sickle Cell Disease

• RCH – Sickle cell disease

• Medscape - Sickle Cell Anemia

Last updated:
Time spent: QID:1510 2023-2-12

627 of 1943
Peter, 25 years of age, presents to the Emergency Department with generalized severe pain in his limbs and joint. He is a known case of sickle cell disease and has just arrived from Paris to Melbourne after a long flight. On scale of 1 to 10, peter rates
the pain as 7. He denies any breathlessness or chest pain. On examination, he has blood pressure of 140/87 mmHg, pulse rate of 104 bpm, respiratory rate of 18 breaths per minute and a temperature of 36.9℃. The only remarkable exam finding is
the generalized tenderness on palpation of his muscles, bones, and joints of upper and lower limbs. Which one of the following would be the next best step in management?

A. Oxygen.

B. Hydration.

C. Enoxaparin.

D. Analgesics.

E. CTPA.

Incorrect. Correct answer is D


45% answered correctly

Explanation:

Correct Answer Is D

The scenario describes a case of sickle cell crisis, presented with an acute pain crisis.

Pain (vaso-occlusive) crises are the most common presentation of sickle cell crises. A painful crisis is caused by ischemia and infarction that typically occurs in the bones, but can also affect the spleen, lungs, or kidneys. With painful crisis,
analgesics should be started promptly as the most important initial step. In mild cases, simple analgesics such as paracetamol plus codeine adequately control the pain; however, most patient have more severe pain and often will need intravenous
opioids. IV morphine (continuous or bolus) is effective and safe. Meperidine should be avoided.

Although dehydration contributes to sickling and may precipitate crises, it is unclear whether vigorous hydration (option B) is helpful during crises. Nevertheless, maintaining normal intravascular volume if the patient is volume depleted is an
important part of management. Patients may even need transfusions using WBC filtered blood.

Although Peter has had a recent long flight, he does not have symptoms suggestive of pulmonary embolism such as chest pain or shortness of breath. Therefore, CTPA (option E) or enoxaparin (option C) are not correct options for now.

Supplementation with oxygen (option A) is required when there is hypoxia, or there is chest syndrome as a manifestation of his sickle crisis. Peter is not hypoxic, nor does he have chest pain; therefore, he is not in immediate need for oxygen.

NOTE – Regardless of the cause, checking for airway, breathing and circulation (ABC) followed by assessment of pain and pain control if necessary is the common approach for every single patient presenting with a medical problem.

TOPIC REVIEW

Sickle cell disease (SCD) is caused by the structurally abnormal hemoglobin (Hb S) that results in rigid and sickle-shaped red blood cells.

There are three common types of SCD:

Sickle cell anemia (SS disease) – the most common type


Sickle β-thalassemia
Sickle hemoglobin C disease

Sickle cells crisis is an acute complication of the disease. Sickle cell crisis may occur spontaneously or be precipitated by the following:

Infections
Dehydration
Hypoxia
Sedatives and local anesthesia

An acute crisis may present in the following forms:

Pain crisis (vaso-occlusive crisis)

Painful crisis is the most common of all forms of sickle cell crises, and is caused by ischemia and infarction, typically of the bones, but also of the spleen, lungs, or kidneys.

Management

Analgesics should be started promptly as the most important initial step. In mild cases simple analgesics such as paracetamol plus codeine satisfactorily controls the pain; however, most patient will need intravenous opioids. IV
morphine (continuous or bolus) is effective and safe.
Although dehydration contributes to sickling and may precipitate crises, it is unclear whether vigorous hydration is helpful during crises. Nevertheless, maintaining normal intravascular volume has been a mainstay of therapy.
Intravenous antibiotics (third-generation generation cephalosporins) if the there is a fever

NOTE – chest pain should be treated as an acute chest syndrome and not simply as a vaso-occlusive episode.

Aplastic crisis

This condition occurs when marrow erythropoiesis slows during acute infection (especially viral). Hemoglobin drops to under the baseline for the patient and reticulocyte count decreases substantially (usually <1%). Enlarged spleen may be noted.

628 of 1943
Management

Intravenous fluids and oral intake to a total of maintenance


Transfusion of red blood cells if patient is symptomatic due to anemia, or hemoglobin is less than 50 g/L
Intravenous antibiotics (third-generation cephalosporins) if the there is fever

Acute chest syndrome

Acute chest syndrome results from pulmonary microvascular occlusion and is a common cause of death in patients with SCD, with mortality rates of up to 10%. It occurs in all age groups but is most common in childhood. Repeated episodes
predispose to chronic pulmonary hypertension presenting with lower respiratory symptoms, hypoxemia and emergence of new infiltrates on chest X-ray. Chest pain and hypoxemia may be the only manifestations. Chest pain should be treated as an
acute chest syndrome, and not simply as a vaso-occlusive crisis.

Management

High flow oxygen to maintain O2 saturation above 96% as the most important initial step
Chest X-ray. This, however, should not delay commencement of treatment. Patients with acute chest syndrome should be admitted regardless of chest X-ray findings.
Intravenous antibiotics (a third-generation cephalosporin and coverage for atypical organisms with antibiotics such as roxithromycin or azithromycin) if there is fever
Early referral to ICU for respiratory support if significant hypoxia or respiratory distress is present.
Transfusion (usually 5ml/kg over 4 hrs) if there is symptomatic anemia or hemoglobin is less than < 50 g/L with close attention and monitoring to prevent volume overload
Consultation with hematologist

Priapism

Priapism is prolonged painful erection of the penis, often starting in the early hours of the morning. It is a serious complication that can cause erectile dysfunction. It is most common in young men. The priapism associated with sickle cell crisis is
low flow.

Priapism occurs in two forms:

Stuttering episodes which last 2-4 hrs but are often recurrent and may precede a severe episode
Severe attack lasting longer than 4 hrs and can result in impotence

Management

A detailed history including the length of current episode, associated symptoms such as fever, dysuria, dehydration or pain at other locations, and history of prior episodes and the previous treatments and effectiveness should be
obtained.
The patient should be encouraged to empty the bladder. Catheterization should be considered if the patient is unable to void
Simple measures – moderate exercise, taking a bath or shower
Application of ice should be avoided
Hydration, oxygen, analgesia and urine alkalinization as soon as possible
Consultation with a general surgeon or urologist and on-call hematologist if priapism has lasted more than 3-4 hours
Transfusion support may also be required to reduce the risks of surgery.

Acute neurological events

Acute neurological events occur in about 10% of patients with the disease. A neurological deficit may occur acutely or as a complication of acute chest syndrome or aplastic crisis. Neurological features include, but not limited to, hemiparesis,
monoparesis, aphasia or dysphasia, seizures, cranial nerve palsies, or even coma.

Management

Neuroimaging is required to determine if there is hemorrhagic or ischemic stroke. MRI is the modality of choice. If MRI is not available, non-contrast CT scan can be used. Contrast matter carries the risk of hyperviscosity and worsening
of neurological status
Transfusion support will be required to reach to a hemoglobin level of 100 g/L followed by red cell exchange (performed in ICU in acute setting).

Acute splenic sequestration

Splenic sequestration is defined as a hemoglobin level of at least 20 g/L below the baseline level for that patient with an acutely enlarged spleen. Mild to moderate thrombocytopenia is often present. Reticulocyte count is normal or elevated. If
reticulocyte count is low, coexisting aplastic anemia should be considered.

Management

Blood transfusion is the mainstay of therapy, but while waiting for blood, 0.9% Saline (10-20ml/kg) should be started for treatment of hypovolemia
Intravenous third-generation cephalosporins if there is a fever

NOTE - Blood transfusion should aim initially for a hemoglobin of 50-60 g/L to ameliorate shock/hemodynamic instability. The initial increase in the hemoglobin level should not exceed 3 g/dL. With an excessive or too-quick raise in
hemoglobin, the spleen will shrink and autotransfusion will occur. This will then increase the hemoglobin excessively and can lead to increased risk of stroke due to hyperviscosity.

Other manifestation of sickle cell disease /crisis include: leg ulcers, avascular necrosis, and renal involvement.

References

• NCBI – Sickle Cell Disease and Venous Thromboembolism

• Merck Manual – Sickle Cell Disease

• RCH – Sickle cell disease

• Medscape - Sickle Cell Anemia

L t d t d

629 of 1943
Last updated:
Time spent: QID:1512 2023-2-12

630 of 1943
After flying from Sydney to Port Vila, a 25-year-old man develops shortness of breath shortly after landing. On further inquiry, you realize that he is a known case of sickle cell trait, whose father died of myocardial infarction at the age of 36 years. He
denies any chest pain and muscle and joint pain but feels slightly short of breath. On examination, he has central cyanosis, a blood pressure of 130/85 mmHg, a pulse of 110 bpm and a respiratory rate of 24 breaths per minute. He is afebrile.
Pulseoxymetry shows an O2 saturation of 86% while on room air. The rest of the exam including a full neurological exam is inconclusive. Which one of the following is the next best step in management?

A. Start intravenous heparin immediately.

B. Do a chest X-ray to confirm pulmonary embolism.

C. Do an ECG.

D. Do CTPA to confirm pulmonary embolism.

E. Give oxygen and review.

Incorrect. Correct answer is E


45% answered correctly

Explanation:

Correct Answer Is E

Patients who are heterozygous (Hb AS) (Sickle cell trait) do not experience hemolysis, painful crises, or thrombotic complications except possibly during hypoxic conditions (e.g. at high altitudes, during sudden decompression in airplanes). In these
patients, rhabdomyolysis and sudden death may occur during sustained, exhausting exercise. Impaired ability to concentrate urine (hyposthenuria) is common. Unilateral hematuria (by unknown mechanisms and usually from the left kidney) can
occur but is self-limited. Typical renal papillary necrosis can occur but is less common than in homozygous patients.

Main presenting symptoms in this man are shortness of breath and central cyanosis. There are conditions to consider as the cause of such clinical picture i.e., long flight in high altitude, pulmonary embolism, acute chest syndrome in the setting of
sickle cell trait triggered by hypoxia, etc. However, the most important step to take first is attending to his hypoxia. This patient should be started immediately on oxygen and his status reviewed. With a desirable response that sustains after
discontinuation of oxygen, other causes such as acute chest syndrome or pulmonary embolism are unlikely. With sustained hypoxia despite initial oxygen supplementation, other measures to consider are a chest X-ray (option C) to exclude other
conditions, an ECG (option C), and CTPA (option D) if there is a high probability for pulmonary embolism. Heparin (option A) is only used when there is a definite diagnosis of pulmonary embolism. Heparin is never given empirically for treatment of
venous thromboembolism.

TOPIC REVIEW

Common causes of central cyanosis:

Upper respiratory tract obstruction


Impaired gas exchange secondary to pneumonia
Pulmonary embolism and ventilation/perfusion mismatch
Impaired gas diffusion in the alveoli
High altitude
Anatomic shunts
Right to left shunts in congenital heart disease
Arteriovenous malformation
Intrapulmonary shunt

References

• NCBI – Sickle Cell Disease and Venous Thromboembolism

• Merck Manual – Sickle Cell Disease

• RCH – Sickle cell disease

• Medscape - Sickle Cell Anemia

Last updated:
Time spent: QID:1514 2023-2-12

631 of 1943
A 3-year-old boy is being evaluated in your general practice. For the past 2 days, he has had bruising over his trunk and limbs and a petechial rash over his body. This morning he had an episode of nose bleed lasting about 10 minutes. On
examination, he looks well and healthy with no distress. He has no fever. There is no splenomegaly or hepatomegaly. He has several non-blanching petechiae over his trunk and legs and recent bruises on his body. Which one of the following is the
most likely diagnosis?

A. Von Willebrand disease.

B. Henӧch-Schonlein purpura.

C. Acute lymphocytic leukemia.

D. Immune thrombocytopenia.

E. Hemophilia A.

Incorrect. Correct answer is D


45% answered correctly

Explanation:

Correct Answer Is D

Petechiae indicate that the problem is either a vascular or platelet problem. Petechiae do not occur in factor types of bleeding (e.g. hemophilia or Von Willebrand disease). The clinical picture is highly-suggestive of immune thrombocytopenic
purpura, characterized by recent-onset easy bruising, bleeding and petechiae in an otherwise healthy child. The history of a preceding upper respiratory tract infection is another pointer towards the diagnosis.

ITP is an acquired immune-mediated thrombocytopenia characterized by shortened lifespan of circulating platelets in the absence of other disturbances of hemostasis or coagulation.

Patients fall broadly into two categories:

Acute (~ 90%): self limiting disease (sometimes preceded by a viral syndrome) with spontaneous resolution within 6 months (usually within 2 months)
Chronic (~ 10%): persists beyond 6 months.

In acute ITP, the patient often presents with bruising and petechiae alone; however, in some instances there is oral bleeding, epistaxis, rectal bleeding or hematuria.

Morbidity in ITP is usually minimal and parents need to be reassured. The incidence of intracranial hemorrhage is less than 1%, and very rare in true uncomplicated ITP.

The clinical diagnosis of ITP relies on manifestations of thrombocytopenia (platelet type of bleeding) without other abnormal findings, in particular no pallor, lymphadenopathy or hepatosplenomegaly. ITP is a diagnosis of exclusion. Other causes of
thrombocytopenia should be ruled out.

(Option A) Von Willebrand disease is a hereditary bleeding disorder presenting similar to ITP with easy bruising, petechiae, epistaxis, and menorrhagea in women. The disorder often runs in families. Patients may become symptomatic at any age.
Although the presentation of von Willebrand disease shares many features with ITP, two points make it less likely than ITP. First is the fact that petechiae are a less common finding in von Willebrand, and second is the absence of a family history.

(Option B) Henӧch-schonlein purpura is usually associated with abdominal pain and arthralgia which is not present in this child.

(Option C) In acute lymphocytic leukemia, the clinical picture is more pronounced and the child has been unwell for several weeks prior to coagulopathy.

(Option E) Hemophilia is a hereditary condition presenting early in life with factor type of bleeding. Petechiae is not a feature.

References

• RCH - Immune thrombocytopenic purpura

Last updated:
Time spent: QID:78 2023-2-12

632 of 1943
A 22-year-old man presents with complaints of weakness and lethargy for the past few months. He denies any abdominal pain, rectal bleeding, or bowel symptoms. His past medical history is not remarkable. A full blood exam (FBE) and iron studies
are ordered with the following results:

RBC: 3.5x1012/L (4.5-6.5x1012/L)


WBC: 6.3x109/L (4.0-11x109/L)
PLT: 210x109/L (150-400x109/L)
Hemoglobin: 70 g/L (130-180g/L)
MCV: 66 fL (76-96 fL)
Ferritin: 15μg/L (30-200μg/L)

Serum iron: 7µmol/L (10.74 – 30.43 µmol/L)

Which one of the following options is most likely to help reach a diagnosis

A. Bone marrow biopsy.

B. Small bowel biopsy.

C. Hemoglobin electrophoresis.

D. Colonoscopy.

E. Sigmoidoscopy.

Incorrect. Correct answer is D


45% answered correctly

Explanation:

Correct Answer Is D

According to WHO, anemia is defined as hemoglobin (Hb) <120 g/L for females and <130 g/L for men. According to Royal College of Pathologists of Australasia iron deficiency is present when serum ferritin level is less than 30 µg/L for an adult.
Therefore, this patient has iron deficiency anemia. Iron deficiency is never a final diagnosis in itself and a cause should always be sought.

The most common cause of iron deficiency is ongoing bleeding, either overt (e.g., heavy periods, or occult such as that occurring in gastrointestinal bleeding or malignancies). In cases with confirmed iron deficiency anemia for which an obvious
source of blood loss cannot be identified in history or physical examination, upper and lower gastrointestinal investigations should be considered. In fact, upper and lower gastrointestinal tract scope are mandatory in all postmenopausal females and
all males with confirmed iron deficiency anemia unless there is a history of significant overt non-GI blood loss such as vaginal bleeding.

In premenopausal women with iron deficiency anemia, upper and lower gastrointestinal (GI) investigation should be reserved for those aged over 50 years who have symptoms suggestive of GI disease or a strong family history of colorectal cancer.

This patient is a male with iron deficiency anemia. Since there is not apparent cause for the blood loss, and since the GI tract is the most common site of occult blood loss in such patients, the first and most important step to consider is GI
investigations starting with a colonoscopy. If no identifiable is found in colonoscopy, upper endoscopy should follow. Sigmoidoscopy (option E) is not an appropriate choice for this purpose because it only provides information about the sigmoid
colon and will miss more proximal potential sources of bleeding.

Celiac disease should also be considered as a differential diagnosis and excluded in all patients with iron deficiency anemia using serologic studies. These tests are screening tests and those with positive test results will require small bowel biopsy
(option B) as the confirmatory test. Not every patient with iron deficiency anemia undergoes small bowel biopsy.

(Option A) Bone marrow biopsy and aspiration may be used for many conditions, such as leukopenia, leukocytosis, thrombocytopenia, thrombocytosis, pancytopenia and polycythemia. This test would be used if anemia could not be justified by iron
deficiency, or if the patient, besides the anemia, has any of the above features.

(Option C) Serum electrophoresis is the test of choice to confirm thalassemia as the cause of anemia. Although the full blood exam (FBE) of this patient resembles thalassemia trait, the low ferritin level in this patient is against it because in patients
with thalassemia, ferritin and iron or either normal or elevated. Since thalassemia cannot cause iron deficiency anemia, serum electrophoresis adds no diagnostic value in this patient.

References

• MJA - Diagnosis and management of iron deficiency anaemia: a clinical update

Last updated:
Time spent: QID:1538 2023-2-12

633 of 1943
A 70-year-old woman presents to your practice with complaints of weakness and easy fatigability for the past 5 months. She is a recent immigrant from Algeria landing on Australia almost 8 months ago. She has been a vegetarian most of her adult
life. Past medical history is insignificant for any chronic condition otherwise. Based on the history and physical findings, you suspect anemia and order a full blood examination (FBE) which is significant for a red cell count (RCC) of 2.9x1012/L (4.5-
6.5x1012/L), hemoglobin of 90 g/L (120-160 g/L), MCV of 62 fL (80-100 fL). Which one of the following could be the most likely cause of this presentation?

A. Cecal cancer.

B. Thalassemia minor.

C. Hookworm infestation.

D. The vegetarian diet.

E. Vitamin B12 deficiency.

Incorrect. Correct answer is C


45% answered correctly

Explanation:

Correct Answer Is C

The decreased RCC and hemoglobin in this patient represent anemia. With the decreased MCV, the anemia is microcytic. The most common causes of microcytic anemia are iron deficiency and thalassemia. This clinical picture, however, is
inconsistent with thalassemia minor (trait) (option B) because although patients with thalassemia minor has isolated mild microcytic anemia, the red cell count, unlike in this patient, is normal of even increased. Moreover, the anemia of thalassemia
minor is asymptomatic.

Strict vegetarians are prone to vitamin B12 deficiency as this vitamin is only available in animal products such as meat, poultry, fish, eggs, and dairy products. Vitamin B12 deficiency (option E) causes macrocytic anemia that does not explain the
microcytosis in this scenario.

A vegetarian diet includes green-leaf vegetables that provide adequate dietary iron to prevent from iron deficiency. Anemia caused by vegetarian diet (option D) is more likely to be macrocytic and have been caused by vitamin B12 deficiency.

Cecal cancer (option A) and right-sided colon cancer in general often present with anemia and its symptoms such as fatigue and weakness (as opposed to left-sided colorectal cancers that usually manifest with altered bowel habits and rectal
bleeding); therefore, should be considered and investigated in patients with microcytic anemia, or iron deficiency anemia to be more specific. This is even more important in older patients. However, colorectal cancer in vegetarians is extremely rare
because high-fiber diets are a protective factor against colorectal cancer.

Given the history and exclusion of other options, hookworm infestation remains the most likely explanation to this scenario.

Human hookworm disease is a common helminth infection predominantly caused by the nematode parasites Necator americanus and Ancylostoma duodenale. Hookworm infection is acquired through skin exposure to larvae in soil contaminated by
human feces.

Worldwide, hookworms infect an estimated 472 million people. It is endemic in many underdeveloped or developing regions in the work including African, south Asia, Mediterranean region, and south America.

The larvae of hookworm from the soil enter the foot sole skin, then migrate through the dermis, enter the bloodstream, and move to the lungs within 10 days. Once in the lungs, they break into alveoli, causing a mild and usually asymptomatic
alveolitis with eosinophilia. Form the alveoli, the larvae are carried to the glottis by the ciliary action of the respiratory tract. During pulmonary migration, the host may develop a mild reactive cough, sore throat, and fever that resolve after the worm
migrates into the intestines. At the glottis, the larvae are swallowed and carried to their final destination, the small intestine. Of all infested people, only 10% develop symptoms which include iron deficiency anemia due to loss from small intestine in
patients with moderate to severe infestation burden.

References

• UpToDate – Hookworm infection

Last updated:
Time spent: QID:1656 2023-2-12

634 of 1943
A 34-year-old pregnant woman presents to your office for evaluation at 12 weeks gestation. Her obstetrical history is significant for 2 miscarriages in previous pregnancies, for which she underwent assessment and was found to have
antiphospholipid syndrome. She is currently on aspirin. Which one of the following would be the next best step in management?

A. Do nothing, as aspirin alone is adequate.

B. Start her on prophylactic dose of low molecular weight heparin.

C. Start her on warfarin.

D. Start her on therapeutic dose of low molecular weight heparin

E. Arrange for elective abortion.

Incorrect. Correct answer is B


45% answered correctly

Explanation:

Correct Answer Is B

For women with laboratory-proven criteria for antiphospholipid antibody syndrome and one or more miscarriages after 10 weeks gestation, combined therapy with low-dose aspirin (50 to 100 mg per day) and prophylactic dose of low molecular
weight heparin (LMWH) upon confirmation of intrauterine pregnancy is recommended. Therefore, addition of low-molecule weight heparin (LMWH) in prophylactic dose would be the next best step in management. Treatment should be continued
throughout the pregnancy. Therapy is withheld at the time of delivery and is restarted after delivery, continuing for 6-12 weeks postpartum.

References

• http://emedicine.medscape.com/article/333221-treat

• http://www.uptodate.com/contents/pregnancy-in-wome

• http://www.guideline.gov/content.aspx?id=25312

Last updated:
Time spent: QID:443 2023-2-12

635 of 1943
A 69-year-old woman has been in the Intensive Care Unit (ICU) for the past 7 days following a complicated hip replacement surgery. She is currently on low-molecular weight heparin (LMWH) as well as intermittent pneumatic compression device.
Today, she has developed new-onset right calf pain. On examination, the leg is edematous and tender to touch. A positive Homan’s sign is elicited. A Doppler ultrasound confirms the presence of a clot in the right lower leg and the diagnosis of deep
vein thrombosis. Blood tests show a platelet count of 78,000 /m3m, but there is no sign of hemorrhage. Which one of the following is the next best step in management?

A. Discontinue application of pneumatic compression device.

B. Start her on warfarin.

C. Discontinue heparin.

D. Perform venography.

E. Transfuse platelets.

Incorrect. Correct answer is C


45% answered correctly

Explanation:

Correct Answer Is C

Development of DVT in the presence of thrombocytopenia is suggestive of heparin-induced thrombocytopenia (HIT) as the most likely diagnosis. HIT is a complication of heparin therapy.

There are two types of HIT:

Type 1 HIT - presents within the first 2 days after exposure to heparin, and the platelet count normalizes with continued heparin therapy. Type 1 HIT has a non-immune pathophysiology.

Type 2 HIT - an immune-mediated disorder that typically occurs 4-10 days after exposure to heparin and has life- and limb-threatening thrombotic complications. In general medical practice, the term HIT refers to type 2 HIT.

HIT is caused by antibodies that bind to complexes of heparin and platelet factor 4 (PF4), activation of the platelets, and promoting a prothrombotic state. HIT is more frequently encountered with unfractionated heparin (UFH) than with low-
molecular weight heparin (LMWH).

The risk of HIT is highest with prolonged use of heparin for postoperative thromboprophylaxis. However, case studies have also shown that HIT can occur even with minimal heparin exposure via intravascular flushes to maintain an arterial or venous
catheter open.

HIT must be suspected when a patient, who is receiving heparin, has a decrease in the platelet count, particularly if the fall is over 50% of the baseline count, even if the platelet count remains above 150,000/mm3. Clinically, HIT may manifest as skin
lesions at heparin injection sites or by acute systemic reactions (e.g., chills, fever, dyspnea, chest pain) after administration of an intravenous bolus of heparin.

Unlike other forms of thrombocytopenia, HIT is associated with thromboembolism (e.g., DVT, PE) as the most common complication, not with bleeding. Sometimes, arterial thrombosis (e.g., myocardial infarction) may occur. For this reason, the
disorder is sometimes termed heparin-induced thrombocytopenia and thrombosis (HITT).

Factors that increase the risk for heparin-induced thrombocytopenia (HIT) are as follows:

Use of unfractionated heparin (UFH) rather than low molecular weight heparin (LMWH)
Intravenous rather than subcutaneous administration of heparin
Longer duration of heparin use
Surgical (especially cardiac or orthopedic surgery) rather than medical patient
Female sex

HIT usually occurs 4-10 days after the start of heparin therapy. However, in patients with recent prior heparin exposure (especially within the previous month, but even within the past 100 days), persistence of circulating HIT antibodies may result in
rapid-onset HIT, in which the platelet count falls within 24 hours of starting heparin. Onset of HIT may also occur after heparin cessation. Delayed-onset HIT should be considered when a patient presents with thrombosis and unexplained
thrombocytopenia up to 3 weeks after heparin therapy has ended.

Patients with heparin-induced thrombocytopenia (HIT) may present with unusual characteristic sequelae, including the following:

Venous limb gangrene


Bilateral adrenal hemorrhagic infarction
Skin lesions at injection sites
Acute systemic reactions following an intravenous heparin bolus

If heparin-induced HIT is suspected, the first step is to discontinue and avoid all heparin products immediately. Even small doses for catheter flushing should be avoided.

(Option A) HIT is not caused by pneumatic compression; therefore, its discontinuation has no benefit in management of this patient.

(Option B) Warfarin should be avoided in such patients because it can lead to microthrombosis. Interestingly, patients with HIT have an INR of above 4. Warfarin can be started once the platelet count is back to normal. If warfarin has already been
started, vitamin K should be given.

(Option D) Once a Doppler ultrasound has confirms the diagnosis of DVT, venography is not required as an extra measure unless some surgical intervention is planned and an accurate venous map is required.

(Option E) Platelet transfusion should be avoided in HIT, as it may increase the thrombogenic effect. Guidelines from the American College of Chest Physicians suggest limiting platelet transfusions to patients with severe thrombocytopenia, who are
experiencing bleeding or undergoing an invasive procedure with a high risk of bleeding.

Once HIT occurs, alternatives to heparin are direct thrombin inhibitors (DTIs) such as argatroban and the indirect factor Xa inhibitor fondaparinux (Arixtra®).

References

• Medscape - Heparin-Induced Thrombocytopenia

Last updated:
Time spent: QID:671 2023-2-12

636 of 1943
Which of the following is the most profound adverse effect of chronic transfusional iron overload in children with thalassemia?

A. Progressive hepatic cirrhosis.

B. Extramedullary hematopoeisis.

C. Productive cough and shortness of breath.

D. Hemolytic anemia.

E. Autosplenectomy.

Correct
45% answered correctly

Explanation:

Correct Answer Is A

The following are are major complications associated with iron overload:

Cardiac problems:

Congestive heart failure, dysrhythmias and pericarditis

Endocrine problems:

Delay in growth and sexual development, hypoparathyroidism, hypothyroidism and diabetes mellitus

Hepatic problems:

Progressive hepatic fibrosis and cirrhosis


Of the given options, only progressive hepatic cirrhosis could be a complication of iron overload.

Of the options, only progressvie hepatic cirrhosis can be direct complication of iron overload.

(Option B) Extra-medullary hematopoiesis is a feature of thalassemia, not a complication of iron overload.

(Option C) Shortness of breath can be caused only if iron overload has already led to heart failure. It is not a direct effect of iron overload.

(Option D) Hemolysis and anemia is the disease mechanism, not an adverse effect of iron overload.

(Option E) Autosplenectomy is a feature of sickle cell disease and occurs because of splenic infarcts.

NOTE - Splenomegaly is not a common feature in thalassemia. If present, it is more likely to have been caused by congestive heart failure rather than the disease itself.

References

• MJA - Management and clinical outcomes of transfusion-dependent thalassaemia major in an Australian tertiary referral clinic

• Australian Guidelines for the assessment of iron overload and iron chelation in transfusion-dependent thalassaemia major, sickle cell disease and other congenital anaemias. *

Last updated:
Time spent: QID:766 2023-2-12

637 of 1943
A 70-year-old woman presents with abdominal discomfort of 6 months duration. On examination, generalized lymphadenopathy and splenomegaly are noted. Which one of the following could be the most likely diagnosis?

A. Acute lymphoblastic leukemia (ALL).

B. Acute myeloid leukemia (AML).

C. Hodgkin lymphoma.

D. Infectious mononucleosis.

E. Chronic lymphoblastic leukaemia (CLL).

Incorrect. Correct answer is E


45% answered correctly

Explanation:

Correct Answer Is E

The clinical picture is suggestive of CLL as the most likely diagnosis. CLL is massive overproduction of mature yet till leukemic lymphocytes usually from the monoclonal production of B lymphocytes.

CLL can often present as an asymptomatic elevation of white cells, found on routine blood tests or during investigations for other problems. The patients are exclusively older with 90% being over the age of 50 years. Symptoms, when present, are
often nonspecific.

CLL can be associated with various autoimmune phenomena such as thrombocytopenia and autoimmune hemolytic anemia.

Enlarged lymph nodes are the most common presenting symptom (87% of symptomatic patients) at the time of diagnosis. A predisposition to repeated infections such as pneumonia, herpes simplex labialis, and herpes zoster may be noted. Early
satiety and/or abdominal discomfort may be related to an enlarged spleen. Splenomegaly and hepatomegaly are seen in 30-45% and 10-20% of cases, respectively. Thrombocytopenia can cause mucocutaneous bleeding and/or petechiae. Ten
percent of patients with CLL will present with an autoimmune hemolytic anemia.

Richter syndrome or Richter transformation refers to the transformation of CLL into an aggressive large B-cell lymphoma and is seen in approximately 3-10% of cases. Patients will often present with symptoms of weight loss, fevers, night sweats,
muscle wasting, (i.e. B symptoms) and increasing hepatosplenomegaly and lymphadenopathy. Treatment remains challenging and prognosis poor, with median survival in months.

Staging of CLL is as follows:

Stage 0 - lymphocytosis alone (usually between 30,000 – 50,000, but may go as high as 150,000 – there is always lymphocytosis)
Stage 1 - lymphadenopathy
Stage 2 - splenomegaly
Stage 3 - anaemia (Hb <100 g/L)
Stage 4 - thrombocytopenia (<100,000/mm3)

Patients with stages 0 and 1 often have a survival of 10-12 years even without treatment. In contrast, the survival of stage 3 and 4 is only 1-2 years.

Early stage CLL with only an elevated white cell count or enlargement of lymph nodes is not treated; however, patients with symptomatic disease always require treatment. Those with more advanced disease should receive initial therapy with
fludarabin. Chlorambucil may be used as second-line therapy. Chlorambucil decreases the white cell count and symptoms.

Prednisolone is used for treatment of thrombocytopenia and autoimmune hemolytic anemia.

(Options A and B) AML and ALL have rapid progressive courses, and without treatment, the survival rate hardly exceeds 12 weeks. ALL is more commonly seen in children. Presentation is with fatigue, anemia, infections and bleeding caused by
reduction in all 3 cell lines - red blood cells, white blood cells and platelets.

(Option C) Hodgkin disease presents with enlarged, painless, rubbery, non-erythematous, non-tender lymph nodes as the hallmark of the disease. In Hodgkin disease lymphadenopathy is limited to cervical and supraclavicular lymph nodes 80-90% of
the time (unlike non-Hodgkin disease where lymphadenopathy is generalized 80-90% of the time). Drenching night sweats, weight loss of >10% and fever, collectively termed as ‘B symptoms’ are another clinical feature. Pruritus is common. Although
extralymphatic involvement (spleen, liver, bone marrow) can be seen, it is more common with non-Hodgkin lymphoma. The disease has a bimodal age distribution at 20s and 60s.

(Option E) Infectious mononucleosis cannot be the diagnosis because firstly, there is no history of fever or pharyngeal symptoms in the history and secondly, acute phase of infectious mononucleosis (lymphadenopathy, fever, pharyngitis) resolve
within 1-2 weeks; the fatigue however may last for months. With this presentation, infectious mononucleosis is a very unlikely diagnosis.

References

• http://emedicine.medscape.com/article/199313-overv

• http://emedicine.medscape.com/article/197802-overv

• http://emedicine.medscape.com/article/201886-overv

Last updated:
Time spent: QID:776 2023-2-12

638 of 1943
An 83-year-old woman is brought to your attention with complaints of progressive fatigue. On examination, axillary rubbery and non-tender lymphadenopathy is noted. Blood tests show a platelet count of 70,000/mm3, hemoglobin of 70 g/L and WBC
count of 40,000/mm3. Which one of the following would be the most appropriate treatment option for this patient?

A. Palliative care.

B. Prednisolone.

C. Platelet transfusion.

D. Radiotherapy.

E. Chemotherapy.

Incorrect. Correct answer is B


45% answered correctly

Explanation:

Correct Answer Is B

Considering the age of the patient and clinical and laboratory findings, chronic lymphoblastic leukemia (CLL) is the most likely diagnosis.

CLL is massive overproduction of mature, but still leukaemic, lymphocytes usually from the monoclonal production of B lymphocytes.

CLL can often present as an asymptomatic elevation of white cells found on routine bloobsod tests or during investigations for other problems. The patients are exclusively older with 90% being over the age of 50 years. Symptoms, when present, are
often nonspecific, and include:

CLL can be associated with various autoimmune phenomena such as thrombocytopenia and autoimmune hemolytic anemia.

Patients with CLL present with a wide range of symptoms and signs. Onset is insidious, and it is not unusual for CLL to be discovered incidentally after a blood cell count is performed for another reason; 25-50% of patients will be asymptomatic at
the time of presentation.

Enlarged lymph nodes are the most common presenting symptom, seen in 87% of symptomatic patients at the time of diagnosis. A predisposition to repeated infections such as pneumonia, herpes simplex labialis, and herpes zoster may be noted.
Early satiety and/or abdominal discomfort may be related to an enlarged spleen. Splenomegaly and hepatomegaly are seen in 30-45% and 10-20% of cases, respectively. Mucocutaneous bleeding and/or petechiae may be due to thrombocytopenia.
Tiredness and fatigue may be present secondary to anemia; 10% of patients with CLL will present with an autoimmune hemolytic anemia.

Patients with chronic lymphocytic leukemia (chronic lymphoid leukemia, CLL) do not need to be treated with chemotherapy until they become symptomatic or display evidence of rapid progression of disease characterized by the following:

Weakness, night sweats, weight loss, painful lymphadenopathy, or fever

Symptomatic anemia and/or thrombocytopenia (stages III or IV)

Autoimmune hemolytic anemia and/or thrombocytopenia poorly responsive to corticosteroid therapy

Progressive disease, demonstrated by increasing lymphocytosis with a lymphocyte doubling time less than 6 months, and/or rapidly enlarging lymph nodes, spleen, and liver. In contrast, transient localized lymphadenopathy, occurring
in response to localized infections, is not necessarily an indication for treatment

Repeated episodes of infection; hypogammaglobulinemia without repeated episodes of infection is not a clear indication for therapy.

NOTE - Autoimmune mediated anemia or thrombocytopenia is treated with therapy directed at the autoimmune process before treatment of the underlying CLL is initiated.

When chemotherapy is indicated, one of the following regimens is used:

Fludarabine, cyclophosphamide, and rituximab (FCR)


Pentostatin, cyclophosphamide, and rituximab (PCR)
Fludarabine, cyclophosphamide, and mitoxantrone (FCM)
Cyclophosphamide, vincristine, and prednisone (CVP)

As mentioned earlier, corticosteroids such as prednisolone are first-line treatment where autoimmune complications of CLL such as autoimmune thrombocytopenia or hemolytic anaemia occurs. This treatment should precede chemotherapy (if
indicated) when there is thrombocytopenia and/or anaemia.

References

• http://www.uptodate.com/contents/overview-of-the-t

• http://emedicine.medscape.com/article/199313-treat

• http://www.bcshguidelines.com/documents/Revised_CL

Last updated:
Time spent: QID:777 2023-2-12

639 of 1943
Which one of the following is caused by the inflammation of meibomian glands?

A. External hordeolum (stye).

B. Internal hordeolum (meibomian abscess).

C. Pinguecula.

D. Pterygium.

E. Blepharitis.

Incorrect. Correct answer is B


45% answered correctly

Explanation:

Correct Answer Is B

Obstruction, infection (usually by Staphylococcus aureus) and the residual nodule in the meibomian gland is referred to as chalazion also called meibomian cyst or internal hordeolum.

(Option A) External hordeolum (stye) is the infection of a lash follicle. It is usually self-limiting. Application of heat and plucking the affected lash are of much help. Topical antibiotics (e.g. chloramphenicol) may be used especially if the surrounding
tissue is inflamed as well.

(Options C and D) Pinguecula is a benign growth of the conjunctiva. The structure is slightly raised, vascular and usually yellow grey. It can be on either side of the cornea, but it is more common on the nasal side. When the growth extends to cover
the cornea, it is referred to as pterygium. Pterygium (surfer’s eye) is more common in those exposed to dust and wind blow. Pinguecula often does not require treatment except for cosmetic issues. If it becomes inflamed a short course of topical
corticosteroids is advised. Keeping the eye moist and wearning sunglasses prevents inflammation. With progression to pterygium, removal is indicated.

(Option E) Blepharitis is inflammation of the lids caused by Staphylococcus aureus infection, seborrheic dermatitis, or rosacea. It presents with marginal eyelid itching and burning. Management is with removal of crusts from the lids and steroid
drops or creams. For blepharitis associated with Staphylococcus aureus, tetracycline or chloramphenicol ointments is used. Oral doxycycline is indicated in those with blepharitis associated with rosacea.

References

• Therapeutic Guidelines – Antibiotics; available from http://tg.org.au

• http://emedicine.medscape.com/article/1212709-over

Last updated:
Time spent: QID:303 2023-2-12

640 of 1943
A 30-year-old woman, who is 34 weeks pregnant, presents with dysuria and frequency. On examination, she has a temperature of 37.3°C. There is no nausea and vomiting, flank pain or tenderness A urine dipstick test is positive for nitrites and
leucocytes. A mid-stream urine sample is sent for culture and sensitivity. In the meanwhile, which one of the following antibiotics is the medication of choice for empiric therapy in this woman?

A. Cephalexin.

B. Trimethoprim plus folic acid supplementation.

C. Doxycycline.

D. Nitrofurantoin.

E. Ciprofloxacin.

Correct
45% answered correctly

Explanation:

Correct Answer Is A

The clinical manifestation is characteristic of urinary tract infection (UTI). With absence of fever and flank pain, infectious cystitis is more likely. During pregnancy the following antibiotics are considered safe for treatment of UTI:

First line

Cephalexin (category A) 500mg orally, 6-hourly for 10 days

Second line

Nitrofurantoin (category A) 50mg orally, 6-hourly for 10 days

Third line

Amoxycillin + clavulanate (category B1) 500+125mg orally, 12-hourly for 10 days

The above drugs are used for treatment of asymptomatic bacteriuria of pregnancy as well.

(Option B) Trimethoprim is an antagonist of folate and there might be increased potential risk of neural tube defects. Manufacturers advise that this drug be avoided during pregnancy. This drug, however, is the first-line therapy for UTI in non-
pregnant women and adult men.

(Option C) Tetracyclines such as doxycyclin and tetracyclin are contraindicated in pregnancy due to their risk of teratogenicity.

(Option D) Nitrofurantoin is inferior ro cephalexin for treatment of UTI during pregnancy. Moreove, it should be avoided near term due to risk of neonatal hemolysis. It also should be avoided in those with renal impairment because the drug has renal
excretion.

(Option E) Ciprofloxacin is a fluoroquinolone. Although fluoroquinolones are categorized as B1 and are almost safe for use during pregnancy, they should not be considered as first-line medications because they are the only orally-active drugs
available for infections due to Pseudomonas aeruginosa and other resistant bacteria to conventional antibiotics.

641 of 1943
A 32-year-old woman presents to your practice at 28 weeks gestation with recurrent vulval warts previously treated with cryotherapy. Which one of the following options would be the most appropriate management option?

A. Electrocautery.

B. Surgical excision.

C. Cervical screening.

D. Podophyllum.

E. Cryotherapy.

Incorrect. Correct answer is E


45% answered correctly

Explanation:

Correct Answer Is E

Genital warts can be managed either medically or surgically:

Medical management

Podophyllotoxin - 0.15% cream or 0.5% paint topically applied by the patient to each wart, twice daily for 3 days followed by a 4-day break. This should be repeated for 4 to 6 cycles until the warts disappear.

Imiquimod - 5% cream topically applied by the patient to each wart, 3 times per week at bedtime (wash off after 6 to 10 hours) until warts are cleared (usually 12-24 weeks)

Podophyllum - 25% solution in compound benzoin tincture topically applied by the clinician to each wart, washed off after 6 hours (repeat once weekly until the warts disappear)

NOTE - Podophyllum and podophyllotoxin should not be used in pregnancy. Although imiquimod is category B1 in pregnancy, its use is not recommended during the pregnancy due to lack of evidence on its safety.

Surgical management

Overall, surgical management of genital warts is associated with fewer recurrences compared to medical treatment. The surgical options include:

Cryosurgery (cryotherapy) - it is the method of choice for simple warts, and is performed in the office with ot without anesthesia and/or analgesia once in a week until the warts disappear. This method is safe in pregnancy and the preferred option.

Surgical excision - surgical excision of the wart is the method with the least recurrence rate. The procedure is painful and needs anesthesia and/or analgesia. Simple surgical excision with a scalpel, scissors, or curette is used to remove warts,
especially the large genital ones. This method is reserved for refractory or extensive warts. The procedure is usually performed in an outpatient surgical suite.

Electrosurgical modalities (diathermy or electrocautery) - these methods use high-frequency current to cut and coagulate warts. Electrodesiccation with a bipolar needle is most effective with external genital warts. LEEP is primarily used to treat
cervical squamous intraepithelial lesions (SILs) after confirmation with a cervical biopsy but may also be used to remove large external genital warts.

Electrosurgical methods usually require only topical anesthesia and may be employed in an outpatient setting if the appropriate equipment is available. Large unresponsive lesions around the rectum or vulva can be treated with scissor excision of
the bulk of the mass followed by electrocautery of the remaining tissue down to the skin surface. Removal of a very large mass of warts is a painful procedure and is best performed with the patient under either general or spinal anesthesia.

Laser therapy – carbon dioxide laser vaporization is typically used for treatment of refractory disease or extensive warts of the anogenital mucosal warts and is particularly useful in the treatment of periurethral and vaginal warts. It is the treatment
of choice for pregnant women with extensive mucosal lesions.

Since this woman is pregnant, podophyllum and podophyllotoxin should not be used due to the risk of teratogenicity. Imiquimod (Aldara®) is not recommended (however, it can be used after informed consent).

Cryotherapy, surgical excision, laser ablation and electrocautery can all be used safely during pregnancy. If the lesions are readily available and are few in numbers, cryotherapy is the method of choice.

References

• SA Health - Wart Treatment Algorithm

• AAFP - Management of Genital Warts

Last updated:
Time spent: QID:349 2023-2-12

642 of 1943
A 75-year-old man presents to your clinic with cough and rusty-colored sputum for2 days. On examination, he has a blood pressure of 110/85 mmHg, pulse of 125bpm, respiratory rate of 32 breaths per minute and temperature of 38.6°C. Chest X-ray
(CXR) shows consolidation in the right middle and lower lobes. Which one of the following is the most appropriate management?

A. Amoxicillin/clavulanate.

B. Ampicillin.

C. Erythromycin.

D. Intravenous penicillin.

E. Intravenous ceftriaxone.

Incorrect. Correct answer is D


45% answered correctly

Explanation:

Correct Answer Is D

The clinical and CXR findings of this patient are highly suggestive of community acquired pneumonia (CAP).

There are several ways to stratify the risk index for patients with CAP such as CURB-65, SMARTACOP, and Pneumonia Severity Index (PSI).

Pneumonia Severity Index (PSI)

PSI is a validated risk stratification instrument that can help in identifying patients with CAP who can safely be treated with outpatient antibiotics.

A point value is given to a variety of clinical and laboratory parameters. According to the score, every patient is placed into any of the five risk classes namely I, II, III, IV, and V.

Age – 1 point for each year of age


Nursing home resident +10
Gender
Male +10
Female -10
Co-morbid illness
Neoplastic disease +30
Liver disease +20
Congestive heart failure +10
Cerebrovascular disease +10
Renal disease +10
Physical examination findings
Altered mental status +20
Systolic blood pressure<90mmHg +20
Respiratory rate≥30/min +20
Temperature<35°C or ≥40°C +15
Pulse≥125bpm +10
Laboratory and radiographic findings
Arterial PH<7.35 +30
BUN≥30mg/dl (11mmol/L) +30
Sodium<130mEq/L +20
Glucose≥250mg/dl (14mmol/L) +10
PaO2<60mmHg or O2saturation<90% +10
Hematocrit<30% +10
Pleural effusion +10

Points Class Risk of 30-day mortality


0-50 I 0.1 %
51-70 II 0.6%
71-90 III 0.9%
91-130 IV 9.3%
131-195 V 27%

The first in management of a patient with CAP is to quickly assess whether the patient is in class I or other classes. The patient is in class I if:

Age ≤50 year +


NO HISTORY OF: NO CLINICAL SIGN OF:

Neoplastic disease Acutely altered mental status


Liver disease Respiratory rate≥30 per minute
Congestive heart failure Systolic blood pressure<90mmHg
Cerebrovascular disease Temperature <35°C or ≥40°C
Renal disease Pulse rate≥125 per minute

in presence of even one of the above risk factors, laboratory tests including full blood count, serum electrolytes and creatinine, ABG (or VBG) analysis should be conducted to calculate the PSI and plan further management.

All patients with PaO2<60mmHg or oxygen saturation <94% should be admitted regardless of their PSI class.

643 of 1943
Class I and II
Treat as outpatient
Oral amoxycillin

If treatment for atypical pneumonia (e.g. Chlamydia pneumophila, mycoplasma pneumoniae, etc) is intended ADD:

Doxycycline (oral)

Or

Clarythromycin or roxithromycin (oral)

Class III and IV


Treat as inpatient – consider hospital-in-the-home care for selected cases

Non-tropical regions (and tropical regions with no risk factors)

Benzylpenicillin (intravenous)

Or

Amoxycillin or ampicillin (intravenous)

PLUS

Doxycycline (oral)

Or

clarythromycin or roxithromycin (oral)

if Gram-negative bacilli are identified in sputum or blood, either add gentamicin to the above regimen or use ceftriaxone/cefotaxime instead of benzylpenicillin or amoxycillin/ampicillin

Tropical regions with risk factors

Ceftriaxone (intravenous)

PLUS

Initial single dose of gentamicin

Class V
Treat as inpatient – consider hospital-in-the-home care for selected cases

Non-tropical regions

Azithromycin (intravenous)

Or either of the following:

Erythromycin (intravenous) + ceftriaxone (intravenous)

Erythromycin (intravenous) + cefotaxime (intravenous)

Erythromycin (intravenous) + benzylpenicillin (intravenous) + gentamicin (intravenous)

Tropical regions (all patients)

Meropenem (intravenous)

Or

Imipenem (intravenous) + either erythromycin (intravenous), or azithromycin (intravenous)

This patient is 75 years old (+75 scores), is male (+10) and has a pulse rate of 125 mmHg (10+). Based on PSI, he has a total score of 95, and is calssified as class IV with a 30-day mortality risk of 9.1%.

For him, the antibiotics of choice isbenzyl penicillin. If the patient is penicillin allergic (rash, but not anaphylactic shock), intravenous ceftriaxone is considered.

TOPIC REVIEW

644 of 1943
SMARTACOP is another validated assessment tool for pneumonoa severity. SMARTACOP score is calculated based on the presence of different clinical, laboratory and CXR findings as the following table:

Clinical/radiological finding Score


Systolic blood pressure<90mmHg 2
Multi-lobar involvement on CXR 1
Albumin level<35g/L 2
Respiratory rate raised 1
Tachycardia≥125/min 1
Aboriginal or Torres Strait islander 1
Confusion (new onset) 1 AMTS<8
Oxygen Low 2 PaO2<60mmHg or O2 saturation<90%
pH (arterial<7.35) 2
Interpretation of total score
0-2 Low risk of needing IRVS*
3-4 Moderate risk (1 in 8) of needing IRVS
5-6 High risk (1 in 3) of needing IRVS
≥7 Very high risk (2 in 3) of needing IRVS
*intensive respiratory/vasopressor support

References

• Therapeutic Guidelines – Antibiotic; available from http://tg.org.au

• https://www.mja.com.au/journal/2010/192/3/pneumoni

Time spent: QID:356 Last updated:


2023-2-12

645 of 1943
A 60-year-old man presents to the emergency department with a 3-day history of increased sputum production, fever and progressive shortness of breath. He has had productive cough with greenish sputum for the past 3 years. He explains that he
coughs up of a cupful of green sputum every morning. A chest X-ray is obtained showing consolidation of the right lower lobe. Which one of the following would be the most appropriate treatment option?

A. Doxycycline.

B. Roxithromycin.

C. Ticarcillin-clavulanate.

D. Amoxicillin-clavulanate.

E. Azithromycin.

Incorrect. Correct answer is C


45% answered correctly

Explanation:

Correct Answer Is C

The current clinical presentation, as well as consolidation of the right lower lobe is suggestive of lobar pneumonia. The 3-year history of the chronic cough and copious amount of purulent sputum indicates that this patient has bronchiectasis as well.
Bronchiectasis is permanent dilation of the bronchi and bronchiols characterized by recurrent or persistent bronchial infection and cough.

Patients with bronchiectasis often have chronically purulent sputum. Sputum cultures in such patients may reveal organisms such as Hemophilus influenza, Streptococcus pneumoniae, Staphylococcus aureus, Branhamella catarrhalis, and
Pseudomonas aeruginosa. The presence of Pseudomonas aeruginosa in the airways of these patients is generally associated with more severe disease. Colonization of the airway with these organisms is an indication for antibiotic treatment.
Exacerbations of bronchiectasis empirically treated with amoxicillin. Ciprofloxacin is used if the causative organism is found to be Pseudomonas aeruginosa.

Every patient with pneumonia and an underlying lung pathology, such as cystic fibrosis or bronchiectasis, that makes Pseudomonas aeruginosa a likely causative organism should be initially treated with either of the following antibiotic regimens:

Ticarcillin + clavulanate
Piperacillin + tazobactam
Cefotaxime
Ceftriaxone
Gentamicin

Any patient with history of chronic bronchiectasis who develops pneumonia should be admitted to the hospital for intravenous antibiotics.

Macrolides such as roxithromycin or azithromycin do not cover Hemophilus influenza and are not options for treatment of infectious exacerbation of bronchiectasis.

Amoxicillin clavulanate or amoxicillin can be used for exacerbations of bronchiectasis where pneumonia is not a concern.

References

• Therapeutic Guidelines – Respiratory; available from http://tg.org.au

• Therapeutic Guidelines – Antibiotic; available from http://tg.org.au

Last updated:
Time spent: QID:357 2023-2-12

646 of 1943
As part of a health check, an immigrant Sudanese woman is found to have a positive tuberculin skin test (TST). A chest X-ray is obtained with inconclusive results. Sputum microscopy and culture are positive for tuberculosis (TB). She is planned to
be started on four-drug therapy. She has a 4-year-old son and you make arrangements for him to have an interferon gamma release assay (IGRA) which turns out positive. The child is healthy and has no symptoms. Which one of the following would
be the next best step in management?

A. No further action is required.

B. Start him on isoniazid.

C. Repeat the IGRA for the child.

D. Start him on 4-drug regimen.

E. Obtain a chest X-ray.

Incorrect. Correct answer is E


45% answered correctly

Explanation:

Correct Answer Is E

This child is has been exposed to active TB infection and has a positive IGRA, but is asymptomatic. By definition, this child has latent tuberculosis infection (LTBI). LTBI means the patient has been infected with Mycobacterium tuberculosis (MTB),
but does not have active tuberculosis. Unlike patients with active TB, those with LTBI cannot transmit the MTB to others. Generally, for patients with LTBI, there is a 10% lifetime risk of developing active TB ( 5% in the first 2 years after infection and
0.1% per year thereafter).

To diagnose the latent TB, 2 tests can be used:

Tuberculin skin test (TST), also known as Mantoux test


Interferon gamma release assays (IGRA)

NOTE - Australia's National Tuberculosis Advisory Committee suggests TST as the preferred method of testing, with IGRA as a supplemental assay in patients older than 2 years. IGRA is easier to interpret in patients who are vaccinated with BCG or
are immunocompromised.

TST (Mantoux)

TST includes assessment of the skin inflammation 48-72 hours after intradermal (not subcutaneous) injection of 0.1ml of tuberculin protein (purified protein derivative or PPD). The diameter of the induration (not the erythema) measured across the
arm (not along it) gives a semi-quantative assessment of the likelihood of LTBI.

TST is considered positive if the induration is ≥5mm in the following groups:

Recent high risk (close) contacts of persons with infectious TB


Indivuduals with organ transplants or immune suppressive therapy equivalent to prednisone >15mg/day for >1 month
Individuals with HIV infection
Individuals with CXR evidence of past untreated TB

TST is considered positive if the induration is ≥10 mm in the following groups:

Individulas born or resident (for greater than 3 months) in countries with high prevalence of TB (>100 cases/100,000)
Children < 4 years of age without any identified risk factors
Individuals who live or spend time in high risk congregate settings (e.g. prisons, homeless shelters, alcohol rehabilitation and drug treatment centers
Health care workers without prior BCG vaccination in the past 10 years
Intravenous drug users

TST is considered positive if the induration is ≥15 mm in the following groups:

Normal population other than the mentioned groups.

Factors that influence interpretation of TST are listed in the following table:

Factors that may DECREASE skin reaction and give false-negative results
Infections:

Viral e.g. HIV, measles, mumps, chickenpox, etc


Bacterial e.g. pertussis, brucellosis, leprosy, etc
Fungal

Live virus vaccination e.g. measles, mumps, polio, etc


Metabolic diseases e.g. chronic renal failure
Malnutrition/ protein depletion
Lymphoid neoplasms e.g. Hodgkin’s disease, lymphoma, CML

647 of 1943
Sarcoidosis
Drugs e.g. corticosteroids, immunosuppressants
Age – newborn and elderly
TB infection acquired within the past 8 weeks
Other causes leading to cell-mediated immune response suppression
Local skin damage e.g. trauma, dermatitis, surgery
Incorrect storage and handling of tuberculin (PPD)
Poor technique of injection and misreading
Factors that may INCREASE skin reaction and give false-positive results
Exposure to or infection with non-tuberculosis mycobacteria
Past BCG vaccination
Trauma and irritation to the site of injection prior to reading
Misreading

Interferon Gamma Release Assay (IGRA)

Unlike TST, IGRA is unaffected by previous BCG vaccination. A positive IGRA suggests that the patient’s immune system recognizes MTB antigens due to either current infection or a past infection. A negative test cannot exclude active TB because
sensitivity is not 100%

NOTE - TST remains the preferred method with IGRAs as a supplemental assay in subjects more than 2 years of age. TST alone is recommended for those ≤2 years of age.

If a TST or IGRA is positive, active disease must be excluded with chest X-ray as the next best step in management. If the chest X-ray is normal, active TB is excluded and the patient should only be treated for LTBI with INH (and pyridoxine) for 6-9
months.

If the chest X-ray shows abnormalities, sputum should be examined for acid-fast bacilli (AFB) stain and culture. An abnormal chest X-ray and 3 negative sputum smears excludes active TB, and the patient should only be treated for LTBI with INH
(300mg/day) and pyridoxine for 6-9 months.

With even one positive sputum exam, the patient isconsidered to have active TB and is treated with standard full-dose 4-drug therapy after discussion with and supervision of a clinician experienced in TB management.

References

• RACGP - Tuberculosis testing

• Australian Prescriber - Testing for tuberculosis

• Therapeutic Guidelines – Antibiotics: available from http://tg.org.au

Time spent: QID:360 Last updated:


2023-2-12

648 of 1943
On a routine health evaluation for immigration, a 45-year-old Asian man is found to have a positive tuberculin skin test (TST). He denies cough, fever, sputum, or weight loss. A complete physical examination is normal. A chest X-ray is obtained that is
shown in the following photograph. Which one of the following is the next best step in management of this patient?

A. Start him on quadruple therapy for active TB.

B. Start him on monotherapy with isoniazid for 9 months.

C. Obtain sputum sample for acid fast bacilli (AFB) stain and culture.

D. Repeat the chest X-ray in 6 months.

E. Repeat the TST in 2 weeks.

Incorrect. Correct answer is C


45% answered correctly

Explanation:

Correct Answer Is C

Latent tuberculosis infection (LTBI) means the patient has been infected with Mycobacterium Tuberculosis (MTB), but does not have active tuberculosis. Unlike patients with active TB, those with LTBI cannot transmit the MTB to others. Generally, for
patients with LTBI there is a 10% lifetime risk of developing active TB ( 5% in the first 2 years after infection and 0.1% per year thereafter).

To diagnose the latent TB, 2 tests can be used:

Tuberculin skin test (TST), also known as Mantoux test


Interferon gamma release assays (IGRA)

NOTE - Australia's National Tuberculosis Advisory Committee suggests TST as the preferred method of testing, with IGRA as a supplemental assay in patients older than 2 years. IGRA is easier to interpret in patients who are vaccinated with
BCG or are immunocompromised.

A positive TST or IGRA indicates that the patient has either active or latent Mycoplasma tuberculosis (MTB) infection. In such cases, the next best step in management is obtaining a chest X-ray. If the chest X-ray is normal, active TB is excluded and
the patient should only be treated for LTBI with INH (and pyridoxine) for 6-9 months.

If the chest X-ray shows abnormalities, sputum should be examined for acid-fast bacilli (AFB) stain and culture. An abnormal chest X-ray and 3 negative sputum smears excludes active TB, and the patient should only be treated for LTBI with INH
(300mg/day) and pyridoxine for 6-9 months. However, with even one positive sputum exam, the patient should be thought as having active TB and started on the standard full-dose four-drug therapy after discussion with and supervision of a clinician
experienced in TB management.

This patient has a positive TST. Furthermore, his chest X-ray shows opacities in the apex of the right lung, consistent with TB infection. With the abnormal chest X-ray, the next best step would be obtaining three samples for acid FAST stain and
culture.

(Option A) Starting the patient on four-drug therapy is the correct option if the patient has active TB. Unless there is at least one positive sputum sample for TB, the provisional diagnosis is LTBI and not active TB infection; therefore, quadruple therapy
is not necessary.

(Option B) Although monotherapy with isoniazid for 6-9 months is used for treatment of LTBI, this patient might have active TB infection. The decision as to whether this patient should be treated for active or latent TB cannot be made until sputum
exam results are available.

(Option D) Repeating the chest X-ray in 6 months is not appropriate because this patient has a positive TST result and chest X-ray findings consistent with current or previous TB infection. Action should be taken for further assessment and prompt
treatment of LTBI or active TB based on assssment results. Repeating the chest X-ray adds no benefit and unnecessarily delays treatment.

(Option E) A TST should never be repeated once it is positive.

References

• RACGP - Tuberculosis testing

• Australian Prescriber - Testing for tuberculosis

• Therapeutic Guidelines – Antibiotics: available on: http://tg.org.au

Last updated:
Time spent: QID:361 2023-2-12

649 of 1943
A 47-year-old woman presents to your practice with complaints of fever, cough and hemoptysis. She is a nurse working in a rural area and has smoked 15 cigarettes a day for the past 20 years. She also has lost eight kg in the past three months. Her
physical examination is unremarkable. A chest X-ray is obtained and shown in the following photograph. Which one of the following is the most appropriate next step in management?

A. Obtain sputum sample for Gram stain.

B. Bronchoscopy and lavage.

C. Obtain sputum sample for Zihel - Neelsen stain.

D. High-resolution CT scan of the chest.

E. Start the patient on full-dose four-drug therapy for tuberculosis.

Incorrect. Correct answer is C


45% answered correctly

Explanation:

Correct Answer Is C

Clinical findings of fever, cough, hemoptysis, and weight loss, in the presence of a lesion seen in the upper lobe of the right lung is highly suggestive of pulmonary tuberculosis (TB). This patient should be considered to have TB unless proven
otherwise. When TB is suspected, the next step will be obtaining three sputum samples for acid-fast (Zihel-Neelsen) bacilli staining and culture.

(Option A) Gram-stain does not show TB bacilli and is not used for diagnosing TB.

(Option B) Bronchoscopy and lavage for obtaining samples are indicated if traditionally-obtained samples are negative but, based on clinical grounds, TB is a likely diagnosis.

(Option D) High resolution CT scan of the chest has no role in diagnosis of TB and is not indicated unless the diagnosis is not certain or other diagnoses than TB are considered..

(Option E) Starting the patient on full-dose 4-drug therapy for tuberculosis is indicated in this patient due to high possibility of TB as the diagnosis, but it should be delayed until samples are taken because antibiotics, if used prior to testing, may alter
test results, mostly in form of false negativity.

References

• RACGP - Tuberculosis testing

• Australian Prescriber - Testing for tuberculosis

• Therapeutic Guidelines – Antibiotics: available from http://tg.org.au

Time spent: QID:362 Last updated:


2023-2-12

650 of 1943
A 25-year-old Asian woman with no known tuberculosis contact has a positive Tuberculin skin test (TST) in a workup for immigration. A chest X-ray is performed and is completely normal. She has no cough, fever or sputum. Physical examination is
unremarkable. Which one of the following is the next best step in management?

A. BCG vaccination.

B. Isoniazid for 9 months.

C. Repeat the chest X-ray in 6 months.

D. Repeat the TST in 2 weeks.

E. Isoniazid for 2 months.

Incorrect. Correct answer is B


45% answered correctly

Explanation:

Correct Answer Is B

Every person with a positive tuberculin skin test (TST), also known as Mantoux test, or interferon gamma release assay (IGRA), also known as Quantiferon-TB Gold test, should undergo a full investigation for active TB infection, including chest X-ray
and sputum smear for acid-fast bacilli (AFB) and culture. If the diagnosis of active disease is established, the patient should receive full four-drug therapy with isoniazid, ethambutole, pyrizinamide, and rifampicin.

If the active disease is ruled out by history, physical exam and a negative chest X-ray, latent TB infection (LTBI) should be considered as the diagnosis. LTBI denotes infection with mycobacterium tuberculosis without any active disease. Ninety
percent of patients with LTBI remain asymptomatic all their lives; however, 5-10% may develop active disease. This is why monotherpay with isoniazid for 6 to 9 months is required for such patients. Supplementation with pyridoxine is necessary
throughout the treatment period. Patients with LTBI cannot transmit the disease.

A positive chest X-ray indicates either healed TB with residual traction scars or active TB infection. For this reason, if an abnormal chest X-ray is the only finding in addition to a positive TST or IGRA, three sputum samples should be obtained for acid
fast bacilli (AFB) stain and culture. Bronchoalveolar lavage may be considered if there is high index of suspicion despite negative sputum results.

If all three samples are negative, active TB infection is excluded, and the patient should be treated for LTBI. With even one positive sample in the presence of chest X-ray abnormalities, active TB infection is the definite diagnosis. Any positive result
needs to be discussed with a clinician experienced in TB management before starting any treatment.

(Option A) It is accepted that in immune-competent neonates and infants, BCG vaccine reduces the likelihood of LTBI progressing to active TB infection, or at least it decreases the chances of a more severe outcome if the active infection
occurs. Such benefit in older age groups is less clear. Generally, BCG vaccine is not recommended for general use in adult patients.

(Option C) With a chest X-ray positive for TB, either healed TB or active infection, there will be no need for repeating it, as this will not affect the diagnosis or the management plan.

(Option D) Repetition of TST is not recommended as retest is associated with inaccurate and misleading results due to sensitization of immune system response by the previous test.

(Option E) Treatment of LTBI requires at least 6 months of isoniazid. Two month is not an effective duration for therapy.

References

• RACGP - Tuberculosis testing

• Australian Prescriber - Testing for tuberculosis

• Therapeutic Guidelines – Antibiotics: available on: http://tg.org.au

Last updated:
Time spent: QID:363 2023-2-12

651 of 1943
Which one of the following is not a feature of mycoplasma pneumonia?

A. Headache.

B. High fever.

C. Pleuretic chest pain.

D. Cough.

E. Myalgia.

Incorrect. Correct answer is B


45% answered correctly

Explanation:

Correct Answer Is B

Generally, acute mycoplasma pneumonia presents as tracheobronchitis, reactive airway disease, wheezing, or a non-specific upper respiratory symptoms.

The prodromal phase is characterized by headache, malaise, myalgia and mild fever. Fever more than 39°C is rare in mycoplasma infection and should raise suspicion against another cause.

Cough in mycoplasma is usually incessant and often non-productive (dry) to mildly productive. Sputum discoloration may be seen late in the course of the disease.

Pleural effusion is seen in 15-20% of patients and is transudate in nature in most of the time. As a rule, pleural effusion caused by pneumonia (parapneumonic effusion) should always be aspirated to prevent empyema.

Pleuretic chest pain and empyema formation are other problems seen in patients with mycoplasma pneumonia.

References

• http://emedicine.medscape.com/article/1941994-clin

Last updated:
Time spent: QID:404 2023-2-12

652 of 1943
A 27-year-old man is seen in the emergency department with complaints of dry cough, fever, dyspnea, and diarrhea, which began 4 days after an upper respiratory tract infection. On physical examination, he has a respiratory rate of 22 breaths per
minute and temperature of 41°C. On auscultation, bilateral basal crackles are noted. Chest X-ray is remarkable for bilateral interstitial infiltrates. Which one of the following is the most appropriate initial therapy for this patient?

A. Amoxicillin/clavulanate.

B. Penicillin.

C. Azithromycin.

D. Ciprofloxacin.

E. Trimethoprim-sulfamethoxazole.

Incorrect. Correct answer is C


45% answered correctly

Explanation:

Correct Answer Is C

This man has the characteristic features of an atypical pneumonia. The most common causative organism in atypical pneumoniae is mycoplasma pneumoniae, but in infections with this organism the fever is unlikely to exceed 39°C.

Signs and symptoms of atypical pneumonia in addition to high fever and gastrointestinal problems make infection with legionella pneumophila pneumonia (legionnaires’ disease) the most likely diagnosis. In the presence of symptoms and the
absence of tests such as PCR, or a positive gram satin, the next best step in management of patient with community acquired pneumoniae is starting empiric therapy that covers atypical pneumonias as well. Macrolides (e.g. azithromycin) are
antibiotics of choice for all atypical pneumonias, including legionnaires disease. They also may cover streptococcus pneumonia.

References

• http://emedicine.medscape.com/article/220163-clini

• http://emedicine.medscape.com/article/220163-overv

Last updated:
Time spent: QID:405 2023-2-12

653 of 1943
A 24-year-old pregnant woman, who is 19 weeks pregnant, presents to your office complaining of frequency, urgency and dysuria. Further investigations establish the diagnosis of urinary tract infection. The culture results are still pending. The patient
has history of allergic reaction to penicillin manifesting as a rash. Which one of the following would be the antibiotic of choice for treating this patient?

A. Amoxicillin.

B. Clarithromycin.

C. Aminoglycosides.

D. Tetracyclines.

E. Cephalexin.

Incorrect. Correct answer is E


45% answered correctly

Explanation:

Correct Answer Is E

The antibiotic of choice for empirical treatment of urinary tract infection (UTI) in pregnancy is cephalexin. Nitrofurantoin and amoxicillin-clavulanate are second- and third-line medications, respectively. Patients allergic to penicillin manifested as a
rash can be still rather safely treated with cephalexin. If the presentation of allergy to penicillin was anaphylaxis, cephalosporins were not recommended, and the patient should be treated with nitrofurantoin.

5
NOTE - Asymptomatic bacteriuria (>10 colony count in urine culture of an asymptomatic woman) in pregnancy should also be treated with a one week course of antibiotics. Urine culture should be repeated 48 hours after completion of treatment to
confirm resolution of the infection.

(Option A) Amoxycillin (without clavulanate) is only recommended if susceptibility of the organism is proven.

(Option B) Macrolides e.g. clarithromycin are not recommended for treatment of UTI.

(Option C) Aminoglycosides are category D and should not be used during pregnancy, unless there is severe gram negative sepsis.

(Option D) Tetracyclins are contraindicated in pregnancy due to their potential teratogenic effects.

654 of 1943
A 6-year-old boy is brought to your practice with fever and cough for 7 days. On examination, he has a blood pressure of 100/70 mmHg, pulse rate of 110 bpm, respiratory rate of 22 breaths per minute and a temperature of 37.7°C. There is no
respiratory distress. Lung auscultation reveals bilateral crackles at lung bases. Chest X-ray is performed that is remarkable for bilateral patchy infiltrates more prominent in the left upper lobe. Blood test is done and the results are as follow:

Hb 130g/L (120-150)
WCC 14x109/L (4-11)
Neutrophils 6.5x 109/L (2.0-7.5)
Lymphocytes 3.7x109/L (1.5-4.0)
Platelets 180x109/L (150-400)

Which one of the following is the most likely diagnosis?

A. Streptococcus viridians.

B. Mycoplasma pneumoniae.

C. Adenovirus.

D. Respiratory syncytial virus.

E. Hemophilus influenza.

Incorrect. Correct answer is B


45% answered correctly

Explanation:

Correct Answer Is B

The clinical findings on exam and radiological findings of diffuse patchy infiltrates are highly suggestive of atypical pneumonia. Mycoplasma pneumoniae is the most common causative organism in this age group (school-aged children).

NOTE - Viral pneumonia is the most common cause of atypical pneumonia in children younger than 2 years.

Patients with mycoplasma pneumoniae usually experiences a prodromal phase characterized by headache, malaise, myalgia and mild fever. Fever more than 39°C is rare in mycoplasma infection and should always raise suspicion against another
cause. More specific manifestations include:

Cough – nonproductive to mildly productive and sometimes sputum discoloration late in the course of the disease. Cough is the most common presenting sign
Pleural effusion - 15-20% of patients.
Pleuretic chest pain.
Empyema.
Usually the patient is not very ill

Mycoplasma pneumonia has a more prolonged course, compared to viral pneumonia.

Like other pulmonary diseases, chest X-ray is the best initial diagnostic test. In atypical pneumoniae including mycoplasma pneumoniae, the chest X-ray is usually normal in the beginning but will show diffuse patchy infiltrates after 72 hours.

Mycoplasma serology is used if the diagnosis is uncertain. IgM antibody level is usually elevated especially after the first week of the disease.

References

• http://emedicine.medscape.com/article/1941994-over

• Therapeutic Guidelines - Respiratory; available on: http://tg.org.au

• AMC Handbook of Multiple Choice Questions – page 543

Time spent: QID:406 Last updated:


2023-2-12

655 of 1943
A 57-year-old man presents to your GP practice with progressive cough, fever and malaise for the past 3 months. His background history includes smoking for 20 years and working in pottery industry for 30 years. Five years ago, he developed
shortness of breath, for which he sought medical care and was diagnosed with silicosis. Which one of the following can be the cause for his recent condition?

A. Bronchogenic carcinoma.

B. Tuberculosis.

C. Chronic obstructive pulmonary disease.

D. Mesothelioma.

E. Pneumonia.

Incorrect. Correct answer is B


45% answered correctly

Explanation:

Correct Answer Is B

People who have chronic exposure to silica (miners, quarry workers, potters, etc.) may develop silicosis, which is an occupational interstitial lung disease. Pulmonary silicosis presents with exertional dyspnea due to progressive fibrosis of the lung
parenchyma. Typical radiologic findings vary based on the severity and course of the disease and may include small nodular opacities (<1 cm) in the upper lung zones and reticular changes and honey-combing (a late finding). Similar to all interstitial
lung diseases, spirometry will show an intrapulmonary restrictive pattern.

Patients with silicosis are at significantly-increased risk of contracting pulmonary tuberculosis, which presents with additional symptoms of chronic fever and cough, malaise, weight loss, or night sweats.

(Option A) Although bronchogenic carcinoma is another possibility in heavy smokers with exposure to silica, it is not usually associated with fever and malaise. With fever and malaise and the history of silicosis, TB is more likely than bronchogenic
carcinoma.

(Option C) Smoking history makes chronic obstructive pulmonary disease (COPD) another possibility, but it does not cause fever and malaise unless an infection superimposes the condition. It is unlikely for a COPD patient to have lung infection as
the first presentation of the disease.

(Option D) Mesothelioma is a very rare malignancy of the pleura and is associated with asbestosis. It presents with pleuretic chest pain, pleural effusion and pleural thickening and calcification on imaging.

(Option E) With symptoms persisting for three months, pneumonia is unlikely.

References

• International Journal of Tuberculosis Lung Disease - Silica, silicosis, and tuberculosis

• Medscape - Silicosis

Last updated:
Time spent: QID:411 2023-2-12

656 of 1943
Which one of the following conditions is likely to cause the oral lesions shown in the accompanying photograph in a 67-year-old man?

A. Human immunodeficiency virus (HIV).

B. Use of systemic corticosteroids.

C. Diabetes mellitus.

D. Chronic xerostomia.

E. All of the above.

Incorrect. Correct answer is E


45% answered correctly

Explanation:

Correct Answer Is E

The photograph shows the typical presentation of oral Candidiasis (thrush). Candida albicans colonizes the oral cavity in 60-75% of population, but the infection often occurs in those with immunodeficiency. Oral Candida infection presents with
tender white or yellowish patches overlying an erythematous mucosa. These patches are easily scraped off, leaving only an underlying red patch.

Risk factors to development of oral Candidiasis include (but not limited to):

Immunodeficiency
Cytotoxic therapy
Broad spectrum antibiotics
Corticosteroids including inhaled corticosteroids
Diabetes mellitus
HIV
Debility
Anemia (iron, folic acid, or B12 deficiency)
Dentures
Chronic xerostomia

All of the given options can predispose to oral Candidiasis.

References

• Murtagh’s General Practice – McGraw Hill – 5th Edition – page 776

• http://www.uptodate.com/contents/overview-of-candi

Last updated:
Time spent: QID:467 2023-2-12

657 of 1943
A 20-year-old man presents to the Emergency Department with acutely painful scrotal swelling for the past 12 hours. Evaluation establishes the diagnosis of epidiymoorchitis. Which one of the following is the most likely causative organism?

A. E.cloi.

B. Staphylococcus aureus.

C. Chlamydia.

D. Neisseria gonorrhea.

E. Treponema pallidum.

Incorrect. Correct answer is C


45% answered correctly

Explanation:

Correct Answer Is C

Causative organism in epididymo-orchitis varies based on the age of the patient:

Children and men older than 35 years:

Urinary coliforms (e.g., E coli, Pseudomonas species, Proteus species, Klebsiella species) are the most common causes. Less frequently, pathogens such as Ureaplasma urealyticum, Corynebacterium species, Mycoplasma species, and Mima
polymorpha have also been isolated. Systemic Hemophilus influenzae and Neisseria meningitides infections are rare yet possible causes.

Sexually active men younger than 35 years:

Chlamydia is the most common cause in sexually active men younger than 35 years (accounting for up to 50% of cases, although laboratory evidence of Chlamydia may be absent in up to 90% of cases). Infections with Neisseria gonorrhea,
Treponema pallidum, Trichomonas species, and Gardnerella vaginalis also occur in this population.

In this sexually active man, who is younger than 35 years, Chlamydia would be the most likely causative organism.

References

• RACGP - AFP - Acute scrotal pain

• Medscape - Epididymitis

Last updated:
Time spent: QID:532 2023-2-12

658 of 1943
A 69-year-old woman presents to your clinic with complaint of vaginal discharge. As a part of workup you perform a urine PCR test for gonorrhea and Chlamydia. The result is positive for gonorrhea. You tell her about the results and that gonorrhea is
a sexually transmissible disease. She mentions that she has not had any sexual relationship with anyone whatsoever. Which one of the following will be the next best step in management?

A. Repeat the PCR test.

B. Give a single dose of ceftriaxone.

C. Give a single dose of azithromycin.

D. Arrange for hysteroscopy.

E. Counsel her about treatment.

Incorrect. Correct answer is E


45% answered correctly

Explanation:

Correct Answer Is E

A physician should be familiar with current STI screening recommendations, and selectively apply STI screening tests.

Currently, a nucleic acid amplification test (NAAT) such as PCR is recommended as the optimal method for the diagnosis of genital tract infections caused by Neisseria gonorrhea and Chlamydia trachomatis in men and women with or without
symptoms. Samples for NAATs can be collected from vagina or endocervix, urine (male and female) or urethra (only men).

Compared with culture, commercially available NAATs offer rapid results (within hours) and are generally more sensitive than culture. The main drawback of NAAT is the fact that no antibiotic susceptibility is provided. Cultures are only recommended
when drug resistance is suspected.

In situations where positive test results are not supported by sexual history or clinical findings (such as in this scenario), retesting the patient with a different NAAT or culture is recommended. A decision to delay treatment until retesting confirms the
diagnosis should be made in consultation with the patient. Patients should be counselled about prompt treatment after a positive screening test because an additional test might be falsely negative. Since the treatment of Chlamydia trachomatis and
Neisseria gonorrhea is safe and relatively inexpensive, the patient might choose to be treated empirically without further testing or wait for the repeated test result.

References

• Australian Government - Health Department - Gonorrhoea

Last updated:
Time spent: QID:590 2023-2-12

659 of 1943
A 60-year-old engineer was admitted to the hospital with complaints of fever, cough and pleuretic chest pain. On admission, he had a fever of 40°C. Physical exam and chest X-ray revealed right lower lobe pneumonia. Sputum was obtained for smear
and culture that wa positive for pneumococcus. He was started on intravenous penicillin. After a few days, fever and leukocytosis subsided and chest X-ray showed clearing of the lobar involvement. On the 7th day of admission, his temperature
spikes to 39.4 °C and his dyspnea and cough worsens. A chest X-ray shows an increase in the pulmonary infiltrate. Which one of the following is the most likely explanation to this clinical picture?

A. Development of penicillin-resistant pneumococcal infection.

B. Laboratory contamination of the original culture.

C. Superinfection by a different type of bacteria.

D. Serum sickness as an adverse effect of antibiotic therapy.

E. Pulmonary thromboembolism.

Incorrect. Correct answer is C


45% answered correctly

Explanation:

Correct Answer Is C

After initial response to antibiotic therapy, this patient has developed a high fever again, as well as respiratory symptoms of cough and dyspnea. A repeated chest X-ray confirms the presence of increased pulmonary infiltrates. With this picture,
superinfection with some other germs not sensitive to penicillin (e.g. gram negative bacilli, methicillin-resistant staph aureus, etc) would be the most likely diagnosis.

(Option A) As a rule, reappearance of fever during antibiotic therapy is almost never due to development of drug resistance, rather it suggests infectious complications.

(Option B) Laboratory contamination of the original culture would have shown other organism, not pneumococcus in isolation.

(Option D) Although serum sickness caused by a type III hypersensitivity reaction to penicillin can cause high spiking fevers between 7-14 days of treatment, absence of features such as arthralgia/arthritis and the skin rash that are almost always
present in serum sickness syndrome make this diagnosis unlikely. Drug fever, as the name implies, is fever caused by administration of certain drugs. Drug fever due to an antimicrobial agent can cause clinical confusion because the return of fever
in an infected patient who has been started on antimicrobial treatment may be interpreted as relapse of the original infection. Unless drug fever is considered with certainty, investigation should follow for infection rather than relating the fever to drug
fever. If the cause is found to be drug fever, the drug should be changed.

(Option E) Pulmonary embloism presents with pleuretic chest pain, dyspnea and tachycardia. Spiking fever is not an expected of pulmonary embolism.

References

• http://www.uptodate.com/contents/drug-fever

• Infectious Diseases - Sherwood L. Gorbach

• John G. Bartlett

• Neil R. Blacklow Lippincott Williams & Wilkins

• 2004 – page 56

• European Respiratory Monograph 63: community acquired pneumonia – pages 211-212

Last updated:
Time spent: QID:710 2023-2-12

660 of 1943
Based on the culture results of a 42-year-old man with infective endocarditis, hemophilus ducreyi is found to be the causative organism. He is treated with a course of intravenous ceftriaxone. Which one of the following would be the most
appropriate investigation in addition to treatment?

A. Colonoscopy.

B. Abdominal CT scan.

C. CT angiography.

D. Trans-thoracic echocardiography (TTE).

E. Trans-esophageal echocardiography (TEE).

Incorrect. Correct answer is C


45% answered correctly

Explanation:

Correct Answer Is C

Infective endocarditis (IE) can be uncommonly caused by HACEK group. HACEK group are oral gram-negative bacilli including Hemophilus, Aggregatibacter actinomycetmcomitans, Cardiobacterium hominis, Eikenella corrodens and Kingella kingae.
These organisms commonly colonize the human oropharynx as normal flora, and may cause mouth infections. HACEK organisms are most often associated with infective endocarditis, accounting for up to 10% of cases. They are also the most
common cause of gram-negative endocarditis among persons who do not abuse intravenous drugs.

Typically, HACEK organisms grow slowly and have an incubation periods of 7 to 21 days; however, with the advent of improved culture media and automated culture systems, prolonged incubation may no longer be necessary, and a 3- t o7-day
incubation period may suffice for culture.

Generally, IE have several complications with embolic events being one of the most lethal ones. Another common complication is mycotic (infective) aneurysms that may occur in 20-40% of patients. In subacute IE, the incidence rises to 60%. Since
HACEK organisms are usually associated with a subacute course, microaneurysms are a common complication. Mycotic aneurysms are commonly seen in the bifurcation of the arteries where bacteria easily vegetate. Mycotic aneurysms are usually
clinically silent until they rupture. Rupture of an intracranial mycotic aneurysm can lead to catastrophic outcomes. This is why it is recommended that imaging studies be performed to follow up the patients with subacute IE.

A number of imaging studies have been used to identify infected aneurysms, including ultrasound, CT scan, MRI and digital subtraction angiography (DSA). Of these, CT angiography is the most useful one for diagnosing mycotic aneurysm.

NOTE - MRI angiography is the alternative when intravenous contrast is contraindicated.

(Option A) Colonoscopy is considered in patients with Streptococcus bovis as the cause of their IE because Streptococcus bovis is found to be associated with increased risk of colon cancer.

(Option B) Abdominal CT scan is not useful for detection of microaneurysms.

(Options D and E) Trans-thoracic or trans-esophageal echocardiography is not useful for detection of micoraneurysms.

References

• AHA Journal - Diagnosis and Management of Infective Endocarditis and Its Complications

• Medscape - HACEK Group Infections Treatment & Management

• UpToDate - Complications and outcome of infective endocarditis

• Public Health England - Identification of Haemophilus species and the HACEK Group of Organisms

Last updated:
Time spent: QID:738 2023-2-12

661 of 1943
A 21-year-old girl is brought to the emergency department with headache and mild neck stiffness. She had kidney transplantation last year. Examination of cerebrospinal fluid (CSF) reveals a cell count of 150 with 70% monocytes. CSF glucose level
is 1.2 mmol/L and protein 1.8 g/L. Which one of the following investigation is most important to consider for this patient?

A. Immunofluorescence assay for herpes simplex virus (HSV).

B. Ziehl - Neelsen stain of the CSF.

C. Viral culture of the CSF.

D. Bacterial culture of the CSF.

E. Cryptococcus assay of the CSF.

Incorrect. Correct answer is E


45% answered correctly

Explanation:

Correct Answer Is E

The clinical picture suggests meningeal irritation. Cerebrospinal fluid (CSF) analysis shows CSF monocytosis, decreased CSF glucose level (normal 2.5-3.5 mmol/L or approximately 60% of simultaneous plasma glucose level), and elevated protein
level (normal 0.15.0.4 g/L).

Monocytosis of the CSF highly suggests fungal meningitis; therefore, Cryptococcus assay of the CSF would be the most appropriate next step to confirm the diagnosis.

Invasive fungal infections are a significant complication in solid organ transplant (SOT) recipient. Cryptococcosis is the third most common invasive fungal infection in patients with cell-mediated immune deficiency.

Between 53-72% of the cryptococcal disease in SOT recipients is disseminated or involves the central nervous system (CNS). Positive serum cryptococcal antigen has been reported in 88-91% of SOT recipient with cryptococcal meningitis.

(Option A) With Herpes meningitis, there are often concomitant genital lesions. In herpes meningitis, like other viral meningitides, the CSF glucose level tends to be normal; however, HSV is sometimes associated with decreased CSF glucose. On the
other, hand the dominant cell group is lymphocytes rather than monocytes.

(Option B) Ziehl - Neelsen stain of the CSF is indicated if TB meningitis is suspected. Although TB meningitis can be a possibility, especially in immunocompromised patients, monocytosis is more consistent with fungal meningitis. In TB meningitis,
polymorphonuclear cells are dominant at early stages, but lymphocytosis follows.

(Option C) With decreased CSF glocuse and increased CSF protein, viral meningitis is less likely. In viral meningitis, CSF gluocose and CSF protein are normal or near normal. CSF protein can be increased.

(Option D) Bacterial culture of the CSF is likely to be positive in bacterial meningitis.Bacterial meningitis often presents with significantly increased protein levels and cell count with PMNs dominating the cell differential.

References

• http://www.ncbi.nlm.nih.gov/pmc/articles/PMC269609

• http://www.ecinsw.com.au/sites/default/files/field

Last updated:
Time spent: QID:748 2023-2-12

662 of 1943
A 31-year-old woman presents with painful genital wart over the labia majora and minora. She is 38-week pregnant. Which one of the following is the next best step in management?

A. Take a piece of wart for polymerase chain reaction (PCR).

B. Obtain blood for culture.

C. Observer until after delivery.

D. Cryotherapy.

E. Excision of the lesion.

Incorrect. Correct answer is D


45% answered correctly

Explanation:

Correct Answer Is D

Higher human papilloma virus (HPV) infection rates have been reported in pregnant women. If condyloma develops, rapid growth can be observed. Contributing factors include immune system suppression during pregnancy and hormonal changes.

Small asymptomatic lesions do not require treatment as most of them resolve after delivery. Large lesions can be treated with cryotherapy or keratolytics such as trichloracetic acid (TCA) or bichloracetic acid (BCA) as the preferred methods during
pregnancy. Recurrences should be treated with same methods if complete response had occurred.

NOTE - Interferon, podophyllin, podophyllotoxin, imiquimod and 5-fluorouracil are topical agents that chemically ablate warts. Except imiquimod, these agents are contraindicated throughout pregnancy. Although imiquimod is category B1,
there is not enough evidence regarding the safety of imiquimod during pregnancy, and it can be used after the pregnant woman is fully informed.

(Option A) Genital warts are diagnosed clinically. Biopsy for PCR is only required if diagnosis is not certain.

(Option B) Blood culture is indicated to diagnose HPV infection.

(Option C) Observation until after the delivery would be the correct answer if the lesions were small and asymptomatic.

(Option E) Excision often after the first trimester is used if the wart is very large and macerated only.

References

• CFP - New approach to managing genital warts

• Medscape - Human Papillomavirus (HPV)

Last updated:
Time spent: QID:769 2023-2-12

663 of 1943
A 56-year-old Victorian farmer presents to your GP clinic with chest tightness and coughing for the past 2 months. He also mentions vague right upper quadrant abdominal pain. On examination, no remarkable findings are noted except for mildly
jaundiced sclera. Which one of the following could be the most likely diagnosis?

A. Hydatid cyst.

B. Q fever.

C. Ross River fever.

D. Brucellosis.

E. Dengue fever.

Correct
45% answered correctly

Explanation:

Correct Answer Is A

The clinical picture, suggestive of lung and hepatic involvement, and the occupation of the patient make echinococcus infection (hydatid cyst) the most likely diagnosis.

In Australia, most hydatid infections are passed between sheep ad dogs, although other animals including horses, kangaroos, dingoes and foxes may be involved.

The hydatid parasite is carried by dogs in their bowels, without any symptoms of infection. Sheep become infected while grazing in areas contaminated with dog feces. Dogs become infected by eating the uncooked organs of the infected sheep.

People become infected by ingesting eggs of the parasite, usually while there is a hand-to-mouth transfer of eggs in dog feces. This can occur when handling dogs or objects (including food and water) soiled with dog feces. Person-to-person or
sheep-to-person transmission does not occur.

The pressure effect of the cyst on the liver can produce symptoms of obstructive jaundice and abdominal pain. With biliary rupture, the classic triad of biliary colic, jaundice and urticaria is observed. Passage of hydatid membranes in vomit (hydatid
emesia) and passage of membrane in the stool (hydatid enterica) may rarely occur.

Involvement of the lungs produces chronic cough, dyspnea, pleuretic chest pain and hemoptysis. Expectoration of cyst membrane and fluid is observed with intrabronchial rupture.

References

• SA Health - Hydatid disease - including symptoms, treatment and prevention

• Medscape - Hydatid Cysts

Last updated:
Time spent: QID:803 2023-2-12

664 of 1943
A 65-year-old man presents to your clinic with a painful swelling below his left eye. The lesion is illustrated in the accompanying photograph. Which one of the following would be the most appropriate next step in management?

A. Incision and drainage.

B. Cold compression.

C. Warm compression.

D. Antibiotics.

E. Topical steroids.

Incorrect. Correct answer is D


45% answered correctly

Explanation:

Correct Answer Is D

The picture shows a swollen, red, puss-filled sac below the medial aspect of the left eye suggestive of dacryocystitis. Dacryocystitis is infection of the lacrimal sac often secondary to obstruction of the nasolacrimal duct. It presents with
inflammation localized over the medial canthus. The condition may vary in clinical presentation from mild (such as in infants) to more severe forms with abscess formation such as in this case. There is often a history of watery eye for months
beforehand.

Mild cases are managed with hot compresses and massaging the tear sac and nasolacrimal duct (the mainstay of the management) and astringent eye drops (e.g. zinc sulfate 0.25% + phenylephrine 0.12%, 1 or 2 drops 3 times daily). Topical
antibiotics such as chloramphenicol are used if there is concomitant conjunctivitis.

More severe cases of dacryocystitis are often caused by Staphylococcus aureus or Streptococcus pyogenes, but occasionally Gram-negative and anaerobes can be the culprits. Systemic antibiotic therapy is always required as the most important
initial step to limit the likelihood of infection spreading to adjacent areas such as the orbit, and serious complications such as orbital cellulitis. Although, the most appropriate choice of antibiotics are guided by Gram-stain and culture,
di/flucloxacillin or amoxicillin-clavulanate, orally, should be started while the results are pending. Cephalexin is an alternative if the patient is allergic to penicillin. Amoxicillin-clavulanate has anti-anaerobic activity and is the option of choice in
patients with diabetes or those who are otherwise immunocompromised.

NOTE - Intravenous route is used if there is orbital celllulitis, significant periorbital/ facial cellulitis, or if the patient has marked systemic symptoms (e.g. fever).

Antibiotic therapy is then followed by surgical treatment after the infection has subsided with adequate antibiotic therapy. For dacryocystitis, an dacryocystorhinostomy is preferred after several days of initiating antibiotic therapy.

665 of 1943
Six days ago, a 30-year-old man presented to your practice with complaints of acute sore throat and a fever of 39.2°C. On examination, he had red swollen tonsils with exudate. He was prescribed amoxicillin. Today, he has presented with a non-
blanching pruritic rash. Which one of the following options best describes the most likely cause for the rash?

A. Infectious mononucleosis.

B. Allergic drug reaction.

C. Hypersensitivity vasculitis.

D. Varicella zoster infection.

E. Streptococcal pharyngitis.

Incorrect. Correct answer is B


45% answered correctly

Explanation:

Correct Answer Is B

The scenario describes a rather common clinical situation: a patient with suspected bacterial pharyngitis develops rash after being started on amoxicillin. This occurs when a patient with infectious mononucleosis is misdiagnosed as bacterial
pharyngitis and started on antibiotics.

Infectious mononucleosis (IM) is a febrile illness caused by Epstein–Barr virus from herpes family. It can mimic diseases such as primay HIV infection, streptococcal tonsillitis, viral hepatitis and acute lymphatic leukemia. It may occur at any age but
is more common between10 and 35 years with the peak incidence among those ages 15-25 years.

IM has an incubation period of 4-6 weeks. The disease may initially present with sore throat (the cardinal finding), lymphadenopathy, fever, rash, and hepatosplenomegaly.

The rash of IM is almost always related to antibiotics given for tonsillitis. The primary rash, most often non-specific, pinkish and maculopapular (similar to that of rubella), occurs in only about 5% of cases. It is usually blanching and non-pruritic.

The secondary rash is most often precipitated by one of the penicillins, especially ampicillin or amoxycillin. About 90–100% of patients prescribed ampicillin or amoxycillin will be affected. This rash is non-blanching and itchy, and develops 5-9 days
after antibiotics are started.

The rash of this patient is most likely to be an allergic reaction to the antibiotic. The exact mechanism of rash following administration of antibiotics in IM is not fully understood. Some authors believe it is not a consequence of a genuine allergic
reaction because most patients will not be sensitive to the implicated antibiotic after IM resolves. Some theories suggest that the rash is caused by both decreased immune system tolerance and enhanced immune response to the implicated drug
during the infection.

(Option A) As mentioned earlier, the rash of IM presents early in the course of the disease. It has a faint color, and is non-pruritic and non-blanching in contrast to the rash here which is non-blanching and itchy.

(Option C) Neither the initial rash of IM nor the drug-related rash developing after antibiotics are given are not caused by hypersensitivity vasculitis.

(Option D) Varicella zoster infection presents with vesicular rash that is pruritic. There is no hint regarding varicella zoster as a cause in the scenario.

(Option E) Streptococcal pharyngitis can be associted with scarlatiniform form rash (similar to the rash in scarlet fever) which might resemble the rash of this patiet. In a suspetible patient, this rash manifests within the first 2 days of symptoms and
cause a papular, blanching and erythematous rash. This patient has developed an itchy, non-blanching rash 6 days after the onset of symptoms that makes such diagnosis less likely.

References

• RACGP - Infectious mononucleosis

• UpToDate - Infectious mononucleosis

• Medscape - Epstein-Barr Virus (EBV) Infectious Mononucleosis (Mono) Clinical Presentation

• DermNet NZ - Infectious mononuclesosis

Last updated:
Time spent: QID:861 2023-2-12

666 of 1943
A 16-year-old boy presents with a rash shown in the following photograph. The rash developed few days after he was started on amoxicillin-clavulanate and paracetamol because of sore throat, fever and cervical lymphadenopathy. He mentions no
improvement despite being on antibiotic. Which one of the following is the most appropriate next step in management?

A. Add gentamicin.

B. Add erythromycin.

C. Stop the antibiotic.

D. Wait and watch.

E. Give systemic corticosteroids.

Incorrect. Correct answer is C


45% answered correctly

Explanation:

Correct Answer Is C

The scenario describes a rather common clinical situation: a patient with suspected bacterial pharyngitis develops rash after being started on amoxicillin. This occurs when a patient with Ebstein-Barr infectious mononucleosis is misdiagnosed as
having bacterial pharyngitis and started on antibiotics accordingly in the majority of patients.

Infectious mononucleosis (IM) is a febrile illness caused by the herpes (Epstein–Barr) virus. It can mimic diseases such as HIV primary infection, streptococcal tonsillitis, viral hepatitis and acute lymphatic leukaemia. It may occur at any age but
usually between 10 and 35 years; it is commonest in 15–25 years age group.

It has a incubation period of 4-6 weeks. The disease may present initially with sore throat (the cardinal finding), lymphadenopathy, fever, rash, hepatosplenomegaly, etc. There might be exudative tonsillitis resembling that of streptococcal pharyngitis.

The rash of IM is almost always related to antibiotics given for tonsillitis when it is mistakenly considered bacterial.

Two types of rash might be encountered in a patient with IM:

The primary rash which is a non-specific, pinkish and maculopapular rash (similar the rash of rubella) occuring only in about 5% of cases - It is usually blanching and non-pruritic.
The secondary rash which is most often precipitated by one of the penicillins, especially ampicillin or amoxycillin. About 90–100% of patients who have been prescribed ampicillin or amoxycillin and up to 50% of those who have given
penicillin will develop the rash. It is non-blanching and itchy, can be extensive, and sometimes has a purplish tinge. The rash develops 5-9 days after antibiotics are started.

There could be another possible cause to the rash: a genuine delayed hypersensitivity reaction to amoxicillin-clavulanate, meaning that the patient has developed a genuine allergy to the antibiotic rather than an interaction with the underlying
infection.

No matter if the rash is a result of interaction between the antibiotic and IM or a drug eruption caused by a genuine drug allergy, the next step in management is cessation of the culprit antibiotic, which is the amoxicillin – clavulanate here..

Addition of other antibiotics to the current regimen is not appropriate because no antibiotic therapy is indicated for IM. Systemic steroids are the treatment of last resort for bothersome rashes not responding to more conservative treatments such
as antihistamines.

References

• http://www.racgp.org.au/afp/200310/20031001charles

• http://www.uptodate.com/contents/infectious-mononu

667 of 1943
• http://emedicine.medscape.com/article/222040-clini

• http://www.dermnetnz.org/viral/ebv.html

• http://onlinelibrary.wiley.com/doi/10.1111/j.1398-

• Murtagh’s General Practice – McGraw Hill – 6th Edition – pages 270-273


Last updated:
Time spent: QID:862 2023-2-12

668 of 1943
A 30-year-old man presented to your GP office with complaints of acute sore throat and a fever of 39.2°C which developed 48 hours prior to his visit. On examination, he had red swollen tonsils with exudate. He was prescribed amoxicillin. After
5 days, he presents with a non-blanching pruritic rash all over body. The rash is shown in the following photograph. Which one of the following tests could be most diagnostic at this stage?

A. Penicillin allergy test.

B. Blood culture.

C. Full blood count.

D. Throat swabs and cultures.

E. Serologic tests for EBV.

Incorrect. Correct answer is C


45% answered correctly

Explanation:

Correct Answer Is C

The scenario can be frequently encountered when Epstein-Barr infectious mononucleosis is misdiagnosed as streptococcal pharyngitis and the patient is started on aminopenicillins i.e. amoxicillin and ampicillin.

Epstein-Barr Infectious mononucleosis (IM) is a febrile illness caused by Epstein–Barr virus from herpes family. It can mimic diseases such as primary HIV infection, streptococcal tonsillitis, viral hepatitis and acute lymphocytic leukemia. It may
occur at any age but usually between 10 and 35 years (most common between 15 and 25 years).

After an incubation period of 4-6 weeks, IM may present with sore throat (the cardinal finding), lymphadenopathy, fever, rash, and hepatosplenomegaly. There might be exudative tonsillitis resembling streptococcal pharyngitis.

Two types of rash might be encountered in a patient with IM:

The primary rash, which is a non-specific, pinkish and maculopapular rash (similar to the rash of rubella) occuring only in about 5% of cases - It is usually blanching and non-pruritic.

The secondary rash, which is most often precipitated by one of the penicillins, especially ampicillin or amoxycillin: about 90–100% of patients, who have been prescribed ampicillin or amoxycillin and up to 50% of those who have given
penicillin will develop the rash. It is non-blanching and itchy, can be extensive, and sometimes has a purplish tinge. The rash develops 5-9 days after antibiotics are started.

With the rash and unresponsiveness to antibiotics, Epstein-Barr IM should be considered and investigated as the most likely cause.

Of the options, a full blood count, is the most appropriate test; however, not the most diagnostic one. In IM, the FBC will invariably demonstrate lymphocytosis (>50% lymphocytes). Lymphocytosis is a rule in IM. With normal or decreased lymphocyte
count, a different diagnosis should be considered. Blood film will show atypical lymphocytes (>10% lymphocytes).

ESR is another test differentiating EBV and streptococcal pharyngitis. ESR is elevated in IM but normal in streptococcal pharyngitis.

Diagnostic tests for Epstein-Barr IM are traditional Paul-Bunnell test (reaction of heterophile antibodies with sheep red blood cells) or the newer Monospot®® test (reaction of heterophile antibodies with horse red blood cells). This test is positive in
almost 90% of patients with IM somewhere in the course of the disease. One drawback to this test is that it may be falsely negative in the early stages of the clinical disease. Up to 25% of patients can have a negative Monospot® test in the first
week, 5-10% in the second week and 5% in the third week.

With Monospot® test available, serological studies (IgG and IgM titres against Epstein-Barr) are rarely indicated and only reserved for patients with signs and symptoms suggestive of the infection but with negative Monospot test results performed
weekly for six weeks.

(Option A) Penicillin skin allergy test adds no more clinical data regarding management. Blood cultures and cultures of throat swabs are very unlikely to be positive if the cause is IM. They are not useful or indicated in this situation.

(Option B) Blood cultures are negative in IM and have no diagnostic value.

(Option D) Throat swabs and cultures are negative in IM. They are not appropriate diagnostic tests to consider for patients with IM.

(Option E) Serologic tests, so early in the course of the disease, are less likely to be diagnostic.Moreover, they are not cliically indicated because Monospot® test is more available and diagnostic.

References

• http://www.uptodate.com/contents/infectious-mononu

669 of 1943
• http://emedicine.medscape.com/article/222040-worku

• http://www.racgp.org.au/afp/200310/20031001charles
Last updated:
Time spent: QID:863 2023-2-12

670 of 1943
A 47-year-old man presents with complaints of swinging fevers, productive cough with a fetid odor and right-sided chest pain that worsens with respiration. These have occurred and progressed over a course of few days. A chest X-ray is obtained
that shows a round opacity in the right middle lobe. There is air-fluid level within the opacity. Expectorated sputum is sent for gram stain and culture. The initial result of the Gram stain test reveals Staphylococcus aureus in the sputum. You start the
patient on intravenous flucloxacillin. Which one of the following is could be the most appropriate next step in management?

A. Transpleural drainage.

B. Lobectomy.

C. Transcutaneous aspiration.

D. Water-sealed chest drain.

E. CT scan of the chest.

Incorrect. Correct answer is E


45% answered correctly

Explanation:

Correct Answer Is E

The clinical presentation as well as the radiographic finding of right middle lobe opacity with an air-fluid level is highly suggestive of lung cavitation. Lung abscess is a common cause of cavitation in lung parenchyma. Other pulmonary lesions that
are not caused by microbes may resemble lung abscess. These include:

Lesions of pulmonary infarction


Bronchiectasis
Necrotizing carcinoma
Pulmonary sequestration
Vasculitides e.g. polyarteritis nodosa, granulomatosis with polyangiitis (Wegener’s), Goodpasture syndrome
Cysts or bullae with fluid collections

Lung abscess is defined as necrosis of the pulmonary parenchyma caused by microbial infection. Some authors use the term “necrotizing pneumonia” or “lung gangrene” to distinguish pulmonary necrosis with multiple small abscesses from a larger
cavitary lesion; however, this actually represents a spectrum of the same process.

The most common cause of lung abscesses are bacteria with predominance of anaerobes (often anaerobic flora of the oral cavity). Many other bacteria can also cause lung abscess but much less frequently. Lung abscess usually has a
polymicrobial cause; however, the following organisms are capable of producing monomicrobial lung abscess:

Staphylococcus aureus
Klebsiella pneumoniae
Gram negative bacilli
Hemophilus influenza type B
Legionella
Nocardia
Actinomycosis

Lung abscess can present with the following features:

An indolent course over days to weeks (or months in specific situations)


Fever (rigors or chills are often absent)
Productive cough that can be foul-smelling
Night sweats
Weight loss
Malaise

NOTE – The most characteristic form of pneumonia caused by Staphylococcus aureus is fulminant disease in young adults or adolescents with underlying influenza infection. The patient often has a fulminant course with shock, neutropenia, lung
necrosis and high mortality rate despite antibiotic treatment.

Antibiotics (often intravenously) are mainstay of therapy for uncomplicated lung abscess, especially for those with air-fluid level that indicates communication of the abscess with airway and spontaneous abscess drainage.

Surgical intervention is reserved for patients with complicated lung abscess. The following are considered as complicated lung abscess:

Failure to respond to medical management


Suspected neoplasm
Abscesses associated with an obstructed bronchus
An extremely large abscess (>6 cm in diameter)
Abscesses involving relatively resistant organisms such as Pseudomonas aeruginosa

The usual procedure in such cases is a lobectomy or pneumonectomy. For those who are poor candidates for surgery alternative methods such as endoscopic drainage or percutaneous drainage may be considered. Percutaneous procedures require
special care to prevent contamination of the pleural space. Bronchoscopy may be done as a diagnostic procedure, especially to detect an underlying lesion, but this procedure is of relatively little use to facilitate drainage and can result in spillage of
abscess contents into the airways. Endoscopic drainage, which requires an experienced operator, is performed by placing a pigtail catheter into the abscess cavity under bronchoscopic visualization and leaving the catheter in place until the cavity
has drained.

Once lung abscess is suspected, the most important component of evaluation are:

Defining the etiologic factor for selection of appropriate choice of antibiotic

Microbiologic studies include stains and cultures of expectorated sputum to detect aerobic bacterial pathogens that in this case scenario has been Staphylococcus aureus.

Anaerobic bacteria are not detected in expectorated sputum cultures. Alternative specimens that may be useful include pleural fluid obtained by thoracentesis in patients who have empyema and quantitative bronchoalveolar lavage
(BAL) specimens if they are processed promptly and appropriately for anaerobic bacteria. Another method with increasing popularity for bypassing contamination by the flora of the upper airways is trans-thoracic (percutaneous) needle
aspiration under CT guidance; the popularity of this procedure has increased in recent years.

Radiographic evaluation for differentia diagnoses or associated conditions

A chest X-ray generally demonstrates the pulmonary lesion to the extent necessary for diagnosis and management; however, better anatomic definition can be achieved with CT scanning. CT is particularly helpful if the diagnosis and
delineation of the cavity is in doubt or in distinguishing between lung abscess and empyema despite looking similar on X-ray sometimes, require completely different managements.

Of the options, CT scan is the most appropriate option. It can give more anatomic definition of the lesion, assess the presence of empyema or other associated conditions, and differentiate between other possible diagnoses.

671 of 1943
(Option A) With the air-fluid level, this abscess is communicating with a major airway and drainage is not reqired because the abscess will drain into that aiway. However, if CT scan found the cause to be an empyema and not an abscess, transpleural
drainage will be indicated.

(Option B) Lobectomy (preferred) and transpleural drainage can be considered if the abscess is complicated, evident by the size, poor response to treatment, obstruction, etc. There is no clue in the scenario pointing towards complications; therefore,
these measures are not appropriate for now.

(Option C) The etiologic agent in this patient has already been determined by sputum exam so aspiration to obtain a sample is not appropriate or useful.

(Option D) Water-sealed chest drain is a treatment option for empyema either in isolation or as a complication of lung abscess. There is no indication for that at present.

References

• http://www.uptodate.com/contents/lung-abscess

• http://emedicine.medscape.com/article/299425-overv

• Harrison’s Principals of Internal Medicine - 18th Edition - McGraw Hill Medical - page 2145

Time spent: QID:866 Last updated:


2023-2-12

672 of 1943
Rob is a 30-year-old patient of yours, who was found to be HIV positive a while back. He was started on antiretroviral therapy after a consultation with an infectious disease specialist. Today, he is in your office for a follow-up visit. He looks quite
calm and avoiding. After breaking the ice, you realize that he has been feeling down and has not taken his medications for the past 6 months. You arrange for laboratory tests, the results of which shows a decreased CD4 count of 46/mm3. You talk
him into restarting his anti HIV medications. He is convinced to follow your instructions. In addition to the anti HIV medications, which one of the drug(s) he should receive?

A. Fluconazole.

B. Azithromycin and trimethoprim-sulfamethoxazole.

C. Dapsone.

D. Azithromycin and fluconazole.

E. Azithromycin, dapsone, and fluconazole.

Incorrect. Correct answer is B


45% answered correctly

Explanation:

Correct Answer Is B

The primary mechanism of HIV is infecting a particular subset of T lymphocytes known as CD4. Decreased amount of CD4 cells results in development opportunistic infections that define AIDS.

CD4 count in a non-infected person is approximately 700/mm3. HIV infection causes this number to drop at a rate of 50-100/mm3/year; therefore, without treatment, it would take 5-10 years for CD4 count to drop to around 200/mm3 when the first
AIDS-defining syndrome develops.

An HIV infected person does not develop opportunistic infection unless CD4 decrease below a certain level; therefore, prophylactic treatment against such infections is not indicated as long as the CD4 count is above the predicted CD4 count for that
infection.

The most common opportunistic infections associated with HIV infection include:

Pneumocystis jiroveci (formerly carini)


Mycobacterium avium complex (MAC)
Toxoplasmosis
Candidiasis
Cryptococcosis
Coccidioidomycosis
CMV
Tuberculosis
Histoplasmosis
Cryptosporidium

Of these infections, routine prophylactic is indicated for pneumocystis jiroveci, toxoplasmosis, and mycobacterium avium complex once CD4 count drops below 200/mm3, 100/mm3, and 50/mm3, respectively.

The following table summarizes the opportunistic infections in HIV infected patients and the CD4 count below which prophylaxis is commenced as well as the appropriate prophylactic regimen:

CD4 count
Infection Prophylaxis recommendations

(/mm3 or μL)
Pneumocystis jiroveci (formerly carini) 200 Trimethoprim-sulfamethoxazole (double-strength tablet daily)
Toxoplasmosis 100 Trimethoprim-sulfamethoxazole (double-strength tablet daily)
Mycobacterium avium complex 50 Azithromycin (1200 mg weekly)

Routine prophylaxis for the following infections is not routinely indicated, unless there are indications other than CD4 count:

Candidiasis
Cryptococcosis
Coccidioidomycosis
CMV
Tuberculosis
Histoplasmosis
Cryptosporidium

With a CD4 count of 46/mm3, this patient should be started on routine prophylaxis against pneumocystis jiroveci, toxoplasmosis, and mycobacterium avium complex. For this purpose, combination of trimethoprim-sulfamethoxazole and
azithromycin should be considered in addition to antiretroviral therapy.

Dapsone can be replaced with trimethoprim-sulfamethoxazole as second-line medication, where the former cannot be tolerated or is contraindicated.

Fluconazole is used for treatment of Candidiasis (oropharyngeal, vaginal, esophageal) and has no role in routine prophylaxis.

References

• http://www.uptodate.com/contents/overview-of-preve

• http://emedicine.medscape.com/article/1529727-over

• https://aidsinfo.nih.gov/guidelines/html/4/adult-a

Time spent: QID:982 Last updated:


2023-2-12

673 of 1943
A 2-year-old boy is brought to your GP clinic with drowsiness, fever and a petechial rash. You suspect meningitis based on clinical grounds. Which one of the following is the most appropriate next step in management?

A. Blood culture.

B. Intravenous access.

C. Lumbar puncture.

D. Intravenous ceftriaxone.

E. Intravenous benzylpenicillin.

Incorrect. Correct answer is B


45% answered correctly

Explanation:

Correct Answer Is B

The presence of rash and fever is highly suggestive of meningococcemia. In approach to an ill child with suspected bacterial meningitis in settings other than hospital, the most important step in management is supplemental oxygen and intravenous
(IV) access. Although there is no comment in the scenario regarding hemodynamic compromise, such children are susceptible to rapid deterioration and collapse. A bolus dose of 20ml/kg (up to 60ml/kg) should be given intravenously. If IV access
cannot be established, interosseous route should be tried.

Pre-hospital antibiotics are recommended due to high mortality rate of potential meningococcal sepsis.Parenteral benzylpenicillin (option E) is an appropriate option as it is readily available; however, since other invasive pathogens such as
Streptococcus pneumoniae and Hemophilus influenza type b can cause invasive disease with similar presentation, a broader spectrum antibiotic such as ceftriaxone (option D) or cefotaxime is preferred if available. If neither is available, ampicillin or
amoxicillin can be used as alternatives. In case of hypersensitivity to cephalosporins or penicillins, chloramphenicol is recommended. Intravenous administration is the preferred route.

If IV or intraosseous access cannot be obtained, the antibiotic may be administered intramuscularly despite the fact that decreased perfusion in the setting of shock may limit absorption. It is, however, preferable to not giving any antibiotic.

Blood sample for culture (option A) should also be obtained if possible when intravenous access is tried. If not, antibiotics should not be delayed.

Lumbar puncture (option C) should be performed in the hospital in consultation with an expert in the field.

References

• http://www.racgp.org.au/download/documents/AFP/201

Last updated:
Time spent: QID:1010 2023-2-12

674 of 1943
An 75-year-old man presents to your GP clinic with complaints of dysuria, frequency and urgency. Investigations, including urine analysis establish the diagnosis of urinary tract infection (UTI). His GFR is 17 ml/min. Which one of the following is the
antibiotic of choice for treatment of his UTI?

A. Amoxicillin.

B. Gentamicin.

C. Ciprofloxacin.

D. Trimethoprim.

E. Nitrofurantoin.

Incorrect. Correct answer is D


45% answered correctly

Explanation:

Correct Answer Is D

The choice of antibiotics for management of pyelonephritis in patients with decreased GFR is generally the same for patients with normal GFR, but dose adjustment according to the GFR is often required.

For empiric treatment of UTI in men, while awaiting the results of the investigations, trimethoprim is the antibiotic of choice.

NOTE - UTI in men is often associated with an underlying urinary tract anomaly. Simultaneous infections of posterior urethra, prostate or epididymis is common. All males with UTI should be investigated for exclusion of an underlying abnormality,
and determining more specific therapy. The duration of antibiotic therapy for UTI in men is 14 days.

(Option A) Amoxicillin is never used for empiric treatment of UTI because the very high rates of resistance to this antibiotic.

(Option B) Gentamicin is not among recommended options for empiric treatment of uncomplicated UTI. Moreover, aminoglycosides such as gentamicin are nephrotoxic.

(Option C) Fluoroquinolones (e.g. ciprofloxacin) should not be used as first-line therapy as they are the only active drugs available for infections due to Pseudomonas aeruginosa and other multiresistant bacteria.

(Option E) Nitrofurantoin can also be used as an alternative, but is not recommended as first-line option. This is true about cephalexin and amoxycillin +clavulanate.

References

• CJASN - Urinary Tract Infections in Patients with Chronic Renal Insufficiency

• RACGP - Medical care of older persons in residential aged care facilities

• Therapeutic Guidelines – Antibiotics; available from: http://tg.org.au

Last updated:
Time spent: QID:1096 2023-2-12

675 of 1943
A 5-year-old boy is brought to the emergency department with lethargy and a fever. On examination, he has neck stiffness. With meningitis as the most likely diagnosis, a lumbar puncture is performed, the result of which is as follows:

Opening pressure: 45mmH2O (5-20)


Appearance: cloudy
Protein: 1.2g/L (0.18-0.45)
Glucose: 1.5mmol/L (2.5-3.5)
WCC: 900/mm3 (<3) neutrophil predominance

The child is started on intravenous cefotaxime. Which one of the following should be considered as the most appropriate next step in management?

A. Oral paracetamol.

B. Aspirin.

C. Intravenous dexamethasone.

D. Acyclovir.

E. CT scan of the head.

Incorrect. Correct answer is C


45% answered correctly

Explanation:

Correct Answer Is C

If possible, an LP should be performed prior to starting antibiotics in suspected cases of meningitis. This child has a CSF analysis consistent with the diagnosis of bacterial meningitis. High protein, low glucose and increased WCC with neutrophil
predominance make bacterial meningitis the most likely diagnosis with high certainty.

Once, on clinical grounds and/or CSF analysis results, bacterial meningitis is suspected, intravenous antibiotics should be started as the most appropriate step management, ideally after blood samples are taken for culture.

Current evidence suggests that steroids may reduce the risk of hearing loss in bacterial meningitis. For children >2months of age, dexamethasone (0.15mg/kg, intravenously) should be administered before antibiotics or within one hour of
commencement of antibiotics.

(Option A) Paracetamol may be considered to lower the fever in a febrile child; however, this does not take precedence over dexamethasone.

(Option B) Aspirin is not used in febrile children to the risk of Reye syndrome.

(Option D) Acyclovir or other antiviral agents are indicated if ,based on clinical and/or laboratory findings, viral encephalitis is suspected. The CSF analysis in this child is not suggestive of a viral etiology, nor are there comments about confusion,
altered mental state or focal neurological findings as is expected in encephalitis.

(Option E) CT scan of the head may be considered for adults before LP is attempted to exclude increased intracranial pressure that can result in brain herniation once LP is performed. It is not a routine practice in children. Moreover, in this child LP
has already been carried out and there is not further indication for CT scanning.

TOPIC REVIEW

CSF analysis results in patients with meningitis are summarized in the following table:

Normal Bacterial Viral Fungal/TTB


Pressure (cmH2O) 5-20 >30 Normal or mildly increased
Appearance Normal Turbid Clear Fibrin web
>1 <1 0.1-0.5
Protein (g/L) 0.18-0.45
but maybe normal but may be normal but maybe normal
<2.2 1.6-2.5
Glucose(mmol/L) 2.5-3.5 Usually normal
but may be normal but maybe normal
Glucose: CSF:Serum ratio 0.6 <0.4 >0.6 <0.4
WCC* <3 >500 <1000 100-500
Gram stain Normal 60-90% positive Normal

Monocytes

Other 90% PMN 10% have >90% PMN

30% have>50% PMN

* WCC is not very reliable for differentiating the causative organism in meningtis. A CSF WCC of 5-100 can be seen in early bacterial meningitis, viral meningitis, TB meningitis and fungal meningitis.

References

• The Royal Children's Hospital (RCH) - Meningitis- encephalitis

• NSW Health Department - Infants and Children: Acute Management of Bacterial Meningitis

Last updated:
Time spent: QID:1130 2023-2-12

676 of 1943
A 46-year-old woman presents to the emergency department with headache and photophobia. Lumbar puncture is performed after CT scan of the head excluded increased intracranial pressure. The CSF analysis result is as follows:

Opening pressure 25cmH2O (normal:10-20)


WCC: 40/mm3 – lymphocytes 85%
Total protein: 1.8 g/L (normal: 0.15-0.45)
Glucose: 1.0 mmol/L (2.5-3.5)
Simultaneous serum glucose: 4.9 mmol/L

Which one of the following is the least likely diagnosis?

A. Partly treated bacterial meningitis.

B. Carcinomatosis meningitis.

C. Viral meningitis.

D. TB meningitis.

E. Fungal meningitis.

Incorrect. Correct answer is C


45% answered correctly

Explanation:

Correct Answer Is C

Normal concentration of glucose in CSF samples is 2.5-3.5 mmol/L, or 60-80% of simultaneous plasma glucose (for glucose plasma concentrations less than 22 mmol/L). A significant decrease CSF glucose level, especially decreased CSF glucose level in relation with serum are usually associated with bacterial,
TB, fungal and carcinomatosis meningitis, but not viral meningitis.

Exceptionally, viral meningoencephalitis caused by mumps, enteroviruses, herpes simplex and herpes zoster can be associated with mild to moderate decrease in CSF glucose; however, a CSF glucose of 1.0 mmol/L and a CSF-to-serum glucose ratio of 20% (normal 60-80%) is very unlikely to be caused by viral
meningitis.

This patient has also an elevated CSF protein level. Typically, protein level is elevated in bacterial, TB, fungal and carcinomatosis meningitides, but decreased in viral meningitis (often <1g/L) but it can be normal as well. An elevated protein level in this patient’s CSF makes viral meningitis less likely than bacterial,
fungal or TB meningitides (but not unlikely).

Finally, the CSF opening pressure in this patient has been 25 cmH2O. Increased opening pressure is more commonly seen in bacterial rather than viral meningitis. In viral meningitis, the opening pressure is normal or just slightly raised above normal. A normal opening pressure does not exclude bacterial
meningitis though.

With a significantly decreased CSF glucose, high protein level and elevated CSF opening pressure, viral meningitis is the LEAST likely diagnosis.

The cell count is not of much help to exclude a causative agent as there is a huge overlap between different types of meningitis in terms of cell count. Interpretation of total white cells in CSF is summarized in the following table:

5-100/mm3 100-1000/mm3 >1000/mm3


Early bacterial meningitis Bacterial meningitis

More common Viral meningitis Viral meningitis Bacterial meningitis

TB meningitis TB meningitis
Less common Encephalitis Encephalitis Mumps, lymphocytic choriomeningitis

The WCC of this patient does not make any of the options least likely. A WCC count of 40/mm3 may be due to early bacterial meningitis, viral meningitis, fungal meningitis or TB meningitis.

(Option A) In patients with partly treated bacterial meningitis‫ و‬culture and Gram stain results may be altered (less frequently positive), but other CSF values such as cell count, protein and glucose remain mostly unchanged. With elevated CSF protein and decreased CSF glucose, bacterial meningitis can be likely
despite a WCC count of 40 with lymphocytic predominance.

(Option D) Carcinomatosis meningitis is a rare complication of cancer in which the disease spreads to the meninges. It occurs in 5% of patients with cancer, usually terminal. With treatment, median survival is 2-3 months. CSF analysis shows decreased glucose and elevated protein levels as in this patient.
However, no cancer in the history and the rarity of the condition make it less likely.

3
(Options D and E) CSF in fungal/TB meningitis often has decreased glucose, increased protein and cell count of 5-100 up to 500 cell/mm . This patient can have either of them based on given CSF findings.

TOPIC REVIEW

CSF analysis results in patients with meningitis are summarized in the following table:

Normal Bacterial Viral Fungal/TTB


Pressure (cmH2O) 5-20 >30 Normal or mildly increased
Appearance Normal Turbid Clear Fibrin web
>1 <1 0.1-0.5
Protein (g/L) 0.18-0.45
but maybe normal but may be normal but maybe normal
<2.2 1.6-2.5
Glucose(mmol/L) 2.5-3.5 Usually normal
but may be normal but maybe normal
Glucose: CSF:Serum ratio 0.6 <0.4 >0.6 <0.4
WCC* <3 >500 <1000 100-500
Gram stain Normal 60-90% positive Normal
Monocytes

Other 90% PMN 10% have >90% PMN

30% have>50% PMN

* WCC is not very reliable for differentiating the causative organism in meningtis. A CSF WCC of 5-100 can be seen in early bacterial meningitis, viral meningitis, TB meningitis and fungal meningitis.

References

• UptoDate - Cerebrospinal fluid: physiology and utility of an examination in disease states

• The Royal Children's Hospital (RCH) - CSF interpretation

Time spent: QID:1182 Last updated:


2023-2-12

677 of 1943
A 35-year-old woman presents to your practice with a painful tender swelling inferior to the medial side of his left eye. The lesion is shown in the following photograph. She is otherwise healthy and not febrile. Which one of the following is the most
appropriate next step in management?

A. Massage and heat.

B. Intravenous dicloxacillin.

C. Incision and drainage.

D. Oral amoxicillin.

E. Oral flucloxacillin.

Incorrect. Correct answer is E


45% answered correctly

Explanation:

Correct Answer Is E

The pus-filled swelling below the medial canthus is highly suggestive of acute dacryocystitis. The surrounding skin is red but not significantly swollen. In most patient with acute dacryocystitis, antibiotics such as di/flucloxacillin, amoxicillin-
clavulanate, or cephalexin are associate with a dramatic response, and should be used as first-line management options. Intravenous route is reserved for patients with severe cellulitis, orbital involvement (orbital cellulitis) or severe adjacent
facial/periseptal cellulitis.

(Option A) Massage and heat is used in patients with mild dacryocystitis. In more severe cases, massage is exquisitely painful and not effective.

(Option B) This patient does not have orbital/periorbital cellulitis, significant facial cellulitis, or signs of systemic toxicity. In such cases intravenous antibiotics are unnecessary and oral route is very likely to provide adequate coverage and treatment.

(Option C) Incision and drainage is not recommended in active infection. However, it is rarely considered in very severe cases associated with serious complications such as orbital cellulitis or when more conservative management fails.

(Option D) Because of increased chances of resistance, amoxicillin alone is not likely to provide appropriate antibiotic coverage, unless there is proven sensitivity.

TOPIC REVIEW

In most cases acute dacryocystitis is caused by complete obstruction of the nasolacrimal duct, preventing normal drainage from the lacrimal sac into the nose. Chronic tear retention and stasis lead to secondary infection.

Clinical findings include edema and erythema with distension of the lacrimal sac below the medial canthal tendon. The degree of discomfort ranges from none to severe pain. Complications include dacryocystocele formation, chronic conjunctivitis,
and spread to adjacent structures (orbital or periorbital/facial cellulitis).

Guidelines for treatment of acute dacryocystitis are as follows:

Avoid irrigation or probing of the canalicular system until the infection subsides. In most cases, irrigation is not needed to establish the diagnosis and is extremely painful in the setting of active infection.

Similarly, diagnostic or therapeutic probing of the nasolacrimal duct is not indicated in adults with acute dacryocystitis.

Topical antibiotics are of limited value. They do not reach the site of the infection because of stasis within the lacrimal drainage system. They also do not penetrate sufficiently within the adjacent soft tissue.

Oral antibiotics are effective in most infections. Gram-positive bacteria are the most common cause of acute dacryocystitis. However, gram-negative organisms should be suspected in patients who are diabetic or
immunocompromised or in those who have been exposed to atypical pathogens (e.g. individuals residing in nursing homes). Parenteral antibiotics are necessary for the treatment of severe cases, especially if cellulitis or orbital
extension is present.

Incision and drainage of a localized abscess involving the lacrimal sac and adjacent soft tissues should be reserved for severe cases and those who are unresponsive to more conservative measures because a chronically draining
epithelialized fistula that communicates with the lacrimal sac can form. The incised abscess is packed open and allowed to heal by second intention. This treatment should be reserved for severe cases and those that do not respond to
more conservative measures.

References

• http://www.rcemlearning.co.uk/references/eye-infec

• http://www.mrcophth.com/eyecasualty/maincontent/sw

678 of 1943
• Therapeutic Guidelines – Antibiotics; available from http://tg.org.au
Last updated:
Time spent: QID:1185 2023-2-12

679 of 1943
A 60-year-old man is booked for elective colonoscopy as a part of evaluation of iron deficiency anemia. He was diagnosed with mitral regurgitaion 15 years ago. Which one of the following is the next best step in management regarding infectious
endocarditis prophylaxis?

A. Amoxicillin 2g before the procedure.

B. Ceftriaxone 1g before the the procedure.

C. No prophylaxis is needed.

D. Amoxicillin 500mg 6 hours before the procedure.

E. Gentamicin 80mg before the procedure.

Incorrect. Correct answer is C


45% answered correctly

Explanation:

Correct Answer Is C

Antibiotic prophylaxis is recommended in patients with the following cardiac conditions who are undergoing a specific procedure:

Prosthetic cardiac valve or prosthetic material used for cardiac valve repair
Previous infective endocarditis
The following congenital heart diseases:

Unrepaired cyanotic defects, including palliative shunts and conduits


Completely repaired defects with prosthetic material or devices, whether placed by surgery or catheter intervention, during the first 6 months after the procedure (after which the prosthetic material is likely to have been
endothelialized)
Repaired defects with residual defects at or adjacent to the site of a prosthetic patch or device (which inhibit endothelialization)

Cardiac transplantation with the subsequent development of cardiac valvulopathy


Rheumatic heart disease in Indigenous Australians only

Mitral valve regurgitation is not an indication for infectious endocarditis prophylaxis.

References

• Therapeutic Guidelines – Antibiotic; available from http://tg.org.au

Last updated:
Time spent: QID:251 2023-2-12

680 of 1943
A 37-year-old man presents to your practice with complaints of fever and three episodes of loose stool since yesterday. He returned back from a four-day stay in Thailand two days ago. On examination, he has a fever of 38.7° with no other
remarkable findings. A full blood exam (FBE) reveals no anemia. Which one of the following options excludes malaria as the diagnosis?

A. The short incubation period.

B. The fever.

C. The absence of splenomegaly.

D. The absence of anemia.

E. Diarrheal episodes.

Correct
45% answered correctly

Explanation:

Correct Answer Is A

It is very important to consider malaria in all febrile returning travelers to areas where malaria is endemic. Following the bite of an infected female anopheles mosquito, the inoculated sporozoites go to the liver within 1-2 hours. The bitten individual
is often asymptomatic for 12 to 35 days; however, symptoms can develop as early as 7 days. The incubation period for plasmodium falaciparum is 7; however, people who have taken antimalarial drugs or have partial immunity due to frequent
exposures may develop symptoms much later. The incubation period for relapsing species plasmodium ovale and plasmodium vivax is approximately 2 weeks but clinical symptoms can occur months later due to activation of hypnozoites. The
incubation period for plasmodium malariae is about 18 days.

This man has been in Thailand for four days and his symptoms have started one day after he has returned. Even if bitten on the first day the diagnosis cannot be malaria given the fact that the minimum reported incubation period for malaria is at
least 7 days (A is correct).

(Option B) In malaria, fever is almost always a feature. The fever is usually fluctuating and is associated with rigors, chills, and sweating.

(Options C and D) In a non-immune person, the spleen can be palpable after several days of symptoms. In such patients, lack of anemia early in the clinical course is common. Given these, a normal spleen, or the lack of anemia does not exclude
malaria as the diagnosis.

(Option E) Clinical malaria often starts with fever, malaise, fatigue, tachycardia, tachypnea, sweating, headache, cough, anorexia, nausea, vomiting, abdominal pain, diarrhea, arthragia and myalgia. Mild jaundice may also be present. Fluctuating fever
is almost always a feature. Since diarrhea can be a manifestation, its presence does not exclude malaria.

NOTE – In a person with malaria, laboratory findings other than anemia may include: thrombocytopenia, elevated transaminases, increased blood urea nitrogen (BUN) and creatinine, mild coagulopathy and hyperbilirubinemia (often indirect).

References

• https://www.uptodate.com/contents/clinical-manifestations-of-malaria-in-nonpregnant-adults-and-children

Last updated:
Time spent: QID:1200 2023-2-12

681 of 1943
A 32-year-old Sudanese man, who is a recent immigrant to Australia, presents for medical assessment. He has been febrile for the past two weeks, is lethargic and has decreased appetite. Laboratory tests are normal expect for mildly elevated WBCs
of 12300/mm3. A chest X-ray is obtained which is shown in the following photograph. Which one of the most important initial step in management of this patient?

A. Commencement of treatment with doxycycline.

B. Isolation.

C. Monotherapy with isoniazid.

D. Quadruple therapy with rifampin, isoniazid, pyrazinamide and ethambutol (RIPE therapy).

E. Sputum smear and culture.

Incorrect. Correct answer is B


45% answered correctly

Explanation:

Correct Answer Is B

The X-ray is remarkable for cavity formation in the upper lobe of the right lung. This X-ray finding and the symptoms of fever, lethargy and decreased appetite in a recently migrated patient from Sudan, makes pulmonary tuberculosis (TB) the most
likely diagnosis.

TB is caused by the Mycobacterium tuberculosis (M. tuberculosis) complex. M. tuberculosis is the causative bacteium for most cases of TB . Mycobacterium bovis (M. bovis), M. africanum, M. canetii and M. caprae also cause a small number of TB
cases in Australia.

A person with pulmonary TB is infectious as long as viable bacilli are being discharged from the sputum. In practice, the greatest risk of transmitting infection is in the period prior to diagnosis and effective treatment of a pulmonary TB case. The risk
of transmitting infection is reduced within days to two weeks after starting TB treatment, providing there is no drug resistance.

NOTE - A sputum smear-positive case is more infectious than a case that is only culture-positive.

Due to high degree of infectivity of pulmonary TB, it is strongly recommended that all suspected or definite cases of pulmonary TB be appropriately isolated regardless of being treated in outpatient and inpatient setting as the very first step in
management while other investigations/treatment is undertaken.

NOTE - A restriction order, e.g. under a Notifiable Diseases Act or similar, may be issued to a person with pulmonary TB who does not comply with prescribed treatment and is not willing to limit their movement within the community.

The criteria for discontinuation of isolation include:

For patients with suspected TB:

Infectious TB is subsequently considered unlikely and either:

an alternative diagnosis has been made which explains the clinical manifestations; or
the patient has three consecutive negative AFB sputum smears on different days

For patients with confirmed TB:

The patient has been started on treatment and:


has received a minimum of two weeks of effective therapy; and
understands and tolerates the medications, and is improving clinically, including improvement of cough; or
there are three consecutive daily negative sputum AFB smears.

NOTE – patients with extra-pulmonary TB do not require isolation.

(Option A) Doxycycline is not the standard treatment option for TB.

682 of 1943
(Option C) Monotherapy with isoniazid may be considered for patients with latent TB infection (LTBI). This regimen is not appropriate for active TB.

(Option D) The initial (also called intensive) phase of TB treatment for patients with no suspected drug resistance is with four antibiotics i.e. rifampin, isoniazid, pyrazinamide, and ethambutol (RIPE regimen). This treatment phase is often started
before antibiotic sensitivity results are available. The duration of the initial intensive phase is for a minimum of two months or at least until sputum is smear negative for pulmonary TB and, in culture positive cases, drug susceptibilities are known,
whichever is longer.

(Option E) Sputum AFB smear, culture and sensitivity are mandatory and should be performed as soon as possible. However, isolation should be undertaken prior to any investigations or treatment in all cases with suspected or confirmed pulmonary
TB.

References

• Management, control and prevention of tuberculosis

• Health Department Australia - National guidelines for the public health management of TB

Time spent: QID:1212 Last updated:


2023-2-12

683 of 1943
A 40-year-old man presents to the emergency department with complaints of fever and cough. He has the history of chronic cough and sputum for the past 2 years. According to him, the sputum is greenish-yellow and very copious especially in the
morning. He mentions that he almost has had a cup-full of sputum every days for the past several months. On examination, he has a blood pressure of 110/95mmHg, pulse rate of 110bpm, temperature of 38.7, and O2 saturation of 92% while on
room air. Lung auscultation reveals consolidation of the right lower lobe and bibasal crackles. A chest x-ray establish the diagnosis of lobar pneumonia. Which one of the following is the most appropriate treatment option for him?

A. Oral roxythromycin.

B. Oral Augmentin.

C. Intravenous ampicillin.

D. Intravenous flucloxacillin.

E. Intravenous piperacillin-tazobactam.

Incorrect. Correct answer is E


45% answered correctly

Explanation:

Correct Answer Is E

The chronic copious sputum production that is yellow-green in color is highly suggestive of bronchiectasis as the underlying lung condition, over which a lobar pneumonia has superimposed.

Bronchiectasis is the irreversible dilation of one or more of the conducing bronchi or airways, most often secondary to an infectious process.

Bronchiectasis presents with the following:

Cough and daily mucopurulent sputum production, often lasting months to years (classic)
Blood-streaked sputum or hemoptysis from airway damage associated with acute infection
Dyspnea, pleuritic chest pain, wheezing, fever, weakness, fatigue, and weight loss
Rarely, episodic hemoptysis with little to no sputum production (i.e. dry bronchiectasis)

Exam findings in a patient with bronchiectasis are often non-specific and include:

Crackles, rhonchi, scattered wheezing, and inspiratory squeaks on auscultation


Digital clubbing (2-3% of patients; more frequent in moderate-to-severe cases)
Cyanosis and plethora with polycythemia from chronic hypoxia (rare)
Wasting and weight loss
Nasal polyps and signs of chronic sinusitis
Physical stigmata of cor pulmonale, in advanced disease

Exacerbation of bronchiectasis is characterized by:

Increased sputum production over baseline


Thickening of the sputum
Malodorous sputum (occasional)
Low-grade fever (a rare finding)
Increased fatigue and/or malaise
Increased dyspnea, shortness of breath, wheezing, or pleuritic chest pain

Exacerbations of bronchiectasis often require oral antibiotic therapy with amoxicillin or doxycycline. Patients in whom pseudomonas aeroginosa is isolated from sputum culture require a quinolone such as ciprofloxacin or moxifloxacin to cover this
organism.

This patient, however, has a lobar pneumonia that demands a different antibiotic therapy from that used for an exacerbation of bronchiectasis. It is estimated that a significant number of patients with bronchiectasis are colonized or have chronic
infection with pseudomonas aeroginosa; therefore, for this patient an antibiotic with adequate coverage against this pathogen should be considered as first-line empiric therapy until susceptibility is available. Ceftazidime or a synthetic penicillins
with anti-pseudomonas activity such as piperacillin + tazobactam or ticarcillin +clavulanic acid, or meropenem are first-line antibiotics to consider for this patient.

Oral roxythromycin (a macrolide) (option A) or oral Augmentin (amoxicillin + clavulanic acid) (option B) were appropriate options if the case was an exacerbation of bronchiectasis not pneumonia.

Ampicillin (option D) and flucloxacillin (option E) do not provide adequate coverage against pseudomonas.

References

• https://emedicine.medscape.com/article/296961-overview

• Therapeutic Guideline – available from: http:tg.org.au

Last updated:
Time spent: QID:1249 2023-2-12

684 of 1943
A 40-year-old man presents to the emergency department with complaints of fever and cough. He has the history of chronic cough and sputum for the past 2 years. According to him, the sputum is greenish-yellow and very copious especially in the
morning. He mentions that he almost has had a cup-full of sputum every days for the past several months. On examination, he has a blood pressure of 110/95mmHg, pulse rate of 110bpm, temperature of 38.7, and O2 saturation of 92% while on
room air. Lung auscultation reveals consolidation of the right lower lobe and bibasal crackles. A chest x-ray establish the diagnosis of lobar pneumonia. Which one of the following is the most appropriate treatment option for him?

A. Oral roxythromycin.

B. Oral Augmentin.

C. Intravenous ampicillin.

D. Intravenous flucloxacillin.

E. Intravenous piperacillin-tazobactam.

Incorrect. Correct answer is E


45% answered correctly

Explanation:

Correct Answer Is E

The chronic copious sputum production that is yellow-green in color is highly suggestive of bronchiectasis as the underlying lung condition, over which a lobar pneumonia has superimposed.

Bronchiectasis is the irreversible dilation of one or more of the conducing bronchi or airways, most often secondary to an infectious process.

Bronchiectasis presents with the following:

Cough and daily mucopurulent sputum production, often lasting months to years (classic)
Blood-streaked sputum or hemoptysis from airway damage associated with acute infection
Dyspnea, pleuritic chest pain, wheezing, fever, weakness, fatigue, and weight loss
Rarely, episodic hemoptysis with little to no sputum production (i.e. dry bronchiectasis)

Exam findings in a patient with bronchiectasis are often non-specific and include:

Crackles, rhonchi, scattered wheezing, and inspiratory squeaks on auscultation


Digital clubbing (2-3% of patients; more frequent in moderate-to-severe cases)
Cyanosis and plethora with polycythemia from chronic hypoxia (rare)
Wasting and weight loss
Nasal polyps and signs of chronic sinusitis
Physical stigmata of cor pulmonale, in advanced disease

Exacerbation of bronchiectasis is characterized by:

Increased sputum production over baseline


Thickening of the sputum
Malodorous sputum (occasional)
Low-grade fever (a rare finding)
Increased fatigue and/or malaise
Increased dyspnea, shortness of breath, wheezing, or pleuritic chest pain

Exacerbations of bronchiectasis often require oral antibiotic therapy with amoxicillin or doxycycline. Patients in whom pseudomonas aeroginosa is isolated from sputum culture require a quinolone such as ciprofloxacin or moxifloxacin to cover this
organism.

This patient, however, has a lobar pneumonia that demands a different antibiotic therapy from that used for an exacerbation of bronchiectasis. It is estimated that a significant number of patients with bronchiectasis are colonized or have chronic
infection with pseudomonas aeroginosa; therefore, for this patient an antibiotic with adequate coverage against this pathogen should be considered as first-line empiric therapy until susceptibility is available. Ceftazidime or a synthetic penicillins
with anti-pseudomonas activity such as piperacillin + tazobactam or ticarcillin +clavulanic acid, or meropenem are first-line antibiotics to consider for this patient.

Oral roxythromycin (a macrolide) (option A) or oral Augmentin (amoxicillin + clavulanic acid) (option B) were appropriate options if the case was an exacerbation of bronchiectasis not pneumonia.

Ampicillin (option D) and flucloxacillin (option E) do not provide adequate coverage against pseudomonas.

References

• https://emedicine.medscape.com/article/296961-overview

• Therapeutic Guideline – available from: http:tg.org.au

Last updated:
Time spent: QID:1249 2023-2-12

685 of 1943
A 25-year-old man presents to the emergency department of a tertiary hospital with complaint of back pain. The pain is felt over the L4/L5 region. He denies any history of preceding back trauma or sprain, but admits to injecting drug use previously.
He also mentions positivity for hepatitis C virus antibody. Remarkable finding on physical examination is erythema and tenderness over the L4/L5 area. Which one of the following would be the most appropriate diagnostic approach to reach a
diagnosis in this patient?

A. CT scan of the lumbar area.

B. MRI of the lumbar area.

C. HIV serology.

D. Bone scan.

E. X-ray of the lumbar area.

Incorrect. Correct answer is B


45% answered correctly

Explanation:

Correct Answer Is B

Local tenderness and erythema over a bony area in a patient with history of injection drug abuse should raise the suspicion of osteomyelitis as a very likely diagnosis.

This possibility is increased given the positive hepatitis C antibody titer that indicates hepatitis C infection and the likelihood of other infections associate with shared needle use. Needle sharing is associated with a significant risk of hematogenous
spread of infections to bones, joints and soft tissue.

NOTE - Osteomyelitis can occur as a result of contiguous spread of infection from adjacent soft tissue and joints, hematogenous seeding, or direct inoculation into the bone as a result of trauma or surgery. Hematogenous osteomyelitis is more
common in children than in adults. In children, long bones are most often affected, while the vertebral column is the most common site of involvement with hematogenous spread in adults.

Typically, acute osteomyelitis presents with gradual onset of symptoms over several days. There is often dull pain in the affected area that can be related or unrelated to movement. There might be local findings such as erythema, swelling,
tenderness or warmth, or systemic manifestations such as fever and rigors. However, patients with osteomyelitis of the hip, vertebrae or pelvis may have few signs and symptoms other than pain (such as in this patient).

MRI is the investigation of choice in diabetic patients with suspected osteomyelitis of the foot and all patients with suspected osteomyelitis of the vertebral column.

(Option A) CT scan is use for evaluation if MRI is contraindicated or not available.

(Option D) A bone scan is considered for patients with suspected osteomyelitis and contraindications to MRI and CT scan (e.g. metal hardware).

(Option C) While HIV and decreased immunity is a risk factor for osteomyelitis as well as other infections, HIV positivity is not diagnostic for osteomyelitis. It is necessary though to check, as well as hepatitis B status, in a patient with history of
intravenous drug use.

(Option E) ESR and full blood exam are tests routinely consider but they are not specific. Findings associated with osteomyelitis are leukocytosis and elevated ESR (and often CRP) which are common findings in many other conditions than
osteomyelitis.

NOTE - An X-ray is the initial diagnostic modality to consider in patients presenting with signs and symptoms of osteomyelitis. X-rays however are usually negative and not sensitive in the first 2-3 weeks of infection. X-ray becomes positive when at
least 50% of the mineral bone is lost due to infection.

References

• https://emedicine.medscape.com/article/1348767-overview

• https://www.uptodate.com/contents/approach-to-imaging-modalities-in-the-setting-of-suspected-osteomyelitis

Last updated:
Time spent: QID:1280 2023-2-12

686 of 1943
An indigenous woman has brought her three-year-old son to your GP clinic on the advice of an Aboriginal liaison in her community. She was diagnosed with pulmonary tuberculosis 13 months ago and was treated with standard quadruple therapy.
She wants to know if her son may have contacted TB from her. On physical examination, the child has no symptoms. You order a Quantiferon-TB Gold test for the child, the result of which comes back positive. Which one of the following is the most
appropriate next step in management of this child?

A. Start him on isoniazid.

B. Obtain a chest X-ray.

C. Repeat the Quantiferon-TB Gold test.

D. Start him on quadruple therapy.

E. Obtain sputum sample for acid fast bacilli microscopy and culture.

Incorrect. Correct answer is B


45% answered correctly

Explanation:

Correct Answer Is B

This child has been in close contact with a person with active TB (his mother), and there are chances that he also has contracted the infection. In fact, the positive Quantiferon-TB Gold test indicates that he has TB infection. The next question to
answer is whether he has latent TB infection (LTBI) or active TB infection.

With a positive Quantiferon-TB Gold (IGRA) or tuberculin skin test (TST) [also termed as Mantoux test] in an asymptomatic patient, the next step is always obtaining a chest X-ray. If the chest X-ray is clear, the patient does not have active TB and
should be treated for LTBI by monotherapy with isoniazid for 6-9 months.

If the chest X-ray shows abnormalities in favor of TB infection, the next is obtaining three sputum samples for acid fast bacilli (AFB) stain and culture. If all the three samples are negative, active TB infection is excluded and the patient should be
treated for LTBI. With even one positive sample, active TB infection is the diagnosis, and the patient should be treated with standard quadruple therapy.

This child is asymptomatic and has a positive Quantiferon-TB Gold test. For him, obtaining an X-ray would be the most appropriate next step in management.

(Option A) Starting the child on isoniazid alone for treatment of LTBI is considered if investigations, as outlined above, show he does not have active TB infection.

(Option C) Quantiferon-TB Gold test has a very high specificity. Once it is positive, TB infection is almost certain, and there is no need for the test to be repeated.

(Option D) Quadruple therapy with rifampicin, isoniazid, pyrazinamide and ethambutol (RIPE regimen) is used for treatment of active TB infection. Active TB is not an established diagnosis for this child yet.

(Option E) Obtaining sputum sample for acid fast bacilli (AFB) stain and culture would be the correct answer if the X-ray shows abnormalities that support TB as a diagnosis.

TOPIC REVIEW

Following exposure and inhalation of airborne tubercle bacilli, cellular immune defenses usually contain the infection so patients do not become symptomatic. When infection is contained by the immune system it remains dormant, usually within the
lung apices. This is termed 'latent tuberculosis infection (LTBI).'

Overall, a person with LTBI has a 10% risk of developing active TB during their lifetime, with the greatest risk being within 2 years of infection. Once reactivated, symptoms such as fever, night sweats, weight loss and cough develop.

A test for LTBI should be performed to identify any person who might be at an increased risk for TB and might benefit from treatment of latent TB infection. In summary, testing for LTBI should be considered in the following situations:

Recent contacts of individuals known to have, or who are suspected of having clinically active TB
Casual contacts (for example, visitors at home, at work or at clubs) should be tested only if the source case is considered highly infectious (laryngeal disease, strongly smear positive cavitary pulmonary disease, or endobronchial
tuberculosis). If the initial test is negative, consider retesting at 8 weeks.
Those with HIV infection
Those with abnormal chest X-rays suggestive of previous TB
Those expected to be at substantially increased risk of TB disease (for example, in preparation for renal transplant or significant immunosuppression)
Groups at high risk of recent infection with M. tuberculosis, such as refugees and asylum seekers. Other groups, including immigrants from Asia, Africa, Central America, Oceania, Eastern Europe and the former Soviet Union, medically
under-served populations, personnel and long-term residents in some hospitals, nursing homes, mental institutions, and correctional facilities do not require routine screening, but may be considered for testing on an individual basis
Healthcare workers

To diagnose the latent TB, 2 tests can be used:

Tuberculin skin test (TST), also known as Mantoux test or PPD test
Interferon gamma release assays (IGRA), also known as Quantiferon-TB Gold test

NOTE - Australia's National Tuberculosis Advisory Committee suggests TST as the preferred method of testing, with IGRA as a supplemental assay in patients older than 2 years. IGRA is easier to interpret in patients who are vaccinated with
BCG or are immunocompromised.

TST (Mantoux)

TST includes assessment of the skin inflammation 48-72 hours after intradermal (not subcutaneous) injection of 0.1ml of tuberculin protein (purified protein derivative or PPD). The diameter of the induration (not the erythema), measured across the
arm (not along it) gives a semi-quantative assessment of the likelihood of LTBI.

TST is considered positive if the induration is ≥5mm in the following groups:

Recent high risk (close) contacts of persons with infectious TB


Individuals with organ transplants or immune suppressive therapy equivalent to prednisone >15mg/day for >1 month
Individuals with HIV infection
Individuals with CXR evidence of past untreated TB

687 of 1943
TST is considered positive if the induration is ≥10 mm in the following groups:

Individuals born or resident (for greater than 3 months) in countries with high prevalence of TB (>100 cases/100,000)
Children < 4 years of age without any identified risk factors
Individuals who live or spend time in high risk congregate settings (e.g. prisons, homeless shelters, alcohol rehabilitation and drug treatment centers
Health care workers without prior BCG vaccination in the past 10 years
Intravenous drug users

TST is considered positive if the induration is ≥15 mm in the following groups:

Normal population other than the mentioned groups.

Factors that influence interpretation of TST are listed in the following table:

Factors that may DECREASE skin reaction and give false-negative results
Infections:

Viral e.g. HIV, measles, mumps, chickenpox, etc


Bacterial e.g. pertussis, brucellosis, leprosy, etc
Fungal

Live virus vaccination e.g. measles, mumps, polio, etc


Metabolic diseases e.g. chronic renal failure
Malnutrition/ protein depletion
Lymphoid neoplasms e.g. Hodgkin’s disease, lymphoma, CML
Sarcoidosis
Drugs e.g. corticosteroids, immunosuppressants
Age – newborn and elderly
TB infection acquired within the past 8 weeks
Other causes leading to cell-mediated immune response suppression
Local skin damage e.g. trauma, dermatitis, surgery
Incorrect storage and handling of tuberculin (PPD)
Poor technique of injection and misreading
Factors that may INCREASE skin reaction and give false-positive results
Exposure to or infection with non-tuberculosis mycobacteria
Past BCG vaccination
Trauma and irritation to the site of injection prior to reading
Misreading

Interferon Gamma Release Assay (IGRA) (Quantiferon-TB Gold test)

Unlike TST, IGRA is unaffected by previous BCG vaccination. A positive IGRA suggests that the patient’s immune system recognizes M. tuberculosis due to either current infection or a past infection. A negative test cannot exclude active TB because
sensitivity is not 100%, but a positive result is highly suggestive of TB infection.

NOTE - TST remains the preferred method with IGRAs as a supplemental assay in subjects more than 2 years of age. TST alone is recommended for those ≤2 years of age.

If a TST or IGRA is positive, active disease must be excluded with chest X-ray as the next best step in management. If the chest X-ray is normal, active TB is excluded and the patient should only be treated for LTBI with isoniazid (INH) for 6-9 months.
Supplementation with pyridoxine (vitamin B6) during is required during therapy.

If the chest X-ray shows abnormalities consistent with TB, sputum should be examined for acid-fast bacilli (AFB) stain (Ziehl Neelsen) and culture. An abnormal chest X-ray and three negative sputum smears excludes active TB and the patient should
only be treated for LTBI as mentioned above.

With even one positive sputum exam, the patient have active TB infection and should be started on standard full-dose four-drug therapy (RIPE regimen) after discussion with and supervision of a clinician experienced in TB management.

References

• RACGP - Tuberculosis testing

• Australian Prescriber - Testing for tuberculosis

• Therapeutic Guidelines – Antibiotics: available from http://tg.org.au

Time spent: QID:1393 Last updated:


2023-2-12

688 of 1943
A 45-year-old man presents to your practice with complaints of fever, joint pain and a generalized rash all over his body. His fever and other symptoms started this morning, five days after he returned from Thailand. On examination, maculopapular
rash over his limbs and trunk are noted. Despite generalized severe joint pain, no arthritis is noted on exam. Abdominal exam is normal and there is no lymphadenopathy. A full blood exam (FBE) is performed, the result of which is as follows:

Hb: 135 g/L (normal: 130-180 g/L)


RBC: 5x106/mm3 (4.5x106-6.5x106/ mm3)
MCV: 85 fL (76-96 fL)
WCC: 3100/mm3 (4000-11000/ mm3)
Platelet: 75000/mm3 (150,000-400,000/mm3)

Which one of the following is the most likely diagnosis?

A. Malaria.

B. Leptospirosis.

C. Dengue fever.

D. HIV infection.

E. Infectious mononucleosis.

Incorrect. Correct answer is C


45% answered correctly

Explanation:

Correct Answer Is C

The clinical findings of fever, arthralgia and a generalized rash, as well thrombocytopenia and leukopenia in a traveller to an endemic area is consistent with Dengue fever as the most likely diagnosis.

Dengue fever is the most prevalent mosquito-borne viral disease; it is estimated that over 50 million dengue virus infections occur each year throughout the world. Dengue fever is widespread in the south-east Pacific and endemic in Queensland.
Symptomatic dengue virus infections can present with a wide range of clinical manifestations, from a mild febrile illness to a life-threatening shock syndrome or organ dysfunction.

In its classic form, dengue fever is an acute febrile illness with the following manifestations:

Fever – 100%
Nauseas – 37%
Headache – 68%
Retro-orbital pain
Marked muscle and joint pains – the disease is sometimes called ‘break bone fever’. Myalgia is seen in 79% of patients
Rash
Sore throat

Fever typically lasts up to 7 days. The fever can be biphasic in a minority of patients. It subsides and then returns in about two days.

Typical rash of Dengue fever is similar to that of rubella. It starts on the limbs and progress to involve the trunk. In some patients petechial rash is present, even in the absence of thrombocytopenia. Hemorrhagic dengue fever is a rare form of the
disease characterized by life-threatening hemorrhage. Patients may experience fatigue and depression after recovery.

Laboratory findings in patients with dengue fever are leukopenia and thrombocytopenia (Plt<100,000). Leukopenia is a characteristic finding seen in both children and adults. A 2-5 times rise in AST is in a frequent finding; however, marked elevation
can also occur.

(Option A) Malaria should be always suspected in every returning traveller who is febrile; however, with the presence of rash, arthralgia, leukopenia, thrombocytopenia and the absence of anemia, dengue fever is a more likely diagnosis.

NOTE – A returned traveller with myalgia and fever <39°C is more likely to have dengue fever than malaria.

(Option B) Leptospirosis is among differential diagnoses of dengue fever and should be thought of in appropriate setting. In Australia, leptospirosis is almost exclusively an occupational disease seen in farmland workers and meat industry workers.
Leptospirosis follows contamination of abraded or cut skin or mucous membrane with Leptospira-infected urine of animals such as sheep, goat, pig, horse, rat and dog. Clinical manifestations include fevers, chills, myalgia, and headache and light-
sensitive conjunctivitis. The absence of such contacts in history makes leptospirosis a remote possibility.

(Option D) HIV infection is a great mimic and can be among differential diagnosis of many febrile illnesses; however, with travel to Thailand in history, characteristic laboratory findings and absence of lymphadenopathy acute HIV infection is less
likely than dengue fever as a diagnosis.

(Option E) Sore throat and generalized lymphadenopathy are characteristic features of infectious mononucleosis, the absence of which make such diagnosis less likely. Furthermore, lymphocytosis is a characteristic finding in infectious
mononucleosis. This patient has leukopenia, not leukocytosis. This makes infectious mononucleosis a less likely diagnosis.

References

• UpToDate - Clinical manifestations and diagnosis of dengue virus infection

Last updated:
Time spent: QID:1403 2023-2-12

689 of 1943
A 45-year-old man, who has recently migrated to Australia, presents with recent history of productive cough and yellow sputum that is sometimes associated with streaks of blood. He has lost seven kg in the past three months. He smokes 10
cigarettes a day. On examination, he has normal vital signs. There are few small painful nodules on his legs that are shown in the following photograph. The rest of the examination is inconclusive. Which one of the following could be the most likely
diagnosis?

A. Bronchogenic carcinoma.

B. Pulmonary tuberculosis.

C. Sarcoidosis.

D. Pneumococcal pneumonia.

E. Atypical pneumonia.

Incorrect. Correct answer is B


45% answered correctly

Explanation:

Correct Answer Is B

The clinical description of the nodules alongside their appearance in the photograph is highly suggestive of erythema nodosum (EN).

EN is an acute, nodular, erythematous eruption usually limited to the extensor aspects of the lower legs. Rarely, EN can be chronic or recurrent. EN assumed to be a hypersensitivity reaction and may occur in association with several systemic
diseases or drug therapies, or it may be idiopathic. EN is seen in the following conditions:

Bacterial infections
Streptococcal infection – one of the most common causes of EN
Mycoplasma tuberculosis – TB should always be considered in developing countries
Yersinia enterocolitica
Mycoplasma pneumonia
Salmonella infection
Fungal infections – Coccidioidomycosis
Drugs
Sulfonamide and halide agents – important causes of drug-induced EN
Gold
Sulfonylureas
Oral contraceptive pills
Enteropathies – ulcerative colitis and Crohn disease can trigger EN. EN associated with enteropathies correlate with flares of the disease. The mean duration of chronic ulcerative colitis before the onset of erythema nodosum is five
years. EN is the most frequent dermatologic symptom in inflammatory bowel disease (IBD), and it is strongly associated with Crohn disease.
Hodgkin disease and lymphoma
Sarcoidosis - the most common cutaneous manifestation of sarcoidosis
Behçet disease
Pregnancy

Pulmonary tuberculosis and sarcoidosis share so many characteristic features that they might be confused clinically. Both pulmonary TB and sarcoidosis present with weight loss, malaise, fever, cough and sputum as well as EN; however,
hemoptysis (coughing up blood) is a rare and unusual finding yet not impossible finding in sarcoidosis. Therefore, pulmonary TB is more likely to be the diagnosis than sarcoidosis (option C).

Streptococcal infections including pneumococcal pneumonia (option D) are very common causes of EN, but weight loss and chronicity of the symptoms make this diagnosis unlikely. Mycoplasma pneumonia causes atypical pneumonia (option E)
and may be associated with EN, but again the weight loss makes it a less likely possibility.

The association between bronchogenic carcinoma (option A) and EN has not yet been established.

References

• Medscape - Erythema Nodosum

• Medscape - Sarcoidosis

Time spent: QID:1502 Last updated:


2023-2-12

690 of 1943
A 22-year-old man presents to your practice in Darwin (Northern Territory) with complaints of rash and malaise and fatigue for the past few weeks. He is originally from Asia but has been living in Darwin for the past 3 months as a backpacker. On
examination, vital signs including temperature are within normal limits. He has a maculopapular rash over the soles and palms that are non-pruritic, and cervical, axillary, and inguinal lymphadenopathy which is not tender. There are also two small
alopecic patches on his head. Which one of the following is the most appropriate treatment option for him?

A. Doxycycline.

B. Benzyl penicillin.

C. Azithromycin.

D. Procaine penicillin.

E. Intramuscular ceftriaxone.

Incorrect. Correct answer is B


45% answered correctly

Explanation:

Correct Answer Is B

The scenario and the clinical findings are highly suggestive of secondary syphilis as the most likely diagnosis for which penicillin is the standard treatment.

Syphilis is a sexually transmissible infection caused by the spirochete Treponema pallidum. Syphilis is transmissible by sexual contact with infectious lesions, from mother to fetus in utero, via blood product transfusion, and occasionally through
breaks in the skin that come into contact with infectious lesions. If untreated, it progresses through 4 stages: primary, secondary, latent, and tertiary.

Primary syphilis

Primary syphilis occurs 10-90 days after contact with an infected case, and mainly manifests on the glans penis in males and on the vulva or cervix in females. Almost 10% of syphilitic lesions are found on the anus, fingers, oropharynx, tongue,
nipples, or other extragenital sites. Regional nontender lymphadenopathy follows invasion.

Lesions (chancres) usually begin as solitary, raised, firm, red papules that can be as large as several centimeters in diameter. The chancre erodes to create an ulcerative crater within the papule, with slightly elevated edges around the central ulcer. It
usually heals within 4-8 weeks, with or without therapy. The lesions may be small and be overlooked or unnoticed by the patients.

Chancre of primary syphilis - penis

Chancre of primary syphilis - lip

691 of 1943
Chancre of primary syphilis - tongue

Secondary syphilis

Secondary syphilis manifests in various ways. It usually presents with a cutaneous eruption within 2-10 weeks after the primary chancre and is most florid 3-4 months after infection. The eruption may be subtle; 25% of patients may be unaware of
skin changes. A localized or diffuse mucocutaneous rash (generally nonpruritic and bilaterally symmetrical) with generalized nontender lymphadenopathy is typical. The rash may be varied and resemble any other skin condition. Patchy alopecia and
condylomata Lata may also be observed.

Mild constitutional symptoms of malaise, headache, anorexia, nausea, aching pains in the bones, and fatigue often are present, as well as fever and neck stiffness. A small number of patients develop acute syphilitic meningitis and present with
headache, neck stiffness, facial numbness or weakness, and deafness.

Other less-common manifestations include gastrointestinal tract involvement, hepatitis, nephropathy, proctitis, arthritis, and optic neuritis.

Rash of secondary syphilis

692 of 1943
Rash of secondary syphilis

rash of secondary syphilis

693 of 1943
alopecia of second syphilis

Condyloma lata of secondary syphilis - vulvar

Latent syphilis

Latency may last from a few years to as many as 25 years before the destructive lesions of tertiary syphilis manifest. Affected patients may recall symptoms of primary and secondary syphilis. They are asymptomatic during the latent phase, and the
disease is detected only by serologic tests.

Latent syphilis is divided into early latent and late latent. The distinction is important because treatment for each is different. The early latent period is the first year after the resolution of primary or secondary syphilis. Asymptomatic patients who
have a newly active serologic test after having a serologically negative test result within 1 year are also considered to be in the early latent period. Late latency syphilis is not infectious; however, women in this stage can spread the disease in utero.

Tertiary syphilis

Tertiary (late) syphilis is slowly progressive and may affect any organ. The disease is generally not thought to be infectious at this stage. Manifestations may include the following:

Impaired balance, paresthesias, incontinence, and impotence


Focal neurologic findings, including sensorineural hearing and vision loss
Dementia
Chest pain, back pain, stridor, or other symptoms related to aortic aneurysms

The lesions of gummatous tertiary syphilis usually develop within 3-10 years of infection. The patient complaints are usually secondary to bone pain, which is described as a deep boring pain characteristically worse at night. Trauma may predispose
a specific site to gumma involvement.

CNS involvement may occur, with presenting symptoms representative of the area affected. Patients with brain involvement may present with headache, dizziness, mood disturbance, neck stiffness, or blurred vision while spinal cord involvement can
present with bulbar symptoms, weakness and wasting of shoulder girdle and arm muscles, incontinence, and impotence.

Some patients may present up to 20 years after infection with behavioral changes and other signs of dementia, which is indicative of paresis.

Treatment of syphilis is by intramuscular injection using procaine penicillin or benzathine penicillin:

694 of 1943
Benzathine penicillin 1.8 g (2.4 mU) IM as a single dose
Procaine penicillin 1.0 g IM daily for 10 days

The best option to consider in this patient is benzathine penicillin because procaine penicillin must be given as a daily injection for 10 days, which can be uncomfortable and inconvenient for the patient. Doxycycline is the alternative for non-pregnant
patients with allergy to penicillin.

Management of the infected individual should include an assessment of sexual risk and provision of information and support to reduce risks for re-infection or acquisition of other STIs.

References

• RACGP – AFP – Syphilis, the great mimicker, is back

Time spent: QID:1660 Last updated:


2023-2-12

695 of 1943
A 35-year-old man, who is a know case of HIV infection, presents to the Emergency Department with complaints of fever, cough, and sputum for the past 48 hours. On examination, he has a blood pressure of 110/67 mmHg, PR of 110 bpm,
temperature of 39.1°C, and respiratory rate of 32 breaths per minute. Chest exam is remarkable for dullness below the right scapula. A chest X-ray confirms a consolidation in the lower lobe of the right lung. In blood tests, white cell count of 14,000,
ESR of 30, CRP of 21 mg/d and CD4 count of 55/mm3 are of significance. Which one of the following could be the most likely diagnosis?

A. Pneumocystis jiroveci pneumonia.

B. Mycoplasma pneumonia.

C. Staphylococcal pneumonia.

D. Streptococcal pneumonia.

E. Viral pneumonia.

Incorrect. Correct answer is D


45% answered correctly

Explanation:

Correct Answer Is D

With cough, sputum, and fever developing acutely and focal findings on physical examination and chest x-ray (CXR), bacterial pneumonia is the most likely diagnosis. Streptococcus pneumoniae is the most common cause of bacterial pneumonia
both in general population and individuals with compromised immunity such as HIV infected patients.

(Option A) Pneumocystis jiroveci (P. jiroveci) (formerly known as pneumocystis carinii) is a ubiquitous fungus. By far, P. jiroveci pneumonia (PJP) is the most common AIDS-defining condition and usually is seen in HIV- infected individuals late in the
course of HIV infection, or those who have poor adherence to PJP prophylaxis. JPJ is uncommon in people with CD4 counts of over 200.

PJP presents sub-acutely with fever, non-productive (dry) cough, chest tightness, dyspnea, and tiredness. Symptoms may be present for 2 to 6 weeks or even more before a certain diagnosis of JPJ is made. Patients typically are immunodeficient
(CD4+ T cell count < 200/μL), with failed suppression of HIV replication, and not taking cotrimoxazole prophylaxis, who present with fatigue and fever. The patient may not necessarily have noticed cough or dyspnea, yet a non-productive cough is
commonly apparent during the clinical assessment. Respiratory examination is often unremarkable except for probable tachypnea and oxygen desaturation on exertion. Crackles may be present on auscultation.

(Option B) Mycoplasma pneumonia comprises up to 40% of CAPs. It has a gradual and insidious onset of several days to weeks. The patient's history may include the following:

Fever, generally low-grade


Malaise
Persistent, slowly worsening, incessant cough. The cough ranges from non-productive to mildly productive with sputum discoloration developing late in the course of the illness. The absence of cough makes the diagnosis
of mycoplasma pneumonia unlikely.
Headache
Chills but not rigors
Scratchy sore throat
Sore chest and tracheal tenderness as results of the protracted cough
Pleuritic chest pain (rare)
Wheezing
Dyspnea (uncommon)

The clinical picture of rapid-onset symptoms of high-grade fever and productive cough and the absence of the other features of mycoplasma pneumonia make such diagnosis less likely.

(Option C) Pneumonia caused by staphylococcus aureus has a rapid course with fever, and productive cough, which often does not satisfactorily respond to antibiotic therapy. Cavitation, abscess formation, empyema, and pleural effusion are more
likely to develop and be present on assessment compared to streptococcal pneumonia. Staphylococcal pneumonia is a possibility in this patient as well, but streptococcal pneumonia remains more likely as it is more common.

NOTE - Staphylococcal pneumonia is less likely of a diagnosis in community acquired pneumonia (CAP) and is more likely to occur in hospitalized or immunocompromised individuals.

(Option E) Viral pneumonia is usually preceded by flu-like symptoms which are absent in this patient’s history. Moreover, CXR findings in viral pneumonia are typically bilateral patchy infiltrates which is different from CXR findings of this patient.

References

• ASHM – Infectious diseases and cancers caused by HIV-induced immunodeficiency: Bacteria pneumonia

Last updated:
Time spent: QID:1680 2023-2-12

696 of 1943
Melany, 75 years old, was admitted to the hospital 5 days ago after she was diagnosed with bacterial community acquired pneumonia. She was started on intravenous antibiotics with a desirable therapeutic response and feeling much better. on her
5th day admission, she suddenly develops a high fever of 39.5°C, chills, and rigors. Which one of the following could be the most likely explanation to this clinical picture?

A. Hospital acquired pneumonia.

B. Lung abscess.

C. IV cannula-related bacteremia.

D. Empyema.

E. Pulmonary embolism.

Incorrect. Correct answer is C


45% answered correctly

Explanation:

Correct Answer Is C

Melany develops high grade fever, chills, and rigors after recovering from her pneumonia owing to a favorable response to intravenous antibiotics. Her current symptoms and sudden nature of onset is well aligned with bacteremia. IV cannula-
associated bacteremia and sepsis is the most common cause of bloodstream infections in hospitalized patients in Australia.

Empyema and lung also can also complicate pneumonia. However, the expected clinical course of empyema and lung abscess in Melany would have been poor or no response to initial antibiotics followed by the worsening of existing or addition of
new symptoms. Melany had a favorable response to antibiotic therapy and was feeling well before new symptoms develop. This makes lung abscess (option B) and empyema (option D) less likely. Moreover, these conditions tend to develop
insidiously rather than suddenly.

Hospital acquired pneumonia (HAP) (option A) is defined as pneumonia occuring 48 hours or more after hospital admission and not incubating at the admission time. Timeline-wise, HAP could also be a possibility for Melany; however, it the case
was pneumonia, worsening of previous symptoms or build-up of new would be expected which is not mentioned in the scenario. On the other hand and from commonality point of view, IV cannula-associated bacteremia is a more likely possibility
compared to HAP.

Acute pulmonary embolism (option E) presents with sudden onset pleuritic chest pain and shortness of breath. It is unusual for pulmonary embolism to present with high fever, chills, and rigor.

References

• Australian Prescriber – Controlling intravascular catheter infections

• Medscape- Management of Catheter-related Infection

Last updated:
Time spent: QID:1682 2023-2-12

697 of 1943
A12-year-old boy is brought to your clinic with complaint of an eyelid swelling that developed one week ago. The lesion is illustrated in the following photograph. The child is otherwise healthy. Which one of the following would be the most likely
diagnosis?

A. Stye.

B. Blepharitis.

C. Carbuncle.

D. Chalazion.

E. Periorbital cellulitis.

Incorrect. Correct answer is D


45% answered correctly

Explanation:

Correct Answer Is D

The photograph shows a swelling of the right upper eyelid. At the first look, it appears to be either hordeolum (stye) or chalazion (infection of a meibomian gland). With the lesion being distal to the eyelid, chalazion would be more likely because stye
tends to be located more marginally. With resolution of acute infection, the abscess gives rise to nodule formation, namely meibomian cyst. It is not inflamed but may be locally irritating.

(Option A) Stye is, in fact, the carbuncle of eyelash where the eyelash follicle becomes obstructed and infected. Stye is located more marginally and has inflammatory features.

(Option B) Blepharitis presents with inflamed margins of the eyelids. It may be caused by Staph aureus, seborrheic dermatitis, or rosacea. It presents with marginal itching and burning. The management is with removal of crusts from the lids and
steroid drops or creams. For blepharitis associated with staph aureus, tetracycline or chloramphenicol ointments are used. Oral doxycycline in indicated in those with blepharitis associated with rosacea.

(Option C) Carbuncle of the eyelash (stye) presents at the margin of the eyelid.

(Option E) Periorbital cellulitis presents differently with warmth, erythema and swelling around the orbit and often systemic upset. Absence of such findings makes this diagnosis unlikely.

References

• http://emedicine.medscape.com/article/1212709-over

Last updated:
Time spent: QID:289 2023-2-12

698 of 1943
A 25-year-old male patient comes to the Emergency Department (ED) complaining of a 3-day history of productive cough, colds, and fever. On further probing, you find that the patient has no other comorbidities and does not have any pertinent travel
history. Vital signs of the patient show a heart rate of 125, respiratory rate of 32, BP of 130/80 mmHg, and O2 saturation of 90% on room air. Physical exam findings indicate bilateral crackles on all lung fields, and the patient speaks only in short
phrases. You arrange for a workup showing the chest x-ray (CXR) as illustrated below and find that the patient is positive for COVID-19 on reverse transcription polymerase chain reaction. Which of the following is the most important in the
management of severe COVID-19?

A. Dexamethasone.

B. Barcitinib.

C. Remdesivir.

D. Azithromycin.

E. Enoxaparin.

Correct
45% answered correctly

Explanation:

Correct Answer Is A

COVID-19 severity is classified according to the following criteria:

Mild:

Adults not presenting any clinical features suggestive of moderate or severe disease or a complicated course of illness.

Characteristics:

No symptoms, or
mild upper respiratory tract symptoms, or
cough, new myalgia, or asthenia without new shortness of breath or a reduction in oxygen saturation

Moderate:

Stable adult patients presenting with respiratory and/or systemic symptoms or signs. Able to maintain oxygen saturation above 92% (or above 90% for patients with chronic lung disease) with up to 4 L/min oxygen via nasal prongs.

Characteristics:

prostration
severe asthenia
fever > 38 ̊C or persistent cough
clinical or radiological signs of lung involvement
no clinical or laboratory indicators of clinical severity or respiratory impairment

Severe:

Characteristics:

Any one of the following:

Respiratory rate ≥ 30 breaths/min


Oxygen saturation ≤ 92% at a rest state
The arterial partial pressure of oxygen (PaO2)/ inspired oxygen fraction (FiO2) ≤ 300

Critical:

699 of 1943
occurrence of severe respiratory failure (PaO2/FiO2 < 200), or
respiratory distress or acute respiratory distress syndrome (ARDS), or
respiratory deterioration despite advanced forms of respiratory support (NIV, HFNO), or
patients requiring mechanical ventilation, or
other signs of significant deterioration
hypotension or shock
impairment of consciousness
other organ failures

The patient’s chest x-ray findings show peripheral focal areas of ground glass opacities and consolidation at the bilateral middle and lower lung zones. When correlated with clinical findings, this patient meets the criteria for severe COVID due to a
respiratory rate of > 30 breaths per minute and oxygen levels of less than 92%.

Of all the following treatment options, only dexamethasone (6mg intravenously or orally for up to 10 days) has been shown to reduce mortality for patients requiring oxygenation. Dexamethasone should be started within 14 days of symptoms onset.
Dexamethasone is also safe and recommended in patients who are pregnant or breastfeeding. It is not recommended for routine use if the patient does not require oxygen, even if patients are at risk for developing disease progression.

NOTE - risk factors for disease progression are as follows:

Older age (e.g., over 65 years, or over 50 years for Aboriginal and Torres Strait Islander people)
Diabetes requiring medication
Obesity (BMI >30 kg/m2)
Renal failure
Cardiovascular disease, including hypertension
Respiratory compromise, including COPD, asthma requiring steroids, or bronchiectasis
Immunocompromising condition

If dexamethasone is unavailable, the following could be the alternatives:

hydrocortisone: intravenous (50 mg), every 6 hours for up to 10 days


prednisolone: oral (50 mg), daily for up to 10 days
methylprednisolone may also be an acceptable alternative, however, the most appropriate dosage is uncertain

Barcitinib (option B) is also an approved medication for the treatment of severe to critical COVID-19 patients (including those on a ventilator or ECMO), however, the guidelines as of September 2022 only recommend this as a conditional treatment.

Remdesivir (option C) is also listed as a conditional treatment for COVID-19 patients who require oxygen, but not in those with non-invasive or invasive ventilation. Although the recommendation differs from that of the WHO, in Australia it is only used
within the context of clinical trials or is considered in unvaccinated individuals.

Azithromycin (option D) is an antibacterial medication that is not recommended for COVID-19.

Enoxaparin (low-molecular-weight heparin) is used for venous thromboembolism (VTE) prophylaxis in moderate to severe COVID-19. It is still under study as to whether there is a clear benefit or harm. It is contraindicated in those with a significant
risk of bleeding.

Additionally, the following medications are not recommended for the treatment of COVID-19:

aspirin
azithromycin
interferon β-1a
ivermectin
lopinavir-ritonavir
convalescent plasma
colchicinefavipiravir
hydroxychloroquine

NOTE - The following medications were approved for use last January 18, 2022:

molnupiravir (Lagevrio®)
nirmatrelvir + ritonavir (Paxlovid®)

References

• Australian National COVID-19 Clinical Evidence Taskforce

• Australian Guidelines for the Clinical Care of people with COVID-19

Time spent: QID:1766 Last updated:


2023-2-12

700 of 1943
A 25-year-old woman presents to your practice complaining of a painful lump in her left eyelid. The lesion is shown in the following photograph. Which one of the following is the most likely diagnosis?

A. ​Stye.

B. Chalazion.

C. Meibomian cyst.

D. Dacryocystitis.

E. Blepharitis.

Correct
45% answered correctly

Explanation:

Correct Answer Is A

Acute abscess of a lash follicle or associated glands of the anterior margin of the lid is termed hordeolum (stye). The causative organism is usually Staphylococcus aureus.

The picture shows a red swelling of the upper eyelid which may be seen in both chalazion and stye, but since stye tends to be located more marginally, the lesion is more likely to be an stye. Stye presents with a red tender swelling of the lid margin.

(Option B and C) Chalazion, also termed meibomian abscess/cyst (internal hordeolum), is the result of an obstructed and secondarily infected meibomian gland. Meibomian (tarsal) glands are special kinds of sebaceous glands at the rim of the
eyelid inside the tarsal plate. They produce and secret meibum, which is an oily substance that prevents evaporation of the eyes’ tear film and tear spillage onto the cheeks by trapping the tear between the oiled edge and the eyeball. It also makes the
closed lids airtight. There are about 50 glands in the upper and 25 in the lower lid. When an acute infection of a meibomian abscess subsides, the abscess within the gland is replaced by granulation tissue, and eventually becomes fibrotic. This will
form a small hard nodule in the eyelid.If the lesion was more away from the eyelid margin, chalazion could have been the better bet.

(Option D) Dacryocystitis is inflammation of the tear sac often secondary to obstruction of naso-lacrimal duct. It present with inflamed swelling over the medial canthus.

(Option E) Blepharitis presents with inflamed margins of the eyelids. It may be caused by Staphylococcus aureus, seborrheic dermatitis, or rosacea, and presents with itching and burning of lid margins. Management is with removal of crusts from the
lids and steroid drops or creams. For blepharitis associated with staph aureus tetracycline or chloramphenicol ointment is used. Oral doxycycline is indicated in those with blepharitis associated with rosacea.

References

• http://emedicine.medscape.com/article/798940-overv

• http://emedicine.medscape.com/article/1213080-over

Last updated:
Time spent: QID:290 2023-2-12

701 of 1943
A 4-year-old boy is brought to the emergency department by his mother because of fever and swelling of his left eye since this morning. Examination reveals an ill child with a fever of 38.5°C and right periorbital swelling. The eyelids are swollen and
erythematous. Further assessment reveals that eye movements are preserved in all direction. The visual acuity is intact. There is no nuchal rigidity. Which one of the following is the most appropriate next step in management?

A. Lumbar puncture.

B. Emergency CT scan of the orbital fossa.

C. Topical chloramphenicol ointment.

D. Intravenous flucloxacillin and ceftriaxone.

E. Oral antihistamine and topical corticosteroids.

Incorrect. Correct answer is D


45% answered correctly

Explanation:

Correct Answer Is D

The periorbital edema and erythema and fever are suggestive of either orbital or periorbital cellulitis. With orbital cellulitis, the eye movements are impaired. This leaves periorbital cellulitis as the most likely diagnosis. Periorbital cellulitis should be
urgently treated with intravenous broad spectrum antibiotics (e.g. flucloxacillin and ceftriaxone). If the child is vaccinated against Hemophilus influenza, some will advise a narrower-spectrum combination of antibiotics.

(Option A) In a child of this age, meningitis would be expected to be associated with neck rigidity. Without this finding,or other pointers, meningitis would be unlikely and lumbar puncture futile.

(Option B) Emergency CT scan of the orbital fossa to determine the extension prior to surgical intervention is the next best in management of orbital cellulitis.

(Option C) Periorbital cellulitis requires treatment with intavenous antibiotics. Topical antibiotics are not effective.

(Option E) Antihistamine or corticosteroids has no role in management of periorbital cellulitis.

References

• http://www.rch.org.au/clinicalguide/guideline_inde

• Therapeutic Guidelines – Antibiotic; available from http://tg.org.au

Last updated:
Time spent: QID:294 2023-2-12

702 of 1943
A 28-year-old man presents to your practice with redness and purulent discharge of his left eye as illustrated in the accompanying photograph. Which one of the following is the most effective antibiotic for treatment of this patient?

A. Tobramycin.

B. Streptomycin.

C. Chloramphenicol.

D. Penicillin.

E. Tetracycline.

Incorrect. Correct answer is C


45% answered correctly

Explanation:

Correct Answer Is C

The red eye, injection of the conjunctiva and mucopurulent discharge is suggestive of bacterial conjunctivitis as the most likely diagnosis. Suspected bacterial conjunctivitis should be empirically treated with a broad-spectrum topical antibiotic. For
this purpose, chloramphenicol is widely used as one of the most appropriate choices. The patient should always be advised about hygienic measures for prevention of spread to the other eye.

Swab culture is not initially indicated and is reserved if the infection persists or recurs despite initial treatment. Specific therapy then is planned based on the culture and susceptibility results.

References

• http://www.australianprescriber.com/magazine/17/3/

• Therapeutic Guidelines – Antibiotic; available from http://tg.org/au

Time spent: QID:295 Last updated:


2023-2-12

703 of 1943
A 34-year-old woman presents with a painful lump in the right upper eyelid. The lump has developed over a course of 3 days. On examination, there is a 1x1.5cm warm tender lump in the upper eyelid. The visual acuity is intact. The eye movements
are completely normal in all directions. The patient is otherwise healthy and afebrile. Which one of the following would be the next best step in management of this patient?

A. CT scan of the orbit.

B. Flucloxacillin and ceftriaxone intravenously.

C. Topical chloramphenicol.

D. Application of heat and massage.

E. Incision and curettage of the lesion under local anesthesia.

Incorrect. Correct answer is D


45% answered correctly

Explanation:

Correct Answer Is D

The painful lump in the eyelid in the presence of normal vision and preserved eye movements can be either stye (external hordeolum) or meibomian abscess (internal hordeolum). Both of these conditions are often managed conservatively with
application of heat and massage to allow drainage of infected contents.

This then is followed by oral di/flucloxacillin in meibomian abscess and sometimes by topical chloramphenicol in stye. Incision and curettage under local anesthesia (option E) is the treatment of last resort or when the abscess is large.

NOTE - In the AMC handbook of multiple choice question the treatment of meibomian cyst in acute phase has been mentioned to be incision of the lesion. However, in Australian therapeutic guidelines, as well as Murtagh’s general practice and many
other references,conservative measures, are mentioned as the initial management.

Intravenous administration of antibiotics (option B) is used if peri-orbital or orbital cellulitis is suspected. Topical chloramphenicol (option C) is used for treatment of conjunctvitis and not indicated for this patient.

CT scan of the orbit (option A) is an important part of management if orbital cellulitis is suspected.

References

• Therapeutic Guidelines – Antibiotic; available from http://tg.org.au

• Murtagh’s General Practice – McGraw Hill – 5th Edition – page 547

Last updated:
Time spent: QID:302 2023-2-12

704 of 1943
An 79-year-old diabetic man presents with isolated third nerve palsy. Which one of the following findings, if present, suggests diabetes as the etiology?

A. Normal pupillary reflex.

B. Normal extraocular movements.

C. Enophthalmos.

D. Involvement of the superior oblique muscle.

E. Absence of ptosis.

Correct
45% answered correctly

Explanation:

Correct Answer Is A

Ischemic third nerve palsies comprise the vast majority of third nerve palsies in adults. Diabetes mellitus is the most common cause, follwoed by hypertension and advanced age.

Isolated third nerve palsy with a normal pupillary reflex and completely paralysed extra-ocular muscles innervated by this nerve (superior rectus, inferior rectus, medial rectus and inferior oblique) are the most common presenting features in diabetic
third nerve palsy.

NOTE - Pupil involvement in third nerve palsy should be assumed to be due to aneurysmal compression until proven otherwise.

(Option B) With third nerve palsy the fucntion of superior rectus, inferior rectus, medial rectus and inferior oblique muscles are impaired regardless of the etiology.

(Option C) Enophthalmos can be seen in palsies occurred in trauma setting and is caused by orbital floor fractures not the paralysis of the nerve.

(Option D) Superior oblique muscle is supplied by the fourth cranial nerve and its function remains intact in diabetic third nerve palsy.

(Option E) Ptosis can be seen in all cases of third nerve palsy regardless of the underlying etiology.

References

• UpToDate - Third cranial nerve (oculomotor nerve) palsy in adults

Last updated:
Time spent: QID:741 2023-2-12

705 of 1943
A 32-year-old man presents with complaint of not being able to use his right arm as usual. On examination, shoulder abduction and elbow extension on the right side are diminished. Moreover, there is loss of sensation over the right deltoid area.
Which one of the following could be the cause of his problem?

A. C5 nerve palsy.

B. Injury to the brachial plexus.

C. Carpal tunnel syndrome.

D. C7 nerve root injury.

E. C6-C7 nerve root injury.

Incorrect. Correct answer is B


45% answered correctly

Explanation:

Correct Answer Is B

To answer this question, one should know the anatomy and distribution of brachial plexus which innervates almost all of the movements and sensation of the upper limb.

The following schematic diagram pictures the structure of the brachial plexus, the rami, trunks, divisions, cords and terminal branches and shows which spinal nerve roots contribute to formation of the nerves:

Nerve branch Contributing spinal nerve roots Action


Musculocutaneous nerve C5, C6, C7 Elbow flexion, supination, and some shoulder flexion
Axillary nerve C5, C6 Shoulder abduction (15°- 110°), lateral rotation of shoulder
Median nerve C5, C6, C7, C8, T1 Wrist and fingers flexion, pronation of forearm, thumb opposition
Radial nerve C5, C6, C7, C8, T1 Extension of the elbow and the wrist
Ulnar nerve C8, T1 Together with median nerve: wrist and fingers flexion

Diminished shoulder abduction and sensory loss over deltoid area suggest that C5 and C6 are affected. With decreased elbow extension radial nerve (C5, C6, C7, C8 and T1) must be affected. With possible involvement of several roots, the injury to
the brachial plexus is the most likely underlying cause to this man’s presentation.

(Option A) C5 nerve palsy, can justify, to some extent, the loss of sensation of deltoid and shoulder abduction, but not the elbow extension which is innervated by the radial nerve.

(Option C) Carpal tunnel syndrome is caused by entrapment of the median nerve in the flexor retinaculum. It does not affect elbow extension, shoulder abduction and sensory loss over the deltoid area.

(Options D and E) C7 nerve injury can justify the weakened elbow extension but not the shoulder abduction and sensory loss over the deltoid area which is supplied by axillary nerve comprised of C5 and C6 nerve roots. This is through about C6-C7
nerve root deficit.

706 of 1943
References

• Medscape - Brachial Plexus Anatomy

Time spent: QID:151 Last updated:


2023-2-12

707 of 1943
A 45-year-old man with history of type II diabetes mellitus comes to the emergency department with diplopia and ptosis of his left eye. On examination, the left eye is deviated slightly out and down in straight-ahead gaze. Upward gaze is impaired.
Pupils have normal reaction to light. Which one of the following would be the most likely underlying cause of this presentation?

A. Aneurysm of the posterior communicating artery.

B. Tumor.

C. TB meningitis.

D. Diabetes mellitus.

E. Trans-tentorial brain herniation.

Incorrect. Correct answer is D


45% answered correctly

Explanation:

Correct Answer Is D

The clinical features described are consistent with left oculomotor (3rd cranial nerve [CN-3]) palsy with sparing of the pupil.

The most common causes of third nerve palsy include:

Ischemia - Ischemic third nerve palsies, are the most common etiologic subset of third nerve palsies in adults. The pathogenesis is hypothesized to be microvascular. Diabetes mellitus, hypertension and advanced age are important risk factors.
While some isolated ischemic third nerve palsies are due to midbrain infarction, most are peripheral.

Intracranial aneurysms - The most dreaded cause of a third nerve palsy is compression by an enlarging intracranial aneurysm. The most common site of an aneurysm causing a third nerve palsy is the posterior communicating artery; however,
aneurysms involving the internal carotid artery and basilar artery are reported to produce third nerve palsies as well. In the setting of an acute third nerve palsy, the aneurysm is believed to be acutely enlarging and therefore at risk of imminent
rupture. In this setting, subarachnoid hemorrhage can occur within hours or days of initial presentation of a third nerve palsy. The mean age of presentation for aneurysmal subarachnoid hemorrhage is 55 years; however, aneurysms have been
reported in young children and in the elderly.

Trauma - Traumatic third nerve palsy usually arises only from severe blows to the head assoiciated with skull fracture and/or loss of consciousness. Thus, a third nerve palsy associated with mild head trauma should prompt evaluation for an
associated pathology.

Migraine - Ophthalmoplegic migraine is a condition affecting children and young adults, and most commonly involves the third cranial nerve, sometimes with permanent deficits.

Other causes - Infections, tumors, vasculitis, and herniation are other less common causes of third nerve palsy.

The most common causes of the oculomotor nerve palsy sparing the pupil reaction is ischemia of the nerve as a result of vascular compromise due to diabetes mellitus.

Third nerve involvement associated with impaired pupil light reaction can be caused with any of the following:

Aneurysm of the posterior communicating artery – it can rarely spare the pupil light reaction
Transtentorial brain herniation
TB meningitis

References

• UpToDate - Third cranial nerve (oculomotor nerve) palsy in adults

Last updated:
Time spent: QID:742 2023-2-12

708 of 1943
A 26-year-old woman sustains a closed head injury in a motor vehicle crash. She is unconscious on arrival at the emergency department. A head CT scan excluded skull fractures and hematomas. She regains consciousness after 25 minutes and
now is complaining of double vision when she looks to the right. Vision ahead is normal, as is when either eye is covered. Which one of the following cranial nerves is more likely to have been injured?

A. Right third cranial nerve.

B. Right fourth cranial nerve.

C. Left fourth cranial nerve.

D. Left sixth cranial nerve.

E. Right sixth cranial nerve.

Incorrect. Correct answer is E


45% answered correctly

Explanation:

Correct Answer Is E

The sixth cranial nerve innervates the lateral rectus muscles; therefore, damage to the nerve produces binocular (when looked with both eyes) horizontal diplopia when the patient looks to the side of the injured nerve. This occurs because the lateral
rectus muscle on the affected side fails to turn the eye to the affected side laterally, while the functioning medial rectus muscle of the contralateral eye turns it medially. The result of uncoordinated move of the two eyes produces two distinct picture
on the retina and causes horizontal diplopia.

The sixth cranial nerve has a long course from the lower pons and reaches the lateral rectus muscle after emerging through the superior orbital fissure. The long course of this nerve makes it susceptible to injuries.

(Option A) Typical presentation of the third cranial nerve palsy is ipsilateral ‘down and out’ gaze and fixed dilated pupil. With ischemic palsy of this nerve (e.g. diabetic palsy, midbrain stroke) the pupil reflex usually remains intact.

(Options B and C) The fourth cranial nerve supplies the superior oblique muscle. The function of this muscle gives eye intorsion which is moving of the eye inwards and downwards. Damage to this nerve will lead to binocular diplopia on downward
gaze.

(Option D) With the left sixth cranial nerve injury, there will be binocular diplopia on looking to the left side (the affected side).

References

• Medscape - Extraocular Muscle Actions

Last updated:
Time spent: QID:743 2023-2-12

709 of 1943
Aaron, 73 years old, is a diabetic patient of yours who has presented to the clinic with sudden onset of horizontal diplopia, better when he looks at a near object and worse when looking at distance. On examination, the left eye is deviated towards
medial side. Which one of the following is the most likely diagnosis?

A. Left sixth cranial nerve palsy.

B. Right Sixth cranial nerve palsy.

C. Right third cranial nerve palsy.

D. Left third cranial nerve palsy.

E. Left fourth cranial nerve palsy.

Correct
45% answered correctly

Explanation:

Correct Answer Is A

The sixth cranial nerve (CN-VI) palsy results in isolated weakness of abduction of the affected eye and horizontal binocular diplopia. On examination, there is an esotropia (inward deviation) that is worsened with gaze into the field of the affected
lateral rectus muscle. Abduction is commonly limited on the side of the lesion. Poorly controlled diabetes is a predisposing factor.

(Option B) In the right sixth cranial nerve palsy the right eye will be in an abnormal position (medially deviated), and the patient is diplopic on looking laterally to the right side.

(Option C) The third cranial nerve supplies the levator palpebrae muscle of the eyelid and 4 extraocular muscles: the medial rectus, superior rectus, inferior rectus, and inferior oblique. These muscles adduct, depress, and elevate the eye. Patients
with acute acquired third nerve palsy usually complain of the sudden onset of binocular horizontal, vertical, or oblique diplopia and a droopy eyelid. Pupil reflex remains intact in ischemic palsies (e.g. due to diabetes or midbrain infarcts).

710 of 1943
(Options D and E) A person with fourth nerve palsy may complain of binocular (both eyes open) vertical diplopia and/or subjective tilting of objects (torsional diplopia). The affected eye is usually extorted because the superior oblique muscle is
responsible for intorsion of the eye. Objects viewed in primary position, especially in down-gaze may appear double when going down a flight of stairs so that the patient does not know which step to take first.

References

• Medscape - Abducens Nerve Palsy (Sixth Cranial Nerve Palsy)

Time spent: QID:744 Last updated:


2023-2-12

711 of 1943
A 78-year old woman presents to the emergency department with acute onset of left-sided ptosis and horizontal diplopia. On examination, there is dilated nonreactive pupil on left side and normal indirect light reflex of the left side. Which one of the
following is the most likely diagnosis?

A. Optic nerve palsy.

B. Third cranial nerve palsy.

C. Fourth cranial nerve palsy.

D. Sixth cranial nerve palsy.

E. Seventh cranial nerve palsy.

Incorrect. Correct answer is B


45% answered correctly

Explanation:

Correct Answer Is B

Patients with acute acquired third nerve palsy usually complain of the sudden onset of binocular horizontal, vertical, or oblique diplopia as well as a droopy eyelid.

On examination, patients with complete non-pupil-sparing third nerve palsy have ptosis, a midriatic (dilated) pupil, and paralysis of adduction, elevation, and depression of the eye on the affected side. The eye rests in a position of abduction, slight
depression, and intorsion (down and out).

NOTE - pupillary reflex is often intact and unaffected in third nerve palsies due to ischemia such as in diabetes or midbrain infarcts.

Left third nerve palsy. The affected eye rests in 'out and down' position. Mild ptosis is noted.

(Option A) An optic nerve lesion leads to monocular visual loss. Other features of optic neuropathy include:

Impaired pupil light reflex (the afferent limb)


Central vision loss (scotoma) on visual field testing
Dyschromatopsia (color blindness) often out of proportion to acuity loss

(Option C) Fourth nerve palsy may result in binocular (with both eyes open) vertical diplopia and subjective torsional diplopia. It causes paralysis of superior oblique muscle. On examination, an ipsilateral hypertropia is present (the involved eye is
deviated upward) because the action of the superior oblique muscle (moving the eye down and inwards) is weak.

(Option D) Patients with sixth nerve palsies typically complain of binocular (both eyes open) horizontal diplopia that worsens with gaze toward the paretic lateral rectus muscle. Lateral rectus muscle paralysis results in weakness of abduction of the
affected eye with the affected eye often resting in medial deviation.

(Option E) Seventh nerve palsy results in inability to close eye, facial droop, inability to form wrinkles on forehead and change in taste sensations in anterior two-thirds of the tongue. When one entire side of the face is weak, the lesion is usually
peripheral. With a central lesion (such as stroke in one cerebral hemisphere), the forehead muscles are often spared because the part of the facial nerve nucleus supplying innervation to the forehead gets input from motor neurons of both cerebral
hemispheres. The portion of the facial nerve nucleus innervating the lower face does not have the same bilateral input; its input is predominantly from the contralateral cortex.

NOTE – with upper motor neuron lesion of the facial nerve (also called central seven), the contralateral lower parts of the face are affected. The contralateral forehead remains unaffected. With peripheral lesions, complete ipsilateral
dysfunction of the face muscles (upper and lower) occurs and the forehead is not spared.

References

• UpToDate - Third cranial nerve (oculomotor nerve) palsy in adults

• UpToDate - The detailed neurologic examination in adults

Last updated:
Time spent: QID:745 2023-2-12

712 of 1943
Which one of the following conditions causes ptosis and dilated pupil?

A. Myasthenia gravis.

B. Mitochondrial myopathy.

C. Horner’s syndrome.

D. Third cranial nerve palsy.

E. Sixth cranial nerve palsy.

Incorrect. Correct answer is D


45% answered correctly

Explanation:

Correct Answer Is D

Third cranial nerve (oculomotor) palsy can cause ptosis, deviation of the eye downwards and outwards, dilated pupils and sluggish or absent light reflex. In the third nerve paralysis due to ischemia (e.g. diabetic palsy or midbrain infarcts) pupils
usually remain intact.

(Option A) Myasthenia gravis can cause ptosis and diplopia with no pupil abnormality.

(Option B) Mitochondrial myopathy is associated with progressive ophthalmoplegia and limb weakness induced by exercise. It does not involve pupils.

(Option C) Horner’s syndrome is characterized by ptosis, miosis (constricted pupils) and ipsilateral loss of sweating. There is no diplopia.

(Option E) Sixth cranial nerve palsy presents with impaired lateral eye movement without affecting the pupils.

References

• UpToDate - Third cranial nerve (oculomotor nerve) palsy in adults

Last updated:
Time spent: QID:746 2023-2-12

713 of 1943
A 34-year-old woman presents to the Emergency Department with complaint of sudden-onset severe unilateral headache on the left side. The pain is felt behind the orbit and is associated with nasal stiffness and conjunctival injection and
lacrimation. She has the history of previous similar headaches. Which one of the following is the the treatment of choice to prevent further attacks?

A. Verapamil.

B. Oxygen 100%.

C. Propranolol.

D. Sumatriptan.

E. Paracetamol.

Correct
45% answered correctly

Explanation:

Correct Answer Is A

The scenario describes a classic case of cluster headache, also known as migrainous neuralgia. Cluster headache is characterized by attacks of severe orbital, supraorbital, or temporal pain, accompanied by autonomic phenomena. The typical
attacks may strike up to eight times a day and are relatively short-lived. Cluster headache is strictly unilateral, and the symptoms remain on the same side of the head during a single cluster attack. However, the symptoms can switch to the other side
during a different cluster attack (side shift) in approximately 15%.

The pain is very severe and as opposed to migraine pain, patients with cluster headaches are restless and prefer to pace about or sit and rock back and forth. The attacks of cluster headache can be so severe and vicious that patients may commit
suicide if the disease is not diagnosed or treated. Unlike migraine headache, cluster headache is not associated with nausea and vomiting.

Autonomic symptoms include ptosis, miosis, lacrimation, conjunctival injection, rhinorrhea, and nasal congestion. These symptoms are due to both sympathetic impairment and parasympathetic hyperactivity. In some patients, the signs of
sympathetic paralysis (miosis and ptosis) persists indefinitely and increases during attacks.

The very first step in management of a cluster headache attack is oxygen 100% through a tightly-sealed face mask for 15 minutes. This treatment alleviates the headache in most patients. For those unresponsive to oxygen, the following could be
tried with indefinite response:

Sumatriptan (intramuscularly or intranasally)


Dihydroergoramine (intramuscularly)
Lidocaine (intranasally)

NOTE - Nasal congestion may render the intranasal route ineffective.

After cluster headache is diagnosed, preventive treatment should be started with verapamil (sustained released) as the first-line medication.

Methysergide, corticosteroids, or lithium have been used as alternatives, but not as first-line. Methysergide is associated with the serious complication of retroperitoneal, cardiac and/or pleural fibrosis.

References

• UpToDate - Cluster headache: Treatment and prognosis

• Therapeutic Guidelines – Neurology

Last updated:
Time spent: QID:790 2023-2-12

714 of 1943
A 65-year-old woman presents to the emergency department with complaint of difficulty walking for the past 24 hours. She also mentions that she has been experiencing progressive pins and needles in hands and feet for the past few days. On
examination, she has a blood pressure of 130/80 mmHg, pulse rate of 90 bpm, respiratory rate of 14 breaths per minute and temperature of 37.5°C. On neurological examination, she is found to have brisk deep tendon reflexes (DTRs). There is an
extensor plantar response. There is also a decrease in sensing pain, temperature, vibration and touch. Romberg test is positive. There is no visual sign or symptom. Which one of the following is the investigation of choice to reach a diagnosis?

A. Lumbar puncture.

B. Nerve conduction studies.

C. Serum vitamin B12 level.

D. Serum creatine kinase (CK).

E. MRI of the brain.

Incorrect. Correct answer is C


45% answered correctly

Explanation:

Correct Answer Is C

The simultaneous presence of clinical findings attributable to peripheral neuropathy and myelopathy are strong indicators of subacute combined spinal cord degeneration that occurs with vitamin B12 deficiency. Vitamin B12-associated cord
degeneration may present with isolated neuropathy (~25%), myelopathy (~14%) or both (~41%).

Patients with the condition complain of distal paresthesia and weakness of the extremities, followed by spastic paresis and ataxia (represented by a positive Romberg test here). On physical exam, there is a combined deficit of vibration and
proprioception with pyramidal signs (plantar extension and hyperreflexia). Other sensory modalities can be affected as well.

Neuropsychiatric manifestations, such as recent memory loss with reduced attention span and otherwise normal cognition, depression, hypomania, paranoid psychosis with auditory or visual hallucinations (megaloblastic madness), violent behavior,
personality changes, blunted affect, and emotional lability are possible presentations of the disease reported in some of the patients.

Ocular findings including a cecocentral scotoma occurres 0.5% of cases. Optic atrophy, nystagmus, small reactive pupils, and chiasmatic lesion causing bitemporal hemianopia are other possible findings.

To establish the diagnosis, measuring the serum vitamin B12 level is the most important investigation. Treatment replacing vitamin B12.

References

• Medscape - Vitamin B-12 Associated Neurological Diseases

Last updated:
Time spent: QID:802 2023-2-12

715 of 1943
A 67-year-old man presents for assessment after he experienced an episode of vision loss of his right eye lasting 30 minutes 24 hours ago. The condition is resolved now. On examination, the visual acuity of the left and right eyes are 6/60 and 6/36,
respectively. A systolic murmur is noted over the sternum. No carotid bruit is heard. Which one of the following is most likely to establish the cause of his condition?

A. CT scan of the brain.

B. Electroencephalography (EEG).

C. MRI of the brain.

D. Doppler ultrasound of carotid arteries.

E. Echocardiography.

Incorrect. Correct answer is D


45% answered correctly

Explanation:

Correct Answer Is D

In adults, transient visual loss is a frequently encountered complaint that in most cases has an identifiable cause. The loss of vision may be monocular or bilateral and may last from seconds to hours. Episodes are usually ischemic in origin.

Ischemic causes of transient visual loss include:

Giant cell arteritis


Cerebrovascular ischemia
Retinal arteriolar emboli
Amaurosis fugax syndrome

Transient visual loss can be a symptom of a serious vision-threatening or even life-threatening condition, requiring urgent investigation and treatment, or it may have a more benign origin (e.g., migraine). Transient visual loss in children is less
common than in adults and is more likely to have a benign origin. Causes of transient visual loss in children include migraine and epileptic seizure.

Embolic occlusions of the arteries supplying the eye are a common cause of transient visual loss in adults. Emboli causing circulatory compromise may originate from the heart or the carotid arteries. Embolic events are usually isolated; therefore,
frequent episodes of visual loss are less likely to be caused by emboli.

In contrast to transient ischemic attacks (TIAs) involving the cerebral hemispheres, retinal ischemia is more commonly associated with emboli originating from carotid stenosis rather than the heart.

Diagnostic evaluation of transient visual loss (TVL):

The overlap in clinical presentations and grave prognosis of some potential diagnoses necessitates that some diagnostic testing is performed in most patients:

Ophthalmologic evaluation - A detailed fundoscopic evaluation is an important part of the evaluation of patients with transient visual loss. Ophthalmology referral is required for all patients with suspected giant cell arteritis, retinal vein disease, and
ocular causes of visual loss.

Erythrocyte sedimentation rate (ESR) and C-reactive protein - All older patients (>50 years) with transient monocular or binocular vision loss should have an ESR and C-reactive protein to exclude giant cell arteritis (GCA). If these are elevated, or if the
history is very suggestive, patients should proceed to a confirmatory temporal artery biopsy. Treatment with predniso(lo)ne should be started empirically.

Carotid imaging - Carotid Duplex Doppler ultrasound, magnetic resonance angiography (MRA), or computed tomographic angiography (CTA) should be ordered in all older patients (>50 years) and in younger patients with vascular risk factors
(diabetes, hypertension, hyperlipidemia), who have experienced transient mono-ocular visual loss (TMVL).

Duplex ultrasound is the preferred initial modality because it is noninvasive, inexpensive and readily available with acceptable sensitivity and specificity.

MRI should be performed in a patient of any age with TMVL, who has a history suggestive of carotid artery dissection.

Cardiac evaluation - Once GCA and carotid disease have been excluded, an evaluation to see if a cardiogenic source of embolism exists should follow in all older patients (and younger patients with risk factors), who have had TMVL. This is also
indicated in patients with transient binocular visual loss (TBVL) due to posterior circulation ischemia. Testing may include Holter monitoring and echocardiography. A baseline electrocardiogram (ECG) should also be included in the evaluation of
these patients because cardiac morbidity and mortality in patients with TMVL and central retinal artery occlusion is significant.

Brain MRI - Older patients with binocular visual symptoms (TBVL) with accompanying symptoms suggestive of vertebrobasilar ischemia should have a brain MRI. Patients whose history or examination suggests optic neuropathy should have an MRI
to look for other evidence of demyelinating disease. A history suggestive of compressive optic neuropathy (gaze evoked amaurosis) or seizure should also prompt a brain MRI with contrast enhancement.

Electroencephalography (EEG) - EEG is not a routine test for TVL, but should be performed in a patient with TBVL whose symptoms suggest possible seizure. EEG monitoring may increase the diagnostic yield, especially in patients with frequently
recurring symptoms.

Hypercoagulable testing - When brain or ocular ischemia is the suspected cause of TVL, hypercoagulable testing should be performed in individuals who have suggestive histories (prior thrombosis, miscarriage, or family history), as well as in
individuals with probable ischemia and otherwise negative workup. A full blood exam should also be obtained to screen for conditions such as polycythemia vera and essential thrombocythemia.

Of the options, the most important initial test to consider is Duplex Doppler ultrasonography of carotid arteries since most visual losses are due to carotid artery stenoses. Absence of carotid bruit does not exclude the possibility of carotid stenosis
as the most likely cause of retinal ischemia in this patient.

(Option A) CT scan of the brain can be considered after CGA and carotid stenosis are excluded as the possible causes of the presentation.

(Option B) As mentioned earlier, EEG is not used routinely for evaluation of a patient with MTVL.

(Option C) Brain imaging (MRI) is indicated in older patients with clues to posterior circulation insufficiency.

(Option E) Echocardiography is considered when CGA and carotid stenosis as the most common causes of TVL are excluded.

716 of 1943
References

• UpToDate - Amaurosis fugax (transient monocular or binocular visual loss)

• Medscape - Atherosclerotic Disease of the Carotid Artery

Last updated:
Time spent: QID:832 2023-2-12

717 of 1943
Which one of the following statements is incorrect regarding Guillain-Barre syndrome?

A. Intravenous immunoglobulin (IVIG) is the treatment of choice.

B. Plasma exchange is the first-line treatment.

C. Neuropathic pain is commonly seen during the illness.

D. Steroids have no role in treatment.

E. Neuropathic pain is responsive to tricyclic antidepressants.

Incorrect. Correct answer is B


45% answered correctly

Explanation:

Correct Answer Is B

Acute inflammatory polyradiculoneuropathy (Guillain-Barre syndrome) often presents acutely, with rapidly progressive widespread weakness and sensory disturbance, often in the absence of sensory signs that usually begins peripherally. Tendon
reflexes are often lost or impaired. Many different clinical presentations are seen including Miller Fisher variant, manifested with extraocular paresis, ataxia and arefelxia. There is also an uncommon variant with bulbar presentation.

Some cases have an infectious trigger including Campylobacter infection.

Confirmatory tests include a cerebrospinal fluid (CSF) examination with typical finding of elevated CSF protein level with an absent or minimal cellular response and, a little while later in the course of the disease, slowing of nerve conduction on nerve
conduction studies.

Patient should always be managed in an inpatient setting. Vital capacity should be monitored 4-hourly and if it falls to less than 20 ml/kg or is declining rapidly, the patient should be transferred to an intensive care unit. Cardiac monitoring is
recommended for such patients due to significant risk of cardiac arrhythmia.

Intravenous immunoglobulin (IVIG) and plasma exchange are both proven treatments with similar efficacy. Both can be used as treatment options; however, plasma exchange is logistically more difficult and associated with greater potential
complications making it less desirable as first-line treatment.

(Option A) IVIG can be more readily available and can be used as the treatment of choice in most centers.

(Options C and E) Neuropathic pain is a frequent complaint in patients with Guillain-Barre syndrome. The management is the same as for any neuropathic pain syndrome – adjuvant analgesia with a tricyclic antidepressant or antiepileptic drug, often
administered conjointly with an opioid is used for pain control. Gabapentin is more effective than carbamazepine.

(Option D) Steroids have no proven role in management of Guillain-Barre syndrome.

References

• Therapeutic Guidelines – Neurology; available from: http://tg.org.au

• The Guillain-Barre Syndrome Association of NSW

• UpToDate - Guillain-Barré syndrome in adults: Clinical features and diagnosis

Last updated:
Time spent: QID:923 2023-2-12

718 of 1943
On neurological assessment of a patient, he is unable to copy a pentagon. Which one of the following is the site of the lesion?

A. Temporal lobe.

B. Temporoparietal lobe.

C. Frontal lobe.

D. Dominant parietal lobe.

E. Non-dominant parietal lobe.

Incorrect. Correct answer is E


45% answered correctly

Explanation:

Correct Answer Is E

Drawing intersecting pentagons assesses the ability of the patient in constructional praxis (a task which requires three-dimensional manipulation). This is a task of non-dominant (right) parietal lobe. Lesions of non-dominant parietal lobe lead to
constructional apraxia.

TOPIC REVIEW

The Mini Mental State Examination (MMSE) is a tool that to systematically and thoroughly assess mental status. It is an 11-question measure that tests five areas of cognitive function: (1) orientation, (2) registration, (3) attention and calculation, (4)
recall, and (5) language. The maximum score is 30. A score of 23 or lower is indicative of cognitive impairment. The MMSE takes only 5-10 minutes to administer, and is practical to use repeatedly and routinely. The questions and their order are
shown in the following table:

Standardized mini-mental status exam


Question Time allowed score
What year is this? 10seconds 1
What season is this? 10 seconds 1
1 What month is this? 10 seconds 1
What date is today? 10 seconds 1
What day of the week is this? 10 seconds 1
What country are we in? 10 seconds 1
What province are we in? 10 seconds 1
What city/town are we in? 10 seconds 1 Oreintation
IN HOME: what is the street address of this house?

2 10 seconds 1
IN FACILITY: what is the name of this building?
IN HOME: what room are we in?

10 seconds 1
IN FACILITY: what floor are we on?
SAY: I am going to name three objects. When I am finished, I want you to repeat them. Remember what they are because I am
3 20 seconds 3 Registration
going to ask you to name them again in a few minutes. Say the following words slowly at 1‐second intervals ‐ ball/ car/ man
Spell the word WORLD. Now spell it backwards; OR ALTERNATIVELY: Serials 7s from 100. 1 point for each correct answer.
4 30 seconds 5 Attention and calculation
Stop after 5 answers.
5 Now what were the three objects I asked you to remember? 10 seconds 3 Recall

719 of 1943
6 SHOW wristwatch. ASK: What is this called? 10 seconds 1
7 SHOW pencil. ASK: What is this called? 10 seconds 1
8 SAY: I would like you to repeat this phrase after me: No ifs, ands or buts. 10 seconds 1
SAY: Read the words on the page and then do what it says. Then hand the person the sheet with CLOSE YOUR EYES on it. If
9 10 seconds 1
the subject reads and does not close their eyes, repeat up to three times. Score only if subject closes eyes
HAND the person a pencil and paper. SAY: Write any complete sentence on that piece of paper. (Note: The sentence must
10 30 seconds 1
make sense. Ignore spelling errors)
PLACE design, eraser and pencil in front of the person. SAY: Copy this design please. Allow multiple tries. Wait until person
is finished and hands it back. Score only for correctly copied diagram with a 4‐sided figure between two 5‐sided figures

11 1 minute 1

Language

ASK the person if he is right or left‐handed. Take a piece of paper and hold it up in front of the person. SAY: Take this paper in
your right/left hand (whichever is non‐ dominant), fold the paper in half once with both hands and put the paper down on the
floor . Score 1 point for each instruction executed Correctly.

1- Takes paper correctly in hand


12 30 seconds

2- Folds it in half

1
3- Puts it on the floor

1
TOTAL 30

Failing to answer the questions or doing the tasks indicates dysfunction of a particular area of the cortex:

Action The area of cortex tested


No ‘if’s, ‘and’s and ‘but’s Left hemisphere
Prosody Right hemisphere
Serial 7s (calculation) Left parietal
3-step task (motor praxis) Left parietal
Writing (write a sentence) Left parietal
Constructional praxis (draw intersecting pentagons, clock) Right parietal
Receptive language (follow simple commands: close your eyes) left temporal
Receptive language (prosody: how words are said (rhythms)) Right temporal
Verbal memory (object recall) Left temporal
Visual memory (redraw the clock from earlier) Right temporal
Object recognition (name the pen and watch) Occipital
Face recognition Occipital

720 of 1943
An 11-year-old boy is brought to your practice with progressive difficulty climbing stairs, walking, and running for the past few days. On examination, bilateral lower limb weakness is noted. All lower limb deep tendon reflexes (DTRs) are lost. The
sensation however is intact. Which one of the following tests is most likely to confirm the diagnosis?

A. CSF analysis.

B. Forced vital capacity.

C. MRI.

D. Nerve conduction studies.

E. X-ray.

Incorrect. Correct answer is D


45% answered correctly

Explanation:

Correct Answer Is D

Asymmetrical lower limb weakness, arelfexia and intact sensation, make acute inflammatory polyradiculoneuropathy (Guillain-Barre syndrome) the most likely diagnosis. Guillain-Barre syndrome (GBS) often presents acutely with rapidly progressive
widespread weakness and sensory disturbances such as pain, often in the presence of a normal sensory examination. Weakness usually begins peripherally. Tendon reflexes are often lost or impaired. Some cases may have a preceding infectious
disease such gastroenteritis with Campylobacter.

GBS occurs world-wide with an overall incidence of 1-2 per 100,000 per year. While all age groups can be affected, the incidence increases by approximately 20% with every 10-year increase in the age beyond the first decade of life.

The initial diagnosis of GBS is based on the clinical picture. The cardinal clinical features of GBS are progressive, mostly symmetric muscle weakness and absent or depressed deep tendon reflexes. The weakness can vary from mild difficulty walking
to nearly complete paralysis of all extremities, facial, respiratory, and bulbar muscles.

The diagnosis of Guillain-Barre syndrome (GBS) is confirmed if cerebrospinal fluid (CSF) and clinical neurophysiology studies show the typical abnormalities. Therefore, lumbar puncture and clinical neurophysiology studies are performed in all
patients with suspected GBS. Of these two, however, nerve conduction studies (NC) and needle electromyography (EMG) are more accurate. They are used not only for confirmation of diagnosis, but also for providing information regarding prognosis.

The typical CSF finding, known as albuminocytologic dissociation, starts approximately 48 hours after symptoms onset and is present in 50-66% of the patients in the first week, and over in 75% in the third week. Nerve conduction studies show a
typical demyelinating pattern. NCV changes often lag behind those of CSF.

Patients with GBS should always be managed in an inpatient setting. Vital capacity should be monitored 4-hourly, using forced vital capacity (FVC), and if it falls to less than 20 ml/kg or is declining rapidly, the patient should be transferred to an
intensive care unit. FVC monitoring is not diagnostic though. Cardiac monitoring is also recommended for such patients due to significant risk of cardiac arrhythmia.

Imaging studies (MRI, X-rays, etc.) are not of diagnostic value.

References

• UpToDate - Guillain-Barré syndrome in adults: Clinical features and diagnosis

• The Guillain-Barre Syndrome Association of NSW

• Therapeutic Guidelines – Neurology; available from: http://tg.org.au

Last updated:
Time spent: QID:959 2023-2-12

721 of 1943
A 22-year-old college student presents to your practice, complaining of recurrent episodes of drowsiness and feeling detached from the surrounding environment, mostly when she is in a shopping mall. Which one of the following would be the next
best step in management?

A. CT scan of the head.

B. EEG.

C. Ask a witness about the episodes.

D. Polysomnography.

E. Advise relaxation techniques.

Incorrect. Correct answer is C


45% answered correctly

Explanation:

Correct Answer Is C

This patient has presented with episodes of transient alteration or absence of consciousness. Evaluation of such episodes is critical to diagnose epileptic seizures, parasomnias, organic enecphalopathies, psychogenic nonepileptic seizure, or
sometimes a false account of the events for malingering.

A witness is a valuable source of information regarding the episode(s), and should be asked about the event if possible. The following are examples of questions to ask:

What did they see?


Did the person become pale or confused before or during the attack?
What does she/he looked like during the attack?
How long does the episode last?
How long did it take before consciousness returned to normal?

A thorough history in conjunction with an account of the event from a witness (if present) is always the very first step in stable patients who present with such episodes.

(Options A, B and D) Investigations such as CT scan, EEG, or polysomnography can be selected for further evaluation of possible diagnoses based on history and physical exam findings.

(Option E) Relaxation techniques are indicated if anxiety is found to be the underlying etiology.

References

• NCBI - Neurology Clinical Practice - Approach to the patient with transient alteration of consciousness

• Medscape - 10 Steps Before You Refer for Syncope

Last updated:
Time spent: QID:981 2023-2-12

722 of 1943
A 70-year-old man is being evaluated for further management after he recovered from an episode of transient ischemic attack (TIA). On examination, his blood pressure is 180/110 mmHg and a bruit is heard over the right carotid artery. A Duplex
Doppler ultrasonography of carotid arteries is performed, which is significant for 50% and 60% stenosis of the right and left carotid arteries, respectively. Which one of the following options is the most appropriate management for prevention of
further thromboembolic phenomena?

A. Hypertension control.

B. Aspirin.

C. Warfarin.

D. Carotid endarterectomy.

E. Clopidogrel.

Incorrect. Correct answer is B


45% answered correctly

Explanation:

Correct Answer Is B

The scenario is about measures for secondary prevention of cerebrovascular accident (CVA) after the patient sustained a TIA. Several preventive measures must be considered for him.

By far, hypertension is the most important preventable risk factor for CVA, accounting for 50% of cases. Unless contraindicated by symptomatic hypotension, all patients should receive blood pressure lowering therapy after stroke or TIA regardless of
being hypertensive or normotensive. With the exception of beta blockers, all antihypertensive medications have proven effective; however, ACE inhibitors and diuretics, either in combination or alone, have shown to be more effective.

This patient should also receive antiplatelet therapy. Antiplatelet therapy significantly reduces further incidences of stroke, MI, or vascular death.

Long-term antiplatelet medications should be prescribed to all individuals with ischemic stroke or TIA, who are not alteady on anticoagulation therapy (e.g., warfarin). The antiplatelet therapy of choice is low-dose aspirin plus modified release
dipyridamole. However, aspirin alone is the most used regimen in practice.

Clopidogrel is indicated in patients who are not tolerant of aspirin, those with contraindications to aspirin, or those who experience another CVA despite being on aspirin.

This patient has also a 50% stenosis in the right and 60% stenosis in the left carotid artery. Current guidelines recommend that carotid endarterectomy (CEA) should be performed in patients with non-disabling carotid artery territory ischemic stroke
or TIA with ipsilateral carotid stenosis measured at 70-99% if surgery can be performed by a specialist surgeon with low (<6%) perioperative mortality/morbidity. For stenoses of 50-69%, CEA is performed in selected patients (considering age, gender,
and comorbidities) with non-disabling carotid artery territory ischemic stroke or TIA if surgery can be performed with very low (<3%) perioperative mortality/morbidity.

Symptomatic stenosis means TIA or stroke attributable to the territory of the stenotic artery. For example, if a patient has stenosis of both left and right carotid arteries, and develops right hemiparesis and left facial weakness, only the stenosis of left
carotid artery is symptomatic.

Other measures to consider for this patient are regular exercise, statins and glycemic control if diabetic.

Of the given options, however, the most appropriate one for prevention of a thromboembolic event is aspirin.

(Option A) Lowering the blood pressure has a significant effect on prevention of CVAs in the future, and reduces the risk by approximately 25%, but it is unlikely to prevent thromboembolism; not directly at least.

(Option C) Warfarin is the standard of care in patients with CVA in the setting of atrial fibrillation and is preferred over aspirin or other antiplatelet agents. Unless this patient has AF, warfarin is not the preferred option for him.

(Option D) Although CEA in patients with symptomatic carotid stenosis of ≥70% is associated with decreased risk of thromboembolism, benefits to those with stenosis at 50-69% seems marginal, when compared to aspirin.

(Option E) Clopidogrel is the antiplatelet medication of choice if the patient has co-existing ischemic heart disease or has recently undergone coronary artery stenting. It is also replaced with aspirin is the patient is intolerant of it, or develops another
ischemic event while on it.

References

• Stroke Foundation – Clinical Guidelines for Stroke Management

Last updated:
Time spent: QID:1075 2023-2-12

723 of 1943
A 65-year-old man presents to the Emergency Department with left-sided hemiplegia starting 30 minutes ago. On arrival, his blood pressure is 140/95 mmHg and pulse rate 82 bpm and regular. A non-contrast CT scan of the head excludes
intracranial hemorrhage. With a diagnosis of ischemic cerebrovascular accident (CVA), which one of the following could have the most significant effect on recovery of this patient?

A. Immediate lowering of his blood pressure.

B. Carotid endarterectomy.

C. Fibrinolytic therapy.

D. Aspirin.

E. Expert nurse care.

Incorrect. Correct answer is C


45% answered correctly

Explanation:

Correct Answer Is C

If not contraindicated, fibrinolytic (thrombolytic) therapy is the treatment of choice in carefully selected patients, who present within 4.5 hours after symptoms onset. Several trials have shown significant reduction in death and disability. Recombinant
tissue plasminogen activator (rTPA) is approved for this purpose in Australia. Despite the benefits, access to thrombolysis remains low in Australia (~3%).

(Option A) Acute lowering of the blood pressure is harmful for patients with stroke due to risk of cerebral hypoperfusion. The only indication is extremely high blood pressures (systolic> 22mmHg, diastolic>11 mmHg).

(Option B) Carotid endarterectomy (CEA) is considered for secondary prevention of ischemic stroke and TIA in selecedt patients. It prevents further ischemic events but does not affect the outcomes of the patient’s current condition.

(Option D) Aspirin (150-300mg) is the most common medication used for initial treatment of ischemic stroke (contraindicated in hemorrhagic stroke) if thrombolysis is not considered and should be given as soon as possible (within 48 hours). If
thrombolysis is done, aspirin should be delayed for 24 hours.

NOTE - current guidelines recommend dual antiplatelet therapy with aspirin and clopidogrel for 3 weeks.

(Option E) Expert nurse care provides rehabilitation and improvement of the quality of life of patients with stroke in and established debilitation but has nothing to do with acute management of stroke.

References

• UptoDate - Intravenous fibrinolytic thrombolytic therapy in acute ischemic stroke

• Stroke Foundation – Clinical Guidelines for Stroke Management

Last updated:
Time spent: QID:1077 2023-2-12

724 of 1943
A 60-year-old man has had three episodes of transient monocular left blindness and right hemianesthesia. On examination, a soft bruit is heard over the left carotid artery. A Duplex Doppler ultrasound reveals a 25% stenosis of the left carotid artery
at the level of the carotid bifurcation. Based on the current recommendations, which one of the following would be the best step in management of this patient?

A. Immediate carotid endarterectomy.

B. Carotid endarterectomy in 2 months.

C. Aspirin 100 mg daily.

D. Warfarin.

E. Intracarotid streptokinase infusion.

Incorrect. Correct answer is C


45% answered correctly

Explanation:

Correct Answer Is C

The clinical picture is remarkable for episodes of transient ischemic attacks (TIA) in the territory of the left carotid artery. Impaired blood flow to this artery can cause ipsilateral visual problems, facial weakness, and facial sensory deficits, as well as
contralateral motor and sensory disturbances below the level of the injury.

Antiplatelet therapy is the pharmacological intervention of choice for secondary prevention of ischemic strokes and TIAs. The most effective treatment option is either low-dose aspirin or combination therapy with aspirin and dipyridamole. Although
the latter appears more effective, aspirin alone is the most commonly used regimen due to adverse effects of dipyridamole (e.g., headache, flushing, nausea, gastrointestinal upset).

(Options A and B) Carotid endarterectomy (CEA) has shown no benefit in patients with symptomatic carotid stenoses of less than 50%. It is not indicated in this patient with a stenosis of 25%.

(Option D) Warfarin in used for prevention of ischemic stroke or TIA in the presence of atrial fibrillation (AF).

(Option E) Intra-carotid streptokinase infusion is not a preventive measure for ischemic stroke or TIA. It is used for treatment of stroke within 4.5 hours of symptoms onset.

References

• Stroke Foundation – Clinical Guidelines for Stroke Management

• Therapeutic Guidelines – Neurology

Last updated:
Time spent: QID:1078 2023-2-12

725 of 1943
A 71-year-old woman presents to the Emergency Department with dysarthria and right sided hemiplegia which started one hour ago. She had a myocardial infarction 18 moths back and is on aspirin 80 mg, daily. A CT scan of the head is obtained
which is negative for intracranial hemorrhage. Which one of the following is the most appropriate next step in management?

A. rTPA.

B. Clopidogrel and aspirin.

C. Clopidogrel.

D. Warfarin.

E. Heparin.

Correct
45% answered correctly

Explanation:

Correct Answer Is A

The constellation of signs and symptoms in the absence of blood on brain CT scan suggests ischemic stroke or transient ischemic attack (TIA) as the diagnosis.

In carefully selected patients with ischemic stroke or TIA, thrombolytic therapy is the treatment of choice if the patient presents within 4.5 hours of symptoms onset. Recombinant tissue plasminogen activator (rTPA) is approved for this purpose in
Australia. Despite the benefits, access to thrombolysis remains low in Australia (~3%).

Current guidelines also recommend dual therapy with aspirin and clopidogrel (option B) for ONLY 3 weeks. This treatment should be started immediately. If thrombolytic therapy has been performed, dual therapy should be started 24 hours after it.
For this patient, thrombolytic therapy is the first step followed by combination therapy after 24 hours.

(Option C) Clopidogrel alone is used as stand-alone therapy in patients who are intolerant of or allergic to aspirin or develop another cardiovascular event while on aspirin. Addition of dipyridamole to aspirin is another option for the latter.

(Options D and E) Anticoagulation is the secondary preventive method of choice in patients with ischemic stroke and TIA in the setting of AF. Heparin is not recommended for acute management of stroke or TIA if the cause is not AF.

References

• Stroke Foundation – Clinical Guidelines for Stroke Management

• UpToDate - Intravenous thrombolytic therapy for acute ischemic stroke: Therapeutic use

Last updated:
Time spent: QID:1079 2023-2-12

726 of 1943
A 67-year-old woman presents to the Emergency Department after she developed a transient left hemiparesis at home 20 minutes ago which has resolved completely at the time of physical examination. She had an acute myocardial infarction 12
months ago and underwent angioplasty. She is currently on aspirin 100 mg, daily. Which one of the following is the most appropriate management option for her?

A. Add warfarin.

B. Add clopidogrel.

C. Continue the same dose of aspirin.

D. Increase the dose of aspirin.

E. Switch to dipyridamole.

Incorrect. Correct answer is B


45% answered correctly

Explanation:

Correct Answer Is B

Antiplatelet therapy is one of the most important measures taken for secondary prevention of ischemic cerebrovascular accidents in patients with the history of such events. Although addition of dipyridamole to aspirin has shown increased benefit
compared to aspirin alone, aspirin alone is more used due to the adverse effects of dipyridamole and the consequent patients' non-compliance.

Current guidelines recommend dual therapy with aspirin and clopidogrel for 3 weeks for initial pharmacological management of strokes and high-risk transient ischemic attacks (TIA). This patient is already on aspirin; therefore, addition of
clopidogrel is the most appropriate next step. Unless also indicated for acute coronary disease or coronary artery stents, extended combination therapy with aspirin and clopidogrel beyond 3 weeks is not recommended.

(Option A) Warfarin is the antithrombotic drug of choice in the presence of atrial fibrillation (AF) as the likely underlying cause of the cerebrovascular event.

(Options C and D) Continuation of aspirin at the same dose does not prevent more episodes of CVA. Increasing the dose of aspirin has not shown to be more beneficial than low-dose aspirin as a preventive measure.

(Option E) Dipyridamole has no considerable benefit over aspirin. However, adding it to aspirin can be used for long-term secondary prevention of cerebrovascular events, Dipyridamole adverse effects include headache, flushing, nausea and
gastrointestinal upset. Such adverse effects are a major cause of non-compliance and abandoning the treatment by patients. Dipyridamole does not cause bleeding, nor does it increase the incidence of aspirin-related bleeding.

References

• UpToDate - Antiplatelet therapy for secondary prevention of stroke

• Stroke Foundation – Clinical Guidelines for Stroke management

• Medscape – Stroke Prevention

• Therapeutic Guidelines – Neurology; available from: http://tg.org.au

Last updated:
Time spent: QID:1080 2023-2-12

727 of 1943
A 72-year-old woman, who is a known case of diabetes and hypertension and on multiple medications, is brought to the Emergency Department with complaint of left-sided weakness for the past 48 hours. On examination, she has a blood pressure
of 150/100 mmHg and an irregular pulse of 98 bpm. Her blood sugar is 8 mmol/L. Three years ago, she had an episode of sudden-onset right vision loss for few hours before she completely recovers. She, however, did not seek any medical attention
at that time. Which one of the following is the most appropriate treatment option for her?

A. Warfarin.

B. Aspirin.

C. Dipyridamole.

D. rTPA.

E. Aspirin and clopidogrel.

Correct
45% answered correctly

Explanation:

Correct Answer Is A

If not contraindicated, antithrombotic therapy should be started for all patients with ischemic stroke. In patients with atrial fibrillation (AF), warfarin is the antithrombotic of choice to start. The need for anticoagulation is assessed based on CHA2DS2-
VASc system. Click here for an online CHA2DS2-VASc score calculator. You can learn more about CHA2DS2-VASc under the 'Atrial Fibrillation' section in 'Circulatory System/ Cardiology' category.

(Options B and C) Aspirin or dipyridamole alone does not appear to provide adequate anticoagulation in patients with AF.

(Option D) Thrombolytic therapy is indicated within the first 4.5 hours of symptoms onset. In this patient, 48 hours has passed since the start of symptom; therefore, thrombolytic therapy with rTPA is not beneficial for her.

(Option E) Combination therapy with aspirin and clopidogrel for 3 weeks is recommended for patients with stroke or high-risk transient ischemic attacks in other settings than AF. Patients who develop cerebrovascular event caused by AS need
anticoagulation with warfarin.

References

• UpToDate - Stroke in patients with atrial fibrillation

• Stroke Foundation – Clinical Guidelines for Stroke management

• Therapeutic Guidelines – Neurology; available from: http://tg.org.au

• Hear Foundation - Atrial Fibrillation

Time spent: QID:1081 Last updated:


2023-2-12

728 of 1943
A 60-year-old woman presents to your clinic for evaluation. She has the history of a recent transient ischemic attack (TIA), and was diagnosed with atrial fibrillation (AF) 3 months ago. Other significant points in her history are cigarette smoking and
hypercholesterolemia. A Duplex Doppler sonography of the carotid arteries has been performed which shows 40% stenosis of the left and 50% stenosis of the right carotid arteries. On examination, she is found to have a blood pressure of 180/110
mmHg on two occasions. Which one of the following would be the most appropriate long-term management?

A. Left carotid endarterectomy.

B. Smoking cessation.

C. Hypertension control.

D. Warfarin.

E. Aspirin.

Incorrect. Correct answer is D


45% answered correctly

Explanation:

Correct Answer Is D

By far, hypertension is the most important preventable risk factor for CVA, accounting for 50% of cases. Unless contraindicated by symptomatic hypotension, all patients should receive blood pressure lowering therapy after stroke or TIA, regardless
of their blood pressure (option C). With the exception of beta blockers, all antihypertensive medications have proven effective; however, ACE inhibitors and diuretics, either in combination or alone have shown to be more effective.

This patient should also receive antiplatelet therapy. Antiplatelet therapy significantly reduces stroke, MI or vascular death. Long term antiplatelet therapy should be prescribed to all patients with ischemic stroke or TIA. The antiplatelet therapy of
choice is low dose aspirin plus modified release dipyridamole.

In this patient, however, in the presence of AF, aspirin (option E) will not provide adequate protection against thromboembolism, and warfarin should be used for such purpose. Warfarin is the standard of care in patients with CVA in the setting of AF,
and is preferred over aspirin or other antiplatelet agents.

This patient has also a 50% stenosis in the right and 40% stenosis in the left carotid arteries. Current guidelines recommend that carotid endarterectomy (CEA) (option A) should be performed in patients with non-disabling carotid artery territory
ischemic stroke or TIA with ipsilateral carotid stenosis measured at 70-99% if surgery can be performed by a specialist surgeon with <5% perioperative mortality/morbidity, or stenoses of 50-69% in selected patients (considering age, gender and
comorbidities) with <3% risk of perioperative mortality/morbidity. The question is not specific about which side was involved in the previous TIA, but if occurred on the right side, CEA would be considered. For symptomatic stenoses of less than 50%,
or asymptomatic stenoses of less than 60%, CEA is not recommended.

Smoking cessation (option B) is another important step that reduces the chance of having another incidence of ischemic stroke or TIA and all-cause mortality/morbidity. It is not, however, as important as warfarin therapy at this stage.

This patient should also be started on statins regardless of her cholesterol level.

References

• UpToDate - Stroke in patients with atrial fibrillation

• Stroke Foundation – Clinical Guidelines for Stroke management

• Therapeutic Guidelines – Neurology; available from: http://tg.org.au

Last updated:
Time spent: QID:1082 2023-2-12

729 of 1943
To perform a dilation and curettage on a woman, she is placed in lithotomy position, after which she is found to have developed a nerve injury. Which one of the muscles is most likely to be affected by this injury?

A. Extensor hallucis longus.

B. Flexor digitorum longus.

C. Tibialis posterior.

D. Soleus.

E. Quadriceps.

Correct
45% answered correctly

Explanation:

Correct Answer Is A

The most commonly injured lower extremity nerve in patients undergoing surgery in lithotomy position is the common peroneal nerve. The injury is thought to be secondary to compression of the nerve between the lateral aspect the fibular head and
the bar holding the leg, especially when candy cane stirrups are used (see below). Low weight, smoking and prolonged surgery are risk factors for this injury.

At the level of the popliteal fossa, the sciatic nerve bifurcates into tibial and common peroneal (fibular) nerves. Medial to the course of the biceps femoris muscle, common peroneal nerve runs laterally and inferiorly, wraps around the fibular head and
enters the lateral compartment of the leg. It passes between the attachments of the fibularis longus muscle, where the nerve divides into the superficial and deep fibular (peroneal) nerves.

Injury to common peroneal nerve can cause dysfunction of both deep and superficial peroneal nerves that can present with the following:

Loss of dorsiflexion and eversion of the foot (equinovarus deformity)


Sensory manifestations along the anterolateral border of the leg and dorsum of the toes except those supplied by saphenous and sural nerves.

The following muscles of the leg are innervated by deep and superficial branches of the common peroneal nerve:

Tibialis anterior
Extensor digitorum longus
Extensor hallucis longus
Peroneous tertius
Peroneous longus
Peroneous brevis

Of the given options, only extensor hallucis longus can be affected as a result of common peroneal nerve injury in lithotomy position. In fact, inability to extend the greater toe is a common finding in patients with injured common peroneal nerve.

(Options B, C and D) Soleus (plantar flexor), tibilalis posterior (plantar flexor and ankle inverter), and flexor muscles of the foot, including flexor digitorum longus are innervated by tibial nerve; therefore, unaffected with common peroneal nerve injury.

(Option E) Quadriceps muscle is supplied by femoral nerve and unaffected by common peroneal injury.

NOTE - Lower leg manifestations of L5 radiculopathy can be exactly similar to the common peroneal nerve injury.

References

• BJA - Perioperative peripheral nerve injuries

• Orhtobullets - Deep Peroneal Nerve

Last updated:
Time spent: QID:1100 2023-2-12

730 of 1943
Which one of the following is caused by damage to the trigeminal nerve?

A. Forehead wrinkling.

B. Vision loss.

C. Double vision.

D. Loss of taste.

E. Difficulty in opening the mouth.

Incorrect. Correct answer is E


45% answered correctly

Explanation:

Correct Answer Is E

Trigeminal nerve (NC V) has three divisions that carry pain, temperature and touch modalities from the skin of the face, the mucosa of sinuses, nose, mouth, the teeth, and portions of the dura. They also carry proprioceptive sensation from the teeth,
hard palate, temporomandibular joint, and muscles of mastication. Motor fibers are included in the maxillary division of the trigeminal nerve.

The three divisions of this nerve are:

Ophthalmic (upper) division - Innervates forehead, upper eyelid, cornea (thus the corneal reflex - different from pupilary reflex), conjunctiva, dorsum of the nose, and the dura of some of the anterior cranial fossa.

This division leaves orbit through the superior orbital fissure. It then proceeds through the lateral wall of the cavernous sinus in close relation to the CN III, CN IV and CN VI and joins the other two divisions to form the trigeminal (semilunar, Gasserian)
ganglion.

Maxillary (middle) division – Innervates the upper lip, lateral and posterior portions of nose, upper cheek, anterior temple, mucosa of the nose, upper jaw, upper teeth, the roof of the mouth, and the dura of part of the middle cranial fossa. The nerve
leaves the pterygopalatine fossa, passes through the foramen rotundum, traverses the inferior part of the cavernous sinus, and enters the trigeminal ganglion.

Mandibular (lower) division – Supplies the lower lip, chin, posterior cheek, temple, external ear, mucosa of the lower part of mouth, anterior two-thirds of the tongue, and portions of the dura of anterior and middle cranial fossae. Proprioceptive
impulses are carried largely in the motor nerve, which is incorporated into the mandibular division. It enters the cranium through the foramen ovale and goes to the trigeminal ganglion.

Sympathetic and parasympathetic fibers join the three divisions and are distributed to the pupil, to the nasal mucosa causing mucus secretion, to the lacrimal, submaxillary, and sublingual glands, and to the arterioles of the face.

Anatomy of trigeminal nerve (CN V) www.teachmeanatomy.com

Sensory territories of the three branches of CN V

The motor division of the nerve (incorporated in the mandibular division) supplies the muscles of mastication including:

Masseter
Temporal
Pterygoid
Mylohyoid
Digastric.

These muscles produce elevation, depression, protrusion, retraction, and the side-to-side movements of the mandible; therefore, lesions of motor portion of this nerve lead to dysfunction of these muscles and difficulties in opening the mouth and
chewing, and impairment of various mandible (jaw) movements. The motor division also supplies the tensor tympani and tensor palati muscles.

When there is unilateral paralysis of the masticatory muscles, on mouth opening, the mandible deviates toward the paralyzed side. This direction of the mandible is due to the action of normal pterygoids on the opposite side. The mandible droops,
and no jaw movement is possible with bilateral paralysis. The involved muscles undergo atrophy in nuclear or infranuclear lesions.

731 of 1943
The jaw jerk is one of the deep tendon or stretch reflexes. When it is normal, tapping the mandible produces a brisk contraction. When abnormal, with upper motor neuron lesions, there is a hyperactive or repeating reflex (clonus). With nuclear or
infranuclear lesions, the reflex is absent.

th
(Option A) Forehead wrinkling is an action of the 7 cranial nerve (facial nerve).

nd
(Option B) Carrying visual stimuli is a function of 2 cranial nerve (optic nerve).

(Option C) Diplopia (squint) is mostly caused by dysfunction of extraocular muscles innervated by CN III, CN IV and CN VI.

(Option D) Sense of taste is mediated by three cranial nerves:

1. Facial nerve (CN VII)


2. Glossopharyngeal nerve (CN IX)
3. Vagus nerve (CN X)

Facial nerve (chorda tympani and the greater petrosal branch nerves) provides the sense of taste in the anterior 2/3 of the tongue. Glossopharyngeal nerve provides that of the posterior 1/3 of the tongue. The superior laryngeal branch of vagus (CN
X) innervates taste buds in the laryngeal surface of the epiglottis. The trigeminal nerve provides the general sensation of the tongue not the taste sense; therefore, CN V lesions do not result in taste loss.

Tongue nerves

References

• Clinical Methods: The History, Physical, and Laboratory Examinations. 3rd edition - Chapter 61 Cranial Nerve V: The Trigeminal Nerve

• Medscape - Taste System Anatomy

Time spent: QID:493 Last updated:


2023-2-12

732 of 1943
One week after a knee surgery, a 24-year-old man presents with complaints of numbness and paresthesia of the right leg. On examination, there is foot drop and weakness of dorsiflexion and eversion of the right ankle. Sensation over the outer
aspect of the right leg is also lost. Right ankle jerk is intact. Which one of the following nerves is most likely to have caused such presentation?

A. L4 nerve root.

B. L5 nerve root.

C. Common peroneal nerve.

D. Tibial nerve.

E. Sciatic nerve.

Incorrect. Correct answer is C


45% answered correctly

Explanation:

Correct Answer Is C

The clinical picture suggests damage to the common peroneal nerve as an adverse outcome of the knee surgery. Foot drop is the result of weakness of the ankle dorsiflexors (extensors) innervated by the deep peroneal nerve (with the exception of
peroneous tertius, a weak ankle dorsiflexor located in the lateral compartment of the leg, and supplied by the superficial peroneal nerve). Sensation over the outer aspect of the leg is supplied by the common peroneal nerve and its superficial branch.
These together make the common peroneal nerve injury the most likely explanation.

After branching off the sciatic nerve, the common peroneal nerve runs down laterally, wraps around the fibular head and enters the leg. The nerve is superficial in this region and susceptible to injuries during knee surgeries, with compression or in
trauma.

(Option A) L4 nerve root is in the lumbar area. L4 radiculopathy can cause weakened knee reflex and partially impaired ankle inversion, but does not explain other manifestations.

(Option B) L5 nerve root damage has a very similar clinical presentation in lower leg to that of the common peroneal nerve injury; however, the history of knee surgery makes the latter more likely.

(Option D)Tibial nerve damage causes impaired ankle jerk, weak or absent plantar flexion and weak ankle inversion. Sensory impairment due to tibial nerve injury affects the back the leg and most parts of the sole of the foot.

(Option E) Sciatic trunk (nerve) is above the knee, and unlikely to have been affected by the knee surgery. Moreover, sciatic damage affects the ankle and foot movements globally, and gives a clinical picture consistent with injuries of both common
peroneal and tibial nerves at the same time.

References

• Orhtobullets - Tibial Nerve

• Orhtobullets - Deep Peroneal Nerve

• BJA - Perioperative peripheral nerve injuries

Last updated:
Time spent: QID:1101 2023-2-12

733 of 1943
A 45-year-old man presents with loss of inversion, eversion, dorsiflexion and plantar flexion of his left foot. His left knee jerk is preserved, but left anjle reflex is weak. Which one of the following can be the cause of this presentation?

A. Sciatic nerve injury.

B. Common peroneal nerve injury.

C. Tibial nerve injury.

D. L4/L5 disc herniation.

E. Femoral nerve damage.

Correct
45% answered correctly

Explanation:

Correct Answer Is A

Loss of ankle inversion in this man can be either due to dysfunction of deep peroneal nerve (a branch of common peroneal nerve), or tibial nerve that supply the two main ankle invertors: tibialis anterior and tibialis posterior.

Ankle eversion is the action of the muscles in the lateral compartment of the leg: peroneous (fibularis) longus, peroneous (fibularis) brevis and peroneous tertius. These muscles are supplied by the other main branch of the common peroneal nerve,
superficial peroneal nerve.

Plantar flexion of the ankle is mediated mostly by the muscles in the posterior compartment of the leg, all of which are innervated by tibial nerve. On the other hand, ankle reflex in this patient is weak and suggests involvement of the S1 (and to a
lesser extend S2).

In fact, the clinical picture is only justified if both common peroneal and tibial nerves are involved and affected. This can be explained by injury to sciatic nerve.

Tibial nerve and common peroneal (fibular) nerve exit from the pelvis together contained in a sheath of connective tissue to form the sciatic trunk (nerve). In fact, the sciatic nerve is the common peroneal nerve and tibial nerves running down
together in the buttock and back of the thigh. In the distal thigh, these two nerves separate and run different paths.

Injury to sciatic nerve (trunk) can present with motor and sensory deficits attributable to both the tibial nerve and common peroneal nerve. The clinical findings include:

Common peroneal nerve:

Weak or absent ankle eversion


Weak or absent ankle dorsiflexion (drop foot)
Weak or absent toes extension
Weak ankle inversion
Paresthesia / impaired sensation over the lateral aspect of the leg and dorsum of the foot (except the areas supplied by saphenous and sural nerves)

Tibial nerve:

Weak or absent plantar flexion (the patient cannot stand on the ball of his foot on the affected side)
Weak ankle inversion
Weak or absent toes flexion
Weak or absent ankle jerk reflex
Paresthesia/impaired sensation over the posterior part of the leg and sole of the foot (except those areas supplied by saphenous and sural nerves)

In summary, injury to sciatic trunk causes weakness of ankle movements in all directions, impaired or absent ankle reflex and impaired sensation below the knee except the medial aspect of the leg and the outer part of the foot. Knee reflex usually
remains intact.

(Option B) Common peroneal nerve damage (in isolation) explains loss of sensation over the lateral leg, as well as weakened dorsiflexion, eversion and inversion, but not the weak plantar flexion.

(Option C) Tibial nerve injury (in isolation) explains the absence of the ankle plantar flexion and impaired ankle jerk, but not the other clinical aspects.

(Option D) Lateral L4/L5 disc herniation affects the L5 nerve root and results in L5 radiculopathy. L5 and common peroneal nerve injuries present similarly in the lower leg. L5 radiculopathy does not cause weak plantar flexion and impaired ankle
reflex.

(Option E) Femoral nerve supplies all the muscles in the anterior compartment of the thigh, as well as the sensation of the thigh except the posterior part (which is supplied by the sciatic trunk) and a small medial part (supplied by the obturator
nerve), and through the saphenous branch, the medial aspect of the leg and outer part of the foot.

References

• Orthobullets - Sciatic Nerve

• Orthobullets - Tibial Nerve

• Orhtobullets - Deep Peroneal Nerve

• Orthobullets - Superficial Peroneal Nerve

• Orthobullets - Femoral Nerve

Last updated:
Time spent: QID:1102 2023-2-12

734 of 1943
A 75-year-old woman underwent total left hip replacement surgery yesterday. Today, she has developed weakness and numbness of her left foot. On examination, there is weakness of all the left ankle movements including dorsiflexion, plantar
flexion, eversion and inversion, and numbness over the dorsum and sole of her foot and lateral aspect of her leg. The left ankle jerk is absent. Which one of the following is the most likely site of the lesion?

A. Left common peroneal nerve.

B. Left femoral nerve.

C. Left obturator nerve.

D. Left sciatic nerve.

E. Left tibial nerve.

Incorrect. Correct answer is D


45% answered correctly

Explanation:

Correct Answer Is D

Ankle dorsiflexion and eversion are actions of the deep peroneal and superficial peroneal nerves, respectively. Inversion is supplied by deep peroneal nerve (anterior tibialis muscle) and tibialis nerve (posterior tibialis muscle). Plantar flexion and
ankle reflex are supplied by the tibial nerve. The efferent limb of the ankle jerk reflex is innervated by S1 and to lesser extend S2 fibers within this nerve. The pattern of the sensory impairment indicates that both nerves are affected.

As a matter of fact, this woman has a clinical presentation consistent with both common peroneal and tibial nerves injuries. When such clinical presentation is encountered, injury to the sciatic nerve is the most likely explanation. This can be the
most likely diagnosis given the history of hip surgery.

The sciatic nerve is derived from lumbosacral plexus. Once formed, it leaves the pelvis and enters the gluteal region via greater sciatic foramen. It emerges inferiorly to the piriformis muscle, and enters the posterior thigh by passing deep to the long
head of the biceps femoris, and descends in an inferolateral direction. Within the posterior thigh, the nerve gives off branches to the hamstring muscles and adductor magnus. When the sciatic nerve reaches the apex of the popliteal fossa, it
terminates by bifurcating into the tibial and common fibular (peroneal) nerves.

NOTE - The sciatic nerve can be described as two individual nerves bundled together in a sheath of connective tissue. These two nerves are the tibial and common peroneal nerves. These two nerves usually separate at the apex of the
popliteal fossa, however in a minority or persons, they separate as they leave the pelvis.

Sciatic nerve injury can occur during hip surgeries. Stretch, compression, ischemia, and direct damage are primary mechanisms. The lithotomy, frog leg, and sitting positions have been implicated in perioperative injury to this nerve. Regional
anesthetic techniques and hip arthroplasty may also cause injury. The common peroneal component is more common to be affected because it is more superficial compared with the tibial component.

Sciatic nerve Injury manifests as paralysis of the hamstring muscles (knee flexion weakness) and all the muscles below the knee. All sensation below the knee is affected with the exception of the sensation of the medial aspect of the leg and lateral
aspect of the foot, which remains intact because these two areas are supplied by saphenous nerve (a sensory branch of the femoral nerve) and sural nerve, respectively.

(Option A) Left common peroneal nerve injury results in foot drop (loss of ankle dorsiflexion), weak inversion and inability of the patient to evert the ankle. Sensory impairment occurs over the lateral aspect of the leg and dorsum of the left foot
(except the lateral margin of left foot).

(Option B) Left femoral nerve injury causes global weakness of all muscles in the anterior compartment of the thigh, impaired knee jerk and sensory disturbances of the lateral, and anteromedial surfaces of the thigh, as well as the medial aspect of
the leg.

(Option C) Obturator nerve consists of L2, L3, and L4 nerve roots. It innervates the muscles adductor longus, adductor brevis, adductor magnus, gracilis, and obturator externus. It provides, along with the femoral nerve, the sensation of the medial
aspect of the thigh. Obturator nerve injury presents with medial thigh or groin pain, weakness of leg adduction and sensory loss in the medial thigh.

(Option E) Tibial nerve damage results in weak plantar flexion. Impaired ankle jerk and sensory disturbances over the posterior aspect of the leg and sole of the foot are other manifestations.

References

• Orthobullets - Sciatic Nerve

• Orthobullets - Obturator Nerve

• Orthbullets - Tibial Nerve

• BJA - Perioperative peripheral nerve injuries

• UpToDate - Acute lumbosacral radiculopathy: Pathophysiology, clinical features, and diagnosis

Last updated:
Time spent: QID:1103 2023-2-12

735 of 1943
A 26-year-old-man develops weakness of foot plantar flexion and inversion. Which nerve is most likely to have been damaged?

A. Common peroneal nerve.

B. Tibial nerve.

C. Sciatic nerve.

D. Lumbosacral plexus.

E. Lumbar nerve roots.

Incorrect. Correct answer is B


45% answered correctly

Explanation:

Correct Answer Is B

Plantar flexion is provided by the ankle flexor muscles in the posterior compartment of the leg, all of which are innervated by the tibial nerve. Ankle inversion is the action of two muscles: tibialis anterior and tibialis posterior Tibialis anterior is
supplied by deep peroneal nerve, while tibialis posterior in the posterior leg compartment is innervated by tibial nerve. This movement can be affected with injuries to the tibial or deep peroneal nerves.

(Option A) With involvement of the common peroneal nerve, foot drop (caused by weakness of the ankle extensors) and sensory disturbances over the outer surface of the leg is expected. Ankle jerk reflex remains intact.

(Option C) With sciatic nerve injury, global weakness of all ankle and foot movements occurs. Ankle reflex is also affected. Weakness of the knee flexion may be noticed as well.

(Option D) Lumbosacral plexus problems have various presentations depending on the affected nerve root. S1 radiculopathy presents with similar manifestation seen in tibial nerve injury; however, back pain radiating from low back and the buttock
down to the posterior aspect of the leg into the foot is a feature not mentioned here. It makes lumbosacral plexus involvement less likely.

(Option E) Lumbar nerve root problems cause deficits in the movements and sensation of the hip, thigh and knee extension and reflex.

References

• http://www.medscape.com/viewarticle/758724_7

• http://www.orthobullets.com/anatomy/10115/sciatic-

• http://www.orthobullets.com/anatomy/10116/tibial-n

• http://ceaccp.oxfordjournals.org/content/early/201

• http://www.uptodate.com/contents/acute-lumbosacral

Last updated:
Time spent: QID:1104 2023-2-12

736 of 1943
A 26-year-old man develops weakness of foot dorsiflexion and eversion. Which nerve is most likely to have been damaged?

A. Common peroneal nerve.

B. Tibial nerve.

C. Sciatic nerve.

D. Lumbosacral plexus.

E. Lumbar nerve roots.

Correct
45% answered correctly

Explanation:

Correct Answer Is A

Ankle eversion is an action of three muscles in the lateral compartment of the leg, including:

Peroneous (fibularis) longus


Peroneous (fibularis) brevis
Peroneous tertius

These muscles are innervated by the superficial peroneal nerve which is one of the two main branches of the common peroneal nerve.

Ankle dorsiflexion is the action of the following:

Tibialis anterior
Extensor hallucis longus
Extensor digitorum longus
Peroneous (fibularis) tertius

Peroneous tertius is innervated by the superficial peroneal nerve. The other three are supplied by deep peroneal nerve, the other main branch of the common peroneal nerve.

With both ankle dorsiflexion and eversion affected, the common peroneal nerve is most likely to be involved.

(Option B) Tibial nerve innervates the posterior compartment of the leg. Muscles in this compartment are mostly ankle plantar flexors. With tibial nerve injury plantar flexion and ankle jerk will be affected.

(Option C) With sciatic nerve involvement, global weakness and impairment of all ankle and foot movements is expected. Ankle reflex will be affected as well. Weakness of the knee flexion could also be problem.

(Option D) Clinical findings associated with lumbosacral plexus problems vary depending on the affected nerve root. S1 radiculopathy presents similar to tibial nerve injury with impaired plantar flexion, ankle jerk and sensory disturbances in the back
of the leg and most parts of the sole of the foot.

(Option E) Lumbar nerve root problems cause deficits in the movements and sensation of the hip, thigh and knee extension and deep tendon reflex.

References

• Orthobullets - Deep Peroneal Nerve

• Orthobullets - Superficial Peroneal Nerve

• Orthobullets - Tibial Nerve

• Othobullets - Sciatic Nerve

• BJA - Perioperative peripheral nerve injuries

• UpToDate - Acute lumbosacral radiculopathy: Pathophysiology, clinical features, and diagnosis

Last updated:
Time spent: QID:1105 2023-2-12

737 of 1943
Which one of the following can be a presentation associated with L5/S1 disc prolapse?

A. Absent knee reflex.

B. Absent ankle reflex.

C. Impaired ankle dorsiflexion.

D. Impaired ankle eversion.

E. Impaired sensation over the lateral aspect of the leg.

Incorrect. Correct answer is B


45% answered correctly

Explanation:

Correct Answer Is B

In L5/S1 disc prolapse, which often occurs laterally, the herniated disc compresses the S1 nerve root causing S1 radiculopathy. S1 radiculopathy presents with pain radiating down the posterior aspect of the leg into the foot from the back. On
examination, weakness of plantar flexion is specific. Weakness of leg extension (gluteus maximus) and toe flexion are other possible features. Sensation is generally reduced on the posterior aspect of the leg and the lateral edge of the foot. Loss of
the ankle reflex is typical.

(Option A) Knee reflex is an action of L3 and L4 nerve roots.

(Options C, D and E) Ankle dorsiflexion, eversion, and sensation over the lateral aspect of the leg are innervated by L5 that can be affected in L4/L5 disc prolapse.

References

• UpToDate - Acute lumbosacral radiculopathy: Pathophysiology, clinical features, and diagnosis

Last updated:
Time spent: QID:1106 2023-2-12

738 of 1943
Which one of the following is most suggestive of sciatic nerve injury?

A. Absent ankle reflex.

B. Foot drop.

C. Inability to flex the hip.

D. Decreased sensation over the anterior thigh and medial leg.

E. Intervertebral disc prolapse at L2/L3 level.

Correct
45% answered correctly

Explanation:

Correct Answer Is A

The sciatic nerve is derived from the lumbosacral plexus. Once formed, it leaves the pelvis and enters the gluteal region via greater sciatic foramen. It emerges inferiorly to the piriformis muscle, and enters the posterior thigh by passing deep to the
long head of the biceps femoris, and descends in an inferolateral (down and out) direction. Within the posterior thigh, the nerve gives off branches to the hamstring muscles and adductor magnus. When the sciatic nerve reaches the apex of
the popliteal fossa, it terminates by bifurcating into the tibial and common fibular nerves.

The sciatic nerve can be described as tibial and common peroneal nerves bundled together in a sheath of connective tissue. Common peroneal nerve and tibial nerve usually separate at the apex of the popliteal fossa, however in a minority of
individuals, they separate as they leave the pelvis.

Sciatic nerve Injury manifests as paralysis of the hamstring muscles (knee flexion weakness) and all the muscles below the knee. All sensation of the leg except the medial aspect is impaired. Absent or weak ankle reflex is most specific to sciatic
nerve injury among other options.

(Option B) Foot drop is seen in sciatic as well as common peroneal nerve injuries and is not specific to sciatic nerve injury.

(Option C) Hip flexion is the action of L1-L2 nerve roots. Hip flexion is not an action of the sciatic nerve which is formed by L4, L5, S1, S2, and S3 nerve roots.

(Option D) Decreased sensation over the anterior thigh and medial leg is caused by femoral nerve injuries.

(Option E) Intervertebral disc lesion at L2/L3 compromises the action of L3 nerve root, which does not contribute to the formation of the sciatic nerve.

References

• UpToDate - Acute lumbosacral radiculopathy: Pathophysiology, clinical features, and diagnosis

• Othobullets - Deep Peroneal Nerve

Last updated:
Time spent: QID:1107 2023-2-12

739 of 1943
A 65-year-old man is being seen in the Emergency Deaprtment because of severe headache starting two hours ago. On a scale of 1 to 10, the patient scores the pain as 10 in intensity. There is no history of similar headaches in the past. Relatives
accompanying the patient inform you that he lost consciousness 30 minutes after the headache started but he was lucid again few minutes later. Which one of the following could be the most likely cause of the headache?

A. Subdural hematoma.

B. Epidural hematoma.

C. Cerebral abscess.

D. Subarachnoid hemorrhage.

E. Migraine.

Incorrect. Correct answer is D


45% answered correctly

Explanation:

Correct Answer Is D

Of the options, subarachnoid hemorrhage (SAH) is most consistent with the clinical scenario. Symptoms of SAH typically begin abruptly, and occur at night in almost 30% of patients. The most common and premier symptom is a sudden, severe
headache occurring in 97% of patients. Patients often describe the headache as the "worst headache of my life." The headache is lateralized in 30% of patients. The onset of the headache may be associated with a brief loss of consciousness,
seizure, nausea, or vomiting. Signs of meningeal irritation may be present but may not develop until several hours after the bleed, because it is caused by the breakdown of blood products within the CSF and the consequent aseptic meningitis.

Meningismus can be elicited by a positive Kernig’s sign (extending the patient’s knee while the hip is flexed at 90° causes pain) and/or Brudzinski’s sing (flexing the patient’s head while the chest is restrained with the physician’s other hand causes
flexion of the hip and the knees). Neurological deficits such as hemiplegia, third cranial nerve palsy, etc. may be found on physical examination as well.

A non-contrast CT scan is the initial investigation of choice and should be performed as soon as possible to confirm the diagnosis.

(Option A) Subdural hematoma is due to a venous bleed between the dura and the arachnoid. It follows injuries that may be trivial, especially in the elderly. It can be acute (less than two days), subacute (2-14 days), or chronic (more than 14 days).

Subdural hematoma should be considered if a person presents with the following:

Personality changes
Slowness and unsteadiness of movements
Headache
Irritability
Fluctuating level of consciousness

CT scan of the head is the confirmatory diagnostic tool of choice for subdural hematoma.

(Option B) Epidural hematoma is a life-threatening head injury that is caused by arterial bleeding between the skull bone and dura mater. The typical clinical scenario is loss of consciousness following trauma to the head which is usually followed by
a short lucid interval before consciousness deteriorates further. The patient with epidural hematoma is usually restless, confused, irritable, and may develop severe headache, seizures, ipsilateral pupil dilatation and facial weakness. Lumbar puncture
is contraindicated in this situation. CT scan of the head is used to confirm the diagnosis.

(Option C) In cerebral abscess, symptoms are more insidious and usually develop over two weeks or so. Presentation is usually with fever, confusion and decreased responsiveness.

(Option E) Classic features of migraine include headache (frontal or temporal) radiating to retro-orbital and/or occipital area. The pain is aggravated by tension and relieved by rest particularly in a dark silent environment. Loss of consciousness is not
a feature.

References

• UpToDate - Aneurysmal subarachnoid hemorrhage: Clinical manifestations and diagnosis

Last updated:
Time spent: QID:1181 2023-2-12

740 of 1943
A 52-year-old man presents to your clinic for evaluation of spasticity and weakness in his left lower limb. The condition has progressively worsened over the past three months and made walking difficult for him. On examination, fasciculation in the
left deltoid muscle and small muscles of the hand are noted; however, the muscle power, reflexes and sensation of the upper limbs are within the normal range. In the lower limb, bilateral extensor plantar reflexes, left-sided foot drop and muscle
weakness and spasticity are noted. Lower limb sensation is intact. Which one of the following investigations is the most appropriate one to reach a diagnosis?

A. CT scan of the brain.

B. MRI of the cervical spine.

C. Electromyography.

D. Lumbar puncture (LP).

E. MRI of the brain.

Incorrect. Correct answer is C


45% answered correctly

Explanation:

Correct Answer Is C

While fasciculation of the deltoid and small muscles of the hand and the foot drop in this patient suggest a lower motor neuron (LMN) problem, increased muscle tone (spasticity) and extensor plantar reflex are pointers toward an upper motor
neuron lesion. The combination of LMN and UMN findings in the same area of the body as well as the intact sensation puts amyotrophic lateral sclerosis (ALS), the most common motor neuron disorder, at the top of the differential diagnoses list.

Motor neuron disorders (MNDs) are a clinically and pathologically heterogeneous group of neurologic diseases characterized by progressive degeneration of motor neurons. Either or both of the following two sets of motor neurons can be affected:

Upper motor neurons (UMNs) – These neurons originate from the primary motor cortex of the cerebrum (precentral gyrus). These neurons have long axons that form corticospinal and corticobulbar tracts.

Lower motor neurons (LMNs) – These neurons originate from the brainstem (cranial nerve [CN] motor nuclei) and spinal cord (anterior horn cells) and directly innervate skeletal muscles.

MNDs can be classified into those affecting primarily the UMNs, those affecting primarily the LMNs, and those affecting both. The patient’s symptoms vary, depending on which set of motor neurons is involved.

Motor neuron disorders include:

Amyotrophic lateral sclerosis (ALS)


Primary lateral sclerosis (PLS)
Hereditary spastic paraparesis (HP)
Progressive bulbar palsy (PBP).
Spinal muscular atrophy.
X-linked spinobulbar muscular atrophy.
Post-polio syndrome

ALS, the most common neurodegenerative disease of adult onset, is a fatal disorder and is characterized by progressive skeletal muscle weakness and wasting or atrophy (i.e., amyotrophy), spasticity, and fasciculations as a result of degeneration of
both the UMNs and LMNs. ALS eventually results in respiratory paralysis. Both LMN and UMN abnormalities should be observed in a single area before ALS could be considered a highly likely diagnoses. ALS is sporadic in 90-95% of cases and
hereditary in only 5-10% of affected individuals.

ALS should be suspected as a diagnosis when there is insidious loss of function or gradual, slowly progressive, painless weakness in one or more regions of the body, without changes in sensation, and there is no other explanation for to this
presentation. With time, patients with ALS develop manifestations of both LMN and UMN involvement.

Findings indicating UMN involvement include the following:

Stiffness (spasticity)
Brisk tendon reflexes (hyperreflexia) – but maybe diminished if there is concurrent LMN
Presence of abnormal reflexes (e.g., Babinski, Chaddock, or Hoffman signs)
Loss of dexterity in the presence of normal strength
Muscle spasms

Findings indicating LMN dysfunction include the following:

Twitching muscles (fasciculations) – this can be an early manifestation most commonly seen in the tongue and limbs
Reduction of muscle bulk (atrophy)
Foot drop (or wrist drop in case of upper limb involvement)
Depressed reflexes
Breathing difficulties

The diagnosis of ALS is primarily clinical. When the disease has progressed far in its course and involves many parts of the body, the patient’s appearance and neurologic examination findings often provide sufficient evidence for the diagnosis.
However, if the patient presents early in the course of the disease, the diagnosis is not straightforward, and often exclusion of other possible causes is required before a diagnosis of ALS is made because there is no pathognomonic test for ALS (or
other motor neuron disorders) and the symptoms are initially nonspecific. Often, several months (average 14 months) are needed before a definite diagnosis of ALS is made.

Of the investigations currently in use for workup and assessment of ALS (and other motor neuron disorders), electromyography (EMG) and nerve conduction studies (NCS) are most useful for confirming the diagnosis of ALS and for exclusion of
peripheral conditions that resemble ALS. The role of EMG/NCS in diagnosis of ALS is so crucial that some consider it as an extension to physical examination.

Imaging studies such as CT scan (option A) or MRI (options B and E) may be primarly used in some patients for assessment of other conditions, such as multiple sclerosis (MS), spinal canal stenosis or other spinal lesions that can mimic initial
presentations of ALS. Imaging studies are usually normal in patients with ALS, and not diagnostic for the condition.

(Option D) LP and examination of the CSF is not necessary unless the patient has a pure UMN or pure LMN presentation, in which case it can be of diagnostic value in ruling out inflammatory conditions, neoplastic infiltrations, or infection. With a
combination of both UMN and LMN manifestations, this patient will not benefit from LP as a diagnostic tool.

References

• RACGP - Motor Neuron Disease

741 of 1943
• Medscape - Amiotrophic Lateral Sclerosis

• Medscape - Pathology of Motor Neuron Disorders

• BMJ - Diagnosis and management of motor neuron disease


Last updated:
Time spent: QID:1216 2023-2-12

742 of 1943
Alan, 45 years, is a new patient to your clinic. He has presented with complaint of frequent tripping of his right foot for the past few months, which has increased in frequency in the past month. On examination, interosseous muscles wasting of the
right foot is noted. Plantar reflex is equivocal on the right but extensor on the left. Lower limb deep tendon reflexes are normal. There is foot drop on the right side. Which one of the following tests is most likely to make a diagnosis?

A. MRI of the head and cervical spine.

B. Acetylcholine receptor antibodies.

C. CT scan of the head and cervical spine.

D. Lumbar puncture (LP).

E. Electromyography (EMG).

Incorrect. Correct answer is E


45% answered correctly

Explanation:

Correct Answer Is E

Alan has signs of lower motor neuron (LMN) involvement of his right foot including the foot drop and muscle wasting. While his left foot has an upward plantar reflex (Positive Babinski) indicating upper motor neuron lesion, the right plantar reflex is
equivocal making it hard to tell if the right foot signs and symptoms are LMN or UMN in origin. However, in general he has clinical findings suggesting both LMN and UMN lesions at different sites.

Among the differential diagnoses justifying such presentation, a motor neuron disorder (MND) is the most likely diagnosis. Spinal canal stenosis can be a differential diagnosis but is far less likely in this scenario because firstly chronic pain is the
most common presenting symptom in spinal canal stenosis which is absent here. Secondly, stenotic lesions cause UMN manifestations at the level of stenosis and LMN features below that. The pattern of neurological deficits here does not match
that of a stenotic spinal canal.

Given the pattern of clinical findings, amyotrophic lateral sclerosis (ALS) as the most common MND could be the most likely diagnosis in this case. ALS often presents with insidious onset of painless motor neurological deficits and intact
sensations. With time, features of both LMN and UMN will be present in the affected area. While LMN involvement manifests with fasciculations (twitching muscles), muscle cramps, muscle atrophy, foot drop (or wrist drop) and depressed reflexes,
features of UMN include spasticity (stiffness), hyperreflexia, presence of abnormal reflexes (such as Babinski, Chaddock, or Hoffman signs), loss of dexterity in the presence of normal muscle strength and muscle spasms.

In ALS, symptoms begin with limb involvement in 75-80% of cases and bulbar symptoms in 20-25%. Patients with upper limb onset have twice the likelihood for onset in the dominant arm. Individuals with upper limb onset may experience reduced
finger dexterity, cramping, stiffness, and weakness or wasting of intrinsic hand muscles. This may lead to difficulty with actions such as buttoning clothes, picking up small objects, or turning a key. Wrist drop may develop.

There is equal likelihood for presentation in either lower extremity. Patients who have lower limb onset initially may complain of tripping, stumbling, or awkwardness when running. Foot drop is common, and patients may report a "slapping" gait.

Bulbar involvement presents with slurred speech, hoarseness, or decreased volume of speech, or aspiration or choking during a meal. Hypernasality of the voice and eventually loss of speech, swallowing difficulty (usually starting with liquids), and
drooling occurs later in the course of the disease. Women have a greater frequency of bulbar (speech dysfunction) onset than men.

Neurologic examination often provides sufficient evidence for the diagnosis. However, early in the course of the disease, the diagnosis may not be clear, and often exclusion of other possible causes is required before a diagnosis of ALS is made. This
is due to lack of a pathognomonic test for ALS (or other motor neuron disorders) and nonspecific nature of initial symptoms.

Of the investigations currently in use for workup and assessment of ALS (and other motor neuron disorders), electromyography (EMG) and nerve conduction studies (NCS) are confirmatory for ALS and for exclusion of peripheral conditions that
resemble ALS. The role of EMG/NCS in diagnosis of ALS is so crucial that some consider it as an extension to physical examination.

(Option B) Assessment of acetylcholine antibodies is used when myasthenia gravis is suspected as a diagnosis. This disease typically affects the extraocular, bulbar, or proximal limb muscles. Droopy eyelids or double vision is the most common
symptom at initial presentation (more than 75%). These symptoms progress from mild to more severe over weeks to months. Difficulty in swallowing, slurred or nasal speech, difficulty chewing, and facial, neck, and extremity weakness often follow.

Although there is significant overlap between myasthenia gravis and ALS with bulbar involvement, the specific pattern of neurological deficits in Alan makes this diagnosis less likely.

Imaging studies such as CT scan (option C) or MRI (choice) (options A) are used to exclude other mimicking conditions such as multiple sclerosis (MS), spinal canal stenosis or other spinal lesions but not for confirming ALS as a diagnosis. Imaging
studies are usually normal in patients with ALS.

(Option D) LP and examination of the CSF is not necessary unless the patient has a pure UMN or pure LMN presentation, in which case it can be of diagnostic value in ruling out inflammatory conditions, neoplastic infiltrations, or infection. With a
combination of both UMN and LMN manifestations, this patient will not benefit from LP as a diagnostic tool.

NOTE - For patients with a new focal presentation, the differential diagnoses by region include the following:

Upper motor neuron (UMN) bulbar signs - Brainstem lesions including syrinx, mass, stroke, and demyelinating forms of other degenerative diseases

Lower motor neuron (LMN) bulbar signs - Cranial nerve palsies

Limb UMN signs - Cervical myelopathy, cord tumor, hereditary spastic paraparesis, transverse myelopathy, HIV-related myelopathy, syrinx

Limb LMN signs - Radiculopathy, plexopathy, neuropathy

Differential diagnoses for patients with more advanced disease most commonly include the following:

UMN signs - Compressive myelopathy, syrinx

LMN signs - Chronic inflammatory demyelinating polyradiculoneuropathy; multifocal motor, toxic, or metabolic neuropathies or myopathies such as inclusion body myositis or polymyositis

The following table summarizes the most important differential diagnoses of MNDs and their diagnostic tests:

743 of 1943
Disorders that might resemble motor neuron disease
Diagnosis Tests to confirm the diagnosis
Multifocal motor neuropathy Nerve conduction study (NCS) – antiganglioside antibody
Chronic inflammatory demyelinating polyradiculoneuropathy neuropathy NCS – LP (elevated CSF protein)
Post-polio syndrome Clinical history – EMG/NCS
Heavy metal poisoning (e.g. lead) Heavy metal urinary or blood screen
Myasthenia gravis Repetitive nerve stimulation, single fiber-electromyography, antibodies for acetylcholine receptor
Inclusion body myositis, polymyositis, dermatomyositis Serum creatine kinase, EMG, muscle biopsy
Spinal cord compression MRI
Vitamin B12 deficiency Serum B12 level
Multiple sclerosis MRI, evoked potentials, CSF oligoclonal bands
Lyme disease Lyme serology

References

• RACGP - Motor Neuron Disease

• Medscape - Amiotrophic Lateral Sclerosis

• Medscape - Pathology of Motor Neuron Disorders

• BMJ - Diagnosis and management of motor neuron disease

Time spent: QID:1217 Last updated:


2023-2-12

744 of 1943
A 68-year-old woman presents with left hemiplegia. A CT scan of the brain is obtained and is shown in the following photograph. Past medical history is significant for a melanoma that was excised 3 years ago. Which one of the following could be
the most likely diagnosis?

A. Metastasis from the melanoma.

B. Glioma.

C. Cerebral hemorrhage.

D. Cerebral abscess.

E. Meningioma.

Correct
45% answered correctly

Explanation:

Correct Answer Is A

The CT scan shows a round hyperattenuated lesion in the right hemisphere. These features are suggestive of either an abscess or a tumor. Since the patient is afebrile, abscess is less likely as the diagnosis, and a brain tumor will top the differential
diagnoses list.

With history of melanoma, a metastatic brain tumor from the melanoma could be the most likely diagnosis.

Although, most cases of malignant melanoma are diagnosed at an early stage, when surgical excision can be curative, a few patients have metastatic disease at presentation, or develop metastases after the initial definitive surgical excision.

Melanoma can metastasise to distant sites. Brain and lung are the most common sites of metastasis.

Melanoma is one of the three most common malignancies metastasising to the brain. The other two are breast cancer and lung cancer.

The following factors are associated with increased risk of systemic metastasis (including brain metastasis):

Male gender
Melanomas arising on mucosal surfaces or the skin of the trunk, head, or neck
Wide, thick, deeply invasive, or ulcerated primary lesions
Acral lentiginous or nodular lesions on histologic examination
Involvement of more than three regional lymph nodes, either at diagnosis or relapse

If amenable, surgical resection of the metastatic brain tumors is the best appropriate treatment option; however, it is not the case often.

References

• http://www.uptodate.com/contents/melanoma-treatmen

• http://www.medscape.com/viewarticle/813109

Last updated:
Time spent: QID:614 2023-2-12

745 of 1943
Lindsay, 35 years of age, presents to your practice with a 48-hours history pf facial weakness. She has no remarkable past medical history, is on no medications currently, and has been fit and well to date. On examination, she is asked to smile. Her
appearance while smiling is shown in the accompanying photograph. She has no tinnitus, vertigo, or hearing loss. Otoscopic exam is normal. Which one of the following options can be the most likely diagnosis?

A. Acoustic neuroma.

B. Bell’s palsy.

C. Central facial nerve lesion.

D. A parotid glad tumor.

E. Ramsay – Hunt syndrome.

Incorrect. Correct answer is B


45% answered correctly

Explanation:

Correct Answer Is B

In the photograph, the forehead lines and nasolabial fold on the right side are lost on smiling. Also, the right mouth corner has failed to be elevated while she is smiling. These features suggest the complete right-sided facial palsy. With no clue on
exam pointing toward a specific underlying cause, as well as the absence of vertigo or hearing problems, Bell's palsy (idiopathic facial nerve palsy) is the most likely diagnosis.

Bell’s palsy is the most common cause of unilateral facial paralysis. This idiopathic condition, is one of the most common neurologic disorders of the cranial nerves. In most patients, Bell palsy gradually resolves over time.

Signs and symptoms of Bell palsy include the following:

Acute onset of unilateral upper and lower facial paralysis (over a 48-hour period)
Posterior auricular pain
Decreased tearing
Hyperacusis (increased hearing sensitivity)
Taste disturbances (anterior 2/3 of the tongue is innervated by the facial nerve)
Otalgia (ear pain)
Weakness of the facial muscles
Poor eyelid closure
Aching of the ear or mastoid
Tingling or numbness of the cheek/mouth
Excessive watering of the eye
Ocular pain
Blurred vision
Flattening of forehead and nasolabial fold on the side affected by palsy
When patient raises eyebrows, palsy-affected side of forehead remains flat
When patient smiles, face becomes distorted and lateralizes to side opposite the palsy

(Option A) Acoustic neuroma presents with hearing loss, tinnitus and vertigo, none of which Lindsay has. Acoustic neuromas, however, can cause facial paralysis due to pressure on the facial nerve. This occurs later in the course of the disease. It is
very unlikely that facial nerve palsy is the first presentation of an acoustic neuroma.

(Option C) A central lesion of the facial nerve, e.g. caused by a lacunar stroke, spares the forehead due to bilateral motor neuron innervation. With an affected forehead, the palsy is always peripheral.

(Option D) Malignant parotid tumors can invade the facial nerve in its course through the superficial and deep layers of the affected parotid gland and cause paralysis of the facial nerve. A painless enlarging mass is the typical initial presentation, the
absence of which makes a parotid tumor as the cause a less likely diagnosis.

(Option E) Ramsay – Hunt syndrome (herpes zoster infection of the facial nerve) is caused by the reactivation of dormant varicella zoster virus in the geniculate ganglion. It typically presents with peripheral facial palsy, ear pain, loss of taste, dry eyes
and a vesicular rash. Although Ramsay- Hunt syndrome shares many clinical features with Bell’s palsy, the absence of vesicular rash in the ear canal (the otoscopic exam is normal) makes it a less likely diagnosis.

References

• RACGP - AFP - A general practice approach to Bell’s palsy

• Medscape - Bell Palsy

Last updated:
Time spent: QID:1231 2023-2-12

746 of 1943
Peter, 48 years old, is on your office concerned about facial weakness, unpleasantly increased perception in the left ear, and that everything tastes bland. His symptoms have developed and progressed since yesterday. He denies hearing loss and
tinnitus. He is an accountant and otherwise healthy with no previous health problems but an upper respiratory viral infection 10 days ago. He does not take any medications except supplemental vitamins. On examination, there is right-sided facial
weakness. The following photograph illustrates his facial expression when he is asked to smile. Neurological examination is otherwise unremarkable. Hearing is unaffected. An otoscopic examination reveals normal ear canal and tympanic
membrane. Which one of the following diagnostic investigations should be considered for him at this stage?

A. Nerve conduction studies (NCS).

B. CT scan of the head.

C. MRI of the head.

D. Swallowing test by a speech pathologist.

E. No investigation is required at this stage.

Incorrect. Correct answer is E


45% answered correctly

Explanation:

Correct Answer Is E

Features evident in the photograph are loss of forehead wrinkles, failure of the right mouth corner to rise, absence of laugh lines, and failure of the eye to tighten up, all on the right side. These features suggest peripheral (lower motor neuron) right
facial nerve palsy (in upper motor neuron palsy the forehead is spared due two dual innervation). Additional features of Hyperacusis and loss of taste are also confirmatory findings. Peter’s physical findings are otherwise normal and there is no
significant past medical history to point towards any specific underlying etiology. These make idiopathic facial nerve palsy (Bell palsy) the most likely diagnosis.

Bell palsy is the most common cause of unilateral facial paralysis. Bell palsy gradually resolves over time. The cause is unknown; however, herpes simplex type 1 virus and herpes zoster virus have been implicated by some authors.

Clinical features of Bell palsy include:

Acute onset of unilateral upper and lower facial paralysis (over a 48-hour period)
Posterior auricular pain
Decreased tearing
Hyperacusis (increased hearing sensitivity)
Taste disturbances (anterior 2/3 of the tongue is innervated by the facial nerve)
Otalgia (ear pain)
Weakness of the facial muscles
Poor eyelid closure
Aching of the ear or mastoid
Tingling or numbness of the cheek/mouth
Excessive watering of the eye
Ocular pain
Blurred vision
Flattening of forehead and nasolabial fold on the side affected by palsy
When patient raises eyebrows, palsy-affected side of forehead remains flat
When patient smiles, face becomes distorted and lateralizes to side opposite the palsy

In many cases, the history and physical examination establish the diagnosis of Bell palsy, and no further investigation including imaging studies, laboratory tests and nerve conduction studies are required because most patients recover within 6-
8 weeks. With the clinical findings highly suggestive of Bell palsy, Peter do not require any further investigations at this stage. Investigations are only indicated if the diagnosis is uncertain or the paralysis last longer 6-8 weeks.

(Option A) NCS and electromyography (EMG) have prognostic value in Bell palsy but are most useful if performed 3-10 days after the onset of paralysis. These tests are not part of acute workup and not indicated at this stage; however, they may be
considered later for outcome estimation. Comparison of the affected to the unaffected side helps to demonstrate the extent of nerve injury and has prognostic value. Nerve conduction responses are abnormal if there is a 50% difference in amplitude
between the paralyzed and normal side; a difference of 90% suggests a poorer prognosis. Generally speaking, NCS/EMG is a tool used for research rather than diagnosis in Bell palsy.

NOTE - Most NCS/EMG studies do not show an abnormality for 3 weeks following a peripheral nerve injury.

(Option B) Ct scan of the head is indicated if there are other associated physical findings or if the paresis is progressive and unremitting. CT scan is used for assessment of the temporal bone architecture, and, if used with contrast enhancement, for
detection of tumors compressing the facial nerve (MRI is a better option for this purpose).

(Option C) In patients with Bell palsy, MRI may show enhancement of the facial nerve at or near the geniculate ganglion. MRI, however, is not indicated unless the diagnosis is in doubt, symptoms do not resolve within 6-8 weeks, or symptoms are
progressive. MRI is the preferred imaging modality for assessment of cerebellopontine angle, or if, based on the history and/or clinical findings, a tumor compressing the facial nerve is suspected. Such tumors include acoustic neuroma
(Shcwannoma), hemangioma, meningioma, and sclerosing hemangioma. Acoustic neuroma is the most common tumor associated with facial nerve palsy.

(Option D) Facial nerve has not a significant role in swallowing and swallowing remains mostly unaffected. Therefore, swallowing tests for assessment of dysphagia in patients with facial nerve palsy are not indicated.

References

747 of 1943
• RACGP - AFP - A general practice approach to Bell’s palsy

• Medscape - Bell Palsy


Last updated:
Time spent: QID:1232 2023-2-12

748 of 1943
A 58-year-old man presents with complaint of progressive difficulty in walking for the past few months. He denies any pain or pins and needles. On examinations, he drags his left foot on walking. On the left side, ankle tendon reflexes are
exaggerated and the strength of plantar flexion and dorsiflexion are decreased to 4/5. Sensation is intact. The right foot is completely normal. Plantar reflexes are equivocal bilaterally. The rest of the physical examination including neurologic exam
of the face and upper extremities are unremarkable. Lesions of which one of the following parts could be the mostly likely cause to this presentation?

A. Common peroneal nerve.

B. L5/S1 nerve root.

C. Cervical spine.

D. Brainstem.

E. Cerebral cortex.

Incorrect. Correct answer is E


45% answered correctly

Explanation:

Correct Answer Is E

One of important points to consider in dealing with patients who have presented with motor neurological deficits, such as this patient, is the possible origin(s) of such presentation. Distinguishing lower motor neuron lesions from those of upper
motor neuron is of paramount importance.

Upper motor neurons (UMNs) neurons originate from the primary motor cortex of the cerebrum (precentral gyrus). These neurons have long axons that form corticospinal and corticobulbar tracts. Lesions of UMN presents with:

Stiffness (spasticity)
Brisk tendon reflexes (hyperreflexia)
Presence of abnormal reflexes (e.g., Babinski, Chaddock, or Hoffman signs)
Loss of dexterity in the presence of normal strength
Muscle spasms

Lower motor neuron (LMNs), on the other hand, originate from the brainstem (cranial nerve [CN] motor nuclei) and spinal cord (anterior horn cells) and directly innervate skeletal muscles. Findings consistent with LMNs lesions include:

Fasciculations (twitching muscles)


Reduction of muscle bulk (atrophy)
Decreased muscle strength
Depressed deep tendon reflexes

While the weakness of muscles responsible for weak ankle flexion and dorsiflexion indicates an LMN lesion, the increased ankle tendon reflexes on the same side reflects an UMN problem. Therefore, this patient has both LMN and UMN disorders at
the same time. Additionally, he has no sensory impairment. This constellation of findings makes a motor neuron disease (MND), more specifically, amyotrophic lateral sclerosis, the most likely diagnosis. Of the given options, only lesions of cerebral
cortex can give rise to this scenario.

Injuries to the common peroneal nerve (option A) can weaken the ankle dorsiflexion and disturbed sensation over the anterolateral aspect of the leg and lateral aspect of the foot. However, such lesions are associated with a depressed ankle reflex.
Moreover, with such injuries, sensation of the lateral aspect of the lower leg and foot that are innervated by the superficial branch of this nerve would be impaired.

Lesions of the L5/S1 nerve root (option B) can cause weak ankle dorsiflexion and drop foot (L5) and decreased plantar flexion (S1). Atrophy of muscles innervated by these nerve roots is another feature, but since the lesions are of lower motor
origin, decreased reflexes would be expected. This patient has also features consistent with UMN deficits that makes this diagnosis unlikely.

Cervical spine lesions (option C) caused by conditions such as multiple sclerosis (MS), or spinal canal stenosis/compression can cause signs and symptoms consistent with UMN lesions. But with cervical spine involvement, neurological deficits are
expected to be more pronounced in upper extremities than the lower extremities. Normal neurologic exam findings in the face and arms make cervical spine lesions a less likely diagnosis.

(Option D) UMN lesions of the brainstem origin is almost an unlikely diagnosis in this scenario due to lack of signs and symptoms suggestive of bulbar involvement such as slurred speech and oropharyngeal dysphagia, especially for liquids, or facial
involvement.

References

• RACGP - Motor Neuron Disease

• Medscape - Amiotrophic Lateral Sclerosis

• Medscape - Pathology of Motor Neuron Disorders

• BMJ - Diagnosis and management of motor neuron disease

Last updated:
Time spent: QID:1239 2023-2-12

749 of 1943
A 56-year-old man presents to the emergency department with complaint of diplopia. On examination, he has right-sided ptosis. His visual acuity and visual fields are unaffected. His right eye is depressed inferiorly and laterally and he is not able to
look at to the left side. The pupillary light reflex of the right eye is sluggish. The neurological examination is otherwise unremarkable. Which one of the following can be the most likely cause to this presentation?

A. Midbrain infarct.

B. Right 6th nerve palsy.

C. Posterior communicating artery aneurysm.

D. Right 4th nerve palsy.

E. A lesion in the occipital cortex.

Incorrect. Correct answer is C


45% answered correctly

Explanation:

Correct Answer Is C

This patient has the classic presentation of a third cranial nerve palsy. The third cranial nerve supplies the levator muscles of the eyelid and four extraoccular muscles: (1) the medial rectus (eye adduction), (2) superior rectus (eye elevation), (3)
inferior rectus (eye depression), (4) and inferior oblique (eye elevation). Additionally, the third cranial nerve constricts the pupil through its parasympathetic fibers that supply the smooth muscles of the ciliary body and the sphincter of the iris. The
third nerve begins as a nucleus in the midbrain that consists of several subnuclei innervating the individual extraoccular muscles, the eyelids, and the pupils.

The etiology of the third cranial nerve palsy is vast and includes conditions such as:

Ischemia (e.g. diabetes and midbrain infarcts)


Compressive effects of aneurysms such as those of the posterior communicating artery, internal carotid artery and basilar artery.
Trauma
Infections
Infiltrative diseases (e.g. neoplasms)
Demyelination
Space occupying tumors
Meningitis
Herniation
Inflammatory diseases

Of the given options, only a posterior communicating aneurysm can cause a third nerve palsy that involves the pupilary reflex. Compression of the third nerve by an enlarging intracranial aneurysm is the most dreaded etiology. The most common site
of an aneurysm causing a third nerve palsy is the posterior communicating artery. In the setting of an acute third nerve palsy, the aneurysm is believed to be progressively and acutely enlarging and is at imminent risk of rupture with catastrophic
outcomes.

(Option A) Midbrain infarcts can cause third cranial nerve palsy. However, since the etiology is ischemic (such as in diabetes) the pupillary light reflex is expected to be unaffected most of the time. Furthermore, with a midbrain infarct, an isolated
third nerve palsy as the sole presenting symptom is very unlikely and other signs and symptoms related to a midbrain infarct are expected.

th
(Option B) Right 6 nerve palsy presents with horizontal binocular diplopia upon looking laterally to the affected side. Furthermore, lesions of the sixth cranial nerve do not cause ptosis.

(Option D) The fourth cranial nerve palsy presents with binocular vertical diplopia. Ptosis is not a feature.

(Option E) Lesions of occipital cortex are associated with impaired vision and visual fields deficits, none of which are present here. Furthermore, ptosis is not a feature.

References

• UpToDate - Third cranial nerve (oculomotor nerve) palsy in adults

• Medscape - Third Nerve Palsy (Oculomotor Nerve Palsy)

Last updated:
Time spent: QID:1245 2023-2-12

750 of 1943
Which one of the following is the most likely sensory disturbance associated with the following x-ray?

A. Paresthesia over the deltoid.

B. Decreased sensation of the ulnar aspect of the forearm and hand.

C. Decreased strength of hand grip.

D. Decreased sensation over dorsum of the hand.

E. Decreased sensation over the 2/3 radial side of the palm.

Incorrect. Correct answer is D


45% answered correctly

Explanation:

Correct Answer Is D

The X-ray shows a fracture in the mid-shaft of the left humerus. Radial nerve injury and palsy associated with fracture of the shaft of the humerus is the most common nerve injury complicating fractures of long bones. Rarely, median or ulnar nerves
can be affected. Injury to the axillary nerve is seen in the fractures of humeral neck.

The primary motor function of the radial nerve is to innervate the muscles responsible for wrist and metacarpophalangeal (MCP) joints extensors, and abduction and extension of the thumb; therefore, proximal radial nerve injuries results in wrist
drop.

Radial nerve also provides the sensation of dorsum of the hand and fingers at the 2/3 radial portion (thumb, index, middle and half of the ring fingers), the web space between the thumb and index fingers, most parts of the posterior aspect of the
forearm. Depending on the anatomical site of the injury, sensory disturbances over these areas can be noted.

Of the options, decreased sensation over dorsum of the hand can be caused by radial nerve injury.

(Option A) Paresthesia over the deltoid area is a sign of damage to the axillary nerve. Axillary nerve damage can be an associated injury in fractures of the humeral neck, not the shaft.

(Option B) Decreased sensation over the ulnar aspect of the forearm and hand is associated with ulnar nerve injuries. Compared to the radial nerve injuries, ulnar nerve injuries are rare in humeral shaft fractures.

(Option C) The strength of grip is the action of flexor muscles of the wrist and fingers. These muscles are innervated by median (mostly) and ulnar nerves.

(Option E) Decreased sensation over the 2/3 radial side of the palm can be caused by injuries of the median nerve.

751 of 1943
References

• Medscape - Humerus Fracture

Last updated:
Time spent: QID:1246 2023-2-12

752 of 1943
Which one of the following could the most likely finding in a patient with mid-shaft humerus fractures?

A. Numbness of the little finger.

B. Inability to extend the thumb.

C. Numbness of the ventral aspect of the thumb.

D. Decreased grip strength.

E. Decreased wrist flexion.

Incorrect. Correct answer is B


45% answered correctly

Explanation:

Correct Answer Is B

Radial nerve injury and palsy associated with fracture of the shaft of the humerus is the most common nerve injury complicating fractures of long bones. Rarely, median or ulnar nerves can be affected. Injury to the axillary nerve is seen in the
fractures of humeral neck.

The primary motor function of the radial nerve is to innervate the muscles responsible for wrist and metacarpophalangeal (MCP) joints extensors, and abduction and extension of the thumb; therefore, proximal radial nerve injuries results in wrist
drop or the decreased or absent thumb extension and abduction.

(Option A) The sensation of the little finger and half of the ring fingers are supplied by terminal braches of the ulnar nerve which is very less like to be damaged in humeral mid-shaft fracture compared to the radial nerve.

(Option C) The sensation of the ventral aspect of the thumb is provided by the terminal branches of the median nerve. Compared to the radial nerve, the median nerve is less likely to be affected in a mid-shaft humeral fracture.

(Options D and E) The motor innervation of hand grip and wrist flexion is supplied by the median nerve.

References

• Medscape - Humerus Fracture

Last updated:
Time spent: QID:1247 2023-2-12

753 of 1943
A 71-year-old man, who has been diagnosed with bilateral osteoarthritis of his knees few years ago, presents to the GP clinic you work at with complaint of paresthesia and numbness of the lateral aspect of his right leg. The condition is worse at
night and improves after walking for about 10 minutes. Which one of the following is the most important physical exam finding to consider in this patient?

A. Positive straight leg raise test.

B. Any tenderness over the right lateral condyle.

C. Any loss of sensation over the lateral side of the right leg.

D. Weakened dorsiflexion of the right foot.

E. Weak distal pulses of the right leg.

Incorrect. Correct answer is D


45% answered correctly

Explanation:

Correct Answer Is D

The chief complaint of this man is paresthesia and numbness below the knee over the lateral side of the right leg. This presentation can be caused by common peroneal nerve injury (CPN). CPN is one of the two main nerves comprising the sciatic
nerve. Along with the tibial nerve, CPN runs down from the buttock down the posterior part of the thigh to the apex of the popliteal fossa where it changes its path lateral to reach the knee. After wrapping around the fibular head, CPN enters the
lateral compartment of the leg. There, this nerve divides in 'deep peroneal' and 'superficial peroneal' nerves. The proximal lateral aspect of the leg is innervated by CPN, while the distal part is innervated by superficial peroneal nerve.

CPN through its branches supplies the muscles of ankle dorsiflexion, eversion and inversion in the anterior and lateral compartments of the leg. CPN nerve injury can cause weak ankle dorsiflexion (extension) resulting in foot drop and ankle and foot
eversion, as well as toes extension.

This patient has presented with sensory deficits without any obvious motor weakness. He has osteoarthritis and bony processes (osteophytes) can compress the CPN in the lateral aspect of the knee, resulting in nerve irritation and damage. It
should be noted that motor weakness, if subtle, might not be always appreciated by the patient. Therefore, a thorough physical examination is required to reveal any motor deficits that have remained unnoticed by the patient. In this case, weakened
dorsiflexion of the right foot makes the diagnosis almost certain. Weakened dorsiflexion is the characteristic feature of CPN nerve injury as well as L5 radiculopathy.

(Option A) This manifestation can also be caused by L5 radiculopathy. Lesions of L5 nerve root and CPN nerve in the leg present similarly. However, these two can be differentiated by other historical and clinical findings. L5 radiculopathy is often
associated with low back pain as a prominent feature. The pain radiates down from the lateral aspect of the thigh, is aggravated by walking (especially uphill) and bending, and is relived by recumbency and lying down. Pain relieved by walking and
aggravated during sleep in unusual for L5 nerve root lesions. If in doubt, tests such as straight leg raise (SLR) or slump test can reproduce the symptoms in L5 radiculopathy, but not in CPN lesions.

(Option B) While tenderness over the lateral condyle may be caused by compressing an injured CPN, it could also be just due to osteoarthritis itself, and is not a distinguishing feature.

(Option C) With this presentation, loss of sensation over the lateral aspect of the leg is very likely as the main complaint is paresthesia and numbness of this area. Such finding adds no significant diagnostic benefit.

(Option E) Weak distal pulses of the leg can be an important finding pointing towards vascular problems as a concomitant condition. It does not justify or adds any diagnostic clue to the presentation.

References

• Medscape - Peroneal Mononeuropathy

Last updated:
Time spent: QID:1248 2023-2-12

754 of 1943
Mother of an 8-year-old girl has brought her for genetic testing for Huntington disease because the father has been recently diagnosed with the disease. She knows that the disease can have been passed to her daughter as well and she is very
worries about this issue. Which one of the following is the most appropriate action in this situation?

A. Do not perform the genetic test.

B. Counsel the daughter about the disease and genetic testing.

C. Perform genetic testing both for the mother and daughter.

D. Genetic counselling.

E. Perform the genetic test only for the daughter.

Correct
45% answered correctly

Explanation:

Correct Answer Is A

Huntington disease is an autosomal dominant genetic disorder affecting approximately 1 in 15000 Australian population. The disease affects males and females equally and is found in all ethnic groups but it is among people of European descent.

The symptoms often starts between 30 and 50 years of age and can include:

Involuntary muscle movements affecting the face, hands, limbs and body that are irregular, jerky and uncoordinated.
Slurred speech and difficulties with swallowing.
Disruptions to thought processes, including short term memory loss, difficulty with making decisions, lack of motivation, reduced insight and awareness of social boundaries.
Unsteadiness and falls.
Difficulty with emotional control, frustration, agitation and mood swings.
Depression and anxiety.

HD progresses slowly. After developing the first symptoms, affected people usually live 15 to 20 years.

For HD, there is genetic testing that predicts whether a person with a family member affected with HD will develop the disease in the future. According to Australian national guideline, in concordance with those of WHO, eligibility criteria for such test
are:

The age of the individual is at least 18 years

AND

There is at least one blood-relative with definite diagnosis of HD in the pedigree.

This means no genetic testing for HD is performed on request of a person, or the carers (e.g. parents) of a person under the age of 18 years.

In 1989 a research group of the World Federation of Neurology declared that children should not be tested for HD on their parents’ request. The age of majority remained the touchstone in the 1994 recommendations of a joint committee of the
International Huntington’s Association and the World Federation of Neurology Research Group on Huntington’s chorea. But the report added, “It seems appropriate and even essential, however, that the child be informed of his or her at-risk status
upon reaching the age of reason.”

In the same year, a working party of the Clinical Genetics Society concluded that although discussion and counselling could and should be offered to minors, “formal genetic testing should generally wait until the ‘children’ request such tests for
themselves, as autonomous adults. Testing should be waited either until the person affected is adult or “is able to appreciate not only the genetic facts of the matter but also the emotional and social consequences.”

The following summarizes the current guidelines for genetic testing for HD:

No genetic testing is performed for children under 18 years upon parents’ (or other legal carers’) request
Genetic counselling (different from testing) will not be performed for the child alone on parents’ request.
No genetic testing will be performed upon the request of a consenting (competent) minor, but they can be offered genetic counselling (different from testing) provided that a belief is formed. On reasonable grounds, that the young
person can handle the emotional and social consequences

Based on the guideline, genetic testing requested by the mother should not be performed for this girl until the age of 18. On the other hand, her very young age excludes her from genetic counselling (not testing) for her. She is too young to
understand and handle the given information or handle the social and emotional ramifications.

References

• https://www.ncbi.nlm.nih.gov/pmc/articles/PMC1115457/

• http://www.genetics.edu.au/publications-and-resources/booklets-and-pamphlets/huntington-disease-and-genetic-testing

• https://www.huntingtonswa.org.au/information/genetics/

Last updated:
Time spent: QID:1272 2023-2-12

755 of 1943
A 16-year-old girl comes to your GP clinic requesting genetic testing for Huntington disease. She is very distressed because her father was diagnoses with the disease two months ago. She surfed the internet about the disease and has known that
the disease is inheritable and she may develop it later in life. She says that this is her decision and her parents do not know anything about her request. She does not want to inform them. Which one of the following is the most appropriate action in
this situation?

A. Arrange the genetic test for her.

B. Tell her that she needs to comeback for the test when she is 18 years old.

C. Arrange for the whole family to have genetic counselling.

D. Tell her parents’ consent is required for genetic testing.

E. Seeking a court order before proceeding to the test.

Incorrect. Correct answer is B


45% answered correctly

Explanation:

Correct Answer Is B

Current national Australian guidelines, in agreement with WHO guidelines, recommend predictive genetic testing for Huntington disease (HD) only for people aged 18 years or older who have at least one blood relative with definite diagnosis of HD.

In cases where a consenting minor fulfilling the criteria for Gillick competence rule requests for such testing, the rule still holds true; however, genetic testing for HD will not be performed for such minor, even if he/she is independent and seems to
understand, and can handle the social and emotional consequences of a positive test result for HD and the fact that he/she will eventually develop the disease and probably die of it. In such instances, the child is needed to be informed that he/she
can request such a test at the age of 18 years or older. However, the child can be offered referral to an accredited genetic unit for genetic counselling (not testing), provided that the clinician, based on reasonable ground, form a belief that this minor
can handle the situation.

This girl should be told that she cannot have the genetic test until she is 18 years old.

(Option A) Arrangement for the genetic test for her is incorrect and she can have the test at the age of 18 years or older.

(Option C) Arrangement for offspring of this family is only performed upon their request provided that they fulfill the criteria of predictive genetic testing for HD.

(Option D) Parents cannot consent to or request for HD predictive genetic testing on the minors’ behalves. Once the minor are 18 years no parental consent for testing is required.

(Option E) A competent adult can request for the test. A court order is required where guidelines are unclear for decision making.

References

• Can children and young people consent to be tested for adult onset genetic disorders?

• Huntington Disease and Genetic Tetsting

• Huntington's Western Australia

Last updated:
Time spent: QID:1273 2023-2-12

756 of 1943
An eight-year-old boy is brought to your practice by his mother for what she calls funny turns in the past several weeks. According to the mother, the boy have episodes of sudden stares along with fidgeting of the right hand fingers and right arm, lip
smacking and chewing movements while his head twitches to the right side. He has 3 to 4 episodes every day with each lasting 60-90 seconds. There is always spontaneous resolution after which the child is confused for 1 to 2 minutes before he
becomes completely himself and back to whatever he was doing before. Which one of the following is the most likely diagnosis?

A. Temporal lobe epilepsy.

B. Juvenile myoclonic epilepsy.

C. Absence seizure.

D. Hypsarrhythmia.

E. Rolandic epilepsy.

Correct
45% answered correctly

Explanation:

Correct Answer Is A

The clinical picture in this child is highly suggestive of temporal lobe epilepsy as the most likely diagnosis. Temporal lobe epilepsy is categorized under complex partial seizures meaning that the seizure has a particular focus in the brain (partial) and
the awareness is impaired during an attack (complex). Temporal lobe epilepsy is characterized by automatism including fidgeting, chewing, lip smacking, fumbling, or complex acts such as speed driving, kissing, violence or other bizarre behaviors.
The duration of an attack is often 1-3 minutes. There is a postictal state of 1 to 2 minutes associated with confusion. The patient has no recollection of the events during the attack.

Other features include memory disturbances such as deja vu or jamais vu, hallucinations, emotional disturbances such as a sense of sudden terror, panic, anger or drealization. Ictal or post-ictal aphasia can be a feature. Abdominal rising sensation
(abdominal aura) with or without nausea and vomiting may occur. Patients may have bizarre delusions.

(Option B) Juvenile myoclonic epilepsy, also known of myoclonic epilepsy of Janz, is the triad of: (1) myoclonic jerks, (2) tonic-clonic seizures, and (3) absence seizures. Myoclonic and tonic-clonic seizures often occur early in the morning after
waking. Typically the onset is around puberty but it may develop earlier in life. The condition is often life-long and responds well to sodium valproate. There is no cognitive impairment.

(Option C) Absence seizure, formerly known as petit mal seizure, is characterized by often over a hundred daily attacks of sudden onset alteration of awareness and blank stare. The common age for absence seizure is 5 to 7 years. There might be
automaticity of face and other body parts. Ethosuximide is the first-line therapy.

(Option D) Hypsarrhythmia, also known as infantile spasms or West syndrome, are generalized tonic seizures featured by sudden flexion of the arms, forward flexion of the trunk and extension of the legs lasting only a few seconds. The age of onset
is often between 3 and 7 months and commonly restricted to the first three years of life before being replaced by other types of seizures later on. There are different cognitive outcomes but the prognosis is often poor. First-line treatment is with
corticotropin (ACTH) intramuscularly. Prednisolone, vigabatrin, sodium valproate and benzodiazepines has been tried as second-line therapies. The age of the child and the seizure features are not consistent with such diagnosis.

(Option E) Benign Rolandic epilepsy is a simple partial seizure often associated with a family history of epilepsy. It affects children between 2 and 13 years with a peak age of 5 to 8 years. There are motor or somatosensory disturbances involving the
mouth and face during sleep and a characteristic ‘glugging’ sound (the sound a liquid pouring from a bottle). The child often wakes up and goes to the parents with a twisted face and inability to speak. Progression to tonic-clonic seizures is likely but
there is often a good prognosis and remission occurs around the puberty. Carbamazepine is the first-line medication for treatment.

References

• Medscape - Temporal Lobe Epilepsy

Last updated:
Time spent: QID:1281 2023-2-12

757 of 1943
A 68-year-old man presents with a history of headache for the past several months. The headache is worse on waking up, coughing, sneezing and bending forward. He also mentions that he has been suffering from cough for the past 3 months. His
history is significant for smoking of 45 cigarettes a day for the past 45 years. Significant finding on examination is left hemiparesis. A chest X-ray reveals an opacity of 3x2 cm in size in the left lung field. A contrast CT scan of the head is obtained
which is shown in the following picture. Which one of the following could be the most likely diagnosis?

A. Primary brain tumor.

B. Metastatic brain tumor.

C. Brain abscess.

D. Cerebral hemorrhage.

E. Ischemic stroke.

Incorrect. Correct answer is B


45% answered correctly

Explanation:

Correct Answer Is B

The clinical picture of headache worse in the morning, and by sneezing, coughing and bending forward is suggestive of a space-occupying lesion in the brain that can be either a tumor or abscess. Considering the age of the patient, history of heavy
smoking and the cough, this patient has lung cancer with high certainty.

Lung cancer is one of the three primary cancers that tend to metastasize to the brain (21%). The other two are melanomas (40%) and breast cancer (9%).

With this history and chest X-ray findings, the lesion is most likely to be a metastatic brain lesion from a primary lung cancer.

Metastatic brain tumors are the most common form of brain tumors. Of all brain tumors, more than a half are metastatic. In patients with systemic malignancies, brain metastases occur in 10-30% of adults and 6-10% of children.

(Option A) With the high probability of an underlying lung cancer in this patient, this tumor is more likely to be metastatic than primary.

(Option C) Brain abscess has similar appearance on contrast CT scan (ring enhancement); however, with absence of fever and considering the history, a brain abscess is not a likely diagnosis.

(Options D and E) The appearance of the lesion as well as the insidious onset and progression of the symptoms make cerebral hemorrhage and ischemic stroke unlikely diagnoses.

References

• http://emedicine.medscape.com/article/1157902-over

• http://www.uptodate.com/contents/overview-of-the-c

Last updated:
Time spent: QID:621 2023-2-12

758 of 1943
A 65-year-old man is brought to your clinic for evaluation after his carer noticed that his movements have been progressively slowing down for the past few weeks. He has been on chlorpromazine for the past 8 weeks for treatment of psychotic
features he developed after his wife passed away. Which one of the following options would suggest a diagnosis of drug induced Parkinsonism rather than Parkinson’s disease?

A. Restlessness of the arms and legs.

B. Bilaterality of the symptoms.

C. Rigidity.

D. Masked face.

E. Bradykinesia.

Incorrect. Correct answer is B


45% answered correctly

Explanation:

Correct Answer Is B

Drug-induced Parkinsonism (DIP) is the second-most-common etiology of Parkinsonism in the elderly after Parkinson's disease (PD). Many patients with DIP may be misdiagnosed with PD because the clinical features of these two conditions are
indistinguishable.

Typical antipsychotics such as phenothiazines (e.g. chlorpromazine) and haloperidol are the leading cause of DIP. Metoclopramide is another drug frequently associated with DIP. Other medications with potential ability to cause DIP include calcium
channel blockers, atypical antipsychotics, and antiepileptic drugs.

The clinical manifestations of DIP are classically described as bilateral and symmetric Parkinsonism without tremor at rest. Although approximately 50% of DIP patients show asymmetrical Parkinsonism and tremor at rest, bilaterality of symptoms is
in favor of PID rather than Parkinson’s disease.

Rigidity (option C), masked face (option D) and bradykinesia/hypokinesia (option E) are shared features of both PID and Parkinson’s disease.

Restlessness of arms and legs (option A) is not seen in any of the conditions.

TOPIC REVIEW

Parkinsonism is defined as the triad of: (1) bradykinesia/hypokinesia, (2) rigidity, and (3) tremors. Atypical Parkinsonism is defined as syndromes characterized by some features of Parkinsonism PLUS additional features (Parkinson plus).

Early clinical features that suggest an atypical Parkinsonism rather than Parkinson’s disease include the following:

Falls at presentation or early in the disease


Poor response to levodopa
Symmetry at disease onset
Rapid disease progression
No tremor
Dysautonomia (e.g., urinary incontinence, fecal incontinence, catheterization for urinary retention, persistent erectile failure, prominent symptomatic orthostatic hypotension)

The atypical parkinsonisms are usually associated with little or no tremor, relatively early speech and balance difficulty, and little or no response to dopaminergic medications.

Parkinson plus syndromes include:

Progressive supranuclear palsy (PSP) - early falls and postural instability; vertical gaze paralysis; rigidity of trunk>limbs; symmetrical onset; speech and swallowing problems; tremor is unusual
Multiple system atrophy (MSA, Shy-Dragger syndrome) - early autonomic features (postural hypotension, bladder dysfunction; cerebellar and pyramidal signs; rigidity>tremor
Cortico-basal degeneration (CBD) - akinetic rigidity involving one limb; cortical sensory loss (e.g. astereognosis); apraxia (in the extreme there may be autonomous interfering activity of affected limb – the ‘alien limb’ phenomenon)
Lewy body dementia - early dementia with fluctuating cognition and visual hallucinations.
Vascular Parkinsonism -Parkinsonism worse in legs than arms; pyramidal signs; prominent gait abnormality.

References

• https://emedicine.medscape.com/article/1831191-differential

• https://www.ncbi.nlm.nih.gov/pmc/articles/PMC3325428/

• Oxford Handbook of Clinical Medicine – Oxford – 8th Edition – Pages 498-499

Last updated:
Time spent: QID:1284 2023-2-12

759 of 1943
Which one of the following, if present, is in favor of Parkinson’s disease rather than drug induced Parkinsonism?

A. Tremors.

B. Rigidity.

C. Symmetrical symptoms.

D. Masked face.

E. Bradykinesia.

Correct
45% answered correctly

Explanation:

Correct Answer Is A

Drug-induced Parkinsonism (DIP) is the second-most-common etiology of Parkinsonism in the elderly after Parkinson's disease (PD). Many patients with DIP may be misdiagnosed with PD because the clinical features of these two conditions are
indistinguishable.

Typical antipsychotics such as phenothiazines (e.g., chlorpromazine) and haloperidol are the leading cause of DIP. Metoclopramide is another drug frequently associated with DIP. Other medications with potential ability to cause DIP include calcium
channel blockers, atypical antipsychotics, and antiepileptic drugs.

The clinical manifestations of DIP are classically described as bilateral and symmetric Parkinsonism without tremor at rest. Although approximately 50% of DIP patients show asymmetrical Parkinsonism and tremor at rest, bilaterality of symptoms is
in favor of DIP rather than Parkinson’s disease.

Of the option, presence of tremors is more (not definitely) suggestive of Parkinson’s disease rather than DIP.

Rigidity (option B), masked face (option D), and bradykinesia/hypokinesia (option E) are shared features in both PID and Parkinson’s disease.

Symmetrical symptoms (option C) is in favor of DIP rather than Parkinson’s disease.

References

• Journal of Clinical Neurology - Drug-induced Parkinsonism

Last updated:
Time spent: QID:1285 2023-2-12

760 of 1943
A 12-year-old boy is brought to your office after accidentally cutting his left wrist with a pocketknife. On examination, there is a deep 2-cm laceration at the base of the thenar eminence. To test the motor injury to the median nerve you would have the
patient:

A. Extend the thumb and fingers.

B. Oppose the thumb and little finger.

C. Flex the wrist.

D. Abduct the thumb and index finger.

E. Adduct the thumb and index finger.

Incorrect. Correct answer is B


45% answered correctly

Explanation:

Correct Answer Is B

The scenario describes a cut in the thenar area of this child.

There are three muscles in the thenar eminence:

1. abductor pollicis brevis


2. flexor pollicis brevis
3. opponens pollicis.

Median nerve injury at wrist level results in the dysfunction of these three muscles (as well as the lumbrical muscles of the digits 2 and 3).

The first two muscles are conjoined with fibers from their counterparts originating from forearm, namely abductor pollicis longus and flexor pollicis longus. While median nerve injury at wrist level causes the dysfunction of thenar region muscles,
thumb abduction and flexion remain preserved due to compensatory action of abductor pollicis longus and flexor pollicis longus muscles. However, thumb opposition becomes severely impaired due to denervation of the opponens pollicis muscle.
Isolated impairment of thumb opposition is the most likely finding in median nerve injuries at wrist level.

(Option A) Extension of the thumb and fingers is an action of radial nerve and is not affected by median nerve injuries.

(Option C) Wrist flexion is mainly the action of flexor carpi radialis and flexor carpi ulnaris muscles, innervated by the median and ulnar nerves, respectively, in the forearm. Median and ulnar nerve injuries in the wrist will not affect wrist flexion;
therefore, this movement will be unaffected in this patient.

(Option D) While median nerve injury at wrist can impair the function of abductor pollicis brevis in the thenar eminence, thumb abduction is preserved to a great extend, because the fibers of abductor pollicis longus are innervated in the forearm by
the anterior interosseous nerve (a branch of median nerve) and unaffected by injury to the median nerve at wrist level. Therefore, this action is preserved. Abduction of other fingers than the thumb are provided by dorsal interosseous muscles and
abductor digiti minimi, all supplied by the ulnar nerve; hence unaffected in this patient.

(Option E) Thumb adduction is mainly provided by the muscle adductor pollicis that is innervated by the deep branch of the ulnar nerve. Median nerve injury at wrist level is unlikely to significantly affect this movement. Adduction of the other fingers
other than the thumb are the action of palmar interosseous muscles all innervated by ulnar nerve and is not expected to be impaired in this patient.

References

• Neuroexam.com - Strength of Individual Muscle Groups

• BMJ - Movements of the thumb in relation to peripheral nerve injuries

Last updated:
Time spent: QID:1303 2023-2-12

761 of 1943
A 35-year old woman presents to your practice with complaint of decreased ability to hold small objects between his right thumb and index finger. He noticed it few months ago when he was unable to properly hold the door or car keys. Which one of
the following conditions could have led to this presentation?

A. Radial nerve injury.

B. Carpal tunnel syndrome.

C. Anterior interosseous nerve injury.

D. Ulnar nerve injury.

E. Posterior interosseous nerve injury.

Incorrect. Correct answer is D


45% answered correctly

Explanation:

Correct Answer Is D

In order to understand clearly the nerve injury associated with this presentation, it is essentinal to know the pinching and its types.

Four types of pinching have been described:

Tip to tip pinch (pulp of the thumb against that of the index/middle finger) – this type of pinch is used for holding and positioning of small objects

Tip-to-tip pinch

Pad to pad pinch (pad of the thumb against index / middle finger) – this type of pinch is used for larger objects or objects that need more stabilization

Pad-to-pad pinch

762 of 1943
Three jaw chalk pinch (thumb against tips of index and middle fingers) – it is used for larger or heavier objects e.g., holding a piece of chalk while writing on the blackboard or holding a pencil

Three-jaw-Chalk pinch

Lateral / key pinch (pulp of thumb against radial/lateral aspect of the index finger) – object is aligned with forearm and with supination/pronation the object is moved through the space (using a key to open a door)

Lateral/key pinch

Different muscles work together to provide precise alignment and power of different types of pinch (pincer grip); however, the two key muscles providing the strength of a grip are adductor pollicis and dorsal interosseous muscle of the index finger,
both of which are supplied by the ulnar nerve. With decreased power of pinching, an ulnar nerve injury is more likely to be the underlying cause of this presentation.

If a patient with ulnar nerve injury is asked to grasp a piece of paper between the thumb and lateral aspect of the index finger, loss of power to adductor pollicis muscle will cause flexion of the distal thumb as the flexor pollicis longus compensates.
Patient makes a “clamp” instead of the naturally-occurring “tongs” to hold the paper (Froment’s sign).

763 of 1943
(Option A) Radial nerve and its branch - the posterior interosseous supply the extensor muscles of the forearm, wrist and fingers. Radial nerve lesions do not cause weakness of pinching between the thumb and index finger.

(Option C) For tip-to-tip approximation during pinching between the thumb and index finger (thumb-index finger pincer grip) the two key muscles involved are flexor pollicis longus (flexing the interphalangeal joint of the thumb) and flexor digitorum
profundus (flexing the distal interphalangeal joint of the index finger). Both flexor pollicis longus and the flexor digitorum profundus muscles of fingers II and III are innervated by anterior interosseous nerve (AIN), a motor branch of the median nerve.
Patients with AIN lesions typically fail to make an “O.K.” sign, because flexion of the interphalangeal joint of the thumb and the distal interphalangeal joint of the index finger are impaired. Another sensitive test is the pinch test: a patient with AIN
lesions will also not be able to pinch a sheet of paper between his thumb and index finger, instead they hold the sheet between his extended thumb and index fingers, giving an appearance similar to a “tong” rather than a “clamp”. The anterior
interosseous nerve, however, does not contribute as significantly to the power of the pincer grip as do adductor pollicis and dorsal interosseous muscle.

NOTE - Anterior interosseous nerve innervates the following muscles:

Flexor pollicis longus – main flexor of the thumb


Flexor digitorum profundus of digits II and III (index and middle fingers) – flexor of the distal interphalangeal joints of forefinger and middle finger
Pronator quadratus – along with the pronator teres, and while the elbow is flexed to a right angle, turns the hand so that the palm of the hand faces the floor.

th
(Options B and E) Carpal tunnel syndrome is caused by compression of the median nerve in the carpal tunnel, and presents with sensory disturbances in palmar aspect of the thumb, index finger, middle finger and half of the ring (4 ) finger. It also
causes the weakness of the thenar muscles the abductor pollicis brevis, flexor pollicis brevis and opponens pollicis. While thumb opposition can be significantly affected in patients with carpal tunnel syndrome, pinching between thumb and index
fingers are NOT markedly impaired because the key muscles to the action, which are flexor pollicis longus and flexor digitorum profundus muscle of the index finger are supplied by AIN that does not run in the carpal tunnel; therefore, unaffected in
carpal tunnel syndrome.

References

• Medscape - Ulnar Neuropathy

• Radiopedia - Anterior interosseous nerve syndrome

• BMJ - Movements of thumb in relation to peripheral nerve injuries

Time spent: QID:1304 Last updated:


2023-2-12

764 of 1943
A 27-year-old woman, typist by profession, presents to your practice with complaint of right wrist pain and numbness of the index, middle and radial half of the ring fingers. On examination, she is unable to appropriately do the opposition of the
thumb and little fingers on the right side. Which one of the following is the most likely diagnosis?

A. Ulnar nerve lesion.

B. Radial nerve lesion.

C. Carpal tunnel syndrome.

D. Anterior interosseous nerve syndrome.

E. Tendonitis.

Incorrect. Correct answer is C


45% answered correctly

Explanation:

Correct Answer Is C

The clinical presentation is highly suggestive of carpal tunnel syndrome (CTS) as the most likely diagnosis.

CTS refers to a constellation of signs and symptoms caused by compression of the median nerve as it travels through the carpal tunnel. CTS presents with pain and paresthesia, and less commonly weakness in the median nerve distribution in hand.
CTS is the most frequent compressive focal mononeuropathy seen in clinical practice.

Classic CTS is associated with pain or paresthesia (numbness and tingling) in the distribution of the median nerve territory in the hand (the first three digits and the radial half of the fourth digit.) Symptoms are typically worse at night and often
awaken patients. The patients often shake or wring their hands or place them under warm running water to alleviate the symptoms. Symptoms of CTS are usually limited to fingers innervated by the median nerve; however, the pain and paresthesia
may be localized to the wrist or involve the entire hand. It is even possible that the sensory symptoms radiate proximally into the forearm, or less frequently to above the elbow or even the shoulder.

CTS symptoms often worsen with activities involving wrist flexion/extension or arm raising. Some of such activities include typing, reading, driving, and holding a telephone.

In more severe cases, motor involvement follows and results in weakness or clumsiness when using the hands, such as difficulty holding objects, turning keys or doorknobs, buttoning, or opening jar lids. Clinical signs may include weakness of thumb
abduction and opposition, and atrophy of the thenar eminence.

Fixed sensory loss is usually a late finding characterized by a distinctive clinical pattern characterized by involvement of the fingers supplied by the median nerve, while the thenar eminence is spared. This pattern occurs because the sensation of the
palm is supplied by a sensory cutaneous nerve coming off the median nerve proximal to the wrist. This nerve over the carpal tunnel, not through it.

(Option A) Ulnar nerve lesions result in sensory disturbances in the sensory territory of the ulnar nerve. In hand, the ulnar nerve provides sensation of the little finger and ½ of the ring finger, as well as ulnar parts of the palm and dorsum of the hand.
Motor involvement results in decreased ability to abduct/adduct fingers (an action of interosseous muscles) as well as weak pincer grip.

(Option B) Radial nerve lesions, presents with disturbed sensation over the dorsal aspect of the thumb, index finger, middle finger and radial ½ half of the ring finger. Thumb opposition is provided by the action of opponens pollicis that is innervated
by median nerve and is not affected in radial nerve injuries.

(Option D) Anterior interosseous nerve (AIN) syndrome is a rare condition associated with entrapment of the anterior interosseous nerve. AIN syndrome presents with forearm pain, inability to make an ‘O.K. sign’ with the thumb and index finger, and
positive pinch test in which the patient pinches in between the thumb and index finger in a fashion similar to tongs rather than clamps. AIN is a pure motor nerve branching off the median nerve; therefore, its injuries do not cause sensory symptoms.

(Option E) Tendonitis is not associated with sensory disturbances.

References

• Medscape - Carpal Tunnel Syndrome

• UpToDate - Carpal tunnel syndrome: Clinical manifestations and diagnosis

Last updated:
Time spent: QID:1305 2023-2-12

765 of 1943
Lesions of which of the following nerves can lead to wrist drop?

A. Median nerve at the wrist.

B. Median nerve at the elbow.

C. Radial nerve at the wrist.

D. Radial nerve at the elbow.

E. Ulnar nerve at elbow.

Incorrect. Correct answer is D


45% answered correctly

Explanation:

Correct Answer Is D

Wrist extension is mainly the action of extensor carpi radialis longus, extensor carpi radialis brevis, extensor carpi ulnaris, and extensor digitorum, all of which originating from around the lateral epicondyle of the humerus and innervated in the vicinity
of the elbow. Therefore, radial nerve injury at the level of the elbow can be associated with wrist drop.

(Option A) Median nerve injury at the wrist causes the weakness of thenar eminence muscles i.e. flexor pollicis brevis, abductor pollicis brevis and opponens pollicis. These muscles are not involved in wrist extension.

(Option B) Median nerve injury at elbow results in dysfunction of all the muscles innervated by the median nerve and its motor branch the anterior interosseous nerve. These muscles include all muscles in the anterior compartment of the forearm
(except flexor carpi ulnaris that is innervated by the ulnar nerve) as well as muscles of thenar eminence and the 1st and 2nd lumbrical muscles. Wrist extension is not affected by median nerve injuries at elbow.

(Option C) Radial nerve injury at wrist is not associated with any motor function impairment and only causes sensory deficit in the territory or radial nerve that includes the dorsal aspect of the thumb, index and middle fingers, the radial half of the
ring finger, and the dorsal aspect of the first web space.

th
and 5th, wrist flexion (paralysis of the flexor carpi ulnaris muscle and radial deviation when flexion is attempted), the 3rd and 4th lumbrical muscles, interosseous muscles of the
(Option E) Ulnar nerve injury at elbow affects the flexion of fingers 4
hand and muscles in the hypothenar eminence. Sensory deficits in the territory of the ulnar nerve can be another physical finding. None of the wrist extensor muscles are innervated by the ulnar nerve and remain unaffected by ulnar nerve injuries.

References

• Neuroexam.com - Strength of Individual Muscle Groups

Last updated:
Time spent: QID:1306 2023-2-12

766 of 1943
Which one of the following can cause weakness in small muscles of the hand while sparing the thenar eminence?

A. Ulnar nerve injury.

B. Median nerve injury.

C. Radial nerve injury.

D. Anterior interosseous nerve injury.

E. Posterior interosseous nerve injury.

Correct
45% answered correctly

Explanation:

Correct Answer Is A

Of the options, ulnar nerve injury can cause weakness of most intrinsic hand muscles while sparing the thenar eminence.

In hand, ulnar nerve innervates all interosseous muscles, the 3rd and 4th lumbrical muscles, and muscles in the hypothenar eminence. While ulnar nerve injuries affect the function of these muscles, thenar eminence muscles are spared because they
are supplied by the median nerve.

(Option B) Low (distal) median nerve injuries affect the thenar muscles flexor pollicis brevis, abductor pollicis brevis and opponens pollicis.

(Option C) Radial nerve supplies the muscles in the posterior compartment of the forearm, but no small muscle in the hand, thenar, or hypothenar eminence.

nd
(Option D) Anterior interosseous nerve is a purely motor branch of the median nerve and supplies the muscles flexor pollicis longus, flexor digitorum profundus (2 and 3rd fingers), and pronator quadratus, all of which originating from the forearm,
not the hand.

(Option E) Posterior interosseous nerve (PIN) is a branch of the radial nerve that supplies the muscles originating from the posterior compartment of forearm. PIN injuries do not affect the function of any of the small hand muscles, thenar, or
hypothenar eminence.

References

• Medscape - Ulnar Neuropathy

Last updated:
Time spent: QID:1307 2023-2-12

767 of 1943
Which one of the following nerves supplies the interosseous muscles of the hand?

A. Radial nerve.

B. Median nerve.

C. Ulnar nerve.

D. Anterior interosseous nerve.

E. Posterior interosseous nerve.

Incorrect. Correct answer is C


45% answered correctly

Explanation:

Correct Answer Is C

Interosseous muscles are intrinsic hand muscles that abduct (dorsal interosseous muscles) and adduct (palmar interosseous muscles) the fingers. These muscles are innervated by deep branch of the ulnar nerve.

768 of 1943
References

• Orthobullets - Dorsal interossei

• Orthobullets - Palmar interossei

Time spent: QID:1308 Last updated:


2023-2-12

769 of 1943
After a left elbow injury, a 55-year-old man develops weakness of the long flexors of the thumb and index finger. He is also unable to do forearm pronation on the same side. Which one of the nerves is involved?

A. Radial nerve.

B. Median nerve.

C. Anterior interosseous nerve.

D. Posterior interosseous nerve.

E. Ulnar nerve.

Incorrect. Correct answer is C


45% answered correctly

Explanation:

Correct Answer Is C

The clinical presentation is suggestive of right anterior interosseous nerve injury (AIN).

AIN is a purely motor branch of the median nerve that innervates all deep muscles of the anterior compartment of forearm except the ulnar part of the muscle flexor digitorum profundus. AIN branches from median nerve 4cm distal to medial
epicondyle and runs along the front of the interosseous membrane of the forearm in the space between the flexor pollicis longus and flexor digitorum profundus. AIN ends distally in the pronator quadratus and wrist joint.

Muscles innervated by AIN include:

Flexor pollicis longus (the long flexor of thumb)


The radial part of flexor digitorum profundus (flexor of the index and (sometimes) middle fingers)
Pronator quadratus (forearm pronation resulting in palm of the hand facing down)

(Option A) Radial nerve supplies thumb and fingers extensors and forearm supinator muscles. Radial nerve injuries or neuropathies do not cause weakness of fingers flexion or forearm pronation.

(Option B) Median nerve injuries distal to where AIN branches off cause weakness of thenar muscles including flexor pollicis brevis, abductor pollicis brevis and opponens pollicis. With compensatory action of flexor pollicis longus (supplied by the
AIN) and abductor pollicis longus (supplied by the radial nerve) flexion and abduction remain almost unaffected; however, thumb opposition becomes impaired. Median nerve injuries proximal to AIN origin result in dysfunction of all median-
innervated muscles in the forearm and hand.

(Option D) Posterior interosseous nerve innervates the common and deep extensors of the fingers. It branches off from the radial nerve at the radiohumeral joint line and travels down in the posterior compartment of the forearm. Injuries or
neuropathies of this nerve cause extension abnormalities, the presentation in this case scenario.

th
(Option D) With ulnar nerve involvement, there would be weak flexion of the 4 and 5th fingers, weak flexion of the wrist, and sensory disturbances in the territory of the ulnar nerve.

References

• Orthobullets - Anterior interosseous nerve

Last updated:
Time spent: QID:1309 2023-2-12

770 of 1943
A 45-year-old man presents to your clinic complaining of weak right wrist flexion. On examination, small muscles of his right hand are wastes. He is not able to flex his wrist against resistance. A lesion of which of the following nerves could have led
to this presentation?

A. Right ulnar nerve at the wrist.

B. Right median nerve at the wrist.

C. Right median nerve at the elbow.

D. Right ulnar nerve at the elbow.

E. Right anterior interosseous nerve.

Incorrect. Correct answer is D


45% answered correctly

Explanation:

Correct Answer Is D

Most small muscles of the hand including dorsal and palmar interossei, lumbrical muscles of digits 4 and 5, and hypothenar eminence muscles are innervated by the ulnar nerve, the paralysis of which results in wasting of most small hand of the
hand (lumbrical muscles of digits 2 and 3 and muscles of thenar eminence are innervated by the median nerve). Ulnar nerve also supplies flexor carpi ulnaris which is a wrist flexor. Ulnar nerve injuries or neuropathies at the level of the elbow can
cause weakness and wasting of most of the small muscles of the hand and weakness of the wrist flexion.

Another important muscle involved in wrist flexion is flexor carpi radialis that is innervated by the median nerve.

(Option A) Although ulnar nerve injuries at the wrist can affect the function of small muscles of the hand, flexor carpi ulnaris is innervated proximal to the wrist, and ulnar nerve injuries at wrist does not cause weak wrist flexion.

(Option B) Median nerve injuries at the wrist results in weakness of thenar muscles and the first two lumbricals but does not affect the action of flexor carpi radialis because it is innervated proximal to the wrist.

(Option C) With median nerve injuries at the elbow, all of the muscles innervated by the median nerve and its branches including anterior interosseous nerve are affected. However, with the exception of the first two lumbrical muscles and muscles of
the thenar eminence, all other small muscles of the hand remain intact as they are supplied by the ulnar nerve.

(Option E) Anterior interosseous nerve does not innervate any of the small muscles of the hand or main flexors of the wrist. Muscles supplies by the anterior interosseous nerve include flexor pollicis longus, the radial half of flexor digitorum
profundus and pronator quadratus.

References

• Medscape - Hand Anatomy

• Medscape - Ulnar Neuropathy

Last updated:
Time spent: QID:1310 2023-2-12

771 of 1943
Robert is a 72-year-old patient of yours, who was diagnosed with Parkinson disease 10 years ago. Recently he developed agitation, for which he was prescribed haloperidol. Today, he is brought to you office by his son with complaint of marked
increase in his tremors. Which one of the following is the most appropriate management?

A. Do nothing.

B. Increase the dose of haloperidol.

C. Increase the dose of anti-Parkinson medications.

D. Decrease the dose of haloperidol.

E. Switch to risperidone.

Incorrect. Correct answer is D


45% answered correctly

Explanation:

Correct Answer Is D

Haloperidol is a first-generation antipsychotic that acts by inhibition of dopamine receptors in CNS. On the other hand, the pathophysiology of Parkinson disease is dopamine depletion from the basal ganglia. Consequently, haloperidol results in
worsening of extrapyramidal symptoms of Parkinson disease such as bradykinesia and tremors.

All antipsychotic drugs are capable of producing extrapyramidal effects in a dose-dependent fashion. Aripiprazole, clozapine and quetiapine are probably exceptions. When extrapyramidal effects develop, it is an indication that the dose of drug has
exceeded the optimal therapeutic range for a given patient. In such circumstances, the next best step would be decreasing the dose of the antipsychotic.

(Option A) While decreasing the dose of haloperidol is very likely to reduce tremor, doing nothing would be an incorrect option.

(Option B) By increasing the dose of haloperidol, the condition becomes worse.

(Option C) Increasing the dose of anti-Parkinson drugs is not an appropriate option because the condition is caused by high doses of haloperidol and can be reversed by dose reduction.

(Option E) Switching to another antipsychotic with the same side effect is not an appropriate option when the problem is likely to resolve with decreasing the dose of haloperidol.

References

• Therapeutic Guidelines – Psychotropic; available from http://tg.org.au

Last updated:
Time spent: QID:692 2023-2-12

772 of 1943
A 28-year-old man presents with distal weakness and atrophy of the small muscles of both hands including interossei and lumbricals as well as thenar and hypothenar eminences. Which one of the following is the most likely cause of his
presentation?

A. Multiple sclerosis.

B. Bilateral median nerve palsy.

C. Syringomyelia.

D. Bilateral ulnar nerve palsy.

E. Brainstem infarction.

Incorrect. Correct answer is C


45% answered correctly

Explanation:

Correct Answer Is C

This man has bilateral global weakness of distal arms and muscle wasting of both thenar (median-innervated) and hypothenar (ulnar-innervated) eminences. Of the options, syringomyelia best justified the bilaterality of the signs and symptoms, as
well as multiple nerves involvement.

Syringomyelia is the development of a fluid-filled cavity (syrinx) within the spinal cord. Syrinx extension into the anterior horns of the spinal cord damages motor neurons (lower motor neuron) and causes diffuse muscle atrophy that begins in the
hands and progresses proximally to include the forearms and shoulder girdles. Claw-hand may develop.

Syrinx also interrupts the decussating spinothalamic fibers that mediate pain and temperature sensibility, resulting in loss of these sensations, while light touch, vibration, and position senses are preserved because their fibers are located in the
posterior column that are not affected unless late in the course of the disease (dissociated sensory loss). When the cavity enlarges to involve the posterior columns, position and vibration senses are lost as well. The sensory disturbances usually
occur in a shawl-like distribution.

(Option A) Cervical spine multiple sclerosis can cause muscle weakness, exaggerated reflexes and decreased sensation of the upper limb; however, bilateral symptoms of this patient makes multiple sclerosis a less likely diagnosis.

(Option B) Bilateral median nerve palsy can cause bilateral atrophy of thenar eminence but not that of hypothenar eminence.

(Option D) Bilateral ulnar nerve palsy can cause bilateral atrophy of hypothenar eminence but not that of thenar eminence.

(Option E) Brainstem infarction is associated with contralateral limb weakness. Bilaterality is against brainstem infarction as a likely diagnosis.

References

• Medscape - Syringomyalia

Last updated:
Time spent: QID:1311 2023-2-12

773 of 1943
A 5-year-old boy is brought to your practice by his parents for assessment. According to the parents, he has several episodes of blank stare while watching TV every day, which are accompanied by bizarre movements of the head and right hand. The
trunk and head turn before starts fidgeting the fingers of his right hand and smacking his lips. He seems lost during an attack. There are up to 10 episodes almost every day, each lasting about one minute before spontaneous resolution occurs. He
seems confused and drowsy for 1 to 2 minutes after each attack. Which one of the following is the most appropriate treatment option for him?

A. Phenytoin.

B. Sodium valproate.

C. Ethosuximide.

D. Carbamazepine.

E. Phenobarbital.

Incorrect. Correct answer is D


45% answered correctly

Explanation:

Correct Answer Is D

The clinical picture in this child is highly suggestive of temporal lobe epilepsy as the most likely diagnosis. Temporal lobe epilepsy is categorized under complex partial seizures meaning that the seizure has a particular focus in the brain (partial) and
the awareness is impaired during an attack (complex). Temporal lobe epilepsy is characterized by automatism including fidgeting, chewing, lip smacking, fumbling, or complex acts such as speed driving, kissing, violence or other bizarre behaviors.
The duration of an attack is often 1-3 minutes. There is a post-ictal state of 1 to 2 minutes associated with confusion. The patient has no recollection of the events during the attack. Other features include memory disturbances such as deja vu or
jamais vu, hallucinations, emotional disturbances such as a sense of sudden terror, panic, anger or drealization. Ictal or post-ictal aphasia can be a feature. Abdominal rising sensation (abdominal aura) with or without nausea and vomiting may occur.
Patients may have bizarre delusions.

Many antiepileptic drugs (AEDs) have been used successfully for treatment of temporal lobe epilepsy. A many as 47-60% of new cases have been treated with the first drug with adequate clinical response. The three major AEDs, carbamazepine,
phenytoin and sodium valproate have equal efficacy in controlling the seizures; however, of these carbamazepine is the drug of choice because it is associated with fewer side effects and better tolerance by patients.

NOTE - The newer AEDs (not mentioned as an option), such as gabapentin, topiramate, lamotrigine, levetiracetam, oxcarbazepine, pregabalin, lacoasmide, and zonisamide, have similar efficacy than the older AEDs, but they are preferred over
older AEDs because they have far less adverse effects and less drug-drug interactions. If present in the options, they should be selected as the most appropriate treatment option.

Vigabatrin and felbamate are reserved for intractable epilepsy patients due to the potential of serious side effects.

NOTE - Temporal lobectomy is the definitive treatment for medically intractable temporal lobe epilepsy, as it has a high seizure-free rate.

Ethosuximide (option C) is a treatment option for absence seizure and phenobarbital (option E) is not a reasonable option for treatment of temporal lobe epilepsy.

References

• MJA - The management of epilepsy in children and adults

Last updated:
Time spent: QID:1313 2023-2-12

774 of 1943
A 35-year-old woman presents with chief complaint of headaches. She describes that she has been having these headaches for the past 8 months. The headaches occurr more often in the evening hours and she took Panadol, ibuprofen, and
sometimes codeine for pain relief. Recently, the headache that is varying in intensity comes early in the morning and wakes her up. The ache is felt in the frontal area bilaterally and it seems that the her regular pain killers are not that effective with
only a partial relief lasting not more than 2 to 3 hours. She denies nausea and vomiting, light and noise sensitivity, or visual problems. She is otherwise in good shape, smokes 5-10 cigarettes a day, and drinks within healthy limit. Physical
examination is inconclusive. Which one of the following condition is most likely to have led to this presentation?

A. Pseudotumor cerebri.

B. Migraine.

C. Tension headaches.

D. Drug rebound headache.

E. Cerebral tumor.

Incorrect. Correct answer is D


45% answered correctly

Explanation:

Correct Answer Is D

Although early morning headache is characteristic of headaches caused by cerebral tumors (option E) or other space-occupying intracranial lesions, absence of other features such as worsening of the pain on bending forward, sneezing or coughing,
no history projectile vomiting or absence of focal neurological findings makes such diagnoses less likely.

Migraine headache is usually unilateral and felt behind the orbit and/or temporal regions. However, it can progress to affect the entire head and neck. In this patient, the pain is bilateral and other features of migraine such as nausea and vomiting and
sensitivity to noise and light are absent. With these, migraine (option B) is an unlikely diagnosis.

Tension headache (option C) is very common. The pain is bilateral and described as band-like pressure. Nausea and vomiting are not prominent features and patient is not sensitive to light and noise. One important characteristic of tension headache
is the fact that it occurs mainly towards the end of the day (evening hours). Adequate response to analgesics is another distinctive feature of tension headaches.

Although the headache in this woman is bilateral and not associated with nausea and vomiting and resembles tension headache, the time of onset (early in the morning) and partial and inadequate response to analgesia makes this diagnosis less
likely for this woman.

Idiopathic intracranial hypertension (IIH), also known as pseudotumor cerebri, is a disorder of unknown etiology. It can occur in all age groups but is most common in obese women of childbearing age. The primary problem in IIH is chronically
elevated intracranial pressure (ICP). With increased ICP, the most important neurological finding is papilledema that can result in progressive optic atrophy, visual loss, and potentially blindness. In general and as the name implies, sign and symptoms
of IIH are similar to any space-occupying brain lesion such as a cerebral tumor, including:

Headache (typically nonspecific and varying in type, location, intensity, and frequency)
Nausea and vomiting
dizziness
Visual loss (typically visual field but rarely visual acuity loss)
Brief episodes of blindness lasting only a few seconds and affecting one or both eye (visual obscuration)
Diplopia (typically horizontal due to non-localizing sixth nerve palsy but rarely vertical)
Pulsatile tinnitus – ringing in the ears that pulses in time with the heartbeat
Pain in the neck, shoulders or arms (not very common)

With just headache in the history and absence of other presenting features of pseudotumor cerebri, such diagnosis is unlikely.

As a matter of fact, with prolonged use of analgesics for chronic headache and absence of alarming findings in the history and physical examinations, this clinical scenario is most consistent with drug rebound headache, also known as medication
overuse headache. Classically, a typical patient is described as a 30-60 years old woman with a history of more than a decade of tension or migraine headache and pain killer use for relief; however, it may affect patients of different ages with a
history of infrequent analgesic use for headache even as short as days or weeks. Therefore, a high index of suspicion should be held for every patient presenting with frequent headaches. The characteristic features of medication overuse headache
is not uniform but often is similar to quality of pain in the original headache, for which medications are taken. The condition is more prevalent in patient who use prophylaxis for their headaches. In General, medication overuse headache occurs every
day or almost every day, often waking up the patient early in the morning when the effect of the painkillers has worn off overnight. The pain is partially improved with medications but returns as the medication wears off again.

Medication overuse headache is not associated with focal or lateralizing neurological symptoms. It is, however, common for patients with migraine with aura to experience aura if they develop medication overuse headaches. It is important to note
that if the patient’s symptoms have been stable months or years, imaging or neurological examination is not required provided that there is no neurological findings in between headaches.

References

• Australian Prescriber - Medication overuse headache

Last updated:
Time spent: QID:1319 2023-2-12

775 of 1943
A 30-year-old woman presents to your GP clinic with complaint of headaches 3 to 5 days before her periods. The headache often starts from the back of her head (occipital area) and radiates to her left temple, and is aggravated by walking or other
types of physical activity. She denies visual, auditory, or taste disturbances, but mentions that during attacks she cannot tolerate noisy or crowded places. According to her, every episode of headache lasts from two to several hours. Physical
examination, including a full neurological exam is inconclusive. Which one of the following could be the most likely diagnosis?

A. Migraine with aura.

B. Migraine without aura.

C. Tension vascular headache.

D. Cerebral tumor.

E. Premenstrual tension headache.

Incorrect. Correct answer is B


45% answered correctly

Explanation:

Correct Answer Is B

The description of headache in the patient’s words implies a unilateral nature for the pain (from occipital area radiating to the left temple). The headache is aggravated by walking and physical activity and the patient is intolerant of noise and crowded
places. These, collectively, make migraine the most likely diagnosis. She also denies visual, taste, or auditory disturbances (aura). So she is very likely to be suffering from migraine without aura. Relation to menstruation (hormonal changes as a
trigger) is also another supporting evidence for such diagnosis.

Migraine is very common and affects four of every ten women and two of every ten men in their lifetime, mostly before the age of 35 years. Over 50% of women with migraine report an association between migraine and menstruation. Such
association has been hypothesized to be due to hormonal changes around menstruation.

(Option A) Absence of visual, taste or auditory disturbances makes migraine with aura (classic migraine) an unlikely diagnosis.

(Option C) Tension-vascular headache is a term used for headaches that cannot be categorized as tension-type headache or migraine headache due to overlapping symptoms. In such patients both migraine and tension headaches are present.
Findings in this woman are nearly classic for migraine without aura.

The following tables outline differences between tension headache and migraine:

Tension headache Migraine


Site of pain Bilateral (bitemporal, frontal or occipital), can be generalized Unilateral – often felt retro-orbital or temporal. It can progress to affect
the entire head and neck
Quality of pain Constant, tight or pressure Pulsating or throbbing
Associated features Can be associated with sleep disturbances Sensory or motor features or aura in migraine with aura
No sensitivity to light, noise or smells Sensitivity to light (photophobia), sensitivity to noise (phonophobia),
There might be nausea but vomiting is rare sensitivity to smells (osmophobia)
Nausea and vomiting are very common (80% and 50% respectively)

Aggravating factors Stress (often at the time of stress) Often when stress has passed
Certain foods (cheese, chocolate, wine) or hunger
Hormonal changes (e.g. premenstrual)

(Option D) Headache caused by space-occupying brain lesions such as cerebral tumors characteristically occur early in the morning and are associated with forceful (projectile) vomiting and focal neurological findings, absence of which in the
scenario make a space-occupying lesion as the cause of the headaches less likely. Also, headache associated with such lesions do not occur just prior to menstruation and in a cyclic fashion.

(Option E) Although this woman has headaches before menstruation, the features of headache are more consistent with migraine rather than tension headache.

References

• Medscape - Migraine Headache

• Menstrual Migraine: Therapeutic Approaches

• RACGP - AFP - Management of chronic headache

Last updated:
Time spent: QID:1320 2023-2-12

776 of 1943
Joan, 60 years of age, is in the Emergency Department with complaint of acute onset of a spinning sensation at home followed by severe nausea and vomiting. She has never had a similar episode. She also denies any recent history of upper
respiratory tract infections. On examination, she has a blood pressure of 164/104 mmHg, heart rate of 103 bpm and regular, respiratory rate of 17 breaths per minute and temperature of 37.0°C. Remarkable exam findings include nystagmus,
decreased hearing of the left ear, left-sided Horner syndrome, ipsilateral sensory loss of the face and contralateral sensory deficits of the limbs. No motor dysfunction of the limbs is noted but ataxia is present. Based on the clinical picture, a
diagnosis of stroke is made. Which one of the following arteries is most likely to be occluded?

A. Anterior communicating artery.

B. Posterior cerebral artery.

C. Anterior cerebral artery.

D. Posterior inferior cerebellar artery.

E. Internal carotid artery.

Incorrect. Correct answer is D


45% answered correctly

Explanation:

Correct Answer Is D

The clinical findings of vertigo, decreased hearing, Horner syndrome, and the pattern of loss of sensation in the face and limbs (ipsilateral and contralateral respectively) is mostly consistent with lateral medullary syndrome, also known as Wallenberg
syndrome.

This syndrome is most often due to vertebral artery and less commonly to posterior inferior cerebellar artery (PICA) occlusion. Patients with the lateral medullary syndrome present with nausea, vomiting, and vertigo from involvement of the
vestibular system. Other findings may include:

Ipsilateral:

Ataxia and dysmetria, due to damage to the inferior cerebellar peduncle and cerebellum
Horner syndrome (ptosis, miosis, hypohidrosis or anhydrosis), due to damage to descending sympathetic fibers
Loss of facial pain and temperature sensation
Reduced corneal reflex due to damage to the descending spinal tract and nucleus of CN V
Nystagmus
Hypoacusis (involvement of cochlear nucleus)
Dysarthria
Dysphagia
Paralysis of the pharynx, palate, and vocal cord
Loss of taste from the posterior third of the tongue (nuclei or fibers of CN IX and X)

Contralateral:

Loss of pain and temperature sense in the body and extremities, indicating involvement of the lateral spinothalamic tract.

Other findings include tachycardia and dyspnea (dorsal nucleus of CN X) and palatal myoclonus, which is a rhythmic involuntary jerking movement of the soft palate, pharyngeal muscles, and diaphragm. Palatal myoclonus sometimes follows
infarction of the dentate nucleus of the cerebellum and inferior oliva.

(Option A) Anterior communicating artery is an artery in the circle of Willis that connects the right and left anterior cerebral arteries. Aneurysms of this artery are the most common circle of Willis aneurysms. The ischemic strokes due to occlusion of
this artery are not that common but if they occur, presentation will be similar to that of anterior cerebral artery strokes with personality changes, motor weakness, and sensory disturbances which are more prominent in the lower limbs.

(Option B) The posterior cerebral arteries (PCAs) are paired arteries, usually branching from the top of the basilar artery. They curve laterally, posteriorly, and superiorly around the midbrain. The PCAs supply parts of the midbrain, subthalamic
nucleus, basal nucleus, thalamus, mesial inferior temporal lobe, and occipital and occipitoparietal cortices. In addition, the PCAs, via the posterior communicating arteries (PCOM), may become important sources of collateral circulation for the
middle cerebral artery (MCA) territory.

Patients with PCA stroke/ TIA often present with the following symptoms:

Acute vision loss


Confusion
New onset posterior cranium headache
Paresthesia
Limb weakness
Dizziness
Nausea
Memory loss
Language dysfunction

(Option C) Anterior cerebral artery (ACA) supplies mostly the frontal lobes and parasagittal areas. Strokes caused by ACA stroke/ TIA may present with lower limb weakness and sensory disturbances, personality changes (inappropriate behavior),
and apraxia and aphasia. The scenario is inconsistent with ACA strokes.

(Option E) With internal carotid occlusion, the presentation is a combination of signs and symptoms seen in both anterior and middle cerebral arteries PLUS vision loss (i.e., amaurosis fugax).

References

• Medscape - Vertebrobasilar Stroke

Last updated:
Time spent: QID:1345 2023-2-12

777 of 1943
A 45-year-old man presents to the Emergency Department with complaints of acute-onset vertigo, right eyelid drooping, and numbness of the lower right half of the face. He also complains of blurred vision and decreased hearing in the right ear. On
examination, he has a blood pressure of 176/95 mmHg, heart rate of 94 bpm, respiratory rate of 20 breaths per minute, temperature of 36.8°C, and oxygen saturation of 96% on room air. There is right-sided miosis, ptosis and anhidrosis. He has
nystagmus and ataxic movements and sensory loss of the left upper and lower limbs. Rinne and Weber tests show sensorineural deafness of the right ear. Which one of the following could be the most likely cause of this constellation of symptoms?

A. Right-sided Horner syndrome.

B. Left vertebral artery thrombosis.

C. Right vertebral artery thrombosis.

D. Obstruction of the anterior communicating artery.

E. Obstruction of the posterior cerebral artery.

Incorrect. Correct answer is C


45% answered correctly

Explanation:

Correct Answer Is C

Constellation of clinical findings in this patient, including vertigo, right Horner syndrome indicated by miosis, ptosis and anhidrosis, right-sided sensorineural hearing loss, blurred vision, and numbness of the right side of the face and left side of the
body is highly suggestive of right-sided vertebrobasilar stroke.

The vertebral arteries arise from the subclavian arteries, and as they course cephalad in the neck, they pass through the costotransverse foramina of C6 to C2. They enter the skull through the foramen magnum and merge at the pontomedullary
junction to form the basilar artery. Each vertebral artery usually gives off the posterior inferior cerebellar artery (PICA). At the top of the pons, the basilar artery divides into 2 posterior cerebral arteries (PCAs).

The vertebrobasilar arterial system provides blood supply to the medulla, cerebellum, pons, midbrain, thalamus, and occipital cortex. Occlusion of large vessels in this system usually leads to major disability or death (mortality rate>85%). Because of
involvement of the brainstem and cerebellum, most survivors have multisystem dysfunction such as quadriplegia or hemiplegia, ataxia, dysphagia, dysarthria, gaze abnormalities, and cranial neuropathies. Fortunately, many vertebrobasilar lesions
arise from small vessels and are small and discrete. The clinical correlates of these smaller lesions consist of a variety of focal neurologic deficits, depending on their location within the brainstem. Patients with small lesions usually have a benign
prognosis with reasonable functional recovery.

Vertebrobasilar strokes have distinct characteristic features that differentiate them from hemispheric strokes caused by lesions of anterior or middle cerebral arteries or internal carotid artery. These features include:

When cranial nerves or their nuclei are involved, the corresponding clinical signs are ipsilateral to the lesion and the corticospinal signs are crossed, involving the opposite arm and leg.

Cerebellar signs (e.g. dysmetria, ataxia) are frequently observed.

Involvement of the ascending sensory pathways may affect the spinothalamic pathway or the medial lemniscus (dorsal columns), resulting in dissociated sensory loss, which is loss of one sensory modality on one side and
preservation of other sensory modalities in the opposite limbs (dissociative sensory loss).

Dysarthria and dysphagia are typically present.

Vertigo, nausea, and vomiting, along with nystagmus, represent involvement of the vestibular system and are seen in vertebrobasilar strokes.

Unilateral Horner syndrome occurs with brainstem lesions.

Occipital lobe lesions result in visual field loss or visuospatial deficits.

Cortical deficits, such as aphasia and cognitive impairments, are absent

(Option A) Horner syndrome is a constellation of signs (miosis, ptosis, and anhidrosis) and a feature of vertebrobasilar stroke. It does not explain this constellation of symptoms.

(Option B) With left vertebral artery lesions, left-sided Horner syndrome, numbness of the left half of the face and that of the right limbs and left-sided hearing loss would have been expected.

(Option D) Anterior communicating artery is an artery in the circle of Willis that connects right and left anterior cerebral arteries. Aneurysms of this artery are the most common circle of Willis aneurysms. However, ischemic strokes due to occlusion
of this artery are not that common, but if they occur presentation will be similar to that of anterior cerebral artery strokes with personality changes, motor weakness and sensory disturbances more prominent in the lower limbs.

(Option E) The posterior cerebral arteries (PCAs) are paired arteries, usually branching from the top of the basilar artery. They curving laterally, posteriorly, and superiorly around the midbrain. The PCAs supply parts of the midbrain, subthalamic
nucleus, basal nucleus, thalamus, mesial inferior temporal lobe, and occipital and occipitoparietal cortices. In addition, the PCAs, via the posterior communicating arteries (PCOM), may become important source of collateral circulation for the middle
cerebral artery (MCA) territory.

Although posterior cerebral artery is a branch of the basilar artery from the vertebrobasilar system, lesions at this rather terminal branch are associated with a narrower range of symptoms, mostly including:

Acute vision loss


Confusion
New onset posterior cranium headache
Paresthesia
Limb weakness
Dizziness
Nausea
Memory loss
Language dysfunction

With the presence of vertigo, Horner syndrome, hearing loss and ataxia indicating lesions of cerebellum and medulla, the lesion is expected to be more proximal to the PCA.

References

• Medscape - Vertebrobasilar Stroke

• Medscape - Anterior Circulation Stroke

Last updated:
Time spent: QID:1347 2023-2-12

778 of 1943
A 56-year-old man is brought to the Emergency Department with presenting complaint of vertigo. He describes that he had mild dizziness and nausea before he hears a popping sound in his right ear. The dizziness intensified afterwards, and he felt a
severe spinning sensation associated with nausea and two episodes of vomiting. On examination, he has a blood pressure of 145/105 mmHg, pulse rate of 100 bpm, respiratory rate of 18 breaths per minute, and a temperature of 36.7°C. He also
has right-sided Horner’s syndrome, complete sensorineural deafness of the right ear and nystagmus. There is paresthesia of the left upper and lower limbs. Which one of the following would be the most likely cause of this clinical presentation?

A. Meniere’s disease.

B. Acoustic neuroma.

C. Acute labyrinthitis.

D. Vertebrobasilar insufficiency.

E. Vestibular neuronitis.

Incorrect. Correct answer is D


45% answered correctly

Explanation:

Correct Answer Is D

With vertigo, Horner’s syndrome and sensorineural deafness and nystagmus on the right side after a popping sound in the ear (which indicates sudden onset tinnitus), a central cause for this presentation should be thought of first. Considering the
fact that this patient has also contralateral (left-sided) sensory impairment, vertebrobasilar insufficiency is the most likely explanation.

The vertebrobasilar arterial system provides blood supply to the medulla, cerebellum, pons, midbrain, thalamus, and occipital cortex. Occlusion of large vessels in this system often leads to major disability or death (mortality rate>85%). Fortunately,
many vertebrobasilar lesions arise from small vessels and are small and discrete. The clinical correlates of these smaller lesions consist of a variety of focal neurologic deficits, depending on their location within the brainstem.

Characteristic features of vertebrobasilar insufficiency/ stroke include the following:

Vertigo, nausea, and vomiting, along with nystagmus, represent involvement of the vestibular system, and are seen in vertebrobasilar strokes.
Unilateral Horner syndrome occurs with brainstem lesions.
Cerebellar signs (e.g., dysmetria, ataxia) are frequently observed.
Dysarthria and dysphagia are typically present.
When cranial nerves or their nuclei are involved, the corresponding clinical signs are ipsilateral to the lesion and the corticospinal signs are crossed, involving the opposite arm and leg.
Involvement of the ascending sensory pathways may affect the spinothalamic pathway or the medial lemniscus (dorsal column), resulting in dissociated sensory loss, which is loss of one sensory modality on one side and preservation
of other sensory modalities in the opposite limbs (dissociative sensory loss).
Occipital lobe lesions result in visual field loss or visuospatial deficits.
Cortical deficits, such as aphasia and cognitive impairments, are absent.

In the absence of visual problems, this patient is more likely to have developed occlusion of the left inferior cerebellar artery (PICA) occlusion. The following diagram depicts the vertebrobasilar circulation:

PICA is the largest branch of the vertebral artery and is one of the three main arteries that supply the cerebellum (the other two are superior cerebellar artery and anterior inferior cerebellar artery). Occlusion of PICA (A) results in a clinical syndrome
called lateral medullary syndrome, Wallenberg syndrome or PICA syndrome.

This syndrome is characterized by sensory deficits that affect the trunk and extremities contralaterally (opposite to the lesion), and sensory deficits of the face and cranial nerves ipsilaterally (same side as the lesion). The cross body finding is the
chief symptom leading to PICA syndrome as the diagnosis.

Other possible symptoms include:

Difficulty walking or maintaining balance (ataxia) or difference in temperature sensation between left and right sides of the body
Attacks of vertigo and nystagmus
Difficulty swallowing (dysphagia) caused by the involvement of the nucleus ambiguus (this nucleus supplies the vagus and glossopharyngeal nerves).
Dysarthria and dysphonia
Ataxia (imbalance on walking) due to damage to the cerebellum or the inferior cerebellar peduncle
Horner’s syndrome caused by damage to hypothalamospinal fibers and disruption of sympathetic nervous system

779 of 1943
Palatal myoclonus
Hoarseness
Nausea and vomiting
Bradycardia and dysregulation of blood pressure (due to injuries of nucleus ambiguous)

(Option A) Meniere’s disease is caused by abnormal accumulation of endolymph in the labyrinth, and presents with vertigo, tinnitus and hearing loss lasting for hours each time. Patients often mention a sensation of fullness in the ear. There is often
a positive family history. In Meniere’s disease, the pathology is contained within the labyrinth, and there is no neurological abnormalities such as Horner’s syndrome or sensory deficits of the body otherwise.

(Option B) Acoustic neuromas are intracranial tumors arising from the Schwann cell sheath of either the vestibular or cochlear nerve. As acoustic neuromas increase in size, they eventually occupy a large portion of the cerebellopontine angle.
Acoustic neuromas account for approximately 80% of tumors found within the cerebellopontine angle. The remaining 20% are principally meningiomas.

Unilateral hearing loss is the most common symptom present at the time of diagnosis and is generally the symptom that leads to diagnosis. The tumor can produce hearing loss through at least two mechanisms: (1) direct injury to the cochlear nerve
or (2) interruption of cochlear blood supply. Progressive injury to cochlear fibers probably accounts for slow progressive neurosensory hearing loss observed in majority of patients with acoustic neuromas. Sudden and fluctuating hearing loss is seen
in 5-15% of patients, probably due to disruption of cochlear blood supply by the tumor.

Other symptoms of acoustic neuroma include headaches (50-60% of patients at the time of diagnosis, but in less than 10% as the presenting symptom) and facial numbness (in 25% of patients but in less than 1% as the presenting symptom).

Vertigo and disequilibrium are uncommon presenting symptoms among patients with acoustic tumors. Overall, approximately 40-50% of patients with an acoustic neuroma might have some balance disturbance. However, balance disturbance is the
presenting symptom in less than 10% of patients. This is due to the fact that destruction of vestibular fibers by the tumor is slow and allows for compensation.

Sudden onset of vertigo and hearing loss and contralateral sensory disturbances are strongly against acoustic neuroma as the diagnosis. In majority of patients with acoustic neuroma, the presenting symptom is progressive unilateral hearing loss.

(Option C) Acute labyrinthitis presents with acute vertigo often followed by nausea and vomiting, tinnitus, and hearing loss. There is often a history of preceding viral upper respiratory tract infection. Change in head position provokes vertigo. Each
episode of vertigo lasts from few seconds to minutes. Although there are similar symptoms of vertigo and hearing loss between labyrinthitis and vertebrobasilar insufficiency, the former is not associate with Horner’s syndrome and abnormal
neurological findings such as sensory impairment.

(Option E) Vestibular neuronitis is the inflammation of the vestibular nerve, often by a viral infection. Patients may have a preceding viral upper respiratory infection or herpes zoster. Vertigo and imbalance are the prominent features of vestibular
neuronitis and there is no hearing loss or tinnitus. Loss of balance is more prominent in vestibular neuronitis compared to other causes of vertigo, and patient commonly presents with vertigo and falls. Similar to benign paroxysmal positional vertigo
(BPPV), symptoms in vestibular neuronitis are aggravated by change in the position of the head. Neurological examination in patients with vestibular neuronitis is otherwise normal and there is no Horner’s syndrome.

References

• Medscape - Vertebrobasilar Stroke

• RACGP – An approach to vertigo in general practice

• Medscape – Acoustic neuroma

Time spent: QID:1371 Last updated:


2023-2-12

780 of 1943
A 54-year-old woman presents to the emergency department with complaints of sudden onset vertigo, nausea, vomiting, and hearing loss in her left ear. On examination, her vital signs are within normal ranges. Hearing is decreased on the left side
on whisper test. Rinne and Weber tests establish sensorineural deafness of the left ear. She has also nystagmus with the rapid eye to the left side. The rest of the examination is inconclusive. Which one of the following could be the most likely
diagnosis?

A. Labyrinthitis.

B. Vestibular neuronitis.

C. Acoustic neuroma.

D. Meniere’s disease.

E. Lateral medullary syndrome.

Correct
45% answered correctly

Explanation:

Correct Answer Is A

Of the options, labyrinthitis, Meniere’s disease and lateral medullary syndrome can cause acute onset vertigo, tinnitus and hearing loss. Ataxia can be a presentation in patients with cerebellar or vestibular disease. Of these three, and given the
inconclusive neurological examination, labyrinthitis is the most likely diagnosis.

Acute labyrinthitis presents with acute vertigo often followed by nausea and vomiting, tinnitus, and hearing loss. A history of preceding viral upper respiratory tract infection is present in up to 50% of patients. Change in head position provokes
vertigo. Each episode of vertigo lasts from few seconds to minutes.

Meniere’s disease (option D) also presents with episodes of acute onset vertigo, tinnitus and hearing loss. However, patients with Meniere’s disease often complain of ear fullness because the pathophysiology is excess endolymph in the labyrinth.
Patients are usually middle-aged women with a positive family history for the condition. Finally, Meniere’s disease is much less common compared to labyrinthitis. Given these, Meniere’s disease in this patient is a less likely diagnosis compared to
labyrinthitis.

(Option B) Vestibular neuronitis is the inflammation of the vestibular nerve often by a viral infection. Patients usually have a preceding viral upper respiratory infection or herpes zoster. Vertigo and imbalance are the prominent features of vestibular
neuronitis and there is no hearing loss or tinnitus. Loss of balance is more prominent in vestibular neuronitis compared to other causes of vertigo, and patient commonly present with vertigo and falls. Symptoms in vestibular neuronitis are
aggravated by change in the position of the head. Neurological examination in patients with vestibular neuronitis is otherwise normal.

(Option C) Acoustic neuromas are intracranial tumors that arise from the Schwann cell sheath of either the vestibular or cochlear nerve. As acoustic neuromas increase in size, they eventually occupy a large portion of the cerebellopontine angle.
Although 5-15% of patients with acoustic neuroma present with acute onset of unilateral hearing loss, deafness has an insidious onset in this condition, making it a less likely diagnosis. Gradual hearing loss is overwhelmingly the most common
presenting symptom of patients with acoustic neuroma. Imbalance and vertigo is not a prominent feature because as the tumor growth disrupts the vestibular nerve function slowly, there is enough time for compensation. Other features that may be
present in patients with acoustic neuroma are headache and facial sensory impairment.

(Option E) Lateral medullary syndrome, also known as Wallenberg syndrome or posterior inferior cerebellar artery (PICA) syndrome has other clinical features in addition to vertigo, hearing loss, and tinnitus. Such features include cross-body sensory
impairment (sensory impairment of the face on the affected side and that of the body on the other side), Horner’s syndrome, and signs and symptoms indicative of the involvement of cranial nerves or their nucleus. Such signs and symptoms may
include dysphagia (due to involvement of nucleus ambiguus that supplies the vagus and glossopharyngeal nerves), dysarthria, dysphonia, disrupted temperature and pain sensation, palatal clonus and heart rate and blood pressure dysregulation (due
to involvement of the vagus nerve).

References

• RACGP – An approach to vertigo in general practice

• Medscape – Acoustic neuroma

• Medscape - Vertebrobasilar Stroke

Last updated:
Time spent: QID:1373 2023-2-12

781 of 1943
A 22-year-old man presents to your GP clinic for assessment. On waking up this morning, he felt pain in his left ear, associated with severe dizziness and loss of balance. He describes that the dizziness was felt as spinning sensation and giddiness
followed by nausea and vomiting. The symptoms significantly improve when he lies down. He denies any hearing loss or ringing in the ears. Physical examination reveals normal vital signs. There is horizontal nystagmus with the rapid component to
the right side. Caloric test shows markedly reduced response on the left side. The rest of the exam, including a full assessment of cranial nerves, is unremarkable. Which one of the following could be the most likely diagnosis?

A. Meniere’s disease.

B. Vestibular neuronitis.

C. Vertebrobasilar ischemia.

D. Paroxysmal positional vertigo.

E. Acute labyrinthitis.

Incorrect. Correct answer is B


45% answered correctly

Explanation:

Correct Answer Is B

Acute onset of vertigo, nausea and vomiting, and being unable to keep balance in the absence of hearing loss and/or tinnitus favors either BPPV or vestibular neuronitis as the most likely etiologies. Spontaneous horizontal nystagmus with the fast
part towards the unaffected site and a reduced caloric test of the left side (see TOPIC REVIEW), on the other hand, strongly suggests vestibular neuronitis as the most likely diagnosis. Another clue to vestibular neuronitis is the significant imbalance
in this patient. Although all causes of vertigo can give rise to imbalance, this feature is more prominent and disabling in vestibular neuronitis compared to other causes of vertigo such as labyrinthitis, BPPV, or Meniere’s disease.

Vestibular neuronitis is the inflammation of the vestibular nerve often by a viral infection. Approximately, 50% of Patients have a preceding viral upper respiratory infection or herpes zoster infection. Vertigo and imbalance are the prominent features
of vestibular neuronitis, and there is no hearing loss or tinnitus. Loss of balance is more prominent in vestibular neuronitis compared to other causes of vertigo, and patients commonly present with vertigo and falls. Symptoms in vestibular neuronitis
are aggravated by change in head position. Neurological examination in these patients is otherwise normal.

Spontaneous, unidirectional, horizontal nystagmus is the most important physical finding. Fast phase oscillations beat toward the healthy ear. Nystagmus may be positional and apparent only when gazing toward the healthy ear, or during Hallpike
maneuvers. Patients may suppress their nystagmus by visual fixation (a differentiating feature in nystagmus caused by central causes). Patient tends to fall toward his or her affected side when attempting ambulation or during Romberg tests. The
affected side has impaired or no response to caloric stimulation.

Patients with BPPV (option D) usually wake up with the condition (like in this patient) and notice the vertigo while trying to sit up suddenly. Thereafter, propensity for positional vertigo may last for days to weeks. In many, the symptoms periodically
wax and wane.

The physical examination findings in patients affected by BPPV are generally unremarkable. All neurotologic examination findings except those from the Dix-Hallpike maneuver is normal.

The Dix-Hallpike maneuver (see TOPIC REVIEW) is the standard clinical test for BPPV. The finding of classic rotatory nystagmus with latency and limited duration is considered pathognomonic. A negative test result is meaningless except to indicate
that active canalithiasis is not present at that moment.

(Option A) Meniere’s disease presents with episodes of acute onset vertigo, tinnitus, and hearing loss. Patients often complain of ear fullness. Absence of tinnitus and hearing loss in this patient make Meniere’s disease an unlikely diagnosis.

(Option C) It is very unusual for a vertebrobasilar ischemic episode to present with vertigo as the sole symptom. There are always other clinical features in addition to vertigo such as hearing loss/ tinnitus, cross-body sensory impairment, Horner’s
syndrome, and signs and symptoms indicative of the involvement of cranial nerves or their nucleus. Such signs and symptoms may include dysphagia (due to involvement of nucleus ambiguus that supplies the vagus and glossopharyngeal nerves),
dysarthria, dysphonia, disrupted temperature and pain sensation, palatal clonus and heart rate and blood pressure dysregulation (due to involvement of the vagus nerve).

(Option E) Acute labyrinthitis presents with acute vertigo often followed by nausea and vomiting, tinnitus, and hearing loss. Absence of hearing loss and tinnitus in this patient is against such diagnosis.

TOPIC REVIEW

Caloric test (vestibuocular reflex test)

Caloric reflex test also termed as vestibular caloric stimulation is a test of the vestibule-ocular reflex. Ice cold or warm water or air is irrigated into the external auditory canal, usually using a syringe. The temperature difference between the body and
the injected water creates a convective current in the endolymph of the nearby horizontal semicircular canal. In patients with an intact brain stem, hot and cold water produce currents in opposite directions and therefore a horizontal nystagmus in
opposite directions.

If the water is warm (44 °C or above) endolymph in the ipsilateral horizontal canal rises, causing an increased rate of firing in the vestibular afferent nerve. This situation mimics a head turn to the ipsilateral side. Both eyes will turn toward the
contralateral ear, with horizontal nystagmus (quick horizontal eye movements) to the ipsilateral ear.

If the water is cold, relative to body temperature (30 °C or below), the endolymph falls within the semicircular canal, decreasing the rate of vestibular afferent firing. This situation mimics a head turn to the contralateral side. The eye then turn toward
the ipsilateral ear, with horizontal nystagmus to the contralateral ear.

Absent reactive eye movement suggests vestibular weakness of the horizontal semicircular canal of the side being stimulated.

One mnemonic used to remember the FAST direction of nystagmus is COWS:

COWS: Cold Opposite, Warm Same.


Cold water: FAST phase of nystagmus to the Opposite side from the cold water filled ear
Warm water: FAST phase of nystagmus to the Same side as the warm water filled ear

782 of 1943
Dix-Hallpike test (Maneuver)

Dix – Hallpike maneuver is performed on all patients who complain of vertigo but do not exhibit nystagmus on routine examination of the extraocular muscles.

Hallpike maneuver requires patient to lie back from sitting to supine position 3 times. The first time, have the patient lie back with the head facing forward and the neck slightly extended; repeat this movement with the patient's head turned 45
degrees to the right and a third time with the head turned 45 degrees to the left.

Instruct patient to keep both eyes open each time he or she lies back.

Check for nystagmus and ask patient about any symptoms of vertigo.

Among the characteristics of an elicited nystagmus that would suggest disease of peripheral origin are a pause before nystagmus appears (latency), unidirectional nystagmus, and fatiguing of nystagmus after approximately 1 minute or repeated
inductions.

Failure either to observe or to provoke unidirectional nystagmus casts doubt on whether the process is localized to the peripheral vestibular system. Either finding suggests a need to consider other diagnostic alternatives.

References

783 of 1943
• RACGP – An approach to vertigo in general practice

• Medscape – Benign Paroxysmal Positional Vertigo

• Medscape – Vestibular Neuronitis


Last updated:
Time spent: QID:1375 2023-2-12

784 of 1943
A 55-year-old man presents to your GP clinic with complaints of tinnitus and hearing loss in his right ear. According to him, he suddenly felt a severe spinning sensation associated with ringing in and hearing loss of his right ear while working. He felt
intensely nauseous but did not vomit. He could not stand so he lied down in bed. After five hours, the spinning partially improved and only has been coming on with head movements, but he still has the ringing ear and deafness. Except for the
sensorineural deafness of the right ear and nystagmus on ocular movements, physical examination, including a full neurological exam, is unremarkable. Which one of the following could be the most likely diagnosis?

A. Labyrinthitis.

B. Vestibular neuronitis.

C. Acoustic neuroma.

D. Benign paroxysmal positional vertigo.

E. Meniere’s disease.

Correct
45% answered correctly

Explanation:

Correct Answer Is A

The clinical picture is a classic description of acute labyrinthitis. Acute labyrinthitis is characterized by sudden, unilateral loss of vestibular function and hearing. The acute onset of severe, often incapacitating, vertigo, frequently associated with
nausea and vomiting, is characteristic of this condition. The patient is often bedridden while the symptoms gradually subside. Each episode of vertigo lasts from seconds to minutes. Vertigo eventually resolves after several days to weeks; however,
unsteadiness and positional vertigo may persist for several months. Hearing loss and tinnitus is common and may be the primary presenting symptom in many patients. Up to 50% of patients with acute labyrinthitis report a preceding viral upper
respiratory tract infection.

Vestibular neuronitis (option B) and benign paroxysmal positional vertigo (BPPV) (option D) also present with vertigo and imbalance; however, tinnitus and hearing loss are not features seen in these conditions.

(Option C) Acoustic neuromas are intracranial tumors that arise from the Schwann cell sheath of either the vestibular or cochlear nerve. As acoustic neuromas increase in size, they eventually occupy a large portion of the cerebellopontine angle.
Acoustic neuromas account for approximately 80% of tumors found within the cerebellopontine angle.

Although 5-15% of patients with acoustic neuroma present with acute onset of unilateral hearing loss, deafness has an insidious onset in most cases. This fact, makes acoustic neuroma a less likely diagnosis for this patient who has presented with
acute hearing loss. Gradual hearing loss is overwhelmingly the most common presenting symptom of patients with acoustic neuroma. Imbalance and vertigo is not a prominent feature because as the tumor disrupts the vestibular nerve function
slowly, there is enough time for compensation. Other features that may be present in patients with acoustic neuroma are headache and facial sensory impairment.

(Option E) Meniere’s disease can also cause vertigo, tinnitus and hearing loss; however, it is an uncommon cause of vertigo. Furthermore, it is more common in middle-aged women. Family history is often positive in patients with Meniere’s disease.

References

• Medscape – Labyrinthitis

• RACGP – An approach to vertigo in general practice

Last updated:
Time spent: QID:1377 2023-2-12

785 of 1943
A 67-year-old man is brought to the Emergency Department by his wife after he suddenly blacked out in the bathroom while urinating and regained consciousness in few seconds. He is diabetic and on antihyperglycemic medications. On assessment,
there is no remarkable point except urinary symptoms of hesitation and terminal dribbling. Which one of the following could be the most likely cause of this presentation?

A. Epilepsy.

B. Vasovagal syncope.

C. Micturition syncope.

D. Hypoglycemia.

E. Transient ischemic attack (TIA).

Incorrect. Correct answer is C


45% answered correctly

Explanation:

Correct Answer Is C

The scenario describes an episode of syncope. Syncope is defined as an episode of sudden loss of consciousness and postural tone with spontaneous recovery. Presyncope refers to symptoms of faintness or lightheadedness without loss of
consciousness. Any condition that leads to loss of oxygen or glucose to the brain can lead to syncope. Almost always, syncope results from loss of cerebral blood flow. Even syncope secondary to hyperventilation results from hypocapnia-induced
cerebral vasoconstriction.

Although other conditions such as epilepsy or cerebral trauma can also cause sudden loss of consciousness and postural tone with spontaneous recovery, in practice the term syncope is reserved for transient reduction in cerebral blood flow, while
other causes are considered ‘syncope mimics’.

Reflex syncope, also termed as neurally-mediated or neurocardiogenic syncope, accounts for at least one-third of syncopal episodes, and is the most common cause of syncope. As these terms imply, the nervous system rather than cardiac
problems is the root cause of the problem. Orthostatic syncope is the second most common cause of syncope.

Reflex syncope includes:

Vasovagal syncope (common faint)


Carotid sinus hypersensitivity
Situational syncope i.e., cough syncope, micturition syncope, swallowing syncope and defecation syncope
Receptor syncope

With urinary symptoms of difficulty starting urination and terminal dribbling, micturition syncope is the most likely diagnosis. Micturition syncope or post-micturition syncope is fainting shortly after or during urination.

Micturition syncope is commonly seen in elderly men with a large prostate that prevents normal urine flow. When the patient strains to overcome the obstruction, intraabdominal pressure rises and leads to increased vagal tone and consequent
bradycardia, drop in blood pressure, and cerebral hypoperfusion.

Micturition syncope is categorized under the entity ‘situational syncope’ which belongs to ‘reflex-mediated’ causes of syncope.

Situational syncope is an uncommon cause of syncope related to specific situations such as urinating, coughing or swallowing. The mechanism through which syncope is triggered may or may not differ from that of vasovagal syncope. Cough
syncope is postulated to be caused be decreased blood flow to the brain due to increased intrathoracic pressure during bouts of coughing, while micturition syncope is caused by vasovagal hyperstimulation.

(Option A) Epilepsy is the term used for seizures for which no apparent cause is found.

(Option B) Although micturition syncope is caused by vasovagal response, it is independently categorized as ‘situational’ rather than ‘vasovagal’ syncope.

(Option D) In fainting due to hypoglycemia, the regain of consciousness is not quick. Furthermore, there is usually presyncopal prodrome.

(Option E) TIA is associated with focal neurological deficits rather than syncope.

TOPIC REVIEW

Causes of syncope (including ‘syncope mimics’):

Reflex-mediated (most common)

Vasovagal syncope (common faint)


Carotid sinus hypersensitivity
Situational syncope (cough, micturition, swallowing, defecation)
Receptor stimulation (cranial nerve irritation, bladder distension, airway stimulation, neck tumor)

Orthostatic hypotension (second most common)

Drug induced
Hypovolemia/hemorrhage
Deconditioning
Autonomic dysfunction: primary (pure autonomic failure, multiple system atrophy, etc.), secondary (diabetic, paraneoplastic neuropathies, amyloidosis)

Cardiac (electrical)

Bradycardias (sinus node dysfunction or AV-conduction-system disease)


Tachycardias (supraventricular or ventricular)

786 of 1943
Cardiac (mechanical)

Valvular disease: aortic, mitral or pulmonary stenosis


Hypertrophic obstructive cardiomyopathy
Aortic dissection
Pulmonary embolus
Pericardial disease/cardiac tamponade
Primary pulmonary hypertension
Cardiac tumors
Critical myocardial ischemia

Non-cardiac (predominantly syncope mimics)

Neurological/cerebrovascular: seizure, migraine, subarachnoid hemorrhage, subclavian steal syndrome, vertebrobasilar insufficiency
Metabolic disturbances: hypoglycemia or hypoxemia, pheochromocytoma, Addison’s disease, drug intoxication including alcohol, hyperventilation (via hypocapnia)

Psychogenic pseudo-syncope

In up to one-third of cases, no diagnosis is made despite thorough investigation. It is very likely that most of these undiagnosed cases are due to reflex-mediated syncope or orthostatic hypotension.

References

• MJA - Postural syncope: mechanisms and management

Time spent: QID:703 Last updated:


2023-2-12

787 of 1943
A 21-year-old woman is in your GP clinic with complaint of recurrent episodes of vertigo. She describes that she has had these episode every month for the past five years. The vertigo is followed by imbalance and sometimes nausea and vomiting.
She denies any hearing impairment or ringing in the ear, or ear fullness during the attacks. She is otherwise healthy and does not drink alcohol or smoke. Physical examination is unremarkable. Which one of the following would be the most
appropriate management for her?

A. Refer her to ENT specialist.

B. Reassure her that this is a benign condition.

C. Tempanostomy.

D. Prescribe decongestants.

E. MRI of the posterior fossa.

Incorrect. Correct answer is B


45% answered correctly

Explanation:

Correct Answer Is B

Recurrent acute attacks of vertigo associated with nausea and vomiting without tinnitus or hearing impairment in a young otherwise healthy young woman is most likely due to benign paroxysmal positional vertigo (BPPV). Although BPPV is more
common in middle-aged people, it can affect any age group.

BPPV is the most common cause of vertigo in clinical practice. The condition is caused by an accumulation of calcium crystals in the posterior semicircular canal. These crystals affect the movement of the endolymph in the semicircular canals and
causes vertigo. The classic symptoms of BPPV are brief episodes of vertigo (often lasting for few seconds), associated with nausea and nystagmus. There is no hearing impairment or tinnitus.

Episodes of vertigo are triggered by rapid changes in the position of the head. Symptoms may persist for weeks and recur after remission. Nystagmus seen in BPPV has a rotational nature. The Dix−Hallpike maneuver is used to diagnose BPPV. The
Epley manoeuvre can be used to treat it.

As the name implies, BPPV is a benign condition and the patient can be reassured. Treatment of BPPV is with Epley maneuver, vestibular rehabilitation and watchful waiting.

(Option A) Referral to ENT specialist is not required at this stage as the diagnosis is almost certain and no further action other than simple measures is required.

(Option D) Although antihistamines such as betahistine can provide modest relief in patients with BPPV and are often prescribed for short term use, they are not the mainstay of the treatment and do not have a significnt role in treatment of BPPV.

(Option C) Tempanostomy is used for relief of pressure in the middle ear. This procedure is not useful for treatment of BPPV, the etiology of which lies withing the inner ear.

(Option E) With a diagnosis of BPPV as a peripheral cause of the vertigo in this patient, MRI or other imaging studies are not indicated. Imaging studies are often considered when a central nervous system lesion is suspected.

References

• Medscape – Benign Paroxysmal Positional Vertigo

• RACGP – An approach to vertigo in general practice

Last updated:
Time spent: QID:1379 2023-2-12

788 of 1943
A 30-year-old woman presents for evaluation of vertigo of three months duration. She also complains of constant ringing of her ears. She is very fond of music and has her earphones on through the day. She has no significant past medical or
surgical history, is not on any regular medications, and otherwise enjoys a healthy life. Her family history is remarkable for hearing impairment in one of her maternal aunts. Whispering test shows reduced hearing in the left ear. Which one of the
following is the most appropriate option to reach a diagnosis?

A. Rinne and Weber tests.

B. Speech discrimination test.

C. Electrocochleography.

D. Pure tone audiometry.

E. MRI.

Incorrect. Correct answer is D


45% answered correctly

Explanation:

Correct Answer Is D

For every patient presenting with suspected hearing loss, tinnitus, vertigo and other ear symptoms, an audiogram is indicated to evaluate the condition, especially if a clear diagnosis of a benign self-limiting condition such as vestibular neuronitis,
acute labyrinthitis, or benign paroxysmal positional vertigo (BPPV) cannot be made based on the history and physical examination alone. Audiometry is also useful for screening for hearing loss in people regularly exposed to loud noises, and for
certain patients on ototoxic medications (e.g., gentamicin).

Pure tone audiometry is the standard test to assess hearing loss in adult patients. It allows for independent thresholds to be determined in each ear, for air conduction (conductive hearing) and bone conduction (sensorineural hearing). Air conduction
measures the ability of the external and middle ear to transmit sound to the cochlea. Conductive hearing loss (CHL) results from any barriers that inhibit sound transmission along this pathway, which may include cerumen impaction, middle ear fluid,
or a tympanic membrane perforation. On the audiogram, CHL is depicted as an air-bone gap. Sensorineural hearing loss (SNHL) is defined by equal air and bone conduction thresholds higher than 25 Db. Tympanometry can complement the pure tone
audiogram by assessing the compliance and mobility of the middle ear system.

(Option A) Rinne and Weber test are office tests for a provisional assessment of hearing loss. In Rinne test bone and air conductions are compared. A vibrating tuning fork is placed over the mastoid bone and the patient is asked to signal the
examiner once he/she cannot hear the vibration. On the signal, the examiner brings the fork in front of the ear and asks the patient if he/she could still hear the vibration. In a normal person air conduction (AC) is better than (BC); therefore, the
vibration should still be heard when the fork is brought to in front of the ear. If not, BC is better than AC in that ear and CHL is suspected.

In Weber test, a vibrating tuning fork is placed in the middle of the patient’s forehead. In CHL the sound is lateralized to the affected ear (louder in the affected ear) while in SNHL the sound is louder in unaffected ear.

Although these tests can assess the hearing loss and differentiate CHL from SNHL to some extend, they are not very reliable compared to formal audiometry because for example in patients with bilateral and equal SNHL Rinne and Weber tests
could be completely normal as no lateralization occurs. None of these tests replace the need for pure tone audiometry for assessment of CHL versus SNHL and the extent of impairment.

(Option B) Speech discrimination test is performed as a part of formal audiometry. Speech discrimination scores are a measure of the patient’s ability to hear words correctly and are a good indication of the integrity of the cochlear nerve. There is a
predictable attenuation with a conductive or a cochlear loss, and if there is an acoustic neuroma the speech discrimination is much worse than expected from the pure tone audiogram. Improvement in speech discrimination scores with increased
intensity is also a good way of predicting if a patient will benefit from a hearing aid or other amplification.

(Option C) Electrocochleography is an objective measure of the electrical potentials generated in the inner ear as a result of sound stimulation. This test is most often used to determine if the cochlea has an excessive amount of fluid pressure.
Excessive fluid pressure in the cochlea can cause symptoms such as hearing loss, aural fullness, dizziness, and/or tinnitus. These symptoms are sometimes indicative of certain ear pathologies such as Meniere’s disease or endolymphatic hydrops.
This test might be later considered for this patient if her symptoms are suspected to have been caused by such pathology.

(Option E) Imaging studies such as magnetic resonance imaging (MRI), computed tomography (CT) or magnetic resonance angiography (MRA) in patients with vertigo and/or hearing loss are indicated if:

the examination is not consistent with a peripheral lesion


prominent risk factors for CVA are present
neurological signs and symptoms are present, or
symptoms of vertigo are accompanied by a headache

At this stage, this patient does not have any of the above to necessitate MRI.

References

• RACGP – Audiology

• RACGP – An approach to vertigo in general practice

• RACGP – Diagnosing and management of hearing loss in elderly patients

Last updated:
Time spent: QID:1381 2023-2-12

789 of 1943
Ben is in your clinic for assessment. He is 60 years old, and at several occasions in the past month has had vertigo, right sided hearing loss and double vision as well as numbness of his left limbs. These episodes last for up to 5-10 minutes and
resolve spontaneously. He smokes 10-15 cigarettes and drinks 2-3 standard drinks every day. His past medical or surgical history is insignificant and he is not on any regular medication. He also denies any recent flu-like symptoms. His family history
is remarkable for Meniere’s disease in his mother. Which one of the following could be the most likely diagnosis?

A. Multiple sclerosis.

B. Vertebrobasilar insufficiency.

C. Meniere’s disease.

D. Labyrinthitis.

E. Meningioma.

Incorrect. Correct answer is B


45% answered correctly

Explanation:

Correct Answer Is B

Episodes of vertigo, right-sided hearing loss, double vision and most importantly simultaneous numbness of the left limbs are highly suggestive of vertebrobasilar insufficiency as the most likely diagnosis.

Vertebrobasilar circulation supplies the medulla, cerebellum, pons, midbrain, thalamus, and occipital cortex.

Characteristic features of vertebrobasilar insufficiency/ stroke include the following:

Vertigo, nausea, and vomiting, along with nystagmus, represent involvement of the vestibular system and are seen in vertebrobasilar strokes.
Unilateral Horner syndrome occurs with brainstem lesions.
Cerebellar signs (e.g., dysmetria, ataxia) are frequently observed.
Dysarthria and dysphagia are typically present.
When cranial nerves or their nuclei are involved, the corresponding clinical signs are ipsilateral to the lesion and the corticospinal signs are crossed, involving the opposite arm and leg.
Involvement of the ascending sensory pathways may affect the spinothalamic pathway or the medial lemniscus (dorsal columns), resulting in dissociated sensory loss, which is loss of one sensory modality on one side and
preservation of other sensory modalities in the opposite limbs (dissociative sensory loss).
Occipital lobe lesions result in visual field loss or visuospatial deficits.
Cortical deficits, such as aphasia and cognitive impairments, are absent.

(Option A) Multiple sclerosis (MS) can cause a variety of neurological symptoms; however, it is unusual for MS to affect the same part of the nervous system with each relapse. MS relapses characteristically are described as CNS demyelinating
lesions that are separate in time and location.

With MS, there is involvement of one particular part of the CNS and the corresponding symptomatology at one time that completely or partially resolves and involvement of another part of the CNS with next relapse. Furthermore, symptoms last for
days to weeks before they improve. This clinical scenario is not consistent with MS.

(Option B) Meniere’s disease present with recurrent episodes of hearing loss, tinnitus, and vertigo as well as a sense of ear fullness that last for hours. Additionally, there are no other focal neurological findings such as numbness because the
pathology is within the inner ear (increased endolymph pressure in labyrinth).

(Option D) Labyrinthitis causes vertigo, hearing loss and tinnitus in episodes lasting from seconds to minutes. However, there is no focal neurological findings in labyrinthitis.

(Option E) Meningiomas are non-cancerous CNS tumors that arise from meningeal lining within the cranium or spinal canal. The symptoms are caused by irritation of the underlying brain or spinal tissue and/or compression effect. Symptoms have
an insidious onset and the presentation depends on the specific location they arise from within the CNS. Headache is a common feature in intracranial meningiomas.

Of the cerebellopontine angle tumors, almost 20% are meningiomas (80% are acoustic neuromas). Unilateral gradual sensorineural deafness is overwhelmingly the cardinal presenting symptom. However, imbalance, vertigo, or facial numbness can
be the presenting feature in a minority of patients. Limb paresthesia is not a presentation. More importantly, patients with intracranial tumors have continuous symptoms that progress with time. The presence of this patient’s symptom in short
periods and being asymptomatic in between is strongly against meningioma as the diagnosis.

References

• Medscape – Vertebrobasilar Atherothrombotic Disease

Last updated:
Time spent: QID:1383 2023-2-12

790 of 1943
A 30-year-old woman presents to your general practice with complaint of blurred vision and pain in the right eye for the past 3 days. She is otherwise healthy and has not had any major medical or surgical history. On examination, her visual acuity of
the right and left eyes are 6/12 and 6/6, respectively. She cannot read the numbers in Ishihara chart and confuses red and green. Other significant finding on examination is worsening of the right eye pain with eye movements that are normal and
symmetric in all directions with no diplopia. Which one of the following could be the most likely diagnosis?

A. Intracranial hemorrhage.

B. Orbital tumor.

C. Optic neuritis.

D. Temporal arteritis.

E. Retinal detachment.

Incorrect. Correct answer is C


45% answered correctly

Explanation:

Correct Answer Is C

Sudden onset of decreased vision and retro-orbital pain in a young female is highly suggestive of optic neuritis as the most likely diagnosis.

Optic neuritis (ON) is a demyelinating inflammation of the optic nerve that often occurs in association with multiple sclerosis (MS) and neuromyelitis optica (NMO). A gradual recovery of visual acuity with time is characteristic of ON; however,
permanent residual deficits in color vision and contrast and brightness sensitivity are common.

Typically, patients with first time acute optic neuritis (ON) are otherwise healthy young adults, who experience rapidly developing impairment of vision in one eye or, less commonly, both eyes during an acute attack. Dyschromatopsia (change in color
perception) in the affected eye occasionally may be more prominent than the decreased vision. Decreased color vision is tested with Ishihara chart shown in the following picture. Patients with impaired color vision cannot read the numbers in the
chart due to color blindness.

791 of 1943
In almost all cases, the visual changes are associated with a retro-orbital or ocular pain, usually aggravated by eye movement. The pain may precede the visual loss.

Patients may complain of vision loss exacerbated by heat or exercise (Uhthoff phenomenon). Objects moving in a straight line may appear to have a curved trajectory (Pulfrich phenomenon), presumably due to asymmetrical conduction between the
optic nerves.

Physical examination findings in patient with ON may include:

Decreased visual acuity


Decreased color vision
Presence of relative afferent pupillary defect
Reduction in subjective brightness
Visual field defect
Abnormal contrast sensitivity
Certain appearances of the optic nerve (swelling, peripapillary hemorrhage or pallor)

(Option A) Intracranial hemorrhage is not likely to present with isolated findings of reduced vision and retro-orbital pain worsening on eye movement. In patients with intracranial hemorrhage, more significant motor and/or sensory disturbances
would be expected.

(Option B) An orbital tumor presents differently with probable findings of eye bulging, decreased vision, impaired visual fields, and manifestations of raised intraocular pressure. It is less likely that an intraorbital tumor is missed on funduscopic
examination in experienced hands. Vision loss and retro-orbital pain are unlikely to be the sole manifestations of an eye tumor.

NOTE – Uveal melanoma is the most common intraocular malignancy in adult.

(Option D) Temporal arteritis is a vasculitis of large vessels of middle-age onset and characterized by a variety of symptoms including headache, scalp tenderness, jaw claudication and vision loss. Polymyalgia rheurmatica can be an association.
Neither the patient’s age, nor the clinical presentation is consistent with temporal arteritis as the diagnosis.

(Option E) Although retinal detachment is associated with vision loss, absence of other features like flashes and floaters in the visual fields and the presence of pain makes such diagnosis unlikely.

References

• Medscape – Adult Optic Neuritis

• RACGP (AFP) – Optic neuritis

Time spent: QID:1429 Last updated:


2023-2-12

792 of 1943
A 38-year-old woman presents with complaints of decreased visual acuity and retro-orbital pain worse on eye movements on the left side for the past four days. On examination, there is vision loss, color vision in particular, and painful eye
movements. Which one of the following is the most appropriate next investigation to consider for her?

A. Temporal artery biopsy.

B. MRI.

C. CT scan of the head.

D. Visual evoked potentials.

E. Gonioscopy.

Incorrect. Correct answer is B


45% answered correctly

Explanation:

Correct Answer Is B

Clinical manifestations of decreased visual acuity, especially the color vision, and retro-orbital pain worse on eye movements are highly suggestive of optic neuritis (ON) as the most likely diagnosis.

ON is acute inflammation of the optic nerve, and the most commonly optic neuropathy encountered in general practice. Demyelination associated with multiple sclerosis (MS) is the most common cause of ON in adults. Other causes include
infections, parainfections (infection source elsewhere other than the optic nerve) and rheumatologic diseases.

Decreased vision, especially color vision, and retro-orbital pain aggravated by eye movements are the cardinal symptoms of ON. On physical examination of the eye, the following features can be present:

Decreased visual acuity


Decreased color vision
Presence of relative afferent pupillary defect
Reduction in subjective brightness
Visual field defect
Abnormal contrast sensitivity, and
Certain appearances of the optic nerve (swelling, peripapillary hemorrhage or pallor)

For patients with suspected ON, magnetic resonance imaging (MRI) is the most appropriate investigation to consider. MRI is highly sensitive and specific in assessing inflammatory changes in the optic nerves and helps to rule out structural lesions.
In addition, MRI may have a value in predicting future development of MS in patients presenting with first-time, acute optic neuritis. Patients with optic neuritis, who also have demyelinating brain lesions on MRI, are more likely to develop MS in the
future. MRI should be considered as the most important initial investigation for all patients presenting with clinical manifestations of optic neuritis.

Visual evoked potentials (VEP) (option D) is another important test to consider, especially if the diagnosis is uncertain. Patients with ON or MS will have a delay in latency with preserved waveforms in conventional VEP. In ON, VEP can be abnormal
when MRI is negative and inconclusive. VEP can confer the highest diagnostic yield in patients with suspected ON and is often performed in such patients. However, MRI is always the first diagnostic modality to consider.

(Option A) Temporal artery biopsy is the definitive diagnostic test for temporal arteritis (giant cell arteritis). Temporal arteritis presents with scalp tenderness, jaw claudication and vision loss (due to involvement of the retinal branch). The condition
usually occurs in older patients (>50 years). This patient is young and does not have other features of temporal arteritis; hence, temporal artery biopsy is not necessary for her.

(Option C) CT scan of the head has no diagnostic value for suspected ON.

(Option E) Gonioscopy is the examination of the anterior chamber, most commonly for assessment of the anterior chamber angle. It adds no diagnostic benefit for patients with ON.

References

• RACGP (AFP) – Optic neuritis

• Medscape – Adult optic neuritis

Last updated:
Time spent: QID:1431 2023-2-12

793 of 1943
A 35-year-old woman presents with complaints of decreased vision and pain of her right eye for the past three days. Eye examination reveals decreased visual acuity of the right eye and retro-orbital pain worse on eye movements. Which one of the
following investigations is most likely to establish a diagnosis for her?

A. Temporal artery biopsy.

B. Lumbar Puncture (LP).

C. CT scan of the head.

D. Visual evoked potentials.

E. Fundoscopy.

Incorrect. Correct answer is D


45% answered correctly

Explanation:

Correct Answer Is D

Sudden-onset decreased vision and retro-orbital pain in a young female is highly suggestive of optic neuritis (ON) as the most likely diagnosis.

For patients with suspected ON, MRI of the brain is always the most initial investigation to consider. MRI is highly sensitive and specific in assessing inflammatory changes in the optic nerve and helps to rule out structural lesions. In addition, MRI
may have a value in predicting future development of multiple sclerosis (MS) in patients presenting with first-time acute optic neuritis. Patients with optic neuritis, who also have demyelinating brain lesions on MRI, are more likely to develop MS in the
future. MRI should be considered as the most important initial investigation for all patients presenting with clinical manifestations of optic neuritis. However, when it comes to diagnostic accuracy, visual evoked potentials (VEP) can give a more
sensitive diagnostic yield compared to MRI. In ON, VEP can be abnormal when MRI is negative and inconclusive. Patients with ON or MS will have a delay in latency with preserved waveforms in conventional VEP.

(Option A) Temporal artery biopsy is the definite diagnostic test for patients with suspected temporal arteritis. Temporal arteritis presents with scalp tenderness, jaw claudication, and vision loss (due to involvement of the retinal branch). The
condition usually occurs in older patients (>50 years). This patient is young and does not have other features of temporal arteritis; hence, temporal artery biopsy is not necessary for her.

(Option B) Lumbar puncture (LP) is now used less frequently and its use is controversial. However, cerebrospinal fluid (CSF) examination is useful in patients with normal or atypical brain MRI. The typical finding is the presence of oligoclonal bands
(69% of patients with ON).

(Option C) CT scan of the head has no diagnostic value in patients with suspected ON.

(Option E) In almost 65% of patients, the pattern of ON is retrobulbar; therefore, no abnormality is noted on fundoscopic examination. In the remaining 35% of patients with anterior ON (involvement of the optic nerve head), optic nerve swelling or
papillitis can be seen. Other possible findings in these patients are retinal exudates or peripapillary hemorrhage. Given the low sensitivity of funduscopy in ON, it cannot be relied on for diagnosis.

References

• RACGP (AFP) – Optic neuritis

• Medscape – Adult optic neuritis

Last updated:
Time spent: QID:1435 2023-2-12

794 of 1943
A 26-year-old woman presents with history of recurrent visual problems. She describes that she frequently experiences seeing glimmering flashes in her vision for about 10 to 15 minutes each time which are completely resolved every time. She also
has experiencing global headaches which worsens on exertion. On examination, visual acuity, visual fields, eye movements and pupillary reflexes are normal. Fundoscopic examination is inconclusive. There are no focal neurological deficits on
central nervous system examination either. Which one of the following could be the most likely diagnosis?

A. Focal epilepsy.

B. Occipital lobe tumor.

C. Optic neuritis.

D. Migraine with aura.

E. Carotid artery stenosis.

Incorrect. Correct answer is D


45% answered correctly

Explanation:

Correct Answer Is D

The history of visual symptoms lasting 10-15 minutes with complete resolution and headache aggravated by exercise are consistent with migraine with aura as the most likely diagnosis among the options. Although migraine headache often tends to
be unilateral first, some patients may have generalized (global) headache. Significant pieces of information in the history suggesting migraine with aura as the probable diagnosis are the presence of visual disturbances in form of glimmering flashes
indicating aura and exacerbation of the headache with exertion. Exertion can trigger and/or aggravate migraine headache.

In migraine, auras most commonly consist of visual symptoms, which may be negative or positive. Negative symptoms include negative scotoma or negative visual phenomena, such as the following:

Homonymous hemianopic or quadrantic field defects


Central scotomas
Tunnel vision
Altitudinal visual defects
Complete blindness

The most common positive visual phenomenon is the scintillating scotoma. This consists of an arc or band of absent vision with a shimmering or glittering zigzag border. The disturbance begins in the paracentral area, and gradually enlarges and
moves across the hemifield, eventually breaking up and resolving. It is often combined with photopsias (uniform flashes of light) or visual hallucinations, which may take various shapes.

Other forms of aura include paresthesias (the next most common form of aura and seen in 40% of patients) and motor symptoms. The typical pattern is numbness that starts at hands, migrates to the arm, and then involves the face, lips, and tongue.

Motor symptoms may occur in 18% of patients and usually are associated with sensory symptoms. Motor symptoms often are described as a sense of heaviness of the limbs before a headache but without any true weakness. Speech and language
disturbances have been reported in 17–20% of patients. These disturbances are commonly associated with upper extremity heaviness or weakness.

NOTE – Auras may or may not be followed by the headache.

The headache often is initially unilateral and localized in the frontotemporal and ocular area, but pain can be felt anywhere around the head or neck. The pain typically builds up over a period of 1–2 hours, progressing posteriorly and becoming global.
The headache typically lasts from 4–72 hours. Among females, more than two thirds of patients report attacks lasting longer than 24 hours.

Pain intensity is moderate to severe and intensifies with movement or physical activity. Many patients prefer to lie quietly in a dark room. The pain usually subsides gradually within a day and after a period of sleep. Most patients report feeling tired
and weak after the attack.

(Option A) Focal epilepsies can present with a variety of symptoms. Visual auras such as flashes, glimmering lights, etc. are common aural phenomena. With the focus of the epilepsy in the visual cortex, visual disturbances or hallucinations can be
the presentation of the epilepsy itself and not just the aura. However, a headache that worsens with exertion more favors the diagnosis of migraine with aura rather than focal epilepsy. Moreover, migraine attacks are more common than focal
epilepsies in terms of commonality.

(Option B) In brain tumors, the neurological findings are usually fixed and not intermittent. Although occipital lobe tumor can cause flashes, the intermittent nature of the flashes in this patients makes an occipital tumor a less likely explanation.

(Option C) Decreased visual acuity, impairment in color perception, and retro-orbital pain that worsens with eye movements are typical clinical presentations in optic neuritis (ON). Patients with ON may also experience diminished perception of
contrast, visual field deficits and flashes in the eye. Intermittent flashes and headaches that are worse with exertion. Intact visual acuity and visual fields, as well as no painful eye movements in this patient are against ON as the diagnosis.

(Option E) The most common visual disturbance in patients with carotid artery stenosis is amaurosis fugax, which is transient unilateral vision loss. Amaurosis fugax is described by the patients as a curtain coming down in front of their eye. While
flashes can be a symptom seen in patients with vertebrobasilar insufficiency, it is not a feature in carotid artery stenosis.

References

• Medscape – Migraine Headache

• RACGP – Diagnosing headache

Last updated:
Time spent: QID:1439 2023-2-12

795 of 1943
A 65-year-old man presents to a GP clinic for what he believe is vertigo. At work, when he stood up from a sitting position, he felt dizzy and nauseated. He feels ok now and does not have the symptoms anymore. He is a known case of diabetes and
ischemic heart disease, and is currently on valsartan, metoprolol, atorvastatin, metformin and glimepiride. On physical examination, his vital signs are within normal limits and the rest of the examination including a full neurological exam is
unremarkable. Laboratory tests are only significant for a random serum glucose level of 11 mmol/L and HbA1C level of 7%. An ECG is obtained which is normal. Which one of the following is the most likely diagnosis?

A. Stroke.

B. Myocardial infarction.

C. Diabetes mellitus.

D. Drug-induced hypotension,

E. Ear problems.

Incorrect. Correct answer is D


45% answered correctly

Explanation:

Correct Answer Is D

Vertigo is a common, distressing presentation in general practice and constitutes approximately 54% of cases presenting with dizziness. Vertigo in a sensation of movement of the environment around the patient. Often, patients describe a ‘spinning’
sensation of either their body or their surroundings. This sensation can be confused with dizziness, which is a non-specific term; therefore, adequate history is required to differentiate dizziness from true vertigo.

Dizziness can be classified into four categories:

1. Vertigo (spinning sensation)


2. Disequilibrium (feeling of imbalance)
3. Light-headedness (sensation of giddiness)
4. Presyncope (faint).

In this man, what he believes was vertigo has occurred after a change in position from sitting to standing. On the other hand, he is on the blood pressure lowering agents valsartan (angiotensin receptor blocker [ABR]) and metoprolol (beta blocker).
With all these in the history and the fact that there are no other symptoms at present, this patient is very likely to have experienced light-headedness or presyncope due to a sudden drop in his blood pressure while standing up from a sitting position
because of his antihypertensive medications.

For this patient, blood pressure should be recorded in both supine and standing positions to see if a postural drop in hypotension (orthostatic hypotension) follows when he stands from supine position. A drop of at least 20 mmHg in systolic or 10
mmHg in diastolic blood pressure confirms this diagnosis with high certainty.

Common symptoms of orthostatic hypotension can include dizziness, lightheadedness, blurred vision, weakness, fatigue, nausea, palpitations, and headache. Syncope, dyspnea, chest pain and neck and shoulder pain are less common symptoms.

(Option A) With stroke, the patient should have focal neurological abnormalities on physical examination. There is no limb weakness, paresthesia, clinical features of involvement of cranial nerves or any other indicators of stroke.

(Option B) Myocardial infarction presents with chest pain as the cardinal symptom. Other symptoms may include shortness of breath and lightheadedness due to impaired cardiac output. Although myocardial infarction (MI) can be painless in
patients with diabetes mellitus, absence of chest pain in history and the normal ECG exclude this diagnosis.

(Option C) Longstanding diabetes can result in autonomic dysregulation. Patients with autonomic dysregulation from diabetes experience symptoms related to heart rate and blood pressure and bowel motility abnormalities. These symptoms may
include bradycardia, tachycardia, orthostatic hypotension, diarrhea or constipation.

Although diabetes can result in impaired blood pressure regulation independently, in this patient and in the presence of blood pressure lowering agents, hypotension from medications seems more likely as the cause of such episode. Autonomic
dysregulation from the diabetes, if present, can act as a predisposing factor.

(Option E) Inner ear problems can cause true vertigo. Benign paroxysmal positional vertigo, vestibular neuronitis, labyrinthitis and Meniere’s disease are diseases originating from inner ear that can cause vertigo. Episodes of vertigo in vestibular
neuronitis and labyrinthitis last from seconds to minutes. Vestibular neuronitis is associated with severe and almost debilitating spinning sensation. The history of dizziness for a few seconds with complete resolution and no residual symptoms is
against vestibular neuronitis. Absence of hearing impairment and ringing ears (tinnitus) strongly excludes labyrinthitis and Meniere’s disease. BPPV can be a possibility as it presents with acute episode of vertigo often followed by nausea and intact
hearing with every episode of vertigo lasting only few seconds. However, given the drug history and the fact that the dizziness has occurred just once and was not present afterwards or reproduced during the physical examination, this diagnosis is
less likely compared to drug-induced postural hypotension.

NOTE – If in history, the dizziness was described as true vertigo (rotation or spinning sensation), BPPV would be a better diagnosis because postural hypotension cannot cause a spinning sensation.

References

• Australian Prescriber – Cardiovascular drugs in older people

• RACGP – An approach to vertigo in general practice

Last updated:
Time spent: QID:1506 2023-2-12

796 of 1943
A 40-year-old man presents to your office with complaints of vertigo and shaky hands every morning after he wakes up. His medical history is significant for long-standing heavy alcohol drinking. When you ask him about the nature of what he
describes as vertigo, he says: “like I am floating, and my head is light”. He denies any sensation of spinning or moving around. Which one of the following could be the most likely diagnosis?

A. Temporal lobe epilepsy.

B. Anxiety.

C. Cerebellar degeneration.

D. Benign paroxysmal positional vertigo (BPPV).

E. Optic neuritis.

Incorrect. Correct answer is B


45% answered correctly

Explanation:

Correct Answer Is B

One of the chief complaints of this patient, is dizziness. It is very important to differentiate between dizziness or lightheadedness (pseudovertigo) and true vertigo.

Vertigo is caused by a variety of etiologies, widely categorized as peripheral and central. While central vertigo is caused by a central nervous system (CNS) pathology, peripheral vertigo is a presentation of the dysfunction of the peripheral component
of the balance system such as vestibular nerve, labyrinths, and utricle and saccule. The majority of vertigoes are have a peripheral etiology.

This patient mentions dizziness but denies any spinning or movement sensation. This indicates that he has dizziness rather than true vertigo. Of the options, only anxiety can justify such presentation. This patient is alcohol dependent and is likely to
be experiencing withdrawal symptoms including anxiety after waking up in the morning. Anxiety can cause dizziness due to hyperventilation and also hand tremors.

(Option A) Temporal lobe epilepsy is characterized by a preceding aura (now called focal aware) including auditory, olfactory or gustatory illusions, hot-cold sensation, and somatosensory, vestibular or visual disturbances. Other feature of focal
aware include heart rate regulation abnormalities (tachycardia, bradycardia r asystole), pallor, piloerection, and flushing, or gastrointestinal disturbances (e.g. the sense of a rising of or presence of butterflies in the stomach as explained by patients).
Cognitive/ psychic phenomena (e.g., Déjà vu or jamais vu, dissociation, depersonalization or derealization, forced thinking, and aphasia/dysphasia), mood and affect lability (e.g., agitation, aggression, anger, anxiety, fear, paranoia, pleasure, crying
or laughing) may be present. This aural phase is followed by motionless stare, dilated pupils, and behavioral arrest. During an attack there can be eye blinking, alimentary, manual or unilateral dystonic limb posturing. There is often a postictal period
with confusion, aphasia, or amnesia. This patient is unlikely to be suffering from temporal lobe epilepsy.

(Option C) Cerebellar degeneration associated with alcohol misuse mainly presents with ataxia. Almost all patients have gait abnormalities which is typically characterized as weakness, unsteadiness, or incoordination in legs. Later, a minority of
patients may develop incoordination and tremor in the upper arm, dysarthria, and intermittent diplopia, or blurred vision. Vertigo, tinnitus, and hearing loss have not been reported in patients with alcohol-related cerebellar degeneration. With dizziness
and hand tremors in the morning as the only complaints and in the absence of gait abnormalities, this patient is unlikely to have cerebellar degeneration.

(Option D) BPPV is characterized by spells of vertigo on head movements and body position. This patient does not have vertigo. Moreover, tremor is not a feature of BPPV. Presence of tremor in the history makes BPPV an unlikely diagnosis.

(Option E) Optic neuritis is the demyelinating inflammation of the optic nerve. Multiple sclerosis is the most common associated condition, but it can be seen in other conditions such as diabetes as well. Symptoms include retro-orbital pain brought
on by eye movements, partial or complete vision loss and blurred vision. None of these features are present in this patient. On the other hand, optic neuritis does not cause dizziness/vertigo and/or tremors.

NOTE - Different causes of true vertigo are as follows:

Peripheral causes

Labyrinthitis.
Vestibule neuronitis.
Meniere’s disease
Benign paroxysmal positional vertigo (BPPV)
Chronic suppurative otitis media (CSOM)
Cervical spondylosis with or without radiculopathy

Central causes

Transient ischemic attack (TIA) / stroke e.g. PICA syndrome


Complex migraine
Multiple sclerosis (MS)
CNS tumors (e.g. acoustic neuroma)

Other causes

Medications/ drugs
Idiopathic/ iatrogenic
Psychogenic
Trauma

References

• Medscape – Withdrawal Syndromes

• UpToDate - Overview of the chronic neurologic complications of alcohol

Last updated:
Time spent: QID:1508 2023-2-12

797 of 1943
A 6-year-old boy is brought to your practice by his parents with recurrent episodes of headache for the past few weeks. He has frequent episodes almost every week. The headache is often associated with visual disturbances and accompanied by
vomiting at times. After sleeping, the headache resolves. Which one of the following is the most appropriate drug for management of acute episodes?

A. Aspirin.

B. Paracetamol.

C. Sumatriptan.

D. Non-steroidal anti-inflammatory drugs (NSAIDs).

E. Propranolol.

Incorrect. Correct answer is B


45% answered correctly

Explanation:

Correct Answer Is B

With recurrent episodes of headache associated with visual disturbances (aura), nausea and vomiting, and resolution after sleep, migraine with aura (classic migraine) is the most likely diagnosis.

In children, a stepwise management of migraine headaches starts with simple analgesic. Paracetamol either alone or in combination with NSAIDs (option D) such as ibuprofen as the most appropriate option.

Over recent years there has been an increasing use of medications from the triptans group, most commonly sumatriptan. These agents work through 5-HT (1B/1D) receptors by three mechanisms:

Promoting vasoconstriction of painfully dilated cerebral blood vessels


Inhibiting the release of vasoactive neuropeptides by trigeminal nerves
Inhibiting nociceptive neurotransmission.

The efficacy of triptans such as sumatriptan (option C) has been established in several large studies and recently there has been more evidence for their safety and effectiveness in the adolescent population. In the pediatric population (less than 12
years of age), triptans have been shown to be reasonably safe but not very effective in the termination of recurrent migraines. Different formulations are available, including nasal sprays and sublingual wafers, and it is likely that these will become a
mainstay of migraine termination treatment, especially in the adolescent population.

As this is the first presentation, and the patient is a child less than 12 years of age, and not an adolescent, the best choice would be paracetamol. However, if paracetamol and NSAIDs fail to control the pain, triptans would be the next best option to
consider.

Some studies have shown that aspirin (option A) may be more effective than some NSAIDs such as ibuprofen, but it should not be used in the pediatric group due to the risk of Reye syndrome.

Propranolol (option E) is often used to prevent migraine attacks in children but it is not an appropriate option for treatment of an acute attack.

References

• RACGP – AFP - Managing childhood migraine

Last updated:
Time spent: QID:1570 2023-2-12

798 of 1943
A 72-year-old man is being assessed for memory loss. On mini mental status exam (MMSE), he cannot answer two questions because of sight impairment. At the end of the test, his score is 23. Which one of the following would be the most
appropriate next step in management?

A. Start him in donezepil.

B. Perform another cognitive assessment.

C. CT scan of the head.

D. MRI of the head.

E. Reassure.

Incorrect. Correct answer is B


45% answered correctly

Explanation:

Correct Answer Is B

MMSE is a practical screening test to evaluate cognitive impairment in older adults. A score of 24 or above rules out cognitive impairment.

Sight impairment has an effect in the results of MMSE (or some similar tools) and may lead to falsely decreased scores. Sight impairment not only can affect completion of tasks requiring vision such as copying a pentagon, but it also has been
shown to be associated with impairment in performing tasks that do not require vision.

In those with sight impairment, it is suggested that the test be repeated after the sight is corrected, or other tests in which the vision is not essential for reliable results are used. With an intact sight, the MMSE score of this patient would increase to at
least 25; a score that excludes cognitive impairment and overrules the previous score of 23 that is indicative of mild impairment; therefore, this patient can be safely reassured as to not having cognitive impairment.

TOPIC REVIEW

The following video demonstrates how to perform an MMSE:

References

• IHPA - Standardised Mini-Mental State Examination (SMMSE)

Last updated:
Time spent: QID:708 2023-2-12

799 of 1943
A 67-year-old woman presents to your practice with complaint of headaches for the past 2 weeks. She describes that the headache is nagging in nature and is present on waking up, worse in the morning and associated with vomiting but no nausea.
She has taken some over-the-counter painkillers for relief to no avail. Which one of the following could be the most likely diagnosis?

A. Migraine headache.

B. Cluster headache.

C. Sinusitis.

D. A brain tumor.

E. Tension-type headache.

Incorrect. Correct answer is D


45% answered correctly

Explanation:

Correct Answer Is D

Common things are more common. Generally, in a patient presenting with headache, benign causes such as tension-type or migraine headache are more likely diagnoses. However, the presence or absence of some features excludes some of these
common and benign pathologies and suggest more serious and less common diagnoses as more likely. In this patient nagging headaches in the morning and the associated vomiting not preceded by nausea (projectile vomiting) are highly
suggestive of a space occupying lesion in the brain such as a tumor or an abscess. Of the options, a brain tumor is most likely to have given rise to such presentation. Other physical exam findings if present can include focal neurological deficits,
papilledema, or personality changes.

(Option A) Migraine headache is throbbing in nature, felt unilateral, and may occur in the morning as well. Nausea and sometimes vomiting are often present. Migraine headache and headache caused by space occupying brain lesions tend to
aggravate by head movement. An untreated migraine headache can last between 4-72 hours. In this patient however, the nagging rather than throbbing nature of the pain, absence of previous migraine headache, and also the fact that there is no
nausea makes migraine less likely.

(Option B) Cluster headaches present completely different. Cluster headache is usually unilateral and tend to occur at night or early in the morning waking the patient from sleep. The pain attack is usually a deep, extremely intensive pain around or
behind the eye or in the temple. Tearing, runny nose, eye congestion, and ptosis of affected side. The patient is agitated and restless with a tendency to pace around and/or push the painful area with his/her hand. Attacks occur about the same time
on each and there may be up to 8 attacks a day. Each attack lasts between 15 and 180 minutes. The nagging nature of the pain in this patient, and that aches do not occur in episodic clusters make such diagnosis less likely.

(Option C) Headaches caused by sinusitis are nagging and associated with a sense of pressure. There might be tenderness over maxillary or frontal sinuses. Other features are nasal discharge and cough due to postnasal discharge. However,
sinusitis headache is not associate with nausea and vomiting.

(Option E) Tension-type headaches is the most common cause of headache. Such headaches though often occur in the evening hours and are not associated with nausea and vomiting. Tension headache responds well to rest and analgesia with
simple painkillers such as paracetamol or NSAIDs.

References

• RACGP – AFP – Management of chronic headache

• RACGP – Theme – Diagnosing headache

• RACGP – Clinical guidance for MRI referral: Unexplained chronic headache

Last updated:
Time spent: QID:1608 2023-2-12

800 of 1943
A 71-year-old man is brought the Emergency Department with right-sided weakness of his arm and leg. His past medical history is significant for myocardial infarction 3 years ago for which he takes aspirin 100 mg daily. Non-contrast brain CT scan
excludes intracranial hemorrhage. Doppler ultrasonography shows bilateral carotid artery stenosis of 50%. Which one of the following would be the most appropriate initial management?

A. Add warfarin.

B. Add clopidogrel.

C. Carotid endarterectomy.

D. Continue the same dose of aspirin.

E. Increase the dose of aspirin.

Incorrect. Correct answer is B


45% answered correctly

Explanation:

Correct Answer Is B

Current guidelines recommend immediate antiplatelet therapy for patients with stroke or TIA after intracranial hemorrhage is excluded using a non-contrast brain CT scan. Aspirin plus clopidogrel should be commenced within 24 hours and continued
short-term. There is strong recommendation against dual therapy with aspirin and clopidogrel for long-term prevention of stroke/ TIA. This patient has developed a stroke while on aspirin. For him, short-term addition of clopidogrel is the most
appropriate immediate step to consider.

It is recommended that asymptomatic patients with carotid artery stenosis of equal to greater than 60% and symptomatic patients with stenosis of equal to or greater than 50% undergo carotid endarterectomy (option C) in an attempt to prevent
further strokes. With a cerebrovascular event and a stenosis of 50% this patent requires carotid endarterectomy, ideally within 2 weeks, as the most appropriate action to prevent further strokes. The question however asks about immediate rather
than long-term management.

(Option A) Warfarin is the preferred anticoagulant to prevent stroke in the setting of atrial fibrillation.

(Option D) With the stenotic carotid artery, continuing the same dose of aspirin is necessary but will not prevent further cerebrovascular events.

(Option E) Increasing the dose of aspirin has not proven effective in prevention of further strokes/TIAs compared to lower doses.

References

• Stroke Foundation - Clinical Guidelines for Stroke Management

Last updated:
Time spent: QID:1678 2023-2-12

801 of 1943
You are assessing Tim, a 65-year-old man, who is brough to the Emergency Department after his wife suspected there may be something wrong with him. She noticed that he had been shaving only the right side of his face for the past 2 days.
Assessment suggests ischemic stroke and you are performing a neurological examination on him. In the physical examination, when you ask him to lift his left arm, he raises his right arm instead. Which one of the following areas is most likely to
have been affected by the stroke?

A. Right frontal cortex.

B. Left temporal cortex.

C. Right parietal cortex.

D. Left parietal cortex.

E. Right occipital cortex.

Incorrect. Correct answer is C


45% answered correctly

Explanation:

Correct Answer Is C

The scenario represents a typical case of hemineglect, also known as unilateral spatial neglect, or unilateral inattention. Hemineglect is a disabling condition that classically presents as non-dominant (often left-sided) spatial disorientation after a
pathological event in the right cerebral hemisphere, classically the right posterior parietal cortex. Patients with syndrome can entirely fail to attend to the side opposite (contralateral) to lesion. The patient often collides into their surroundings, ignores
food on one side of the plate, and attends to only one side their bodies.

This patient is neglecting the left side evident by not shaving the left side of his face (as if it does not exist) and lifting the right arm instead of the left arm because he is unaware of his left arm. Right parietal lobe pathology is the most likely
explanation. Ischemic injuries to the parietal lobe are often caused by stroke in the territory of middle cerebral artery.

NOTE - Spatial neglect not only can cause sensory and motor impairment, but can also affect other components like perceptual, representational, visuospatial, behavioral deficits, etc.

Frontal cortex lesions (option A) may include disinhibited speech and behavior, primitive reflexes (e.g., grasping, sucking reflexes), altered mental status, impaired judgment, contralateral weakness (greater in legs than arms), contralateral cortical
sensory deficits, and gait apraxia.

Temporal cortex lesions (option B) may cause different symptoms which are less noticeable by others. These symptoms include problems with language and speech, forgetfulness, and visual disturbances.

Left parietal lobe involvement (option D) results in right side hemineglect. This patient has left side hemineglect; therefore, the lesion is in the right parietal lobe.

Occipital lobe lesions (option E) usually present with contralateral homonymous hemianopsia, cortical blindness, visual agnosia (not recognizing familiar objects), altered mental status, and impaired memory.

References

• Medscape – Spatial Neglect

Last updated:
Time spent: QID:1692 2023-2-12

802 of 1943
Ten days ago, Jenna, 65-year-old, received Covid19 vaccine. Now she has presented with complaints of persistent headaches, anorexia, and nausea. The headache has been resistant to painkillers and progressive in nature. He has no other
symptoms and has been otherwise in good health. She only takes thyroxine for hypothyroidism.

On examination, she is systemically well and has a blood pressure (BP) of 130/75 mmHg while lying down, a regular pulse rate of 78 beats per minute, a respiratory rate of 16 breaths per minute, and oxygen saturation of 98% on room air. Ear, nose,
and throat examination is unremarkable as is the cardiovascular exam. A full neurological exam is performed with no concerning findings. She has a Glasgow coma scale (GCS) of 15. Which one of the following is the most important investigation to
perform?

A. 24-hour BP monitoring.

B. CT scan of the head.

C. D-Dimers, fibrinogen, and platelet levels​​.

D. Urgent retinal scans.

E. Radionucleotide imaging of the thyroid.

Incorrect. Correct answer is C


45% answered correctly

Explanation:

Correct Answer Is C

Persistent headaches in recipients of adenovirus-based vaccines such as Covid19 vaccines should always alarm cerebral venous thrombosis (CVST) until proven otherwise.

CVST is a recognized complication of the ChAdOx1 nCoV-19 vaccine that should elicit a high index of clinical suspicion when patients of any age present with persistent headaches 4–42 days post-administration of the vaccine.

Headache is the most frequent symptom in CVST, and it may occur isolated or accompanied by other symptoms. As in other secondary headache disorders, CVST may be recognized by the presence of red flags.

CVST and other thrombotic complications in adenovirus-based vaccine recipients share a unique feature, delayed presentation. In most of the reported cases, symptoms started one week after immunization.

Although headache is a common symptom after vaccination, it typically presents and resolves within the same day or a few days later.

All patients with symptoms consistent with CVST (persistent, severe, and exacerbated by recumbency and Valsalva) within the relevant time criteria should be assessed initially by blood investigation assessing for (relative) thrombocytopenia and
elevated D-dimer (>5 times the upper limit of normal), and subsequently by imaging such as CT scan (option B) if the screening tests yield positive results. Consideration for hematological advice should be sought early for possible CVST cases.

It is crucial that all general practitioners are aware of the signs/symptoms of CVST and other thrombotic complications of the ChAdOx1 nCoV-19 vaccine and have a low threshold for consideration of an investigation of thrombosis and
thrombocytopenia syndrome (TTS).

24-hour BP monitoring (Option A)would be indicated if there were high fluctuations in blood pressure, which is not the case here.

Retinal scans (option D) are indicated in cases of acute visual loss which is not a complaint in this scenario. Radionucleotide imaging (option E) would be considered in case of some thyroid disorders.

References

• RACGP - AJGP: Case study of thrombosis and thrombocytopenia syndrome following administration of the AstraZeneca COVID-19 vaccine

Last updated:
Time spent: QID:1768 2023-2-12

803 of 1943
A 71-year-old man presents to the Emergency Department with weakness of the left arm and leg. His past medical history is significant for coronary angioplasty 5 years ago for which he is currently on daily low-dose aspirin. A non-contrast brain CT
scan excludes cerebral hemorrhage. Doppler ultrasonography shows bilateral carotid artery stenosis of 50%. Which one of the following would be the most appropriate action to takef or long-term secondary prevention?

A. Add warfarin.

B. Add clopidogrel.

C. Carotid endarterectomy.

D. Continue the same dose of aspirin.

E. Increase the dose of aspirin.

Incorrect. Correct answer is C


45% answered correctly

Explanation:

Correct Answer Is C

It is recommended that asymptomatic patients with carotid artery stenosis of equal to or greater than 60% and symptomatic patients with stenosis of equal to or greater than 50% undergo endarterectomy in an attempt to prevent further strokes as
the most appropriate management. With a cerebrovascular event and a stenosis of 50% this patent requires carotid endarterectomy as the most appropriate action to prevent further strokes.

(Option A) Warfarin is the preferred anticoagulant to prevent stroke in the setting of atrial fibrillation.

(Option B) Although short-term combination theray with aspirin and clopidogrel for 3 weeks is recommended for initial pharmacological treatment of patients with stroke or high-risk TIA, long-term use is discouraged and should be avoided unless
they are indicated for acute coronary syndrome or coronary artery stents. If an individual has a stroke while on aspirin, dipyridamole should be added to aspirin or aspirin be switched to clopidogrel.

(Option D) With the stenotic carotid artery, continuing the same dose of aspirin is necessary but will not prevent further cerebrovascular events.

(Option E) Increasing the dose of aspirin has not proven effective in prevention of further strokes/TIAs compared to lower doses.

References

• Stroke Foundation - Clinical Guidelines for Stroke Management

Last updated:
Time spent: QID:732 2023-2-12

804 of 1943
A 4-year-old black Sudanese boy from a refugee camp presents with irritability and lethargy. Laboratory assessment is significant for a decreased 25-hydroxy vitamin D level, hemoglobin of 48 g/L, platelet count of 170x106/L, and white cell count of
8x106/L. Which one of the following can be the cause of his clinical picture of irritability and lethargy?

A. Acute lymphocytic leukemia (ALL).

B. Acute myelocytic leukemia (AML).

C. Low level of 25-hydroxy vitamin D.

D. Low hemoglobin.

E. Infection.

Incorrect. Correct answer is C


45% answered correctly

Explanation:

Correct Answer Is C

Vitamin D deficiency is a problem seen in people of low socioeconomic status, particularly those living in camps such as refugees, those living near Poles where there is not enough sun exposure, and dark-skinned individuals.

Moreover, according to an article recently published in the Medical Journal of Australia (MJA), vitamin D deficiency is prevalent among adult Aboriginal Australians, particularly in autumn, spring and winter. The article, however, is not clear about the
prevalence of vitamin D deficiency among Aboriginal children.

Vitamin D insufficiency leads to decreased absorption of dietary calcium and hypocalcemia. The earliest effects of hypocalcemia on CNS include irritability and lethargy. Severe cases may develop into seizures.

(Options A and B) With acute leukemia, fatigue would be the most common presenting symptom. Moreover, this patient has normal WBC and platelet counts, making leukemia a less likely diagnosis.

(Option D) Anemia commonly presents with fatigue and pallor rather than irritability and lethargy.

(Option E) In the absence of fever and a normal WBC count, infection is very unlikely.

References

• MJA - Vitamin D insufficiency in Aboriginal Australians

Last updated:
Time spent: QID:107 2023-2-12

805 of 1943
A 34-year-old Aboriginal woman is found to be IgM positive, but IgG negative after exposure to a child with rubella during the first trimester of pregnancy. Rubella infection is confirmed with repeated serology testing showing not only a positive IgM,
but also rising titers of IgG. Which one of the following would be the most appropriate next step in management?

A. Termination of the pregnancy.

B. Give natural human immunoglobulin (NHIG).

C. Check rubella serology again and get the test reported from another laboratory.

D. No action is required.

E. Give MMR vaccine.

Correct
45% answered correctly

Explanation:

Correct Answer Is A

The positive IgM for rubella indicates active rubella infection. In such cases, repeating the test is indicated. If IgM is still poitive, diagnosis of rubella infection is confirmed. No further testing from another laboratory (option C) is required.

If rubella infection occurs in the first trimester of pregnancy, the risk of fetal infection and damage is high (85% in the first 2 months of pregnancy, and 50 -80% in the third month); therefore, termination of pregnancy is usually recommended.

NHIG (option B) has been used in trials for prophylaxis and prevention of congenital rubella syndrome after contact to a case with rubella infection. The results have been discouranging; therefor, it is not advised for prophylaxis. This woman, on the
other hand, has establsihed infection and NHIG wil not be of benefit as primary prevention.

(Option E) Rubella-containing vaccines are contraindicated throughout pregnancy and shoud be avoided. Women who remain susceptible to rubella should receive MMR vaccine postpartum.MMR vaccine is a live attenuate vaccine and is
contraindicated during pregnancy.

806 of 1943
A 52-year-old lady presents to your GP clinic for breast cancer screening. She has been on combined hormone replacement therapy (HRT) for the past year. Mammogram is the only available method of screening for breast cancer and because of
HRT she is expected to have dense breast tissue. Which one of the following is the best recommendation to this patient regarding HRT and breast cancer screening?

A. Stop HRT six months before mammography.

B. Stop HRT 1one year before mammography.

C. Continue HRT and perform mammography as recommended for other women.

D. Change combined HRT to estrogen-only hormone replacement therapy.

E. Decrease the dose of combined hormone replacement therapy.

Incorrect. Correct answer is C


45% answered correctly

Explanation:

Correct Answer Is C

About 10% of women on combined HRT will have extremely dense breasts; nonetheless, presently, there is not enough evidence to recommend that patients on combined hormone replacement therapy should stop HRT for any length of time before
their mammography in an attempt to reduce breast density. Therefore, this patient should continue HRT and have mammography based on current recommendations for other women.

Women making decisions as to whether to commence combined HRT should be aware of the fact that HRT may lead to inaccurate mammography results. Furthermore, they should be informed about the increased risk of breast cancer if HRT
extends beyond the recommended period (3 to 5 years).

807 of 1943
A 34-year-old multigravida, whose fetus is known to be in breech position, presents to the hospital for vaginal delivery. After spontaneous rupture of the membranes, you notice bradycardia and variable deceleration on fetal heart rate monitoring.
Vaginal examination confirms cord prolapse; however the cord is still pulsating. Which one of the following would be the most appropriate next step in management?

A. Push the presenting part away, push the cord up as far as possible, and keep it back with digital pressure.

B. Continue the vaginal delivery.

C. Give oxytocin to enhance contractions.

D. Prophylactic antibiotics.

E. Fetal scalp blood sampling.

Correct
45% answered correctly

Explanation:

Correct Answer Is A

Cord prolapse presents with abnormalities in fetal heart rate patterns (bradycardia, variable decelerations, etc.). If these changes begin after the rupture of the membranes, either spontaneously or with amniotomy, cord prolapse should be strongly
suspected.

Umbilical cord prolapse is abnormal position of the cord in front of the fetal presenting part, resulting in cord compression during labor and fetal hypoxemia.

Once the diagnosis of the cord prolapse is confirmed on pelvic examination and the cord is still pulsating (indicating the fetus is still alive), the presenting part should be pushed up with the examiner's finger, followed by pushing the cord up as far as
possible and maintaining the digital pressure.

The mother should be placed in knee-chest position and with the examiners hand in place and \rushed to the operating room for emergency cesarean delivery.

With the child being unstable and severely hypoxemic (reflected by variable decelerations on heart rate monitoring) there is no place for vaginal delivery and emergency cesarean section should be performed.

References

• South Australian Perinatal Practice Guideline - Cord Presentation and Prolapse

Last updated:
Time spent: QID:436 2023-2-12

808 of 1943
Which one of the following is the most common cause of cesarean section delivery in Australia?

A. Failed progress of labor.

B. Previous cesarean section.

C. Cephalopelvic disproportion.

D. Cord prolapse.

E. Fetal distress.

Correct
45% answered correctly

Explanation:

Correct Answer Is B

The most common cause of performing a cesarean section in Australia is a history of previous cesarean section.

Other causes of performing cesarean section include:

Failed progress of the labor


Cephalopelvic disproportion
Cord prolapse
Fetal distress
Fetal malpresentation especially breech
Placenta previa
Failed induction of labor

References

• http://www.aihw.gov.au/WorkArea/DownloadAsset.aspx

• https://www.sahealth.sa.gov.au/wps/wcm/connect/49c

Last updated:
Time spent: QID:437 2023-2-12

809 of 1943
A 26-year-old woman with history of chronic immune thrombocytopenic purpura (ITP) presents to your clinic and seeks advice regarding pregnancy. She has a platelet count of
70000/mm3. Which one of the following is the correct statement in counselling her?

A. She should have splenectomy before pregnancy.

B. She should avoid pregnancy in the next 2 years.

C. She can become pregnant.

D. If she becomes pregnant, the mode of delivery should be cesarean section.

E. She cannot become pregnant while the platelet count is below 70000/mm3.

Incorrect. Correct answer is C


45% answered correctly

Explanation:

Correct Answer Is C

While uncommon, immune thrombocytopenia (ITP) is an important cause of thrombocytopenia in pregnant women either as a pre-existing condition or occurring at any time during
pregnancy.

ITP developed in pregnancy is important to be distinguished from ‘incidental’ or gestational thrombocytopenia, which, is responsible for up to 80% of cases of thrombocytopenia in
pregnant women, or from more serious conditions such as HELLP syndrome.

Gestational thrombocytopenia usually causes an approximately 10% decrease in platelet counts and is usually characterized by counts that are above 70,000/mm3. When platelets
are lower than that, it is mostly ITP.

A diagnosis of ITP is usually made in 1-4% of cases of thrombocytopenia. ITP, either pre-exiting or developed during pregnancy, does not prevent a woman from becoming pregnant
or safely delivering a healthy baby. The best advice for this woman is that she can become pregnant and ITP does not preclude vaginal delivery; however, the pregnancy is
categorized as high risk and she will need extra care and probably treatment, especially around the delivery.

In pregnant women with ITP, treatment is usually not necessary as long as the platelet count is above 30,000, unless there is bleeding or easy bruising. For pregnant women with
symptomatic platelet counts of above 30,000, or women with platelet count of below 30,000 regardless of symptoms, treatment either with prednisolone or intravenous
immunoglobulin (IVIG) is required due to significant concern of bleeding, especially uterine bleeding.

It is important to maintain a platelet count that allows the pregnant woman with the condition to go through delivery. This safe count is at least 50,000. For regional anesethesia, a
count between 70,000 and 100,000 is often demanded.

References

• Medpage Today - Clinical Challenges: ITP in Pregnancy

• Immune Thrombocytopenia in Pregnancy

Last updated:
Time spent: QID:1596
2023-2-12

810 of 1943
The fetus of a 27-year-old woman is found to have breech presentation at 32 weeks gestation. She has gestational diabetes and assessment reveals macrosomia of the fetus. The fetus is otherwise normal. Which one of the following would be the
best advice regarding the mode of delivery?

A. Induction of labor now.

B. Elective cesarean delivery.

C. Urgent cesarean delivery now.

D. Trial of vaginal delivery and then cesarean if needed.

E. Elective vaginal delivery.

Incorrect. Correct answer is B


45% answered correctly

Explanation:

Correct Answer Is B

Contraindications to vaginal delivery in breech presentation are as follows:

Cord presentation
Any presentation other than frank or complete breech with a flexed or neutral head attitude
Clinically inadequate maternal pelvis
Fetal anomaly incompatible with vaginal delivery
The fetal weight less than 2500 gr or over 3800 gr

Because of macrosomia of the baby, vaginal delivery is contraindicated and elective cesarean delivery would be the best advice.

NOTE - if ultrasound is not available for estimation of fetal weight or macrosomia, cesarean section is recommended.

The following are recommendations for vaginal delivery of a baby with breech presentation:

Continuous electronic fetal heart monitoring is preferable in the first stage and mandatory in second stage of the labor.
When membrane rupture, immediate vaginal examination is recommended to rule out prolapsed cord.
In the absence of adequate progress in labor, cesarean section is advised.
Induction of labor is not recommended for breech presentation.
Oxytocin augmentation is acceptable in the presence of uterine dystocia.
A passive second stage without active pushing may last up to 90 minutes, allowing the breech to descend adequately into the pelvis. Once active phase starts, cesarean section is recommended if delivery is not imminent after 60
minutes.
The active second stage of labor should take place in or near an operating room with equipment and personnel available to perform a timely cesarean section.
A health care professional skilled in neonatal resuscitation should be present at the time of delivery.

References

• http://www.ncbi.nlm.nih.gov/pubmed/19646324

• http://www.kemh.health.wa.gov.au/development/manua

Last updated:
Time spent: QID:439 2023-2-12

811 of 1943
You are a resident medical officer in a high-risk pregnancy clinic. One of your patients with past obstetrical history of placenta previa and cesarean section is found to have breech presentation at 36 weeks gestation. Which one of the following is a
risk factor for increased chance of term breech presentation?

A. Nulliparity.

B. Prior cesarean section.

C. Fetal anomalies.

D. Polyhydramnios.

E. All of the above.

Incorrect. Correct answer is E


45% answered correctly

Explanation:

Correct Answer Is E

Contributing factors to breech presentation include the following:

Maternal factors:

Nulliparity
Previous breech birth
Uterine (anatomical) anomaly
Placental abnormalities (previa, cornual)
Oligohydramnios
Polyhydramnios
Multiple pregnancy
Grand multiparity

Fetal factors:

Extended fetal legs


Short umbilical cord
Early gestation
Fetal abnormality
Poor fetal growth

All the options are contributing factors to breech presentation.

References

• https://www.sahealth.sa.gov.au/wps/wcm/connect/227

Last updated:
Time spent: QID:440 2023-2-12

812 of 1943
The fetus of a 35-year-old woman was found to be in breech position at 32 weeks gestation. At 37 weeks gestation, ultrasonography confirms that the breech presentation has presisted . External cephalic version is attempted but fails to reposition
the fetus to cephalic. Which one of the following is the next best step in management?

A. Systemic steroids.

B. Elective cesarean delivery at 38 weeks gestation.

C. Emergency cesarean delivery.

D. Labor induction now.

E. Repeat external cephalic version after 38 weeks again.

Incorrect. Correct answer is B


45% answered correctly

Explanation:

Correct Answer Is B

In some cases, breech presentation spontaneously turns into cephalic presentation. The chance of spontaneous version from breech to cephalic diminishes with advancing gestation age; however, this may still occur in up to 25 % of breech
pregnancies after 36 weeks. With persisting breech presentation beyond 36-37 weeks, external cephalic version (ECV) is recommended to be tried from 36 weeks for nulliparous women and 37 weeks for multiparous women. If ECV fails, delivery
either through elective caesarean section or, if no contraindication, vaginal delivery is booked and performed after 38.5 weeks.

References

• https://www.sahealth.sa.gov.au/wps/wcm/connect/227

• http://www.kemh.health.wa.gov.au/development/manua

Last updated:
Time spent: QID:441 2023-2-12

813 of 1943
A 32-year-old woman is diagnosed with deep vein thrombosis (DVT) of the iliofemoral vein at 18 weeks gestation. Which one of the following is the next best step in management?

A. Prophylactic dose of low molecular weight heparin for 6 months.

B. Therapeutic dose of low molecular weight heparin for 3 months.

C. Therapeutic dose of low molecular weight heparin for 6 months.

D. Termination of the pregnancy.

E. Start her on warfarin and continue for 6 months with strict INR monitoring.

Incorrect. Correct answer is C


45% answered correctly

Explanation:

Correct Answer Is C

As a general rule, any pregnant patient with DVT should undergo anticoagulation with therapeutic dose of low-molecular weight heparin (LMWH) for 6 months if the DVT is in veins above the knee and 3 months if the DVT has occurred below the
knee. The treatment is then followed by prophylactic dose of LMWH until delivery. Anticoagulation is required to be continued for another 6 weeks postpartum. After delivery, warfarin can be used.

NOTE – LMWH should be changed with unfractionated heparin four weeks before the due date, as LMWH is more likely to be associated with spinal hematoma if the pregnant woman undergoes neuraxial anesthesia.

Termination of pregnancy is not required as this condition is manageable. Warfarin is contraindicated in pregnancy due to risk of warfarin embryopathy.

References

• South Australian Perinatal Practice Guideline Thromboprophylaxis and Thromboembolic Disease in Pregnancy

Last updated:
Time spent: QID:442 2023-2-12

814 of 1943
Which one of the following is an indication for cervical cerclage at 14 weeks gestation?

A. Two or more consecutive previous second-trimester pregnancy losses.

B. Short cervix before 24 weeks gestation with no previous preterm birth.

C. One preterm birth before 34 weeks gestation.

D. Carcinoma of the cervix.

E. Vaginal polyp.

Correct
45% answered correctly

Explanation:

Correct Answer Is A

Decision as to whether to perform a cervical cerclage is made based upon the following conditions:

History-indicated: consider cervical cerclage at 12-14 weeks gestation for women with a history of:

Two or more second trimester pregnancy losses in the presence of cervical shortening
Each loss earlier than the previous pregnancy

Ultrasound-inidcated: perform cervical cerclage at 14-26 weeks if:

There is progressive cervical shortening on serial ultrasounds and the external os is closed​

Rescue-indicated: consider cerclage for women with a combination of:

Cervix dilated > 2 cm with no perceived uterine contractions


Premature cervical effacement > 50 %
Presence of pelvic pressure
Heavy mucoid vaginal discharge or bulging membranes through the cervical os

Short cervix before 24 weeks gestation without any history of preterm birth or miscarriages is not an indication for performing cerclage. Similarly carcinoma of the cervix and vaginal polyps are not indications to performing cerclage.

Cerclage removal should take place at 36-37 weeks, after fetal lung maturity has achieved, but before the usual onset of spontaneous labor that could result in avulsion of the sutures.

References

• http://www.sahealth.sa.gov.au/wps/wcm/connect/76cd

• http://3centres.com.au/guidelines/complications-in

Last updated:
Time spent: QID:444 2023-2-12

815 of 1943
A 23-year-old Aboriginal woman at 10 weeks gestation presents with 2-week history of nausea and vomiting. She feels dizzy. She has not seen any doctor during this period. On examination, she is dehydrated and her blood pressure is 95/60 mmHg
with a drop in systolic blood pressure by more than 20% when she stands. She is unable to tolerate oral intake. Laboratory results show ketonuria. Which one of the following would be the next best step in management?

A. Start intravenous fluids and keep her nil by mouth.

B. Give metoclopramide and intravenous normal saline.

C. Give metoclopramide and intravenous dextrose.

D. Give ondansetron and intravenous fluids.

E. Give steroids and admit in intensive care unit.

Incorrect. Correct answer is B


45% answered correctly

Explanation:

Correct Answer Is B

The clinical picture is indicative of hyperemesis gravidarum with resultant hypovolemia and pre-shock state. This patient needs urgent fluid resuscitation and intravenous antiemetic medications. The first-line fluid for resuscitation is normal saline
(0.9% NaCl) and the anti-emetic drug of choice in this situation is metoclopramide which is safe for use in pregnancy (category A).

Dextrose should be avoided because once the sugar in the solution is consumed a hypotonic fluid is left that leads to more hyponatraemic states and carries the risk of encephalopathy due to edema.

Ondansetron is the second-line antiemetic (category B1) and considered in the following situations:

More refractory vomiting


No response to metoclopramide
Recurrent hospital admissions due to hyperemesis gravidarum

Steroids such as prednisolone are third-line medications used in resistant cases of hyperemesis gravidarum only after consultation with an expert in the field.

References

• http://www.wnhs.health.wa.gov.au/development/manua

Last updated:
Time spent: QID:445 2023-2-12

816 of 1943
Which of the following abnormalities is not seen in a pregnant woman with hyperemesis gravidarum?

A. Ketosis.

B. Hypothyroidism.

C. Hyponatremia.

D. Hypokalemia.

E. Hypochloremic alkalosis.

Incorrect. Correct answer is B


45% answered correctly

Explanation:

Correct Answer Is B

Hyperemesis gravidarum affects many women, particularly in the first trimester of pregnancy. It presents with severe nausea and vomiting and may lead to following electrolyte and metabolic derangements:

Hyponatremia – caused by vomiting and GI loss


Hypokalemia – caused by vomiting and GI loss
Hypochloremic alkalosis – caused by vomiting and GI loss
Ketosis – resulting from decreased oral intake, starvation and dehydration
Abnormal liver enzymes (ALT>AST)
Increased serum amylase and lipase

Beta hCG is a glycoprotein with structural similarities to the beta subunit of thyroid stimulating hormone (TSH). Significantly increased amounts of beta hCG, particularly around weeks 10 to 12 of pregnancy can cause subclinical hyperthyroidism and
on rare occasions, overt hyperthyroidism. This condition is usually self-limiting, but occasionally anti-thyroid medications are required.

Hypothyroidism is not a feature of hyperemesis gravidarum.

References

• http://www.uptodate.com/contents/clinical-features

Last updated:
Time spent: QID:446 2023-2-12

817 of 1943
A 48-year-old woman presents complaining of hot flushes, insomnia and mood swings. She has been on sertraline for mood swings and also takes over-the-counter vitamins. Which one of the following is incorrect regarding menopause?

A. Decreased estrogen level.

B. Increased serum LH level.

C. Decreased serum FSH.

D. Increased LDL level.

E. Decreased bone density.

Incorrect. Correct answer is C


45% answered correctly

Explanation:

Correct Answer Is C

Menopause is characterized by irregular menstrual cycles and marked hormonal fluctuations, often accompanied by hot flushes, sleep disturbances, mood symptoms, and vaginal dryness.

Eventual depletion of oocytes and ovarian follicles in the ovaries result in drop in estrogen level and elevation of serum FSH and LH levels. Decrease in serum estrogen results in the characteristic features of menopause. Long term effects of low
estrogen include osteoporosis and vaginal atrophy. After menopause there is a rise in LDL level.

Serum FSH is elevated (not decreased) in perimenopausal period and causes hot flushes.

References

• Medscape - Menopause

Last updated:
Time spent: QID:89 2023-2-12

818 of 1943
Which one of the following is the most common symptoms of hyperemesis gravidarum?

A. Nausea.

B. Vomiting.

C. Fever.

D. Headache.

E. Dizziness.

Correct
45% answered correctly

Explanation:

Correct Answer Is A

The most common symptom of hyperemesis gravidarum is nausea. It occurs in 80% to 85% of the cases. Vomiting is present in about 50% of the cases.

The peak severity for hyperemesis is around 12 weeks (when beta hCG is at tits peak). The symptoms resolve in most of women by 20 weeks, but 10% will continue to have the problem throughout pregnancy.

Hyperemesis gravidarum is a severe form of nausea and vomiting occuring in 0.3%1 to 1.5% of pregnancies. Excessive vomiting may lead to the following:

Dehydration
Ketosis
Electrolyte imbalance
Vitamin deficiency in pregnancy (very rare)

The term 'hyperemesis', however, should be used only where one or more of the following exist:

Persistent symptoms that have led to attendance at the hospital, and the need for intravenous (IV) therapy
Weight loss of > 4 kg (or >5%) since conception associated with persistent vomiting/anorexia
Lack of response to usual antiemetic and other medications

References

• http://www.wnhs.health.wa.gov.au/development/manua

• https://www.sahealth.sa.gov.au/wps/wcm/connect/a81

• http://www.australianprescriber.com/magazine/37/2/

Last updated:
Time spent: QID:447 2023-2-12

819 of 1943
A 26-year-old lady at 18 weeks gestation presents with clear vaginal discharge. She has the history of preterm labor at 24 weeks in her last pregnancy. On examination, you notice clear fluid coming out of the vagina. Which one of the following is the
best predictor of preterm labour?

A. Cervical length of 15 mm.

B. Cervical length of 40 mm.

C. Elevated serum beta hCG.

D. Fibronectin test.

E. Abdominal pain.

Correct
45% answered correctly

Explanation:

Correct Answer Is A

Measurement of cervical length by transvaginal ultrasound is the best way to predict preterm delivery between 14 and 24 weeks gestation. The predictive value is even higher between 20 and 24 weeks of pregnancy.

At 20 weeks, the cervical length is expected to be 40mm. This length is decreased to 34mm at 34 weeks gestation. A cervical length of 15 mm is definitely short for this gestational age and is associated with increased risk of pregnancy loss.

Cervical length screening is required at least twice between 14-24 weeks gestation in the following situations to predict preterm labor:

Prior preterm birth less than 34 weeks gestation


Previous cervical cone biopsy
Women with suspected cervical incompetency
Multiple pregnancies

Fetal fibronectin (fFN) level is another test that could be performed to predict the chance of preterm labor. Candidates for testing should meet the following criteria:

Intact fetal membranes


Cervical dilation less than 3 cm
Gestational age of between 22+ 0d and 34+6d weeks

The absence of fetal fibronectin (fFN) in the cervical secretions is a very useful negative predictor of imminent birth (negative predictive value for birth within 7 days 97-98%). By negative prediction its meant that with a negative test the preterm labor
is unlikely, but a positive test does not mean that preterm labor will occur.

NOTE – fFN is not useful for prediction of preterm labor if performed earlier than 22 weeks.

A cervical length, on the other hand, is a good positive predictor, meaning that with a cervical length of more than 30 mm the preterm labor is unlikely. In cases of threatened preterm labor, a threshold of 30 mm has been consistently reported to
exclude preterm labor, but there is no threshold of cervical length that establishes the diagnosis. In women with contractions and cervical length less than 30 mm, fetal fibronectin may help predict the patient's risk of preterm delivery within the next
seven days.

(Option B) A cervical length of 40mm is quite normal and against preterm labor as a diagnosis.

(Option D) fFN test is not an appropriate option here because the gestational age is less than 22 weeks and secondly, clear vaginal discharge might be due to rupture of membranes and leaking amniotic fluid. This excludes the patient as a potential
candidate.

(Option C) Elevated beta hCG is not helpful in predicting miscarriage in second trimester of pregnancy.

(Option E) Premenstrual-like cramping lower abdominal pain is one of the features seen in cervical insufficiency, but it is very nonspecific with no predictive value for miscarriage.

References

• http://www.kemh.health.wa.gov.au/development/manua

• http://www.uptodate.com/contents/fetal-fibronectin

Last updated:
Time spent: QID:448 2023-2-12

820 of 1943
A 29-year-old woman presents to your clinic at 16 weeks pregnancy with complaints of premenstrual-like cramping, backache and increased clear vaginal discharge for the past week. She has the past history of miscarriages at 14 and 18 weeks in
her last two pregnancies. On pelvic examination, the cervix is 4 cm dilated and 80% effaced. Which one of the following is the most likely diagnosis?

A. Placental abruption.

B. Twin pregnancy.

C. Cervical insufficiency.

D. Normal finding.

E. Placenta previa.

Incorrect. Correct answer is C


45% answered correctly

Explanation:

Correct Answer Is C

The clinical features are suggestive of cervical insufficiency.

Cervical insufficiency or cervical incompetence is defined as inability of the uterine cervix to retain a pregnancy to viability in the absence of uterine contractions.

Risk factors for cervical insufficiency include:

Trauma from rapid forceful cervical dilation associated with second trimester abortion procedures
Cervical laceration from rapid delivery
Injury from deep cervical conisation
Congenital weakness due to exposure to diethylstilbestrol (DES)

Presentation

The usual presentation is the following sign and symptoms present for days to weeks:

Pelvic pressure
Backache
Premenstrual-like lower abdominal cramping (different form uterine contractions of labour)
Increased vaginal discharge or bulging membranes through the cervix on vaginal exam

Diagnosis

The diagnosis is based on:

1. A history of recurrent (two or more) miscarriages occurring after the 12th weeks gestation, usually starting with painless leaking of amniotic fluid
2. The easy passage of a size 9 cervical dilator through the internal os of the cervix when the woman is not pregnant, and the absence of a ‘snap’ on its withdrawal
3. A cervical length of less than 25mm or cervical funneling >40% prior to 24 weeks gestation

Placental abruption (option A) presents with painful vaginal bleeding, which could lead to shock. Twin pregnancy (option B) has other different features such as anaemia, polyhydramnios, large for gestational age and more severe hyperemesis
gravidarum. Placenta previa (option E) is characterized by painless vaginal bleeding.

References

• http://emedicine.medscape.com/article/1979914-clin

• http://www.sahealth.sa.gov.au/wps/wcm/connect/76cd

• Llewellyn-Jones - Fundamentals of obstetric and gynaecology Elsevier – Mosby 9th Edition

Last updated:
Time spent: QID:449 2023-2-12

821 of 1943
A 35-year-old woman presents with regular uterine contractions at 34 weeks gestation. Sterile speculum examination is performed showing cervical dilation of 2 cm and intact membranes. Which one of the following investigations is the best
negative predictor of imminent birth within the next 7 days?

A. Transvaginal ultrasound.

B. Mid-stream urine sample.

C. The absence of fetal fibronectin in cervical secretions.

D. The presence of fetal fibronectin in cervical secretions.

E. Irregular uterine contractions.

Incorrect. Correct answer is C


45% answered correctly

Explanation:

Correct Answer Is C

Preterm labor is defined as regular uterine contractions after 20 weeks but before 37 weeks gestation, associated with cervical shortening and effacement.

Risk factors for preterm labour include:

Infections
Antepartum hemorrhage
Uterine distension
Spontaneous rupture of membranes

Fetal fibronectin (fFN) is thought to be a "trophoblast glue" that promotes cellular adhesion at uterine-placental and decidual-fetal membrane interfaces. It is released into cervicovaginal secretions when the extracellular matrix of the
chorionic/decidual interface is disrupted; this is the rationale for measurement of fFN as a predictor of PTD.

Absence of fFN in cervical secretions is the most useful negative predictor of imminent birth within the next 7 days. This test is performed at bedside using a commercial test kit according to instructions which are provided by the kit. It is very
expensive but useful.

Candidates for fibronectin test are pregnant women who meet the following criteria:

Intact fetal membranes


Cervical dilation less than 3 cm
Gestational age of between 22 +0d – 34+6d weeks

At 34 weeks gestation, with a cervical dilation of 2 cm and intact membrane fibronectin test is the best negative predictor, meaning that with a negative test, preterm labor within the next 7 days would be unlikely. In the presence of fFN (positive test),
one cannot tell with certainty that preterm labor will follow.

Other steps to take are obtaining vaginal swabs and midstream urine for evaluation of vaginal infection and colonization with group B streptococcus. Ultrasonography is performed to assess cervical length, placental position and presentation before
the fFN is performed. A sterile speculum examination should be considered to exclude rupture of membranes.

References

• http://www.kemh.health.wa.gov.au/development/manua

• http://www.uptodate.com/contents/fetal-fibronectin

Last updated:
Time spent: QID:450 2023-2-12

822 of 1943
A 28-year-old pregnant woman is involved in a car accident at 26 weeks gestation, while wearing seatbelt. On examination, there are visible bruises on the abdomen. She is otherwise normal. Fetal heart sounds are audible and within normal
parameters and CTG is reassuring. Which one of the following is the next best step in management?

A. Reassure and discharge home.

B. Perform 24-hour cardiotocography (CTG) monitoring.

C. Admit her and observe for 24 hours.

D. Perform a continuous 30-minute CTG and discharge home if reassuring.

E. Observe for six hours and discharge home.

Incorrect. Correct answer is C


45% answered correctly

Explanation:

Correct Answer Is C

Trauma is a major contributor to maternal mortality, and is the leading cause of pregnancy-associated maternal deaths.

Maternal trauma can compromise the fetus as a result of maternal hypotension or hypoxemia, placental abruption, uterine rupture, or fetal trauma.

With bruises on the abdomen (seatbelt marks) this woman has positioned the seat belt incorrectly over the uterus and there is a good chance that the uterus and its contents, including the fetus, has been affected by the impact. The correct position
of the seat belt is when the lap belt is placed on the hip below the uterus and the sash between breasts and above the uterus.

Correct (green) vs. incorrect position of seat belt for pregnant women

Apart from routine trauma workup necessary for non-pregnant patients, a minimum 24-hour period of monitoring is recommended for all pregnant women who have sustained trauma if any of the following is present:

Regular uterine contractions


Vaginal bleeding
A non-reassuring fetal heart rate tracing
Abdominal/uterine pain
Significant trauma to the abdomen

With bruises over the abdomen this patient should be considered as having significant abdominal trauma and observation for a minimum of 24 hours, should be considered. These patients should not be discharged until the clinician reasonably
makes sure that they do not have an abruption or preterm labor.

References

• http://www.kemh.health.wa.gov.au/development/manua

• http://www.health.qld.gov.au/qcg/documents/g-traum

• https://www.sahealth.sa.gov.au/wps/wcm/connect/abd

Last updated:
Time spent: QID:451 2023-2-12

823 of 1943
A 20-year-old woman presents to the emergency department with premature uterine contractions. He is 28 weeks pregnant and has the history of untreated mitral valve stenosis. After necessary evaluation and absence of contraindications, tocolysis
is planned. Which one of the following would be the drug of choice for tocolysis?

A. Nifedipine.

B. Salbutamol.

C. Beta blockers.

D. Magnesium sulphate.

E. Oxytocin antagonists.

Incorrect. Correct answer is E


45% answered correctly

Explanation:

Correct Answer Is E

First-line medication for tocolysis is the calcium channel blocker nifedipine. Beta agonists such as terbutaline and salbutamol are second-line choice.

As this patient has untreated mitral valve stenosis, nifedipine and beta agonists are contraindicated. Nifedipine, by decreasing peripheral vascular resistance leads to a decrease in an already diminished cardiac output. Beta agonists do the same by
inducing tachycardia.

Oxytocin is a hormone which increases uterine contractions. Oxytocin can be counteracted by atosiban. The counteraction is achieved through competitive binding of atosiban to oxytocin and vasopressin receptors, and by this, suppressing uterine
contractions. It has about the same efficacy of nifedipine. This drug is the option of choice when other tocolytic agents such as calcium channel blockers or beta agonists are contraindicated.

Contraindications to oxytocin antagonists are generally the same for tocolysis and include:

Gestational age below 24 or over 33 completed weeks


Premature rupture of the membranes at >30 weeks of gestation
Intrauterine growth retardation
Abnormal fetal heart rate
Antepartum uterine haemorrhage requiring immediate delivery
Eclampsia and severe pre-eclampsia requiring delivery
Intrauterine fetal death
Suspected intrauterine infection
Placenta praevia or abruptio placenta
Any other conditions of the mother, or fetus, where continuation of pregnancy is hazardous
Any known hypersensitivity to the active substance or any of the ingredients

Athough magnesium sulfate acts as a calcium antagonist, and inhibits the muscle contractions of the uterus, studies have shown that magnesium sulfate (previously used as a tocolytic agent) is unlikely to stop preterm labor. But if it is given to
women in preterm labor, who are less than 32 weeks pregnant, it may help reduce the risk of cerebral palsy in preterm babies.

Beta blockers are not used to prevent uterine contractions.

References

• http://www.ncbi.nlm.nih.gov/pmc/articles/PMC297173

• http://www.kemh.health.wa.gov.au/development/manua

• http://www0.health.nsw.gov.au/policies/pd/2011/pdf

• http://www.rcpi.ie/content/docs/000001/779_5_media

Last updated:
Time spent: QID:452 2023-2-12

824 of 1943
A 34-year-old woman from country-side of Victoria presents to the hospital at 37 weeks gestation after she noticed a sudden gush of clear fluid per vagina. Uterine contractions are absent. Speculum examination shows pooling of liquor in the
posterior fornix. After 12 hours, she develops fever, tachycardia and chills. Apart from giving antibiotics, which one of the following would be the next best step in management?

A. Perform vaginal examination.

B. Continue the pregnancy until natural delivery.

C. Induce labour now.

D. Give corticosteroids.

E. Discharge from hospital after 14 days of antibiotics.

Incorrect. Correct answer is C


45% answered correctly

Explanation:

Correct Answer Is C

This patient presented with premature rupture of membranes. Premature rupture of membranes (PROM) refers to membrane rupture before the onset of uterine contractions.

The classic clinical presentation of premature rupture of membranes is a sudden gush of clear or pale yellow fluid from the vagina. This patient also has developed signs of infection (chorioamnionitis) indicated by fever, tachycardia and chills. Once
chorioamnionitis develops the induction of labour (IOL) should be considered as soon as possible.

Vaginal examination should be avoided in PROM unless delivery is imminent. Continuation of the pregnancy carries the significant risk of maternal and fetal jeopardy.

Corticosteroids are given up to 34 weeks gestation. At 37 weeks gestation maternal systemic corticosteroids will have no effect on fetal lung maturity. In the presence of chorioamnionitis where immediate induction of labour is indicated,
corticosteroids would be of no use even if the gestational age was under 34 weeks.

References

• http://www.sahealth.sa.gov.au/wps/wcm/connect/f3f6

• http://www.kemh.health.wa.gov.au/development/manua

• https://www.ranzcog.edu.au/Statements-Guidelines/Obstetrics/Preterm-Prelabour-Rupture-of-Membranes

Last updated:
Time spent: QID:453 2023-2-12

825 of 1943
A 28 weeks pregnant woman comes to your clinic with presenting complaint of a gush of clear vaginal fluid. On speculum examination, premature preterm rupture of membranes (PPROM) is confirmed. The cervical os is closed. In addition to transfer
to a tertiary care center, which one of the following would be the most appropriate management?

A. Systemic corticoseroids.

B. Nifedipine.

C. Salbutamol.

D. Cardiotocography.

E. Pelvic ultrasound.

Correct
45% answered correctly

Explanation:

Correct Answer Is A

Premature rupture of membranes (PPROM) is defined as spontaneous rupture of the membranes before the onset of labour prior to 37 weeks gestation. It complicates 2-4% of all singleton and 7- 20% of twin pregnancies and is associated with over
60% of preterm births.

The etiology is multifactorial. Risk factors for PPROM include:

Intra-amniotic infections – the most common cause


Placental abruption
Invasive uterine procedures e.g. amniocentesis, cordocentesis, chorionic villus sampling, cervical cerclage

Typically, women with PPROM present with a large gush or steady trickle of clear vaginal fluid. The clinical signs of PPROM may become less accurate after one hour.

The interval between PPROM and the onset of labor is influenced by many factors including gestational age. Women with PPROM have a 50% chance of going into labor within 24 to 48 hours. The chance increases to70 to 90% chance within 7 days.
If PPROM occurs between 24 and 28 weeks gestation the latency period before birth is generally longer than if occurring closer to term.

Perinatal complications of POM include:

Respiratory distress syndrome


Infections
Fetal intraventricular hemorrhage
Pulmonary hypoplasia
Skeletal deformities
Cord prolapse
Malpresentation

PPROM is diagnosed by sterile speculum exam and the following criteria:

Positive pooling – there is pooling of fluid in the posterior fornix


Positive nitrazine test – the pH paper will turn blue with the fluid
Positive fern test – the fluid shows a ferning pattern when the fluid is allowed to air-dry and seen under light microscope

Management

In the absence of chorioamnionitis the management depends on the gestational age:

Before 23 weeks

The labor may be induced or the patient sent home for bed rest for signs of spontaneous delivery to start.

Between 23 and 34 + 0/7 weeks

The patient should be transferred to a tertiary hospital and be admitted - while being transferred to the tertiary hospital, it is important to administer systemic corticosteroids for fetal lung maturity.

Bed rest
Cervical and vaginal swabs for microscopy and culture
Prophylactic antibiotics for prevention of chorioamnionitis

Tips for management of PROM

Digital vaginal examination should be avoided unless the woman is in active labor or birth is imminent.

Between 23 and 23+6/7 weeks gestation the decision for corticosteroids administration is made after consultation with obstetric/pediatric medical team and the parents. Corticosteroids are not effective in promoting long maturation
before 23 weeks.

A single course of antenatal corticosteroids should be administered to women with PPROM without signs of infection between 23 and 34 weeks gestation. In the presence of chorioamnionitis prompt delivery is indicated and there is
not enough time for corticosteroids to be effective on lung maturity.

If gestation is less than 34 weeks and in the absence of infection or complications and in circumstances when a course of corticosteroids has not been completed, tocolysis may be considered for threatened premature labor.

Broad spectrum antibiotic administration is recommended following PPROM to prevent infection and prolongation of the pregnancy in the short term.

It is the obstetric consultant’s decision as to when to deliver a preterm baby. If expectant management continues >34weeks, women should be advised of the increased risk for chorioamnionitis and the decreased risk of respiratory
problems in the neonate.

NOTE - chorioamnionitis is an absolute indication to terminate the pregnancy regardless of the gestational age

826 of 1943
At 28 weeks gestation, this patient should be immediately transferred to a tertiary hospital. She should also receive systemic steroids to promote fetal lung maturation in case preterm delivery ensues.

References

• https://www.ranzcog.edu.au/Statements-Guidelines/Obstetrics/Preterm-Prelabour-Rupture-of-Membranes

Time spent: QID:454 Last updated:


2023-2-12

827 of 1943
A 29-year-old woman at 32 weeks gestation presents with regular uterine contractions. Cardiotocography shows the presence of a live fetus with regular accelerations. Blood pressure is 200/120 mmHg. Liver function tests shows mildly raised
serum alkaline phosphatase. Sterile speculum examination shows foul-smelling fluid leakage from vagina. Which one of the following is the next best step in this situation?

A. Suppression of labor by giving oral nifedipine with two further doses if contractions still persist.

B. Intravenous salbutamol should be used as first line drug to stop uterine contractions.

C. Both nifedipine and salbutamol are indicated in this patient.

D. Further investigations are needed to find whether this patient needs tocolysis.

E. Tocolysis is absolutely contraindicated in this patient.

Incorrect. Correct answer is E


45% answered correctly

Explanation:

Correct Answer Is E

The presence of regular uterine contraction at 32 weeks gestation is suggestive of preterm labor. If there are no contraindications, tocolyitc agents should be used to delay the labour. In the meanwhile, betamethasone should be given to promote
accelerated maturation of the fetus’ lungs, but with fluid leaking from the vagina the membranes are very likely to have ruptured and premature rupture of the membranes is one of the contraindications to tocolysis. The foul-smelling discharge
suggests chorioamnionitis that is another contraindication to tocolysis.

Despite having a blood pressure of 200/120mmHg, this patient cannot be said to have severe pre-eclampsia. It is true that elevated alkaline phosphatase can be a pointer to hepatic involvement, and by this, one may conclude that the patient has pre-
eclampsia, but it should be noted that alkaline phosphatase can go up in late pregnancy as a physiologic response. Nonetheless, hypertension would be another clue for not using tocolytics because the diagnosis is uncertain.

The following are the contraindications to tocolysis in preterm labour:

Gestation > 34 weeks or <24 weeks


Labor is too advanced – advanced cervical dilation (>4cm) -
Chorioamnionitis
In utero fetal death
Abnormal CTG suggesting non-reassuring fetal status
Lethal fetal anomalies
Intrauterine fetal demise
Suspected fetal compromise
Significant antepartum haemorrhage, such as placental abruption/ active vaginal bleeding
Suspected intrauterine infections (e.g. chorioamnionitis)
Maternal hypotension haemorrhage with haemodynamic instability
Pregnancy-induced hypertension/ eclampsia/ pre-eclampsia
Placenta praevia
Plancental insufficiency
Intrauterine growth retardation
Severe preeclampsia
Maternal allergy to specific tocolytic agents, or where tocolytics are contraindicated with specific co-morbidities (e.g. β agonists should not be given in case of cardiac disease)

Known or suspected fetal pulmonary maturity is not an absolute contraindication to tocolysis, as there are nonpulmonary morbidities associated with preterm birth. These fetuses could potentially benefit from prolongation of pregnancy and the
nonpulmonary benefits of glucocorticoid therapy.

Inhibition of preterm labor is less likely to be successful when cervical dilation is greater than 3 cm. Tocolysis can still be considered in these cases, especially when the goal is to administer antenatal corticosteroids or safely transport the mother to
a tertiary care centre.

References

• http://www.health.qld.gov.au/qcg/documents/ed-ptl.

• http://www.uptodate.com/contents/inhibition-of-acu

Last updated:
Time spent: QID:455 2023-2-12

828 of 1943
Which one of the following is not a contraindication to tocolysis?

A. A fetus with congenital malformations inconsistent with life.

B. Suspected fetal compromise determined by cardiotocography.

C. Fetal death in utero.

D. Placental abruption.

E. Maternal hypothyroidism.

Incorrect. Correct answer is E


45% answered correctly

Explanation:

Correct Answer Is E

The following are the contraindications to tocolysis in preterm labor:

Gestation > 34 weeks or <24 weeks


Labor is too advanced – advanced cervical dilation (>4cm)
Chorioamnionitis
In utero fetal death
Abnormal CTG suggesting non-reassuring fetal status
Lethal fetal anomalies
Intrauterine fetal demise
Suspected fetal compromise
Significant antepartum hemorrhage, such as placental abruption/ active vaginal bleeding
Suspected intrauterine infections (e.g. chorioamnionitis)
Maternal hypotension
Hemorrhage with hemodynamic instability
Pregnancy-induced hypertension/ eclampsia/ pre-eclampsia
Placenta previa
Plancental insufficiency
Intrauterine growth retardation
Severe preeclampsia
Maternal allergy to specific tocolytic agents, or where tocolytics are contraindicated due to specific co-morbidities (e.g. beta agonists should not be given in case of cardiac disease)

Known or suspected fetal pulmonary maturity is not an absolute contraindication to tocolysis, as there are nonpulmonary morbidities associated with preterm birth. These fetuses could potentially benefit from prolongation of pregnancy and the
nonpulmonary benefits of glucocorticoid therapy.

Inhibition of preterm labor is less likely to be successful when cervical dilation is greater than 3 cm. Tocolysis can still be considered in these cases, especially when the goal is to administer antenatal corticosteroids or safely transport the mother to
a tertiary care center.

Maternal hypothyroidism is usually treated with thyroxine and is not a contraindication to labor suppression.

References

• http://www.health.qld.gov.au/qcg/documents/ed-ptl.

• http://www.uptodate.com/contents/inhibition-of-acu

Last updated:
Time spent: QID:456 2023-2-12

829 of 1943
Jane, a 65-year-old patient of yours, has presented to you with complaint of uterine prolapse. Which one of the following ligaments has the most important role in pathophysiology of uterine prolapse?

A. Broad ligament.

B. Round ligament.

C. Uterosacral ligament.

D. Ovarian ligament.

E. Levator ani muscle.

Incorrect. Correct answer is C


45% answered correctly

Explanation:

Correct Answer Is C

The uterus is normally held in place by surrounding and attached muscles and ligaments. Prolapse happens when the ligaments supporting the uterus become so incompetent that uterus cannot stay in place and slips down from its normal
anatomic position. These ligaments are:

1. Round ligament
2. Uterosacral ligament
3. Broad ligament
4. Ovarian ligament

Of these, incompetency of the uterosacral ligament has the most important role in pathophysiology of uterine prolapse.

830 of 1943
A 36-year-old woman with asthma and a 5-year history of hypertension is keen to become pregnant. She has never been pregnant before and has stopped using contraception methods recently. Her only currennt medication is ramipril 10 mg daily.
Her blood pressure is 130/85. Which one of the following is the most appropriate initial management of her hypertension?

A. Cease ramipril and start her on methyldopa.

B. Cease ramipril and start her on atenolol.

C. Cease ramipril and start her on irbesartan.

D. Cease the ramipril and start her on hydralazine.

E. Continue ramipril.

Correct
45% answered correctly

Explanation:

Correct Answer Is A

Some commonly prescribed anti-hypertensive drugs are contraindicated, or are better avoided before conception and during pregnancy. These drugs include:

Angiotensin converting enzyme (ACE) inhibitors e.g. ramipril, captopril


Angiotensin receptor blockers (ARBs) e.g. losartan, irbesartan
Diuretics
Most beta blockers

Methyldopa is the first-line antihypertensive agent during pregnancy for controlling mild to moderate hypertension during pregnancy.

(Option A) Beta blockers can cause fetal bradycardia and with long-term use result in growth restriction. Labetolol is an exception and can be used in hypertensive emergencies.

(Options C and E) Angiotensin receptor blockers and ACE inhibitors both are teratogenic in first trimester. In the second and third trimesters they can result in fetal renal dysfunction, oligohydramnios and skull hypoplasia.

(Option D) Hydralazine can be safely used for treatment of hypertensive emergencies during pregnancy. Otherwise, methyldopa remains the choice.

Other anithypertensice medications:

Diuretics - Diuretics can cause fetal electrolyte disturbances and reduction in maternal blood volume. This can consequently lead to restricted fetal growth.

Calcium channel blockers (CCBs) - with the exception of nifedipine, CCBs should be avoided in pregnancy due to risk of maternal hypotension and fetal hypoxia. Nifedipine is commonly used as first-line tocolytic agent, but is not used as an
antihypertensive medication during pregnancy.

NOTE - Hydralazine and labetalol are safe to use during pregnancy. However, these two drugs are used for treatment of hypertensive emergencies. For routin management of hypertension in pregnanr women or in those who are planning pregnancy,
methyldopa is the first-line option.

References

• Therapeutic Guidelines – Cardiovascular: available from http://tg.org.au

Last updated:
Time spent: QID:457 2023-2-12

831 of 1943
A 26-year-old woman presents for her first antenatal visit at 10th week of her pregnancy. On history, her mother has type II diabetes mellitus. Which one of the following would be the recommended screening test for gestational diabetes mellitus
(GDM) for her?

A. 2-hour 75 gr OGTT between 24 and 28 weeks.

B. 2-hour 75 gr OGTT now.

C. Fasting blood glucose (FBS) now.

D. Fasting blood glucose (FBS) between 24 and 28 weeks.

E. 3-hour 100 gr OGTT between 24 and 28 weeks.

Incorrect. Correct answer is B


45% answered correctly

Explanation:

Correct Answer Is B

All women should be assessed for risk of developing gestational diabetes mellitus in the first antenatal visit. The risk index is calculated according to the following table:

Risk factors Score


BMI: 25-35 kg/m2 1
BMI (pre-pregnancy or first antenatal visit) 2
BMI>35 kg/m 2
Ethnicity: Asian, Indian subcontinent, Aboriginal, Torres Strait Islander, Pacific Islander, Maori, Middle Eastern, non-white African 1
Previous GDM 2
Previous elevated blood glucose 2
Maternal age≥40 years 2
Family history DM: (first degree relative with diabetes) or sister with GDM 2
Previous macrosomia: (birth weight > 4500 g or > 90th percentile)2 2
Previous perinatal loss 2
Polycystic ovarian syndrome 2
Medications (corticosteroids, antipsychotics) 2

Every woman with a total score of 2 or greater is high risk and should have a 2-hour 75-gr oral glucose tolerance test (OGTT) now. If normal, another 2-hour 75-gr OGTT should be performed between 24-28 weeks of pregnancy.

With an abnormal 2-hour OGTT test, the patient should be advised to repeat the second 2-hour 75 gr OGTT after 10 hours of fasting while on her regular medications (if any).

Pregnant women whose risk score is less than 2 should have their first 2-hour 75gr OGTT at 24-28 weeks gestation.

With a positive family history for diabetes of her mother this patient has a total score of 2; therefore she should undergo 2-hour 75 gr OGTT now.

References

• RACGP - AJGP - Gestational diabetes mellitus: A pragmatic approach to diagnosis and management

• ADIPS Consensus Guidelines for the Testing and Diagnosis of Gestational Diabetes Mellitus in Australia

Last updated:
Time spent: QID:468 2023-2-12

832 of 1943
A 26-year-old woman with menstrual cycles ranging from 4 to 6 weeks, presents with vaginal bleeding following amenorrhea of 6 weeks duration, which is heavier than ever and not like her previous periods. A urine pregnancy test is positive. On
vaginal examination, the cervical os is closed. An endovaginal ultrasound scan is performed that reveals an endometrial thickness of 12 mm and an empty uterus. Adenexa are clear and there is no fluid in the pouch of Douglas. There is a corpus
luteum cyst in the left ovary. Which one of the following would be the most likely diagnosis?

A. Ectopic pregnancy.

B. False positive pregnancy test result.

C. Complete abortion.

D. Incomplete abortion.

E. Complicated corpus luteam cyst.

Incorrect. Correct answer is C


45% answered correctly

Explanation:

Correct Answer Is C

A positive pregnancy test, while the uterus is empty on ultrasound exam can be due to either of the following:

A false positive pregnancy test


Complete abortion
Ectopic pregnancy
Early normal pregnancy

With an endometrial thickness of 12 mm on ultrasoound, false positive pregnancy test result is less likely. If the woman is not pregnant and she is experiencing just a normal menstruation at the 6th week (as she often does according to the history)
the endometrial thickness would be expected to be 2-4 mm.

Several studies have been conducted to see whether measuring endometrial thickness would be helpful for telling ectopic pregnancy, early normal pregnancy, and complete abortion apart, but no meaningful results are yet obtained. In fact, the
endometrial thickness could be similar in all these conditions; therefore, the only condition this parameter excludes is a false negative result. Considering the fact that a corpus luteum cyst is found on ultrasound, false positive result would become
even less likely.

After ovulation, the remnant of the follicle,turns into a temporary endocrine structure that produces progesterone (pro=in favor of + gesterone= pregnancy) - an essential hormone for egg implantation and maintaining the pregnancy until the placenta
fully takes over this function by 10-12 weeks gestation.

Corpus luteal cyst may internally or externally bleed and cause pain and peritoneal irritation (resembling acute abdomen). Ovarian torsion is another very serious complication but the clinical picture would be quite different.

If pregnancy does not take place, the copus luteum involutes and disappears; otherwise, it grows in size and persists. If the patient was not pregnant, the corpus luteum cyst would be expected to have resolved by the time she starts her
menstruation. Since it has persisted, it is more likely that pregnancy has occurred.

With clear adenexa on ulrtasoudn, ectopic pregnancy is less likely but not excluded, and should be thoroughly investigated. However, the question asks about the ‘most likely diagnosis’ not the ‘most appropriate management’.

The prevalence of ectopic pregnancies in terms of the location is as follow:

Distal tubal (ampullary) – 80%


Isthmic – 12%
Fimbrial – 5%
Cornual/interstitial – 2%
Abdominal – 1.4%
Ovarian – 0.2%
Cervical – 0.2%

With clear adenexa almost 97% of EPs are excluded; however, since ultrasound is highly operator dependent, ectopic pregnancy cannot be cleared with certainty. It shoudl be noted that in uncomplicated ectopic pregnancies there is no fluid collection
in the pouch of Douglas; therefore, an empty pouch does not exclude ectopic pregnancy.

The closed OS, excludes incomplete abortion because OS will not completely close unless all the products of pregnancy are expelled and uterine contractions have stopped.

Since the US findings do not favor EP, of the options, complete abortion remains the most likely diagnosis. This does not mean that EP is ruled out whatsoever. In fact what was seen as corpus luteum cyst could be an ovarian EP that has been
misinterpreted but comparing to complete abortion that would far less likely.

TOPIC REVIEW

Common causes of falsely positive pregnancy test include:

Dirty urine collecting cup (detergent residue for example is known to cause false-positive results)
Faulty test kit (for example, the kit is damaged, past its use-by date or has been exposed to heat or moisture)
Blood in the urine (e.g. from cystitis)
Protein in the urine (e.g. from kidney damage)
Certain medications including anti-convulsants, some fertility drugs, diuretics and tranquillizers
Recent birth or miscarriage -a woman’s blood and urine may still contain detectable levels of hCG for a few weeks afterwards
An ovarian tumor or some other type of hCG-producing growth

References

• Medscape - Ectopic Pregnancy

• Radiopaedia - Endometrial thickness

L t d t d

833 of 1943
Last updated:
Time spent: QID:492 2023-2-12

834 of 1943
A 32-year-old woman presents to the Emergency Department at 38 weeks gestation because she has not felt the fetal movements since yesterday. Investigations confirm fetal demise. Delivery is induced and she gives birth to a dead fetus. Which
one of the following is least likely to reveal the cause the fetal death?

A. Autopsy of the fetus.

B. Indirect Coomb’s test of the mother.

C. Chromosomal analysis of the mother.

D. Kleihauer-Betki test.

E. Fetal X-ray.

Incorrect. Correct answer is C


45% answered correctly

Explanation:

Correct Answer Is C

It is essential that the cause of fetal demise (still birth) is investigated and determined. The purposes of such investigations are to provide an explanation for the death of their baby to the parents and family, who need to know what went wrong to
relieve suffering, enable appropriate counselling about recurrent risk, and guide the management of future pregnancies. Providing an explanation to the incidence may be helpful in going through the grieving process and decreasing the feelings of
guilt.

For all still births without an obvious cause, the following Investigations must be undertaken with the parents' consent. Consideration should be given to omitting screening tests when the cause of death is absolutely clear.

At diagnosis of a fetal death (prior to delivery):

Ultrasound scan to detect possible fetal abnormalities and to assess amniotic fluid volume
Amniocentesis (where available and warranted) for cytogenetic and infection investigations
A low vaginal and peri-anal swab, to culture for anaerobic and aerobic organisms
Full blood examination - maternal blood must be collected for:

Serology for cytomegalovirus, toxoplasmosis, and parvovirus B19


Rubella and syphilis if not already undertaken in the pregnancy
Blood group determination and antibody screen if not already undertaken in this pregnancy
Kleihauer –Betke test
Renal function tests including uric acid
Liver function tests
Bile acids
HbA1c
Anticardiolipin antibodies
Lupus Anticoagulant
Activated protein C (APC)

Following birth:

External examination of the baby (by a perinatal pathologist, neonatologist or a pediatrician if possible)
Clinical photographs
Surface swabs (ear and throat) for microbiological cultures
Babygram (X-ray of the entire body in one film) (or ultrasound (where post-mortem autopsy is refused)
Post-mortem examination
Blood samples from the cord or cardiac puncture for investigations of infection
Blood samples for chromosomal analysis
Detailed macroscopic examination of the placenta and cord
Placental microbiological cultures
Placental and amnion biopsy for chromosomal analysis
Placental histopathology

Further investigation for thrombophilia must be undertaken 8-12 weeks after the birth where:

- Fetal death is associated with:

Fetal growth restriction


Preeclampsia
Maternal thrombosis and/or maternal family history of thrombosis

- The stillbirth remains unexplained following the standard investigations

- ​Tests for thrombophilia were positive at the time of the intrauterine fetal death (IUFD) as follows:

Anticardiolipin antibodies and Lupus anticoagulant repeated if positive at the time of the intrauterine fetal death or initial testing if not previously undertaken
APC resistance if it was not undertaken at birth
Factor V Leiden mutation if APC resistance was positive at birth
Fasting Homocysteine and if there is a positive test for MTHFR gene mutation
Protein C and S deficiency
Prothrombin gene mutation 20210A

Of the given options, chromosomal analysis of the mother is not recommended, because this test does not add any further explanation to the cause of fetal demise.

References

• Prenatal Society of Australia and New Zealand - Investigations of stillbirths

• Queensland Clinical Guidelines - Stillbirth care

Last updated:
Time spent: QID:498 2023-2-12

835 of 1943
A 16-year-old girl is brought to your GP clinic because her parents are concerned about her development. She has not started her menses yet, and according to the parents she is the shortest of all her classmates at school. On examination, the
breasts have not started budding, but pubic and axillary hair growth is present. On examination, no abdominal mass, tenderness or rebound tenderness is noted. The remainder of the exam is unremarkable. The patient refuses a vaginal exam. Which
one of the following can be the most likely diagnosis?

A. Ovarian dysgenesis.

B. Mullerian agenesis.

C. Imperforated hymen.

D. A pituitary tumor.

E. Absent vagina.

Correct
45% answered correctly

Explanation:

Correct Answer Is A

Menses, the final piece of puberty, is expected by the age of 16 years. Failure to menstruate beyond this age is termed primary amenorrhea. Primary amenorrhea can be a feature of all the given options; however, short stature makes ovarian
dysgenesis (e.g., Turner syndrome) the most likely diagnosis.

Female puberty consists of three components:

1. Adrenarche – growth of pubic and axillary hair as a response to increased production of androgens by zona reticularis layer of the adrenal gland
2. Thelarche – breast development (the first sign of puberty in girls is breast budding)
3. Menarche – commencement of menses (the final event in puberty)

In patients with ovarian dysgenesis, adrenarche often occurs around the expected time like a normal female, but thelarche and menarche are almost always delayed to various extents, depending on the underlying cause, because of decreased levels
of estrogen, the adequate levels of which is crucial to thelarche and menarche. Estrogen deficiency usually leads to short stature as a characteristic finding.

In mullerian agenesis (option B), imperforated hymen (option C) and absent vagina (option D), sexual development occurs normally because ovaries have full function but there is no menstruation because of menstrual flow obstruction (e.g.,
imperforated hymen, absent vagina), or absent or abnormal menstruation (e.g., absent vagina).

Many infiltrative disease and tumors of the hypothalamus and pituitary (option E) can result in diminished GnRH release or gonadotropin destruction and consequently amenorrhea. Examples are craniopharyngioma, germinoma and Langerhans cell
histiocytosis. Such diseases, however, are often associated with other manifestations such as visual field defects and headaches, none of which present in history.

References

• UpToDate - Evaluation and management of primary amenorrhea

• Medscape – Disorders of Sex Development

Last updated:
Time spent: QID:506 2023-2-12

836 of 1943
A 28-year old woman visits your office for her first prenatal check-up. She has smokes one pack of cigarettes per day for the past five years. Which one of the following is incorrect regarding disadvantages of smoking during pregnancy?

A. Small for gestational age.

B. Increased risk of sudden infant death syndrome.

C. Increased risk of developing small teeth with faulty enamel.

D. Higher perinatal mortality.

E. Developmental delay of the baby at least in early years.

Incorrect. Correct answer is C


45% answered correctly

Explanation:

Correct Answer Is C

Effects of tobacco smoking (active or passive) during pregnancy on fetal, childhood and adult life include:

Preterm labor
Low birth weight
Intrauterine growth restriction
Birth defects – limb reduction, club foot, oral clefts, eye defects, gastrointestinal effects and many other insignificant anomalies
Sudden infant death syndrome (SIDS)
Developmental delay (at least at early stages of development)
Type II diabetes
Obesity
Hypertension
Decreased HDL
Nicotine dependence
Asthma, lower respiratory tract infections
Impaired academic performance and cognitive abilities
Conduct disorders, ADHD, antisocial behaviour

Small teeth and faulty enamel are features seen in fetal alcohol syndrome (FAS), and not smoking.

References

• http://www.racgp.org.au/afp/2014/januaryfebruary/s

• http://www.uptodate.com/contents/cigarette-smoking

Last updated:
Time spent: QID:509 2023-2-12

837 of 1943
An Aboriginal woman is in your practice for antenatal care. She is a smoker and has the history of hypertension. She does not seem to be motivated to quit smoking. You want to consult her on harms of smoking to her child, which one of the
following is not an effect smoking on the fetus during pregnancy if she continues to smoke?

A. Still birth.

B. Premature birth.

C. Placental abruption.

D. Hydrops fetalis.

E. Spontaneous abortion.

Incorrect. Correct answer is D


45% answered correctly

Explanation:

Correct Answer Is D

Effects of tobacco smoking (active or passive) during pregnancy include:

Obstetric complications

Spontaneous abortion and miscarriage


Placental abruption (double the risk for smokers who consume more than 20 cigarettes a day)
Placenta previa
Preterm birth (<37 weeks)
Pre-eclampsia
Premature rupture of membranes
Still birth
Ectopic pregnancy

Fetal complications

Low birth weight


Intrauterine growth restriction
Birth defects – limb reduction, club foot, oral clefts, eye defects, gastrointestinal defects and many other less insignificant anomalies

Childhood and adult life complications:

Sudden infant death syndrome (SIDS)


Developmental delay (at least at early stages of development)
Type II diabetes
Obesity
Hypertension
Decreased HDL
Nicotine dependence
Asthma, lower respiratory tract infections
Impaired academic performance and cognitive abilities
Conduct disorders, ADHD, antisocial behavior

Hydrops fetalis is a serious fetal condition, defined as abnormal accumulation of fluid in two or more fetal compartments, including ascites, pleural effusion, pericardial effusion, and skin edema. Hydrops fetalis can be caused by the following
conditions:

Hemolytic disease of the newborn


Severe anemia
Chromosomal abnormalities
Congenital heart disease

Smoking is not a cause of hydrops fetalis.

References

• RACGP - AFP - Management of smoking in pregnant women

• UpToDate - Cigarette and tobacco products in pregnancy: Impact on pregnancy and the neonate

• Medscape - Pediatric Hydrops Fetalis

Last updated:
Time spent: QID:510 2023-2-12

838 of 1943
A 33-year-old woman presents at 18th week of her pregnancy complaining of fishy-smelling, thin, white homogenous and offensive vaginal discharge. A sample of the discharge is positive for clue cells under light microscopy. Which one of the
following statements is true regarding this condition?

A. Metronidazole is contraindicated in this patient.

B. There is a relapse rate of over 50% within 6 months.

C. The condition is a sexually transmitted disease (STD).

D. Her sexual partner needs to be treated as well.

E. Reassurance is the only action required.

Incorrect. Correct answer is B


45% answered correctly

Explanation:

Correct Answer Is B

The clinical case is a typical description of bacterial vaginosis (BV). This condition is caused by overgrowth of predominantly anaerobic organisms such as Gardnerella vaginalis. Although bacterial vaginosis is exclusively seen in sexually active
women (or sometimes after menopause), it is not a sexually transmitted disease.

BV is suspected in the presence of ‘fishy-smelling’ thin white vaginal discharge. The criteria (Amsel’s criteria) for diagnosis of BV includes:

Clue cells on microscopy


Vaginal pH> 4.5
Positive whiff test: addition of potassium hydroxide to the discharge results in a fishy odor

Since the cause of BV is unclear, current treatment is directed toward alleviation of symptoms and restoration of normal flora rather than eradication of a specific etiologic agent. Internationally recommended therapies include:

First-line: 7 days of oral metronidazole (400 mg twice daily) or,


Second-line: Vaginal clindamycin (1 g at night)

These antibiotics are associated with cure rates of 70–90%, but there is a recurrence in more than 50% of patients within 6 months of treatment. Single dose therapies such as 2g metronidazole have been shown to be less effective than 7-day course
of metronidazole.

Australian national guidelines recommend either clindamycin 300 mg orally 12 hourly for 7 days (category A) or metronidazole 400 mg orally 12 hourly for 7 days (category B2). Oral therapy is currently recommended over vaginal therapy, as there is
some concern that topical therapy may not be effective against BV organisms in the endometrial cavity; however, this has not been supported by recent clinical trials.

NOTE – clindamycin (300 mg orally, 12 hourly for 7 days) is first-line medication for treatment of BV during pregnancy, but metronidazole (400 mg orally, 12 hourly for 7 days) can also be used safely as the second-line therapy.

Bacterial vaginosis is only seen in women, and men will never have it; therefore, treating the male partner is not necessary.

Symptomatic bacterial vaginosis in non-pregnant women and even asymptomatic BV in pregnant women need treatment, and reassuring the patient is not a correct option. BV in pregnancy may lead to obstetric complications and pelvic infections.

References

• RACGP - Bacterial vaginosis

Last updated:
Time spent: QID:511 2023-2-12

839 of 1943
A 27-year-old woman presents to the emergency department of the hospital after giving birth to a healthy male baby four days ago with complaint of fever. During the vaginal delivery, she sustained small vaginal laceration. The lesion was small and
did not require sutures for repair. She is breastfeeding her baby. On examination, no uterine tenderness is noted. The rest of the exam is unremarkable. Which one of the following can be the most likely cause of her fever?

A. Infection of the unrepaired vaginal laceration.

B. Endometritis.

C. Urinary tract infection (UTI).

D. Breast engorgement.

E. Atelectasis.

Correct
45% answered correctly

Explanation:

Correct Answer Is A

The time of fever (4th day postpartum) and absence of uterine tenderness on exam makes infection of the vaginal laceration the most likely cause of this presentation.

(Option B) With endometritis, exquisite uterine tenderness was expected. However, sometimes malodorous lochia is the only finding in addition to fever. Furthermore, endometritis would be expected earlier on days 2-3 postpartum (not a strict rule).

(Option C) There are no urinary symptoms suggesting UTI as the likely cause of the presentation. Moreover, UTI is often expected on days one or two postpartum.

(Option D) It is too soon for breast engorgement to develop (7-21 days). Besides, there are no findings suggesting breast engorgement as the likely cause of the fever.

(Option E) Atelectasis is expected during the first 24 hours postpartum.

TOPIC REVIEW

Postpartum fever

Postpartum fever is a common complication, with a frequency of approximately 5-7% of births and the majority of occurrences happening more than two days after birth.

Postpartum fever is defined as a temperature of 38.7°C or greater for the first 24 hours, or greater than 38.0°C of any two of the first 10 days postpartum.

Local spread of colonized bacteria is the most common etiology for postpartum infection following vaginal delivery.

The following conditions are associated with increased risk of postpartum infections and fever:

History of cesarean delivery


Premature rupture of membranes
Frequent cervical examination (sterile gloves should be used in examinations. Other than a history of cesarean delivery, this risk factor is most important in postpartum infections.)
Internal fetal monitoring
Preexisting pelvic infection including bacterial vaginosis
Diabetes
Malnutrition
Obesity

Significant causes of postpartum fever, in order of temporal occurrence after delivery, are summarised in the following table:

Cause of postpartum fever Expected time of onset (days postpartum)


Atelectasis 0 (within the first 24 hours)
Urinary tract infections / pyelonephritis 1-2
Endometritis 2-3
Wound infection 4-5
Septic thrombophlebitis 5-6
Mastitis 7-21

Atelectasis (and resultant respiratory tract infections)

Clinical picture include tachypnea, rales, crackles, rhonchi, and consolidation. General anesthesia is the most common risk factor, followed by cigarette smoking.

No chest X-ray is required.

Management includes respiratrory toliet and postural drainage.

Urinary tract infection (UTI)

Patients with pyelonephritis or UTIs may have costovertebral angle tenderness, suprapubic tenderness, and an elevated temperature.

Bacteria most frequently found in UTIs are normal bowel flora, including E.coli, Klebsiella, Proteus, and Enterobacter species. Any form of invasive manipulation of the urethra (e.g. Foley catheterization) increases the likelihood of a UTI. UIT is more
commonly associated with repeated catheterization and vaginal exam in prolonged labor.

840 of 1943
Endometritis

Endometritis is the most common infection during the postpartum period and is characterized by lower abdominal tenderness on one or both sides of the abdomen, adnexal and parametrial tenderness elicited with bimanual examination, and
temperature elevation (most commonly >38.3°C).

Some women have foul-smelling lochia without other evidence of infection. Some infections, most notably caused by group A beta-hemolytic streptococci, are frequently associated with scanty, odourless lochia.

The mode of delivery is the single most important factor in the development of endometritis. The risk increases dramatically after cesarean delivery. However, there is some evidence that hospital readmission for management of postpartum
endometritis occurs more often in those who delivered vaginally.

Other risk factors include prolonged rupture of membranes, prolonged use of internal fetal monitoring, anemia, and lower socioeconomic status.

Perioperative antibiotics have greatly decreased the incidence of endometritis.

In most cases of endometritis, the bacteria responsible are those that normally reside in the bowel, vagina, perineum, and cervix.

The uterine cavity is usually sterile until the rupture of the amniotic sac. As a consequence of labor, delivery, and associated manipulations, anaerobic and aerobic bacteria can contaminate the uterus.

Management includes provision of antibiotic coverage against polymicrobial infection. The recommended regimen is ampicillin/amoxicillin + gentamicin + metronidazole (all intravenously)

Wound infections

Patients with wound infections, or episiotomy infections, or traumatic vaginal lacerations have erythema, edema, tenderness out of proportion to expected postpartum pain, and discharge from the wound or episiotomy site.

Drainage from wound site should be differentiated from normal postpartum lochia and foul-smelling lochia suggestive of endometritis.

Most often, the causative organisms associated with perineal cellulitis and episiotomy site infections are Staphylococcus or Streptococcus species and gram-negative organisms, as in endometritis. Vaginal secretions contain as many as 10 billion
organisms per gram of fluid. Yet, infections develop in only 1% of patients who had vaginal tears or who underwent episiotomies.

Those who underwent cesarean delivery have a higher readmission rate for wound infection and complications than those who delivered vaginally.

Wound toilet, abscess drainage and broad spectrum antibiotics are mainstay of treatment.

Septic pelvic thrombophlebitis

Numerous factors cause pregnant and postpartum women to be more susceptible to thrombosis. Pregnancy is known to induce a hypercoagulable state secondary to increased levels of clotting factors. Also, venous stasis occurs in the pelvic veins
during pregnancy.

Patients with septic pelvic thrombophlebitis, although rare, may have palpable pelvic veins. These patients also have tachycardia that is out of proportion to the fever. Septic pelvic thrombosis and thrombophlebitis are rather rare cause of postpartum
fever.

Broad-spectrum antibiotics should be administered. Initial choice of antibiotics should cover gram-positive, gram-negative, and anaerobic organisms. Ampicillin and gentamicin with metronidazole or clindamycin is a common regimen.

Anticoagulation either with unfractionated heparin (UFH) or low molecular weight heparin (LMWH) may be used. There is no universal guideline or recommendation for anticoagulation therapy in septic pelvic thrombosis.

Mastitis

Patients with mastitis have very tender, engorged, erythematous breasts. Infection is frequently unilateral.

The most common organism reported in mastitis is Staphylococcus aureus. The organism usually comes from the breastfed neonate/infant's mouth or throat.

Mastitis should be treated with antibiotics to prevent abscess formation. Oral di/flucloxacillin, 6-hourly, provides good coverage against the most common causative organisms. For those with hypersensitivity to penicillin (immediate hypersensitivity
excluded) cephalexin is an alternative. For those with immediate hypersensitivity (anaphylaxis) clindamycin is used.

In patients with evidence of cellulitis and more pronounced systemic symptoms, intravenous di/flucloxacillin is the preferred. For patients with penicillin hypersensitivity (immediate hypersensitivity excluded) intravenous cefazolin/cephalothin is an
alternative. For those with immediate hypersensitivity, intravenous clindamycin can be used.

References

• http://emedicine.medscape.com/article/796892-overv

• http://www.medscape.com/viewarticle/804263

Time spent: QID:531 Last updated:


2023-2-12

841 of 1943
A 37-year-old woman presents to your practice with complaint of malodorous grey vaginal discharge. A wet mount prep under light microscopy reveals bacteria attached to vaginal epithelial cells. Which one the following organisms could be the
most likely pathogen?

A. Chlamydia.

B. Trichomonas.

C. Candida albicans.

D. Gardnerella vaginalis.

E. Mycoplasma hominis.

Incorrect. Correct answer is D


45% answered correctly

Explanation:

Correct Answer Is D

The malodorous greyish discharge and more importantly the bacteria attached to vaginal epithelial cells are suggestive of bacterial vaginosis (BV) as the most likely diagnosis.

BV represents a complex change in the vaginal flora characterized by a reduction in concentration of normally dominant hydrogen-peroxidase producing lactobacilli and an increase in concentration of other organisms, especially anaerobic gram
negative rods, such as:

Gardnerella vaginalis
Prevotella species
Porphyromonas species
Bacteroids
Peptostreptococcus species
Mycoplasma hominis
Ureaplasma urealyticum
Mobiluncus species
Fusobacterium species
Atopobium vaginae

Increasing evidence suggests that Gardnerella vaginalis plays the most important role in pathogenesis of BV.

Most of women with BV are asymptomatic. The symptoms, if present, include a grayish white, thin, and homogenous discharge with the unpleasant ‘fishy smell’ that may be more noticeable after sexual intercourse (when the semen has made the
vagina more alkaline) and during the menses.

BV alone does not cause dysuria, dyspareunia, pruritus, burning, or vaginal inflammation. The presence of these symptoms suggests mixed vaginitis.

Although gram stain is the gold standard for diagnosis of BV, the diagnosis is usually based on Amstel criteria, which are simple and useful in an office practice where microscopy is available. The first 3 findings are sometimes also present in
Trichomoniasis:

1. Homogeneous, thin, grayish-white discharge that smoothly coats the vaginal walls
2. Vaginal pH >4.5
3. Positive whiff-amine test, defined as the presence of a fishy odor when a drop of 10% potassium hydroxide (KOH) is added to a sample of vaginal discharge
4. Clue cells on saline wet mount - clue cells are vaginal epithelial cells studded with adherent coccobacilli that are most evident at the edge of the cell. For a positive result, at least 20% of the epithelial cells on wet mount should be clue
cells. The presence of clue cells diagnosed by an experienced microscopist is the single most reliable predictor of BV.

References

• RACGP - Bacterial vaginosis

• Medscape - Vaginitis

Last updated:
Time spent: QID:533 2023-2-12

842 of 1943
A 38-year-old primigravida woman is admitted to the Labor Unit at 39 weeks gestation due to the onset of regular uterine contractions. Her cervix is 8 cm dilated and 100% effaced, with the fetus’ vertex at +1 station. The fetal heart rate is 150 bpm.
As the labor progresses, the fetal heart rate falls to 80 bpm without any changes in the mother’s general condition. Which one of the following would be the next best step in management?

A. Fetal scalp blood sampling.

B. Cardiotocography.

C. Vaginal examination to exclude any cord prolapse.

D. Emergency delivery by cesarean section.

E. Percutaneous umbilical sampling.

Incorrect. Correct answer is B


45% answered correctly

Explanation:

Correct Answer Is B

The case describes fetal bradycardia detected on fetal heart auscultation. Bradycardia of <100bpm for more than five minutes or <80 bpm for more than three minutes is always considered abnormal.

Severe bradycardia is caused by:

Prolonged cord compression


Cord prolapse
Epidural and spinal anesthesia
Maternal seizures
Rapid fetal descent

In clinical situations where the fetal heart rate pattern is considered abnormal, immediate management should include:

Identification of any reversible cause of the abnormality and initiation of appropriate action (i.e., maternal repositioning, correction of maternal hypotension, rehydration with intravenous fluid, cessation of oxytocin, tocolysis for
excessive uterine activity, and initiation or maintenance of continuous CTG.

Consideration of further fetal evaluation or delivery if a significant abnormality persists.

In this scenario where the baby is 1+ station and abnormal fetal heart rate is detected on auscultation the next step would be performing a confirmatory cardiotocography (CTG). If CTG confirms the condition despite initial measures mentioned
above, prompt action should be taken. If CTG indicates fetal compromise, measures such as fetal scalp blood sampling (option A), or emergency delivery by caesarean section (option D) can be undertaken.

With the mother having normal general condition, cord compression or prolapse should come on the top of the differential diagnoses list, but since the cervix is 8 cm dilated, 100% effaced and the fetal head is already engaged, cord prolapse would
be unlikely; therefore, repeating vaginal exam (option C) is not as important as confirmatory CTG; however, if the scenario indicated cord prolapse as a possibility, a vaginal exam should have been performed to exclude cord compression or prolapse.

In case a blood sample of the fetus is required, scalp blood sampling is the preferred method. Percutaneous blood sampling (option E) is not routinely performed.

NOTE - In cases of severe prolonged bradycardia, if the cause cannot be identified and corrected, immediate delivery is recommended.

References

• RANZCOG Intrapartum Fetal Surveillance Clinical Guideline

• WA Health Department – Obstetrics and Gynaecology Guidelines

Last updated:
Time spent: QID:94 2023-2-12

843 of 1943
A 28-year-old woman is admitted to the Maternity Unit in early labor. She has had an uncomplicated pregnancy to date and is otherwise healthy. She asks you about fetal monitoring during labor and says she has heard that cardiotocography (CTG) is
helpful for assessment of the baby’s wellbeing and preventing fetal problems. In counselling regarding to the use of CTG as a predictor of fetal outcome and satisfactory labor in comparison to intermittent auscultation and whether CTG monitoring
will reduce the risk of neonatal developmental abnormalities, which one of the following would be the most appropriate advice?

A. There is no evidence to support admission CTG.

B. CTG is a predictor of previous fetal oxygenation.

C. Compared to intermittent auscultation, CTG is superior in monitoring the baby’s wellbeing.

D. CTG is a good predictor of the fetal outcome.

E. CTG will reduce the risk of neonatal developmental abnormalities.

Correct
45% answered correctly

Explanation:

Correct Answer Is A

Continuous fetal heart rate monitoring with CTG is mandatory in all high-risk pregnancies but has been shown to have limited value in low-risk pregnancies. Intermittent fetal heart auscultation has the same efficacy in such pregnancies.

In fact, CTG in low-risk pregnancies, by recognizing abnormalities that are of little if any significance, leads to unnecessary obstetrical interventions such as instrumental delivery or cesarean section.

CTG has not reduced the incidence of neonatal developmental abnormalities such as cerebral palsy. CTG is a predictor of the current fetal condition, not the previous, unless there are severe abnormalities as soon as the CTG is applied on admission.

In conclusion, there is no evidence to support routine admission CTG.

References

• NCBI - PubMed - Cardiotocography versus intermittent auscultation of fetal heart on admission to labour ward for assessment of fetal wellbeing.

Last updated:
Time spent: QID:534 2023-2-12

844 of 1943
Christina, 38 years old, presents for discussion about her risk of developing ovarian cancer. Her detailed history shows that she is the mother of 4 children, all breastfed. She used oral contraceptive pills for 5 years after the birth of her last child. Two
year ago, she underwent hysterectomy to ‘get rid of OCPs’. Apart from a BMI of 31, she is otherwise healthy on examination. Which one of her conditions is considered a risk factor for developing ovarian cancer?

A. History of oral contraceptive pills.

B. Hysterectomy.

C. Her body mass index.

D. Breastfeeding.

E. Multiparity.

Incorrect. Correct answer is C


45% answered correctly

Explanation:

Correct Answer Is C

The following are associated with INCREASED risk of ovarian cancer:

Family history of ovarian cancer - Family history of ovarian cancer is a strong risk factor, present in approximately 10-15% of women who develop the disease. The risk of ovarian cancer is increased when the family history suggests a
sporadic case, but is substantially greater when there is a hereditary cancer syndrome.
Familial ovarian cancer syndromes - Clues to the presence of a hereditary cancer syndrome include the presentation of ovarian cancer in a first- or second-degree family member (both maternal and paternal lineage) at an early age
(under 50 years) or the occurrence of ovarian or related cancers in multiple members (i.e. 2 to 4 generations). Familial ovarian cancer syndromes are uncommon, accounting for 5-10% of ovarian cancer cases. These hereditary
syndromes include:
Lynch II syndrome (cancers of the colon, breast, endometrium, and ovary with hereditary nonpolyposis colorectal cancer or HNPCC)
Breast-ovarian cancer syndrome (usually associated with a BRCA1 or BRCA2 mutation) – the most common hereditary ovarian cancer syndrome.
Age - The incidence of ovarian cancer increases with age.
Infertility or Nulliparity
Early menarche or late menopause - these factors are associated with greater numbers of ovulation that are associated with increased risk of ovarian cancer
Endometriosis
Premenopausal or postmenopausal hormone replacement therapy (HRT)
Obesity – The mechanism through which obesity increases the chance of contracting ovarian cancer is increased oestrogen production by adipose tissue.

The risk of ovarian cancer is DECREASED in women with history of:

Pregnancy –the more the number of pregnancies the less the chance of ovarian cancer
Use of the oral contraceptive pill
Breastfeeding
Tubal ligation or hysterectomy

Of the given options, the BMI of this woman (obese) is a risk factor for developing ovarian cancer.

Oral contraceptive pills, multi-parity, breastfeeding and hysterectomy are protective factors against ovarian cancer.

References

• http://emedicine.medscape.com/article/255771-overv

• http://www.uptodate.com/contents/screening-for-ova

Last updated:
Time spent: QID:535 2023-2-12

845 of 1943
A young woman with history of valvular heart disease comes to your clinic for advice regarding pregnancy. She is planning to become pregnant. Which one of the following, if present, can lead to most significant complications during pregnancy?

A. Mitral valve regurgitation.

B. Mitral valve stenosis.

C. Aortic valve regurgitation.

D. Aortic valve stenosis.

E. Tricuspid valve regurgitation.

Incorrect. Correct answer is B


45% answered correctly

Explanation:

Correct Answer Is B

Of the given options, mitral valve stenosis is the cardiac condition associated with the most significant complications during pregnancy.

The pregnancy-induced increase in blood volume, cardiac output, and tachycardia can increase the trans-mitral pressure gradient and cause pulmonary edema in women with mitral valve stenosis.

Pregnancy associated with long-standing mitral valve stenosis may result in pulmonary hypertension. On the other hand, pregnant women with mitral stenosis are at increased risk for the development of atrial fibrillation and other tachyarrhythmias.

Mitral valve stenosis, grade III or IV heart failure, and atrial fibrillation are the most significant cardiac conditions with high risk for complications..

Mitral valve regurgitation (option A), aortic valve regurgitation (option C), aortic valve stenosis (option D) and tricuspid valve regurgitation (option E) are often well tolerated during pregnancy, as the pregnancy-induced decrease in systemic vascular
resistance reduces the risk of cardiac failure with these conditions.

References

• KEMH - Clinical guidelines

• Medscape - Cardiovascular Disease and Pregnancy

Last updated:
Time spent: QID:536 2023-2-12

846 of 1943
A 25-year-old woman presents to the Emergency Department with an episode of migraine headache. She is 33 weeks pregnant and has had migraine since she was 18 years old. She has vomited twice in the past two hours and is nauseous now.
Which one of the following would be the most appropriate management?

A. Paracetamol.

B. Paracetamol and metoclopramide.

C. Regular mefenamic acid until delivery.

D. Dihydroergotamine.

E. Propranolol.

Incorrect. Correct answer is B


45% answered correctly

Explanation:

Correct Answer Is B

Migraine attacks often disappear or diminish during the second and third trimesters. However, a small number of women develop migraine during pregnancy. Because of the potential risks to the fetus, drug treatment options are limited and include:

Paracetamol (first-line) - For acute treatment, paracetamol is safe but often inadequate to control the symptoms. Migraine not responding to paracetamol alone may be relieved with combination therapy such as paracetamol (650 to 1000 mg) and
metoclopramide (10 mg), or paracetamol-codeine.

For migraine with nausea and vomiting, metoclopramide (category A) can be safely added. Prochlorperazine can be used as an alternative to metoclopramide.

Women with migraine that has not responded to these drugs after several days should be evaluated for provoking factors and treated more aggressively with the following medications in a step-wise approach.

NSAIDs and aspirin (second-line) - NSAIDs are second-line options and safest in the second trimester. In the first trimester, an association with miscarriage and some birth defects (ventricular septal defect, gastroschisis) has been suggested. In the
third trimester, their use should be limited to fewer than 48 hours due to concerns about the premature closure of ductus arteriosus, platelet inhibition, and oligohydramnios.

Opioids (third-line) - Opioids (e.g., oxycodone, meperidine, morphine, etc.) can be given by rectal, intravenous, or intramuscular administration. Opioids can be useful for treatment of women with nausea and vomiting.

Triptans (fourth-line) - For moderate to severe symptoms in patients, who do not respond to other drugs, triptans can be considered.

NOTE – Ergotamine is absolutely contraindicated throughout pregnancy.

Treatment of this patient should be started with paracetamol as the firs-line medication, but since she has nausea and vomiting as well, metoclopramide should be added.

(Option A) Paracetamol alone is unlikely to help with nausea and vomiting.

(Option C) Mefenamic acid is an NSAID and should be avoided in the third trimester. If inevitable, it should not be used for more than 48 hours.

(Option D) Ergotamine is absolutely contraindicated throughout pregnancy.

(Option E) Propranolol can be used with extreme caution for prophylaxis in selected cases with recurrent severe migraine attacks. Its use has no role in acute management of migraine.

References

• AAFP - Treatment of Acute Migraine Headache

• UpToDate - Headache in pregnant and postpartum women

• Australian Prescriber - Treatment of nausea and vomiting in pregnancy

• Therapeutic Guidelines – Neurology

Last updated:
Time spent: QID:537 2023-2-12

847 of 1943
Which one the following is unlikely to require RhoGAM (anti-D immunoglobulin) administration in an Rh negative woman, who has not actively formed their own Anti-D?

A. Threatened abortion before 12 weeks' pregnancy.

B. Spontaneous abortion.

C. External cephalic version.

D. Significant abdominal blunt trauma.

E. Termination of pregnancy.

Correct
45% answered correctly

Explanation:

Correct Answer Is A

There is insufficient evidence to suggest that a threatened abortion before 12 weeks gestation requires Anti-D antibody unless the bleeding is significant. Royal college of Obstetricians and Gynaecologists (UK) recommends that RhoGAM is not
required in cases of threatened abortion in the first 12 weeks if there is no significant bleeding. This recommendation is endorsed and practiced by Royal Australian and New Zealand College of Obstetricians and Gynaecologists.

TOPIC REVIEW

An Rh-negative mother has no antibodies against Rh (D) antigen. If she is pregnant and the fetus is Rh-positive any mixing up the fetus’ blood to the mother’s will trigger an immune response by lymphocytes and will lead to production of anti-Rh
antibodies in the mother’s blood (anti-D antibody). RhoGAM® is anti-D IgG passive antibody that will eliminate the D-antigen from the mother’s blood, before the mother’s immune system start to sensitize. It is administered intramuscularly (IM).

RhoGAM is available in two forms:

CLS-250 IU – one dose contains 50μg of anti-D antibody


CLS-265 IU – one dose contains 125μg of anti-D antibody

Each 300μg of anti-D antibody neutralizes 15 mL of fetal packed red cells (30 mL of whole blood)

RhoGAM should be administered within the first 72 hours after the precipitating even, however, with much less success rate it can be given up to 9-10 days.

INDICATIONS:

First trimester indications (up to and including 12 weeks):

Spontaneous abortion (complete, incomplete, or missed abortion) probably excluding threatened abortion
Induced abortion (medically or by D&C)
Ectopic pregnancy
Chorionic villous sampling (this procedure is performed at 10-12 weeks)
Molar pregnancy – chorionic villi may contain D-antigen

Management – in the first 12 weeks the maximum amount of fetal blood that can mix with mother’s is 2.5 mL of RBC (5 mL whole blood). A single dose of CLS 250-IU is sufficient for neutralization of circulating fetal D-antigen. Multifetal pregnancies
need extra doses. In a singleton pregnancy Kleihauer-Betki test is not indicated because the amount of fetomaternal blood mix is not significant and a single dose CLS – 250 will be enough.

Second and third trimester indications:

Spontaneous or induced abortions of all kind after 12 weeks


Amniocentesis (this procedure is performed > 15 weeks)
Cordocentesis (this procedure is performed >20 weeks)
Fetal blood sampling
Fetal death
External cephalic version of breech presentation (successful or unsuccessful)
Blunt abdominal trauma in pregnancy considered sufficient to cause fetomaternal haemorrhage
Antepartum haemorrhage (revealed or concealed) in the second or third trimester (e.g. placental abruption, placenta praevia, etc)

Management – in the second or third trimester CLS 625 (containing 125μg anti D-antibody) should be used. A Kleihauer-Betki test is indicated in the second or third trimester events to assess the need for extradoses of CLS 625 IU. Ideally the sample
for the test should be collected within 15 minutes of the precipitating event, but if not possible it can be performed up to 72 hours.

NOTE (1) - RhoGAM will remain in maternal circulation for up to 6 weeks.

NOTE (2) - RhoGAM should not be given to women in which anti-D antibody has already formed.

Routine administration of RhoGAM – in the absence of a precipitating event, RhoGAM (CLS – 625 IU) is routinely given to all Rh-negative pregnant women at 28 weeks and 34 weeks of pregnancy.

Within the first 72 hours postpartum all R-negative women whose baby is Rh-positive should receive a dose of CLS-625 IU and undergo quantification of fetomaternal blood mix using Kleihauer-Betki test to evaluate whether extra doses of CLS-625 IU
are indicated.

References

• http://www.australianprescriber.com/magazine/23/2/

• https://www.nhmrc.gov.au/_files_nhmrc/publications

• Royal Australian and New Zealand College of Obstetricians and Gynaecologists – College Statement C-Obs 6 (2015)

L t d t d

848 of 1943
Last updated:
Time spent: QID:538 2023-2-12

849 of 1943
A 38-year-old Rh-negative woman was given Anti-D antibody (RhoGAM®) at 28 and 34 weeks of pregnancy three years ago during her first pregnancy, and extra postpartum doses after the baby was found to be Rh-positive. Six month ago, she
underwent an emergency orthopedic surgery and received blood transfusion. She has had no miscarriages since then. She is planning to become pregnant again. In the first antenatal visit, anti-D antibody level is found to be positive. Which one of the
following is most likely to be the cause of sensitzation to D antigen in this woman?

A. Occult fetomaternal bleeding.

B. Blood transfusion.

C. Previous pregnancy.

D. RhoGAM injection.

E. Orthopedic surgery.

Incorrect. Correct answer is B


45% answered correctly

Explanation:

Correct Answer Is B

An Rh-negative woman with an Rh-positive fetus is likely to become sensitized against fetal D antigen in the following situations:

Delivery
Spontaneous abortion (complete, incomplete, missed or threatened abortion)
Induced abortion (medically or by D&C)
Ectopic pregnancy
Chorionic villous sampling, amniocentesis, cordocentesis, or fetal blood sampling
Fetal death
External cephalic version of breech presentation (successful or unsuccessful)
Blunt abdominal trauma in pregnancy considered sufficient to cause fetomaternal hemorrhage
Antepartum hemorrhage (overt or concealed) in the second or third trimester (e.g. placental abruption, placenta previa, etc)
Molar pregnancy

Timely administration of adequate anti D immunoglobulin, removes fetal D-antigen from maternal circulation before sensitization occurs in the mother.

This patient has received RhoGAM during her previous pregnancies and has had no abortion or any obstetrical procedures to pose the risk of fetomaternal blood mixing. Therefore, the only likely cause of exposure to Rh antigen can be transfusion of
mismatched blood during the orthopedic surgery.

References

• http://www.australianprescriber.com/magazine/23/2/

• https://www.nhmrc.gov.au/_files_nhmrc/publications

Last updated:
Time spent: QID:539 2023-2-12

850 of 1943
A 42 year-old woman presents to your office complaining of marked lack of energy, breast tenderness and insomnia. These symptoms start a week before the onset of menses. She is severely depressed as she has been unable to concentrate at
work during these times. Her symptoms improve after the onset of menses. She neither smokes nor drinks alcohol. She has no history of any mental illness in the past. Which one of the following would be the most appropriate treatment?

A. Evening primrose oil.

B. Fluoxetine.

C. Oral contraceptive pills.

D. Non-steroidal anti-inflammatory drugs.

E. Bromocriptine.

Incorrect. Correct answer is B


45% answered correctly

Explanation:

Correct Answer Is B

With severe depression, lack of energy, difficulty concentrating, insomnia, and breast tenderness this patient has 5 out of 11 symptoms of premenstrual dysphoric disorder (PMDD). The symptoms are affecting her work (lack of concentration at
work), are related to menstruation, and have been present for more than 2 cycles. Furthermore, these symptoms cannot be attributed to any other underlying psychiatric conditions as the history suggests. With these, this patient has PMDD and
selective serotonin reuptake inhibitors (SSRIs) are the most effective (not first line) medications for treatment. This does not mean that SSRIs should be start immediately because every pharmacological approach should be considered if
conservative measures such as CBT, regular exercise, life style modification and supplementation with vitamin E, B6, calcium and magnesium proved ineffective after at least 2-3 menstrual cycles.

Systematic review of SSRIs for PMDD concluded that these drugs are effective. Fluoxetine 20mg/day has been shown to decrease the symptoms of PMS and PMDD with an overall response of 60-75%. Sertraline (50-150mg/day), paroxetine (20-
30mg/day) and citalopram (20-30mg/day) can be used with aboout the same efficacy. Higher doses are associated with increased incidence of adverse effects and no added benefit. Adverse effects of SSRIs occur in 15% of the patients and include,
but not limited to, headache, anxiety, nausea and impaired sexual function. Sexual dysfunction including anorgasmia and decreased libido may be the most problematic adverse effects.

Some women who do not respond to one SSRI may respond to a second SSRI and it is worth attempting this switch in non-responders. Additionally, women who do not respond completely to intermittent therapy may benefit from daily therapy and
some women who undergo daily treatment may do better with intermittent treatment.

GnRH antagonists and danazol would be considered as next step if SSRIs are ineffective. The rationale behind their use is the fact that ovarian steroids have been proved to be the underlying cause of PMS and PMDD through a rather unknown
mechanism.

GnRH antagonists and danazol, by suppression of ovarian steroid production, decrease the symptoms. Their adverse effect profile, however, makes them a last-resort therapy for PMS and PMDD. Acne and hirsutism are the most common
complications.

Vitamins such as B6, D, and E have been studied with controversial results. Despite lack of solid evidence these agents can be used, as well as regular exercise and healthy life style, as the first –line treatment for mild to moderate PMS. PMDD,
however, demands more prompt and aggressive treatment with SSRIs as the first-line treatment option.

Oral contraceptives may have an effect on reducing the symptoms of PMS and are the simplest method of inducing anovulation. Older combined oral contraceptives were not more effective than placebo in several studies; however, new meta-
analysis suggests that an oral contraceptive containing the progestin, drospirenone, and use of a 4 rather than 7 day pill-free interval, is more effective than placebo for symptom relief in women with PMDD. The long-term safety and efficacy of this
compound for treating PMS/PMDD are not known. Oral contraceptives may be used as first-line treatment if only physical symptoms of PMS are of great concern and significance.

NSAIDs are ineffective in treatment of PMDD, but they occasionally might be considered for aches and pain associated with PMS.

Evening prim rose oil has no effect over placebo in PMS overall, but studies has shown that it can be effective in management of cyclic mastalgia.

TOPIC REVIEW

Premenstrual dysphoric disorder (PMDD)

PMDD is severe form of premenstrual syndrome and is characterized by severe feeling of sadness, emotional labiality with frequent tearfulness, loss of interest in daily activities, decreased concentration, fatigue, insomnia and feeling of being
overwhelmed or out of control. Symptoms are more related to affections rather than being somatic. In other words, affective impairment is the dominating feature.

The Diagnostic and Statistical Manual of Mental Disorders, Fifth Edition (DSM-5), established 4 research criteria (A through D) for the diagnosis of PMDD.

Criterion A - in most menstrual cycles during the past year, at least 5 of the following 11 symptoms (including at least 1 of the first 4 listed) were present:

1. Markedly depressed mood, feelings of hopelessness, or self-deprecating thoughts


2. Marked anxiety, tension, feelings of being 'keyed up' or 'on edge'
3. Marked affective lability (e.g. feeling suddenly sad or tearful or experiencing increased sensitivity to rejection)
4. Persistent and marked anger or irritability or increased interpersonal conflicts
5. Decreased interest in usual activities (e.g. work, school, friends, and hobbies)
6. Subjective sense of difficulty in concentrating
7. Lethargy, easy fatigability, or marked lack of energy
8. Marked change in appetite, overeating, or specific food cravings
9. Hypersomnia or insomnia
10. A subjective sense of being overwhelmed or out of control
11. Other physical symptoms, such as breast tenderness or swelling, headaches, joint or muscle pain, a sensation of bloating, or weight gain

The symptoms must have been present for most of the time during the last week of the luteal phase, must have begun to remit within a few days of the onset of menstrual flow, and must be absent in the week after menses.

Criterion B - the symptoms must be severe enough to interfere significantly with social, occupational, sexual, or scholastic functioning. For example, the patient may avoid social activities or exhibit decreased productivity and efficiency at work or
school.

Criterion C - the symptoms must be discretely related to the menstrual cycle and must not merely represent an exacerbation of the symptoms of another disorder, such as major depressive disorder, panic disorder, dysthymic disorder, or a personality
disorder (although the symptoms may be superimposed on those of any of these disorders).

Criterion D- criteria A, B, and C must be confirmed by prospective daily ratings during at least 2 consecutive symptomatic menstrual cycles. The diagnosis may be made provisionally before this confirmation.

Of the 11 symptoms listed in DSM-5, 10 are emotional and behavioral in nature; only 1 includes multiple common physical symptoms. Thus, PMDD defines a narrow group of women with the most severe premenstrual emotional symptoms, with
functional impairment, and without a concurrent Axis I or Axis II disorder that is exacerbated premenstrually.

851 of 1943
References

• Royal College of Obstetricians and Gynaecologists – Green-top Guideline – No.48

• https://www.nps.org.au/__data/assets/pdf_file/0003

• http://www.uptodate.com/contents/treatment-of-prem

• http://www.guidelines.co.uk/obstetrics_gynaecology

Last updated:
Time spent: QID:540 2023-2-12

852 of 1943
A 34-year-old woman presents with history irritability around her periods. She also mentions breast tenderness at these times. She is not sexually active and is not planning to start any relationship with a male partner in the future. Which one of the
following would be the next best step in management?

A. Hysterectomy.

B. Selective serotonin reuptake inhibitors (SSRIs).

C. Oral contraceptive pills.

D. Topical anesthetics.

E. Evening primrose oil.

Incorrect. Correct answer is E


45% answered correctly

Explanation:

Correct Answer Is E

Most women of reproductive age experience one or more mild emotional or physical symptoms for 1 to 2 days before the onset of menses. These symptoms, such as breast soreness and bloating, are mild, do not cause distress or functional
impairment, and are not considered to represent premenstrual syndrome (PMS).

In contrast, clinically significant premenstrual syndrome (PMS) is defined by the American College of Obstetricians and Gynecologists as at least one symptom associated with 'economic or social dysfunction' that occurs during the 5 days before the
onset of menses and is present in at least 3 consecutive menstrual cycles. Symptoms may be affective (e.g. anger outbursts, depression) or physical (e.g. breast pain and bloating).

Over 150 physical, behavioral, emotional, and cognitive symptoms have been attributed to PMS. Common symptoms of PMS include:

Psychological and behavioral

Mood swings and depression


Tiredness, fatigue or lethargy
Irritability
Difficulty concentrating
Sleep disturbances
Aggression
Tearfulness or feeling low
Tension or unease
Clumsiness/poor coordination
Altered libido
Food cravings/increased appetite
Loss of self-control

Physical

Bloated abdomen and fullness


Breast pain/tenderness
Headaches
Puffiness of face, abdomen or fingers
Constipation or diarrhea
General aches and pains, especially backache
Weight gain
Acne or other skin rashes
Muscle or joint stiffness
Abdominal pain/cramp
Exacerbation of epilepsy, migraine, asthma, rhinitis or urticaria.

This patient has irritability and cyclic mastalgia as her two main symptoms. Irritability is the most common psychological symptom in PMS. Of physical symptoms, abdominal bloating is the most common one (90%) followed by headache (>50%)
and mastalgia/breast tenderness (>50%)

Treatment of PMS starts with non-drug strategies (e.g. CBT, relaxation training) and lifestyle changes (e.g. regular exercise). Vitamin E, B6, calcium and magnesium have shown some efficacy and can be used in conjunction with CBT, regular exercise,
relaxation therapy, etc. This treatment should be continued for at least 2-3 cycles.

Evening primrose oil (EPO) has been shown to have no more effect on PMS symptoms in general, but it can be effective on cyclic mastalgia either in isolation or as a symptom of PMS. For cyclic mastalgia addition of EPO is an appropriate initial step
in management. EPO contains an essential fatty acid claimed to be lacking in the diet, and replacement allows for the production of prostaglandin E, which counteracts the effects of estrogen and prolactin on breasts.

If this patient was considering contraception, OCPs (option C) could be an option for her. Some evidence suggest that OCP preparations, especially those with shortened pill-free intervals can reduce physical symptoms of PMS and (including
mastalgia).

Topical anesthetics (option D) are not typically used for management of mastalgia; however, topical analgesics may benefit some patients.

SSRIs (option B) are used in treatment of women with premenstrual dysphoric disorder (PDD) where symptoms are negatively impacting the patient’s economics and/or social functioning. Even so, a trial of more conservative management with the
emphasis on lifestyle modification should be considered first.

Cyclical mastalgia (or PMS/PDD) in general are hypothesized to be caused by hormonal imbalance. Hysterectomy (option A) does not affects a woman’s hormones; therefore, not effective in treatment of such conditions.

References

• Royal College of Obstetrics and Gynaecology – Green top Guideline No.48

• RACGP - Management of benign breast conditions

• UpToDate - Breast pain

L t d t d

853 of 1943
Last updated:
Time spent: QID:541 2023-2-12

854 of 1943
A 32-year-old office secretary presents with symptoms of premenstrual syndrome associated with severe mastalgia, unresponsive to conservative measures, evening prim rose oil and analgesic. Which one of the following is the most appropriate
treatment?

A. Mefenamic acid.

B. Spironolactone.

C. Danazol.

D. Bromocriptine.

E. Vitamin E.

Incorrect. Correct answer is C


45% answered correctly

Explanation:

Correct Answer Is C

Although never first-line, danazol is the most effective treatment for cyclic mastalgia either in isolation or as a part of premenstrual syndrome (PMS) . The regimen includes 200mg danazol daily from the onset of the symptoms to the onset of
menstruation.

GnRH antagonists and bromocriptine (option D) are also very effective but not appropriate for use in severe cyclic mastalgia because their significant adverse effect profiles.

(Option A) NSAIDs such as mefenamic acid are more effective for management of dysmenorrhea than PMS.

(Option B) Spironolactone, with controversial effects, has been considered for treatment of choice for PMS if fluid retention is the dominating feature. It should be used from 3 days before the anticipated time for development of symptoms to day
1 of menstruation.

(Option E) Evidence favoring use of vitamin E for treatment of cyclic mastalgia is very weak.

TOPIC REVIEW

Approach to cyclical mastalgia often starts with non-pharmacological measures, including:

Well-fitting brassiere - this approach is widely advocated. For patients with pendulous breasts a support bra with steel underwire is more effective. In addition, use of a 'sports bra' during exercise has been shown to reduce pain related to breast
movements.

Compress - Some women obtain relief from application of warm compresses or ice packs or gentle massage.

Evening primrose oil (EPO) - this natural remedy has shown to have some effects on cyclic mastalgia (but not on PMS in general)

Medical therapy as the next step if conservative measures fail:

First line: paracetamol or NSAIDs or both can be used to relieve breast pain. Topical NSAIDs may also be useful

Second line: danazol is the only medication approved by FDA for the treatment of mastalgia with the greatest efficacy; however, it causes significant androgenic effects, which generally limits its use and result in discontinuation of treatment.

Third line: Tamoxifen can be used in patients with more severe mastalgia failing to respond to other measures; however, this medication is associated with side effects including menopause-like symptoms such as hot flushes, vaginal dryness, joint
pain, and leg cramps. Tamoxifen also increases the risk of blood clots, strokes, uterine cancer, and cataracts. Tamoxifen is infrequently used for this indication.

NOTE - Bromocriptine and GNRH antagonists have been studied, but are not advocated for use in patients with severe mastalgia because of significant adverse effects..

References

• Royal College of Obstetrics and Gynaecology – Green top Guideline No.48

• http://www.uptodate.com/contents/breast-pain

Last updated:
Time spent: QID:542 2023-2-12

855 of 1943
A 27-year-old woman presents to your practice with complaint of premenstrual abdominal bloating, irritability and uneasiness for the past 6 months. These symptoms begin 3-4-days before her anticipated periods and ease off within 1 to 2 days of
her menstrual flow. Recently, she had to take few days off work because of the intensity of her symptoms. Which one of the following is the most appropriate next step in management?

A. Relaxation therapy.

B. Sertraline 50mg daily in the second half of anticipated luteal phase.

C. Evening primrose oil.

D. NSAIDs.

E. Low dose OCP with shortened pill free intervals.

Correct
45% answered correctly

Explanation:

Correct Answer Is A

Most women of reproductive age experience one or more mild emotional or physical symptoms for 1 to 2 days before the onset of their menses. These symptoms, such as breast soreness and bloating, are mild, do not cause distress or functional
impairment, and are not considered to represent premenstrual syndrome (PMS).

In contrast, clinically significant premenstrual syndrome (PMS) is defined by the American College of Obstetricians and Gynecologists as at least one symptom associated with 'economic or social dysfunction' that occurs during the 5 days before the
onset of menses and is present in at least 3 consecutive menstrual cycles. Symptoms may be affective (e.g. irritability, anger outbursts, depression) or physical (e.g. abdominal bloating, headache, breast pain, etc).

The characteristic features in the history, as well as apparent interference of symptoms with her job (economic dysfunction) make PMS the most likely diagnosis with high index of certainty.

Management of PMS (and PMDD) always starts with non-drug measures including the following:

Cognitive behaviour therapy (CBT)


Relaxation therapy (Yoga included)
Regular exercise
Healthy lifestyle and diet - cessation of alcohol/smoking, high-fiber diet, low-glycemic index carbohydrates and cutting down on caffeine intake. Caffeine has not shown to be related to PMS but generally, intake reduction is advised.
Vitamin E, B6, calcium and magnesium

NOTE – evening primrose oil has no more effect than placebo on PMS, but studies show that it may be helpful in treatment of mastalgia as a main symptom.

Low-dose SSRIs (fluoxetine, sertraline, paroxetine and citalopram) are used if the above measures have been tried to no avail for at least 2-3 cycles. SSRIs are particularly effective if the mood and affective symptoms dominate.

SSRIs have shown greatest effectiveness, as well as most acceptable adverse effect profile among other treatment options.

Combined new-generation pill, such as Yasmin®, Cilest® (cyclically or continuously) can be tried particularly if physical symptoms are pronounced and bothersome. Their effect on PMS, however, is still controversial.

Estradiol patches, danazol and GnRH antagonists with add-back hormone replacement therapy, bilateral oophorectomy and total hysterectomy with adding back HRT and testosterone are other treatment options considered in a step-wise manner.

Of the given options, only relaxation therapy falls in the category of measures considered for initial management of PMS (and PMDD).

TOPIC REVIEW

A step-wise approach to management of PMS and PMDD

First line

Treatment Starts with non-drug strategies (e.g. CBT, relaxation training) and lifestyle changes (e.g. regular exercise). Vitamin E, B6, calcium and magnesium have shown some efficacy and can be used in conjunction with CBT, regular exercise and
healthy diet. This treatment should be continued for at least 2-3 cycles. If proved ineffective consider the next steps:

Second line

Continuous or in luteal phase (day 15–28) low-dose SSRIs – especially if psychological symptoms are of main concern.
Combined new-generation pill, such as Yasmin®, Cilest® (cyclically or continuously) – these contraceptive are not as effective of SSRs but may be considered the preferred option if the physical symptoms are bothersome. They do not
relieve mood and psychological symptoms.

Third line

Estradiol patches (100 micrograms) PLUS either oral progestogen such as duphaston or Mirena®
Higher-doses of SSRIs continuously or in luteal phase may be tried with care as evidence suggest that there are more adverse effects, with added benefit compared to lower doses

Fourth line

GnRH antagonist or danazol , as well as adding back hormone replacement therapy (either continuous or cyclical) to counteract effects of induced estrogen deficiency.

Fifth line

Total abdominal hysterectomy and bilateral oophorectomy PLUS add-back HRT (including testosterone)

NOTE – Treatment of PMS and PMDD always starts with non-pharmacological measures for at least 2-3 menstrual cycles.

References

856 of 1943
• http://www.uptodate.com/contents/premenstrual-synd

• http://www.aafp.org/afp/2002/1001/p1239.html

• https://www.nps.org.au/__data/assets/pdf_file/0003

• Royal College of Obstetricians and Gynaecologists – Green-top Guideline – No.48


Last updated:
Time spent: QID:543 2023-2-12

857 of 1943
A 28-year-old woman presents to the Maternity Unit in labor. Her last pregnancy ended in a cesarean section due to breech presentation. After delivery of her baby, spontaneous complete expulsion of the placenta is followed by 1000 ml of blood loss,
after which the woman entered a state of shock. Which one of the following is the most likely cause of her bleeding?

A. Uterine atony.

B. Cervical laceration.

C. Uterine inversion.

D. Uterine rupture.

E. Retained products of pregnancy.

Incorrect. Correct answer is D


45% answered correctly

Explanation:

Correct Answer Is D

Uterine atony, cervical lacerations, uterine inversion and retained products of pregnancy can all cause postpartum hemorrhage.

Normally after delivery, 600-800 ml of blood is lost per vagina. The plasma volume is increased by 50% during pregnancy. Assuming that a regular adult has 5000 ml of blood, during the pregnancy this volume increases to approximately 7500 ml. A
blood loss of 1000 ml amounts to approximately 13% of total blood volume. Loss of this amount is very unlikely to result in shock; therefore, a concealed source of bleeding should exist. Considering the fact that shock has occurred with only 1000
ml of blood loss, either uterine rupture or uterine inversion should be suspected; however, given the history of cesarean section and the rarity of uterine inversion, uterine rupture would be the most likely diagnosis.

In uterine rupture, there are often signs of hemorrhagic shock unjustifiable with the amount of per vaginal hemorrhage, because the blood spillage occurs into the abdominal cavity.

With uterine atony and hemorrhage to shock (option A), more blood is expected per vagina, as is in cervical or vaginal lacerations (option B).

(Option C) Uterine inversion is a potentially lethal condition that occurs when the fundus of the uterus turns inside out. Uterine inversion can be incomplete or complete. It is a rare condition. Diagnosis of uterine inversion is usually based on clinical
signs and symptoms. When there is complete inversion, the diagnosis is most easily made by palpating the inverted fundus at the cervical os or vaginal introitus. In incomplete inversion, palpating the fundal wall in the lower uterine segment and
cervix might be required for diagnosis.

(Option E) With a complete placenta, retained products of pregnancy as the underlying cause of uterine atony and postpartum bleeding is not likely.

References

• Medscape - POstpartum Hemorrhage

• UpToDate - Overview of postpartum hemorrhage

Last updated:
Time spent: QID:102 2023-2-12

858 of 1943
A 20-year-old woman presents to your clinic because she has never had a period so far. On examination, the genitalia appear completely normal. Breasts are immature, but pubic and axillary hair are present. Pelvic ultrasound shows normal uterus
and fallopian tubes, but ovaries contain small amount of connective tissue and no follicles. Which one of the following at is the most likely cause of her amenorrhea?

A. A hypothalamic lesion.

B. Mullerian agenesis.

C. Turner syndrome.

D. Hyperprolactinemia.

E. A pituitary lesion.

Incorrect. Correct answer is C


45% answered correctly

Explanation:

Correct Answer Is C

The clinical features described in the scenario are consistent with diagnosis of Turner syndrome. Turner syndrome is characterized by the Karyotype of 45-XO.

Clinical features of Turner syndrome include:

Short stature
Webbed neck
Puffy hands and feet
Coarctation of the aorta
Cardiac abnormalities
High-arched palate
Absent secondary sexual characteristics when puberty is expected

In Turner syndrome, the ovaries consist of small amounts of connective tissue and no or just a few atrophic follicles (streak gonads). The external genitalia, vagina, uterus and Fallopian tubes are well-developed at birth and anatomically normal, but
fail to function when estrogen-induced maturity should take place.

In older adolescents and adults, presenting symptoms usually involve issues of puberty and fertility as well as short stature. Adrenarche, the beginning of pubic hair growth, occurs as predicted, but is not an indication that puberty will progress
normally. Breast development is absent when ovarian failure occurs before puberty, but pubic hair growth is normal. Up to 13% of girls with Turner syndrome have spontaneous breast development or menses; some of these are XO/XX mosaics, with
normal gonadotropin responses to luteinizing hormone–releasing hormone (LHRH). Turner syndrome should be suspected in individuals with primary or secondary amenorrhea and in adult women with unexplained infertility, particularly when such
individuals also have short stature.

Approximately 30% of girls with Turner syndrome have some spontaneous pubertal development.

References

• Medscape - Turner Syndrome

• Medscape – Disorders of Sex Development

• UpToDate - Evaluation and management of primary amenorrhea

Last updated:
Time spent: QID:546 2023-2-12

859 of 1943
A 32-year-old female presented for antenatal follow-up. She is 14 weeks pregnant. On examination, the fundal height is 18 cm. which one of the following would be the next best step in management of this patient?

A. Check serum beta hCG level.

B. Perform an ultrasound scan.

C. Reassure that this is normal.

D. Refer to a gynecologist.

E. Review in four weeks.

Incorrect. Correct answer is B


45% answered correctly

Explanation:

Correct Answer Is B

The fundus is expected to be palpable above the pubis symphysis at 12 weeks gestation. At 20 weeks it is expected to be felt at the level of umbilicus. Between 12 and 20 weeks the uterus stands in between. After 20 weeks the fundal height in
centimeters is equivalent to the number of weeks of pregnancy.

A fundal height of 18cm is definitely large for a 14-week pregnant uterus. The most common cause of such discrepancy is dating errors; hence, the next best step would be performing an ultrasound scan for more accurate estimation of the
gestational age. Furthermore, ultrasonography can give additional information about the conditions that might have led to a large-for-date uterus such as polyhydramnious, multiple gestation, etc, if the case is not simply a dating error.

The most common causes of a large-for-gestational-age are:

Dating errors - the most common


Twin pregnancy
Gestational diabetes
Polyhydramnios
Gestational trophoblastic disease (molar pregnancy)

References

• http://www.health.gov.au/internet/publications/pub

Last updated:
Time spent: QID:547 2023-2-12

860 of 1943
Which one of the following is a protective factor against developing uterine fibroids?

A. Obesity.

B. Early menarche.

C. Smoking.

D. First-degree relatives with uterine fibroid.

E. Nulliparity.

Incorrect. Correct answer is C


45% answered correctly

Explanation:

Correct Answer Is C

Uterine leiomyoma (fibroid) is a benign uterine tumor commonly seen in females of reproductive age. Fibroid growth is highly dependent on estrogen.

The following are associated with INCREASED risk of uterine leiomyomas:

Black race - the condition is 2- to 3-fold greater in black women than in white women
Family history of uterine leiomyomas
Early menarche
Prenatal exposure to diethylstilbestrol
Obesity - most studies show a relationship between fibroids and increasing body mass index
Significant consumption of beef and other red meats or ham
Consumption of alcohol, especially beer (controversial)
Frequent uterine infections (controversial)
Hypertension (controversial)

The following are associated with DECREASED risk for uterine leiomyomas:

Smoking - by decreasing estrogen levels


Green vegetables and fruit (especially citrus fruit)
Parity (having one or more pregnancies extending beyond 20 weeks) - it has been hypothesized that the postpartum remodeling of the uterus may have the effect of clearing smaller fibroids
Long-acting progestogen

NOTE – Combined oral contraceptive pills (COCPs) do not seem to increase the risk of developing uterine leiomyomas.

Of the given options, smoking is associated with decreased risk of uterine leiomyomas.

References

• UpToDate - Uterine fibroids (leiomyomas): Epidemiology, clinical features, diagnosis, and natural history

Last updated:
Time spent: QID:549 2023-2-12

861 of 1943
Which one of the following is a risk factor for endometrial cancer?

A. Combined hormone replacement therapy.

B. Age more than 30 years.

C. Early menarche.

D. Early menopause before the age of 45.

E. Multiparity.

Incorrect. Correct answer is C


45% answered correctly

Explanation:

Correct Answer Is C

Risk factors for endometrial hyperplasia are essentially the same for endometrial cancer. The following table summarizes the risk factors for endometrial hyperplasia/cancer and their relative risk:

Risk factor Relative Risk (%)


Unopposed estrogen therapy 2-10
Increasing age 1.4 (50-70 years)
Late menopause (>55 years) 2
Nulliparity 2
Chronic anovulation (e.g. in PCOS) 3
Obesity 2-4
Diabetes mellitus 2
Tamoxifen therapy 2
Lynch syndrome (hereditary nonpolyposis colorectal cancer) 22-50% (lifetime risk)
BRCA1/BRCA2 gene mutation 20%
Early menarche Not specified
Estrogen secreting tumor Not specified
Family history of endometrial, ovarian, breast, or colon cancer Not specified

Of the options, only early menarche is associated with increased risk of endometrial hyperplasia and endometrial cancer.

(Option A) Combined HRT does not seem to increase the risk of endometrial hyperplasia/cancer. Estrogen-alone HRT, however, is associated with an increased risk.

(Option B) While age over 50 appears to be a risk factor, age over 30 is not a risk factor for endometrial cancer.

(Option D) Early menopause, by shortening the time of exposure of endometrium to estrogen, decreases the risk of endometrial hyperplasia and cancer.

(Option E) Multiparity is associated with decreased risk of endometrial hyperplasia.

References

• UpToDate - Overview of endometrial carcinoma

Last updated:
Time spent: QID:550 2023-2-12

862 of 1943
A 55-year-old woman presents to your clinic complaining of vaginal dryness and irritation, which has resulted in painful sexual intercourse. Speculum examination confirms the presence of atrophic vaginitis. No other abnormality is noted. She has
the history of breast cancer and is on tamoxifen. Which one of the following would be the most appropriate management?

A. Non-hormonal vaginal lubricants.

B. Vaginal estrogen cream.

C. Oral estrogen replacement therapy.

D. Combined oral contraceptive pills.

E. Progesterone-only pills.

Incorrect. Correct answer is B


45% answered correctly

Explanation:

Correct Answer Is B

The primary indication for treatment of vaginal atrophy is the presence of bothersome genitourinary symptoms related to estrogen deficiency that cause distress in women with low estrogen such as menopausal women.

As the hypoestrogen state continues, symptoms of vaginal atrophy progress and often worsen; therefore, the condition demands treatment once it is encountered. The first-line therapy for post-menopausal atrophic vaginitis is topical vaginal
estrogen creams.

With tamoxifen in the history, the patient is very likely to have had an estrogen receptor-positive tumor. Studies show that topical hormone therapy in not associated with an increased risk in patients with estrogen-dependent cancers such as breast
cancer; therefore, estrogen cream (in low-doses) can be safely used for treatment of atrophic vaginitis.

NOTE - All hormonal contraception methods and systemic hormone replacement therapies are contraindicated in the presence of a history of breast cancer.

References

• Local estrogen therapy and risk of breast cancer recurrence among hormone-treated patients

Last updated:
Time spent: QID:552 2023-2-12

863 of 1943
Which one of the following is not associated with increased risk of cervical cancer?

A. Sexual activity in the past.

B. Current sexual activity.

C. Age older than 25 years at first sexual intercourse.

D. Multiple sexual partners.

E. Human papilloma virus infection.

Incorrect. Correct answer is C


45% answered correctly

Explanation:

Correct Answer Is C

Risk factors for carcinoma of cervix include:

Persistent infection with high-risk HPV types (the most significant risk factor – without HPV infection, there is no risk of cervical cancer even in the presence of multiple other factors)
Smoking
Lack of regular cervical screening tests
Age – cervical cancer risk increases after the age of 35 years
Prolonged use of OCPs (more than 5 years)
Immunosuppression
Previous screening abnormalities or previous cervical cancer
Multiparity (5 or more)
Exposure to diethylstilbestrol (DES)

Age over 25 at first sexual intercourse decreases the risk of cervical cancer, as women at this age group are more likely to take precautionary measures in sexual relationships.

References

• RACGP – Guidelines for preventive activities in general practice (The Red Book) – cervical cancer

• Cancer Australia – Cervical cancer: what are the risk factors for cervical cancer?

Last updated:
Time spent: QID:553 2023-2-12

864 of 1943
A 27-year-old woman presents to your clinic complaining of a painful swelling down below. She is unable to sit and feels very uncomfortable. On examination, there is vulval redness and tenderness. Which one of the following you should inspect to
confirm the diagnosis?

A. Posterior end of the labia minora.

B. Posterior end of the labia majora.

C. Anterior end of the labia minora.

D. Anterior end of the labia majora.

E. Anterior and posterior ends of the labia majora.

Correct
45% answered correctly

Explanation:

Correct Answer Is A

Labia minora (singular: labium minus) are two flaps of skin on either side of the vaginal opening situated between the labia majora. They extend from clitoris obliquely downwards, laterally and backwards.

On the front, each lip divides into two portions surrounding around the clitoris. The upper part of each lip passes above the clitoris to meet the upper part of the other lip, forming a fold that overhangs the clitoris called clitoral hood. The posterior
ends of labia minora are joined across the midline by a fold of skin called frenulum of labia minora or fourchette. The part of each lip that is near clitoris is called anterior end of the labium minus (the nomenclature is based on a woman in a standing
position where the clitoris is in the front and fourchette in the back). When a woman is in lithotomy position and the vulva observed anteriorly, the posterior end is in the bottom.

Bartholin glands are two pea-sized glands, each being in the posterior (bottom) part of the labium minus. Observed in the lithotomy view, the glands are approximately at 4 o’clock and 8 o’clock positions. The gland secretions, through a 2- to 2.5-cm
duct, drains into the vestibule. Cysts and abscesses of these glands leads to gland enlargement and tenderness (in case of abscess) felt in the posterior end of the labium minus.

Schematic anatomy of Bartholin's glands and the ducts and their position in the labia minora - A extensive Bartholin abscess in he left labium minus. The swelling has extended from the posterior end of the labium minus to invlove
http://aafp.org almost the entire labium minus - http://emedicine.medscape.com/

References

• AAFP - Management of Bartholin's Duct Cyst and Gland Abscess

• Medscape - Bartholin Gland Diseases

Last updated:
Time spent: QID:560 2023-2-12

865 of 1943
A 25-year-old female presents with pain and swelling of her vulva. She has been experiencing discomfort during sexual intercourse and while walking for the past few days. She denies any past history of sexually transmitted infections. Her partner is
asymptomatic. She is afebrile and generally well. On pelvic examination, a red, hot swelling is noted in the posterior end of the labia majora. Speculum exam reveals normal appearing mucosa without significant discharge. There is no regional
lymphadenopathy. Which one of the following is the most appropriate initial treatment?

A. Antibiotics.

B. Marsupialization.

C. Word catheter.

D. Excision.

E. Hot compress and analgesics.

Incorrect. Correct answer is C


45% answered correctly

Explanation:

Correct Answer Is C

This patient has signs and symptoms suggestive of Bartholin gland abscess. Management of Bartholin’s cyst is different from Bartholin’s abscess.

The Bartholin’s glands develop from buds in the epithelium of the posterior area of the vestibule. The glands are located bilaterally at the base of labia minora and drain through 2- to 2.5-cm-long ducts into the vestibule at about 4 o’clock and 8 o’clock
positions. The glands are usually the size of a pea and rarely exceed 1 cm. they are not palpable except in the presence of disease or infection or in very thin women.

Obstruction of the distal Bartholin's duct may result in retention of secretions, with consequent dilation of the duct and cyst formation. The cyst may become infected, and an abscess may develop in the gland. However, a Bartholin cyst does not
necessarily have to be present before a gland abscess develops.

Bartholin's gland abscesses are polymicrobial. Although Neisseria gonorrheae is the predominant aerobic isolate, generally, anaerobes are the most common pathogens. Chlamydia trachomatis also may be a causative organism.

Bartholin's duct cysts and gland abscesses are no longer considered to be exclusively the result of sexually transmitted infections because for instance vulvovaginal surgery is an uncommon cause of these cysts and abscesses too.

The treatment of a Bartholin's duct cyst depends on the patient's symptoms. An asymptomatic cyst may require no treatment, but symptomatic Bartholin's duct cysts and gland abscesses require drainage. Unless spontaneous rupture occurs, an
abscess rarely resolves on its own.

Incision and drainage –this procedure is a relatively quick and easy method with immediate relief; however, due to high tendency for recurrence, it is not recommended.

Word catheter - the recommended initial management of a Bartholin’s gland abscess is by incision and drainage and Word catheter insertion to reduce recurrence rates.

Word catheter is a balloon-tipped device that is inserted into the cyst/abscess cavity immediately after incision and drainage. The bulb of the catheter is then inflated with 4mL of saline and left in place for at least 4 weeks to promote formation of an
epithelialized tract for drainage of glandular secretions. The end of the catheter is tucked into the vagina to minimize discomfort. When the tract appears well-epithelialized, the balloon is deflated and the catheter is removed. Anesthesia is not
necessary. With this procedure, recurrence rates decrease to approximately 3%.

Word catheter - http://aafp.org

Marsupialization - an alternative to Word catheter placement is marsupialisation of Bartholin cysts. This procedure takes longer and is more invasive than placement of Word catheter. Marsupialization (option B) should be reserved for patients who
have failed one or two placements of a Word catheter. Some authors advise that this method should not be used for abscesses.

In this office-based procedure, under local anesthetics, a vertical 1.5- to 3-cm incision is made in the vestibule over the center of the cyst and outside the hymen ring. The abscess cavity is irrigated with saline solution, and loculations (if present) are
broken up with a hemostat. The cyst wall is then everted and approximated to the edge of the vestibular mucosa.

866 of 1943
Marsupialisation - http://aafp.org

Excision - excison of the affected Bartholin gland (option D) is the definite treatment. The procedure should be performed in the operating room under general anesthesia. Since it can result in vulval disfigurement, excisional removal of Bartholin
cysts is reserved for cases in whom all other measures have failed.

Antibiotics – antibiotics (option A) are not generally recommended for treatment of Bartholin’s glad abscess unless there is:

Suspected gonorrhea or Chlamydia infection


Extensive surrounding cellulitis
Immunosuppression
Systemic signs of infection

In these cases antibiotics should be used after abscess drainage.

Hot compress (option E ) is not effective in treatment of Bartholin’s gland cysts or abscesses.

References

• UpToDate - Disorders of bartholin gland

• Medscape - Bartholin Gland Diseases Treatment & Management

Time spent: QID:561 Last updated:


2023-2-12

867 of 1943
Which one of the following is highly suggestive of polycystic ovarian syndrome (PCOS)?

A. Elevated FSH.

B. Elevated LH.

C. Elevated testosterone levels.

D. Decreased free testosterone.

E. Hyperprolactinemia.

Incorrect. Correct answer is C


45% answered correctly

Explanation:

Correct Answer Is C

Polycystic ovarian syndrome (PCOS) is a common condition associated with the following:

1. Clinical or biochemical hyperandrogenism. Clinical features suggestive of PCOS include hirsutism, acne, deepened voice, etc
2. Menstrual dysfunction, irregularity or lack of ovulation.
3. Polycystic ovaries are present on the ultrasound.

Only the first two mentioned features suffice to establish the diagnosis of PCOS, because cystic structures on ultrasound are often not seen, particularly in women who have been on treatment.

The following hormonal changes are seen in PCOS:

Serum FSH (follicle stimulating hormone) is either normal or low

Elevation of LH (luteinizing hormone)

The LH/FSH ration is a way to differentiate several hormonal abnormalities associated with different conditions. This is best measured on day 3 of the menstrual cycles.

A normal LH/FSH ratio in premenopausal women is about 1:1, while in PCOS this ration may be 2:1 or 3:1. It should be noted that approximately 30% of patients with established diagnosis of PCOS will have a LH/FSH ration of 1:1

Serum estrogen level is either decreased or normal. Overall, estrogen level is very nonspecific.

Serum free testosterone is the first-line investigation in women suspected of having PCOS. Hyperandrogenism is one of the essential criteria to establish the diagnosis of PCOS.

Prolactin level is usually normal or mildly elevated.

​An elevated LH or FHS level can be seen in other conditions such as premature ovarian failure, menopause, etc; therefore, not highly suggestive PCOS.

References

• http://emedicine.medscape.com/article/256806-worku

• http://www.ncbi.nlm.nih.gov/pmc/articles/PMC106906

Last updated:
Time spent: QID:562 2023-2-12

868 of 1943
Jessica, 51 years old, presents to your practice with complaints of hot flushes, irritability, and disturbed sleep for the past two months which have been progressively worsening. She had been otherwise in good health prior to that. she has her last
period 6 months ago after a 12-month history of irregular, widely spaced periods. Her recent symptoms have negatively affected her family and social lives. Which one of the following options would you recommend for treatment?

A. Fluoxetine.

B. Continuous daily estrogen therapy, with medroxy progesterone acetate (MPA) given daily for 12 days each month.

C. Continuous estrogen and MPA.

D. Daily progesterone therapy with MPA.

E. Continuous estrogen therapy alone.

Incorrect. Correct answer is B


45% answered correctly

Explanation:

Correct Answer Is B

Jessica has classic symptoms of menopause. Bothersome vasomotor symptoms of menopause i.e., hot flushes are the main indication for hormone replacement therapy in perimenopausal women.

There are two recommended regimens for combined HRT therapy:

Cyclical HRT with daily estrogen PLUS MPA given only for 12 days each month – this method is best used within the first 1-2 years of cessation of periods because compared to continuous method, occurrence of unpredictable
breakthrough bleeding - an adverse effect of progesterone - will be reduced. Such bleedings can be troublesome for patient and my result in noncompliance with the therapy.

Continuous therapy with daily estrogen and MPA – this method is based on taking both estrogen and MPA on regular daily basis. Unpredictable breakthrough bleeding as an effect of progesterone makes this method appropriate 1-2
years after cessation of menses.

Since the menses has been ceased for only the past 6 months, cyclical HRT with continuous daily estrogen and MPA for 12 days each month will the most appropriate management option for her.

(Option A) Selective serotonine reuptake inhibitors (SSRIs) are used for treatment of hot flushes of menopause if HRT is contraindicated. Paroxetine is approved for such purpose.

(Option C) Continuous daily estrogen and progesterone is appropriate for women after 1 to 2 years of menopause.

(Option D) Progesterone-only method is not an effective HRT because estrogen is the main component counteracting menopausal symptoms.

(Option E) Estrogen-only method is the best and most effective method of HRT, but it can be used if the patient has no uterus. With a uterus in place, estrogen therapy alone increases the risk of endometrial hyperplasia and cancer.

References

• http://www.ncbi.nlm.nih.gov/pmc/articles/PMC254436

• https://www.nhmrc.gov.au/_files_nhmrc/publications

• AMC Handbook of Multiple Choice Questions – page 530

Last updated:
Time spent: QID:563 2023-2-12

869 of 1943
Which one of the following is unlikely to predispose to postpartum hemorrhage?

A. Uterine fibroids.

B. Multiple gestation.

C. Von Willebrand disease of the mother.

D. Oligohydramnios.

E. Prolonged labor.

Incorrect. Correct answer is D


45% answered correctly

Explanation:

Correct Answer Is D

Predisposing factors to postpartum hemorrhage include:

Uterine fibroids
Multiple pregnancy
Polyhydramnios
Maternal history of bleeding disorders
Prolonged labor
Retained placenta
Instrumental delivery
Pregnancy-induced hypertension and pre-eclampsia
Past history of postpartum hemorrhage
Multiparity
Obesity

Oligohydramnios does not increase the risk of postpartum hemorrhage.

References

• http://emedicine.medscape.com/article/275038-overv

• http://www.uptodate.com/contents/overview-of-postp

• https://thewomens.r.worldssl.net/images/uploads/do

Last updated:
Time spent: QID:103 2023-2-12

870 of 1943
A 28-year-old woman, who is 22 weeks pregnant in her second pregnancy, presents for evaluation of a vulval ulcer. The swab taken confirms the diagnosis of herpes simplex type II (HSV-II) infection. When she is informed, she becomes quite
surprised as neither she, nor hes husband has ever had this infection before and insists to know the source of the infection. Furthermore, she is very concerned about her baby’s wellbeing and asks whether her condition may affect it. Which one of
the following is the most appropriate advice in this regard?

A. Most of neonates with neonatal herpes present with mucocutaneous lesions.

B. Although treatment with antiviral agents such as aciclovir will reduce the rate recurrence of the disease, they cannot be used during pregnancy because of their adverse effects on the fetus.

C. The primary infection is commonly asymptomatic.

D. Unless she has had a new sexual partner recently, this problem could not have been acquired sexually.

E. The risk of neonatal herpes is much higher with recurrent maternal infection compared with primary infection.

Incorrect. Correct answer is C


45% answered correctly

Explanation:

Correct Answer Is C

Unlike what is usually thought, genital herpes infection is asymptomatic in 75% of the cases. Therefore, this woman may have contracted the infection from his asymptomatic husband.

Herpes simplex virus type II exclusively passes on through skin-to-skin contact during sexual activity; therefore having a sexual partner infected with HSV-II is essential for contracting the disease. Herpes simplex virus type I, on the other hand can be
transmitted from non-sexual skin contacts e.g. kissing.

With herpetic infection during pregnancy, antiviral therapy can be considered. Studies have shown that aciclovir is the safest to use in pregnancy.

The risk of fetal infection is significantly higher in primary infection, because in recurrences the fetus has already received IgG against HSV, passively from the mother. It should be born mind that neonatal infection with recurrent infection is also
possible, but far less likely compared with primary infection. If primary HSV is contracted before 30 weeks, the risk of shedding HSV during a normal birth is 7 % with an overall risk of ≤ 3 % for neonatal HSV disease.

References

• https://www.sahealth.sa.gov.au/wps/wcm/connect/91b

• Australian society for infectious diseases – Management of Perinatal Infections (2014)

Last updated:
Time spent: QID:564 2023-2-12

871 of 1943
Which one of the following is not helpful in prevention of transmission of HIV infection from a pregnant woman to her baby?

A. Maternal antiretroviral therapy.

B. Peripartum intravenous zidovudine.

C. Elective cesarean section.

D. Neonatal antiretroviral treatment.

E. Breastfeeding.

Incorrect. Correct answer is E


45% answered correctly

Explanation:

Correct Answer Is E

Interventions to prevent perinatal transmission of HIV include:

Maternal antiretroviral therapy


Peripartum intravenous zidovudine
Elective cesarean section
Neonatal antiretroviral treatment
Bottle feeding

Breast feeding is contraindicated in HIV positive mothers because of increased chance of transmission of HIV infection.

Without intervention, the risk per cent of HIV transmission to the fetus is approximately 20-30%. With intervention this rate the risk decrease to less than 2%.

References

• https://www.sahealth.sa.gov.au/wps/wcm/connect/72e

Last updated:
Time spent: QID:567 2023-2-12

872 of 1943
A 27-year-old woman, diagnosed with gestational diabetes at 28 weeks pregnancy, has been being managed at a high-risk pregnancy clinic until 38 weeks when she vaginally deliveres a healthy 4-kg baby without any complications. Which one of the
following is correct regarding follow-up?

A. 75g oral glucose tolerance test performed between weeks 6 and 12 after delivery.

B. Check fasting blood glucose level in 6 months after delivery.

C. HbA1C as soon as possible.

D. Fasting lipids.

E. No further action is required.

Correct
45% answered correctly

Explanation:

Correct Answer Is A

Less than 10% of women with GDM remain hyperglycemic after delivery. The management of these women requires ongoing care from a diabetes or medical clinic in collaboration with their general practitioner.

The following are recommendations for women with gestational diabetes after delivery:

Checking a random blood glucose level the day after delivery


4-point blood glucose level (BGL) measurement on the day prior to discharge (fasting and two hours post meals for three meals)
Cease blood glucose monitoring if BGL is within normal range.
If there is elevated BGL (above 10 mmol/L) contact the medical registrar / diabetes educator – if BGLs are considerably elevated, after-hours contacts are justified.
It is recommended that women who have had GDM visit their GP for a follow-up oral glucose (75 gr) tolerance test at 6-12 weeks postpartum, and every 1-2 years thereafter.
Lifestyle counselling.
A summary letter is provided to the woman’s GP for follow-up care.

This woman should have a random BGL the day after delivery, 4-point BGL test the day before discharge, and an oral glucose tolerance test using 75g glucose between weeks 6 and 12 after delivery.

Checking HbA1C and fasting lipids would be required if this patient is found to have type 2 diabetes mellitus.

References

• KEMH - Clinical guidelines

Last updated:
Time spent: QID:568 2023-2-12

873 of 1943
A 17-year-old girl presents to your office for consultation because she has never had a period. On examination. she has normal height for her age. She also has normally developed breasts, pubic hair, axillary hair and genitalia. Which one of the
following would be the most likely cause of her amenorrhea?

A. Turner syndrome.

B. Prolactinoma.

C. Absent uterus.

D. A hypothalamic lesion.

E. Congenital adrenal hyperplasia.

Incorrect. Correct answer is C


45% answered correctly

Explanation:

Correct Answer Is C

Primary amenorrhea is defined as the absence of menses by the age of 16 in the presence of normal growth and secondary sexual characteristics, or by the age of 14 in the absence of these features. Primary amenorrhea, in the presence of normal
sexual development is suggestive of abnormalities such as absent or malformed uterus or absent or septated vagina or obstructed menstrual flow. Ultrasonography is the best initial investigation to find whether uterus is present or absent.

If uterus is present, amenorrhea may be due to menstruation flow obstruction caused by conditions such as imperforated hymen or vaginal septum. If uterus is absent or abnormal, chromosomal abnormalities are the most likely cause.

In the presence of normal breast development, normal pubic and axillary hair growth, normal genitalia and normal height, hypothalamic hypogonadism (option D), pituitary lesions and chromosomal abnormalities such as Turner syndrome (option A)
are very unlikely to be the cause of amenorrhea in this girl.

With prolactinoma (option B), patients are expected to have galactorrhea, headache, or visual symptoms along with menstrual abnormalities.

Congenital adrenal hyperplasia (option E) has different clinical picture with manifestations much earlier in life.

References

• Medscape - Amenorrhea

• NSW Health - Amenorrhoea

Last updated:
Time spent: QID:570 2023-2-12

874 of 1943
Which one of the following is increased during pregnancy?

A. Serum alkaline phosphatase.

B. Serum albumin.

C. Serum calcium.

D. Parathyroid hormone.

E. TSH in the first trimester of pregnancy.

Correct
45% answered correctly

Explanation:

Correct Answer Is A

Normal pregnancy is associated with profound changes in almost every organ system to accommodate the demands of the fetoplacental unit.

Serum albumin, transaminases (AST and ALT) and total bilirubin are lowered compared with the non-pregnant state due to expansion of extracellular volume. The only exception is serum alkaline phosphatase (ALP) which is elevated due to ALP of
placental origin.

During pregnancy, the serum total calcium, phosphate and magnesium tend to be low due to the expanded intravascular space. Concentrations of calcium are also affected by the reduced albumin concentration.

The concentration of serum parathyroid hormone tends to be 50% lower in pregnancy, despite the fact that serum concentration of calcium is decreased.

The reasons for hypocalcemia during pregnancy are (1) plasma expansion that through dilutional effect causes decreased plasma calcium and albumin concentration and (2) increased urinary excretion of calcium due to increased GFR

Thyroid stimulating hormone (TSH) normally decreases during the first trimester of pregnancy, during which time there is maximal cross-stimulation of the TSH receptor by human chorionic gonadotropin hormone (HCG).

References

• http://www.australianprescriber.com/magazine/28/4/

Last updated:
Time spent: QID:571 2023-2-12

875 of 1943
A 38-year-old female at 33 weeks’ gestation sustains a road traffic accident at 90 km/hour. In the emergency department and on examination, she is found to be pale, with a heart rate of 112 bpm, blood pressure of 95/55 mmHg and respiratory rate
of 18 breaths per minute. Her oxygen saturation is 95% on room air. Fetal heart rate is audible at 102bpm. The uterus is tense and tender. Which one of the following is the most likely diagnosis?

A. Ruptured uterus.

B. Liver laceration.

C. Placental abruption.

D. Ruptured spleen.

E. Placenta previa.

Incorrect. Correct answer is C


45% answered correctly

Explanation:

Correct Answer Is C

This patient has signs and symptoms consistent with clinical diagnosis of placental abruption.

Trauma in last trimester of pregnancy could be dangerous to both the mother and the fetus. Motor vehicle accidents, by force of deceleration, cause placental separation.

Placental abruption is characterized by painful, tender uterus which is often contracting. The condition leads to maternal hypovolemic hypotension and consequent fetal distress represented by fetal bradycardia and repetitive late decelerations.

The amount of vaginal bleeding is not usually an appropriate indicator to severity of placental abruption, because bleeding could be very severe or it may be concealed between uterine wall and placenta in form of a hematoma.

(Option A) Uterine rupture is characterized by severe abdominal pain and tenderness, cessation of contractions and loss of uterine tone. It is associated with mild to moderate vaginal bleeding and fetal bradycardia or loss of heart sound. Compared
to placental abruption, there uterus is less tense and tender.

(Options B and D) Ruptured spleen and liver laceration present may justify the low blood pressure, tachycardia and fetal bradycardia but not the tense, tender and contracting uterus.

(Option E) Placenta previa presents with sudden, painless bleeding with bright red blood. There is no uterine tenderness. This diagnosis is not consistent with the clinical picture.

References

• http://www.aafp.org/afp/2004/1001/p1303.html

• http://www.uptodate.com/contents/placental-abrupti

Last updated:
Time spent: QID:572 2023-2-12

876 of 1943
A 24-year-old woman comes for preconception evaluation and consultation. She has the past history of tonic-clonic epilepsy, but has been seizure-free for the past 2 years while on phenytoin. She is currently on oral contraceptive pills. An EEG is
obtained that excludes any epileptiform activity. On examination, there is no focal neurological finding. Which one of the following would be the most appropriate advice?

A. Cease phenytoin and change to carbamazepine.

B. Discontinue phenytoin.

C. Advise that she should not become pregnant.

D. Continue phenytoin, but add high dose folic acid according to current recommendations.

E. Check serum phenytoin level and decrease the dose if appropriate.

Incorrect. Correct answer is B


45% answered correctly

Explanation:

Correct Answer Is B

Antiepileptic therapy may be ceased after the patient has been seizure-free for at least 2 years, particularly if there is no epileptiform activity on EEG. Medication may be withdrawn earlier in women planning a pregnancy; however, the epileptic
disorder needs to be reviewed beforehand.

The decision to withdraw the drug and the time it should be considered depends on the patient’s specific epileptic syndrome and previous history. Factors associated with a poor prognosis associated with withdrawal include:

A symptomatic epilepsy
Neurological abnormalities on examination
History of difficult-to-control seizures (such as patients with juvenile myoclonic epilepsy)
Epileptiform changes on the EEG
Abnormalities on MRI
A history of seizure recurrence after previous attempts to withdraw medication

Antiepileptic drugs are potential teratogens and assumed not safe to the fetus. Therefore, withdrawal of antiepileptics should always be considered provided that all the following are considered and strictly followed:

1. If the patient has been seizure-free for 2 years or more


2. There is no epileptic activity on EEG
3. There is no abnormal focal neurological findings on physical exam and imaging studies
4. Withdrawal from antiepileptic drugs is managed or guided by a specialist.

The dose is gradually decreased over several months and even more slowly for barbiturates. Ideally, complete cessation should take place at least 6 months before conception. If seizures recur, the previously effective dose is resumed.

NOTE – In juvenile myoclonic epilepsy it is best not to withdraw the mediation, or at least to do so after the patient has been seizure free for many years.

Since this woman has been stable for two years and has a normal EEG, gradual withdrawal would be the next best option in management.

References

• Therapeutic Guidelines – Neurology

• Australian Prescriber - Antiepileptic drugs in pregnancy and lactation

Last updated:
Time spent: QID:573 2023-2-12

877 of 1943
Which one of the following is the most common cause of postpartum hemorrhage?

A. Uterine atony.

B. Laceration of genital tract.

C. Uterine rupture.

D. Uterine inversion.

E. Coagulopathy.

Correct
45% answered correctly

Explanation:

Correct Answer Is A

The traditional definition of a primary postpartum hemorrhage (PPH) is a blood loss of 500 mL or more in the first 24 hours. Postpartum hemorrhage can be minor (500-1,000 mL) or major (> 1,000 mL). A major PPH can be further described as
moderate (1,000-2,000 mL) or severe (> 2,000 mL).

A widely accepted definition of postpartum hemorrhage (PPH) in many institutions is a blood loss of 600 ml for vaginal delivery and 750 ml for cesarean delivery. The classification of PPH in relation to the amount of blood loss is problematic, largely
due to a well-documented underestimation of blood loss. A clinically relevant alternative is a substantial fall in the hematocrit e.g. 10 %.

PPH causes include:

Abnormalities of uterine contraction (Tone) 70 %


Genital tract trauma (Trauma) 20 %
Retained products of conception or invasive placenta (Tissue) 10 %
Abnormalities of coagulation (Thrombin) < 1 %

Postpartum uterine atony is by far the most common cause of PPH.

The most common risk factors for uterine atony include:

Multiple pregnancy
Polyhydramnions
Macrosomia
Prolonged labor
Multiparity

References

• https://www.sahealth.sa.gov.au/wps/wcm/connect/7a6

• https://www.health.qld.gov.au/qcg/documents/g_pph.

Last updated:
Time spent: QID:574 2023-2-12

878 of 1943
A 26-year-old primigravida presented with vaginal bleeding at 16 weeks gestation. She is Rh-negative, but her partner RH-positive. On examination, cervix is dilated and products of conception are visible. Pelvic ultrasound confirms the diagnosis of
spontaneous abortion. Which one of the following would be the most appropriate advice regarding Anti-D antibody (RhoGAM)?

A. Give Anti-D at 28 weeks gestation in next pregnancy.

B. Anti-D is not indicated in this situation.

C. Genetic analysis should be performed before making any decision.

D. Give anti-D now.

E. Advice that Anti-D antibodies would develop within 7 days.

Incorrect. Correct answer is D


45% answered correctly

Explanation:

Correct Answer Is D

Rhesus (Rh) negative women who deliver an Rh positive baby or who are otherwise exposed to Rh positive red blood cells are at risk of developing anti-Rh antibodies (RhD) and should receive RhD antibody (RhoGAM®) in current pregnancy.

Rh positive fetuses/neonates of these mothers are at risk of developing hemolytic disease of the fetus and newborn, which can be lethal or associated with serious morbidity.

TOPIC REVIEW

An Rh-negative mother has no antibodies against Rh (D) antigen. If she is pregnant and the fetus is Rh-positive, any mixing up the fetus’ blood with the mother’s will trigger an immune response by lymphocytes and will lead to production of anti-Rh
antibodies in the mother’s blood (anti-D antibody). RhoGAM® is anti-D IgG passive antibody that eliminates the D-antigen from the mother’s blood, before the mother’s immune system start to sensitize. It is administered intramuscularly (IM).

RhoGAM is available in two forms:

CLS-250 IU – one dose contains 50μg of anti-D antibody


CLS-265 IU – one dose contains 125μg of anti-D antibody

Each 300μg of anti-D antibody neutralizes 15ml of fetal packed red cells (30 mL of whole blood)

RhoGAM should be administered within the first 72 hours after the precipitating even, however, with much less success rate it can be given up to 9-10 days.

INDICATIONS:

First trimester indications (up to and including 12 weeks):

Spontaneous abortion (complete, incomplete, or missed abortion) probably excluding threatened abortion
Induced abortion (medically or by D&C)
Ectopic pregnancy
Chorionic villous sampling (this procedure is performed at 10-12 weeks)
Molar pregnancy – chorionic villi may contain D-antigen

Management – in the first 12 weeks ,the maximum amount of fetal blood that can mix with mother’s is 2.5 ml of RBC (5 mL whole blood). A single dose of CLS 250-IU is sufficient for neutralizing circulating fetal D-antigen. Multifetal pregnancies need
extra doses. In a singleton pregnancy Kleihauer-Betki test is not indicated because the amount of fetomaternal blood mix is not significant and a single dose CLS – 250 will be enough.

Second and third trimester indications:

Spontaneous or induced abortions of all kind after 12 weeks


Amniocentesis (this procedure is performed > 15 weeks)
Cordocentesis (this procedure is performed >20 weeks)
Fetal blood sampling
Fetal death
External cephalic version of breech presentation (successful or unsuccessful)
Blunt abdominal trauma in pregnancy considered sufficient to cause fetomaternal hemorrhage
Antepartum haemorrhage (revealed or concealed) in the second or third trimester (e.g. placental abruption, placenta praevia, etc)

Management – in the second or third trimester CLS 625 (containing 125μg anti D-antibody) should be used. A Kleihauer-Betki test is indicated in the second or third trimester events to assess the need for extradoses of CLS 625 IU. Ideally the sample
for the test should be collected within 15 minutes of the precipitating event, but if not possible it can be performed up to 72 hours.

NOTE (1) - RhoGAM will remain in maternal circulation for up to 6 weeks.

NOTE (2) - RhoGAM should not be given to women in which anti-D antibody has already formed.

Routine administration of RhoGAM

In the absence of a precipitating event, RhoGAM (CLS – 625 IU) is routinely given to all Rh-negative pregnant women at 28 weeks and 34 weeks of pregnancy.

Within the first 72 hours postpartum all R-negative women, whose baby is Rh-positive should receive a dose of CLS-625 IU and undergo quantification of fetomaternal blood mix using Kleihauer-Betki test to evaluate whether extra doses of CLS-625
IU are indicated.

References

• http://www.australianprescriber.com/magazine/23/2/

• https://www.nhmrc.gov.au/_files_nhmrc/publications

879 of 1943
• Royal Australian and New Zealand College of Obstetricians and Gynaecologists – College Statement C-Obs 6 (2015)
Last updated:
Time spent: QID:576 2023-2-12

880 of 1943
A 28-year-old lady comes to your clinic at 10 weeks pregnancy. She is keen to know how she can minimize the chance of toxoplasma infection. Which one of the following pieces of advice is not helpful in preventing toxoplasma infection?

A. Wash fruits and vegetables before consumption.

B. Wash hands and kitchen surfaces thoroughly after contact with raw meat.

C. Daily shower.

D. Avoid untreated water while travelling.

E. Avoid changing cat litter boxes.

Incorrect. Correct answer is C


45% answered correctly

Explanation:

Correct Answer Is C

Risk of transmission of Toxoplasma can be minimized by washing fruits and vegetables before consumption, washing hands and kitchen surfaces after contact with raw meat and avoiding changing cat litter boxes.

She should also avoid using untreated water while travelling especially in developing countries.

These measures should be adopted throughout the pregnancy.

Daily shower is not recommended for prevention of Toxoplasma.

References

• http://www.cdc.gov/parasites/toxoplasmosis/prevent

Last updated:
Time spent: QID:577 2023-2-12

881 of 1943
Which one of the following is the most common cause of postpartum hemorrhage requiring hysterectomy?

A. Genital lacerations.

B. Uterine atony.

C. Uterine inversion.

D. Placenta accreta.

E. Retained products of conception.

Incorrect. Correct answer is D


45% answered correctly

Explanation:

Correct Answer Is D

The most common reasons for hysterectomy as the inevitable treatment of postpartum hemorrhage are placental abnormalities such as placenta previa and placenta accreta.

Placental villi normally invade only the superficial layers of endometrial deciduas basalis. When the invasion is too deep into the uterine wall, the condition is termed as placenta accreta, increta or percreta depending on the depth of invasion:

Placenta accreta – the villi invade the deeper layers of the endometrial deciduus basalis, but not the myometrium. This is the most common type of decidual invasion, accounting for approximately 75% of the cases.

Placenta increta – the villi invade the myometrium, but do not reach the uterine serosa or the bladder. This type accounts for 15% of cases.

Placenta percreta – the villi invade into the uterine serosa or the bladder. This happens in 5% of cases.

Prior uterine surgery is the main risk factor for placenta accreta. The best management is elective cesarean hysterectomy.

Post-partum hemorrhage can also be caused by following conditions:

Genital lacerations
Uterine atony
Retained products of conception
Uterine inversion

In most of above cases, hysterectomy would not be required and remains the last resort in really desperate situations.

NOTE - Uterine atony is the most common cause of post-partum hemorrhage and is often manageable medically.

References

• http://www.ncbi.nlm.nih.gov/pmc/articles/PMC323414

Last updated:
Time spent: QID:104 2023-2-12

882 of 1943
Which one of the following is not elevated in the third trimester of pregnancy?

A. Serum prolactin.

B. Serum alkaline phosphatase.

C. Serum iron binding capacity.

D. Serum free T4.

E. Serum cortisol.

Incorrect. Correct answer is D


45% answered correctly

Explanation:

Correct Answer Is D

Normally, estrogen stimulates and progesterone inhibits prolactin secretion; despite this action and counteraction, there is a slight raise in prolactin level throughout pregnancy.

Serum alkaline phosphatase is increased in pregnancy due to placental ALP.

Physiologically, the concentration of thyroid stimulating hormone (TSH) normally decreases during the first trimester of pregnancy when there is maximal cross-stimulation of the TSH receptor by hCG. The TSH concentration then returns to its pre-
pregnancy level in the second trimester and then rises slightly in the third trimester. However, most of changes still occur within the normal non-pregnant range. Serum free T3 and T4 concentrations remain essentially unchanged during pregnancy
but total concentrations, which include both free and protein-bound fractions, are significantly elevated due to increased circulating binding globulins..

Iron binding capacity reflects the protein used for iron transport, transferrin. Transferrin is a globulin in the beta band on electrophoresis. Both transferrin and iron binding capacity are elevated in pregnancy to counteract decreased plasma iron during
pregnancy.

Cortisol levels can be elevated up to three times normal when compared to non-pregnant levels.

References

• http://www.australianprescriber.com/magazine/28/4/

• http://www.ncbi.nlm.nih.gov/pmc/articles/PMC398211

Last updated:
Time spent: QID:578 2023-2-12

883 of 1943
Which one of the following is incorrect regarding hypothyroidism in pregnancy?

A. Thyroxin requirement increases by 25- 30% during pregnancy.

B. Children born to women whose hypothyroidism was inadequately treated in pregnancy are at increased risk of neuropsychological impairment.

C. Thyroxine dose should be increased by 30% at the beginning of pregnancy.

D. TSH should be monitored every 4 to 6 weeks in the first half of the pregnamcy, and thyroxin dose adjusted if necessary.

E. Thyroxin requirement does not increase in pregnancy and maintenance dose must be continued.

Incorrect. Correct answer is E


45% answered correctly

Explanation:

Correct Answer Is E

During pregnancy, the size of the thyroid gland increases 15%, mostly due to increased vasculature. Estrogen causes the liver to produce more thyroid binding globulin (TBG). This results in increased total T3 and T4 levels; however, free T3 and T4
remain unchanged. Increased glomerular infiltration rate (GFR) leads to more renal excretion of iodine.

Pre-existing maternal hypothyroidism can lead to neuropsychological deficits of the baby; therefore, the thyroxin requirement should be precisely adjusted during pregnancy. Pregnancy is associated with an increase in demand of thyroxin by 25 to
30% requiring a 30% increase is thyroxin dose as soon as the pregnancy is diagnosed. This is best achieved by taking 2 additional doses of thyroxin per week on confirmation of pregnancy. (In most cases increasing from 7 to 9 doses per week)

TSH and free T4 should be rechecked after 4 weeks and monitored every 4-6 weeks in the first half of pregnancy and at least once between 26 and 32 weeks. The values should be maintained within the trimester specific range.

After delivery, the thyroxin dose can be reduced to pre-pregnancy dose. Breastfeeding mothers do not need extra doses of thyroxin.

NOTE – subclinical hypothyroidism is associated with adverse pregnancy outcomes, particularly miscarriage but not impaired cognitive function.

References

• RACGP - AFP - Thyroid disease in the perinatal period

Last updated:
Time spent: QID:580 2023-2-12

884 of 1943
Which one of the following statements is correct regarding the management of deep venous thrombosis in pregnancy?

A. Warfarin therapy is contraindicated only in the first trimester of pregnancy.

B. Warfarin therapy is contraindicated throughout pregnancy but safe during breast feeding.

C. Warfarin therapy is contraindicated throughout pregnancy and postpartum period.

D. Both warfarin and heparin are safe to use for treatment of deep vein thrombosis in pregnancy.

E. Low-molecular weight heparin is contraindicated in the first trimester of pregnancy.

Incorrect. Correct answer is B


45% answered correctly

Explanation:

Correct Answer Is B

Pregnancy is a hypercoagulable state with increased risk of deep venous thrombosis (DVT) and pulmonary embolism (PE). PE is a significant cause of maternal death in Australia.

In pregnant women, venous thromboembolism should be treated with heparin because warfarin is contraindicated. Warfarin crosses the placenta and should be avoided throughout pregnancy, especially during the first and third trimesters.

Warfarin at 6-12 weeks’ gestation results in fetal warfarin syndrome characterized by the following features:

A characteristic nasal hypoplasia


Short fingers with hypoplastic nails
Calcified epiphyses, namely chondrodysplasia punctuta (evident by stippling of epiphyses on X-ray)
Intellectual disability
Low birth weight

Recent studies show the risk of fetal warfarin syndrome in babies of women who require warfarin throughout pregnancy is around 5 %. The risk is dose dependent.

Later exposure (after 12 weeks) is associated with central nervous system anomalies including microcephaly, hydrocephalus, agenesis of corpus callosum, Dandy-Walker malformation (complete absence cerebellar vermis and enlarged fourth
ventricle), and mental retardation, as well as eye anomalies such as optic atrophy, microphthalmia, and Peter anomaly (anterior segment dysgenesis). Blindness can be seen in newborns exposed to warfarin in all three trimesters. Perinatal
intracranial hemorrhage and other major bleeding is another complication in neonates exposed to warfarin.

Warfarin is not secreted into the breast milk, and is safe to use in post-partum period.

References

• MJA - Consensus guidelines for warfarin therapy

Last updated:
Time spent: QID:581 2023-2-12

885 of 1943
A 32-year-old female presented in a regional Australian hospital with acute onset shortness of breath, tachycardia, and sweating after landing from a long-distance flight from California. She is 34 weeks pregnant. On examination, she has a blood
pressure of 110/87 mmHg, pulse rate of 110 bpm, and respiratory rate of 26 breaths per minute. Heart and lung auscultation is clear. There is no leg swelling, warmth o tenderness. Which one of the following is the best investigation in this situation?

A. CT pulmonary angiogram (CTPA).

B. Electrocardiography.

C. Bilateral lower extermity Doppler ultrasound.

D. D-dimer.

E. Ventilation-perfusion scan.

Incorrect. Correct answer is E


45% answered correctly

Explanation:

Correct Answer Is E

In developed countries, pulmonary embolism (PE) remains a significant cause of mortality in pregnant women. Despite advances in diagnostic modalities, diagnosis of PE in this patient group is challenging due to the following reasons:

1. There are two patients at risk rather than one


2. Overdiagnosis results in unnecessary and dangerous treatment, jeopardizing both the mother and fetus
3. Choosing a modality for diagnosis of PE in pregnant women is more complicated than in non-pregnant patients.

There is about the equal risk of venous thromboembolism in all three trimesters of pregnancy. DVT is three times more common than PE. In 85% of the cases, pregnancy-associated DVT occurs on the left side of the body, probably due to
compression of the left iliac vein by the pregnant uterus.

Isolated pelvic DVT is much more common in pregnancy or in early postpartum period than in non-pregnant patients. All three elements of Virchow's triad (hypercoagulability, vascular damage, and venous stasis) are present in pregnancy and early
postpartum period.

Neither clinical judgment nor clinical decision rules (e.g., Well's score) that are commonly used in non-pregnant patients have proven ineffective for PE during pregnancy. For example, D-dimer levels rise gradually during pregnancy and then drop in
immediate postpartum period but do not return to normal until 4-6 weeks postpartum.

A normal D-dimer level appears to have a high negative predictive value in patients with a low clinical suspicion for VTE, but false-positive levels are very common.

The decision to use imaging modalities associated with radiation exposure in pregnant patients is difficult due to concern of teratogenicity. Radiation exposure more than 1 Gy at any stage of the pregnancy may induce congenital malformations.
Currently, the most commonly used diagnostic tests for PE are ventilation/perfusion scintigraphy (V/Q scan) and computed tomographic pulmonary angiography (CTPA):

V/Q scan

The fetal radiation dose of VQ/ scan is 0.00028-0.00051 Gy. The negative predictive value of a normal V/Q scan in pregnant patients is excellent. For a V/Q scan to be of predictive value, a normal chest X-ray is mandatory. With an underlying lung
pathology, CTPA would be the modality of choice.

CTPA

The fetal radiation dose of CTPA is 0.000003-0.000131 Gy. Although the radiation dose to fetus is lower in CTPA compared to V/Q scanning, CTPA produces higher levels of radiation exposure to women's breasts.

Breast cancer risk in 20-year-old women after a CTPA is estimated to be 1 event in 143 exposures, with a lifetime attributable risk of 0.7%. The risk for malignancy is inversely related to age at the time of exposure. Therefore, it is recommended that
women use breast shields when undergoing CTPA to decrease radiation dose from 0.05-0.08 Gy to 0.02-0.06 Gy.

A major benefit of CTPA over other imaging modalities is that it often provides alternative diagnoses. Thus far, no studies have addressed the accuracy or outcome of CTPA in pregnant patients. However, technical limitations might produce poor
vessel opacification in pregnant patients that simulates PE and leads to false-positive results. This appears to be more common when apparent subsegmental PEs are identified. Hence, it is usually recommended that subsegmental PEs identified on
CTPA be confirmed with further testing (e.g., VQ scan or lower extremity ultrasonography) before making a final diagnosis.

Which test to choose?

V/Q scan is the option of choice for pregnant women provided there is no underlying chest X-ray abnormality to make interpretation difficult. The pregnant woman should be informed that there is a slight risk of increased childhood malignancies
for her baby. But the risk of radiation to the mother is virtually negligible.

(Option A) CTPA is the second best option if V/Q scan is not an option. The fetal radiation is almost zero, but there is a slight risk of breast cancer for the mother. The risk, however, can be reduced by shield protection of the breasts during the
procedure.

(Option B) Electrocardiography is among initial tests for exclusion of other causes of chest pain but it cannot diagnose PE.

(Option C) Doppler ultrasound of both legs is the next best step for diagnosing suspected PE if there are signs suggestive of DVT. If a clot is detected, no further diagnostic test is required and the patient should be started on anticoagulation therapy.
If no clot is found the routine pathway will be followed. This woman has no leg symptoms; therefore, Doppler ultrasound is unlikely to have diagnostic benefit for her.

(Option D) Negative predictive value of a normal D-dimer level for exclusion of VTE including PE is high, especially when there is a low pretest probability; however, this test is not of much use in pregnant women because D-dimer levels progressively
increase throughout pregnancy. After delivery, D-dimer levels drop but not to the baseline for 4-6 weeks.

References

• UpToDate - Deep vein thrombosis and pulmonary embolism in pregnancy: Treatment

• Medscape - PE in Pregnancy: A Complicated Diagnosis

• Diagnostic Imaging Pathways - PE in pregnancy

• Medscape - Pulmonary Embolism in Pregnancy - Workup

L t d t d

886 of 1943
Last updated:
Time spent: QID:582 2023-2-12

887 of 1943
A 32-year-old female presented in a regional Australian hospital with acute onset shortness of breath, tachycardia, and sweating after landing from a long-distance flight from California. She is 34 weeks pregnant. On examination, she has a blood
pressure of 110/87 mmHg, pulse rate of 110 bpm, and respiratory rate of 26 breaths per minute. Heart and lung auscultation is clear. There is no leg swelling, warmth o tenderness. Which one of the following is the best investigation in this situation?

A. CT pulmonary angiogram (CTPA).

B. Electrocardiography.

C. Bilateral lower extermity Doppler ultrasound.

D. D-dimer.

E. Ventilation-perfusion scan.

Incorrect. Correct answer is E


45% answered correctly

Explanation:

Correct Answer Is E

In developed countries, pulmonary embolism (PE) remains a significant cause of mortality in pregnant women. Despite advances in diagnostic modalities, diagnosis of PE in this patient group is challenging due to the following reasons:

1. There are two patients at risk rather than one


2. Overdiagnosis results in unnecessary and dangerous treatment, jeopardizing both the mother and fetus
3. Choosing a modality for diagnosis of PE in pregnant women is more complicated than in non-pregnant patients.

There is about the equal risk of venous thromboembolism in all three trimesters of pregnancy. DVT is three times more common than PE. In 85% of the cases, pregnancy-associated DVT occurs on the left side of the body, probably due to
compression of the left iliac vein by the pregnant uterus.

Isolated pelvic DVT is much more common in pregnancy or in early postpartum period than in non-pregnant patients. All three elements of Virchow's triad (hypercoagulability, vascular damage, and venous stasis) are present in pregnancy and early
postpartum period.

Neither clinical judgment nor clinical decision rules (e.g., Well's score) that are commonly used in non-pregnant patients have proven ineffective for PE during pregnancy. For example, D-dimer levels rise gradually during pregnancy and then drop in
immediate postpartum period but do not return to normal until 4-6 weeks postpartum.

A normal D-dimer level appears to have a high negative predictive value in patients with a low clinical suspicion for VTE, but false-positive levels are very common.

The decision to use imaging modalities associated with radiation exposure in pregnant patients is difficult due to concern of teratogenicity. Radiation exposure more than 1 Gy at any stage of the pregnancy may induce congenital malformations.
Currently, the most commonly used diagnostic tests for PE are ventilation/perfusion scintigraphy (V/Q scan) and computed tomographic pulmonary angiography (CTPA):

V/Q scan

The fetal radiation dose of VQ/ scan is 0.00028-0.00051 Gy. The negative predictive value of a normal V/Q scan in pregnant patients is excellent. For a V/Q scan to be of predictive value, a normal chest X-ray is mandatory. With an underlying lung
pathology, CTPA would be the modality of choice.

CTPA

The fetal radiation dose of CTPA is 0.000003-0.000131 Gy. Although the radiation dose to fetus is lower in CTPA compared to V/Q scanning, CTPA produces higher levels of radiation exposure to women's breasts.

Breast cancer risk in 20-year-old women after a CTPA is estimated to be 1 event in 143 exposures, with a lifetime attributable risk of 0.7%. The risk for malignancy is inversely related to age at the time of exposure. Therefore, it is recommended that
women use breast shields when undergoing CTPA to decrease radiation dose from 0.05-0.08 Gy to 0.02-0.06 Gy.

A major benefit of CTPA over other imaging modalities is that it often provides alternative diagnoses. Thus far, no studies have addressed the accuracy or outcome of CTPA in pregnant patients. However, technical limitations might produce poor
vessel opacification in pregnant patients that simulates PE and leads to false-positive results. This appears to be more common when apparent subsegmental PEs are identified. Hence, it is usually recommended that subsegmental PEs identified on
CTPA be confirmed with further testing (e.g., VQ scan or lower extremity ultrasonography) before making a final diagnosis.

Which test to choose?

V/Q scan is the option of choice for pregnant women provided there is no underlying chest X-ray abnormality to make interpretation difficult. The pregnant woman should be informed that there is a slight risk of increased childhood malignancies
for her baby. But the risk of radiation to the mother is virtually negligible.

(Option A) CTPA is the second best option if V/Q scan is not an option. The fetal radiation is almost zero, but there is a slight risk of breast cancer for the mother. The risk, however, can be reduced by shield protection of the breasts during the
procedure.

(Option B) Electrocardiography is among initial tests for exclusion of other causes of chest pain but it cannot diagnose PE.

(Option C) Doppler ultrasound of both legs is the next best step for diagnosing suspected PE if there are signs suggestive of DVT. If a clot is detected, no further diagnostic test is required and the patient should be started on anticoagulation therapy.
If no clot is found the routine pathway will be followed. This woman has no leg symptoms; therefore, Doppler ultrasound is unlikely to have diagnostic benefit for her.

(Option D) Negative predictive value of a normal D-dimer level for exclusion of VTE including PE is high, especially when there is a low pretest probability; however, this test is not of much use in pregnant women because D-dimer levels progressively
increase throughout pregnancy. After delivery, D-dimer levels drop but not to the baseline for 4-6 weeks.

References

• UpToDate - Deep vein thrombosis and pulmonary embolism in pregnancy: Treatment

• Medscape - PE in Pregnancy: A Complicated Diagnosis

• Diagnostic Imaging Pathways - PE in pregnancy

• Medscape - Pulmonary Embolism in Pregnancy - Workup

L t d t d

888 of 1943
Last updated:
Time spent: QID:582 2023-2-12

889 of 1943
A 35-year-old woman develops deep vein thrombosis (DVT) of her left lower leg at 20 weeks pregnancy. Which one of the following statements is true regarding DVT in pregnancy?

A. Decreased levels of protein C and S is a precipitating factor.

B. DVT of right leg is more common than left leg.

C. Decreased levels of factor V and Vll is a precipitating factor.

D. Deep vein thrombosis is a rare condition in pregnancy.

E. Warfarin is the treatment of choice.

Correct
45% answered correctly

Explanation:

Correct Answer Is A

Pregnancy is considered a hypercoagulable state caused by a decrease in anticoagulation activity due to a drop in proteins C and S, and an increase in activity of pro-coagulants, factor V and Vll.

Such physiological changes make DVT a common complication in pregnancy with an incidence rate of 1 in 2200 of pregnancies in Australia.

In pregnant women, deep vein thrombosis occurs much more commonly in the left leg than in the right leg (85%) due to compression of the left iliac vein by the pregnant uterus.

Anticoagulant therapy with low-molecular weight heparin (LMWH) or unfractionated heparin is the treatment of choice in pregnant women with DVT.

Warfarin therapy is contraindicated throughout pregnancy due to association with fetal warfarin syndrome and increased risk of fetal intracranial hemorrhage in the second and third trimesters.

References

• UpToDate - Deep vein thrombosis in pregnancy: Epidemiology, pathogenesis, and diagnosis

• Medscape - PE in Pregnancy: A Complicated Diagnosis

Last updated:
Time spent: QID:583 2023-2-12

890 of 1943
A 30-year-old woman developed deep vein thrombosis at 30 weeks pregnancy and was started on the therapeutic dose of low molecular weight heparin (LMWH). Now and at 34 weeks gestation, she is in your office for a follow-up visit. Her expected
date of delivery is due in 4 weeks. Which one of the following would be the best advice for her today?

A. The LMWH should be replaced with unfractionated heparin.

B. Continue the LMWH until delivery.

C. Switch to warfarin and check INR in 2 days.

D. Add warfarin to the LMWH.

E. Cease anticoagulation.

Correct
45% answered correctly

Explanation:

Correct Answer Is A

A patient with deep vein thrombosis during pregnancy requires 3-6 months of treatment with therapeutic dose of LMWH or subcutaneous unfractionated heparin (UFH). The duration of therapy depends on the site of the thrombosis.

LMWH may be associated with an increased risk of epidural hematoma in women receiving epidural anesthesiae in labor; therefore, it is recommended that it be switched to subcutaneous UFH at 36 weeks or sooner if delivery is expected earlier to
make regional anesthesia at labor (if indicated) safe.

Subcutaneous UFH can be discontinued 24-36 hours prior to anticipated delivery or induction. If there is high risk of thromboembolism in the absence of anticoagulation, the patient can be started on intravenous UFH because it has a shorter half-life.
Intravenous UFH can be discontinued 4-6 hours before delivery.

Prior to neuraxial anesthesia, an ATTP should be checked.

Anticoagulation can be restarted with UFH or LMWH 4 to 6 hours after vaginal delivery or 6-12 hours after cesarean delivery. If a neuraxial catheter was used, anticoagulation should not be restarted for at least 12 hours after it is removed.

References

• Medscape - Thromboembolism in Pregnancy

• UpToDate - Deep vein thrombosis and pulmonary embolism in pregnancy: Treatment

Last updated:
Time spent: QID:584 2023-2-12

891 of 1943
Warfarin is contraindicated during pregnancy. Which of the following complications develop if warfarin is used in second trimester of pregnancy?

A. Fetal chondrodysplasia punctata.

B. Hypercalcemia.

C. Facial anomalies.

D. Maternal cerebral bleeding.

E. Fetal optic atrophy.

Incorrect. Correct answer is E


45% answered correctly

Explanation:

Correct Answer Is E

Warfarin crosses the placenta and should be avoided throughout pregnancy, especially during the first and third trimesters. Warfarin at 6-12 weeks’ gestation results in fetal warfarin syndrome characterized by the following features:

A characteristic nasal hypoplasia


Short fingers with hypoplastic nails
Calcified epiphyses, namely chondrodysplasia punctuta (evident by stippling of epiphyses on X-ray)
Intellectual disability
Low birth weight

Recent estimates indicate the risk of fetal warfarin syndrome in babies of women who require warfarin throughout pregnancy is around 5 %. These effects are dose dependent.

Later exposure (after 12 weeks) is associated with central nervous system anomalies including microcephaly, hydrocephalus, agenesis of corpus callosum, Dandy-Walker malformation (complete absence cerebellar vermis and enlarged fourht
ventricle) and mental retardation, as well as eye anomalies such as optic atrophy, microphthalmia, and Peter anomaly (anterior segment dysgenesis). Blindness can be seen in newborns exposed to warfarin in all three trimesters. Perinatal
intracranial and other major bleeding is another complication in neonates exposed to warfarin.

NOTE – There may be a place for mid-trimester warfarin in pregnant women with prosthetic heart valves, but this choice should be made only after full discussion of the implications with the patient. Unfractionated heparin and low molecular
weight heparin are alternatives that do not cross the placenta.

References

• SA Health - Cardiac disease in pregnancy

• Australian Prescriber - Managing warfarin therapy in the community

Last updated:
Time spent: QID:585 2023-2-12

892 of 1943
A 31-year-old woman develops deep vein thrombosis (DVT) one week after delivery. She is started on enoxaparin. You are planning to start her on warfarin on discharge, but the mother is reluctant to take warfarin because she wants to breastfeed
her baby and she thinks that warfarin may cause problems to her child. Which of the following is correct about warfarin and breastfeeding?

A. She cannot breastfeed her baby as long as on any type of anticoagulation as this increases he risk of hemorrhage in the baby.

B. She can breastfeed her baby while she is on warfarin.

C. Warfarin is contraindicated in breastfeeding mothers.

D. She should take enoxaparin injections for six months before warfarin can be started.

E. She can go home without anticoagulation as breastfeeding of her baby is a priority.

Incorrect. Correct answer is B


45% answered correctly

Explanation:

Correct Answer Is B

Low molecular weight heparin (enoxaparin) is the medication of choice for antenatal thromboprophylaxis.

After delivery, warfarin can be used safely as the anticoagulation method. Both warfarin and heparin are safe to use in breastfeeding mothers. These do not increase the risk of bleeding in the baby because only a very small amount is secreted in
breast milk.

References

• QLD Health - Guidelines for warfarin management in the community

Last updated:
Time spent: QID:586 2023-2-12

893 of 1943
Of the following options, which one is NOT associated with an increased risk of preeclampsia?

A. Nulliparity.

B. History of preeclampsia in a first-degree relative.

C. Chronic renal disease.

D. Age between 18 and 40 years.

E. Prolonged intervals between pregnancies.

Incorrect. Correct answer is D


45% answered correctly

Explanation:

Correct Answer Is D

Preeclampsia refers to the onset of hypertension and proteinuria after 20 weeks of gestation in a previously-normotensive woman.

Risk factors for pre-eclampsia include:

Preeclampsia in a previous pregnancy


Family history of preeclampsia
Poor outcome in a prior pregnancy (placental abruption, IUGR, fetal death in utero)
Interdelivery interval greater than 10 years
Nulliparity (8 times more risk)
Pre-existing medical conditions or chronic hypertension
Diabetes (pre-existing or gestational)
Renal disease
Thrombophilias e.g. protein C and S deficiency, antithrombin III deficiency, or Factor V Leiden mutation
Antiphospholipid syndrome
Systemic lupus erythematosus
Maternal age greater than or equal to 40 years
Body Mass Index (BMI) greater than 30 kg/m2
Multiple pregnancy
Raised blood pressure at booking
Gestational trophoblastic disease
Fetal triploidy

Age between 18 and 40 years is associated with decreased risk of preeclampsia, not an increased risk.

NOTE - Previously, age 16 years or younger was considered a risk factor for preeclampsia; however, recent studies failed to establishe a meaningful relationship.

References

• Risk factors for pre-eclampsia at antenatal booking: systematic review of controlled studies

Last updated:
Time spent: QID:587 2023-2-12

894 of 1943
A 20-year-old Asian woman with background history of primary pulmonary hypertension attends your clinic. She is planning to become pregnant in the next few months. She enjoys good health at the moment. Which one of the following would be the
most appropriate advice?

A. Pulmonary hypertension becomes better during pregnancy.

B. If she becomes pregnant, termination of pregnancy is not required.

C. Pregnancy is contraindicated for her.

D. Maternal mortality in this setting is low.

E. Sudden death is a rare complication.

Incorrect. Correct answer is C


45% answered correctly

Explanation:

Correct Answer Is C

Primary pulmonary hypertension is a contraindication to pregnancy and the patient should be advised on the risks of increased maternal mortality in the presence pulmonary hypertension.

With unwanted pregnancy, this woman should be encouraged to terminate the pregnancy, as pulmonary hypertension becomes worse in pregnancy and can result in the mother’s death. Sudden death secondary to hypotension is a common and
feared complication of pulmonary hypertension during pregnancy.

References

• SA Health - Cardiac disease in pregnancy

Last updated:
Time spent: QID:588 2023-2-12

895 of 1943
Which one of the following is the most important single warning sign of diminishing blood volume within the first four hours postpartum?

A. Decrease in blood pressure.

B. Tachycardia.

C. Tachypnea.

D. Sweating.

E. Vomiting.

Incorrect. Correct answer is B


45% answered correctly

Explanation:

Correct Answer Is B

For early recognition of postpartum hemorrhage, routine observation of all postpartum women for blood loss, fundal tone, blood pressure and pulse is carried out during the first four hours after delivery.

The single most important sign of diminished blood volume and mild shock is tachycardia, which often precedes a fall in blood pressure. Weakness, sweating and tachypnea may accompany. Vomiting is not of any significance for early recognition
of shock.

References

• http://www.ranzcog.edu.au/college-statements-guide

• http://www.elsevieradvantage.com/samplechapters/97

Last updated:
Time spent: QID:105 2023-2-12

896 of 1943
A 28-year-old woman presents for antenatal visit at 39 weeks gestation. On examination, the fetus is found to be in transverse lie. Which one of the following would be the most appropriate next step in management?

A. Immediate ultrasonography.

B. Vaginal examination.

C. External cephalic version.

D. Lower segment cesarean section.

E. Ask her to come back in two weeks.

Correct
45% answered correctly

Explanation:

Correct Answer Is A

Shoulder presentations, unstable lie, transverse lie and oblique lie may be detected in late pregnancy.

Contributing factors include:

High parity
Pendulous abdomen
Placenta previa
Polyhydramnios
Pelvic inlet contracture and/or fetal macrosomia
Uterine abnormalities (e.g. bicornuate uterus or uterine fibroids)
Fetal anomaly (e.g. tumors of the neck or sacrum, hydrocephaly, abdominal distension)
Distended maternal urinary bladder
Poorly formed lower segment
Wrong dates i.e. more premature than appears
Undiagnosed twins

The diagnosis of transverse lie is made clinically by the presence of a broad asymmetric uterus with a firm ballotable round head in one iliac fossa and a softer mass (buttock) in the other.

An ultrasonography is required as the next step to is if there is:

Placenta previa
Twins
Fetal anomaly
Pelvic tumor
Polyhydramnios

When placenta previa has been excluded, a gentle cephalic version may be tried; in many cases, however, the fetus returns to its previous presentation. If the lie remains unstable, the expectant mother should be advised to report to hospital
immediately once the labor starts. Those with unfavorbale social conditions may be admitted to the hospital for expectant management.

In some cases after the cephalic version, the head is held over the pelvic brim and with the head in place amniotomy is performed to let the head be pressed into the pelvis as the amniotic fluid is released. Cesarean section is another option.

If these presentations are found once the labor has been established, cesarean section is the preferred method.

References

• https://www.sahealth.sa.gov.au/wps/wcm/connect/249

• http://www.gpnotebook.co.uk/simplepage.cfm?ID=1697

Last updated:
Time spent: QID:591 2023-2-12

897 of 1943
An 18-week pregnat woman is brought to the Emergency Department with complaint of sudden-onset severe frontal headache. On examination, she has a blood pressure of 80/60 mmHg, pulse of 110bpm, respiratory rate of 17 breaths per minute
and temperature of 37.3°C. The rest of the examination is unremarkable. There is no neck stiffness, abdominal pain or tenderness, or uterine contractions. Which one of the following is the investigation more likely to establish the diagnosis?

A. Lumbar puncture.

B. CT scan of the head.

C. Ultrasonography.

D. MRI of the head.

E. Full blood exam (FBE) and liver function tests (LFT).

Incorrect. Correct answer is D


45% answered correctly

Explanation:

Correct Answer Is D

With the clinical findings of frontal headache and hypotension in a pregnant woman, pituitary apoplexy should be considered as one of the most important differential diagnoses. Pituitary apoplexy is defined as sudden hemorrhage into the pituitary
gland. Hemorrhage often occurs into a pituitary adenoma.

The most dramatic presentation of apoplexy is the sudden onset of excruciating headache, diplopia due to pressure on the oculomotor nerve, and hypopituitarism.

All pituitary hormonal deficiencies can occur, but a fall in ACTH and, consequently, cortisol is most serious because it can cause life-threatening hypotension.

In a series of 35 patients with pituitary apoplexy seen at one center, 97% had headache, 71% had visual field impairment, and 66% had decreased visual acuity. Only a minority had clinical manifestations of hormonal excess or deficiency, but there
was biochemical evidence of gonadotropin deficiency in 79%, ACTH deficiency in 76%, and TSH deficiency in 50%.

CT scan and MRI can be used for detection of intra-pituitary hemorrhage; however, MRI is more sensitive for evaluation of the pituitary gland.

Hypopituitarism and the diplopia may improve after surgical decompression of the pituitary. Both problems may also improve spontaneously as the blood is absorbed over a course of weeks to months after the hemorrhage.

For hypotension, high-dose corticosteroids are the mainstay of therapy.

(Options A and B) With headache and hypotension, subarachnoid hemorrhage can be another differential diagnoses, but headache of SAH tends to be much more severe (the worst headache in life) and often associated with neck stiffness. On the
other hand, skull cannot accommodate enough blood to justify the hypotension and the tachycardia in this patient. CT scan of the head (non-contrast) and LP (if CT was inconclusive) are used if SAH is suspected.

(Option C) Ultrasonography is of no use in assessment of this patient with a possible intracranial pathology.

(Option E) Preeclampsia is defined as persistent hypertension and proteinuria after 20 weeks’ gestation. FBE, LFT, and urinalysis should be considered for excluding preeclampsia. This woman with hypotension is less likely to have preeclampsia.

References

• RANZCOG - Pituitary apoplexy in pregnancy: a case report

• Pituitary apoplexy in pregnancy: A case series and literature review

Last updated:
Time spent: QID:592 2023-2-12

898 of 1943
A 31-year-old woman presents to your GP clinic for planning her mode of delivery at 38 weeks’ gestation. She insists that her mode of delivery be a caesarean section because she strongly believes that this is best for both her and her baby. She has
had an uneventful pregnancy so far, and has no indications for cesarea. You fully explain about the risks of cesarean delivery and its complications to her and the baby, but she insists. Which one of the following statements is correct regarding this
situation?

A. Comply with her wish because she is right about the safety of cesarean section.

B. She should take a second opinion from an obstetrician.

C. Refuse to refer her for cesarean delivery because it is not indicated.

D. Refer her to an obstetrician for cesarean delivery.

E. Ask her to bring her husband for more discussion on the issue.

Incorrect. Correct answer is B


45% answered correctly

Explanation:

Correct Answer Is B

The scenario is a case of cesarean delivery on maternal request (CDMR) that is defined as elective cesarean delivery for singleton pregnancy on maternal request at term in the absence of any medical or obstetrical indication.

In such cases and as a GP, if the patient is still adamant to undergo cesarean delivery, referral to an obstetrician for further counselling is the next best step in management.

If after full discussion the patient maintains a request for delivery by cesarean section, the obstetrician may consider either of the following:

Agree to perform the cesarean section, providing the patient is able to demonstrate an understanding the risks and benefits.

Decline to perform the cesarean section in circumstances where: (1) the obstetrician believes there are significant health concerns to the mother or baby if cesarean section is performed or (2) the patient appears to not have an
understanding sufficient to enable informed consent to the procedure.

References

• The Royal Australian and New Zealand College of Obstetricians and Gynaecologists - Caesarean Delivery on Maternal Request (CDMR) C-Obs 39

Last updated:
Time spent: QID:593 2023-2-12

899 of 1943
A 32-year-old woman presents with placenta previa. You offer cesarean section as the most appropriate mode of delivery. She wants to know about the risks of cesarean delivery.
Which one of the following is correct regarding cesarean section?

A. Decreased risk of maternal mortality.

B. Decreased need for repeated cesarean sections.

C. No damage to adjacent viscera.

D. Increased risk of adhesions.

E. Decreased risk of infections.

Incorrect. Correct answer is D


45% answered correctly

Explanation:

Correct Answer Is D

Complications of caesarean section delivery may include:

Maternal complications

Increased risk of maternal mortality


Increased risk of thromboembolism
Hemorrhage
Infection
Incidental surgical injuries
Extended hospitalization
More postpartum pain
Poor birth experience
Re-admission to the hospital
Adhesion formation
Infertility/subfertility
Uterine rupture in subsequent pregnancies
Abnormal placentation in subsequent pregnancies

Neonatal complications

Increased risk of neonatal death


Increased neonatal respiratory problems
Asthma
Iatrogenic prematurity
Trauma especially lacerations with cuts (particularly in emergency cesarean sections)
Failure to breastfeed

Cesarean section is associated with increased risk of infections as a complication.

References

• http://www.medscape.org/viewarticle/512946_4

Last updated:
Time spent: QID:594
2023-2-12

900 of 1943
A 25-year-old pregnant woman comes to your clinic with vaginal bleeding at 14 weeks’ gestation. Which one of the following is the main reason for performing ultrasonography?

A. Excluding ectopic pregnancy.

B. Excluding neural tube defects.

C. Determining the cause of bleeding.

D. Determining if the fetus is alive.

E. Excluding congenital heart defects.

Incorrect. Correct answer is C


45% answered correctly

Explanation:

Correct Answer Is C

Transvaginal ultrasound is the cornerstone in evaluation of vaginal bleeding in the second trimester. The main reason for ultrasonography is to find an underlying cause for the bleeding. Reasons to perform an ultrasound include:

To determine whether the placenta is covering the cervical os (placenta previa).


Whether there is evidence of decidual hemorrhage causing placental separation (abruption of placenta)
Whether the cervix shows signs suggestive of cervical insufficiency (short length, dilated internal os, funneling of the fetal membranes).
Whether there is a cervical polyp

The common causes of vaginal bleeding in second and third trimesters of pregnancy are:

Bloody show associated with cervical insufficiency or labor


Placenta previa
Placental abruption
Uterine rupture
Vasa previa

References

• http://www.uptodate.com/contents/overview-of-the-e

Last updated:
Time spent: QID:595 2023-2-12

901 of 1943
You are working at a busy GP clinic. Your next patient is 24 -year-old woman with irregular menstrual and a positive pregnancy test. She is keen to know the age of her baby. Which one of the following methods would be the most accurate one for
estimation of gestational age?

A. Bimanual examinations at 10 weeks.

B. Ultrasound at 16 weeks.

C. Transvaginal ultrasound at 8 weeks.

D. Transvaginal ultrasound at 20 weeks.

E. Transabdominal ultrasound at 20 weeks.

Incorrect. Correct answer is C


45% answered correctly

Explanation:

Correct Answer Is C

By eight weeks gestation, the fetus and its heart beat can be detected relatively easy with transvaginal ultrasound. Dating scans are usually recommended if there is doubt about the validity of the last menstrual period, such as in the following
conditions:

Patient does not know when the first day of her last period or the likely day of conception was
Patient with irregular periods
Patients who has become pregnant while on hormonal contraceptives

Transvaginal ultrasonography performed between weeks 8 and 12 (within the first trimester) can predict the gestational with three to five days difference and is the most accurate method of determining the gestational age.

As the pregnancy advances, sonography becomes less accurate in estimation of gestational age.

References

• http://sogc.org/wp-content/uploads/2014/02/gui303C

Last updated:
Time spent: QID:596 2023-2-12

902 of 1943
A 37-year-old woman presents to you with history of irregular periods and decreased libido for the past 6 months. Which one of the following would be the investigation of choice?

A. Serum prolactin level.

B. Ultrasound scan of the ovaries.

C. Serum FSH and LH.

D. Serum FSH and estradiol.

E. Thyroid function tests.

Incorrect. Correct answer is D


45% answered correctly

Explanation:

Correct Answer Is D

In women younger than 40 years, who present with secondary amenorrhea or menstruation irregularities and signs of estrogen deficiency such as decreased libido, atrophic vaginitis, etc, the most common cause is found to be primary ovarian failure
(POF) (premature menopause) or premature ovarian insufficiency (POI), for which an elevated FSH associated with decreased estradiol level is diagnostic.

Measuring serum FSH level is the core study to establish the diagnosis of POI/POF after pregnancy has been excluded. By convention, 2 FSH levels in the menopausal range for the specific assay (>40 µIU/mL by radioimmunoassay), measured at
least 1 month apart, are diagnostic of POI/POF. A parallel test of serum estradiol is necessary. As a rule, serum estradiol is low in women with POI/POF and is similar to or less than the early follicular phase estradiol of women who cycle normally.
The combination of low estradiol and high gonadotropins defines POI/POF.

Measurement of serum LH is also important. In most cases of spontaneous POI/POF, FSH is higher than LH. If autoimmune oophoritis is present, FSH may be only mildly elevated, sometimes below the cutoff of 40 µIU/mL, while LH is markedly
elevated.

Occasionally, women with POI/POF may have spontaneous follicular activity. In such women, if hormonal tests are performed during such episodes, FSH, LH, and estradiol levels could be in the normal range or only minimally elevated. This may lead
to an erroneous exclusion of POI/POF. In these cases, persistent amenorrhea or oligomenorrhea accompanied by menopausal symptoms necessitates a repeat of the above tests in 1-2 months.

References

• http://emedicine.medscape.com/article/271046-worku

Last updated:
Time spent: QID:597 2023-2-12

903 of 1943
Which of the following is not a risk factor for isolated spontaneous abortion?

A. Age more than 35 years.

B. Cigarette smoking.

C. High doses of caffeine.

D. Uterine adhesions.

E. Retroverted uterus.

Incorrect. Correct answer is E


45% answered correctly

Explanation:

Correct Answer Is E

The following are known risk factors for spontaneous abortion or miscarriage:

Advanced maternal age (≥35)


Previous miscarriage(s)
Antiphospholipid syndrome
Parentral chromosomal derangements
Embryonic chromosomal abnormalities
Congenital uterine malformations
Cervical weakness
Diabetes mellitus (subclinical disease excluded) and thyroid disease (subclinical disease excluded)
Immune factors
Infections
Inherited thrombophilic defects
Caffeine, smoking and alcohol use (dose dependent)

A retroverted uterus has not shown association with increased risk of spontaneous abortions. A retroverted uterus is a uterus that is tilted backwards instead of forwards.

References

• The royal college of obstetricians and Gynaecologists – Green-top Guideline No.17

Last updated:
Time spent: QID:598 2023-2-12

904 of 1943
A 24-year-old woman underwent dilation and curettage for septic abortion. Now, she has developed amenorrhea of six months duration. An office pregnancy test excludes pregnancy. She smokes 10 cigarettes and drinks two standard units of
alcohol every day. Which one of the following is the most appropriate next investigation to reach a cause for this presentation?

A. Liver function tests.

B. Urine analysis and microscopy.

C. Transvaginal ultrasound.

D. Full hormone assay.

E. MRI of the brain.

Incorrect. Correct answer is C


45% answered correctly

Explanation:

Correct Answer Is C

Intrauterine adhesions are a common complication of curettage. Approximately 90% cases of severe intrauterine adhesions are related to curettage for pregnancy complications such as missed or incomplete abortion, postpartum hemorrhage, or
retained placental tissue.

Intrauterine adhesions can be asymptomatic and of no clinical significance. If there are symptoms they can include:

Infertility
Menstrual irregularities (amenorrhea)
Cyclic pelvic pain
Recurrent miscarriages

When intrauterine adhesions are suspected, transvaginal ultrasonography is the next best investigation to confirm the diagnosis. Although not always necessary, diagnostic hysteroscopy remains the gold standard diagnostic investigation.

References

• UpToDate - Intrauterine adhesions

Last updated:
Time spent: QID:599 2023-2-12

905 of 1943
A 20-year-old female presents to your practice at 18 weeks pregnancy with right iliac fossa pain that is particularly brought on by getting up from a chair, sneezing and coughing. On examination she has normal vital signs and is otherwise healthy.
Abdominal examination reveals no tenderness, rebound tenderness or guarding. Which one of the following is the most likely diagnosis?

A. Round ligament pain.

B. Acute appendicitis.

C. Ovarian torsion.

D. Ruptured ectopic pregnancy.

E. Uncomplicated ectopic pregnancy.

Correct
45% answered correctly

Explanation:

Correct Answer Is A

This patient has signs and symptom suggestive of round ligament pain. It is considered a normal finding during pregnancy and does not require any intervention.

Round ligament pain most frequently occurs during the second trimester of pregnancy when women report sharp unilateral or bilateral pain in the iliac fossa that may radiate to the groin. The pain is often sudden-onset, sharp and spastic ,lasting few
seconds. The pain is aggravated by standing, getting off chairs, sneezing, laughing or rolling in bed. Sudden change of position is a well-known trigger.

Rest and avoiding sudden changes in body position is the cornerstone of management.

(Option B) Although appendicitis presents with pain in the right iliac fossa, absence of tenderness makes this diagnosis less likely.

(Option C) Ovarian torsion presents with aute onset severe pain followed by signs and symptoms of peritoneal irritation such as tenderness, rebound tendernee, guarding, etc.

(Options D and E) Presentation of ectopic pregnancy and its complications (e.g. rupture) almost always occurs in the first trimester; furthermore, absence of tenderness, rebound tenderness and other localized findings make this diagnosis even less
likely.

NOTE - It should be noted that the question asks about the most likely diagnosis, not the next best step in management. Although, round ligament strain comes top on the list, but more serious conditions should be excluded first through appropriate
examination and investigations.

References

• Danforth’s Obstetrics and Gynaecology – 10th Edition – page 18

Last updated:
Time spent: QID:600 2023-2-12

906 of 1943
A 30-year-old woman presents with a 12-month history of secondary infertility. Her first child, fathered by the same partner, was born 4 years ago after she was conceived spontaneously, and through an uneventful vaginal delivery. She has always had
irregular periods occurring every 2 to 4 months. On examination, she is obese (BMI>32) and otherwise normal. Ultrasonography of the pelvis reveals 12 small cyst of 3-6 mm in diameter in the left and 20 cysts of about the same size in the right
ovary. A sperm analysis of the partner is normal. Which one of the following is the most appropriate next step in management?

A. Metformin.

B. Clomiphene citrate.

C. Ovulation induction with gonadotropins.

D. Laparoscopic ovarian drilling.

E. In-vitro fertilization.

Incorrect. Correct answer is B


45% answered correctly

Explanation:

Correct Answer Is B

The findings on ultrasonography are suggestive of polycystic ovarian syndrome (PCOS). PCOS is the most common cause of infertility due to anovulation. Infertility in women with PCOS, however, is not absolute and many women can conceive even
without treatment. In women with infertility due to PCOS, different options are available:

Non-pharmacological treatment:

If a woman is younger than 35 years of age and has a BMI>25, and no other cause of infertility is suspected an intensive lifestyle program addressing weight loss, without any pharmacological treatment for the first 6 months, is recommended. Small
amounts of weight loss (~5%) may restore menstrual cycle regularity and ovulation, providing benefit even if pharmacological intervention is subsequently required

Pharmacological treatment:

If pharmacological treatment is required, the best first-line treatment is clomiphene citrate, which has a pregnancy rate of 30–50% after six ovulatory cycles.

In women with a BMI <30–32 kg/m2, metformin may have a similar efficacy to clomiphene citrate, and is the first-line treatment (with or without clomiphene citrate) if there is concomitant impaired glucose tolerance).

If clomiphene citrate, metformin or a combination of the two is unsuccessful in achieving pregnancy, gonadotropins are the next pharmacological options.

Laparoscopy with ovarian surgery/drilling (LOS) is an appropriate second-line treatment if clomiphene citrate with metformin has failed. The pregnancy rate with LOS is as effective as 3-6 cycles of gonadotropin ovulation induction.

If all of the above are unsuccessful or if there are other factors contributing to infertility such as endometriosis or male factors, in vitro fertilization or intra-cytoplasmic sperm injection is recommended.

References

• http://www.racgp.org.au/afp/2012/october/polycysti

• Therapeutic Guidelines – Endocrinology; available from http://tg.org.au

Last updated:
Time spent: QID:121 2023-2-12

907 of 1943
A 30-year-old pregnant woman presents to the Emergency Department with severe right-sided throbbing head ache, nausea, and vomiting. She is 24 weeks pregnant. Her medical history is remarkable for migraine. Which one of the following is the
most appropriate management of this patient?

A. Paracetamol.

B. Paracetamol and codeine.

C. Codeine and metoclopramide.

D. Codeine and promethazine.

E. Sumatriptan.

Incorrect. Correct answer is C


45% answered correctly

Explanation:

Correct Answer Is C

This woman is suffering from a migraine attack associate with nauseas and vomiting. In pregnant women with migraine paracetamol is the treatment of choice for mild attacks. For more severe attacks, codeine alone or in combination with
paracetamol can be used.

Codeine is not shown to lead to miscarriage or have teratogenic effects on fetus; however, long-term use of opiates can cause withdrawal (abstinence) syndrome in the neonate.

Because paracetamol alone or with codeine would not stop vomiting, an antiemetic should be added as well to control the nausea and vomiting. Metoclopramide is the antiemetic of choice during pregnancy (category A) . Promethazine is category C
and should be avoided.

TOPIC REVIEW

A step-wise approach to management of an acute migraine attack in a pregnant woman is as follows:

Paracetamol (first-line) - For acute treatment, paracetamol is safe but often inadequate to control the symptoms. Migraine that does not respond to paracetamol alone may be relieved with combination therapy such as paracetamol (650 to 1000 mg)
and metoclopramide (10 mg); paracetamol-codeine.

For migraine with nausea and vomiting, metoclopramide (category A) can be safely added. Prochlorperazine can be used as an alternative to metoclopramide.

Women with migraine that has not responded to these drugs after several days should be evaluated for provoking factors and treated more aggressively with the following medications in a step-wise approach.

NSAIDs and aspirin (second-line) - NSAIDs are second-line options, and safest in the second trimester. In the first trimester, an association with miscarriage and some birth defects (ventricular septal defect, gastroschisis) has been suggested. In the
third trimester, their use should be limited to fewer than 48 hours due to concerns about premature ductus arteriosus closure, platelet inhibition, and oligohydramnios.

Opioids (third-line) - Opiates (e.g., oxycodone, meperidine, morphine, etc.) can be given by rectal, intravenous, or intramuscular administration. Opioids can be useful for treatment in women with nausea and vomiting.

Triptans (fourth-line) - For moderate to severe symptoms in patients who do not respond to other drugs, triptans can be considered.

NOTE – Ergotamine is absolutely contraindicated throughout pregnancy.

OF the options, codeine plus metoclopramide is the most appropriate one for a severe headache and nausea of this woman.

References

• AAFP - Treatment of Acute Migraine Headache

• UpToDate - Headache in pregnant and postpartum women

• Australian Prescriber

• Therapeutic Guidelines – Neurology

Last updated:
Time spent: QID:602 2023-2-12

908 of 1943
A 35-year-old woman pregnant woman presents to the Dmergency Department with left-sided retro-orbital and occipital severe headache associated with nausea and vomiting. She has been suffering from migraine for the past 10 years, and has
been on treatment with sumatriptan. Which one of the following drugs if used for treatment of migraine would lead to premature closure of fetal ductus arteriosus?

A. Codeine.

B. Paracetamol.

C. Metoclopramide.

D. Non-steroidal anti-inflammatory drugs (NSAIDs).

E. Sumatriptan.

Incorrect. Correct answer is D


45% answered correctly

Explanation:

Correct Answer Is D

For treatment of migraine in a pregnant woman, NSAIDs should be used with caution and only if paracetamol with or without codeine/metoclopramide fails to control the pain. If NSAIDs are indicated, they should be used not more than 48 hours, and
not in late pregnancy, because they are associated with premature closure of fetal ductus arteriosus. Aspirin has the same effect and should be avoided as well.

Other effects of NSAIDs on fetus include:

Delayed labor and birth


Oligohydramnios via decreasing the glomerular filtration rate in the fetus

Other mentioned drugs are not associated with premature closure of ductus arteriosus.

References

• UpToDate- Headache in pregnant and postpartum women

• Australian Prescriber - Treatment of nausea and vomiting in pregnancy

Last updated:
Time spent: QID:603 2023-2-12

909 of 1943
A pregnant woman has come to you because two days after babysitting her friend’s son, he was diagnosed with infectious mononucleosis. She is worried about herself and her baby’s well-being. Currently, she is asymptomatic. Which one of the
following will be the most appropriate action for now?

A. Order an ultrasound for assessment of fetal hydrops.

B. Check serology for antibody titres.

C. Reassure her.

D. Tell her to come back if any symptoms develop.

E. Refer her to specialist.

Incorrect. Correct answer is D


45% answered correctly

Explanation:

Correct Answer Is D

Epstein-Barr virus (EBV) is a human herpes virus with variable incubation period that may cause infectious mononucleosis. This virus can remain latent in the body and become reactivated at a later time.

In Australia, EBV is more common among women aged 15 and 19 years, but in developing countries it is more common among children. The route of transmission is sharing oral secretions (saliva). EBV has an incubation period of 2-7 weeks after
exposure.

Clinical features of EBV include:

Fever
Sore throat
Lymphadenopathy
Characteristic increase in the percentages of monocytes and lymphocytes (mononucleosis and lymphocytosis)
Hepatosplenomegaly
Rise in hepatic transaminases

Of all pregnant women, only 3.0% to 3.4% are susceptible and of those infected, only 50% develop clinical disease.

In several recent studies, EBV infection was not transmitted to the fetus and there were no adverse effects. The risk of intrauterine transmission of EBV infection is considered to be low, even when the mother is symptomatic; however, reactivation of
EBV in pregnancy may carry a small risk of a shortened pregnancy duration and lower birth weight.

Recent primary EBV infection is diagnosed by the presence of viral capsid antigens (VCA) IgG and IgM antibodies in the absence of antibodies to EBV-associated nuclear antigen which develop 3 to 4 weeks after primary infection.

It is recommended that serology for IgG and IgM antibodies against viral capsid antigens (VCA) be obtained soon after symptoms of infection. About 80% of those infected form antibodies to early antigens, which usually fall to undetectable levels by
six months afterwards. The presence of antibodies against early antigens at later times after acute infection indicates possible viral reactivation.

Management is supportive with rest, fluids and analgesia if required. Most pregnant women will have a gradual, uneventful recovery after an acute phase lasting several days to 3 or 4 weeks. Fever usually resolves within two weeks. Abnormal liver
transaminases occur in about 10 % of cases. Nausea, anorexia and possibly vomiting can be expected. Significant organomegaly usually resolves within 1 to 3 months. Recovery from severe fatigue may occur quickly, however a full recovery to a
feeling of wellbeing may take several months.

This woman has a 3.0 – 3.4% risk of infection with EBV. Even if she shows clinical infection, the potential risks to the fetus are negligible, and she can be safely reassured that no harm from EBV threatens her pregnancy. She, however, may have
become infected and show clinical infection, in that case further management (including serology) would be required. For this reason she should be warned that the risk of developing the disease is small yet possible and she should come back for
further management if any symptom develops.

References

• Perinatology.com - Perinatal Infections

Last updated:
Time spent: QID:613 2023-2-12

910 of 1943
Yasmin®, containing drospirenone 3mg and ethinylestradiol 30 mcg, has been released for use in Australia. Which one of the following has made it the preferred oral contraceptive pill among Australian women?

A. It has not weight gain as an adverse effect and may be associated with weight loss.

B. It has a failure rate of less than other OCPs.

C. It has a protective effect against cervical cancer.

D. It causes less spotting even at the very beginning of use.

E. It has a good effect on acne.

Correct
45% answered correctly

Explanation:

Correct Answer Is A

Yasmin® has been shown to be associated with less fluid retention and weight gain as one of the complication of COCs, and this the main reason Yasmin® is preferred by most women suffering from this adverse effect. Unlike older progestogens,
drospirenone is associated with no increase in weight or eve slight weight loss due to its anti-mineralocorticoid effects.

(Option B) The failure rate Yasmin® is about the same as other COCs. There is no study showing that use of Yasmin® is associated with less incidence of cervical cancer as a long-term adverse effect of COCs (option C) . As with all COCs, Yasmin
can cause irregular bleeding and spotting within the first few months of use (option D) .

(Option E) The progesterone component – drospirenone has antiadorgenic effects and is slightly more effective in treatment of acne, but compared to other COCs, the difference is not significant enough to make it preferable in terms of acne
treatment of prevention.

References

• AMC Handbook of Multiple Choice Questions – page 533

• http://www.ncbi.nlm.nih.gov/pubmed/16203653

• http://www.uptodate.com/contents/hormonal-therapy-

Last updated:
Time spent: QID:619 2023-2-12

911 of 1943
A 29-year-old obese woman comes to you for prescription of oral contraceptive pill (OCP). She weighs 115kg and has a BMI of 35. She also has hirsutism and acne. She mentions that she has migraine headaches at occasions, associated with pins
and needles in her left arm. Which one of the following would be the most appropriate contraception method for her?

A. Condoms.

B. Implanon®.

C. An OCP containing ethinylestradiol and norgestrel.

D. An OCP containing ethinylestradiol and cyproterone acetate.

E. An OCP containing ethinylestradiol and drospirenone.

Correct
45% answered correctly

Explanation:

Correct Answer Is A

This woman has classic migraine associated with focal neurological findings. In such patients use of OCP preparation of any kind containing estrogen is absolutely contraindicated. Progesterone has androgenic effects such as hirsutism, acne and
weight gain. For a woman of her weight, progesterone of any kind (norgestrel, drospirenone, cyproterone, etc) is better avoided; therefore, a barrier method such as male condoms will be the most appropriate advice.

When choosing a combined oral contraceptive pill (COCP), it is recommended that preparations containing 20-30 mcg ethinylestradiol be considered first. The progesterone component can be norgestrel, drospirenone, cyproterone, etc; however, the
preparations containing norgestrel are cheaper and more affordable for patients.

Considerations should be given to the progesterone type for particular patients:

Patients with bothersome fluid retention and weight gain as a side effect of COCPs can be prescribed preparations containing drospirenone (Yaz®, Yasmin®). Drospirenone has anti-mineralocorticoid activity and does not lead to fluid
retention. It may even be associated with slight weight loss.
If the patient has probable polycystic ovarian syndrome (PCOS), a preparation containing cyproterone acetate is preferred.

TOPIC REVIEW

Combination oral contraceptives (COCs) with antiandrogenic progestins are a subclass of COCs. These include agents that contain cyproterone acetate, drospirenone, or dienogest plus an estrogen (ethinylestradiol).

Yasmin ® (ethinylestradiol 30mcg/drospirenone 3 mg) and Yaz ® (ethinylestradiol 20mcg/drospirenone 3mg) are the two drospirenone-containing oral contraceptives. These two COCs are shown in studies to be effective in treatment of acne and
other androgenic effects such as hirsutism, PCOS, etc; however, they are only slightly superior to other COCs. The effective component of COCs against acne is estradiol and all low-dose COCs are estrogen dominant, which effectively makes all of
these agents antiandrogenic and effective.

Some studies,however, suggest that their use is associated with a 2- to 3-time increase in venous thromboembolic events. Although their use is only contraindicated in the presence of active venous thromboembolism (e.g. DVT, PE, etc)

References

• AMC Handbook of Multiple Choice Questions – page 533

• http://www.uptodate.com/contents/hormonal-therapy-

• http://www.fpnotebook.com/mobile/gyn/pharm/Ysmn.ht

Last updated:
Time spent: QID:620 2023-2-12

912 of 1943
A 36-year-old woman presents to your clinic for advice regarding diabetes mellitus. She has 3 children, and was diagnosed with gestational diabetes mellitus in her second pregnancy at the age of 32 years. Which one of the following tests would be
the most appropriate screening test for her?

A. Fasting blood sugar (FBS), 3 yearly.

B. Oral glucose tolerance test (OGTT), 2 yearly.

C. OGTT, yearly.

D. HbA1C, now.

E. FBS, 2 yearly.

Incorrect. Correct answer is D


45% answered correctly

Explanation:

Correct Answer Is D

All women diagnosed with GDM should have a 75 g OGTT at 6-12 weeks postpartum. Additionally and due to the fact that women with GDM have a 50% risk of developing type 2 DM within 20 years, they need to be tested for DM. Based on current
guidelines by the Royal Australian College of General Practitioners (RACGP) all women with GDM should undergo a fasting OGTT with 75 g glucose at weeks 6-12 postpartum and fasting blood sugar (FBS) or glycated hemoglobin (HbA1C) every 3
years thereafter. Since this woman has not been screened for diabetes until now, the most important step would be ordering an FBS or HbA1c now.

NOTE - Different guidelines mention different intervals for DM screening in women with history of GDM. For example Therapeutic Guidelines and Australian Diabetes in Pregnancy Society, recommend 2-yearly 75 g OGTT as the screening test
of choice. We have chosen the correct answer based on RACGP guidelines which are important AMC MCQ references for the AMC MCQ exam.

References

• RACGP - Gestational diabetes mellitus

• Therapeutic Guidelines

Last updated:
Time spent: QID:629 2023-2-12

913 of 1943
A 28-year-old primigravida woman presents at 24 weeks gestation after she noticed vaginal bleeding of approximately 50ml. Last week, she also had a 10ml vaginal bleeding, for which ultrasonography was performed revealing placenta previa grade
IV. Which one of the following would be the most appropriate next step in management?

A. Transvaginal ultrasonography.

B. Blood group and cross match.

C. Anti-D (RhoGAM).

D. Abdominal ultrasonography.

E. Induction of labor.

Incorrect. Correct answer is B


45% answered correctly

Explanation:

Correct Answer Is B

The scenario describes vaginal bleeding in the second half of the pregnancy (antepartum hemorrhage), most likely caused by the previously diagnosed grade IV placenta previa.

The usual history includes a first episode of vaginal bleeding occurring at home. Bleeding is not often heavy. This can be followed by more episodes of bleeding.

The first priority in cases presenting with antepartum hemorrhage is assessment of vital signs, estimation of the blood loss (both clinically and by using laboratory tests), determination of blood group and cross matching.

Even an asymptomatic currently-stable placenta previa can lead to massive bleeding at anytime. This is even more likely with higher grades of placenta previa, or if the first episode of bleeding occurred prior to 30 weeks’ gestation.

The abdomen should be palpated for any uterine tenderness, contraction and fetal presentation. An abdominal ultrasonography is needed to confirm the diagnosis as well as gathering additional information regarding the situation.

Fetal well-being should be assessed by cardiotocography, and corticosteroids administered (if indicated) to promote fetal lung maturity because the feared risk of premature labor.

No vaginal examination should be made before placenta previa is excluded by ultrasound examination, because this can lead to torrential bleeding. Transvaginal ultrasonography is more accurate than transabdominal and first choice of investigation
if indicated. The probe is entered into the vaginal to the extent and in an angle that prevents the probe from reaching the cervical os and is safe to perform in experienced hands.

Anti-D (RhoGAM) should be administered, but not as the first priority.

TOPIC REVIEW

Placenta previa is implantation of placenta, either partially or wholly, in the lower uterine segment below (previa) the fetal presenting part. Placenta previa is classified as:

Grade I – placenta is in lower segment, but the lower edge does not reach the internal os.
Grade II – Lower edge of placenta reaches the internal os, but does not cover it.
Grade III – Placenta covers the internal os partially.
Grade IV – placenta covers the internal os completely

Placenta previa occurs is 0.5-1.0% of pregnancies and accounts for 20% of all cases of antepartum hemorrhage. It is three times more common in multiparous women.

Risk factors for placenta previa include:

Large placental area e.g. multiple pregnancy


Advanced age
High parity
Deficient endometrium due to pre-existent uterine scar (e.g. previous cesarean section)
Endometritis
Manual removal of placenta
Curettage (especially for miscarriage or termination of pregnancy)
Submucous fibroids

NOTE – if a routine ultrasound examination is made at 18 weeks, the report may show that there is a low-lying placenta, but in over 85% of cases, the placenta will be normally situated by the time of delivery, as the lower uterine segment does not
develop fully until late in the third trimester. Another ultrasound examination should be performed at about the 34th week, or earlier if vaginal bleeding occurs.

Presentation

Most cases of placenta previa present with painless bright red vaginal bleeding of different amount without uterine contractions. Rarely, uterine contraction and tenderness may be present.

In approximately one-third of affected pregnancies, the initial bleeding episode occurs prior to 30 weeks of gestation; this group is more likely to require blood transfusions and is at greater risk of preterm delivery and perinatal mortality than women
whose bleeding begins later in the course of pregnancy. An additional one-third of patients become symptomatic between 30 and 36 weeks, while most of the remaining patients have their first bleed after 36 weeks. About 10% of women reach term
without bleeding.

For an individual patient, it is not possible to predict whether a bleed will occur, nor the gestational age, volume, or frequency of bleeding. However, higher grades of placenta previa bleed earlier and more compared with lower grades.

There is often fetal malpresentation or usually high and mobile fetal presenting part.

Management

Check vital signs, establish IV access and start fluids (if indicated)
Cross-matched blood and blood products should be readily available in anticipation of massive hemorrhage.
Gentle abdominal palpation to see is uterine tenderness/contractions are present, and to estimate the gestational age as well as the fetal presenting part.
Ultrasonography to confirm the diagnosis
Assessment of fetal well-being
Anti-D (RhoGAM) if indicated
Corticosteroids (if indicated)

914 of 1943
Decide for outpatient versus inpatient management. Outpatient management maybe considered:

If the patient is stable and there is no current hemorrhage, the patient can be managed in outpatient setting:

Explain the frequency and severity of recurrent bleeding is unpredictable and carries the risk of fetal and maternal complications
Advise the woman to seek immediate hospital care if contractions or vaginal bleeding occurs
Ensure emergency transport access to hospital
Admit if active bleeding
If inpatient care is indicated:

Cesarean delivery should be considered where there is maternal or fetal instability despite vigorous management, regardless of gestational age; otherwise, cesarean delivery at 37 weeks should be performed.

Vaginal delivery may be considered in low-risk women with low-grade placenta praevia.

References

• http://www.sahealth.sa.gov.au/wps/wcm/connect/b1c6

• Llewellyn-Jones – Fundamentals of Obstetrics and Gynaecology – Elsevier Mosby – 9th Edition

• Royal College of Obstetrics and Gynaecology – Green-top Guideline No.27

Time spent: QID:641 Last updated:


2023-2-12

915 of 1943
On examination of a 28-year-old nulliparous woman at 37 weeks gestation, her fundal height is found to be 35cm and her fetus in transverse lie. An ultrasound exam at 18th week, revealed a low-lying placenta. Which one of the following is the most
likely cause of this presentation?

A. Placenta previa.

B. Ruptured membrane.

C. Small for gestational age.

D. Nulliparity.

E. A normal variation.

Correct
45% answered correctly

Explanation:

Correct Answer Is A

Shoulder presentations, unstable lie, transverse lie and oblique lie may be detected in late pregnancy. These conditions occur in 1 in 200 pregnancies, usually in multiparous women. The etiology is varied. They may occur in a lax multiparous uterus
with no other complications of pregnancy but may be associated with other contributing factors.

Contributing factors to fetal transverse lie include:

High parity
Pendulous abdomen
Placenta previa
Polyhydramnios
Pelvic inlet contracture and / or fetal macrosomia
Uterine abnormalities (e.g. bicornuate uterus or uterine fibroids)
Fetal anomaly (e.g. tumors of the neck or sacrum, hydrocephaly, abdominal distension)
Distended maternal urinary bladder
Poorly formed lower segment
Wrong dates i.e. more premature than appears
Undiagnosed twins
Preterm delivery (fetus)

When transverse lie is found, an ultrasound exam is required to exclude placenta previa as a likely cause and a contraindication to vaginal delivery.

85% of low-lying placentas found on ultrasound at 18 weeks will be normally situated at term. In 15%, placenta remains in the lower segment of the uterus. This woman still has a 15% chance of having low placentation as the main contributing factor
to the transverse lie.

After 20weeks gestation, the fundal height in centimeters correlates to the gestational age in weeks. A discrepancy of >2 cm between the gestational age and the fundal height (more than 2 cm) is considered significant and can be caused by:

Fundal height < gestational age:

Dating errors
Oligohydramnios
Transverse/oblique lie
Small for gestational age

Fundal height > gestational age:

Dating errors
Large for gestational age
Multiple pregnancy
Polyhydramnios
Molar pregnancy

In this case, a discrepancy of only 2 cm is not that significant but can be caused by the transverse lie.

Over all, some reports show that extreme prematurity is the most common finding in pregnancies complicated by transverse lie, but in this case and with low-lying placenta in history, placenta previa remains the most likely cause to consider and
exclude.

Transverse lie near term is not a normal finding. In early pregnancy, it is very common to see in fetus in transverse lie, but not a normal finding near term.

References

• South Australian Perinatal Practice Guideline - Unstable lie of the fetus

• Williams Obstetrics – McGraw Hill - 24th Edition – page 468

Last updated:
Time spent: QID:642 2023-2-12

916 of 1943
A 31-year-old G2P1 woman presented to the maternity unit at 38 weeks gestation and in labour. Her previous pregnancy led to caesarean section and delivery of a healthy baby. The current pregnancy had been uneventful without any remarkable
problems in antenatal visits except first trimster nausea and vomiting. On arrival, she had a cervical dilation of 4 cm and the fetal head was at -1 station. After 5 hours, the cervical length and fetal head station are still the same despite regular uterine
contractions. Suddenly, there is sudden gush of blood of approximately 1000 mL and the fetal heart rate drops to 80 bpm on CTG. Which one of the following could be the most likely cause?

A. Lower genital tract lacerations.

B. Placenta previa.

C. Placenta accreta.

D. Uterine atony.

E. Ruptured uterus.

Incorrect. Correct answer is E


45% answered correctly

Explanation:

Correct Answer Is E

With previous caesarean section in history and prolonged active phase of the first stage of the labour, uterine rupture would be the most likely cause of the bleeding.

Maternal manifestations of uterine rupture are variable and include the following

Constant abdominal pain – pain may not be present in sufficient amount, character, or location to suggest uterine rupture and pain may be partially or completely masked by regional analgesia.
Signs of intraabdominal hemorrhage (a strong indicator) - Although hemorrhage is common, the signs and symptoms of intraabdominal bleeding in cases of uterine rupture, especially those cases not associated with prior surgery, may
be subtle.
Vaginal bleeding – Vaginal bleeding is not a cardinal symptom, because it may be modest, despite major intraabdominal hemorrhage.
Maternal tachycardia and hypotension
Cessation of uterine contractions
Loss of station of the fetal presenting part
Uterine tenderness

Fetal bradycardia (as seen in this case) is the most common and characteristic clinical manifestation of uterine rupture. Variable or late decelerations may precede the bradycardia, but there is no fetal heart rate pattern pathognomonic of rupture.
Furthermore, fetal heart rate changes alone have low sensitivity and specificity for diagnosing uterine rupture.

Postpartum uterine rupture is characterized by pain and persistent vaginal bleeding despite use of uterotonic agents. Hematuria may occur if the rupture extends into the bladder.

NOTE – A history of previous uterine surgery (e.g. cesarean section) typically alerts the obstetrician to the possibility of uterine rupture in symptomatic women.

When uterine rupture is suspected, immediate cesarean section should be performed to save both the mother and the baby. Definite diagnosis of uterine rupture is always made after laparotomy.

References

• http://www.sahealth.sa.gov.au/wps/wcm/connect/0f2c

• http://www.uptodate.com/contents/rupture-of-the-un

Last updated:
Time spent: QID:643 2023-2-12

917 of 1943
A 50-year-old woman presents to your GP practice because of persistent vaginal bleeding for the last 3 days. Her last menstrual period occurred one year and a half ago. She mentions that just before the bleeding she felt reduction in hot flushes she
has been experiencing for the past year. She is sexually active and has had regular pap smears with a normal one 6 months ago. Which one of the following is the most likely diagnosis?

A. Endometrial cancer.

B. Cystic glandular endometrial hyperplasia.

C. Atrophic vaginitis.

D. Cervical cancer.

E. An episode of ovarian follicular activity.

Incorrect. Correct answer is E


45% answered correctly

Explanation:

Correct Answer Is E

When menopause occurs, estrogen levels fall. Decreased estrogen levels result in increased production of follicular stimulating hormone (FSH) and very high levels of circulating FSH. At times, these high levels of FSH cause a remaining follicle to
become active, resulting in menstruation. At such occasions, produced estrogen by the active follicle leads to decreased symptoms of menopause such as hot flushes, vaginal dryness, etc.

(Option A) Endometrial cancer could be a possibility but it is very uncommon in premenopausal women or those women within the first 2-3 years of their menopause.

(Option B) Cystic glandular hyperplasia is a result of long-term exposure of endometrium to unopposed estrogen such as in women with anovulatory cycles (e.g., in PCOS). There is no clue in history to suggest endometrial hyperplasia.

(Option C) Atrophic vaginitis presents differently with decreased vaginal lubrication and dyspareunia, vaginal discharge or bleeding, dysuria, etc. Although not impossible, it is uncommon for vaginal atrophy to occur within the first 2-3 years of
menopause.

(Option D) With a normal cervical screening 6 months ago, the cervical cancer is very unlikely.

References

• AMC Handbook of Multiple Choice Questions – page 528

Last updated:
Time spent: QID:651 2023-2-12

918 of 1943
Which one of the following is highly suggestive of polycystic ovarian syndrome (PCOS)?

A. Elevated FSH.

B. Elevated LH.

C. Elevated testosterone levels.

D. Decreased free testosterone.

E. Hyperprolactinemia.

Incorrect. Correct answer is C


45% answered correctly

Explanation:

Correct Answer Is C

Polycystic ovarian syndrome (PCOS) is associated with the following:

1. Clinical or biochemical hyperandrogenism. Clinical features suggestive of PCOS include hirsutism, acne, deepened voice, etc
2. Menstrual dysfunction, irregularity or lack of ovulation.
3. Polycystic ovaries on the ultrasound.

Only the first two mentioned features will suffice for establishing the diagnosis of PCOS, because cystic structures on ultrasound are often not seen, particularly in women who have been on treatment.

The following hormonal changes are seen in PCOS:

Serum FSH (follicle stimulating hormone) is either normal or low.

Elevation of LH (luteinizing hormone)

A normal LH/FSH ratio in premenopausal women is about 1:1, while in PCOS this ration may be 2:1 or 3:1. It should be noted that approximately 30% of patients with established diagnosis of PCOS will have a LH/FSH ration of 1:1.

Serum estrogen level is either decreased or normal. Overall, estrogen level is very nonspecific for diagnosis of PCOS..

Serum free testosterone is the first-line investigation in patients suspected of having PCOS. Hyperandrogenism is one of the essential criteria to establish the diagnosis of PCOS.
Prolactin level is usually normal or mildly elevated, possibly due to chronic estrogen exposure.

Of the options, an elevation of testosterone is diagnostic for PCOS.

References

• https://www.mja.com.au/journal/2011/195/6/assessme

• https://www.nhmrc.gov.au/_files_nhmrc/publications

• http://emedicine.medscape.com/article/256806-wor

Last updated:
Time spent: QID:122 2023-2-12

919 of 1943
A 21-year-old woman, who underwent dilation and curettage for termination of an unwanted pregnancy 4 days ago, presents to the emergency department with fever and purulent vaginal discharge. Her pregnancy was the outcome of sex with an
stranger, while intoxicated in a party. Which one of the following organisms is most likely to be the cause of her problem?

A. Mycoplasma hominis.

B. E.cloi.

C. Chlamydia trachomatis.

D. Chlamydia trachomatis and vaginal pathogens.

E. Neisseria gonorrhea.

Incorrect. Correct answer is D


45% answered correctly

Explanation:

Correct Answer Is D

With the presence of fever and purulent vaginal discharge, this woman has inflammatory pelvic disease. The predominance of microbial etiology depends on whether the PID is sexually-acquired or not:

Non-sexually acquired PID – This type is usually caused by mixed pathogens originating from vaginal flora, including anaerobes, facultative bacteria, Mycoplasma hominis and sometimes E.cloi. This type of PID often occurs postpartum or following
instrumentation or gynecological surgeries.

Sexually acquired PID – Chlamydia trachomatis and Neisseria gonorrhea are the two most common causes of pelvic inflammatory disease (PID) in sexually active women. In Australia chalmydia is much more common.

In patients presenting for termination of pregnancy, especially those with risk factors for Chlamydia, such as not having a constant sexual partner, preoperative swabs have shown that Chlamydia infection is present in up to 15%. For this patient
combination of vaginal pathogens and Chlamydia trachomatis would be the most likely etiology.

If the symptom was purulent vaginal discharge, and no fever was present, the diagnosis would have been cervicitis (and not PID). Chlamydia trachomatis is the most common cause of cervicitis. Vaginal pathogens do not cause cervicitis.

NOTE – for the exam purpose remember the following:

The most common cause of post-procedural PID is vaginal pathogens


The most common cause of pos- procedural PID in women, who are high risk for STDs as well is Vaginal pathogens PLUS Chlamydia
The most common cause of PID, unrelated to surgical procedures is Chlamydia trachomatis

References

• Therapeutic Guidelines – Antibiotic; available from: http://tg.org.au

• AMC Handbook of Multiple Choice Questions – page 526

• http://emedicine.medscape.com/article/256448-overv

Last updated:
Time spent: QID:652 2023-2-12

920 of 1943
A 40-year-old woman with history of tubal ligation presents with complaint of irregular heavy menstrual bleeding. She is known to have multiple uterine fibroids. Other significant point in history is dilation and curettage 6 months ago with no
improvement in her symptoms, or no diagnosis for a cause. On examination, the uterus has the size of a 12-week pregnant uterus. Laboratory studies are significant for a hemoglobin level of 85 g/L. Which one of the following is the next best step in
management?

A. Total abdominal hysterectomy.

B. Endometrial ablation.

C. Myomectomy.

D. Combined oral contraceptive pills.

E. NSAIDs.

Correct
45% answered correctly

Explanation:

Correct Answer Is A

Uterine leiomyomas (fibroids) are benign tumors of uterus. Since histological confirmation of the clinical diagnosis in not necessary in most cases, asymptomatic uterine leiomyomas only require follow-up without any specific intervention.

Symptoms and complications demand intervention. Hysterectomy is the definitive treatment. Other alternatives include myomectomy with various techniques, endometrial ablation, uterine artery embolization and myolysis.

Hysterectomy is indicated in the following situations:

Women with acute hemorrhage who do not respond to other therapies

Women who have completed childbearing and have current or increased future risk of other diseases (cervical intraepithelial neoplasia, endometriosis, adenomyosis, endometrial hyperplasia, or increased risk of uterine or ovarian
cancer) that would be eliminated or decreased by hysterectomy

Women who have failed prior minimally invasive therapy for leiomyomas

Women who have completed childbearing and have significant symptoms, multiple leiomyomas, and a desire for a definitive treatment

Total abdominal hysterectomy is the procedure of choice if hysterectomy is planned.

For women, who desire to preserve the ability of child bearing, a course of gonadotropin releasing hormone (GnRH) agonists, followed by myomectomy is the treatment of choice.

NOTE - Leiomyomas are the most common indication for hysterectomy, accounting for 30% of hysterectomies in white women and over 50% of hysterectomies in black women.

This woman, who does not wish to have more children and has undergone tubal ligation, and has bothersome symptoms and anemia, total abdominal hysterectomy is the most appropriate treatment option.

(Option B) Endometrial ablation is inferior to hysterectomy. With endometrial ablation, amenorrhea is achieved; however, leiomyomas remain untreated.

(Option C) Myomectomy is not advisable, unless further fertility is desired. There is a likelihood of recurrence that may necessitate further surgeries. Furthermore, when the uterus is enlarged with multiple leiomyomas, it will be very difficult to remove
them all. Remaining leiomyomas can grow over time and produce the symptoms again.

(Option D) Hormonal methods including combined contraceptive pills, progesterone-only method, or diazole, has been used with anecdotal effects on some symptoms such as menorrhagia. Some have controversial efficacy and some with proven
efficacy have undesirable adverse effect profiles that limit their use.

(Option E) NSAIDs have not been extensively studied in leiomyomas-related menorrhagia. NSAIDs do not appear to reduce blood loss in women with leiomyomas, but they decrease painful menses and can be useful for this purpose.

References

• UpToDate - Uterine fibroids (leiomyomas): Treatment overview

Last updated:
Time spent: QID:653 2023-2-12

921 of 1943
You are giving advice to a young couple about contraception. After explanation of different methods, they choose to use ‘rhythm method’ (calendar calculation). The woman has menstrual cycles of between 26 and 29 days. Which one of the
following abstinence periods would be the correct advice?

A. From day 4 to day 17.

B. From day 6 to day 17.

C. From day 8 to day 19.

D. From day 9 to day 19.

E. From day 12 to day 17.

Incorrect. Correct answer is B


45% answered correctly

Explanation:

Correct Answer Is B

Regardless of the length of the menstrual cycle, ovulation occurs 14 days (2 weeks) before menstruation. In other words, luteal phase has always a fixed duration of 14 days; therefore, in this woman ovulation occurs on day 12 at earliest and on day
15 at latest. After ovulation, there is a window of 24-36 hours, during which the egg can be fertilzied by sperm; therefore, intercourse should not be resumed until the latest day when ovulation is possible plus an additional 48 hours (15 + 2=17).

In the presence of adequate and normal cervical mucus, sperm can survive up to 6 days. To calculate the day from which abstinence should be started, 6 should be subtracted from the earliest date of possible ovulation (12 here) (12-6=6).

The appropriate advice for this couple will be abstinence between days 6 and 17.

NOTE – calculation of the abstinence period is by the following formula:

Beginning of the abstinence = subtract 6 days from the earliest day of the cycle on which ovulation can occur
End of abstinence = Add 2 days to the latest day of the cycle on which ovulation may occur

For example, in a patient with menstrual cycles between 26 and 32 days:

Possible days of ovulation: earliest=26-14=12th day of menstrual cycle; latest=32-14=18th day of menstrual cycle
Beginning of the abstinence = 12-6=6th day of menstrual cycle
End of abstinence =18 +2 =20th day of menstrual cycle

References

• AMC Handbook of Multiple Choice Questions – pages 525-526

Last updated:
Time spent: QID:654 2023-2-12

922 of 1943
A 32-year-old woman presents with complaint of mucopurulent greenish vaginal discharge. Which one of the following organisms is the most likely cause of this presentation?

A. Candida albicans.

B. Chlamydia trachomatis.

C. Gardnerella vaginalis.

D. Mycoplasma hominis.

E. E.cloi.

Incorrect. Correct answer is B


45% answered correctly

Explanation:

Correct Answer Is B

Of the given options, only Chlamydia trachomatis can cause a thick mucopurulent greenish yellow vaginal discharge. Trichomonas vaginalis (not an option here) can also present with thin frothy yellow-green vaginal discharge that is offensive.
Neisseria gonorrhea can also present with mucopurulent vaginal discharge.

(Option A) The discharge in candida vaginitis is thick, white and non-offensive.

(Options C and D) Gardnerella vaginalis is the most common cause of bacterial vaginosis. Vaginal discharge of bacterial vaginosis is thin, grey-white and has an offensive (fishy) odor. Mycoplasma hominis is another organism found in bacterial
vaginosis.

(Option E) E.coli may colonize vagina but does not result in infections presenting with green vaginal discharge.

References

• Australian STI management guideline - Vaginal Discharge

• Medscape - Cervicitis

Last updated:
Time spent: QID:655 2023-2-12

923 of 1943
A 65-year-old woman presents to your clinic with complaints of a brownish vaginal discharge for the past 2 months. On examination, atrophic vagina is evident. Which one of the following is the most like diagnosis?

A. Vaginal atrophy.

B. Endometrial cancer.

C. Cervical cancer.

D. Chlamydia infection.

E. Endometrial hyperplasia.

Correct
45% answered correctly

Explanation:

Correct Answer Is A

With vaginal discharge of brownish color in a 65-year-old woman, endometrial cancer should always be the first diagnosis to exclude; however, the question asks about the most likely cause of the presentation and not the most important diagnosis
to consider.

The brown color of vaginal discharge is often caused by blood. The source of bleeding can be either the uterine cavity or the vagina. Of all women with postmenopausal vaginal bleeding, only 5-10% were found to have endometrial cancer,
while approximately 60% had atrophic vaginitis. Of the options, vaginal atrophy due to estrogen-deficient status of menopausal women is the most likely explanation for this presentation.

Estrogen deficiency in postmenopausal years results in urogenital atrophy.

Symptoms of urogenital atrophy include:

Vaginal dryness
Vaginal burning or irritation
Decreased vaginal lubrication during sexual activity
Dyspareunia, including vulvar or vaginal pain (at the introitus or within the vagina)
Vulvar or vaginal bleeding (e.g. postcoital bleeding, fissures)
Vaginal discharge (leucorrhea or yellow and malodorous)
Pelvic pressure or a vaginal bulge
Urinary tract symptoms (e.g. urinary frequency, dysuria, urethral discomfort, hematuria)

TOPIC REVIEW

The differential diagnoses of vaginal bleeding in postmenopausal women is less broad than that for abnormal bleeding in premenopausal women. A series of 1138 women aged 41 to 91 years with postmenopausal bleeding reported the following
etiologies and prevalence:

Vaginal/endometrial atrophy: 50%


Polyps: 12%
Endometrial cancer: 10%
Endometrial hyperplasia: 9.8%
Hormonal effects: 7%
Cervical cancer: <1%
Other (e.g., hydrometra, pyometra, hematometra): 2%

References

• Family Practice Notebook - Postmenopausal Bleeding

• UpTodate - Post menopausal uterine bleeding

Last updated:
Time spent: QID:656 2023-2-12

924 of 1943
Which one of the following options is the most common presenting symptom in women with atrophic vaginitis?

A. Painless vaginal bleeding.

B. Endometritis.

C. Difficult intercourse.

D. Postcoital bleeding.

E. Scanty purulent discharge.

Incorrect. Correct answer is C


45% answered correctly

Explanation:

Correct Answer Is C

Atrophic vaginitis, also referred to as vulvovaginal atrophy, urogenital atrophy, or vaginal atrophy, is characterized by dryness, inflammation, and thinning of the epithelial lining of the vagina and lower urinary tract due to estrogen deficiency. It typically
occurs in menopausal women, but can occur in women of any age who experience a decrease in estrogenic stimulation of urogenital tissue.

Premenopausal woman are in hypo-estrogenic state in postpartum period, lactation, and when on anti-estrogenic medication. In such conditions, atrophic vaginitis can also occur.

Symptoms of urogenital atrophy include:

Vaginal dryness
Vaginal burning or irritation
Decreased vaginal lubrication during sexual activity
Dyspareunia, including vulvar or vaginal pain (at the introitus or within the vagina)
Vulvar or vaginal bleeding (e.g., postcoital bleeding (option D), fissures)
Vaginal discharge (leukorrhea or yellow or brown discharge that can be malodorous) (option E)
Pelvic pressure or a vaginal bulge
Urinary tract symptoms (e.g. urinary frequency, dysuria, urethral discomfort, hematuria)

Symptoms of vaginal atrophy are usually progressive and worsen with time. Vaginal dryness and lack of lubrication on sexual arousal is the earliest and the most common symptom of atrophic vaginitis. This is the first signs of estrogen insufficiency
in urogenital system. Urinary symptoms present later as the estrogen deficiency continues.

Painless vaginal bleeding (option A) and endometritis (option B) are not clinical features associated with atrophic vaginitis.

References

• AAFP - Diagnosis and Treatment of Atrophic Vaginitis

Last updated:
Time spent: QID:657 2023-2-12

925 of 1943
Which one of the following is the most common site of endometriosis?

A. Round ligament.

B. Uterosacral ligament.

C. Ovary.

D. Bladder.

E. Broad ligament.

Incorrect. Correct answer is C


45% answered correctly

Explanation:

Correct Answer Is C

Endometriosis is defined as the presence of endometrial glands and stroma at extrauterine sites. These ectopic endometrial implants are usually located in the pelvis, but can occur nearly anywhere in the body.

The most common sites of endometriosis, in decreasing order of frequency, are:

Ovaries
Posterior cul-de-sac
Broad ligament
Uterosacral ligament
Rectosigmoid colon
Bladder
Distal ureter

Less common, yet possible, sites are vagina, cervix, rectovaginal septum, caecum, ileum, inguinal canals, abdominal or perineal scars, and umbilicus. Rare sites include breast, pancreas, liver, gallbladder, kidney, urethra, extremities, vertebrae,
peripheral nerves, lungs, spleen, diaphragm and central nervous system.

Ovaries are the most common site of endometriosis.

References

• Medscape - Endomteriosis

Last updated:
Time spent: QID:659 2023-2-12

926 of 1943
Which one of the following is the least likely site of endometriosis?

A. Ovaries.

B. Cervix.

C. Uterine wall.

D. Pouch of Douglas.

E. Bladder.

Incorrect. Correct answer is B


45% answered correctly

Explanation:

Correct Answer Is B

Endometriosis is defined as the presence of endometrial glands and stroma at extrauterine sites. These ectopic endometrial implants are usually located in the pelvis, but can occur
nearly anywhere in the body.

The most common sites of endometriosis, in descending order of prevalence, are:

Ovaries
Posterior cul-de-sac
Broad ligament
Uterosacral ligament
Rectosigmoid colon
Bladder
Distal ureter

Less common, yet possible, sites are vagina, cervix, rectovaginal septum, cecum, ileum, inguinal canals, abdominal or perineal scars, and umbilicus. Rare sites include breast,
pancreas, liver, gallbladder, kidney, urethra, extremities, vertebrae, peripheral nerves, lungs, spleen, diaphragm and central nervous system.

Of the given options, cervix is the least likely site for endometriosis.

References

• Medscape - Endomteriosis

Last updated:
Time spent: QID:660
2023-2-12

927 of 1943
A 47-year-old woman presents to your clinic with heavy irregular periods every 2 to 4 months for the past 12 months. Before that, her periods were normal, occurring every 32 days.
Physical examination is unremarkable. Blood tests show a hemoglobin level of 110 g/L (normal: 115-165). Which one of the following is the most likely cause of this presentation?

A. Endometrial cancer.

B. Endometrial polyps.

C. Endometriosis.

D. Anovulatory cycles.

E. Uterine fibroids.

Incorrect. Correct answer is D


45% answered correctly

Explanation:

Correct Answer Is D

By far, anovulatory cycles are the most common cause irregular heavy periods. After ovulation, the follicle turns into the progesterone-secreting corpus luteum. The progesterone
stabilizes the endometrium. If pregnancy fails to take place, corpus luteum regresses and the progesterone declines. When progesterone falls, endometrial shedding and menstrual
flow occurs at the predicted time.

Without ovulation, unopposed estrogen continues to cause endometrial growth. This results in unpredictable shedding of a hypertrophied endometrium and irregular, often heavy
vaginal bleeding.

Anovulatory cycles have different etiologies, but polycystic ovary syndrome and approaching menopause are the most common causes.

(Option A) Endometrial cancer in premenopausal woman is not common.

(Options B, C and E) Endometrial polyps, endometriosis and uterine fibroids can cause heavy menstrual bleeding (menorrhagia) in a predictable fashion. Menstrual irregularities are
not a common feature of these conditions.

References

• Clinical Care Standards - Heavy Menstrual Bleeding Clinical Care Standard

• AAFP - Evaluation and Management of Abnormal Uterine Bleeding in Premenopausal Women

• UpToDate - Approach to abnormal uterine bleeding in nonpregnant reproductive-age women

Last updated:
Time spent: QID:661
2023-2-12

928 of 1943
A30-year-old woman on oral contraceptive pills (Microgynon 30®) presents to your clinic for a repeat prescription. On examination, you find out that she has a blood pressure of
160/100mmHg, confirmed by a second reading 20 minutes later. She mentions that her husband and she are planning to become pregnant in 6 months. Which one of the following
is the best advice for her?

A. She can continue the same OCP but should use methyldopa for hypertension.

B. She should change to an OCP preparation with lower dose of estrogen.

C. She should stop OCP, use condoms for contraception and reassess her blood pressure in 3 months.

D. She should stop the OCP and be started on methyldopa for hypertension.

E. She should stop the OCP and be started on a diuretic or ACE inhibitor for hypertension.

Incorrect. Correct answer is C


45% answered correctly

Explanation:

Correct Answer Is C

Eestrogen-containing hormonal contraception methods are NOT absolutely contraindicated in hypertensive women, whether the hypertension has existed before or developed as
result of the hormonal contraception use. With hypertension, the patient should be started on appropriate antihypertensive agent if she insists on using the method.

This woman, however, is willing to stop the OCP; hence, cessation of the estrogen-containing method and using an alternative is the best advice. Since this woman is planning to
conceive, using condoms for contraception in the next 6 month, during which she does not want to become pregnant, would be the best advice among the options.

If OCPs are ceased, she does not need to be started on antihypertensive drugs at this stage because with cessation of estrogen, there is a good chance that her blood pressure
returns to normal. Antihypertensive medications will be indicated if blood pressure remains high despite stopping the OCP.

If use of antihypertensive medications is inevitable, methyldopa should be considered as first line because it is safe to use during pregnancy.

ACE inhibitors and diuretics are contraindicated during pregnancy and should be avoided.

References

• http://www.aafp.org/afp/2010/0915/p621.html

• http://www.gpnotebook.co.uk/simplepage.cfm?ID=7721

Last updated:
Time spent: QID:662
2023-2-12

929 of 1943
A 51-year-old menopausal woman presents with frequent hot flushes disturbing her sleep and occupational performance. She has the history of breast cancer diagnosed 10 years ago. The umor was positive for estrogen receptors on pathology.
Which one of the following would be the most appropriate management?

A. Cyclical estrogen and progesterone.

B. Progesterone-only HRT.

C. Paroxetine.

D. Black cohoch.

E. Mefenamic acid.

Incorrect. Correct answer is C


45% answered correctly

Explanation:

Correct Answer Is C

Hot flush is a sudden feeling of warmth that is generally most intense over the face, neck, and chest. The duration is variable but on average lasts about 3-4 minutes. It is often accompanied by sweating that can be profuse and followed by a chill.
Hot flushes are seen in perimenopausal women as one of the most common complaints, and is the only indication to start hormone replacement therapy (HRT). HRT is the most effective treatment for severe hot flushes and is a reasonable choice in
the absence of contraindications.

(Options A and B) Hormonal therapy including estrogen and progesterone is avoided in patients with history of breast cancer, although the effect of progesterone on breast cancer is not established.

Women with hot flushes and contraindications to hormonal therapy may benefit from selective serotonin reuptake inhibitor (first-line) such as paroxetine, or other medications such as gabapentin or velnafaxine.

SSRI are widely used in Australia for management of hot flushes when there is contraindication to hormone therapy. It also improves mood symptoms. Paroxetine is the most common prescribed SSRI (and the only one approved by FDA in US). SSRIs
are not as effective as HRT, but definitely more effective than placebo. It is important to note that paroxetine, fluoxetine, duloxetine, and bupropion should not be used in patients who are currently on tamoxifen because it interferes with the
conversion of tamoxifen to its active metabolite endoxifen.

(Option D) Black Cohosh acts by unknown mechanism and is used for hot flushes. Many studies have found no significant benefit over placebo.

(Option E) Mefenamic acid may help for short-term control of menopausal symptoms; however, its long term use is controversial and not recommended.

930 of 1943
On a routine health exam, a 40-year-old woman is found to have hypercholesterolemia. She mentions that her older sister has hypertension. Which one of the following suggests a familial cause for her hypercholesterolemia?

A. Tendon xanthomas.

B. Xanthelasma of the eye.

C. The family history of hypertension.

D. Age of the patient.

E. The presence of corneal arcus.

Correct
45% answered correctly

Explanation:

Correct Answer Is A

Familial hypercholesterolemia (FH) is an autosomal-dominant inherited condition characterized by a genetic defect in one of several genes affecting receptor-mediated uptake of low-density lipoprotein (LDL). Affected individuals present with
characteristic metabolic and clinical features including high cholesterol levels and increased risk of premature cardiovascular disease.

Clinical features of FH are:

Premature cardiovascular disease (CVD)


Aortic stenosis
Tendon xanthomas (11%)
Corneal arcus (27%)
Xanthelasmas (12%)

There are many other potential causes of premature CVD and aortic stenosis. Likewise, corneal arcus and xanthelasma are non-specific signs, but tendon xanthomas, which may gradually develop in Achilles' tendons and extensor tendons of the
dorsum of the hand, are pathognomonic for FH. However, they are rarely identifiable before adulthood.

Corneal arcus in a young adult is suggestive of FH, but not pathognomonic.

Achilles tendon xanthoma

Xanthelasma

931 of 1943
Corneal arcus

References

• RACGP - AJGP- Familial hypercholesterolaemia: A guide for general practice

Last updated:
Time spent: QID:142 2023-2-12

932 of 1943
A 28-year old woman presents with her husband complaining of inability to conceive after 18 months of regular sexual activity. During the discussion on an etiology, further work-up and the treatment, she asks about the possible outcome for
treatment of different etiologies. Which one of the following conditions has the most favorable treatment outcome?

A. Azoospermia.

B. Tubal obstruction.

C. Polycystic ovarian syndrome.

D. Pelvic tuberculosis.

E. Turner syndrome.

Incorrect. Correct answer is C


45% answered correctly

Explanation:

Correct Answer Is C

Treatment of infertility due to ovulation problems has had the most satisfactory outcomes by far. Polycystic ovarian syndrome (PCOS), also known as Stein-Leventhal syndrome is characterized by hyperandrogenism, small cysts in ovaries, acne and
hirsutism as well as problems of ovulation leading to anovulatory cycles due to hormonal imbalance.

70% of females with anovulatory cycles start ovulation on medical management and of those, 50% conceive within 6 to 9 months.

Low or zero sperm counts, poor sperm motility and dysmorphic sperms can all cause infertility. Fertility drugs for promotion of sperm production, artificial insemination with donor sperm and intracytoplasmic injection of sperm can be used
(Option A)
as treatment modalities, but success rate is low (less than 25%).

(Options B and D) Obstruction of or damage to the Fallopian tubes follows pelvic inflammatory disease, prior surgery, or tuberculosis and prevents the sperm from reaching to the ovum. Laparoscopic surgery is used to treat tubal obstructions or
adhesions. Success rates of conception vary widely from a minimum of 10% to a maximum of 70% depending on the severity of the blockage.

(Option E) Turner syndrome is associated with ovarian dysgenesis and infertility. There is no option for treatment of gonadal dysgenesis and reversal of infertility. Success rate is very low even after egg donation due to high rate of miscarriages.

References

• http://www.ncbi.nlm.nih.gov/pmc/articles/PMC270066

• http://emedicine.medscape.com/article/274143-overv

Last updated:
Time spent: QID:123 2023-2-12

933 of 1943
A 28-year-old unmarried woman presents to your clinic with slight left-sided lower abdominal pain for the past 2 days. On examination, she has a blood pressure of 125/90 mmHg,
pulse rate of 90 bpm and temperature of 37.3°C. She is otherwise healthy. Abdominal exam elicits no tenderness, rebound or guarding. Ultrasonographic assessment shows a 6 cm
solid mass lateral to the uterus on the left side. Which one of the following is the most likely diagnosis?

A. Mucinous cystadenoma.

B. Ovarian teratoma.

C. Endometriosis.

D. Corpus luteum cyst.

E. Ovarian cancer.

Incorrect. Correct answer is B


45% answered correctly

Explanation:

Correct Answer Is B

Ovarian cystic teratomas and ovarian dermoid cysts are encapsulated tumors with mature tissue or organ component. They are composed of well-differentiated derivations from at
least two of the three germ cell layers, namely ectoderm, mesoderm, and endoderm; therefore, they may contain developmentally mature skin components such as hair follicles,
sweat glands, pockets of sebum, blood, fat, bone, nail, teeth, eyes, cartilage, thyroid tissue, etc.

Real organoid structures such as teeth, fragments of bone, etc. may be present in approximately 30% of cases.

The diameter of an ovarian teratoma is typically less than 10 cm and very rarely more than 15 cm.

Uncomplicated ovarian dermoids are often asymptomatic and discovered incidentally. There is, however, an increased risk of ovarian torsion that may present with acute pelvic pain.

Ultrasound shows a solid mass in the adnexal (next to the uterus). Teratomas are common in young women around the age of 30 years and the most common pelvic tumors in
women younger than 20 years.

(Option A) Mucinoid cystadenoma, as the name implies, has cystic nature and is not solid. This tumor is at the benign end of the spectrum of mucin-containing epithelial ovarian
tumors. On ultrasonography, large cystic adnexal mass that is multilocular with numerous septations is seen.

(Option C) Ovarian endometriosis has a different appearance on ultrasonography including the acoustic enhancement of ovaries with homogenous low-level internal echoes as a
result of the hemorrhagic debris. On the other hand, the predominant feature of endometriosis is dysmenorrhea.

(Option D) Corpus luteal cyst is a type of functional ovarian cyst that forms when a corpus luteum does not regress, either normally due to pregnancy or without the pregnancy
taking place. Corpus luteum cyst is the most common pelvic mass within the first trimester. When corpus luteal cysts are associated with pregnancy, most of them will involute by
the end of the second trimester. On ultrasonography, they might have a range of sonographic appearances depending on the stag of evolution and the age of the associated
intracystic hemorrhage, but general characteristics are as follows:

Diffusely thick wall


Peripheral vascularity
Diameter less than 3 cm (with the wall)

(Option E) Although ovarian cancer is seen as an adnexal solid tumor, it is a remote possibility in a 28-year-old woman.

Ovarian teratoma - solid appearance

934 of 1943
Mucinoid cystadenoma - multiloculated cyst

Corus luteum cyst (hemorrhagic)

References

• Radiopaedia - Mature cystic ovarian teratoma

• Radiopaedia - Ovarian mucinous cystadenoma

• Radiopaedia - Endometrioma

Last updated:
Time spent: QID:663
2023-2-12

935 of 1943
A 26-year-old woman presents to your clinic on the 6th day of her menstrual cycle with mild lower abdominal pain. She lives with her male partner and has no children. On
examination, she has a blood pressure of 110/70 mmHg, pulse rate of 90bpm, and temperature of 37.5°C. No adnexal mass is palpated; however, on vaginal exam cervical motion
tenderness is noted. Which one of the following would be the next best step in management?

A. Transvaginal ultrasonography.

B. Cervical swabs for culture.

C. Urine analysis and culture.

D. Thyroid stimulating hormone.

E. Abdominal CT scan.

Incorrect. Correct answer is B


45% answered correctly

Explanation:

Correct Answer Is B

Cervical motion tenderness (also called cervical excitation or chandelier sign) suggests a pelvic pathology. Classically, it is present in pelvic inflammatory disease (PID) and ectopic
pregnancy (EP). Sometimes the test is used to differentiate PID from appendicitis.

This patient is menstruating and less likely to have an EP; therefore, PID re,mains the most likely diagnosis. EP is less likely.

Because of the relatively poor specificity and sensitivity of clinical findings in PID, the Center for Disease Control and (CDC) has established minimal criteria for diagnosis of PID.

In a female patient PID is the diagnosis if:

She is at risk for sexually transmitted disease (STD) AND


She has lower abdominal or pelvic pain without no other identifiable cause for her illness other than PID, AND
on pelvic examination there is (1) cervical motion tenderness or (2) uterine tenderness or (3) adnexal tenderness

In such patients, based on diagnosis of PID, empiric antibiotics should be started after cervical swabs are taken for culture and DNA probe.

Since EP is an important differential diagnosis in a patient with cervical motion tenderness, EP should be excluded. This is correct that in the presence of normal menstruation EP is
very unlikely, but excluding pregnancy would be a good choice if it was among the options.

Ultrasonography is the next best step in management if ectopic pregnancy is suspected based on history, clinical findings and a positive pregnancy test.

References

• http://emedicine.medscape.com/article/256448-diffe

• http://emedicine.medscape.com/article/256448-worku

Last updated:
Time spent: QID:665
2023-2-12

936 of 1943
A 31-year-old woman comes to your clinic complaining of unpredictable vaginal bleeding for the past 6 months. She also mentions that recently the amount of bleeding has
increased. Which one of the following is the most appropriate next step in management?

A. Full blood exam (FBE).

B. Transvaginal ultrasonography.

C. Serum beta HCG.

D. Thyroid stimulating hormone (TSH).

E. Endometrial sampling.

Correct
45% answered correctly

Explanation:

Correct Answer Is A

Abnormal uterine bleeding (AUB) affects 9-14% of women. AUB generally can be divided into anovulatory and ovulatory patterns. In ovulatory pattern, the bleeding is regular but
heavy, whereas in anovulatory pattern the bleeding is irregular and unpredictable.

Ovulatory pattern is usually caused by uterine problems such as leiomyomas, endometriosis, adenomyosis, polyps, etc. anovulatory pattern is a result of hormonal problems such as
polycystic ovarian syndrome (PCOS), hypothyroidism, hyperthyroidism, hyperprolactinemia, and Cushing syndrome.

The approach to women with ovulatory versus anovulatory pattern is different. It is recommended that if one pattern of bleeding cannot be told from the other, the patient be
assessed as having irregular bleeding because this pattern includes investigation for endometrial hyperplasia/cancer for more diagnostic safety.

In approach to abnormal uterine bleeding in reproductive years, the first thing to exclude is pregnancy. It is done using an office urine pregnancy test and not serum beta HCG)
(option C).

The next step, after exclusion of pregnancy is checking whether the patient has developed anemia due to blood loss with a full blood exam (FBE). Transvaginal ultrasound (option
B) is also a very important investigation in this patient because she has anovulatory pattern of AUB, bringing up endometrial hyperplasia or even cancer as a possibility for which
transvaginal ultrasound can be used as a screening tool. Endometrial sampling (option E) using curettage follows if any endometrial abnormalities are found on ultrasound.

A thyroid function test (option D) should only be ordered if there are indicators of thyroid disease. Testing for coagulation disorders such as Von Willebrand disease is
recommended if indicated

References

• Cancer Australia - Abnormal Vaginal Bleeding in Pre- and Peri-menopausal Women

• AAFP - Evaluation and Management of Abnormal Uterine Bleeding in Premenopausal Women

• Approach to diagnosis and management of abnormal uterine bleeding

Last updated:
Time spent: QID:666
2023-2-12

937 of 1943
A 29-year-old woman with past history of pregnancy with an anencephalic fetus presents to your clinic for preconception counselling. Which one of the following is the most
appropriate advice regarding folic acid supplementation for her?

A. She should use supplemental folic acid 0.4 to 0.5 mg per day.

B. She should use supplemental folic acid 10 mg per day.

C. She should increase consuming more green vegetables.

D. She should use supplemental folic acid 1 mg per day.

E. She should use supplemental folic acid 4 to 5 mg per day.

Incorrect. Correct answer is E


45% answered correctly

Explanation:

Correct Answer Is E

Folic acid deficiency of mother can result in fetal neural tube defects and for this reason every woman should be started on folic acid from at least one month before conception and
continue during the first trimester. In addition, all women should be encourage to include green vegetables in their diets, at least during this period.

Current recommendation is daily use of folic acid 0.5 mg daily or a multivitamin containing at least 0.4 mg of folic acid.

Women should take 10 times the minimal dose (4 to 5 mg per day) if:

There is a family, or personal history of neural tube defects (NTDs) or a previous pregnancy with NTD
Use of medications that affect folic acid absorption e.g. antiepileptic medications
Diabetes (type I/II)
Mother’s BMI>35 kg/m2

According to National Health and Medical Research Council (NHMRC), the father’s personal history of NTDs is another condition requiring prescription if high dose folic acid.

This woman has previous history of a pregnancy associated with NTD; hence, she should be started on folic acid 4 to 5 mg per day from at least one month before conception and 3
months into the pregnancy.

NOTE – the above recommendations apply for women with all current nutritional status. For example, if a woman is vegetarian, just assuming that she is receiving adequate amount
of folic acid through green vegetables should not result in altered management of folic acid supplementation for her.

References

• http://www.kemh.health.wa.gov.au/development/manua

• https://www.nhmrc.gov.au/_files_nhmrc/file/nics/ma

Last updated:
Time spent: QID:668
2023-2-12

938 of 1943
A 27-year-old woman presents for a pre-pregnancy consultation. She is vegetarian. Which one of the following would be the best recommendation with regard to folic acid
supplementation?

A. She does not need folic acid supplementation because she is receiving adequate amount through her diet.

B. She does not need folic acid supplementation but she should increase consumption of green vegetables.

C. She should be started on folic acid 0.5 mg per day.

D. She should be started on folic acid 0.1 mg per day.

E. She should be started on folic acid 5 mg per day.

Incorrect. Correct answer is C


45% answered correctly

Explanation:

Correct Answer Is C

Folic acid deficiency of mother can result in fetal neural tube defects and for this reason every woman should be started on folic acid from at least one month before conception and
continue during the first trimester. In addition, all women should be encouraged to include green vegetables in their diets, at least during this period.

Current recommendation is daily use of folic acid 0.5 mg daily or a multivitamin containing at least 0.4 mg of folic acid.

Women should take 10 times the minimal dose (4 to 5 mg per day) if:

There is a family, or personal history of neural tube defects (NTDs) or a previous pregnancy with NTD
Use of medications that affect folic acid absorption e.g. antiepileptic medications
Diabetes (type I/II)
Mother’s BMI>35 kg/m2

According to National Health and Medical Research Council (NHMRC), the father’s personal history of NTDs is another condition requiring prescription if high dose folic acid.

The above recommendations apply for women with all current nutritional status, and being a vegetarian does not exclude her. Therefore, she should be started on folic acid
supplementation 0.4 to 0.5 mg per day. She should also be advised to include more green vegetables in her diet if she already has not.

References

• http://www.kemh.health.wa.gov.au/development/manua

• https://www.nhmrc.gov.au/_files_nhmrc/file/nics/ma

Last updated:
Time spent: QID:669
2023-2-12

939 of 1943
A 32-year-old vegetarian woman presents for a pre-pregnancy consultation. She is epileptic and has been on phenytoin for the past 3 years. Which one of the following would be the
best recommendation regarding folic acid supplementation?

A. She does not need folic acid supplementation because she is receiving adequate amount through her diet.

B. She does not need folic acid supplementation but she should increase consumption of green vegetables.

C. She should be started on folic acid 0.5 mg per day.

D. She should be started on folic acid 5 mg per day.

E. She should be started on folic acid 0.1 mg per day.

Incorrect. Correct answer is D


45% answered correctly

Explanation:

Correct Answer Is D

Folic acid deficiency of mother can result in fetal neural tube defects; therefore, every woman should be started on folic acid from at least one month before conception through the
first trimester. In addition, all women should be encouraged to include green vegetables in their diets, at least during this period.

The recommendation is daily use of folic acid 0.5 mg daily or a multivitamin containing at least 0.4 mg of folic acid.

Woman should take 10 times the minimal dose (4 to 5 mg per day) if:

There is a family, or personal history of neural tube defects (NTDs) or a previous pregnancy with NTD
Use of medications that affect folic acid absorption e.g. antiepileptic medications
Diabetes (type I/II)
Mother’s BMI>35 kg/m2

According to National Health and Medical Research Council (NHMRC), the father’s personal history of NTDs is another condition requiring prescription of high dose folic acid.

The above recommendations apply to women with all current nutritional status. Being a vegetarian does not exclude her or change the recommendations. Since she is on
antiepileptic drugs that decrease the absorption of folic acid, she should be started on folic acid supplementation 4 to 5 mg per day. She should also be advised to include more
green vegetables in her diet if she already has not.

References

• http://www.kemh.health.wa.gov.au/development/manua

• https://www.nhmrc.gov.au/_files_nhmrc/file/nics/ma

Last updated:
Time spent: QID:670
2023-2-12

940 of 1943
A 41-year-old woman presents with her cervical screening results reporting CIN2. Colposcopic examination is arranged and performed but reveals no gross abnormality. Which one
of the following would be the next best step in management?

A. Cone biopsy.

B. LEEP.

C. Repeat pap smear in 12 months.

D. Reassure the patient as it could be a false positive result.

E. Repeat pap smear in 6 months.

Correct
45% answered correctly

Explanation:

Correct Answer Is A

Patients with CIN2 cervical screening result (equivalent to HSIL in modified Bethesda system) should be referred for colposcopy. If the lesion cannot be visualized on colposcopy,
performing a cone biopsy would be the next best step in management.

Cervical squamous cell abnormalities in different nomenclatures, i.e. CIN (Cervical Intraepithelial Neoplasia) and Australian Modified Bethesda system are defined and compared in
the following table:

Australian modified
Description CIN grade
Bethesda system
Normal Normal Within normal limits
Atypia: reactive or
neoplastic
Atypia ASCUS1
HPV HPV LSIL4
Mild dysplasia 2 LSIL
CIN 1
Moderate dysplasia CIN 2 HSIL
Severe dysplasia CIN 3 HSIL5
Carcinoma in situ 3 HSIL
CIS
Invasive carcinoma Invasive carcinoma Invasive carcinoma
1
ASCUS – Atypical Squamous Cells of Undetermined Significance

2
CIN – Cervical Intraepithelial Neoplasia

3
CIS – Carcinoma in situ

4
LSIL – Low-grade Squamous Intraepithelial Lesion

5
HSIL – High-grade Squamous Intraepithelial Lesion

CIN 2 and 3 are equivalents of HSIL.

Women with HSIL on cervical screening should be referred to specialist for colposcopy. If colposcopy is unsatisfactory, the next step to consider is cone biopsy.

Cone biopsy is an excisional technique in which a significant length of tissue along the cervical canal (from 1.5–3 cm) is removed. Traditionally, cone biopsy has been performed
using the ‘cold-knife’ technique.

The specific indications for cone biopsy are:

Failure to visualize the upper limit of the cervical transitional zone (TZ) in a woman with a HSIL on her referral cervical smear i.e. unsatisfactory colposcopy
Suspicion of an early invasive cancer on cytology, biopsy or colposcopic assessment
The suspected presence of an additional significant glandular abnormality (i.e. AIS) on cytology or biopsy (i.e., a mixed lesion)

References

• Australian Government, Department of Health – Cancer screening

• AAFP - Management of Histologic Abnormalities of the Cervix

Last updated:
Time spent: QID:674
2023-2-12

941 of 1943
A 26-year-old woman comes to you with a cervical screening test result, reporting high-grade squamous intraepithelial lesion (HSIL). She is 34 weeks’ pregnant. Which one of the
following is the most appropriate management at this stage?

A. Loop excision.

B. Cone biopsy.

C. Continuation of the pregnancy.

D. Offer termination of the pregnancy now.

E. Refer for colposcopy.

Incorrect. Correct answer is E


45% answered correctly

Explanation:

Correct Answer Is E

Based on recommendations from National Health and Medical Research Council (NHMRC), management of cervical neoplasia in pregnancy is as follows:

LSIL (CIN 1)

Women with low-grade cytologic lesions should be managed in the same way as for non-pregnant women with such lesions, with a repeat smear after 12 months, if they have had a
normal cervical screening in the past 2-3 years. Otherwise, they should be referred to a specialist for colposcopy.

HSIL (CIN 2,3)

With a high-grade squamous intraepithelial lesion (HSIL) (CIN 2,3) the woman should be referred for colposcopy.

Colposcopy is safe during pregnancy. The colposcopic evaluation of the cervix may be more difficult due to vaginal laxity preventing complete visualization of the transformation
zone. The increased vascularity due to pregnancy may also be difficult to interpret.

Experienced colposcopists will not usually perform a biopsy if they are confident that they have excluded an invasive cancer. If no lesion is identified at colposcopy, it is advisable to
request a review of all the cytological slides. If the diagnosis of a HSIL is confirmed, a second opinion from another colposcopist with wide experience in the colposcopic evaluation
of pregnant women is recommended.

With HSIL, it will be prudent to review the woman at approximately 20–24 weeks with cytology and colposcopy to determine, as far as possible, that she does not have an invasive
lesion.

Definitive treatment of a high-grade lesion, with the exception of invasive cancer, may be deferred safely until after pregnancy.

Invasive carcinoma

A diagnosis of cervical cancer in pregnancy poses major challenges to the patient, her family, and the treating physician(s). The general principles in management of invasive
cervical carcinoma are as follows:

If the fetal lungs are mature at the time of diagnosis, or if the fetus is at/near the gestational age of expected lung maturity, immediate delivery and definitive treatment of
the mother is the preferred approach (antenatal corticosteroid therapy is given if indicated).

If the pregnancy is previable and the patient decides not to continue the pregnancy, immediate therapy of the mother should be initiated.

In all other patients, decisions regarding timing of treatment and delivery require careful consideration of the stage of disease, the trimester in which the diagnosis is
made, and the preferences of the affected woman and her family regarding the pregnancy.

References

• National cervical screening program

• UpToDate - Cervical cancer in pregnancy

Last updated:
Time spent: QID:675
2023-2-12

942 of 1943
A 31-year-old woman gave birth to a baby of normal weight through vaginal delivery. The delivery was complicated by a small perianal tear that was taken care of without stitching.
Today on the fifth day postpartum, she presents with heavy bright red vaginal bleeding. She mentions that the lochia was in scant amounts after delivery compared to her previous
pregnancy. On examination, she has a temperature of 38.8°C. The uterus is mildly tender to palpation. Which one of the following could be the most likely diagnosis?

A. Endometritis.

B. Infection of the perianal tear.

C. Retained products of conception.

D. Cervical tear.

E. Uterine rupture.

Incorrect. Correct answer is C


45% answered correctly

Explanation:

Correct Answer Is C

The presence of secondary postpartum hemorrhage (between 24 hours and 12 weeks postpartum) that is bright red and fever suggests retained products of conception (RPOC).

After separation of placenta, the basal portion of the decidua remains. This decidua divides into two layers: the superficial layer that is shed and the deep layer regenerates new
endometrium, which covers the entire endometrial cavity by the 16th postpartum day. Normal shedding of blood and decidua is referred to as lochia rubra (red, red brown), and lasts
for the first few days following delivery. Vaginal discharge then becomes increasingly watery, called lochia serosa (pinkish brown) lasting for 2 to 3 weeks. Ultimately, the discharge
becoms yellowish-white, the lochia alba.

Scant amount of lochia in the first few days after delivery suggests that the placental site may have not undergone involution probably due the RPOC. Usually, retained products
undergo necrosis with fibrin deposition and may eventually form a so-called placental polyp. As the scar of the polyp detaches from the myometrium, brisk hemorrhage may occur.

The necrotic products may also become infected and cause uterine infection presented by fever, lower abdominal pain and uterine tenderness.

(Option A) Endometritis can cause fever, offensive-odor lochia and abdominal pain/tenderness. Endometritis is the most common cause of postpartum fever. It occurs within the
first 5 days postpartum with the peak incidence between days 2 and 3 postpartum. Although vaginal bleeding can be a feature at occasions, bright red bleeding makes endometritis
less likely.

(Option B) Genital lacerations are another cause of postpartum fever. The peak incidence of wound infection is between days 4 and 5. Although the temporal appearance of the
fever favors wound infection as a cause, this diagnosis is less likely because wound infection do not affect the normal course of lochia, and does not have heavy bright red bleeding
as an expected feature. Moreover, there are no clues to wound infection such as erythema, tenderness or discharge in history.

(Option D) Cervical tear can be another cause of bleeding or fever, but cervical tear tends to present with primary rather than secondary postpartum hemorrhage (occurring after the
first 24 hours postpartum). An infected overlooked minor cervical laceration can cause fever but is not very likely to cause bright red bleeding. Furthermore, genital tract lacerations
do not affect lochia.

(Option E) Uterine rupture is very unlikely to occur after 24 hours of delivery.

References

• http://www.uptodate.com/contents/retained-products

• http://www.sahealth.sa.gov.au/wps/wcm/connect/dc33

Last updated:
Time spent: QID:677
2023-2-12

943 of 1943
A 21-year-old woman presents to your clinic asking for emergency contraception. Which one of the following is the most important initial question to ask her before further
management?

A. Previous use of OCP.

B. History of headaches.

C. The date of the unprotected sex.

D. The date of the last menstrual period (LMP).

E. Family history of hypercoagulable state.

Incorrect. Correct answer is C


45% answered correctly

Explanation:

Correct Answer Is C

Emergency contraception (also known as postcoital contraception, the morning-after pill, or plan B) refers to the use of drugs or a device as an emergency measure to prevent
pregnancy in women who have had recent unprotected intercourse, including those who have had a failure of another method of contraception.

Although conception is possible on only a few days of the menstrual cycle, emergency contraception is offered regardless of the timing of the menstrual cycle due to uncertainty in
timing of ovulation; therefore, LMP is not a significant determining factor for considering emergency contraception.

Of the options, the most important question to ask is the date of unprotected sex. Most methods are useful within a short time of unprotected sex. The window to use an emergency
contraception method is simplified to up to 5 days (120 hours) post-coitus. There is a linear relationship between efficacy and the time from intercourse. The sooner a method is
used the lesser the chance of pregnancy would be.

There is no medical contraindication to using pharmacological emergency contraception methods. Contraindications to use of IUCD are the same for routine use of intrauterine
devices.

References

• http://www.uptodate.com/contents/emergency-contrac

• http://kemh.health.wa.gov.au/development/manuals/O

Last updated:
Time spent: QID:683
2023-2-12

944 of 1943
The anomaly of the child, illustrated in the following photograph, is more likely to be due to which one of the following drugs taken by the mother during the pregnancy?

A. Phenytoin.

B. Sodium valproate.

C. Levodopa.

D. Carbamazepine.

E. Lamotrigine.

Incorrect. Correct answer is B


45% answered correctly

Explanation:

Correct Answer Is B

Previously, women with epilepsy were informed that their risk for fetal malformations was increased 2- to 3 folds, but more recent data suggest that the risk may not be as great as once thought, particularly for newer agents. More recent studies
showed that the risk of fetal malformation following intrauterine exposure to anticonvulsants that are currently in use is approximately the same or only slightly elevated compared to children without exposure.

Sodium valproate is one important exception to these findings. Women treated with sodium valproate are at significantly increased risk of fetal malformations.

The most frequently reported anomalies are:

Orofacial clefts
Cardiac anomalies
Neural tube defects

One study showed that major malformations occurred in 9% of fetuses with first-trimester valproate exposure; however, different data suggest the rate to be as high as 16%.

References

• PubMed - Teratogenic effects of antiepileptic drugs

• William’s Obstetrics – 2014 - McGraw Hill – pages 246-7

Last updated:
Time spent: QID:130 2023-2-12

945 of 1943
Which one of the following is unlikely to cause fetal growth restriction?

A. Cytomegalovirus infection.

B. Trisomy 13.

C. Thalassemia minor with hemoglobin of 80 g/L.

D. Mother with essential hypertension on treatment with methyldopa.

E. Lupus nephritis.

Incorrect. Correct answer is C


45% answered correctly

Explanation:

Correct Answer Is C

Of the options, thalassemia minor has not been shown to be associated with negative pregnancy outcomes such as fetal growth restriction. Although a hemoglobin level of 80 g/L
may appear low at the first look, it should be noted that in pregnant women the anemia is defined as a hemoglobin level of less than 100 g/L (not 120 g/L in non-pregnant women).
The reason is that plasma volume and hematocrit increase by 50% and 30% respectively, resulting in diluted plasma.

(Option A) Cytomegalovirus (CMV) is the most common prenatal viral infection. Fetal growth restriction is a known complication of intrauterine infection with CMV.

(Option B) Trisomy 21 (Down syndrome), trisomy 18 (Edward syndrome), Trisomy 13 (Patau syndrome) and Turner’s syndrome are congenital causes of fetal growth restriction.
Trisomy 13 is characterized by profound mental retardation, cyclopia, proboscis, holoprosencephaly and severe orofacial clefts.

(Option D) Maternal hypertension is another important cause of fetal growth restriction. Studies suggest that treatment of hypertension during pregnancy and consequent drop in
maternal mean arterial pressure is associated with a lower birth weight. In fact, it is postulated that mild to moderate hypertension of the mother should be left untreated to prevent
treatment-induced fetal growth restriction.

(Option E) Hypertension can be seen in 10-20% of pregnant women with systemic lupus erythematosus, especially if the kidneys are involved (lupus nephritis). Maternal
hypertension can result in fetal growth restriction.

TOPIC REVIW

Causes of intrauterine growth restriction:

Medical

Chronic hypertension
Preeclampsia
Diabetes mellitus
Systemic lupus erythematous
Chronic renal disease
Inflammatory bowel disease
Severe hypoxic lung disease

Maternal

Smoking
Alcohol use
Cocaine use
Warfarin
Phenytoin
Malnutrition
Prior history of pregnancy with intrauterine growth restriction
Residing at altitude above 5000 feet

Infections

Syphilis
CMV
Toxoplasmosis
Rubella
Hepatitis B
Herpes simplex 1,2
HIV

Congenital

Trisomy 21
Trisomy 18
Trisomy 13
Turner’s syndrome

946 of 1943
References

• RACGP - Intrauterine growth restriction: Diagnosis and management

• IJRM - The pregnancy outcome in patients with minor β-thalassemia

• CMR - Lupus and Pregnancy: Complex Yet Manageable

Last updated:
Time spent: QID:700
2023-2-12

947 of 1943
A 58-year-old woman presents to your practice with complaints of hot flushes and mood swings for the past 12 months. Her menstrual periods ceased 2 years ago. She has the
history of breast cancer at the age of 35 that was treated successfully. Which one of the following is be the most appropriate management for her symptoms?

A. Clonidine.

B. Fluoxetine.

C. Hormone replacement therapy.

D. Estrogen patch.

E. No treatment at this age.

Incorrect. Correct answer is B


45% answered correctly

Explanation:

Correct Answer Is B

Menaopausal hormone replacement therapy (HRT) is the most effective treatment for vasomotor symptoms (e.g. hot flushes) of menopause. Menopause is associated with several
complications, but vasomotor symptoms are the only indication to start HRT in a menopausal woman. However, personal history of a estrogen-dependent cancer makes HRT of any
form an unfavorable treatment option. For those in whom HRT is not an appropriate option either due to a medical condition or the patient’s preferences, the selective serotonin
reuptake inhibitors (SSRIs) and selective serotonin norepinephrine reuptake inhibitors (SNRIs) are first choices for treatment of vasomotor symptoms.

The following are recommendations by Royal Australian and New Zealand College of Obstetrics and Gynaecology (RANZCOG) regarding menopausal hormone therapy (MHT):

Recommendation1 - The primary indication for the use of MHT is the alleviation of distressing menopausal vasomotor symptoms.

Recommendation 2 - In women with primary ovarian insufficiency, MHT should be continued until the normal age of the menopause.

Recommendation 3 - MHT is also effective and appropriate for the prevention of osteoporosis related fracture in at risk women within 10 years of the menopause.

Recommendation 4 - The risk of VTE and stroke increases with oral MHT but the absolute risk is very small before age 60 years.

Recommendation 5 - In women within 10 years of the menopause MHT does not increase the risk of coronary heart disease.

Recommendation 6 - Combined MHT use for more than 5 years may be associated with an increased risk of breast cancer. This risk appears to be related to the use of a
progestogen and duration of therapy.

Recommendation 7 - Estrogen-only MHT does not increase risk of breast cancer.

Recommendation 8 - Current safety data do not support the use of MHT in breast cancer survivors.

Recommendation 9 - Estrogen-only therapy is appropriate for women who have undergone hysterectomy.

Recommendation 10 - Eestrogen plus progestogen should be used in women with an intact uterus.

Recommendation 11 - The dose and duration of therapy should be consistent with treatment goals.

References

• http://www.uptodate.com/contents/menopausal-hot-fl

• https://www.ranzcog.edu.au/doc/hormone-replacement

Last updated:
Time spent: QID:706
2023-2-12

948 of 1943
A 32-year-old woman is referred to you at 36 weeks gestation for management of eclampsia. While examining, the patient develops a seizure in front of you. Which one of the
following is the most appropriate immediate management?

A. Phenytoin.

B. Intravenous diazepam.

C. Ensuring airway patency.

D. Magnesium sulphate.

E. Induction of labor.

Incorrect. Correct answer is C


45% answered correctly

Explanation:

Correct Answer Is C

In approaching a patient, especially in an emergency setting such as this one, the most immediate management is checking and stabilizing airway, breathing and circulation (ABC),
regardless of the underlying etiology.

Ensuring and maintaining airway patency should always be the first priority, followed by sufficient breathing and oxygenation, and circulation.

Magnesium sulphate is the anticonvulsant of choice for eclampsia. Intravenous diazepam can be given while magnesium sulphate is prepared and the seizure is prolonged.

Arrangements for delivery should be decided once the woman’s condition is stable. In the meantime, close fetal monitoring should be maintained.

References

• http://www.ranzcog.edu.au/doc/somanz-hypertensive-

• http://www.kemh.health.wa.gov.au/development/manua

Last updated:
Time spent: QID:739
2023-2-12

949 of 1943
A 29-year-old woman presents with two episodes of bleeding, one hour apart at 39 weeks gestation. She has no abdominal pain or any other complaints. Physical exam is
inconclusive. An ultrasound scan reveals placenta previa. Which one of the following is the most appropriate next step in management?

A. Admit her for supervision.

B. Discharge her home.

C. Induction of labor.

D. Cesarean section.

E. Vaginal examination.

Incorrect. Correct answer is D


45% answered correctly

Explanation:

Correct Answer Is D

Most women who initially present with symptomatic placenta previa respond to supportive therapy, and do not require immediate delivery. Even a large bleed does not preclude
conservative management. Management of placenta previa when mother and fetus are stable is conservative until 38 weeks pregnancy when delivery should be performed by
cesarean section. Since this woman is already at 39 weeks gestation, cesarean section without further need to document fetal lung maturity would be the most appropriate option.

Admission for supervision (option A) was the appropriate option at earlier gestational ages (<37 weeks) where expectant management (if mother and fetus were stable)
was considered until adequate fetal maturity is ensured.

Vaginal delivery is contraindicated in placenta previa most of the time, so induction of labor (option C) is inappropriate. Digital vaginal examination (option E) should be avoided in
patients with placenta previa.

References

• http://www.sahealth.sa.gov.au/wps/wcm/connect/b1c6

• https://www.ranzcog.edu.au/doc/rcog-placenta-praev

Last updated:
Time spent: QID:770
2023-2-12

950 of 1943
A 37-year-old woman presents to your practice for contraception advice. She has otosclerosis and uses hearing aids. She had 3 pregnancies, during which her hearing deteriorated
rapidly. Which one of the following is the most appropriate method to recommend?

A. Low-dose combined oral contraceptive pills.

B. High-dose combined oral contraceptive pills.

C. Progestin-only pills.

D. Condoms.

E. Intrauterine contraceptive device (IUCD).

Incorrect. Correct answer is E


45% answered correctly

Explanation:

Correct Answer Is E

Otosclerosis is an often inherited condition, which is the most common cause of progressive deafness in young adults. It involves hardening of the bones of the middle ear resulting
in progressive hearing loss that is often irreversible. In most cases of otosclerosis, the smallest bone (stapes) is involved. The bone becomes calcified or hardened by overgrowth of
new bone. Treatment for otosclerosis is by a surgical procedure called stapedectomy, duirng which the top portion of the stapes is removed and a small prosthesis is inserted
instead.

Although not supported by solid evidence, there have been suggestions that the combined oral contraceptives (COCs) can worsen otosclerosis, especially if the patient has
experienced onset or deterioration during pregnancy or steroid use. For this reason and depsite lack of evidence, avoiding systemic hormonal methods may be appropriate. Of the
given non-hormonal methods, IUCD provides more contraception benefit and less failure rates and can be advised as an acceptable method for her.

References

• https://www.ncbi.nlm.nih.gov/pubmed/19121641

Last updated:
Time spent: QID:775
2023-2-12

951 of 1943
A 27-year-old woman presents with severe nausea and vomiting at 8 weeks pregnancy. She cannot take solid foods but is able to drink small sips of liquids. She is concerned that
she might have gastroenteritis, because her partner has recently had it. Which one of the following is the next best investigation in this situation?

A. Pelvic ultrasound.

B. Urinalysis and culture.

C. Stool exam and culture.

D. Quantitative β-HCG.

E. Abdominal ultrasound.

Incorrect. Correct answer is B


45% answered correctly

Explanation:

Correct Answer Is B

Although this woman might have gastroenteritis, the first thing to consider, given the gestational age, is hyperemesis gravidarum/nausea and vomiting of pregnancy. The mean
onset of hyperemesis gravidarum/nausea and vomiting of pregnancy symptoms is at 5 to 6 weeks of gestation, peaking at about 9 weeks, and usually abating by 16 to 20 weeks of
gestation; however, symptoms may continue until the third trimester in 15-20% of pregnant women and until delivery in 5%. Sixty percent of women are asymptomatic six weeks
after onset of nausea. If vomiting persists beyond a few days postpartum, other etiologies should be investigated.

Diagnosis of hyperemesis gravidarum is mainly clinical; however, investigations are needed both for assessment of the patient’s status and excluding other possible causes of
nausea and vomiting.

The standard initial evaluation of pregnant women with persistent nausea and vomiting includes measurement of weight, vital signs, orthostatic blood pressure, serum electrolytes,
and urine ketones and specific gravity and urine microscopy and culture.

Attending to hydration status and electrolyte imbalance is of paramount importance before proceeding to investigations for and underlying cause. A spot urine dipstick at the
office/ED or a formal urinalysis, microscopy, and culture is as well as electrolyte assays is the first step while arrangements for other investigations are made. An obstetric
ultrasound exam (pelvic ultrasound) (option A) should be performed to look for gestational trophoblastic disease or multiple gestation as possible underlying causes. At this
gestational age, transvaginal ultrasound is the preferred method. Pelvic ultrasound allows quick visualization of the female pelvic organs and structures including the uterus, cervix,
vagina, fallopian tubes, ovaries, and in pregnant women, the fetus and the gestational sac.

(Option C) Stool exam and culture may be considered for evaluation of the selected patients if gastroenteritis is suspected as the cause of the clinical picture.

(Option D) With conditions such as multiple or molar pregnancy, quantitative serum beta HCG is higher than normal pregnancies. However, the most common cause for a β-HCG
level unproportionate to gestational age is dating errors. If such discrepancy is found on quantitative β-HCG test, ultrasound will be required for accurate estimation of gestational
date; therefore, this test is neither necessary nor useful as initial investigation.

(Option E) Abdominal ultrasound is inferior to transvaginal ultrasound for evaluation of female reproductive organs and the fetus.

References

• South Australian Perinatal Practice Guideline Nausea and Vomiting in Pregnancy and Hyperemesis Gravidaru

• UpToDate - Nausea and vomiting of pregnancy: Clinical findings and evaluation

Last updated:
Time spent: QID:786
2023-2-12

952 of 1943
A 24-year-old woman, who underwent an episiotomy during labor four days ago, has presented with severe vaginal pain. On examination, an 8-cm hematoma is noted at the site of
the episiotomy. The woman is hemodynamically stable. Which one of the following is the most appropriate next step in management?

A. Aspirate the mass.

B. Explore the hematoma.

C. Reassure the patient as this will resolve spontaneously.

D. Antibiotics.

E. Oral analgesics and review in one week.

Incorrect. Correct answer is B


45% answered correctly

Explanation:

Correct Answer Is B

Most puerperal hematomas arise from bleeding lacerations related to operative deliveries or episiotomy, but it is also possible for a hematoma to result from spontaneous injury to a
blood vessel in the absence a of laceration/incision of the surrounding tissue.

The most common locations for puerperal hematomas are vulva, vagina/paravaginal area, and retroperitoneum.

The diagnosis of most puerperal hematomas is based upon the presence of characteristic symptoms and findings on physical examination including:

Vulvar hematomas - usually present with rapid development of a severely painful, tense, compressible mass covered by skin with purplish discoloration. A vulvar hematoma may be
an extension of a vaginal hematoma that has dissected through loose subcutaneous tissue into the vulva.

Vaginal hematomas - often present with rectal pressure; however, hemodynamic instability due to bleeding into the ischiorectal fossa and paravaginal space may be the first sign of
a vaginal hematoma, and can result in hypovolemic shock. On physical examination, a large mass protruding into the vagina is often seen.

Retroperitoneal hematomas - they extend between the folds of the broad ligament and may be asymptomatic initially. Due to the significant amount of blood that can accumulate in
the retroperitoneal space, these patients often present with tachycardia, hypotension, or even shock. Patients with a retroperitoneal hematoma usually do not present with pain
unless the hematoma is associated with trauma. Palpation of an abdominal mass or fever can also be signs of a retroperitoneal hematoma.

Treatment of a hematoma depends on the size and location:

Non-expanding hematomas that are <3cm can be managed conservatively with analgesics and ice packs.

Management of an expanding hematoma or one greater than 3cm is with surgical exploration under anesthesia. An incision is made and the hematoma is evacuated. The surgical
site should not be sutured. Vagina is often packed for 12-24 hours. An indwelling urinary catheter may be indicated.

This patient has a large haematoma (>3cm) that needs surgical excision and evacuation.

Aspiration of the hematoma is not an appropriate treatment. If surgical intervention is indicated excision and evacuation is the preferred option, followed by vaginal packing for 12-
24 hours.

References

• https://www.sahealth.sa.gov.au/wps/wcm/connect/149

Last updated:
Time spent: QID:841
2023-2-12

953 of 1943
A 28-year-old primigravida woman presents to the maternity unit at 38 weeks gestation after spontaneous rupture of membrane. On vaginal examination, the cervix is 5 cm dilated
and the membranes are ruptured. Meconium-stained liquor is noted in the vagina. A cardiotocography (CTG) unveils a fetal heart rate of 130 bpm, variability of 5 bpm and variable
decelerations. Which one of the following would be the most appropriate next step in management?

A. Urgent cesarean section.

B. Fetal Scalp blood sampling.

C. Continue CTG monitoring.

D. Start infusion of oxytocin to facilitate the labor.

E. High forceps delivery.

Incorrect. Correct answer is B


45% answered correctly

Explanation:

Correct Answer Is B

Meconium can be found in the gastrointestinal tract of the fetus from as early as 10 -16 weeks gestation. Although 75% of meconium is water, the remaining 25% consists of
gastric secretions, bile salts, mucous, vernix, lanugo, blood, pancreatic enzymes, free fatty acids and squamous cells. Meconium staining of the amniotic fluid (MSAF) occurs in
approximately 10-15% of labors. MSAF rarely occurs before 30 weeks gestation. In a preterm fetus, MSAF can suggest possible infection or hypoxia. The incidence of this condition
increases with longer gestations (mature fetus) and approximately 20% of newborns have MSAF at term. Neonatal aspiration of MSAF can become a serious problem if pulmonary
hypertension and severe hypoxemia develop. Such conditions require expert neonatal intensive care.

Meconium stained liquor is one of the conditions considered as intrapartum risk factors for the fetus’ wellbeing. With meconium present, continuous cardiotocography (CTG) should
follow for ongoing fetal assessment. Further Actions depend on CTG results:

Reassuring CTG: a CTG is reassuring if the following criteria are met:

Baseline fetal heart rate (FHR) is between 110-160 bpm


Variability of FHR is between 5-25 bpm
Decelerations are absent or early
The significance of the presence or absence of accelerations is unclear and are exclude for interpretation.

With a reassuring CTG, watchful waiting for labor to progress is recommended, unless there are other contraindications. Continuous CTG and close monitoring of the mother and
fetus is essential.

Non- reassuring CTG: a CTG is non-reassuring if any of the following is present:

Baseline FHR is between 100-109 bpm or between 161-170 bpm


Variability of FHR is reduced (3-5 bpm for >40 minutes)
Decelerations are variable without complicating features

Abnormal CTG: a CTG is abnormal if at least two of the features described in non-reassuring CTG are present OR one or more of the following features exists:

Baseline FHR is <100 bpm or >170bpm


Variability is absent or <3 bpm
Decelerations are prolonged for >3 minutes OR late OR have complicated variables

With a non-reassuring or abnormal CTG, fetal blood sampling should follow unless contraindicated.

Contraindications to fetal blood sampling include the following:

Clear evidence of serious fetal compromise (e.g., complete absence of reassuring features) – in such conditions urgent preparation to expedite birth should be made.
Prolonged deceleration
Mobile presenting part
Unknown presentation
Face presentation
Undilated cervix (<3 cm)
Active second stage of labor
Fetal hereditary bleeding disorders (e.g., suspected fetal thrombocytopenia, hemophilia)
Maternal infection (e.g., HIV, hepatitis, herpes simplex virus, suspected intrauterine sepsis)
Prematurity (gestation < 34 weeks) as delayed birth due to the procedure may be associated with an increase in adverse outcomes because the small “at risk” fetus will
have a lower threshold than a term infant for fetal compromise.

This fetus has a non-reassuring CTG and no contraindications to fetal scalp blood sampling; therefore, this procedure is the most appropriate next step in management.

Proceeding to delivery is indicated if fetal blood sampling shows genuine fetal compromise.

References

954 of 1943
• WA Health Department – Obstetrics and Gynaecology Guidelines

• RANZCOG Intrapartum Fetal Surveillance Clinical Guideline


Last updated:
Time spent: QID:842
2023-2-12

955 of 1943
A 31-year-old woman is admitted to the Maternity Unit for labor. She has had regular antenatal visits and her pregnancy has been uneventful thus far. During the labor, meconium
liquor passage is noted. Cardiotocography shows a fetal heart rate (FHR) of 149 bpm. There are no decelerations or accelerations. There is beat-to-beat variability of 15 bpm.
Vaginal exam is normal. Which one of the following is the most appropriate next step in management?

A. Fetal scalp blood sampling as there is a 10% chance of hypoxia.

B. Fetal scalp blood sampling as there is a 50% chance of hypoxia.

C. Fetal scalp blood sampling as there is a 75% chance of hypoxia.

D. Proceeding to immediate cesarean section.

E. The CTG is normal and close monitoring until delivery is all required for now.

Incorrect. Correct answer is E


45% answered correctly

Explanation:

Correct Answer Is E

Meconium staining of the amniotic fluid (MSAF) occurs in around 10-15% of labors. MSAF rarely occurs before 30 weeks gestation. The incidence of this condition increases with
longer gestations (mature fetus). Approximately 20% of newborns have MSAF at term. Meconium stained liquor is one of conditions considered as intrapartum risk factors for the
fetus’ wellbeing and one of the indications for continuous cardiotocography (CTG). With meconium present, continuous cardiotocography (CTG) should follow for ongoing fetal
assessment. Further Actions depend on CTG results:

Reassuring CTG: a CTG is reassuring if the following criteria are met:

Baseline fetal heart rate (FHR) is between 110-160 bpm


Variability of FHR is between 6-25 bpm
Decelerations are absent or early
The significance of the presence or absence of accelerations is unclear and exclude for interpretation.

The CTG in the scenario is normal (reassuring) and as long as so, meconium-stained liquor is not clinically significant and the labor is allowed to progress normally; however, since
meconium-stained liquor is an intrapartum risk factor continuous CTG and close monitoring of the fetus should be maintained.

Non- reassuring CTG: a CTG is non-reassuring if any of the following is present:

Baseline FHR is between 100-109 bpm or between 161-170 bpm


Reduced FHR variability (3-5 bpm for >40 minutes)
Decelerations are variable without complicating features

NOTE - The absence of accelerations in intrapartum interpretation is not considered abnormal.

Abnormal CTG: a CTG is abnormal if two or more features described in non-reassuring CTG are present OR one or more of the following features exists:

Baseline FHR is <100 bpm or >170bpm


Variability is absent or <3 bpm
Decelerations are prolonged for >3 minutes OR late OR have complicated variables

With non-reassuring or abnormal CTG, reversible causes leading to the abnormality should promptly be investigated and reversed if possible. Measures such as repositioning of the
mother, rehydration, etc may result in improved CTG readings. Persistent non-reassuring or abnormal features in CTG should be followed by fetal scalp blood sampling unless
contraindicated.

Contraindications to fetal scalp blood sampling include:

Clear evidence of serious fetal compromise (e.g. complete absence of reassuring features) – in such conditions urgent preparation to expedite birth should be made
Prolonged deceleration
Mobile presenting part
Unknown presentation
Face presentation
Undilated cervix (<3 cm)
Active second stage of labor
Fetal hereditary bleeding disorders (e.g. suspected fetal thrombocytopenia, hemophilia)
Maternal infection (e.g. HIV, hepatitis, herpes simplex virus, suspected intrauterine sepsis)
Prematurity (gestation < 34 weeks) as delayed birth due to the procedure may be associated with an increase in adverse outcomes because the small ‘at risk’ fetus will
have a lower threshold than a term infant for fetal compromise.

References

• WA Health Department – Obstetrics and Gynaecology Guidelines

• RANZCOG Intrapartum Fetal Surveillance Clinical Guideline

956 of 1943
Last updated:
Time spent: QID:843
2023-2-12

957 of 1943
A 25-year-old woman presents to the emergency department at 14 weeks pregnancy with complaint of urine retention for the past 18 hours. She had an obstetric ultrasound at 10
weeks pregnancy confirming the date. In the emergency department, a transvaginal ultrasound is performed confirming the gestational age and revealing a retroverted uterus. On
abdominal examination, there is a tender mass midway below the umbilicus. Which one of the following is the most likely diagnosis?

A. Ectopic pregnancy.

B. Ovarian fibroid.

C. Red degeneration of uterine fibroid.

D. Ovarian cyst.

E. Incarcerated uterus.

Incorrect. Correct answer is E


45% answered correctly

Explanation:

Correct Answer Is E

The presentation is classic for incarcerated gravid uterus.

The term incarcerated gravid uterus refers to a pregnant uterus entrapped in the pelvis by the sub-promontory sacrum.

In very early pregnancy, the uterus is retroverted (retroflexed) in up to 20% of women. As the uterus enlarges during the first trimester, the fundus normally rises from the hollow of
the sacrum to an anterior ventral position and corrects the retroversion. However, in rare cases, the fundus becomes wedged below the sacral promontory, where it continues to
enlarge. Concomitantly, the cervix becomes displaced upwards against or above the symphysis pubis and pushes against the urethra and bladder, interfering with normal voiding.

Eventually, the posterior pelvis becomes too small to accommodate the increasing size of the fundus; this typically occurs when the pregnancy progresses beyond 20 weeks of
gestation. At this point, the anterior lower uterine wall begins to thin and balloon into the upper abdomen to form a sacculation. Concomitantly, both the bladder and the cervix are
pulled up into the abdominal cavity, towards the umbilicus. The cervix can stretch to 10 cm or more in length, such that the internal os becomes located above the symphysis pubis,
and occasionally above the bladder.

Normal gravid uterus Incarcerated uterus


UpToDate - http://www.uptodate.com/contents/incarcerated-gravid-uterus

958 of 1943
History and symptoms:

Typically, the patient presents at 14 to 16 weeks of pregnancy with symptoms related to pressure on the anatomic structures adjacent to the entrapped enlarging uterus. The most
common symptoms are pain and progressive difficulty voiding. The pain could be abdominal, suprapubic, or felt in the back; or may be just a discomfort or feeling of fullness in the
pelvis.

Urinary symptoms include dysuria, frequency, sensation of incomplete emptying, stress incontinence, and urinary retention (most common). Pressure on rectum can cause rectal
pressure, tenesmus and worsening constipation. Vaginal bleeding can infrequently occur.

Symptoms can be intermittent, resolving for a period of time and then returning weeks later.

Exam finding:

Severe anterior displacement of the cervix behind the pubic symphysis, making the physician unable to visualize the cervix with a speculum or palpate the external os on
pelvic examination
The vagina is angulated anteriorly and a large, soft, smooth, nontender mass (the incarcerated uterus) fills the cul-de-sac
Initially, the uterus may be difficult to palpate abdominally, and later in pregnancy the fundal height may lag behind that expected for gestational age. The abdomen may
appear full or distended due to an overdistended bladder.

Ultrasonography is the most appropriate initial investigation when the condition is suspected.

References

• http://www.uptodate.com/contents/incarcerated-grav

Last updated:
Time spent: QID:846
2023-2-12

959 of 1943
Which one of the following drugs has the highest rate of congenital malformations if used in pregnancy?

A. Carbamazepine.

B. Phenytoin.

C. Sodium valproate.

D. Levetiracetam.

E. Lamotrigine.

Incorrect. Correct answer is C


45% answered correctly

Explanation:

Correct Answer Is C

Sodium valproate carries the highest teratogenicity rate among all antiepileptic drugs. The potential congenital defects caused by sodium valproate are shown in the following table:

Neural tube defects Spina bifida, anencephaly


Heart defects Congenital ventricular septal defect, aortic stenosis, patent ductus arteriosus, aberrant pulmonary artery
Radial ray defect, polydactyly (more than 5 fingers), oligodactyly (less than 5 fingers), absent fingers,
Limb defects overlapping toes, camptodactyly (fixed flexion deformity of one or more proximal interphalangeal joints), split
hand, ulnar or tibial hypoplasia,
Genitourinary defects Hypospadias, renal hypoplasia, hydronephrosis, duplication of calyceal system
Hydranencephal (1), porencephaly(2), arachnoid cysts, cerebral atrophy, partial agenesis of corpus callosum,
Brain anomalies agenesis of septum pellucidum, lissencephaly (3) of medial sides of occipital lobes, Dandy-Walker anomaly
(4)
Bilateral congenital cataract, optic nerve hypoplasia, tear duct anomalies, microphthalmia, bilateral iris
Eye anomalies
defects, corneal opacities
Tracheomalacia , lung hypoplasia, severe laryngeal hypoplasia, abnormal lobulation of the right lung, right
Respiratory tract defects
oligemic (less blood flow) lung
Abdominal wall defects Omphalocele
Skin abnormalities Capillary hemangioma, aplasia cutis congenital of the scalp

All other medications mentioned carry the risk of congenital malformations at a lower rate compared to valproic acid.

(1) Also called hydrancephaly is absence of cerebral hemispheres and replacement of the vacant parts of the cranium with cerebrospinal fluid.

(2) A congenital disorder characterised by presence of cysts or cavities within the cerebral hemisphere.

th th
(3) Literally meaning ‘smooth brain’ is caused by defective central nervous migration during the 12 and 24 weeks of gestation resulting in lack of development of the brain folds (gyri) and grooves(sulci)

(4) Dandy – Walker syndrome is a congenital malformation of the cerebellum with the key feature being complete absence of the cerebellar vermis.

References

• http://www.medscape.com/viewarticle/724671

• http://www.australianprescriber.com/magazine/31/3/

• http://www.gpnotebook.co.uk/simplepage.cfm?ID=6511

Last updated:
Time spent: QID:131 2023-2-12

960 of 1943
Sarah, 18 years old, is in your office for a visit because she is concerned about her delayed puberty and absence of menses. She has never had a period before. On examination, she
has no pubic or axillary hair, and her breasts are at Tanner I stage. Which one of the following is the most appropriate next step in management?

A. Pelvic ultrasonography.

B. Serum follicular stimulating hormone (FSH) and luteinizing hormone (LH).

C. Bone age assay.

D. Serum thyroid stimulating hormone (TSH).

E. Karyotype.

Incorrect. Correct answer is B


45% answered correctly

Explanation:

Correct Answer Is B

This scenario represents a case a primary amenorrhea as wells as delayed puberty evident by complete lack of breast development and axillary and pubic hair.

The most important step in assessment of primary amenorrhea is always a thorough history and complete physical exam to check if secondary sexual characteristics are present.
Breast development reflects estrogen exposure and is the best indicator that puberty has begun. In fact, the diagnostic approach to primary amenorrhea starts with breast
assessment. Breasts at Tanner III stage or higher are equivalent to presence of secondary sexual characteristics (Tanner III or higher in females older than 13 years). With
developing or developed breasts, the next thing to check is whether a normal uterus is present. This is achieved by physical exam (if possible) and a pelvic ultrasound.

With Tanner I breasts, secondary sexual characteristics are absent. In such cases, investigations start with measurement of FSH either alone or with LH for assessment of
hypothalamus – pituitary – ovary (HPO) axis.

This patient has not started breast development; therefore, the next step in management is measuring the FSH (and LH) for evaluation of HPO axis. TSH (option D) and prolactin are
also routinely assessed as a part of hormonal assay in delayed puberty. Exclusion of pregnancy should always be considered as well.

An algorithmic approach is illustrated in the following diagram:

Pelvic ultrasonography (option A) is considered first choice of investigation in females with primary amenorrhea and developed secondary sexual characteristics.

Bone age assay (option C) is used to establish a diagnosis of constitutional growth delay (CGD). CGD is a diagnosis of exclusion, meaning that all possible causes of delayed
puberty should be excluded by thorough history, physical examination, and investigation. Some conditions to exclude are hormonal deficiencies, occult systemic illness, or
syndromes associated with impaired growth and development. After exclusion of all possible causes of CGD, radiographic study of the left hand and wrist to assess skeletal
maturation is critical in diagnosing constitutional growth delay. Typically, in children with CGD, the bone age begins to lag behind chronologic age during early childhood and is
delayed in adolescence by an average of 2-4 years.

Karyotyping (option E) is considered in patients with syndromic features such as short stature and amenorrhea or where hypergonadotropic hypogonadism (e.g., Turner syndrome)
is suspected in the presence of an elevated FSH above the prepubertal reference range.

TOPIC REVIEW

Primary amenorrhea is defined as the absence of menses at age 15-16 in the presence of normal growth and secondary sexual characteristics (with a breast of at least Tanner II) or
13-14 in the absence of secondary sexual characteristics.

Primary amenorrhea is usually the result of a genetic or anatomical abnormality. In a large case series of primary amenorrhea, the most common etiologies were:

Gonadal dysgenesis including turner syndrome - 43%


Mullerian dysgenesis (absence of vagina, sometimes with absence of uterus) – 15%
Physiological delay of puberty (constitutional delay of puberty, chronic systemic disease, acute illness) – 14%
Polycystic ovarian syndrome (PCOS) – 7%
Isolated gonadotropin-releasing hormone (GnRH) deficiency – 5%
Transverse vaginal septum – 3%
Weight loss / anorexia nervosa / exercise induced – 2%

961 of 1943
Hypopituitarism – 2%
Rare causes - <1% (imperforated hymen, complete androgen insensitivity syndrome, hyperprolactinemia/prolactinoma), other pituitary tumors, congenital adrenal
hyperplasia, hypothyroidism, central nervous system defects, craniopharyngioma, and Cushing’s disease)

Investigations:

According to latest guidelines, indications for investigating a cause of primary amenorrhea include:

An adolescent who has not had menarche by age 15-16 years


Adolescent who has not had menarche and more than 3 years have elapsed since thelarche
An adolescent who has not had a menarche by age 13-14 years and no secondary sexual development
An adolescent who has not had menarche by age 14 years and:

(1)There is a suspicion of an eating disorder or excessive exercise, or


(2) There are signs of hirsutism, or
(3) there is suspicion of genital outflow obstruction

The following tests are considered basic for all patients with primary amenorrhea:

Pregnancy test (to exclude pregnancy prior to first menstruation)


Pelvic ultrasound (complementary to physical exam)
FSH (and LH)
Thyroid stimulating hormone (TSH)
Prolactin

For the exam purposes though, one should know how to prioritize these tests.

In assessment of female patients with primary amenorrhea two critical questions should be answered:

Are secondary sexual characteristics present or absent?

A physical examination will evaluate secondary sexual characteristics such as breast development, axillary and pubic hair, and growth. Breasts are an endogenous assay for
estrogen. Breast development consistent with a Tanner stage of II or greater indicates the presence of estrogen and ovarian function (although it could be insufficient or prematurely
failed). With breasts present, the next step is always checking for the presence or absence of a uterus, its anatomy and possible defects, vagina, and hymen. If there is no breast
development, FHS (and LH) comes first.

Is a uterus absent or present?

Ultrasonography is the best method to reach an answer for this question. Both transvaginal (if the patient not virgin) and transabdominal ultrasonography should be performed to
assess whether mullerian structures are absent or present.

EXAM TIP

The following are characteristic features of some important conditions associated with primary amenorrhea:

Gonadal dysgenesis (including Turner)

1. No or abnormally developed breasts


2. Present uterus (but streak ovaries)
3. Often normally-developed axillary and pubic hair
4. Increased serum FSH

Androgen insensitivity

1. Normally-developed breasts (Tanner II or higher)


2. Absent uterus
3. Absent axillary and pubic hair

Hypothalamic-pituitary failure

1. No or abnormally developed breasts (Tanner stage inconsistent with predicted age)


2. Present uterus
3. Decreased serum FSH

Mullerian dysgenesis

1. Normal breast development


2. Normal axillary and pubic hair
3. Normal hormonal assay (including FSH, LH, prolactin, TSH)
4. Absent uterus (and upper part of vagina)

Transverse vaginal septum/imperforated hymen

1. Normal breast development (Tanner II or higher)


2. Normal axillary and pubic hair
3. Normal hormonal assay (including FHS, LH, prolactin, TSH)
4. Cyclical pelvic pain and/or suprapubic mass

962 of 1943
References

• UpToDate - Evaluation and management of primary amenorrhea

• American Family Physician - Disorders of Puberty: An Approach to Diagnosis and Management

• MSD Manual - Amenorrhea

Last updated:
Time spent: QID:867
2023-2-12

963 of 1943
A 16-year-old girl is brought to your clinic by her mother for evaluation because she has not started menstruation yet. She is a dedicated ballet dancer. On examination, she has a
body mass index (BMI) of 16. Of all secondary sexual characteristics, only breast buds consistent with a Tanner stage II are present. You order serum FSH, LH, TSH and prolactin
levels all of which are in normal range. Which one of the following is the most appropriate next step in management?

A. Pelvic ultrasonography.

B. Refer to an adolescent health center.

C. Reassure that it could be a normal variant of puberty.

D. Perform a Pap smear.

E. Karyotyping.

Correct
45% answered correctly

Explanation:

Correct Answer Is A

The case scenario describes a girl with primary amenorrhea. Primary amenorrhea is defined as the absence of menses by the age of 15-16 years in the presence of normal growth
and secondary sexual characteristics or by the age of 3-14 years in the absence of secondary sexual characteristics. According to this definition, this girl has primary amenorrhea
and should be thoroughly investigated.

Etiologies of primary amenorrhea diverse but generally can be considered as anatomical or hormonal.

In a large case series of primary amenorrhea, the most common etiologies were:

Gonadal dysgenesis including turner syndrome - 43%


Mullerian dysgenesis (absence of vagina, sometimes with absence of uterus) – 15%
Physiological delay of puberty (constitutional delay of puberty, chronic systemic disease, acute illness) – 14%
Polycystic ovarian syndrome (PCOS) – 7%
Isolated gonadotropin-releasing hormone (GnRH) deficiency – 5%
Transverse vaginal septum – 3%
Weight loss / anorexia nervosa / exercise induced – 2%
Hypopituitarism – 2%
Rare causes - <1% (imperforated hymen, complete androgen insensitivity syndrome, hyperprolactinaemia/prolactinoma), other pituitary tumours, congenital adrenal
hyperplasia, hypothyroidism, central nervous system defects, craniopharyngioma, and Cushing’s disease)

There are a few points in the history that can narrow down the diagnosis. Firstly, this girl has Tanner II breast development indicating that she has been exposed to estrogen;
however, at 16 years age, more developed breasts with higher Tanner stages would be expected.

Puberty in girls starts with breast development. Growth spurt then occurs and is followed by axillary and pubic hair development. Menstruation often does not take place until two
years from the onset of puberty.

Another point to consider is the fact that this girl has no pubic or axillary hair. This finding can be either due to arrested or delayed progression of puberty which once has been
started (as evident by breast buds), or incomplete androgen insensitivity in which breast development occurs but the uterus and axillary and pubic hair are absent. In this girl who is
a professional ballet dancer with a BMI of 16, the former is more likely.

Low body weight, excessive physical exercise, and starvation are associated with hypogonadotropic hypogonadism (central hypogonadism). This is a very common finding among
patients with anorexia nervosa or bulimia nervosa, but is also seen in female athletes as a cause of primary or secondary amenorrhea. If it occurs prior to complete puberty, it could
impede puberty from normal progression.

In hypogonadotropic hypogonadism, the characteristic finding is a decreased serum FSH (and LH). Since this patient has normal hormonal assays, an ultrasonography should be
performed for assessment of possible uterine anomalies or those of genital tract, resulting in outflow obstruction, some of which include: mullerian agenesis, transverse vaginal
septum, Asherman’s syndrome, and imperforated hymen

One interesting point in the question is that, in fact, with breasts at Tanner II stage, ultrasound should have been preceded the hormonal assays because, for instance, if this girl was
found on ultrasound to have no uterus, there was no need to measure FSH and other hormones, and the diagnostic pathway then should be continued with karyotyping (option E)
and testosterone assays to investigate complete or incomplete androgen insensitivity as the most likely diagnosis.

Referral to adolescent health centre (option B) may be indicated somewhere down the line once other causes of primary amenorrhea other than those related to excessive exercise,
weight loss and eating disorders are excluded with high certainty. It is not appropriate at this stage when other probable causes of primary amenorrhea are not yet fully investigated.

This girl lags behind normal puberty and cannot be reassured (option C) unless the cause is ensured to be constitutional delay of puberty. Interestingly and unlike the common
belief, constitutional delay of puberty is not that common in females compared to males.

Screening with Pap smear (option D) starts at 25 years of age. It is not indicated for this girl now.

TOPIC REVIEW

In assessment of female patients with primary amenorrhea two critical questions should be answered:

964 of 1943
Are secondary sexual characteristics present or absent?

A physical examination will evaluate secondary sexual characteristics such as breast development, axillary and pubic hair, growth, etc). Breasts are an endogenous assay for
estrogen. Breast development over Tanner II stage indicates the presence of estrogen and ovarian function. With breasts above Tannger II, the next step is always checking for the
presence or absence of a uterus, uterine anomalies, vaginal septum or absence of vagina, imperforated hymen, etc. With no breast development, hormonal assay including FSH, LH,
TSH, and prolactibe comes first.

Is a uterus present or absent?

A step by step approach to primary amenorrhea is shown in the following diagram:

EXAM TIP

Following are characteristic features of some important conditions associated with primary amenorrhea:

Gonadal dysgenesis (including Turner)

1. No or abnormally-developed breasts
2. Present uterus (but streak ovaries)
3. Often normally-developed axillary and pubic hair
4. Increased serum FSH

Androgen insensitivity

1. Normally-developed breasts (Tanner II or higher)


2. Absent uterus
3. Absent axillary and pubic hair

Hypothalamic-pituitary failure

1. No or abnormally-developed breasts (Tanner stage inconsistent with predicted age)


2. Present uterus
3. Decreased serum FSH

Mullerian dysgenesis

1. Normal breast development


2. Normal axillary and pubic hair
3. Normal hormonal assay (including FSH, LH, prolactin, TSH)
4. Absent uterus (and upper part of vagina)

Transverse vaginal septum/imperforated hymen

1. Normal breast development Tanner II or higher)


2. Normal axillary and pubic hair
3. Normal hormonal assay (including FHS, LH, prolactin, TSH)
4. Cyclical pelvic pain and/or suprapubic mass

References

• UpToDate - Evaluation and management of primary amenorrhea

• NSW Health – Amenorrhoea

• ARUP Consult - Primary Amenorrhea Testing

Last updated:
Time spent: QID:868
2023-2-12

965 of 1943
A mother has brought her 16-year-old daughter to your practice for evaluation with the complaint that she has not started her periods yet. On examination, her height is 140 cm. She
has breast buds consistent with stage II Tanner. There is axillary and pubic hair growth appropriate for her age. She gives no history of galactorrhea. Which one of the following is
the most likely cause to her presentation?

A. A normal variant in sexual development.

B. Hypothalamic hypogonadism.

C. Mullerian agenesis.

D. Ovarian dysgenesis.

E. Pituitary tumour.

Incorrect. Correct answer is D


45% answered correctly

Explanation:

Correct Answer Is D

The main complaint presented in the scenario is primary amenorrhea. Primary amenorrhea is defined as the absence of menses at age 15-16 in the presence of normal growth and
secondary sexual characteristics, or 13-14 in the absence of secondary sexual characteristics. According to this definition, this girl has primary amenorrhea and should be
thoroughly investigated.

Etiology of primary amenorrhea is diverse but generally can be considered as anatomical or hormonal.

In a large case series of primary amenorrhea, the most common etiologies were:

Gonadal dysgenesis including turner syndrome - 43%


Mullerian dysgenesis (absence of vagina, sometimes with absence of uterus) – 15%
Physiological delay of puberty (constitutional delay of puberty, chronic systemic disease, acute illness) – 14%
Polycystic ovarian syndrome (PCOS) – 7%
Isolated gonadotropin-releasing hormone (GnRH) deficiency – 5%
Transverse vaginal septum – 3%
Weight loss / anorexia nervosa / exercise induced – 2%
Hypopituitarism – 2%
Rare causes - <1% (imperforated hymen, complete androgen insensitivity syndrome, hyperprolactinaemia/prolactinoma), other pituitary tumors, congenital adrenal
hyperplasia, hypothyroidism, central nervous system defects, craniopharyngioma, and Cushing’s disease)

By far, the gonadal dysgenesis (ovarian dysgenesis) including Turner syndrome is the most common cause of primary amenorrhea in female patients.

Gonadal dysgenesis is caused by complete or partial deletion of one X chromosome with the classic Karyotype being (45,OX). The female may also have a mosaic genotype –
(45,Ox/46,xx).

Short stature and a square appearance is the cardinal common clinical feature of Turner syndrome.

In older adolescents and adults, presenting symptoms of the syndrome usually involves problems of puberty and fertility as well as short stature. Pubic and axillary hair growth
occurs at a normal age, but is not an indicator that puberty will progress normally. Breast development may be absent if ovarian failure occurs before puberty, or partially developed
if it occurs at some time after puberty has begun. Up to 13% of girls with Turner syndrome have spontaneous breast development or menses; some of these are XO/XX mosaics,
with normal gonadotropin responses to luteinizing hormone–releasing hormone (LHRH). Overall, approximately 30% of girls with Turner syndrome have some spontaneous pubertal
development.

Turner syndrome should be considered in individuals with primary or secondary amenorrhea and in adult women with unexplained infertility, particularly when such individuals also
have a short stature.

Considering the fact that gonadal dysgenesis is the most common cause of primary infertility, and more importantly, with the short stature, gonadal (ovarian) dysgenesis is most
likely to underlie the condition. A Tanner II stage breast advocates the diagnosis; however, even more developed breasts do not exclude the diagnosis. Primary amenorrhea and
short stature remain the most important pointers towards ovarian dysgenesis.

(Option A) A 17-year-old amenorrheic girl with a height of 140 is far beyond parameters of normal puberty and cannot be reassured as just having a normal variation in puberty.

(Options B and C) Mullerian agenesis and hypothalamic hypogonadism are not associated with short stature.

(Option E) Pituitary tumors are the cause of primary amenorrhea in less than 0.5% of cases. Short statue is not a characteristic feature.

References

• Medscape - Turner Syndrome

• UpToDate - Clinical Manifestations and diagnosis of Turner syndrome

• UpToDate - Evaluation and management of primary amenorrhea

Last updated:
Time spent: QID:871
2023-2-12

966 of 1943
A 16-year-old girl presents with primary amenorrhea. On examination, she has satisfactory development of breasts and axillary and pubic hair for her age. She is virgin and does not
allow you to perform a vaginal exam for further assessments to exclude an imperforated hymen. Which one of the following would be the most appropriate next step in
management?

A. CT scan of the pelvis.

B. Ultrasound scan of the pelvis.

C. Convince her to a vaginal exam.

D. Refer her to another doctor.

E. Serum follicle stimulating hormone (FSH).

Incorrect. Correct answer is B


45% answered correctly

Explanation:

Correct Answer Is B

Physical exam is an important part in assessment of females with primary amenorrhea. It should include examination for assessment of presence and development of secondary
sexual characteristics, genitalia and clitoris, and a vaginal exam to check for anomalies of the vagina and hymen if feasible. A vaginal exam may reveal an imperforated hymen,
absence of vagina (in vaginal agenesis), or short vagina (in mullerian agenesis).

Ultrasonography of the pelvis is another very important part in assessment of every patient with primary amenorrhea, particularly if breasts are over Tanner II stage. With ultrasound
the following are checked:

Presence or absence of the uterus


Possible mullerian anomalies
Ovaries
Presence of a transverse vaginal septum
Evidence of menstrual flow obstruction

Since this patient does not consent to vaginal examination, pelvic ultrasonography can provide details as a surrogate, and additionally, further information regarding pelvic
structures. Therefore, in this situation, a pelvic ultrasound is the next best in management.

Ideally, both transvaginal and transabdominal sonography should be performed unless the patient’s virginity or refusal precludes the former. It should be noted that if the patient had
consented to vaginal exam, the need for ultrasonography was not eliminated.

NOTE - vaginal examination should be restricted to women who are or have been sexually active.

With satisfactory development of breasts and pubic and axillary hair, it is very likely that the cause of this girl’s amenorrhea is mullerian agenesis. Vaginal transverse septum and
imperforated hymen are also possible; however, with these conditions cyclical pelvic pain and/or discomfort and/or suprapubic mass would be expected.

(Option A) CT scan of the pelvis adds nothing to ultrasound findings at this stage. It is more expensive and unnecessarily poses the patient to radiation exposure.

(Option C) Convincing the patient to the examination as long as ultrasound can provide almost the same efficacy is not appropriate.

(Option D) Referral to another doctor just because the patient does not consent to vagial exam is not an approriate action.

(Option E) Serum FSH is unlikely to provide further clarification. With reassuring development of secondary sexual characteristics, FSH is very likely to be within the normal range.
Measuring FHS might be indicated somewhere down the road but not at this stage.

References

• UpToDate - Evaluation and management of primary amenorrhea

• NSW Health – Amenorrhoea

Last updated:
Time spent: QID:872
2023-2-12

967 of 1943
A 16-year-old ballet dancer girl is brought to your office by her mother because she has not had any periods until now. On examination, the girl has a body mass index (BMI) of 16,
and breasts at Tanner II stage. No axillary or pubic hair is present. Which one of the following should come next in management?

A. Reassurance.

B. Hormonal assays.

C. Abdominal pelvic ultrasound.

D. Combined oral contraceptive pills (COCPs).

E. Referral to adolescent health center.

Incorrect. Correct answer is B


45% answered correctly

Explanation:

Correct Answer Is B

Puberty in girls starts with breast development. Growth spurt then occurs and is followed by axillary and pubic hair development. Menstruation often does not take place until
two years after the onset of puberty.

There are a few points in the history that can narrow down the diagnosis. Firstly, this girl has Tanner II breast at the age of 16 which shows lack of breast development as the first
sign of puberty (Tanner II breasts after the age of 13 means absence of breast development).

Another point to consider is the fact that this girl has no pubic or axillary hair. This finding can be either due to arrested or delayed progression of puberty, or androgen insensitivity
presented with breast development, but absence of a uterus and axillary and pubic hair. In this girl, who is a professional ballet dancer with a BMI of 16, the former is more likely.

In approaching the female patients with primary amenorrhea, the diagnostic approach depends on the presence or absence of secondary sexual characteristics. In patients with
absence of such characteristics, hormonal studies including FSH, LH, TSH, and prolactin is the most appropriate next step in investigations, while in the presence of such
characteristics, a pelvic ultrasound for uterine or menstrual outflow abnormalities come first.

This girl has amenorrhea and absence of secondary sexual characteristics; therefore, hormonal assay is the most appropriate next step in management.

Low body weight, excessive physical exercise, and starvation are associated with hypogonadotropic hypogonadism (central hypogonadism). This is frequently observed in patients
with anorexia nervosa or bulimia nervosa, but also can be seen in female athletes. Hypogonadotropic hypogonadism may result in primary or secondary amenorrhea. If it occurs
during puberty, it could impede puberty from normal progression.

Although a decreased serum FSH will be the expected finding associated with the condition, with breast buds present, the most appropriate next step in management is a pelvic
ultrasound. Along with physical examination, a pelvic ultrasound is always the very first initial assessment of patients with primary amenorrhea who have secondary sexual
characteristics (evident by Tanner II or higher stages of breast development). Ultrasound helps to check for the presence of a uterus, mullerian agenesis, imperforated hymen, etc.
as the potential causes of primary amenorrhea. Ideally, both transvaginal and transabdominal pelvic ultrasound should be performed unless virginity precludes transvaginal
ultrasound.

(Option A) This girl lags behind normal puberty and cannot be reassured unless the cause is ensured to be constitutional delay of puberty.

(Option C) Pelvic ultrasound would be the correct answer if secondary sexual characteristics were present.

(Option D) Combined oral contraceptives (COCs) have no role in diagnosing or treatment of this girl.

(Option E) Referral to adolescent health center may be indicated later once other causes of primary amenorrhea other than those related to excessive exercise, weight loss, and
eating disorders are excluded with high certainty. It is not appropriate at this stage when other probable causes of primary amenorrhea are not yet fully investigated.

References

• American Family Physicians - Amenorrhea: An Approach to Diagnosis and Management

• NSW Health - Amenorrhoea

Last updated:
Time spent: QID:873
2023-2-12

968 of 1943
A 32-year-old woman presents to the emergency department with vaginal bleeding at 28 weeks pregnancy. After prompt investigations, the diagnosis of placental abruption is
made. Of the following options, which is not expected to be caused by the condition?

A. Tense and tender uterus.

B. Blood pressure of 180/110mmHg.

C. Shock out of proportion to the amount of vaginal bleeding.

D. Fetal head mobile above the pelvic brim.

E. Fetal demise.

Incorrect. Correct answer is B


45% answered correctly

Explanation:

Correct Answer Is B

Placental abruption occurs when a normally implanted placenta (not in the lower uterine segment) separates from the uterine wall before delivery of the fetus. Separation can be
partial or complete. The condition is associated with vaginal bleeding of varying extent and a tense tender uterus (option A).

Commonly, the bleeding is overt and external (80% of cases); however, at occasions the bleeding may be concealed with blood accumulating between the uterine wall and the
placenta. This can lead to uterine hematoma and an increased size of the fundal height. Concealed bleeding, if severe, can result in hemorrhagic shock not justified by the amount of
external bleeding (option C). Fetal head can be felt in the pelvic brim or above it (option D), depending on the gestational age. An already engaged head can lose its position if
placental abruption occurs.

Placental abruption is a serious condition capable of putting both the mother and the fetus in jeopardy. In utero fetal demise (option E) is a major concern.

There are several conditions associated with increased risk of placental abruption, with hypertension being one, but placental abruption does not result in hypertension.

References

• http://www.uptodate.com/contents/placental-abrupti

• http://www.aafp.org/afp/2004/1001/p1303.html

Last updated:
Time spent: QID:899
2023-2-12

969 of 1943
A 25-year-old woman, on oral contraception pills, presents to your office because she is concerned about 3 episodes of painless vaginal bleeding in the past week. She has had
regular cervical screening tests with normal results to date, with the last one taken 12 months ago. Which one of the following is the most likely cause of her postcoital bleeding?

A. Chlamydia infection.

B. Cervical cancer.

C. Endometrial cancer.

D. A cervical polyp.

E. Cervical ectropion.

Incorrect. Correct answer is E


45% answered correctly

Explanation:

Correct Answer Is E

Postcoital bleeding can certainly be caused by Chlamydia cervicitis, a cervical polyp or cervical carcinoma. However, the most likely cause in this instance is a cervical ectropion,
where the single layer of columnar epithelium of the endocervix has extended onto the ectrocervix and is exposed to trauma during coitus.

Cervicitis and ectropion are the two most common causes of postcoital bleeding in women younger than 30 years. Although Chlamydia infection (option A) and cervicitis can be a
cause as well, absence of other symptoms of Chlamydia infection such as discharge makes such diagnosis less likely than an ectropion cervix.

Cervical polyps (option D) are more common in women of reproductive age, especially those over 30-40 years.

Cervical cancer (option B) and endometrial cancer (option C) are far less likely at this age. Cervical intraepithelial neoplasia does not cause postcoital bleeding per se. However,
such lesions often reside in ectropion and can bleed with contact during sexual intercourse.

Other possible causes are pregnancy and its complications such as abortion, HRT, and hormonal contraception.

TOPIC REVIEW

The following table outlines the age-specific common causes of irregular uterine bleeding:

Relation of age to common causes of irregular uterine bleeding


AGE GROUP
15-20 20-30 30-45 45-55 55+
STIs (especially Chlamydia cervicitis) HRT

Cervical ectropion Endometrial cancer


Endometrial polyps

Endometrial hyperplasia

Uterine fibroids
Intrauterine device Atrophic vaginitis
Pregnancy and complications; miscarriage/ectopic pregnancy

Contraceptive steroids, especially progestogen

Endometriosis
Trauma / surgery

References

• Cancer Australia - Abnormal vaginal bleeding in pre- and peri-menopausal women

• AMC Handbook of Multiple Choice Questions – page 530

Last updated:
Time spent: QID:910
2023-2-12

970 of 1943
An 18-year-old girl presents to your office with painful menses for the past 12 months. She has been prescribed NSAIDs initially and then OCPs after the NSAIDs failed to control her
symptoms. OCP has been unable to control the painful menses as well. Which one of the following would be the most appropriate next step in management?

A. Transabdominal ultrasound.

B. Transvaginal ultrasound.

C. CT scan of the pelvis and abdomen.

D. Dilation and curettage.

E. Laparoscopy.

Incorrect. Correct answer is B


45% answered correctly

Explanation:

Correct Answer Is B

The main complaint in this patient is with painful periods, also termed dysmenorrhea.

Dysmenorrhea can be divided into two broad categories of primary and secondary. Primary dysmenorrhea is defined as recurrent, crampy pain occurring with menses in the
absence of identifiable pelvic pathology. Secondary dysmenorrhea is menstrual pain associated with underlying pelvic pathology such as endometriosis.

Primary dysmenorrhea usually begins in adolescence after the establishment of ovulatory cycles. Primary dysmenorrhea is caused by myometrial activity and consequent uterine
ischemia. This myometrial activity is modulated and augmented by prostaglandin synthesis; therefore, non-steroidal anti-inflammatory drugs (NSAIDs), by inhibition of prostaglandin
synthesis, are the first-line treatment option.

Oral contraceptive pills (OCPs) are used as second-line if NSAIDs fail. OCPs can also be used as the treatment of choice (first-line) if contraception is desired as well. OCPs prevent
menstrual pain by suppressing ovulation and decreasing uterine prostaglandin levels. An additional mechanism is the reduction of menstrual flow after several months of use.

If treatment with either of these medications fails after 2 or 3 menstrual cycles, a course of treatment with the other modality should be used. Treatment with both hormonal
contraceptives and NSAIDs may be effective in women who do not respond to either drug alone.

Adolescents who fail to respond to first- or second-line treatment, have recurrent symptoms, or have worsening symptoms should be re-evaluated for the possible causes of
secondary dysmenorrhea such as endometriosis, uterine leiomyomas, polyps, or other pelvic pathologies.

In this patient, treatment has failed despite use of both NSAIDs and OCP; therefore, secondary dysmenorrhea should be considered and investigated. When pelvic pathology is
suspected, abdominal and transvaginal ultrasonography should be used as first-line investigation. They are inexpensive, effective, and readily available. Transvaginal ultrasound is
more accurate and the preferred option if possible.

(Option A) Transabdominal ultrasound is inferior to transvaginal ultrasound for evaluation of dysmenorrhea; but it can be used instead of transvaginal ultrasound if transvaginal is
not possible e.g., in virgin girls.

(Option C) CT scan and MRI are not routinely used for assessment of secondary dysmenorrhea.

(Options D and E) Dilation and curettage or laparoscopy might be indicated at some stage after initial evaluation points towards a specific pathology such as endometriosis or
endometrial problems, but not as an initial step.

References

• MedScape - Dysmenorrhea

• UpToDate - Primary dysmenorrhea in adolescents

• AAFP - Diagnosis and Initial Management of Dysmenorrhea

Last updated:
Time spent: QID:930
2023-2-12

971 of 1943
A 34-year-old primigravida woman presents to the emergency department at 36 weeks gestation after she passed approximately 200ml of clotted blood per vagina at home 45
minutes ago. Currently, there is no active vaginal bleeding. On examination, she has a pulse rate of 120 bpm and blood pressure of 90/60 mmHg. Abdominal examination reveals a
very tense uterus and moderate tenderness on palpation. Fetal heart sound is audible at a rate of 140 bpm. Which one of the following is the most appropriate next step in
management?

A. Intravenous fluids.

B. Ultrasound.

C. Cardiotocography (CTG).

D. Induction of labor.

E. Immediate cesarean section.

Correct
45% answered correctly

Explanation:

Correct Answer Is A

The clinical picture of a tense and tender uterus in the presence of vaginal bleeding is suggestive of placental abruption as the most likely diagnosis. The important clue is
tachycardia and hypotension unproportionate to the amount of vaginal bleeding indicating that the bleeding is concealed. In concealed bleeding blood accumulates between the
placenta and the uterine wall.

In the presence of hypotension and tachycardia, fluid resuscitation with intravenous isotonic fluids is the most appropriate next step in management.

Blood typing and cross match should also be performed and packed cells reserved in case of severe bleeding requiring blood transfusion.

An ultrasound scan (option B) is then indicated for assessment of the placental and fetal status. Continuous CTG (option C) should be performed for fetal monitoring. Labor
induction (option D) or cesarean section (option E) are indicated if there is maternal or fetal jeopardy.

References

• http://www.sahealth.sa.gov.au/wps/wcm/connect/b1c6

• http://3centres.com.au/guidelines/complications-in

• http://www.kemh.health.wa.gov.au/development/manua

Last updated:
Time spent: QID:960
2023-2-12

972 of 1943
A 29-year-old woman with history of schizophrenia for the past nine years presents to your practice, accompanied by her husband, because of amenorrhea of 2 months
duration. She is currently stable on clozapine. Which one the following is most important step in management?

A. Ultrasound.

B. Measurement of serum follicle stimulating hormone (FSH).

C. Measurement of serum luteinizing hormone (LH).

D. Urine pregnancy test.

E. Full blood count (FBC).

Incorrect. Correct answer is D


45% answered correctly

Explanation:

Correct Answer Is D

Universally, the most common cause of secondary amenorrhea is pregnancy. Hence, in every woman of reproductive age presenting with amenorrhea, the first thing to check is a
urine pregnancy test.

An ultrasound or measuring FSH and LH may be used later (if indicated) for assessment of the condition, once pregnancy has been excluded.

FBC is used to monitor adverse outcomes of clozapine such as neutropenia or agranulocytosis. It is not indicated for evaluation of amenorrhea.

References

• NSW Health – Amenorrhoea

Last updated:
Time spent: QID:968
2023-2-12

973 of 1943
A 20-year-old woman is planning to conceive and has presented for your advice. She has the history of juvenile myoclonic epilepsy, and has been stable on sodium valproate. Despite your full explanations about the risks of the teratogenicity of
antiepileptic drugs, she insists that she wants to become pregnant and asks you to show her a way. Which one of the following would be the most appropriate advice?

A. Continue sodium valproate.

B. Discontinue sodium valproate.

C. Switch to carbamazepine.

D. Switch to lamotrigine.

E. Switch to phenytoin.

Correct
45% answered correctly

Explanation:

Correct Answer Is A

When approaching a woman, who wishes to become pregnant while on antiepileptic drugs, two major groups of epilepsies should be distinguished because they typically respond differently to different drugs.

Partial epilepsies respond to most antiepileptic drugs, but for idiopathic generalized epilepsies, especially in juvenile myoclonic epilepsy, seizure can be controlled with a reasonably low dose of sodium valproate.

Although lamotrigine may be helpful, it is not as effective as sodium valproate, and even sometimes worsens the myoclonic seizures of juvenile myoclonic epilepsy. Therefore, substitution of sodium valproate with lamotrigine (option D) is not the
right advice.

Topiramate and levetiracetam may be effective in idiopathic generalized epilepsy, while carbamazepine (option C), phenytoin (option E) and gabapentin may worsen some seizure types, especially myoclonic and absence seizures. For some women
with idiopathic generalized epilepsies, there may be no effective alternative to sodium valproate. Cessation of sodium valproate (option A) is associated with recurrence of the generalized seizures, especially juvenile myoclonic epilepsy; therefore, it
is not advisable to discontinue the drug.

On the other hand, sodium valproate has the highest teratogenicity potential among antiepileptic drugs. The Australian Pregnancy Register has reported the risk to be as high as 16% for the first trimester. Sodium valproate should therefore be
avoided in women of reproductive age. If a patient is willing to become pregnant, she should be fully informed of the risk of teratogenicity and the decision is left to her. If she decides to accept the risks, and pregnancy is unavoidable, the lowest
effective dose should be used.

If sodium valproate dose has been lowered to a minimum during pregnancy to reduce teratogenesis, the prepartum effective dose may need to be re-established before the onset of labor. This is a time of increased seizure risk, especially in patients
with idiopathic generalized epilepsy who are very sensitive to sleep deprivation. Breastfeeding is safe during valproate therapy.

References

• Australian Prescriber - Antiepileptic drugs in pregnancy and lactation

Last updated:
Time spent: QID:132 2023-2-12

974 of 1943
A pregnant elementary school teacher, aged 24 years, presents to your GP office after she found that one of the students in her class has been recently diagnosed with rubella by his
GP; the diagnosis however was not confirmed by serologic studies. Her last vaccination against rubella was when she was 12 years. Which one of the following would be the next
best step in management?

A. No action is required; reassure her.

B. Check rubella serology.

C. Confirm the diagnosis of rubella in the sick child with rubella serology testing.

D. Give her a booster dose of MMR vaccine.

E. Advise her to terminate the pregnancy.

Correct
45% answered correctly

Explanation:

Correct Answer Is B

All pregnant women with contact with a person, who has confirmed diagnosis of rubella infection or is suspicious of having the disease, should be offered serologic testing (both
IgM and IgG) for rubella as the most appropriate next step in management. However, if the patient had been screened for rubella IgG in the first trimester and had a positive IgG with
titers >10 IU/ml, no further serologic testing would have been indicated.

(Option A) Taking no action is incorrect as serologic testing is indicated in all pregnant patients after contact to a rubella or rubella-like infection, unless the woman has been
documented to be immune in the current pregnancy using IgG titers as mentioned earlier.

(Option C) There is no need for serologic testing in the index case (the child in this scenario).

(Option D) Rubella-containing vaccines (MMR, MMRV) are live attenuated vaccines and contraindicated throughout pregnancy.

Advising the patient to terminate the pregnancy is recommended if there is a with definite diagnosis of rubella infection (confirmed by serology) in the first trimester, as it is
associate with fetal anomalies in almost 85% of cases.

TOPIC REVIEW

Rubella, also called German measles, is usually a mild infectious disease in children and adults. It is clinically difficult to diagnose due to transient clinical features that are also
common to some other viral infections.

Rubella has an incubation period of 14-23 days. The infectivity period starts from one week before until 4 days after onset of rash.

Rubella is asymptomatic in 25 to 50 % of cases. If there are symptms, they may include:

Low grade fever


Transient erythematous rash
Lymphadenopathy involving post-auricular and sub-occipital nodes
Occasionally arthritis and arthralgia (commonly observed in women of child-bearing age)
Rarely neurological disorders and thrombocytopenia
Rash - the rash characteristically begins on the face and spreads to the trunk and extremities. It will usually resolve within three days in the same order in which it
appeared (face first and then body)

During pregnancy, maternal viremia may occur 5 to 7 days after exposure with spread of the virus throughout the body as well as transplacental infection of the fetus.

Vertical transmission (transplacental) from mother to fetus can only occur during viremia of a primary infection. The effect of maternal infection on the fetus depends on the
gestational age:

<8 weeks: up to 85% of fetuses will be infected and all will have clinical manifestations of the congenital rubella syndrome.
8≤ but <12 weeks: 50-80% of fetuses will be infected and of those 65-85% will be clinically affected.
13-16 weeks: 30% of the fetuses will be infected and of those, 1/3 will have sensorineural deafness.
16-19 weeks: 10% of the fetuses will be infected. Clinical features are rare, yet deafness can be a possibility.
>19 weeks: there is no apparent risk.

NOTE – reinfection is associated with far less risk of fetal infection (~5%.)

Abnormalities associated with congenital rubella syndrome include:

Central nervous system dysfunction (10-25%, intellectual impairment, developmental delay, microcephaly)
Eye abnormalities (10-25%, cataracts, retinopathy, glaucoma, strabismus, micropthalmos)
Sensorineural deafness (60-75%)
Cardiac abnormalities (10-20%, PDA, PA stenosis)
Intrauterine growth restriction, short stature
Inflammatory lesions of the brain, liver, lungs and bone marrow

All pregnant women, who have contact with rubella or clinical features consistent with rubella – like illness should be screened for the presence of rising antibody titre (IgG) and
rubella specific IgM, regardless of previous history of vaccination or infection. Interpretation of the results and corresponding action is as follows:

975 of 1943
IgM IgG Interpretation Recommendation
Serology tests should be repeated. If positive IgM is confirmed or
+ + Possible recent infection of reinfection, depending on history
there is a rise in IgG, the infection is confirmed.
If the test has been performed < 3 weeks since contact or < 7days
- - The patient is susceptible since onset of illness, it should be repeated. Seroconversion or
positive IgM indicates infection
Repeat the test for confirmation. If positive IgM is confirmed or
+ - Recent infection is possible
seroconversion occurs, the infection is confirmed.
- + Immunity to rubella or past infection No further action is needed

If maternal infection is confirmed, antenatal testing is recommended at least 6 weeks after known maternal infection. Rubella PCR, rubella culture and fetal IgM can be performed
following chorionic villus sampling (CVS) / amniocentesis or cordocentesis; however, due to the very high risk of fetal infection and consequent fetal anomalies, if maternal infection
occurs in the first trimester, termination of pregnancy should be recommended.

NOTE - In all pregnant women, serologic testing for rubella (IgG only) should be offered and performed in the first antenatal visit. With titers ≥10 IU/ml, risk of reinfection is minimal.
However, if the titers are ≤15, vaccination after delivery should be performed.

References

• https://www.sahealth.sa.gov.au/wps/wcm/connect/d81

• https://www.asid.net.au/documents/item/368

Last updated:
Time spent: QID:983
2023-2-12

976 of 1943
A kindergarten teacher presents to your clinic at 10 weeks pregnancy. Recently, she has found that one of the children in her class has rubella infection. Which one of the following is
the most appropriate next step in management?

A. Exclude the sick child and reassure her.

B. Terminate the pregnancy before 13 weeks.

C. Check anti-rubella IgM and IgG levels.

D. Tell her to come back if she becomes symptomatic.

E. Give her MMR vaccine.

Incorrect. Correct answer is C


45% answered correctly

Explanation:

Correct Answer Is C

All pregnant women should be offered serologic testing for rubella as the most appropriate next step in management if there is exposure to rubella infection through contact with an
infectious case. This is true for all pregnant women regardless of the past history of immunization or infection with rubella. However, if the patient was screened for rubella IgG in
the first trimester and had a positive IgG with titers >10 IU/ml no further serologic testing would have been indicated.

(Option A) Exclusion of the child until full recovery or at least four days after the onset of the rash is recommended, but this will not affect the management plan for the woman. She
is at risk of contracting rubella and its detrimental impacts on her fetus, and cannot be reassured until investigations exclude rubella infection.

(Option B) Termination of the pregnancy is recommended for pregnant woman with established diagnosis of rubella within the first trimester of pregnancy, confirmed with serology
studies. Rubella infection in the first trimester is associated with severe fetal anomalies in 85% of cases.

(Option D) Telling her to come back if she is symptomatic is inappropriate. She should undergo serologic studies now.

(Option E) Rubella-containing vaccines (MMR, MMRV) are live attenuated vaccines and are contraindicated during pregnancy.

TOPIC REVIEW

Rubella, also called German measles, is usually a mild infectious disease in children and adults. It is clinically difficult to diagnose due to transient clinical features that are also
common to some other viral infections.

Rubella has an incubation period of 14-23 days. The infectivity period starts from one week before until 4 days after onset of rash.

Rubella is asymptomatic in 25 to 50 % of cases. If there are symptms, they may include:

Low grade fever


Transient erythematous rash
Lymphadenopathy involving post-auricular and sub-occipital nodes
Occasionally arthritis and arthralgia (commonly observed in women of child-bearing age)
Rarely neurological disorders and thrombocytopenia
Rash - the rash characteristically begins on the face and spreads to the trunk and extremities. It will usually resolve within three days in the same order in which it
appeared (face first and then body)

During pregnancy, maternal viremia may occur 5 to 7 days after exposure with spread of the virus throughout the body as well as transplacental infection of the fetus.

Vertical transmission (transplacental) from mother to fetus can only occur during viremia of a primary infection. The effect of maternal infection on the fetus depends on the
gestational age:

<8 weeks: up to 85% of fetuses will be infected and all will have clinical manifestations of the congenital rubella syndrome.
8≤ but <12 weeks: 50-80% of fetuses will be infected and of those 65-85% will be clinically affected.
13-16 weeks: 30% of the fetuses will be infected and of those, 1/3 will have sensorineural deafness.
16-19 weeks: 10% of the fetuses will be infected. Clinical features are rare, yet deafness can be a possibility.
>19 weeks: there is no apparent risk.

NOTE – reinfection is associated with far less risk of fetal infection (~5%.)

Abnormalities associated with congenital rubella syndrome include:

Central nervous system dysfunction (10-25%, intellectual impairment, developmental delay, microcephaly)
Eye abnormalities (10-25%, cataracts, retinopathy, glaucoma, strabismus, micropthalmos)
Sensorineural deafness (60-75%)
Cardiac abnormalities (10-20%, PDA, PA stenosis)
Intrauterine growth restriction, short stature
Inflammatory lesions of the brain, liver, lungs and bone marrow

All pregnant wome,n who have contact with rubella or clinical features consistent with rubella – like illness should be screened for the presence of rising antibody titre (IgG) and
rubella specific IgM, regardless of previous history of vaccination or infection. Interpretation of the results and corresponding action is as follows:

977 of 1943
IgM IgG Interpretation Recommendation
Serology tests should be repeated. If positive IgM is confirmed or
+ + Possible recent infection of reinfection, depending on history
there is a rise in IgG, the infection is confirmed.
If the test has been performed < 3 weeks since contact or < 7days
- - The patient is susceptible since onset of illness, it should be repeated. Seroconversion or
positive IgM indicates infection
Repeat the test for confirmation. If positive IgM is confirmed or
+ - Recent infection is possible
seroconversion occurs, the infection is confirmed.
- + Immunity to rubella or past infection No further action is needed

If maternal infection is confirmed, antenatal testing is recommended at least 6 weeks after known maternal infection. Rubella PCR, rubella culture and fetal IgM can be performed
following chorionic villus sampling (CVS) / amniocentesis or cordocentesis; however, due to the very high risk of fetal infection and consequent fetal anomalies, if maternal infection
occurs in the first trimester, termination of pregnancy should be recommended.

NOTE - In all pregnant women, serologic testing for rubella (IgG only) should be offered and performed in the first antenatal visit. With titers ≥10 IU/ml, risk of reinfection is minimal.
However, if the titers are ≤15, vaccination after delivery should be performed.

References

• https://www.sahealth.sa.gov.au/wps/wcm/connect/d81

• https://www.asid.net.au/documents/item/368

• http://www.sahealth.sa.gov.au/wps/wcm/connect/publ

Last updated:
Time spent: QID:984
2023-2-12

978 of 1943
A 27-year-old woman presents to your practice at 39 weeks pregnancy with complaint of intermittent watery vaginal discharge for the past 20 hours. Speculum examination, shows
pooling of clear fluid in the posterior vaginal fornix. The cervical os is closed and there are no uterine contractions. Which one of the following is the most important next step in
management?

A. Start antibiotics.

B. Transfer her to a tertiary hospital.

C. Tocolysis.

D. Intramuscular betamethasone.

E. Admit and observer for labor to start.

Correct
45% answered correctly

Explanation:

Correct Answer Is A

The scenario is typical for rupture of membranes (ROM) after 37 weeks (term or near term) and can be a harbinger of labor. However, if labor does not start within 4 hours after
ROM, the term premature (prelabor) rupture of membranes (PROM) is applied to differentiate the condition from ROM, as PROM can be considered a deviation from normal process,
accompanied by maternal or fetal complications.

A different term is used if the rupture occurs before 37 weeks gestation – preterm premature rupture of membrane (PPROM). Approach to PROM (≥37 weeks) and PPROM (<37
weeks) are different because with the former, there are concerns of unfavorable fetal outcomes due to prematurity.

The classic presentation of rupture of the membranes (ROM), regardless of the gestational age, is with a sudden gush of watery fluid per vagina; however, many women describe
continuous or intermittent leakage of fluid or the sensation of wetness within the vagina or on the perineum. Presence of liquor flow from the cervical os or pooling in the posterior
vaginal fornix is pathognomonic.

PROM occurs in 6-19 % of women at term. Even in the presence of an unfavorable cervix, many women will enter labor within 12 hours.

Fifty percent of women with PROM women will be in labor in12 hours, 86 % within 24 hours, and 94 % will be established in labor within 48- 95 hours. Six percent will not establish in
labor within 96 hours of PROM.

The risks of PROM at term relate to maternal and neonatal infection, prolapsed cord and fetal compromise resulting in operative delivery or low 5-minute Apgar score.

Chorioamnionitis is a feared feature of PROM of over 18 hours. The incidence of chorioamnionitis in women at term with PROM is 6-10 % and occurs in up to 40 % of women with
membrane rupture > 24 hours.

The most important risk factors for development of chorioamnionitis include:

Increasing numbers of digital vaginal examinations


Longer duration of active labor
Meconium staining of the amniotic fluid

To prevent chorioamnionitis, it is recommended that all women with PROM more than 18 hours be started on parenteral antibiotic. This woman has presented with established
diagnosis of PROM. Since 19 hours has elapsed, the most important next step in management is starting of intravenous antibiotics as recommended above.

The patient then can be counselled regarding expectant versus active management. Active management includes induction of labor (IOL), whereas in expectant management, the
patient is admitted in the Labor and Maternity unit until labor begins spontaneously.

(Option B) Transferring the patient to a tertiary hospital is recommended for women with PROM before 37 weeks if labor is not imminent and prematurity is a concern, in which case
optimal care for the premature baby in a neonatal intensive care unit (NICU) will be of paramount importance.

(Option C) Tocolysis is not indicated because the fetus is mature. If labor starts it should not be stopped.

(Option D) At 39 weeks the fetal lung maturity has already has completed; therefore, corticosteroids are not indicated.

(Option E) Admission and observation for the labor to begin and progress may be considered if the patient prefers expectant management and there are no contraindications. This,
however, should not delay antibiotic coverage for GBS, when the PROM has lasted more than 18 hours. IOB of labor should not be delayed beyond 24 hours.

NOTE – In patients with positive undetermined GBS status, antibiotic coverage against GBS is always indicated regardless of the duration of PROM and should be administered as
soon as possible.

References

• https://www.ranzcog.edu.au/Statements-Guidelines/Obstetrics/Preterm-Prelabour-Rupture-of-Membranes

Last updated:
Time spent: QID:986
2023-2-12

979 of 1943
A 24-year-old woman presents at 39 weeks gestationy with complaint of intermittent watery vaginal discharge, which started last night after she had sex with her husband. On
speculum examination, pooling of straw-colored fluid in the posterior vaginal fornix is noted. The cervical os is closed. Furthermore, there is leak of liquor from cervical os when she
is asked to cough or strain. Which one of the following best explains these clinical findings?

A. Retained semen from the last night sex.

B. Vaginal infection.

C. Premature rupture of membranes (PROM).

D. Urine leakage.

E. Cervical insufficiency.

Incorrect. Correct answer is C


45% answered correctly

Explanation:

Correct Answer Is C

The presentation is classic for premature rupture of membranes (PROM), probably caused by trauma during intercourse.

By definition, premature rupture of membranes (PROM) is defined as rupture of embryonic membranes before the labor regardless of the age of pregnancy. If PROM occurs before
37 weeks gestation, it is termed as premature PROM (PPROM).

The classic presentation of rupture of the membranes (ROM), regardless of gestational age, is with a sudden gush of watery fluid per vagina; however, many women, such as in this
scenario may complain of continuous or intermittent leakage of fluid or the sensation of wetness within the vagina or on the perineum. Presence of liquor flow from the cervical os
or pooling in the posterior vaginal fornix is pathognomonic.

If there is doubt, further tests can be conducted to clinch the diagnosis:

Nitrazine test – the normal vaginal pH is between 4.5 and 6.0, whereas amniotic fluid is more alkaline, with a pH of 7.1 to 7.3. Nitrazine paper will turn blue when the pH is above 6.0;
however, the presence of contaminating substances such as blood, semen, alkaline antiseptics can also cause nitrazine paper to turn blue, giving a a false-positive result. Bacterial
vaginosis alkalinizes the vagina as well.

Amnisure test - a non-invasve and approximately 99% accurate for definite diagnosis of PROM. It is based on immunoassay, can be done at any gestational age and does not need a
speculum exam.

NOTE - The above tests are currently used in practice in Australia.

Ferning on microscopy – using a swab, fluid is obtained from the posterior fornix or vaginal sidewalls. Once the fluid is dried on the slide, the slide is checked for ferning
(arborization) under low-power microscope. The presence of ferning indicates PROM. It is noteworthy that vaginal blood can obscure presence of ferns, and that cervical mucus can
result in false-positive result if the cervical os has been swabbed. Ferning test is no more performed in Australia.

NOTE – ultrasonography is not used merely for diagnosis of PROM; however, in rare cases when the diagnosis cannot be certain using history, physical exam, and mentioned tests
ultrasonography may help. An example is when the patient’s history suggests PROM, but physical findings findings are not consistent with the diagnosis. Ultrasonography, however,
is a part of investigation plan for assessment of fetal well-being, the position of the fetus, placental location, estimated fetal weight and presence of any anomalies in PROM and
PPROM.

(Option A) Retained semen will have a different appearance and will not result in findings in the clinical scenario.

(Option B) Infections will be associated with characteristic features including purulent cervical discharge, malodorous vaginal discharge, etc. Pooling of clear fluid in the posterior
fornix is pathognomonic for ROM.

(Option C) Urine leakage is common during the pregnancy, but it is inconsistent with such a clinical scenario.

(Option E) Although cervical insufficiency is associated with increased vaginal discharge, absence of other findings on speculum exam such as cervical dilation, and bulging
membranes makes this diagnosis unlikely.

References

• https://www.ranzcog.edu.au/Statements-Guidelines/Obstetrics/Preterm-Prelabour-Rupture-of-Membranes

Last updated:
Time spent: QID:987
2023-2-12

980 of 1943
In which one of the following women groups, endometrial hyperplasia is most likely to be found?

A. An ovulating woman.

B. An obese diabetic woman.

C. A woman on cyclic combined oral contraceptive pills.

D. A woman on Depo-Provera® for treatment of endometriosis.

E. A woman with an intrauterine device.

Incorrect. Correct answer is B


45% answered correctly

Explanation:

Correct Answer Is B

Endometrial hyperplasia is characterized by proliferation of endometrial glands. The condition may progress to or coexist with endometrial carcinoma. Endometrial hyperplasia
virtually always results from chronic exposure of endometrial tissue to estrogen (sometimes in excess amounts) unopposed by the counterbalancing effects of progesterone.

Excess estrogen exposure can be exogenous or endogenous. Exogenous estrogen exposure includes postmenopausal estrogen therapy and tamoxifen, while endogenous exposure
may result from obesity anovulatory menstrual cycle, or estrogen-secreting tumors.

Of the options, diabetes and obesity are associated with increased risk of endometrial hyperplasia. Diabetes mellitus type 2 is associated with increased levels of insulin-like growth
factor (IGF). IGF can cause endometrial proliferation. Increased body mass index (BMI) causes excessive peripheral conversion of androgens to estrogen in adipose tissue.

NOTE - Risk factors for endometrial hyperplasia are essentially the same.

Risk factors for endometrial cancer/hyperplasia and their relative risk are summarized in the following table:

Risk factor Relative Risk (%)


Unopposed estrogen therapy 2-10
Increasing age 1.4 (50-70 years)
Late menopause (>55 years) 2
Nulliparity 2
Chronic anovulation (e.g. in PCOS) 3
Obesity 2-4
Diabetes mellitus 2
Tamoxifen therapy 2
Lynch syndrome (hereditary nonpolyposis colorectal cancer) 22-50% (lifetime risk)
BRCA1/BRCA2 gene mutation 20%
Early menarche Not specified
Estrogen secreting tumor Not specified
Family history of endometrial, ovarian, breast, or colon cancer Not specified

(Option A) Ovulating women has cyclical exposure to progesterone. This leads to reduces risk of endometrial hyperplasia. For the same reason, women on cyclical combined oral
contraceptive are unlikely to be at increased risk of endometrial hyperplasia.

(Option C) Combined oral contraceptive pills (COCs) contain progesterone that counterbalances the effects of estrogen. Women on COCs do not have an increased risk of
endometrial hyperplasia/cancer.

(Option D) Depo-Provera® is injectable progesterone. Progesterone counterbalances the effect of estrogen on endometrial hyperplasia. It is a protective factor rather than risk
factor.

​(Option E) Intrauterine devices have not been shown to be associated with increased risk of endometrial hyperplasia.

References

• UpToDate - Classification and diagnosis of endometrial hyperplasia

• RCOG - Management of Endometrial Hyperplasia

Last updated:
Time spent: QID:998
2023-2-12

981 of 1943
Which one of the following is the most likely finding in a pregnant woman with placental abruption?

A. Decreased fetal movements.

B. Preterm labor.

C. Vaginal bleeding.

D. Uterine tenderness.

E. Shock.

Incorrect. Correct answer is C


45% answered correctly

Explanation:

Correct Answer Is C

Placental abruption is defined as the premature separation of the placenta from the uterus and presents with bleeding, uterine contractions, and fetal distress. Placental abruption is
a significant cause of third-trimester bleeding and is associated with fetal and maternal mortality and morbidity. The condition should be considered as a differential diagnoses in all
pregnant women with vaginal bleeding in the second half of the pregnancy.

Vaginal bleeding is the most common presenting symptom reported by 80% of women with placental abruption. In 20% of women with placental abruption, vaginal bleeding is
concealed; therefore, absence of vaginal bleeding does not exclude placental abruption.

Frequency of symptoms in women with placental abruption is as follows:

Vaginal bleeding – 80%


Abdominal or back pain and uterine tenderness – 70%
Fetal distress – 60%
Abnormal uterine contractions (e.g. hypertonic, high frequency) – 35%
Idiopathic premature labor – 25%
Fetal death – 15%

Exam findings may include:

Vaginal bleeding
Uterine contractions and/or tenderness
Shock
Absence of fetal heart sounds
Increased fundal height do to an expanding hematoma

Shock is seen in class 3 placental abruption that represents 24% of all cases.

TOPIC REVIEW

Placental abruption can be classified clinically, based on the extent of separation (partial versus complete) or location of the separation (marginal versus central). Clinical
classification of placental abruption, the prevalence of each class and its clinical features of each class is as follows:

Class 0 – asymptomatic

Presentation

A diagnosis of class 0 is made retrospectively by finding an organized blood clot or a depressed area on a delivered placenta.

Class 1 – Mild (represents approximately 48% of all cases)

Presentation

No vaginal bleeding to mild vaginal bleeding


Slightly tender uterus
Normal maternal blood pressure and heart rate
Absence of coagulopathy
No fetal distress

Class 2 – moderate (represents approximately 27% of all cases)

Presentation

No vaginal bleeding to moderate vaginal bleeding


Moderate to severe uterine tenderness with possible tetanic contractions
Maternal tachycardia with orthostatic changes in blood pressure and heart rate
Fetal distress
Hypofibrinogenemia (i.e. 50-250 mg/dL)
( )
982 of 1943
Class 3 - Severe (represents approximately 24% of all cases)

Presentation

No vaginal bleeding to heavy vaginal bleeding


Very painful tetanic uterus
Maternal shock
Hypofibrinogenemia (ie, < 150 mg/dL)
Coagulopathy
Fetal death

References

• Medscape - Abruptio Placentae Clinical Presentation

Last updated:
Time spent: QID:1019
2023-2-12

983 of 1943
A 28-year-old 10-week pregnant woman presents 24 hours after eruption of a rash, characteristic of measles. The history and exam findings establish the diagnosis with certainty.
Which one of the following is the most appropriate next step in management?

A. Give MMR vaccine.

B. Administer immunoglobulin.

C. Contact tracing.

D. Check her antibody status.

E. No action is required.

Incorrect. Correct answer is C


45% answered correctly

Explanation:

Correct Answer Is C

The measles (rubeola) virus is a single-stranded RNA virus of the family Paramyxoviridae.

Measles is highly infectious. The main route of transmission is by respiratory airborne droplets; however, it can rarely be passed on to others by means of articles soiled with
respiratory droplets.

Measles has an incubation period of 10-14 days.

Initially, the illness presents with the following:

Fever
Malaise
Cough
Coryza (inflammation of the mucous membranes of the nose)
Conjunctivitis
Koplik’s spots (white spots, each surrounded by a red ring, found on the buccal mucosa)

Two to 4 days after the initial symptoms, a maculopapular rash develops. The rash typically starts from face and upper neck and later spreads to involve the whole body. The
infectivity periods starts 2 days before the onset of the rash and continues for 4 days after the eruption.

Measles is often a severe disease. Complication may occur, and include otitis media in 7% and bronchopneumonia in 6% of immunocompetent patients. Acute encephalitis is a rare
complication occurring in between 2 and 10 per 10,000 reported cases with measles in the general population, with an associated mortality rate of 10 -15 %. Around 15-40 % of
survivors will have permanent brain damage.

In pregnant women measles can be associated with increased risk of the following:

Preterm labor
Spontaneous abortion
Fetal / neonatal loss
Maternal mortality

This woman has established diagnosis of measles and is already at increased risk of above-mentioned complications. There is no active treatment for her except conservative
management; however, since measles is a ‘notifiable’ disease, notification should be made as soon as possible as the most appropriate next step in management.

(Option A) MMR is vaccine has no role in management of established measles. Furthermore, MMR is a live attenuated vaccine and is contraindicated throughout the pregnancy.

(Option B) Normal human immunoglobulin (NHIG) is used for prophylaxis in contacts of an index case with measles in whom MMR vaccine is contraindicated, e.g., pregnant
woman.

(Option D) Checking antibody status in this patient with established measles is of no use, and will not change the management plan.

(Option E) Taking no action while measles is a notifiable disease is not appropriate. A mentioned before all cases of measles should be reported.

References

• SA Health - Measles and measles contacts in pregnancy

Last updated:
Time spent: QID:1022
2023-2-12

984 of 1943
A 27-year-old pregnant woman presents to your practice at 17 weeks gestation with complaint of eruption of a rash, following a 2-day history of malaise, low grade fever and
rhinorrhea. You suspect measles and order serology tests for her. IgM against measles is positive; IgG is negative. Which one of the following is the most appropriate next step in
management?

A. MMR vaccine.

B. Serologic testing of the fetus for measles.

C. Administration of natural human immunoglobulin (NHIG) within 6 days.

D. Contact tracing.

E. Repeat the serology in 2 weeks.

Incorrect. Correct answer is D


45% answered correctly

Explanation:

Correct Answer Is D

The scenario describes a typical case of measles infection in a pregnant woman, confirmed by serologic studies. Positive IgM suggests acute infection, while negative IgG shows
that the infection is in its early phase and seroconversion has not occurred yet.

After contracting measles, conservative management of the symptoms and its potential complications is the only management plan therapeutically. Another very important issue is
‘notification’ and contact tracing. Measles is a notifiable disease and healthcare professionals are mandated to report cases of measles to the authorized public health units.
Contact tracing is the main objective of this notification.

Control of measles relies on early diagnosis and notification of the cases, prompt isolation of infectious cases, and timely and effective identification of contacts, with provision of
advice and post-exposure prophylaxis, as appropriate.

Since measles is primarily transmitted by air-borne means, a contact is defined as anyone, who has or may have shared the same air-space (enclosed area) for any length of time
with an infectious case. Contact tracing should aim to identify those most susceptible to measles or at greater risk from infection, including infants, immunocompromised people
and pregnant women.

In general, contacts may be prioritized in the following order, recognizing that it may not be feasible for the PHU to identify and arrange post-exposure prophylaxis for all susceptible
contacts, given the constraints of time, resources and logistics:

The household and other settings where people share communal facilities (for example, in a hospital, boarding school or military barracks)

Early childhood education and care services, school or other educational settings where people share a classroom with the case

People who shared a waiting area at the same time as the case (for example, patients in a healthcare facility’s waiting room and any people accompanying these patients)
and people who used the waiting area or who were seen in the same consulting room for up to 30 minutes after the case left. The latter period is based on the recognition
that normal room ventilation systems ensure that levels of airborne viruses and virus survival diminish rapidly.
Work settings where work colleagues have shared the same work area or communal facilities at the same time as a case.

(Option A) MMR vaccine is used for prevention of measles including post-exposure prophylaxis. It is not useful once measles is contracted. Even for prophylaxis, MMR was
contraindicated throughout pregnancy. For those with contact to a case of measles, MMR vaccine within 72 hours of contact may have a protective effect. All measles-containing
vaccines (MMR, MMRV) are contraindicated throughout pregnancy.

(Option B) Serologic testing of the fetus is not useful because circulating maternal antibodies cross the placenta into fetal ciculation. A positive test does not mean that the fetus is
infected.

(Option C) NHIG is used as a post-measles exposure prophylactic means for those persons with contraindications to MMR vaccine such as pregnant women, premature babies, etc.
It is not useful for treatment of established measles.

(Option E) Repeating measles-specific serologic test is not necessary in this case because both symptoms and IgM levels indicate measles as the diagnosis. In general no test is
indicated, unless the result has an impact on further management or is of prognostic value. Repeating serologic tests in a patient with established measles is futile.

References

• http://www.health.gov.au/internet/main/publishing.

Last updated:
Time spent: QID:1024
2023-2-12

985 of 1943
A 27-year-old woman presents to the primary care center where you work at 26 weeks gestation after she has been noticing intermittent leakage of watery liquor per vagina for the
past eight hours, especially after straining, coughing or sneezing. You perform a speculum vaginal exam which reveals clear fluid in the posterior vaginal fornix, as well as flow of
liquid through the cervical os. Further evaluation, establishes preterm premature rupture of the membranes (PPROM) as the diagnosis. No uterine contraction is felt. There is a
tertiary hospital 50 km away. Which one of the following is the most appropriate next step in management?

A. Administration corticosteroids.

B. Commencement of tocolysis.

C. Transferring her to the tertiary hospital.

D. Sending her home on oral antibiotics and warning signs.

E. Admission for observation.

Correct
45% answered correctly

Explanation:

Correct Answer Is A

The scenario is a classic presentation of preterm premature rupture of membranes (PPROM). Rupture of membrane harbingers labor in term or near term women. If ROM does not
end up in established labor in 4 hours, it is termed premature ROM (PROM). In other words, PROM is defined by ROM befire the laor onset. If it occurs before 37 + 0 days weeks,
PPROM is the preferred entity, as approach to PROM and PPROM are different.

The classic presentation of rupture of the membranes (ROM), regardless of the gestational age, is with a sudden gush of watery fluid per vagina or continuous or intermittent
leakage of fluid or the sensation of wetness within the vagina or on the perineum. Presence of liquor flow from the cervical os or pooling in the posterior vaginal fornix is
pathognomonic.

PPROM is associated with the following risks:

Preterm labor
Cord prolapse
Placental abruption
Chorioamnionitis
Fetal pulmonary hypoplasia and other features of prematurity
Limb positioning defects
Perinatal mortality

Diagnosis of women with PPROM, as usual, begins with history and physical exam. Fetal heart rate auscultation and monitoring (+/- CTG) is another important step. A speculum
vaginal exam should be performed to exclude cord prolapse, to assess the cervical length, and visualize pooling of liquor in the posterior fornix. High and low vaginal swabs should
be obtained. Samples should be taken for nitrazine test.

Ultrasound may be used for estimation of amniotic fluid volume if the diagnosis is not certain, as well as for assessment of fetal wellbeing, estimated fetal weight, location of
placenta, etc.

Once the diagnosis is certain the following should be considered as components of a management plan:

Maternal corticosteroids

Corticosteroids are effective in reducing adverse perinatal outcomes such as respiratory distress syndrome, intraventricular hemorrhage, and necrotizing enterocolitis. In case
respiratory distress develops, duration of neonatal respiratory support is significantly reduced if corticosteroids have been administered.

Corticosteroids are indicated in the following situations:

Gestational age is between 23+0d and 34+6d weeks if preterm labor is a concern
Preterm birth is planned or expected within the next 7 days

Recommended regimens are IM betamethasone in two doses of 11.4 mg, 24 hours apart, to the woman. If betamethasone is unavailable, IM dexamethasone in two doses of 12 mg,
24 hours apart, is given.

When the gestational age is less than 32 + 6d, a single repeat dose of corticosteroid may be given seven days or more after the first dose if the woman is still considered to be at risk
of preterm labor. Up to 3 repeat doses might be considered.

Antibiotics

The rationale for antibiotic prophylaxis is that infection appears to be both a cause and consequence of PPROM and is related to preterm delivery. The objective of antibiotic therapy
is to reduce the frequency of maternal and fetal infection and delay the onset of preterm labor.

Antibiotic prophylaxis is indicated in all women with PPROM after low and high vaginal swabs are taken. In the absence of chorioamnionitis recommended antibiotic prophylaxis of
choice is erythromicin given for 10 days intravenously.

Chorioamnionitis is a feared complication of PPROM. The diagnosis of chorioamnionitis relies on the clinical presentation and may be difficult in its early manifestations. The
clinical picture may include maternal fever with 2 or more of the following:

986 of 1943
Increased white cell count (> 15 x 109 / L)
Maternal tachycardia (> 100bpm)
Fetal tachycardia (>160bpm)
Uterine tenderness
Offensive smelling vaginal discharge
C-Reactive Protein > 40

Tocolytics

Chorioamnionitis is an absolute contraindication to tocolysis, while PPROM in the absence of intrauterine infection is only relatively contraindicated. The rationale for PPROM being
so is that it is often difficult to exclude infections in the presence of PPROM because at times uterine contractions can be the only presenting symptom.

However, where contractions are present, nifedipine, as a tocolytic, may be started to prolong pregnancy for 48 hours while corticosteroid cover is established if there are no other
signs of chorioamnionitis.

If not in labor or labor is not anticipated immediately, transfer to a tertiary care facility should be considered if possible. This ensures prompt care of the premature neonate if
preterm labor and delivery occurs.

In this woman with PPROM at 26 weeks gestation, administration of corticosteroids as outlined above is the most appropriate next step in management.

For the moment, there are no signs and symptoms suggesting intrauterine infection; nonetheless, prophylactic antibiotics should be started to prevent infections as well as to delay
labor.

(Option B) Tocolysis using nifedipine was indicated if the woman was in labor. Cessation of labor for at least 48 hours provides a window for corticosteroid to establish its effects.
In the absence of uterine contractions suggesting labor, tocolysis is not needed.

(Option C) Transferring this woman to a tertiary hospital is an appropriate option after first doses of corticosteroid and antibiotic are given. Being in a tertiary hospital ensures
optimal neonatal care in case of premature delivery.

(Option D) Discharging this woman home on oral antibiotics is not an appropriate option because she needs investigations and fetal monitoring as well as close observation for
dvelopment of any signs of infection and preterm labor.

(Option E) Admission a primary care center with no neonatal ICU (NICU) adds no benefit to the outcome.

References

• https://www.ranzcog.edu.au/Statements-Guidelines/Obstetrics/Preterm-Prelabour-Rupture-of-Membranes

Last updated:
Time spent: QID:1025
2023-2-12

987 of 1943
A 23-year-old woman presents to your clinic at 26 weeks gestation after a sudden gush of watery fluid while cooking. Examination establishes the diagnosis of preterm premature
rupture of membranes (PPROM). Fetal assessment using ultrasound and cardiotocography (CTG) is reassuring. Vaginal examination using a speculum shows fluid in the posterior
vaginal fornix and a 1cm dilated cervical os. Uterine contractions are absent. There are two hospitals in the vicinity: a primary hospital and a tertiary hospital, 50km and150 km away,
respectively. After administration of antibiotics and corticosteroids, which one of the following would be the most appropriate step in management of this patient?

A. Give him the warning signs and send her home.

B. Send her to the secondary care hospital 50km away.

C. Send her to the tertiary care chospital 150km away.

D. Tocolysis.

E. Labor induction.

Incorrect. Correct answer is C


45% answered correctly

Explanation:

Correct Answer Is C

PROM is defined as rupture of fetal membranes before labor at any gestational age. If PROM occurs before, it is termed premature PROM (PPROM). The main concerns in PPROM
are preterm labor, which is associated with neonatal complications due to prematurity if labor establishes, and intrauterine infections with chorioamnionitis being of greatest
significance.

If PPROM occurs before 34+6d, corticosteroids should be given to the mother in an attempt to decrease neonatal complications such as respiratory distress syndrome,
intraventricular hemorrhage, and necrotizing enterocolitis.

Intravenous prophylactic antibiotics are given in anticipation of chorioamnionitis as a major complication of PPROM. Currently, intravenous erythromicin is the common practice is
Australia as it is safe and mildy tocolytic.

Women with PPROM are at significantly elevated risk of preterm labor. The premature baby is required to be cared for in a center with optimal neonatal care in a neonatal intensive
care unit (NICU). This can be found in a tertiary hospital. Therefore, if a woman with PPROM is not in labor and the labor is not anticipated immediately, the most appropriate next
step will be transferring her to a tertiary hospital with NICU.

(Option A) Expectant management at home may be considered for all man after 72 hours of hospitalization if there is:

No further fluid leakage.


Singleton pregnancy AND
Cephalic presentation > 23 weeks AND
Easy access to the hospital

This woman just had PPROM and has not been intensively evaluated in a hospital. Moreover, the nearest tertiary hospital is 150km away. Therefore, she does not meet criteria for
expectant home care.

(Option B) Secondary care hospitals do not have NICU and add no benefit to the care of the patient in this regard.

(Option D) If the patient was in labor, evident by presence of uterine contractions and a dilating cervix, use of tocolytic agents were indicated to stop labor for at least 48 hours. This
could provide a window for corticosteroids to exert their effects, as well as providing time for transfer to a tertiary care facility. Tocolysis is contraindicated in the presence of
intrauterine infection, but only relatively contraindicated in PPROM; therefore, in the absence of intrauterine infection benefits of tocolysis outweigh risks of prematurity in women
with (P)PROM and with uterine comtractions.

(Option E) Induction of labor is not indicated due to prematurity unless:

Signs of chorioamnionitis are present


There is anterpartum hemorrhage
The woman requests so
There is neonatal jeopardy.

References

• https://www.ranzcog.edu.au/Statements-Guidelines/Obstetrics/Preterm-Prelabour-Rupture-of-Membranes

Last updated:
Time spent: QID:1026
2023-2-12

988 of 1943
Which one of the following drugs can potentially cause the most harmful neurological defects in a fetus if given during pregnancy?

A. Amphetamine.

B. Morphine.

C. Cocaine.

D. Amitriptyline.

E. Alcohol.

Incorrect. Correct answer is C


45% answered correctly

Explanation:

Correct Answer Is C

The role of cocaine in producing congenital malformations is controversial. Some studies suggest that congenital birth defects including those of face, nervous system and urinary system are possible. These problems, however, are very rare and
somewhat negligible. Although cocaine is not associated with congenital anomalies, it has been shown, especially in late pregnancy, to be associated with increased risk of fetal intracranial hemorrhage with the worst possible outcome among the
options.

The following features are definitely associated with maternal cocaine use during the pregnancy:

IUGR
Placental abruption
Still birth
Premature delivery
Need for neonatal resuscitation
Developmental delay
Intraventricular hemorrhage

(Option A) The role of amphetamines in fetal malfomrations are controversial. While some studies have shown no teratogenic effects associated with amphetamines, other suggest that orofacial clefts are possible effects of amphetamine use
during pregnancy; the incidence, however, has been exceedingly rare. Other complications of amphetamines during pregnancy include:

Intrauterine growth restriction (IUGR)


Preterm delivery
Increased neonatal mortality and morbidity
Hypertension and pre-eclampsia

(Option B) Morphine or other opiates have a number of complications. One example is transitory CNS depression. This is not of major concern if promptly being taken care of.

(Option D) Tricyclic antidepressants (e.g. amitriptyline) are not associated with congenital anomalies or concerning complications of the fetus or neonate and are rather safe for use during pregnancy.

(Option E) Alcohol, if used in excess amounts can cause foetal alcohol syndrome which has developmental delays as part the complications spectrum; however, intracranial hemorrhage associated with cocaine use has a much worse outcome
compared to alcohol.

TOPIC REVIEW

The following table lists the most important drugs associated with congenital malformations:

Drug Associated congenital malformations


Alcohol Fetal alcohol syndrome
Isotretinoin Congenital deafness, microtia, CNS defects, congenital heart defects
Craniofacial dysmorphism (epicanthal folds, depressed nasal bridge, oral clefts), mental retardation, microcephaly, nail hypoplasia, heart
Phenytoin
defects
Neural tube defects, mental retardation and a many other problems. Sodium valproate is the most harmful antiepileptic drug with the
Sodium valproate/carbamazepine and some other anticonvulsants
worst congenital malformations.
Lithium Ebstein anomaly (right heart defect)
Tetracycline Enamel hypoplasia, discolored teeth, bone problems
Warfarin Facial dimorphism and chondrodysplasia, microcephaly, optic atrophy
ACE inhibitors Craniofacial anomalies
Diethylstilbestrol (DES) T-shaped uterus, vaginal adenosis (with predisposition to vaginal clear cell carcinoma)
Aminoglycosides e.g. streptomycin 8th nerve damage, hearing loss
Thalidomide Phocomelia (1), limb reduction defects, ear/nasal anomalies, cardiac defects, pyloric or duodenal stenosis
Cocaine and amphetamines Cleft lip

(1) An extremely rare congenital disorder presenting with limb malformation (dysmelia)

References

• http://www.merckmanuals.com/professional/gynecolog

• Llewellyn-Jones Fundamentals of Obstetrics and Gynaecology – Elsevier –Mosby - 9th Edition

Last updated:
Time spent: QID:133 2023-2-12

989 of 1943
A 22-year-old woman presents with complaint of amenorrhea of one year duration. Her menses were regular before. She has no other complaints or symptoms. Physical
examination is inconclusive. Pelvis ultrasonography is unremarkable. Thyroid function tests show a TSH of 3.5mU/L (0.5-5mU’/L). Which one of the following is most likely to be the
cause of her amenorrhea?

A. Mullerian agenesis.

B. Ovarian dysgenesis.

C. Hormonal dysfunction.

D. Hyperthyroidism.

E. Subclinical hyperthyroidism.

Incorrect. Correct answer is C


45% answered correctly

Explanation:

Correct Answer Is C

Secondary amenorrhea is defined as cessation of periods after they have started. As opposed to secondary amenorrhea is defined as lack of menstrual flow by age 14 and absence
of secondary sexual characteristics or lack of menstrual flow by age 16 and presence of secondary sexual characteristics

Since this woman's case is the cessation of previously present periods, she has secondary amenorrhea. Mullerian agenesis (option A) and ovarian dysgenesis (option B) result in
primary amenorrhea. With previous menstruation, these two conditions are unlikely. Of the uterine causes, only Asherman’s syndrome can be the cause of secondary amenorrhea.

Hyperthyroidism can cause menstruation abnormalities, particularly amenorrhea or oligomenorrhea; however, a normal TSH excludes hyperthyroidism (option D) as the etiology of
her amenorrhea. Subclinical hyperthyroidism is defined as a subnormal TSH while T3 and T4 are within normal range. This woman has a normal TSH level and does not have
subclinical hyperthyroidism (option E).

Of the options, the only explanation can be hormonal dysfunction. A hormonal dysfunction in this case could be due to conditions such as a pituitary tumor, hyperprolactinemia,
ovarian insufficiency, or polycystic ovary syndrome (PCOS).

References

• UpToDate - Evaluation and management of secondary amenorrhea

Last updated:
Time spent: QID:1028
2023-2-12

990 of 1943
A 37-year-old woman presents with secondary amenorrhea of 7 months duration. She has a body mass index (BMI) of 24. On laboratory studies, she has a FSH level of 55 U/L (2-
8U/L in luteal phase; >25U/L in menopause), LH of 54 U/L and estradiol the lower limit of normal. Serum prolactin level is also normal. A urine pregnancy test excludes pregnancy.
On ultrasonography, each ovary contains 3-4 cysts. She desires to become pregnant in the future. Which one of the following would be the most appropriate management of her
condition?

A. Progestin-only pills (POP).

B. Menopausal hormone replacement therapy (HRT).

C. Combined oral contraceptives (COCs).

D. Metformin.

E. Danazol.

Incorrect. Correct answer is B


45% answered correctly

Explanation:

Correct Answer Is B

In this woman, FSH and LH are within menopausal range (cut-off point> 40 U/L) and estradiol in the normal lower limit. Prolactin level is normal and excludes hyperprolactinaemia
as a cause of her amenorrhea. The clinical picture and laboratory values are highly suggestive of ovarian failure.

Primary ovarian insufficiency (POI), commonly referred to as premature ovarian failure (POF), is defined as ovarian failure before 40 years of age. The incidence is 1 in 250 by age 35
and 1 in 100 by age 40.

In its full-blown presentation, ovarian failure is associated with amenorrhea, symptoms of estrogen deficiency, and increased gonadotropin levels (In the menopausal range) before
the age of 40 years. The term 'failure' means that ovarian function is not normal, but it does not necessarily imply total cessation of ovarian function. Patients diagnosed with
primary ovarian insufficiency may intermittently produce estrogen, ovulate or even conceive and have a normal pregnancy and delivery in 5-10% of cases

Measuring serum FSH level is the core study to establish the diagnosis of POI/POF after pregnancy has been ruled out. By convention, two FSH levels in the menopausal range for
the specific assay (>40 U/L by radioimmunoassay), measured at least 1 month apart, are diagnostic of POI/POF.

Measurement of serum LH is also important. In most cases of spontaneous POI/POF, FSH is higher than LH. If autoimmune oophoritis is present, FSH may be only mildly elevated,
sometimes below the cutoff of 40 U/L, while LH is markedly elevated.

All women with POI/POF should be started on menopausal hormone replacement therapy (HRT) to prevent menopause complications and this should be continued at least until the
average age of normal menopause (50-51 years). The main objective of early HRT is prevention of bone loss and early osteoporosis. Other measures to consider are
supplementation of calcium and vitamin D.

Standard postmenopausal HRT does not provide effective contraception and spontaneous ovarian activity may resume; therefore, women with POI/POF, who require contraception,
should use oral contraceptives for both hormone replacement and contraception.

This woman desires pregnancy; therefore, the most appropriate option for her is postmenopausal HRT. OCP would have been the correct answer if she also had asked for
contraception.

References

• Medscape - Ovarian Insufficiency

• Clinical manifestations and diagnosis of spontaneous primary ovarian insufficiency (premature ovarian failure)

• Management of spontaneous primary ovarian insufficiency (premature ovarian failure)

Last updated:
Time spent: QID:1044
2023-2-12

991 of 1943
Which one of the following foods should be avoided during pregnancy?

A. Liver.

B. Smoked salmon.

C. Beans.

D. Fruits.

E. Rice.

Incorrect. Correct answer is B


45% answered correctly

Explanation:

Correct Answer Is B

Liver, beans, fruits and rice can all be safely used during pregnancy. Salmon and other sea foods are generally safe to use, unless they are smoked and ready-to-use. Smoked salmon
and trout are susceptible to colonization with Listeria monocytogenes even in the refrigeration temperature and should be avoided during pregnancy.

Soft cheese is another source of Listeria, unless it is cooked above 65°C and served hot.

References

• http://www.babycenter.com.au/c4209/food-and-drink-

• http://www.nutritionaustralia.org/sites/default/fi

• https://www.nhmrc.gov.au/_files_nhmrc/publications

Last updated:
Time spent: QID:1064
2023-2-12

992 of 1943
Lisa is in your office at 8 weeks pregnancy for an antenatal visit. She asks you about foods that can be harmful to her pregnancy. Which one of the following she should avoid during
pregnancy?

A. Soft cheese.

B. Tinned salmon.

C. Fish rich is iodine.

D. Lobster.

E. Liver.

Correct
45% answered correctly

Explanation:

Correct Answer Is A

Of the options, soft cheese should be avoided due to the risk of listeriosis, unless it is thoroughly cooked and heated.

Listeria monocytogenes can cause invasive diseases including meningitis, meningoencephalitis, or bacteremia in immunosuppressed patients, individuals at the extremes of age
including neonates and elderly adults, and pregnant women. Listeriosis can be associated with miscarriage and early neonatal sepsis.

Some foods, such as soft cheese, are more prone to contamination by Listeria, which can even grow at refrigeration temperature. Such foods should be avoided during pregnancy.

During pregnancy it is recommended that the following dairy products be avoided:

Soft, semi-soft and surface ripened cheeses (e.g. brie, camembert, ricotta, feta and blue cheese)
Soft serve ice cream
Unpasteurized dairy foods - almost all dairy foods produced in Australia must be pasteurized, however some specialty imported cheeses may be unpasteurized.

NOTE - Soft cheese is safe if it is cooked above 65˚C Sand served hot e.g. ricotta and spinach cannelloni, cheese topping on pizza.

Ready-to-eat fish, such as smoked salmon or trout is also considered high-risk for Listeria contamination and should be avoided during pregnancy, but canned salmon or
appropriately cooked salmon can be is used safely.

Liver, fish rich in iodine (if not smoked or ready-to-use), and lobster can be safely used if appropriately cooked and served.

References

• http://www.nutritionaustralia.org/sites/default/fi

• http://www.babycenter.com.au/c4209/food-and-drink

• https://www.nhmrc.gov.au/_files_nhmrc/publications

Last updated:
Time spent: QID:1065
2023-2-12

993 of 1943
Kate is 22 years old and a patient in your clinic for the past 3 years. She was diagnosed with hypothyroidism 12 months ago, and was started on levothyroxine 100 mcg, daily. Today,
she has presented with complaints of hot flushes and irregular periods for the past 4 months. Her periods were previously regular occurring every 28-30 days. Which one of the
following hormonal assessments is more important to do next?

A. Estrogen.

B. Follicular stimulating hormone (FSH).

C. Luteinizing hormone (LH).

D. Thyroid stimulating hormone (TSH).

E. Prolactin.

Incorrect. Correct answer is D


45% answered correctly

Explanation:

Correct Answer Is D

The scenario represents a case of irregular periods and hot flushes. In approach to menstruation abnormalities, thyroid hormone problems should always be considered. In fact with
Kate’s history, the diagnostic approach is completely clear. Kate is on thyroid hormones for treatment of hypothyroidism. Excess exogenous thyroid hormones either used for
treatment can have led to hyperthyroidism, which in turn has resulted in menstruation abnormalities. The most common menstruation abnormalities associated with
hyperthyroidism are oligomenorrhea and amenorrhea; however, menorrhagia is also possible.

Also, hyperthyroidism is a well-known cause of non-menopausal hot flushes. Hypertension is another cause.

Another possibility is that her hypothyroidism is still undertreated. Hypothyroidism, however, is most likely to present with menorrhagia. Additionally, the presence of hot flushes
goes against hypothyroidism.

In this case, the next best step in approach to Kate's problem, after taking a complete history, performing a thorough physical exam, and excluding pregnancy is to order a TSH.

Estrogen (option A), FSH (option B), LH (option C), and prolactin (option E) should be considered next if TSH is normal and thyroid problems are excluded because premature
ovarian failure is another possibility to consider next.

References

• UpToDate - Overview of the clinical manifestations of hyperthyroidism in adults

Last updated:
Time spent: QID:1069
2023-2-12

994 of 1943
Regarding oxytocin, which one of the following is true?

A. It has antidiuretic effect.

B. It is a steroid.

C. It is produced by the posterior pituitary.

D. It has an action on most involuntary muscles.

E. It causes milk production.

Correct
45% answered correctly

Explanation:

Correct Answer Is A

Oxytocin has an antidiuretic effect through an unknown mechanism.

(Options B and C) Oxytocin is a neurohypophysial peptide mostly produced in hypothalamus and stored in the posterior pituitary; however, recent studies showed that this hormone
is also produced in uterus, placenta, amnion, corpus luteum, testis and heart.

(Option D)The most prominent action of oxytocin is on smooth muscles of the uterus and breast. The most widely known effect of oxytocin on the uterine smooth muscles is
uterine contraction. Its effect on smooth muscles of the breast is milk ejection during lactation. Oxytocin does not have an action on most of involuntary muscles such as those in
digestive system or respiratory tract, or smooth muscles within the wall of vessels; at least not while within physiologic limits.

(Option E) Although oxytocin is the main hormone in milk ejection process and milk-let-down reflex, it has no known role in milk production.

References

• http://www.ncbi.nlm.nih.gov/pmc/articles/PMC239673

• http://www.ncbi.nlm.nih.gov/pubmed/11274341

• http://physrev.physiology.org/content/81/2/629

Last updated:
Time spent: QID:1112
2023-2-12

995 of 1943
Which one of the following is not an effect of oxytocin?

A. Mammalian glands stimulation.

B. Uterine stimulation.

C. Labor induction.

D. Progesterone release.

E. Antidiuretic effect.

Incorrect. Correct answer is D


45% answered correctly

Explanation:

Correct Answer Is D

Oxytocin stimulates the smooth muscles of the uterus and breast. The most widely known effect of oxytocin on the uterine smooth muscles is uterine contraction and labor
induction. Its effect on smooth muscles of the breast is milk ejection during lactation. It has no role in milk production.

Also, oxytocin has antidiuretic effect through an unknown mechanism.

Progesterone release is not an effect of oxytocin.

References

• http://www.ncbi.nlm.nih.gov/pmc/articles/PMC239673

• http://www.ncbi.nlm.nih.gov/pubmed/11274341

• http://physrev.physiology.org/content/81/2/629

Last updated:
Time spent: QID:1113
2023-2-12

996 of 1943
A 17-year-old girl is brought to your practice for evaluation because her periods have not started yet. She also complains of abdominal pain every month and about the same time.
On examination, she has fully developed breasts and axillary and pubic hair. She does not consent to a vaginal exam. Which one of the following could be the most likely diagnosis?

A. Imperforated hymen.

B. Absent vagina.

C. Mullerian agenesis.

D. Turner syndrome.

E. Normal variant.

Correct
45% answered correctly

Explanation:

Correct Answer Is A

This girl has presented with primary amenorrhea as her chief complaint.

A girl is expected to have her periods until the age of 16 years. Primary amenorrhea is defined as absence of periods after the age 15-16 years in the presence of secondary sexual
characteristics, or after the age of 13-14 years in the absence of such characteristics. The most common cause of primary amenorrhea is gonadal dysgenesis such as in Turner
syndrome, followed by müllerian agenesis as the second most common cause. Patients with müllerian agenesis typically present with primary amenorrhea in the presence of
normal growth and development in adolescence.

On physical examination, patients with müllerian agenesis have normal height, secondary sexual characteristics, body hair, and external genitalia, but a vagina is either absent, or
present as a short blind-ended structure without a cervix at the vaginal apex. Patients with müllerian agenesis have a normal 46,XX karyotype and a normal hormonal profile. Most
patients have a rudimentary nonfunctioning uterus. In other words, cyclical endometrial shedding does not occur in most of women with müllerian agenesis; however, in 2 to 7%
cases, there is a uterus with functioning endometrium. In such women, chronic or cyclic abdominopelvic pain occurs secondary to hematocolpos (accumulation of blood in the
vagina), hematosalpinx (accumulation of blood in the ovarian tubes), hematometra (accumulation of blood in the uterus), or endometriosis.

Imperforated hymen can be another cause of primary amenorrhea. In contrast to most patients with müllerian agenesis, patients with an imperforate hymen will not have the typical
fringe of hymenal tissue.

The patient with a low transverse vaginal septum will have a normal hymen with more proximal obstruction of the vaginal canal. In addition to presenting with primary amenorrhea,
imperforated hymen and transverse vaginal septum are often associated with symptoms of cyclic abdominal or pelvic pain and a pelvic mass due to the obstructed outflow tract
and associated hematocolpos.

With findings of fully developed sexual characteristics, absent periods, and cyclical pain, an imperforated hymen is the most likely diagnosis both clinically and statistically.

(Options B and C) Absent vagina can be a presentation of a müllerian agenesis. Absent vagina /müllerian dysgenesis can present similar to imperforated hymen in 2 to 7% of cases
in whom a functioning uterus exists. However, imperforated hymen is more likely statistically.

(Option D) In Turner syndrome, usually no menstruation occurs. In fact, the gonads are dysplastic and malfunctioning (streak gonads). The patients often have secondary sexual
characteristics (often delayed), but there is no cyclical abdominal pain because they do not menstruate. Short stature is commonly noted.

(Option E) Absence of periods after the age of 16 years is always alarming and cannot be considered a normal variant.

References

• UpToDate - Evaluation and management of primary amenorrhea

Last updated:
Time spent: QID:1120
2023-2-12

997 of 1943
A 53-year-old woman presents to your practice with vaginal bleeding after sexual intercourse 7 hours ago. She has not had any menses for the past 18 months. Her last cervical
screening test, performed 12 months ago, was normal. She does not complain of any other symptoms and is otherwise healthy. Which one of the following could be the most likely
underlying cause of postcoital bleeding in this woman?

A. Endometrial cancer.

B. Cervical cancer.

C. Cervical polyp.

D. Cervical ectropion.

E. Vaginal atrophy.

Incorrect. Correct answer is E


45% answered correctly

Explanation:

Correct Answer Is E

Postmenopausal vaginal bleeding, including post coital bleeding, can have many causes. Some menopausal women may experience ‘rouge ovulation’ which is an episode of
ovulation and followed by menstruation after more than a year without episodes. Hormone replacement therapy (HRT) is another important cause of post-menopausal vaginal
bleeding. Apart from those, vaginal bleeding in postmenopausal women should be taken very seriously and investigated promptly.

Fortunately, the most common causes of post-menopausal vaginal bleeding are benign and far less serious. Of these causes, atrophic vaginitis due to ongoing estrogen deficiency
has been shown to be the most common cause of vaginal bleeding, especially post coital bleeding, in menopausal women.

It should be noted the although serious condition such as endometrial cancer or hypertrophy and cervical cancer are less common and comprise up to 20% of cases presenting with
post-menopausal bleeding, these conditions should always considered first and excluded using prompt investigations.

Guidelines by the RACGP recommend that any post coital bleeding in a post-menopausal woman should be investigated with a co-test (HPV and LBC) and referral for gynecological
assessment to rule out cervical cancer.

(Option A) Although endometrial cancer can be another possibility, compared to atrophic vaginitis, it is not as common; nonetheless, assessment to exclude endometrial cancer
should be undertaken with transvaginal ultrasound and endometrial biopsy if there is an abnormally thick endometrium.

(Option B) With a normal pap smear 12 months ago, cervical cancer is very unlikely.

(Options C and D) Cervical ectropion and cervical polyps can be other causes of postcoital bleeding. Cervical ectropion is more commonly a result of high estrogen levels such as
during ovulatory phase in younger women, pregnancy and in those women on oral contraceptive pills. Cervical ectropion in postmenopausal women is not very common.

TOPIC REVIEW

Common causes of irregualr vaginal bleeding, including postcoital bleeding, are summarized in the following table:

Relation of age to common causes of irregular uterine bleeding


AGE GROUP
15-20 20-30 30-45 45-55 55+
STIs (especially Chlamydia cervicitis) HRT

Cervical ectropion Endometrial cancer


Endometrial polyps

Endometrial hyperplasia

Uterine fibroids
Intrauterine device atrophic vaginitis
Pregnancy and complications; miscarriage/ectopic pregnancy

Contraceptive steroids, especially progestogen

Endometriosis
Trauma / surgery

References

• AAFP - Diagnosis and Treatment of Atrophic Vaginitis

998 of 1943
Last updated:
Time spent: QID:1136
2023-2-12

999 of 1943
Robert and Janet are in your clinic for consultation. Robert complains of malaise, mild headache and a generalized rash in form of both intact and unroofed vesicles, as well as
macules and papules. Three weeks ago, Alan, their 7-year-old was diagnosed with chickenpox. Based on the history and the appearance of the rash, you establish the diagnosis of
chickenpox for Robert. Janet is 13 weeks pregnant and is concerned if the infection can affect her and her baby too. Janet is not sure if she had chickenpox in childhood. Which one
of the following is the most appropriate next step in management of Janet?

A. Start her on oral acicolvir.

B. Give her varicella zoster vaccine (Varivax).

C. Give her varicella zoster immunoglobulin (VZIG).

D. Arrange for checking her varicella zoster antibody (IgG) status.

E. Start her on oral famcilovir.

Incorrect. Correct answer is D


45% answered correctly

Explanation:

Correct Answer Is D

In approaching a pregnant woman, who has been exposed to varicella zoster virus (VZV) by contact to a case of chicken pox, the first question to be answered is whether the
pregnant woman has had a history of chickenpox. If she remembers chickenpox infection, no further action is required. If she has had not the infection or the history is uncertain,
the next step in management would be serologic tests for VZV (IgG). If the test is positive, the woman is immune. If not, management depends on the time elapsed since exposure:

1. If the presentation is within the first 96 hours after exposure, passive immunization with VZV immunoglobulin (VZIG) is the most appropriate next step in management.
2. If the presentation is after 96 hour, VZIG is of no use; however, the following women should still receive prophylaxis with the antiviral agents aciclovir (first line) or
valaciclovir:

In the second half of pregnancy


With a history of an underlying lung disease
Immunocompromised
Smokers

Since Janet is not certain about contracting chicken pox before and there is no history of vaccination against VZZ, the next best step for her would be checking her immunity status
against VZV by VZV IgG (D is correct).

(Option A) Oral aciclovir (or valaciclovir as a second option) is used for women with established VZV infection who present within the 24 hours after the rash appears or for those
who present later and are at risk of complicated VZV infection indicated by: neurological symptoms, fever longer than 6 days, eruption of new pocks after 6 days, VZV pneumonia, or
those who are immunocompromised.

(Option B) Varivax contains live attenuated virus and is contraindicated throughout pregnancy.

(Option C) VZIG would be the correct option if IgG levels show that Janet is not immune to VZV.

(Option E) while there is adequate data that aciclovir and valaciclovir are safe to use during pregnancy, safety of famciclovir for use in a pregnant has yet to be established; hence,
famciclovir is not as safe.

References

• Australian Society of Infectious Diseases - Management of Prenatal Infections

• South Australia Health Department - Varicella Zoster (chicken pox) in Pregnancy Clinical Guideline

Last updated:
Time spent: QID:1202
2023-2-12

1000 of 1943
A 35-year-old woman is brought to the emergency department after she sustaind ankle twisting. After initial management of her current problem, you realize that she is 10 weeks along her second pregnancy. On further questioning, she admits to
heroin addiction. She also says that ‘Doc, I sometimes need to get high on meth. My favorite wings to fly are cocaine though, but since I cannot afford it, I a bit when I manage to crash a party’. She drinks a bottle of Whisky every day. During the past
few weeks, she has started to worry about not being able to be a good mother. For this reason she is taking diazepam at night. She says that she managed to illegally get a bottle of these sleeping pills. Which one of the following in her history is
most likely to cause fetal malformations?

A. Diazepam.

B. Alcohol.

C. Amphetamine.

D. Cocaine.

E. Heroine.

Incorrect. Correct answer is B


45% answered correctly

Explanation:

Correct Answer Is B

This woman is taking a bottle of Whisky every day. A small glass of Whisky (1.5 Oz) is equivalent of a standard drink. A bottle definitely exceeds 12 standard drinks; therefore this woman’s fetus would be at significant risk of fetal alcohol syndrome
(FAS) which is associated with the following congenital malformations:

Low-set ears
Midfacial hypoplasia
Elongated philtrum
Upturned nose
Microcephaly
Skeletal and cardiac malformations

(Option A) The health risks of benzodiazepines in pregnancy have not been clearly established. There have been inconsistent reports of teratogenic effects associated with fetal exposure to benzodiazepines. Regular benzodiazepine use in pregnancy
may be associated with a neonatal abstinence syndrome, which may be of delayed onset..

(Option C) Amphetamine use in controlled doses during pregnancy is unlikely to pose a substantial teratogenic risk, but women who use amphetamines are at higher risk of a range of obstetric complications such as reduced birth weight — many of
these outcomes are not specific to amphetamines but influenced by other drug use and lifestyle factors in addition to amphetamine use. Exposure to amphetamines in utero may influence prenatal brain development, but the nature of this influence
and potential clinical significance are not well established.

3,4- methylenedioxymethamphetamine (MDMA), commonly known as ecstasy, is an amphetamine derivative. Existing evidence suggests that use of ecstasyduring first trimester poses a potential teratogenic risk; therefore, it is strongly
recommended that use of ecstasy be avoided during this dangerous period of organogenesis (between week two and week eight post conception or between week four and ten of last menstrual period)

(Option D) The role of cocaine in congenital malformations is controversial, and the malformations caused by cocaine are extremely rare. However, if it leads to fetal intracranial haemorrhage the outcome would be devastating.

(Option E) Opiate addiction carries the significant risk of several perinatal complications, but there is no associated congenital malformation.

References

• Llewellyn-Jones Fundamentals of Obstetrics and Gynaecology – Elsevier –Mosby - 9th Edition

• http://www.merckmanuals.com/professional/gynecol

Last updated:
Time spent: QID:134 2023-2-12

1001 of 1943
A 54-year-old woman presents to your clinic, with complaint of a vulvar lesion associate with severe itching. Her problem started nearly 6 months ago and has progressed in
severity. She has been menopausal since the age 50 years. On examination, atrophic white plaques are evident as well as excoriation marks over and around the vulva. The labia
minora is slightly atrophic and there is interoitus stenosis. Vaginal examination is normal. The appearance of the vulva is shown in the accompanying photograph. Which one of the
following is the most appropriate option for treatment of this patient?

A. Vaginal estrogen cream.

B. Clobetasol cream.

C. Hydrocortisone cream.

D. Topical antifungals.

E. Oral prednisone.

Incorrect. Correct answer is B


45% answered correctly

Explanation:

Correct Answer Is B

The clinical findings of pruritic white vulvar plaques, in the absence of vaginal symptoms and signs, make genital lichen sclerosus (LS) the most likely diagnosis.

LS is a chronic inflammatory dermatosis of unknown etiology. The disease most commonly affects the genitalia, but it can occur at any skin site (extragenital lichen sclerosus).
Balanitis xerotica obliterans is an outdated termed used to describe the LS of glans penis. Lichen sclerosus can occur in males or females of any age. In female, however, there are
two peaks of onset during prepubertal and perimenopausal/postmenopausal ages, both of which are estrogen-deficient states; however, the relationship between LS and estrogen
deficiency is unclear. In males it is more common between the puberty and age 60 years.

Typical clinical presentation is with white, often atrophic, plaques associated with pruritus and pain that result in genital scarring and adhesions. The most common complaint in
vulvar lichen sclerosus is progressive pruritus that can be followed by dyspareunia, dysuria, or genital bleeding. Female genital lesions may be confined to the labia majora but
usually involve, and eventually obliterate the labia minora and stenose the interoitus. Often, an hourglass, butterfly, or figure-8 pattern involves the perivaginal and perianal areas, with
minimal involvement of the perineum in between.

Penile LS is associated with sexual/urinary dysfunction. It is usually preceded by pruritus but may present with sudden phimosis of previously retractable foreskin, and urinary
obstruction can result.

Extragenital lichen sclerosus (LS) may be asymptomatic (approximately one third of cases) or it may itch or be tender.

NOTE - With genital LS in women, there is an increased risk of squamous cell carcinoma (<5%). Progression to malignancy in genital LS in men is rare.

First-line therapy for LS includes patient education and super-potent topical corticosteroids (e.g. clobetasol propionate). Prolonged use of potent corticosteroids does not result in
atrophy of the labia minora and clitoris, owing to the resistant nature of their modified mucous membranes. However, perianal and hair-baring areas (e.g. labia majora) can atrophy
within 2-3 weeks of use. Intralesional corticosteroid injections are also considered at occasions.

Second-line therapies include the calcineurin inhibitors, tacrolimus and pimecrolimus, which can be a helpful adjunct to topical corticosteroids for maintenance.

Third-line therapies that could be considered in treatment-resistant genital lichen sclerosus could include topical or oral retinoid, steroid injections, systemic cyclosporine (topical
use has not shown effective), methotrexate, or hydroxyurea. For extragenital lichen sclerosus, phototherapy or methotrexate could be considered.

(Option A) Vaginal estrogen cream is the first-line treatment option for women with atrophic vaginitis and not effective against LS. Normal vaginal exam excludes atrophic vaginitis
as the diagnosis.

1002 of 1943
Although extensively used in the past, topical estrogen, topical testosterone, topical progesterone, and hormone replacement therapy are not recommended anymore due to lack
evidence for effectiveness.

(Option C) Hydrocortisone is a weak corticosteroid and not appropriate for treatment of LS where potent to very potent (super potent) topical corticosteroids are indicated.

(Option D) Topical antifungal agents were the correct answer if the diagnosis would have been vulvovaginal candidiasis. Candidiasis is associated with inflamed skin and mucosa
and cheesy- white vaginal discharge that are not present in this woman.

(Option E) Oral (systemic) predniso(lo)ne is not used for treatment of LS due to their adverse effect profile that is not outweighed by the benefits.

References

• Medscape - Lichen Sclerosus

• Localized provoked vestibulodynia (vulvodynia): assessment and management

Last updated:
Time spent: QID:1219
2023-2-12

1003 of 1943
A 39-week pregnant woman is admitted to the Labor Unit due to commencement of labor pain and contractions. The woman is placed in left lateral decubitus position and started
on supplemental oxygen by face mask. An intravenous line is established and isotonic fluids and cyntocinon drip is started according to the protocol. A CTG is performed that
shows a fetal heart rate of 140 which drops to 70 bpm and returns back to the base line of 140 bpm in two minutes. Which one of the following is the most appropriate immediate
management?

A. Stop cyntocinon.

B. Fetal scalp blood sampling.

C. Immediate cesarean section.

D. Titrate and increase the dose of cyntocinon.

E. Continuous CTG monitoring.

Correct
45% answered correctly

Explanation:

Correct Answer Is A

The scenario represents a case of prolonged fetal heart rate deceleration during labor. Prolonged deceleration is defined as a drop in the fetal heart rate below the baseline that lasts
from 90 seconds up to five minutes. Prolonged bradycardia is defined as a fetal heart rate <100 bpm lasting more than five minutes.

The following are the possible causes of prolonged deceleration or prolonged bradycardia:

Maternal hypotension
Cord prolapse or compression
Uterine hypertonia
Scar dehiscence
Placental abruption
Rapid fetal descent

Management of prolonged decelerations or prolonged bradycardia includes the following steps:

Reposition the woman – e.g. lateral position


Checking maternal blood pressure and administration of bolus IV fluids in case of hypotension.
Discontinuation of oxytocin or decreasing rate of infusion (if in progress)
Check the maternal blood pressure.
Checking the maternal pulse to differentiate maternal pulse rate from the fetal heart rate (FHR).
If indicated, performing a vaginal exam to exclude cord prolapse or rapid cervical dilatation if the bradycardia persists.
Consideration of applying a fetal scalp electrode.
Assessment of abdominal tone to exclude a hypertonic uterus.
Preparation e for assisted delivery (vacuum or forceps delivery) or emergency cesarean section if bradycardia does not resolve.

It is very likely that prolonged decelerations have been caused by excess uterine contraction as a result of oxytocin (cyntocinon) infusion; therefore, cessation of cyntocinon is the
most important immediate action to consider. CTG monitoring (option E) should also be continued throughout the labor for assessment of fetal status.

Fetal scalp blood sampling (option B) is considered if measures such as repositioning the woman, cessation of cyntocinon, blood pressure correction or other measures fail to
correct the situation. This will determine if there is fetal jeopardy necessitating immediate delivery.

(Option C) Urgent cesarean section or assisted vaginal delivery may be required if fetal compromise persists despite more conservative measures.

Increasing the dose of cyntocinon (option D), by increasing the contractions, will escalate the fetal condition and is an incorrect option.

References

• WA Health Department – Obstetrics and Gynaecology Guidelines

• RANZCOG Intrapartum Fetal Surveillance Clinical Guideline

Last updated:
Time spent: QID:1267
2023-2-12

1004 of 1943
A 38-year-old pregnant woman, G1P0, is brought to the emergency department after spontaneous rupture of membranes. She is admitted to the labor unit. After three hours, there is
still no uterine contractions present; hence, she is started on cyntocinon. After four hours, reassessment shows that there are adequate uterine contractions. A CTG shows a base
line fetal heart rate of 143 bpm dropping to 60 bpm and slowly returning to the baseline in four minutes. On vaginal exam, the fetal head is in LOA position, the station is 1+ and there
is 1+ molding. The cervix is dilated at 9 cm. Which one of the following is the most appropriate next step in management?

A. Forceps delivery.

B. Vacuum delivery.

C. Addition of one liter of normal saline.

D. Cessation of cyntocinon.

E. Emergent cesarean delivery.

Incorrect. Correct answer is D


45% answered correctly

Explanation:

Correct Answer Is D

With adequate uterine contractions, head engagement and appropriate descent, the labor is progressing appropriately; however, with FHR dropping to 60 bpm (<100 bpm) there is
prolonged decelerations that make the CTG abnormal.

CTG is considered abnormal if there are at least two of the following features on CTG:

Baseline FHR is between 100-109 bpm or between 161-170 bpm


Variability of FHR is reduced (3-5 bpm for >40 minutes)
Decelerations are variable without complicating features

OR

There is any of the following:

Baseline FHR is <100 bpm or >170bpm


Variability is absent (<3 bpm)
Decelerations are prolonged for >3 minutes OR late OR have complicated variables

When there is CTG abnormalities, the first steps to consider are reversing the possible causes of decelerations:

Maternal hypotension
Cord prolapse or compression
Uterine hypertonia
Scar dehiscence
Placental abruption
Rapid fetal descent

Management of prolonged decelerations or prolonged bradycardia includes the following steps:

Reposition the woman – e.g. lateral position


Checking maternal blood pressure and administration of bolus IV fluids in case of hypotension
Discontinuation of oxytocin or decreasing rate of infusion (if in progress)
Check the maternal blood pressure
Checking the maternal pulse to differentiate maternal pulse rate from the fetal heart rate (FHR)
If indicated, performing a vaginal exam to exclude cord prolapse or rapid cervical dilatation if the bradycardia persists.
Consideration of applying a fetal scalp electrode.
Assessment of abdominal tone to exclude a hypertonic uterus
Preparation e for assisted delivery (vacuum or forceps delivery) or emergency cesarean section if bradycardia does not resolve.

Uterine hyperstimulation and contractions due to excess cyntocinon is a common cause of prolonged deceleration; therefore, cessation of cyntocinon is the most appropriate
immediate step once prolonged decelerations are encountered on CTG. It is very likely that prolonged decelerations have been caused by excess uterine contraction as a result of
oxytocin (cyntocinon) infusion; therefore, cessation of cyntocinon is the most important immediate action to consider.

It is also important to check the maternal blood pressure as the mother’s hypotension is also a cause of prolonged deceleration. In case of hypotension, intravenous fluids should be
administered (option C) to correct maternal hypotension. CTG monitoring should be continued for assessment of fetal status and response to treatment.

Assisted vaginal delivery using forceps or vacuum (options A and B) or emergent cesarean delivery (option E) are measures to consider if the above conservative steps fail to
correct the underlying cause(s) of fetal compromise.

References

• WA Health Department – Obstetrics and Gynaecology Guidelines

1005 of 1943
• RANZCOG Intrapartum Fetal Surveillance Clinical Guideline
Last updated:
Time spent: QID:1268
2023-2-12

1006 of 1943
A 24-year-old woman presents to a GP clinic with concerns about breastfeeding. She gave birth to a healthy male child 20 days ago through an uneventful vaginal delivery. She is
worried that she is not producing milk to feed her baby and feels he is always hungry. Which one of the following conditions could be the most likely cause of decreased mild
production?

A. Poor positioning of the baby.

B. Exhaustion of the mother.

C. Short duration of feeding at each session.

D. Decreased frequency of breastfeeding.

E. Insufficient fluid intake by the mother.

Incorrect. Correct answer is D


45% answered correctly

Explanation:

Correct Answer Is D

Although most women are capable of producing more milk than their infants require, more than half of breastfeeding mothers perceive that their milk supply as inadequate. Results
of an online survey of 12 World Health Organization Western Pacific offices revealed that the second most commonly reported reason women did not breastfeed was "not having
enough milk" (17%).

For some mothers, this is a perceived rather than real issue. In such cases, providing information on what constitutes ‘normal’ newborn feeding behavior may provide reassurance.
For example, it is normal for infants to have frequent but small amount feeds during a particular time of the day. A change to an infant’s feeding pattern, such as increased frequency
of feeds, or a feeling of softer breasts, may concern the mother; however, these may be normal changes that are unrelated to decreased supply.

Of the causes that lead to genuine lowered milk production, decreased frequency of breastfeeding is the most common one. In the human mammary gland, lactation is under
autocrine control, in which the frequency and degree of milk removal by expression regulate an inhibitory peptide present in the milk. This means that if the milk is not removed, this
inhibitory peptide accumulates and subsequently decreases the synthesis of milk. If the milk is frequently removed, this inhibitory peptide does not accumulate, and milk synthesis
increases.

References

• https://www.racgp.org.au/afp/2016/august/overcoming-challenges-faced-by-breastfeeding-mothers/

• https://emedicine.medscape.com/article/979458-overview#a4

Last updated:
Time spent: QID:1283
2023-2-12

1007 of 1943
Which one of the following women is most likely to develop pre-eclampsia during pregnancy?

A. A 40-year-old woman, gravida 6, para 5, with no previous history of any pregnancy-related complications.

B. A 16-year-old primigravida woman.

C. A 35-year-old woman with primary hypertension.

D. A 25-year-old woman who is gravida 4, para 3.

E. A 25-year-old primigarvida with family history of pre-eclampsia in her mother and sister.

Incorrect. Correct answer is C


45% answered correctly

Explanation:

Correct Answer Is C

Pre-eclampsia is defined as hypertension and proteinuria after 20 weeks of pregnancy.

The following table summarizes the established risk factors for development of pre-eclampsia and their associated increased risks:

Condition Risk of preeclampsia


Previous preeclampsia 7 times more likely
Chronic hypertension 5 times more likely
Pre-existing diabetes 4 times more likely
Multiple pregnancy (e.g. twin pregnancy) 3 times more likely
Autoimmune disease e.g. SLE or antiphospholipid syndrome 3 times more likely
Nulliparity 3 times more likely
Family history of preeclampsia in first-degree relatives 3 times more likely
Age 40 or older 2 times more likely
Pre-existing kidney disease 2 times more likely
BMI>30 2 times more likely

According to the above table, a woman with chronic hypertension is five time more likely to develop preeclampsia compared to other options.

(Option A) In a 40-year-old woman with no previous obstetrical history, the risk of preeclampsia is almost doubled.

(Option B) Nulliparity almost triples the risk of preeclampsia. Age 16 years or younger has been considered as a risk factor but recent studies failed to establish a meaningful
relationship.

(Option D) This woman has no known risk factor for preeclampsia.

(Option E) In this woman, the family history of preeclampsia in her mother and sister, triples the risk of preeclampsia for her.

References

• Risk factors for preeclampsia at antenatal booking: systematic review of controlled studies

Last updated:
Time spent: QID:1323
2023-2-12

1008 of 1943
In the event of chord prolapse during labor, which one of the following findings is more likely on CTG monitoring?

A. Sinusoidal pattern.

B. Early decelerations.

C. Variable decelerations.

D. Late decelerations.

E. Fetal tachycardia.

Incorrect. Correct answer is C


45% answered correctly

Explanation:

Correct Answer Is C

The most common CTG abnormality associated with umbilical cord prolapse is variable decelerations. Other less common CTG findings include persistent fetal bradycardia and
prolonged decelerations of over one minute.

References

• RCOG - Greentop Guidelines - Umbilical Cord Prolpase

Last updated:
Time spent: QID:1339
2023-2-12

1009 of 1943
Dorothy, 21 years of age, presents to your GP clinic for advice regarding commencement of combined oral contraceptive pills (COCPs). You take a careful medical and family history,
which is significant for gastro-esophageal reflux disease (GERD) and premenstrual headaches. She has smoked 15-20 cigarettes a day for the past four years and drinks 2 to 3
glasses of wine every day. Her family history includes breast cancer of her mother diagnosed at the age of 53 years and hypertension in her father. On examination, her blood
pressure is 130/78 mmHg. She has a body mass index (BMI) of 28. She wants to know if there is anything wrong with her taking OCPs. In counselling her, which one of the following
will you discuss further as a possible absolute contraindication to COCPs?

A. Her drinking history.

B. Her family history of breast cancer.

C. Her premenstrual headaches.

D. Her BMI.

E. Her smoking history.

Incorrect. Correct answer is C


45% answered correctly

Explanation:

Correct Answer Is C

According to the UK Medical eligibility criteria for contraceptive use, the following are considered absolute risk factors for COCPs:

Breastfeeding and ≤6 weeks postpartum


Smoker ≥35 year and ≥ 15 cigarettes/day
Presence of multiple risk factors for CVD including older age, smoking, diabetes, hypertension
Hypertension with systolic ≥160mmHg or diastolic ≥95mmHg
Vascular disease
Major surgery with prolonged immobilization
Current or past history of venous thromboembolism (VTE)
Known thrombogenic mutations (Factor V Leiden, Prothrombin mutation, Protein S, Protein C and Antithrombin deficiencies)
Migraine with aura
Current or past history of Ischemic Heart Disease (IHD);
Complicated valvular heart disease
Diabetes complicated by nephropathy, retinopathy or vascular disease
Breast cancer
Severe Liver disease including cirrhosis hepatocellular adenoma and hepatoma
Raynaud’s with lupus anticoagulant
SLE with antiphospholipid antibodies

In the history, Dorothy has mentioned headaches before her menses. The relation of the headaches to her periods makes migraine a possibility, as one of the triggers for migraine is
hormonal changes in the premenstrual period. If that is the case, further scrutinizing the history is required for characteristic features of aura. If her headaches are associated with
aura, COCPs will be absolutely contraindicated for her.

(Option A) Alcohol use is not a contraindication to COCPs use.

(Option B) While personal history of breast cancer is an absolute contraindication to COCPs, family history of breast cancer is not associated with increased risk of breast cancer in
COCPs users; hence, the history of breast cancer in Dorothy’ mother does not preclude use of COCPs for her.

(Option D) Overweightness and obesity alone are not contraindications to use of combines oral contraceptive pills, unless associated with other cardiovacular risk factors such as
smoking, diabetes, etc.

(Option E) COCPs are contraindicated in woman who are 35 years or older AND smoke ≥ 15 cigarettes/day. Although Dorothy smokes 15-20 cigarettes a day, she is 21 years;
therefore, her smoking history is not an absolute contraindication.

References

• RANZCOG - Combined Hormonal Contraceptives

Last updated:
Time spent: QID:1351
2023-2-12

1010 of 1943
A 55-year-old woman presents to your GP practice with complaints of dyspareunia, dysuria, and vaginal itch. She has been menopausal since the age of 50 years. She relates that
sexual intercourse has been becoming progressively painful for the past 6 months. She describes the pain as burning. The vaginal itching and dysuria has developed in the past few
weeks. Her past medical history is unremarkable, and she is currently on no medications except for supplemental vitamins. She denies any hot flushes. On examination, atrophy of
the labia minora is evident, but labia majora and other parts of the vulva are normal. Vaginal exam reveals a rather stenotic vagina with pale and atrophic walls but no vaginal or
cervical inflammation or discharge. Which one of the following would be the most appropriate first-line treatment for her?

A. Estrogen cream.

B. Topical antifungal agents.

C. Conization.

D. Hydrocortisone cream.

E. Topical antibiotics.

Correct
45% answered correctly

Explanation:

Correct Answer Is A

This woman has atrophic changes of the vagina 5 years after menopause. For her, atrophic vaginitis is the most likely diagnosis.

Atrophic vaginitis results from estrogen deficiency and is experienced by almost 50% of postmenopausal women. Symptoms include itching, burning, dryness, and irritation, all of
which can lead to dyspareunia. On the other hand, a decline in estrogen alters the vaginal flora, leading to bacterial overgrowth. This can cause bacterial vaginosis and vaginal
discharge.

Declining estrogen also affects the urinary tract, leading to thinning of the bladder and urethral linings. This can result in chronic dysuria and an increased incidence of urinary tract
infections. Estrogen creams are the mainstay of treatment for atrophic vaginitis. As this may take a while to come to effect, lubricants can be used to facilitate sexual intercourse.

(Option B) Antifungal agents were the treatment of choice if vaginal candidiasis would have been the diagnosis. The condition most commonly presents with itching. Burning and
dyspareunia can also be present; however, an inflamed vaginal with or without cheesy white discharge rather than an atrophic vagina would have been expected on examination.

(Option C) Conization has no role in treatment of atrophic vaginitis. It is often used to treat premalignant and malignant cervical lesions.

(Option D) Potent to very potent topical corticosteroids are used, as first-line therapy, for treatment of lichen sclerosus (LS). This woman is not likely to have LS, as LS does not
involve vagina. On the other hand, hydrocortisone is a weak corticosteroid and not effective in treatment of LS.

(Option E) Topical antibiotics (e.g., metronidazol) may be used for concomitant bacterial infection or vaginosis in atrophic vaginitis, but it is not effective for atrophic vaginitis itself.

References

• MJA – Management of common vulval conditions

• RACGP - Localized provoked vestibulodynia (vulvodynia): assessment and management

• Medscape – Treating Atrophic Vaginitis

Last updated:
Time spent: QID:1391
2023-2-12

1011 of 1943
Hanna is a 32-year-old patient of yours, who have decided to start combined oral contraceptive pills (COC) for contraception, but she is worried about the risk of cancer because she
has heard from her friends and read on different websites that COC increase the risk of some cancers. In consulting her regarding long-term use COC, which one of the cancers will
you mention to be of highest risk for her if she takes COC in the long run?

A. Endometrial cancer.

B. Colon cancer.

C. Breast cancer.

D. Cervical cancer.

E. Ovarian cancer.

Incorrect. Correct answer is D


45% answered correctly

Explanation:

Correct Answer Is D

Studies have shown that COCs has different effects on cancers.

COCs cease ovulation and is associated with decreased risk of ovarian cancer (option E). women who have ever used COCs have a 30% to 50% lower risk of ovarian cancer than
women who have never used COCs. This protective effect is increased with the length of time COCs are used, and last up to 30 years after cessation. This effect is caused by
reducing the number of ovulations a woman experiences in her lifetime, and consequently reducing exposure to naturally occurring female hormones as a risk factor.

The risk of colorectal cancers (option B) is also decreased by 15% to 20% in women who have ever taken COCs compared to women never on COC. This effect is caused by
reduction in bile acids in women on COCs.

Another cancer with decreased risk associated with COCs use is endometrial cancer (option A). There is at least 30% reduction in risk of developing endometrial cancer in women
who have ever used COCs compared to those who have not. The risk reduces more with prolonged use of COCs. This reduction is even more significant in long-time users of oral
COCs who are smokers, obese, or physically inactive. Such protective effect is caused by suppression of endometrial proliferation as a result of naturally occurring sex hormones
when a woman uses COCs (synthetic sex hormones).

COCs, however, are associated with an increased risk of cervical cancer. Women who have used oral COCs for 5 or more years have a higher risk of cervical cancer than women who
have never used COCs. The longer a woman uses COCs, the greater the increase in her risk of cervical cancer will be. One study found a 10% increased risk for less than 5 years of
use, a 60% increased risk with 5–9 years of use, and a doubling of the risk with 10 or more years of use. This risk declines over time after cessation of COCs. Such effect is probably
due to changes in the susceptibility of cervical cells to persistent infection with high-risk HPV types which are virtually the cause of all cervical cancers. Another hypothesis is that
women on COCs have more frequent unprotected sex which increases the risk of HPV infection.

COCs are also associated with a slight increase in risk of breast cancer (option C) compared to women who have never taken COCs. The risk declines after cessation, and after 10
years of cessation, will be the same as that of women who have never used COCs. Generally, the benefits of preventing unwanted pregnancy and its harms outweighs the slightly
increased risk of breast cancer associated with COCs.

In consulting Hanna, she should be told the main concern would be the increased risk of cervical cancer if COCs are taken longer than 5 years as the main concern, and the fact that
there is a just slightly increased risk of breast cancer according to studies. COCs would not be appropriate for her in the presence of any personal history of breast cancer but even
with breast cancer history in her first degree relatives, she can still take COCs.

It should be explained to her than in fact OCPs decrease her risk of developing ovarian, endometrial and colon cancers.

References

• National Cancer Institute – Oral contraceptives and cancer risk

• Cancer Council - Oral contraceptives

Last updated:
Time spent: QID:1455
2023-2-12

1012 of 1943
Miranda is a 33-year-old patient of your, who has presented for cervical screening test today. Her previous tests were all normal. She is married, has never been pregnant, and started
her sexual relationships from the age of 17 years. She has been using oral contraceptive pills (OCPs) for the past 8 years as the means of contraception. She does not smoke but
drinks alcohol on social occasions. She is obese with a BMI of 31 kg/m2. In consulting her, which one of the following will you mention as the most significant risk factor for
development of cervical cancer in the future?

A. Sexual relationship.

B. Alcohol.

C. Obesity.

D. Prolonged use of OCPs.

E. Nulliparity.

Incorrect. Correct answer is D


45% answered correctly

Explanation:

Correct Answer Is D

Some conditions are associated with increased risk of developing cervical cancer. Of these, the most important one is infection with high risk human papilloma virus (HPV) types,
such as HPV type 16 and 18. Virtually, all the cases of cervical cancers are caused by HPV infection. In the absence of HPV infection, no cervical cancer occurs. Other risk factors
predispose to the development of cervical cancer only in the presence of HPV infection.

All women, who have ever been sexually active, are at an average risk of developing cervical cancer through receiving the HPV infection either by sexual intercourse or skin to skin
contact. However, sexual relationship (option A) without contracting HPV infection is not a risk factor for cervical cancer.

OCP use in another important risk factor for cervical cancer. Women, who have used oral OCPs for 5 years or more \, have a higher risk of cervical cancer than women who have
never used OCPs.

The longer a woman uses oral contraceptives, the greater the increase in her risk of cervical cancer. One study found a 10% increased risk for less than 5 years of use, a 60%
increased risk with 5–9 years of use, and a doubling of the risk with 10 or more years of use. Such effect is probably due to changes in the susceptibility of cervical cells to
persistent infection with high-risk HPV types. Another hypothesis is that women on OCPs have more frequent unprotected sex, with consequent increased the risk of HPV infection.
This should be mentioned to Miranda as the most significant risk factor among other options.

Alcohol (option B) have not shown to be associated with an increased risk of cervical cancer. Although obesity and overweightness have been implicated for increasing the risk of
cervical cancer, such association is not significant.

Nulliparity (option E) is a protective factor rather than a risk factor. Studies suggest that giving birth to 5 or more children may slightly increase the cervical cancer for women who
have HPV infection.

TOPIC REVIEW

Important risk factor for cervical cancer are as follows:

Persistent infection with high-risk HPV types (the most significant risk factor – without HPV infection, there is no risk of cervical cancer even in the presence of multiple
other factors)
Smoking
Lack of regular cervical screening tests
Age – cervical cancer risk increases after the age of 35 years
Prolonged use of OCPs (more than 5 years)
Immunosuppression
Previous screening abnormalities or previous cervical cancer
Multiparity (5 or more)
Exposure to diethylstilbestrol (DES)

References

• RACGP – Guidelines for preventive activities in general practice (The Red Book) – cervical cancer

• Cancer Australia – Cervical cancer: what are the risk factors for cervical cancer?

Last updated:
Time spent: QID:1459
2023-2-12

1013 of 1943
A 27-year-old woman comes to antenatal clinic at 10 weeks gestation. It is her first pregnancy. She smokes 2 packs of cigarettes a day and drinks 5-7 standard drinks every day. Recently, she has started using crack cocaine and heroin. She mentions
that she is going through a lot these days because her employer found out about her addiction and fired her. She is very anxious and worried about her future and cannot sleep well. This is why she uses diazepam every night. Which one of the
following in the history may have the worst effect on the fetus?

A. Cocaine.

B. Heroin.

C. Cigarette smoking.

D. Diazepam.

E. Alcohol.

Correct
45% answered correctly

Explanation:

Correct Answer Is A

Although cocaine is not associated with congenital anomalies, its use is associated with increased risk of fetal intracranial hemorrhage with the worst possible outcome if it occurs.

(Option B) Heroine and other opiates can be associated with the following complications:

Low birth weight usually associate with intrauterine growth restriction(IUGR)


Preterm labor
Drug withdrawal for the neonate (neonate abstinence syndrome)
Increased risk of sudden infant death syndrome (SIDS)

(Option C) Smoking is associated with multiple complications usch as IUGR, preterm labor, limb reduction, and GI malformations; however, none of them is as life-threatening as fetal intracranial hemorrahge should it occur.

(Option D) The health risks of benzodiazepines in pregnancy have not been clearly established. There have been inconsistent reports of teratogenic effects associated with fetal exposure to benzodiazepines. Regular benzodiazepine use in pregnancy
may be associated with a neonatal abstinence syndrome, which may be of delayed onset.

(Option E) Although alcohol use is associated with fetal alcohol syndrome (FAS), the amount needed for occurrence of FAS often has to be more than 12 standard drinks (120gr) per day. With only 5 to 7 standard drinks per day, the likelihood of FAS
is insignificant.

NOTE - There is a long list of therapeutic medications and recreational drugs that can cause neonatal malformations. Malformations are caused by teratogenic effects of these drugs. Bear in mind that malformations are different from perinatal
complications. For example, marijuana can be associated with several perinatal risks, but there are no associated congenital malformations. Alcohol, on the other hand, not only causes complications such as intrauterine growth restriction (IUGR),
but it use can also give rise to neonatal alcohol syndrome (FAS), characterized by congenital malformations such as midfacial hypoplasia, upturned nose, long philtrum and low-set ears.

References

• Llewellyn – Jones Fundamentals of Obstetrics and Gynaecology Elsevier – Mosby 9th Editionhttp://www.merckmanuals.com/professional/gynecology_and_obstetrics/drugs_in_pregnancy/drugs_in_pregnancy.html

• http://www.sahealth.sa.gov.au/wps/wcm/connect/fad9

Last updated:
Time spent: QID:135 2023-2-12

1014 of 1943
A couple present to your clinic for evaluation of infertility because they have failed to conceive after one year of unprotected sexual intercourse in a timely fashion at mid-cycles. The
female is 45 years old and has three children from her former spouse. She has regular periods at 30-day intervals. The male is 50 years old and has never fathered a child. They have
been married for five years now. Which one of the following could be the most likely cause of the infertility?

A. Maternal age.

B. Paternal age.

C. Semen abnormalities.

D. Anovulatory cycles.

E. Pelvic adhesions from previous pregnancies.

Incorrect. Correct answer is C


45% answered correctly

Explanation:

Correct Answer Is C

Infertility is defined by the World Health Organisation (WHO) as the inability of a couple to conceive after 12 months of regular unprotected intercourse in women less than 35 years
of age; and after six months of regular unprotected intercourse in women 35 years of age or older; or the inability to carry pregnancies to live birth. Based on this definition, this
couple are infertile.

Infertility can be caused by a male factor, female factor, or both. Causes of infertility can be categorized as follows:

Female infertility factors:

Hormonal disorders
Damaged or blocked fallopian tubes
Endometriosis
Excessively thick cervical mucus

Male infertility factors:

Semen abnormalities (volume, PH, count, morphology, motility, vitality, etc.)


Any obstruction in the course of sperms out and/or ejaculation failure.

In general, advances maternal age is the most important factor influencing infertility. This is followed by male infertility as the second most important factor.

In this case, the female partner has already given birth to three children via vaginal delivery. There has been no risk factor in the history, such as caesarean section or
instrumentation, to make pelvic adhesions from previous pregnancies (option E) a likely explanation to the infertility.

She is 45 years old and, as mentioned before, advanced maternal age (option A) has the most important role in infertility due to factors such as poor follicle reserve and anovulatory
cycles, decreased quality of the egg, and undesirable endometrium for the fertile egg to implant. This woman has regular periods at 30-day intervals indicating that she very likely to
be still ovulating because anovulatory cycles (option E) are associated with irregular periods as the most common symptom.

Although having ovulatory cycles does not exclude problems such as poor-quality eggs or endometrium for fertility due to advanced age, the fact that the male partner has never
fathered a child makes male-infertility, and more specifically semen abnormalities, the most likely explanation for infertility in this couple.

Paternal age (option B) is a risk factor for semen abnormalities, but again since he has never fathered a child before (even at younger ages), semen abnormalities independent of
the paternal age could be a better and more likely explanation.

For this couple, the first investigation to consider should be a semen analysis. Once male infertility, as the most likely explanation in this scenario, is excluded, attention should be
turned to female factors.

References

• RACGP – AFP – We’re having trouble conceiving

Last updated:
Time spent: QID:1516
2023-2-12

1015 of 1943
Mary, 27 years old, is admitted to the Maternity Ward after her labor pain started. After amniotomy, she is placed in the left lateral position and on supplemental oxygen by nasal
canula, and intravenous fluids and Syntocinon® (oxytocin) infusion is started. A while later and during fetal heart auscultation, fetal heart rate (FHR) of 70 bpm is noted. CTG is
applied which reveals a baseline fetal heart rate of 140 bpm dropping to 70 bpm periodically with each episode of bradycardia lasting approximately three minutes. Which one of the
following is the most appropriate next step in management?

A. Fetal scalp blood sampling.

B. Stop Syntocinon.

C. Immediate cesarean delivery.

D. Continuous CTG monitoring.

E. Reposition to supine.

Incorrect. Correct answer is B


45% answered correctly

Explanation:

Correct Answer Is B

The FHR pattern on CTG is typically interpreted as reassuring or non-reassuring. The presence of a reassuring pattern indicates that there is a minimal possibility of fetal acidemia at
that point in time. It does not predict the future fetal status because tracing pattern can change.

A CTG pattern is reassuring if all of the following are present:

A baseline fetal heart rate of 110 to 160 bpm


Absence of late or variable FHR decelerations
Moderate FHR variability (6 to 25 bpm)
Age-appropriate FHR accelerations

Any other CTG pattern except the above is considered non-reassuring and prompt action should be taken. When a non-reassuring CTG is encountered, management includes
the following steps:

Calling for assistance


Administration of oxygen via a tight-fitting face mask
Changing the maternal position to left lateral or knee chest (to remove the pressure of the gravid uterus from the inferior vena cava and improve maternal circulation and
consequently placental circulation)
Administration of fluids (bolus)
Discontinuation of any uterine stimulant (e.g. oxytocin)
Continuous CTG monitoring
Performing a vaginal examination (to exclude cord prolapse) and fetal scalp stimulation
Determination and correction of the cause of the non-reassuring CTG pattern if possible
Considering tocolysis if indicated
Determining whether operative intervention (e.g., cesarean delivery) is warranted and, if so, how urgently it is needed

In this scenario, the presence of fetal bradycardia (FHR<110 bpm) makes the CTG non-reassuring and expeditious action and investigation (as mentioned above) is warranted. Mary
has already been placed in the left lateral position and is receiving oxygen and intravenous fluids. Of the options, cessation of oxytocin infusion is the most appropriate next step in
management because it is likely that oxytocin has led to uterine hyperstimulation. Uterine hyperstimulation is defined as tachysystole (more than five active labor contractions in 10
minute) or uterine hypertonus (contractions lasting more than two minutes or contractions occurring within 60 seconds of each other) in the presence of fetal heart rate
abnormalities.

(Option A) Fetal scalp blood sampling is indicated to assess the presence and severity of fetal acidosis if CTG remains non-reassuring despite initial measures.

(Option C) Immediate cesarean delivery is considered if there are persisting CTG abnormalities indicative of fetal jeopardy. Unless conditions dictate otherwise, a fetal scalp blood
sample is performed to evaluate fetal acidemia prior to making such decision.

(Option D) Continuous CTG monitoring is indicated in all cases of non-reassuring CTGs after initial measures are undertaken. CTG monitoring is a very important step in this case,
but does not take precedence over cessation of oxytocin.

(Option E) Placing Mary in supine position will cause the pregnant uterus to compress the inferior vena cava and reduce the venous return which will lead to decreased maternal
cardiac output. This will result in decreased blood flow to the placenta and deteriorate the fetus’s condition.

References

• RANZCOG Intrapartum Fetal Surveillance Clinical Guideline

Last updated:
Time spent: QID:1528
2023-2-12

1016 of 1943
A 25-year-old woman presents at 20 weeks gestation for an antenatal check-up. Laboratory studies reveal a platelet count of 90,000/mm3 with no other abnormalities in test results.
She has no symptoms and physical examination is unremarkable. Which one of the following could be the most likely diagnosis?

A. Idiopathic thrombocytopenia.

B. Immune thrombocytopenia.

C. Gestational thrombocytopenia.

D. Disseminated intravascular coagulation (DIC).

E. Preeclampsia.

Incorrect. Correct answer is C


45% answered correctly

Explanation:

Correct Answer Is C

Thrombocytopenia is the second most common hematological finding during pregnancy after anemia. Thrombocytopenia can have many etiologies, some of which are not specific
to pregnancy.

Causes of thrombocytopenia in pregnancy include:

Pregnancy-related

Acute fatty liver


Gestational thrombocytopenia
HELLP syndrome
Hypertensive disorders such as preeclampsia/eclampsia

General

Autoimmune conditions (SLE, antiphospholipid syndrome)


Bone marrow disorders
Disseminated intravascular coagulation
Drugs
Heparin induced thrombocytopenia
Hypersplenism
Inherited, Type IIB Von Willebrand disease
B12, or folate deficiency
Immune (idiopathic) thrombocytopenia
Pseudothrombocytopenia
Secondary immune thrombocytopenia due to viral infections (e.g., HIV, Hep C, CMV, EBV)
Thrombotic microangiopathies ( TTP/HUS)

Of these, gestational thrombocytopenia is the most common cause of thrombocytopenia in pregnancy, followed by ITP as the second most common cause.

Gestational thrombocytopenia (incidental thrombocytopenia) occurs in approximately eight percent of all pregnancies and accounts for more than 70% of cases with
thrombocytopenia in pregnancy. Although the pathophysiology of gestational thrombocytopenia is unknown, it is thought to be related to hemodilution, increased platelet
consumption, and increased platelet aggregation (as a result of increased levels of thromboxane A2). Platelet count may be lower in women with twins perhaps due to a greater
increase in thrombin generation.

In summary, gestational thrombocytopenia is defined by the following five criteria:

1. Mild and asymptomatic thrombocytopenia


2. No past history of thrombocytopenia (except possibly during a previous pregnancy)
3. Occurrence during late gestation (from mid-second trimester to third trimester)
4. No association with fetal thrombocytopenia
5. Spontaneous resolution after delivery

This woman has an isolated and asymptomatic mild thrombocytopenia. This picture is consistent with either gestational thrombocytopenia or ITP. Given the high prevalence of
gestational thrombocytopenia, it could be the most likely diagnosis.

(Options A and B) Immune thrombocytopenia and idiopathic thrombocytopenia are in fact two names for one condition characterized by isolated thrombocytopenia of autoimmune
origin that can present similar to gestational thrombocytopenia; however, gestational thrombocytopenia is much more common during pregnancy and more likely to be the
underlying cause of this woman’s presentation.

(Option D) DIC is a serious life-threatening condition presenting with low-platelet count, intravascular coagulation, and thrombus formation potentially in every organ system. The
two most common conditions associated with DIC during pregnancy are placental abruption and fetal demise. Laboratory studies show low levels of all of coagulation factors. An
asymptomatic isolated thrombocytopenia is not a picture consistent with DIC.

(Option E) Although thrombocytopenia is a common feature of pregnancy-related hypertensive disorders such as pre-eclampsia or HELLP syndrome. With pre-eclampsia other
manifestations clinical or laboratory findings would be expected (e.g., proteinuria, hypertension, elevated liver enzymes, headache, abdominal pain, etc.) An isolated mild
thrombocytopenia in a completely asymptomatic pregnant woman is unlikely to have been caused by pre-eclampsia.

1017 of 1943
NOTE - Compared to non-pregnant women with thrombocytopenia, pregnant women with thrombocytopenia tend to have fewer bleeding complications due to the
procoagulant state induced by increased levels of fibrinogen, factor VIII and von Willebrand factor, suppressed fibrinolysis and reduced protein S activity. There are several
other pregnancy-related conditions that can also lead to thrombocytopenia. Thrombocytopenia in pregnancy is a common reason for hematology consultation.

References

• Medscape – Thrombocytopenia in Pregnancy

• UpToDate – Thrombocytopenia in pregnancy

Last updated:
Time spent: QID:1530
2023-2-12

1018 of 1943
A 31-year-old woman presents with complaints of 10 kg weight loss and heavy periods for the past six months. However, her periods were and still are regular occurring at intervals
of 32 days lasting for five days with no increased pain or discomfort. The number of pads she uses indicates blood loss of more than 80 mL per cycle. She has no known medical
condition. The rest of the physical examination, including pelvic exam is inconclusive. An office urine pregnancy test is negative. Which one of the following would be the most
appropriate investigation to consider first?

A. Transvaginal ultrasonography.

B. Abdominal ultrasonography.

C. Thyroid stimulating hormone (TSH) level.

D. Prolactin level.

E. Follicle stimulating hormone (FSH) level.

Correct
45% answered correctly

Explanation:

Correct Answer Is A

This woman has presented with an altered pattern of menstrual bleeding in from of increased menstrual flow at regular intervals, namely ‘menorrhagia’. Menorrhagia is a form of
abnormal uterine bleeding (AUB).

Normal menstrual periods last 3–6 days and is associated with blood loss of up to 80 ml. Menorrhagia is defined as menstrual periods lasting more than 7 days and/or
involving blood loss greater than 80 mL.

AUB affects 9-14% of women. AUB generally can be divided into anovulatory and ovulatory patterns. Ovulatory pattern of AUB is characterized by abnormal volume or length of
blood loss at regular intervals, whereas in anovulatory pattern the bleeding is irregular and unpredictable (no regular periods)

Ovulatory pattern is usually caused by uterine problems such as leiomyomas, endometriosis, adenomyosis, polyps, etc. as opposed to anovulatory pattern that is a result of
hormonal problems such as polycystic ovarian syndrome (PCOS), hypothyroidism, hyperthyroidism, hyperprolactinemia, or Cushing syndrome.

The approach to women with ovulatory versus anovulatory pattern is different. It is recommended that if the pattern cannot be specified, the patient be assessed as having irregular
bleeding because this pattern includes investigation for endometrial hyperplasia/cancer for more diagnostic safety.

In approach to every woman of reproductive age presenting with AUB, the most important to consider and exclude first is pregnancy regardless of the pattern. If anemia is
suspected based on the history or clinical findings, a full blood exam (FBE) is indicated. A serum TSH to exclude thyroid disease is indicated if the history and/or physical
examination raise suspicion against thyroid disease, especially hypothyroidism. Routine use of TSH in every woman presenting with AUB is not recommended.

Other investigations depend on the pattern and whether it is ovulatory or anovulatory. Since this woman has menorrhagia, indicating ovulatory AUB and the high possibility of
structural anomalies such as fibroids, adenomyosis, polyps, etc., a transvaginal ultrasound is very likely to pick up the etiology and is the most important initial diagnostic approach
to consider first and go with. Abdominal ultrasound (option B) is not as accurate as transvaginal ultrasound for this purpose yet can be used if transvaginal ultrasound is not
possible (e.g., adolescent girls).

NOTE – A very significant finding in the history is the weight loss. For significant weight loss in this woman in the presence of AUB, the two most important differential
diagnoses to think of are hyperthyroidism and cancer. Of these two, the greatest concern is cancer, especially endometrial cancer. Even if cancer is the case, transvaginal
ultrasound is the initial option of choice for assessment of endometrial thickness and other abnormalities of the uterine cavity and adjacent structures. In the presence of any
endometrial abnormality, the patient will require endometrial curettage and biopsy. Hyperthyroidism is more likely to be associated with amenorrhea or oligomenorrhea than
menorrhagia; however, thyroid function tests can be considered as well once cancer as a main concern is safely excluded.

Hormonal assay like FSH (option E), LH, and prolactin (option D) are investigations to consider in patients with anovulatory AUB, where hormonal derangements are the most
important underlying etiology to consider and think of.

References

• Cancer Australia - Abnormal Vaginal Bleeding in Pre- and Post-menopausal Women

• Clinical Care Standard - Heavy Menstrual Bleeding Clinical Care Standard

Last updated:
Time spent: QID:1542
2023-2-12

1019 of 1943
Angelina, accompanied by her mother, is in your office for consultation. She is 15 years old, and is concerned because she has not started her periods yet. She says: “I am about the
same height as everybody else in my class at school. Their periods have started but mine not.” Which one of the following is the most appropriate initial question to ask her?

A. When she had her breast buds developed.

B. When she had her pubic hair.

C. Whether she gets cyclic abdominal pain.

D. Whether she had accelerated growth.

E. When she had her axillary hair.

Correct
45% answered correctly

Explanation:

Correct Answer Is A

The complaint presented in the scenario is primary amenorrhea. Primary amenorrhea is defined as the absence of menses after the age of 16 in the presence of normal growth and
secondary sexual characteristics, or 13-14 in the absence of secondary sexual characteristics. In case breast budding occurs before the age of 10, amenorrhea is defined as absent
periods within the next 5 years.

Puberty in girls start with breast development followed by the growth spurt, growth of axillary and pubic hair, and finally menstruation. Breast above Tanner II stage indicates the
commencement of puberty and exposure to estrogen.

In this case, it is very important to know whether puberty has started and if so, how long has she been through puberty. This can be known by simply asking if she has had any
breast developments as the most appropriate question. Other question are relevant if puberty has already begun indicated by breast development.

References

• NSW Health – Amenorrhoea

• RACP – Investigating primary and secondary amenorrhoea

Last updated:
Time spent: QID:1552
2023-2-12

1020 of 1943
A 26-year-old woman with history of chronic immune thrombocytopenic purpura (ITP) presents to your clinic and seeks advice regarding pregnancy. She has a platelet count of
70000/mm3. Which one of the following is the correct statement in counselling her?

A. She should have splenectomy before pregnancy.

B. She should avoid pregnancy in the next 2 years.

C. She can become pregnant.

D. If she becomes pregnant, the mode of delivery should be cesarean section.

E. She cannot become pregnant while the platelet count is below 70000/mm3.

Incorrect. Correct answer is C


45% answered correctly

Explanation:

Correct Answer Is C

While uncommon, immune thrombocytopenia (ITP) is an important cause of thrombocytopenia in pregnant women either as a pre-existing condition or occurring at any time during
pregnancy.

ITP developed in pregnancy is important to be distinguished from ‘incidental’ or gestational thrombocytopenia, which, is responsible for up to 80% of cases of thrombocytopenia in
pregnant women, or from more serious conditions such as HELLP syndrome.

Gestational thrombocytopenia usually causes an approximately 10% decrease in platelet counts and is usually characterized by counts that are above 70,000/mm3. When platelets
are lower than that, it is mostly ITP.

A diagnosis of ITP is usually made in 1-4% of cases of thrombocytopenia. ITP, either pre-exiting or developed during pregnancy, does not prevent a woman from becoming pregnant
or safely delivering a healthy baby. The best advice for this woman is that she can become pregnant and ITP does not preclude vaginal delivery; however, the pregnancy is
categorized as high risk and she will need extra care and probably treatment, especially around the delivery.

In pregnant women with ITP, treatment is usually not necessary as long as the platelet count is above 30,000, unless there is bleeding or easy bruising. For pregnant women with
symptomatic platelet counts of above 30,000, or women with platelet count of below 30,000 regardless of symptoms, treatment either with prednisolone or intravenous
immunoglobulin (IVIG) is required due to significant concern of bleeding, especially uterine bleeding.

It is important to maintain a platelet count that allows the pregnant woman with the condition to go through delivery. This safe count is at least 50,000. For regional anesethesia, a
count between 70,000 and 100,000 is often demanded.

References

• Medpage Today - Clinical Challenges: ITP in Pregnancy

• Immune Thrombocytopenia in Pregnancy

Last updated:
Time spent: QID:1596
2023-2-12

1021 of 1943
Adele is a 23 years old primigravida woman who has a spontaneous abortion at 12 weeks gestation 2 weeks ago. She has presented to you because she wants to have another
pregnancy as soon as possible as she thinks this could help her husband and her get over with their loss. She asks you when she can become pregnant again. Which one of the
following would be the best advice?

A. She should not become pregnant for at least 6 months and should be on OCPs for now.

B. She can start trying to conceive again immediately if she feels fit for that.

C. She can conceive after 12 months.

D. She can conceive after 3 months.

E. She can conceive after 2 menstrual cycles.

Incorrect. Correct answer is B


45% answered correctly

Explanation:

Correct Answer Is B

Despite the emotional pain of pregnancy loss, most couples go onto having another pregnancy. It is recommended that women with pregnancy loss wait for one menstrual period
after a pregnancy loss and then try the next pregnancy when they are feeling emotionally strong and ready.

For some, this could be 2 months, for others 2 years. It is generally recommended that women with pregnancy loss can conceive immediately after they feel fit both physically and
emotionally to have another pregnancy. For this woman, next pregnancy can be planned once she feels emotionally ready.

The following are recommendations for women who are planning conceiving again after a pregnancy loss:

Reducing the chance of another pregnancy loss by starting the next pregnancy as healthy as possible. Some measures are quitting smoking and alcohol, achievement of
a healthy weight through exercise and diet and adoption of healthy lifestyle in general.
Checking on any long term health problems such as depression, high blood pressure, or diabetes. Medicines might need to be altered or more emphasis placed on non-
medical treatments after appropriate advice from health professionals.
Having a health check-up before the next pregnancy.

For couples with a lost pregnancy, peer support can be invaluable. Other mothers who have suffered pregnancy loss can have a real understanding of a woman’s experience as she
goes through another pregnancy.

References

• RANZCOG – Pregnancy loss

Last updated:
Time spent: QID:1606
2023-2-12

1022 of 1943
A 27-year-old woman is in labor at 39 weeks gestation when passage of meconium is noted. A cardiotocography (CTG) is arranged that shows a fetal heart rate (FHR) of 149 bpm, a
beat-to-beat variability of 15, no acceleration, and no deceleration. Which one of the following should be the next best step in management?

A. Fetal scalp blood sampling as there is a 10% chance of hypoxia.

B. Fetal scalp blood sampling as there is a 50% chance of hypoxia.

C. Fetal scalp blood sampling as there is 75% chance of hypoxia.

D. Emergency cesarean section.

E. Close monitoring until delivery as there is no abnormality.

Incorrect. Correct answer is E


45% answered correctly

Explanation:

Correct Answer Is E

Baseline fetal heart rate (FHR) is the mean level of the FHR when this is stable, excluding accelerations and decelerations. It is determined over a period of 5-10 minutes, expressed
as beats per minute (bpm). Preterm fetuses tend to have values towards the upper end of the normal range.

Baseline variability is the minor fluctuation in baseline FHR. It is assessed by estimating the difference between the highest peak and lowest trough of fluctuation in one-minute
segments of the fetal hear rate trace. Baseline variability is categorized as follows:

Normal variability: 6-25 beats per minute


Reduced variability: 3-5 beats per minute
Absent variability: <3 beats per minute
Increased (salutatory) variability: > 25 beats per minute.

Accelerations are transient increases in FHR of 15 bpm or more above the baseline that last at least 15 seconds. Accelerations in preterm fetuses may be of lesser amplitude and
shorter duration.

Decelerations are transient episodes of decreased FHR below the baseline of more than 15 bpm lasting at least 15 seconds. The specific features of the deceleration inform the
classification. Decelerations should never be described as ‘unprovoked’; the fetus will not decelerate its heart rate without physiological provocation. Uterine activity, even in its
mildest form, will result in decelerations in a fetus whose oxygenation is already compromised.

Decelerations are categorized as:

Early decelerations - benign and associated with the sleep cycle and often in the range of 4-8 cm of cervical dilatation. They are caused by head compression and in general are a
normal physiological response to a mild increase in intracranial pressure. Importantly they are uniform in shape and start and finish with the contraction. They may be said to mirror
the contraction.

Variable decelerations - repetitive or intermittent decreasing of FHR with rapid onset and recovery. Time relationships with contraction cycle may be variable but most commonly
occur simultaneously with contractions. The significance of variable decelerations depends on the overall clinical picture and specific features of the decelerations themselves, as
well as other features of the CTG. Variable decelerations in association with other non-reassuring or abnormal features change the category of the deceleration to ‘complicated’.

Prolonged decelerations - a decrease of FHR below the baseline of more than 15 bpm for longer than 90 seconds but less than 5 minutes.

Late decelerations - defined as uniform, repetitive decreasing of FHR with, usually, slow onset mid to end of the contraction and nadir more than 20 seconds after the peak of the
contraction and ending after the contraction. Late decelerations are caused by contractions in the presence of hypoxia. This means that they will occur with each contraction and
the fetus is already hypoxic. There will be no features of a well oxygenated fetus, like early or typical variable decelerations, normal baseline variability or shouldering. They start after
the start of the contraction and the bottom of the deceleration is more than 20 seconds after the peak of the contraction. Importantly, they return to the baseline after the
contraction has finished. In the hypoxic fetus, this will include decelerations of less than 15 bpm (and occasionally less than 5 bpm).

A CTG is considered abnormal and demands further action or investigations if:

There is at least two of the following:

Baseline FHR is between 100-109 bpm or between 161-170 bpm


Variability of FHR is reduced (3-5 bpm for >40 minutes)
Decelerations are variable without complicating features

OR

There is any of the following:

Baseline FHR is <100 bpm or >170bpm


Variability is absent (<3 bpm)
Decelerations are prolonged for >3 minutes OR late OR have complicated variables

This CTG has none of the above and is completely normal; therefore, close monitoring until the delivery is all that is needed at this stage.

References

1023 of 1943
• WA Health Department – Obstetrics and Gynaecology Guidelines

• RANZCOG Intrapartum Fetal Surveillance Clinical Guideline


Last updated:
Time spent: QID:1622
2023-2-12

1024 of 1943
A 29-year-old woman presents to your practice with complaints of annoying pain and cramping with her periods as well as slight increase in her menstrual flow. She has had this
problem since she started menstruating and investigations revealed no apparent cause for the presentation. Recently, she was advised to take combined oral contraceptives (COCs)
for both contraception and treatment of her dysmenorrhea and was started on microgynon 30 mcg. Her dysmenorrhea, however, persists. Which one of the following would be the
next best step in management?

A. Start her on NSAIDs during her menses.

B. Increase the dose of estrogen.

C. Decrease the dose of estrogen.

D. Prescribe progestogen-only pills (POP).

E. Advise Mirena®.

Correct
45% answered correctly

Explanation:

Correct Answer Is A

Based on the history and the normal investigation, this woman has primary dysmenorrhea. The characteristic symptoms of primary dysmenorrhea include lower abdominal or pelvic
pain with or without radiation to the back or legs, with initial onset six to 12 months after menarche. Pain typically lasts 8 to 72 hours and usually occurs at the onset of menstrual
flow. Other associated symptoms may include low back pain, headache, diarrhea, fatigue, nausea, or vomiting. For primary dysmenorrhea to be the diagnosis, other underlying
causes such as uterine leiomyoma, endometriosis, etc. should have been excluded by thorough investigations.

A Cochrane review of 73 randomized controlled trials, demonstrated strong evidence to support nonsteroidal anti-inflammatory drugs (NSAIDs) as the first-line treatment for primary
dysmenorrhea. The choice of NSAID should be based on effectiveness and tolerability for the individual patient, because no NSAID has been proven more effective than others.
NSAIDs should be taken 1 to 2 days before the anticipated onset of menses and continued for 2 to 3 days into the periods.

Oral, intravaginal, and intrauterine hormonal contraceptives have been recommended for management of primary dysmenorrhea; however, the evidence supporting their
effectiveness is limited. Such methods are often considered second-line for treatment of dysmenorrhea. In patients who also desire contraception a 2-3 months trial of oral
contraceptives is considered first. Both 28-day and extended-cycle oral contraceptives are reasonable options in women with primary dysmenorrhea who also desire contraception.
If unresponsive, NSAIDs are added.

References

• Medscape – Dysmenorrhea

• AAFP - Diagnosis and Initial Management of Dysmenorrhea

Last updated:
Time spent: QID:1632
2023-2-12

1025 of 1943
A 23-year-old woman presents to your clinic with amenorrhea for 6 weeks. Which one of the following, if found on history or exam, will make you decide to proceed to emergency
Intervention?

A. Vaginal bleeding.

B. Lower abdominal pain.

C. Lower abdominal tenderness.

D. Shoulder tip pain.

E. Rebound tenderness.

Incorrect. Correct answer is D


45% answered correctly

Explanation:

Correct Answer Is D

Abdominal pain in women may be related to pathology in the pelvic organs. Ovarian cysts, uterine fibroids, tuboovarian abscesses, and endometriosis are common causes of lower
abdominal pain in women. In women of reproductive age, special attention to pregnancy complications, including ectopic pregnancy and loss of pregnancy is critical in forming an
appropriate differential diagnosis. The possibility of pregnancy modifies the likelihood of disease and significantly changes the diagnostic approach.

In every woman of reproductive age, abdominal pain, especially in the presence of a positive pregnancy test, diagnoses such as ectopic pregnancy (EP) or miscarriage should come
on top of the list of differential diagnoses and investigated thoroughly.

Specific clinical findings point to different conditions. Vaginal bleeding can be seen in EP as well as almost all types of abortions and failed pregnancies. Lower abdominal pain and
tenderness is often another shared finding in the mentioned conditions. Lower abdominal rebound with minimal or no guarding is the most common abdominal findings in women
with EP even when it is still unruptured. Such finding warrants immediate investigation and action because, for example, rupture of an EP can rapidly result in shock and catastrophic
outcomes.

Of the option, however, shoulder tip pain is the most important sign demanding urgent action. This sign is not common but if present in suspected EP, demands urgent surgical
action because indicates free blood in the peritoneum and peritoneal irritation. The mechanism such pain is produced is thought to be irritation of the diaphragm and the phrenic
nerve by the free blood in the peritoneal cavity. Then pain can radiate to the shoulder tip especially on the right side.

NOTE – Generally, abdominal rebound tenderness indicates acute abdomen and surgical emergency. In specific circumstances such as in EP, however, the rebound tenderness
could have been caused by an unruptured EP that can be taken care of non-surgically (e.g., methotrexate). Other indications of surgical acute abdomen include rigidity and
peritonism.

References

• RANZCOG – Ectopic pregnancy and miscarriage

• Medscape – Ectopic Pregnancy

• World Journal of Emergency Medicine - An unusual emergency department case: ruptured ectopic pregnancy presenting as chest pain

Last updated:
Time spent: QID:1638
2023-2-12

1026 of 1943
A 32-year-old woman with history of primary infertility presents to your clinic with a 6-month history of secondary amenorrhea. Which one of the following hormones, if significantly elevated, would signify that pregnancy has already occurred?

A. Serum prolactin (PRL).

B. Serum leuteinizing hormone (LH).

C. Serum follicle stimulating hormone (FSH).

D. Serum progesterone.

E. Serum estradiol.

Incorrect. Correct answer is B


45% answered correctly

Explanation:

Correct Answer Is B

Estradiol, prolactin and progesterone all are elevated during the pregnancy, but they may be also raised in other pathological conditions without pregnancy.

FSH is suppressed throughout pregnancy due to the negative feedback signaled by elevated eestradiol.

During the pregnancy β-HCG reaches it maximum plasma levels around 11-12 weeks of gestation and remains high throughout the pregnancy.

LH, FSH, TSH and HCG have identical alpha subunits. But only LH has a beta subunit (physiologically active component of LH) with similar structure to that of HCG; therefoe, virtually in all LH assays during gestation the LH will be reported high.
Although LH levels of 100-150 IU/L may be seen in other conditions such as polycystic ovarian syndrome (PCOS), levels above 200 IU/L are highly indicative of pregnancy.

1027 of 1943
Janet, 32 years of age, who is 33-week pregnant, presents to the Emergency Department with lower abdominal pain starting few hours ago and increasing in intensity. Her
pregnancy so far has been uncomplicated, and she is not on any medications apart from supplements. Few days ago, she had an upper respiratory infection associated with
sneezing, bouts of cough, runny nose, and mild fever from which she is recovering. She denies vaginal bleeding. On examination, her vitals are within normal limits. Abdominal exam
reveals tenderness over the right lower part of her uterus, but no abnormal uterine contractions are note. A cardiotocograph (CTG) is normal with a fetal heart rate of 155 bpm.
Which one of the following could be the most likely explanation to this presentation?

A. Diastasis recti abdominis.

B. Rectus sheath hematoma.

C. Placental abruption.

D. Placenta previa.

E. Vasa previa.

Incorrect. Correct answer is B


45% answered correctly

Explanation:

Correct Answer Is B

The scenario represents a rather sudden-onset abdominal pain in a pregnant woman. The history is unremarkable except for preceding sneeze and bouts of cough and abdominal
tenderness in the absence of vaginal bleeding and fetal compromise which could be justified best by rectus sheath hematoma. Rectus sheath hematoma in pregnancy is
uncommon or even rate yet the best explanation for this scenario.

Etiologies of recuts sheath hematoma are as follows:

Anticoagulation therapy
Severe cough
Pregnancy
Previous or recent abdominal surgery
Abdominal trauma
Chronic kidney disease
Steroid/immunosuppressive therapy
Vigorous uncoordinated rectus muscle contraction
Some general medical condition

Janet already has two of these etiologies for the condition: pregnancy and cough as the most common inciting factor.

Rectus sheath hematoma can mimic acute abdomen; therefore, it is of paramount importance that other causes of acute abdomen be considered and fully investigate through
history, physical exam, and relevant investigations. With pregnancy, other pregnancy-related conditions presenting with abdominal pain such as placental abruption should be highly
suspected and excluded. For Janet however placental abruption (option C) is less likely of a diagnosis. Placental abruption can cause uterine tenderness and overt vaginal bleeding
or even without vaginal bleeding if bleeding forms a retroplacental hematoma (concealed bleeding). There is often fetal compromise on CTG monitoring. The CTG in this scenario is
within physiologic limits making placental abruption less likely. On the other hand, sneezing and bouts of cough are very unlikely to result in placental abruption. What is elicited as
tenderness of the uterus is in fact overlying abdominal wall tenderness due to the hematoma.

Diastasis recti abdominis (DRA) (option A) is a midline separation at the linea alba of the rectus muscles of the abdominal wall. A diastasis is a palpable midline gap of more than
2.5 cm or any visible bulging on exertion or when the intrabdominal pressure is increased (Valsalva maneuvers, coughing, straining, etc.) DRA often develops around the umbilicus
but can also occur anywhere between the xiphoid and the pubic bone. DRA can happen in varying degrees in pregnancy due to abdominal musculature stretch weakness from
maternal hormonal changes and increased tension by the growing uterus. DRA presents as a painless midline swelling which increases in size and accentuates with increased
abdominal pressure such as with coughing. The presence of tenderness and its locations in this scenario (RLQ) make DRA less likely of a diagnosis.

Placenta previa (option D) and vasa previa (option E) could present with painless vaginal bleeding. CTG usually shows compromised fetal parameters. With abdominal pain and
normal CTG, these are less likely to be the cause of such presentation.

References

• BMC – Journal of medical case report - Spontaneous rectus sheath hematoma in pregnancy and a systematic anatomical workup of rectus sheath hematoma: a case report

Last updated:
Time spent: QID:1702
2023-2-12

1028 of 1943
Which one of the following is the most important factor for labor to start and progress?

A. Head molding.

B. Rupture of membranes.

C. Cervical effacement.

D. Cervical dilation.

E. Uterine contractions.

Incorrect. Correct answer is E


45% answered correctly

Explanation:

Correct Answer Is E

Labor is a physiologic process during which the fetus, membranes, umbilical cord, and placenta are expelled from the uterus.

Labor has 3 stages:

First stage of labor

Begins with regular uterine contractions and ends with complete cervical dilatation at 10 cm.

First stage of the labor divides into 2 phases:

Latent phase:

Begins with mild, irregular uterine contractions that soften and shorten the cervix. Contractions become progressively more rhythmic and stronger.

Active phase:

Usually begins at about 3-4 cm of cervical dilation and is characterized by rapid cervical dilation and descent of the presenting fetal part

Second stage of labor

Begins with complete cervical dilatation and ends with the delivery of the fetus

Third stage of labor

The period between the delivery of the fetus and the delivery of the placenta and fetal membranes. Delivery of the placenta often takes less than 10 minutes, but the third stage may
last as long as 30 minutes.

For the labor to start and progress, uterine contractions are the most important inciting factor precipitating the chain of events at different stages of labor such as cervical dilation
(option D) and cervical effacement (option C). Rupture of membrane (option A) happens due to the force imposed on over-stretched amniotic membranes by the uterine contraction.

Skull sutures and fontanelles are flexible in fetus to accommodate brain growth. Head molding happens as a result of the fetal head passing through the birth canal during labor.
The flexibility of the sutures may cause slight temporary disfigurement of the skull which is referred to as head molding. Head molding (option A) is an effect of the labor not a
cause for it.

References

• MSD Manual – Management of Normal Labor

• Medscape – Normal Labor and Delivery

Last updated:
Time spent: QID:1738
2023-2-12

1029 of 1943
Alison is 33 years old and has come to you for contraception advice. Two years ago, while on combined oral contraceptive pills (COCP), she developed deep vein thrombosis;
therefore, she was switched to a progestogen-only pill (POP). Last year, she came off the pill to start a family, but her pregnancy turned out ectopic, and aborted 6 months ago. She
wants to start contraception again as she is not planning any pregnancies in near future. Which of the following would be the correct advice in this regard?

A. She cannot take POP.

B. She can start taking POP.

C. She can start taking COCP, but she will need prophylactic anticoagulation.

D. She can take POP after 12 months of her ectopic pregnancy.

E. She can start taking COCP.

Incorrect. Correct answer is B


45% answered correctly

Explanation:

Correct Answer Is B

The stem in question asks about the best approach to contraception after an ectopic pregnancy. Important to note in history is a contraindication to the use of contraceptives, in this
case, developing deep venous thrombosis after COCP use, which makes it an absolute contraindication to use of COCPs (options C and E) in the future.

There is controversy regarding the use of POPs, with some studies reporting an increase in the incidence of ectopic pregnancy by reducing fallopian tube activity of cilia and altered
tubal motility. The same mechanism is postulated to be responsible for the increased incidence of ectopic pregnancies following emergency contraception with POPS (progestogen-
only emergency contraception, (POEC).

For the recommendations of contraception after contraception, women should be advised that any method of contraception can be safely initiated immediately after surgical or
medical treatment for an ectopic pregnancy. Therefore, in this case, the woman can be safely advised to use POP. Telling her she cannot take POP (option A) is incorrect.

Telling her to start POP after 12 months (option D) is not recommended because contraception can be started immediately. An additional contraceptive (barrier or abstinence) may
be required if hormonal contraception started 5 days or more after miscarriage (can be omitted if used immediately or within 5 days after the miscarriage.

All contraceptives are effective in preventing intrauterine and ectopic pregnancies. However, LARC (long-acting reversible contraceptive) is most effective in the reduction of any
type of pregnancy including ectopic.

Women with a history of ectopic pregnancy are at risk of another one and they should be informed of the small risk of ectopic pregnancy while on contraceptive use if a pregnancy
occurs. In this context, they should be offered LARCs since they are the most effective method in reducing all pregnancies including ectopic ones.

References

• RANZCOG - FSRH Guideline Contraception after pregnancy

• RCH - Oral hormonal contraception in special circumstances

• Harrison-Woolrych M, Woolley J. Progestogen-only emergency contraception and ectopic pregnancy. BMJ Sexual & Reproductive Health. 2003 Jan 1;29(1):5-6.

Last updated:
Time spent: QID:1740
2023-2-12

1030 of 1943
Mary is in your GP clinic for consultation. She is 35 years old and is annoyed by her periods which are regular yet heavy and prolonged. She has been already assessed with
transvaginal ultrasound and blood tests with normal results except for mild iron-deficiency anemia. She is married and has no plans to start a family, not at least in one year. Which
one of the following would be the most appropriate treatment option for her?

A. Progestogen-only pill (POP).

B. Combined oral contraceptive pills (COCP).

C. Mirena.

D. Implanon.

E. NSAIDs.

Incorrect. Correct answer is C


45% answered correctly

Explanation:

Correct Answer Is C

Heavy menstrual bleeding (HMB) is the most common form of abnormal vaginal bleeding (AUB) in women of reproductive age and is defined as an excessive blood loss that
impairs the woman's quality of life either physically, emotionally, or socially. It is benign, and not associated with pregnancy or any other gynecological or systemic disease.

HMB can have a significant impact on quality of life and lead to time work, and fatigue related to iron deficiency anemia. Moreover, HMB can have a significant burden on healthcare
resources. HMB is a common reason for referral to a gynecologist.

Treatment for HMB can be medical or surgical. Hysterectomy has traditionally been regarded as the 'definitive' treatment, but surgical options such as hysterectomy and the less
invasive endometrial ablation are associated with risks and complications. Medical options enable women to retain their fertility and avoid the risks of surgery. The UK NICE
guidelines on HMB recommend the following medical treatments: hormonal (levonorgestrel-releasing intrauterine system (LNGIUS), combined oral contraceptives, and
progestogens), and non-hormonal (non-steroidal anti-inflammatory drugs (NSAIDs) and antifibrinolytics).

The choice of medication depends upon its appropriateness, likely acceptability to a woman, and whether she requires contraception. Combination contraception methods, in the
form of a pill, the vaginal ring, and the transdermal patch, have all been shown to regulate the menstrual cycle in premenopausal women, with the added benefit of reducing MBL.

Management of HMB has to take into account the woman's preferences, any comorbidities, the presence or absence of fibroids (including size, number, and location), polyps,
endometrial pathology, or adenomyosis, and other symptoms such as pressure and pain.

The most appropriate treatment option for this woman is Mirena (C is correct), as she is young and has heavy menstrual bleeding. As a part of long-acting reversible contraceptives
(LARCs), which provide highly effective ‘set-and-forget’ contraception and are suitable for provision in general practice. They have very few contraindications and can be
recommended as first-line contraceptive options across the reproductive lifespan, from adolescence to perimenopause. Mirena is 99.7-99.9% effective and lasts up to 5 years with
efficacy for contraception and HMB.

Implanon (etonogestrel single-rod contraceptive implant) (option D) provides highly effective (99.95%) contraception for up to three years. However, troublesome bleeding is the
most common side effect leading to discontinuation of the implant. This makes it less desirable for this woman.

COCP (option C) could be a choice but compared to LNGIUS is less effective.

Progesterone only pill (option A) is associated with irregular and unpredictable blood loss and is not usually recommended as a treatment for HMB.

NSAIDs (option E) can be used to treat heavy menstrual bleeding but is not a contraceptive.

References

• NICE guidelines UK for heavy menstrual bleeding, 2018

• AJGP - Long-acting reversible contraceptives: New evidence to support clinical practice

Last updated:
Time spent: QID:1746
2023-2-12

1031 of 1943
A 49-year-old woman presents with heavy irregular periods every 2 to 3 months lasting 7-10 days for the past seven years. She undergoes a diagnostic dilation and curettage (D&C) in the premenstrual period. Which one of the following findings on
histological examinations, if present, is the most likely cause of her problem?

A. Normal secretory endometrium.

B. Cystic glandular hyperplasia.

C. Atypical hyperplasia.

D. An endometrial polyp.

E. Endometrial cancer.

Incorrect. Correct answer is B


45% answered correctly

Explanation:

Correct Answer Is B

Endometrial exposure to unopposed estrogen leads to endometrial hyperplasia and irregular endometrial shedding and periods. Endometrial hyperplasia can be seen in the following different forms:

Simple hyperplasia (cystic without atypia) aka cystic glandular hyperplasia


Complex hyperplasia (adenomatous without atypia)
Atypical simple hyperplasia (cystic with atypia)
Atypical complex hyperplasia (adenomatous with atypia)

Of the above forms, simple hyperplasia, also termed cystic glandular hyperplasia (CGH) is the most common form of endometrial hyperplasia seen in perimenopausal women.

Endometrial hyperplasia can present with:

Bleeding between periods


Heavy and/or prolonged periods
Vaginal discharge
Abdominal pain

Statistically, the most common cause of heavy and/or irregular periods in a peri-menopausal woman is anovulatory cycles, giving rise to endometrial hyperplasia, mostly CGH form.

(Option A) The clinical picture is not caused by a normal secretory endometrium.

(Options C and E) Endometrial cancer and atypical hyperplasia can result in irregular heavy menstrual bleeding but they are less common compared with CGH.

(Option D) Endometrial polyp can be a cause of heavy menstrual bleeding but firstly, it is less common compared to CGH and secondly, it only causes menorrhagea and not irregular periods.

1032 of 1943
A 48-year-old woman presents with complaint of heavy irregular bleedings for the past five months. Her periods were completely regular before, occurring every 30 days and lasting for 6 days. Her last pap smear was performed 6 months ago with
normal result. Physical examination is unremarkable. Which one of the following is the most likely cause of her presentation?

A. Endometriosis.

B. Endometrial polyps.

C. Endometrial carcinoma.

D. Anovulatory cycles.

E. Submucosal fibroids.

Incorrect. Correct answer is D


45% answered correctly

Explanation:

Correct Answer Is D

This woman is most likely experiencing perimenopausal erratic bleeding due to fluctuating levels of estrogen and progesterone. Failure to ovulate causes unopposed estrogen effect and endometrial hyperplasia that when undergoes shedding leads
to metrorrhagia, menorrhagia or both.

Endometrial polyps (option B), endometriosis (option A) and fibroids (option E) are associated with increased volume of bleeding, namely metrorrhagia, but not irregular patterns of bleeding.

Although endometrial carcinoma (option C) should always be borne in my mind in perimenopausal women, statistically it is less likely of a cause compared to anovulatory cycles as the underlying etiology of irregularities in bleeding pattern during
the perimenopausal period.

1033 of 1943
A 50-year-old woman presents to you complaining of hot flushes disturbing her daily life. Her medical history includes hysterectomy due to uterine fibroid and an episode of deep vein thrombosis 18 months ago. She asks for hormone replace therapy
(HRT). Which one of the following would be the best treatment option for her?

A. Low-dose estradiol valerate.

B. Low-dose combined estrogen and progesterone.

C. Progesterone.

D. Estrogen dermal patch.

E. Estrogen implants.

Incorrect. Correct answer is D


45% answered correctly

Explanation:

Correct Answer Is D

Estrogen-only HRT is more effective than combined HRT and indicated whenever endometrial cancer caused by unopposed estrogen is not a concern, such as in this woman who does not have a uterus.

Oral estrogen is known to stimulate the coagulation pathway and is associated with increased risk of thromboembolism; hence, systemic estrogen is not a good option for her. However, estrogen dermal patches have not been shown to be
associated with increased risk of venous thromboembolism and can be safely use for this woman. Estrogen implants are the second-line option if the transdermal patches fail to control the symptoms.

TOPIC REVIEW

Combination HRT is associated with a 2- to 5-fold relative risk of venous thromboembolism (VTE). The risk is 1.2 -1.5 in estrogen HRT. Natural progesterone is not associated with increased risk of VTE, whereas, synthetic progesterones such as
medroxy progesterone acetate (Depo Provera®) has been shown to be associated with a slight increase in risk of VTE.

Studies comparing oral and transdermal estorgen preparations have demonstrated that transdermally administered estrogen has little, if any, effect in increasing prothrombotic substances (due to first-pass effect in the liver) and may have beneficial
effects on proinflammatory markers, including C-reactive protein, prothrombin activation peptide, and antithrombin activity. Also, in contrast to oral estrogen, transdermal estrogen may have a suppressive effect on tissue plasminogen activator and
plasminogen activator inhibitor activity.

1034 of 1943
Which of the following breast diseases has been shown to be associated with vitamin D deficiency?

A. Fibroadenoma.

B. Periductal mastitis.

C. Ductal carcinoma in situ.

D. Intraductal papilloma.

E. Ductal ectasia.

Incorrect. Correct answer is C


45% answered correctly

Explanation:

Correct Answer Is C

Studies suggest that vitamin D deficiency has been associated with an increased risk of cancer, especially colon cancer, prostate cancer, and breast cancer. Vitamin D may play a role in controlling normal breast cell growth and may be able to stop
breast cancer cells from growing.

References

• Medscape - Vitamin D Linked to Enhanced Breast Cancer Survival

Last updated:
Time spent: QID:688 2023-2-12

1035 of 1943
Eve, 19 years old, comes to your office seeking advice on contraception. She is planning a trip to Thailand with her new boyfriend. She has no contraindication to contraception. Which one of the following would be the most appropriate advice?

A. Condoms.

B. Intrauterine contraceptive device (IUCD).

C. Oral contraceptive pills (OCPs).

D. Oral contraceptive pills and condoms.

E. Diaphragm and spermicide gel.

Incorrect. Correct answer is D


45% answered correctly

Explanation:

Correct Answer Is D

For the following groups, it is recommended that contraception be adivsed with combined oral contraceptives (COCs) if not contraindicated PLUS condoms to prevent sexually transmissbles infections (STIs) such as chalmidya, gonorrhea, HIV, etc.:

Young women (<25 years)


Women older than 25 years with a new partner
Women older than 25 year with two or more partners in the last year
Women older than 25 years whose regular partner has multiple partner

Eve falls is younger than 25 years and must be recommended to use both COCs and condomes. Addittionally, she has started a new relationship, and this stresses more on such recommendation as the best advice.

1036 of 1943
A 25-year-old woman presents for your advice regarding choice of oral contraceptive pills after delivery of her male baby two weeks ago. She used combined oral contraceptive pills before her last pregnancy. She is non-smoker and breastfeeds her
baby on demand. Which one of the following is the best choice in this situation?

A. Restarting combined oral contraceptive pills.

B. Start progesterone-only pills

C. No contraception needed.

D. Advise condoms.

E. Refer to a gynecologist.

Incorrect. Correct answer is C


45% answered correctly

Explanation:

Correct Answer Is C

Elevated prolactin levels and a reduction in gonadotropin-releasing hormone from hypothalamus during lactation suppress ovulation. This results in a reduction in leuteinizing hormone (LH) release and inhibition of follicular maturation. The duration
of suppression varies and is influenced by the frequency and duration of breastfeeding and the length of time since birth.

Lactational amenorrhea (LAM) is an excellent method of contraception provided that all the following criteria are met:

The woman remains amenorrheic


Less than 6 months since giving birth
The baby is fully breastfed (or breastfed with very infrequent supplements)

NOTE – if the woman is not breastfeeding or breastfeeds but not on a regular basis, ovulation recurs within 45 days but it is possible after 21 days. Ovulation may occur in the absence of menstruation.

The following contraception methods can beused in postpartum period in a lactating woman:

Condoms - can be used immediately; spermicide gels are not recommended.

Minipill (POP) - Can be started in breastfeeding women. Although it does not have an adverse effect on breast milk, because of a theoretical concern about the effect of sex steroids on the neonate the commencement of POP is recommended after
3-4 weeks.

Etonorgestrel implant (Implanon®) - It can be inserted anytime from 6 weeks postpartum. If the woman is amenorrheic, pregnancy should be excluded first. Small amounts of contraceptive steroids are excreted in breast milk and concerns about
exposure prior to 6 weeks exist. If it is considered, risks and benefits should be fully discussed with the woman.

Progesterone intramuscular injection (Medroxy progesterone acetate e.g. Depo-Provera® or Depo-Ralovera®) – can be started anytime postpartum; however, it is recommended by WHO that it not be used until 6 weeks postpartum, unless more
appropriate methods are not avaiable. Pregnancy should be excluded if >21 days.

NOTE - this method may be considered in women, who are breastfeeding and are less than 6 weeks postpartum if risk of subsequent pregnancy is high and other forms of contraception are unacceptable.

Mirena® (Levonorgestrel) IUD, or the Copper Intrauterine Devices (Cu-IUD) - Unless the IUD can be inserted within 48 hours postpartum, it should be delayed until 4 weeks after childbirth. Prior to insertion after 4 weeks, pregnancy should be excluded.
Women who have had a cesarean section should not have an IUD inserted prior to 6 weeks postpartum due to the increased risk of perforation. Mirena® is effective within 7 days, and Cu-IUDs are effective immediately.

This woman is regularly breastfeeding her baby on demand; therefore, she probably does not need further contraception at least while breastfeeding in the first 6 months, but in reality there is an slight risk of pregnancy (about 2%) if menstruation
resumes.

NOTE - Postpartum women should not have intercourse within the first 2 weeks after giving birth because of the risk of air embolism or infections. Overall, intercourse is recommended to be deferred to until 6 weeks postpartum to avoid various
complications. If a women, who does not fullfill criteria for LAM, decides to start having sexual intercourse before that time offer condoms for the first 3-4 weeks and POP afterwards.

1037 of 1943
A 27-year-old woman presents to your practice on the 10th day of her menstrual cycle because she missed her pills on 6th and 7th days and had unprotected intercourse on 8th and 9th days. She is on 35mcq ethinylestradiol. She wants your advice
regarding contraception because she does not want to become pregnant. Which one of the following would be the most appropriate advice?

A. She should take the last missing pill now and continue the same OCP as usual.

B. She should stop taking pills from the current package and starts a new one from now.

C. Give emergency contraception now using Postinor® (2 doses 12 hours apart).

D. Stop OCP and wait for 4 weeks and check if she is pregnant.

E. She should take the last missed pill now and continue the same OCP as usual and use condoms or abstain for 7 days.

Incorrect. Correct answer is C


45% answered correctly

Explanation:

Correct Answer Is C

The following are the latest recommendations regarding missed oral contraceptive pills and emergency contraception if sexual intercourse occurs within the ‘missed pill’ period:

If one pill (active) is missed, anywhere in the pack (i.e. more than 24 and up to 48 hours late):

The last pill missed should be taken now, even if it means taking two pills in one day.
The rest of the pack should be taken as usual.
No additional contraception is needed.
The seven-day break is taken as normal.

Emergency contraception – Emergency contraception is not needed if just one pill has been missed. However, it should be considered if other pills have been missed recently, either earlier in the current packet, or at the end of the previous packet.

If two or more pills are missed (i.e. more than 48 hours late):

The last missed pill should be taken now, even if it means taking two pills in one day.
Any earlier missed pills should be left.
The rest of the pack should be taken as usual and additional precautions (e.g. condoms or abstinence) should be taken for the next seven days.

The next step then depends on where in the packet the pills are missed:

If the pills are missed in the first week of a pack (pills 1-7): emergency contraception should be considered if the patient had unprotected sex in the pill-free interval or the first week of the pill packet. She should finish the packet and have the usual
pill-free interval.

If the pills are missed in the second week of a pack (pills 8-14): there is no need for emergency contraception as long as the pills in the preceding seven days have been taken correctly. The packet should be finished and the usual pill-free interval
taken.

If the pills are missed in the third week of a pack (pills 15-21): the next pack of pills should be started without a break – i.e. the pill-free interval is omitted. If taking a packet with dummy/placebo pills, these should be discarded, and the new packet
started. Emergency contraception is not required.

NOTE - If more than seven pills are missed, the woman should start again as if starting for the first time. (Exclude pregnancy, and start a new pack on the first day of the next menstrual period.)

This woman has two missed pills on days 6 and 7 (within the first) week; therefore, in order to prevent pregnancy she should have emergency contraception using Postinor® (750mcq of levonorgestrel, 2 tablets, 12 hours apart) as the most important
step. Postinor is effective if given within the 72 hours (up to 96 hours) of unprotected sexual intercourse. She should also continue with the rest of the package as well as using condoms or abstinence for 7 days. As a rule when missed pills (even 1)
are within the first 7 pills, emergency contraception is needed if sexual intercourse has been done in the missed pill period.

1038 of 1943
A 32-year-old woman comes to your practice because she forgot to take her 7th and 8th p pills from a pack of OCPs containing 20mcg ethinyloestradiol (EO) and had unprotected intercourse on days 6 and 7. Today is the 10th day of her menstrual
cycle. She wants advice regarding contraception because she does not want to become pregnant. Which one of the following would be the best advice for her?

A. She should take the last missing pill now and continue with the same pack of OCP.

B. Give emergency contraception now using Postinor® (2 doses 12 hours apart).

C. She should take the last missed pill now and continue the same OCP as usual and use condoms or abstain for seven days.

D. Stop OCP and wait for 4 weeks and check if she is pregnant.

E. She should stop taking pills from the current package and starts a new one with her new period.

Incorrect. Correct answer is B


45% answered correctly

Explanation:

Correct Answer Is B

The following are the latest recommendations regarding missed oral contraceptive pills and emergency contraception in case sexual intercourse occurs within the ‘missed pill’ period:

If one pill (active) is missed, anywhere in the pack (i.e. more than 24 and up to 48 hours late):

The last pill missed should be taken now, even if it means taking two pills in one day.
The rest of the pack should be taken as usual.
No additional contraception is needed.
The seven-day break is taken as normal.

Emergency contraception – Emergency contraception is not needed if just one pill has been missed. However, it should be considered if other pills have been missed recently, either earlier in the current packet, or at the end of the previous packet.

If two or more pills are missed (i.e. more than 48 hours late):

The last pill missed should be taken now, even if it means taking two pills in one day.
Any earlier missed pills should be left.
The rest of the pack should be taken as usual and additional precautions (e.g. condoms or abstinence) should be taken for the next seven days.

The next step then depends on where in the packet the pills are missed:

If the pills are missed in the first week of a pack (pills 1-7): emergency contraception should be considered if the patient had unprotected sex in the pill-free interval or the first week of the pill packet. She should finish the packet and have the usual
pill-free interval.

If the pills are missed in the second week of a pack (pills 8-14): there is no need for emergency contraception as long as the pills in the preceding seven days have been taken correctly. The packet should be finished and the usual pill-free interval
taken.

If the pills are missed in the third week of a pack (pills 15-21): the next pack of pills should be started without a break – i.e. the pill-free interval is omitted. If taking a packet with dummy/placebo pills, these should be discarded, and the new packet
started. Emergency contraception is not required.

NOTE - If more than seven pills are missed, the woman should start again as if starting for the first time. (Exclude pregnancy, and start a new pack on the first day of the next menstrual period.)

This woman has missed two pills (>48 hours); therefore she should follow the following steps:

She should use emergency contraception now – although one pill is missed on day 8, since there has been no complete seven-day coverage, emergency contraception is needed.
She should take other pills and use condoms or abstain for 7 days.

The main concern of this woman is preventing the pregnancy so the most important advice would be emergency contraception. Post-coital methods have no effect on an implanted pregnancy. It is recommended that emergency contraception be
performed within the first 72 hours post-coitus; however, it can be effective up to 96-120 hours (4-5 days) post-coitus.

1039 of 1943
A 28-year-old woman comes to your practice because of nausea, vomiting and diarrhea after she had a burger in a local restaurant. She is on oral contraceptive pills. She has not missed her pills even when she was severely sick. Which one of the
following would be the most appropriate advice to give her regarding contraception and the pills?

A. She should continue the pills and use condom for 7 days.

B. She should stop the pills now and starts a new pack for the next cycle.

C. She should discontinue the pills and use a barrier method until diarrhea settles.

D. Give her ciprofloxacin and review in 2 days.

E. She should avoid sexual activity for 3 weeks.

Correct
45% answered correctly

Explanation:

Correct Answer Is A

In the following situations, the effectiveness of oral contraceptive pills is reduced and additional measures should be taken:

An active pill is forgotten to be taken for more than 24 hours from the last pill.
Severe diarrhea or vomiting lasting more than 24 hours. Virtually, this has the same effect of not having the next pill for more than 24 hours after the last.

In situations like these, the woman should take the next pill on the routine schedule but an extra protection method such as barrier methods should be used, or intercourse avoided for 7 days.

1040 of 1943
A 32-year-old woman, with past medical history of deep vein thrombosis, comes to your clinic for advice. She has been taking progesterone only pills (POP) until 4 months ago when she planned to become pregnant. The pregnancy, however, turned
out to be ectopic. Fortunately, she survived through. She is not willing to become pregnant again and asks whether she can start the pills again. Which one of the following would be the best advice you can give?

A. She cannot take progesterone only pills.

B. She should use combined oral contraceptive pills.

C. She can take progesterone only pills after 12 months of first ectopic pregnancy.

D. She should take anticoagulants along with combined oral contraceptive pills.

E. Tell her she should get pregnant as that is the only option.

Correct
45% answered correctly

Explanation:

Correct Answer Is A

With history of DVT, estrogen-containing contraceptives are contraindicated in this woman. This is probably why she was previously started on POP instead of regular combined pills. With the history of ectopic pregnancy, progesterone
is contraindicated as well and should never be used again. Other options such as barrier methods could be used in this woman.

Absolute contraindications to using progesterone only pills include:

Suspected pregnancy
Breast cancer
Undiagnosed vaginal bleeding
Past history of ectopic pregnancy or high risk for ectopic pregnancy

Relative contraindications to progesterone only pills include:

Active viral hepatitis


Severe chronic liver disease
Malabsorption syndrome
Severe arterial disease
Successfully treated breast cancer more than 5 years ago
concomitant use of hepatic enzyme enducing medications

1041 of 1943
A 20-year-old woman presents to your practice with complaints of episodes of erratic vaginal bleeding for the past four months after she started taking combined oral contraceptive pills (Microgynon 30). She is sexually active and uses condoms for
prevention of sexually transmitted infections as well. Which one of the following would be the most appropriate advice?

A. She should switch to progestogen-only pills.

B. She should switch to a new combined pill with ethinylestradiole 50mcg.

C. She should use intrauterine contraceptive device.

D. She should continue the same oral contraceptive pill.

E. She should use Implanon®.

Correct
45% answered correctly

Explanation:

Correct Answer Is B

Erratic breakthrough vaginal bleeding is seen in 30% of women who are recently started on combined oral contraceptive pills (COCPs). This percentage, however, decrease to only 10% after 3 months.

This problem is more common with preparations that contain low-dose estrogen. If the history of erratic bleeding is less than three months, only reassurance is required as the condition resolves after three months of use in most women. Once the
problem persists beyond three months, the next best step would be increasing the dose of estrogen (e.g. to 50mcg).

1042 of 1943
A 24-year-old woman presents to your clinic for advice regarding contraception. She has been on oral combined oral contraceptive pills for the past two years. Two months ago, she developed deep vein thrombosis and has been on treatment with
warfarin. Her mother and sister also have history of deep vein thrombosis. Which one of the following would be the most appropriate advice?

A. She should use male condoms.

B. Continuation of combined oral contraceptive pills.

C. Estrogen patch.

D. Progestogen-only pill.

E. Discontinuation of warfarin.

Correct
45% answered correctly

Explanation:

Correct Answer Is A

Any patient with history of DVT should discontinue estrogen-containing oral contraceptive pills.

Guidelines released by the WHO (World Health Organization) advocate that for most women with a family or personal history of deep vein thrombosis (DVT) or pulmonary embolism (PE) the benefits of using a POP outweigh the risks, and a POP can
generally be used.

The guideline, however, mentions that the use of a POP is not recommended in the presence of current venous thromboembolism (VTE) unless other methods are not available or are unacceptable. Medicines Adverse Reaction Committee (MARC)
considers that POPs are absolutely contraindicated in women with a current thromboembolic process.

Since this woman has a current DVT, for which she is on therapy, not only combined contraceptive are absolutely contraindicated, but progesterone-only pills are better avoided in this situation, and an alternative non-hormonal contraceptive methods
such as condoms is used.

NOTE - Without making any distinction between minor and major surgery, WHO guidelines recommend that POPs can be continued for women undergoing any surgical procedure. Where there is prolonged immobilization following surgery, the
guidelines mention that it is generally appropriate to continue POP use. Based on this advice, the MARC suggests that when determining the most appropriate oral contraceptive, a POP may also be considered for women with other risk factors that
put them at a high risk of VTE with a combined oral contraceptive, especially for those with more than one risk factor (e.g. family history and BMI > 30).

1043 of 1943
Which one of the following is the first-line contraceptive choice for women with epilepsy, who are on enzyme-inducing anti-epileptic drugs?

A. Combined oral contraceptive pills with low-dose estrogen.

B. Levonorgestrel-releasing intrauterine contraceptive device.

C. Barrier methods.

D. Progestin-only pills.

E. Combined oral contraceptive pills with high-dose estrogen.

Incorrect. Correct answer is B


45% answered correctly

Explanation:

Correct Answer Is B

There are a few management options for women on enzyme-inducing medications, levonorgestrel-releasing intrauterine contraceptive device (Mirena®) is the first-line contraceptive choice. Other methods to consider are male condom or increasing
the dose of estrogen.

Mirena® is a very effective form of contraception. The effect of progesterone is mediated locally and is not affected by enzyme-inducing antiepileptic drugs. Failure rate is 1%, with prompt reversion of fertility once the contraception is withdrawn.

Contraindications to intrauterine devices include:

Pregnancy
Active pelvic inflammatory disease (PID) or the history of acute PID within the past 3 months
Postpartum or post-abortion endometritis in the past 3 months
Uterine abnormalities resulting in anatomic distortion
Genital bleeding of unknown etiology
Untreated vaginitis or cervicitis, including bacterial vaginosis
Previously inserted IUD still in place

Relative contraindications:

Menorrhagea
Dysmenorrhea
Very large or very small uterus (>5.5cm or <0.9cm)
Anemia
Defective immune system
Impaired clotting mechanism
Valvular heart disease
Patient or partner with multiple sexual partners

Combined oral contraceptive pills are not the first-line option for contraception in patients taking enzyme inducing anti-epileptic drugs. If a patient chooses to use combined oral contraceptive pills, those with higher estrogen content should be
considered.

Progestin-only pills are not suitable for women who are on enzyme inducing antiepileptic drugs which can cause reduction in circulating levels of progestin..

1044 of 1943
A 38-year-old woman presents to your clinic for advice. She is smoker and has been on progesterone-only pills for the past 3 years. Recently, following an episode of tonic-clonic convulsion, a neurologist has started her on carbamazepine. Which one
of the following would be the most appropriate advice regarding contraception?

A. She should stop progesterone-only pill and use condoms.

B. She shoudl cease the progesterone-only pill and start taking combined oral contraceptive pills.

C. She should stop carbamazepine.

D. She can continue both carbamazepine and progesterone-only pills as there is no interaction.

E. The dose of progesterone-only pill should be doubled.

Correct
45% answered correctly

Explanation:

Correct Answer Is A

Enzyme-inducing drugs can affect the efficacy of oral contraception. Concomitant use of enzyme-inducing drugs may cause a 40-50% decrease in plasma levels of both progesterone and estrogen, resulting in unwanted pregnancy.

Enzyme inducing drugs affect the following:

Combined oral contraceptive pills (COCs)


Progestin-only pills
Progestin implants (Implanon®)

NOTE – the efficacy of Mirena® and Depo Provera® (medroxy progesterone acetate) are not affected by enzyme-inducing drugs.

Carbamazepine is an enzyme inducer that can reduce the efficacy of oral contraceptive pills. Women using COCs (option C) , POPs or Implanon® should be advised that the efficacy of these methods may be reduced by enzyme-inducing drugs and
that they should use an alternative method, such as male condoms. While there is a significant interaction between these two advising continuing use of both drugs together (option D) is not appropriate.

As this woman is smoker and older than 35 years, estrogen-containing contraceptives are contraindicated.

Doubling the dose of the POP (option E) is not an appropriate option. Rather the women should be advised to use levonorgestrel (LNG) IUCD (Mirena®), injectable medroxy progesterone (Depo Provera®), or male condoms.

(Option C) Cessation of carbamzepine is not appropriate as this can result in uncontrolled seziures.

TOPIC REVIEW

Enzyme-inducing drugs and contraception:

All women starting enzyme-inducing drugs should be advised to use a reliable contraceptive method unaffected by enzyme inducers (e.g. progesterone-only injectable, copper-bearing intrauterine devices (Cu-IUDs) or the
levonorgestrel-containing intrauterine system (LNG-IUS))

Women who do not wish to change from a combined method while on short-term treatment with an enzyme-inducing drug (and for 28 days after stopping treatment), may choose to continue using a combined oral contraceptive (COC)
containing at least 30 mcg ethinylestradiol, the patch or ring along with additional contraception. An extended or tricyclic regimen should be used with a hormone-free interval of 4 days. Additional contraception should be continued for
28 days after cessation of the enzyme inducing drug. Tricycling is taking three packs of COC (3x21) in a row with 4-days hormone-free intervals (3x21)

With the exception of the very potent enzyme inducers rifampicin and rifabutin, women who are on an enzyme-inducing drugs, who do not wish to change from COC may increase the dose of COC to a preparation with at least 50mcg
ethinyl estradiol (up to 75mcq) and use an extended or tricycling regimen with a pill-free interval of 4 days

NOTE- with rifampicin and rifabutin, advising to increase the dose of the COC is inappropriate. An alternative method should be used.

In women using enzyme-inducing drugs with COC, breakthrough bleeding may indicate low serum ethinylestradiol concentrations. If other causes (e.g. Chlamydia) have been excluded, the dose of ethinylestradiol can be increased up to
a maximum of 75 mcg (or according to some references 100mcg)

Women, who do not wish to use additional contraception, or women on long-term treatment (>2 months) with an enzyme-inducing drug, who do not wish to change to another method, may be offered an increased dose of COC
containing at least 50mcq ethinylestradiol during treatment and for 28 days afterwards. An extended or tricycling regimen and pill-free interval of 4 days are recommended but additional contraception is not essential.

Progesterone-only pills, implants, and enzyme-inducing drugs

Women using progesterone-only pill (POP) or implant should be advised that the efficacy of these methods may be reduced by enzyme-inducing drugs and that they should use an alternative method.

Women using POP or implant with a short-term course of enzyme-inducing drug (<2 months) could be offered an injection of DMPA. Women using enzyme-inducing drugs in the short term, who do not wish to change their contraceptive
method, should be advised that they must take additional contraceptive precautions while on the enzyme-inducing drug and for 28 days after cessation of treatment

Women who do not wish to change from the progesterone-only pill or implant while on short term treatment with an enzyme-inducing drug or within 28 days of stopping treatment may choose to continue the method together with
additional contraceptive precautions (e.g. condoms). Additional precautions should be continued for 28 days after stopping the enzyme-inducing drug.

1045 of 1943
A 50-year-old menopausal female presents with frequent hot flushes disturbing her sleep and her work. Her mother had breast cancer at 50 years of age. She is otherwise in good health, with no significant previous medical or surgical history. Which
one of the following is the best management option for her symptoms?

A. Combined hormone replacement therapy.

B. Progesterone.

C. Paroxetine.

D. Black cohoch.

E. Estrogen dermal patch.

Correct
45% answered correctly

Explanation:

Correct Answer Is A

Currently, hormone replacement therapy (HRT) is the most effective treatment for troublesome vasomotor symptoms of menopause such as hot flushes. Studies suggest that by using HRT, the frequency of hot flushes is reduced by approximately 18
episodes per week, and the severity of hot flushes by 87% compared with placebo. Other benefits include: significant reduction in fracture risk, improvement of vaginal dryness and sexual function, improvement of sleep, decreased muscle aches and
pain, and improved quality of life in symptomatic women.

For women with a uterus, the regimen consists of both estrogen and progesterone. Although HRT with estrogen alone has shown to be more effective, progesterone should be added to counteract the effect of unopposed estrogen on uterus and
endometrial hyperplasi. However, in a woman without a uterus, estrogen-only regimens are preferred.

Current evidence-based guidelines advise consideration of HRT for troublesome vasomotor symptoms in perimenopausal and early postmenopausal women without contraindications and after individualized discussion of likely risks and benefits.

HRT reduces the fracture risk and improve mood and libido, but none of these is an indication for commencement of HRT. Vaginal symptoms alone do not require systemic HRT and can be managed with topical estrogen.

Starting HRT in women over the age 60 years is generally not recommended. For women with premature (age <40 years) or early (<45 years) menopause, current guidelines recommend HRT until the age of 50 for treatment of vasomotor symptoms
and bone preservation.

As a history of hysterectomy is not mentioned in the question, it should be assumed that the patient has a uterus; therefore the treatment of choice would be combination therapy with estrogen and progesterone if there are no contraindications.
There is no consensus on absolute contraindications to HRT. However, HRT should be avoided or discontinued in patients with the following conditions:

Personal history of breast cancer - HRT may increase the risk of breast cancer recurrence and development of new breast cancers. Family history of breast cancer is not a contraindication. Any breast abnormality should be excluded
before starting HRT. Counsel women considering HRT that it may increase their risk of an abnormal mammogram and that combined HRT may increase their risk of breast cancer after 4-5 years of use.

A personal history or known risk factor for venous or arterial thromboembolic disease, including stroke and cardiovascular disease. If HRT is decided in such patients, a transdermal preparation with minimal estrogen is preferred.

NOTE - In the absence of a personal or family history of venous thromboembolism, screening for inherited thrombophilias before starting HRT is not indicated.

Uncontrolled hypertension.
Abnormal vaginal bleeding - HRT should not be started in women with undiagnosed abnormal vaginal bleeding. Combined HRT may often cause unscheduled bleeding in the first six months of use. Persistent or new onset (after six
months) unscheduled bleeding requires investigation to exclude pelvic disease.

The following precautions should be taken in using HRT:

Avoid oral HRT products in the presence of abnormal liver function tests, since these estrogen and progesterone are metabolized in the liver.
Migraine does not seem to be exacerbated by HRT, but if the patient has migraine, low-dose transdermal preparations are preferred.
History of endometrial or ovarian cancer - seek specialist review before considering HRT.
High risk of gallbladder disease - Advise that HRT may increase the risk gallbladder disease. The risk may be lower with transdermal therapy.

While family history of breast cancer is not a contraindication to use HRT, it should be avoided in patients with personal (not family) history of breast cancer with estrogen or progesterone receptors or both being positive.

(Option B) The safety or efficacy of progesterone alone for treatment of menopausal symptoms in a patient with history of breast cancer is not known.

(Option C) SSRIs are widely used in Australia for management of hot flushes when HRT is contraindicated. They also help with improving mood symptoms. But they lack other effects of HRT such as increasing libido and predervation of bone density.
SSRIs are used for treatment of hot flushes if HRT is contraindicated.

(Option D) Black Cohosh is sometimes used for hot flushes. The mechanism of potential is unknown. Many studies have found no significant benefit over placebo.

(Option E) Oral route is the most effective way of taking HRT. This woman has no condition to necessitate the use of dermal patches of estrogen (which delivers less estrogen)

1046 of 1943
A 64-year-old man is brought to the Emergency Department with confusion. He is a known case of hypertension and heart failure and is on indapamide and enalapril for treatment. Laboratory studies are significant for a serum sodium level of 120
mmol/L. Which one of the following would be the most appropriate next step in management?

A. Restrict oral water intake to 500 ml/day.

B. Hypertonic saline.

C. Stop indapamide.

D. Stop enalapril.

E. Stop both enalapril and indapamide.

Incorrect. Correct answer is B


45% answered correctly

Explanation:

Correct Answer Is B

The clinical presentation in this scenario is most likely to have been caused by hyponatremia. If symptomatic, hyponatremia and hypernatremia present with CNS-related symptoms such as weakness, lethargy, confusion, seizure, coma, and even
death.

Hyponatremia can be caused by a variety of conditions including:

Medications, especially diuretics, ACH inhibitors, Angiotensin receptor blockers (ARBs), etc.
Dehydration
Syndrome of inappropriate ADH (SIADH)
Congestive heart failure
Renal failure
Liver failure
Hormonal diseases e.g. Addison’s disease, hypothyroidism, etc.

Mild hyponatremia is managed with fluid restriction; whereas severe hyponatremia should be treated with prompt sodium replacement. There are debates as to whether normal saline or hypertonic saline should be used, but it is generally accepted
that patients with severe neurological symptoms should initially be treated with hypertonic saline. The following conditions, when present, are indications for the administration of hypertonic saline after expert consultation and with great caution:

Confusion
Seizures
Coma

The first step to consider in the management of this patient is dealing with hyponatremia while an investigation for an underlying cause is carried out. With the presence of confusion, cautious use of hypertonic saline would be the next best step in
management.

The sodium level is not an indicator of severity; the clinical status is. A very low serum sodium level can be well tolerated if it has occurred chronically, whereas an acute fall may result in severe CNS problems and death.

According to Royal ChildreChildren'sal guidelines, lethargy is an indication for the administration of hypertonic saline (3%) in children. It remains unclear whether this sign signifies the need for hypertonic saline use in adults.

Indapamide is a thiazide-like diuretic used for the treatment of hypertension. Hyponatremia and hypokalemia are two well-known adverse effects of thiazide and thiazide-like diuretics. The hyponatremia caused by diuretics can be either hypovolemic
or euvolemic. ACE inhibitors, through a not-fully-understood mechanism, can cause both hypovolemic and euvolemic hyponatremia as well. Enalapril and indapamide should also be stopped (options C, D, and E) and their use and dose, either as
stand-alone medication or in combination, should be reassessed once the patient is stable. Response to the cessation of the medications implicated in hyponatremia is delayed and depends on the half-lives of the drugs. Therefore, stopping the
offending drug alone will not help with the rapid resolution of life-threatening neurological symptoms of severe hyponatremia.

Fluid restriction (option A) is used for patients with asymptomatic or mildly symptomatic patients with hyponatremia and is not appropriate as a stand-alone therapy for this patient with severe CNS symptoms.

References

• Medscape - Hyponatremia in Emergency Medicine

• RCH - Hyponatraemia

Last updated:
Time spent: QID:704 2023-2-12

1047 of 1943
Which one of the following antiepileptic medications does not affect the efficacy of hormonal contraceptive pills?

A. Topiramate.

B. Carbamazepine.

C. Primidone.

D. Levetiracetam.

E. Phenytoin.

Incorrect. Correct answer is D


45% answered correctly

Explanation:

Correct Answer Is D

Epilepsy does not pose any restrictions on the use of hormonal contraceptive methods, but restrictions may apply if certain antiepileptic drugs (AEDs) are used.

Some AEDs can reduce the efficacy of the following hormonal methods:

Combined oral contraceptive pills


Progestin-only pills
Progestin implants (Implanon®)

NOTE – the efficacy of Mirena® and Depo Provera® (medroxy progesterone acetate) are not affected by enzyme inducing AEDs.

The mechanism of action is through inducing more cytochrome P450 activity and more accelerated clearance of hormonal contraceptives. On the other hand, circulating sex-hormone binding globulins are increases and results in decreased levels of
freely circulating contraceptive medications.

The following drugs reduce the efficacy of hormonal contraceptive pills with the mentioned mechanisms:

Phenobarbital
Primidone
Phenytoin
Carbamazepine
Oxcarbazepine
Topiramate

Following antiepileptic drugs are non-inducers and do not affect the efficacy of hormonal contraceptive pills:

Valproic acid
Gabapentin
Levetiracetam
Tigabine
Zonisamide
Pregabalin
Vigabatrin
Ethosuximide
Benzodiazepines
Lamotrigine

If hormonal methods are considered for contraception while the patient is on enzyme-inducing antiepileptic drugs, an increased dose of estrogen, or alternative methods should be used. Mirena® or Depo Provera® can be an excellent choice if not
contraindicated.

NOTE – Combined oral contraceptives (COCs) increases the clearance of lamotrigine and reduce serum lamotrigine levels. Women using lamotrigine should be advised that seizure frequency may increase when initiating COCs while on lamotrigine.

1048 of 1943
A 37-year-old smoker woman presents for contraception advice. She smokes 20 cigarettes a day. Which one of the following is an appropriate option for her?

A. Combined oral contraceptive pills.

B. Condoms.

C. Progestogen-only pills.

D. Diaphragm.

E. Cervical cap.

Incorrect. Correct answer is C


45% answered correctly

Explanation:

Correct Answer Is C

Smoking more than 15 to 20 cigaarettes a day in a woman aged more than 35 year is a contraindication to combined oral contraceptive pills (option A) . Progesterone-only preparations (e.g. progesterone-only pills) are options of choice when
estrogen-containing preparations are contraindicated. Failure rate per year is 3 to 10%, compared to 2 to 6% for combined pill.

(Option B) Male condoms are the next best appropriate option with a Pearl index* of 3 compared to 0.5 of POPs.

(Option D) Diaphragm is dome-shaped rubber cup with a flexible rim that fits over the cervix. It creates barrier to the sperm. The fitness should initially be checked by a medical practitioner. It should remain in place for more than 8 hours after the
coitus. Pregnancy rate is 6% in the first year. It is not routinely used anymore in Australia.

(Option E) Cervical cap is slightly more rigid than diaphragm and is smaller in size. It comes in several sizes. It needs to be fitted by health care practitioner. It can be left in place for 48 hours. Pregnancy rate is over 9%. It is not a favorite and
advisable method of contraception anymore.

*The Pearl index, also called the Pearl rate, is the most common technique used in clinical trials for reporting the effectivenss of a birth control method

1049 of 1943
A 23-year-old university student visits your clinic for advice about contraception. She reports migraine-type headaches once or twice a month for the past 3 months. She already has been on combined oral contraceptive pills for the past 2 years.
Which one of the following would be the most appropriate advice?

A. Discontinuation of combined oral contraceptive pills.

B. Stopping the combined oral contraceptive pills and starting progesterone only pills (POP).

C. Continuation of combined oral contraceptive pills.

D. Refer her to an obstetrician for further management.

E. Tell her to use barrier methods, as those would be safer.

Incorrect. Correct answer is B


45% answered correctly

Explanation:

Correct Answer Is B

Since this patient has developed migraine headache while on combined oral contraceptives (COCs), the best advice would be discontinuation of COCs and using progestogen-only pills (POPs) after discussion of potential adverse effects with her.

COCs are generally considered safe in women who have migraine without aura under the age of 35; however, if the migraine has started after the initiation of COCs, COCs will be relatively contraindicated.

Migraine with aura remains the absolute contraindication to the use of COCs because of the increased risk of ischemic stroke.

Progestogen-only methods provide a useful contraceptive option for any woman, in whom estrogen is contraindicated, irrespective of age or parity. No harmful effects have been found on long-term follow-up of children whose mother used
progesterone-only methods. This makes POPs a good contraceptive option for lactating women as well.

Indications to use progesterone-only methods as the first-line options include:

Hypertension
Superficial thrombophlebitis
History of thromboembolism
Biliary tract disease
Thyroid disease
Epilepsy
Diabetes mellitus without vascular disease
Migraine – see the explanations above
lactation
Chloasma

Contraindications to progesterone-only methods are:

Pregnancy
Breast cancer
Undiagnosed genital tract bleeding
History of or risk factor for ectopic pregnancy
Active liver disease
Liver cirrhosis

1050 of 1943
Which one of the following statements is correct regarding post-coitus contraception with Postinor-2®?

A. Nausea and vomiting is a very common complication seen with Postinor-2®.

B. Virilization of a female fetus is likely if Postinor-2® fails to prevent pregnancy.

C. Menstruation is likely within 7 days of the treatment.

D. Taking the two doses at the same time is associated with lower risk of failure and less adverse effects.

E. Vaginal spotting (breakthrough bleeding) is a common complication seen within 3 days of treatment.

Incorrect. Correct answer is D


45% answered correctly

Explanation:

Correct Answer Is D

Postinor-2 ®, used for post-coital contraception, is a progesterone-only method. The package contains 2 tablets of levonorgestrel 750mcgs. Often one tablet is given within 72 hours of unprotected sexual intercourse and the other 12 hours later;
however, taking the two pills of the package at once seems to be more effective and associated with less adverse effects. The failure rate is 2-3%.

NOTE – according to latest recommendation, the efficacy of Postinor-2® significantly declines after 72 hours of coitus, it is recommended that it be given even beyond 72 hours and up to 5 days.If pregnancy occurs there will be no risk of virilization
of female fetuses.

(Option A) Nausea and vomiting are not a usual complication of Postinor-2 ®.

(Option B) Postinor-2 ® does not cause female fetus virilization if pregnancy occurs.

(Option E) If Postinor-2 ®.is given before days 8 to 10 of the menstrual cycle, breakthrough bleeding within few days of administration is possible, but if it is given in mid-cycle vaginal bleeding would be unlikely and the timing of period (option C) will
not be affected unless the method fails and the patient becomes pregnant.

1051 of 1943
A 36-year-old woman comes to your office for advice regarding oral contraception. She is currently on Microgynon 30 and is satisfied with it. She has 2 children, does not smoke and enjoys good health. On examination, you find that her blood
pressure is 150/100mmHg. Another reading in 20 minutes shows the sameblood pressure. Which one of the following would be the next best step in management?

A. Continue the same OCP, but start her on antihypertensive medications.

B. Stop OCP and give antihypertensive medications.

C. Use an OCP with lower doses of estrogen.

D. Start her on progestogen-only pills (POPs).

E. Continue the same OCP without further action.

Incorrect. Correct answer is D


45% answered correctly

Explanation:

Correct Answer Is D

Evidence suggests that combined oral contraceptive pills (COC) have a small adverse effect on blood pressure with an average increase of 8mmHg for systolic and 6mmHg for diastolic pressure. However, a minority of women may suffer severe
hypertension induced by COC.

Unlike COCs, progesterone-only pills do not appear to induce an increase in blood pressure; therefore the next best step in management is switching to POPs.

In women with well-controlled and monitored hypertension ,who are 35 years or younger, a trial of combined oral contraceptives may be appropriate as long as the patient is otherwise healthy, shows no signs of end-organ vascular disease, and does
not smoke. If blood pressure remains well controlled several months after the trial is started, combination contraceptive may be continued.

1052 of 1943
A 24-year-old woman is at your office for prescription of oral contraceptive pills. She has the history of epilepsy for the past 5 years and is on carbamazepine. She has not had any episode of seizure in the past 6 months. Which one of the following
would be the most appropriate advice?

A. Cease carbamazepine.

B. Reduce the dose of carbamazepine.

C. Prescribe high-dose oral contraceptive pills (microgynon 50).

D. Prescribe low-dose oral contraceptive pills (microgynon 30) and follow up in 3 months.

E. Advise her to use condoms only.

Incorrect. Correct answer is C


45% answered correctly

Explanation:

Correct Answer Is C

Since this patient has been stable on carbamazepine, cessation of carbamazepine or dose reduction is not wise, but it should be remembered that some antiepileptic drugs such as carbamazepine can induce liver cytochrome enzyme activity and
increase the metabolism of the active gradients of oral contraceptive pills (OCPs), progestin-only pills and progesterone implants. Concomitant use of anti-epileptic drugs (AEDs) reduces the efficacy of oral contraceptive pills by 40-50%. This can
cause less effective protection against pregnancy and ‘pill failure’.

With low-dose OCPs (microgynon 30), this reduction in efficacy will result in a 6% failure rate. This is why it is recommended that women on antiepileptic drugs take high-dose OCPs (microgynon 50) to minimize the risk of ‘pill failure’. Even by
such increase, there is a small risk of unplanned pregnancy, and barrier methods can be considered along with microgynon 50 after discussion with the patient.

Mirena® is often the best option to offer, as it is unaffected by enzyme-inducing drugs with provision of excellent protection, and should be offered as an option unless IUCDs are contraindicated. Depo Provera® is another good choice.

NOTE - COCs, POPs and Implanon® are affected by enzyme-inducing drugs. These drugs have no imapct on Mirena® and Depo Provera®.

The following antiepileptic drugs can induce cytochrome enzymes and cause pill failure:

Phenobarbital
Primidone
Phenytoin
Carbamazepine
Oxcarbazepine
Topiramate

1053 of 1943
A 30-year-old woman presents to the emergency department with lower abdominal pain and vomiting. She is restless and anxious. On examination she is sweaty and disturbed. Brown vaginal discharge is noted on vaginal examination. Her last
menstrual period occured six weeks ago. Vital signs include a blood pressure of 105/55 mmHg, heart rate of 102 bpm and respiratory rate of 20 breaths per minute. Which of the following is the best investigation to establish the diagnosis?

A. CT scan of the abdomen.

B. Serum beta hCG.

C. Transvaginal ultrasound.

D. Full blood count.

E. Urine beta hCG.

Incorrect. Correct answer is C


45% answered correctly

Explanation:

Correct Answer Is C

This patient has suspected diagnosis of ruptured ectopic pregnancy that has led to pre-shock.

The most accurate test (not the first) for assessment of suspected ectopic pregnancy is transvaginal ultrasound, which has a sensitivity of 69 to 96% and specificity of 85 to 99%. The first test is alwasy urine pregnancy test to establish a pregnancy.
However, this test is a screenign test, meaning that a negative result at 6 weeks excludes ectopic pregnancy with certainty, while a positive result prompts more evaluation, the most important and diagnostic of which is a transvaginal ultrasound scan
for detection of gestational sac outside the uterine cavity.

(Option A) CT scan of abdomen is not needed and should be avoided considering possible pregnancy.

(Option C) Patients with indeterminate ultrasonography findings require further evaluation with quantitative beta HCG levels. Serum beta HCG and laparoscopy are usually used as secondary tests in women with inconclusive findings on ultrasound.

(Option D) Full blood count helps to determine the degree of blood loss and in guiding fluid resuscitation or blood transfusion. It is not diagnostic.

(Option E) Urine beta hCG is the very first diagnostic test when ectopic pregnancy is suspected. With most current pregnancy test kits (sensitive for as little as 25 units per litre) urine may reveal results 3-4 days after implantation; by 7 days (the time
of expected period) 98% will be positive. With an ectopic pregnancy a urine beta hCG may be falsely negative in more than 50% of women in early pregnancy; however, at 6 weeks it will be positive in as many as 99% of women, and a negative urine
test can safely exclude the ectopic pregnancy.

Another very important test to consider without delay in this situation (not mentioned here) is blood group typing and cross matching, as this patient may need blood transfusion.

1054 of 1943
A 27-year-old woman presents to the Emergency Department with vaginal bleeding and lower abdominal pain. She had her last period 6 weeks ago. She has irregular periods, each lasting variably between 4 and 7 days. On examination, her blood
pressure is 110/75 mmHg, pulse rate 98 bpm and respiratory rate 18 breaths per minutes. The rest of the exam is inconclusive. Which one of the following is the next best step in management?

A. Abdominal ultrasonography.

B. Transvaginal ultrasonography.

C. Laparoscopy.

D. Urine pregnancy test.

E. Serum beta hCG.

Incorrect. Correct answer is D


45% answered correctly

Explanation:

Correct Answer Is D

The triad of amenorrhea, vaginal bleeding, and abdominal pain in women of reproductive age is ectopic pregnancy (EP) until proven otherwise. In order to establish EP as a diagnosis, pregnancy should be diagnosed first. Therefore, the next best step
would be a simple urine pregnancy test. In early pregnancy, the urine pregnancy test may be falsely negative; however, 99% of women with an EP will have a positive urine pregnancy later.

Other steps depend on the result of the urine beta hCG test. A negative test at 6 weeks excludes ectopic pregnancy with high certainty, but if positive the next step would be an transvaginal ultrasonography:

If the gestational sac is seen within the uterus cavity, EP is excluded.

If the gestation sac is seen outside the uterine cavity diagnosis of EP is established. Medical (methotrexate) or surgical (laparoscopy or laparotomy) management should be contemplated as the next step.
It the ultrasonography is inconclusive, serial serum beta hCG is considered. In an intrauterine pregnancy, beta hCG is expected to almost doubles every 48 hours. A less than predicted rise in serum beta hCG is suggestive of EP.

NOTE - transvaginal ultrasonography often shows the EP if the beta-HCG is above 1500-2000. An abdominal sonography is likely to detect the EP only if beta-HHCG is above 5000-6500.

References

• Diagnostic Imaging Pathways - Ectopic Pregnancy (Suspected)

Last updated:
Time spent: QID:191 2023-2-12

1055 of 1943
During an ultrasonsography scan for suspected renal stones, a 28-year-old woman is found to have two ovarian cysts on the left side. The cysts are 10 cm and 12 cm in diameter. Which one of the following is the the most appropriate management
option?

A. Aspiration.

B. Laparoscopy.

C. Laparotomy.

D. Oophorectomy.

E. Conservative management.

Incorrect. Correct answer is B


45% answered correctly

Explanation:

Correct Answer Is B

Ovarian cysts are frequently noted incidentally during ultrasonography for other reasons.

The following incidentally-found ovarian cysts can be ignored without any furtehr follow-up:

Asymptomatic simple cysts ≤3cm in women of reproductive age


Asymptomatic simple cysts ≤1cm in postmenopausal women

Such cysts are almost always of no clinical importance in asymptomatic women and can be safely ignored.

Simple cysts larger than 3 cm in women of reproductive age or larger than 1 cm in postmenopausal women should be described in ultrasound reports.

Although simple cysts of any size are unlikely to be malignant lesions, it is reasonable to perform yearly sonographic follow-up of cysts, including classic-appearing hemorrhagic cysts larger than 5 cm in premenopausal women and larger than 1 cm
in postmenopausal women.

Based on new (2010) recommendations from Radiological Society of North America (RSNA) asymptomatic ovarian cysts in premenopausal women are managed as follow:

Cysts ≤3 cm are normal physiologic findings that do not need follow-up.


For cysts >3 cm, but ≤5cm repeat ultrasound in 6 to 12 weeks.
For cysts>5 cm, but smaller than 7cm, repeat ultrasound in 6 to12 weeks and then yearly follow-up with ultrasound.
As cysts over 7cm may be difficult to be completely assessed with ultrasonography; therefore, further imaging with magnetic resonance (MRI) or surgical evaluation should be considered. Laparoscopy is the method of choice.

This woman has two cysts larger than 7cm in diameter. According to the mentioned guidelines, laparoscopic removal of these cysts (cystectomy) is the most appropriate management option. Laparoscopy can be used for both diagnosis and
treatment.

1056 of 1943
A 20-year-old woman presents to your office with secondary amenorrhea. Pregnancy is excluded by urine pregnancy test. You order a pelvic ultrasound as a part of work-up that reports back a cyst of 1.8 x 1.3 x 1 cm. The cyst is thin-walled and fluid-
filled. There is no pain in history. On examination, not tenderness is elicited. Which one of the following is the most appropriate next step in management?

A. Laparoscopy for cytology.

B. Repeat the ultrasound in 6 weeks.

C. Oral contraceptive pills.

D. Do nothing.

E. Oophorectomy.

Incorrect. Correct answer is D


45% answered correctly

Explanation:

Correct Answer Is D

Ovarian cysts are frequnetly noted incidentally during ultrasonography for other reasons.

The following incidentally-found ovarian cysts can be ignored without any furtehr follow-up:

Asymptomatic simple cysts ≤3cm in women of reproductive age


Asymptomatic simple cysst ≤1cm in postmenopausal women

Such cysts are almost always of no clinical importance in asymptomatic women and can be safely ignored.

Simple cysts larger than 3 cm in women of reproductive age or larger than 1 cm in postmenopausal women should be described in ultrasound reports.

Although simple cysts of any size are unlikely to be malignant lesions, it is reasonable to perform yearly sonographic follow-up of cysts, including classic-appearing hemorrhagic cysts larger than 5 cm in premenopausal women and larger than 1 cm
in postmenopausal women.

Based on new (2010) recommendations from Radiological Society of North America (RSNA) asymptomatic ovarian cysts in premenopausal women are managed as follow:

Cysts ≤3 cm are normal physiologic findings that do not need follow-up.


For cysts >3 cm, but ≤5cm repeat ultrasound in 6 to 12 weeks.
For cysts>5 cm, but smaller than 7cm, repeat ultrasound in 6 to12 weeks and then yearly follow-up with ultrasound.
As cysts over 7cm may be difficult to be completely assessed with ultrasonography; therefore, further imaging with magnetic resonance (MRI) or surgical evaluation should be considered. Laparoscopy is the method of choice.

Since the cyst is less than 3 cm in diameter, no further action or follow-up is required for the cyst. The cause for the presenting symptom (amenorrhea) should be investigated though.

Aspiration of ovarian cysts, either vaginally or laparoscopically, is less effective and associated with a high rate of recurrence.

Oophorectomy is not an appropriate option for women of reproductive age.

1057 of 1943
A 53-year-old woman, who has been on hormone replacement therapy (HRT) for the past two years asks how frequent she should undergo screening for breast cancer. Which one of the following would be the most appropriate response?

A. Annual breast screening is recommended as HRT increases the risk of breast cancer.

B. Mammogram every two years until the age of 70.

C. Breast cancer screening every 6 months.

D. Mammography every 3 months as long as HRT continues.

E. Self breast examination every month and mammography every 6 months.

Incorrect. Correct answer is B


45% answered correctly

Explanation:

Correct Answer Is B

Current Australian guidelines recommend that all asymptomatic women aged 50-69 years undergo screening tests for breast cancer every 2 years.

Currently, there is no evidence that more frequent mammograms improve the ability of early detection of breast cancer in women on HRT. These women should have their screening test similar to other women not on HRT. Women who are started on
HRT should be encouraged to join Australia Breast Screen Program.

1058 of 1943
A 14-year-old girl presents to your clinic with a dark rash on her axillae and around her neck for the past few months. She also complains of fatigue and lethargy. She has regular periods which started at age 12. On examination, she has a BMI of 32,
and blood pressure of 120/80mmHg. She also has abdominal striae. Which one of the following investigations is most likely to establish the diagnosis?

A. 24-hour urinary cortisol.

B. Fasting blood glucose (FBS).

C. Serum TSH.

D. Pelvic ultrasound.

E. Serum ACTH.

Incorrect. Correct answer is B


45% answered correctly

Explanation:

Correct Answer Is B

This girl with a BMI of 34 is obviously obese, and the rash on her axillae and around the neck is acanthosis nigricans, an indicator of insulin resistance. Insulin resistance and increased plasma glucose can also cause lethargy and fatigue, which are
present in history. An FBS, therefore, is most likely to establish the diagnosis.

OF note, the oral glucose tolerance test is more sensitive than FBS for the detection of impaired glucose tolerance or early type 2 diabetes.

Obesity, lethargy, acanthosis nigricans, irregular menstruation, and striae are features seen in metabolic syndrome and polycystic ovarian syndrome (PCOS). With regular periods PCOS is unlikely but with irregular periods both PCOS and metabolic
syndrome could be possible diagnoses.

This girl has regular menstruation. Her blood pressure is also normal. These two, make Cushing’s syndrome less likely; therefore, serum ACTH (option E) and urinary or plasma cortisol levels (option A) are not likely to be abnormal. The striae can be
caused by obesity and overstretching of the abdominal wall.

Fatigue can also be a feature seen in thyroid hormone problems. Although hypothyroidism has fatigue and obesity as its clinical features, the presence of acanthosis nigricans makes this diagnosis less likely; therefore, THS measurement (option C)
probably will not add a diagnostic benefit.

Metabolic syndrome is seen in 25% of patients with PCOS. In patients with PCOS, ultrasonography can show multiple ovarian cysts. PCOS is associated with menstruation abnormalities and is the most common cause of anovulatory cycles and
infertility in women. With regular periods, this girl is unlikely to have PCOS, and ultrasound (option D) is unlikely to be helpful.

References

• UpToDate - The metabolic syndrome, insulin resistance syndrome , or syndrome X

• UpToDate - Epidemiology and clinical manifestations of Cushing's syndrome

Last updated:
Time spent: QID:1070 2023-2-12

1059 of 1943
Which one of the following statements is incorrect regarding Down syndrome screening in a 40-year-old pregnant woman?

A. Dating ultrasound together with second trimester serum screening test has detection rate of 75%.

B. Dating ultrasound together with second trimester serum screening test has detection rate of 97%.

C. Without dating ultrasound, second trimester serum screening test has detection rate of 65%.

D. Chorionic villous sampling has a miscarriage risk of 1 in 100.

E. Amniocentesis is associated with 1 in 200 miscarriage risk.

Incorrect. Correct answer is B


45% answered correctly

Explanation:

Correct Answer Is B

(Option A) Correct statement: second-trimester measurement of biochemical markers for screening has a lower sensitivity of 75% if a first-trimester dating ultrasound has established an accurate gestational age and quadruple marker screening test
(free beta hCG, alpha-fetoprotein, unconjugated estriol and inhibin A level) is used.

(Option B) Incorrect statement: combinatoin of second trimester screening tests and a first trimester dating ultrasound has a maximum detection rate of 75%, not 97%.

(Option C) Correct statemen: with a triple marker screening test (free beta hCG, alpha-fetoprotein and unconjugated oestriol) without first-trimester dating ultrasound the sensitivity declines to as low as 65%.

(Options D and E) Correct satements: chorionic villous sampling and amniocentesis are associated with miscarriage risks of 1 in 100 and 1 in 200, respectively.

NOTE - In the first-trimester, using a combination of ultrasonography for nuchal translucency, pregnancy associated plasma protein A (PAPP-A) and free beta hCG can be as sensitive as 90%; hence the screening tests of choice.

References

• https://www.rcpa.edu.au/getattachment/2ab50a32-563

Last updated:
Time spent: QID:198 2023-2-12

1060 of 1943
Which one of the following maternal serum screening markers can be used in both first and second trimesters for screening of Down syndrome?

A. Alpha fetoprotein level.

B. Inhibin A level.

C. Unconjugated estriol level.

D. Pregnancy-associated plasma protein A level.

E. Free beta HCG level.

Incorrect. Correct answer is E


45% answered correctly

Explanation:

Correct Answer Is E

For first-trimester Down syndrome screening, maternal serum free beta hCG (along with pregnancy-associated plasma protein A (PAPPA)) is measured, whereas total HCG is measured for second-trimester screening. Free beta hCG can be used
instead of total hCG for second-trimester screening. The difference between the detection rates of free beta hCG and total hCG for Down syndrome remains controversial; however, recent studies showed that application of free beta hCG is
associated with a 4% increase in detection of Down syndrome (not certain yet).

Alpha fetoprotein (option A) , inhibin A (option B) and unconjugated estriol levels (option C) are used for screening Down syndrome solely in the second trimester. ‘Pregnancy-associated plasma protein A level (option D) is used as a screening
method only appropriate during the first-trimester.

TOPIC REVIEW

Down syndrome (Trisomy 21) is one of the most common genetic birth defects in Australia, and the most common cause of intellectual disability. The proportion of pregnancies involving women over the age of 35 has increased from about 5% to
20% over the past 20 years, making Down syndrome one of the few congenital defects with an increasing prevalence. The previously quoted incidence of 1:750 has now increased to more than 1:500.

Once a woman is over the age of 35, her risk of a Down syndrome in a pregnancy is 1:300. By the age of 43, her risk increases to about 1:30. It is, therefore, necessary to use screening tools for early detection of Down syndrome in utero.

Tools used for assessment, screening and antenatal diagnosis of Down syndrome include:

SCREENING TESTS

At present there are two recommended sets of antenatal screening tests for Down syndrome:

First trimester screening tests for Down syndrome (9-12 weeks):

Ultrasonography for nuchal translucency (>0.5mm)

PLUS

Maternal serum biomarkers:

Maternal Serum Biomarker in Down syndrome pregnancies


Free beta hCG Increased
Pregnancy-associated plasma protein A Decreased

NOTE - The combined first-trimester screening is the screening test of choice. First-trimester ultrasound of the nuchal translucency (measurement of the translucent area between the fetal skin and the underlying tissue at the nape of the neck) has a
sensitivity of 60-80%, and first-trimester serum screening has a sensitivity of 60-65%. In combination, they can detect 90% of Down syndrome pregnancies (90% sensitivity), while maintaining 95% specificity (i.e. only 5% of normal pregnancies will
give a positive screen).

Second trimester screening tests for Down sydnrome (15-18 weeks)

Triple marker screening tes


Free beta hCG Increased
Alpha-fetoprotein (AFP) Decreased
Unconjugated estriol Decreased
Sensitivity 65-70%

Quadruple marker screening test


Free beta hCG Increased
Alpha-fetoprotein (AFP) Decreased
Unconjugated estriol Decreased
Inhibin A level Increased
Sensitivity 70-75%

NOTE- provided that a first-trimester dating ultrasound has determined a good estimate of the gestational age, the sensitivity of second-trimester screening testing can be as high as 70% and 75% with triple and quadruple marker screening tests,
respectively. Without first-trimester dating ultrasound, the sensitivity of the second-trimester screening tests can be as low as 65%.

DIAGNOSTIC TESTS

Chorionic villous sampling – performed at 12-14 weeks of pregnancy and carries a 1:100 risk of pregnancy loss 1:100
Amniocentesis – performed at 15-18 weeks of pregnancy and carries a 1:200 risk pregnancy loss

1061 of 1943
Consider and discuss these two tests with the patients where:

1. Screening tests whether in first- or second-trimester suggest Down syndrome.


2. The risk of the mother having a Down syndrome exceeds the risk of fetal loss.

References

• https://www.rcpa.edu.au/getattachment/2ab50a32-563

• http://www.ncbi.nlm.nih.gov/pubmed/7687460

Time spent: QID:199 Last updated:


2023-2-12

1062 of 1943
A 35-year old primigravida woman presents to your clinic for her first antenatal check-up. She is 8 weeks pregnant, and is concerned that her baby might have chromosomal abnormalities. She asks for antenatal screening tests. Which one of the
following results at this time would indicate further assessment for trisomy 21?

A. Decreased pregnancy-associated plasma protein A (PAPP-A).

B. Decreased hCG.

C. Increased alpha-fetoprotein.

D. Increased unconjugated estriol.

E. Decreased inhibin A.

Correct
45% answered correctly

Explanation:

Correct Answer Is A

The presence of decreased pregnancy-associated plasma protein (PAPP-A) in the first-trimester of pregnancy is an indication for further diagnostic testing for Down syndrome. PAPP-A is the only serum marker that can be used in the first timester
of pregnancy in combination with free beta-hCG.

Tools used for assessment, screening and antenatal diagnosis of Down syndrome include:

SCREENING TESTS

At present there are two recommended sets of antenatal screening tests for Down syndrome:

First trimester screening tests for Down syndrome (9-12 weeks):

Ultrasonography for nuchal translucency (>0.5mm)

PLUS

Maternal serum biomarkers:

Maternal Serum Biomarker in Down syndrome pregnancies


Free beta hCG Increased
Pregnancy-associated plasma protein A Decreased

NOTE - The combined first-trimester screening is the screening test of choice. First-trimester ultrasound of the nuchal translucency (measurement of the translucent area between the fetal skin and the underlying tissue at the nape of the neck) has a
sensitivity of 60-80%, and first-trimester serum screening has a sensitivity of 60-65%. In combination, they can detect 90% of Down syndrome pregnancies (90% sensitivity), while maintaining 95% specificity (i.e. only 5% of normal pregnancies will
give a positive screen).

Second trimester screening tests for Down sydnrome (15-18 weeks)

Triple marker screening tests


Free beta hCG Increased
Alpha-fetoprotein (AFP) Decreased
Unconjugated estriol Decreased
Sensitivity 65-70%

Quadruple marker screening test


Free beta hCG Increased
Alpha-fetoprotein (AFP) Decreased
Unconjugated estriol Decreased
Inhibin A level Increased
Sensitivity 70-75%

NOTE- provided that a first-trimester dating ultrasound has determined a good estimate of the gestational age, the sensitivity of second-trimester screening testing can be as high as 70% and 75% with triple and quadruple marker screening tests,
respectively. Without first-trimester dating ultrasound, the sensitivity of the second-trimester screening tests can be as low as 65%.

DIAGNOSTIC TESTS

Chorionic villous sampling – performed at 12-14 weeks of pregnancy and carries a 1:100 risk of pregnancy loss 1:100
Amniocentesis – performed at 15-18 weeks of pregnancy and carries a 1:200 risk pregnancy loss

Consider and discuss these two tests with the patients where:

1. Screening tests whether in first- or second-trimester suggest Down syndrome.


2. The risk of the mother having a Down syndrome exceeds the risk of fetal loss.

References

• https://www.rcpa.edu.au/getattachment/2ab50a32-563
L t d t d

1063 of 1943
Last updated:
Time spent: QID:200 2023-2-12

1064 of 1943
A 37-year-old pregnant woman presents to your practice for consultation regarding the chance of her baby having Down syndrome. She has read somewhere that advanced maternal age is a risk factor for Down syndrome. She is concerned and asks
you to advise her on ways she can know whether her baby can be affected. She is 8 weeks pregnant. Which one of the following would you offer as both safest and the most accurate disgnostic tool for exclusion of Down syndrome?

A. Chorionic villus sampling at 10 to 12 weeks of gestation.

B. Ultrasonography at 10 to 12 weeks gestation.

C. Amniocentesis at 16 weeks gestation.

D. Triple marker screen test.

E. Cordocentesis at 18 weeks of gestation.

Incorrect. Correct answer is C


45% answered correctly

Explanation:

Correct Answer Is C

Karyotyping of the fetal cells is the only definite method to exclude Down syndrome with almost 100% accuracy. For this reason, fetal cells should be obtained through either of the following:

Chorionic Villus sampling (CVS)

CVS is an outpatient office procedure performed at 10-12 weeks gestation without anesthesia. Under the guide of ultrasound, a catheter is placed directly into the placental tissue without entering the amniotic cavity. Chorionic villi, which are
placental precursors, are aspirated and sent for karyotyping. The chromosomes of chorionic villi are almost always identical to those of the fetus.

The risk of miscarriage associated with CVS is 0.7%, roughly equivalent to 1 in 100-150 depending on the technique and the operator’s skills.

Amniocentesis

This office-based procedure is performed after 15 and up to 20 weeks gestation without anesthesia. Under the guide of ultrasound, a needle is placed into the amniotic cavity and amniotic fluid is aspirated. The fluid contains desquamating live fetal
cells (amniocytes) which can be used for karyotyping. Furthermore, the amniotic fluid itself can be analyzed for alpha-fetoprotein and acetylcholine esterase as screening parameters in neural tube defects (NTD). NTDs are associated with increased
alpha-fetoprotein and elevated acetylcholinesterase activity.

The risk of miscarriage associated with amniocentesis is 0.5% equivalent to 1 in 200 pregnancies.

Percutaneous umbilical blood sample (PUBS) or cordocentesis

Under ultrasound guidance, blood is aspirated from the umbilical vein. The procedure is performed after 20 weeks gestation without anesthesia. The blood sample can be used for karyotyping, ABG, or titration of IgM and IgG antibodies.

The risk of miscarriage with PUBS is 1-2% equivalent to 1 in 50-100 pregnancies.

Although all of the above methods can obtain fetal cells for karyotyping, the method that carries the least risk is amniocentesis.

Triple marker screen test (maternal serum alpha-phetoprotein, estradiol and β-HCG) and ultrasonography for nuchal fold translucency are screening tests and never used as definitive methods of diagnosis.

References

• https://www.nhmrc.gov.au/_files_nhmrc/file/your_he

• https://www.rcpa.edu.au/getattachment/2ab50a32-563

Last updated:
Time spent: QID:202 2023-2-12

1065 of 1943
A 35-year-old woman presents for counselling regarding Down syndrome. She is planning to conceive in the next few months and is concerned because she was told by one of her friends that she is at risk of having a baby with Down syndrome due
to advanced age. She wants to discuss this issue with you. Which of the following statements is true regarding second-trimester screening tests for Down syndrome?

A. Elevated alpha-phetoprotein in the second trimester suggests Down syndrome.

B. Elevated unconjugated estriol in the second trimester suggests Down syndrome.

C. Decreased free beta hCG in the second trimester suggests Down syndrome.

D. Decreased inhibin A level in the second trimester suggests Down syndrome.

E. Every screening test should be interpreted in conjunction with the gestational age.

Incorrect. Correct answer is E


45% answered correctly

Explanation:

Correct Answer Is E

Triple marker screen test is done in the window of 15 to18 (up to 20) weeks gestation. Since the reference values are gestational age-specific, accurate dating is important.

Biomarkers measured in triple-marker screening test include:

1. Maternal serum alpha-fetoprotein (AFP)


2. Maternal serum free beta hCG
3. Maternal serum unconjugated estriol

AFP is the major serum glycoprotein of embryo. AFP concentration peaks at 12 weeks in the fetus and the amniotic fluid, then rises in the maternal serum until 30 weeks. Fetal structural defects such as neural tube or ventral wall defects cause
spillage of more AFP into the amniotic fluid and maternal blood. Other causes of increased AFP include:

Multiple pregnancies
Placental bleeding
Fetal renal disease
Sacrococcygeal teratoma

Maternal serum AFP (MS-AFP) is reported in ‘multiples of median’ (MoM). The most common causes of increased (>2.5 MoM) or decreased (<0.85 MoM) MS-AFP on screen test is dating errors. Therefore, the next best step, when the values are not
within the normal range, would be ultrasonography for accurate dating.

MS-AFP alone is only 20% sensitive in detection of Down syndrome. With triple marker screen, the sensitivity rises to 70% if accurate dating based on first-trimester dating sonography is performed.

With Down syndrome (trisomy 21), MS-AFP and estriol are decreased, but β-hCG is increased.
With Edward syndrome (trisomy 18) all three markers are decreased

Quadruple marker screen:

By adding the forth marker, inhibin A, to triple marker screen, there will be an increase of up to 75% in detection of Down syndrome. Inhibin A is increased in Down syndrome.

None of these markers, however, are predictive of Down syndrome unless they are interpreted in conjunction with accurate gestational age. This is why ultrasonography for accurate dating is an indispensible part of marker screen tests.

Once the test is abnormal with reference to normal values of an accurately estimated gestational age, the next step would be amniocentesis for amniotic fluid AFP, acetylcholinesterase activity and karyotyping.

References

• https://www.rcpa.edu.au/getattachment/2ab50a32-563

Last updated:
Time spent: QID:203 2023-2-12

1066 of 1943
Katherine, 28 years old, presents at your office with soreness down below at 37 weeks pregnancy. The physical exam is highly suggestive of genital herpes. She has not had such lesions before and this is the first time she is experiencing such
problem. You take swabs for PCR and culture with the results confirming the diagnosis of primary herpes simplex infection. Which one of the following would be the most appropriate next step in management?

A. Vaginal delivery.

B. Prophylactic antiviral therapy.

C. Give acyclovir to the neonate after delivery.

D. Topical lignocaine.

E. Antibiotics.

Incorrect. Correct answer is B


45% answered correctly

Explanation:

Correct Answer Is B

The risk of transmission of herpes simplex virus to the neonate and neonatal infection significantly increases if a pregnant woman develops primary herpes simplex infection after 30 weeks gestation.

Risk factors for intrapartum herpes simplex infection of the baby include:

Premature labor
Premature rupture of membrane
Primary herpes simplex infection near the time of delivery
Multiple lesions in the genital area

The most appropriate management includes:

Checking for herpes simplex infection with PCR testing (cervical swab)
Prophylactic antiviral therapy of the mother from 36th week until delivery
Cesarean section delivery

For this pregnant woman, antiviral therapy should be started and cesarean section be performed to minimize the risk of vertical transmission.

References

• Australian society for infectious diseases – Management of Perinatal Infections (2014)

• https://www.sahealth.sa.gov.au/wps/wcm/connect/91b

Last updated:
Time spent: QID:204 2023-2-12

1067 of 1943
Martha, 42 years of age, has developed a rash characteristic of chicken pox today, just 5 days prior to her expected date of delivery. She is 37 weeks pregnant. Which one of the following is the most appropriate management to prevent neonatal
chickenpox?

A. Antiviral therapy.

B. Consultation with an infectious diseases expert.

C. Varicella zoster immunoglobulin to the mother before delivery.

D. Varicella zoster immunoglobulin to the baby after delivery.

E. Follow up for early onset sepsis.

Incorrect. Correct answer is D


45% answered correctly

Explanation:

Correct Answer Is D

Management of neonates exposed to maternal varicella zoster (maternal chickenpox) is according to the following guidelines:

Presentation of the symptoms >7 days before delivery:

No varicella zoster immunoglobulin (VZIG) required.


No isolation required.
Encourage breastfeeding.
No other interventions required even if baby has chickenpox at or very soon after birth, unless preterm < 28 weeks gestation or low birth weight <1,000 g.

NOTE - Very preterm neonates (≤ 28 weeks gestation) born with chickenpox should receive intravenous aciclovir 20 mg/kg every 8 hours as a slow infusion (in 1-2 hours)

Maternal chickenpox 7 days before to 2 days after birth

Give newborn VZIG 200 IU (one vial) intramuscularly (IM) immediately after birth. VZIG should be given as soon as possible within the first 24 hours of birth but may be given up to 72 hours.
Discharge term neonates as soon as possible.
No isolation required.
Encourage breastfeeding.

Maternal chickenpox > 2 to 28 days after birth

If neonate < 28 weeks gestation or 1000 g birth weight, give VZIG (preferably within 96 hours but can be given up to 10 days post-maternal rash.
Due to the increased risk of severe varicella in newborns of seronegative women (if the mother has no personal history of infection with VZV), give VZIG to neonates exposed to varicella between 2 to 28 days of age.
Discharge term neonates as soon as possible.
No isolation required.
Encourage breastfeeding.

NOTE - Some experts give VZIG 200 IU (one vial) IM when mothers develop chickenpox to term babies who are more than 2 to 28 days of age but there is little evidence to support this.

This woman has developed chickenpox 5 days before delivery; therefore, the neonate should receive ZIG immediately after birth. Although antiviral therapy, preferrably with acyclovir, will benefit the mother if she presents within 24 hours of
appearance of the rash, it will not effectively reduce the risk of transmission to the neonate at this stage of pregnancy.

References

• http://www.sahealth.sa.gov.au/wps/wcm/connect/a69e

• https://www.asid.net.au/documents/item/368

Last updated:
Time spent: QID:205 2023-2-12

1068 of 1943
Which one of the following is an indication of cesarean delivery in a young woman at 37 weeks gestation?

A. Hepatitis C.

B. Hepatitis B.

C. Primary herpes simplex infection with multiple cervical lesions.

D. Recurrent genital herpes infection before 30 weeks.

E. Vaginal Candidiasis.

Incorrect. Correct answer is C


45% answered correctly

Explanation:

Correct Answer Is C

The following are the indications of cesarean section if a pregnant woman has genital herpes:

There are active lesions in the cervix or vulva at the time of delivery or within the preceding 4 days
Rupture of membrane for more than four hours in a mother with history of recent genital herpes

Risk of intrapartum and vertical transmission of herpes simplex to the neonate is high if the mother develops the infection after 30 weeks.

Cesarean delivery is not required in pregnant women with hepatitis C, B, or vaginal Candidiasis; however, invasive procedures such as fetal scalp electrodes and fetal scalp blood sampling should be avoided, because they may increase the risk of
neonatal infection with hepatitis B and C.

References

• Australian society for infectious diseases – Management of Perinatal Infections (2014)

Last updated:
Time spent: QID:206 2023-2-12

1069 of 1943
A 10-month-old female infant is brought to you for assessment. She was born via an uneventful vaginal delivery. Her mother was found to have positive titers of anti-hepatitis C virus antibodies. Which one of the following would the most appropriate
next step in management of this baby?

A. Wait until 18 months and check the baby’s anti HCV antibody titre.

B. Check HCV RNA serology (PCR) now.

C. Refer to specialist for further management.

D. Start the baby on ribavirin and interferon.

E. Reassure that these antibodies are from the mother’s blood and would be cleared with time.

Correct
45% answered correctly

Explanation:

Correct Answer Is A

The Royal Australian and New Zealand College of Obstetricians and Gynecologists recommend that all infants of HCV positive mothers should be screened at 12-18 months of age to determine whether they have been infected. The delay allows full
clearance of maternal antibody in the infant and avoiding confusing results. Screening should be performed by antibody and, if positive, HCV RNA by PCR.

Since this child is only 10 months, the next best step would be waiting until she reaches 18 months to let the circulating antibodies cleared. If the antibody titer remains positive beyond this time, HCV RNA serology would be the next step.

Positive HCV RNA serology indicates presence of hepatitis C virus and infection of the infant. If that is the case, the neonate should be referred to a specialist for further management.

References

• Australian society for infectious diseases – Management of Perinatal Infections (2014)

• http://www.sahealth.sa.gov.au/wps/wcm/connect/1a77

Last updated:
Time spent: QID:207 2023-2-12

1070 of 1943
A 27-year-old pregnant woman presents to your practice in the first trimester of her pregnancy for routine antenatal care. On investigations, she is found to be positive for hepatitis C virus antibody (HCVAb). On discussing the matter with her, she
becomse extremely concerned about transmission of the disease to her baby. Which one of the following is correct about the condition?

A. Cesarean section can prevent the transmission.

B. Fetal scalp blood sampling should be avoided.

C. Breastfeeding is contraindicated for her.

D. The baby should be screened for hepatitis C shortly after delivery.

E. Co-existence of HIV has no effect on HCV transmission.

Incorrect. Correct answer is B


45% answered correctly

Explanation:

Correct Answer Is B

Hepatitis C infection is usually contracted from needle sharing. Fifty to 70% of intravenous drug users are hepatitis C positive. Needle sharing is the most common mode of hepatitis C transmission in Australia. Other modes of transmission include
transfusion of contaminated blood products, and contaminated instruments e.g. tattooing and body piercing.

If a mother is serologically positive for hepatitis C virus, there is a 5% average risk of vertical transmission to the fetus. If there is co-infection with HIV, the transmission rate will increase to as high as 25%. Measures such as fetal scalp electrode and
fetal scalp sampling should be avoided, as these measures increase the risk of fetal infection.

Currently, there is no clear evidence suggesting that cesarean delivery (option A) or avoiding breast feeding (option C) decreases the rate of transmission from mother to baby; however, it is recommended that if the nipples are cracked and there is
bleeding, breast milk be expressed and given to the baby.

NOTE - Ribavirin and interferon are used to treat chronic hepatitis in non-pregnant women, but should be avoided in pregnant women as ribavirin is teratogenic.

All infants of HCV positive mothers should be screened at about 18 months of age (not shortly after devlivery (option D)) to determine whether they have been infected. The delay allows full clearance of maternal antibody from the infant and
avoiding confusing results. The mother should be screened by means of antibody titer, and if positive, HCV RNA by PCR. The HIV shoud also be checked if not previously done.

One very important precautionary measure is avoiding intrauterine procedures that can lead to skin breach exposure of the fetus to maternal blood (e.g. fetal scalp blood sampling).

References

• Australian society for infectious diseases – Management of Perinatal Infections (2014)

• http://www.sahealth.sa.gov.au/wps/wcm/connect/1a77

Last updated:
Time spent: QID:208 2023-2-12

1071 of 1943
Which one of the following is definitely not an adverse effect of long-term combined hormone replacement therapy (HRT)?

A. Myocardial infarction.

B. Breast cancer.

C. Deep vein thrombosis.

D. Pulmonary embolism.

E. Bowel cancer.

Incorrect. Correct answer is E


45% answered correctly

Explanation:

Correct Answer Is E

The following table (from NHMRC) shows the effect of combined HRT and estrogen-only HRT on incidence of different conditions:

Assessed system or condition Effects of HRT


Cancers
Combined HRT increases the risk of cancer.
Estrogen-only HRT shows little or no increased incidence.
Breast cancer

Combined HRT shows little or no increased incidence.


Estrogen-only HRT increases the risk of cancer.
Endometrial cancer

For combined HRT, there is not enough evidence.


Estrogen-only HRT slightly increases the incidence.
Ovarian cancer

Combined HRT reduces the incidence.


No change with estrogen-only HRT.
Colon cancer

Cardiovascular
Combined HRT increases the incidence.
For estorgen-only HRT, there is not enough evidence.
DVT and PE

CAD HRT does not seem to protect post-menopausal women against coronary artery disease as it was previously thought.
Stroke HRT increases the incidence.
Bones
Osteoporosis and fractures HRT decreases the incidence of osteoporosis and fractures.
Other conditions
Gallbladder diseases HRT increases the incidence very slightly if at all.
Cognition and dementia There is not enough evidence whether HRT helps to preserves the cognitive function and prevents dementia.

According to the above table, HRT does not increase the risk of bowel cancer.

1072 of 1943
A 14-year-old girl presents to your clinic with complaints of significant fatigue and dark rash in her axillae and around her neck that have been present for the past few months. Her menses are regular. Her family history is only notable for type 2
diabetes mellitus in her grandmother. On examination, she has a BMI of 32, and blood pressure of 120/80mmHg. She has also abdominal striae. Which one of the following could be the most likely diagnosis?

A. Cushing's syndrome.

B. Addison disease.

C. Metabolic syndrome.

D. Polycystic ovarian syndrome (PCOS).

E. Hypothyroidism.

Incorrect. Correct answer is C


45% answered correctly

Explanation:

Correct Answer Is C

Obesity and the dark rash which is characteristic of acanthosis nigricans are suggestive of insulin resistance. The presence of lethargy and fatigue is also suggestive of hyperglycemia. This presentation can be seen in a variety of conditions such as
Cushing’s syndrome, metabolic syndrome, and PCOS.

This girl, however, has normal blood pressure, making Cushing’s syndrome (option A) less likely. Between 74% to 87% of patients with Cushing's syndrome are hypertensive. Abdominal striae can be simply due to obesity and abdominal wall
overstretch.

Menstrual irregularities or amenorrhea is a characteristic feature seen in PCOS as well as in obesity, especially centripetal obesity. Patients with obesity and metabolic syndrome might or might not have menstrual abnormalities, but in the absence of
such complaint PCOS (option D) is excluded, as impaired menstruation is one of the criteria required for PCOS to be the diagnosis.

Hypothyroidism (option E) can also present with obesity and fatigue, but acanthosis nigricans is not a feature seen in hypothyroidism.

Addison disease (option B)should always be among differential diagnoses in patients with fatigue. Adrenal deficiency results in increased production of ACTH with melanocyte-stimulating hormone (MSH) as a co-product. The elevated MSH level
results in increased melanin synthesis and hyperpigmentation. The hyperpigmentation is generalized but is most prominent and conspicuous in light-exposed areas such as the face, neck, and back of the hands, areas exposed to chronic friction or
pressure such as elbows, knees, spine, knuckles, waist (belt), and shoulders (brassier straps), and in areas that are normally pigmented, such as the areola, axillae perineum, and umbilicus. The rash only in the axillae and around the neck makes
Addison's disease less likely, but not excluded from differentials. Cardiovascular symptoms of adrenal insufficiency include postural dizziness or syncope. In most patients the blood pressure is low, but some have only postural (orthostatic)
hypotension. Normal blood pressure in this patient makes Addison’s disease less likely, but again not impossible.

By exclusion, metabolic syndrome (Mets) could be the most likely diagnosis in this patient.

Mets is a cluster of risk factors including:

Excess abdominal weight


Lipid abnormalities
Hypertension
Elevated glucose levels

According to current guidelines in Australia, metabolic syndrome is defined by the presence 3 of the 5 risk factors summarized in the following table (2001 National Cholesterol Education Program/ATP III):

Measure Categorical cut point


Increased waist circumference Depends on the population
Elevated triglyceride levels or treatment for elevated triglycerides ≥1.7 mmol/L (>150mg/dl)
Reduces HDL-C (or treatment for reduced HDL-C) <1.0mmol/L (40mg/dl) for men <1.3mmol/L (50mg/dl) for women
Elevated blood pressure (or treatment for blood pressure) Systolic≥130 or diastolic≥85
Elevated fasting blood glucose or treatment for elevated blood glucose >5.5mmol/L (>100mg/dl)

Although none of these criteria are explicitly mentioned in the scenario, metabolic syndrome is the most likely diagnosis by exclusion.

References

• UpToDate - The metabolic syndrome, insulin resistance syndrome, or syndrome X

• UpToDate - Epidemiology and clinical manifestations of Cushing's syndrome

Last updated:
Time spent: QID:1071 2023-2-12

1073 of 1943
A 27-year-old woman, previously intravenous drug abuser, presents at 8 weeks pregnancy for evaluation. On assessment, she is found to have positive hepatitis C antibody (HCVAb). Which one of the following would be the next best step in
management?

A. Termination of the pregnancy.

B. Maternal blood PCR for hepatitis C, perinatally.

C. Screen the infant at 12 months.

D. Reassure her that the risk of vertical transmission is negligible and nothing needs to be done.

E. Screen the infant at 18 months.

Incorrect. Correct answer is B


45% answered correctly

Explanation:

Correct Answer Is B

It is recommended that pregnant women wth a positive hepatitis C virus (HCV) antibody titer, have a PCR test for HCV RNA and liver function tests because the risk of perinatal transmission depends on the presence of HCV RNA. The PCR detects
the presence or absence of the virus and the viral load in the blood, and the genotype.

Liver function tests should be performed at the time of checking HCV RNA status. Since concomitant HIV infection increases the risk of transmission, HIV status should also be checked if not already performed.

With a mother serologically positive for hepatitis C, transmission rate to fetus would be 5%, and higher if the mother becomes infected during pregnancy.

Termination of pregnancy is not advised in this situation; rather the patient should be educated about the risk of transmission of hepatitis C to the infant and screening the infant at 12-18 month of age for hepatitis C. If the infant is found to have
hepatitis C, specialist care would be required.

References

• Australian society for infectious diseases – Management of Perinatal Infections (2014)

• http://www.health.gov.au/internet/publications/pub

Last updated:
Time spent: QID:209 2023-2-12

1074 of 1943
Jane, 25 years old, presents to you at 14 weeks gestation, and is extremely concerned because this morning she developed a rash characteristic of chickenpox after 2 days of low-grade fever and mild malaise. Serology studies show positive anti-
varicella IgM. Which one of the following would be the most appropriate course of action in this situation?

A. Antiviral therapy and pelvic ultrasound.

B. Varicella zoster immunoglobulin.

C. Repeat serology in one week.

D. Give antibiotics.

E. No action is needed.

Correct
45% answered correctly

Explanation:

Correct Answer Is A

Mary has clinical varicella zoster infection (chickenpox), confirmed by a positive titre of IgM against varicella zoster. Pregnant patients with clinical varicella zoster infection should be managed as follows:

Chicken pox without complications:

Rash≤24 hours – give oral antivirals (aciclovir)


Rash>24 hours – no treatment is required.

Chicken pox with complications or in immunocompromised patients:

Intravenous acyclovir

NOTE - chicken pox is considered complicated if:

There are respiratory symptoms


There is hemorrhagic rash or bleeding
New pocks develop >6 days
There is persistent fever >6 days
There are neurological symptoms

This patient should also have ultrasound to check if the baby has developed any anomaly.

(Option B) Varicella zoster immunoglobulin (VZIG) is the treatment of choice if a pregnant woman comes to contact to a case with active varicella zoster. VZIG is not effective once the clinical disease establishes.

(Option C) Since the serology is positive for varicella zoster, further serological tests would not be necessary.

(Option D) Varicella zoster is a viral illness and antibiotics have no role in treatment.

(Option E) Maternal infection can follow after exposure to infectious cases of chicken pox. Doing nothing or reassurance is not a correct action because the mother’s infection carries the risk of fetal infection and resultant congenital anomalies
including:

Skin scarring
Eye defects
Limb hypoplasia
Prematurity and low birth weight
Cortical atrophy, mental retardation
Poor sphincter control
Early death

References

• Australian society for infectious diseases – Management of Perinatal Infections (2014)

• http://www.sahealth.sa.gov.au/wps/wcm/connect/a69e

Last updated:
Time spent: QID:210 2023-2-12

1075 of 1943
A 30-year-old woman presents to your clinic at 26 weeks gestation. She is worried because 24 hours ago, she came in contact with a child who had chickenpox. Her pre-pregnancy IgG against chickenpox was negative, and she missed her prenatal
chickenpox vaccine. On examination, she is asymptomatic. Which one of the following would be the next best step in management?

A. Check varicella zoster virus IgM level.

B. Give varicella vaccine.

C. Check IgG level.

D. Give varicella zoster immunoglobulin (VZIG).

E. No action is needed because she is asymptomatic.

Incorrect. Correct answer is D


45% answered correctly

Explanation:

Correct Answer Is D

In the absence of a postive IgG antibody in prenatal assessment, the patient should be informed of her susceptibility to varicella infection, and warned about risks of exposure to other individuals, who may have acute varicella.

If a susceptible pregnant woman is exposed to someone with chickenpox, serologic assessment for varicella zoster virus (VZV IgG) should be performed and varicella zoster immunoglobulin (VZIG) given as prophylactic treatment if the patient is
within the 96 hours of exposure.

If >96 hours, the following patients should still be treated prophylactically with aciclovir (preferred) or valacilovir after exposure:

If in the second half of pregnancy


There is a history of an underlying lung disease
The woman is immunocompromised
The woman is smoker

The patient has no history of chickenpox or vaccination against VZV, and a prenatal IgG has been negative. For this patient serology is not required. Since she has presented within 96 hours of exposure, the next step in management would be treating
her with prophylactic IVIG (VZIG).

(Option A) Measuring IgM level could be helpful if there was an atypical presentation of varicella zoster infection. This patient is completely asymptomatic at this stage; hence, checking IgM level would be unnecessary.

(Option B) Chicken pox vaccine is a live attenuated vaccine and contraindicated throughout pregnancy.

(Option C) This patient has a recent negative IgG level and has had no infection to make her immune. Her IgG is negative and no IgG measurement is required.

(Option E) This patient needs follow-up for possible infection. Taking no action is incorrect.

References

• https://www.asid.net.au/documents/item/368

• http://www.sahealth.sa.gov.au/wps/wcm/connect/a69e

Last updated:
Time spent: QID:211 2023-2-12

1076 of 1943
Which one of the following will best minimize the risk of mother-to-child transmission of human immune deficiency virus?

A. Cesarean section.

B. Use of zidovudine.

C. Elective cesarean section at term and giving zidovudine to the neonate.

D. Combined triple antiviral therapy for the neonate.

E. Induction of delivery as early as possible in pregnancy consistent with maintaining good fetal maturity.

Incorrect. Correct answer is C


45% answered correctly

Explanation:

Correct Answer Is C

Perinatal HIV transmission rates have been reduced by more than 70 % through:

Treatment of the mother and baby with antiretroviral treatment (ART)


Elective cesarean section
Not breastfeeding

The Pediatric AIDS Clinical Trial Group (PACTG 076) study showed that administration of zidovudine alone, or in combination with other retroviral drugs, to HIV infected women and their newborn, reduces the risk of perinatal transmission by
approximately two thirds (from 25.5 % to 8.3 %). When antiretroviral therapy is combined with elective cesarean section, the risk of vertical transmission falls to 2%.

Rates of transmission are increased in case of:

Advanced maternal illness


High maternal viral load (due to advanced infection or viral activity)
Poor maternal immune status e.g. low CD4 count (also known as T cell count)
Rupture of membranes > 4 hours before birth
Preterm labor
Vaginal delivery
Procedures that may jeopardize the integrity of natural barriers (e.g. fetal scalp electrodes, vigorous suctioning, injections through unwashed skin)
Breastfeeding

References

• http://www.sahealth.sa.gov.au/wps/wcm/connect/72e4

Last updated:
Time spent: QID:212 2023-2-12

1077 of 1943
A 27-year-old woman comes to your clinic at 16 weeks gestation for review of her blood tests, which is positive for cytomegalovirus (CMV) IgM. She is asymptomatic and clinical examination is unremarkable. Which one of the following options is the
most appropriate next step in management?

A. Termination of the pregnancy.

B. Antiviral medications during pregnancy.

C. Amniocentesis.

D. Reassurance.

E. Isolation from the family until delivery.

Incorrect. Correct answer is C


45% answered correctly

Explanation:

Correct Answer Is C

CMV (a DNA virus) is the most common cause of intrauterine infection and the most common viral cause of birth defects. Primary infection of the mother is often asymptomatic but may present as flue-like symptoms. Up to 50% of pregnant women
are CMV IgG seropositive.

Vertical transmission from mother to fetus occurs mainly during the viremia of a primary infection. However, since the result of primary infection is predisposition to a residual life-long latency, fetal infection can occur with reactivation as well.

Transplacental fetal infection is 50% with primary infection, regardless of the gestational age but less than 1% with reactivation of a latent infection.

The infected newborn will be symptomatic in 10 % of cases, but of these 10% almost 90% carry a risk of sequelae including:

Microcephaly
Ascites
Hydrops fetalis
Oligo or polyhydramnios
Hepatomegaly
Pseudomeconium ileus
Hydrocephalus (ventricular dilation)
Intrauterine growth restriction (IUGR)
Pleural or pericardial effusions
Intracranial calcification
Abdominal calcification
Hearing loss

Serologic testing for cytomegalovirus is recommended for the following women in pregnancy:

History suggestive of CMV illness


Exposure to known CMV infected individual or blood product
Immunocompromised
Abnormalities on routine antenatal ultrasound (usually at 18 weeks)

The serology results are interpreted as follows:

A patient with positive IgG but negative IgM has a had past exposure

Changing from an IgG-negative to an IgG-positive state (seroconversion) or a significant rise in IgG indicates a recent primary CMV infection

If the patient has a positive IgM with or without a positive IgG, the result is equivocal and the test should be repeated in 2-4 weeks. The reasons for this include:

IgM can remain positive for over one year after an acute infection; therefore, presence of CMV IgM is not helpful for timing of the onset of infection.
IgM is only positive in only 75-90% of women with acute infection
IgM can revert from negative to positive in women with CMV reactivation or reinfection with a different strain

NOTE - Interpretation of CMV IgM results in pregnancy requires specialist opinion.

Fetal diagnosis is best achieved by a combination of fetal ultrasound, amniocentesis + /- fetal serology; however, the definite diagnosis of fetal infection is by amniocentesis and PCR or the amniotic fluid for CMV. It should be born in mind that
positive results do not predict any degree of fetal damage.

Since this woman is 16 weeks pregnant, amniocentesis for definite diagnosis is the most appropriate option. Amniocentesis is perfomed in the rather small window of 15-18 (up to 20) weeks. However, repeating the test in 2-4 weeks was the most
appropraite option if it was an option.

References

• Australian society for infectious diseases – Management of Perinatal Infections (2014)

• Murtagh’s General Practice – McGraw Hill – 5th Edition – page 1023

• http://www.sahealth.sa.gov.au/wps/wcm/connect/9317

Last updated:
Time spent: QID:213 2023-2-12

1078 of 1943
Which one of the following conditions will require intra-partum antibiotics prophylaxis against group B streptococcus (GBS)?

A. Elective cesarean section at 37 weeks gestation in a woman with positive swab culture for GBS.

B. GBS carrier in the previous pregnancy, but negative carrier state in current pregnancy.

C. Threatened preterm labor with intact membranes.

D. GBS carrier state established by low vaginal swab culture results at 34 weeks.

E. Labor at 38 weeks pregnancy with no maternal risk factor for GBS sepsis in the baby.

Incorrect. Correct answer is D


45% answered correctly

Explanation:

Correct Answer Is D

In the following situations, prophylaxis against group B streptococcus is NOT required:

Elective cesarean section (no labor, no rupture of membrane), irrespective of carriage state or gestational age.

Threatened preterm labor with intact membranes and labor after 37 weeks with no maternal risk factors for GBS sepsis in the baby (threatened preterm labor is different from preterm labor that needs GBS prophylaxis if the woman is a
carrier or the carrier state is not known).

Group beta streptococcus carriage in a previous pregnancy, but negative carriage state in current pregnancy, provided that no other indications for prophylaxis against GBS exists.

Of the given options, only ‘group B streptococcus colonization in current pregnancy at 34 weeks is the only indication for intra-partum antibiotic prophylaxis.

TOPIC REVIEW

Group B streptococcus (GBS or Streptococcus agalactiae) is an encapsulated gram-positive coccus that colonises the gastrointestinal and genital tracts of 15 to 40% of pregnant women. Although GBS colonisation usually remains asymptomatic in
these women, maternal colonisation is the critical determinant of infection in neonates and young infants (less than 90 days of age), in whom GBS is the most common cause of bacterial infection. Vertical (mother-to-child) transmission primarily
occurs when GBS ascends from the vagina to the amniotic fluid after onset of labor or rupture of membranes, but can occur with intact membranes as well.

To reduce the risk of mortality and morbidity from neonatal GBS infections, prophylactic antibiotics are used in certain groups.

Generally, there are two approaches for deciding as to whether intrapartum antibiotic prophylaxis should be given:

Culture-based approach:

Women are screened for GBS by taking low vaginal and rectal swabs. All women with who are culture-positive should receive intrapartum prophylactic antibiotics (penicillin; cefazolin if penicillin-allergic)

Risk factor-based approach:

Women with the following risk factors should receive prophylactic antibiotics:

Intrapartum fever ≥38ºC


Delivery before 37 weeks of gestation (preterm labour)
Rupture of membranes ≥18 hours
Previous delivery of an infant affected by GBS disease
GBS bacteriuria (≥10 4 cfu/mL) in the current pregnancy

NOTE – according to recent studies, culture-based approach is superior to risk factor-based one, as most woman whose baby developed early complications of GBS sepsis had no risk factors. For this reason it is recommended that all
women be screened at 35-37 weeks’ pregnancy and receive prophylactic antibiotics intrapartum.

Latest recommendations suggest that the following receive intrapartum prophylaxis:

Positive screening culture for GBS from either vagina or rectum in the current pregnancy
Positive history of birth of an infant with early-onset GBS disease
GBS bacteriuria during the current pregnancy
Unknown culture status (culture not performed or result not available) and either of the following:

Intrapartum fever (≥100.4ºF, ≥38ºC)


Preterm labor (<37 weeks of gestation)
Prolonged rupture of membranes (≥18 hours)
Intrapartum NAAT (nucleic acid amplification test) e.g. PCR positive for GBS

Intrapartum antibiotic prophylaxis against GBS is NOT recommended for women with:

In a previous pregnancy, a positive GBS rectovaginal culture or GBS bacteriuria and none of the indications for prophylaxis listed above

Women with a positive GBS culture who undergo elective caesarean delivery (at any gestational age) without labour or rupture of membranes; these patients however should undergo routine vaginal and rectal screening for GBS at 35 to
37 weeks because onset of labor or rupture of membranes may occur before the planned cesarean delivery.

Pregnant women with negative GBS cultures at 35 to 37 weeks of gestation, even if they have one or more of the following intrapartum risk factors: intrapartum fever ≥38ºC, preterm labor (<37 weeks of gestation), or prolonged rupture
of membranes (≥18 hours). However, the use of broad-spectrum intrapartum antibiotics for the treatment (not prophylaxis) is indicated for febrile women in labor if they have clinical evidence of chorioamnionitis.

1079 of 1943
References

• Queensland Clinical Giudelines - Early onset Group B Streptococcal disease

Last updated:
Time spent: QID:214 2023-2-12

1080 of 1943
Mary, 33 years old, presents to the Maternity Unit for delivery at 38 weeks gestation. She has had all her antenanal visits until now, but there is no record of GBS status in her file. When you discuss the issue with her, you learn that her general
practitioner failed to take swabs for culture. Mary has a 3-year-old son, Michael, who suffered an early GBS sepsis shortly after birth, but survived and enjoys good health now. Mary’s current pregnancy has been uneventful until now. Which one of the
following is the most appropriate next step in management?

A. Do vaginal swab for GBS now.

B. Give antibiotics during the labor.

C. Give prophylactic antibiotics to the baby immediately after the delivery.

D. Ask her general practitioner why he has not performed the swabs.

E. Refer the matter to the Medical Board.

Incorrect. Correct answer is B


45% answered correctly

Explanation:

Correct Answer Is B

Group B Streptococcus (GBS) is found in the rectum, vagina and urinary tract of 10-30% of healthy Australian women. These women are usually asymptomatic. Giving these women prophylactic intravenous penicillin is an important measure to take
for reduction of the risk of neonatal infections with GBS and neonatal sepsis.

The following women should be treated for GBS during labour:

All women with a history of a GBS-related disease - these women should be given intrapartum antibiotics in all their subsequent pregnancies regardless of the swab culture results
All women with a GBS positive swab or urine culture result in the current pregnancy
Premature rupture of membranes for more than 18 hours, or when the time is unknown
Maternal pre- or intra-partum fever of ≥38°C
Women with unknown status of GBS colonization

Mary have two indications for intrapartum treatment with antibiotics. The first one is the history of a previous baby affected by GBS. The second one is her unknow GBS status in this pregnancy. Either of these conditions makes intrapartum
prophylaxis the best management option for her.

(Option A) With a previous history of GBS infected baby, no swabs are necessary, and prophylactic antibiotics are indicated regardless of the culture results.

(Option C) Best prophylaxis is achieved while the baby is in utero. The objective of prophylaxis is suppressing the maternal colonization before the child passes the birth canal, not afterwards.

(Option D) Asking Mary's GP why he has not perform the test does not change the management plan.

(Option E) Referring the case to the medical board is not the appropriate next step prior to taking care of Mary's GBS status.

References

• http://www.kemh.health.wa.gov.au/development/manua

• http://www.health.qld.gov.au/qcg/documents/g_gbs5-

• Australian society for infectious diseases – Management of Perinatal Infections (2014)

Last updated:
Time spent: QID:215 2023-2-12

1081 of 1943
A 24-year-old pregnant woman comes for her first antenatal visit at 10 weeks pregnancy. Which one of the following investigations will change management at this gestational age?

A. HIV screening.

B. Group B streptococcus.

C. Glucose challenge test.

D. Varicella antibodies.

E. Preeclampsia.

Correct
45% answered correctly

Explanation:

Correct Answer Is A

Among the given options, HIV is the only screening test, the result of which may alter the management of the pregnancy at this gestational age.

It is recommended that all pregnant women undergo HIV screening at the first antenatal visit. If the pregnant woman is found to be HIV positive, antiretroviral therapy should be started immediately to decrease the risk of passing the infection to the
fetus.

(Option B) Screening for group B streptococcus colonization in a pregnant woman is routinely performed between 35-37 weeks. Those with positive culture results should receive intrapartum intravenous penicillin for prevention of neonatal sepsis.

(Option C) The glucose challenge test lacks both sensitivity and specificity and is no longer part of the diagnostic algorithm. Fasting oral glucose tolerance test with 75mg glucose has substituted this test.

(Option D) Checking for varicella antibodies is recommended in the first trimester antenatal visit if there is no or uncertain history of prior varicella infection. Negativity of anti-varicella antibody does not add any further steps to the management,
unless the pregnant woman comes to contact with a case with varicella zoster or develops the clinical disease.

th
(Option E) By definition, preeclampsia occurs after 20 week of gestation.

References

• Therapeutic Guidelines – Antibiotic; available on: http://tg.org.au

• Llewellyn – Jones – Fundamentals of Obstetrics and Gynaecology – Elsevier Mosby 9th Edition

• Australian society for infectious diseases – Management of Perinatal Infections (2014)

Last updated:
Time spent: QID:216 2023-2-12

1082 of 1943
A 28-year-old pregnant woman presents to your clinic at 32 weeks gestation with vaginal discharge and itching. A low vaginal swab is taken as a part of assessment. The result is postive for group B streptococcus (GBS). No other abnormal finding is
reported. Which one of the following would be the appropriate management regarding the positive GBS culture?

A. No treatment is needed before labor.

B. Give trimethoprim for 3 days.

C. Penicillin.

D. Repeat swab in a week and treat if still positive.

E. Check full blood count.

Correct
45% answered correctly

Explanation:

Correct Answer Is A

Group B Streptococcus positive vaginal swabs are not indication for treatment at the time of diagnosis, as recolonization may occur; instead, the mother should receive intravenous penicillin during labor for prevention of neonatal sepsis. It is of great
importance to advise this woman to report signs of labor, or rupture of the membranes so that antibiotics can be commenced before delivery.

A negative culture is only current for 5 weeks.

NOTE - Positive vaginal or rectal swabs for GBS do not need treatment, but urine cultures positive for GBS should be treated with oral antibiotics. Cultures should be repeated for assessment of response to treatment.

TOPIC REVIEW

Group B streptococcus (GBS or Streptococcus agalactiae) is an encapsulated gram-positive coccus that colonizes the gastrointestinal and genital tracts of 15 to 40% of pregnant women. Although GBS colonization usually remains asymptomatic in
these women, maternal colonization is the critical determinant of infection in neonates and young infants (less than 90 days of age) in whom GBS is the most common cause of bacterial infection. Vertical (mother-to-child) transmission primarily
occurs when GBS ascends from the vagina to the amniotic fluid after onset of labor or rupture of membranes, but can occur with intact membranes as well.

To reduce the risk of mortality and morbidity from neonatal GBS infections, prophylactic antibiotics are used in certain groups.

Generally, there are two approaches for deciding as to whether intrapartum antibiotic prophylaxis should be given:

Culture-based approach

Women are screened for GBS by taking low vaginal and rectal swabs. All women, who are culture-positive should receive intrapartum prophylactic antibiotics (penicillin; cefazolin if penicillin-allergic)

Risk factor-based approach

Women with the following risk factors should receive prophylactic antibiotics:

Intrapartum fever ≥38ºC


Delivery before 37 weeks gestation (preterm labor)
Rupture of membranes ≥18 hours
Previous delivery of an infant affected by GBS disease
GBS bacteriuria (≥10 4 cfu/mL) in the current pregnancy

NOTE – according to recent studies, culture-based approach is superior to risk factor-based one, as most woman whose baby developed early complications of GBS sepsis had no risk factors. For this reason it is recommended that all
women be screened at 35-37 weeks’ pregnancy and receive prophylactic antibiotics intrapartum.

Latest recommendations suggest that the following receive intrapartum prophylaxis:

Positive screening culture for GBS from either vagina or rectum in the current pregnancy
Positive history of birth of an infant with early-onset GBS disease
GBS bacteriuria during the current pregnancy
Unknown culture status (culture not performed or result not available) and either of the following:

Intrapartum fever (≥100.4ºF, ≥38ºC)


Preterm labor (<37 weeks of gestation)
Prolonged rupture of membranes (≥18 hours)
Intrapartum NAAT (nucleic acid amplification test) e.g. PCR positive for GBS

Intrapartum antibiotic prophylaxis against GBS is NOT recommended for women with:

a positive GBS rectovaginal culture or GBS bacteriuria in a previous pregnancy and none of the indications for prophylaxis listed above

a positive GBS culture, who undergo elective cesarean delivery (at any gestational age) without labor or rupture of membranes; these patients however should undergo routine vaginal and rectal screening for GBS at 35 to 37 weeks
because onset of labor or rupture of membranes may occur before the planned cesarean delivery.

negative GBS cultures at 35 to 37 weeks of gestation, even if they have one or more of the following intrapartum risk factors: intrapartum fever ≥38ºC, preterm labor (<37 weeks of gestation), or prolonged rupture of membranes (≥18
hours). However, the use of broad-spectrum intrapartum antibiotics for the treatment (not prophylaxis) is indicated for febrile women in labor if they have clinical evidence of chorioamnionitis.

References

1083 of 1943
• QLD Health - Early onset Group B streptococcal disease

• Australian society for infectious diseases – Management of Perinatal Infections (2014)


Last updated:
Time spent: QID:217 2023-2-12

1084 of 1943
A 25-year-old woman is found to have a urine culture positive for group B streptococcus at 30 weeks gestation. She denies no urinary symptoms such as frequency, dysuria, or urgency, and is otherwise healthy. Which one of the following would be
the next best step in management?

A. Prophylactic penicillin during labour.

B. Nitrofurantoine.

C. Cephalexin.

D. Do vaginal swab in a week and then treat if positive.

E. Follow up to see if she develops fever.

Incorrect. Correct answer is C


45% answered correctly

Explanation:

Correct Answer Is C

At the first look and if the question is not read carefully, one may mistake the diagnosis and management of an asymptomatic bacteriuria in a pregnant woman with the notorious group B streptococcus (GBS) colonization.

In cases of coloniation of pregnant women with GBS, the management includes intravenous antibiotics once the labor starts, as an attempt to significantly decrease the chances of early neonatal GBS infection and sepsis with consequent
catastrophic outcomes.

In this case, however, this pregnant woman has asymptomatic bacteriuria. and should be treated with appropriate antibiotics once diagnosed. The recommedned antibiotic regimens include:

First-line: cephalexin 500mg orally, 12 hourly for 10 days (category A)


Second-line: nitrofurantoin 50mg orally, 6 hourly for 10 days (category A)
Third-line: amoxicillin + clavulanate 500+125mg orally, 12-hourly for 10 days (category B1)

There is increased resistance to amoxicillin. This drug should only be used if the susceptibility test is proven.

Women with confirmed bacteriuria should have repeat cultures sent at each antenatal visit or monthly to monitor for recurrent bacteriuria. Ongoing prophylaxis for the duration of the pregnancy should be offered to women with persistent bacteriuria.

Group B streptococcus positive vaginal swabs are not treated in the antenatal period, as group B streptococcus can recolonize at any time after treatment.

References

• Therapeutic Guidelines – Antibiotic; available on: http://tg.org.au

• http://www.kemh.health.wa.gov.au/development/manua

• http://www.health.qld.gov.au/qcg/documents/g_gbs5-

Last updated:
Time spent: QID:218 2023-2-12

1085 of 1943
A 51-year-old woman comes to your GP clinic to discuss the choice of hormone replacement therapy for 'hot flushes' and other menopausal symptoms. Which one of the following statements is correct regarding continuous combined hormone
replacement therapy?

A. It increases the risk of breast cancer.

B. It does not increase the risk of breast cancer.

C. It increases the risk of endometrial cancer.

D. It decreases the risk of endometrial cancer.

E. It provides primary prevention from coronary artery disease.

Correct
45% answered correctly

Explanation:

Correct Answer Is A

According to Australian National Health and Medical Research Council (NHMRC) guidelines, estrogen-only hormone replacement therapy does not increase the risk of breast cancer; however combined hormone replacement does. The risk is directly
related to the duration of HRT.

The following table (from NHMRC) shows the effect of combined HRT and oestrogen-only HRT on incidence of different conditions:

Assessed system or condition Effects of HRT


Cancers
Combined HRT increases the risk of cancer.
Estrogen-only HRT shows little or no increased incidence.
Breast cancer

Combined HRT shows little or no increased incidence.


Estrogen-only HRT increases the risk of cancer.
Endometrial cancer

For combined HRT, there is not enough evidence.


Estrogen-only HRT slightly increases the incidence.
Ovarian cancer

Combined HRT reduces the incidence.


No change with estrogen-only HRT.
Colon cancer

Cardiovascular
Combined HRT increases the incidence.
For estorgen-only HRT, there is not enough evidence.
DVT and PE

CAD HRT does not seem to protect post-menopausal women against coronary artery disease as it was previously thought.
Stroke HRT increases the incidence.
Bones
Osteoporosis and fractures HRT decreases the incidence of osteoporosis and fractures.
Other conditions
Gallbladder diseases HRT increases the incidence very slightly if at all.
There is not enough evidence whether HRT helps to preserves the cognitive function and prevents dementia.

Cognition and dementia

1086 of 1943
A 5-year-old boy is brought to the emergency department by ambulance after he had a generalized tonic-clonic seizure at home 20 minutes ago. Upon arrival at the hospital, he is assessed immediately. He is not having a seizure now but is lethargic
and confused. His blood pressure is 105/65 mmHg, pulse rate 76 bpm, respiratory rate 16 breaths/min, and temperature 37.℃. His mucous membranes are not dry, skin turgor is normal, and capillary refill time is 2 seconds. The rest of the
examination, including cardiovascular, respiratory, and neurological, is completely normal. Pathology results are as follows:

FBE: Normal
Random blood sugar: 8.3 mmol/L (4 - 11.1 mmol/L)
Sodium: 120 mmol/L (135-145 mmol/L)
Potassium: 4.1 mmol/L (3.5 – 5.5 mmol/L)
Bicarobonate: 24 mmol/L (22-32 mmol/L)
Creatinine: 80 µmol/L (60-110 µmol/L)
Urea: 4.5 mmol/L (2.5-7.1 mmol/L)
Calcium: 2.5 mmol/L (2.2-2.7 mmol/L)

Which of the following could be the most likely cause of this presentation?

A. Acute renal failure.

B. Addison's disease.

C. Congestive heart failure.

D. Hyponatremia due to dehydration.

E. SIADH.

Incorrect. Correct answer is E


45% answered correctly

Explanation:

Correct Answer Is E

Afebrile seizures in children can have a variety of etiologies including space-occupying intracranial lesions, CNS infections (e.g., meningitis, encephalitis), metabolic derangements (e.g., hypoglycemia, hyponatremia, and hypernatremia,
hypercalcemia, uremia, epileptic disorders, medications, head trauma, etc.

The abnormal finding in the lab values of this child is serum sodium of 120 mmol/L indicating hyponatremia as the most likely cause of the seizure episode in this child. Both hyponatremia and hypernatremia can present with lethargy, headache,
altered mental status, seizure coma, and even death.

Hyponatremia can be caused by a variety of conditions including:

Medications, especially diuretics, ACH inhibitors, Angiotensin receptor blockers (ARBs), etc.
Dehydration
Syndrome of inappropriate ADH (SIADH)
Congestive heart failure
Renal failure
Liver failure
Hormonal diseases e.g. Addison’s disease, hypothyroidism, etc.

Hyponatremia, depending on the volume state, is classified as hypovolemic hyponatremia, hypervolemic hyponatremia, and euvolemic hyponatremia.

In hypovolemic hyponatremia, volume depletion of any cause such as dehydration results in a surge in the secretion of antidiuretic hormone (ADH). ADH, by retaining water and its dilutional effect results in hyponatremia. Since this child has no
clinical findings suggestive of dehydration, hyponatremia due to dehydration (option D) is not an unlikely diagnosis.

Hypervolemic hyponatremia is when the presence of excess total body water dilutes sodium. This kind of hyponatremia can be seen in congestive heart failure (CHF), renal failure, and chronic liver disease.

In renal failure, sodium loss and water retention caused by faulty kidneys result in hyponatremia. This child has normal creatinine and urea, and acute renal failure (option A) is far less likely to be the underlying etiology for hyponatremia and seizure
in this child.

CHF, another cause of hypervolemic hyponatremia, is associated with low cardiac output and blood pressure. This activates several neurohormonal pathways to preserve arterial blood volume and pressure. The primary mechanism for such
compensation is the increased activity of arginine vasopressin (AVP). AVP increases free-water reabsorption in the renal collecting ducts; therefore, the blood volume is increased and plasma sodium is diluted. In this child, however, the absence of
clinical manifestations of CHF in the history and physical examinations makes CHF (option C) to be an unlikely underlying cause of hyponatremia.

In euvolemic hyponatremia, the total amount of water in the body is normal. The most common cause of euvolemic hyponatremia is the syndrome of inappropriate secretion of antidiuretic hormone (SIADH).

Addison's disease can be another cause of hyponatremia. Addison's disease presents with an adrenal crisis if it occurs acutely. Chronic symptoms result from cortisol deficiency, aldosterone deficiency, and excess ACTH. Cortisol deficiency
manifests with weakness and fatigue, weight loss, anorexia, nausea and vomiting, diarrhea or constipation, flank or abdominal pain, hypoglycemia, and hyperthermia.

Clinical manifestations of aldosterone deficiency include hyponatremia, hyperkalemia, acidosis, tachycardia, and hypotension. Suggestive symptoms are postural hypotension and salt cravings.

With low sodium but normal potassium in this child and the absence of other clinical and laboratory manifestations, Addison disease (option B) is unlikely to cause hyponatremia and consequent seizure.

Given the fact that there is no abnormal laboratory value except the hyponatremia, and by the exclusion of other options as above, SIADH is the most likely diagnosis in this child. Excess ADH in SIADH results in increased retained water in the body.
With excess water, sodium is diluted and hyponatremia occurs. Hyponatremia is the most common clinical presentation in patients with SIADH.

Any of the following can cause SIADH:

CNS pathologies - Guillain-Barre syndrome, subarachnoid hemorrhage, head trauma, meningitis, encephalitis
Lung pathologies - TB, sarcoidosis, pneumonia, pneumothorax, atelectasis, asthma
Malignancies - Small cell lung cancer, nasopharyngeal cancers, mesotheliomas, gastrointestinal cancers, pancreatic cancer, lymphoma, sarcoma
Medications - Antidepressants, antipsychotics, anticonvulsants, anti-inflammatory drugs, ecstasy, cyclophosphamide
Other - prolactinoma, Waldenstrom macroglobulinemia

1087 of 1943
The mainstay of treatment in SIADH is the treatment of symptomatic patients with water restriction with or without sodium replacement, depending on the severity of symptoms, while a meticulous investigation is carried out for an underlying
etiology. Although this child does not have overt symptoms or clinical findings indicating the presence of any of the above conditions, for now, there might be an occult pathology such as an infection or malignancy.

References

• RACGP – AFP – The suspect – SIADH

• RACGP – AFP – Addison disease

• RCH – Hyponatraemia

• Medscape - Hyponatremia in Emergency Medicine

Time spent: QID:1600 Last updated:


2023-2-12

1088 of 1943
Which one of the following risk factors is not associated with increased risk of group-B streptococcus (GBS) neonatal sepsis?

A. Preterm labor.

B. GBS bacteriuria in current pregnancy.

C. Previous baby with GBS-related early onset sepsis.

D. Rupture of membranes of 10 hours duration.

E. Maternal temperature greater than or equal to 38°C within 24 hours of giving birth.

Incorrect. Correct answer is D


45% answered correctly

Explanation:

Correct Answer Is D

Risk factors for early neonatal GBS sepsis include:

Maternal fever more than or equal to 38°C either intrapartum or within 24 hours of giving birth
Group-B streptococcus bacteriuria and colonization in current pregnancy
Rupture of membranes prior to birth for more than 18 hours
Preterm labor (less than 37 weeks)
Previous GBS related early onset sepsis

Rupture of membranes of 10 hours duration is not associated with increased risk of group B streptococcus neonatal sepsis.

References

• http://www.kemh.health.wa.gov.au/development/manua

• http://www.health.qld.gov.au/qcg/documents/g_gbs5-

• Australian society for infectious diseases – Management of Perinatal Infections (2014)

Last updated:
Time spent: QID:219 2023-2-12

1089 of 1943
A newly married24-year-old woman is found to have multiple fibroids, palpable up to the umbilicus. She has been trying to get pregnant, but every attempt has failed so far. Laboratory investigations show hemoglobin of 82 g/L. Which one of the
following would be the most appropriate management?

A. Open myomectomy after correcting her anemia with blood transfusion.

B. GnRH analogues for 3 months followed by myomectomy.

C. OCP for 3 months followed by myomectomy.

D. Immediate myomectomy.

E. Hysterectomy.

Incorrect. Correct answer is B


45% answered correctly

Explanation:

Correct Answer Is B

Gonadotropin-releasing hormone (GnRH) agonists are the most effective medical therapy for uterine myomas. These drugs work by increasing the release of gonadotropins, followed by desensitization and down-regulation to a hypogonadotropic
hypogonadal state that clinically resembles menopause.

Most women will develop amenorrhea, improvement in anemia, and a significant reduction (35 to 60%) in uterine size within 3 months of initiating the therapy (most of size reduction occurs in the first month of use).

Surgical removal of myomas then follows. The only surgical procedure that preserves fertility and effectively ameliorates fibroid-related symptoms is myomectomy.

For women with poor prior infertility, in whom the only finding is a distorted uterine cavity from the presence of one or more uterine fibroids (most commonly submucous fibroids), myomectomy can promote fertility and successful pregnancy
outcome.

Combined hormonal contraceptives and progesterone are commonly prescribed to regulate abnormal uterine bleeding, but appear to have limited efficacy in the treatment of uterine leiomyomas.

Hysterectomy would be a wrong option for this woman and almost every woman of reproductive age (unless they are not willing to have more children).

References

• AAFP - Uterine Fibroid Tumors: Diagnosis and Treatment

• An Evidence-based Guideline for the Management of Uterine Fibroids

Last updated:
Time spent: QID:220 2023-2-12

1090 of 1943
A 27-year-old pregnant lady at 13 weeks’ gestation presents to your clinic after she came to contact with a child with parvovirus infection four days ago. She is concerned that her baby may become affected. You order serum IgM and IgG antibody
levels for parvovirus, both of which are negative. Which one of the following would be the most appropriate next step in management?

A. Repeat serologic tests in two weeks.

B. Ultrasonography.

C. Reassure her as the antibody titres are negative.

D. Arrange for intrauterine blood transfusion.

E. Arrange for fetal umbilical cord blood sampling.

Correct
45% answered correctly

Explanation:

Correct Answer Is A

Parvovirus B19 is a single-stranded DNA virus. It is the causative virus for erythema infectiosum, also known as fifth disease or slapped cheek syndrome.

Maternal infection with parvovirus B19 is associated with the risk of transplacental fetal infection throughout the pregnancy. Fetal infection may result in fetal parvovirus syndrome. The syndrome is characterized by anemia-hydrops with cardiac
failure and possibly death.

The earlier the exposure occurs, the more likely the development of fetal parvovirus syndrome would be. With third trimester infection still births are common.

Women who have been exposed to parvovirus in early pregnancy should be informed of the possible risks to the fetus and offered serology for parvovirus B19 specific IgG:

If parvovirus specific IgG is positive reassure that pregnancy is not at risk


If parvovirus specific IgG is negative, serology for IgM should be performed.

After infection with parvovirus, IgM is expected to become positive in 1 to 3 weeks and remain high until 8-12 weeks. IgG levels start to rise. IgG levels start to rise 2 to 4 weeks after the infection.

This woman has a negative IgG titer indicating that she is not immune to the infection. Although her IgM titer is negative, this does not exclude infection because it takes approximately 1 to 3 weeks for IgM to become positive after infection. IgM will
remain high for 8 to 12 weeks. In such cases, it is recommended that serologic tests be repeated in 2 weeks when the IgM may become positive while IgG starts to rise.

Positive IgM titers confirm maternal parvovirus infection. If that is the case, the next step would be fetal monitoring with ultrasound for development of hydrops at 1-2 weeks intervals for the next 6-12 weeks (needs referral). Once the fetus is found to
have hydrops, fetal umbilical cord sampling and intrauterine blood transfusion are considered.

Positive IgG and negative IgM indicates maternal immunity to parvovirus.

The following table summarizes interpretation of serologic tests results and recommended further actions:

IgM IgG Interpretation Further action


Negative Negative Mother is not immune, infection is possible Repeat serology tests in 2 weeks
Positive Negative Infection is established Fetal monitoring with ultrasound at 1- to 2-week intervals for the next 6-12 weeks
Positive Positive Infection is established Fetal monitoring with ultrasound at 1- to 2-week intervals for the next 6-12 weeks
Negative Positive Mother is immune Reassure

References

• http://www.sahealth.sa.gov.au/wps/wcm/connect/04ab

Last updated:
Time spent: QID:325 2023-2-12

1091 of 1943
Which one of the following is the most common symptom of endometriosis?

A. Dysmenorrhea.

B. Dysuria.

C. Infertility.

D. Deep dyspareunia.

E. Painful abdominal bloating.

Correct
45% answered correctly

Explanation:

Correct Answer Is A

Endometriosis can presents with the following features:

Dysmenorrhea (moderate to severe) – 60 to 80% of cases


Chronic pelvic pain – 40-50% of cases
Dysuria and hematuria – 5% of cases
Infertility – 30 to 50% of cases
Deep dyspareunia – 40 to 50% of cases
Dyschezia, tenesmus, and painful abdominal bloating - 10 to 40% of cases
Premenstrual spotting lasting 1-2 days
Heavy menstrual bleeding – 10-20%

Dysmenorrhea has been by far the most common symptoms in patients with endometriosis.

References

• Australian Prescriber - Medical management of endometriosis

Last updated:
Time spent: QID:330 2023-2-12

1092 of 1943
Which one of the following investigations can help in diagnosing endometrioses earliest?

A. Transvaginal ultrasound.

B. Pelvic examination.

C. Laparoscopy.

D. Hysteroscopy.

E. Colposcopy.

Incorrect. Correct answer is C


45% answered correctly

Explanation:

Correct Answer Is C

The definite diagnosis of endometriosis can be made by laparoscopy. This method can detect endometriosis at the earliest stages.

(Option A) Ultrasonography can only visualize endometriomas (tumors of endometrial tissue), not endometriosis.

(Option B) Pelvic examination does not often help with the diagnosis.

(Options D and E) Hysteroscopy and colposcopy are not helpful in diagnosing endometriosis, as the former can just visualize inside the uterine cavity and the latter only the vagina and the cervix.

References

• Medscape - Endomteriosis

Last updated:
Time spent: QID:331 2023-2-12

1093 of 1943
Which one of the following statements is incorrect regarding endometriosis?

A. Laparoscopy is the best diagnostic test.

B. It is the commonest cause of chronic pelvic pain in women in most developed countries.

C. The usual delay in diagnosis is 8 to 10 months with onset in adolescence.

D. Medical treatment for endometriosis usually does not improve fertility.

E. Excisional surgery is the treatment of choice for infertility caused by endometriosis.

Incorrect. Correct answer is C


45% answered correctly

Explanation:

Correct Answer Is C

Option A is correct - Endometriosis is a highly variable condition and diagnosis can be difficult. Confirmation of diagnosis requires laparoscopy in most situations.

Option B is correct - Endometriosis is the most common cause of chronic pelvic pain in most developed countries.

Option C is incorrect - usual delay in diagnosis is about 8 to 10 years and not 8 to 10 months.

Option D is correct - There is strong evidence that medical treatment only relieves symptoms and does not improve fecundity.

Option E is correct - Excisional surgery for endometriosis is usually the treatment of choice as it improves the symptoms, confirms the diagnosis and improves the quality of life in 60-80% of patients.

References

• Medscape - Endomteriosis

Last updated:
Time spent: QID:332 2023-2-12

1094 of 1943
A 24-year-old Aboriginal woman presents to the antenatal clinic at 19 weeks gestation. This is the best time for which one of the following?

A. Ultrasonography.

B. Maternal serum screening for Down syndrome.

C. Amniocentesis.

D. Chorionic villous sampling.

E. Rubella screening.

Correct
45% answered correctly

Explanation:

Correct Answer Is A

Ultrasound for identification of physical abnormalities, including neural tube defects is best performed between 18- 20 weeks of gestation, and should be considered in this visit.

(Option B) Maternal serum screening for Down syndrome is best performed between 15-18 weeks of gestation. The screening tests include alpha fetoprotein, oestriol, and free beta hCG in the maternal blood (triple screening test). Accurate date of
gestation is needed for reliable interpretation of the results. This patient is young and the risk of having a baby with Down sydrome is not significant. Even otehrwise, sonography can be used for assessment of nuchal translucency as well as other
fetal parameters.

(Option C) Amniocentesis is best performed between 15-18 weeks of gestation and carries a risk of miscarriage of 1 in 200. It is a very accurate measure for diagnosing chromosomal anomalies including Down syndrome.

(Option D) Chorionic villous sampling is best performed between 10-12 weeks of gestation. The procedure carries a miscarriage risk of 1 in 100. It is also accurate for diagnosis of chromosomal anomalies.

(Option E) Screening for rubella is best performed before conception and not during the pregnancy.

References

• http://www.racgp.org.au/your-practice/guidelines/n

• http://www.babycenter.com.au/a557390/18---20-week-

Last updated:
Time spent: QID:346 2023-2-12

1095 of 1943
A 31-year-old woman G2P1 presents at 10 weeks’ gestation for antenatal visit. She is found to have a twin pregnancy. Her previous pregnancy was complicated with placental abruption at 34 weeks. Which one of the following is the next best step in
management in addition to routine antenatal care?

A. Vitamin supplementation.

B. Cervical suture.

C. Increased iron and folic acid supplementation.

D. Hospital admission after 34 weeks.

E. Serial CTGs after 34 weeks.

Incorrect. Correct answer is C


45% answered correctly

Explanation:

Correct Answer Is C

This woman has a high-risk pregnancy due to a multiple pregnancy and the history of placental abruption in the previous pregnancy, each of which alone is significant enough to warrant extra care of the patient. Risks and care associated with and
needed for easc condition is as follows:

Placental abruption: A pregnancy following a placental abruption (PA) must be considered a high risk pregnancy, not only in terms of increased risk of recurrence, but also due to excess risk of small for gestational age, preterm labor and birth and
pregnancy-induced hypertension (PIH), irrespective of recurrence of placental abruption. Threrefore, all pregnancies following a pregnancy complicated by placental abruption should be offered close antenatal surveillance. A significant risk factor for
PA is abruption in a previous pregnancy. It is estimated that a woman with a previous PA has a 5-10% chance of recurrent PA in subsequent pregnancies. With two or more PAs, the risk increases to 19-25%.

It is recommended that once a woman had a PA, close surveillance be performed in the subsequent pregnancies starting from 2-3 weeks (up to 3 months) before the gestational age the PA occurred. There are limited measures to prevent
subsequent PAs to develop except intervention in modifiable risk factors (e.g. hypertension, smoking, amphetamine and cocaine).

Multiple pregnancies: multiple pregnancy is associated with several potential maternal and fetal complications. The summary of recommendations for multiple pregnancies is reviewed in the following table:

PROCEUDRE ADDITIONAL IFORMATION

Routine antenatal investigations

Management as for a singleton pregnancy

Frequency of antenatal clinic visits:

Women without complications are seen:

4-weekly until 28 weeks gestation


2-weekly until 34 weeks gestation
weekly from 34 weeks gestation

Women with high-risk multiple pregnancy:

Frequency of care is individualized. The Consultant must be involved in decision making.

Genetic Screening

Offer Nuchal translucency in the first trimester. Serum screening tests are not as sensitive in multiple gestations

Refer to Genetic Services if required

Ultrasound The ultrasound at this time identifies whether the twins are dichorionic or monochorionic.

Perform an ultrasound at 12-13 weeks gestation. Congenital malformations are more common in multiple pregnancies.

Perform an anatomy scan at 19 (18-20) weeks gestation.


The incidence of malformations are higher in monozygotic twins than dizygotic twins than
singleton pregnancies.

Frequency of antenatal scans performed are: Serial ultrasounds provides the best method for monitoring discordant growth, with evaluation
of fetal well-being by biophysical profiles, and Doppler studies.

Monochorionic twins – 12,16,19,22,26,30,33 and 36 weeks


Dichorionic twins –12,20,28,32 and 36 weeks

1096 of 1943
Cardiotocograph Monitoring (CTG)

CTG monitoring should be done for:

Discordant growth – weekly after 34 weeks


Multiple pregnancy increases calorie, protein, mineral, and vitamin requirements.
Diet and Nutrition

The risk for anemia increases in multiple pregnancy.


Offer referral to Dietician Services

Iron deficiency anemia is associated with pre-term delivery and low ferritin levels are linked to
Recommend twice daily iron and folic acid supplementation.
prematurity.

Recommend multivitamin supplementation for woman with poor nutritional status.

For this woman several measures should be contemplated, starting with increasing the dose of folate and iron supplementation as the best initial step at this time. Multifetal pregnancies are associated with increased risk of iron and folate
deficiencies. Iron deficiency anemia is associated with increased risk of preterm delivery and low ferritin level with prematurity. Folate deficiency may result in neural tube defect and anaemia (megaloblastic).

Ultrasonographic assessment is recommended to start at 12-13 weeks gestation where it can demonstrate whether the twins are monochorionic or dichorionic. If initial ultrasonography reveals a dichorionic twin pregnancy serial sonographies
should be carried out at 20, 28, 32 and 36 weeks. For monochorionic twin pregnancies the time of further subsequent sonographies will be at 16,19,22,26,30,33 and 36 weeks.

CTG should be performed after 34 weeks on a weekly basis. CTG and serial sonography, as mentioned above, provide the best method for monitoring discordant growth, with evaluation of fetal well-being by use of CTG monitoring, biophysical
profiles, and Doppler studies.

Previously it was suggested that patients such as this woman be admitted around the time the previous PA occurred, but currently there is not enough evidence to support a policy of routine hospitalization for bed rest in multiple pregnancy. No
reduction in the risk of preterm birth or perinatal death is evident, although there is a suggestion that fetal growth is improved. In women with just multiple pregnancy and no other complications, this measure is not only useful, but it may result in
increased risk of very preterm birth. Until furthe evidence is available to the contrary, the policy cannot be recommended for routine clinical practice.

References

• http://www.ncbi.nlm.nih.gov/pubmed/11279677

• http://apps.who.int/rhl/pregnancy_childbirth/anten

• https://www.rcog.org.uk/en/guidelines-research-ser

• https://www.rcog.org.uk/globalassets/documents/gui

• http://www.kemh.health.wa.gov.au/development/manua

• http://www.ncbi.nlm.nih.gov/pubmed/11279677

• http://apps.who.int/rhl/pregnancy_childbirth/anten

• http://3centres.com.au/guidelines/complications-in

Last updated:
Time spent: QID:347 2023-2-12

1097 of 1943
A 29-year-old woman is planning to become pregnant. She is extremely concerned about rubella infection during pregnancy and its impacts on her baby because 2 years ago her elder sister had to undrgo an abortion after she was diagnosed with
rubella infection. She remembers that she was vaccinated for measles, mumps and rubella when she was a child. Which one of the following statements is correct regarding rubella infection?

A. A history of immunization for rubella is enough and no further rubella testing is needed.

B. Rubella vaccine can be given safely during pregnancy.

C. In Australia, 25% of women aged 15-40 years are immune to rubella.

D. Serology testing of rubella antibodies titer should be ordered now.

E. Still birth following infection of the mother in the second trimester is common.

Incorrect. Correct answer is D


45% answered correctly

Explanation:

Correct Answer Is D

Every woman who is planning for pregnancy should be tested for rubella serology (IgG only), regardless of her previous immunization status. The serology results should be documented.

(Option B) Rubella vaccine is a live attenuated vaccine and is contraindicated during pregnancy, as there is theoretical risk of harm to the developing fetus; however, rubella vaccine has not been proven to cause problems for the fetus so far.

(Option C) In Australia, 95% of women aged 15-40 years of age are rubella IgG seropositive and immune to rubella.

(Option E) If the fetus is infected during the second trimester, still birth will be an extremely rare event.

TOPIC REVIEW

Rubella, also called German measles, is usually a mild infectious disease in children and adults. It is clinically difficult to diagnose due to transient clinical features that are also common to some other viral infections.

Rubella has an incubation period of 14-23 days. The infectivity period starts from one week before until 4 days after onset of rash.

Rubella is asymptomatic in 25 to 50 % of cases. In some cases prodromal symptoms may be evident, such as:

Low grade fever


Transient erythematous rash
Lymphadenopathy involving post-auricular and sub-occipital nodes
Occasionally arthritis and arthralgia (commonly observed in women of child-bearing age)
Rarely neurological disorders and thrombocytopenia
Rash - the rash characteristically begins on the face and spreads to the trunk and extremities. It will usually resolve within three days in the same order in which it appeared (face first and then body)

During pregnancy, maternal viremia may occur 5 to 7 days after exposure with spread of the virus throughout the body as well as transplacental infection of the fetus.

Vertical transmission (transplacental) from mother to fetus can only occur during viremia of a primary infection. The effect of maternal infection on the fetus depends on the gestational age:

<8 weeks: up to 85% of fetuses will be infected and all will have clinical manifestations of the congenital rubella syndrome.
8≤ but <12 weeks: 50-80% of fetuses will be infected and of those 65-85% will be clinically affected.
13-16 weeks: 30% of the fetuses will be infected and of those, 1/3 will have sensorineural deafness.
16-19 weeks: 10% of the fetuses will be infected. Clinical features are rare, yet deafness can be a possibility.
>19 weeks: there is no apparent risk.

NOTE – reinfection is associated with far less risk of fetal infection of approximately 5%.

Abnormalities associated with congenital rubella syndrome include:

Central nervous system dysfunction (10-25%, intellectual impairment, developmental delay, microcephaly)
Eye abnormalities (10-25%, cataracts, retinopathy, glaucoma, strabismus, micropthalmos)
Sensorineural deafness (60-75%)
Cardiac abnormalities (10-20%, PDA, PA stenosis)
Intrauterine growth restriction, short stature
Inflammatory lesions of the brain, liver, lungs and bone marrow

All pregnant women, who have contact with rubella or clinical features consistent with rubella – like illness should be screened for the presence of rising antibody titre (IgG) and rubella specific IgM, regardless of previous vaccination history or
infection. Interpretation of the results and corresponding action is as follows:

IgM IgG Interpretation Recommendation


Serology tests should be repeated. If positive IgM is confirmed or there is a rise in IgG, the
+ + Possible recent infection of reinfection, depending on history
infection is confirmed.
If the test has been performed < 3 weeks since contact or < 7days since onset of illness, it
- - The patient is susceptible
should be repeated. Seroconversion or positive IgM indicates infection
Repeating the test for confirmation must be performed. If positive IgM is confirmed or
+ - Recent infection is possible
seroconversion occurs, the infection is confirmed.
- + Immunity to rubella or past infection No further action is needed

If maternal infection is confirmed, antenatal testing is recommended at least 6 weeks after known maternal infection and is best performed after the 20th week of gestation. Rubella PCR, rubella culture and fetal IgM can be performed following
chorionic villus sampling (CVS) / amniocentesis or cordocentesis; however, due to the very high risk of fetal infection and consequent fetal anomalies, if maternal infection occurs in the first trimester, termination of pregnancy should be
recommended.

1098 of 1943
NOTE - In all pregnant women, serologic testing for rubella (IgG only) should be offered and performed in the first antenatal visit. With titers ≥10 IU/ml, risk of reinfection is minimal. However, if the titers are ≤15, vaccination after delivery should be
performed.

References

• https://www.sahealth.sa.gov.au/wps/wcm/connect/d81

• https://www.asid.net.au/documents/item/368

Time spent: QID:350 Last updated:


2023-2-12

1099 of 1943
A 20-year-old pregnant woman presents to your practice at 10 weeks pregnancy. She is worried because 3 days ago she came to contact with one of her friends from work, who had rubella. She has never been vaccinated against rubella, nor
contracted the disease. On examination, she is asymptomatic with no signs or symptoms. Which one of the following is the next best step in management?

A. Check serum IgM and IgG titres for rubella.

B. Give intravenous immunoglobulin.

C. Give MMR Vaccine.

D. Give only rubella vaccine.

E. Terminate the pregnancy.

Correct
45% answered correctly

Explanation:

Correct Answer Is A

All pregnant women with suspected rubella or exposure to rubella should be serologically tested (IgM/IgG), irrespective of a history of previous vaccination, clinical rubella or a previous positive rubella antibody result.

In Australia, over 90% of women are immune against rubella. This can be due to either previous vaccination or infection. In some cases, infection occurs without clinical significance; this means that even though this woman denies previous rubella
infection, she might have been infected without even knowing, therefore immune to rubella. This would be evident by an IgG titer of equal to or greater than 10 IU/ml. On the other hand, a positive IgM titer maskes a recent acute infection likely. A
positive IgM demands the test to be repeated.

After exposure of a pregnant woman to a case with rubella infection, the next best step is always rubella serology testing regardless of previous history of immunization, infection or previous IgG titres. One exception is an IgG level of equal to or
greater than 10 IU/ml documented in current pregnancy. In such case, serologic studies can be skipped and the patient be reassured.

Rubella infection during pregnancy is potentially teratogenic with the risk of fetal infection being greatest if maternal infection occurs during the first trimester. Fetal damage occurs in up to 85% cases of maternal rubella infection in the first 8 weeks
and 50-80% if infection occurs within the third month of pregnancy.

Risk of fetal damage declines to about 10 to 20% by 16 weeks and would be rather rare beyond this time.

Vaccination against rubella (options C and D) is contraindicated during pregnancy and there is no single rubella vaccine availabel. The value of immunoglobulin (option B) is controversial. Therapeutic abortion (option E) is generally recommended
after proven infection during the first trimester. The doctor must counsel the patient regarding the risks associated with congenital rubella syndrome, but if the patient, by taking the risks, prefers to continue the pregnancy, her wish should be
followed.

TOPIC REVIEW

Rubella, also called German measles, is usually a mild infectious disease in children and adults. It is clinically difficult to diagnose due to transient clinical features that are also common to some other viral infections.

Rubella has an incubation period of 14-23 days. The infectivity period starts from one week before until 4 days after onset of rash.

Rubella is asymptomatic in 25 to 50 % of cases. In some cases prodromal symptoms may be evident, such as:

Low grade fever


Transient erythematous rash
Lymphadenopathy involving post-auricular and sub-occipital nodes
Occasionally arthritis and arthralgia (commonly observed in women of child-bearing age)
Rarely neurological disorders and thrombocytopenia
Rash - the rash characteristically begins on the face and spreads to the trunk and extremities. It will usually resolve within three days in the same order in which it appeared (face first and then body)

During pregnancy, maternal viremia may occur 5 to 7 days after exposure with spread of the virus throughout the body as well as transplacental infection of the fetus.

Vertical transmission (transplacental) from mother to fetus can only occur during viremia of a primary infection. The effect of maternal infection on the fetus depends on the gestational age:

<8 weeks: up to 85% of fetuses will be infected and all will have clinical manifestations of the congenital rubella syndrome.
8≤ but <12 weeks: 50-80% of fetuses will be infected and of those 65-85% will be clinically affected.
13-16 weeks: 30% of the fetuses will be infected and of those, 1/3 will have sensorineural deafness.
16-19 weeks: 10% of the fetuses will be infected. Clinical features are rare, yet deafness can be a possibility.
>19 weeks: there is no apparent risk.

NOTE – reinfection is associated with far less risk of fetal infection of approximately 5%.

Abnormalities associated with congenital rubella syndrome include:

Central nervous system dysfunction (10-25%, intellectual impairment, developmental delay, microcephaly)
Eye abnormalities (10-25%, cataracts, retinopathy, glaucoma, strabismus, micropthalmos)
Sensorineural deafness (60-75%)
Cardiac abnormalities (10-20%, PDA, PA stenosis)
Intrauterine growth restriction, short stature
Inflammatory lesions of the brain, liver, lungs and bone marrow

All pregnant women, who have contact with rubella or clinical features consistent with rubella – like illness should be screened for the presence of rising antibody titre (IgG) and rubella specific IgM, regardless of previous vaccination history or
infection. Interpretation of the results and corresponding action is as follows:

IgM IgG Interpretation Recommendation


Serology tests should be repeated. If positive IgM is confirmed or there is a rise in IgG, the
+ + Possible recent infection of reinfection, depending on history
infection is confirmed.
If the test has been performed < 3 weeks since contact or < 7days since onset of illness, it
- - The patient is susceptible
should be repeated. Seroconversion or positive IgM indicates infection
Repeating the test for confirmation must be performed. If positive IgM is confirmed or
+ - Recent infection is possible
seroconversion occurs, the infection is confirmed.

1100 of 1943
- + Immunity to rubella or past infection No further action is needed

If maternal infection is confirmed, antenatal testing is recommended at least 6 weeks after known maternal infection and is best performed after the 20th week of gestation. Rubella PCR, rubella culture and fetal IgM can be performed following
chorionic villus sampling (CVS) / amniocentesis or cordocentesis; however, due to the very high risk of fetal infection and consequent fetal anomalies, if maternal infection occurs in the first trimester, termination of pregnancy should be
recommended.

NOTE - In all pregnant women, serologic testing for rubella (IgG only) should be offered and performed in the first antenatal visit. With titers ≥10 IU/ml, risk of reinfection is minimal. However, if the titers are ≤15, vaccination after delivery should be
performed.

References

• https://www.sahealth.sa.gov.au/wps/wcm/connect/d81

• https://www.asid.net.au/documents/item/368

Time spent: QID:351 Last updated:


2023-2-12

1101 of 1943
Which one of the following statements is incorrect regarding the use of combined hormone replacemet therapy (HRT) with estrogen and progesterone in a 55-year-old postmenopausal woman?

A. If used longer than 5 years, it significantly increases the risk of breast cancer.

B. It is not recommended for treating postmenopausal women presenting with osteoporosis, who are older than 60 to 65 years and where the management of osteoporosis is the only concern.

C. It should be avoided in patient with history of deep vein thrombosis.

D. Combination with alendronate is usually discouraged.

E. In women who are more than three years post menopausal, raloxifene is effective in reducing vertebral fracture risk.

Correct
45% answered correctly

Explanation:

Correct Answer Is A

Studies suggest that combined HRT is associated with a slight increase in the incidence of breast cancer if used more than five years. However, the risk is not significant unless there is personal history of breast cancer. The benefits of HRT definitely
outweigh the slightly increased risk of breast cancer.

(Option B) Vasomotor symptoms, such as hot flushes are the only indications for commencement of hormone replacement therapy (HRT). Although HRT decreases mineral bone loss, osteoporosis is not an indication to start HRT. HRT prevents
osteoporosis, but do not treat it.

(Option C) Estrogen should be avoided in women at risk of venous thromboembolism.

(Option D) Combination therapy with estrogen and alendronate should generally be avoided unless there is specialist guidance.

(Option E) Raloxifene is an estrogen receptor modulator. It has an agonist effect on bone, but not the endometrium. Raloxifene is effective in reducing vertebral fracture risk but not the non-vertebral fractures. Raloxifene is not the first-line treatment
option for osteoporosis even in the presence of personal history of breast cancer.

References

• http://www.ncbi.nlm.nih.gov/pmc/articles/PMC254436

• https://www.nhmrc.gov.au/_files_nhmrc/publications

Last updated:
Time spent: QID:86 2023-2-12

1102 of 1943
Zara is a recent 28-year-old immigrant from Sudan, who is being assessed for several medical issues including fatigue, insomnia, forgetfulness, and irritability. Her older sister accompanies her and says they have recently lost two family members
due to the conditions in their county and believes Zahra could be depressed. She is vegetarian and back at home her diet mostly consisted of corn and wheat. On examination, she has desquamated plaques on an erythematous skin base, especially
on the back of her hands, around her eyes, and on her neck. Which one of the following could be the most likely diagnosis?

A. Vitamin B12 deficiency.

B. Niacin deficiency.

C. Vitamin D deficiency.

D. Zinc deficiency.

E. Vitamin E deficiency.

Incorrect. Correct answer is B


45% answered correctly

Explanation:

Correct Answer Is B

Zara’s background (an immigrant from Sudan), her diet, and the constellation of symptoms are highly suggestive of pellagra. Pellagra, a systemic disease, results from niacin (nicotinic acid or vitamin B3) deficiency.

Primary pellagra due to a deficient diet is common, particularly in developing countries where corn is a major food source or following prolonged disasters including famine or war. Although corn is rich in niacin, it is in a bound form unable to be
accessed unless treated with alkaline solutions, and the proteins in corn are low in tryptophan. Pellagra remains endemic in India, China, and some African countries. In developed countries, pellagra is rare because many foods, such as bread and
cereal, are fortified with niacin. However, it may still result from chronic alcohol abuse, anorexia nervosa, or self-imposed diets. Alcoholism is the most common cause of pellagra in developed countries.

Secondary pellagra develops despite an adequate diet due to:

Malabsorption — inflammatory bowel disease, chronic diarrhea, Whipple disease, and gastrointestinal surgery including gastric bypass surgery
Drug-induced e.g., azathioprine, chloramphenicol, 5-fluorouracil (5FU), isoniazid, ethionamide, carbamazepine, leucine.
Metabolic steal — carcinoid syndrome steals tryptophan from the niacin pathway to make serotonin.

A form of pellagra is seen in Hartnup disease (MIM 234500), an autosomal recessive inherited disorder of the SLC6A19 gene resulting in impaired absorption of tryptophan from the bowel and excessive excretion by the kidneys.

Pellagra presents with the mnemonic 4D’s (1) photosensitive dermatitis, (2) diarrhea, (3) dementia, and (4) death. This is the only photosensitivity syndrome where death is included as a cardinal clinical feature. Of note, not every patient, especially
infants and children, with pellagra has the whole tetrad.

Early symptoms of pellagra include lassitude, weakness, loss of appetite, mild digestive disturbances, and psychiatric or emotional distress (e.g., anxiety, irritability, depression).

Characteristic and pathognomonic skin changes in pellagra are also known as pellagroderma, and often follow the following clinical order:

Early findings:

Acute pellagra resembles sunburn in the beginning: the skin is red with large blebs or blisters that may exfoliate and leave large areas of denuded epithelium resembling sunburns. Over time and with subsiding lesions, dusky, brown-red skin
discoloration may happen.

Late findings:

In the second stage, dermatosis becomes hard, rough, cracked, blackish, and brittle. The skin may look like that of a goose (goose skin). Patients have thickened skin that is dry, scaly, and hyperkeratotic with a parchmentlike appearance and a
yellowish-brown hue. The skin is darkly pigmented.

Pellagra can affect any part of the body surface, but it more frequently appears in certain areas. The usual sites are the dorsal surfaces of the hands, face, neck, arms, and feet. The backs of the hands are the most common sites for the lesions (in
77-97% of cases), with accentuation of the radial border of the dorsal aspect.

In a nutshell, photosensitive skin changes in pellagra can be summarized as:

Bilateral and symmetrical with a well-demarcated sharp edge


Initially resembles sunburn – acute onset, red, swollen, painful burning
Occasional vesicles and/or bullae
Persistent redness, scaling, pigmentation
Becomes hyperpigmented, thickened, dry, and rough
Casal necklace — hyperpigmented band or collar around the neck
Pellagrous glove and boot — hyperpigmented plaques and fissuring of the hands and feet

1103 of 1943
Pellagra skin changes (early stages) - courtesy of MSD Manuals

Pellagra skin changes (later stages) - courtesy of MSD Manuals

Other mucocutaneous manifestations of pellagra may present as anogenital and mucosal lesions, glossitis, stomatitis, cheilitis, and sebaceous gland changes.

gastrointestinal manifestations:

Early symptoms — loss of appetite, vomiting, abdominal pain


Diarrhea — develops in 50% of cases, watery, mucoid, or bloody, persists longer than 4 weeks

Neuropsychiatric manifestations:

Late manifestation
Initial apathy and weakness
Progresses to headache, confusion, irritability, anxiety, tremor, depression
Dementia and encephalopathy

NOTE - Pellagra is usually treated with oral supplementation of niacin.

References

• DermNet - Pellagra

• Medscape - Dermatologic Manifestations of Pellagra

Time spent: QID:1792 Last updated:


2023-2-12

1104 of 1943
An 18-year-old girl was vaccinated against rubella as a part of antenatal work up and was advised to avoid pregnancy for at least 3 months. She was found to be pregnant 4 weeks after the rubella vaccination and is concerned about the outcome.
Which one of the following would be the correct management?

A. Termination of the pregnancy.

B. Reassurance.

C. Re-vaccination against rubella.

D. Check serum IgG level.

E. Check serum IgM level.

Incorrect. Correct answer is B


45% answered correctly

Explanation:

Correct Answer Is B

It is recommended that women who receive rubella vaccine avoid pregnancy for 3 months post-vaccination, despite the fact that no study has proven the increased risk of congenital rubella syndrome if the patient becomes pregnant during this
period.

If a patient becomes pregnant within 3 months of receiving the vaccine, reassurance is all needed and pregnancy should be continued.

Termination of pregnancy is considered if there is serological evidence of rubella infection during first trimester of pregnancy.

References

• https://www.sahealth.sa.gov.au/wps/wcm/connect/d81

• https://www.mja.com.au/journal/2002/176/5/1-infect

Last updated:
Time spent: QID:352 2023-2-12

1105 of 1943
A 23-year-old woman presents to the Emergency Department with lower abdominal pain and bleeding per vagina after a 6-week history of amenorrhea. Which one of the following sets of symptoms, if present, is highly suggestive of a tubal ectopic
pregnancy?

A. Board-like abdominal rigidity with both guarding and rebound tenderness.

B. Little guarding but marked rebound tenderness in the suprapubic region.

C. Rapid pulse and upper abdominal rebound tenderness.

D. Tenderness in the pouch of Douglas and tender adnexal mass.

E. Profound shock, tachycardia and hypotension.

Incorrect. Correct answer is B


45% answered correctly

Explanation:

Correct Answer Is B

Marked rebound tenderness in the suprapubic region with only little or absent guarding is the most common exam finding in patients with EP. The rebound tenderness is caused by the presence of blood in the peritoneal cavity.

Board-like rigidity is evident when chemicals or purulent material irritate the peritoneal cavity. Therefore, board-like rigidity is not an expected finding in EP.

NOTE - The pain caused by a tubal ectopic pregnancy in often felt in the lower abdomen and suprapubic region rather than the affected side.

Patients with EP have cervical motion tenderness rather than tenderness of the pouch of Douglas. Pelvic inflammatory disease (PID) is another differential diagnosis for cervical motion tenderness.

In patients with EP, adnexal tenderness, or pelvic tenderness in general, is much more common than pelvic mass on exam. A mass, if present, is more likely to have been caused by blood collection from an ectopic pregnancy rather than the ectopic
pregnancy itself.

Shock and its presentations (e.g., tachycardia, hypotension) are present when the ectopic pregnancy has ruptured and caused internal hemorrhage.

References

• AMC Handbook of Multiple-choice Questions – page 529

Last updated:
Time spent: QID:390 2023-2-12

1106 of 1943
A 32-year-old woman comes to your office to discuss about her recent cervical screening test result with you. The result reports low-grade squamous intraepithelial lesion (LSIL). Her last testing wilth the old pap smear 2 years ago was reported
normal. Which one of the following would be the most appropriate action to take?

A. Repeat the test in one year.

B. Refer for colposcopy.

C. Refer to an oncologist.

D. Repeat the test smear in 6 months.

E. Repeat the test in 2 years.

Correct
45% answered correctly

Explanation:

Correct Answer Is A

The following table includes guidelines for management of different pap smear results:

Pap smear report Investigation and management


Negative test – inflammatory cells Repeat the test in 5 years
Unsatisfactory test Repeat the smear in 6-12 weeks – this time period allows regeneration of the cells
Low-grade squamous intraepithelial lesion (LSIL)
Possible LSIL

And Repeat the test in 12 months High-grade squamous intraepithelial lesion (HSIL)

Definite LSIL
Possible HSIL
Referral to specialist for colposcopy
Definite HSIL
Glandular abnormalities including adenocarcinoma in situ Referral to gynecologist
Invasive squamous cell carcinoma or adenocarcinoma Referral to appropriate specialist gynecologist or unit
Inconclusive – raising possibility of high-grade disease Referral for colposcopy and possible biopsy

According to this guideline, as this woman had a normal pap smear 2 years ago, the next step would be repeating the smear in 12 months (one year).

References

• National Cervical Screening Program

Last updated:
Time spent: QID:417 2023-2-12

1107 of 1943
On a routine cervical screening, a 34-year-old woman is diagnosed with high-grade squamous intraepithelial lesion (HSIL) of the cervix. She was referred to a gynecologist, who treated her successfully. Now this patient has been referred back to you.
Which one of the following would be the next best step regarding surveillance?

A. Colposcopy and cervical cytology in 4 to 6 months.

B. Start 2-yearly colposcopy after 2 years.

C. Annual human papilloma virus typing.

D. No further testing is required.

E. Cervical screening every 2 years.

Correct
45% answered correctly

Explanation:

Correct Answer Is A

Once a woman has been treated for high grade squamous intraepithelial lesion(HSIL), she should undergo monitoring by:

Colposcopy and cervical cytology in 4 to 6 months after treatment.

Cervical cytology and human papilloma virus typing at 12 months after treatment and annually until tested negative for both tests on 2 consecutive occasions. She then can be safely returned to the standard 5-yearly screening intervals
for cervical cancer.

References

• National Cervical Screening Program

Last updated:
Time spent: QID:418 2023-2-12

1108 of 1943
A 43-year-old woman presents to your GP clinic with a cervical screening test result reporting invasive squamous cell carcinoma of the cervix. Which one of the following would be the next best step in management?

A. Repeat the test in 6 weeks.

B. Repeat the test in 12 months.

C. Colposcopy in your clinic.

D. Refer to a gynecological oncologist.

E. Reassure the patient and perform speculum examination.

Incorrect. Correct answer is D


45% answered correctly

Explanation:

Correct Answer Is D

According to the current guidelines, as shown in the following table, with a cervical screening test result significant for invasive carcinoma, the next best step would always be urgent referral to a gynecological oncologist.

TOPIC REVIEW

Action plans based on the cervical screening result are summarized in the following table:

Pap smear report Investigation and management


Negative smear – inflammatory cells Repeat the test in 5 years
Unsatisfactory test Repeat the test in 6-12 weeks – this period allows regeneration of the cells
Low-grade squamous intraepithelial lesion (LSIL)
Possible LSIL

And Repeat the test in 12 months

Definite LSIL
High-grade squamous intraepithelial lesion (HSIL)
Possible HSIL
Referral to specialist for colposcopy
Definite HSIL
Glandular abnormalities including adenocarcinoma in situ Referral to gynecologist
Invasive squamous cell carcinoma or adenocarcinoma Referral to appropriate specialist gynecologist or unit
Inconclusive – raising possibility of high-grade disease Referral for colposcopy and possible biopsy

References

• Cervical Screening Essentials

Last updated:
Time spent: QID:419 2023-2-12

1109 of 1943
A 35-year-old woman had a cervical screening at your clinic one year ago, which was reported as low-grade squamous intraepithelial lesion (LSIL). You repeat the test after 12 months reporting the same result. Which one of the following would be
the next best step in management?

A. Refer for colposcopy.

B. Repeat the cervical screening in 12 months again.

C. Perform cervical screening every 3 months.

D. Advise the patient for safe sex practices to reduce the risk of cervical cancer.

E. No action is required.

Correct
45% answered correctly

Explanation:

Correct Answer Is A

If a woman is older than 30 year old and cervical screening is positive for low-grade squamous intraepithelial lesion (LSIL), the test should be repeated in 12 months, provided that she has had a normal smear within the past 2-3 years. After 12
months, if the repeated cervical screening still shows LSIL, the patient should be referred for colposcopy as the most appropriate next step in management.

References

• Cervical Screening Essentials

• Murtagh’s General Practice – McGraw Hill – 7th Edition

Last updated:
Time spent: QID:420 2023-2-12

1110 of 1943
A 24-year-old pregnant woman presents in the first trimester of her pregnancy. Her pap smear report is significant for high-grade cervical squamous intraepithelial lesion (HSIL). Which one of the following would be the next best step in management?

A. Monitor her symptoms until after delivery.

B. Cone biopsy with endocervical curettage.

C. Hysterectomy.

D. Colposcopy.

E. Abortion induction before 9 weeks.

Incorrect. Correct answer is D


45% answered correctly

Explanation:

Correct Answer Is D

All women (pregnant or non-pregnant) with high-grade cervical squamous intraepithelial neoplasia on cervical screening test should have colposcopy of the cervix and biopsy of any abnormal areas observed during the procedure.

If no lesion is identified at colposcopy, it is advisable to request a review of all the cytological slides. If the diagnosis of a high-grade abnormality is confirmed, a second opinion from another colposcopist with wide experience in the colposcopic
evaluation of pregnant women is recommended. Re-evaluation in 20-24 weeks for cytology and colposcopy should be performed.

References

• Cancer Council - Cervical cancer screening: Screening in pregnancy

Last updated:
Time spent: QID:421 2023-2-12

1111 of 1943
An 18-year-old girl comes to see you at a medical centre. She just has become sexually active with her new boyfriend. She is concerned about her risk of cervical cancer and is keen to follow your advice. Which one of the following would be the most
appropriate advice in this regard?

A. She does not need cervical screening until she gets pregnant.

B. She should start 5-yearly HPV cervical screening from the age of 25 years and continue until the age of 70-74.

C. She is not at risk of contracting cervical cancer.

D. She should have 2 cervical screening tests at 5-year intervals and stop if both are normal.

E. She should not have sex with her boyfriend before marriage.

Incorrect. Correct answer is B


45% answered correctly

Explanation:

Correct Answer Is B

Guidelines for national cervical cancer screening program has been changed since December 2017. It is postulated that implementation of new guidelines will result in a 30% reduction in the mortality rate from cervical cancer. The following table
outlines the differences between the new and old guidelines:

Previous recommendations (no longer in use) Current recommendations (since December 2017)
Who? HPV vaccinated and unvaccinated women HPV vaccinated and unvaccinated women
What to do? Pap test screening HPV test
How often? Every 2 years Every 5 years
When to start? From 18-20 years of age or 2 years after the first sexual intercourse, From 25 years of age or 2 years after the first sexual intercourse,
whichever is later whichever is later
When to end? At 70 years of age for women who have had 2 normal pap test results Between 70 and 74 years of age
within the last 5 years. Women>70 years of age, who have never had a pap
test, or who request a pap test should be screened.

NOTE -

Women who have never engaged in sexual relationship do not need pap smears
Lesbian women require pap testing similar to heterosexual women even if they never had a male sexual partner

Since this girl has just started sexual relationship she should start 5-yearly HPV testing after the age of 25 years and continue unitl the age of 70-74 years.

References

• National Cervical Screening Program

Last updated:
Time spent: QID:422 2023-2-12

1112 of 1943
A 29-year-old woman comes to your clinic for advice regarding cervical screening. She has never had sexual activity with men or women. Regarding cervical screening, which one of the following would be the best advice for her?

A. Annual cervical screening.

B. As long as sexually inactive, no cervical screening is required for her.

C. 2-yearly cervical screening.

D. She should start sexual activity as it is safe now.

E. She should see a psychiatrist, as she might have underlying mental illness.

Incorrect. Correct answer is B


45% answered correctly

Explanation:

Correct Answer Is B

Current guidelines recommend 5-yearly HPV testing of all vaccinated and unvaccinated women from the age of 25 years or 2 years after the first sexual relationship, whichever is later, to 70-74 years. This has replaced the previous recommendations
of performing 2-yearly pap smears for women from the age of 18 years or 2 years after the first sexual relationship.

However, women, who never had sexual relationship (neither with men, nor women), like this woman, do not need cervical screening. Once this woman starts sexual relationship, cervical screening would be indicated in 2 years and repeated every 5
years until the age of 70-74.

Cervical cancer is so strongly associated with sexual relationship, which can be classified as a sexually transmitted disease (STD); therefore, not having sexual relationship usually means no cervical cancer.

Sexual relationship without precautionary measures increases the risk of cervical cancer at any age. The risk of cervical cancer rises after the first unprotected sexual relationship, regardless of the age at which it is started.

Advising her to see a psychiatrist is not appropriate because it is not uncommon for women to not have experienced any sexual relationship for many years through their puberty. On the other hand, there is no history of mental illness to necessitate
psychiatric evaluation.

References

• Cervical Screening Essentials

Last updated:
Time spent: QID:423 2023-2-12

1113 of 1943
A 30-year-old woman presents to the emergency department at 37 weeks’ gestation after sudden onset of severe abdominal pain, vaginal bleeding (approximately 1200 cc) , and cessation of contractions after 18 hours of active pushing at home.
The pregnancy has been uneventful until the event. On examination, she is conscious and pale, with a blood pressure of 70/45 mmHg and pulse rate of 115bpm. The abdomen is irregularly distended. Shifting dullness and fluid thrill are present. Fetal
heart sounds are not audible. Which one of the following is the most likely diagnosis?

A. Placenta previa.

B. Placental abruption.

C. Uterine rupture.

D. Cervical laceration.

E. Disseminated intravascular coagulation.

Incorrect. Correct answer is C


45% answered correctly

Explanation:

Correct Answer Is C

The scenario is typical for uterine rupture. The condition presents with sudden abdominal pain, cessation of uterine contraction, the urge to push, and vaginal bleeding. On examination, the fetal heart rate is decreased or lost. Signs of fluid collection
are often present. These signs include fluid thrill and shifting dullness caused by entrance of the blood into the peritoneal cavity.

Other manifestations of uterine rupture include:

Loss of the station of the fetal presenting part


Vaginal bleeding that might not be proportionate to the hemodynamic status
Maternal tachycardia and hypotension ranging from subtle to severe (shock)
Uterine tenderness
Change in uterine shape and contour
Easily palpable fetal parts
No fetal presentation on vaginal examination
Hematuria if the rupture extends to the bladder

The most common site of spontaneous uterine rupture is the anterior lower transverse segment.

(Option A) Placenta previa causes painless vaginal bleeding, so cannot be the diagnosis here.

(Option B) Placental abruption presents with vaginal blood loss and often a tender and tense uterusl. Contrary to uterine rupture, in placental abruption uterine contractions continue and do not stop.

(Option D) Cervical laceration can be a possibility, but if it was the source of bleeding, more amount of vaginal blood loss was expected in this patient with hemodynamic instability. Furthermore, deformed uterus, abdominal distention and cessation
of the contractions are inconsistent with cervical laceration as the cause of the bleeding.

(Option E) Disseminated intravascular coagulation (DIC) is a condition resulting from excessive generation of thrombin and fibrin in the circulating blood. It leads to increased platelets aggregation and consumption of coagulation factors with
consequent bleeding at one site and thromboembolism at another. The most common obstetrical causes of DIC are placental abruption and retained products of conception in the uterine cavity. DIC does not fit this clinical scenario.

References

• http://www.sahealth.sa.gov.au/wps/wcm/connect/0f2c

• http://www.uptodate.com/contents/rupture-of-the-un

Last updated:
Time spent: QID:425 2023-2-12

1114 of 1943
48-year-old woman with past history of hysterectomy comes to discuss about hormone replacement therapy (HRT) for her post-menopausal symptoms, while she is also concerned about the associated risk of breast cancer. Which one of the
following is the best advice for her?

A. Postmenopausal women with BMI <25 have a lower relative risk of developing breast cancer associated with estrogen-only HRT than those women with higher BMIs.

B. There is consistent evidence indicating that tibolone may be associated with an increased risk of breast cancer.

C. Use of estrogen-alone HRT for up to seven years appears to have no effect on risk of breast cancer.

D. She does not need hormone replacement therapy.

E. She should use hormone replacement therapy only for few weeks.

Incorrect. Correct answer is C


45% answered correctly

Explanation:

Correct Answer Is C

The best option for HRT in women who do not have a uterus due to previous hysterectomy is estrogen alone. These patients should be advised that this regimen seems not to increase the risk of developing breast cancer if used for only up to
seven years. Beyond this time, however, the risk increases proportionate to the duration of HRT.

Post-menopausal women with a BMI less than 25 have a higher relative risk of contracting breast cancer associated with estrogen-only hormone replacement therapy, than women with higher BMI. The reason can be attributed to less metabolization
of estrogen in the adipose tissue.

Tibolone is used for hormone replacement therapy and for treatment of endometriosis. There is very inconsistent evidence indicating that tibolone may be associated with an increased risk of breast cancer.

While this woman has menopausal symptoms, advising against HRT is not appropriate. She can use HRT for a longer duration compared to women who still have uterus and are at risk of developing endometrial cancer.

1115 of 1943
A 3-year-old Sudanese boy is being assessed as a part of a health check for refugees. He has dark skin, and Further inquiry reveals that he has a history of multiple fractures at different times, which the parents attribute to his clumsiness. When
asked why they feel he is clumsy, they answer that he started walking very late and prefers to sit down rather than walk or play with other children. Based on multiple risk factors in the history, you order a serum vitamin D level among other tests
which is positive for vitamin D deficiency. Which of the following is the most appropriate treatment option for him at this stage?

A. Calcitonin.

B. Calcitriol.

C. Calcium supplementation.

D. Cholecalciferol.

E. More sun exposure.

Incorrect. Correct answer is D


45% answered correctly

Explanation:

Correct Answer Is D

There is increasing recognition that many Australians and people from specific groups within the community suffer from vitamin D deficiency.

Vitamin D deficiency can be classified as:

mild (25–50 nmol/L)


moderate (12.5–25.0 nmol/L)
severe (<12.5 nmol/L) with levels of 50–100 nmol/L thought of as being vitamin D insuffici><12.5 nmol/L)

With levels of 50–100 nmol/L, vitamin D insufficiency rather than deficiency is used.

Vitamin D is important for muscle and bone health. Vitamin D deficiency is associated with:

Cortical bone loss


Increased bone turnover
Increased parathyroid hormone levels
Predisposing to osteoporosis
Vitamin D supplementation increases bone density in established deficiency.
Vitamin D deficiency has associations with:
Cardiovascular disease
Insulin resistance and β-cell dysfunction
The development of autoimmune diseases (including type 1 diabetes, rheumatoid arthritis, and multiple sclerosis)
Colon, breast, and prostate cancers

The following groups are at significant risk of vitamin D deficiency:

The institutionalized or housebound individuals


Those with sun-avoidant behavior
Refugees
Those with celiac disease (or other malabsorptive conditions)
Those from areas of high vitamin D deficiency prevalence
Those in whom osteoporosis therapy is considered
Those with chronic idiopathic musculoskeletal pain
Pregnant women (particularly if dark-skinned or veiled)

Vitamin D deficiency is often a silent disease. Prolonged vitamin D deficiency results in Rickets in children whose growth plates have not fused. These children are often found to have started walking late or prefer to sit down for prolonged periods. In
adults, vitamin D deficiency results in osteomalacia, which presents as a poorly mineralized skeletal matrix. Adults in these cases can experience chronic muscle aches and pains.

Current guidelines recommend screening only those individuals who are at high risk for vitamin D deficiency, including patients with osteoporosis or a malabsorption syndrome, as well as black and Hispanic individuals, obese persons (BMI >30
kg/m2), and those with several other medical conditions.

The daily maintenance dose of vitamin D varies by age, but most children and adults generally require 600-2000 IU of vitamin D daily. For vitamin D-deficient children and adults, higher doses of vitamin D given either daily or weekly are recommended,
followed by an increase in the daily dose of vitamin D.

Current guidelines endorse supplementation with cholecalciferol (25- hydroxyvitamin D). also known as D3, or ergocalciferol (plant bases vitamin D). Cholecalciferol is the best treatment option for this child at this stage.

For maintenance, a diet rich in vitamin D should also be recommended.

Sun exposure is the most important source of vitamin D production in the human body. Safe sun exposure is recommended for maintaining healthy levels of vitamin D; however, with vitamin D deficiency or insufficiency, supplementing vitamin D is
required first to push vitamin D levels within normal ranges. Moreover, dark-skinned people may take less benefit from sun exposure (option E). In fact, their skin color could have contributed to such deficiency in the first place.

Vitamin is required for calcium absorption and metabolism. Reduced amounts of vitamin D result in impaired calcium absorption and this child may have calcium issues as well. However, calcium supplementation (option C) does not treat vitamin D
deficiency. Moreover, vitamin D must reach a safe level before calcium supplementation is considered.

Calcitonin (option A) is used to treat hypercalcemia due to Paget’s disease of the bone, or hypercalcemia due to other causes. It also may be used to prevent bone loss in postmenopausal women with osteoporosis, but it has no role in the treatment
of vitamin D deficiency.

Calcitriol (1,25-dihydroxyvitamin D3) (option B) is the most active form of vitamin D and generally not suitable for the treatment of vitamin D deficiency as it has a narrow therapeutic window resulting in an increased risk of hypercalcemia or
hypercalciuria. This is especially true in nursing home residents who often have severe vitamin D deficiencies. Calcitriol has a role in the treatment of vitamin D deficiency in renal failure where there is an inability to convert 25-hydroxyvitamin D
(vitamin D3) to 1,25-dihydroxy vitamin D. Serum calcium concentrations and renal function must be monitored closely under these circumstances.

References

1116 of 1943
• Medscape - Vitamin D Deficiency and Related Disorders

• RCH - Vitamin D deficiency

• Australian Prescriber - Vitamin D deficiency in Adults


Last updated:
Time spent: QID:1798 2023-2-12

1117 of 1943
A 37-year-old woman presents to your clinic for an antenatal checkup. She has past obstetric history of breech presentation, premature rupture of membranes and twin pregnancy. She also had postpartum depression during her last two pregnancies as well as anemia and
gestational diabetes in her last pregnancy.Which one of the following would not increase the risk for cord prolapse during delivery?

A. Breech presentation.

B. Multiple gestation.

C. Premature rupture of membrane.

D. Anemia.

E. Gestational diabetes.

Incorrect. Correct answer is D


45% answered correctly

Explanation:

Correct Answer Is D

Cord prolapse should be suspected where there is an abnormal fetal heart rate pattern (bradycardia, variable decelerations etc), particularly if such changes commence soon after the ruputre of the membranes, spontaneously or with amniotomy.

The following are risk factors predisposing to cord prolapse:

Although fetal malpresentation is a risk factor, most cord prolapses occur with vertex presentations because of the relatively
Fetal malpresentation (breech, transverse lie, etc)
low incidence of noncephalic presentation
Unengaged fetal presenting part ---
Probably due to the smaller size of the fetus relative to the amniotic fluid volume and the increased frequency of
Prematurity
malpresentation among premature fetuses
The risk of cord prolapse in a term twin pregnancy is confined to the second born twin, in whom there is an increased
Multiple gestation
probability of malpresentation
Iatrogenic or spontaneous rupture of membranes can lead to cord prolapse because a forceful gush of fluid may carry the
Premature rupture of membranes cord beyond the presenting fetal part. The highest risk of this occurrence is with preterm premature rupture of membranes,
polyhydramnios, or an unengaged fetal presenting part
Abnormal placentation ---
Cord prolapse in multiparous women may be related to the increased likelihood of rupture of membranes prior to engagement
Multiparity
of the presenting part, since engagement in multiparas often occurs after labor has begun and later than in nulliparas
Polyhydramnios is often associated with an unstable lie or unengaged presenting part, as well as copious flow of amniotic
Polyhydramnios e.g. in gestational diabetes
fluid after membrane rupture
Long umbilical cord ---
Pelvic deformities ---
Uterine tumours and malformations ---
Congenital anomalies ---
Interventions that may predispose to cord prolapse include:

Iatrogenic rupture of membranes


Application of an internal scalp electrode
Insertion of an intrauterine pressure catheter
Manual rotation of the fetal head
Amnioinfusion or amnioreduction
External cephalic version in patients with ruptured membranes
Application of forceps or vacuum

Anemia is not a predisposing factor to cord prolapse.

References

• Royal College of Obstetricians and Gynaecologists – Green-top Guideline – No.50

• UpToDate - Umbilical cord prolapse

Last updated:
Time spent: QID:426 2023-2-12

1118 of 1943
A 21-year-old woman, G1P0 comes to your clinic at 12 weeks pregnancy. She is complaining of mild vaginal bleeding for the past 12 hours, associated with bouts of mild cramping lower abdominal pain. On vaginal examination, the cervical os is
closed and there is mild discharge containing blood clots. Ultrasonography confirms the presence of a live fetus with normal heart rate. Which one of the following is the most likely diagnosis?

A. Threatened abortion.

B. Inevitable abortion.

C. Incomplete abortion.

D. Missed abortion.

E. Complete abortion.

Correct
45% answered correctly

Explanation:

Correct Answer Is A

Uterine bleeding in the presence of a closed cervix and sonographic visualization of an intrauterine pregnancy with detectable fetal cardiac activity is diagnostic of threatened abortion.

The term ‘threatened’ is used because abortion does not always follow uterine bleeding in early pregnancy, even after repeated episodes or large amounts of bleeding.

In fact, in 90 to 96% of cases who present with vaginal bleeding in the presence of a closed os and a detectable fetal heart rate, the pregnancy continues. The more advance the gestational age, the less likely the condition will end in miscarriage.

(Option B) In inevitable abortion, there is a dilated cervix as well as progressive uterine bleeding and painful uterine contractions. The gestational tissue often can be felt or seen through the cervical os and passage of this tissue typically occurs
within a short time.

(Option C) In incomplete abortion, the membranes may have ruptured, and the conception products may have partly passed, but significant amounts of placental tissue are still left in the uterus. This is the most common presentation of an abortion
in late first and early second trimester. On examination, the cervical os is open, gestational tissue may be observed in the cervix, and the uterine size is smaller than expected for gestational age. The uterine is not well contracted. The amount of
bleeding varies but can be severe enough to cause hypovolemic shock. Painful contractions are often present. Ultrasound reveals tissue in the uterus.

(Option D) A missed abortion refers to in utero death of the embryo or fetus prior to 20 weeks gestation. Women may notice that symptoms associated with early pregnancy (nausea, breast tenderness, etc.) have abated and they do not 'feel
pregnant' anymore. Vaginal bleeding may occur, and the cervix usually remains closed. Ultrasound reveals an intrauterine gestational sac with or without an embryonic/fetal pole, but no embryonic/fetal cardiac activity.

(Option E) In complete abortion, a miscarriage occurs before week 12 and the entire contents of the uterus is expelled. If this has occurred, the uterus is small on physical examination and well contracted with an open or closed cervix. There is scant
vaginal bleeding, and only mild cramping. Ultrasound will reveal an empty uterus and no extra-uterine pregnancy.

References

• UpToDate - Pregnancy loss (miscarriage): Risk factors, etiology, clinical manifestations, and diagnostic evaluation

Last updated:
Time spent: QID:427 2023-2-12

1119 of 1943
A 35-year-old woman presents to your GP clinic with complaints of irritability, anxiety, breast tenderness and headaches about 5 days before the onset of menses every months for the past 6 months. These symptoms are relieved with the onset of
menses. She has to take a week off from work because of these symptoms. She is non-alcoholic and does not use any regular medication. Which one of the following is the most likely diagnosis?

A. Premenstrual dysphoric disorder.

B. Premenstrual syndrome.

C. Normal menstrual physiology.

D. Generalized anxiety disorder.

E. Depression.

Incorrect. Correct answer is B


45% answered correctly

Explanation:

Correct Answer Is B

This woman fufils diagnostic criteria for premenstrual syndrome (PMS) as the most likely diagnosis.

Diagnostic criteria for premenstrual syndrome include affective and somatic symptoms starting within one week before menstruation and resolving within 4 days after menstrual flow starts. The symptoms should be recurrent for at least 3 menstrual
cycles and must be absent in the preovulatory phase of the menstrual cycle. The specific symptoms are less important than their temporal relationship to the menstrual cycle.

Affective symptoms include, but not limited to:

Depression
Anger outbursts
Irritability
Social withdrawal
Anxiety
Tearfulness
Helplessness/hopelessness
The patient suffers from identifiable dysfunction in social or economic performance

Somatic symptoms include, but not limited to:

Breast tenderness
Abdominal bloating and pain
Headache
Muscle and joint pain
Eedema
Weight gain
Heart pounding
Confusion, dizziness

Symptoms are usually relieved within 4 days of onset of menses.

It is important that these symptoms cannot be attributed to any medical or psychological disorder, medications, drug, or alcohol.

(Option A) Premenstrual dysphoric disorder is severe form of premenstrual syndrome and is characterised by severe feeling of sadness, emotional labiality with frequent tearfulness, loss of interest in daily activities, decreased concentration, fatigue,
insomnia and feeling of being overwhelmed or out of control. Symptoms are more related to affections rather than being somatic. In other words, affective impairment is the dominating feature.

The Diagnostic and Statistical Manual of Mental Disorders, Fifth Edition (DSM-5), established 4 research criteria (A through D) for the diagnosis of PMDD.

Criterion A - in most menstrual cycles during the past year, at least 5 of the following 11 symptoms (including at least 1 of the first 4 listed) were present:

1. Markedly depressed mood, feelings of hopelessness, or self-deprecating thoughts


2. Marked anxiety, tension, feelings of being “keyed up” or “on edge”
3. Marked affective lability (e.g. feeling suddenly sad or tearful or experiencing increased sensitivity to rejection)
4. Persistent and marked anger or irritability or increased interpersonal conflicts
5. Decreased interest in usual activities (e.g. work, school, friends, and hobbies)
6. Subjective sense of difficulty in concentrating
7. Lethargy, easy fatigability, or marked lack of energy
8. Marked change in appetite, overeating, or specific food cravings
9. Hypersomnia or insomnia
10. A subjective sense of being overwhelmed or out of control
11. Other physical symptoms, such as breast tenderness or swelling, headaches, joint or muscle pain, a sensation of bloating, or weight gain

The symptoms must have been present for most of the time during the last week of the luteal phase, must have begun to remit within a few days of the onset of menstrual flow, and must be absent in the week after menses.

Criterion B - the symptoms must be severe enough to interfere significantly with social, occupational, sexual, or scholastic functioning. For example, the patient may avoid social activities or exhibit decreased productivity and efficiency at work or
school.

Criterion C - the symptoms must be discretely related to the menstrual cycle and must not merely represent an exacerbation of the symptoms of another disorder, such as major depressive disorder, panic disorder, dysthymic disorder, or a personality
disorder (although the symptoms may be superimposed on those of any of these disorders).

Criterion D- criteria A, B, and C must be confirmed by prospective daily ratings during at least 2 consecutive symptomatic menstrual cycles. The diagnosis may be made provisionally before this confirmation.

With only pyschological symptoms of irritability and anxiety and phyical symptoms of headache and breast tenderness as one symptom, this woman does not fulfil the diagnostic criteria PMDD.

(Option C) Symptoms so severe cannot be attributed to normal physiology of menstruation.

(Options D and E) As these symptoms are temporally andf related to menstrual cycles, times generalized anxiety disorder and depression are unlikely diagnoses.

References

1120 of 1943
• http://www.uptodate.com/contents/premenstrual-synd

• Royal College of Obstetricians and Gynaecologists – Green-top Guideline – No.48

• The Diagnostic and Statistical Manual of Mental Disorders - 5th Edition (DSM-5)
Last updated:
Time spent: QID:428 2023-2-12

1121 of 1943
A 27-year-old female, who is 37 weeks pregnant, presents to the Emergency Department with sudden onset of severe headache. She feels nauseous and dizzy. Her mother noted slurred speech and repetition of words after the headache started. She
is clinically stable and has not had any obstetric complication so far. She has no history of migraine. Which one of the following would be the most appropriate next step in management?

A. Aspirin.

B. CT scan of the brain.

C. MRI of the brain.

D. Lumbar puncture.

E. Observation in the ward.

Incorrect. Correct answer is B


45% answered correctly

Explanation:

Correct Answer Is B

The clinical picture is suggestive of subarachnoid hemorrhage (SAH), and non-contrast CT scan of the brain would be the next best step in management.

Pregnancy is a recognized risk factor for aneurysmal SAH. SAH is a very rare complication of pregnancy and may be confused with eclampsia, due to similar presentation with high blood pressure, seizures and altered levels of consciousness. Once
the condition is suspected, the diagnosis is confirmed with CT scan (preferred), MR, or cerebral angiography. Although CT scanning exposes the fetus to radiation, the benefits significantly outweighs the risks. Appropriate covering of the uterus may
reduce the risks to the fetus.

(Option A) Aspirin should be avoided unless full assessment excludes the possibility of intracranial hemorrhage.

(Option C) CT scan is more sensitive than MRI in detection of blood and is the first-line investigation where intracranial hemorrhage is suspected.

(Option D) If CT scan of the brain is inconclusive, lumbar puncture comes next. It should be performed within 12 hours after the onset of the symptoms.

(Option E) Subarachnoid hemorrhage is a real emergency demanding immediate workup; therefore, just observing the patient in the ward would be inappropriate.

References

• Aneurysmal Subarachnoid Haemorrhage in Pregnancy: A Case Series

• UpToDate - Cerebrovascular disorders complicating pregnancy

Last updated:
Time spent: QID:429 2023-2-12

1122 of 1943
In which one of the following conditions, it might be advised that breastfeeding be stopped?

A. Engorged breasts.

B. Inverted nipples.

C. Cracked nipples.

D. Breast abscess.

E. Mastitis.

Incorrect. Correct answer is D


45% answered correctly

Explanation:

Correct Answer Is D

It is recommended that breastfeeding be continued from both breasts in cases of breast abscess. However, as the treatment of the abscess would be incisional drainage, the location of the incision may prevent the baby from being directly breastfed.
In case of breast abscess it is recommended that breastfeeding be discontinued until the abscess resolves. Milk should be expressed manually and be given to the baby.

(Option A) Engorged breast occurs with excessive milk production or decreased output due to inappropriate breastfeeding technique or the baby not taking enough milk. It results in accumulation of milk in breasts and engorgement. Breast feeding
should be continued. Proper breastfeeding on baby’s demand is the key management of this condition.

(Option B) Inverted nipple occurs when one or both nipples invert into the breast instead of pointing outwards. Breast feeding should be continued in mothers with inverted nipples.

(Option C) Cracked nipples are usually due to the baby clamping the mouth around the nipple instead of applying the jaws behind them. The best approach is to rest the nipple for 1 to 2 days. The milk of the affected breast should be expressed
manually and given to the baby with bottle. After 1 to 2 days of rest, breastfeeding should be resumed with short feeds and proper technique.

(Option E) Mastitis is cellulitis of the interlobular connective tissue of the breast. It is mainly associated with cracked nipples or poor milk drainage (engorgement) in a lactating mother. The infecting organism is Staphylococcus aureus. Breast
feeding from the affected side can be continued as infection is confined to the interstitial breast tissue.

References

• RACGP - Lactational mastitis and breast abscess - Diagnosis and management in general practice

• Australian Breastfeeding Association - Breast Abscess

Last updated:
Time spent: QID:430 2023-2-12

1123 of 1943
A 32-year old woman comes to your clinic for advice. She is 12-week pregnant, and has already been on folic acid for the past 24 weeks. Which one of the following should be started for her?

A. Iodine for a few weeks.

B. Vitamin B12 injections.

C. Vitamin A supplements.

D. Vitamin C.

E. Iodine throughout the pregnancy.

Incorrect. Correct answer is E


45% answered correctly

Explanation:

Correct Answer Is E

Clinical guidelines for antenatal care by National Health and Medical Research Council of Australia recommend folic acid 0.5 mg daily 12 weeks before conception and during first trimester. The dose of folic acid is increased to 5mg daily in the
following situations:

History of neural tube defects in previous pregnancies


The pregnant woman is diabetic
The pregnant woman is on anticonvulsant medications

The mother is also recommended to take 150 microgram of iodine on a daily basis throughout the pregnancy for prevention of iodine deficiency in the fetus.

She has taken folic acid for the first trimester; therefore it can safely be stopped.

While there is evidence to support routine supplementation with folic acid and iodine in pregnancy, other vitamin and mineral supplements are not of benefit unless there is an identified deficiency. Vitamin A, B and C supplements are not routinely
used in pregnancy.

References

• http://www.health.gov.au/internet/main/publishing.

Last updated:
Time spent: QID:431 2023-2-12

1124 of 1943
A 32-year-old woman presents to your clinic for advice. She has been married for the past 2 years and has not been able to conceive. Blood tests show the following results:

Luteinizing hormone (LH): 1 U/L (3-16)


Follicle stimulating hormone (FSH): 0.5 U/L (2 – 8)
Thyroid stimulating hormone (TSH): 0.3 mU/L (0.5 -5)
Prolactin: 8750 U/L (<600 U/L)

Which one of the following is most likely to result in feritility in this woman?

A. Clomiphene citrate.

B. In vitro fertilisation.

C. Metformin.

D. Bromocriptine.

E. Surgical resection of the pituitary tumour.

Incorrect. Correct answer is D


45% answered correctly

Explanation:

Correct Answer Is D

The laboratory picture suggests hyperprolactinemia probably due to a prolactinoma. High levels of circulating prolactin, through a negative feedback, lead to co-inhibition of THS, FHS, LH and probably ACTH secretion. Once the prolactinoma is
treated successfully with dopamine agonists such as bromocriptine, the condition is controlled. Prolactin-secreting adenomas of the pituitary gland (prolactinomas) can lead to hyperprolactinemia-induced infertility.

Bromocriptine is initially used for induction of tumor shrinkage. As the tumor shrinks, the prolactin levels become more normal and fertility resumes. Once the prolactinoma is biochemically diagnosed, MRI scan of the head should be considered to
confirm and localize the adenoma before commencement of the therapy.

References

• Therapeutic Guidelines – Endocrinology; available from http://tg.org.au

Last updated:
Time spent: QID:432 2023-2-12

1125 of 1943
A 34-year-old pregnant woman presents at 32 weeks pregnancy for an antenatal visit. On pelvic examination, breech presentation is suspected. Ultrasound shows that fetal hips are flexed, and the knees extended. Which one of the following is this
presentation called?

A. Complete breech.

B. Frank breech.

C. Footling breech.

D. Kneeling breech.

E. Transverse lie.

Incorrect. Correct answer is B


45% answered correctly

Explanation:

Correct Answer Is B

Types of breech presentation include:

Frank breech: The fetal hips are flexed, and the knees extended.

Complete breech: The fetus seems to be sitting with hips and knees flexed.

Footling breech: One or both legs are completely extended and present before the buttocks.

Kneeling breech: The baby is in a kneeling position, with one or both legs extended at the hips and flexed at the knees.

Fetal position is transverse if the fetal long axis is oblique or perpendicular rather than parallel to the maternal long axis.

With the hips flexed and knees extended, this fetus has Frank breech presentation. This type is the most common breech presentation.

References

• https://www.sahealth.sa.gov.au/wps/wcm/connect/227

Time spent: QID:433 Last updated:


2023-2-12

1126 of 1943
A 23-year old Asian woman comes to your clinic at 36 weeks gestation. She was diagnosed with breech at 32 weeks. She is not in labor and manual examination of the uterus is suggestive of breech position. Which one of the following would be the
next best step in management?

A. Induction of labor.

B. Cesarean delivery.

C. Steroids.

D. Pelvic ultrasound.

E. Admission to the hospital and arrange for blood transfusion.

Incorrect. Correct answer is D


45% answered correctly

Explanation:

Correct Answer Is D

When a breech presentation suspected by manual exam, ultrasonography is needed as the next best step in management for confirmation, as well as evaluation of maternal pelvis, the fetal size and viability. This should be considered in every future
visit. Breech presentation has the chance to spontaneously turn into cephalic 36 to 37 weeks. Once the breech position persists beyond this time, the chance of spontaneous version reduces to 25%. It is recommended that all women with breech
presentation be offered an external cephalic version provided that there is no contraindication and cesarean delivery is not indicated for other reasons..

References

• http://www.kemh.health.wa.gov.au/development/manua

• https://www.sahealth.sa.gov.au/wps/wcm/connect/227

Last updated:
Time spent: QID:434 2023-2-12

1127 of 1943
Which one of the following is not a contraindication to vaginal delivery in breech presentation at term?

A. Placenta previa.

B. Footling breech presentation.

C. Estimated fetal weight > 3800g.

D. Clinically inadequate pelvis.

E. Anemia.

Incorrect. Correct answer is E


45% answered correctly

Explanation:

Correct Answer Is E

Vaginal breech delivery may be offered provided that there are no contraindications to vaginal delivery.

Contraindications to vaginal breech delivery include:

Contraindication to vaginal birth (e.g. placenta previa, contracted pelvis, cord presentation)
Prior cesarean deliveries
Fetal anomalies that may cause dystocia
Estimated fetal weight <2500 g and >3800 g.
Gestational age <36 weeks
Hyperextension of the fetal head (i.e. an extension angle of greater than 90 degrees)
Any presentation other than frank or complete breech presentation (incomplete breech)
Anticipated mechanical difficulty

Anemia is not a contraindication to vaginal delivery for breech presentation.

References

• http://www.kemh.health.wa.gov.au/development/manua

Last updated:
Time spent: QID:435 2023-2-12

1128 of 1943
A 28-year-old man is brought to the Emergency Department after he had an accident while driving a motorcycle and had his right ankle injured. On examination, his vital signs are
stable. The right ankle joint is laterally displaced and there is a laceration over the joint. Which one of the following is the most important initial step in management?

A. Wound debridment.

B. Tetanus immunization.

C. Intravenous antibiotics.

D. Reduction of the displacement.

E. X-ray of the joint.

Incorrect. Correct answer is D


45% answered correctly

Explanation:

Correct Answer Is D

Open fractures and/or dislocations are defined as the bone and/or joint being exposed to the external environment, or when the fracture or dislocation is caused by blunt or
penetrating forces sufficient to disrupt or penetrate skin, subcutaneous tissue, muscle fascia, muscle, and/ or the bone or joint. Open fractures are often contaminated by foreign
material (e.g., clothing, grass, dirt, gravel), dead or devitalized tissue, and bacteria.

Always follow the following rules in open fractures/ dislocations:

Remove any gross contamination off the wound. No provisional irrigation or debridement is performed at this stage.

Take photographs of the wound and dress it with wet sterile cover (the photographs are taken, so that other treating physicians do not need to uncover the wound to see
it)

Give patient analgesic (preferably intravenously to both control the pain and prepare for a reduction in the emergency department). Morphine is a good option.

By gentle traction, reduce the fracture and correct misalignments as much as possible.

Start the patient on intravenous prophylactic antibiotics.

Give the patient tetanus prophylaxis if indicated.

Obtain X-rays.

Urgently arrange for transferring the patient to the operating room for surgical wound debridement and definite treatment of the fracture and dislocation.

NOTE - Initial irrigation or debridement in the emergency department is not recommended and should be avoided.

In this scenario the most appropriate initial management would be reduction of the dislocation by gentle traction after non-surgical removal of any gross contamination, and
analgesia. The patient then should be started on intravenous antibiotics and receive tetanus prophylaxis.

X-ray of the joint (option E) is required pre-operatively to visualize the anatomical disruption. Wound debridement (option A) is crucial but should be performed in the operating
room ideally with 1-2 hours of presentation.

NOTE - In open fractures, wound treatment may sometimes be more time-consuming and require more work than the treatment of the fracture itself. Debridement may be
carried out several times before and after definitive fracture treatment is performed.

TOPIC REVIEW

Wound debridement including surgical removal of all devitalized soft tissues and bone is a crucial step in management of any open fracture/dislocation because ensuing infections
may resulting in poor outcomes. However, debridement should be performed in the sterile environment of the operating room, not in the non-sterile emergency department where
more exploration of the wound can lead to increased risk of infection.

The golden time for treatment of open fractures/ dislocations is 6 hours.

In fractures/ dislocations where there are concomitant vascular and nerve injuries the order of priorities is as follows:

1. Bone – first correct any misalignment and dislocation and fix if needed because firstly, some neurovascular compromise might be reverted by alignment of the dislocated
bone/ joint. Secondly, by manipulation of the bone after neurovascular repair has been carried out, there is a risk that neurovascular disruption occurs again accidentally.
2. Vessels
3. Nerves

1129 of 1943
A 72-year-old man was brought to the emergency department after he sustained a fracture of his left hip following a fall at home. Initial pain management was considered and the
operation carried out with intramedullary nail placement. Today, he is being discharged. Which one of the following is the most important management option to consider for him on
discharge?

A. Painkillers.

B. Walker.

C. Crutches.

D. Low molecular weight heparin.

E. Warfarin.

Correct
45% answered correctly

Explanation:

Correct Answer Is A

According to guidelines provided by the Agency for Clinical Innovation (ACI), it is crucial to mobilize patients with hip fracture within 24 hours of the surgery. Pain following hip
fracture should be presumed severe and continuous. Effective pain management is a primary goal for patients with a hip fracture, because immobility caused by pain has been
associated with the increased risk of the following:

Pressure ulcers
Pneumonia
Venous thromboembolism (VTE)

With greater pain, the likelihood of the following will increase:

Delirium
Sleep disturbances
Depression

Pain control is of significant importance for early mobilization of patients who have undergone hip surgery, because the pain brought on by physical activity is the most important
factor preventing from being mobile; therefore, pain control with painkillers play a crucial role in early mobilization.

Pain is often undertreated or poorly controlled in orhtogeriatric patients due to:

1. Reluctance or inability of an older patient to request analgesia


2. Reluctance of medical staff to prescribe analgesia in older frail patients
3. Cognitive impairment in patients, making assessment of pain level difficult

Pain management techniques for patients with hip fracture include pharmacological approaches such as paracetamol and opioids and use of femoral nerve block.

(Options B and C) Walking aids such as crutches or walkers should be advised to provide more convenient mobilization, but they are not superior to analgesics, because with pain
the patient is unlikely to become physically active and changes posture (e.g. from sitting to standing or walking).

(Option D) LMWH is also important in prevention of VTE in this patient, but this has already been started immediately after the surgery and continued during the hospital stay.

(Option E) Warfarin is not routinely used for prophylaxis of VTE in patients with hip surgery.

References

• https://anzhfr.org/wp-content/uploads/2016/07/ANZ-Guideline-for-Hip-Fracture-Care.pdf

Last updated:
Time spent: QID:101
2023-2-12

1130 of 1943
A 35-year-old woman is brought to the Emergency Department after she sustained a motor vehicle accident as a frontseat passenger, and had her left ankle injured. On examination,
her vital signs are stable. The left ankle joint is laterally displaced and there is a 13-cm laceration over the joint. The left dorsal pedis pulse is barely felt and the foot is cold and pale.
The condition is diagnosed as open fracture/dislocation of the ankle joint. Which one of the following is the most important step in preventing wound infection and ensuing
complications?

A. Intravenous antibiotics.

B. Wound debridment.

C. Tetanus prophylaxis.

D. Reduction of the displacement.

E. X-ray of the joint.

Incorrect. Correct answer is B


45% answered correctly

Explanation:

Correct Answer Is B

This patient has an open fracture/dislocation of the ankle joint with vascular compromise. Open fractures/dislocations are characterized by the bone and/ or joint exposure to
external environment, or when the fracture or dislocation is caused by blunt or penetrating forces sufficient to disrupt or penetrate the skin, subcutaneous tissue, muscle fascia,
muscle, and/ or the bone or joint. Open fractures are often contaminated by foreign material (e.g., clothing, grass, dirt, gravel), dead or devitalized tissue and bacteria.

Always follow the following rules in open fractures / dislocations

Remove any gross contamination off the wound. No provisional irrigation or debridement is performed at this stage.

Take photographs of the wound and dress it with wet sterile cover (the photographs are taken, so that other treating physicians do not need to uncover the wound to see
it.)

Give patient analgesic (preferably intravenously to both control the pain and prepare for a reduction in the emergency department). Morphine is a good option.

By gentle traction, reduce the fracture and correct misalignments as much as possible.

Start the patient on intravenous prophylactic antibiotics.

Give the patient tetanus prophylaxis if indicated.

Obtain X-rays.

Urgently arrange for transferring the patient to the operating room for surgical wound debridement and definite treatment of the fracture and dislocation.

The question, however, asks about the most important step in preventing infections and consequent complications. Debridement of all devitalized soft tissues and bone all the way
down to the bone is the most crucial step in preventing the infection and the consequent adverse outcomes such as delayed healing, amputation, etc.

Prophylactic antibiotics are started prior to debridement and definite treatment, and are of great importance; however, debridement remains the cardinal step in preventing
infections. Tetanus prophylaxis should be given to the patient if indicated but this does not prevent other infections if adequate debridement is not carried performed.

Wound debridement is always crucial but should be performed in the operating room after the above measures have been undertaken; therefore, it is not a priority but an extremely
important step in preventing infections.

1131 of 1943
A 57-year-old construction worker man presents to your practice with the hand deformity illustrated in the accompanying photograph. He works with a drilling machine and
has tingling sensation in his hand after work. He drinks 2 bottles of beers every day and even more on weekends. Which one of the following could be the most likey cause of his
problem?

A. Alcohol consumption.

B. An autoimmune process.

C. Vibration injury.

D. Direct trauma.

E. Injury to the ulnar nerve.

Correct
45% answered correctly

Explanation:

Correct Answer Is A

The picture shows puckering of palmar skin and a nodule at the base of fourth (ring) finger which are highly suggestive of Dupuytren’s contracture (DC).

DC is caused by fibrous hyperplasia of palmar fascia leading to nodular formation and contracture over the fourth and fifth fingers commonly. This condition occurs in 10% of males
over the age of 65 years particularly in those of North European descent.

There is a genetic predisposition for the disease and the condition is assumed to have an autosomal dominant inheritance, but for clinical presentation to develop other factors are
hypothesized to be involved.

An association between DC and some conditions have been well-established. These conditions are as follows:

Male gender
Alcohol Excess
Smoking
Liver cirrhosis
COPD
Diabetes mellitus
Heavy manual labour

Of the above factors, the most common risk factors in descending order are:

Age
Alcohol excess
Sex (male)
Previous hand injuries

In this man’s case, excess alcohol drinking can be the most likely contributing factor to his condition. Each bottle of beer contains 1.5 - 2 units of alcohol depending on the size and
the alcohol concentration.

The pathophysiology of DC is not well-understood. Proliferation of fibrocytes and inflammatory processes are speculated to be the most likely pathophysiology. The disease is not
autoimmune. No nerve is involved in DC.

References

• PubMed - Dupuytren's contracture, alcohol consumption, and chronic liver disease

• RCAGP - A man with contracted fingers

Last updated:
Time spent: QID:507
2023-2-12

1132 of 1943
While playing basketball, a 23-old-man landed on his left foot awkwardly and injured his ankle. He is in the emergency department now. On examination, lateral dislocation of the
ankle is evident. Dorsal pedis pulse is absent and the foot looks pale and is cold to touch. Which one of the following would be the next best step in management?

A. Reduce the dislocation in the emergency department.

B. Send the patient to the operating room for reduction under general anesthesia.

C. Immobilize the joint with plaster.

D. Send the patient to the operating room for open reduction and internal fixation.

E. Angiography.

Correct
45% answered correctly

Explanation:

Correct Answer Is A

Early reduction/alignment of a dislocation/fracture is always the most important step in management. This is even more crucial when a dislocation or fracture has caused
neurovascular compromise.

In the field (prehospital), if there is evidence of neurovascular compromise such as a cold, discolored, and pulseless or insensate foot, reduction of the dislocation or alignment of
the fracture should be tried after the patient is given analgesia with intravenous opiates (morphine). If morphine is not available, intravenous benzodiazepines can be used instead.
Reduction and alignment is achieved by gentle in-line traction. Alignment should be maintained en route to the hospital.

In the emergency department, if the reduction/alignment has not been performed in the field, reduction should be attempted as soon as possible even before radiological
examination if there is vascular compromise.

By definition, dislocation of the ankle is always considered unstable due to accompanying disruption of the lateral or medial ligaments or the tibiofibular syndesmosis. Ankle
dislocation requires immediate orthopedic or podiatric consultation for surgical intervention such as internal or external fixation of any accompanying fracture and repair of capsular
or ligamentous tears.

None of the other options take precedence over early reduction of the joint.

References

• Medscape - Ankle Dislocation in Emergency Medicine Treatment & Management

Last updated:
Time spent: QID:516
2023-2-12

1133 of 1943
Which one of the following is a recognized feature of polymyalgia rheumatica?

A. Weakness of the distal muscle groups.

B. Elevated serum creatine phosphokinase (CPK).

C. An association with bronchial carcinoma.

D. Weight loss.

E. A peak incidence in the fourth decade of life.

Incorrect. Correct answer is D


45% answered correctly

Explanation:

Correct Answer Is D

Polymyalgia rheumatica (PMR) is a relatively common inflammatory disease in elderly people (almost never seen before <50) and a common indication for long-term steroid use.
The incidence increases with advancing age. It is more common in women.

PMR has many nonspecific features and a wide range of differential diagnoses. No gold standard diagnostic test is available and the diagnosis is mainly made clinically. About 15-
20% of patients with PMR develop giant cell arteritis (GCA) and 40-50% of those with GCA are found to have associated PMR. Despite the similarities in the age of onset and some
clinical manifestations, the relationship between PMR and GCA is not yet clearly established; however, these two disorders are thought to represent different manifestations of a
shared disease process.

The symptoms of polymyalgia rheumatica (PMR) include pain and stiffness of the shoulder and hip girdle. The neck may be involved in some patients. The stiffness can be so
severe that the patient may have great difficulty getting off the chair, turning over in bed, or raising the arms above shoulder height. Stiffness after periods of rest (gel phenomenon)
as well as morning stiffness of more than 30 minutes typically occurs.

Muscle weakness is not a feature of PMR. However, this can be difficult to assess in the setting of pain, especially if symptoms are protracted and untreated, resulting in disuse
atrophy. Some patients report the following systemic features:

Low-grade fever (high, spiking fevers are rare and should prompt evaluation for underlying infection, malignancy, or vasculitis)
Weight loss
Malaise and/or fatigue
Depression
Anorexia

Of the options, weight loss can be a manifestation of PMR; however, not all patients with PMR have weight loss or other systemic symptoms.

(Option A) Weakness of distal muscle groups is not a feature of PMR.

(Option B) Creatine phosphokinase elevation is associated with inflammatory processes of muscles or muscle breakdown, none of which occurs in PMR.

(Option C) There is no association between PMR and lung cancer including small cell carcinoma of the lung.

(Option E) PMR/GCA is almost never seen before the age of 50. The average age of onset is 70 years.

TOPIC REVIEW

Several diagnostic criteria for PMR exist. One set of diagnostic criteria is as follows:

Age of onset 50 years or older


Erythrocyte sedimentation rate ≥40 mm/h
Pain persisting for ≥1 month and involving 2 of the following areas: neck, shoulders, and pelvic girdle
Absence of other diseases capable of causing the same musculoskeletal symptoms
Morning stiffness lasting ≥30 minutes
Rapid response to prednisone (≤20 mg)

References

• Medscape - Polymyalgia Rheumatica

Last updated:
Time spent: QID:554
2023-2-12

1134 of 1943
Michael is a 66-year-old patient of yours, who has osteoarthritis of the left knee. Six months ago and after non-pharmacological measures and paracetamol on an as-needed basis
failed to relieve the pain, you started him on regular full-dose paracetamol. Today he is at your practice with the complaint that he is still suffering from severe knee pain despite
taking paracetamol. He is diabetic and has ischemic heart disease and peptic ulcer disease. Which one of the following is the most appropriate option to add to paracetamol at this
stage?

A. Celecoxib.

B. Tramadol.

C. Codeine.

D. Slow-release morphine preparations.

E. Diclofenac.

Incorrect. Correct answer is C


45% answered correctly

Explanation:

Correct Answer Is C

For mild to moderate osteoarthritic pain without evidence of inflammation, pharmacological treatment starts with paracetamol. Regular full-dose of paracetamol (up to 4gr/day)
even in the absence of pain is superior to taking the medication on an as-needed basis.

If the clinical response to paracetamol is not satisfactory, or if the clinical presentation of osteoarthritis is inflammatory, NSAIDs should be considered with the lowest effective dose
or intermittent dosing if symptoms are intermittent. NSAIDs are associated with an increased risk of gastrointestinal problems such as dyspepsia and peptic ulcer disease and are
contraindicated in patients with active peptic ulcer disease. Selective COX-2 inhibitors are preferred in these patients. The addition of prophylactic proton pump inhibitors or
misoprostol is an alternative.

On the other hand, selective COX-2 inhibitors are associated with an increased risk of cerebrovascular or cardiovascular diseases and should be avoided in patients with a history of
these conditions.

This patient has peptic ulcer disease and cardiovascular disease; therefore, NSAIDs and COX-2 inhibitors are inappropriate options for him.

Opiates and tramadol are drugs of choice for the treatment of patients with moderate to severe osteoarthritis, for whom NSAIDs are contraindicated or ineffective in controlling
pain.

In this patient with a contraindication to both non-selective NSAIDs (e.g. ibuprofen, diclofenac, naproxen) and celecoxib, the addition of a weak opiate such as codeine will be the
next best step in management.

(Option A) Celecoxib is not an appropriate option for this man because he has cardiovascular disease.

(Option B) Although tramadol is effective in the management of OA pain, either as a single therapy or added to paracetamol is associated with more adverse effects and is not
considered an option at this stage. Studies showed that tramadol has more adverse effects than diclofenac.

(Option D) Strong opiates (e.g. oxycodone, morphine) are associated with more adverse effects, but about the same efficacy as weak opiates for the management of OA.

(Option E) With peptic ulcer disease, diclofenac is not an appropriate option to consider for this man.

TOPIC REVIEW

Pharmacological management of osteoarthritis of the knee

Paracetamol – paracetamol (or other simple analgesics) are first-line pharmacological therapy for pain control. Paracetamol is safe and can be used up to 1mg every 4 hours.
Patients should be advised to use paracetamol on a regular basis (even in pain-free periods) rather than as-needed because the latter often fails to adequately control the pain.

Non-steroidal anti-inflammatory drugs (NSAIDs) – addition of NSAIDs is the next best step in management if:

Optimal therapy with paracetamol fails to control the pain


There is evidence of inflammation, such as pain that is worse with rest, nocturnal pain, and gelling of the joint (stiffness with rest).
COX-2 inhibitors have all the adverse effects of NSAIDs, although they seem to be less irritating to the stomach lining; therefore, better choices if there is a risk of GI
bleeding

Opiate analgesics and tramadol

Opiate analgesics are used if:

NSAIDs are contraindicated


NSAIDs fail to control the symptoms

Opiates have a modest effect in managing moderate to severe OA pain in patients for whom paracetamol is ineffective, and who do not respond to, or have contraindications for
NSAIDs. However, most of the research on opioid use has been in short-term trials and long-term efficacy has not been shown.

1135 of 1943
Intra-articular treatments

Corticosteroids – a single injection can provide rapid relief for up to 4 weeks. The patient should be advised to rest 24 hours after the injection. This therapy can be used if the
patient has an important occasion to participate in or wants to travel. Repeated injections are not as effective.

Hyaluronan – hyaluronan is given weekly for 3 to 5 weeks. Effect sizes are small. They provide slower onset but longer-lasting (up to 12 weeks) pain relief than corticosteroids.

There is conflicting evidence of the benefit of glucosamine in the treatment of the symptoms of knee OA. There is insufficient evidence to support the benefit of preventing the
progression of cartilage loss.

PRACTICE POINTS

Regular use of full-dose paracetamol (500-1000 mg/4-6 hours, up to 4 mg /day) even when there is no pain, is superior to its use on an as-needed basis.

Whenever NSAIDs are considered, they should be given with the minimum effective dose for 2-4 weeks and close monitoring of blood pressure, renal function, and
gastrointestinal symptoms.

​ SAIDs are contraindicated in patients with active peptic ulcer disease. In those at risk of or with a history of peptic ulcer disease, either the selective COX-2 inhibitor –
N
celecoxib should be used or non-selective NSAIDs be given with prophylactic PPIs.

Celecoxib must be avoided in patients with a history of cardiovascular diseases such as ischemic heart disease, cerebrovascular disease, etc.

References

• RACGP - Musculoskeletal Health

• Medscape - Osteoarthritis

• UpToDate - Initial pharmacological therapy of osteoarthritis

• Therapeutic Guidelines - Rheumatologyhttp://tg.org.au

Last updated:
Time spent: QID:555
2023-2-12

1136 of 1943
A 67-year-old woman presents to your clinic with complaints of painful hips and shoulders and headache. She relates that muscle pain is felt in both arms and hips. She feels her
shoulders are stiff and her hips cannot move when she wakes up. It almost takes 40 minutes of painful physical activity for the stiffness to relent. Her appetite is decreased and she
has lost 5 kg. “To make me even more miserable, the terrible headaches have just been started”, she mentions. The headache is felt on the right side and is throbbing with no
response to analgesics. On physical examination, she is noted to have bilateral painful restricted movements of shoulder and hip joints. Her right temple is tender to touch. She is
not febrile and the rest of the exam is inconclusive. Which one of the following would be the most appropriate next step in management?

A. Check serum creatine kinase (CK) level.

B. Check the rheumatoid factor.

C. Check ESR.

D. Perform a muscle biopsy.

E. Star the patient on prednisolone.

Incorrect. Correct answer is C


45% answered correctly

Explanation:

Correct Answer Is C

The clinical picture is highly suggestive of two closely-associated clinical syndromes, polymyalgia rheumatica and giant cell (temporal) arteritis (GCA). The etiology of these two
syndromes is unknown. Either of these syndromes can develop solitarily, although in 15-20% of cases both are present. If so, polymyalgia rheumatica invariably precedes temporal
arteritis.

The patients are usually older (the 60s or 70s). These conditions are very rare among those younger than 50 years. Women are more commonly affected.

Polymyalgia rheumatica presents with pain and stiffness of the shoulders and hips symmetrically. The cervical spine may be involved. There is marked stiffness usually early in the
morning lasting over 30 minutes. The symptoms tend to subside or at least relieve by the end of the day. Constitutional symptoms such as malaise, low-grade fever, anorexia, and
weight loss may be present.

Giant cell arteritis presents with headaches often unresponsive or poorly responsive to analgesics, scalp tenderness, jaw claudication, and weak or absent pulsation of the temporal
artery. Not all patients with CGA present with headaches.

When these two conditions are suspected, the next best step in management would be measuring the erythrocyte sedimentation rate (ESR). With an elevated ESR (>40), it is prudent
that corticosteroids be started immediately to prevent permanent visual loss. With visual problems at presentation, intravenous corticosteroids (methylprednisolone) replace the
oral route for the first 3 days of therapy.

(Options A and D) Creatine kinase levels or muscle biopsy would come first if myositis was suspected.

(Option B) Serologic tests, such as rheumatoid factor (RF), antinuclear antibodies (ANA), and cyclic citrullinated peptide (CCP) antibodies are typically negative.

(Option E) Corticosteroids (e.g. prednisolone) are indicated if ESR is >40mm/h.

References

• Medscape - Polymyalgia Rheumatica

• Therapeutic Guidelines - Rheumatology

Last updated:
Time spent: QID:556
2023-2-12

1137 of 1943
Which one of the following cannot be associated with HLA- B27?

A. Acute anterior uveitis.

B. Ankylosing spondylitis.

C. Reactive arthritis.

D. Behçet disease.

E. Psoriatic arthritis.

Incorrect. Correct answer is D


45% answered correctly

Explanation:

Correct Answer Is D

HLA-B27 is found in almost 90% of patients with seronegative spondyloarthropathies. Seronegative spondyloarthropathies include:

Ankylosing spondylitis (AS)


Reactive arthritis
Psoriatic arthritis
Inflammatory bowel disease

Anterior uveitis is another possible finding in seronegative spondyloarthropathies; hence, from another standpoint, it could be said that those with anterior uveitis may suffer from a
seronegative spondyloarthropathy and consequently be HLA-B27 positive.

Behçet disease is a diffuse vasculitis of unknown origin. The disease is characterized by a triple-symptom complex of:

1. Recurrent oral aphthous ulcers


2. Recurrent genital ulcers
3. Uveitis

Behçet disease has no association with HLA-B27.

References

• Medscape - HLA-B27 Syndromes

Last updated:
Time spent: QID:557
2023-2-12

1138 of 1943
A 67-year-old man presents to your office with complaints of reduced dexterity of his left hand and difficulty grasping objects. His hand is shown in the following photograph. Which
one of the following investigations will confirm the diagnosis?

A. X-ray of the hand.

B. MRI.

C. Ultrasonography of the hand.

D. CT scan of the hand.

E. Blood sugar level.

Incorrect. Correct answer is C


45% answered correctly

Explanation:

Correct Answer Is C

The picture shows palmar skin puckering and a nodule at the base of the fourth finger, indicative of Dupuytren contracture. The diagnosis is almost always made clinically and no
imaging is needed. However, ultrasonography can demonstrate thickening of the palmar fascia and the presence of a nodule. At cases, ultrasonography of the hand is considered if
the patient is planned for injection of corticosteroids directly into the cord and avoiding injecting into the flexor tendon.

Since there is an association between Dupuytren contracture and diabetes mellitus, blood sugar level can be measured if the physician is suspicious of diabetes based on history
and/or clinical findings.

X-ray, CT scan or MRI is almost never indicated.

TOPIC REVIEW

Treatment options for Dupuytren contracture:

Physiotherapy (for mild cases): e.g. heat and ultrasonographic waves, physical therapy, custom splint or brace for finger stretching
Occupational therapy
Intralesional corticosteroid injection
Collagenase injection
Other measures: 5-fluorouracil, imiquimod, botulinum toxin, and hyperbaric oxygen
Surgery: surgery is indicated if MCP contracture is more than 30° or there is contracture of PIP.

References

• Medscape - Dupuytren Contracture

Last updated:
Time spent: QID:707
2023-2-12

1139 of 1943
A 32-year-old man presents to your office with complaint of low back pain in the setting of ankylosing spondylitis (AS), which he has had for the past 5 years. He has been on
treatment with naproxen 500 mg, 3 times a day, for the past few years with no significant improvement. An X-ray of the lumbosacral area is obtained and is shown in the following
photograph. Which one of the following is the most appropriate management option for this patient?

A. Start him on sulfasalazine.

B. Start him on methotrexate.

C. Start him on infliximab.

D. Start him on cyclosporine.

E. Double the dose of naproxen.

Incorrect. Correct answer is C


45% answered correctly

Explanation:

Correct Answer Is C

Non-steroidal anti-inflammatory drugs (NSAIDs) are first-line therapy for symptomatic patients with AS. Recent studies suggest that regular NSAID use in AS slows radiographic
progression more than on-demand use. An individualized assessment of the risk of long-term NSAID use should be made in consultation with the rheumatologist before long-term
daily NSAID use is recommended.

Tumour Necrotizing Factor (TNF) inhibitors such as infliximab are used when there is not adequate response to at least two different NSAIDs. TNF inhibitor therapy has strikingly
improved the quality of life in over two-thirds of the patients. These agents are listed on the Pharmaceutical Benefits Scheme (PBS) for active AS not adequately responsive to
exercise and NSAIDs, but not for non-radiographic axial AS, meaning that these drugs are not indicated where the X-ray changes of sacroiliitis are absent. Randomized controlled
trials, however, suggest better clinical responses in early non-radiographic disease.

Traditional disease-modifying anti-rheumatic drugs (DMARDs) such as methotrexate and sulfasalazine have little effect on spinal disease, but can be useful for an associated
peripheral arthritis. Other biologic agents available for rheumatoid arthritis in Australia have not been effective in treatment of axial AS.

This patient has not adequately responded to maximum dose of naproxen (an NSAID). On the other hand, his lumbosacral X-ray shows the classic bamboo spine (squaring and
fusion of lumbosacral vertebrae) seen in AS; therefore, he will be an appropriate candidate to be started on infliximab as the most appropriate next step in management.

(Options A and B) Sulfasalazine and methotrexate are not indicated where there is axial involvement.

(Option D) Cyclosporine has not shown effective in management of axial AS.

(Option E) Doubling the dose of naproxen will exceed the maximum dose of this drug and is associated with significant adverse effects and probably no improved outcome.

NOTE - If there was an option suggesting switching to another NSAID, it would be the correct answer because some patients unresponsvie to one NSAID may improve on another.

References

• http://www.racgp.org.au/afp/2013/november/ankylosi

• Therapeutic Guideline – Rheumatology; available from: http://tg.org.au

Last updated:
Time spent: QID:824
2023-2-12

1140 of 1943
A 68-year-old woman presents to your practice complaining of pain in her right hip. The pain started 2 days ago and has escalated since. She refuses to bear weight on the affected
side, as this brings on severe pain. The patient denies any history of osteoarthritis or other joint diseases, but admits to long-term use of oral corticosteroids for Behçet disease. On
examination, all movements of the right hip joint are diminished and very painful. Which one of the following would be the most likely diagnosis?

A. Gout.

B. Avascular necrosis of the femoral head.

C. Osteoarthritis of the hip joint.

D. Septic arthritis.

E. Bony metastasis from a primary cancer.

Correct
45% answered correctly

Explanation:

Correct Answer Is B

There are many conditions that can cause hip or groin pain in adults. The following 2 are the most common causes in adults:

A flare of a chronic inflammatory condition such as rheumatoid arthritis


Degenerative changes of the hip joint, such as in osteoarthritis

Other causes, especially in the elderly may include:

Fracture of the femoral head or neck


Degenerative spondylosis of lumbosacral spine (neurogenic claudication)
Arterial occlusions (vascular claudication)
Polymyalgia rheumatica
Trochanteric bursitis
Bony metastases from a primary cancer
Atraumatic avascular necrosis of the femoral head

Atraumatic avascular necrosis of the femoral neck, although not a very common condition, should always be suspected in an elderly patient, particularly if the following risk factors
are present in the history:

Long-term steroid use


Chronic alcohol use and resulting liver disease
Sickle cell disease
Past hip fracture

Of the given options, considering the history of long-term steroid use, avascular necrosis of the femoral head seems more likely.

The initial step in diagnosis would be an X-ray of the hip. Typical finding is variable density of the femoral head.

Sonography can be used to exclude joint effusion due to an acute inflammation such as in a flare up of rheumatoid arthritis or septic arthritis. The most accurate diagnostic tool
would be an MRI with characteristic ‘geographic’ subchondral lesions. It is often considered if the diagnosis remains equivocal.

(Option A) Gout can cause acute inflammation and pain of the joint, but the hip joint is unlikely to be affected by this disease.

(Option C) Sudden onset of the hip pain, without any previous history, is a very unlikely to be caused by osteoarthritis (OA) of the hip. The history of OA often includes insidious onset
and progression to more severe states over a long period of time.

(Option D) Septic arthritis is a serious condition that can lead to complete joint destruction and permanent disability within hours if untreated, and needs to be excluded; however, it
is not the most common diagnosis in this case scenario. The patient with septic arthritis is expected to be more systematically ill, but the range of motions of the affected hip joint
would be painfully limited, just such as this patient.

(Option E) Bony metastases from a primary cancer does not seem to be a likely diagnosis, as a cancer so advanced to metastasize to distant sites, is expected to have presented
with sign and symptoms of the primary tumor earlier. The examination is mentioned to be unremarkable otherwise.

References

• http://www.australiandoctor.com.au/cmspages/getfil

• Murtagh’s General Practice - McGraw Hill- 5th edition – page 687

Last updated:
Time spent: QID:108
2023-2-12

1141 of 1943
A 35-year-old man presents to your GP clinic with severe back pain for several years. The pain is worse on waking up in the morning. Pain and stiffness make him stay in bed for at
least 30 minutes. The pain ameliorates by walking and physical activity throughout the day. His condition has deteriorated recently despite being on the maximum dose of naproxen.
An X-ray of his lumbosacral spine is shown in the accompanying photograph. Which one of the following would be the most appropriate next step in management?

A. Sulfasalazine.

B. Methotrexate.

C. Adalimumab.

D. Aspirin.

E. Prednisolone.

Incorrect. Correct answer is C


45% answered correctly

Explanation:

Correct Answer Is C

The X-ray shows severe deformities of the lumbosacral spine evident by the fusion of vertebrae resulting in the characteristic bamboo spine, a hallmark radiographic feature in
ankylosing spondylitis (AS).

Non-steroidal anti-inflammatory drugs (NSAIDs) are first-line therapy for symptomatic patients with AS; however, when the disease is rapidly progressive and/or there is not
adequate response despite the maximum dose of NSAIDs, biological disease-modifying anti-rheumatic drugs (bDMARDs) are next step in management.

Of these drugs, tumor Necrotizing Factor (TNF) inhibitors have strikingly improved the quality of life in more than two-thirds of the patients. These agents are indicated for active AS
not adequately responsive to exercise and NSAIDs with radiographic axial AS (as in this case), but not for non-radiographic axial AS, meaning that these drugs are not indicated
where the X-ray changes typical for AS are absent. Randomized controlled trials, however, suggest better clinical responses in early non-radiographic disease. Infliximab, etanercept,
adalimumab, and golimumab are TNF inhibitors approved for the treatment of AS when indicated.

With inadequate response to NSAIDs and characteristic X-ray findings, this patient is eligible for treatment with adalimumab as the next best step in management.

Traditional DMARDs such as methotrexate, azathioprine, and sulfasalazine have little effect on spinal disease, but can be useful for associated peripheral arthritis. Other biologic
agents available for rheumatoid arthritis in Australia have not been effective for the treatment of AS.

(Options A and B) Sulfasalazine and methotrexate are not effective treatments when there is axial involvement.

(Option D) If among the options, a trial of a second NSAID would be the correct answer because one of the criteria required for bDMARD to be covered by the pharmaceutical
benefits scheme (PBS) is the failure of at least two NSAIDs before the commencement of bDMARDs. Aspirin is a salicylate, not an NSAID.

(Option E) Systemic steroids may be used for extra-spinal presentations of AS such as uveitis, inflammatory bowel disease (IBD), etc. but are not indicated nor effective for the
treatment of spinal involvement.

References

1142 of 1943
• RACGP - AFP - Ankylosing Spondylitis

• Therapeutic Guideline – Rheumatology


Last updated:
Time spent: QID:825
2023-2-12

1143 of 1943
A 51-year-old man presents to your practice for back pain. He was diagnosed with ankylosing spondylitis 20 years ago and has had mild back pain all this time. He had received no
medical treatment and has been on an exercise program on an on-and-off basis. The patient suffers from peptic ulcer disease and has been treated for the condition several times.
Which one of the following is the most appropriate next step in management?

A. Paracetamol.

B. Naproxen.

C. Methotrexate.

D. Sulfasalazine.

E. Infliximab.

Correct
45% answered correctly

Explanation:

Correct Answer Is A

First-line management of ankylosing spondylitis (AS) is NSAIDs. Regular use of NSAIDs has been shown to reduce the symptoms as well as the rate at which skeletal changes
occur. This patient has been diagnosed with AS 20 years ago and has not received any medication. On the other hand, mild low back pain is the patient's current complaint. Without
gastric problems, NSAIDs (naproxen) was the drug of choice to consider first, but with peptic ulcer disease and because the pain is mild, simple analgesics such as paracetamol
should be tried first in an attempt to alleviate the patient’s pain without significantly increasing the risk of his peptic ulcer disease deteriorate.

NOTE - In general, simple analgesics and opioids are considered where there is residual pain despite NSAID use or when NSAIDs are contraindicated.

(Option B) Despite the increased risk, NSAIDs may be considered at the lowest doses possible, once simple analgesics proved ineffective in pain control. COX2-selective NSAIDs
such as celecoxib are preferred over non-selective NSAIDs if the patient does not have cardiovascular or cerebrovascular diseases.

(Options C and D) Methotrexate and sulfasalazine may occasionally be considered for the treatment of extra-spinal manifestation of AS.

(Option E) Infliximab is a tumor necrotizing factor (TNF) inhibitor approved to use in patients with axial involvement with inadequate response to a maximum dose of NSAIDs or in
those with severely progressive disease.

References

• UpToDate - Assessment and treatment of ankylosing spondylitis in adults

Last updated:
Time spent: QID:826
2023-2-12

1144 of 1943
An 80-year-old woman sustains a fall in the bathroom and is brought to the emergency department by her daughter. She has severe pain and decreased range of motion in all
directions in the right hip. The right leg appears shorter than the left and is externally rotated. A bedside portable X-ray of the pelvis is taken which is shown in the following
photograph. Which one of the following is the most appropriate management option for this patient?

A. Intramedullay nail placement.

B. External fixation.

C. Hip arthroplasty.

D. Analgesics and bed rest.

E. Traction.

Incorrect. Correct answer is C


45% answered correctly

Explanation:

Correct Answer Is C

The X-ray shows an intracapsular (Garden) fracture of the right femoral neck. The fracture can be subcapital, transcervical, or basicervical. Only such fractures can be considered
genuine femoral neck fractures.

Garden classification of intracapsular (subcapital) fractures of the femoral neck is as follows:

Garden type I: incomplete fracture with valgus impaction


Garden type II: Complete fracture without displacement
Garden type III: Complete fracture with partial displacement of the fracture fragments
Garden type IV: Complete fracture with total displacement allowing the femoral head to rotate back to an anatomic position

In reality the distinction between classes is difficult; however, complete fracture and displacement of the femoral head makes Garden type III (or IV) femoral neck fracture the most
likely diagnosis.

Garden types I and II femoral neck fractures are surgically stabilized with closed reduction and internal fixation. Garden types III and IV are controversial in the type of implant used
for treatment. In younger patients, closed or open reduction is recommended. In less active older patients, however, prosthetic replacement is recommended.

None of other options are superior to hip arthroplasty.

1145 of 1943
www.radiologymasterclass.com

References

• http://www.radiologymasterclass.co.uk/tutorials/mu

• http://emedicine.medscape.com/article/87043-treatm

Last updated:
Time spent: QID:957
2023-2-12

1146 of 1943
Examination of an aspirate from an acutely painful swollen knee is as follows:

White blood cell count: 4100 (<2000μL)


Red blood cells: few
Crystals: negative
Culture: no growth

Which of the following is the most accurate interpretation of the results?

A. Traumatic tap.

B. Inflammation.

C. TB arthritis.

D. Viral arthritis.

E. Gonococcal arthritis.

Incorrect. Correct answer is B


45% answered correctly

Explanation:

Correct Answer Is B

Biochemically, the synovial fluid is an ultrafiltrate of plasma across the synovial membrane enriched with various compounds produced by the synoviocytes. In normal physiological conditions, the biochemical composition of
the synovial fluid is similar to that of plasma. In pathological conditions, laboratory evaluation of synovial fluid provides information about the pathology of the disease affecting the joint (e.g., arthritis).

Similar to other body fluids, common laboratory evaluation of synovial fluid includes the following three steps:

Physical examination of synovial fluid (e.g., appearance, including color, viscosity, and other physical characteristics)
Chemical analysis (e.g., glucose, total protein level, uric acid)
Microscopic evaluation (e.g., cell count and differential count, crystal identification)

Based on physical and laboratory findings, synovial fluid can be categorized as:

Normal
Non-inflammatory
Inflammatory
Septic
Hemorrhagic

The following table summarizes the characteristic feature of synovial fluid in different conditions:

PMN cell count


Color Transparency Viscosity Cell count (/mm3) Gram stain Culture Crystals
(%)
Normal Clear Transparent High-thick <200 <25% Negative Negative Negative
Non-inflammatory Straw Translucent High-thick 200-2000 <25% Negative Negative Negative
Inflammatory – crystalline
Yellow Cloudy Low-thin 2000-100000 >50% Negative Negative Negative
disease
Inflammatory – non-crystalline
Yellow Cloudy Low-thin 2000-100000 >50% Negative Negative Positive
disease
Infectious (Lyme) Yellow Cloudy Low 3000-100000 (mean 25,000) >50% Negative Negative Negative
Variable Negative
Infectious (gonococcal) Yellow Cloudy-opaque Low 34000-68000 >75% Positive (25-75%)
(<50%)
Negative
>50,000 (>100,000 more
Infections (non-gonococcal) Yellow-green Cloudy Very low >75% Positive (60-80%) Positive (>90%)
specific)

Although a cell count of 4800 in the aspirate analysis could be seen in both inflammatory and infectious conditions, a very high cell count of >50,000 or even >100,000 is more specific for infectious causes (read the Exam Tip
below). Furthermore, a negative culture does not exclude infection but makes it less likely.

In practice, a cell count of 2000-50,000 is the typical finding in inflammatory joint diseases such as rheumatoid arthritis, lupus arthritis, gout, and pseudogout. The absence of crystals, however, makes crystal-induced
inflammation unlikely.

(Option A) Although a traumatic tap results in blood in the synovial fluid, other features such as elevated white cell count are absent unless a concomitant condition is present. A few red blood cells (RBCs) are usually present
even in a normal synovial fluid.

(Option C) In TB arthritis, synovial fluid analysis is usually not helpful. The white cell count can be high or low with the dominance of either neutrophils or lymphocytes and no other specific diagnostic features. In fact, this
aspirate analysis can also be caused by TB as well, yet an inflammatory condition is more likely.

(Option D) Viral arthritis presents with a high white cell count with a predominance of polymorphonuclears (PMNs), but the cultures are negative.

(Option E) A white cell count of 4100, makes gonococcal arthritis less likely. Gonococcal arthritis often presents with a white cell count of 34000-68000.

EXAM TIP

Remember the following tips for the exam:

The WBC count of a normal joint aspirate is <200.


With degenerative diseases (e.g., OA) or trauma, the WBC count of joint fluid is 200-2000.
Typical inflammatory joint fluid either with the crystalline or non-crystalline disease has a WBC of 2000-50,000 (more commonly 5,000-50,000)
The typical WBC count of joint fluid in septic arthritis is usually >50,000; however, septic arthritis should be considered a possibility in every joint aspirate with WBC>5000, especially if the arthritis is monoarticular and
the patient has already received antibiotics.

It is not common for gout and pseudogout to present with WBC>50,000. If so, co-infection is a possibility to consider.

References

• Lab tests Online -

1147 of 1943
Last updated:
Time spent: QID:979
2023-2-12

1148 of 1943
A 37-year-old man is brought to the Emergency Department after he sustained a motor vehicle accident. Upon arrival, a quick review excludes any life-threatening internal injuries.
He has a deformed right leg highly indicative of a fracture. There is a bleeding laceration over the deformity. An X-ray shows fractures of the tibial and fibula. As the initial
management, intravenous fluid and antibiotics are started. Which one of the following is the most appropriate next step in management?

A. External fixation.

B. Internal fixation.

C. Tetanus prophylaxis.​

D. Wound debridment.

E. Wound closure.

Incorrect. Correct answer is C


45% answered correctly

Explanation:

Correct Answer Is C

The vignette describes a typical case of open fracture of the right leg. Open fractures are characterized by the fracture exposure to the environment trough a breach of the skin and
underlying soft tissue. Such fractures are particularly at increased risk of infection that can compromise the fracture and healing process.

Always follow the following rules in open fractures / dislocations

Remove any gross contamination off the wound. No provisional irrigation or debridement is performed at this stage.

Take photographs of the wound and dress it with wet sterile cover (the photographs are taken, so that other treating physicians do not need to uncover the wound to see
it.)

Give patient analgesic (preferably intravenously to both control the pain and prepare for a reduction in the emergency department). Morphine is a good option.

By gentle traction, reduce the fracture and correct misalignments as much as possible.

Start the patient on intravenous prophylactic antibiotics.

Give the patient tetanus prophylaxis if indicated.

Obtain X-rays.

Urgently arrange for transferring the patient to the operating room for surgical wound debridement and definite treatment of the fracture and dislocation.

There is no comment regarding significant misalignment. The patient’s wound should be covered by wet dressing after photographs are taken (if possible). He also has been started
on intravenous antibiotics; therefore, the next best step for him would tetanus prophylaxis if indicated.

(Options A and B) Fixation of fractures is performed in the operating room after surgical irrigation and wound debridement.

(Option D) Wound debridement including surgical removal of all devitalized soft tissues and bone is a crucial step in management of any open fracture/dislocation because
infections may ensue and result in poor outcomes. However, debridement should be performed in the sterile environment of the operating room, not in the non-sterile environment
of the Emergency Department where more exploration of the wound can lead to increased risk of infection.

(Option E) Wound closure can be performed (if not contraindicated such as in highly contaminated wounds or delayed presentation) after vigorous irrigation and debridement of the
wound and fixation of the fracture.

References

• Medscape - Open Fractures

Last updated:
Time spent: QID:1099
2023-2-12

1149 of 1943
You are visiting a 75-year-old woman in your practice, who is complaining of bilateral pain and stiffness in her shoulders and hips. She explains that the pain and stiffness are worst
on waking, but becomes better after 40 to 60 minutes of physical activity. On examination, there is no remarkable finding except generalized stiffness of her hip and shoulder
girdles. Which of the following investigations is most consistent with the diagnosis?

A. Elevated serum calcium.

B. Elevated erythrocyte sedimentation rate (ESR).

C. Positive rheumatoid factor (RF).

D. Increased serum alkaline phosphatase level.

E. Joint space narrowing and decreased bone density on X-rays of the shoulders and hips.

Incorrect. Correct answer is B


45% answered correctly

Explanation:

Correct Answer Is B

Pain and morning stiffness of hip and shoulder girdles taking 40-60 minutes of activity to relieve is consistent with polymyalgia rheumatic as the most likely diagnosis.

Polymyalgia rheumatica is an inflammatory rheumatic condition characterized clinically by aching and morning stiffness in the shoulders, hip girdle, and neck. The condition can be
associated with giant cell (temporal) arteritis (GCA). The 2 disorders are thought to represent different manifestations of a shared disease process.

Polymyalgia rheumatica is almost exclusively a disease of adults over the age of 50, and 2- to 3-times more common in women than men. The prevalence increases with age and
the average age at diagnosis is over 70 years.

Clinical manifestations of polymyalgia rheumatica include:

The subacute or chronic onset of aching and morning stiffness in the shoulders, hip girdles, neck, and torso (usually symmetric)

Morning stiffness that typically lasts more than 30 minutes

Pain – shoulder pain is more common at presentation (70-95%) than hip and neck (50-70%). The pain is worse with movement

Decreased range of motion

Synovitis, tenosynovitis, and bursitis - ~50% of patients can exhibit distal musculoskeletal involvement; some patients may have swelling and pitting edema of the hands,
wrists, ankles, and top of the feet. Tenosynovitis can also cause carpal tunnel syndrome, which occurs in approximately 10 to 15% of patients with polymyalgia rheumatica

Subjective weakness - Muscle strength is usually normal, but weakness is frequently reported by patients and may be a problem diagnostically due to diminished effort on
examination because of pain, or clinically because of disuse atrophy. Thus, careful testing is required if weakness is reported.

Systemic manifestations - Up to 40% of patients may have malaise, fatigue, depression, anorexia, weight loss, and fever

NOTE – Muscle tenderness is not a prominent feature despite what the name implies. If present, tenderness is more likely to be due to synovial or bursal inflammation than
muscle involvement.

The most characteristic finding in polymyalgia rheumatica (and giant cell arteritis) is an elevated erythrocyte sedimentation rate (ESR) that is above 40mm/h. According to some
studies, an elevated ESR can be seen in 78-93%. ESR may exceed 100mm/h in some patients.

(Option A) Serum calcium is unaffected in polymyalgia rheumatica.

(Option C) Serologic tests, such as rheumatoid factor (RF), antinuclear antibodies (ANA), and cyclic citrullinated peptide (CCP) antibodies are typically negative.

(Option D) Some patients may have some rise in their liver enzymes, especially alkaline phosphatase; however, this is more common in patients with giant cell arteritis than
polymyalgia rheumatica alone.

(Option E) Joints are not affected in polymyalgia rheumatica. No X-ray abnormalities are expected.

References

• UpToDate- Clinical Manifestations and diagnosis of polymyalgia rheumatica

• UpToDate - Clinical manifestations of giant cell temporal arteritis

Last updated:
Time spent: QID:1138
2023-2-12

1150 of 1943
A 74-year-old woman presents to the emergency department with sudden-onset left visual loss that occurred within a few hours. She also mentions having left-sided headaches for
the past few days and hip and shoulder pain and stiffness for 7 months. She has tried over-the-counter painkillers for her headache with no response. Which one of the following is
the most appropriate next step in management?

A. Muscle biopsy.

B. CT scan of the head.

C. MRI of the head.

D. Erythrocyte sedimentation rate (ESR).

E. Ocular massage.

Incorrect. Correct answer is D


45% answered correctly

Explanation:

Correct Answer Is D

Sudden-onset visual loss and headache in a patient over the age of 50 is very likely to be giant cell (temporal) arteritis until proven otherwise. Additionally, this patient has had hip
and shoulder pain, suggestive of polymyalgia rheumatica as a strong association.

Giant cell arteritis (GCA) and polymyalgia rheumatica are closely associated syndromes with unknown etiology. GCA and polymyalgia rheumatica can develop alone, although in
20% of cases both are present (as in this case)

Polymyalgia rheumatica presents with pain and stiffness of shoulders and/or hips often symmetrically. Neck pain and/or stiffness may be present. Shoulder pain is the presenting
symptom in 70-95% of patients. Hip and/or shoulder pain is present in 50-70% of patients. Tenosynovitis and bursitis may be present in 50% of patients. A decreased range of
motion can be found on examination. A subjective sense of muscle weakness may be reported by the patient despite the fact that muscles are not involved. Constitutional
symptoms such as fever, weight loss, malaise, fatigue, and anorexia are present in 40% of patients.

GCA presents with headaches often unresponsive or poorly responsive to analgesics, scalp tenderness, jaw claudication, and weak or absent pulsation of the temporal artery. A new
headache occurs in at least 2/3 of patients. The headache may have a variety of presentations in GCA, but being new-onset is the critical feature. In its classic form, the headache of
GCA is located in temporal regions, but it may also occur in frontal or occipital areas or may be generalized. Headache is the chief complaint in most patients, but in some, it may be
absent or elicited by direct questioning.

Systemic symptoms of GCA include fever, fatigue, and weight loss. Up to 50% of patients have a fever that is often low-grade but can exceed 39°C in 15% of patients which can be
mistaken for infection. Importantly, in approximately 10% of patients, constitutional symptoms are the only manifestations of GCA.

NOTE – in patients with polymyalgia rheumatica, ongoing monitoring for symptoms or physical findings suggestive of GCA is recommended. Evaluation including biopsy should be
performed if symptoms of GCA develop, even if patients are on corticosteroids.

Once GCA/polymyalgia rheumatica is suspected on clinical grounds, ESR is the most appropriate next step in management. An elevated ESR above 40mm/h is the most
characteristic finding in GCA/polymyalgia rheumatica. With an elevated ESR, immediate treatment with systemic corticosteroids should be started and a temporal artery biopsy for a
definite diagnosis arranged.

(Option A) Although some patients may complain of muscle pain and tenderness, muscles are not involved in GCA/polymyalgia rheumatica. If present, tenderness is more likely to
be due to synovial or bursal inflammation than muscle involvement. Muscle biopsy has no role in the diagnosis of these conditions.

(Options B and C) Imaging studies such as CT scans or MRIs are not routinely used for diagnosis of GCA/polymyalgia rheumatica unless the diagnosis is equivocal and other
diagnoses are considered.

(Option E) ocular massage is used initially for urgent management of retinal artery occlusion that not the case here.

References

• UpToDate - Clinical manifestations and diagnosis of polymyalgia rheumatica

• UpToDate - Clinical manifestations of giant cell temporal arteritis

Last updated:
Time spent: QID:1139
2023-2-12

1151 of 1943
A 76-year-old woman presents with painful and stiff shoulders that she has had for the past 2 months. The stiffness is worse in the morning and improves after 30-60 minutes of
physical activity. She denies headaches, visual problems, or symptoms such as fever or weight loss. Initial laboratory tests results are:

RBC: 5x106 cells/mm3 (4.5-6.5x106)


Hb:115g/L (130-180)
MCV: 85 fL (76-96)
WCC: 9000 cells/m m3 (4000-11000)
Platelet: 500,000/mm3 (150,000-400,000)
Metabolic panel: normal
ESR: 65mm/h
CRP: Positive
RF: negative
ANA: negative

Which of the following would you consider for treating this patient?

A. Paracetamol.

B. Prednisolone.

C. Antibiotics.

D. Non-steroidal anti-inflammatory drugs (NSAIDs).

E. Physical therapy.

Incorrect. Correct answer is B


45% answered correctly

Explanation:

Correct Answer Is B

The painful stiff shoulder and/or hip girdle in a person over the age of 50 years is mostly consistent with polymyalgia rheumatica as the most likely diagnosis.

Polymyalgia rheumatica is an inflammatory rheumatic condition characterized clinically by aching and morning stiffness in the shoulders, hip girdle, and neck. The condition can be
associated with giant cell (temporal) arteritis (GCA). The two disorders are thought to represent different manifestations of a shared disease process.

Polymyalgia rheumatica is almost exclusively a disease of adults over the age of 50, and 2- to 3-times more common in women than men. The prevalence increases with age and
the average age at diagnosis is over 70 years.

Polymyalgia rheumatica presents with pain and stiffness of shoulders and/or hips often symmetrically. Neck pain and/or stiffness may be present. Shoulder pain is the presenting
symptom in 70-95% of patients. Hip and/or neck pain is present in 50-70% of patients. Tenosynovitis and bursitis may be present in 50% of patients. A decreased range of motion
can be found on examination. A subjective sense of muscle weakness may be reported by the patient despite the fact that muscles are not involved. Constitutional symptoms such
as fever, weight loss, malaise, fatigue, and anorexia are present in 40% of patients.

The most characteristic finding in polymyalgia rheumatica (and giant cell arteritis) is an elevated erythrocyte sedimentation rate (ESR) that is above 40mm/h. According to some
studies, an elevated ESR (and CRP) can be seen in 78-93% .ESR may exceed 100mm/h in some patients.

Normocytic normochromic anemia is seen in some patients. WCC is normal, but platelet count can be raised, as platelets are acute phase reactants and can be elevated in
inflammatory conditions. Serologic tests, such as rheumatoid factor (RF), antinuclear antibodies (ANA), and cyclic citrullinated peptide (CCP) antibodies are typically negative.

Some may have an increase in their liver enzymes, especially alkaline phosphatase; however, this is more common in patients with giant cell arteritis than polymyalgia rheumatica
alone.

Corticosteroids are recommended as the initial therapy in patients with polymyalgia rheumatica. The primary objective of treatment is improvement in symptoms. Therapy is mostly
symptomatic and has not been proven to improve prognosis or prevent progression to giant cell arteritis. Most patients experience dramatic improvement in symptoms within days
of therapy.

Prednisone/prednisolone 0-20mg, daily in the morning, is the recommended regimen. Therapy is often required for 2 to 3 years. The minimum duration of therapy is unlikely to be
less than 12 months. Long-term use of corticosteroid use in such patients is associated with corticosteroid-induced bone loss and osteoporosis. Prophylactic calcium/vitamin D
supplementation is recommended and bisphosphonates (e.g. alendronate) may be indicated for prevention.

In patients with CGA, daily doses of 40-60mg are required. The addition of aspirin is recommended. For patients with visual symptoms, who present early, a 3-day course of
intravenous prednisolone, followed by oral therapy is the management of choice.

NOTE – Response to treatment is monitored with ESR and CRP.

(Option A) Polymyalgia rheumatica is an inflammatory condition. Paracetamol is unlikely to be of considerable benefit in patients with polymyalgia rheumatica.

(Option C) Antibiotics have no role in the management of polymyalgia rheumatica and/or GCA.

(Option D) NSAIDs have been tried as corticosteroid-sparing alternatives, but can be associated with significant adverse effects. NSAIDs are occasionally used in patients on very
low doses of corticosteroids, who also require analgesic or anti-inflammatory therapy for other disorders, such as osteoarthritis or rotator cuff tendonitis.

1152 of 1943
(Option E) Physical therapy is recommended as adjunctive therapy in patients who have difficulty regaining good mobility and a full range of motion despite control of the
inflammatory process.

References

• UpToDate - Treatment of polymyalgia rheumatica

• Medscape - Polymyalgia Rheumatica

• Therapeutic Guidelines – Rheumatology

Last updated:
Time spent: QID:1140
2023-2-12

1153 of 1943
A 73-year-old woman presents to your practice with a complaint of shoulder pain for the past 8 months. The pain is mostly felt in both shoulders and is aching and vague in nature.
She also has shoulder girdle stiffness in the morning, often taking 30-60 minutes of physical activity and sometimes a hot bath to improve. She denies any fever, weight loss,
headache, or visual symptoms. Past medical history is significant for osteoporosis, for which she is on alendronate and supplemental calcium and vitamin D. Laboratory tests show
normocytic, normochromic anemia and an erythrocyte sedimentation rate (ESR) of 100mm/h. Which one of the following is the most appropriate next step in management?

A. Corticosteroids.

B. Non-steroidal anti-inflammatory drugs (NSAIDs).

C. Paracetamol.

D. Methotrexate.

E. Infliximab.

Correct
45% answered correctly

Explanation:

Correct Answer Is A

The clinical picture is highly suggestive of polymyalgia rheumatica an inflammatory disease of uncertain etiology that can be associated with giant cell arteritis in some patients. If
both conditions are present polymyalgia rheumatica invariably precedes GCA.

As in this patient, polymyalgia rheumatica presents with pain and stiffness of the shoulders and hips in a symmetrical fashion. The cervical spine may be involved. There is marked
stiffness usually early in the morning. The symptoms tend to subside or at least relieve by the end of the day. Constitutional symptoms such as malaise, low-grade fever, anorexia,
and weight loss may be present in 40% of patients.

Corticosteroids are recommended as the initial therapy in patients with polymyalgia rheumatica. The primary objective of treatment is the improvement in symptoms.

Prednisone/prednisolone 10-20mg, daily in the morning is the recommended regimen. Therapy is often required for 2 to 3 years. The minimum duration of therapy is unlikely to be
less than 12 months. Long-term use of corticosteroid use in such patients is associated with corticosteroid-induced bone loss and osteoporosis. In this patient with pre-existing
osteoporosis, long-term corticosteroid therapy may result in the deterioration of bone loss; nonetheless, there is no alternative effective therapy for myalgia rheumatica to replace
corticosteroids.

This patient should also be started on preventive measures to decrease the adverse effects of corticosteroids on bone, if the duration of therapy is predicted to exceed one month:

Weight-bearing exercises where possible


Adequate calcium intake
Measurement and addition of vitamin D if indicated
Monitoring bone density from the outset of therapy
Bisphosphonates if indicated

(Option B) NSAIDs have been tried as corticosteroid-sparing alternatives, but can be associated with significant adverse effects. NSAIDs are occasionally used in patients on very
low doses of corticosteroids who also require analgesic or anti-inflammatory therapy for other disorders, such as osteoarthritis or rotator cuff tendonitis. NSAIDs alone are unlikely
to be useful for the treatment of polymyalgia rheumatica.

(Option C) Polymyalgia rheumatica is an inflammatory condition. Paracetamol is unlikely to be of considerable benefit in patients with polymyalgia rheumatica.

(Option D) Methotrexate alone is not shown to be effective in managing polymyalgia rheumatica Addition of methotrexate to corticosteroids may be used to reduce the dose of
corticosteroids in patients who are at increased risk of corticosteroid-induced adverse effects such as osteopenia/osteoporosis. This has been suggested by some but not all
studies. In general, methotrexate is rarely used in patients with polymyalgia rheumatica without GCA, because low doses of corticosteroids are often sufficient to control the
disease.

(Option E) The use of TNF inhibitors, such as infliximab and etanercept, is effective in the treatment of polymyalgia rheumatica as an alternative to corticosteroids.

References

• UpToDate - Treatment of polymyalgia rheumatica

• Medscape - Polymyalgia Rheumatica

• Therapeutic Guidelines – Rheumatology

Last updated:
Time spent: QID:1141
2023-2-12

1154 of 1943
A 70-year-old man presents with complaints of headache and blurry vision in his right eye. He also mentions a history of shoulder pain that has been bilateral and associated with
morning stiffness of at least 30 minutes for the past few months. Which one of the following investigations is most important to establish the diagnosis?

A. Erythrocyte sedimentation rate (ESR).

B. ANA.

C. Temporal artery biopsy.

D. CT scan of the head.

E. Evoked visual potential. ​

Incorrect. Correct answer is C


45% answered correctly

Explanation:

Correct Answer Is C

The clinical picture of headache and visual impairment in a patient over the age of 50 is highly suggestive of giant cell arteritis. The preceding history of shoulder pain and stiffness
indicates polymyalgia rheumatica and makes the diagnosis more certain.

When CGA is suspected the initial step is measuring the erythrocyte sedimentation rate (ESR). An elevated ESR is the most characteristic finding in patients with GCA/polymyalgia
rheumatica. An ESR level of less than 40mm/h makes CGA/polymyalgia rheumatica an unlikely diagnosis while measurings above 40mm/h prompt treatment with corticosteroids to
prevent permanent visual loss. Arrangements then should be made to perform a temporal artery biopsy. Superficial temporal artery biopsy is the criterion standard for diagnosis
of CGA and should be obtained almost without exception in patients with suspected GCA on clinical grounds. It is essential because the treatment course for GCA is long and often
complicated, and many of the nonspecific symptoms of GCA such as headache, body aches, and fatigue may occur in many other diseases. A positive temporal artery biopsy is
100% specific but has a relatively low sensitivity of 15-40% for the diagnosis of GCA.

NOTE - Patients who only have the symptoms of polymyalgia rheumatica do not need a biopsy. However, such patients should be carefully questioned about headache, jaw or
arm claudication, visual symptoms, and any unusual pain in the face, throat, or tongue. Any of these symptoms suggest the possibility of GCA.

(Option B) ESR is performed to determine if prompt treatment with corticosteroids should be started. It is not diagnostic for CGA.

(Option C) Serologic tests, such as rheumatoid factor (RF), antinuclear antibodies (ANA), and cyclic citrullinated peptide (CCP) antibodies are typically negative, and not used for
diagnosis of CGA.

(Option D) Imaging studies such as CT scans are not routinely used to diagnose GCA unless another diagnosis is considered.

(Option E) Evoked visual potentials test is used for the diagnosis of optic neuritis and is not indicated here.

References

• UpToDate - Diagnosis of giant cell arteritis

• Medscape - Giant Cell Arteritis (Temporal Arteritis)

Last updated:
Time spent: QID:1142
2023-2-12

1155 of 1943
Mr. Johnson comes to your practice, complaining of progressively increasing left knee pain. He is 67 years old and otherwise healthy. He elaborates that the pain is brought on by
walking. Generally, he feels well in the morning except for the stiffness of the left knee which often resolves within minutes of physical activity. When he starts walking, the pain is
brought on and builds up to a maximum intensity throughout the day. On physical examination, everything is normal except the crepitations of the left knee. The knee is not tender or
warm to the touch and there is no swelling either. You order an X-ray of the left knee. Which one of the following would be the least likely finding on the X-ray?

A. Joint space narrowing.

B. Subchondral cysts.

C. Periarticular osteoporosis.

D. Osteophytes.

E. Increased density of bone ends.

Incorrect. Correct answer is C


45% answered correctly

Explanation:

Correct Answer Is C

This patient's clinical presentation fits the diagnosis of osteoarthritis of the left knee. Osteoarthritis is the most common arthropathy worldwide.

The diagnosis of OA is mainly clinical. An X-ray of the affected joint is necessary to confirm the diagnosis. The characteristic findings on X-ray of an osteoarthritic joint are:

Joint space narrowing


Presence of subchondral cysts
Sclerosis of subchondral bone
Osteophytes at the outer edges of the joint lines

The pathophysiology of the disease includes chronic destruction of bone cartilage. With the diminished protective function of the cartilage, the periarticular bone undergoes
repetitive trauma leading to bone destruction, which is followed by abnormal bone formation in form of bony spurs also called osteophytes. Osteophyte formation plays a major
role in the pathophysiology of pain in OA.

The healing process is affected by bony areas leads to sclerosis of the bony edges of the joint, signified by hyperdense sclerotic subchondral bones. Joint destruction and
degeneration lead to joint space narrowing. Subchondral cyst formation would be another finding.

Osteopenia or osteoporosis around the joint is seen in inflammatory conditions such as rheumatoid arthritis or neuropathic arthropathy.

Neuropathic arthropathy (Charcot's joint) may be seen in the following conditions:

Diabetes mellitus neuropathy – is the most common cause in Australia and worldwide. It is related to long-term poor blood glucose control.
Alcohol neuropathy
Cerebral palsy
Leprosy
Syphilis (tabes dorsalis)
Spinal cord injury
Myelomeningocele
Syringomyelia
Intra-articular steroid injection
Congenital insensitivity to pain
Peroneal muscular atrophy

The classic findings on an X-ray of a joint with neuropathic arthropathy would be:

Juxta-articular osteopenia
Joint space narrowing

Osteopenia/osteoporosis around the joint (juxta-articular) is unlikely to be seen in OA.

References

• Radiopaedia - Osteoarthritis

• Medscape - Osteoarthritis

Last updated:
Time spent: QID:124
2023-2-12

1156 of 1943
A 17-year-old boy falls on his outstretched right arm, with subsequent shoulder pain and limited movement of his right shoulder, especially when attempting to elevate his arm over
the head. His appearance is illustrated in the following photograph. Which one of the following is the most likely diagnosis?

A. Fracture of the surgical neck of humerus.

B. Rupture of the rotator cuff tendon.

C. Dislocated glenohumeral joint.

D. Rupture of the long head of biceps tendon.

E. Dislocated acromioclavicular joint.

Incorrect. Correct answer is C


45% answered correctly

Explanation:

Correct Answer Is C

The flattened shoulder, as well as the way the patient is holding the affected arm to limit shoulder for preventing pain is typical for anterior glenohumeral dislocation.

Shoulder dislocations account for 50% of all major joint dislocations. In over 90% of cases, such dislocation occurs anteriorly. Posterior, inferior, or superior dislocations account for
the remaining 10%.

An anterior shoulder dislocation is usually caused by a blow to the abducted, externally rotated, and extended arm (e.g. blocking a basketball shot). Less commonly, a blow to the
posterior humerus or a fall on an outstretched arm may cause an anterior dislocation.

An anteriorly dislocated shoulder causes the arm to be slightly abducted and externally rotated. The patient resists all movement. There is loss of the normal round appearance of
the shoulder resulting in a flat shoulder as seen in the photograph.

(Option A) Fracture of the surgical neck of the humerus does not cause a flat shoulder. Pain and tenderness are almost always present. Bruising, swelling, or deformity may occur.

(Option B) Rotator cuff ruptures does not cause the shoulder to become flat. Pain and varying inability to abduct the arm are characteristic features.

(Option D) Rupture of the long head of the biceps tendon does not cause a flattened shoulder. Resisted flexion can elicit pain in the antecubital fossa. On forearm flexion, the bulk of
the contracted biceps is lower than the unaffected arm.

(Option E) Acromioclavicular (AC) joint dislocation causes pain, tenderness and the classic step deformity in the AC joint. Arm raise is limited due to exquisite pain when attempted.

References

• http://www.acc.co.nz/PRD_EXT_CSMP/groups/external_

• http://www.uptodate.com/contents/shoulder-dislocat

• http://www.racgp.org.au/afp/2012/april/shoulder-in

Last updated:
Time spent: QID:1157
2023-2-12

1157 of 1943
A young man presents to the emergency department after injuring his shoulder in a fall. His X-ray is shown in the following photograph. Testing the motor function by resisted
isometric contraction of which one of the following muscles would be most likely to confirm the presence of an associated nerve injury?

A. Latissimus dorsi.

B. Pectoralis major.

C. Trapezius.

D. Serratus anterior.

E. Deltoid.

Incorrect. Correct answer is E


45% answered correctly

Explanation:

Correct Answer Is E

Significant findings on this X-ray of the shoudler are a humeral head out of the glenoid fossa and displaced inferiorly. These findings are typical for anterior dislocation of the
glenohumeral joint.

Shoulder dislocations often result in injury of the axillary nerve. Axillary nerve runs inferiorly to the humeral head and wraps around the surgical neck of the humerus. Axillary nerve
innervates the deltoid and terese minor muscles and the skin of the lateral shoulder (shoulder badge region). Axillary nerve dysfunction manifests as loss of sensation in a 'shoulder
badge' distribution and deltoid muscle weakness. It is impractical to ask the patient to move the affected arm due to pain; however, in the presence of axillary nerve injury, the patient
is often unable to isometrically contracts the deltoid muscle.

No other muscle mentioned in the options are innervated by the axillary nerve; hence, unaffected by axially nerve injury.

References

• http://www.uptodate.com/contents/shoulder-dislocat

Last updated:
Time spent: QID:1158
2023-2-12

1158 of 1943
A patient with anterior dislocation of the shoulder is unable to perform which one of the following actions?

A. Turning the door knob.

B. Combing hair.

C. Taking a knife from the table.

D. Shaking hand.

E. Wrist extension.

Incorrect. Correct answer is B


45% answered correctly

Explanation:

Correct Answer Is B

Shoulder dislocation often result in injury of the axillary nerve. Axillary nerve runs inferiorly to the humeral head and wraps around the surgical neck of the humerus. Axillary nerve
innervates the deltoid and terese minor muscles and the skin of the lateral shoulder (shoulder badge region) Axillary nerve dysfunction manifests as loss of sensation in a 'shoulder
badge' distribution and deltoid muscle weakness. Even in the absence of such injury, a patient with anterior shoulder dislocation cannot raise the arm; hence not able to comb hair.

Patients with anterior shoulder dislocation can use their elbow, wrists and fingers, so they can grasp and turn the door knob, pick and hold objects with their hand, shake hand or
extend the wrist.

References

• http://www.uptodate.com/contents/shoulder-dislocat

Last updated:
Time spent: QID:1159
2023-2-12

1159 of 1943
Which one of the following are the most appropriate medications to give prior to reduction of an anterior dislocation of the shoulder?

A. IV diazepam and morphine.

B. IV midazolam.

C. Intranasal fentanyl and IV midazolam.

D. IV midazolam and morphine.

E. IM diazepam and nasal fentanyl.

Incorrect. Correct answer is C


45% answered correctly

Explanation:

Correct Answer Is C

Reduction of anterior shoulder dislocation is performed in the emergency department after adequate muscle relaxation and sedation is provided. Intravenous midazolam plus
intranasal fentanyl (an opiate) is the most commonly used drug combination.

Ketamine, etomidate and propofol are other appropriate alternatives.

Use of parenteral morphine is not justified while intranasal fentanyl can provide the same pain relief. Fentanyl is easier to administer. Diazepam is a long-acting benzodiazepine.
While a short duration of muscle relaxation is required for joint reduction, use of midazolam in preference of diazepam is more appropriate. Furthermore, IM diazepam has an
delayed and unpredictable effect.

References

• http://www.uptodate.com/contents/shoulder-dislocat

Last updated:
Time spent: QID:1160
2023-2-12

1160 of 1943
After a fall on an outstretched hand, a 32-year-old woman develops pain and tenderness of her wrist. An X-ray is obtained that is shown in the following photograph. Which one of
the following is the most common complication of this fracture?

A. Joint stiffness.

B. Nonunion.

C. Misalignment.

D. Avascular necrosis.

E. Pain.

Incorrect. Correct answer is D


45% answered correctly

Explanation:

Correct Answer Is D

The X-ray shows a scaphoid fracture of the left wrist. Scaphoid fracture is common and often results from fall on an outstretched hand. Tenderness of anatomical snuff box and
pain elicited by applying axial pressure to the thumb of the affected hand are typical clinical findings.

Avascular necrosis is the most common complication of scaphoid fracture seen in 13-40% of patients. Nonunion is the second most common complication occurring in 5-12%.

Joint stiffness and pain are other possible complications but are not as common.

Misalignment is not a complication of scaphoid fracture because the fracture is either non-displaced or displaced. Displaced fractures are treated with open reduction and internal
fixation. During the procedure the displacement is corrected and fixed.

References

• http://emedicine.medscape.com/article/328658-follo

• Rosen’s Emergency Medicine – 8th Edition – Lange

Last updated:
Time spent: QID:1161
2023-2-12

1161 of 1943
A 34-year-old woman trips over and falls on her outstretched right hand. Immediately after the fall she fells severe pain in her right wrist. On examination in the emergency
department, there is tenderness over the lateral aspect of the head of the radius and the anatomical snuffbox. Application of axial pressure to the thumb of the affected hand
causes pain. An X-ray is ordered which is completely normal. Which one of the following is the next best step in management?

A. Reassurance.

B. Place the wrist in thumb spica cast and repeat the X-ray in 7-10 days.

C. Repeat the X-ray in 6 weeks.

D. Analgesics.

E. Place the wrist in a sugar tongue splint.

Incorrect. Correct answer is B


45% answered correctly

Explanation:

Correct Answer Is B

The mechanism of injury as well as the exam findings is suggestive of scaphoid fracture. Scaphoid fracture often results from falling on an outstretched hand. Tenderness of
anatomical snuff box and pain elicited by applying axial pressure to the thumb of the affected hand are typical clinical findings.

Once scaphoid fracture is suspected, plain X-ray films are the most appropriate next step in management. If fracture is evident on X-ray, definite treatment with casting or surgery to
fix a displaced fracture follows.

Some patients, however, will have normal X-rays, despite high index of clinical suspicion. In such cases, further management depends on the availability of advanced imaging
studies such as MRI or CT scan. MRI is the preferred modality, as it is radiation-free and can assess both the scaphoid and the surrounding soft tissue. CT scan can detect fractures
missed on plain radiographs but it cannot visualize possible soft tissue injuries.

If advanced imaging is not available, the wrist should be immobilized by a thumb spica cast for 7-10 days (up to 14 days) and X-ray repeated after this period. By 7-10 days, the
fracture line should be visualized on plain films. A negative control X-ray film in the presence of symptoms warrants referral for MRI.

(Option A) Scaphoid fracture may not be evident on initial X-rays, but seen on follow-up films. A negative X-ray in a symptomatic patient does not exclude fracture; so reassurance is
not appropriate.

(Options C and D) When symptoms suggest scaphoid fracture but X-rays are clear, the wrist should be immobilized and X-ray repeated in 7-10 days. Analgesic may be prescribed for
pain control, but giving only analgesics without wrist immobilization is incorrect.

(Option E) Sugar tongue splints are insufficient for immobilization of the wrist after scaphoid fracture. A thumb spica cast stabilizes the wrist in a neuter position with an slight ulnar
deviation.

References

• http://www.imagingpathways.health.wa.gov.au/index.

Last updated:
Time spent: QID:1162
2023-2-12

1162 of 1943
A 76-year-old lady presents to the emergency department after she sustained a fall on her outstretched hand. The X-ray obtained is shown in the following photograph. Which one of
the following should be the position of the the wrist in cast?

A. Full flexion of the wrist with radial deviation.

B. Partial flexion of the wrist with radial deviation.

C. Full flexion of the wrist with ulnar deviation.

D. Partial extension of the wrist with radial deviation.

E. Partial flexion of the wrist with ulnar deviation.

Incorrect. Correct answer is E


45% answered correctly

Explanation:

Correct Answer Is E

The X-ray shows fracture of the distal radius, also known as Colles fracture. Falling on an outstretched (dorsiflexed) hand is the most common mechanism of this fracture. Colles
fracture is very common in elderly women, and can occur with minimal trauma if there is osteoporosis.

There can be posterior dislocation of the head of the radius, giving rise to the 'fork deformity'. Lateral displacement may also occur. Casts are often used for treatment of Colles
fractures. Severely displaced fractures may be treated with external or even internal fixation.

For casting, the wrist should be in 10° flexion and 10° ulnar deviation, and the forearm must be in slight pronation. Often a below-elbow cast for 4-6 week is sufficient; however, the
cast may be extend to include above the elbow if there is significant displacement. Every significant displacement should be reduced under anesthesia before casting.

References

• Murtagh’s General Practice – McGraw Hill – 6th Edition – pages 1468-1469

Last updated:
Time spent: QID:1163
2023-2-12

1163 of 1943
A 72-year-old woman develops pain and deformity of the left wrist after she falls on her outstretched left hand. The x-ray is as shown in the following photograph. Which one of the
following is the characteristics of the cast that should be used for treatment?

A. Should include the wrist but not the thumb.

B. Should include the wrist and thumb.

C. Should include the wrist, thumb and elbow.

D. Should include the wrist and elbow.

E. Should include the thumb, wrist, elbow and the upper arm.

Correct
45% answered correctly

Explanation:

Correct Answer Is A

The X-ray shows fracture of the distal 3cm of the radius, namely Colles fracture. Casting after meticulous reduction of any displacement remains the most commonly used
treatment option for Colles fracture. The wrist is flexed at 10° and deviated to the ulnar at 10°. The cast includes the wrist and the forearm but not the thumb and the elbow.
However, if there is severe displacement and casting is considered for treatment, the cast may be extended to include up to above the elbow is selected patients.

References

• Murtagh’s General Practice – McGraw Hill – 6th Edition – pages 146-1469

Last updated:
Time spent: QID:1165
2023-2-12

1164 of 1943
Which one of the following could be the earliest complication of the fracture shown in the following photograph?

A. Malunion.

B. Nonunion.

C. Ischemic Volkmann contracture.

D. Wrist drop.

E. Rupture of the extensor pollicis longus tendon.

Incorrect. Correct answer is C


45% answered correctly

Explanation:

Correct Answer Is C

The photograph shows fracture of the distal 3cm of the radius, referred to as Colles fracture. Potentially, Colles fracture can be associated with complications including:

Rupture of extensor pollicis longus tendon


Malunion and/or nonunion
Stiffness of the elbow, metacarpophalangeal joints (MCPs), interphalangeal joints (IPs)
Discomfort at distal radiolunalr joint
Regional pain syndrome
Volkmann contracture

Of these, Volkmann contracture develops earliest.

Volkmann ischemic contracture is the permanent shortening of forearm muscles, usually resulting from compartment syndrome in the forearm. It causes a claw-like deformity of
the wrist, hand, and fingers. The clinical presentation of Volkmann contracture includes five Ps: pain, pallor, pulselessness, paresthesia, and paralysis. Of these, pain is the earliest
sign.

On physical examination, pain accentuated by passive stretching seems to be the most reliable finding. Firmness of the tissues often is noted on palpation. Pulselessness and
paralysis are late findings. Induration of the forearm is another useful diagnostic finding.

NOTE - Any fracture of the humerus, elbow, forearm and wrist can result in compartment syndrome and Volkmann contracture.

Malunion (option A), non-union (option B) and rupture of the extensor pollicis longus tendon (option E) do not develop as early as Volkmann contracture. Rupture of extensor pollicis
longus tendon often occurs 4-8 weeks after the fracture. For wrist drop (option D), radial nerve injuries should happen at a more proximal level. Colles fracture does not cause wrist
drop.

References

• PubMed - NON-OSSEOUS COMPLICATIONS FOLLOWING DISTAL RADIUS FRACTURE

• Medscape - Volkmann Contracture

Last updated:
Time spent: QID:1166
2023-2-12

1165 of 1943
Which one of the following is the most common complication of Colles fracture?

A. Malunion.

B. Nonunion.

C. Ischemic Volkmann contracture.

D. Osteoarthritis of the wrist.

E. Wrist joint stiffness.

Correct
45% answered correctly

Explanation:

Correct Answer Is A

Colles’ fracture is defined as fracture of the distal 3cm of the radius.

The following are possible complications of Colles’ fracture:

Rupture of extensor pollicis longus tendon


Malunion and/or nonunion
Stiffness of the elbow, metacarpophalangeal joints (MCPs), interphalangeal joints (IPs)
Discomfort at distal radiolunalr joint
Regional pain syndrome
Volkmann contracture

Post-reduction redisplacement is the most common complication of Colles’ fracture that can result in malunion; therefore, the most likely complication among other options.

Nonunion (option B), wrist joint stiffness (option E), osteoarthritis of the wrist (option D), regional pain syndrome, median nerve injury and inferior radioulnar joint laxity are other
less common complications associated with Colles’ fracture. Volkmann’s contracture (option C) is a grave complication if compartment syndrome occurs in forearm fractures
(including distal radius fracture) but it is not as common.

References

• BMJ - Some complications of Colles' fracture and their treatment

Last updated:
Time spent: QID:1167
2023-2-12

1166 of 1943
Which one of the hip joint movements is initially affected by osteoarthritis?

A. External rotation.

B. Internal rotation.

C. Flexion.

D. Adduction.

E. Abduction.

Incorrect. Correct answer is B


45% answered correctly

Explanation:

Correct Answer Is B

In osteoarthritis of the hip joint (and other hip joint pathologies), the first movement restricted is internal rotation of the hip followed by hip flexion.

References

• PubMed - Hip Pain and Mobility Deficits – Hip Osteoarthritis

Last updated:
Time spent: QID:125
2023-2-12

1167 of 1943
A 48-year-old man presents to the Emergency Department with complaint of sudden onset pain and tenderness that developed after he lifted an armchair at home for redecoration.
One examination, restricted and painful movements of the lumbar area including flexion, extension and rotation, as well as tenderness over the L4-L5 region is noted. Which one of
the following is the most appropriate step in initial management of this patient?

A. Arrange for an MRI scan of the lumbosacral spine.

B. Prescribe lumbar corset.

C. Prescribe analgesics and advise that he continue his daily activities.

D. Prescribe analgesics and advise bed rest.

E. Prescribe pelvic traction.

Incorrect. Correct answer is C


45% answered correctly

Explanation:

Correct Answer Is C

This scenario represents a case of acute low back pain. Low back pain is a very common condition faced in general practice. It is estimated that up to 80% of adults have low back
pain at some times in their lives. Despite the fact that there is a long list of potential causes for low back pain, most patients seen in primary care (>85%) will have nonspecific low
back pain meaning that there is back pain in the absence of a specific underlying condition that can be reliably identified. Only less than 1% are found to have a serious underlying
disease such as cancer or infection.

In the middle of this spectrum, there are other less serious but specific etiologies, the most important of which are vertebral compression fractures, radiculopathies, and spinal canal
stenosis. Other conditions include osteoarthritis of the lumbosacral spinal column, scoliosis and hyperkyphosis, psychologic stress and extraspinal causes such as pancreatitis,
nephrolithiasis, pyelonephritis, abdominal aortic aneurysms and herpes zoster.

In approaching patients with low back pain, the first step in management is making a judicious decision as to which patient can be managed conservatively versus those who may
need more investigations and probably referral for specialist care. As always, a thorough history and physical examination is the most important step in approaching patients with
low back pain. The very first thing to consider in history and during physical exam is to assess the presence of alarming features or risk factors that may point toward a serious
underlying condition.

Such features and risk factors are known as ‘red flags’ and are summarized in the following table:

Red flags for low-back-pain


Feature/risk factor Condition to consider
History of cancer with new onset back pain

Unexplained weight loss

Failure to improve after one month

Age >50 years or <20 years


Cancer

Nocturnal pain

Fever

Intravenous drug use

Infection
Recent infection

Immunocompromised state

Rest pain

Urinary retention

Fecal incontinence

Saddle anesthesia
Cauda equina syndrome

Lower limb weakness or paresthesia

1168 of 1943
History of osteoporosis

Use of corticosteroid Vertebral fracture

Older age (>50 years)

History of fall or other trauma

Morning stiffness

Improvement with exercise

Alternating buttock pain

Ankylosing spondylitis
Waking up because of pain during the second part of the night

Younger age

Red flags should be evaluated in the context of the clinical presentation as a whole, not as the sole guide for management because over 80% of patients with non-specific low back
pain have a red flag in their histories irrelevant to their back pain. High index of suspicion should be kept when there is more than one red flag present.

Physical examination of patients with low back pain includes inspection for any deformities in the spinal column, hip and lower limb such as scoliosis, kyphosis, skin changes, signs
of trauma like bruises or abrasions, and hip and lower limb deformities or discrepancies. Checking the range of motion of lumbar or lumbosacral spine should be performed in all
patients if possible, as they might give clues about the extent of disability and, at occasions, the probable source of the pain (e.g., a restricted flexion in disc problems versus a
limited extension in facet-joint disorders).

Palpation of the spine and paraspinal areas is another part of the physical exam. It is important to note that local tenderness in the lumbar area is a common finding in non-specific
low back pain. This tenderness is caused mostly by the spasm of the paravertebral muscles and is not an alerting sign (red flag). For example, a tear in a facet joint or a prolapsed or
degenerated disc can result in muscle spasm leading to tenderness over the underlying area. However, bone tenderness directly over lumbar spinous processes should be
considered as an alarming sign pointing toward serious bone disorders such as neoplasm or infection.

Another important point to consider is that as long as the ow back pain remains above the sacroiliac joint, a neurological exam is not required.

As a rule, all patients with low back pain without a red flag should be initially managed conservatively with:

1. Education and explanations


2. Simple analgesics such as paracetamol or NSAIDs for pain control
3. Encouraging maintenance of daily life and remaining physically active
4. Recommending and referral for physiotherapy for those with debilitating pain and/or severe restriction of movements
5. Review and reassessment (recommended in one and 4-6 weeks)

It is very important to note that NO further investigation including imaging studies such as plain radiographs, CT scan or MRI are indicated in patients with low back pain in the
absence of red flags. Such studies should be considered if a red flag is present or the symptoms persist despite effective conservative management (as above) for at least 4 weeks.
This holds true for patients with radicular pain (e.g. sciatica); however, any patient with symptoms of cauda equina, spinal cord compression, or progressive and/or severe
neurologic deficit should have immediate MRI for evaluation as well as urgent specialist referral. Symptoms indicating these conditions include: new bowel or bladder incontinence,
urinary retention, saddle anesthesia and significant motor deficit not localized to a single unilateral nerve root. This means that, for example, a patient with low back pain, unilateral
foot drop and no red flags does not need further investigations until conservative management proves ineffective after 6 weeks of treatment,

For patients with a red flag further investigation and/or referral may be indicated. The investigations of choice differ based on the probability diagnoses formed through history and
physical findings. Despite being non-sensitive and non-specific, plain radiography is often the initial imaging study. Detailed and more specific investigations are then considered
based on the probability diagnosis.

This man has low back pain with typical findings of painfully restricted movements and tender L4-L5. Importantly, he has no red flags in the history and physical examination;
therefore, a diagnosis of non-specific low back pain can be considered for now. Of the options, analgesia and maintenance of physical activity is the best initial management for
him. Physiotherapy can be discussed and offered for symptom relief thereafter.

(Option A) Imaging studies such as plain radiography, CT scan, or MRI are not indicated in evaluation of non-specific low back pain, unless there is at least one red flag present.
Choice of further investigation depends on other pieces of information from history and physical examination.

(Option B) Although widely used, several studies found little or no difference between individuals with low-back pain who used lumbosacral corsets as a means of back support and
those who received no treatment.

(Option D) Patients with non-specific low back pain should be encouraged to continue their daily routines and remain physically active. Bed rest should be strongly discouraged.

(Option E) Pelvic traction is an irrelevant and inappropriate option for treatment of non-specific low back pain.

References

• Diagnostic Imaging Pathways – Low Back Pain

• Australian Prescriber - Managing low back pain in primary care

• FP Notebook - Low Back Pain Red Flag

1169 of 1943
• RACGP - Approach to low back pain
Last updated:
Time spent: QID:1220
2023-2-12

1170 of 1943
A 47-year-old woman presents to your practice with complaint of progressive right elbow pain for the past two months. She is an office secretary and her job entails lots of computer
keyboard and mouse use. The pain increases through the day as she performs her tasks. Recently, the pain is also brought up by gripping, and sometimes persists at night. On
examination, there is no deformity, redness or warmth of the right elbow, and the range of motion is normal. However, palpation elicits tenderness approximately 2 cm below the
right lateral epicondyle. Fingers and wrist have normal range of motion but passive terminal flexion of the wrist and resisted extension of the wrist and middle finger reproduce the
pain. Which one of the following is the most likely cause of this presentation?

A. Olecranon bursitis.

B. Lateral epicondylitis.

C. Medial epicondylitis.

D. Rupture of the wrist extensor tendons.

E. Stress fracture of the right epicondyle.

Incorrect. Correct answer is B


45% answered correctly

Explanation:

Correct Answer Is B

The scenario represents a typical case of humeral lateral epicondylitis, also known as lateral tennis elbow. Lateral epicondylitis is the most common overuse syndrome of the upper
limb and is related to excessive wrist extension.

The typical age of onset is 40 to 50 years. There is almost always a history of overuse without specific trauma.

Lateral epicondylitis presents with pain in the lateral elbow that worsens with activity and alleviates with rest. The pain often can be pinpointed 1-2 cm distal to the proximal origin of
the extensor carpi radialis brevis muscle (ECRB) at the lateral epicondyle. Affected individuals commonly describe aggravating activities such as a backhand stroke in tennis or the
overuse of a screwdriver, computer keyboard or mouse. Pain can vary from being mild (e.g. with aggravating activities like tennis or the repeated use of a hand tool), or it can be
such severe that simple activities like gripping of even light objects such as coffee cup (i.e. "coffee cup sign") are painful. The pain may also radiate down the posterior aspect of the
forearm.

On examination, the elbow has normal appearance and range of motion. However, pronation at its extreme can be painful. Resisted extension of the wrist and the middle finger as
well as the passive terminal flexion of the wrist while the elbow is fully extended reproduce the pain. It is important to notice that lack of such findings does not exclude lateral
epicondylitis as a likely diagnosis.

Lateral epicondylitis is a benign self-limiting condition that often resolves within 6 to 12 months in most cases. Activity modification, counterforce bracing, and home-based exercise
programs constitute the initial conservative management in almost all affected persons.

(Option A) Olecranon bursa is a subcutaneous synovial-lined sac overlying the olecranon process at the proximal end of the ulna. This bursa cushions the olecranon and decreases
friction between it and the skin. Olecranon bursitis is the inflammation of this bursa that most commonly presents with a focal swelling at the posterior elbow. Pain may or may not
be reported by the patient but is often absent in noninflammatory aseptic cases. If present, pain is usually exacerbated by pressure, such as when the patient leans on the elbow or
the elbow is rubbed against a surface like tabletop during writing. Many cases are idiopathic. Infection is a less common but important cause occasionally (septic bursitis).

(Option C) Medial epicondylitis is a condition similar to lateral epicondylitis in pathophysiology. It affects the tendons of wrist flexor muscles the pronator teres and flexor carpi
radialis at their proximal insertion at the humeral medical epicondyle. Overuse of the wrist flexor muscles by excessive wrist flexion is the main etiology. The pain is felt in the medial
aspect of the forearm and can be reproduced by resisted flexion of the wrist or passive terminal wrist extension with the elbow in full flexion. There is often tenderness over the
medial epicondyle and the proximal wrist flexor muscles mass. The ulnar groove, ulnar nerve and medial collateral ligament should NOT be tender. Medial epicondylitis is managed
similar to lateral epicondylitis with activity modification, rest from the causative activity, home-based exercises to strengthen the wrist flexor muscles, ice, NSAIDs, or physiotherapy.

(Option D) With the rupture of the wrist extensor muscles tendons, wrist extension was expected to be defected which is not the case here.

(Option E) Although the stress fracture of the right lateral epicondyle can present with an almost similar presentation (expect for the maximal tenderness being elicited over the
lateral epicondyle itself rather than 1-2 cm below it), it is not a likely diagnosis because it is a very rare condition that often occurs in children. Stress fractures of the medial
epicondyles are much more common than those of right epicondyle.

References

• https://www.uptodate.com/contents/epicondylitis-tennis-and-golf-elbow

• https://emedicine.medscape.com/article/96969-overview

Last updated:
Time spent: QID:1233
2023-2-12

1171 of 1943
Which one of the following wrist movements is most likely to reproduce the pain in a patient with lateral epicondylitis?

A. Radial deviation.

B. Resisted extension.

C. Supination.

D. Ulnar deviation.

E. Resisted flexion.

Incorrect. Correct answer is B


45% answered correctly

Explanation:

Correct Answer Is B

Lateral epicondylitis, also referred to as ‘lateral tennis elbow’ is the most common overuse syndrome related to excessive wrist extension. Patients with lateral epicondylitis typically
present with pain in the lateral aspect of the forearm that may radiate down into the forearm. There is often maximal tenderness 1-2 cm distal to the origin of the extensor carpi
radialis brevis muscle (ECRB) at the lateral epicondyle.

The pain in lateral epicondylitis is reproduced with resisted wrist extension with the elbow fully extended, or passive terminal wrist extension while the elbow is fully extended.
Resisted extension of the middle finger is also painful secondary to stress placed on the ECRB tendon, as it is preferentially stressed in this position when it must contract
synergistically to anchor the third metacarpal, such that extension can take place at the digits. Resisted pronation of the wrist and gripping while the elbow is extended can also
bring up the pain.

Of the given options, only resisted extension of the wrist can reproduce the pain in a patient with lateral epicondylitis.

(Options A and D) Radial and ulnar deviation of the wrist do not exert a significant force on extensor muscles and are unlikely to reproduce the pain.

(Option C) supination of the wrist, especially if resisted, can reproduce the pain in medial epicondylitis (golf elbow), as does the resisted flexion of the wrist (option E).

References

• https://www.uptodate.com/contents/epicondylitis-tennis-and-golf-elbow

• https://emedicine.medscape.com/article/96969-clinical#b4

Last updated:
Time spent: QID:1234
2023-2-12

1172 of 1943
Overuse of which one the following is most likely the underlying mechanism of lateral epicondylitis?

A. Wrist extension.

B. Wrist flexion.

C. Elbow flexion.

D. Elbow extension.

E. Wrist pronation.

Correct
45% answered correctly

Explanation:

Correct Answer Is A

Lateral epicondylitis, also referred to as ‘lateral tennis elbow’ is the most common overuse syndrome related to excessive wrist extension.

The lateral epicondyle of the humerus serves as the bony common origin of the wrist extensors. Overuse injury to the extensor carpi radialis brevis muscle (ECRB), which is felt at
the tip of lateral epicondyle and occasionally, the extensor digitorum communis muscle (EDC) (felt just posterior and lateral to the tip of lateral epicondyle) constitutes lateral
epicondylitis.

‘Tennis elbow’ and ‘lateral epicondylitis’ are, however, misnomers because the condition is seen more commonly in non-tennis players than tennis players and the underlying
pathophysiology is degenerative rather than inflammatory. Microscopic evaluation of the tendons does not show signs of inflammation, but rather angiofibroblastic degeneration
and collagen disarray. Light microscopy reveals both an excess of fibroblasts and blood vessels that are consistent with neovessels or angiogenesis. The injury to the tendon often
occurs 1 to 2 cm below the attachment point of ECRB to lateral epicondyle.

References

• https://emedicine.medscape.com/article/96969-overview

Last updated:
Time spent: QID:1235
2023-2-12

1173 of 1943
A 45-year-old man presents to your GP clinic with complaint of a painful right elbow. The pain is felt laterally radiating to the back of the forearm, aggravates with activity and
relieves with rest. He is a carpenter and makes cabinets. His job includes lots of sawing, drilling, and using screw drivers. On examination, you notice a tender point 1-2 cm distal to
the right lateral epicondyle. Extension of the wrist with the elbow in full extension brings up the pain. Which one of the following is the most appropriate next step in management?

A. Physiotherapy.

B. Advising himr to quit his job.

C. Intralesional corticosteroids.

D. Surgery.

E. Band support below the elbow.

Incorrect. Correct answer is E


45% answered correctly

Explanation:

Correct Answer Is E

The scenario describes a typical case of lateral epicondylitis, also known as ‘lateral tennis elbow’, which is the most common overuse syndrome of the upper limb and is related to
excessive wrist extension. The condition is typically seen in persons of 40-50 years of age.

Affected individuals often report an insidious onset, with a history of overuse without specific trauma almost always present in the history. Patients with the condition complain of
pain over the lateral elbow that worsens with activity and alleviates with rest. The pain often can be pinpointed 1-2 cm distal to attachment of the extensor carpi radialis brevis
muscle (ECRB) tendon to the lateral epicondyle.

Lateral epicondylitis is benign self-limiting condition that often resolves within 6 to 12 months in most cases. Activity modification, counterforce bracing, and home-based exercise
programs constitute the initial conservative management in almost all cases. This patient should be advised to modify all his daily activities in a way that limits forearm pronation,
the position in which wrist extension exerts great force on wrist flexor tendons. This often includes using a palm-up position to avoid or limit pronation.

Of the given options band support below the elbow (counterforce bracing) is the most appropriate option to consider for him. Counterforce braces encircle the forearm 6 to 10 cm to
reduce the tension forces on the wrist extensor tendons and provide rest for the extensor tendons. Despite conflicting evidence, they are still widely used in management of lateral
epicondylitis. Counterforce braces are inexpensive, convenient, and have shown to decrease pain and increase grip strength in most cases within 3-6 weeks.

A ciunterforce brace for lateral epicondylitis

Application of ice and short-term use of topical or oral non-steroidal anti-inflammatory drugs (NSAIDs) are other measures that might be considered for acute management of
lateral epicondylitis.

(Option A) Some studies have shown that physiotherapy can be more effective than a wait-and-watch approach or topical corticosteroid injection in both the short and long-term
outcomes; however, it is often reserved for those who fail to respond to the above measures such as home-based exercise programs.

(Option B) Although this patient has developed lateral epicondylitis most likely due to the overuse of his elbow in his carrier, advising him to quit the job he is doing for a living is not
appropriate and practical.

(Options C) Intralesional corticosteroid injection is reserved for patients with refractory severe pain and disability despite conservative management. Corticosteroid injection is
associated with a rapid response in short term (usually within 4 weeks) but poorer outcome in long term.

(Option D) Surgery is rarely used for treatment of patients who fail to response to all available measure for treatment of lateral epicondylitis.

1174 of 1943
References

• https://www.uptodate.com/contents/epicondylitis-tennis-and-golf-elbow

• http://www.flexphysiotherapy.com.au/upload/pages/info2v2/coombes-le-guidelines-2015.pdf

• http://www.cwp.nhs.uk/media/1266/cc26-clinical-guideline-to-standardise-the-management-of-tennis-elbow-issue-1.pdf

• https://www.racgp.org.au/your-practice/guidelines/handi/interventions/musculoskeletal/physiotherapy-for-tennis-elbow/

Last updated:
Time spent: QID:1236
2023-2-12

1175 of 1943
Janet is a 43-year-old IT worker, who has presented to you with complaint of pain in the right elbow. She describes that the pain increases through the day as she does her job which
includes lots of typing with computer keyboard and use of computer mouse. On examinations, the elbow looks normal with preserved range of motion, but slight pain at the extreme
of pronation. Tenderness is elicited over the lateral epicondyle of the right humerus when palpated deeply. The pain is reproduced on resisted extension of the wrist while her
forearm is fully extended. The arm is otherwise normal. Which one of the following would be the best treatment option for Janet at this stage?

A. Immobilization of the elbow.

B. Corticosteroid injection.

C. Encircling support strap for the right forearm.

D. Oral NSAIDs.

E. Topical NSAIDs.

Incorrect. Correct answer is C


45% answered correctly

Explanation:

Correct Answer Is C

The scenario is a typical description of lateral epicondylitis, which is caused by degenerative changes of the wrist extensor muscles at their origin in the humeral lateral epicondyle.
Of these muscles, the extensor carpi radialis brevis muscle (ECRB) is the most commonly affected one.

The diagnosis is almost always clinical, but x-rays, ultrasound, or MRI can be used when the diagnosis is in doubt or for further management planning in refractory cases.

A myriad of treatment options has been investigated and applied for treatment of lateral epicondylitis; however, conservative management including activity modification to prevent
further tendon overuse, counterforce bracing, and home-based exercises to increase the strength of the extensor muscles is almost always the best initial option to consider.
Although controversial, short term use of topical (or sometimes oral) non-steroidal anti-inflammatory drugs (NSAIDs) may be prescribed for acute and very painful cases.
Application of ice can also help with pain reduction in an acute phase. The outcome is then reviewed, often in 4 to 6 or 8 weeks for further management.

Patients with lateral epicondylitis should be advised and taught to modify their arm movements in a way that most of their activities are done with the palm facing up rather down in
an attempt to prevent or limit forearm pronation in which wrist extension can exert excess force on the wrist extensor tendons. However, for some patients, whose daily activity,
including their job, entails tasks such as typing or computer mouse clicking, such instruction may be impractical to follow.

Another important step in management is counterforce bracing of the forearm by placing an encircling band around it. Counterforce braces are strapped around the forearm below
the elbow to reduce the tension forces on the wrist extensor tendons and provide rest, an important part of the initial management, for the extensor tendons. Despite conflicting
evidence, counterforce braces are still widely used. Counterforce braces are cost-effective and easy to use, and may provide benefit during the first 6 weeks of treatment by pain
relief and grip strengthening. An encircling support strap below the elbow acts as a counterforce brace for Janet and is the best choice among the given options.

(Option A) Immobilization of the elbow has been shown to be inferior to counterforce bracing below the elbow in pain reduction and increasing grip strength.

(Option B) Intralesional corticosteroid injection is reserved for patients with refractory severe pain and disability despite conservative management. Corticosteroid injection is
associated with a rapid response in short term but poorer outcome in long term.

(Options D and E) Lateral epicondylitis is a tendinosis rather than tendinitis with minimum or absent inflammatory changes on histopathology. Despite the absence of apparent
inflammation, NSAIDs either topical or oral (if not contraindicated) have shown benefit in short term (up to 14 days) in some studies. Although controversial, they are often used for
patients with lateral epicondylitis, specially during the acute initial phase. Diclofenac is the preferred NSAIDs as it has showed efficacy in some studies (to date naproxen has proved
not more effective than placebo).

References

• https://www.uptodate.com/contents/epicondylitis-tennis-and-golf-elbow

• http://www.flexphysiotherapy.com.au/upload/pages/info2v2/coombes-le-guidelines-2015.pdf

• http://www.cwp.nhs.uk/media/1266/cc26-clinical-guideline-to-standardise-the-management-of-tennis-elbow-issue-1.pdf

Last updated:
Time spent: QID:1237
2023-2-12

1176 of 1943
A 48-year-old man presents to your practice, complaining of the soreness of his right elbow. He is an office worker who uses the computer keyboard and mouse a lot at work, to
which he thinks his pain is related. On examination, the right elbow has both normal appearance and range of motion. There is, however, tenderness over the lateral epicondyle. Wrist
extension while the forearm is extended reproduces the pain. Which one of the following is the most appropriate treatment option for him?

A. Finger immobilization splint.

B. Rest and paracetamol.

C. Immobilization of the elbow.

D. Using encircling support orthotic on forearm.

E. Topical diclofenac.

Incorrect. Correct answer is D


45% answered correctly

Explanation:

Correct Answer Is D

This man has presented with a clinical presentation that is very typical for lateral epicondylitis. Lateral epicondylitis is caused by degenerative changes of the wrist extensor muscles
at their origin in the humeral lateral epicondyle. The extensor carpi radialis brevis muscle (ECRB) is the most commonly affected wrist extensor muscle.

For patients with lateral epicondylitis, conservative management with activity modification to prevent further tendon overuse, counterforce bracing, and home-based exercises to
strengthen the wrist extensor muscles is almost always the best initial management option.

Persons with lateral epicondylitis should be instructed to modify their wrist and elbow movements in a way and direction that makes the palm facing up rather than down to prevent
or limit forearm pronation in which wrist extension can exert excess force on the wrist extensor tendons. This, however, is not applicable for all patients because their daily activity,
including their job, entails activities such as typing or computer mouse clicking.

Another important step in management is counterforce bracing of the forearm by placing an encircling band around it. Counterforce braces are wrapped around the forearm below
the elbow and reduce the tension forces on the wrist extensor tendons and provide rest, which is a very important portion of initial conservative management. Although evidence is
conflicting and controversial, counterforce braces are still widely used and deemed useful for treatment because they are cheap and easy to use and may provide benefit during the
first 6 weeks of treatment. An encircling support strap below the elbow acts as a counterforce brace for this patient and the best choice among other options.

(Option A) Fingers extension does not exert force on wrist extensor tendons; therefore, finger immobilization is not effective in providing rest for the wrist extensor tendons at the
elbow.

(Option B) Use of paracetamol as a non-prescribed analgesic has been used for alleviation of pain in short term. Advice for rest, however, is not practical in this scenario as the
nature of his job entails use of computer keyboard and mouse, during which the causative wrist and forearm movements are continued. For him, counterforce strapping decreased
tension on wrist extensor tendons while he can continue his daily activities.

(Option C) Immobilization of the elbow has been shown to be inferior to counterforce bracing below the elbow in pain reduction and increasing strength of grip.

(Option E) Although controversial, short term use of topical (or sometimes oral) non-steroidal anti-inflammatory drugs (NSAIDs) may be considered for acute and very painful cases.
However, rest from the implicated activity by means of counterforce bracing remains the most appropriate initial option.

References

• https://www.uptodate.com/contents/epicondylitis-tennis-and-golf-elbow

• https://emedicine.medscape.com/article/96969-overview

• http://www.cwp.nhs.uk/media/1266/cc26-clinical-guideline-to-standardise-the-management-of-tennis-elbow-issue-1.pdf

• http://www.flexphysiotherapy.com.au/upload/pages/info2v2/coombes-le-guidelines-2015.pdf

Last updated:
Time spent: QID:1238
2023-2-12

1177 of 1943
Which one of the following is not considered an extra-articular manifestation of rheumatoid arthritis?

A. Hepatomegaly.

B. Osteoporosis.

C. Pericardial effusion.

D. Cutaneous nodules.

E. Peripheral neuropathy.

Correct
45% answered correctly

Explanation:

Correct Answer Is A

Extra-articular RA is a serious condition, and RA patients with extra-articular manifestations should be treated and monitored aggressively. Extra-articular manifestations of RA occur
in approximately 40% of patients, either in the beginning or during the course of their disease. Extra-articular manifestations of RA include the following:

Bone manifestations:

Osteopenia and osteoporosis are very common in patients with RA. The development of osteopenia in RA patients appears to occur independently of corticosteroid use

Skin manifestations:

Rheumatoid nodules (the most frequent skin manifestation seen in 20% of patients with RA)
Pyoderma gangrenosum

Ocular manifestations:

Keratoconjunctivitis sicca – affects at least 10% of RA patients. It is frequently observed together with xerostomia in a secondary Sjögren syndrome.
Episcleritis (in 1% of RA patients)
Scleritis
Peripheral ulcerative keratitis

Oral manifestations:

Oral dryness (xerostomia) and salivary gland swelling can be found in patients with RA. These patients can also develop secondary Sjögren syndrome.

Gastrointestinal manifestations:

Gastrointestinal manifestations of RA are more related to treatment rather than the disease. A rare gastrointestinal problem seen in RA patients is caused by mesenteric
vasculitis leading to intestinal infarctions, bleeding, and perforation.

Pulmonary manifestations:

Pleural disease (in 50% of RA patients)


Parenchymal pulmonary nodules
Interstitial lung disease

Cardiac manifestations:

Pericarditis (the most common cardiac manifestation of RA)


Myocardial infarction
Myocarditis
Congestive heart failure
Endocarditis

Renal manifestations:

Renal manifestations of RA are rare and include:

Mesangial glomerulonephritis
Amyloidosis

Neurological manifestations:

Peripheral neuropathy occurs in a small subset of patients


Cervical myelopathy caused atlantoaxial subluxation or pannus formation.

Hematological manifestations:

( )
1178 of 1943
Anemia (one of the most common extra-articular manifestations of RA)
Neutropenia
Thrombocytosis
Lymphadenopathy

Of the given options hepatomegaly is not an extra-articular manifestation of RA.

References

• Medscape - Systemic Extra-articular Manifestations of Rheumatoid Arthritis

• MAEDICA - Extra-articular Manifestations in Rheumatoid Arthritis

Last updated:
Time spent: QID:1257
2023-2-12

1179 of 1943
While getting off the car, a 40-year-old man developed sudden-onset severe low back pain and got locked in the middle of the way, according to him. He is now in the emergency
department for evaluation, complaining of lower back pain, shooting sharp pain in the right leg and foot, and restricted and painful lumbar movements. He runs a grocery store and
has been otherwise fit. He drinks alcohol on social occasions and smokes 3 to 4 cigarettes a day. His past medical history is insignificant. On examination, he has stable vital signs
and tenderness over the L4-L5 region. Except for positive slump test and straight leg raise test, neurological exam findings are inconclusive with no motor or sensory deficits. Which
one of the following is the most appropriate management option for him?

A. Obtaining a lumbosacral spine x-ray.

B. Obtaining a lumbosacral spine CT scan.

C. Obtaining a lumbosacral spine MRI.

D. Nerve conduction studies.

E. Observation.

Incorrect. Correct answer is E


45% answered correctly

Explanation:

Correct Answer Is E

It is estimated that up to 80% of adults have low back pain at some times in their lives. Despite the fact that there is a long list of potential causes for low back pain, most patients
seen in primary care (>85%) will have nonspecific low back pain, meaning that there is back pain in the absence of a specific underlying condition that can be reliably identified. Only
less than 1% are found to have a serious underlying disease such as cancer or infection.

In the middle of this spectrum, there are other less serious but specific etiologies, the most important of which are vertebral compression fractures, radiculopathies, and spinal canal
stenosis. Other conditions include osteoarthritis of the lumbosacral spinal column, scoliosis and hyperkyphosis, psychological stress and extraspinal causes such as pancreatitis,
nephrolithiasis, pyelonephritis, abdominal aortic aneurysm and herpes zoster.

In this patient, the acute onset of back pain shooting down the leg and foot and positive slump and straight leg tests are highly suggestive of radicular pain (sciatica). Moreover, he
has no ‘red flag’ (see ‘NOTE’ below).

Current guidelines recommend that patients with radicular pain, who have no red flag, be managed conservatively with:

1. Education and explanations


2. Simple analgesics such as paracetamol or NSAIDs for pain control
3. Encouraging maintenance of daily life and remaining physically active
4. Recommending and referral for physiotherapy for those with debilitating pain and/or severe restriction of movements
5. Review and reassessment (recommended in one and 4-6 weeks)

This means that patient with radicular pain do not require any studies such as plain X-rays (option A), CT scan (option B), MRI (option C), or nerve conduction studies (option D). An
exception is the presence of significant motor deficit not localized to a single unilateral nerve root. In such patients urgent MRI and referral is the management of choice. This
means that, for example, a patient with low back pain, unilateral foot drop and no red flags does not need further investigations until conservative management proves ineffective
after six weeks of treatment.

Of the options, observation while the patient is on conservative management as mentioned above is the most appropriate next step in management.

NOTE - Red flags for low back pain are summarized in the following table:

Red flags for low-back-pain


Feature/risk factor Condition to consider
History of cancer with new onset back pain

Unexplained weight loss

Failure to improve after one month

Age >50 years or <20 years


Cancer

Nocturnal pain

1180 of 1943
Fever

Intravenous drug use

Infection
Recent infection

Immunocompromised state

Rest pain

Urinary retention

Fecal incontinence

Saddle anesthesia
Cauda equina syndrome

Lower limb weakness or paresthesia

History of osteoporosis

Use of corticosteroid Vertebral fracture

Older age (>50 years)

History of fall or other trauma

Morning stiffness

Improvement with exercise

Alternating buttock pain

Ankylosing spondylitis
Waking up because of pain during the second part of the night

Younger age

References

• Diagnostic Imaging Pathways – Low Back Pain

• Australian Prescriber - Managing low back pain in primary care

• NSW Agency for Clinical Innovation – Managing people with low back pain

• FP Notebook - Low Back Pain Red Flag

Last updated:
Time spent: QID:1263
2023-2-12

1181 of 1943
A 55-year-old man presents with sudden onset severe pain over the L4/L5 region which is sharp and shooting in nature and radiated down to both legs. He has never had such pain
before and there is no history of preceding trauma or strain. He has lost six kilograms in the past two months. Which one of the following in the history is the most important factor
necessitating further investigations in this patient?

A. The first episode ever.

B. No history of preceding trauma or strain.

C. His age.

D. Weight loss.

E. Radicular nature of his pain.

Incorrect. Correct answer is D


45% answered correctly

Explanation:

Correct Answer Is D

Despite the long list of conditions resulting in low back pain, most patients seen in primary care (>85%) will have nonspecific low back pain meaning that the patient has back pain in
the absence of specific underlying condition that can be reliably identified. Of those who present to primary care, only less than 1% are found to have a serious underlying disease
such as cancer or infection.

In the middle of this spectrum, there are other less serious but specific etiologies, the most important of which are vertebral compression fractures, radiculopathies, and spinal canal
stenosis, etc.

In management of patients with low back pain, the first step is making a judicious decision as to which patient can be managed conservatively versus those who may need more
investigations and probably referral for specialist care. Such assessment starts with history taking and physical examination and checking for the presence of any alarming feature
or risk factors that may point toward a serious underlying condition.

Such features and risk factors are known as ‘red flags’ and are summarized in the following table:

Red flags for low-back-pain


Feature/risk factor Condition to consider
History of cancer with new onset back pain

Unexplained weight loss

Failure to improve after one month

Age >50 years or <20 years


Cancer

Nocturnal pain

Fever

Intravenous drug use

Infection
Recent infection

Immunocompromised state

Rest pain

Urinary retention

Fecal incontinence

Saddle anesthesia
Cauda equina syndrome

Lower limb weakness or paresthesia

1182 of 1943
History of osteoporosis

Use of corticosteroid Vertebral fracture

Older age (>50 years)

History of fall or other trauma

Morning stiffness

Improvement with exercise

Alternating buttock pain

Ankylosing spondylitis
Waking up because of pain during the second part of the night

Younger age

The presence of red flags is an indication for further evaluation and referral for further specialized management. One important issue is that red flags should be evaluated in the
context of the clinical presentation as a whole, not as the sole guide for management because over 80% of patients with non-specific low back pain have a red flag in their histories
irrelevant to their back pain. High index of suspicion should be raised when there is more than one red flag present.

Of the options, the patient’s age (>50 years) and the recent history of weight loss are alarming features with the latter being of significant importance. Back pain in the presence of a
history of recent unintentional weight loss, especially in someome older than 50 years, should always raise suspicion about malignancies for which meticulous assessment is
prompted.

Being the first episode ever (option A) is not a red flag, neither is absence of a preceding trauma or sprain (option B). In fact, in most patients with low back pain no precipitating
event is identified.

(Option C) Age older than 50 years or younger than 20 years is a red flag. The age alone, however, is not as significant as the recent weight loss in this man.

(Option E) Radicular pain is quite common. Radiculopathies are often benign and managed conservatively with resumed daily activities, analgesics, and in more severe cases
physiotherapy. Investigations such as imaging or electrophysiological studies are unnecessary unless there is significant motor deficits not justified by a single nerve root
involvement, suspicion against cord compression, or cauda equina syndrome, for which urgent MRI is mandatory.

References

• Diagnostic Imaging Pathways – Low Back Pain

• Australian Prescriber - Managing low back pain in primary care

• NSW Agency for Clinical Innovation – Managing people with low back pain

• FP Notebook - Low Back Pain Red Flag

Last updated:
Time spent: QID:1264
2023-2-12

1183 of 1943
A 72-year-old patient of yours has presented to your clinic for advice regarding osteoarthritis of his right knee. He finds walking difficult due to pain. You recommend that he use a
stick as a walking aid. He needs to know how he should use it. Which one of the following is the appropriate recommendation for holding the stick and walking?

A. He should hold the stick in his right hand and step with the right leg first.

B. He should hold the stick in his right hand and step with the left leg first.

C. He should hold the stick in his left hand and step with the right leg first.

D. He should hold the stick in his left hand and step with the left leg first.

E. He should hold the stick in his right hand and can step with either leg.

Incorrect. Correct answer is C


45% answered correctly

Explanation:

Correct Answer Is C

When using a stick as a walking aid for problems of a lower limb (foot, ankle, knee, or hip), the stick should be held with the hand on the unaffected side (good leg) and the first step
should be taken with the affected limb (bad leg). As this patient’s problem is in his right leg, he should hold the stick with his left hand (the side of the good leg) and put his right foot
(bad leg) forward first.

When walking up stairs the good leg should take the first step and when walking down take the first step with the injured or affected leg.

References

• wikiHow - How to Hold and Use a Cane Correct

Last updated:
Time spent: QID:126
2023-2-12

1184 of 1943
While gardening, a 45-year-old woman develops sudden onset low back pain. She is in the Emergency Department for evaluation now. Her past medical history is significant for
breast cancer 10 years ago that was treated with breast-preserving surgery, adjuvant chemotherapy and radiotherapy. She has been disease-free since then. On examination, there is
tenderness over the L4/L5 region, and painfully restricted flexion, extension and rotation. Neurological exam is unremarkable. Which one of the following is the most appropriate
next step in management?

A. Physiotherapy.

B. MRI of the lumbosacral spine.

C. X-ray of the lumbosacral spine.

D. Advise that she continue daily activities.

E. Advise strict bed rest for 48 hours and analgesics.

Incorrect. Correct answer is C


45% answered correctly

Explanation:

Correct Answer Is C

Acute episodes of low back pain are very common and often managed conservatively if a thorough history and physical examination is negative for alarming signs, symptoms, and
risk factors necessitating prompt investigations for a serious underlying cause. Such alarming features are termed ‘red flags’ for low back pain and are summarized in the following
table:

Red flags for low-back-pain


Feature/risk factor Condition to consider
History of cancer with new onset back pain

Unexplained weight loss

Failure to improve after one month

Age >50 years or <20 years


Cancer

Nocturnal pain

Fever

Intravenous drug use

Infection
Recent infection

Immunocompromised state

Rest pain

Urinary retention

Fecal incontinence

Saddle anesthesia
Cauda equina syndrome

Lower limb weakness or paresthesia

History of osteoporosis

Use of corticosteroid Vertebral fracture

Older age (>50 years)

History of fall or other trauma

1185 of 1943
Morning stiffness

Improvement with exercise

Alternating buttock pain

Ankylosing spondylitis
Waking up because of pain during the second part of the night

Younger age

With the presence of an active, treated or currently-in-remission malignancy in history, the back pain should be considered to have been caused by bone metastasis until proven
otherwise.

The bone is one of the most common sites of metastasis. A history of cancer, except non-melanoma skin cancers, is a very serious alarming factor for back pain from bone
metastasis. Breast, prostate, lung, thyroid, and kidney cancers account for 80% of metastases. Almost 60% of patients with multiple myeloma have skeletal lytic lesions present at
diagnosis.

Pain is the most common symptom in of metastatic bone disease. In patients with history of cancer, sudden, severe pain raises concern for pathologic fracture. Neurologic
symptoms from spinal cord compression or spinal instability could be another manifestation.

Where a pathologic fracture is suspected, plain radiography is often the initial imaging study. Detailed and more specific investigations are then considered based on the probability
diagnosis. For this woman, an X-ray of the lumbosacral spine to look for possible vertebral fracture(s) due to metastatic disease is the most appropriate initial step in management
among other options.

Conservative management with advice of continued activity (option D), short-term use of analgesics, or physiotherapy (option A) if the pain is severe and disabling are steps to
consider once a serious underlying pathology for the back pain has been safely excluded.

(Option B) MRI is often indicated in patients with radicular pain that cannot be solely attributed to one nerve root. Generally, radiculopathies are managed conservatively with
analgesia, physiotherapy and encouraging maintained physical activity.

(Option E) Although simple analgesics or a short course of NSAIDs are often advised for management of nonspecific or benign low back pain, strict bed rest should be discouraged.
Patients should return and maintain daily physical activity as soon as possible.

References

• Diagnostic Imaging Pathways – Low Back Pain

• Australian Prescriber - Managing low back pain in primary care

• NSW Agency for Clinical Innovation – Managing people with low back pain

• FP Notebook - Low Back Pain Red Flag

Last updated:
Time spent: QID:1265
2023-2-12

1186 of 1943
A 58-year-old woman presents to your GP clinic with complaint of back pain that started suddenly after one hour of gardening. Her past medical history is unremarkable. She has
been healthy and active until the event. She does not smoke and drinks alcohol in moderation. On examination, flexion, extension and rotation of the lumbar spine are significantly
limited and tenderness between L4 and L5 is elicited. There is no tenderness over the vertebral bodies. After prescribing analgesics, which one of the following would be the most
appropriate next step in management?

A. Bed rest.

B. Referral for orthopedic consultation.

C. Use of corsets.

D. Advise that she should maintain physical activity.

E. Magnetic resonance imaging of the lumbar spine.

Incorrect. Correct answer is D


45% answered correctly

Explanation:

Correct Answer Is D

Low back pain in the absence of alarming features is history and physical examination does not require specific management or evaluation other than reassurance, advising
maintained physical activity, short-term use of analgesics and review in 4 to 6 weeks. Physiotherapy may be considered for patients with protracted severe pain despite initial
measures.

Alarming features are a set of signs, symptoms, or specific risk factors, the presence of which raise concerns regarding a serious underlying etiology for the low back pain such as
malignancies, infections, cord compression, cauda equina syndrome, etc. This occurs in approximately 1% of patients presenting with acute low back pain.

Alarming features, also known as ‘red flags’ are summarized in the following table:

Red flags for low-back-pain


Feature/risk factor Condition to consider
History of cancer with new onset back pain

Unexplained weight loss

Failure to improve after one month

Age >50 years or <20 years


Cancer

Nocturnal pain

Fever

Intravenous drug use

Infection
Recent infection

Immunocompromised state

Rest pain

Urinary retention

Fecal incontinence

Saddle anesthesia
Cauda equina syndrome

Lower limb weakness or paresthesia

1187 of 1943
History of osteoporosis

Use of corticosteroid Vertebral fracture

Older age (>50 years)

History of fall or other trauma

Morning stiffness

Improvement with exercise

Alternating buttock pain

Ankylosing spondylitis
Waking up because of pain during the second part of the night

Younger age

Red flags should be evaluated in the context of the clinical presentation as a whole, not as the sole guide for management, because over 80% of patients with non-specific low back
pain have a red flag in their histories irrelevant to their back pain. High index of suspicion is raised when there is more than one red flag present.

With the age older than 50 years, this woman has a red flag. Although low back pain in patients older than 50 can have been caused by a pathologic fracture due to metastasis or
osteoporosis, negative findings in history and physical findings make such diagnoses weak possibilities. The age alone in this context is not a red flag justifying further assessment;
imaging studies in particular.

NOTE - There are often abnormal findings on imaging studies of adults without low back pain which can make it difficult to correlate symptoms with imaging findings in
patients with low back pain. Disc herniation is seen on MRI scans of 22 to 67% of asymptomatic adults and spinal stenosis in 21% of adults older than 60 years. Evidence of
osteoarthritis is often seen on imaging but correlates poorly with symptoms.

Even when the radiographic findings are consistent with clinical presentation, the magnitude of radiographic findings does not necessarily correlate with clinical severity and
outcome, and clinical improvement may not correlate with resolution of the radiographic defect.

For this woman, advising maintained physical activity is the best management option. She can take analgesics if the pain is bothersome. A review in 4 to 6 weeks should follow to
see the response to treatment and further management planning.

(Option A) Bed rest should be discourage because immobility is associated with poorer outcomes.

(Option B) Referral for orthopedic or neurological consultation could be a correct option if there was any pointers towards a more serious cause other than non-specific low back
pain.

(Option C) Corsets provide support and correction of posture and have shown benefit in some studies on patients with chronic low back pain due to anatomical and structural
deformities. Their use in patients with acute low back pain has not been associated with significant improved outcomes. Therefore, they are not recommended for management of
acute low back pain.

(Option E) MRI would be the option of choice if there was any significant motor deficit not justified by a single nerve root pathology, any suspicion of cord compression, or cauda
equina syndrome. This woman does not have any clinical features suggestive of these.

References

• Diagnostic Imaging Pathways – Low Back Pain

• Australian Prescriber - Managing low back pain in primary care

• NSW Agency for Clinical Innovation – Managing people with low back pain

• FP Notebook - Low Back Pain Red Flag

Last updated:
Time spent: QID:1266
2023-2-12

1188 of 1943
Jane is a 37-year-old woman who has presented to your clinic for the first time. She has a facial rash shown in the following photograph. According to her, the rash is more
prominent when she is exposed to sunlight. She also complains of 4-kg weight loss in the past 3 months, occasional low-grade fever, and fatigue. She has had intermittent painful
and mildly swollen wrists and knees to which recent finger pain has been added. On examination, oral ulcers that she has never noticed before are found. You order laboratory
tests, which are remarkable for mild normocytic, normochromic anemia; mild thrombocytopenia; an elevated ESR of 110; positive ANA, and positive ds-DNA antibody. Which one of
the following is the most appropriate long-term treatment option for her?

A. Naproxen.

B. Systemic corticosteroids.

C. Hydroxychloroquine.

D. Methotrexate.

E. Cyclophosphamide.

Incorrect. Correct answer is C


45% answered correctly

Explanation:

Correct Answer Is C

The characteristic malar rash seen in the photograph that is sensitive to sun exposure, and other clinical and laboratory findings in this woman are suggestive of systemic lupus
erythematosus (SLE). SLE is a chronic multisystem autoimmune disease 9 times more common in women than men. The disease has a relapsing-remitting nature and patients
experience episodes of exacerbations interspersed with periods of absent or relatively mild symptoms.

Clinical presentation of SLE is diverse because the disease has the potential to involve any organ system virtually. Patients often present with problems of the skin, mucosa, joints,
CNS, kidneys, and hematological aberrancies.

According to current criteria, diagnosis of definitive SLE requires 4 or more of the following 11 criteria with at least one clinical and one laboratory, with the exception of biopsy-
proven LN (which requires fewer criteria). Criteria are cumulative and need not be present concurrently.

1. Acute of subacute cutaneous lupus:


2. Chronic cutaneous lupus
3. Oral or nasal ulcers
4. Non-scarring alopecia
5. Synovitis involving 2 or more joints
6. Serositis (e.g. pericarditis, pleuritis)
7. Renal involvement i.e., lupus nephritis
8. Neurologic manifestations
9. Hemolytic anemia
10. Leukopenia (<4000/mm3) or lymphopenia (<1000/mm3)
11. Thrombocytopenia (<100,000/mm3)

There are also immunologic criteria associated with specific presentations of SLE including:

Positive ANA: acute of subacute lupus


Positive Anti-ds-DNA antibody: chronic cutaneous lupus
Positive Anti-Sm antibody: oral and/or nasal ulcers
Low complement (C3, C4.CH50): non-scarring alopecia
Positive direct Coombs’ test: synovitis
Anti-phospholipid antibody: serositis

This woman has 3 clinical criteria (malar rash, mouth ulcers, and joint pain), one clinical criterion (thrombocytopenia), and 2 immunological criteria (positive ANA and ant- ds-DNA
antibody). She has a definitive diagnosis of SLE.

Management of SLE often depends on disease severity and disease manifestations; however, hydroxychloroquine is the cornerstone of treatment in all SLE patients and should be
used as the most appropriate long-term treatment option for Jane. Hydroxychloroquine is associated with decreased flare-ups and survival benefits in SLE patients. It is well-
tolerated and has few adverse effects. One concern is the risk of ocular toxicity that can be eliminated with appropriate dosing.

Nonsteroidal anti-inflammatory drugs (NSAIDs) such as naproxen (option A) or low-dose systemic steroids can be considered for SLE patients with mild cutaneous lupus,
tenosynovitis, or serositis. In long term, however, such drugs are associated with undesirable adverse effects.

1189 of 1943
A number of steroid-sparing immunosuppressive drugs beyond hydroxychloroquine are used to treat severe lupus manifestations other than skin manifestations, tenosynovitis, and
serositis, the involvement of which is considered mild lupus. For example, cyclophosphamide (option E) and mycophenolate are medications of choice for lupus nephritis. Janet
does not have renal involvement and such drugs are not indicated for her. Methotrexate (option D) is another steroid-sparing agent that can be used in SLE patients.

NOTE - Patients with severe lupus such as those with neuropsychiatric features, renal involvement, hemolytic anemia, or severe thrombocytopenia might need moderate to
high doses of steroids for acute management. In the long term, corticosteroids should be replaced by steroid-sparing agents.

References

• RACGP - Focus - Systemic lupus erythematosus

• Medscape - Systemic Lupus Erythematosus (SLE)

Last updated:
Time spent: QID:1297
2023-2-12

1190 of 1943
Mary is 32 years old and 36 weeks pregnant. She has had an uneventful pregnancy to date and has attended her antenatal care regularly. At today's visit, fetal auscultation is
significant for bradycardia. A CTG confirms a persistently decreased fetal heart rate of 67 bpm. Due to the risk of fetal jeopardy, emergency cesarean section is considered and
performed. The neonate has still bradycardia at the rate of 70 bpm. Which one of the following antibodies is most likely to have contributed to such a presentation?

A. Anti-Ro antibody.

B. Anti-La antibody.

C. Anti-cardiolipin antibody.

D. Anti-Sm (Smith) antibody.

E. U1-ribonucleoprotein.

Correct
45% answered correctly

Explanation:

Correct Answer Is A

With the options all being immunologic markers, the question requires you to know the congenital rheumatologic disease associated with neonatal bradycardia, which is congenital
(neonatal) lupus erythematosus (NLE).

Neonatal lupus erythematosus (NLE) is thought to be caused by the transplacental passage of maternal autoantibodies; however, only 1-2% of infants with positive maternal
autoantibodies develop NLE. The most common clinical manifestations of NLE are dermatologic, cardiac, and hepatic. Some infants may also have hematologic, central nervous
system, or splenic abnormalities.

Cardiac involvement in NLE is common, occurring in nearly 65% of patients. Rhythm abnormalities and conduction defects may be observed in various forms, but congenital
complete heart block is the most common rhythm disturbance observed, with an incidence of 15-30%. Complete heart block can present with profound bradycardia resulting in
congestive heart failure. Cardiac blocks usually develop in utero between the 18th and 20th weeks of pregnancy. The risk is more significant for neonates born to mothers with
primary Sjögren syndrome or undifferentiated autoimmune syndrome than those with systemic LE (SLE).

The mother produces immunoglobulin G autoantibodies against Ro (SSA), La (SSB), and/or U1-ribonucleoprotein (U1-RNP), and they are passively transported across the placenta.
The presence of maternal anti-SSA/Ro and anti-SSB/La antibodies increases the risk of bearing infants with NLE; rarely, NLE is due to maternal passage of U1-RNP antibodies.
These autoantibodies can be found alone or in combination; however, anti-Ro is present in almost 95% of patients and is most likely to have caused this problem.

Anti-La antibody (option B) and U1-ribonucleoprotein (option E) can also cause NLE but anti-Ro antibody is most likely to be detected.

There is no strong link between NLE and maternal anti-cardiolipin (option C) or anti-Sm (option D) antibody.

References

• https://emedicine.medscape.com/article/1006582-clinical

Last updated:
Time spent: QID:1298
2023-2-12

1191 of 1943
A 60-year-old man presents to the Emergency Department with a complaint of a red and severely painful right big toe as shown in the following photograph. He has had such
episodes in the same toe and the left big toe before. Based on the history, physical examinations, and laboratory tests, he is diagnosed with an acute attack of gouty arthritis. He is
obese and has a BMI of 32. He drinks 5 to 6 standard units of alcohol every day. He also has hypertension currently controlled on hydrochlorothiazide. His lab results
include hypercholesterolemia, high LDL, low HDL, a serum creatinine level of 160 mmol/L (normal 50-110 mmol/L), and a serum uric acid level of 670 mmol/L (normal: 120-420
mmol/L) are remarkable. Which one of the following would you consider the first line for his treatment?

A. Prednisolone.

B. Codeine.

C. Naproxen.

D. Allopurinol.

E. Colchicine.

Correct
45% answered correctly

Explanation:

Correct Answer Is A

First-line medications for the treatment of acute gouty arthritis are NSAIDs and colchicine. Of NSAIDs, indomethacin and naproxen are the potent ones mostly prescribed for the
condition, but other NSAIDs such as ibuprofen or diclofenac can be effective too. The COX2 inhibitor celecoxib has approximately the same efficacy and can replace NSAIDs where
gastrointestinal bleeding makes use of traditional NSAIDs a concern.

Although naproxen (option C) is one of the first-line options for the treatment of acute gout, it should be avoided for this patient, as he has an increased serum creatinine level of 180
mmol/L indicating renal impairment.

Colchicine is another first-line medication for the treatment of acute gouty arthritis. Treatment starts with 1.2 mg statim, followed by 0.6 mg six hours later, and then 0.6 mg once or
twice daily for two to three days for those with normal renal function. In patients with reduced kidney function e.g. chronic kidney disease (CKD), who have a GFR of less than 50
ml/min/1.73m2, or those receiving CYP3A4 or P-glycoprotein inhibitors, the dose of colchicine should be reduced. Colchicine is better avoided or used with extreme caution (e.g.
longer dose intervals) in patients with GFR<30 ml/ml/1.73m2.

Oral steroids, such as prednisolone or intra-articular steroids are also effective for the treatment of acute gouty arthritis but are reserved for patients who cannot tolerate or have
contraindications to first-line medications. The recommended dose is 0.5mg/kg of prednisolone (or equivalent) for 5 to 10 days with gradual dose reduction. Intra-articular steroid
injection is the preferred method with only one affected joint and contraindications to first-line agents.

For this patient, with renal impairment, either colchicine (option E) at a reduced dose or prednisolone can be used; however, since GFR is not known (not at least in the question),
prednisolone would be safer than colchicine and the preferred option here. It should be noted that (for the purpose of the exam) colchicine can be an appropriate option in the
absence of prednisolone.

(Option B) Codeine can relieve the pain but is not effective in counteracting the inflammation; hence, not an appropriate option.

(Option D) Allopurinol is a uric acid lowering agent and is indicated for patients with high serum uric acid levels as a measure of prophylaxis and prevention of further recurrences.
Despite being effective in the prevention of further attacks, it has no role in managing acute gout.

References

• https://www.racgp.org.au/afp/2016/may/the-management-of-gout-much-has-changed/

• https://www.nps.org.au/radar/articles/colchicine-for-acute-gout-updated-information-about-dosing-and-drug-interactions

• Calculation of GFR based on sex, weight and creatinine level

• Creatinine level: mmol/L to mg/dL converter

1192 of 1943
Last updated:
Time spent: QID:1322
2023-2-12

1193 of 1943
Sue, 56 years old, presents to the Emergency Department with acute-onset right knee pain starting 3 hours ago and worsening gradually. On examination, she has a BMI of 32, blood
pressure of 158/105 mmHg, pulse rate of 90 bpm, and temperature of 37°C. A joint aspirate excludes septic arthritis; however, the presence of birefringent crystals is highly
suggestive of gouty arthritis. Her medical history is remarkable for chronic kidney disease due to long-standing hypertension. Laboratory tests are ordered, of which a serum uric
acid of 0.6 mmol/L (normal: 0.12-0.42 mmol/L) and a GFR of 30 (normal: 90-120 ml/min/1.73m2) are remarkable. Which one of the following is the most appropriate treatment to
consider for her?

A. Paracetamol.

B. Naproxen.

C. Codeine.

D. Colchicine.

E. Allopurinol.

Incorrect. Correct answer is D


45% answered correctly

Explanation:

Correct Answer Is D

This patient has an acute attack of gouty arthritis. The aims of treatment for gouty arthritis are the treatment of acute attacks and the prevention of further episodes.

Non-steroidal anti-inflammatory drugs (NSAIDs) or colchicine are first-line management options for first-time or recurrent acute gout attacks. Of the NSAIDs, indomethacin has been
traditionally used, as a potent NSAID, for the treatment of acute gouty arthritis. Naproxen is another potent NSAID for this purpose. These two are the most often used NSAIDs for
those under 60 years of age, for whom there is no contraindication to use of this drug class. Other less potent NSAIDs such as ibuprofen or diclofenac can be used as well. Some of
the contraindications to using NSAIDs are:

History of peptic ulcer disease or gastrointestinal bleeding


Chronic kidney disease
Hepatic disease
Any kind of other adverse drug reaction to NSAIDs

NOTE - the cyclooxygenase (COX) 2 inhibitor celecoxib has been used with success in the treatment of acute gouty arthritis with almost the same efficacy as traditional NSAIDs.
Celecoxib is the preferred option when NSAIDs are considered for treatment, but the patient has a history of GI bleeding, peptic ulcer disease, or gastritis.

Colchicine is another first-line medication for the treatment of acute gouty arthritis. Treatment starts with 1.2 mg statim, followed by 0.6 mg six hours later, and then 0.6 mg once or
twice daily for two to three days for those with normal renal function. This dosing is not associated with gastrointestinal adverse effects such as diarrhea and vomiting which were
commonly seen with higher doses and shorter intervals that were traditionally used.

NOTE - the 0.6 mg tablet size is not available in Australia and the 0.5 mg tablet is usually substituted.

For patients with reduced kidney function e.g., chronic kidney disease (CKD), who have a GFR of less than 50 ml/min/1.73m2, or those receiving CYP3A4 or P-glycoprotein inhibitors,
the dose of colchicine should be reduced. Colchicine is better avoided or used with extreme caution (e.g., longer dose intervals) in patients with GFR<30 ml/min/1.73m2.

Oral steroids are reserved for patients who cannot tolerate or have contraindications to first-line medications. The recommended is 0.5mg/kg of prednisolone (or equivalent) for 5 to
10 days with gradual dose reduction. Intra-articular steroid injection is the preferred method with only one affected joint and contraindications to first-line agents.

Sue has a GFR of 30 ml/min/1.73m2. For her, NSAIDs such as naproxen (option B) are contraindicated. For her, either reduced doses of colchicine or steroids (not an option) would
be the regimen of choice.

NOTE – if an option, oral or intra-articular steroid will be a better choice for acute gouty arthritis in patients with renal impairment.

Although paracetamol (option A) or codeine (option C) can provide pain relief, they do not have an anti-inflammatory effect, which is required for the treatment of gouty arthritis.

Allopurinol (option E) is a urate-lowering agent and is used for gout prophylaxis where indicated. It is not effective in the treatment of acute gout.

References

• RACGP - AFP - The management of gout: Much has changed

• NPS - Colchicine for acute gout: updated information about dosing and drug interactions

Last updated:
Time spent: QID:1349
2023-2-12

1194 of 1943
Laura, 56 years old, presents to the Emergency Department with an acutely painful left index finger shown in the following photograph. On examination, she is afebrile, and the right
finger's distal interphalangeal (DIP) joint is red, warm, and extremely tender to touch with decreased motion due to pain. She has had 4 episodes of such incidents in the past 2 years
in the same finger diagnosed as recurrent gouty arthritis. She is currently on a prophylactic dose of colchicine and allopurinol 100 mg daily. She is also a known case of long-
standing hypertension, controlled on hydrochlorothiazide and metoprolol, and also a known case of hypertensive nephropathy. Which one of the following would you consider for her
as the first-line treatment option at this stage?

A. Cessation of allopurinol.

B. Indomethacin.

C. Celecoxib.

D. Prednisolone for 10 days.

E. Increasing the dose of allopurinol to 150 mg, daily.

Incorrect. Correct answer is D


45% answered correctly

Explanation:

Correct Answer Is D

First-line treatment for patients with acute gouty arthritis is either potent NSAIDs such as indomethacin and naproxen or colchicine. The COX 2 inhibitor celecoxib is an acceptable
but off-lable option when NSAIDs are considered first line, but the patient has a history of peptic ulcer disease, gastrointestinal bleeding ,or gastritis.

NOTE - Since celecoxib is not approved for treatment of gout, it is better not to choose it as an option in the exam.

In general, oral steroids, such a prednisolone or intra-articular steroids should be considered for patients who cannot tolerate or have contraindications to first-line medications. The
recommended dose is 0.5mg/kg of prednisolone (or equivalent) for 5 to 10 days with gradual dose reduction. Intra-articular steroid injection is the preferred method with only one
affected joint and contraindications to first-line agents. Colchicine in reduced dose (not an option here) can be used as well.

For this patient with renal impairment, prednisolone for 10 days is the most appropriate option.

(Option A) Allopurinol is a urate-lowering agent and used for prevention of further attacks of gouty arthritis. It should never be ceased during a gouty arthritis flare-up, as the
resultant fluctuating serum urate level may worsen the condition.

(Options B and C) NSAIDs such as indomethacin and the COX2 inhibitor celecoxib are contraindicated in patients with chronic kidney disease and should be avoided.

(Option E) Increasing the dose of allopurinol may considered later for prevention of further attacks but is not useful for treatment of acute flares. It is also recommended to avoid
changing the dose of allopurinol in patient who is already on it and has presented with an acute attack.

References

• RACGP - AFP - The management of gout: Much has changed

• NPS - Colchicine for acute gout: updated information about dosing and drug interactions

Last updated:
Time spent: QID:1355
2023-2-12

1195 of 1943
A 58-year-old man presents to your GP clinic with an acutely painful left knee complaint. On examination, the left knee is warm, swollen, and tender. and erythematous. He is afebrile,
and the rest of the physical examination is inconclusive. A joint aspirate shows typical features of gouty arthritis. Which of the following is the most appropriate next step in the
management of this patient?

A. Celecoxib.

B. Aspirin.

C. Colchicine.

D. Prednisolone.

E. Allopurinol.

Incorrect. Correct answer is C


45% answered correctly

Explanation:

Correct Answer Is C

For treatment of gouty arthritis, either non-steroidal anti-inflammatory drugs (NSAIDs) or colchicine are used as first-line options. Of NSAIDs, indomethacin and naproxen are most
potent and usually selected over other NSAIDs for acute treatment of gouty arthritis.

Of the options, colchicine can be used for this patient as the first-line medication (NSAIDs would be a better choice in options). Treatment starts with 1.2 mg statim, followed by 0.6
mg six hours later and then 0.6 mg once or twice daily for two to three days for those with normal renal function. In patients with reduced kidney function e.g. chronic kidney disease
(CKD), who have a GFR of less than 50 ml/min/1.73m2, or those receiving CYP3A4 or P-glycoprotein inhibitors, the dose of colchicine should be reduced. Colchicine is better
avoided or used with extreme caution (e.g. longer dose intervals) in patients with GFR<30 ml/ml/1.73m2.

(Option A) Celecoxib, a COX2 inhibitor, is categorized under NSAIDs. Although celecoxib is sometimes used off-label for patients with gout in whom traditional NSAIDs are
contraindicated due to gastrointestinal problems such as history of gastrointestinal bleeding, peptic ulcer disease or gastritis, it has not been officially approved for treatment of
gout. It should be noted that celecoxib still has the hepatic, renal, and cardiovascular toxic effects similar to NSAIDs.

(Option B) Although aspirin has anti-inflammatory effects, it can impair excretion of urate from kidneys and should be avoided in patients with gout.

(Option D) Oral steroids, such a prednisolone or intra-articular steroids should be considered for patients who cannot tolerate or have contraindications to first-line medications such
as NSAIDs and colchicine.

(Option E) Allopurinol is a uric acid lowering agent, and is indicated for patients with high serum uric acid levels as a measure of prophylaxis and prevention of further recurrences.
Despite being effective in prevention of further attacks, it has no role in management of acute gout.

References

• RACGP - AFP - The management of gout: Much has changed

Last updated:
Time spent: QID:1357
2023-2-12

1196 of 1943
A 49-year-old woman presents to your GP clinic with progressive low back pain that is shooting and radiates to her left buttock. She denies any preceding trauma or sprain. Her past
medical history is significant for breast cancer treated with mastectomy, chemotherapy, and radiotherapy six year ago and osteoarthritis of the lumbosacral spine. On examination,
there is tenderness over the L4/L5 area, a positive straight leg raise (SLR) test on the left side. Which one of the following features in the clinical assessment is an indication for
further assessments of this patient?

A. Absence of a preceding trauma or sprain.

B. Osteoarthritis of the lumbosacral spine.

C. Positive SLR test.

D. The radicular nature of her pain.

E. Past history of breast cancer.

Incorrect. Correct answer is E


45% answered correctly

Explanation:

Correct Answer Is E

This patient has progressive back pain, shooting in nature, that makes radiculopathy a possibility.

Acute episodes of low back pain are very common and often managed conservatively if a thorough history and physical examination is negative for alarming signs, symptoms, and
risk factors. These alarming features are termed ‘red flags’ for low back pain.

In this patient, the history of breast cancer is an alarming feature and requires prompt further assessment because bone is one of the most common sites of metastasis. A history
of cancer, except non-melanoma skin cancers, is the strongest alarm in the history of any patient with back pain. Breast, prostate, lung, thyroid, and kidney cancers account for 80%
of metastases. Almost 60% of patients with multiple myeloma have skeletal lytic lesions present at diagnosis.

Pain is the most common symptom in of metastatic bone disease. In patients with history of cancer, sudden severe pain raises concern for pathologic fracture. Neurologic
symptoms from nerve root/ spinal cord compression or spinal instability could be another manifestation.

Where a pathologic fracture is suspected, plain radiography is often the initial imaging study. Detailed and more specific investigations are then considered based on the probability
diagnosis.

(Option A) Absence of a preceding trauma or sprain is not an indication for further evaluation because many episodes of non-specific mechanical back pain are not preceded by
trauma or sprain remembered by the patient.

(Options B) Osteoarthritis of the lumbosacral spine is not a red flags for back pain to mandate further assessment.

(Option C and D) Radicular nature of the pain, confirmed by the SLR test, is not an indication for further assessment of this patient. In general, radicular pain is managed
conservatively with analgesia, physiotherapy and maintained daily physical activity. For radicular pain without alarming features, MRI is not necessary unless the presentation is
unusual, such as sign and symptoms that cannot be attributed to only one nerve root involvement.

TOPIC REVIEW

Red flags for low back pain are summarized in the following table:

History of cancer with new onset back pain

Unexplained weight loss

Failure to improve after one month

Age >50 years or <20 years


Cancer

Nocturnal pain

1197 of 1943
Fever

Intravenous drug use

Infection
Recent infection

Immunocompromised state

Rest pain

Urinary retention

Fecal incontinence

Saddle anesthesia
Cauda equina syndrome

Lower limb weakness or paresthesia

History of osteoporosis

Use of corticosteroid Vertebral fracture

Older age (>50 years)

History of fall or other trauma

Morning stiffness

Improvement with exercise

Alternating buttock pain

Ankylosing spondylitis
Waking up because of pain during the second part of the night

Younger age

References

• Diagnostic Imaging Pathways – Low Back Pain

• Australian Prescriber - Managing low back pain in primary care

• NSW Agency for Clinical Innovation – Managing people with low back pain

• FP Notebook - Low Back Pain Red Flag

Last updated:
Time spent: QID:1359
2023-2-12

1198 of 1943
A 68-year-old patient of yours presents for medical evaluation. He is a known-case of benign prostatic enlargement. Today he is complaining of urinary problems in form of difficulty
starting micturition and dribbling at the end of urination, as well as constipation. He also has back pain and left thigh pain. On examination, there is decreased sensation over the
medial aspect of the left lower leg. Which one of the following indicates intervertebral disc prolapse in this patient?

A. Urinary problems.

B. Thigh pain.

C. Back pain.

D. Decreased sensation of the lower leg.

E. Constipation.

Incorrect. Correct answer is D


45% answered correctly

Explanation:

Correct Answer Is D

Of the options, only decreased sensation of the lower leg can be specific to an intervertebral disc prolapse and others can be manifestations of non-neurological problems as well.

Radicular lower back pain can happen due to chemical or mechanical irritation of nerve roots. The pain is sharp, shooting and has an electric quality. Unilateral leg pain is the more
pronounced symptoms and is often worse than pain in the back. Pain concentrates distally, running into the lower limb, usually extending below the knees. Pain, numbness and
paresthesia follow a dermatomal distribution. Reflexes may be reduced or even absent. Motor weakness not always would be present.

Lesions of the cauda equina can present with back pain, leg pain, paresthesia around the anus, and urinary incontinence. However, in this patient, the urinary problem is described as
difficulty starting micturition and terminal dribbling, which is consistent with bladder outlet obstruction by causes such as an enlarged prostate. Urinary problems (option A) of this
patient is very unlikely to have been caused by an intervertebral disc prolapse.

Although disc prolapse in the lumbar area can present with back pain (option C) and thigh pain (option B), such pain may have been caused by other factors than radiculopathy.
Back pain may be due to a mechanical stressor. In fact, back pain is the common factor between mechanical and neural pain in the back. The accompanying symptoms are the only
indicators helping to differentiate between the two. Leg pain may have other causes such as mechanical injuries and vascular problems (e.g., venous insufficiency, arterial disease,
etc.). Neural impairments other than radiculopathy may give rise to pain as well.

Constipation (option E) in this patient may have many other causes as well, with one being bladder obstruction and pressing of a distended bladder on the rectum. Decreased
dietary fibers, limited physical activity, and many other factors could have caused the constipation. Disc prolapse is more likley to be associated with fecal incontinence rather than
constipation.

References

• Australian Family Physician Vol. 33, No. 6, June 2004

Last updated:
Time spent: QID:1540
2023-2-12

1199 of 1943
Which one of the following statements is correct regarding osteoarthritis (OA)?

A. Immobilization of the affected joint at the early stages of the disease slows further damage.

B. OA is the disease of the subchondral bone that progresses to involve the overlying cartilage later in the course of the disease.

C. OA is often associated with morning stiffness of over 30 minutes.

D. Secondary OA commonly affects the metacarpophalangeal joints.

E. OA can develop secondary to rheumatoid arthritis (RA).

Incorrect. Correct answer is E


45% answered correctly

Explanation:

Correct Answer Is E

OA characteristically starts with thinning and destruction of the cartilage (not the bone). Less protection due to loss of cartilage renders the underlying bones susceptible to
repetitive traumas resulting in bone destruction and impaired bone healing and remodeling.

OA has two types:

Idiopathic (primary) OA - idiopathic OA comprises the majority of cases. Joint involvement is usually symmetrical. The most common joint involved is the first carpometacarpal joint
of the thumb (base of the thumb), followed by the first metatarsophalangeal of the big toe and the distal interphalangeal (DIP) joints of the hand. Other joints that may be affected
are the proximal interphalangeal joints of the hand, the knees, the hips, acromioclavicular joints, and spinal joints especially the facet joints between C5 and C6, C6 and C7, L3 and
L4, and L4 and L5. Generally, primary OA is more common in women (with the exception of OA of the hip, which affects both sexes equally)

Secondary OA - in secondary OA, there is an underlying cause for OA. Joints are usually affected unilaterally, with the knee being the most commonly affected joint. Secondary OA is
more common in men. Secondary OA typically involves joints with pres-existing damage such as those with destructions and deformities caused by rheumatoid arthritis, chronic
gout, etc. Generally speaking, any structural abnormality makes a joint prone to OA.

The clinical presentation of OA is pain that comes on with activity and is relieved by rest. The pain often becomes worse throughout the day. If there is morning stiffness, it should
be less than 30 minutes.

The first-line pharmacological treatment for OA is simple analgesics such as paracetamol. It is strongly recommended that paracetamol be taken on a regular basis, even when
there is no pain, rather than on an as-needed basis because the former provides more pain control.

NSAIDs are used as second-line medications only if:

Optimal therapy with paracetamol fails to control the pain


There is evidence of inflammation, such as pain that is worse with rest, nocturnal pain, and gelling of the joint (stiffness with rest)

Immobilization of the affected joint is never a part of the management plan because It makes the affected joint stiffer and the condition worse.

References

• RACGP - Musculoskeletal Health

• Medscape - Osteoarthritis

Last updated:
Time spent: QID:129
2023-2-12

1200 of 1943
A 47-year-old woman presents to your clinic with an 8-month right ankle swelling and pain history. The patient relates that the ankle is stiff in the morning and takes over one hour of
walking and physical activity to ease up. No other joint is involved. On examination, she has a temperature of 36.9°C with other vital signs within normal limits. The right ankle joint is
swollen and has a restricted range of motion and is slightly tender. An X-ray of the right ankle and foot shows periarticular osteopenia and narrowed joint space of the right ankle.
Which of the following could be the most likely underlying cause of this presentation?

A. Septic arthritis.

B. Osteomyelitis.

C. Systemic lupus erythematosus (SLE).

D. Rheumatoid arthritis (RA).

E. Osteoarthritis (OA).

Incorrect. Correct answer is D


45% answered correctly

Explanation:

Correct Answer Is D

This patient has a painful and swollen right ankle joint associated with a morning stiffness of over one hour which indicates there is an inflammatory process either infective or
rheumatologic involved. Osteoarthritis (OA) (option E) is not inflammatory and unlike inflammatory arthritides is aggravated by activity and relieved by rest. Joint space narrowing
can be a feature of OA, but subchondral sclerosis and cysts rather than periarticular osteopenia/osteoporosis are expected on x-ray.

X-ray changes in OA of weight-bearing joints such as the hip, knee, and ankle include:

Joint space narrowing


Osteophytes
Subchondral sclerosis and cysts

Septic arthritis (option A) has an acute presentation with fever, joint pain, and tenderness, swelling, warmth, and redness in a matter of hours to days. An afebrile patient with joint
pain and swelling of 8 months duration is very unlikely to have septic arthritis.

X-ray findings in septic arthritis include the following:

X-rays may be normal in the very early stage of the disease


The joint effusion may be seen
Juxta-articular osteoporosis due to hyperemia
Joint space narrowing due to cartilage destruction in the acute phase
Destruction of the subchondral bone on both sides of a joint
If left untreated, reactive juxta-articular sclerosis and, in severe cases, ankylosing will develop

Local presentations of osteomyelitis (option B) are usually seen in the affected bone rather than the adjacent joint. Osteomyelitis has an insidious onset with constitutional
symptoms such as fatigue, fever, chills, local bone tenderness, etc. Early plain radiological findings in osteomyelitis are subtle and not often obvious until 5-7 days from the onset in
children and 10-14 days in adults. Radiographs taken after this time may show the following:

Effusion at the adjacent joint (usually the only joint finding)


Regional osteopenia of the affected bone
Periosteal reaction (often thickening or elevation caused by periostitis)
Focal bony lysis or cortical loss
Loss of trabecular architecture
New bone formation
Eventual peripheral sclerosis.

This patient's, history, physical examination, and radiological findings are inconsistent with osteomyelitis as a diagnosis.

SLE (option C) is a multisystem rheumatologic disease with joints involved in a large number of patients (70-90%). Often two or more joints are involved. SLE arthritis is non-erosive,
meaning that no erosive lesions or joint space narrowing or destruction is expected on the x-ray of an SLE-affected joint. This patient has joint space narrowing which is inconsistent
with SLE. Expected X-ray findings of an affected joint in SLE include:

Soft-tissue swelling
Periarticular (juxta-articular) osteopenia or osteoporosis.
Preserved joint space

Of the options, only RA best fits the x-ray findings described in the scenario. X-ray findings of an affected joint in RA include the following:

Soft tissue swelling (mostly synovium)


Joint space narrowing or even destruction in more advanced cases
Erosions of joint margins (in more advanced cases)
Periarticular (juxta-articular) osteopenia or osteoporosis.

NOTE – Typically, patients with RA arthritis present with symmetrical arthritis affecting the wrists and the metacarpophalangeal and proximal interphalangeal joints of the
hands. Involvement of the metatarsophalangeal joints of the feet is also common. Occasionally, large joints such as the knee and ankle can be affected in isolation, in which
case septic or crystal arthropathy may need to be excluded.

References

( )
1201 of 1943
• Radiopaedia: Rheumatoid arthritis (musculoskeletal manifestations)

• Radiopaedia: Systemic lupus erythematosus (musculoskeletal manifestations)

• Radiopaedia: Osteoarthritis

• Radiopaedia: Septic arthritis

• Radiopaedia – Osteomyelitis
Last updated:
Time spent: QID:1610
2023-2-12

1202 of 1943
Which one of the following is NOT an expected physical finding in infectious tenosynovitis of finger?

A. Pain on passive flexion of the affected finger.

B. Pain on passive extension of the affected finger.

C. Finger in fixed flexion.

D. Fusiform swelling of the whole finger.

E. Tenderness along the flexor tendon sheath.

Correct
45% answered correctly

Explanation:

Correct Answer Is A

Flexor sheath infections, also known as infective tenosynovitis, are relatively common hand infections.

Digital flexor sheaths are a closed continuous synovial system that contain the flexor digitorum profundus and flexor digitorum superficialis. Flexor sheath infections are caused by
direct inoculation due to trauma, spread of infection from adjacent tissues, or via hematogenous spread. Common injuries include animal bites, puncture wounds, IV drug use, and
wounds exposed to fresh/saltwater. Such injuries, if associated with severe tenderness, should raise a high suspicion of flexor tenosynovitis.

Infective flexor tenosynovitis of the middle finger

Flexor sheath infections begin with buildup of exudative fluid within the tendon sheath, followed by the development of purulent fluid. Bacterial overgrowth within the synovial sheath
leads to increased fluid and pressure, leading to ischemia, and resulting in a septic necrosis of the tendon sheath and surrounding structures. If left untreated, complications may
include soft tissue necrosis, osteomyelitis, and necrotizing fasciitis.

Infective flexor tenosynovitis presents with the four cardinal signs referred to as Kanavel signs:

Flexed resting position of the affected finger


Fusiform swelling of the whole finger
Tenderness along the flexor tendon sheath
Pain (usually exquisite) with passive extension of the affected finger

Of the options, pain on passive flexion of the affected finger is not a feature.

References

1203 of 1943
• BJGP - Kanavel signs of flexor sheath infection: a cautionary tale
Last updated:
Time spent: QID:1688
2023-2-12

1204 of 1943
Which one of the following is the earliest sign of flexor sheath infection?

A. Fusiform swelling of the finger.

B. Pain on passive extension of the affected finger.

C. Fever.

D. Tenderness along the tendon sheath.

E. Flexed resting position of the affected finger.

Incorrect. Correct answer is B


45% answered correctly

Explanation:

Correct Answer Is B

Flexor sheath infections, also known as infective tenosynovitis, is a serious hand infection which if not attended to urgently can result in a septic necrosis of the tendon sheath and
surrounding structures. If left untreated, complications may include soft tissue necrosis, osteomyelitis, and necrotizing fasciitis.

Flexor sheath infections are caused by direct inoculation due to trauma, spread of infection from adjacent tissues, or via hematogenous spread.

Common signs of flexor sheath infections are:

Flexed resting position of the affected finger


Fusiform swelling of the whole finger
Tenderness along the flexor tendon sheath
Pain (usually exquisite) with passive extension of the affected finger

These signs are also known as Kanavel signs. Collectively, Kanavel signs have a specificity of over 90% for diagnosis of flexor sheath infection.

Of these signs, the earliest is usually pain on passive infection of the affected sign. A later sign is tenderness along the flexor sheath (option D) that suggest proximal spread of the
infection, followed by fusiform swelling (option A) and fixed resting position of the finger (option E). Fever (option C) is often absent but if present indicates systemic spread of the
infection which happens late in the course of the disease if left untreated.

References

• BJGP - Kanavel signs of flexor sheath infection: a cautionary tale

Last updated:
Time spent: QID:1690
2023-2-12

1205 of 1943
Ruth, 70 years of age, has presented to your office with a complaint of muscle weakness for the past couple of weeks. She has hypertension and coronary artery disease for which
she is on ramipril and atorvastatin. Physical examination is remarkable for bilateral shoulder girdle muscle weakness and tenderness. You order blood tests that are significant for
an elevated ESR of 48 mm and normal creatine kinase (CK). Which one of the following is the most likely diagnosis?

A. Polymyositis.

B. Statin-induced myositis.

C. Dermatomyositis.

D. Polymyalgia rheumatica.

E. Adhesive capsulitis.

Incorrect. Correct answer is D


45% answered correctly

Explanation:

Correct Answer Is D

Polymyositis (option A), statin-induced myositis (option B), and dermatomyositis (option C) all can cause inflammation and damage of the muscles, primarily proximal muscles,
with elevated inflammatory markers such as ESR and CRP as well CK as a result. For Ruth, a normal CK excludes such conditions with rather a high certainty.

Adhesive capsulitis (option E), if bilateral, can present with rather similar physical findings but inflammatory ESR and CRP are normal.

Polymyalgia rheumatica (PMR) on the other hand fits the scenario. PMR is a relatively common chronic inflammatory condition of unknown etiology that affects elderly individuals.
It is characterized by symmetrical myalgia of the hip and/or shoulder girdles with accompanying morning stiffness lasting more than 45 minutes. PRM is a diagnosis of exclusion.
Laboratory findings in PMR include elevated ESR (often above 40) and CRP, and normocytic normochromic anemia.

References

• RACGP – AFP – Polymyalgia rheumatica: clinical update

• Medscape - Polymyalgia Rheumatica

Last updated:
Time spent: QID:1718
2023-2-12

1206 of 1943
The 9-year-old Tim is brought to the Emergency Department by his mum after he fell off his bike and hurt his left shoulder. The left shoulder is painful, and he is barely moving his
left arm due to the pain. On physical examination, there is no bruise or deformity of the left shoulder, but there is tenderness over the middle third of the left clavicle. Neurovascular
examination of the left arm and hand is normal. Which one of the following is the most appropriate next step investigation?

A. X-ray of the left clavicle; PA view.

B. X-ray of the left clavicle; AP view.

C. X-ray of both clavicles; AP view.

D. CT scan.

E. X-ray of the left clavicle; oblique view.

Incorrect. Correct answer is B


45% answered correctly

Explanation:

Correct Answer Is B

Tim has clinical features of fracture/injury of the middle third of his left clavicle. Clavicular fractures are common in children, often a as result of a fall on outstretched arm. When
clinical suspicion of clavicle exists, an standard anteroposterior (AP) x-ray and AP with 15° cephalic tilt is the initial and, in most cases, the only investigation needed.

The AP film visualizes the entire clavicle including its articulation with the acromion and the sternum. AP with cephalic tile straightens out the clavicle and is used to better visualize
occult fractures.

CT scan (option D) may be considered in fractures of the medial third of the clavicle for assessment of potential tracheal impingement and thoracic anatomy, or in displaced lateral
third injuries to assess the coracoclavicular ligament.

PA views (option A) and obliques views (option E) add no diagnostic value and are not recommended for suspected clavicular fractures. AP views of both clavicles (option C) are
indicated only of comparison is required or the injury to both clavicles is suspected.

References

• RCH - Clavicle fractures - Emergency Department

• Radiopaedia - Clavicle series

Last updated:
Time spent: QID:1726
2023-2-12

1207 of 1943
4. A 52-year-old man comes to your GP practice complaining of difficulty holding and grasping objects. He has smoked 20 sticks per day for the past 15 years but is otherwise well.
You immediately notice his hands as shown in the photo below. On further probing, the patient tells you that he has similar findings on his feet. Which of the following is the most
likely diagnosis?

A. Buerger Disease.

B. Wegener’s polyangiitis.

C. Raynaud phenomenon.

D. Diabetes mellitus.

E. Cogan’s Syndrome.

Correct
45% answered correctly

Explanation:

Correct Answer Is A

The clinical picture is suggestive of thromboangiitis obliterans, also known as Burger’s disease, as the most likely diagnosis.

Thromboangiitis obliterans also called Buerger disease is a nonatherosclerotic, segmental, inflammatory disease that most commonly affects the small to medium-sized arteries
and veins of the extremities, resulting in occlusive thrombus and distal extremity ischemia It occurs most commonly in men, with the age of onset in the 40s, is strongly associated
with smoking, and is often progressive. Amputation is commonly required to manage pain or secondary infection.

Wegener’s polyangiitis or granulomatosis (option B) with polyangiitis is a rare vasculitis of unknown cause. The classic triad is (1) upper respiratory tract granuloma, (2) fleeting
pulmonary shadows (nodules), and (3) glomerulonephritis, none of which is present in this scenario.

Raynaud phenomenon (option C) is a clinical syndrome of episodic arteriolar vasospasm usually involving the fingers and toes (one or two at a time). The characteristic finding in
the Raynaud phenomenon is being triggered by cold temperatures, anxiety, or stressful events. The fingers and toes change from blue to red to a very pale appearance. It may also
involve the nose, ear, or nipple. It does not cause or result in digital amputation.

Diabetes mellitus is an established and common underlying cause of digital amputation. However, it is unlikely in this scenario in the absence of a history or suggestive clinical
findings (option D).

Cogan’s syndrome (option E) is a rare autoimmune vasculitis that affects the eyes and inner ears. Symptoms of the syndrome include irritation and pain in the eyes, decreased
vision, hearing loss, and vertigo.

References

• Murtagh's General Practice - 8th Edition

Last updated:
Time spent: QID:1772
2023-2-12

1208 of 1943
A 30-year-old female with severe muscle aches and fatigue for the past 4 months. She denies fever or weight loss but admits to mood and sleep disturbances. On examination,
there are 13 tender points in different areas including the neck, upper chest, inner aspects of the knee, upper thighs, and lateral epicondyles. ESR, CRP, and CK are normal. Which of
the following could be the most appropriate pharmacological treatment option?

A. Paracetamol​​.

B. Naproxen.

C. Amitriptyline.

D. Prednisolone.

E. Fluoxetine.

Incorrect. Correct answer is C


45% answered correctly

Explanation:

Correct Answer Is C

For this patient, fibromyalgia seems to be the best-fitting explanation for the clinical presentation. Fibromyalgia is characterized by widespread pain affecting all four body
quadrants, along with systemic symptoms like fatigue, sleep problems, and cognitive impairment. It is more common in females with a female-to-male ratio of 4:1. The usual age of
onset is between 29- 37 years, but it is often diagnosed by around 44 -53 years. There may be a family history.

Pain should be present for at least 3 months and there must be 11 out of 18 characteristic tender points for a diagnosis.

Fibromyalgia has multiple, varied, and fluctuating symptoms. It should be considered in a patient with chronic musculoskeletal pain, fatigue, and poor sleep. These symptoms are
usually accompanied by depression or anxiety, sensitivity to chemicals, irritable bowel, or restless legs. The symptoms often start or worsen during a period of severe psychosocial
or physical stress. Thorough examination and investigation are warranted to rule out other pathological causes of disease, especially in the presence of red flags which are as
follows:

Older age at new symptom onset


Weight loss
Night pain
Focal pain
Fever or sweats
Neurological features
History of malignancy

Management includes explanation and support. Stress management and CBT are beneficial. Concomitant depression should be treated. Of the given pharmacological treatment
options, amitriptyline seems to be the most beneficial for short-term pain management.

Tricyclic anti-depressants (TCAs) such as amitriptyline and serotonin-noradrenaline reuptake inhibitors (SNRIs) such as duloxetine seem to be of short-term value for pain control in
fibromyalgia. Low-dose TCAs or SNRIs elevate levels of serotonin and noradrenaline in the descending inhibitory nociceptive pathways of the CNS, providing a significant benefit in
fibromyalgia independent of effects on mood.

SSRIs such as fluoxetine (option E) can be used for the treatment of underlying or concomitant depression but are not as effective as TCAs and SNRIs for pain control and
symptomatic relief for fibromyalgia.

Paracetamol (option A) can be given initially but is not the most appropriate treatment. European League Against Rheumatism (EULAR) recommends the use of simple analgesics
like paracetamol; however, there is insufficient data to back this. Paracetamol use has not been studied in fibromyalgia patients, other than in combination with tramadol, where the
combination resulted in a modest (18%) improvement in pain.

NSAIDs such as naproxen (option B) are of no proven benefit, and steroids such as prednisolone (option D) are not indicated in the condition.

References

• Therapeutic Guidelines - Rheumatology

• Murtagh's General Practice - 8th Edition, p.778

• Medscape - Fibromyalgia

Last updated:
Time spent: QID:1774
2023-2-12

1209 of 1943
A 56-year-old woman presents with swelling in front of his right ear. He also complains of dry itchy eyes and a dry mouth. On examination, right parotid and xerostomia are
remarkable findings. Fine needle aspiration cytology (FNAC) is performed which is significant for lymphocytes without any clues of malignancy. Which diagnostic test would be the
best diagnostic test to confirm the diagnosis?

A. Salivary gland biopsy.

B. Schirmer test.

C. CT scan of the neck.

D. Ultrasound scan of the swelling.

E. ANA.

Correct
45% answered correctly

Explanation:

Correct Answer Is A

Sjӧgren syndrome is a chronic inflammatory disease characterized by lymphocytic infiltration and fibrosis of exocrine glands, especially lacrimal and salivary glands. The disease is
much more prevalent in women (female: male~ 9:1), with the usual age of onset being in the 4th and 5th decades of life.

Sjӧgren syndrome for which no underlying etiology can be identified is termed primary, whereas when it is secondary to other connective tissue disorders, it is called secondary
Sjögren syndrome. Autoimmune diseases associated with Sjӧgren syndrome include:

Rheumatoid arthritis
Systemic lupus erythematosus
Scleroderma

Considering the overlap of Sjӧgren syndrome with many other rheumatic disorders, it is sometimes difficult to determine whether a clinical manifestation is solely a result of Sjӧgren
syndrome or is due to one of the overlapping disorders.

Clinical features of the disease include the following:

Dry eyes (xerophthalmia) and keratoconjunctivitis sicca due to decreased tear production- the patient may complain of feeling as sand under the eyelid or itchy eyes.

Dry mouth (xerostomia) due to decreased salivation – dry mouth can lead to the following manifestations:

Red smooth and dry tongue


Severe and progressive dental caries
Cracks at the corners of the mouth
Chronic oral candidiasis

Parotid gland swelling

​NOTE - Dry eyes and mouth are the most common presenting features in adults, whereas, children often present with parotid gland enlargement.

Other clinical features of Sjӧgren disease include:

Nasal dryness – can result in discomfort and bleeding


Vaginal dryness – can result in dyspareunia, vaginitis, and pruritus
Myalgia and fatigue
Arthralgia or arthritis – similar to that of SLE with symmetrical involvement of small joints. arthritis is of non-erosive nature
Raynaud’s phenomenon
Recurrent miscarriages or stillbirths in women and a history of venous or arterial thrombosis related to the presence of antiphospholipid antibodies (e.g. lupus
anticoagulant or anticardiolipin antibodies)
Leukopenia, anemia
Lymphadenopathy
Non-Hodgkin lymphoma

Currently, a minor salivary gland biopsy is the best single test to establish a diagnosis of Sjögren syndrome. In this procedure, an incision is made on the inner lip, and some minor
salivary glands are removed for examination. In patients with a possible diagnosis of this disease but with severe extraglandular symptoms, a lip biopsy is often performed to firmly
establish the diagnosis of Sjögren syndrome.

While this is the most definitive test, performing it is not necessary from a clinical standpoint. Patients with Sjögren syndrome are essentially treated symptomatically and are
observed for the development of other rheumatic disorders or lymphoma. This can be initiated without performing a biopsy. If the diagnosis is in doubt, or a definitive diagnosis is
needed, a biopsy is the best test.

(Option B) Schirmer test shows decreased tear production. A strip of filter paper is put under the lower eyelid, and the distance along the paper that tears are absorbed is measured.
Less than 5mm in 5 minutes is considered positive. It is one of the tests routinely performed when Sjӧgren disease is suspected but it is not diagnostic.

(Options C and D) Imaging studies are not diagnostic; however, they may be sometimes needed, particularly with extraglandular manifestations of the disease.

( ) ( )
1210 of 1943
(Option E) ANA, particularly anti-Ro (SSA) and anti-La (SSB) may be positive, but again these could be elevated in other conditions as well.

References

• Medscape - Sjogren Syndrome

• Therapeutic Guidelines – Rheumatology

Last updated:
Time spent: QID:146
2023-2-12

1211 of 1943
A 66-year-old man presents to your practice with complaints of left-sided cheek swelling, joint pain, and fatigue for the past 2 months. He has a 20-year history of type II diabetes
mellitus. He also mentions eye and mouth dryness for the past 6 months. On examination, there is non-tender swelling of the left parotid gland, several dental carries, and reddened
eyes. Which one of the following is the most likely diagnosis?

A. Left salivary duct stone.

B. Sjӧgren syndrome.

C. Parotid gland carcinoma.

D. Chronic parotitis.

E. Behcet’s syndrome.

Incorrect. Correct answer is B


45% answered correctly

Explanation:

Correct Answer Is B

The clinical findings of dry mouth (xerostomia), dry eye (xerophthalmia), fatigue, and joint pain are strongly consistent with Sjӧgren’s syndrome as the most likely diagnosis.

Sjӧgren syndrome is a chronic inflammatory disease characterized by lymphocytic infiltration and fibrosis of exocrine glands, especially lacrimal and salivary glands. The disease is
much more prevalent in women (female: male~ 9:1), with the usual age of onset being in the 4th and 5th decades of life.

Sjӧgren syndrome for which no underlying etiology can be identified is termed primary, whereas when it is secondary to other connective tissue disorders, it is called secondary
Sjogren syndrome.

Considering the overlap of Sjӧgren syndrome with many other rheumatic disorders, it is sometimes difficult to determine whether a clinical manifestation is solely a result of Sjӧgren
syndrome or is due to one of its overlapping disorders.

Extra-glandular manifestations of Sjögren syndrome are common and include:

Cutaneous manifestations

About 50% of patients with Sjögren syndrome have cutaneous findings, such as dry skin (xeroderma), palpable and nonpalpable purpura, and/or urticaria. Blepharitis and
erythema annulare may be other features.

Gastrointestinal manifestations

Difficulty in swallowing (dysphagia) due to dryness of the pharynx and esophagus - Patients usually describe food becoming stuck in the upper throat.
Lack of saliva may lead to impaired acid clearance and may result in gastroesophageal reflux and esophagitis.
Abdominal pain and diarrhea can occur
Rarely, patients develop acute or chronic pancreatitis and malabsorption due to pancreatic insufficiency.
Patients with Sjögren syndrome are at increased risk for delayed gastric emptying, which can cause early satiety, upper abdominal discomfort, nausea, and vomiting.

Pulmonary manifestations

The dryness of tracheobronchial mucosa (xerotrachea) can manifest as a dry cough.


Less often, there might be mild dyspnea from trivial interstitial lung disease.
Recurrent bronchitis or even pneumonitis (infectious or noninfectious)

Cardiac manifestations

Pericarditis and pulmonary hypertension can occur in Sjögren syndrome.


Orthostatic symptoms related to dysfunction of autonomic control of blood pressure and heart rate are associated with increased severity of Sjögren syndrome.

Neurologic Manifestations

Different studies estimate the prevalence of CNS symptoms in those with Sjogren syndrome from 8 to 40%. When CNS manifestations are present, other possible causes
including concomitant SLE, multiple sclerosis, cerebrovascular disease, and Alzheimer's disease should be assessed and meticulously investigated.

Renal manifestations

The most common form of renal involvement in Sjögren syndrome is interstitial nephritis. Renal calculi, renal tubular acidosis, osteomalacia, nephrogenic diabetes
insipidus, and hypokalemia can occur secondary to tubular damage caused by interstitial nephritis,
Interstitial cystitis, with dysuria, frequency, urgency, and nocturia, is strongly associated with Sjögren syndrome.
Glomerulonephritis can be caused by Sjögren syndrome but is uncommon and is usually attributable to another disorder, such as SLE or mixed cryoglobulinemia.

Other possible manifestations

Nasal dryness – can result in discomfort and bleeding


Vaginal dryness – can result in dyspareunia, vaginitis, and pruritus

1212 of 1943
Myalgia and fatigue
Arthralgia or arthritis – similar to SLE with symmetrical involvement of small joints. arthritis is of non-erosive nature
Raynaud’s phenomenon
Recurrent miscarriages or stillbirths in women and a history of venous or arterial thrombosis related to the presence of antiphospholipid antibodies (e.g. lupus
anticoagulant or anticardiolipin antibodies)
Leukopenia, anemia
Lymphadenopathy
Non-Hodgkin lymphoma

(Options A, C, and D) Left salivary duct stone can lead to swelling of the left parotid gland, as can chronic parotitis and parotid gland carcinoma, but dry mouth, dry eye, and
arthralgia would be absent in these conditions.

(Option E) Behcet’s disease is a systemic vasculitis that presents mostly with mouth and genital ulcers as well as the involvement of other organs. Xerophthalmia and xerostomia
are not the features seen in Behcet’s disease. Chronic parotitis has a different clinical picture.

References

• Medscape - Sjogren Syndrome

• Therapeutic Guidelines – Rheumatology

Last updated:
Time spent: QID:147
2023-2-12

1213 of 1943
A 43-year-old woman complains of increasing sensation of foreign body in her eyes, eye itching, and dry mouth for the past 6 months. She also reveals that she has recently
developed a dry cough and pain in the hand joints. On examination, there is mild bilateral swelling of the parotid glands. The metacarpophalangeal (MCP) and proximal
interphalangeal (PIP) are tender to touch, but not warm or swollen. She denies any morning stiffness. Serologic tests are significant for a positive ANA and RF. ESR and CRP are
elevated. Urinalysis is normal. Which one of the following would be the most likely diagnosis?

A. Systemic lupus erythematosus.

B. Sjӧgren syndrome.

C. Bulimia nervosa.

D. Mumps.

E. Uveoparotid fever.

Incorrect. Correct answer is B


45% answered correctly

Explanation:

Correct Answer Is B

The presentation of dry eye (xerophthalmia), dry mouth (xerostomia), and parotid gland enlargement are pointers toward Sjӧgren syndrome as the most likely diagnosis.

Sjӧgren syndrome is a chronic inflammatory disease characterized by lymphocytic infiltration and fibrosis of exocrine glands, especially lacrimal and salivary glands. The disease is
much more prevalent in women (female: male~ 9:1), with the usual age of onset being in the 4th and 5th decades of life.

Sjӧgren syndrome for which no underlying etiology can be identified is termed primary, whereas when it is secondary to other connective tissue disorders, it is called secondary
Sjӧgren syndrome.

Considering the overlap of Sjӧgren syndrome with many other rheumatic disorders, it is sometimes difficult to determine whether a clinical manifestation is solely a result of Sjӧgren
syndrome or is due to one of its overlapping disorders.

Extra-glandular manifestations of Sjӧgren syndrome are common and include:

Cutaneous manifestations

About 50% of patients with Sjögren syndrome have cutaneous findings, such as dry skin (xeroderma), palpable and nonpalpable purpura, and/or urticaria. Blepharitis and
erythema annulare may be other features.

Gastrointestinal manifestations

Difficulty in swallowing (dysphagia) due to dryness of the pharynx and esophagus - patients usually describe food sticking in the throat.
Lack of saliva may lead to impaired clearance of acid and may result in gastroesophageal reflux and esophagitis.
Abdominal pain and diarrhea can occur
Rarely, patients develop acute or chronic pancreatitis and malabsorption due to pancreatic insufficiency
Patients with Sjögren syndrome are at increased risk for delayed gastric emptying, which can cause early satiety, upper abdominal discomfort, nausea, and vomiting.

Pulmonary manifestations

Dryness of tracheobronchial mucosa (xerotrachea) that can manifest as a dry cough


Less often, there might be mild dyspnea from a trivial interstitial lung disease
Recurrent bronchitis or even pneumonitis (infectious or noninfectious)

Cardiac manifestations

Pericarditis and pulmonary hypertension can occur in Sjögren syndrome


Orthostatic symptoms related to dysfunction of autonomic control of blood pressure and heart rate are associated with increased severity of Sjögren syndrome

Neurologic Manifestations

Different studies estimate the prevalence of CNS symptoms in those with Sjӧgren syndrome from 8 to 40%. When CNS manifestations are present, other possible causes
including concomitant SLE, multiple sclerosis, cerebrovascular disease, and Alzheimer's disease should be assessed and meticulously investigated.

Renal manifestations

The most common form of renal involvement in Sjögren syndrome is interstitial nephritis. Renal calculi, renal tubular acidosis, osteomalacia, nephrogenic diabetes
insipidus, and hypokalemia can occur secondary to tubular damage caused by interstitial nephritis,
Interstitial cystitis, with symptoms of dysuria, frequency, urgency, and nocturia is strongly associated with Sjögren syndrome.
Glomerulonephritis can be caused by Sjögren syndrome but is uncommon and is usually attributable to another disorder, such as SLE or mixed cryoglobulinemia.

Other possible manifestations

Nasal dryness – can result in discomfort and bleeding


Vaginal dryness – can result in dyspareunia, vaginitis, and pruritus

1214 of 1943
Myalgia and fatigue
Arthralgia or arthritis – similar to that of SLE with symmetrical involvement of small joints. arthritis is of non-erosive nature
Raynaud’s phenomenon
Recurrent miscarriages or stillbirths in women and a history of venous or arterial thrombosis related to the presence of antiphospholipid antibodies (e.g. lupus
anticoagulant or anticardiolipin antibodies)
Leukopenia, anemia
Lymphadenopathy
Non-Hodgkin lymphoma

Laboratory findings may include (but are not limited to) elevated ESR, positive CRP, positive RF, and positive ANA (Anti Ro and Anti La).

(Option A) SLE can be a possible cause of Sjögren syndrome, especially with the presence of features such as positive ANA and symmetrical involvement of hand joints, but even
so, the diagnosis would be Sjögren syndrome secondary to SLE because the dry mouth and dry eyes are not features commonly seen in SLE.

(Option C) Bulimia nervosa can cause bilateral parotid enlargement due to repetitive vomiting, but dry eye and respiratory signs would not be a feature. Furthermore, the history
lacks a stereotypical eating pattern of bulimia nervosa.

(Option D) Parotid gland enlargement and dry mouth due to decreased salivation are seen in mumps, but this patient's prolonged course of the disease makes this diagnosis very
unlikely.

(Option E) Uveoparotid fever or Heerfordt’s syndrome is a rare manifestation of sarcoidosis with the following presentation:

1. Uveitis
2. Parotiditis
3. Chronic fever
4. Palsy of facial nerves in some cases

With the absence of fever and uveitis in this patient, uveoparotid fever is very unlikely.

References

• Medscape - Sjogren syndrome

• Therapeutic Guidelines – Rheumatology

Last updated:
Time spent: QID:148
2023-2-12

1215 of 1943
Which of the following organism is the most common cause of urinary tract infections in children?

A. Proteus mirabilis.
B. E.coli.
C. Staphycoccis aureus.
D. Enterobacter.
E. Pseudomonas.

Incorrect. Correct answer is B


45% answered correctly

Explanation:

Correct Answer Is B

In children, between 75 to 90% of urinary tract infections are caused by E.coli. Other organisms may include
Klebsiella pneumonia, proteus mirabilis, staphylococci species, pseudomonas and enterobacters.

References

• Medscape - Pediatric Urinary Tract Infection

• RCH - Urinary tract infection

Last updated:
Time spent: QID:117 2023-2-12

1216 of 1943
A 15-year-old girl presents with an upper respiratory tract infection (URTI). On examination, she has a temperature of 38.1°C and an erythematous pharynx with no exudate. She is
otherwise healthy. A urine dipstick is positive for blood for which you order a urinalysis (UA). UA is reported back significant for red blood cells, 1+ proteinuria, no WBCs, and
negative for nitrite. Which one of the following would be the next best step in management?

A. Repeat UA after the URTI resolves.

B. Urine culture.

C. 24-hour urine exam.

D. Ultrasonography.

E. Start her on corticosteroids.

Correct
45% answered correctly

Explanation:

Correct Answer Is A

Asymptomatic hematuria in children, in the setting of a febrile illness other than urinary tract infection, should be assessed after the febrile illness subsides; provided that the illness
is not due to urinary tract infection (UTI). With UTI, the next step would be a urine culture. This patient neither has urinary symptoms, nor laboratory findings consistent with UTI;
therefore, no urine culture is indicated.

Asymptomatic microscopic hematuria should always be confirmed with 2 positive urine exams out of 3 in 2-3 weeks before any further assessment is considered. However, in the
presence of symptoms such as hypertension, edema, or renal failure, prompt action is mandated.

1217 of 1943
A 5-year-old boy is brought to the Emergency Department by ambulance after he had a generalized tonic-clonic
seizure at home 20 minutes ago. Upon arrival to the hospital, he was assessed immediately. He is not having a
seizure now but is lethargic and confused. His blood pressure is 105/65 mmHg, pulse rate 76 bpm, respiratory rate
16 breaths/min, and temperature 37°C. His mucous membranes are not dry, skin turgor is normal, and capillary refill
time is 2 seconds. The rest of the examination, including cardiovascular, respiratory and neurological exam is
completely normal. Pathology results are as following:

FBE: Normal
Random blood sugar: 8.3 mmol/L (4 - 11.1 mmol/L)
Sodium: 120 mmol/L (135-145 mmol/L)
Potassium: 4.1 mmol/L (3.5 – 5.5 mmol/L)
Bicarbonate: 24 mmol/L (22-32 mmol/L)
Creatinine: 80 µmol/L (60-110 µmol/L)
Urea: 4.5 mmol/L (2.5-7.1 mmol/L)
Calcium: 3.1 mmol/L (2.2-2.7 mmol/L)

Which one of the following could be the most likely cause to this presentation?

A. Acute renal failure.


B. Addison disease.
C. Congestive heart failure.
D. Hyponatremia due to dehydration.
E. SIADH syndrome.

Incorrect. Correct answer is E


45% answered correctly

Explanation:

Correct Answer Is E

Seizure can be caused by a variety of causes including:

Metabolic derangements such as hyponatremia, hypernatremia, hypocalcemia, and hypoglycemia


Structural abnormalities such as space occupying lesions in the brain
CNS Infections such as meningitis
Medication intoxication or withdrawal
Epilepsy

There are a few clues in the history, physical examination, and laboratory findings that point towards hyponatremia
as the most likely explanation for this presentation. Hyponatremia (and hypernatremia) present with CNS
manifestations including lethargy, headache, alteration in consciousness, and in worse cases, seizure, coma, or even
death. He also has hypercalcemia. But it is it is uncommon for hypercalcemia to cause seizures while hypocalcemia
is notorious for that.

All the given options can potentiate and cause hyponatremia; hence, such presentation. However, this child has no
signs of dehydration, and this excludes hyponatremia due to dehydration (option D) as a cause. Also, there is no
history of congestive heart failure (option C) nor any clinical findings such as abnormal cardiovascular exam
findings, volume overload, etc. to support such diagnosis.

Acute renal failure (option A) can also cause hyponatremia but the normal urea and creatinine rules out such
diagnosis.

Addison disease (option B) can be another explanation for hyponatremia in general but not as a likely diagnosis in
this scenario. In Addison disease, adrenal insufficiency and lack of aldosterone result in hyponatremia and
hyperkalemia at the same time. This child has a normal potassium. Furthermore, hypotension is one of the main
1218 of 1943
clinical findings in patients with Addison disease. This child has normal blood pressure. Collectively, these make
Addison disease a very remote possibility.

Of the options, SIADH is most likely to have resulted in such presentation. SIADH (syndrome of inappropriate anti-
diuretic hormone) is caused by excess amount of anti-diuretic hormone (ADH) from the hypothalamus-pituitary unit.
ADH affects kidneys to concentrates urine and retain water. In SIADH, excess ADH results in a very concentrated
urine, increased body water, and hyponatremia through dilutional effect.

Depending on the rate at which hyponatremia occurs, patients with SIADH can be asymptomatic or have classic
symptoms of hyponatremia including the following:

Nausea and vomiting


Headache
Problems with balance that may result in falls
Mental changes, such as confusion, memory problems, strange behavior
Seizures or coma, in severe cases

Common causes of SIADH are:

Medicines, such as certain type 2 diabetes medications, anti-epileptic medications, antidepressants, heart and
blood pressure drugs, cancer drugs, drugs used for anesthesia
Surgery under general anesthesia
Disorders of the brain, such as injury, infections, stroke
Brain surgery in the region of the hypothalamus
Lung disease, such as pneumonia, tuberculosis, cancer, chronic infections
Cancer of the lung, small intestine, pancreas, brain, leukemia

References

• RACGP – AFP – The suspect: SIADH

Time spent: QID:1648 Last updated:


2023-2-12

1219 of 1943
Concerned parents of a 6-year-old girl has brought her to your GP clinic as they have noticed scattered growth of
pubic hair in their daughter recently. On examination, she has sparse pubic hair growth and Tanner I breast. Her
height and weight are on 75% and 50% percentiles respectively. Which one of the following is the most appropriate
next step in management?

A. Ultrasounds scan of the pelvis.


B. Assessment of the bone age.
C. FSH and LH.
D. Review in 3 months.
E. 17-hydroxyprogesterone and estradiol.

Incorrect. Correct answer is D


45% answered correctly

Explanation:

Correct Answer Is D

The scenario represents development of pubic hair before the age of 8 years in girls (or 9 in boys) in the absence
other sexual characteristics which are thelarche (breast development) and menarche (periods), which is referred to
as premature isolated adrenarche. Adrenarche is the maturation of the adrenal zona reticularis in both boys and girls,
resulting in the development of pubic hair, axillary hair, and adult apocrine body odor. Premature isolated adrenarche
is considered a benign variant of normal development if the weight and height are normal for the age.

No investigations are required for premature isolated adrenarche as FSH and LH (option C), 17-hydroxyprogesterone
and estradiol (option E), and testosterone all normal. The only laboratory finding might be slight elevation of
dehydroepiandrosterone sulfate (DHEAS). It is recommended that children with premature isolated adrenarche are
followed every 3-6 months for surveillance of puberty development and linear progression and velocity of growth.

Bone age assessment (option B) is considered in constitutional growth delay (CGD) alongside exclusion of other
causes of delayed puberty/growth to confirm the diagnosis or where the child is small for age. This child has normal
weight and height for age.

Ultrasound of the pelvis (option A) is the study of choice for girls whose periods are overdue despite development of
other secondary sexual characteristics, normal body measures, and absence of features suggesting chromosomal
abnormalities such as Turner syndrome.

NOTE– puberty starts with thelarche. A Tanner II breast before the age of 13 years suggests estrogen exposure and
commencement of puberty in girls. This is equivalent to testicular enlargement in boys. A Tanner II or less after 13
years is abnormal and indicated absence of breast development.

References

• American Family Physicians: Disorders of Puberty: An Approach to Diagnosis and Management

• RACGP – AFP – Growth disorders in children

Last updated:
Time spent: QID:1672 2023-2-12

1220 of 1943
Concerned parents of a 5-year-old boy have brough him to the Emergency department because he has been having
palpitations and dizziness since this morning. He is otherwise healthy with no significant medical history, and this is
the first time he is experiencing it. An ECG as well as blood tests are arranged. On examination, he has a weak
thready pulse which is so rapid that cannot be counted correctly. His blood pressure is 98/60 mmHg. He seems
restless but does not seem to be breathless. Extremities are of normal temperature and color. A 12-lead ECG is
obtained and is shown in the following photograph. Which one of the following is the most appropriate next step in
management of him?

A. Immersion of face in cold water.


B. Intravenous adenosine.
C. Intravenous amiodarone.
D. Synchronized cardioversion.
E. Intravenous esmolol.

Incorrect. Correct answer is C


45% answered correctly

Explanation:

Correct Answer Is C

The ECG shows wide-QRS complex tachycardia (WCT) at a rate of approximately 300 bmp. Most WCTs in children
are in fact supraventricular tachycardias (SVT) with aberrancy. Compared to SVT with aberrancy, ventricular
tachycardia (VT) is rare in children; however, due to seriousness of the condition, Australian Advanced Paediatric Life
Support (APLS) and Royal Children Hospital (RCH) guidelines recommend that WCTs are considered and treated as
VT until proven otherwise. This approach is different from recommendations by American Heart Association (AHA)
that advise to treat WCTs in a hemodynamically stable children as SVT first by a trial of intravenous adenosine
(option B) before making a diagnosis of VT.

APLS recommends intravenous amiodarone under close monitoring for treatment of symptomatic VT in children
with stable hemodynamics, and synchronized DC cardioversion (option D) in those with hemodynamic instability/
shock if there is pulse and defibrillation in the absence of a pulse.

This child is hemodynamically stable; therefore, intravenous amiodarone will be the next best step in management.

Vagal maneuvers such as immersion of the face in cold water (option A), application of ice pack to the face or
Valsalva maneuvers are initial treatment for patients with narrow-QRS complex tachycardia i.e., SVT. Vagal
maneuvers suppress the AV node and might worsen the condition in VT.

Intravenous beta blockers such as esmolol (option E) are not generally recommended for treatment of VT or SVT in
children.

References

1221 of 1943
• Advanced Paediatric Life support (APLS) – Algorithms – Ventricular Tachycardias

• Medscape – Pediatric Ventricular Tachycardia Overview of Ventricular Arrhythmias


Last updated:
Time spent: QID:1710 2023-2-12

1222 of 1943
Emad is 6 years old and is being rushed to the Emergency Department by an ambulance after he experienced light-
headedness, shortness of breath, chest tightness, and anxiousness at home 15 minutes ago. When you visit him, he
is lying on a stretched and is receiving oxygen by non-re-breathable face mask. He is mumbling and does not seem
to be oriented to time and place however he seems to recognize his parents. Examination reveals a very rapid barely
perceptible pulse and blood pressure of 80/45 mmHg. Cardiac monitoring shows the following rhythm. Which one of
the following is the most appropriate immediate management in this situation?

A. Defibrillation.
B. Synchronized DC cardioversion.
C. Intravenous amiodarone.
D. Intravenous lignocaine.
E. Intravenous adenosine.

Incorrect. Correct answer is B


45% answered correctly

Explanation:

Correct Answer Is B

Emad’s ECG shows tachycardia of approximately 300 bpm with regular wide QRS complexes (wide-QRS complex
tachycardia [WCT]). In children, ventricular tachycardia (VT) is very rare, and most WTCs are in fact supraventricular
tachycardia with conduction aberrancy (SVT with aberrancy). However, due to seriousness of VT compared to SVT,
guidelines such Advanced Paediatric Life Support (APLS) as those by the Royal Children Hospital (RCH) recommend
that WCTs should be considered and treated as VT until prove otherwise.

As such, Emad’s ECG should be considered and treated as VT. Treatment of VT with a pulse is determined by the
presence or absence of shock/hemodynamic instability (hypotension, hypoperfusion, altered mental status)

For every patient presenting with hemodynamic instability caused by a cardiac tachyarrhythmia, DC cardioversion
should always be considered as the most appropriate treatment option (defibrillation for ventricular fibrillation and
pulseless VT; synchronized for the rest.)

Emad has an ECG with WCT in addition to hypotension and hypoperfusion evident by altered mental status
(disorientation to time and place) and cold pale sweaty extremities. Of the options, synchronized DC cardioversion is
the next step in treatment to consider for him.

Defibrillation (unsynchronized) (option A) is used as first line for treatment of ventricular fibrillation (VF) and
pulseless VT.

If Emad was not hemodynamically unstable, intravenous amiodarone (option C) would be considered first.
Amiodarone is the medication of choice for children presenting with symptomatic wide QRS tachycardia who are not
in shock.

Intravenous amiodarone (class III antiarrhythmic) is almost always the medication of choice for treatment of
symptomatic VT in stable infants and young children. The medication has a broad efficacy and is readily available.
However, major adverse reactions have always been concerning.

1223 of 1943
Lignocaine (lidocaine) (option D) is a class I (sodium channel blocker) antiarrhythmic medication. It is no longer
recommended for treatment of VT in children.

NOTE - American Heart Association recommends a trial of intravenous adenosine (option E) in symptomatic yet
stable children with WCT because most of such WTCs are in fact SVT with aberrancy. This way, the child is not
unnecessarily exposed to adverse effects of amiodarone or other antiarrhythmics. The Australian Advanced
Paediatric Life Support (APLS) and Royal Children Hospital (RCH) guidelines on the other hand endorse
considering and treating WCTs as VT rather than SVT until proven otherwise. In any case, Emad is unstable, and
adenosine has no role in his management based on either of those sets of guidelines.

References

• APSL – Ventricular tachycardia algorithm

Time spent: QID:1714 Last updated:


2023-2-12

1224 of 1943
Parents of a 4-year-old girl has brough her to the Emergency Department after she started feeling short of breath and
dizzy half an hour ago at home. Parents deny any other previous similar episodes or any significant recent or past
medical conditions. On examination, a very rapid thready and hardly perceptible pulse is felt. She has a blood
pressure of 98/65 mmHg and respiratory rate of 35 breaths per minute. He is afebrile and has normal peripheral
perfusion. You start oxygen via non-rebreathable face mask, put him on cardiac monitoring, and stablish an
intravenous line. Cardiac monitoring shows the following rhythm. Which one of the following is the most appropriate
next step in management?

A. Put an icepack on his face.


B. Immerse his head in cold water.
C. Intravenous adenosine.
D. Intravenous verapamil.
E. Synchronized DC cardioversion.

Correct
45% answered correctly

Explanation:

Correct Answer Is A

The photograph is typical for supraventricular tachycardia (SVT) at a rate of approximately 170 bpm. SVT is an
umbrella term used to describe tachycardias (atrial and/or ventricular rates above 100 bpm at rest) that originates
from the His bundle or above. These SVTs include inappropriate sinus tachycardia, atrial tachycardia (AT) including
focal and multifocal AT, macro-reentrant AT (including typical atrial flutter), junctional tachycardia, atrioventricular
reentrant tachycardia (AVRT), atrioventricular nodal reentrant tachycardia (AVNRT), and various forms of accessory
pathway-mediated reentrant tachycardias e.g., Wolff-Parkinson, White syndrome (WPW).

In younger children, SVT is often caused by AVRT, including WPW while in adolescents it is caused by AVNRT. Other
causes are very rapid sinus tachycardia, atrial flutter, ectopic atrial tachycardia, and junctional ectopic tachycardia.

SVT typically has a fixed rate, usually >220 bpm. Onset and offset are abrupt, and p-waves are either not visible or
seen after the QRS complexes. Conditions such as sepsis, pain, dehydration, anxiety, and fever can contribute to
development of SVT and should always be considered and addressed in management of children with a
tachyarrhythmia

NOTE- SVT typically causes narrow complex tachycardia; however, in SVT with aberrancy, QRS complexes could be
wide, resembling ventricular tachycardia (VT). Although the majority (95%) of wide-QRS complex tachycardias in
children are SVT with aberrancy, sustained wide-QRS complex tachycardia should always be considered and treated
as VT until proven otherwise (RCH recommendation).

1225 of 1943
Symptomatic SVT often presents with palpitations, shortness of breath, dizziness, and decreased exercise tolerance.
If protracted, signs and symptoms of congestive heart failure can develop.

Management of children (and adults) with SVT depends on the presentation. For all patients, oxygen, securing
intravenous access, cardiac monitoring, and an a 12-lead ECG should are the initial steps to take. The rest of the
management is determined by the presence or absence of indicators of hemodynamic instability which are
hypotension, hypoperfusion, or hypoperfusion-related altered mental status.

In the absence of hemodynamic instability, vagal stimulation maneuvers are tried first:

Neonates and infants (<6 months):

Immersion of the face in ice water for 5 seconds to elicit diving reflex (not to be use for those with hemodynamic
instability/shock). While attached to cardiac monitor and arms are wrapped in a towel, immerse the whole face in ice
water for five seconds. It is unnecessary to occlude the nostrils. This technique is safe and 90% effective in
terminating SVT.

Older infants/ toddlers:

Apply an ice-cold facecloth or bag filled with ice to the face for 15-30 seconds (elicits the ‘dive reflex’). It is less
effective than facial immersion but better tolerated in older infants.

School-aged children:

Valsalva technique: with the child in the supine position, ask them to blow on their thumb or a 10 mL syringe, after
full inspiration, for 10-15 seconds. There should be no air escape and the child should be seen to strain. In modified
Valsalva, the same manoeuvre is performed in a semi-recumbent position. Then at the end of the strain, the child is
immediately repositioned into a supine position with passive leg raise to 45 degrees.

This child has symptomatic SVT but is hemodynamically stable. For her, vagal maneuvers are likely to end the SVT
and should be tried first. At this age group, putting an ice pack or ice-cold towel/cloth on the face for 15-30 seconds
is the preferred method.

Immersion of face in ice water (option B) for five seconds is more effective than placing ice pack on the face.
However, it is less likely to be tolerated by children older than 6 months and is not recommended for this age group
as the preferred method despite higher efficacy.

Intravenous adenosine (option C) is the preferred medication for terminating SVT in children and adults, but it should
be considered if vagal maneuvers fail to work.

Despite the increasing use of the calcium-channel blocker verapamil (option D) for treatment of SVT in adults, it is
not routinely used for such purpose in children. It is also contraindicated in children less than 12 months because it
can cause irreversible hypotension and fatal ventricular dysfunction in this age group.

Synchronized DC cardioversion (option E) would be the correct answer if the child was in hemodynamic
instability/shock. However, some guidelines including Advances Paediatric Life Support (APLS) recommend vagal
maneuvers are tried while preparation for DC cardioversion is made, provided it does not cause any delay in
delivering the latter.

NOTE - American Heart Association (AHA) recommends that based on rarity of ventricular tachycardias (VT) in
children, wide-complex QRS tachycardias with a pulse be considered and treated as supraventricular tachycardia
(SVT) with aberrancy if the QRS complex are regular and a pulse is present. Based on recommendations by the
AHA (and also UpToDate) a trial of adenosine in such cases should be considered first for children who are
hemodynamically stable, both as therapeutic and diagnostic measures (cessation of tachycardia in response to
adenosine favors SVT with aberrancy rather than VT). With no response to adenosine, the tachycardia should be
treated as VT (e.g., with IV amiodarone).

References

1226 of 1943
• RCH – Supraventricular tachycardia (SVT)

• APLS – Supraventricular tachycardia algorithm


Last updated:
Time spent: QID:1716 2023-2-12

1227 of 1943
Sarah has brought Jesse, her 4-year-old son, Jesse, because of bedwetting. Jesse is her first child and is the
outcome of an uncomplicated pregnancy with good health so far. He mingles with his peers and has met all
milestones otherwise. He is very attached to his Teddy bear and will not sleep without it. He also has some
imaginary friends to whom he talks sometimes. He is dry during the daytime but wets the bed a few times per
month. Physical examination is normal. Which one of the following is the most likely diagnosis?

A. Autism.
B. ADHD.
C. Psychosis.
D. Normal development.
E. Anxiety.

Incorrect. Correct answer is D


45% answered correctly

Explanation:

Correct Answer Is D

Bedwetting happens when the bladder empties involuntarily during sleep, most children will gain daytime urinary
continence by the age of 3 years, and night continence by the age of 6 years. Usually, no treatment is required for
night-time bed wetting by the age of 6 years as there is a high rate of spontaneous resolution. Also, attachment to
toys or objects such as the Teddy bear in this scenario is completely normal up to the age of 7 years and is a healthy
form of play for children.

Jesse is 4 years old and otherwise normal. Sarah should be advised that for now, it can be normal to wet the bed a
few nights in a month and that his talking to imaginary friends and attachment to his toys are not concerning at this
age and there is no need to be alarmed.

Autism (option A) presents with communication problems such as finding friends and playing with peers and
repetitive ritualistic behaviors, none of which are present in Jesse.

ADHD (option B) presents with poor concentration and impulse control. The main clinical presentation of ADHD
includes:

Inattention: difficulty concentrating, forgetting instructions, moving from one task to the other without
completing the previous one
Impulsivity: acting without thinking, talking over the top of others, losing control of emotions easily, and being
accident prone
Overactivity: constant fidgeting and restlessness

None of the above is present in Jesse.

Having imaginary friends and talking to or playing with them is quite normal in children and part of their social
development; therefore, a diagnosis of psychosis (option C) cannot be based on that unless other features are
psychosis are present e.g., hallucinations, delusions, thought process problems, apathy, etc.

Anxiety (option E) presents with affective symptoms such as subjective discomfort, erratic concentration, or
hypervigilance or somatic manifestations such as tachycardia, hyperventilation, nausea, vomiting, urinary frequency,
or so on. Jesse doe does not seem to have any signs or symptoms suggestive of anxiety.

References

• RCH – Enuresis: Bed wetting and Monosymptomatic Enuresis

1228 of 1943
Last updated:
Time spent: QID:1744 2023-2-12

1229 of 1943
A 35-year-old woman gives birth to her second male baby through an uneventful vaginal delivery. The baby is preterm
at 34 weeks and weighs 3100 gr. Apgar's scores at the first and 5th minutes are 6 and 9, respectively. Almost one
hour after birth, he develops tachypnea and grunting as well as intercostal recession. He is placed under the oxygen
hood. In the next 4 hours, his condition remarkably improves. Which one of the following could be the most likely
diagnosis?

A. Meconium aspiration.
B. Birth asphyxia.
C. Transient tachypnea of the newborn.
D. Respiratory distress syndrome.
E. Tension pneumothorax.

Incorrect. Correct answer is C


45% answered correctly

Explanation:

Correct Answer Is C

The scenario represents early-onset respiratory distress in a newborn with a near normal and normal APGAR score
of 6 and 9 at 1 and 5 minutes. These scores exclude birth asphyxia (option B), which is defined as failure to establish
breathing at birth. This baby started breathing right after birth but developed respiratory problems shortly after.

Of the other options, transient tachypnea of the newborn (TTN) and respiratory distress syndrome (RDS) (option D)
seem more likely compared to others.

Transient tachypnea of the newborn (TTN) is a benign, self-limiting condition that can present in infants of any
gestational age shortly after birth. The pathophysiology is a delay in the clearance of fetal lung fluid after birth, which
leads to ineffective gas exchange, respiratory distress, and tachypnea. RDS on the other hand is caused by
surfactant deficiency in preterm newborns.

Both TTN and RDS present with the following:

Onset within the first few minutes to hours after birth.


Physical exam findings usually include signs of respiratory distress:

Tachypnea (respiratory rate greater than 60 per minute)


Nasal flaring
Grunting
Intercostal/subcostal/suprasternal retractions

The distinctive feature between these two is the clinical course. While TTN improves with supportive treatment
usually within 6 hours of onset, RDS worsens over 48-72 hours before it improves. The fact that this newborn has
made a remarkable recovery in 4 hours makes TTN more likely of a diagnosis compared to RDS.

Meconium aspiration syndrome (option A) is a common cause of early-onset neonatal respiratory distress in near-
term, term or post-term neonates. One important clue to such diagnosis is the presence of meconium on the
neonate’s body (meconium-stained baby) or in the amniotic fluid. Prematurity and absence of any clues on the
presence of meconium in the scenario are less likely.

Tension pneumothorax (option D) presents with acute onset respiratory distress. Diagnostic clues include
tachycardia, tachypnea, hypotension, asymmetrical respiratory movements, absence of breath sounds, and hyper-
resonance of the affected side and engorged neck and forehead veins. As none is mentioned in the scenario, it is
less likely of a diagnosis.

1230 of 1943
References

• Star Pearls – Transient Tachypnea of the Newborn

• Safe Care Victoria – Respiratory Distress Syndrome (RDS) in Neonates

• Safe Care Victoria – Meconium Aspiration Syndrome

• RCH – Recognition of the seriously unwell neonate and young infant

• AAFP - Newborn Respiratory Distress

Last updated:
Time spent: QID:1754 2023-2-12

1231 of 1943
A 9-year-old girl was brought to your practice two weeks ago with a fever. Your assessment established an upper
respiratory tract infection (URTI) as the diagnosis; however, you were also concerned about 3+ blood and 1+
proteinuria on a dipstick urine exam; therefore, you advised a follow-up in two weeks. Today, she is back for it and
dipstick urine shows 2+ blood and no proteinuria. You arrange for a formal urinalysis which comes back negative for
glomerular red blood cells and casts. Which one of the following is the most important next step in management?

A. Urine culture.
B. DMSA.
C. Ultrasound.
D. IVP.
E. ASOT.

Correct
45% answered correctly

Explanation:

Correct Answer Is A

Based on current guidelines by the Royal Children’s Hospital (RCH), asymptomatic isolated hematuria in a child with
a febrile disease other than urinary tract infection should be followed up once the febrile disease settles. Based on
this guideline, this child has undergone another urine exam in 2 weeks which still shows isolated hematuria of non-
glomerular origin.

For this child, a urine culture/sensitivity to exclude urinary tract infection is the most appropriate next step in
management.

DMSA scan (DMSA, or dimercaptosuccinic acid, bound to technetium 99m) (option B) isotope is circulated through
the body. 95% of the radiopharmaceutical is bound to the renal cortex and the remaining 5% is usually excreted into
the urine. Modern gamma cameras capable of performing single-photon emission tomography (SPECT), are able to
reconstruct 3-dimensional images of the kidneys. In children, the accepted indications for a DMSA scan include renal
scars as a result of an insult (usually recurrent urinary tract infections) or reflux-associated nephropathy.

Ultrasound (option C) has a place in the diagnosis of renal morphology and assessing renal damage, especially in
the setting of hematuria, edema, and hypertension where underlying nephritis is suspected.

IVP (Intravenous Pyelography) (option D) is used to check urine flow if there is concern regarding kidney, ureter, or
bladder blockage, or to check the function and appearance of the urinary tract after surgery to prevent blockages
from causing permanent damage to the kidneys.

ASOT (Anti-streptolysin O Titer) (option E) is used in the case of post-streptococcal GN in a school-aged child with
macroscopic hematuria, edema due to fluid retention, and possibly a headache due to hypertension. Investigation
shows elevation of urea and creatinine, normocytic anemia, elevated streptococcal markers (ASOT/anti-DNase B),
and a depressed complement (C3) level.

TOPIC REVIEW

Pediatric hematuria can be divided into the following categories:

Microscopic hematuria - (>10 RBC/microlitre) can only be detected by urinalysis as urine color remains
normal
Macroscopic hematuria - visible blood in urine without microscopy, and is more likely to come from the
bladder or urethra rather than the kidney
1232 of 1943
Persistent microscopic hematuria – three positive separate samples for blood, each taken at least one week
apart, without prior exercise nor during menstruation
Isolated asymptomatic microscopic hematuria - the presence of microscopic hematuria without clinical
symptoms or any other abnormalities in the urine such as proteinuria

Assessment of pediatric hematuria should include:

Previous history of hematuria


Symptoms of urinary infection such as dysuria, frequency, pain, fever
Systemic symptoms such as fatigue, edema, rash, arthralgia, or coryza
Recent surgery or trauma including non-accidental injury
Family history of hematuria
History of underlying bleeding disorder or immunodeficiency
Medication history
Food intake (beetroot and berries can color urine to pink or red)
Recent exercise, especially if strenuous

Causes of hematuria in the pediatric population:

Common
Urinary tract infection
Perineal irritation
Trauma
Meatal stenosis with ulceration
Uncommon
Glomerulonephritis
Renal calculi
Coagulation abnormalities
Tumors

Investigation of macroscopic hematuria

Urine Dipstick urinalysis


Mid-stream urine Microscopy, culture, and sensitivity (MSU MCS)
Calcium/creatinine ratio
Blood Electrolytes including creatinine, full blood count, Antistreptiterin titer (ASOT)/anti-DNase B
Complement (C3)
Renal ultrasound

The following algorithm provides an outline for the assessment of microscopic hematuria in children:

References

• RCH - Haematuria

Time spent: QID:1760 Last updated:


2023-2-12

1233 of 1943
Tom, 5 years old, is in the emergency department with a swollen face, ankles, and a distended abdomen. Dipstick
urine is positive for blood (trace) and protein (+++). He is hemodynamically stable. Which one of the following is the
most appropriate next investigation to consider for him?

A. Urine culture.
B. Urinary Protein to Creatinine Ratio.
C. Urine cytology.
D. Blood chemistry.
E. Renal ultrasound.

Incorrect. Correct answer is B


45% answered correctly

Explanation:

Correct Answer Is B

With proteinuria and edema, Tom must be fully assessed for renal disease and probably nephrotic syndrome.

Proteinuria is likely to present either as a symptomatic disease (i.e., nephrotic syndrome), an incidental finding
during an assessment of either renal or nonrenal symptoms or detected on urinary screening in asymptomatic
patients.

Nephrotic syndrome is defined by the occurrence of generalized edema, heavy proteinuria (+++ or ++++ on dipstick),
hypoalbuminemia, and hypercholesterolemia. Nephrotic syndrome occurs most commonly in young children, with
peak incidence for the initial episode at 2 years of age.

Children with steroid-responsive nephrotic syndrome usually have no nephritic features such as hematuria,
hypertension, or raised serum creatinine. However, microscopic hematuria is occasionally seen (15–30%).

Most children will turn out to have minimal change disease (80%) or focal segmental GN (5–10%), while mesangial
proliferative glomerulonephritis (GN), membranoproliferative GN, and membranous GN are all rare.

In as many as 30–50% of children, proteinuria is transient and resolves over 1–2 weeks. Transient proteinuria is
common during febrile illnesses and can occur with strenuous exercise, emotional stress, and following seizures or
abdominal surgery.

Isolated proteinuria resolves spontaneously after the cessation of the causal factor and an extensive workup is not
necessary. Isolated proteinuria is common in children while persistent proteinuria is much less common.

Evaluation of proteinuria should begin with a careful history and thorough physical examination, urine microscopic
examination, and determination of the amount of protein excretion rate (PER). The PER has been traditionally
measured using 24-hour urine collections. However, the collection of 24-h urine is often cumbersome, and spot
urinary protein-to-creatinine ratio (PCR), expressed in g/g or mg/mg, has become a simple and attractive yet reliable
alternative. A spot urine PCR has been found to have a significant linear correlation with a 24-h urine PCR. because
the PCR compares urinary protein concentration with urinary creatinine concentration, urinary dilution or
concentration does not influence this value.

In young children, accurately timed collections are difficult to obtain and the protein/creatinine ratio (PCR) on an
untimed urine specimen has been the accepted standard for many years. A random urine specimen is acceptable.

(Option A) Urine microscopy and culture (Urine MCS) is considered if the patient is febrile

1234 of 1943
(Option C) Urine cytology is considered when the glomerular or non-glomerular origin or under debate and the
presence of urinary casts provides a clue

(Options D & E) Blood chemistry and renal ultrasound are considered the next steps in order to look for an underlying
disease but not the next step.

References

• RCH -Proteinuria

• RCH - Nephrotic syndrome

Time spent: QID:1762 Last updated:


2023-2-12

1235 of 1943
The parents of John, 6 years of age, have brought him to your GP practice due to being bullied in school. When you
start talking to him by asking how he is feeling today, he says anxiously “I I I…wan ‘n ‘n ttt to ttoo go o home”. He is
otherwise normal with no remarkable physical findings. According to the parents, he has met developmental
milestones otherwise. Which one of the following is the most important initial step in management?

A. Referring the child to a speech therapist.


B. Referring the patient to a pediatric psychiatrist.
C. Telling the child to speak slowly.
D. Enforcing reward and punishment strategy.
E. Arranging for school observation.

Correct
45% answered correctly

Explanation:

Correct Answer Is A

Stuttering or stammering is a speech disorder characterized by interruptions to speech such as hesitating, repeating
sounds and words, or prolonging sounds. The cause is unknown, but genetics may be a factor. Anxiety and stress do
not cause stuttering but can make it worse.

About 1 in 100 Australians stutter. The condition can affect children, adolescents, and adults. It usually starts in
childhood, between the ages of around 2 and 4 years, although it can also start later. Stuttering can start overnight,
or it can build up over time.

Up to 1 in 12 three-year-old children stutter, but approximately three-quarters of them will recover without any
treatment – although it might take a few years. Persisting stutter beyond adolescence is unlikely to spontaneously
recover.

It is important that any child who stutters is promptly referred to a speech therapist/pathologist for appropriate
assessment. The earlier a child starts treatment, the better the prognosis. The best evidence from clinical trials for
treating children who stutter is the Lidcombe Program of Early Stuttering Intervention. This is a behavior modification
treatment. The main principles involve praising a child when words are spoken clearly, and occasionally noting when
stuttering has occurred.

A psychiatrist referral (option B) is not the appropriate initial step if the child appears to have no developmental
delays otherwise.

Asking the child to speak slowly (option C) is not the appropriate response or treatment.

A reward and punishment strategy (option D) is also not appropriate, because punishing the child may induce further
stress and cause even more stuttering (D is incorrect).

School observation (option E) is not indicated at this stage since the child has already presented to your clinic with
the condition.

References

• The Sydney Children’s Hospital Network

• Better Health Victoria

Last updated:
Time spent: QID:1770 2023-2-12

1236 of 1943
A mother brings her 10-month old boy to the Emergency Department with fever, nausea and vomiting. She mentions
that she has used fewer nappies in the past 48 hours because he has been less wet. She also mentions that his urine
has an offensive odor. A urine analysis is perfomed that is positive for nitrite and leukocytes consistent with urinary
tract infection (UTI). This child has never had an UTI before. Which one of the following is the next best step in
management?

A. Antibiotics and ultrasound of kidneys, ureter and bladder.


B. Antibiotics, intravenous fluids and ultrasound scan of the kidneys, ureter and bladder.
C. Antibiotics, oral fluids and discharge home once the child improves.
D. Refer to a pediatrician.
E. Antibiotics.

Incorrect. Correct answer is B


45% answered correctly

Explanation:

Correct Answer Is B

The scenario describes a baby who has developed dehydration (decreased urine output) as a result of vomiting in
the background of UTI. This child needs rehydration as the most appropriate initial management.

Dehydration is an indication for hospital admission and fluid resuscitation. Antibiotics should also be started for
treatment of UTI.

In the following conditions, ultrasound scan of the kidney, ureter and bladder is always indicated:

Concurrent bacteremia
Atypical UTI organisms: i.e., Staphylococcus aureus, Pseudomonas
UTI <3 years old
No/inadequate response to 48 hrs of IV antibiotics
Abdominal mass
Abnormal voiding (e.g., dribbling)
Recurrent UTI
First febrile UTI and no prompt follow up assured
Renal impairment
Significant electrolyte derangement
No antenatal renal tract imaging in second to third trimester

Since this child is younger than 12 months age, he needs an ulrasound as well. It is best to perform the ultrasound
within the first 3 days of presentation. If the child is younger than 1 year, either ultrasound or micturating
cystoureterogram or both can be used to exclude vesicoureteric reflux (VUR). If the child is older than 1 year,
ultrasound is adequate. Early detection of VUR and control of recurrent urinary tract infections could prevent the
development of reflux nephropathy and its complications including hypertension and chronic renal failure.

NOTE – According to RCH guidelines, ultrasonography is indicated in the following situation (different from
international guidelines and even National Guidelines by the Therapeutic Guidelines or RACGP):

Children with atypical UTI, those not responding to treatment within 48 hours, and boys <3 months of age
should have a renal ultrasound to exclude renal obstruction.
Children <6 months should have a renal ultrasound within 6 weeks of diagnosis. It should be performed during
the illness if the UTI is atypical or not responding to antibiotics within 48 hours.
Older children do not require an ultrasound after the first UTI, but should have a renal ultrasound for recurrent
UTI.

1237 of 1943
References

• RCH - Urinary tract infection

• Medscape - Pediatric Urinary Tract Infection

• Therapeutic Guidelines – Antibiotics; available on http://tg.org.au

Last updated:
Time spent: QID:118 2023-2-12

1238 of 1943
Lucas, 16 years old, is brought for medical attention after he developed a widespread rash yesterday. He has asthma
and a history of eczema but is otherwise fit and healthy. The rash started on his face and spread downwards to
involve his torso. He feels unwell and has flu-like symptoms and a fever of 38.2°C. Which of the following is the most
likely diagnosis?

Click to enlarge (Photo courtesy of DermNetNZ.org)

A. Impetigo.
B. Stevens-Johnson syndrome.
C. Allergic contact dermatitis.
D. Erysipelas.
E. Eczema herpeticum.

Incorrect. Correct answer is E


45% answered correctly

Explanation:

Correct Answer Is E

The photograph shows multiple facial lesions on an erythematous base, which are fluid-filled and some with central
dimpling. The characteristics of the rash and importantly the presence of eczema in history make eczema
herpeticum the most likely diagnosis in this scenario.

Eczema herpeticum is a disseminated viral infection characterized by fever and clusters of itchy blisters or punched-
out erosions. It is most often seen as a complication of eczema/atopic dermatitis. The lesions resemble the
chickenpox/varicella rash.

Most cases of eczema herpeticum are due to Herpes simplex type 1 or 2. Eczema herpeticum usually arises during
the first episode of infection with herpes simplex (primary herpes). Signs appear 5–12 days after contact with an
infected individual, who may or may not have visible cold sores. Eczema herpeticum may also
complicate recurrent herpes. However, repeated episodes of eczema herpeticum are unusual.

Eczema herpeticum can affect males and females of all ages but is more commonly seen in infants and children
with atopic dermatitis. Patients with atopic dermatitis appear to have reduced immunity to herpes infection. Their
underlying dermatitis can be mild to severe, active, or inactive.

Eczema herpeticum starts with clusters of itchy and painful blisters. It may affect any site but is most often seen on
the face and neck. Blisters can occur in normal skin or sites actively or previously affected by atopic dermatitis or

1239 of 1943
another skin disease. New patches form and spread over 7–10 days and may rarely be widely disseminated
throughout the body. The patient is unwell, with a fever and swollen local lymph nodes.

The following are the characteristics of the rash in eczema herpeticum:

The blisters are monomorphic, that is, they all appear similar to each other.
They may be filled with clear yellow fluid or thick purulent material.
They are often blood-stained i.e., red, purple, or black.
New blisters have central dimples (umbilication).
They may weep or bleed.
Older blisters crust over and form sores (erosions)
Lesions heal over 2–6 weeks.
In severe cases where the skin has been destroyed by infection, small white scars may persist long-term.

Severe eczema herpeticum may affect multiple organs, including the eyes, brain, lungs, and liver. It can rarely be
fatal.

Secondary bacterial infection with staphylococci or streptococci may lead to impetigo (option A), erysipelas (option
D), or cellulitis; however, this patient does not have any clinical features suggestive of any of the mentioned
conditions.

Contact dermatitis (option C) is localized at and limited to the site of the contact with the culprit allergen. Moreover,
systemic symptoms are not a feature. Stevens-Johnson syndrome (option B) is a rare serious disease of the skin
and mucous membranes usually as a reaction to a medication. It starts with flu-like symptoms and is followed by a
painful rash that spreads and blisters and results in sloughing off of the skin and mucous membranes. The absence
of a triggering medication and different rash characteristics make such a diagnosis unlikely.

References

• DermNet - Eczema herpeticum

Time spent: QID:1794 Last updated:


2023-2-12

1240 of 1943
A 12-year-old child has history of recurrent urinary tract infections since the age of 3 years. Which one of the
following is the investigation of choice to exclude renal scarring?

A. Renal biopsy.
B. Renal ultrasound.
C. Serum urea and creatinine.
D. DMSA scan.
E. CT scan of the abdomen.

Incorrect. Correct answer is D


45% answered correctly

Explanation:

Correct Answer Is D

Recurrent urinary tract infections (more than 2 times during childhood) can result in renal scarring, hypertension, and
end-stage renal disease. The underlying etiology is vesicoureteral reflux (VUR) -retrograde passage of urine from the
bladder into the upper urinary tract.

Dimercaptosuccinic acid scintigraphy (DMSA) scan is the gold standard for diagnosis of kidney scarring and
assessment of renal functions.

DMSA is indicated in the following conditions:

Clinical suspicion of renal injury


Reduced renal function
Suspicion of VUR
Suspicion of obstructive uropathy on ultrasound in older toilet-trained children

For this child with recurrent episodes of UTI, a DMSA should be considered as the most diagnostic modality for
assessment of renal function and possible renal scaring.

(Option A) Biopsy provides clues to histopathological abnormalities in kidney associated with certain type of renal
diseases such as glomerulonephritis but not the overall kidney function. Moreover, it is unnecessarily invasive.

(Option B) Ultrasound is capable of assessing the presence of fluid collections, bladder volume and the size, shape
and location of kidneys. However, it gives no clue regarding kidney function or scarring. Ultrasound is indicated for
assessment of children with UTI in the following situations:

Concurrent bacteremia
Atypical UTI organisms: i.e., Staphylococcus aureus, Pseudomonas
UTI <3 years old
Non/inadequate response to 48hrs of IV antibiotics
Abdominal mass
Abnormal voiding
Recurrent UTI
First febrile UTI and no prompt follow up assured
Renal impairment
Significant electrolyte derangement
No antenatal renal tract imaging in second to third trimester

(Option C) Urea and creatinine abnormalities are common among patients with urinary system problems. Elevated
levels of urea and creatinine indicated renal impairment but gives no clue for each indivudual kideny or at times
kidneys in general because urea and creatinine may be normal in early stages of pre-renal or post-renal renal failure.
1241 of 1943
On the other hand, normal values do not exclude the presence of abnormalities in only one kidney as the other kidney
can compensate. Many patients may have normal ranges in the presence of severe damage to one kidney while the
other is functional and compensating.

(Option E) Abdominal CT scan will visualize the kidneys, adrenal glands and adjacent structure but is not capable of
assessment of renal function.

References

• RCH - Urinary tract infection

• Medscape - Pediatric Urinary Tract Infection

Time spent: QID:119 Last updated:


2023-2-12

1242 of 1943
An eight-month-old male infant is brought to your office with history of vomiting and diarrhea. Urine dipstick
examination suggests urinary tract infection. The child is not dehydrated but seems slightly ill. Which one of the
following is the next best step in management?

A. Give antibiotics.
B. Give antibiotics and refer for ultrasound.
C. Investigate for a sexually transmitted infection.
D. Notify child protection services.
E. Oral rehydration and discharging home.

Incorrect. Correct answer is B


45% answered correctly

Explanation:

Correct Answer Is B

Urinary tract infection (UTI) in children may arise from urogenital abnormalities such as vesicoureteric reflux. For this
reason, in addition to treatment with antibiotics, an ultrasound scan of the kidney, ureter and bladder should be
considered in all children younger than 3 years of age (2-36 months in most guidelines) with the first episode of UTI.

Also, ultrasound is also indicated for children with UTI if:

Concurrent bacteremia
Atypical UTI organisms: i.e., Staphylococcus aureus, Pseudomonas
Non/inadequate response to 48hrs of IV antibiotics
Abdominal mass
Abnormal voiding
Recurrent UTI
First febrile UTI and no prompt follow-up assured
Renal impairment
Significant electrolyte abnormalities
No antenatal renal tract imaging in second to third trimester

I​n children less than 1 year, an ultrasound should be performed first. If the ultrasound is normal, voiding
cystoureterogram (VCU) would be the next step. This approach is adopted for early detection of urologic structural
abnormalities as well as possible vesicoureteric reflux.

In children older than 1 year of age, ultrasound alone is the investigation of choice for excluding vesicoureteric reflux.
Approximately 40% of the children with UTI have vesicoureteric reflux.

Vesicoureteric reflux can result in scarring of the kidneys, hypertension and chronic renal failure. Early detection of
vesicoureteric reflux helps prevent such complications.

References

• RACGP - Paediatric urinary tract infections: Diagnosis and treatment

Last updated:
Time spent: QID:120 2023-2-12

1243 of 1943
A mother has brought her 2-year-old boy to your GP clinic with complaint of chronic constipation. She mentions that
her boy opens bowels every 5 days. She is also concerned about her child’s recent weight loss. Which one of the
following piece of information, if in history, will help to make a diagnosis?

A. Delayed passage of meconium.


B. Cystic fibrosis of the mother’s brother in family history.
C. The child is premature at 35 weeks.
D. The child's weight is on 25th percentile.
E. Diarrhea after introducing cow’s milk.

Correct
45% answered correctly

Explanation:

Correct Answer Is A

Of the options, delayed meconium passage if in the history suggests Hirschsprung disease as the most likely cause
of this infant’s constipation.

Hirschsprung disease is a congenital anomalyy characterized by absence of ganglia in a segment of colon and
paralysis of this aganglionic segment. The condition starts at the anus and progresses up the rectum towards the
colon. It rarely affects more than the first 30 cm of the rectum and colon. The disease in more common in males.
Classic presentation is with the history of delayed meconium passage after birth and constipation in early infancy.

(Option B) With an affected brother with CF, the mother may be a carrier. If so, the baby had a 25% chance of
inheriting the disease if the father is a carrier, or 50% if affected. The history and clinical presentation is completely
inconsistent with CF as the diagnosis. The diagnostic triad of CF includes (1) chronic cough and recurrent chest
infections, (2) loose bowel motions and (3) failure to thrive. With constipation and no significant history of chest
infections, CF is unlikely.

(Options C and D) Prematurity and the child’s growth centile are irrelevant to the presenting condition.

(Option E) Diarrhea after introducing cow’s milk is suggestive of cow’s milk allergy. Symptoms cow's milk allergy may
include a rash (eczema or hives), swelling, abdominal pain or vomiting, diarrhea or breathing difficulties. Constipation
is not a common feature of cow’s milk allergy.

TOPIC REVIEW

Hirschsprung disease and its clinical presentation

Hirschsprung disease (HD) results from the absence of enteric ganglions in the rectum and/or colon. Hirschsprung
disease is approximately 4 times more common in males than females. Nearly all children with Hirschsprung
disease are diagnosed during the first 2 years of life. Approximately 50% of affected children are diagnosed before
the age of 1 year. The presenting features of HD vary depending on the length of the aganglionic segment and the
severity. Common presenting features include:

Delayed passage of meconium - In healthy term neonates, delay of more than 48 hours is indicative of HD. However,
this feature occurs in only about 50% of neonates with HD.

Neonatal bowel obstruction - Distal bowel obstruction is a presenting feature in about 25% of newborns with HD. The
classical features are distended abdomen, bilious vomiting, fever, dehydration, lethargy, not passed meconium, and
occasionally dilated peristaltic loops visible on the per abdominal examination in the neonate with a normal anus.
1244 of 1943
Neonatal bowel perforation - 5% percent of children with Hirschsprung’s disease have bowel perforation. Features of
perforative peritonitis predominate and in this situation the diagnosis of HD is not clear. Hence, during exploration of
such neonates, the colon must be examined and appropriate seromuscular biopsies must be done so as not to miss
the diagnosis of HD.

Neonatal enterocolitis - About 30% of neonates with HD develop enterocolitis. Sudden onset of diarrhea with or
without blood in the stools in neonates should raise the suspicion of HD. The stool is very foul smelling, explosive
and is associated with severe abdominal distension, fever, lethargy, dehydration and occasionally generalized sepsis.
Hirschsprung’s enterocolitis is a life threatening condition if toxic megacolon develops. Early diagnosis and
treatment is of significant importance.

Constipation- Older infants and children typically present with chronic constipation. This constipation often is
refractory to usual treatment protocols and may require daily enema therapy.

References

• Journal of Neonatal Surgery

• MedScape - Pediatric Hirschsprung Disease

Time spent: QID:140 Last updated:


2023-2-12

1245 of 1943
Six days after an uneventful tonsillectomy on a 7-year-old boy, he is brought back to the emergency department after
coughing up blood 30 minutes earlier. Which one of the following would be the next best step in management?

A. Packing.
B. Oral antibiotics.
C. Intravenous antibiotics.
D. Take him back to the operating room for exploration.
E. Give him ice cream and cold fluids.

Incorrect. Correct answer is C


45% answered correctly

Explanation:

Correct Answer Is C

Hemorrhage after tonsillectomy is the most common complication of the procedure. The bleeding may occur in the
early post-operative period or may be as delayed as 5 to 10 days. Late bleeding is normally secondary to infection of
the tonsillar fossae. Typically, there is initially minor bleeding, but blood-pouring develops afterwards; therefore any
bleeding even trivial must be considered serious in all cases. Post-tonsillectomy hemorrhage is a life-threatening
condition, which requires prompt management in all cases (even if only little blood is spat out and bleeding stops
again – all these cases should be dealt with caution – particularly the children).

NOTE – The chance of post-tonsillectomy hemorrhage increases with age.

On physical examination, a hint of blood in a corner of one of the tonsillar fossae may be noticed. Small grey or
clotted blood is another likely fkinding. Parents of younger children may describe finding blood on the child’s
pillowcase, or an episode of hemoptysis or hematemesis. Excessive swallowing may also be an indicator of ongoing
bleeding in young children

Management of post-tonsillectomy hemorrhage includes:

Resuscitation – 2 large bore cannula with bloods


Reservation of cross-matched packed red cells
Intravenous antibiotics: with 1.2g Benzyl Penicillin, 6-hourly + 500mg of metronidazole, 12-hourly
Analgesia without NSAIDs (NSAIDs may increase the chance of bleeding)
Hydrogen peroxide gargle - 20mls of hydrogen peroxide diluted with water in a ratio of 1:6. This gargle should
be used every 4 hours; it should be used when a large clot overlies one of the tonsil beds.

Of the given options, the only one in the management plan of this patient is administration of intravenous antibiotics.

(Option A) Packing the tonsils is neither practical nor effective.

(Option B) Antibiotics in oral forms are not recommended for management.

(Option D) Exploration might be indicated; however, most patients will respond to more conservative managements.

(Option E) The patient should be kept nil by mouth (NBM) because it is very likely that emergency surgery becomes
inevitable; therefore giving him ice cream or cold fluids is an incorrect option.

PRACTICE POINT

If a clot, as describes above, overlies a tonsil bed, it should be removed to see if there is bleeding behind it. Inform
the ENT registrar and the anesthetic team, then prepare yourself with a Magill’s forceps, some gauze, 1:10,000
1246 of 1943
adrenaline, tongue depressor and a headlight. Use the Magill’s forceps to pull off the clot and watch what happens. If
there is no bleeding, you can follow the advice given above and admit the patient with H2O2, IV fluids, and NBM.

If the patient starts bleeding, quickly soak a gauze with 1:10,000 adrenaline. Grip the gauze tightly with the tip of the
Magill’s forceps and push it into the fossa and hold it there for as long as your patient can cope with it. The idea is to
soak the fossa with adrenaline while also blocking the flow of blood. Lean the patient forward so he/she can spit out
the blood and saliva. The direction of the pressure should be directly into wall of the mouth (so laterally rather than
any posterior or inferior pressure which will cause the patient to gag). Continue this until either the bleeding stops or
help arrives.

References

• Clinical Junior

• Entsho

Time spent: QID:153 Last updated:


2023-2-12

1247 of 1943
Tanya has brought her 2-year-old son to your clinic, disgruntled and annoyed by the child’s condition. She tells you
that 2 months ago, she started to wean the child onto solid foods and currently his diet includes breastfeeding 3
times a day 5 minutes from each of her breasts and a bowl of cereal and rice every morning. The child has dropped
from the 50th to 25th percentile. The child’s past medical history is remarkable for one episode of admission due to
bronchiolitis. Which one of the following is the most likely cause of this baby’s diminished weight gain?

A. Cystic fibrosis.
B. Celiac disease.
C. Low calorie intake.
D. Child abuse.
E. Normal physiologic response to weaning.

Incorrect. Correct answer is C


45% answered correctly

Explanation:

Correct Answer Is C

With a drop in growth percentile, this child has developed failure to thrive.

Failure to thrive is defined as either of the following:

Body mass index for age less than the 5th percentile
Length for age less than the 5th percentile
Weight deceleration crossing two major percentile lines
Weight for age less than 50th percentile
Weight less than 75% of median weight for age
Weight less than 75% of median weight for length
Weight velocity less than the 5th percentile

The most common cause of failure to thrive is inadequate caloric intake. With 3 sessions of breast feeding and only
one bowl of cereal, this baby is definitely receiving far less than his caloric needs and therefore, low caloric intake
would be the most likely cause of his problem.

(Option A) With only one episode of bronchiolitis, the child is unlikely to have cystic fibrosis. Patients with cystic
fibrosis often have respiratory infection recurrences.

(Option B) Celiac disease can be a possibility, especially considering the fact that this child has been started on
cereal and rice, both of which contain gluten, but celiac disease is far less common than inadequate calorie intake.
Furthermore, the child with celiac disease has more pronounced symptoms e.g., diarrhea.

(Option D) Child abuse seems unlikely because firstly there is no comment in the question pointing towards the
likelihood of child abuse and secondly, the mother seems concerned about her child’s condition and has sought
medical care.

(Option E) Normal physiologic response to weaning could be considered if the baby was receiving adequate
nutrition.

1248 of 1943
A mother has brought her 9-month old boy to the Emergency Department with dyspnea and poor feeding. On
examination, his chest is hyperinflated, and subcostal recession is noted. On auscultation, there are widespread fine
inspiratory crackles and expiratory wheezes. Which one of the following is the next best step in management?

A. Nebulized adrenaline.
B. 100% oxygen via nasal prongs.
C. Intravenous corticosteroids.
D. Intravenous ceftriaxone.
E. 100% oxygen via facemask.

Incorrect. Correct answer is B


45% answered correctly

Explanation:

Correct Answer Is B

This clinical presentation in this 9-month old infant is classic for acute bronchiolitis, a viral illness most commonly
caused by respiratory syncytial virus (RSV). The usual age for the disease is between 2 weeks to 9 months of age.

The classic clinical presentation is prodromal symptoms (cough, coryza) followed by wheezy breathing, tachypnea
and hyperinflated chest with subcostal recessions. In children with bronchiolitis, oxygen therapy should be used
when oxygen saturations are persistently less than 90%. Infants with bronchiolitis will have brief episodes of
mild/moderate desaturations to levels less than 90% but these brief desaturations are not a reason to start oxygen
therapy.

If required, oxygen 100% should be given via nasal cannula and discontinued when oxygen saturations are
persistently greater than or equal to 90%.

Dehydration is also a serious problem in children with severe bronchiolitis and requires rehydration using intravenous
fluids or nasogastric tube.

Antibiotics are not indicated, unless a bacterial infection is suspected. There is no evidence to support the use of
nebulized adrenaline, bronchodilators or corticosteroids in children with bronchiolitis.

References

• RCH - Bronchiolitis

Last updated:
Time spent: QID:333 2023-2-12

1249 of 1943
Which of the following is regarded as the most important clinical exam finding in a child with acute viral
bronchiolitis?

A. Fever.
B. Expiratory wheeze.
C. Nasal discharge.
D. Inspiratory fine crackles.
E. Cough with productive sputum.

Incorrect. Correct answer is D


45% answered correctly

Explanation:

Correct Answer Is D

Bronchiolitis is an acute viral illness most commonly caused by respiratory syncytial virus (RSV). Usual age is from 2
weeks to 9 months (up to 12 months). It is the most common cause of lower respiratory tract infection in infants.

It presents initially with prodromal symptoms such irritating cough and rhinorrhea and then becomes worse in the
next 3 to 5 days.

Findings on examination include:

Wheezy breathing which is often distressed


Bilateral fine inspiratory crackles.
Tachypnea
Hyper-inflated chest (barrel-shaped)
Use of accessory muscles and subcostal recession

Chest X-rays are not necessary for diagnosis, but if obtained bilateral perihilar congestion and fullness will be
noticed.

Inspiratory fine crackles are regarded as the most characteristic feature of bronchiolitis.

(Option A) Children with bronchiolitis may have low-grade fever; however, fever is a very nonspecific finding seen in
many other respiratory tract infecions.

(Option B) Expiratory wheezing may be presents in bronchiolitis, as well as asthma and foreing body aspiration. It is
NOT a specific feature for bronchiolitis.

(Option C) Nasal discharge can be a featue in the prodromal phase of bronchiolitis, common cold, influenza, and
allergies. It is not specific for bronchiolitis.

(Option E) Cough can be a feature of bronchiolitis, but firstly it is not productive, and secondly, it may be present in
many other respiratory infections such as pneumonia, bronchitis, and asthma.

References

• MJA - Bronchiolitis: assessment and evidence-based management

• RCH - Bronchiolitis

Last updated:
Time spent: QID:334 2023-2-12

1250 of 1943
A 2-year-old boy is brought to the Emergency Department by his parents, who are concerned about his fever and
difficulty breathing. The fever has started this afternoon. On examination, he looks ill and pale and is grunting. He
has a temperature of 39.6°C and his respiration is shallow with a rate of 38 breaths per minute. Chest auscultation
is normal. Which one of following could be the most likely diagnosis?

A. Lobar pneumonia.
B. Acute bronchitis.
C. Atypical pneumonia.
D. Bronchiolitis.
E. Foreign body aspiration.

Correct
45% answered correctly

Explanation:

Correct Answer Is A

The findings of a high fever associated with pallor and respiratory distress in an infant is strongly suggestive of lobar
pneumonia. In this age group, chest auscultation may deceitfully be normal due to shallow rapid breathing and
grunting. The chest X-ray, however, will show consolidation. The most common causative organism is this situation
would be streptococcus pneumoniae.

(Option B) Acute bronchitis has a more insidious onset and the child is not often as severely ill as this child.

(Option C) With atypical pneumonia, a prodromal state and more insidious onset is expected. Moreover, fever is not
often that high.

(Option D) Bronchiolitis is the viral infection of bronchioles, caused by respiratory syncytial virus (RSV) in most
cases, and is more commonly seen in infants younger than 12 months. There may be low-grade fever, wheezing,
bilateral fine crackles and respiratory distress. There is a prodrome of cough and rhinorrhea, followed by more severe
symptoms in the following 3 to 5 days. The high fever and acute onset of symptoms favors the diagnosis of
pneumonia rather than bronchiolitis.

(Option E) Although post-obstructive atelectasis can give rise to pneumonia in children with foreign body aspiration,
this usually occurs over a course of weeks. The respiratory symptoms such as unilateral wheezing would be evident.

References

• Medscape - Pediatric Pneumonia

• AAFP- Community-Acquired Pneumonia in Infants and Children

• RCH - Community acquired pneumonia

Last updated:
Time spent: QID:335 2023-2-12

1251 of 1943
A 13-year-old boy is brought to your practice with fever and malaise for the past 2 days. On examination, he has a temperature of 38°C. There rest of the exam is inconclusive. In an
attempt to find the source of the infection, you order a set of laboratory tests including urinalysis (UA). The UA is positive for 1+ proteinuria and RBCs 30/hpf. No white blood cells
are present. The urine is negative for nitrates. Which one of the following would be the most appropriate step in management of this patient in terms of the abnormal UA?

A. Renal ultrasonography.

B. Repeat UA after fever settles.

C. Urine culture.

D. Start him on antibiotics.

E. Renal biopsy.

Incorrect. Correct answer is B


45% answered correctly

Explanation:

Correct Answer Is B

It is recommended that asymptomatic hematuria during a febrile illness should be assessed after the patient recovers from the illness, provided that the illness is not due to urinary
tract infection (UTI). Asymptomatic microscopic hematuria should always be confirmed with 2 positive urinalysis (UA) out of 3 in 2-3 weeks before any further assessment is
considered. With hypertension, edema, or renal failure, however, prompt action is mandated.

1252 of 1943
A mother brings her 2-week-old male neonate with noisy breathing, which becomes worse when the baby is laid on
his back. The child is otherwise healthy. Which one of the following is the most likely cause of this presentation?

A. Acute asthma.
B. Laryngomalacia.
C. Croup.
D. Acute bronchiolitis.
E. Acute epiglotitis.

Incorrect. Correct answer is B


45% answered correctly

Explanation:

Correct Answer Is B

The clinical history is suggestive of laryngomalacia. Laryngomalacia is congenital softening of the tissues of the
larynx above the vocal cords. This is the most common cause of noisy breathing in infancy. In laryngomalacia, the
laryngeal structure is malformed and floppy, causing the tissues to fall over the airway opening and partially block it.
Laryngomalacia symptoms are usually present at birth, and can become more obvious within the first few weeks of
life. Most children outgrow laryngomalacia by 18 to 20 months of age.

Symptoms of laryngomalacia include:

Noisy breathing - an audible wheeze when a baby inhales. It is often worse when the baby is agitated, feeding,
crying or sleeping on his back
High pitched sound
Difficulty feeding (in severe cases)
Poor weight gain (in severe cases)
Choking while feeding (in severe cases)

(Option A) Acute asthma makes breathing difficult, and wheeze is audible in any position. Asthma is an extremely
rare possibility in a 2-week-old neonate.

(Option C) Croup (laryngotracheobronchitis) is most commonly caused by para-influenza virus type 1 infection.
Croup is characterized by a distinctive seal barking cough and inspiratory stridor. Croup affects mainly children aged
6 month to 3 years. This child is in no respiratory distress, is otherwise healthy, and does not have any cough.

(Option D) Acute bronchiolitis is an acute viral infection of the lower respiratory tract, typically affecting infants
younger than 12 months of age, and is characterized by respiratory distress, wheezing, and inspiratory fine crackles.
It is primarily caused by respiratory syncytial virus (RSV). Treatment is supportive with oxygen and hydration.
Prognosis is generally excellent; however, some children may develop apnea or respiratory failure.

(Option E) Epiglottitis is a rapidly progressive bacterial infection of the epiglottis and surrounding tissues that may
lead to sudden respiratory obstruction and death. Symptoms include severe sore throat, dysphagia, high fever,
drooling, and inspiratory stridor. Treatment includes airway protection and antibiotics. This child does not have any
of these symptoms.

NOTE - While expiratory stridor indicates an obstruction in the lower trachea, inspiratory stridor suggests
obstruction above the glottis.

References

• Medscape - Laryngomalacia Clinical Presentation


1253 of 1943
• UpToDate - Congenital anomalies of the larynx

• AFP - Diagnosis of Stridor in Children


Last updated:
Time spent: QID:336 2023-2-12

1254 of 1943
A 6-month-old male infant with Tetralogy of Fallot (TF) is brought to your clinic by his mother with history of nasal
discharge, low-grade fever, feeding difficulty, wheezy breathing, and cough for 3 days. The infant's father smokes a
pack of cigarettes per day at home. Physical examination is significant for inspiratory crackles and expiratory
wheeze. Respiratory rate is 60 breaths per minute and oxygen saturation is 92 % on room air. Which one of the
following is the next best step in management?

A. Reassure the mother and educate about minimal handling and frequent feeds.
B. Admit the baby to the hospital and consider symptomatic care with supplemental oxygen, minimal
handling and use of intravenous fluids.
C. Start corticosteroid therapy immediately.
D. Start nebulized ribavirin and monitor the response.
E. Start the child on nebulized saline 3% immediately.

Incorrect. Correct answer is B


45% answered correctly

Explanation:

Correct Answer Is B

This infant has clinical diagnosis of acute viral bronchiolitis. The diagnosis of bronchiolitis is clinical and based on
typical history of increased respiratory rate, cough, expiratory wheezing and bilateral fine crackles, preceded by a
prodromal syndrome of low-grade fever and rhinorrhea.

For children with increased risk of progression to severe bronchiolitis, special consideration and probably admission
to the hospital and discussion with a senior doctor is required. Such infants are those with:

History of significant apnea before assessment


Known structural cardiac anomaly, especially large left-to-right shunt (e.g., ventricular septal defect)
Known pre-existing lung disease (e.g., cystic fibrosis)
Age less than 6 weeks
Significant prematurity (< 32 weeks) and/or chronic neonatal lung disease
Severe degree of respiratory distress or apnea
Significant dehydration
Hypoxemia (Spo2 < 93%)
Re-presentation to the emergency department within 24 hours
Uncertain diagnosis.

In the following situations, hospital admission is definitely indicated:

Difficulty in feeding
Increased work of breathing - use of accessory respiratory muscles
Respiratory rate>50 breaths per minute
Oxygen saturation less than 93% on room air.
Apnea
Dehydration
Patient or parental exhaustion

This child has SpO2 of 92% and a breathing rate of 60 breaths per minute. He also has as an underlying congenital
heart disease. For him, hospital admission is the first thing to consider.

The main issues in management are parental reassurance, education about minimal handling, and frequent feeds.
The child should receive intravenous fluids because the illness itself, as well as tachypnea makes the child
dehydrated.

1255 of 1943
(Option A) Although mentioned measures are appropriate, these should be taken in the hospital after admission.
This option does not include hospital admission and is not correct.

(Option C) Corticosteroids have not shown any benefit in management of bronchiolitis.

(Option D) Nebulized ribavirin may be used in immunocompromised patients but only provides marginal benefit in
respiratory syncytial virus infection. It is expensive, potentially toxic, and rarely indicated. Its use is not recommended
without specialist consultation.

(Option E) There is some evidence that the use of inhaled 3 % hypertonic saline may reduce the length of stay in
hospital and improve clinical severity score; however, it is not considered first-line treatment and its use remains
controversial.

References

• MJA- Bronchiolitis: assessment and evidence-based management

• RCH - Bronchiolitis

Time spent: QID:337 Last updated:


2023-2-12

1256 of 1943
A 4-year-old child is brought to your practice with a 2-week history of harsh cough, fever, runny nose and dyspnea.
He is short of breath on examination. Which one of the following would be the next best step in management?

A. Nasopharyngeal aspiration for pertussis.


B. Pertussis serology.
C. Chest X-ray.
D. Pulmonary function tests.
E. Blood culture.

Incorrect. Correct answer is C


45% answered correctly

Explanation:

Correct Answer Is C

With dyspnea, this child is likely to have a pulmonary pathology. Considering this, obtaining a chest X-ray is the most
appropriate initial step in management, as an attempt to assess the lungs for the cause of this presentation.

(Options A and B) Nasopharyngeal aspiration in children (nasopharyngeal swab in adults) for culture or serology is
to confirm the diagnosis of pertussis. However, the findings in this scenario are against pertussis, because (1)
patients with pertussis are not febrile and (2) lower respiratory tract is not involved. Unless during the bouts of
cough, shorteness of breath is not a common feature in pertussis.

(Option D) Pulmonary function test are not indicated here; at least not as an initial step.

(Option E) Blood culture can be considered as part of workup later on if indicated. An example is when chest X-ray
suggests pneumonia and measures to isolate the causative organism is required. At this stage, however, obtaining a
chest X-ray would be the most appropriate initial step.

References

• RACGP - AFP - Pertussis presentation, investigation and management

• Australian Prescriber - Managing pertussis in adults

• Medscape - Pertussis

Last updated:
Time spent: QID:338 2023-2-12

1257 of 1943
A mother brings her 8-year-old daughter for consultation because the girl avoids school and goes to games and
parlors. Her parents were advised by her teachers to seek medical help. When you talk to the girl, she becomes
tearful and looks withdrawn. Which one of the following is most suspected from the history?

A. Separation anxiety.
B. Truancy.
C. Depression.
D. Conduct disorder.
E. ADHD.

Incorrect. Correct answer is B


45% answered correctly

Explanation:

Correct Answer Is B

Truancy is any intentional unauthorized or illegal absence from compulsory schooling. Children who run away from
school to do other things (truancy) usually have different psychosocial issues when compared to those who do not
want to go to school and have the preference of staying at home.

Truancy may be an attempt to draw attention or to make an impression on peers. It also can be caused by anger at
school or problems at home. Truancy may simply be due to learning problems. Truant children sometimes go on to
break laws more seriously when they are older.

A feature of truancy is that the child tries to hide his/her action from the parents. On the other hand, the parents of
such children often are not very interested in their children educational state. They are more commonly of low
socioeconomic status.

Schools usually expect parents to inform the school in advance if their child is absent through the student diary, or by
phoning the school on the day of absence. It is a good way of ensuring child's safety. After 3 days of unexplained
absence, schools are required to contact the parent (s) and report the absence.

(Option A) This child does not fulfil the criteria for separation anxiety. Children with separation anxiety:

Want to stay at home with parents


Get upset about going to school and may have stomach aches or headaches, or do not feel well without a
physical cause
Do not have any serious behavior problems
Do not try to hide their wish regarding not going to school from their parents
Are more likely to be the youngest member of a family

(Option C) Children with depressive disorders feel sad, lack interest in activities they previously enjoyed, criticize
themselves, and are pessimistic or hopeless about the future. They may also be irritable and aggressive. They may
be indecisive, and have problems concentrating. They tend to lack energy and to have sleeping problems.

(Option D) The typical behavior of those with conduct disorder involves bullying, frequent physical fights, deliberate
destruction of other people's property, breaking into houses or cars, staying out until late night despite parental
prohibitions, running away from home, or frequent truancy from school. In this scenario, there is no misconduct in
the history to make this diagnosis likely.

(Option E) ADHD is characterized by lack of concentration and hyperactivity at least at two distinct environments
(e.g. school and home) which is different from this child’s problem.

References

1258 of 1943
• RACGP - School Refusal
Last updated:
Time spent: QID:339 2023-2-12

1259 of 1943
Which one of the following can be an alarming sign in a 2-month old infant?

A. Inability to smile at people.


B. Inability to hold neck.
C. Inability to explore things by holding and looking at them and putting them in mouth.
D. Does not seem to recognize mother.
E. Does not seem interested in activities around them.

Correct
45% answered correctly

Explanation:

Correct Answer Is A

Normally, social smile develops at 6 weeks of age; therefore its absence in a 2-month-old baby could be concerning.

(Option A) The ability to hold the neck is obtained at around 3 months.

(Option B) Ability to explore things by holding, looking at them and putting them in mouth is achieved by 6 months of
age.

(Option D) A skill to recognize mother’s face is achieved at the age of 3 months.

(Option E) The child starts to show interest in the activities around them at about 9 months of age.

References

• http://childrenandfamilies.nt.gov.au/library/scrip

• http://files.acecqa.gov.au/files/ACECQA/2014/devel

• http://www.parenting.sa.gov.au/pegs/peg77.pdf

Last updated:
Time spent: QID:342 2023-2-12

1260 of 1943
A male neonate born at 36 weeks gestation is noted to have many clinical features suggestive of Down syndrome.
Six hours after birth, he starts vomiting. The vomit contains caseated milk and green material suggestive of bile. On
examination, the abdomen is mildly distended. Which one of the following is the most likely diagnosis?

A. Hirschprung disease.
B. Duodenal atresia.
C. Necrotizing enterocolitis.
D. Large bowel volvulus.
E. Meconium plug syndrome.

Incorrect. Correct answer is B


45% answered correctly

Explanation:

Correct Answer Is B

Children with Down syndrome have an increased risk of gastrointestinal tract anomalies, occurring in approximately
5%. These anomalies include:

Duodenal atresia or stenosis, sometimes associated with annular pancreas - the most characteristic lesion,
occurring in 2.5%
Imperforate anus
Esophageal atresia with tracheoesophageal fistula
Hirschsprung disease (<1%)

Of the options, early onset of bilious vomiting is explicable by duodenal atresia as the most likely diagnosis.

(Option A) Although Hirschsprung disease can present in neonatal period, especially in children with Down
syndrome, typical presentation occurs after 4 to 5 days after birth with constipation, progressive abdominal
distention. Delayed passage of meconium is an early characteristic feature in history. Vomiting is not common.

(Option C) Necrotizing enterocolitis is almost only seen in sick premature infants who are on maximum support
during the acute stage of their complicated illness.

(Option D) In neonatal period, malrotation with midgut volvulus may occur. Volvulus presents with abdominal pain
that may be associated with bilious or nonbilious vomiting; however, Down syndrome, very early onset of vomiting,
and more importantly vomiting immediately after breastfeeding in more consistent with duodenal atresia.

(Option E) Meconium plug syndrome tends to present later (2 to 3 days after birth), and is not associated with Down
syndrome.

References

• RACGP - AJGP - Common neonatal presentations to the primary care physician

• Medscape - Pediatric Duodenal Atresia

Last updated:
Time spent: QID:344 2023-2-12

1261 of 1943
Lucy has presented to your office concerned about her 3-years-old daughter. Lucy has past medical history of
asthma and eczema and heard that these conditions can run in families. She wants to know about the risk of her
daughter developing asthma or eczema as well. In consulting her, which one of the following would be the most
appropriate advice you can give to minimize the risk allergic reactions in the child?

A. Avoid monosodium glutamate in foods.


B. Avoid contact with sheepskin clothes.
C. Minimizing household dust mites.
D. No specific advice is needed.
E. Giving the child antihistamines in case allergy develops.

Incorrect. Correct answer is C


45% answered correctly

Explanation:

Correct Answer Is C

Household dust mites are the most common allergens that can cause allergy and asthma in genetically susceptible
individuals. About 90% of children with atopic symptoms and asthma demonstrate positive skin-prick responses to
dust mite extract.

In general, reducing dust mites is the most appropriate advice here. Avoiding contact with sheepskin bedding and
clothes is a part of measures taken to reduce dust mites.

Anti-histamines and avoiding mono-sodium glutamate in food is not recommended for minimizing the risk of atopy
in children.

Current evidence suggests that exposure to dust mite allergen can be minimized by:

Encasing mattresses, pillows and blankets/doonas in dustproof covers


Polishing floors and replacing carpet
Washing bed clothes in hot water (>55°C) weekly
Not using sheepskin bedding

References

• ACAAI - Dust Allergy

Last updated:
Time spent: QID:368 2023-2-12

1262 of 1943
Emily, 18 months old, is brought to your clinic by her parents for a routine health visit. They are new to then town and
this is the first time you visit her. She is the outcome of an uneventful pregnancy and has had no major medical
problem so far. On examination, you hear a cardiac murmur. Which one of the following clinical findings suggests
that the murmur is an innocent murmur?

A. The murmur is diastolic.


B. The murmur is pansystolic.
C. The murmur is associated with a thrill.
D. The murmur increases when the child is positioned supine.
E. The murmur is associated with reduced exercise tolerance.

Incorrect. Correct answer is D


45% answered correctly

Explanation:

Correct Answer Is D

Physiologic (innocent) murmurs are common in healthy infants, children, and adolescents.

Innocent murmurs have the following characteristics (7 ‘S’s):

1. Sensitive – changes with child’s position or with respiration


2. Short duration (not holosystolic)
3. Single – no associated clicks or gallops
4. Small – murmur limited to a small area and non-radiating
5. Soft – low amplitude (up to 2/6 or very rarely to 3/6)
6. Sweet – no harsh sounding (one innocent murmur may be harsher than the other, but they are not generally
harsh sounding like pathologic murmurs), no thrills
7. Systolic – occurs during and is limited to systole

With the presence of either of the following, the murmur is pathological:

Grade 3/6 or higher murmurs


Harsh quality
Abnormal S2
the presence of systolic click
Symptoms related to cardiac conditions e.g., shortness of breath, chest pain, poor feeding, failure to thrive,
etc.
Increased intensity when the venous return decreases (e.g., when the patient stands)

Also with any of the following every murmur should be considered pathologic until proven otherwise:

Family history of sudden cardiac death or congenital heart disease


In utero exposure to certain medications or alcohol
Maternal diabetes mellitus
History of rheumatic fever
History of Kawasaki disease

Physiologic (innocent murmurs) always change in intensity (increase) with maneuvers that increase venous flow to
the heart (squatting, lying down, leg raising, etc.) and are decreased in intensity when there is a reduction in venous
return (increased intra-abdominal pressure by e.g. Valsalva maneuvers). It should be noted, however, that pathologic

1263 of 1943
murmurs can be affected by different maneuvers as well. An innocent murmur changes with position, but not every
murmur that changes with position is innocent.

NOTE - There only 3 murmurs that increase in intensity when the venous return to the heart is decreased:

1. The murmur caused by hypertrophic obstructive cardiomyopathy (HOCM) – pathologic


2. The murmur caused by mitral valve prolapse – pathologic
3. The venous hum – venous hum is a roaring sound caused by flow of the blood from the superior vena cava
best heard on the neck above the clavicle, mostly on the right side.

(Options A and B) Innocent murmurs are never diastolic. Even holosystolic murmurs are frequently found to be
pathologic on further evaluation.

(Option C) A thrill is never physiologic and an underlying cardiac pathology is always present.

(Option E) An innocent murmur is always asymptomatic; therefore, symptoms such as decreased exercise tolerance,
failure to thrive, poor feeding, etc. are against such diagnosis.

(Option E) Reduced exercise tolerance is highly suggestive of a cardiac disease, either congenital or acquired.

TOPIC REVIEW

Types of physiological (innocent) murmurs

Still’s murmur

Still’s murmurs are low-pitched sounds best heard at the lower left sternal border. They are musical or have
relatively pure tone or may be squeaky. They most commonly occur between 3 years of age and adolescence.
They are related to flow; therefore, they change with position.

Pulmonary flow murmur

These murmurs are harsher and best heard over the upper left sternal border. They are flow-dependent and
change in intensity with changing positions. They disappear or decrease with maneuvers that increase the
intraabdominal pressure resulting in decreased venous return. These murmurs originate from the right
ventricular outflow tract and radiate along the pulmonary arteries and may be best heard in the back and axilla
bilaterally.

Pulmonary flow murmurs can occur at any age, but they are common particularly in adolescents or in children
with pectus excavatum. They are prominent in high-flow situations, such as when a child has a fever or is
anemic.

In infants, these sounds may be most prominent in the back or axilla because turbulence occurs when the
blood flows from the larger main pulmonary artery to the smaller and less-developed distal pulmonary
arteries. In fetal life, the main pulmonary artery transports about 90% of the blood to the ductus arteriosus and
only about 10% to the distal pulmonary arteries. The main pulmonary artery is thus large, whereas the distal
pulmonary arteries are relatively smaller and come off at more acute angles than they do later as the child’s
chest grows. An analogy from nature would be the noise that is created as a large stream narrows into
smaller streams.

Systemic flow murmurs (supraclavicular systemic bruits)

These are harsh high-pitched murmurs caused by normal blood flow into the aorta and into the head and neck
vessels and are heard best high up in the chest and above the clavicles. There is no ejection click. It has been
said that because of these sounds “all children have carotid bruits”; however, the sounds are different in
quality from true carotid bruits and are not associated with carotid outflow pathology.

Venous hums
1264 of 1943
Venous hums are low-pitched continuous murmurs produced by blood returning from the great veins to the
heart. They are heard best with the bell of the stethoscope. By changing the position of the patient’s head or
by pressing in the area of the major neck veins, the flow may be changed and these murmurs will change or
disappear. Having the child look down or to the side while listening, will often make these murmurs or sounds
disappear.

NOTE – Still’s murmur and venous hum are low-pitched and best heard by the bell of the stethoscope, whereas
pulmonary flow murmurs and systemic flow murmurs are high-pitched and best detected by the diaphragm of
the stethoscope.

References

• AAFP - Evaluation and Management of Heart Murmurs in Children

• GP Notebook - Pediatric murmur

Time spent: QID:392 Last updated:


2023-2-12

1265 of 1943
A one-year-old female child is presented to you for a health checkup. She has normal growth parameters. She can sit
unsupported, stand with assistance, and say ‘mama’. On examination, a 2/6 systolic murmur is heard over the apex.
The rest of the exam is completely normal. Which one of the following would be the most appropriate next step?

A. Order ECG and chest X-ray.


B. Refer her to pediatric cardiologist.
C. Reassure the parents as this is an innocent murmur.
D. Refer her for emergency echocardiography.
E. Tell the parents that she can have a congenital heart disease.

Incorrect. Correct answer is B


45% answered correctly

Explanation:

Correct Answer Is B

Heart murmurs are common in healthy infants, children, and adolescents. Despite the fact that most heart murmurs
are not pathologic, a murmur may be the only manifestation of a serious underlying heart disease.

With either of the following present, the murmur should be considered pathological unless proven otherwise:

Family history of sudden cardiac death or congenital heart disease


In utero exposure to certain medications or alcohol
Maternal diabetes mellitus
History of rheumatic fever
History of Kawasaki disease

When a murmur is present on heart auscultation, a thorough physical exam, including the following, should be
performed:

Vital signs
Peripheral pulses and perfusion
Exercise tolerance (age-matched)
Auscultation over the heart valves
Looking for respiratory or gastrointestinal manifestations of cardiac diseases

An innocent murmur is a physiologic murmur with the following characteristics (7 ‘S’s):

1. Sensitive – changes with child’s position or with respiration


2. Short duration (not holosystolic)
3. Single – no associated clicks or gallops
4. Small – murmur limited to a small area and non-radiating
5. Soft – low amplitude (up to 2/6 or very rarely to 3/6)
6. Sweet – no harsh sounding (some innocent murmurs my sound harsher than the others, but generally
physiologic murmurs are not harsh in nature), no thrills
7. Systolic – occurs during and is limited to systole

​With the presence of either of the following, the murmur is pathologic:

Grade 3/6 or higher murmurs


Harsh quality
1266 of 1943
Abnormal S2
The presence of a systolic click
Increased intensity with decreased venous return (e.g., when the patient stands)
The patient has any symptoms that could be related to a cardiac condition (e.g. shortness of breath, chest
pain, poor feeding, etc.)

In the following situations, the patient should be referred to a pediatric cardiologist:

Murmurs in infancy, especially in the first 6 months


There is a family history of sudden cardiac death or congenital heart disease
There are cardiac symptoms – shortness of breath, poor feeding, failure to thrive, chest pain, fatigability, etc.
There are chromosomal abnormalities
The diagnosis is equivocal

This child is completely healthy and has no associated physical finding suggestive of a cardiac pathology. The
murmur itself, on the other hand, has all characteristic features of an innocent physiologic murmur, but since she is
in the infancy period, referral to a specialist (preferably a pediatric cardiologist) is the most appropriate option to
consider.

(Option A) Chest X-ray and electrocardiography are of little value in assessment of the underlying cause of a murmur
in a child. Chest X-rays were obtained previously as a part of assessment, but not an acceptable practice today.

(Option C) Although this murmur is likely to be innocent, parents cannot be reassured unless full evaluation by a
pediatric cardiologist excludes any pathologic etiology.

(Option D) Echocardiography is the investigation of choice that may be considered by the pediatric cardiologist for
further assessment.

(Option E) Although parents should be informed of all possibilities, telling them that their baby has heart disease
while the murmur is more likely to be innocent and no further assessment has been carried out is not appropriate.

References

• AAFP - Evaluation and Management of Heart Murmurs in Children

• FP Notebook - Pediatric Murmurs

• GP Notebook - cardiac murmur (innocent)

Time spent: QID:393 Last updated:


2023-2-12

1267 of 1943
Ten hours after uneventful delivery of a 3200 g male baby, he develops central cyanosis. He is immediately started
on supplemental oxygen but there is no improvement. On auscultation, no murmur is heard. Which one of the
following is the most likely cause of this presentation?

A. Tetralogy of Fallot.
B. Patent ductus arteriosus.
C. Ventricular septal defect.
D. Transposition of the great arteries.
E. Arterial septal defect.

Incorrect. Correct answer is D


45% answered correctly

Explanation:

Correct Answer Is D

The cyanotic congenital heart lesions are associated with right to left shunts. Of the given options, patent ductus
arteriosus, ventricular septal defect, and arterial septal defect are associated with left to right shunts; therefore, no
cyanosis is expected.

The only two options associated with cyanosis are Tetralogy of Fallot (TOF) and Transposition of the Great Arteries
(TGA).

In TOF, as the name implies, there are 4 defects:

1. Pulmonary infundibular stenosis


2. Overriding aorta
3. Ventricular septal defect
4. Right ventricular hypertrophy

In TOF, the VSD gives rise to a systolic murmur, but this child has no cardiac murmurs on auscultation. This
excludes TOF as a diagnosis.

TGA is the most common cyanotic congenital heart lesion that presents in neonates. In TGA the aorta arises from
the right ventricle and the pulmonary artery from the left ventricle. Infants with TGA are usually born at term, with
cyanosis developing within hours of birth. The clinical course and manifestations depend on the extent of
intercirculatory mixing and the presence of associated anatomic lesions. If there are no associated defects such as
ventricular septal defect (VSD) no murmur is heard.

References

• Medscape - Transposition of the Great Arteries

Last updated:
Time spent: QID:394 2023-2-12

1268 of 1943
Regarding patent ductus arteriosus, which one of the following statements is incorrect?

A. It frequently occurs as an isolated phenomenon.

B. Cyanosis is usually present.

C. It causes a pansystolic ‘machinery’ murmur at the left sternal border.

D. There is a wide pulse pressure.

E. Definite treatment is often by surgical closure.

Incorrect. Correct answer is B


45% answered correctly

Explanation:

Correct Answer Is B

During fetal life, ductus arteriosus, a conduit connecting the pulmonary artery to the proximal descending aorta, is patent (open). This patency
is vital to the fetus because it allows most of the blood from the right heart to bypass the fetal fluid-filled lungs and enter the aorta, and from
there, flow into the systemic circulation. After birth and when the neonate should take over breathing and oxygenation, it is essential for
ductus arteriosus to close for prevention of mixing oxygenated and deoxygenated blood; otherwise, a shunt of oxygenated blood from higher-
pressure aorta to the lower-pressure pulmonary artery will occur. With left-to-right shunts, such as in patent ductus arteriosus, cyanosis is not
a feature until very late in the course of the disease.

(Option A) Patent ductus arteriosus (PDA) is often an isolated congenital cardiac problem with no association with other cardiac anomalies.

(Option B) PDA is associated with left-to-right shunting; therefore, cyanosis would not be a presenting feature; however, if untreated, right-to-
left shunting and cyanosis eventually follows.

(Option D) Small lesions are expected to be asymptomatic, but larger ones present with bounding (strong) arterial pulses, wide pulse
pressure, the typical systolic machinery murmur, and congestive heart failure in severe or prolonged cases.

(Option E) In some cases, PDA closure occurs spontaneously within one week. Lesions persisting beyond one week are very unlikely to close
on their own. NSAIDs, by inhibiting prostaglandin synthesis play a role in management; nonetheless, closure by surgical intervention is needed
for cases with persisting PDA.

References

• Medscape - Patent Ductus Arteriosus (PDA)

Last updated:
Time spent: QID:100
2023-2-12

1269 of 1943
A 4-year-old boy is brought to your practice with a second urinary tract infection (UTI) in the past 4 months. Last
time, an ultrasound scan revealed a grade II vesicoureteral reflux (VUR). In addition to treatment of the current
infection, which one of the following would be the most important action to take for prevention of further UTIs?

A. Intermittent antibiotics.
B. Practicing good hygiene.
C. Surgical correction of the VUR.
D. Continuous antibiotics.
E. Circumcision.

Incorrect. Correct answer is D


45% answered correctly

Explanation:

Correct Answer Is D

Vesicoureteral reflux (VUR) (retrograde flow or urine from the bladder into the ureter and kidneys) is an anatomic and
functional disorder that potentially can result in significant morbidity, both from acute urinary tract infections and
from the sequelae of reflux nephropathy.

After entering the bladder through the muscular hiatus of the detrusor muscle, the normal distal ureter passes
through a submucosal tunnel before opening into the bladder lumen via the ureteral orifice. If the length of the
submucosal tunnel or its muscular backing is inadequate, the valve mechanism is incompetent and causes reflux.

Voiding cystoureterogram is the criterion standard for both diagnosis and staging of the VUR.

The International Classification System for VUR is as follows:

1. Grade I – Reflux into the nondilated ureter


2. Grade II – Reflux into the ureter and renal calyces without dilation
3. Grade III – Reflux with mild to moderate dilation and minimal blunting of the fornices
4. Grade IV – Reflux with moderate ureteral tortuosity and dilation of pelvis and calyces
5. Grade V – Reflux with gross dilation of ureter, pelvis and calyces, loss of papillary impressions, and ureteral
tortuosity

The condition often resolves with growth. The lower the grade, the higher the chance of resolution would be.

Prophylaxis against urinary tract infection (UTI) should be started after a child has completed treatment of the first
episode of UTI and continued at least until a VCUG has been performed. If no VUR is detected, prophylaxis can be
discontinued. If VUR is present, prophylactic antibiotics are continued until:

1. VUR resolves
2. VUR is surgically repaired
3. The child grows old enough and prophylaxis is no longer considered necessary

Since a significant number of children will have spontaneous resolution of VUR (50-85% of cases with grade I-III),
medical treatment spares this group the morbidity from surgery, while protecting the kidneys from more damage.

Medical treatment consists of antibiotics dosed at 1/4 – 1/3 of the therapeutic dosage and regular follow-up and
imaging. A typical routine includes renal ultrasound and VCUG, or nuclear cystography every 12-18 months.

In boys with persistent VUR, who have not had recurrent UTIs, antibiotics are often discontinued as the boys
approach puberty. Surgery may be recommended in girls approaching puberty who have persistent VUR.

1270 of 1943
NOTE – In the past few years, the role of prophylaxis with antibiotics has been challenged. Several studies failed
to find any decrease in the incidence of UTIs in children with VUR who take antibiotics. However, these studies
had significant flaws, making it difficult to state with certainty that antibiotic prophylaxis is ineffective.
Accordingly, the National Institutes of Health (NIH) sponsored a large prospective clinical trial of prophylaxis in
an effort to answer this key question. In 2014, the results of the Randomized Intervention for children with
Vesicoureteral Reflux (RIVUR) study showed that antibiotic prophylaxis with trimethoprim-sulfamethoxazole was
associated with a decrease of approximately 50% in the incidence of recurrent UTIs among children with VUR, in
comparison with placebo. Based on these results prophylactic antibiotics for all grades of VUR remains in
practice.

This boy with recurrent UTIs needs continuous prophylactic antibiotics.

(Option A) While continuous prophylactic antibiotics have shown to decrease UTI recurrences, intermittent use of
antibiotics has not proven effective.

(Option B) Although practicing good hygiene is an appropriate advice to give, it does not specifically reduce the risk
of recurrent UTIs in children with VUR.

(Option C) Surgery is considered for sever VURs (grade V), girls approaching puberty whose VUR persist, or patients
with recurrent UTIs despite prophylaxis. A grade II reflux is very likely to resolve spontaneously with growth and only
needs medical management.

(Option E) Circumcision of infant males with VUR may be considered based on an increased risk of urinary tract
infections in boys who are not circumcised compared to those who are. Although there are insufficient data to
evaluate the degree of this increased risk and its duration, parents need to be made aware of this association so that
they can decide whether their children are circumcised.

References

• UCSF - Vesicoureteral Reflux (VUR)

• Medscape - Pediatric Vesicoureteral Reflux

• American Urological Association - Management and Screening of Primary Vesicoureteral Reflux in Children

Time spent: QID:461 Last updated:


2023-2-12

1271 of 1943
You are called by the nurse on duty to assess a 24-hour old female neonate in the neonatal ward. Physical
examination is remarkable for the knees being at unequal levels when the hips and knees are flexed, asymmetric
gluteal folds, and limited abduction of the right hip. The history is significant for breech delivery. Which one of the
following is the most likely diagnosis?

A. Osgood-Schlatter disease.
B. Slipped capital femoral epiphysis.
C. Developmental dysplasia of the hip.
D. Femoral head fracture.
E. Legg-Calve-Perthes disease.

Incorrect. Correct answer is C


45% answered correctly

Explanation:

Correct Answer Is C

The exam findings are characteristic of developmental dysplasia of the hip (DDH), also called congenital hip
dislocation.

DDH can be unilateral or bilateral. Both genetic and environmental factors contribute to the condition. With positive
family history, bilateral involvement is more common. Genetic as well as environmental factors has been implicated
for DD. Environemental factors include intrauterine malposition (especially breech with extended legs) and maternal
production of relaxin shortly before delivery (relaxin is a ligament-relaxing hormone)

Female newborns are 6 times more likely to be affected. Every newborn should be screened for the condition
because if left untreated it can lead to disabling morbidity.

DDH can present with the following features on examination:

Diminished abduction in flexion of the affected hip


Apparent inequality of legs with the affected leg being shortened and externally rotated
Asymmetrical skin creases of the groin and thigh
‘clicking’ on hip movements

In DDH the femoral head of the affected side is subluxed, dislocatable or dislocated. This, if left untreated, leads to
early degenerative joint disease of the hips.

Two tests are used to diagnose DDH:

Barlow test – The affected femoral head can be telescoped in and out of the acetabular socket by piston-like
‘telescoping’ movements in the long axis of the flexed and abducted thighs. This maneuver is often done at the same
time as Ortolani test. Great caution should be taken to avoid further ligamental injuries.

Ortolani test – The child’s flexed hips are abducted. The thighs are the grasped between the thumbs in front and
other fingers behind. The child’s knees are flexed and hip flexed to a right angle. Each thigh is steadily abducted
towards the examining surface while the fingers behind apply forward pressure. Positive findings include reduction
of the dislocated femoral head into the acetabulum with an audible and palpable ‘jerk’ or ‘clunk’ (not a click).
(Positive Ortolani sign)

1272 of 1943
NOTE - Barlow test is always performed before Ortolani test.

(Option A) Osgood-Schlatter disease is the overuse apophysitis of the tibial tubercle resulting in localized pain,
especially with quadriceps contraction in active young boys.

(Option B) Slipped capital femoral epiphysis is the separation of the proximal femoral epiphysis through the growth
plate, possibly as a result of imbalance between growth hormone and sex hormones, and often seen in early
adolescence (the typical patient is an overweight boy of 10-14 years of age).

(Option D) With no history of trauma, femoral head fracture is unlikely. Furthermore, femoral head fracture presents
differently.

(Option E) Legg-Calve-Perthes disease is an idiopathic avascular necrosis of the femoral head most commonly
found in boys of 4 to 10 years of age.

References

• Royal Prince Alfred Hospital – Developmental dysplasia of the hip

• SA Health – Neonatal Hip Screening and Management of Developmental Dysplasia of the Hip

• Diagnostic Imaging Pathways – Paediatrics, Hip Developmental Dysplasia

Time spent: QID:518 Last updated:


2023-2-12

1273 of 1943
A 6-year-old boy is brought to your practice by his mother with complaints of pain in the right knee and limping. He
has had flu-like symptoms 10 days ago. On examination, he has a temperature of 38 °C. Hip movements are limited
in all directions, but limitation of internal rotation and abduction are more pronounced. An ultrasonography of the
right hip is arranged that reveals widening of the right hip joint space. Which one of the following is the most
appropriate management?

A. Start him on antibiotics.


B. Start him on analgesics.
C. Immobilize the affected joint.
D. Start him on systemic corticosteroids.
E. Bed rest.

Incorrect. Correct answer is E


45% answered correctly

Explanation:

Correct Answer Is E

Hip joint pain and limp in children may have different etiologies. In children aged 4 -10 years, transient synovitis (TS)
also known as irritable hip is the most common cause of hip pain. Legg-Calve-Perthes disease is another cause with
a predilection for male children. TS is the most common cause of nontraumatic hip pain in children.

Clinical manifestations of TS include:

Arthralgia and arthritis secondary to a transient inflammation of the synovium of the hip
Unilateral hip or groin pain is the most common symptom reported; however, some patients may report medial
thigh or knee pain
There is often no fever, or only a mild fever
Some patients with transient synovitis may not report pain and may present with only a limp

NOTE - High fever is rare, and if present should raise suspicion against more serious conditions such as septic
arthritis

Diagnosis is based on history, physical examination‫ و‬and ultrasonography. Hip X-rays are performed to exclude other
possible causes. On physical examination, mild restriction of motion, especially abduction and internal rotation is
noted; however, one-third of patients with transient synovitis do not have limitation of hip movements. The hip may
be painful even with passive movement, or tender to palpation.

The most sensitive test for transient synovitis is the log roll tes‫و‬t in which the patient lies supine and the examiner
gently rolls the involved limb from side to side. This may detect involuntary muscle guarding of one side when
compared to the other side.

Radiographs exclude bony lesions such as occult fractures or osteoid osteoma. Medial joint space may be slightly
wider in the affected hip. Radiographs are often used as the initial diagnostic test.

Ultrasonography (US) is the most accurate modality for detection of intracapsular effusion, but does not help in
determining the etiology. An effusion is present if US demonstrates capsular distention greater than 2 mm. US
cannot exclude osteomyelitis or soft tissue infection. US is used if X-rays are normal despite clinical suspicion of TS.

Management of TS is protection of the hip by bed rest for 7-10 days and use of crutches to avoid weight bearing of
the affected joint. Paracetamol or NISAIDs (e.g.‫ و‬ibuprofen) may help.

1274 of 1943
(Option A) TS is not an infectious diseases. Antibiotics have no role in treatment of TS.

(Option B) NSAIDs and paracetamol may have a role in management, but relative joint rest is the mainstay of therapy
in most cases.

(Option C) While bed rest and use of crutches is advised, strict immobilization of the hip is not recommended.

(Option D) TS is a benign condition that resolves spontaneously within 1 to 4 weeks. Corticosteroids are
unnecessary for treatment.

References

• UpToDate- Overview of hip pain in childhood

• Medscape - Transient Synovitis

• Therapeutic Guidelines – Rheumatology; available from http://tg.org.au

Time spent: QID:519 Last updated:


2023-2-12

1275 of 1943
A 4-year-old boy is brought to you with a painful left knee and limping. Recently, the child had a diagnosis of 'irritable
hip'. On examination, internal rotation and abduction of the left hip joint is limited. An X-ray of the hip shows
increased joint space. Which one of the following is the next best step in management?

A. Encourage activity.
B. Use crutches and encourage bed rest.
C. Do a follow up X-ray next week.
D. Hip joint aspiration under general anesthesia.
E. Referral to an orthopedic surgeon.

Incorrect. Correct answer is E


45% answered correctly

Explanation:

Correct Answer Is E

The clinical and radiological findings are highly suggestive of Legg - Calve - Perthes disease as the most likely
diagnosis.

Legg-Calve-Perthes disease is caused by idiopathic avascular necrosis of the femoral epiphysis, possibly due to
repeated multiple vascular occlusive episodes that involve the femoral head. It typically presents with hip/knee pain
and/or limp of acute or insidious onset, most commonly in Caucasian boys between the ages of 4 and 10 years, with
peak incidence at 5 to 7 years of age. In 15% of the patients the condition is bilateral.

In children older than 8 years, the condition is associated with higher incidence of osteoarthritis in adulthood.
Younger children (less than 6 years old) have a greater capacity for the acetabulum to undergo remodeling to
accommodate irregularities in the shape of the femoral head.

As there is a significant risk of femoral head destruction due to avascular necrosis, immediate referral to orthopedic
specialist is essential and the most appropriate next step in management.

(Option A) Encouraging activity is not an appropriate option because reducing the weight on the hip joint is an
essential part of treatment.

(Option B) Activities should be restricted and the patient should be offered to use crutches at least until the time he
is reviewed by an orthopedic surgeon. Although, use of crutches and encouraging bed rest are partially effective,
they are not the definite management for such patients and are only temporary measures to prevent worsening of
the problem.

(Option C) Legg-Calve-Perthes requires urgent referral and delayed referral with further investigations is not
appropriate. The condition will not improve spontaneously and follow-up X-rays unnecessarily delays appropriate
treatment.

(Option D) Aspiration under general anesthesia is performed to exclude septic arthritis if history suggest such
diagnosis. This child is afebrile and his problem has had an insidious development, making septic arthritis less
likely.

References

• Medscape - Legg-Calve-Perthes Disease

• UptoDate – Approach to hip pain in childhood

Last updated:
Time spent: QID:520 2023-2-12
1276 of 1943
A 2-year-old boy is brought to your clinic due to a 3-day history of fever with occasional cough and coryza. On clinical
history, viral upper respiratory tract infection is suspected. On physical examination, the patient is noted to have
down-slanting of palpebral fissures, eyes that are apart more than usual and low-set ears. The height of the patient is
smaller than the standard. Which one of the following is not consistent with this condition?

A. Pulmonary valve stenosis.


B. Myopia and ectopic ocular lens.
C. An autosomal disorder involving mutation in chromosome 11.
D. Webbed neck.
E. Abnormalities in cardiac conduction and rhythm.

Incorrect. Correct answer is B


45% answered correctly

Explanation:

Correct Answer Is B

This clinical picture is suggestive of Noonan syndrome. This is an autosomal dominant disorder with mutation of
chromosome 11. It has been described as the male Turner's syndrome, but affects both sexes.

Clinical features of the syndrome include the following:

Facial/eye features

Triangular face
Hypertelorism (increased distance between the eyes)
Down-slanting eyes
Ptosis
Strabismus – seen in 48%
Amblyopia – seen in 33%
Refractive errors – seen in 61%
Low-set ears with thickened helices
High nasal bridge
Short webbed neck

Chest/back features

Pectus carinatum/excavatum
Scoliosis

Cardiac features

Stenotic/dysplastic pulmonic valve is the characteristic cardiac anomaly


Hypertrophic cardiomyopathy (obstructive and non-obstructive) – seen in as many as 30%
Virtually all types of congenital heart defects have been described in patients with Noonan syndrome

Abdominal features

Hepatosplenomegaly unrelated to cardiac disease – seen in 25% of the patients

Genitourinary features

Renal anomalies are present in 10% of patients but are not clinically significant
More than half of male patients have undescended testes
1277 of 1943
Skeletal features

Joint laxity is present – seen in more than 50% of the patients


Less common findings include talipes equinovarus (club foot) that is an excessively turned-in foot and high
medial longitudinal arc, radioulnar synostosis (fusion), cervical spine fusion, and joint contractures.

Skin findings

Lymphedema
Prominent pads of fingers and toes – seen in 67%
Follicular keratosis of face and extensor surfaces – seen in 14%
Multiple lentigines (a lentigo is a small, sharply circumscribed, pigmented macule surrounded by normal-
appearing skin) – seen in 3%

Neurological findings

Hypotonia
Seizure disorders – seen in 13%
Unexplained peripheral neuropathy – not seen frequently
Mental retardation

Previously, patients with Noonan syndrome were thought to have a form of Turner syndrome, as clinical picture is
similar to that of Turner’s.

Myopia and ectopic ocular lens is consistent with Marfan syndrome and not are features seen in Noonan syndrome.

References

• Medscape - Noonan Syndrome

Time spent: QID:523 Last updated:


2023-2-12

1278 of 1943
During physical examination of a 3-kilogram full-term male newborn, you notice that the baby’s oxygen saturation
decreases when he is laid on his back. The fall in oxygen saturation is sometimes associated with mild cyanosis.
However, when the baby starts crying the oxygen saturation increases again. Which one of the following will be the
next best step in management?

A. Rigid nasogastric tube.


B. Chest X-ray.
C. Surgery.
D. Intubation and ventilation.
E. Oxygen via nasal cannula.

Correct
45% answered correctly

Explanation:

Correct Answer Is A

The presentation is classic for choanal atresia. Choanal atresia is a congenital abnormality in which there is failure
of canalization of the bucconasal membrane. This can be either unilateral or bilateral and is usually due to a
combination of bone and soft tissue anomalies. It occurs in 1:5000 to 1:8000 births. Unilateral atresia is more
common, with a predilection for females. CHARGE Syndrome (coloboma, heart defect, atresia choanae, retarded
growth and development, genital abnormality and ear abnormality) or other congenital abnormalities are present in
50% of patients with bilateral choanal atresia.

Bilateral choanal atresia is a relatively rare anomaly of the upper airway. As neonates are obligatory nasal breathers,
presentation may be with life-threatening respiratory distress, retractions and paradoxical cyanotic episodes which
are relieved by crying as the infant begins to breathe via the mouth. Unilateral choanal atresia does not usually
produce severe symptoms.

When choanal atresia is suspected the diagnosis can be confirmed with trying to pass a nasogastric tube. It is not
possible to pass a nasogastric tube through the nares and choanae if there is choanal atresia.

In the presence of bilateral choanal atresia, as the history and physical suggest, oxygen through nasal cannula will be
of no benefit.

Other measures to consider as parts of management plan include:

Oxygen
Oropharyngeal airway
Intubation and ventilation if there is life-threatening hypoxemia
CT scan
ENT consultation and referral for surgical treatment

Since the manifestation is typical of choanal atresia, chest X-ray is not indicated now, but it may be obtained to
exclude other possible associated respiratory conditions or anomalies.

References

• Western Australia Health Department - Choanal Atresia

• Medscape - Choanal Atresia

Last updated:
Time spent: QID:530 2023-2-12

1279 of 1943
A 3-year-old boy is brought to the Emergency Department with complaints of fever and shortness of breath. His
symptoms began 2 days ago with mild fever and rinorrhea. Today, he has a fever of 39°C, nasal flaring, tracheal tug
and intercostal recession. He has a harsh cough as well. Which one of the following would be the next best step in
management of this child?

A. Nebulized salbutamol.
B. Nebulized budesonide.
C. Intramuscular adrenaline.
D. Intravenous penicillin.
E. Oxygen by nasal prongs.

Incorrect. Correct answer is B


45% answered correctly

Explanation:

Correct Answer Is B

The clinical picture is highly suggestive of croup as the most likely diagnosis. Croup (acute
laryngotracheobronchitis) presents with a coryzal prodrome, hoarseness (or husky voice in those old enough to
speak), biphasic stridor, a harsh barking ‘brassy’ cough and variable airway obstruction due to inflammatory edema
within the subglottis. Fever is often mild by can be as high as 39°C.

It is most common in 1- to 3-year-old children and is generally self-limiting, with a duration of 2 to 5 days.
Parainfluenza viruses are the most common causes of croup.

Treatment of croup depends on the severity. In terms of severity, croup can be classified as mild, moderate, and
severe based on the criteria outlined in the following table:

Mild Moderate Severe


Behavior Normal Intermittent/ mild agitation Increasing agitation/
Drowsiness
Stridor No stridor, or only when Intermittent stridor at rest Persistent stridor at rest
active or upset
Respiratory Normal Increased respiratory rate Marked increase or
rate decrease
Use of None or minimal Moderate chest wall Marked chest wall retraction
accessory retraction
muscles
Oxygen Hypoxia is a late sign
indicating life-threatening
croup

In mild cases (a croupy cough but no history of noisy breathing in a child older than 2 years), no treatment is
required, but early review is essential.

In moderate to severe cases (e.g., a history of noisy breathing or age less than 2 years, or presence of stridor on
examination), a single dose of corticosteroid is indicated. Use either:

Dexamethasone 0.3 mg/kg orally (first-line), OR


Predniso(lo)ne 1mg/kg orally, or
Budesonide 2mg by nebulizer

Further doses are usually not needed, but can be considered if response to therapy is suboptimal at 24 hours

1280 of 1943
In more severe cases with significant airway obstruction or fatigue, treatment in hospital may be required with an
initial dose of:

Adrenaline 1% (1:1000, 10 mg/ml) solution 0.05 ml/kg/dose up to 0.5 ml diluted with NaCl 0.9%, by nebulizer

PLUS EITHER

Dexamethasone 0.6mg/kg orally (or IM if vomiting). or


Predniso(lo)ne 1mg/kg orally

Maintenance dose and its frequency vary with the severity of the condition and the response to treatment.

This child with nasal flaring and tracheal tug and stridor is classified as having moderate croup and should be
treated with oral or IM dexamethasone, or oral predniso(lo)ne, or nebulized budesonide.

(Option A) Nebulized salbutamol would be used if the child had an asthma attack.

(Option C) Intramuscular adrenalin has no role in management of croup, but nebulized adrenaline is indicated in
severe croup.

(Option D) Antibiotics are not indicated in treatment of croup.

(Option E) Supplemental oxygen is not often required, except in children with severely obstructed airway. Even so,
nebulized adrenaline takes precedence to provide a patent airway for oxygen supplementation.

References

• The Royal Children's Hospital (RCH) - Croup (Laryngotracheobronchitis)

• Therapeutic Guidelines – Antibiotic; available from: http://tg.org.au

Time spent: QID:676 Last updated:


2023-2-12

1281 of 1943
A mother comes to your clinic because she is concerned about her 5-year-old son. He has started soiling his
underwear every night since he entered school 3 months ago. His teacher also has asked her to come for a
discussion about his behavior that seems inappropriate for his age at school. On examination, the boy seems small
for his age. Which one of the following can be the most likely cause of the soiling?

A. Constipation.
B. Autism.
C. Cystic fibrosis.
D. Hirschsprung disease.
E. Anorectal malformations.

Incorrect. Correct answer is A


45% answered correctly

Explanation:

Correct Answer Is A

The scenario represents a case of encopresis. Encopresis is defined as the voluntary or involuntary passage of
formed, semiformed, or liquid stool into a place other than the toilet for more than one time per month for at least 3
months in a child older than 4 years.

Encopresis is either primary or secondary:

Primary encopresis: the child has never been continent


Secondary encopresis: fecal incontinence in a child who was previously continent

Encopresis is a common condition reported in approximately 1-4% of school-aged children.

Most fecal incontinence in children is functional, meaning there is no known organic defect. Approximately 80-90%
of fecal incontinence results from chronic constipation, which leads to impaction and overflow soiling.

Constipation can be caused by the child repetitively attempting to avoid defecation or holding the stool because of
pain, fear, or not liking defecation or using the toilet. This can lead to further constipation. It may also be associated
with behavioral, emotional, or psychiatric issues and impacts the child’s physical and psychological development.

For many children with encopresis, there may be a history of an event that made having a bowel movement
uncomfortable or frightening. This could range from fear of pain or toilet flushing to repeated sexual abuse.

Non-retentive encopresis occurs if the child refuses to defecate in an appropriate place, e.g., the toilet, but has no
history of constipation. These children typically soil daily or regularly but their bowel movements are of normal size
and consistency. While most children with encopresis have not experienced sexual abuse, if there is a history of early
sexual abuse, encopresis may be more likely.

Non-functional encopresis (caused by medical or physical problems) accounts for about 5- 10% of cases. This is
known as organic encopresis. Common organic causes of encopresis include:

Anorectal malformations
Spinal cord abnormalities
Hirschsprung disease
Certain medications.

This child is small for his age. Moreover, it is mentioned, by the school teacher, that his behavior is not appropriate
for his age. With these in history, this child should be further evaluated for underlying problems, yet constipation
remains the most likely cause of his problem.

1282 of 1943
NOTE – In the following situations the child should be referred for further assessment:

Soiling is prolonged and resistant to treatment


Associated significant behavioral problems
Soiling associated with day wetting
Soiling not associated with fecal retention and overflow

(Option B) Gastrointestinal symptoms such as constipation may be seen with increased frequency in autistic
children but this child has no features suggestive of autism in clinical exam or history. Such features include
impaired language, stereotypic behavior, social inappropriateness, etc.

(Option C) Cystic fibrosis can cause constipation and encopresis; however, cystic fibrosis would have presented
earlier with recurrent chest infection if it was the underlying cause. Cystic fibrosis presents with meconium ileus
(20%of patients), respiratory symptoms (45% of patients), and failure to thrive (28% of patients).

(Option D) Hirschsprung disease is one of the non-functional (organic) causes of encopresis, but it presents much
earlier in the neonatal and infantile period.

(Option E) Anorectal anomalies can cause non-functional encopresis; however, it is less common than constipation
as the underlying etiology.

References

• Functional fecal incontinence in infants and children: Definition, clinical manifestations and evaluation

Time spent: QID:679 Last updated:


2023-2-12

1283 of 1943
A mother brings her 7-year-old son who has measles and wants to know if the child needs to be excluded from
school. The diagnosis was made when the typical rash developed 6 days ago and he has not attended his class
since then. The teacher has informed her that all children in the class has been immunized against measles. Which
one of the following would be the most appropriate advice?

A. No more exclusion is required.


B. He should be excluded from school for a total period of 14 days after the rash.
C. He should be excluded for another 4 days.
D. He shoudl receive MMR vaccine before he can go back to school.
E. He can go to school when the rash completely resolves.

Incorrect. Correct answer is A


45% answered correctly

Explanation:

Correct Answer Is A

All children with measles should be excluded from school for at least 4 days after the onset of the rash.

Immunized and immune contacts are not excluded. Non-immunized contacts of a case are to be excluded from child
care until 14 days after the first day of appearance of rash in the last case, unless immunized within 72 hours of first
contact during the infectious period with the first case. All immunocompromised children should be excluded until
14 days after the first day of appearance of rash in the last case.

Since more than 4 days has passed since the onset of the rash no more exclusion is required and he can go back to
school.

Vaccination is not indicated anymore because this child will have life-long immunity to measles. However, MMR
vaccine should still be given for protection against rubella and mumps at due time.

References

• https://www.nhmrc.gov.au/_files_nhmrc/publications

Last updated:
Time spent: QID:709 2023-2-12

1284 of 1943
The baby whose photograph is shown in the following photograph was born to an epileptic mother who had been on
treatment with carbamazepine throughout the pregnancy. She is smoker but has cut down on it after the first
trimester. She also had had one standard drink throughout the first trimester but stopped when she found out these
might have detrimental effects on her baby. She was not supplemented with folic acid as recommended. Which one
of the following could be the most likely cause of the birth defect of this child?

A. Smoking.
B. A sporadic or familial genetic condition.
C. Folic acid deficiency.
D. Carbamazepine.
E. Alcohol.

Incorrect. Correct answer is B


45% answered correctly

Explanation:

Correct Answer Is B

The picture shows a cleft lip. Orofacial clefts are the outcome of interaction between genetic and environmental
factors; however, genetic defects either sporadic or familial are the most important factors in developing orofacial
clefts.

Environmental factors can increase the risk of orofacial clefts in a genetically susceptible fetus. These factors
include:

Medications - antiepileptic drugs (e.g., phenytoin, sodium valproate, and topiramate), methotrexate (folic acid
antagonist). It should be noted that while the relative risk of orofacial abnormality is increased, the absolute
risk of such defects after drug exposure remains small.
Smoking
Alcohol
Folic acid deficiency
1285 of 1943
Maternal obesity
Maternal diabetes

References

• UpToDate - Etiology, prenatal diagnosis, obstetric management, and recurrence of cleft lip and/or palate

• RCH - Cleft lip and cleft palate

• Medscape - Pediatric Cleft Lip and Palate

Time spent: QID:712 Last updated:


2023-2-12

1286 of 1943
A 3-year-old Aboriginal boy is presented to you after he sustained a fit at home 20 minutes ago. According to his mother, the fit started with
the boy falling on the ground and having jerky movements of the arms and legs. He was unresponsive during the attack. He never had such an
episode before. On examination, the boy looks lethargic but oriented. His temperature is 37.2°C.The rest of examination is inconclusive. Of
the following options, which one is most likely to reveal the cause of this seizure?

A. Vitamin D level.

B. EEG.

C. CT scan of the brain.

D. Chest X-ray.

E. Toxicology screen.

Correct
45% answered correctly

Explanation:

Correct Answer Is A

The following (VITAMINS) can be the causes of an episode of seizure:

Vascular (stroke, bleeding, arteriorvenous malformations)


Infections (encephalitis, meningitis, abscess)
Trauma (especially penetrating)
Autoimmune (CNS vasculitis)
Metabolic (hypo/hyper natremia, hypocalcaemia, hypomagnesemia, hypoglycemia, hypoxia, drug overdose/withdrawal)
Idiopathic (epilepsy)
Neoplasms
pSychiatric (pseudoseizures)

It is very important to consider an episode of afebrile seizure as a symptom rather than a disease. The next best step in management of a
seizure for the first time invariably starts with looking for an underlying cause after the patient is stabilized. Once no precipitating factor could
be found for the seizure, the condition is termed as epilepsy.

The minimum investigations for a first-episode afebrile seizure include the following:

Serum calcium, magnesium, and the basic metabolic panel (sodium, potassium, blood sugar, chloride, bicarbonate, urea and
creatinine)
Fasting blood glucose
ABG – to exclude hypoxia as a cause of the seizure
Toxicology screen
An MRI of the brain (first-choice) or CT scan

According to an article recently published in Medical Journal of Australia (MJA), vitamin deficiency is prevalent among adult Aboriginal
Australian, particularly within autumn, spring and winter. The article, however, is not clear about the prevalence of vitamin D deficiency among
Aboriginal children.

Other populations susceptible to vitamin D deficiency are those who live in refugee camps, people with limited sun exposure (residents in
areas near Antarctica or North Pole or dark-skinned people).

Vitamin D insufficiency, leads to decreased absorption of dietary calcium and hypocalcemia. Hypocalcemia has several manifestations
including seizure and, as mentioned earlier, serum calcium level should always be checked in the very first workup of any first-episode seizure.

The question does not ask about initial investigations. Measuring vitamin D level is not among initial assessments of a first-time seizure, but
serum calcium level is. Once hypocalcemia is established, further investigation to determine the etiology of the hypocalcemia will include
measurement of serum vitamin D level.

However, for the purpose of this question, if vitamin D was measured initially, a decreased level is likely to be present as the most likely cause
of the seizure.

(Option B) EEG is often considered if no secondary cause for the seizure is found (not among the very first initial steps).

(Option C) A CT scan or preferably MRI of the brain, although should be considered as part of the investigation are not very likely to unveil an
underlying brain pathology such as brain tumor or abscess, because there are no pointers towards space occupying lesion in this child on
exam.

1287 of 1943
(Option D) The boy is afebrile; therefore, studies such as chest X-ray or urinalysis, that are recommended as a part of work for identifying the
source of the fever in febrile seizures are not indicated here.

(Option E) Toxicology screen would be of great value in older patients such as adolescents or adults in whom intentional drug use is more
common. A 3-year-old boy is unlikely to overdose on drugs deliberately.

References

• MJA - Vitamin D insufficiency in Aboriginal Australians

Last updated:
Time spent: QID:106
2023-2-12

1288 of 1943
A 16-month-old child is brought to your attention by his parents because of their concerns about their child’s
language delay. He was born at 36 weeks through normal vaginal delivery with a birth weight of 2500 g. At 8 months
of age, a 'mother and child health screening test' for hearing was performed that was completely normal. The
parents say that he babbles but has not spoken any words so far. On physical examination, the child appears quite
normal. Which one of the following will you recommend?

A. Reassure the parents that this is a normal variant.


B. Arrange for formal audiometry testing.
C. Reassess the child at 18 months of age.
D. Repeat the 'mother-child health' hearing test.
E. Check for dysmorphic features.

Incorrect. Correct answer is C


45% answered correctly

Explanation:

Correct Answer Is C

Parents are usually the first to be suspicious of their child having a hearing problem and they are almost always
right. Concerns of parents about possible developmental delays should never be ignored and have to be addressed
promptly. In this child, the parents are concerned about language delay.

Language delay is defined as either of the following:

Late to talk and first words do not appear by the age of 15-18 months.
The child gets their first words but then does not go on to develop new words quickly.
By two years of age, the child is saying less than 50 words and is not using any two word combinations (e.g.
‘more drink’, ‘Daddy gone’, ‘car go’).
The child has difficulty understanding what is being said to them and has difficulties following instructions.
The child’s language sounds immature for their age.
Difficulties attending at group time at kindergarten or school.
The very young child may have difficulties with eye contact, attending to activities and to speech and using
sounds and gestures.
Difficulty answering questions.
Difficulty sequencing words together in sentences.
Difficulty reading and writing.
Difficulty getting their message across.
Uses incorrect grammar (e.g., ‘me want that red one’ instead of ‘I want the red one’).

By the age of 2 years a child should be able to:

Say first name


Say many words (mostly naming words)
Begin to use one- to two-word sentences such as ‘want milk’

Not saying even a word by the age of 15-18 months is definitely an indicator of language delay for which
investigation should follow. This child is 16 months now; therefore, it is best to wait until 18 months of age to see if
he can speak his first word. Sine hearing problems are the most common cause of language delay, a formal
audiometry assessment (option B) would be the most appropriate option to consider if he fails to speak his first
word by the age of 18 months.

(Option A) Parents cannot be reassured unless their child can speak his first word by the age of 18 months.

1289 of 1943
(Option D) As the name implies, UNNHS test is a screening test. Screening tests are to find asymptomatic cases.
Since this child seems to have hearing problems (symptomatic) repeating a test would be useless and formal
audiological assessment is needed as the most appropriate step in management. Several techniques are available
for children who are still so young to engage in tests where speaking is necessary.

(Option E) The hearing deficit as a possible cause of language delay may or may not be associated with dysmorphic
features indicative of a congenital developmental problem or syndrome. This does not change the necessity for
confirmation of hearing deficit by audiological assessments when indicated. This child still has until 18 months of
age to say his word before language delay is even considered.

References

• Kid Sence - Language Delay

• Developmental milestones and the Early Years Learning Framework and the National Quality Standards

• RACGP - Audiology

Time spent: QID:749 Last updated:


2023-2-12

1290 of 1943
A 3-year-old boy is brought to the Emergency Department with sever stridor and suprasternal notch and chest wall
retraction. Based on the history and clinical findings, the diagnosis of croup is established. Which one of the
following findings, if present, indicates severe respiratory distress?

A. Suprasternal notch and chest-wall retraction.


B. Use of accessory muscles.
C. Loudness of the stridor.
D. Restlessness and agitation.
E. Oxygen saturation by oxymetry.

Incorrect. Correct answer is D


45% answered correctly

Explanation:

Correct Answer Is D

Croup affects about 2% of preschool-aged children every year. Most children have mild croup and are managed at
home often after review by a general practitioner. A minority of children develop moderate or severe croup. This
group should be reviewed in an emergency department and may need hospital admission.

Determining the degree of airway obstruction (based primarily on the history) is the most important consideration
when assessing children with croup. Since airway obstruction can deteriorate rapidly, repeated careful clinical
assessment is crucial.

Based on clinical findings, airway obstruction can be classified as mild, moderate or severe:

Mild – A child with mild airway obstruction appears happy and is prepared to drink, eat, play and be interested
in the surroundings. Most child presenting to general practitioners fall within this category. There might be
mild chest-wall retraction and mild tachycardia, but there is no stridor at rest.

Moderate – Moderate airway obstruction is indicated by persisting stridor at rest, chest-wall retraction, use of
accessory respiratory muscles and increased heart rate. The child can be consoled and remains interactive
with people and the surroundings.

Severe – The child appears increasingly worried, tired and exhausted. There is marked persistent tachycardia
unprortionate to fever. Restlessness, agitation, irrational behavior, decreased level of consciousness,
hypotonia, cyanosis and marked pallor are late signs of life-threatening airway obstruction.

Assessment of croup severity is summarized in the following table:

Mild Moderate Severe


Behavior Normal Intermittent/ mild agitation Increasing agitation/
Drowsiness
Stridor No stridor, or only when Intermittent stridor at rest Persistent stridor at rest
active or upset
Respiratory Normal Increased respiratory rate Marked increase or
rate decrease
Use of None or minimal Moderate chest wall Marked chest wall retraction
accessory retraction
muscles

1291 of 1943
Oxygen Hypoxia is a late sign
indicating life-threatening
croup

Of the given options, restlessness and agitation, especially if the child cannot be consoled and calmed down,
indicates severe airway obstruction. Other indicators of severe airway obstruction include: confusion, lethargy,
cyanosis, pallor, or confusion.

(Options A and B) Suprasternal notch and chest wall retraction can be seen in moderate to severe or even mild
airway obstruction. This is also true about use of accessory respiratory muscles; therefore, these findings cannot be
reliably used to determine the severity of airway obstruction. Moreover, children with severe obstruction may have
decreased chest-wall retraction and accessory respiratory muscle use as respiratory fatigue develops.

(Option C) Loudness of the stridor cannot be used as a predictor because firstly, it could be underestimated or
overestimated depending on the ambient noises, and secondly, the intensity of the stridor may decrease when
respiratory fatigue and diminished air entry develops.

(Option E) Oxymetry is not a reliable indicator because it may be close to the normal level in a child with severe
croup, or, conversely, substantially decreased in a child with mild to moderate croup.

References

• MJA - Croup: assessment and evidence-based management

• RCH - Croup (Laryngotracheobronchitis)

Last updated:
Time spent: QID:755 2023-2-12

1292 of 1943
A 6-month-old boy is brought to your office by his mother with persistence bouts of cough for the past 2 weeks.
Every bout of cough is associated with cyanosis of the face and usually followed by vomiting. Which one of the
following is the next step in management?

A. Nasopharyngeal aspirate and review in 24 hours.


B. Immunization at this visit with dTP and review in 24 hours.
C. Start him on clarythromycin and review in 24 hours.
D. Trial of salbutamol by mask.
E. Admission to the hospital.

Incorrect. Correct answer is E


45% answered correctly

Explanation:

Correct Answer Is E

The clinical picture suggests respiratory infection with Bordetella pertussis.

Pertussis (whooping cough) is a bacterial respiratory infection caused by the organism, Bordetella pertussis (a Gram
negative bacillus). It is highly infectious and is spread by respiratory droplets to 70–100% of susceptible household
contacts and 50–80% of susceptible school contacts. Pertussis infection occurs worldwide, it affects all age groups,
and is most serious in young, unprotected infants. It commonly occurs in teenagers and young adults who have not
been immunized or with waning immunity. Patients are infectious just before, and for 21 days after, the onset of
cough if left untreated.

If treated early with antibiotics, the period of infectivity usually lasts 5 days or less after the commencement of
therapy. Maternal antibody does not give adequate protection against pertussis; therefore, babies can be infected
before being immunized.

Pertussis presents with mild upper respiratory tract symptoms (catarrhal stage) and can progress to severe
paroxysms of cough (paroxysmal stage), often with a characteristic inspiratory whoop, followed by vomiting. Fever is
usually absent or minimal. Symptoms gradually resolve (convalescent stage). The paroxysmal cough and inspiratory
whoop may be absent, particularly in infants less than 6 months of age and in adults. Mild illness is common,
particularly in immunized individuals. The cough may persist for up to 3 months.

The indications for hospitalization/pediatric consultation include the following:

Infants less than 6 months of age


Any child who has had apnea, cyanosis, pneumonia, or encephalopathy as a complication of pertussis.

With cyanosis following the attacks of cough, this child needs to be admitted to the hospital as the most important
next step in management.

(Option A) Nasopharyngeal aspirate can be considered later, if indicated, after the child is admitted to the hospital.

(Option B) Immunization, at a later date, should be encouraged even if the child has had pertussis, but this step is
not a part of acute management plan.

(Option C) Antibiotics should be considered if:

The patient is diagnosed in catarrhal or early paroxysmal phase (may reduce severity)
Cough for less than 14 days (may reduce spread - reduces school exclusion period)
Admitted to hospital
There are complications (pneumonia, cyanosis, apnea)

1293 of 1943
Antibiotics of choice are azithromycin or clarithromycin. They can be used after the child is admitted to the hospital.

(Option D) Salbutamol or other bronchodilators has not been shown to be helpful in management of patients with
pertussis.

References

• RCH - Whooping cough (pertussis)

• RACGP - Pertussis Presentation, investigation and management

Time spent: QID:757 Last updated:


2023-2-12

1294 of 1943
Paul, aged 4 months, is diagnosed with pertussis. The father is an engineer and the mother a housewife. They both
have completed childhood vaccination against pertussis. Paul has a 5-year-old sister. Which one of the following is
the most appropriate management plan to consider for Paul's parents regarding pertussis?

A. Immediate booster immunization against pertussis.


B. A 10-day course of erythromycin.
C. Scheduling a 3-dose course of pertussis vaccination.
D. Nasopharyngeal swabs.
E. Immediate immunization with pertussis immunoglobulin.

Correct
45% answered correctly

Explanation:

Correct Answer Is A

Current guidelines in Australia for the public health management of pertussis recommend Antibiotic treatment of the
index case with exclusion from childcare, school, work, or other environment where high-risk contacts may be
present, until they are non-infectious (that is, after 5 days of antibiotic treatment). Treatment must be commenced in
the first 21 days of illness to be effective. It does not shorten the duration of the illness, but does limit duration of
infectivity.

Chemoprophylaxis of contacts to prevent secondary transmission is not recommended in most situations because
of the delayed presentation of the index case, and the cost and adverse effects of antibiotics. However, given the
high risk of mortality and morbidity associated with infection of the newborn, particularly in the context of the rising
incidence of pertussis in the community and the high transmission rate, chemoprophylaxis is recommended to limit
transmission to those most at risk of the infection. Data to support this recommendation are limited.

Australian guidelines recommend post-exposure chemoprophylaxis for contacts to whom transmission is most
likely, and when there is significant risk of morbidity or mortality or risk of transmission to other high-risk groups.

The following criteria must be met for commencement of prophylactic antibiotics:

Close contact with confirmed case of pertussis (index case) while index case is infectious, which is within the
first 21 days of cough and <5 days of antibiotics

AND

First contact was within 14 days (or within 21 days for infants <6 months) – beyond that time prophylaxis is
not effective

If the above criteria are met, the following groups should receive prophylactic antibiotics:

Children

Age <6 months; OR


<3 doses pertussis vaccine; OR
There is a household member aged <6 months; OR
Children who attend childcare in same room as infant <6 months

Adults (regardless of immunization status)

Expectant parents in last month of pregnancy; OR

1295 of 1943
Health care worker in maternity hospital or newborn nursery; OR
Childcare worker in close contact with infants <6 months; OR
Household member aged <6 months

In the given scenario, there is no other child less than 6 months in the house other than Paul who is the index case,
and none of the parents are in close contacts with other children younger 6 than months, no prophylactic antibiotic is
required; however, it is recommended that close contacts that are not up-to-date with their pertussis immunization
should be given DTPa or dTpa as soon after exposure as possible. dTpa should be considered for adults who have
not had pertussis-containing vaccine in the last 10 years.

(Option B) Parents do not fulfill criteria to receive prophylactic antibiotics.

(Option C) Parents have received their 3 doses in childhood and do not need to have it repeated again. All they need
is a booster dose of pertussis vaccine.

(Option D) Nasopharyngeal swab results do not affect the management of close contacts in terms of prophylaxis;
therefore, not indicated.

(Option E) Immunoglobulin (passive immunity) is not effective in reducing the chance of catching pertussis.

References

• RCH - Whooping cough (pertussis)

• CDNA National Guidelines for Public Health Units - Pertussis

• RACGP - Pertussis Presentation, investigation and management

Time spent: QID:758 Last updated:


2023-2-12

1296 of 1943
An unvaccinated 3-year-old child has had severe paroxysms of cough with inspiratory whoop for the past 4 weeks.
Bouts of cough have been followed by vomiting. Cough bouts occur more at night. Physical examination is normal.
Laboratory investigations is significant for lymphocytosis. Which of the following is not true regarding this condition?

A. Antibiotics at this stage are not likely to reduce the communicability of the disease.
B. Antibiotics often do not improve the symptoms of the patient.
C. School exclusion for him is not necessary anymore.
D. Given the time of presentation, antibiotic prophylaxis for the close contacts is not likely to be effective.
E. Antibiotic prophylaxis for his household is required at this stage.

Incorrect. Correct answer is E


45% answered correctly

Explanation:

Correct Answer Is E

This child has signs and symptoms of whooping cough or pertussis. Pertussis is a respiratory infection caused by
Bordetella pertussis. The clinical illness is divided into three stages- catarrhal, paroxysmal, and convalescent:

Catarrhal phase - Mild cough and coryza often lasting 1 to 2 weeks. Fever is uncommon, and if present, is usually low
grade. Instead of improving, the cough gradually increases, unlike a typical upper respiratory infection.

Paroxysmal phase - In the paroxysmal phase, coughing persists and severity increases, occurring in paroxysmal
attacks. The classic cough of pertussis is distinctive. The paroxysm is a long series of coughs, during which the child
may develop gagging and cyanosis and appear to be struggling for breath. Whooping, which is characterized by the
noise of the forced inspiratory effort, may be observed following a coughing attack during the paroxysmal phase.
Post-tussive vomiting occurs frequently.

Convalescent phase - In convalescent phase, the cough begins to subside, and the child enters the convalescent
stage. The cough continues to decrease gradually over several weeks to months.

Antibiotics are usually given to reduce the spread of the disease to others; however there is little evidence about
improvement of the patient's symptoms with antibiotics, unless it is given in the catarrhal phase (less than 14 days
of symptoms) which is not the case most of the time. This child has had the symptoms for 4 weeks and is over the
infectious period. Therefore, antibiotics at this stage are not likely to reduce the communicability of the disease
(option A); neither do they decrease the symptoms (option B).

Unimmunized (<3 doses) household and close children contacts less than 7 years of age must be excluded from
school or child care for 14 days from the last exposure to infection OR until they have taken 5 days of effective
antibiotics. The index case should be excluded from school while in the infectious period or until he receives 5 days
of antibiotics.This child is beyond the infectivity period is school exclusion for him is not necessary anymore (option
C).

Chemoprophylaxis to prevent secondary transmission is not recommended in most situations because of the
delayed presentation of the index case and the cost and adverse effects of antibiotics. However, given the high risk
of mortality and morbidity associated with infection of newborns, particularly in the context of the rising incidence of
pertussis in the community and the high transmission rate, chemoprophylaxis is recommended to limit transmission
to those most at risk of the infection (young infants). Data to support this recommendation are limited.

Current guidelines in Australian recommend post-exposure chemoprophylaxis for contacts to whom transmission is
most likely, and when there is significant risk of morbidity or mortality or risk of transmission to other high-risk
groups.

The following criteria must be met for commencement of prophylactic antibiotics:

1297 of 1943
The close contact with confirmed case of pertussis (index case) while index case is infectious which is within
the first 21 days of cough and <5 days of antibiotics

AND

First contact was within 14 days (or within 21 days for infants <6 months) – beyond this period time
prophylaxis is not effective

If the above criteria are met, the following groups should receive prophylactic antibiotics:

Children

Age <6 months; OR


<3 doses pertussis vaccine; OR
There is a household member aged <6 months; OR
Children who attend childcare in same room as infant <6 months

Adults (regardless of immunization status)

Expectant parents in last month of pregnancy; OR


Health care worker in maternity hospital or newborn nursery; OR
Childcare worker in close contact with infants <6 months; OR
Household member aged <6 months

Since the above criteria are not met for the household in this scenario, no antibiotic prophylaxis for the index case’s
household is required at this stage.

References

• RCH - Whooping cough (pertussis)

• CDNA National Guidelines for Public Health Units - Pertussis

• RACGP - Pertussis Presentation, investigation and management

Time spent: QID:759 Last updated:


2023-2-12

1298 of 1943
A 6-month-old male infant was brought to your clinic due to characteristic cough of pertussis. Which one of the
following is the best advice for the patient’s parents to reduce their risk of infection?

A. Commence a 3-dose pertussis revaccination.


B. Immediate booster immunizations against pertussis.
C. A 5-day course of azithromycin.
D. No action is required.
E. Immediate immunization with pertussis immunoglobulin.

Incorrect. Correct answer is B


45% answered correctly

Explanation:

Correct Answer Is B

Pertussis is a respiratory infection caused by Bordetella pertussis. The disease often has an incubation period of 7
to 10 days. In unvaccinated individuals, Bordetella pertussis is highly infectious; spreading by aerosols to 90% of
susceptible household contacts.

Antibiotics, if given early during the paroxysmal and catarrhal stages, will cause amelioration of the symptoms.
However, more patients do not present at this stage and come to medical attention when the been disease
has established. At this point the, role of antibiotics is limited to preventing the disease from more spread to the
patient’s contacts. It should be remembered that patients are rarely infectious after 3 weeks.

PatiePatientsld be advised to avoid contact with other people, until 5 days after antibiotics are started. The
antibiotics used in pertussis for prevention of the disease spread for index case are:

(First –line) Azithromycin 500 mg (child 6 months or older: 10 mg/kg up to 500 mg) orally, for the first dose,
then 250 mg (child 6 months or older: 5 mg/kg up to 250 mg) orally, daily for a further 4 days (neonate and
child younger than 6 months: 10 mg/kg orally, daily for 5 days)

OR

(First-line) Trimethoprim + sulfamethoxazole 160+800 mg (child 1 month or older: 4+20 mg/kg up to 160+800
mg) orally, 12-hourly for 7 days

OR

(Second-line) Clarithromycin 500 mg (child: 7.5 mg/kg up to 500 mg) orally, 12-hourly for 7 days

NOTE - There is no clinical evidence to support roxithromycin for the treatment of pertussis

regarding prophylactic measures, the following guidelines are currently in place:

Prophylaxis with antibiotics for household and close contacts:

Chemoprophylaxis to prevent secondary transmission is not recommended in most situations because of the
delayed presentation of the index case and the cost and adverse effects of antibiotics. However, given the high risk
of mortality and morbidity associated with infection of the newborn, particularly in the context of the rising incidence
of pertussis in the community and the high transmission rate, chemoprophylaxis with antibiotics is recommended to
limit transmission to those most at risk of the infection (young infants). Data to support this recommendation are
limited.
1299 of 1943
Australian guidelines recommend post-exposure chemoprophylaxis with antibiotics for contacts to whom the
transmission is most likely, and when there is a significant risk of morbidity or mortality or risk of transmission to
other high-risk groups. These groups include:

all household contacts of an index case when the household includes children less than two years who have
received less than three doses of vaccine (including newborn infants)
any woman in the last month of pregnancy
all adults and children in a childcare arrangement with an index case, if the group contains children less than
two years who have received less than three doses of vaccine
healthcare workers in maternity and neonatal units
infants in maternity and neonatal units where a healthcare worker was the infected case.

Therapy must be started within 21 days of exposure to the index case to be effective.

Vaccination

Australian guidelines recommend a single dose of an acellular vaccine for contacts of pertussis older than eight
years, and catch-up vaccination for unvaccinated or partially vaccinated (incomplete infant vaccination) contacts up
to their eighth birthday.

This child's parents do not require prophylaxis with antibiotics (option C) because they do not fulfill the mentioned
criteria; however, based on current guidelines, they should receive a booster dose of the pertussis vaccine. Passive
immunization with pertussis immunoglobulin (option E) is not effective in preventing disease spread. A full
vaccination with 3 doses of pertussis-containing vaccines as a catch-up program is recommended for children up to
8 years old.

References

• RCH - Whooping cough (pertussis)

• CDNA National Guidelines for Public Health Units - Pertussis

• RACGP - Pertussis Presentation, investigation and management

• Australian Prescriber - Pertussis prophylaxis

Time spent: QID:760 Last updated:


2023-2-12

1300 of 1943
A 7-year-old boy is brought to your clinic with a 2-week history of paroxysms of cough and vomiting at the end of
coughing episodes during school holidays. Cough is dry and is worse at night. His mother is worried whether he
should go to school, which is going to start in 2 days. Which one of the following would be the most appropriate
approach regarding this situation?

A. No exclusio is required.
B. Exclude from school.
C. Exclude from the school and start antibiotics.
D. The child can go to school after antibiotics are started.
E. The decision depends on the child’s immunization status.

Correct
45% answered correctly

Explanation:

Correct Answer Is A

The clinical picture is consistent with whooping cough (pertussis) caused by Bordetella pertussis. The incubation
period is of 7 to 14 days (up to 20 days), and the disease then appears with the following three stages:

1. Catarrhal – Cough and coryza for one week


2. Paroxysmal – more pronounced cough in spells or paroxysms
3. Convalescence

Antibiotics for the case with pertussis (index case) are indicated in the following situations:

The patient is diagnosed in catarrhal or early paroxysmal phase (may reduce severity)
Cough for less than 14 days (may reduce spread; reduces school exclusion period)
The patient is admitted to the hospital
There are complications (e.g., pneumonia, cyanosis, apnea)

Exclusion of the case and close contacts are based on the following recommendations from National Health and
Medical Research Council (NHMRC) and Australian Health Department:

Exclusion of the index case: exclude until 5 days after starting appropriate antibiotic treatment, or for 21 days from
the onset of any cough (catarrhal phase), or 14 days after the onset of paroxysmal cough if the date is known.

Exclusion of close contacts: unimmunized (< 3 doses) household and close childcare contacts less than 7 years of
age must be excluded from school or child care for 14 days from the last exposure to infection OR until they have
taken 5 days of effective antibiotics for prophylaxis.

Since school time is after two weeks (14 days) of the date the cough paroxysm has been started, the case is not
infectious and no exclusion from school will be require

Antibiotics would have been indicated and could have reduced exclusion period if the presentation had been within
14 days of cough. With 14 days of paroxysmal cough (that would be 21 days of cough of catarrhal phase) this child
is at the end of the infectious period (21 days).

Immunization status guides exclusion of the close contacts from school or childcare settings, not that of the index
case.

References

1301 of 1943
• RCH - Whooping cough (pertussis)

• CDNA National Guidelines for Public Health Units - Pertussis

• RACGP - Pertussis Presentation, investigation and management


Last updated:
Time spent: QID:761 2023-2-12

1302 of 1943
A 5-month-old child is brought to your practice with a 10-day history of cough paroxysms followed by inspiratory
whoop and vomiting. There is no cyanosis associated with bouts of cough, and the child is well between the attacks.
Which one of the following is the most appropriate action now?

A. Reassure.
B. Prescribe a 5-day course of antibiotics and discharge the child home.
C. Admit the child to the hospital for observation.
D. Chemoprophylaxis of the household.
E. Vaccinate the household.

Incorrect. Correct answer is C


45% answered correctly

Explanation:

Correct Answer Is C

The clinical picture is consistent with whooping cough (pertussis) caused by Bordetella pertussis.

Pertussis is a prolonged coughing illness with varying clinical manifestations depending on age and immunization
status. An initial catarrhal phase is characterized by insidious onset of runny nose, sneezing absent or low-grade
fever, and a mild occasional cough. The cough gradually becomes paroxysmal usually after 1-2 weeks, and may end
in vomiting, cyanosis, and/or a characteristic high-pitched inspiratory whoop. Paroxysms can recur with subsequent
respiratory illnesses for many months after the onset of pertussis. Fever is generally minimal or absent throughout
the course of the disease. Infants are less likely to have the inspiratory whoop and a significant catarrhal phase and
are more likely to present with gagging, gasping, cyanosis, apnea or non-specific signs such as poor feeding or
seizures.

Current Guidelines in Australia recommend that the following patients with pertussis are admitted and managed
in hospital:

Infants less than 6 months of age


Any child with complications (i.e. apnea, cyanosis, pneumonia, encephalopathy)

Since this child is younger than 6 months, admission to hospital and further management is the most appropriate
next step.

Antibiotics are indicated only in the following situations:

The patient is diagnosed in catarrhal or early paroxysmal phase (may reduce severity)
Cough for less than 14 days (may reduce spread; reduces school exclusion period)
The patient is admitted to hospital
There are complications (e.g., pneumonia, cyanosis, or apnea)

After admission, this child will require antibiotics as well, but this not take precedence over hospital admission.
Azithromycin is an appropriate option.

Chemoprophylaxis would have been indicated if there was a child <6 months or someone with <3 doses of vaccine
among the household (besides the index case). Catching up with immunization, at a later date, should be
encouraged even if the child has had pertussis, but this step is not a part of acute management.

References

• RCH - Whooping cough (pertussis)

1303 of 1943
• CDNA National Guidelines for Public Health Units - Pertussis

• RACGP - Pertussis Presentation, investigation and management


Last updated:
Time spent: QID:762 2023-2-12

1304 of 1943
A 7-month-old child is brought to your practice with paroxysmal cough and vomiting for 2 weeks and relative
wellness and health in between paroxysms. You suspect he may have whooping cough (pertussis). He has already
been started on azithromycin. He lives at home with his mother, father and older brothers, aged 3 and 5 years.
Neither of his brothers have been immunized against pertussis. Which one of the following options is the most
appropriate next action to take?

A. Arrange to have the child admitted to hospital, and isolated immediately.


B. Report the family to the child protection agency in your state for failing to immunize their children.
C. Immediately vaccinate his brothers.
D. Prescribe azithromycin for the whole family.
E. Take no action.

Incorrect. Correct answer is C


45% answered correctly

Explanation:

Correct Answer Is C

This child is older than 6 months and does not have any serious complication of Bordetella pertussis infections such
as cyanosis, apnea or pneumonia; therefore, admission to hospital is not required.

The decision as to whether a child is immunized is made by the parents. Parents can decide to not immunize their
children and it is not an act subject to notification to child protection authorities.

Parents can be talked into vaccinating their children for their benefit but they cannot be forced to because this is
their right to refuse vaccination of their children. Moreover, vaccination at this point will not help the current
situation.

Prophylaxis against pertussis in an attempt to decrease spread to the community is indicated in the following
situations:

The close contact with confirmed case of pertussis (index case) while index case is infectious, which is within
the first 21 days of cough and <5 days of antibiotics

AND

First contact was within 14 days (or within 21 days for infants <6 months) – beyond that time prophylaxis is
not effective

If the above criteria are met, the following groups should receive prophylactic antibiotics:

Children

Age <6 months; OR


<3 doses pertussis vaccine; OR
There is a household member aged <6 months; OR
Children who attend childcare in same room as infant <6 months

Adults (regardless of immunization status)

Expectant parents in last month of pregnancy; OR


Healthcare workers in maternity hospital or newborn nursery; OR
Childcare worker in close contact with infants <6 months; OR
Household members aged <6 months

1305 of 1943
Prophylaxis is indicated for the 3- and 5-year-old children, but not for parents. Therefore, the option suggesting that
all family members should receive prophylactic antibiotics is incorrect. It is recommended that close contacts that
are not up-to-date with their pertussis immunization should be given DTPa or dTpa as soon after exposure as
possible. In the absence of an option suggesting prophylaxis for 3- and 5-year-old children, immediate vaccination of
these two children would be the most appropriate alternative option. Vaccinated adults, who have not had pertussis-
containing vaccine in the last 10 years, should receive a booster dose by dTpa.

References

• RCH - Whooping cough (pertussis)

• CDNA National Guidelines for Public Health Units - Pertussis

• RACGP - Pertussis Presentation, investigation and management

Time spent: QID:763 Last updated:


2023-2-12

1306 of 1943
Which one of the following statements is correct about pertussis?

A. Febrile convulsions are not a contraindication for vaccination against pertussis.


B. DTPa cannot be given with another live virus-containing vaccine.
C. Pertussis vaccine should not be given to a child with infantile convulsions.
D. Erythromycin reduces the severity of the signs and symptoms.
E. Prophylaxis is always indicated for the entire household.

Correct
45% answered correctly

Explanation:

Correct Answer Is A

Febrile convulsions are very rarely reported following DTPa-containing vaccines within 48 hours of vaccination. The
risk is even lower in infants who complete their primary course at 6 months of age because febrile convulsions are
uncommon in children <6 months of age. Children who experience a febrile convulsion after a dose of DTPa-
containing vaccine have a slightly increased risk of further febrile convulsions following a subsequent dose of DTPa-
containing vaccine. The risk can be minimized by appropriate measures to prevent fever, so vaccination is still
recommended.

(Option B) Simultaneous administration (that is administration on the same day) of the most widely used live and
inactivated vaccines does not result in decreased antibody responses or increased rates of adverse reactions.
Simultaneous administration of all vaccines for which a child is eligible is very important in childhood vaccination
programs because it increases the probability that a child will be fully immunized at an appropriate age; therefore,
DTPa can be given with other live-virus vaccines.

(Option C) Pertussis-containing vaccines do not cause infantile spasms or epilepsy. Infants and children known to
have active or progressive neurological disease can be safely vaccinated with DTPa-containing vaccines.

(Option D) Antibiotics are not likely to reduce the severity of signs and symptoms; however, they can reduce the
infectious period, and decrease the communicability of the disease.

(Option E) Chemoprophylaxis to prevent secondary transmission is not recommended in most situations because of
the delayed presentation of the index case and the cost and adverse effects of antibiotics. However, given the high
risk of mortality and morbidity associated with infection of the newborn, particularly in the context of the rising
incidence of pertussis in the community and the high transmission rate, chemoprophylaxis is recommended to limit
transmission to those most at risk of the infection (young infants). Data to support this recommendation are
limited. Australian guidelines recommend post-exposure chemoprophylaxis for contacts to whom transmission is
most likely, and when there is significant risk of morbidity or mortality or risk of transmission to other high-risk
groups. Routine chemoprophylaxis of all household is not indicated unless certain criteria are met.

References

• Australian Immunisation Handbook - Pertussis (whooping cough)

• CDC - Immunology and Vaccine-Preventable Diseases – Pink Book –General Recommendations

• CDNA National Guidelines for Public Health Units - Pertussis

Last updated:
Time spent: QID:764 2023-2-12

1307 of 1943
Which one of the following statements is correct about pertussis?

A. Febrile convulsions are not a contraindication for vaccination against pertussis.


B. DTPa cannot be given with another live virus-containing vaccine.
C. Pertussis vaccine should not be given to a child with infantile convulsions.
D. Erythromycin reduces the severity of the signs and symptoms.
E. Prophylaxis is always indicated for the entire household.

Correct
45% answered correctly

Explanation:

Correct Answer Is A

Febrile convulsions are very rarely reported following DTPa-containing vaccines within 48 hours of vaccination. The
risk is even lower in infants who complete their primary course at 6 months of age because febrile convulsions are
uncommon in children <6 months of age. Children who experience a febrile convulsion after a dose of DTPa-
containing vaccine have a slightly increased risk of further febrile convulsions following a subsequent dose of DTPa-
containing vaccine. The risk can be minimized by appropriate measures to prevent fever, so vaccination is still
recommended.

(Option B) Simultaneous administration (that is administration on the same day) of the most widely used live and
inactivated vaccines does not result in decreased antibody responses or increased rates of adverse reactions.
Simultaneous administration of all vaccines for which a child is eligible is very important in childhood vaccination
programs because it increases the probability that a child will be fully immunized at an appropriate age; therefore,
DTPa can be given with other live-virus vaccines.

(Option C) Pertussis-containing vaccines do not cause infantile spasms or epilepsy. Infants and children known to
have active or progressive neurological disease can be safely vaccinated with DTPa-containing vaccines.

(Option D) Antibiotics are not likely to reduce the severity of signs and symptoms; however, they can reduce the
infectious period, and decrease the communicability of the disease.

(Option E) Chemoprophylaxis to prevent secondary transmission is not recommended in most situations because of
the delayed presentation of the index case and the cost and adverse effects of antibiotics. However, given the high
risk of mortality and morbidity associated with infection of the newborn, particularly in the context of the rising
incidence of pertussis in the community and the high transmission rate, chemoprophylaxis is recommended to limit
transmission to those most at risk of the infection (young infants). Data to support this recommendation are
limited. Australian guidelines recommend post-exposure chemoprophylaxis for contacts to whom transmission is
most likely, and when there is significant risk of morbidity or mortality or risk of transmission to other high-risk
groups. Routine chemoprophylaxis of all household is not indicated unless certain criteria are met.

References

• Australian Immunisation Handbook - Pertussis (whooping cough)

• CDC - Immunology and Vaccine-Preventable Diseases – Pink Book –General Recommendations

• CDNA National Guidelines for Public Health Units - Pertussis

Last updated:
Time spent: QID:764 2023-2-12

1308 of 1943
A fully breast-feed baby of a mother on a healthy, balanced diet could have the deficiency of which one of the
following vitamins?

A. Vitamin A.
B. Vitamin B complex.
C. Vitamin C.
D. Vitamin D
E. Vitamin K.

Incorrect. Correct answer is E


45% answered correctly

Explanation:

Correct Answer Is E

Breast milk lacks or has insufficient vitamin K, but as this vitamin is given intramuscularly at birth, there is no more
need for supplementation. Without supplementaion at birth, the baby may suffer vitamin K deficiency and
develop severe complications such as intracranial hemorrhage. Vitamin K deficiency is more common among
premature babies and those whose mothers have been on antiepileptic drugs.

References

• PubMed - Are breastfed infants vitamin K deficient?

Last updated:
Time spent: QID:111 2023-2-12

1309 of 1943
Which one of the following is the most common cause of persistent cough in children?

A. Post-viral cough.
B. Asthma.
C. Post-pertussis.
D. Gastro-esophageal reflux disease (GERD).
E. Passive smoking.

Incorrect. Correct answer is B


45% answered correctly

Explanation:

Correct Answer Is B

By duration, cough is defined as acute (<3 weeks), subacute (3-8 weeks) and chronic or persistent (>8 weeks).
Asthma is the most common cause of persistent (chronic) cough in children and the second most common cause in
adults (after chronic bronchitis).

Post-viral cough is the most common cause of acute and subacute (not chronic) cough in children.

Other causes of persistent cough in children are as follows:

Persistent bacterial bronchitis


Chronic suppurative lung disease and bronchiectasis
Foreign body
Gastro-esophageal reflux disease (GERD)
Upper respiratory cough syndrome (formerly known as postnasal drip syndrome)
Post-pertussis cough
Increased cough receptor sensitivity
Habit (psychogenic) cough
Otogenic cough

References

• UpToDate - Causes of chronic cough in children

• UpToDate - Evaluation of subacute and chronic cough in adults

• Murtagh’s General Practice – 6th Edition - McGraw Hill – page 469

Last updated:
Time spent: QID:765 2023-2-12

1310 of 1943
A mother brings her 3-week-old male baby to the Emergency Department because of his loose bloody bowel
motions. The mother says the diarrhea has started 2 days ago after she started her son on formula milk. The mother
is lactose-intolerant and does not use dairy products. On examination, the child is mildly dehydrated and has an
eczematous rash on his right cheek. He is not febrile. The rest of the exam is inconclusive. Family history is
remarkable for atopy including asthma and eczema in his father and older sister. Mother is concerned that his son
may have the same allergy to milk and dairy product she has. Which one of the following is the most likely
diagnosis?

A. Lactose intolerance.
B. Gastroenteritis.
C. Galactosemia.
D. Fructose intolerance.
E. Cow’s milk protein intolerance.

Incorrect. Correct answer is E


45% answered correctly

Explanation:

Correct Answer Is E

The onset of bloody diarrhea shortly after introduction of formula, the presence of eczema, and family history of
atopy makes cow’s milk protein allergy (intolerance) (CMPA) the most likely diagnosis with high certainty.

CMPA results from an immunological reaction to one or more milk proteins. This immunological basis distinguishes
CMPA from other adverse reactions to cow’s milk protein such as lactose intolerance.

CMPA may be immunoglobulin E (IgE) or non-IgE associated. In IgE-associated cases, CMPA may be a manifestation
of the atopic diathesis. These reactions may occur as short as minutes after ingestion of cow’s milk or cow’s milk-
based formula. These early reactions usually manifest as urticaria, angioedema, vomiting or an acute flare of atopic
dermatitis. The remaining 42% show a later reaction, typically atopic dermatitis or gastrointestinal disturbances.

Even small amounts of cow’s milk in breast milk of mother’s who take dairy products may trigger the condition.

Other options do not justify the clinical presentation.

References

• MJA - Guidelines for the use of infant formulas to treat cows milk protein allergy: an Australian consensus panel
opinion

• BMJ - Guidelines for the diagnosis and management of cow’s milk protein allergy in infants

Last updated:
Time spent: QID:815 2023-2-12

1311 of 1943
A 2-year-old boy is brought to the Emergency Department, at 11 pm, by his parents with complaints of a harsh
barking cough. On examination, he is febrile with a temperature of 39°C and has a respiratory rate of 32 breaths per
minute. There is inspiratory stridor as well as scattered crackles over both lung fields. He is easily irritated but can be
consoled by the parents. Which one of the following is most important to give him first?

A. Intravenous prednisolone.
B. Nebulized adrenaline.
C. Nebulized salbutamol.
D. Oral steroids.
E. Antibiotics.

Incorrect. Correct answer is D


45% answered correctly

Explanation:

Correct Answer Is D

The age of the child, the fever, the harsh cough and more importantly the stridor is suggestive of croup until proven
otherwise.

The term croup has been used to describe a variety of respiratory conditions in children, including:

Laryngitis - Inflammation is limited to the larynx and manifests as hoarseness. It usually occurs in older
children and adults and, similar to croup, it is frequently caused by a viral infection.

Laryngotracheitis - Refers to inflammation of the larynx and trachea. Although lower airway signs are absent,
the typical barking cough will be present.

Laryngotracheobronchitis - The condition occurs when inflammation extends into the bronchi, resulting in
lower airway signs (e.g. wheezing, crackles, air trapping, tachypnea) and sometimes more severe illness than
laryngotracheitis alone. This term commonly is used interchangeably with laryngotracheitis, and the entities
are often indistinct clinically. Further extension of the inflammation into the lower airways results in
laryngotracheobronchopneumonitis, which sometimes can be complicated by bacterial superinfection.
Bacterial superinfection can manifest as pneumonia, bronchopneumonia, or bacterial tracheitis.

NOTE - Laryngotracheitis and laryngotracheobronchitis are the conditions that have been interchangeably
considered as classic croup.

Bacterial tracheitis - Bacterial tracheitis (also called bacterial croup) describes bacterial infection of the
subglottic trachea, resulting in a thick, purulent exudate, which causes symptoms of upper airway obstruction.
The bronchi and lungs are typically involved, as well (i.e. bacterial tracheobronchitis). Bacterial tracheitis may
occur as a complication of viral respiratory infections or as a primary bacterial infection.

Spasmodic croup - Spasmodic croup is characterized by the sudden onset of inspiratory stridor at night, short
duration (several hours), and sudden cessation. This is often in the setting of a mild upper respiratory
infection, but without fever or inflammation. A striking feature of spasmodic croup is its recurrent nature.
Because of some clinical overlap with atopic diseases, it is sometimes referred to as 'allergic croup'.

Croup in most commonly seen in children aged 6 to 36 months of age. It is also seen in children as young as 3
months and preschool children but rare beyond these age extremes.

Viruses are almost always the cause with parainfluenza virus type 1 being the most common culprit. Other common
viruses are respiratory syncytial virus (RSV), influenza virus, and rhinovirus. Bacterial croup can be primary, for which

1312 of 1943
mycoplasma species are the most common etiology, or secondary due to superinfection with staphylococcus aureus
(most common cause of superinfection), streptococcus pyogenes or streptococcus pneumoniae

The most common presentation is a harsh cough that in younger children takes the quality of barking, inspiratory
stridor and hoarseness. When the inflammation extends to involve the structures below the trachea, e.g. bronchi,
crackles and wheezes may be present as well. Fever is not a prominent feature; however, there may be a fever as
high as 39°C, especially when the lower parts of the respiratory tract are involved or when there is bacterial croup.

Based on the clinical findings suggestive of the severity of airway obstruction, croup can be categorized as mild,
moderate or severe:

Mild – A child with mild airway obstruction appears happy and is prepared to drink, eat, play and interested in
the surroundings. Most children presenting to general practitioners fall within this category. There might be
mild chest wall retraction and mild tachycardia, and stridor with activation or agitation, but there is no stridor
at rest.

Moderate – Moderate airway obstruction is indicated by persisting stridor at rest, chest wall retraction, use of
accessory muscles and increased heart rate. The child can be consoled and remains interactive with people
and the surroundings.

Severe – The child appears increasingly worried, tired and exhausted. There is marked tachycardia unjustified
by the fever. Restlessness, agitation, irrational behavior, decreased level of consciousness, hypotonia,
cyanosis and marked pallor are late signs of life-threatening airway obstruction.

The following table outlines the clinical findings in different forms of croup:

Mild Moderate Severe


Some/intermittent Increasing irritability
Behavior Normal
irritability and/or lethargy
Barking cough
Stridor Some stridor at rest Stridor present at rest
Stridor only when
active or upset
Increased respiratory Marked increased
rate respiratory rate
Respiratory rate Normal
Tracheal tug Tracheal tug

Nasal flaring Nasal flaring


Accessory muscle Moderate chest wall Marked chest wall
None or minimal
use retraction retraction
Hypoxia is a late sign
No oxygen No oxygen
Oxygen of significant upper
requirements requirements
airway obstruction

NOTE – Loudness of stridor is not a good indicator of severity and should not be relied upon.

Management of croup depends on the severity:

In mild cases (a croupy cough but no history of noisy breathing in a child older than 2 years), no treatment is
required, but early review is essential.
1313 of 1943
In moderate to severe cases (e.g., a history of noisy breathing or age less than 2 years, or presence of stridor on
examination), a single dose of corticosteroid is indicated. Use either:

Dexamethasone 0.3 mg/kg orally (first-line); or


Predniso(lo)ne 1mg/kg orally, or
Budesonide 2mg by nebulizer

Further doses are usually not needed, but can be considered if response to therapy is suboptimal at 24 hours

In more severe cases with significant airway obstruction or fatigue, treatment in hospital may be required with an
initial dose of:

Adrenaline 1% (1:100, 10mg/ml) solution 0.05ml/kg/dose up to 0.5ml diluted with NaCl 0.9%, by nebulizer

PLUS EITHER

Dexamethasone 0.6mg/kg orally (or IM if vomiting), or


Predniso(lo)ne 1mg/kg orally

Maintenance dose and frequency vary with the severity of the condition and the response to treatment.

This child has stridor and is irritable but consolable; therefore, he is most likely having moderate croup. He should be
given a single dose of oral dexamethasone or predniso(lo)ne or nebulized budesonide.

(Option A) Intravenous administration of steroids is not indicated in patients with croup.

(Option B) Nebulized adrenalin should be reserved for cases with severe croup.

(Option C) Bronchodilators (e.g., salbutamol) in any form have no role in treatment of croup.

(Option E) Antibiotics would be indicated if the underlying cause is found to be bacterial tracheitis, but airway always
takes precedence.

References

• UpToDate - Croup: clinical features, evaulation and diagnosis

• RCH - Croup (Laryngotracheobronchitis)

Time spent: QID:856 Last updated:


2023-2-12

1314 of 1943
A 4-year-old girl is presented with the proptosis of her left eye and a white cornea which was noticed yesterday by
her parents. Based on the exam findings, you suspect retinoblastoma. Which one of the following is the most
appropriate initial diagnostic test?

A. MRI.
B. Ultrasound.
C. CT scan.
D. X-ray.
E. Biopsy.

Incorrect. Correct answer is B


45% answered correctly

Explanation:

Correct Answer Is B

This child has suspected diagnosis of retinoblastoma.

Retinoblastoma is the most common intraocular tumor in children. Leukocoria (white cornea or Cat’s eye) is the
most common presenting sign in developed countries, followed by strabismus as the second most common feature.
Other findings include decreased vision, ocular inflammation, vitreous hemorrhage, hyphema, orbital cellulitis,
proptosis, glaucoma, eye pain, and fever. A family history is present in 5 -10% of cases.

The diagnosis is usually made by dilated indirect ophthalmoscopic examination under anesthesia. The characteristic
finding is a chalky, white-grey retinal mass with soft and friable consistency. Ultrasonography is cheap and readily
available and is used in conjunction with physical exam to establish the diagnosis. Characteristic finding on
ultrasonography is intraocular calcification.

(Option A) Together with ultrasonography, high-resolution MRI has emerged as an important imaging modality for
pretreatment assessment and diagnostic confirmation, detection of local tumor extension, involvement of the optic
nerve, detection of associated developmental malformations of the brain, and detection of associated intracranial
primitive neuroectodermal tumors (trilateral retinoblastoma). However, MRI is not the initial investigation and is
considered after physical exam, funduscopy and ultrasound established the diagnosis.

(Options C and D) In the past, CT was used to determine tumor size, retro-orbital spread and intracranial growth;
however, because of its superior soft-tissue contrast, MRI is more sensitive and specific than CT in detection of
tumor extent and metastatic risk factors. Furthermore, CT and x-rays scan poses children with hereditary
retinoblastoma at increased risk of other malignancies given the fact that hereditary retinoblastoma is caused by
mutation in RB1 gene which is a tumor suppressor. Routine use of CT scan as a diagnostic modality is
discouraged; X-ray carries the same risk.

(Option E) Biopsy has no role in diagnosis of retinoblastoma.

References

• Guidelines for imaging retinoblastoma: imaging principles and MRI standardization

• UpToDate - Overview of retinoblastoma

Last updated:
Time spent: QID:887 2023-2-12

1315 of 1943
Which one of the following is the most common cause of painful rectal bleeding in children?

A. Intussusception.
B. Gastroenteritis.
C. Anal fissure.
D. Meckel’s diverticulum.
E. Hemorrhoids.

Incorrect. Correct answer is C


45% answered correctly

Explanation:

Correct Answer Is C

Anal fissure is the most common cause of painful rectal bleeding in childhood. It is usually caused by passage of
hard stool associated with constipation. The common presentation is bright blood on the surface of stool, on the
nappy or toilet paper.

(Option A) Intussusception is associated with abdominal pain and rectal bleeding, not usually with pain on
defecation and not as common as anal fissure.

(Option B) Gastroenteritis and juvenile polyps are the most common causes of rectal bleeding in children aged 2 to 5
years, but these conditions are not associated with painful defecation.

(Option D) Meckel's diverticulum can cause blood in the stool and abdominal pain but the pain is not related to
defecation. Moreover, it is far less common than anal fissure.

(Option E) Hemorrhoids neither are associated with painful defecation, nor as common in childhood.

References

• Medscape - https://emedicine.medscape.com/article/1955984-overview#a5

• UpToDate - Lower gastrointestinal bleeding in children: Causes and diagnostic approach

• AMC Handbook of Multiple Choice Questions – page 126

Last updated:
Time spent: QID:904 2023-2-12

1316 of 1943
Concerned parents of a 7-year-old boy have brought him to your clinic after he passed a large bloody stool this
morning. No pain or discomfort is reported. Which one of the following can be the most likely cause?

A. Meckel’s diverticulum.
B. Duodenal ulcer.
C. Colon polyp.
D. Diverticulitis of the colon.
E. Intussusception.

Incorrect. Correct answer is C


45% answered correctly

Explanation:

Correct Answer Is C

A large bloody stool, especially if painless, in a 7-year-old child is very likely to be caused by a juvenile colonic polyp.

Juvenile polyps are benign hamartomas, occurring between the ages 2 and 8 years, with a peak at 3 to 4 years.
Patients usually present with painless rectal bleeding; however, a few children may have lower abdominal pain from
traction of the polyp. More than 60% of patients have a palpable polyp on rectal examination.

(Option A) Meckel’s diverticulum can result in massive rectal bleeding, often associated with central abdominal pain.
However, the presentation is expected earlier in life and is also less common than juvenile polyps.

(Option B) Duodenal ulcer is most likely to present with melena or hematemesis rather than large rectal bleeding.

(Options D and E) Diverticular disease is not common among children of this age group, neither is intussusception.
Moreover, intussusception is often painful.

References

• Medscape - Pediatric Gastrointestinal Bleeding

• Canadian Journal of Gastroenterology - Colonic polyps in children and adolescents

Last updated:
Time spent: QID:905 2023-2-12

1317 of 1943
A 5-year-old boy is brought to your clinic with generalized petechial rash and bruises all over his body. He had an
episode of upper respiratory tract infection (URTI) 3 weeks ago. On examination, he is otherwise healthy. His blood
tests is only significant for a platelet count of 74,000/mm3. Red blood cells and white cells are within normal range.
Which one of the following is the most appropriate management option in this child?

A. Strict bed rest.


B. Intravenous immunoglobulin (IVIG).
C. Steroids.
D. Antibiotics.
E. Plasma exchange.

Correct
45% answered correctly

Explanation:

Correct Answer Is A

Isolated thrombocytopenia in an otherwise healthy child following an URTI is highly suggestive of immune
(idiopathic) thrombocytopenic purpura (ITP). ITP in children is a benign disease of unknown etiology.

ITP can be acute (~90%) or chronic (~10%). Both forms have similar presentation. Twenty percent of those with
acute ITP will go on to have the chronic form, defined as persistent thrombocytopenia more than 6 months. However,
some authors have suggested that 12 months is a more appropriate cut-off for defining chronic ITP.

The diagnosis is made based on manifestations of thrombocytopenia in the absence of abnormal findings, in
particular no pallor, lymphadenopathy or hepatosplenomegaly. Confirmation relies on exclusion of other causes of
thrombocytopenia, acute leukemia in particular.

The management of children with ITP is controversial. There are generally two different approaches: (1) watchful
waiting and (2) pharmacologic intervention. None has been proved superior to the other.

Most patients with a platelet count > 20 x 109/L and some of those with a platelet count < 20 x 109/L can be
managed in outpatient setting with no specific treatment if the following criteria are met:

The diagnosis is certain, and there is no pallor, hepatosplenomegaly or lymphadenopathy, and there is only
isolated thrombocytopenia without anemia, leukopenia or blood film changes on laboratory studies
There is no active bleeding other than bruising and petechiae in isolation. There should be no mucosal,
gastrointestinal or renal tract bleeding
The child is otherwise well
Social circumstances allow confidence about the degree of parental supervision and relative safety of the
home environment, particularly for younger children.
Parental reassurance and education in the emergency department can be provided.
Follow up is guaranteed within a few days.

(Option B) Intravenous immunoglobulin (IVIG) is not routinely used and is reserved for patients with severe life-
threatening hemorrhage.

(Option C) This child has platelet count of >20,000/mm3, has no hemorrhage other than bruises and petechial rash
and is otherwise healthy. He does not need pharmacological treatment. However, if pharmacological intervention is
planned, oral steroids are first-line.

(Option D) Antibiotics have no role in management of ITP.

(Option E) Plasma exchange is not an option for treatment of ITP.

1318 of 1943
References

• RCH - Immune thrombocytopenic purpura

• UpToDate - Immune thrombocytopenia (ITP) in children: Management of chronic disease

• Medscape - Immune Thrombocytopenia (ITP)

Last updated:
Time spent: QID:906 2023-2-12

1319 of 1943
A mother brings her 7-year-old son 6 days after eruption of a generalized rash that, based on the history and clinical
findings, is found to be measles. She asks if her son is required to be excluded from school. She also mentions that
according to the class teacher, all students in the class have complete vaccination against measles. Which one of
the following would be the most appropriate advice regarding exclusion from school for this boy?

A. No exclusion is required.
B. Exclusion for 4 days.
C. Exclusion until the rash completely disappears.
D. Exclusion is required only if there is an immunocompromised child in the class room.
E. Exclusion for 6 days.

Correct
45% answered correctly

Explanation:

Correct Answer Is A

Measles is highly infective and all suspected, probable and confirmed cases should be excluded from work, school,
and early childhood education and care services. Patients with established measles should be excluded until 4 days
after the onset of rash. This child has had eruption of his rash 7 days ago; therefore, he is not infective anymore and
exclusion is not required.

TOPIC REVIEW

Since measles is highly infective, it is very important that persons who has been in contact with a person with
measles (index case) are traced and excluded from contacts to others accordingly. It is even more crucial in
childcare setting. The following are recommendations regarding exclusion of contacts of known case of measles in
childcare setting:

Immunized contacts of a child with measles do not require exclusion.


Non-immunized children, who have been in contact with a child with measles should be excluded from school
until 14 days after the day the infected child developed the rash, unless they are immunized with MMR within
72 hours of contact with the infected person.

Example: John develops a rash 3 days ago. There are four other children in his class: Alan, Josh, Mary, and Ana. These
children are all 7 years old. Ana contracted measles at the age of 3, and Josh has had all his childhood vaccines to date.
Alan and Mary have not received any vaccines because their parents are against immunization. So exclusion from
school for these children would be:

John (the index case): until tomorrow which is 4 days after the onset of the rash (3 days ago).
Ana (contact): no exclusion is required because she had measles and is immune now.
Josh (contact): no exclusion is required because he is up-to-date with his vaccination.
Alan (contact): he is not immune so he should be excluded from school for 11 days from today because John
developed the rash 3 days ago.
Mary (contact): she is not immune so she should be excluded from school for 11 days from today because John
developed the rash 3 days ago.

References

• Australian Government, Department of Health – Measles - Measles National guidelines for public health units

Last updated:
Time spent: QID:924 2023-2-12
1320 of 1943
A 5-year-old boy is presented to your GP clinic by his parents with intermittent abdominal pain and vomiting usually
lasting for 12 hours. The last painful episode was felt in the left flank. Physical examination is quite inconclusive.
You order a urine analysis and culture, the results of which are negative. Which one of the following investigations is
most likely to reveal the underlying cause of this presentation?

A. Plain abdominal films.


B. Small intestine barium meal and follow-through.
C. Serum creatinine and BUN.
D. Serum electrolytes.
E. Ultrasound scan during a painful episode.

Incorrect. Correct answer is E


45% answered correctly

Explanation:

Correct Answer Is E

Intermittent abdominal pain in children has a myriad of causes. In approximately 40%, no apparent cause is found
after extensive investigations. Such pains are referred to as functional abdominal pain.

Other causes to consider are:

Most common

Functional (most cases)


Abdominal migraine
Irritable bowel syndrome
Non-specific inflammation of the mesenteric lymph node
Constipation

Less common

Hydronephrosis
Intussusception
Inflammatory bowel disease (IBD)
Pancreatic disease
Liver or gallbladder disease
Urinary tract infections
Splenomegaly
Aerophagia
Intraabdominal tumors
Intraabdominal infections

Of this list, however, most conditions are excluded as the child is otherwise healthy and asymptomatic between
episodes of abdominal pain. Considering the presence of flank pain, a renal origin should come to mind. A urinalysis
and culture are negative. This makes using urinary tract infections unlikely and renal stones less likely with rather
high certainty. With these results, hydronephrosis caused by ureteropelvic junction obstruction (UPJ) obstruction
could be the most likely diagnosis.

UPJ obstruction is by far the most common cause of pediatric hydronephrosis, occurring in 1 per 1000-2000
newborns. Widespread use of antenatal ultrasonography and the advent of modern imaging techniques have
resulted in earlier and more common diagnosis of the condition.

UPJ obstruction can be broadly divided into the following two categories:
1321 of 1943
Lesions that involve the UPJ intrinsically
Lesions that are extrinsic

In older children, periodic abdominal pain with vomiting is a common symptom. These symptoms are probably
caused by intermittent kinking of the UPJ. If obstruction does not resolve, severe hydronephrosis and pelvic
distention can follow that presents with flank pain or even tenderness. Although rarely seen in developed countries,
massive dilation of the renal pelvis and kidney can fill the entire flank and abdomen.

In younger adults, the typical presentation is episodic flank pain following diuresis.

Ultrasonography is the initial diagnostic choice for most cases of hydronephrosis and UPJ obstruction. Since the
condition is present intermittently, sonography during a painful attack is most likely to delineate the diagnosis.

(Option A and B) Abdominal X-ray films and barium and follow through are indicated if intussusception is the
suspected diagnosis. Intussusception can present with intermittent abdominal pain; however, most cases present
within the first 2 years of life (2 months -2 years). This child is 5 years old. Moreover, flank pain is unlikely to be a
presentation of intussusception.

(Options C and D) Electrolytes, BUN, and creatinine might be indicated for evaluation of kidney function but they are
of little, if any, diagnostic value.

References

• Medscape - Pediatric Ureteropelvic Junction Obstruction

• Medscape - Hydronephrosis and Hydroureter

• UpToDate - Clinical manifestations and diagnosis of urinary tract obstruction and hydronephrosis

Time spent: QID:962 Last updated:


2023-2-12

1322 of 1943
A 4-year-old is boy is brought to your attention with complaints of intermittent abdominal pain. Each episode lasted
for a maximum of 2 to 3 hours before it completely subsided. He has been quite asymptomatic in between the
episodes. This time, however, the pain has lasted for 12 hours. On examination, he is afebrile but a mass is palpated
in the right upper quadrant (RUQ). The mass is not tender. Which one of the following could be the most likely
diagnosis?

A. Hepatoblastoma.
B. Neuroblastoma.
C. Wilms tumor.
D. Pyelonephritis.
E. Pancreatic tumor.

Incorrect. Correct answer is C


45% answered correctly

Explanation:

Correct Answer Is C

The scenario is consistent with Wilms tumor (nephroblastoma) as the most likely diagnosis. Wilms tumor is the
most common intraabdominal tumor of childhood that often is diagnosed at 2 to 3 years of age.

Most cases are sporadic, and only a few percent have a family history. Wilms tumor suppressor gene, WT1, is
located on chromosome 11 and regulates normal kidney development. In approximately 20% of cases with Wilms
tumor, there is a mutation of this gene.

Most children with Wilms tumor presents with abdominal mass or swelling without other signs and symptoms. Other
symptoms, if present, can include abdominal pain (between 25-40% according to different studies), hematuria (12-
25%), fever, and hypertension (25%). Hypertension is the result of the tumor compressing the renal artery, renal
hypoperfusion, and activation of renin-angiotensin-aldosterone system.

The characteristic finding on examination is a smooth firm non-tender palpable abdominal mass that usually does
not cross the midline.

(Option A) Hepatoblastoma is rare hepatic malignancy in children. Patients with hepatoblastoma are usually
asymptomatic. The disease is advanced at diagnosis in approximately 40% of patients, and 20% have pulmonary
metastases. Children with advanced disease may have anorexia. Severe osteopenia is present in most patients and
regresses with resection of the tumor; osteopenia is often asymptomatic. Rarely, patients in whom the tumor has
ruptured present with symptoms consistent with acute abdomen. Occasionally, patients present with severe anemia
resulting from tumor rupture and hemorrhage.

(Option B) Neuroblastoma, which is almost exclusively a disease of children, is the third most common childhood
cancer after leukemia and brain tumors, and is the most common solid extracranial tumor in
children. Neuroblastoma can also present with an abdominal mass, but the condition is expected to be
diagnosed earlier compared to Wilms tumor, usually before the age of 2 years. Two-thirds (~65%) of neuroblastomas
arise within the abdomen, of these 2/3 originate from adrenal glands. The abdominal mass seen in neuroblastoma is
hard, irregular and non-tender and can extend beyond the midline. Other symptoms may include loss of appetite and
weight loss, malaise, protrusion of one or both eyes. Other symptoms may be caused by compression effect the of
tumor or metastases on the adjacent structures.

(Option D) Pyelonephritis can be associated with costovertebral angle tenderness but not a palpable mass. Fever is
often a feature. Most importantly, it does not manifest as long standing intermittent abdominal pain with full
resolution.

(Option E) Pancreatic tumors are extremely rare in children.

1323 of 1943
References

• http://emedicine.medscape.com/article/989398-overv

• http://www.aafp.org/afp/2000/0401/p2144.html

• http://www.uptodate.com/contents/presentation-diag

Last updated:
Time spent: QID:963 2023-2-12

1324 of 1943
A 6-year-old girl is brought to your practice by her parents with a 4-day history of bilateral knee and ankle pain. On
examination, she has a temperature of 37.6 C. Her joints are tender but there is no swelling, inflammation‫ و‬or
synovial thickening. The full range of motion of the joints is preserved. Palpable purpuric rashes are noted on
her legs and buttocks with some ecchymoses around her ankles. Full blood count, blood urea, electrolytes and liver
function tests are normal. ESR is 10mm/hr. which one of the following is a known complication of this girl’s
condition?

A. Pleurisy.
B. Fasciitis.
C. Nephritis.
D. Thrombocytopenia.
E. Diabetes mellitus.

Incorrect. Correct answer is C


45% answered correctly

Explanation:

Correct Answer Is C

The clinical picture is highly suggestive of Henӧch-Schonlein purpura (HSP).

HSP, also known as anaphylactoid purpura, is an acute immune-mediated vasculitis of unknown cause. It is
characterized by arthralgia (not arthritis), and non-thrombocytopenic purpura (the purpura is vascular due to
leukoclastic activity) with typical distribution over legs and buttock.

Other clinical manifestations HSP include colicky abdominal pain, melena, swelling of the joints especially the
ankles, and nephritis. The arthralgia and abdominal pain may persist for 2 to 4 weeks.

Nephritis of HSP, which is histologically the same as the IgA nephropathy, may occur in a small percentage of
patients. For this reason, all patients with HSP should have a urinalysis. If the patient is found to have hematuria,
long-term follow-up is required.

References

• Medscape - Henoch-Schonlein Purpura

• RCH - Henoch-Schonlein purpura

Last updated:
Time spent: QID:112 2023-2-12

1325 of 1943
A 4-year-old child is brought to your practice with complaint of intermittent right upper quadrant (RUQ) and right flank
pain for 2 years. These painful episodes frequently have been associated with vomiting and lasted for 3 hours. He
has been well in between the episodes. This time, the pain has lasted for 12 hours. On examination, a mass is
palpated in the RUQ. The mass is tender to palpation. Which one of the following is the most likely diagnosis?

A. Wilms tumor.
B. Neuroblastoma.
C. Ureteropelvic junction obstruction.
D. Vesicoureteral reflux (VUR).
E. Pyelonephritis.

Incorrect. Correct answer is C


45% answered correctly

Explanation:

Correct Answer Is C

Intermittent abdominal pain in children is a frequent reason of seeking medical attention by concerned parents. The
causes of intermittent abdominal pain are diverse, ranging from benign conditions such as functional abdominal
pain to grave diseases such as childhood malignancies.

As usual, a thorough history and physical examination is the cornerstone of approach to such children. Associated
symptoms can also help narrow down the differential diagnoses.

Of the options, ureteropelvic junction (UPJ) obstruction is the only likely diagnosis justifying the clinical scenario.

UPJ obstruction is by far the most common cause of pediatric hydronephrosis, occurring in 1 per 1000-2000
newborns. Widespread use of antenatal ultrasonography and the advent of modern imaging techniques have
resulted in earlier and more common diagnosis of the condition.

In older children, periodic abdominal pain with vomiting is a common symptom. These symptoms are probably
caused by intermittent kinking of the UPJ. If obstruction does not resolve, severe hydronephrosis and pelvic
distention can follow, presenting with flank pain or even tenderness. Although rarely seen in developed countries,
massive dilation of the renal pelvis and kidney can fill the entire flank and abdomen.

In younger adults the typical presentation is episodic flank pain following diuresis.

(Option A) Wilms tumor is the most common intra-abdominal malignant tumor of childhood that often presents with
a smooth firm abdominal mass that usually do not cross the midline. Abdominal pain, either constant and vague or
intermittent, is another finding. Hematuria and hypertension are other possible manifestations. Unlike the mass in
the scenario, the abdominal mass found in Wilms tumor is non-tender, making Wilms tumor a less likely diagnosis.

(Option B) Neuroblastoma is the most common extracranial malignant tumor of childhood. The initial presentation is
most commonly within the first 2 years of life. The tumor is intraabdominal in two-thirds of cases. Of these cases,
two-thirds originating from adrenal glands. It can also present with a palpable abdominal mass that is non-tender.
However, the age of the child and the tenderness of the mass are against neuroblastoma as the diagnosis.

(Options D and E) Based on physical finding of a palpable mass, pyelonephritis or vesicoureteral reflux are very
unlikely to be the diagnosis as, these do not cause a palpable abdominal mass.

References

• Medscape - Pediatric Ureteropelvic Junction Obstruction

1326 of 1943
• Medscape - Hydronephrosis and Hydroureter

• UpToDate - Clinical manifestations and diagnosis of urinary tract obstruction and hydronephrosis

• AAFP - Recognition of Common Childhood Malignancies


Last updated:
Time spent: QID:964 2023-2-12

1327 of 1943
Mother of a 6-month-old boy brings him for evaluation after she felt a hard mass in the left hypochondrium while she
was bathing him. The mass is non-tender. Which one of the following could be the most likely diagnosis?

A. Wilms tumor.
B. Neuroblastoma.
C. Polycystic kidney disease.
D. Ureteropelvic junction obstruction.
E. Hepatoblastoma.

Incorrect. Correct answer is B


45% answered correctly

Explanation:

Correct Answer Is B

In approach to a child with an abdominal mass two possibilities should always be considered: (1) Wilms tumor
(nephroblastoma) and (2) neuroblastoma.

Wilms tumor is the most common intra-abdominal malignant tumor of childhood that often presents with a smooth
firm abdominal mass that usually do not cross the midline. Abdominal pain either constant and vague or intermittent
is another finding. Hematuria and hypertension are other possible manifestations. The median age of diagnosis is
3.5 years. Although not impossible, it is less likely to present within the first year of life.

Neuroblastoma is the most common extracranial malignant tumor of childhood with the initial presentation most
commonly being within the first 2 years of life. The tumor is intraabdominal in two-thirds of cases, of these two-
thirds originating from adrenal glands. It can also present with a palpable abdominal mass that is non-tender.

Given the age, neuroblastoma is more likely than Wilms tumor to be diagnosis. Wilms tumor and less frequently
neuroblastoma are often found when the child is being bathed or hugged.

(Option A) Given the age of the baby, Wilms tumor is a less likely dagnosis compared to neuroblastoma.

(Option C) An enlarged cystic kidney may be palpated as an abdominal mass in patients with polycystic kidney
disease, but the mass is not hard.

(Option D) Ureteropelvic junction obstruction is not associated with a hard mass. In fact, if a mass is present, it is the
significantly dilated kidney which is not hard, but can be tender.

(Option E) Hepatoblastoma is a rare hepatic malignancy in children. It does not give rise to a hard mass on the left
side.

References

• http://emedicine.medscape.com/article/989398-overv

• http://www.aafp.org/afp/2000/0401/p2144.html

• http://www.uptodate.com/contents/presentation-diag

Last updated:
Time spent: QID:969 2023-2-12

1328 of 1943
A previously healthy 18-month boy is brought to the Emergency Department with wheezing and cough starting 24
hours ago. On examination, he has a respiratory rate of 35 breaths per minute. Bilateral wheeze is evident on chest
auscultation, as well as mildly diminished breath sounds of the left hemithorax. There is family history of asthma, as
well as allergy in history. Which one of the following is the most appropriate next step in management of this child?

A. Nebulized salbutamol 4-hourly.


B. Amoxicillin.
C. Admission and administration of oxygen.
D. Chest physiotherapy.
E. Chest X-ray (CXR).

Incorrect. Correct answer is C


45% answered correctly

Explanation:

Correct Answer Is C

Wheezing in children have a multitude of causes. The age of the child, physical findings, and the pattern and
associations can help narrow down the differential diagnoses.

For instance, wheezing associated with feeding points towards reflux disease as the most likely cause, while wheeze
and cough may suggest asthma. Positional wheeze, for example when the child is supine, is suggestive of
laryngomalacia, whereas, wheeze in the presence of fever may be due to infections. Sudden onset unilateral
wheezing is a strong clue to foreign body aspiration.

The age of onset is another important clue to the diagnosis. Early onset wheeze can reflect a congenital problem.
Bronchiolitis is a very common cause of wheezing in children younger than 12 months, and asthma is the most
common cause of episodic wheezing in children younger than 5 years. Depsite this, a definite diagnosis of asthma is
avoided at this age group and other terms suchs wheeze syndrom, viral wheeze, etc. are used instead.

In this child, wheezing and cough is of sudden onset, suggesting aspiration; however, bilaterality of the wheeze on
exam makes this diagnosis very unlikely. The presence of cough and the strong family history of asthma and allergy
put asthma at the top of the differential diagnoses list. This child though has also diminished breath sounds of the
right lung that mayu suggest collection either in the lung or pleural space. This child needs to be admitted and
through investigation carried out; however, making sure of adequate oxygenation regardless of the diagnosis comes
first.

(Option A) Based on the provisional diagnosis of asthma, a trial of salbutamol is required both diagnostically and
therapeutically because adequate response to bronchodilators is in favor of a diagnosis of asthma. This should be
performed after oxygenation has started. Salbutamol is given by nebulizer in this age group. An adequate response
to therapy almost clinches the diagnosis with high certainty.

(Option B) Antibiotics such as amoxicillin may be necessary later on if investigations suggest bacterial infections
such as pneumonia.

(Option D) Chest physiotherapy is of little, if any, value at this point.

(Option E) CXR is indicated in children who present with unexplained wheezing that is unresponsive to
bronchodilators or with recurrent wheezing. It is not indicated in this child who has presented with first-time wheeze
that is very likely to be due to asthma. It, however, might be indicated if the wheeze is proved to be due to other
causes than asthma, e.g., infections, foreign body, etc.

1329 of 1943
TOPIC REVIEW

The potential causes of wheezing in children include:

Common

Allergies
Asthma and reactive airway disease
Gastroesophageal reflux disease
Infections (pneumonia, bronchiolitis, bronchitis, upper respiratory infection)
Obstructive sleep apnea

NOTE - wheeze occurs when there is cliically significant narrowing of any part of the respiratory tract and not
just; so, URTI specialluy in younger children may also cause wheeze.

Uncommon

Bronchopulmonary dysplasia
Foreign body aspiration

Rare

Congenital vascular abnormalities


Congestive heart failure
Cystic fibrosis
Bronchiolitis obliterans
Immunodeficiency diseases
Mediastinal masses
Tracheobronchial anomalies
Primary ciliary dyskinesia
Tumor or malignancy
Vocal cord dysfunction

References

• RACGP – AFP – The wheezing child: an algorithm

• AAFP - The Diagnosis of Wheezing in Children

Time spent: QID:980 Last updated:


2023-2-12

1330 of 1943
Which one of the following is the most common cause of failure to breastfeed?

A. Exhaustion of the mother.


B. Reduced frequency of breastfeeding.
C. A reduction in the time the baby feeds at each session.
D. Maternal dehydration.
E. Nipple problems.

Incorrect. Correct answer is B


45% answered correctly

Explanation:

Correct Answer Is B

The benefits of breastfeeding for infants and mothers are well proven. Recommendations to breastfeed exclusively
for 6 months (EBF6) have been widely adopted by relevant organizations in Australia.

Advantages of breastfeeding for the child include increased resistance to diseases, lower rates of diseases such as
gastroenteritis, recurrent otitis media, and pneumonia. Additionally, there are lower rates of sudden infant death
syndrome (SIDS), colitis, hypertension, obesity, hyperlipidemia, atopic disease and diabetes, and a higher IQ.

For the mother, there is a delay in ovulation, decreased risks of breast and ovarian cancer and the bonding effects of
breastfeeding.

For many women, difficulties in breastfeeding result in early termination of breastfeeding before the recommended
period of 6 months.

By far, inadequate milk intake or the perception of inadequate milk production is the most common reason for early
termination of breastfeeding. Inadequate milk intake may be due to failure of the infant to extract milk or insufficient
milk production.

Inappropriate early feeding routines are the most common cause of insufficient milk intake. They include infrequent
feeding, poor latch-on, maternal-infant separation, and the use of supplemental formula.

Other less common causes include oral-motor or neurologic abnormalities of the baby and poor emptying of the
breast.

Of the given options, reduced frequency of breastfeeding is the most common cause to decreased milk production.
With infrequent feeds, milk production reduces resulting in both inadequate milk production by the mother and intake
by the baby.

(Option A) Maternal exhaustion can contribute to decreased frequency of breastfeeding and eventually failure of
lactation.

(Option C) Reduced time of latching during each feed does not have such significant negative impact on milk
production as has reduced frequency of feeds.

(Option D) Maternal dehydration can be associated with decreased milk production but is not as common a cause as
is infrequent feeding.

(Option E) Nipple problems can prevent from appropriate and adequate breastfeeding; however, infrequent feeds
remain the most common cause of failure to breastfeed.

References

1331 of 1943
• UpToDate - Common problems of breastfeeding and weaning

• Infant and Young Child Feeding


Last updated:
Time spent: QID:996 2023-2-12

1332 of 1943
A 10-year-old girl is being assessed in your clinic with complaints of labial fusion. She has the past history of
vulvovaginitis which has been successfully treated. The child reports no voiding difficulties. Which one of the
following is the most appropriate next step in management?

A. Reassurance.
B. Surgical separation of adhesions.
C. Encourage good hygiene.
D. Antibiotics.
E. Manual separation of adhesions.

Correct
45% answered correctly

Explanation:

Correct Answer Is A

Labial adhesion (labial agglutination) occurs when the medial edges of the labia minora becomes adherent, often in
the background history of vulvovaginitis. This is probably due to combination of thin vaginal mucosa (the normal
prepubescent state) and minor irritation (such as in vulvovaginitis).

This is a normal variant and will resolve spontaneously in late childhood. Provided that the child is able to void easily,
no treatment other than reassurance is required.

(Option B) Surgical separation of the adhesions, followed by application of Vaseline and/or estrogen cream can be
considered for children with urinary symptoms. This method, however, is not generally recommended because of the
high recurrence rates.

(Option C) Encouraging good hygiene, althoug a good advice, is not effective in treatment of the condition.

(Option D) Antibitoics have no role in management of labial adhesions.

(Option E) Manual separation of adhesions can be distressing for the child and is associated with a high risk of
recurrence and not recommended.

References

• RCH - Prepubescent gynaecology

Last updated:
Time spent: QID:1008 2023-2-12

1333 of 1943
A 9-year-old boy is brought to your attention by his parents because of poor exercise tolerance. He is a member of
the school basketball team but recently, it has been hard for him to play because he becomes short of breath and
tired soon after he starts playing. On examination, his height is at 97 percentile and weight at 3 percentile. Which one
of the following is the most important step in management?

A. Karyotyping.
B. Bone scan.
C. Cardiac ultrasound.
D. Growth hormone test.
E. CT scan.

Incorrect. Correct answer is C


45% answered correctly

Explanation:

Correct Answer Is C

Marfan syndrome (MFS) should always be considered as a probable diagnosis in children with tall stature and low
weight,

MFS is an autosomal dominant connective tissue disorder with involvement of the cardiovascular, skeletal and
ocular systems, as well as the skin, lungs and dura. The prevalence is at least 1 in 5000. In 90-93% of cases, MFS is
caused by mutations in FBN1. In a minority, MFS is caused by mutations in a second gene called TGFBR2. MFS
caused by this mutation is termed MFS2. Based on current statistics, MFS2 accounts for up to 10% of cases with
MFS.

Cardinal manifestations include aortic aneurysm and dissection, ocular lens dislocation and long bone overgrowth.

Cardiac disease is the leading cause of morbidity and mortality in patients with MFS. When the syndrome is
suspected based on clinical grounds, an echocardiogram (cardiac ultrasound scan) should follow as the most
appropriate next step in management for evaluation of aortic root dilation (the most important) and mitral valve
prolapse – conditions frequently seen in patients with MFS. The former is more common and significant clinically.

Serial echocardiographic surveillance is indicated for all affected individuals. Frequency should be tailored to each
individual by their cardiologist. Based on current evidence, the use of beta blockers remains the first-line treatment
(except where contra-indicated e.g., in asthmatic patients) in aortic dilatation in MFS even in young children if a
diagnosis of MFS is clear, or if there is a known FBN1 mutation in a young child without clinical features of MFS but
with affected first degree relative(s) with a known mutation and aortic root dilatation. If beta blockers are ineffective
or contraindicated, verapamil or ACE inhibitors would be the appropriate second-line treatment options.

References

• The Cardiac Society of Australia and New Zealand - Guidelines for the diagnosis and management of Marfan
Syndrome

• UpToDate - Genetics, clinical features, and diagnosis of Marfan syndrome and related disorders

Last updated:
Time spent: QID:1029 2023-2-12

1334 of 1943
A 14-year-old boy has clinically apparent Marfan syndrome. Which of the following cardiac murmurs would you
expect to hear on cardiac auscultation?

A. Midsystolic ejection murmur at the second right intercostal space.


B. Decrescendo high-pitched diastolic murmur at the left sternal edge.
C. Low-pitched rumbling diastolic murmur at the apex.
D. Pansystolic murmur at the left sternal edge with no radiation.
E. Continuous machinery murmur at the second left intercostal space.

Incorrect. Correct answer is B


45% answered correctly

Explanation:

Correct Answer Is B

Aortic root disease is the most common cardiac manifestations in Marfan syndrome (MFS). This results in
aneurysmal dilation, aortic regurgitation and aortic dissection. Such conditions are the most common causes of
mortality and morbidity in patients with MFS.

Dilation of the aorta is found in approximately 50% of children and 60-80% of adults with MFS. Aortic regurgitation
(AR) is often present. On cardiac exam, murmurs caused by chronic AR are most likely to be heard.

The murmur of AR is a decrescendo murmur occurring during diastole usually as a high-pitched sound that is
loudest at the left sternal border. The duration of the murmur correlates with the severity of AR; loudness of the
murmur does not.

Other possible murmurs in patients with MFS and AR include:

Austin-Flint murmur - During diastole, blood jet backs through the incompetent aortic valve and strikes
anterior leaflet of the mitral valve and results in premature disclosure of this valve. This produces a mid-
diastolic rumbling murmur best heard over the apex. Austin-Flint murmur is heard in patients with severe AR.

A functional systolic flow murmur may also be present due to increased stroke volume.

NOTE - Patients with MFS may also have mitral valve prolapse. The classic auscultation finding in mitral valve
prolapse is a mid-to-late systolic click, which is present due to the leaflets prolapsing into the left atrium resulting in
tensing of the mitral valve apparatus. This click may or may not be followed by a high-pitched, mid-to-late systolic
murmur at the cardiac apex.

(Option A) Midsystolic ejection murmur at the second right intercostal space is the characteristic finding in aortic
stenosis and hypertrophic obstructive cardiomyopathy (HOCM).

(Option C) A low-pitched rumbling mid-diastolic murmur over the apex (Austin-Flint murmur) is also possible in
patients with MFS with severe AR. this is, however, a less common finding.

(Option D) Pansystolic murmur at the left sternal edge with no radiation is characteristic for ventricular septal defect

(Option E) A continuous machinery murmur at the second left intercostal space is characteristic of patent ductus
arteriosus (PDA).

References

• The Cardiac Society of Australia and New Zealand - Guidelines for the diagnosis and management of Marfan
Syndrome

1335 of 1943
• UpToDate - Genetics, clinical features, and diagnosis of Marfan syndrome and related disorders

• Medscape - Aortic Regurgitation


Last updated:
Time spent: QID:1030 2023-2-12

1336 of 1943
A 5-week-old female infant is brought to your office by her parents because of ‘feeding problems’. \arents tell you
that their child has been vomiting after many of her feeds for the past week or so. They mention that this is a new
problem, and she was previously quite healthy. For the first time, the baby’s vomit was projectile and forceful last
night. On examination, a mass is palpated in the epigastric area. Which one of the following, can the most likely
metabolic profile in this infant?

A. Increased pH, increased K, increased Cl, increased pCO2, and increased HCO3.
B. Decreased pH, decreased K, decreased Cl, increased pCO2, and decreased HCO3.
C. Increased pH, increased K, decreased Cl, increased pCO2, and decreased HCO3.
D. Increased pH, decreased K, decreased Cl, increased pCO2, and increased HCO3.
E. Increased pH, decreased K, decreased Cl, decreased pCO2, and decreased HCO3.

Incorrect. Correct answer is D


45% answered correctly

Explanation:

Correct Answer Is D

The scenario describes a classic presentation of infantile hypertrophic pyloric stenosis (IHPS). IHPS is more
common in males than females (4:1 to 6:1).

IHPS typically presents with immediate post-prandial non-bilious projectile vomiting in a 3- to 6-week old baby. After
vomit, the baby demands to be re-fed immediately (hungry vomiter).

On physical examination, the baby is dehydrated with a palpable ‘olive-like’ mass at lateral edge of the rectus
abdominus muscle in the right upper quadrant of the abdomen. The mass is most easily felt immediately after
vomiting.

Gastric fluid is rich in hydrochloric acid and potassium. Loss of gastric fluid by emesis results is hypochloremic
metabolic alkalosis. Initially the potassium is normal, but as emesis persists hypokalemia develops. Metabolic
alkalosis is associated with increased HCO3. To compensate the metabolic alkalosis, respiratory acidosis occurs
that is associated with increased arterial pressure of CO2 (pCO2).

Given these, the expected metabolic picture in a patient with prolonged emesis will be increased PH (metabolic
alkalosis), decreased serum potassium and chloride, increased HCO3 and increased PCO2.

References

• RCH - Pyloric stenosis

• Medscape - Pediatric Hypertrophic Pyloric Stenosis

• UpToDate - Infantile hypertrophic pyloric stenosis

Last updated:
Time spent: QID:1042 2023-2-12

1337 of 1943
Kyle, 5 months old, is brought to your practice by his mother for evaluation. He has had a runny nose and a fever or
40°C for the past 48 hours, but otherwise quite active and healthy. Today, Kyle’s temperature dropped to normal but
he developed a maculopapular rash on his trunk. He is still in good health. Which one of the following is the most
likely diagnosis?

A. Measles.
B. Chicken pox.
C. Erythema infectiosum.
D. Rubella.
E. Roseola infantum.

Incorrect. Correct answer is E


45% answered correctly

Explanation:

Correct Answer Is E

The scenario is typical for roseola infantum as the diagnosis. Roseola infantum, also known as exanthem subitum,
sixth disease, and three-day fever, is a clinical syndrome most frequently caused by human herpes virus 6. Other
causes include human herpes virus 7 (HHV-7), enteroviruses (coxackievirus A and B, echovirus), adenovirus, and
parainfluenza virus type 1.

It is a clinical syndrome of infants (mostly 6-18 months) characterized by 3 to 5 days of high fever of commonly up
to 40°C (or even higher). The fever then resolves abruptly, followed by development of a rash.

Typical rash of roseola infantum is a macular or maculopapular rash, starting on the neck and trunk and spreading to
the face and extremities. Uncommonly, the rash is vesicular. It is unusual for the rash to be pruritic. The rash typically
persists for 1 to 2 days, but occasionally may come and go within 2 to 4 hours. In children receiving antibiotics due
to the high fever, the onset of rash can be confused with drug allergy. Diagnosis is almost always clinical. The
disease has a benign course and resolves spontaneously. Febrile convulsions may occur.

Lymphadenopathy and erythematous tympanic membrane are common. During the febrile phase, the disease can be
confused with otitis media. The combination of high fever and bulging fontanelle occurs in as many as 26% of
infants and can be mistaken for meningitis.

In one series of 80 children with roseola infantum, associated findings included:

Lymphadenopathy – 98%
Erythematous tympanic membranes – 93%
Irritability – 92%
Nagayama spots – 87%
Anorexia – 80%
Upper respiratory tract symptoms – 25%
Diarrhea – 15%
Cough – 11%

References

• http://www.uptodate.com/contents/roseola-infantum-

Last updated:
Time spent: QID:1066 2023-2-12

1338 of 1943
Which one of the following is suggestive of hypertrophic pyloric stenosis?

A. Vomiting after 1 hour of feeding.


B. Vomiting immediately after feeding.
C. Vomiting after few minutes of feeding.
D. Bilious vomiting.
E. Vomiting after 2 hours of feeding.

Incorrect. Correct answer is C


45% answered correctly

Explanation:

Correct Answer Is C

Hypertrophic pyloric stenosis (HPS) is caused by progressive thickening of the circular muscle of the pylorus. This
leads to gastric outlet narrowing. The condition usually presents between 2-4 and 6-9 weeks of age.

Predisposing factors to HPS are:

Male gender
First born
Caucasian
Parental history of HPS (higher if mother affected)

HPS typically presets initially with non-bloody, nonbilious vomiting at 2-9 (typically 4-8) weeks of age. Vomiting may
initially be infrequent, but over several days it becomes more predictable, occurring at nearly every feeding.

The vomiting occurs within the first hour after meals, but the typical time is within the first few minutes (up to 30
minutes). Vomiting intensity increases until pathognomonic projectile vomiting develops.

Slight hematemesis of either bright-red flecks or a coffee-ground appearance is sometimes observed. Patients are
usually not ill-looking or febrile. The baby in the early stage of the disease remains hungry and sucks vigorously after
episodes of vomiting (hungry vomiter).

Prolonged delay in diagnosis can lead to dehydration, poor weight gain, malnutrition, metabolic alterations, and
lethargy. Parents often report trying several different baby formulas because they (or their physicians) assume
vomiting is due to intolerance.

References

• RCH - Pyloric stenosis

• Medscape - Pediatric Hypertrophic Pyloric Stenosis

• Kidshealth - Pyloric Stenosis

Last updated:
Time spent: QID:1118 2023-2-12

1339 of 1943
A 13-year-old boy presents to his general practitioner, accompanied by his mother, complaining of poorly localized
pain in his knee for the past 6 weeks. His mother has observed a limp for the same period of time. He walks with his
foot externally rotated. Examination of the knee is normal. Which one of the following is the most likely diagnosis?

A. Osteochondritis dissecans of the knee.


B. Perthes disease of the hip.
C. Undiagnosed congenital dislocation of the hip.
D. Slipped capital femoral epiphysis (SCFE).
E. Osgood-Schlatter osteochondritis of the knee.

Incorrect. Correct answer is D


45% answered correctly

Explanation:

Correct Answer Is D

The clinical features described, as well as the age of the patient suggests slipped capital femoral epiphysis (SCFE)
as the most likely diagnosis.

SCFE is more commonly seen in adolescents of 10 to15 years of age. The classic case would be an oversized
prepubertal boy. The condition is bilateral in 20% of cases.

CSFE presents with the following:

Limp and irritability of hip on movement


Knee pain – referred from the affected hip
On flexion of the hip, it rotates externally. Hip is often in external rotation on walking.
Most movements restricted, especially internal rotation.

The first symptom is hip stiffness that subsides with rest. Later on, limping and hip pain radiating down the
anteromedial thigh to knee follows. Early hip examination neither detects pain nor movement limitation. In more
advanced stages, hip movements become painful and there is decreased flexion, abduction and internal rotation.
The affected leg is externally rotated on walking.

The most significant aspect of the SCFE is the great number of patients who develop avascular necrosis of the
femoral head despite expert treatment. Therefore, diagnosis of the condition before major slipping occurs is
important. This necessitates early investigation and referral.

Any adolescent with a limp or knee pain should have X-rays (AP and frog view) of both hips. Otherwise, this
important condition will be overlooked.

The most important management principles include:

Cease weight-bearing and refer urgently.


If acute slip, gentle reduction via traction is better than manipulation for prevention of later avascular necrosis.
Once reduced, pinning is performed.

1340 of 1943
(Option A) Osteochondritis dissecans is characterized by separation of an osteochondral fragment from the articular
surface. The underlying bone from which the fragment separates has normal vascularity. Knee pain and swelling are
clinical features of the disease. Hip is not affected.

(Option B) Legg-Calvé-Perthes disease, also called Perthes disease, is a temporary condition in the hip joint
characterized by decreased blood supply to the femoral head and consequent avascular necrosis. As a result,
femoral head collapses and the area becomes inflamed and irritated. Legg-Calvé-Perthes disease causes the hip
joint to become painful and stiff. Affected children are usually between 4 and 10 years old, physically active and
small for their age.

(Option C) It is very unlikely for congenital dislocation of the hip to remain asymptomatic until this age. The recent
onset of the problem makes this diagnosis unlikely.

(Option E) Osgood-Schlatter disease is seen in preadolescent children and is characterized by pain localized to the
tibial tubercle and occasionally the patellar tendon. The pathophysiology of the disease is by repetitive traction effect
of patellar tendon on an immature tibial tubercle. There is often tenderness over the tibial tubercle on examination.
Hip movements are not painful or restricted.

References

• RCH - Slipped upper femoral epiphysis (SUFE)

• RACGP – AFP - Slipped upper femoral epiphysis in children

Time spent: QID:113 Last updated:


2023-2-12

1341 of 1943
A 4-year-old boy is brought to your practice by his parents because of what they think to be vertigo. The boy suddenly
starts feeling that the world is spinning around him. Each episodes lasts for about one to two minutes, while the
child is fully conscious. This has happened once a month for the past three months. His past medical history is
unremarkable. He has had no headache, nausea, or vomiting before, during, or after the attacks. Physical
examination including ear exam is normal. Which one of the following is the most appropriate next step in
management?

A. Reassure the parents as this is very likely to be benign paroxysmal positional vertigo (BPPV).
B. EEG and CT scan of the head.
C. Audiology and ENT referral.
D. MRI of the head.
E. Psychiatric assessment.

Incorrect. Correct answer is B


45% answered correctly

Explanation:

Correct Answer Is B

Although dizziness and vertigo can be confused by patients, the spinning sensation described by the child is most
likely true vertigo rather than dizziness. Unlike in adults, vertigo in children often has a sinister cause such as brain
tumors (e.g., medulloblastoma) or temporal lobe epilepsy.

In every child with vertigo, epilepsy and CNS tumors should be excluded as the most appropriate next step in
management, and early referral for specialist review must be considered.

In addition to epilepsy and CNS tumors, the following can also cause vertigo in children:

CNS infections
Trauma, especially to the temporal area
Middle ear infections
Prescription drugs
Alcohol and other illicit drugs
BPPV (rare)
Labyrinthitis

(Option A) BPPV in children is considered a migraine variant of childhood and presents quite similar to this scenario.
However, BPPV is rare in children and is a diagnosis of exclusion, only made once other causes have been excluded.
While CNS tumors are more common and serious diagnoses, reassuring the parents based on a hasty diagnosis of
BPPV is inappropriate.

NOTE – BPPV in childhood often precedes migraine in adulthood.

(Option C) Generally, neurology referral (not ENT referral) is an appropriate and safe option to select because CNS
tumors and epilepsy are the major differential diagnosis in such presentation in a child. Audiology assessment and
ENT referral are appropriate once CNS-related causes have been safely excluded.

(Option D) MRI may be considered later if a CNS tumor is suspected during initial evaluation with CT scan.

(Option E) This child does not seem to have a psychiatric problem, and psychiatric assessment is not an appropriate
initial management option.

References

1342 of 1943
• John Murtagh - Murtagh’s General Practice – McGraw Hill – 6th Edition – page 530

• ILAE - Epilepsy Imitators


Last updated:
Time spent: QID:1122 2023-2-12

1343 of 1943
A 3-year-old girl is brought to your practice by her mother with complaint of multiple painful small oral ulcers. The
child refuses to eat because of the pain. On examination, the child is found to have a temperature of 38°C. Similar
ulcers are noted on his hand and feet. Which one of the following is the causative organism?

A. Herpes simplex virus (HSV).

B. Group B streptococcus (GBS).

C. Coxsackie A virus.

D. Adenovirus.
E. Coronavirus.

Incorrect. Correct answer is C


45% answered correctly

Explanation:

Correct Answer Is C

The clinical picture of ulcers in mouth and on hands and feet is highly suggestive of ‘hand, foot and mouth (HFM)
disease’. This often self-limiting mild viral infection is causes by Coxsackie A virus. The disease has an incubation
period of 3 to 5 days. The illness begins with a prodromal syndrome of mild fever, headache and malaise, followed by
appearance of the rash after 1 to 2 days. The rash starts as an erythematous macular rash, progressing to gray
vesicles with surrounding erythema. These vesicles form shallow ulcers on buccal mucosa, gums and tongue.
Hands and feet, especially the lateral borders, are affected as well. It is not uncommon to see the rash on buttocks
and genitalia as well.

The rash resolves in 3 to 5 days with no scarring. The child is infectious until blisters disappear, but the virus can be
secreted into saliva and feces.

References

• http://www.uptodate.com/contents/hand-foot-and-mou

Last updated:
Time spent: QID:1145 2023-2-12

1344 of 1943
A 10-year-old boy is referred to your clinic from his school for assessment of probable hearing deficit because he
has difficulty in hearing resulting in dropped school performance. You carry out a Rinne test that is positive in both
ears. A Weber test shows no lateralization; however, he has symmetrical decreased hearing threshold. Which one of
the following options could be the most likely diagnosis?

A. Presbycusis.
B. Otosclerosis.
C. Congenital sensorineural deafness.
D. Chronic perforation of the tympanic membrane.
E. Chronic secretory otitis media with effusion.

Incorrect. Correct answer is C


45% answered correctly

Explanation:

Correct Answer Is C

In clinical practice it is common to see patients complaining of hearing loss. It is useful to be able to have a simple
test that can distinguish where the site of the cause of the hearing loss is. This can help guide the need for further
examination, investigation and management. Rinne’s test and Weber’s test are done to differentiate between a
conductive (middle and outer ear causes) and a sensorineural deafness (caused by damage to the cochlea or to the
8th nerve – or its central connections). These tests are always done together. The Rinne test is done first.

The function of the external ear is to collect sounds vibrations form the air and focus these onto the tympanic
membrane. These vibrations are then transmitted through the middle ear cavity by the ossicular chain (Malleus,
Incus and Stapes). The stapes transmits these vibrations to the cochlea through the oval window (fenestra ovalis).
Sound can also be transmitted through the bones of the skull to the cochlea. The hair cells in the cochlea convert the
physical vibrations into action potentials that are transmitted via the nerves in the vestibulo-cochlear (auditory) nerve
to the brainstem for further processing.

The Rinne and Weber tests are used for quick screening of conductive versus sensorineural hearing loss. The Rinne
test is performed for evaluation of hearing loss in one ear (unilateral hearing loss) and compares perception of
sound transmitted by air conduction to those transmitted by bone conduction through the mastoid. Therefore, there
presence of conductive hearing loss is screened for.

For the Rinne test, a vibrating tuning fork (typically 512 Hz) is placed initially on the mastoid process behind each ear
until sound is no longer heard, signaled by the patient. The fork is then immediately positioned just outside the ear
with the patient asked to report when the sound caused by the vibration is no longer heard. A normal or positive
Rinne test is when the sound heard outside the ear (air conduction or AC) is louder than the initial sound heard when
the tuning fork end is placed on mastoid process behind the ear (bone conduction or BC). Therefore, AC > BC; which
is how it is reported clinically for a normal or positive Rinne result. In conductive hearing loss, bone conduction is
better than air or BC > AC, a negative Rinne.

In the Weber test, a vibrating tuning fork (often 256Hz or 512Hz) is placed in the middle of the forehead, or above the
upper lip under the nose over the teeth, or on top of the head in an equal distant from the patient’s ears. The patient
is then asked to report in which ear the sound is louder. A Weber test result is normal (positive) if the patient reports
the sound equally on both sides. Lateralization (hearing the sound better in one ear) is due to either a sensorineural
or conductive problem. In a patient with sensorineural hearing loss, the normal ear hears the sound better than the
defective one. In a patient with conductive hearing loss, the sound is lateralized to the affected side.

NOTE – Patients with symmetrical hearing loss have also a positive (normal) Weber test result because both
ears are affected equally and no lateralization occurs despite the fact that there is hearing loss.

1345 of 1943
The results of these two tests are compare to localize and characterize the nature of any detected hearing loss.
These test are, however, screening tests and are not replacements for formal audiometry.

In this boy, a positive (normal) Rinne test for both ears excludes conductive impairment. The Weber test is also
positive for both ears indicating that there is no lateralization. Based on these findings, chronic perforation of the
tympanic membrane (option D), chronic secretory otitis media (option E), and otosclerosis (option B) are excluded
from the differential diagnoses. This boy should have a hearing problem affecting both ears equally.

NOTE - Otosclerosis is a disease characterized by fusion of the stapes to incus resulting in conductive hearing
loss. This disease is more common among young adult. This boy has normal Rinne test and this condition is
unlikely to be the cause of his hearing loss.

Presbycusis (option A) is the most common cause of sensorineural hearing loss in aging individual. A 10-year-old
boy is unlikely to have presbycusis.

Given the exam findings and the age of this child, congenital sensorineural hearing loss either in isolation or
associated with a syndromic condition could be the most likely diagnosis among others.

For more details about Rinne and Weber tests watch the following video:

Web page not available

The web page at


https://www.youtube.com/embed/FgF91K7dU8Y?
wmode=transparent&jqoemcache=Dn3TA could not be
loaded because:

net::ERR_INTERNET_DISCONNECTED

References

• Medscape - Hearing Impairment

• Medscape - Genetic Sensorineural Hearing Loss

Time spent: QID:1211 Last updated:


2023-2-12

1346 of 1943
A 15-year-old boy is brought by his parents for his behavioral problems. According to the parents, he has had marked
aggressiveness with several fights and arguments with the school staff, anger outbursts, and being uneasy to
control. He seemingly is not obedient at home and defies his parents and the home rules. Parents say that they have
fights over every simple issue at home. They, however, deny any act of vandalism, cruelty towards animals or people,
or drug use. He has declined school performance. On examination, he has a normal attention span and does not
appear hyperactive. If pharmacological intervention is required, which one of the following is most likely to benefit
him?

A. Carbamazepine.
B. Sodium valproate.
C. SSRIs.
D. Risperidone.
E. Olanzapine.

Incorrect. Correct answer is D


45% answered correctly

Explanation:

Correct Answer Is D

The clinical picture and the age of the child are mostly consistent with oppositional defiant disorder (ODD) as the
diagnosis.

ODD is a disruptive behavior disorder in children and teenagers characterized by patterns of unruly and
argumentative behavior and attitudes toward authority figures. This pattern is often considered by parents simply as
stubbornness, emotionalism, and strong will. However, in ODD the behavior is significantly more extreme than what
is considered normal. Interestingly, ODD occurs much more frequently than the type of childhood stubbornness,
whininess, and rebellion that often occurs at different stages of a child’s development.

ODD manifests with a constant extremely negative, defiant, and hostile behavior leading to disruption of the social,
school, and home life for at least 6 months. Symptoms may appear as early as the late preschool years.

Children with ODD often direct their anger and resentment toward their parents, teachers, and other authority figures;
however, they can have such problems with their peers as well. They are often uncooperative, vindictive, and easily
annoyed. They usually defy the rules, have anger outburst, blame others for their mistakes, seek revenge, and disturb
others on purpose. ODD symptoms may be directed at one person or many people, and may occur only at home, at
school, or may occur in a number of settings.

Since the child is unlikely to understand that he has a problem, seeking treatment is often from the parents’ side. A
careful history is essential because many other childhood conditions may have some shared features with ODD.
Also, ODD may co-exist with other psychiatric conditions such as anxiety disorder, ADHD, learning disorders and
language disorders.

It is important to differentiate whether the abnormality in the behavioral pattern is due to ODD or simply a response
to a temporary situation.

The international Classification of Diseases 10th Revision (ICD-10) classifies ODD as a mild form of conduct
disorder. It has been estimated that up to 60% of patients with ODD will develop conduct disorder.

Treatment is necessary at earlier stages to prevent it from developing into a more serious conduct disorder (most
important), mental health disorder, or criminal behavior. Treatment entails a combination of behavioral therapy,
family therapy, and at occasions, medications.

1347 of 1943
In more than 50% of patients with ADHD, ODD is also part of the clinical picture. There is strong evidence suggesting
that ODD and ADHD overlap and many medications that are used to treat ADHA may also be efficacious in the
treatment of ODD too. A few studies have reported the positive effects of psychostimulants or atomoxetine in the
treatment of ODD associated with ADHD. Patients with ODD and conduct disorder with severe aggression may well
respond to risperidone, with or without psychostimulants.

Mood regulators, alpha2 agonists, and antidepressants may also have a second-line role in the treatment of ODD and
its comorbidities.

Of the options, risperidone is the only option that can be used if pharmacotherapy is considered.

Mood stabilizers such as sodium valproate (option B) or olanzapine (option A), and antidepressants such as SSRIs
(option C) are second-line options.

There is no evidence supporting carbamazepine (option E) as a pharmacological treatment for ODD.

References

• Medscape - Oppositional Definat Disorder

• PubMed - Psychopharmacological treatment of oppositional defiant disorder.

Time spent: QID:1274 Last updated:


2023-2-12

1348 of 1943
A 7-year-old boy presents with a generalized rash illustrated in the following photograph. The rash developed after 24
hours of mild malaise and fever. When he can go back to school?

A. After the resolution of the blisters.


B. After 2 days.
C. After 5 days.
D. No exclusion is required.
E. After clearance of the rash.

Correct
45% answered correctly

Explanation:

Correct Answer Is A

The photograph shows blisters of varying stages, macules and papules. Of the blisters, some are intact, some
unroofed and some dried. This, along with the history, is highly diagnostic for chicken pox.

Children with chickenpox should be excluded from school or other daycare setting until all blisters are dried out. This
usually takes 5 days from the onset of the rash but may be less in previously immunized children.

Dried blisters take more time to clear, during which the child is not infectious. Hence, waiting until the complete
clearance of the rash is not necessary.

References

• https://www2.health.vic.gov.au/public-health/infectious-diseases/school-exclusion/school-exclusion-table

Time spent: QID:1275 Last updated:


2023-2-12

1349 of 1943
The parents of a 7-year-old girl have brought her to you for assessment because she frequently soils her underwear.
In the past 2 months, they have received calls from her school about her problems there. She is easily irritated and
becomes angry and aggressive with her classmates and other children at school. At home, they found her soiled
underwear hidden. She becomes angry, cries, and fights with her older brother when he calls her ‘smelly’. This
happens several times a week. Which one of the following is the most likely diagnosis?

A. Conduct disorder.
B. Oppositional defiant disorder.
C. Regression.
D. Delayed developmental milestones.
E. Depression.

Incorrect. Correct answer is B


45% answered correctly

Explanation:

Correct Answer Is B

The scenario suggests encopresis. However, full diagnostic criteria for such diagnosis is uncertain (frequency and
duration of symptoms are lacking in the scenario).

Encopresis is the voluntary or involuntary passage of formed, semi-formed, or liquid stool into a place other than the
toilet for more than one time per month in a child older than 4 years of age for at least 3 months. If the child has
never been continent, the condition is termed primary encopresis, whereas secondary encopresis is fecal
incontinence in a previously continent child. Encopresis is reported in 1-4% of school-aged children. Encopresis is
more common in boys than girls.

There are two different types of encopresis:

With constipation and overflow Incontinence: children with the constipation and overflow incontinence type have
less than 3 bowel movements per week. Due to constipation, only part of the total available stool is emptied during
each of these movements. Parts of the remaining stool leak out, often during the child's daily activities. Once the
constipation is resolved, the encopresis usually no longer exists.

Without constipation and overflow Incontinence: in this type, there is no constipation, and the child’s feces have
normal consistency. Unlike in cases associated with constipation and overflow, soiling of this type is intermittent.
Feces may be emitted in a prominent location (e.g., as an act of defiance) or maybe an unintentional consequence of
anal self-stimulation (e.g., a variety of masturbation). Encopresis without constipation and overflow incontinence is
less common than the first type of encopresis and is often associated with oppositional defiant disorder and
conduct disorder. It has been estimated that 3% of children with psychiatric issues may have encopresis.

With aggressiveness, anger outbursts, and being easily annoyed and irritated, oppositional defiant disorder is the
most likely underlying cause of encopresis and the diagnosis. This girl has no history of violence, a serious breach of
law, or vandalism to suggest conduct disorder (option A).

(Option C) Regression is a defense mechanism characterized by the reversion of an individual’s personality to an


earlier stage of development and adopting more childish mannerisms. In children, regression presents with returning
to behaviors that they have already grown out of. Examples are wanting a bottle or pacifier, temper tantrums,
whining, aggression, thumb-sucking, and baby talk. Enuresis (bet wetting) or encopresis may occur. Regression is
often triggered by a stressful life event such as bereavement, parental separation, or sexual abuse. In the absence of
such history, regression is less likely as the diagnosis. Furthermore, the behavioral pattern of this child is more
consistent with oppositional defiant disorder.

(Option E) Encopresis is unlikely to have been caused by delayed development in a girl who has otherwise fulfilled
other developmental milestones.
1350 of 1943
(Option E) There is no strong link between depression and encopresis. Moreover, there are no other symptoms such
as low mood, sadness, or altered sleep pattern and appetite to suggest depression as a diagnosis.

References

• Mental Help - Elimination Disorders: Encopresis

• Science Direct - Encopresis

Time spent: QID:1286 Last updated:


2023-2-12

1351 of 1943
A 24-month female child is brought for assessment of development. Which one of the following is expected to
develop at this age?

A. Knowing two pronouns.


B. Naming four colors.
C. Knowing her age.
D. Knowing her family name.
E. Speaking in full sentences.

Incorrect. Correct answer is A


45% answered correctly

Explanation:

Correct Answer Is A

Of the options knowing pronouns and prepositions is a language developmental milestone that could have been
achieved at this age. Other options are often achieved after the age of 3 years (36 months)

(Option B) Naming some colors is the ability achieved between 3 and 5 years of age. It is too soon for this child for
this.

(Option C) knowing the age is an ability achieved often after 5 years of age.

(Option D) Knowing the family name is an expected ability around the age of 4 years. Assessment should be
considered if a child cannot say their family name by the age of 5 years.

(Option E) Speaking in full sentences using many words is a language development milestone normally achieved
between the ages 3 and 5 years. It would not be expected in a 24-month-old child.

References

• Developmental milestones and the Early Years Learning Framework and the National Quality Standards

Last updated:
Time spent: QID:1287 2023-2-12

1352 of 1943
Parents of a 14-month-old boy has brought him for evaluation because they think their baby lags behind language
development milestones because he does not speak any words and only babbles. He is the outcome of an
uneventful pregnancy and was born through an uncomplicated vaginal delivery. He has not had any medical
problems to date except few episodes of upper respiratory tract infections. He started rolling over at the age of 4
months, can sit without support from the age of 8 months and has started unaided walking recently. He
transfers objects from one hand to the other and plays and enjoys peak-a-boo. Which one of the following this child
have?

A. Language delay.
B. Social delay.
C. Normal development.
D. Fine motor delay.
E. Gross motor delay.

Incorrect. Correct answer is C


45% answered correctly

Explanation:

Correct Answer Is C

Developmental milestones are categorized under the following developmental areas:

Physical
Social
Emotional
Cognitive
Language

This child has been able to roll over since the age of 4 months (normal: 0-4 months), can sit without support from the
age of 8 months (normal: 8-12 months), can transfer objects from one hand to the other (normal: 8-12 months) and
has started walking without support (normal: 1-2 years). In terms of physical development, he is fully developed and
does not have any fine motor (option D) or gross motor (option E) developmental delay. He plays peak-a-boo
(normal: 4-8 months). This is an example of cognitive developmental milestone often achieved between 4-8 months
of age.

There is no information regarding social developmental milestones in the question but parents do not seem to be
concerned about it. This exclude social delay (option B) as his problem as parents are first to suspect something
wrong in their baby and they are often right for that matter.

In fact the parent’s concern is only language delay.

A child aged 1 to 2 years is expected to:

Say first name


Say many words (mostly naming words)
Begin to use one- to two-word sentences such as ‘want milk’

This child is 14 months and have enough time to fulfill the above milestones. He has been developing normal so far
and has not lagged behind. Language delay would be a concern if the child still babbles beyond 24 months of age.

References

• Developmental milestones and the Early Years Learning Framework and the National Quality Standards
1353 of 1943
Last updated:
Time spent: QID:1288 2023-2-12

1354 of 1943
You order an ultrasound scan for a 9-year-old girl, who has been brought for treatment of urinary tract infection for
the second time this year. On the ultrasound, the right kidney is reported smaller than the left beyond the normal
discrepancy. Which one of the following is the most appropriate investigation to consider for assessment of renal
status?

A. DMSA.
B. DTPA scintigraphy.
C. Urea and electrolytes.
D. Abdominal CT scan.
E. Urine culture.

Correct
45% answered correctly

Explanation:

Correct Answer Is A

The dimercaptosuccinic acid scintigraphy (DMSA) scan is the gold standard for diagnosis of kidney scarring, which
can have been resulted from recurrent urinary tract infections.

DMSA is indicated in the following conditions:

Clinical suspicion of renal injury


Reduced renal function
Suspicion of VUR
Suspicion of obstructive uropathy on ultrasound in older toilet-trained children

For this child with recurrent episodes of UTI and a smaller-than-normal kidney, DMSA should be considered as the
most diagnostic modality for assessment of possible renal scaring.

(Option B) DTPA scintigraphy is a type of radioisotope renography using DTPA and Tc99 as radio-labeled material.
This test also provides information regarding renal function and/or scarring. Compared with DMSA, this test is faster
and associated with less radiation but DMSA remains the gold standard criterion for assessment of renal scarring.

(Option C) Urea and creatinine abnormalities are common among patients with urinary problems. Elevated levels of
urea and creatinine indicated renal impairment but gives no clue regarding the kidney function per se. Urea and
creatinine may raise in patients with pre-renal renal failure or post-renal renal failure in the initial course of which the
kidneys are normal or near normal. On the other hand, normal values do not exclude the presence of abnormalities in
a single kidney. Many patients may have normal ranges in the presence of severe damage to one kidney while the
other is functional and compensating.

(Option D) Abdominal CT scan will visualize the kidneys, adrenal glands and adjacent structure. It is not capable of
assessment of renal function.

(Option E) Urine culture will show the presence or absence of infection in kidneys but does not provide any clue
regarding current kidney function.

References

• RACGP - Paediatric urinary tract infections: Diagnosis and treatment

Last updated:
Time spent: QID:1296 2023-2-12

1355 of 1943
A five-year-old boy is brought for evaluation by his parents concerning unusual behavior. He loves Popeye cartoon
and always remembers when it is shown on TV. At that time he turns on the TV. He becomes distraught and irritated
if he misses the cartoon. He has a goldfish and spends several hours staring at it with curiosity and fascination. His
favorite toy is wooden building blocks that he uses to make towers, break them and build them again. He did not say
his first word until the age of 2.5 years and could not use a communicative phrase at the age of 3 years. He does not
play with his peers and is not interested in making friends. During the examination, he adamantly avoids eye contact.
Which one of the following is the most likely diagnosis?

A. Asperger syndrome.
B. Autism.
C. Mental retardation.
D. Delayed developmental milestones.
E. Obsessive-compulsive disorder.

Correct
45% answered correctly

Explanation:

Correct Answer Is B

The persistent ritualism and preoccupations with activities (watching the Popeye cartoon) and objects (the building
blocks and the goldfish), delayed speech language development, poor communication with friends and avoiding eye
contact make Autistic disorder the most likely diagnosis in this child.

Autistic disorder is pervasive developmental disorder (PDD) affecting at least 4 children in 10000 with a male to
female ration of 4:1.

Characteristic features of autistic disorder include the following:

1- Onset during infancy and early childhood

2- An impairment of social interactions shown by at least two of the following:

lack of awareness of the feelings of others


absent or abnormal comfort seeking in response to distress
lack of imitation
absent or abnormal social play
impaired ability to socialize, including avoiding eye contact

3- Impairment in communication as shown by at least one of the following:

lack of babbling, gesture, mime or spoken language


absent or abnormal non-verbal communication
abnormalities in the form or content of speech
poor ability to initiate or sustain conversation
abnormal speech production

4- Restricted or repetitive activities, interests and imaginative development, shown in at least one of the following:

stereotyped body movements


persistent and unusual preoccupations and rituals with objects or activities
severe distress over changes in routine or environment
an absence of imaginative and symbolic play

1356 of 1943
5- Behavioral problems:

tantrums
hyperactivity
destructiveness
risk-taking activity

In a nutshell diagnosis of autistic disorder requires the presence of the 3 following core features by the age of 3
years:

1. Qualitative impairment of social interaction


2. Qualitative impairment of communication
3. Restricted, repetitive and stereotyped patterns of activities, behavior and interest

A majority of children with autistic have intellectual disability. Approximately 30% of autistic children have normal
intellectual development.

(Option A) Asperger syndrome, also called high-functioning autism, shares many features with autistic disorder.
However, children with Asperger syndrome have normal language development and intellectual ability. With language
problems in this child, Asperger syndrome is an unlikely diagnosis.

(Option C) Although intellectual disability is common in children with Autism, this is a condition not a diagnosis.

(Option D) Children with Autism have language and cognitive problems but again these are specific conditions not a
diagnosis. On the other hand, language development is impaired not delayed. ‘Delayed’ implies that such milestones
may be reached at a later stage which will not happen in autistic children.

(Option E) Routines and ritualistic behavioral patterns are features seen in obsessive-compulsive disorder (OCD);
however, delayed language, stereotypic behavior, and poor communications skills present in this child make OCD an
unlikely diagnosis.

References

• Medscape - Autism Spectrum Disorder

• UpToDate - Autism spectrum disorder: Evaluation and diagnosis

Time spent: QID:1300 Last updated:


2023-2-12

1357 of 1943
A 4-year-old girl is brought to you by her kindergarten teacher. She states that the child has not been well since this
morning, vomited once, and complained of central abdominal pain. On examination, the child looks unwell and has a
temperature of 39°C. Which one of the following would be most expected in this child?

A. Numerous pus cells on urine microscopy.


B. Inflamed ear drums.
C. Neck stiffness.
D. Inflamed tonsils.
E. Localized tenderness over the right iliac fossa.

Correct
45% answered correctly

Explanation:

Correct Answer Is A

It is not uncommon for young children to have urinary tract infection (UTI) without classic urinary symptoms of
urgency, frequency and/or dysuria. In fact, the younger the child, the more likely the symptoms are vague and
nonspecific. In case the cause of such presentation is a UTI, abnormally high counts of white blood cells (puss cells)
are the most common and expected finding on urinalysis.

(Option B) Inflamed ear drums are seen in otitis media. Otitis media is not associated with abdominal pain; therefore,
not a likely diagnosis with this constellation of symptoms.

(Option C) Neck stiffness could be elicited in meningitis. Fever and unwellness are expected features, but abdominal
pain is less likely.

(Option D) Tonsillitis can make a child ill and febrile, but does not cause abdominal pain.

(Option E) Central abdominal pain, vomiting and localized tenderness over the right iliac fossa can be caused by
appendicitis, but it is uncommon for appendicitis to cause a fever as high as 39°C, unless it is complicated due to
perforation. Since the symptoms has commenced this morning, perforation of an appendicitis would not be
expected. It is unusual for an appendicitis to perforate this early.

References

• RCH - Urinary tract infection

• Medscape - Pediatric Urinary Tract Infection

Last updated:
Time spent: QID:115 2023-2-12

1358 of 1943
A 7-year-old boy weighs 30 kg and has a body mass index of 19.5 kg/m2. Which one of the following is correct
regarding his weight and BMI (use the following chart for calculation)?

click to enlarge

A. He is underweight.
B. He is overweight.
C. He has normal weight.
D. He is obese.
E. BMI is not applicable in children.

Incorrect. Correct answer is D


45% answered correctly

Explanation:

Correct Answer Is D

Since the body composition changes with normal growth and stage of puberty, BMI in children is not a fixed measure
like it is in adults. BMI interpretation in children 2-18 years of age must be plotted on the sex-specific BMI percentile
charts. A 17-year-old boy with a BMI of 20 kg/m2 is within the normal range while a 6-year-old boy with the same
BMI is obese.

To check the weight status of a child, his/her BMI should be calculated by dividing the weight by the square of the
height in meter. The BMI then should be plotted on the sex-specific BMI percentile to see in which zone it is. This
child’s BMI lies in the ‘obesity’ zone (see the chart below).

1359 of 1943
Captclick to enlarge

If used with the sex-specific percentile chart, BMI can be used for children with the same efficacy as for adults.

References

• RACGP - Adolescent overweight and obesity

• NHMRC - Clinical Practice Guidelines for the management of overweight and obesity

Time spent: QID:1312 Last updated:


2023-2-12

1360 of 1943
Rachel, 5 years old, is brought to your GP clinic by her mother because of pain and redness in the back of her left ear.
According to the mother, Rachel developed fever and left ear ache one week ago, for which he was seen by another
GP and started on paracetamol and amoxicillin. She was feeling better for the first 4 days but spiked a fever again
and developed a painful red swelling behind her left ear. On examination, she has a warm, red, and tender swelling
behind the left ear shown in the accompanying photograph. Which one of the following could be the most likely
pathogenic factor for this presentation?

A. Staphylococcus aureus.
B. Streptococcus pneumoniae.
C. Hemophilus influenzae.
D. Moraxella catarrhalis.
E. Streptococcus viridans.

Incorrect. Correct answer is B


45% answered correctly

Explanation:

Correct Answer Is B

With the history of unilateral earache and fever one week ago, and development of a red and tender swelling behind
the affected ear, acute mastoiditis complicating acute otitis media (AOM) is the most likely diagnosis.

Acute mastoiditis is rare; however, it is the most common suppurative complication of AOM. Acute mastoiditis may
result in intracranial complications; therefore, it should be taken very seriously. Treatment of acute mastoiditis starts
with intravenous antibiotics (Flucloxacillin plus a third-generation cephalosporin). Immediate involvement of an ENT
consultant is of paramount importance as some cases may even require surgical intervention.

Acute mastoiditis is diagnoses based on postauricular (behind the ear) inflammatory signs such as erythema,
edema, tenderness, and fluctuance. Other clinical findings include edema of auricle and/or external canal and
associated signs of AOM. Although acute mastoiditis follows AOM, it can be the first presentation of AOM in some
cases.

Common pathogens for acute mastoiditis are the same for AOM, with streptococcus pneumoniae being the most
common one and the cause in over 50% of cases. Hemophilus influenza (option C) is the second most common
causative organism responsible for such presentation.

Staphylococcus aureus (option A), Moraxella catarrhalis (option D), and streptococcus viridans are less common
pathogenic organisms in AOM and acute mastoiditis.

1361 of 1943
References

• Medscape - Acute Otitis Media

• RCH - Acute otitis media

Last updated:
Time spent: QID:1337 2023-2-12

1362 of 1943
A 5-year-old child is brought to your practice by his parents with complaints of fever and ear pain. On examination, he
is in mild distress and has a fever of 38.3°C. Otoscopic examination reveals a red bulging tympanic membrane on
the right side with decreased mobility. Which one of the following is more likely to have caused this presentation?

A. Hemophilus influenza.
B. Streptococcus pneumoniae.
C. Pseudomonas aeruginosa.
D. RSV virus.
E. Moraxella catarrhalis.

Incorrect. Correct answer is B


45% answered correctly

Explanation:

Correct Answer Is B

A red bulging tympanic membrane with decreased mobility is the classic finding in acute otitis media (AOM).

In children older than 6 weeks, AOM is caused by bacteria in majority of cases. Streptococcus pneumoniae,
Hemophilus influenzae, Moraxella catarrhalis, and Streptococcus pyogenes are responsible for the majority of
episodes of AOM in persons older than 6 weeks. Other bacteria implicated in AOM include Staphylococcus aureus,
streptococcus viridans, and Pseudomonas aeruginosa.

Of these, Streptococcus pneumoniae is the most common etiologic agent responsible for AOM and for invasive
bacterial infections in children of all age groups, followed by Hemophilus influenzae as the second most common
etiologic factor.

RSV virus is the etiologic factor, mostly in neonates. Infection with this virus is associated with pneumonia,
bronchiolitis and otitis media in this age group; however, streptococcus pneumoniae remains the most common
etiologic factor for AOM even in this age group.

References

• Therapeutic Guidelines - Antibiotics

• Medscape - Actue Otitis Media

Last updated:
Time spent: QID:1343 2023-2-12

1363 of 1943
Parents of an 11-year-old boy have brought him to your clinic for assessment because they believe he is obese. On
examination, his weight and height are on 90 and 50 percentiles for sex- and age-matched growth charts
respectively. Which one of the following is the most appropriate investigation to consider for him?

A. GH.
B. FSH and LH.
C. TSH.
D. Bone age.
E. Reassure he has normal growth and investigation is required.

Incorrect. Correct answer is C


45% answered correctly

Explanation:

Correct Answer Is C

For children and adolescents aged 2–18 years, growth is monitored based on age, height, and weight, using sex-
specific Body Mass Index (BMI) percentile charts. BMI is not a fixed measure in this age group but varies with normal
growth, stage of puberty, and sex. Either the United States Centers for Disease Prevention and Control (US-CDC) or
WHO BMI percentile charts may be used, with the same chart used over time to allow for consistent monitoring of
growth. The US-CDC categorizes overweight as between the 85th and 95th percentile and obesity as above the 95th
percentile. The WHO categorizes overweight as between the 85th and 97th BMI percentiles and obesity as above the
97th percentile. These categories are not diagnostic but contribute to the overall clinical impression of the child or
adolescent being measured.

This boy, based on either chart, is overweight (>85th). The likelihood that childhood overweight and obesity will
persist into adulthood increases with the age of the child and with the presence of parental obesity. One of the
strongest predictors of a child’s weight is the weight status of his/her parents. For overweight or obese children,
initial assessment by history and clinical assessment should determine current health problems and risks for future
disease.

History taking includes developmental history, physical and mental health (including family history of obesity), and
current health behaviors.

Clinical assessment includes pubertal stage, possible causes for overweightness or obesity (e.g., hypothyroidism),
and indicators of comorbidities (e.g., raised blood pressure, joint pain, gastrointestinal symptoms, insulin resistance,
intertrigo, dental health etc.)

It is also very important that secondary causes for obesity are considered and excluded. Of the options, TSH is an
appropriate investigation for exclusion of hypothyroidism as a potential cause of overweightness.

(Options A and B) There is no clinical finding to suggest FSH/LH or GH as an initial assessment. Such clinical
findings may include but not limited to short stature (necessitating GH evaluation) and delayed puberty (for TSH,
FSH and LH).

(Option D) Bone age is often used as the initial assessment for short stature that is not the problem here.

(Option E) This boy is overweight and reassurance cannot be given because there might be an underlying cause
present and also the child is at increased risk of obesity-relates health problems in the future.

References

• RACGP - Adolescent overweight and obesity


1364 of 1943
• NHMRC - Clinical practice guidelines for the management of overweight and obesity in adults, adolescents and
children in Australia
Last updated:
Time spent: QID:1397 2023-2-12

1365 of 1943
An 8-year-old boy is found to have persistent proteinuria of 2+ on dipstick urine. There is no hematuria. Physical
examination including blood pressure is unremarkable. Which one of the following is the next best investigation to
consider for this boy?

A. Renal biopsy.
B. 12-hour urine protein.
C. Urine examination.
D. C3 and C4 level measurement.
E. Ultrasonography.

Incorrect. Correct answer is C


45% answered correctly

Explanation:

Correct Answer Is C

Proteinuria is defined as presence of 1+ protein or more on urine dipstick exam. To confirm the diagnosis of
proteinuria, two consecutive urine dipsticks or urinalysis, 1 to 2 days apart must indicate proteinuria.

If proteinuria persists on dipstick, the next best will be a complete urine exam (urinalysis) on a midstream urine
sample obtained in the office for a more accurate assessment of the urine. With urinalysis (urine examination), the
urine sediment, as well as other signs of glomerular and/or parenchymal disease such as hematuria, red cell casts,
pyuria and/or lipiduria are assessed. Particular findings on urine examination may suggest glomerulonephritis,
nephrotic syndrome, or other renal parenchymal disorders.

Urinalysis may also suggest urinary tract infection with pyuria, bacteriuria, and positive nitrites or leukocyte esterase
along with mild proteinuria.

Other initial laboratory evaluation includes renal function tests (blood urea nitrogen and creatinine), serum
electrolytes, cholesterol, albumin, and total protein.

Another very important investigation to consider for this child is urine protein/creatinine ratio on a first morning void
obtained at home as an estimation of the amount of protein excreted in 24 hours.

(Option A) The role of renal biopsy in children with asymptomatic persistent proteinuria is controversial and is not
recommended routinely.

(Option B) If indicated, urine protein measurement is performed on a 24-hour urine collection (not 12 hours). This
test is cumbersome and not necessary unless at times if the patient is found to have an abnormal urine
protein/creatinine ratio.

(Options D and E) Other tests such as renal ultrasound, serum complement levels (C3 and C4), antinuclear antibody
(ANA), streptozyme testing, hepatitis B and C serology, and human immunodeficiency virus (HIV) testing should be
considered based on the clinical setting. A voiding cystourethrogram should be considered if there is an abnormal
ultrasound with scarring or a history of febrile urinary tract infections.

References

• The Royal Children's Hospital (RCH) - Proteinuria

• UpToDate - Evaluation of proteinuria in children

Last updated:
Time spent: QID:1399 2023-2-12

1366 of 1943
A 3-year-old child is brought to your office by his mother with complaints of diarrhea and colicky abdominal pains for
the past three days. Which one of the following can be the most likely diagnosis?

A. Giardiasis.
B. Norovirus gastroenteritis.
C. Perforated appendicitis.
D. Staphylococcal gastroenteritis.
E. Shigellosis.

Incorrect. Correct answer is B


45% answered correctly

Explanation:

Correct Answer Is B

Of the options, gastroenteritis caused by Norovirus is the most likely diagnosis.

Norovirus is a major human pathogen across the world. It is estimated to account for 95% of non-bacterial
gastroenteritis in outbreaks in industrialized countries such as Australia and 50% of all gastroenteritis outbreaks
worldwide. All age groups are susceptible.

The most common route of transmission is via fecal-oral route. Consumption of contaminated food such as salads,
ice, water, fresh fruit, bakery goods, and cold meats is a major cause of infection. In a minority of cases transmission
occurs via aerosol/personal contact or fomites from toys, carpet, handles, surfaces etc.

The incubation period after digestion is 24-48 hours but it can occur as early as 12 hours.

Clinical manifestation includes:

Common

Nausea
Vomiting
Diarrhea
Abdominal cramps

Less common

Headache
Low-grade fever
Chills
Muscle aches
Malaise

Diarrhea of Norovirus gastroenteritis is non-inflammatory and is not bloody or mucoid. Except in outbreaks, no
diagnostic test is necessary. PCR of the stool is the method of choice. Treatment is only symptomatic.

(Option A) Giardiasis should be thought of in hikers, campers, or those with history of drinking fresh water.
Symptoms of acute giardiasis include:

Diarrhea (sudden in onset; initially may be watery) - 90%


Malaise - 85%
Foul-smelling and fatty stools (steatorrhea) - 70%
Abdominal cramps and bloating - 70%
Flatulence - 75%
1367 of 1943
Nausea - 70%
Weight loss – 65%
Vomiting – 30%
Fever -10-15%

Symptoms usually develop after an incubation period of 7 to 14 days (range 1 to 45 days) and may last 2-4 weeks.
Significant weight loss can occur in about 50% of symptomatic patients.

With diarrhea and abdominal cramps, giardiasis can be a possibility; however, no history of using fresh water makes
this diagnosis less likely.

(Option C) Perforated appendicitis presents with a completely different clinical picture including systemic toxicity
and sign and symptoms of peritonitis.

(Option D) Staphylococcal gastroenteritis is a self-limiting condition caused by contaminated foods. Nausea and
vomiting are more pronounced, and diarrhea is less common.

(Option E) Shigellosis is an infection of the lower gastrointestinal tract. Patients with Shigella gastroenteritis typically
present with high fever, abdominal cramps, and bloody and mucoid diarrhea. The incubation period ranges from 1 to
7 days. The disease typically begins with constitutional symptoms such as fever, anorexia, and malaise. Initially,
diarrhea is watery, but subsequently may contain blood and mucus. Tenesmus is a common complaint.

References

• SA Health - Norovirus infection - including symptoms, treatment and prevention

Time spent: QID:1401 Last updated:


2023-2-12

1368 of 1943
A mother has brought her four-year-old son with complaint of nighttime bed wetting. According to her, he wets his
bed 3-4 time a week at night despite achievement of daytime continence at the age of two years. He is otherwise
asymptomatic. Physical examination is unremarkable. Which one of the following is the most appropriate
management regarding this complaint?

A. Bed wetting alarms.


B. Nasal desmopressin.
C. Amitriptyline.
D. Waiting for spontaneous resolution.
E. Ultrasonography of the bladder, ureters and kidneys.

Incorrect. Correct answer is D


45% answered correctly

Explanation:

Correct Answer Is D

According to the International Children’s Continence Society (ICCS), enuresis, also termed intermittent nocturnal
incontinence, refers to discrete episodes of urinary incontinence during sleep in children of five years of age or older.

Monosymptomatic nocturnal enuresis refers to children with normal daytime voiding patterns and night time wetting
only. Non-monosymptomatic enuresis refers to enuresis in children with daytime wetting and / or additional lower
urinary tract symptoms (such as abnormal urine stream, hesitancy, urgency, dribbling or pain).

This child is only 4 years old, and does not fulfill the criteria for enuresis. He is very likely to grow out of the problem
until 5 years age; therefore, no further action is required except waiting for spontaneous resolution. Approximately,
84% of children achieve full night-time bladder control by the age of 5 years.

(Option A) Bed wetting (enuresis) alarms are the most effective means of controlling nocturnal enuresis and
preventing relapse. An arousal device, which is often an auditory alarm and/or a vibrating belt or pager, activates
when a sensor placed in the underwear or on a bed pad, detects moisture. The alarms work through conditioning: the
child learns to wake or inhibit bladder contraction in response to the physiologic conditions present before wetting.
Enuresis alarms are a first-line treatment for children whose bedwetting has not responded to advice about fluid
intake, toileting, or an appropriate reward system. Enuresis alarms work best for well-motivated families and children
with frequent enuresis (more than twice per week). Other treatment options should be used if:

Rapid or short-term improvement is the goal


The child or parents do not want to try the enuresis alarm
The child wets the bed only once or twice per week
The parents are having emotional difficulty coping with the burden of bedwetting
The parents are expressing anger, negativity, or blame toward the child

Bedwetting of this 4-year-old child is normal and does not require investigation or treatment for now.

(Option B) Desmopressin, a synthetic vasopressin analog, is the first-line treatment for enuresis in children older
than 5 years, whose bedwetting has not responded to advice about fluid intake, toileting, or an appropriate reward
system. It is an alternative to enuresis alarms for children and families who seek rapid or short-term improvement of
enuresis, or have failed, refused, or are unlikely to adhere to enuresis alarm treatment; and for whom an enuresis
alarm is unsuitable.

(Option C) Tricyclic antidepressants (TCAs) decrease the amount of time spent in REM sleep, stimulate vasopressin
secretion, and relax the detrusor muscle. Given the efficacy and safety of enuresis alarms and desmopressin,
tricyclic antidepressants (e.g., amitriptyline, despiramine, imipramine) are used as third-line treatment for
monosymptomatic enuresis after failed alarm therapy and/or desmopressin).

1369 of 1943
(Option E) Ultrasonography of urinary system and urine examination are initial investigation to consider in children
with enuresis. This child is only 4 years old and does not fulfill criteria for enuresis; therefore, no evaluation is
indicated for him at this stage.

References

• The Royal Children's Hospital (RCH) - Enuresis - Bed wetting and Monosymptomatic Enuresis

• RACGP - Behavioural interventions including alarms: bedwetting (enuresis)

Time spent: QID:1411 Last updated:


2023-2-12

1370 of 1943
A 6-yer-old girl is brought to your general practice by her mother, who is concerned about a neck mass in her
daughter. The mass is shown in the following photograph. The child is otherwise healthy. On examination, there is a
non-tender neck mass in the midline moving with swallowing and tongue protrusion. The rest of the physical
examination is unremarkable. Which one of the following could be the most likely complication of this neck mass?

A. Infection.
B. Malignancy.
C. Fistula formation.
D. Overgrowth and compression of underlying structures.
E. Rupture.

Correct
45% answered correctly

Explanation:

Correct Answer Is A

The clinical presentation and the photograph are highly suggestive of thyroglossal duct cyst (TDC) as the most likely
diagnosis. TDC is the most common type of developmental cyst encountered in the neck region. TDC occurs when
the thyroglossal duct fails to obliterate and forms a bridge between the base of the tongue and the thyroid gland.

A TDC usually presents with an asymptomatic palpable midline neck mass usually at or below the level of the hyoid
bone, above the thyroid cartilage. The mass is most often in the midline, although it can present slightly off the
midline to one side or the other. A TDC may present in childhood (less than 50%) or later in life, usually as a young
adult below the age of 20 years.

Characteristically on examination, a TDC moves up when the tongue is protruded, and moves up and down upon
swallowing reflecting the attachment of these cysts to the base of the tongue by the thyroglossal tract. Some
patients have neck or throat pain, or dysphagia, but most patients are just concerned by the lump itself.

A TDC may become complicated. Complications of TDCs include:

Infection.
Malignancy.
Overgrowth and pressure of the underlying structures.
Rupture and fistula formation.

1371 of 1943
Of these, infection is the most common complication. An infected TDC usually presents with redness and swelling
(image below).

An infected thyroglossal duct cyst

Malignancy (option B) is the second most common complication of TDCs. It can complicate the cyst in
approximately 1% of patients. Papillary carcinoma is the most common malignancy arising from a TDC. Follicular-
papillary and Squamous cell carcinoma follow in order of commonality.

(Option D) Cyst enlargement and compression of the underlying structures is another complication of TDCs.
Obstruction of the airway is of significance importance. However, compared to infection and malignancy, this
complication is less common.

Rupture (option E) and fistula formation (option C) are other rare complications of a Thyroglossal duct cyst.

References

• Medscape – Thyroglossal Duct Cyst Imaging

• PubMed - Thyroglossal Duct Cyst—More Than Just an Embryological Remnant

Last updated:
Time spent: QID:1423 2023-2-12

1372 of 1943
Concerned mother of a 6-year-old girl has brought her to your attention because she has frequently noticed blood-
stained yellow discharge on her panties. She is otherwise healthy. There is no complaint of itching or urinary
symptoms. The girl lives with her mother and birth father. On examination, the vitals are within normal ranges. On
inspection, there is mild erythema of the vulva and perianal area and offensive blood-stained yellow discharge in the
introitus. Which one of the following could be the most likely diagnosis?

A. Monilial vulvovaginitis.
B. Vaginal foreign body.
C. Atrophic vulvovaginitis.
D. Chlamydia infection.
E. Threadworms.

Correct
45% answered correctly

Explanation:

Correct Answer Is B

Blood-stained vaginal discharge in female children should always be taken seriously and investigated promptly. One
exception is vaginal bleeding in the first week of birth in female neonates that is caused by withdrawal from maternal
estrogen upon birth.

Causes of vaginal bleeding in children include the following:

Vaginal foreign body


Severe vulvovaginitis
Trauma (including straddle injury and sexual abuse)
Excoriation associated with threadworms
Onset of first menstruation
Hematuria
Urethral prolapse (an inflamed "doughnut" of tissue is visible at the urethral meatus

Of the above, a vaginal foreign body is the most common cause of bloody vaginal discharge. The foreign body is
often toilet t papers. Foreign body in the vagina causes offensive purulent vaginal discharge that recurs frequently
despite successful initial management with antibiotic unless the foreign body is removed. It can also cause vaginal
bleeding. Children with suspected vaginal foreign body should be referred to pediatrics specialist for removal of the
foreign body.

(Option A) Monilial (candida) vulvovaginitis is almost never seen in prepubertal girls except as an association with
nappy rash. It cannot be the cause of such presentation in a 6-year-old girl.

(Option C) Atrophic vulvovaginitis is a less common condition in prepubertal girls and if present should raise
suspicion against lichen sclerosus. Even in case of atrophic vaginitis, itching and mucoid discharge would be the
expected presentation. With a blood-stained yellow discharge, atrophic vaginitis is very unlikely.

(Option D) Infection with chlamydia, gonorrhea and trichomonas can cause offensive bloody vaginal discharge.
These infections in children are highly suggestive of sexual abuse. For every child with vaginal discharge associated
with unusual features such as persistent and significant discharge or blood in the discharge, sexual abuse should be
considered as a possibility and approached appropriately. Although sexual abuse should also be considered and
thought of in such situation, the most likely cause of such presentation remains a vaginal foreign body, unless
investigations establish the presence of chlamydia, gonorrhea or trichomonas, in which case sexual abuse is almost
always the cause.

(Option E) Infection with threadworm may vulvovaginitis, in which case, itching is the most prominent symptom.
Excoriation can cause bleeding; however, absence of itching makes this diagnosis almost unlikely.
1373 of 1943
References

• RCH – Prepubescent gynaecology

• Government of Western Australia - Child and Adolescent Health Service – Vulvovaginitis

• Australian Prescriber – Vulval disease in childhood

Last updated:
Time spent: QID:1427 2023-2-12

1374 of 1943
Parents of a 12-year-old girl has brought her to a clinic after they noticed a rash on hers back. She had sore throat 3
weeks ago which subsided gradually in a course of days. Last week she had a very painful left wrist pain that
improved after 2 days before she develops pain in her right knee. Today, she has a painful right ankle. On physical
examination, she looks ill and has a temperature of 38.1°C. The rest of her vital signs are within normal limits.
Examination of the musculoskeletal system is notable for a painful, slightly hot and swollen right ankle with
decreased range of motion due to pain. There is a rash on her back which is shown in the following photograph.
Which one of the following is the most appropriate next step in management?

A. Echocardiography.
B. Blood culture.
C. ESR.
D. Throat swab.
E. Full blood examination (FBE).

Incorrect. Correct answer is D


45% answered correctly

Explanation:

Correct Answer Is D

The photograph shows well demarcated bright red and pink macules characteristic of erythema marginatum. The
presence of the rash, migratory arthralgia/arthritis (involvement of one joint, improvement and then involvement of
another joint), and more importantly, a history of sore throat 3 weeks before such presentation is highly suggestive
of acute rheumatic fever as the most likely diagnosis.

Acute rheumatic fever (ARF) occurs predominantly in childhood and is secondary to infection with Group A
streptococcus (GAS). It is an acute illness presenting with a cluster of signs and symptoms including carditis and
polyarthralgia. Without preventive treatment, ARF may progress to chronic rheumatic heart disease (RHD)
associated with damaged heart valves.

An accurate diagnosis of ARF is important because overdiagnosis results in unnecessary treatment over a long time,
while underdiagnosis leads to further attacks of ARF, cardiac damage and premature death. Diagnosis is mostly
clinical because there is no specific laboratory test.

The diagnosis of ARF is usually guided by the Jones criteria and the more recent World Health Organization (WHO)
criteria. To increase sensitivity for ARF diagnosis in Australia’s unique high-risk population, the Jones and WHO
criteria have been further modified to form the 2012 Australian criteria for the diagnosis. According to Australian
guidelines, aseptic monoarthritis or polyarthralgia is a major criteria for populations at risk, including Aboriginal and
Torres Strait Islanders communities living in rural or remote areas or in disadvantaged suburban areas, it is a minor
criteria (in agreement with international guidelines) for other not-at-risk populations.

1375 of 1943
The following table summarizes the Australian ARF diagnostic criteria for high risk population and other populations:

High risk population Other populations


Major criteria Major criteria
Carditis (including subclinical evidence of Carditis (including subclinical evidence of
rheumatic valve disease on echocardiogram) rheumatic valve disease on echocardiogram)
Polyarthritis or aseptic mono-arthritis or Polyarthritis
polyarthralgia Sydenham chorea
Sydenham chorea Erythema marginatum
Erythema marginatum Subcutaneous nodules
Subcutaneous nodules

Minor criteria Minor criteria


Fever Polyarthralgia or aseptic mono-arthritis
ESR ≥30 mm/hr or CRP ≥30 mg/L ESR ≥30 mm/hr or CRP ≥30 mg/L
Prolonged P-R interval on ECG Prolonged P-R interval on ECG

To make a diagnosis of an initial episode of ARF, there should be:

2 major criteria OR one major and two minor criteria

PLUS

Evidence of preceding GAS infection

To make a diagnosis of recurrent attack of ARF (previous ARF or established rheumatic heart disease [RHD]) in past
medical history) there should be

Two major criteria OR one major and two minor criteria OR three minor criteria

PLUS

Evidence of preceding GAS infection

When signs and symptoms suggest ARF (based on the criteria), the most important step is to look for evidence of a
preceding group A streptococcus infection. This child fulfills clinical criteria for ARF; however, it should be
determined that the cause of her sore throat has been GAS before a definite diagnosis of GAS is made. Of the
options, a throat swab for microscopy and culture is the most important next step in management. Cultures for GAS
are the gold standard; however, any of the following can be uses instead:

A positive rapid GAS carbohydrate antigen test in a child whose clinical presentation suggests a high pretest
probability of streptococcal pharyngitis
Elevated or rising streptococcal antibody titer (a rise in titer is better evidence than a single titer result)
Antistreptococcal antibodies include the following:

Antistreptolysin O (ASO)
Antideoxyribonuclease B (anti-DNAse B)
Antistreptokinase
Antihyaluronidase
Anti-DNAse (anti-DNPase)

(Option A) Echocardiography with Doppler should be performed in all cases of confirmed or suspected ARF.
Echocardiography may show abnormalities if there is heart involvement. However, abnormal echocardiographic
findings will not direct the management plan unless there is evidence of a preceding GAS and a definite diagnosis of
ARF.

(Option B) Blood culture was indicated if the source of GAS infection is unknown. With the sore throat in history,
which is possibly the source of the infection, cultures from throat swab (or other methods mentioned above) can
provide sufficient clue regarding preceding GAS infection, and blood cultures are not necessary.
1376 of 1943
(Option C) an ESR≥30mm or CRP≥30mg/L is a minor criteria for ARF. This child already has adequate clinical criteria
for ARF, and a positive ESR or CRP adds no diagnostic benefit at this stage. ESR or CRP could have been an option if
further clinical or laboratory criteria were required to fulfill the diagnostic criteria.

(Option E) Although FBE is often routinely evaluated as a part of basic laboratory tests, it will not add any diagnostic
benefit in this case. FBE in patients with ARF is often normal or inconclusive.

References

• RACGP - The Australian guideline for prevention, diagnosis and management of acute rheumatic fever and
rheumatic heart disease

• RACGP – Rheumatic Fever

Time spent: QID:1443 Last updated:


2023-2-12

1377 of 1943
For which one of the following children with urinary tract infection (UTI) hospital admission is indicated?

A. An infant younger than 12 months of age.


B. A 6-year old male child with family history of urinary tract infection.
C. A 5-year old female child with family history of urinary tract infection.
D. An infant younger than 2 months of age.
E. A 2-year old child with allergy to penicillin.

Incorrect. Correct answer is D


45% answered correctly

Explanation:

Correct Answer Is D

According to the Royal Children Hospital (RCH) guidelines, the following children with UTI should be admitted to the
hospital for intravenous antibiotics and close monitoring:

Most of children less than 3 months.


Children who are seriously unwell such as those with toxic appearance or dehydration.
Oral antibiotics cannot be tolerated.

NOTE - The cut-off age for hospitalization can be different according to other guidelines. For example, Medscape
recommends admission of all children who have febrile UTI and are younger than 2 months of age.

References

• RCH - Urinary tract infection

• Medscape - Pediatric Urinary Tract Infection

Last updated:
Time spent: QID:116 2023-2-12

1378 of 1943
Which one of the following statements is correct about Perthes disease?

A. Hip mobility is reduced especially abduction and external rotation.


B. Ultrasound is the modality of choice to make a diagnosis.
C. Progress is evaluated with serial radiological examination.
D. Osteotomy is the treatment of choice.
E. The inflammatory markers such as WBC, ESR and CRP are usually raised.

Incorrect. Correct answer is C


45% answered correctly

Explanation:

Correct Answer Is C

Legg-Calvé-Perthes disease, also called Perthes disease is avascular necrosis (AVN) of the proximal femoral head
resulting from compromise of blood supply to this area. Perthes disease usually occurs in children aged 2-12 years
with a peak incidence in children between 4 and 8 years. It is more common in boys than girls. The disease has an
insidious onset and may follow an injury to the hip. Perthes disease is unilateral in most cases (80% -90%), and in
only 10%-20% of cases both hips are involved. Even in bilateral cases, the hip joints are involved successively, not
simultaneously.

Affected children often have delayed bone age, disproportionate growth, and a mildly shortened stature (smaller
than age). Perthes disease can be idiopathic, or it may result from a slipped capital femoral epiphysis, trauma,
steroid use, sickle-cell crisis, toxic synovitis, or congenital dislocation of the hip.

The earliest sign of Perthes disease is an intermittent limp, especially after exertion, with mild or intermittent pain in
the anterior part of the thigh.

NOTE- Perthes disease is the most common cause of a limp in the 4- to 10-year-old age group.

The classic presentation of Perthes disease has been described as a 'painless limp'. The child may present with
limited range of motion of the affected extremity. The most common symptom is persistent pain.

Hip pain may develop and is a result of necrosis of the involved bone. This pain may be referred to the medial aspect
of the ipsilateral knee or to the lateral thigh. The quadriceps muscles and adjacent thigh soft tissues may atrophy,
and the hip may develop adduction flexion contracture. The patient may have an antalgic gait with limited hip motion,
or a Trendelenburg gate (abductor lurch). Pain may be present with passive range of motion and limited hip
movement, especially internal rotation and abduction.

Laboratory studies including a full blood count, ESR and CRP are usually normal and are often ordered to exclude
other possible causes of hip pain and limp such as septic arthritis, transient synovitis, or juvenile rheumatoid
arthritis.

In most cases X-rays are normal early in the course of the disease but are used serially to assess the progression of
the disease. The following are radiographic stages of Perthes disease in time:

Stage 1 - Cessation of femoral epiphyseal growth


Stage 2 - Subchondral fracture
Stage 3 - Resorption
Stage 4 - Reossification
Stage 5 - Healed or residual stage

1379 of 1943
These stages can take 12-40 months. A bone scan can be used to evaluate the site for avascular necrosis (AVN).

Early radiographic findings in acute phase can be a nonspecific effusion of the joint evident by slight widening of the
joint space. Other early findings may include decreased bone density around the joint and a bulging joint capsule.
The acute phase often lasts 1-2 weeks. Decreasing bone density in and around the joint is noted after a few weeks.

Once Perthes disease is suspected, the child should be urgently referred to orthopedic specialist for management
and treatment. In the meanwhile, limited movement and reducing the weight from the hip joint should be encouraged
to prevent or minimize further damage of the femoral head. For this purpose, crutches are prescribed and
encouraged to use.

Treatment goals in Pethes disease are:

Eliminating hip irritability


Restoring and maintaining good range of motion in the hip
Preventing femoral epiphyseal collapse
Having a spherical femoral head when the hip heals

Initial therapy includes protecting the hip joint and making it non-weightbearing. This can be achieved by maintaining
the femur abducted and internally rotated to keep it held well inside the rounded portion of the acetabulum. Most
cases are treated conservatively using splinting (e.g. Scottish Rite brace). At occasions surgery (osteotomy) may be
considered. Surgery does not speed femoral head healing, rather it results in a more spherical oreossification of the
femoral head. Limb shortening is a potential complication of surgical treatment.

Scottish Rite Brace

(Option A) Although Perthes disease is associated with hip pain and decreased range of motion, the two most
severely affected movements are abduction and internal rotation, not external rotation.

(Option B) Perthes disease is mostly diagnosed clinically. The imaging modality of choice to help with diagnosis and
also assessment of the disease progression is X-ray, not ultrasound.

(Option D) Osteotomy can be a treatment option depending on the child’s age and the disease severity; however,
more conservative strategies such as bracing are applied as the treatment option of choice where feasible.

(Option E) Perthes disease is not an inflammatory condition; therefore, inflammatory markers such as WBC, ESR and
CRP are usually negative. Positivity of such markers points toward an alternative diagnosis such as septic arthritis,
rheumatoid arthritis, synovitis, etc.

References

1380 of 1943
• UptoDate – Approach to hip pain in childhood

• Medscape - Legg-Calve-Perthes Disease

• RACGP – Check (Unit 493 April 2013)

• Murtagh’s General Practice – 7th Edition – Pages 745, 746


Last updated:
Time spent: QID:1496 2023-2-12

1381 of 1943
The mother of a four-year-old female child has brought her to you after she noticed a mass in her neck while giving
her a bath. The mas is 2x1.5 cm in size, located in the anterior triangle of the neck, and not tender. The child is
otherwise healthy and has no other concerning feature on physical examination. Which one of the following would
be the most appropriate next step in management?

A. Excision of the mass.


B. Fine needle aspiration cytology (FNAC).
C. Reassurance.
D. Ultrasonography of the neck.
E. Chest X-ray.

Incorrect. Correct answer is D


45% answered correctly

Explanation:

Correct Answer Is D

Childhood cervical masses are a common presentation in general practice. Often, such masses cause significant
parental anxiety and also a potential diagnostic dilemma. While these masses are commonly benign, there are other
possibilities that should be taken into account. A good knowledge of relevant anatomy and a thorough and
systematic history, examination, and investigations are crucial to ensure an appropriate and efficient approach.

Cervical masses in children are divided in the following broad categories:

1. Congenital
2. Inflammatory/ infective
3. Neoplastic

While the majority of childhood neck lumps seen in general practice occur as a result of an inflammatory/infective
process, other possibilities such as persistent congenital abnormalities and red flag of neoplastic diseases always
must be considered.

In history, three main components should always be covered:

1. Timeline (since when and for how long has the mass been present?
2. Time course (how fast is the mass growing/has grown?)
3. Associated features/ red flags.

Cervical masses in the neonatal period and early infancy are often congenital. Examples are thyroglossal duct cyst,
teratomas, sternocleidomastoid tumors of infancy and vascular or lymphatic malformations. Of note, vascular and
lymphatic malformations typically grow with the growth of the child. Reactive lymphadenopathy is most common in
infancy and early childhood, with 40–55% of young children found to have palpable cervical lymph nodes.

Congenital masses may present later in childhood/adolescence because of continuous growth or superimposed
infection, while the likelihood of malignancy also rises in this age group.

Rapidly developed masses are typically inflammatory, including reactive lymphadenopathy, lymphadenitis, or
secondary infection of underlying congenital or neoplastic masses. Inflammatory pathology typically resolves within
four weeks. Cervical masses that persist past six weeks require further evaluation.

Rapidly growing masses should be immediately referred if there is a concern that they have the potential to affect
the airway or have features suggestive of abscess formation. Masses that grow at a slower rate, for months to years,
are suggestive of benign neoplasms or a slowly enlarging congenital malformation.

1382 of 1943
In terms of associated symptoms, a viral prodrome, fevers, and cervical tenderness are associated features
suggestive of reactive lymphadenopathy. It is, however, important to consider suppurative lymphadenitis or infection
of an underlying congenital or neoplastic mass, as this may present similarly.

On physical examination, location, size, consistency, and tenderness give important clues. Location-wise, midline
masses are more likely congenital and are typically thyroglossal duct cysts or dermoid cysts. Thyroglossal duct
cysts will elevate with tongue protrusion or swallowing, while dermoid cysts are tethered to the overlying skin.
Thyroid masses are potentially malignant and need further evaluation. Lymphadenopathy commonly arises as a
lateral lump in the anterior or posterior triangle, and may present an inflammatory or neoplastic process.

Lymphadenopathy in the posterior triangle has a higher risk of malignancy, while supraclavicular lymphadenopathy is
considered a red flag. Other masses of the lateral neck include lymphatic and vascular malformations and branchial
cleft cysts.

Reactive lymphadenitis is characterized by a local collection of small, tender, mobile lumps. The possibility of
suppurative lymphadenitis should be considered if there is palpable warmth, fluctuation, induration, or severe
tenderness. If the mass is firm, irregular, or immobile, malignancy should be thought with high suspicion.

In terms of size, a palpable cervical lymph node less than 1 cm in size can be considered normal in children, while
increasing size is associated with a significantly increased risk of malignancy. Lymph nodes greater than 1 cm in
size that persist for longer than six weeks or despite antibiotic therapy should be evaluated with medical imaging
and a possible tissue biopsy e.g., FNAC

The following features are red flags for a neck lump in children, the presence of which warrants a more
aggressive approach:

Weight loss
Sustained fever
Night sweats
Generalized lymphadenopathy
Signs and symptoms of pancytopenia
Mass persisting> 6 weeks
Lymph node> 3 cm
Thyroid mass
Supraclavicular mass
Hard, irregular mass
Fixed mass

This child has presented with a solitary neck mass that is incidentally found, and for that no exact duration can be
determined. The size of the mass is over 1 cm, and there is no history of a preceding infection or any physical
findings for that matter. Although there is no red flag in history or physical examination, an ultrasound scan of the
lump would be the most appropriate way to go among other options.

Ultrasonography is a readily available, inexpensive and radiation-free imaging study that can provide important
information about the location, size and consistency of a cervical mass. For this reason, ultrasound is the preferred
initial imaging study for most children with a neck mass. Ultrasound can identify features typical of numerous
congenital masses as well as benign and malignant lymphadenopathy.

FANC (option B) or excision of the mass (option A) is considered if, based on the presence of red flags in history/
physical examination or alarming features on ultrasound, malignancy is suspected. In most cases, ultrasound-guided
FANC is preferred over excisional biopsy.

(Option C) Watchful waiting when the diagnosis is in favor of benign diagnoses such as reactive lymphadenopathy is
the approach of choice. This, however, does not mean that reassurance can be given, especially in this case where
supporting findings for reactive lymphadenopathy such as a preceding viral illness, tenderness, etc. are lacking. If an
option, watchful waiting and review could have been an acceptable option given the fact that there are no alarming
features.

(Option E) Chest X-ray does not seem to provide additional diagnostic value. However, in the presence of red flags, it
could be used for diagnoses such as TB or for further evaluation and/ or staging in malignancies.
1383 of 1943
TOPIC REVIEW

There is no proven algorithm for management of pediatric neck masses, and management depends on the use of
sound clinical judgment and judicious surveillance for red flags.

However the following approaches are often recommended and practiced:

Watchful waiting for up to six for patients with suspected reactive lymphadenitis often evident by bilateral
lymphadenopathy with no red flag features for malignancy or deep cervical abscess).

Although the use of empirical antibiotics is controversial, it is the wide accepted practice in patients thought
to have suppurative lymphadenitis (e.g., lymphadenopathy with marked erythema and tenderness, asymmetric
lymphadenopathy and systemic symptoms).

If symptoms have not improved with antibiotics or if the mass persists longer than four weeks, investigation
with ultrasound and serological screens for atypical infections including mycobacteria is warranted. Targeted
serological investigations for atypical infections may be indicated if the patient has associated risk factors or
if a presumed reactive lymphadenitis has not improved with conservative management.

While there is no strong evidence for the value of a routine full blood count (FBC), it is recommended if
systemic disease is suspected or the diagnosis of infection is uncertain. An FBC and blood film may help
identify pancytopenia and atypical cells suggestive of hematological malignancy, while the differential may
suggest a viral or bacterial pathology.

It is recommended that patients with suspected reactive lymphadenitis are referred to a head and neck
surgeon if the patient’s condition deteriorates while on antibiotics, the mass persists longer than six weeks of
observation or a collection requiring incision and drainage develops.

Timely referral to a head and neck surgeon for consideration of surgical management is advised for all
suspected congenital neck masses. Medical imaging (typically ultrasound) should be considered prior to
referral for determining the urgency of the referral process. Secondary infection of a congenital mass may
require more urgent specialist review when the patient does not respond to oral antibiotics.

All neck masses suspicious for malignancy are best referred urgently to a head and neck surgeon for further
evaluation including possible biopsy. Medical imaging, including ultrasound (most commonly), CT or MRI,
may be considered as part of the referral.

References

• RACGP – AJGP - Paediatric neck lumps: An approach for the primary physician

• AAFP - Evaluation and Management of Neck Masses in Children

Time spent: QID:1544 Last updated:


2023-2-12

1384 of 1943
A 20-month-old male infant is brought to the Emergency Department, by his parents, with complaints of cough and
shortness of breath. On examination, there is bilateral wheezing over lung fields on auscultation. Which one of the
following could be the most likely diagnosis?

A. Bronchiolitis.
B. Croup.
C. Epiglottitis.
D. Asthma.
E. Foreign body aspiration.

Incorrect. Correct answer is D


45% answered correctly

Explanation:

Correct Answer Is D

Wheeze is a common presentation in young children. About 20% of infants have wheeze in infancy, and at least 40%
of children <6 years of age have at least one wheezing episode.

According to Australian pediatric guidelines, the most common cause for cough and wheeze in toddlers are foreign
body aspiration and viral infections. While the wheeze caused by foreign body aspiration tends to be local and
usually unilateral, viral infections manifest with a more widespread and bilateral wheeze. According to the algorithm
by the Royal college of Australian general practice (RACGP), if there is no symptom of coryza, the first diagnosis in
that age group would be foreign body, especially if the wheeze is localized or unilateral.

Although no coryza symptoms are mentioned in the scenario, bilaterality of the wheeze makes foreign body
aspiration (option E) less likely, and either asthma or a viral infection should be considered as the most likely
possibilities.

Bronchiolitis (option A) matches the scenario at first glance; however, the condition is more common within the first
12 months of life and is rarely seen beyond this age. This child is 20 months old and unlikely to have bronchiolitis. Of
the options, only asthma could be an acceptable explanation for this scenario.

NOTE – Asthma is very rare in children younger than 12 months of age and not very common in children between
1 and 2 years age. In fact, according to RACGP guidelines, for wheezing between ages 2 and 5 years terms such
as viral wheezing, pre-school wheeze, episodic viral wheeze and multiple trigger wheeze are preferred to a
definite diagnosis of asthma. In this question, however, the only option that matches the scenario is asthma.

Croup (option B) is a common childhood disease presenting with sudden onset of a distinctive barking cough that is
usually accompanied by stridor, hoarse voice, and respiratory distress resulting from upper airway obstruction. Since
there is no lower respiratory tract involvement, wheeze is not a feature.

Epiglottitis (option C) is caused by infection (mostly bacterial) of the epiglottis. The condition presents with painful
swallowing, hyperextended neck, drooling, and signs of upper respiratory tract obstruction such as difficulty
breathing and stridor. The child is often toxic and miserable. Cough and wheezing in particular are not features of
epiglottitis.

References

• RACGP – AFP – The wheezing child: an algorithm

• RCH – Acute upper airway obstruction

1385 of 1943
Last updated:
Time spent: QID:1548 2023-2-12

1386 of 1943
Kim, 3 years old, is brought to the Emergency Department by her parents reporting that their daughter is not feeling
well at all. According to them, she developed a fever this morning and started to refuse food and crying and throwing
it out when she was fed. A short while later she became miserable and inconsolable, started drooling, and her breath
became noisy. On examination, she has a pulse rate of 140 bpm, respiratory rate of 34 breaths per minute with
expiratory stridor, and a fever of 39.1°C. She maintains her neck extended. Lungs are clear on auscultation. Which
one of the following could be the most likely diagnosis?

A. Croup.
B. Acute bronchiolitis.
C. Retropharyngeal abscess.
D. Acute epiglottitis.
E. Bacterial pharyngitis.

Incorrect. Correct answer is D


45% answered correctly

Explanation:

Correct Answer Is D

The given clinical picture is very suggestive of acute epiglottitis as the most likely diagnosis.

Epiglottitis is an acute inflammation in the supraglottic region of the oropharynx with inflammation of the epiglottis,
vallecula, arytenoids, and aryepiglottic folds. Acute epiglottitis affects all age groups. In children it is more common
between ages 2 and 4 years; however, introduction of Hemophilus Influenza vaccine has resulted in a significant
decline in incidence of the condition in children. The condition is not as common in children anymore.

Epiglottitis often has a short course of progression and presents with high fever, sore throat,
dysphagia/odynophagia, and drooling due to inability to swallow the saliva because swallowing is painful and
difficult. The child is miserable and often adopts a tripod position with a hyperextended neck to maintain the airway
open. Muffled voice (hot potato voice), as if the patient is struggling with a mouthful of hot food, is another feature
seen in epiglottitis. Lungs are clear on auscultation but later as the condition escalates, expiratory stridor develops
which is an ominous sign.

1387 of 1943
Tripod position in a child with epiglottitis

How the tripod position maintains the airway patent

(Option A) Croup is the inflammation of larynx with or without involvement of the trachea and bronchi. The condition
is caused by viruses with the RSV virus being the most common etiology. Children with croup present with a typical
seal barking cough and inspiratory stridor. Fever is often minimal or absent. There is no drooling, dysphagia,
odynophagia. There is usually a preceding viral prodrome including runny nose and sneezing. Lungs are clear on
auscultation unless more distal airways are involved as well.

(Option B) Bronchiolitis, as the name implies, is the inflammation of bronchiole by viral respiratory infections. It is
usually seen in infants younger than 12 months, and present with cough, tachypnea, chest retraction, and
widespread wheeze and crackles on lung auscultation.

1388 of 1943
(Option C) Retropharyngeal abscess has a similar presentation to epiglottitis; however, the rapid onset of symptoms
in the absence a preceding bacterial upper respiratory tract infection makes retropharyngeal abscess a far less likely
diagnosis.

(Option E) Although bacterial pharyngitis can have a rapid course similar to epiglottitis, there are differences in
presentation. Sore throat is a shared feature but acute pharyngitis does not cause the tripod position, hyperextension
of the neck, and expiratory stridor. There might be refusal of food due to the sore throat but drooling is rarely seen in
acute pharyngitis.

References

• RACGP – I can’t breathe

• RCH – Acute upper airway obstruction

Time spent: QID:1564 Last updated:


2023-2-12

1389 of 1943
Caleb, 7 years old, is brought to your clinic by his mother with complaint of bilateral leg pain for the last couple of
days. The pain is more prominent on the right side and is felt in the thighs behind the knee and in the calves. The
pain occurs intermittently and more often in late evening and in the night and sometimes wakes him up. Caleb had
an upper respiratory tract infection 10 days ago presenting with sore throat, mild fever and runny nose. He is
otherwise healthy and has had no serious medical problem to date. His vaccination is up to date. On physical
examination, the only remarkable finding is mildly-tender cervical lymphadenopathy. Hip, knee, and ankle joints of the
right leg are normal with no swelling, warmth, or tenderness, and have preserved range of motion. He walks normally
without limp. Which one of the following could be the most likely diagnosis?

A. Perthes disease.
B. Slipped capital femoral epiphysis (EFCS).
C. Growing pains.
D. Transient tenosynovitis.
E. Septic arthritis of the hip.

Incorrect. Correct answer is C


45% answered correctly

Explanation:

Correct Answer Is C

The scenario is a typical presentation of growing pain.

Growing pains refers to pain in the lower extremities of growing active children. The condition is the cause of leg
pain in 20-30% of children between 2 and 12 years with the peak incidence in preschool children. Unlike what the
name implies, growing pains are not caused by growth, rather they are hypothesized to be related to excessive
physical activity and exercise.

Although accepted as an independent entity, growing pain is a diagnosis of exclusion, and all other potential
diagnoses need to be excluded before such diagnosis is made. To make a diagnosis of growing pain, there must be
no associated constitutional symptoms such as fever, malaise or change in appetite, no signs of joint involvement
(swelling, tenderness, warmth or redness) and no related significant clue in history or physical examination pointing
toward another etiology.

The pain is mostly felt in the thighs, calves and behind the knee. The pain is intermittent and in mostly present in the
afternoon and evening after activity during the day. Pain may also wake up the child at night. The pain responds to
conservative measures such as heat, massage and over-the-counter analgesics, and resolves by the morning.

Growing pains are bilateral in 80%. Although in 20% cases growing pain may be unilateral, it is recommended that to
remain on the safe side by considering an alternative diagnosis rather than growing pain is considered and promptly
investigated until proven otherwise.

(Option A) Perthes disease is avascular necrosis of the femoral head. The condition is seen mostly in children
between 4 and 10 years of age and is associated with constant hip pain and often a limp. Affected hip often has
limited range of motion.

(Option B) EFCS is mostly seen in obese boys between ages 10 and 14. Limp and limited range of motion of the
affected hip is usually features seen in the condition.

(Option D) Transient tenosynovitis (irritable hip) is the most common cause of limp in the preschool age group
usually occurring in children between 3 and 8 years old. There is often a history of preceding viral illness 1 to 2
weeks before. Children with transient tenosynovitis are able to walk but with pain. Affected children are otherwise
well and have no fever. The range of motion, especially internal rotation, is mildly decreased. Although the scenario
also resembles transient tenosynovitis, the normal range of motion of the hip in Caleb as well as the intermittent
nature of the pain and also absence of pain on walking make such diagnosis less likely than growing pains.
1390 of 1943
(Option E) In septic arthritis of the hip, the child is often ill and febrile. The range of motion of the hip is decreased
and joint motion causes pain. The child would refuse to put weight on the affected leg.

References

• UpToDate – Growing pain

• Sports Health - Growing pains

• RCH - The limping or non-weight bearing child

Time spent: QID:1566 Last updated:


2023-2-12

1391 of 1943
A 5-year-old boy is presented to your practice with headache and vomiting for the past 6 weeks. On examination, he
has a temperature of 37.8°C. A non-tender lymph node is palpated in the posterior triangle of the neck. Which one of
the following is the investigation to consider at this stage?

A. Blood culture.
B. CT scan of the head.
C. Urine culture.
D. Lumbar puncture.
E. Fine needle aspiration of the lymph node.

Incorrect. Correct answer is E


45% answered correctly

Explanation:

Correct Answer Is E

Lymphadenopathy in children, or neck masses in this age group in general, is a common presenting symptom in
general practice causing remarkable parental concern as well as a diagnostic dilemma for the doctor.

While lymphadenopathy in children is often a result of a benign inflammatory/ infectious process such as a viral
upper respiratory tract infection, more serious diagnoses such as systemic infections, atypical infections and
malignancies must also be considered. Examples are: TB, Infectious mononucleosis, cytomegalovirus infection, cat
scratch disease, hematological and non-hematological malignancies, and Kawasaki disease.

As always, a thorough history and physical examination is the main component of approaching children with
lymphadenopathy or neck mass. Three important areas to inquire are (1) the timeline of the lump (when and for how
long?), (2) time course (how fast has t grown/ is it growing), and (3) associated symptoms and potential red flags.

On physical examination, location, size, consistency, and tenderness give important clues. Lymphadenopathy
commonly arises as a lateral lump in the anterior or posterior triangle, and may present an inflammatory or
neoplastic process. Lymphadenopathy in the posterior triangle has higher risks of malignancy. Supraclavicular
lymphadenopathy is very concerning and considered a red flag.

A very important and crucial part of approach to neck lumps in children is to look for the presence of alarming signs
and symptoms in physical exam and history – the red flags.

Red flags include the following:

Weight loss
Sustained fever
Night sweats
Generalized lymphadenopathy
Signs and symptoms of pancytopenia
Mass persisting> 6 weeks
Lymph node> 3 cm
Thyroid mass
Supraclavicular mass
Hard, irregular mass
Fixed mass

While watchful waiting for 6 weeks is recommended for lymphadenopathy with high likelihood of being reactive (e.g.,
history of a recent viral infection, node tenderness, etc.), prompt and more invasive approach should be taken in the
presence of red flag(s).

1392 of 1943
This child has a mild fever on examination but it is not clear if it has been sustained. There is also a lymph node
found but it is not clear if it has been present for more than 6 weeks. Presence of headache and vomiting are not red
flags either but these all together indicate an unwell child, who has painless lymphadenopathy in the posterior
triangle. Lymphadenopathies in the posterior triangle have higher chances of association with malignancies. For
these reasons, an FNA probably guided by ultrasound is the most appropriate option to consider first among others.

While FNA can be traumatic and carries a risk of inadequate tissue sampling, it is considered a highly sensitive and
specific investigation (both reported up to >90%) and is less invasive than open biopsy in all patients carrying red
flags.

The presence of fever and lymphadenopathy, may also suggest an infectious process. While there is no strong
evidence for the value of a routine full blood count (FBC), it is recommended if systemic disease is suspected or if
the diagnosis of infection is uncertain. An FBC and blood film may help identify pancytopenia and atypical cells
suggestive of haematological malignancy, while the differential may suggest a viral or bacterial pathology. Although
a blood culture (option A) may be required later on bases on the FBC or other findings, it does not seem to take
precedence over FNA at this stage.

This child has headache, vomiting and a mild fever. This constellation of symptoms can be seen in children with
meningitis. In children, classical signs of meningeal irritation (e.g., Kernig’s and Brudzinski’s signs) might be absent.
However, the duration of headache and vomiting for 6 weeks, make meningitis a less likely diagnosis; therefore, an
LP (option D) is not necessary for now.

Ultrasonography (USS) is a readily accessible, inexpensive and radiation-free imaging modality that can provide
crucial information about the location, size and consistency of a cervical mass. For this reason, ultrasound is the
preferred initial imaging study for most children with a neck mass. USS can identify features typical of numerous
congenital masses as well as benign and malignant lymphadenopathy, and can provide guidance for fine needle
aspiration (FNA) if deemed necessary. USS is the initial imaging modality of choice for thyroid gland pathology
including malignancy and thyroglossal duct cysts. CT scan (option B) involves radiation exposure and should be
avoided in children unless a malignancy or deep cervical abscess is suspected.

Magnetic resonance imaging (MRI) is an increasingly available and popular alternative to CT and is considered the
gold standard imaging study for vascular malformations. While MRI does not involve radiation exposure, the
frequent requirement for sedation in the pediatric population does decrease its utility. Histopathology may be
required to confirm diagnoses and guide treatment in certain cases.

For this child, urine culture (option C) is not indicated unless investigations exclude other more likely causes of such
presentation. This boy is old enough to have more specific symptoms of urinary tract infection such as dysuria or
frequency or, at least, abdominal pain. It is unlikely that urinary infections present with painless cervical
lymphadenopathy and headache.

TOPIC REVIEW

There is no proven algorithm for management of pediatric neck masses, and management depends on the use of
sound clinical judgment and judicious surveillance for red flags.

However the following approaches are often recommended and practiced:

Watchful waiting for up to six for patients with suspected reactive lymphadenitis often evident by bilateral
lymphadenopathy with no red flag features for malignancy or deep cervical abscess).

Although the use of empirical antibiotics is controversial, it is the wide accepted practice in patients thought
to have suppurative lymphadenitis (e.g., lymphadenopathy with marked erythema and tenderness, asymmetric
lymphadenopathy and systemic symptoms).

If symptoms have not improved with antibiotics or if the mass persists longer than four weeks, investigation
with ultrasound and serological screens for atypical infections including mycobacteria is warranted. Targeted
serological investigations for atypical infections may be indicated if the patient has associated risk factors or
if a presumed reactive lymphadenitis has not improved with conservative management.
1393 of 1943
While there is no strong evidence for the value of a routine full blood count (FBC), it is recommended if
systemic disease is suspected or the diagnosis of infection is uncertain. An FBC and blood film may help
identify pancytopenia and atypical cells suggestive of hematological malignancy, while the differential may
suggest a viral or bacterial pathology.

References

• RACGP – AJGP - Paediatric neck lumps: An approach for the primary physician

• AAFP - Evaluation and Management of Neck Masses in Children

Time spent: QID:1572 Last updated:


2023-2-12

1394 of 1943
Tom, 10 years old, is brought to your clinic by his mother who is concerned about his height and weight. She says
Tom has always been the biggest boy in his classes since he started school. On examination, his weight and height
are on 96 and 97 percentile, respectively. Which one of the following would be the most important initial evaluation
to consider?

A. TSH.
B. Urine cortisol level.
C. Bone age.
D. Blood sugar.
E. MRI of the head.

Incorrect. Correct answer is D


45% answered correctly

Explanation:

Correct Answer Is D

For children and adolescents aged 2–18 years, growth is monitored based on age, height, and weight, using sex-
specific Body Mass Index (BMI) percentile charts. BMI is not a fixed measure in this age group but varies with normal
growth, stage of puberty, and sex. Either the United States Centers for Disease Prevention and Control (US-CDC) or
WHO BMI percentile charts may be used, with the same chart used over time to allow for consistent monitoring of
growth.

The US-CDC categorizes overweight as between the 85th and 95th percentile and obesity as above the 95th
percentile. The WHO categorizes overweight as between the 85th and 97th BMI percentiles and obesity as above the
97th percentile. These categories are not diagnostic but contribute to the overall clinical impression of the child or
adolescent being measured.

Tom, based on either chart, is obese. The likelihood that childhood overweightness and obesity will persist into
adulthood increases with the age of the child and with the presence of parental obesity. One of the strongest
predictors of a child’s weight is the weight status of his/her parents. For overweight or obese children, initial
assessment by history and clinical assessment should determine current health problems and risks for future
disease.

History taking includes developmental history, physical and mental health (including family history of obesity), and
current health behaviors.

Clinical assessment includes pubertal stage, possible causes for overweightness or obesity (e.g., hypothyroidism),
and indicators of comorbidities (e.g., metabolic syndrome, raised blood pressure, joint pain, gastrointestinal
symptoms, insulin resistance, intertrigo, dental health etc.)

Of the options, measuring blood glucose is the most important investigation to consider at this stage as well as a
lipid profile and liver function tests. Obese children are at increased risk of metabolic syndromes and the condition
should be identified and addressed early in the course of assessment and treatment.

It is also very important that secondary causes for obesity are considered and excluded. Hypothyroidism can be a
potential cause of obesity; however, the prevalence of hypothyroidism in overweight and obese children and
adolescents is not greater than in normal-weight ones. Given these, measuring TSH (option A) would be an
appropriate option if the clinical picture is suggestive of hypothyroidism or if no apparent cause for overweightness
or obesity can be identified. It is however important to note that the prevalence of hypothyroidism is not greater than
in normal-weight children and adolescents.

(Option B) Urine cortisol level would have been indicated if Tom had clinical features suggestive of Cushing
syndrome such as moon face, central obesity, etc.
1395 of 1943
(Option C) Bone age is often used as the initial assessment for short stature that is not the problem here.

(Option E) This child does not have any alarming features necessitating brain imaging using MRI or CT scan.

References

• RACGP - Adolescent overweight and obesity

• NHMRC - Clinical Practice Guidelines for the management of overweight and obesity

Time spent: QID:1580 Last updated:


2023-2-12

1396 of 1943
A 6-year-old Indigenous boy presents to your practice in a rural area presents with purulent left ear discharge for the
past 2 months. History and physical examination establish a diagnosis of chronic suppurative otitis media. Culture
of the ear discharge is positive for pseudomonas aeruginosa. After ear toileting, which one of the following is the
most appropriate treatment option to consider for this child?

A. Oral Amoxicillin.
B. Oral Augmentin.
C. Ciprofloxacin ear drop.
D. Paramycetin with steroid ear drops.
E. Gentamycin ear drops.

Incorrect. Correct answer is C


45% answered correctly

Explanation:

Correct Answer Is C

Chronic suppurative otitis media (CSOM)is the infection of the middle ear with a perforated tympanic membrane and
discharge of at least 6 weeks duration.

CSOM can cause hearing impairment. Occasionally and in less developed countries and Aboriginal and Torres Strait
Islander people, serious complications such as intracranial infections and mastoiditis can occur.

Treatment of CSOM includes ear toileting and topical antibiotics. Ear toileting is a very important step and should be
performed before instillation of antibiotic drops. Ear toileting can be performed by a healthcare professional with
measures such as mechanical suction, or cotton wool on a probe under direct visualization. It can be also done by
the patient or cared using dry mopping the ear with rolled tissue spears or similar, 6-hourly.

Quinolones such as ciprofloxacin 3% ear drop is the antibiotic of choice after ear toileting achieves a dry and clean
ear canal. It is given as 5 drops into the affected ear, 12-hourly, until the middle ear has been free of discharge for at
least 3 days.

Oral antibiotics such as amoxicillin (option A) or Augmentin (option B) are not appropriate options because the most
common organism in CSOM is pseudomonas aeruginosa which is a gram-negative pathogen. Since topical
treatment eradicates the infection, systemic antibiotic therapy is unnecessary.

Paramycetin (chloramphenicol) (option D) do not adequately cover pseudomonas aeruginosa and is not an
appropriate treatment option. Also, there is inadequate evidence to support use of topical steroids in combination
with topical antibiotics for treatment of CSOM.

Aminoglycoside-based ear drops such as gentamycin (option E) have been previously used for CSOM; however, due
to concerns of safety, especially the risk of auditory and vestibular toxicity are not preferred options now.

References

• Therapeutic Guideline – Antibiotics

Last updated:
Time spent: QID:1616 2023-2-12

1397 of 1943
An 8-year-old boy is brought to the Emergency Department with right ear pain and purulent discharge since yesterday
and a red swelling behind the right ear since this morning. He has the history of chronic bilateral otitis media for
which he underwent bilateral grormmet procedure. Which one of the following is the most appropriate next step in
management?

A. Take samples for culture.


B. Hearing assessment.
C. Antibiotics.
D. CT scanning of the temporal bone.
E. Involvement of an ENT consultant.

Correct
45% answered correctly

Explanation:

Correct Answer Is A

The scenario is a typical presentation of acute mastoiditis (AM). Acute mastoiditis is a rare yet the most common
suppurative complication of acute otitis media (AOM) and may be associated with intracranial complications. The
diagnosis of AM is almost always made clinically based on the presence a current or preceding history of AOM plus
the following findings:

Post auricular inflammatory signs such as erythema, edema, tenderness, or fluctuance,

and/ or

A protruding auricle and/or external auditory canal edema

The mainstay of initial treatment is intravenous antibiotics (flucloxacillin + a third-generation cephalosporin) (option
D). Before commencement of antibiotics, however, a sample from the ear discharge should be taken for microscopy,
culture, and sensitivity because antibiotics may alter the result. If the tympanic membrane is intact, tympanocentesis
or myringotomy should be obtained to both relief the middle ear pressure and obtain a sample. This should be
performed by an ENT consultant. Some cases may require surgical treatment; therefore, an ENT consultant must be
involved as well (option E).

CT scanning of the temporal bone (option D) is the standard for evaluation of the extend and complications of
mastoiditis and may be considered for further management planning. The results do not affect the initial
management; therefore, not the best appropriate next step. It may later be considered for further management, often
by the ENT specialist.

A hearing assessment (option B) using audiometry must be obtained after the convalescence
from the acute phase mostly due to medicolegal issues. Audiometry is rarely appropriate or
useful during the acute phase of AM.

NOTE - In general, mastoiditis includes all inflammatory processes of the mastoid air cells of the temporal bone.
Since the mastoid is contiguous to and an extension of the middle ear cleft in the temporal bone, by definition
and virtually, every patient with acute otitis media or chronic middle ear inflammatory disease has mastoiditis.
However, in practice, acute mastoiditis refers to instances where middle ear infection spreads beyond the
mucosa of the middle ear cleft, and there is involvement of mastoid air-cell system or periosteitis of the middle
ear cleft. While acute mastoiditis is associated with acute otitis media (AOM), chronic mastoiditis is most
associated with chronic suppurative otitis media (CSOM) and particularly with cholesteatoma formation.

References

1398 of 1943
• RCH- Acute otitis media

• Medscape – Mastoiditis
Last updated:
Time spent: QID:1618 2023-2-12

1399 of 1943
Raymond, 9 years of age, is brought to your GP clinic with right sided hearing impairment. A Rinne test shows that air
conduction is better than bone conduction in both ears. Weber test indicates localization to the left ear. If the
following options, which one justifies these findings?

A. Acute otitis media.


B. Acute otitis externa.
C. Glue ear.
D. Otosclerosis.
E. Acoustic neuroma.

Incorrect. Correct answer is E


45% answered correctly

Explanation:

Correct Answer Is E

Hearing loss is a common presentation to clinical practice. In such cases Rinne and Weber tests are performed to
differentiate between a conductive (middle and outer ear causes) and a sensorineural deafness (caused by damage
to the cochlea or to the 8th nerve – or its central connections). These tests are always done together. The Rinne test
is performed first.

The function of the external ear is to collect sounds vibrations form the air and focus these onto the tympanic
membrane. These vibrations are then transmitted through the middle ear cavity by the ossicular chain (Malleus,
Incus and Stapes). The stapes transmits these vibrations to the cochlea through the oval window (fenestra ovalis).
Sound can also be transmitted through the bones of the skull to the cochlea. The hair cells in the cochlea convert the
physical vibrations into action potentials that are transmitted via the nerves in the vestibulo-cochlear (auditory) nerve
to the brainstem for further processing.

The Rinne test is performed for evaluation of hearing loss in one ear (unilateral hearing loss) and compares
perception of sound transmitted by air conduction to those transmitted by bone conduction through the mastoid.

For the Rinne test, a vibrating tuning fork (typically 512 Hz) is placed initially on the mastoid process behind each ear
until sound is no longer heard, signaled by the patient. The fork is then immediately positioned just outside the ear
with the patient asked to report when the sound caused by the vibration is no longer heard. A normal or positive
Rinne test is when the patient can still hear the vibration while the fork is in front of the ear (air conduction or AC)
after he/she could not hear it while the tuning fork was in the mastoid bone (bone conduction or BC) [AC>BC]. In
conductive hearing loss, bone conduction is better than air or BC > AC, a negative Rinne.

In the Weber test, a vibrating tuning fork (often 256Hz or 512Hz) is placed in the middle of the forehead, or above the
upper lip under the nose over the teeth, or on top of the head in an equal distant from the patient’s ears. The patient
is then asked to report in which ear the sound is louder. A Weber test result is normal (positive) if the patient reports
the sound equally on both sides. Lateralization (hearing the sound better in one ear) is due to either a sensorineural
or conductive problem. In a patient with sensorineural hearing loss, the normal ear hears the sound better than the
defective one. In a patient with conductive hearing loss, the sound is lateralized to the affected side.

In this child, air conduction is greater than bone conduction in both ears (bilateral positive Rinne). This means that
the cause of hearing loss is not conductive. In other words, acute otitis media, (option A), acute otitis externa (option
B), Glue ear (otitis media with effusion) (option C), or otosclerosis (option D) [affecting the ossicular chain the
middle hear, and impairs sound transmission] cannot be the underlying cause of the hearing impairment in this child.

Weber test, on the other hand, shows lateralization the left side. Given the normal Rinne test results, there is right-
sided sensorineural hearing impairment that only can be justified by acoustic neuroma among options.

Acoustic neuromas are intracranial, extra-axial tumors arising from the Schwann cell sheath investing either the
vestibular or cochlear nerve. As acoustic neuromas increase in size, they eventually occupy a large portion of the
1400 of 1943
cerebellopontine angle. Acoustic neuromas account for approximately 80% of tumors found within the
cerebellopontine angle. The remaining 20% are principally meningiomas.

Unilateral hearing loss is overwhelmingly the most common presenting symptom. Any unilateral sensorineural
hearing loss should be assumed to have been caused by an acoustic neuroma until proven otherwise. The tumor can
produce hearing loss through at least two mechanisms, direct injury to the cochlear nerve or interruption of cochlear
blood supply. Progressive injury to cochlear fibers probably accounts for slow progressive neurosensory hearing loss
observed in a significant number of patients with acoustic neuromas. Sudden and fluctuating hearing losses are
likely to be caused by disruption of cochlear blood supply.

Headache and imbalance are other symptoms that may develop later in the course of the disease but are less
common as presenting symptoms compared to hearing loss.

Acoustic neuroma is rare in children, yet it is the only explanation for this case scenario among other options.

For more details about Rinne and Weber tests watch the following video:

References

• RACGP – Ear examination

• Medscape – Acoustic Neuroma

Time spent: QID:1620 Last updated:


2023-2-12

1401 of 1943
Which one of the following medications will not cause serotonin syndrome if given along with selective serotonin re-uptake inhibitors (SSRIs)?

A. Tryptophan.

B. Amitriptyline.

C. Moclobemide.

D. Chlorpromazine.

E. Lithium.

Incorrect. Correct answer is D


45% answered correctly

Explanation:

Correct Answer Is D

Serotonin syndrome is a potentially life-threatening condition associated with increased serotonergic activity in the central nervous system. It
is seen with therapeutic medication use, inadvertent interactions between drugs, and intentional self-poisoning.

The clinical features of serotonin syndrome include:

Cognitive features: confusion, agitation, hypomania, hyperactivity, restlessness


Autonomic features: hyperthermia, sweating, tachycardia, hypertension, mydriasis, flushing, shivering
Neuromuscular features: clonus (spontaneous/inducible/ocular), hyperreflexia, hypertonia, ataxia, tremor

NOTE - hypertonia and clonus are always symmetrical and are often much more dramatic in the lower limbs

The drugs implicated to cause serotonin syndrome and their mechanism of action are summarized in the folloiwng table:

Drug Mechanism of action


L-Tryptophan Serotonin precursor
SSRIs (fluoxetine, sertraline, citalopram, …) Inhibit serotonin reuptake
Tricyclic antidepressants (amitriptyline, clomipramine,
Inhibit serotonin reuptake
nortriptyline, …)
Monoamine oxidase inhibitors (A>B) (moclobemide,
Inhibit metabolism of 5-HT
selegiline, …)
Pethidine Serotonin agonist
Tramadol Inhibits serotonin reuptake
LSD Partial serotonin agonist
Buspirone Partial serotonin agonist
Amphetamines and anorectics ↑ 5-HT release & ↓ reuptake
Atypical antidepressants Various
St John’s wort Unknown – may be all of the above
Lithium Unknown

Chlorpromazine is a HT2 receptor antagonist used for treatment of serotonin syndrome. It is not does not contribute to development of
serotonin syndrome.

TOPIC REVIEW

Management of serotonin syndrome

Meticulous supportive care is the mainstay of therapy:

All serotonergic drugs should be stopped at once and care should be taken that no other precipitant be inadvertently administered

1402 of 1943
Those with moderate to severe serotonergic symptoms should be admitted to the hospital. Those with hyperthermia require
admission to the ICU.

Intravenous hydration to establish a reassuring urine output

Regular monitoring of temperature, pulse, blood pressure and urine output

If the patient is hyperthermic, the temperature should aggressively be lowered using cool water sprays, ice packs, and even paralysis
and ventilation

Benzodiazepines may be used to control seizures and muscle hyperactivity

Specific treatment of hypertension is usually not required

Specific therapy

The serotonin antagonist cyproheptadine is the first-line medication for treatment of moderate to severe serotonin syndrome: an initial does
of 4-8mg is given orally. This can be repeated in 2 hours if needed. Cyproheptadine should be discontinued if there is no response after a total
dose of 16mg. If there is a response, it may be continued in divided doses up to 32 mg/day (e.g. up to 8 mg, 6-hourly)

Other suggested medications are chlorpromazine and propranolol, but these drugs are associated with more adverse effects. If sedation is
desired, chlorpromazine may be more appropriate to use rather than other routine sedative agents. Since it can cause hypotension, patients
must receive sufficient volume loading.

1403 of 1943
A 68-year old woman presents with migratory superficial thrombophlebitis and recurrent deep vein thrombosis (DVT) of his lower extremities.
Which one of the following malignancies is most likely to be the cause?

A. Ovarian cancer.

B. Endometrial cancer.

C. Gastric cancer.

D. Pancreatic cancer.

E. Lung cancer.

Incorrect. Correct answer is D


45% answered correctly

Explanation:

Correct Answer Is D

Trousseau's syndrome is a variant of venous thrombosis characterized by recurrent and migratory pattern of superficial thrombophlebitis,
frequently in unusual sites such as the arm or chest. Patients Trousseau's syndrome usually have an occult tumor (usually adenocarcinoma)
which is not always detectable at the time of presentation, and might be present months to years before the malignancy is clinically evident.

The most common cancers associated with Trousseau syndrome and the prevalence of the sydnrome in each are as follows:

Pancreatic cancer – 24%


Lung cancer – 20%
Prostatic cancer – 13%
Gastric cancer – 12%
Acute leukemia – 9%
Colon cancer – 5%

Of the given options, pancreatic cancer has the highest association with Trousseau sign.

Trousseau syndrome is extremely rare in ovarian and endometrial cancer; however, these cancers can result in hypercoagulable states and
recurrent episodes of DVT.

References

• Medscape - What are the signs and symptoms of migratory thrombophlebitis

• PubMed - Thrombophlebitis migrans in a man with pancreatic adenocarcinoma: a case report

Last updated:
Time spent: QID:500
2023-2-12

1404 of 1943
A 56-year-old man is admitted to the hospital for treatment of methicillin resistant staphylococcus aureus (MRSA) pneumonia. He is a known
case of hypertension and on amilodipine and hydrochlorothiazide. You decide to start him on vancomycin. Twenty minutes after intravenous
infusion of vancomycin is started, he develops a generalized erythematous rash all over his body. There is no respiratory symptom and the
vitals are within the normal range. Which one of the following is the most likely cause of this presentation?

A. Idiosyncratic drug reaction.

B. Toxic shock syndrome.

C. Stevens-Johnson syndrome.

D. Anaphylaxis.

E. Fixed drug eruption.

Correct
45% answered correctly

Explanation:

Correct Answer Is A

Vancomycin can cause several different types of hypersensitivity reactions, ranging from localized skin reactions to generalized
cardiovascular collapse. Red man syndrome (RMS) is the most reported adverse reaction. This patient's presentation is more consistent with
Red man syndrome (RMS).

RMS, also called “red neck syndrome” is an idiosyncratic rate-dependent infusion reaction, not a true allergic reaction. The etiology is
unknown (idiopathic) but it is not thought to involve drug-specific antibodies and, in contrast to allergic reactions, may develop with first time
administration of vancomycin.

RMS is characterized by flushing, erythema, and pruritus, usually affecting the upper body, neck, and face more than the lower body. Pains and
muscle spasms in the back and chest, dyspnea, and hypotension may also occur. RMS is rarely life-threatening; however, severe
cardiovascular toxicity and even cardiac arrest can occur.

IgE-mediated anaphylaxis can present with symptoms similar to those of severe RMS. Unlike RMS, an IgE-mediated reaction to vancomycin
does not occur with initial administration. Such reaction requires previous sensitization. Severe RMS and anaphylaxis can present with similar
signs and symptoms but wheezing and respiratory distress are more common in anaphylaxis. On the other hand, chest pain is more common
in RMS than anaphylaxis.

NOTE - Since it may not be possible to distinguish anaphylaxis from severe RMS based upon clinical presentation, the patient should be
assumed to have anaphylaxis and treated promptly if in doubt.

Concomitant use of some drugs that are capable of inducing histamine release may increase the risk of adverse drug reaction associated
with vancomycin. Opiates and contrast media are the most implicated medications. Some studies suggest that the dihydropyridine calcium
channel blockers nifedipine may increase the risk of vancomycin-related adverse drug interaction. No significant interaction with amilodipine
has been reported thus far.

NOTE - Other possible adverse reactions associated with vancomycin include:

Maculopapular eruptions

Vancomycin-related linear IgA bullous dermartosis

Leukocytosis, eosinophilia, neutropenia, and immune thrombocytopenia - neutropenia tends to occur with longer courses of therapy.
Weekly monitoring of the white blood cell count and differential leukocyte counts during prolonged administration is indicated

Drug-induced fever - uncommonly, vancomycin been implicated as a cause of drug-induced fever

Nephrotoxicity, especially in patients receiving nephrotoxic drugs (e.g. aminoglycosides), or who have renal insufficiency or altered
hemodynamics. Interstitial nephritis is seen at occasions

References

• http://www.medsafe.govt.nz/profs/datasheet/v/Vanco

• https://ainotes.wikispaces.com/file/view/Vancomyci

1405 of 1943
• http://www.ncbi.nlm.nih.gov/pmc/articles/PMC270616
Last updated:
Time spent: QID:504
2023-2-12

1406 of 1943
During hospital stay of a 62-year-old woman, she develops pneumonia. Since methicillin resistant staphylococcus aureus (MRSA) is highly
suspected, intravenous vancomycin is started. After 20 minutes of infusion, the patient develops a generalized pruritic erythematous rash all
over her face, torso and arms. She also complains of chest tightness. The infusion is stopped immediately, and within few minutes the rash
resolves. A quick drug history reveals that he is on aspirin and amilodipine for treatment of her hypertension. Which one of the following is
correct regarding continuation of vancomycin?

A. Change the concentration of vancomycin.

B. Restart the infusion at a slower rate.

C. Switch to another antibiotic.

D. Never give her vancomycin again.

E. Give vancomycin along with prednisolone.

Incorrect. Correct answer is B


45% answered correctly

Explanation:

Correct Answer Is B

Vancomycin can cause several different types of hypersensitivity reactions, ranging from localized skin reactions to generalized
cardiovascular collapse. Red man syndrome (RMS) is the most reported adverse reaction.

RMS, also called “red neck syndrome” is a rate-dependent infusion reaction, not a true allergic reaction. The etiology is unknown but it does
not involve drug-specific antibodies and, in contrast to allergic reactions, may develop with the first administration of vancomycin.

RMS is characterized by flushing, erythema, and pruritus, usually affecting the upper body, neck, and face more than the lower body. Pains and
muscle spasms in the back and chest, dyspnea, and hypotension may also occur. Chest pain and chest tightness are other reprted clinical
features. RMS is rarely life-threatening; however, severe cardiovascular toxicity and even cardiac arrest can occur.

IgE-mediated anaphylaxis can present with symptoms similar those of severe RMS. Unlike RMS, an IgE-mediated reaction to vancomycin does
not occur with first administration. However, there may be some characteristics distinguishing features. While wheezing, respiratory, and
angioedema are more common in anaphylaxis, chest pain or a sense of chest tightness is seen more frequently in RMS.

NOTE - Since it may not be possible to distinguish anaphylaxis from severe RMS based on clinical presentation, the patient should be
assumed to have anaphylaxis and treated promptly if in doubt.

Concomitant use of some drugs associated with histamine release may increase the risk of vancomycin adverse reactions. Opiates and
contrast media are the most implicated medications. Some studies suggest that the dihydropyridine calcium channel blockers – nifedipine
may increase the risk of vancomycin-related adverse drug interaction.

Management of RMS is as follows:

For mild to moderate reactions, in which the patient is uncomfortable due to flushing or pruritus, but without hemodynamic instability, chest
pain or muscle spasms, the infusion should be interrupted and patient be treated with diphenhydramine (50 mg orally or intravenously) and
ranitidine (50 mg intravenously). Symptoms usually subside promptly. The infusion can then be restarted at one-half of the initial rate or
10 mg/min, whichever is slower.

For Severe reactions associated with muscle spasms, chest pain, or hypotension, the infusion should be interrupted and the patient be treated
with diphenhydramine (50 mg intravenously) and ranitidine (50 mg intravenously), and intravenous fluids if there is hypotension. Once
symptoms have resolved, the infusion can be restarted, and given over 4 or more hours. For future administration in such patients,
premedication with antihistamines before each dose and infusion over 4 hours is recommended.

If the reaction to vancomycin is anaphylaxis type, immediate treatment would be intramuscular adrenaline. Vancomycin should never be given
again, unless desensitization is performed. Hives, laryngeal edema, and wheezing are suggestive of anaphylaxis.

This patient has typical presentation of RMS . Since chest tightness is present, the management would be restarting the infusion at a slower
rate (over 4 hours) as well as premedication with histamine (and ranitidine).

Unlike antihistamines, addition of prednisolone to vancomycin has not shown to decrease the risk of recurrence of RMS.

References

• http://www.medsafe.govt.nz/profs/datasheet/v/Vanco
1407 of 1943
• https://ainotes.wikispaces.com/file/view/Vancomyci

• http://www.ncbi.nlm.nih.gov/pmc/articles/PMC270616
Last updated:
Time spent: QID:505
2023-2-12

1408 of 1943
Catherine, 76 years old, is rushed to the Emergency Department of the local hospital you work at after she was found drowsy. She has end-
stage ovarian cancer and is on methadone, codeine, paracetamol, naproxen, and diazepam for pain control. On her way to the hospital, she
was given naloxone 0.4 mg intravenously because she has pinpoint pupils. On examination, you notice that her pupils are still pin-point sized,
but she is less drowsy and able to communicate. The first thing she says to you is that she is still in severe pain. Which one of the following
would be the most appropriate next step in management?

A. Give her naproxen.

B. Give her another dose of naloxone.

C. Give her diazepam.

D. Give her more methadone for adequate pain control.

E. Give her morphine, intravenously.

Incorrect. Correct answer is B


45% answered correctly

Explanation:

Correct Answer Is B

With methadone and codeine on her drug list, opiate overdose is the cause of her drowsiness and pin-point pupils until proven otherwise. She
has been given naloxone by paramedics in the ambulance, but her pupils are still pin-point, and she is still drowsy. These are pointers towards
the fact that opiate toxicity is not still fully reversed. In such situations, repeated doses of naloxone are required as the most appropriate next
step in management.

(Option A) It is unlikely that naproxen helps with pain control in a patient who has taken opiates in excess doses. Furthermore, reversal of
opiate overdose is the main and most important objective for now. Efficient pain control may be carefully and judiciously planned for this
patient once she is out of this emergency.

(Option C) Diazepam can lead to more CNS depression and drowsiness; moreover, it may suppress the respiratory drive in a patient who is
already at increased risk of respiratory suppression and apnea due to opiate overdose.

(Options D and E) Giving a patient overdosed with opioids, more doses of opiates of any kind is not an appropriate option, as this deteriorates
the patient’s condition.

References

• Medscape - Opioid Toxicity

Last updated:
Time spent: QID:831
2023-2-12

1409 of 1943
A 16-year-old girl, who is a known case of epilepsy for 5 years, has been started on sodium valproate and lamotrigine 4 weeks ago after her
previous medications failed to control her seizures. She also has the history of recurrent urinary tract infections (UTI) with last episode
6 weeks ago for which she received amoxicillin. Today, she has presented with a maculopapular rash and a fever of 38°C. Which one of the
following is the most likely cause of her presentation?

A. Adverse drug reaction to sodium valproate.

B. Adverse drug reaction to lamotrigine.

C. Adverse drug reaction to amoxicillin.

D. Septicemia.

E. Drug interaction between lamotrigine and sodium valproate.

Incorrect. Correct answer is B


45% answered correctly

Explanation:

Correct Answer Is B

To date, medications have been the most common cause of adverse allergic reactions. Allergic reactions can vary from immediate- to late-
onset ,and from a not-clinically-significant rash to potentially life-threatening complications and systemic involvement.Lamotrigine, on the
other hand, is well known for causing rash as a less frequent adverse effect. Simultaneous use of lamotrigine and sodium valproate has been
associated with the higher chances of rash development.

Rash as an adverse effect of lamotrigine occurs between 5 days and 8 weeks (2 months) of taking lamotrigine. The rash might be
maculopapular that often do not coalesce. Lesions are not tender on palpation but may be pruritic. The rash is not significant if there are no
associated systemic symptoms such as fever, malaise, etc either before the appearance of the rash or contemporaneously.

There is also a more serious, but rarer form of rash that starts as a morbiliform rash progressing more or less rapidly to a diffuse confluent,
and infiltrated erythema with follicular accentuation (hair follicles are raised and bumpy). This rash of lamotrigine is often associated with
complications such as Stevens-Johnson syndrome, toxic epidermal necrolysis (TEN), or drug reaction with eosinophilia and systemic
symptoms (DRESS).

On the sight of any rash while the patient is on lamotrigine, the drug should be immediately stopped, and careful evaluation performed.

(Option A) Sodium valproate has many potential adverse effects, but rash has been a very rare finding as an adverse reaction associated with
this drug. Direct association of the rash and sodium valproate, although not impossible, seems very unlikely. There have been only few reports
about such association.

(Option C) Amoxicillin and other penicillins are among the most common causes of allergic drug reactions while the patient is taking them.
This patient has completed a course of amoxicillin for her UTI. She is not on the medication now; hence amoxicillin is not likely to be the
cause of her presentation.

(Option D) There is no clue in history suggesting septicaemia as a cause to the rash; furthermore, with septicaemia a higher fever would be
expected.

(Option E) Rash is not the result of interaction between lamotrigine and sodium valproate. It is more attributable to lamotrigine with
enhancement by concomitant use of sodium valproate.

TOPIC REVIEW

DRESS is a rare, potentially life-threatening, drug-induced hypersensitivity reaction that includes rash, hematologic abnormalities (eosinophilia,
atypical lymphocytosis), lymphadenopathy, and internal organ involvement (liver, kidney, lung, etc). DRESS is characterized by a long latency
(2 to 8 weeks) between drug exposure and disease onset, a prolonged course with frequent relapses despite the discontinuation of the culprit
drug, and frequent association with the reactivation of a latent human herpes virus infection.

The following medications are the most common causes of DRESS:

Carbamazepine
Lamotrigine
Phenytoin
Phenobarbital
Allopurinol
1410 of 1943
Other drugs less commonly associate with DRESS are sulfonamides (particularly sulfasalazine), dapsone, minocycline and
vancomycin.

Systemic symptoms of DRESS are:

Fever (38-40°C)
Malaise
Lymphadenopathy (30-60% of patients) – slightly enlarged (1-2 cm) and tender nodes at several sites
Symptoms related to visceral involvement

At least one internal organ is involved in 90% of patients with DRESS.

Organs that can become involved in DRESS are:

Liver (60-80%)
Kidneys (10-30%)
Lungs
Heart (eosinophilic myocarditis, pericarditis)
Gastrointestinal tract (diarrhea, mucosal erosions, bleeding)
Pancreas (pancreatitis)
Thyroid (autoimmune thyroiditis, appearing often late, as a sequel of DRESS)
Brain (encephalitis, meningitis)
Muscle (myositis, increase in creatine kinase)
Peripheral nerves (polyneuritis)
Eye (uveitis)

References

• http://www.blackdoginstitute.org.au/docs/Lamotrigi

• https://www.nlm.nih.gov/medlineplus/druginfo/meds/

• http://psycheducation.org/treatment/mood-stabilize

• http://www.uptodate.com/contents/drug-reaction-wit

Last updated:
Time spent: QID:865
2023-2-12

1411 of 1943
Which one of the following medications can cause tachycardia?

A. Propranolol.

B. Verapamil.

C. Diltiazem.

D. Nifedipine.

E. Metoprolol.

Incorrect. Correct answer is D


45% answered correctly

Explanation:

Correct Answer Is D

Propranolol and metoprolol are beta blockers. Beta blockers, by inhibition of cardiac conductive system, result in bradycardia.

Verapamil and diltiazem are non-dihydropyridine calcium channel blockers (CCBs). Non-dihydropyridine CCBs has less effect on peripheral
arteries and more on cardiac conductive system. They can cause myocardial depression and bradycardia.

Nifedipine is a dihydropyridine CCB. This group of CCBs has no clinically significant effect on the heart (i.e. myocardial depression and
bradycardia), but significant effect on vessel wall and vasodilation. Vasodilation results in hypotension (the therapeutic effect). Excess
vasodilation and hypotension, seen in overdose with dihydropyridine CCBs, can be followed by compensatory tachycardia.

References

• Therapeutic Guidelines – Cardiovascular; available from: http://tg.org.au

Last updated:
Time spent: QID:922
2023-2-12

1412 of 1943
A 54-year-old man presents to your GP clinic with complaints of light-headedness and palpitation for 3 days. He is hypertensive and has been
on nifedipine and hydrochlorothiazide for the past 5 years. Recently, he was diagnosed with generalized anxiety disorder and started on
fluoxetine. On examination, he has a blood pressure of 90/60 mmHg, heart rate of 110 bpm, respiratory rate of 14 breaths per minute, and
temperature of 37.5°C. The rest of the exam is inconclusive. An ECG is obtained, which shows sinus tachycardia with no other arrhythmias.
Which one of the following is the most appropriate next step in management?

A. Stop nifedipine.

B. Stop fluoxetine.

C. Stop both.

D. Reduce fluoxetine.

E. Reduce nifedipine.

Incorrect. Correct answer is E


45% answered correctly

Explanation:

Correct Answer Is E

The exam findings are hypotension and tachycardia which can be justified by increased plasma levels of nifedipine. Nifedipine is
dihydropyridine calcium channel blocker (CCB). Dihydropyridine CCBs reduce the peripheral vascular resistance and by this decrease blood
pressure. These drugs generally do not have clinically significant cardiac effects such as suppression of myocardium or cardiac conductive
system.

SSRIs, such as fluoxetibe, can inhibit hepatic metabolism of CCBs and result in increased plasma levels of nifedipine leading to hypotension
and compensatory tachycardia as seen in this patient.

Other adverse effects of CCBs such as headache, flushing, ankle edema, constipation, etc may be seen or worsen in this situation as well.

In such circumstances the dose of CCB should be reduced, or the drug temporarily stopped if the symptoms, particularly cardiovascular
symptoms are profound.

References

• Drug Interactions in Psychiatry – Lippincott Williams & Wilkins – 3rd Edition – pages 105-106

• Therapeutic Guidelines – Cardiovascular; available from http://tg.org.au

Last updated:
Time spent: QID:955
2023-2-12

1413 of 1943
A 24-year-old woman has been started on an antipsychotic drug due to poorly-controlled schizophrenia 3 weeks ago. She has now presented
with complaint of palpitations. Which one of the following drug is more likely to have been prescribed for her?

A. Clozapine.

B. Mirtazapine.

C. Olanzapine.

D. Quetiapine.

E. Venlafaxine.

Correct
45% answered correctly

Explanation:

Correct Answer Is A

Clozapine is an effective antipsychotic for treatment of resistant schizophrenia, but is associated with serious adverse effects including
sedation, postural hypotension, hypersalivation, severe constipation, dyslipidemia, myoclonus and epileptic seizures. Excessive weight gain
and glucose intolerance may occur, precipitating type 2 diabetes.

The two serious adverse effects of neutropenia (2-3%), agranulocytosis (1%) are of significant concern as well. Tachycardia is another
common side effect of clozapine, observed in 25% of patients. Arrhythmias may also occur. Moreover, a minority of clozapine-treated patients
experience ECG changes similar to those seen with other antipsychotic drugs, including ST segment depression and flattening or inversion of
T-waves, which normalizes after discontinuation of clozapine.

Of the given options, clozapine can be associated with tachycardia and palpitations as an adverse effect.

NOTE - Myocarditis is a reported serious adverse effect of this drug which necessitates application of close monitoring protocols for patients
on treatment with this drug.

References

• https://www.mja.com.au/journal/2009/190/4/clozapin

• http://www.medsafe.govt.nz/profs/PUarticles/clozca

• Therapeutic Guidelines – Psychotropic; available from http://tg.org.au

Last updated:
Time spent: QID:965
2023-2-12

1414 of 1943
Which one of the following is the most important diagnostic test to consider for a patient on clozapine who has developed palpitation?

A. Troponin.

B. Echocardiography.

C. Holter monitoring.

D. Fool blood count.

E. Clozapine level.

Correct
45% answered correctly

Explanation:

Correct Answer Is A

Clozapine is an effective antipsychotic for treatment of resistant schizophrenia, but is associated with serious adverse effects including
sedation, postural hypotension, hypersalivation, severe constipation, dyslipidemia, myoclonus and epileptic seizures. Excessive weight gain
and glucose intolerance aand eventually type 2 diabetes mellitus can occur.

Neutropenia (2-3%) and agranulocytosis (1%) are other adverse effects. All patients taking clozapine should be registered in an approved
clozapine monitoring service where ongoing monitoring primarily occurs for the detection of neutropenia and agranulocytosis

Tachycardia is another common side effect of clozapine, observed in 25% of patients. Arrhythmias may occur. Similar to other atypical
antipsychotics, ST segment depression or T-wave flattening may be seen on ECG.

A range of cardiac disorders have also been associate with the use of clozapine, the most serious being myocarditis, cardiomyopathy and
death. Myocarditis is most commonly observed early in treatment. Seriousness of such conditions requires engaging patients in a monitoring
protocol devised for this purpose. The monitoring protocol recommends:

Obtaining baseline troponin I or T, CRP, ECG and echocardiography

Weekly monitoring of CRP and troponin for the first 4 weeks of treatment

During the first 4 weeks, vital signs must be measured and direct enquiry regarding symptoms performed at least every alternate day
while the patient is an inpatient, and weekly if the patient has been transferred to an outpatient clinic

In the presence of relevant symptoms, an abnormally increased heart rate, or raised CRP (50 mg/L), it is recommended that troponin
and CRP be measured daily

If troponin levels are only slightly raised (less than twice the upper limit of normal) and CRP remains less than 100 mg/L, clozapine
may be continued

Discontinuation of clozapine and investigation by echocardiography is advised if either troponin is more than double the normal
maximum, or CRP is more than 100 mg/L

Routine monitoring for myocarditis up to day 28 is recommended, in comparison to the previous guidelines which extended
monitoring only to day 14.

With a high proportion of cases of myocarditis occurring during week 3, this recommendation as to active monitoring for myocarditis during
the first 4 weeks proposes that this regime will have sufficient sensitivity to pick up all symptomatic cases of myocarditis developing between
days 14 and 21.

In this patient with palpitations, troponin levels should be monitored along with CRP on a daily basis as the most appropriate next step in
management.

(Option B) Echocardiography is used for baseline assessment (prior to commencement of clozapine) and if there are symptoms suggestive of
heart failure.

(Option C) Holter monitoring is not routinely used for monitoring and assessment of clozapine-treated patients with tachycardia or other
cardiac complications.

(Option D) Full blood count is performed on a regular basis to monitor development of neutropenia and agranulocytosis.

1415 of 1943
(Option E) Clozapine level measurement is not indicated for management of adverse effects as there is no reliable cut-off level.

References

• https://www.mja.com.au/journal/2009/190/4/clozapin

• http://www0.health.nsw.gov.au/policies/pd/2012/pdf

Last updated:
Time spent: QID:966
2023-2-12

1416 of 1943
A 61-year-old man is brought to your clinic by his wife with complaint of sudden-onset tongue swelling. He is diabetic and is on treatment with
metformin and chlorpropramide, has hypercholesterolemia for which he takes simvastatin, and also suffers from hypertension that
is controlled with ramipril. Four weeks ago, he presented to his GP with complaints of fever and productive cough, diagnosed as acute
bronchitis, and was prescribed amoxicillin. On examination, his tongue is swollen. He has difficulty speaking and points to his tongue as the
cause of his inability to speak. On lung auscultation, wheezes are heard. No rash is noted. Which one of the following drugs can be the cause
of this presentation?

A. Metformin.

B. Simvastatin.

C. Amoxicillin.

D. Ramipril.

E. Chlorpropamide.

Incorrect. Correct answer is D


45% answered correctly

Explanation:

Correct Answer Is D

The clinical presentation of tongue swelling and respiratory tract involvement, in the absence of rash, is highly suggestive of angioedema
caused by other reasons than anaphylaxis.

Angiotensin converting enzyme (ACE) inhibitors are the leading cause of drug-induced angioedema. The reason is that this drug group is
widely prescribed. ACE inhibitor-induced angioedema most commonly affects the lips, tongue, face, and upper airway. Angioedema of the
pharynx, larynx, and subglottic area have also been reported. Early signs of laryngeal edema include hoarseness and inspiratory stridor, which
may progress to airway obstruction in up to 10% of cases. Rarely, fatalities due to massive tongue swelling and asphyxiation have been
reported.

Less often, intestines can be involved, presenting as acute abdominal pain with diarrhea or other gastrointestinal symptoms.

ACE inhibitor-related angioedema occurs most commonly in the first years of treatment but can occur after years of use. Patients may have
multiple episodes before the condition is recognized. Initial mild episodes may progress to severe life-threatening ones.

NOTE - Presence of rash and/or itching excludes the diagnosis of ACE inhibitor-induced angioedema. If present, other etiologies than ACE
inhibitors should be considered.

After an episode of ACE inhibitor-related angioedema, the patient must not take this class of drug again. The common practice is switching to
an angiotensin receptor blocker (ARB). ARBs can also induce angioedema but at a much lower rate and less severity than ACE inhibitors.

Of the current drugs the patient is on, ramipril is an ACE inhibitor and the most likely cause of this presentation.

(Option A) Metformin is not associated with such presentation, nor is chlorpropramide (a sulfonylurea).

(Option B) Statins (e.g. simvastatin) has not been shown to be associated with angioedema.

(Option C) Amoxicillin can cause a hypersensitivity reaction type I (anaphylaxis) presenting with angioedema, rash (wheels) and itching;
however, absence of rash and itch makes amoxicillin a less likely cause of this presentation.

(Option E) Chlorpropamide is a sulfonylurea used for treatment of diabetes. Sulfonylureas have been reported as rare cause of angioedema.

References

• http://www.racgp.org.au/afp/2013/december/ace-inhi

• http://www.ncbi.nlm.nih.gov/pmc/articles/PMC363840

• http://www.uptodate.com/contents/ace-inhibitor-ind

Last updated:
Time spent: QID:967
2023-2-12

1417 of 1943
A 60-year-old woman is prescribed sertraline for management of depression. Her medical history includes hypertension that is well-controlled
on nifedipine and atenolol. On review after 2 months, she feels better but her signs and symptoms of depression have not resolved
completely; therefore, the dose of sertraline is increased. After 10 days, she presents again complaining of palpitations. On examination, she
has a blood pressure of 100/78mmHg and heart rate of 110bpm. The rest of the examination is inconclusive. Which one of the following is
most likely to have led to this presentation?

A. Sertraline.

B. Nifedipine.

C. Interaction between sertraline and nifedipine.

D. Increased metabolism of nifedipine.

E. A new rhythm disturbance irrelevant to her current medications.

Incorrect. Correct answer is C


45% answered correctly

Explanation:

Correct Answer Is C

Nifedipine is a dihydropyridine calcium channel blocker (CCB). Dihydropyridine CCBs reduce the peripheral resistance and by this reduce the
blood pressure. These drugs generally do not have clinically significant cardiac effects. Although nifedipine can cause compensatory
tachycardia, the patient has been asymptomatic before the dose of sertraline was increased.

Tachycardia and palpitation are not expected adverse effects of SSRIs; However, SSRIs can inhibit hepatic metabolism of CCBs, and result in
increased plasma levels of nifedipine leading to hypotension and compensatory tachycardia as seen in this patient. Therefore, the most likely
explanation to the patient’s tachycardia and palpitation is an interaction between the two drugs.

(Option A) Sertraline alone does not cause tachycardia.

(Option B) Since tachycardia has been developed after increasing the dose of sertraline, it is unlikely that nifedipine alone has led to
tachycardia.

(Option D) As mentioned earlier, SSRIs result in decreased metabolism of nifedipine, and by this, increase of the plasam level and effects of
nifedipine.

(Option E) It is unlikely that a new rhythm disturbance is the underlying cause of tachycardia, while a known drug interaction exists and
explains the symptoms.

References

• Drug Interactions in Psychiatry – Lippincott Williams & Wilkins – 3rd Edition – pages 105-106

• Therapeutic Guidelines – Cardiovascular; available from http://tg.org.au

Last updated:
Time spent: QID:999
2023-2-12

1418 of 1943
In which one of the following conditions nitrates are contraindicated?

A. Chronic left ventricular failure.

B. Unstable angina pectoris.

C. Acute left ventricular failure.

D. Myocardial infarction.

E. Hypotension.

Incorrect. Correct answer is E


45% answered correctly

Explanation:

Correct Answer Is E

The following are the contraindications to nitrate use:

Pulse rate less than 50 bpm.


Pulse rate over 100 bpm.
Systolic blood pressure less than 90mmHg or more than 30 mmHg under the baseline
Right ventricular infarction.
Severe aortic stenosis.
Hypertrophic cardiomyopathy.
The patient has taken sildenafil in the past 24 hours or tadalafil in the past 5 days.

Of the given options, only hypotension is a contraindication to nitrate use. Other options are in fact indications for nitrates.

References

• http://circ.ahajournals.org/content/110/5/588.full

Last updated:
Time spent: QID:221
2023-2-12

1419 of 1943
A 25-year-old woman comes to your clinic concerned about a breast lump in her left breast. She says that the lump is mobile and tender to palpation. You find out that the lump is
benign and is due to hormonal changes. She is concerned about her risk of having breast cancer. Her mother died of breast cancer at 45 years of age, and one of her aunts died of
ovarian cancer. Which one of the following is not a risk indicator of familial breast-ovarian cancer in her?

A. Onset of the cancer > 50 years old.

B. Two first-degree or second-degree relatives on one side of family with ovarian or breast cancer.

C. Individuals with ovarian cancer.

D. Breast cancer in a male relative.

E. Bilateral or multifocal breast cancer.

Correct
45% answered correctly

Explanation:

Correct Answer Is A

Hereditary breast-ovarian cancer (HBOC) syndrome is mainly caused by a pathogenic mutation in either BRCA1 or BRCA2 genes. There result of this mutation is strong
predisposition for breast, ovarian cancer, prostatic cancer and pancreatic cancer.

The lifetime risk for these cancers in individuals with a pathogenic variant in BRCA1 or BRCA2 is:

40%-80% for breast cancer (~ 50%)


11%-40% for ovarian cancer
1%-10% for male breast cancer
Up to 39% for prostate cancer
1%-7% for pancreatic cancer

The following features in history, if present, are suggestive of an increased risk of familial breast -ovarian cancer syndrome:

Two first-degree or second-degree relatives on one side of the family with ovarian or breast cancer
Breast cancer occurring in an affected relative younger than 50 years
One or more relatives with two cancers (breast and ovarian cancer or two independent breast cancers)
Individuals with bilateral or multifocal breast cancer
Individuals with ovarian cancer (especially if <50 years)
Breast cancer in a male relative
Ashkenazi Jewish descent (doubled risk)

​of the options, cancer after the age of 50 in not a risk indicator for HBOC syndrome.

There are other less common hereditary syndromes associated with increased risk of breast/ovarian cancer:

Ataxia telangectasia - Ataxia-telangiectasia (AT) is an autosomal recessive disorder characterized by progressive cerebellar degeneration (ataxia), dilated blood vessels in
the eyes and skin (telangiectasia), immunodeficiency, chromosomal instability, increased sensitivity to ionizing radiation, and a predisposition to cancer, in particular
lymphoma and leukemia. Solid tumors associated with AT include cancers of the breast, stomach, ovary, and melanoma.

Cowden syndrome - Cowden syndrome (also known as multiple hamartoma syndrome) is an autosomal dominant condition characterized by multiple hamartomatous
tumors, mucocutaneous findings, and an increased risk of early onset breast, uterine, and non-medullary thyroid cancer. Up to 75% of women with Cowden syndrome have
benign breast conditions such as ductal hyperplasia, intraductal papillomatosis, adenosis, lobular atrophy, fibroadenomas, and fibrocystic changes, and breast cancer
develops in 25% to 50% percent of female carriers. Most breast cancers are diagnosed premenopausally.

Peutz-Jeghers syndrome - Peutz-Jeghers syndrome (PJS) is an autosomal dominant condition characterised by hamartomatous polyps in the gastrointestinal tract and
mucocutaneous melanin deposits in the buccal mucosa, lips, fingers, and toes. Affected patients are at increased risk for both gastrointestinal (small bowel, stomach,
colorectal, and pancreas) and extraintestinal cancers, including cancers of the lung, breast, uterus, and ovary.

Inherited CDH1 mutations and Hereditary Diffuse Gastric Cancer Syndrome (HDGC) - HDGC is an inherited form of diffuse type gastric cancer, a highly invasive tumor that
is characterized by late presentation and a poor prognosis. The syndrome is highly penetrant, with the lifetime risk of gastric cancer exceeding 80%. It is also associated
with development of lobular breast cancer in women, with a cumulative lifetime risk estimated to be as high as 60% for some families, and possibly colon cancer.

Hereditary Nonpolyposis Colorectal Cancer (HNPCC) - HNPCC, also called Lynch Syndrome, is a cancer susceptibility syndrome caused mostly by mutations in the
mismatch repair genes, MSH2, MLH1, MSH6, PMS1, and PMS2. Mutation carriers are at risk of colorectal and endometrial cancer and, less frequently, cancer of the ovaries,
stomach, small bowel, hepatobiliary tract, ureter, renal pelvis and brain. Although information is limited, some studies have suggested that breast cancer is a part of the
spectrum of HNPCC-associated tumors.

Mutations in PALB2 gene - Mutations in a gene known as partner and localizer of BRCA2 (PALB2) are associated with an increased risk of pancreatic cancer and also
appear to be associated with a 2- to 4-fold increased risk of breast cancer in women (roughly a 18 to 35 percent lifetime risk). Risks of male breast cancer are also elevated.

References

• http://canceraustralia.gov.au/sites/default/files/

• http://emedicine.medscape.com/article/1947145-over

• http://www.ncbi.nlm.nih.gov/books/NBK1247/

• https://www.nhmrc.gov.au/_files_nhmrc/file/your_he

• http://www.australianprescriber.com/magazine/34/2/

1420 of 1943
Last updated:
Time spent: QID:548
2023-2-12

1421 of 1943
Lithium is commonly used in treatment of bipolar disorder. Which one of the following is the least common side effect of lithium?

A. Renal impairment.

B. Hypothyroidism.

C. Hyperparathyroidism.

D. Weight gain.

E. Teratogenicity.

Incorrect. Correct answer is E


45% answered correctly

Explanation:

Correct Answer Is E

Lithium can potentially cause many acute and chronic adverse effects even if the plasma levels are within therapeutic range.

The most common acute adverse effects of lithium include:

Nausea
Tremor
Polyuria (related to nephrogenic diabetes insipidus) and thirst
Weight gain
Loose stools
Cognitive impairment (including apathy, decreased creativity, and changes in verbal learning, memory, and concentration)

Chronic (long-term) adverse effects include:

Renal involvement - Renal function is adversely affected by lithium. Lithium initially reduces the ability of the kidney to concentrate
urine, leading to dilute urine and polyuria (nephrogenic diabetes insipidus). In the long run, interstitial nephritis or even fibrosis can
follow. This process can lead to increased serum creatinine and, rarely, to progressive renal failure.

Thyroid involvement - the most common adverse effect of lithium on thyroid gland is hypothyroidism, but goiter and chronic
autoimmune thyroiditis have been reported as well. Hyperthyroidism is also possible. However, pre-existing hypothyroidism or
hypothyroidism caused by lithium is not a contraindication to lithium use. TSH should be monitored regularly. Once elevated,
treatment with T4 should be started, while lithium is continued.

NOTE - Renal and thyroid problems are the most common log-term adverse effects of lithium.

Parathyroid gland involvement - lithium can reslt in hyperparathyroidism and hypercalcemia. Elevated serum calcium level should be
followed by serum PTH measurement, and an endocrine consultant involved if elevated.

Cardiac involvement - rarely, lithium may cause cardiac arrhythmias in patients without pre-existing cardiac disease. Moreover, lithium
may lead to ECG abnormalities such as abnormalities of T wave or ST segment, findings consistent with sick sinus syndrome, or an
unmasked Brugada pattern.

Although lithium is generally considered as a teratogenic medication due to increased risk cardiac anomalies in a fetus (1:1000-2000
compared to 1:20000 in general population, the absolute risk is low (0.05%). Of the options, teratogenicity is the least common adverse effect.

References

• http://www.uptodate.com/contents/bipolar-disorder-

• https://www.drugs.com/sfx/lithium-side-effects.htm

• Therapeutic Guidelines – Psychotropics; available from http://tg.org.au

Last updated:
Time spent: QID:1007
2023-2-12

1422 of 1943
Which one of the following is not a contraindication to ACE inhibitor use?

A. Bilateral renal artery stenosis.

B. Pregnancy.

C. Cough.

D. Angioedema.

E. Previous allergic reaction to ACE inhibitors.

Incorrect. Correct answer is C


45% answered correctly

Explanation:

Correct Answer Is C

The following are absolute contraindications to ACE inhibitors:

History of angioedema regardless of cause (even if not due to ACE inhibitor)


Pregnancy (due to harm to fetus)
Bilateral renal artery stenosis.
Previous allergic reaction to ACE inhibitors

Relative contraindications are:

Aortic stenosis
Hypertrophic cardiomyopathy

A dry cough is a common adverse effect of ACE inhibitors and a main cause of non-compliance and abandonment of treatment; however,
neither pre-existing cough nor ACE inhibitor-induced cough is not a contraindication to their use.

References

• http://www.fpnotebook.com/cv/pharm/AcInhbtr.htm

Last updated:
Time spent: QID:1014
2023-2-12

1423 of 1943
Janet, 32 years old, has been prescribed sertraline for treatment of her major depression. She comes to you for follow-up, and admits to
taking ecstasy at occasions. She says that she take the pill because she wants to enjoy more from time to time. Which one of the following is
a correct advice to give her in this regard?

A. Sertraline and ecstasy are synergistic.

B. Ecstasy is contraindicated in patients on sertraline.

C. The combination can lead to irreversible psychosis.

D. Ecstasy decreases the therapeutic effect of sertraline.

E. Ecstasy and sertraline have no interactions. ​

Correct
45% answered correctly

Explanation:

Correct Answer Is A

Selective serotonin reuptake inhibitors (SSRIs) such as sertraline are first-line medications for treatment of depression. Serotonin toxicity
occurs when the amount of the serotonin rises beyond the therapeutic range. Concomitant use of certain drugs is associated with increased
risk of serotonin toxicity. These drugs often lead to a synergistic effect on serotonin, and increase the risk of serotonin toxicity.

Certain illicit drugs such as ecstasy, amphetamine, and methamphetamine are examples of the drugs that can result in increased
concentration of serotonin in the body.

3,4- methylenedioxy-N-methamphetamine (MDMA), also known as ecstasy, is a unique psychedelic amphetamine with stimulant, euphoriant
and empahtogenic/entactogenic effect. The effect on serotonin is larger than amphetamine or methamphetamine. MDMA causes increased
release of serotonin in the body and has a synergistic effect on SSRIs.

Although this synergistic effect associated with simultaneous use of ecstasy and SSRIs increases the risk of serotonin toxicity, ecstasy is not
contraindicated to SSRIs use, and vice versa. However, advice should be given against the concomitant use. Of antidepressants, only
monoamine oxidase inhibitors (MAO inhibitors) is a contraindication to ecstasy use.

Other contraindications to ecstasy use are use of drugs metabolized through the same liver enzyme as MDMA (i.e.CYP2D6) such as ritonivar
(protease inhibitor), codeine and other opiates and dextromethorphan (DXM) which is found in over-the counter cough medications.

NOTE - Studies have shown that combining ecstasy with SSRIs may reduce the effects of both ecstasy and the SSRI, making the treatment
with SSRIs less effective and prevents from desired effects of ecstasy to occur.

References

• https://www.mja.com.au/journal/2007/187/6/serotoni

• https://dancesafe.org/drug-information/mdma-contra

Last updated:
Time spent: QID:1119
2023-2-12

1424 of 1943
A 67-year-old man presents to your office complaining of a painful swollen right thigh. He has diabetes well controlled on metformin 500mg
12-hourly and hypertension for which he is taking losartan 25mg 12-hourly. He is also on simvastatin 20 mg/daily hypercholesterolemia. His
recent medical history is significant for atrial fibrillation (AF) under treatment with warfarin. One week ago, he was started on amiodarone
after he was diagnosed with ventricular tachycardia (VT). On physical examination, he has a blood pressure of 140/95mmHg, pulse rate of
98bpm and a temperature of 37.3°C. The right thigh is painful, swollen and slightly tender but not warm or red, and has a circumference 4 cm
greater than that of the left thigh. Which one of the following could be the most likely cause of this presentation?

A. Deep vein thrombosis (DVT).

B. Drug reaction.

C. Cellulitis.

D. A thigh hematoma.

E. Rhabdomyolysis.

Incorrect. Correct answer is D


45% answered correctly

Explanation:

Correct Answer Is D

A painful swollen thigh can be caused by a number of different conditions such as DVT, cellulitis and hematoma. This patient is on warfarin
for treatment of AF. Although not impossible, it is less likely that he develops DVT (which is treated with warfarin as well). This makes DVT
(option A) a less likely yet possible diagnosis.

Interaction between warfarin and amiodarone is well-known. Amiodarone results in decreased metabolization of warfarin through hepatic
pathways and leads to increased bleeding tendency. Considering that the patient has been recently started on amiodarone while on warfarin,
a hematoma can also be possibility and in fact the most likely one. Hematomas are a frequent result of increased bleeding tendency
occurring in over-anticoagulated patients.

Drug reaction (option B) is different from drug interaction. Drug reaction means an adverse reaction caused by a single drug while in drug
interaction the effect of one drug leads to unwanted or exaggerated response to another. While this scenario is very likely to have been
caused by the interaction between amiodarone and warfarin, resulting in increased untoward effects of warfarin, none of the drugs in the
medications he is taking are not likely to result in such presentation.

Cellulitis (option C) presents with a warm and red areas of swollen skin. Fever is often present. This patient has no fever and the thigh
swelling is not warm and red. This makes cellulitis a less likely diagnosis compared to hematoma.

Rhabdomyolysis (option E) is a serious condition caused by muscle fibers breakdown and release of muscle cell contents such as myoglobin
and potassium. Myoglobinuria results in deposition of myoglobin in kidneys and renal failure. Release of excess potassium from damaged
muscle cells leads to hyperkalemia and serious complications such as cardiac arrhythmias. Statins such as atorvastatin and simvastatin in
particular are metabolized by cytochrome P450-3A4 (CYP3A4), and amiodarone is a potent inhibitor of this cytochrome. Concomitant use of
amiodarone and statins can result in increased activity of statins and rises the likelihood of statin-related adverse effects such as muscular
pain, myopathy and rarely rhabdomyolysis. However, in the event of statin-induced rhabdomyolysis, a systemic presentation is expected.
Rhabdomyolysis never causes focal signs mentioned in the scenario.

References

• https://www.nzwcs.org.nz/images/publications/Pagan-Hunter_WPR_19-01.pdf

• https://www.racgp.org.au/download/documents/AFP/2010/July/201007tadros_warfain.pdf

• https://emedicine.medscape.com/article/214222-overview

Last updated:
Time spent: QID:1210
2023-2-12

1425 of 1943
A 52-year-old man presents to the emergency department with complaint of a swollen tongue after he had seafood 2 hours ago. His medical
history is remarkable for diabetes mellitus for which he is taking metformin 500md 8-hourly, hypertension controlled on ramipril and
indapamide, and hypercholesterolemia treated with atorvastatin 20mg daily. He mentions no itching. On examination, he has stable vital signs
with no respiratory or abdominal symptoms. Which one of the following could be the most likely cause of this presentation?

A. Anaphylaxis.

B. C1 esterase inhibitor deficiency.

C. Ramipril.

D. Atorvastatin.

E. Metformin.

Incorrect. Correct answer is C


45% answered correctly

Explanation:

Correct Answer Is C

The scenario represents a case of angioedema manifested as a tongue swelling and no other symptoms pointing towards allergy or
anaphylaxis.

Angioedema is localized deep dermis, subcutaneous, or submucosal swelling which is self-limiting and results from extravasation of fluid into
interstitial tissue (blood vessel leakage). Depending on the underlying cause, angioedema can occur in isolation, accompanied by urticarial, or
as a component of anaphylaxis.

Angioedema typically affects the gravity-independent areas with loose connective tissue such as the face, lips, tongues, throat, uvula, larynx,
extremities and genitalia.it also may affect bowel wall (most commonly the jejunum) with the clinical picture of colicky abdominal pain,
obstruction, or ascites. Angioedema of the bowel wall often presents as a diagnostic dilemma.

Angioedema can be clinically differentiated from other forms of edema by the following characteristic features:

Onset is in minutes to hours and resolution occurs spontaneously in hours to days.


Asymmetric distribution.
Tendency not to involve gravitationally-dependent areas of the body.
Involvement of the face, lips, larynx, and bowels.
Association of some forms of angioedema with other clinical features of allergic reaction or anaphylaxis.

Based on the pathophysiology, there are three main types of angioedema:

Mast cell-related angioedema – in mast cell-related angioedema, seen in allergic reactions to food or insect stings, there are often (not
always) other clinical features of mast cell-mediator release including urticarial, flushing, pruritus (generalized), bronchospasms, throat
tightness or obstruction, and/or hypotension. Mast cell-mediated angioedema often begins within minutes of exposure to the allergen, build
up over a few hours and resolve within 24-48 hours.

Histamine-related angioedema – angioedema might be histamine- dependent without clear evidence of mast cell degranulation. This is often
the cause in cases of idiopathic (spontaneous) angioedema. In this type, angioedema may occur in isolation or with urticaria, but is not
associated with respiratory or circulatory symptoms.

Bradykinin- induced angioedema – this type of angioedema is not associated with other manifestations of the other two groups such, i.e.
signs and symptoms of an allergic reaction such as urticaria, respiratory and/or circulatory symptoms, pruritus, etc. There is a more
prolonged time course. The sign and symptoms usually develop over 24-36 hours and resolve within 2 to 4 days. In this type of angioedema,
there is no definite relationship between the trigger and the onset of symptoms.

This man has presented with a swollen tongue as the sole manifestation of angioedema and the most likely diagnosis for him is bradykinin-
induced angioedema. Angiotensin converting enzyme inhibitors (ACIs) such as captopril, ramipril, lisinopril, enalapril, and perindopril are the
most common triggering factors for bradykinin-induced angioedema; therefore, the most likely cause to this presentation would be the ACEI
ramipril. Angiotensin receptor blockers (ARBs) such a losartan and valsartan are also capable of inducing such presentation, but the
association is not as strong.

(Option A) Although anaphylaxis can have angioedema as a manifestation, the absences of urticaria, circulatory/respiratory symptoms and
pruritus makes this diagnosis makes anaphylaxis (e.g. brought up by the seafood) a remote possibility.

(Option B) acquired C1esterase inhibitor (C1-INH) deficiency, also called acquired angioedema (AAE), is a rare syndrome presenting with
recurrent episodes of angioedema without urticaria. This syndrome is sometimes associated with B cell lymphoproliferative disorders. Like

1426 of 1943
ACEI-induced angioedema, manifestations include swelling of the face, tongue, upper respiratory tract, or bowel walls. C1-INH deficiency
presents similar to hereditary angioedema (HAA); however, unlike patients with HAA who are often young, patients with this disease are often
older than 40 years. Although C1-INH deficiency can be a diagnosis in this man, its rarity makes it a less likely possibility.

Statins such as atorvastatin (option D) and metformin (option E) are not associated with angioedema, especially the bradykinin-induced type.

References

• https://emedicine.medscape.com/article/135208-overview

• https://www.uptodate.com/contents/ace-inhibitor-induced-angioedema

• https://www.uptodate.com/contents/acquired-c1-inhibitor-deficiency-clinical-manifestations-epidemiology-pathogenesis-and-diagnosis

Last updated:
Time spent: QID:1215
2023-2-12

1427 of 1943
A 62-year-old man presents to your practice complaint of recent-onset shortness of breath which is brought up by physical activity. He is
diabetic and takes metformin and rosiglitazone for treatment and also has hypertension controlled on enalapril and metoprolol. Recently, he
has been started on amiodarone due to episodes of sustained ventricular tachycardia. Which one of the following medications is more likely
have caused this clinical picture?

A. Rosiglitazone.

B. Metformin.

C. Enalapril.

D. Metoprolol.

E. Amiodarone.

Correct
45% answered correctly

Explanation:

Correct Answer Is A

Of the options, rosiglitazone is more likely to have caused this clinical picture.

Rosiglitazone (Avandia®) is a thiazolidinedione and is prescribed as an oral antidiabetic agent and is used in management of type 2 diabetes
mellitus. The primary mechanism of action has been hypothesized to be through increasing insulin sensitivity.

Fluid retention may occur in patients on rosiglitazone. This may lead to or exacerbate heart failure in such patients, who often already have
risk factors in the setting of diabetes. Diuretic therapy may be necessary for treatment of fluid retention. Weight gain, probably due to fluid
overload has been observed during therapy.

It is estimated that 1-10% of patients on rosiglitazone develop adverse effects such as:

Edema
Hypertension
Heart failure
Myocardial ischemia
Diarrhea
Upper respiratory tract infection

Increased total cholesterol, LDH and HDL are other findings seen in >10% of patients on this medication.

While on rosiglitazone, patients should be monitored for signs and symptoms of fluid retention and heart failure, including rapid weight gain,
edema and dyspnea, and the drug should be discontinued if any deterioration is cardiac status occurs.

(Option B) Metformin is an anti-hyperglycemic agent with adverse effects including hypoglycemia and lactic acidosis, abdominal pain,
decreased appetite, muscle pain and cramps, somnolence, etc. Dyspnea is not a known adverse effect of this medication.

(Option C) Enalapril is an angiotensin converting enzyme inhibitor (ACEI). ACEI adverse effects include dry cough, hyperkalemia, fatigue,
dizziness, headache, and loss of taste. Dyspnea is not an adverse effect.

(Option D) Beta blockers such as metoprolol can exacerbate an existing heart failure and result in sign and symptoms such as dyspnea, fluid
retention and edema. This patient does not have heart failure and metoprolol is not likely to have resulted in this presentation.

(Option E) Amiodarone use is also associated with several adverse effects including pulmonary toxicity that can lead to dyspnea as a
presenting symptom. However, months to even years of use is required before the cumulative dose of amiodarone reaches toxic levels and
results in pulmonary toxicity. This patient has just recently been started on amiodarone and his dyspnea is unlikely to have been caused by it.

There are several different types of pulmonary toxicity caused by amiodarone. Interstitial pneumonitis is the most common; others include
organizing pneumonia, acute respiratory distress syndrome, and a solitary lung mass. A non-productive cough and dyspnea are often present
in 50% to 75% of patients with amiodarone-induced pulmonary toxicity. Pleuritic chest pain, weight loss, fever and malaise can also occur.

References

• Medscape - Rosiglitazone

• PubChem
1428 of 1943
Last updated:
Time spent: QID:1221
2023-2-12

1429 of 1943
Joan is 60-year old woman who is suffering migraine. The pain has been crushing and disabling despite medical therapy with maximum dose
of paracetamol. Seven hours ago, in a desperate attempt she took 20 500mg paracetamol tablets to make the pain go away. She is now in the
Emergency Department of the tertiary hospital you work in with complaints of right upper quadrant abdominal pain, nausea, and vomiting. She
weighs 65 kilograms. Which one of the following is the next best step in management?

A. Give her intravenous N-acetyl cysteine immediately.

B. Obtain a serum paracetamol level and give her N-acetyl cysteine.

C. Do immediate gastric lavage and give her activated charcoal.

D. Check liver function tests prior to decision making on giving N-acetyl cysteine.

E. Refer the patient to toxicology registrar.

Incorrect. Correct answer is B


45% answered correctly

Explanation:

Correct Answer Is B

Before making a decision as to treatment of potential paracetamol poisoning, it should be determined whether the ingested dose can be toxic.
The following table from the Medical Journal of Australia summarizes the doses associated with hepatic toxicity:

Adults and children >6 years Children (0-6 years)*


Acute single dose ingestion Acute single ingestion >200 mg/kg >200 mg/kg over a period of 8 hours
or 10 g (whichever is lower) over a
period of 8 hours
Repeated supratherapeutic ingestion >200 mg/kg or 10 g (whichever is >200 mg/kg over a single 24-hour
lower) over a single 24-hour period period
>150 mg/kg or 6 g (whichever is >150 mg/kg per 24-hour period in the
lower) per 24-hour periods in the preceding 48 hours
preceding 48 hours
>100 mg/kg or 4 g (whichever is >100 mg/kg per 24-hour period for
lower) per 24-hour periods for more more than 48 hours
than 48 hours in those who also have
symptoms indicating possible hepatic
injury (e.g. abdominal pain, nausea,
vomiting)
*for obese children, the body weight used for calculation should be an ideal body weight

Joan has ingested at least 10 g (500mgX20) paracetamol in less than 8 hours that puts her in a significant risk of toxicity and liver damage.
Management of potential paracetamol toxicity depends on the time of presentation and is summarized in the following table (from Medical
Journal of Australia):

Since the presentation is within 8 hours, the next best step is obtaining a blood sample for serum paracetamol level and starting N-acetyl
cysteine (NAC) immediately because it is unlikely that Joan’s serum paracetamol levels is available within 8 hours. NAC can be safely stopped
if the paracetamol level is within the safe range and continued if in the toxic range.

(Option A) Administration of NAC should not be delayed more than 8 hours of ingestion. Ingestion has occurred 7 hours ago and it is very
unlikely that the results of the serum paracetamol level is available within 1 hour. Therefore, NAC should be started but taking blood samples
for paracetamol level should be obtained first.

(Option C) Gastric lavage is indicated if the presentation is within the first hour of toxic ingestion. Active charcoal is indicated if the
presentation is within the first 2 hours of ingestion.

(Option D) Although assessing the potential damage to the liver demands liver function testing, this step is not of priority at this stage and can
be considered once other appropriate measures are taken.

(Option E) Referring the patient to toxicology registrar is incorrect. Action should be taken immediately now. Seeking expert advice can come
later in the course of treatment.
1430 of 1943
References

• MJA - Summary statement: new guidelines for the management of paracetamol poisoning in Australia and New Zealand

Last updated:
Time spent: QID:1251
2023-2-12

1431 of 1943
A 72-year-old man is brought to the emergency department after he collapsed while walking back home from shopping. After administration
of oxygen, an ECG strip is obtained emergently which is shown in the following photograph. He lost her wife 12 months ago and is currently
on sertraline 100 mg/day due to depression as a result of complicated grief. One week ago, he was prescribed azithromycin for treatment of
atypical pneumonia. His other medications include aspirin 80 mg/day for treatment of ischemic heart disease (IHD), atorvastatin 20 mg/day
for hyperlipidemia and multivitamin. Which one of the following is most likely to have lead in such presentation?

A. Interaction between aspirin and azithromycin.

B. Sertraline.

C. Azithromycin and sertraline.

D. Azithromycin.

E. Ischemic heart disease.

Incorrect. Correct answer is C


45% answered correctly

Explanation:

Correct Answer Is C

The ECG is characteristic of Torsades de pointes (TdP). TdP is a specific form of polymorphic ventricular tachycardia (PVT) occurring in the
context of QT prolongation. It has a characteristic morphology in which the QRS complexes “twist” around the isoelectric line. For TdP to be
the diagnosis, the patient has to have evidence of both PVT and QT prolongation.

QT interval prolongation (QTc>500 ms) is the essential predisposing factors for TdP. QT interval prolongation is cause by a variety of
conditions including:

Electrolyte Disturbances, in particular hypokalemia, hypomagnesemia and more rarely hypocalcemia


Concomitant use of more than one drug that prolongs the QT interval
Congenital Long QT Syndrome
Cardiac Disease such as congestive heart failure, ventricular hypertrophy, recent conversion from AF
Thyroid Disease - more common with hypothyroidism and usually normalizes with treatment
Drugs

NOTE – Female gender and age 65 or older are risk factors for QT interval prolongation.

Of the above list, drugs play an important role in a marked number of patients. There is a long list of drugs with potential capability of
prolongation of the QT interval as the underlying cause for TdP. Some examples are:

Antibiotics:

Macrolides (azithromycin, clarithromycin, erythromycin)


Moxifloxacin.
Ketoconazole.
Fluconazole.

Antidepressants:

SSRIs (e.g. citalopram, escitalopram, sertraline)


TCAs (e.g. amitriptyline, imipramine, clomipramine, doxepin)

Antipsychotics

1432 of 1943
First-generation anti-psychotics (e.g. haloperidol, chlorpromazine)
Atypical antipsychotics (e.g. risperidone, pimozide, quetiapine, clozapine)

Anti-arrhythmics:

Sotalol
Quinidine
Flecainide
Amiodarone

Anti-emetics

Ondansetron
Domperidone

Other:

Methadone
Protein kinase inhibitors e.g. sunitinib
Lofepramine
Some antimalarials
Some antiretrovirals
Telaprevir
Boceprevir

NOTE - Often a combination of two or more of these drugs are necessary for QT interval prolongation.

Both sertraline (an SSRI) and azithromycin (a macrolide) can cause QT interval prolongation and TdP. This patient, however, has been on
sertraline for months without any complication and just has become problematic after azithromycin was added. This makes the concomitant
use and accumulative effect of sertraline and azithromycin the most likely cause for this presentation.

Theoretically, azithromycin can result in QT interval prolongation, but as mentioned earlier often more than one drug with such effect in
required for this. Therefore, azithromycin alone (option D) is less likely to have caused this problem. This holds through about sertraline
(option B). Moreover, this patient has been on sertraline for a rather long time with no complaints.

QT interval prolongation is not a known effect of aspirin and aspirin does not increase the effect of azithromycin on QT interval prolongation
(option A).

Although heart disease such as IHD (option E) can be a cause of QT interval prolongation and TdP, the presence of concomitant use of a
macrolide and an SSRI in the history, makes IHD a less likely trigger for this presentation than the drug combination.

References

• https://www.ncbi.nlm.nih.gov/pmc/articles/PMC4110870/

• http://www.ggcprescribing.org.uk/media/uploads/ps_extra/pse_21.pdf

Last updated:
Time spent: QID:1289
2023-2-12

1433 of 1943
A 69-year-old man presents to the emergency department with a painful swollen left thigh. His current medications include metformin 500 mg
8-hourly for diabetes, valsartan 80 mg daily for hypertension, atorvastatin 10mg daily for hypercholesterolemia, and warfarin 5mg daily for
chronic atrial fibrillation. One week earlier, he was started on amiodarone after he was diagnosed with ventricular tachycardia (VT). On
physical examination, he has a blood pressure of 138/87mmHg, pulse rate of 82 pm and a temperature of 36.6°C. The left thigh is painful,
swollen and slightly tender but not warm or red. The affected leg circumference is 5 cm greater than the unaffected one. Which one of the
following options is the most appropriate next step in management?

A. Duplex Doppler ultrasound of the left leg.

B. Blood culture.

C. INR.

D. Commencement of antibiotics.

E. Decrease the dose of warfarin.

Incorrect. Correct answer is C


45% answered correctly

Explanation:

Correct Answer Is C

A painful swollen thigh can be caused by a number of different conditions such as deep vein thrombosis (DVT), cellulitis and hematoma. This
patient is on warfarin for treatment of AF.

Interaction between warfarin and amiodarone is well-known. Amiodarone results in decreased metabolization of warfarin through hepatic
pathways and leads to increased bleeding tendency. Considering that the patient has been recently started on amiodarone while on warfarin,
a hematoma is the most likely diagnosis. Given the normal temperature, cellulitis is a less likely diagnosis and while the patient is on warfarin
it is less likely but not impossible to develop DVT.

With hematoma being on the top of the differential diagnoses, measurement of INR would be the most appropriate initial step to take in
management of this patient. An increased INR beyond the therapeutic range of 2-3 makes the diagnosis more certain; however, it is not
uncommon that bleeding develops in the presence of therapeutic or even subtherapeuthic INR ranges.

(Option A) Duplex Doppler ultrasound for assessment of possible DVT may be considered after hematoma, as the most likely diagnosis, is
excluded, or even for assessment of hematoma.

(Options B and D) Blood culture and antibiotics are measures for suspected cellulitis. Absence of fever, warmth and erythema make cellulitis
a far less likely diagnosis and these measures not appropriate steps at least for now.

(Option E) Dose reduction or cessation of warfarin may be considered if the INR is found to be above the therapeutic range.

References

• https://www.nzwcs.org.nz/images/publications/Pagan-Hunter_WPR_19-01.pdf

• https://www.racgp.org.au/download/documents/AFP/2010/July/201007tadros_warfain.pdf

• https://emedicine.medscape.com/article/214222-overview

Last updated:
Time spent: QID:1316
2023-2-12

1434 of 1943
The laboratory test results of a 55-year-old man on polypharmacy shows a total cholesterol level of 4.5mmol/L and HDL of 0.8mmol/L. Which
one of the following medications can be the cause of this lipid profile?

A. Allopurinol.

B. Aspirin.

C. Hydrochlorothiazide.

D. NSAIDs.

E. Calcium channel blockers.

Incorrect. Correct answer is C


45% answered correctly

Explanation:

Correct Answer Is C

This patient has an elevated total cholesterol level of 4.5 mmol/L (normal: <4mmol/L) and a decreased HDL level of 0.8mmol/L (normal:
men:1-1.3mmol/L, women: 1.3-1.5mmol/L). Of the options, only hydrochlorothiazide in doses greater than 25mg/day or prolonged use is
capable of producing such lipid profile.

Hydrochlorothiazide has not shown to adversely affect lipid profile in doses of 6.25-12mg/day. In dose of 25mg/day, it increases TG levels but
not total cholesterol or HDL levels. With doses more than 25mg/day, hydrochlorothiazide is associated with an increase in total cholesterol
and TG levels, and decreased HDL levels especially in diabetic patients.

(Option A) Allopurinol is a uric acid-lowering agent. Studies suggest that allopurinol is associated with an increase in TG, total cholesterol and
LDL and HDL levels. HDL level was shown to increase later on after 3 months of therapy. With decreased HDL in this patient, allopurinol would
be a less likely cause to this lipid profile.

(Options B and D) Salicylates such as aspirin or NSAIDs have not shown to have an adverse effect on lipid profile.

(Option E) Calcium channel blockers appear to have no effect on lipid profile.

TOPIC REVIEW

Effects of some cardiovascular drugs on lipid profile:

Medication Effects on lipid profile


Non-selective beta blockers (e.g. propranolol) ↑TG, ↓LDL
Selective beta1 blockers (e.g. atenolol, metoprolol) ↑TG
Hydrochlorothiazide ↑TG, ↑total cholesterol, ↓ HDL in diabetics or in longer-term use
Indapamide ↑LDL
Chlorthalidone ↑LDL
Orlistat ↓HDL
Bile acid sequestrants ↑TG
Omega 3 ↑ LDL in patients with severe hypertriglyceridemia
Fibrates (gemfibrozil) ↑LDL in patient with hypertriglyceridemia
Allopurinol ↑TG, ↑total cholesterol, ↑LDL, ↑HDL
ACE inhibitors Inconsistent: ↓TG, ↓total cholesterol, ↓LDL, ↑HDL
Calcium channel blockers No effect

References

• Journal of the American Society Nephrology - Thiazide Diuretics in the Treatment of Hypertension: An Update

• PubMed - Dyslipidemia Induced by Drugs Used for the Prevention and Treatment of Vascular Diseases

Last updated:
Time spent: QID:1405
2023-2-12

1435 of 1943
Which one of the following is the most common pathologic ECG abnormality in intoxication with amitriptyline?

A. Ventricular tachycardia.

B. Widening of QRS complexes.

C. Prolongation of QT interval.

D. Prolongation of PR interval.

E. Premature ventricular contractions.

Incorrect. Correct answer is B


45% answered correctly

Explanation:

Correct Answer Is B

Widening of QRS complexes is probably the most prominent pathological ECG abnormality in patients intoxicated with tricyclic
antidepressants. If untreated, this can lead to ventricular tachycardia (VT) and ventricular fibrillation (VF). Other possible abnormalities
include:

Prolongation of QT (option C) and/or PR intervals (option D). The latter can cause first-degree AV block if PR>200 milliseconds.
VT (option A) and VF are seen in approximately 4% of patients and are the most common cause of death in TCA intoxication.
Sinus tachycardia due to anticholinergic effect of TCAs (probably the most common benign ECG finding in TCA intoxication).

NOTE - QRS prolongation >100ms is associated with a 30% chance of seizure. With QRS>160ms, there is a 50% risk of serious
arrhythmias.

Premature ventricular contractions (option E) are the most common ECG abnormality seen in patients with digoxin intoxication.

References

• UpToDate - Tricyclic antidepressant poisoning

Last updated:
Time spent: QID:228
2023-2-12

1436 of 1943
Which one of the following malignancies is most likely to have hypokalemia as an associated feature?

A. Colon cancer.

B. Breast cancer.

C. Lung cancer.

D. Renal cell carcinoma.

E. Cancer of the pancreas.

Incorrect. Correct answer is C


45% answered correctly

Explanation:

Correct Answer Is C

Paraneoplastic syndromes are rare disorders that are triggered by an altered immune system response to a neoplasm and are defined as
clinical syndromes involving non-metastatic systemic effects that accompany malignant disease.

These syndromes are collections of symptoms that result from substances produce remote from the tumor itself.

Manifestations of symptoms can be related to either of the following:

Endocrine
Renal
Neuromuscular
Musculoskeletal
Cutaneous
Hematologic
Gastrointestinal
Miscellaneous in nature

Paraneoplastic syndromes may be the first or most prominent manifestation of a cancer. When a patient without a known cancer presents
with a paraneoplastic syndrome, cancer should be considered and promptly investigated.

Hypokalemic nephropathy, which is characterized by urinary potassium leakage of more than 20 mEq per 24 hours, may develop in patients
with tumors that secrete adrenocorticotropic hormone (ACTH) or ACTH-like substances. It occurs in 50% of individuals with ACTH-secreting
tumors of the lung (i.e. small cell cancer)

Cushing syndrome is another paraneoplastic syndrome (endocrine syndrome) that may be caused by ACTH or ACTH-like substances
produced by small cell carcinoma of the lung.

References

• http://emedicine.medscape.com/article/280744-clini

Last updated:
Time spent: QID:650
2023-2-12

1437 of 1943
A 73-year-old woman presents to the Emergency Department with vomiting, abdominal pain, and abdominal distention. She has congestive
heart failure (CHF) in the setting of long-standing hypertension. She had been taking carvedilol, ramipril, and atorvastatin until 10 days ago
when she had digoxin and hydrochlorothiazide added to her medications for a tighter control of her rather poorly controlled hypertension and
CHF. On examination, she has a blood pressure of 130/85 mmHg, an irregular pulse of 110 bpm in rate, respiratory rate of 22 breaths per
minute, and temperature of 37.5°C. Her abdomen is distended but not tender. There is also no guarding or rigidity. Bowel sounds are absent.
Which one of the following is the most likely cause of her abdominal distention?

A. Hypokalemia.

B. Digoxin toxicity.

C. Hypocalcemia.

D. Mesenteric ischemia.

E. Hyperkalemia.

Correct
45% answered correctly

Explanation:

Correct Answer Is A

This patient has the clinical picture of paralytic ileus. As the name implies, the condition is associated with paralysis of colon wall leading the
clinical picture of bowel obstruction in the absence of a mechanical blockage. Hypokalemia is one the causes of paralytic ileus and can be the
most likely explanation for this clinical scenario. The problem has started after introduction of hydrochlorothiazide and digoxin to her
medications. Thiazide diuretics are notorious for causing hypokalemia. Other metabolic derangements induced by thiazide diuretics are
hyponatremia, increased blood glucose, increase uric acid, and hypercalcemia.

Hypercalcemia often presents with symptoms such as loss of appetite, nausea and vomiting, constipation and abdominal pain, increased
thirst and frequent urination, fatigue, weakness, muscular pain, confusion and disorientation, headaches, and depression. Although both
hypokalemia and hypercalcemia share constipation as a feature, hypercalcemia does not cause a clinical picture consistent with bowel
obstruction.

Thiazide-induced hypokalemia in this patient may predispose her to digoxin toxicity (option B), and digoxin toxicity results in hyperkalemia
(option E); however, gastrointestinal manifestations of digoxin toxicity are anorexia, nausea, vomiting, abdominal pain, and diarrhea.
Abdominal distention is not a feature of digoxin toxicity or the consequent hyperkalemia.

Hypocalcemia (option C) presents with muscle spasms, numbness and tingling in the hands, feet, and face, and in more severe case, central
nervous system problems such as hallucinations. There are no clues in the history suggestive of hypocalcemia as the cause, nor is there an
identifiable etiology for that.

(Option D) The rapid irregular pulse in this patient indicated atrial fibrillation (AF). AF predisposes to mesenteric ischemia; however, acute
mesenteric ischemia presents with abdominal pain and bloody diarrhea, especially after meals. Chronic mesenteric ischemia has
postprandial periumbilical and/or epigastric pain, fear of eating, and weight loss as typical symptoms. Less common features include nausea,
vomiting, diarrhea, constipation, and flatulence. Although some of this patient’s symptoms are seen in chronic mesenteric ischemia as well,
abdominal distention goes against this diagnosis.

NOTE – Carvedilol is a beta blocker. Beta blockers do not commonly cause hypokalemia. In fact, they more tend to cause hyperkalemia
by impairing potassium transport into the cells.

References

• Medscape – Hypokalemia

• Medscape – Digital Toxicity

• AHA – Thiazides Effects and Adverse Effects

Last updated:
Time spent: QID:1504
2023-2-12

1438 of 1943
A 72-year-old woman presents to the Emergency Department complaining of nausea and vomiting. Apparently, she visited the on-call GP a
few days earlier and was prescribed clarithromycin for a respiratory tract infection. Past medical history of significance includes chronic
obstructive pulmonary disease (COPD) for which she takes high-dose oral corticosteroid, tiotropium and oral theophylline, ischemic heart
disease for which she takes ramipril, amlodipine and indapamide, and chronic kidney disease. On examination, she has a blood pressure of
135/70 mmHg and heart rate of 105 bpm that is irregularly irregular. There is also bilateral basal lung crackles consistent with mild left
ventricular failure.

Her laboratory results are as follows:

Hemoglobin: 11.4 mg/dl (11.5-16.5)


WBC: 7 x 109/L (4x109 – 11x109)
Platelet: 197 x 109/L(150x109 – 400x109)
Serum sodium: 128 mmol/L (135 - 46)
Serum potassium: 4.0 mmol/L (3.5 – 5)
Creatinine: 178 μmol/L (79 – 118 μmol/L)

Which one of the following medication is most likely to have resulted in her presentation?

A. Amlodipine.

B. Indapamide.

C. Ramipril.

D. Theophylline.

E. Tiotropium.

Incorrect. Correct answer is B


45% answered correctly

Explanation:

Correct Answer Is B

Multiple medications, especially in a fragile elderly, is a common problem in general practice. Adverse effects and/or drug interactions can
leave the patient with problems that in fact can outweigh the benefits of medications.

This woman has some abnormal laboratory findings. One is an elevated serum creatinine level which can be justified by the chronic kidney
disease. Another is hyponatremia. Although both uremia in chronic kidney disease and hyponatremia can cause nausea and vomiting,
considering the level of creatinine and the fact that she has chronic kidney disease, hyponatremia is more likely to have resulted in such
presentation.

Hyponatremia is decrease in serum sodium concentration. Common causes of hyponatremia are:

Diuretic use
Diarrhea
Heart failure
Liver disease
Renal disease
Syndrome of inappropriate ADH secretion (SIADH)

Signs and symptoms of hyponatremia include nausea and vomiting, headache, short-term memory loss, confusion, lethargy, fatigue, loss of
appetite, irritability, muscle weakness, spasms or cramps, seizures, and decreased consciousness or coma.

This patient has congestive heart failure, kidney disease. These can result in hyponatremia in isolation but none are an option. COPD can
cause SIADH, but it is less likely to be the underlying cause compared to more probable scenarios.

This woman is also taking amlodipine (option A), indapamide, ramipril (option C), and also theophylline all of which can result in
hyponatremia.

Amlodipine is a calcium channel blocker. Even low doses of this drug class can cause a marked increase in urinary sodium by up to four fold.
Another mechanism of hyponatremia may be via direct action on the renal tubule with resultant increased sodium excretion and inhibition of
renal sodium reabsorption.

Ramipril is an angiotensin converting enzyme inhibitor. Syndrome of Inappropriate Anti-diuretic Hormone (SIADH) and subsequent
hyponatremia has been observed in some patients treated with ramipril. It is recommended that serum sodium levels be monitored regularly

1439 of 1943
in the elderly who are taking this drug class.

Theophylline has sometime been associated with hyponatremia. A thiazide-like action of the drug on the stimulation of SIADH could be the
underlying mechanism.

Indapamide is a thiazide-like diuretic. The mechanism of action is excretion of sodium and water via the loop of Henle.

Of these medications, however, indapamide is very well-known for hyponatremia and more likely to have caused hyponatremia among others.
A mentioned before, diuretic use is a very common cause of hyponatremia.

For this patient, it is very important to stop indapamide and monitor the response very closely. Gentle fluid restriction and daily weighing is
also recommended.

Tiotropium (Spiriva) (option E) is a member of the drug class anticholinergic bronchodilators which is used in inhalation form mostly in
patients with COPD. Inhaled tiotropium is not a known cause of hyponatremia.

References

• Therapeutic Guidelines

Last updated:
Time spent: QID:1568
2023-2-12

1440 of 1943
Which one of the following is the most common adverse effect of aripiprazole?

A. Light-headedness.

B. Dizziness.

C. Bloating.

D. Sleepiness.

E. Tiredness.

Correct
45% answered correctly

Explanation:

Correct Answer Is A

Aripiprazole is an atypical antipsychotic used in treatment of schizophrenia, bipolar disorder, and depression with psychotic features.
Reported adverse effects of aripiprazole in order of prevalence include:

>10%

Weight gain (8-30%)


Headache (27%)
Agitation (19%)
Insomnia (18%)
Anxiety (17%)
Nausea and vomiting (11-15%)
Akathisia (10-13%)
Lightheadedness (11%)
Constipation (10-11%)

1-10%

Dizziness (10%)
Dyspepsia (9%)
Somnolence (5-8%)
Fatigue (6%)
Restlessness (6%)
Tremor (6%)
Dry mouth/xerostomia (5%)
Extrapyramidal disorder (5%)
Orthostatic hypotension (1-5%)
Musculoskeletal stiffness (4%)
Abdominal discomfort (3%)
Blurred vision (3%)
Cough (3%)
Pain (3%)
Myalgia (2%)
Rash
Rhinitis

<1%

Altered mental status


Autonomic instability
Dysphagia
Hyperpyrexia
Muscle rigidity
Neuroleptic malignant syndrome (NMS)
Seizure
Tardive dyskinesia

1441 of 1943
Of the options, light-headedness is the most prevalence one (11%) followed by dizziness (10%) (option B). Light-headedness is defined as a
pre-faint state while dizziness is used by patients to imply or describe a sense of movement or even vertigo which is absent in light-
headedness.

NOTE – Aripiprazole is a non-sedative antipsychotic. Insomnia is commonly reported adverse effect (18%), sleepiness (somnolence) has
a prevalence of 5-8%.

References

• Medscape – Drugs Reference – Aripiprazole

Last updated:
Time spent: QID:1652
2023-2-12

1442 of 1943
Janet, 55 years of age, has presented to your GP clinic with complaint of recent-onset hair loss for the past few weeks. The hair loss is
generalized and not local to a specific area. She has bipolar disorder and had been on lithium for a long time until recently when her
psychiatrist increased the dose and added carbamazepine because she had an acute manic attack despite being on lithium. She also has
migraine for which she takes sumatriptan. Her other medications are aspirin for coronary artery disease and prophylactic dose of enoxaparin
after she had a deep vein thrombosis of her left lower leg. Which one of the following medications is more likely to have caused the hair loss?

A. Aspirin.

B. Lithium.

C. Carbamazepine.

D. Enoxaparin.

E. Sumatriptan.

Incorrect. Correct answer is B


45% answered correctly

Explanation:

Correct Answer Is B

Hair loss and alopecia from medications is characteristically a diffuse nonscarring hair loss that occurs within days to weeks of starting a
new medication or changing the dose. The development and severity of the hair loss depends on the medication and individual predisposition
of the patient. While some medication cause hair loss in almost all patients (e.g., chemotherapy agents), others may affect only some
patients.

There are two types of drug-induced hair loss:

Anagen effluvium – the shedding of actively growing hairs which is usually caused by chemotherapy drugs, or rarely with gold, colchicine, or
poisoning with arsenic, bismuth, thaliumm or boric acid.

Telogen effluvium – the shedding of resting, or bulb hairs

Telogen effluvium is the mechanism of virtually all other medication-induced hair loss. The list of possible drug causes is very long and
includes:

Anti-coagulants: heparin, warfarin and possibly newer anticoagulants such as rivaroxaban, dabigatran and apixaban

Anti-hypertensives: beta-blockers, ACE inhibitors

hormones : oral contraceptives pills (during/after/changing), hormone replacement therapy, androgens

Anticonvulsants — valproic acid 12–28% (dose-dependent), carbamazepine up to 6%, phenytoin

Mood stabilizers and antidepressants — most, such as lithium 12–19%

Others — cimetidine, retinoids (acitretin > isotretinoin), antithyroid drugs, cholesterol-lowering drugs, interferons, anti-infective agents,
amphetamines, nonsteroidal anti-inflammatory drugs (NSAIDs), bromocriptine, levodopa, some antipsychotics and anti-anxiety drugs,
rarely tricyclic antidepressants such as amitriptyline

Of the given options, lithium with the highest association of 12-19% among other options is the most likely cause for the hair loss. The second
most likely cause could be carbamazepine (option C) (1.6-6%). Although the patient has been on lithium for a long time, the increased dose of
lithium is still more likely to have resulted in the hair loss than carbamazepine.

Enoxaparin-induced hair loss is rather rare, and enoxaparin (option D) is less likely, yet not impossible, to have caused the issue. Aspirin
(option A) and triptans e.g., sumatriptan (option E) are not associated with any clinically meaningful correlation with hair loss to date.

References

• DermNet NZ – Alopecia from drugs

Last updated:
Time spent: QID:1676
2023-2-12

1443 of 1943
A 70-year-old man presents to the Emergency Department for an ongoing central chest pain that started about 1 hour ago. He has the history
of recurrent chest pains that are brought on with exertion and relieved with rest, and type II diabetes mellitus. Further assessment establishes
the diagnosis of unstable angina. You start him on aspirin, clopidogrel, heparin, and glyceryl trinitrate. The on-call cardiologist advises that you
start him on tirofiban as well if it is not contraindicated. Which one of the following is not a contraindication for tirofiban?

A. Non-ST elevation myocardial infarction (NSTEMI).

B. Acute pericarditis.

C. Vasculitis.

D. Aortic dissection.

E. History of intracranial neoplasm.

Correct
45% answered correctly

Explanation:

Correct Answer Is A

Tirofiban and abciximab are glycoprotein IIb/IIIa inhibitors. Their mechanism of action is preventing fibrin from binding to platelets by
occupying the glycoprotein IIIa/IIb receptors. They may be indicated in management of unstable angina, or NSTEMI, particularly if early
cardiac catheterization is planned.

Contraindications to their use include:

History of stroke within 30 days or any history of hemorrhagic stroke


Known history of intracranial disease (e.g., neoplasm, arteriovenous malformation, aneurysm)
Active or recent (within the previous 30 days) clinically relevant bleeding (e.g., gastrointestinal bleeding)
Malignant hypertension
Trauma or major surgical intervention within the past 6 weeks
Thrombocytopenia (platelet count <100,000/mm3); disorders of platelet function
Clotting disturbances (e.g. prothrombin time >1.3 times normal or INR >1.5)
Severe liver failure

Tirofiban is not recommended in the following situations:

Traumatic or protracted cardiopulmonary resuscitation, organ biopsy or lithotripsy within the past 2 weeks
Severe trauma or major surgery >6 weeks but <3 months
Active peptic ulcer within past 3 months
Uncontrolled hypertension (>180/110 mm Hg)
Acute pericarditis
Active or a known history of vasculitis
Suspected aortic dissection
Hemorrhagic retinopathy
Occult blood in the stool or hematuria
Thrombolytic therapy
Concurrent use of drugs that increase the risk of bleeding

NSTEMI is an indication for use of tirofiban, not a contraindication.

References

• EMC - Tirofiban 50 micrograms/ml solution for infusion

Last updated:
Time spent: QID:232
2023-2-12

1444 of 1943
A 45-year-old man is diagnosed with a primary lymphoma of the small bowel. In check for predisposing factors, which one of the following
risk factors is most likely to be present?

A. Crohn disease.

B. Peutz-Jeghers syndrome.

C. Celiac disease.

D. Familial adenomatous polyposis.

E. Previous gastrectomy.

Incorrect. Correct answer is C


45% answered correctly

Explanation:

Correct Answer Is C

Approximately, 50% of small bowl malignancies are adenocarcinomas. The rest include carcinoid tumor, lymphoma, gastrointestinal stromal
tumors, and metastatic deposits.

Of the given options, the most important risk factor for small bowl lymphoma is celiac disease. Other options are potential risk factors for
development of small bowel adenocarcinoma (not lymphoma).

(Option A) Crohn disease is associated with a 100-fold increase in risk of small bowel adenocarcinoma in the affected areas, especially the
distal ileum.

(Option B) Peutz – Jeghers syndrome, characterized by development of hamartomas which may undergo malignant changes, increases the
risk of adenocarcinoma.

(Option D) Although patients with familial adenomatous polyposis are at significantly increased risk of colorectal cancer, duodenal and small
bowel adenocarcinoma can also occur.

(Option E) Patients who have undergone gastrectomy more than 25 years ago are at increased risk of developing adenocarcinoma in the
gastric stump.

References

• http://www.ncbi.nlm.nih.gov/pubmed/11903028

• AMC Handbook of Multiple Choice Questions – pages 450-451

Last updated:
Time spent: QID:693
2023-2-12

1445 of 1943
Which one of the following statements is correct about adenosine?

A. It blocks impulse transmission through atrioventricular (AV) node.

B. It has a short half-life of 10-15 minutes.

C. It is not contraindicated in asthmatic patients.

D. It is the first-line therapy for ventricular tachycardias.

E. It does not reduce heart rate.

Correct
45% answered correctly

Explanation:

Correct Answer Is A

Adenosine is the treatment of choice for narrow-complex supraventricular tachycardias (SVTs). The mechanism of action is by transient
blockage of impulse transmission through AV node. It has no or little effect on myocytes or other parts of the cardiac conductive system. The
half-life is short and 10-15 seconds. Adenosine is given as bolus, followed by saline flush.

It efficiently reduces the heart rate and improves cardiac output once the heart rate slowed down.

The following conditions are contraindications to adenosine administration:

Second- or third-degree AV block (except patients with a functioning artificial pacemaker)


Sinus node disease, such as sick sinus syndrome, or symptomatic sinus bradycardia (except in patients with a functioning artificial
pacemaker)
Known or suspected bronchoconstrictive or bronchospastic disease such as asthma or COPD
Known hypersensitivity to adenosine

References

• Therapeutic Guidelines – Cardiovascular

• Drugs.com - Adenosine

Last updated:
Time spent: QID:243
2023-2-12

1446 of 1943
Which one of the following malignancies is least treatable?

A. Hodgkin lymphoma (HL).

B. Non-Hodgkin lymphoma (NHL).

C. Breast cancer.

D. Testicular cancer.

E. Choriocarcinoma.

Incorrect. Correct answer is B


45% answered correctly

Explanation:

Correct Answer Is B

Of the options, NHL, with a 5-year survival of 66%, is the least treatable malignancy. Some malignancies and their 5-year survival rates are
summarized in the following table:

cancer 5-year survival


Testicular 99%
Melanoma 90%
Thyroid 92%
Breast 87%
Uterus 84%
Prostate 84%
Hodgkin lymphoma 82%
Choriocarcinoma >80%
Non-Hodgkin lymphoma 66%
Colon 63%
Bladder 51%
Ovary 41%
Stomach 25%
Lung 11%
Liver 10%
Pancreas 5%

References

• Murtagh’s General Practice – McGraw Hill – 6th Edition – page 257

• http://www.cancerresearchuk.org/about-cancer/type/

Last updated:
Time spent: QID:779
2023-2-12

1447 of 1943
Which one of the following is not an ECG finding associated with digoxin use?

A. Prolonged QT interval.

B. Prolonged PR interval.

C. Inverted T waves.

D. Ventricular bigeminy.

E. ST depression.

Correct
45% answered correctly

Explanation:

Correct Answer Is A

There are 4 benign ECG changes associated with therapeutic levels of digoxin, including:

1. T-wave changes such as flattening or inversion


2. QT interval shortening
3. “Scooped” appearance of ST segment with ST segment depression
4. Increased amplitude of U-wave

ECG changes in toxic levels include:

Frequent PVCs, especially in a sick heart – the most common arrhythmia


Supraventricular tachyarrhythmias
AV block (including PR interval lengthening in 1st degree AV block)
Junctional escape rhythms
Prolongation of PR interval
Bidirectional ventricular tachycardia with an alternating axis of the QRS complex is rare but the most specific ECG finding of digoxin
toxicity.

Prolongation of QT interval is not an ECG feature of digoxin use or toxicity.

References

• GP Notebook - ECG changes in digoxin

• Medscape - Digitalis Toxicity

Last updated:
Time spent: QID:283
2023-2-12

1448 of 1943
Which one of the following is not an ECG finding associated with digoxin use?

A. Prolonged QT interval.

B. Prolonged PR interval.

C. Inverted T waves.

D. Ventricular bigeminy.

E. ST depression.

Correct
45% answered correctly

Explanation:

Correct Answer Is A

There are 4 benign ECG changes associated with therapeutic levels of digoxin, including:

1. T-wave changes such as flattening or inversion


2. QT interval shortening
3. “Scooped” appearance of ST segment with ST segment depression
4. Increased amplitude of U-wave

ECG changes in toxic levels include:

Frequent PVCs, especially in a sick heart – the most common arrhythmia


Supraventricular tachyarrhythmias
AV block (including PR interval lengthening in 1st degree AV block)
Junctional escape rhythms
Prolongation of PR interval
Bidirectional ventricular tachycardia with an alternating axis of the QRS complex is rare but the most specific ECG finding of digoxin
toxicity.

Prolongation of QT interval is not an ECG feature of digoxin use or toxicity.

References

• GP Notebook - ECG changes in digoxin

• Medscape - Digitalis Toxicity

Last updated:
Time spent: QID:283
2023-2-12

1449 of 1943
Which one of the following is the most common cancer in Australia?

A. Lung cancer.

B. Skin cancer.

C. Breast cancer.

D. Prostate cancer.

E. Colorectal cancer.

Incorrect. Correct answer is B


45% answered correctly

Explanation:

Correct Answer Is B

Non-melanoma skin cancers (basal cell carcinoma and cutaneous squamous cell carcinoma) are the most common cancers in Australia.

Excluding non-melanoma skin cancers, the top five concerns in the Australian population in the order of prevalence are as follows:

1. Prostate cancer
2. Breast cancer (in females)
3. Melanoma of the skin
4. Colorectal cancer
5. Lung cancer

However, once non-melanoma skin cancers (e.g., squamous and basal cell carcinoma) are included, skin cancer remains the most prevalent
in Australia.

References

• Cancer Australia - Cancer in Australia statistics

Last updated:
Time spent: QID:811
2023-2-12

1450 of 1943
A mother brings her 5-year-old boy to your practice for consultation. He has recently been started on FLixotide® (fluticasone propionate)
inhaler, 4-hourly, after he was diagnosed with asthma. She wants to know about the adverse effects of this drug. Which one of the following is
not an adverse effect of this medication?

A. Linear growth retardation.

B. Candidiasis.

C. Osteoporosis.

D. Weight gain.

E. Osteomalacia.

Incorrect. Correct answer is E


45% answered correctly

Explanation:

Correct Answer Is E

Long-term inhaled corticosteroid use in asthmatic patients is associated with a number of complications. These complications can be local
or systemic.

Local effects- local effects are the most common adverse effects of inhaled corticosteroids, and include:

Oropharyngeal Candidiasis
Dysphonia (hoarseness)

These adverse effects can be easily managed by rinsing the mouth and gargling with water after use.

Systemic effects- systemic adverse effects are far less common and may be seen in those on long-term high-dose inhaled corticosteroids:

Adrenal suppression
Bone loss (osteopenia and osteoporosis)
Skin thinning
Increased rate of cataract formation
Weight gain
Metabolic changes (alteration of glucose and lipid metabolism)
Decreased linear growth in children
Psychiatric effects

Osteomalacia is not an adverse effect of inhaled corticosteroids.

References

• UpTODate - Major side effects of inhaled glucocorticoids

Last updated:
Time spent: QID:371
2023-2-12

1451 of 1943
Which one of the following malignancies is associated with poorest prognosis?

A. Hodgkin lymphoma.

B. Non-Hodgkin lymphoma.

C. Testicular cancer.

D. Metastatic breast cancer.

E. Melanoma.

Incorrect. Correct answer is D


45% answered correctly

Explanation:

Correct Answer Is D

Hodgkin lymphoma has a 5-year survival of 86%, which is higher than that of non-Hodgkin lymphoma, 69%. Testicular cancers often have an
excellent prognosis and a 5-year-survival rate of approximately 98%. 5-year survival rate of melanoma is 86%.

Of the given options, metastatic breast cancer has the poorest prognosis. Unlike non-metastatic breast cancers that are associated with good
prognosis, stage IV breast cancer or metastatic breast cancer is associated with a 5-year survival rate of 22% which is the lowest among
other options.

An estimate of 5-year survival rates for different cancers is summarized in the following table: (reproduced from Murtagh's General Practice -
6th Edition)

Cancer 5-year survival rate


Testicular 98%
Thyroid 92%
Melanoma 91%
Breast 89%
Prostate 89%
Hodgkin lymphoma 86 86%
Uterus 84%
Non-Hodgkin lymphoma 69%
Colon 63%
Bladder 51%
Ovary 41%
Stomach 25%
Liver 10%
Pancreas 5%

References

• Murtagh’s General Practice – McGraw Hill – 6th Edition – page 257

• http://www.cancer.org/cancer/breastcancer/detailed

• http://seer.cancer.gov/csr/1975_2013/results_merge

Last updated:
Time spent: QID:974
2023-2-12

1452 of 1943
During a routine visit, a 50-year-old woman with a 10-year history of type 2 diabetes mellitus is found to have a blood pressure of 145/90
mmHg. Further assessment reveals microalbuminuria. The patient is planned to be started on ACE inhibitors. Which one of the following
would be an absolute contraindication to use of an ACE inhibitor in this patient?

A. A previous history of angioneurotic edema.

B. Renal insufficiency.

C. Asthma.

D. A history of recent myocardial infarction.

E. A cardiac ejection fraction of <25%.

Correct
45% answered correctly

Explanation:

Correct Answer Is A

Angiotensin converting enzyme (ACE) inhibitors are drugs of choice in diabetic patients who develop hypertension. Even in normotensive
diabetics who develop microalbuminuria, ACE inhibitors or angiotensin receptor blockers (ARBs) are strongly recommended. ACE inhibitors,
by inhibiting the angiotensin converting enzyme, result in dilation and consequently decreased hydrostatic pressure of the glomerular
capillaries. The decreased hydrostatic pressure of the glomerular space leads to less protein extravasation into glomerulae and slowing down
of the rate at which diabetic nephropathy develops.

The only absolute contraindications to use of this drug class are:

Pregnancy
Previous angioedema following use of ACE inhibitors
Hypersensitivity to a specific ACE inhibitor.

(Option B) Although ACE inhibitors are associated with decreased GFR, the benefit of delayed progression to renal failure outweighs the
transient reduction in GFR in the long run. However, with significant reduction in GFR, either pre-existing or as a result of ACE inhibitor use, is a
relative contraindication to using this drug class.

(Option C) Asthma is not a contraindication to use of ACE inhibitors.

(Options D and E) Myocardial infarction and heart failure (diastolic or systolic) are among main indications for ACE inhibitors.

References

• RXmed.com - ACE Inhibitors

• Medscape - Medication Update - Angiotensin Converting Enzyme (ACE) Inhibitors

Last updated:
Time spent: QID:395
2023-2-12

1453 of 1943
A 78-year-old man with advanced prostate cancer is found dead in his house. Which one of the following could be the most likely primary
cause of his death?

A. Multi-organ failure.

B. Chronic renal failure.

C. Urinary tract infection.

D. Sepsis.

E. Heart attack.

Correct
45% answered correctly

Explanation:

Correct Answer Is A

Studies show that off all deaths in patients with metastatic (advanced) prostate cancer, almost 77.8% are directly related to the prostatic
cancer, 5.5% from other cancers and, 16.7% from non-cancer causes.

This case represents a sudden death in a patient with advanced (terminal cancer) which is defined as an incident in which a patient who had
been ambulatory, suddenly experiences a change in condition and dies within a day. Of the options, multiorgan failure seems to be the most
common cause of such cancer-related sudden deaths.

Urinary tract infection (option C) can result in sepsis (option D), especially in the elderly with comorbid conditions. Additionally, sepsis is
prevalent in patients with prostate cancer and the prevalence increases with time. However, this is multiorgan failure, as the end-result of the
sepsis, that leads to death.

Cardiovascular disease, cerebrovascular disease, and lung diseases such as COPD and pulmonary embolism are the most common causes of
non-cancer deaths in patients with advanced prostate cancer. An acute condition such as heart attack or pulmonary embolism might as well
be the cause of this sudden death. Although there is a good chance that this patient has died of an acute coronary event i.e., heart attack
(option E) or pulmonary embolism as well, the question asks about the most likely cause of primary cause of the death rather than conditions
not directly related to the disease process (secondary causes).

While acute renal failure (either in isolation or as a part of multiorgan failure) could cause sudden death, chronic renal failure (option B) is less
likely to have caused sudden death in this patient.

NOTE – the primary cause of death is the disease, situation, or event that started the chain of events resulting in death. Consequences or
complication of this are usually considered secondary causes of death.

References

• A study of ‘sudden death’ in end-stage cancer patients receiving home care.

• The oncologist - What Do Prostate Cancer Patients Die Of?

Last updated:
Time spent: QID:1694
2023-2-12

1454 of 1943
A 65-year-old man, who is on selegiline for treatment of Parkinson disease, recently has been started on sertraline after he was diagnosed
with major depression. Today, he is brought to the Emergency Department with abdominal pain, diarrhea, and confusion. Which one of the
following would be the most appropriate next step in management?

A. Reduce sertraline.

B. Reduce selegiline.

C. Stop sertraline.

D. Stop selegiline.

E. Stop both sertraline and selegiline.

Incorrect. Correct answer is E


45% answered correctly

Explanation:

Correct Answer Is E

Sertraline is a selective serotonin reuptake inhibitor (SSRI) and selegiline is a monoamine oxidase inhibitor (MAOI) type B. The combination of
the two is notorious as the most common cause of serotonin syndrome.

Serotonin syndrome is a potentially life-threatening condition associated with increased serotonergic activity in the central nervous system.
Serotonin syndrome can occur with therapeutic medication use, inadvertent interactions between drugs, and intentional self-poisoning.

The clinical features of serotonin syndrome include:

Cognitive features: confusion, agitation, hypomania, hyperactivity, restlessness


Autonomic features: hyperthermia, sweating, tachycardia, hypertension, mydriasis, flushing, shivering
Neuromuscular features: clonus (spontaneous/inducible/ocular), hyperreflexia, hypertonia, ataxia, tremor

NOTE - Hypertonia and clonus are always symmetrical and are often much more dramatic in the lower limbs

The drugs implicated to cause serotonin syndrome and their mechanism of action are shown in the following table:

DRUG Mechanism of action


L-Tryptophan Serotonin precursor
SSRIs Inhibit serotonin reuptake
Tricyclic antidepressants Inhibit serotonin reuptake
Monoamine oxidase inhibitors (A>B) Inhibit metabolism of 5-HT
Pethidine Serotonin agonist
Tramadol Inhibits serotonin reuptake
LSD Partial serotonin agonist
Buspirone Partial serotonin agonist
Amphetamines and anorectics ↑ 5-HT release & ↓ reuptake
Atypical antidepressants Various
St John’s wort Unknown – may be all of the above
Lithium Unknown

Meticulous supportive care is the mainstay of therapy. Once serotonin syndrome issuspected, the most important initial management is to
stop all serotonergic drugs at once and care be taken that no other precipitant be inadvertently administered.

In the presence of abdominal pain, diarrhea and confusion, serotonin is very likely; hence, both sertraline and selegiline should be stopped as
the most important initial management.

If medical treatment is required, the serotonin antagonist cyproheptadine (antagonist of HT2) would be the first-line medication for treatment
of moderate to severe serotonin syndrome.

References
1455 of 1943
• MJA - Serotonin toxicity: a practical approach to diagnosis and treatment

• Australian Prescriber - Serotonin syndrome


Last updated:
Time spent: QID:466
2023-2-12

1456 of 1943
A 20-year-old girl presents with history of episodes of trembling, palpitation, shortness of breath,
and sweating. Each episode lasts for 5 minutes and then gradually resolves. She has no history of
heart disease, and is not on any regular medication. She denies any use of illicit drugs. Which one
of the following would be the best course of action for management?

A. Reassurance, explanation, and support.


B. Selective serotonin reuptake inhibitors.
C. Breathing in and out of a paper bag when attacks develop.
D. Tricyclic antidepressants.
E. Referral to a psychiatrist.

Correct
45% answered correctly

Explanation:

Correct Answer Is A

This patient has typical presentation of panic attacks.

Panic attack is characterized by the presence of at least 4 of the following features:

Palpitations, pounding heart, or accelerated heart rate


Sweating
Trembling or shaking
Sense of shortness of breath or smothering
Feeling of choking
Chest pain or discomfort
Nausea or abdominal distress
Feeling dizzy, unsteady, lightheaded, or faint
Derealization or depersonalization (feeling detached from oneself)
Fear of losing control or going crazy
Fear of dying
Numbness or tingling sensations
Chills or hot flushes

The cornerstone of treating panic attacks is explaining the condition to the patient, reassurance
and supporting the patient to understand the situation. For residual anxiety following a panic attack
benzodiazepines might be considered.

(Options B and D) Selective serotonin re-uptake inhibitors (SSRIs) and tricyclic antidepressants
(TCAs) have been used as pharmacological treatment of panic disorder (not panic attacks) as
complements to nonpharmacological management (e.g. CBT), which is first-line treatment of panic
disorder.

(Option C) Breathing in and out of paper bag is helpful if patient is hyperventilating during a panic
attack. It can provide relief in short time and can be taught to the patient after explanation and
1457 of 1943
reassurance.

(Option E) Referring to a psychiatrist is not needed at this stage because the diagnosis is
completely straightforward and clear.

References

• RACGP - Managing panic disorder in general practice

• Medscape - Panic Disorder

• Therapeutic Guidelines - Psychotropic


Last updated:
Time spent: QID:128
2023-2-12

1458 of 1943
An 82-year-old man male presents to the emergency department with a knife in his hand. He claims
that he hears voices saying his neighbors want to kill him and he should kill them before they kill
him. You, as the attending physician, manage to verbally de-escalate him and convince him to
surrender his knife. Which one of the following would be the next best step in management?

A. Call the hospital security.


B. Call the police.
C. Call his family to help you with dealing with him.
D. Calm him down and talk to him.
E. Offer him tea and biscuit.

Incorrect. Correct answer is A


45% answered correctly

Explanation:

Correct Answer Is A

The presentation is typical of acute psychosis and command hallucinations. Any patient with
command hallucinations to harm self or others is at significant risk of developing violence sooner
or later.

Although apparently this patient is ready to surrender his knife now, with command hallucinations
he may still pose risk to himself or others.

Under circumstances such as this one, calling the hospital security would be the next best step in
management. This will ensure the safety of you and the staff while you are planning further
measures.

(Option B) Calling the police was an appropriate option if this situation happened in the community
and not in a medical facility.

(Option C) Calling his family will provide an opportunity to obtain more information about his
medical and psychiatric history; however, security and safety comes first.

(Options D and E) Verbal de-escalation to calm the patient or measures to establish rapport such
as offering tea and biscuits is appropriate after saefty is ensured by the hospital security.

References

• http://www0.health.nsw.gov.au/policies/pd/2015/pdf

• Therapeutic Guidelines - Psychotropic; available from http://tg.org.au


Last updated:
Time spent: QID:63
2023-2-12

1459 of 1943
John, 24 years, had a diagnosis of depression and was started on sertraline (Zoloft) 50 mg/day 23
days ago, and was given a mental health care plan to see a psychologist for cognitive behavioral
therapy (CBT). He is in your office today for follow-up, and believes there has been no improvement
whatsoever. Which one of the following is the most appropriate next step in management?

A. Continue the same dose of sertraline.


B. Increase the dose of sertraline.
C. Switch to another SSRI.
D. Switch to an SNRI.
E. Add a different antidepressant medication to sertraline.

Correct
45% answered correctly

Explanation:

Correct Answer Is A

Selective serotonin reuptake inhibitors (SSRIs) and tricyclic antidepressants (TCAs) are the two
most prescribed medications for major depression.

Other major drug classes include serotonin norepinephrine reuptake inhibitors (SNRIs), (e.g.,
venlafaxine), reversible monoamine oxidase inhibitors (e.g., moclobemide), and non-selective
monoamine oxidase inhibitors (e.g., phenelzine).

SSRIs have safer adverse effect profile and less likely to cause dropout compared to TCAs.

Antidepressant are started at initial dose and gradually and incrementally increased in dosed until
therapeutic doses are reached. For most SSRIs the initial dose is the same as therapeutic dose. As
such, 50mg per day is both the initial and therapeutic dose for sertraline. The therapeutic response
to therapeutic dose is often delayed for 1-2 weeks, and as a rule of thumb, all antidepressants
should be trialed for at least 4-6 weeks after the therapeutic dose is reached before any change in
treatment is considered.

John has been started on sertraline 20 days ago and it is still early to consider a change to
treatment such as increasing the dose of sertraline (option B), switching to another SSRI (option C)
or an SNRI (option D), unless for adverse effects issues which feel bothersome to the patient. John
should be advised to continue the same dose for at least to 6 weeks under close monitoring for
any complications (e.g., adverse effects, suicidality, etc.)

Using two antidepressants together is NOT recommended. Therefore, addition of another


antidepressant to sertraline (option E) is not a correct option.

References

• RACGP – use of antidepressant medications in the general practice setting


Last updated:
Time spent: QID:1720
2023-2-12

1460 of 1943
Janet presents to you concerned about her 15-year-old son, Joey, after she found out he had been
wearing his older sister’s underwear. She is a single parent as her husband left them after Joey was
born. She works two jobs and has been under a lot of stress and has not had enough time to spend
with her kids. She is quite frustrated and distressed with Joey’s behaviour and believes there is
something seriously wrong with him and wishes she has not given birth to him at all. Which of the
following is the correct statement to share with Janet?

A. Joey may feel as if he is a girl trapped in a boy's body.


B. He is most likely doing it for fun.
C. Not having a father figure in his life is the reason for this behavior.
D. Janet’s lifestyle may be the main reason for Joey’s behavior.
E. It is a part of normal development at this age.

Correct
45% answered correctly

Explanation:

Correct Answer Is A

Joey’s interest in wearing clothing suggests differential diagnoses such as cross dressing,
transvestic disorder, gender dysphoria, and transgenderism. All these diagnoses should be born in
mind, discussed with Janet, and assessed thoroughly.

It is normal for children and teenagers to experiment with gender. For example, a girl might refuse
to wear skirts or dresses, or a boy wants to play ‘mum’. For most children and teenagers,
experimenting with gender does not mean they are gender diverse or transgender. Most children go
on to feel comfortable with their birth gender. In contrast, gender dysphoria in children and
adolescents is not a phase. Gender dysphoria is when one feels distressed because their gender
identity differs from the birth sex. This distress might affect their school or home life. Those with
gender dysphoria feel like they are trapped in a body of the opposite sex. It should be explained to
Janet while going over potential diagnoses and how one would feel if they were gender dysphoric
or transgender.

Janet is quite worried about her son and his behavior; therefore, telling her that he is most likely
doing this for fun (Option B) would not be reassuring to her, especially when you do not have all the
facts to draw such conclusion. It would be more prudent to counsel her and explain to her Joey
may be experiencing and encourage her to bring him in for counselling if he wants it. Personal
counselling and family insight therapy should be recommended.

There is not enough information in the scenario to suggest that not having a father figure in his life
(option C) or Janet’s lifestyle (option D) could be the reason for Joey’s manner. Although both
these reasons are disruptive to the normal functioning of their family life, Joey does not exhibit any
other characteristics or behaviors to suggest that he is acting out in response to these disruptions.

By about the age 6 or 7 years, children begin to understand that sex is permanent across situations
and over time. Once they develop this understanding, they begin to act as members of their sex.
Therefore, it would be inaccurate to tell Janet that Joey’s behavior is a part of normal development
(Option E) at the age 15.

1461 of 1943
TOPIC REVIEW

Cross-dressing/ transvestitism

Cross-dressing is defined as typically heterosexual men wearing women's clothing. There is no


correlation between crossdressing and transgenderism or homosexuality. Cross dressers choose
to dress as women only some of the time and enjoy experiencing both the masculine and feminine
parts of themselves. As opposed to transvestitism, a transgender person lives fulltime in the
gender that they identify with.

DSM-5 considers cross-dressing as a psychiatric disorder (transvestic disorder) if cross-dressing


or thoughts of cross-dressing are always or often accompanied by sexual excitement. The main
difference between cross-dressing and transvestitism is that the latter is associated with sexual
arousal.

Transgenderism

Children whose gender identity differs from the gender they were assigned at birth are known as
transgender or gender diverse. Transgender is a non-medical term describing individuals whose
gender identity (inner sense of gender) or gender expression (outward performance of gender)
differs from the sex or gender to which they were assigned at birth. Not all transgender people
suffer from gender dysphoria and that distinction is important to keep in mind. Gender dysphoria
and/or coming out as transgender can occur at any age.

People who are transgender may pursue multiple domains of gender affirmation, including social
affirmation (e.g., changing one’s name and pronouns), legal affirmation (e.g., changing gender
markers on one’s government-issued documents), medical affirmation (e.g., pubertal suppression
or gender-affirming hormones), and/or surgical affirmation (e.g., vaginoplasty, facial feminization
surgery, breast augmentation, masculine chest reconstruction, etc.). Of note, not all people who are
transgender will desire all domains of gender affirmation, as these are highly personal and
individual decisions.

References

• RCH - Gender dysphoria

• American Psychiatric Association - Gender Dysphoria

• RACGP - AFP: Gender dysphoria


Last updated:
Time spent: QID:1728
2023-2-12

1462 of 1943
Eve is 45 years old and has presented to your clinic for consultation in tears. Her only son died 6
weeks ago from a car accident, and since then she has been having difficulty sleeping and terrible
headaches. She does not enjoy doing her life routines anymore and does not feel like eating.
Further probing reveals that she hears her son speaking to her and calling her name and
sometimes she feels his presence in her bedroom at night. She wants something to help her with
her sleep. Which one of the following is the most appropriate action in this situation?

A. Referral to hospital for admission, assessment, and treatment.


B. Referral to a psychiatrist.
C. A short course of benzodiazepines and referral for grief counselling.
D. A short course of selective serotonin re-uptake inhibitors (SSRI) and referral for grief
counselling.
E. A short course of antipsychotics and referral for grief counselling.

Incorrect. Correct answer is C


45% answered correctly

Explanation:

Correct Answer Is C

Given the symptoms and their duration, a normal grief reaction is the most likely explanation to this
woman’s problem. Janet is grieving for her deceased son, and this has affected her in many ways,
including a disruption of her daily routine, weight loss and decreased sleep and appetite. Patients
with normal grief reaction return to normal social functioning within 2 months. The symptoms
however might last up to one year with waxing and waning.

Eve has come to your clinic seeking help; in particular, she wants something to help her with her
sleep. In the absence of any red flags (suicidality, drug seeking behaviour, etc.), the most
appropriate action would be to prescribe a short course of benzodiazepines (< 7 days) for her
insomnia and referral to grief counselling to help her cope with her loss.

Sadness, despair, tearfulness, decreased sleep, decreased appetite and decreased interest in life
and the world, are some of the common findings in normal grief reaction. While guilt and shame
are not common in normal grief reaction, they are still possible. Suicidality is not usually a concern
unless the patient has said something indicative of her intent and plans to do so which is not the
case here.

Eve might require referral to psychiatrist (option B) down the line if grief counselling does not
resolve the issue and symptoms persist. It is not necessary for now.

SSRIs would be indicated if Eve had a diagnosis of moderate to severe depression. Eve does not
fulfill criteria for a diagnosis of major depression. While referral for grief counselling is necessary,
SSRIs are not indicated neither for depressive symptoms nor for the sleep issue.

Simple visual and auditory hallucinations of the deceased person are common and may lead the
bereaved person to fear he/she is losing their mind. These are a result of her grief reaction, and not
new onset psychosis. Similarly, Eve is hearing her son speaking to her and calling her name and
sometimes, she feels his presence in her bedroom at night. This could be considered normal in

1463 of 1943
grief as long as there are no other psychotic features necessitating antipsychotic treatment. Eve
does not have psychotic illness and antipsychotics (option E) would not be indicated.

References

• RACGP – Theme: Grief and loss

• World Psychiatry - Grief and bereavement: what psychiatrists need to know


Last updated:
Time spent: QID:1730
2023-2-12

1464 of 1943
A middle-age famer from Queensland presents with symptoms of poor sleep, lack of concentration
and energy, decreased appetite, and low mood for the past few weeks. He believes that all these
started after the drought and blames the government for being reckless and irresponsible. When
you mention that his symptoms are of depressive nature, he objects it and says he is just
exhausted and not depressed. Which one the following defense mechanisms is he using?

A. Displacement.
B. Projection.
C. Denial.
D. Rationalization.
E. Reaction formation.

Incorrect. Correct answer is C


45% answered correctly

Explanation:

Correct Answer Is C

This patient is experiencing symptoms of depression (poor sleep, lack of concentration and energy,
decreased appetite, and low mood) brought on by his recent losses after the drought. As the doctor
attempts to explain to him that his symptoms are of a depressive nature, he refuses to accept it
and insists that his symptoms are a result of exhaustion and not depression. Based on the facts in
the scenario, this patient is most likely utilizing denial as a defense mechanism. Denial is a form of
psychotic defense mechanism evident by the replacement of external reality with wishful fantasy
(behaving as if an aspect of reality does not exist).

When counselled by the doctor, he said he blamed the government for being reckless and
irresponsible. This is an example of projection (Option B), another form of a defense mechanism.
Projection refers to the interpretation of internal impulses as though they are outside oneself
(attributing one’s own feelings to others). In this scenario, the drought could have been a cause for
his symptoms as it might have affected his livelihood as a farmer, but the patient places blame on
the government without accepting his share of responsibility to have taken adequate safety
measures in case of natural causes. Displacing his feelings helps him subconsciously place the
root cause of this feeling onto someone or something else. In other words, he is projecting the
negative feelings of recklessness and irresponsibility elsewhere to protect his ego. However, this
question refers to the patient’s reaction to being informed that his symptoms are of a depressive
nature. Therefore, projection (option B) is not the correct option.

Displacement (option A), another defense mechanism and occurs when a person represses
affection, fear, or impulses that they feel towards another person as they believe it is irrational or
socially unacceptable to demonstrate such feelings; therefore, these feelings are displaced toward
another person or thing.

Rationalization (option D) defense mechanism involves using rational explanations to justify


behaviors that are unacceptable (justifying behaviour to avoid difficult truths). In doing so, they
avoid accepting the true cause or reason resulting in the present situation. This patient would be
using rationalization if, for example, he said: “of course I am depressed. The drought has destroyed

1465 of 1943
all my crops and left me with losses”. In this scenario, however, he is denying his depressive
symptoms altogether.

In reaction formation (option E) an individual expresses the opposite of their true feelings,
sometimes to an exaggerated extent. It is an intentional effort to compensate for conscious
dislikes. For example, if he felt that his self-esteem was threatened by being diagnosed with
depression, he would have acted overly aggressive. This is not the case here, and therefore
reaction formation is not the correct option.

References

• Simple Psychology - Defense Mechanisms

• Psychology Today - The Essential Guide to Defense Mechanisms


Last updated:
Time spent: QID:1732
2023-2-12

1466 of 1943
Ali, 45 years old is in your office with his wife who is concerned about him and insisted that he sees
a doctor because he seems to have been ‘lost’ and ‘confused’ since he had a severe car accident
and lost one of his best friends who was in the car with him 5 months ago. He only sustained a
head injury in the accident which was cleared as a minor one with no serious complications at that
time. Which of the following could be the most likely cause to this presentation?

A. Major depressive disorder.


B. Post-traumatic stress disorder.
C. Post-concussion syndrome.
D. Late-onset schizophrenia.
E. Dementia.

Incorrect. Correct answer is C


45% answered correctly

Explanation:

Correct Answer Is C

This Scenario represents a common undiagnosed and mistreated condition seen in general
practice after a head trauma, namely post-concussion syndrome (PCS).

Concussion is a temporary disturbance in brain function following a trauma to the head. It can also
occur after a blow to the body. Concussion can present with a variety of signs and symptoms
including:

Cognitive related symptoms:

Difficulty concentrating
Difficulty finding things
Difficulty reading
Memory problems
Brain fog
Easily distracted

Mood-related symptoms:

Anxiety
Depression
Feeling overwhelmed
Irritability
Low-energy or motivation
Various other mood/personality changes

Sensory-related symptoms:

Blurred vision
Car sickness or nausea with motion
Change in (or loss of) taste or smell
Ringing ears
1467 of 1943
Blood dysregulation symptoms:

Headache
Fatigue
Nausea
Dizziness
Sensitivity to light and noise
Sleep problems
Persistent neck pain
Pressure in the head
Tried eyes

Almost 90% of patients with concussive symptoms experience a quick recovery within few days to
weeks. Those in whom symptoms persist beyond 3 moths are defined as having post-concussion
syndrome (PCS).

Since Ali has been in a car accident resulting in his friend’s death, post-traumatic stress disorder
(PTSD) (option B) should be considered as well. Symptoms may include flashbacks, nightmares,
and severe anxiety, as well as uncontrollable thoughts about the event. Symptoms should persist
for at least 1 month before a diagnosis of PTSD is made. The DSM-5 has a more expansive set of
criteria including 20 different symptoms across the domains of re-experiencing, avoidance,
negative cognitions and moods, and hyperarousal. Ali has none of such symptoms; therefore,
unlikely to have PTSD.

Major depressive disorder (option A) requires the persistence of 5 or more of the 9 Criteria A
symptoms listed in the Diagnostic and Statistical Manual of Mental Disorders, 5th edition (DSM-5)
– depression, loss of pleasure, weight change, sleep change, retardation, loss of energy, feelings of
worthlessness, diminished concentration, thoughts of death – in a person who has never had an
episode of mania. Ali does not meet the criteria for major depressive disorder.

Late-onset schizophrenia (option D) is defined as schizophrenia starting after the age of 45 years.
Patients with late-onset schizophrenia typically present with the same positive psychotic
symptoms (paranoid delusions and hallucinations) as do younger schizophrenic patients, although
these symptoms are less severe in older patients. In contrast with early-onset disease,
disorganised thoughts and negative symptoms (flat affect, alogia, avolition) are less likely to occur
in patients with late-onset disease. Late-onset schizophrenia is also less likely to be associated
with impaired learning and cognitive functions. Ali has no paranoia, delusions or hallucinations;
therefore, not likely to have late-onset schizophrenia.

Dementia is a clinical syndrome that is caused by a number of underlying diseases. Such as


vascular dementia, frontotemporal dementia, dementia with Lewy body, Alzheimer’s. The DSM-5
diagnostic criteria for dementia nclude the following:

Significant cognitive decline from a previous level of performance in one or more cognitive
domains (i.e., complex attention, executive function, learning and memory, language,
perceptual-motor or social cognition.

The cognitive deficits interfere with independence in everyday activities (paying bills,
managing medications).

The cognitive deficits are not better explained by another mental disorder (e.g., major
depressive disorder, schizophrenia).

1468 of 1943
Ali has no memory loss, language problem, or cognitive deficit; therefore, unlikely to have dementia
(option E).

References

• Medscape - Post-concussion Syndrome

• RACGP – News GP – The potentially long-lasting effects of concussion

• The American Journal of Psychiatry – Late-Onset Schizophrenia and Very-Late-Onset


Schizophrenia-Like Psychosis: An International Consensus
Last updated:
Time spent: QID:1734
2023-2-12

1469 of 1943
During taking a history from a patient in a psychiatry ward, you ask him if there is any history of
mental illnesses in the family. He answers: ‘Good question. Now that you’ve asked, I must tell you
something. Whales are good creatures. We take their oil and use it to light the world. My father
hunted whales as did my grandpa. The business runs in the family and of course, we are all sane
and sound. Do you think we are crazy?”

Which one of the following is present in this patient’s speech?

A. Flight of ideas.
B. Word salad.
C. Circumstantiality.
D. Derailment.
E. Tangentiality.

Incorrect. Correct answer is C


45% answered correctly

Explanation:

Correct Answer Is C

The speech given by this patient as the answer to the question of whether there is a family history
of mental illnesses starts with a comment on an unrelated topic, whales. Thereafter, each next
sentence uses a clue in the previous sentence for continuation. At the end and after many detours
the answer to the question is provided: ‘We are all sane and sound’ (meaning there is no family
history of mental illness and they all have been sane and sound.

This pattern fits ‘circumstantiality’ best. Circumstantiality occurs when the patient drifts from one
topic to the other but eventually returns to the starting point. In other words, if a question is
imagined as a destination, there are many detours, but the destination is reached at the end. Unlike
circumstantiality, patients with derailment (option D) (loosening of association) never come back
to the topic they started off with. Tangentiality (option E) or tangential speech is a milder form of
derailment in which there is a hint linking two consecutive topics (whales and the whale oil that
light up the world [whale oil in the past had industrial use including for lighting and as fuel]).

Flight of ideas (option A) is characterized by over-productive speech with rapid shifting from one
topic to another. There is often a hint in the previous topic leading to the next one. In the flight of
ideas, there is a subjective feeling that the thoughts are racing. In the flight of ideas, the topic
spoken by the patient is organized but over-productive and in excess of details. In other words, the
general concept of the current topic is adequately understood, but a hint in one part leads the
patient to another. This patient follows a direct line made of pieces that are relevant and justify his
sanity and that of his family for that matter, rather than jumping from one topic to another.

In word salad (option B), the patient throws words together without any sensible and intelligible
meaning.

References

• Doctors Lounge - Formal Thought Disorders


1470 of 1943
• Stat Pearls - Circumstantiality
Last updated:
Time spent: QID:1756
2023-2-12

1471 of 1943
Jarred, 15 years old, is brought to you by his concerned parents for assessment. According to
them, he recently has significantly declined school performance. He is quite withdrawn and barely
leaves his room. He does not engage in social activities and even rarely talks with his parents.
Which one of the following, if present in the history of the mental exam, is most likely to help reach
a diagnosis?

A. Sleep issues.
B. Anxiety.
C. Loosening of association.
D. Use of recreational drugs.
E. Shyness.

Incorrect. Correct answer is C


45% answered correctly

Explanation:

Correct Answer Is C

The scenario illustrates social withdrawal represented by not engaging in social activities, barely
talking with the parents, and not leaving the room. He also has a significant decline in school
performance. This constellation of problems is likely to have been caused either by a mood
disorder or by the prodromal phase of early-onset schizophrenia. Of the options, the presence of
loosened association is highly predictive and suggestive of the latter.

“Loose associations” is a psychological term to describe a lack of connection between ideas. This
can manifest in speech as an individual moving quickly from one idea to an unrelated one in the
same sentence, expressing a random jumble of words and phrases. An example of a loose
association would be: “I like to dance; my feet are wet.” Loosening of association is a key symptom
in psychotic disorders such as schizophrenia.

Schizophrenia typically manifests with a prodrome of negative symptoms and psychosis (e.g.,
social withdrawal) that precedes the positive psychotic symptoms (e.g., hallucinations and bizarre
delusions).

childhood-onset and early-onset schizophrenia are more severe and debilitating forms of
schizophrenia. Early signs and symptoms may include problems with thinking, behavior, and
emotions:

Thinking:

Problems with thinking and reasoning


Bizarre ideas or speech
Confusing dreams or television for reality

Behavior:

Withdrawal from friends and family


Trouble sleeping
Lack of motivation e.g., presenting with a drop in performance at school
1472 of 1943
Not meeting daily expectations, such as bathing or dressing
Bizarre behavior
Violent or aggressive behavior or agitation
Recreational drug or nicotine use

Emotions:

Irritability or depressed mood


Lack of emotion, or emotions inappropriate for the situation
Strange anxieties and fears
Excessive suspicion of others

Sleep issues (option A) could be present in schizophrenia and other psychotic disorders as well as
a wide variety of other mental conditions such as mood disorders or anxiety disorders. In and of
itself, sleep issues are neither specific nor diagnostic.

Anxiety (option B) may indicate anxiety disorders; however, other symptoms are not consistent
with anxiety disorders. Therefore, its presence would not help with justifying this clinical
presentation and in fact will add more of a diagnostic challenge.

Individuals with mental illnesses are more prone to using recreational drugs. Also, the use of
recreational drugs may lead to mental issues (e.g., acute psychosis, withdrawal syndromes, etc.) If
there is a history of recreational drug use in Jarred (option D), it could be a behavioral issue
associated with a mental illness such as schizophrenia, or completely irrelevant to his
presentation. Either way, it is neither specific nor diagnostic.

Shyness (option E) is not uncommon among children and teenagers and could be normal behavior.
Shyness in history has no diagnostic significance or importance.

TOPIC REVIEW

According to DSM-V, diagnostic criteria for schizophrenia are as follows

At least two of the following symptoms of which at least one is the first three:

1. Delusions
2. Hallucinations (almost always auditor)
3. Disorganized speech (e.g., frequent derailment or incoherence)
4. Grossly disorganized or catatonic behavior
5. Negative symptoms (i.e., flattened affect, alogia, or avolition)

Active symptoms must persist for ≥ 1 month (or less if successfully treated) while the
continuous disturbance for ≥ 6 months
Symptoms must cause social, occupational, or personal function impairment lasting ≥ 6
months
Other possible causes for the symptoms are excluded

Terminology

Childhood-onset schizophrenia – Childhood-onset (or very early-onset) schizophrenia starts


prior to the age of 13 years
1473 of 1943
Early-onset schizophrenia – Early-onset schizophrenia starts prior to age 18
Adult-onset schizophrenia – Adult-onset schizophrenia starts at or after age 18

References

• Medscape - Schizophrenia

• UpToDate - Schizophrenia in children and adolescents: Epidemiology, pathogenesis, clinical


manifestations, course, assessment, and diagnosis
Last updated:
Time spent: QID:1758
2023-2-12

1474 of 1943
A 17-year-old girl is diagnosed with major depression associated with psychotic features. She is
planned to be started on selective serotonin reuptake inhibitors (SSRIs). For how long, should the
treatment be continued?

A. Six months.
B. One year.
C. Five years.
D. Life-long.
E. Two years.

Incorrect. Correct answer is C


45% answered correctly

Explanation:

Correct Answer Is C

All patients with major depression require continuation of treatment for up to 6 to 12 months;
however, the course of therapy should be extended in the following situations:

Two depressive episodes within 5 years


Three prior episodes
Severe psychotic depression
Serious suicidal attempt

If a patient is started on long-term treatment, this should probably be continued for at least 3 to 5
years, after which time the need for further management should be reviewed. Some patients may
even need life-long treatment.

1475 of 1943
A 27-year-old man is found to have major depression after he has psychiatric evaluation following
an attempted suicide. He is started on sertraline. For how long he should receive the medication?

A. Six months.
B. One year.
C. Two years.
D. Five years.
E. Life-long.

Incorrect. Correct answer is D


45% answered correctly

Explanation:

Correct Answer Is D

For most patients with depression the medical treatment should be continued for at least 6 months
to ideally 12 months.

In the following situations, however, the duration of therapy should be extended to 3 to 5 years:

Two episodes of major depression in 5 years


Three previous episodes of major depression
Depression with psychotic features
Depression with a serious suicidal attempt

After 3-5 years, the patient should be reassessed for the need for further management. Some
patient might need lifelong antidepressant therapy.

As this patient has had a suicidal attempt, he should receive antidepressant therapy for at least 3-5
years.

References

• Therapeutic Guidelines – Psychotropic; available from http://www.tg.org.au


Last updated:
Time spent: QID:508
2023-2-12

1476 of 1943
Which one of the following does not increase the risk of suicide?

A. Alcohol abuse.
B. Poor social support.
C. Inquiring the patient about suicidal ideation.
D. Conduct disorder.
E. Schizophrenia.

Incorrect. Correct answer is C


45% answered correctly

Explanation:

Correct Answer Is C

A variety of factors are associated with an increased risk of suicide:

Psychiatric disorders - Psychiatric illness is a strong predictor of suicide. More than 90% of
patients who attempt suicide have a major psychiatric disorder, and 95% of patients who
successfully commit suicide have a psychiatric diagnosis. Patients with psychiatric diagnoses kill
themselves at rates 3 to 12 times higher than other patients.

Hopelessness and impulsivity - Across psychiatric disorders, hopelessness is strongly associated


with suicide. Hopelessness can persist even when other symptoms of depression have remitted.

Impulsivity, particularly among adolescents and young adults, is also associated with acting on
suicidal thoughts.

History of previous suicide attempts or threats - The strongest single factor predictive of suicide is
prior history of attempted suicide. Patients with a previous history of suicide attempts are 5 to 6
times more likely to make another attempt; furthermore, up to 50% of successful victims have
made a prior attempt. One of every 100 suicide attempt survivors will die by suicide within one year
of the previous attempt which amounts to 100 times that of the general population.

Age, sex, and race - The risk of suicide increases with increasing age; however young adults
attempt suicide more often than older adults. Females attempt suicide 4 times more frequently
than males, but males are successful 3 times more often. These age and sex differences appear to
be primarily related to the lethality of the method chosen (e.g. firearms, hanging, jumping, etc.)
rather than a difference in completion rates for the same method. Elderly white men, aged 85 years
and older, and young black males have the highest suicide rate.

Marital status - based on marital status the suicide risk in descending order is increased in:

Those who never married


Widowed, separated, or divorced
Married without children
Married with children

Occupation - Unemployed and unskilled individuals are at higher risk for suicide than those who are
employed and skilled.
1477 of 1943
Health - Suicide risk increases with physical illness such as chronic pain, recent surgery, and
chronic or terminal disease.

Adverse childhood experiences - Childhood abuse and other adverse childhood experiences appear
to increase the risk of suicide in adults. Conduct disorders in children is shown to be associated
with an increased risk of suicide.

Family history and genetics - The risk of suicide increases in patients with a family history of
suicide. A first-degree relative who committed suicide increases the risk six-fold.

Antidepressants - Antidepressants can have potential association with suicide.

Other - The risk of suicide increases in following situations:

Accessibility to weapons
Sociopolitical, cultural, and economic forces
Violence and political coercion
Economic downturns
Living in rural areas
Being lesbian, gay, or bisexual
Lower intelligence

Of the given options, inquiring the patient about suicidal ideation is not associated with an
increased risk of attempting suicide. In fact, asking the patient directly about suicidal ideation is an
essential part in assessment of suicide risk.

References

• http://www.health.vic.gov.au/mentalhealth/suicide/

• http://www.uptodate.com/contents/suicidal-ideation
Last updated:
Time spent: QID:512
2023-2-12

1478 of 1943
Which one of the following will not increase the risk of depression?

A. High socioeconomic status.


B. Elderly male with cognitive decline.
C. Unemployment.
D. Family history of depression.
E. Substance misuse.

Correct
45% answered correctly

Explanation:

Correct Answer Is A

The following are the condition associated with increased risk of depression:

Family history of depression


Chronic illness
Co-occurring mental conditions such as anxiety, personality disorders, etc
Physical illness, physical or intellectual disability
Low self-esteem, distorted body image, social incompetence
Entering puberty and schooling, transition into workforce and independent living
(adolescents)
Language problems, generational culture clashes, cultural nonrecognition of mental health
problems, stresses from living between two cultures
Uncertainty, fear of rejection by family and friends, desire to ‘fit in’ with perceived societal
expectations, being bullied, being subjected to homophobic abuse (more in adolescents)
Domestic violence, poverty, family discord, sexual or physical abuse
Bereavement, separation from loved ones, divorce, trauma
Smoking, alcohol, drug use, internet use affecting sleep
Marginalisation, homelessness, refugee status, fostering, unemployment

The high socioeconomic status is a protective factor against depression, not a risk factor.

References

• https://www.nhmrc.gov.au/_files_nhmrc/publications

• http://psychcentral.com/lib/risk-factors-for-depre
Last updated:
Time spent: QID:513
2023-2-12

1479 of 1943
A 24-year-old woman presents with history of low mood, psychomotor retardation, decreased
appetite and decreased sleep for the past 2 weeks. She has family history of bipolar disorder.
Which one of the following is less common in bipolar depression than unipolar depression?

A. Psychomotor retardation.
B. Hypersomnia.
C. Decreased appetite.
D. Positive family history of bipolar disorder.
E. Delusions and hallucinations.

Incorrect. Correct answer is C


45% answered correctly

Explanation:

Correct Answer Is C

The following features are more commonly seen in bipolar depression than unipolar depression:

Psychomotor retardation
Increased appetite (hyperphagia)
Increased sleep (hypersomnia)
Early onset of first depression before 25 years of age
Delusions and hallucinations
Positive family history of bipolar disorder

It is very important to identify these features in patients with possible diagnosis of bipolar disorder,
who initially present with depression. This patient has positive family history of bipolar disorder,
psycho-motor retardation and age of onset of depression below 25 years, all favoring bipolar
depression.

NOTE - Psychomotor retardation is seen in depressive phase of bipolar disorder. It is not a feature
of mania associated with bipolar disorder.

Decreased sleep and decreased appetite are not common features of bipolar depression and are
seen in major depression more frequently.

References

• http://ajp.psychiatryonline.org/doi/10.1176/appi.a
Last updated:
Time spent: QID:517
2023-2-12

1480 of 1943
Accompanied by his wife, a 63-year-old man presents to your practice for consultation. She is
concerned about his husband because he has been recently behaving childish and bizarre. Last
week he was dismissed from his job as a manager in a local restaurant, because of treating rude to
customers and shouting at his colleagues. He does not shave, bathe or change his clothes as he
did before and is disheveled and unkempt all the time. She denies any falls, gait abnormalities, or
hallucinations in her husband. His memory is not significantly affected. Which one of the following
could be the most likely diagnosis?

A. Depression.
B. Alzheimer disease.
C. Lewy body dementia.
D. Frontotemporal dementia.
E. Schizophrenia.

Incorrect. Correct answer is D


45% answered correctly

Explanation:

Correct Answer Is D

Cognitive and behavioral changes in aged people are frequently faced in general practice, with
dementia and delirium being the most common underlying etiologies.

Cognitive function is measured by various mental functions, including memory, concentration,


praxis, language, executive functions, and visuospatial skills. Dementia refers to memory loss with
impairment of any other cognitive function that can interfere with social or occupational
functioning.

A myriad of causes have been identified for dementia. These causes can be reversible or
irreversible.

The most common reversible causes of dementia include:

Hypothyroidism
Vitamin B12 deficiency
Hepatic or uremic encephalopathy
Vasculitides affecting CNS
Space occupying brain lesions i.e. abscess/tumors either primary or metastatic
Medications – anticholinergics in particular
Normal pressure hydrocephalus
Central or obstructive sleep apnea
Subdural hematoma
Trauma
Depression

Some of the most common irreversible causes of dementia are:

Alzheimer disease (60-80% of cases )


1481 of 1943
Vascular dementia including multi-infarct dementia and Binswanger disease
Lewy body dementia
Frontotemporal degeneration (dementia) including Pick disease
Multifocal leukoencephalopathy

The case scenario describes a patient with social inappropriateness as the most concerning
presenting symptom without memory being significantly involved. Of the options, the most
consistent one with such scenario is frontotemporal dementia (Pick disease). In this disease,
social disgrace is the earliest symptom with memory impairment and forgetfulness following later.

Frontotemporal dementia is characterized by focal degeneration of the frontal and/or temporal


lobes. The typical age of onset is in the late 50s or early 60s, and the primary initial clinical
manifestations are changes in personality and social behavior or language, progressing over time
to a more global dementia. Other features include impaired initiation and planning, disinhibited
behavior and social disgrace and mild abnormalities on cognitive testing. Apathy and memory
deficits develop later in the course of the disease. A subset of patients may also exhibit symptoms
of extrapyramidal or motor neuron involvement at some point in the disease process.

(Option A) Patients with depression may present with pseudodementia which is different from
dementia in some aspects. It is less common for patients with pseudodementia to have
disinhibition or social disgrace. The history of disturbances in pseudodementia is often short and
abrupt onset, while dementia is more insidious. On cognitive testing, people with pseudodementia
often answer that they do not know the answer to a question, and their attention and concentration
are intact and they may appear upset or distressed. Those with true dementia will often give wrong
answers, have poor attention and concentration, and appear indifferent or unconcerned.

(Option B) In Alzheimer disease, forgetfulness is usually the presenting symptom. It is very unlikely
for a patient with Alzheimer disease to present with disinhibition and social inappropriateness early
in the course of the disease.

(Option C) although misbehavior and disinhibition is a common early feature in patients with Lewy
body dementia, the absence of other manifestations such as fluctuating cognition, hallucinations,
extrapyramidal deficits (Parkinsonism) and repeat falls makes this diagnosis less likely.

(Option E) Psychotic features such as hallucinations and delusion are a significant diagnostic
component in schizophrenia that is absent here. Moreover, development of schizophrenia at this
age is unusual.

References

• UpToDate - Frontotemporal dementia: Clinical features and diagnosis

• Therapeutic guidelines – Psychotropics


Last updated:
Time spent: QID:1151
2023-2-12

1482 of 1943
Which of the following does not increase the risk of postaprtum depression?

A. Adverse life events.


B. Lack of social support.
C. Past history of depression.
D. Emergency cesarean section.
E. Elective cesarean section.

Incorrect. Correct answer is E


45% answered correctly

Explanation:

Correct Answer Is E

Postpartum depression is most commonly seen during the first 1-8 weeks after delivery. The risk
factors for development of postpartum depression (and postpartum anxiety disorders) include:

Psychological

Antenatal anxiety, depression or mood swings


Previous history of anxiety, depression (option C), or mood swings, especially if occurred
perinatally
Family history of anxiety, depression or alcohol abuse, especially in first degree relatives
Severe baby blues
Personal characteristics like guilt-prone, perfectionistic, feeling unable to achieve, low self-
esteem
EPDS (Edinburgh postnatal depression) score ≥ 12

Social

Lack of emotional and practical support from partner and/or others (option B)
Domestic violence, history of trauma or abuse (including childhood sexual assault)
Many stressful life events recently (option A)
Low socioeconomic status, unemployment
Unplanned or unwanted pregnancy
Expecting first child or has many children already
Child care stress

Biological / medical

Recent cessation of psychotropic medications


Medical history of serious pregnancy or birth complications (including emergency cesarean
section (option D)), neonatal loss, poor physical health, chronic pain or disability, or
premenstrual syndrome
Perinatal sleep deprivation
Neonatal medical problems

Elective cesarean section does not increase the risk of postnatal depression.

1483 of 1943
References

• Medscape - Postpartum Depression


Last updated:
Time spent: QID:565
2023-2-12

1484 of 1943
A 76-year-old man is brought to your clinic by his son because of progressive decline in memory for
the past 18 months, as well as weakness of his upper and lower limbs and deterioration of motor
function. On examination, right hemiparesis is evident. Which one of the following could be the
most likely diagnosis?

A. Alzheimer's disease.
B. Lewy body dementia.
C. Multi-infarct dementia.
D. Parkinson's disease.
E. Pick’s disease.

Incorrect. Correct answer is C


45% answered correctly

Explanation:

Correct Answer Is C

The clinical picture is suggestive of multi-infarct dementia as the most likely cause. Multi-infarct
dementia is the second most common cause of dementia after Alzheimer disease (10% of all
cases with dementia).

Alzheimer disease (option A), Lewy body dementia (option B) and Pick’s disease (frontotemporal
dementia) (option E) are not associated with focal motor dysfunction. Parkinson's disease (option
D) as a cause of dementia is associated with motor dysfunction in the form of tremors and
extrapyramidal symptoms. Features include gate disturbances, tremor, rigidity and micrographia.
Hemiparesis is not a feature.

TOPIC REVIEW

Diseases associated with dementia:

Alzheimer disease - Alzheimer disease is the most common cause of dementia. Typically, patients
with Alzheimer disease present with problems in memory and visuospatial abilities that occur early
in the course of the disease. Despite severe memory impairment, social grace remains intact until
very late in the course of the disease when hallucinations and personality changes develop.
Alzheimer disease is not associated with motor or sensory dysfunction at least not very late in the
course of the disease.

Lewy body dementia - Lewy body dementia is characterized by fluctuating level of consciousness,
social disinhibition and Parkinsonism. Dementia often follows later. Lewy body dementia can be
confused with delirium.

Vascular dementia - Vascular dementia is divided into multi-infarct dementia, which typically has a
stepwise progression associated with frequent discrete cerebrovascular events, and Binswanger
disease, involving the subcortical white matter, that presents with a slowly progressive course.

1485 of 1943
Normal pressure hydrocephalus - It presents with prominent gait abnormalities early in the course
of the disease that usually precedes the onset of memory impairment. Urinary incontinence is
another distinguishing feature.

Pick’s disease (frontotemporal dementia) - Patients with Pick’s disease present with personality
changes early in the course of the disease, with relative sparing of visuospatial function. Social,
interpersonal, and emotional abnormalities precede memory impairment. The condition is first
noted by the family because the patient does not have insight into their problem.

Parkinson's disease - Dementia secondary to Parkinson disease is associated with typical features
of the disease such as gate disturbances, rigidity, tremors, micrographia, etc. Recurrent visual
hallucinations can be a feature that usually develop later in the course of the disease.

Creutzfeldt – Jacob disease (CJD) - Dementia of CJD develops in shorter time (weeks to months)
and has a course more aggressive than Alzheimer’s disease. Myoclonus is a distinguishing feature.
Diagnosis of CJD is by rapidly progressive dementia, myoclonus and the presence of 14-3-3 protein
in the CSF.

References

• Medscape - Dementia Pathology


Last updated:
Time spent: QID:687
2023-2-12

1486 of 1943
Which one of the following diseases of the central nervous system is caused by infectious
proteins?

A. Alzheimer’s disease.
B. Creutzfeldt- Jakob disease.
C. Parkinson’s disease.
D. Pick's disease.
E. Guillain-Barre syndrome.

Incorrect. Correct answer is B


45% answered correctly

Explanation:

Correct Answer Is B

Creutzfeldt-Jakob disease is caused by an infectious protein particle called prion. The disease is
often contracted at about mid-seventies and presents with dementia and myoclonus. It has a
progressive course and death follows in one year.

References

• Medscape - Prion-Related Diseases


Last updated:
Time spent: QID:691
2023-2-12

1487 of 1943
A 36-year-old man is brought to the emergency department by his relatives due to what they call an
outburst. When you step in the examining room, you realise that he is very agitated and is yelling at
the staff angrily and threatening to kill anyone who touches him. You are informed by the staff that
he has the past history of schizophrenia and has been aggressive and violent at previous
presentations. Which one of the following would be the next best step in management?

A. Tell the hospital security guards to hold him until he is calm.


B. Alert the hospital security to intervene and sedate him with haloperidol or
benzodiazepine.
C. Call hospital security and request the patient to behave properly.
D. Ask the hospital security to escort the patient out of the hospital.
E. Hand him over to the police.

Incorrect. Correct answer is B


45% answered correctly

Explanation:

Correct Answer Is B

Angry and agitated patients usually present a challenge to the health care providers. The best
approach is to calm down the patient in tactful professional manners and ideally without use of
sedatives; however when the patient is feared to pose harm to him/her or medical staff, hospital
security is required to be called and the patient sedated.

Since this patient is high-risk for violence due to his psychiatric condition and past history of
violence, calling the hospital security and sedating him would be the next best step in
management.

In all Australian emergency departments, the’ zero tolerance policy to violence’ is adopted. This
policy mandates sedation on the slightest concern of harm from the patients either to themselves
or the staff. The medications of choice for this purpose include haloperidol (preferred) and
midazolam. Midazolam carries the risk of respiratory depression ad hypotension and should be
used with great caution.

Benzodiazepines are first line when agitation is likely to have caused by alcohol intoxication or
withdrawal.

Patients in need of medical attention should be treated, not handed over to the Police, or escorted
out.

1488 of 1943
During assessment of a 32-year-old woman, she says ‘Oh well, my food! My food is cheese, cheese
is in the air, air is blue, and I came by bus’. Which one of the following can be the most likely
condition she is suffering from?

A. Schizophrenia.
B. Depression.
C. Dissociation.
D. Psychosis.
E. Adjustment disorder.

Incorrect. Correct answer is D


45% answered correctly

Explanation:

Correct Answer Is D

The scenario describes a typical example of ‘tangentiality'. Tangentiality occurs when one idea
connects to the next with one word or phrase, but the thoughts become confusing because they go
off on a tangent and end in a different subject. In the above example the word ‘food’ ends in
another comment about food unrelated to the previous sentence. The word ‘food’ is the only
connection. Loosening of association (derailment), on the other hand, happens when one idea
does not connect to the next at all.

Following are sentences told by patients in real situations. The first two are examples of derailment
(loosening of association), while the third is 'tangentiality':

The next day when I’d be going out you know, I took control. Like uh, I put bleach on my hair …
The traffic is rumbling along the main road. They are going to the north. Why do girls always
play pantomime heroes?
I think someone has infiltrated my copies of the cases. We’ve got to case the joint. I don’t
believe in joints, but they do hold your body together.

Derailment, loosening of association, poverty of content of speech, and thought blocking are
examples of disorganized thought content as characteristic features of psychosis and psychotic
disorders. A brief psychotic episode, schizophreniform disorder, schizoaffective disorder and
schizophrenia can have tangentiality/derailment as a presentation. Psychosis is the option
encompassing all these disorders and the correct answer for this question.

References

• Frontier Psychiatrist - Formal thought disorder

• Doctors Lounge - Formal though disorders

• Medscape - History and Mental Status Examination


Last updated:
Time spent: QID:788
2023-2-12

1489 of 1943
An alcoholic man is brought to your clinic because of alcohol intoxication. He is successfully
resuscitated. Which one of the following conditions in the history will direct you towards alcohol
dependence in this patient?

A. Any compulsory alcohol drinking first thing in the morning.


B. He drinks when he is anxious.
C. He drinks socially.
D. He drinks when he is driving.
E. He drinks more than 4 standard drinks in one single session.

Correct
45% answered correctly

Explanation:

Correct Answer Is A

There are screening tools to assess the likelihood of alcohol dependence in primary care setting.
CAGE questionnaire and AUDIT (Alcohol Use Disorders Identification Test) questionnaires are the
two most commonly use tools for this purpose.

The CAGE test consists of 4 questions. The letters of the acronym are the initial letters of key
words in questions:

1. Have you ever felt the need to CUT down on your drinking?
2. Have you ever felt ANNOYED by others asking you about your drinking?
3. Do you feel GUILTY about your drinking?
4. Do you ever have an EYE-OPENER in t the morning?

A positive response to any item on the CAGE questionnaire is a pointer towards alcohol
dependence and warrants a detailed assessment.

AUDIT questionnaire consists of 10 questions and a maximum score of 40. A score of 8 or more
out of the total score of 40 suggests alcohol dependence. AUDIT questionnaire has minimal false
positive or negative results.

Of the given options, compulsory morning drinking (eye opener) is suggestive of alcohol
dependence.

Other options may or may not be seen in alcohol dependence and are not useful in determining the
possibility of alcohol dependence.

References

• http://www.australianprescriber.com/magazine/25/3/

• http://www.health.nt.gov.au/library/scripts/object
Last updated:
Time spent: QID:801
2023-2-12

1490 of 1943
Jane, 32 years, gave birth to her child 3 days ago and has been in the ward since. On the second
day, she went to the nurses and told them that she is very worried and afraid about her child and
needs to keep her safe in the room. Last night she was seen walking around and moving objects
whole night. She took an insecticide and sprayed on the floor to get rid of imaginary bugs. On
examination, she is agitated with a heart rate of 110 bpm, respiratory rate of 22 breaths per minute
and temperature of 37.5°C. Which one of the following is most important initial step in
management?

A. Blood culture.
B. Full blood count (FBC).
C. Urine drug screen.
D. CT scan of the head.
E. Urine analysis (UA).

Incorrect. Correct answer is C


45% answered correctly

Explanation:

Correct Answer Is C

The scenario is consistent with diagnosis of an acute psychotic episode in early postpartum
period. A few explanations should be considered and investigated. The most important ones
include:

Substance-induced psychotic disorder (substance intoxication/withdrawal)


Brief psychotic disorder
Postpartum psychosis

Considering the time of the presentation (shortly after delivery) and other physical finding, drug
intoxication as a likely cause should be excluded first. For this purpose, a urine drug screen test is
the most appropriate measure.

A rare yet important diagnosis to consider is postpartum psychosis. The condition most commonly
presents within 2 weeks of childbirth. Hallucinations and delusions are usually present, often with
thought disorganization and/or bizarre behavior.

Although one should have postpartum psychosis as one possible diagnosis, more prevalent
etiologies for an acute psychotic episode should be consider and excluded first, especially with the
very early onset of symptoms which seems somewhat unusual (but not impossible) for
postpartum psychosis.

(Options A, B and E) With a temperature of 37.5°C, it is very unlikely that an infectious process is
the cause of this presentation; hence, blood culture, FBC, and UA are not appropriate steps; at least
not as the most important initial approach.

(Option D) CT scan of the head would have been indicated as a part of initial management if there
was a pointer towards an intracranial pathology.

1491 of 1943
References

• http://emedicine.medscape.com/article/294416-worku
Last updated:
Time spent: QID:813
2023-2-12

1492 of 1943
Alcohol-related dementia acounts for what percent of all cases of dementia in Australia?

A. 10%.
B. 5%.
C. 90%.
D. 80%.
E. 35%.

Incorrect. Correct answer is B


45% answered correctly

Explanation:

Correct Answer Is B

Incidence of alcohol-related dementia in Australia is about 5% (5.4% according to a study by


Panegyres and Frencham (2000)) of total demented population. Vascular dementia accounts for
10% and Alzheimer for 80-85% of all cases of dementia in Australia.

References

• Australian Institute of Health and Welfare


Last updated:
Time spent: QID:816
2023-2-12

1493 of 1943
A 23-year-old woman is referred to you for psychiatric assessment. When you ask her what she
thinks to be her problem, she answers: “Oh, isn’t it a long story? The train always runs on rails; the
birds keep flying up the hills and I have to remember to count my chickens before they hatch.” At
the end she does not come back to answer the question you asked. Which one of the following is
the most likely thought disorder she has?

A. Derailment.
B. Flight of ideas.
C. Pressured speech.
D. Circumstantiality.
E. Word salad.

Correct
45% answered correctly

Explanation:

Correct Answer Is A

The speech pattern given in the question is characteristic of derailment, also known as “loosening
of associations”, in which there is no connection between one topic and the next. One sentence is
spoken and then another sentence unrelated to the previous one follows.

Tangentiality is a milder form of derailment in which there is a linking hint between the two
consecutive sentences. An example is: I think someone has infiltrated my copies of the CASEs.
We’ve got to CASE the JOINT. I don’t believe in JOINTs, but they do hold your body together.

Words in upper case are the links between each sentence and the previous one. These connecting
hints are characteristic of tangentiality. In tangentiality (like derailment) there is no returning to the
initial topic.

(Option B) Flight of ideas is characterized by over-productive speech with rapid shifting from one
topic to another. There is often a hint in the previous topic leading to the next one. In the flight of
ideas, there is a subjective feeling that the thoughts are racing. In flight of ideas, the topic spoken
by the patient is organized but over-productive and in excess details. In other words, the general
concept of the current topic is adequately understood, but a hint in one part leads the patient to
another topic. The flight of idea is a matter of switching between coherent ideas.

(Option C) Pressure of speech (pressured speech) is a tendency to speak rapidly and excitedly, as if
motivated by an urgency not apparent to the listener. The speech is difficult to interrupt. It may be
too fast, or too tangential for the listener to understand.

(Option D) Circumstantiality occurs when the patient drifts from one topic to the other, but
eventually returns to the starting point. In other words, if a question is imagined as a destination,
there are many detours, but the destination is reached at the end. Unlike circumstantiality, patients
with derailment (loosening of association) never come back to the topic they started off with.
Tangentiality or tangential speech is a milder form of derailment in which there is a hint linking two
consecutive topics.

1494 of 1943
(Option E) In word salad, the patient throws words together without any sensible and intelligible
meaning.

References

• Frontier Psychiatrist - Formal thought disorder

• Doctors Lounge - Formal though disorders

• Medscape - History and Mental Status Examination


Last updated:
Time spent: QID:822
2023-2-12

1495 of 1943
A 47-year-old homeless man is brought to the Emergency Department by the police after he was
found agitated and fearful in the streets while carrying a bag full of rubbish. He says to you, as the
treating doctor, that he was caught by special military forces because he did not returned the DVDs
to a shop that belongs to the Jewish brotherhood. He also mentions that he has been followed by
demons because he has not done his prayers. He does not seem to be drunk but looks exhausted
and agitated. Which one of the following options is the most appropriate next step in management
of this patient?

A. Admit him to psychiatric unit for evaluation and treatment.


B. Ask the police about him.
C. Arrange urgent parenteral antipsychotics.
D. Arrange admission to general ward for assessment of his medical status.
E. Perform a urinary toxicology screen.

Incorrect. Correct answer is B


45% answered correctly

Explanation:

Correct Answer Is B

Psychiatric evaluation of the agitated patient includes:

De-escalation (verbal de-escalation; medical de-escalation [e.g. sedation] if necessary)

Visual observation of the patient before direct patient interview and paying careful attention
to the patient's verbal and nonverbal interaction with the examiner during de-escalation

Collecting collateral information - Collecting collateral information can be very helpful. While
de-escalation is in process, another team member can obtain verbal reports from family,
paramedics, or police officers or review written material that may accompany the patient.
Medical records are also an important source of information, and electronic records, if
available, can be readily accessed to determine previous diagnoses and medications. These
sources of information can be invaluable in determining the cause of agitation.

Of the given options, asking the police about him has the highest priority as it can provide
additional information for further management.

Once it is determined that the patient does not have an acute medical problem, there are several
important questions, the answers to which will guide the next step in management of the patient.

The first question is whether the patient has a delirium. It is not uncommon for a patient to go
through initial screening and have a diagnosis of delirium overlooked. The patient may be
mistakenly diagnosed as being psychotic, or the signs and symptoms of delirium may be subtle
and easily overlooked. In delirium, the patient has an altered level of awareness and problems
directing, focusing, sustaining, or shifting attention. The examiner must pay close attention to how
the patient interacts during the encounter and recognize these often subtle signs. Does the patient
seem confused and unable to focus? Are there perseverative behaviors? Does the patient appear to
be responding to visual hallucinations? Are there signs of language impairment, problems naming,

1496 of 1943
or other cognitive deficits? If agitation is associated with any of these findings, especially in the
setting of drug or medication use or medical illness, the presumptive diagnosis is delirium.

Risk of suicide or self-harm should be assessed and if presents managed accordingly. The
possibility of substance intoxication/withdrawal should be borne in mind and investigated if
indicated.

Other measures include assessment of general medical condition, followed by a detailed formal
psychiatric assessment if an underlying psychiatric condition is suspected. Antipsychotics may be
used for treatment of an agitated patient if verbal de-escalation fails.

References

• WestJM - Psychiatric Evaluation of the Agitated Patient: Consensus Statement of the American
Association for Emergency Psychiatry Project BETA Psychiatric Evaluation Workgroup
Last updated:
Time spent: QID:848
2023-2-12

1497 of 1943
A 35-year-old man presents to your practice with complaint of impotence for the past few months.
He smokes 20 cigarettes and regularly drinks 3-4 glasses of wine a day. He is marrying his fiancee
in one month. Which one of the following would be the most appropriate management option for
him at this stage?

A. Cessation of smoking.
B. Cessation of alcohol drinking.
C. Referral for psychotherapy.
D. Prescribe sildenafil.
E. Prescribe clomipramine.

Incorrect. Correct answer is D


45% answered correctly

Explanation:

Correct Answer Is D

Unless contraindicated, oral phosphodiesterase type 5 (PDE5) inhibitors such as sildenafil,


vardenafil, tadalafil and avanafil are first-line therapy for erectile dysfunction, regardless of the
etiology. In patients refractory to oral PDE5 inhibitors, one of these agents can be combined with an
injection of the prostaglandin PGE1.

Although not proven, it is likely that erectile dysfunction can be prevented by good general health,
paying particular attention to body weight, exercise, and cigarette smoking and alcohol
consumption. These measures should be considered but are not first-line treatment plan.

TOPIC REVIEW

Impotence is defined as male erectile dysfunction (ED), that is, inability to achieve or maintain an
erection sufficient for satisfactory sexual performance.

ED usually has a multifactorial etiology. Organic, physiologic, endocrine, and psychogenic factors
are involved in the ability to obtain and maintain erections. In general, ED is divided into 2 broad
categories, organic and psychogenic. Although psychologic factors have been mostly implicated
as the contributing factor, pure psychogenic ED is in fact uncommon; however, many men with
organic etiologies may also have an associated psychogenic component.

Conditions that may be associated with ED include diabetes, hypertension, and coronary artery
disease as well as neurologic disorders, endocrinopathies, benign prostatic hyperplasia, sleep
apnea, COPD, and depression. In fact, almost any disease may affect erectile function by altering
the nervous, vascular, or hormonal systems. Various diseases may produce changes in the smooth
muscle tissue of the corpora cavernosa or influence the patient’s psychological mood and
behavior.

Conditions associated with reduced nerve and endothelium function (e.g. aging, hypertension,
smoking, hypercholesterolemia, and diabetes) alter the balance between contraction and relaxation

1498 of 1943
factors. These conditions cause circulatory and structural changes in penile tissues, resulting in
arterial insufficiency and defective smooth muscle relaxation. In some patients, sexual dysfunction
may be the presenting symptom of these disorders.

Given the multiplicity of possible etiologic factors, it may be difficult to determine how much any
given factor is contributing to the problem. A thorough evaluation is necessary for correct
identification of the specific cause or causes in any given individual.

NOTE – hypertension has been found to have the highest association with ED. Benign prostatic
hyperplasia (BPH) has also a strong association. The cause exact mechanism, however, is unclear.

The first step in the management of ED is a thorough history that includes the following:

Sexual history
Medical history
Psychosocial history

A physical examination is necessary for every patient, emphasizing the genitourinary, vascular, and
neurologic systems. A focused examination entails evaluation of the following:

Blood pressure
Peripheral pulses
Sensation
Status of the genitalia and prostate
Size and texture of the testes
Presence of the epididymis and vas deferens
Abnormalities of the penis (e.g. hypospadias, peyronie plaques)

Management options for ED include the following:

Sexual counseling, if no organic causes can be found for the dysfunction. Sexual counselling,
however, can be used as an ongoing treatment along with other therapies
Oral medications
Injected, implanted, or topically applied medications
External vacuum and constriction devices
Surgery (penile prosthesis, vascular surgery)

References

• http://emedicine.medscape.com/article/444220-overv

• https://www.andrologyaustralia.org/wp-content/uplo
Last updated:
Time spent: QID:853
2023-2-12

1499 of 1943
A 17-year-old boy is brought to your office by his mother for evaluation. History revleas that he had
always been a good student until the age 15 when his grades began dropping. His mother has also
noticed that he has stopped socializing with friends and begun spending more time alone in his
room. He dropped the school earlier this year due to poor school performance. His mother reports
that he has been trying to home-school him, but he does not seem interested in learning. Days go
before he takes bath and he does not seem to care about his appearance. What concerns the
mother most and has led her to bringing him to see a psychiatrist is the fact that for the past 2
months, she has overheard him having what sounds like conversations with someone imaginary in
his room. Which one of the following could be the most likely diagnosis?

A. Major depressive disorder.


B. Substance abuse disorder.
C. Schizophrenia.
D. Brief psychotic disorder.
E. Schizophreniform disorder.

Incorrect. Correct answer is C


45% answered correctly

Explanation:

Correct Answer Is C

The scenario, describes a condition of 2-year duration, with social withdrawal as the initiating event
(dropping grades and poor school performance), refusing to home-learn, unkempt appearance, and
more importantly conversations with an imaginary individual that is a strong pointer towards
hallucinations. These features are characteristic of schizophrenia as the most likely diagnosis.

To establish the diagnosis of schizophrenia the following criteria should be met:

Two (or more) of the following symptoms:

Hallucinations
Delusions
Disorganized speech
Grossly disorganized or catatonic behavior
Negative symptoms

AND

The symptoms are severe enough to cause significant social or occupation disability

AND

The symptoms have lasted for at least 6 months

AND

Other psychotic conditions such as schizophreniform disorder, brief psychotic episode, etc.
are excluded
1500 of 1943
AND

Drugs or medical conditions as the cause of symptoms are excluded

AND

The condition could not be solely attributed to pervasive developmental disorders such as
autistic disorders

This young man's conversations with imaginary represents hallucinations. His lack of motivation,
isolation, and poor self-care are examples of negative symptoms for 2 years or more. He has been
like this for more than 6 months now and there is marked social or occupational dysfunction as
evidenced by his poor school performance. These fulfill the criteria for schizophrenia as the
diagnosis.

(Option A) Major depressive disorder can present with psychotic symptoms occurring in the
context of a major depressive episode. The patient's history does not suggest major depression.
He has had 2 years of negative symptoms followed by positive symptoms, suggesting
schizophrenia rather than major depression.

(Option B) A medical or substance-related condition is always among the differential diagnoses in


patients with psychotic features. There is no hint about a medical condition as the underlying
cause in the scenario, and it is unlikely that substance abuse of a teenager who lives with his
parents goes unnoticed for 2 years.

(Option D) Brief psychotic disorder can share many features with schizophrenia. The distinctive
feature; however, is the symptoms duration of less than one month with eventual return to
premorbid functioning.

(Option E) Although schizophreniform disorder shares many features with schizophrenia, the
symptoms last no longer than 6 months in schizophreniform disorder.

References

• RANZCP - clinical practice guidelines for the management of schizophrenia and related disorders

• UpToDate - Schizophrenia in adults: Clinical manifestations, course, assessment, and diagnosis


Last updated:
Time spent: QID:857
2023-2-12

1501 of 1943
A 50-year-old man presents to your practice, significantly concerned about amnesia. He relates that
he has been to a place for work for the first time with one of his colleagues. He thought that this is
his first time he had ever visited that place but his colleague was surprised and told him that he
had visited this place before and wondered why he could not remember. His past medical history is
significant for electroconvulsive therapy (ECT) sessions due to major depression with last session
being 3 months ago. Which one of the following in the history best justifies his presenting
complaint?

A. History of dementia in his cousin.


B. History of depression.
C. Previous ECT sessions.
D. De javu.
E. Relapse of his major depression.

Incorrect. Correct answer is C


45% answered correctly

Explanation:

Correct Answer Is C

Of the given options, previous ECT sessions are the most appropriate and justifying explanation to
this man’s current problem that is forgetting past events i.e. retrograde amnesia.

Electroconvulsive therapy (ECT) is a treatment option for several psychiatric conditions. Major
depression associated with psychotic features, strong suicidal ideation, unresponsiveness to other
therapies and previous favorable response to ECT are the most frequent indications.

Most patients report some adverse cognitive effects during and after a course of ECT. The
incidence depends upon electrode placement, stimulus type and dose, anesthesia, and the
patient's pretreatment cognitive status. A systematic review found four studies in which the
proportion of patients who reported any memory loss ranged from 51 -79%.

ECT can cause tree types of cognitive impairment:

Acute confusion - the acute confusional state is the result of both the seizure and the anesthesia. It
typically resolves 10 to 30 minutes after the procedure.

Anterograde amnesia - anterograde amnesia is the decreased ability to retain newly acquired
information. It occurs during a course of ECT and typically resolves within two weeks after
completing the course.

Retrograde amnesia - retrograde amnesia is defined as forgetting recent memories and is the most
anxiety-producing and frightening cognitive effect of ECT. The affected memories are for events
that occur during the course of ECT and a period of weeks to a few months prior to that. The
deficits are greatest and most persistent for knowledge about public or world events compared
with knowledge about the self (personal memory). Bilateral ECT causes more retrograde amnesia
than right unilateral. Retrograde amnesia recovers more slowly than anterograde amnesia. Some of

1502 of 1943
the lost memories of events prior to the course of ECT may be expected to return, while others may
not.

(Option A) Although dementia starts by disorientation, typically first to places, this man is young for
dementia, and while dementia in a first-degree relative can be risk factor, a demented cousin does
not appear to be of importance.

(Options B and E) Neither the history of depression in itself, nor a relapse of depressive episode
justifies dementia.

(Option D) In Déjà vu is the sense familiarity with unfamiliar things. For example the patient feels
that he/she has been in place before while he/she has not. This is not the case here.

References

• SAn Health - Electroconvulsive Therapy Policy Guideline

• Overview of electroconvulsive therapy (ECT) for adults

• NSW Health - Electroconvulsive Therapy: ECT Minimum Standard of Practice in NSW


Last updated:
Time spent: QID:877
2023-2-12

1503 of 1943
Which one of the following is not considered a basic investigation for patients with anorexia
nervosa?

A. Electrolytes.
B. Urinalysis.
C. Liver function tests.
D. DXA scan.
E. Electrocardiogram (ECG).

Incorrect. Correct answer is D


45% answered correctly

Explanation:

Correct Answer Is D

Anorexia nervosa is characterized by failure to maintain a normal body weight, fear of weight gain
and preoccupation with a distorted body image and unrealistic self-evaluation as overweight. There
are two types: (1) restrictive and (2) binge eating/purging.

In restrictive type, the patient strictly reduces food intake and maintains diets of low-calorie foods.
In purging type, weight loss is achieved through vomiting, laxatives, diuretics, or enemas.

Clinical manifestations of anorexia nervosa are mostly related to prolonged starvation and
malnutrition, and include:

Hypothermia
Acrocyanosis
Resting bradycardia (resting heart rate often 40-49 beats per minute)
Hypotension
Orthostatic lowered blood pressure or pulse
Loss of muscle mass
Low blood glucose (impaired insulin clearance)
Low parathyroid hormone levels
Elevated liver function tests
Low white blood cell (WBC) count

Since eating disorders are clinical diagnoses, no definitive diagnostic tests are available for
anorexia nervosa; however, with negative impacts of starvation on different multiple organ
systems, the following basic tests should be performed for every patient with anorexia nervosa as
baseline tests:

Physical and mental status evaluation


Full blood exam (FBE) and ESR
Metabolic panel (Na, K, Bun, urea, Cl, BS, HCO3)
Urinalysis for ketones and specific gravity
Liver function tests
Pregnancy test (in patients of childbearing age)

1504 of 1943
Both dehydration and increased fluid intake can occur in anorexia nervosa, leading to electrolytes
imbalances with negative impacts on cardiac conduction system, central nervous system, etc.
Therefore, electrolyte assessment (option A) should be considered in all patients with anorexia
nervosa and bulimia nervosa.

Cardiac problems is the leading cause of death in patients with anorexia nervosa and an ECG
(option E) should be obtained in all such patients as a very important basic assessment. In
anorexic patients with BMI<14kg/m2, an echocardiography is mandated.

Urinalysis (option B) is another component of basic assessment. Specific gravity reflects hydration
status, and ketonuria indicates starvation.

Prolonged starvation has deleterious effects on liver function; hence, liver function tests (option C)
are indicated as basic investigations in all patients with anorexia nervosa.

Patients with anorexia nervosa and bulimia nervosa may have central hypogonadism and estrogen
deficiency. Estrogen deficiency can result in osteopenia and osteoporosis. A Bones scan (DXA
scan) is indicated for all patients with anorexia nervosa of longer than 6 months duration; this test
however is not among base line investigations. It can be considered later in the course of
management.

References

• RACGP - GP Learning - Eating disorders: a quick guide

• Medscape - Anorexia Nervosa

• UpToDate - Anorexia nervosa in adults and adolescents: Medical complications and their
management
Last updated:
Time spent: QID:886
2023-2-12

1505 of 1943
Which one of the following features does not help to differentiate mania from hypomania?

A. Delusions.
B. Hallucinations.
C. Functional impairment.
D. Need for hospitalization.
E. Elevated mood.

Incorrect. Correct answer is E


45% answered correctly

Explanation:

Correct Answer Is E

Features of mania versus hypomania is summarized in the following table:

Mania Hypomania
Duration of symptoms at least 7 days Duration of symptoms at least 4 days
Marked functional impairment No functional impairment
Delusions and hallucinations present Delusions and hallucinations absent
Hospitalization is often required Hospitalization usually not required

Elevated mood is seen both in mania and hypomania and does not help to differentiate between
these two.

Mania versus hypomania is the parameter by which bipolar disorder is classified into I and II.
Patients with mania are calssified as bipolar I disorder, whereas those with hypomania are labeled
as having bipolar II disorder.

1506 of 1943
A 17-year-old girl has bizarre eating behavior noted by her mother. She indulges in episodes of
binge eating at least once or twice a week. She spends several hours a day doing excessive
workouts, and follows a restrict low-calorie diet because she believes she is fat. She also spends
considerable amount of time in front of mirror checking her body. On examination, she has a BMI of
19 kg/m2. Which one of the following could be the most likely predisposing factor to this
presentation?

A. Alcohol abuse.
B. Childhood sexual abuse.
C. Family history of obesity.
D. Family history of athleticism.
E. History of substance abuse.

Incorrect. Correct answer is B


45% answered correctly

Explanation:

Correct Answer Is B

The case scenario describes a typical case of bulimia nervosa, indicated by binge eating,
compensatory behavior to lose weight, and normal BMI that differentiates it from binging/purging
type of anorexia nervosa.

There is no consensus on the causes of eating disorders. A combination of genetic, biologic,


psychological, family, environmental, and social factors probably contribute to developing an eating
disorder.

Factors associated with the development of eating disorders include:

Genetics – genetic factors are involved in the pathogenesis of eating disorders. As an


example, concordance for either anorexia nervosa or bulimia nervosa is greater in
monozygotic twins compared with dizygotic twins.

Family distress – family characteristics associated with eating disorders may include high
perceived parental expectations for achievement and appearance, families who communicate
poorly, have members who are enmeshed with or estranged from each other, devalue the
mother or maternal role, have marital tension, or have difficulty managing conflicts.

Sexual abuse

History of dieting​

Preoccupation with a thin body and social pressure about weight

Athletic and artistic endeavors such as running, ballet, etc. that emphasizes leanness, and
sports in which scoring is partly subjective (e.g., skating or gymnastics)

Of the given options, sexual abuse is the most important risk factor for development of eating
disorders, as well as other psychiatric conditions such as substance abuse, depressive disorders,
1507 of 1943
and anxiety disorders.

Sexual abuse can have many different effects on eating habits and body image of survivors. Sexual
abuse violates the boundaries of the self so dramatically that inner sensations of hunger, fatigue,
or sexuality become difficult to identify. Victims of sexual abuse may turn to food to relieve a wide
range of different states of tension unrelated to hunger. It is their confusion and uncertainty about
their inner perceptions that leads them to focus on the food.

Many survivors of sexual abuse often work to become very fat or very thin in an attempt to render
themselves unattractive. In this way, they try to de-sexualize themselves. Other survivors
obsessively diet, starve, or purge to make their bodies “perfect.” A perfect body is their attempt to
feel more powerful, invulnerable, and in control, so as not to re-experience the powerlessness they
felt as children.

Several studies have found that childhood trauma such as childhood sexual abuse is a risk factor
for onset of bulimia nervosa as well as discontinuation of psychotherapy for bulimia nervosa; In
addition, a history of childhood maltreatment in patients with bulimia nervosa is associated with
comorbid depressive disorders, anxiety disorders, and borderline personality disorder.

In all eating disorders, there is an increased genetic heritability and frequency of a family history. A
family history of ‘leanness’ or ‘thinness’ or 'elite athleticism' (option D) may be associated with
anorexia nervosa, whereas bulimic eating disorders are associated with a personal or family history
of obesity (option C). Although these risk factors are specific to anorexia nervosa and bulimia
nervosa, they are less likely to trigger the eating disorder compared to sexual child abuse.

Substance abuse (option E) for weight reduction has been frequently observed among patients
with anorexia nervosa and bulimia nervosa as a consequent of the disorder, but not as an
underlying etiology.

Alcohol abuse (option A) does not seem to increase the risk of developing anorexia/bulimia
nervosa.

References

• RACGP - GP Learning - Eating disorders: a quick guide

• PubMed - Childhood sexual abuse and eating disorders in females: findings from the Victorian
Adolescent Health Cohort Study.

• UpToDate - Bulimia nervosa in adults: Clinical features, course of illness, assessment, and
diagnosis
Last updated:
Time spent: QID:888
2023-2-12

1508 of 1943
A 17-year-old girl has bizarre eating behavior noted by her mother. She indulges in episodes of
binge eating at least once or twice a week. She spends several hours a day doing excessive
workouts, and follows a restrict low-calorie diet because she believes she is fat. She also spends
considerable amount of time in front of mirror checking her body. On examination, she has a BMI of
19 kg/m2. Which one of the following could be the most likely predisposing factor to this
presentation?

A. Alcohol abuse.
B. Childhood sexual abuse.
C. Family history of obesity.
D. Family history of athleticism.
E. History of substance abuse.

Incorrect. Correct answer is B


45% answered correctly

Explanation:

Correct Answer Is B

The case scenario describes a typical case of bulimia nervosa, indicated by binge eating,
compensatory behavior to lose weight, and normal BMI that differentiates it from binging/purging
type of anorexia nervosa.

There is no consensus on the causes of eating disorders. A combination of genetic, biologic,


psychological, family, environmental, and social factors probably contribute to developing an eating
disorder.

Factors associated with the development of eating disorders include:

Genetics – genetic factors are involved in the pathogenesis of eating disorders. As an


example, concordance for either anorexia nervosa or bulimia nervosa is greater in
monozygotic twins compared with dizygotic twins.

Family distress – family characteristics associated with eating disorders may include high
perceived parental expectations for achievement and appearance, families who communicate
poorly, have members who are enmeshed with or estranged from each other, devalue the
mother or maternal role, have marital tension, or have difficulty managing conflicts.

Sexual abuse

History of dieting​

Preoccupation with a thin body and social pressure about weight

Athletic and artistic endeavors such as running, ballet, etc. that emphasizes leanness, and
sports in which scoring is partly subjective (e.g., skating or gymnastics)

Of the given options, sexual abuse is the most important risk factor for development of eating
disorders, as well as other psychiatric conditions such as substance abuse, depressive disorders,
1509 of 1943
and anxiety disorders.

Sexual abuse can have many different effects on eating habits and body image of survivors. Sexual
abuse violates the boundaries of the self so dramatically that inner sensations of hunger, fatigue,
or sexuality become difficult to identify. Victims of sexual abuse may turn to food to relieve a wide
range of different states of tension unrelated to hunger. It is their confusion and uncertainty about
their inner perceptions that leads them to focus on the food.

Many survivors of sexual abuse often work to become very fat or very thin in an attempt to render
themselves unattractive. In this way, they try to de-sexualize themselves. Other survivors
obsessively diet, starve, or purge to make their bodies “perfect.” A perfect body is their attempt to
feel more powerful, invulnerable, and in control, so as not to re-experience the powerlessness they
felt as children.

Several studies have found that childhood trauma such as childhood sexual abuse is a risk factor
for onset of bulimia nervosa as well as discontinuation of psychotherapy for bulimia nervosa; In
addition, a history of childhood maltreatment in patients with bulimia nervosa is associated with
comorbid depressive disorders, anxiety disorders, and borderline personality disorder.

In all eating disorders, there is an increased genetic heritability and frequency of a family history. A
family history of ‘leanness’ or ‘thinness’ or 'elite athleticism' (option D) may be associated with
anorexia nervosa, whereas bulimic eating disorders are associated with a personal or family history
of obesity (option C). Although these risk factors are specific to anorexia nervosa and bulimia
nervosa, they are less likely to trigger the eating disorder compared to sexual child abuse.

Substance abuse (option E) for weight reduction has been frequently observed among patients
with anorexia nervosa and bulimia nervosa as a consequent of the disorder, but not as an
underlying etiology.

Alcohol abuse (option A) does not seem to increase the risk of developing anorexia/bulimia
nervosa.

References

• RACGP - GP Learning - Eating disorders: a quick guide

• PubMed - Childhood sexual abuse and eating disorders in females: findings from the Victorian
Adolescent Health Cohort Study.

• UpToDate - Bulimia nervosa in adults: Clinical features, course of illness, assessment, and
diagnosis
Last updated:
Time spent: QID:888
2023-2-12

1510 of 1943
Which one of the following is the most important risk factor for developing anorexia nervosa?

A. Alcohol abuse.
B. Childhood sexual abuse.
C. Family history of obesity.
D. Family history of athleticism.
E. History of substance abuse.

Incorrect. Correct answer is B


45% answered correctly

Explanation:

Correct Answer Is B

Of the given options, sexual abuse is the most important risk factor for development of bulimia
nervosa, anorexia nervosa and other eating disorders, as well as other psychiatric conditions such
as substance abuse, depressive disorders, and anxiety disorders.

Sexual abuse can have many different effects on the eating habits and body image of survivors.
Sexual abuse violates the boundaries of the self so dramatically that inner sensations of hunger,
fatigue, or sexuality become difficult to identify. Victims of sexual abuse may turn to food to relieve
a wide range of different states of tension unrelated to hunger. It is their confusion and uncertainty
about their inner perceptions that leads them to focus on the food.

Many survivors of sexual abuse often work to become very fat or very thin in an attempt to render
themselves unattractive. In this way, they try to desexualize themselves. Other survivors
obsessively diet, starve, or purge to make their bodies “perfect.” A perfect body is their attempt to
feel more powerful, invulnerable, and in control, so as not to re-experience the powerlessness they
felt as children.

Several studies have found that childhood trauma such as childhood sexual abuse is a risk factor
for onset of bulimia nervosa as well as discontinuation of psychotherapy for bulimia nervosa.
Studies show the connection between childhood sexual abuse and anorexia nervosa is not as
strong as that observed in bulimia nervosa; however, child sexual abuse remains a very important
risk factor for all eating disorders including anorexia nervosa.

Two studies showed that a history of childhood sexual abuse was present in 47-50% of patients
with bulimia nervosa and a less strong connection between childhood sexual abuse and anorexia
nervosa when compared with that of bulimia nervosa (23-27%); In addition, a history of childhood
maltreatment in patients with bulimia nervosa is associated with comorbid depressive disorders,
anxiety disorders, and borderline personality disorder.

In all eating disorders there is an increased genetic heritability and frequency of a family history. A
family history of ‘leanness’ or ‘thinness’ or 'elite athleticism' (option D) may be associated with
anorexia nervosa, whereas bulimic eating disorders are associated with a personal or family history
of obesity (option C). Although these risk factors are specific to anorexia nervosa and bulimia
nervosa, they are less likely to be trigger the eating disorder compared to sexual child abuse.

1511 of 1943
Substance abuse (option E) for weight reduction has been frequently observed among patients
with anorexia nervosa and bulimia nervosa as a consequent of the disorder, but it is not the
underlying etiology.

Alcohol abuse (option A) does not seem to increase the risk of developing anorexia/bulimia
nervosa.

References

• RANZCP - clinical practice guidelines for the treatment of eating disorders

• PubMed - Childhood sexual abuse and eating disorders in females: findings from the Victorian
Adolescent Health Cohort Study
Last updated:
Time spent: QID:889
2023-2-12

1512 of 1943
A 67-year-old woman comes to your GP clinic 6 weeks after her husband passed away due to
prostate cancer. She is sad and tearful and complains that she cannot sleep at night and is also
afraid of the dark. She mentions similar feelings 32 years ago after she had a still birth. Which one
of the following will help with prompt relief of her symptoms?

A. Venlafaxine.
B. Electroconvulsive therapy (ECT).
C. Olanzapine.
D. Temazepam.
E. Citalopram.

Incorrect. Correct answer is D


45% answered correctly

Explanation:

Correct Answer Is D

Given the symptoms and their duration, a normal grief reaction is the most likely explanation to this
woman’s problem.

Sadness, despair, tearfulness, decreased sleep, decreased appetite, and decreased interest in life
and the world are present as common findings in normal grief reaction. Visual and auditory
hallucinations of the deceased person are common and may lead the bereaved person to fear he or
she is ‘going crazy’. Guilt and shame are not common in normal grief reaction, yet possible.
Suicidality is not usually a concern

Patients with normal grief reaction return to normal social functioning within 2 months; the
symptoms, however, might last up to one year with waxing and waning.

Conventional support (e.g., family, social, religious, social) are usually sufficient to help the patient
over her grief. Although there is normally little place for pharmacotherapy, anxiolytics and
hypnotics can provide prompt relief where indicated due to excessive insomnia and anxiety. A
short course of benzodiazepines (< 7 days) can be used to help with the insomnia and anxiety.

The patient has no psychotic feature necessitating antipsychotics such as olanzapine (option C).
Antidepressants such as venlafaxine (option A) and citalopram (option E) might be indicated in
patients with complicated grief or grief associated with major depression, none of which is the
case here.

ECT (option B) might very infrequently be indicated in selected patients with complicated grief
reaction.

References

• UpToDate - Bereavement and grief in adults: Clinical features

• Medscape - What are the DSM-5 criteria for diagnosis of major depressive disorder (clinical
depression)?
1513 of 1943
• RACGP - AFP - Psychological triage in general practice
Last updated:
Time spent: QID:891
2023-2-12

1514 of 1943
A 32-year-old woman presents to your practice with complaint of back pain, felt between her
shoulder blades. Her past medical history is remarkable for anorexia nervosa for 16 years. On
examination, she is severely emaciated, weighs 38 kg and has a body mass index (BMI) of 14
kg/m2. Palpation of the interscapular region elicits tenderness over the vertebral column. Which
one of the following could be the most likely diagnosis?

A. Vertebral compression fracture.


B. Pneumonia.
C. Pulmonary embolism.
D. Pericardial effusion.
E. Metastatic bone lesion.

Correct
45% answered correctly

Explanation:

Correct Answer Is A

Patients with longstanding anorexia nervosa (>6 months) are at significantly increased risk of
pathological fractures due to osteopenia and osteoporosis caused by malnutrition and estrogen
deficiency.

Osteopenia and osteoporosis is a complication of long-standing anorexia nervosa. These changes


are often treated once anorexia nervosa are successfully treated and adequate weight gain is
obtained. Vertebral column is the most common site of pathologic fractures due to osteoporosis.

In this patient with long-standing anorexia nervosa, the pain and tenderness over thoracic vertebrae
makes compression fracture the most likely diagnosis.

Pneumonia (option B), pulmonary embolism (option C), and pericardial effusion (option D) may be
associated with referred pain to the back, but there is be no tenderness.

Metastatic bone lesions (option E) can cause pathologic fractures; however, with the presence of
anorexia nervosa as a significant risk factor on one hand, and no pointers towards an underlying
primary malignancy on the other, metastatic bone disease is less likely.

References

• Eating Disorders: Treatment and management of anorexia nervosa


Last updated:
Time spent: QID:892
2023-2-12

1515 of 1943
Which one of the following is a common feature seen in anorexia nervosa?

A. Bradycardia.
B. Normally-developed secondary sexual characteristics.
C. Calloused knuckles.
D. Hirsutism.
E. Heat intolerance.

Correct
45% answered correctly

Explanation:

Correct Answer Is A

Of the options, bradycardia is the most common finding in patient with anorexia nervosa.

(Option B) Sexual characteristics in patients with anorexia nervosa are often absent or
underdeveloped depending on time of the onset of the disease.

(Option C) Calloused knuckles are seen in patients with binge eating and purging subtype of
anorexia nervosa where vomiting induction frequently occurs as a compensatory means. It is not
as common as bradycardia.

(Option D) Hirsutism is not a feature of anorexia nervosa; rather such patients have lanugo (the fine
thin hair covering the body) which is different from hirsutism.

(Option E) Cold intolerance (not heat tolerance) is a feature of anorexia nervosa; however, shivering
in the cold is not seen).

TOPIC REVIEW

Different complications associated with anorexia nervosa are as follows:

Endocrine/metabolic

Menstrual dysfunction
Delayed menarche/ secondary amenorrhea
Delayed growth and development
Regression of secondary sexual characteristics
Decreased metabolic rate and decreased resting energy expenditure
Appetite and thirst dysregulation
Low parathyroid hormone levels
Elevated liver function
Anemia
Hypoglycemia
Decreased white cell count

1516 of 1943
Protein-calorie malnutrition
Vitamin, mineral and essential fatty acids deficiencies
Ketonuria
Cold intolerance
Depressed immune function
Atrophied breasts

Musculoskeletal

Decreased bone mass


Osteopenia
Stress fractures
Decreased muscle mass

Cardiovascular

Cardiac arrhythmias
ECG abnormalities
Bradycardia (HR<60bpm)
Hypotension
Orthostatic hypotension
Decreased left ventricular mass and stroke volume
Congestive heart failure
Cardiac arrest
Peripheral edema

Gastrointestinal

Parotid and submandibular gland hypertrophy (usually in binge eating/purging type)


Delayed gastric emptying
Constipation/obstipation
Hypoactive bowel sounds
Postprandial discomfort, bloating after small meals

Psychoneurological

Depression
Anxiety
Structural brain abnormalities/brain tissue loss
Impaired concentration
Lack of insight
Sleep disturbances

Dermatological / other

Acrocyanosis
Lanugo – fine soft hair
Xerosis (dry, scaly skin)
Brittle hair
Hair loss
Calloused knuckles (in binge eating/purging type where vomiting induction occurs as a
compensatory behavior)
Eroded enamel (in binge eating/purging type)
1517 of 1943
Negative signs

Normal fundi or visual fields


No organomegaly
No lymphadenopathy

References

• Medscape - Anorexia Nervosa

• UpToDate - Anorexia nervosa in adults: Clinical features, course of illness, assessment, and
diagnosis
Last updated:
Time spent: QID:893
2023-2-12

1518 of 1943
A 19-year-old woman with a two-year history of anorexia nervosa is brought for assessment. She is
extremely thin, weighs 37.5 kg and has a body mass index of 15 kg/m2. Her menses started at 14
years of age, but stopped one year ago. Which one of the following investigations is not indicated
in this patient?

A. Liver function tests (LFTs).


B. Electrocardiogram.
C. Serum gonadotropins (FSH and LH).
D. Electrolytes.
E. DXA scan.

Incorrect. Correct answer is C


45% answered correctly

Explanation:

Correct Answer Is C

The following are the investigations to consider in every patients presenting with anorexia
nervosa:

Serum electrolytes (option D)


Blood urea nitrogen
Serum creatinine
Serum glucose
Serum calcium, phosphorous, and magnesium
Thiamine
Serum albumin
Liver function tests (option A) (aspartate aminotransferase, alanine aminotransferase, and
alkaline phosphatase)
Internationalized Normalized Ratio (INR)
Complete blood count (CBC) including differential
Erythrocyte sedimentation rate
Thyroid stimulating hormone (TSH)
Electrocardiogram (ECG) (option B)
Urinalysis for specific gravity

A dual energy X-ray absorptiometry (DXA) (option E) scan is also indicated for all patients with
anorexia nervosa of longer than 6 months duration. For this patient with 2-year history of anorexia
nervosa, a DXA scan should be performed to assess the presence of osteopenia or osteoporosis.

An echocardiography should be performed on all patients with anorexia nervosa with body mass
index (BMI) of less than 14 kg/m2.

FSH and LH are not diagnostically valuable where diagnosis of anorexia nervosa is clear based on
the presence of emaciation in conjunction with self-induced starvation and intense fear of weight
gain. In such cases, amenorrhea is the result of the underlying malnutrition and estrogen deficiency
(hypopituitary hypogonadism).

1519 of 1943
Measurement of FSH/LH is only indicated if the diagnosis of anorexia nervosa cannot be clinched
through physical exams and history and other cause of amenorrhea such as ovarian failure have
been considered.

NOTE - If the diagnosis of anorexia nervosa is not clear, other causes that can present with
weight loss, malabsorption, or amenorrhea should be investigated. The following conditions are
possible causes to consider:

Neoplasm
Chronic infections (e.g. tuberculosis or acquired immunodeficiency syndrome)
Uncontrolled diabetes mellitus
Hyperthyroidism
Malabsorption syndromes (e.g., celiac disease)
Inflammatory bowel disease
Pregnancy
Primary ovarian failure
Polycystic ovarian syndrome
Pituitary prolactinoma

References

• RANZCP - Clinical practice guidelines for the treatment of eating disorders

• UpToDate - Anorexia nervosa in adults: Clinical features, course of illness, assessment, and
diagnosis
Last updated:
Time spent: QID:894
2023-2-12

1520 of 1943
A 57-year-old man, who is a chronic alcohol user, is admitted to the hospital after he was found out
to have developed chronic liver disease. Three days after the admission, he progressively becomes
aggressive and starts to shout at the ward staff and threatening them. Which one of the following
is the most appropriate medication to give him?

A. Diazepam.
B. Lorazepam.
C. Haloperidol.
D. Olanzapine.
E. Droperidol.

Incorrect. Correct answer is B


45% answered correctly

Explanation:

Correct Answer Is B

With chronic alcohol use in this patient, the presentation is most likely due to alcohol withdrawal
syndrome (AWS). AWS is characterized by a range of signs and symptoms including tremor,
sweating, nausea and vomiting, anxiety, agitation, headache, diarrhea, tachycardia, delirium,
perceptual disturbances, tachycardia and hypertension. Seizures occasionally occur. In 50%, there
is only one episode of seizure.

Clinical presentation begins within the first 6 to 24 hours after last ingestion and typically persists
72 hours, but may last for weeks.When medications are indicated for treatment of AWS,
benzodiazepines are first-line. Diazepam 10-20 mg is given orally every 2 hours until symptoms
subside or the maximum daily dose of 120mg is reached. Diazepam can be iven on subsequent
days if symptoms persist.

In patients with severe liver disease, however, a short-acting benzodiazepine without active
metabolite such as lorazepam or oxazepam should replace diazepam.

Diazepam is first metabolized by hepatic oxidation resulting in production of active metabolite. The
drug then undergoes glucuronidation. In the elderly and patients with liver disease, benzodiazepine
oxidation is decreased and can results in accumulation of the drug, resulting in excessive sedation
and respiratory suppression.

Lorazepam and oxazepam metabolism is minimally affected by age and liver disease; therefore,
these drugs appear to be safest choices among the various benzodiazepines for treating AWS in
the elderly and patients with liver disease.

Haloperidol and droperidol are used when psychotic features or agitation persists despite
treatment with benzodiazepine.

NOTE – Benzodiazepines and, to a lesser extent, haloperidol and droperidol, may worsen the
symptoms of hepatic encephalopathy.

References

1521 of 1943
• http://www.ncbi.nlm.nih.gov/pubmed/8700792

• Therapeutic Guidelines – Psychotropics; available from http://tg.org.au


Last updated:
Time spent: QID:903
2023-2-12

1522 of 1943
A 73-year-old woman with end-stage renal disease (ESRD), who has been well-controlled on routine
peritoneal dialysis, and has been coping well thus far, suddenly declines to have her dialysis
session today. On examination, she is found to have a temperature of 38.3°C and mild tenderness
in upper abdomen. Which one of the following could be the most likely additional finding in this
patient?

A. Disorientation.
B. Depressed mood.
C. Blunted affect.
D. Delusions.
E. Hypervigilance.

Correct
45% answered correctly

Explanation:

Correct Answer Is A

Depression is common among patients with ESRD and on dialysis. It is a major cause of treatment
withdrawal in such patients. However, sudden decline associated with findings of fever and
abdominal tenderness are against depression as the cause of treatment refusal in this patient.

This patient’s fever is a pointer towards infection, as is the abdominal tenderness. Infections may
cause delirium, especially in aging people. For delirium to be the underlying cause to this
presentation, disorientation should be present on examination.

Depressed mood (option B) is an unlikely finding in a patient who has been coping well so far, but
all of the sudden refuses treatment. Blunted affect (option C), delusions (option D) and
hypervigilance (option E) would have been expected in psychotic mental disorders. Such disorders
usually have a more insidious onset making them less likely explanations in this scenario.

References

• UpToDate - Psychiatric illness in dialysis patients

• Indian Journal of Nephrology - Psychiatric issues in renal failure and dialysis


Last updated:
Time spent: QID:915
2023-2-12

1523 of 1943
A 37-year-old man presents to your practice with complaint of erectile dysfunction. A few months
ago, he found out that her wife had an extramarital affair and developed low mood. After 5 weeks,
he went to see a doctor and was diagnosed with depression and prescribed sertraline. Recently, he
has started a new relationship but is experiencing erectile dysfunction. Which one of the following
would be the most appropriate option for management of his current problem?

A. Stop sertraline.
B. Switch to fluoxetine.
C. Switch to fluvoxamine.
D. Prescribe sildenafil.
E. Prescribe clomipramine.

Incorrect. Correct answer is D


45% answered correctly

Explanation:

Correct Answer Is D

Many drugs are capable of producing sexual dysfunction. The reported sexual issues by the
patients include: decreased libido, erectile or ejaculatory problems, and decreased lubrication in
women.

The most important drug classes associated with such problems are: antihypertensives,
antipsychotics, antidepressants and recreational drugs.

Selective serotonin reuptake inhibitors (SSRIs) such as sertraline, fluoxetine, paroxetine,


fluvoxamine, citalopram, etc are all well-known to be associated with sexual dysfunction as one of
their most prominent adverse effects that could lead to noncompliance.

The reported sexual adverse outcomes of SSRIs are:

Decreased libido
Delayed ejaculation (this effect has been used for treatment of premature ejaculation)
Erectile dysfunction
Painful ejaculation

The issue of sexual dysfunction associated with antidepressants is complex. The problem may be
caused by other factors than the medication, while strongly believed otherwise by the patient.
There might be an unrecognizable interaction between the drug and psychological effects. Several
strategies approaches have been proposed

Where the medication is considered drug-induced, several strategies might be tried to reverse the
condition including:

Dose reduction
Switching to another drug from a different drug group
Drug holidays

1524 of 1943
However, prescribing a phosphodiesterase type 5 inhibitor such as sildenafil or tadalafil in
anticipation of intercourse has become the standard of care for men. In women, these drugs have
shown some benefits such as increased lubrication.

Switching to other SSRIs such as fluoxetine, fluvoxamine, paroxetine, etc is unlikely to resolve the
problem because these drugs are associated with sexual dysfunction as well.

Low-dose clomipramine has proven effective in treatment of premature ejaculation. Recent studies
suggest that it could have an effect in delayed ejaculation; however, sildenafil remains the standard
of care in most patients.

NOTE - For patients with depression who develop sexual side effects while on SSRIs, switching to
nafazodone, bupropion, or mirtazapine is an alternative. These drugs have minimal, if any, impact
on sexual function.

References

• PubMed - Sildenafil in the Treatment of SSRI-Induced Sexual Dysfunction: A Pilot Study

• PubMed - Antidepressant-associated sexual dysfunction: impact, effects, and treatment

• RACGP - Much more than prescribing a pill – Assessment and treatment of erectile dysfunction
by the general practitioner
Last updated:
Time spent: QID:931
2023-2-12

1525 of 1943
A 48-year-old man presents with complaint of significantly delayed ejaculation for the past 2
months. He has the history of major depression, for which he is on sertraline. Four months ago, the
dose of sertraline was increased to 100mg, daily, after he felt that his previously well controlled
symptoms are returning. At present, he is stable and symptom free. Which one of the following
would be the most appropriate management option regarding his ejaculation problem?

A. Switch to fluoxetine.
B. Stop sertraline.
C. Advise sildenafil in anticipation of intercourse.
D. Reduce the dose of sertraline.
E. Prescribe clomipramine.

Incorrect. Correct answer is C


45% answered correctly

Explanation:

Correct Answer Is C

Selective serotonin reuptake inhibitors (SSRIs) such as sertraline, fluoxetine, paroxetine,


fluvoxamine, citalopram, etc are all well-known to be associated with sexual dysfunction as one of
their most prominent adverse effects that could lead to noncompliance to the treatment from the
patients.

The reported sexual adverse outcomes of SSRIs are:

Decreased libido
Delayed ejaculation (this effect has been used for treatment of premature ejaculation)
Erectile dysfunction
Painful ejaculation

Strategies have been proposed for dealing with sexual adverse effects of SSRIs, including:

Drug holidays
Dose reduction
Switching to other antidepressant with minimal or no impact on sexual functioning such as
bupropion, mirtazapine, or nafazodone
Use of phosphodiesterase type 5 inhibitors such as sildenafil or tadalafil
Cognitive behavioral therapy

Of these options, however, using a phosphodiesterase type 5 inhibitor such as sildenafil before
intercourse has become the standard of care.

(Option A) Switching to other SSRIs such as fluoxetine, fluvoxamine, paroxetine, etc is unlikely to
resolve the problem because these drugs are associated with sexual dysfunction as well.

(Option B) Cessation of sertraline will result in relapse of the recently-controlled depression.

(Option D) Decreasing the current dose of sertraline in a patient who has just responded the
incremental dose increase due to uncontrolled depressive symptoms is very likely to result in
1526 of 1943
relapse; therefore, not an appropriate option.

(Option E) Clomipramine is a tricyclic antidepressant used for treatment of premature ejaculation.


Addition of clomipramine will deteriorate delayed ejaculation.

References

• http://www.australianprescriber.com/magazine/36/2/

• http://www.ncbi.nlm.nih.gov/pmc/articles/PMC181091

• http://www.ncbi.nlm.nih.gov/pmc/articles/PMC310869
Last updated:
Time spent: QID:932
2023-2-12

1527 of 1943
A 24-year-old woman presents with elevated mood, pressured speech, agitation and flight of ideas
over the past 2 weeks. There is no history of drug abuse in the past. The patient is fully
cooperative. Which one of the following is the drug of choice in this situation?

A. Sodium valproate.
B. Lithium.
C. Carbamazepine.
D. Haloperidol.
E. Quetiapine.

Incorrect. Correct answer is B


45% answered correctly

Explanation:

Correct Answer Is B

The clinical history of this patient is consistent with acute mania, probably due to bipolar I disorder.

Bipolar I disorder is a mood disturbance characterized by manic symptoms of at least one week
duration. The symptoms should be significant enough to cause considerable impairment or
distress in the level of functioning.

Characteristic features of bipolar disorder include:

Persistent abnormally elevated mood lasting for at least one week


Increased self-esteem and grandiosity
Distractibility
Excessive involvement in activities
More talkative than usual – pressured speech is often noticed on examination
Psychomotor agitation
Flights of ideas
Increased sexual activity
Increase in goal-directed activities

Of the given options, lithium is the drug of choice for both acute treatment and maintenace therapy.
Antipsychotics such as olanzapine, quetiapine (option E), risperidone and aripiprazole can be used
in combination with lithium or alone for acute mania. Sodium valproate (option A) in combination
with atypical antipsychotics is an alternative in specific patients.

Haloperidole (option D) is used for patients with acute manic features, who need sedation. If the
patient is not cooperative and acutely agitated, intramuscular haloperidol would be the best initial
management. Carbamazepine (option C) and some other anticonvulsants has been used as a
mood-stabilzer but evidence shows it is less effective than lithium or atypical antipsychotics in
management of acute mania in bipolar disorder.

References

• International Journal of Bipolar Disorder


1528 of 1943
• Therapeutic Guidelines - Psychotropic; available from http://tg.org.au
Last updated:
Time spent: QID:66
2023-2-12

1529 of 1943
A 32-year-old is brought to the emergency department after she was arrested by the police in a
shopping mall. She is dressed in colorful dresses. According to the police, she was shouting at
everyone in the mall. She was claiming in an excited manner that her mother was virgin and
married God. In the emergency department and during the interview she keeps saying that she
hears the voice of God, and that she is planning to become pregnant from him. Which one of the
following is the most likely cause to her presentation?

A. Antisocial personality disorder.


B. Opium abuse.
C. Alcohol intoxication.
D. Schizophrenia.
E. Borderline personality disorder.

Incorrect. Correct answer is D


45% answered correctly

Explanation:

Correct Answer Is D

Two points are of diagnostic significance in this case scenario. First is the delusional thought
content evident by strongly-held bizarre false beliefs that the patient’s mother married God, and
that she is planning to be conceived by him. Second is the presence of auditory hallucinations.
These two together makes a psychotic episode the most likely diagnosis. Of the given option, the
only one that can represent these is schizophrenia.

(Option A) Antisocial personality disorder is characterized by continuous antisocial or criminal


acts, inability to conform to social rules, impulsivity, disregard for the rights of others,
aggressiveness, lack of remorse, and deceitfulness. There are no delusions or hallucinations,
unless caused by another reason.

(Option B) Opium abuse is not associated with delusions or hallucinations.

(Option C) Acute alcohol intoxication presents varying manifestations depending on the severity,
and can include slurred speech, nystagmus, disinhibited behavior, incoordination, unsteady gait,
memory impairment, stupor or even coma. Hallucinations and delusions are not typical
presentations.

(Option E) Borderline personality disorder is often characterised by an unstable affect, mood


swings, marked impulsivity, unstable relationships, recurrent suicidal behaviours and self-inflicted
injures, chronic feelings of emptiness and boredom, identity disturbances, and inappropriate anger.
If they are stressed they may become psychotic. They use splitting as their main defence
mechanism.

References

• http://www.uptodate.com/contents/schizophrenia-cli

• http://psychcentral.com/disorders/borderline-perso
1530 of 1943
• http://psychcentral.com/disorders/antisocial-perso
Last updated:
Time spent: QID:941
2023-2-12

1531 of 1943
A nurse calls you to see a 33-year-old woman, who has just been admitted to the general ward and
apparently is making trouble by arguing with the nursing staff. When you step into her room and
she sees you, she says seductively: ‘Oh, my lovely doctor. I am sure you are a gentleman and
understand my needs. I want a private room with classy and beautiful furniture. I can’t eat hospital
foods because they are rubbish. I need to order French dishes as they are my favorite. I need
access to my makeup artist, and I want my little dog here for comfort. I am sure you are brilliant
and know what I am talking about. You are the one who understands me. These idiot nurses don’t’.
Which of the following conditions is most likely to be present in this woman?

A. Narcissistic personality disorder.


B. Histrionic personality disorder.
C. Schizophrenia.
D. Hallucinations.
E. Schizotypal personality disorder.

Correct
45% answered correctly

Explanation:

Correct Answer Is B

The behavioral pattern in this woman is most likely due to histrionic personality disorder.
Remarkable findings are seductiveness and trying to be charming, exaggeration in paying
compliments to you as a person she has met for the first time, and self-centeredness and self-
dramatization.

Individuals with histrionic personality disorder exhibit emotionalism, which is a tendency to regard
things emotionally. They are attention seekers and uncomfortable in situations where they are not
the center of attention. Behaviors may include constant seeking of approval or attention, self-
dramatization, theatricality (acting out as if they are on the theatre stage), and striking self-
centeredness or sexual seductiveness in inappropriate situations including social, occupational,
and professional relationships beyond what is appropriate for the social context. They may be lively
and dramatic and initially charm new acquaintances with their passion, enthusiasm, apparent
openness, or flirtatiousness. Personal interests and conversations will be self-focused. They use
physical appearance to draw attention to themselves. Emotional expression may be shallow and
rapidly shifting. Their style of speech is excessively impressionistic and lacking in detail.

This patient is also using a splitting defense mechanism by seeing all white or black. Although
splitting is commonly seen in individuals with borderline personality disorder, other behavioral
findings are not consistent with the diagnosis.

(Option A) This patient is unlikely to have a narcissistic personality disorder. Although obvious self-
centeredness in this patient may resemble grandiosity, the characteristic feature in narcissistic
personality disorder, this personality disorder is associated with condescending behavior, while this
woman is trying to be in the center of attention and charming at the same time. This makes
histrionic personality disorder more likely.

1532 of 1943
(Option D) Patients with personality disorders do not hallucinate. No hallucinations are evident in
the presentation.

(Option C) Schizophrenia is characterized by bizarre delusions and hallucinations, mostly auditory,


none of which are present in this case.

(Option E) A schizotypal personality disorder is characterized by significant difficulty in establishing


and maintaining close relationships with others. A person with a schizotypal personality disorder
may have extreme discomfort with such relationships. Someone with this disorder usually has
cognitive or perceptual distortions as well as eccentricities in their everyday behavior. Individuals
with schizotypal personality disorder often have ideas of reference (e.g., they have incorrect
interpretations of casual incidents and external events as having a particular and unusual meaning
specifically for the person). People with this disorder may be unusually superstitious or
preoccupied with paranormal phenomena outside their subculture's norms.

References

• Psych central - What Is Histrionic Personality Disorder? Symptoms, Treatments, and More

• Psych Central - Narcissistic Personality Disorder: Symptoms, Causes, and Treatment

• Psych Central - Schizotypal Personality Disorder: Symptoms, Causes, and Treatment


Last updated:
Time spent: QID:942
2023-2-12

1533 of 1943
A 27-year-old woman is about to get discharged from the hospital where she was admitted 2
weeks ago due to a psychological breakdown. When you tell her about the discharge plan, she
says: ‘doctor, you cannot discharge me. If you do, I will hurt myself and you will be responsible for
that. Doctor Barkley knows me very well". Which one of the following best describes the underlying
cause of her behavior?

A. Narcissistic personality disorder.


B. Histrionic personality disorder.
C. Borderline personality disorder.
D. Factitious disorder.
E. Malingering.

Incorrect. Correct answer is C


45% answered correctly

Explanation:

Correct Answer Is C

Of the options, borderline personality disorder (BPD) is the best explanation for this scenario. The
excessive fear of being abandoned is one of the key features of BPD. These abandonment fears are
related to intolerance of being alone and the need to have other people around. Frantic efforts are
made to avoid imagined abandonment. Some of these efforts include impulsive actions such as
self-mutilating or suicidal behavior. To this woman, discharge conveys abandonment, and she is
making every effort to prevent it.

(Option A) Narcissistic personality disorder is characterized by a sense of self-importance,


grandiosity, and preoccupation with fantasies of success. Someone with this personality disorder
believes they are special, require excessive admiration, and reacts with anger when criticized. They
lack empathy, are envious of others, and are interpersonally exploitative.

(Option B) Histrionic personality disorder is characterized by colorful, exaggerated behavior and


shallow expression of emotions. People with this disorder use their appearance to draw attention,
are sexually seductive and uncomfortable when they are not the center of attention.

(Option D) Factitious disorder is characterized by the conscious production of signs and symptoms
of medical or mental disorders with the main objective of assuming the sick role and eventually
hospitalization. The motivation behind this behavior is not clear to the patient and occurs at a
subconscious level. This patient is not producing any signs or symptoms.

(Option E) Malingering is also the conscious production of signs and symptoms, but unlike
factitious disorder, a clear gain such as money, avoidance of work, free bed and board, etc. is
intended. Again, this patient is not producing any signs or symptoms.

NOTE – although not an option, dependent personality disorder (DPD) can be another diagnosis.
Individuals with DPD are preoccupied with the need to be taken care of. They have clinging
behavior and are unrealistically worried about abandonment. They feel inadequate and helpless
and avoid disagreements with others. They usually focus dependency on a family member or
spouse and desperately seek substitutes if this person becomes unavailable. Associated
1534 of 1943
features include self-doubt, excessive humility and humbleness, poor independent functioning,
mood disorders, anxiety disorders, adjustment disorder, and other personality disorders.
Patients with dependent personality disorder lack opinion and are distressed when they must
make decisions for themselves. Studies suggest that over 50% of patients with BPD also meet
the criteria for DPD. The prevalence of these comorbidities could owe to the fact that some of
the features of DPD are very similar to the features of BPD. For example, people with BPD
experience rejection sensitivity. They tend to feel desperate at even the slightest perceived
rejection. Individuals with DPD may react similarly to criticism or perceived abandonment by
loved ones.

References

• Pysch Central - Symptoms of Borderline Personality Disorder (BPD)

• Psych Central - Narcissistic Personality Disorder: Symptoms, Causes, and Treatment

• Psych Central - What Is Histrionic Personality Disorder? Symptoms, Treatments, and More

• Very Well Mind - The Link Between Borderline and Dependent Personality Disorders
Last updated:
Time spent: QID:943
2023-2-12

1535 of 1943
A 32-year-old woman has brought her 10-week-old male infant to your attention for consultation for
the fifth time since his birth. She complains that the baby is crying too much, is irritable, and very
hard to console. On examination, he looks well with a weight on 90 percentile. The rest of exam is
unremarkable. Which one of the following is the most important thing to look for in history or
clinical findings?

A. Mood.
B. Marital conflicts with her husband.
C. Weight.
D. Previous attitude.
E. Financial issues.

Correct
45% answered correctly

Explanation:

Correct Answer Is A

One in five Australian mothers of full-term infants suffers from a perinatal mental health disorder
with depression and anxiety being more encountered. Postpartum blues is quite common, but
often has a benign and self-limiting course. Postpartum psychosis and postpartum obsessions are
other conditions encountered. Psychiatric problems pose both physiologic and psychologic risks to
mother and the baby. Therefore, it is of paramount importance to check for the presence of such
conditions in perinatal period.

Depression and anxiety are common in postnatal period and are associated with poor parenting
and attachment and adverse outcomes. Interestingly, many postnatal women will not identify
themselves as depressed or anxious; rather they may visit for “other” reasons such as an
inconsolable or unsettled babies (such as in this case). In such cases, healthcare professionals
such as GPs or Child Health Nurses should routinely inquire about mood, anxiety, and how the
woman is coping, and if deemed necessary, use Edinburgh Postnatal Depression Scale (EPDS)
questionnaire as a screening tool.

Marital conflicts, support from husband or partner, or other issues that might have triggered or
exacerbated such a psychological breakdown should be inquired judiciously if indicated, but
probing for mood problems, anxiety or other psychiatric issues take precedence over such matters.

Previous attitude is another important point to be taken into consideration during evaluation. A
woman may have a previous history of depression or anxiety posing her at greater risk of
recurrences, but more important is her current condition.

Weight loss, anorexia and decreased sleep time are frequently seen in depressed persons. Weight
gain, an increase in appetite and increased sleep time are atypical features in some depressed
patients, the presence of which is associated with an unfavorable outcome. Weight changes may
be seen in depression, but it could be also physiologic or caused by other medical conditions.

NOTE – Frequent visits to the GP or obstetrician can signal underlying depression.

1536 of 1943
Common signs and symptoms of mental health problems which can be used to guide questioning
at clinical interview include the following:

Depressed / irritable / anxious mood most of the day nearly every day
Mood swings
Loss of interest, pleasure or motivation in usual activities
Problems with thinking clearly, concentration, memory, making decisions
Negative thoughts (death, disaster)
Suicidal ideation or plans
Feelings of worthlessness or guilt; low self-esteem
Unusual change in weight or appetite
Significant change in sleep
Significant fatigue or loss of energy
Significant speeding up or slowing down of psychomotor activity
Excessive anxiety, panicky feelings, worry or rumination
Avoidance (of people, places, things, activities)
Misuse of alcohol or other substances
Very unusual behavior or thinking: psychotic symptoms

References

• RACGP - Theme - Perinatal Depression

• Medscape - Postpartum Depression


Last updated:
Time spent: QID:970
2023-2-12

1537 of 1943
A 28-year-old woman presents 3 months after giving birth to her daughter. She is concerned about
her ‘silly’ thoughts of harming her baby. This has led her to avoid her baby and keeping her away
from her. When you ask her to give you an example of how she is afraid of harm to the baby, she
says ‘I am afraid my baby would fall off my hand if I lift her’. Which one of the following could be
the most likely diagnosis?

A. Postpartum depression.
B. Postpartum blues.
C. Postpartum obsession.
D. Postpartum psychosis.
E. Overvalued ideas.

Incorrect. Correct answer is C


45% answered correctly

Explanation:

Correct Answer Is C

The history is completely consistent with postpartum obsessions as the most likely diagnosis.
Women are at increased risk for obsessive-compulsive disorder (OCD) during or following
pregnancy, including new-onset OCD, recurrent OCD, or exacerbation of a chronic OCD.

The obsessional thinking and compulsive behaviors often focus on the pregnancy or baby. In
postpartum period, obsessive thoughts or mental images of harming and fears of contamination of
the baby are common. Examples of such thoughts or images include dropping the baby onto the
floor, drowning the baby in bathtub, throwing the baby out of the window ,crushing the baby’s skull,
microwaving the baby, etc.

Prenatally, obsessions are often about fears of fetal death or contamination. An example of
contamination is a mother's belief that she is infected and if she continues the pregnancy, the
infection will pass on to her baby too.

Compulsive behaviors may include the mother’s repeated requests for ultrasound to check fetal
well-being prior to birth, or subsequent avoidance of touching the baby, or repeated washing or
changing diapers.

Other examples are shown in the following table:

Obsession Associated compulsion


Checking for fetal
Fear of fetal death
movement
Fear of contaminating the baby with toxic Excessive washing and
Pregnancy agents cleaning
Aggressive obsessions towards fetus

1538 of 1943
Fear of intentional or accidental harm to the Avoidant behavior (e.g.
infant (including sexual abuse) avoiding knives, infant)
Intense fear of sudden infant death syndrome
Postpartum Excessive washing and
Fear of contaminating the infant
cleaning
Fear of criticism of mothering skills

By definition, patients with OCD have insight into their obsessions, recognizing them as intrusive
and inappropriate, like in this women, by describing her thoughts as ‘silly’; however, their degree of
insight can vary. When severe obsessive thoughts persist over a long period of time, they can
resemble fixed delusional thinking. On very rare occasions, these thoughts can progress to
psychosis, and the patient believes the ideas/thoughts are real.

NOTE - The likelihood that aggressive obsessions toward the baby results in harmful behaviors in
not known, but generally is believed to be very small.

References

• http://www.kemh.health.wa.gov.au/brochures/health_

• http://www.uptodate.com/contents/obsessive-compuls
Last updated:
Time spent: QID:971
2023-2-12

1539 of 1943
Which one of the following is the treatment of choice for and acute manic episode of bipolar
disorder during the first trimester of pregnancy?

A. Carbamazepine.
B. Lithium.
C. Clozapine.
D. Sodium valproate.
E. Quetiapine.

Incorrect. Correct answer is B


45% answered correctly

Explanation:

Correct Answer Is B

Of the options, lithium is both the most effective and safest medication to be used for treatment of
acute mania during pregnancy.

Despite recent controversy about lithium’s efficacy, a series of meta-analyses confirms its value for
both acute episodes and for prophylaxis, and lithium remains the cornerstone of therapy in patients
with bipolar disorder (BPD).

Although lithium use during the first trimester of pregnancy has been reported to be associated
with fetal cardiovascular anomalies (e.g. Epstein’s anomaly) and midfacial and other defects,
maternal benefits substantially outweigh the risks of fetal anomaly. Generally speaking, lithium can
be used rather safely during all trimesters of pregnancy.

However, if a pregnant woman has been exposed to lithium during the first trimester, an ultrasound
and echocardiogram should be performed at 16-20 weeks gestation to exclude fetal anomalies,
especially cardiac ones.

Lithium is even safer during the second and third trimesters, because cardiogenesis has already
occurred and the risk of fetal malformations is significantly decreased. If lithium has been tapered
during the first trimester or before the conception, it can be safely resumed in second or third
trimester if indicated.

Lithium requirements increase in the third trimester; however, dose reduction by 25% is
recommended before delivery to prevent possible neonatal toxicity that can present with a floppy
baby syndrome (hypotonia, cyanosis, poor suckling). The dose should then be increased after
delivery.

NOTE - Lithium is contraindicated in breast-feeding women. Renal clearance of lihtium in infant is


low and may result in high levels of lithium and life-threatening toxicity.

(Option A) With a few exceptions, the quality of the randomized controlled trials (RCTs) of
carbamazepine in acute mania has been surprisingly poor; therefore, it is not an appropriate option
for treatment of acute mania in this patient.

1540 of 1943
(Option D) Sodium valproate has significant teratogenic effect and should be avoided during
pregnancy.

(Options C and E) Clozapine or mirtazapine is not indicated or useful for treatment of an acute
manic episode in bipolar disorder.

NOTE – A recent meta-analysis has confirmed that the most efficacious acute treatments for
mania are the antipsychotics rather than the traditional ‘mood stabilizers’ lithium, valproate and
carbamazepine. Specifically, that report found that the preferred options – after taking into account
both efficacy and tolerability – were risperidone, olanzapine and haloperidol. An added benefit of
haloperidol in acute management of a pregnant woman is the rapid-onset therapeutic effect
compared to a delayed effect of other mood stabilizers. These drugs, on the other hand, are
associated with potential adverse effects such as metabolic syndrome with olanzapine and
extrapyramidal side effects with haloperidol, requiring a careful balance between risks and benefits,
often in consultation with an expert in the field. Generally, where antipsychotics (as mentioned
above) are among options, it is safest to choose them as the preferred treatment options and
correct answer for a pregnant woman, especially during the first trimester.

References

• https://www.ranzcp.org/Files/Resources/Publication

• http://www.racgp.org.au/afp/2013/september/bipolar
Last updated:
Time spent: QID:992
2023-2-12

1541 of 1943
Which one of the following is the treatment of choice for an acute manic episode of bipolar
disorder during the third trimester of pregnancy?

A. Carbamazepine.
B. Lithium.
C. Clozapine.
D. Sodium valproate.
E. Mirtazapine.

Incorrect. Correct answer is B


45% answered correctly

Explanation:

Correct Answer Is B

Of the given options, lithium is the only acceptable one in terms of both safety and efficacy during
pregnancy. However, it is contraindicated in breast feeding woman because reports suggest that
there is low renal clearance of lithium in infants resulting in toxic levels of lithium. This may result
in life-treathening infantile complications.

Reportedly, lithium use during the first trimester of pregnancy has been associated with
cardiovascular anomalies (e.g. Epstein’s anomaly), midfacial defects, and other defects;
nonetheless, the benefits to the mother substantially outweigh the risks of fetal anomaly. However,
if a pregnant woman has been exposed to lithium during the first trimester, an ultrasound and
echocardiogram should be performed at 16-20 weeks gestation to exclude fetal anomalies,
especially cardiac ones. Lithium is even safer during the second and third trimesters when
cardiogensis has already occurred and the risk of fetal malformations is significantly decreased. If
lithium has been tapered during the first trimester or before the conception, it can be safely
resumed in second or third trimester if indicated.

Lithium requirements increase in the third trimester; however, dose reduction by 25% is
recommended before delivery to prevent possible neonatal toxicity presenting with floppy baby
syndrome (hypotonia, cyanosis, poor suckling). The dose should then be increased after delivery.

(Option A) Carbamazepine has been shown poor efficacy in controlling an acute manic episode in
bipolar disorder.

(Options C and E) Clozapine or mirtazapine is not indicated or useful for treatment of an acute
manic episode in bipolar disorder.

(Option D) Sodium valproate has significant teratogenic effect and should not be used during
pregnancy.

NOTE - A recent meta-analysis has confirmed that the most efficacious acute treatments for mania
are the antipsychotics rather than the traditional ‘mood stabilizers’ lithium, valproate and
carbamazepine. Specifically, that report found that the preferred options – after taking into account
both efficacy and tolerability – were risperidone, olanzapine and haloperidol.

1542 of 1943
An added benefit of haloperidol in acute management of a pregnant woman is the rapid-onset
therapeutic effect compared to a delayed effect of other mood stabilizers. These drugs, on the
other hand, are associated with potential adverse effects such as metabolic syndrome with
olanzapine and extrapyramidal side effects with haloperidol, requiring a careful balance between
risks and benefits, often in consultation with an expert in the field.

Generally, where antipsychotics (as mentioned above) are among options, it is safest to choose
them as the preferred treatment options and correct answer for a pregnant woman, especially
during the first trimester or if the woman desires to breastfeed.

References

• https://www.ranzcp.org/Files/Resources/Publication

• http://www.racgp.org.au/afp/2013/september/bipolar
Last updated:
Time spent: QID:993
2023-2-12

1543 of 1943
Which one of the following is the treatment of choice for an acute manic episode of bipolar
disorder in a breastfeeding woman?

A. Carbamazepine.
B. Lithium.
C. Clozapine.
D. Sodium valproate.
E. Lamotrigine.

Incorrect. Correct answer is D


45% answered correctly

Explanation:

Correct Answer Is D

Postpartum period is a critical time during which a woman is at increased risk of emotional
challenges. The risk is particularly higher in those with bipolar disorder. In such women,
postpartum period is the time they may experience exacerbations of bipolar disorder in forms of
mania, depression and psychosis.

Recurrence rate of BPD in the first 3 to 6 months is 20-50%. Recurrence rates without treatment
may be as high as 70%. Symptoms often develop rapidly and may occur in late pregnancy or within
the first few days to weeks after delivery.

Lithium has been classically used for treatment of both manic and depressive espisodes as well as
maintenance therapy in patients with bipolar disorder. It is considered safe for use in pregnant
women, especially in the second and third trimesters of pregnancy. There are concerns regarding
Epstein anomaly as well as other congenital defects if used in the first trimester. The absolute risk
is insignificant though. Recently, haloperidol, risperidone and olanzapine have been proved more
effective than lithium and other mood stabilizers for acute treatment of mania in bipolar disorder.
There are concerns of adverse effects such as extrapyramidal effects of haloperidol as a first-
generation antipsychotic and weight gain and metabolic derangements with risperidone and
olanzapine as atypical antipsychotics. Sodium valproate is another drug used for treatment of
manic episodes in bipolar disorder; however, it is not an effective treatment if psychotic features
such as hallucinations or delusions are present.

On making a decision as to whether which drug is prescribed, the clinician should balance risks
and benefits of each, often in consultation with an expert in the field, and also consider specific
circumstances such as pregnancy and breastfeeding.

Of the options, sodium valproate is the only accpetable one for treatment of this patient. It hase
been proven as efficacious as lithium for acute management of mania. Although its use is
contraindicated in pregnancy, it can be effectively used in breastfeeding women. However, if there
are psychotic features such as delusions or hallucinations, sodium valproate will not be an
appropriate option either.

(Option A) Thus far, carbamazepine has not been shown to be associated with harm to the
breastfed baby, but studies suggest it is not more effective than placebo in management of an

1544 of 1943
acute manic episode.

(Options B and C) Lithium and clozapine are absolutely contraindicated in breastfeeding women.
Furthermore, clozapine is not recommended or approved for treatment of mania, depression or
maintenance therapy for bipolar disorder.

(Option E) Lamotrigine has being increasingly used to prevent relapses of bipolar depression;
however, the effects for treatment or prevention of manic episodes are minimal. It is also secreted
in breast milk and should not be used in breastfeeding woman due to lack of data regarding its
safety profile.

NOTE - If a pregnant woman has been stable on lithium or another mood stabilizer or
antipsychotic, with no contraindication in pregnant women, but not recommended during
breastfeeding, withdrawal from the drug or switching to another drug that is safe in breastfeeding
is not advisable, as any alteration in medication(s) may result in relapse of bipolar disorder. In such
circumstance, the most appropriate approach is continuation of the drug, avoiding breastfeeding,
and feeding of the baby with formula.

References

• https://www.ranzcp.org/Files/Resources/Publication
Last updated:
Time spent: QID:994
2023-2-12

1545 of 1943
A man brings his wife to your practice for your opinion, because she has started excessive cleaning
of the house and alcohol drinking. She has a lot of fights with him and flirts with his friends. During
consultation, she is initially flirtatious, but then becomes angry and starts talking to you very rudely.
According to her husband, 4 days ago she was quite normal. Which one of the following is the
most likely diagnosis?

A. Obsessive compulsive disorder.


B. Hypomania.
C. Histrionic personality disorder.
D. Passive-aggressive personality disorder.
E. Mania.

Incorrect. Correct answer is B


45% answered correctly

Explanation:

Correct Answer Is B

The patient seems to have some characteristic features of a manic episode. According to DSM-IV,
diagnostic criteria for a manic episode include:

A. A distinct period of abnormally and persistently elevated, expansive, or irritable mood, lasting at
least 1 week (or any duration if the symptoms are severe enough to result in hospitalization).

B. during the period of mood disturbance, three (or more) of the following symptoms have
persisted (four if the mood only irritable) and has been present to a significant degree:

1. Inflated self-esteem or grandiosity


2. Decreased need for sleep (e.g. feels rested after only 3 hours of sleep)
3. More talkative than usual or pressure to keep talking
4. Flight of ideas or subjective experience that thoughts are racing
5. Distractibility (i.e. attention too easily drawn to unimportant or irrelevant external stimuli)
6. Increase in goal-directed activity (either socially, at work or school, or sexually) or
psychomotor agitation
7. Excessive involvement in pleasurable activities that have a high potential for painful
consequences (e.g. engaging in unrestrained buying sprees, sexual indiscretions, or foolish
business investments)

C. The symptoms do not meet criteria for a Mixed Episode.

D. The mood disturbance is sufficiently severe to cause marked impairment in occupational or in


usual social activities or relationships with others, or to necessitate hospitalization to prevent harm
to self or others, or there are psychotic features.

E. the symptoms are not due to the direct physiological effects of a substance (e.g. a drug of
abuse, a medication or other treatments) or a general medical condition (e.g. hyperthyroidism).

NOTE - Manic-like episodes that are clearly caused by somatic antidepressants treatment (e.g.
medication, light therapy) should not count toward a diagnosis of bipolar disorder.
1546 of 1943
The patient’s symptoms do not appear to be severe enough to need hospitalization and the
duration of the symptoms has not lasted for at least 7 days; therefore, hypomania will be the most
likely diagnosis for now.

A hypomanic episode has the same symptoms of a manic episode with 2 important differences:

1. The mood disturbances usually are not severe enough to cause problems with the person
working or socializing with others, or to require hospitalization
2. There is no psychotic feature

A hypomanic episode is characterized by a distinct period of persistently elevated, expansive, or


irritable mood, lasting at least 4 days and present for most of the day nearly every day. This
hypomanic mood is clearly different from the person’s usual mood.

(Option A) In this scenario, engagement in excessive cleaning is different from what happens in
obsessive-compulsive disorder which is repetitive cleaning (that might not be
excessive). Obsessive-compulsive disorder (OCD) is characterized by recurrent intrusive thoughts,
images, or urges (obsessions) that typically cause anxiety or distress, and by repetitive mental or
behavioral acts (compulsions) that the individual feels driven to perform, either in response to an
obsession or according to rules that he or she believes must be applied rigidly.

(Option C) Some symptoms of hypomania overlap with those of histrionic personality disorder,
including shallow emotions and flirtatiousness. However, flirting in hypomania is due to increased
sexual drive, whereas, flirting in histrionic personality disorder is for attention seeking without any
intention for sex. Features of a person with histrionic personality disorder include being self-
dramatic, egocentric, immature, seductive and attention seeking.

(Option D) Features of passive-aggressive personality disorder include childish stubbornness,


argumentativeness, egocentricity, deliberate inefficiency, and hypercriticism of authority figures.
The scenario is not consistent with passive aggressive personality disorder.

(Option E) A minimum duration of 7 days is required for symptoms in order to make a diagnosis of
mania. With symptoms present for only 4 days this patient has hypomania for now.

References

• https://www.racgp.org.au/afp/2013/september/bipola
Last updated:
Time spent: QID:1000
2023-2-12

1547 of 1943
A man brings his wife to your practice for consultation, because in the past 5 days has been
excessively cleaning the house and drinking alcohol. She has a lot of fights with him and flirts with
his friends. During consultation, she initially is flirtatious, but then becomes angry and starts talking
to you very rudely. According to her husband, she was previously quite normal. Which one of the
following is the next best step in management?

A. Lithium.
B. Lamotrigine.
C. ​Olanzapine
D. Carbamazepine.
E. Diazepam.

Incorrect. Correct answer is C


45% answered correctly

Explanation:

Correct Answer Is C

The scenario fulfills the classic definition of hypomania. In fact this woman can have her first
hypomanic episode of a bipolar disorder (type II) disorder. A hypomanic episode is characterized
by a distinct period of persistently elevated, expansive, or irritable mood, lasting throughout at least
4 days and present for most of the day nearly every day. This hypomanic mood is clearly different
from the person’s usual mood.

Features seen in hypomania (as well as mania) include inflated self-esteem and grandiosity,
decreased need for sleep, more talkativeness than usual and pressures speech, distractibility,
increased goal-directed activities, increased sexual drive, and excessive involvement in pleasurable
activities that have a high potential for painful consequences (e.g. engaging in unrestrained buying
sprees, sexual indiscretions, or foolish business investments)

Hypomania is distinct from mania by milder magnitude of symptoms and less interference with
level of social and/or occupational. Patients with hypomania are unlikely to require hospitalization.

Some symptoms of hypomania overlap with those of histrionic personality disorder, including
shallow emotions and flirtatiousness. However, flirting in hypomania is due to increased sexual
drive, whereas flirting in histrionic personality disorder is for attention seeking without any intention
for sex. In this scenario, engagement in excessive cleaning is different from what occurs in
obsessive compulsive disorder which is repetitive cleaning (that might not be excessive).

For patients with hypomania and mild to moderate manic episodes, monotherapy with risperidone
or olanzapine is currently the first-line recommendation. Reasonable alternatives are lithium,
sodium valproate, carbamazepine, aripiprazole, quetiapine, ziprasidone and haloperidol.

(Options A and D) According to a recent metanalysis, the antipsychotics risperidone and


olanzapine are superior to lithium or carbamazepine for treatment of acute mania. Lithium,
however, may be added for maintenance therapy.

1548 of 1943
(Option B) Lamotrigine is an anticonvulsant successfully used for prevention of depression
episodes in patients with bipolar disorder. Its effect in treatment or prophylaxis of mania or
hypomania has not proved satisfactory.

(Option E) Benzodiazepines are considered in manic patients who cannot tolerate any of other
mood stabilzers. With the wide range of mood stabilizers available, monotherapy with
benzodiaepines is almost never indicated; however, lorazepam and clonazepam are used in short
term and in combination with mood stabilizers for rapid resolution of mania. Diazepam is not used
for such purpose.

References

• https://www.racgp.org.au/afp/2013/september/bipola

• http://www.uptodate.com/contents/bipolar-disorder-

• https://www.ranzcp.org/Files/Resources/Publication
Last updated:
Time spent: QID:1001
2023-2-12

1549 of 1943
Which one of the following is not associated with body dysmorphic disorder?

A. Anorexia nervosa.
B. Delusional disorders.
C. Social phobia.
D. Depression.
E. Decrease alcohol intake.

Incorrect. Correct answer is E


45% answered correctly

Explanation:

Correct Answer Is E

Body dysmorphic disorder is characterized by the belief that some body part is abnormal, defective,
or misshapen. Patients are preoccupied with an imagined defect in their appearance or excessive
concern about even the slightest physical anomaly. This preoccupation causes significant distress
or impairment in social, occupational, or other areas of function.

This disorder affects women more than men, typically between ages of 15 and 20 years.

The characteristic features of body dysmorphic disorder are as follow:

Preoccupation with alleged physical flaws - of all body parts, imagined facial flaws are most
common
Constant mirror checking
Attempt to hide the imagined deformity
Housebound and avoiding social situations
Impaired levels of functioning

The following comorbid conditions are often found as an association with body dysmorphic
disorder:

Depression
Delusional disorders
Obsessive compulsive disorder
Social phobia
Increased alcohol intake

Body dysmorphic disorder is not associated with decreased alcohol intake.

1550 of 1943
A 37-year-old woman is in your office accompanied by her husband for consultation. The man is
concerned about her wife because for the past 8 months, she has been progressively edgy at
home. She is always fighting with their 8-year-old daughter as to why she is not doing her
homework, and extremely anxious and distressed about her health. She worries a lot about
financial issues and is concerned about not having money for the future despite the stable financial
state of the family. She feels tense and is easily irritated. She has lost all her interest in hobbies
and leisure time, but denies depressed mood. Which one of the following is the most likely
diagnosis?

A. Major depression.
B. Generalized anxiety disorder.
C. Adjustment disorder.
D. Dysthymic disorder.
E. Obsessive compulsive disorder.

Incorrect. Correct answer is B


45% answered correctly

Explanation:

Correct Answer Is B

The scenario is a typical presentation of general anxiety disorder (GAD). GAD is characterized by
experiencing excessive anxiety and worry, often about health, family, financial issues, or work. This
worrying goes on every day, possibly all day long. It negatively interferes with social activities and is
associated with impaired function at work, school, or family.

GAD is diagnosed in adults when they experience at least 3 of the following symptoms on more
days for at least 6 months:

Restlessness or feeling keyed up or on edge


Being easily fatigued
Difficulty concentrating or mind going blank
Irritability
Muscle tension
Sleep disturbance (difficulty falling or maintaining, unrefreshing sleep, etc.)

To make the diagnosis in children, only one symptom is required.

(Option A) Major depression is associated with depressed mood which is not present in this
woman. However, depression and dysthymic disorder can be seen in patients with GAD.

(Option C) For adjustment disorder to be the diagnosis, one provocative event should be present.
There is no recent stressor in the history.

(Option D) Dysthymic disorder is characterized by mild to moderate symptoms of depression


lasting for at least 2 years.

1551 of 1943
(Option E) OCD is associated with obsessions and impulse. There is no clue in the history to
suggest such diagnosis.

References

• RACGP - Anxiety disorders

• UpToDate - Generalized anxiety disorder in adults: Epidemiology, pathogenesis, clinical


manifestations, course, assessment, and diagnosis
Last updated:
Time spent: QID:1002
2023-2-12

1552 of 1943
Aida is a pregnant patient of yours, who has presented for an antenatal visit in her first trimester.
This is her second pregnancy. Her previous pregnancy, 3 years ago, was followed by development
of postpartum psychosis, for which she was successfully treated with risperidone. The medication
was then tapered and stopped. She has been stable since. She wants to know if there is anything
she can do to prevent postpartum psychosis in this pregnancy. Which one of the following would
be the most appropriate advice?

A. Review her later in pregnancy.


B. Start prophylactic dose of antidepressants.
C. Start her on antipsychotics now.
D. Start her on risperidone if she develops postpartum psychosis again.
E. Start risperidone immediately after delivery.

Incorrect. Correct answer is E


45% answered correctly

Explanation:

Correct Answer Is E

With either of the following conditions present in history, a woman is at increased risk of
postpartum psychosis::

History of postpartum psychosis


History of bipolar disorder
Family history of postpartum psychosis
First pregnancy
Recent discontinuation of lithium or other mood stabilizers

The risk of recurrence in woman with previous postpartum psychosis is high in subsequent
pregnancies. The period during which the recurrence risk of post-partum psychosis is the highest,
is the first 28 days after delivery. The symptoms generally appear within days or even hours of
delivery.

It is strongly recommend that, in a woman with history of post-partum psychosis, prophylactic


mood-stabilizers be started immediately after delivery to prevent relapse. Choice of mood stabilizer
depends on different factors including breastfeeding, or previous response. Aida had desirable
response to risperidone in the past; therefore, commencement of risperidone immediately after
delivery would be the most appropriate option for her. Risperidone has not been associated with
increased risk to the breastfed baby; however, the data are limited.

This approach prevents unnecessary treatment of an asymptomatic woman and potential adverse
outcomes associated with psychotropic medications.

(Option A) This woman should also be closely monitored for development of psychiatric problems
throughout pregnancy; however, this does not prevent postpartum psychosis.

(Option B) Antidepressants, either therapeutically or prophylactically, have no effect on postpartum


psychosis. A mood stabilizer is indicated in such situations.

1553 of 1943
(Option C) In an asymptomatic woman, who is currently stable, initiation of prophylactic treatment
is not likely to be of significance benefit compared to immediately after delivery.

(Option D) With a high recurrence rate as high as 30-50%, waiting for the postpartum psychosis to
develop as an indication for starting treatment is not appropriate and prophylactic measures
should be contemplated.

References

• https://www.sahealth.sa.gov.au/wps/wcm/connect/8fb

• http://www.uptodate.com/contents/treatment-of-post
Last updated:
Time spent: QID:1003
2023-2-12

1554 of 1943
Which one of the following is not a risk factor for schizophrenia?

A. Genetic susceptibility.
B. Winter birth.
C. Advanced parental age.
D. Fetal hypoxia.
E. High educational achievements.

Incorrect. Correct answer is E


45% answered correctly

Explanation:

Correct Answer Is E

The genetic linkage in schizophrenia is well-established. Although schizophrenia appears to be


multifactorial, genetics plays a significant role.

Winter births are statistically associated with increased occurrence of schizophrenia. This fact may
augment the hypothesis that schizophrenia may be associated with viral infections.

Antenatal and perinatal complications such as hypoxia, neonatal sepsis, neonatal jaundice, etc are
associated with increased risk of schizophrenia. Advanced parental age at conception is
associated with increased risk of development of schizophrenia in offsprings.

High educational achievements are not risk factors for schizophrenia. In fact, they are protective
against schizophrenia.

References

• http://www.schizophreniaresearch.org.au/library/br

• http://www.uptodate.com/contents/schizophrenia-epi
Last updated:
Time spent: QID:1004
2023-2-12

1555 of 1943
A 30-year-old male is found not to interact with people and prefers to stay at home. He has no
desire to become involved in any relationships and feels uncomfortable around people. A female
psychologist recently saw him at his family's request, and he wishes he could make friends with
her by means of spells and supernatural solutions. Which one of the following options is the most
likely explanation for his behavior?

A. Schizoid personality disorder.


B. Schizotypal personality disorder.
C. Paranoid personality disorder.
D. Avoidant personality disorder.
E. Dependent personality disorder.

Incorrect. Correct answer is B


45% answered correctly

Explanation:

Correct Answer Is B

This patient's clinical and historical features are suggestive of schizotypal personality disorder.

Schizotypal personality disorder is characterized by significant discomfort in social relationships;


thought distortions, and eccentricity.

Individuals with this disorder are socially isolated and uncomfortable with others. The most
differentiating feature of this disorder from schizoid personality disorder is the presence of
particular patterns of thinking such as ideas of reference, persecution, and magical thinking that
are absent in schizoid personality disorder. Bizarre preoccupations, odd affection, and peculiar
speech are other features. Eccentric attire is another commonly observed feature.

(Option A) Schizoid personality is characterized by social withdrawal, isolation, and emotional


coldness. Affected individuals usually remain absorbed and sunken in their thoughts and feelings.
They fear closeness and intimacy with other people. They prefer theoretical speculations (such as
math and physics) to practical action. However, the thinking pattern is normal.

(Option C) Paranoid individuals find offense in even the most benign remarks or circumstances.
These individuals are often defending an extremely fragile self-concept.

(Option D) A person with an avoidant personality disorder is interpersonally sensitive, is afraid of


being criticized, and is constantly concerned about saying or doing something foolish or
humiliating. They have an intense desire to connect with others; however, they will never let anyone
get close unless they are sure of being liked.

(Option E) Individuals with dependent personality disorder are passive, submissive, and have the
fear of abandonment. They need constant reassurance. Dependent patients assume that they are
incapable of functioning independently and that being assertive will be experienced by others as
aggressiveness.

References

1556 of 1943
• Psych Central - Schizotypal Personality Disorder: Symptoms, Causes, and Treatment

• Psych Central - Schizoid Personality Disorder: Symptoms, Causes, and Treatment

• Psych Central - All About Paranoid Personality Disorder

• Psych Central - So Close, Yet So Far: Understanding Avoidant Personality Disorder

• Psych Central - Symptoms of Dependent Personality Disorder


Last updated:
Time spent: QID:1005
2023-2-12

1557 of 1943
Which one of the following is the preferred medical treatment for obsessive compulsive disorder
(OCD)?

A. Selective serotonin re-uptake inhibitors.


B. Tricyclic antidepressants.
C. Serotonin norepinephrine re-uptake inhibitors.
D. Benzodiazepines.
E. Alcohol.

Correct
45% answered correctly

Explanation:

Correct Answer Is A

Obsessive-compulsive disorder (OCD) is characterized by recurrent obsessions or compulsions


that are recognized by the individual as unreasonable.

There are two components in OCD:

Obsessions: obsessions are anxiety provoking, intrusive thoughts, commonly concerning


contamination, doubt, guilt, aggression and sex.

Compulsions: peculiar behaviors that reduce the anxiety such as hand washing, organizing,
checking, counting, etc.

Abnormalities of serotonin metabolism are most implicated as the underlying cause of OCD. The
prevalence is 2% in general population, and there is a 1:1 male-to-female ratio. The onset is
insidious and occurs during childhood, adolescence, or early adulthood. The symptoms are usually
of waxing and waning nature. Comorbid conditions such as depression, anxiety, and substance
abuse are frequently seen.

OCD is best treated with a specific form of behavior therapy - exposure and preventive response
and/or selective serotonin reuptake inhibitors (SSRIs) as the first-line medications. All SSRIs have
about the same efficacy. The TCA clomipramine has also been used as a treatment option and was
the medication of choice before SSRIs take over.

NOTE - Antipsychotics have been shown to add to the benefits of SSRIs or clomipramine.
Although adjunctive therapy with antipsychotics may be considered, they are not used for single
therapy in OCD.

(Option C) Serotonin norepinephrine re-uptakes inhibitors (SNRIs) e.g., venlafaxine can be used as
an alternative, but SSRIs remain the preferred option.

(Option B) Among tricyclic antidepressants (TCAs), clomipramine is the only one that seems to
have an impact on serotonin metabolism and can be used as second-line management in
treatment of OCD. The side effects of clomipramine include sedation, urinary retention, confusion,
delirium, and hypotension.
1558 of 1943
(Option D) Occasionally benzodiazepines may be considered for anxiety relief in short term, but
they are not effective in treatment of OCD.

(Option E) Although alcohol consumption is associated with decreased compulsive behavior in


short-term, its use is not recommended for treatment of OCD due to the high risk of dependence
and abuse.

References

• RACGP - AFP - Obsessive-compulsive disorder

• Medscape - Obsessive-Compulsive Disorder

• UpToDate - Pharmacotherapy for obsessive-compulsive disorder in adults


Last updated:
Time spent: QID:1006
2023-2-12

1559 of 1943
A 30-year-old woman presents with complaint of binge eating, after which she often induces
vomiting. She has a body mass index (BMI) of 21 kg/m2. Which one of the following is most likely
to be present in her family history?

A. Pica.
B. Family history of obesity.
C. Family history of elite athleticism.
D. Schizophrenia.
E. Substance abuse.

Incorrect. Correct answer is B


45% answered correctly

Explanation:

Correct Answer Is B

Based on the history and the BMI, bulimia nervosa is the most likely diagnosis. Both bulimia
nervosa and binge eating/purging type anorexia nervosa are associated with episodes of binge
eating, followed by compensatory purging with laxative or diuretic use, induction of vomiting, etc.
Patients with bulimia nervosa, however, tend to have normal weight or even can be overweight.

Of the given options, a family history of obesity is the most relevant finding in a patient with bulimia
nervosa.

In all eating disorders, there is an increased genetic heritability and frequency of a family history. A
family history of leanness or athleticism may be associated with anorexia nervosa; whereas
bulimic eating disorders are associated with a personal or family history of obesity.

Sexual abuse (not an option) is also a significant risk factor for development of eating disorders as
well as a wide range of other psychiatric disturbances.

(Option A) Pica is defined as persistent ingestion of nonnutritive substances for at least one month
at an age for which this behavior is developmentally inappropriate. Personal or family history of
pica has not been shown to be a risk factor for eating disorders.

(Option C) Family history of elite athleticism has been proposed as a risk factor for anorexia
nervosa.

(Option D) Although abnormal eating patterns can be seen in schizophrenic patients, family history
of schizophrenia does not appear to be a risk factor for development of eating disorders.

(Option E) Substance abuse is a comorbid conditions often noticed in family or personal history of
patients with eating disorders; this, however, is less likely to be directly related to development of
eating disorders.

References

• RANZCP - Clinical practice guidelines for the treatment of eating disorders

1560 of 1943
• UpToDate - Bulimia nervosa in adults: Clinical features, course of illness, assessment, and
diagnosis
Last updated:
Time spent: QID:1009
2023-2-12

1561 of 1943
A 25-year-old woman presents to your practice , accompanied by her husband, because the
husband believes that she has not been herself in the past few weeks. She also has been suffering
from delusional states at times. She is 20 weeks’ pregnant. On examination, she is noticed to have
pressured speech and flight of ideas represented by swift changes between irrelevant topics.
Which of the following will be the most appropriate treatment option for her?

A. Sodium valproate.
B. Clozapine.
C. Diazepam.
D. Lithium.
E. Carbamazepine.

Incorrect. Correct answer is D


45% answered correctly

Explanation:

Correct Answer Is D

The scenario is classic for a manic episode. Features seen in mania (as well as hypomania)
include inflated self-esteem and grandiosity, decreased need for sleep, more talkativeness than
usual and pressured speech, flight of ideas, distractibility, increased goal-directed activities,
increased sexual drive, and excessive involvement in pleasurable activities that have a high
potential for painful consequences (e.g. engaging in unrestrained buying sprees, sexual
indiscretions, or foolish business investments). Hypomania has the same symptoms but with
milder intensity. The presence of delusions in the scenario, however, is consistent with severe
mania. In fact, this woman is very likely to have presented with a manic episode of bipolar disorder.

Mood stabilizers are the cornerstone of management of a manic episode in mild to moderate
disease. Mood stabilizers include, the antipsychotic medication, such as haloperidole, risperidone,
aripiprazole, quetiapine, ziprasidone, or lithium, or anticonvulsants such as carbamazepine and
sodium valproate.

According to a recent meta-analysis, antipsychotics, especially risperidone, haloperidol and


olanzapine (not among options) are recommended as first-line treatment of acute mania in bipolar
disorder both in pregnant and nonpregnant patients.

Lithium can be used as well and is the most appropriate option among others, considering the fact
that at 20 weeks gestation (second trimester) fetal cardiogenesis has already taken place and the
risk of cardiac anomalies, a feared adverse outcome of lithium on the developing fetus, is minimal
if any.

Reportedly, lithium use during the first trimester of pregnancy has been associated with
cardiovascular anomalies (e.g. Epstein’s anomaly), midfacial defects, and other defects;
nonetheless, the benefits to the mother substantially outweigh the risks of fetal anomaly.

Lithium is even safer during the second and third trimesters when cardiogenesis has already
occurred and the risk of fetal malformations is significantly decreased. Lithium requirements
increase in the third trimester; however, dose reduction by 25% is recommended before delivery to

1562 of 1943
prevent possible neonatal toxicity that can present with a floppy baby syndrome (hypotonia,
cyanosis, poor suckling). The dose should then be increased after delivery.

(Option A) Sodium valproate has major teratogenic effects and is contraindicated throughout the
pregnancy.

(Option B) Although an atypical antipsychotic, clozapine is not among recommended medications


for treatment of mania.

(Option C) Benzodiazepines, mostly clonazepam or lorazepam are used as monotherapy for


patients with mild to moderate manic or mixed episodes who cannot tolerate lithium,
anticonvulsant or antipsychotics. This is very unusual to occur, given the large number of available
options for treatment of bipolar disorder. However, benzodiazepines are generally used as
adjunctive therapy for treatment of insomnia, anxiety and agitation in patients with elevated mood.

(Option E) Carbamazepine is an antiepileptic medication used as a mood stabilizer in treatment of


bipolar disorder. Recent studies, however, has questioned its efficacy. There is also concern about
teratogenicity pregnancy due to reported cases of neural tube defects.

References

• https://www.ranzcp.org/Files/Resources/Publication

• http://www.racgp.org.au/afp/2014/april/perinatal-m

• http://www.uptodate.com/contents/bipolar-disorder-
Last updated:
Time spent: QID:1013
2023-2-12

1563 of 1943
A 26-year-old woman with background history of difficult-to-treat bipolar disorder is planning to
conceive. She has been stable on lithium and quetiapine for the last 6 months. Which one of the
following would be the next best step in management?

A. Explain the pros and cons of lithium use during pregnancy and continue the medication.
B. Switch to sodium valproate.
C. Switch to lamotrigine.
D. Start the patient on antipsychotics.
E. Taper and stop lihtium before conception.

Correct
45% answered correctly

Explanation:

Correct Answer Is A

Although lithium use during the first trimester of pregnancy has been reported to be associated
with fetal cardiovascular anomalies (e.g. Epstein’s anomaly) and midfacial and other defects,
benefits to the mother substantially outweigh the risks of fetal anomaly.

For pregnant women stable on lithium, one approach can be careful withdrawal of lithium (or other
mood stabilizers) to avoid potential risks to the developing fetus. Some authorities discourage this
method because of the risk or relapse.

In this patient, however, the history of difficult-to-treat bipolar disorder that has just recently
responded to lithium after failed trials of other mood stabilizers makes such approach less
favorable. She should be maintained on lithium to avoid relapse. In such situations, the woman
should be fully informed of the risks, and decision making left to the patient. If the woman decides
to conceive, lithium should be continued throughout the pregnancy and an ultrasound and
echocardiogram performed at 16-20 weeks gestation to exclude fetal anomalies, especially cardiac
anomalies.

(Option B) Sodium valproate is teratogenic and contraindicated in pregnancy. Switching to sodium


valproate can be associated with high risk of relapse as well as more harm to the fetus.

(Option C) Lamotrigine is an option for prevention of bipolar depression. It is not effective in


prevention or treatment of manic episodes.

(Option D) According to a recent meta-analysis, antipsychotics, especially risperidone, olanzapine,


and haloperidol are superior to other mood stabilizers such as lithium, sodium valproate, and
carbamazepine for treatment of manic episodes and are recommended as first-line options for this
purpose, but if the risk of relapse is high, lithium should be used concomitantly for a maintained
therapeutic response. While this patient is stable on lithium, switching to withdrawal of lithium and
commencement of antipsychotics is likely to result in a relapse.

(Option E) As mentioned earlier, stopping medications in a stable patient might be considered, but
for this patient such action is can result in a relapse of the disease.

1564 of 1943
References

• http://www.ncbi.nlm.nih.gov/pmc/articles/PMC428404

• http://www.racgp.org.au/afp/2014/april/perinatal-m
Last updated:
Time spent: QID:1017
2023-2-12

1565 of 1943
A 30-year-old woman decides to conceive while she is stable on lithium for severe relapsing bipolar
disorder. Lithium is continued after discussion with the patient. Which one of the following would
be most important to consider during pregnancy?

A. Monitoring the liver function.


B. High resolution ultrasound at16-20 weeks gestation.
C. Increasing the dose of the lithium 25% in late third trimester.
D. Giving a diet rich in vitamin C.
E. Monitoring vitamin B12 level.

Incorrect. Correct answer is B


45% answered correctly

Explanation:

Correct Answer Is B

The benefits of lithium prophylaxis during pregnancy, in cases of severe bipolar disorder, may
outweigh the risks. For pregnant women stable on lithium, one possible approach is careful
withdrawal of lithium (or other mood stabilizers) to avoid potential risks to the developing fetus.

In this patient with history of severe relapsing bipolar disorder, however, continuation of the
treatment has been decided. Lithium use during the first trimester of pregnancy has been reported
to be associated with fetal cardiovascular anomalies (e.g. Ebstein’s anomaly) and midfacial and
other defects.

Ebstein's anomaly is a cardiac defect characterized by downward displacement of the tricuspid


valve into the right ventricle. The absolute risk for Ebstein’s anomaly associated with lithium use in
the first trimester is approximately 1 in 1000 to 2000 compared with 1 in 20000 in the general
population. Despite significant increased risk (10-20 folds), the absolute risk is small (0.05%).
Coarctation of the aorta and mitral atresia are other reported cardiac anomalies.

If a pregnant woman has been exposed to lithium during the first trimester, an ultrasound and
echocardiogram should be performed at 16-20 weeks gestation to exclude fetal anomalies,
especially cardiac anomalies.

(Option A) While the patient is on lithium, regular renal and thyroid and parathyroid function tests
as well as serum lithium levels are indicated. Liver function tests are not required to be monitored
as lithium does not have an adverse effect on the liver.

(Option C) Lithium requirements increase in the third trimester; however, dose reduction (not
increase) by 25% is recommended before delivery to prevent possible neonatal toxicity that can
present with a floppy baby syndrome (hypotonia, cyanosis, poor suckling). The dose should then be
increased immediately after delivery due to increased risk of relapse in postpartum period.

(Options D and E) Lithium is not associated with vitamin C or B12 deficiency. Supplementation of
these vitamines are not recommended while the pregnant woman is on lithium, unless indicated for
other reasons.

1566 of 1943
References

• http://www.ncbi.nlm.nih.gov/pmc/articles/PMC428404

• http://www.racgp.org.au/afp/2014/april/perinatal-m
Last updated:
Time spent: QID:1018
2023-2-12

1567 of 1943
A 27-year-old woman is being assessed in your general practice after her marriage was broken
8 weeks ago. One year ago, she was referred for psychotherapy from work due to frequent conflicts
with colleagues and being easily irritated and shouting at them. She complains that after finishing
work, she is not able to relax at home. Which one of the following options is more likely to be the
cause of this presentation?

A. Bipolar II disorder with depression.


B. Cyclothymic disorder.
C. General anxiety disorder.
D. Major depression.
E. Borderline personality disorder.

Incorrect. Correct answer is E


45% answered correctly

Explanation:

Correct Answer Is E

Of the given options, borderline personality disorder is most consistent with the history. Important
clues to this diagnosis are frequent conflicts with colleagues, being irritated easily, bursts of anger
represented by shouting at colleagues, failed marriage, and not being able to relax.

Borderline personality disorder (BPD) is characterized by instability of interpersonal relationships,


self-image, and emotions and by impulsivity.

BPD has three components:

1- Impaired relatedness - unstable relationships with others, identity disturbance, and chronic
emptiness

Patients with BPD usually have stormy relationships, especially in their close relationships. At one
moment a friend or romantic partner may be viewed as trusted or an ideally perfect person, and
then this same individual can suddenly be seen as cruel, betraying, very limited, and damaged.
When the idealized person is present and supportive, the patient feels strong and solid.

However, if the support person leaves (or is unable to meet the patient's needs) for a limited period
of time, or if the patient thinks that the support person is about to leave, the patient can
immediately become angry, demeaning, depressed, hopeless, and suicidal.

Splitting, which is rigidly classifying other as good as bad, is common among patients with BPD
and can lead the patient to shift between extreme points of view and to selectively attend to
information in a way that confirms his or her current opinion. This tendency to "split" can impact
treatment.

Patients with BPD often interpret neutral events, words, or faces as 'negative'; therefore, the patient
is prone to misinterpret relatively minor disagreements or adverse events as a sign that the
1568 of 1943
caretaker wants to terminate the relationship. The patient often reacts with anger or threats of self
harm, which can gradually alienate the support person, and result in their real tendency to end the
relationship. This heightened sensitivity to actual or perceived rejection can lead patients to feel
more comfortable or secure with a transitional object, such as a pet or stuffed animal, than with
other people.

2- Affective dysregulation - affective lability, excessive anger, and efforts to avoid abandonment

Patients with BPD experience repeated and marked mood changes throughout the course of a
single day, with moment to moment fluctuations often triggered by environmental stressors.
Periods of euthymia alternate with intense, episodic dysphoria that includes depression, anxiety,
and irritability. Angry outbursts triggered by dissatisfaction with a caregiver are often followed by
feelings of shame, guilt, and worthlessness.

3- Behavior dysregulation - impulsivity, suicidality, and self-injurious behavior

Impulsive and self-damaging behavior is common and can take many forms. Patients abuse
substances, binge eat, engage in unsafe sex, spend money irresponsibly, and drive recklessly. In
addition, patients can suddenly quit a job that is promising or end a relationship that has the
potential to last, thereby sabotaging their own success. Impulsivity can also manifest with
immature and regressive behavior.

The following are characteristic features of BPD and their prevalence:

Affective instability – 95%


Inappropriate anger – 87%
Impulsivity – 81%
Unstable relationships – 79%
Feelings of emptiness – 71%
Paranoia or dissociation – 68%
Identity disturbance – 61%
Abandonment fears – 60%
Suicidality or self-injury – 60%

(Option A) There are no symptoms indicating either mania/hypomania or depression to suggest


bipolar disorder. Bipolar disorder is often characterized by shifting episodes of mania/hypomania
and depression, none of which are present in the history.

(Option B) Cyclothymic disorder presents with many periods of depressed mood and many
episodes of hypomanic mood for at least 2 years. Interestingly, the condition frequently coexists
with BPD. In the scenario, however, there are no clues to indicate depression or mania.

(Option C) Generalized anxiety disorder (GAD) is characterized by excessive, poorly controlled


anxiety about life circumstances that continues for more than 6 months. To establish the
diagnosis, both physiologic and psychologic symptoms should be present.

(Option D) Major depression presents with at least a 2-week course of the following symptoms:

1. Depressed mood most of the day


2. Anhedonia during most of the day
3. Significant weight loss or gain
1569 of 1943
4. Insomnia or hypersomnia
5. Psychomotor agitation or retardation
6. Fatigue or loss of energy almost everyday
7. Feelings of helplessness/hopelessness or guilt
8. Diminished ability to concentrate
9. Recurrent thoughts about death

Of these symptoms, the first two are necessary for major depression to be present. The history is
not consistent with major depression as diagnosis.

References

• Psych Central - Symptoms of Borderline Personality Disorder (BPD)

• Psychology Today - Borderline Personality Disorder: Big Changes in the DSM-5


Last updated:
Time spent: QID:1020
2023-2-12

1570 of 1943
Which one of the following is not a feature of bipolar disorder?

A. Flight of ideas.
B. Persistently elevated mood for more than one week.
C. Delusions of grandiosity.
D. Decrease in goal directed activities.
E. Impairment of occupational functioning.

Incorrect. Correct answer is D


45% answered correctly

Explanation:

Correct Answer Is D

Features of bipolar disorder include:

Inflated self-esteem or grandiosity


Decreased need for sleep (e.g. feels rested after only three hours of sleep)
More talkative than usual or pressure to keep talking
Flight of ideas or subjective experience that thoughts are racing
Distractibility (i.e. attention too easily drawn to unimportant or irrelevant external stimuli)
Increase in goal-directed activity (either socially, at work or school, or sexually) or
psychomotor agitation
Excessive involvement in pleasurable activities that have a high potential for painful
consequences (e.g. engaging in unrestrained buying sprees, sexual indiscretions, or foolish
business investments)

These all must be associated with impairment of social or occupational functioning.

Patients with bipolar disorder have increased (not decreased) goal-oriented activities.

1571 of 1943
Which one of the following drugs is not used in the treatment of cocaine overdose?

A. Diazepam.
B. Aspirin.
C. Diltiazem.
D. Propranolol.
E. Hydralazine.

Incorrect. Correct answer is D


45% answered correctly

Explanation:

Correct Answer Is D

Acute cocaine use is associated with arterial vasoconstriction and enhanced thrombus formation.
It causes tachycardia, hypertension, increased myocardial oxygen demand, and increased vascular
shearing forces. Chest pain is the presenting symptom in 40% of patients.

The following cardiovascular complications can also occur:

Coronary vasoconstriction in a dose dependent fashion leading to myocardial ischemia


Acute left ventricular depression and heart failure at high blood concentrations
Supraventricular and ventricular arrhythmia (both through direct action or indirectly by
producing myocardial ischemia)
Aortic rupture (rare)

CNS manifestations of cocaine intoxication include:

Psychomotor agitation
Headache
Seizure
Coma
Intracranial hemorrhage
Focal neurological symptoms

NOTE - Cocaine-induced psychomotor agitation can cause hyperthermia due to peripheral


vasoconstriction that prevents the body from dissipating the heat generated from persistent
agitation. Hyperthermia in the setting of cocaine intoxication can have a mortality of as high as
33%.

Crack cocaine is associated with upper and lower airway injuries because its use in requires high
temperatures for evaporation. Such injuries are not caused by direct toxic effects. Cocaine use can
also have a variety of effects on gastrointestinal tract, eyes and muscles.

In majority of cases with acute cocaine intoxication, benzodiazepines are generally sufficient to
improve cardiovascular symptoms because most cardiovascular stimulation from cocaine is
mediated by sympathetic nervous system. Benzodiazepines also help alleviate the agitation.
Intravenous diazepam is the treatment of choice if the patient is not hypoxic or hypoglycemic.

1572 of 1943
However, in patients with severe, refractory, or symptomatic cocaine-induced hypertension other
antihypertensive medications can be tried. Phentolamine, given as an intravenous bolus, is the
treatment of choice, but other medications used for acute management of hypertension such as
hydralazine can be used as well.

Aspirin is given to patients with pain associated with cocaine intoxication; after aortic dissection as
potential cause has been excluded. As mentioned earlier, cocaine use is associated with enhanced
thrombus formation.

Beta blockers should not be used in the treatment of cocaine-induced cardiovascular symptoms
and complications owing to the fact that beta blockers may result in unopposed alpha-adrenergic
stimulation, resulting in coronary vasoconstriction and end-organ ischemia. In very rare occasion
when use of beta blockers is inevitable, phentolamine should be given beforehand to prevent
unopposed alpha-adrenergic stimulation.

References

• http://www.uptodate.com/contents/cocaine-acute-int
Last updated:
Time spent: QID:1090
2023-2-12

1573 of 1943
A 21-year-old woma presents to your practice with complaints of anorexia, weight loss, loss of
interest in daily life, and a depressed mood for the past 3 weeks. Evaluation reveals the diagnosis
to be major depression. You advise that she take antidepressants as well as non-pharmacological
therapy, but she refuses to be treated. Which one of the following would be appropriate advice for
her?

A. Your symptoms will progressively increase until you inevitably commit suicide.
B. Your symptoms will not become better untill 6 months later.
C. Your symptoms will disappear spontaneously without medications in 5 years.
D. Only partial remission could be expected as the best outcome.
E. Your symptoms will remit in 2 months.

Incorrect. Correct answer is B


45% answered correctly

Explanation:

Correct Answer Is B

Without treatment, an episode of major depressive disorder in adults lasts from 6 months to 18
months (average 8 months). The course of the disease in adolescents appears to be different from
that of adults. Although there is no solid evidence, it is suggested that major depression in
adolescents may last from 2 weeks to many years.

Based on such data, the most appropriate advice for this patient is that without treatment she is
unlikely to recover until 6 months.

(Option A) Depression has the most association with suicide. Lifetime risk of suicide among
patients with untreated depression ranges from 2.2% to 15%. Most of depressed patients never
commit suicide.

(Option C) Most patients with major depression witll improve in 6 to 18 months even without
treatment. Telling the patient that her symptoms will last for 5 years is incorrect.

(Option D) Recovery from depression can be complete without residue; although the recurrence
rate is significant.

(Option E) It is unlikely that the patient's symptoms remit in 2 months without treatment. The
minimum expected time for resolution of major depression is 6 months.

References

• http://www.aihw.gov.au/WorkArea/DownloadAsset.aspx

• http://www.suicidology.org/portals/14/docs/resourc
Last updated:
Time spent: QID:1094
2023-2-12

1574 of 1943
During a car chase, a 24-year-old man hits a post lamp and sustains injuries. Accompanied by the
police, he is brought to the Emergency Department. According to the police, he has had multiple
occasions of violence, offense, bar fights, and vandalism, for which he has been arrested several
times during the past 5 years. He drinks alcohol, smokes marijuana, and sometimes uses cocaine.
Which one of the following pieces of information is most helpful in making a diagnosis?

A. History of non-accidental injuries.


B. History of trying to set fire on a neighbor’s car in adolescence.
C. Amphetamine use of his mother while she was pregnant with him.
D. Failure to response to methylphenidate prescribed for suspected diagnosis of ADHD in
childhood.
E. Truancy.

Incorrect. Correct answer is B


45% answered correctly

Explanation:

Correct Answer Is B

This scenario suggests antisocial personality disorder (ASPD) as the most likely diagnosis. ASPD
is characterized by a pattern of disregard for and violation of others’ rights, routine engagement in
illegal activities (e.g., drug use, assault, and theft), endangering the well-being of others, and
blatantly disregarding their legal rights. They also tend to be aggressive and impulsive and have
difficulty maintaining employment.

ASPD typically begins by mid-adolescence and continues into adulthood. Patients must be 18
years or older to fulfill the diagnostic criteria for the condition. Individuals with ASPD often have a
history of conduct disorder in adolescence presented by aggression towards people or animals,
destruction of property, deceitfulness or theft, or a serious violation of rules. Conduct disorder is
considered a warning sign for the development of ASPD in the future and helps with the diagnosis
in adulthood.

Of the options, the history of trying to set fire to a neighbor’s car is consistent with conduct
disorder in childhood as a risk factor and a warning sign for ASPD.

(Option A) Non-accidental injury in childhood is a significant cause of mortality and morbidity,


especially in infants and young children, and should raise suspicion of child abuse and neglect.
Child abuse increases the risk of developing antisocial personality disorder but is not essential for
establishing the diagnosis.

(Option C) Amphetamines are considered unsafe to use during pregnancy. However, there is no
evidence that in-utero exposure to amphetamines is a risk factor for ASPD. However, ASPD patients
commonly are noted to abuse amphetamine.

(Option D) Individuals diagnosed with ADHD during childhood are at greater risk for developing a
personality disorder in late adolescence. Specifically, studies found elevated rates of narcissistic,
paranoid, borderline, and antisocial personality disorders. Antisocial and paranoid personality
disorder appears to emerge primarily when ADHD is persistent.

1575 of 1943
(Option E) Some young people, who are not in school due to either truancy or exclusion, are
particularly vulnerable to drug misuse. Others at risk include those who have been in foster care
and those who are homeless. Moreover, people with antisocial personality disorder tend to start
abusing their substance of choice at an earlier age than those who do not have the disorder.
Truancy, however, is not a pointer towards ASPD or necessary to make such a diagnosis.

References

• HHS Public Access - Childhood ADHD and the Emergence of Personality Disorders in
Adolescence: A Prospective Follow-up Study
Last updated:
Time spent: QID:1098
2023-2-12

1576 of 1943
A 55-year-old woman presents with insomnia for the past few days after she found out she has
breast cancer. She is a successful accountant and has enjoyed a healthy life. Although distressed
about the diagnosis, her mood is not significantly down. You recommend sleep hygiene and routine
as the most appropriate initial management of her sleep problem, but she insists that you prescribe
some medications to help her immediately. Which one of the following is the most appropriate
medication for her?

A. Temazepam.
B. Diazepam.
C. Fluoxetine.
D. Olanzapine.
E. Haloperidol.

Correct
45% answered correctly

Explanation:

Correct Answer Is A

Insomnia is a common problem in women with breast cancer with a prevalence of 23% to 61%.

Insomnia may present as difficulty initiating or maintaining sleep, or non-refreshing sleep. If


persistent enough, it can cause clinically significant distress or impairment in social, occupational,
or other important areas of functioning.

In approach to patients with insomnia, it is important to educate the patient about sleep hygiene,
which is encouraging habits that promote good sleep and avoiding those that may inhibit sleep.

Patients with anxiety can benefit from cognitive behavioral therapy (CBT). Behavioral therapeutic
approaches for insomnia seem particularly suitable to use in the breast cancer populations
because they have lower risk of interacting with the cancer treatment, do not burden the patients
with additional pharmacological treatments, and can target the treatment towards ameliorating
specific symptoms, like fatigue, that are characteristic in this population.

If such measures fail or the patient requests immediate relief, pharmacologic therapy is indicated.
Temazepam, zolpidem or zopiclone are first-line medications for such purpose. If they are
prescribed, the duration of therapy should be for the shortest time possible. The duration of use
should be determined and discussed with the patient. Also, patient should be fully informed of
potential adverse effects and chances of dependence.

This woman requests medication for immediate relief of insomnia, and of the options, temazepam
is the most appropriate one to consider for her.

(Option B) Diazepam is a long-acting benzodiazepine with unpredictable effects when used orally.
It is not recommended for management of insomnia.

(Options C) Fluoxetine and other SSRIs are used for pharmacological treatment of depression. This
woman, although distressed and anxious, is not depressed and does not need antidepressants.

1577 of 1943
SSRIs can cause sedation and improve sleep; however, they can induce insomnia as one of their
adverse effects, particularly early in the course of treatment.

(Options D and E) Haloperidol is a first-generation antipsychotic and olanzapine a second-


generation one. This woman does not have psychotic features justifying the use of antipsychotics.
Haloperidole in particular and olanzapine has sedative effects, but should not be used for the
purpose of insomnia in the absent of other main indications to their use.

References

• PubMed - Insomnia and its Treatment in Women with Breast Cancer

• Therapeutic Guidelines – Psychotropics; available from: http://tg.org.au


Last updated:
Time spent: QID:1121
2023-2-12

1578 of 1943
Anne is a 65-year-old part-time nurse, who is reported for evaluation by her colleagues. For the past
12 months, she has been increasingly apathetic and has lost interest in her daily activities including
gardening that she has been doing for the past 40 years. Recently, she has started to accuse her
colleagues of stealing things from the ward every time something is lost. During the interview, she
has flat affect and reduced eye contact. She can answer to simple questions, but is unable to
process more complex ideas. She denies hallucinations, falls or movement disorders. Which one of
the following could be the most likely diagnosis?

A. Delusional misidentification syndrome.


B. Alzheimer disease.
C. First episode of psychosis.
D. Frontotemporal dementia.
E. Paranoid schizophrenia.

Incorrect. Correct answer is D


45% answered correctly

Explanation:

Correct Answer Is D

The clinical scenario is most consistent with dementia. Dementia is a progressive decline in
general cognitive function, with normal consciousness and attention. There is impairment of
memory, abstract thinking, judgment, verbal fluency and the ability to perform complex tasks. It is
associated with behavioral and psychological changes, and impairment of social and physical
functioning. Behavioral and psychological symptoms of dementia include psychosis, depression,
agitation, aggression and social disgrace.

Of the given options, frontotemporal dementia (Pick disease) is most consistent with the clinical
scenario.

Frontotemporal dementia is characterized by focal degeneration of the frontal and/or temporal


lobes. The typical age of onset is in the late 50s or early 60s, and the primary clinical
manifestations are changes in personality and social behavior or language, progressing over time
to a more global dementia. Other features include impaired initiation and planning, disinhibited
behavior and social disgrace and mild abnormalities on cognitive testing. Apathy and memory
deficits develop later in the course of the disease. A subset of patients may also exhibit symptoms
of extrapyramidal or motor neuron involvement at some point in the disease process.

(Option A) Delusional misidentification syndromes are rare psychopathologic phenomena that may
occur within the context of schizophrenia or affective or organic illnesses. They include Capgras
syndrome, Fregoli syndrome, intermetamorphosis syndrome, syndrome of subjective doubles,
mirrored self, delusional companions, and clonal pluralization of the self. Misidentification
syndromes show a great degree of overlap and do not represent distinctive syndromes. There is no
clue in the scenario to point towards delusional syndromes.

(Option B) Alzheimer disease is the most common cause of irreversible dementia and is
characterized by forgetfulness early in the course of the disease. Behavioral changes develop late

1579 of 1943
in the course. Anne’s presenting symptom is behavioral changes that are unlikely to develop early in
Alzheimer disease.

(Option C) Psychosis typically presents with hallucinations and delusions, the absence of which in
the scenario makes this diagnosis unlikely.

(Option E) Patients with paranoid type schizophrenia present with preoccupation with one or more
delusions and/or hallucinations, usually involving grandeur or persecution.

References

• UpToDate - Frontotemporal dementia: Clinical features and diagnosis

• Therapeutic guidelines – Psychotropics


Last updated:
Time spent: QID:1150
2023-2-12

1580 of 1943
Accompanied by his wife, a 63-year-old man presents to your practice for consultation. She is
concerned about his husband because he has been recently behaving childish and bizarre. Last
week he was dismissed from his job as a manager in a local restaurant, because of treating rude to
customers and shouting at his colleagues. He does not shave, bathe or change his clothes as he
did before and is disheveled and unkempt all the time. She denies any falls, gait abnormalities, or
hallucinations in her husband. His memory is not significantly affected. Which one of the following
could be the most likely diagnosis?

A. Depression.
B. Alzheimer disease.
C. Lewy body dementia.
D. Frontotemporal dementia.
E. Schizophrenia.

Incorrect. Correct answer is D


45% answered correctly

Explanation:

Correct Answer Is D

Cognitive and behavioral changes in aged people are frequently faced in general practice, with
dementia and delirium being the most common underlying etiologies.

Cognitive function is measured by various mental functions, including memory, concentration,


praxis, language, executive functions, and visuospatial skills. Dementia refers to memory loss with
impairment of any other cognitive function that can interfere with social or occupational
functioning.

A myriad of causes have been identified for dementia. These causes can be reversible or
irreversible.

The most common reversible causes of dementia include:

Hypothyroidism
Vitamin B12 deficiency
Hepatic or uremic encephalopathy
Vasculitides affecting CNS
Space occupying brain lesions i.e. abscess/tumors either primary or metastatic
Medications – anticholinergics in particular
Normal pressure hydrocephalus
Central or obstructive sleep apnea
Subdural hematoma
Trauma
Depression

Some of the most common irreversible causes of dementia are:

Alzheimer disease (60-80% of cases )


1581 of 1943
Vascular dementia including multi-infarct dementia and Binswanger disease
Lewy body dementia
Frontotemporal degeneration (dementia) including Pick disease
Multifocal leukoencephalopathy

The case scenario describes a patient with social inappropriateness as the most concerning
presenting symptom without memory being significantly involved. Of the options, the most
consistent one with such scenario is frontotemporal dementia (Pick disease). In this disease,
social disgrace is the earliest symptom with memory impairment and forgetfulness following later.

Frontotemporal dementia is characterized by focal degeneration of the frontal and/or temporal


lobes. The typical age of onset is in the late 50s or early 60s, and the primary initial clinical
manifestations are changes in personality and social behavior or language, progressing over time
to a more global dementia. Other features include impaired initiation and planning, disinhibited
behavior and social disgrace and mild abnormalities on cognitive testing. Apathy and memory
deficits develop later in the course of the disease. A subset of patients may also exhibit symptoms
of extrapyramidal or motor neuron involvement at some point in the disease process.

(Option A) Patients with depression may present with pseudodementia which is different from
dementia in some aspects. It is less common for patients with pseudodementia to have
disinhibition or social disgrace. The history of disturbances in pseudodementia is often short and
abrupt onset, while dementia is more insidious. On cognitive testing, people with pseudodementia
often answer that they do not know the answer to a question, and their attention and concentration
are intact and they may appear upset or distressed. Those with true dementia will often give wrong
answers, have poor attention and concentration, and appear indifferent or unconcerned.

(Option B) In Alzheimer disease, forgetfulness is usually the presenting symptom. It is very unlikely
for a patient with Alzheimer disease to present with disinhibition and social inappropriateness early
in the course of the disease.

(Option C) although misbehavior and disinhibition is a common early feature in patients with Lewy
body dementia, the absence of other manifestations such as fluctuating cognition, hallucinations,
extrapyramidal deficits (Parkinsonism) and repeat falls makes this diagnosis less likely.

(Option E) Psychotic features such as hallucinations and delusion are a significant diagnostic
component in schizophrenia that is absent here. Moreover, development of schizophrenia at this
age is unusual.

References

• UpToDate - Frontotemporal dementia: Clinical features and diagnosis

• Therapeutic guidelines – Psychotropics


Last updated:
Time spent: QID:1151
2023-2-12

1582 of 1943
An 84-year-old man is brought to the Emergency Department by his neighbors. He lives alone and
has developed forgetfulness and peculiar behavior recently. At night, the neighbors hear him
shouting and saying that somebody is trying to break into his house while there is no body there
when they come and check. This behavior started 2 months ago and has worsened progressively.
Which one of the following conditions is most likely to have caused this presentation?

A. Lewy body dementia.


B. Alzheimer disease.
C. Delusional disorder.
D. Schizophrenia.
E. Frontotemporal dementia.

Correct
45% answered correctly

Explanation:

Correct Answer Is A

Some features in the scenario are of diagnostic importance. Recent onset forgetfulness and
peculiar behavior and importantly visual hallucinations suggested by seeing people that do not
exist. These features suggest Lewy body dementia as the most consistent diagnosis among other
options.

Lewy body dementia is a progressive, degenerative dementia of unknown etiology, presenting with
dementia, motor signs, episodes of reduced cognition and visual hallucinations. Unlike Parkinson
disease, in Lewy body dementia, cognitive impairment and dementia precedes extrapyramidal
manifestations.

In summary, Lewy body dementia presents with the following:

Dementia and fluctuations in cognitive function with varying levels of alertness and attention
during a day
Episodes of long staring and disorganized speech
Well-formed, detailed Visual hallucinations (or less commonly, auditory)
Parkinsonian motor features
Early extrapyramidal features
Anterograde memory loss
Visuospatial impairment

(Option B) Although forgetfulness is a prominent feature and an early symptom in Alzheimer


disease, spontaneous motor Parkinsonism, hallucinations and peculiar behavior develops late in
the course of the disease, often in a matter of years.

(Option C) There is no false belief in the scenario suggesting a delusional disorder. The imagined
people entering his house are in fact visual hallucinations, not delusions. .

(Option D) Although hallucinations are a characteristic feature is schizophrenia and other psychotic
disorders, patients with schizophrenia usually have auditory rather than visual hallucinations.
1583 of 1943
Moreover, the age of the patient is unusual for development of schizophrenia.

(Option E) Despite the fact that early behavioral changes are a feature also seen in frontotemporal
dementia, the presence of visual hallucinations makes Lewy body dementia the more likely
diagnosis. Furthermore, forgetfulness is not a prominent feature early in the course of
frontotemporal dementia.

References

• Medscape - Lewy Body Dementia

• UpToDate - Clinical features and diagnosis of dementia with Lewy bodies


Last updated:
Time spent: QID:1152
2023-2-12

1584 of 1943
A 71-year-old woman is brought to your practice for evaluation by her daughter. According to the
daughter, she has developed progressive forgetfulness for the past 7 months. Recently, she
frequently forgets days of the week and the date. Her past medical history is insignificant. She has
not had any ischemic heart disease (IHD) or hypertension. On physical examination, she has a
blood pressure of 110/65mmHg, pulse of 53 and regular, and a mini-mental status exam (MMSE)
score of 14. You decide to start her on medications. Which one of the following would be the most
appropriate option for her?

A. Donepezil.
B. Memantine.
C. Rivastigmine.
D. Haloperidol.
E. Galantamine.

Incorrect. Correct answer is B


45% answered correctly

Explanation:

Correct Answer Is B

This patient has classic presentation of Alzheimer disease, and an MMSE score of 14 indicates
moderate dementia in this patient.

The maximum MMSE score is 30 points. A score of 20 to 24 suggests mild dementia, 13 to 20


suggests moderate dementia, and less than 12 indicates severe dementia. On average, the MMSE
score of a person with Alzheimer disease declines 2 to 4 points per year.

Currently available pharmacological treatments for Alzheimer disease are cholinesterase inhibitors
(e.g., donepezil, rivastigmine and galantamine) and memantine.

Patients with Alzheimer disease (AD) have reduced cerebral production of acetylcholine
transferase, resulting in decreased acetylcholine production and impaired cortical cholinergic
function. Cholinesterase inhibitors increase cholinergic transmission by inhibiting cholinesterase at
the synaptic cleft.

Cholinesterase inhibitors have a modest benefit to patients with dementia due to Alzheimer
disease. These drugs improve alertness and function and maintain cognitive scores at or above the
baseline for up to 12 months, but do not prevent disease progression.

Cholinesterase inhibitors are associated with prominent adverse effects including:

Gastrointestinal symptoms as the most prominent adverse effects, including anorexia,


nausea, vomiting and diarrhea
Insomnia
Fatigue
Vivid dreams
Hallucinations
Asthma

1585 of 1943
Bradyarrhythmias and cardiac conduction abnormalities
hypertension
Dizziness
Cramps
Headache

Memantine is an N-methyl-D-aspartate (NMDA) receptor antagonist, with a distinct mechanism of


action from that of cholinesterase inhibitors. It appears to be neuroprotective. Memantine may be
helpful in patients with moderate to severe dementia who are deteriorating despite initial response
to a cholinesterase inhibitor (e.g., donepezil), or in previously untreated patients with moderate to
severe dementia.

This patient has moderate dementia. She also has a baseline bradycardia of 52 bpm. Use of
cholinesterase inhibitors for this patient, with pre-existing bradycardia, may result in profound
bradycardia and cardiac complication; therefore, initiation of memantine in preference of a
cholinesterase inhibitor would be the most appropriate treatment.

(Options A, C and E) Donepezil, rivastigmine, and galantamine can be associated with severe
bradycardia in this patient who already has bradycardia.

(Option D) Haloperidol is not used for treatment of dementia.

References

• MJA - Clinical practice guidelines for dementia in Australia

• Medscape - Donepezil

• Therapeutic Guidelines – Psychotropics


Last updated:
Time spent: QID:1153
2023-2-12

1586 of 1943
Which one of the following is the earliest feature of Alzheimer disease?

A. Impairment in visuospatial skills.


B. Forgetfulness.
C. Disorientation to familiar faces and objects.
D. Social inappropriateness.
E. Ataxia and motor function deficits.

Incorrect. Correct answer is B


45% answered correctly

Explanation:

Correct Answer Is B

Memory impairment and forgetfulness is an essential feature of Alzheimer disease and often its
earliest manifestation. Even when not the primary complaint, memory deficits can be elicited in
most patients with Alzheimer disease at presentation. Recent memory is affected early in the
course of the disease while remote memory impairment develops later.

Memory impairment in Alzheimer disease has a distinctive pattern. It starts with memory
impairment for the time of facts and events. Loss or orientation to place follows. Recognizing
familiar faces becomes affected later, as does memory for vocabulary and concepts.

(Option A) Loss of visuospatial skills also occurs, but usually not before forgetfulness. It can be
very prominent at presentation. Visuospatial impairments manifest as misplacement of objects
and difficulty navigating in unfamiliar and then familiar places. A patient with Alzheimer disease
becomes easily lost in unfamiliar places and then familial places such as the neighborhood or even
at home.

(Option C) Visual agnosia that is inability to recognize objects and prosopagnosia, defined as
inability to recognize faces, are due to visuospatial skills. None of these features are common to
precede forgetfulness and memory impairment.

(Options D and E) Misbehavior, social disgrace and motor function deficits usually occur late in the
course of the disease.

References

• UpToDate - Clinical features and diagnosis of Alzheimer disease


Last updated:
Time spent: QID:1154
2023-2-12

1587 of 1943
A 75-year-old woman is brought to your practice by her husband, who complains that she is not
herself anymore. The problem started with reduced self care and episodic disorientation and
forgetfulness. She refuses to bathe and barely changes her clothes. Recently, she has developed
difficulty walking and has had several falls. Last night, she started yelling and screaming because
she thought there are snakes slithering on the floor. On examination, she is a disheveled and
disoriented to time, place, and person. Fine resting tremors of her hands, mostly prominent on the
right, are noted. When she is asked to walk, she has difficulty starting and cannot stop when you
ask her to. Which one of the following is the most likely diagnosis?

A. Alzheimer disease.
B. Parkinson disease.
C. Schizophrenia.
D. Vascular dementia.
E. Lewy body dementia.

Incorrect. Correct answer is E


45% answered correctly

Explanation:

Correct Answer Is E

They key findings of cognitive impairment and unorganized behavior followed by deficits in motor
functions consistent with extrapyramidal involvement and visual hallucinations (snakes on the
floor), make Lewy body dementia the most likely diagnosis.

Lewy body dementia is a progressive, degenerative dementia of unknown etiology presenting with
dementia, motor signs, fluctuating cognition, and visual hallucinations. In Lewy body dementia,
cognitive impairment and dementia precedes extrapyramidal manifestations.

In summary, Lewy body dementia manifests with the following:

Dementia and fluctuations in cognitive function with varying levels of alertness and attention
Episodes of long staring and disorganized speech
Well-formed, detailed visual hallucinations
Early extrapyramidal features
Anterograde memory loss
Visuospatial impairment

(Option A) In Alzheimer disease, behavioral changes, hallucinations and motor deficits occur late in
the course of disease. In this woman this features have occurred early which is against Alzheimer
disease as a diagnosis.

(Option B) In Parkinson, memory and cognition remain intact until late in the course of the disease.
In Lewy body dementia, cognitive impairment and memory problems precede extrapyramidal
manifestations. On the other hand, visual hallucinations are not a feature of Parkinson disease.

(Option C) Schizophrenia is associated with hallucinations that are mostly auditory, delusions, and
other psychotic feature for at least 6 months. Patients with schizophrenia do not have motor
1588 of 1943
symptoms. In addition, the age of patient is unusual for development of schizophrenia.

(Option E) In vascular dementia, there is often a history of stroke or cerebrovascular disease. The
condition presents with focal neurological signs on examination, abrupt symptom onset related to
stroke in a step-wise fashion, and evidence of old infarcts on CT scan. Risk factors for
cardiovascular disease such as hypertension, diabetes, smoking, and atrial fibrillation are often
present. It is more common in men.

References

• Medscape - Lewy Body Dementia

• UpToDate - Clinical features and diagnosis of dementia with Lewy bodies


Last updated:
Time spent: QID:1155
2023-2-12

1589 of 1943
A 50-year-old man with past medical history of bipolar disorder with psychotic features presents to
you, complaining of decreased mood and appetite for the past 2 weeks. On examination, you notice
that he does not talk much and only gives short answers to your questions. He seems to have
decreased concentration and avoid eye contact. He, however, denies hallucinations and paranoid
ideation. He is currently on olanzapine, which he has been taking for the past 2 years. Which one of
the following is the most appropriate initial management of this patient?

A. Continue olanzapine and add escitalopram.


B. Gradually decrease olanzapine and add escitalopram.
C. Increase the dose of olanzapine and wait for the effect.
D. Switch to venlafaxine.
E. Both A and C.

Incorrect. Correct answer is E


45% answered correctly

Explanation:

Correct Answer Is E

Many mood stabilizers (olanzapine here) have bimodal effect. They can be used for treatment of
both depression and mania. If a patient, who has successfully been stable on prophylactic dose of
a particular mood-stabilizer, develops acute depression, two approaches can be considered with
about the same efficacy:

Adding an antidepressant to the prophylactic mood stabilizer: the choices of the drug would
be the same as for major depression. SSRIs are first line options.

Increasing the dose of prophylactic mood stabilizer.

Either addtion of escitalopram or increasing the dose of olanzapine can be effectively used for this
patient.

NOTE - Cessation or dose reduction of a mood stabilzer can result in relapse of mania and is not
appropriate.

References

• Therapeutic Guidelines - Psychotropic; available from http://tg.org.au


Last updated:
Time spent: QID:69
2023-2-12

1590 of 1943
A 79-year-old woman presents with tiredness and forgetfulness of recent onset. She scores 28/30
on a mini mental status exam (MMSE). Which one of the following is the most likely diagnosis?

A. Alzheimer disease.
B. Pick disease.
C. Anxiety.
D. Multi-infarct dementia.
E. Pseudodementia.

Incorrect. Correct answer is E


45% answered correctly

Explanation:

Correct Answer Is E

Mini-mental status exam (MMSE) is a practical test performed for initial assessment of cognitive
function. The total score is 30. An MMSE score≥25 makes dementia less likely, while scores less
than 25 suggest dementia.

Patients with dementia often have MMSE scores of less than 25. Alzheimer disease, Pick disease
(frontotemporal dementia) and multi-infarct (vascular dementia) are associated with genuine
dementia and decreased MMSE scores of less than 25. This patient has a MMSE score of 28 and is
very unlikely to be demented. For him, pseudodementia could be the most likely diagnosis.

Depressive pseudodementia is a term commonly used to describe a condition in a patient who


experiences a cognitive deficit secondary to a primary psychiatric disturbance including major
depressive disorder, bipolar disorder, schizophrenia, Ganser’s syndrome, and adjustment disorder.
Pseudodementia mimics dementia, and is usually reversible once the primary diagnosis has been
treated.

(Options A, B and D) While an MMSE score of <25 is likely in both dementia and pseudodementia, a
score of >25 almost excludes dementia. Alzheimer disease, Pick disease and multi-infarct
dementia are often associated with a marked decrease in MMSE score.

(Option C) Although tiredness can be seen in some patients with anxiety disorders, particularly in
those with generalized anxiety disorder, forgetfulness in not a common feature.

NOTE - The history of disturbances in pseudodementia is often short and of abrupt onset,
whereas dementia is often more insidious. Clinically, people with pseudodementia differ from
those with true dementia when their memory is tested. They typically answer that they do not
know the answer to a question, and their attention and concentration are often intact, and they
may appear upset or distressed when they fail to answer. Those with true dementia will give
wrong answers, have poor attention and concentration, and appear indifferent or unconcerned.

References

• Medscape - What is pseudodementia in patients with major depressive disorder (clinical


depression)?
1591 of 1943
Last updated:
Time spent: QID:1156
2023-2-12

1592 of 1943
Which one of the following is the method of choice in the management of borderline personality
disorder?

A. Cognitive behavior therapy.


B. Dialetical behavior therapy.
C. Exposure therapy.
D. Family therapy.
E. Interpersonal therapy.

Incorrect. Correct answer is B


45% answered correctly

Explanation:

Correct Answer Is B

Borderline personality disorder (BPD), a member of cluster B (dramatic, emotional) personality


disorders, is characterized by a pattern of unstable interpersonal relationships and marked
impulsivity. Patients with this condition swing wildly between devaluing and idealizing others,
labeling people as wholly good or wholly bad -a phenomenon known as ‘splitting’. Borderline
patients generally have difficulty calming down and controlling anger once they have become
upset. Such patients tend to be unpredictable. As a result, they frequently have anger outbursts and
engage in impulsive behavior such as substance abuse, overspending, binge eating, and suicidal or
self-mutilating behavior. Chronic feelings of emptiness are common.

The first step in management is by building a positive therapeutic relationship that plays a central
role in the management of BPD and that helps identify and break the self-defeating interpersonal
patterns characteristic of the previous relationships. Referral for longer-term psychotherapy with a
psychiatrist and/or clinical psychologist is an important consideration in long-term treatment
planning.

A specialized form of very intensive cognitive therapy, known as dialectical behavior therapy (DBT),
is currently considered to be the treatment of choice for women (not approved for men or
adolescents) with BPD for whom reduction in self-harm, anger, anxiety, or depression are treatment
goals. DBT significantly reduces self-harm and substance use disorder. People aged 14–18 years
with BPD or clinically significant features of BPD should be offered time-limited structured
psychological therapies that are specifically designed for BPD. For adolescents, younger than 14
years with features of BPD, clinical psychological support and monitoring and involving their
families are recommended.

Medicines should not be used as primary therapy for BPD, because they have only modest and
inconsistent effects, and do not change the nature and course of the disorder. There is no current
medication that is approved for the management of BPD, although there is some evidence that low-
dose atypical antipsychotics and/or mood stabilizers may be effective in treating core symptoms.

(Option A) Cognitive behavior therapy was adapted for use with personality disorders, but the
results are still preliminary. Manual-assisted cognitive therapy failed to reduce the repetition of self-
harm in borderline personality disorder.

1593 of 1943
(Option C) Exposure therapy is one of the commonly used strategies of cognitive behavioral
therapy (CBT). CBT has been found to be the most effective treatment for anxiety disorders. In
exposure therapy, the psychologist guides the person through a series of real or imaginary
scenarios to confront specific fears. Through a gradual process of exposure, the person learns to
cope more effectively with these fears, and with practice, the anxious response naturally
decreases.

(Option D) Family therapy is a type of psychological counseling (psychotherapy) that helps family
members improve communication and resolve conflicts. A psychologist or licensed therapist
usually provides the therapy. It helps family members solve family problems and conflicts, through
improved communication, greater awareness of childhood and adolescence issues, and a better
understanding of special family situations such as separation, divorce, remarriage, serious illness,
or death in the family as well as mixed marriages in terms of culture, race, or religion.

(Option E) Interpersonal psychotherapy (IPT) is a time-limited, empirically validated treatment for


mood disorders. IPT is an add-on therapy to antidepressant medication.

References

• RACGP - Managing borderline personality disorder and substance use

• Australian Psychology Society - Anxiety Disorders

• World Psychiatry - Interpersonal psychotherapy: principles and applications


Last updated:
Time spent: QID:1168
2023-2-12

1594 of 1943
A 23-year-old woman presents to your clinic asking for a medical certificate to give to her employer.
She says she was not able to go to work for the past two days because she had diarrhea and a
fever. You explain that you cannot do that because she has never been your patient and you can
make a false statement. Upon your refusal, she becomes irritated and starts shouting at you and
threatening you. You check the clinic records and realize that she was previously treated for a self-
inflicted wrist cut in the same clinic. Which one of the following is the most likely diagnosis?

A. Malingering.
B. Narcissistic personality disorder.
C. Borderline personality disorder.
D. Histrionic personality disorder.
E. Munchhausen syndrome.

Incorrect. Correct answer is C


45% answered correctly

Explanation:

Correct Answer Is C

In this case, the woman seems to malinger to obtain a secondary gain. The term denotes the
deliberate simulation or exaggeration of symptoms for the purpose of obtaining some gain, such
as financial compensation. The presence of a personality disorder is a risk factor for malingering.
Malingering, her history of self-inflicted wrist cuts, and her behavior of difficulty controlling her
anger suggest borderline personality disorder as the most likely diagnosis.

Borderline personality disorder (BPD) is classified under cluster B (dramatic, emotional) personality
disorders, and is characterized by a pattern of unstable interpersonal relationships and marked
impulsivity. Patients with this condition swing wildly between devaluing and idealizing others,
labeling people as wholly good or wholly bad-a phenomenon popularly known as ‘splitting’.
Borderline patients generally have a hard time calming down, and they have difficulty controlling
anger once they have become upset, their moods tend to be unpredictable. As a result, they
frequently have anger outbursts and engage in impulsive behaviors such as substance abuse,
overspending, binge eating, or suicidal or self-mutilating behavior. Chronic feelings of emptiness
are common.

(Option A) Malingering is not a medical diagnosis but a description of behavior. DSM-5 describes
malingering as the intentional production of false or grossly exaggerated physical or psychological
problems. Motivation for malingering is usually external (e.g., avoiding military duty or work,
obtaining financial compensation, evading criminal prosecution, or obtaining drugs). In other
words, malingering is assuming a sick role for a planned gain.

(Option B) Narcissistic personality disorder is classified under cluster B (dramatic, emotional)


personality disorders, and is characterized by arrogance, grandiosity, a need for admiration, and a
tendency to exploit others. Individuals with the condition are preoccupied with fantasies about
success, power, brilliance, beauty, or the perfect mate. There is often a sense of excessive
entitlement. They usually demand special treatment.

1595 of 1943
(Option D) People with histrionic personality disorder are highly emotional and dramatic, have an
excessive need for attention and approval, and may be obsessed with their appearance. Sexual
seductiveness is another feature.

(Option E) Factitious disorder (Munchausen syndrome) is characterized by the intentional


production of false physical or psychological signs or symptoms to assume the sick role. Unlike
patients with malingering, those with the factitious disorder receive no secondary gain. It is called
Munchausen's syndrome when patients display symptoms of the factitious disorder that are
predominantly physical in nature. Multiple tests usually show no abnormality, but they will not
accept that they do not have an illness. They may undergo numerous surgeries resulting in scarring
and adhesions.

References

• The Department of Health - What are the main types of personality disorder?

• American Psychiatric Association - Diagnostic and Statistical Manual of Mental Disorders - Fifth
Edition- (DSM-5)- Washington DC: American Psychiatric Press Inc; 2013. 726
Last updated:
Time spent: QID:1169
2023-2-12

1596 of 1943
A 25-year-old man, who has history of agitations, multiple crimes, drug abuse, and theft, comes to
you with complaints of feeling sad ,losing interests, insomnia, fatigue, and loss of appetite. He says
that he is not using any drugs now but he has been recently charged with a minor assault. Which
one of the following is the most likely explanation to his behavior?

A. Antisocial personality disorder.


B. Malingering.
C. Bipolar disorder.
D. Substance abuse.
E. Conduct Disorder.

Incorrect. Correct answer is B


45% answered correctly

Explanation:

Correct Answer Is B

Apparently, this man has antisocial personality disorder (ASPD), which is characterized by a pattern
of disregard for and violation of the rights of others, which typically begins by middle adolescence
and continues into adulthood. Individuals with this condition are aged ≥18 years and have had
conduct disorder in childhood.

Individuals with ASPD routinely engage in illegal activities (e.g. drug use, assault, or theft),
endanger the well-being of others, and blatantly disregard others’ legal rights. They tend to be
aggressive and impulsive, and have difficulty maintaining employment. They frequently lie, and this
explains the malingering behavior in this young man to obtain secondary gain to provide the court
with.

DSM-IV defines malingering as ‘the intentional production of false or grossly exaggerated physical
or psychological symptoms, motivated by external incentives’. It stresses that clinicians should
suspect malingering when two or more of the following conditions are present:

The symptoms are reported within a forensic context


They sharply contrast with objective findings
There is lack of cooperation during diagnostic evaluation
The patient meets criteria for ASPD

The new edition of the DSM (i.e. the DSM-V) does not contain substantial revisions of how it
portrays malingering. Despite these features, however, affected individuals frequently display a
superficial charm that allows for the manipulation of others.

(Option A) Antisocial personality disorder is a psychiatric diagnosis and is considered a risk factor
for malingering. However, the question is asking about the behavior intention not the diagnosis.

(Option C) Bipolar disorder is a mood disorder characterized by periods of mania and depression
Manic episodes involve abnormally elevated mood, grandiosity, decreased need for sleep, racing
thoughts, talkativeness, and impulsivity (e.g. spending exorbitant amounts of money, gambling,
sexual indiscretion) This patient exhibits impulsivity and tension, which are specific to stealing. She

1597 of 1943
demonstrates none of the other concurrent manic symptoms required for a diagnosis of bipolar
disorder.

(Option D) While this patient denies substance abuse, patients do not always tell the truth. A urine
toxicology screen would be indicated if suspected. However, malingering is more likely here for
evading criminal prosecution not for obtaining a drug.

(Option E) Children and adolescents with conduct disorder often become adults with antisocial
personality disorder. The diagnosis of conduct disorder requires at least three symptoms from the
following:

Aggression towards people or animals


Destruction of property
Deceitfulness
Theft
A serious violation of rules

Adult patients have ASPD not conduct disorder, which is by definition a disorder of childhood and
adolescence.

References

• http://emedicine.medscape.com/article/293206-overv
Last updated:
Time spent: QID:1170
2023-2-12

1598 of 1943
A 27-year-old woman presents to your office for evaluation. She left home when she was 12 and
lived on streets where she started using alcohol and marijuana. Her criminal records show multiple
arrests for shop-liftings and prostitution. During the interview, she is calm and communicative.
When you ask if she has ever considered going back home, she answers I have nothing to do at
home, and I am not going back there. Which one of the following is the most likely underlying
cause of this history?

A. Borderline personality disorder.


B. Obsessive compulsive disorder (OCD).
C. Substance abuse.
D. Histrionic personality disorder.
E. Narcissistic personality disorder.

Incorrect. Correct answer is C


45% answered correctly

Explanation:

Correct Answer Is C

This woman has started abusing substances in an early age, and has multiple jail stays due to drug
issues and shop-lifting. At first glance, one might consider the diagnosis of antisocial personality
disorder; however, this woman does not fulfill criteria for such a diagnosis. Evidence of conduct
disorder with onset before age 15 is one of the criteria to make a diagnosis of antisocial
personality disorder. Although this woman has left home at an early age (<15 years) and has been
involved in drug use as well as criminal charges, these do not fulfill criteria for conduct disorder.

For conduct disorder to be present, one of the important criteria is the presence of at least 3 out of
the following 15 in the history:

1. Often bullies, threatens, or intimidates others.


2. Often initiates physical fights.
3. Has used a weapon that can cause serious physical harm to others (e.g., a bat, brick, broken
bottle, knife, gun).
4. Has been physically cruel to people.
5. Has been physically cruel to animals.
6. Has stolen while confronting a victim (e.g., mugging, purse snatching, extortion, armed
robbery).
7. Has forced someone into sexual activity.
8. Has deliberately engaged in fire setting with the intention of causing serious damage.
9. Has deliberately destroyed others’ property (other than by fire setting).
10. Has broken into someone else’s house, building, or car.
11. Often lies to obtain goods or favors or to avoid obligations (i.e., “cons” others).
12. Has stolen items of nontrivial value without confronting a victim (e.g. shoplifting, but without
breaking and entering; forgery).
13. Often stays out at night despite parental prohibitions, beginning before age 13 years.
14. Has run away from home overnight at least twice while living in the parental or parental
surrogate home, or once without returning for a lengthy period.
15. Is often truant from school, beginning before age 13 years.
1599 of 1943
This woman only has had 2 from the above criteria that are (1) leaving home early at age 12 and (2)
shop-lifting. So she did not have conduct disorder as an essential part for diagnosis of antisocial
personality disorder in adulthood. Moreover, oshe lacks other aspects of antisocial personality
disorder such as aggressiveness, recklessness towards self and others' safety.

As a matter of fact, drug abuse appears to be the most likely cause of her presentation. Shop-lifting
and prostitution are the most common crimes committed to fund drug habits (economic related
crime).

(Option A) The two most common personality disorders associated with substance use disorders
are antisocial personality disorder and borderline personality disorder. This patient does not meet
the criteria for none.

(Option B) Obsessive-compulsive disorder (OCD) is characterized by obsessions or intrusive


thoughts that cause significant anxiety, or forced engagement in repetitive compulsions in an
attempt to avoid otherwise significant psychological stress. Patients with OCD often recognize the
unreasonable nature of their thoughts and behaviors, but are unable to stop them due to anxiety.
No points in the history favor OCD as a diagnosis.

(Option D) People with histrionic personality disorder are highly emotional and dramatic, have an
excessive need for attention and approval, and may be obsessed with their appearance. This
disorder usually begins by late teens or early 20s and seems to be caused by genetic
predisposition and early childhood events. This patient does not meet the criteria for histrionic
personality disorder.

(Option E) Narcissistic personality disorder is classified as a Cluster B (dramatic, emotional)


personality disorder and is characterized by arrogance, grandiosity, a need for admiration, and a
tendency to exploit others. They are preoccupied with fantasies about success, power, brilliance,
beauty or the perfect mate. Patients with this condition often have a sense of excessive
entitlement and may demand special treatment, Fifty percent of those with narcissistic personality
disorder abuse alcohol and drugs, with many facing addiction and dependency. This patient does
not meet the criteria for narcissistic personality disorder.

TOPIC REVIEW

Diagnosis of antisocial personality disorder:

According to the DSM-5, there are four diagnostic criterion, of which Criterion A has seven sub-
features.

(A) Disregard for and violation of others rights since age 15, as indicated by one of the seven sub
features:

1. Failure to obey laws and norms by engaging in behavior which results in criminal arrest, or
would warrant criminal arrest
2. Lying, deception, and manipulation, for profit or self-amusement
3. Impulsive behavior
4. Irritability and aggression, manifested as frequently assaults others, or engages in fighting
5. Blatantly disregards safety of self and others
6. A pattern of irresponsibility
7. Lack of remorse for actions
1600 of 1943
The other diagnostic Criterion are:

(B) The person is at least age 18

(C) Conduct disorder was present by history before age 15

(D) The antisocial behavior does not occur in the context of schizophrenia or bipolar disorder

References

• https://www.ncadd.org/about-addiction/alcohol-drug

• http://epublications.marquette.edu/cgi/viewcontent
Last updated:
Time spent: QID:1171
2023-2-12

1601 of 1943
A 27-year-old woman presents to your practice with complaints about compromised interpersonal
relationships. She has no friends and has been fired from work several times in the past few years.
She relates she could not make any friends at school as well and dropped school early. She also
admits to a lack of concentration. Which one of the following could be the most likely diagnosis?

A. Borderline personality disorder.


B. Antisocial personality disorder.
C. Attention deficit - hyperactivity disorder (ADHD).
D. Depression.
E. Asperger syndrome.

Correct
45% answered correctly

Explanation:

Correct Answer Is A

Compromises in interpersonal relationships and the inability to hold a job are most consistent with
the diagnosis of borderline personality disorder.

Borderline personality disorder is classified under Cluster B (dramatic, emotional) personality


disorders, characterized by a pattern of unstable interpersonal relationships and marked
impulsivity. Patients with this condition swing wildly between devaluing and idealizing others,
labeling people as wholly good or wholly bad - a phenomenon known as ‘splitting’. Borderline
patients generally have a hard time calming down, and they have difficulty controlling anger once
they have become upset, their moods tend to be unpredictable. As a result, they frequently have
anger outbursts and engage in impulsive behavior such as substance abuse, overspending, binge
eating, or suicidal or self-mutilating behavior. Chronic feelings of emptiness are common.

(Option B) Antisocial personality disorder is diagnosed in patients aged 18 years or older, who
engage in illegal activities (e.g., theft or assault) and disregard others’ rights. These individuals
often display evidence of conduct disorder as minors.

(Option C) Although this woman admits to a lack of concentration, there is insufficient information
to substantiate a diagnosis of attention deficit hyperactivity disorder (ADHD), characterized by
hyperactivity, short attention span, and easy distractibility.

(Option D) This patient has only one symptom of major depressive disorder which is ‘lack of
concentration, but does not meet the full criteria including five or more of the following symptoms
for at least two weeks:

Depressed mood
Loss of interest
Early-morning awakening
Weight loss
Low energy
Impaired concentration
Feelings of worthlessness

1602 of 1943
Excessive guilt

(Option E) Patients with Asperger syndrome present in early childhood. It is four times more
common in males. Patients with Asperger's have impairments in reciprocal social interactions and
restricted interests. They often desire relationships but lack awareness of social conventions,
including the use of nonverbal communication. The absence of language delay (not a feature in
Asperger syndrome) helps distinguish Asperger syndrome from autism.

References

• Psych Central - Symptoms of Borderline Personality Disorder (BPD)

• Psych Central - What Is a Personality Disorder?


Last updated:
Time spent: QID:1173
2023-2-12

1603 of 1943
A 26-year-old accountant comes to your clinic complaining of flu-like symptoms for three days. He
arrives ten minutes late for his appointment but insists on being seen immediately. He says that he
has some very important matters to attend to and is, therefore, unable to wait. The receptionist
calmly informs him that the official policy is to see patients in order of arrival and that she will call
him when it is his turn. Upon hearing this, he flies into a rage and screams ‘How dare you! Don't you
realize who I am?’ Given this presentation, which of the following is the most likely diagnosis?

A. Paranoid personality disorder.


B. Obsessive-compulsive personality disorder.
C. Antisocial personality disorder.
D. Narcissistic personality disorder.
E. Borderline personality disorder.

Incorrect. Correct answer is D


45% answered correctly

Explanation:

Correct Answer Is D

The clinical scenario is consistent with narcissistic personality disorder as the most likely
diagnosis.

DSM-5 criteria for narcissistic personality disorder include most of all of these features:

Grandiosity - an exaggerated sense of self-importance


Fixated on fantasies of power, success, intelligence, attractiveness, etc.
Self-perception of being unique, superior, and associated with high-status people and
institutions
Needing constant admiration from others
Sense of entitlement to special treatment and to obedience from others
Exploitative of others to achieve personal gain
Unwilling to empathize with others' feelings, wishes, or needs
Being envious of others and believing others envy him
Pompous and arrogant demeanor
Behaving in an arrogant or haughty manner

(Option A) Paranoid personality disorder is characterized by distrust and fear. Individuals with this
disorder frequently have unfounded suspicions and misinterpret the motives of others. They may
find it difficult to confide in or forgive people.

(Option B) Patients with obsessive-compulsive personality disorder have a preoccupation with


orderliness and perfectionism that negatively impacts their functioning. They are often overly
devoted to their work, are stubborn, and believe there is only one way to do things. They can have
difficulty throwing out worn items, often have trouble with relationships because of their need for
perfection, and even need to play games by following the rules precisely. They often do not see
their behavior as a problem. Despite the similar names, There is no specific association between
OCD and any specific personality disorder, including obsessive-compulsive personality disorder.

1604 of 1943
(Option C) Antisocial personality disorder can be a diagnosis in patients aged 18 years or older,
who engage in illegal activities (e.g. theft or assault) and disregard others’ rights. These individuals
often display evidence of conduct disorder as minors.

(Option E) Individuals with borderline personality disorder show a pattern of instability in


relationships and marked impulsivity They swing wildly between scorning and idealizing people
(with others considered wholly good or wholly bad, a phenomenon known as "splitting") They tend
to be impulsive and reckless and may demonstrate suicidal or self-mutilating behavior. Feelings of
anger and chronic emptiness are common.

References

• Psych Central - Narcissistic Personality Disorder: Symptoms, Causes, and Treatment

• Psych Central - https://psychcentral.com/disorders/personality


Last updated:
Time spent: QID:1174
2023-2-12

1605 of 1943
A young woman, who lives alone, comes to your practice with a complaint of insomnia. She left
high school to spend countless hours in her room practicing some supernatural medicine. She is
extremely fascinated by her home medicine and says: ‘I have some powers in the cure, I am willing
to discuss it happily only with my clients and those who are interested'. She has no social life or close
friends and does not enjoy attending parties or leaving home that much. Which one of the following
personality disorders she most likely has?

A. Paranoid personality disorder.


B. Obsessive-compulsive personality disorder.
C. Schizotypal personality disorder.
D. Schizoid personality disorder.
E. Avoidant personality disorder

Incorrect. Correct answer is C


45% answered correctly

Explanation:

Correct Answer Is C

Intentional reclusiveness, odd beliefs about supernatural issues, and lack of interpersonal
relationships make schizotypal personality disorder the most likely diagnosis.

Schizotypal personality disorder is characterized by a pattern of odd and eccentric behavior and
beliefs and a reduced capacity for close relationships. These patients usually exhibit ‘magical
thinking’. They may have bizarre fantasies or believe in telepathy, clairvoyance, or the concept of a
sixth sense. They often have paranoid ideation and unusual perceptual experiences.

(Option A) Paranoid personality disorder is characterized by a pervasive distrust and


suspiciousness of others in a way that their actions are interpreted as malevolent as indicated by:
reluctance to confide in others, preoccupation with unjustified doubts, suspicions that others are
being exploitative, and finding hidden meaning in benign remarks.

(Option B) Obsessive-compulsive personality disorder is characterized by a preoccupation with


orderliness and perfectionism that negatively impacts functioning. They are often overly devoted to
their work, are stubborn, and believe there is only one way to do things. They can have difficulty
throwing out worn-out and useless items, and often have trouble with relationships because of
their need for perfection. They often do not see their behavior as a problem.

(Option D) Schizoid personality disorder is characterized by the pervasive pattern of detachment


from social relationships and restricted emotions, as evident by four or more of the following
characteristics:

Lack of desire for close relationships


No interest in sexual experiences
Indifference to praise or criticism
Emotional coldness
Choosing solitary activities

1606 of 1943
NOTE - Schizoid personality disorder shares many features with schizotypal disorder. The
distinctive features are bizarre and eccentric appearance and beliefs which are present in
schizotypal personality disorder and absent in schizoid personality disorder.

(Option E) Avoidant personality disorder is characterized by hypersensitivity to criticism, social


inhibition, and feelings of inadequacy. Although these individuals want friendships, they avoid (as
the name suggests) such intimate relationships because they fear ridicule. They also perceive
themselves as inferior and are reluctant to engage in new activities or take risks for fear of being
embarrassed.

References

• Psych Central - Schizotypal Personality Disorder: Symptoms, Causes, and Treatment

• Psych Central - What Is a Personality Disorder?


Last updated:
Time spent: QID:1175
2023-2-12

1607 of 1943
Which one of the following is the most common cause of death in people with schizophrenia?

A. Cardiovascular disease.
B. Diabetes mellitus.
C. Suicide.
D. Smoking-related lung disease.
E. Accidents.

Correct
45% answered correctly

Explanation:

Correct Answer Is A

On average, people with schizophrenia have a lifespan 20 years shorter than the general
population. More than two-thirds of people with schizophrenia, compared with about a half in the
general population, die of coronary heart disease. This is partly due to factors such as smoking,
increased rates of diabetes, and metabolic problems caused by the use of some antipsychotic
medications. These factors often worsen once a cardiac condition arises because people with
schizophrenia are less likely to make the necessary lifestyle changes, such as diet and exercise, to
offset the problem.

Although death due to suicide is a contributing factor, the lifetime risk of suicide in schizophrenia is
usually quoted to be around 10%. However, overall, the evidence is for a lower rate (around 5%). The
highest risk is soon after diagnosis.

References

• https://www.sciencedaily.com/releases/2012/10/1210

• http://apt.rcpsych.org/content/aptrcpsych/14/3/217
Last updated:
Time spent: QID:1176
2023-2-12

1608 of 1943
A 54-year-old man is brought to the Emergency Department on a stretcher while accompanied by
the police. He is agitated and says: ‘I know you are taking me to the prison. I know that you will kill
me.’ Which one of the following conditions could be the most likely explanation to this
presentation?

A. Delirium.
B. Dementia.
C. Delusional disorder.
D. Drug abuse.
E. Paranoid personality disorder.

Correct
45% answered correctly

Explanation:

Correct Answer Is A

Of the options, delirium best explains the agitation and delusional state of mind in this man.

Delirium, also called acute confusional state, is a decline from a previously attained baseline level
of cognitive function due to organic causes. Delirium is characterized by a fluctuating course,
attentional deficits, and generalized severe disorganization of behavior. It typically involves other
cognitive deficits, changes in arousal (hyperactive, hypoactive, or mixed), perceptual deficits, and
psychotic features such as hallucinations and delusions that are often short-lived. Delirium itself is
not a disease, but a clinical syndrome.

(Option B) Dementia is characterized by impairment of memory and at least one other cognitive
domain such as aphasia, apraxia, agnosia, or executive function. These must represent a decline
from previous level of function and be severe enough to interfere with daily function and
independence. Although this patient may also have dementia, his current presentation of agitation
and delusions are acute and most likely due to delirium.

(Option C) Delusional disorder is an illness characterized by at least one month of delusions but no
other psychotic symptoms. Delusions are false beliefs based on incorrect inference about external
reality that persist despite the evidence to the contrary. These beliefs are not ordinarily accepted by
other members of the person’s culture or subculture.

(Option D) Drug abuse can cause delirium; however, it is an underlying cause of the condition, not a
diagnosis.

(Option E) Paranoid personality disorder is characterized by distrust and fear. Individuals with this
disorder frequently have unfounded suspicions and misinterpret the motives of others. They may
find it difficult to confide in or forgive people. Agitation, as in this man, is not a feature.

References

• Mind - Caring for a person experiencing Delusions

1609 of 1943
• Medscape - Delirium
Last updated:
Time spent: QID:1177
2023-2-12

1610 of 1943
Which one of the following is the most common anxiety disorder encountered in general practice?

A. Phobic disorders.
B. Post-traumatic stress disorder (PTSD).
C. Panic disorder.
D. Obsessive compulsive disorder (OCD).
E. Generalized anxiety disorder (GAD).

Incorrect. Correct answer is E


45% answered correctly

Explanation:

Correct Answer Is E

Anxiety disorders are one of the most common psychiatric disorders encountered in general
practice and at primary care level. In general, anxiety disorders are categorized into the following
six groups:

Panic disorder - sudden bursts of extreme anxiety that are accompanied by symptoms like a
pounding heart, sweaty palms, and shortness of breath or nausea
Agoraphobia - anxiety about being in places or situations from which it is difficult to escape if
a panic attack occurs
Social phobia (also called social anxiety disorder) - strong fear of social interaction or
performance situations because of the potential for embarrassment of humiliation
Generalized anxiety disorder (GAD) - long periods of uncontrollable worry about everyday
issues or events, which is typically accompanied by feelings of fatigue, restlessness or
difficulty in concentrating.
Posttraumatic stress disorder (PTSD) - recurrent and intrusive memories of a trauma, feelings
of emotional numbing and detachment, and increases in emotional arousal, such as
irritability and disturbed sleep, resulting from a previous traumatic event
Obsessive-compulsive disorder (OCD) - repeated thoughts, images or impulses that the
person feels are inappropriate, and repetitive behaviors, designed to reduce the anxiety
generated by the thoughts

Of the above disorders, generalized anxiety disorder (GAD) is the most common anxiety disorder
encountered in general practice and at primary care level. GAD is also the most common anxiety
disorder among older population.

Note that the question asks about the most common anxiety disorder faced in general practice
setting, not the most common anxiety disorder overall. According to the ‘Mental Health of
Australia’, a survey in 2007 showed that PTSD was the most common reported anxiety disorder in a
12-month period.

NOTE - An estimated 12% of all general practice encounters are related to mental health. Of
these mental conditions, depression is tne most common one (34%), followed by anxiety
disorders (14.8%).

1611 of 1943
Michael, 35 years, has recently been started on fluoxetine for treatment of major depression
diagnosed based on findings of low mood, decreased sleep and appetite and loss of concentration.
Today he has presented because despite the fact that his depressive symptoms has satisfactorily
improved with treatment, he has been experiencing terrifying sleep disturbances. He explains that
he wakes up in the middle of the night and sees scary black figures around his bed. He wants to
get off the bed, but is paralyzed to do so. He cannot even move a finger, shout or call for help. The
whole experience is so frightening that he is afraid of going to sleep at all. He does not have such a
problem while awake. Which one of the following is the most appropriate management for him?

A. Reassure that this is normal and resolves with time.


B. Add an antipsychotic medication.
C. Advise that he uses a sleep mask during sleep.
D. Switch to another antidepressant.
E. Advise sleep hygiene and routine.

Incorrect. Correct answer is D


45% answered correctly

Explanation:

Correct Answer Is D

The sleep disturbance described is characteristic of sleep paralysis. Sleep paralysis is a sleep
disorder in which either during falling asleep or waking up, an individual temporarily experiences
inability to move, speak and react. Sleep paralysis is a transitional state between wakefulness and
sleep, characterized by an inability to move muscles, often accompanied by terrifying
hallucinations to which no reaction can be made due to paralysis. These hallucinations are often
described as a person or supernatural creature suffocating or terrifying the individual,
accompanied by a feeling of pressure on one's chest and difficulty breathing. Another common
hallucination type involves intruders (human or supernatural) entering one's room or lurking outside
one's window, accompanied by a feeling of dread. Feelings such as being abducted by aliens have
also been reported.

The pathophysiology of sleep paralysis has been hypothesized to be a disrupted REM sleep that
normally induces complete muscle atonia to prevent sleepers from acting out their dreams.
Normally muscle tone is regained before the individual wakes up. In sleep paralysis the sequence is
reversed resulting in the person regains consciousness while muscles are still in atonic state.

Proposed risk factors for sleep paralysis (and other sleep disturbances) include sleep disruption,
traumatic life events, anxiety and depression and use of antidepressants.

This patient has developed sleep paralysis after he has been started on fluoxetine. Multiple studies
have shown a link between antidepressants use and sleep architecture disturbances. Interestingly,
selective serotonin reuptake inhibitors (SSRIs) have been used for treatment of some sleep
disorders including sleep paralysis, while they can also induce such disorders. Less so are
serotonin-norepinephrine reuptake inhibitors (SNRIs).

In this scenario, sleep paralysis has developed after starting the patient on the SSRI fluoxetine,
making the medication the likely cause of his problem. Considering this possibility, changing
1612 of 1943
fluoxetine with another antidepressant with less likelihood of inducing sleep disturbances can be
an appropriate management option.

(Option A) Sleep paralysis is usually a horrifying experience and the patient cannot be reassured.
No evidence suggests that the condition is temporary and resolves with time.

(Option B) Addition of an antipsychotic has not shown to be of proven benefit for management of
sleep paralysis.

(Options C and E) In sleep paralysis the problem is not going to or maintaining sleep. It often
occurs while the patient is falling asleep or is waking up; therefore, measures such as wearing a
sleep mask or sleep hygiene and routine are unlikely to be useful.

References

• PubMed - Parasomnias and Antidepressant Therapy: A Review of the Literature


Last updated:
Time spent: QID:1178
2023-2-12

1613 of 1943
A 45-year-old woman presents to your practice with complaints of sadness, tearfulness when
alone, and difficulty initiating and maintaining sleep for the past 2 months. She works as a part-
time secretary and seems to have difficulties concentrating at work. Four weeks ago, she was
started on St. John’s wort tablets 300 mg, 8-hourly, by a naturopath; however, she has become even
more distracted, tearful and sleepless. Assessment establishes the diagnosis of major depression
but no suicidal ideation or attempts. Which one of the following is the most appropriate
management option?

A. Add lithium, 600mg daily, to augment the effect of St. John’s wort.
B. Add Omega 3, 5 g daily.
C. Stop St. John’s wort, wait for one week as the washout period, and start her on
venlafaxine 75 mg daily.
D. Increase the doe of St. John’s wort to 600 mg 8-hourly.
E. Continue the same dose of St. Johns wort and add paroxetine 20 mg, 12-hourly.

Incorrect. Correct answer is C


45% answered correctly

Explanation:

Correct Answer Is C

St. John’s wort (SJW) is a natural, herbal medicine that is reported to treat depression. It also has
been reported to be useful for treatment of premenstrual syndrome (PMS), menopausal symptoms,
anxiety, seasonal affective disorder (SAD), and even obsessive-compulsive disorder (OCD).

Therapeutic ingredients of SJW include hypericin, pseudohypericin and various xanthones.


Although the mechanism of action is not fully understood, it appears that the antidepressant
activity of SJW is through a combination of mechanisms including inhibition of serotonin,
norepinephrine and dopamine synaptic reuptake and via the neurotransmitters gamma-
aminobutyric acid (GABA) and glutamate.

Studies have shown that SJW can be as effective as standard antidepressants in treatment of mild
to moderate depression, but no more effective than placebo in severe depression. However,
patients are more compliant because of less adverse effects. Despite these facts, SWJ has not
been included in mainstream treatment guidelines due to uncertainty about appropriate dose,
persistence of effects, variations in the nature of preparations that makes the effects
unpredictable, and serious drug interactions. One major concern about SJW is its interactions with
other antidepressants, mood stabilizers, and anticonvulsants. Serotonin syndrome is the most
feared condition associated with such interactions. For this reason, concomitant use of St. John’s
wort with such drugs is not recommended and should be discouraged.

NOTE - Another concern is the fact that concomitant use of SWJ and oral contraceptives can
reduce the efficacy of the contraceptives because SWJ is a liver enzyme inducer and, through
this, increases the metabolism of oral contraceptives. It is recommended that concomitant use
be avoided.

This woman has been started on usual dose of SJW for depression (300mg, TDS), but her
symptoms have failed to improve. Under such circumstance, the most appropriate approach is
1614 of 1943
cessation of SJW and switching to a prescription antidepressant such as a selective serotonin
reuptake inhibitor (e.g., fluoxetine) or a serotonin-norepinephrine reuptake inhibitor
(e.g., venlafaxine). SJW should be withdrawn gradually and the prescription antidepressant started
after the washout period of SWJ, which is one week, to avoid the risk of serotonin syndrome.

(Option A) Lithium is a mood stabilizer used for treatment of bipolar disorder. It has no effect on
unipolar depression. Moreover, addition of lithium to SWJ is associated with increased risk of
serotonin syndrome.

(Option B) Some research has shown that omega 3 has a slight effect on major depression
compared to placebo. While this patient’s depressive symptoms have not improved with SJW, it is
very unlikely that addition of omega 3 adds any benefits.

(Option D) The recommended dose of SJW used for treatment of depression and mood symptoms
by manufacturers is 300mg, 8-hourly. Increasing the dose to 600 mg, 8-hourly neither is safe nor
likely to help this patient’s depression.

(Option E) Addition of the selective serotonin reuptake inhibitor (SSRI) paroxetine (or other
antidepressants) is associated with increased risk of serotonin syndrome and should be avoided.

St. John's wort flower

References

• RACGP - AFP - Your questions about complementary medicines answered: St John’s wort
Last updated:
Time spent: QID:1193
2023-2-12

1615 of 1943
Ali, 45 years, suffers an ischemic stroke and is admitted to the hospital. He is started on life-long
aspirin 80 mg/day for prevention of stroke. He has no past history of a bleeding event such as
gastrointestinal bleeding and is currently on no other medications. During his hospital stay he is
noted to develop sign and symptoms consistent with diagnosis of depression. He is planned to be
started on cognitive behavior therapy (CBT) as well as an antidepressant medication. Of the
following options, which one is most suitable to consider for Ali?

A. Citalopram.
B. Fluoxetine.
C. Venlafaxine.
D. Sertraline.
E. Amitriptyline.

Incorrect. Correct answer is B


45% answered correctly

Explanation:

Correct Answer Is B

Post-stroke depression (PSD) is the most common neuropsychiatric consequence of stroke up to


6-24 months after the stroke onset. Elderly patients are at risk in particular. When depressive
symptoms develop in the first few days after the stroke, remission is more likely, whereas later
onset (i.e. after 3 months) is more likely to be associated with a prolonged course.

PSD is less likely to present with dysphoria and more frequently is associated with signs and
symptoms of vegetative state.

Antidepressants are not generally indicated in mild PSD because the adverse effects outweigh the
benefits, but are used often in conjunction with cognitive behavior therapy (CBT) for treatment of
patients with moderate or severe PSD.

Selective serotonin reuptake inhibitors (SSRIs) are the first-line treatment options for
pharmacological treatment of PSD. Of SSRIs, the drug with the most randomized controlled trials
(RCTs) supporting its use in PSD is fluoxetine both in terms of tolerability and effectiveness is
fluoxetine that is often safe at a dose of 20mg/day.

One concern with SSRIs, however, is the inhibitory effect of this drug class in the inhibition of
serotonin reuptake into the platelets that may result in bleeding tendency of bleeding and increased
incidence of gastrointestinal (most frequent), genitourinary, and intracranial hemorrhage.

This effect can be more pronounced when the patient is concomitantly on ulcerogenic medications
such as non-steroidal anti-inflammatory drugs (NSAIDs), aspirin and other antiplatelet medications,
corticosteroids, etc. However, a recent study suggest that SSRIs are associated with only a slight
increase in absolute risk of gastrointestinal bleeding with a number need to harm of 3177 in low-
risk and 881 in high-risk patients (the relative risk have been shown to be high, while the absolute
risk is small). Considering this fact and in general, such risk does not preclude SSRIs from being
used as first-line options for treatment of PSD.

1616 of 1943
(Option A) The SSRI citalopram is highly selective but less potent. It is the second-choice option for
treatment of PSD after fluoxetine.

(Option C) Venlafaxine is a serotine-norepinephrine reuptake inhibitor (SNRI). This drug family


have shown efficacy in treatment of depression as well increased bleeding tendency in few studies.
Whether this class of antidepressants be considered a treatment option for PSD remains
controversial.

(Option D) This SSRI sertraline is the second most studies medication for treatment of PSD. The
starting dose is 50mg/day with gradual increase up to 200mg/day if indicated and tolerated. It can
also be considered a treatment option but not superior to fluoxetine.

(Option E) Amitriptyline is a tricyclic antidepressant (TCA). SSRIs and tricyclic antidepressants are
equally effective in treatment of PSD; however, grade A recommendations are in favor of SSRIs as
the first-line treatment option due to better tolerability and their significantly lower adverse effects
potentials. TCAs are associate with anticholinergic effects such as hypotension, dry mouth,
constipation, urinary retention and confusion.

NOTES –

If SSRIs are considered a treatment option for PSD in patients with high risk of
gastrointestinal bleeding, proton pump inhibitors should be prescribed for prevention of
bleeding.

No evidence supports prophylactic use of antidepressants for prevention of PSD.

The effective duration of antidepressant therapy for PSD is 4 to 6 months with gradual
tapering if appropriate.

References

• https://www.ncbi.nlm.nih.gov/pmc/articles/PMC2938303/

• https://www.ncbi.nlm.nih.gov/pmc/articles/PMC2515899/

• https://www.medscape.com/viewarticle/750528
Last updated:
Time spent: QID:1227
2023-2-12

1617 of 1943
Exposure and response prevention is the treatment option for which one of the following
conditions?

A. Bipolar disorder.
B. Depression.
C. Agoraphobia.
D. Obsessive – compulsive disorder (OCD).
E. Schizophrenia.

Incorrect. Correct answer is D


45% answered correctly

Explanation:

Correct Answer Is D

Exposure and response (or ritual) prevention (ERP) is the important and specific core element in
behavior therapy for obsessive-compulsive disorder (OCD).

OCD is characterized by distressing, intrusive obsessive thoughts and/or repetitive compulsive


physical or mental acts often in form of rituals.

The mainstays of treatment of OCD include the use of selective serotonin reuptake inhibitors
(SSRIs) or the TCA clomipramine, ERP, some forms of cognitive behavior therapy), education and
family interventions, and, in extremely refractory cases, neurosurgery.

In ERP, the situations the patients perceive as threatening are ranked and then the patient is
systematically exposed to symptoms triggers of gradually increasing intensity, and is taught to
suppress his or her usual ritualized response. This is generally challenging and often quite
distressing for the patient, but when effectively done, it promotes unlearning of the strong link that
has existed between having an urge and giving into the urge.

References

• Medscape - Obsessive-Compulsive Disorder Treatment & Management


Last updated:
Time spent: QID:1258
2023-2-12

1618 of 1943
Which one of the following criteria is essential to make a diagnosis of schizophrenia?

A. Flat affect.
B. Lack of insight.
C. Suicidal ideation.
D. Visual hallucinations.
E. Auditory hallucinations.

Incorrect. Correct answer is E


45% answered correctly

Explanation:

Correct Answer Is E

According to the Diagnostic and Statistical Manual of Mental Disorders, Fifth Edition, (DSM-5), to
meet the criteria for diagnosis of schizophrenia, the patient must have experienced at least 2 of the
following symptoms:

Delusions
Hallucinations – almost always auditory
Disorganized speech
Disorganized or catatonic behavior
Negative symptoms

At least 1 of the symptoms must be the presence of delusions, hallucinations, or disorganized


speech.

Continuous signs of the disturbance must persist for at least 6 months, during which the patient
must experience at least 1 month of active symptoms (or less if successfully treated), with social
or occupational deterioration problems occurring over a significant amount of time. These
problems must not be attributable to another condition.

Of the given options, only auditory hallucinations are a criterion for a diagnosis of schizophrenia.

(Option A) Flat affect is one of the negative symptoms that can be encountered in patients with
schizophrenia but its absence dose not exclude such diagnosis.

(Option B) Lack of insight is seen in schizophrenia as well as many other psychiatric conditions. It
is not a diagnostic criteria for schizophrenia.

(Option C) Suicidal ideation is a feature common in many psychiatric illnesses such as psychotic
disorders, mood disorders, etc. It is not characteristic of schizophrenia and a diagnostic criterion.

(Option D) Although hallucinations are one of the main criteria to establish a diagnosis of
schizophrenia, such hallucinations are almost always auditory. With visual hallucinations
alternative diagnoses should be considered first.

References

1619 of 1943
• https://emedicine.medscape.com/article/288259-overview
Last updated:
Time spent: QID:1259
2023-2-12

1620 of 1943
In which one of the following periods, a pregnant woman is most likely to require treatment for
psychiatric problems?

A. First trimester of pregnancy.


B. Second trimester of pregnancy.
C. Third trimester of pregnancy.
D. Puerperium.
E. After 6 weeks postpartum.

Incorrect. Correct answer is D


45% answered correctly

Explanation:

Correct Answer Is D

There are a wide range of overwhelming negative emotions such as guilt, anxiety, confusion,
frustration, helplessness, sadness and positive emotions such as anticipation, fulfillment,
happiness and excitement that can be experienced during the pregnancy but more importantly and
most commonly experienced in the postpartum period. This makes the postpartum period a highly
vulnerable time for development of psychiatric disorders such as maternity blues, puerperal
psychosis, and postnatal depression.

Puerperium is the first 6 weeks after the delivery. The incidence of the above psychiatric conditions
as well as relapse of pre-existing conditions such as schizophrenia or bipolar disorder
necessitating active psychiatric treatment is highest during this critical period.

References

• https://www.ranzcp.org/Files/Resources/Publications/CPG/Clinician/Mood-Disorders-CPG.aspx

• https://www.ncbi.nlm.nih.gov/pmc/articles/PMC4539865/
Last updated:
Time spent: QID:1276
2023-2-12

1621 of 1943
You are assessing a 15-year-old girl for low weight. She has a body mass index (BMI) of 15. She
hates food because she think it makes her fat. She avoids eating as long as she can but when she
is urged to eat due to hunger, she induces vomiting or uses laxative to get rid of it. During the
interview you ask if she take part in any activity of her interest. She replies: ‘Why should I? What
does the entire world means? Nothing’. Which one of the following is more likely to be elicited from
the rest of the history?

A. Ideas of guilt.
B. Excessive socializing with friends.
C. Sleep disturbances.
D. Feelings of inferiority.
E. Fatigue.

Incorrect. Correct answer is D


45% answered correctly

Explanation:

Correct Answer Is D

This girl has characteristic features of anorexia nervosa (AN). AN is characterized by a markedly
low body weight, intense fear of weight gain, and body image distortion.

Although not central to the diagnostic criteria of AN, emerging evidence suggests additional
deficits in key aspects of social functioning. AN (and other eating disorders) patients appear to be
socially withdrawn, and they report having smaller social networks, less social interactions, and a
reduced number of close friends. There is also evidence for premorbid social problems such as
increased levels of loneliness, feelings of inferiority, and shyness, and comorbidity with anxiety
disorders, such as social phobia.

Of these, feelings of inferiority is most commonly cited by affected women during clinical
interviews. Interestingly, feelings of inferiority is one of the most important factors leading to the
development of eating disorders.

Ideas of guilt (option A) and sleep disturbances (option C) might as well present in the history but
not as commonly as feelings of inferiority.

(Option B) Patients with eating disorders including AN are often socially withdrawn, feel lonely and
may have no or few friends. Excessive socializing with friends is very unlikely to be present in the
history.

(Option E) Fatigue is common in AN patients due to a number of physical and psychological


factors. Low calorie intake, malnutrition and electrolyte disturbances can all lead to fatigue. This
girl may or may not be present in the history depending on the duration of the disease and severity
of malnutrition, electrolyte disturbances, or other complications of the disorder, but feelings of
inferiority are present in the past or current history.

References

1622 of 1943
• Science Direct - Theory of Mind and the Brain in Anorexia Nervosa: Relation to Treatment
Outcome

• Emotional Meanings Assigned to Eating Disorders: Narratives of Women with Anorexia and
Bulimia Nervosa
Last updated:
Time spent: QID:1302
2023-2-12

1623 of 1943
A 17-year-old boy presents to your office for assessment with complaint of excess distress and
anxiety because his college exams are starting. He admits to insomnia, looks anxious, distressed,
and restless. On mental examination, you realize that he adamantly believes the meat in his food is
poisonous and ‘they’ are trying to kill him. Which one of the following does he have?

A. Hallucinations.
B. Delusions.
C. Hypochondriasis.
D. Malingering.
E. Illusions.

Incorrect. Correct answer is B


45% answered correctly

Explanation:

Correct Answer Is B

The adamant belief that the meat in his food is poisonous is an example of delusional thinking.

Delusions are fixed false beliefs that are held despite contradictory evidence. Delusions are
considered "bizarre" if they are clearly implausible and peers within the same culture cannot
understand them. An example of a bizarre delusion is when an individual believes that his or her
brain is replaces with someone else’s. An example of a nonbizarre delusion is the belief that one is
under police surveillance, or like in this case the meat is poisonous despite lack of evidence.

(Option A) A hallucination is a sensation or sensory perception that a person experiences in the


absence of a relevant external stimulus. That is a person experiences something that does not
really exist except in their mind. A hallucination can occur in any sensory modality — visual,
auditory, olfactory, gustatory, or tactile.

(Option C) Hypochondriasis (recently changed to the term illness anxiety disorder [IAD]) is a strong
belief that one has a serious or life-threatening illness despite having no or only mild symptoms.

(Option D) Malingering is the purposeful production of falsely or grossly exaggerated physical or


psychological complaints with the goal of receiving a reward.

(Option E) Illusions are wrong or misinterpreted perception of sensations. Illusions can occur in
any sensory modality. The difference between a hallucination and an illusion is that in the former
there is an external stimulus which is misinterpreted or perceived differently, while for
hallucinations there is no external stimulus.

References

• Lexicon on psychiatric and mental health terms (2nd edition)


Last updated:
Time spent: QID:1365
2023-2-12

1624 of 1943
A 28-year-old man with obsessive-compulsive disorder (OCD) has come for consultation. He has
already been started on clomipramine, and asks if there are other non-pharmacological treatment
options for him. In response to his question, which one of the following options would you consider
for him as a treatment for OCD?

A. Insight-oriented psychotherapy.
B. Explain to him that his compulsive behavior is a means of prevention from obsession -
related anxiety.
C. Exposure and response prevention therapy.
D. Psychoanalytic therapy.
E. Dialectic therapy.

Incorrect. Correct answer is C


45% answered correctly

Explanation:

Correct Answer Is C

OCD is a chronic illness that usually can be treated in an outpatient setting. The mainstays of
treatment of OCD include pharmacotherapy with clomipramine or selective serotonin re-uptake
inhibitors (SSRIs) or behavior therapy.

Exposure and response (or ritual) prevention is the important and specific behavior therapy for
OCD. In exposure and response prevention (ERP), the situations the patients perceive as
threatening are ranked and then the patient is systematically exposed to symptom triggers of
gradually increasing intensity and is taught to suppress his or her usual ritualized response. This is
generally challenging and often quite distressing for the patient, but when effectively done, it
promotes unlearning of the strong link that has existed between having an urge and giving into the
urge.

(Option A) Insight-orientation psychotherapy, also known as insight therapy, is a form of client-


centered therapy that guides patients towards developing an improved understanding of self.
During each session, the person receiving the therapy will recall situations from his/her life. The
therapist highlights observed patterns of behavior or feeling and encourages the patient to
examine them more closely. This method works through examination and analysis of issues that
have negatively affected the patient’s life, and helping the client understand and change destructive
patterns. Insight-orientation therapy is not an effective treatment for OCD.

(Option B) Patients with OCD are usually aware of and have adequate insight about their
bothersome urge for compulsive behavior as a means to relief the anxiety caused the obsessions.
Their problem is with the irresistible urge for such relief and not unawareness of their condition;
therefore, explaining why their compulsive behavior occurs will not help them as a treatment.

(Option D) Psychoanalytic therapy is used to bring unconscious or deeply buried thoughts and
feelings to the conscious mind so that repressed experiences and emotions, often from childhood,
can be brought to the surface and examined. Working together, the therapist and patient look at
how these repressed early memories have affected the patient’s thinking, behavior, and
relationships in adulthood. Psychoanalytic therapy is used for people suffering from depression,
1625 of 1943
emotional struggles, neurotic behavior, self-destructive behavior patterns, personality disorders or
relationship issues. Psychoanalytic therapy has not proved significantly effective in treatment of
OCD.

(Option E) Dialectic therapy, a very specialized form of intensive cognitive therapy, is currently
considered the treatment of choice for women with borderline personality disorder. This treatment
is not approved for treating male patients with borderline personality disorder. Dialectic therapy is
not effective for treatment of OCD.

References

• RACGP - AFP - Obsessive-compulsive disorder

• Medscape - Obsessive-Compulsive Disorder Treatment & Management


Last updated:
Time spent: QID:1417
2023-2-12

1626 of 1943
It is well-known that countertransference is frequently observed in doctors treating patients with
self-harm or anorexia nervosa. Which one of the following can be the cause?

A. These conditions are rarely seen.


B. These conditions are frequently seen in rich people.
C. These conditions are self-inflicted.
D. The doctor projects his/her inner feelings towards the patient.
E. These conditions are not socially accepted.

Incorrect. Correct answer is E


45% answered correctly

Explanation:

Correct Answer Is E

Countertransference is defined as the unconscious development of feelings toward the patient


from the treating doctor. Negative countertransference is when such feelings are negative and
prevent appropriate empathy and communication.

Studies suggest that countertransference is especially evident in the context of substance use,
self-harm and anorexia. For substance use and self-inflicted injuries there is often an underlying
undiagnosed borderline personality disorder as well as a comorbid condition. The pattern of
behavior in patients with this co-morbidity further deteriorates the condition and makes the patient-
doctor relationship more difficult and delays access to appropriate treatment.

Recent research showed that the most important cause for the negative countertransference in
treating personalities who causing self-harm is stigmatism and not socially being accepted.

Stigma associated with patients with self-inflicted injury and harm (including anorexia nervosa) is
high and in healthcare setting reflects a lack of skills and knowledge in relation to the specific
needs of this population, as well as the negative countertransference commonly experienced in
working with these patients. Indeed, it has been suggested that individuals with BPD constitute the
‘most psychologically challenging patients a primary care physician ever encounters’, and this is
especially true when substance use is also prominent.

Working successfully with patients with BPD requires many layers of support in order to prevent
any negative countertransference adversely impacting treatment planning or the response to crisis
presentations.

References

• RACGP - Managing borderline personality disorder and substance use

• Countertransference in the Treatment of Borderline Personality Disorder


Last updated:
Time spent: QID:1560
2023-2-12

1627 of 1943
Jane, 75 years old, had a heart attack 18 months ago. She feels a little anxious, but isvery
independent and mobile. However, she ia afraid of leaving the house, because sometimes as she
thinks of leaving, she feels dizzy, unsteady, unable to breathe, heavy in the chest, clammy, and also
develops palpitation. These symptoms also occur unexpectedly at other times. These often last
approximately 10 minutes before she feels better, but she is really scared as to when they might
occur next. Which one of the following is the most likely diagnosis?

A. Anxiety.
B. Phobia.
C. Axiety with panic attacks.
D. Anxiety and ongoing angina pectoris.
E. Depression and phobia of going out.

Incorrect. Correct answer is C


45% answered correctly

Explanation:

Correct Answer Is C

Jane has the classical symptoms of panic attacks with some pre-existing anxiety.

Panic attacks are defined as a period of intense fear with at least 4 of the following 13
symptoms:

1. Palpitations, pounding heart, or accelerated heart rate


2. Sweating
3. Trembling or shaking
4. Sense of shortness of breath or smothering
5. Feeling of choking
6. Chest pain or discomfort
7. Nausea or abdominal distress
8. Feeling dizzy, unsteady, lightheaded, or faint
9. Derealization or depersonalization (feeling detached from oneself)
10. Fear of losing control or going crazy
11. Fear of dying
12. Numbness or tingling sensations
13. Chills or hot flushes

Other describes symptoms include:

Headache
Cold hands
Diarrhea
Insomnia
Fatigue
Intrusive thoughts
Ruminations

1628 of 1943
Symptoms must peak within 10 min and usually subside within few minutes, leaving little to
observe, except the person's fear of another terrifying panic attack.

Many people experience panic attacks without further episodes or complication. In some, however,
panic attacks progress to panic disorder characterized by repeated panic attacks, combined with
major changes in behavior or persistent anxiety of having further attacks.

According to the Diagnostic and Statistical Manual of Mental Disorders, Fifth edition, the following
criteria must be met in order to make a diagnosis of panic disorder:

Panic attacks must be associated with longer than 1 month of subsequent persistent worry
about:

1. Having another attack or complications of attack


2. Significant maladaptive behavioral changes related to the attacks
3. The attacks cannot directly or physiologically result from substance use (intoxication or
withdrawal), medical condition or another psychiatric disorder.

Agoraphobia was traditionally thought to involve a fear of public and open spaces. However, it is
now considered a complication of panic attacks. With agoraphobia, the person fears that they may
have an attack in a situation where escape would be difficult or embarrassing.

References

• Medscape - Panic Disorder


Last updated:
Time spent: QID:127
2023-2-12

1629 of 1943
A 16-year-old boy presents to your GP clinic with complaint of recent decrease in school
performance. He broken up with his girlfriend 3 weeks ago. He has been suffering from lack of
energy and sleep disturbances, and feels like crying almost all the time. Which one of the following
would be the most appropriate next step in management?

A. Empathy about the breakup.


B. Cognitive behavior therapy (CBT).
C. Start him on fluoxetine.
D. Start him on fluvoxamine.
E. Refer him to a psychiatrist

Correct
45% answered correctly

Explanation:

Correct Answer Is A

This young boy has presented with symptoms suggestive of depression after an identifiable
stressor (the breakup). He has the criteria for ‘adjustment disorder’ as the most likely diagnosis.

The DSM-5 diagnostic criteria for adjustment disorder are as follows:

Emotional or behavioral symptoms develop in response to an identifiable stressor or


stressors within 3 months of the onset of the stressor(s) plus either or both of (1) marked
distress that is out of proportion to the severity or intensity of the stressor, even when
external context and cultural factors that might influence symptom severity and presentation
are taken into account and/or (2) significant impairment in social, occupational, or other
areas of functioning.

The stress-related disturbance does not meet criteria for another mental disorder and is not
merely an exacerbation of a preexisting mental disorder.

The symptoms do not represent normal bereavement

After the termination of the stressor (or its consequences), the symptoms persist for no
longer than an additional 6 months

Royal College of Australian General Practitioners (RACGP) has proposed a model for interacting
with patients with psychological distress and disturbances regardless of the underlying etiology.
This approach model consists of three steps:

STEP 1- Establishing rapport using active listening and empathy, non-directional supportive
psychotherapy, and psycho-education. Then if required:

STEP 2- Cognitive behavioral therapy (CBT), interpersonal psychotherapy and interpersonal


counselling, structured problem solving, behavioral therapy, and family therapy. Then if
required:

STEP 3- Intensive therapy (e.g., use of medications)


1630 of 1943
Differing levels of intervention
BATHE method: Background, Affect, Trouble, Handling, Empathy
DIG technique: Dream the miracle, Initiate the first small step, Get going and implement
the first move
afp, Vol.41, No.7 July 2012

For this patient establishing rapport in this session is a good start. Being empathetic about his
breakup and what he is going through is the most appropriate next step at this stage. This builds
therapeutic connection and builds up trust, and paves the path for further intervention.

CBT (option B) can be an option, among others, to consider for this patient in the STEP 2
afterwards.

Patients with depression in the setting of adjustment disorder usually respond to brief non-
pharmacological interventions as mentioned above and do not require anti-depressants such as
fluoxetine (option C) and fluvoxamine (option D).

Referral to a psychiatrist (option E) is not often required for adjustment disorder. Even so, this
should be considered after establishing an appropriate therapeutic relationship with the patient.

References

• RACGP – AFP - Psychological triage in general practice


Last updated:
Time spent: QID:1578
2023-2-12

1631 of 1943
Monaro is a 19-year-old Aboriginal man who has presented to your practice for consultation. He
has been in a correctional facility for the past 3 years for multiple felonies including street fights,
battery, minor drug dealing and possession of illicit drugs, and has been released just few weeks
ago. H has always been thinking about his past and childhood neglect and feels sad, angry, and
frustrated about it. He recently has had anger outburst in confronting other people and is very
worried that he might get himself into trouble and go back to jail again. He seeks advice for
treatment. Which one of the following is the most appropriate treatment option for him?

A. Cognitive behavior therapy.


B. Dialectical behavior therapy.
C. Antidepressants.
D. Psychoanalytic therapy.
E. Anger management program.

Incorrect. Correct answer is C


45% answered correctly

Explanation:

Correct Answer Is C

Cognitive behavior therapy (CBT) (option A), dialectical therapy (option B), psychoanalytic therapy
(option D), and anger management program (option E) are all therapeutic options with potential
benefit for Monaro. For buried frustration from childhood, psychoanalytic therapy may be
beneficial. He has spent 3 years in a correctional facility and is very likely to be suffering from post-
traumatic stress disorder (PTSD). His anger may respond properly to CBT and anger management
program. These methods, however, take time to come to effect.

In the meanwhile, Monaro has the acute problem of anger outbursts that are very likely to put him
or others at risk. It is recommended that for patients in immediate risk of harm to self or others, or
escalation of the current situation, medical therapy is started as a means of rapid stabilization
while other non-pharmacological measures are planned and carried out. This is often true
regardless of the underlying etiology that can be PTSD, depression, substance abuse, etc.

For anger outbursts, antidepressants, especially selective serotonin reuptake inhibitors (SSRIs)
have the highest supportive evidence for efficacy, and should be considered acutely for Monaro
while other forms of therapy are arranged.

TOPIC REVIEW

A summary on CBT, DBT, psychoanalytic therapy, and anger management:

Cognitive behavioral therapy (CBT)

CBT is a form of psychological treatment that has proven effective for a range of problems
including depression, anxiety disorders, alcohol and drug use problems, marital problems, eating
disorders, and severe mental illness. Numerous research studies suggest that CBT leads to
1632 of 1943
significant improvement in functioning and quality of life. In many studies, CBT has been
demonstrated to be as effective as, or more effective than, other forms of psychological therapy or
psychiatric medications.

Advances in CBT have been made on the basis of both research and clinical practice. Indeed, CBT
is an approach for which there is ample scientific evidence showing that it actually produce
change. In this manner, CBT differs from many other forms of psychological treatments.

CBT usually involves efforts to change thinking patterns. These strategies might include:

Learning to recognize one's distortions in thinking that are creating problems, and then to re-
evaluate them in light of reality.
Gaining a better understanding of the behavior and motivation of others.
Using problem-solving skills to cope with difficult situations.
Learning to develop a greater sense of confidence is one's own abilities.

CBT treatment also usually involves efforts to change behavioral patterns.

These strategies might include:

Facing one's fears instead of avoiding them.


Using role playing to prepare for potentially problematic interactions with others.
Learning to calm one's mind and relax one's body.

Dialectical behavior therapy (DBT)

DBT is a type of talking therapy originally developed by an American psychologist named Marsha
Linehan. DBT is based on cognitive behavior therapy (CBT), but has been adapted to meet the
particular needs of people who experience emotions very intensely. It is mainly used to treat
problems associated with borderline personality disorder (BPD), such as:

Repeated self-harming
Suicide attempt
Using alcohol or drugs to control emotions
Eating problems, such as binge eating and purging
Unstable relationships

The goal of DBT is to help patients learn to manage their difficult emotions by letting themselves
experience, recognize, and accept them. Then as they learn to accept and regulate their emotions,
they also become more able to change their harmful behavior. To help patient achieve this, DBT
therapists use a balance of acceptance and change techniques.

Psychoanalytic therapy

Psychoanalytic therapy is a form of in-depth talk therapy that aims to bring unconscious or deeply
buried thoughts and feelings to the conscious mind so that repressed experiences and emotions,
often from childhood, can be brought to the surface and examined.

1633 of 1943
Working together, the therapist and client look at how these repressed early memories have
affected the client’s thinking, behavior, and relationships in adulthood. People with depression,
emotional struggles, emotional trauma, neurotic behavior patterns, self-destructive behavior
patterns, personality disorders, or ongoing relationship issues, may benefit from psychoanalytic
therapy.

Anger management

Anger management is a psycho-therapeutic program for anger prevention and control. It has been
described as deploying anger successfully. Anger is frequently a result of frustration, or of feeling
blocked or thwarted from something the subject feels is important. Anger can also be a defensive
response to underlying fear or feelings of vulnerability or powerlessness.

Anger management programs consider anger to be a motivation caused by an identifiable reason


which can be logically analyzed, and if suitable worked toward. Drug addiction, alcoholism, mental
disability, biochemical changes, and PTSD can all lead to a person committing an aggressive act
against another person.

References

• RACGP – AFP - PTSD – an update for general practitioners

• UpToDate - Intermittent explosive disorder in adults: Treatment and prognosis


Last updated:
Time spent: QID:1588
2023-2-12

1634 of 1943
Janet and Li present to your practice for consultation. They met 6 months ago and now they have
decided to move in and live together. Li has a dog and loves it very much but Janet is severely
afraid of dogs. She says that I try to overcome my fear but even speaking of dogs makes me panic
to death. She says that when she was a child, she was bitten by a dog and has been afraid of them
her whole life. They ask for advice as to how to overcome her fear. Which one of the following is
the most appropriate advice she should follow as the first step in overcoming her fear of dogs?

A. She should keep a picture of a dog in her diary.


B. She should keep a picture of a dog in her room and at sight.
C. She should learn and perform relaxation techniques.
D. She should try to imagine patting a dog.
E. She should be exposed to dogs to overcome her fear.

Incorrect. Correct answer is C


45% answered correctly

Explanation:

Correct Answer Is C

Current guidelines recommend systematic desensitization for patients with specific phobias.
Systematic desensitization is a type of behavior therapy based on the principle of classical
conditioning.

This therapy aims to remove the fear response of a phobia, and substitute a relaxation response to
the conditional stimulus gradually using counter conditioning. There are three phases to the
treatment:

Phase 1 - The patient is taught a deep muscle relaxation technique and breathing exercises, e.g.,
control over breathing, muscle de-tensioning or meditation. This step is very important because of
reciprocal inhibition, where once response is inhibited because it is incompatible with another. In
the case of phobias, fears involves tension and tension is incompatible with relaxation.

Phase 2 - The patient creates a fear hierarchy starting at stimuli that create the least anxiety (fear)
and building up in stages to the most fear provoking images. The list is crucial as it provides a
structure for the therapy. The following table is an example rated by a patient:

Behavior Fear rating


Think about a dog 10
Look at the photo of a dog 20
Look at a real dog while 40
detained
Look at a real dog while 50
unleashed
Look at the dog while it is 60
approaching
Let the dog get close to you 70
Pat the dog 80
1635 of 1943
Hold the dog 90
Play with the dog 100

Phase 3 - The patient works their way up the fear hierarchy, starting at the least unpleasant
stimulus (thinking about a dog) and practicing their relaxation technique as they go. When they feel
comfortable with this (they are no longer afraid) they move on to the next stage in the hierarchy. If
the client becomes upset they can return to an earlier stage and regain their relaxed state.

The client repeatedly imagines (or is confronted by) this situation until it fails to evoke any anxiety
at all, indicating that the therapy has been successful. This process is repeated while working
through all of the situations in the anxiety hierarchy until the most anxiety-provoking stimulus is
reached.

Based on this therapy, the first step in approaching Janet’s phobia would be teaching her relaxation
techniques and methods to overcome the anxiety brought up by exposure to the gradually-
intensifying stimuli.

Other options are stimuli she should face after she learns how to overcome anxiety and relax in a
hierarchical fashion.

References

• Simply Psychology – Systematic desensitization


Last updated:
Time spent: QID:1590
2023-2-12

1636 of 1943
A 24-year-old woman presents to your office extremely worried about her weight and body size. She
adamantly believes that she is obese and must do something for her obesity. On examination, she
has a BMI of 25 and there is no abnormal finding on the rest of the exam. Laboratory values are all
within normal range. Which one of the following is the most likely diagnosis?

A. Bulimia nervosa.
B. Body dysmorphic disorder.
C. Anorexia nervosa.
D. Hypochondriasis.
E. Generalized anxiety disorder.

Incorrect. Correct answer is B


45% answered correctly

Explanation:

Correct Answer Is B

This woman is presenting significant yet unrealistic concerns about and is preoccupied with her
weight and body size while she has a near healthy BMI (healthy BMI: 18.5-24.9 kg/m2). Also, there
is no pointers towards other conditions such as anorexia nervosa or bulimia nervosa. One
important diagnostic criterion for anorexia nervosa is a BMI of less than 17.5 kg/m2. A BMI of 25
kg/m2 in this woman excludes anorexia nervosa (option C) as a diagnosis, and lack of binge eating
and purging in history makes bulimia nervosa (option A) very unlikely.

Given these, body dysmorphic disorder (BDD) is the most likely diagnosis in this scenario.

Specific DSM-5 criteria for BDD are as follows:

The individual is preoccupied with 1 or more perceived defects or flaws in physical


appearance that are not observable by or appear slight to others
At some point during the course of the disorder, the individual has performed repetitive
behaviors (e.g., mirror checking, excessive grooming, skin picking, or reassurance seeking) or
mental acts (e.g., comparing his or her appearance with that of others) in response to the
appearance concerns
The preoccupation causes clinically significant distress or impairment in social, occupational,
or other important areas of functioning
The appearance preoccupation cannot be better explained by concerns with body fat or
weight in an individual whose symptoms meet diagnostic criteria for an eating disorder

(option D) Hypochondriasis is associated with excessive and unduly concerns about a serious
disease. This woman does not have any concerns about a disease rather is obsessively
preoccupied with her physical appearance.

(Option E) Generalized anxiety disorder (GAD) is a disorder characterized by excessive or


unrealistic anxiety about two or more aspects of life such as work, social relationships, financial
issues, etc., often accompanied by physical symptoms of anxiety such as palpitation, shortness of

1637 of 1943
breath, chest pain, or dizziness. Although preoccupation with the weight can cause anxiety in this
woman, given the absence of worry in other aspects, GAD cannot be a diagnosis.

References

• Medscape – Body Dysmorphic Disorder


Last updated:
Time spent: QID:1628
2023-2-12

1638 of 1943
Sid, a 47-year-old local farmer, presents to your practice for a visit due to problem sleeping due his
worries about his debts to the bank after a recent drought this year. During the consultation, you
get to know that his mood has been low in the past 2 months. He does not go out anymore and
stays at home most of the time because he lacks energy. He has poor appetite and lost almost 10
kg. Based on your diagnosis of severe depression, you advise him to be started on
antidepressants. He refuses and says: “Oh doctor. I am not depressed. This is what everybody
says. But I am just exhausted. You were too if in my place.” Which one of the following is the
defense mechanism he is using?

A. Displacement.
B. Reaction formation.
C. Projection.
D. Denial.
E. Rationalization.

Incorrect. Correct answer is D


45% answered correctly

Explanation:

Correct Answer Is D

This case scenario is a clear example of denial defense mechanism. Sid has a clear diagnosis of
severe depression, but he denies it and believes it is just the exhaustion and that he has just been
through too much.

Denial is one of the most common defense mechanisms. When a situation or fact becomes too
much to handle, the person copes by refusing to experience it. By denying reality, one essentially
protects himself from the need to face and deal with the unpleasant consequences and pain that
accompany acceptance.

Examples of denial are:

A smoker denies that his habit has negative health consequences so he can continue smoking.
A parent denies that her son has dropped out of college even though the school administrator
has left her three voicemails telling her so.
A business owner denies that her business is failing despite declining profits.
A problem drinker denies his alcohol dependence and believes he is just a social drinker.
Someone with failing marriage believes that every couple eventually loses the romance.

(Option A) Displacement occurs when a person represses affection, fear, or impulses that they feel
towards another person. Accepting that it is irrational or socially unacceptable to demonstrate
such feelings prevents them from being converted into actions. However, the feelings are instead
displaced towards a person or animal whom it is acceptable to express such feelings for. A person
who dislikes their teacher after being given low grades may feel that they would be punished if they
express their hostility towards them. Therefore, they may unconsciously displace their negative
emotions onto their best friend.

Examples of displacement defense mechanism are:

1639 of 1943
A woman who is frustrated because her boss made her work late again comes home and yells
at her son for asking what’s for dinner.
A man who is mad that he can’t seem to get ahead at work constantly criticizes his wife for not
making more money.
An employer who is frustrated with his business’ losses blames his employees instead of
rethinking his business strategy.

(Option B) With reaction formation defense mechanisms, one goes beyond denial and behaves in
the opposite way of which they think or feel. Typically, reaction formation is marked by a blatant
display. For example, a man with strong homosexual desires, vigorously and overtly reasons
against homosexuality.Other examples of reaction formation are:

A woman cares for her aging mother by cooking her meals, cleaning her home and taking her to
appointments while she secretly despises her.
A person with a drinking problem preaches about the virtues of sobriety and not drinking alcohol
instead of taking steps toward moderate drinking.

(Option C) When we experience feelings or desires that cause anxiety, or that we are unable to act
on owing to the negative impact that they would have on us or those around us, we may defend the
ego from resulting anxieties by projecting those ideas onto another person. A person who is afraid
of crossing a bridge with a friend might accuse them of having a fear of heights, for example, and
in doing so, avoids accepting their own weaknesses.

Examples of projection defense mechanism

You really dislike your manager at work who treats you just like they treat everyone else. Instead
of admitting your dislike, you tell everyone the manager has a grudge against you and is trying to
sabotage your career.
A woman is in a bad mood all day and comes home to her husband. After greeting her warmly
and asking about her day, she instantly accuses him of being in a bad mood and ruining her
evening.
A father has body image issues but refuses to make lifestyle changes that would help him lose
weight. Instead, he blames his daughter for her weight, projecting his lack of self-confidence
onto her.

(Option E) Rationalization occurs when a person attempts to explain or create excuses for an event
or action in rational terms. In doing so, they are able to avoid accepting the true cause or reason
resulting in the present situation.

Examples of rationalization defense mechanism are:

A man forgets to take out the garbage even though his wife has reminded him four times.
Instead of apologizing, he rationalizes that she hasn’t loaded the dishwasher so it’s only fair that
he not take out the garbage.
A woman stands up her friend for lunch and tells herself it’s okay because her friend is always
late for their morning yoga sessions.
A student rationalizes cheating on a test by saying the test was too demanding anyway.

References

• Psychologist World – 31 Psychological Defense Mechanisms Explained

1640 of 1943
Last updated:
Time spent: QID:1642
2023-2-12

1641 of 1943
A 50-year-old farmer is in significant financial debts dues to a recent severe drought. He is planning
to commit suicide so that his family can use his life insurance. He feels anxious and is irritable but
does not show symptoms of depressive disorders. You know him well and consulted him for
alcohol use and problem gambling and know, from his wife, that he is reckless around money,
spends a lot in pubs and casinos, and has problem drinking issues. He blames the government and
authorities for what he is going through and believes that if they were more responsible and did
their jobs right, he was not like this. Which one of the following defense mechanisms is he using?

A. Denial.
B. Projection.
C. Displacement.
D. Rationalization.
E. Regression.

Incorrect. Correct answer is B


45% answered correctly

Explanation:

Correct Answer Is B

Base on the facts in the scenario, this man is most likely utilizing projection as a defense
mechanism.

Projection is the misattribution of a person’s undesired thoughts, feelings, or impulses onto another
person who does not have those thoughts, feelings, or impulses. Projection is used especially
when the thoughts are considered unacceptable for the person to express. For example, a man
may be angry at their wife for not listening, when in fact it is the angry man who does not listen.
Projection is often the result of a lack of insight and acknowledgement of one’s own motivations
and feelings.

In this scenario, the drought might really has caused his financial problems, but putting all the
blame on the government and authorities and not accepting his share of irresponsibility helps him,
subconsciously, to misattribute the root cause of this feeling on someone or something else. In
other words, he is projecting the negative feelings of recklessness and irresponsibility elsewhere to
protect his ego. In projection, in fact, some negative feelings or perceptions are completely or
partially mirrored out.

Some other examples of projection are:

You really dislike your manager at work who treats you just like they treat everyone else. Instead
of admitting your dislike, you tell everyone the manager has a grudge against you and is trying to
sabotage your career.

A woman is in a bad mood all day and comes home to her husband. After greeting her warmly
and asking about her day, she instantly accuses him of being in a bad mood and ruining her
evening.

1642 of 1943
A father has body image issues but refuses to make lifestyle changes that would help him lose
weight. Instead, he blames his daughter for her weight, projecting his lack of self-confidence
onto her.

(Option A) Denial is one of the most common defense mechanisms. When a situation or fact
becomes too much to handle, the person copes by refusing to experience it. By denying reality, one
essentially protects himself from the need to face and deal with the unpleasant consequences and
pain that comes with acceptance.

Examples of denial are:

A smoker denies that his habit has negative health consequences so he can continue smoking.

A parent denies that her son has dropped out of college even though the school administrator
has left her three voicemails telling her so.

A business owner denies that her business is failing despite declining profits.

A problem drinker denies his alcohol dependence and believes he is just a social drinker.

Someone with failing marriage believes that every couple eventually loses the romance.

(Option C) Displacement occurs when a person represses affection, fear, or impulses that they feel
towards another person because he/she beleives it is irrational or socially unacceptable to
demonstrate such feelings. However, the feelings are instead displaced towards another person or
animal. A person who dislikes their teacher after being given low grades may feel that they would
be punished if they express their hostility towards them. Therefore, they may unconsciously
displace their negative emotions onto their best friend, making excuses for treating them badly
without justification.

Examples of displacement defense mechanism are:

A woman who is frustrated because her boss made her work late again comes home and yells
at her son for asking what’s for dinner.

A man who is mad that he can’t seem to get ahead at work constantly criticizes his wife for not
making more money.

An employer who is frustrated with his business’ losses blames his employees instead of
rethinking his business strategy.

(Option D) Rationalization occurs when a person attempts to explain or create excuses for an event
or action in rational terms. In doing so, they are able to avoid accepting the true cause or reason
resulting in the present situation.

Examples of rationalization defense mechanism are:

A man forgets to take out the garbage even though his wife has reminded him four times.
Instead of apologizing, he rationalizes that she hasn’t loaded the dishwasher so it’s only fair that
he not take out the garbage.

A woman stands up her friend for lunch and tells herself it’s okay because her friend is always
late for their morning yoga sessions.

1643 of 1943
A student rationalizes cheating on a test by saying the test was too demanding anyway.

A shoplifter blames the high price of sweets to justify their theft of a chocolate bar, when in
reality they simply enjoyed the act of shoplifting.

NOTE – The case scenario may be mistakenly interpreted as rationalization as he might sound
as if he is rationalizing his problems by something else which in this case is the government.
There is, however, a very fine differentiating factor between these two in this scenario; that is, he
is attributing his exact recklessness and irresponsibility to an outer source. The key difference is
the ‘mirroring process’ which is specific to projection defense mechanism.

(Option E) Regression occurs when a person reverts to the types of behavior that they exhibited at
an earlier age. Stress of adult life and the associated anxiety may lead to a person seeking comfort
in things which they associate with more secure, happier times. They might regress by eating
meals that they were given as a child, watching old films or cartoons, acting without thought for the
consequences of their actions.

Examples of regression are:

A 9-year-old boy is in the hospital to get his tonsils removed and begins sucking his thumb like
he did when he was a toddler.

A woman gets in an argument with someone at work and starts sobbing uncontrollably.

Instead of utilizing mature parenting techniques like empathy and boundaries, a father resorts to
temper tantrums to scare his children into obedience.

References

• Psychologist World – 31 Psychological Defense Mechanisms Explained


Last updated:
Time spent: QID:1644
2023-2-12

1644 of 1943
A 75-year-old man is being assessed for insomnia. He has metastatic prostate cancer for which he
is taking morphine for pain control. Because of the sleeplessness, he had been using cannabis until
recently when it was no more effective. He feels agitated and edgy most of the time in daytime.
Which one of the following is most likely to help him with his problem?

A. Supportive psychotherapy.
B. Interpersonal therapy.
C. Sleep hygiene and routine.
D. Motivational therapy.
E. Syntonic phototherapy

Incorrect. Correct answer is C


45% answered correctly

Explanation:

Correct Answer Is C

Insomnia disorder is characterized by inadequate sleep despite adequate sleep opportunity,


accompanied by daytime dysfunction. Both pharmacologic and psychological interventions could
be used for treatment of insomnia disorder; however, selecting the right treatment depends on the
chronicity of symptoms, taking into account medical and psychiatric factors.

Current guidelines recommend cognitive behavioural therapy for insomnia (CBT-i) as the gold
standard and first-line treatment for insomnia. This may be complemented with short-term
pharmacological intervention.

The following diagram illustrates a stepwise approach to treatment of insomnia:

Adapted form Morgan, 2011

The very first step in management of patients with insomnia is gathering a detailed sleep history
and treatment of any contributing factors and comorbidities. The second step is utilizing CBT-I.

CBT-I includes the following components:

Implementation of good sleep hygiene


Sleep restriction* and stimulus control programs**, and relaxation procedures***
Cognitive therapy for insomnia

1645 of 1943
Sleep hygiene and routine is a term that encompasses most of the above and is the next best step
and in fact the most important arm of treatment for this patient. However, if CBT was an option, it
would be a more exact and appropriate one to choose.

Sleep hygiene and routing include the following pieces of advice for patients suffering from
insomnia:

Following a nightly routine:

Budgeting 30 minutes for winding down before sleep (soft music, light stretching, reading
and/or relaxation exercise
Dimming the light
Unplugging from electronics (e.g., TV, mobiles, tablets, etc.)
Relaxation techniques
Not tossing and turning in the bed and walking out of the bedroom if not fell sleep within 15-
30 minutes

Cultivation of healthy daily habits:

Adequate daytime light exposure


Day time regular exercise
Cutting down on alcohol
Cutting down on caffeine in the afternoon and evening hours
Smoking cessation
Avoiding late dining
Avoiding using the bed for anything but sleep and sex

Bedroom optimization:

Comfortable beddings
Setting a cool yet comfortable temperature
Blocking out noise and light in the bedroom
Using calming scents

(Option A) Supportive psychotherapy is a talking-based therapy designed to allow a person with


mental health issues to voice their concerns and receive encouragement and help in finding
practical solutions. Supportive psychotherapy is used to help people dealing with serious addiction,
eating disorders, stress, and other mental health issues. This method works to build a person’s
adaptability and resilience and better coping strategies for the future. Although this treatment
could be beneficial to treatment the underlying mental issues associated with or underlying
insomnia, it is not a recommended stand-alone therapy for it.

(Option B): Interpersonal therapy focuses on the interpersonal relationships of the depressed
person. The idea of interpersonal therapy is that depression can be treated by improving the
communication patterns and how people relate to others.

Techniques of interpersonal therapy include:

Identification of emotion — Helping the person identify what their emotion is and where it is
coming from.

1646 of 1943
Example - Roger is upset and fighting with his wife. Careful analysis in therapy reveals that he
has begun to feel neglected and unimportant since his wife started working outside the home.
Knowing that the relevant emotion is hurt and not anger, Roger can begin to address the
problem.

Expression of Emotion - This involves helping the person express their emotions in a healthy
way.

Example - When Roger feels neglected by his wife, he responds with anger and sarcasm. This in
turn leads his wife to react negatively. By expressing his hurt and his anxiety at no longer being
important in her life in a calm manner, Roger can now make it easier for his wife to react with
nurturance and reassurance.

Dealing with emotional baggage - Often, people bring unresolved issues from past
relationships to their present relationships. By looking at how these past relationships affect
their present mood and behavior, they are in a better position to be objective in their present
relationships.

Example — Growing up, Roger’s mother was not a nurturing woman. She was very involved in
community affairs and often put Roger’s needs on the back burner. When choosing a wife, Roger
subconsciously chose a woman who was very attentive and nurturing. While he agreed that the
family needed the increased income, he did not anticipate how his relationship with his own
mother would affect his reaction to his wife working outside the home.

(Option D) Motivational therapy/interviewing is a therapy technique that empowers patients to


make actionable behavioral changes in their lives through an interview-style discussion with a
licensed mental health professional. This technique focuses on change talk where patients
verbalize their need and desire to make changes in their lives.

Since the benefits of motivational interviewing for mental health are numerous, it has become a
common treatment method for people struggling with various mental health problems and co-
occurring disorders like substance abuse. This therapy technique identifies and accepts the fact
that individuals who need to make changes in their lives approach counselling at different levels of
readiness to change their behavior. In some cases, patients may have thought about making a
behavior change but have not taken the necessary steps to make that change happen. During this
fragile time, people may express a fear of change and deal with internal struggles that need to be
broken down. This is where motivational interviewing steps in. A motivational interview allows
individuals to learn how to effectively change unhealthy behaviors to aid the addiction recovery
process.

(Option E) Syntonic phototherapy (light therapy) uses visible colored lights to enhance visual
attention and decrease the symptoms of stress and trauma. The goal of this therapy is to balance
the autonomic nervous system which controls visual perceptual fields. Colored light delivered
through the eyes helps control biological development and function. For example, red light
activates the sympathetic nervous system (‘fight-or-flight’ responses such as heart rate and blood
pressure) and blue/indigo stimulates the parasympathetic nervous system (‘rest and digest’
responses like salivation, urination, and even arousal).

Syntonic phototherapy consists of patients focusing their eyes at a dot of color at the end of a
50cm tube. Patients view specific safe light frequencies for approximately 20 minutes at a time
over a prescribed number of sessions (usually around 20, done over the course of 4-5 weeks). As
light enters the eyes, it travels from nerves connected to the retina to brain centers that influence

1647 of 1943
chemical, hormonal, and electrical balances. These centers control the balance of all body
functions, and years of study have proven that selected light frequencies delivered through the
retina can create favorable results that benefit the entire body. In addition to red and blue lights and
their systemic healing of the parasympathetic and sympathetic nervous systems, green lights
promote physiological balance. Once this balance is achieved, the effect is long lasting for most
individuals.

Because the practice affects the entire nervous system, the potential benefits of syntonic therapy
are numerous and varied. Syntonics can enhance visual acuity by improving blurred vision,
contrast, night vision issues, and light and allergy sensitivity. This therapy is also used to help
alleviate pain, headaches, eyestrain, and mental and emotional stress levels. Based on lack of
evidence for usefulness of this method for insomnia, it is not currently a recommended treatment
option for sleep problems.

*Sleep restriction – The aim of sleep restriction is to increase sleep drive and reduce the time awake
in bed. Time in bed must be aimed to align with the patient’s sleep duration. Gradually, more time is
spent in bed as sleep improves. Sleep restriction is typically administered by trained professionals and
requires close monitoring of daily sleep–wake patterns.

**Stimulus control – Behaviors that require wakefulness (e.g., watching TV, reading books) in bed
could result in bed being associated with hyperarousal, thereby perpetuating sleep difficulties.
Stimulus control helps to re-associate bed with being asleep. The patient is instructed to go to bed
only when sleepy. This helps re-associate bed with sleepiness. Activities in bed should be limited to
only sleeping and sex. The patient is instructed to leave the bed and bedroom if unable to fall sleep
within what feels like 15–30 minutes (without looking at the clock). During the time out, the patient is
instructed to complete a non-stimulating task, returning to bed when they feel comfortable. The same
morning wake-time is recommended even if sleep the night before is poor.

**Relaxation – Relaxation strategies can include progressive muscle relaxation and diaphragmatic
breathing. The goal is to release tension and arousal; however, close monitoring of patients is
important as relaxation may become sleep effort in disguise.

References

• RACGP – AJGP – Insomnia management


Last updated:
Time spent: QID:1646
2023-2-12

1648 of 1943
In which one of the following situations a depressed patient has the highest risk of attempting
suicide?

A. When the depression deteriorates.


B. On commencement of treatment.
C. On improvement with treatment.
D. After discharge from the hospital.
E. On admission to the hospital.

Incorrect. Correct answer is D


45% answered correctly

Explanation:

Correct Answer Is D

Studies suggest that in patients with depression severe enough to mandate hospital admission, the
highest risk of suicide is within 4 weeks post-discharge after inpatient care of a depressed patient.
One study showed that 43% of those who were treated as inpatient for depression attempted
suicide within 1 month of discharge and 47% died before first appointment. Overall, the risk of
suicide within 4 weeks after psychiatric inpatient care is 100 higher compared to that of general
population.

A strong belief has long been held that during the early improvement phase after commencement
of treatment, the risk of suicide is high because the apathy symptoms resolve but depressed mood
remains; this believe, however, has been questioned according to recent studies.

NOTE – In general, suicide risk in depressed patients is highest when there is an initial
symptoms improvement brought about by treatment. In this period, the depressed patient is out
of the low mood and apathy and more likely to act on a suicide plan. However, admitted patients
are monitored and cared for during their stay and risk of suicide is lower compared to after
discharge when patient is more likely to feel left on his/her own and there is no ongoing support
or monitoring.

References

• Medscape - Early Postdischarge Period Linked to Very High Suicide Risk

• Post-Discharge Suicidal Behavior Risk

• CHIR - Are Patients With Depression at Heightened Risk of Suicide as They Begin to Recover?
Last updated:
Time spent: QID:1664
2023-2-12

1649 of 1943
A 35-year-old man is brought from prison to the Emergency Department by the police and prison
staff for assessment. He is doing time in prison. Recently, he has caused problems inside. He has
picked up fights with inmates over trivial issues and had several anger outbursts. He writes sanity
words on his cell walls such as ‘I’m all ok’, ‘beat it and keep it up', ‘you are doing just fine’, etc. He
has mutilated himself several times and has cuts on both his wrists. According to the prison staff,
his condition has worsened progressively in the past few days. Which one of the following could be
the most likely?

A. Psychosis.
B. Schizophrenia.
C. Antisocial personality disorder.
D. Borderline personality disorder.
E. Factitious disorder.

Incorrect. Correct answer is D


45% answered correctly

Explanation:

Correct Answer Is D

At first glance, there might be diagnostic confusion because critical pieces of information in history
and mental status exam (MSE) are missing. The question, however, asks about the most likely
diagnosis based on the given information rather than a certain diagnosis.

There are some pieces of the puzzle that can be elicited from the scenario. This patient has been
having anger outbursts and fights over trivial issues that suggest ‘impulse control’ problems. He
has been self-mutilating, self-cutting, and refusing to eat or drink. Moreover, his sanity writings on
the cell walls indicates reassurance-seeking behavior. Since no one can reassure him, he is doing it
himself. These features make borderline personality disorder (BPD) the most likely explanation
with just this amount of available information (read the DSM 5 diagnostic criteria for BPD below).

This patient has been in imprisoned, and this could suggest antisocial personality disorder (ASPT)
as well. In ASPD there is a persistent display of impulsive, reckless, and aggressive behaviors and
no remorse about them. There is often a history of violent relationships, legal challenges, and
substance abuse if you have this personality disorder.

BPD and antisocial ASPD share some similarities. Both ASPT and BPD belong to cluster B
personality disorders (dramatic, emotional, and unpredictable behavior) and have disinhibited
behavior as a presentation. They both also have impulsivity in common. However, in ASPT the
person involves in violence and aggressiveness because they can (there is the element of control
and decision) while in BPD it happens because it cannot be controlled. Another differentiating
factor is that in ASPT the harm targets others while in BPD the harm is directed towards self. This
makes ASPD less likely of a diagnosis.

Psychosis (option A) and schizophrenia (as a psychotic disorder) (option B) are associated with
features such as impaired speech, auditory hallucinations, and delusional thinking, none of which is

1650 of 1943
present in the scenario. This makes them a less likely diagnosis.

Factitious disorder (option E) is a falsification of illness in oneself or another person (factitious by


proxy) without any obvious gain. By obvious gain, it is meant that the patient does not have a
known external motivation or achievement for producing the symptoms.

Features seen in factitious disorder are:

Inconsistencies between patient history and medical observations.


Vague details that seem plausible on the surface but that don’t hold up to scrutiny.
Lengthy medical records with multiple admissions at different hospitals.
Willingness to accept any discomfort and risk from many medical procedures, even surgery.
Overdramatic or outlandish presentation of a factitious illness, or hostility when challenged.

Although a detailed mental history might make this condition a diagnosis in this patient, later on,
the absence of the above features in the scenario makes such a diagnosis less likely compared to
BPD.

DSM-5 diagnostic criteria for BPD

A pervasive pattern of instability of interpersonal relationships, self-image and affects, and


marked impulsivity beginning by early adulthood and present in a variety of contexts, as indicated
by five (or more) or the following:

1. Frantic efforts to avoid real or imagined abandonment (Note: Do not include suicidal or self-
mutilating behavior covered in Criterion 5)
2. A pattern of unstable and intense interpersonal relationships characterized by alternating
between extremes of idealization and devaluation
3. Identity disturbance: markedly and persistently unstable self-image or sense of self
4. Impulsivity in at least two areas that are potentially self-damaging (e.g. spending, sex,
substance abuse, reckless driving, binge eating) (Note: Do not include suicidal or self-
mutilating behavior covered in Criterion 5)
5. Recurrent suicidal behavior, gestures, threats, or self-mutilating behavior
6. Affective instability due to a marked reactivity of mood (e.g., intense episodic dysphoria,
irritability , or anxiety usually lasting a few hours and only rarely more than a few days)
7. Chronic feelings of emptiness
8. Inappropriate, intense anger or difficulty controlling anger (e.g., frequent displays of temper,
constant anger, recurrent physical fights)
9. Transient, stress-related paranoid ideation or severe dissociative symptoms

References

• Psych Central - Symptoms of Borderline Personality Disorder (BPD)

• Australian BPD Foundation Limited

• Very Well Minfd - Antisocial vs. Borderline Personality Disorders

• Medscape - Factitious Disorder Imposed on Self (Munchausen's Syndrome)

1651 of 1943
Last updated:
Time spent: QID:1666
2023-2-12

1652 of 1943
Which one of the following conditions is more likely to be associated with morbid jealousy?

A. Alcoholism.
B. Schizophrenia.
C. Bipolar disorder.
D. Depression.
E. Obsessive compulsive disorder.

Incorrect. Correct answer is B


45% answered correctly

Explanation:

Correct Answer Is B

Jealousy a common yet complex emotion which has been well-recognized throughout mankind
history. Jealousy is defined as ‘feeling or showing a resentful suspicion that one’s partner is
attracted to or involved with someone else’. This definition indicates that it is a belief in the
presence of rivalry that is the key issue and that whether or not such a rivalry truly exists is less
important.

While normal jealousy is considered a normal human emotion, morbid jealousy is pathological.
Morbid jealousy is not a diagnosis rather it describes a range of irrational thoughts and emotions
that is accompanied by unacceptable or extreme behavior, in which the dominant theme is a
preoccupation with a partner’s sexual unfaithfulness based on unfounded evidence.

The following features in morbid jealousy can distinguish it from normal jealousy:

Excessive amount of time taken up by jealous concerns


Difficulty in putting the concerns out of the mind
Impairment of the relationship
Limitation of the partner's freedom
Checking on the partner's behavior

There are three forms of morbid jealousy: (1) delusional jealousy, (2) obsessional jealousy, and (3)
overvalued jealousy.

In delusional jealousy the characteristic feature is the presence of strong false belief of the partner
being unfaithful (certainty) while in obsessive jealousy the affected individual suffers from
unpleasant and irrational jealous rumination that the partner could be unfaithful (considering a
possibility not in the form of a strong belief) which is followed by compulsive checking of the
partner’s behavior. The difference between these two is the presence of a strong beliefs in
unfaithfulness in delusional jealousy versus the bothersome lingering thoughts of the possibility of
unfaithfulness. Overvalued jealousy shared many features with obsessional jealousy. In overvalued
jealousy, there is an acceptable and comprehensible idea which is excessively pursued by the
patient beyond the bounds of reason. The main difference could be lack of compulsions and
compulsive behavior in overvalued jealousy

1653 of 1943
Morbid jealousy is not a diagnosis rather a comorbidity seen in many psychiatric conditions,
personality disorders, and other settings including but not limited to drugs and alcohol abuse,
psychotic disorders and schizophrenia, mania, depression, obsessive compulsive disorder (OCD),
and anxiety disorders.

A recent study showed that of all psychiatric illnesses, schizophrenia is most likely to be
associated with morbid jealousy, most commonly in form of delusional jealousy. The study
concluded that depression (option D) could be the second most significant condition associated
with morbid jealousy. Interestingly, all three forms of morbid jealousy were identified across the
patients with depression in the study. Other conditions identified to be potentially associated with
morbid jealousy in order of strength of such association were drug and alcohol abuse (option A),
bipolar mood disorders (option C), and lastly other conditions such as anxiety disorders, OCD
(option E), dementia and other.

The same study concluded that morbid jealousy in more common in men than woman, and also
more common among married versus single men.

Treatment of morbid jealousy is with psychotherapy (e.g., CBT) and pharmacotherapy (e.g., SSRIs
or antipsychotics depending on the underlying condition and diagnosis).

NOTE - Of the personality disorders, borderline personality disorder is most prone to morbid
jealousy.

References

• HHS Public Access - Phenomenology and predisposing factors of morbid jealousy in a


psychiatric outdoor: a cross-sectional, descriptive study

• Cambridge University Press - Aspects of morbid jealousy


Last updated:
Time spent: QID:1700
2023-2-12

1654 of 1943
A 52-year-old man presents to your clinic, seeking advice on screening for colon cancer. Although he has no gastrointestinal symptoms, he is concerned because his younger
brother has been recently diagnosed with colon cancer at the age of 50 years. There is no other family history of colorectal cancer. Which one of the following is the most
appropriate advice regarding current and future management?

A. He should have a fecal occult blood test (FOBT) now.

B. No screening procedure is required now.

C. Colonoscopy should be performed now, and if normal, no further testing is required.

D. Colonoscopy should be performed now and yearly thereafter.

E. Colonoscopy should be performed now and 5-yearly thereafter.

Incorrect. Correct answer is E


45% answered correctly

Explanation:

Correct Answer Is E

Colorectal cancer (CRC) screening methods and intervals depend on the individual’s risk category. Based on the risk of developing CRC, the population is classified in
four categories:

Risk Category Screening Method / Other measures


Category 1 - average or slightly increased risk (RR: x1-2) Immunochemical faecal occult blood test (iFOBT) should be considered every two years
from age 45 years, given the risk of colorectal cancer at this age is approximately
equivalent to the population risk at age 50 years
Asymptomatic people who have: For patients aged 50–70 years, low-dose aspirin (100 mg) daily should be considered

no personal history of bowel cancer, colorectal adenomas, inflammatory bowel disease or


family history of colorectal cancer (CRC)

OR

one first-degree relative with CRC diagnosed at 55 years or older

OR

one first-degree and one second-degree relative with CRC diagnosed at 55 years or older.

Category 2: Moderately increased risk (RR: x 3-6) iFOBT should be performed every two years from age 40 to 50 years
Colonoscopy should be performed every five years from age 50 to 74 years. Computed
tomography (CT) colonography can be offered if colonoscopy is contraindicated
(1-2% of the population) Low-dose (100 mg) aspirin daily should be considered
As a result of the possibility of Lynch syndrome, a complete family history should be taken
Asymptomatic people who have: and updated regularly, and the accuracy of the cancer diagnoses and polyp pathology
should be checked carefully
Genetic testing is not appropriate at present for people with category 2 risk
one first-degree relative with CRC diagnosed before age 55 years

OR

two first-degree relatives with CRC diagnosed at any age

OR

one first-degree relative and at least two second-degree relatives diagnosed with CRC at any
age.

Category 3: High risk (RR: x7-10) iFOBT should be performed every two years from age 35 to 45 years
Five-yearly colonoscopy from age 45 to 74 years. CT colonography can be offered if
colonoscopy is contraindicated
(<1% of the population) Low-dose (100 mg) aspirin daily should be considered
Referral to a genetic centre for hereditary cancer syndromes should be considered
Asymptomatic people who have:

at least three first-degree or second-degree relatives with CRC, with at least one diagnosed
before age 55 years

OR

at least three first-degree relatives with CRC diagnosed at any age.

( ) ( )
1655 of 1943
This patient is asymptomatic, but has a brother (first-degree relative) diagnosed with CRC at the age of 50 years (<55 years). Therefore, he should be placed in category 2. People in
category 2 should be offered iFOBT every two years fron the age of 40 to 50 years and colonoscopy everu five years from 50 to 74 years of age. Since he is 52 years now,
colonoscopy now and then every five years afterwards is the most appropriate option.

References

• RACGP - The Red Book: colorectal cancer

Last updated:
Time spent: QID:490
2023-2-12

1656 of 1943
Sarah, whose husband has a plasma cholesterol of 6.4 mmol/L (normal <5.5mmol/l), wants to know which cooking oil she should use when she cooks. Which one of the following
would you advise?

A. There is not much difference between cooking oils.

B. Any margarine is suitable.

C. An oil rich in saturated fat rather than those rich in unsaturated fats.

D. She should use either canola or sunflower oil.

E. None of the above.

Incorrect. Correct answer is D


45% answered correctly

Explanation:

Correct Answer Is D

Epidemiological studies have shown reduced mortality from cardiovascular causes if diets containing increased levels of mono- and polyunsaturated fatty acids are used. Canola oil
and olive oil have a high concentration of monounsaturated fatty acids, while sunflower oil is rich in polyunsaturated fatty acids. Both canola and sunflower oil are appropriate
dietary oils for Sarah's husband.

For lowering the plasma cholesterol levels, the National Heart Foundation of Australia recommends that saturated fat in the diet be replaced with a combination of mono- and
polyunsaturated fats.

Cholesterol content of foods does increases LDL cholesterol. LDL cholesterol is directly related to the amount of saturated and trans fat contents of dietary intake.

References

• http://www.heartfoundation.org.au/healthy-eating/f

Last updated:
Time spent: QID:109
2023-2-12

1657 of 1943
A 42-year-old olive-skinned man comes to your GP practice because he is concerned about contacting melanoma after he watched a TV program about it. He has no family history of melanoma or other
skin cancers. On examination, there is no abnormal finding. You reassure him but he says that he will come back to you if he finds anything unusual. Which one of the following will you recommend
instead for follow-up?

A. Follow-up every six months.

B. Follow-up every two years.

C. Follow-up every five years.

D. Start him on regular medication.

E. Follow-up every 12 months.

Incorrect. Correct answer is C


45% answered correctly

Explanation:

Correct Answer Is C

Australia has the highest incidence of skin cancer in the world. Current clinical guidelines do not recommend systemic skin cancer screening, but in clinical practice many general practitioners do
provide skin checks for their patients.

A ‘skin-check’ can be defined as a comprehensive assessment of any asymptomatic patient for any evidence of skin cancer. Current Australian guidelines advise against general population screening
for skin cancer, based on lack of evidence that justifies organized screening as an effective method to reduce mortality. Patient self examination with opportunistic screening is the current standard.

One’s risk of contracting skin cancer is classified as ‘high-risk’, ‘intermediate risk’, or ‘low risk’ according to the following table:

High-risk
Red hair
Type I skin and age more than 45 years
Type II skin and age more than 65 years
Family history of melanoma in a first-degree relative in a patient aged more than 15 years
More than 100 nevi (more than 10 dysplastic nevi)
Past history of melanoma
Past history of non-melanoma skin cancers or more than 20 solar keratoses

Medium-risk
Blue eyes
Type I skin and age between 25-45 years
Type II skin and age between 45-65 years
Type III skin and age more than 65 years
Family history of non-melanoma skin cancer in first-degree relatives
Past history of solar keratosis
Multiple episodes of previous sunburns

Low-risk
Type I skin and age less than 25 years
Type II skin and age less than 45 years
Type III skin and age less than 65 years
Type IV or V skin

Recommendations for skin check are as follows:

High-risk: 3-monthly self examination and 12 monthly skin check with doctor
Medium-risk: 3- to 6-monthly self examination and 2- to 5-yearly skin check with doctor
Low-risk: 12-monthly self examination and check with doctor for assessment of risk and advice regarding skin care

With olive-colored skin, this man has a skin type of IV to V. Considering the additional fact that he has no family history of skin cancer, he is low-risk for developing skin cancer. The recommendations for
this patient are annual self examination and one-shot check with doctor. However, he has not been fully reassured despite your efforts. For this reason and for putting his mind at ease 5-yearly check –
up can be offered.

TOPIC REVIEW

The Fitzpatrick Skin Type is a skin classification system that classifies skin types based on a scoring system:

1658 of 1943
Type I – pale white skin, blue/hazel eyes, blonde/red hair, always burns, does not tan

Type II - fair skine, blue eyes, burns easily, tans poorly

Type III - darker white skin, tans after initial burn

Type IV - light brown skin, burns minimally, tans easily

Type V - brown skin, rarely burns, tans darkly easily

Type VI - dark brown or black skin, never burns, always tans darkly

References

• RACGP - Skin checks

• Australia Government - Australian Nuclear Protection and Nuclear Safety Agency - Fitzpatrick skin Phototype

• DermNEt Newzealand - Fitzpatrick skin phototype

Last updated:
Time spent: QID:612
2023-2-12

1659 of 1943
A 37-year-old computer software engineer man comes to you because he is concerned about developing malignant melanoma. His concerned arose when his father was diagnoses with melanoma eight months ago. He does not drink alcohol nor does he smoke. On
examination, only multiple benign nevi are noted. Which one of the following would be the most appropriate management of this patient?

A. Excision of his benign nevi.

B. Refer him to a dermatologist.

C. Review him in 12 months.

D. Reassure him.

E. Review him in 2-5 years.

Incorrect. Correct answer is C


45% answered correctly

Explanation:

Correct Answer Is C

Australia has the highest incidence of skin cancer in the world. Current clinical guidelines do not recommend systemic skin cancer screening, but, in clinical practice many general practitioners do provide skin checks for their patients.

A ‘skin-check’ can be defined as a comprehensive assessment of any asymptomatic patient for any evidence of skin cancer. Current Australian guidelines advise against general population screening for skin cancer, based on lack of evidence to justy organized screening as an
effective method to reduce mortality. Patient self-examination with opportunistic screening is the current standard.

One’s risk of contracting skin cancer is classified as ‘high-risk’, ‘intermediate risk’, or ‘low risk’ according to the following table:

High-risk
Red hair
Type I skin and age more than 45 years
Type II skin and age more than 65 years
Family history of melanoma in a first-degree relative in a patient aged more than 15 years
More than 100 nevi (more than 1 dysplastic nevi)
Past history of melanoma
Past history of non-melanoma skin cancers or more than 20 solar keratoses

Medium-risk
Blue eyes
Type I skin and age between 25-45 years
Type II skin and age between 45-65 years
Type III skin and age more than 65 years
Family history of non-melanoma skin cancer in first-degree relatives
Past history of solar keratosis
Multiple episodes of previous sunburns

Low-risk
Type I skin and age less than 25 years
Type II skin and age less than 45 years
Type III skin and age less than 65 years
Type IV or V skin

Recommendations for skin check are as follows:

High-risk: 3-monthly self examination and 12-monthly skin check with doctor
Medium-risk: 3- to 6-monthly self examination and 2- to 5-yearly skin check with doctor
Low-risk: 12-monthly self examination and one-shot skin check with doctor for assessment of risk and advice regarding skin care

With the father being diagnosed with melanoma, this man is categorised as ‘high-risk’ for skin cancers. It is recommended that high-risk people have 3-6 monthly self examination and check up with doctor every 12 months.

(Option A) Removing simple nevi to prevent melanoma is not recommended, because melanoma often arise de novo from other sites other than the nevi.

(Option B) Referring the patient to a dermatologist is not necessary at this stage because the patient is asymptomatic now.

(Option D) Reassuring the patient is not appropriate because he is high risk for melanoma.

(Option E) Check up with doctor every 2-5 years would be the option if this man was categorised as medium risk.

TOPIC REVIEW

The Fitzpatrick Skin Type is a skin classification system that classifies skin types based on a scoring system:

Type I – pale white skin, blue/hazel eyes, blonde/red hair, always burns, does not tan

1660 of 1943
Type II - fair skine, blue eyes, burns easily, tans poorly

Type III - darker white skin, tans after initial burn

Type IV - light brown skin, burns minimally, tans easily

Type V - brown skin, rarely burns, tans darkly easily

Type VI - dark brown or black skin, never burns, always tans darkly

References

• RACGP - Skin checks

• Australia Government - Australian Nuclear Protection and Nuclear Safety Agency - Fitzpatrick skin Phototype

• DermNEt Newzealand - Fitzpatrick skin phototype

Time spent: QID:618 Last updated:


2023-2-12

1661 of 1943
You are giving a lecture in a primary school regarding skin cancer awareness. Which one of the following is the most appropriate advice you should emphasize on?

A. Using sunscreens in the morning.

B. Avoiding or lessening sun-exposure between 10am and 4pm.

C. Annual skin checks and screening.

D. Excision of moles.

E. Avoiding sun-exposure.

Incorrect. Correct answer is B


45% answered correctly

Explanation:

Correct Answer Is B

Australia has the highest rate of skin cancers in the world. Protective measures help reduce development of skin cancers. Of all known risk factors for skin cancer, sun-exposure is the most important MODIFIABLE one (not the most important one in
gneral).

All people (especially children aged ≤ 10 years) should be advised to use protective measures when UV levels are 3 and above. UV level throughout the day depends on the season, but generally UV levels are highest between 10am and 4pm, the time
during which protection against sun-exposure should be advised.

These measures include:

Broad-brimmed, bucket or legionnaire-style hats


Protective clothing
Sunglasses
Sunscreens with at least a sun protection factor (SPF) of (needs reapplication every 2 hours)

(Option A) Using sunscreens only in the morning is not because sunscreens are required to be reapplied every 2 hours. UV level in the early hours of the morning is not expected to be that high compared to later hours such as noon and afternoon.

(Option C) Annual skin checks is advisable for those who are high risk for skin cancers, and is not recommended for general population.

(Option D) Excision of moles has not shown to be associated with decreased risk of skin cancers. Furthermore, melanomas have shown to often arise de novo (from areas of the skin with no pre-existing moles).

(Option E) Complete avoidance from sun-exposure results in vitamin D insufficiency and its complications, and is not recommended.

References

• http://www.racgp.org.au/your-practice/guidelines/r

• http://www.bom.gov.au/uv/

Last updated:
Time spent: QID:681 2023-2-12

1662 of 1943
In a randomized controlled trial (RCT) conducted to study the effect of aspirin on prevention of coronary artery events among diabetic smokers, the results in the two arms of the study are as follows:

ASA group Placebo group


Coronary event 1 2
No coronary event 99 98

Which one of the following is the relative risk of not using ASA?

A. 1%.

B. 2%.

C. 100%.

D. 200%.

E. 50%.

Incorrect. Correct answer is D


45% answered correctly

Explanation:

Correct Answer Is D

In statistics and epidemiology, relative risk or risk ratio (RR is the ratio of the probability of an event occurring in an exposed group to the probability of the event occurring in a non-exposed, comparison group.

Of 100 diabetic smokers who are on aspirin, 1 person has developed a coronary event. So the incidence of coronary event in this group is 1% [1/(1+99)x100], while the incidence of coronary events in the group taking placebo instead of aspirin is 2%
[2/(2+98)x100].

In this scenario the exposure is taking aspirin. Exposed group has a 1% chance of developing a coronary event versus 2% in those who do not take aspirin.

The RR is then calculated by dividing the odds of the condition in the exposed group (1%) by that of the non-exposed group:

RR= P(exposed) / P(non-exposed) : RR=1% / 2%=0.5

Here, the RR indicates that the odds of developing a coronary event in those diabetic smokers who are on aspirin is half compared to those on placebo. In other words, those who are on aspirin has a 50% risk reduction. Inversely, those who are not
taking aspirin are twice as likely to develop a coronary event compared to those who are taking it. So the RR. This means that not taking aspirin is associated with a 200% increase in incidence of coronary events.

References

• http://www.wikihow.com/Calculate-Relative-Risk

Last updated:
Time spent: QID:1299 2023-2-12

1663 of 1943
A 67-year-old woman presents for receiving her annual influenza vaccination. She also mentions that her daughter is going to have a baby in 2 months and asks if she should receive other vaccines that may help. Which one of the following vaccines
will you advise?

A. Pneumococcal vaccine.

B. Varicella vaccine.

C. Hepatitis B vaccine.

D. DPT vaccine.

E. Hemophilus influenza vaccine.

Incorrect. Correct answer is D


45% answered correctly

Explanation:

Correct Answer Is D

According to vaccination national program every child should be vaccinated against pertussis (whooping cough infection at ages 6 weeks, 4 months and 6 months.

The mother, the father and other adults in close contact with young babies can be the source of whooping cough infection in children who are still too young to be vaccinated. For this reason, they should seek advice from their GPs about the benefits
of getting an adult pertussis-containingvaccine.

Vaccination against whooping cough (pertussis) is strongly recommended for adults in contact with children too young to be vaccinated. These people should be vaccinated before or as early as possible after the birth of the baby if they have not
had a pertussis vaccine in the past 10 years.

Pneumococcal was indicated for this woman for her own sake, and not the baby's, if she was older than 65.

References

• http://www.health.nsw.gov.au/immunisation/Pages/wh

Last updated:
Time spent: QID:705 2023-2-12

1664 of 1943
You are going to give a lecture about obesity and body mass index (BMI). Which one of the following is the most appropriate advice?

A. BMI alone is the best way to estimate obesity.

B. BMI gives false estimates in elderly people because of a fatty abdomen.

C. Waist circumference is the best way to estimate obesity.

D. BMI in conjunction with waist circumference is the best way to estimate obesity.

E. BMI alone is the best predictor of cardiovascular risk.

Incorrect. Correct answer is D


45% answered correctly

Explanation:

Correct Answer Is D

An adult’s BMI can be compared to thresholds to define whether a person is underweight, of healthy weight, overweight or obese based on the WHO classifications. BMI is calculated by dividing weight in kilograms by the square of height in meters.

WHO classification is shown in the following table:

BMI Classification
<18.5 Underweight
18.5 – 24.9 Normal
24.9. 29.9 Overweight
30-34.9 Obese I
35-39.9 Obese II
≥40 Obese III

Individuals with the same BMI may have different ratios of body fat to lean mass. People with high muscle mass (e.g. athletes) may have a lower proportion of body fat than less muscular people, so a higher BMI threshold must be considered.
Women have more body fat than men with equal BMIs. People lose lean tissue with age, so an older person will have more body fat than a younger one at the same BMI.

This fact necessitates waist circumference as an additional factor. Waist circumference is a good indicator of total body fat and is also a useful predictor of visceral fat. Compared to BMI, waist circumference is a better predictor of cardiovascular
risk and type 2 diabetes (in women, but not in men).

The best method for estimation of obesity in adults is a combination of BMI and waist circumference. The latter takes into account fat distribution, and in combination with BMI, gives a more accurate benchmark for obesity. Alhtough this
combination is useful for determining the cardiovacular risk due to obesity, the most accurate measure for prediction of cardiovascular risk and ischmeic heart disease is 'waist to hip ratio' (not an option here).

(Option A) BMI alone is never the best predictor of cardiovascular risk in adults.

(Option B) BMI gives false negative results in older people due to decreased total lean mass, not only the abdomen. Even in an elderly with a flat abdomen, BMI may not be accurate due to the fact the most of their weight is comprised of fat rather
than lean mass.

(Option C) Waist circumference alone is not accurate and should not be interpreted as an indicator of cardiovascular risk.

(Option E) BMI is not a predictor of cardiovascular risk an as mentioned earlier, may not be an accurate tool for estimation of obesity in certain groups such as muscular athletes, the elderly, etc.

References

• https://www.nhmrc.gov.au/_files_nhmrc/publications/attachments/n57_obesity_guidelines_140630.pdf

Last updated:
Time spent: QID:718 2023-2-12

1665 of 1943
A 49-year-old male, commercial truck driver by profession, presents with left-sided chest pain radiating to his left arm and jaw. Electrocardiography shows ST segment elevation myocardial infarction (STEMI). Thrombolysis is done with tenecteplase,
which resulted in resolution of symptoms. Which one of the following would be the best advice regarding driving?

A. He can drive a private car after four weeks.

B. He can drive his commercial truck after four weeks.

C. There is no driving restriction.

D. He can never drive his truck.

E. He should drive carefully and avoid driving on main highways.

Incorrect. Correct answer is B


45% answered correctly

Explanation:

Correct Answer Is B

Following an acute myocardial infarction, the patient cannot drive private vehicles for two weeks and commercial vehicles for four weeks.

TOPIC REVIEW

There are several conditions or procedures which lead to reduced ability to drive. Under these circumstances, the treating physician should give appropriate advice regarding driving. It is the patients’ duty (not the treating physician) to inform the
Road Safety Department about their condition. Failing to do so will lead to prosecution. The most commonly encountered medical problems/procedures that may be faced and the consequent driving limitations, including non-driving periods, are
listed in the following table.

EVENT /PROCEDURE Minimum non-driving period (advisory)


Private drivers Commercial drivers
Ischaemic heart disease
Acute myocardial infarction 2 weeks 4 weeks
Percutaneous coronary intervention e.g. angioplasty 2 days 4 weeks
Coronary artery bypass graft 4 weeks 3 months

Disorders of the rate, rhythm and conduction

Cardiac arrest 6 months 6 months


6 months
Implantable cardioverter defibrillator (ICD) Not applicable
after cardiac arrest
Generator change of an ICD 2 weeks Not applicable
ICD therapy associated with associated with symptoms of haemodynamic compromise 4 weeks Not applicable
Cardiac pacemaker insertion 2 weeks 4 weeks
Vascular disease
Aneurysm repair 4 weeks 3 months
Valvular repair 4 weeks 3 months
Other cardiovascular conditions
Deep vein thrombosis 2 weeks 2 weeks
Pulmonary embolism 6 weeks 6 weeks
Heart/lung transplant 6 weeks 3 months
Syncope (due to cardiovascular event) 4 weeks 3 months
Epilepsy
First seizure OR isolated seizure 6 months 5 years
Recently diagnosed with epilepsy 12 months 10 years
12 months after last seizure (if on
Chronic epilepsy + uncontrolled seizure in the past 10 years
treatment)
Seizure only in sleep OR treated with surgery 12 months after last seizure 10 years
3 months after dose reduction (if no other No driving license except when the
Reducing the dose of one or more antiepileptic drugs.
seizure occurs) dose is reduced due to its side effects
4 weeks if a provocative cause can be
Seizure in a person whose epilepsy was previously well controlled
identified. 10 years, provided that the EEG shows
no epileptiform activity; and the person
follows medical advice, including
‘Well controlled’ is defined as: There were no seizures during the 12 months leading up to
3 months if a provocative cause cannot be adherence to medication if prescribed.
the last seizure.
identified.
Vertigo
Benign paroxysmal positional vertigo 3 months 6 months
Conditional license if there are alarming
Conditional license if there are alarming
Meniere’s disease symptoms to aware the driver of the
symptoms to aware the driver of the attack
attack
Stroke / intracranial hemorrhage
Intracranial surgery 6 months 12 months
4 weeks 3 months

Stroke (ischemic or hemorrhagic)


Conditional licensing depends on the Conditional licensing depends on the
residual defects residual defects
TIA 2 weeks 4 weeks
Subarachnoid hemorrhage 3 months 6 months
Visual acuity / visual fields
No driving license if acuity in the better eye
No driving license if acuity in the better
Visual acuity or with both eyes together is worse than
eye is worse than 6/9.
6/12.

References

• Austroads - Assessing Fitness to Drive

L t d t d

1666 of 1943
Last updated:
Time spent: QID:719 2023-2-12

1667 of 1943
A 65-year-old man presented to the emergency department with complaint of chest pain that turned out to be of cardiac origin. Electrocardiographic changes were consistent with inferior ST elevation myocardial infarction (STEMI). He
underwent coronary artery bypass graft the next day. He is a commercial driver and on discharge wants to know if he can continue driving. Which one of the following would be the most appropriate advice?

A. He can drive private and commercial vehicles as long as he feels fine.

B. He should write a letter to road safety department.

C. No commercial driving for four weeks.

D. No commercial driving for three months

E. He can never drive commercial vehicles, but he can drive private vehicles after three months.

Incorrect. Correct answer is D


45% answered correctly

Explanation:

Correct Answer Is D

After coronary artery bypass grafting, one should not drive private vehicles for at least four weeks and commercial vehicles for three months.

TOPIC REVIEW

There are several conditions or procedures which lead to reduced ability to drive. Under these circumstances, the treating physician should give appropriate advice regarding driving. It is the patients’ duty (not the treating physician) to inform the
Road Safety Department about their condition. Failing to do so will lead to prosecution. The most commonly encountered medical problems/procedures that may be faced and the consequent driving limitations, including non-driving periods, are
listed in the following table:

EVENT /PROCEDURE Minimum non-driving period (advisory)


Private drivers Commercial drivers
Ischaemic heart disease
Acute myocardial infarction 2 weeks 4 weeks
Percutaneous coronary intervention e.g. angioplasty 2 days 4 weeks
Coronary artery bypass graft 4 weeks 3 months

Disorders of the rate, rhythm and conduction

Cardiac arrest 6 months 6 months


6 months
Implantable cardioverter defibrillator (ICD) Not applicable
after cardiac arrest
Generator change of an ICD 2 weeks Not applicable
ICD therapy associated with associated with symptoms of haemodynamic compromise 4 weeks Not applicable
Cardiac pacemaker insertion 2 weeks 4 weeks
Vascular disease
Aneurysm repair 4 weeks 3 months
Valvular repair 4 weeks 3 months
Other cardiovascular conditions
Deep vein thrombosis 2 weeks 2 weeks
Pulmonary embolism 6 weeks 6 weeks
Heart/lung transplant 6 weeks 3 months
Syncope (due to cardiovascular event) 4 weeks 3 months
Epilepsy
First seizure OR isolated seizure 6 months 5 years
Recently diagnosed with epilepsy 12 months 10 years
12 months after last seizure (if on
Chronic epilepsy + uncontrolled seizure in the past 10 years
treatment)
Seizure only in sleep OR treated with surgery 12 months after last seizure 10 years
3 months after dose reduction (if no other No driving license except when the
Reducing the dose of one or more antiepileptic drugs.
seizure occurs) dose is reduced due to its side effects
4 weeks if a provocative cause can be
Seizure in a person whose epilepsy was previously well controlled
identified. 10 years, provided that the EEG shows
no epileptiform activity; and the person
follows medical advice, including
‘Well controlled’ is defined as: There were no seizures during the 12 months leading up to
3 months if a provocative cause cannot be adherence to medication if prescribed.
the last seizure.
identified.
Vertigo
Benign paroxysmal positional vertigo 3 months 6 months
Conditional license if there are alarming
Conditional license if there are alarming
Meniere’s disease symptoms to aware the driver of the
symptoms to aware the driver of the attack
attack
Stroke / intracranial hemorrhage
Intracranial surgery 6 months 12 months
4 weeks 3 months

Stroke (ischemic or hemorrhagic)


Conditional licensing depends on the Conditional licensing depends on the
residual defects residual defects
TIA 2 weeks 4 weeks
Subarachnoid hemorrhage 3 months 6 months
Visual acuity / visual fields
No driving license if acuity in the better eye
No driving license if acuity in the better
Visual acuity or with both eyes together is worse than
eye is worse than 6/9.
6/12.

1668 of 1943
References

• Austroads - Assessing Fitness to Drive

Last updated:
Time spent: QID:720 2023-2-12

1669 of 1943
A 65-year-old was found to have an abdominal aortic aneurysm. The aneurysm was repaired by a vascular surgeon, and the patient was transferred to the Intensive Care Unit (ICU) where he stayed for 24 hours. The recovery period was uneventful.
The patient is now ready to be discharged, and wants to know whether he can drive his private car. Which one of the following is the most appropriate advice regarding driving?

A. He is unfit to drive for six months post-repair.

B. He is unfit to drive for six months as he has been in intensive care unit.

C. He can drive after he spends a week at home without any symptoms.

D. After his general practitioner considers him fit to drive.

E. He is unfit to drive for four weeks.

Incorrect. Correct answer is E


45% answered correctly

Explanation:

Correct Answer Is E

After repair of an aortic aneurysm or cardiac valvular repair, one is unfit to drive their private motor vehicles for at least four weeks. This extends to three months for commercial vehicles. They can then have their unconditional driving license again.

ICU admission for 24 hours is a normal routine after some surgeries and does not pose any restriction on driving by itself.

Referral to general practitioner is not the correct answer as the patient should be informed of the restrictions upon discharge. The patient may plan to visit his general practitioner in a week and without knowing about his driving limitations.

References

• Austroads - Assessing Fitness to Drive

Last updated:
Time spent: QID:721 2023-2-12

1670 of 1943
You are counselling a patient who is concerned about the cholesterol content of foods. Which one of the following foods contains the most cholesterol content?

A. Yoghurt.

B. Avocado.

C. Coconut oil.

D. Peanut butter.

E. Canola.

Correct
45% answered correctly

Explanation:

Correct Answer Is A

Cholesterol is only found in animal products such as meat, poultry, fish, dairy products and egg. Although vegetable products have different levels of fat, they do not contain
cholesterol. Of the given options, only yoghurt (a dairy product) contains cholesterol.

References

• http://www.betterhealth.vic.gov.au/bhcv2/bhcarticl

Last updated:
Time spent: QID:110
2023-2-12

1671 of 1943
A 45-year-old man presents to the emergency department with a self-limiting episode of seizure. He is known to have epilepsy which has been well-controlled with carbamazepine for the last 12 months. Full investigations including blood tests and
CT scan of the head reveals no apparent cause for the seizure. The patient is keen to know about the driving restriction. Which one of the following is the appropriate advice regarding driving?

A. He cannot drive for 4 weeks.

B. He cannot drive for 3 months.

C. He cannot drive for 6 months.

D. He cannot drive for 12 months.

E. He cannot drive for 3 years.

Incorrect. Correct answer is B


45% answered correctly

Explanation:

Correct Answer Is B

If one develops an episode of seizure after at least 12 months of being well-controlled by antiepileptic drugs, they cannot drive for 4 weeks if a provocative factor (sleep deprivation, alcohol, electrolyte abnormality, CNS lesion, etc) can be identified,
and for 3 months if no cause is found.

The patient then may be eligible to hold a conditional driving license provided that the patient does not experience another attack during the mentioned periods.

References

• https://www.onlinepublications.austroads.com.au/it

Last updated:
Time spent: QID:722 2023-2-12

1672 of 1943
A 65-year-old commercial driver sustianed stork and presented with left hemiparesis and left homonyms hemianopia. He is now making a good recovery. Which of the following is most appropriate step regarding driving?

A. Permanent restriction of driving.

B. Driving assessment supervised by an occupational therapist.

C. He should not drive for two weeks.

D. Refer him to a neurologist to decide about fitness to drive.

E. He can continue driving.

Incorrect. Correct answer is B


45% answered correctly

Explanation:

Correct Answer Is B

Cerebrovascular events (e.g. stroke, TIA) make the patient unfit to drive non-commercial vehicles for at least four weeks. Once there is no residual deficit and the risk of recurrence is minimized by appropriate measures such as prophylactic
anticoagulation, the patient should be assessed by an occupational therapist (or any other relevant consultants such as ophthalmologist, neurologist, etc. depending on the residual defects) for evaluation of fitness to drive.

There are a wide range of practical assessments available, including off-road, on-road and driving simulator assessments, each with strengths and limitations. Assessments may be conducted by occupational therapists trained in driver assessment
or by others approved by the particular driver licensing authority. Processes for initiating and conducting driver assessments vary between the states and territories and choice of assessment depends on resource availability, logistics, cost and
individual requirements.

The assessments may also be initiated by the examining health professional, other referrers (e.g. police, self, family) or by the driver licensing authority.

In this case, improvement should be assessed by an occupational therapist and ophthalmologist. Any further process regarding driving private or commercial depends on expert opinions from these disciplines.

References

• Austroads - Assessing Fitness to Drive

Last updated:
Time spent: QID:723 2023-2-12

1673 of 1943
A 39-year-old woman presents with second episode of seizure within one week. An EEG confirms the diagnosis of epilepsy. She is started on carbamazepine. She asks you when she can drive again. Which one of the following would be the correct
answer?

A. Six months.

B. Four weeks.

C. She cannot drive as long as she is on carbamazepine.

D. She can drive as long as she is on carbamazepine.

E. She cannot drive for five years.

Correct
45% answered correctly

Explanation:

Correct Answer Is A

If a patient is diagnosed with epilepsy for the first time, a conditional license may be considered by the driver licensing authority subject to at least annual review, taking into account information provided by the treating doctor as to whether the
following criteria are met:

The patient has been treated for at least six months


There have been no seizures in the preceding six months
If any seizures occurred after the start of treatment, they happened only in the first six months after starting treatment and not in the last six months*
The person follows medical advice, including adherence to medication

For this patient, who has been diagnosed with epilepsy for the first time, a limitation of six months and she cannot drive non-commercial vehicles during this period.

* When treatment with an anti-epileptic drug is started in a previously untreated person, sufficient time should pass to establish that the drug is effective before driving is recommenced. However, effectiveness cannot be established until the person reaches
an appropriate dose. For example, if a drug is being gradually introduced over three weeks and a seizure occurs in the second week, it would be premature to consider the drug ineffective. The standard allows seizures to occur within the first six months
after starting treatment without lengthening the required period of seizure freedom. However, if seizures occur more than six months after starting therapy, a longer seizure-free period is required. For commercial drivers, the default standard applies.

Example: if a patient has a seizure three months after starting therapy, they may be fit to drive six months after the most recent seizure (nine months after starting therapy). However, if a person experiences a seizure 8 months after starting therapy, the
default standard applies and they may not be fit to drive until 12 months after the most recent seizure.

References

• Austroads - Assessing Fitness to Drive

Last updated:
Time spent: QID:724 2023-2-12

1674 of 1943
A 37-year-old epileptic man seeks advice regarding driving. He was diagnosed with epilepsy five years ago and was started on carbamazepine; however, he experienced intermittent seizures despite treatment. Three months ago, carbamazepine was
switched to phenytoin.He has not had any seizures since then. Which one of the following would be the most appropriate advice regarding driving a non-commercial vehicle?

A. He cannot drive for three months.

B. He cannot drive for six months.

C. He cannot drive for one year.

D. He cannot drive for two years.

E. He can never drive.

Incorrect. Correct answer is C


45% answered correctly

Explanation:

Correct Answer Is C

All patients with seizures and epilepsy should avoid driving non-commercial vehicles for 12 months and commercial vehicles for 10 years as default standards.

NOTE - There are circumstances under which these periods may be subject to reduction. Some of these circumstances are listed in the following table:

Condition Non-commercial vehicle Commercial vehicle


No exclusion provided that no seizure has occurred after 11 years No limitation provided that no seizure has occurred after 11 years
History of a benign seizure or epilepsy syndrome usually limited to
of age. With seizure after 11 years of age general rule applies of age. With seizure after 11 years of age general rule applies
childhood
unless the situation matches one of those below unless the situation matches one of those below
First-time seizure 6 months 5 years
Epilepsy treated for the first time:

6 months Default period applies (no reduction)


This applies when antiepileptic treatment has been started for the
first time within the preceding 18 months.
Acute symptomatic seizure caused by head injury, metabolic
6 months 12 months
disorders, etc in which a cause for the seizure can be found
Seizure in a person whose epilepsy was previously well controlled.

‘Well controlled’ is defined as:


3 months Default period applies (no reduction)

There were no seizures during the 12 months leading up to the last


seizure
‘Safe’ seizures:

No exclusion provided that ‘safe’ seizures have been present for at


These are defined as seizures that do not impair driving ability least two years; and there have been no seizures of other type for at Default period applies (no reduction)
(which requires consciousness and ability to control the vehicle at least two years
all times). Consciousness must be verified by witnesses or video-
EEG.
Sleep-only seizures:
12 months Default period applies (no reduction)
seizures occurring only during sleep
No driving during the period in which the dose is being tapered; and
for three months after the last dose.

Planned withdrawal of one or more antiepileptic medications in a If any anti-epileptic medication is to be withdrawn, the person will
If seizures recur, the driver licensing authority may allow the person
person who satisfies the standard to hold a conditional license no longer meet the criteria to hold a conditional license.
to resume driving on a conditional license subject to at least
annual review if the previously effective medication regime is
resumed; and there have been no seizures for four weeks after
resuming the medication regime
No driving during the period in which the dose reduction is being If any anti-epileptic medication is to be withdrawn, the person will
made; and for 3 months after completion of the dose reduction. no longer meet the criteria to hold a conditional license.
Reduction in dosage of anti-epileptic medication in a person who
satisfies the standard to hold a conditional license
Driving may continue if the dose reduction is due only to the Driving may continue if the dose reduction is due only to the
presence of side-effects. presence of side-effects.

This patient does not fulfill any of the above criteria to be subject to an exception to the general rules; therefore, he should not drive a non-commercial vehicle for at least 12 months after his last seizure, provided that no seizures occurred during the
preceding 12 months.

References

• Austroads - Assessing Fitness to Drive

Last updated:
Time spent: QID:725 2023-2-12

1675 of 1943
A 23-year-old woman presents for advice regarding driving after a first-time generaliZed seizure. She had episodes of sudden spasms and twitching of muscles in the past few years. Which one of the following would be the most appropriate advice
regarding driving a non-commercial vehicle?

A. No driving for six months.

B. No driving for three months.

C. No driving for one month.

D. She cannot drive anymore.

E. No driving for 12 months.

Correct
45% answered correctly

Explanation:

Correct Answer Is A

All patients with seizures and epilepsy should avoid driving non-commercial vehicles for 12 months and commercial vehicles for 10 years as per default standards for patients with seziure. However, there are circumstances under which these
periods may be subject to reduction. These circumstances are listet in the following table:

Condition Non-commercial vehicle Commercial vehicle


No exclusion provided that no seizure has occurred after 11 years No limitation provided that no seizure has occurred after 11 years
History of a benign seizure or epilepsy syndrome usually limited to
of age. With seizure after 11 years of age general rule applies of age. With seizure after 11 years of age general rule applies
childhood
unless the situation matches one of those below unless the situation matches one of those below
First-time seizure 6 months 5 years
Epilepsy treated for the first time:

6 months Default period (10 years) applies (no reduction)


This applies when antiepileptic treatment has been started for the
first time within the preceding 18 months.
Acute symptomatic seizure caused by head injury, metabolic
6 months 12 months
disorders, etc in which a cause for the seizure can be found
Seizure in a person whose epilepsy was previously well controlled.

‘Well controlled’ is defined as:


3 months Default period (10 years) applies (no reduction)

There were no seizures during the 12 months leading up to the last


seizure
‘Safe’ seizures:

No exclusion provided that ‘safe’ seizures have been present for at


These are defined as seizures that do not impair driving ability least two years; and there have been no seizures of other type for at Default period (10 years) applies (no reduction)
(which requires consciousness and ability to control the vehicle at least two years
all times). Consciousness must be verified by witnesses or video-
EEG.
Sleep-only seizures:

12 months Default period (10 years) applies (no reduction)


seizures occurring only during sleep
No driving during the period in which the dose is being tapered; and
for three months after the last dose.

Planned withdrawal of one or more antiepileptic medications in a If any anti-epileptic medication is to be withdrawn, the person will
If seizures recur, the driver licensing authority may allow the person
person who satisfies the standard to hold a conditional license no longer meet the criteria to hold a conditional license.
to resume driving on a conditional license subject to at least
annual review if the previously effective medication regime is
resumed; and there have been no seizures for four weeks after
resuming the medication regime
No driving during the period in which the dose reduction is being If any anti-epileptic medication is to be withdrawn, the person will
made; and for 3 months after completion of the dose reduction. no longer meet the criteria to hold a conditional license.
Reduction in dosage of anti-epileptic medication in a person who
satisfies the standard to hold a conditional license
Driving may continue if the dose reduction is due only to the Driving may continue if the dose reduction is due only to the
presence of side-effects. presence of side-effects.

Those who experience first-time seizure are exceptions to the general rule. They should not drive non-commercial vehicles for six months and commercial vehicles for five years. Thereafter, a conditional driving license for non-commercial vehicles
may be considered by driving authority subject to at least annual review, if there has been no seizures (with or without treatment for at least six months).

References

• Austroads - Assessing Fitness to Drive

Last updated:
Time spent: QID:726 2023-2-12

1676 of 1943
A 42-year-old epileptic man, who had intermittent seizures despite being on treatment with sodium valproate, was switched to carbamazepine one month ago. Since starting the medication, there has been no seizure. Which one of the following
would be the most appropriate advice regarding driving a non-commercial vehicles for him?

A. He can never drive.

B. He can drive after six months of seizure-free period.

C. He can only drive after 12 months of seizure-free period.

D. He can drive after 10 years of seizure-free period.

E. He can drive now.

Incorrect. Correct answer is C


45% answered correctly

Explanation:

Correct Answer Is C

The scenario describes a case of chronic seizure. Generally, patients with seizures and epilepsy should not drive non-commercial vehicles for 12 months and commercial vehicles for 10 years as default standards after the last episode of their
seizures, unless their condition is one of the exceptions for them different limitations is applied. These conditions are listed in the following table:

Condition Non-commercial vehicle Commercial vehicle


No exclusion, provided that no seizure has occurred after 11 years No limitation, provided that no seizure has occurred after 11 years
History of a benign seizure or epilepsy syndrome usually limited to
of age. With seizure after 11 years of age general rule applies of age. With seizure after 11 years of age general rule applies
childhood
unless the situation matches one of those below unless the situation matches one of those below
First-time seizure 6 months 5 years
Epilepsy treated for the first time:

6 months Default period (10 years) applies (no reduction)


This applies when antiepileptic treatment has been started for the
first time within the preceding 18 months.
Acute symptomatic seizure caused by head injury, metabolic
6 months 12 months
disorders, etc. in which a cause for the seizure can be found
Seizure in a person whose epilepsy was previously well controlled.

‘Well controlled’ is defined as:


3 months Default period (10 years) applies (no reduction)

There were no seizures during the 12 months leading up to the last


seizure
‘Safe’ seizures:

No exclusion provided that ‘safe’ seizures have been present for at


These are defined as seizures that do not impair driving ability least two years; and there have been no seizures of other type for at Default period (10 years) applies (no reduction)
(which requires consciousness and ability to control the vehicle at least two years
all times). Consciousness must be verified by witnesses or video-
EEG.
Sleep-only seizures:

12 months Default period (10 years) applies (no reduction)


seizures occurring only during sleep
No driving during the period in which the dose is being tapered; and
for three months after the last dose.

Planned withdrawal of one or more antiepileptic medications in a If any anti-epileptic medication is to be withdrawn, the person will
If seizures recur, the driver licensing authority may allow the person
person who satisfies the standard to hold a conditional license no longer meet the criteria to hold a conditional license.
to resume driving on a conditional license subject to at least
annual review if the previously effective medication regime is
resumed; and there have been no seizures for four weeks after
resuming the medication regime
No driving during the period in which the dose reduction is being If any anti-epileptic medication is to be withdrawn, the person will
made; and for 3 months after completion of the dose reduction. no longer meet the criteria to hold a conditional license.
Reduction in dosage of anti-epileptic medication in a person who
satisfies the standard to hold a conditional license
Driving may continue if the dose reduction is due only to the Driving may continue if the dose reduction is due only to the
presence of side-effects. presence of side-effects.

This patient fulfills none of the above-mentioned condition; therefore, general rule applies for him: he should not drive non-commercial vehicles for 12 months.

References

• AustRoads - Fitness to Drive

Last updated:
Time spent: QID:727 2023-2-12

1677 of 1943
An 85-year-old man presents to your clinic for annual check to renew his driver's license. He has long-standing history of hypertension which is well-controlled on antihypertensive medications. Which one of the following is the investigation you
should conduct before you issue a certificate?

A. Visual acuity.

B. Mini-mental status exam.

C. Blood sugar.

D. Cholesterol.

E. Liver function tests.

Correct
45% answered correctly

Explanation:

Correct Answer Is A

Individuals, who have blood pressure consistently greater than 200 systolic or greater than 110 diastolic (treated or untreated), are not fit to hold an unconditional driving license for non-commercial vehicles. The threshold for commercial vehicles is
170 mmHg and 100 mmHg for systolic and diastolic blood pressure, respectively.

A conditional license may be considered by the driver licensing authority subject to periodic review, taking into account the nature of the driving task and information provided by the treating doctor as to whether the following criteria are met:

Blood pressure is well controlled; AND


There are no side effects from the medication that will impair safe driving; AND
There is no evidence of damage to target organs relevant to driving

As far as driving is concerned, eyes are the most important end-organ potentially affected by chronic hypertension. Those with hypertension are at risk of hypertensive retinopathy and impaired vision. For that reason, examining the visual acuity will
be the most crucial investigation before a certificate is issued for driving license renewal in this patient.

References

• Austroads - Assessing Fitness to Drive

Last updated:
Time spent: QID:734 2023-2-12

1678 of 1943
Which one of the following is the best predictor of obesity and its associated risks?

A. Body mass index (BMI).

B. Waist circumference.

C. Mid arm circumference.

D. Waist-to-hip ratio.

E. BMI and waist circumference together.

Incorrect. Correct answer is D


45% answered correctly

Explanation:

Correct Answer Is D

BMI is advocated by World Health Organization (WHO) as the epidemiological measure of obesity; nevertheless, BMI is a crude index that does not take into account the distribution of body fat, resulting in variability in different individuals and
populations. For example, individuals with the same BMI may have different ratios of body fat to lean mass. A muscular athlete may have the same BMI of a less muscular person. Women have more body fat than men at equal BMIs and people lose
lean tissue with age so an older person will have more body fat than a younger one with same BMI.

Waist circumference has been recommended as a simple and practical measure for indentifying overweight and obese patients, but it does not take into account body size and height.

Waist-to-hip ratio (WHR) has been suggested as the preferred measure of obesity for predicting cardiovascular disease, with more universal application in individuals and population groups of different body builds. This parameter reflects abdominal
(central) fat which is strongly associated with ischemic heart disease, hypertension and type II diabetes mellitus. In terms of predicting obesity-related mortality, WHR is more reliable than BMI and waist circumference together. Waist circumference
alone comes next and BMI alone last.

References

• https://www.mja.com.au/journal/2003/179/11/waist-h

• https://www.nhmrc.gov.au/_files_nhmrc/publications

Last updated:
Time spent: QID:756 2023-2-12

1679 of 1943
You are asked to give advice regarding breast cancer to a 37-year-old woman who has come to your clinic with concerns about the disease. Her mother was diagnosed with breast cancer at the age of 60 years. She is asymptomatic and her clinical
examination is normal. Which one of the following is the next best step in management?

A. She should start mammography now and every two years until the age of 74 years.

B. She should start ultrasonography now and every two years until the age of 74 years.

C. She should perform six-monthly self-breast examination.

D. She should start mammography now and then yearly until the age of 74 years.

E. Reassure her.

Incorrect. Correct answer is E


45% answered correctly

Explanation:

Correct Answer Is E

Woman with family history of breast cancer in one first-degree relative older than 50 years are considered to be at slightly elevated risk above the normal population for breast cancer. For these women, two-yearly screening mammography starting
from the age of 50 is the currently recommended screening; therefore, reassurance for now would be the most appropriate action. This woman however, should be advised that she should start mammography from the age of 50 years.

In general population, breast cancer screening is aimed at asymptomatic women aged 50-69 years; however, all women between 40 and 74 are eligible to enter the program if they wish.

Mammography is not recommended for women younger than 40 years due to dense breast tissue. Other modalities such as ultrasound or MRI can be used as alternatives if indicated.

References

• RACGP - The Red Book - Breast Cancer

• Cancer Council - Breast Cancer

Last updated:
Time spent: QID:808 2023-2-12

1680 of 1943
Julian, 35 years old, presents to your office for breast cancer screening after she found out that one of her maternal aunts was diagnosed with breast and ovarian cancer at the age of 40 years. Which one of the following would be the most
appropriate advice for her?

A. Two-yearly mammography.

B. Two-yearly ultrasound.

C. Genetic risk screening.

D. Referral for BRCA gene screening.

E. Six-monthly self-breast examination.

Incorrect. Correct answer is C


45% answered correctly

Explanation:

Correct Answer Is C

The following groups are at increased risk of breast and/or ovarian cancer due to a gene mutation (mostly BRCA1 and BRCA2):

Multiple relatives affected by breast (male or female) or ovarian cancer


Young age at cancer diagnosis in relatives
Relatives affected by both breast and ovarian cancer
Relatives affected with bilateral breast cancer
Ashkenazi Jewish ancestry

Of all breast cancers, 5-10% are caused by inherited genetic mutation. BRCA1 and BRCA2 mutation are the most important causes for hereditarily increased risk of breast/ovarian cancer. It is recommended that primary care provider screen high-risk
woman with screening tools designed to identify a family history that may be associated with an increased risk for potentially harmful mutations in breast cancer susceptibility genes (BRCA1, BRCA2). These screening tools are questionnaires used
by the health provider to take into account the family history of the person, and estimating the likelihood of developing breast and/or ovarian cancer in future, or whether they are high risk for having a faulty gene mutation.

FRA-BOC is one of the most commonly used screening tools in Australia for this purpose. FRA-BOC is available from: https://canceraustralia.gov.au/clinical-best-practice/gynaecological-cancers/fra-boc/evaluate

Women with positive screening results should receive genetic counselling and, if indicated after counselling, BRCA testing. For Julian, genetic risk assessment using screening tools such as FRA-BOC is the most appropriate next step. Referral for
pretest counselling and genetic testing is an appropriate option once she is found to be at high risk.

References

• Cancer Australia - Advice about familial aspects of breast cancer and epithelial ovarian cancer

• Cancer Australia - Genetic testing for breast/ovarian cancer risk

• Australian Prescriber - BRCA testing for familial breast cancer

Last updated:
Time spent: QID:809 2023-2-12

1681 of 1943
You are working in a rural area. A 4-year-old Somali boy is presented to you by his mother for polio vaccine. He has received 3 doses of oral polio vaccine (OPV) at 2, 4 and 6 months
of age back at his country with the last dose being given approximately 3 years ago. You only have injectable polio vaccine available in your office. Which one of the following would
be the best appropriate management?

A. Check his immune status.

B. No further vaccination is needed.

C. Refer him to another clinic.

D. Give the injectable polio vaccine.

E. Try to find oral polio vaccine for him.

Incorrect. Correct answer is D


45% answered correctly

Explanation:

Correct Answer Is D

Injectable Inactivated polio vaccine (IPV) is the polio vaccine currently in use in Australia, and is given intramuscularly. Oral polio vaccine (OPV) is no longer in use in Australia. OPV
and IPV are interchangeable. Children, who have been started on OPV should complete their polio vaccination schedule using IPV (IPOL®) or IPV-containing vaccines.

IPV (IPOL) or IPV-containing vaccines are recommended for infants at 2, 4 and 6 months of age. The 1st dose of an IPV-containing vaccine can be given as early as 6 weeks of age.
If the 1st dose is given at 6 weeks of age, the next scheduled doses should still be given at 4 months and 6 months of age.A booster dose of IPV (IPOL®) or IPV-containing vaccine
is recommended at 4 years of age. This is commonly provided as DTPa-IPV, which can be given as early as 3.5 years, but if DTPa-IVP is not available, IPV alone is used.

The only absolute contraindications to IPV (IPOL®) or IPV-containing vaccines are:

Anaphylaxis following a previous dose of any IPV-containing vaccine


Anaphylaxis following any vaccine component

References

• http://www.health.gov.au/internet/immunise/publish

Last updated:
Time spent: QID:143
2023-2-12

1682 of 1943
A 30-year-old woman is concerned about breast cancer because her mother was diagnosed with breast cancer at the age of 48 years. Clinical examination of the breast is normal. Which one of the following would be the most appropriate advice for
her?

A. Mammography yealy from the age of 40 years.

B. Mammography every 2 years from 50 to 74 years.

C. Refer to a surgeon.

D. Refer for genetic studies.

E. Reassure her that she is not at increased risk of breast cancer.

Correct
45% answered correctly

Explanation:

Correct Answer Is A

With a first-degree relative affected by breast cancer diagnosed before the age of 50, this woman is considered as having moderate risk for breast cancer. For such patients yearly screening mammography starting from the age of 40 is the best
advice to give.

(Option B) Mammography every 2 years is the most appropriate recommendation for general population and those with just slightly increased risk above average for breast cancer.

(Option C) Referral to a surgeon would have been indicated if a diagnosis of breast cancer was established.

(Option D) Referral for genetic testing is an acceptable step for those women who are likely to have a predisposition for familial forms of breast/ovarian cancer. Referral for genetic testing should be considered once screening tools raise suspicion
against strong risks of such cancers.

(Option E) Although this woman is not in high-risk category and is categorized as having moderate risk, reassuring is not appropriate because she needs a more extensive surveillance program for breast cancer than general population.

References

• RACGP - The Red Book - Breast Cancer

Last updated:
Time spent: QID:810 2023-2-12

1683 of 1943
A 67-year-old woman presents to your clinic for advice regarding reducing the risk of shingles infection after her husband was diagnosed with ophthalmic shingles 3 days ago. He had a vesicular rash on his face involving the eye, for which he was
referred to an ophthalmologist for consultation. She wants to know if she should be vaccinated. She clearly remembers that she had chicken pox at the age of 12 years. Which one of the following is the most appropriate advice for her regarding
herpes zoster prophylaxis?

A. Vaccination at this age will not benefit her.

B. Vaccination is indicated if she develops clinical shingles.

C. Vaccination will reduce her risk of infection regardless of her previous exposure.

D. A varicella serology testing is required for further decision making.

E. With the childhood history of chicken pox, she will not benefit from vaccination.

Incorrect. Correct answer is C


45% answered correctly

Explanation:

Correct Answer Is C

Varicella-zoster virus (VZV) is a DNA virus and a member of the herpes virus family. Primary infection with VZV is known as varicella or ‘chickenpox’. Herpes zoster (HZ), or ‘shingles’, is caused by reactivation of latent VZV, which typically resides
dormant in the dorsal root or trigeminal nerve ganglia following primary infection. Cranial nerves most commonly involved are 5th cranial nerve (CN V) and 7th cranial nerve (CN VII) resulting in herpes ophthalmicus and Ramsay-Hunt syndrome,
respectively.

Reactivation of VZV causing HZ is thought to be particularly due to a decline in cellular immunity to the virus, and presents clinically as a unilateral vesicular rash in a dermatomal distribution in most cases. A prodromal phase occurs 48 to 72 hours
prior to the appearance of the lesions in 80% of patients. Prodromal symptoms may include headache, photophobia, malaise, and an itching, tingling or severe pain in the affected dermatome. In most patients, HZ is an acute and self-limiting disease,
with the rash lasting 10 to 15 days. However, complications can occur, especially in older adults.

Post-herpetic neuralgia (PHN), the most frequently faced and debilitating complication of HZ, is a neuropathic pain syndrome that persists or develops after the dermatomal rash has resolved. By definition, PHN is established when pain persists for
longer than 3 months after the onset of the rash.

Other complications may occur, depending on the site of reactivation. These include ophthalmic disease (such as keratitis and chorioretinitis), neurological complications (e.g. meningoencephalitis and myelitis), secondary bacterial skin infection,
scarring and pneumonia. Rarely, disseminated HZ may develop, with widespread vesicular rash, and visceral, central nervous system and pulmonary involvement. Disseminated disease is more common in people who are immunocompromised.
Dermatomal pain without the appearance of rash is also documented (zoster siné herpéte).

Zostavax® is a live attenuated vaccine formulated from the same VZV vaccine strain (Oka/Merck) as the registered varicella (chickenpox) vaccine Varivax®, but is much more potent (on average, at least 14 times). This higher potency is owed to
higher viral titer in Zostavax, and is required to elicit a boost in immune response of adults who usually remain seropositive to VZV following primary infection, but have declining cellular immunity with increasing age.

Zostavax® is used for the prevention of HZ in people>50 years of age. It is important to note that the registered varicella vaccines are not indicated for use in preventing HZ in older people and Zostavax is not indicated for use in younger people who
have not been previously immunized or infected with VZV. Zostavax® is used solely for prophylaxis in a potentially-exposed asymptomatic patient. It has no therapeutic effect on established HZ infection or PHN; therefore, not indicated once
infection develops. However, if given prophylactically, it can reduces the risk of HZ and PHN.

(Option A) A single dose of Zostavax® is recommended for adults ≥60 years of age who have not previously received a dose of it regardless of their childhood VZ vaccination status. Routine serological testing prior to vaccination is not indicated.
Zostavax® is given regardless of a history of previous varicella (chickenpox) infection; therefore, inquiring about previous chicken pox infection is not required.

Routine population-based use of zoster vaccine in persons aged 50–59 years is not recommended. Although the incidence of HZ in persons 50–59 years of age is higher than in younger age groups, and zoster vaccine seems effective in those 50–
59 years of age, the likelihood of developing PHN and other complications of HZ is lower in this age group than in those ≥60 years of age. People aged 50–59 years who wish to protect themselves against HZ can be vaccinated; however, the duration
of efficacy, and need for a booster dose at a later age, is not yet determined. The routine use of vaccine is not recommended for those younger than 50 years.

References

• Medical Journal of Australia - The prevention and management of herpes zoster

• Australian Immunization Handbook

Last updated:
Time spent: QID:821 2023-2-12

1684 of 1943
A 29-year-old woman presents to your clinic at 14 weeks pregnancy for consultation regarding smoking cessation. She tried to give up smoking when she found out she is pregnant but she failed. She is worried about her baby’s wellbeing and wants
to quit smoking. Which one of the following is the most appropriate management option for her?

A. Nicotine replacement therapy.

B. Non-nicotinic chewing gums.

C. Bupropion.

D. Varenicline.

E. Advise that she smoke less cigarettes.

Correct
45% answered correctly

Explanation:

Correct Answer Is A

Smoking in pregnancy is the most important preventable cause of a wide range of adverse pregnancy outcomes. Smoking causes obstetric and fetal complications. Furthermore, there is growing evidence that harms can extend into childhood and
even adulthood. Unfortunately, most smokers who become pregnant continue to smoke, and most of those who quite relapse after delivery.

Pregnancy is a window of opportunity for health professionals to help smokers quit. Women are motivated to protect their baby’s health, and quitting smoking during pregnancy reduces the risk of complications.

Interventions during pregnancy are modestly effective and assist 6 in every 100 smokers to quit. Many light smokers quit without assistance when they find out they are pregnant. The remaining smokers may need more intensive treatment.

The Australian Smoking Cessation guidelines recommend that pregnant smokers first try to quit with behavioral counselling and support as the first-line treatment. Smoking should be addressed at every GP visit during pregnancy in view of its
serious health impact. Counselling in pregnancy produces a 4%-6% increase in the quit rate, compared with no counselling.

Nicotine Replacement Therapy (NRT) should then be considered if the patient is unable to succeed without it. NRT should be used under the supervision of a qualified health professional.

Intermittent, short-acting forms of NRT such as the lozenge or mouth spray are recommended to deliver a lower total daily nicotine dose. However, this may result in under-dosing and reduced effectiveness. The guidelines also advise that if patches
are used they should be removed at bedtime.

Although guidelines recommend the smallest effective dose of nicotine, larger doses or even combination therapy may be required. Adequate doses to relieve cravings and withdrawal symptoms, and a full course of at least 8 weeks treatment are
supported. The risks and benefits of NRT during pregnancy should be explained without making the patient unduly concerned.

There is no evidence of increased rates of miscarriage, stillbirth, admission to neonatal intensive care unit (NICU) or neonatal death between NRT and control groups. There is currently insufficient evidence to determine whether NRT is safe in
pregnancy, but available data and expert opinion suggest it is less harmful than continuing to smoke.

This patient should be started on counselling and support (not an option). NRT should be considered if the patient requires it.

Australian Smoking Cessation Guidelines recommend against the two prescription medicines for smoking cessation, varenicline and bupropion during pregnancy and breastfeeding.

NOTE - Quitting smoking before conception or in the first trimester results in similar rates of adverse pregnancy outcomes compared with non-smokers; however, quitting at any time during produces health benefits. Quitting before pregnancy
also allows the use of full range of pharmacotherapies.

References

• RACGP - AFP - Management of smoking in pregnant women

Last updated:
Time spent: QID:823 2023-2-12

1685 of 1943
A 72-year-man presents to your practice for assessment. He does not smoke and has no history of diabetes mellitus or cardiovascular disease. On examination, he has a blood pressure of 172/100 mmHg, confirmed with a second reading. On blood
tests, he has a total cholesterol level of 6.2 and HDL of 1mmol/L. According to the following chart which one of the following is his absolute cardiovascular risk?

A. 10-15%.

B. 16-19%.

C. 20-24%.

D. 25-29%.

E. ≥30%.

Incorrect. Correct answer is D


45% answered correctly

Explanation:

Correct Answer Is D

Assessment of cardiovascular disease (CVD) risk on the basis of the combined effect of multiple risk factors (absolute CVD risk) is more accurate than the use of individual risk factors because the cumulative effects of multiple risk factors may be
additive or synergistic. In Australia, 64% of the adult population have 3 or more modifiable risk factors. Since CVD is largely preventable, an approach focusing on comprehensive risk assessment will enable effective management of identified
modifiable risk factors through lifestyle changes and, where needed, pharmacological therapy.

Absolute risk is the numerical probability of a CVD event occurring within 5 years expressed as a percentage. For example, if one patient’s risk is 15%, he/she could be told he/she has a 15% chance of having a CVD event within the next 5 years.

In Australia, the following charts are used for calculation of absolute CVD risk in any given patient. The risk percentages are extracted from Framingham Risk Equation (FRE). It takes into account the following variables for calculation of absolute CVD
risk:

Age
Gender
Smoking
Systolic blood pressure
Total cholesterol to HDL ratio

NOTE - Different rules apply to diabetic versus nondiabetic individuals and Aboriginal and Torres Strait Islanders.

These charts are used as follows:

Identify the chart relating to the person’s sex, diabetes status, smoking history and age. The charts should be used for all adults aged 45 years or over (and all Aboriginal and Torres Strait Islander adults aged 35 - 74 years) without known history of
CVD, and not already known to be at clinically determined high risk. On the chart, choose the cell nearest to the person’s age, systolic blood pressure (SBP) and total cholesterol (TC):HDL ratio. The color of the cell that can be the person falls into
provides their 5 year absolute cardiovascular risk level. People who fall exactly on a threshold between cells are placed in the cell indicating higher risk. For those older than 74 years, calculate as if they are 74.

This patient is male, non-smoker and 72 years. The part of the chart that should be used for risk calculation is shown in the following picture:

1686 of 1943
When the box is determined, consider an imaginary vertical line for cholesterol-to-HDL ratio and a horizontal one for systolic blood pressure:

The colour of the box is assigned to a 25-29%


absolute cardiovascular risk.

NOTE – Absolute risk calculation should not be used for the following groups as they are already high risk for CVD:

Diabetes and age >60 years


Diabetes with microalbuminuria (>20 mcg/min or urinary albumin:creatinine ratio >2.5 mg/mmol for males, >3.5 mg/mmol for females)
Moderate or severe CKD (persistent proteinuria or estimated glomerular filtration rate (eGFR) <45 mL/min/1.73 m2 )
A previous diagnosis of familial hypercholesterolaemia • Systolic blood pressure ≥180 mmHg or diastolic blood pressure ≥110 mmHg
Serum total cholesterol >7.5 mmol/L
A previous diagnosis of familial hypercholesterolemia
Systolic blood pressure ≥180 mmHg or diastolic blood pressure ≥110 mmHg
Serum total cholesterol >7.5 mmol/L
Aboriginal and Torres Strait Islander adults aged over 74

References

• https://heartfoundation.org.au/images/uploads/publ

• https://heartfoundation.org.au/images/uploads/publ

Time spent: QID:845 Last updated:


2023-2-12

1687 of 1943
Hypertension is the most important preventable risk factor for which one of the following conditions?

A. Coronary artery disease.

B. Peripheral arterial disease.

C. Renal arterial disease.

D. Ischemic stroke.

E. Atrial fibrillation.

Incorrect. Correct answer is D


45% answered correctly

Explanation:

Correct Answer Is D

Although hypertension is a risk factor for all of the given to some extent, it remains the most important modifiable risk factor for stroke and intracerebral hemorrhage (ICH). The risk of stroke increases progressively with increasing blood pressure,
independent of other factors. Hypertension is the cause of stroke in >50% cases worldwide. Most estimates for hypertension indicate a relative risk of approximately 4 when hypertension is defined as systolic blood pressure ≥160mmHg and/or
diastolic blood pressure ≥95mmHg.

Hypertension and hypertensive heart disease results in development of left ventricular hypertrophy and diastolic dysfunction that are important risk factors for atrial fibrillation accounting for 20% of cases.

Hypertension is also a major risk factor for coronary artery disease (CAD), and treatment of hypertension reduces the risk of a coronary artery disease by one-third; however, the most important reversible risk factor for CAD is hypercholesterolemia
and cigarette smoking. Cessation of cigarette smoking is the single most important preventive measure for coronary artery disease. Moreover, smoking has the highest association with peripheral arterial disease.

References

• http://stroke.ahajournals.org/content/28/7/1507.fu

Last updated:
Time spent: QID:849 2023-2-12

1688 of 1943
Which one of the following risk factors is most signficant for ischemic stroke?

A. Hypercholesterolemia.

B. Smoking.

C. Obesity.

D. Type 2 diabetes mellitus.

E. Hypertension.

Incorrect. Correct answer is E


45% answered correctly

Explanation:

Correct Answer Is E

All of the given options can be potential risk factors for ischemic stroke; however, hypertension is associated with the highest risk. Hypertension is the cause of stroke in >50% cases worldwide. Most estimates for hypertension indicate a relative risk
of approximately 4 when hypertension is defined as systolic blood pressure ≥160mmHg and/or diastolic blood pressure ≥95mmHg.

The following table compares the relative risk of different preventable risk factors for stroke:

Risk factor Relative risk


Hypertension ~4
Diabetes mellitus 1.8-3
Cigarette smoking 2
Lifestyle factors (obesity, inactivity, unhealthy diet) 1.5-2

NOTE - Age is the single most important risk factor for stroke. For each successive 10 years after age 55, the stroke rate more than doubles in both men and women. Stroke incidence rates are 1.25 times greater in men, but because women tend to
live longer than men, more women than men die of stroke each year. Obesity is associated with elevated vascular risk in population studies. Furthermore, it has been associated with glucose intolerance, insulin resistance, hypertension, physical
inactivity and dyslipidaemia.

(Option A) Hypercholesterolemia seems to act in a complex fashion and does not directly act as a risk factor; however, it is an important modifiable risk factor for coronary artery disease.

(Options B and C) Smoking and obesity are associated with relative risks of 1.5 and 1.5-2.0 respectively.

(Option D) Diabetes mellitus is associated with a relative risk of 1.8-3.0.

References

• http://stroke.ahajournals.org/content/28/7/1507.fu

Last updated:
Time spent: QID:850 2023-2-12

1689 of 1943
A 46-year-old man presents to your clinic for a routine check-up. He runs a butchery, and is an smoker. Sometimes, he also uses cocaine and cannabis at parties. On examination, he has a blood pressure of 135/95 mmHg. His BMI is 29. Which one of
the following is the most significant risk factor for developing coronary artery disease in this patient?

A. Obesity.

B. Cannabis.

C. Age.

D. Cocaine.

E. Cigarette smoking.

Incorrect. Correct answer is E


45% answered correctly

Explanation:

Correct Answer Is E

Major traditional risk factors for cardiovascular diseases are:

Hypercholesterolemia (high LDL)


Smoking
Hypertension
Diabetes mellitus

Other traditional risk factors include:

Family history of premature coronary artery disease in a first-degree relative (female<65 years; male<55 years)
Low physical activity
Obesity

Of the options, smoking is a major risk factor for coronary artery disease (CAD). People who smoke more than 20 cigarettes a day have a 2- to 3-fold increase in total incidence of heart disease. Continued smoking is a major risk factor for recurrent
heart attack. Cessation of cigarette smoking is the single most important preventive measure for CAD.

Although family history of premature heart disease (male<55 years and female<65 years), low level of physical activity, and obesity are considered risk factors for CAD, these factors work through major risk factors to a large extent.

Age, by itself, is not a risk factor unless other major risk factors are present. The incidence of CAD increases after the age of 45 years in men and 55 in women.

References

• http://circ.ahajournals.org/content/97/18/1837.ful

Last updated:
Time spent: QID:851 2023-2-12

1690 of 1943
A 60 year-old man presents to your clinic for a routine medical check-up. He smokes 45 cigarettes a day, and drinks 10 standard drinks daily. He has a body mass index (BMI) of 32 kg/m2.He wears seat-belt almost 60% of the times when he drives.
He is motivated to follow lifestyle modification advice. Which one of the following is the most significant advice for prevention of cardiovascular disease and premature death in this patient?

A. Smoking cessation.

B. Reduction of alcohol intake.

C. Wearing seatbelt all the time when he drives.

D. Weight reduction.

E. Regular exercise.

Correct
45% answered correctly

Explanation:

Correct Answer Is A

By far, the most common cause of coronary artery disease (CAD) is atherosclerotic changes of coronary arteries. The most common underlying cause is hypercholestrolemia (high LDL). Smoking is also strongly associated with increased risk of
cardiovascular diseases, and its cessation is considered the most effective life style modification to prevent cardiovascular events (more than lipid modification). In a patient with coronary artery disease, quitting smoking is associated with a 36%
reduction in mortality.

Although family history of premature heart disease (male<65 and female<55), low level of physical activity and obesity are considered risk factors for coronary artery disease, these factors exert their effects in the presence of major risk factors.

(Option B) This patient should also be advised to reduce alcohol consumption to a maximum of 2 standard drinks (one for women) with 2 alcohol-free days every week. This measure is beneficial for general health but does not have a specific effect
on cardiovascular status as does smoking cessation.

(Option C) Wearing a properly-adjusted seatbelt reduces the risk of mortality by 50%; however this reduction is related to trauma and severe injuries, not to cardiovascular diseases.

(Option D) Blood pressure and body weight are directly related. For every 1kg reduction in weight, the systolic blood pressure will drop by 1mmHg. Weight reduction is particularly beneficial if the patient has insulin resistance and/or hyperlipidemia.
Weight reduction, however, is not as effective as smoking cessation in improving cardiovascular risk.

(Option E) Regular exercise increases high-density lipoprotein cholesterol and reduces body weight, and contribute to improved cardiovascular status, but not as smoking cessation does.

TOPIC REVIEW

Major traditional risk factors for coronary artery disease (CAD) are as follow:

Hypercholesterolemia (high LDL)


Smoking (especially smoking cigarettes)
Hypertension
Diabetes mellitus

Other traditional risk factors include:

Family history of heart disease (premature)


Low level of physical activity
Obesity

References

• http://emedicine.medscape.com/article/164163-overv

• http://circ.ahajournals.org/content/97/18/1837.ful

Last updated:
Time spent: QID:852 2023-2-12

1691 of 1943
A 62-year-old man presents for 'absolute cardiovascular risk' assessment. His past medical history includes type 2 diabetes mellitus for which he is on metformin 500 mg 12-hourly and gliclizide 60 mg daily. Laboratory studies show an
albumin/creatinine ratio of 5mg/mmol. Which one of the following options is correct regarding this scenario?

A. Inform him that he is already in high risk group for cardiovascular disease and do not need risk calculation.

B. Perform absolute risk calculation for him now and 2-yearly.

C. Reassure him.

D. Start him on insulin.

E. Increase the dose of his medications.

Correct
45% answered correctly

Explanation:

Correct Answer Is A

Assessment of cardiovascular disease risk on the basis of the combined effect of multiple risk factors (absolute CVD risk) is more accurate than the use of individual risk factors, because the cumulative effects of multiple risk factors may be
additive or synergistic. In Australia, 64% of the adult population have 3 or more modifiable risk factors. As CVD is largely preventable, an approach focusing on comprehensive risk assessment will enable effective management of identified
modifiable risk factors through lifestyle changes and, where needed, pharmacological therapy.

Absolute cardiovascular risk assessment should be conducted at least every 2 years in all adults aged 45 years or older who are not known to have CVD or to be at clinically-determined high risk. Calculation of the absolute risk requires information
on the patient’s age, sex, smoking status, total and HDL cholesterol, systolic blood pressure and if the patient is known to have diabetes or left ventricular hypertrophy.

Absolute risk calculation is not indicated if:

The patient has current or history of cardiovascular disease e.g. stroke, transient ischemic attack (TIA), ischemic heart disease (IHD), or peripheral arterial disease (PAD).

The patient is already clinically-determined high risk:

Diabetes and age >60 years


Diabetes with microalbuminuria (>20 µg/min or urine albumin:creatinine ratio (UACR) >2.5 mg/mmol for males, >3.5 mg/mmol for females)
Moderate or severe CKD (persistent proteinuria or estimated glomerular filtration rate (eGFR) <45 mL/min/1.73 m2)
Previous diagnosis of familial hypercholesterolaemia (FH)
SBP ≥180 mmHg or diastolic blood pressure (DBP) ≥110 mmHg
Serum total cholesterol >7.5 mmol/L
Aboriginal or Torres Strait Islander peoples aged over 74 years

With diabetes and age of 62 (>60) on one hand and an albumin/creatinine ratio of 5mg/mmol (>2.5mg/mmol), this patient has 2 conditions that categorizes him as clinically-determined high risk; therefore, risk calculation is not required.

(Option B) He does not need absolute cardiovascular risk assessment now or any other time because he is already clinically-determined high risk.

(Option C) Reassurance of a patient who high-risk for CVD is not wise.

(Options D and E) There are no pointers in the scenario towards inadequacy of diabetes treatment; hence, increasing the dose of the current medications or commencement of insulin appears irrelevant.

References

• http://heartfoundation.org.au/images/uploads/publi

• http://www.racgp.org.au/your-practice/guidelines/r

Last updated:
Time spent: QID:855 2023-2-12

1692 of 1943
A 47-year-old man presents for cholesterol check. He smokes 20 cigarettes a day. On examination, he has a blood pressure of 140/90 mmHg. Among the blood tests performed, a cholesterol level of 5.5 mmol/L is remarkable. Which one of the
following would be the most appropriate advice for him at this stage?

A. He should start statins.

B. He should start antihypertensive medications.

C. He should be involved in a smoking cessation program.

D. His cholesterol level should be checked in 3 months.

E. He should start regular exercise and diet.

Incorrect. Correct answer is C


45% answered correctly

Explanation:

Correct Answer Is C

By far, smoking is the single most modifiable risk factor for coronary artery diseases (CAD). Smoking is associated with a 2- o 3-fold increase in CAD. Smoking cessation alone reduces the risk of CAD by 32%; therefore, advice against smoking and
encouraging the patient to actively participate in a smoking cessation program will be the most appropriate step.

Adopting a healthy lifestyle including regular exercise and diet is also important for reduction in CAD risk but smoking remains the most important factor.

The patient’s blood pressure is at the cut-off point for hypertension. Presently, antihypertensive medications are not indicated.

The target cholesterol level is <4mmol/L. With a cholesterol level of 5.5 mmol/L, this patient should initially be advised to start regular exercise and a healthy diet. Statins as first-line medications for hypercholesterolaemia are indicated if diet and
exercise fail to lower the cholesterol levels in 3-6 months.

References

• http://heartfoundation.org.au/images/uploads/publi

• http://www.australiandoctor.com.au/cmspages/getfil

Last updated:
Time spent: QID:859 2023-2-12

1693 of 1943
Which one of the following measures is of most importance in reducing the prevalence of trachoma infection in a susceptible community such as indigenous Australians?

A. Childhood vaccination.

B. Prophylactic use of tetracycline.

C. Avoidance of community bathing.

D. Regular hand and face washing.

E. Eradication of mosquitoes.

Incorrect. Correct answer is D


45% answered correctly

Explanation:

Correct Answer Is D

To reduce the prevalence of trachoma infection, the strategy defined and implemented by the World Health Organization (WHO) should be followed. This strategy is called SAFE that
stands for surgery, antibiotics, facial hygiene (cleanliness) and environmental factors.

Surgery is used to correct the entropion before it leads to permanent damage to the cornea.
Azithromycin is the antibiotic of choice in this strategy and is given, in oral form, to family members with active trachoma.
Facial hygiene is achieved through regular hand and face washing.
Environmental factors should be addressed by improving water supply, better community hygiene and dust and fly control.

Among the following measures surgery and antibiotics are used for treatment not prevention; hence they have no effect on prevalence. The most important measure, which with
proper education will significantly reduce the prevalence of the disease, is regular hand and face washing.

(Option A) There is no effective vaccine against Chlamydia trachomatis.

(Optipn B) Tetracycline is not used for prophylaxis or treatment of Chlamydia. Azithromycin is the drug of choice for such purposes.

(Option C) Chlamydia trachomatis is spread via direct contact e.g. from mother to the child. Avoidance of community bathing does not seem to have a significant impact on the
prevalence.

(Option E) Trachoma is transmitted by flies not mosquitoes; therefore, eradication of mosquitoes would not be an effective measure.

References

• AMC Handbook of Multiple Choice Questions – page 623

• 624

Last updated:
Time spent: QID:297
2023-2-12

1694 of 1943
A 37-year-old man is about to travel to Central Africa and has presented for advice regarding prevention of yellow fever. Which one of the following options is the most appropriate advice for him?

A. He should avoid swimming or wading in fresh water.

B. He should use bottled or boiled water.

C. He should be careful with fecal-oral hygiene.

D. He should use yellow fever vaccine.

E. He should receive prophylactic doxycycline.

Incorrect. Correct answer is D


45% answered correctly

Explanation:

Correct Answer Is D

Yellow fever is caused by a flavivirus. The virus is transmitted to humans from the bite of insect mosquito vectors such as Aedes Egypti. The reservoir for the virus is the monkey population of tropical rainforests of South and Central America and
West and South Africa. Travellers to these areas are prone to the disease. Yellow fever presents with rigors and fever, jaundice, petechiae, hematemesis and circulatory failure. Treatment is almost supportive and symptomatic.

Stamaril ® is the live, attenuated yellow fever virus (17D-204 strain) freeze-dried vaccine available in Australia. It is given as a single intramuscular or subcutaneous injection of 0.5mL of reconstituted vaccine. Stamaril ® is almost 100% effective
against yellow fever and should be advised as the most important preventive means for those travelling to endemic areas. Protective immunity occurs within 1–2 weeks in 90–95% of people, and a single dose lasts at least 10 years. Under
international health regulations, revaccination is required every 10 years.

NOTE - Stamaril ® is contraindicated in patients with history of egg allergy or previous anaphylaxis to the vaccine. It is also contraindicated in pregnancy. Pregnant woman should be advised against travelling to endemic areas; however, if the travel is
inevitable, the benefits of vaccination outweigh the risks.

Other protective measures include staying indoors or under mosquito nets to avoid contributing to the transmission cycle. Advice should be given regarding avoidance of epidemics and minimization of high risk behavior (e.g. jungle visits). Personal
protective measures are an important part of any travel medicine consultation where the traveller is heading to any region of mosquito-borne disease. Mosquito avoidance includes the use of treated mosquito nets and screening of living and work
areas.

References

• http://www.racgp.org.au/download/documents/AFP/201

Last updated:
Time spent: QID:896 2023-2-12

1695 of 1943
A previously healthy 2-year-old child is brought to your practice with signs and symptoms of meningitis. She has been fully immunized with pneumococcal conjugated vaccine. You refer the child to the hospital, where she is found to have
pneumococcal meningitis. Which one of the following is the most likely cause of the infection despite immunization?

A. An underlying primary immune deficiency.

B. A pneumococcal serotype that the vaccine did not protect against.

C. Failure of the vaccine to elicit a protective immune response against a pneumococcal serotype contained in the vaccine.

D. A cardiac defect with right-to-left shunt.

E. Contamination of the specimens used for testing in the hospital.

Incorrect. Correct answer is B


45% answered correctly

Explanation:

Correct Answer Is B

Streptococcus pneumoniae (Pneumococcus) is a Gram-positive coccus. Capsule polysaccharide is the most important virulence factor of pneumococci. Over 90 capsular antigen types (serotypes) have been recognized, each of which elicits type-
specific immunity in the host. The natural reservoir of pneumococci is the mucosal surface of the upper respiratory tract of humans. Different pneumococcal serotypes vary in their propensity to cause nasopharyngeal colonization or the disease.

Worldwide, only a limited number of serotypes are responsible for most cases of invasive pneumococcal disease (IPD), but the predominant serotypes vary by age group and geographic area.

The introduction of 7-valent pneumococcal conjugate vaccine (7vPCV) in Australia has led to a dramatic reduction in the overall incident of IPD. However, since the implementation of the universal 7vPCV program, increased rates of IPD caused by
serotypes not contained in 7vPCV (replacement disease) have been observed in Australia and several other countries. This is particularly so common among non-Indigenous children aged<2 years, and is the most likely cause to IPD in immunized
children.

This child has been diagnosed with meningitis as an IPD despite a full course of vaccination against pneumococci; therefore, the most likely explanation is infection with a specific serotype which has not been covered by 7vPCV.

(Option A) An underlying immune deficiency is also a possible explanation; however, this is unlikely in a previously healthy child.

(Option C) Failure of the vaccine to elicit an immune response in a healthy patient with intact immune system is a very remote possibility.

(Option D) A cardiac defect with right-to-left shunt does not explain the event.

(Option E) Sample contimantion would have caused growth of different types of organisms on culture. With growth of streptococcus in isolation, such an event is not likely.

References

• http://www.immunise.health.gov.au/internet/immunis

Last updated:
Time spent: QID:911 2023-2-12

1696 of 1943
A mother brings her 15-month-old son to your office for vaccination. The child has not received any vaccines since birth because the mother had been strongly against vaccination until last week when she was convinced that immunization is in best
interest of her child. Now the mother is asking you how she should catch up. Which one of the following would be the most appropriate action now?

A. A full course of vaccination should be started now.

B. Vaccination should be started now with the vaccines scheduled for this age.

C. The vaccination should be postponed as there is no need for vaccination at this stage.

D. The family should be reported to Child Protection Service.

E. The child should receive both initial and catch-up doses at once right now.

Correct
45% answered correctly

Explanation:

Correct Answer Is A

A physician should take every opportunity to review one’s vaccination history and administer the appropriate vaccine(s). For those, who do not have a documented receipt of vaccines scheduled in the National Immunisation Program (NIP)
appropriate for their age, a catch-up schedule should be planned, documented and discussed with the person or their caretakers/parents. The number of doses to give and the intervals may differ according to the age at which the catch-up is
planned. The objective of catch-up vaccination is to complete a course of vaccination and provide optimal protection as quickly as possible. The assessment of vaccination status should be based on the schedule for the state or territory where the
person to be vaccinated is residing.

(Option B) Commencement of vaccination with vaccines appropriate for the child’s current age while previous doses have been missed will not be appropriate because adequate protection may not be achieved.

(Option C) Postponidng vaccination is not appropraite because this child should receive his vaccines right now.

(Option D) The decision regarding vaccination of children completely lies within parents hands. They can refuse vaccination of their chidlren as their legal right, and reporting the family to child protection services not appropriate.

(Option E) Receiving both initial and catch-up doses at the same time is not appropriate. This child shoud receive initial doses at this visit followed by catch-up doses according to the vaccination guidelines.

References

• http://www.immunise.health.gov.au/internet/immunis

• http://www.rch.org.au/genmed/clinical_resources/Ca

Last updated:
Time spent: QID:919 2023-2-12

1697 of 1943
In which one of the following infections, administration of immunoglobulin as prophylaxis is not useful?

A. Hepatitis B.

B. Hepatitis A.

C. Mumps.

D. Varicella.

E. Rubella.

Incorrect. Correct answer is C


45% answered correctly

Explanation:

Correct Answer Is C

Hepatitis B specific immunoglobulin (HBIG) is used for neonates born to mothers with hepatitis B infection and non-immune persons who are exposed through occupational exposure to the blood of unidentified persons, or to persons who are
chronically infected with hepatitis B or whose hepatitis status cannot be established in time.

Hepatitis A vaccination is recommended in preference to NHIG for post-exposure hepatitis A prophylaxis in persons ≥12 months of age who are immunocompetent. NHIG can be used when hepatitis A vaccine administration is contraindicated such
as in infants <12 months of age, or in persons who are immunocompromised and who might not mount a sufficient response following vaccination. NHIG contains sufficiently high levels of antibody against hepatitis A to prevent or ameliorate
infection in susceptible persons, if administered within 2 weeks of exposure.

Specific zoster immunoglobulin (ZIG) is highly efficacious. Normal high-titer zoster immunoglobulin is available from the Australian Red Cross Blood Service on a restricted basis for the prevention of varicella in high-risk subjects who report a
significant exposure to varicella or herpes zoster. ZIG has no proven use in the treatment of established varicella or zoster infection. ZIG must be given early in the incubation period (within 96 hours of exposure), but may have some efficacy if
administered as late as 10 days post exposure. ZIG is able to prevent or ameliorate varicella in infants <1 month of age, in children who are being treated with immunosuppressive therapy, and in pregnant women.

Administration of NHIG for rubella infection is not recommended and does not prevent infection in non-immune contacts; however it may prolong incubation period and marginally reduce fetal risk and maternal clinical symptoms. Its use may be
considered in rare situations (within 72 hours of exposure), or in outbreaks as a pre-exposure prophylactic for non-immune pregnant women in high risk occupations. Serological follow-up is required for up to 2 months.

NIHG has not proven effective in prevention or treatment of mumps.

TOPIC REVIEW

Immunoglobulin preparations are used to provide passive immunization, that is, the direct administration of antibodies to a non-immune person to provide immediate protection against infection or disease.

There are two types of immunoglobulin:

Normal human immunoglobulin (NHIG) - NHIG is derived from the pooled plasma of blood donors. It contains antibody to microbial agents that are prevalent in the general population.

Specific immunoglobulins - Specific immunoglobulin preparations are obtained from pooled blood donations from patients convalescing from the relevant infection, donors recently vaccinated with the relevant vaccine, or those who, on screening,
have been found to have sufficiently high antibody concentrations. These blood-derived specific immunoglobulins contain concentrations of antibody to an individual organism or toxin at a higher titer than would be present in NHIG.

Specific immunoglobulins available in Australia include:

Hepatitis B specific immunoglobulin


Rabies specific immunoglobulin
Varicella zoster specific immunoglobulin
Botulism antitoxin
Tetanus immunoglobulin
Respiratory syncytial virus monoclonal antibodies
Cytomegalovirus immunoglobulin

For infections or disease other than above, NHIG is used if indicated and there is proven efficacy. NHIG use for passive prophylaxis has proven effective for measles and hepatitis A if vaccination is contraindicated; however, in selected cases, NHIG is
used for poliomyelitis and rubella as well with less efficacy.

References

• http://www.kemh.health.wa.gov.au/development/manua

• https://www.health.qld.gov.au/cdcg/index/rubella.a

• https://www.blood.gov.au/NHIg

• http://www.immunise.health.gov.au/internet/immunis

• http://bestpractice.bmj.com/best-practice/monograp

Last updated:
Time spent: QID:939 2023-2-12

1698 of 1943
One of the nurses of the ward you are working in sustains a needle stick injury while she is setting up a new intravenous line for a patient. The patient is HBsAg negative. History reveals that the patient is unlikely to be within the window period. The
nurse is unsure of her hepatitis B immunization status. Which one of the following is the most appropriate next step in management?

A. No further follow up is required.

B. Give hepatitis B immunoglobulin and full course of hepatitis B vaccination.

C. Organize a full course of hepatitis B immunization.

D. Report the incidence to the infectious disease specialist.

E. Perform a blood test to check the HBsAg status in 3 months.

Incorrect. Correct answer is C


45% answered correctly

Explanation:

Correct Answer Is C

If the index case is HBsAg negative and unlikely to be within the window period, no post-exposure prophylaxis for the nurse was required if she had been immunized. There is no risk for the nurse to contract hepatatiis B from this exposure; however,
she should receive a full course of hepatitis B vaccine. This ideally should be started within 24 hours.

Another appropriate approach (not an option) is to check for HBV antibody titer of the nurse. If the titre was equal or greater than 10mIU/ml, no vaccination is required either.

(Option A) Although the nurse is not likely to become infected from this exposure, she should have follow-up for immunization due to uncertain immunization status.

(Option B) Since the index case is HBsAg negative and unlikely to be in window period, post-exposure prophylaxis is not required.

(Option D) It is important that the incident is reported to a team manager or supervisor, but reporting to an infectious disease specialist in not necessary.

(Option E) Checking for the nurse’s HBsAg status was indicated if the index case was HBsAg positive.

TOPIC REVIEW

Recommendations regarding post-occupational exposure prophylaxis:

Hepatitis B

If the source is HBsAg-negative, and unlikely to be in the window period, no further follow-up testing is required for source or healthcare worker. The healthcare worker should have full HBV immunization if it has not been already
performed. It should be commenced as soon as possible, and preferably within 24 hours of exposure.

If hepatitis B immunoglobulin (HBIG) is indicated, it should be given as soon as possible and preferably within 24 hours exposure. Efficacy of HBIG beyond 7 days of exposure is unknown.

Hepatitis C

If the source is HCV antibody negative, and unlikely to be in the window period, no further follow-up testing is required for the source or healthcare worker.

Follow-up testing of the healthcare worker should be done in all other circumstances, and should include HCV RNA testing at 4 to 6 weeks, and HCV antibody and ALT at 4 to 6 months.

There is no effective or active immunoprophylaxis. Early therapy if seroconversion occurs should be considered.

HIV

If the source is HIV antibody negative, and unlikely to be in the window period, no further follow-up testing is required for the source or healthcare worker.

In all other circumstances, the healthcare worker should have follow-up antibody testing at 6 weeks and 3 months, and up to 6 months, along with tests for other blood-borne viruses as above.

If post-occupational exposure prophylaxis (PEP) for HIV is indicated, it should be commenced as soon as possible after exposure. it is substantially less effective in animal studies when started more than 24-36 hours after exposure,
but the interval after which no benefit is gained is unknown.

Recommendation for PEP include a basic regimen of 2 nucleoside/nucleotide reverse transcriptase inhibitors for most HIV exposures, and an expanded regimen with the addition of a third drug when the exposure poses an increased
risk for transmission.

Expert advice from an HIV physician or adherence to local agreed guideline is essential before PEP for HIV infection is initiated.

References

• Therapeutic Guidelines – Antibiotics; available from http://tg.org.au

Last updated:
Time spent: QID:991 2023-2-12

1699 of 1943
Mother of an 11-month-old boy brings him to your practice for vaccination. Three weeks ago, the boy had a fever and rash suggestive of rubella infection. Which one of the following will be the most appropriate step in management?

A. Vaccinate him with MMR vaccine now.

B. Order a serum anti-rubella antibody to guide further decision.

C. Do not give rubella vaccine as he is already immune.

D. Tell her she should wait until the boy turns 12 months before MMR vaccination.

E. Vaccinate him at 15 months of age.

Correct
45% answered correctly

Explanation:

Correct Answer Is A

MMR-containing vaccines are not routinely recommended for infants younger than 12 months of age because maternal antibodies to measles (if the mother has immunity) are known to persist in many infants until approximately 11 months of age
and may interfere with active immunization if performed before 12 months of age. However, there is some evidence that the first dose if given at ≥11 months (but prior to 12 months) is sufficiently immunologic.

Rubella infection provides life-long immunity in an immunocompetent person, but not against measles and mumps. This child is still in risk of catching measles and mumps, and MMR vaccine is still indicated regardless of immune status for rubella.
He is 11 months of age; therefore, the first dose of MMR vaccine can be administrated now.

(Options B) The results of anti-rubella serology will not affect further plans regarding MMR vaccination; therefore, serologic studies for rubella is inappropriate.

(Option C) Although the child will be immune against rubella after the infection, he is still in need for MMR for measles and mumps. A monovalent vaccine for rubella does not exist.

(Option D) Although the recommended time for first dose of MMR is at 12 months of age, for the sake of convenience, this child can receive his first dose of MMR vaccine in this visit and at the age of 11 months. MMR vaccine can also be
administered as early as 9 months of age if the infant is high risk for measles, mumps, or rubella, e.g. travel to endemic area, outbreaks, etc.

(Option E) The first dose of MMR vaccine should be given at 12 months of age (≥11 months). Deferring the first dose untill 15 months is not appropriate.

TOPIC REVIEW

Update on MMR vaccination

Two doses of measles-containing vaccine are recommended for all children. The first dose should be given at 12 months of age as MMR vaccine. MMRV (measles, mumps, rubella, varicella) vaccines are NOT recommended for use as the 1st dose of
MMR-containing vaccine in children <4 years of age, due to a small but increased risk of fever and febrile seizures when given as the 1st MMR-containing vaccine dose in this age group.

The second dose of measles-containing vaccine is recommended to be given routinely at 18 months of age as MMRV vaccine. The recommended age for administration of the second dose of measles-containing vaccine has been moved down from
4 years of age, to provide earlier 2-dose protection against measles, mumps and rubella, and to improve vaccine uptake.

Catch-up vaccination of children who did not receive the second dose of MMR-containing vaccine at 18 months of age can occur at the 4-year-old schedule point. Use of MMRV vaccine at the 4-year-old schedule point is preferred when varicella
vaccination is also indicated.

Children >12 months of age who have received 1 dose of MMR vaccine can be offered their second dose of MMR-containing vaccine early (if at least 4 weeks after the 1st dose has elapsed) if they are considered at risk of coming in contact with
measles.

If MMRV vaccine is inadvertently administered as dose 1 of MMR-containing vaccine, the dose does not need to be repeated (providing it was given at ≥12 months of age). However, parents/carers should be advised regarding the small but increased
risk of fever and febrile seizures (compared with the expected risk following MMR vaccine).

References

• http://www.immunise.health.gov.au/internet/immunis

Last updated:
Time spent: QID:1021 2023-2-12

1700 of 1943
A 31-year-old woman presents to your GP clinic quite concerned about contracting measles. Three days ago she met one of her old friends from school in a shopping mall. After having a coffee, her friend drove her back home. Last night, she called
her friend to invite her for lunch, but her friend was down with measles confirmed by her GP after she developed a rash yesterday morning. She does not remember if she has been vaccinated in childhood, nor is she sure if she had measles before.
She asks about what she should do. Which one of the following is the most appropriate advice for her?

A. Perform measles serology.

B. She should receive MMR vaccine now.

C. She should receive natural human immunoglobulin (NHIG).

D. She is not at risk of the infection as she is immune.

E. She should come back if symptoms develop.

Incorrect. Correct answer is B


45% answered correctly

Explanation:

Correct Answer Is B

Measles is highly infectious. The main route of transmission is by respiratory airborne droplets; however, it can rarely be passed on to others by means of particles soiled with respiratory droplets.

The infectivity of a person with measles begins 2 days before the onset of the symptoms until 4 days after eruption of the rash. This woman has been in contact with her friend 2 days before the rash; therefore, within the infectivity period, and is at
risk of contracting measles if not already immune.

Any non-immunocompromised individual with a definite positive history of measles disease, documented evidence of having received 2 doses of a measles-containing vaccine (MMR) administered at least 4 weeks apart and with both doses
administered ≥ 12 months of age, or positive measles antibody test is considered to be immune.

This woman does not remember if she has been immunized or contracted measles. In such situations, where the immunity status is unknown, it is recommended that all immunocompetent, non-pregnant patient receive MMR of vaccine within 72
hours of exposure. A second dose should then follow at least 4 weeks later.

(Option A) Measles serology was the correct answer if the woman was pregnant or immunocompromised and considered for prophylaxis with natural human immunoglobulin (NHIG). In such cases, if time allows, serology testing is recommended,
as a positive IgG against measles will indicate immunity and no need for prophylaxis.

(Option C) NHIG is used if the patient is in need for prophylaxis but there is a contraindication to MMR vaccination such as pregnancy or immunodeficiency.

(Option D) This woman is uncertain about her immune status. She does not remember contracting measles or if she had been vaccinated. She cannot be considered immune and post-exposure prophylaxis should be considered.

(Option E) While post-exposure is indicated in this patient in an attempt to decrease the risk of measles infection, telling the patient to wait until she develops active infection is not appropriate.

TOPIC REVIEW

Measles (rubeola) virus is a single-stranded RNA virus of the family Paramyxoviridae. Humans and monkeys are the only known hosts. There are no carrier states. Measles has an incubation period of 10-14 days. Initially, the illness presents with the
following:

Fever
Malaise
Cough
Coryza (inflammation of the mucous membranes of the nose)
Conjunctivitis
Koplik’s spots (white spots, each surrounded by a red ring, found on the buccal mucosa)

Two to 4 days after the initial symptoms, a maculopapular rash develops. The rash typically starts from face and upper neck and later spreads to involve the whole body. The infectivity periods starts 2 days before the onset of rash untill 4 days after
the rash develops.

Measles is often a severe disease. Complication may occur, and include otitis media in 7% and bronchopneumonia in 6% of immunocompetent patients. Acute encephalitis is a rare complication occurring in between 2 and 10 per 10,000 reported
cases with measles in the general population, with an associated mortality rate of 10 -15 %. Around 15 - 40 % of survivors of measles encephalitis will have permanent brain damage.

Measles can be more sever in the following groups:

Immunocompromised people, especially those with T-cell deficiencies such as certain leukemias, lymphomas and acquired immunodeficiency syndrome (AIDS), in whom measles can be severe, atypical (e.g. without rash) and
prolonged, with virus shedding for several weeks after the acute illness

Malnourished children, particularly those with vitamin A deficiency

Children younger than 5 years and adults 20 years of age and older, in whom complications of measles are more common

Pregnant women in whom infection is associated with increased risk of premature labor, spontaneous abortion, and low birth weight. Birth defects have not conclusively been associated with measles virus infection.

Management of contacts

Neonates or infants less than 12 months of age

This group is at high risk of developing complications from measles infection if they have not acquired maternal antibodies from a measles-immune mother; therefore, NHIG should be administered to infants of non-immune mothers as
early as possible, and at the latest within 6 days (144 hours) of potential exposure.

NHIG should be administered to pre-term neonates (< 37 weeks) or immune-suppressed infants regardless of maternal history or antibody status.

Infants 6-8 months of age exposed within 144 hours should receive NHIG (irrespective of mother's immune status).

1701 of 1943
If exposure has occured within 72 hours, immune competent infants 9-11 months should receive MMR now then second dose at 12 months or 4 weeks later (whichever is later).

If exposure within 73-144 hours, give NHIG

Non-immunosuppressed non-vaccinated people over 1 year of age

Consider MMR vaccination within 72 hours of exposure for all non-immune contacts.

Individuals in whom protection is desirable, but live vaccination is contraindicated

Pregnant women and immunocompromised children (or adults) should not be given live vaccine. NHIG should be administered as early as possible and up to 6 days (144 hours) of exposure.

References

• http://www.health.gov.au/internet/main/publishing.

• http://www.sahealth.sa.gov.au/wps/wcm/connect/b9f4

• http://www.ncirs.edu.au/assets/provider_resources/

Time spent: QID:1023 Last updated:


2023-2-12

1702 of 1943
A 6-month-old male infant is brought to your clinic by his mother for receiving his third dose of DTPa vaccine. He has received his previous doses at 6 weeks and 4 months of age. The child sneezes, has a runny nose and a fever of 38.2°. Exam
findings are consistent with upper respiratory tract infection (URTI). According to the mother, last time he received his DTPa vaccine, he developed a huge swelling at the injection site. Which one of the following is the most appropriate management
regarding vaccination with DTPa in this situation?

A. Vaccinate him now.

B. Vaccinate after his UTRI settles.

C. Do not give the pertussis component.

D. Use dTpa instead.

E. Vaccinate him after 2 months.

Correct
45% answered correctly

Explanation:

Correct Answer Is A

Pertussis vaccine is available in Australia only in combination with diphtheria, tetanus and other antigens.

The acronym DTPa, using capital letters, signifies child formulations of diphtheria, tetanus and acellular pertussis-containing vaccines. The acronym dTpa is used for formulations that contain substantially lesser amounts of diphtheria toxoid and
pertussis antigens than child (DTPa-containing) formulations; dTpa vaccines are usually used in adolescents and adults.

Pertussis-containing vaccine is recommended in a 3-dose primary schedule for infants at 2, 4 and 6 months of age. Due to the high morbidity and occasional mortality associated with pertussis in the first few months of life, the first dose can be
given as early as 6 weeks of age. Giving a first dose at 6 weeks of age rather than 2 months of age is estimated to prevent an additional 8% of infant pertussis cases. The next scheduled doses should still be given at 4 months and 6 months of age.

Extensive limb swelling, defined as swelling and/or redness involving at least half the circumference of the limb and the joints both above and below the injection site is a recognized adverse event that occurs rarely following booster doses of DTPa.
Such reaction begins within 48 hours of vaccination, last for 1 to 7 days and resolve completely without sequelae. History of such event is not a contraindication to the vaccine use. No change in the formulation is required or recommended either.

NOTE – The only absolute contraindications to pertussis containing vaccines are:

Anaphylaxis following a previous dose of any acellular pertussis-containing vaccine


Anaphylaxis following any vaccine component

Children with mild disease and fever <38.5°C can be safely vaccinated. However, if the fever is ≥38.5°C, or the child has serious systemic illness, vaccination should be deferred until the child improves. The rationale behind such recommendations is
to avoid adverse events in an already unwell child, or to avoid attributing symptoms to vaccination. Conditions such as mild diarrhea or URTI without systemic symptoms are safe for vaccination.

This child had an adverse reaction to the DTPa vaccine that is known to be benign and self-limiting, and is not a contraindication to vaccination with pertussis containing vaccines. Although febrile, his fever is less than 38.5°C associate with a mild
disease; therefore, he can be safely vaccinated with DTPa now.

Other facts about pertussis-containing vaccines

Febrile convulsions are very infrequently reported following DTPa-containing vaccines within 48 hours of vaccination. The risk is even lower in infants who complete their primary course at 6 months of age, as febrile convulsions are
uncommon in children <6 months of age. Children who experience a febrile convulsion after a dose of DTPa-containing vaccine have a slightly greater risk of a further febrile convulsion following a subsequent dose of a DTPa-
containing vaccine. This risk can be minimized by appropriate measures to prevent fever, so vaccination still recommended.

Pertussis-containing vaccines do not cause infantile spasms or epilepsy. Infants and children known to have active or progressive neurological disease can be safely vaccinated with DTPa-containing vaccines.

Sudden infant death syndrome (SIDS) is not associated with either DTPa or any pertussis-containing vaccine. Some studies suggest a decreased risk of SIDS in children who have been vaccinated.

Brachial neuritis (inflammation of a nerve in the arm, causing weakness or numbness) has been described following the administration of tetanus toxoid-containing vaccines, with an estimated excess risk of approximately 0.5–1 in 100
000 doses in adults.

References

• http://www.immunise.health.gov.au/internet/immunis

• https://www.cdc.gov/vaccines/hcp/patient-ed/conver

Last updated:
Time spent: QID:1027 2023-2-12

1703 of 1943
A mother had brought her 18-month-old female child for MMR vaccination. The baby has flu-like symptoms of runny nose, cough and sneezes. She looks slightly ill. On examination, she has a fever of 38.7°C. Which one of the following is the most
appropriate management?

A. Give MMR vaccine without the rubella portion.

B. Give MMR vaccine without the rubella portion and give the rubella portion after she recovers from her illness.

C. Give MMR vaccine now.

D. Postpone the vaccination untill after recovery from her current illness.

E. She should receive the second dose at 4 years of age.

Incorrect. Correct answer is D


45% answered correctly

Explanation:

Correct Answer Is D

Children with mild disease and fever <38.5°C can be safely vaccinated. However, if the fever is ≥38.5°C, or the child has serious systemic illness, vaccination should be deferred until the child improves. The rationale behind such recommendations is
to avoid adverse events in an already unwell child, and to avoid attributing symptoms to vaccination. Conditions such as mild diarrhea or URTI without systemic symptoms and fever <38.5°C are safe for vaccination.

This child has a fever of 38.7°C; therefore, vaccination is recommended to be deferred until she is out of the disease and the fever subsides.

The only vaccines available in Australia is MMR (measles, mumps, rubella) and MMRV (measles, mumps, rubella, varicella). Separate monovalent vaccines against rubella, measles or mumps do not exist.

Current guidelines recommend two doses of MMR vaccine at 12 months and 18 months of age. Postponing MMR vaccine until the age of 4 years is against recommendations and not appropriate.

References

• http://www.immunise.health.gov.au/internet/immunis

Last updated:
Time spent: QID:1033 2023-2-12

1704 of 1943
A 62-year-old man presents to your GP clinic for a routine health check after he sustained a myocardial infarction (MI) few months ago. After the event, he stopped smoking and has been following a healthy diet. He also does regular exercise. On
examination, he has a blood pressure of 137/85 mmHg. His cholesterol level is 4.0 mmol/L. Which one of the following would be the most appropriate advice for him?

A. Maintaining the diet and regular exercise.

B. He should have his cholesterol checked in 3 months.

C. He should be started on lipid lowering agents.

D. He should be started on antihypertensive medications.

E. He should have cholesterol level checked annually.

Incorrect. Correct answer is C


45% answered correctly

Explanation:

Correct Answer Is C

With a previous MI, this man is already high risk (absolute risk >15%), and absolute cardiovascular risk calculation should not be performed for him. This patient is following strict lifestyle modification. He quit smoking which is the single most
modifiable risk factor for cardiovascular diseases. He takes exercise on a regular basis, and follows a healthy diet.

Statins should be started in all patients with coronary heart disease (CHD) regardless of they current lipid profile status as it is associated with decreased mortality. His lipid profile then should be checked evey 3-6 moths.

Target values for patients with CHD are:

Low-density lipoprotein cholesterol (LDL-C) < 1.8 mmol/L


High-density lipoprotein cholesterol (HDL-C) > 1.0 mmol/L
Triglyceride (TG) < 2.0 mmol/L
Non–high-density lipoprotein cholesterol (NHDL-C) < 2.5 mmol/L

Although the target blood pressure in patients with CHD is ≤130/85mmHg, his blood pressure of 137/85mmHg is not of significant concern compared to importance of commencement of lipid-lowering agents. He should also be advised to keep on
diet and regular exercise.

References

• http://heartfoundation.org.au/images/uploads/publi

• https://heartfoundation.org.au/images/uploads/publ

Last updated:
Time spent: QID:1034 2023-2-12

1705 of 1943
Which one of the following conditions requires major attention for prevention of blindness in Aboriginal Australians?

A. Trichiasis.

B. Diabetes mellitus.

C. Bacterial conjunctivitis.

D. Herpetic keratitis.

E. Chronic simple glaucoma.

Correct
45% answered correctly

Explanation:

Correct Answer Is A

Blindness from Chlamydia trachomatis is a major cause of blindness where the socioeconomic status is poor and people disadvantaged. Many Aboriginal Australian communities
exemplify such situation.

Recurrent infections with Chlamydia trachomatis cause chronic keratoconjunctivitis. This is then followed by scarring of the tarsal conjunctiva. A the condition progresses, tarsal
plates become distorted eyelids and eyelashes turn inwards (entropion) as do eyelashes. Inwardly-turned eyelashes will irritate the cornea (trichiasis). Chronic irritation leads to
abrasions, corneal opacification and eventually blindness.

Although all other options can potentially lead to blindness, infection with Chlamydia trachomatis remains the major risk and concern in Aboriginal communities.

References

• AMC Handbook of Multiple Choice Questions - pages 623- 624

Last updated:
Time spent: QID:298
2023-2-12

1706 of 1943
Which one of the following is the study of choice for determining the state of vitamin D deficiency in Australia?

A. Cohort study.

B. Case control study.

C. Randomized controlled trial (RCT).

D. Cross sectional study.

E. Case series.

Incorrect. Correct answer is D


45% answered correctly

Explanation:

Correct Answer Is D

The objective of the study is determining the prevalence of vitamin D deficiency in Australia. When prevalence of a specific condition in a population is the aim, cross sectional studies are used as the most appropriate method.

Cross sectional studies are also called prevalence studies, because one of the main measures available is the 'population prevalence'. These studies consist of assessing a population, as represented by the study sample, at a single point in time.
Cross-sectional study investigates the presence or absence of any disease or other condition in each member of the study. Cross sectional studies can also describe the odd ratio, absolute risk and relative risk. These parameters cannot be elicited
from case-control studies. Cross-sectional are also able to support (not establish) the interference of cause and effect.

Cross-sectional studies are different from case-control studies in that they aim to provide data on the entire population under study, whereas case-control studies typically include only individuals with a specific characteristic.

(Option A) In cohort studies, the population group is identified. Subjects who have been exposed to a certain risk factor are followed over time and compared with a group not exposed to the risk factor. Cohort studies are prospective (forward-
looking), and subjects are tracked forward in time. It can determine incidence and causal relationship; however, the population must be followed long enough for the incidence to appear.

(Option B) Case-control studies are observational studies that compare subjects who have a disease or outcome (cases) with subjects who do not have the disease or outcome (controls), and look back retrospectively to compare how frequently the
exposure to a risk factor is present in each group to determine the relationship between the risk factor and the disease.

Option C) Randomized controlled trials (RCTs) are interventional studies conducted for the assessment of effectiveness and safety of a drug, procedure or any other intervention.

(Option E) Case series is objective report of a clinical characteristic or outcome from a group of clinical subjects (n>1). For example, patient at a local hospital with hospital acquired pneumonia.

NOTE - Cross sectional studies are the method of choice for determining prevalence in a population.

References

• http://www.ncbi.nlm.nih.gov/pmc/articles/PMC350529

• http://www.ncbi.nlm.nih.gov/pmc/articles/PMC350529

• https://himmelfarb.gwu.edu/tutorials/studydesign10

• https://himmelfarb.gwu.edu/tutorials/studydesign10

• https://himmelfarb.gwu.edu/tutorials/studydesign10

Last updated:
Time spent: QID:1045 2023-2-12

1707 of 1943
To study the prevalence of cancer in a city, which one of the following studies is most appropriate?

A. Double blind clinical trial.

B. Cross sectional study.

C. Cohort.

D. Case control.

E. Randomized controlled trial (RCT).

Incorrect. Correct answer is B


45% answered correctly

Explanation:

Correct Answer Is B

For determining the prevalence of a disease or a condition in a population, cross sectional studies are used as the method of choice.

Cross sectional studies are also called prevalence studies because one of the main purposes and measures available is determining the ‘prevalence’ of a condition (e.g. disease). These studies consist of assessing a population, as represented by
the study sample, at a single point in time. Cross-sectional study investigates the presence or absence of any disease or other condition in each member of the study.

(Option A) Double blind studies are gold-standard for interventional studies conducted for the assessment of effectiveness and safety of a drug, procedure or any other intervention.

(Option C) The main objective in cohort studies is to see if there is an established link between one variable and the disease. In cohort studies, the population group is identified. Subjects who have been exposed to a certain risk factor are followed
over time and compared with a group not exposed to the risk factor. The outcome is the disease incidence in each group, as well as the linkage between the cause (risk factor) and the effect (disease). Cohort studies are prospective (forward-
looking), and subjects are tracked forward in time. A cohort study can determine incidence and causal relationship (such as hypercholesterolemia and myocardial infarction); however, the population must be followed long enough for the incidence to
appear.

(Option D) Case-control studies (also known as retrospective studies and case-referent studies) are observational studies that compares subjects who have a disease or outcome (cases) with subjects who do not have the disease or outcome
(controls), and looks back retrospectively to compare how frequently the exposure to a risk factor is present in each group to determine the relationship between the risk factor and the disease.

NOTE – The main differences between cohort study and case control study are:

In case control study the subjects in the study population have the disease, while in cohort subjects are not diseased.
Cohort study is prospective (forward-looking), while case control study is retrospective (back-looking)

(Option E) Similar to randomized double blind studies, RCTs are also interventional studies.

References

• http://www.ncbi.nlm.nih.gov/pmc/articles/PMC350529

• http://www.ncbi.nlm.nih.gov/pmc/articles/PMC350529

• https://himmelfarb.gwu.edu/tutorials/studydesign10

• https://himmelfarb.gwu.edu/tutorials/studydesign10

• https://himmelfarb.gwu.edu/tutorials/studydesign10

Last updated:
Time spent: QID:1046 2023-2-12

1708 of 1943
Which one of the following studies should be used to investigate the relationship between the number of nevi and melanoma?

A. Cross sectional study.

B. Cohort study.

C. Case-control study.

D. Comparative analysis.

E. Case series report.

Incorrect. Correct answer is B


45% answered correctly

Explanation:

Correct Answer Is B

Cohort studies are the option of choice when a causal relationship between a variable (e.g. a risk factor) and a disease or condition is the objective of the study. In fact they are best for identification of risk factors, as well as the incidence.

Cohort studies are usually prospective (forward-looking). They are planned in advance and carried out over a future period of time.

In a prospective cohort study, a research question is raised first, forming a hypothesis about potential causes of a disease. The researcher then observe a group of people (the cohort) over a period of time (often several years), collecting data that
may be relevant to the disease. This allows the researchers to detect any changes in health in relation to the potential risk factors they have indentified.

To establish a relationship between the number of nevi (risk factor) and melanoma (the disease), cohort study is the study of choice.

(Option A) Cross sectional studies are the studies of choice whenever determination of the prevalence of particular condition of disease is desired.

(Option C) Case-control studies (also known as retrospective studies and case-referent studies) are observational studies that compares subjects who have a disease or outcome (cases) with subjects who do not have the disease or outcome
(controls), and looks back retrospectively to compare how frequently the exposure to a risk factor is present in each group to determine the relationship between the risk factor and the disease.

(Option D) Comparative analysis is the item-by-item comparison of two or more comparable alternatives, processes, products, qualifications, set of data, etc. It can be used, for example, for comparing two treatments for one condition, but generally it
is not a distinct research entity, nor can it establish or exclude a relationship between a risk factor and a condition.

(Option E) Case report (study) is an article describing and interpreting an individual case, often written in the form of a detailed story. Case reports are considered the lowest level of evidence, but they are also the first line of evidence, because they
are where new issues and ideas emerge. Case series report (study) is similar to case repot (study) with the only difference being the number of cases compared to case report (n=1 versus n>1).

References

• http://www.ncbi.nlm.nih.gov/pmc/articles/PMC350529

• http://www.ncbi.nlm.nih.gov/pmc/articles/PMC350529

• https://himmelfarb.gwu.edu/tutorials/studydesign10

• https://himmelfarb.gwu.edu/tutorials/studydesign10

• https://himmelfarb.gwu.edu/tutorials/studydesign10

Last updated:
Time spent: QID:1048 2023-2-12

1709 of 1943
You want to find the causality between hypertension and myocardial infarction. Which one of the following studies should you use?

A. Cohort study.

B. Cross-sectional study.

C. Case-control study.

D. Randomization.

E. Double-blind clinical trial (RCT).

Correct
45% answered correctly

Explanation:

Correct Answer Is A

To see if there is a relationship between hypertension as a risk factor and myocardial infarction as the outcome, cohort studies are the most appropriate option.

Cohort studies are the option of choice when a casual relationship between a variable (e.g. a risk factor) and a disease or condition is investigated. They are best for identification of risk factors, as well as the incidence.

Cohort studies are usually prospective (forward-looking). They are planned in advance and carried out over a future period of time.

In a prospective cohort study, a research question is raised first, forming a hypothesis about potential causes of a disease. The researcher then observe a group of people (the cohort) over a period of time (often several years), collecting data that
may be relevant to the disease. This allows the researchers to detect any changes in health in relation to the potential risk factors they have indentified.

(Option B) Cross sectional studies are also called prevalence studies because one of the main purposes and measures available is population prevalence. These studies consist of assessing a population, as represented by the study sample, at a
single point in time. Cross-sectional study investigates the presence or absence of any disease or other condition in each member of the study.

(Option C) Case-control studies (also known as retrospective studies and case-referent studies) are observational studies that compare subjects who have a disease or outcome (cases) with subjects who do not have the disease or outcome
(controls), and look back retrospectively to compare how frequently the exposure to a risk factor is present in each group to determine the relationship between the risk factor and the disease.

(Option D) Randomization is a sampling method used for unbiased sample definition and selection for research studies which is a necessary element for a study to be valid and trustable. It is not a study.

(Option E) Double blinded studies are gold-standard for interventional studies conducted for the assessment of effectiveness and safety of a drug, procedure or any other intervention.

References

• http://www.ncbi.nlm.nih.gov/pmc/articles/PMC350529

• http://www.ncbi.nlm.nih.gov/pmc/articles/PMC350529

• https://himmelfarb.gwu.edu/tutorials/studydesign10

• https://himmelfarb.gwu.edu/tutorials/studydesign10

• https://himmelfarb.gwu.edu/tutorials/studydesign10

Last updated:
Time spent: QID:1049 2023-2-12

1710 of 1943
Which one of the following is the study of choice to assess the efficacy of a vaccine on a group of people?

A. Randomized controlled trial.

B. Cohort study.

C. Case control study.

D. Cross-sectional study.

E. Case series report.

Correct
45% answered correctly

Explanation:

Correct Answer Is A

Randomized controlled trials (RCTs) are the gold-standard study for assessment of safety and effectiveness of an intervention such as a vaccine, drug, or method of treatment.

Cohort study, cross-sectional study, case-control study and case series report are all observational, and not interventional studies. Observational studies are used for assessment and examining the etiology and potential risk factors of a disease.

References

• http://www.ncbi.nlm.nih.gov/pmc/articles/PMC350529

• http://www.ncbi.nlm.nih.gov/pmc/articles/PMC408357

• https://himmelfarb.gwu.edu/tutorials/studydesign10

Last updated:
Time spent: QID:1050 2023-2-12

1711 of 1943
A young scientist has decided to study the cause of neonatal jaundice. He selects 150 babies with and 150 babies without jaundice, and investigates their histories including the maternal obstetric history for factors that could have caused jaundice
in them. Which one of the following is the type of study she is using?

A. Cohort study.

B. Case report.

C. Case control study.

D. Prospective study.

E. Randomized controlled trial (RCT).

Incorrect. Correct answer is C


45% answered correctly

Explanation:

Correct Answer Is C

When evaluation of a causal relationship between a variable such as a risk factor and a disease or condition is desired, either a case control or a cohort study should be considered and designed. The decision as to whether which one should be used
depends on several factors.

Cohort study is a study designed when one or more samples (called cohorts) are followed prospectively (forward-looking) to see if there is a linkage between a variable, such as a risk factor, and an outcome of interest. Cohort studies are used to
investigate the cause of disease, and establishing links between risk factors and health outcomes. Cohort studies are usually prospective (forward-looking). They are planned in advance and carried out over a future period of time.

Case control studies compare patients who have a disease or outcome of interest (cases) with patients who do not have the disease or outcomes (controls), and looks back (retrospective) to compare how frequently the exposure to a risk factor is
present in each group to determine the relationship between the risk factor and the disease. Case control studies are observational because no intervention is attempted and no attempt is made to alter the course of the disease. The goal is to
retrospectively determine the exposure to the risk factor of interest from each of the two groups of individuals: cases and controls. These studies are designed to estimate odds. Case control studies are also known as "retrospective studies" and
"case-referent studies”.

This scientist wants to compare a group of neonates with jaundice (disease present) with a group of neonates without jaundice, by looking back at the history to see how exposure to hypothesized parameters could have resulted in jaundice.
Therefore, he should use a case control study as the most appropriate method.

(Option A) Cohort study is a forward-looking study for assessment of the relationship between a variable (risk factor) and a condition e.g. the relationship between hypertension and ischemic heart disease. The relation can also be determined by
case-control studies; however, cohort study has many advantages over the latter, making it the preferred study.

(Option B) Case report (study) is an article describing and interpreting an individual case, often written in the form of a detailed story.

(Option D) Prospective study is the term sometimes used for a cohort study.

(Option E) RCTs are interventional studies conducted for the assessment of effectiveness and safety of a drug, procedure or any other intervention.

References

• http://www.ncbi.nlm.nih.gov/pmc/articles/PMC408357

• https://himmelfarb.gwu.edu/tutorials/studydesign10

• https://himmelfarb.gwu.edu/tutorials/studydesign10

• https://himmelfarb.gwu.edu/tutorials/studydesign10

• http://www.ncbi.nlm.nih.gov/pmc/articles/PMC350529

Last updated:
Time spent: QID:1051 2023-2-12

1712 of 1943
You want to design a study about the asthmatic children to find out if there is a relationship between asthma and exposure to smoking and its severity. Which one of the following studies is the most appropriate study for this purpose?

A. Case control study.

B. Cross-sectional study.

C. Cohort study.

D. Observational study.

E. Randomized controlled trial (RCT).

Correct
45% answered correctly

Explanation:

Correct Answer Is A

This study is intended to assess if there is a causal linkage between a risk factor (exposure to smoke and the severity) and an outcome of interest (asthma) in a group, who already have the condition (asthmatic patients). For such an approach, case
control study should be conducted.

Case control studies compare patients who have a disease or outcome of interest (cases) with patients who do not have the disease or the outcomes (controls), and looks back (retrospective) to compare how frequently the exposure to a risk factor
is present in each group to determine the relationship between the risk factor and the disease. Case control studies are observational because no intervention is attempted and no attempt is made to alter the course of the disease. The goal is to
retrospectively determine the exposure to the risk factor of interest from each of the two groups of individuals: cases and controls. These studies are designed to estimate odds.

(Option B) Cross sectional studies are also called prevalence studies because one of the main purposes and measures available is population prevalence. These studies consist of assessing a population, as represented by the study sample, at a
single point in time. Cross-sectional study investigates the presence or absence of any disease or other condition in each member of the study.

(Option C) Cohort study is another method to see if a relationship between cause and effect exist, in a retrospective (forward-looking) manner. It consists of observing the development of the condition of interest (e.g. asthma) in case who do not
have asthma at the beginning of the study and seeing whether the case develops the condition in the future, and if so, how relevant it can be to the cause (smoke in this question). Unlike case control study, cohort is usually prospective, and the
population studied has not the condition in the outset.

(Option D) Observational study is a term used for studies in which no intervention is attempted during the study. Cohort, case control, cross sectional, case report, and case series report are all observational studies.

(Option E) RCTs are interventional studies conducted for the assessment of effectiveness and safety of a drug, procedure or any other intervention.

References

• http://www.ncbi.nlm.nih.gov/pmc/articles/PMC408357

• https://himmelfarb.gwu.edu/tutorials/studydesign10

• https://himmelfarb.gwu.edu/tutorials/studydesign10

• https://himmelfarb.gwu.edu/tutorials/studydesign10

• http://www.ncbi.nlm.nih.gov/pmc/articles/PMC350529

Last updated:
Time spent: QID:1052 2023-2-12

1713 of 1943
A medical doctor wants to investigate the causal relationship between alcohol intake and dementia. He designs a study and starts to collect cases with dementia and the history of alcohol use. Which one of the following is the type of study he
intends to use?

A. Case series.

B. Cohort study.

C. Case control study.

D. Cross sectional study.

E. Randomized control trial (RCT).

Incorrect. Correct answer is C


45% answered correctly

Explanation:

Correct Answer Is C

The objective of this study is to see if there in a causal relationship between a risk factor (alcohol) and a condition of interest (dementia). When establishment of such association or linkage is the purpose, two studies can be used:

Case control studies - Compare patients who have a disease or outcome of interest (cases) with patients who do not have the disease or outcomes (controls), and looks back to compare how frequently the exposure to a risk factor is present in each
group to determine the relationship between the risk factor and the disease. These studies are designed to estimate odds.

Cohort studies - Cohort study is another method to investigate a relationship between cause and effect, but it is prospective (forward-looking), and observes the development of the condition of the interest (e.g. dementia) in cases who do not have
the disease at the beginning of the study and sees if they develop the condition in the future, and if so how relevant it can be to the cause (amalgam in this question). Unlike case control study, cohort is usually prospective and the population studied
has not the condition in the beginning of the study.

This doctor is using a sample of people who already have dementia and wants to look back at their history of alcohol intake to see if he can find a relationship. These features are characteristics of a case control study.

(Option A) Case report (study) is an article describing and interpreting an individual case, often written in the form of a detailed story. Case series report (study) is similar to case repot (study) with the only difference being the number of cases
compared to case report (n=1 versus n>1).

(Option B) In cohort study, the target group has not the disease yet. In the study conducted by this doctor, dementia (the disease) has already established in the study group.

(Option D) Cross sectional studies consist of assessing a population, as represented by the study sample, at a single point in time. Cross-sectional study investigates the presence or absence of any disease or other condition in each member of the
study. They are used mostly to see how prevalent a condition is in a population.

(Option E) RCTs are interventional studies conducted for the assessment of effectiveness and safety of a drug, procedure or any other intervention.

References

• http://www.ncbi.nlm.nih.gov/pmc/articles/PMC408357

• https://himmelfarb.gwu.edu/tutorials/studydesign10

• https://himmelfarb.gwu.edu/tutorials/studydesign10

• https://himmelfarb.gwu.edu/tutorials/studydesign10

• http://www.ncbi.nlm.nih.gov/pmc/articles/PMC350529

Last updated:
Time spent: QID:1053 2023-2-12

1714 of 1943
You are a doctor in a hospital who has decided to conduct a research about the outcomes of babies born to diabetic mothers compared to those born to non-diabetics mothers. Which one of the following studies should you select?

A. Cross sectional study.

B. Cohort study.

C. Case control study.

D. Randomized controlled trial (RCT).

E. Case report study.

Incorrect. Correct answer is B


45% answered correctly

Explanation:

Correct Answer Is B

When evaluation of a causal relationship between a variable such as a risk factor and a disease of condition is desired, either a case control or a cohort study should be considered and designed. The decision as to whether which one should be used
depends on several factors.

Cohort studies are used to investigate the cause of disease, and establishing links between risk factors and health outcomes. Cohort studies are usually prospective (forward-looking). They are planned in advance and carried out over a future period
of time.

In a prospective cohort study, a research question is raised first, forming a hypothesis about potential causes of a disease. The researcher then observe a group of people (the cohort), over a period of time (often several years), collecting data that
may be relevant to the disease. This allows the researchers to detect any changes in health in relation to the potential risk factors they have indentified. For example, scientists may ask participants to record specific lifestyle details over the course of
a study, before going on to analyze any possible correlations between lifestyle factors and disease.

In contrast, case control studies are retrospective, looking back at data that already exist and, through this, try to indentify risk factors for particular conditions. The drawback to this study is the fact that the researchers are limited in their
interpretations because they cannot retrospectively retrieve missing data.

Here, being born to a diabetic mother has been considered a risk factor, and the intended future outcomes related to this risk factor is the objective of this study. For this purpose, a cohort study serves best.

(Option A) Cross sectional studies consist of assessing a population, as represented by the study sample, at a single point in time. Cross-sectional study investigates the presence or absence of any disease or other condition in each member of the
study group. They are used mostly to see how prevalent a condition is in a population.

(Option C) Case control study was the choice if the aim of the study was to find out how a current problem in the babies could be related to the maternal obesity (retrospective approach).

(Option D) RCTs are interventional studies conducted for the assessment of effectiveness and safety of a drug, procedure or any other intervention.

(Option E) Case report (study) is an article describing and interpreting an individual case, often written in the form of a detailed story.

References

• http://www.ncbi.nlm.nih.gov/pmc/articles/PMC408357

• https://himmelfarb.gwu.edu/tutorials/studydesign10

• https://himmelfarb.gwu.edu/tutorials/studydesign10

• https://himmelfarb.gwu.edu/tutorials/studydesign10

• http://www.ncbi.nlm.nih.gov/pmc/articles/PMC350529

Last updated:
Time spent: QID:1054 2023-2-12

1715 of 1943
You are a dentist and you want to find the relationship between mercury amalgam and the risk of development of dementia. You select 2 groups of patients, one with dementia and the other with mercury amalgam. Which one of the following studies
is the study you are using?

A. Cross sectional study.

B. Randomized controlled trial (RCT).

C. Case control study.

D. Cohort study.

E. Case series.

Incorrect. Correct answer is C


45% answered correctly

Explanation:

Correct Answer Is C

The objective of this study is to see if there in a causal relationship between a risk factor (amalgam) and dementia. When investigating such association or linkage is the purpose, two studies can be used:

Case control studies - Compare patients who have a disease or outcome of interest (cases) with patients who do not have the disease or outcomes (controls), and looks back to compare how frequently the exposure to a risk factor is present in each
group to determine the relationship between the risk factor and the disease. These studies are designed to estimate odds.

Cohort studies - Cohort study is another method to investigate a relationship between cause and effect, but it is prospective (forward-looking), and observes the development of the condition of the interest (e.g. dementia) in cases who do not have
the disease at the beginning of the study and sees if they develop the condition in the future, and if so how relevant it can be to the cause (amalgam in this question). Unlike case control study, cohort is usually prospective and the population studied
has not the condition in the beginning of the study.

Of the two of your samples, one consists of the cases who already have dementia, and you want to look back at their history of dental fillings with amalgam to see if there is a meaningful linkage between amalgam as a risk factor and dementia as an
outcome. These features are characteristics of a case control study.

(Option A) Cross sectional study has different structure and is mostly used for prevalence studies.

(Option B) RCTs are interventional studies conducted for the assessment of effectiveness and safety of a drug, procedure or any other intervention.

(Option D) In cohort study, the target group has not the disease yet. In this study, dementia (the disease) has already established in the study group.

(Option E) Case report is an article describing and interpreting an individual case, often written in the form of a detailed story. Case series report (study) is similar to case repot with the only difference being the number of cases compared to case
report (n=1 versus n>1).

References

• http://www.ncbi.nlm.nih.gov/pmc/articles/PMC408357

• https://himmelfarb.gwu.edu/tutorials/studydesign10

• https://himmelfarb.gwu.edu/tutorials/studydesign10

• https://himmelfarb.gwu.edu/tutorials/studydesign10

• http://www.ncbi.nlm.nih.gov/pmc/articles/PMC350529

Last updated:
Time spent: QID:1055 2023-2-12

1716 of 1943
A junior doctor in the hospital had a needle stick injury while setting up a line for intravenous fluids. The patient status for hepatitis B is unknown. The doctor has no history of
previous hepatitis B vaccination. A blood sample of the doctor is sent to laboratory for hepatitis B serology which came back positive for HBsAg. Which one of the following is the
next best step in management?

A. Give immunoglobulin.

B. Give hepatitis B vaccine and immunoglobulin.

C. Counselling and follow-up.

D. Lamivudine.

E. Give hepatitis B vaccine.

Incorrect. Correct answer is C


45% answered correctly

Explanation:

Correct Answer Is C

This health care worker had a needle stick injury and blood test came back positive for hepatitis B surface antigen meaning that he has hepatitis B infection. This healthcare worker
should be counseled about the result and should have follow-up testing at 3 months and 6 months. If the blood tests remain positive for hepatitis surface antigen beyond 6 months,
the patient should be referred to specialist.

References

• Therapeutic Guidelines – Antibiotic; available on: http://tg.org.au

Last updated:
Time spent: QID:316
2023-2-12

1717 of 1943
Which one of the following is the study of choice to investigate the effect of vitamin C in treatment of influenza?

A. Cohort study.

B. Case control study.

C. Case series.

D. Randomized controlled trial (RCT).

E. Randomization.

Incorrect. Correct answer is D


45% answered correctly

Explanation:

Correct Answer Is D

When it comes to assessment of the efficacy or safety of a drug, procedure, or any other interventional treatment of any kind, RCTs or randomized double blind trials are the option of choice.

RTCs in humans are used to test the safety and potential benefit of a treatment. While the harms of a treatment sometimes prove to outweigh the benefits, this form of testing is acceptable to the participants because the investigators are aiming to
develop a new treatment and usually have a reasonable expectation of safety at least, if not a positive effect of treatment.

RCTs, especially if double blind, are the gold-standard study for medical interventions. They have become the standard for assessment of efficacy and safety of an experimental drug or method of treatment.

(Options A and B) Cohort and case control are observational studies and not used for assessment of an intervention.

(Option C) Case report (study) is an article describing and interpreting an individual case, often written in the form of a detailed story. Case series report (study) is similar to case repot (study) with the only difference being the number of cases
compared to case report (n=1 versus n>1). Case reports, as the name implies, just report a condition and are not able to define a cause.

(Option E) Randomization is a sampling method used for unbiased sample definition and selection for research studies which is a necessary element for a study to be valid and trustable. It is not a study.

References

• http://www.ncbi.nlm.nih.gov/pmc/articles/PMC350529

• http://www.ncbi.nlm.nih.gov/pmc/articles/PMC408357

• https://himmelfarb.gwu.edu/tutorials/studydesign10

• https://himmelfarb.gwu.edu/tutorials/studydesign10

• https://himmelfarb.gwu.edu/tutorials/studydesign10

Last updated:
Time spent: QID:1056 2023-2-12

1718 of 1943
Which one of the following studies should be used to assess the effect of vitamin C on the efficacy of iron tablets in treatment of iron deficiency anemia?

A. Cohort.

B. Case control study.

C. Case series study.

D. Double blind clinical trial.

E. Randomization.

Incorrect. Correct answer is D


45% answered correctly

Explanation:

Correct Answer Is D

Logically, when the effect of vitamin C on the efficacy of iron tablets is intended as the objective of this study, an interventional study should be conducted. Two groups are selected randomly from persons who are taking iron tablet in a similar
controlled way. The intervention will be the addition of vitamin C to one group only, and to see how effective vitamin can be in treatment outcomes compared to the control group. This is called randomized controlled trial (RCT).

A double blind clinical trial is an RTC, in which neither the group populations, nor the researchers are aware of who is being intervened during the trial. No one in trial groups knows if he/she is taking vitamin C, nor does the researcher. This method is
the gold standard study in clinical trials, and should be used as first-line study (if possible) for assessment of the efficacy or safety of a drug or therapy.

(Options A, B and C) Cohort, case control and case series are all observational studies. Observational are not interventional and inappropriate and irrelevant when efficacy of an intervention is the objective of the study.

(Option E) Randomization is a sampling method used for unbiased sample definition and selection for research studies which is a necessary element for a study to be valid and trustable. It is not a study.

References

• http://www.ncbi.nlm.nih.gov/pmc/articles/PMC350529

• https://himmelfarb.gwu.edu/tutorials/studydesign10

• http://www.ncbi.nlm.nih.gov/pmc/articles/PMC408357

Last updated:
Time spent: QID:1057 2023-2-12

1719 of 1943
There was an outbreak of salmonella infection in your area 3 weeks ago. As a GP you are asked to perform a study to find out the cause. Which one of the following is the most appropriate study for this purpose?

A. Case control.

B. Case series.

C. Cross sectional study.

D. Randomization.

E. Cohort study.

Correct
45% answered correctly

Explanation:

Correct Answer Is A

Case control studies are the most appropriate option to find the possible cause(s) of the disease in outbreaks.

Case-control studies are useful when epidemiologists investigate an outbreak of a disease, because the study design is powerful enough to identify the cause of the outbreak especially when the sample size is small. Attributable risks may also be
calculated.

(Option B) Case report (study) is an article describing and interpreting an individual case, often written in the form of a detailed story. Case series report (study) is similar to case repot (study) with the only difference being the number of cases
compared to case report (n=1 versus n>1). Case report, as the name implies, just reports a condition, and is not able to define a cause.

(Option C) Cross sectional studies consist of assessing a population, as represented by the study sample, at a single point in time. Cross-sectional study investigates the presence or absence of any disease or other condition in each member of the
study. They are used mostly to see how prevalent a condition is in a population.

(Option D) Randomization is a sampling method used for unbiased sample definition and selection for research studies which is a necessary element for a study to be valid and trustable. It is not a study.

(Option E) Cohort studies are used to establish linkage between a cause (e.g. risk factor) and the effect (disease) in a prospective approach. They are also excellent for determining incidence.

References

• https://onlinecourses.science.psu.edu/stat507/node

Last updated:
Time spent: QID:1059 2023-2-12

1720 of 1943
A 17-year-old high school student boy presents to your clinic with a fever and diarrhea for the past 24 hours. Evaluation, including a stool exam and culture, establishes the diagnosis of salmonella infection. You have been already informed that there
is an outbreak of salmonella in your area. Which one of the following is the most important question to ask the patient?

A. Where do you go to school?

B. What is your part time job?

C. Have you been engaged in school excursion recently?

D. Are you using public transport?

E. Where do you live?

Incorrect. Correct answer is B


45% answered correctly

Explanation:

Correct Answer Is B

Salmonella gastroenteritis is febrile diarrheal disease presenting with fever, vomiting, bloody diarrhea and abdominal pain. The disease is food-borne, and the main rout of spread is food handling. Therefore, of the questions, the most important
question to ask is if the patients job has the potentials to spread the infection to others. The risk of disease spread is particularly high in those who handle or prepare food and dishes. If this patient has a job related to food or food handling, or other
jobs that might pose risk to others, it is very important that he is excluded from work until 48 hours after the diarrhea stops. Exclusion from school until 24 hours after resolution of the diarrhea is also important.

Salmonella infection transmission occurs through oral-fecal route. Public transport is NOT a significant route of transmission if general hygienic measures, such as frequent hand washing, are considered.

Other questions are not as important as the question asking whether he has a part time job that includes food handling.

References

• NSW Health - Investigating food borne outbreaks - frequently asked questions

Last updated:
Time spent: QID:1060 2023-2-12

1721 of 1943
Which one of the following is the least reliable study to consider for studying patients with acute ascending cholangitis?

A. Randomized controlled trial.

B. Cohort study.

C. Case control study.

D. Case report.

E. Systematic review.

Incorrect. Correct answer is D


45% answered correctly

Explanation:

Correct Answer Is D

To answer this question, one should be familiar with the hierarchy of studies used for medical research. Each study provides some evidence. The more qualified evidence a specific type of study provides, the higher it is in the order of hierarchy. The
research studies used in medical research, in order of their ability to reliably provide evidence are as follows:

1. Meta-analysis
2. Systematic review
3. Practice guideline
4. Randomized controlled trial
5. Cohort study
6. Case control study
7. Cross sectional study
8. Case report
9. Individual opinion papers

Of the given options case report is weakest in providing evidence. Meta-analysis provides the strongest evidence.

References

• http://canberra.libguides.com/content.php?pid=5914

Last updated:
Time spent: QID:1061 2023-2-12

1722 of 1943
Tim is 6 years old and is brought to your GP clinic by his mother, who is concerned about him getting chickenpox after he was in contact with a child with the disease 2 days ago. He is asymptomatic now. Which one of the following is the most
appropriate step in management?

A. Varicella zoster immunoglobulin (VIZG).

B. Acyclovir.

C. Chickenpox vaccine.

D. Observe for a few days.

E. Exclude him from school.

Incorrect. Correct answer is C


45% answered correctly

Explanation:

Correct Answer Is C

If varicella-containing vaccines are not contraindicated, vaccination can be offered to non-immune, age-eligible children and adults, who have a significant exposure to varicella or herpes zoster, and wish to protect against primary infection with
varicella zoster virus (VZV).

Post-exposure vaccination is generally successful when given within 3 days (72 hours) and up to 5 days after exposure; the earlier, the better.

Live attenuated varicella vaccine (VV) is currently available in Australia as a monovalent vaccine. Two quadrivalent combination vaccines containing live attenuated measles, mumps, rubella and varicella viruses (MMRV) are also registered in
Australia. MMRV vaccine can be given to children in this setting, particularly if MMR vaccination is also indicated.

(Option A) VZIG is indicated for patients in whom prophylaxis is indicated, but vaccination is contraindicated.

(Option B) Acyclovir is the treatment of choice and indicated once the child has the clinical symptoms of varicella zoster.

(Option D) While post-exposure prophylaxis can reduce the risk of infection, just observation to see if symptoms develop is not an appropriate option.

(Option E) Exclusion from school until the blisters are dried out (usually 5 days) is indicated if the child develops chickenpox. The child is not affected yet.

References

• http://www.immunise.health.gov.au/internet/immunis

Last updated:
Time spent: QID:1092 2023-2-12

1723 of 1943
A 24-year-old woman presents concerning about the risk of breast cancer after her mother was recently diagnosed with breast cancer and was found to have BRCA1 gene mutation. She is keen to know if she should undergo genetic testing for
BRCA1 and the expenses. Which one of the following is correct in this regard?

A. She can be screened at her own expense.

B. She can be screened at her employer’s expense.

C. Screening for her is funded by the government.

D. She does not require screening.

E. She can be screened at the expense of Medicare.

Incorrect. Correct answer is C


45% answered correctly

Explanation:

Correct Answer Is C

Despite the fact that a family history of breast and/or ovarian cancer is common in women with breast or ovarian cancer, only less than 10% of all breast cancers and less than 15% of ovarian cancers are associated with inherited genetic mutations.
Mutations in BRAC1 and BRCA2 account for the majority of hereditary breast/ovarian cancers. Less common mutations are TP53 (Li-Fraumeni syndrome) and PTEN (Cowden syndrome).

Hereditary syndromes associated with BRCA1 and BRCA2 are inherited in an autosomal dominant fashion, meaning that 50% of offspring of a person with such mutations will have the faulty gene.

There are national guidelines to use when offering publically funded BRCA gene testing in Australia. It is generally accepted that when offering BRCA testing in a family for the first time, the person who is offered testing must be either affected by a
relevant cancer (usually breast or ovarian cancer) and have a more than 10% likelihood of a gene mutation, or if unaffected and have more than a 20% chance of a gene mutation.

This woman is unaffected; however, with a mother with proven BRCA1 mutation, she has a 50% of chance of having a BRCA1 gene mutation; therefore, she is eligible for BRCA gene testing at the expense of government.

BRCA1 and BRCA2 testing is not subsidized by Medicare. These tests are done through the public hospital system, funded by state health department.

The following patietns are eligibe for BRCA gene testing funded by public health system:

Breast cancer <30 years; triple negative breast cancer <40 years*, ovarian cancer (high grade serous, endometrioid or clear cell) <70 years
Males with breast cancer < 60 year would usually be eligible for testing while older men require additional family history
Other (usually eligible):

Ashkenazi Jewish heritage and either a personal or family history of breast or ovarian cancer
An affected woman with a family history of breast and ovarian cancer
Three first- or second-degree relatives with breast cancer, especially if <50 years
Bilateral breast cancer with the first cancer occurring in the 30’s or 40’s
Affected patients whose family history scores >16 on the Manchester risk score

TOPIC REVIEW

The following screening strategy is recommended by expert groups for women with hereditary breast/ovarian cancer (HBOC) syndrome, who have not undergone risk-reducing surgery (i.e., bilateral mastectomy, oophorectomy):

Monthly breast self-examination beginning at age 18

Clinical breast examination 2 to 4 times annually beginning at age 25

Annual mammography and breast magnetic resonance imaging (MRI) screening (commonly alternated every 6 months) beginning at age 25 or individualized based on the earliest age of onset in the family

Twice yearly ovarian cancer screening with transvaginal ultrasound and serum CA-125 levels (preferably drawn on day 1 to 10 of menstrual cycle for premenopausal women) beginning at age 35, or 5 to 10 years earlier than the earliest
age of first diagnosis of ovarian cancer in the family

NOTE - The lack of efficacy of ovarian cancer screening has prompted many clinicians to recommend risk-reducing bilateral salpingo-oophorectomy at the completion of childbearing rather than intensified screening for ovarian cancer.

*Any breast cancer than does not express the genes for estrogen receptor, progesterone receptor (PR) or Her2/neu

References

• Cancer Australia - Genetic testing for breast/ovarian cancer risk

• Nature Reviews - Clinical Oncology: Review Article Published: 20 October 2015 Population genetic testing for cancer susceptibility: founder mutations to genomes

• UpToDate - Cancer risks and management of BRCA1/2 carriers without cancer

Last updated:
Time spent: QID:1093 2023-2-12

1724 of 1943
While establishing an intravenous live, a 26-year-old nurse sustains a needle stick injury from a patient who is known to be HBsAg positive. She is 15 weeks pregnant. Serologic tests of nurse shows both HBsAg and HBsAb are negative. Which one of
the following is the most appropriate management for her?

A. Reassure her as she is HBsAg negative.

B. A full course of hepatitis B vaccine.

C. Hepatitis B immunoglobulin (HBIG).

D. A full course of hepatitis B vaccine and HBIG.

E. Repeat HBsAg and HBsAb in 3 months.

Incorrect. Correct answer is D


45% answered correctly

Explanation:

Correct Answer Is D

Post-exposure prophylaxis for hepatitis B during pregnancy is the same for non-pregnant persons and follows the same general guidelines.

After exposure to hepatitis B, serology testing for the person who has had the exposure should be performed urgently. A negative HBsAg and a HBsAb titer of >10IU/mL indicates sufficient immunity against hepatitis B, and no further action is
required.

With HBsAb titers <10 (and HBsAg negative), the person has not enough immunity, and should receive both hepatitis B vaccine and HBIG within 72 hours of exposure, ideally within 24 hours. Vaccine should also be given at 1 and 6 months after the
first dose (0, 1 and 6 months). Serology testing should be repeated in 3 months. If the mother is found to be HBsAg positive despite prophylaxis, the baby should receive vaccine and HBIG within 12 hours of birth.

References

• https://www.asid.net.au/documents/item/368

• Therapeutic Guideline – Antibiotics; available from: http://tg.org.au

Last updated:
Time spent: QID:1097 2023-2-12

1725 of 1943
A pharmaceutical company receives approval from the ethical committee to start trial of a new antihypertensive drug. The ethical committee sets a target P-value of ≤0.02. At the end of the trial, the company claims that the efficacy of their new drug
is superior to other available drugs in the market. The computed P-value of the trial is 0.04. What does the P value of 0.04 indicates in this trial?

A. The new drug is safer than other available drugs.

B. The new drug is as useful as other available drugs but has fewer side effects.

C. The new drug is superior to other available drugs.

D. The new drug is not superior to other available drugs.

E. The new drug is inferior to other available drugs.

Incorrect. Correct answer is D


45% answered correctly

Explanation:

Correct Answer Is D

A P-value is used to interpret output from a statistical test (a clinical trial here). P-value is used to control if the new results are statistically different from previous trials. A standard P-value is set in the beginning of the trial. After the study is
concluded, the P-value of the study is computed and compared with the standard P-value:

If the computed P-value from the study is ≤ standard P-value then the results has statistical significance.
If the computed P-value from the study is > standard P-value then the results has not reached statistical significance.

For this study, a standard P-value of ≤0.02 has been set by the ethical committee, meaning that if the computed P-value from the study is ≤0.02 the new drug has statistical significance and can be claimed to be superior to other available drugs. With
a computed P-value of >2, this new drug cannot be told to be superior to other available drugs.

At the end of the trial, this new drug has a computed P-value of 0.04 that is > 0.02. This means that the new drug has no superiority over the available drugs.

(Options A and B) P-value does not indicate adverse effects. It can only determine if a new drug has superiority over others or not.

(Option C) With a P-value> standard P-value, this drug cannot be claimed to be superior to other available drugs.

(Option E) P-value predicts if a drug can be superior or not superior to other drugs. Inferiority cannot be predicted by P-value.

References

• https://onlinecourses.science.psu.edu/statprogram/

Last updated:
Time spent: QID:1129 2023-2-12

1726 of 1943
Which one the following test is a mass population screening test which has been demonstrated to reduce cancer mortality significantly?

A. Annual fecal occult blood testing in people over the age of 50 years.

B. Annual colposcopy in sexually active women.

C. Annual plasma CA125 in post menopausal women.

D. Annual colonoscopy in siblings of patients with colon cancer.

E. 2-yearly mammography in women aged 35-45 years.

Correct
45% answered correctly

Explanation:

Correct Answer Is A

There have been at least three randomized control trials conducted which show that 1- to 2-yealy fecal occult blood testing (FOBT) in the population over the age of 50 years reduces the mortality from colorectal cancer by approximately 20%.

(Option B) Colposcopy is a diagnostic, not a screening test. The appropriate screening test for cancer of the cervix in sexually active women is the Papanicolaou (Pap) smear.

(Option C) Plasma CA125 levels may be raised in asymptomatic women with ovarian cancer but there is not any current evidence to support it as a mass screening test.

(Option D) Colonoscopy for siblings of patients with colorectal cancer may reduce their mortality, but this is a 'selective' screening targeting a high-risk group. It is not a mass screening.

(Option E) While mammography in the 50-70 age group has been shown to reduce mortality from breast cancer by up to 30%, the benefit for women between 40-50 years is small. This benefit is even smaller in younger women.

NOTE – Regular pap smear has been shown to reduce the incidence and mortality of cervical cancer by up to 80% and would be the correct answer if it was an option.

References

• National Cancer Institute - Colorectal Cancer Screening (PDQ®)–Health Professional Version

• National Cancer Institute - Cervical Cancer Screening (PDQ®)–Health Professional Version

• National Cancer Institute - Breast Cancer Screening (PDQ®)–Health Professional Version

• Current Oncology - Impact of screening mammography on mortality from breast cancer before age 60 in women 40 to 49 years of age

Last updated:
Time spent: QID:1295 2023-2-12

1727 of 1943
A young female intern accidentally pricked herself with the needle while taking blood sample from an HIV positive patient. Which one of the following is the most appropriate
action?

A. Give antiretroviral drugs for 4 weeks.

B. Test both the patient and doctor for HIV viral load now.

C. Do not allow the doctor to draw blood for 3 months.

D. Give her intravenous immunoglobulin.

E. Tell her that the risk of disease transmission is very low.

Correct
45% answered correctly

Explanation:

Correct Answer Is A

I​n the event of needle stick injury and exposure to an HIV positive person, post-exposure HIV prophylaxis is recommended. Provision of post exposure prophylaxis should not be
delayed while establishing the HIV status of the source. Early initiation of post-exposure prophylaxis (PEP), as soon as possible after exposure, is strongly recommended. PEP
should not be delayed more than 72 hours post-exposure. A 28-day (4-week) course of PEP is recommended.

This health care worker should have follow-up HIV antibody testing at baseline, 6 weeks, 3 months, and 6 months, along with tests for other blood-borne viruses.

Recommendations for occupational post-exposure prophylaxis include a basic regimen of two nucleoside/nucleotide reverse transcriptase inhibitors for most HIV exposures, and
an expanded regimen with the addition of a third drug when the exposure poses an increased risk for transmission.

References

• http://www.health.qld.gov.au/qhpolicy/docs/gdl/qh-

• http://www.som.uq.edu.au/media/418890/national_gui

Last updated:
Time spent: QID:317
2023-2-12

1728 of 1943
In a randomized controlled trial (RCT) conducted to study the effect of aspirin on prevention of coronary artery events among diabetic smokers, the results in the two arms of the study are as follows:

ASA group Placebo group


Coronary event 1 2
No coronary event 99 98

Which one of the following is the relative risk of not using ASA?

A. 1%.

B. 2%.

C. 100%.

D. 200%.

E. 50%.

Incorrect. Correct answer is D


45% answered correctly

Explanation:

Correct Answer Is D

In statistics and epidemiology, relative risk or risk ratio (RR is the ratio of the probability of an event occurring in an exposed group to the probability of the event occurring in a non-exposed, comparison group.

Of 100 diabetic smokers who are on aspirin, 1 person has developed a coronary event. So the incidence of coronary event in this group is 1% [1/(1+99)x100], while the incidence of coronary events in the group taking placebo instead of aspirin is 2%
[2/(2+98)x100].

In this scenario the exposure is taking aspirin. Exposed group has a 1% chance of developing a coronary event versus 2% in those who do not take aspirin.

The RR is then calculated by dividing the odds of the condition in the exposed group (1%) by that of the non-exposed group:

RR= P(exposed) / P(non-exposed) : RR=1% / 2%=0.5

Here, the RR indicates that the odds of developing a coronary event in those diabetic smokers who are on aspirin is half compared to those on placebo. In other words, those who are on aspirin has a 50% risk reduction. Inversely, those who are not
taking aspirin are twice as likely to develop a coronary event compared to those who are taking it. So the RR. This means that not taking aspirin is associated with a 200% increase in incidence of coronary events.

References

• http://www.wikihow.com/Calculate-Relative-Risk

Last updated:
Time spent: QID:1299 2023-2-12

1729 of 1943
Marian, 55 years old, is in your office for consultation. She is obese and has a BMI of 33, waist circumference of 125 and hip circumference of 110cm. Laboratory studies you ordered before show an LDL of 3mmol/L (normal <0.2 mmol/L) and HDL of
0.7 (normal:0.9-1.93 mmol/L). Which one of her physical or biochemical markers is the most important predictor of her health risk?

A. BMI.

B. Waist-to-hip ratio.

C. Waist circumference.

D. High LDL.

E. Low HDL.

Incorrect. Correct answer is B


45% answered correctly

Explanation:

Correct Answer Is B

Waist-to-hip ratio (WHR) has been suggested as the preferred measure of obesity for predicting cardiovascular disease, with more universal application in individuals and population groups of different body builds. This parameter reflects abdominal
(central) fat which is strongly associated with ischemic heart disease, hypertension and type II diabetes mellitus. In terms of predicting obesity-related mortality, WHR is more reliable than BMI and waist circumference together. Waist circumference
alone comes next and BMI alone last.

(Option A) BMI is advocated by World Health Organization (WHO) as the epidemiological measure of obesity; nevertheless, BMI is a crude index that does not take into account the distribution of body fat, resulting in variability in different individuals
and populations. For example, individuals with the same BMI may have different ratios of body fat to lean mass. A muscular athlete may have the same BMI of a less muscular person. Women have more body fat than men at equal BMIs and people
lose lean tissue with age so an older person will have more body fat than a younger one with same BMI.

(Option C) Waist circumference has been recommended as a simple and practical measure for identifying overweight and obese patients, but it does not take into account body size and height.

High LDL (option D) and low HDL (option E) are associated with increased cardiovascular disease risk, but these two are not strong predictors per se. For example they are not strong predictors for development of hypertension or insulin resistance.

NOTE – for each 1 cm increase in waist circumference, there is an increase of 2% in the relative risk (RR) for a cardiovascular event. For a 0.01 unit increase in waist-to-hip ratio, this relative risk is 5%.

References

• https://www.nhmrc.gov.au/_files_nhmrc/publications/attachments/n57_obesity_guidelines_140630.pdf

• https://academic.oup.com/eurheartj/article/28/7/850/2887789

Last updated:
Time spent: QID:1314 2023-2-12

1730 of 1943
A 36-year-old man presents to a general practice for advice about vaccination before he goes on a business trip to Cambodia. His past medical history is significant for splenectomy two years ago after he sustained a car accident and had his spleen
severely injured. To which one of the following infections would he be most susceptible during this trip?

A. Dengue fever.

B. Hepatitis B.

C. Hepatitis A.

D. Malaria.

E. Traveler’s diarrhea.

Incorrect. Correct answer is D


45% answered correctly

Explanation:

Correct Answer Is D

Patients who are asplenic (no spleen) or hyposplenic (for example, decreased function of spleen due to ongoing microinfarcts caused by sickle cell disease) are at increased risk of severe septicemia with Streptococcus pneumoniae (S.pneumoniae),
Hemophilus influenza type b (Hib), Neisseria meningitidis, and Capnocytophaga canimorsus (acquired by dog or cat bites). Infection with these organisms in asplenic (or hyposplenic) individuals is called overwhelming post-splenectomy infection
(OPSI). Risk of OPSI is highest in the first two years of splenectomy.

S. pneumoniae accounts for approximately 50% of patients. Therefore, it is recommended that all patients receive pneumococcal 23 valent polysaccharide vaccine PPV23 (Pneumovax 23). It is recommended that the vaccine be given at least 2
weeks before elective splenectomy or, in case of emergency splenectomy, after 7 days but not later than 14 days post-splenectomy (or at discharge). A booster dose is required at 5 years. It is also recommended that such patients receive
prophylactic antibiotic (e.g. amoxicillin 250mg/day) until two years after splenectomy.

Other recommended vaccines are Neisseria meningitidis, Hib, and annual influenza vaccine. The reason for recommending influenza vaccine even though it is not among OPSI, is that influenza infection can result in S. pneumoniae being
superimposed.

Another important issue to consider for such patients is travel advice and prophylaxis. Although all the given options can pose a risk to the patient’s health during his visit to Cambodia, malaria is the most serious condition to take care of. People
with asplenia or hyposplenia are at increased risk of severe malaria when traveling to endemic areas. He should be advised for prevention when travelling to a malaria endemic region such as Cambodia. This advice includes vector avoidance (using
protective measures such as wearing long sleeved clothing, sleeping in closed and protected areas, and insect repellent), antimalarial medications, and seeking immediate medical advice if there is any symptom.

Dengue fever (option A) is endemic in Cambodia and the patient should be advised regarding it; however, health risks caused by this viral infection is almost the same as patients with intact spleen. This is true about hepatitis A (option C), hepatitis B
(option B) and traveler’s diarrhea (option E).

References

• RACGP – Post-splenectomy infection

Last updated:
Time spent: QID:1449 2023-2-12

1731 of 1943
Daniel, 52 years old, is in your office with complaint of excessive daytime sleepiness for the past six months. He says he has never felt his night sleeps refreshing enough, and he takes every opportunity to take a nap during the day. On further
questioning, he admits to snoring and that some nights his wife leaves the bedroom to sleep on the couch in the living room because his snoring does not let her sleep, and in fact, she has made him to see you for this problem. He denies smoking or
any other significant medical condition. On examination, his blood pressure is 150/88 mmHg, pulse rate of 80 bpm and respiratory rate of 17 breaths per minute. He has a BMI of 37 and waist circumference of 135 cm. The rest of the examination is
inconclusive. You suspect sleep apnea as the diagnosis, and arrange for him to see a sleep specialist. You advise him not to drive until a full assessment is undertaken by the specialist, including sleep studies. He becomes upset and says he cannot
stop driving as this is his job. Which one of the following would be the most appropriate next step to take in this situation?

A. Explain to him about the serious risks he could pose to himself and others if he keeps driving while he is untreated.

B. Ask him to report his condition to the relevant driver licensing authority.

C. Inform the driver licensing authority (DLA).

D. Ask him to submit his driver's license to you now.

E. Advise weight reduction to alleviate the condition and decrease the risk of accidents.

Correct
45% answered correctly

Explanation:

Correct Answer Is A

Assessment of fitness to drive is one of the most challenging situations in medical practice. On one side is the patient’s important issues such as independence, and in this case financial problems Daniel will face if he is prevented from driving as a
commercial driver, and on the other side, lies health and safety of the public. A medical practitioner should be able to balance between their duty to the patient and the duty to act in public’s interest.

In case a patient is likely to pose risk to health and wellbeing of others, the following measures should be taken:

STEP 1 - most importantly, it should become clear to the patient that driving under their medical condition is dangerous both for him/her and the public. Many patients are likely to choose not to drive when the condition is explained to them in an
empathetic and reasonable manner.

STEP 2 - the patient should become aware of his/her civil responsibility to self-report the medical condition to the driver licensing authorities. This discussion and the given advice should be documented in the patient’s notes.

STEP 3 - reporting to the licensing authorities if the patient:

is unable to understand the impact of their condition; OR


is unable to take notice of the health professional’s recommendations due to cognitive impairment; OR
continues driving despite appropriate advice and is likely to endanger the public

Daniel has the provisional diagnosis of sleep apnea (or other sleep disorders) that has resulted in daytime sleepiness. Driving while drowsy or sleepy poses a significant risk to his or others’ health. This should be explained to him as the first step in
management. He should also be asked to self-report to driving licensing authorities (option B). If you find out that Daniel keeps driving despite the advice, you can breach the confidentiality and personally inform the authorities (option C).

(Option D) asking a patient to surrender his/her driver’s license is not a task of doctors. Doctors are not in a position, legally or morally, to make such a request.

(Option E) Weight reduction will benefit Daniel not only for the sleep apnea but also in terms of general health and should be advises; however, it is a long-term management with future results and not likely to resolve his daytime sleepiness in short
term.

NOTE - The take-home message of this question is that talking to the patient and convincing them to stop driving voluntarily and follow your recommendations is always the most important first step to take in cases of unfitness to drive.

References

• Austroads - Assessing Fitness to Drive

Last updated:
Time spent: QID:1453 2023-2-12

1732 of 1943
Peter, 33 years of age, is concerned about colon cancer because his father had it at the age of 49 and his elder brother at the age of 46 years. He denies any symptoms such as rectal bleeding, altered bowel habits or those related to anemia. Which
one of the following would be the most appropriate advice for him regarding colon cancer screening?

A. Screening with FOBT 2-yearly from the age of 50 years.

B. Screening with FOBT 2-yearly from the age 40 years.

C. Screening with yearly FOBT starting from now.

D. Screening with colonoscopy 5-yearly from the age of 50 years.

E. Screening with colonoscopy 5-yearly starting from now.

Incorrect. Correct answer is B


45% answered correctly

Explanation:

Correct Answer Is B

Recommendations for colorectal cancer screening is based on individual risk. The risk categories, criteria, and recommended screening program for colorectal cancer currently in use in Australia (recently updated) is outlined in the following table:

Category 1 – average or slightly increased risk: (Relative risk: 1-2); 95-98% of population
Asymptomatic people with:

no personal history of bowel cancer, colorectal adenomas, Fecal occult blood test (FOBT) every 2 years beginning from the age of 50 years.
inflammatory bowel disease or family history of colorectal
cancer

OR

one first-degree or one second-degree relative with


colorectal cancer diagnosed aged ≥55 years

NOTE – fore those with a family history of colorectal cancer, as mentioned for this category, it is recommended that 2-yearly FOBT
starts from the age of 45 years.
Category 2 – Moderately increased risk: (relative risk: 3-6); 2–5% of the population
Asymptomatic people with:

one first-degree relative with colorectal cancer diagnosed


aged <55 years

OR

two first-degree relatives with colorectal cancer diagnosed


at any age
FOBT every 2 years from 40 to 49 years of age.

OR
AND

one first-degree relative AND at least two second-degree


relatives diagnosed with colorectal cancer at any age Colonoscopy every 5 years from 50 to 74 years of age.

Category 3 – High risk : (relative risk: 7-10); <1% of the population


Asymptomatic people with:

at least three first-degree or second‑degree relatives FOBT every two years from 35 to 44 years of age
diagnosed with colorectal cancer at aby age.

AND

Colonoscopy every five years from 45 to 74 years of age

Peter has two first-degree relatives diagnosed with colorectal cancer. With this in history, he falls in category 2, and requires FOBT testing 2-yearly from the age of 45 years until 49, and colonoscopy every 5 year from the age of 50 years.

References

• Australian Journal of General Practice (AJGP) – Colorectal cancer screening in Australia: An update

Last updated:
Time spent: QID:1457 2023-2-12

1733 of 1943
Ted, 63 years old, is a regular smoker patient of your clinic. Last year, he was diagnosed with chronic obstructive pulmonary disease (COPD) and was advised to give up smoking. On every visit, you have briefly advised him about smoking cessation
as the best step thing he can do for his COPD and his health in general, but he did not seem interested. Today he says he has decided to quit and asks for help. He confides in you that he has tried three times before but to no avail because every time
he tried to quit, he had severe craving and agitation, so he had to smoke again. He is frustrated and believes there is no way for him to get rid of it. After consulting him regarding his good decision and that there are ways you can help him, you decide
to start him on nicotine replacement therapy. Which one of the following is the most important factor in the history supporting such decision?

A. His failed previous attempts.

B. Withdrawal symptoms.

C. His COPD.

D. His frustration with his failed attempts.

E. Nicotine replacement therapy is recommended for all patients attempting to quit smoking.

Incorrect. Correct answer is B


45% answered correctly

Explanation:

Correct Answer Is B

The five important steps in consulting every patient with drug, alcohol or smoking problems are known as the 5 'A's (ask, assess, advice, assist and arrange).

One important part is assessing the degree of dependence to that drug. For smokers trying to quit, assessment of dependence is a key step in management because it determines whether pharmacotherapy (nicotine, bupropion, or varenicline
[Champix®]) or a combination of these is required assist the patient in quitting smoking.

While patients who are not nicotine dependent are managed non-pharmacologically by counselling, cognitive and behavioral coping strategies, written information (e.g. Quit Pack), those with dependence need pharmacotherapy to increase the
chances of successful quit attempt.

In assessment of dependence, the following questions should be asked:

How many minutes after waking to you smoke your first cigarette?
How many cigarettes do you smoke a day?
Have you had cravings or withdrawal symptoms in previous quit attempts?

The following strongly indicate nicotine dependence and the need for pharmacotherapy:

Smoking with 30 minutes of waking.


Smoking more than 10 cigarettes a day.
Craving or withdrawal symptoms in previous attempt.

Based on the above, Ted requires pharmacotherapy because he has had craving and agitation (a symptom of nicotine withdrawal) in previous attempts, indicating nicotine dependence. Other options are bupropion or varenicline that could be added
to nicotine based on his condition and preferences.

(Option A) failed previous attempts alone do not justify use of pharmacotherapy for Ted. Such failure may have been caused by social and psychological factors, and not necessarily nicotine dependence. Unless there is nicotine dependence, one can
still make another attempt to quit without pharmacotherapy.

A medical condition (COPD) (option C) does not justify the use of pharmacological therapy for a patient if there is no nicotine dependence, neither does frustration with previous failed attempts (option D) unless withdrawal symptoms have caused
such failure.

(Option E) Nicotine replacement therapy is only indicated for patients with nicotine dependence. Not all smokers are nicotine dependent.

References

• RACGP – Supporting smoking cessation: A guide for health professionals

Last updated:
Time spent: QID:1461 2023-2-12

1734 of 1943
Patricia is a 28-year-old sex worker, working in a licensed brothel. She is in your office for sexual health assessment and receiving a sexual health certificate. In consulting her, which one of the following is the correct advice?

A. She needs to perform vaginal and anal swabs every 12 months.

B. She should be screened for chlamydia and gonorrhea using a mid-stream urine every 3 months.

C. She should be screened for hepatitis C infection every 12 months.

D. She should be screened for chlamydia and gonorrhea using high vaginal swabs every 3 months.

E. She should be screened for syphilis every 12 months.

Incorrect. Correct answer is D


45% answered correctly

Explanation:

Correct Answer Is D

This question asks about requirements by law in order to renew a license for sex work practice. Requirements for such issue is different from current guidelines by the RACGP and STI guidelines in Australia. Based on clinical guidelines, it is
recommended that high risk individuals for STIs (such as sex workers) undergo screening every 12 months; however, these guidelines advise that state legislations should be followed regarding the intervals of such tests for legal matters such as
issuance or renewal of a sexual health certificate for sex workers. Based on current legislations in Australia, sex workers require to be screened for STIs quarterly (every 3 months) to have their licenses renewed.

Legislations for STI screening are in accordance with the Public Health and Wellbeing Act 2008. Based on this act, sex workers are required to undergo the following tests and measures every 3 months:

Blood tests:

Syphilis Antibody
HIV Antigen/Antibody
Hepatitis B core antibody and surface antibody (HBcAb and HBsAb)
Hepatitis A antibody (where appropriate)

Swabs (Women):

Chlamydia (High Vaginal Swab)


Gonorrhea (High Vaginal Swab)
Trichomoniasis (High Vaginal Swab) (only at first presentation or if the woman is a contact of infection)
Cervical Screening when required as per policy
Gonorrhea (pharyngeal)
Chlamydia (pharyngeal)

Genital Examination (Women):

Vulval examination is performed at each visit to exclude visible lesions of genital wart or genital herpes
A speculum examination is not routinely performed on asymptomatic women unless they require a cervical screening
A speculum examination should be considered if the woman is symptomatic, has experienced a condom break/slip, or they have a retained sponge/tampon/condom.

Gonorrhea and chlamydia testing for Male Sex Workers:

As per the policy - Screening of Asymptomatic Men for Sexually Transmitted Infections by Sexual Health Nurses.
Men must also undergo a genital examination to exclude visible lesions of genital warts and genital herpes.

Provision of vaccinations:

vaccinations should be advised, when appropriate against:

Hepatitis B
Hepatitis A

Based on current legislation, the only correct option would be high vaginal swabs for chlamydia and gonorrhea testing using Nucleic acid amplification test (NAAT) every 3 months.

(Option A) Vaginal swabs should be performed every 3 months for renewal of sex work certificate. Anal swabs are required only if there has been anal sex.

(Option B) If instead of high vaginal swabs, a urine sample is considered, the specimen used should be a first catch urine, not midstream urine.

(Option C) Screening for hepatitis C is not routinely required unless there is a high risk for such infection, e.g., injection drug users.

(Option E) Although testing for syphilis is required in sex workers, the interval proposed by law is 3 months, not 12 months.

TOPIC REVIEW

The following is the recommendation by the RACGP regarding STI screening:

Risk assessment of asymptomatic sexually active person What should be done? How often?
Low–average risk: Urine, cervical or genital swab polymerase chain reaction Opportunistically if indicated (evidence is unclear on
(PCR; or self-collected) for chlamydia testing interval)

Heterosexual asymptomatic up to 29 years of age


requesting sexually transmissible infection (STI) check up Consider other infections based on risk assessment
Medium–high risk: As above Opportunistically if indicated (evidence is unclear on
testing interval)

1735 of 1943
<20 years of age Consider other infections, particularly gonorrhea and
Rural and remote syphilis, based on risk assessment

Higher risk: Testing for chlamydia, gonorrhea, syphilis Every 12 months (evidence is unclear on testing interval)

Aboriginal or Torres Strait Islander peoples Serology for human immunodeficiency virus (HIV), syphilis
and, if the person is not vaccinated or immune, hepatitis A
and B

Offer hepatitis A and B vaccination


Other higher risk: Testing for chlamydia, gonorrhea, syphilis; Serology for HIV, Every 12 months (evidence is unclear on testing interval)
syphilis; if the person is not vaccinated or immune,
hepatitis A and B
People who inject drugs
Sex workers
Offer hepatitis A and B vaccination

Hepatitis C testing if the patient injects drugs


Highest risk: Urine, throat and rectal swab for chlamydia PCR Every 12 months; every 3-6 months in higher risk men

Asymptomatic men who have sex with men Throat and rectal swab for gonorrhea PCR

Highest risk in those who: Serology for HIV, syphilis and, if the person is not
vaccinated or immune, hepatitis A and B

have unprotected anal sex


had >10 partners in past six months Offer hepatitis A and B vaccinations
participate in group sex or use recreational
drugs during sex

References

• Melbourne Sexual Health Centre (MSHC) - Screening of sex workers for STIs and BBV: Policy Procedure

• RACGP – The Red Book: Sexually transmissible infections

• Australian STI management guidelines – Sex worker testing advice

Last updated:
Time spent: QID:1465 2023-2-12

1736 of 1943
John is 72 years old and a regular patient of yours in your GP clinic. He has depression and has been treated for that a few times. He also smokes 15-20 cigarettes a day and drinks alcohol in moderation. He is in the clinic today and wants to quit
smoking. Which one of the following would be the most important option to consider for him to assist his quitting?

A. Cognitive behavioral therapy (CBT).

B. Nicotine replacement therapy (NRT).

C. Varenicline (Champix®).

D. Bupropion.

E. Coping skills and lifestyle counselling.

Incorrect. Correct answer is B


45% answered correctly

Explanation:

Correct Answer Is B

There is a strong association between smoking and depression. Patients with current or past history of depression are approximately twice as likely to be current smokers and smoke more cigarettes per day than those who are not depressed.
Quitting is a health priority for smokers with depression because they are at higher risks of smoking related diseases than the general population of smokers.

Compared to the general population of smokers, the chances for successful quitting are lower and relapse rates are higher in depressed smokers. Depressed smokers are more nicotine dependent and usually experience more severe withdrawal
symptoms.

First-line pharmacological treatments for quitting smoking are NRT, bupropion, and varenicline. All these treatments can be used for depressed smokers as well with about the same efficacy. Varenicline (option C) is more effective than bupropion
(option D) for quitting, but the additional antidepressant action of bupropion may be beneficial in some cases. However, NRT would be the mainstay of treatment for John because of its safety profile and the fact that depressed smokers are more
nicotine dependent. On the other hand, nicotine has some antidepressant effects as well. Varenicline or bupropion, if given along with NRT can provide additional benefits. The combination of a nicotine patch with a quick acting form of nicotine such
as gum or lozenge and precessation use of nicotine patches are likely to increase success rates further.

CBT (option A) is a good option to address the depression and has been shown to be of a small positive effect on quitting. The effect is greater for patients with recurrent episode of depression. CBT can be considered along with NRT for John but is
not the most important arm of treatment for him.

Coping skills and lifestyle counselling (e.g. exercise, diet, sleep, pleasurable activities) (option E) as well as other psychological strategies such as problem solving, stress management, mindfulness, distraction have been applied and tested for their
efficacy but with mixed results and uncertain efficacy. Such measures can be recommended for their overall benefits but not for a definite result for smoking cessation.

References

• RACGP - AFP: Smoking and depression

Last updated:
Time spent: QID:1476 2023-2-12

1737 of 1943
A 25-year-old woman presents with a history of greenish-yellow vaginal discharge. Testing for sexually transmissible infections (STIs) is positive for chlamydia infection. The woman has had multiple sex partners in the past six months. Which one of
the following would be the most appropriate advice for her in terms of priority?

A. Trace all her sexual contacts in the past six months and treat them.

B. Ask her to bring her most recent sexual contacts for testing and treatment.

C. Ask her to practice safe sex.

D. Advise that she should not use intrauterine contraception devices until there is no infection.

E. Tell that she should report her infection to the relevant health authorities.

Correct
45% answered correctly

Explanation:

Correct Answer Is A

Chlamydia infection is a communicable disease and contact tracing is mandatory. For Chlamydia infection, it is recommended that sexual contacts of the index case in the past six months be traced and treated if infected.

Initiation of contact tracing is the responsibility of the diagnosing physician. Contact tracing starts with a conversation with the patient about informing their partners. The patient can decide to inform their own contacts (patient referral) or organize
for someone else to inform them (provider referral). Patient referral is the most common type of contact tracing used in general practice. For this type of contact tracing to be successful, it is important that the diagnosing doctor informs the patient
about who needs to be informed and what information needs to be given. If the patient decides to use provider referral, the diagnosing doctor can collect the contacts’ details and either notify the contacts directly or pass the details to a practice
nurse or a sexual health clinic who can undertake this.

NOTE - Currently in Victoria, reporting Chlamydia infection is the responsibilty of the testing laboratories, not the doctor.

(Option B) This patient’s sexual contacts in the past six months must be traced. Only testing and treating her most recent contacts is inadequate and inappropriate.

(Option C) Asking the patient to practice safe sex is an essential part of consultation but not the most appropriate option here. This woman should avoid sex until the infection is treated. Her current partner(s) should also be treated to reduce the risk
of re-infection.

(Option D) Intrauterine devices are contraindicated in the presence of pelvic infection. This patient should be advised not to use intrauterine devices while she has the infection but contact tracing should take precedence in priority.

(Option E) Chlamydia infection is a notifiable disease, and notifying such diseases is the diagnosing physician’s responsibility not that of the patient.

References

• STI Management Guidelines

• RACGP – AFP – Contact tracing for STIs

Last updated:
Time spent: QID:1522 2023-2-12

1738 of 1943
A 31-year-old woman, sex worker by profession, presents to your practice for a regular check up. Testing for sexually transmissible infections (STIs) shows she has gonorrhea infection. In addition to treating her for gonorrhea, which one of the
following would be the next best action?

A. Trace her sexual contacts in the past 12 months and treat them.

B. Trace her sexual contacts in the past 6 months and treat them.

C. Ask her to avoid sex.

D. Inform the Health Department.

E. Tell that she should tell her clients to use male condoms.

Incorrect. Correct answer is D


45% answered correctly

Explanation:

Correct Answer Is D

Gonorrhea infection is a notifiable disease and confirmed cases should be reported to the relevant health authorities (health department). A confirmed case requires laboratory definitive evidence only. Confirmation may be through isolation
of Neisseria gonorrhea, or detection of Neisseria gonorrhoeae by nucleic acid testing, or detection of typical Gram-negative intracellular diplococci in a smear from a genital tract specimen.

This woman has confirmed gonorrhea infection and informing the health department is the most appropriate of the options.

(Options A and B) Contact tracing of this woman’s sexual contacts in the past two months is another important step. The patient can decide to inform their own contacts (patient referral) or organize for someone else to inform them (provider
referral). Often, a sex worker is reluctant to disclose the name of his/her clients and provider referral is the method of choice to consider. However, tracing her contacts for the past 6 or 12 months is unnecessary and inappropriate.

(Option C) Sex workers should not practice sex until their current STI is adequately treated. In case of HIV infection, they cannot work as a sex worker anymore. Gonorrhea is a curable disease. She should avoid sex as long as she is infected and
undergoing treatment but asking to avoid sex forever is not appropriate.

(Option E) Advising male condoms is important but not a priority because protected sex is almost practiced universally in sex industry. Re-emphasizing is a good idea; however, as long as this woman is under treatment for her current infection and
within exclusion period, no sex of any kind must be practiced because although male condoms reduce the risk of infection transmission to or re-infection from male partners, it does not eliminate the risk.

References

• The Department of Health - Australian national notifiable diseases and case definitions

• Health VIC - Notifications procedures for infectious diseases

• MSHC - Screening of sex workers for STIs and BBV: Policy Procedure

• STI Management Guideline – Gonorrhoea

Last updated:
Time spent: QID:1524 2023-2-12

1739 of 1943
A nurse at your clinic sustains a needle stick injury at workplace. She was previously vaccinated for hepatitis B. Her anti HBs antibody is measured and is below 10 mIU/ml. The
patient is known to be HBsAg positive. Which one of the following is the most appropriate next step?

A. Hepatitis B vaccine.

B. Hepatitis B vaccine and immunoglobulin.

C. Hepatitis B serology.

D. Reassure and follow up.

E. No prophylaxis is needed.

Incorrect. Correct answer is B


45% answered correctly

Explanation:

Correct Answer Is B

Post-exposure prophylaxis for hepatitis B (hepatitis B vaccine and immunoglobulin) is recommended in the following situations:

The exposed person has been vaccinated in the past and post-vaccination anti-HBs level is below 10mIU/ml
Unvaccinated exposed individuals
Unknown vaccination status of the exposed person

In previously vaccinated individuals, who have been exposed to either an HBsAg-positive source or a source whose hepatitis B status cannot be determined, post-exposure
prophylaxis is not necessary if there is a documented protective response (anti-HBs level ≥10 mIU/mL) at any time after vaccination.

References

• http://www.health.qld.gov.au/qhpolicy/docs/gdl/qh-

Last updated:
Time spent: QID:318
2023-2-12

1740 of 1943
Steven is a 67-year-old patient of yours, who has presented a year and a half ago for a health review before renewal of his driver’s license. Your examination revealed 6/24 vision for his both eyes; therefore, you advised that he should not drive
anymore and encouraged him to self-report to licensing authorities and is referred for assessment by an ophthalmologist. Today, he has presented again and asks if you could change your mind and give him a letter than he can drive. You examine
him again and realize he still has poor eye sight and is unfit to drive. When you tell him so, he becomes irritated and says that last year he saw another doctor who give him a letter and he has been driving since then, and why you cannot just go on
and give him a similar letter as well. Which one of the following is the most appropriate action now that you are aware of him driving despite his unfitness to drive?

A. Call the police.

B. Call the doctor who issued the letter and ask him why he has done so.

C. Report the patient to the relevant Road and Traffic department.

D. Ask him to drive only if there is someone else in the car to supervise him.

E. It is not your duty to report him to anyone. Just tell him again that he is unfit to drive and has to submit his license to authorities.

Incorrect. Correct answer is E


45% answered correctly

Explanation:

Correct Answer Is E

According to Australian Road Safety (Austroads), a person is not fit to hold a driver licence if:

The corrected visual acuity (with glasses or contact lenses) is less than 6/12 for the better eye for private vehicle drivers.
The corrected visual acuity is less than 6/18 for the worse eye and less than 6/9 for the better eye.

This patient has a visual acuity of 6/24 both eyes which is less than the minimum standard; so, he is not fit to drive for now and should be referred to ophthalmologist or optometrist for clinical assessment with regards to the driving task.

Assessment of fitness to drive is one of the most challenging situations in medical practice. On one side is the patient’s important issues such as independence, and probably financial issues especially for commercial vehicles drivers, and on the
other side is the health and safety of the public. A medical practitioner should be able to balance between their duty to the patient and the duty to act in public’s interest.

When a patient is likely to pose risk to health and wellbeing of others, it should become clear to the patient that driving under their medical condition is dangerous both for him/her and the public.

Many patients are likely to choose not to drive when the condition is explained to them in an empathetic and reasonable manner. The patient should also become aware of his/her civil responsibility to self-report the medical condition to the driver
licensing authorities. This discussion and the given advice should be documented in the patient’s notes.

However, it is important to note that the relationship between a patient and the treating doctor is confidential; therefore, doctors will not normally communicate directly with the Driver Licensing Authority and will provide the patient with advice about
the ability to drive safely as well as a letter or report to take to the authority. It is not the doctor’s duty to report the patient to the police (option A) or driving licensing authority (option C) directly. What they must do though is refusing to sign the
‘fitness to drive’ form and encourage the patient to self-report himself.

In terms of the duty to the public, if a doctor believes that patient is not heeding advice to cease driving, he/she may report directly to the Driver Licensing Authority; however, except in South Australia (SA) and Northern Territory (NT) this reporting is
not mandatory.

In this case, Steven should be clearly made aware of all the risks involved, his responsibility to self-report and not driving. If you form a belief that he will not follow the instructions and is likely to place the public at risk, you may choose to report or
must report to authorities depending on the state you are practicing in.

(Option C) It is not a doctor’s responsibility to investigate other doctors as to their decision and actions. Calling the doctor who allegedly has given the patient a certificate of fitness to drive is beyond a doctor’s limits of practice.

(Option D) The presence of a supervisor does not make an unfit person fit to drive.

References

• Austroads - Assessing Fitness to Drive

Last updated:
Time spent: QID:1534 2023-2-12

1741 of 1943
Which one of the following is correct regarding chronic use of marijuana?

A. It is associated with decreased incidence of COPD.

B. It masks psychosis.

C. It reduces anxiety.

D. It impairs the ability to drive.

E. It is associated with improves job performance.

Incorrect. Correct answer is D


45% answered correctly

Explanation:

Correct Answer Is D

Marijuana is the second most commonly smoked substance worldwide after tobacco. The constituents of marijuana smoke are qualitatively and, to a large extent, quantitatively similar to those of tobacco smoke, with the exceptions of 9-
tetrahydrocannabinol (THC), found only in marijuana, and nicotine, found only in tobacco.

Given these similarities, there is concern that the health risks of regular marijuana smoking may be similar to those of habitual tobacco smoking. Chronic obstructive pulmonary disease (COPD), which is associated with high morbidity and mortality,
is among those risks. Firm conclusions cannot be drawn about the association between use of marijuana and COPD based on the limited and inconsistent data available. The conducted studies are limited by their small numbers of participants and
by the uncertain accuracy of self-reported use of marijuana, particularly in view of its illegality and the difficulty of accurately recalling amounts previously used. Some of these studies are also limited by their cross-sectional design, and most are
limited by the young age (40 years or younger) of participants. Nevertheless, the consistency of some aspects of the available data allows us to more firmly conclude that smoking marijuana by itself can lead to respiratory symptoms because of
injurious effects of the smoke on larger airways. Given the consistently reported absence of an association between use of marijuana and abnormal diffusing capacity, and signs of macroscopic emphysema, it can be close to concluding that
smoking marijuana by itself does not lead to COPD.

Cannabis use is likely to increase the risk of developing schizophrenia and other psychoses; the higher the use, the greater the risk. Cannabis use does not mask the psychosis (option B); rather results in it or worsens it. However, in individuals with
schizophrenia and other psychoses, a history of cannabis use may be linked to better performance on learning and memory tasks.

Cannabis use does not appear to increase the likelihood of developing depression, anxiety, and posttraumatic stress disorder in general; however, evidence suggest that regular cannabis use is likely to increase the risk for developing social anxiety
disorder.

In patients diagnosed with bipolar disorders, near daily cannabis use may be linked to greater symptoms of bipolar disorder than for nonusers. Moreover, heavy cannabis users are more likely to report thoughts of suicide than are nonusers.

Although there has been no strong link between regular cannabis use and increased incidence of anxiety disorders in general, it has not shown to reduce anxiety (option C) either.

Because cannabis use acutely impairs cognitive processes, there is a concern that chronic cannabis use may cause chronic cognitive impairment. Such a chronic effect is not necessarily permanent but it can persist even after the elimination of
cannabinoids from the body, and therefore would be the result of secondary changes induced by cumulative exposure to cannabinoids. Such chronic effects could produce relatively enduring behavioural deficits which presumably reflect changes in
brain function. Given these, cannabis is could potentially decrease the work performance and is not associated with increased work performance (option E).

Marijuana significantly impairs judgment, motor coordination, and reaction time, and studies have found a direct relationship between blood THC concentration and impaired driving ability.

References

• The Health Effects of Cannabis and Cannabinoids: The Current State of Evidence and Recommendations for Research.

• NIH – Does marijuana use affects driving?

• CMAJ - Does smoking marijuana increase the risk of chronic obstructive pulmonary dis-ease?

• Australian Government DoH - The health and psychological consequences of cannabis use - chapter 7

Last updated:
Time spent: QID:1556 2023-2-12

1742 of 1943
As a general practitioner, you are trying to follow the current Australian guidelines to reduce the incidence of HIV infection in your community by administration of Pre-exposure prophylaxis (PrEP) to those who are at risk. Which one of the following
is in need for PrEP?

A. Commercial sex workers.

B. Intravenous drug users.

C. Men who have sex with men.

D. Female partner of an HIV-positive man with undetectable viral load.

E. HIV-positive male partner of a female.

Incorrect. Correct answer is C


45% answered correctly

Explanation:

Correct Answer Is C

PrEP is used to significantly reduce the risk of HIV transmission in the community using antiretroviral drugs, usually tenofovir/ emtricitabine, in people who are HIV negative but at a significantly increased risk of HIV infection.

One important point to consider is that PrEP is used in those who are currently HIV negative as means of prophylaxis; therefore, those who are already HIV positive do not require PrEP and should be referred for HIV prescription.

The following groups are considered to be at “high risk” if they had these risks in the previous 3 months, or if they foresee these risks in the upcoming 3 months. For such patients, PrEP is recommended:

Men who have sex with men


Receptive CLI with any casual male partner
Rectal gonorrhea, rectal chlamydia or infectious syphilis
Methamphetamine use
CLI with a regular HIV-positive partner who is not on treatment and/or has a detectable viral load

Trans and gender diverse people


Receptive CLI with any casual male partner
Rectal or vaginal gonorrhea, chlamydia or infectious syphilis
Methamphetamine use
CLI with a regular HIV+ partner who is not on treatment and/or has a detectable viral load.

Heterosexual people
Receptive CLI with any casual MSM partner
A woman in a serodiscordant heterosexual relationship, who is planning natural conception in the next 3 months
CLI with a regular HIV+ partner who is not on treatment and/or has a detectable viral load

People who inject drugs


Shared injecting equipment with an HIV+ individual or with MSM of unknown HIV status

CLI: condomless intercourse

Of the options, only men who have sex with men (MSM) are high-risk for HIV Infection and require PrEP.

(Option A) Commercial sex worker are usually on routine STI follow-up programs and do not require PrEP merely for their carrier. However, once they have any of the high-risk conditions as outlined in the above table, they will require PrEP.

(Option B) Unless the intravenous drug user is not sharing needles with an HIV-positive individual or with MSM of unknown HIV status, PrEP is not indicated.

(Option D) As long as a man has undetectable viral load and take precautionary measures such as use of male condoms, PrEP for his female partner is not indicated as the risk of transmission is extremely low.

(Option E) Prophylaxis aims to prevent the infection. Once a person is HIV infected, prophylaxis has no point, and treatment with antiretroviral regimens should be started.

References

• ASHM – Decision Making in PrEP

Last updated:
Time spent: QID:1582 2023-2-12

1743 of 1943
A 47-year-old man presents for cholesterol check. On examination, he has a blood pressure of 140/90 mmHg. His laboratory studies show a cholesterol level of 6 mmol/L and fasting blood sugar of 5.4 mmol/L. He smokes 20 cigarettes per day and
drinks alcohol on weekends and social occasions. Which one of the following would be the most appropriate management of this patient?

A. Advise that he should start smoking cessation program.

B. Order an oral glucose tolerance test (OGTT).

C. Start him on antihypertensive medications.

D. Start him on statins.

E. Start him on aspirin.

Correct
45% answered correctly

Explanation:

Correct Answer Is A

The remarkable findings in this scenario are an upper limit normal for blood pressure both systolic and diastolic (SBP>140mmHg, DBP>90 mmHg), smoking history, and a cholesterol level of 6 mmol/L (normal <5.5 mmol/L) are significant risk factors
for cardiovascular diseases.

Of the given options, advice that she should start smoking cessation program is the most appropriate one. Smoking is associated with significantly increased risk of cardiovascular disease such as myocardial infarction, stroke, and limb ischemia, as
well as other harms to health such as increased risk of various malignancies just to name one. Every smoker should be briefly consulted about benefits of quitting at each visit.

This patient has a normal blood glucose and does not any specific interventions for now except; however, based on current Australian guidelines, he should have his blood glucose monitored every 3 years using fasting blood sugar (FBS) or HbA1C.
OGGT (option B) is not required now or used as test to monitor this patient for diabetes in the future. He also has slightly elevated cholesterol level for which regular exercise and a healthy diet for 6 months should be advised before decision as to
commencement of anti-lipid therapy using statins is made. Therefore, starting him in statins (option D) is not appropriate for now.

This patient has systolic and diastolic blood pressures right at the upper limit of normal blood pressure. He can be consider pre-hypertensive and advised for life style modification such as regular exercise, healthy diet (including low salt intake),
weight reduction (if overweight or obese), and smoking cessation. At this stage he does not need to be started on hypertensive medications (option C). Generally, pharmacotherapy for hypertension is not considered as long as the blood pressure is
less than 160 mmHg (systolic) or 10 mmHg (diastolic).

In general, aspirin (option E) is not recommended for primary prevention of cardiovascular diseases in patients with low and moderate cardiovascular risk bases on absolute CVD risk assessment (see the topic review).

TOPIC REVIEW

Absolute CVD risk assessment

In non-aboriginal patients over the age of 45 without cardiovascular disease and aboriginal and Torres Strait Islander patients over the age of 35 without cardiovascular absolute cardiovascular disease (CVD) assessment should be performed.

Absolute CVD risk assessment combines risk factors to calculate the probability of development of cardiovascular event or vascular event within a specified time frame (usually 5 years). Based on calculations, patients fall into either low-risk,
medium risk, or high-risk for CVD.

Absolute CVD risk assessment should be conducted at least every two years. This calculation requires information on the patient’s age, sex, smoking status, total and high-density lipoprotein-cholesterol (HDL–C), systolic blood pressure (SBP), and
whether the patient is known to have diabetes or left ventricular hypertrophy (LVH). In adults at low absolute CVD risk, blood test results within 5 years may be used for review of absolute CVD risk unless there are reasons to the contrary.

Adults >74 years of age may have their absolute CVD risk assessed with age entered as 74 years. This is likely to underestimate five-year risk but will give an estimate of minimum risk. Patients with a family history of premature CVD (in a first-degree
relative – men aged <55 years, women aged <65 years) or obesity (body mass index [BMI] above 30 kg/m2 or more) may be at greater risk. Similarly, patients with depression and atrial fibrillation (AF) may also be at increased risk.

Adults with any of the following DO NOTE require absolute CVD risk assessment using the absolute risk calculator, because they are already known to be at clinically determined high risk of CVD:

Diabetes and >60 years of age


Diabetes with microalbuminuria (>20 μg/min or urine albumin-to-creatinine ratio (UACR) >2.5 mg/mmol for males, >3.5 mg/mmol for females)
Moderate or severe chronic kidney disease (CKD; persistent proteinuria or estimated glomerular filtration rate [eGFR] <45 mL/min/1.73 m2)
Previous diagnosis of familial hypercholesterolemia (FH)
Systolic blood pressure (SBP) ≥180 mmHg or diastolic blood pressure (DBP) ≥110 mmHg
Serum total cholesterol >7.5 mmol/L
Aboriginal or Torres Strait Islander peoples aged >74 years (Practice Point)

According to the RACGP guideline, aspirin is not recommended for low-risk and moderate-risk patients. Also blood pressure lowering and/or lipid-lowering agents should be considered in addition to lifestyle advice if 3-6 months of lifestyle
intervention does not reduce the risk or:

Blood pressure is persistently ≥160/100 mmHg


There is a family history of premature CVD
Specific population where the absolute CVD risk assessment underestimates the risk such as in Aboriginal and Torres Strait Islander, South Asian, Maori, Pacific Islander and Middle Eastern peoples

Lifestyle modification includes:

Smoking cessation (if smoker)


Following a diet rich in vegetables and fruit, low in salt and saturated and trans fats
At least 30 minutes of physical activity on most or preferably every day of the week
Limit alcohol intake

References

• Absolute cardiovascular disease risk management

1744 of 1943
• RACGP – The Red Book: Assessment of absolute cardiovascular risk
Last updated:
Time spent: QID:1602 2023-2-12

1745 of 1943
A mother has brought her 7-year-old son for evaluation because her husband has been recently diagnosed with familial hypercholesterolemia and low-density lipoprotein receptor (LDLR) gene mutation after evaluation for established diagnosis of
acute coronary syndrome. She insists that her son undergo genetic testing for the condition. Which one of the following will be the most appropriate next step in management?

A. Refer the son for LDLR gene mutation testing.

B. Pre-test counselling.

C. Send the whole family for LDLR gene mutation testing.

D. Refer the son to a specialist dietitian.

E. Refer the mother for LDLR gene mutation testing.

Incorrect. Correct answer is B


45% answered correctly

Explanation:

Correct Answer Is B

Familial hypercholesterolaemia (FH) is a dominantly inherited condition due to a genetic defect in one of several genes that affect receptor-mediated uptake of low-density lipoprotein (LDL). Affected individuals have metabolic and clinical features
including impaired uptake of plasma LDL cholesterol, resulting in high cholesterol levels and increased risk of premature cardiovascular disease. Untreated, men have a 50% chance of coronary heart disease (CHD) before the age of 50 years and
women a 30% risk by the age of 60 years. Atherosclerosis caused by FH starts in childhood and adolescence, highlighting the need to identify cases early and commencement of preventive measures.

Offspring of FH patients will inherit either the normal gene or the defective gene; therefore, they have a 50% chance of being affected. This leads to quite a high prevalence in the general population. Estimates range between 1:200 and 1:500, but
some groups exhibit a “founder gene effect” that enriches the prevalence of the disorder. It is more common in Mediterranean countries, Christian Lebanese, French Canadians and Afrikaaner South Africans. In these populations the prevalence may
exceed 1:100. The prevalence of homozygous FH is around 1:1 million in the general population.

Index cases of FH present with one or more of the following features:

Severe hypercholesterolaemia that is not explained by secondary causes - relative hypercholesterolaemia is present from birth, but levels rise with age
A strong personal or family history of premature atherosclerotic cardiovascular disease
Tendon Xanthomas

Although the clinical picture of FH will be clear-cut in many cases, the diagnostic criteria suggest that genetic testing can provide certainty of diagnosis in some cases where confounding factors such as borderline cholesterol levels, inconclusive
family histories or tendon injuries have resulted in a diagnostic dilemma. The major value in making a molecular diagnosis is its use in predictive testing of other family members for FH. This is useful in early detection of cases that need intervention
to prevent CVD and in re-assuring family members who may not have the condition. Individuals in whom predictive genetic tests are required should be offered pre-test genetic counselling prior to consenting to sample collection for genetic analysis
because the genetic testing is expensive and time consuming, and sometimes no mutation can be detected with current methods. These facts should be shared with the patients in full details and informed consent is obtained before proceeding to
genetic testing.

Since the father is an established case of, there is a 50% chance that the son has the condition as well. The best option after pre-test counselling and informed consent would be referring the son for genetic testing.

References

• Guidelines for the Diagnosis and Management of Familial Hypercholesterolaemia

• RACGP – AFP – Detecting hypercholesterolaemia in general practice

Last updated:
Time spent: QID:1634 2023-2-12

1746 of 1943
You, as a GP, are assigned to give a lecture in a girls’ high school. At school, you are told that girls you will be talking to today are 13 years old, and that sex education topic has already been covered by another doctor. Which one of the following topics
is the most important one you would consider to cover at this session?

A. Sunscreening.

B. Regular cancer screening.

C. Bullying.

D. Cervical screening.

E. Alcohol

Correct
45% answered correctly

Explanation:

Correct Answer Is A

According to the RACGP guidelines for age-related health checks in children and young people, promotion of sunscreen use to prevent screen damage and skin cancers in future is the only option that should be covered for this age group.

The following are the assessment and preventive measures recommended by the RACGP for children aged 6-13 years

Assessment:

Measure growth and BMI routinely


Promote oral health
Promote healthy eating and drinking
‘Lift the lip’ dental check. Encourage regular dental reviews
Promote healthy physical exercise and reduction of sedentary behaviour
Enquire about progress at school as an index of wellbeing
When behaviour is a concern, explore possible contributing factors within the family and the wider social environment

Preventive counselling and advice

Injury prevention
Promote social and emotional wellbeing
Promote sun protection

(Options B and D) Regular cancer screening programs currently in place in Australia are colorectal cancer screening starting at 50 years, breast cancer screening at starting at 50 years, and cervical cancer screening starting at 25 years. No cancer
screening topic is recommended to be covered for children 6-19 years.

(Option C) Assessment of progress at school as part of wellbeing is part of preventive programme for children aged 6-19 years; however, bullying by itself is not one of the quoted factors that has to be questioned and covered.

(Option E) Alcohol is covered for children aged 14-19 years under the topic ‘healthy eating and drinking’ as is asking about smoking and provision of a strong anti-smoking message.

References

• RACGP – The Red Book – Preventive activities in children and young people

Last updated:
Time spent: QID:1640 2023-2-12

1747 of 1943
Tom, 18 months of age is brought to your general practice for vaccination. Today, he has a runny nose and fever of 38.1°C but otherwise is doing well and healthy. He had an episode of febrile seizure 7 months ago when he had an upper respiratory
tract infection. Which one of the following vaccine is best to not be given to him today?

A. Pneumococcal vaccine.

B. Polio vaccine.

C. DTPa vaccine.

D. MMR vaccine.

E. Influenza vaccine.

Incorrect. Correct answer is D


45% answered correctly

Explanation:

Correct Answer Is D

Infants and young children are most at risk for febrile seizures. Up to 5% of young children will have a febrile seizure at some time in their life. Febrile seizures happen in children between the ages of 6 months and 5 years, with a peak prevalence
between 14 and 18 months of age. About one third of children with one episode of febrile seizure will have at least one more later during childhood.

Fevers can be caused by common childhood illnesses like colds, ear infection, roseola, or any other febrile conditions. Although most vaccines can cause a mild fever as an adverse effect, febrile seizures are uncommon after vaccination. However,
caution should be exercised about MMR (measles, mumps, and rubella) and MMRV (measles, mumps, rubella, and varicella) vaccines because studies suggest a small increased risk of febrile seizures during the 5 to 12 days after the vaccine.
Studies have not shown an increased risk for febrile seizures after the separate varicella (chickenpox) vaccine.

For this child with a current mild fever (38.1°C) and history of febrile seizures, it is best to postpone the MMR vaccine to a later date when he is out of his current illness and there is least chance of febrile seizure following vaccination.

NOTE – while children with mild illnesses and fever less than 38.5°C can be safely vaccinated, it is still best to avoid the MMR vaccine as precautionary measure in this child. Children with fevers of 38.5°C or higher should receive their
vaccines when they have improved.

One specific formulation of trivalent influenza vaccine (TIV) had a link with febrile seizures in the past, especially if co-administered with pneumococcal vaccine, but current TIV formulations (option E) do not cause febrile seizures, nor does the
pneumococcal vaccine (option A).

Currently, there is no evidence to link Polio vaccine (option B) or DTPa vaccine (option C) to febrile seizures.

NOTE - febrile convulsion following vaccination is not a contraindication to vaccination.

References

• RACGP – AJGP - Seizures following vaccination in children: Risks, outcomes and management of subsequent revaccination

Last updated:
Time spent: QID:1696 2023-2-12

1748 of 1943
1A 35-year-old Caucasian lady comes to your GP practice for a follow-up. She initially had a BMI of 32 kg/m2 and was referred to a dietician for nutritional advice. She otherwise has no comorbid but has a strong family history of type 2 diabetes
mellitus. She has come to you 6 months later feeling discontented with her weight loss of 1.5kg. She tells you that she is motivated to continue her weight loss but would like further advice for better results. What is the most appropriate next step in
management?

A. Semaglutide.

B. Orlistat.

C. Bariatric surgery.

D. Psychotherapy.

E. Reduce meals to 2x a day with no snacks in between.

Incorrect. Correct answer is E


45% answered correctly

Explanation:

Correct Answer Is E

Supervised lifestyle interventions are the essential component of all weight loss strategies and are the first line of management for individuals with a BMI of 30-40kg/m2. Options for initial weight loss strategies include a reduced energy diet (RED) or
low energy diet (LED). When these options have failed a very low-energy diet (VLED) is recommended as the next step. (E is correct)

VLEDs are often recommended for 12 weeks. However, VLEDs can be continued for 6-12 months under careful supervision. Individuals may follow a partial or a complete VLED regimen. The partial regimen is the most common program
recommended and is based on 2-meal replacements per day (typically breakfast and lunch) and 1 serving of lean protein, usually for dinner, with vegetables. Milk (other than small quantities in tea or coffee) should be avoided. The complete VLED
regimen is based on 3 meal replacements per day, plus vegetables. The choice of the program (partial vs. complete) depends on the target weight and the individual’s ability to tolerate the VLED. Baseline blood tests should be taken and reviewed
every 4 to 6 weeks.

Contraindications to VLED:

Pregnancy or lactation
Severe psychological disturbance (e.g., unstable anxiety disorders, major depression), alcoholism or drug dependence
Recent myocardial infarction, cerebrovascular event, or unstable angina
Porphyria
Age >65 years (VLED in this age group should be advised with caution)

Caution in the following groups:

Diabetes on insulin or sulphonylureas


The dose of sulphonylureas or insulin should be reduced by 50% on the commencement of the VLED. It should be stressed that if hypoglycemia occurs, it takes precedence over the diet and must be treated with
appropriate carbohydrate ingestion.
Chronic kidney disease
Individuals with an estimated glomerular filtration rate (eGFR)<60 mL/min/1.73m2 need closer supervision and should have electrolytes assessed more frequently.
Taking Warfarin
The diet increases vegetable intake and may alter the international normalized ratio (INR). Individuals on Warfarin should be instructed to test INR one week after commencing the VLED to adjust the warfarin dose.

Individuals are considered to be responding to the VLED if they lose on average 1.0 to 1.5 kg per week. Once the target weight is reached, individuals are weaned off the VLED, and food is progressively re-introduced over a period of 8 weeks

If no weight is lost or less than 1.0 kg is lost per week over 4 weeks, review the person’s adherence to the VLED or other factors that may be affecting weight loss. Otherwise, pharmacotherapy is the next step.

Semaglutide (option A), a glucagon-like peptide-1 (GLP-1) receptor agonist, was recently approved for the treatment of obesity and type 2 diabetes mellitus. It is given as a weekly subcutaneous injection. However, VLED would still be the most
appropriate next step.

Orlistat (option B) inhibits pancreatic and gastric lipase, and in the long-term results in the deficiency of the fat-soluble vitamins A, D, E, and K, and the development of oxalate kidney stones. It is not recommended as a first-line option for weight loss.

Other approved medications for weight loss are phentermine (centrally acting adrenergic agonist) and liraglutide (GLP-1 receptor agonist). Off-label medication includes topiramate.

Although considered the most efficacious means of weight reduction, bariatric surgery (option C) which is an invasive procedure is reserved for those who are unresponsive to more conservative means; therefore, not the best initial option.

NOTE - Indications for bariatric surgery:

BMI >40 kg/m2


BMI >35 kg/m2 and comorbidities that may improve with weight loss
BMI >30 kg/m2 who have poorly controlled type 2 diabetes and are at increased cardiovascular risk

Psychotherapy (option D) would be pertinent to review the underlying factors as to why this patient failed her initial diet plan but not as a weight loss management option per se.

References

• The Australia Obesity Management Algorithm

Last updated:
Time spent: QID:1776 2023-2-12

1749 of 1943
A medical student sustained needle stick injury while providing an intravenous access for a patient. He is not immunized for hepatitis B. The patient's hepatitis B status is unknown.
Which one of the following is the most appropriate next step in management?

A. Immunoglobulin.

B. Immunoglobulin and hepatitis B vaccine.

C. Hepatitis B vaccine.

D. Request serology and wait for the result before commencing the vaccine.

E. Reassure and counseling regarding safe occupational practice.

Incorrect. Correct answer is B


45% answered correctly

Explanation:

Correct Answer Is B

Since the exposed person is not immune to hepatitis B, he should be given a single dose of hepatitis B immunoglobulin (preferably within the first 24 hours post-exposure) and 3
doses of hepatitis B vaccine over 6 months.

The status of the source and exposed person should be followed up by requesting full hepatitis B serology. However, if the exposed person has previously been vaccinated for
hepatitis B, and has a blood test documenting an adequate response to the vaccine (HBsAB≥10mIU/ml), no prophylaxis is necessary.

References

• http://www.health.qld.gov.au/qhpolicy/docs/gdl/qh-

Last updated:
Time spent: QID:319
2023-2-12

1750 of 1943
A 77-year-old man presents with shortness of breath and right-sided chest discomfort. A chest X-
ray shows a right-sided pleural effusion. A thoracocentesis is performed and the pleural fluid sent
to laboratory for evaluation. The results are as follows:

Pleural fluid protein toserum protein ratio: 0.7


Pleural fluid LDH to serum LDH: 0.8
Glucose of 45mg/dl

Which one of the following is the most likely underlying cause of for this pleural effusion?

A. Congestive heart failure.


B. Hepatic cirrhosis.
C. Autoimmune hemolytic anemia.
D. Malignancy.
E. Nephrosis.

Incorrect. Correct answer is D


45% answered correctly

Explanation:

Correct Answer Is D

Pleural effusion is accumulation of fluid in the pleural cavity. It can be transudative, exudative, or
hemorrhagic.

Transudate

Transudate effusions are caused by either increased hydrostatic pressure e.g. congestive heart
failure, or decreased oncotic pressure, usually due to decreased serum albumin level, such as in
chronic liver disease and cirrhosis or nephrosis in which protein is lost through the urine. Because
of systemic involvement, transudate pleural effusion is often bilateral and equal in both lungs.

Exudate

Exudate effusions are caused by a local pathology such as pneumonia, tuberculosis, or


malignancies. This type of effusion is usually unilateral and needs more investigation.

Hemorrhaic

Hemorrhagic pleural effusion is seen in malignancies, trauma or pulmonary effusion.

The following table summarizes the most common causes of each type:

Types of pleural effusion and their common etiologies


Transudative Exudative Hemorrhagic
Heart failure Parapneumonic effusion Mesothelioma
Nephrotic syndrome (pneumonia) Metastatic lung cancer
1751 of 1943
Liver disease Malignancies (lung, breast, Metastatic breast cancer
Pulmonary embolism lymphoma) Pulmonary embolism (if
atelectasis Tuberculosis with infarction)
Pulmonary embolism Trauma
Collagen vascular diseases
(RA, SLE)
Drugs
Pancreatitis

RA=rheumatoid arthritis; SLE=systemic lupus erythematosus

Pulmonary embolism (PE) can cause both transudative and exudative types. Always consider PE if
the transudate effusion cannot be attributed to any other underlying cause.

Exudate versus transudate pleural effusion:

After thoracocentesis, the effusion fluid albumin and lactated dehydrogenase (LDH) should be
measured, as well as those of the serum at the same time. The effusion is exudative if there is any
of the following:

Effusion fluid albumin to serum albumin ration>0.5


Effusion fluid LDH to serum LDH ration>0.6
Effusion fluid LDH>200 IU/ml

If none of the above criteria are met, the effusion is transudative.

A low pleural glucose concentration (1.6-2.8 mmol/L or 30-50 mg/dL) is seen in the following
conditions:

Malignant effusion
Empyema
Tuberculosis pleuritis
Esophageal rupture
Lupus pleuritis

A very low glucose concentration (<1.6 mmol/L) narrows down the possibilities to rheumatoid
pleurisy or empyema.

A normal pleural fluid has a pH of 7.6 – 7.64.

Pleural fluid pH highly correlated with pleural fluid glucose levels. A pleural fluid pH of less than
7.30 with a normal arterial blood pH level is caused by the same diagnoses for low pleural fluid
glucose. However, for parapneumonic effusion a low pleural fluid pH level is more predictive of
complicated effusion (that requires drainage) than is a low pleural fluid glucose level.

This patient has exudative pleural effusion and of the given options only malignancies (lung, breast,
lymphoma) can be the underlying cause.

References

• http://emedicine.medscape.com/article/299959-worku
1752 of 1943
• http://emedicine.medscape.com/article/299959-worku
Last updated:
Time spent: QID:366
2023-2-12

1753 of 1943
A 57-year-old man, who has been a smoker for most of his adult life, presents to your clinic
complaining of coughing up blood after he wakes up in the morning. Which one of the following is
the most important condition to exclude?

A. Bronchiectasis.
B. Leukemia.
C. Laryngeal cancer.
D. Bronchogenic carcinoma.
E. Pulmonary embolus.

Incorrect. Correct answer is D


45% answered correctly

Explanation:

Correct Answer Is D

Coughing up blood indicates a pulmonary pathology, requiring a thorough investigation. Pulmonary


tuberculosis and bronchogenic carcinoma should always be considered. Although bronchiectasis,
COPD, bronchitis, etc can all cause cough as the most common cause of hemoptysis, since
bronchogenic carcinoma has a significant aggressive course than other mentioned conditions and
should be considered and excluded first.

1754 of 1943
Which one of the following could trigger an asthma attack?

A. Sulphites.
B. Benzyl benzoate.
C. Lecithin.
D. Terbutaline.
E. Ephedrine.

Correct
45% answered correctly

Explanation:

Correct Answer Is A

The most common adverse reaction with exposure to sulfites is asthma like symptoms, estimated
to occur in 5 to 10% of patients with asthma. These symptoms are more likely to occur in patients
with severe asthma, or those in whom asthma is poorly controlled. Reactions can vary from mild to
life-threatening.

Sulfites are used to preserve many foods and beverages. The addition of sulfites to some foods
like beer and wine is permitted in most countries. However, it is illegal to add these to foods like
fresh salads or fruit salads, or to meats like mincemeat. Unfortunately, these can be added from
time to time illegally.

The following is a list of the most common sources of accidental exposure to sulfites.

Drinks: cordials and some fruit juices as well as beer and wine, occasionally soft drinks and
instant tea.
Other liquids: commercial preparations of lemon and lime juice, vinegar, grape juice.
Commercial foods: dry potatoes, gravy and sauces and fruit toppings, maraschino cherries,
pickled onions, Maple syrup, jams, jellies, some biscuits and bread, or pie or pizza dough.
Fruit: dried apricots, and sometimes grapes will be transported with sachets of the sulfite
containing preservative.
Salads and fruit salads: sometimes restaurant salads and fruit salads will have sulfites added
to preserve their color.
Crustaceans: sulphur powder is sometimes added over the top of crustaceans to stop them
discoloring.
Meat: sulfites are sometimes illegally added to mincemeat or sausage meat.
Other foods: gelatin, coconut.

References

• http://www.ncbi.nlm.nih.gov/pmc/articles/PMC401744

• http://www.asthmaaustralia.org.au/onAIR/Food_addit

• http://www.allergy.org.au/patients/product-allergy
1755 of 1943
Last updated:
Time spent: QID:367
2023-2-12

1756 of 1943
Which one of the following is never indicated in acute management of an asthma attack in
children?

A. Steroids.
B. Salbutamol.
C. Ipratropium bromide.
D. Sodium cromoglycate.
E. Adrenaline.

Incorrect. Correct answer is D


45% answered correctly

Explanation:

Correct Answer Is D

Sodium cromoglycate is indicated in the prophylactic treatment of asthma, including prevention of


exercise-induced bronchospasm. It is not useful in management of an acute asthma attack.

The management of acute asthma depends on the severity of the symptoms:

Salbutamol (short-acting β2 agonist) is the best initial treatment in all cases of acute asthma.
Systemic steroids are useful in moderate to severe attacks adn should be continued for 3
days and stopped without tapering.
Ipratropium bromide (an anticholinergic agent) can also be helpful in management of
moderate to severe asthma attack if maximum dose of short-acting β2 agonists fail to break
the attack.
Adrenaline can be used in severe asthma attack where there is imminent cardiorespiratory
arrest or when there is anaphylactic reaction.

References

• Therapeutic Guidelines – Respiratory; available from http://tg.org.au


Last updated:
Time spent: QID:370
2023-2-12

1757 of 1943
A mother brings her 3-year-old boy to your practice with episodic asthma and persistent interval
symptoms of nocturnal cough and exercise-induced wheezing. You plan to start her on fluticasone.
Which one of the following delivery methods is most appropriate for this age group?

A. Oral suspension.
B. Metered dose inhaler (MDI).
C. MDI and spacer device with mask.
D. Nebulizer and pump.
E. Breath-actuated inhaler (Accuhaler®).

Incorrect. Correct answer is C


45% answered correctly

Explanation:

Correct Answer Is C

(Option A) Fluticasone is not available in form oral suspension or solution. Moreover, systemic use
of corticosteroids unnecessarily poses the child at adverse effects assoiciated with systemic
corticosteroids.

(Option B) Metered dose inhalers are not an effective delivery mode in this age group, as it needs
breath-holding.

(Option C) MDI and spacer device with mask is the preferred mode of delivery for inhalers in young
children unable to efficiently hold their breath.

(Option D) Nebulizers are also effective but are more expensive and difficult to use compared with
spacers with face mask. They should be considered if the patient cannot cope with spacer and
face mask. They are the second best option.

(Option E) Breath-actuated inhalers are inhalers that automatically release a spray of medication
when the person begins to inhale. Like MDIs this mode of delivery is difficult to use in young
children.

References

• http://www.asthmawa.org.au/PageFiles/181/AA-Medica
Last updated:
Time spent: QID:372
2023-2-12

1758 of 1943
A 5-year-old child is brought to your clinic with complaint of cough. His mother explains that the
coughs started 2 months ago, and sometimes wakes him up at night. He also has them when he
plays with other children or runs and sometimes is associated with small amount of clear phlegm.
On history, you realize that he had flexural eczema when he was 2 years old. There child is afebrile.
On chest examination, no wheeze is heard. The rest of the physical exam is inconclusive. Which
one of the following is the most likely diagnosis?

A. Epiglottitis.
B. Chronic sinusitis.
C. Asthma.
D. Gastro-esophageal reflux disease.
E. Whooping cough.

Incorrect. Correct answer is C


45% answered correctly

Explanation:

Correct Answer Is C

The age of the child, history of atopy (eczema) and chronic intermittent cough favor the diagnosis
of asthma. Asthma is common in this age group and is usually underdiagnosed. In atopic asthma,
the child usually has the history of other atopies such as allergic rhinitis, eczema, etc.

(Option A) Epiglotitis has more pronounced and acute presentation with fever and cough,
respiratory distress and drooling. The child is usually very ill.

(Option B) Chronic sinusitis may present with nocturnal cough caused by post nasal drip, but
exertional cough is not usually a feature. Besides, the discharges tend to be more thick and
purulent. There is also history of facial tenderness or fullness and recurrent upper respiratory
infections.

(Option D) Gastro-esophageal reflux is rare in this age group. If present, it may cause chronic calf
and mimic cough-variant asthma especially at night, but not exertional cough. Moreover, absence
of other features such as heart burn makes this diagnosis far less unlikely.

(Option E) Whooping cough (pertussis) has a different presentation with paroxysms of cough and
the terminal inspiratory whoop.

References

• http://emedicine.medscape.com/article/296301-clini
Last updated:
Time spent: QID:373
2023-2-12

1759 of 1943
A 4-year-old boy, with background history of asthma, presents with a severe attack of asthma
following an upper respiratory infection. On physical examination, he appears distressed with a
pulse rate of 130 bpm, respiratory rate of 48 breaths per minute and temperature 37.1°C. Bedside
pulseoxymetry shows an oxygen saturation of 85% while on room air. Which one of the following is
the most appropriate initial step in management?

A. Give salbutamol inhalation and recommend salbutamol inhaler 4-hourly at home and
review in 24 hours.
B. Start 4-hourly salbutamol and ipratropium bromide inhalers and oral prednisolone, and
review in 24 hours.
C. Start inhaled salbutamol and oral amoxicillin 4-hourly, and review in 24 hours.
D. Give oxygen and continuous salbutamol inhalations and transfer to the hospital by
ambulance.
E. Refer for admission to the hospital for salbutamol and ipratropium inhalations.

Incorrect. Correct answer is D


45% answered correctly

Explanation:

Correct Answer Is D

Initial assessment of the severity of an asthma attack in children is according to the following
table:

Initial assessment of the severity of an acute attack of asthma in children


Severe and life-
Clinical features Mild Moderate
threatening
Altered
No No No
consciousness
Increased accessory
No Some marked
muscle use
Speaking Sentences Phrases Words
Pulse rate* <100 100 – 200 >200
PEF (% predicted) >60% 40%-60% <40%**
FEV1 (% predicted) >60% 40%-60% <40%***
Oxymetry while
>94% 90%-94% <90%
breathing air
Probably, especially if
Hospital admission No poor initial treatment Yes, consider ICU
response
PEF= peak expiratory flow; FEV1= forced expiratory volume in 1 second

*bradycardia may be seen when respiratory arrest is imminent

**patient may be incapable of performing the test

***patient may be incapable of performing the test


1760 of 1943
With an oxygen saturation of less than 90%, this child has a severe asthma attack and should be
transferred to the hospital for close monitoring and treatment; however, initial treatment with
oxygen and short-acting β2 agonists (e.g. salbutamol) should be started without delay and
maintained on the way to the hospital.

Options suggesting outpatient management are incorrect. Transferring to the hospital without
oxygen supplementation and bronchodilators is inappropriate, because this boy is hypoxic and at
increased risk of respiratory failure.

References

• Therapeutic Guidelines – Respiratory; available from http://tg.org.au

• http://www.asthmahandbook.org.au/uploads/Australia
Last updated:
Time spent: QID:374
2023-2-12

1761 of 1943
A 25-year-old woman presents to the emergency department with exacerbation of asthma. Which
one of the following is the most accurate tool to assess the severity of the attack?

A. Oxygen pressure of arterial blood (PaO2).


B. Loudness of the wheeze.
C. Severity of the dyspnoea.
D. Forced expiratory volume in one second (FEV1).
E. Pulse rate.

Incorrect. Correct answer is D


45% answered correctly

Explanation:

Correct Answer Is D

Oxygen pressure of arterial blood (PaO2) is reduced in severe asthma as a result of


ventilation/perfusion mismatch, but it must be interpreted in relation to the concentration of
inhaled oxygen to in order to determine the arterial-alveolar (a-A) oxygen gradient.

Wheezing and its loudness is not a reliable indicator. In sever disease, the wheezing may decrease
significantly, resulting in ‘silent-chest’ that is an ominous sign. The wheezing is minimal both with
mild attack and just prior to respiratory arrest in very severe attacks.

In an asthma attack, the severity of the dyspnea is usually a marker of the disease severity, but not
the best one because it may be affected by physiologic conditions such as anxiety.

Forced expiratory volume in one second (FEV1), or peak expiratory flow rate (PEFR) is the best
guide to assess the severity, as well as the response to treatment. Of these two, FEV1 is the best
single indicator.

Pulse rate is usually increased in an asthma attack but it might be a consequence of anxiety or the
effect of beta 2 agonist or atropine –like drugs such as ipratropium bromide.

References

• http://www.ncbi.nlm.nih.gov/pmc/articles/PMC211718
Last updated:
Time spent: QID:375
2023-2-12

1762 of 1943
A 12-year-old boy is brought to the hospital by his mother with acute shortness of breath and
wheezing. He has background history of asthma and uses inhalers on an as-required basis. Which
one of the following is the best bedside measure to assess the severity of asthma?

A. Severity of the wheeze.


B. Response to inhalers.
C. The degree of dyspnea.
D. Peak expiratory flow rate.
E. Formal pulmonary function tests.

Incorrect. Correct answer is D


45% answered correctly

Explanation:

Correct Answer Is D

The peak expiratory flow rate (PEF, also known as a peak flow) is the maximum rate that a person
can exhale during a short maximal expiratory effort after a full inspiration.

In patients with asthma, the PEF correlates reasonably well with the the forced expiratory volume in
one second (FEV1). FEV1 (not an option) is the most useful tool for bedside assessment of
asthmaesponse to treatment. In the abence of FEV1, PEF is the best option for this objective.

Peak flow meters have a limited role in establishing the diagnosis of asthma. However, they are
very useful in measuring the severity of asthma exacerbation, or response to treatment. Peak flow
meter readings become higher while patients are recovering and lower when the airways are
constricted.

(Option A) Severity of wheeze is not reliable sign of severity of asthma. Severe asthma usually
presents with silent chest and low grade wheeze.

(Option B) Response to inhalers and degree of dyspnea are also very unreliable signs to assess the
severity of asthma.

(Option C) Dyspnea is not a reliable predictor of asthma severity because a patient with mild
asthma and one with severe asthma may be equally breathless.

(Option E) Formal pulmonary function testing is performed at specialized centers and is used to
diagnose asthma. In contrast, peak flow measurement is quick and useful in emergency situation
to assess the severity of an asthma attack.

References

• http://www.ncbi.nlm.nih.gov/pmc/articles/PMC211718
Last updated:
Time spent: QID:376
2023-2-12

1763 of 1943
Which of the following statements is correct regarding asthma in children?

A. Inhaled bronchodilators are very effective under the age of 12 months.


B. Oral bronchodilators are very effective under the age of 12 months.
C. A spacer with face mask should be used for children aged between 1 to 2 years.
D. Most of the children do not grow out of asthma when they become adults.
E. Giving more than 6 inhaled doses of beta-2 agonist can be life-threatening in severe
asthma.

Incorrect. Correct answer is C


45% answered correctly

Explanation:

Correct Answer Is C

For children aged 1-2 years (or even older) a spacer with face mask provides adequate delivery of
the inhaled drug and is a the preferred mode to deliver asthma medicine.

(Options A and B) Bronchodilators, inhaled or oral, are not very effective in children younger than
12 months.

(Option D) Most of the children grow out of asthma in adulthood and very few continue to be
asthmatic adults.

(Option E) Giving 6 to 12 inhaled doses of beta-2 agonist is very safe and appropriate if used in
severe asthma.

References

• Therapeutic Guidelines – Respiratory; available from http://tg.au.org


Last updated:
Time spent: QID:377
2023-2-12

1764 of 1943
A 6-month-old child, with confirmed diagnosis of asthma, is given 6 puffs of salbutamol during an
acute asthma attack without adequate response. Which one of the following would be the next
best step in management?

A. Repeat another 6 puffs of salbutamol.


B. Give 4 puffs of ipratropium bromide.
C. Give oral steroids.
D. Give intravenous steroids.
E. Give inhaled steroids.

Correct
45% answered correctly

Explanation:

Correct Answer Is A

Asthma is less common in children younger than 12 months of age. However, if contracted at this
age, it is poorly responsive to bronchodilators.

Oxygen and short-acting beta-2 agonists such as salbutamol are used routinely despite poor
response to β2 agonists, because a mild asthma attack can still be managed by salbutamol.

Initially up to 6 puffs for children ages 0-5 years and up to 12 puffs for those 6 years or older are
given. 20 minutes is waited and the respond assessed. If the attack does not subside, up to 6 puffs
can be given every 20 minutes for one hour. If these measures failed ipratropium bromide would be
added (4 puffs by mask and spacer or 250 mcg by nebuliser for children 0-5 years, or 8 puffs by
mask and spacer or 500 mcg by nebuliser for children older than 6 years.

The role of ipratropium in management of an acute asthma attack is controversial. It should be


considered if the asthma is severe and unresponsive to maximum dose of short-acting beta
agonists.

Oral prednisolone 1 mg/kg should also be given for the first 3 days after the asthma attack.

Corticosteroids have minimal role in acute management of asthma, but should be considered to
prevent relapses. Oral forms of steroids (suspension) are available in Australia.

Once the acute attack subsides, the child should be reviewed daily for revision of management
plan.

1765 of 1943
A 27-year-old woman presents to your clinic with fever and cough since yesterday. On examination,
she looks ill and unwell. Her blood pressure is 110/70 mmHg, pulse rate 110 bpm, respiratory rate
22 breaths per minute and temperature 38.7°C. Pulse oxymetry shows oxygen saturation of 89%
while on room air. Which one of the following is the next best step in management?

A. Continuous positive airway pressure (CPAP).


B. 60% Oxygen through nasal prongs.
C. 100 % Oxygen through Hudson’s mask.
D. Oxygen through Venturi mask – 28%.
E. Intubation and ventilation.

Incorrect. Correct answer is C


45% answered correctly

Explanation:

Correct Answer Is C

This patient has clinical features of pneumonia. With an oxygen saturation of 89% (less than 90%),
she should be considered hypoxemic until ABG results are available.

Patients with serious acute respiratory illnesses such as acute asthma, pneumonia, lung cancer,
pulmonary embolism, pleural effusion, and pneumothorax require moderate levels of supplemental
oxygen. If the patient is hypoxemic on pulse oxymetry (oxygen saturation less than 90%). This may
be 100% oxygen by nasal cannulae at 2 to 6 L/minute or a simple face mask at 5 to 10 L/min
initially, but if the oxygen saturation is less than 85% and the patient is not at risk of hypercapnic
respiratory failure, treatment should be commenced with 100% oxygen through reservoir mask at
10 to 15 L/min. Mechanical ventilation such as CPAP and intubation and ventilation are considered
if the patient fails to maintain the target O2 saturation despite adequate supplementation of
oxygen with other delivery systems such as nasal cannulae, simple face mask, reservoir masks or
no-rebreather masks.

Oxygen through Venturi mask is first-line option for patients at risk developing hypercapnic
respiratory failure (type II). These patients include the following:

1. Known cases of chronic obstructive pulmonary disease or bronchiectasis


2. Severe kyphoscoliosis or ankylosing spondylitis
3. A known history of heavy smoking
4. Severe obstructive sleep apnoea
5. Morbid obesity (BMI more than 40 kg/m2)
6. Musculoskeletal disorders with respiratory muscle weakness, especially if on home
ventilation
7. Overdose of opioids, benzodiazepines or other respiratory depressant drugs.

NOTE - ‘Hudson mask’ is a confusing name. ‘Hudson RCI’ is the manufacturer and produces a
variety of oxygen delivery devices including nasal prongs, simple face masks, reservoir face masks
and non-rebreather masks, but historically the Hudson mask is the inappropriate equivalent of
simple face masks.

1766 of 1943
References

• https://www.brit-thoracic.org.uk/document-library/

• http://www.ncbi.nlm.nih.gov/pmc/articles/PMC111390

• http://www.respiratoryupdate.com/members/Levels_of

• Therapeutic Guidelines – Respiratory; available on http://www.tg.org.auOxford Handbook of


Clinical Medicine – 8th Edition – pages 180-181
Last updated:
Time spent: QID:137
2023-2-12

1767 of 1943
A 12-year-old boy is brought to the emergency department with a progressively worsening asthma
attack. He has past medical history of previous asthma attacks and recurrent chest infections.
Which one of the following, if present, would indicate life-threatening asthma?

A. Wheeze.
B. Cyanosis.
C. Speaking in sentences.
D. Peak expiratory flow more than 75%.
E. Oxygen saturation 95%.

Incorrect. Correct answer is B


45% answered correctly

Explanation:

Correct Answer Is B

If present, cyanosis indicates a life-threatening asthma episode. Other indicators of severity are
FEV1<50%, peak expiratory flow <50%, the patient speaking in words, physical exhaustion, pulse
rate>120bpm, oxygen saturation<92%, and significant accessory muscles use.

(Option A) Wheeze is not a reliable indicator of the severity of an asthma attack. In a severe attack,
the wheeze disappears and results in the 'silent chest' just prior to respiratory collpase. A loud
wheeze may not be heard in the noisy environmet of the emergency department.

(Option C) A patient can speak in sentences in mild attack, in phrases in moderate and in words in
severe attacks.

(Option D) Peak expiratory flow meter is usually more than 75% in mild asthma, 50-75% in
moderated and <50% is severe asthma attacks.

(Option E) Oxygen saturation of more than 94% is seen in mild, between 92-94% in moderate and
<92% in a severe attacks.

1768 of 1943
A 5-year-old boy is brought to your practice, by his mother, because of intermittent attacks of
wheezing, which have beein occurring 4-6 times a year over the past 2 years. These attacks are
usually precipitated by viral upper respiratory tract infections and last 2-3 days. The symptoms
respond to treatment with inhaled salbutamol. There are no interval symptoms. His older brother
had flexural eczema until the age of 18 months. Which one of the following is the most appropriate
management of this child?

A. Inhaled sodium cromoglycate.

B. Inhaled fluticanose.

C. Oral montelukast.

D. Oral prednisolone when symptomatic.

E. Inhaled salbutamol when symptomatic.

Incorrect. Correct answer is E


45% answered correctly

Explanation:

Correct Answer Is E

With infrequent episodes of wheezing and absence of interval symptoms, this child is most likely to
have mild intermittent asthma. Mild asthma in children and adults is managed by short-acting β2
agonists such as salbutamol.

Inhaled fluticasone or other corticosteroids are not used for mild intermittent asthma, and is
reserved for more severe forms of asthma. Montelukast is often considered and tried for a period
of 2-4 weeks before the child is started on inhaled corticosteroids in an attempt to spare the child
from corticosteroids and their potential adverse effects.

References

• http://www.asthmahandbook.org.au/uploads/Australia

• Therapeutic Guidelines – Respiratory; available from http://tg.org.au


Last updated:
Time spent: QID:380
2023-2-12

1769 of 1943
Which one of the following is correct about combination of long-acting β2 agonists with inhaled
corticosteroids in children with asthma?

A. It reduces the frequency of severe exacerbations.


B. It may increase the frequency of severe exacerbations.
C. It has no effect on asthma.
D. It is excellent for exercise-induced asthma.
E. Long acting β2 agonists decrease the need for inhaled steroids.

Incorrect. Correct answer is B


45% answered correctly

Explanation:

Correct Answer Is B

Long-acting beta2 agonists are currently overprescribed in children. They are also often used
inappropriately as first-line therapy and are not recommended for children aged five years or less.

Due to the paucity of pediatric clinical trials, the evidence for the efficacy and safety of long-acting
beta2 agonists in children is limited. There is little evidence that they reduce the risk of severe
exacerbations and some evidence that they may actually increase the risk.

The regular use of long-acting beta2 agonists may also result in a loss of protection against
exercise-induced bronchoconstriction, and the development of tolerance to short-acting beta2
agonists. Long-acting beta2 agonists are only one option for children whose asthma is not
adequately controlled with inhaled corticosteroids alone – the other options being an increase of
inhaled corticosteroid dose or the addition of a leukotriene receptor antagonist.

For children whose major ongoing symptoms are activity related, the addition of a leukotriene
receptor antagonist is the preferred option.

(Option A) There is little evidence for efficacy of LABAs in reducing the acute asthma exacerbation,
and in fact, some studies have shown that LABA use can increase the frequency of asthma
exacerbations in children.

(Option C) Evidence has shown some effectiveness for LABAs in adult patients and under some
circumstances for children, often as measure of last resort. But it is associated with the problems
like possible increased frequency of acute exacerbations and tolerance to short-acting beta2
agonists.

(Option D) First-line medications for prevention of exercise-induced asthma are short-acting beta2
agonists (SABA) such as salbutamol, 20 minutes before the anticipated exercise. LABAs not only
are inappropriate for this purpose, but they also have been shown to reduce the protective effects
of SABAs.

(Option E) LABAs do not decrease the need for preventers such as inhaled corticosteroids (ICS).
Low to medium doses of ICS are best preventers to consider for children with asthma as first-line.
Combination therapy with ICS and LABA should only be considered if other measure such as
1770 of 1943
maximum dose of ICS and leukotriene receptor antagonists (e.g. Montelukast) fails or is
inadequate.

1771 of 1943
A 10-year-old male child is brought in by his mother with complaint of ongoing exercise-induced
asthma, despite being on maximum dose of inhaled corticosteroids. Which one of the following is
the most appropriate next step in management?

A. Refer to respiratory physician.


B. Add leukotriene receptor antagonists.
C. Cease inhaled corticosteroids and start intravenous steroids.
D. Add long-acting β2 agonists.
E. Add short-acting β2 agonists.

Incorrect. Correct answer is B


45% answered correctly

Explanation:

Correct Answer Is B

For children aged 4-16 years, who are already taking an inhaled corticosteroid, but their exercise-
induced symptoms do not improve, addition of regular leukotriene receptor antagonist
(montelukast, zafirlukast) is the next best step in management.

References

• http://www.asthmahandbook.org.au/uploads/Australia
Last updated:
Time spent: QID:382
2023-2-12

1772 of 1943
A 69-year-old man is brought to the emergency department with severe shortness of breath and
cough. He has had 2 similar episodes in the past 6 months. His background history includes
smoking of 30 cigarettes a day for the past 25 years. On examination, he is in moderate distress
with a blood pressure of 110/75mmHg, pulse of 120 bpm and respiratory rate of 34 breaths per
minute. he is started on oxygen 24% through Venturi mask. A bedside pulseoxymetry shows
oxygen saturation of 85%. Which one of the following is the next best step in management?

A. Chest X-ray.
B. ABG.
C. Serum electrolytes.
D. Full blood count.
E. Assisted ventilation with CPAP.

Incorrect. Correct answer is B


45% answered correctly

Explanation:

Correct Answer Is B

The clinical findings, the history of smoking, and the oxygen saturation of 85%, are suggestive of an
acute exacerbation of COPD. When hypoxemia is suspected on pulseoxymetry, an arterial blood
gas (ABG) analysis is mandatory as the next best step to not only confirm the hypoxia, but also to
assess the pH and partial arterial pressure of carbon dioxide (PaCO2) for further managment
planning, as well as having a baseline for monitoring the response to treatment.

Chest X-rat (option A) , serum electrolytes (option C) and full blood count (option D) are part of the
initial assessment, but the most important step remains ABG.

Assisted ventilation with CPAP (option E) or intubation and ventilation are considered if the patient
remains hypoxic and hypercapnic despite judicious administration of oxygen or if the patient is in
severe respiratory distress (e.g. accessory muscle use, etc).

References

• https://www.mja.com.au/journal/2003/178/6/copdx-pl

• http://www.uptodate.com/contents/management-of-exa

• Therapeutic Guidelines – Respiratory; available on: http://tg.org.au


Last updated:
Time spent: QID:383
2023-2-12

1773 of 1943
A 67-year-old man with background history of smoking for the past 40 years and chronic
obstructive pulmonary disease (COPD) presents with acute shortness of breath, pleuretic chest
pain, increased mucus production and severe cough. Last week, he has had an episode of gout in
his first metatarsophalangeal joint, for which he was started on indomethacin 75 mg, 8-hourly.
Sever pain made him bedbound for 5 days. On examination, he has a blood pressure of 130/95
mmHg, pulse rate of 110 bpm, respiratory rate of 34 breaths per minute and temperature of 37.5°C.
Which one of the following is the next best step in management?

A. Chest X-ray.
B. Arterial blood gas (ABG) analysis.
C. Computed tomography pulmonary angiography (CTPA).
D. Ventilation/perfusion scan.
E. Chest CT scan.

Incorrect. Correct answer is B


45% answered correctly

Explanation:

Correct Answer Is B

Pleuretic chest pain, tachycardia, and tachypnea and the history of immobility is highly suggestive
of pulmonary embolism (PE). PE on the other hand, can precipitate a COPD exacerbation that
justifies the increased amount of sputum production, hypoxia and hypercapnia. Under these
circumstances, an ABG is always the next best investigation to check for hypoxia (the most
important concern in both PE and COPD), worsened hypercapnia due to COPD exacerbation, and
the blood pH. A chest X-ray (option A) should be obtained for this patient to investigate any other
underlying lung pathology that might, other than the PE, has led to the condition, but immediate
pulseoxymetry and ABG always come first in priority.

Ventilation/perfusion scan (option D) or CTPA (preferred) (option C) are diagnostic tools to


establish the definite diagnosis of PE. They should be performed to prove the presence if PE prior
to treatment with anticoagulation.

A conventional CT scan of the chest (option E) has no role in management of this patient for now.

With the underlying COPD and baseline chest X-ray abnormalities, ventilation/perfusion scan is
unlikely to be of diagnostic value. Conventional CT scan of the chest would add no relevant piece
of information.

References

• Therapeutic Guidelines – Respiratory; available from http://tg.org.au


Last updated:
Time spent: QID:384
2023-2-12

1774 of 1943
A 60-year-old man is brought to the emergency department by paramedics because of severe
shortness of breath. On his arrival, a bedside pulse oxymetry shows an O2 saturation of 85% on
room air. His son states that this is the fifth time he is admitted to the emergency department due
to the same problem. On further inquiry, he is found to have smoked 60 packs per year for the past
30 years. He is given oxygen 28% by Venturi mask. 30 minutes later an arterial blood gas (ABG)
reports his PaO2 and PaCO2 68 mmHg and 60 mmHg respectively. The O2 saturation is 88% while
on oxygen. Which of the following is the next best step in the management of this patient?

A. Stop oxygen and check ABG after 30 minutes.


B. Continue oxygen until normal PaO2 is achieved.
C. Immediate intubation and ventilation.
D. Provide immediate antibiotic coverage against gram negative bacteria.
E. Start intravenous aminophylline infusion.

Incorrect. Correct answer is B


45% answered correctly

Explanation:

Correct Answer Is B

Oxygen therapy, bronchodilators, systemic corticosteroids and antibiotic are mainstay of therapy in
an acute exacerbation of COPD. The main goal of oxygen therapy is delivering adequate oxygen,
while preventing from hypercapnia and acidosis. Since this patient has not deteriorated since
presentation and commencement of oxygen therapy, maintaining the same rate of oxygen delivery
is the best appropriate management.

Adequate oxygenation must be assured, even if it leads to acute hypercapnia. Hypercapnia is


generally well tolerated in patients whose arterial carbon dioxide tension (PaCO 2 ) is chronically
elevated. However, mechanical ventilation may be required if hypercapnia is associated with
depressed mental status, profound acidemia, or cardiac dysrhythmias.

Stopping oxygen causes the patient’s condition to worsen again and is not an appropriate option.
Immediate intubation and ventilation is not necessary at this point because the patient is rather
stable, with no signs of hypercapnia as mentioned above.

Antibiotic are indicated if a bacterial infection is suspected base on the presence of (1) increased
dyspnea, (2) increased sputum volume, or (3) purulent sputum. The most common culprits in
bacterial infections are streptococcus pneumoniae, hemophilus influenza and moraxella
catarrhalis. These are gram positive organisms and are adequately covered by amoxicillin or
doxycycline.

References

• Therapeutic Guidelines – Respiratory; available on: http://tg.org.au

• http://www.uptodate.com/contents/management-of-exa

1775 of 1943
Last updated:
Time spent: QID:385
2023-2-12

1776 of 1943
A 68-year-old nursing home resident man, with long standing chronic obstructive pulmonary
disease (COPD), has been brought by ambulance to the emergency department because of an
exacerbation of his condition. En route to the emergency department, he was severely short of
breath and was given oxygen 10 L/min via nasal canula. On examination, he is drowsy and
disoriented. Which one of the following could be the most likely result of his arterial blood gas
(ABG)?

A. Ph=7.29, PaCO2=65mmHg, PaO2=85mmHg.


B. Ph=7.15, PaCO2=50mmHg, PaO2=68mmHg.
C. Ph=7.25, PaCO2=25mmHg, PaO2=100mmHg.
D. Ph=7.35, PaCO2=40mmHg, PaO2=40mmHg.
E. Ph=7.45, PaCO2=85mmHg, PaO2=40mmHg.

Correct
45% answered correctly

Explanation:

Correct Answer Is A

The clinical picture is consistent with carbon dioxide (Co2) narcosis. Agitation, confusion, tremors,
convulsions, and possible coma may occur if blood levels of carbon dioxide rise to 70 mmHg or
higher. Individuals with chronic obstructive pulmonary disease (COPD) can have CO2 narcosis with
no symptoms other than confusion and/or drowsiness, because they have already developed
tolerance to elevated amounts of CO2. When ventilation is sufficient to maintain a normal PaO2 in
the arteries, the carbon dioxide partial pressure is generally expected to be near 40 mmHg.

This patient has been on high-flow oxygen so his blood oxygen content is expected to be normal or
even high. On the otehr hand, this patient is dependent on hypoxia rather than hypercapnia as the
main stimulant of respirotry drive. With excessive oxygenation respiration will be suppressed and
he is likely to have CO2 retention and increased blood CO2. CO2 retention also results in repiratroy
acidosis. Of the options, option A is consistent with these findings.

TOPIC REVIEW

If a patient with acute exacerbation of COPD is hypoxemic, oxygen should be used as the most
appropriate next step in management to maintain the arterial oxygen saturation at about 90%
(88%-92%). Oxygen therapy starts with 2L/minute through nasal cannula (prongs) or 24- 28%
oxygen via Venturi mask.

Serial ABGs are used to monitor for hypercapnia or acidosis and accurate measurement of blood
oxygen content.

With high flow or high concentration of oxygen some patients may develop hypercapnia, probably
because supplemental oxygen contributes to the following:

1. Increasing the perfusion/ventilation mismatch


2. Suppression of respiratory drive

Ventilatory support may be needed if hypercapnia develops or worsens despite:


1777 of 1943
Optimal drug therapy
Treatment of other complications
Attempt to minimize inspired oxygen while maintaining adequate O2sat≥88% to 90%

Non-invasive ventilatory support (NIVS) with CPAP or BiPAP may avoid the need for intubation.

Supplemental oxygen should be maintained at minimum levels required for adequate oxygen
saturation, peripheral oxygen delivery, and stable pressure of PaCO2 and pH. Adjust the oxygen or
decide for ventilation according to the following table:

PaCO2
PaO2 (mmHg) pH Adjustment
(mmHg)
>>60 Reduce oxygen flow to
Normal (36- Normal (7.36-
maintain PaO2 closer to 60
44) or high 7.44) or low
(SaO2>>90% by mmHg (SaO2 90%-92%) and
(>44) (<7.36)
oxymetry) monitor ABG
Normal (36-
Normal Continue same oxygen flow
44)
Consider assisted
>60 (SaO2>90% ventilation and monitor ABG
by oxymetry) Large rise
Low
(>50) Reduce oxygen flow to
maintain PaO2 at 60 mmHg
(SaO2 88%-92%)
Increase oxygen flow but
Normal or low Normal
monitor ABG
<60 (SaO2< 90% Consider assisted
by oxymetry) ventilation and monitor ABG
Large rise Low

>= greater than; >>=substantially greater than

References

• Therapeutic Guidelines – Respiratory; available from http://tg.org.au

• http://www.uptodate.com/contents/management-of-exa
Last updated:
Time spent: QID:386
2023-2-12

1778 of 1943
A 72-year-old man, who is a known case of chronic obstructive pulmonary disease (COPD),
presents to the emergency department with severe shortness of breath and cough. Based on the
diagnosis of a COPD exacerbation, he is started on oxygen 28% by Venturi mask. After one hour, an
ABG is obtained that shows a PaO2 of 42mmHg and a PaCO2 of 68mmHg. Which one of the
following is the next best step in management?

A. Increase the oxygen flow and use face mask.


B. Assisted ventilation.
C. Decrease the oxygen flow.
D. Continue with the same amount of oxygen and repeat the ABG after one hour.
E. Add inhaled short-acting β2 agonists.

Incorrect. Correct answer is B


45% answered correctly

Explanation:

Correct Answer Is B

This patient has been started on standard oxygen therapy with 28% oxygen via Venturi mask, but
has not responded to the treatment, and now has a PaO2 of less than 60 mmHg. The next step for
this patient would be assisted ventilation either by CPAP or BiPA (preferred), or intubation and
mechanical ventilation.

Recommendations for oxygen adjustment for patients with an episode of COPD exacerbation is
summarized in the following table:

PaCO2
PaO2 (mmHg) pH Adjustment
(mmHg)
>>60 Reduce oxygen flow to
Normal (36- Normal (7.36-
maintain PaO2 closer to 60
(SaO2>>90% by 44) or high 7.44) or low
mmHg (SaO2 90%-92%) and
oxymetry) (>44) (<7.36)
monitor ABG
Normal (36-
Normal Continue same oxygen flow
44)
Consider assisted
>60 (SaO2>90% ventilation and monitor ABG
by oxymetry) Large rise
Low
(>50) Reduce oxygen flow to
maintain PaO2 at 60 mmHg
(SaO2 88%-92%)
Increase oxygen flow but
Normal or low Normal
monitor ABG
<60 (SaO2< 90% Consider assisted
by oxymetry) ventilation and monitor ABG
Large rise Low

1779 of 1943
>: greater than; >>:substantially greater than

(Option A) Patients with COPD are dependent on hypoxia rather than hypercapnia to increase their
respiratory drive. Increasing the oxygen flow in this patient may initially result in increased levels of
blood oxygen, which results in decreased respiration and deterioration of their hypercapnia.

(Option C) Decreasing the oxygen flow will lead to more hypoxia and deterioration of the condition.

(Option D) Current oxygen flow has failed to provide adequate oxygenation for this patient and has
also resulted in further hypercapnia. Continuation of treatment with the same amount of oxygen
will not benefit the patient.

(Option E) Adding β2 agonists and anticholinergic agents are an essential part of the management
plan, but should be administered with nebulisers. Inhaled drugs are of trivial benefit in COPD
exacerbations.

References

• Therapeutic Guidelines – Respiratory; available from http://tg.org.au

• http://www.uptodate.com/contents/management-of-exa
Last updated:
Time spent: QID:387
2023-2-12

1780 of 1943
A 71-year-old man, known case of COPD for the past 8 years, is brought to the emergency
department by ambulance due to severe difficulty in breathing. On the way to the hospital, he is
given oxygen 100% by nasal cannula. Upon arrival to the emergency department, his oxygen
saturation is 81% and his shortness of breath is even worse, according to the paramedics. Which
one of the following would be the next best step in management?

A. Intubation.
B. Cessation of oxygen.
C. Increasing the oxygen flow.
D. Decreasing the oxygen flow.
E. Starting the patient on bronchodilators.

Incorrect. Correct answer is D


45% answered correctly

Explanation:

Correct Answer Is D

COPD patients has type II respiratory failure, which by definition is hypoxia (PaO2<60mmHg (8.0
kPa)) and hypercapnia (PaCO2> 50mmHg (6.6 kPa)). Under normal condition, both decrease in
PaO2 and increase PaCO2 stimulate respiratory centers in the brainstem and increase the
respiratory drive as a compensatory mechanism, but in chronic type II respiratory failure, such as in
COPD, sustained exposure to high levels of PaCO2, desensitizes respiratory centers to elevated
concentrations of CO2 in the blood. Administration of high-concentration oxygen to these patients
decreases the respiratory drive and results in more hypercapnia, dyspnea, paradoxically decreased
oxygen saturation and respiratory acidosis.

In such patients with deteriorating PaO2 and oxygen saturation despite being on oxygen the next
best step in management is to reduce the delivery of oxygen, provided that the patient is not at risk
of imminent respiratory arrest or severe acidemia (PH<7.3) in which immediate case assisted
ventilation should be considered.

If patient remains hypoxemic despite adequate controlled oxygen delivery (e.g. by Venturi mask)
assisted ventilation such as CPAP or intubation (option A) and mechanical ventilation should be
considered next.

Both increasing the oxygen flow (option C) and cessation of oxygen (option B) will increase the
hypoxemia deteriorate the patient’s condition as the first decreases the respiratory drive even more
while the second deprives him of oxygen.

(Option E) While the respiratory drive is suppressed, administration of bronchodilators is not likely
to be beneficial for this patient.

References

• Therapeutic Guidelines – Respiratory; available on: http://tg.org.au

• UpToDate - Management of exacerbations of chronic obstructive pulmonary disease


1781 of 1943
Last updated:
Time spent: QID:397
2023-2-12

1782 of 1943
A 57-year-old man presents to the emergency department with severe shortness of breath and
tachypnea. As a part of emergency evaluation, an arterial blood gas (ABG) analysis is performed,
the result of which are as follows:

PH: 7.2 (normal 7.35-7.45)


PaCo2: 48 (normal 35-45 mmHg)
PaO2: 50 (normal 80 -100 mmHg)

Which one of the following is the next best step in management?

A. 100% oxygen by Hudson mask.


B. 28% oxygen by Venturi mask.
C. Intubation and ventilation.
D. Positive pressure ventilation.
E. Oxygen 6 litres/minute by nasal prongs.

Incorrect. Correct answer is D


45% answered correctly

Explanation:

Correct Answer Is D

In case of respiratory failure, oxygen supplementation is the next best step in management. This
can be achieved either by high concentration oxygen through nasal cannulae, simple face mask,
reservoir masks or non-rebreathing masks in type I (hypoxemic respiratory failure) or low-
concentration oxygen through Venturi mask in type II (hypercarbic) respiratory failure or for those
patient at increased risk of type II respiratory failure.

As a rule, however, patients with the following conditions should be supported with assisted
mechanical ventilation, either with non-invasive measures (CPAP, BiPAP) (preferred option) or
conventional intubation or mechanical ventilation as the last resort if there is:

persistent or worsening hypoxemia despite supplemental oxygen or inability to achieve an O2


saturation above 90% OR
Worsening hypercapnia (PaCO2>50 mmHg) OR
Worsening respiratory acidosis (PH<7.25) OR
Clinical manifestations of respiratory distress such as:

Severe dyspnea
Tachypnea (RR>30)
Nasal flaring
Accessory muscle use
Tracheal tugging
Recession of suprasternal and intercostal spaces
Pulsus paradoxus
Diaphoresis
Paradoxic motion of the rib cage and abdomen

1783 of 1943
When the patient is not conscious enough to maintain their airway patency or when there is
productive cough, invasive mechanical ventilation (intubation) should be considered first.

This patient has a PH of 7.2 and requires positive pressure assited ventilation by C-PAP or BiPAP
as the most appropriate next step in management.

References

• http://www.merckmanuals.com/professional/critical_

• Davidson’s Principles and Practice of Medicine – 20th Edition – pages 193-195 and 669
Last updated:
Time spent: QID:138
2023-2-12

1784 of 1943
A 78-year-old man is found unconscious in the backyard garden by his wife and is brought to the
emergency department. He was diagnosed with chronic obstructive pulmonary disease (COPD) 7
years ago and is currently on inhaled ipratropium bromide. He also has ischemic heart disease, for
which he takes aspirin, verapamil and statins. On examination, he is drowsy and has a blood
pressure of 110/70 mmHg, pulse rate of 100 bpm, and respiratory rate 24 breaths per minute. On
chest examination, bilateral wheezing is noted. Bedside pulseoxymetry shows oxygen saturation of
85% on room air. An ABG shows PaO2 of 60 mmHg and PaCO2 of 50 mmHg. Which one of the
following is the most likely cause of this clinical picture?

A. Infection.
B. Electrolyte disturbances.
C. Sedative overdose.
D. Left ventricular failure.
E. Right ventricular failure.

Correct
45% answered correctly

Explanation:

Correct Answer Is A

The most likely explanation to this clinical presentation is either hypoxia (the most important
concern) or hypercapnia and CO2 narcosis, or both in the setting of a COPD exacerbation. All the
given options can be potential causes of the decompensation of the COPD in this man, but viral
infections remain the most common underlying etiology.

Electrolyte disturbances (option A) are another important cause of confusion and drowsiness, but
not as common as viral infections.

(Option C) Although sedatives overdose, by suppressing the respiratory drive, can lead to both
hypoxia and hypercapnia, there is nothing in the history pointing towards this etiology as a
possibility.

(Option D) If the left ventricular failure was the cause more pronounced presentation such as
pulmonary edema (basal crackles), and S3 gallop were expected.

(Option E) With right ventricular failure, signs of increased venous pressure such as a distended
jugular vein, peripheral edema, and hepatojugular reflux would be present.

References

• Therapeutic Guidelines – Respiratory; available on: http://tg.org.au

• UpToDate - Management of exacerbations of chronic obstructive pulmonary disease


Last updated:
Time spent: QID:398
2023-2-12

1785 of 1943
Mr. Browny brings her wife to the emergency department because he found her walking in the front
yard in her night gown and speaking to herself incoherently. She is 68 years old and has the
established diagnosis of chronic obstructive pulmonary disease (COPD) made 3 years ago. She
has had 3 episodes of COPD exacerbation in the past 18 months, for which she has been
hospitalized. On examination, she looks confused, with a mini mental status exam (MMSE) score
of 24. Which one of the following investigations is more likely to establish the cause of her
presentation?

A. ABG.
B. Chest X-ray.
C. CT scan of the head.
D. Urinalysis.
E. Basic metabolic panel.

Correct
45% answered correctly

Explanation:

Correct Answer Is A

Patients with exacerbation of COPP have deterioration of the pre-existing hypoxia and hypercapnia,
both of which can cause a confusional state and delirium. With COPD in history, the initial
investigation would always be an arterial blood gas (ABG) analysis to for hypoxia, reflected by a
decreased PaO2 level, or hypercapnia indicated by an elevated PaCO2.

Management of hypoxia always takes precedent over correction of hypercapnia.

(Option B) A chest X-ray is always required for patient with exacerbations of COPD to look for an
underlying cause for the deterioration, but this is not the first step.

(Option C) CT scan of the head may be indicated to exclude intracranial bleeding, tumors, etc, but
as mentioned earlier, ABG always comes first.

(Option D) Urinalysis is considered when urosepsis is suspected. Although there is no comment


about ‘fever’, it should be borne in mind and investigated, as a part of workup plan in elderly
patients with new-onset confusion.

(Option E) Electrolyte disturbances (e.g. hypo/hypernatremia) are other possible causes of


suppressed mental status and confusion, and should always be investigated promptly.

References

• Therapeutic Guidelines – Respiratory; available on: http://tg.org.au

• UpToDate - Management of exacerbations of chronic obstructive pulmonary disease


Last updated:
Time spent: QID:399
2023-2-12

1786 of 1943
Which one of the following is the most common cause of secondary pulmonary hypertension and
cor pulmonale?

A. Emphysema.
B. Bronchiectasis.
C. Pulmonary embolism.
D. Pneumothorax.
E. Foreign body.

Correct
45% answered correctly

Explanation:

Correct Answer Is A

Emphysema as a component of chronic obstructive pulmonary disease (COPD) is the most


common cause of cor pulmonale worldwide and in Australia.

Causes of cor pulmonale are as follows:

Acute

Pulmonary embolism
Exacerbation of chronic cor-pulmonale
Acute respiratory distress syndrome (ARDS)

Chronic

Chronic obstructive pulmonary disease


Primary pulmonary hypertension
Uncontrolled Persistent Asthma
Loss of lung tissue following trauma or surgery
End stage pneumoconiosis
Sarcoidosis
T1-4 vertebral subluxation
Untreated obstructive sleep apnea
Interstitial lung disease
Sickle cell anemia
Bronchopulmonary dysplasia (in infants)
Severe thoracic kyphoscoliosis

References

• http://www.uptodate.com/contents/cor-pulmonale

• http://www.merckmanuals.com/professional/cardiovas
Last updated:
Time spent: QID:400
2023-2-12

1787 of 1943
Which one of the following is the best indicator of development of right heart failure in a patient
with chronic obstructive pulmonary disease (COPD)?

A. Hypoxia.
B. Hypercapnia.
C. Reduced forced vital capacity.
D. Increased jugular venous pressure.
E. Hepatomegaly.

Incorrect. Correct answer is D


45% answered correctly

Explanation:

Correct Answer Is D

Right heart failure is a common complication of long-standing COPD. The underlying


pathophysiology is constriction of pulmonary vasculature resulting in pulmonary hypertension.
Pulmonary hypertension leads to right ventricular failure with the following features:

Increased jugular venous pressure


Hepatomegaly
Ascites
S3 gallop (right-sided)
Ascites (rarely)
Edema, particularly that of the lower limb

Of the given options, raised JVP is the most specific indicator of right heart failure.

(Options A and B) Although hypoxia and hypercapnia may become worse with right heart failure,
they often pre-exist as a result of COPD.

(Option C) Reduced FVC can be seen in severe obstructive and restrictive lung diseases. It is not a
sign of right heart failure.

(Option E) Hepatomegaly is often a late sign of right heart failure, but it may also be present in
other conditions such as portal hypertension, and is not specific to right heart failure.

References

• Medscape - Cor Pulmonale

• MSD Manuals - Cor Pulmonale


Last updated:
Time spent: QID:401
2023-2-12

1788 of 1943
A 72-year-old woman presents to your clinic for medical evaluation. Her past medical history
includes chronic obstruction pulmonary disease (COPD), hypertension and smoking of 30
cigarettes a day for the past 40 years. On examination, there is bilateral ankle edema, raised jugular
vein pressure and positive hepatojugular reflux. Chest exam reveals loud P2 and, globally
decreased air entry and fine bilateral crackles. Which one of the following is the most likely
diagnosis?

A. Right heart failure.


B. Exacerbation of chronic obstructive pulmonary disease.
C. Pneumonia.
D. Cor pulmonale.
E. Left heart failure.

Incorrect. Correct answer is D


45% answered correctly

Explanation:

Correct Answer Is D

With clinical findings of right heart failure (peripheral edema, raised jugular venous pressure and
positive hepatojugular reflux), and a loud P2 indicative of pulmonary artery hypertension, cor
pulmonale would be the most likely diagnosis.

Cor pulmonale refers to hypertrophy or dilatation and/or impaired function of the right ventricle
that results from pulmonary hypertension, caused by the diseases of the lung (e.g. chronic
obstructive pulmonary disease), vasculature (e.g. idiopathic pulmonary arterial hypertension),
upper airway (e.g. obstructive sleep apnea), or chest wall deformities (e.g. kyphoscoliosis).

NOTE - right-sided heart disease due to left-sided heart disease or congenital heart disease is not
considered cor pulmonale.

Cor pulmonale is the failure of the right heart due to a pulmonary pathology as the underlying
cause. Hypoxia leads to constriction of the pulmonary vasculature and pulmonary hypertension.
This eventually results in right heart failure.

Many patients may remain asymptomatic for many years. Others may develop acute onset cor
pulmonale. Once symptomatic, the clinical manifestation can be as follow:

Dyspnea on exertion, fatigue, lethargy, and exertional syncope are the results of an inability to
sufficiently increase cardiac output during exertion because of increased pulmonary vascular
resistance. Dyspnea on exertion generally presents first, followed by fatigue, lethargy, and,
eventually, exertional syncope.

Exertional angina can occur in patients with cor pulmonale, even in the absence of coronary
artery disease. There are two major mechanisms: (1) patients with cor pulmonale have
increased myocardial demand; when myocardial demand increases further due to elevated
transmural wall tension during exertion, the demand may exceed the supply, precipitating
subendocardial right ventricular ischemia. This may be exacerbated by exertional

1789 of 1943
hypoxaemia due to the pulmonary hypertension. (2) angina can be induced during exertion by
dynamic compression of the left main coronary artery due to an enlarged pulmonary artery.
This, however, do not occur until the pulmonary artery trunk diameter exceeds 40 mm.

Anorexia and right upper quadrant discomfort due to passive congestion of the liver and
bowel (in advanced cases).

Increased pulmonic component of the second heart sound (may be palpable)

A narrowly split second heart sound that may be absent if there is a right bundle branch block

A holosystolic murmur at the left lower sternal border characteristic of tricuspid insufficiency

A diastolic pulmonary valve regurgitation murmur (in advanced cases)

As the right ventricle becomes hypertrophic, elevated right-sided pressures can produce a
prominent a wave (corresponding to atrial contraction) in the jugular venous pulse, as well as a
right-sided fourth heart sound and either a left parasternal heave or a downward subxiphoid thrust.
When the right ventricle becomes dilated and fails, the resultant systemic venous hypertension can
produce an elevated jugular venous pressure with a prominent v wave, a right ventricular third heart
sound, peripheral edema and, rarely, ascites. The liver can become enlarged and pulsatile, reflecting
tricuspid insufficiency. Although peripheral edema is often cited as evidence of cor pulmonale, it is
important to keep in mind that peripheral edema can be associated with many of the diseases that
cause cor pulmonale, even in the absence of any hemodynamic abnormalities.

The right-sided murmurs and gallops described above are augmented during inspiration, although
this may be obscured by atrial fibrillation or the underlying disease. As examples, the distant heart
sounds that are characteristic of COPD and the increased soft tissue of morbidly obese patients
who have obstructive sleep apnea make it difficult to identify murmurs and gallops, as well as to
appreciate their augmentation with inspiration.

Patients with end-stage cor pulmonale may develop signs of cardiogenic shock, including
hypotension, tachycardia, oliguria, and cool extremities due to low poor stroke volume. Pulmonary
edema may also occur in late-stage disease because elevated right-sided heart pressures can
cause the interventricular septum to bow to the left, impairing left ventricular diastolic function.

This patient has signs of advanced right heart failure including edema, raised jugular venous
pressure, and hepatojugular reflux. Since she also has COPD, cor pulmonale is the most likely
diagnosis.

References

• http://emedicine.medscape.com/article/154062-overv

• http://www.merckmanuals.com/professional/cardiovas
Last updated:
Time spent: QID:402
2023-2-12

1790 of 1943
A 48-year-old man presents to the Emergency Department with complaints of sudden onset chest
pain that worsens on respiration and shortness of breath. On physical examination, he has a blood
pressure of 110/85 mmHg, a pulse rate of 110 bpm and a respiratory rate of 24 breaths per minute.
His jugular vein is distended and a prominent ‘a’ wave of the jugular vein is noted. On heart
auscultation, there is an S4. Which one of the following is the most likely diagnosis?

A. Pulmonary embolism.
B. Infectious endocarditis.
C. Congestive heart failure.
D. Rheumatic fever.
E. Myocardial infarction.

Correct
45% answered correctly

Explanation:

Correct Answer Is A

It is rare for pulmonary embolism to instantly lead to acute right heart failure; however, if the
embolism is massive enough, a significantly elevated pulmonary hypertension caused by the
massive pulmonary embolism can result in acute right heart failure which presents with raised
jugular venous pressure (JVP), prominent ‘a’ waves, and the emergence of an S4 on auscultation as
a result of an ischemic and/or stiff ventricle.

NOTE - Prominent ‘a’ ways are seen when there is an increased pressure of the right atrium during
atrial systole. It can be seen in:

Tricuspid valve stenosis.


Diastolic right heart failure.
Pulmonary hypertension.

(Option B) Infectious endocarditis usually presents differently with fever and murmurs due to
valvular involvement. Prominent ‘a’ wave and elevated JVP are not common features of infectious
endocarditis.

(Option C) Congestive heart failure is not associated with acute-onset pleuritic chest pain or ‘a-
wave’. However, raised JVP can be noted. An S3 gallop can be present due to volume overload.

NOTE - An S3 is usually normal in children and adults younger than 40 years of age.

(Option D) Myocardial infarction (MI) is not associated with pleuritic chest pain. However, MI
should always be considered as a differential diagnosis and excluded. A massive MI of right
ventricle with subsequent right hear failure can cause non-pleuritic chest pain, raised JVP and 'a'
wave.

(Option E) Acute rheumatic fever is characterized by group A streptococcal infection followed by


the following clinical manifestations:

1791 of 1943
Major manifestations:

1. Migratory arthritis (predominantly involving the large joints)


2. Carditis and valvulitis
3. Central nervous system involvement (e.g. Sydenham chorea)
4. Erythema marginatum
5. Subcutaneous nodules

Minor manifestations:

1. Arthralgia
2. Fever
3. Elevated acute phase reactants such as ESR and CRP
4. Prolonged PR interval

To establish the diagnosis of acute rheumatic fever, 2 major manifestations or one major plus two
minor manifestations are required. This patient does not meet criteria for acute rheumatic fever.

References

• Medscape - Pulmonary Embolism (PE)

• UpToDate - Overview of acute pulmonary embolism in adults


Last updated:
Time spent: QID:403
2023-2-12

1792 of 1943
A 37-year-old man presents to the Emergency Department with fever, malaise, and cough for the
past 10 hours. He mentions that he has had four similar episodes before, but he is quite well in
between the episodes. On examination, he has a blood pressure of 130/90 mmHg, pulse rate of
110 bpm, respiratory rate of 24 breaths per minute and temperature of 38.2°C. On chest
examination, bilateral crackles are heard. Which one of the following is the most likely diagnosis?

A. Asthma.
B. Asbestosis.
C. Allergic bronchitis.
D. Extrinsic allergic alveolitis.
E. Left ventricular failure.

Incorrect. Correct answer is D


45% answered correctly

Explanation:

Correct Answer Is D

Recurrent episodes of flulike symptoms (fever, malaise, cough) associated with acute shortness of
breath are characteristic of extrinsic alveolitis, also known as hypersensitivity pneumonitis. This
disease is caused by a hypersensitivity reaction to specific antigens. The subtypes and specific
antigens are listed in the following table:

Disorder Etiology (antigen)


Farmers’ lung Spores of actinomycetes from moldy hay
Bird fanciers’ lung Antigens from feathers, excreta, droplets
Mushroom workers’ lung Spores of actinomycetes from compost
Malt workers’ lung Spores of Aspergillus clavatus in grain
Grain handlers’ slung Grain weevil dust
Bagassosis Spores of actinomycetes from sugarcane
Air conditioner lung Spores of actinomycetes from air conditioner

Chronic exposure leads to gradual fibrosis of the lungs and bilateral crackles (crepitations).
Removing the offending antigen is the management of choice. Corticosteroids are used to prevent
fibrosis, but ineffective if the source of exposure is not removed.

In early stages of the disease, the patient usually does not have interval symptoms. With
progressive fibrosis, exertional dyspnea, pulmonary hypertension, and other characteristic findings
of interstitial lung disease develops. In chronic cases, the pulmonary function tests will show
intrapulmonary restrictive pattern.

(Option A) Asthma has an intermittent presentation, but the patient does not have fever or malaise.

(Option B) Asbestosis is an occupational lung disease presenting with progressive shortness of


breath, exercise intolerance, and lung fibrosis. There is no intermittent acute exacerbation.
Moreover, fever and malaise are not seen in asbestosis.

1793 of 1943
(Option C) Allergic bronchitis is a variant of asthma in which involvement of bronchi rather than
smaller airways are prominent. The main presenting symptom is cough or, less commonly,
wheezing. Patients allergic bronchitis are not febrile and do not have malaise during attacks.

(Option E) Although left ventricular failure can result in cough, shortness of breath, and bibasal
crepitations, fever and malaise are not expected features. Moreover, the patient is less likely to be
asymptomatic in between exacerbations.

References

• Medscape - Hypersensitivity pneumonitis

• Therapeutic Guidelines – Respiratory


Last updated:
Time spent: QID:407
2023-2-12

1794 of 1943
A 55-year-old man presents to your practice with increasing wheeze and shortness of breath for the
past 2 days. His past medical history includes asthma with recurrent episodes with similar
symptoms controlled with oral corticosteroids. His current medications are inhaled
budesonide and salmetrol. On examination, his blood pressure is 120/80 mmHg, pulse 94 bpm,
and respiratory rate 22 breaths per minute. His oxygen saturation is 94% on room air. Respiratory
examination is remarkable for widespread wheeze. The rest of the examination is inconclusive.
Blood tests shows mild eosinophila and elevated IgE level. Chest X-ray shows central
bronchiectasis and patchy opacities. Which one of the following is the most likely diagnosis?

A. Asthma exacerbation.
B. Autoimmune interstitial lung disease.
C. Bronchiectasis.
D. Chronic bronchitis.
E. Allergic bronchopulmonary aspergillosis.

Incorrect. Correct answer is E


45% answered correctly

Explanation:

Correct Answer Is E

The findings of recurrent asthma exacerbations, patchy opacities and central bronchiectasis on
chest X-ray and eosinophilia and elevated IgE level are suggestive of allergic bronchopulmonary
aspergillosis.

The findings of allergic bronchopulmonary aspergillosis are:

Recurrent episodes of asthma-like attacks


Migratory pulmonary opacities on chest X-ray
Central atelectasis on chest X-ray
Peripheral blood eosinophilia
Elevated IgE levels
Positive skin test for aspergillus fugamitus

Treatment is with oral corticosteroids. Inhaled corticosteroids are not effective. Antifungal agents
may be added.

References

• Medscape - Aspergillosis

• UpToDate - Allergic bronchopulmonary aspergillosis


Last updated:
Time spent: QID:408
2023-2-12

1795 of 1943
A 20-year-old man presents to the Emergency Department with complaints of cough, fever, joint
pain, and malaise for the past few days. He denies night sweats, weight loss, and chills. He does
not smoke. On examination, painful raised erythematosus rashes are observed over his anterior
tibia. The rest of the physical exam is inconclusive. A chest X-ray is done that shows bilateral hilar
lymphadenopathy. Which one of the following is the most likely diagnosis?

A. Bronchiectasis.
B. Chronic obstructive pulmonary disease (COPD).
C. Tuberculosis.
D. Lung cancer.
E. Sarcoidosis.

Incorrect. Correct answer is E


45% answered correctly

Explanation:

Correct Answer Is E

Fever, malaise, joint pain, cough, and bilateral hilar lymphadenopathy are highly suggestive of
sarcoidosis. The painful raised nodule is consistent with description of erythema nodosum which
adds more support to such diagnosis.

Sarcoidosis is a systemic disease of unknown etiology. Histologically, it is characterized by the


presence of non-specific non- caseating granulomas in the lung and other organs. It is more
common among black race with a usual age of onset of 20-30 years.

Sarcoidosis commonly involves the lung but may affect any other organ system. There are two
distinct sarcoid syndromes with acute presentation:

Lofgren syndrome - erythema nodosum, arthritis, and hilar lymphadenopathy


Heerfordt - Waldenstrom syndrome: fever, parotid gland enlargement, uveitis, and facial palsy

(Option A) Bronchiectasis presents with copious sputum production and recurrent chest infections.
The scenario does not fit the diagnosis of bronchiectasis.

(Option B) Chronic obstructive pulmonary disease (COPD) can cause cough, but joint pain and rash
are not associated features.

(Option C) Tuberculosis (TB) can be another diagnosis. TB can cause cough, fever, and erythema
nodosum; however, the history is negative for any risk factors for TB. Moreover, bilateral hilar
lymphadenopathy and arthritis make Lofgren syndrome caused by sarcoidosis more likely than TB.

(Option D) Lung cancer is very unlikely in a non-smoker young patient. On the other hand, erythema
nodosum, fever, and joint pain are not expected findings in lung cancer.

References

• http://emedicine.medscape.com/article/301914-overv

1796 of 1943
• Therapeutic Guidelines – Respiratory
Last updated:
Time spent: QID:409
2023-2-12

1797 of 1943
A 45-year-old non-smoker man comes to your practice with complaint of progressive dyspnea. He
has worked in an insulating factory for the last 15 years. Significant physical findings on physical
examination are clubbing, peripheral cyanosis, and inspiratory crackles. A chest X-ray
shows bilateral calcified opacities in both lung fields. For further visualization of the lesions ,you
order a high resolution CT scan of the chest. The CT is remarkable for calcified pleural plaques and
small opacities in middle lung fields. There is no history of weight loss. Which one of the following
is the most likely diagnosis?

A. Pleural effusion.
B. Mesothelioma.
C. Asbestosis.
D. Small cell lung cancer.
E. Squamous cell lung cancer.

Incorrect. Correct answer is C


45% answered correctly

Explanation:

Correct Answer Is C

The history, clinical findings, and radiological characteristics are consistent with nodular
asbestosis which is a common finding in people with the history of exposure to asbestos. The
absence of weight loss in history makes the bronchogenic carcinomas a less likely possibility.
Asbestosis is highly associated with both non-malignant and malignant lung disease.

Chest X-ray and spiral CT scan in patients with asbestosis show calcified pleural plaques
suggestive of asbestos exposure.

Clacified pleural plaques of asbestosis on CXR

1798 of 1943
Clacified pleural plaques of asbestosis on CT scan

Pleural mesothelioma (option B) is associated with pleural thickening and calcification, pleuritic
chest pain and weight loss, as well as recurrent pleural effusions. There is a connection between
asbestos exposure and mesothelioma, but the most common malignancy associated with
asbestosis is bronchogenic carcinoma, not mesothelioma.

Bronchogenic lung cancers are usually seen after 20 years of exposure to asbestosis. However, in
the absence of weight loss and lack of supporting findings of a tumor on imaging studies, cancers
such as small cell lung cancer (option D) or squamous cell lung cancer (option E) are less likely to
be the diagnosis.

Pleural effusion (option A) can cause shortness of breath. With pleural effusion, the lungs will be
dull to percussion over the affected areas. It is unlikely for pleural effusion to be missed on both
CXR and CT scan.

References

• Therapeutic Guidelines - Respiratory

• Medscape - Asbestosis
Last updated:
Time spent: QID:410
2023-2-12

1799 of 1943
A 37-year-old woman presents to the emergency department with shortness of breath. She is a
known case of asthma and smokes heavily. On pulseoxymetry, she is found to have an O2
saturation of 85% on room air. Which one of the following would be the next best step in
management of this patient?

A. 100% oxygen by Hudson mask.


B. Intubation and mechanical ventilation.
C. 100% oxygen by nasal prongs at 3 litres/minute.
D. Oxygen 28% by Venturi mask.
E. Continuous positive airway pressure.

Incorrect. Correct answer is D


45% answered correctly

Explanation:

Correct Answer Is D

With an O2 saturation of 85% on room hair, this woman is definitely hypoxemic and requires oxygen
supplementation. There is no comment about the patient CO2 status to differentiate whether this
patient has type I or type II respiratory failure, but patients with the following conditions, if
hypoxemic, are at increased risk of hypercapnia if they are given oxygen with FiO2 between 35-60%
(either by nasal cannula or face mask) as the respiratory drive relies on hypoxemia and not
hypercarbia:

1. Known chronic obstructive pulmonary disease or bronchiectasis


2. Severe kyphoscoliosis or ankylosing spondylitis
3. A known history of heavy smoking
4. Severe obstructive sleep apnea
5. Morbid obesity (BMI more than 40 kg/m2)
6. Musculoskeletal disorders with respiratory muscle weakness, especially if on home
ventilation
7. Overdose of opioids, benzodiazepines or other respiratory depressant drugs.

Under above-mentioned condition, the first-line oxygen therapy would be oxygen 24-28% through
Venturi mask. If Venturi mask is not available, oxygen through nasal prongs at a rate of 1- 2
litres/minute could be an acceptable option.

This woman is asthmatic (obstructive) and smokes heavily. High-flow oxygen is very likely to pose
her at the risk of hypercapnia; hence she should be started on low-flow oxygen either through
Venturi mask or 1-2 liter/min oxygen through nasal prongs.

References

• Therapeutic Guidelines – Respiratory; available on: http://www.tg.org.au


Last updated:
Time spent: QID:139
2023-2-12

1800 of 1943
Which one of the following lung cancers is most common in non-smokers?

A. Squamous cell carcinoma.


B. Small cell carcinoma.
C. Adenocarcinoma.
D. Mesothelioma.
E. Large cell carcinoma.

Incorrect. Correct answer is C


45% answered correctly

Explanation:

Correct Answer Is C

95% of all of bronchogenic carcinomas are directly related to cigarette smoking. If non-smokers
develop lung cancer, it is more likely to be of adenocarcinoma type.

References

• Medscape - Non-small Cell Lung Cancer


Last updated:
Time spent: QID:413
2023-2-12

1801 of 1943
A 57-year-old man presents with progressive shortness of breath and cough for the past 4 months.
He has smoked 40 cigarettes a day for the past 30 years and worked in a coal mine for the past 20
years. On chest auscultation, expiratory wheezes are heard over the left lung. The only abnormal
finding on chest X-ray is a 3x4 opacity in the lower lobe of the left lung. Which one of the following
is the most likely diagnosis?

A. Bronchogenic carcinoma.
B. Coal workers’ pneumoconiosis.
C. Mesothelioma.
D. Chronic obstructive lung disease (COPD).
E. Extrinsic allergic alveolitis.

Correct
45% answered correctly

Explanation:

Correct Answer Is A

In this patient, with wheezing over the left lung, indicating local obstruction of the small airways
and a single opacity in the left lower lobe, as well as long-standing history of heavy smoking,
bronchogenic carcinoma would be the most likely diagnosis.

(Option B) Coal workers’ pneumoconiosis is an occupational interstitial lung disease seen in coal
miners. It can present with progressive shortness of breath and respiratory crackles. Chest X-ray
shows small round densities in the parenchyma, usually involving the upper half of the lungs.

(Option C) Mesothelioma is a rare malignancy, which is associated with asbestosis and presents
with pleural thickening on chest X-ray.

(Option D) With the chest X-ray findings, COPD is it less likely. Chest X-ray findings in COPD are
hyper-inflated lungs, flattened hemi-diaphragms if emphysema is dominant and increased
pulmonary markings if the chronic bronchitis is a feature.

(Option E) Although there are no typical findings on chest X-ray characteristic of extrinsic allergic
pneumonitis, just a single opacity on X-ray makes this diagnosis less likely.

References

• http://www.nhmrc.gov.au/_files_nhmrc/publications/

• http://emedicine.medscape.com/article/279960-clini

• http://www.merckmanuals.com/professional/pulmonary
Last updated:
Time spent: QID:414
2023-2-12

1802 of 1943
A 70-year-old man presents with recurrent right-sided pleural effusions and 10kg weight loss in the
past 6 months. He has the history of smoking 40cigarettes a day for the past 40 years. Chest X-ray
shows pleural thickening with no consolidation in the lung fields. Pleural fluid cytology does not
show any malignant cells. Which one of the following is the next best step in management?

A. Thoracoscopy.
B. Repeat the chest X-ray.
C. Permanent intercostal catheter.
D. Diuretics.
E. Fluid restriction.

Correct
45% answered correctly

Explanation:

Correct Answer Is A

The pleural thickening on chest X-ray, the pleural effusion and significant weight loss, in the
absence of any abnormalities in lung fields, are suggestive mesothelioma as the most likely
diagnosis.

More than 90% of patients with pleural mesothelioma present with pleural effusion. Cytologic
examination of the pleural fluid is diagnostic in only 32% of patient, and suggestive in 56%. If a
diagnosis cannot be made based on cytologic exam of the pleural fluid, thoracoscopy-guided
pleural biopsy should be performed as the next best step in management. The results are 98%
diagnostic.

References

• http://emedicine.medscape.com/article/280367-worku
Last updated:
Time spent: QID:415
2023-2-12

1803 of 1943
A 55-year-old man presents with complains of dry cough and Hemoptysis. He had worked in a gold
mine for 15 years and in a petrol station for another 9 years. Physical examination is inconclusive.
A chest X-ray is obtained that is remarkable for a 2 x 3 cm opacity in the right lung field. Which one
of the following would be the next best step in management?

A. Bronchoscopy.
B. Computed tomogram (CT) of the chest.
C. Fine needle aspiration.
D. Ultrasonography.
E. Chemotherapy.

Incorrect. Correct answer is B


45% answered correctly

Explanation:

Correct Answer Is B

Considering the age and the history of working in gold mine and petrol station, this man has
bronchogenic cancer until proven otherwise. For every patient with suspected pulmonary
pathology, chest X-ray is always the best initial diagnostic tool. With an opacity in the right lung
field, two more steps would be ahead: (1) spiral CT scan of the chest to further visualize the lesion
and its extend, and (probably) (2) sputum cytology. Once the lesion is confirmed on the spiral CT,
referral specialist for bronchoscopy or open lung biopsy is the most appropriate step.

References

• http://www.imagingpathways.health.wa.gov.au/index.

• http://www.nhmrc.gov.au/_files_nhmrc/publications/
Last updated:
Time spent: QID:416
2023-2-12

1804 of 1943
Which one of the following is unlikely to cause central cyanosis?

A. Polycythemia.
B. Right-to-left cardiac shunt.
C. Left-to-right cardiac shunt.
D. Acute pulmonary edema.
E. Respiratory depression.

Incorrect. Correct answer is C


45% answered correctly

Explanation:

Correct Answer Is C

All the given options are associated with central cyanosis except left-to-right shunts. With left-to-
right shunts oxygenated blood goes from the left heart to the right heart and no unsaturated blood
enters the systemic circulation to cause cyanosis.

Examples of left-to-right shunt include patent foramen ovale, atrial septal defects, ventricle septal
defects and patent ductus arteriosus.

Right-to-left shunting is seen in transposition of the great vessels, persistent truncus arteriosus and
tetralogy of Fallot.

Polycythemia, decreased respiration and acute pulmonary edema can cause central cyanosis.

References

• http://www.health.vic.gov.au/neonatalhandbook/asse

• http://emedicine.medscape.com/article/303533-overv
Last updated:
Time spent: QID:522
2023-2-12

1805 of 1943
A 27-year-old asthmatic man presents to the Emergency Department with sudden onset of right-
sided pleuretic chest pain. On physical examination, he has a blood pressure of 120/75 mmHg,
pulse rate of 94 bpm and respiratory rate of 18 breaths per minute. Chest auscultation reveals
diminished breath and hyper-resonance in the right lung. No respiratory distress is noted. A chest
X-ray is obtained that is significant for a 20% right-sided pneumothorax. Which one of the following
will be the most appropriate next step in management?

A. Admit the patient for observation.


B. Perform thoracotomy.
C. Needle aspiration.
D. Water-sealed catheter thoracostomy.
E. Discharge the patient and review in 24 hours.

Incorrect. Correct answer is D


45% answered correctly

Explanation:

Correct Answer Is D

Pneumothorax is defined as presence of air in the pleural space. Based on the etiology,
pneumothorax can be defined as traumatic or spontaneous.

Traumatic pneumothorax is caused by blunt or penetrating chest traumas or invasive procedures


(iatrogenic) such as central vein catheterization, pleural biopsy, etc.

Spontaneous pneumothorax is the term used when the condition occurs in the absence of an
apparent trauma or procedure. Spontaneous pneumothorax is classified as primary or secondary
based on absence or presence of an underlying lung pathology or chest X-ray abnormalities.

Primary spontaneous pneumothorax (PSP) – spontaneous pneumothorax is primary if all of


following criteria are met:

There is no respiratory finding on exam except those related to the pneumothorax


There is no history of lung disease
There is no chest X-ray finding other than those related to the pneumothorax
The patient is young (≤50 years)
There is no significant history of smoking

Secondary spontaneous pneumothorax (SSP) – spontaneous pneumothorax is considered


secondary if either of the following is present:

Respiratory findings other than those related to pneumothorax


History of lung disease such as COPD (the most common cause of SSP), asthma, cystic
fibrosis sarcoidosis, TB, lung cancer, chronic bronchitis, bronchiectasis, extra- or
intrathoracic restrictive lung disease, etc
Any chest X-ray abnormality other than those related to pneumothorax
Age >50 years
Significant history of smoking

1806 of 1943
NOTE – the typical patient with PSP is a thin tall man of 20- to 40-years old. PSP is rarely seen
after 40 year and almost never after the age of 50; therefore, spontaneous pneumothorax in
those older than 50 is considered secondary and treated accordingly until proven otherwise.

Size of pneumothorax - Accurate estimation of the size of a pneumothorax is difficult. There


different methods for estimation:

Average interpleural distance (AID) method - approximates the size of a pneumothorax from a
PA CXR in standing position by taking the sum of the distances (measured in millimeters)
between the ribs and the visceral pleura at the apical level (A), midthoracic (B), and basal
level (C), then dividing the sum by 3.

Light Index – An upright PA chest X-ray is obtained. The width of the lung and the hemithorax
are measured (in centimeters). The percentage of pneumothorax is calculated from the
following formula:

Pneumothorax percentage = (1 – (width of the lung)3/(width of the hemithorax)3)x100

For example if the width of the lung and the hemithorax are 5 cm and 10 cm respectively, the
percentage of pneumothorax will be:

1 – (5)3 / (10)3 = 1 – 0.125 = 0.875 or 87.5 % (87.5% of the affected hemithorax is occupied with
the pneumothorax)

These methods are difficult to apply in practice and often underestimate the size of the
pneumothorax. As a result, some clinicians tend to describe a pneumothorax as large or small,
rather than using the percentage.

Chest wall – pleural line distance at the hilum level - British Thoracic Society guidelines define
a pneumothorax as small if the distance from chest wall to the visceral pleural line (at the
level of the hilum) is less than 2 cm or large if the distance from the chest wall to the visceral
pleural line is 2 cm or greater. Some clinicians prefer 3 cm laterally and 4 cm apically as the
threshold to distinguish small and large pneumothoraces.

Symptoms:

Small pneumothoraces are often asymptomatic.


Larger pneumothoraces can present with:

Pleuretic chest pain (pain may be referred to shoulder tip)


Shortness of breath

Clinical findings:

Clinical findings associated with pneumothoraces include:

Decreased breath sounds over the affected area due to decreased air entry
Hyperresonance over the affected area
Decreased tactile fremitus
Tracheal deviation to the affected side

Management:

1807 of 1943
Primary spontaneous hemothorax (PSP) – management of PSP depends on the presence of the
symptoms and/or the size of the pneumothorax:

Consider discharging the patient and review in 24 hours, and every 1-2 weeks until the
pneumothorax is resolved if:

The patient is asymptomatic (or minimal symptoms) AND


The size of the pneumothorax is less than 15% of the affected lung, or the distance
between the chest wall and the visceral plural line is ≤2cm

Consider needle aspiration and REVIEW if:

The patient is symptomatic (pleuretic chest pain or dyspnoea), OR


The size of the pneumothorax is ≥15%of the affected lung, or the distance between the
chest wall and the visceral plural line >2cm

After needle aspiration, admission and catheter drainage (chest tube) would be indicated as
the next best step in management if:

The aspirated air is ≥3 litres


The distance between the chest wall and visceral pleural line is still >2cm on a chest x-
ray taken 4 hours after needle aspiration
There is significant shortness of breath

NOTE – the rationale behind less invasive initial management of PSP is based upon the fact that
the patients with PSP are young and otherwise healthy, and there is no underlying pathology to
perpetuate the pneumothorax or prevent it from spontaneous healing

Seconadary spontaneous pneumothorax (SSP) – in SSP, the underlying lung problem prevents the
pneumothorax from spontaneous healing. All patients with SPS need to be admitted to the hospital
and undergo:

Needle aspiration and REVIEW if there are no significant symptoms AND the size of the
pneumothorax is less than 15% of the affected lung (or the distance between the chest wall
and the visceral plural line is ≤2cm)

Catheter drainage (chest tube) if the patient is symptomatic (pleuretic chest pain or
dyspnoea) OR the size of the pneumothorax is ≥15%of the affected lung (or the distance
between the chest wall and the visceral plural line >2cm)

​Traumatic pneumothorax

Unless the patient is asymptomatic and the size of pneumothorax is <15% (or the distance
between the chest wall and the visceral plural line is ≤2cm), he patient should undergo catheter
drainage (chest tubes). Asymptomatic patients whose pneumothorax is 15% (or the distance
between the chest wall and the visceral plural line is ≤2cm) cab be closely observed for
spontaneous resolution of the pneumothorax.

NOTE – pneumothorax of any kind and size should be treated with chest catheter (chest tube)
insertions if:

The patient is undergoing general anaesthesia for any reason


The patient is planned to be intubated and mechanically ventilated
1808 of 1943
The patient is planned to be transported by air (air transport)

With the history of asthma, this patient has SSP until proven otherwise. Furthermore, not only is
he is symptomatic (pleuretic chest pain), but the size of the pneumothorax is also large. The best
management option for him will be water-sealed chest catheter thoracostomy (chest tube
insertion).

References

• UpToDate - Treatment of secondary spontaneous pneumothorax in adults

• Merck Manuals - Pneumothorax

• AMC Handbook of Multiple Choice Questions – page 179


Last updated:
Time spent: QID:527
2023-2-12

1809 of 1943
A 68-year-old woman, with history of stage IV chronic kidney disease (CKD) presents to the
Emergency Department with sudden-onset acute chest pain and shortness of breath. The chest
pain is pleuretic in nature. On examination, she has a blood pressure of 130/90 mmHg, pulse rate
of 120 bpm and respiratory rate of 24 cycles per minute. ECG shows no abnormalities. A chest X-
ray is obtained that is completely clear. Ventilation/perfusion scan is performed
showing decreased perfusion in the lower lobe of the left lung. Which one of the following is the
most appropriate next step in management?

A. Low-molecular weight heparin (LMWH).


B. Thrombolysis.
C. CTPA.
D. Unfractionated heparin.
E. Warfarin.

Incorrect. Correct answer is D


45% answered correctly

Explanation:

Correct Answer Is D

Clinical findings suggest pulmonary embolism (PE) as the most likely diagnosis. Decreased
perfusion confirms the diagnosis. V/Q scan can be reliable if there is no pre-existing underlying
lung pathology leading to misinterpretation of the result. The validity of the V/Q scan in this
scenario can be trusted as there is no abnormal finding on chest X-ray.

Although both unfractionated heparin and low-molecular weight heparin (LMWH) have about the
same efficacy for treatment of PE, unfractionated heparin is the preferred option in patients with
renal disease because LMWH is excreted renally.

(Option A) LMWH is better avoided in patients with renal disease. If LMWH is considered for
treatment, the dose should be adjusted according to glomerular filtration rate (GFR).

(Option B) Thrombolysis is reserved for selected patients with hemodynamic instability, in whom
there is no contraindication to thrombolysis.

(Option C) Since V/Q scan has already established the diagnosis, CTPA would not be required.
Furthermore, CTPA needs intravenous injection of contrast material that is contraindicated in
patients with renal impairment. This is the reason why V/Q scan has been used for the diagnosis of
PE in the first place.

(Option E) Warfarin is a part of management after INR reaches the therapeutic dose of 2-3.
Warfarin has delayed action, and has initial pro-thrombotic effects. Anticoagulation never starts
with warfarin alone.

References

• MJA - Venous thromboembolism: diagnosis and management of pulmonary embolism

1810 of 1943
• RACGP - Focus: Pulmonary embolism

• Medscape - Pulmonary Embolism (PE)


Last updated:
Time spent: QID:658
2023-2-12

1811 of 1943
A 47-year-old man has developed cough and dyspnea over the past few months. There is no history
of smoking or occupational exposure. Marked findings on examination are clubbing of the fingers
and inspiratory crepitations. Which one of the following is the most likely diagnosis?

A. Mesothelioma.
B. Sarcoidosis.
C. Pulmonary fibrosis.
D. Lung cancer.
E. Mycoplasma infection.

Incorrect. Correct answer is C


45% answered correctly

Explanation:

Correct Answer Is C

The history and physical findings are characteristic of interstitial lung disease (IDL).

Interstitial lung diseases are a group of heterogenous lung diseases with different clinical,
physiologic, radiographic, and pathologic presentations. However, a number of common features
justify their inclusion in a single disease category.

IDL is characterized by chronic inflammation and fibrosis of the interstitium and lung parenchyma.

The interstitium of the lung (supporting structure) is the area in and around the small blood vessels
and alveoli where the exchange of oxygen and carbon dioxide takes place. Inflammation and
scarring of the interstitium, and eventually extension into the alveoli, will disrupt normal gas
exchange. Although the progression of IDL may be variable from one disease to another, they have
common clinical, radiographic, and spirometric findings:

Dyspnea - all patients with IDL develop exertional dyspnea that is the most common
complaint bringing them to medical attention
Nonproductive cough
Bibasal inspiratory crackles
Evidence of pulmonary hypertension (in advanced disease) such as increased pulmonic
sound, right heart failure, etc.
Clubbing (not always) especially in idiopathic pulmonary fibrosis and asbestosis
Reticular or reticulonodular pattern (‘ground-glass appearance) on chest X-ray
Intrapulmonary restrictive pattern and decreased DLCO on pulmonary function testing

Of the given options, only pulmonary fibrosis as an ILD fits the scenario.

(Option A) Mesothelioma is a rare malignancy of the pleural and peritoneum, highly associated
with asbestos exposure. It presents with weight loss, recurrent pleural effusions, and pleural
thickening and calcification on imaging. Neither the history, nor the clinical findings is consistent
with mesothelioma.

(Option B) Sarcoidosis is a multisystem disease characterized by non-caseating granuloma


formation in different organ systems. Pulmonary system is the most common affected system.
1812 of 1943
Although pulmonary sarcoidosis is classified as ILD, most patients with pulmonary involvement
(90%) are asymptomatic. Symptoms, if present, resemble those of pulmonary TB including
productive cough, hemoptysis, and constitutional symptoms such as weight loss, fever, night
sweats and malaise.

(Option D) Absence of smoking in the history makes lung cancer less likely. Moreover, lung cancer
is associated with more systemic manifestation such as weight loss.

(Option E) Mycoplasma infection such as TB or atypical pneumonia caused by mycoplasma


pneumoniae presents with a completely different picture. Pulmonary TB presents with signs and
symptoms such as cough, weight loss, night sweats and malaise. Mycoplasma pneumoniae
presents with a prodromal syndrome followed by dry cough, sore throat, and low-grade fever. None
of these infections are associated with clubbing.

References

• Medscape - Interstitial (Nonidiopathic) Pulmonary Fibrosis

• UpToDate - Approach to the adult with interstitial lung disease: Diagnostic testing
Last updated:
Time spent: QID:699
2023-2-12

1813 of 1943
A 28-year-old woman presents to the Emergency Department with sudden onset dyspnea and
pleuretic chest pain. She is 18 weeks pregnant and has a previous history of deep venous
thrombosis (DVT). Which one of the following would be the most appropriate initial investigation in
this patient leading to a definite diagnosis?

A. D-dimer assay.
B. Ventilation/perfusion scan.
C. ECG.
D. CTPA.
E. Doppler ultrasound scan.

Incorrect. Correct answer is B


45% answered correctly

Explanation:

Correct Answer Is B

Investigation for pulmonary embolus (PE) in pregnancy is a complicated issue. PE is the leading
cause of death in pregnancy in the developed world. Pregnant women have all the 3 components
of the Virchow’s triad that are (1) venous stasis, (2) vessel damage and, (3) hypercoagulable state.

Difficulty and confusion in assessment of PE in a pregnant woman is due to the following:

Normal physiological changes in pregnancy such as dyspnea, tachycardia, and leg swelling
are also symptoms that PE can present with.

Pre-test probability tests such as Wells score, simplified Wells score (SWS) or revised Geneva
score, cannot be applied for a pregnant woman because pregnant women were excluded
from the analysis group for criteria validation.

The d-dimer will start to rise in the second trimester and remain elevated for 4-6 weeks post-
partum; therefore, it is not a reliable test to use as a screening tool.

If PE is suspected, it is important to make a definitive diagnosis, as management of a PE during


pregnancy needs to include choice of anticoagulation, mode of delivery, and consideration of
prophylaxis in future pregnancies. There are debates as to optimal methods of diagnostic work-up
for pregnant women with suspected PE. For PE, we use latest guidelines by The Royal Australian
and New Zealand College of Radiologists as the most accurate and reliable reference for the exam.

The sudden-onset chest pain and shortness of breath and previous history of DVT make PE a very
likely diagnosis. If PE is suspected in a pregnant woman, a chest radiograph (not an option) should
be performed as the most appropriate initial step to exclude other causes of the symptoms such
as pneumonia, and to determine if V/Q scan or CTPA should be performed as the diagnostic test
for suspected PE. If the chest radiograph is clear, VQ scan using a lower dose technique is
recommended because it is associated with a lower breast dose than CTPA and a comparably
small dose to the fetus.
1814 of 1943
(Option A) D-dimer levels become elevated in most patients after the first trimester. Normal values
for each trimester have yet to be validated in a large patient population to enable them to be used
to screen for the possibility of pulmonary embolism during pregnancy in routine clinical practice.
Therefore, D dimer is generally not used to screen pregnant women for suspected PE, particularly
during the second and third trimester when it is physiologically elevated.

(Option C) Ischemia is among differential diagnoses of chest pain. ECG may be considered to
exclude cardiac causes of the chest pain, and also to look for ECG changes favoring PE as a
diagnosis. Given the sudden-onset of chest pain and shortness of breath in an otherwise healthy
young woman who has risk factors for PE, ECG is not likely to be of significant benefit. The
pleuretic nature of the pain, on the other hand, makes cardiac ischemia almost unlikely; however, it
may be considered in the initial work-up along with chest radiograph. It is not a priority in this
woman though.

(Option D) CTPA is associated with increased risk of breast cancer in women of reproductive age
and should be replaced with V/Q scan in this patient group if possible. The most common
indication for CTPA instead of V/Q scan is an abnormal chest radiograph.

(Option E) Lower limb venous ultrasound is negative in more than 90% of pregnant women with PE.
Lower limb venous ultrasound is not recommended as a first line investigation unless there are
clinical symptoms of DVT (e.g., unilateral leg swelling, calf tenderness). This is due to the very low
rate of lower limb deep venous thrombosis in pregnant women with PE. Almost no V/Q scans or
CTPAs would be avoided in pregnant women with possible PE by performing ultrasound first,
because ultrasound is almost invariably negative in the absence of specific leg symptoms in favor
of DVT. Therefore, the reduction in radiation exposure to the population of women with possible PE
and their fetuses that would result from an “ultrasound first” strategy is negligible. However, with
leg symptoms, Doppler ultrasound is the most appropriate initial investigation.

References

• Diagnostic Imaging Pathways - Pulmonary Embolism (Pregnancy)

• Medscape - Thromboembolism in Pregnancy


Last updated:
Time spent: QID:750
2023-2-12

1815 of 1943
Which one of the following is the most appropriate initial investigation in a patient with suspected
pulmonary embolism?

A. Chest X-ray.
B. CTPA.
C. V/Q scan.
D. D-dimer assay.
E. Doppler ultrasound scan.

Correct
45% answered correctly

Explanation:

Correct Answer Is A

When pulmonary embolism is suspected based on history and clinical findings, chest X-ray is the
most appropriate investigation to consider first. There are two objectives; the first one is to exclude
other lung pathologies that might have similar presentation. The second is to determine whether a
V/Q scan or is CTPA is the most appropriate next investigation, because patients with an abnormal
chest X-rays are more likely to have a non-diagnostic V/Q scan than those with a normal chest X-
rays.

An ECG would be indicated if there is chest pain which cannot be solely attributed to PE as the
cause.

After other possible conditions are excluded by chest X-ray and ECG, a clinical pretest probability
(PTP) assessment using Well score system, simplified Wells score, or revised Geneva scoring
system should be performed. D-dimer assay is then used, as a screening test, to exclude PE
in patients with low to moderate likelihood of having PE.

(Options B and C) CTPA is considered the imaging modality of choice for definite diagosis of PE,
provided that it is available and there is no condition on the patient’s side to make its use
undesirable (e.g. pregnancy, women of reproductive age, allergic reaction to contrast media, renal
impairment, etc), in which case V/Q should be considered the first-line diagnostic modality..

(Option D) D-dimer assay is indicated in patients with low and moderate pretest probability. A
negative D-dimer excludes PE, but positive results will require further assessment with CTPA or
V/Q scan. Even if D-dimer is considered, an initial chest X-ray is required to estimate the pretest
probability.

(Option E) Doppler ultrasound is used if there are leg symptoms suggesting a DVT as the source of
PE. However, CXR remains the most important initial investigation.

References

• RANZCR - Using clinical decision rules to guide appropriate imaging for suspected pulmonary
embolism.

1816 of 1943
• RACGP - Focus: Pulmonary embolism

• Medscape - Pulmonary Embolism (PE)


Last updated:
Time spent: QID:751
2023-2-12

1817 of 1943
A 45-year-old man presents to your office concerned about asbestosis. He is a construction worker
and gets involved in demolitions of old houses for renovation. He has read smewhere that old
houses are a source of asbestos and that asbestosis is a serious condition associated with cancer.
He says that he and his coworkers do not use protective measures during the work. Which one of
the following would be the most appropriate action to take in this situation?

A. Refer him to respiratory specialist.


B. Chest X-ray now and in 5 years’ time.
C. Inform the authorities about the breach of workplace safety.
D. Do nothing.
E. Inform him that asbestos carries a very low risk of cancer.

Incorrect. Correct answer is B


45% answered correctly

Explanation:

Correct Answer Is B

Asbestos has been linked to an increased risk of bronchogenic carcinoma, mesothelioma,


gastrointestinal cancers, and asbestosis among occupationally-exposed workers. Adequate
screening for asbestos-related diseases does not exist at present. However, some tests,
particularly chest X-rays and pulmonary function tests, may indicate that an individual has been
overexposed to asbestos. It is important for physicians to become familiar with the operating
conditions that are associated with potential asbestos exposure. This is particularly important in
evaluating medical and work histories and in performing physical examinations.

The current recommendations include taking a complete history, performing physical exam with
close attention to respiratory system and the following diagnostic measures:

Chest X-ray – A chest X-ray should be taken for all patients with exposure to asbestos;
however, initially, the chest X-rays are often inconclusive because it takes approximately 3-
5 years for radiographic signs of asbestos exposure to be evident on X-rays. It is
recommended that chest X-rays be repeated at 3-5 years intervals.

Pulmonary function tests (PFTs) – simple PFTs available at general practice level should be
applied to recognize the possible restrictive patterns caused by asbestosis.

If the condition is work-related, involvement of a respiratory or occupational physician is needed to


consider factors such as the effect of the exposure on other workers, how to control the exposure
at the workplace and appropriate use of personal protective equipment.

When a current worker has been identified to have been overexposed to asbestos, measures taken
by the employer to eliminate or mitigate further exposure should also lower the risk of serious long-
term consequences. The employer is required to institute a medical surveillance program for all
employees who are or will be exposed to asbestos at or above the permissible exposure limit (0.1
fiber per cubic centimeter of air).
1818 of 1943
NOTE – Although most patients with asbestos-related lung disease have a strong exposure
history, significant disease can occur with even minimal exposure; therefore, even minor or
accidental exposures mandates evaluation.

Referring the patient to respiratory specialist (option A) is not necessary at this stage because he
is asymptomatic and does not seem to have any issues right now. The surveillance program can be
carried out by the general practitioner.

Infromin the case to authorities for the breach of workplace safety (option C) may be considered if
there is a significant risk that other employees are at risk of harm inflicted on their health. At this
stage, however, the most important step is to perform an initial evaluation and setting a
surveillance plan for thes patient.

(Option D) While this patient is in need of assessment and being monitored, doing nothing is the
most inappropriate action to take.

(Option E) Asbestos exposure and asbestosis are significant risk factor for bronchogenic cancer
and mesothelioma, and telling the patient that such correlation is not strong and there is only a
slightly increased risk of cancer is incorrect.

References

• AAFP - Asbestos-Related Lung Disease

• ATSDR - Asbestos Toxicity Clinical Assessment - Tests

• ATSDR - Asbestos Toxicity: Patient Education Care Instruction Sheet


Last updated:
Time spent: QID:144
2023-2-12

1819 of 1943
A 26-year-old woman presents to the Emergency Department with dyspnea and sudden-onset chest pain that
worsens with respiration. On examination, she has a blood pressure of 130/95 mmHg, pulse rate of 110 bpm
and respiratory rate of 24 breaths per minute. Her family history is significant for protein C deficiency in her
mother and older sister. Which one of the following is the most diagnostic investigation in this patient?

A. Doppler ultrasound scan.


B. Chest X-ray.
C. V/Q scan.
D. CTPA.
E. D-dimer assay.

Incorrect. Correct answer is C


45% answered correctly

Explanation:

Correct Answer Is C

Family history of thrombophilia is associated with increased risk of venous thromboembolism (moderate risk
factor). With this risk factor present in the history, acute-onset pleuretic chest pain (worsened by respiration)
and shortness of breath, pulmonary embolism (PE) would be the most likely possibility.

Not all patients presenting with possible symptoms of PE need to undergo imaging tests such as computer
tomography pulmonary angiography (CTPA), V/Q scan, Doppler ultrasound, etc. Decision as to whether this
tests are performed depends on pretest probability of PE. A reasonable way to stratify patient risk is to use
one of the validated clinical decision rules (CDR). Simplified Wells Score (SWS) is one of recently validated
ones.

SWS takes into account different variables to generate a score. In SWS, each element in the CDR is scored 1
point. The cut-off value distinguishing between “PE likely” and “PE unlikely” is 2, so patients with a score of 1
or 0 are classified as “PE unlikely” (chance of having PE ≤10%), while those with scores 2 or above are
classified as PE likely.

Elements for calculation of SWS and an algorithmic approach based on SWS score are shown in the following
table and diagram respectively:

Variable points
Clinical signs and symptoms of DVT (minimum of leg swelling and pain elicited
+1
upon deep vein palpation)
No alternative diagnosis more likely than PE +1
Heart rate >100 +1
Immobilization at least 3 days or surgery in previous 4 weeks +1
Previous DVT or PE +1
Hemoptysis +1
+1
Malignancy (on treatment or treated in the past 6 weeks or palliative)

1820 of 1943
The question however asks about the most diagnostic test and not the approach sequence. CTPA and V/Q
scan are used to confirm the diagnosis of PE. Diagnostic accuracy of CTPA and V/Q scan is similar. However,
CTPA detects clots in smaller vessels. CTPA may have the advantage of widespread availability where V/Q
scanning may not be available outside working hours. Radiation dose of V/Q is significantly less than
CTPA, making V/Q scan the preferred option for women of reproductive age (<55 years) to avoid the risk of
breast cancer associated with higher radiation exposure in CTPA.

(Option A) Doppler ultrasound, when leg symptoms associated with deep vein thrombosis (DVT) are absent, is
not very helpful to establish a diagnosis.

(Option B) Chest X-ray is the initial imaging in all patients with suspected PE to exclude other pathologies and
guide further assessment. However, the chest X-ray findings are often non-specific and non-diagnostic.

(Option D) CTPA is a very useful diagnostic test for PE, but not preferred over V/Q scan in this young woman
because it is associated with more radiation exposure and increased risk of breast cancer.

(Option E) D-dimer assay is used to exlcude PE in patients, who are unlikely to have PE, based on pretest
probability. It is sensitive enough to be used as a screening test, but not specific to establish a diagnosis.

References

• RACGP - AFP - Pulmonary embolism: assessment and imaging

• RANZCR - Using clinical decision rules to guide appropriate imaging for suspected pulmonary embolism.

• Medscape - Pulmonary Embolism (PE)

• Diagnostic Imaging Pathways - Pulmonary Embolism (Haemodynamically Stable)


Last updated:
Time spent: QID:752
2023-2-12

1821 of 1943
A 45-year-old man, who had a hip replacement surgery 2 weeks ago, presents with sudden-onset chest pain
and shortness of breath. Pulse oxymetry reveals an oxygen saturation of 89% on room air. On physical
examination, his blood pressure is 100/75mmHg, heart rate 113 bpm, respiratory rate 23 breaths per minute
and temperature 37.3°C. Which one of the following is the most appropriate next step in management of this
patient?

A. CTPA.
B. V/Q scan.
C. ECG.
D. Chest X-ray.
E. D-dimer assay.

Incorrect. Correct answer is C


45% answered correctly

Explanation:

Correct Answer Is C

Both major surgery and possible immobilization of longer than 3 days are strong risk factors for
thromboembolism; therefore, with sudden-onset chest pain, shortness of breath and hypoxia, PE tops the list
of differentials. CTPA and V/Q scan are used to confirm the diagnoses; however, not every patient with clinical
manifestations of PE and a suggestive history should undergo imaging studies, unless other potential causes
of the presentation have been excluded.

Although the acute onset of the chest pain is not in favor of ischemic heart problems such as acute coronary
syndrome, an ECG should be considered the very first step to exclude cardiac problems as a cause.
Moreover, some ECG findings such as Q1S3T3 (Q wave in lead I, prominent S wave and inverted T-wave on lead
III) or T –wave inversion in leads V1 through V4, although non-specific, might support PE as diagnosis (but not
diagnostic).

(Options A and B) CTPA and V/Q scan are used for definite diagnosis of PE in this patient when other potential
causes are excluded.

(Option D) Once cardiac disease is excluded, CXR comes next in line to exclude other potential causes, as well
as to guide further diagnostic approach regarding selection of CTPA versus V/Q. Patients with baseline CXR
abnormalities are more likely to have equivocal V/Q scan results.

(Option E) D-dimer assay is used to exclude PE in patients with low pretest probability of PE. A negative D-
dimer has a high negative predictive value in exclusion of PE. In post-surgical patients, however, D-dimer levels
are often high, making this test of less importance. Furthermore, with a pulse rate of >100bpm and the history
of surgery, this man has a simplified Wells score of 2. Patients with simplified Wells score of >1 do not need D-
dimder assay.

TOPIC REVIEW

Definite diagnosis of PE relies on imaging studies such as CTPA or V/Q scan as the most diagnostic
investigations for PE, and are used to establish the diagnosis with high pretest probability or those with
moderate prettiest probability and positive D-dimer results. However, not everyone with signs and symptoms
of PE should undergo diagnostic imaging just based on the presentation.

1822 of 1943
To avoid unnecessary imaging studies on one hand, and missing patients with PE as a potential fatal
condition on the other, clinical decision rules (CDRs) are devised to help clinicians identify patients who are
likely or unlikely to have PE and to avoid performing imaging in those who have low or intermediate risk of
having PE.

In a nutshell, CDRs are used to:

Avoid unnecessary exposure to ionizing radiation.


Reduce financial cost to the patient and health system of unnecessary testing.
Minimizing the frequency of encountering incidental findings on imaging that often have no clinical
significance for the patient but trigger further imaging follow up to exclude the very small possibility of
something significant.
Select a test other than medical imaging that may be faster and less expensive for ruling out clinically
important pathology.

There are a number of validated CDRs for clinical uses with Wells and revised Geneva being most commonly
applied in clinical situations.

More recently, the Simplified Wells Score (SWS) is developed. In SWS each element in the CDR is scored 1
point. The cut off value distinguishing between “PE likely” and “PE unlikely” is 2, so patients with a score of 1
or 0 are classified as “PE unlikely” (chance of having PE ≤10%), while those with scores 2 or above are
classified as PE likely.

Simplified Wells Score

Variable points
Clinical signs and symptoms of DVT (minimum of leg swelling and pain elicited
+1
upon deep vein palpation)
No alternative diagnosis more likely than PE +1
Heart rate >100 +1
Immobilization at least 3 days or surgery in previous 4 weeks +1
Previous DVT or PE +1
Hemoptysis +1
Malignancy (on treatment or treated in the past 6 weeks or palliative) +1

NOTE – With the following conditions present, Wells (either formal or simplified) should not be used
(exclusion criteria) for pre-test probability assessment:

Suspected upper extremity DVT as source of PE


No symptoms of PE for more than 3 days before presentation
Use of anticoagulation for more than 72hrs
Expected survival less than 3 months
Contraindication to contrast media
Pregnancy

The following diagram shows an algorithmic approach to patients with suspected PE according to the risk of
PE calculated by simplified Wells score:

1823 of 1943
References

• Medscape - Pulmonary Embolism (PE)

• RANZCR - Using clinical decision rules to guide appropriate imaging for suspected pulmonary embolism.

• RACGP - Focus: Pulmonary embolism


Last updated:
Time spent: QID:753
2023-2-12

1824 of 1943
A 40-year-old woman comes to your clinic with complaint of acute onset shortness of breath. She
had one round of chemotherapy 3 weeks ago with both chest exam and X-ray being normal at that
time. You assess her and found that her clinical Well score is 4. Which one of the following would
be the diagnostic investigation of choice in this patient?

A. D-dimer assay.
B. CTPA.
C. Chest X-ray.
D. Doppler ultrasound scan.
E. V/Q scan.

Incorrect. Correct answer is E


45% answered correctly

Explanation:

Correct Answer Is E

Not all patients presenting with possible symptoms of pulmonary embolism (PE) need to undergo
imaging tests such as computer tomography pulmonary angiography (CTPA), V/Q scan, Doppler
ultrasound, etc. Decision as to whether this tests are performed depends on pretest probability of
PE. A reasonable way to stratify patient risk is to use one of the validated clinical decision rules
(CDR). simplified Wells Score (SWS) is one the most commonly used CDR for this purpose. It takes
into account different variables to generate a score, based on which the probability of PE will be
estimated to be low, moderate or high. The variables and their designated score are according to
the following table:

Wells score
Variable Points
Predisposing factors
Previous DVT or PE + 1.5
Recent surgery (within 4 weeks) or
+ 1.5
immobilisation (>3 days)
Malignancy +1
Symptoms
Hemoptysis +1
Clinical signs
Heart rate > 100 bpm +1.5
Clinical signs of DVT +1.5
Clinical judgment
Alternative diagnosis less likely than DVT +3

Clinical probability of PE based on Wells score:


1825 of 1943
Wells score Probability of PE
0-1 Low
2-6 Intermediate
≥7 High

Or

Wells score ≤ 4: PE unlikely


Wells score >4: PE likely

Those with low or intermediate pretest probability should have a D-dimer assay as the most
appropriate step. D-dimer, a degradation product of cross-linked fibrin, is elevated in the presence
of clot because of the activation of fibrinolysis. A negative D-dimer using a quantitative enzyme-
linked immunoabsorbent assay (ELISA) has a sensitivity of >95% and effectively excludes PE in
low- and intermediate-probability groups. However, D-dimer levels are elevated after surgery, in
cancer patients, and during pregnancy and cut-off levels considered for other patients may not be
as reliable. In the presence of cancer D-dimer is not a useful tests and imaging studies should be
considered.

CTPA and V/Q scan are used to confirm the diagnosis of PE. Diagnostic accuracy of CTPA and V/Q
scan is similar. However, CTPA detects clots in smaller vessels. CTPA may have the advantage of
widespread availability whereas V/Q scanning may not be available outside working hours.
Radiation dose of V/Q is significantly less than CTPA, making V/Q scan preferable for women of
reproductive age (<55 years) to avoid the risk of breast cancer associated with higher radiation
exposure in CTPA, Provided that the patient has a clear chest X-ray and no underlying lung
pathology to make V/Q scan results equivocal.

NOTE - Although CXR is the initial test of choice for every patient with suspected PE, it is only used
to exclude other causes of such presentation as well as eligibilty for V/Q scan. It is not diagnostic
for PE. However, V/Q or CTPA are not used as first tests for every patient with suspected PE, unless
there is a high index of suspicion based on pre-test probability.

References

• RANZCR - Using clinical decision rules to guide appropriate imaging for suspected pulmonary
embolism.
Last updated:
Time spent: QID:754
2023-2-12

1826 of 1943
You are called to see a 60-year-old man presenting with dyspnea and chest pain that worsens on
inspiration. Five days ago, he underwent laparotomy and gastric surgery. On physical examination,
he has a temperature of 37.5°C, heart rate of 90 bpm and blood pressure of 130/95 mmHg. Chest
auscultation reveals no abnormal heart or respiratory sounds. There is good air entry into both
lungs and the percussion note is resonant in all areas. A chest X-ray and a ventilation/perfusion
scan are performed. Which one of the following test results indicates a high probability of
pulmonary embolus in a particular zone of the lung?

A. Normal chest X-ray, abnormal ventilation, normal perfusion scan.


B. Normal chest X-ray, normal ventilation, reduced perfusion.
C. Consolidation on chest X-ray, normal ventilation, reduced perfusion.
D. Consolidation on chest X-ray, abnormal ventilation scan, normal perfusion scan.
E. Normal chest X-ray, abnormal ventilation scan, reduced perfusion scan.

Incorrect. Correct answer is B


45% answered correctly

Explanation:

Correct Answer Is B

Chest radiographs are the first imaging modality in assessment of a patient with suspected
pulmonary embolism (PE) to exclude other pulmonary causes of the patient’s presentation.

Initially, chest X-rays are normal in most cases of PE; however, in later stages, most patients
will develop chest X-ray abnormalities including, atelectasis, small pleural effusions, parenchymal
opacities and elevation of the hemidiaphragms.

Once pulmonary infarction occurs, the classic radiographic finding is a wedge-shaped, pleural-
based triangular opacity with an apex pointing towards the hilus (Hampton hump). This finding is
neither common, nor specific. Another infrequent and non-specific finding is decreased pulmonary
vascularity and dilated pulmonary vessels (Westermark sign). In general chest X-rays cannot be
used to include or exclude PE; however, radiography and ECG may be useful for excluding
alternative diagnoses.

Ventilation/perfusion scan (V/Q scan) or radioisotope lung scanning is performed using injecting
of particles of albumin labeled with technetium-99m. As these particles perfuse the lung, the lungs
are imaged by using a gamma camera to obtain anterior, posterior, lateral, and oblique views. In
normal lungs, the isotope particles distribute evenly and produce two dark lung-shaped shadows.
In PE, the embolus blocks the branches of the pulmonary artery producing a filling defect on the
scan. When findings in the perfusion scan are abnormal, a ventilation scan is obtained by using
inhalational radioactive xenon-133. Uncomplicated PE does not make any alteration in ventilation
scan. As a result a patient with a high probability of PE has an abnormal perfusion scan with a
normal ventilation component.

Any pre-existing lung abnormality such as COPD, pneumonia, atelectasis, etc, by altering the
ventilation scan results, can make interpretation of a V/Q scan difficult and inconclusive; hence, a
clear chest X-ray is the essential prerequisite for performing a V/Q scan.

1827 of 1943
With PE, the findings on V?Q scan of this patient at early stages of PE will be a normal chest X-ray,
normal ventilation scan, and reduced perfusion on perfusion scan.

References

• Medscape - Pulmonary Embolism (PE)

• Medscape - Pulmonary Hypertension Imaging


Last updated:
Time spent: QID:805
2023-2-12

1828 of 1943
A 34-year-old man presents to the Emergency Department with acute-onset shortness of breath.
Medical history is only remarkable for essential thrombocytosis. On examination, he has a blood
pressure of 110/70 mmHg, pulse rate of 90 bpm, respiratory rate of 22 breath per minute, and
temperature of 37.1°C. He has a BMI of 24 kg/m2. Chest radiograph is significant for a prominent
right hilum. With the provisional diagnosis of pulmonary embolism, a CT pulmonary angiogram
(CTPA) is ordered which is shown in the following photograph. Which one of the following is the
most appropriate treatment option for this patient?

A. Warfarin.
B. Unfractionated heparin or low-molecular weight heparin (LMWH).
C. Caval filter.
D. Thrombolytic therapy.
E. Dabigatran.

Incorrect. Correct answer is B


45% answered correctly

Explanation:

Correct Answer Is B

The photograph shows a filling defect in the right branch of the pulmonary artery consistent with
the diagnosis of pulmonary embolism (PE).

PE may only present with shortness of breath and tachypnea. However, other symptoms and signs
such as pleuretic chest pain, cough, tachycardia and, in severe cases, hemodynamic instability may
be present.

With confirmed PE, the patient should be started on unfractionated heparin or low-molecular
weight heparin (LMWH) as the most appropriate next step in management.

(Option A) Warfarin should be commenced within 48 hours of heparin administration and


continued for at least 3 months or more, depending on individual risk of recurrent PE or DVT in the
future.

(Option C) Inferior vena cava (IVC) filters provide a screen in the inferior vena cava, allowing blood
to pass through, while large emboli from the pelvis or lower extremities are blocked or fragmented
1829 of 1943
before reaching the lung. Placement of an IVC filter is generally considered in patients who have
contraindications to anticoagulation, failed anticoagulation, or developed a complication due to
anticoagulation, or are non-adherent to treatment. Other indication is hemodynamic or respiratory
compromise significant enough to make another PE may be fatal.

(Option D) Thrombolytic therapy may be considered for patients with PE associated with significant
hemodynamic compromise. This patient does not have hemodynamic instabiliy justifying the use
of thrombolytics.

(Option E) Dabigatran is a direct thrombin inhibitor, approved in Australia for stroke prevention in
patients with non-valvular atrial fibrillation and at least one other risk factor for stroke. It is not
recommended to substitute warfarin or heparin therapy in PE.

References

• Australian Prescriber - Dabigatran – a new safe drug to replace an old poison?

• Therapeutic Guidelines – Cardiovascular


Last updated:
Time spent: QID:839
2023-2-12

1830 of 1943
A 52-year-old woman presents to your GP clinic with severe upper arm pain and progressive
shortness of breath for the past 6 months. She is none-smoker with a medical history significant
only for an operation on her right thigh approximately 2 years ago for a strangulated right femoral
hernia. On examination, weakness and muscle wasting of the right hand is noted, as well as an
increased respiratory rate of 20 breaths per minute. A chest X-ray is obtained that is shown in the
following photograph. Which one of the following would be the most appropriate next step in
management?

A. Lung biopsy.
B. Electromyographic studies of the right arm.
C. Electromyographic studies of the right thigh.
D. Bronchoscopy.
E. Sputum cytology exam.

Incorrect. Correct answer is A


45% answered correctly

Explanation:

Correct Answer Is A

The photograph shows a cap-like opacity covering the apex of the right lung. This finding in
addition to hand weakness and muscular atrophy and upper arm pain is suggestive of Pancoast
tumor as the most likely diagnosis.

The terms “Pancoast tumors”, “superior sulcus tumors”, or “superior pulmonary sulcus tumors” are
used to describe neoplasms located at the apical pleuropulmonary groove, adjacent to the
subclavian vessels.

Lesions in the superior sulcus can result in shoulder or arm pain in the distribution of the C8, T1,
and T2 dermatomes. T1 involvement causes sensory disturbances in the medial forearm and hand
and wasting of intrinsic hand muscles.

Horner’s syndrome can be another finding. It is caused by invasion to the stellate sympathetic
ganglion in the neck. The constellation of Horner’s syndrome and weakness and atrophy of hand
muscles is referred to as “Pancoast’s syndrome”.

1831 of 1943
The majority of patients with superior sulcus tumors present with one or more of the following:

Shoulder pain - The most common initial symptom of superior sulcus tumors is shoulder pain,
present in 44%-96% of patients. Pain is produced by invasion to the brachial
plexus and/or extension of the tumor into the parietal pleura, endothoracic fascia, first and second
ribs, or vertebral bodies. Pain can progress and radiate up to the head and neck, or down to the
medial aspect of the scapula, axilla, anterior chest, or ipsilateral arm in the distribution of the ulnar
nerve. Patients frequently receive treatment for presumed cervical osteoarthritis or shoulder
bursitis and there is often a delay in diagnosis. Pain ultimately becomes persistent and often so
severe that the patient may need to support the involved arm with the uninvolved hand for pain
relief.

Horner's syndrome - Horner's syndrome consists of ipsilateral ptosis with narrowing of the
palpebral fissure, miosis, enophthalmos, and anhidrosis. It is caused by the involvement of the
paravertebral sympathetic chain and the inferior cervical (stellate) ganglion. It is seen in 14%-50%

Ipsilateral flushing and decreased sweating of the face may occur before the development of the
full Horner's syndrome, presumably due to irritation of the sympathetic chain by the tumor prior to
frank invasion. Ipsilateral complex regional pain syndrome (reflex sympathetic dystrophy) and
facial pain similar to paroxysmal hemicrania rarely may occur. Contralateral facial sweating and
flushing, especially with exercise, due to an excessive response by the intact sympathetic pathway
have also been reported; this phenomenon is known as the Harlequin sign.

Neurologic complications involving the upper extremity - Extension of the tumor to the C8 and T1
nerve roots results in upper extremity neurologic findings in approximately 8%-22% of cases.
Involvement of these nerve roots may result in weakness and atrophy of the intrinsic hand
muscles, or pain and paresthesia of the fourth and fifth digits and the medial aspect of the arm and
forearm. Abnormal sensation and pain in the T2 territory (the axilla and medial aspect of the upper
arm) may also be an early finding. The triceps reflex may be lost.

Superior sulcus tumors invade the intervertebral foramina early in the course of disease in
approximately 5% of patients and may cause spinal cord compression and paraplegia.
Approximately 25% of patients ultimately develop spinal cord compromise.

Other findings - Supraclavicular lymph node enlargement and prominent weight loss are seen in
some patients. Superior sulcus tumors may produce a phrenic or recurrent laryngeal neuropathy or
superior vena cava syndrome in 5%-10% of patients.

Radiologic findings include a unilateral cervical cap of more than 5 mm, asymmetry of bilateral
cervical cap of more than 5 mm, an apical mass and local bone destruction.

Biopsy is used for definite diagnosis. The location of the superior sulcus tumors allows
percutaneous needle biopsy in most cases. The technique can be performed via a posterior or
cervical approach with the use of ultrasound or CT to localize the lesion. Video-assisted
thoracoscopy (VATS) or thoracotomy can be performed if less invasive methods are non-
diagnostic.

CT scanning of the chest is often used to provide additional information about a superior sulcus
tumor and its extension, including the presence of satellite pulmonary nodules, parenchymal
disease, and mediastinal lymphadenopathy. MRI is a better option for evaluation of brachial plexus,
subclavian vein and artery and chest wall involvement. CT scan and MRI are often performed prior
to biopsy and were the best choices if present among the given options. In their absence, lung
biopsy would be the most appropriate next step and the correct option.
1832 of 1943
(Options B and C) Electromyographic studies do not add any useful infrmation regarding diagnosis
or management plan.

(Options D and E) Due to the peripheral location of superior sulcus tumors, the diagnostic yields of
bronchoscopy and sputum cytology are less desirable. Fiberoptic bronchoscopy with cytology and
biopsies are diagnostic in only 30% to 40% of patients, but there is the benefit of detecting
unexpected endobronchial tumors that will significantly change the management plan.

References

• UpToDate - Superior Pulmonary Sulcus Tumors

• Medscape - Pancoast Syndrome Workup


Last updated:
Time spent: QID:858
2023-2-12

1833 of 1943
A 65-year-old man undergoes left hemicolectomy for colon cancer. Four days after the surgery, he
suddenly develops dyspnea. On examination, he has a blood pressure of 110/76 mmHg, heat rate
of 110 bpm and respiratory rate of 22 breaths per minute. He is afebrile. Which one of the following
is the diagnostic modality of choice to confirm the diagnosis?

A. Pulmonary angiography.
B. D-dimer.
C. Chest X-ray (CXR).
D. ECG.
E. MRI.

Correct
45% answered correctly

Explanation:

Correct Answer Is A

In patients with risk factors for pulmonary embolism (PE), any sign and symptoms pointing
towards the diagnosis should raise suspicion against PE, and prompt investigations follow.
Imaging by V/Q scan and CTPA is the cornerstone of diagnosis of PE; however, clinical
presentations of PE such as pleuretic chest pain, tachycardia, tachypnoea, hypoxia, etc. can be
seen in a good variety of other pulmonary conditions. For this reason an appropriate clinical
diagnostic rule (CDR) should be considered if possible in an attempt to decrease unnecessary
diagnostic tests. One of the most recent and validated tools is simplified Wells score (SWS), which
takes into account a number of items to determine whether the pretest probability of PE is likely
(PE likley more than 10% ) or unlikely ( PE likley less than 10%). Patients with SWS scores of 0 and
1 have a low pretest probability versus those with SWS scores of equal to or greater than 2.

An algorithmic approach to patients with suspected PE is as follows:

With recent major surgery in history and pulse rate of 110 bpm, this patient has SWS of 2; therefore,
PE should be considered ‘likely’ in this patient. When PE is likely the next step would be imaging
studies either by V/Q scan or computed tomography CT angiography (CTPA). Generally, CTPA is
preferred over V/Q scan because is often readily available, easy to use and more convenient to the
patient. Unlike, V/Q scan, underlying lung disease evident on CXR does not significantly affect the
specificity of the tests. Besides, it can detect smaller and more distal emboli in pulmonary arterial
tree, as well as other conditions that may mimic PE. Drawbacks to this method are more radiation

1834 of 1943
exposure and detection of clinically insignificant emboli which can trigger unnecessary
anticoagulation.

CTPA is always the method of choice except in the following patient groups:

Women of reproductive age (<55 years)


Those with poor GFR (<40) or those with progressively declining renal function
Those with known allergic reaction to contrast media

Conventional angiography, although practically supplanted by CTPA, remains the gold-standard


method and gives the higher diagnostic yield.

NOTE - Duplex ultrasound scan of lower limbs is very unlikely to be of usefulness in visualizing
clot(s) in lower limb as a source of PE and making a definite diagnosis if leg symptoms such as
swelling or pain elicited by deep palpation of leg veins.

(Option B) D-dimer might have a role in case the pre-test probability was low (=<10%). Howeve with
recent surgery in the history, D-dimer assay may not be very useful as D-dimer levels are elevated
after surgery, in pregnant patients and in those with cancer.

(Option C) A CXR should always be the initial investigation in all patients with suspected PE to
exclude other possible causes of the presentation, and to guide the clinician regarding selection of
the appropriate imaging modality. With few exceptions, a pristine chest X-ray is a ‘must’ for V/Q
scanning. With an abnormal chest X-ray or pre-existing lung pathology, V/Q scan results are likely
to be equivocal. The exceptions are where despite the presence of CXR abnormalities, contrast
media used in CTPA is remarkably harmful to the patient. As such is the use of CTPA in patients
with poor renal function evident by a GFR of <40 or rapidly declining renal function.

(Option D) ECG is used in initial assessment of patients with PE to exclude other serious causes of
chest pain, but it is not diagnostic for PE.

(Option E) Currently, MRI is not routinely used in patients with suspected PE. It may, however, be
performed for selected patients.

References

• RANZCR - Using clinical decision rules to guide appropriate imaging for suspected pulmonary
embolism
Last updated:
Time spent: QID:878
2023-2-12

1835 of 1943
A 45-year-old man presents with sudden onset severe chest pain and shortness of breath 2 weeks
after a hip replacement surgery. One year ago, he was diagnosed with chronic renal disease due to
IgA nephropathy. Which one of the following is the most appropriate investigation to consider for
this patient?

A. Computed tomography pulmonary angiography (CTPA).


B. V/Q scan.
C. ECG.
D. Chest X-ray (CXR).
E. D-dimer.

Incorrect. Correct answer is B


45% answered correctly

Explanation:

Correct Answer Is B

Hip or knee surgery is a strong risk factor for venous thromboembolism i.e. deep vein thrombosis
(DVT) and pulmonary embolism (PE). In patients with risk factors for PE, strong risk factors in
particular, any suggestive sign and symptoms should prompt detailed investigations. V/Q scan and
CTPA are the diagnostic methods to confirm the diagnosis of PE in whom, based on their pretest
probability test, PE is considered likely.

Generally, CTPA is preferred over V/Q scan because it is often readily available, easy to use and
more convenient to the patient. Unlike V/Q scan, underlying lung disease evident on CXR does not
significantly affect the specificity of the tests. Besides, it can detect smaller and more distal emboli
in pulmonary arterial tree, as well as other conditions that may mimic PE. Drawbacks to this
method are more radiation exposure and detection of clinically insignificant emboli which can
trigger unnecessary anticoagulation.

CTPA is always the diagnostic method of choice except in:

Women of reproductive age (<55 years)


Those with poor GFR (<40) or those with progressively declining renal function
Those with known allergic reaction to contrast media

This patient has poor GFR due to chronic kidney disease. In patients with poor GFR or those with
rapidly declining renal function, contrast media used in CTPA is relatively contraindicated and
should be avoided where possible. Contrast media can result in more kidney damage by (1) direct
toxic effect on kidney tubules and (2) triggering allergic reactions resulting in allergic interstitial
nephritis, both of which have detrimental effects on a previously damaged kidney. Having said
these, V/Q scan should be considered in this patient for definite diagnosis and the most
appropriate step.

ECG and CXR should be performed for all patients with suspected PE to exclude other mimickers,
but they are not diagnostic whatsoever.

1836 of 1943
D-dimer assay is used to exclude PE in patients in whom PE is unlikely based on pretest probability
assessment with one of validated clinical decision rules (CDR) such as Wells score, revised
Geneva score, etc.

NOTE - Patients with contraindication to contrast material, such as this patient, are excluded from
most CDRs.

References

• RANZACR - Using clinical decision rules to guide appropriate imaging for suspected pulmonary
embolism.
Last updated:
Time spent: QID:879
2023-2-12

1837 of 1943
An inpatient 65-year-old man develops acute onset-chest pain and dyspnea 7 days after
hemicolectomy due to colon cancer. On examination, his BP is 140/95 mmHg, heart rate 106 bpm
and respiratory rate 22 breaths per minute. Which one of the following is the most appropriate next
step in management of this patient?

A. Computed tomography pulmonary angiogram (CTPA).


B. Chest X-ray (CXR).
C. ECG.
D. D-dimer.
E. V/Q scan.

Incorrect. Correct answer is C


45% answered correctly

Explanation:

Correct Answer Is C

Sudden onset dyspnea with or without other signs and symptoms of PE such as pleuretic chest
pain, persistent tachycardia, hypotension, cough, etc. in a background of recent surgery and the
absence of a reasonable alternative explanation should raise the suspicion of PE. In general,
individual clinical signs and symptoms do not allow exclusion or confirmation of PE. However, the
history of recent major surgery increases the probability of this diagnosis.

While PE is a possible diagnosis, other conditions that can mimic PE should always be kept in mind
and excluded first. Although pleuretic chest pain goes against acute coronary syndrome (ACS) as a
likely diagnosis, in the presence of dyspnea, especially in the post-operative period, performing an
ECG is mandatory to exclude cardiac ischemia both as a cause of the chest pain, a concomitant
event, or as a result of hypoxia and cardiac ischemia triggered by the suspected PE.

By ECG, ACS as a potential cause to the presentation, can be excluded and possible concomitant
abnormalities be assessed for risk stratification.

The most common ECG abnormalities in the setting of PE are tachycardia and non-specific ST-T
wave abnormalities. The classic findings of right heart strain and acute cor pulmonale caused by
PE, as an underlying cause, are tall peaked P waves in lead II (P pulmonale), right axis deviation,
right bundle-branch block, an S1Q3T3 pattern, or arterial fibrillation. These findings, however, are
present in only about 20% of patients with PE. If they are present, they can only be suggestive (not
diagnostic), but their absence does not exclude PE whatsoever.

A chest X-ray comes next in line to not only exclude other caused to the presentation, but also to
decide upon the imaging modality of choice for definite diagnosis of PE.

(Option A) CTPA (or V/Q scan) was the answer if the most appropriate diagnostic test was asked.
Generally CTPA is preferred over V/Q scan as it is faster to perform and more available.
Furthermore, it can detect other pulmonary abnormalities other than PE, or where an abnormal
chest X-ray precludes use of V/Q scan due to potential non-diagnostic results.

1838 of 1943
(Option B) In general, ECG and chest-x ray are always among initial steps to exclude PE not to
diagnose it. After an ECG, chest X-ray is the most appropriate next step in management.

(Option D) D-dimer assay is a useful screening tool for patients in whom PE is considered ‘unlikely’
based on clinical decision rules (CDRs) such as Wells score, simplified Wells score, etc.

(Option E) V/Q scan is the most appropriate diagnostic modality in pregnant patients, in those with
allergy to contrast media, in patients with decreased GFR due to renal disease and in women of
reproductive age due to increased radiation exposure and potential risk of breast cancer
associated with CTPA.

References

• Medscape - Pulmonary Embolism (PE)

• Australian Medical Student Journal - Venous thromboembolism: a review for medical students
and junior doctors

• RANZACR - Using clinical decision rules to guide appropriate imaging for suspected pulmonary
embolism.

• MJA - Venous thromboembolism: diagnosis and management of pulmonary embolism


Last updated:
Time spent: QID:880
2023-2-12

1839 of 1943
A 42-year-old woman sustained a motor vehicle accident (MVA) and had her femoral neck
fractured which is surgically fixed. Five days after the surgery, she develops sudden-onset chest
pain and dyspnea. She is on oral contraceptive pills. On examination, she is afebrile, has a heart
rate of 100 bpm, BP of 110/70 mmHg, and respiratory rate of 24 breaths per minute. Lungs are
clear to auscultation. Chest X-ray and ECG are normal. Which one of the following is the most
appropriate next step in management?

A. Duplex doppler ultrasound scan of lower limbs.


B. D-dimer assay.
C. Computed tomography pulmonary angiography (CTPA).
D. Arterial blood gas (ABG) analysis.
E. V/Q scan.

Incorrect. Correct answer is E


45% answered correctly

Explanation:

Correct Answer Is E

Orthopedic surgery is a strong risk factor for development of venous thromboembolism in the post-
op period. For every patient with major surgery within the past 4 weeks, or immobilization longer
than 3 days, who have signs and symptoms such as dyspnea, chest pain (especially pleuretic or
sudden onset), cough, hemoptysis, or hypoxia, pulmonary embolism (PE) should always be
suspected and promptly investigated.

CTPA and V/Q scan are confirmatory tests for PE if, through pretest probability assessment, PE is a
likely diagnosis. There are rules referred to as Clinical Decision Rules (CDR) for this purpose. CDRs
take into account different items to calculate the likelihood of PE as a diagnosis to
prevent unnecessary diagnostic testing or treatment.

One of the most recently validated and practical of these CDRs is Simplified Wells Score (SWS).
SWS gives seven items one score each (if present). Patients with scores 2 or higher are likely to
have PE and should undergo imaging studies either by CTPA or V/Q scan, while patients with
scores 0 or 1 are unlikely to have PE. In this group, a D-dimer assay is used to safely exclude PE.

D-dimer is a fibrin degeneration product. High levels of D-dimer represent the presence of
thrombus and triggers further confirmatory diagnostic tests for definite diagnosis. Negative results
have high negative predictive value and are sensitive enough to exclude PE in patients with low to
moderate pretest probability of PE. D-dimer levels are, however, elevated in post-surgery (first 2
weeks) period, pregnancy, inflammation, infection, and cancer, making a negative result less likely
and the test less desirable where these conditions are present.

The following diagram shows the approach to patients with suspected PE bases on SWS:

1840 of 1943
This patient has had a major surgery within the past four weeks (1 score). She also has a clear
chest X-ray and a normal ECG, excluding other lung pathologies and cardiac problems as a cause
to the presentation; hence, no other condition seems to be more likely (1 score). She has a SWS 2
and is likely to have PE.

When PE is likely, imaging tests follow. Generally, CTPA is preferred over V/Q scan because it is
often readily available, easy to use and more convenient to the patient. Unlike V/Q scan, underlying
lung disease evident on CXR does not significantly affect the specificity of the test. Moreover, it can
detect smaller and more distal emboli in pulmonary arterial tree, as well as other conditions that
may mimic PE. Drawbacks to this method are more radiation exposure and detection of clinically
insignificant emboli which can trigger unnecessary anticoagulation.

CTPA is always the method of choice except in the following groups:

Women of reproductive age (<55 years)


Those with poor GFR (<40) or those with progressively declining renal function
Those with known allergic reaction to contrast media

Since this patient is a female younger than 55 years, V/Q is scan is the preferred method, as
radiation exposure is greater with CTPA and associated with higher rates of breast cancer in future,
at least theoretically.

When leg symptoms are present (a minimum of leg swelling or tenderness of deep veins on
palpation), a Doppler ultrasonography of the lower limbs can be used as an alternative. The usual
source of PE is thrombosis in the deep veins (DVT) of the lower limbs or pelvis. If a clot is present

1841 of 1943
in lower limb or pelvis on Doppler ultrasound, no further imaging is required, and treatment should
be started. In the absence of leg symptoms, however, Doppler ultrasound is unlikely to be useful.

NOTE – Although ABG is an essential part of PE workup alongside the ECG and CXR, it is not the
correct answer here because even with a normal ABG or an ABG inconsistent with expected
result in PE (hypoxia and hypocarbia at the same time) is neither sensitive nor specific. ABG is
never used to diagnose PE with certainty.

References

• RANZCR - Using clinical decision rules to guide appropriate imaging for suspected pulmonary
embolism.

• RANZCR - Suspected Pulmonary Embolism


Last updated:
Time spent: QID:881
2023-2-12

1842 of 1943
A 28-year-old man sustains multiple limb fractures in a motor vehicle accident and undergoes
surgical fixation of the fractures. On the third day after the surgery, he develops sudden right-sided
pleuretic chest pain and shortness of breath. On examination, the findings are otherwise
inconclusive. Which one of the following would be the most likely diagnosis?

A. Pulmonary embolism.
B. Fat embolism.
C. Atelectasis.
D. Pneumonia.
E. Air embolism.

Correct
45% answered correctly

Explanation:

Correct Answer Is A

The history of orthopedic surgery and clinical findings of pleuritic chest pain and shortness of
breath make pulmonary embolism (PE) the most likely diagnosis. PE may have been caused by clot
(thromboembolism) or fat from a fractured bone where fat can escape from bone marrow into
circulation.

NOTE - differential diagnoses of pleuritic chest pain include the following 5 ‘P’s:

1. Pulmonary embolism
2. Pericarditis
3. Pneumonia
4. Pneumothorax
5. Pneumomediastinum.

(Option B) Fat embolism syndrome (FES) is systemic involvement due to occlusion of vessels by
free-circulating fat drops and droplets. Brain, eyes, kidneys, skin, and lungs are the most common
affected organs. Lung involvement is termed as pulmonary embolism, not fat embolism because
the latter is, by definition, a systemic condition not just confined to the lungs.

The history of a patient with fat embolism syndrome (FES) may include the following:

Trauma to a long bone or the pelvis, including trauma sustained in orthopedic procedures
Parenteral lipid infusion
Recent corticosteroid administration

Major physical criteria of FES are respiratory symptoms, cerebral signs without other etiologies and
petechial rash. Minor criteria are pulse rate higher than 110 bpm, fever higher than 38.5° C, retinal
changes of fat globules or petechiae, renal dysfunction, jaundice, acute drop in hemoglobin level,
platelet count, or both and elevated erythrocyte sedimentation rate

A formal diagnosis of FES requires the presence of at least one major criterion and four minor
criteria, plus fat microglobulinemia. With only shortness of breath and pleuretic chest pain and
otherwise normal findings on exam this patient is unlikely to have FES.
1843 of 1943
(Options C and D) Atelectasis after the first day of operation, and pneumonia without preceding
atelectasis are less likely.

(Option E) Air embolism is a rapidly lethal condition with sudden death being the most common
presentation. It does not cause pleuritic chest pain. Often the patient will not have enough time to
even become short of breath.

References

• Emedicine - Fat Embolism

• Therapeutic Guideline – Cardiovascular


Last updated:
Time spent: QID:323
2023-2-12

1844 of 1943
A 20-year-old man is admitted after a motor vehicle crash, in which he sustained multiple lower
limb fractures. He underwent surgical fixations of his fractures. Five days after the surgery, he
becomes pale, clammy and short of breath. His BP is 110/70 mmHg and pulse rate 100 bpm.
Which one of the following investigations is most likely to diagnose his current problem?

A. Duplex Doppler ultrasound of the calf veins.


B. Computed tomography pulmonary angiography(CTPA).
C. D-dimer assay.
D. Positron emission tomogram (PET).
E. Chest X-ray (CXR).

Incorrect. Correct answer is B


45% answered correctly

Explanation:

Correct Answer Is B

Orthopedic surgery and immobilization (bed rest) more than 3 days are strong risk factors for
development of venous thromboembolism during the post-op period. For every patient with risk
factors for PE, who have signs and symptoms such as dyspnea, chest pain (especially pleuretic or
sudden onset), cough, hemoptysis, or hypoxia, pulmonary embolism should always be suspected
and promptly investigated.

CTPA and V/Q scan are confirmatory tests for definite diagnosis of PE. Generally, CTPA is preferred
over V/Q scan because it is often readily available, easy to use and more convenient to the patient.
CTPA also comes first when the patient appears to be hemodynamically unstable. Although this
patient has normal blood pressure and heart rate, his pallor and clamminess are ominous signs
heralding shock. He requires the fastest method to establish the diagnosis prior to starting
treatment.

CXR (option E), ECG, and ABG are all performed on patients with suspected PE but none of them
are diagnostic for it.

D-dimer assay (option C) is not a reliable tool in post-op patient as it is usually high after surgery
and has no positive predictive value for PE in such patients. Other conditions where D-dimer assay
is unreliable are during pregnancy and shortly in postpartum period and in patients with
malignancies.

Doppler ultrasound (option A) could be diagnostic in patients with suspected PE and simultaneous
leg symptoms suggestive of deep vein thrombosis (DVT), sparing the patient the need to undergo
CTPA or VQ scanning. However, in the absence of such symptoms, doppler ultrasound is very
unlikely to be diagnostic.

Positron-emission tomogram (PET) (option D) is of little diagnostic value in suspected PE and nor
recommended.

NOTE - CTPA is always the method of choice except in the following groups:

Women of reproductive age (<55 years)


1845 of 1943
Those with poor GFR (<40) or those with progressively declining renal function
Those with known allergic reaction to contrast media

References

• Medscape - Pulmonary Embolism (PE)

• RANZCR - Suspected Pulmonary Embolism

• RANZACR - Using clinical decision rules to guide appropriate imaging for suspected pulmonary
embolism.
Last updated:
Time spent: QID:882
2023-2-12

1846 of 1943
A 62-year-old man presents to the Emergency Department with sudden-onset chest pain and
shortness of breath. He has chronic kidney disease and a documented reduced GFR of 25 ml/min.
On examination, his blood pressure is 110/75mHg and pulse 112 bpm. An ECG is performed that is
is normal. A chest X-ray shows a wedge-shaped opacity in the periphery of the right lung. Which
one of the following should be considered the best diagnostic tool in this patient?

A. Duplex Doppler ultrasound of lower limbs.


B. D-dimer assay.
C. Computed tomography pulmonary angiography (CTPA).
D. Arterial blood gas (ABG) analysis.
E. V/Q scan.

Incorrect. Correct answer is E


45% answered correctly

Explanation:

Correct Answer Is E

Sudden onset of shortness of breath can have a variety of etiologies. Spontaneous pneumothorax
and pulmonary embolism (PE) are typical examples. Pneumonia and other respiratory infections
are also possible explanations, as is cardiac ischemia. Pericarditis and pneumomediastinum are
also possible.

In approach to such patients, an ECG and chest radiograph are the initial steps to narrow down the
possibilities. An arterial blood gas (ABG) analysis is indicated as well in the presence of hypoxia.

There is no mention of fever in the question; therefore, infectious causes are weak possibilities.
ECG is normal and excludes cardiac ischemia as a cause.

A wedge-shaped opacity on CXR, however, is suggestive of pulmonary infarction, making


pulmonary embolism likely.

With this amount of information, PE comes first on the differentials list. In other words, no other
etiology could be as likely as PE. At least PE can be as possible as other causes of this
presentation.

Definite diagnosis of PE relies on imaging studies such as CTPA or V/Q scan; hiowever, not
everyone with signs and symptoms of PE should undergo diagnostic imaging just based on the
presentation. To avoid unnecessary imaging studies on the one hand and missing patients with PE
as a potential fatal condition on the other, clinical decision rules (CDRs) are devised to help
clinicians identify patients who are likely or unlikely to have PE and to avoid performing imaging in
those who have low or intermediate risk.

There are a number of validated CDRs for clinical use with Wells and revised Geneva being most
commonly applied in clinical situations; however, with either of the following conditions present,
patients with suspected PE should be excluded from assessment with Wells score, simplified Wells
score, and revised Geneva score:

Ongoing anticoagulant treatment


1847 of 1943
Contraindication to CT (known allergy to iodinated contrast agents or risk for allergic reaction,
creatinine clearance (GFR) <0.5ml/s or <30ml/minute)
Pregnancy
Suspected massive PE with shock
Estimated life expectancy less than 3 months

This patient, who has chronic kidney disease and poor GFR, is not an appropriate candidate for
pretest probability assessment using Wells score, either classic or simplified, or revised Geneva
score, and should be considered for investigation with imaging studies. Moreover, with PE being
more likely than other possible conditions (1 score) and tachycardia (1 score), he has a simplified
Wells score of 2 and should have imaging tests as the next best step.

Generally, CTPA is preferred over V/Q scan because it is often readily available, easy to use and
more convenient to the patient. Unlike V/Q scan, underlying lung disease evident on CXR, does not
significantly affect the specificity of the test. Moreover, it can detect smaller and more distal
emboli in pulmonary arterial tree, as well as other conditions that may mimic PE. Drawbacks to this
method are more radiation exposure and detection of clinically insignificant emboli which can
trigger unnecessary anticoagulation.

CTPA is always the method of choice except in the following patient groups:

Women of reproductive age (<55 years)


Those with poor GFR (<40) or those with progressively declining renal function
Those with known allergic reaction to contrast media

This patient has poor GFR due to chronic kidney disease. In patients with poor GFR or those with
rapidly declining renal function, contrast media used in CTPA is relatively contraindicated and
should be avoided where possible. Contrast media can result in more kidney damage by (1) direct
toxic effect on kidney tubules and (2) triggering allergic reactions resulting in allergic interstitial
nephritis. For such patients, V/Q scan should be considered for definite diagnosis and as the most
appropriate step. CXR abnormalities (the wedged-shape opacity here) makes V/Q scan less ideal
because the chances are that the result is misinterpreted, yet V/Q scan is the only alternative given
the risk of contrast media to this patient.

References

• RANZCR - Suspected Pulmonary Embolism

• RANZACR - Using clinical decision rules to guide appropriate imaging for suspected pulmonary
embolism.

• Medscape - Pulmonary Embolism (PE)


Last updated:
Time spent: QID:883
2023-2-12

1848 of 1943
A 26-year-old woman presents to the emergency department with shortness of breath and
pleuretic chest pain. Based on the history and clinical findings, she is suspected to have pulmonary
embolism. She has a family history of thrombophilia. Which one of the following is the
investigation of choice for her?

A. Duplex Doppler ultrasound scan of lower limbs.


B. Chest X-ray (CXR).
C. V/Q scan.
D. Computed tomogram pulmonary angiogram (CTPA).
E. D-dimer assay.

Incorrect. Correct answer is C


45% answered correctly

Explanation:

Correct Answer Is C

Computed tomography pulmonary angiography (CTPA) and V/Q scan are currently imaging
modalities used for definite diagnosis of pulmonary embolism (PE).

Generally, CTPA is preferred over V/Q scan because it is often readily available, easy to use, and
more convenient to the patient. Unlike V/Q scan, underlying lung disease evident on chest X-ray
does not affect the specificity of the test. Besides, it can detect smaller and more distal emboli in
pulmonary arterial tree, as well as other conditions that may mimic PE. Drawbacks to this method
are more radiation exposure and detection of clinically insignificant emboli which can trigger
unnecessary anticoagulation.

CTPA is always the method of choice except in the following patient group:

Women of reproductive age (<55 years)


Those with poor GFR (<40) or those with progressively declining renal function
Those with known allergic reaction to contrast media
Pregnant women

This woman is young (<55 years) and V/Q scan is the preferred diagnostic option for her. It is
recommended that V/Q scan be used in women of reproductive age (<55 years) to avoid the
increased risk of breast cancer associated with higher radiation exposure in CTPA.

(Option A) Doppler ultrasound is used for detection of lower limb clot(s) as the source of PE. Lower
limb clots indirectly point towards PE as the most likely cause to the presentation. In the absence
of leg symptoms, however, this method does not yield high diagnostic values.

(Option B) CXR is performed in all patients with suspected PE to exclude other possible causes
and guide as to selection of the imaging modality of choice. V/Q scan is less accurate when there
are chest X-ray abnormalities. CXR is not a diagnostic test for PE ecause it is neither sensitive, nor
specific.

(Option D) CTPA is better avoided in women younger than 55 years due to increased risk of breast
cancer associated with higher radioation exposure compared to V/Q scan.
1849 of 1943
(Option E) D-dimer assay is indicated as a screening tool in patients with low or intermediate
pretest probability of PE estimated with current clinical decision rules (CDR). It is not diagnostic.

References

• RANZCR - Suspected Pulmonary Embolism

• RANZACR - Using clinical decision rules to guide appropriate imaging for suspected pulmonary
embolism.

• Medscape - Pulmonary Embolism (PE)


Last updated:
Time spent: QID:884
2023-2-12

1850 of 1943
A 40-year-old woman presents to the Emergency Department with sudden-onset chest pain,
dyspnea and a fever of 39°C. She has arrived from Canada to Sydney 2 days ago by air. Which one
of the following would be the most appropriate next step in management?

A. Chest X-ray (CXR).


B. ABG.
C. Computed tomography pulmonary angiography (CTPA).
D. V/Q scan.
E. D-dimer assay.

Correct
45% answered correctly

Explanation:

Correct Answer Is A

Travel, especially by air, longer than 3 hours is a risk factor for development of thromboembolism
either in form of deep vein thrombosis (DVT) or pulmonary embolism (PE). Sudden onset dyspnea
is the most common presenting symptom in PE. Pleuretic chest pain, cough, hemoptysis, hypoxia,
and even collapse and hemodynamic instability can be present as well. Fever is less common, yet a
possible presenting sign. However, in the presence of fever, more common causes should be
excluded before PE is considered as a likely diagnosis. A CXR is always required as one of the very
initial measure to investigate other likely causes other than PE. This is even more important in the
presence of fever that makes an infectious etiology more likely than or at least as likely as, PE.

(Option B) ABG is performed in initial assessment. Typical finding for PE is concomitant presence
of hypoxia and hypocapnia; however, this pattern is seen in only a minority of patients. Hypoxia is
frequently present but may be due to a wide variety of other cardiopulmonary complications. To put
it simple, ABG does not add diagnostic value to this case.

(Options C and D) CTPA and V/Q scan are used for definite diagnosis of PE if initial assessment
favors it but not as the next best step here.

(Option E) D-dimer assay is used when PE is considered as a likely diagnosis in patients with low to
moderated pretest probability for PE, after other potential causes are excluded.

References

• RANZCR - Suspected Pulmonary Embolism

• RANZACR - Using clinical decision rules to guide appropriate imaging for suspected pulmonary
embolism.
Last updated:
Time spent: QID:885
2023-2-12

1851 of 1943
A 6-year-old boy is brought to your attention because of frequent episodes of wheezing and cough
at night and during exercise. You prescribe salbutamol on as-needed basis and regular low-dose
inhaled fluticasone. Which one of the following medications is most effective in preventing further
episodes?

A. Long-acting beta agonists (LABA).


B. Short-acting beta agonists (SABA).
C. Sodium cromoglycate (SCG).
D. Fluticasone.
E. Montelukast.

Incorrect. Correct answer is D


45% answered correctly

Explanation:

Correct Answer Is D

This child has persistent asthma evident by frequent episodes of night symptoms more than 2
nights. The scenario does not provide adequate information as to whether this is moderate or
severe persistent asthma.

Asthma severity classification based on frequency of symptoms is as follows:

Intermittent

Symptoms occur more than one week apart AND


Night symptoms occur less than 2 time per month (1 night per month only or not at all)

Mild persistent

Symptoms occur more than once a week but not on a daily basis
Night symptoms occur more than 2 times per month

Moderate persistent

Symptoms occur on a daily basis


Night symptoms more than once in a week

Severe persistent

Symptoms occur on a daily basis


Frequent night symptoms

For persistent asthma, the treatment of choice is regular use of an inhaled corticosteroid (ICS) and
a short acting beta agonist (SABA) on a as-needed basis. Treatment should be started at the
lowest dose possible and titrated according to the response.

ICS (fluticasone, beclomethasone, budesonide, ciclesonide) are the most effective preventive
therapy in children with asthma. In Australia, they are referred to as ‘preventers’ and are first-line
1852 of 1943
maintenance treatment.

This child has already been started on inhaled fluticasone. None of the options are superior to
fluticasone as a preventive measure.

(Option A) LABAs are not routinely used as preventers.

(Option B) SABA is the treatment of choice for intermittent asthma, or for prevention of wheeze
and cough that is only induced by exercise.

(Option C) Cromones (cromoglycate, nedocromil) can be used as an alternative to low-dose ICS or


montelukast in children with frequent intermittent or persistent asthma, but they are not as
effective of ICS and require a more complex regimen. They should be used more frequently (e.g. 4
times a day) and meticulous daily care needed care to prevent clogging of the inhaler device.
Nedocromil has an unusual unpleasant taste and is not tolerated by many children.

(Option E) Although not as effective as ICS, a 2-week trial of montelukast maybe considered first in
an attempt to spare the child of ICSs.

References

• Australian Asthma Handbook - Guide to preventers: inhaled corticosteroids

• Therapeutic Guidelines – Respiratory; available from http://tg.org.au


Last updated:
Time spent: QID:956
2023-2-12

1853 of 1943
A 12 year-old boy is brought to the emergency department by his mother with complaints of cough
and wheezing. He has had previous episodes in the past, and has not any specific treatment so far.
On examination, he has a respiratory rate of 40 breaths per minute and a temperature of 37.5°C. On
chest auscultation, widespread wheeze is noted bilaterally. The rest of the exam is inconclusive.
Which one of the following is most likely to make a diagnosis now and in the emergency setting?

A. CT scan of the chest.


B. Chest X-ray (CXR).
C. Spirometry.
D. Improvement with salbutamol.
E. Measurement of peak expiratory flow (PEF).

Incorrect. Correct answer is D


45% answered correctly

Explanation:

Correct Answer Is D

The scenario is typical for an exacerbation of a previously-undiagnosed asthma. Asthma is the


most common cause cough and wheezing in children.

By definition, asthma is reversible inflammatory response of the bronchial tree triggered by factors
such as allergens, cold, exercise, stress, noxious inhalants, dust mite, etc. Reversibility and
symptom relief after administration of bronchodilators (e.g. inhaled salbutamol) is the key issue in
diagnosis of asthma.

Imaging studies such as plain chest X-ray (option B) or CT scan (option A) are not routinely used
for diagnosis of asthma, unless bases on clinical grounds, an alternative diagnosis other than
asthma is considered.

Spirometry (option C) can help in diagnosis by showing a reversible obstructive pattern; however, it
is of little value during an exacerbation of asthma when patient cannot efficiently follow the
instructions such as taking a very deep breath, or blowing with the maximum force they can. In
patients with intermittent asthma, pulmonary function test using spirometry may be inconclusive if
carried out in between attacks. If so, bronchoconstriction is induced by metacholine (metacholine
challenge test). A bronchodilator is then administered to see whether this is reversible. Reversibility
suggests asthma with high certainty.

PEF (option E) is a valuable tool to use for monitoring the response to treatment e.g. inhaled
salbutamol. Increasing values read by PEF indicates appropriate response to treatment. The
question though asks about the best method for diagnosis now in the acute setting the patient has
presented with not how to assess the response to treatment in which case PEF would be the
correct answer.

References

• Australian Asthma Handbook

1854 of 1943
Last updated:
Time spent: QID:1015
2023-2-12

1855 of 1943
Which one of the following is the most common finding in a patient with pleural effusion?

A. Cough.
B. Chest tightness.
C. Dyspnea.
D. Weight loss.
E. Pleuretic chest pain.

Incorrect. Correct answer is C


45% answered correctly

Explanation:

Correct Answer Is C

Dyspnea is the most common and prominent clinical finding in pleural effusion.

(Option A) Cough may be present in patients with pleural effusion and is often mild and non-
productive. More severe cough or bloody or purulent sputum associated with cough suggest an
underlying pneumonia or endobronchial lesions.

(Option B) Chest tightness is not a feature caused by pleural effusion.

(Option D) Pleural effusion per se does not cause weight loss. Weight loss, however, can be caused
by the underlying condition such as TB, lymphoma, etc.

(Option E) pleuretic chest pain is present in some patients and suggests an exudative etiology,
such as pleural infection, pulmonary infarction, or mesothelioma.

References

• http://emedicine.medscape.com/article/299959-clini

• http://www.nevdgp.org.au/info/lungf/pleural-effusi
Last updated:
Time spent: QID:1149
2023-2-12

1856 of 1943
A 35-year-old woman presents to the Emergency Department with sudden onset shortness of
breath and right-sided pleuretic chest pain. On examination, she has a blood pressure of 130/89
mmHg, pulse rate of 102 bpm, and respiratory rate of 25 breaths per minute. Neck and forehead
veins are flat and the trachea is slightly deviated to the right side. Her right-lung field has reduced
breath sounds and is hyper-resonance to percussion. A chest X-ray shows a 25% right-sided
pneumothorax. There are no other abnormalities on chest X-ray. Which one of the following is the
most appropriate next step in management?

A. Chest tube drainage.


B. Needle aspiration.
C. Intubation and mechanical ventilation.
D. Observation for 24 hours.
E. Thoracotomy.

Incorrect. Correct answer is B


45% answered correctly

Explanation:

Correct Answer Is B

This woman has developed pneumothorax in the absence of a history of trauma or an invasive
procedure. Such pneumothoraces are termed 'spontaneous pneumothorax'. She has no chest X-ray
abnormalities apart from those related to the pneumothorax. She also has no history of lung
disease, making ‘primary spontaneous pneumothorax (PSP)’ the most likely diagnosis. The
management option of choice for patients with symptomatic PSPs or asymptomatic PSPs that are
large (>15%) is needle aspiration.

(Option A) Chest tubs insertion is the management of choice in patients with traumatic
pneumothorax, patients with a large (>15%) secondary spontaneous pneumothorax, or when a
patient with pneumothorax is about to undergo mechanical ventilation or being transferred by air
(see topic review).

(Option C) Mechanical ventilation or positive pressure ventilation of any kind makes pneumothorax
worse.

(Option D) Observation is an acceptable management option for patients with primary


spontaneous pneumothoraces that are asymptomatic and small (≤15%).

(Option E) Thoracotomy has no role in management of pneumothorax but may be indicated for the
underlying airway disease as the cause of the pneumothorax.

TOPIC REVIEW

Pneumothorax is defined as presence of air in the pleural space. Based on the etiology,
pneumothorax can be defined as traumatic or spontaneous.

1857 of 1943
Traumatic pneumothorax is caused by blunt or penetrating chest traumas or invasive procedures
(iatrogenic) such as central vein catheterization, pleural biopsy, etc.

Spontaneous pneumothorax is the term used when the condition occurs in the absence of an
apparent trauma or procedure. Spontaneous pneumothorax is classified as primary or secondary
based on absence or presence of an underlying lung pathology or chest X-ray abnormalities.

Primary spontaneous pneumothorax (PSP) – spontaneous pneumothorax is primary if all of


following criteria are met:

There is no respiratory finding on exam except those related to the pneumothorax


There is no history of lung disease
There is no chest X-ray finding other than those related to the pneumothorax
The patient is young (≤50 years)
There is no significant history of smoking

Secondary spontaneous pneumothorax (SSP) – spontaneous pneumothorax is considered


secondary if either of the following is present:

Respiratory findings other than those related to pneumothorax


History of lung disease such as COPD (the most common cause of SSP), asthma, cystic
fibrosis sarcoidosis, TB, lung cancer, chronic bronchitis, bronchiectasis, extra- or
intrathoracic restrictive lung disease, etc
Any chest X-ray abnormality other than those related to pneumothorax
Age >50 years
Significant history of smoking

NOTE – the typical patient with PSP is a thin tall man of 20- to 40-years old. PSP is rarely seen
after 40 year and almost never after the age of 50; therefore, spontaneous pneumothorax in
those older than 50 is considered secondary and treated accordingly until proven otherwise.

Size of pneumothorax - Accurate estimation of the size of a pneumothorax is difficult. There


different methods for estimation:

Average interpleural distance (AID) method - approximates the size of a pneumothorax from a
PA CXR in standing position by taking the sum of the distances (measured in millimeters)
between the ribs and the visceral pleura at the apical level (A), midthoracic (B), and basal
level (C), then dividing the sum by 3.

Light Index – An upright PA chest X-ray is obtained. The width of the lung and the hemithorax
are measured (in centimeters). The percentage of pneumothorax is calculated from the
following formula:

Pneumothorax percentage = (1 – (width of the lung)3/(width of the hemithorax)3)x100

For example if the width of the lung and the hemithorax are 5 cm and 10 cm respectively, the
percentage of pneumothorax will be:

1 – (5)3 / (10)3 = 1 – 0.125 = 0.875 or 87.5 % (87.5% of the affected hemithorax is occupied with
the pneumothorax)

1858 of 1943
These methods are difficult to apply in practice and often underestimate the size of the
pneumothorax. As a result, some clinicians tend to describe a pneumothorax as large or small,
rather than using the percentage.

Chest wall – pleural line distance at the hilum level - British Thoracic Society guidelines define
a pneumothorax as small if the distance from chest wall to the visceral pleural line (at the
level of the hilum) is less than 2 cm or large if the distance from the chest wall to the visceral
pleural line is 2 cm or greater. Some clinicians prefer 3 cm laterally and 4 cm apically as the
threshold to distinguish small and large pneumothoraces.

Symptoms:

Small pneumothoraces are often asymptomatic.


Larger pneumothoraces can present with:

Pleuretic chest pain (pain may be referred to shoulder tip)


Shortness of breath

Clinical findings:

Clinical findings associated with pneumothoraces include:

Decreased breath sounds over the affected area due to decreased air entry
Hyperresonance over the affected area
Decreased tactile fremitus
Tracheal deviation to the affected side

Management:

Primary spontaneous hemothorax (PSP) – management of PSP depends on the presence of the
symptoms and/or the size of the pneumothorax:

Consider discharging the patient and review in 24 hours, and every 1-2 weeks until the
pneumothorax is resolved if:

The patient is asymptomatic (or minimal symptoms) AND


The size of the pneumothorax is less than 15% of the affected lung, or the distance
between the chest wall and the visceral plural line is ≤2cm

Consider needle aspiration and REVIEW if:

The patient is symptomatic (pleuretic chest pain or dyspnoea), OR


The size of the pneumothorax is ≥15%of the affected lung, or the distance between the
chest wall and the visceral plural line >2cm

After needle aspiration, admission and catheter drainage (chest tube) would be indicated as
the next best step in management if:

The aspirated air is ≥3 litres


The distance between the chest wall and visceral pleural line is still >2cm on a chest x-
ray taken 4 hours after needle aspiration
There is significant shortness of breath

1859 of 1943
NOTE – the rationale behind less invasive initial management of PSP is based upon the fact that
the patients with PSP are young and otherwise healthy, and there is no underlying pathology to
perpetuate the pneumothorax or prevent it from spontaneous healing

Seconadary spontaneous pneumothorax (SSP) – in SSP, the underlying lung problem prevents the
pneumothorax from spontaneous healing. All patients with SPS need to be admitted to the hospital
and undergo:

Needle aspiration and REVIEW if there are no significant symptoms AND the size of the
pneumothorax is less than 15% of the affected lung (or the distance between the chest wall
and the visceral plural line is ≤2cm)

Catheter drainage (chest tube) if the patient is symptomatic (pleuretic chest pain or
dyspnoea) OR the size of the pneumothorax is ≥15%of the affected lung (or the distance
between the chest wall and the visceral plural line >2cm)

​Traumatic pneumothorax

Unless the patient is asymptomatic and the size of pneumothorax is <15% (or the distance
between the chest wall and the visceral plural line is ≤2cm), he patient should undergo catheter
drainage (chest tubes). Asymptomatic patients whose pneumothorax is 15% (or the distance
between the chest wall and the visceral plural line is ≤2cm) can be closely observed for
spontaneous resolution of the pneumothorax.

NOTE – Pneumothorax of any kind and size should be treated with chest catheter (chest tube)
insertions if:

The patient is undergoing general anesthesia for any reason


The patient is planned to be intubated and mechanically ventilated
The patient is planned to be transported by air (air transport)

References

• UpToDate - Treatment of secondary spontaneous pneumothorax in adults

• Merck Manuals - Pneumothorax

• AMC Handbook of Multiple Choice Questions – page 179


Last updated:
Time spent: QID:1187
2023-2-12

1860 of 1943
Which one of the following options is the most appropriate test to differentiate pulmonary
tuberculosis (TB) from sarcoidosis?

A. Chest X-ray (CXR).


B. Erythrocyte sedimentation rate (ESR).
C. C-reactive protein (CRP).
D. Mantoux test.
E. Sputum exam.

Incorrect. Correct answer is D


45% answered correctly

Explanation:

Correct Answer Is D

Sarcoidosis and pulmonary TB have many similarities in presentation. They both can be associated
with vague systemic symptoms of fever, night sweats, and weight loss. They both cause cough and
might appear similar on CXR and other imaging modalities. There might be also an overlap
syndrome in which both conditions coexist.

Both TB and sarcoidosis are granulomatous diseases; however, tuberculosis has a caseating
granuloma as opposed to sarcoidosis, which presents with non-caseating epithelioid cell
granuloma.

Misdiagnosis of sarcoidosis as TB results in futile anti-TB treatment while the sarcoidosis remains
unattended. Conversely, a patient with misdiagnosed TB as sarcoidosis may be treated with
corticosteroids that worsen the TB due to induction of immunodeficiency. Therefore, it is
imperative to distinguish between these two. A tuberculin skin test (TST), also known as Mantoux
test, is the most appropriate initial tool to apply for differentiation between these two. A negative
TST suggests sarcoidosis versus TB. A positive test, however, should prompt extensive
investigations for TB. A patient with a positive TST may be infected with TB, or the result might just
be false positive e.g. due to previous BCG vaccination.

(Option A) CXR can be quite similar in both TB and sarcoidosis; hence, not a good means of
differentiation.

(Options B and C) ESR and CRP are non-specific inflammatory markers that could be elevated in
both sarcoidosis and TB and are not of discriminatory value.

(Option E) Sputum exam can be inconclusive in patients with TB, and smear-negative TB is
frequently encountered. Therefore, negative sputum satin for AFB or culture cannot exclude TB as
a diagnosis.

NOTE – histopathology study on specimens from lung biopsy is the most accurate and gold
standard method for distinguishing sarcoidosis from TB.

References

1861 of 1943
• UpToDate – Clinical manifestations and diagnosis of pulmonary sarcoidosis

• PubMed - Tuberculin skin test among pulmonary sarcoidosis patients with and without
tuberculosis
Last updated:
Time spent: QID:1415
2023-2-12

1862 of 1943
A 30-year-old woman, recently migrated from China to Australia, is undergoing a health
assessment. Before coming to Australia, she had been working in a cotton mill for 18 months and
describes that upon start of the workweek she developed chest tightness, shortness of breath,
cough, and wheeze. These symptoms eased up through the week, were almost absent in
weekends, and started over the next workweek. Physical examination including a thorough
examination of the chest and lungs are inconclusive. Her chest x-ray is normal as is her pulmonary
function tests. Which one of the following could be the most likely diagnosis?

A. Anxiety.
B. Berylliosis.
C. Byssiniosis.
D. Silicosis.
E. Occupational asthma.

Incorrect. Correct answer is C


45% answered correctly

Explanation:

Correct Answer Is C

The clinical picture given in this scenario is highly suggestive of byssinosis as the most likely
diagnosis. Byssinosis is a type of occupational lung disease almost exclusively seen in workers
who are in contact with unprocessed raw cotton especially those who are exposed to open bales of
cotton or work in cotton spinning or carding rooms.

Often, byssinosis occurs in workers with at least 10 years of exposure, but it can also develop
acutely in a matter of hours or days.

The etiology of the condition is suggested to be a bacterial endotoxin in cotton dust. The endotoxin
results in bronchoconstriction acutely, and in longer term exposures, chronic bronchitis and gradual
decreases in pulmonary function, particularly in genetically susceptible people. Cotton dust does
not cause chronic obstructive lung disease as (COPD) as previously thought.

Symptoms are chest tightness and dyspnea that reduce with repeated exposure. Symptoms
develop on the first day of work after a weekend or vacation and diminish or disappear by the end
of the week. With repeated exposure over a period of years, chest tightness tends to return and
persist through midweek and occasionally to the end of the week or as long as the person
continues to work. This typical temporal pattern distinguishes byssinosis from asthma (option E);
however, symptoms are relieved by asthma medications. Physical findings of acute exposure are
tachypnea and wheezing while with chronic exposure crackles can be a finding.

During attacks, pulmonary function tests show characteristic patterns of reversible air flow
obstruction and reduced vital capacity, especially if measured during the first work shift. Other than
such times, pulmonary function tests are inconclusive unless chronic disease has set in, in which
case a restrictive pattern is more likely. Hyperresponsiveness to methacholine is often observed.

1863 of 1943
(Option A) Anxiety-induced panic attacks may mimic parts of this clinical picture such as chest
tightness and shortness of breath. If anxiety triggers an asthma attack in an asthmatic patient, the
clinical picture can be identical to the scenario. However, with the exposure to cotton in history and
the temporal course of symptoms in this patient (only present when she is exposed to the irritant)
byssinosis is a better explanation and the more likely cause of this clinical picture.

(Option B) Exposure to and inhalation of beryllium (a metal used in aerospace, computer, electrical
devices, and car industries) is associated with two pulmonary syndromes, i.e., (1) an acute
chemical pneumonitis and (2) a granulomatous lung disease known as chronic beryllium disease
(CBD), or berylliosis . In acute beryllium disease, the metal acts as a direct chemical irritant,
causing a nonspecific inflammatory reaction (acute chemical pneumonitis). Berylliosis (chronic
form) presents with shortness of breath, unexplained cough, fatigue, weight loss, fever, and night
sweats. Some workers may develop severe symptoms very quickly, while others may not
experience signs and symptoms until months or years after their exposure to beryllium. Berylliosis
can continue to progress even after exposure has been removed. With no beryllium exposure in
history, berylliosis is unlikely to be the cause of such presentation. Moreover, the clinical picture
and temporal course of the symptoms in this patient are inconsistent with berylliosis.

(Option D) Silicosis is a progressive, irreversible, and incurable fibrotic pulmonary disease caused
by the inhalation of respirable crystalline silica dust (dust diseases). There three types of silicosis:

Acute silicosis – Induced by short-term (<3 years) exposure to very high levels of silica. It
presents with rapidly progressive dyspnea and respiratory failure with a high mortality rate.

Accelerated silicosis – Associated with exposure to high levels of silica over 3-10 years and
presenting with clinical and radiological features of both acute and chronic silicosis. There is
a higher rate of disease progression compared to chronic silicosis.

Chronic silicosis – occurs with exposures over 10 years. It can manifest as simple silicosis or
complicated silicosis. Simple silicosis is often asymptomatic with small, predominately upper
lobe nodules less than 1cm in size. It may progress to complicated silicosis. In complicated
silicosis, (also known as progressive massive fibrosis (PMF)) nodules conglomerate into
masses greater than 1cm in size. Calcification in masses and in hilar and mediastinal lymph
nodes in common. As the condition progresses, lung function becomes impaired with
development of dyspnea, and potentially cor pulmonale, respiratory failure and death.

The absence of exposure of to silica dust as well as the temporal course of the symptoms in this
patient make silicosis unlikely.

References

• MSD manual – Byssiniosis


Last updated:
Time spent: QID:1612
2023-2-12

1864 of 1943
Peta, aged 3 years, is brought to the Emergency Department with an acute attack of asthma. She is
initially treated with salbutamol inhaler through spacer and mask and oxygen by face mask. She
improves symptomatically, but becomes breathless and wheezy again after 90 minutes. Which one
of the following would be the most appropriate next step in management?

A. Double the dose of salbutamol.


B. Repeat salbutamol and commence oral corticosteroids.
C. Assess the severity of the asthma with spirometry.
D. Measure oxygen saturation to determine the need for hospitalization.
E. Start her on in intravenous aminophylline.

Incorrect. Correct answer is B


45% answered correctly

Explanation:

Correct Answer Is B

Peta initially responded to short-acting beta 2 agonists (salbutamol), but has deteriorated after 90
minutes. The best initial course of action would be repeating salbutamol to treat the current
symptoms as well as early commencement of systemic corticosteroids to prevent further attacks.
Every dose of short-acting beta 2 agonists is up to 12 puffs for those 6 years or older and up to 6
puffs for children 0-5 years. Exceeding the safe limits by doubling the dose (option A) is not
appropriate.

Assessment of the severity of the attack, O2 saturation (option D) and the need for hospitalization
are components of the management plan, but not the first priority. spirometry (option C) in the
acute setting is not a reliable indicator of severity.

Intravenous aminophylline (option E) may be rarely indicated for patients with severe asthma
attack and no or inadequate response to other measures.

References

• Australian Asthma Handbook - Managing acute asthma in clinical setting


Last updated:
Time spent: QID:358
2023-2-12

1865 of 1943
Ashley is brought to your clinic by his mother. He is a 9-year-old boy, who has been diagnosed with
intermittent asthma two years ago. He is on inhaled salbutamol on an as-needed basis. His mother
mentions that he has had to use his medication more frequently recently. Moreover, he has
developed night coughs at least 2-3 times a week. Examination reveals diffuse wheezing over the
lung fields. Which one of the following would be the most appropriate next step in management?

A. Start him on inhaled fluticasone and salmeterol.


B. Start him on inhaled fluticasone.
C. Start him on a 14-day course of oral steroids.
D. Start him on montelukast.
E. Start him on sodium cromoglycate.

Incorrect. Correct answer is D


45% answered correctly

Explanation:

Correct Answer Is D

This child had been diagnosed with mild intermittent asthma and started on a short-acting β2
agonist (salbutamol) on an as-needed basis, but his asthma is poorly controlled. Once adherence
to therapy and inappropriate medication use are excluded as the possible causes of
unresponsiveness to treatment, the next step would be a 2- to 4-week course of montelukast. If the
symptoms improve, montelukast should be continued; otherwise, it should be replaced with the
lowest effective dose of inhaled corticosteroids as the most appropriate next step in
management. Montelukast is used to spare the child from inhaled corticosteroids and their
potential adverse effects if there is a response.

References

• Australian Asthma Handbook - Managing asthma in children


Last updated:
Time spent: QID:359
2023-2-12

1866 of 1943
A 55-year-old man presents to your practice with complaints of shortness of breath and cough for
the past 2 months. After initial evaluation, you arrange a pulmonary function test for him, the result
of which is as follows:

FVC (forced vital capacity) = 60% (normal >80% of predicted)


FEV1 (Forced expiratory volume in 1 second) = 35% predicted (normal >80% of predicted)
FEV1/FVC = 0.58

Which one of the following is the most likely diagnosis?

A. Obstructive pulmonary disease.


B. Restrictive pulmonary disease.
C. Obesity.
D. Myasthenia gravis.
E. Concurrent obstructive and restrictive lung pathologies.

Incorrect. Correct answer is E


45% answered correctly

Explanation:

Correct Answer Is E

Forced vital capacity (FVC) is the amount of air that a patient can exhale in a single expiration after
a deep inspiration, no matter how long it takes. FVC is diminished (less than 80% predicted) in
restrictive pulmonary disease and severe obstructive diseases.

Forced expiratory volume in the first second (FEV1) reflects the airway flow; therefore it is expected
to be within normal range in patients with restrictive pulmonary disease, as there is no obstruction
against the flow. FEV1 is diminished in obstructive diseases.

The best indicator of obstructive vs. restrictive pulmonary disease is the FEV1/FVC ratio (normal
0.75 – 0.85). In obstructive diseases, the FEV1 is decreased, but the FVC remains unaffected or
mildly decreased (unless there is severe obstruction); hence, the ratio will decrease. In restrictive
diseases both FEV1 and FVC decrease so the ratio is expected to be normal or even slightly
increased.

When the FEV1/FVC is reduced, the next step would be looking at the FVC. If FVC is normal a pure
obstructive disease (e.g. asthma) is the expected underlying cause, whereas a decreased FVC is
either due to a reduction in lung volume (e.g. concomitant restrictive lung disease), namely mixed
pattern, or severe obstruction that can result in increased residual volume and consequently a
decreased vitally capacity.

Since this patient has decreased FEV1, FEV1/FVC, and FVC, a severe obstructive disease or a mix
obstructive/restrictive lung pathology is the best explanation and the correct option.

References

• Interpreting pulmonary function tests: Recognize the pattern, and the diagnosis will follow
1867 of 1943
Last updated:
Time spent: QID:364
2023-2-12

1868 of 1943
A 35-year-old man with history of sore throat three weeks ago, presents to the Emergency
Department with a puffy face and decreased urine output for the past 48 hours. On examination, he
has a blood pressure of 160/120 mmHg. His face is swollen especially in the periorbital area. On
lung auscultation, bilbasal crepitations are heard. Which one of the following is correct regarding
his condition?

A. Hematuria is a grave prognostic factor.


B. He is at immediate risk of death from left ventricular failure.
C. Increase in oral intake will result in diuresis.
D. If there is renal tenderness, a renal biopsy should be performed.
E. Dialysis is contraindicated during the acute phase of the illness.

Incorrect. Correct answer is B


45% answered correctly

Explanation:

Correct Answer Is B

Facial edema, hypertension and oliguria are strong pointers towards glomerulonephritis. With the
sore throat in the history, post-streptococcal glomerulonephritis (PSGN) is the most likely cause of
this presentation.

PSGN is induced by infection with specific nephritogenic strains of group A streptococcus (GAS)
such as type 12 and type 49. The clinical presentation can vary from asymptomatic, microscopic
hematuria to full-blown acute nephritic syndrome, characterized by red to brown urine, proteinuria
(which can reach the nephrotic range), edema, hypertension and acute renal failure.

A latent period always occurs between the streptococcal infection and the onset of signs and
symptoms of acute glomerulonephritis. In general, the latent period is 1-2 weeks after a throat
infection and 3-6 weeks after a skin infection. The onset of signs and symptoms at the same time
as pharyngitis is points towards immunoglobulin A (IgA) nephropathy rather than PSGN.

Dark urine (brown-, tea-, or cola-colored) is often the first clinical manifestation of PSGN. Dark urine
is caused by lysis of red blood cells that have penetrated the glomerular basement membrane and
have passed into the tubular system.

Periorbital edema is typical. The onset of puffiness of the face or eyelids is sudden. It is usually
prominent upon waking up and, if the patient is active, tends to subside toward the end of the day.
In some cases, generalized edema and other features of circulatory congestion, such as dyspnea,
may be present. Edema is the result of a defect in renal excretion of salt and water. The severity of
edema is often disproportionate to the degree of renal impairment. Nonspecific symptoms of
PSGN can include general malaise, weakness, and anorexia that are present in 50% of patients.
Approximately 15% of patients complain of nausea and vomiting.

Early death is extremely rare in children (<1%) but is significantly more common in adults (25%).
This is secondary to congestive heart failure and azotemia. Left ventricular failure (congestive
heart failure) is more common in adults (43%) than in children (<5%); therefore, this patient is
potentially at immediate risk of congestive heart failure and death due to volume overload.

1869 of 1943
Nephrotic-range proteinuria is also more common in adults (20%) than in children (4-10%).
Approximately 83% of adults have azotemia compared to 24-40% of children.

(Option A) Hematuria is seen in most patients with PSGN as in all other forms of
glomerulonephritis. It can range from microscopic to gross (cola-or tea-colored urine). Hematuria
is not associated with poor prognosis.

(Option C) Oliguria is present in 10-50% of cases. The oliguria is often transient and diuresis occurs
within 1-2 weeks. Increased water intake does not result in diuresis.

(Option D) Kidney biopsy is not required, unless for those patient in whom other glomerular
disorders are considered due to deviation from the natural course of PSGN or lack of a history of
streptococcal infection. Recognition of PSGN in this patient is clear from the history and the
clinical findings. Renal tenderness is not an indication for renal biopsy.

(Option E) Patients with PSGN have variable reduction in renal function, and some patients require
dialysis during the acute episode.

References

• Medscape - Poststreptococcal Glomerulonephritis

• UpToDate - Poststreptococcal glomerulonephritis


Last updated:
Time spent: QID:804
2023-2-12

1870 of 1943
A 72-year-old man presents with complaint of one episode or blood in the urine. Five years ago, he
underwent colectomy after he was diagnosed with colon cancer. A while back, he developed back
pain for which he was assessed and diagnosed with metastatic bone disease. Three months ago
he was started on tramadol for management of the back pain. He has no urinary symptoms. Which
one of the following is the most appropriate next best step in management?

A. Perform a pelvic and abdominal CT scan.


B. Perform a renal ultrasound.
C. Urine culture.
D. Stop tramadol.
E. Perform an intravenous pyelogram (IVP).

Incorrect. Correct answer is C


45% answered correctly

Explanation:

Correct Answer Is C

Hematuria should be always considered as a sign; therefore, for established cases of hematuria an
underlying cause should be investigated through history, physical examinations and
laboratory/imaging studies. For established hematuria, the most common causes such as vigorous
exercise, menstruation, trauma, viral illnesses, and infections should be excluded first.

One of the most common causes of hematuria even in the absence of symptoms is urinary tract
infection (UTI), for which a urine analysis and culture should be performed.

(Option A) Genitourinary malignancies (either primary or metastatic) can cause hematuria.


Malignancies should always be excluded and CT scan is one of the most accurate and commonly
used means. However, other common causes of hematuria should be excluded first.

(Option B) Renal ultrasound may be later needed as further work-up but not as the first priority
because the hematuria has been painless, making renal stones less likely. Small stones may cause
painless hematuria, but since they almost always pass spontaneously, their detection on
sonography does not change the management plan.

(Option D) Although hematuria has been reported as a rather rare adverse effect of tramadol, initial
assessment should be focused on more common possibilities such as UTI. Even so, cessation of
tramadol in an almost end-stage patient would not be recommended.

(Option E) If imaging studies are required during the evaluation process, CT scan is preferred over
IVP. In the presence of more modern and convenient modalities, IVP is rarely done these days.

References

• http://www.imagingpathways.health.wa.gov.au/index.

• http://www.ncbi.nlm.nih.gov/pmc/articles/PMC265826

1871 of 1943
• http://www.aafp.org/afp/2001/0315/p1145.html

• http://www.rxlist.com/ultram-er-side-effects-drug-
Last updated:
Time spent: QID:150
2023-2-12

1872 of 1943
A 65-year-old man presents to your practice with complaint of blood in the urine. These episodes of
hematuria have all been painless. Which one of the following is the least likely cause of painless
hematuria?

A. Cancer within the kidney.


B. Anticoagulantion therapy.
C. Glomerulonephritis.
D. Benign prostatic hyperplasia (BPH).
E. Use of cyclophosphamide.

Incorrect. Correct answer is E


45% answered correctly

Explanation:

Correct Answer Is E

All the given options can present with painless hematuria except cyclophosphamide.
Cyclophosphamide use can result in hemorrhagic cystitis. With cystitis, the hematuria is more likely
to be painful rather than painless.

The following can result in hematuria (painless or painful):

Urothelial cancers (kidney cancers, bladder cancers, etc)


Hydronephrosis/distention
Renal vein thrombus / renal artery embolism
Arteriorvenous malformation
Papillary necrosis (sickle cell disease)
Hypertension
Glomerulonephritis
Structural abnormalities (polycystic kidney disease, medullary sponge kidney, etc)
Nephrolithiasis
Urinary tract infections (pyelonephritis, cystitis, parasitic infections, etc)
Ureteral strictures
BPH
Prostate cancer
Prostatic procedures
Trauma (including traumatic catheterization)
Exercise-induced hematuria
Bleeding diathesis / anticoagulation
Urethritis
Urethral diverticulum
Hypercalciuria/hyperuricosuria
Urinary tract fistulas

Mimics of hematuria:

Menstruation
Drugs (phenazopyridine, pyriduim, rifampin, nitrofurantoin, etc)
1873 of 1943
Pigmenturia
Beeturia

References

• http://www.uptodate.com/contents/etiology-and-eval

• http://emedicine.medscape.com/article/2056130-over
Last updated:
Time spent: QID:835
2023-2-12

1874 of 1943
A couple has presented to your office for infertility consult. They have been trying to start a family
for over a year. The woman’s history and examination results are normal, but the man is found to
have absent vas deferens. Which one of the following would be the most appropriate advice for
them?

A. They should have sperm donation.


B. The vas deferens should be fused to the ejaculatory duct.
C. Sperm aspiration and intrauterine fertilization should be tried.
D. Sperm aspiration for frozen sample and repeated tubal insemination.
E. In vitro fertilization.

Incorrect. Correct answer is E


45% answered correctly

Explanation:

Correct Answer Is E

One to two percent of infertile men have congenital absence of the vas deferens. Most have
mutations of the cystic fibrosis transmembrane conductance regulator (CFTR) gene. Many infertile
men with mutations of CFTR present with infertility in the absence of other typical features of
cystic fibrosis (e.g. respiratory and pancreatic disease).

Using in vitro fertilisation is the most appropriate reproductive technique for this couple. Sperms
are aspirated from epididymis or testis and then injected into the aspirated oocytes in vitro.

NOTE - Patients with congenital bilateral absence of the vas deferens may have genetic mutation
commonly present in cystic fibrosis. Such men and their partners who are considering assisted
reproductive techniques to achieve pregnancy should have genetic screening and counselling.
Screening the female partner may be more cost effective than screening the patient because if she
is negative, the risk that their children having cystic fibrosis is almost zero.

(Option A) Sperm donation, while the male sperms are retrievable, is not an appropriate treatment
option.

(Option B) Fusion of vas deferens to ejaculatory duct is considered appropriate for male patients,
who have vas deferens but has undergone vasectomy.

(Options C and D) Aspirated sperms are not enough in number to make intrauterine infertilization
or tubal insemination acceptable methods.

References

• http://www.uptodate.com/contents/treatment-of-male

• http://humrep.oxfordjournals.org/content/15/2/431.
Last updated:
Time spent: QID:897
2023-2-12

1875 of 1943
A 65-year-old man, who is on chronic hemodialysis every other day due to chronic renal failure,
presents to the Emergency Department with generalized weakness and dyspnea. He is non-
compliant with his dialysis sessions and had his last session 5 days ago. Which one of the
following would be the most appropriate next action to take?

A. Call the dialysis unit.


B. Arterial blood gas (ABG) analysis.
C. ECG.
D. Chest X-ray (CXR).
E. Furosemide.

Correct
45% answered correctly

Explanation:

Correct Answer Is A

This patient has skipped dialysis sessions. Patients with end stage renal disease have volume
overload and hazardous metabolic derangements such as metabolic acidosis and hyperkalemia,
both of which can be fatal. Volume overload can cause congestive heart failure and pulmonary
edema presenting with dyspnea as the most common manifestation. Hyperkalemia is also of great
concern because it can cause lethal cardiac arrhythmias. No matter what the underlying cause
of this presentation is, arrangement for urgent dialysis is the most appropriate next step in
management as it shoud be arranged at earliest time. While the arrangements are in process,
investigations such as ECG (option C), CXR (option D) and ABG (option B) can be undertaken. ECG
is of paramount importance because with evidence of arrhythmias induced by hyperkalemia,
calcium gluconate should be administered promptly for cardiac protection.

Urgent dialysis can remove excess fluid and the congestion; in the mean time, standard protocols
should be used for such patients. In case of pulmonary edema, furosemide (option E) can be used
in patients with preserved renal function as a component of acute management of pulmonary
edema.

References

• Family Practice Notebook - Emergency Care in ESRD


Last updated:
Time spent: QID:917
2023-2-12

1876 of 1943
A 70-year-old woman presents with increasing generalized abdominal pain for the past 2 hours.
She is a known case of chronic renal failure and has been on peritoneal dialysis for the past 18
months. Her last dialysis session was hours ago. On examination, she appears toxic. Her blood
pressure is 140/95mmHg, heart rate 110bpm, and temperature 39°C. Her abdomen is distended
and diffusely tender to deep palpation. Rebound tenderness is also noted. Blood exam is
significant for a white cell count of 18,000/mm3. Which one of the following is the most
appropriate next step in management?

A. Erect and supine abdominal films.


B. Ultrasonography of the abdomen.
C. CT scan of the abdomen and pelvis.
D. Peritoneal fluid analysis and Gram stain.
E. Comparison of ascitic fluid amylase with serum amylase.

Incorrect. Correct answer is D


45% answered correctly

Explanation:

Correct Answer Is D

In the setting of peritoneal dialysis (PD), peritonitis is the most likely explanation to this
presentation.

Peritonitis is a common complication of peritoneal dialysis (PD), and is associated with significant
morbidity, catheter loss, transfer to hemodialysis, transient loss of ultrafiltration, possible
permanent membrane damage, and occasionally death.

Among PD patients, peritonitis may be PD-related or secondary (enteric). PD-related peritonitis is


due to touch contamination with pathogenic skin bacteria or to catheter-related infection.
Secondary peritonitis is caused by underlying pathology of the gastrointestinal tract. PD-related
peritonitis is much more common than secondary peritonitis; the latter, however, is associated with
higher rates of complications and mortality. Conditions that may lead to secondary peritonitis
include cholecystitis, appendicitis, ruptured diverticulum, treatment of severe constipation,
perforation during endoscopy, bowel ischemia, and incarcerated hernia.Secondary peritonitis may
also be caused by seeding from the blood or vagina but this is less common compared to intra-
abdominal causes.

The most common signs and symptoms of peritonitis among PD patients are abdominal pain and
cloudy peritoneal effluent. Other signs and symptoms include fever, nausea, diarrhea, abdominal
tenderness, rebound tenderness, and occasionally systemic signs, including hypotension.

The diagnosis of peritonitis should be suspected in a PD patient with characteristic clinical


presentation. Analysis of the peritoneal fluid is the most appropriate next step when PD-related
peritonitis is suspects.

A presumptive diagnosis is made if the peritoneal fluid white cell count is greater than 100
cells/mm3 and the percentage of neutrophils is greater than 50%. The white cell count is greatly
dependent on the dwell time of the dialysis catheter; therefore, with a white cell count of less than
1877 of 1943
100,but neutrophil percentage of greater than 50%, the diagnosis remains PD-related peritonitis
untill proven otherwise.

Peritoneal fluid culture and Gram stain should also be performed. Although it is usually negative,
the identification of any organisms is a helpful guide to therapy since the gram stain is predictive of
the culture results. Gram stain may be particularly useful in the early diagnosis of fungal peritonitis.
Peritoneal fluid cultures are always indicated and performed, but empiric antibiotic therapy should
not be delayed until culture results are available.

Culture of purulent drainage from the exit site should be performed since isolation of the same
organism as from peritoneal fluid suggests that the exit site infection may be the cause of
peritonitis.

Peritoneal fluid amylase and lipase should also be measured. The peritoneal fluid amylase and
lipase concentrations are occasionally elevated (>50IU/L) among patients with secondary
peritonitis, but not among patients with PD-related peritonitis. This may help to distinguish between
PD-related and secondary peritonitis. Among patients with secondary peritonitis, the elevated
amylase concentration may be due to a leak from a bowel perforation or directly from an inflamed
pancreas. An elevated level of peritoneal lipase above 15IU/L suggests pancreatitis as the cause of
the peritonitis.

NOTE – PD-related peritonitis is a different entity from spontaneous bacterial peritonitis (SBP).

(Options A, B and C) Radiographic manifestations of peritonitis are nonspecific and such studies
are not routinely performed among patients with suspected PD-related peritonitis. However, they
are indicated when secondary peritonitis is suspected.

(Option E) Amylase and lipase levels in peritoneal fluid are not used in comparison with their serum
levels.

In the presence of systemic symptoms, blood cultures should be obtained, although they are
seldom positive.

References

• http://www.pdiconnect.com/content/30/4/393.full

• http://www.uptodate.com/contents/clinical-manifest

• http://homedialysis.org.au/wp-content/uploads/2013
Last updated:
Time spent: QID:925
2023-2-12

1878 of 1943
A 60-year-old man with end-stage renal failure presents to the dialysis unit for a session of
hemodialysis. His current medications are enalapril, hydrochlorothiazide, and aspirin. He has a
blood pressure of 140/95 mmHg. After the session, which was uneventful, his blood pressure is
found to be 160/90mmHg. Which one of the following is more likely to be the cause of his
increased blood pressure?

A. Allergic reaction to the dialysis fluid.


B. Overdialysis.
C. Enalapril overdose.
D. Hypokalemia.
E. Anemia.

Incorrect. Correct answer is B


45% answered correctly

Explanation:

Correct Answer Is B

While hemodialysis lowers blood pressure (BP) in most hypertensive end-stage renal disease
patients, approximately 15% of patients show a paradoxical increase in BP during haemodialysis,
termed intradialytic hypertension (IDH). IDH is defined as either of the following:

An increase in mean arterial blood pressure (MAP) ≥ 15 mmHg during or immediately after
hemodialysis
An increase in systolic BP (SBP) >10 mmHg from pre- to post-dialysis
Hypertension during the second or third hour of hemodialysis after significant ultrafiltration
has taken place
An increase in BP that is resistant to ultrafiltration
Aggravation of pre-existing hypertension or development of de novo hypertension with
erythropoietin stimulating agents.

Despite extensive investigations as to the mechanism and pathophysiology of IDH, the exact
pathogenesis remains unclear. Numerous factors have been implicated, including:

1. Renin-angiotensin system activation because of ultrafiltration (UF) induced hypovolemia


2. Sympathetic overactivity
3. Intradialytic Ca++/k+ variations
4. Blood viscosity/haemoconcentration-induced vasoconstriction caused by erythropoietin
treatment
5. Fluid overload
6. Increased cardiac output
7. Endothelin-driven vasoconstriction
8. Antihypertensive drug removal by dialysis treatment

Several studies and reports highlight the important role of fluid overload, hemodynamic changes
and increased endothelin level. The importance of other hypothesis such as renin-angiotensin
system activation, sympathetic overactivity and ionic variations seems secondary. Fluid removal
remains the key point for treatment of IDH.
1879 of 1943
Of the options, overdialysis is the most appropriate one. In majority of dialysis patient, overdialysis
results in hypotension as a complication; in some, however, this excess fluid removal leads to
activation of compensatory mechanisms by which the blood pressure can increase. Also, excess
removal of enalapril with dialysis can be an explanation.

(Option A) Allergic reaction to dialysis fluid has not been shown as a possible
mechanism responsible for intradialytic hypertension

(Option C) Enalapril overdose can cause hypotension, not hypertension.

(Option D) Hypokalemia induces vasoconstriction and increased blood pressure;however, it is does


not appear to be a mainn contributor to IDH.

(Option E) Anemia, by itself, is not associated with hypertension during or shortly after dialysis, but
erythropoietin stimulating agents used for treatment of anemia in patients with CKD during dialysis
is among postulated causes of hypertension associated with dialysis.

References

• PubMed - Intradialytic Hypertension: A Less-Recognized Cardiovascular Complication of


Hemodialysis

• KARGER - Intradialytic Hypertension: It Is Time to Act


Last updated:
Time spent: QID:926
2023-2-12

1880 of 1943
A 64-year-old man with end-stage renal disease presents to the dialysis unit for a pre-scheduled
hemodialysis session. He proceeds through the session uneventfully, but his blood pressure
reading is 80/60 mmHg at the end of the session. His pre-dialysis blood pressure was 150/90
mmHg. A blood panel shows a hemoglobin level of 80 g/L. Which one of the following could be the
most likely cause of this drop in blood pressure?

A. Allergic reaction to the dialysis fluid.


B. Overdialysis.
C. Anemia.
D. Hypokalemia.
E. Enalapril overdose.

Incorrect. Correct answer is B


45% answered correctly

Explanation:

Correct Answer Is B

The scenario is a typical example of intradialytic hypotension. Intra- or post-dialysis hypotension is


a well recognized and common complication of hemodialysis.

The incidence of a symptomatic reduction in blood pressure during (or immediately following)
dialysis ranges from 15-50% of dialysis sessions. In some patients, the development of orthostatic
hypotension necessitates intravenous fluid replacement before they are able to leave the dialysis
unit. This problem contributes to the excessive morbidity associated with the dialysis procedure.

There are two clinical patterns of dialysis-associated hypotension:

Episodic hypotension, which typically occurs during the later stages of dialysis and presents
with vomiting, muscle cramps, and other vagal symptoms (such as yawning)

Chronic persistent hypotension, which may occur in patients with long-term dialysis. A pre-
dialysis systolic blood pressures of less than 100 mmHg is frequently observed

The etiology of intradialytic hypotension is divers and includes:

A rapid reduction in plasma osmolality, which causes extracellular water to move into the
cells
Rapid fluid removal in an attempt to attain "dry weight", particularly among those with large
inter-dialytic weight gains
Inaccurate determination of true ‘dry weight’
Autonomic neuropathy
Diminished cardiac reserve
Use of acetate rather than bicarbonate as a dialysate buffer
Intake of antihypertensive medications that can impair cardiovascular stability
Use of a lower sodium concentration in the dialysate
Sudden release of adenosine during organ ischemia
Ingestion of a meal immediately before or during dialysis

1881 of 1943
Arrhythmias or pericardial effusion with tamponade, which are volume-unresponsive
Reactions to the dialyzer membrane, which may cause wheezing and dyspnea as well as
hypotension
Increased synthesis of endogenous vasodilators, such as nitric oxide
High magnesium concentrations in the dialysate
Failure to increase plasma vasopressin levels

Overdialysis, by excess fluid removal, is an important, and probably the most common cause of
intradialytic hypotension.

(Option A) Allergic reaction to dialyzer membrane (not fluid) can be a possible cause of
hypotension in few cases. In addition to hypotension, wheezing and dyspnoea are also expected.

(Option C) Anemia has not been recognized as a cause of intradialytic hypotension.

(Option D) Hypokalemia induces vasoconstriction, with increased blood pressure being the
expected result. Hypokalemia is unlikely to cause hypotension in this setting.

(Option E) Enalapril overdose can cause hypotension regardless of dialysis. Circulatiing enalapril is
expected to decrease during dialysis; therefore, enalapril overdose, while the patient has not have
hypotension before the session is very unlikely.

References

• http://www.uptodate.com/contents/intradialytic-hyp
Last updated:
Time spent: QID:927
2023-2-12

1882 of 1943
A 72-year-old woman has been on routine dialysis sessions for the past 6 months due to end-stage
renal disease. At the beginning of each session, she is found to have a high blood pressure (BP).
During the session the BP normalizes, but goes up again after the session. Which one of the
following would be the most appropriate management option for this patient?

A. Addition of hypertensive medications.


B. Addition of furosemide, daily.
C. Increasing the dialysis time.
D. Sedation before dialysis.
E. Decreasing the dialysis time.

Incorrect. Correct answer is C


45% answered correctly

Explanation:

Correct Answer Is C

Although hypotension during hemodialysis is a frequent complication, some patients (5-15%)


develop paradoxical hypertension in the later stages of dialysis or when the patients comes off the
dialysis machine, a time at which most of the excess fluid has already been removed. The
pathogenesis is unclear; however, the following mechanisms have been hypothesized as the cause:

1. Renin-angiotensin system activation because of ultrafiltration (UF) induced hypovolemia


2. Sympathetic overactivity
3. Intradialytic Ca++/k+ variations
4. Blood viscosity/hemoconcentration-induced vasoconstriction caused by erythropoietin
treatment
5. Fluid overload
6. Increased cardiac output
7. Endothelin-driven vasoconstriction
8. Antihypertensive drug removal by dialysis treatment

The optimal therapy for this problem is not known. While antihypertensive medications such as
angiotensin converting enzyme inhibitors (ACE inhibitors) and alpha-blockers have been used
before (or during) dialysis, they have not been predictably effective. Carvedilol, which blocks
endothelin-1 release, appears to be effective.

Although there are no validated universal guidelines regarding management of such patients, fluid
removal has been accepted as the first-line treatment for intradialytic hypertension (IDH).
Theoretically, increasing the time of the dialysis session and ultrafiltration (UF) rate would be
efficient; however, this decision faces many difficulties such as patient refusal or the unit
limitations. This treatment should be done with caution to avoid hazardous blood pressure drop
that may occur in the elderly or patients with severe comorbidity.

The dry weight of patients should be gradually reduced by increasing the dialysis time and the UF
rate. In addition, patients should be advised to decrease their daily salt and water intake in
between their dialysis sessions.

1883 of 1943
(Options A and B) Addition of anti-hypertensive or other medications ACE inhibitors or angiotensin
receptor blockers (ARBs), beta blockers, endothelin-1 receptor blockers, furosemide, etc has been
associated with conflicting results. In some patients, hypertensive crises may occur. These
hypertensive crises are not persistent and usually the blood pressure level quickly decreases
spontaneously; however, addition of antihypertensive medications might be indicated. There is no
comment regarding a hypertensive crisis in this patient to necessitate addition of
antihypertensives. Furosemide is not the first-line option for management of hypertension crisis if
it occurs.

(Option D) Sedation before dialysis has no role in management of IDH or post- dialytic
hypertension.

(Option E) Decreasing the dialysis time results in insufficient excess fluid removal and hypertension
due to volume overload.

References

• https://www.karger.com/Article/FullText/313031

• http://www.medscape.com/viewarticle/733263#vp_1

• http://www.ncbi.nlm.nih.gov/pmc/articles/PMC283036
Last updated:
Time spent: QID:928
2023-2-12

1884 of 1943
A 50-year-old man presents with complaints of several episodes of painless gross hematuria and a
mass he felt in his left loin recently. The significant finding on examination is a non-tender loin
mass. Urine analysis shows blood in the urine. Urine culture is negative. Which one of the following
is the most appropriate next step in management?

A. CT scan.
B. Cystoscopy.
C. MRI.
D. Intravenous pyelography (IVP).
E. Intravenous urogram (IVU).

Correct
45% answered correctly

Explanation:

Correct Answer Is A

Gross hematuria is always concerning and warrants thorough investigations, because the
prevalence of urinary tract malignancies among patients with macroscopic hematuria has been
reported to be as high as 19%, but usually ranges from 3-6%.

Risk factors can help in determining which patients are at higher risk of urinary tract and bladder
malignancies. Risk factors include:

Age >40 years


A history of smoking
History of gross hematuria
History of chronic cystitis or irritative lower urinary tract symptoms (e.g. frequency, urgency,
dysuria, nocturia, hesitancy, sensation of incomplete emptying)
History of pelvic irradiation
Exposure to occupational chemical and dyes (e.g. heavy phenacetin use, treatment with high
doses of cyclophosphamide, aristolochic acid)

One of the most common causes of hematuria is urinary tract infection, which can be investigated
with urine microscopy, culture and sensitivity (MC&S). Ureteric and renal stones are another
common cause, but these typically present with pain and microscopic or gross hematuria. In cases
where macroscopic hematuria or risk factors are present, or if another cause cannot be
determined, more extensive investigations are recommended to exlcude an underlying malignancy.

Despite extensive investigation, studies have shown that in up to 50% of patients with macroscopic
hematuria and 70% with microscopic hematuria have no identifiable cause is found. This could be
attributed to transient benign physiological conditions, including vigorous physical exercise, sexual
intercourse or menstrual contamination.

In this patient, urinalysis is negative but for blood. There is no increase in WBC to indicated
infection; furthermore, a urine culture is negative, excluding infections as a cause. There are also
no red cell casts or other pathologic findings indicating conditions such as glomerulonephritis as

1885 of 1943
the underlying pathophysiology. An enlarged or otherwise abnormal prostate would have come to
attention on physical exam.

Given the presence of gross hematuria, the age (>40) and the loin mass, the most important
concern would be a renal tumor until proven otherwise.

Such patients should undergo non-contrast CT scan of kidney, ureter and bladder (KUB) (first-line)
or ultrasound (second-line). CT-KUB is a non-contrast study – the current gold standard in
identifying ureteric and renal stones, with 94-98% sensitivity; however, the sensitivity is much lower
for indentifying genitourinary malignancies.

CT scan findings associated with increased risk of urinary tract malignancies justify CT-KUP IVP.
CT-KUB IVP (multidetector CT scanning of kidneys, ureter and bladder after intravenous contrast
media is administered), also known as CT urography (CTU) is as accurate and diagnostic as a
combination of ultrasonography, IVP and CT-KUB. This, however, should be performed by,or at least
under consultation with an urologist. Sensitivity for detecting pathology in patients with hematuria
varies from 94-100%, with a 97.4% specificity.

With the loin mass, the cause of hematuria is more likely to originate from upper parts of urinary
tract that are beyond the reach of cystoscopy. Cystoscopy, however, can be considered somewhere
along the diagnostic pathway, for patients with voiding symptoms or where there is suspicion of
bladder cancer, but not as the next best step in this scenario.

IVP (intravenous pyelogram), also called IVU (intravenous urogram) is inferior to CT scan.

References

• http://www.imagingpathways.health.wa.gov.au/index.

• http://www.racgp.org.au/afp/2013/march/macroscopic
Last updated:
Time spent: QID:977
2023-2-12

1886 of 1943
A 32-year-old man presents to the Emergency Deaprtment with left loin pain and hematuria. He is
HIV positive and on an anti-HIV treatment regimen including indinavir. This is the first time he is
having sych problem. Urine dipstick is positive for blood but negative for urinary tract infection.
Which one of the following is the most appropriate next step in management?

A. Intravenous pyelogram (IVP).


B. Ultrasound (US).
C. Non-contrast CT scan.
D. KUB X-ray.
E. Triple-phase CT scan.

Incorrect. Correct answer is B


45% answered correctly

Explanation:

Correct Answer Is B

Indinavir sulfate is an HIV protease inhibitor. Indinavir-induced nephrolithiasis is a well-recognized


adverse effect of this drug, occurring in almost 12.4% of patients. If indinavir-induced
nephrolithiasis develops, the drug should be discontinued and an alternative antiretroviral be used.

Non-contrast helical CT scan (option C) is the criterion standard for evaluation of nephrolithiasis.
This modality can detect both stones and urinary tract obstruction. Stones not visualized on IVP or
KUB, usually are detected on CT scan; however, since stones secondary to indinavir are not
radiopaque and signs of obstruction may be minimal, this modality may not be as accurate and
miss the diagnosis. This holds true about x-ray KUB (option D) and ultrasound.

Although not accurate as non-contrast helical CT scan, ultrasound can detect obstruction caused
by large, clinically significant stones and probably the stone itself. This modality has been
recommended as the initial diagnostic option by many authors. It is safe, readily available, cost-
effective, and spares patients from the risk of radiation in most cases. Again, the accuracy is
reduced in detection of stones secondary to indinavir.

In fact, studies have shown that contrast CT-scan is more likely to visualize indinavir-induced
stones, but since most of such stones, especially if not large enough to cause obstruction and be
detected on ultrasonography, will pass with conservative management, contrast enhanced imaging
studies as contrast CT or IVP (option A) unnecessarily puts the patient at risk of radiation and/or
contrast media.

Contrast CT scan, however, should be considered for patients in whom the symptoms persist or
other diagnosis is suspected based on clinical grounds.

Triple phase CT scan (option E) is mostly used for diagnosis of liver lesions. It is not indicated
diagnosis of renal stones but may be considered if an alternative diagnosis

References

• Diagnostic Imaging Pathways - Loin Pain (Renal Colic)


1887 of 1943
• UpToDate - Diagnosis and acute management of suspected nephrolithiasis in adults

• PubMed - HIV medication-based urolithiasis


Last updated:
Time spent: QID:997
2023-2-12

1888 of 1943
During a routine health assessment, a 43-year-old man is found to have established asymptomatic
hematuria. Which one of the following is the most appropriate next step in management?

A. Ultrasonography of the renal system.


B. CT scan.
C. Cystoscopy.
D. Urine culture.
E. Intravenous pyelogram (IVP).

Incorrect. Correct answer is D


45% answered correctly

Explanation:

Correct Answer Is D

Hematuria may be seen in a variety of situations including, but not limited to:

Infections
Renal stones
Glomerulonephritis
Urinary tract malignancies
Trauma

Infections are the most common cause of hematuria in both symptomatic and asymptomatic
patients. For this reason, the next best step in management of every patient with hematuria would
be a urine analysis and culture. Urologic malignancies (mostly cancer of the bladder) should be
considered and ruled out once urine culture is negative and renal stones are excluded using
ultrasound (the best initial test) or spiral CT (the most accurate test).

References

• http://www.imagingpathways.health.wa.gov.au/index.
Last updated:
Time spent: QID:321
2023-2-12

1889 of 1943
A 75-year-old man presents to your practice after he noticed blood in his urine. A four-phase
contrast CT scan is obtained that shows a 1cm mass in the left kidney, highly suggestive of renal
cell carcinoma. His medical history is significant for acute myocardial infarction 1 month ago for
which he underwent angioplasty and drug-eluting stent placement. Currently, he is on dual
antiplatelet therapy (DAPT) with daily clopidogrel 75mg and aspirin 100mg. Which one of the
following would be the most appropriate next step in management ?

A. Nephrectomy.
B. Heminephrectomy.
C. Repeat the CT scan in 12 months.
D. CT-guided percutaneous biopsy.
E. Repeat the CT scan in 3 months.

Incorrect. Correct answer is C


45% answered correctly

Explanation:

Correct Answer Is C

Small renal masses can be found incidentally, or during evaluation for urinary symptoms, often by
ultrasound. Once a mass is found in the kidney on ultrasound, a four-phase contrasted CT scan
should follow for further evaluation, provided renal function allows use of contrast media. These
four phases include: (1) arterial, (2) corticomedullary, (3) nephrographic and (4) excretory phases.
This study allows a detailed examination of each aspect of the functional anatomy of the kidney.

The majority of renal lesions are benign simple cysts, which would require no further work-up.
However, completely solid, mixed (solid and cystic), and cystic renal lesions that enhance with
contrast are likely to be malignant.

A small renal mass is defined as a mass <4 cm that has enhancement on abdominal contrast
imaging. For such lesions, surgical resection of the mass is the most appropriate option if life
expectancy is >5 years and the patient is a good candidate for surgery.

However, this patient has had an MI one month ago. Major surgery within the first 6 months after
MI is associated with high risk of mortality; therefore, this patient is not a candidate for surgical
resection of the mass. In such cases active surveillance of a small renal mass with CT or MRI in 6-
to 12-month intervals is recommended.

Even if a small renal mass has imaging characteristics highly suspicious for RCC, active
surveillance may be appropriate, particularly in patients with medical comorbidities that can
increase the risk of active intervention such as surgery, in elderly patients, and in those with
decreased life expectancy (≤5 years). Renal impairment may also be an indication for active
surveillance in some patients. Active surveillance means that the patient will either have delayed
treatment or no treatment at all.

NOTE - Active surveillance is not generally recommended for healthy younger patients
because while there is some evidence to suggest such approach if the lesion is less than 1 cm;
more data is needed before adopting this as a standard protocol in this patient group. Another

1890 of 1943
issue of concern, when considering active surveillance in young patients, is the number of scans
they will require over a lifetime, which is associated with considerable exposure to radiation.

(Options A and B) Tumor resection is the management of choice for small renal masses that are
suspected of malignancy. Such masses are solid, mixed solid, or cystic renal lesions that
enhance with contrast.

If possible, nephron-sparing surgery or partial nephrectomy rather than a total nephrectomy is


performed to allow for preservation of renal function. However, in the following situation total
nephrectomy is the treatment of choice:

Tumor size ≥7 cm
Those tumors that have a more central location
Suspected lymph node involvement
Tumor with associated renal vein or inferior vena cava (IVC) thrombus
Direct extension into the ipsilateral adrenal gland

Even in the presence of the above, patients with any of the following conditions must have partial
rather than total nephrectomy:

A solitary kidney
Multiple, small, and/or bilateral tumors
Patients with or at risk for chronic renal disease

(Options D) The role of renal biopsy is controversial in the setting of a small renal mass. In
particular, there is disagreement as to whether it is necessary to biopsy these lesions before
planning management and in what settings. Circumstances under which a renal biopsy might be
considered are where the patient is not a surgical candidate (controversial), life expectancy is ≤5
years (controversial), or the patient requests a definite diagnosis before proceeding with the
surgery. Biopsy is not recommended for patients who are candidates for active surveillance.

NOTE - With available evidence, it may be safe to assume that routine discontinuation of the
antiplatelet therapy is not necessary in low risk patients planned to undergo percutaneous renal
biopsies.

(Option E) Recommended interval for active surveillance with CT scan or MRI is 6-12 months, not 3
months.

References

• RACGP - Incidentally detected small renal masses

• Diagnostic Imaging Pathways - Renal Mass

• UpToDate - Definitive surgical management of renal cell carcinoma


Last updated:
Time spent: QID:1108
2023-2-12

1891 of 1943
A 77-year-old man presents to your clinic with vague abdominal pain. Among initial laboratory
tests, hematuria is notable. A repeated urinalysis confirms hematuria. You order a 4-phase
contrasted abdominal CT scan that shows a 1cm complex mass in the lower pole of the right
kidney. He is otherwise asymptomatic, enjoys a healthy life, and do not have any comorbidities.
Which one of the following is the most appropriate inital management option for him?

A. Repeat the CT scan in 12 months.


B. Nephrectomy.
C. CT-guided percutaneous biopsy.
D. Reassurance.
E. Thermal ablation.

Incorrect. Correct answer is B


45% answered correctly

Explanation:

Correct Answer Is B

Abdominal pain and hematuria in the presence of a renal mass is renal cell carcinoma (RCC) until
proven otherwise. Small renal masses (<4 cm) can be found incidentally, or during evaluation for
urinary symptoms, often by ultrasound as the initial imaging modality. Once a mass is found in the
kidney on ultrasound, a 4-phase contrasted CT scan should follow for further evaluation if contrast
media is not contraindicated.

Solid or complex masses (a lesion demonstrating both cystic and solid structures), as well as
cystic masses enhanced with contrast are suspected to be malignant and treatment is warranted.

NOTE - By definition, a renal lesion <4 cm in its largest diameter that shows contrast
enhancement on abdominal imaging is a small renal mass. These lesions can be solid or
complex cystic. Simple cysts not enhancing with contrast are excluded.

If the patient is a good candidate for surgical resection and has a life expectancy >5 years, surgical
resection of the small renal mass would be the most appropriate initial management. This patient
is otherwise fit and healthy with no comorbid condition precluding him from surgical tumor
resection. Partial nephrectomy is the procedure of choice for masses less than 7cm that are not
centrally located. This is to preserve as much renal function as possible.

(Option A) Active surveillance with CT scan or MRI in 6- to 12-monht intervals is recommended for
patients with small renal masses who are not good candidates for surgical resection (e.g., have a
comorbid condition), or have a life expectancy of ≤5 years. Renal impairment may also be an
indication for active surveillance in some patients. Active surveillance means that the patient will
either have delayed treatment or no treatment at all. This patient has no comorbid condition
precluding him from tumor resection.

1892 of 1943
(Option C) The role of renal biopsy is controversial in the setting of a small renal mass. There is
disagreement as to whether it is necessary to biopsy such lesions before planning management
and in what settings. Circumstances under which a renal biopsy might be considered are where the
patient is not a surgical candidate (controversial), life expectancy is ≤5 years (controversial), or the
patient requests a definite diagnosis before proceeding with the surgery. Biopsy is not
recommended for patients who are candidates for active surveillance.

(Option D) With this clinical picture, the chances are that the tumor is malignant; therefore,
reassurance cannot be given unless investigations exclude malignancy.

(Option E) Ablation of renal mass can be performed with either freezing (cryoablation) or heat
(radiofrequency or microwave ablation). Using a percutaneous approach, a needle is used to ablate
the tumor with heat of freezing. After ablation, all patients should undergo surveillance to assess
for residual or recurrent tumor that would require additional therapy. Although initial studies have
shown favorable short-term results with thermal ablation, long-term outcomes are yet to be
studied. Thermal ablation is reserved for patients with a small RCC who are not candidates for
surgery. In addition, ablation is a treatment option for RCC after partial nephrectomy, those with a
unilateral or transplanted kidney, or for patients at risk for multiple RCCs over their lifetime.

References

• RACGP - Incidentally detected small renal masses

• Diagnostic Imaging Pathways - Renal Mass

• UpToDate - Definitive surgical management of renal cell carcinoma


Last updated:
Time spent: QID:1110
2023-2-12

1893 of 1943
A 62-year-old man is referred to your clinic for a health checkup by his insurance company. An
ultrasound of the kidney, ureter, and bladder reveals a 3-cm heterogeneous mass in the upper pole
of his right kidney. A contrast-enhanced CT scan is ordered that confirms the presence of a
contrast-enhancing cystic mass. Which one of the following would be the most appropriate
management for this patient?

A. Total nephrectomy.
B. Review in 12 months.
C. Percutaneous biopsy.
D. Partial nephrectomy.
E. Thermal ablation.

Incorrect. Correct answer is D


45% answered correctly

Explanation:

Correct Answer Is D

Rising use of imaging studies as a means of diagnosis has increased the rate at which small renal
masses are detected. Incidentally-found small renal masses (incidentalomas) are now a common
clinical scenario. Recent data have revealed that over 50% of renal cell carcinomas (RCCs) are
incidentally found, and the classic presentation with a triad of gross hematuria, flank pain, and
abdominal mass is not commonly encountered.

The majority of incidentally-detected renal masses are benign simple cysts that require no further
work-up. However the following renal masses are more likely to be malignant:

Completely solid renal masses


Mixed solid and cystic renal lesions
Cystic lesions that enhance with contrast

By definition, a renal lesion <4cm in its largest diameter that shows contrast enhancement on
abdominal imaging is a small renal mass. These lesions can be solid or complex cystic. Simple
cysts not enhancing with contrast are not considered renal small masses.

For small renal masses, surgical resection is the most appropriate option if life expectancy is >5
years and the patient is a good candidate for surgery. This man has a 3-cm complex tumor and
needs surgical resection of the tumor as the most appropriate management option.

For renal masses smaller than 7-cm in size, nephron-sparing surgery, or partial nephrectomy rather
than a total nephrectomy is performed to allow for preservation of renal function. This patient with
a 1cm mass is most likely to benefit from partial nephrectomy as the most appropriate
management option.

(Option A) Total nephrectomy is the treatment of choice in the following situations:

Tumor size ≥7 cm
Tumors with a more central location
Suspected lymph node involvement
1894 of 1943
Tumor with associated renal vein or inferior vena cava (IVC) thrombus
Direct extension into the ipsilateral adrenal gland

Even in the presence of the above, patients with any of the following conditions must have partial
rather than total nephrectomy:

A solitary kidney
Multiple, small, and/or bilateral tumors
Patients with or at risk for chronic renal disease

(Option B) Active surveillance with CT scan or MRI in 6- to 12-monht intervals is recommended for
patients with a small renal mass, who are not good candidates for surgical resection (e.g., have a
comorbid condition), or have a life expectancy of ≤5 years. Renal impairment may also be an
indication for active surveillance in some patients. Active surveillance means that the patient will
either have delayed treatment or no treatment at all. This patient has no comorbid condition
precluding him from tumor resection.

(Option C) The role of renal biopsy is controversial in the setting of a small renal mass. In
particular, there is disagreement about necessity to biopsy these lesions before planning
management. Circumstances under which a renal biopsy might be considered are:

The patient is not a surgical candidate (controversial)


Life expectancy is ≤5 years (controversial)
The patient requests a definite diagnosis before proceeding with the surgery.

NOTE - Biopsy is not recommended for patients who are candidates for active surveillance.

(Option E) Ablation of renal mass can be performed with either freezing (cryoablation) or heat
(radiofrequency or microwave ablation). Using a percutaneous approach, a needle is used to ablate
the tumor with heat or freezing. After ablation, all patients should undergo surveillance for
assessment of residual or recurrent tumor that would require additional therapy. Although initial
studies have shown favorable short-term results with thermal ablation, long-term outcomes are yet
to be studied. Thermal ablation is reserved for patients with a small RCC, who are not candidates
for surgery.

Ablation is also considered for:

Treatment of residual RCC after partial nephrectomy


Those with a unilateral or transplanted kidney
Patients at risk for multiple RCCs over their lifetime.

References

• RACGP - Incidentally detected small renal masses

• Diagnostic Imaging Pathways - Renal Mass

• UpToDate - Diagnostic approach, differential diagnosis, and management of a small renal mass
Last updated:
Time spent: QID:1111
2023-2-12
1895 of 1943
A 69-year-old man presents to your GP clinic with complaint of uriary incontinence. He explains
that he cannot make it to the bathroom once he feels the urge to pass urine. He denies

You might also like